ABC Amber CHM Converter Trial version, http://www.processtext.com/abcchm.html
яШяа JFIF
яЫC
Page 1
ABC Amber CHM Co...
119 downloads
2095 Views
24MB Size
Report
This content was uploaded by our users and we assume good faith they have the permission to share this book. If you own the copyright to this book and it is wrongfully on our website, we offer a simple DMCA procedure to remove your content from our site. Start by pressing the button below!
Report copyright / DMCA form
ABC Amber CHM Converter Trial version, http://www.processtext.com/abcchm.html
яШяа JFIF
яЫC
Page 1
ABC Amber CHM Converter Trial version, http://www.processtext.com/abcchm.html
Page 2
ABC Amber CHM Converter Trial version, http://www.processtext.com/abcchm.html
!
"$"$
яЫC
яА
ф –
"
яД
яДµ
} !1A Qa "q 2Ѓ‘Ў #B±Б RСр$3br‚ %&'()*456789:CDEFGHIJSTUVWXYZcdefghijstuvwxyzѓ„…†‡€‰Љ’“”•–— ™љўЈ¤Ґ¦§Ё©ЄІіґµ¶·ё№єВГДЕЖЗИЙКТУФХЦЧШЩЪбвгдежзийкстуфхцчшщъяДяДµ w !1 AQ aq "2Ѓ B‘Ў±Б #3Rр brС $4б%с &'()*56789:CDEFGHIJSTUVWXYZcdefghijstuvwxyz‚ѓ„…†‡€‰Љ’“”•–— ™љўЈ¤Ґ¦§Ё©ЄІіґµ¶·ё№єВГДЕЖЗИЙКТУФХЦЧШЩЪвгдежзийктуфхцчшщъяЪ
Page 3
ABC Amber CHM Converter Trial version, http://www.processtext.com/abcchm.html
?тЧvу жn§Ѕ7sя}ї:эc}M%}Йу§GрН›ю 7†юfяђҐ·•ъhµчий_|3я’Ќбїы [•иЕЇїЗJщьзгЏЎйа> QExЗy‰г•щ .>©яЎ ућ}•Zфo яИ·qхOэ WњgЬW~ аfsЬ_уЦЉLыЉ3о+¤‘ sюsG>яќ&}Е ч јыюtsюM&}Е• yчьизЯу¤ПТЏКЂ џ•ОЋ}OзIщRP №х?ќыюtЪ( Щ>§уЈ'Яу¤ хҐПЅ Йч пFG ч ¬ дыС“пI‘ к(Их X Йч¤ЙчЈ>яҐ чўА.OЅ >ф™Ј>фX жЊљLэ(ЭпEЂ\Р3пIёъС»Ю‹јыТупI»Ь~T›ЅЗе@зЮЋ} й7{ЏКЌЮгт зЮЏО“wёьЁЭо( yчЈљMЮвЌЮгт зФРsкi7{ЏК‚Ьu Ух4 ¦‘‰х РyлL єh P¶г?Ѕ_ж+ЦkЙ4НЯЪ Ш ~х{{ЉхєвЕn‹ЂQE ИXQE щВгчЌх4 §Iю±ѕ¦ЃЉы”|сїрМ• ЕЖрЯэ…-їфjЧЯВѕ шiя% Г•ц ¶яС«_~ЉщьзгЏЎй`> QExЗy‡гЇщ о?ЮOэ WљnЇHшЂЫ|-rЮЊ џъ Ї,у‡z Uо?S)о[ЭFк©з Z<б]6&еНФf©щЮфyГЦ‹ Л{ЁЭUГ7ЩчфЯ•ю™ь3[— 1jю рЬq\}‹ы•XХ7>2b #дыа ?ќ}F!Їh|Х[8ЪЗ%б{Щх Њє5ХЕ”– RZХ mМҐ]G5чpЇ€ x›г¶Љ±•ФlД‡элэ6ЧЫўјМТЮе»?Мх2Жќ+Ї/И(ўЉтЏDб>nэ µу/ќп_K~Р'oВЅLя· юЋJщgНчЇs-W¤эN:пЮ4<пzM щЉ·еUO ью Vя¦ЇьЕjн®vхgОвгzТ+y4ХEo єA>Ж¬КїєoЎ¬• ЭщЪО`ь›i#)юл 'хюt\зTЫz >Ubx®Фэ‹н*ї4 •ь Ј~њю Тm М 4- Њ«)Sф4\Ё~оjkЎЗшZMъхЄз®пэ Ч]°ЇтЇ5уOz~Љ7x•Nlu·_еZTV±лf:У^Ј|ЄlСb&"®m¦Nї№n+;ћ#††f‰"ЭAwЗП ЛВ•
Page 4
ABC Amber CHM Converter Trial version, http://www.processtext.com/abcchm.html
ъ »дЦ'‚ж YЧмЙд]yЂ~й]&ЪIљФўўХ»/Ижх›–†tп %ЬНтНs• §6~ ѕppН Ќ•Юsґ•:мь“н^mсћд%Ґ•ђ<Нx цQI?®ЪЁкмL)^q^fПВф ВOЏбќЂь–є,Ц В жxB_ъъ•дµЦyЩНzс†ј{в™Щв‹~ЩіяЩНU'yЎбcj«жaщЈЦ¶ф \йJ°K ]ZѓА ‡O¦zЏjеьПzµЫ„ѓnжёu«‘sМЈМЄЫиЯ]ц:yЌя Iџ ияхя юЊZыЦѕ рcяЕ_Ј•Чьъ1kпкріЏЉ?3» о QE гЎE P з њHЯSZ ич f—іMmmo ЬЇ$ “ш.N~ўідяXЯS]EґўГГ~d)2ґCНhUјЖV8 іЎ^г…a_YЊ RљЉ§»vю»}МсhB2o›ў+|2я’Ќбїы [•иЕЇї…| рЛюJ7†ям)mяЈ ѕю жg?}М ВВЉ(Ї о<љм?б&Т•зв ыьїг_* #qуэђЛЋ3е“ЏjЊШВ ѓfЂ·Aех® аЬ¤еr\[w№хlѕ$Т^6OґА21џ9• Жѕyш™W’ HЛЗ†Cђ•vЅл™:|bC І Aь>W?•*B"Њ Њ" Ф. 5t0Оњ№® ‹Nч$ЯЋrGТѕ р§‹флп й— ч7P‹™bЫ2 Hu;XђOr3шЧМю\›Вyoјт o&Ўx!”‚с#·A‘љУ CЪҐaЛSкяшIґџщш‡юя/шСя 6“я? яЯея щCм yѕ_Ш—•ч|®•*UУђЎqb ¬"аW/Ф_r9•Аъ»ю }'ю~!яїЛю5Г|jХl/ј ?Що!gубВ “уЏC^ Ц * {5Un„ЗЂh[8•УmЄ‡8)„дэ*Ј‚qiЬj-;¶Mїћµф—ЃхЭ.УАЪ ЭCж‹ чЇљ ЇM|ЬРLІщO
Page 5
ABC Amber CHM Converter Trial version, http://www.processtext.com/abcchm.html
‹'ч ђЯ•@Ц‘ ИZG¶VЗЮb™ЗЦє+Сц©$КwjЗХяр“i?ус эю_с¦Mв=&Hљ?ґА3ЯО_сЇ”ТКЭШ"ZЖМx&I4ᦫ; °щ —п(‹‘хв№~ўы‘ИЯ_Аиѕ!Нћ;Ц^ D3Њ2њѓт/q]пмщЪЩГ¬ЩЮЬG b9гЮАdт¤
Page 6
ABC Amber CHM Converter Trial version, http://www.processtext.com/abcchm.html
яАkИгµxЈm–М€¤оВ ч¤kf™UZ nTlОqй]RЈz\—/ҐЏЄяб&Т•звыьїгGь$ЪOььCя•— ьkе%УUҐт…†dЖvщ\юX¦-”
Page 7
ABC Amber CHM Converter Trial version, http://www.processtext.com/abcchm.html
ы Т2Э6„ж№~ўы‘Иыю Х Џ€ф©ё ё“2фъЧЛўб®^K§?4т4НхbOхЁОњћbЗц є/•Йь*U†Rж$…Л/U r? иЎ‡цNн• Шч_Ѓъ†џа'ыUФ)#^Йµ @ Њ/їЛю5тЊ¶сИш– fa —‘ HЦ1(bЦh њ11ф>†І©ѓr“•ч “zЬъїю m'ю~!яїЛю5в•.но|Oa%ґСКїc м`Ш;П\WћЌ9 lXѓ·®"йх§[Z
Page 8
ABC Amber CHM Converter Trial version, http://www.processtext.com/abcchm.html
±¶¶зш¶'шSҐ…pљ•З ґпqылёр/Д{я $vWЁчљzp ЮД?Щ'‚=ЌqLЫ ь©6©Б;N LЮ_™е>Ппm8ьлЄҐ8ФVczо}-бяшoW ¶ъ„ Sя,ќ„rЏш •Jй#ѕІe НЖ}VѕB’Ь±+$$• ЇAлVmоµ{ SoЁjVЁ~оЛ‰ • њW
Page 9
ABC Amber CHM Converter Trial version, http://www.processtext.com/abcchm.html
°/мІy_F}oц»щшJFЅ±тр§и |ЇэївЕ@ЯЫљ°CЊ ь{sЉ†гXс%Бd›\ХЫ2‰ЩqхЖ+?©LV—‘ фц©в=#N„ЛssH?ЉW ЇжMyџЊ>0X,ooЎ'цЊЗ 4yX эзю/ўЧЋ\Y»?ќy ДџїpYО~N Йеy‰ ББ`§ъf¶† ¬ r_вdє…хц©Ё>ҐЄ\ ‹· О0ЁїЭQШ•:Џ}+[Ь.7A(ЙАКґС„ ЋNЮ •OwF1ЉІ4Ш7љ7R ҐW(bђ63·iО)»[hm§i8 жЗѓ ю+ юВ •иЕЇРZьчрgьЋ:'э„ яС‹_Ў аg? ~gЈЃЪAE WЉw… Q@њR•¬o©®»M)7‡– |“Jё [LQ]єeјН ђsњW#'ъЖъљк!Љ;П€Щѓyp ¬ЪњH ’ЅcЖ•„Nx фЩ’јaµZйъяГћN ЩЛРрЗюJ7†ям)mяЈ ѕэН| рЛюJ7†ям)mяЈ ѕщЭ\YПЗCЈ рІMФnЁчQєј[ќЗљюФ'ю,¶±я]mїфzWЖ ѕОэ§ “аО°ЈЇ™oяЈТѕ5т$х фyLвЁ;ѕїдrWБb+Лљ” ^GЁ ь7ћЗю VЇ¦j…RЛSЦaі’Cя,™ЎsћЫd O°5С_YMбпxѓД— ¶ZmЖ•agciqsЅў[§… ыЉДб NѓёЇ т¦Ы·wЛњг' ҐtёpCИX “–'љй•(№7Нїь т —гRKЩ= m7Z—‡х¦‰«ЬГҐ\iЪЪgЩжeЌ|й#;Рq‚2Лћ .• љЯГ Ј‡ЦЯВ7MҐ»ђ\Э™X‚¤я7cѕkГ¶O·aй· ЏOOҐ'—7ЛуэЯ»ЙгйSмao‹ъµ‡х
Page 10
ABC Amber CHM Converter Trial version, http://www.processtext.com/abcchm.html
mя„ПyрЅОЎg{б;ЅpИnyк №ЕeЯшfУ]С|#ћИљ^і\Я’ ъ€|Ёљu>› H? ^6ЛrТ Vg=X±$ю4Ѓ. B‰ 8 Ћ zСмbќФя•М>ЎЊjО‹ю¬} §4$сз„|Ui}c~рк :tпf® #)$– кCЄћ ІзВ9Ї ]#UС|U§ZкъuХ„Нq ¬w bҐЗ8=ё5”‰:}Й ућ ЋhtќШ3И]‡BМI)B4Ыґґ·щя™3Л±Ія— Lъ^[ЛCсEј[ѕ!~5sбХ^7o $пЗэ{ь№ч®:ак_р¦н…„~'xМъџћt§Её яеа`еqџN3^4 }ЫЊ‡9ЭќЗЇЇЦ•EК ЎE™‚Єв±Ћ ·Ѕµї я™o Ќ•тйћыpuUЏD:Ї›я Oь!сэ»ПяЏs/ђыvgЏ7~Мmжі4М}§K ;?б > ѓы 8ЭцЏ1іі?тУfм~5вl“І…i *ф ‰ ЃЫ•‹њЁВњћ>”э„mс•_Цвъ†7ю}3Ф|!kвk-[Vсw‹по4эCKіHlо5Ѓ.Vi‹,|mf8Q!zVц©бйntЇ€‹бЫ WФ"Ц Уµ+$ґ‰›zЙ33l О г W‰Лц™I2Мтg®з'?ќ"‹• dОё бИвЄTУ|ЬЛ§¦–•ЧЁ– _ЊJЮЙќWВ«ќGDсґў= Rїё УЫП љ‘un ЪТGБЫ"ц?ZфН NїТoю!ZЪк^&ХЇЩЇ 1JH,ЩААдсC$нќТ “ёеЏ'ЦЄT ФW6Ль„ІьmЫцOSиЏ ЕвX&ш… †Eкл Нў Є‹2|Б€nџГлћ=iЦ…7шЧы u'ФEЋ’ /ябeYЌЮжуьІ ~ц?ЪЇќяТІЗО|·Ю;П?ZHЦxуеJИO]¬Fk/«FЦж]?Oт+к8Ыя яWя3ЫфЏ Еб_‡¶WъЛл…Ћ№~ІШ8RnШЁ .w Ьдрyм+;Е‡R•„>6°ш—мmҐѓ/Щ ю%лыц8 cЇNю•д м0ОXg<±ЁkВ€№( Ме Aјвђ¤Зvdo›–щЏ?ZҐB*ЮцЯж ћ Їо™и• µ^ыDр| љќЭМShђЬК ’LМ]О7ђz¶8НiЫяiкя JHєЗ‡бС¬<ШҐRRГTC.@#ЊН»Ў ОЮХдн НЌНњ
Page 11
ABC Amber CHM Converter Trial version, http://www.processtext.com/abcchm.html
“А§2ЬґB&•Њjr ±Ъ? їgU ЦЋдяgcnЫ¤х=¦_ кr]ь%“VЦ®¤¶џlчfyЙWu№eW|ћHdф п Ш]иНЈXк¶ІЩЭ\шрO
Page 12
ABC Amber CHM Converter Trial version, http://www.processtext.com/abcchm.html
s&Ц’ ё. к№ gҐx“E13d/@IвњвеЩYжfe R\’ >•Њ-e%э7юe}C Ц“=§UЦµп йW:§ЉјцХ!с яdG{юєKSјN‡?7’КTsЖHЕrҐґ±с KєЌХ к7W‹sјуЇЪi€ш3¬ yу-яфzWЗdџЯ5цн2Щш3¬•ЧKoэ•сРН}.O и;®їў?ЮЗy’ •|СжIэуM4 9#Ш>№€яџЏпcјЙ?ѕhу$ющ¦СG${ Ч1усэмp’Oпљ?ЮЗ $ющЈМ“ыж›[> рѕ№вНAфэ Пнw1Жedу 0№ 9b;‘JJ Wv°,^%»)ЛпfG™'чН'™'чНzь)/‰ я‘ uїр&/ю*ёmoKѕСµkќ+RЂБyk!ЋhЙ k}G ќ ЋСi”с8ЁьS—ЮКВI?ѕhу$ющ¦ц®— БЮ с_Њ-о.|=Ґ=дVО Vу b2 МFxЄџі‚ј¬ђ–/ 'e9}мжьЙ?ѕiD’•|ЦЧЊ
Page 13
ABC Ambe r CHM Conve rte r Tria l ve rsion, http://w w w .proce sste x t.com/a bcchm.html
Editors: Wolfsthal, Susan T itle: NMS Medicine, 6th Edition Copyri ght ©2008 Li ppi ncot t W i l l i ams & W i l ki ns > Front of Book > Edit ors
Editor
Susan Wolfsthal MD Cel es t e L. W oodward Profes s or i n Humani t ari an and Et hi cal Medi cal Pract i ce; As s oci at e Chai r for Educat i on; Res i dency Program Di rect or, Int ernal Medi ci ne Uni vers i t y of Maryl and Sc hool of Medi c i ne, Bal t i more, Maryl and
Contributors to the Sixth Edition Pamela J. Amelung MD As s i s t ant Profes s or Depart ment of Medi c i ne, Di vi s i on of Pul monary and Cri t i c al Care Medi c i ne, Uni vers i t y of Maryl and Sc hool of Medi c i ne, Bal t i more, Maryl and Yanis Bellil MD Chi ef Res i dent 2007–2008; Cl i ni cal Ins t ruct or Depart ment of Medi c i ne, Uni vers i t y of Maryl and Medi c al Cent er, Bal t i more, Maryl and R. Michael Benitez MD As s oci at e Profes s or Depart ment of Medi c i ne, Di vi s i on of Cardi ol ogy, Uni vers i t y of Maryl and Sc hool of Medi c i ne, Bal t i more, Maryl and Sonia E. Blome MD Chi ef Res i dent 2007–2008; Cl i ni cal Ins t ruct or Depart ment of Medi c i ne, Uni vers i t y of Maryl and Medi c al Cent er, Bal t i more, Maryl and Mary E. Bollinger DO As s oci at e Profes s or Depart ment of Pedi at ri c s , Di vi s i on of Pedi at ri c Pul monol ogy and Al l ergy, Uni vers i t y of Maryl and Sc hool of Medi c i ne, Bal t i more, Maryl and Marc F. Brazie MD Res i dent Depart ment of Medi c i ne, Uni vers i t y of Maryl and Medi c al Cent er,
Pa g e 1 4
ABC Ambe r CHM Conve rte r Tria l ve rsion, http://w w w .proce sste x t.com/a bcchm.html
Bal t i more, Maryl and Jennifer Zahn Cooper MD As s i s t ant Profes s or Depart ment of Dermat ol ogy, Uni vers i t y of Maryl and Sc hool of Medi c i ne, Bal t i more, Maryl and Raymond H. Flores MD As s oci at e Profes s or Depart ment of Medi c i ne, Di vi s i on of Rheumat ol ogy and Cl i ni c al I mmunol ogy, Uni vers i t y of Maryl and Sc hool of Medi c i ne, Bal t i more, Maryl and Evonne G. Fontanilla MD Chi ef Res i dent 2006–2007; Cl i ni cal Ins t ruct or Depart ment of Medi c i ne, Uni vers i t y of Maryl and Medi c al Cent er, Bal t i more, Maryl and Bruce D. Greenwald MD As s oci at e Profes s or Depart ment of Medi c i ne, Di vi s i on of Gas t roent erol ogy and Hepat ol ogy, Uni vers i t y of Maryl and Sc hool of Medi c i ne, Bal t i more, Maryl and Donna S. Hanes MD, FACP As s oci at e Profes s or Depart ment of Medi c i ne, Di vi s i on of Nephrol ogy, Uni vers i t y of Maryl and Sc hool of Medi c i ne, Bal t i more, Maryl and Kerri Kissell MD Res i dent Depart ment of Medi c i ne, Uni vers i t y of Maryl and Medi c al Cent er, Bal t i more, Maryl and Janaki C. Kuruppu MD As s i s t ant Profes s or Depart ment of Medi c i ne, Di vi s i on of I nfec t i ous Di s eas es , Uni vers i t y of Maryl and Sc hool of Medi c i ne, Bal t i more, Maryl and Jefferson H. Lee MD Chi ef Res i dent 2007–2008; Cl i ni cal Ins t ruct or Depart ment of Medi c i ne, Uni vers i t y of Maryl and Medi c al Cent er, Bal t i more, Maryl and Dan Lender MD
Pa g e 1 5
ABC Ambe r CHM Conve rte r Tria l ve rsion, http://w w w .proce sste x t.com/a bcchm.html
As s oci at e Profes s or Depart ment of Medi c i ne, Di vi s i on of Endoc ri nol ogy, Di abet es , and Nut ri t i on, Uni vers i t y of Maryl and Sc hool of Medi c i ne, Bal t i more, Maryl and Jeffrey C. Liu MD Chi ef Res i dent 2007–2008; Cl i ni cal Ins t ruct or Depart ment of Medi c i ne, Uni vers i t y of Maryl and Medi c al Cent er, Bal t i more, Maryl and Rebecca L. Manno MD Chi ef Res i dent 2006–2007; Cl i ni cal Ins t ruct or Depart ment of Medi c i ne, Uni vers i t y of Maryl and Medi c al Cent er, Bal t i more, Maryl and Devang M. Patel MD Chi ef Res i dent i n Medi ci ne-Pedi at ri cs 2006–2007 Depart ment of Medi c i ne and Depart ment of Pedi at ri c s , Uni vers i t y of Maryl and Medi c al Cent er, Bal t i more, Maryl and Neil C. Porter MD As s i s t ant Profes s or Depart ment of Neurol ogy, Uni vers i t y of Maryl and Sc hool of Medi c i ne, Bal t i more, Maryl and Darryn R. Potosky MD Chi ef Res i dent 2006–2007; Cl i ni cal Ins t ruct or Depart ment of Medi c i ne, Uni vers i t y of Maryl and Medi c al Cent er, Bal t i more, Maryl and Alexandra K. Pratt MD Chi ef Res i dent 2007–2008; Cl i ni cal Ins t ruct or Depart ment of Medi c i ne, Uni vers i t y of Maryl and Medi c al Cent er, Bal t i more, Maryl and Jennifer A. T aylor MD Chi ef Res i dent 2006–2007; Cl i ni cal Ins t ruct or Depart ment of Medi c i ne, Uni vers i t y of Maryl and Medi c al Cent er, Bal t i more, Maryl and Katherine H. Rak T kaczuk MD As s oci at e Profes s or Depart ment of Medi c i ne, Di vi s i on of Hemat ol ogy/Onc ol ogy, Uni vers i t y of Maryl and Sc hool of Medi c i ne, Bal t i more, Maryl and
Pa g e 1 6
ABC Ambe r CHM Conve rte r Tria l ve rsion, http://w w w .proce sste x t.com/a bcchm.html
Ann B. Zimrin MD As s i s t ant Profes s or Depart ment of Medi c i ne, Di vi s i on of Hemat ol ogy/Onc ol ogy, Uni vers i t y of Maryl and Sc hool of Medi c i ne, Bal t i more, Maryl and
Fifth Edition Editor-in-Chief Allen R. Myers MD Profes s or of Medi ci ne Sec t i on of Rheumat ol ogy, Depart ment of Medi c i ne, T empl e Uni vers i t y Sc hool of Medi c i ne, Phi l adel phi a, Penns yl vani a
Contributors to the Fifth Edition E. Victor Adlin MD As s oci at e Profes s or of Medi ci ne Emeri t us Endoc ri nol ogy and Met abol i s m Sec t i on, Depart ment of Medi c i ne, T empl e Uni vers i t y Sc hool of Medi c i ne; At t endi ng Phys i c i an, T empl e Uni vers i t y Hos pi t al , Phi l adel phi a, Penns yl vani a Jack M. Becker MD As s oci at e Profes s or of Pedi at ri cs Drexel Uni vers i t y Col l ege of Medi c i ne; Chi ef, Sec t i on of Al l ergy, Sai nt Chri s t opher's Hos pi t al for Chi l dren, Phi l adel phi a, Penns yl vani a Hossein Borghaei DO, MS As s i s t ant Member Depart ment of Medi c al Onc ol ogy, Fox Chas e Canc er Cent er, Phi l adel phi a, Penns yl vani a Blase A. Carabello MD Profes s or of Medi ci ne Bayl or Col l ege of Medi c i ne; Chi ef, Depart ment of Medi c i ne, Hous t on Vet erans Affai rs Medi c al Cent er, Hous t on, T exas Gerard J. Criner MD Profes s or of Medi ci ne; Di rect or Pul monary and Cri t i c al Care Medi c i ne, T empl e Uni vers i t y Sc hool of Medi c i ne, Phi l adel phi a, Penns yl vani a Anthony J. DiMarino Jr. MD W i l l i am Rorer Profes s or of Medi ci ne T homas Jeffers on Medi c al Col l ege; Chi ef, Di vi s i on of Gas t roent erol ogy, T homas Jeffers on Uni vers i t y Hos pi t al ,
Pa g e 1 7
ABC Ambe r CHM Conve rte r Tria l ve rsion, http://w w w .proce sste x t.com/a bcchm.html
Phi l adel phi a, Penns yl vani a T homas Fekete MD Profes s or of Medi ci ne I nfec t i ous Di s eas es Sec t i on, Depart ment of Medi c i ne, T empl e Uni vers i t y Sc hool of Medi c i ne; At t endi ng Phys i c i an, T empl e Uni vers i t y Hos pi t al , Phi l adel phi a, Penns yl vani a Stanley Goldfarb MD As s oci at e Dean for Cl i ni cal Educat i on Uni vers i t y of Penns yl vani a Sc hool of Medi c i ne, Phi l adel phi a, Penns yl vani a Donald P. Goldsmith MD Profes s or of Pedi at ri cs Drexel Uni vers i t y Col l ege of Medi c i ne; Chi ef, Sec t i on of Rheumat ol ogy, St . Chri s t opher's Hos pi t al for Chi l dren, Phi l adel phi a, Penns yl vani a S. Christine Kovacs MD, MPH Di vi s i on of Rheumat ol ogy, Lahey Cl i ni c , Burl i ngt on, Mas s ac hus et t s Stuart R. Lessin MD Seni or Member and Di rect or of Dermat ol ogy Fox Chas e Canc er Cent er; Profes s or of Medi c i ne (Dermat ol ogy), Chi ef, Di vi s i on of Dermat ol ogy, Depart ment of Medi c i ne, T empl e Uni vers i t y Sc hool of Medi c i ne, Phi l adel phi a, Penns yl vani a Allen R. Myers MD Profes s or of Medi ci ne Sec t i on of Rheumat ol ogy, Depart ment of Medi c i ne, T empl e Uni vers i t y Sc hool of Medi c i ne, Phi l adel phi a, Penns yl vani a Ronald N. Rubin MD Profes s or of Medi ci ne Sec t i on of Hemat ol ogy, Depart ment of Medi c i ne, T empl e Uni vers i t y Sc hool of Medi c i ne; Chi ef, Cl i ni c al Hemat ol ogy, T empl e Uni vers i t y Hos pi t al , Phi l adel phi a, Penns yl vani a Barney J. Stern MD Profes s or of Neurol ogy Depart ment of Neurol ogy, Uni vers i t y of Maryl and Sc hool of Medi c i ne, Bal t i more, Maryl and Louis M. Weiner MD
Pa g e 1 8
ABC Ambe r CHM Conve rte r Tria l ve rsion, http://w w w .proce sste x t.com/a bcchm.html
Profes s or of Medi ci ne T empl e Uni vers i t y Sc hool of Medi c i ne; Chai rman, Depart ment of Medi c al Onc ol ogy; Seni or Member, Di vi s i on of Medi c al Sc i enc e, Fox Chas e Canc er Cent er, Phi l adel phi a, Penns yl vani a Fuad N. Ziyadeh MD Profes s or of Medi ci ne Di vi s i on of Renal , El ec t rol yt e, and Hypert ens i on, Depart ment of Medi c i ne, Uni vers i t y of Penns yl vani a Sc hool of Medi c i ne; At t endi ng Phys i c i an, Hos pi t al of t he Uni vers i t y of Penns yl vani a, Vet erans Affai rs Medi c al Cent er, Phi l adel phi a, Penns yl vani a
Pa g e 1 9
ABC Ambe r CHM Conve rte r Tria l ve rsion, http://w w w .proce sste x t.com/a bcchm.html
Editors: Wolfsthal, Susan T itle: NMS Medicine, 6th Edition Copyri ght ©2008 Li ppi ncot t W i l l i ams & W i l ki ns > Front of Book > Dedic at ion
Dedication
To my l ovi ng hus band, Bi l l , my daught ers , Rebecca and Jenni fer, and my ment ors who have s upport ed and encouraged my career i n medi cal educat i on at t he Uni vers i t y of Maryl and.
Pa g e 2 0
ABC Ambe r CHM Conve rte r Tria l ve rsion, http://w w w .proce sste x t.com/a bcchm.html
Editors: Wolfsthal, Susan T itle: NMS Medicine, 6th Edition Copyri ght ©2008 Li ppi ncot t W i l l i ams & W i l ki ns > Front of Book > Prefac e
Preface
“Onc e s t udent s have t he opport uni t y t o get t hei r hands on a w arm pat i ent , t hey bei ng t o unders t and w hat i t 's al l about .― T heodore E. Woodw ard, MD, MACP (1914–2005), Profes s or and Chai rman, Depart ment of Medi c i ne (1954–1981) On t he oc c as i on of hi s ret i rement , Uni vers i t y of Maryl and Sc hool of Medi c i ne, 1981. He c ont i nued t eac hi ng s t udent s at t he Uni vers i t y of Maryl and for t he next 23 years . The fi rs t expos ure t o cl i ni cal medi ci ne duri ng medi cal s chool i s an e ye-openi ng experi ence—a t i me when t he s t udent bri ngs t oget her t he vas t amount of i nformat i on l earned i n t he bas i c s ci ences and merges t hat wi t h act ual pat i ent care. It i s a gl ori ous t i me for l earni ng and t he amount t hat needs t o be abs orbed grows exponen t i al l y wi t h t i me. It i s wi t h t hes e fi rs t s t eps i nt o cl i ni cal medi ci ne t hat t he young s t udent begi ns t o comprehend t he i mport ance of bei ng a l i fel ong l earner. W e abs orb bas i c concept s and bui l d upon t hem, modi fy what we know as new advances are di s covered, and cont i nual l y add t o our knowl edge for t he res t of our l i ves . Thi s al l t oo s hort revi ew t ext of i nt ernal medi ci ne i s our at t empt t o capt ure t he es s ent i al concept s and key el ement s of our s peci al t y at t hi s poi nt i n t i me. Al t hough i nt ernal medi ci ne i s cons t ant l y evol vi ng, t here are bas i c pri nci pl es and t hought proces s es t hat remai n t he es s ence of our s peci al t y. Learni ng t he fact s i s onl y t he begi nni ng. Medi cal s t udent s mus t devel op t hei r s ki l l s i n deduct i ve reas oni ng and s ynt hes i ze t hes e fact s , wei ghi ng t he pros and cons of t he eval uat i on and management choi ces for t hei r pat i ent s . Thi s newes t edi t i on of NMS-Medi c i ne was not onl y wri t t en by s peci al i s t s i n t he fi el d but , for t he fi rs t t i me, coaut hored by chi ef res i dent s and s eni or i nt ernal medi ci ne res i dent s at t he Uni vers i t y of Maryl and. The chi efs and s eni or res i dent s are much cl os er t o t he medi cal s t udent s i n t erms of t hei r t rai ni ng and experi ence, and hence, bri ng a fres h and rel evant approach t o t hi s mat eri al . W e
Pa g e 2 1
ABC Ambe r CHM Conve rte r Tria l ve rsion, http://w w w .proce sste x t.com/a bcchm.html
wel come t hem as aut hors of t hi s t ext not onl y for t hei r i nput , but al s o becaus e we enjoy ment ori ng our young col l eagues and provi di ng t hem wi t h an opport uni t y t o advance t hei r academi c careers . Thi s edi t i on has been compl et el y updat ed, i ncl udi ng t he chapt ers , ques t i ons , and probl em s ol vi ng exerci s es . W e have al s o pl aced a s i gni fi cant amount of mat eri al onl i ne, i ncl udi ng addi t i onal chapt er cont ent , t abl es , and i l l us t rat i ons . A
i con s i gnal s t o t he reader
where t o fi nd t hes e s uppl ement al onl i ne res ources . Al s o avai l abl e onl i ne i s a comprehens i ve exam cons i s t i ng of approxi mat el y 150 board-s t yl e revi ew ques t i ons wi t h ans wers and expl anat i ons . Al t hough t hi s book i s geared t oward medi cal s t udent s and res i dent s l earni ng i nt ernal medi ci ne, we ant i ci pat e t hat i t i s al s o us eful for ot hers l earni ng t he es s ent i al pri nci pl es , concept s , and i nformat i on i n our s peci al t y. As a reader of t hi s t ext , we encourage you t o us e t he mat eri al pres ent ed as merel y t he fi rs t s t ep t o readi ng about t hes e t opi cs i n more dept h, ei t her i n t he l arger and more cl as s i c t ext books of i nt ernal medi ci ne or, more i mport ant l y, i n t he pri mary s ource l i t erat ure. The proces s of bei ng a l i fel ong l earner begi ns here. W e hope you accept t hi s chal l enge, are exci t ed by t he fi el d of i nt ernal medi ci ne, and s hare i n our ent hus i as m for our chos en profes s i on. Sus an D. W ol fs t hal MD
Pa g e 2 2
ABC Ambe r CHM Conve rte r Tria l ve rsion, http://w w w .proce sste x t.com/a bcchm.html
Editors: Wolfsthal, Susan T itle: NMS Medicine, 6th Edition Copyri ght ©2008 Li ppi ncot t W i l l i ams & W i l ki ns > T able of Cont ent s > Chapt er 1 - Cardiovasc ular Diseases
Chapter 1
Cardiovascular Diseases Jennifer A. T aylor Michael Benitez
I. Congestive Heart Failure (CHF) A. Definition Conges t i ve heart fai l ure (CHF) i s t he i nabi l i t y of t he heart , worki ng at normal or el evat ed fi l l i ng pres s ure, t o pump enough bl ood t o meet t he oxygen requi rement s of t he body t i s s ues . CHF s houl d never be cons i dered a di agnos i s . Rat her, i t i s a s yndrome res ul t i ng from many di s eas es t hat i nt erfere wi t h cardi ac funct i on. In act i ng as a mus cul ar pump, t he heart does onl y t wo t hi ngs : i t cont ract s (s ys t ol e) and i t rel axes (di as t ol e). Therefore, heart fai l ure can res ul t from onl y t wo broad abnormal i t i es —s ys t ol i c dys funct i on and di as t ol i c dys funct i on.
B. Etiology
1. Systolic dysfunction. Sys t ol e i s governed by t hree cardi ac propert i es : contractility—t he abi l i t y of myocardi um t o generat e force, afterload—t he force agai ns t whi ch t he heart mus t cont ract , and preload —t he s arcomere s t ret ch before cont ract i on. o
o
a. Decreased contractility. Mos t cas es of CHF occur when an i ns ul t t o t he myocardi um reduces i t s abi l i t y t o generat e force, t hereby reduci ng i t s cont ract i l i t y.
Pa g e 2 3
ABC Ambe r CHM Conve rte r Tria l ve rsion, http://w w w .proce sste x t.com/a bcchm.html
(1) Myocardial infarction (MI). In MI, a port i on of t he myocardi um undergoes necros i s and can no l onger generat e force, res ul t i ng i n weakeni ng of t he vent ri cl e. If ext ens i ve areas of t he myocardi um are i nfarct ed, CHF res ul t s .
(2) Valvular heart disease res ul t s i n s t enos i s or regurgi t at i on of t he cardi ac val ves , whi ch pl aces a pressure or volume overload, res pect i vel y, on t he vent ri cl es . Ini t i al l y, compens at ory mechani s ms [s ee I B 1 c] accommodat e t hes e overl oads and mai nt ai n normal cardi ac out put at accept abl e fi l l i ng pres s ures . However, event ual l y t hes e mechani s ms fai l and heart fai l ure ens ues .
(3) Hypertension. Hypert ens i on may cont ri but e t o ei t her s ys t ol i c or di as t ol i c dys funct i on.
(4) Cardiomyopathies are di s eas es t hat di rect l y i njure t he myocardi um.
(a) T oxic. Subs t ances di rect l y t oxi c t o t he myocardi um (e.g., et hanol and cat echol ami nes ) may damage i t s force-generat i ng abi l i t y. Prol onged expos ure t o t hes e agent s may l ead t o t he devel opment of CHF.
(b) Idiopathic. W hen t he cont ract i l e
Pa g e 2 4
ABC Ambe r CHM Conve rte r Tria l ve rsion, http://w w w .proce sste x t.com/a bcchm.html
funct i on of t he myocardi um fai l s i n t he abs ence of a known et i ol ogy, a vi ral caus e oft en i s i mpl i ed but frequent l y cannot be proven. Ot her cl i ni cal pres ent at i ons , i ncl udi ng peri part um cardi omyopat hy, are of unknown et i ol ogy.
(c) Infiltrative. Infi l t rat i on of t he myocardi um by a vari et y of s ubs t ances (e.g., amyl oi d) may reduce cont ract i l i t y.
o
o
b. Increased afterload. Increas i ng t he aft erl oad makes i t harder for t he vent ri cul ar mus cl e fi bers t o s hort en, t hereby reduci ng cardi ac out put . Aft erl oad can be quant i fi ed by cal cul at i ng t he s ys t ol i c force on t he myocardi um us i ng t he Lapl ace equat i on for s t res s : St res s = (pres s ure × radi us )/(2 × t hi cknes s ) Thus , di s eas e s t at es t hat i ncreas e ei t her t he s ys t ol i c pres s ure (hypert ens i on, aort i c s t enos i s ) or chamber radi us (di l at ed cardi omyopat hy, val vul ar regurgi t at i on) i ncreas e aft erl oad unl es s t he wal l t hi cknes s i ncreas es proport i onat el y.
P.2
o
o
c. Compensatory mechanisms devel op i n res pons e t o t he vent ri cul ar pres s ure and vol ume overl oad t hat accompany decreas ed cont ract i l i t y.
Pa g e 2 5
ABC Ambe r CHM Conve rte r Tria l ve rsion, http://w w w .proce sste x t.com/a bcchm.html
(1) T he Frank-Starling mechanism i s act i vat ed when reduced vent ri cul ar empt yi ng res ul t s i n more vol ume ret ai ned i n t he vent ri cl es at t he end of s ys t ol e, whi ch l eads t o a great er vol ume at t he end of di as t ol e. Increas ed end-di as t ol i c vol ume i ncreas es s arcomere s t ret ch (prel oad), whi ch i ncreas es t he number of s ys t ol i c act i n–myos i n cros s -bri dges t hat devel op. The great er number of cros s -bri dges i ncreas es t he s t rengt h of cont ract i on.
(2) Cardiac hypertrophy provi des addi t i onal mus cl e mas s t o bear t he burden of vari ous overl oads .
(3) Adrenergic stimulation by endogenous cat echol ami nes i ncreas es t he i not ropi c s t at e.
2. Diastolic dysfunction. Di as t ol e i s governed by active and passive propert i es . Act i ve rel axat i on occurs earl y i n di as t ol e as cal ci um i s pumped out of t he myocardi um, res ul t i ng i n t he near ces s at i on of act i n–myos i n cros s -bri dge i nt eract i on. Pas s i ve fi l l i ng occurs as t he mi t ral val ve opens , al l owi ng t he bl ood s t ored i n t he at ri a t o fi l l t he vent ri cl es . o
o
a. Abnormalities in active relaxation. Act i ve rel axat i on i s i mpai red when t here i s del ay i n cal ci um reupt ake at t he begi nni ng of di as t ol e.
o
Pa g e 2 6
ABC Ambe r CHM Conve rte r Tria l ve rsion, http://w w w .proce sste x t.com/a bcchm.html o
b. Abnormalities of passive filling. Pas s i ve rel axat i on i s i mpai red when t he myocardi um i s s t i ffer t han normal . St i ffnes s i s defi ned as a change i n pres s ure (ΔP) per uni t change i n vol ume (ΔV), or ΔP/ΔV. W hen s t i ffnes s i s i ncreas ed, any change i n vol ume requi res or caus es a great er i ncreas e i n pres s ure. Thus , t o fi l l t he heart t o an adequat e vol ume, hi gh fi l l i ng pres s ure occurs , whi ch i n t urn l eads t o pul monary and s ys t emi c conges t i on. Increas ed pas s i ve s t i ffnes s of t he vent ri cl es occurs when concent ri c hypert rophy caus es t he chamber wal l t o be t hi cker t han normal , as mi ght occur i n hypert ens i on or when t he myocardi um i s i nfi l t rat ed by abnormal s ubs t ances s uch as amyl oi d.
3. T he neurohumoral hypothesis of heart failure. Heart fai l ure l eads t o t he pers i s t ent act i vat i on of many neurohumoral s ys t ems and hormones , i ncl udi ng t he reni n-angi ot ens i n-al dos t erone s ys t em, t he adrenergi c nervous s ys t em, i nfl ammat ory cyt oki nes , endot hel i n, and vas opres s i n. Al t hough once t hought of as compens at ory, pers i s t ent overact i vat i on of t hes e agent s i s cardi ot oxi c, i n t urn l eadi ng t o a progres s i ve decl i ne i n cardi ac funct i on. Thus , bl ockade of t hes e s ys t ems s houl d be benefi ci al i n t reat i ng CHF.
C. Types of Heart Failure
1. High-output failure i s charact eri zed by cardi ac out put t hat may be s everal t i mes hi gher t han normal but s t i l l i s not adequat e t o mai nt ai n t i s s ue perfus i on needs or, i f adequat e, i s mai nt ai ned wi t h a hi gher-t han-normal fi l l i ng pres s ure. A cl as s i c exampl e of hi gh-out put fai l ure
Pa g e 2 7
ABC Ambe r CHM Conve rte r Tria l ve rsion, http://w w w .proce sste x t.com/a bcchm.html
i s chronic severe anemia, whi ch caus es reduced oxygen-carryi ng capaci t y. In chroni c s evere anemi a, t he fol l owi ng occurs : o
o
a. Compens at i on i s provi ded by i ncreas ed forward cardi ac out put , whi ch i s faci l i t at ed by cardi ac enl argement , decreas ed t ot al peri pheral res i s t ance, and i ncreas ed venous ret urn t o t he heart . Thi s caus es a vol ume overl oad of vent ri cl es .
o
o
b. Event ual l y t he demands on t he heart l ead t o cardi ac fai l ure; cardi ac out put , al t hough hi gh, s t i l l i s not adequat e t o meet t he ci rcul at ory demands pl aced on t he heart by t he anemi a. Some ot her caus es of hi gh-out put fai l ure i ncl ude art eri ovenous fi s t ul a, beri beri , and t hyrot oxi cos i s .
2. Left-sided failure i ndi cat es t hat t he l eft vent ri cl e i s t he fai l i ng chamber. A di s eas e t hat pri mari l y affect s t he l eft vent ri cl e (e.g., MI) may reduce i t s cont ract i l e force, whereas t he ri ght vent ri cl e cont i nues t o pump normal l y. Thus , l eft vent ri cul ar fai l ure can occur wi t hout ri ght vent ri cul ar fai l ure.
3. Right-sided failure i ndi cat es t hat t he ri ght vent ri cl e has fai l ed, ei t her as a res ul t of l eft vent ri cul ar fai l ure or i n i s ol at i on from t he l eft vent ri cl e. o
o
a. The most common cause of ri ght vent ri cul ar fai l ure i s left ventricular failure. W hen l eft vent ri cul ar fai l ure occurs , t he fi l l i ng pres s ure i n t he l eft vent ri cl e becomes el evat ed, i ncreas i ng t he
Pa g e 2 8
ABC Ambe r CHM Conve rte r Tria l ve rsion, http://w w w .proce sste x t.com/a bcchm.html
workl oad of t he ri ght vent ri cl e (t he chamber res pons i bl e for fi l l i ng t he l eft vent ri cl e). Thus overt axed, t he ri ght vent ri cl e event ual l y fai l s al s o. P.3
o
o
b. The ri ght vent ri cl e al s o may fai l i n i s ol at i on from t he l eft vent ri cl e. In t he pres ence of chronic obstructive pulmonary disease (COPD), i ncreas ed pul monary vas cul ar res i s t ance devel ops as a res ul t of archi t ect ural changes i n t he l ungs . The hi gher pul monary vas cul ar res i s t ance produces a pres s ure overl oad on t he ri ght vent ri cl e, whi ch l eads t o i ncreas ed ri ght vent ri cul ar work and event ual fai l ure. Pul monary embol i s m and pri mary pul monary hypert ens i on are s ome ot her caus es of ri ght -s i ded fai l ure.
D. Clinical features
1. Symptoms o
o
a. Dyspnea i s t he mos t frequent l y encount ered s ympt om of CHF.
(1) The feel i ng of breat hl es s nes s i s caus ed by vascular congestion, whi ch reduces pul monary oxygenat i on. In addi t i on, t he vas cul ar conges t i on di mi ni s hes l ung compl i ance, i ncreas i ng t he work of breat hi ng, t hus addi ng t o t he feel i ng of breat hl es s nes s .
Pa g e 2 9
ABC Ambe r CHM Conve rte r Tria l ve rsion, http://w w w .proce sste x t.com/a bcchm.html
(2) Dys pnea al s o res ul t s from reduced cardiac output to the periphery, whi ch t ri ggers t he s ympt om t hrough neurohumoral mechani s ms . In t he earl y s t ages of CHF, dys pnea occurs onl y wi t h exert i on. As heart fai l ure progres s es , t he amount of exert i on requi red t o produce dys pnea becomes progres s i vel y l es s unt i l dys pnea may occur at res t .
o
o
b. Orthopnea refers t o dys pnea t hat occurs i n t he recumbent pos i t i on and i s rel i eved by el evat i on of t he head. Ort hopnea res ul t s from vol ume pool i ng i n t he cent ral vas cul at ure duri ng recumbency, whi ch l eads t o i ncreas ed cardi ac vol ume and, i n t urn, t o i ncreas ed l eft vent ri cul ar fi l l i ng pres s ure, pul monary conges t i on, and t he feel i ng of dys pnea. The phys i ci an may gauge t he degree of ort hopnea by not i ng t he number of pi l l ows t he pat i ent us es t o s l eep. However, i t s houl d be recogni zed t hat many pat i ent s s l eep on more t han one pi l l ow out of habi t , not becaus e of breat hl es s nes s . Nocturnal cough, whi ch has t he s ame pat hophys i ol ogy as ort hopnea, may occur t oget her wi t h, or i ns t ead of, noct urnal dys pnea.
o
o
c. Paroxysmal nocturnal dyspnea i s t he occurrence of s udden dys pnea t hat awakens t he pat i ent from s l eep. Li ke ort hopnea, i t occurs duri ng recumbency as a res ul t of pool i ng i n t he cent ral vas cul at ure, whi ch i ncreas es l eft vent ri cul ar fi l l i ng pres s ure. Paroxys mal noct urnal dys pnea may occur
Pa g e 3 0
ABC Ambe r CHM Conve rte r Tria l ve rsion, http://w w w .proce sste x t.com/a bcchm.html
i n t he ort hopnei c pat i ent who i nadvert ent l y s l i ps off t he pi l l ows us ed t o el evat e t he upper body. Us ual l y, t he pat i ent awakens from s l eep and feel s t he need t o s i t upri ght or t o go t o an open wi ndow for i ncreas ed vent i l at i on. The s ympt om us ual l y s ubs i des aft er t he pat i ent has been i n t he upri ght pos i t i on for 5–20 mi nut es . o
o
d. Nocturia devel ops i n CHF as a res ul t of i ncreas ed renal bl ood fl ow when t he pat i ent i s recumbent and as l eep.
(1) During the day, when t he s kel et al mus cl es are act i ve, l i mi t ed cardi ac out put i s s hi ft ed away from t he ki dney t oward t he s kel et al mus cul at ure. The ki dney i nt erpret s t hi s reduct i on i n bl ood fl ow as hypovolemia and becomes s odi um avi d vi a act i vat i on of t he renin–angiotensin system.
(2) At night, when t he pat i ent i s at res t , cardi ac out put i s s hi ft ed t oward t he ki dney, and di ures i s ens ues .
o
o
e. Edema. There are many caus es of peri pheral edema, s everal of whi ch are noncardi ac. Cardiac edema occurs when t he s ys t emi c hydros t at i c venous pres s ure i s great er t han t he s ys t emi c oncot i c venous pres s ure. Thus , cardi ac edema i s a s i gn of right-sided failure; i t occurs becaus e of t he i ncreas ed s ys t emi c venous pres s ure t hat res ul t s from ri ght vent ri cul ar dys funct i on.
Pa g e 3 1
ABC Ambe r CHM Conve rte r Tria l ve rsion, http://w w w .proce sste x t.com/a bcchm.html
2. Physical signs o
o
a. T achycardia. Increas ed heart rat e occurs i n heart fai l ure due t o i ncreas ed rel eas e of cat echol ami nes as a compens at ory mechani s m for mai nt ai ni ng cardi ac out put i n t he pres ence of decreas ed s t roke vol ume. Cat echol ami nes i ncreas e bot h t he force and t he rat e of cardi ac cont ract i on. However, i n chroni c heart fai l ure, adrenergi c downregul at i on occurs , and heart rat es over 100 bpm i n t he abs ence of arrhyt hmi a are di s t i nct l y unus ual .
o
o
b. Pulmonary rales. The i ncreas ed l eft -vent ri cul ar fi l l i ng pres s ure as s oci at ed wi t h CHF i s referred t o t he l eft at ri um and t he pul monary vei ns . The i ncreas ed hydros t at i c pres s ure produces t rans udat i on of fl ui d i nt o t he al veol i . As ai r ci rcul at es t hrough t he al veol i , crackl i ng s ounds (ral es ) are produced. Not e t hat t here are mul t i pl e caus es of pul monary ral es ; t he mere pres ence of ral es does not neces s ari l y i ndi cat e CHF. P.4
o
o
c. Cardiac enlargement. As t he fai l i ng heart rel i es more and more on t he Frank-St arl i ng mechani s m, i t di l at es and may devel op eccent ri c hypert rophy. In t he pres ence of cardi ac enl argement , t he poi nt of maxi mal i mpul s e (PMI) of t he l eft vent ri cl e i s s hi ft ed downward and t o t he l eft . Thi s s hi ft i s
Pa g e 3 2
ABC Ambe r CHM Conve rte r Tria l ve rsion, http://w w w .proce sste x t.com/a bcchm.html
det ect ed duri ng a phys i cal exami nat i on wi t h t he pat i ent l yi ng s upi ne. o
o
d. Fourth heart sound (S 4 ). Pat i ent s i n s i nus rhyt hm and heart fai l ure oft en have an S 4 (at ri al gal l op). The S 4 i s produced as l eft at ri al s ys t ol e propel s vol ume i nt o t he l eft vent ri cl e. In CHF, t he l eft vent ri cl e i s noncompl i ant and t he S 4 probabl y res ul t s from t he reverberat i on of t he bl ood eject ed i nt o t he l eft vent ri cl e. In el derl y pat i ent s , however, an S 4 may i ndi cat e reduced compl i ance of a s t i ff l eft vent ri cl e as a res ul t of agi ng rat her t han heart fai l ure. The S 4 al s o may be heard over t he ri ght vent ri cl e i n ri ght vent ri cul ar fai l ure.
o
o
e. T hird heart sound (S 3 ). An S 3 (ventricular gallop), whi ch occurs earl y i n di as t ol e, probabl y i s t he s i ngl e mos t reliable sign of left heart failure reveal ed duri ng phys i cal exami nat i on. The S 3 occurs duri ng rapi d fi l l i ng of t he l eft vent ri cl e. Increas ed l eft at ri al pres s ure (whi ch propel s t he bl ood forward wi t h i ncreas ed force) and noncompl i ance of t he l eft vent ri cl e are t wo i mport ant fact ors i n t he product i on of t hi s ext ra s ound. Al t hough an S 3 i s a rel i abl e s i gn of heart fai l ure i n i ndi vi dual s ol der t han age 40, a s i mi l ar s ound i s a normal fi ndi ng i n young, heal t hy at hl et es .
o
f. Neck vein distention. The neck vei ns can be cons i dered manomet ers at t ached t o t he ri ght at ri um and, as s uch, refl ect ri ght at ri al pres s ure [
det ect i on of cent ral venous pres s ure i s
des cri bed onl i ne (Onl i ne Fi gure 1-1)]. o
To es t i mat e cent ral venous pres s ure i n cm H 2 O, t he pat i ent 's back i s el evat ed or l owered s o t he poi nt
Pa g e 3 3
ABC Ambe r CHM Conve rte r Tria l ve rsion, http://w w w .proce sste x t.com/a bcchm.html
demarcat i ng t he di s t ended from t he nondi s t ended port i on of t he neck vei n can be di s cerned cl earl y. The vert i cal hei ght i s meas ured from t hi s poi nt t o t he manubri um. The average dept h of t he ri ght at ri um i ns i de t he ches t cavi t y (5 cm) i s added t o t he hei ght of t he neck vei n. Thi s s um approxi mat es t he ri ght at ri al pres s ure.
Online Figure 1-1 As s es s ment of jugul ar venous pres s ure s houl d be done wi t h t he head of t he bed at an angl e of 30 degrees . The hi ghes t l evel of jugul ar venous pul s at i on s houl d be not ed and a l i ne or rect angul ar object ext ended from t hi s poi nt paral l el t o t he ches t wal l . Meas ure t he hei ght above t he ches t wal l at t he s t ernal angl e and add 5 cm t o account for t he di s t ance bet ween t he ri ght at ri um and t he ches t wal l . o
o
g. Hepatic enlargement. El evat ed cent ral venous pres s ure can l ead t o hepat i c conges t i on, i n t urn caus i ng hepat omegal y. On occas i on, rapi d hepat i c enl argement may al s o caus e l i ver t endernes s .
o
o
h. Edema. Lower ext remi t y and pres acral edema occur i n ri ght -s i ded fai l ure as i ncreas ed venous
Pa g e 3 4
ABC Ambe r CHM Conve rte r Tria l ve rsion, http://w w w .proce sste x t.com/a bcchm.html
pres s ure res ul t s i n t rans udat i on of fl ui d i nt o t hes e areas . For edema t o be at t ri but abl e t o CHF, di s t ended neck vei ns i ndi cat i ve of el evat ed ri ght -s i ded fi l l i ng pres s ure al s o s houl d be pres ent . o
o
i. Ascites. Trans udat i on of fl ui d i nt o t he peritoneal space al s o may occur as a res ul t of i ncreas ed s ys t emi c venous pres s ure. W hen as ci t es i s caus ed by CHF, t he neck vei ns t ypi cal l y are el evat ed and t he l i ver i s di s t ended from pas s i ve conges t i on.
E. Diagnosis Management of CHF mus t focus on t he caus e of t he heart fai l ure, not s i mpl y on rel i evi ng t he s ympt oms . Al t hough a careful hi s t ory and phys i cal exami nat i on are t he mos t i mport ant t ool s avai l abl e i n arri vi ng at a di agnos i s , i n many cas es a di agnos i s may not be reached. In t hes e i ns t ances , t he fol l owi ng s t udi es oft en are hel pful :
1. The electrocardiogram (ECG) frequent l y i s nons peci fi c. However, t he pres ence of Q waves hel ps confi rm t hat MI has been t he caus e of t he CHF.
2. The chest radiograph i s us eful i n demons t rat i ng cardi ac chamber enl argement and i n document i ng conges t i on i n t he l ungs .
3. The echocardiogram i s es s ent i al i n i dent i fyi ng chamber enl argement and i n quant i fyi ng l eft vent ri cul ar funct i on and val vul ar funct i on. The mos t commonl y us ed des cri pt or of vent ri cul ar funct i on i s t he ejection fraction. Eject i on fract i ons bet ween 55% and 76% are normal .
Pa g e 3 5
ABC Ambe r CHM Conve rte r Tria l ve rsion, http://w w w .proce sste x t.com/a bcchm.html
4. Doppler interrogation meas ures t he di rect i on and vel oci t y of bl ood fl ow. Thi s t echni que i s us eful for det ect i ng bl ood fl ow movi ng i n an abnormal di rect i on, whi ch i s charact eri s t i c of val vul ar regurgi t at i on and i nt racardi ac s hunt s . In addi t i on, Doppl er i nt errogat i on can det ect and quant i fy val vul ar s t enos es by meas uri ng how much vel oci t y i s neces s ary t o mai nt ai n cons t ant bl ood fl ow t hrough a s t enot i c val ve.
5. Radionuclide ventriculography i s us ed t o meas ure ri ght and l eft vent ri cul ar eject i on fract i on. It i s an excel l ent noni nvas i ve procedure t o us e when i t i s neces s ary t o quant i fy preci s el y t he degree of s ys t ol i c cardi ac dys funct i on.
6. Duri ng cardiac catheterization, i nt racardi ac pres s ures , chamber s i ze, val vul ar s t enos i s , val vul ar regurgi t at i on, and coronary anat omy can be eval uat ed. Gi ven t he current accuracy of echocardi ography, cat het eri zat i on i s performed pri nci pal l y for t he eval uat i on of i s chemi a.
F. Therapy
1. Etiologic therapy. It i s i mport ant , when pos s i bl e, t o di rect t reat ment at t he et i ol ogi c agent of CHF. For exampl e, i f aort i c s t enos i s i s t he caus e, aort i c val ve repl acement i s t he mos t effect i ve t herapy. P.5
Pa g e 3 6
ABC Ambe r CHM Conve rte r Tria l ve rsion, http://w w w .proce sste x t.com/a bcchm.html
2. Systolic heart failure o
o
a. Therapy s hown t o enhance s urvi val and del ay progres s i on of t he di s eas e. Compens at i on for devel opment of heart fai l ure by i ncreas i ng t he neurohormones res pons i bl e for i ncreas i ng cardi ac cont ract i l i t y and heart rat e can i mprove s ympt oms of heart fai l ure i n t he s hort t erm. However, chroni c s t i mul at i on l eads t o cardi ac remodel i ng and decompens at i on (s ee I B 3). Bl ockade of t hes e agent s have been s hown t o revers e remodel i ng, i mprove mort al i t y, and del ay t he progres s i on of heart fai l ure.
(1) Renin-angiotensin system blockade. Angi ot ens i n-convert i ng enzyme (ACE) i nhi bi t ors have been s hown t o have a mort al i t y benefi t . Becaus e many vas odi l at ors have fai l ed t o i mprove s urvi val des pi t e reduci ng aft erl oad, propert i es of ACE i nhi bi t ors ot her t han vas odi l at i on are t hought t o be operat i ve, namel y revers al of remodel i ng. In pat i ent s who devel op coughi ng due t o ACE i nhi bi t ors , angi ot ens i n recept or bl ockers (ARBs ) may be s ubs t i t ut ed wi t h s i mi l ar effi cacy. Addi t i onal bl ockade of t he reni n-angi ot ens i n s ys t em, us i ng s pi ronol act one or epl erenone t o bl ock t he fi nal product of t hi s s ys t em, al dos t erone, i s al s o benefi ci al i n s el ect ed pat i ent s .
(2) β-Blocking agents. Becaus e s t i mul at i on of t he β-recept or i ncreas es t he force of
Pa g e 3 7
ABC Ambe r CHM Conve rte r Tria l ve rsion, http://w w w .proce sste x t.com/a bcchm.html
cardi ac cont ract i on, β-agoni s t s have been us ed i n t he t herapy of end-s t age CHF. [See 2 b (3) (b)]. Paradoxi cal l y, caut i ous us e of β-recept or ant agoni s t s has al s o been effect i ve i n revers i ng t he s ame s yndrome. The mechani s m of act i on for t hi s cl as s of agent s i n t he t reat ment of heart fai l ure s t ems from prot ect i on of t he heart from t he t oxi c effect s of prol onged expos ure t o t he hi gh l evel s of ci rcul at i ng cat echol ami nes .
(3) Hydralazine and nitrates. The combi nat i on of hydral azi ne and ni t rat es us ed i n cl as s II and III heart fai l ure has al s o been s hown t o i mprove mort al i t y by decreas i ng prel oad and aft erl oad and i mprovi ng cardi ac out put . However, t he mort al i t y benefi t wi t h ACE-i nhi bi t ors i s great er.
o
o
b. Symptomatic therapy. Al t hough not s hown t o i mprove mort al i t y, many drugs can i mprove t he s ympt oms and morbi di t y of heart fai l ure.
(1) Reducing afterload. Agent s t hat caus e art eri ol ar di l at i on reduce i mpedance of t he out fl ow of bl ood from t he l eft vent ri cl e. By di mi ni s hi ng res i s t ance t o eject i on, t hes e agent s caus e cardi ac out put t o ri s e becaus e t he l eft vent ri cl e can eject more compl et el y agai ns t a l ower aft erl oad. The net effect i s i ncreas ed cardi ac out put wi t hout a s eri ous fal l i n bl ood pres s ure l eadi ng t o s ympt omat i c i mprovement . Several vas odi l at ors are us ed i n
Pa g e 3 8
ABC Ambe r CHM Conve rte r Tria l ve rsion, http://w w w .proce sste x t.com/a bcchm.html
t he t reat ment of CHF t o reduce aft erl oad, i ncl udi ng ACE i nhi bi t ors , ni t rat es , and hydral azi ne.
(2) Reducing preload and left ventricular filling pressure. The i ncreas ed prel oad res ul t i ng from vol ume ret ent i on i n t he vent ri cl es i s a compens at ory mechani s m t hat hel ps i ncreas e forward cardi ac out put by us e of t he Frank-St arl i ng mechani s m; however, an exces s i ve i ncreas e i n prel oad i s as s oci at ed wi t h i ncreas ed l eft vent ri cul ar and ri ght vent ri cul ar fi l l i ng pres s ures , whi ch i s res pons i bl e for s ympt oms of pul monary and s ys t emi c conges t i on. Judi ci ous reduct i on i n fi l l i ng pres s ures wi t hout exces s i ve reduct i on i n prel oad i s i ndi cat ed i n t he t herapy of CHF.
(a) Di uret i cs reduce renal t ubul ar abs orpt i on of s odi um and wat er and i ncreas e t he cl earance of t hes e s ubs t ances from t he body. The res ul t i s a reduct i on i n cent ral vol ume and i n cardi ac fi l l i ng pres s ure.
(b) Vas odi l at ors , whi ch i ncreas e t he capaci t y of t he s ys t emi c venous s ys t em, t rans fer cent ral vol ume t o t he peri phery, t hus reduci ng cent ral prel oad and fi l l i ng pres s ure. Ni t rat es and ACE i nhi bi t ors are effect i ve prel oad-reduci ng vas odi l at ors .
(3) Increasing the contractile state.
Pa g e 3 9
ABC Ambe r CHM Conve rte r Tria l ve rsion, http://w w w .proce sste x t.com/a bcchm.html
Cont ract i l e dys funct i on i s t he mos t common mechani s m t hat produces heart fai l ure; t herefore, i ncreas i ng t he cont ract i l e s t at e may res ul t i n s ympt omat i c i mprovement .
(a) Cardi ac gl ycos i des (e.g., di goxi n) i ncreas e t he cont ract i l e s t at e by +
+
i mpedi ng t he Na /K -ATPas e-cont rol l ed i nt racel l ul ar pump. Thi s res ul t s i n t he net i nfl ux of cal ci um i nt o t he myocardi um, whi ch i ncreas es cont ract i l e s t rengt h.
(b) β-Adrenergi c agoni s t s (cat echol ami nes , e.g., dobut ami ne) i ncre as e cont ract i l e funct i on by i ncreas i ng t he product i on of cycl i c adenos i ne monophos phat e (cAMP), whi ch res ul t s i n great er myocardi al cal ci um rel eas e. They are admi ni s t ered as a cont i nuous i nfus i on i n end-s t age heart fai l ure as a t empori zi ng meas ure. P.6
(c) Phos phodi es t eras e i nhi bi t ors (e.g., mi l ri none) i ncreas e cont ract i l e funct i on by i nhi bi t i ng t he breakdown of cAMP. Si mi l ar t o β-adrenergi c agoni s t s , t hey are al s o admi ni s t ered as a cont i nuous i nt ravenous i nfus i on i n end-s t age heart fai l ure for s hort -t erm us e.
Pa g e 4 0
ABC Ambe r CHM Conve rte r Tria l ve rsion, http://w w w .proce sste x t.com/a bcchm.html
3. Diastolic heart failure. Unl i ke s ys t ol i c heart fai l ure, no medi cal t reat ment i s avai l abl e t hat reduces mort al i t y i n di as t ol i c heart fai l ure. Treat ment i s ai med at s ympt omat i c rel i ef. o
o
a. Diuretics. Di uret i cs are t he mai ns t ay of t reat ment . By reduci ng vent ri cul ar vol ume t hey l ower vent ri cul ar pres s ure and reduce t he s ympt oms of conges t i on. The dos i ng mus t be careful l y regul at ed, however, as reduced vent ri cul ar vol ume al s o l owers s t roke vol ume and cardi ac out put . The opt i mum range woul d be one t hat prevent s exces s dys pnea and hepat i c conges t i on but al l ows for adequat e cardi ac out put .
o
o
b. Induction of bradycardia. Sl owi ng t he heart rat e i ncreas es t he t i me avai l abl e for vent ri cul ar fi l l i ng. β-bl ockers and nondi hydropyri di ne cal ci um channel bl ockers (verapami l and di l t i azem) are us ed t o caus e rel at i ve bradycardi a.
o
o
c. Relief of ischemia. In pat i ent s wi t h coronary di s eas e, i s chemi a i mpai rs t he act i ve rel axat i on phas e of di as t ol e by i mpai ri ng cal ci um reupt ake by t he s arcopl as mi c ret i cul um. Thus , s t andard t herapy for angi na i s al s o effect i ve i n i mprovi ng di as t ol i c dys funct i on. [See III A 5 a (4)].
o
o
d. Maintenance of sinus rhythm. The normal at ri al “ki ck― afforded by at ri al cont ract i on i mproves t he effi ci ency of vent ri cul ar fi l l i ng t hat i s l os t i n
Pa g e 4 1
ABC Ambe r CHM Conve rte r Tria l ve rsion, http://w w w .proce sste x t.com/a bcchm.html
at ri al fi bri l l at i on. Thus , every effort s houl d be made t o mai nt ai n s i nus rhyt hm (s ee II).
4. Nonpharmacologic therapy o
o
a. Cardiac resynchronization. Many pat i ent s wi t h advanced heart fai l ure devel op el ect ri cal conduct i on di s t urbances , i ncl udi ng l eft bundl e branch bl ock, whi ch del ays t he i mpul s e s i gnal i ng cont ract i on from get t i ng t o t he l eft vent ri cl e. Thi s del ay di s coordi nat es cont ract i on, i n t urn caus i ng furt her reduct i on i n cardi ac out put . Ins ert i on of a pacemaker t o res ynchroni ze cont ract i on i n pat i ent s wi t h conduct i on del ays i mproves funct i onal capaci t y and qual i t y of l i fe and decreas es mort al i t y.
o
o
b. Physical conditioning. An i mport ant adjunct t o t he medi cal t reat ment of CHF, phys i cal condi t i oni ng permi t s t he peri pheral t i s s ues t o us e cardi ac out put more effi ci ent l y. Thus , t he pat i ent experi ences an i ncreas e i n t ol erance t o phys i cal act i vi t y wi t hout an i ncreas e i n cardi ac out put .
o
o
c. Cardiac transplantation may offer an i mproved qual i t y of l i fe t o s el ect ed pat i ent s i n whom cont rol of CHF i s not pos s i bl e and prognos i s i s poor. Current l y, approxi mat el y 75% of pat i ent s undergoi ng cardi ac t rans pl ant at i on achi eve a 5-year s urvi val rat e. The pauci t y of cardi ac donors i s t he pri mary fact or l i mi t i ng t he us e of t hi s t herapy. Left vent ri cul ar as s i s t devi ces (LVADs ) may provi de a bri dge t o t rans pl ant at i on.
Pa g e 4 2
ABC Ambe r CHM Conve rte r Tria l ve rsion, http://w w w .proce sste x t.com/a bcchm.html
5. Pulmonary edema i s t he mos t ext reme exampl e of CHF i n whi ch profound t rans udat i on of fl ui d i nt o t he pul monary al veol i occurs becaus e of a hi gh l eft vent ri cul ar fi l l i ng pres s ure. The res ul t i s i mpai red oxygenat i on and, i f unt reat ed, deat h. The goal of t herapy i s t o i mprove oxygenat i on, t o reduce l eft vent ri cul ar fi l l i ng pres s ure, and t o i ncreas e forward cardi ac out put . o
o
a. Oxygen s houl d be admi ni s t ered by facemas k becaus e pat i ent s i n pul monary edema are s o dys pnei c t hat t hey breat he pri mari l y t hrough t hei r mout hs .
o
o
b. Diuretics. Furos emi de i s l i kel y t he s i ngl e mos t commonl y us ed medi cat i on i n t he t reat ment of acut e pul monary edema. Thi s rapi d-act i ng l oop di uret i c promot es an i mmedi at e di ures i s i n mos t cas es .
o
o
c. Morphine sulfate. Thi s opi oi d reduces pat i ent anxi et y, whi ch may hel p rel i eve t he art eri al vas ocons t ri ct i on oft en pres ent i n acut e pul monary edema. Thi s , i n t urn, hel ps i ncreas e forward cardi ac out put . Morphi ne i s al s o a venodi l at or; t herefore, i t reduces cent ral vol ume and l eft vent ri cul ar fi l l i ng pres s ure.
o
o
d. Other vasodilators. Ni t rogl yceri ne (admi ni s t ered s ubl i ngual l y or i nt ravenous l y) or ni t roprus s i de (admi ni s t ered i nt ravenous l y) i s oft en effect i ve i n
Pa g e 4 3
ABC Ambe r CHM Conve rte r Tria l ve rsion, http://w w w .proce sste x t.com/a bcchm.html
t reat i ng pul monary edema when ot her t herapi es fai l . Nes i ri t i de, an anal og of B-t ype nat ri uret i c hormone t hat caus es venous and art eri ol ar vas odi l at i on may al s o be benefi ci al . However, t he pot ent vas odi l at i ng abi l i t y of t hes e drugs requi res t hat bl ood pres s ure be moni t ored cons t ant l y duri ng admi ni s t rat i on t o avoi d hypot ens i on. P.7
o
o
e. Intravenous positive inotropic agents (dobut ami ne, mi l ri none) may be us ed i f peri pheral perfus i on i s compromi s ed. They wi l l hel p t o i ncreas e cont ract i l i t y and i mprove cardi ac out put and al s o have s ome aft erl oad reduct i on propert i es t hat wi l l al l ow i ncreas ed forward fl ow i n acut e decompens at i on.
o
o
f. Intubation and positive-pressure ventilation. If t he pat i ent 's oxygenat i on does not i mprove rapi dl y wi t h t he previ ous l y not ed t herapi es , i nt ubat i on or pos i t i ve-pres s ure vent i l at i on may be neces s ary t o provi de mechani cal vent i l at i on and i mprove oxygenat i on.
o
o
g. Invasive hemodynamic monitoring. Mos t cas es of pul monary edema res ol ve qui ckl y, maki ng hemodynami c moni t ori ng unneces s ary. However, i n cas es of refract ory pul monary edema wi t h s evere cardi ac compromi s e, exact knowl edge of i nt racardi ac fi l l i ng pres s ure may be us eful i n gui di ng t herapy. Hemodynami c moni t ori ng vi a
Pa g e 4 4
ABC Ambe r CHM Conve rte r Tria l ve rsion, http://w w w .proce sste x t.com/a bcchm.html
Swan-Ganz cat het eri zat i on provi des t hi s i nformat i on s o t hat opt i mal fi l l i ng pres s ure and cardi ac out put may be obt ai ned.
II. Cardiac Arrhythmias A. Introduction Bradyarrhyt hmi as res ul t from i nadequat e s i nus nodal i mpul s e product i on or from bl ocked i mpul s e propagat i on, and us ual l y are not caus e for concern unl es s s yncope or pres yncope devel ops . Sus t ai ned at ri al t achyarrhyt hmi as us ual l y permi t adequat e cardi ac out put and are l es s dangerous t han s us t ai ned vent ri cul ar arrhyt hmi as , whi ch oft en caus e col l aps e or deat h.
B. Atrial tachyarrhythmias The at ri al t achyarrhyt hmi as can be cl as s i fi ed i nt o t wo s ubcat egori es : t hos e t hat produce a regul ar cardi ac rhyt hm and t hos e t hat produce an i rregul ar cardi ac rhyt hm. In general , at ri al t achyarrhyt hmi as do not i nt erfere wi t h i nt er- or i nt ravent ri cul ar conduct i on of t he cardi ac i mpul s e, and t herefore t he QRS compl ex (whi ch i s generat ed from t he vent ri cl es ) remai ns narrow i n form. Occas i onal l y, however, at ri al arrhyt hmi as caus e aberrant vent ri cul ar conduct i on wi t h a wi de QRS compl ex, whi ch may mi mi c arrhyt hmi as of vent ri cul ar ori gi n.
1. Regular atrial tachycardias o
o
a. Sinus tachycardia. Si nus t achycardi a repres ent s an i ncreas e i n t he s i nus rat e (>100 bpm) and i s us ual l y s econdary t o s ome ot her di s eas e proces s . In general , t he phys i ci an s houl d t reat t he condi t i on t hat i s caus i ng t he s i nus t achycardi a, not t he t achycardi a i t s el f. However, i n s ome pat i ent s , s uch as t hos e wi t h coronary art ery di s eas e, s i nus t achycardi a mus t be cont rol l ed t o prevent
Pa g e 4 5
ABC Ambe r CHM Conve rte r Tria l ve rsion, http://w w w .proce sste x t.com/a bcchm.html
myocardi al i s chemi a. In t hi s i ns t ance, β-bl ocki ng agent s or cal ci um ant agoni s t s (ei t her verapami l or di l t i azem) may be effect i ve i n cont rol l i ng heart rat e. o
o
b. Supraventricular tachycardia (SVT ). The mos t common form of SVT i s AV nodal reent ry t achycardi a (AVNRT).
(1) ECG identification. In SVT, t he ECG s hows regul ar, narrow QRS compl exes wi t hout i dent i fi abl e P waves at a rat e of 150–200 bpm (Fi gure 1-2A).
(2) T herapy. AVNRT i s due t o re-ent ry wi t hi n t he compact AV node. Therapi es t hat s l ow conduct i on or i ncreas e refract ori nes s us ual l y s ucceed.
(a) Mechanical maneuvers s uch as carotid sinus massage and t he Valsalva maneuver are oft en effect i ve i n t ermi nat i ng t he arrhyt hmi a. Thes e are es peci al l y i mport ant becaus e t hey can be s el f-admi ni s t ered by t he pat i ent pri or t o s eeki ng medi cal as s i s t ance.
(b) Medical therapy. Intravenous (IV) adenosine i s t he medi cal t reat ment of choi ce. Les s commonl y us ed agent s i ncl ude verapamil or β-blockers.
o
Pa g e 4 6
ABC Ambe r CHM Conve rte r Tria l ve rsion, http://w w w .proce sste x t.com/a bcchm.html
o
c. Atrial flutter us ual l y occurs i n pat i ent s wi t h ant ecedent heart di s eas e, i ncl udi ng coronary art ery di s eas e, peri cardi t i s , val vul ar heart di s eas e, and cardi omyopat hy. At ri al fl ut t er i s charact eri zed by an at ri al rat e of 220–400 bpm and i s us ual l y conduct ed t o t he vent ri cl e wi t h bl ock s o t hat t he vent ri cul ar rat e i s a fract i on of t he at ri al rat e. Occas i onal l y, at ri al fl ut t er may be conduct ed i rregul arl y. Treat ment i s t he s ame as for regul ar at ri al fl ut t er.
(1) ECG identification. As s hown i n Fi gure 1-2B, t hi s arrhyt hmi a produces a cl as s i c s awt oot h pat t ern on t he ECG. In t hi s exampl e, t here i s 4:1 AV bl ock wi t h an at ri al rat e of 300 bpm and a vent ri cul ar rat e of 75 bpm.
(2) T herapy
(a) Al t hough i nt ravenous admi ni s t rat i on of digoxin, esmolol, or verapamil may be effect i ve i n convert i ng t he arrhyt hmi a t o normal s i nus rhyt hm, convers i on i s l es s l i kel y t han P.8
i n paroxys mal at ri al t achycardi a. Us ual l y t hes e medi cat i ons cont rol t he vent ri cul ar res pons e, whi ch, i n t urn, hel ps mai nt ai n hemodynami c s t abi l i t y. Once t he vent ri cul ar rat e has s l owed as a res ul t of i ncreas ed AV bl ock (3:1 or 4:1 conduct i on), ibutilide or anot her ant i arrhyt hmi c
Pa g e 4 7
ABC Ambe r CHM Conve rte r Tria l ve rsion, http://w w w .proce sste x t.com/a bcchm.html
agent can be admi ni s t ered t o res t ore s i nus rhyt hm.
FIGURE 1-2 At ri al t achycardi as : A. Supravent ri cul ar t achycardi a, B. At ri al fl ut t er, C. At ri al fi bri l l at i on, D. Mul t i focal at ri al t achycardi a. (From St ei n E. Rapi d Anal ys i s of Arrhyt hmi as : A Sel f-St udy Program. 3rd ed. Phi l adel phi a: Li ppi ncot t W i l l i ams & W i l ki ns , 1999:91,66,68. )
(b) If medi cal t herapy does not convert t he pat i ent t o normal s i nus rhyt hm, at ri al fl ut t er wi l l us ual l y s el f-convert over t i me, ei t her t o at ri al fi bri l l at i on or t o normal s i nus rhyt hm.
(c) As i n al l arrhyt hmi as , direct current (DC) cardioversion i s neces s ary i f t he arrhyt hmi a has al ready produced hemodynami c i ns t abi l i t y.
2. Irregular atrial tachycardias o
o
a. Atrial fibrillation. At ri al fi bri l l at i on i s an
Pa g e 4 8
ABC Ambe r CHM Conve rte r Tria l ve rsion, http://w w w .proce sste x t.com/a bcchm.html
irregularly irregular arrhyt hmi a i n whi ch t here i s no ordered cont ract i on of t he at ri a but , rat her, mul t i pl e di s coordi nat e wave front s of depol ari zat i on t hat s end a l arge number of i rregul ar i mpul s es t o depol ari ze t he AV node. The i rregul ar i mpul s es produce an i rregul ar vent ri cul ar res pons e, t he rat e of whi ch depends on t he number of i mpul s es t hat are conduct ed t hrough t he AV node. Sel ect ed caus es of at ri al fi bri l l at i on i ncl ude s t res s , fever, exces s i ve al cohol i nt ake, vol ume depl et i on, peri cardi t i s , coronary art ery di s eas e, MI, pul monary embol i s m, mi t ral val ve di s eas e, t hyrot oxi cos i s , and i di opat hi c at ri al fi bri l l at i on.
(1) ECG identification. An exampl e of at ri al fi bri l l at i on i s s hown i n Fi gure 1-2C.
(2) T herapy
(a) If pat i ent s wi t h at ri al fi bri l l at i on are hemodynami cal l y uns t abl e or demons t rat e an i ncreas e i n angi na pect ori s or wors eni ng of CHF, i mmedi at e DC s ynchronous cardi overs i on i s i ndi cat ed.
(b) If pat i ent s are hemodynami cal l y s t abl e, t he phys i ci an s houl d fi rs t focus on cont rol l i ng t he vent ri cul ar res pons e t o t he at ri al fi bri l l at i on by t he i nt ravenous admi ni s t rat i on of β-bl ockers , nondi hydropyri di ne cal ci um channel bl ockers , or di goxi n.
Pa g e 4 9
ABC Ambe r CHM Conve rte r Tria l ve rsion, http://w w w .proce sste x t.com/a bcchm.html
(c) In t he abs ence of a cont rai ndi cat i on, systemic anticoagulation s houl d be i ns t i t ut ed wi t h unfract i onat ed or l ow–mol ecul ar-wei ght hepari n (LMW H). Oral warfari n s houl d be s t art ed aft er hepari n.
(d) Cardioversion can be cons i dered i mmedi at el y aft er a t rans es ophageal echocardi ogram, provi ded no i nt racardi ac t hrombus i s vi s ual i zed, or aft er 4–6 weeks of adequat e oral ant i coagul at i on wi t h warfari n.
(e) In t he abs ence of a revers i bl e caus e, long-term anticoagulation s houl d be cons i dered.
o
o
b. Multifocal atrial tachycardia. In t hi s arrhyt hmi a, t here i s s ynchronous at ri al cont ract i on, but t he cont ract i on ari s es from many s i t es i n t he at ri a, not from t he s i nus node. In t he majori t y of cas es of mul t i focal at ri al t achycardi a, pat i ent s have s i gni fi cant medi cal comorbi di t i es .
(1) ECG identification. The mul t i pl e s i t es of ori gi n of t he at ri al cont ract i on produce many di fferent P-wave confi gurat i ons and di fferent R-R i nt erval s . At l eas t t hree di fferent P-wave morphol ogi es are requi red t o make t hi s
Pa g e 5 0
ABC Ambe r CHM Conve rte r Tria l ve rsion, http://w w w .proce sste x t.com/a bcchm.html
di agnos i s (Fi gure 1-2D). P.9
(2) T herapy. Treat ment i s di rect ed pri mari l y at managi ng medi cal comorbi di t i es . If t hes e meas ures fai l , β-bl ockers or nondi hydropyri di ne cal ci um channel bl ockers may be us eful i n cont rol l i ng t he heart rat e.
C. Bradyarrhythmias Bradyarrhythmias occur when s i nus node i mpul s e generat i on i s s l owed or when normal s i nus node i mpul s es cannot be conduct ed t o t he vent ri cl es becaus e of AV nodal bl ock or vent ri cul ar conduct i ng s ys t em di s eas e. In general , bradyarrhyt hmi as are a caus e for concern onl y when pat i ent s have become s ympt omat i c wi t h pres yncope or s yncope from t he reduced cardi ac out put t hat t he l ow heart rat e produces . Medi cat i ons s uch as β-bl ockers , nondi hydropyri di ne cal ci um channel bl ockers , and di goxi n may cont ri but e t o bradyarrhyt hmi as .
1. Sinus bradycardia o
o
a. Si nus bradycardi a may be a phys i ol ogi c and normal res pons e t o cardi ovas cul ar condi t i oni ng, as i n t rai ned at hl et es . In s uch cas es , t he arrhyt hmi a i s obvi ous l y a normal fi ndi ng and requi res no t herapy. However, ext reme s i nus bradycardi a (< 35 bpm) as a res ul t of s i nus node dys funct i on may caus e s ympt oms .
o
o
b. T herapy. Atropine may be us eful i n t emporari l y
Pa g e 5 1
ABC Ambe r CHM Conve rte r Tria l ve rsion, http://w w w .proce sste x t.com/a bcchm.html
i ncreas i ng t he s i nus rat e. Withdrawal of offending medications may be neces s ary.
2. Sinus pauses. Thi s bradyarrhyt hmi a i s caus ed by t he fai l ure of t he s i nus node t o generat e an i mpul s e on t i me. Such paus es may l as t for s everal s econds and i nduce s yncope. Defi ni t i ve t herapy requi res pacemaker i mpl ant at i on.
3. AV block. In AV bl ock, s ome of t he i mpul s es generat ed from t he s i nus node are not conduct ed t o t he vent ri cl es . o
o
a. T ypes of AV block (Fi gure 1-3)
(1) First-degree AV block. Every at ri al beat i s conduct ed t o t he vent ri cl e, but t he P-R i nt erval exceeds 0.20 s econds and remai ns cons t ant (Fi gure 1-3A). Thi s i s mos t commonl y due t o a del ay i n t he AV node. Treat ment i s rarel y neces s ary.
(2) Second-degree AV block
(a) Mobitz type I (Wenckebach) block. There i s a progres s i ve prol ongat i on i n t he P-R i nt erval unt i l a generat ed P wave i s not conduct ed (Fi gure 1-3B). Thi s t ype of bl ock us ual l y occurs at t he l evel of t he AV node. It rarel y produces s ympt oms .
Pa g e 5 2
ABC Ambe r CHM Conve rte r Tria l ve rsion, http://w w w .proce sste x t.com/a bcchm.html
(b) Mobitz type II block. There i s no prol ongat i on of t he P-R i nt erval before t he dropped beat (Fi gure 1-3C). Oft en conduct i on i n a 2:1 rat i o i s of s uffi ci ent durat i on t o caus e s ympt omat i c bradycardi a. Thi s t ype of bl ock can occur at t he AV node or i n t he Hi s -Purki nje s ys t em. Mobi t z t ype II bl ock i s t he t ype of great es t concern becaus e i t can progres s t o compl et e heart bl ock.
FIGURE 1-3 AV bl ocks . A. Fi rs t degree AV bl ock. B. Type II AV bl ock, Mobi t z I. C. Type II AV bl ock, Mobi t z II. D. Thi rd degree/compl et e AV bl ock. (From St ei n E. Rapi d Anal ys i s of Arrhyt hmi as : A Sel f-St udy Program. 3rd ed. Phi l adel phi a: Li ppi ncot t W i l l i ams & W i l ki ns , 1999:127,131,132,135. ) o
P.10
o
(3) T hird degree/complete heart block. No i mpul s es are conduct ed (Fi gure 1-3D), and t he vent ri cul ar rat e becomes dependent on s pont aneous vent ri cul ar depol ari zat i ons .
Pa g e 5 3
ABC Ambe r CHM Conve rte r Tria l ve rsion, http://w w w .proce sste x t.com/a bcchm.html
Severe s ympt omat i c bradycardi a charact eri zed by a heart rat e of 25–40 bpm i s s een.
b. T herapy. Atropine and isoproterenol are effect i ve i n t emporari l y i ncreas i ng t he vent ri cul ar res pons e for bl ock occurri ng at t he AV node. Conduct i on bl ock bel ow t he l evel of t he AV node requi res i ns ert i on of ei t her a t emporary or a permanent cardiac pacemaker.
D. Ventricular tachyarrhythmias
1. T ypes of ventricular arrhythmias (Figure 1-4) o
o
a. Premature ventricular contraction (PVC). In t hi s arrhyt hmi a, heart beat s ari s e di rect l y from t he vent ri cl es , bypas s i ng t he s peci al i zed cardi ac Hi s -Purki nje conduct i on s ys t em.
(1) ECG identification
(a) Becaus e t he Hi s -Purki nje s ys t em i s bypas s ed, t he QRS configuration i s t ypi cal l y widened and bizarre i n appearance.
(b) PVCs us ual l y do not affect at ri al depol ari zat i on, whi ch proceeds normal l y and i n di s s oci at i on wi t h t he PVC. Thus , t he next s i nus beat occurs at t he s ame
Pa g e 5 4
ABC Ambe r CHM Conve rte r Tria l ve rsion, http://w w w .proce sste x t.com/a bcchm.html
t i me i t woul d have occurred i f no PVC had been pres ent . Accordi ngl y, a full compensatory pause us ual l y fol l ows a PVC (Fi gure 1-4A). The normal l y occurri ng P wave i s us ual l y buri ed i n t he PVC–QRS compl ex.
(2) T herapy. Mos t i s ol at ed PVCs are beni gn and s houl d not be t reat ed.
o
o
b. Ventricular tachycardia i s a regul ar rhyt hm t hat occurs paroxys mal l y and exceeds 120 bpm. AV dissociation, whi ch caus es t he vent ri cul ar arrhyt hmi a t o proceed i ndependent l y of t he normal at ri al rhyt hm, i s t he hal l mark of t he arrhyt hmi a. Duri ng vent ri cul ar t achycardi a, cardi ac rel axat i on i s i mpai red. Thi s fact or, t oget her wi t h l os s of AV s ynchrony (i .e., l os s of t he at ri al “ki ck―) and l os s of t he el ect ri cal coordi nat i on of t he cont ract i on by t he Hi s -Purki nje s ys t em, us ual l y l eads t o s everel y reduced cardi ac out put , produci ng hypot ens i on. Sustained ventricular tachycardia is usually a life-threatening arrhythmia that degenerates into ventricular fibrillation and death if untreated.
(1) Physical examination. In many cas es of vent ri cul ar t achycardi a, s evere decompens at i on precl udes performance of a det ai l ed phys i cal exami nat i on.
(a) If pat i ent s are rel at i vel y s t abl e, phys i cal di agnos i s reveal s AV
Pa g e 5 5
ABC Ambe r CHM Conve rte r Tria l ve rsion, http://w w w .proce sste x t.com/a bcchm.html
di s s oci at i on, mani fes t ed by t he appearance of cannon a waves i n t he neck vei ns . Thes e waves occur i nt ermi t t ent l y when ri ght at ri al cont ract i on occurs duri ng vent ri cul ar cont ract i on. Becaus e t he at ri al bl ood cannot go forward agai ns t t he cl os ed t ri cus pi d val ve, backward fl ow produces a l arge bul ge i n t he neck vei ns .
FIGURE 1-4 Vent ri cul ar arrhyt hmi as . A. Premat ure vent ri cul ar cont ract i on, B. Monomorphi c vent ri cul ar t achycardi a, C. Pol ymorphi c vent ri cul ar t achycardi a (t ors ades de poi nt es ), D. Vent ri cul ar fi bri l l at i on. (From St ei n E. Rapi d Anal ys i s of Arrhyt hmi as : A Sel f-St udy Program. 3rd ed. Phi l adel phi a: Li ppi ncot t W i l l i ams & W i l ki ns , 1999:100,110–112. )
P.11
(b) Variability in intensity of the S 1 , caus ed by vari abl e pos i t i oni ng of at ri al and vent ri cul ar cont ract i on, can be aus cul t at ed.
Pa g e 5 6
ABC Ambe r CHM Conve rte r Tria l ve rsion, http://w w w .proce sste x t.com/a bcchm.html
(2) ECG identification (Fi gure 1-4B,C)
(a) The QRS complex is widened and bizarre in appearance becaus e t he arrhyt hmi a does not us e t he s peci al i zed conduct i ng s ys t em of t he heart . Becaus e t he vent ri cl es are operat i ng i ndependent l y of t he at ri a, t here i s no constant relationship between the P wave and the QRS complex.
(b) The QRS compl ex may be monomorphic (i .e., of one s hape), as s hown i n Fi gure 1-4B, or i t may be polymorphic, as s hown i n Fi gure 1-4C. W hen t he axi s of a pol ymorphi c arrhyt hmi a appears t o revol ve about a cent ral poi nt and i s as s oci at ed wi t h a prol onged QT i nt erval , t he arrhyt hmi a i s t ermed torsades de pointes.
(3) T herapy
(a) In hemodynami cal l y uns t abl e pat i ent s , DC cardioversion i s urgent l y requi red.
(b) St abl e pat i ent s wi t h monomorphi c vent ri cul ar t achycardi a may be t reat ed wi t h IV amiodarone.
Pa g e 5 7
ABC Ambe r CHM Conve rte r Tria l ve rsion, http://w w w .proce sste x t.com/a bcchm.html
(c) Pol ymorphi c vent ri cul ar t achycardi a requi res defi bri l l at i on. Common caus es are i s chemi a, el ect rol yt e abnormal i t i es , and drugs t hat prol ong t he QT i nt erval .
c. Ventricular fibrillation i s charact eri zed by a l ack of
ordered cont ract i on of t he vent ri cl es ; t herefore, t here i s no cardi ac out put . Thus , ventricular fibrillation is synonymous with death unless conversion to an effective rhythm can be accomplished. The phys i ci an s houl d begi n cardi ac res us ci t at i on, i ncl udi ng mechani cal vent i l at i on, cardi ac compres s i on, and drug and el ect ri cal t herapy, i mmedi at el y on recogni zi ng t he pres ence of vent ri cul ar fi bri l l at i on (Fi gure 1-4D).
2. Prevention and treatment of ventricular arrhythmias. Onl i ne Tabl e 1-1 l i s t s t he current cl as s i fi cat i on and s ome of t he s i de effect s of drugs current l y avai l abl e for l ong-t erm prevent i on of vent ri cul ar arrhyt hmi as . Drug t herapy i s us ed when a pat i ent has had s ympt omat i c or s us t ai ned vent ri cul ar t achycardi a or fi bri l l at i on. Today, i ns ert i on of an i mpl ant abl e cardi ac defi bri l l at or accompani es ant i -arrhyt hmi c agent s i n mos t cas es of l i fe-t hreat eni ng arrhyt hmi as . Some pat i ent s wi t h heart fai l ure s ympt oms and decreas ed l eft vent ri cul ar eject i on fact i on may qual i fy for i ns ert i on of an i mpl ant abl e cardi ac defi bri l l at or for pri mary prevent i on of s udden cardi ac deat h.
ONLINE TABLE 1-1 Antiarrhythmic Agents
Pa g e 5 8
ABC Ambe r CHM Conve rte r Tria l ve rsion, http://w w w .proce sste x t.com/a bcchm.html
Cl Maj a
or
ss Elec trop hysi olog ic P S M r p aj o e o p ci r e fi Si rt c d ie A e s g Ef e fe nt ct s s S Ia Q Pr o In ui o di hi ni ar u bi di rh m t n yt ch ra e h a pi
m
n d
ia
n in el w ar d cu rr e nt Bl Pr
Di
oc ol
ar
Pa g e 5 9
ABC Ambe r CHM Conve rte r Tria l ve rsion, http://w w w .proce sste x t.com/a bcchm.html
k o
rh
er n
e
s g
a
re p ol ar iz at io n Pr Pr oc o ai ar n rh a yt mh id m e ia L u p u sli k e sy n dr o m e Di Pr s o o ar
Pa g e 6 0
ABC Ambe r CHM Conve rte r Tria l ve rsion, http://w w w .proce sste x t.com/a bcchm.html
p rh yr yt a h mm id ia e C o n g e st iv e h e ar t fa il ur e A nt ic h ol in er gi c ef fe ct s Ib Li C
Pa g e 6 1
ABC Ambe r CHM Conve rte r Tria l ve rsion, http://w w w .proce sste x t.com/a bcchm.html
In d N hi oc S bi ai ef t n fe ra e ct pi
s
d in w ar d cu rr e nt Pr o ar rh yt h m ia C N S ef fe ct s A T N cc oc e el ai ut er ni ro at d p e e e
Pa g e 6 2
ABC Ambe r CHM Conve rte r Tria l ve rsion, http://w w w .proce sste x t.com/a bcchm.html
re
ni
p
a
ol ar iz at io n M C e N xi S l e ef t i fe n ct e s P C h N e S n ef yt fe oi ct n s Ic Fl C In ec o hi ai n bi ni g t d e ra e s t pi
iv
d
e
in
h
w
e
ar
ar
d
t
cu
fa
rr
il
Pa g e 6 3
ABC Ambe r CHM Conve rte r Tria l ve rsion, http://w w w .proce sste x t.com/a bcchm.html
e
ur
nt Li
e Pr
tt
o
le
ar
ef
rh
fe
yt
ct
h
o
m
n
ia
re p ol ar iz at io n β II Pr Br -b R o a l o e pr d ck d a yc er uc n ar s e ol di i s ol a ch e m ia Br o nc h o s p
Pa g e 6 4
ABC Ambe r CHM Conve rte r Tria l ve rsion, http://w w w .proce sste x t.com/a bcchm.html
a s m C o n g e st iv e h e ar t fa il ur e R A C e ce o d b n uc ut g e ol e s y ol s t m
iv
p
e
at
h
h
e
et
ar
ic
t
ar
fa
rh
il
yt
ur
h
e
m
Pa g e 6 5
ABC Ambe r CHM Conve rte r Tria l ve rsion, http://w w w .proce sste x t.com/a bcchm.html
o g e ni ci ty P II A P ot I m ul a Pr i o m s s ol d o i u o ar n m n o ar ch g n y a ac e fi n ti
br
n o
o
el n
si
p
s
ot e nt ia l d ur at io n bl
H
oc
y
k
p
er
o-
s
a n d h
Pa g e 6 6
ABC Ambe r CHM Conve rte r Tria l ve rsion, http://w w w .proce sste x t.com/a bcchm.html
y p er th yr oi di s m Br O et rt yl h iu o m st at ic h y p ot e n si o n S Pr ot o al ar ol rh yt h m ia Ib Pr ut o i l i ar
Pa g e 6 7
ABC Ambe r CHM Conve rte r Tria l ve rsion, http://w w w .proce sste x t.com/a bcchm.html
d rh e yt h m ia C IV V Br al D er a ci e a d u pr p yc m e a ar ch s s m di a sl il a n o (a n w tr el i n i a bl w l oc ar ar k d rh er cu yt s rr h e m nt i a s) CNS, cent ral nervous s ys t em.
III. Ischemic Heart Disease A. Atherosclerotic coronary artery disease (ASCAD)
1. Definition. ASCAD i s t he focal narrowi ng of t he coronary art eri es as a res ul t of a plaque compos ed of:
Pa g e 6 8
ABC Ambe r CHM Conve rte r Tria l ve rsion, http://w w w .proce sste x t.com/a bcchm.html o
o
a. Lipids (chol es t erol es t ers and crys t al s ), whi ch are depos i t ed at t he cent er of t he pl aque and accumul at e wi t hi n macrophages
o
o
b. Intimal secretory smooth muscle cells, whi ch prol i ferat e
o
o
c. A fibrous cap made of connect i ve t i s s ue
2. Incidence and risk factors. Current l y i n t he Uni t ed St at es , t he overal l i nci dence of deat h as a res ul t of ASCAD i s 0.5 i n 1,000 and decreas i ng. However, ASCAD di ffers i n frequency i n s ubpopul at i ons wi t h t he fol l owi ng ri s k fact ors : o
o
a. Age. The i nci dence of ASCAD i ncreas es progres s i vel y wi t h age. The ri s k of deat h i s 1.5 i n 1,000 i ndi vi dual s at age 50.
o
o
b. Gender. ASCAD i s more preval ent i n men t han i n women. Thi s di fference i s mos t marked i n premenopaus al women compared wi t h men of s i mi l ar age. By t he t i me men reach t he age of 50 years , t hey are affect ed fi ve t i mes more oft en t han women of t he s ame age. Thi s di fference decl i nes as age i ncreas es .
o
c. Serum cholesterol. The i nci dence of ASCAD i ncreas es wi t h i ncreas i ng t ot al s erum chol es t erol l evel s
(Onl i ne Fi gure 1-5).
Pa g e 6 9
ABC Ambe r CHM Conve rte r Tria l ve rsion, http://w w w .proce sste x t.com/a bcchm.html o
ONLINE FIGURE 1-5 Graph s howi ng t he effect of i ncreas i ng s erum chol es t erol concent rat i on on t he i nci dence of at heros cl erot i c coronary art ery di s eas e i n men age 30–49 years . (Adapt ed from Cohn PF: Di agnos i s and T herapy of Coronary Art ery Di s eas e. Bos t on: Li t t l e, Brown, 1979:27. )
(1) Tot al s erum chol es t erol i s carri ed i n t he bl ood by low-density lipoprotein (LDL), very low-density lipoprotein (VLDL), and high-density lipoprotein (HDL).
(a) The hi gher t he percent age of t ot al chol es t erol carri ed by LDL i n rel at i on t o HDL, t he hi gher t he ri s k of ASCAD. Hi gh l evel s of HDL s eem t o be prot ect i ve. P.12
Pa g e 7 0
ABC Ambe r CHM Conve rte r Tria l ve rsion, http://w w w .proce sste x t.com/a bcchm.html
(b) Tot al chol es t erol l evel s houl d be l es s t han 200 mg/dL.
(i) The LDL chol es t erol l evel s houl d be l es s t han 130 mg/dL; i n pat i ent s wi t h known coronary di s eas e, i t s houl d be l es s t han 100 mg/dL. In hi gh-ri s k pat i ent s , cons i der l oweri ng t o 70 mg/dL.
(ii) The HDL chol es t erol l evel s houl d exceed 40 mg/dL.
(2) Several t ypes of hyperl i pi demi a exi s t , and many are as s oci at ed wi t h an i ncreas ed i nci dence of coronary art ery di s eas e.
Onl i ne Tabl e 1-2 pres ent s an overvi ew of t he
hyperl i pi demi as .
ONLINE TABLE 1-2 The Hyperlipidemias T Li Li Cl y pi p in p d o ic e
p al
of
r
H
ot
y
ei
p e rli pi d e
n A A M T b b a h n n ni e o o fe r r r st a
Pa g e 7 1
ABC Ambe r CHM Conve rte r Tria l ve rsion, http://w w w .proce sste x t.com/a bcchm.html
m m m at p ia al al io y it it n y y s I â â P F Li †‘ †‘ a at p Tr C nc -f o i g h re re pr l y yl at e ot ce o i t i di ei ri m s et n d i c Er l i e ro u p s n pt a
s iv
s
e
e
x
d
a
ef
nt
ic
h
ie
o
nc
m
y
a s Li p e m ia re ti n al
is II â â C R a †‘ †‘ or e
Pa g e 7 2
ABC Ambe r CHM Conve rte r Tria l ve rsion, http://w w w .proce sste x t.com/a bcchm.html
L C L o st D h D n ri L ol L ar ct re e
y e
ce s t
di d
pt er
s di
or ol
e et
d
a Bi
ef
s le
ic
e ac
ie
T id
nc
e â
y
n €“ d bi o n n di x n a g nt re h si o n ms a Ni s co X ti a ni nt c h ac el i d a H s M mG a -C o A
Pa g e 7 3
ABC Ambe r CHM Conve rte r Tria l ve rsion, http://w w w .proce sste x t.com/a bcchm.html
re d uc ta s e in hi bi to rs II â â C R b †‘ †‘ or e C L o st h D n ri ol L ar ct e â y e s t †‘ di d er V s di ol L e et â D a Bi †‘ L s l e Tr
e ac
ig
id
ly
â
ce
€“
ri
bi
d
n
e
di
s
n g re si n s
Pa g e 7 4
ABC Ambe r CHM Conve rte r Tria l ve rsion, http://w w w .proce sste x t.com/a bcchm.html
Ni co ti ni c ac id G e m fi br o zi l H M G -C o A re d uc ta s e in hi bi to rs II â â P G I †‘ †‘ al e D Tr V m m
Pa g e 7 5
ABC Ambe r CHM Conve rte r Tria l ve rsion, http://w w w .proce sste x t.com/a bcchm.html
ys i g L ar fi b l y D fa br et ce L t t o al ri R y zi ip d e st l o e m re pr s n a ot â a ks ei †‘ nt T n C s u e h â b m ol †‘ er ia e V o st L u er D s ol L x a nt h o m a s C or o n ar y di s e a s e
Pa g e 7 6
ABC Ambe r CHM Conve rte r Tria l ve rsion, http://w w w .proce sste x t.com/a bcchm.html
P er ip h er al v a sc ul ar di s e a s e H y p ot h yr oi di s m IV â â Er R †‘ †‘ u e Tr V pt s t i g L i v ri l y D e ct ce L x e ri
a d
d
nt di
Pa g e 7 7
ABC Ambe r CHM Conve rte r Tria l ve rsion, http://w w w .proce sste x t.com/a bcchm.html
e
h et
s
o G me a m s fi P br a o nc zi re l at iti s Di a b et e s  ± C or o n ar y ar te ry di s e a s e
Pa g e 7 8
ABC Ambe r CHM Conve rte r Tria l ve rsion, http://w w w .proce sste x t.com/a bcchm.html
V â â Er R †‘ †‘ u e Tr C pt s t i g h i v ri l y yl e ct ce o x e ri m a d d i c nt di e ro h et s n o G s me a m s fi P br a o nc zi re l at iti s Di a b et e s  ± C or o n ar y ar
Pa g e 7 9
ABC Ambe r CHM Conve rte r Tria l ve rsion, http://w w w .proce sste x t.com/a bcchm.html
te ry di s e a s e HMG-CoA, β-hydroxyβ-met hyl g l ut aryl coenzyme A; LDL, l ow-dens i t y l i poprot ei n s ; VLDL, very-l ow-d ens i t y l i poprot ei n s.
d. Smoking. Compared wi t h nons mokers , ci garet t e s mokers are 60% more l i kel y t o devel op ASCAD when ot her ri s k fact ors are cont rol l ed for s t at i s t i cal l y. Smoki ng i ncreas es carbon monoxi de l evel s i n t he bl ood, whi ch may, i n t urn, damage t he coronary endot hel i um. Smoki ng al s o i ncreas es pl at el et adhes i venes s and t hus t he l i kel i hood of t hrombot i c coronary occl us i on.
e. Hypertension. The hi gher t he s ys t ol i c or
Pa g e 8 0
ABC Ambe r CHM Conve rte r Tria l ve rsion, http://w w w .proce sste x t.com/a bcchm.html
di as t ol i c bl ood pres s ure, t he more l i kel y t he devel opment of ASCAD. Thi s l i kel i hood i s apparent i n bot h men and women and becomes more pronounced wi t h advanci ng age.
f. Diabetes mellitus i s as s oci at ed wi t h a 50% i ncreas e i n t he i nci dence of ASCAD i n men and a 100% i ncreas e i n women. In l i ght of t hi s , di abet es mel l i t us s houl d be cons i dered an ASCAD equi val ent .
g. Family history. A fami l y hi s t ory of premat ure heart di s eas e i s cons i dered t o be pres ent when coronary art ery di s eas e has occurred i n a fi rs t -degree mal e rel at i ve age 55 or younger or a femal e fi rs t -degree rel at i ve age 65 or younger. A fami l i al predi s pos i t i on t o coronary art ery di s eas e exi s t s i n part due t o i nheri t ance of t he above ri s k fact ors (except s moki ng). However, fami l y hi s t ory i s t he onl y ri s k fact or i n about one-t hi rd of i ndi vi dual s wi t h ASCAD.
o
3.
Pathogenesis (Online Figures 1-6 and 1-7). The previ ous l y
ment i oned ri s k fact ors do not cons t i t ut e a known mechani s m for ASCAD. The major t heory of at herogenes i s i s t he response to injury theory.
Pa g e 8 1
ABC Ambe r CHM Conve rte r Tria l ve rsion, http://w w w .proce sste x t.com/a bcchm.html o
ONLINE FIGURE 1-6 Di agrammat i c repres ent at i on of an at heros cl erot i c pl aque, s howi ng i t s compos i t i on. The fi brous c ap of t he pl aque i s l i nked t o cl i ni cal event s becaus e of i t s t endency t o fract ure and ul cerat e. The nec rot i c c ore of t he pl aque has cl i ni cal cons equence as a res ul t of i t s s i ze, cons i s t ency, and t hrombopl as t i c component s . (Adapt ed from Braunwal d E: Heart Di s eas e. 2nd ed. Phi l adel phi a: W B Saunders , 1984:1186. )
ONLINE FIGURE 1-7 Schemat i c evol ut i on of t he at heros cl erot i c pl ague. 1, Li poprot ei n part i cl es accumul at e i n t he i nt i ma. The modi fi cat i on of t hes e l i poprot ei ns i s depi ct ed by t he darker col or. 2, Oxi dat i ve s t res s (i ncl udi ng product s found i n modi fi ed l i poprot ei ns ) i nduce l ocal cyt oki ne el aborat i on. 3, The cyt oki nes i nduce i ncreas ed expres s i on of mol ecul es t hat di rect t hei r mi grat i on i nt o t he i nt i ma. 4, Bl ood monocyt es ent er t he art ery wal l i n res pons e t o chemoat t ract ant cyt oki nes s uch as monocyt e
Pa g e 8 2
ABC Ambe r CHM Conve rte r Tria l ve rsion, http://w w w .proce sste x t.com/a bcchm.html
chemoat t ract ant prot ei n 1 (MCP-1). They encount er s t i mul i s uch as macrophage col ony s t i mul at i ng fact or (M-CSF) t hat augment t hei r expres s i on of s cavenger recept ors . 5, Scavenger recept ors medi at e t he upt ake of modi fi ed l i poprot ei n part i cl es and promot e devel opment of foam cel l s . Foam cel l s produce cyt oki nes and effect or mol ecul es i ncl udi ng hypochl orous aci d, s uperoxi de ani on (O 2
), and mat ri x met al l oprot ei nas es . 6, Smoot h mus cl e cel l s i n t he
i nt i ma prol i ferat e and ot her s moot h mus cl e cel l s mi grat e i nt o t he i nt i ma from t he medi a. 7, The s moot h mus cl e cel l s el aborat e ext racel l ul ar mat ri x, promot i ng i t s accumul at i on i n t he growi ng at heros cl erot i c pl aque. The fat t y s t reak evol ves i nt o a fi brofat t y l es i on. 8, In l at er s t ages , cal ci fi cat i on can occur (not depi ct ed) and fi bros i s cont i nues , s omet i mes accompani ed by s moot h mus cl e cel l deat h. The res ul t i s a rel at i vel y acel l ul ar caps ul e s urroundi ng a l i pi d-ri ch core t hat may al s o cont ai n dyi ng or dead cel l s and t hei r debri s . LDL, l ow-dens i t y l i poprot ei n; IL-1, i nt erl euki n-1. (Modi fi ed from Zi pes D, Li bby P, Bonow RO, Braunmwal d Ed, eds . Braunw al d's Heart Di s eas e: A T ext book of Cardi ovas c ul ar Medi c i ne. 7t h ed. Phi l adel phi a: El s evi er Saunders , 2005:925. )
a. Thi s t heory s t at es t hat s ome i njuri ous s t i mul us (e.g., hypert ens i on or hyperchol es t erol emi a) caus es endot hel i al damage, res ul t i ng i n t he rel eas e of vari ous growt h fact ors . Thes e growt h fact ors caus e s moot h cel l prol i ferat i on and mi grat i on of macrophages i nt o t he ves s el wal l . At t he s ame t i me, t he now i njured endot hel i um becomes more permeabl e, admi t t i ng l i pi d and chol es t erol i nt o t he i nt i ma.
b. Thes e changes res ul t i n pl aque format i on, whi ch may event ual l y compromi s e t he ves s el
Pa g e 8 3
ABC Ambe r CHM Conve rte r Tria l ve rsion, http://w w w .proce sste x t.com/a bcchm.html
l umen enough t o i mpede bl ood fl ow. If t he pl aque i s di s rupt ed, pl at el et s are act i vat ed, l eadi ng t o t hrombus format i on and wors eni ng obs t ruct i on. o
o
4. Pathophysiology of ischemia
a. Supply–demand relationship. As wal l s t res s and heart rat e i ncreas e (e.g., wi t h exerci s e), myocardi al oxygen cons umpt i on ri s es . Aut oregul at ed i ncreas es i n coronary bl ood fl ow normal l y meet t hi s i ncreas ed demand. However, i f demand exceeds s uppl y, i s chemi a res ul t s . The product of heart rat e and l eft vent ri cul ar s ys t ol i c pres s ure or wal l s t res s roughl y approxi mat e t he oxygen needs of t he myocardi um (s ee I B 1 b).
(1) Increased demand. In pat i ent s wi t h ASCAD, s t enos i s of t he coronary art ery prevent s t he i ncreas e i n coronary bl ood fl ow needed t o compens at e for an i ncreas ed demand, res ul t i ng i n an oxygen demand t hat exceeds t he oxygen s uppl y. Myocardial ischemia i s t he res ul t of t hi s i mbal ance.
(2) Reduced supply. At heros cl erot i c s t enos i s was once vi ewed as a fi xed obs t ruct i on t o coronary bl ood fl ow. In fact , t he di s eas ed area of t he coronary art ery oft en remai ns dynami c, and t he effect i ve l umen of t he art ery undergoes
Pa g e 8 4
ABC Ambe r CHM Conve rte r Tria l ve rsion, http://w w w .proce sste x t.com/a bcchm.html
cons t ant change. Changes are produced by vas ocons t ri ct i on of t he coronary art ery, by product i on and degradat i on of l ocal t hrombi at t he s i t e of s t enos i s , and by progres s i ve enl argement of t he at heros cl erot i c pl aque. Acut e changes i n l umen di amet er may reduce t he s uppl y of coronary bl ood fl ow and, t hus , produce i s chemi a wi t hout an i ncreas e i n demand.
b. Myocardial infarction (MI). The necros i s of myocardi al t i s s ue occurs as a res ul t of prol onged i s chemi a. The rapi di t y and ext ent of t he i nfarct i on proces s are det ermi ned by t he ext ent of reduct i on of bl ood fl ow t o t he area. In s ome cas es , collateral flow from ot her coronary art eri es may s uppl y enough bl ood fl ow t o prevent i nfarct i on des pi t e a t ot al coronary occl us i on.
(1) ST -segment elevation myocardial infarction (ST EMI) i s as s oci at ed wi t h occl us i on of a coronary art ery by a thrombus. Aggres s i ve l ys i s of t he t hrombus wi t h agent s s uch as streptokinase and tissue plasminogen activator (t-PA) or bal l oon angi opl as t y can rees t abl i s h coronary bl ood fl ow, rel i eve pai n, rees t abl i s h cont ract i l e funct i on of t he s egment of myocardi um s uppl i ed by t he t hrombos ed art ery, and reduce myocardi al damage [s ee III A 5 b (4) (b) i i ].
Pa g e 8 5
ABC Ambe r CHM Conve rte r Tria l ve rsion, http://w w w .proce sste x t.com/a bcchm.html
(2) Non–ST EMI (NST EMI) i s due t o s evere coronary art ery obs t ruct i on wi t hout t ot al occl us i on.
5. Clinical consequences. At heros cl erot i c pl aques may produce s t abl e angi na or an acute coronary syndrome (ACS) (uns t abl e angi na, NSTEMI, or STEMI).
a. Angina pectoris i s ches t pai n or pres s ure produced by myocardial i s chemi a. P.13
(1) Characteristic features
(a) Relation to exertion. The s i ngl e mos t i mport ant feat ure of angi na pect ori s i s i t s preci pi t at i on by exert i on. Exert i on i ncreas es myocardi al oxygen demand beyond t he s uppl y capabi l i t i es of di s eas ed coronary art eri es , produci ng i s chemi a. Ot her fact ors t hat i ncreas e myocardi al oxygen demand (e.g., emot i onal ups et , eat i ng a meal , or t he peri pheral vas ocons t ri ct i on
Pa g e 8 6
ABC Ambe r CHM Conve rte r Tria l ve rsion, http://w w w .proce sste x t.com/a bcchm.html
caus ed by wal ki ng i n col d weat her) al s o may preci pi t at e angi na.
(b) Quality of pain. Al t hough many pat i ent s percei ve angi na as chest pain, ot hers report a feel i ng of pressure i n t he ches t area or compl ai n of a burning s ens at i on. In s ome pat i ent s , exertional dyspnea may repres ent an angi nal equi val ent .
(c) Radiation of pain. Radi at i on of angi nal pai n t o t he l eft arm i s wel l known. Pai n al s o may radi at e t o t he ri ght arm, jaw, t eet h, or t hroat . Occas i onal l y, t hes e radi at i on s i t es may be t he onl y s i t es of pai n, and t he ches t i s free of di s comfort ; or t he ches t di s comfort , when pres ent , may not radi at e at al l .
(d) Progression of ischemia and duration of symptoms. W hat ever t he angi nal s ympt om qual i t y for a gi ven pat i ent , repeat ed epi s odes of i s chemi a us ual l y reproduce t hat s ame
Pa g e 8 7
ABC Ambe r CHM Conve rte r Tria l ve rsion, http://w w w .proce sste x t.com/a bcchm.html
qual i t y. The s ympt om compl ex us ual l y begi ns at a l ow i nt ens i t y, i ncreas es over 2–3 mi nut es , and l as t s a t ot al of l es s t han 15 mi nut es . Epi s odes l onger t han 30 mi nut es s ugges t t hat MI may have occurred.
(2) T ypes of ischemic episodes
(a) Chronic stable angina i s angi na t hat recurs under s i mi l ar ci rcums t ances and wi t h a s i mi l ar frequency over t i me.
(b) Unstable angina (UA) i s a t erm appl i ed t o angi na when a change i n s t at us occurs (e.g., new-ons et angi na; angi na of i ncreas i ng s everi t y, durat i on, or frequency; or angi na occurri ng at res t for t he fi rs t t i me). It repres ent s a more s eri ous cl i ni cal s i t uat i on t han chroni c s t abl e angi na becaus e uns t abl e angi na i ndi cat es a progres s i on of di s eas e and may be an i mmedi at e precurs or of MI.
(i) Rest angina. Angi na at res t i s part i cul arl y
Pa g e 8 8
ABC Ambe r CHM Conve rte r Tria l ve rsion, http://w w w .proce sste x t.com/a bcchm.html
worri s ome becaus e i t i mpl i es t hat decreas ed s uppl y, rat her t han i ncreas ed demand, i s caus i ng t he angi na. Thi s concept i n t urn s ugges t s t hat art eri al occl us i on and pos s i bl e i nfarct i on may be i mmi nent .
(ii) New-onset angina. In cas es of new-ons et angi na, i t i s di ffi cul t t o general i ze about cl i ni cal out come. New-ons et angi na t hat progres s es i n frequency, s everi t y, or durat i on over 1 or 2 mont hs i s worri s ome. Convers el y, s ome cas es of new-ons et angi na may s i mpl y be t he fi rs t epi s ode i n what becomes a chroni c s t abl e angi nal pat t ern.
(c) Variant (Prinzmetal' s) angina
(i) The hallmark of vari ant angi na i s t he appearance of t rans i ent ST -segment elevation
Pa g e 8 9
ABC Ambe r CHM Conve rte r Tria l ve rsion, http://w w w .proce sste x t.com/a bcchm.html
on t he ECG duri ng t he angi na at t ack. The ST-s egment el evat i on repres ent s transmural ischemia produced by a s udden reduct i on i n coronary bl ood fl ow.
(ii) The reduct i on i n fl ow res ul t s from transient coronary spasm, whi ch may or may not be as s oci at ed wi t h a fi xed at heros cl erot i c l es i on. The s pas m produces t ot al but t rans i ent coronary occl us i on.
(iii) Vari ant angi na us ual l y occurs at res t (oft en at ni ght ), and epi s odes frequent l y are compl i cat ed by compl ex vent ri cul ar arrhyt hmi as .
(3) Diagnosis. W hen a pat i ent exhi bi t s ches t pai n charact eri s t i c of angi na, t he di agnos i s can be s us pect ed s t rongl y on t he bas i s of pat i ent hi s t ory al one. The s us pi ci on t hat coronary di s eas e i s pres ent i s hei ght ened by t he pres ence of one or more coronary ri s k fact ors .
Pa g e 9 0
ABC Ambe r CHM Conve rte r Tria l ve rsion, http://w w w .proce sste x t.com/a bcchm.html
(a) Physical examination. Pat i ent s experi enci ng an epi s ode of angi na are us ual l y uncomfort abl e and anxi ous . Bl ood pres s ure and pul s e rat e are i ncreas ed i n mos t cas es .
(b) Resting electrocardiography. The ECG t aken i n t he abs ence of pai n i n pat i ent s wi t h angi na pect ori s wi t h no hi s t ory of MI i s normal i n 50% of cas es . Every effort s houl d be made t o obt ai n an ECG whi l e t he pat i ent i s experi enci ng ches t pai n.
(i) The pres ence of new horizontal or downsloping ST segments on t he ECG i s hi ghl y s ugges t i ve of myocardi al i s chemi a. New T -wave inversion al s o may occur, but t hi s fi ndi ng al one wi t hout ST s egment depres s i on i s l es s s peci fi c. P.14
Pa g e 9 1
ABC Ambe r CHM Conve rte r Tria l ve rsion, http://w w w .proce sste x t.com/a bcchm.html
(ii) In t he pres ence of vari ant angi na, an acut e current of i njury i ndi cat ed by t rans i ent ST s egment el evat i on i s di agnos t i c. The ST s egment el evat i on normal i zes as t he pai n wanes and no Q waves appear.
(c) Stress electrocardiography. Recordi ng t he ECG duri ng exerci s e s ubs t ant i al l y i ncreas es t he s ens i t i vi t y and s peci fi ci t y of el ect rocardi ography. In addi t i on, a formal exerci s e t es t permi t s quant i fi cat i on of t he pat i ent 's exerci s e t ol erance and obs ervat i on of t he effect s of exerci s e on t he pat i ent 's s ympt oms , heart rat e, and bl ood pres s ure.
(i) The appearance of hori zont al or downs l opi ng ST-s egment depres s i on of 1 mm or more duri ng exerci s e has a s ens i t i vi t y of approxi mat el y 70% and a s peci fi ci t y of 90% for t he det ect i on of
Pa g e 9 2
ABC Ambe r CHM Conve rte r Tria l ve rsion, http://w w w .proce sste x t.com/a bcchm.html
coronary di s eas e.
(ii) The ST cri t eri a for pos i t i vi t y are l es s accurat e i n women t han i n men. An abnormal ST s egment on a res t i ng ECG, as s een i n l eft bundle branch block, left ventricular hypertrophy, or t he us e of digoxin by t he pat i ent all reduce the accuracy of this test.
(d) Stress scintigraphy, when us ed i n combi nat i on wi t h t he s t res s ECG, has yi el ded i ncreas ed s ens i t i vi t y (80%) and s peci fi ci t y (92%) over t he s t andard s t res s ECG al one. Therefore, i t i s a part i cul arl y us eful di agnos t i c t ool when t he s t andard s t res s ECG i s expect ed t o be of l ow yi el d (e.g., i n women and i n pat i ent s wi t h bundl e branch bl ock) and i n pat i ent s i n whom a previ ous s t res s ECG has produced equi vocal res ul t s .
(i) Method. W hen t he radi oact i ve i s ot ope thallium 201 (
201
TI) or
t he t echnet i um-bas ed i s oni t ri l e sestamibi i s i nject ed i nt o t he peri pheral venous bl ood, t he myocardi al di s t ri but i on of t he
Pa g e 9 3
ABC Ambe r CHM Conve rte r Tria l ve rsion, http://w w w .proce sste x t.com/a bcchm.html
s ubs t ance i s affect ed by bl ood fl ow and i s chemi a, wi t h areas of l es s bl ood fl ow and i s chemi a t aki ng up l es s
201
TI or s es t ami bi t han
areas of normal bl ood fl ow. W i t h exerci s e, bl ood fl ow i ncreas es , but i n pat i ent s wi t h coronary art ery di s eas e, t hos e part s of t he myocardi um s uppl i ed by di s eas ed coronary art eri es and areas of MI t ake up l es s s ci nt i gram i n
201
TI or s es t ami bi t han normal areas , as s hown i n t he Onl i ne Fi gure 1-8. In pat i ent s who are unabl e t o
exerci s e, i nfus i on of dobut ami ne (t o i ncreas e oxygen demand) or di pyri damol e or adenos i ne (t o caus e coronary vas odi l at i on) are us ed t o al t er coronary fl ow.
ONLINE FIGURE 1-8 Si ngl e Phot on Emi s s i on Comput ed Tomography (SPECT) i mages of myocardi al perfus i on ut i l i zi ng a radi o-l abel ed t racer were obt ai ned at peak s t res s (t op rows ) and at res t (mat ched bot t om rows ). Tomographi c “s l i ces ― of t he heart were obt ai ned i n t hree axes (from t op t o bot t om): s hort axi s i mages “s l i ci ng― from apex t o bas e; vert i cal l ong-axi s i mages “s l i ci ng― from s ept um t o l at eral wal l ; and hori zont al l ong axi s i mages “s l i ci ng― from pos t eri or t o ant eri or. The i mage i n (A) demons t rat es mat ched perfus i on of al l s egment s at bot h peak
Pa g e 9 4
ABC Ambe r CHM Conve rte r Tria l ve rsion, http://w w w .proce sste x t.com/a bcchm.html
s t res s and at res t . Thi s woul d be cons i dered an exampl e of a “normal ― myocardi al perfus i on s t res s t es t . The i mage i n (B) demons t rat es a rel at i vel y phot openi c regi on on t he peak s t res s i mages i nvol vi ng t he i nt ervent ri cul ar s ept um, ant eri or wal l and apex. Thi s defect normal i zes on t he res t i mages , and woul d be t ermed a ‘revers i bl e’ defect , whi ch i s charact eri s t i c of i s chemi a. The pat i ent had s evere proxi mal l eft ant eri or des cendi ng s t enos i s on angi ography.
(e) Stress echocardiography. Two-di mens i onal echocardi ography can det ect regi onal i s chemi a by i dent i fyi ng areas of wal l mot i on abnormal i t i es t hat occur wi t h s t res s and are not pres ent at res t . Thi s regi onal dys funct i on i ndi cat es t hat t he area i nvol ved i s not recei vi ng adequat e bl ood fl ow. The s ens i t i vi t y i s s i mi l ar t o radi onucl eot i de i magi ng.
(f) Cardiac catheterization with coronary arteriography al l ows for di rect vi s ual i zat i on of t he coronary art eri es by s el ect i ve i nject i on of radi ographi c cont ras t mat eri al . Thi s procedure i s t he most sensitive and specific test commonly used for coronary artery disease.
Pa g e 9 5
ABC Ambe r CHM Conve rte r Tria l ve rsion, http://w w w .proce sste x t.com/a bcchm.html
(i) Risk. Unl i ke t he previ ous l y ment i oned t es t s , cardi ac cat het eri zat i on i s an i nvas i ve procedure t hat carri es a s mal l but fi ni t e ri s k. The overal l ri s k of mort al i t y duri ng coronary art eri ography i s approxi mat el y 0.2%.
(ii) Applications. Cardi ac cat het eri zat i on s houl d be res erved for cas es i n whi ch t he di agnos i s i s uncert ai n aft er noni nvas i ve t es t i ng, when more i nformat i on i s needed t o hel p det ermi ne whet her medi cal or s urgi cal t herapy i s mos t appropri at e for t he pat i ent 's coronary di s eas e, or when i nt ervent i on i s neces s ary, as i n pat i ent s wi t h acut e STEMI or UA. If s urgery i s cont empl at ed, t he art eri ograms obt ai ned at cat het eri zat i on gui de t he s urgeon's pl acement of t he bypas s graft s .
Pa g e 9 6
ABC Ambe r CHM Conve rte r Tria l ve rsion, http://w w w .proce sste x t.com/a bcchm.html
(g) Cardiac computed tomography (CT ). An emergi ng and pot ent i al l y powerful t ool for det ect i ng coronary art ery di s eas e i s cardi ac CT. The rol e of t hi s modal i t y has not yet been cl earl y es t abl i s hed.
(4) T herapy. Treat ment of angi na pect ori s i s di rect ed ei t her at reduci ng myocardi al oxygen demand t o compens at e for i mpai red fl ow t hrough di s eas ed coronary art eri es or at i ncreas i ng myocardi al oxygen s uppl y (i .e., bl ood fl ow).
(a) Nitrates. Thi s cl as s of drugs produces venodi l at i on and, t o a l es s er ext ent , art eri ol ar vas odi l at i on. They may i ncreas e coronary bl ood fl ow and decreas e myocardi al oxygen demand by decreas i ng prel oad and aft erl oad. P.15
(b) β-Adrenergic blocking agents. β-Adrenergi c recept or s t i mul at i on res ul t s i n an i ncreas e i n heart rat e and i n
Pa g e 9 7
ABC Ambe r CHM Conve rte r Tria l ve rsion, http://w w w .proce sste x t.com/a bcchm.html
t he force of myocardi al cont ract i on. Bot h event s i ncreas e myocardi al oxygen demand. β-bl ockers count eract t hes e effect s and reduce myocardial oxygen demand.
(c) Calcium channel blockers. Cal ci um regul at es t he cont ract i on of s moot h mus cl e, whi ch i s pres ent i n t he wal l s of t he coronary and peri pheral art eri es . Cal ci um channel bl ockers are part i cul arl y effective in preventing the coronary spasm that causes variant angina. They are al s o us eful i n t reat i ng cas es of t ypi cal angi na, i n whi ch t hey act as coronary and peripheral arterial vasodilators. The nondi hydropyri di ne cal ci um channel bl ockers , di l t i azem and verapami l , al s o reduce heart rat e and al l cal ci um channel bl ockers may decreas e bl ood pres s ure.
(d) Percutaneous coronary intervention (PCI). Removi ng or reduci ng t he obs t ruct i ve coronary at heros cl erot i c l es i on
Pa g e 9 8
ABC Ambe r CHM Conve rte r Tria l ve rsion, http://w w w .proce sste x t.com/a bcchm.html
can al l evi at e t he angi na.
(i) Duri ng angi opl as t y, a s mal l bal l oon i s i ns ert ed i nt o a femoral or brachi al art ery and gui ded t o t he obs t ruct i on of t he affect ed coronary art ery. The bal l oon i s i nfl at ed, di l at i ng t he s t enos i s and reduci ng t he obs t ruct i on.
(ii) The i ni t i al s ucces s rat e of s i mpl e bal l oon angi opl as t y approaches 90%, al t hough t here i s a 33% res t enos i s rat e aft er 6 mont hs , maki ng i t neces s ary t o repeat t he procedure i n s ome pat i ent s . Pl acement of s mal l wi re stents reduces t he rat e of res t enos i s and now accompani es t he majori t y of angi opl as t i es . Today s ome s t ent s are coat ed wi t h s i rol i mus or pacl i t axel , whi ch reduces t he res t enos i s rat e t o approxi mat el y 5%. Co-admi ni s t rat i on of abci xi mab, a IIb–IIIa pl at el et recept or ant i body, or ept i fi bat i de,
Pa g e 9 9
ABC Ambe r CHM Conve rte r Tria l ve rsion, http://w w w .proce sste x t.com/a bcchm.html
a IIb–IIIa recept or ant agoni s t , al s o enhances s hort - and l ong-t erm pat ency. The us e of a t hi enopyri di ne, s uch as cl opi dogrel , i s es s ent i al aft er s t ent i ng t o prevent acut e t hrombos i s .
(e) Coronary artery bypass grafting (CABG). CABG ext ends s urvi val i n pat i ent s wi t h s evere l eft mai n coronary art ery di s eas e or pat i ent s wi t h s evere t hree-ves s el di s eas e and depres s ed l eft vent ri cul ar funct i on. There i s cont i nued debat e regardi ng t he rol e of CABG vers us mul t i ves s el PCI i n pat i ent s wi t h s evere mul t i ves s el di s eas e and normal l eft vent ri cul ar funct i on. CABG may be preferabl e i n di abet i c pat i ent s wi t h mul t i ves s el di s eas e t hat i ncl udes t he l eft ant eri or des cendi ng art ery.
(f) Refractory angina. Several new t herapi es have emerged for t reat ment of refract ory angi na i ncl udi ng t rans myocardi al l as er
Pa g e 1 0 0
ABC Ambe r CHM Conve rte r Tria l ve rsion, http://w w w .proce sste x t.com/a bcchm.html
revas cul ari zat i on, enhanced ext racorporeal count er pul s at i on, and ranol azi ne. Thes e met hods can be cons i dered i n pat i ent s who cont i nue t o experi ence angi na des pi t e maxi mal t reat ment on convent i onal medi cal t herapy and revas cul ari zat i on.
b. Myocardial infarction (MI) occurs when t he myocardi um i s depri ved of i t s bl ood s uppl y (and t herefore, oxygen) for a s i gni fi cant amount of t i me.
(1) Pathogenesis. An abrupt change i n t he at heros cl erot i c pl aque s eems t o be one of t he event s preci pi t at i ng an MI. Pl aque rupt ure at t ract s pl at el et s t hat t ri gger t hrombus format i on, l eadi ng t o s evere s t enos i s or t ot al occl us i on of t he ves s el .
(2) Symptoms. The pat i ent us ual l y experi ences severe, oppressive chest pain or pressure t hat pers i s t s for more t han 30 mi nut es and i s unrel i eved by ni t rogl yceri n. The pai n radi at es i n a pat t ern s i mi l ar t o t hat of angi na pect ori s .
(a) Frequent l y, nausea,
Pa g e 1 0 1
ABC Ambe r CHM Conve rte r Tria l ve rsion, http://w w w .proce sste x t.com/a bcchm.html
vomiting, diaphoresis, and shortness of breath accompany t he pai n.
(b) An unus ual l y l arge number of i nfarct i ons occur between 6 A.M. and 10 A.M., when catecholamine levels increase on awakening.
(3) Diagnosis
(a) Physical examination. The pat i ent experi enci ng MI i s i n obvi ous pai n, i s qui t e apprehens i ve, and oft en appears as hen. If t he i nfarct i on i s ext ens i ve, hypot ens i on and t achycardi a may be pres ent . There al s o may be s i gns of CHF (e.g., el evat i on of t he neck vei ns , pul monary ral es , and a cardi ac gal l op rhyt hm). The new murmur of mi t ral regurgi t at i on may be pres ent .
(b) Electrocardiography. The ECG i s di agnos t i c i n approxi mat el y 85% of cas es . The remai ni ng 15% of pat i ent s may experi ence MI wi t hout mani fes t i ng cl ear-cut evi dence
Pa g e 1 0 2
ABC Ambe r CHM Conve rte r Tria l ve rsion, http://w w w .proce sste x t.com/a bcchm.html
on t he ECG. P.16
(i) STEMI i s demons t rat ed by ST-s egment el evat i on i n t hos e l eads refl ect i ng t he area of t he MI. As t he ST s egment s fal l , Q waves appear, and t he T waves become i nvert ed
(onl i ne Fi gure 1-9).
Online Figure 1-9 Sequence of ECG changes duri ng t he evol ut i on of a myocardi al i nfarct . A. Smoot hi ng of ST s egment and T waves , maki ng i t di ffi cul t t o s ee where one ends and t he ot her begi ns . B. St rai ght eni ng of t he ST s egment , l os i ng t he upward concavi t y. C. El evat i on of t he s t rai ght ened ST s egment . D. ST el evat i on may al s o occur wi t hout l os s of t he upward concavi t y. E. Invers i on of t he T waves wi t h concave downward ST s egment s . F. Devel opment of Q waves wi t h i nvert ed T waves . The ST s egment remai ns el evat ed and concave downward.
(ii) NSTEMI wi l l have l es s s peci fi c fi ndi ngs , s uch as ST depres s i on or T-wave i nvers i on.
(c) Cardiac enzyme studies
Pa g e 1 0 3
ABC Ambe r CHM Conve rte r Tria l ve rsion, http://w w w .proce sste x t.com/a bcchm.html
(i) As myocardi al necros i s occurs , t he myocardi um rel eas es creatine kinase (CK), including the MB isoform, and troponins, t hereby i ncreas i ng s erum concent rat i ons of t hes e enzymes .
(ii) CK and t roponi n el evat i on appear 3 hours aft er i nfarct i on. CK and CK-MB us ual l y ret urn t o bas el i ne wi t hi n 2 days , but t roponi n may remai n el evat ed for up t o 10 days fol l owi ng an MI.
(4) T herapy
(a) Initial medical therapy. For pat i ent s pres ent i ng wi t h ei t her an STEMI or NSTEMI, s everal medi cal t herapi es s houl d be i ni t i at ed, provi di ng t here are no cont rai ndi cat i ons .
(i) Aspirin s houl d be gi ven i mmedi at el y t o bl ock pl at el et aggregat i on.
Pa g e 1 0 4
ABC Ambe r CHM Conve rte r Tria l ve rsion, http://w w w .proce sste x t.com/a bcchm.html
(ii) Oxygen s houl d be del i vered t o ens ure adequat e oxygenat i on of t i s s ues .
(iii) Nitroglycerin can al l evi at e pai n by vas odi l at i on, whi ch i ncreas es bl ood fl ow, t hereby decreas i ng oxygen demand and coronary vas os pas m.
(iv) Heparin s houl d be s t art ed i nt ravenous l y t o decreas e furt her cl ot format i on. IV hepari n i s preferred for STEMI, when an earl y i nt ervent i on i s pl anned, as i t can eas i l y be t urned off. LMW H may be preferabl e i n NSTEMI, when i nt ervent i on can be del ayed, as t here i s no need for dos e adjus t ment , and t he mort al i t y ri s k may be s l i ght l y l ower.
(v) β-blockers reduce vent ri cul ar arrhyt hmi as and t he ri s k of rei nfarct i on and decreas e t he workl oad and oxygen demand of t he heart . They s houl d be us ed wi t h caut i on i n pat i ent s wi t h s evere CHF, bradyarrhyt hmi as , or bronchos pas m.
Pa g e 1 0 5
ABC Ambe r CHM Conve rte r Tria l ve rsion, http://w w w .proce sste x t.com/a bcchm.html
(b) ST EMI. In STEMI, pers i s t ent t hrombot i c occl us i on i s pres ent i n t he majori t y of pat i ent s , l eadi ng t o necros i s of myocardi al t i s s ue t hat i s not bei ng perfus ed. Mort al i t y and morbi di t y are decreas ed wi t h t he es t abl i s hment of earl y reperfus i on of t he occl uded ves s el s , res t ori ng bl ood fl ow t o t he myocardi um.
(i) PCI. If t he pat i ent pres ent s wi t hi n 12 hours of s ympt om ons et and i s abl e t o undergo PCI wi t hi n 90 mi nut es of pres ent at i on or i s i n cardi ogeni c s hock, PCI i s t he preferred met hod of t reat ment for an STEMI as i t offers t he bes t chance for reperfus i on and has i mproved out comes compared t o fi bri nol ys i s .
(ii) Fibrinolysis. If PCI i s not readi l y avai l abl e, fi bri nol ys i s t o di s s ol ve t he t hrombus and promot e reperfus i on s houl d be cons i dered. Fi bri nol yt i c agent s (s t rept oki nas e, t -PA, and uroki nas e) s houl d be admi ni s t ered wi t hi n 30 mi nut es of pres ent at i on. There i s a s i gni fi cant ri s k of bl eedi ng
Pa g e 1 0 6
ABC Ambe r CHM Conve rte r Tria l ve rsion, http://w w w .proce sste x t.com/a bcchm.html
wi t h t hrombol yt i c t herapy and cont rai ndi cat i ons need t o be rul ed out before admi ni s t rat i on.
(c) UA/NST EMI. In NSTEMI or UA, t here i s s evere coronary obs t ruct i on us ual l y wi t hout t ot al occl us i on of t he coronary art ery, and t hus t reat ment i s focus ed at s t abi l i zi ng t he pl aque and t reat i ng res i dual i s chemi a.
(i) PCI. Immedi at e PCI i s not i ndi cat ed i n mos t pat i ent s , and i s as s oci at ed wi t h wors ened out comes i n UA/NSTEMI. Angi ography and pos s i bl e PCI are performed aft er s t abi l i zat i on i n hi gh-ri s k pat i ent s , pat i ent s who cont i nue t o have pai n des pi t e ant i -i s chemi c t herapy, or i n l ow-ri s k pat i ent s wi t h evi dence of i s chemi a on noni nvas i ve s t res s t es t i ng.
(ii) Additional antiplatelet agents are admi ni s t ered wi t h as pi ri n and have been s hown t o decreas e t he mort al i t y and morbi di t y as s oci at ed wi t h UA/NSTEMI. Clopidogrel
Pa g e 1 0 7
ABC Ambe r CHM Conve rte r Tria l ve rsion, http://w w w .proce sste x t.com/a bcchm.html
furt her i nhi bi t s pl at el et act i vi t y by bl ocki ng adenos i ne di phos phat e (ADP) recept ors and prevent i ng aggregat i on. A l oadi ng dos e s houl d be gi ven fol l owed by dai l y oral dos i ng. Glycoprotein IIb/IIIa inhibitors s uch as ept i fi bat i de, t i rofi ban or abci xi mab prevent t he fi nal pat hway of pl at el et aggregat i on and are admi ni s t ered vi a IV i nfus i on.
(d) Long-term secondary prevention therapy. In addi t i on t o modi fyi ng ri s k fact ors t hrough s moki ng ces s at i on and cont rol of hypert ens i on, di abet es , and hyperl i pi demi a, P.17
s everal cl as s es of medi cat i ons have been s hown t o have a mort al i t y benefi t when us ed pos t -MI.
(i) Aspirin. Long-t erm as pi ri n t herapy i ncreas es t he l i kel i hood of pat ency of t he affect ed art eri es . If pat i ent s have an as pi ri n al l ergy, cl opi dogrel may be s ubs t i t ut ed.
Pa g e 1 0 8
ABC Ambe r CHM Conve rte r Tria l ve rsion, http://w w w .proce sste x t.com/a bcchm.html
(ii) β-blockers. Long-t erm mort al i t y benefi t i s l i kel y due t o a combi nat i on of an ant i arrhyt hmi c effect and neurohormonal modul at i on.
(iii) 3-hydroxy-3-methylglutaryl coenzyme A (HMG-CoA) reductase inhibitors (statins). Decreas i ng LDL i s es s ent i al i n l oweri ng t he ri s k of coronary heart di s eas e. St at i n t herapy al s o has an ant i -i nfl ammat ory effect t hat benefi t s mort al i t y i ndependent l y of l i pi d reduct i on.
(iv) Inhi bi t i on of t he reni n–angi ot ens i n–al dos t er one s ys t em wi t h ACEinhibitors or ARBs prevent s remodel i ng and decreas es t he ri s k of i s chemi c event s , and s houl d be us ed i n pat i ent s wi t h ant eri or MI or s ys t ol i c dys funct i on.
(v) Clopidogrel added t o as pi ri n i mproves out comes aft er hos pi t al i zat i on i n pat i ent s wi t h NSTEMI,
Pa g e 1 0 9
ABC Ambe r CHM Conve rte r Tria l ve rsion, http://w w w .proce sste x t.com/a bcchm.html
regardl es s of i n-hos pi t al t reat ment approach.
(e) Automated implantable cardioverter-defibrillator (AICD). Pat i ent s wi t h pri or MI and a l eft vent ri cul ar eject i on fract i on (LVEF) of ≤40% or pat i ent s who have s urvi ved res us ci t at i on aft er cardi ac col l aps e not as s oci at ed wi t h acut e i nfarct i on s houl d be eval uat ed by a cardi ac el ect rophys i ol ogi s t . Such pat i ent s may be candi dat es for i ns ert i on of an AICD. AICDs moni t or t he heart rhyt hm and defi bri l l at e t he heart i f a l et hal rhyt hm i s det ect ed. Pat i ent s who recei ve frequent s hocks may be candi dat es for t he addi t i on of ant i arrhyt hmi c drugs or cat het er-bas ed abl at i on.
(5) Prognosis. Fol l owi ng an MI, a pat i ent 's prognos i s i s l argel y det ermi ned by pump funct i on and res i dual i s chemi c burden.
(a) Pump function. LV funct i on s houl d be as s es s ed fol l owi ng an MI by l eft vent ri cul ography, mul t i pl e gat ed acqui s i t i on s can (MUGA), or echocardi ogram. Prognos i s i s markedl y wors e for pat i ent s wi t h an LVEF of ≤40%.
Pa g e 1 1 0
ABC Ambe r CHM Conve rte r Tria l ve rsion, http://w w w .proce sste x t.com/a bcchm.html
(b) Residual ischemic risk. Al l pat i ent s who have an MI s houl d be eval uat ed by cardi ac cat het eri zat i on or s t res s t es t i ng t o det ermi ne t he ri s k of recurrent i s chemi a or i nfarct i on.
(6) Complications. An MI can occur wi t h l i t t l e cl i ni cal cons equence; i ndeed, many are s i l ent . The compl i cat i ons of MI, however, produce cl i ni cal l y s i gni fi cant event s .
(a) Arrhythmias. A pat i ent havi ng an acut e MI i s s ubject t o acut e, lethal ventricular arrhythmias (i .e., vent ri cul ar t achycardi a or vent ri cul ar fi bri l l at i on), as wel l as less serious atrial arrhythmias (e.g., at ri al fi bri l l at i on, at ri al fl ut t er). The ri s k for a l et hal arrhyt hmi a i s great es t wi t hi n t he fi rs t 48 hours , requi ri ng cl os e moni t ori ng.
(b) Acute conduction system abnormalities. The s peci al i zed conduct i ng s ys t em of t he heart i s i t s el f myocardi um, whi ch may become i s chemi c or i nfarct ed duri ng an MI. Thi s may l ead t o bradyarrhyt hmi as , heart bl ock, or
Pa g e 1 1 1
ABC Ambe r CHM Conve rte r Tria l ve rsion, http://w w w .proce sste x t.com/a bcchm.html
bot h.
(i) Inferior MI us ual l y occurs when t he ri ght coronary art ery i s di s eas ed. Becaus e t hi s art ery s uppl i es t he SA node i n 55% of pat i ent s and t he AV node i n 85% of pat i ent s , sinus bradycardia and varyi ng degrees of AV nodal block can occur. Heart bl ock occurri ng duri ng i nferi or i nfarct i on i s al mos t al ways t rans i ent .
(ii) Anterior MI us ual l y occurs from occl us i on of t he ant eri or des cendi ng coronary art ery, whi ch s uppl i es t he i nt ervent ri cul ar s ept um. Becaus e t he bundl e branches cours e t hrough t he s ept um, acut e right or left bundle branch block may occur duri ng ant eri or MIs . Complete heart block al s o may occur due t o dys funct i on of bot h bundl e branches or t he bundl e of Hi s .
(c) Pump failure. W hen 30% of t he myocardi um i s i nfarct ed from one or more MIs , CHF i s l i kel y t o ens ue. If more t han 40% of t he myocardi um becomes i nfarct ed, cardiogenic
Pa g e 1 1 2
ABC Ambe r CHM Conve rte r Tria l ve rsion, http://w w w .proce sste x t.com/a bcchm.html
shock i s l i kel y t o devel op.
(d) Mitral regurgitation. The papillary muscles, whi ch are project i ons of t he myocardi um, t et her t he mi t ral val ve. Dys funct i on or i nfarct i on of t he papi l l ary mus cl es t oget her wi t h vent ri cul ar di l at at i on may l ead t o s ys t ol i c prol aps i ng of t he mi t ral val ve i nt o P.18
t he l eft at ri um, caus i ng varyi ng degrees of mi t ral regurgi t at i on. Thi s may l ead t o a preci pi t ous ri s e i n t he l eft vent ri cul ar fi l l i ng pres s ure t hat i s t rans mi t t ed t o t he l ungs , res ul t i ng i n pul monary edema.
(e) Ventricular septal defect. The l eft vent ri cul ar s ept um may become i nfarct ed i n ei t her ant eri or or i nferi or MI, l eadi ng t o rupt ure of t he s ept um. Thi s occurs i n approxi mat el y 2% of pat i ent s , us ual l y 2–5 days aft er i nfarct i on.
(f) Cardiac rupture. MI of t he free wal l may l ead t o event ual perforat i on of t he heart . Thi s compl i cat i on, whi ch res ul t s i n overwhel mi ng cardiac tamponade, i s nearl y al ways fat al . Rarel y,
Pa g e 1 1 3
ABC Ambe r CHM Conve rte r Tria l ve rsion, http://w w w .proce sste x t.com/a bcchm.html
rupt ure may be cont ai ned by adherent peri cardi um formi ng a ps eudoaneurys m. Surgi cal correct i on i s neces s ary.
(g) Left ventricular aneurysm. The i nfarct ed zone of t he myocardi um may evagi nat e and heal wi t h fi brous connect i ve t i s s ue, formi ng an aneurys m. Left vent ri cul ar aneurys ms may produce cardi ac fai l ure and angi na, and t hey al s o may be t he s ource of s evere l eft vent ri cul ar arrhyt hmi as and s ys t emi c embol i .
(7) T reatment of complications
(a) Antiarrhythmic therapy. If s eri ous vent ri cul ar arrhyt hmi as occur, ami odarone or l i docai ne i s i nfus ed. Addi t i onal drugs may be neces s ary t o cont rol arrhyt hmi as i f l i docai ne i s i neffect i ve. Bretylium tosylate, procainamide, and intravenous β-blockers may be us eful i n cont rol l i ng acut e recal ci t rant arrhyt hmi as . Thes e agent s are gi ven i nt ravenous l y wi t h caut i on.
(b) Correction of serious conduction disturbances. As
Pa g e 1 1 4
ABC Ambe r CHM Conve rte r Tria l ve rsion, http://w w w .proce sste x t.com/a bcchm.html
not ed, hi gh-degree AV nodal bl ock may occur duri ng acut e MI, produci ng s i gni fi cant bradycardi a and hypot ens i on. Therapi es for res t ori ng heart rat e i ncl ude:
(i) Atropine (1 mg i nt ravenous l y) may res t ore conduct i on and i ncreas e heart rat e, es peci al l y i n i nferi or i nfarct i ons . If t hi s fai l s , an i nfus i on of a pos i t i ve chronot ropi c agent s uch as isoproterenol (or us e of a transcutaneous electronic pacemaker) i ncreas es heart rat e. Thes e t herapi es are di rect ed at mai nt ai ni ng heart rat e unt i l temporary transvenous pacemaking can be performed.
(i i ) In cas es of s evere l eft vent ri cul ar dys funct i on, at ri al s ys t ol e mus t be pres erved t o mai nt ai n cardi ac out put , and AV sequential pacemaking i s t he preferred t reat ment .
(i i i ) The occurrence of new bundle branch block —part i cul arl y t he combi nat i on of ri ght bundl e branch bl ock
Pa g e 1 1 5
ABC Ambe r CHM Conve rte r Tria l ve rsion, http://w w w .proce sste x t.com/a bcchm.html
and l eft ant eri or hemi bl ock—may be an i ndi cat i on for temporary prophylactic pacemaking becaus e t hes e di s t urbances may pres age t he occurrence of compl et e heart bl ock; however, t hi s t act i c i s cont rovers i al and obvi at ed by t he avai l abi l i t y of t rans cut aneous paci ng.
(c) T reatment of heart failure
(i) Mi l d CHF i n pat i ent s wi t h MI can be t reat ed wi t h diuretics.
(ii) The us e of digitalis duri ng acut e MI i s s afe but of l i mi t ed effi cacy.
(iii) In more advanced cas es of CHF, vasodilators may be us eful i n reduci ng cardi ac aft erl oad, al l owi ng i ncreas ed cardi ac out put .
(d) T reatment of cardiogenic shock. Shock i n t he pres ence of MI us ual l y i s at t ri but abl e t o i nadequat e l eft vent ri cul ar fi l l i ng,
Pa g e 1 1 6
ABC Ambe r CHM Conve rte r Tria l ve rsion, http://w w w .proce sste x t.com/a bcchm.html
s evere mus cl e damage, or a mechani cal compl i cat i on of t he MI. W hen s hock ens ues , an echocardiogram i s performed t o as s es s vent ri cul ar funct i on, and a Swan-Ganz catheter s houl d be pl aced t o meas ure l eft vent ri cul ar fi l l i ng pres s ure.
(i) If t he pul monary capi l l ary wedge pres s ure i s l es s t han 18 mm Hg, volume is infused to maximize left ventricular filling and cardi ac out put . In addi t i on, i f a new cardi ac murmur i s det ect ed, t he echocardi ogram and t he Swan-Ganz cat het er are us eful i n maki ng t he di agnos i s of acut e mi t ral regurgi t at i on or acut e vent ri cul ar s ept al defect .
(ii) Al t ernat i vel y, i f cardi ogeni c s hock i s caus ed by s evere mus cl e damage, inotropic agents (e.g., dobut ami ne, mi l ri none) and intra-aortic balloon pumping may be us ed t o s t abi l i ze t he pat i ent unt i l coronary art eri ography and rees t abl i s hment of coronary bl ood fl ow by PCI are carri ed
Pa g e 1 1 7
ABC Ambe r CHM Conve rte r Tria l ve rsion, http://w w w .proce sste x t.com/a bcchm.html
out . However, t he prognos i s for s uch pat i ent s remai ns very poor des pi t e t herapy.
(e) T reatment of mitral regurgitation and acute ventricular septal defect
(i) Arteriolar vasodilator therapy t o l ower s ys t emi c vas cul ar res i s t ance i s t he mai ns t ay of medi cal t herapy for t hes e compl i cat i ons . Reduct i on of s ys t emi c vas cul ar res i s t ance preferent i al l y i ncreas es forward cardi ac out put and reduces nonproduct i ve cardi ac out put , ei t her i nt o t he l eft at ri um (i n t he cas e of mi t ral regurgi t at i on) or t hrough t he vent ri cul ar s ept al defect .
(ii) Intra-aortic balloon pumping, whi ch al s o i ncreas es forward cardi ac out put and reduces nonproduct i ve cardi ac out put , i s us eful i n s t abi l i zi ng pat i ent s wi t h mi t ral regurgi t at i on or acut e vent ri cul ar s ept al defect , es peci al l y i n hypot ens i ve pat i ent s , where t he us e of
Pa g e 1 1 8
ABC Ambe r CHM Conve rte r Tria l ve rsion, http://w w w .proce sste x t.com/a bcchm.html
vas odi l at ors i s cont rai ndi cat ed.
(iii) Surgical correction of mechanical complications oft en i s requi red.
c. Sudden death i n pat i ent s wi t h coronary art ery di s eas e i s common; i n fact , approxi mat el y one-t hi rd of pat i ent s wi t h coronary di s eas e experi ence s udden deat h wi t hout ant ecedent angi na or MI.
(1) Precipitating causes. It i s bel i eved t hat mos t pat i ent s di e of acute ventricular arrhythmias preci pi t at ed by i s chemi a. Al t hough s udden deat h may be t he res ul t of an MI s econdary t o coronary art ery di s eas e, mos t pat i ent s who di e s uddenl y and are res us ci t at ed do not have an acut e MI document ed. It i s l i kel y t hat i n t hes e pat i ent s , i s chemi a produces het erogeneous depol ari zat i on of t he vent ri cl e, whi ch l eads t o vent ri cul ar t achycardi a and vent ri cul ar fi bri l l at i on.
(2) Acute therapy
(a) Cardiopulmonary resuscitation (i .e., mout h-t o-mout h res us ci t at i on and ext ernal ches t
Pa g e 1 1 9
ABC Ambe r CHM Conve rte r Tria l ve rsion, http://w w w .proce sste x t.com/a bcchm.html
compres s i on) mus t be i ni t i at ed i n t he eut hermi c pat i ent wi t hi n 4 mi nut es of t he ces s at i on of effect i ve vent ri cul ar cont ract i on t o pres erve neurol ogi c and myocardi al funct i on.
(b) Electrical defibrillation and drug support s houl d be provi ded as s oon as pos s i bl e.
B. Nonatherosclerotic coronary artery disease Al t hough t he majori t y of cardi ac i s chemi c event s are caus ed by at heros cl erot i c coronary di s eas e, nonat heros cl erot i c di s eas e al s o may produce cl i ni cal i s chemi a.
1. Coronary embolism occurs i n i nfect i ve endocardi t i s , from mural t hrombus format i on fol l owi ng MI, and i n t he pres ence of at ri al fi bri l l at i on. Coronary embol i s m frequent l y produces MI.
2. Collagen vascular disease. The col l agen vas cul ar di s eas es t hat affect medi um-s i zed art eri es , i ncl udi ng t he coronary art eri es , are: polyarteritis nodosa, Wegener' s granulomatosis, systemic lupus erythematosus (SLE), and, occas i onal l y, rheumatoid arthritis.
3. Radiation therapy. Tumor i rradi at i on, i n whi ch t he
Pa g e 1 2 0
ABC Ambe r CHM Conve rte r Tria l ve rsion, http://w w w .proce sste x t.com/a bcchm.html
fi el d of radi at i on i ncl udes t he heart , damages t he coronary art eri es and l eads t o nonat heros cl erot i c coronary art ery di s eas e.
4. T ransplantation. The devel opment of coronary di s eas e fol l owi ng cardi ac t rans pl ant at i on i s a major fact or i n l i mi t i ng t he s ucces s of t hi s t herapy. Pos t -t rans pl ant at i on coronary di s eas e t ends t o be di s t al i n l ocat i on and di ffus e i n nat ure. It i s probabl y part i al l y at t ri but abl e t o chroni c reject i on of t he organ and i s not cl os el y rel at ed t o t he pres ence of t he s t andard ri s k fact ors for coronary di s eas e.
IV. Valvular Heart Disease A. Aortic stenosis
1. Etiology o
o
a. Congenital aortic stenosis us ual l y i s det ect ed i n pedi at ri c pat i ent s but occas i onal l y becomes apparent i n earl y adul t hood.
o
o
b. Senile calcific aortic stenosis occurs when s carri ng and cal ci fi cat i on of a t ri cus pi d aort i c val ve l ead t o ori fi ce narrowi ng i n t he s i xt h, s event h, and ei ght h decades of l i fe. Al t hough once cons i dered a “degenerat i ve― i di opat hi c di s eas e, i t i s now cl ear t hat t he pat hol ogy l eadi ng up t o s evere aort i c s t enos i s i s due t o prol i ferat i ve and i nfl ammat ory changes l eadi ng t o cal ci fi cat i on, s i mi l ar t o t he devel opment of t he pl aque i n ASCAD.
o
Pa g e 1 2 1
ABC Ambe r CHM Conve rte r Tria l ve rsion, http://w w w .proce sste x t.com/a bcchm.html
o
c. Bicuspid aortic valve i s a common congeni t al cardi ac abnormal i t y. The fl ow charact eri s t i cs of t he bi cus pi d val ve are more t urbul ent t han t hos e of t he normal val ve, l eadi ng t o val ve i njury, cal ci fi cat i on, and s t enos i s i n t he fourt h and fi ft h decades of l i fe.
o
o
d. Rheumatic aortic stenosis never occurs al one and i s al ways as s oci at ed wi t h mi t ral val ve di s eas e.
2. Pathophysiology. Aort i c val ve s t enos i s produces a pres s ure overl oad on t he l eft vent ri cl e due t o t he great er pres s ure t hat mus t be generat ed t o force bl ood pas t t he s t enot i c val ve. Thi s commonl y l eads t o compens at ory l eft vent ri cul ar hypert rophy.
3. Clinical features o
o
a. Symptoms. As ympt omat i c pat i ent s wi t h aort i c s t enos i s are at l i t t l e ri s k for s udden deat h. However, t hi s ri s k i ncreas es dramat i cal l y when s ympt oms devel op.
(1) Angina occurs i n 35%–50% of pat i ent s wi t h aort i c s t enos i s .
(a) Fi ft y percent of pat i ent s who devel op t hi s s ympt om di e wi t hi n 5 years of i t s ons et unl es s aort i c val ve repl acement i s performed.
Pa g e 1 2 2
ABC Ambe r CHM Conve rte r Tria l ve rsion, http://w w w .proce sste x t.com/a bcchm.html
(b) Al t hough t he exact mechani s m of angi na i s unknown, current dat a s ugges t t hat coronary bl ood fl ow res erve i s i mpai red i n t he s everel y hypert rophi ed l eft vent ri cl e. Impai rment of t he coronary bl ood fl ow res erve l i mi t s oxygen del i very t o t he myocardi um and produces angi na duri ng exerci s e.
(2) Syncope occurs duri ng exerci s e when t ot al peri pheral res i s t ance fal l s due t o l ocal aut oregul at ory mechani s ms [s ee IX B, C 1 b (2) (b)]. W hen aort i c s t enos i s i s pres ent , cardi ac out put acros s t he s t enot i c aort i c val ve cannot i ncreas e duri ng exerci s e. Becaus e t ot al peri pheral res i s t ance fal l s , bl ood pres s ure mus t al s o fal l , and s yncope occurs .
(a) Other causes of s yncope i n aort i c s t enos i s i ncl ude a refl ex vas odepres s or res pons e t o hi gh i nt ravent ri cul ar pres s ure, atrial or ventricular arrhythmias, and heart block as a res ul t of conduct i on s ys t em cal ci fi cat i on. P.19
(b) Aft er s yncope occurs i n pat i ent s wi t h aort i c s t enos i s , expect ed s urvi val i s 2–3 years wi t hout val ve repl acement .
Pa g e 1 2 3
ABC Ambe r CHM Conve rte r Tria l ve rsion, http://w w w .proce sste x t.com/a bcchm.html
(3) Heart failure. Fi ft y percent of pat i ent s who devel op heart fai l ure di e wi t hi n 1–2 years of pres ent at i on i f t he s t enos i s i s not correct ed. Heart fai l ure occurs becaus e t he aft erl oad pl aced on t he myocardi um becomes exces s i ve and bot h cont ract i l e dys funct i on and di as t ol i c mus cl e dys funct i on occur when t he myocardi um i s expos ed t o a prol onged, s evere pres s ure overl oad.
o
o
b. Physical signs
(1) Delayed carotid upstroke. In t he pres ence of aort i c s t enos i s , t he carot i d ups t roke t ypi cal l y i s del ayed i n t i mi ng and reduced i n vol ume (pul s us parvus et t ardus ). Thi s fi ndi ng i s t he mos t rel i abl e phys i cal s i gn i n gaugi ng t he s everi t y of t he di s eas e.
(2) Systolic ejection murmur. A hars h, l at e-peaki ng s ys t ol i c eject i on murmur i s heard i n t he aort i c area and i s t rans mi t t ed t o t he carot i d art eri es . The murmur al s o may be refl ect ed t o t he mi t ral area, produci ng t he fal s e i mpres s i on t hat mi t ral regurgi t at i on al s o i s pres ent (Gallavardin' s phenomenon).
(3) Soft, single S 2 . Becaus e t he aort i c val ve i s s t enot i c, i t s mot i on i s s everel y i mpai red. The reduct i on i n mot i on of t he val ve caus es t he aortic component (A 2 ) of t he S 2 t o be abs ent . Thus , t he onl y component of t he S 2
Pa g e 1 2 4
ABC Ambe r CHM Conve rte r Tria l ve rsion, http://w w w .proce sste x t.com/a bcchm.html
t hat i s heard i s t he pulmonic component (P 2 ) , whi ch i s normal l y s oft .
(4) S 4 . An S 4 us ual l y i s heard as a res ul t of t he reduced l eft vent ri cul ar compl i ance t hat occurs i n l eft vent ri cul ar hypert rophy.
(5) Sustained, forceful apex beat. The poi nt of maxi mal cardi ac i mpul s e us ual l y i s not di s pl aced unl es s heart fai l ure has occurred. However, t he i mpul s e i s s us t ai ned and forceful t hroughout s ys t ol e.
4. Laboratory diagnosis o
o
a. Electrocardiography. The ECG us ual l y s hows evi dence of l eft vent ri cul ar hypert rophy.
o
o
b. Echocardiography can rul e out s i gni fi cant aort i c s t enos i s i f val ve mot i on i s s hown t o be normal . However, Doppl er exami nat i on of t he aort i c out fl ow t ract duri ng echocardi ography can accurat el y meas ure t he pres s ure gradi ent acros s t he aort i c val ve, and can be us ed t o cal cul at e t he val ve area.
o
o
c. Cardiac catheterization. Di agnos i s and eval uat i on of t he s everi t y of aort i c s t enos i s may be confi rmed by cardi ac cat het eri zat i on, duri ng whi ch t he pres s ure gradi ent acros s t he val ve i s meas ured and t he degree of s t enos i s i s cal cul at ed.
Pa g e 1 2 5
ABC Ambe r CHM Conve rte r Tria l ve rsion, http://w w w .proce sste x t.com/a bcchm.html
5. T herapy o
o
a. Palliative therapy
(1) Medical therapy has no defi ni t i ve rol e i n t he t reat ment of aort i c s t enos i s , but diuretics may t emporari l y rel i eve t he s ympt oms of vol ume overl oad unt i l mechani cal rel i ef of t he obs t ruct i on i s performed.
(2) Ins ert i on and i nfl at i on of a l arge bal l oon i n t he aort i c val ve ori fi ce (balloon valvuloplasty) al s o may produce a moderat e i mprovement i n t he amount of obs t ruct i on and i n t he s ympt oms , but rel i ef from us i ng t hi s t echni que i s us ual l y onl y t emporary. It does not reduce t he mort al i t y expect ed i f t he di s eas e i s l eft unt reat ed.
b. Curative therapy requi res aortic valve replacement, whi ch may be performed us i ng a pres erved human homograft val ve, a bi opros t het i c het erograft val ve, a mechani cal val ve, or a pul monary aut ograft .
(1) Homograft valves. The hemodynami c fl ow pat t ern i s excel l ent , and pat i ent s wi t h homograft val ves do not requi re ant i coagul at i on t herapy. Avai l abi l i t y of t hes e val ves i s l i mi t ed, becaus e mos t s ui t abl e donors are al s o
Pa g e 1 2 6
ABC Ambe r CHM Conve rte r Tria l ve rsion, http://w w w .proce sste x t.com/a bcchm.html
accept abl e for whol e heart donat i on i n cardi ac t rans pl ant at i on.
(2) Heterograft valves. Pat i ent s wi t h het erograft val ves do not requi re ant i coagul at i on t herapy, but t he durabi l i t y of t he val ve i s l i mi t ed, and det eri orat i on aft er 10 years i s common.
(3) Mechanical valves. Pat i ent s wi t h mechani cal val ves do requi re ant i coagul at i on t herapy. However, t hes e val ves are more durabl e t han bi opros t hes es .
(4) Autograft (Ross procedure). In t hi s procedure, t he pat i ent 's normal pul monary val ve i s t rans pl ant ed i nt o t he aort i c pos i t i on, where i t has excel l ent durabi l i t y and l ongevi t y. A P.20
homograft i s t hen i ns ert ed i nt o t he pul monary pos i t i on, where l ow pres s ure enhances homograft l ongevi t y. In pract i ced hands , t he res ul t s of t hi s procedure are excel l ent .
B. Mitral stenosis
1. Etiology. Al mos t al l cas es of mi t ral s t enos i s i n adul t s are secondary to rheumatic heart disease. Mos t cas es
Pa g e 1 2 7
ABC Ambe r CHM Conve rte r Tria l ve rsion, http://w w w .proce sste x t.com/a bcchm.html
occur i n women.
2. Pathophysiology o
o
a. Mi t ral val ve s t enos i s i mpedes l eft vent ri cul ar fi l l i ng, t hereby i ncreas i ng l eft at ri al pres s ure as a pres s ure gradi ent devel ops acros s t he mi t ral val ve. El evat ed l eft at ri al pres s ure i s referred t o t he l ungs , where i t produces pulmonary congestion. As t he s t enos i s becomes more s evere, i t may s i gni fi cant l y reduce forward cardi ac out put .
o
o
b. Becaus e t he ri ght vent ri cl e i s res pons i bl e for fi l l i ng t he l eft vent ri cl e, t he burden of propel l i ng bl ood acros s t he s t enot i c mi t ral val ve i s borne by t he ri ght vent ri cl e. The overl oad on t he ri ght vent ri cl e may be i ncreas ed furt her when s econdary pul monary vas ocons t ri ct i on occurs . Thus , t he ri ght vent ri cl e mus t generat e enough force bot h t o overcome t he res i s t ance offered by t he s t enot i c val ve and t o propel bl ood t hrough cons t ri ct ed pul monary art eri es . Cons equent l y, pul monary art eri al pres s ure may i ncreas e t o t hree t o fi ve t i mes normal , event ual l y res ul t i ng i n right ventricular failure.
3. Clinical features o
o
a. Symptoms
(1) Left-sided failure. Dyspnea on exertion,
Pa g e 1 2 8
ABC Ambe r CHM Conve rte r Tria l ve rsion, http://w w w .proce sste x t.com/a bcchm.html
orthopnea, and paroxysmal nocturnal dyspnea occur as a res ul t of reduced l eft vent ri cul ar out put and i ncreas ed l eft at ri al pres s ure. In mi t ral s t enos i s , t he s ympt oms of l eft vent ri cul ar fai l ure us ual l y are not at t ri but abl e t o l eft vent ri cul ar dys funct i on but , rat her, t o t he mi t ral s t enos i s i t s el f.
(2) Right-sided failure. W hen pul monary hypert ens i on occurs , t he ri ght vent ri cl e may fai l , produci ng edema, ascites, anorexia, and fatigue.
(3) Hemoptysis. The hi gh l eft at ri al pres s ure produced i n mi t ral s t enos i s may l ead t o rupt ure of s mal l bronchi al vei ns , produci ng hemopt ys i s .
(4) Systemic embolism. St agnat i on of bl ood i n t he enl arged l eft at ri um and l eft at ri al appendage occurs i n mi t ral s t enos i s , part i cul arl y i f at ri al fi bri l l at i on i s pres ent . Under t hes e ci rcums t ances , a t hrombus may form i n t he l eft at ri um and can become a s ource of s ys t emi c embol i s m.
(5) Hoarseness may occur i n mi t ral s t enos i s as t he enl arged l eft at ri um i mpi nges on t he l eft recurrent l aryngeal nerve (Ort ner's s yndrome).
o
o
b. Physical signs
Pa g e 1 2 9
ABC Ambe r CHM Conve rte r Tria l ve rsion, http://w w w .proce sste x t.com/a bcchm.html
(1) Atrial fibrillation. Frequent l y, an i rregul arl y i rregul ar cardi ac rhyt hm i ndi cat i ve of at ri al fi bri l l at i on i s pres ent .
(2) Pulmonary rales. Bi l at eral pul monary ral es occur s econdary t o el evat ed l eft at ri al and pul monary venous pres s ures .
(3) Increased intensity of the S 1 . The S 1 us ual l y i ncreas es i n i nt ens i t y becaus e t he t rans mi t ral gradi ent l i mi t s s pont aneous di as t ol i c mi t ral val ve cl os ure. Thus , t he mi t ral val ve remai ns open unt i l vent ri cul ar s ys t ol e cl os es i t forci bl y, res ul t i ng i n an i ncreas e i n S 1 i nt ens i t y. Lat e i n t he cours e of t he di s eas e, t he val ve may become s o s t enot i c t hat i t no l onger opens or cl os es , reduci ng t he i nt ens i t y of S 1 .
(4) Increased intensity of the P 2 component of t he S 2 . The P 2 component of t he S 2 i s us ual l y i ncreas ed i n i nt ens i t y i f pul monary hypert ens i on has devel oped.
(5) Opening snap. An opening snap is heard following the S 2 as t he s t enot i c val ve i s forced open i n di as t ol e by t he hi gh l eft at ri al fi l l i ng pres s ure. The hi gher t he pres s ure, t he s ooner t he mi t ral val ve opens . Thus , a s hort i nt erval (<0.10 s econd i n durat i on) i ndi cat es rel at i vel y hi gh l eft at ri al pres s ure and s evere
Pa g e 1 3 0
ABC Ambe r CHM Conve rte r Tria l ve rsion, http://w w w .proce sste x t.com/a bcchm.html
s t enos i s .
(6) Diastolic rumble. The murmur of mi t ral s t enos i s i s a l ow-pi t ched api cal rumbl e, whi ch begi ns aft er t he openi ng s nap. If t he pat i ent i s i n s i nus rhyt hm, at ri al s ys t ol e produces a pres ys t ol i c accent uat i on of t hi s murmur.
(7) Sternal lift. Enl argement of t he ri ght vent ri cl e as a res ul t of pul monary hypert ens i on produces a s ys t ol i c l i ft of t he s t ernum. P.21
(8) Other symptoms. Neck vein distention, edema, hepatic enlargement, and ascites may be pres ent i f ri ght vent ri cul ar fai l ure occurs .
4. Laboratory diagnosis o
o
a. Electrocardiography. The ECG may s how at ri al fi bri l l at i on as wel l as s i gns of l eft at ri al enl argement and ri ght vent ri cul ar hypert rophy.
o
o
b. Chest radiography
(1) St rai ght eni ng of t he l eft heart border and
Pa g e 1 3 1
ABC Ambe r CHM Conve rte r Tria l ve rsion, http://w w w .proce sste x t.com/a bcchm.html
a doubl e dens i t y al ong t he ri ght heart border (formed by t he ri ght and l eft at ri a) occur as a res ul t of l eft at ri al enl argement .
(2) Si gns of pul monary venous hypert ens i on, i ncl udi ng an i ncreas e i n pul monary vas cul ar marki ngs and Kerley' s lines, are l i kel y t o be pres ent .
(3) W hen pulmonary hypertension l eads t o ri ght vent ri cul ar enl argement , t he l at eral vi ew s hows a loss of the retrosternal airspace.
o
o
c. Echocardiography us ual l y provi des excel l ent i mages of t he mi t ral val ve.
(1) The echocardi ogram s hows reduct i on i n t he excurs i on of t he val ve l eafl et s and t hi ckeni ng of t he val ve. Two-di mens i onal echocardi ography can be us ed t o vi s ual i ze and meas ure t he res i dual mi t ral val ve ori fi ce. Invari abl y, l eft at ri al enl argement i s pres ent .
(2) Doppl er exami nat i on of t he mi t ral val ve may al s o hel p t o quant i fy t he s everi t y of t he s t enos i s .
5. T herapy o
o
a. Medical therapy i s res erved for pat i ent s wi t h
Pa g e 1 3 2
ABC Ambe r CHM Conve rte r Tria l ve rsion, http://w w w .proce sste x t.com/a bcchm.html
mi l d-t o-moderat e s ympt oms of l eft -s i ded fai l ure.
(1) Diuretics. The mai ns t ay of t reat ment , t hes e agent s are us ed t o control pulmonary congestion and t o limit dyspnea and orthopnea.
(2) Digitalis. Becaus e l eft vent ri cul ar mus cl e funct i on us ual l y i s normal i n mi t ral s t enos i s , t he us e of di gi t al i s i s of l i t t l e benefi t t o pat i ent s i n s i nus rhyt hm. In pat i ent s i n atrial fibrillation, however, di gi t al i s i s us ed t o s l ow vent ri cul ar rat e. A rapi d vent ri cul ar rat e i n mi t ral s t enos i s s hort ens di as t ol e, t hereby reduci ng l eft vent ri cul ar fi l l i ng, whi ch, i n t urn, furt her i ncreas es l eft at ri al pres s ure and reduces cardi ac out put . β-Blockers and diltiazem or verapamil may be added t o di goxi n i f furt her heart rat e cont rol i s neces s ary.
(3) Anticoagulants. Pat i ent s wi t h mi t ral s t enos i s and coexi s t ent at ri al fi bri l l at i on have a high incidence of systemic embolism. In s uch pat i ent s , ant i coagul at i on t herapy (e.g., wi t h warfarin) us ual l y i s i ndi cat ed.
o
o
b. Balloon valvuloplasty. Unl i ke bal l oon val vul opl as t y for aort i c s t enos i s , bal l oon val vul opl as t y for mi t ral s t enos i s can offer effect i ve l ong-t erm i mprovement .
Pa g e 1 3 3
ABC Ambe r CHM Conve rte r Tria l ve rsion, http://w w w .proce sste x t.com/a bcchm.html
(1) Val vul opl as t y for mi t ral s t enos i s produces a commi s s urot omy s i mi l ar t o t hat produced at open heart s urgery. Duri ng bal l oon mi t ral val vul opl as t y, t rans s ept al cat het eri zat i on of t he i nt erat ri al s ept um i s performed, al l owi ng pas s age of t he bal l oon cat het er from t he ri ght at ri um t o t he l eft at ri um. From t he l eft at ri um, t he bal l oon cat het er i s advanced t o t he mi t ral val ve and i s i nfl at ed.
(2) Echocardi ography i s es s ent i al i n predi ct i ng i mmedi at e and l ong-t erm s ucces s . Fact ors i nfl uenci ng t he feas i bi l i t y and s ucces s of val vul opl as t y i ncl ude t he degrees of mi t ral regurgi t at i on, val vul ar t hi ckeni ng, cal ci fi cat i on, and mobi l i t y, and t he degree of s ubval vul ar i nvol vement .
o
o
c. Surgical therapy i s effect i ve i n rel i evi ng t he s ympt oms of mi t ral s t enos i s and i n prol ongi ng l i fe i n s ympt omat i c pat i ent s . Surgery s houl d be performed pri or t o t he devel opment of pul monary hypert ens i on, whi ch i ncreas es s urgi cal ri s k. However, i f pul monary hypert ens i on i s pres ent and s urgery i s s ucces s ful , pul monary hypert ens i on us ual l y regres s es pos t operat i vel y.
C. Aortic regurgitation (or aortic insufficiency)
1. Etiology o
Pa g e 1 3 4
ABC Ambe r CHM Conve rte r Tria l ve rsion, http://w w w .proce sste x t.com/a bcchm.html
o
a. Idiopathic aortic root dilation. Aort i c root di l at i on, a common caus e of aort i c regurgi t at i on, occurs more frequent l y i n pat i ent s wi t h hypert ens i on but correl at es bes t wi t h i ncreas i ng age. It i s al s o s een more frequent l y i n pat i ent s wi t h bi cus pi d aort i c val ves .
o
o
b. Rheumatic heart disease. Aort i c i ns uffi ci ency us ual l y i s pres ent t o s ome degree i n mos t cas es of rheumat i c heart di s eas e. Mi t ral s t enos i s us ual l y predomi nat es , but occas i onal l y aort i c i ns uffi ci ency i s t he mos t s evere mani fes t at i on of rheumat i c heart di s eas e. P.22
o
o
c. Infective endocarditis. Infect i on of t he aort i c val ve may l ead t o perforation or partial destruction of one or more aortic leaflets, produci ng aort i c i ns uffi ci ency.
o
o
d. Marfan syndrome may produce aort i c i ns uffi ci ency i n t wo ways .
(1) Proximal root dilation. The ext reme expans i on of t he proxi mal aort i c root s een i n Marfan s yndrome may produce aort i c i ns uffi ci ency.
(2) Aortic root dissection. The advanced
Pa g e 1 3 5
ABC Ambe r CHM Conve rte r Tria l ve rsion, http://w w w .proce sste x t.com/a bcchm.html
cys t i c medi al necros i s pres ent i n Marfan s yndrome may l ead t o an i nt i mal t ear and di s s ect i on of t he aort a. If t he di s s ect i on i nvol ves t he proxi mal aort i c root , t he s upport i ng s t ruct ures of t he aort i c val ve are di s rupt ed, and t he val ve i s rendered i ncompet ent . o
o
e. Aortic dissection. Any caus e of aort i c di s s ect i on ot her t han Marfan s yndrome may l ead t o aort i c i ns uffi ci ency.
o
o
f. Syphilis may produce aortitis, whi ch may ext end t o t he aort i c val ve and produce aort i c i ncompet ence.
o
o
g. Collagen vascular disease. Sys t emi c l upus eryt hemat os us (SLE) and ankyl os i ng s pondyl i t i s may caus e aort i c i ns uffi ci ency.
2. Pathophysiology o
o
a. A port i on of t he l eft vent ri cul ar s t roke vol ume eject ed duri ng s ys t ol e regurgi t at es i nt o t he l eft vent ri cl e duri ng di as t ol e. If no compens at i on occurs , l eft vent ri cul ar forward out put decreas es . However, chroni c regurgi t at i on of bl ood i nt o t he l eft vent ri cl e produces eccent ri c cardi ac hypert rophy and an i ncreas e i n end-di as t ol i c vol ume. The s t roke vol ume (end-di as t ol i c vol ume mi nus end-s ys t ol i c vol ume) t herefore i ncreas es , hel pi ng t o compens at e for t he vol ume t hat i s regurgi t at ed. The i ncreas e i n
Pa g e 1 3 6
ABC Ambe r CHM Conve rte r Tria l ve rsion, http://w w w .proce sste x t.com/a bcchm.html
t ot al s t roke vol ume l eads t o an i ncreas e i n pul s e pres s ure and i ncreas ed s ys t ol i c pres s ure. The addi t i onal devel opment of concent ri c hypert rophy compens at es t hi s s econd t ype of overl oad. T he additional volume and pressure that the left ventricle must generate eventually lead to left ventricular dysfunction and CHF. o
o
b. An addi t i onal pat hophys i ol ogi c cons equence of aort i c i ns uffi ci ency i s a reduction in systemic diastolic blood pressure.
3. Clinical features o
o
a. Symptoms
(1) Left ventricular failure
(a) Chronic aortic insufficiency may caus e l eft vent ri cul ar dys funct i on, l eadi ng t o s ympt oms of dyspnea, orthopnea, and paroxysmal nocturnal dyspnea.
(b) In acute aortic insufficiency, normal mus cl e funct i on may coexi s t wi t h heart fai l ure. In t hi s ci rcums t ance, reduced forward out put and el evat ed l eft vent ri cul ar fi l l i ng pres s ure occur pri or t o compensatory left ventricular enlargement.
Pa g e 1 3 7
ABC Ambe r CHM Conve rte r Tria l ve rsion, http://w w w .proce sste x t.com/a bcchm.html
(2) Syncope. Reduct i on i n di as t ol i c s ys t emi c art eri al pres s ure produces a reduct i on i n mean art eri al pres s ure. If t he mean art eri al pres s ure i s reduced s i gni fi cant l y, cerebral perfus i on i s compromi s ed, and s yncope may occur.
(3) Angina occurs l es s commonl y i n aort i c i ns uffi ci ency t han i n aort i c s t enos i s . The caus e of angi na i n aort i c i ns uffi ci ency i s reduced coronary blood flow. Coronary bl ood fl ow occurs pri mari l y i n di as t ol e and i s dri ven by t he aort i c di as t ol i c bl ood pres s ure. Thi s dri vi ng pres s ure i s reduced i n aort i c i ns uffi ci ency, i n t urn reduci ng coronary bl ood fl ow.
o
o
b. Physical signs
(1) Left ventricular impulse. The PMI i s hyperdynamic and i s displaced downward and to the left as a res ul t of l eft vent ri cul ar enl argement .
(2) Diastolic murmur. The murmur of aort i c i ns uffi ci ency i s a high-pitched, diastolic blowing murmur heard al ong t he l eft s t ernal border. Oft en t he murmur i s heard bes t when t he pat i ent i s s i t t i ng up and l eani ng forward.
Pa g e 1 3 8
ABC Ambe r CHM Conve rte r Tria l ve rsion, http://w w w .proce sste x t.com/a bcchm.html
(3) Austin Flint murmur. A low-pitched diastolic rumble s i mi l ar t o t hat heard i n mi t ral s t enos i s may be pres ent i n pat i ent s wi t h aort i c i ns uffi ci ency. The Aus t i n Fl i nt murmur usually indicates moderate-to-severe insufficiency. The murmur i s bel i eved t o be caus ed by reverberat i on of t he regurgi t ant fl ow agai ns t t he mi t ral val ve, al t hough t he exact mechani s m i s uncl ear.
(4) T otal stroke volume and cons equent l y, pulse pressure, increases i n chroni c aort i c i ns uffi ci ency. The i ncreas ed s t roke vol ume and pul s e pres s ure l ead t o many phys i cal s i gns , s ome of whi ch are i ncl uded i n t he fol l owi ng. P.23
(a) Corrigan' s pulse. The carot i d pul s e has a rapi d ri s e and ful l ups t roke wi t h a rapi d fal l i n di as t ol e.
(b) Hill' s sign refers t o a di s proport i onat e i ncreas e of s ys t ol i c bl ood pres s ure (i .e., >30 mm Hg) when meas ured i n t he l eg, as compared wi t h t he s ys t ol i c bl ood pres s ure meas ured i n t he arm. Thi s s i gn s ugges t s s evere aort i c i ns uffi ci ency.
Pa g e 1 3 9
ABC Ambe r CHM Conve rte r Tria l ve rsion, http://w w w .proce sste x t.com/a bcchm.html
(c) Pistol-shot femoral pulses. Aus cul t at i on over t he femoral art eri es reveal s a pul s e t hat s ounds l i ke a pi s t ol s hot .
(d) Duroziez' s sign. A s t et hos cope i s pl aced over t he femoral art ery wi t h enough pres s ure t o produce a s ys t ol i c brui t . The concomi t ant occurrence of a di as t ol i c brui t cons t i t ut es Durozi ez's s i gn.
(e) de Musset' s sign refers t o a bobbi ng movement of t he head caus ed by t he i ncreas ed s t roke vol ume and pul s e pres s ure.
(f) Quincke' s pulse i s s ys t ol i c bl us hi ng and di as t ol i c bl anchi ng of t he nai l bed when gent l e pres s ure i s pl aced on t he nai l .
(5) Acute aortic insufficiency. The precedi ng s i gns are us ual l y abs ent i n ac ut e aort i c i ns uffi ci ency becaus e compens at ory i ncreas es i n end-di as t ol i c vol ume and s t roke vol ume have not yet occurred. In fact , t he cl i ni cal pi ct ure of acut e s evere aort i c i ns uffi ci ency i s remarkabl y bl and. The api cal i mpul s e i s not enl arged. S 1 i s s oft becaus e i ncreas ed l eft vent ri cul ar end-di as t ol i c pres s ure cl os es t he mi t ral val ve before s ys t ol e. Thi s fi ndi ng marks
Pa g e 1 4 0
ABC Ambe r CHM Conve rte r Tria l ve rsion, http://w w w .proce sste x t.com/a bcchm.html
a poor prognos i s for pat i ent s t reat ed wi t hout val ve repl acement .
4. Diagnosis o
o
a. Electrocardiography. The ECG us ual l y s hows l eft vent ri cul ar hypert rophy. In endocardi t i s , a prol onged PR i nt erval may i ndi cat e abs ces s format i on i nvol vi ng t he conduct i on s ys t em.
o
o
b. Chest radiography. Unl es s t he aort i c i ns uffi ci ency i s mi l d or acut e, cardiac enlargement i s us ual l y pres ent , and oft en t he proxi mal aort a i s di l at ed. The abs ence of cardi ac enl argement i s evi dence agai ns t t he di agnos i s of s evere chroni c aort i c i ns uffi ci ency.
o
o
c. Echocardiography. Evi dence of an enl arged l eft vent ri cul ar cavi t y i s us ual l y pres ent i n aort i c i ns uffi ci ency. Frequent l y, di as t ol i c vi brat i on of t he mi t ral val ve i s pres ent , produced by t he regurgi t ant fl ow s t ri ki ng t he val ve. Doppl er exami nat i on of t he aort i c out fl ow t ract reveal s abnormal di as t ol i c fl ow from t he aort a t o t he l eft vent ri cl e, whi ch may be anal yzed quant i t at i vel y.
o
o
d. Cardiac catheterization. Aort ography may be performed at t he t i me of cardi ac cat het eri zat i on. Thi s i s us eful i f noni nvas i ve t es t i ng i s not di agnos t i c or di s cordant wi t h cl i ni cal fi ndi ngs .
Pa g e 1 4 1
ABC Ambe r CHM Conve rte r Tria l ve rsion, http://w w w .proce sste x t.com/a bcchm.html
5. T herapy. If aort i c i ns uffi ci ency i s s evere, event ual aortic valve replacement i s neces s ary. o
o
a. Ti mi ng of s urgery i s di ffi cul t , however, becaus e t he l es i on may be t ol erat ed for s everal years . Careful fol l ow-up i s requi red t o det ect earl y s i gns of decompens at i on; at t hi s t i me, val ve repl acement i s advi s abl e. In mos t cas es , val ve repl acement s houl d be performed before t he l eft vent ri cul ar echocardi ographi c end-s ys t ol i c di mens i on exceeds 55 mm and t he eject i on fract i on fal l s bel ow 55%.
o
o
b. For t hos e who do not yet meet t he cri t eri a for s urgery, vasodilator therapy wi t h di hydropyri di ne cal ci um channel bl ockers or ACE i nhi bi t ors can i mprove hemodynami cs and may del ay ons et of l eft vent ri cul ar dys funct i on and t he need for s urgery.
D. Mitral regurgitation (or mitral insufficiency)
1. Etiology o
o
a. Mitral valve prolapse is charact eri zed by redundant mi t ral val ve l eafl et s or chordae t hat permi t s ys t ol i c prol aps e of t he mi t ral val ve i nt o t he l eft at ri um wi t h res ul t ant mi t ral regurgi t at i on.
(1) Thi s s yndrome us ual l y i s beni gn, but i n s ome cas es i t may be as s oci at ed wi t h s i gni fi cant mi t ral regurgi t at i on. Addi t i onal
Pa g e 1 4 2
ABC Ambe r CHM Conve rte r Tria l ve rsion, http://w w w .proce sste x t.com/a bcchm.html
compl i cat i ons i ncl ude at ypi cal ches t pai n, cardi ac arrhyt hmi as , and an i ncreas ed ri s k of endocardi t i s . Mos t cl i ni cal l y i mport ant s equel ae occur i n t hos e pat i ent s whos e mi t ral val ves are cl earl y t hi ckened and echocardi ographi cal l y abnormal .
(2) A mi ds ys t ol i c cl i ck and a l at e s ys t ol i c murmur t ypi cal l y are heard on phys i cal exami nat i on.
o
o
b. Coronary artery disease may l ead t o i s chemi a or i nfarct i on of t he papi l l ary mus cl es t o whi ch t he mi t ral val ve i s t et hered, t hereby produci ng mi t ral i ncompet ence. P.24
o
o
c. Rheumatic heart disease. Scarri ng and ret ract i on of t he mi t ral l eafl et s as a res ul t of rheumat i c heart di s eas e caus e mi t ral regurgi t at i on.
o
o
d. Ruptured chordae tendineae. Spont aneous rupt ure of t he chordae t endi neae may occur i n ot herwi s e heal t hy i ndi vi dual s . Chordal rupt ure permi t s prol aps e of a port i on of a mi t ral val ve l eafl et i nt o t he l eft at ri um, renderi ng t he val ve i ncompet ent .
o
o
e. Infective endocarditis. Infect i on of t he mi t ral val ve may caus e i t s des t ruct i on wi t h s ubs equent
Pa g e 1 4 3
ABC Ambe r CHM Conve rte r Tria l ve rsion, http://w w w .proce sste x t.com/a bcchm.html
regurgi t at i on.
2. Pathophysiology. Mi t ral regurgi t at i on permi t s a port i on of t he l eft vent ri cul ar s t roke vol ume t o be pumped backward i nt o t he l eft at ri um i ns t ead of forward i nt o t he aort a, res ul t i ng i n increased left atrial pressure and decreased forward cardiac output. Prel oad i s i ncreas ed by t he vol ume overl oad, and aft erl oad i s i ni t i al l y decreas ed as t he l eft vent ri cl e empt i es a port i on of i t s cont ent s i nt o t he rel at i vel y (i .e., compared wi t h t he aort a) l ow-pres s ure l eft at ri um. Thi s augment s eject i on performance and hel ps compens at e for t he regurgi t at i on. o
o
a. Ini t i al l y, compl i ance of t he l eft at ri um i s l ow, and t he regurgi t ant vol ume produces hi gh l eft at ri al pres s ure wi t h res ul t ant conges t i ve s ympt oms .
o
o
b. W i t h t i me, t he l eft at ri al compl i ance and vol ume i ncreas e, al l owi ng accommodat i on of t he regurgi t ant vol ume at more phys i ol ogi c fi l l i ng pres s ures .
o
o
c. The devel opment of l eft vent ri cul ar eccent ri c cardi ac hypert rophy res t ores forward s t roke vol ume.
o
o
d. Aft er a prol onged peri od of compens at i on, l eft vent ri cul ar mus cl e dys funct i on event ual l y occurs , res ul t i ng i n a fal l i n eject i on fract i on from s upranormal t o normal or even s ubnormal val ues .
Pa g e 1 4 4
ABC Ambe r CHM Conve rte r Tria l ve rsion, http://w w w .proce sste x t.com/a bcchm.html
3. Clinical features o
o
a. Symptoms. Charact eri s t i cs i ncl ude t hos e of l eft vent ri cul ar fai l ure (i .e., dyspnea, orthopnea, and paroxysmal nocturnal dyspnea).
(1) If mi t ral regurgi t at i on i s s evere and chroni c, pulmonary hypertension and symptoms of right-sided failure al s o may occur.
(2) Pat i ent s i n at ri al fi bri l l at i on may experi ence symptoms of systemic embolization. The ri s k of embol i zat i on appears t o be l es s i n pat i ent s wi t h mi t ral regurgi t at i on t han i n t hos e wi t h mi t ral s t enos i s , al t hough t hi s i s debat abl e.
o
o
b. Physical signs
(1) Left ventricular impulse. As wi t h aort i c regurgi t at i on, t he PMI i s hyperdynami c and di s pl aced downward and t o t he l eft .
(2) Murmur. The murmur of mi t ral regurgi t at i on i s a hol os ys t ol i c api cal murmur t hat radi at es t o t he axi l l a. It does not vary i n i nt ens i t y wi t h vari at i on i n R-R i nt erval .
(3) An S 3 us ual l y i s heard i n mi t ral
Pa g e 1 4 5
ABC Ambe r CHM Conve rte r Tria l ve rsion, http://w w w .proce sste x t.com/a bcchm.html
regurgi t at i on and may occur even i n t he abs ence of overt heart fai l ure. The S 3 i s caus ed by t he rapi d fi l l i ng of t he l eft vent ri cl e by t he l arge vol ume of bl ood accumul at ed i n t he l eft at ri um duri ng s ys t ol e.
4. Diagnosis o
o
a. Electrocardiography. The ECG s hows s i gns of l eft vent ri cul ar hypert rophy and l eft at ri al enl argement .
o
o
b. Chest radiography s hows cardi ac enl argement . Vas cul ar conges t i on i ndi cat es heart fai l ure.
o
o
c. Echocardiography
(1) In cas es of a ruptured chorda or mitral valve prolapse, t he mi t ral val ve can be s een ext endi ng i nt o t he l eft at ri um duri ng s ys t ol e.
(2) W hen t he mi t ral val ve has been damaged by endocarditis, veget at i ons on t he mi t ral l eafl et s frequent l y are demons t rat ed. Trans es ophageal echocardi ography i s bet t er t han t rans t horaci c echocardi ography for det ect i ng veget at i ons .
(3) Regardl es s of t he caus e of t he mi t ral regurgi t at i on, left atrial and left ventricular
Pa g e 1 4 6
ABC Ambe r CHM Conve rte r Tria l ve rsion, http://w w w .proce sste x t.com/a bcchm.html
enlargement occur i f t he condi t i on i s bot h chroni c and s evere.
(4) Doppl er exami nat i on reveal s abnormal s ys t ol i c fl ow from t he l eft vent ri cl e i nt o t he l eft at ri um. Quant i t at i ve doppl er t echni ques can accurat el y as s es s t he regurgi t ant ori fi ce area and t he regurgi t ant vol ume, bot h of whi ch are i mport ant i n det ermi ni ng s everi t y.
o
o
d. Cardiac catheterization. Ri ght -heart cat het eri zat i on yi el ds a pulmonary capillary wedge tracing t hat oft en di s pl ays a large v wave repres ent at i ve of t he s ys t ol i c vol ume overl oad on P.25
t he l eft at ri um. Left ventriculography demons t rat es s ys t ol i c regurgi t at i on of cont ras t mat eri al i nt o t he l eft at ri um.
5. T herapy o
o
a. Medical treatment. The goal of medi cal t herapy i s t o rel i eve s ympt oms by i ncreas i ng forward cardi ac out put and reduci ng pul monary venous hypert ens i on.
(1) Digitalis. W hen at ri al fi bri l l at i on occurs , di gi t al i s i s us eful i n cont rol l i ng heart rat e. In chroni c mi t ral regurgi t at i on wi t h mus cl e dys funct i on, t hi s agent may be us eful i n
Pa g e 1 4 7
ABC Ambe r CHM Conve rte r Tria l ve rsion, http://w w w .proce sste x t.com/a bcchm.html
i ncreas i ng t he i not ropi c s t at e. In cas es of acut e mi t ral regurgi t at i on when no i not ropi c defi ci t exi s t s , i t i s not i ndi cat ed.
(2) Diuretics are us ed t o reduce cent ral vol ume overl oad, whi ch i n t urn reduces pul monary venous hypert ens i on and conges t i on.
(3) Vasodilators. Art eri ol ar vas odi l at ors are particularly useful i n managi ng acut e mi t ral regurgi t at i on. Thes e agent s reduce resistance to aortic outflow, t hereby preferent i al l y i ncreas i ng forward out put whi l e reduci ng t he amount of regurgi t at i on. Vas odi l at ors al s o reduce left ventricular size , whi ch hel ps t o rees t abl i s h mi t ral compet ence.
(4) Anticoagulants. Pat i ent s wi t h mi t ral regurgi t at i on and at ri al fi bri l l at i on are at s ome ri s k for s ys t emi c embol i s m; t herefore, ant i coagul ant s us ual l y are i ndi cat ed.
o
o
b. Surgical treatment. Mi t ral val ve repl acement or repai r i s i ndi cat ed for chroni c mi t ral regurgi t at i on, even i f s ympt oms are mi l d, i f t here i s evi dence of vent ri cul ar dys funct i on.
(1) Valve replacement mus t be performed pri or t o t he ons et of s i gni fi cant mus cl e dys funct i on, whi ch l i mi t s t he s ucces s of
Pa g e 1 4 8
ABC Ambe r CHM Conve rte r Tria l ve rsion, http://w w w .proce sste x t.com/a bcchm.html
operat i ve i nt ervent i on. To hel p ens ure pres ervat i on of vent ri cul ar funct i on, s urgery s houl d occur before t he eject i on fract i on fal l s bel ow 60% or t he end-s ys t ol i c di mens i on exceeds 40 mm.
(2) Valve repair offers s everal advant ages over repl acement , i ncl udi ng el i mi nat i ng t he i nt roduct i on of a pros t hes i s and decreas i ng t he need for ant i coagul at i on t herapy. Furt hermore, repai ri ng, rat her t han repl aci ng, t he val ve hel ps pres erve l eft vent ri cul ar funct i on becaus e t he mi t ral val ve apparat us , whi ch pl ays an i mport ant rol e i n vent ri cul ar cont ract i on i s pres erved.
E. Tricuspid regurgitation
1. Etiology o
o
a. Infective endocarditis. In drug abus ers who i nject drugs under s ept i c condi t i ons , i nfect i ve endocardi t i s i s a common caus e of t ri cus pi d regurgi t at i on.
o
o
b. Right ventricular failure. Sus t ai ned pres s ure or vol ume overl oad on t he ri ght vent ri cl e l eads t o ri ght vent ri cul ar di l at i on and i mproper al i gnment of t he papi l l ary mus cl es , whi ch produces t ri cus pi d regurgi t at i on.
o
o
c. Rheumatic heart disease. In rheumat i c heart
Pa g e 1 4 9
ABC Ambe r CHM Conve rte r Tria l ve rsion, http://w w w .proce sste x t.com/a bcchm.html
di s eas e, t ri cus pi d regurgi t at i on may occur, s econdary t o ri ght vent ri cul ar pres s ure overl oad from l eft -s i ded val vul ar l es i ons . Tri cus pi d regurgi t at i on al s o may occur as a res ul t of pri mary rheumat i c i nvol vement of t he t ri cus pi d val ve.
2. Pathophysiology. Duri ng s ys t ol e, t he dys funct i oni ng t ri cus pi d val ve al l ows bl ood t o fl ow backward i nt o t he ri ght at ri um, l eadi ng t o s ys t emi c venous conges t i on and venous hypert ens i on.
3. Clinical features o
o
a. Symptoms. Ri ght -s i ded fai l ure (i .e., edema, ascites) occurs . In s evere and acut e cas es , hepatic congestion may be s uffi ci ent l y ext ens i ve t o produce right upper quadrant pain. Pas s i ve hepat i c conges t i on al s o may l ead t o hepat ocel l ul ar damage and jaundice.
o
o
b. Physical signs
(1) Right ventricular lift. The enl arged ri ght vent ri cl e may be pal pat ed as a s ys t ol i c l i ft of t he s t ernum.
(2) Murmur. A hol os ys t ol i c murmur t hat i ncreas es wi t h i ns pi rat i on i s heard al ong t he l eft s t ernal border.
Pa g e 1 5 0
ABC Ambe r CHM Conve rte r Tria l ve rsion, http://w w w .proce sste x t.com/a bcchm.html
(3) Jugular venous pulsation. A l arge v wave i s s een i n jugul ar vei ns duri ng s ys t ol e.
(4) Pulsatile liver. Sys t ol i c expans i on of t he l i ver frequent l y i s pres ent .
4. Diagnosis o
o
a. Chest radiography s hows ri ght vent ri cul ar enl argement as an obl i t erat i on of t he ret ros t ernal ai rs pace on t he l at eral vi ew. P.26
o
o
b. Echocardiography demons t rat es enl argement of t he ri ght at ri um and ri ght vent ri cl e. Doppl er exami nat i on i s hi ghl y effect i ve i n demons t rat i ng t ri cus pi d regurgi t at i on.
5. T herapy. Left -s i ded fai l ure frequent l y i s t he caus e of ri ght -s i ded fai l ure and t ri cus pi d regurgi t at i on. Effect i ve t reat ment of l eft -s i ded fai l ure reduces ri ght vent ri cul ar pres s ure overl oad, whi ch may decreas e ri ght vent ri cul ar s i z e, t hereby res t ori ng val vul ar compet ence. If t ri cus pi d regurgi t at i on i s caus ed by organi c val vul ar di s eas e, s urgi cal repai r or repl acement of t he t ri cus pi d val ve may be neces s ary.
V. Cardiomyopathies A. Dilated (congestive) cardiomyopathy
Pa g e 1 5 1
ABC Ambe r CHM Conve rte r Tria l ve rsion, http://w w w .proce sste x t.com/a bcchm.html
1. Definition. Di l at ed cardi omyopat hy i s defi ned as a di mi nut i on i n t he cont ract i l e funct i on of t he l eft , ri ght , or bot h vent ri cl es i n t he abs ence of pres s ure overl oad, vol ume overl oad, or coronary art ery di s eas e. The l os s of cardi ac mus cl e funct i on res ul t s i n CHF.
2. Etiology. The caus e of mos t cas es of di l at ed cardi omyopat hy i s unknown. Vi ral i nfect i on has been i mpl i cat ed i n t he pat hogenes i s of t hi s di s eas e, but proof of caus e general l y i s l acki ng. The fol l owi ng ot her condi t i ons have been l i nked t o cardi omyopat hy. o
o
a. Prolonged ethanol abuse i s t he mos t common revers i bl e caus e of cardi omyopat hy.
o
o
b. Doxorubicin therapy. Hi gh dos es of doxorubi ci n, a commonl y us ed chemot herapeut i c drug, may res ul t i n i rrevers i bl e di l at ed cardi omyopat hy.
o
o
c. Exposure to mercury, lead, or high-dose catecholamines may caus e myocardi al damage and di l at ed cardi omyopat hy.
o
o
d. Endocrinopathies, i ncl udi ng thyrotoxicosis, hypothyroidism, and acromegaly, have been report ed t o caus e di l at ed cardi omyopat hy. In t hyrot oxi cos i s and i n hypot hyroi di s m, t he myopat hy us ual l y i s revers ed when t he endocri nopat hy i s correct ed.
Pa g e 1 5 2
ABC Ambe r CHM Conve rte r Tria l ve rsion, http://w w w .proce sste x t.com/a bcchm.html o
o
e. Metabolic disorders (e.g., hypophosphatemia, hypocalcemia, thiamin deficiency) may produce revers i bl e cardi omyopat hy.
o
o
f. Hemoglobinopathies (e.g., sickle cell anemia, thalassemia) are as s oci at ed wi t h myocardi al dys funct i on.
o
o
g. Genetic abnormalities. In s ome fami l i es , t he devel opment of di l at ed cardi omyopat hy i s l i nked t o s peci fi c genet i c abnormal i t i es .
o
o
h. Prolonged tachycardia (pers i s t i ng for weeks or mont hs ) may res ul t from uncont rol l ed at ri al arrhyt hmi as , caus i ng a di l at ed cardi omyopat hy t hat may be revers ed wi t hi n weeks , aft er heart rat e i s cont rol l ed.
3. Clinical features, diagnosis, and treatment of di l at ed cardi omyopat hy are s i mi l ar t o l eft - and ri ght -s i ded CHF as des cri bed i n I D–F. o
a.
a. Symptoms of di l at ed cardi omyopat hy are t hos e of bot h l eft -
and ri ght -s i ded CHF as des cri bed i n I D 1.
(1) General l y, t he s ympt oms of l eft -s i ded fai l ure (i .e., orthopnea, paroxysmal nocturnal dyspnea, and dyspnea on exertion ) precede t hos e of ri ght -s i ded fai l ure.
Pa g e 1 5 3
ABC Ambe r CHM Conve rte r Tria l ve rsion, http://w w w .proce sste x t.com/a bcchm.html
(2) Chest pain may occur i n t he abs ence of obs t ruct i ve coronary di s eas e. The caus e of t he ches t pai n may be t he exces s i ve oxygen demands of an enl arged, t hi n-wal l ed vent ri cl e wi t h hi gh wal l s t res s .
o
b. Ph ysi cal sig ns in di l at e d car di o my op at h y are t ho se of CH F. A gal l op rhy th m
Pa g e 1 5 4
ABC Ambe r CHM Conve rte r Tria l ve rsion, http://w w w .proce sste x t.com/a bcchm.html
is us u al l y pre s en t. Th e mu rm ur of mi t ral reg urg ita tio n al s o ma y be pre s en t. Mi t ral reg urg ita tio n occ
Pa g e 1 5 5
ABC Ambe r CHM Conve rte r Tria l ve rsion, http://w w w .proce sste x t.com/a bcchm.html
urs as a res ul t of ven t ri c ul a r di l at i on an d im pro per al i gn me nt of t he pa pi l l ary mu s cl es .
4. Diagnosis o
o
a. Electrocardiography reveal s frequent l eft vent ri cul ar hypert rophy and nons peci fi c ST- and
Pa g e 1 5 6
ABC Ambe r CHM Conve rte r Tria l ve rsion, http://w w w .proce sste x t.com/a bcchm.html
T-wave abnormal i t i es . Left bundl e branch bl ock i s common. o
o
b. Chest radiography s hows an enl arged heart , and t here i s evi dence of pul monary vas cul ar conges t i on.
o
o
c. Echocardiography reveal s di l at ed and poorl y cont ract i ng l eft and ri ght vent ri cl es . In addi t i on, s econdary l eft and ri ght at ri al enl argement us ual l y i s s een.
o
o
d. Gated blood pool scanning i n di l at ed cardi omyopat hy reveal s reduct i on of t he eject i on fract i on of bot h vent ri cl es . There us ual l y i s gl obal dys funct i on, but regi onal cont ract i l e abnormal i t i es al s o may exi s t .
o
o
e. Cardiac catheterization us ual l y i s not neces s ary t o make t he di agnos i s of di l at ed cardi omyopat hy. However, becaus e s urgi cal correct i on of i s chemi c heart di s eas e can occas i onal l y i mprove l eft vent ri cul ar funct i on, i s chemi c heart di s eas e s houl d be excl uded pri or t o maki ng t he di agnos i s of cardi omyopat hy. In s uch cas es , cardi ac cat het eri zat i on may be i ndi cat ed.
5. T herapy o
o
a. Removal of an offending agent. The mos t hopeful s i t uat i on i s one i n whi ch i nces s ant
Pa g e 1 5 7
ABC Ambe r CHM Conve rte r Tria l ve rsion, http://w w w .proce sste x t.com/a bcchm.html
t achycardi a or a known t oxi n has caus ed vent ri cul ar dys funct i on. Heart rat e cont rol or removal of t he t oxi n from t he pat i ent 's envi ronment may l ead t o s i gni fi cant i mprovement i n vent ri cul ar funct i on. o
o
b. Supportive therapy. W hen di l at ed cardi omyopat hy i s i di opat hi c, t he s ympt oms of CHF can be i mproved by s uch meas ures as salt restriction and administration of cardiac glycosides, diuretics, vasodilators, and neurohumoral blockers. Evi dence s hows t hat t he addi t i on of ACE i nhi bi t ors t o a s t andard regi men of di uret i cs i ncreas es l ongevi t y. Gradual i nt roduct i on of β-bl ockers al s o prol ongs l i fe.
o
o
c. Cardiac transplantation. Cardi ac t rans pl ant at i on may offer an i mproved qual i t y of l i fe t o s el ect ed pat i ent s when cont rol of CHF i s not pos s i bl e and prognos i s i s poor.
B. Hypertrophic cardiomyopathy
1. Definition. Hypert rophi c cardi omyopat hy i s a di s order i n whi ch regi onal hypert rophy of t he l eft vent ri cl e occurs i ndependent of l oadi ng condi t i ons . It mos t commonl y i nvol ves t he i nt ervent ri cul ar s ept um, but can al s o i nvol ve t he apex. W hen t he s ept um i s i nvol ved, t he hypert rophi ed s ept um and t he ant eri or l eafl et of t he mi t ral val ve may produce dynami c l eft vent ri cul ar out fl ow obs t ruct i on.
2. Etiology. Mos t cas es are i nheri t ed t hrough an
Pa g e 1 5 8
ABC Ambe r CHM Conve rte r Tria l ve rsion, http://w w w .proce sste x t.com/a bcchm.html
aut os omal domi nant mode of t rans mi s s i on, but s poradi c cas es al s o occur. Speci fi c abnormal i t i es i n t he genes codi ng for cardi ac myos i n and ot her cardi ac prot ei ns have been i dent i fi ed.
3. Pathophysiology o
o
a. Methods of obstruction i ncl ude t he fol l owi ng:
(1) As s hown i n Fi gure 1-10, t he hypert rophi ed s ept um encroaches on t he l eft vent ri cul ar out fl ow t ract and comes i nt o cl os e approxi mat i on wi t h t he ant eri or l eafl et of t he mi t ral val ve.
FIGURE 1-10 Cardi ac cros s -s ect i on cut from t he apex t o t he bas e i n a pat i ent wi t h hypert rophi c obs t ruct i ve cardi omyopat hy. The upper port i on of t he s ept um i s t hi ckened and comes i nt o cl os e proxi mi t y wi t h t he ant eri or l eafl et of t he mi t ral val ve. (Adapt ed from Johns on R, et al . T he Prac t i c e of Cardi ol ogy. Bos t on: Li t t l e, Brown, 1980:648.
Pa g e 1 5 9
ABC Ambe r CHM Conve rte r Tria l ve rsion, http://w w w .proce sste x t.com/a bcchm.html
)
P.27
(2) Duri ng s ys t ol e, a l ow-pres s ure zone may devel op as bl ood fl ow accel erat es t hrough t he narrowed area bet ween t he s ept um and t he ant eri or l eafl et , generat i ng a Bernoulli effect. Thus , t he ant eri or l eafl et of t he mi t ral val ve i s drawn i nt o t he s ept um (s ys t ol i c ant eri or mot i on), l eadi ng t o out fl ow obs t ruct i on.
(3) The s ept um i t s el f s hort ens very l i t t l e duri ng s ys t ol e becaus e of i t s cat enoi d s hape. Becaus e t he s ept um does not s hort en, i t cannot t hi cken. Therefore, i t i s t he ant eri or l eafl et of t he mi t ral val ve t hat pl ays t he act i ve rol e i n creat i ng t he obs t ruct i on.
o
o
b. The degree of outflow obstruction vari es from pat i ent t o pat i ent and from t i me t o t i me i n t he s ame pat i ent .
(1) Phys i ol ogi c condi t i ons t hat enl arge t he l eft vent ri cl e (e.g., i ncreas es i n prel oad and aft erl oad) s eparat e t he s ept um and ant eri or l eafl et of t he mi t ral val ve and reduce t he obs t ruct i on.
(2) Phys i ol ogi c condi t i ons t hat make t he vent ri cl e s mal l er or t hat i ncreas e t he vel oci t y
Pa g e 1 6 0
ABC Ambe r CHM Conve rte r Tria l ve rsion, http://w w w .proce sste x t.com/a bcchm.html
of bl ood fl ow (e.g., dehydrat i on, pos i t i ve i not ropi c drugs ) i ncreas e t he degree of obs t ruct i on. o
o
c. The obs t ruct i on t o out fl ow may caus e s econdary cardi ac hypert rophy of t he nons ept al port i ons of t he vent ri cl e, but s ept al t hi cknes s general l y remai ns great er t han t hat of t he free wal l of t he vent ri cl e.
4. Clinical features o
o
a. Symptoms
(1) Angina. Pat i ent s wi t h obs t ruct i ve cardi omyopat hy frequent l y compl ai n of ches t pai n.
(a) The pai n us ual l y has atypical features; t hat i s , t he pai n may occur at res t and i s not al ways rel at ed t o exerci s e.
The pat hophys i ol ogy of angi na i n hypert rophi c obs t ruct i ve cardi omyopat hy i s uncl ear, but coronary bl ood fl ow i s s ubnormal , pot ent i al l y caus i ng i s chemi a.
(2) Syncope
Pa g e 1 6 1
ABC Ambe r CHM Conve rte r Tria l ve rsion, http://w w w .proce sste x t.com/a bcchm.html
(a) Syncope us ual l y occurs duri ng or aft er exerci s e i n pat i ent s wi t h obs t ruct i ve cardi omyopat hy as a res ul t of reduced l eft vent ri cul ar s i ze and t he cons equent i ncreas ed obs t ruct i on t o out fl ow.
(i) Aft er exerci s e, afterload is reduced becaus e of peri pheral vas odi l at i on.
(ii) Preload is reduced becaus e of t he decreas ed act i vi t y of t he cont ract i ons of t he l eg mus cl es , whi ch hel p t o ret urn bl ood t o t he heart .
(iii) The inotropic state remains elevated becaus e of t he i ncreas ed cat echol ami ne l evel aft er exerci s e.
P.28
(b) Arrhythmias, whi ch are common i n t hi s di s order, al s o may preci pi t at e s yncope or s udden deat h.
(3) CHF. Dys pnea on exert i on, ort hopnea, and paroxys mal noct urnal dys pnea occur i n pat i ent s wi t h obs t ruct i ve cardi omyopat hy.
Pa g e 1 6 2
ABC Ambe r CHM Conve rte r Tria l ve rsion, http://w w w .proce sste x t.com/a bcchm.html
Sys t ol i c funct i on us ual l y i s normal or s upranormal , and t he eject i on fract i on oft en exceeds 80%.
(a) The s ympt oms of heart fai l ure us ual l y are not caus ed by s ys t ol i c mal funct i on, but rat her occur as a res ul t of i ncreas ed di as t ol i c s t i ffnes s .
(b) The t hi ckened myocardi um requi res an i ncreas ed fi l l i ng pres s ure for adequat e di as t ol i c di s t ent i on. The i ncreas ed fi l l i ng pres s ure i s refl ect ed t o t he l ungs and produces pul monary conges t i ve s ympt oms .
(c) In t he l at er s t ages of t he di s eas e, however, s ys t ol i c dys funct i on al s o may occur, cont ri but i ng t o t he s ympt oms of CHF.
b. Physical signs o
(1) Carotid upstroke. In pat i ent s wi t h t he obs t ruct i ve form of t he di s eas e, t he carot i d ups t rokes have a spike and dome character (Onl i ne Fi gure 1-11). Thi s confi gurat i on i ndi cat es earl y s ys t ol i c out fl ow fol l owed by a peri od of obs t ruct i on, duri ng whi ch fl ow fal l s . The dome port i on of t he curve refl ect s t he peri od near t he end of s ys t ol e when obs t ruct i on di mi ni s hes and aort i c out fl ow agai n commences .
Pa g e 1 6 3
ABC Ambe r CHM Conve rte r Tria l ve rsion, http://w w w .proce sste x t.com/a bcchm.html o
ONLINE FIGURE 1-11 Di agram s howi ng s i mul t aneous recordi ng of t he el ect rocardi ogram (ECG), l eft vent ri cul ar pres s ure t raci ng (LV), and aort i c pres s ure t raci ng (Ao) i n a pat i ent wi t h hypert rophi c obs t ruct i ve cardi omyopat hy. A l arge pres s ure gradi ent exi s t s bet ween t he l eft vent ri cl e and aort a. The aort i c pres s ure t raci ng (s i mi l ar t o t he carot i d pul s e) demons t rat es a s pi ke and dome confi gurat i on. (Repri nt ed from Cohn PF, W ynne J. Di agnos t i c Met hods i n Cl i ni c al Cardi ol ogy. Bos t on: Li t t l e, Brown, 1982:147, wi t h permi s s i on. ) o
o
(2) Murmur. The murmur i s a s ys t ol i c eject i on murmur heard al ong t he l eft s t ernal border. Unl i ke t he murmur i n aort i c s t enos i s , i t does not us ual l y radi at e t o t he neck.
(a) Increasing the intensity of the murmur
(i) Maneuvers t hat di mi ni s h l eft vent ri cul ar s i ze caus e an i ncreas e i n bot h t he obs t ruct i on t o out fl ow and t he i nt ens i t y of t he cardi ac murmur. Thus , t he Valsalva maneuver, whi ch
Pa g e 1 6 4
ABC Ambe r CHM Conve rte r Tria l ve rsion, http://w w w .proce sste x t.com/a bcchm.html
di mi ni s hes t he murmur i n val vul ar aort i c s t enos i s by di mi ni s hi ng fl ow, i ncreas es t he murmur i n obs t ruct i ve cardi omyopat hy by i ncreas i ng obs t ruct i on.
(ii) Havi ng t he pat i ent s t and or i nhal e amyl ni t ri t e al s o di mi ni s hes l eft vent ri cul ar s i ze and t herefore i ncreas es t he i nt ens i t y of t he murmur.
(b) Diminishing the intensity of the murmur. Squatting, whi ch i ncreas es myocardi al aft erl oad and venous ret urn t o t he heart , i ncreas es cardi ac s i ze and, t herefore, di mi ni s hes t he murmur.
5. Diagnosis o
o
a. Electrocardiography al mos t al ways i s abnormal . The ECG us ual l y s hows evi dence of l eft vent ri cul ar hypert rophy, nons peci fi c ST- and T-wave abnormal i t i es , and l eft at ri al enl argement .
o
o
b. Echocardiography es t abl i s hes t he di agnos i s i n mos t pat i ent s .
(1) In pat i ent s wi t h asymmetric septal hypertrophy without obstruction, i ncreas ed s ept al t hi cknes s res ul t s i n a septum-to-free wall thickness ratio of 1.3:1 or great er.
Pa g e 1 6 5
ABC Ambe r CHM Conve rte r Tria l ve rsion, http://w w w .proce sste x t.com/a bcchm.html
(2) Findings in the obstructive form of t he di s eas e i ncl ude systolic anterior motion of the mitral valve, systolic fluttering of the aortic valve leaflets, and early closure of the aortic valve, corres pondi ng t o t he s pi ke and dome s een i n t he carot i d pul s e.
6. T herapy. Unl i ke aort i c s t enos i s , i n whi ch rel i ef of val vul ar obs t ruct i on rel i eves s ympt oms and prol ongs l i fe, t here i s no concl us i ve evi dence t hat s urgi cal rel i ef of obs t ruct i on i n obs t ruct i ve cardi omyopat hy prol ongs l i fe. Therefore, medi cal t herapy i s us ed fi rs t i n an at t empt t o i mprove s ympt oms . o
o
a. Medical therapy
(1) β-blockers are effect i ve i n rel i evi ng s ympt oms i n t hi s di s eas e.
(a) β-bl ockade slows the heart rate, whi ch i ncreas es l eft vent ri cul ar fi l l i ng and s i ze, di mi ni s hi ng obs t ruct i on.
(b) β-bl ockade al s o reduces t he vi gor of l eft vent ri cul ar cont ract i on and, t hus , decreases the velocity of blood flow, whi ch al s o reduces t he degree of obs t ruct i on.
Pa g e 1 6 6
ABC Ambe r CHM Conve rte r Tria l ve rsion, http://w w w .proce sste x t.com/a bcchm.html
(2) Calcium channel blocking agents. Thes e agent s are an al t ernat i ve t o β-bl ockers and have been s hown t o diminish the left ventricular outflow gradient. Verapamil i s t he cal ci um channel bl ocker mos t wi del y us ed i n t he t reat ment of t hi s di s eas e. Caut i on mus t be exerci s ed i n pat i ent s wi t h CHF becaus e verapami l may wors en fai l ure and preci pi t at e acut e pul monary edema. P.29
(3) Digitalis i s contraindicated in the hyperdynamic phase of t he di s eas e when obs t ruct i on i s pres ent and t he l eft vent ri cul ar cavi t y i s s mal l , becaus e di gi t al i s i ncreas es t he vi gor of l eft vent ri cul ar cont ract i on and t hus i ncreas es t he out fl ow obs t ruct i on.
o
o
b. Surgical therapy
(1) Myomectomy. Surgi cal reduct i on of t he t hi cknes s of t he l eft vent ri cul ar s ept um rel i eves t he out fl ow gradi ent and s ympt oms i n t hos e pat i ent s who have not res ponded t o medi cal t herapy.
(2) Mitral valve replacement. Becaus e i t i s t he ant eri or l eafl et of t he mi t ral val ve t hat produces t he obs t ruct i on, mi t ral val ve repl acement i s al s o effect i ve i n rel i evi ng obs t ruct i on.
Pa g e 1 6 7
ABC Ambe r CHM Conve rte r Tria l ve rsion, http://w w w .proce sste x t.com/a bcchm.html
(3) Intentional septal infarction. Trans cat het er i ns t i l l at i on of et hanol i nt o t he s ept al art ery i s performed t o i nfarct t he s ept um and reduce obs t ruct i on.
o
o
c. Antiarrhythmic therapy. Mos t pat i ent s wi t h hypert rophi c myopat hy di e s uddenl y. Pat i ent s wi t h a fami l y hi s t ory of s udden deat h or a pers onal hi s t ory of s yncope or vent ri cul ar t achycardi a are at hi gh ri s k and s houl d undergo el ect rophys i ol ogi c t es t i ng. Many pat i ent s recei ve i mpl ant abl e defi bri l l at ors .
C. Restrictive cardiomyopathy
1. Definition. The res t ri ct i ve cardi omyopat hi es are a group of di s eas es i n whi ch t he compos i t i on of t he myocardi um has changed s o t hat i t becomes s t i ffer. The increased stiffness of the myocardium res t ri ct s l eft vent ri cul ar fi l l i ng, t hereby reduci ng s t roke out put and i ncreas i ng l eft vent ri cul ar fi l l i ng pres s ure.
2. Etiology. Infiltrative diseases of t he myocardi um, whi ch produce res t ri ct i ve cardi omyopat hy, i ncl ude amyloidosis, hemochromatosis, idiopathic eosinophilia, carcinoid syndrome, sarcoidosis, and endomyocardial fibroelastosis.
3. Pathophysiology. Sys t ol i c funct i on us ual l y i s normal i n t he earl y s t ages of t he di s eas e, but t he al t ered propert i es of t he myocardi um i ncreas e di as t ol i c
Pa g e 1 6 8
ABC Ambe r CHM Conve rte r Tria l ve rsion, http://w w w .proce sste x t.com/a bcchm.html
s t i ffnes s . Thus , t he l eft vent ri cul ar pres s ure i s above normal at any di as t ol i c l eft vent ri cul ar vol ume. Increas ed fi l l i ng pres s ure produces pul monary conges t i on. As t he i nfi l t rat i ve proces s progres s es , s ys t ol i c funct i on al s o i s compromi s ed.
4. Clinical features o
o
a. Symptoms of both left-sided and right-sided CHF us ual l y are pres ent ; t he s ympt oms of ri ght -s i ded fai l ure are us ual l y more promi nent .
o
o
b. Physical signs i ncl ude t hos e pres ent i n l eft -s i ded and ri ght -s i ded CHF.
5. Diagnosis o
o
a. Electrocardiography. The ECG frequent l y s hows l ow QRS vol t ages and nons peci fi c ST- and T-wave abnormal i t i es . Conduct i on abnormal i t i es are common.
o
o
b. Radiographs. Si gns of pul monary vas cul ar conges t i on may coexi s t wi t h normal heart s i ze, becaus e even when l eft vent ri cul ar s ys t ol i c funct i on fai l s i n t he l at er s t ages of t he di s eas e, t he res t ri ct i on t o cardi ac fi l l i ng prevent s cardi ac di l at i on.
o
o
c. Echocardiography
Pa g e 1 6 9
ABC Ambe r CHM Conve rte r Tria l ve rsion, http://w w w .proce sste x t.com/a bcchm.html
(1) The echocardi ogram may demons t rat e t hi ckeni ng of t he l eft and ri ght vent ri cl es . The combi nat i on of i ncreas ed l eft vent ri cul ar t hi cknes s on t he echocardi ogram and decreas ed l eft vent ri cul ar vol t age on t he ECG i s hi ghl y s ugges t i ve of res t ri ct i ve cardi omyopat hy.
(2) Doppl er exami nat i on may reveal evi dence of abnormal vent ri cul ar di as t ol i c fi l l i ng or al t ered compl i ance.
(3) Left and ri ght vent ri cul ar chamber s i zes us ual l y are normal , whereas t he l eft and ri ght at ri a are i ncreas ed i n s i ze.
(4) In amyl oi dos i s , t he myocardi um may appear bri ght er t han normal .
o
o
d. Cardiac catheterization. Oft en i t i s di ffi cul t t o di s t i ngui s h res t ri ct i ve cardi omyopat hy from cons t ri ct i ve peri cardi t i s .
(1) A dip and plateau i n t he l eft and ri ght vent ri cul ar fi l l i ng pres s ures may be s een i n bot h di s eas es .
(2) In res t ri ct i ve cardi omyopat hy, left and right atrial pressures and left and right
Pa g e 1 7 0
ABC Ambe r CHM Conve rte r Tria l ve rsion, http://w w w .proce sste x t.com/a bcchm.html
ventricular filling pressures us ual l y are not identical, as t hey are i n cons t ri ct i ve peri cardi t i s .
(3) Endomyocardial biopsy duri ng cardi ac cat het eri zat i on may hel p es t abl i s h t he di agnos i s .
P.30
6. T herapy. Treat ment for t hi s group of di s eas es i s l i mi t ed. o
o
a. In cas es wi t h a reversible etiology (e.g., hemochromat os i s ), direct therapy s uch as i ron chel at i on may res ul t i n i mprovement .
o
o
b. W hen t he caus e of t he di s eas e cannot be t reat ed, symptomatic therapy with diuretics t o reduce t he s ympt oms of conges t i on i s i ndi cat ed.
VI. Pericardial Disease A. Acute pericarditis
1. Etiology o
o
a. Myocardial infarction (MI). Peri cardi t i s may occur i n t he fi rs t 24 hours fol l owi ng t rans mural MI becaus e t he i nfl amed s urface of t he i nfarct ed area
Pa g e 1 7 1
ABC Ambe r CHM Conve rte r Tria l ve rsion, http://w w w .proce sste x t.com/a bcchm.html
of myocardi um produces pericardial irritation. A s econd t ype of peri cardi t i s , cal l ed Dressler' s syndrome, al s o may be s een from 1 week t o s everal mont hs aft er MI and may occur as t he res ul t of an aut oi mmune react i on t o t he damaged heart mus cl e. o
o
b. Infection. Pericarditis frequently follows upper respiratory tract viral infections and i s s een i n many vi ral i nfect i ons , i ncl udi ng HIV, hepat i t i s , and many more. Tubercul os i s , s t rept ococcal i nfect i on, s t aphyl ococcal i nfect i on, and t he s equel ae of i nfect i ve endocardi t i s al s o may produce peri cardi t i s .
o
o
c. Collagen vascular disease. Acut e peri cardi t i s may be a cl i ni cal mani fes t at i on of SLE, rheumat oi d art hri t i s , or, l es s commonl y, s cl eroderma.
o
o
d. Drugs. Commonl y us ed drugs t hat may caus e acut e peri cardi t i s i ncl ude procainamide, hydralazine, and isoniazid.
o
o
e. Malignancy. Peri cardi t i s may occur s econdary t o met as t at i c i nvol vement of t he peri cardi um. Pulmonary and breast carcinomas are t he mos t common primary sites.
o
o
f. Uremia. Peri cardi t i s i s common i n unt reat ed or undert reat ed s evere chroni c renal fai l ure.
o
Pa g e 1 7 2
ABC Ambe r CHM Conve rte r Tria l ve rsion, http://w w w .proce sste x t.com/a bcchm.html
o
g. Postpericardiotomy syndrome. Duri ng open heart s urgery, t he peri cardi um i s i nci s ed. Us ual l y, t he peri cardi t i s t hat ari s es from t hi s i njury i s s hort -l i ved; however, i t may be prot ract ed and s evere i n s ome pat i ent s .
o
o
h. Radiation. Radi at i on t herapy del i vered t o t he ches t for t horaci c mal i gnanci es may caus e peri cardi t i s .
2. Clinical features o
o
a. Symptoms. The mos t common s ympt om i n peri cardi t i s i s inspiratory chest pain.
(1) The pai n i s l ocat ed i n t he l eft s i de and oft en i s l es s ened when t he pat i ent s i t s up and l eans forward.
(2) Occas i onal l y t he pai n may be s i mi l ar t o t hat of myocardi al i s chemi a and may radi at e t o t he neck and arm.
o
o
b. Physical signs. The classic sign of acut e peri cardi t i s i s t he pericardial friction rub, whi ch i s a s crat chy, l eat hery s ound wi t h t hree component s corres pondi ng t o vent ri cul ar s ys t ol e, earl y di as t ol i c fi l l i ng, and at ri al cont ract i on.
Pa g e 1 7 3
ABC Ambe r CHM Conve rte r Tria l ve rsion, http://w w w .proce sste x t.com/a bcchm.html
3. Diagnosis o
o
a. Physical examination. The pres ence of a pericardial friction rub confi rms t he di agnos i s of peri cardi t i s .
o
o
b. Electrocardiography. Epi cardi al i nfl ammat i on produces a di ffus e current of i njury wi t h ST-s egment el evat i on t hroughout t he ECG. There i s no reci procal ST-s egment depres s i on, as i s s een i n acut e MI. Depression of the PR segment is unique to pericarditis.
o
o
c. Echocardiography. The echocardi ogram frequent l y demons t rat es a peri cardi al effus i on, whi ch hel ps confi rm t he di agnos i s .
4. T herapy o
o
a. Specific therapy s houl d be di rect ed t oward t he caus e of t he peri cardi t i s , i f t he caus e i s known.
o
o
b. Nonsteroidal anti-inflammatory drugs (NSAIDs) s uch as aspirin, indomethacin, and ibuprofen us ual l y are effect i ve i n reduci ng t he i nfl ammat i on and rel i evi ng t he ches t pai n.
o
o
c. Colchicine. Int ract abl e cas es of peri cardi t i s , as may occur wi t h Dres s l er's s yndrome and pos t peri cardi ot omy s yndrome, may requi re
Pa g e 1 7 4
ABC Ambe r CHM Conve rte r Tria l ve rsion, http://w w w .proce sste x t.com/a bcchm.html
gl ucocort i coi d t herapy for rel i ef of s ympt oms . Recent l y col chi ci ne has repl aced s t eroi ds at many cent ers . P.31
B. Pericardial effusion
1. Pathophysiology. The i nfl ammat i on caus ed by acut e peri cardi t i s oft en produces exudat i on of fl ui d i nt o t he peri cardi al s pace. W hen fl ui d accumul at es s l owl y, t he peri cardi um expands t o accommodat e i t , but when fl ui d accumul at es rapi dl y, i t compres s es t he heart , t hus i nhi bi t i ng cardi ac fi l l i ng. Thi s l at t er condi t i on i s known as cardiac tamponade (s ee VI C).
2. Clinical features o
o
a. Symptoms. The mere pres ence of a peri cardi al effus i on does not caus e s ympt oms . However, s ympt oms of acut e peri cardi t i s may coexi s t wi t h a peri cardi al effus i on.
o
o
b. Physical signs. As t he effus i on accumul at es , i t act s as a cus hi on around t he heart .
(1) The precordi um becomes qui et , pal pat i on of t he PMI becomes di ffi cul t , and t he heart t ones become di s t ant and s oft .
Pa g e 1 7 5
ABC Ambe r CHM Conve rte r Tria l ve rsion, http://w w w .proce sste x t.com/a bcchm.html
(2) Al t hough t he accumul at i on of fl ui d bet ween t he l ayers of peri cardi um may di mi ni s h a peri cardi al fri ct i on rub, a fri ct i on rub s t i l l may exi s t i n t he pres ence of a l arge effus i on.
o
o
3. Diagnosis
a. Electrocardiography. The ECG demons t rat es l ow vol t age; el ect ri cal al t ernans may be pres ent n l arge effus i on.
b. Chest radiography. Cardi ac enl argement occurs as t he effus i on devel ops . Typi cal l y, t he cardi ac s i l houet t e has a “ water bottle†• appearance. The pres ence of an ext remel y enl arged heart wi t hout s i gns of vas cul ar conges t i on s ugges t s t he di agnos i s of peri cardi al effus i on.
c. Echocardiography. An echocardi ogram demons t rat i ng an echo-free space between the two layers of the pericardium i s di agnos t i c of a peri cardi al effus i on.
d. Pericardiocentesis. The pres ence of a peri cardi al effus i on may be confi rmed by t he as pi rat i on of fl ui d from t he peri cardi al s ac. Exami nat i on of t he fl ui d hel ps es t abl i s h t he caus e of t he effus i on.
Pa g e 1 7 6
ABC Ambe r CHM Conve rte r Tria l ve rsion, http://w w w .proce sste x t.com/a bcchm.html
(1) The fl ui d s houl d be s ent for a cel l count and di fferent i al , bact eri al and fungal cul t ures , s t ai ns and cul t ures for Myc obac t eri um t uberc ul os i s , prot ei n cont ent , and l act at e dehydrogenas e (LDH) cont ent .
(2) An addi t i onal al i quot of fl ui d s houl d be cent ri fuged and exami ned for t umor cel l s .
(3) Bl oody effus i ons are charact eri s t i c of cert ai n et i ol ogi es (e.g., neopl as i a, t ubercul os i s ). However, bl oody effus i ons can al s o occur i f t he needl e i s pas s ed t oo far and vent ri cul ar bl ood i s as pi rat ed by mi s t ake. It i s pos s i bl e t o di s t i ngui s h t he t wo becaus e vent ri cul ar bl ood cl ot s , whereas a bl oody effus i on does not .
e. T herapy. Treat ment for a peri cardi al effus i on i s t he s ame as t hat for acut e peri cardi t i s but may al s o i nvol ve as pi rat i on.
C. Cardiac tamponade
1. Definition and pathophysiology. Cardi ac t amponade i s a life-threatening condition i n whi ch a peri cardi al effus i on has devel oped s o rapi dl y or has become s o l arge t hat i t compres s es t he heart . o
Pa g e 1 7 7
ABC Ambe r CHM Conve rte r Tria l ve rsion, http://w w w .proce sste x t.com/a bcchm.html o
a. The heart cannot fi l l adequat el y, and becaus e t he heart can pump out onl y what i t t akes i n, i mpai red fi l l i ng caus es a profound reduct i on i n cardi ac out put .
o
o
b. The ext ernal pres s ure produced by t he fl ui d on t he four chambers of t he heart i s di s pers ed equal l y. Becaus e ext ernal pres s ure us ual l y ri s es t o a great er l evel t han t he normal cardi ac fi l l i ng pres s ures , i nt raperi cardi al pres s ure, l eft and ri ght at ri al pres s ures , and l eft and ri ght vent ri cul ar pres s ures al l become equal i n di as t ol e.
2. Clinical features o
o
a. Symptoms. Mos t pat i ent s wi t h cardi ac t amponade compl ai n of dyspnea, fatigue, and orthopnea.
o
o
b. Physical signs
(1) Pulsus paradoxus. The normal fal l i n s ys t ol i c bl ood pres s ure t hat occurs duri ng i ns pi rat i on i s exaggerat ed i n t amponade. A decreas e of more t han 10 mm Hg occurs i n 95% of pat i ent s wi t h cardi ac t amponade. The pres ence of pul s us paradoxus i mpl i es t hat s t roke vol ume i s fal l i ng duri ng i ns pi rat i on, probabl y as a res ul t of t he fol l owi ng mechani s ms :
Pa g e 1 7 8
ABC Ambe r CHM Conve rte r Tria l ve rsion, http://w w w .proce sste x t.com/a bcchm.html
(a) Septal shift. Duri ng i ns pi rat i on, ri ght vent ri cul ar fi l l i ng i s augment ed by negat i ve i nt rat horaci c pres s ure, whi ch i ncreas es venous ret urn. Thi s caus es t rans i ent enl argement of t he ri ght vent ri cl e and pus hes t he vent ri cul ar s ept um i nt o t he l eft vent ri cl e, t hus reduci ng t he s i ze and out put of t he l eft vent ri cl e. P.32
(b) T ensing of the pericardium. Ins pi rat i on produces downward t ract i on on t he peri cardi um, furt her compres s i ng t he cardi ac s t ruct ures and reduci ng l eft vent ri cul ar out put .
(c) Right ventricular enlargement. The enhanced ri ght vent ri cul ar fi l l i ng duri ng i ns pi rat i on al s o di s t ends t he ri ght vent ri cl e, caus i ng i t t o t ake up more room i n t he peri cardi al s pace. Thi s furt her l i mi t s l eft vent ri cul ar fi l l i ng.
(d) Negative intrathoracic pressure. Duri ng i ns pi rat i on, t he negat i ve pres s ure i ns i de t he ches t s ubt ract s pres s ure from t he ext rat horaci c vas cul at ure, furt her reduci ng bl ood pres s ure.
(e) Expansion of the pulmonary
Pa g e 1 7 9
ABC Ambe r CHM Conve rte r Tria l ve rsion, http://w w w .proce sste x t.com/a bcchm.html
vascular bed. The pul monary vas cul ar bed expands duri ng i ns pi rat i on, i ncreas i ng i t s capaci t y and, t hus , reduces l eft at ri al fi l l i ng.
(2) Neck vein distention. The i nt raperi cardi al pres s ure and ri ght at ri al pres s ure i s refl ect ed by ext reme el evat i on of t he jugul ar venous pres s ure. However, Kus s maul 's s i gn (i .e., i ncreas ed neck vei n di s t ent i on wi t h i ns pi rat i on) us ual l y i s abs ent i n t hi s condi t i on.
(3) Narrowed pulse pressure. Reduct i on i n l eft vent ri cul ar s t roke vol ume l eads t o a reduct i on i n s ys t ol i c pres s ure; t he t achycardi a t hat us ual l y occurs as a compens at ory mechani s m di mi ni s hes di as t ol i c runoff and mai nt ai ns di as t ol i c pres s ure. Thus , pul s e pres s ure i s narrowed; however, l es s s evere cas es of cardi ac t amponade may coexi s t wi t h a normal pul s e pres s ure.
(4) Shock. The carot i d ups t roke i s di mi ni s hed i n vol ume, t he s ys t ol i c bl ood pres s ure i s reduced, and t he peri phery i s col d and cl ammy becaus e of t he vas ocons t ri ct i on pres ent i n reduced cardi ac out put s t at es .
3. Diagnosis. El evat ed neck vei ns , pul s us paradoxus , and an enl arged cardi ac s i l houet t e on ches t x-ray i n a pat i ent exhi bi t i ng s ympt oms of compromi s ed cardi ac out put s t rongl y s ugges t t he di agnos i s .
Pa g e 1 8 0
ABC Ambe r CHM Conve rte r Tria l ve rsion, http://w w w .proce sste x t.com/a bcchm.html o
o
a. Echocardiography i s an i ndi s pens abl e t ool i n t he eval uat i on of t amponade. Feat ures cons i s t ent wi t h t amponade i ncl ude:
(1) Pres ence of a peri cardi al effus i on,
(2) Col l aps e of t he ri ght at ri um and/or ri ght vent ri cl e, whi ch occurs i n di as t ol e as t he peri cardi al pres s ure exceeds t he i nt racavi t ary pres s ure,
(3) Enhanced i ns pi rat ory t rans -t ri cus pi d fl ow and s i mul t aneous l y reduced t rans -mi t ral fl ow. Thi s i s t he echo equi val ent of a pul s us paradoxus .
o
o
b. Cardiac catheterization, whi ch coul d confi rm t he di as t ol i c equal i zat i on of pres s ures , i s l es s commonl y us ed for di agnos i s of t amponade.
4. T herapy. The onl y effect i ve t herapy for cardi ac t amponade i s removal of fl ui d from t he peri cardi al s ac. Thus , emergency pericardiocentesis i s i ndi cat ed. The us e of pressor agents and volume expansion i s of l i mi t ed benefi t unt i l peri cardi ocent es i s can be performed.
D. Constrictive pericarditis
Pa g e 1 8 1
ABC Ambe r CHM Conve rte r Tria l ve rsion, http://w w w .proce sste x t.com/a bcchm.html
1. Definition. Cons t ri ct i ve peri cardi t i s i s t he di ffus e t hi ckeni ng of t he peri cardi um i n react i on t o pri or i nfl ammat i on, whi ch res ul t s i n reduced di s t ens i bi l i t y of t he cardi ac chambers . Cardi ac out put i s l i mi t ed, and fi l l i ng pres s ures are i ncreas ed t o mat ch t he ext ernal cons t ri ct i ve force pl aced on t he heart by t he peri cardi um.
2. Etiology. Mos t condi t i ons t hat caus e acut e peri cardi t i s may l ead t o chroni c cons t ri ct i ve peri cardi t i s .
3. Clinical features o
o
a. Symptoms. The cl i ni cal pi ct ure t ypi cal l y i s domi nat ed by symptoms of right-sided failure rat her t han l eft -s i ded fai l ure.
(1) Mos t pat i ent s wi t h cons t ri ct i ve peri cardi t i s compl ai n of dyspnea on exertion as a res ul t of l i mi t ed cardi ac out put . Al t hough approxi mat el y 50% of pat i ent s compl ai n of orthopnea, paroxys mal noct urnal dys pnea i s rare.
(2) Sympt oms rel at ed t o s ys t emi c venous hypert ens i on frequent l y are report ed and i ncl ude ascites, edema, and jaundice.
o
o
b. Physical signs
Pa g e 1 8 2
ABC Ambe r CHM Conve rte r Tria l ve rsion, http://w w w .proce sste x t.com/a bcchm.html
(1) Jugular venous distention. The jugul ar vei ns are di s t ended, i ndi cat i ng s ys t emi c venous hypert ens i on. Neck vei n di s t ent i on i ncreas es wi t h i ns pi rat i on (Kussmaul' s sign). P.33
(2) Heart sounds. The heart s ounds are di s t ant . Earl y i n di as t ol e, a peri cardi al knock may be heard, whi ch fal l s i n t he s ame cadence as an S 3 but i s hi gher pi t ched and corres ponds t o earl y, abrupt ces s at i on of vent ri cul ar fi l l i ng.
(3) Other signs of systemic venous hypertension. Ascites, edema, hepatic tenderness, and hepatomegaly are frequent l y pres ent . It i s not uncommon for cons t ri ct i on t o mas querade as end-s t age l i ver di s eas e.
4. Diagnosis o
o
a. Electrocardiography. The ECG s hows l ow vol t age i n t he l i mb l eads . Atrial arrhythmias are common.
o
o
b. Chest radiography reveal s pericardial calcification i n 50% of pat i ent s . Thi s fi ndi ng i s s een as a radiopaque ring around the heart i n t he l at eral vi ew. The heart us ual l y i s normal i n
Pa g e 1 8 3
ABC Ambe r CHM Conve rte r Tria l ve rsion, http://w w w .proce sste x t.com/a bcchm.html
s i ze, al t hough cardi omegal y occas i onal l y i s not ed. o
o
c. Echocardiography. Al t hough peri cardi al t hi ckeni ng oft en can be det ect ed, rel i abl e di agnos i s of cons t ri ct i ve peri cardi t i s by echocardi ography i s di ffi cul t . However, Doppl er i nt errogat i on of t he mi t ral val ve us ual l y demons t rat es an abnormal decreas e i n fl ow duri ng i ns pi rat i on.
o
o
d. Magnetic resonance imaging (MRI) gat ed t o t he cardi ac cycl e i s an i magi ng t echni que capabl e of meas uri ng peri cardi al t hi cknes s .
o
o
e. Cardiac catheterization reveal s equal pres s ures i n t he four cardi ac chambers duri ng di as t ol e; i n addi t i on, al l pres s ures us ual l y are el evat ed.
5. T herapy. Surgi cal removal of t he peri cardi um i s curat i ve. However, i mmedi at e rel i ef of cons t ri ct i ve s ympt oms may not occur for up t o 6 weeks aft er pericardiectomy.
VII. Congenital Heart Disease in the Adult A. Atrial septal defect (Figure 1-12)
1. Classification o
o
a. An ostium secundum atrial septal defect occurs i n t he midportion of t he i nt ra-at ri al s ept um and i s caus ed by fai l ure of t he s ept um s ecundum t o form
Pa g e 1 8 4
ABC Ambe r CHM Conve rte r Tria l ve rsion, http://w w w .proce sste x t.com/a bcchm.html
properl y. o
o
b. An ostium primum atrial septal defect res ul t s from i mproper s ept at i on of t he endocardi al cus hi on port i on of t he s ept um. It i nvari abl y i nvol ves t he mitral valve, whi ch i s cl eft and oft en regurgi t ant .
o
o
c. A sinus venosus–type atrial septal defect occurs high in the atrial septum and frequent l y i s as s oci at ed wi t h anomal ous drai nage of one or more of t he pul monary vei ns i nt o t he ri ght at ri um.
FIGURE 1-12 At ri al s ept al defect s . Shown here are s everal t ypes of at ri al s ept al defect s : s i nus venos us defect s of t he s uperi or vena caval (SVC) and i nferi or vena caval (IVC) t ypes , os t i um s ecundum and os t i um pri mum defect s , and a coronary s i nus defect . (Adapt ed from Perl off JK. T he Cl i ni c al Rec ogni t i on of Congeni t al Heart Di s eas e. 4t h ed. Phi l adel phi a: W B Saunders , 1994:295. )
P.34
o
Pa g e 1 8 5
ABC Ambe r CHM Conve rte r Tria l ve rsion, http://w w w .proce sste x t.com/a bcchm.html
o
d. Holt-Oram syndrome i s charact eri zed by t he pres ence of a secundum defect t oget her wi t h bony abnormalities of the forearms and hands. Thi s s yndrome i s a hereditary di s eas e t hat i s t rans mi t t ed i n an aut os omal domi nant fas hi on.
2. Pathophysiology o
o
a. Left and ri ght at ri al pres s ures us ual l y are equal i n at ri al s ept al defect ; t hus , no pres s ure gradi ent exi s t s bet ween t he at ri a. However, t he i ncreas ed t hi cknes s of t he l eft vent ri cl e as compared wi t h t he ri ght vent ri cl e makes t he l eft vent ri cl e l es s compl i ant and, t herefore, harder t o fi l l . Bl ood fl ow t akes t he pat h of l eas t res i s t ance and t hus i s s hunt ed from t he l eft at ri um t o t he ri ght at ri um. The net effect i s t o i ncreas e t he vol ume work of t he ri ght vent ri cl e.
o
o
b. The i ncreas ed vol ume pumped t hrough t he pul monary vas cul at ure may l ead t o archi t ect ural changes i n t he pul monary vas cul at ure and t o t he devel opment of i rrevers i bl e pul monary hypert ens i on—a s eri ous but rare l at e compl i cat i on.
3. Clinical features o
o
a. Symptoms. Pat i ent s wi t h at ri al s ept al defect may have a prolonged symptom-free period. Event ual l y, s ympt oms devel op and may i ncl ude
Pa g e 1 8 6
ABC Ambe r CHM Conve rte r Tria l ve rsion, http://w w w .proce sste x t.com/a bcchm.html
palpitations as a res ul t of at ri al arrhyt hmi as , fatigue, dyspnea on exertion, orthopnea, frequent respiratory tract infections, and symptoms of right ventricular failure.
b. Physical signs o
o
(1) Wide and fixed splitting of the S 2 is the classic finding i n at ri al s ept al defect . The i ncreas ed cardi ac fl ow t hrough t he ri ght vent ri cl e del ays pul moni c val ve cl os ure, wi deni ng t he normal s pl i t t i ng of t he S 2 . Ins pi rat i on produces rel at i vel y l i t t l e change i n ri ght -s i ded fl ow, s o t here i s l i t t l e res pi rat ory vari at i on i n t he s pl i t t i ng of t he S 2 .
o
o
(2) Murmur. Under l ow pres s ure, bl ood fl ow from t he l eft t o t he ri ght at ri um occurs t hrough a wi de apert ure and produces no t urbul ence or murmur. However, t he i ncreas ed pul monary bl ood fl ow i n at ri al s ept al defect produces a systolic ejection murmur, whi ch i s heard in the pulmonic area. The i ncreas ed fl ow al s o may produce a diastolic rumble across the tricuspid valve i f t he l eft -t o-ri ght s hunt rat i o i s great er t han 3:1.
o
o
(3) Neck vein distention, ascites, and edema are i ndi cat i ve of ri ght vent ri cul ar fai l ure.
4. Diagnosis o
o
a. Electrocardiography. In os t i um s ecundum
Pa g e 1 8 7
ABC Ambe r CHM Conve rte r Tria l ve rsion, http://w w w .proce sste x t.com/a bcchm.html
defect s , i ncompl et e ri ght bundl e bl ock and ri ght axi s devi at i on are common fi ndi ngs . Os t i um pri mum defect s us ual l y i nvol ve t he ant eri or fas ci cl e of t he l eft bundl e, produci ng l eft ant eri or hemi bl ock and l eft axi s devi at i on. o
o
b. Chest radiography
(1) Increas ed pul monary bl ood fl ow produces i ncreas ed pul monary vas cul ar marki ngs i n t he l ungs , whi ch i s cal l ed shunt vascularity.
(2) Ri ght vent ri cul ar enl argement may encroach on t he ret ros t ernal ai rs pace, reduci ng i t i n t he l at eral vi ew.
(3) Enl argement of t he pul monary art ery s egment i n t he pos t eroant eri or vi ew al s o may be s een.
o
o
c. Echocardiography
(1) The echocardi ogram s hows enl argement of t he ri ght vent ri cl e, and t he at ri al s ept al defect i t s el f may be s een i n many cas es .
(2) A saline injection, whi ch carri es wi t h i t mi cro bubbl es of ai r, s hows a negative-contrast image at the site of t he defect .
Pa g e 1 8 8
ABC Ambe r CHM Conve rte r Tria l ve rsion, http://w w w .proce sste x t.com/a bcchm.html
(3) Doppler examination of t he i nt erat ri al s ept um demons t rat es t he abnormal pres ence of l eft -t o-ri ght bl ood fl ow acros s t he s ept um.
o
o
d. Cardiac catheterization
(1) Duri ng cardi ac cat het eri zat i on, t he di agnos i s can be confi rmed by pas s age of t he cat het er acros s t he at ri al s ept al defect .
(2) Left and ri ght at ri al pres s ures us ual l y are equal .
(3) Oxygen s ampl es drawn from t he s uperi or vena cava and ri ght at ri um demons t rat e a step-up in oxygen concentration i n t he ri ght at ri um, as hi ghl y oxygenat ed l eft at ri al bl ood i s s hunt ed i nt o t he ri ght at ri um. Oxygen s at urat i ons can be us ed t o quant i t at e t he magni t ude of t he l eft -t o-ri ght s hunt .
P.35
o
o
5. T herapy
a. Surgical correction, whi ch has a l ow operat i ve mort al i t y rat e, i s i ndi cat ed for s hunt s wi t h a pul monary-t o-s ys t emi c fl ow
Pa g e 1 8 9
ABC Ambe r CHM Conve rte r Tria l ve rsion, http://w w w .proce sste x t.com/a bcchm.html
rat i o of great er t han 2:1, even i n as ympt omat i c pat i ent s . Shunt s of t hi s magni t ude may l ead t o t he devel opment of pul monary hypert ens i on, us ual l y become s ympt omat i c, and wors en wi t h age.
b. Al t ernat i vel y, s everal cat het er-bas ed devi ces for defect cl os ure are now approved for us e.
B. Ventricular septal defect
1. Pathophysiology. In vent ri cul ar s ept al defect , t he l eft vent ri cl e act i vel y propel s t he bl ood i nt o t he ri ght vent ri cl e, res ul t i ng i n t he t axat i on of bot h vent ri cl es and i n i ncreas ed pul monary bl ood fl ow. Pul monary hypert ens i on i s more s evere and more frequent i n vent ri cul ar s ept al defect t han i n at ri al s ept al defect .
2. Clinical features. Becaus e mos t vent ri cul ar s ept al defect s l ead t o s ympt oms and are correct ed i n chi l dhood, s i gni fi cant congeni t al vent ri cul ar s ept al defect rarel y i s di agnos ed for t he fi rs t t i me i n adul t hood. o
o
a. Symptoms of vent ri cul ar s ept al defect are t hos e of bot h left- and right-sided CHF.
o
o
b. Physical signs
(1) Displacement of the PMI t o t he l eft i s
Pa g e 1 9 0
ABC Ambe r CHM Conve rte r Tria l ve rsion, http://w w w .proce sste x t.com/a bcchm.html
i ndi cat i ve of l eft vent ri cul ar enl argement .
(2) Sternal lift i s i ndi cat i ve of ri ght vent ri cul ar enl argement .
(3) Murmur. A harsh, holosystolic murmur i s heard al ong t he l eft s t ernal border. The murmur oft en i s accompani ed by a thrill and radi at es t o t he ri ght of t he s t ernum.
(4) Aortic regurgitation. Vent ri cul ar s ept al defect s may i nvol ve t he ri ght coronary cus p of t he aort i c val ve, produci ng i ns uffi ci ent s upport for t hi s val ve l eafl et and, hence, aort i c regurgi t at i on. Approxi mat el y 6% of pat i ent s wi t h vent ri cul ar s ept al defect have s i gns of aort i c i ns uffi ci ency.
3. Diagnosis o
o
a. Electrocardiography. The ECG t ypi cal l y s hows biventricular hypertrophy.
o
o
b. Chest radiography. Cardi ac enl argement i s t he rul e. If t he s hunt i s great er t han 2:1 i n magni t ude, shunt vascularity us ual l y i s pres ent .
o
o
c. Echocardiography. The s ept al defect frequent l y can be demons t rat ed duri ng t wo-di mens i onal echocardi ography. Left and ri ght vent ri cul ar
Pa g e 1 9 1
ABC Ambe r CHM Conve rte r Tria l ve rsion, http://w w w .proce sste x t.com/a bcchm.html
enl argement i s s een as wel l . Doppl er exami nat i on reveal s abnormal bl ood fl ow from t he l eft vent ri cl e t o t he ri ght vent ri cl e. o
o
d. Cardiac catheterization
(1) A l eft vent ri cul ogram obt ai ned i n t he l eft ant eri or obl i que pos i t i on demons t rat es fl ow of cont ras t from t he l eft vent ri cl e acros s t he s ept um i nt o t he ri ght vent ri cl e.
(2) Duri ng cardi ac cat het eri zat i on, an oxygen step-up occurs at t he l evel of t he ri ght vent ri cl e. Pul monary hypert ens i on, i f pres ent , can be quant i fi ed.
4. T herapy. Becaus e pat i ent s wi t h vent ri cul ar s ept al defect s are prone t o pul monary vas cul ar compl i cat i ons and bact eri al endocardi t i s , vent ri cul ar s ept al defect s wi t h a magni t ude of 2:1 or great er s houl d be correct ed s urgi cal l y.
C. Patent ductus arteriosus
1. Pathophysiology. In pat ent duct us art eri os us , bl ood fl ows from t he aort a i nt o t he pul monary art ery aft er t he t akeoff of t he l eft s ubcl avi an art ery. Vol ume overl oad i s i mpos ed on t he l eft vent ri cl e, whi ch mus t pump bl ood i nt o bot h t he s ys t emi c and pul monary ci rcul at i ons , and, i n t i me, may l ead t o l eft vent ri cul ar fai l ure. The i ncreas ed pul monary bl ood fl ow creat ed by t hi s l es i on may l ead t o t he devel opment of pul monary hypert ens i on,
Pa g e 1 9 2
ABC Ambe r CHM Conve rte r Tria l ve rsion, http://w w w .proce sste x t.com/a bcchm.html
i mpos i ng a pres s ure overl oad on t he ri ght vent ri cl e.
2. Physical signs o
o
a. Murmur. Throughout t he cardi ac cycl e, t he vas cul ar res i s t ance and pres s ure i n t he pul monary ci rcui t are l ower t han t he res i s t ance and pres s ure i n t he aort a. Therefore, bl ood i s s hunt ed from l eft t o ri ght i n bot h s ys t ol e and di as t ol e, and a continuous murmur wi t h s ys t ol i c and di as t ol i c component s i s heard.
o
o
b. Pulses. The pres ence of a l ow-pres s ure, l ow-res i s t ance pat hway al l ows for i ncreas ed aort i c runoff i n di as t ol e, whi ch produces bounding, full pulses s i mi l ar t o t hos e found i n aort i c i ns uffi ci ency. P.36
o
o
3. Diagnosis
a. Chest radiography reveal s an enl arged cardi ac s i l houet t e wi t h t he pres ence of s hunt vas cul ari t y. In adul t s , t he pat ent duct us may become cal ci fi ed, renderi ng i t vi s i bl e on t he ches t radi ograph.
b. Echocardiography may reveal t he pat ent duct us . Doppl er i nt errogat i on det ect s
Pa g e 1 9 3
ABC Ambe r CHM Conve rte r Tria l ve rsion, http://w w w .proce sste x t.com/a bcchm.html
abnormal fl ow of bl ood from t he aort a t o t he pul monary art ery.
c. Cardiac catheterization
(1) Duri ng cardi ac cat het eri zat i on, t he cat het er us ual l y can be pas s ed from t he pul monary art ery i nt o t he des cendi ng aort a, confi rmi ng t he pres ence of a pat ent duct us art eri os us .
(2) Oximetry can be us ed t o quant i fy t he magni t ude of t he l eft -t o-ri ght s hunt .
(3) Aortography demons t rat es t he fl ow of cont ras t from t he aort a t hrough t he pat ent duct us i nt o t he pul monary art ery.
d. Cardiac multislice CT or MRI may al s o demons t rat e a pat ent duct us art eri os us .
o
o
4. T herapy. Cat het er-bas ed cl os ure or s urgi cal cl os ure of t he pat ent duct us i s i ndi cat ed i n adul t s wi t h a s hunt rat i o of great er t han 2:1.
D. Coarctation of the aorta Thi s defect i s a s t enos i s of t he aort a, us ual l y at t he s i t e of t he duct us art eri os us .
1. Pathophysiology. Coarct at i on of t he aort a oft en l eads t o hypert ens i on.
Pa g e 1 9 4
ABC Ambe r CHM Conve rte r Tria l ve rsion, http://w w w .proce sste x t.com/a bcchm.html o
o
a. If t he s t enos i s i s s evere, i t l i mi t s aort i c bl ood fl ow di s t al t o t he cons t ri ct i on. Di s t al t i s s ues are perfus ed by an ext ens i ve col l at eral art eri al ci rcul at i on.
o
o
b. W hereas renal bl ood fl ow and renal funct i on us ual l y are normal i n adul t s wi t h coarct at i on of t he aort a, t he ki dneys s t i l l are perfus ed at a s ubnormal bl ood pres s ure.
2. Clinical features. If t he coarct at i on does not caus e heart fai l ure due t o pres s ure overl oad i n chi l dhood, i t may not be det ect ed unt i l i t mani fes t s as hypert ens i on i n t he adul t . o
o
a. Symptoms. Pat i ent s wi t h coarct at i on may compl ai n of headache, claudication, and leg fatigue.
o
o
b. Physical signs
(1) Blood pressure det ermi ned i n t he arms us ual l y i s el evat ed, whereas pul s es and bl ood pres s ure i n t he l egs us ual l y are reduced, repres ent i ng t he gradi ent acros s t he coarct at i on.
(2) Habitus. The upper body us ual l y i s wel l devel oped, whereas t he l egs occas i onal l y
Pa g e 1 9 5
ABC Ambe r CHM Conve rte r Tria l ve rsion, http://w w w .proce sste x t.com/a bcchm.html
appear underdevel oped.
(3) Murmur. Typi cal l y, a midsystolic murmur i s heard over t he back. If t he s t enos i s i s s evere, a cont i nuous murmur may be heard. Cont i nuous murmurs al s o may be heard di ffus el y over t he ches t cavi t y as t he res ul t of i ncreas ed fl ow t hrough col l at eral ves s el s .
3. Diagnosis o
o
a. Electrocardiography. The ECG s hows l eft vent ri cul ar hypert rophy.
o
o
b. Chest radiography. Cardiac enlargement us ual l y i s s een. Di l at i on of t he aort a proxi mal and di s t al t o t he coarct at i on wi t h i ndent at i on at t he s i t e of t he coarct at i on may caus e t he aort a t o as s ume a figure “ 3― appearance. Di l at i on of ches t wal l art eri es formi ng t he col l at eral pat hways produces rib notching.
o
o
c. Cardiac catheterization. Duri ng cardi ac cat het eri zat i on, t he gradi ent acros s t he coarct at i on can be meas ured. Aort ography al s o al l ows vi s ual demons t rat i on of t he coarct at i on.
o
o
d. Cardiac multislice CT or MRI may al s o reveal a coarct at i on of t he aort a.
Pa g e 1 9 6
ABC Ambe r CHM Conve rte r Tria l ve rsion, http://w w w .proce sste x t.com/a bcchm.html
4. T herapy. Surgical correction of t he coarct at i on i s s t andard t herapy. Percutaneous balloon aortoplasty wi t h or wi t hout s t ent i ng may be a s ui t abl e al t ernat i ve for s el ect ed pat i ent s .
5. Complications. Hypertension, infective endocarditis, dissection of the thoracic aorta, and rupture of cerebral (berry) aneurysms are s een frequent l y. Hypert ens i on may pers i s t even aft er t he coarct at i on i s repai red.
E. Ebstein's anomaly of the tricuspid valve
1. Pathophysiology. In Ebs t ei n's anomal y, t he t ri cus pi d val ve i s s i t uat ed abnormal l y l ow i n t he ri ght vent ri cl e. Part of t he t ri cus pi d val ve i s t et hered di rect l y t o t he ri ght vent ri cl e. Thus , a port i on of t he ri ght vent ri cl e act ual l y l i es above t he AV groove and i s “at ri al i zed,― reduci ng t he s i ze of t he ri ght vent ri cl e and us ual l y res ul t i ng i n tricuspid regurgitation. A coexistent atrial septal defect occurs i n approxi mat el y 75% of cas es . P.37
2. Clinical features o
o
a. Symptoms. Dependi ng on t he degree of t ri cus pi d regurgi t at i on and whet her an at ri al s ept al defect exi s t s , a pat i ent 's s t at us may range from asymptomatic to cyanotic.
Pa g e 1 9 7
ABC Ambe r CHM Conve rte r Tria l ve rsion, http://w w w .proce sste x t.com/a bcchm.html
(1) Dys pnea on exert i on, peri pheral edema, and ot her symptoms of right ventricular failure frequent l y are encount ered.
(2) Pal pi t at i ons al s o are common i n t hi s anomal y, whi ch i s as s oci at ed wi t h Wolff-Parkinson-White (WPW) syndrome i n approxi mat el y 10% of pat i ent s . W PW s yndrome i s charact eri zed by abnormal vent ri cul ar conduct i on as t he res ul t of a congeni t al s hort ci rcui t of t he conduct i ng s ys t em. Tachyarrhyt hmi as are common.
o
o
b. Physical signs
(1) T ricuspid regurgitation. A large v wave i n t he neck vei ns and a pul s at i l e l i ver refl ect t ri cus pi d regurgi t at i on.
(2) Heart sounds. W i de s pl i t t i ng of t he S 1 and S 2 i s heard. Becaus e an S 3 and an S 4 oft en al s o exi s t , a quadrupl e or qui nt upl e cadence i s a common aus cul t at ory fi ndi ng.
(3) Murmur. The holosystolic murmur of tricuspid regurgitation i s heard al ong t he s t ernal border and may be accompani ed by a s ys t ol i c t hri l l .
Pa g e 1 9 8
ABC Ambe r CHM Conve rte r Tria l ve rsion, http://w w w .proce sste x t.com/a bcchm.html
3. Diagnosis o
o
a. Electrocardiography. The ECG may s how evi dence of W PW s yndrome (a s hort PR i nt erval and a s l urred QRS ups t roke). Ot her fi ndi ngs i ncl ude ri ght at ri al enl argement and ri ght bundl e branch bl ock.
o
o
b. Echocardiography. The echocardi ogram i n Ebs t ei n's anomal y s hows del ayed cl os ure of t he t ri cus pi d val ve i n rel at i on t o t he mi t ral val ve. The i nferi or and l eft ward di s pl acement of t he t ri cus pi d val ve us ual l y can be demons t rat ed.
4. T herapy. Tri cus pi d val ve repl acement and cl os ure of t he at ri al s ept al defect may be us eful i n pat i ent s who have devel oped earl y s i gns of ri ght vent ri cul ar fai l ure.
F. Eisenmenger's syndrome
1. Pathophysiology. In Ei s enmenger's s yndrome, whi ch can occur wi t h any i nt racardi ac s hunt , t he left-to-right shunt is reversed t o produce a ri ght -t o-l eft s hunt . Revers al occurs as a res ul t of pul monary vas cul ar di s eas e t hat l eads t o i ncreas ed pul monary vas cul ar res i s t ance. Increas ed pul monary vas cul ar res i s t ance l eads t o decreas ed ri ght -s i ded compl i ance and i ncreas ed ri ght -s i ded pres s ures , whi ch produce ri ght -t o-l eft s hunt i ng.
2. Clinical features o
Pa g e 1 9 9
ABC Ambe r CHM Conve rte r Tria l ve rsion, http://w w w .proce sste x t.com/a bcchm.html
o
a. Cyanosis may be cons t ant or not ed onl y duri ng exerci s e. Differential cyanosis may occur i n t he pres ence of a pat ent duct us art eri os us ; t he preduct al t i s s ues (i ncl udi ng t he upper t runk) are pi nk, and t he pos t duct al t i s s ues are cyanot i c.
o
o
b. Angina. Pat i ent s wi t h Ei s enmenger's s yndrome may experi ence exert i onal ches t pai n, whi ch occurs even i n t he pres ence of normal coronary art eri es . Reduced myocardi al oxygenat i on and i ncreas ed ri ght vent ri cul ar wal l s t res s may be fact ors caus i ng t he s ympt om.
o
o
c. Heart failure. Dys pnea on exert i on, as ci t es , and peri pheral edema are common.
3. Diagnosis o
o
a. Electrocardiography. Ri ght vent ri cul ar hypert rophy i nvari abl y i s pres ent .
o
o
b. Echocardiography. Saline injection demons t rat es ri ght -t o-l eft s hunt i ng of mi cro bubbl es i n t he pres ence of ei t her an at ri al or a vent ri cul ar s ept al defect . Doppler examination al s o demons t rat es t he abnormal ri ght -t o-l eft bl ood fl ow at t he s i t e of t he s hunt .
o
o
c. Laboratory data. Due t o chroni c hypoxemi a, pat i ent s wi t h Ei s enmenger's s yndrome are
Pa g e 2 0 0
ABC Ambe r CHM Conve rte r Tria l ve rsion, http://w w w .proce sste x t.com/a bcchm.html
polycythemic. Hemogl obi n concent rat i ons i n exces s of 20 g/dL are common. o
o
d. Cardiac catheterization. Ri ght -s i ded pres s ures are ext remel y el evat ed. Oxi met ry i s us ed t o quant i t at e t he ri ght -t o-l eft s hunt . Admi ni s t rat i on of 100% oxygen vi a a rebreat hi ng mas k does not s i gni fi cant l y correct t he art eri al des at urat i on.
4. T herapy. Surgi cal t herapy general l y i s not s ucces s ful . o
o
a. Closure of the shunt site, whi ch act s as an es cape val ve for t he ri ght vent ri cl e, i ncreas es ri ght vent ri cul ar pres s ures and caus es wors eni ng of ri ght vent ri cul ar fai l ure.
o
o
b. Phlebotomy may be neces s ary t o avoi d hypervi s cos i t y by mai nt ai ni ng t he hemogl obi n l evel at l es s t han 20 g/dL.
P.38
VIII. Venous Thrombosis A. Deep venous thrombosis
1. Definition. Deep venous t hrombos i s occurs when a bl ood cl ot forms i n t he l ower ext remi t i es or i n t he pel vi c vei ns . The gravi t y of deep venous t hrombos i s s t ems from t he t endency of t he t hrombi t o become pul monary embol i . Thi s t endency i s es peci al l y pronounced for cl ot s
Pa g e 2 0 1
ABC Ambe r CHM Conve rte r Tria l ve rsion, http://w w w .proce sste x t.com/a bcchm.html
l ocat ed above t he popl i t eal fos s a.
2. Predisposing factors o
o
a. Immobilization. The mus cl es i n t he l egs act as pumps t o mai nt ai n venous ret urn from t he l ower ext remi t i es . Inact i vi t y of t hes e mus cl es l eads t o venous stasis, wi t h s ubs equent devel opment of thrombophlebitis. St as i s i s l i kel y t o occur duri ng s urgery, prol onged bed res t , and prol onged peri ods i n one pos i t i on.
o
o
b. Venous incompetence. Venous val vul ar i ncompet ence and t he pres ence of vari cos e vei ns i ncreas e t he i nci dence of t hrombophl ebi t i s .
o
o
c. CHF. In CHF, cardi ac out put i s reduced, as i s venous ret urn from t he l egs .
o
o
d. Injury. Di rect mechani cal i njury t o t he l ower ext remi t i es may l ead t o bl ood cl ot format i on and t he devel opment of t hrombophl ebi t i s .
o
o
e. Hypercoagulable states. Malignancy, estrogen use, and hyperviscosity syndrome may produce a hypercoagul abl e s t at e, i ncreas i ng t he ri s k of t hrombophl ebi t i s .
3. Clinical features o
Pa g e 2 0 2
ABC Ambe r CHM Conve rte r Tria l ve rsion, http://w w w .proce sste x t.com/a bcchm.html
o
a. Symptoms. The pat i ent us ual l y pres ent s wi t h unilateral leg pain and swelling.
o
o
b. Physical signs. In general , t he phys i cal exami nat i on i s unrel i abl e. Tendernes s on compres s i on of t he cal f mus cl es , i ncreas ed res i s t ance duri ng dors i fl exi on of t he foot (Homans' sign), and an i ncreas e i n t he ci rcumference of t he affect ed l eg by at l eas t 1 cm s ugges t t he pres ence of deep venous t hrombos i s .
4. Diagnosis o
o
a. Noninvasive studies. Impedance plethysmography and Doppler ultrasonography are us eful t es t s for t he det ect i on of deep venous t hrombos i s .
o
o
b. Invasive studies. Contrast venography current l y i s t he mos t effect i ve way t o demons t rat e t he area of bl ood cl ot . Thi s t echni que i s as s oci at ed wi t h compl i cat i ons , i ncl udi ng advers e react i ons t o t he cont ras t agent and pos t venography t hrombophl ebi t i s .
5. T herapy. Anticoagulants prevent addi t i onal cl ot format i on and al l ow t he body's aut ol yt i c s ys t em t o l ys e effect i vel y and heal deep venous t hrombos i s . Ant i coagul at i on t herapy i s us ual l y mai nt ai ned for 3–6 mont hs . o
Pa g e 2 0 3
ABC Ambe r CHM Conve rte r Tria l ve rsion, http://w w w .proce sste x t.com/a bcchm.html
o
a. Ant i coagul at i on wi t h intravenous heparin i s i ndi cat ed i n t he acut e t reat ment of deep venous t hrombos i s . LMW H appears t o be as effect i ve as unfract i onat ed hepari n. Al t hough t he l ow–mol ecul ar-wei ght form i s more expens i ve, i t does not requi re l aborat ory moni t ori ng. In cas es of hepari n-i nduced t hrombocyt openi a, a di rect t hrombi n i nhi bi t or s houl d be s ubs t i t ut ed for hepari n.
o
o
b. Aft er adequat e t reat ment wi t h hepari n, oral anticoagulation with warfarin i s begun.
6. Prophylaxis. There i s s ubs t ant i al medi cal evi dence t hat t he i nci dence of deep venous t hrombos i s for hos pi t al i zed pat i ent s can be reduced by t he fol l owi ng met hods . o
o
a. Rapid mobilization. Prol onged bed res t s houl d be avoi ded when pos s i bl e. The i ncreas i ngl y rapi d mobi l i zat i on of pat i ent s fol l owi ng MI has s i gni fi cant l y reduced t he i nci dence of t hromboembol i c compl i cat i ons fol l owi ng t hi s di s eas e.
o
o
b. Increasing deep venous flow
(1) Antithromboembolic stockings and pneumatic compression devices compres s t he s uperfi ci al vei ns , t hereby i ncreas i ng deep venous fl ow and reduci ng s t as i s and t he
Pa g e 2 0 4
ABC Ambe r CHM Conve rte r Tria l ve rsion, http://w w w .proce sste x t.com/a bcchm.html
i nci dence of t hromboembol i s m.
(2) Foot exercises and avoidance of leg crossing are furt her met hods of prevent i ng deep venous t hrombos i s .
o
o
c. “ Minidose― heparin. Int ermi t t ent dos es of s ubcut aneous hepari n gi ven at 8- hour i nt erval s i nhi bi t fact ors X and XI i n t he cl ot t i ng cas cade wi t hout produci ng overt ant i coagul at i on. Thi s t reat ment s i gni fi cant l y reduces t he i nci dence of deep venous t hrombos i s i n bot h medi cal and s urgi cal pat i ent s on bed res t .
P.39
B. Superficial thrombophlebitis Unl i ke deep venous t hrombos i s , i n whi ch a t hrombus may break off and become a pul monary embol i s m, s uperfi ci al t hrombophl ebi t i s has little potential for embolic complications. Pat i ent s wi t h s uperfi ci al t hrombophl ebi t i s may pres ent wi t h a pai nful t ender cord t hat can be eas i l y pal pat ed i n t he l ower ext remi t i es . In t he abs ence of concomi t ant deep venous t hrombos i s , anticoagulation is not indicated. Superfi ci al t hrombophl ebi t i s i s treated with elevation of the legs, heat , and administration of salicylates or other NSAIDs.
IX. Cardiovascular Syncope A. Definition Syncope i s a s udden l os s of cons ci ous nes s of bri ef durat i on.
B. Pathophysiology Cardi ovas cul ar s yncope occurs when t he brai n's met abol i c needs cannot be met by t he avai l abl e bl ood s uppl y. Adequat e perfus i on i s
Pa g e 2 0 5
ABC Ambe r CHM Conve rte r Tria l ve rsion, http://w w w .proce sste x t.com/a bcchm.html
dependent on an adequat e s ys t emi c bl ood pres s ure: BP = CO × SVR (where BP = bl ood pres s ure, CO = cardi ac out put , and SVR = s ys t emi c vas cul ar res i s t ance). Therefore, a fal l i n cardi ac out put or a fal l i n s ys t emi c vas cul ar res i s t ance can preci pi t at e a fal l i n bl ood pres s ure, l eadi ng t o s yncope. Becaus e CO = SV × HR (where SV = s t roke vol ume and HR = heart rat e), ei t her i nadequat e s t roke vol ume or i nadequat e heart rat e reduces cardi ac out put , pot ent i al l y l eadi ng t o hypot ens i on and s yncope.
C. Etiology
1. Reduced cardiac output o
o
a. Bradycardia
(1) Heart block. A bl ock i n t he cardi ac conduct i on pat hway may prevent t he SA nodal el ect ri cal s i gnal for vent ri cul ar cont ract i on from bei ng t rans mi t t ed, i n t urn caus i ng bradycardi a. W het her s yncope occurs depends on whet her an al t ernat i ve, l ower pacemaker (e.g., t he AV junct i on) produces an es cape rat e t hat i s s uffi ci ent l y fas t t o mai nt ai n bl ood pres s ure.
(2) Sick sinus syndrome occurs when t here i s a defi ci t i n i mpul s e generat i on from t he SA node, whi ch may l ead t o bradycardi a and s yncope.
(a) Profound s i nus bradycardi a, s i nus arres t , and t he t achycardi a–bradycardi a
Pa g e 2 0 6
ABC Ambe r CHM Conve rte r Tria l ve rsion, http://w w w .proce sste x t.com/a bcchm.html
s yndrome are t he arrhyt hmi as t hat cons t i t ut e t he s i ck s i nus s yndrome. The t achycardi a–bradycardi a s yndrome i s one of at ri al i ns t abi l i t y where s upravent ri cul ar t achycardi a hal t s abrupt l y and i s fol l owed by s evere bradycardi a.
(b) Si ck s i nus s yndrome may res ul t from i s chemi c heart di s eas e and i di opat hi c or i nfl ammat ory degenerat i on of t he SA node.
o
o
b. Impaired stroke volume. The rhyt hmi c fi l l i ng and empt yi ng of t he l eft vent ri cl e generat es i t s s t roke vol ume; t herefore, condi t i ons t hat ei t her i nhi bi t l eft vent ri cul ar fi l l i ng or i nhi bi t l eft vent ri cul ar empt yi ng can s everel y reduce s t roke vol ume, l eadi ng t o hypot ens i on and s yncope.
(1) Conditions that limit left ventricular filling
(a) Obstruction to inflow. Any mechani cal bl ock i n t he cardi ovas cul ar s ys t em t hat i nhi bi t s fi l l i ng of t he l eft vent ri cl e i mpai rs i t s out put . Such obs t ruct i ons i ncl ude mi t ral s t enos i s , l eft at ri al myxoma, ri ght at ri al myxoma, pul monary embol i s m, and pul moni c s t enos i s .
Pa g e 2 0 7
ABC Ambe r CHM Conve rte r Tria l ve rsion, http://w w w .proce sste x t.com/a bcchm.html
(b) T achycardia. Bot h vent ri cul ar t achycardi a and very rapi d s upravent ri cul ar t achycardi a reduce t he di as t ol i c fi l l i ng peri od of t he l eft vent ri cl e, l i mi t i ng i t s fi l l i ng and reduci ng i t s s t roke vol ume. In vent ri cul ar t achycardi a, t he s hort ened di as t ol i c fi l l i ng peri od i s compounded by i ncompl et e vent ri cul ar rel axat i on, whi ch furt her l i mi t s fi l l i ng.
(c) Impaired systemic venous return. Fai l ure of adequat e s ys t emi c venous ret urn t o t he ri ght heart s ubs equent l y i mpai rs i t s out put t o t he l eft heart , i mpai ri ng l eft vent ri cul ar s t roke vol ume.
(i) Typi cal l y, i mpai red venous ret urn occurs when t he s upi ne pat i ent as s umes t he upri ght pos t ure.
(ii) Normal l y, t he t endency for gravi t y-i nduced venous pool i ng of bl ood i n t he l egs i s offs et by venous vas ocons t ri ct i on, whi ch hel ps mai nt ai n venous ret urn. However, i n t he face of dehydrat i on, ant i hypert ens i ve drugs , or aut onomi c dys funct i on, i mpai red venous ret urn may produce orthostatic syncope. Aut onomi c dys funct i on may be
Pa g e 2 0 8
ABC Ambe r CHM Conve rte r Tria l ve rsion, http://w w w .proce sste x t.com/a bcchm.html
i di opat hi c; fami l i al ; s urgi cal l y i nduced; or res ul t from di abet es , al cohol i s m, or pyri doxi ne defi ci ency. P.40
(2) Conditions that impair left ventricular emptying. The l eft vent ri cl e may be i mpai red from empt yi ng ei t her as a res ul t of a s evere, s udden depres s i on i n myocardi al cont ract i l e funct i on or as a res ul t of out fl ow obs t ruct i on.
(a) Decreased myocardial contractility. The s udden and s evere degree of cont ract i l e depres s i on requi red t o caus e s yncope i s al mos t i nvari abl y caus ed by gl obal i s chemi a produced by l eft mai n or t ri pl e-ves s el coronary di s eas e, acut e MI, vent ri cul ar t achycardi a, or vent ri cul ar fi bri l l at i on.
(b) Obstruction to outflow. Obs t ruct i on of l eft vent ri cul ar out fl ow t hat produces s yncope i s caus ed by val vul ar aort i c s t enos i s and hypert rophi c cardi omyopat hy.
2. Reduced total peripheral resistance. If cardi ac out put i s mai nt ai ned but t ot al peri pheral res i s t ance fal l s , bl ood pres s ure al s o fal l s , pot ent i al l y caus i ng s yncope. o
Pa g e 2 0 9
ABC Ambe r CHM Conve rte r Tria l ve rsion, http://w w w .proce sste x t.com/a bcchm.html
o
a. An i nappropri at e fal l i n t ot al peri pheral res i s t ance i s us ual l y operat i ve i n t he common fainting spell. Increas ed bl ood fl ow t o t he s kel et al mus cl es due t o a fal l i n t ot al peri pheral res i s t ance may di vert fl ow from t he brai n and res ul t i n fai nt i ng. Venodi l at i on and rel at i ve bradycardi a may furt her compound t he “ vasovagal faint― by reduci ng venous ret urn and cardi ac out put .
o
o
b. Reduced t ot al peri pheral res i s t ance l eadi ng t o s yncope may al s o occur i n drug-i nduced, fami l i al , or i di opat hi c aut onomi c dys funct i on.
D. Diagnosis A s i ngl e fai nt i ng epi s ode or epi s ode of l i ght -headednes s occurs i n more t han 50% of t he popul at i on at s ome poi nt i n a l i fet i me. It woul d be i mpos s i bl e t o expl ore t he caus e of t he event ext ens i vel y i n every affect ed pat i ent . A good hi s t ory and phys i cal exami nat i on s houl d be adequat e t o excl ude pot ent i al l y s eri ous caus es of a s i ngl e epi s ode of s yncope. However, recurrent s yncope requi res a more ext ens i ve workup.
1. History. A t horough pat i ent i nt ervi ew can reveal cl ues t hat may poi nt t o a s peci fi c et i ol ogy for t he recurrent s yncope. o
o
a. A hi s t ory of pal pi t at i ons mi ght i ndi cat e an arrhyt hmi a.
o
o
b. The obs ervat i on t hat s yncope occurred upon as s umpt i on of an upri ght pos i t i on s ugges t s ort hos t at i c hypot ens i on.
Pa g e 2 1 0
ABC Ambe r CHM Conve rte r Tria l ve rsion, http://w w w .proce sste x t.com/a bcchm.html o
o
c. A hi s t ory of ches t pai n mi ght i ndi cat e an i s chemi c event or pul monary embol i s m.
o
o
d. A change i n ant i hypert ens i ve medi cat i on or a recent epi s ode of dehydrat i on are addi t i onal cl ues .
o
o
e. A hi s t ory of a pri or MI and reduced l eft vent ri cul ar funct i on s ugges t s t he pos s i bi l i t y of a vent ri cul ar arrhyt hmi a.
2. Physical examination. Thos e maneuvers t hat mi ght reveal a reas on for hypot ens i on and pos s i bl e s yncope s houl d be emphas i zed. o
o
a. Blood pressure
(1) The bl ood pres s ure s houl d be recorded i n bot h arms i n bot h t he s upi ne and t he s i t t i ng or s t andi ng pos i t i ons .
(2) On as s umi ng an upri ght pos t ure, i t i s normal for s ys t ol i c bl ood pres s ure t o fal l s l i ght l y whi l e di as t ol i c pres s ure i ncreas es . There i s al s o us ual l y a s l i ght i ncreas e i n heart rat e.
(a) A frank decl i ne i n s ys t ol i c and di as t ol i c pres s ure on as s umi ng an
Pa g e 2 1 1
ABC Ambe r CHM Conve rte r Tria l ve rsion, http://w w w .proce sste x t.com/a bcchm.html
upri ght pos t ure may i ndi cat e vol ume depl et i on or s ympat het i c compens at i on t hat i s i nadequat e t o count eract t he change i n pos t ure.
(b) A fal l i n di as t ol i c pres s ure of more t han 10 mm Hg i s s i gni fi cant and may s ugges t an ort hos t at i c et i ol ogy of t he s yncope.
o
o
b. Heart rate and rhythm. The pul s e s houl d be exami ned for an ext ended peri od of t i me i n an effort t o det ect arrhyt hmi a or bradycardi a.
o
o
c. Valvular obstruction. The murmurs and phys i cal fi ndi ngs as s oci at ed wi t h mi t ral s t enos i s , aort i c s t enos i s , pul moni c s t enos i s , or i di opat hi c hypert rophi c s ubaort i c s t enos i s s houl d be recogni zed as i ndi cat i ons of pot ent i al l y correct abl e mechani s ms for s yncope.
o
o
d. T hromboembolism. Thrombophl ebi t i s i n t he l ower l i mbs i ndi cat es a s ource of pul monary embol i , whi ch can caus e s yncope. Phys i cal evi dence t hat a pul monary embol us i s pres ent i ncl udes wheezi ng, i ncreas ed i nt ens i t y of t he pul monary component of t he S 2 , and jugul ar venous di s t ent i on.
P.41
3. Electrocardiography. If s econd- or t hi rd-degree AV
Pa g e 2 1 2
ABC Ambe r CHM Conve rte r Tria l ve rsion, http://w w w .proce sste x t.com/a bcchm.html
bl ock i s det ect ed, i t demons t rat es t he l i kel y caus e of t he s yncope. Bundl e branch bl ock and arrhyt hmi as , or bot h, on t he s t andard ECG s houl d rai s e s us pi ci on t hat heart bl ock or arrhyt hmi a are s yncopal et i ol ogi es .
4. Holter monitoring. If t he hi s t ory, phys i cal exami nat i on, and ECG poi nt t o an arrhyt hmi a as t he pot ent i al caus e of t he s yncope, Hol t er moni t ori ng may be performed. It records each heart beat over a 24-hour peri od, whi ch i s t hen revi ewed for arrhyt hmi a. Unfort unat el y, becaus e mos t arrhyt hmi as occur s poradi cal l y, mos t Hol t er moni t or exami nat i ons are negat i ve even when an arrhyt hmi a i s t he s ource of t he s yncope.
5. Event monitors. Thes e devi ces are worn or carri ed for s everal weeks . They are act i vat ed by t he pat i ent at t he t i me of s ympt oms and document t he rhyt hm before and aft er act i vat i on. The recordi ng i s t hen revi ewed for arrhyt hmi a.
6. Electrophysiologic testing. If t he i ni t i al work-up demons t rat es t hat heart di s eas e i s pres ent but fai l s t o demons t rat e a s peci fi c caus e of s yncope, el ect rophys i ol ogi c s t i mul at i on may provoke t he arrhyt hmi a res pons i bl e for t he s yncope. Havi ng es t abl i s hed an arrhyt hmi c caus e, t he proper t herapy may t hen be i ns t i t ut ed.
E. Therapy
1. T herapy for bradyarrhythmias. W hen a bradyarrhyt hmi a has been es t abl i s hed as t he caus e of
Pa g e 2 1 3
ABC Ambe r CHM Conve rte r Tria l ve rsion, http://w w w .proce sste x t.com/a bcchm.html
t he s yncope, drug-i nduced bradycardi a s houl d be rul ed out as a caus e by di s cont i nui ng pot ent i al l y offendi ng drugs . If s ympt omat i c bradycardi a pers i s t s , a permanent pacemaker i s i ndi cat ed.
2. T herapy for tachyarrhythmias o
o
a. Drug therapy for bot h vent ri cul ar and s upravent ri cul ar t achyarrhyt hmi as t hat have caus ed an epi s ode of s yncope i s cl earl y i ndi cat ed [s ee II D 1 b (3)]. In general , s uch t herapy s houl d be gui ded by el ect rophys i ol ogi c t es t i ng.
o
o
b. Antitachycardia pacemakers or implantable defibrillators may be us ed t o el ect ri cal l y correct arrhyt hmi as i f drug t herapy fai l s .
3. T herapy for autonomic dysfunction. If aut onomi c dys funct i on i s t he caus e of ort hos t at i c hypot ens i on and s yncope, l i t t l e can be done di rect l y t o t reat t he underl yi ng caus e. Ins t ead, t herapi es t o prot ect t he pat i ent from pos s i bl e hypot ens i on s houl d be i ns t i t ut ed. Thes e i ncl ude hi gh s al t i nt ake or fl udrocort i s one t o ens ure vol ume expans i on, s upport s t ocki ngs t o prevent venous pool i ng, or mi dodri ne t o i ncreas e peri pheral vas cul ar res i s t ance. There may be a rol e for permanent cardi ac paci ng i n a s el ect group of pat i ent s for whom ot her forms of t herapy have fai l ed.
4. Correction of mechanical obstructions to cardiac inflow or outflow. Any fi xed val vul ar l es i on t hat has caus ed an epi s ode of s yncope s houl d be correct ed. If
Pa g e 2 1 4
ABC Ambe r CHM Conve rte r Tria l ve rsion, http://w w w .proce sste x t.com/a bcchm.html
i di opat hi c hypert rophi c s ubaort i c s t enos i s i s det ermi ned t o be t he caus e of t he s yncope, s t andard t herapy wi t h propranol ol or verapami l i s i ndi cat ed t o reduce t he amount of out fl ow obs t ruct i on. If medi cal t herapy fai l s , myomect omy or al cohol s ept al abl at i on may be neces s ary. P.42
Study Questions/Answers and Explanations A 52-year-ol d man pres ent s wi t h fever, chi l l s , and art hral gi a. On phys i cal exami nat i on: t emperat ure i s 102.2°F, pul s e i s 106 bpm, bl ood pres s ure i s 100/60 mm Hg, res pi rat ory rat e i s 22. S 1 i s s oft . There i s a s hort II/VI di as t ol i c bl owi ng murmur at t he l eft s t ernal border. There are no ras hes or pet echi ae. The res ul t s of t he res t of t he exami nat i on are unremarkabl e. 1. What is the most likely diagnosis? A Vi ral s yndrome wi t h fl ow murmur B Acut e s ys t emi c l upus eryt hemat os us wi t h aort i c val ve i nvol vement C Infect i ve endocardi t i s of t he aort i c val ve wi t h probabl y mi l d i ns uffi ci ency D Infect i ve endocardi t i s of t he mi t ral val ve E Infect i ve endocardi t i s of t he aort i c val ve wi t h probabl y s evere i ns uffi ci ency Vi ew Ans wer 1. T he answer is E [IV C 3 b (2)]. The di as t ol i c murmur i s t ypi cal of t hat of aort i c i ns uffi ci ency. The fever chi l l s and art hral gi a s ugges t i nfect i on, maki ng i nfect i ve endocardi t i s t he mos t l i kel y di agnos i s . The s oft S 1 s ugges t s mi t ral val ve precl os ure, i ndi cat i ng s evere di s eas e. Thi s s yndrome coul d be s een i n acut e l upus , but t hi s i s l es s l i kel y i n a man wi t hout ot her evi dence of t he di s eas e. Increas ed fl ow from any caus e does not produce aort i c i ns uffi ci ency. A l es i on on t he mi t ral val ve creat es s ys t ol i c, not di as t ol i c, murmurs .
Pa g e 2 1 5
ABC Ambe r CHM Conve rte r Tria l ve rsion, http://w w w .proce sste x t.com/a bcchm.html
2. Which of the following statements is true of the condition of the patient in question 1? A The cardi ac phys i cal exami nat i on i s hyperdynami c. B S 1 i s s oft becaus e of aort i c val ve precl os ure. C Mi t ral val ve precl os ure i ndi cat es a poor prognos i s wi t hout aort i c val ve repl acement . D The appearance of a di as t ol i c murmur i s us ual l y beni gn. E Hi l l 's s i gn i s a good predi ct or of s everi t y. Vi ew Ans wer 2. T he answer is C [IV C 3 b]. Mi t ral val ve precl os ure, caus ed by hi gh vent ri cul ar di as t ol i c fi l l i ng pres s ure, great er t han l eft at ri al pres s ure, i ndi cat es s evere di s eas e t hat i s us ual l y fat al wi t hout aort i c val ve repl acement . In acut e aort i c i ns uffi ci ency s uch as t hat s een i n endocardi t i s , Left vent ri cul ar di l at i on has not yet occurred, s t roke vol ume i s not i ncreas ed very much, and t hus t he ci rcul at i on i s not hyperdynami c. Therefore Hi l l 's s i gn i s al s o abs ent . In general , di as t ol i c murmurs are not beni gn and i ndi cat e val ve pat hol ogy. 3. T he diagnostic test(s) that should be performed next is/are A A ches t x-ray B Bl ood cul t ures C Cardi ac cat het eri zat i on D A radi onucl i de vent ri cul ogram E Expl orat ory t horacot omy Vi ew Ans wer 3. T he answer is B [IV C 4]. Bl ood cul t ures t o confi rm a bl oods t ream i nfect i on and echocardi ography t o i dent i fy val ve l es i ons and val ve funct i on are t he mai ns t ays of di agnos i s i n i nfect i ve endocardi t i s . Al t hough a ches t x-ray mi ght be us eful , i t i s never di agnos t i c of endocardi t i s . Val ve s urgery woul d not be cont empl at ed wi t hout t he di agnos i s of endocardi t i s bei ng es t abl i s hed fi rs t . Cardi ac cat het eri zat i on i s rarel y i ndi cat ed i n endocardi t i s t oday becaus e echocardi ography provi des more i nformat i on more s afel y. A radi onucl i de vent ri cul ogram woul d gi ve i nformat i on about cardi ac performance but woul d not confi rm t he
Pa g e 2 1 6
ABC Ambe r CHM Conve rte r Tria l ve rsion, http://w w w .proce sste x t.com/a bcchm.html
di agnos i s . A 56-year-ol d man ent ers t he emergency depart ment compl ai ni ng of dys pnea t hat began about 3 weeks ago and has progres s ed s o t hat he now has di ffi cul t y wal ki ng acros s a room. He has begun s l eepi ng on t hree pi l l ows . On phys i cal exami nat i on: t emperat ure i s 99(F, pul s e 102 bpm, BP 130/90 mm Hg, res pi rat ory rat e 24. There i s jugul ar venous di s t ent i on, and es t i mat ed cent ral venous pres s ure i s 10 cm H 2 O. Ot her fi ndi ngs i ncl ude bi bas i l ar ral es and an S 3 gal l op. 4. What is the most likely diagnosis in this patient? A Pul monary embol i s m B Conges t i ve heart fai l ure C Emphys ema D Pneumoni a E At ri al s ept al defect Vi ew Ans wer 4. T he answer is B [I D 1–2]. The gradual ons et of dys pnea, t he pul monary ral es , and t he S 3 gal l op are al l t ypi cal of conges t i ve heart fai l ure. Al t hough a pul monary embol us coul d caus e al l of t he fi ndi ngs i n t hi s pat i ent , even a ri ght -s i ded S 3 , s udden ons et i s t he norm i n t hat condi t i on. The ot her condi t i ons al l coul d caus e dys pnea but woul d not caus e gal l op rhyt hm. 5. Which of the following tests is most appropriate to aid in establishing therapy for this patient? A A ches t x-ray B An echocardi ogram C An el ect rocardi ogram D A heart cat het eri zat i on E A radi onucl i de vent ri cul ogram Vi ew Ans wer 5. T he answer is B [I E (3)]. An echocardi ogram wi l l yi el d dat a about s ys t ol i c and di as t ol i c funct i on, chamber s i ze, and val vul ar abnormal i t i es . Al l of t he ot her t es t s are us eful , but al l except cat het eri zat i on gi ve l es s i nformat i on t han t he echocardi ogram. Cardi ac cat het eri zat i on has a hi gher ri s k and i s onl y empl oyed when t he i nformat i on gai ned out wei ghs t hat ri s k. Thus , i n CHF,
Pa g e 2 1 7
ABC Ambe r CHM Conve rte r Tria l ve rsion, http://w w w .proce sste x t.com/a bcchm.html
echocardi ography provi des t he “bi gges t bang for t he buck.― 6. Which of the following is true about the treatment of the condition of the patient in question 4? A The caus e of t he condi t i on s houl d be t reat ed whenever pos s i bl e. B Sys t ol i c vers us di as t ol i c dys funct i on us ual l y cannot be es t abl i s hed. C ACE i nhi bi t ors i mprove s ympt oms but do not prol ong l i fe. D Di uret i cs are t he court of l as t res ort . E β-bl ockers are dangerous and s houl d be avoi ded. Vi ew Ans wer 6. T he answer is A [I F (1)]. Conges t i ve heart fai l ure i s a s yndrome, and i t s caus e s houl d be s ought and t reat ed di rect l y whenever pos s i bl e. It i s us ual l y hel pful t o es t abl i s h whet her t he root caus e i s s ys t ol i c or di as t ol i c dys funct i on, a di s t i nct i on made eas i l y wi t h echocardi ography. Di uret i cs form t he mai ns t ay of t herapy, but addi ng bot h ACE i nhi bi t ors and β-bl ockers prol ongs l i fe. P.43
On a rout i ne offi ce vi s i t , a 45-year-ol d man compl ai ns t hat recent l y he has not ed ri ght -s i ded ches t pai n whi l e mowi ng hi s l awn wi t h a pus h l awn mower. The pai n devel ops s uddenl y, l as t s 2–3 mi nut es , and s ubs i des when he res t s . He deni es s moki ng or a hi s t ory of hypert ens i on, di abet es , or hyperl i pi demi a. Hi s phys i cal exami nat i on i s unremarkabl e. An ECG s hows nons peci fi c T-wave abnormal i t i es . 7. T his patient most likely has which of the following? A Angi na pect ori s B Hi at al herni a C Pl euri t i s D A nons peci fi c ches t pai n s yndrome E There i s not enough i nformat i on t o arri ve at a di agnos i s . Vi ew Ans wer 7. T he answer is E [III A 5 a (1)]. Hi s pres ent at i on wi t h exert i onal
Pa g e 2 1 8
ABC Ambe r CHM Conve rte r Tria l ve rsion, http://w w w .proce sste x t.com/a bcchm.html
pai n i s t ypi cal of angi na, but t he l ocat i on, durat i on, and l ack of ri s k fact ors are at ypi cal . No di agnos i s can be made bas ed on t hi s i nformat i on. 8. What should be the next step in establishing the diagnosis for this patient? A Repeat t he ECG B Perform a cardi ac cat het eri zat i on C Obt ai n cardi ac enzymes and a t roponi n l evel D Perform a s t res s ECG E Perform a s t res s echocardi ogram Vi ew Ans wer 8. T he answer is E [III A 5 a (3)]. A s t res s echocardi ogram wi l l gi ve i nformat i on about cardi ac funct i on and t he pres ence of coronary di s eas e (90% s ens i t i vi t y). Repeat i ng t he ECG i s unl i kel y t o gi ve new i nformat i on. The brevi t y of t he pai n makes i t very unl i kel y t hat myocardi al damage has occurred, and t hus t roponi n i s l i kel y t o be normal . The s t res s ECG wi l l be of l i mi t ed us e becaus e t he res t i ng ECG i s al ready abnormal . Cardi ac cat het eri zat i on coul d be empl oyed, but becaus e of i t s i nvas i ve nat ure i t i s us ual l y not t he fi rs t s t ep i n arri vi ng at a di agnos i s . 9. If coronary disease is found in this patient, indications for surgical revascularization would include which of the following? A Occas i onal angi na B Left mai n coronary s t enos i s of 20% C Three-ves s el coronary art ery di s eas e wi t h l eft vent ri cul ar s ys t ol i c dys funct i on D Di s eas e of t he ri ght and ci rcumfl ex art eri es E A s evere l es i on i n one coronary art ery Vi ew Ans wer 9. T he answer is C [III A 5 a (4) (e)]. Surgi cal revas cul ari zat i on has s hown a mort al i t y benefi t for pat i ent s wi t h di s eas e of >70% of al l t hree epi cardi al coronary art eri es and as s oci at ed l eft vent ri cul ar dys funct i on. In addi t i on, a mort al i t y benefi t has been s hown for pat i ent s wi t h >50% s t enos i s of t he l eft mai n coronary art ery, i rres pect i ve of vent ri cul ar funct i on. Gi ven t he ri s ks of s urgi cal
Pa g e 2 1 9
ABC Ambe r CHM Conve rte r Tria l ve rsion, http://w w w .proce sste x t.com/a bcchm.html
revas cul ari zat i on, medi cal t herapy and/or percut aneous i nt ervent i on s houl d be cons i dered fi rs t for pat i ent s who are not l i kel y t o experi ence a mort al i t y benefi t from bypas s s urgery. A 56-year-ol d man wi t h a hi s t ory of hypert ens i on i s s een for t he eval uat i on of ches t pai n t hat began an hour ago. The pain was cent ered i n t he l eft s i de of t he ches t and radi at es t o t he l eft arm. It was as s oci at ed wi t h naus ea and vomi t i ng. Hi s phys i cal exami nat i on fi ndi ngs are: Bl ood pres s ure 80/60 mm Hg, pul s e 58 bpm, and res pi rat ory rat e 16 Ches t : cl ear Heart : no gal l ops or murmurs ECG: Acut e ant eri or myocardi al i nfarct i on and s i nus bradycardi a 10. What should be the next step in management of this patient? A Ins ert i on of a t emporary pacemaker B Admi ni s t rat i on of ni t rat es C Fl ui d res us ci t at i on D Ins ert i on of an i nt ra-aort i c bal l oon pump E Admi ni s t rat i on of a β-bl ocker Vi ew Ans wer 10. T he answer is C [III A 5 b (4)]. The pat i ent i s hypot ens i ve, as he has no s i gns of vol ume overl oad or heart fai l ure; t hus , fl ui d res us ci t at i on s houl d be performed fi rs t . Al t hough bot h β-bl ockers and ni t rat es are i ndi cat ed i n MI, t hei r us e here woul d onl y exacerbat e t he hypot ens i on. A pacemaker mi ght i mprove bl ood pres s ure but onl y i f AV s equent i al paci ng were us ed, a s omet i mes compl ex procedure. In fact , pacemakers are rarel y us ed for mi l d s i nus bradycardi a. Int ra-aort i c bal l oon pumpi ng woul d be us ed onl y i f ot her meas ures fai l ed t o res t ore bl ood pres s ure. 11. After the patient in question 10 is stabilized, he should: A Be t rans ferred t o t he cri t i cal care uni t B Undergo i mmedi at e percut aneous coronary angi opl as t y i f avai l abl e C Recei ve warfari n D Recei ve ni fedi pi ne E Recei ve i nt ravenous l i docai ne
Pa g e 2 2 0
ABC Ambe r CHM Conve rte r Tria l ve rsion, http://w w w .proce sste x t.com/a bcchm.html
Vi ew Ans wer 11. T he answer is B [III A 5 b (4) (b)]. If acut e angi opl as t y i s avai l abl e, i t s houl d be performed i mmedi at el y t o res t ore coronary bl ood fl ow wi t hout t rans ferri ng t he pat i ent t o t he coronary care uni t (CCU), becaus e every mi nut e count s i n pres ervi ng myocardi um. Di hydropyri di ne cal ci um channel bl ockers s uch as ni fedi pi ne are cont rai ndi cat ed i n MI becaus e t hey i ncreas e mort al i t y. Li docai ne i s no l onger us ed prophyl act i cal l y agai ns t cardi ac arrhyt hmi as becaus e of pos s i bl e cardi ac s t ands t i l l . Al t hough hepari n i s an es s ent i al part of t herapy, warfari n, whi ch t akes days t o become effect i ve, i s not . 12. On the second hospital day, the patient becomes diaphoretic and hypotensive. A III/VI holosystolic murmur is heard. Which of the following is likely? A He has devel oped peri cardi al t amponade. B There has been acut e vent ri cul ar s ept al rupt ure. C He has an acut e at ri al s ept al defect . D He has devel oped mi t ral val ve endocardi t i s . E The murmur was ol d but obs cured by t he reduced cardi ac out put from hi s MI. Vi ew Ans wer 12. T he answer is B [III A 5 b (6) (e)]. Hemodynami c decompens at i on and a new cardi ac murmur aft er MI i ndi cat e ei t her acut e vent ri cul ar s ept al rupt ure or acut e mi t ral val ve dys funct i on. At ri al s ept al defect i s not a cons equence of MI. There i s no i ndi cat i on t hat t he pat i ent has devel oped endocardi t i s . If anyt hi ng, t he pat i ent 's out put has been s t i l l furt her reduced, as i ndi cat ed by hi s change i n vi t al s i gns . There are no s i gns of t amponade, s uch as pul s us paradoxus or neck vei n di s t ent i on. P.44
13. Ultimately this patient' s prognosis will be determined most by which of the following? A The amount of myocardi al damage he has s us t ai ned. B Hi s LDL chol es t erol l evel
Pa g e 2 2 1
ABC Ambe r CHM Conve rte r Tria l ve rsion, http://w w w .proce sste x t.com/a bcchm.html
C Hi s HDL chol es t erol l evel D The rat i o of LDL t o HDL chol es t erol E Bl ood pres s ure cont rol Vi ew Ans wer 13. T he answer is A [III A 5 b (5) (a)]. Prognos i s i s dependent mos t on t he amount of mus cl e damage (and t herefore t he amount t he vent ri cul ar dys funct i on t hat devel ops ), t he pat i ent 's age, and t he ext ent of coronary di s eas e. Al t hough i mprovi ng t he s t at us of known coronary ri s k fact ors s uch as hyperl i pi demi a and hypert ens i on reduces s ubs equent ri s k, t he effect on prognos i s i s not as l arge as are mus cl e damage, age, and ext ent of di s eas e. A 25-year-ol d woman pres ent s wi t h ches t pai n t hat wors ens when s he i ns pi res . Her phys i cal exami nat i on fi ndi ngs are bl ood pres s ure of 120/70 mm Hg, pul s e 76 bpm, res pi rat ory rat e 14, Heart : t hree-component fri ct i on rub. 14. Which of the following statements is true of the friction rub? A It i s generat ed by movement of t he pari et al and vi s ceral l ayers of t he peri cardi um. B It i s generat ed by t he vi s ceral l ayers of t he peri cardi um and pl eura. C It i ndi cat es t he abs ence of an effus i on. D It i ndi cat es t hat t he caus e of t he peri cardi t i s i s a mal i gnancy. E It oft en pers i s t s t hrough effect i ve t herapy. Vi ew Ans wer 14. T he answer is A [VI A 2 b]. The rub i s caus ed by movement of t he i nfl amed pari et al and vi s ceral l ayers of t he peri cardi um. A rub i s i ndi cat i ve of peri cardi t i s from any caus e and does not i mpl y mal i gnancy. Rubs can s t i l l occur even when an effus i on s eparat es t he t wo l ayers of t he peri cardi um. Rubs us ual l y di s appear wi t h effect i ve t herapy. 15. What would be the best first-line therapy for the patient in question 14? A Acet ami nophen B As pi ri n C Ibuprofen
Pa g e 2 2 2
ABC Ambe r CHM Conve rte r Tria l ve rsion, http://w w w .proce sste x t.com/a bcchm.html
D Predni s one E Col chi ci ne Vi ew Ans wer 15. T he answer is C [VI A 4]. Nons t eroi dal ant i -i nfl ammat ory drugs (NSAIDs ) s uch as i buprofen form t he fi rs t l i ne of t herapy. Hi gh-dos e as pi ri n i s effect i ve but i s more l i kel y t o caus e gas t roi nt es t i nal t ract s i de effect s . Al t hough acet ami nophen mi ght rel i eve t he pai n, i t woul d not t reat t he i nfl ammat i on. Predni s one and col chi ci ne are res erved for NSAID fai l ures . 16. Several days later, the patient develops dyspnea and jugular venous distension. T he likely diagnosis now is: A Ri ght -s i ded heart fai l ure B Myocardi al i nfarct i on C Pul monary embol i s m D Peri cardi al t amponade E Pneumoni a Vi ew Ans wer 16. T he answer is D [VI C 2]. The ons et of dys pnea and neck vei n di s t ent i on s houl d i mmedi at el y t ri gger concern for t amponade i n a pat i ent wi t h known peri cardi t i s . As fl ui d bui l ds up i n t he peri cardi al s ac, i t compres s es t he heart , l i mi t s i t s out put , and rai s es t he pres s ure i n al l four cardi ac chambers ; hence t he neck vei n di s t ent i on. W hereas MI, ri ght -s i ded fai l ure, pul monary embol i s m, and pneumoni a are al l pos s i bl e occurrences , t here are no fi ndi ngs t o confi rm t hei r pres ence i n t hi s ot herwi s e heal t hy young woman. A 75-year-ol d man compl ai ns of ches t pai n whi l e cl i mbi ng s t ai rs . On phys i cal exami nat i on, t here i s a II/VI s ys t ol i c eject i on murmur t hat radi at es t o t he neck. The carot i d ups t rokes are del ayed and di mi ni s hed i n vol ume. 17. T he most likely diagnosis is: A Hypert rophi c cardi omyopat hy B Aort i c s t enos i s C Mi t ral s t enos i s D Pul monary s t enos i s E Vas ovagal s yncope
Pa g e 2 2 3
ABC Ambe r CHM Conve rte r Tria l ve rsion, http://w w w .proce sste x t.com/a bcchm.html
Vi ew Ans wer 17. T he answer is B [IV A 3 a–b]. The murmur and delayed carot i d ups t rokes are t ypi cal of t he fi xed LV out fl ow obs t ruct i on of aort i c s t enos i s . Pul monary s t enos i s al s o can caus e ches t pai n and a s ys t ol i c eject i on murmur but woul d not caus e carot i d del ay. Hypert rophi c cardi omyopat hy caus es a s pi ke and dome of t he carot i d ups t rokes ; t hat i s , a s harp ups t roke fol l owed by fal l and a fl at t er s econdary ri s e. The murmur of mi t ral s t enos i s i s di as t ol i c. Al t hough t he s yncope coul d have been at t ri but abl e t o a vas ovagal fai nt , t hi s coul d onl y be a di agnos i s of excl us i on i n t he face of obvi ous aort i c s t enos i s . 18. T he best test to confirm the diagnosis is: A An ECG B An exerci s e s t res s t es t C An echocardi ogram D A radi onucl i de vent ri cul ogram E A ches t x-ray Vi ew Ans wer 18. T he answer is C [IV A 4]. Echocardi ography wi t h Doppl er i nt errogat i on of t he val ve wi l l s how t he aort i c s t enos i s , quant i fy i t s s everi t y, and as s es s l eft vent ri cul ar funct i on. The ECG and ches t x-ray are nons peci fi c i n t hi s di s eas e. Al t hough us eful i n as ympt omat i c pat i ent s , s t res s t es t i ng i s dangerous i n s ympt omat i c aort i c s t enos i s . A radi onucl i de s t udy woul d gi ve i nformat i on about l eft vent ri cul ar funct i on but not about l es i on s everi t y. 19. T he recommended therapy is: A Urgent aort i c val ve repl acement B An ACE i nhi bi t or P.45
C Ni t rogl yceri ne D A cal ci um channel bl ocker E A β-bl ocker
Pa g e 2 2 4
ABC Ambe r CHM Conve rte r Tria l ve rsion, http://w w w .proce sste x t.com/a bcchm.html
Vi ew Ans wer 19. T he answer is A [IV A 5 b]. The onl y accept ed t herapy for s ympt omat i c aort i c s t enos i s i s aort i c val ve repl acement . Ni t rat es can be us ed caut i ous l y for angi na unt i l t he val ve i s repl aced but onl y as a t empori zi ng meas ure. The ot her agent s l i s t ed coul d caus e hypot ens i on and s houl d not be us ed. A murmur i s det ect ed on t he rout i ne exami nat i on of a 35-year-ol d woman. She i s ent i rel y as ympt omat i c and engages i n aerobi c exerci s e cl as s es wi t hout di ffi cul t y. On phys i cal exami nat i on, t here i s II/VI s ys t ol i c eject i on murmur heard bes t i n t he l eft s econd i nt ers pace. S 1 i s normal . S 2 i s wi del y s pl i t and does not vary wi t h res pi rat i on. 20. T he likely diagnosis is: A Pul monary s t enos i s B Aort i c s t enos i s C Vent ri cul ar s ept al defect D At ri al s ept al defect E A fl ow (i nnocent ) murmur Vi ew Ans wer 20. T he answer is D [VII A 3 b]. The wi del y s pl i t S 2 t hat does not vary wi t h res pi rat i on i s pat hognomoni c of at ri al s ept al defect . The murmur i s caus ed by i ncreas ed fl ow acros s t he pul moni c val ve, whi ch i s not s t enot i c. The murmur of a vent ri cul ar s ept al defect i s hol os ys t ol i c. The murmur of aort i c s t enos i s i s as s oci at ed wi t h a s oft s i ngl e S 2 becaus e t he aort i c val ve nei t her opens nor cl os es wel l . 21. Which of the following is true about this abnormality? A Thi s abnormal i t y, when i dent i fi ed, s houl d al ways be repai red. B Al l t ypes of t hi s defect are as s oci at ed wi t h an i ncreas ed ri s k of endocardi t i s . C Uncorrect ed, t hi s defect may l ead t o pul monary hypert ens i on and ri ght heart fai l ure. D Vent ri cul ar arrhyt hmi as are frequent l y as s oci at ed wi t h t hi s abnormal i t y. E The murmur i s due t o t urbul ent fl ow acros s t he defect .
Pa g e 2 2 5
ABC Ambe r CHM Conve rte r Tria l ve rsion, http://w w w .proce sste x t.com/a bcchm.html
Vi ew Ans wer 21. T he answer is C [VII A 3 a]. At ri al s ept al defect s may occur i n di fferent l ocat i ons , and may be of di fferent s i zes . Large, nonres t ri ct i ve defect s may l ead t o pul monary hypert ens i on and event ual ri ght -s i ded heart fai l ure i f l eft uncorrect ed. Not al l at ri al s ept al defect s are as s oci at ed wi t h an i ncreas ed i nci dence of endocardi t i s . Endocardi t i s i s more common i s os t i um pri mum at ri al s ept al defect s due t he as s oci at ed abnormal i t y of t he ant eri or mi t ral val ve (cl eft mi t ral val ve). Os t i um s ecundum defect s , t he mos t common t ype, are not as s oci at ed wi t h an i ncreas ed ri s k. At ri al arrhyt hmi as commonl y occur l at e i n t he nat ural hi s t ory of t he di s eas e, but vent ri cul ar arrhyt hmi as are not commonl y as s oci at ed wi t h at ri al s ept al defect s . The murmur as s oci at ed wi t h an at ri al s ept al defect i s due t o t he rel at i ve i ncreas e i n bl ood fl ow acros s t he pul moni c val ve, and i s not di rect l y rel at ed t o s hunt fl ow acros s t he at ri al s ept al defect . A 50-year-ol d man pres ent s wi t h mi l d dys pnea on exert i on of recent ons et . He was t ol d t hat he had “a murmur― duri ng chi l dhood, but he has not s een a phys i ci an i n many years . On exami nat i on, hi s pul s es are boundi ng and hi s bl ood pres s ure i s 160/60 mm Hg. S 1 i s s oft and S 2 i s normal . There i s a s oft api cal di as t ol i c l ow-pi t ched rumbl e heard at t he apex, and t here i s a di as t ol i c decres cendo murmur heard al ong t he l eft s t ernal border ext endi ng t o S 1 . Ches t x-ray demons t rat es cardi omegal y. 22. T he most likely diagnosis is A Aort i c s t enos i s B Aort i c i ns uffi ci ency C Mi t ral s t enos i s D Mi t ral i ns uffi ci ency E Mi xed aort i c i ns uffi ci ency and mi t ral s t enos i s Vi ew Ans wer 22. T he answer is B [IV C 1-2]. The boundi ng pul s es and wi dened pul s e pres s ure are charact eri s t i c of s i gni fi cant aort i c i ns uffi ci ency and repres ent bot h t he i ncreas ed s t roke vol ume and t he enhanced aort i c “run-off― due t o t he i ncompet ent val ve. Becaus e of
Pa g e 2 2 6
ABC Ambe r CHM Conve rte r Tria l ve rsion, http://w w w .proce sste x t.com/a bcchm.html
ret rograde di as t ol i c fl ow acros s t he aort i c val ve i nt o t he l eft vent ri cl e, t he l eft vent ri cul ar di as t ol i c pres s ure ri s es rapi dl y, l eadi ng t o a nearl y or compl et el y cl os ed mi t ral val ve at t he t i me of vent ri cul ar s ys t ol e, account i ng for t he s oft S 1 . In mi t ral s t enos i s , S 1 i s us ual l y l oud unt i l very l at e s t ages of t he di s eas e. The “precl os ure― of t he mi t ral val ve i s al s o pos s i bl y t he caus e of a rel at i ve mi t ral s t enos i s /di as t ol i c rumbl e, t ermed t he “Aus t i n Fl i nt ― murmur. Cardi omegal y i s pres ent i n t he vol ume overl oaded s t at e of s evere aort i c i ns uffi ci ency. By compari s on, t he l eft vent ri cl e i s prot ect ed from vol ume overl oad wi t h pure mi t ral s t enos i s , and i t i s general l y s mal l i n s i ze. 23. An echocardiogram confirms your clinical suspicions. T he left ventricular function is normal, but the left ventricle is dilated to 6 cm at end-systole. You should recommend which of the following treatments? A Careful t i t rat i on of a β-bl ocker B Aft erl oad reduct i on wi t h an ACE i nhi bi t or C Mi t ral val ve repl acement D Aort i c val ve repl acement E Mi t ral and aort i c val ve repl acement Vi ew Ans wer 23. T he answer is D [IV C 5]. Pat i ent s wi t h s evere aort i c val vul ar i ns uffi ci ency wi t h l eft vent ri cul ar di l at i on t o t hi s degree s houl d be referred for aort i c val ve repl acement . Medi cal t herapy at t hi s poi nt i s more l i kel y t o res ul t i n wors e pos t operat i ve l eft vent ri cul ar funct i on and more s ympt oms of heart fai l ure. The us e of β-bl ockers i n pat i ent s wi t h s i gni fi cant aort i c i ns uffi ci ency i s cont rovers i al , as t hey prol ong t he di as t ol i c i nt erval and may, t herefore, i ncreas e t he regurgi t ant fract i on of bl ood. Aft erl oad reduct i on i n pat i ent s wi t h s i gni fi cant aort i c regurgi t at i on, but wi t hout t hi s degree of vent ri cul ar di l at i on woul d be prudent and can del ay l eft vent ri cul ar di l at i on and compromi s e of funct i on. A 35-year-ol d whi t e woman ent ers t he emergency depart ment compl ai ni ng of epi s odi c ches t pai n t hat us ual l y l as t s for 5–10 mi nut es . Somet i mes i t i s rel at ed t o exerci s e, but on ot her
Pa g e 2 2 7
ABC Ambe r CHM Conve rte r Tria l ve rsion, http://w w w .proce sste x t.com/a bcchm.html
occas i ons i t occurs at res t . The pai n does not radi at e. The woman i s a nons moker and has no hi s t ory of hypert ens i on. Two ot her fami l y members have di ed of heart di s eas e, one at 50 years of age and t he ot her at 56 years of age. On phys i cal exami nat i on, t he pat i ent i s i n no acut e di s t res s . Her bl ood pres s ure i s 120/70 mm Hg and her pul s e i s 70 bpm. Exami nat i on of t he precordi um fi nds t hat t he PMI i s forceful . There i s a II/VI s ys t ol i c eject i on murmur heard al ong t he l eft s t ernal border t hat i ncreas es i n i nt ens i t y when t he pat i ent s t ands up. The ECG s hows nons peci fi c ST-s egment and T-wave abnormal i t i es . P.46
24. Which of the following is the most likely diagnosis? A Innocent fl ow murmur B Aort i c s t enos i s C Hypert rophi c cardi omyopat hy D Mi t ral s t enos i s E Pul moni c s t enos i s Vi ew Ans wer 24. T he answer is C [V B 4]. The mos t l i kel y di agnos i s i s hypert rophi c cardi omyopat hy, as evi denced by t he i ncreas ed i nt ens i t y of t he s ys t ol i c eject i on murmur when t he pat i ent s t ands . W hen a pat i ent wi t h hypert rophi c cardi omyopat hy s t ands , bl ood pool s i n t he l egs , decreas i ng l eft vent ri cul ar s i ze and bri ngi ng t he ant eri or l eafl et of t he mi t ral val ve i n cl os er cont act wi t h t he hypert rophi ed vent ri cul ar s ept um. Thi s i ncreas es t he obs t ruct i on and makes t he murmur l ouder. Convers el y, i nnocent fl ow murmurs and t he murmurs as s oci at ed wi t h pul moni c and aort i c s t enos i s decreas e when t he pat i ent s t ands , becaus e t he t emporary pool i ng of cent ral vol ume i n t he l egs decreas es forward cardi ac out put , t hereby decreas i ng t urbul ent fl ow i n t he val ve. The murmur of mi t ral s t enos i s i s a di as t ol i c murmur, not a s ys t ol i c murmur. 25. Which of the following tools would be best to use when diagnosing this patient?
Pa g e 2 2 8
ABC Ambe r CHM Conve rte r Tria l ve rsion, http://w w w .proce sste x t.com/a bcchm.html
A Ches t radi ograph B Cardi ac cat het eri zat i on C Thal l i um s canni ng D Echocardi ography E Myocardi al bi ops y Vi ew Ans wer 25. T he answer is D [V B 5(b)]. The echocardi ogram i s a hi ghl y effect i ve di agnos t i c t ool i n hypert rophi c cardi omyopat hy, provi ded t he pat i ent can be vi s ual i zed adequat el y. As ymmet ri c hypert rophy of t he s ept um compared wi t h t he free cardi ac wal l confi rms t he di agnos i s . If obs t ruct i on i s pres ent , t here wi l l al s o be s ys t ol i c ant eri or mot i on of t he mi t ral val ve. There are no part i cul ar feat ures of hypert rophi c cardi omyopat hy demons t rabl e on a ches t radi ograph. Thal l i um s ci nt i graphy may s how t he hypert rophi ed s ept um, but t hi s i s not t he opt i mum form of i magi ng. Cardi ac cat het eri zat i on can cert ai nl y confi rm t he di agnos i s , but t hi s i nvas i ve t es t needs t o be performed i n onl y a mi nori t y of pat i ent s when echocardi ography cannot adequat el y vi s ual i ze t he pat i ent 's heart . 26. Which of the following therapies is most appropriate for this patient? A Immedi at e s urgery B A β-bl ocker C Vas odi l at ors D Di goxi n E Furos emi de Vi ew Ans wer 26. T he answer is B [V B 6 a]. Sympt oms of hypert rophi c cardi o myopat hy may be rel i eved wi t h propranol ol , a β-adrenergi c bl ocki ng agent . By decreas i ng heart rat e, propranol ol al l ows i ncreas ed l eft vent ri cul ar fi l l i ng, t hereby i ncreas i ng s eparat i on of t he ant eri or l eafl et of t he mi t ral val ve and t he s ept um and reduci ng t he amount of obs t ruct i on. Unl i ke val vul ar aort i c s t enos i s (where deat h may be i mmi nent aft er t he devel opment of s ympt oms unl es s s urgery i s performed) i n hypert rophi c cardi omyopat hy, t here i s no
Pa g e 2 2 9
ABC Ambe r CHM Conve rte r Tria l ve rsion, http://w w w .proce sste x t.com/a bcchm.html
evi dence t hat s urgery prol ongs l i fe. Bot h di goxi n (by i ncreas i ng t he force of t hat cont ract i on) and furos emi de (by decreas i ng l eft vent ri cul ar s i ze) woul d wors en t he obs t ruct i on and l i kel y exacerbat e t he pat i ent 's s ympt oms . 27. Which of the following is true regarding percutaneous coronary intervention? A There i s no benefi t over t he us e of i nt ravenous t hrombol yt i c agent s for t he t reat ment of acut e myocardi al i nfarct i on. B In pat i ent s wi t h s t abl e angi nal s ympt oms i t provi des s ympt om rel i ef and a mort al i t y benefi t . C St ent s bonded wi t h drugs s uch as s i rol i mus or pacl i t axel have el i mi nat ed t he ri s k of re-s t enos i s . D One t hi rd of pat i ent s who undergo bal l oon angi opl as t y al one wi l l devel op re-s t enos i s wi t hi n 6 mont hs . E Peri procedural admi ni s t rat i on of gl ycoprot ei n IIb/IIIa ant agoni s t s enhances s hort -t erm, but not l ong-t erm pat ency rat es and cl i ni cal s ucces s . Vi ew Ans wer 27. T he answer is (D) [III A 5 a (4) (d)]. The res t enos i s rat e as s oci at ed wi t h bal l oon angi opl as t y al one i s 33% wi t hi n 6 mont hs . St ent s were devel oped, i n part , t o decreas e t he i nci dence of res t enos i s . W hi l e t hey have achi eved t hi s goal , t hey have not el i mi nat ed t he ri s k t ot al l y. The us e of gl ycoprot ei n IIb/IIIa ant agoni s t s has i mproved bot h s hort - and l ong-t erm cl i ni cal s ucces s rat es and angi ographi c pat ency rat es . W hen percut aneous i nt ervent i on i s readi l y acces s i bl e for pat i ent s undergoi ng t reat ment for acut e myocardi al i nfarct i on, t he angi ographi c pat ency rat es and cl i ni cal out comes are bet t er wi t h t he us e of di rect PCI. Pat i ent s wi t h s t abl e angi nal s ympt oms may have s ympt omat i c benefi t from PCI, but do not enjoy a mort al i t y benefi t rel at ed t o t he procedure.
Pa g e 2 3 0
ABC Ambe r CHM Conve rte r Tria l ve rsion, http://w w w .proce sste x t.com/a bcchm.html
Editors: Wolfsthal, Susan T itle: NMS Medicine, 6th Edition Copyri ght ©2008 Li ppi ncot t W i l l i ams & W i l ki ns > T able of Cont ent s > Chapt er 2 - Pulmonary Diseases
Chapter 2
Pulmonary Diseases Alexandra Pratt Pamela J. Amelung
I. Pulmonary Function Studies (
Onl i ne Fi gures 2-1, 2-2, 2-3, 2-4, 2-5, 2-6; Onl i ne Tabl e 2-1)
ONLINE FIGURE 2-1. Typi cal fl ow-vol ume l oop (obs erved duri ng s pi romet ry) i n (A) a s ubject wi t h normal l ung funct i on, and i n s ubject s wi t h (B) obs t ruct i ve or (C) res t ri ct i ve l ung di s eas e. FEF, forced expi at ory fl ow; TLC, t ot al l ung capaci t y; VC, vi t al capaci t y; RV, res i dual vol ume.
Pa g e 2 3 1
ABC Amber CHM Converter Trial version, http://www.processtext.com/abcchm.html
ONLINE FIGURE 2-2. The s ubdi vi s i ons of l ung vol ume as recorded by a s pi romet er. The record i s generat ed on paper cal i brat ed for vol ume i n t he vert i cal di rect i on and t i me i n t he hori zont al . The t erm capacity i s appl i ed t o a s ubdi vi s i on compos ed of t wo or more vol umes .
Page 232
ABC Ambe r CHM Conve rte r Tria l ve rsion, http://w w w .proce sste x t.com/a bcchm.html
ONLINE FIGURE 2-3. Pres s ure–vol ume curves of t he l ung. The curves for emphys ema and as t hma (duri ng bronchos pas m) are s hi ft ed upward and t o t he l eft , whereas t hos e for rheumat i c val ve di s eas e and i nt ers t i t i al fi bros i s are fl at t ened. (Repri nt ed wi t h permi s s i on from Bat es DV, Mackl em PT, Chri s t i e RV. Res pi rat ory Funct i on i n Di s eas e, 2nd ed. Phi l adel phi a: W B Saunders , 1971:9. )
Pa g e 2 3 3
ABC Amber CHM Converter Trial version, http://www.processtext.com/abcchm.html
ONLINE FIGURE 2-4 Exampl es of vent i l at i on perfus i on and t he qual i t y: (A) normal i deal i zed al veol ar capi l l ary exchange uni t ; (B) exampl es of decreas ed vent i l at i on–perfus i on uni t s , as a res ul t of al veol ar s ecret i ons or ai rway obs t ruct i on; (C) exampl es of i ncreas ed vent i l at i on–perfus i on uni t s owi ng t o t he pres ence of emphys ema or probl ems i n t he pul monary vas cul ar bed, s uch as pul monary embol i s m or pul monary vas os pas m. (Repri nt ed wi t h permi s s i on from Cri ner GJ, D'Al onzo GE, eds .: Pul monary Pat hophys i ol ogy, Fence Creek Publ i s hi ng, LLC: Madi s on, Connect i cut , 1999. )
Page 234
ABC Amber CHM Converter Trial version, http://www.processtext.com/abcchm.html
ONLINE FIGURE 2-5. Exampl es of i nt rapul monary s hunt : (A) col l aps e of fl ui d-fi l l ed al veol us ; (B) t he effect of anomal ous bl ood ret urn of mi xed bl ood bypas s i ng t he al veol us , and cont ri but i ng t o i nt rapul monary s hunt . (Repri nt ed wi t h permi s s i on from Cri ner GE, D'Al onzo GE, eds .: Pul monary Pat hophys i ol ogy, Fence Creek Publ i s hi ng, LLC: Madi s on, Connect i cut , 1999. )
ONLINE FIGURE 2-6. Res pons e of vent i l at i on–perfus i on i nequal i t y and i nt rapul monary s hunt t o s uppl ement al oxygen: (A) Even wi t h t he pres ence of s evere [V wi t h dot above]/[Q wi t h
Page 235
ABC Ambe r CHM Conve rte r Tria l ve rsion, http://w w w .proce sste x t.com/a bcchm.html
dot above] i nequi t y, hi gh l evel s of s uppl ement al oxygen have a profound i mpact on i ncreas i ng art eri al PO 2 ; (B) By cont ras t , i nt rapul monary s hunt s of 30% or great er are rel at i vel y refract ory t o s uppl ement al oxygen. (Adapt ed from Dant z ker MD, Davi d R. Pul monary Gas Exchange, i n Bone R. ed., Pul monary and Cri t i cal Care Medi ci ne. St . Loui s : Mos by, 1997. )
A. Introduction Tes t s of pul monary funct i on provi de t hree bas i c ki nds of i nformat i on:
1. Lung volumes are t he vol umes of t he vari ous i nt rapul monary compart ment s . o
o
a. Static lung volumes refl ect t he el as t i c propert i es of t he l ungs and ches t wal l .
o
o
b. Dynamic lung volumes refl ect t he pat ency of t he ai rways .
2. The expiratory flow rate i s t he maxi mum rat e of ai rfl ow duri ng forced expi rat i on. o
o
a. The rat e of ai rfl ow i s i nfl uenced by l ung vol ume and by effort (i .e., force of expi rat i on). Ai rfl ow i ncreas es wi t h i ncreas i ng effort , es peci al l y at hi gh l ung vol umes [>75% of t he vi t al capaci t y (VC)].
o
o
b. Ot her fact ors i nfl uenci ng fl ow rat e i ncl ude t he el as t i c recoi l of t he l ung, s mal l peri pheral ai rway res i s t ance, and t he cros s -s ect i onal area of l arger
Pa g e 2 3 6
ABC Ambe r CHM Conve rte r Tria l ve rsion, http://w w w .proce sste x t.com/a bcchm.html
cent ral ai rways .
3. Diffusing capacity of the lung for carbon monoxide (D L C O ) i s t he effi ci ency of gas t rans fer from t he al veol i t o pul monary capi l l ary bl ood.
B. Spirometry
1. Definition. Spi romet ry i s a s i mpl e t es t of pul monary funct i on. The s pi romet er devi ce pl ot s a t raci ng (t he spirogram) of t he l ung vol ume agai ns t t i me (i n s econds ) whi l e t he pat i ent t akes as deep a breat h as pos s i bl e and t hen exhal es al l of t he i ns pi red ai r as rapi dl y and forceful l y as pos s i bl e.
2. Uses. Spi romet ry can ai d i n di s t i ngui s hi ng obs t ruct i ve from res t ri ct i ve l ung di s eas es (onl i ne Tabl e 2-1) as wel l as s ugges t t he s everi t y of funct i onal i mpai rment and i t s revers i bi l i t y wi t h t reat ment . It i s us eful bot h as a di agnos t i c ai d and as a moni t ori ng t ool .
ONLINE TABLE 2-1 Obstructive and Restrictive Lung Disorders
Pa g e 2 3 7
ABC Ambe r CHM Conve rte r Tria l ve rsion, http://w w w .proce sste x t.com/a bcchm.html
Pri ma rily ob str uct ive ve ntil ato ry def ect s (di s or der s cha rac t eri z ed by red uce d fl o w rat es ) Chr oni c obs t ru
Pa g e 2 3 8
ABC Ambe r CHM Conve rte r Tria l ve rsion, http://w w w .proce sste x t.com/a bcchm.html
ct i ve pul mo nar y di s eas e Chr oni c bro nch itis Pul mo nar y em phy se ma As t hm a Cys tic fi br os i s Pri ma rily res
Pa g e 2 3 9
ABC Ambe r CHM Conve rte r Tria l ve rsion, http://w w w .proce sste x t.com/a bcchm.html
tric tiv e ve ntil ato ry def ect s (di s or der s cha rac t eri z ed by red uce d l un g vol um es ) Par enc hy ma l di s ord ers
Pa g e 2 4 0
ABC Ambe r CHM Conve rte r Tria l ve rsion, http://w w w .proce sste x t.com/a bcchm.html
Al v eol ar an d i nt ers titi al pro ces s es (e. g., ed em a, fi br os i s, i nf ect i on ) Lar ge s pa ceocc upy i ng l es i on s At e
Pa g e 2 4 1
ABC Ambe r CHM Conve rte r Tria l ve rsion, http://w w w .proce sste x t.com/a bcchm.html
l ec t as is Re s ec tio n of pul mo nar y tis s ue Ch es t wal l di s eas e Ob es i ty Ky ph os c ol i os i s An kyl os i ng s po
Pa g e 2 4 2
ABC Ambe r CHM Conve rte r Tria l ve rsion, http://w w w .proce sste x t.com/a bcchm.html
ndy liti s Pl e ura l di s eas e Eff us i on Pn eu mo t ho rax Fi b rot hor ax Ne uro mu s cu l ar di s eas es Gui l l ai n Bar rà ©
Pa g e 2 4 3
ABC Ambe r CHM Conve rte r Tria l ve rsion, http://w w w .proce sste x t.com/a bcchm.html
s yn dro me Spi nal cor d i nj ury Mu s cu l ar dys t ro phi es Pol io my el i t is
C. Values obtainable from the spirogram Many s pi rogram meas urement s are s t at ed as a percent age of predicted values t hat are det ermi ned from many normal i ndi vi dual s grouped on t he bas i s of s ex, age, and hei ght . The range of normal i s 80%–120% of t he predi ct ed val ue.
1. T idal volume (VT ) i s t he vol ume of ai r i n one breat h duri ng normal qui et breat hi ng. The VT (normal , 500–800 mL) vari es accordi ng t o effort and l evel of vent i l at i on. The port i on of t he VT t hat part i ci pat es i n gas exchange i s t he alveolar volume (VA); t he remai n der, approxi mat el y 30% of t he VT, i s “was t ed― or dead space.
Pa g e 2 4 4
ABC Ambe r CHM Conve rte r Tria l ve rsion, http://w w w .proce sste x t.com/a bcchm.html
2. Vital capacity (VC) i s t he maxi mum vol ume of ai r t hat can be expel l ed from t he l ungs aft er a maxi mal i ns pi rat i on. Becaus e VC decreas es progres s i vel y wi t h res t ri ct i ve l ung di s eas e, i t i s us eful , i n conjunct i on wi t h D LC O (s ee I E 1), for moni t ori ng t he cours e and res pons e t o t herapy i n a pat i ent wi t h a res t ri ct i ve l ung di s order.
3. Forced vital capacity (FVC) i s t he s ame as VC, except t hat t he i nhal at i on i s performed as rapi dl y and forceful l y as pos s i bl e. Forced expi rat i on caus es t he ai rways t o narrow, t hereby s l owi ng t he rat e of expi rat i on.
4. Forced expiratory volume in 1 second (FEV 1 ) i s t he vol ume of ai r forceful l y expi red duri ng t he fi rs t s econd aft er a deep breat h, or t he port i on of t he FVC exhal ed i n 1 s econd. The FEV 1 pri mari l y refl ect s t he s t at us of l arge ai rways .
5. FEV 1 /FVC i s t he rat i o of t he FEV 1 t o FVC, expres s ed as a percent age (normal , ≥70%). (Not e: In t hi s cas e, percent age i s not a percent age of predi ct ed normal .) The FEV 1 /FVC i s effort dependent (i .e., i t i ncreas es wi t h i ncreas i ng expi rat ory effort ). FEV 1 /FVC i s part i cul arl y us eful i n eval uat i ng obs t ruct i ve di s orders but i s al s o hel pful i n t he eval uat i on of res t ri ct i ve di s orders . If onl y t he FEV 1 i s l ow (FEV 1 /FVC <70%), obs t ruct i on i s s ugges t ed; i f bot h t he FEV 1 and FVC are l ow (FEV 1 /FVC ≥70%), res t ri ct i on i s s ugges t ed.
6. FEF 2 5 % â € “ 7 5 % i s t he forced expiratory flow rat e over
Pa g e 2 4 5
ABC Ambe r CHM Conve rte r Tria l ve rsion, http://w w w .proce sste x t.com/a bcchm.html
t he mi ddl e hal f of t he FVC (i .e., bet ween 25% and 75%); i t i s al s o cal l ed t he maximal mid-expiratory flow rate (MMEFR or MMFR). The FEF 2 5 % â € “ 7 5 % pri mari l y refl ect s t he s t at us of t he s mal l ai rways , and i t i s more s ens i t i ve t han t he FEV 1 for i dent i fyi ng earl y ai rway obs t ruct i on. The FEF 2 5 % â € “ 7 5 % i s effort independent. The FEF 2 5 % â € “ 7 5 % has a wi de range of normal i t y, and i s l es s reproduci bl e t han FEV 1 .
7. Becaus e s pi romet ry cannot meas ure res i dual vol ume (RV), i t can not meas ure any l ung vol ume cont ai ni ng RV [e.g., RV, funct i onal res i dual capaci t y (FRC), t ot al l ung capaci t y (TLC)] (
Onl i ne Fi gure
2-1)
D. Other lung volumes (
onl i ne Fi gure 2-2). Obt ai ni ng t he fol l owi ng val ues requi res t he
us e of s pi romet ry and ei t her hel i um di l ut i on (whi ch meas ures t he vol ume of gas i n t he l ungs ) or, preferabl y, body pl et hys mography (whi ch meas ures i nt rat horaci c gas vol ume).
1. T otal lung capacity (T LC) i s t he vol ume of ai r i n t he l ungs aft er a maxi mal i ns pi rat ory effort .
2. Functional residual capacity (FRC) i s t he vol ume of ai r remai ni ng i n t he l ungs at t he end of a normal expi rat i on. The FRC refl ect s t he res t i ng pos i t i on of t he l ungs and ches t wal l ; i t i s t he l ung vol ume at whi ch t he i nward recoi l of t he l ungs i s bal anced by t he out ward recoi l of t he ches t wal l . The FRC has t wo component s : o
o
a. The expiratory reserve volume (ERV) i s t he amount of t he FRC t hat can be expel l ed by a maxi mal expi rat ory effort .
Pa g e 2 4 6
ABC Ambe r CHM Conve rte r Tria l ve rsion, http://w w w .proce sste x t.com/a bcchm.html o
o
b. The RV i s t he vol ume of ai r remai ni ng i n t he l ungs aft er a maxi mal expi rat ory effort (normal , 25%–30% of TLC).
3. Lung volume relationships are as fol l ows : o
o
a. TLC = VC + RV
o
o
b. RV = FRC + ERV
E. Other tests of pulmonary function
1. Diffusing capacity of the lung for carbon monoxide (D L C O ). The D LC O i ndi cat es t he adequacy of t he al veol ar–capi l l ary membrane. o
o
a. The D LC O i s det ermi ned by meas uri ng t he amount of carbon monoxi de (CO) t rans ferred from t he al veol ar gas t o t he pul monary capi l l ary bl ood aft er t he pat i ent i nhal es a known amount of CO (0.1%); i t i s expres s ed i n mL/mi n/mm Hg.
o
o
b. The D LC O has a number of us es . In obs t ruct i ve di s eas e, i t hel ps di s t i ngui s h bet ween as t hma/chroni c bronchi t i s and emphys ema, and can i ndi cat e t he s everi t y of emphys ema. In res t ri ct i ve di s eas e, t he D LC O can di s t i ngui s h bet ween i nt ers t i t i al l ung di s eas e and neuromus cul ar di s eas e.
Pa g e 2 4 7
ABC Ambe r CHM Conve rte r Tria l ve rsion, http://w w w .proce sste x t.com/a bcchm.html o
o
c. The effect s of vari ous di s orders on t he D LC O are s hown i n onl i ne Tabl e 2-2.
ONLINE TABLE 2-2 Effect of Various Disorders on Diffusing Capacity (D L C O ) Dis or de rs tha t de cre as e D LC O
Em phy se ma Int ers titi al fi br os i s Mul t i pl e pul mo
Pa g e 2 4 8
ABC Ambe r CHM Conve rte r Tria l ve rsion, http://w w w .proce sste x t.com/a bcchm.html
nar y em bol i Pul mo nar y ed em a Sar coi dos is Pul mo nar y al v eol ar pro t ei nos is Pul mo nar y res ect i on An
Pa g e 2 4 9
ABC Ambe r CHM Conve rte r Tria l ve rsion, http://w w w .proce sste x t.com/a bcchm.html
em ia Dis or de rs tha t inc re as e D LC O
Pul mo nar y he mo rrh ag e Int rac ard i ac l eft -t o -ri g ht s hu nt Va s cu l ar
Pa g e 2 5 0
ABC Ambe r CHM Conve rte r Tria l ve rsion, http://w w w .proce sste x t.com/a bcchm.html
con ges tio n, but onl y pri or to ed em a Pol ycy t he mi a ver a (ea rl y) As t hm a CO , car bo n mo nox i de ; RB
Pa g e 2 5 1
ABC Ambe r CHM Conve rte r Tria l ve rsion, http://w w w .proce sste x t.com/a bcchm.html
Cs , red bl o od cel l s.
2. Compliance curve. The elastic properties of the lung can be as s es s ed from t he rel at i ons hi p bet ween change i n l ung vol ume and change i n t rans pul monary pres s ure (t he pres s ure i n t he al veol i mi nus t he pres s ure wi t hi n t he pl eural s pace; PA - PPL). o
o
a. A gi ven vol ume of ai r i n t he l ung requi res a cert ai n amount of pres s ure t o achi eve t hat degree of i nfl at i on. Thi s pres s ure i s a combi nat i on of t he el as t i c (i nward) recoi l of t he l ung and t he el as t i c (out ward) recoi l of t he ches t wal l .
(1) At t he normal res t i ng end-expi rat ory pos i t i on of t he l ungs (i .e., at FRC), t he el as t i c recoi l of t he l ung i s exact l y bal anced by t he el as t i c recoi l of t he ches t wal l .
(2) At ful l i ns pi rat i on (i .e., at TLC), t he l ungs reach t hei r maxi mal el as t i c recoi l .
Pa g e 2 5 2
ABC Ambe r CHM Conve rte r Tria l ve rsion, http://w w w .proce sste x t.com/a bcchm.html
(3) At ful l expi rat i on (i .e., at RV), t he ches t wal l reaches i t s maxi mal el as t i c recoi l , unl es s age or ai rway di s eas e caus es t he ai rways t o cl os e premat urel y, t rappi ng gas wi t hi n t he l ungs .
o
o
b. Pl ot t i ng t he l ung vol ume agai ns t a range of t rans pul monary pres s ures gi ves t he compliance curve for t he l ung. Compliance (C) i s det ermi ned from t he s l ope of t he pres s ure–vol ume (P–V) curve over t he t i dal vol ume range. It i s defi ned as t he change i n vol ume for any gi ve change i n pres s ure, C = ΔV/ΔP (normal , 200 mL/cm H 2 O) (onl i ne Fi gure 2-3).
o
o
c. Changes i n l ung el as t i c recoi l i nvers el y affect t he compl i ance curve.
(1) Los s of l ung el as t i c recoi l (e.g., i n emphys ema) i ncreas es compl i ance, s hi ft i ng t he compl i ance curve t o t he l eft .
(2) Increas ed l ung el as t i c recoi l (e.g., i n i nt ers t i t i al l ung di s eas es s uch as i di opat hi c pul monary fi bros i s ) decreas es compl i ance, s hi ft i ng t he compl i ance curve downward and t o t he ri ght .
3. Airway resistance (Raw). Meas uri ng Raw pri mari l y refl ect s t he s t at us of l arge ai rways , becaus e 80%–90% of t he res i s t ance t o ai rfl ow i s i n t he l arge cent ral ai rways .
Pa g e 2 5 3
ABC Ambe r CHM Conve rte r Tria l ve rsion, http://w w w .proce sste x t.com/a bcchm.html o
o
a. Raw i s us ual l y det ermi ned from dynami c l ung vol umes and expi rat ory fl ow rat es ; when a more accurat e meas ure i s needed, body pl et hys mography i s us ed.
o
o
b. Raw i s i ncreas ed i n obs t ruct i ve l ung di s eas e and decreas ed i n res t ri ct i ve l ung di s eas e.
F. Patterns of pulmonary function impairment
1. Obstructive lung disorders o
o
a. Flow rates. A reduced FEV 1 /FVC (<70%) i s t he t i me-honored i ndi cat or of obs t ruct i ve ai rway di s eas e. However, t he FEV 1 /FVC may be normal even wi t h cons i derabl e peri pheral ai rway obs t ruct i on. A reduced FEF 2 5 % â € “ 7 5 % (60% or l es s of predi ct ed val ue) may det ect ai rway obs t ruct i on when t he FEV 1 /FVC i s normal . However, t he range of normal FEF 2 5 % â € “ 7 5 % val ues i s wi de.
o
o
b. Lung volumes. Changes i n l ung vol ume may be s een i n moderat e-t o-s evere obs t ruct i ve ai rway di s eas e.
(1) Lung vol ume meas urement s are us eful i n i dent i fyi ng hyperi nfl at i on caus ed by premat ure ai rway cl os ure.
Pa g e 2 5 4
ABC Ambe r CHM Conve rte r Tria l ve rsion, http://w w w .proce sste x t.com/a bcchm.html
(a) Duri ng a forced expi rat i on, i f t he t ermi nal ai rways cl os e before al l t he ai r i s expel l ed, hyperi nfl at i on res ul t s , caus i ng an i ncreas e i n t he FRC, RV, and RV/TLC.
(b) In s mal l ai rway di s orders , becaus e of ai r t rappi ng, t he RV may i ncreas e whi l e t he FRC and FEV 1 remai n normal .
(2) In emphys ema, t he al veol ar wal l des t ruct i on and l os s of l ung el as t i c recoi l caus e an i ncreas e i n t he TLC.
o
o
c. Compl i ance i s i ncreas ed i n emphys ema, becaus e l ung el as t i c recoi l i s reduced.
o
o
d. Raw i s i ncreas ed i n obs t ruct i ve l ung di s eas e.
2. Restrictive lung disorders o
o
a. Flow rates. FEV 1 /FVC and FEF 2 5 % â € “ 7 5 % may be normal or i ncreas ed becaus e of i ncreas ed t ract i on on t he i nt rat horaci c ai rway wal l s .
o
o
b. Lung volumes
Pa g e 2 5 5
ABC Ambe r CHM Conve rte r Tria l ve rsion, http://w w w .proce sste x t.com/a bcchm.html
(1) A reduction in VC and T LC i s t he mos t us eful i ndi cat or of a res t ri ct i ve vent i l at ory defect .
(2) Lung s t i ffnes s i n res t ri ct i ve di s eas es i ncreas es t he l ung el as t i c recoi l , t hus l oweri ng t he FRC.
(3) Ches t wal l s t i ffnes s (e.g., i n kyphos col i os i s ) l owers l ung vol umes becaus e i t res t ri ct s l ung expans i on.
o
o
c. Compliance i s reduced becaus e l ung el as t i c recoi l i s i ncreas ed.
o
o
d. Raw i s decreas ed becaus e t he el as t i c forces mai nt ai n wi der ai rways at any l ung vol ume.
G. Arterial blood gases The part i al art eri al pres s ures of oxygen (PaO 2 ) and carbon di oxi de (PaCO 2 ) as wel l as t he pH of art eri al bl ood are i mport ant i n as s es s i ng pul monary funct i on, becaus e t hes e dat a i ndi cat e t he s t at us of gas exchange bet ween t he l ungs and t he bl ood.
1. PaO 2 and PaCO 2 . The PaO 2 and PaCO 2 s how t he net effect of l ung di s eas e on gas exchange. o
o
a. The PaO 2 normal l y decreas es wi t h age as a res ul t of t he l os s of l ung el as t i ci t y (normal PaO 2 i s approxi mat el y 90 mm Hg at age 20 years and approxi mat el y 75 mm Hg by age 70 years ). A l ower
Pa g e 2 5 6
ABC Ambe r CHM Conve rte r Tria l ve rsion, http://w w w .proce sste x t.com/a bcchm.html
t han normal PaO 2 i ndi cat es hypoxemi a. However, t i s s ue oxygenat i on i s not s i gni fi cant l y reduced unt i l t he PaO 2 decreas es t o l es s t han approxi mat el y 60 mm Hg. o
o
b. The PaCO 2 (normal , 35–45 mm Hg) refl ect s al veol ar vent i l at i on—hypercapnia (res pi rat ory aci dos i s , a hi gh PaCO 2 ) i ndi cat es hypoventilation, and hypocapnia (res pi rat ory al kal os i s , a l ow PaCO 2 ) i ndi cat es hyperventilation.
2. pH o
o
a. Compari ng t he art eri al pH (normal , 7.35–7.45) wi t h t he PaCO 2 hel ps di s t i ngui s h res pi rat ory from met abol i c abnormal i t i es . For exampl e, i f t he PaCO 2 and pH are rel at ed inversely (one decl i ni ng whi l e t he ot her i ncreas es ), t he aci d–bas e i mbal ance i s res pi rat ory.
o
o
b. Cl i ni cal l y, however, res ul t s oft en are not s t rai ght forward, bei ng compl i cat ed by t he nat ure of t he pat i ent 's di s orders and any medi cat i ons bei ng t aken.
H. Ventilation–perfusion ([V with dot above]/[Q with dot above]) mismatch (inequality of pulmonary gas exchange)
1. [V with dot above]/[Q with dot above]
Pa g e 2 5 7
ABC Ambe r CHM Conve rte r Tria l ve rsion, http://w w w .proce sste x t.com/a bcchm.html
relationships o
o
a. Vent i l at i on and bl ood fl ow t hrough t he l ung s houl d mat ch i f t here i s t o be adequat e upt ake of oxygen and adequat e el i mi nat i on of carbon di oxi de (onl i ne Fi gure 2-4).
o
o
b. The overal l [V with dot above]/[Q with dot above] of t he l ung i s 0.8. Thus , t here i s a normal “ physiologic [V with dot above]/[Q with dot above] mismatch,― equi val ent t o a 2%–8% shunting of pul monary art eri al (mi xed venous ) bl ood di rect l y i nt o t he pul monary venous ci rcul at i on wi t hout gas exchange.
2. Significance of [V with dot above]/[Q with dot above] mismatch o
o
a. A low [V with dot above]/[Q with dot above] s i gni fi es i nadequat e vent i l at i on of an adequat el y perfus ed area of t he l ung. The res ul t i s a l owered PaO 2 or hypoxemi a.
(1) Unl es s t he al veol i are occl uded or fl ui d fi l l ed, t he hypoxemi a can be correct ed by admi ni s t eri ng oxygen, becaus e t he oxygen reaches t he areas of al veol ar hypoxi a.
(2) If an area of t he l ung has no alveolar ventilation, i t has no gas exchange at al l ([V with dot above]/[Q with dot above] = 0)
Pa g e 2 5 8
ABC Ambe r CHM Conve rte r Tria l ve rsion, http://w w w .proce sste x t.com/a bcchm.html
(onl i ne Fi gure 2-5). The res ul t i s right-to-left shunting of bl ood; t hat i s , t rue venous bl ood mi xes wi t h art eri al bl ood. Thi s form of hypoxemi a i s refract ory t o oxygen t herapy becaus e t he oxygen cannot reach t he al veol ar–capi l l ary membrane (onl i ne Fi gure 2-6). o
o
b. A high [V with dot above]/[Q with dot above] s i gni fi es adequat e vent i l at i on of a poorl y perfus ed area of t he l ung. Oxygen exchange i s i neffi ci ent becaus e t he avai l abl e hemogl obi n can onl y t ake up s o much oxygen.
(1) If an area of t he l ung has no blood flow at al l , i t has no gas exchange at al l ([V with dot above]/[Q with dot above] = 0). Al l of t he oxygen goi ng t o t hat area of al veol ar dead s pace i s was t ed vent i l at i on.
(2) Al veol ar dead s pace res ul t s i n carbon di oxi de ret ent i on as wel l as hypoxi a. The hypercapni a s t i mul at es t he res pi rat ory cent er, t hereby i ncreas i ng t he work of breat hi ng but al s o i ncreas i ng vent i l at i on. Even t hough t hi s may normal i ze t he PaCO 2 , i t does not i mprove t he l owered PaO 2 .
o
o
c. Typi cal l y, i n pat i ent s wi t h [V wi t h dot above]/[Q wi t h dot above] abnormal i t i es , t he reduct i on i n PaO 2
i s much more marked t han t he i ncreas e i n PaCO 2 .
However, as l ung di s eas e progres s es , vent i l at i on
Pa g e 2 5 9
ABC Ambe r CHM Conve rte r Tria l ve rsion, http://w w w .proce sste x t.com/a bcchm.html
cannot i ncreas e any furt her. The res ul t i s hypoxemi a and hypercapni a (i .e., acut e res pi rat ory fai l ure).
3. Alveolar–arterial oxygen gradient (A–a DO 2 ) i s t he di fference (i .e., gradi ent ) bet ween t he alveolar PO 2 (PAO 2 ) and t he arterial PO 2 (PaO 2 ). An increase in the A–a DO 2 reflects worsening of [V with dot above]/[Q with dot above] matching. o
o
a. Cal cul at i ng t he A–a DO 2 may hel p di s t i ngui s h hypovent i l at i on from ot her caus es of hypoxemi a and i ndi cat e t he s everi t y of l ung di s eas e.
(1) The PaO 2 i s cal cul at ed from t he s i mpl i fi ed al veol ar gas equat i on: PaO 2 = FIO 2 (P B – PH 2 O) – PaCO 2 /R where FIO 2 = t he fract i on (percent age) of oxygen i n t he i ns pi red ai r (0.21 mm Hg for room air at s ea l evel ); P B = at mos pheri c pres s ure (760 mm Hg); PH 2 O = part i al pres s ure of wat er vapor (47 mm Hg—i ncl uded i n t he formul a becaus e i ns pi red ai r i s s at urat ed wi t h wat er vapor); and R = res pi rat ory exchange rat i o (0.8).
(2) The PaO 2 i s det ermi ned di rect l y from a bl ood s ampl e.
(3) The A–a DO 2 is therefore: A–a DO 2 = FIO 2 (P B – PH 2 O) – PaCO 2 /R –
Pa g e 2 6 0
ABC Ambe r CHM Conve rte r Tria l ve rsion, http://w w w .proce sste x t.com/a bcchm.html
PaO 2
o
o
b. The PaO 2 i s normal l y 5–15 mm Hg l ower t han t he PAO 2 becaus e of a normal degree of phys i ol ogi c [V wi t h dot above]/[Q wi t h dot above] mi s mat ch. Thus , t he A–a DO 2 i s normal l y l es s t han 15 mm Hg and i ncreas es wi t h age.
II. Chronic Obstructive Pulmonary Disease (Copd) A. Introduction
1. Definition. COPD (chroni c obs t ruct i ve pul monary di s eas e) i s defi ned as a di s eas e s t at e charact eri zed by t he pres ence of ai rfl ow obs t ruct i on caus ed by chroni c bronchi t i s or emphys ema. The ai rfl ow obs t ruct i on i s general l y progres s i ve, may be accompani ed by ai rway react i vi t y, and may be part i al l y revers i bl e (Fi gure 2-7). COPD i s a common di s order us ual l y charact eri zed by progres s i ve obs t ruct i on t o ai rfl ow and a hi s t ory of i nhal at i on of i rri t ant s (e.g., t obacco s moke). COPD al s o i s referred t o as chronic obstructive lung disease (COLD), chronic airway obstruction (CAO), and, ei t her i ndi vi dual l y or t oget her, as chroni c bronchi t i s and emphys ema. o
o
a. Chronic bronchitis may be defi ned i n t erms of cl i ni cal s ympt oms (i .e., exces s i ve s ecret i on of mucus i n t he bronchi al t ree l eadi ng t o product i ve
Pa g e 2 6 1
ABC Ambe r CHM Conve rte r Tria l ve rsion, http://w w w .proce sste x t.com/a bcchm.html
cough for at l eas t 3 mont hs duri ng each of t wo s ucces s i ve years ). o
o
b. Emphysema can be des cri bed i n t erms of anat omy (i .e., des t ruct i on of al veol ar wal l s and abnormal enl argement of ai r s paces di s t al t o t he t ermi nal nonres pi rat ory bronchi ol e).
o
o
c. Many pat i ent s have, i n varyi ng degrees , a combi nat i on of t hes e t wo ent i t i es . Therefore, t he more general t erm COPD i s appropri at e.
2. The social and economic consequences of COPD are s t aggeri ng. Fourt een mi l l i on Ameri cans have COPD. Approxi mat el y 18 bi l l i on dol l ars are expended annual l y i n heal t h care dol l ars t o t reat COPD pat i ent s ; anot her $9.9 bi l l i on are l os t annual l y becaus e of decreas ed work product i vi t y.
3. The death rate at t ri but abl e t o COPD has doubl ed every 5 years i n t he l as t t wo decades . COPD i s t he fourt h l eadi ng caus e of deat h i n t he Uni t ed St at es . Unl i ke many ot her di s eas es (coronary heart di s eas e, s t roke, and ot her cardi ovas cul ar di s eas es ) for whi ch t here has been a 20%–60% decl i ne i n mort al i t y rat e over t he l as t 20 years , t he mort al i t y rat e of COPD has i ncreas ed by 70% over t he s ame peri od.
B. Etiology The preci s e s ci ent i fi c et i ol ogy of COPD i s unknown; however, t he mos t i mport ant envi ronment al et i ol ogi c agent i s chroni c i nhal at i on of t obacco s moke (Fi gure 2-8). Premat ure bi rt h, poor l ung mat urat i on, l ower s oci oeconomi c s t at us , and ai rways
Pa g e 2 6 2
ABC Ambe r CHM Conve rte r Tria l ve rsion, http://w w w .proce sste x t.com/a bcchm.html
hyperreact i vi t y are bel i eved t o be addi t i onal i mport ant fact ors .
1. T obacco smoke. Smoki ng i n pack-years (t he number of years a pat i ent has s moked × t he number of packs s moked per day) i s di rect l y rel at ed t o vent i l at ory dys funct i on and pat hol ogi c changes i n t he l ung. Smoki ng s t i mul at es i nfl ammat ory cyt oki nes and depres s es al veol ar macrophages , reduces t he funct i onal i nt egri t y of pul monary s urfact ant , ret ards t rans port of mucus , enhances t he rel eas e of l ys os omal enzymes , and produces numerous ot her effect s bel i eved t o be i nvol ved i n t he pat hogenes i s of COPD. P.52
FIGURE 2-7. Exampl es of normal and COPD s mal l ai rways . Normal peri pheral ai rways have no evi dence of i nfl ammat i on or gobl et cel l hypert rophy, and t he ai rways are hel d open by al veol ar at t achment s . By cont ras t , i n chroni c obs t ruct i ve pul monary di s eas e, a l os s of al veol ar at t achment s , due t o emphys ema, cont ri but es t o s mal l ai rway cl os ure. In addi t i on, mucous hypers ecret i on, and peri bronchi al i nfl ammat i on and fi bros i s cont ri but e t o obl i t erat i ve bronchi ol i t i s and l umi nal obs t ruct i on. (Repri nt ed wi t h permi s s i on from
Pa g e 2 6 3
ABC Ambe r CHM Conve rte r Tria l ve rsion, http://w w w .proce sste x t.com/a bcchm.html
Barnes P. Chroni c obs t ruct i ve pul monary di s eas e. N Engl J Med 2000;3(43)269–280. )
2. Other environmental factors o
o
a. The ext ent t hat urban and i ndus t ri al air pollution cont ri but es t o t he pat hogenes i s of COPD i s not ent i rel y known. St udi es compari ng pat i ent s who l i ve i n areas of cl ean ai r wi t h t hos e who res i de i n hi ghl y pol l ut ed areas i ndi cat e a hi gher i nci dence of COPD i n t he l at t er group.
o
o
b. The rol e of infections i n t he devel opment and progres s i on of COPD i s recei vi ng i ncreas i ng at t ent i on. Evi dence s ugges t s t hat upper res pi rat ory viral infections i n chi l dhood may predi s pos e pat i ent s t o COPD i n adul t hood, and vi ral i nfect i ons are frequent preci pi t at i ng fact ors i n s ympt omat i c exacerbat i ons of COPD. Bacterial infections al s o have been report ed t o be i nvol ved i n acut e exacerbat i on of chroni c bronchi t i s .
3. α 1 -Antitrypsin deficiency i s a wel l -recogni zed genet i c fact or t hat predi s pos es pat i ent s t o emphys ema. It account s for approxi mat el y 2% of al l cas es of emphys ema. o
o
a. Pres umabl y, t hi s defi ci ency i ncreas es t he s us cept i bi l i t y of pul monary t i s s ue t o aut odi ges t i on by nat ural l y occurri ng prot eas es .
Pa g e 2 6 4
ABC Ambe r CHM Conve rte r Tria l ve rsion, http://w w w .proce sste x t.com/a bcchm.html
FIGURE 2-8. Age-rel at ed rat e of decl i ne i n l ung funct i on i n vari ous pat i ent groups . FEV 1 , forced expi rat ory vol ume i n 1 s econd. (Modi fi ed wi t h permi s s i on from Sni der GL, Sal i ng LJ, Renard SI. Chronic bronchi t i s and emphys ema. In Murray JF, Nadel JA, eds . Text book of Res pi rat ory Medi ci ne. Phi l adel phi a: W B Saunders , 1994:1342. )
P.53
o
o
b. Emphys ema may devel op earl y i n l i fe i n i ndi vi dual s wi t h homozygous defi ci ency. Ci garet t e s moki ng accel erat es t he proces s . In i t s pure form (s ee al s o Chapt er 5 IX C 1), α 1 -ant i t ryps i n defi ci ency mani fes t s as hepat i c ci rrhos i s , abs ence of t he α 1 -gl obul i n peak on s erum prot ei n el ect rophores i s , negl i gi bl e amount s of s erum α 1 -ant i t ryps i n, and advanced panl obul ar emphys ema, predomi nant l y i n t he bas e of t he l ungs .
C. Pathogenesis Evi dence s ugges t s t hat COPD begi ns i n t he s mal l ai rways (i .e., t hos e <2 mm i n di amet er). Normal l y, t hes e ai rways cont ri but e onl y a s mal l amount of t he t ot al res i s t ance t o ai rfl ow i n t he
Pa g e 2 6 5
ABC Ambe r CHM Conve rte r Tria l ve rsion, http://w w w .proce sste x t.com/a bcchm.html
t racheobronchi al t ree. In COPD, however, t he s mal l ai rways are t he s i t es of ext ens i ve di s eas e and ai rfl ow l i mi t at i on. Recent evi dence s ugges t s t hat i nfl ammat i on i s commonl y found i n hi s t opat hol ogi c s ampl es , i nvol vi ng t he ai rway, al veol ar t i s s ue, and i n s ome s t udi es , t he pul monary vas cul ar bed.
1. Hi s t opat hol ogi c abnormal i t i es of t he s mal l ai rways are common fi ndi ngs at aut ops y i n young s mokers . Thi s s mal l -ai rway di s eas e i s pres umed t o progres s over approxi mat el y 30 years t o t he charact eri s t i c cl i ni cal pi ct ure of di s abl i ng COPD.
2. If young s mokers s t op s moki ng, t he abnormal i t i es obs erved i n t es t s of s mal l -ai rway funct i on t end t o res ol ve. It i s uncl ear at what poi nt duri ng t he cours e of t he di s eas e t he changes become i rrevers i bl e and i nevi t abl y progres s t o COPD.
D. Pathology and pathophysiology The pat hol ogi c changes of COPD are s een i n bot h t he ai rways and t he pul monary parenchyma.
1. Bronchitic component o
o
a. In earl y di s eas e, t he s mal l ai rways demons t rat e mucous pl uggi ng, i nfl ammat i on, narrowi ng, and obl i t erat i on.
o
o
b. In es t abl i s hed di s eas e, t he bronchi t i c component i ncl udes varyi ng degrees of mucus gl and hyperpl as i a, mucos al i nfl ammat i on and edema, bronchos pas m, i mpact ed s ecret i ons , and
Pa g e 2 6 6
ABC Ambe r CHM Conve rte r Tria l ve rsion, http://w w w .proce sste x t.com/a bcchm.html
peri bronchi ol ar fi bros i s . Thes e el ement s cont ri but e t o ai rway narrowi ng and i ncreas ed ai rway res i s t ance.
2. Emphysematous component o
o
a. The des t ruct i on of al veol ar wal l s and t he s upport i ng s t ruct ure produces wi del y di l at ed ai r s paces . The l os s of t i s s ue s upport for t he ai rways i s bel i eved t o cont ri but e t o ai rway narrowi ng; t he uns upport ed ai rways t end t o col l aps e dynami cal l y duri ng expi rat i on t o l ow l ung vol umes . In addi t i on, t he l os s of t he al veol ar capi l l ary membrane reduces D LC O .
o
o
b. Anat omi cal l y, emphys ema i s cl as s i fi ed as ei t her centrilobular or panlobular (Fi gure 2-9).
(1) Centrilobular emphysema oft en i s as s oci at ed wi t h chronic bronchitis and bronchial inflammation. It i s t he mos t common t ype of emphys ema encount ered i n cl i ni cal pract i ce. The condi t i on i s rare i n nons mokers .
(a) Cent ri l obul ar emphys ema i s bel i eved t o repres ent a des t ruct i ve l es i on of t he res pi rat ory bronchi ol e. It ori gi nat es at t he cent er of t he l obul e and i s di s t i nct from t he peri phery of t he aci nus wi t h i t s s ept a and ves s el s .
Pa g e 2 6 7
ABC Ambe r CHM Conve rte r Tria l ve rsion, http://w w w .proce sste x t.com/a bcchm.html
(b) Cent ri l obul ar emphys ema i s vari abl e and pat chy and has a predi l ect i on for upper l ung zones . The great l y di l at ed res pi rat ory bronchi ol es coal es ce and may produce bul l ous cys t s .
(2) Panlobular emphysema, i n cont ras t t o cent ri l obul ar emphys ema, has l i t t l e as s oci at i on wi t h chroni c bronchi t i s and i s s een commonl y i n pat i ent s wi t h α 1 -ant i t ryps i n defi ci ency, or i n t hos e wi t h pri or i nt ravenous met hyl pheni dat e (Ri t al i n) abus e. It i s t hought t o evol ve from di l at ed al veol ar duct s at t he peri phery of t he l obul es t o markedl y enl arged ai r s paces and occurs mos t commonl y i n t he l ung bas es .
E. Clinical features
1. T ypical history. COPD i s an i ns i di ous , l ong-t erm proces s t hat t ypi cal l y devel ops as fol l ows : o
o
a. In t eenagers who s moke, mi l d but as ympt omat i c changes devel op i n t he s mal l ai rways .
o
o
b. As adul t s , t hes e s mokers experi ence chroni c cough and s ympt oms s ugges t i ve of an upper res pi rat ory i nfect i on.
o
o
c. By mi ddl e age, t hes e s mokers have s i gni fi cant bronchi al di s eas e charact eri zed by progres s i ve
Pa g e 2 6 8
ABC Ambe r CHM Conve rte r Tria l ve rsion, http://w w w .proce sste x t.com/a bcchm.html
ai rway obs t ruct i on t hat produces dys pnea on exert i on. Thi s condi t i on occurs i n 15%–25% of s mokers and frequent l y goes unrecogni zed wi t h t he pat i ent s ' more s edent ary l i fes t yl e. P.54
FIGURE 2-9. Emphys ema i s cl as s i fi ed accordi ng t o t he pul monary l obul e t hat i s di l at ed. (A) A normal pul monary l obul e. (B) In cent rol obul ar (cent ri aci nar) emphys ema t he mi dport i on of t he l obul e i s di l at ed, i ncl udi ng t he res pi rat ory bronchi ol e and al veol ar duct s . The t ermi nal al veol i are s pared. (C) In panl obul ar (panaci nar) emphys ema t he ent i re pul monary l obul e i s di l at ed. (From Rubi n E, et al . Rubi n's Pat hol ogy: Cl i ni copat hol ogi c Foundat i ons of Medi ci ne, 4t h ed. Bal t i more: Li ppi ncot t W i l l i ams & W i l ki ns , 2005. ) o
o
d. W hen an unrel at ed heal t h probl em pl aces t he s mokers ' res pi rat ory s ys t em under s t res s , t he
Pa g e 2 6 9
ABC Ambe r CHM Conve rte r Tria l ve rsion, http://w w w .proce sste x t.com/a bcchm.html
pres ence of COPD becomes evi dent . Pneumoni a, s urgery, and t rauma are common preci pi t at i ng event s .
2. Clinical syndromes o
o
a. T wo classic types of COPD exi s t and are gi ven vari ous names . Pat i ent s wi t h “emphys emat ous ,― “dys pnei c,― or “t ype A― COPD are referred t o as pink puffers; t hos e wi t h “bronchi t i c,― “t us s i ve,― or “t ype B― COPD are referred t o as blue bloaters.
(1) Pink puffers have predomi nant emphys ema and s how s ympt oms at a rel at i vel y advanced age (oft en >60 years ). There i s progres s i ve exert i onal dys pnea, wei ght l os s , and l i t t l e or no cough and expect orat i on.
(a) Pul monary funct i on t es t i ng i ndi cat es mi l d hypoxi a, hypocapni a, decreas ed D LC O , onl y a mi l d i ncreas e i n Raw, and l i t t l e i mprovement i n ai rfl ow aft er t reat ment wi t h bronchodi l at ors .
(b) Thes e pat i ent s us ual l y undergo a s l owl y progres s i ve downhi l l cours e.
(2) Blue bloaters have predomi nant chroni c
Pa g e 2 7 0
ABC Ambe r CHM Conve rte r Tria l ve rsion, http://w w w .proce sste x t.com/a bcchm.html
bronchi t i s and, at a rel at i vel y young age, experi ence chroni c cough and expect orat i on, epi s odi c dys pnea, and wei ght gai n. W heezi ng and rhonchi frequent l y are heard i n t he ches t , and cor pul monal e oft en devel ops , accompani ed by edema and cyanos i s .
(a) Pul monary funct i on t es t i ng i ndi cat es s evere hypoxi a, hypercapni a, pol ycyt hemi a, i ncreas ed Raw, i mproved ai rfl ow aft er t reat ment wi t h bronchodi l at ors , and rel at i vel y pres erved l ung vol umes and D LC O .
(b) Pat hol ogi cal l y, t here i s mi ni mal emphys ema but s i gni fi cant bronchi ol i t i s , bronchi t i s , mucous gl and hyperpl as i a, and ri ght vent ri cul ar hypert rophy. Thi s cont ras t s wi t h pat hol ogi c fi ndi ngs i n pi nk puffers , whi ch s how pri mari l y emphys ema.
o
o
b. Pat i ent s wi t h emphys ema have proport i onal and mat ched l os s es of vent i l at i on and perfus i on and, hence, are s pared s evere hypoxemi a. In cont ras t , pat i ent s wi t h chroni c bronchi t i s have P.55
marked [V wi t h dot above]/[Q wi t h dot above] mi s mat ch, res ul t i ng i n s evere hypoxemi a. The hypoxi a i n chroni c bronchi t i s i s wors ened by t he hypercapni a, whi ch may be t he res ul t of res pi rat ory mus cl e dys funct i on.
Pa g e 2 7 1
ABC Ambe r CHM Conve rte r Tria l ve rsion, http://w w w .proce sste x t.com/a bcchm.html
F. Diagnosis The di agnos i s of COPD may be s us pect ed bas ed on t he pat i ent 's hi s t ory, s ympt oms , and phys i cal s i gns , or i t may become obvi ous t hrough ches t radi ograph and cl i ni cal ci rcums t ances .
1. Spi romet ri c s creeni ng may be us ed t o di agnos e COPD i n mi ddl e-aged s mokers . Object i ve document at i on of expi rat ory fl ow obs t ruct i on by pul monary funct i on t es t i ng i s neces s ary for t he di agnos i s of COPD.
2. On phys i cal exami nat i on, t he fi ndi ngs mi ght i ncl ude hyperi nfl at i on, poor di aphragmat i c movement , t he us e of acces s ory mus cl es of res pi rat i on, and decreas ed breat h s ounds and wheezi ng on aus cul t at i on.
3. The ches t radi ograph may s how hyperi nfl at i on, a l os s of vas cul ari t y, a fl at t ened di aphragm, and a s mal l heart . In addi t i on, pat i ent s wi t h chroni c bronchi t i s oft en exhi bi t t hi ckened bronchi al wal l s and “di rt y― l ung fi el ds . Di agnos i s s houl d not res t s ol el y on t he bas i s of radi ographi c fi ndi ngs .
G. Clinical course and prognosis
1. COPD t ends t o be a progres s i ve di s order unl es s t here i s s ome form of i nt ervent i on (i .e., ces s at i on of s moki ng, removal of ot her i rri t ant s , or medi cal t herapy). Once COPD i s wel l advanced, s peci fi c medi cal t reat ment may not s l ow t he progres s i on of di s eas e. However, ces s at i on of s moki ng may al t er t he decl i ne i n pul monary funct i on i n al l but far-advanced di s eas e.
Pa g e 2 7 2
ABC Ambe r CHM Conve rte r Tria l ve rsion, http://w w w .proce sste x t.com/a bcchm.html
2. The FEV 1 decl i nes by approxi mat el y 50–75 mL/yr i n t ypi cal pat i ent s wi t h COPD, as compared wi t h a normal decl i ne of approxi mat el y hal f t hat rat e.
3. Survi val i s s t at i s t i cal l y rel at ed t o t he degree of vent i l at ory funct i on t hat exi s t s when pat i ent s are fi rs t eval uat ed. For exampl e, among pat i ent s wi t h an i ni t i al FEV 1 of l es s t han 0.75 L, t he 5-year s urvi val rat e i s 25%, whereas among t hos e wi t h an i ni t i al FEV 1 of 1 L, t he 5-year s urvi val rat e i s approxi mat el y 50%.
H. Therapy (see Figure 2-10)
1. Bronchodilation. Overal l , mos t pat i ent s wi t h COPD s how i mprovement i n pul monary funct i on aft er s uffi ci ent bronchodi l at or t herapy. o
o
a. COPD i s not al ways an i rrevers i bl e proces s . A pat i ent may have ai rfl ow react i vi t y even when pul monary funct i on s t udi es performed aft er bron chodi l at or t herapy have i ndi cat ed nonrevers i bl e ai rfl ow obs t ruct i on. It appears t hat 15%–20% of pat i ent s wi t h COPD i n t hi s cat egory have revers i bl e ai rfl ow obs t ruct i on.
o
o
b. The t hree major cl as s es of bronchodi l at ors are β-adrenergic agonists (al but erol and ot hers ), anticholinergics (i prat ropi um bromi de), and xanthines (t heophyl l i ne-t ype agent s ). Thes e agent s may be us ed s eparat el y or i n combi nat i on. The s el ect i ve β-adrenergi c agoni s t s offer
Pa g e 2 7 3
ABC Ambe r CHM Conve rte r Tria l ve rsion, http://w w w .proce sste x t.com/a bcchm.html
advant ages over t he ol der agent s i n t erms of a l onger durat i on of bronchodi l at i ng act i on (β 2 effect ) and reduced cardi ac s t i mul at i on (β 1 effect ). Some newer β agoni s t s have acut e as wel l as l ong-act i ng effect s .
2. Corticosteroid therapy. The us e of cort i cos t eroi ds i n t he t reat ment of t he bronchos pas m as s oci at ed wi t h COPD i s cont rovers i al . However, s ome pat i ent s cl earl y have an as t hma-l i ke component t o t hei r di s eas e and may l i kel y res pond t o s t eroi d t herapy t o s ome ext ent . Del i very of i nhal ed cort i cos t eroi ds by a met ered-dos e i nhal er (MDI) i s t he preferred rout e of admi ni s t rat i on. Current gui del i nes s t at e t hat i nhal ed cort i cos t eroi ds are appropri at e for s ympt omat i c COPD pat i ent s wi t h an FEV 1 <50% and frequent exacerbat i ons .
3. Sputum mobilization. Tradi t i onal expect orant s and mucol yt i c agent s appear t o have l i t t l e benefi ci al effect i n COPD pat i ent s .
4. Management of infection. Pat i ent s pres ent i ng i n exacerbat i on are more l i kel y t o be i nfect ed wi t h Haemophilus influenzae, Moraxella catarrhalis, or Streptococcus pneumoniae. Ant i bi ot i cs have been s hown t o be us eful i f pat i ent s have at l eas t t wo of t he fol l owi ng t hree s ympt oms : i ncreas ed dys pnea, i ncreas ed s put um vol ume, or i ncreas ed s put um purul ence. In addi t i on, pat i ent s wi t h COPD s houl d recei ve annual vacci nat i on agai ns t i nfl uenz a, as wel l as a vacci ne for pneumococcal i nfect i on every 5–6 years . P.56
Pa g e 2 7 4
ABC Ambe r CHM Conve rte r Tria l ve rsion, http://w w w .proce sste x t.com/a bcchm.html
FIGURE 2-10. Management of COPD as adapt ed from t he gl obal i ni t i at i ve for chroni c obs t ruct i ve l ung di s eas e (Gol d), 2006. ht t p://www.gol dcopd.org/gui del i nei t em.as p?l 1=2&l 2=1&i nt i d=989 FEV 1 , forced expi rat ory vol ume i n 1 s econd; FVC, forced vi t al capaci t y.
5. Pulmonary rehabilitation programs. Part i ci pat i on i n t hes e programs may i mprove exerci s e performance, decreas e dys pnea, and i mprove qual i t y of l i fe. Pat i ent s are i ns t ruct ed i n exerci s e programs ai med at i ncreas i ng exerci s e t ol erance and res pi rat ory mus cl e s t ami na. In addi t i on, couns el i ng and nut ri t i onal gui dance are avai l abl e.
6. Surgery o
Pa g e 2 7 5
ABC Ambe r CHM Conve rte r Tria l ve rsion, http://w w w .proce sste x t.com/a bcchm.html
o
a. The resection of l arge l ocal i zed bul l ae (e.g., bul l ect omy) may benefi t pat i ent s wi t h l arge bul l ae t hat occupy more t han one-t hi rd of a hemi t horax, wi t hout evi dence of di ffus e underl yi ng emphys ema.
o
o
b. Si ngl e- and doubl e-l ung t rans pl ant at i ons have been performed i n pat i ent s wi t h COPD, and t hey i mprove qual i t y of l i fe and exerci s e t ol erance. However, bronchi ol i t i s obl i t erans cont i nues t o be t he major obs t acl e t o l ong-t erm s ucces s ; 5-year mort al i t y s t i l l approaches 50%.
o
o
c. Lung vol ume reduct i on s urgery (LVRS) has been s hown t o i mprove l ung funct i on, exerci s e performance, qual i t y of l i fe, and even s urvi val i n s el ect pat i ent s wi t h emphys ema. Upper l obe predomi nant emphys ema by comput ed t omography (CT) s can appears t o be t he key prognos t i c fact or i n predi ct i ng s ucces s wi t h LVRS. By decreas i ng l ung vol ume i n hyperi nfl at ed pat i ent s , LVRS i mproves res pi rat ory mus cl e funct i on, ches t wal l mechani cs , and i ncreas es l ung recoi l , t hereby faci l i t at i ng enhanced expi rat ory fl ow.
7. Oxygen. Oxygen admi ni s t rat i on i s t he onl y t herapeut i c modal i t y ot her t han LVRS t hat can i mprove s urvi val . It s val ue, however, i s onl y demons t rabl e i n t hos e pat i ent s wi t h a PaO 2 of 55 mm Hg or l es s on room ai r at res t .
8. α 1 -Antitrypsin. For t hos e pat i ent s wi t h a defi ci ency of α 1 -ant i t ryps i n, repl acement t herapy i s avai l abl e.
Pa g e 2 7 6
ABC Ambe r CHM Conve rte r Tria l ve rsion, http://w w w .proce sste x t.com/a bcchm.html
I. Prevention Avoi dance of s moki ng i s by far t he bes t means of di s eas e prevent i on. In addi t i on, pat i ent s s houl d avoi d chroni c expos ure t o ot her bronchi al i rri t ant s . Si mpl e s pi romet ri c s creeni ng P.57 of hi gh-ri s k pat i ent s can hel p det ect earl y di s eas e s o appropri at e i nt ervent i ons can be t aken t o hel p prevent furt her det eri orat i on.
J. Complications of COPD Complications of COPD i ncl ude cor pul monal e, pol ycyt hemi a, i nfect i on, res pi rat ory fai l ure, bronchogeni c carci noma, noct urnal hypoxi a, general di s abi l i t y, and (for unknown reas ons ) a procl i vi t y for pept i c ul cer di s eas e.
III. Asthma A. Definition As t hma i s a revers i bl e ai rway obs t ruct i on t hat i s charact eri zed by hyperi rri t abi l i t y and i nfl ammat i on of t he ai rways . Subs t ances t hat have no effect when i nhal ed by normal i ndi vi dual s can caus e bronchocons t ri ct i on i n pat i ent s wi t h as t hma. As t hma may be a s i gni fi cant caus e of di s abi l i t y. A pri nci pal feat ure of as t hma i s i t s extreme variability, bot h from pat i ent t o pat i ent and from t i me t o t i me i n t he s ame pat i ent . Anot her feat ure of as t hma t hat i s i mport ant t o i t s pat hophys i ol ogy and t reat ment i s t he pres ence of airway inflammation.
B. Incidence and etiology
1. As t hma occurs i n 3%–8% of t he popul at i on. Et i ol ogi c or pat hol ogi c cl as s i fi cat i on of t he di s eas e i s di ffi cul t ; however, as t hma t radi t i onal l y i s di vi ded i nt o t wo forms . o
Pa g e 2 7 7
ABC Ambe r CHM Conve rte r Tria l ve rsion, http://w w w .proce sste x t.com/a bcchm.html
o
a. An allergic form, res pons i bl e for mos t cas es of as t hma i n chi l dren and many i n adul t s , i s i mmunol ogi cal l y medi at ed—i t i s caus ed by t ype I (i mmedi at e) hypers ens i t i vi t y t o i nhal ed ant i gens .
o
o
b. An intrinsic form, whi ch occurs pri mari l y i n adul t s , s hows no evi dence of i mmedi at e hypers ens i t i vi t y t o s peci fi c ant i gens .
2. In pat i ent s i n whom t he evi dence of i mmedi at e hypers ens i t i vi t y t o ant i gen i s abs ent or equi vocal , mos t at t acks do not appear t o be provoked by i nhal at i on of ant i gens , and t here i s a poor correl at i on bet ween t he s everi t y of s ympt oms and t he l evel s of s peci fi c ant i gens ci rcul at i ng as ai rborne part i cl es . Thes e obs ervat i ons i ndi cat e t hat as t hmat i c bronchos pas m may not neces s ari l y requi re an i mmunol ogi cal l y medi at ed hypers ens i t i vi t y react i on.
C. Pathogenesis
1. Biochemical mediators o
o
a. The mediators of immediate hypersensitivity and t hei r mechani s ms of act i on are di s cus s ed i n Chapt er 7 II A.
o
o
b. Other biochemical mediators, i ncl udi ng s erot oni n, pros t agl andi ns , t hromboxanes , and endoperoxi des , al s o caus e t i s s ue i nfl ammat i on and may be part i cul arl y i mport ant i n t he pat hogenes i s
Pa g e 2 7 8
ABC Ambe r CHM Conve rte r Tria l ve rsion, http://w w w .proce sste x t.com/a bcchm.html
of nonallergic as t hma. o
o
c. The phys i ol ogi c changes s een i n as t hma can be di rect l y i nduced by medi at ors t hat di ffus e l ocal l y t o mucous gl ands , ves s el s , and s moot h mus cl e. Cl earl y, however, t he s i mpl e rel eas e of medi at ors i s not s uffi ci ent t o caus e an as t hmat i c at t ack.
2. Airway hyperirritability o
o
a. The t racheobronchi al t ree of as t hmat i c i ndi vi dual s appears t o have an exaggerat ed react i vi t y, s omet i mes cal l ed nonspecific bronchial hyperreactivity, t o di s t i ngui s h i t from t he bronchos pas m provoked by i mmunol ogi cal l y s peci fi c ant i gens .
o
o
b. The mechani s m underl yi ng bronchi al hyperreact i vi t y i s unknown, but a number of fact ors have been s ugges t ed.
(1) Muscle reactivity. A change i n t he cont ract i l e mechani s ms of ai rway s moot h mus cl e may i nduce hyperreact i vi t y.
(2) Autonomic reactivity. The abnormal i t y may exi s t i n t he nerves t hat regul at e t he t one of t he mus cl e, not i n t he mus cl e i t s el f. The parasympathetic system appears t o medi at e t he refl ex bronchi al cons t ri ct i on caus ed by i nhal at i on of nons peci fi c i rri t ant s s uch as dus t
Pa g e 2 7 9
ABC Ambe r CHM Conve rte r Tria l ve rsion, http://w w w .proce sste x t.com/a bcchm.html
or s ul fur di oxi de. A defi ci ency i n t he sympathetic nervous system may al s o be res pons i bl e for bronchi al hyperreact i vi t y.
(3) Environmental factors. Bronchi al react i vi t y i s t emporari l y i ncreas ed by upper res pi rat ory vi ral i nfect i ons and by expos ure t o pol l ut ant s s uch as ozone, ni t rogen di oxi de (NO 2 ), P.58
and i ndus t ri al fumes . By ampl i fyi ng t he res pons e t o t he mat eri al s rel eas ed from mas t cel l s , t hi s i ncreas ed react i vi t y coul d be res pons i bl e for t he exacerbat i ons of as t hma s een wi t h vi ral i nfect i ons , occupat i onal expos ures , and expos ure t o s evere pol l ut i on.
D. Pathophysiology
1. Pat hophys i ol ogi cal l y, as t hma i s charact eri zed by cons t ri ct i on of ai rway s moot h mus cl e, hypers ecret i on of mucus , edema and i nfl ammat ory cel l i nfi l t rat i on of t he ai rway mucos a, and t hi ckeni ng of t he bas ement membrane underl yi ng t he ai rway epi t hel i um.
2. Thes e pat hophys i ol ogi c changes are not uni forml y di s t ri but ed. Some ai rways may di s pl ay a predomi nance of bronchos pas m, ot hers may be occl uded by mucous pl uggi ng, and s t i l l ot hers may appear unaffect ed.
E. Clinical features
Pa g e 2 8 0
ABC Ambe r CHM Conve rte r Tria l ve rsion, http://w w w .proce sste x t.com/a bcchm.html
1. Cl as s i c presentation i nvol ves epi s odi c bout s of coughi ng, dys pnea, ches t t i ght nes s , and wheezi ng. Upper res pi rat ory vi ral i nfect i on, expos ure t o al l ergens , emot i onal s t res s , and many nons peci fi c preci pi t at i ng event s may provoke as t hmat i c at t acks .
2. The symptoms exhi bi t a wi de s pect rum of s everi t y, refl ect i ng t he vari abi l i t y of t he underl yi ng ai rway obs t ruct i on. Some pat i ent s di s pl ay onl y occas i onal at t acks of exert i onal dys pnea and wheezi ng, whi ch res pond t o i nhal ed bronchodi l at ors al one. Ot her pat i ent s have chroni c s ympt oms requi ri ng cont i nuous us e of oral l y i nhal ed medi cat i ons . Al t hough t hi s l at t er group of pat i ent s may have i rrevers i bl e t hi ckeni ng of t he ai rway wal l s , t hei r i l l nes s s t i l l i s epi s odi c i n occurrence.
3. Pat i ent s are cl as s i fi ed as mi l d i nt ermi t t ent , mi l d pers i s t ent , moderat e pers i s t ent , or s evere pers i s t ent bas ed on s ympt om frequency, us e of s hort -act i ng β 2 -agoni s t s , and peak expi rat ory fl ow rat es or pul monary funct i on (Tabl e 2-3).
4. The t erm status asthmaticus us ual l y i s res erved for a prol onged, s evere as t hmat i c at t ack t hat does not res pond t o t reat ment and i nvol ves bronchos pas m s o s evere t hat t he pat i ent i s at ri s k for vent i l at ory fai l ure. o
o
a. The cl i ni cal mani fes t at i ons of s evere as t hma i ncl ude fat i gue, a pul s e rat e of more t han 100 bpm, and cyanos i s . The us e of acces s ory mus cl es for
Pa g e 2 8 1
ABC Ambe r CHM Conve rte r Tria l ve rsion, http://w w w .proce sste x t.com/a bcchm.html
res pi rat i on i s frequent l y evi dent . o
o
b. An i ns pi rat ory decreas e of more t han 20 mm Hg i n s ys t ol i c bl ood pres s ure (i .e., pulsus paradoxus) i ndi cat es gros s overi nfl at i on of t he l ung and wi de s wi ngs i n pl eural pres s ure.
F. Diagnosis
1. Physical examination t ypi cal l y s hows t achycardi a; t achypnea wi t h prol onged expi rat i on; overi nfl at i on of t he ches t wi t h poor movement of t he di aphragm; and di ffus e, hi gh-pi t ched expi rat ory wheezi ng.
2. Sputum analysis. Sput um may appear purul ent becaus e of an i ncreas ed eos i nophi l cont ent or an i nfl ammat ory res pons e t o a vi ral t racheobronchi t i s . Sput um s mears may reveal Curschmann' s spirals (i .e., mucus t hat forms a cas t of t he s mal l ai rways ) or Charcot–Leyden crystals (i .e., breakdown product s of t he eos i nophi l s ).
3. Hematologic studies i ndi cat e a modes t l eukocyt os i s and eos i nophi l i a i n bot h t he al l ergi c and i nt ri ns i c forms of t he di s eas e.
4. Pulmonary function testing o
o
a. Pul monary funct i on t es t s are t ypi cal l y normal . Duri ng at t acks t he FVC i s reduced; t he FEV 1 /FVC i s reduced but may i mprove aft er i nhal at i on of a
Pa g e 2 8 2
ABC Ambe r CHM Conve rte r Tria l ve rsion, http://w w w .proce sste x t.com/a bcchm.html
bronchodi l at or. RV, TLC, and l ung compl i ance us ual l y are i ncreas ed, and t he D LC O frequent l y i s i ncreas ed. o
o
b. Aft er s ympt omat i c recovery, TLC and l ung compl i ance ret urn t o normal , but t he maxi mum expi rat ory fl ow rat e may remai n reduced at l ow l ung vol umes , and an abnormal di s t ri but i on of vent i l at i on may pers i s t , refl ect i ng res i s t ant obs t ruct i on of s mal l ai rways .
5. Chest radiography us ual l y s hows not hi ng more t han hyperi nfl at i on. Occas i onal fi ndi ngs i ncl ude l ocal i zed dens i t y caus ed by a l arge mucous pl ug or t he omi nous s i gn of pneumot horax or pneumomedi as t i num, refl ect i ng t he rupt ure of al veol ar t i s s ue caus ed by hi gh i nt ra-al veol ar pres s ure. P.59
TABLE 2-3 Asthma Management in Adults According to Classification of Severity
Pa g e 2 8 3
ABC Ambe r CHM Conve rte r Tria l ve rsion, http://w w w .proce sste x t.com/a bcchm.html
R e c o m m e n d e d m a Fr Fr
n
e e
a
q q
g
u u
e
e e P m n n E e c c F nt y y o (i of of r n d ni F cl S a g E u e yt ht V 1 d v i ti P e e mm E s ri e e F s ty s s v h of y y a o a m m ri rt st pt pt a h o o bi a m m m lit ct a s s y in
Pa g e 2 8 4
ABC Ambe r CHM Conve rte r Tria l ve rsion, http://w w w .proce sste x t.com/a bcchm.html
g in h al e d β a g o ni st a s n e e d e d ) Mi â â â N l d ‰‰‰o in ¤ ¤ ¥ n te 2 2 8 e r d ni 0 n m a g % ec i t ys ht < e te p s 2 ss nt er p 0 ar w er % y e m e o k nt
Pa g e 2 8 5
ABC Ambe r CHM Conve rte r Tria l ve rsion, http://w w w .proce sste x t.com/a bcchm.html
A h sy m pt o m at ic a n d n or m al P E F b et w e e n at ta ck s Mi > > â L l d 2 2 ‰o p t i ni ¥ w er m g 8 -d s i e ht 0 o st s s % s e p p 2 e
Pa g e 2 8 6
ABC Ambe r CHM Conve rte r Tria l ve rsion, http://w w w .proce sste x t.com/a bcchm.html
nt er er 0 i n w m %h e o â al e nt €“ e k h 3 d b
0 co
ut
% rt
<
ic
1
o
ti
st
m
er
e
oi
p
d
er d a y At ta ck s m a y af fe ct ac ti vi ty M D > 6 L o ai 1 0 o d l y ni % w er At g â -
Pa g e 2 8 7
ABC Ambe r CHM Conve rte r Tria l ve rsion, http://w w w .proce sste x t.com/a bcchm.html
at t a ht €“ m e ck p 8 e p s er 0 di er af w % u s i fe e > m s t ct e 3 d e ac k 0 o nt t i
%s
vi
e
ty
in h al e d co rt ic o st er oi d pl u s lo n gac ti n g β -a g
Pa g e 2 8 8
ABC Ambe r CHM Conve rte r Tria l ve rsion, http://w w w .proce sste x t.com/a bcchm.html
o ni st Al te r n at iv e: m e di u m d o s e in h al e d st er oi d pl u s or al a nt
Pa g e 2 8 9
ABC Ambe r CHM Conve rte r Tria l ve rsion, http://w w w .proce sste x t.com/a bcchm.html
i -i nf la m m at or y * S C Fr â Hi e o e ‰g v nt q ¤ h er i n u 6 d e u e 0 o p o nt % s er u
> e
si s
3 in
s t Li
0 h
e m
% al
nt i t
e
e
d
d
co
p
rt
h
ic
ys
o
ic
st
al
er
ac
oi
ti
d
vi
pl
ty
u s lo n g-
Pa g e 2 9 0
ABC Ambe r CHM Conve rte r Tria l ve rsion, http://w w w .proce sste x t.com/a bcchm.html
ac ti n g β -a g o ni st a n d or al a nt i -i nf la m m at or y if n e e d e d a n d
Pa g e 2 9 1
ABC Ambe r CHM Conve rte r Tria l ve rsion, http://w w w .proce sste x t.com/a bcchm.html
or al gl uc oc or ti co id a s n e e d e d PEF, peak expi rat ory fl ow; FEV 1 , forced expi rat ory vol ume i n 1 s econd. *Oral ant i -i nfl am mat ory medi cat i on s i ncl ude: l eukot ri ene modi fi ers s uch as z afi rl ukas t or z i l eut on;
Pa g e 2 9 2
ABC Ambe r CHM Conve rte r Tria l ve rsion, http://w w w .proce sste x t.com/a bcchm.html
l ong-act i ng t heophyl l i n e. Adapt ed from t he Nat i onal As t hma Educat i on and Prevent i on Program. ht t p://www .nhl bi .ni h. gov/gui del i nes /as t hm a/execs um m.pdf
6. Arterial blood gas studies. The PaCO 2 us ual l y i s l ow (i .e., <36 mm Hg). An i ncreas ed PaCO 2 or a normal PaCO 2
(40 mm Hg) i ndi cat es s evere obs t ruct i on. Art eri al
hypoxemi a i s common des pi t e t he i ncreas ed vent i l at i on and i s due t o undervent i l at i on of l ung s egment s s uppl i ed by narrowed ai rways (i .e., t here i s [V with dot above]/[Q with dot above] mi s mat ch).
G. Therapy T herapy i s bas ed on an unders t andi ng of t he underl yi ng pat hophys i ol ogi c mechani s ms . Effect i ve management of as t hma rel i es on four i nt egral component s : object i ve meas ures of l ung funct i on, pharmacol ogi c t herapy, envi ronment al meas ures t o cont rol al l ergens and i rri t ant s , and pat i ent educat i on.
1. Goals of therapy. The goal s of t herapy are t o:
Pa g e 2 9 3
ABC Ambe r CHM Conve rte r Tria l ve rsion, http://w w w .proce sste x t.com/a bcchm.html o
o
a. Mai nt ai n near-normal pul monary funct i on
o
o
b. Mai nt ai n normal act i vi t y l evel s
o
o
c. Prevent chroni c and t roubl es ome s ympt oms (e.g., cough, noct urnal s ympt oms )
o
o
d. Prevent recurrent exacerbat i ons
o
o
e. Mi ni mal us e of s hort -act i ng i nhal ed β 2 -agoni s t
o
o
f. Avoi d advers e effect s from as t hma medi cat i ons
2. Principles of treatment. As t hma i s a chronic condition with acute exacerbations. o
o
a. Prevention of exacerbat i ons i s part i cul arl y i mport ant .
o
o
b. Early intervention when t reat i ng acut e exacerbat i ons of as t hma i s i mport ant t o reduce t he l i kel i hood of devel opi ng s evere ai rway narrowi ng. P.60
o
Pa g e 2 9 4
ABC Ambe r CHM Conve rte r Tria l ve rsion, http://w w w .proce sste x t.com/a bcchm.html o
c. Control of airway inflammation i s a key fact or i n t reat i ng as t hma, as s ugges t ed by evi dence of t he pres ence of ai rway i nfl ammat i on i n al l s ubject s wi t h as t hma. The i ncreas ed l evel s of i nfl ammat ory medi at ors are as s oci at ed wi t h ai rway hyperres pons i venes s .
3. Pharmacologic therapy (s ee Tabl e 2-3) o
o
a. Anti-inflammatory agents (e.g., corticosteroids), whi ch i nt errupt t he devel opment of bronchi al i nfl ammat i on, have a prophyl act i c or prevent i ve act i on. Thes e agent s decreas e t he s everi t y of acut e as t hma at t acks as wel l as chroni c as t hma.
(1) Admi ni s t rat i on may be oral or vi a an i nhal er.
(2) Oral or i nt ravenous rout es are recommended i n acut e at t acks ; i nhal ed cort i cos t eroi ds are for l ong-t erm us e.
o
o
b. Bronchodilators act pri nci pal l y t o di l at e t he ai rways by rel axi ng bronchi al s moot h mus cl e. Agent s i ncl ude s ympat homi met i cs , met hyl xant hi nes , and ant i chol i nergi cs .
(1) Sympathomimetics (β 2 -agoni s t s ) are us ed duri ng acut e at t acks of as t hma. β 2 -agoni s t s are i nhal ed vi a an MDI or a
Pa g e 2 9 5
ABC Ambe r CHM Conve rte r Tria l ve rsion, http://w w w .proce sste x t.com/a bcchm.html
nebul i zer.
(2) Anticholinergic agents (e.g., i prat ropi um) are us ed pri mari l y as s uppl ement s t o ot her bronchodi l at ors duri ng acut e at t acks .
(3) Methylxanthines (t heophyl l i ne) are no l onger us ed as wi del y as t hey once were. They are pri mari l y us ed for noct urnal s ympt oms not wel l cont rol l ed by ot her agent s .
o
o
c. Leukotriene modifiers are a newer cl as s of drugs t hat are us eful i n l ong-t erm cont rol of a s el ect group of as t hmat i cs . They act by i nhi bi t i ng di fferent part s of t he arachi doni c aci d cas cade, whi ch pl ays an i mport ant rol e i n as t hma and ai rway i nfl ammat i on.
o
o
d. Ant i -IgE ant i body t herapy may be hel pful i n s ome as t hmat i cs wi t h el evat ed IgE s erum l evel s .
IV. Bronchiectasis, Cystic Fibrosis, and Lung Abscess A. Bronchiectasis
1. Definition. Bronchi ect as i s i s a pat hol ogi c, i rrevers i bl e di l at i on of t he bronchi caus ed by des t ruct i on of t he bronchi al wal l , us ual l y res ul t i ng from s uppurat i ve i nfect i on i n an obs t ruct ed bronchus .
Pa g e 2 9 6
ABC Ambe r CHM Conve rte r Tria l ve rsion, http://w w w .proce sste x t.com/a bcchm.html
2. Etiology and pathogenesis o
o
a. The s mal l bronchi of chi l dhood are mos t s us cept i bl e t o bronchi al i nfect i on and t o obs t ruct i on by i mpact ed s ecret i ons , forei gn bodi es , or compres s i ng l ymph nodes . Sevent y-fi ve percent of pat i ent s can recal l experi enci ng s ympt oms of bronchi ect as i s as earl y as t he age of 5 years .
o
o
b. The mos t common caus e of bronchi ect as i s i s bact eri al pneumoni a, whi ch may be pri mary or may be a compl i cat i on of meas l es , as pi rat i on of gas t ri c cont ent s or part i cul at e mat t er, or t umor.
o
o
c. Predi s pos i ng condi t i ons i ncl ude congeni t al di s orders (e.g., congeni t al cys t i c di s eas e of t he l ung, bronchi al s t enos i s , or compres s i on of bronchi by anomal ous art eri es t o t he l ung); i mmune defi ci enci es [e.g., hypogammagl obul i nemi a or i mmunogl obul i n A (IgA) defi ci ency]; and cys t i c fi bros i s .
3. Clinical features. Sympt oms i ncl ude a chroni c cough product i ve of purul ent s put um, recurrent ches t col ds or pneumoni as , occas i onal hemopt ys i s , and pl euri t i c pai n. Thes e s ympt oms cannot be di fferent i at ed from t hos e of chroni c s uppurat i ve bronchi t i s . Progres s i ve dys pnea, cyanos i s , di gi t al cl ubbi ng, and cor pul monal e are s een i n advanced cas es .
4. Diagnosis
Pa g e 2 9 7
ABC Ambe r CHM Conve rte r Tria l ve rsion, http://w w w .proce sste x t.com/a bcchm.html o
o
a. Physical examination i ndi cat es ral es over t he area of i nvol vement on repeat ed exami nat i ons .
o
o
b. Pulmonary function testing produces normal res ul t s i n mi l d cas es , but , i n moderat e or s evere cas es , i t may reveal ei t her res t ri ct i ve or a mi xt ure of res t ri ct i ve and obs t ruct i ve vent i l at ory pat t erns .
o
o
c. Chest radiography s hows peri bronchi al fi bros i s i n t he i nvol ved s egment . Segment al l ung col l aps e i n areas of bronchi ect as i s i s common.
o
o
d. CT scanning, part i cul arl y high-resolution computed tomography (HRCT ), can oft en i dent i fy bronchi ect as i s .
P.61
5. T herapy. The proper t herapy can markedl y i mprove s ympt oms . o
o
a. Medical treatment i s t he mai ns t ay and cons i s t s of t herapy wi t h ant i bi ot i cs on a frequent or regul ar bas i s (e.g., ampi ci l l i n, t et racycl i ne, eryt hromyci n, or as i ndi cat ed by cul t ure and s ens i t i vi t y t es t i ng), pos t ural drai nage, and i mmuni zat i on agai ns t i nfl uenz a and pneumococcal pneumoni a.
o
Pa g e 2 9 8
ABC Ambe r CHM Conve rte r Tria l ve rsion, http://w w w .proce sste x t.com/a bcchm.html o
b. Bronchodilation may be effect i ve i f t he obs t ruct i on i s revers i bl e.
o
o
c. Oxygen therapy i s appropri at e i f PaO 2 l evel s are depres s ed.
o
o
d. Surgical resection general l y i s not effect i ve for el i mi nat i ng chroni c cough and s put um unl es s t he operat i on i s performed i n earl y chi l dhood for s i ngl e l ocal ai rway di s eas e.
B. Cystic fibrosis
1. Definition. Cys t i c fi bros i s (CF) i s an aut os omal reces s i ve di s eas e charact eri zed by dys funct i on of t he exocri ne gl ands , l eadi ng t o obs t ruct i on i n s uch organs as t he l ungs , pancreas , and gas t roi nt es t i nal t ract . In 99% of pat i ent s wi t h CF, deat h i s caus ed by res pi rat ory fai l ure, wi t h port al hypert ens i on s econdary t o bi l i ary ci rrhos i s account i ng for t he remai nder of deat hs .
2. Incidence o
o
a. An es t i mat ed 30,000 i ndi vi dual s wi t h CF l i ve i n t he Uni t ed St at es . The di s eas e i s mos t common among whi t es , occurri ng i n 1 of 2000–2500 bi rt hs , and 5% of t he whi t e popul at i on are carri ers . Becaus e i nheri t ance i s reces s i ve and het eroz ygot es have no di s eas e, a negat i ve fami l y hi s t ory does not rul e out t he di s eas e.
o
Pa g e 2 9 9
ABC Ambe r CHM Conve rte r Tria l ve rsion, http://w w w .proce sste x t.com/a bcchm.html
o
b. Once t hought t o be uni que t o chi l dren, CF now i s recogni zed as t he mos t common caus e of obs t ruct i ve ai rway di s eas e among i ndi vi dual s up t o age 30 years . The medi an age of pat i ent s wi t h CF has ri s en from t he t eens i n t he 1960s t o approxi mat el y 37 years i n 2006.
3. Etiology and pathogenesis. The genet i c abnormal i t y t hat i s res pons i bl e for CF has been i dent i fi ed on t he l ong arm of chromos ome 7. o
o
a. The gene for CF produces a membrane t rans port prot ei n cal l ed cystic fibrosis transmembrane regulator (CFT R). More t han 200 mut at i ons have been di s covered i n t he CF gene; 70% of pat i ent s have a mut at i on t hat l eads t o a del et i on of t he ami no aci d at pos i t i on 508 i n t he prot ei n. Thi s del et i on produces an abnormal CFTR prot ei n.
o
o
b. The abnormal i t y s eems t o be an el ect rochemi cal defect i n t he epi t hel i al cel l t hat i nvol ves chl ori de i mpermeabi l i t y and i ncreas ed s odi um res orpt i on. Thi s abnormal i t y l eads t o t he vi s ci d mucus t hat i s obs erved and t he s ubs equent obs t ruct i on of organs s erved by exocri ne gl ands .
4. Clinical features. The mani fes t at i ons of CF are vari ed, wi t h pul monary abnormal i t i es bei ng t he overri di ng cl i ni cal concern. o
o
a. Pulmonary manifestations
Pa g e 3 0 0
ABC Ambe r CHM Conve rte r Tria l ve rsion, http://w w w .proce sste x t.com/a bcchm.html
(1) The earl i es t pul monary mani fes t at i on i s peri pheral ai rway obs t ruct i on res ul t i ng from pl ugged bronchi . Repeat ed bout s of i nfect i on l ead t o a cycl e of obs t ruct i on, t i s s ue damage, and i nfect i on t hat ul t i mat el y progres s es t o a l os s of pul monary funct i on.
(2) The predomi nant organi s m t o col oni ze t he l ung i s t he hi ghl y res i s t ant mucoi d s t rai n of Pseudomonas aeruginosa. Ini t i al i nfect i ons may be due t o Staphylococcus aureus. Compl et e eradi cat i on of t he organi s ms i s vi rt ual l y i mpos s i bl e.
(3) Many pat i ent s wi t h CF devel op s i nus i t i s and nas al pol yps , and mos t have cl ubbi ng of t he di gi t s .
(4) Mos t pat i ent s di e of res pi rat ory compl i cat i ons becaus e of a gradual l os s of l ung funct i on, t he res ul t of chroni c i nfl ammat i on perpet uat ed by P. aeruginosa.
o
o
b. Nonpulmonary manifestations al s o are common and i ncl ude meconi um i l eus , mal abs orpt i on, fat t y i nfi l t rat i on of t he l i ver, focal bi l i ary ci rrhos i s , gl ucos e i nt ol erance, s t eri l i t y i n mal es , and a predi l ect i on for heat pros t rat i on due t o s evere s al t depl et i on. As pat i ent s wi t h CF l i ve l onger, os t eoporos i s has become a probl em i n mal es as wel l as femal es . In addi t i on, t here appears t o be
Pa g e 3 0 1
ABC Ambe r CHM Conve rte r Tria l ve rsion, http://w w w .proce sste x t.com/a bcchm.html
s ome epi demi ol ogi c evi dence of an i ncreas e i n s ome gas t roi nt es t i nal cancers .
5. Diagnosis o
o
a. Sweat test. An abnormal s weat t es t i s obs erved i n vi rt ual l y al l cas es , and, i n combi nat i on wi t h cert ai n cl i ni cal hal l marks , confi rms t he di agnos i s .
(1) The s odi um and chl ori de concent rat i ons i n s weat are el evat ed dramat i cal l y, whereas normal concent rat i ons of t hes e i ons exi s t el s ewhere. The quantitative pilocarpine iontophoresis sweat test P.62
defi nes t he upper l i mi t of normal as a s weat chl ori de concent rat i on of 60 mEq/L.
(2) A pos i t i ve s weat t es t i s di agnos t i c of CF i n t he pres ence of at l eas t one of t he fol l owi ng t hree cri t eri a:
(a) A rel i abl e fami l y hi s t ory of CF
(b) Obs t ruct i ve pul monary di s eas e
(c) Pancreat i c i ns uffi ci ency
o
Pa g e 3 0 2
ABC Ambe r CHM Conve rte r Tria l ve rsion, http://w w w .proce sste x t.com/a bcchm.html
o
b. Pulmonary function testing s hows l i mi t at i on of forced expi rat ory fl ows and an el evat ed RV. The D LC O
i s us ual l y wi t hi n normal l i mi t s . As t he di s eas e
progres s ed, t he FEV 1 and rat i o of FEV 1 t o FVC decl i ne. o
o
c. Chest radiography s hows s t ri ki ng mani fes t at i ons t hat are more pronounced i n t he upper l obes . Fi ndi ngs i ncl ude hyperi nfl at i on, cys t format i on, at el ect as i s , bronchi ect as i s , and s egment al i nfi l t rat i on.
6. T herapy o
o
a. In t he early stages of t he di s eas e, t herapy mus t be i ndi vi dual i zed accordi ng t o s peci fi c cl i ni cal mani fes t at i ons .
(1) Sal t depl et i on i s a pot ent i al probl em i n warmer cl i mat es .
(2) Nut ri t i onal s uppl ement at i on and pancreat i c enzyme repl acement oft en are i ndi cat ed.
(3) Pul monary i nfect i ons may requi re hos pi t al i zat i on and vi gorous t reat ment wi t h parent eral ant i bi ot i cs , hydrat i on, humi di fi cat i on, and s uppl ement al oxygen.
o
Pa g e 3 0 3
ABC Ambe r CHM Conve rte r Tria l ve rsion, http://w w w .proce sste x t.com/a bcchm.html
o
b. In t he late stages of t he di s eas e, t herapy i s ai med at s uppres s i ng i nfect i on wi t h s peci fi c ant i bi ot i cs , i nduci ng s put um t hrough phys i cal mani pul at i on, and admi ni s t eri ng s uppl ement al oxygen as requi red. Dornas e-α, a drug t hat hydrol yzes ext racel l ul ar DNA and makes s put um l es s vi s cous , has been approved for us e i n pat i ent s wi t h CF. Thi s drug has been s hown t o decreas e res pi rat ory t ract i nfect i ons and i ncreas e pul monary funct i on. Aeros ol i zed t obramyci n i s an ami nogl ycos i de ant i bi ot i c t hat has demons t rat ed i mproved pul monary funct i on and decreas ed s ympt oms when us ed on a chroni c bas i s .
o
o
c. In end-stage di s eas e, bi l at eral l ung t rans pl ant at i on (t he procedure of choi ce) can be cons i dered. Res ul t s are s i mi l ar t o t hos e obt ai ned wi t h bi l at eral l ung t rans pl ant at i on i n ot her end-s t age l ung di s eas es .
o
o
d. New t herapi es ai med at count eract i ng t he pat hophys i ol ogi c proces s of CF are bei ng t es t ed. Thes e i ncl ude gene t herapy; mani pul at i on of i on t rans port and prot ei n t raffi cki ng; as wel l as admi ni s t rat i on of ant i -i nfl ammat ory agent s , ant i bi ot i cs , and ant i -el as t as es .
7. Complications. Cor pulmonale t hat i s onl y part i al l y res pons i ve t o oxygen t herapy devel ops l at e i n t he di s eas e, wi t h al l of t he mani fes t at i ons of ri ght -s i ded heart fai l ure (e.g., hepat omegal y, peri pheral edema). Ot her pul monary compl i cat i ons of CF i ncl ude hemoptysis and pneumothorax.
Pa g e 3 0 4
ABC Ambe r CHM Conve rte r Tria l ve rsion, http://w w w .proce sste x t.com/a bcchm.html
C. Lung abscess (see also Chapter 8 V C 5)
1. Definition. A l ung abs ces s i s a l ocal i zed area of i nfect i on wi t hi n t he l ung parenchyma t hat devel ops from an i ni t i al pneumoni c s t age. The cent er of t he i nfect ed area fi rs t becomes gangrenous , necrot i c, and purul ent and t hen becomes wel l demarcat ed from t he s urroundi ng l ung t i s s ue. The wal l of t he abs ces s becomes i nt ens el y i nfl amed and l i ned wi t h fi brous and granul at i on t i s s ue and abundant bl ood ves s el s .
2. Etiology. A s ol i t ary l ung abs ces s mos t commonl y res ul t s from as pi rat i on of s ecret i ons i n t he upper res pi rat ory t ract . Ot her l es s common caus es i ncl ude bronchi al obs t ruct i on, bact eri al pneumoni a, pul monary embol i s m wi t h i nfarct i on, t rans di aphragmat i c s pread from i nt ra-abdomi nal i nfect i ons , ches t t rauma, and bact eremi c i nfect i on.
3. Clinical features o
o
a. Ini t i al s ympt oms are s i mi l ar t o t hos e of acut e pneumoni a.
o
o
b. Chroni cal l y, l ung abs ces s i s as s oci at ed wi t h cons t i t ut i onal s ympt oms t hat i ncl ude wei ght l os s , l ow-grade fever, fat i gue, and mal ai s e.
4. Diagnosis o
Pa g e 3 0 5
ABC Ambe r CHM Conve rte r Tria l ve rsion, http://w w w .proce sste x t.com/a bcchm.html
o
a. Physical examination may reveal rel at i vel y normal fi ndi ngs , al t hough cl ubbi ng of t he nai l beds occas i onal l y i s not ed. Amphori c breat h s ounds may be heard over t he abs ces s cavi t y.
o
o
b. Pulmonary function testing us ual l y i s not affect ed by a l ung abs ces s . P.63
o
o
c. Bronchoscopy i s i ndi cat ed when an abs ces s does not res ol ve compl et el y wi t h ant i bi ot i c t herapy, or when t he pos s i bi l i t y of a mal i gnancy or forei gn body i n t he l ung exi s t s .
5. T herapy. The t reat ment of choi ce i s ant i bi ot i cs , ei t her peni ci l l i n or cl i ndamyci n. The t ot al durat i on of t herapy i s 4–8 weeks .
V. Acute Respiratory Failure A. Definition Acut e res pi rat ory fai l ure i s defi ned as hypoxemi a (i .e., a PaO 2 of <50 mm Hg) wi t h or wi t hout as s oci at ed hypercapni a (i .e., a PaCO 2 of >45 mm Hg).
B. Classification Acut e res pi rat ory fai l ure can be di vi ded i nt o t wo t ypes .
1. T ype I: respiratory failure without carbon dioxide retention (i .e., l ow PaO 2 wi t h l ow or normal PaCO 2 ). Thi s t ype of res pi rat ory fai l ure i s charact eri zed by
Pa g e 3 0 6
ABC Ambe r CHM Conve rte r Tria l ve rsion, http://w w w .proce sste x t.com/a bcchm.html
marked [V wi t h dot above]/[Q wi t h dot above] abnormal i t i es and i nt rapul monary s hunt i ng. T ype I res pi rat ory fai l ure occurs i n s uch cl i ni cal s et t i ngs as : o
o
a. Acute res pi rat ory di s t res s s yndrome (ARDS) [s ee VI]
o
o
b. Di ffus e pneumoni a (vi ral and bact eri al )
o
o
c. As pi rat i on pneumoni t i s
o
o
d. Fat embol i s m
o
o
e. Pul monary edema
2. T ype II: respiratory failure with carbon dioxide retention (i .e., l ow PaO 2 wi t h el evat ed PaCO 2 ). Type II res pi rat ory fai l ure, or ventilatory failure, has t wo bas i c phys i ol ogi c abnormal i t i es —[V wi t h dot above]/[Q wi t h dot above] i mbal ance and i nadequat e al veol ar vent i l at i on. Pat i ent s wi t h t ype II res pi rat ory fai l ure are di vi ded i nt o t wo cat egori es . o
o
a. Pat i ent s wi t h intrinsic lung disease charact eri zed by both [V with dot above]/[Q with dot above] imbalance and inadequate alveolar ventilation. Res pi rat ory fai l ure i s preci pi t at ed by addi t i onal cl i ni cal i ns ul t , us ual l y i nfect i on, whi ch wors ens t he phys i ol ogi c abnormal i t i es . Exampl es of s uch l ung di s eas es i ncl ude:
Pa g e 3 0 7
ABC Ambe r CHM Conve rte r Tria l ve rsion, http://w w w .proce sste x t.com/a bcchm.html
(1) COPD (chroni c bronchi t i s , emphys ema) and cys t i c fi bros i s
(2) Acut e obs t ruct i ve l ung di s eas e (as t hma, s evere acut e bronchi t i s )
o
o
b. Pat i ent s with intrinsically normal lungs but wi t h inadequate ventilation due t o:
(1) Di s orders of res pi rat ory cont rol [e.g., as a res ul t of drug overdos e, cent ral nervous s ys t em (CNS) di s eas e, t rauma, or cerebrovas cul ar acci dent (CVA)]
(2) Neuromus cul ar abnormal i t i es (e.g., pol i omyel i t i s , myas t heni a gravi s , Gui l l ai n Barré s yndrome)
(3) Ches t wal l t rauma, kyphos col i os i s , upper ai rway obs t ruct i on
C. Pathophysiologic mechanisms of hypoxemia
1. [V with dot above]/[Q with dot above] imbalance, t he mos t common pat hophys i ol ogi c caus e of hypoxemi a, ari s es when al veol ar vent i l at i on decreas es wi t h res pect t o perfus i on i n t he l ung. Hypoxemi a res ul t i ng from a moderat e al t erat i on i n t he [V wi t h dot above]/[Q wi t h
Pa g e 3 0 8
ABC Ambe r CHM Conve rte r Tria l ve rsion, http://w w w .proce sste x t.com/a bcchm.html
dot above] can be revers ed wi t h rel at i vel y s mal l i ncreas es i n t he i ns pi red oxygen concent rat i on (i .e., 24 %–40% i ns pi red oxygen).
2. Intrapulmonary shunting occurs when vent i l at i on approaches or reaches zero i n perfus ed areas (e.g., due t o col l aps ed or fl ui d-fi l l ed al veol i ), s o t hat venous bl ood i s s hunt ed di rect l y t o t he art eri al ci rcul at i on wi t hout fi rs t bei ng oxygenat ed. Hypoxemi a due t o a s hunt frequent l y cannot be correct ed, even wi t h 100% i ns pi red oxygen.
3. Hypoventilation with resulting hypercapnia may cont ri but e t o hypoxemi a. Thi s res ul t s i n t ype II res pi rat ory fai l ure.
4. An abnormality in the diffusion of oxygen across the alveolar–capillary membrane may cont ri but e t o hypoxemi a duri ng exerci s e or i n condi t i ons of l owered i ns pi red oxygen cont ent [mos t commonl y due t o hi gh al t i t ude, e.g., duri ng a commerci al ai rl i ne fl i ght ). However, t he cont ri but i on of t hi s mechani s m t o res pi rat ory fai l ure, i f any, i s i ns i gni fi cant .
D. Therapy
1. Principles. Treat ment i s di rect ed t oward t he underl yi ng di s eas e as wel l as t oward t he vent i l at ory and hypoxi c component s . In addi t i on, t he acut e and chroni c as pect s of res pi rat ory fai l ure mus t P.64
Pa g e 3 0 9
ABC Ambe r CHM Conve rte r Tria l ve rsion, http://w w w .proce sste x t.com/a bcchm.html
be cons i dered. Pat i ent s wi t h chroni c res pi rat ory fai l ure frequent l y can t ol erat e a l ower PaO 2 and a hi gher PaCO 2 t han t hos e wi t h acut e res pi rat ory fai l ure.
2. Oxygen therapy o
o
a. In type I respiratory failure, pat i ent s may be gi ven hi gh concent rat i ons of i ns pi red oxygen, becaus e carbon di oxi de ret ent i on i s not a ri s k. Oxygen may be del i vered by mas k or nas al cannul a. The us e of devi ces t o i ncreas e end-expi rat ory l ung vol umes , s uch as cont i nuous pos i t i ve ai rway pres s ure (CPAP) or, i n more s evere cas es , i nt ubat i on wi t h mechani cal vent i l at i on (s ee VI E 1 b), may be requi red.
o
o
b. In type II respiratory failure, t reat ment depends on t he caus e.
(1) W hen t he caus e i s an exacerbat i on of COPD, t he bas i s of t herapy i s cont rol l ed admi ni s t rat i on of oxygen (i .e., l ow-fl ow oxygen t reat ment ), wi t h care t aken not t o i ncreas e t he PaCO 2 . Mechani cal vent i l at i on may be needed. Noninvasive mask ventilation wi t h bi l evel pos i t i ve ai rway pres s ure (Bi PAP) or pres et vol ume vent i l at i on has been us ed wi t h s ome s ucces s , t hereby avoi di ng t he need for ai rway i nt ubat i on.
Pa g e 3 1 0
ABC Ambe r CHM Conve rte r Tria l ve rsion, http://w w w .proce sste x t.com/a bcchm.html
(2) Type II res pi rat ory fai l ure t hat ari s es from caus es ot her t han COPD us ual l y i s an i ndi cat i on for ei t her noni nvas i ve or i nvas i ve mechani cal vent i l at i on.
VI. Acute Respiratory Distress Syndrome (ARDS) A. Definition
1. Clinical definition. ARDS i s an i mport ant form of acut e hypoxemi c, hypocapni c (i .e., t ype I) res pi rat ory fai l ure charact eri zed by s evere dys pnea, hypoxi a, l os s of l ung compl i ance, and pul monary edema. The s ynonym “ wet lung― emphas i zes t he pres ence of i ncreas ed ext ravas cul ar l ung wat er (t he bas i c pat hophys i ol ogi c mechani s m underl yi ng t hi s condi t i on).
2. Physiologic definition o
o
a. The rat i o of PaO 2 t o FIO 2 i s ≤200, regardl es s of t he pres ence or l evel of pos i t i ve end-expi rat ory pres s ure (PEEP).
o
o
b. There i s a fi ndi ng of bi l at eral pul monary i nfi l t rat es .
o
o
c. There i s pul monary capi l l ary wedge pres s ure (PCW P) of ≤18 mm Hg or no cl i ni cal evi dence of el evat ed l eft at ri al pres s ure.
B. Etiology Pa g e 3 1 1
ABC Ambe r CHM Conve rte r Tria l ve rsion, http://w w w .proce sste x t.com/a bcchm.html
ARDS can be i ni t i at ed by many di fferent event s and condi t i ons , i ncl udi ng s hock, as pi rat i on of fl ui d, di s s emi nat ed i nt ravas cul ar coagul at i on (DIC), bact eri al s ept i cemi a, t rauma, bl ood t rans fus i on, pancreat i t i s , s moke i nhal at i on, and heroi n overdos e.
C. Pathogenesis and pathophysiology
1. An i ns ul t t o t he capi l l ary endot hel i um or al veol ar epi t hel i um preci pi t at es ARDS. Thi s i ns ul t res ul t s i n capi l l ary conges t i on and i nt ers t i t i al edema, l eadi ng t o di s rupt i on of capi l l ary i nt egri t y and t he ext ravas at i on of fl ui d, fi bri n, red bl ood cel l s (RBCs ), and whi t e bl ood cel l s (W BCs ) i nt o t he l ung i nt ers t i t i um, l ymphat i cs , and, ul t i mat el y, t he al veol i . o
o
a. Tumor necros i s fact or-α (TNF-α) and i nt erl euki n-1 (IL-1), “earl y res pons e― cyt oki nes , are cri t i cal for i ni t i at i ng t he i nfl ammat ory res pons e.
o
o
b. Thes e cyt oki nes t hen s t i mul at e IL-8, whi ch perpet uat es i nfl ammat i on and coagul at i on.
2. The s evere hypoxemi a i s caus ed by ext reme [V wi t h dot above]/[Q wi t h dot above] i mbal ance and t he s hunt i ng of bl ood i n t he fl ui d-fi l l ed areas of t he l ung.
3. The l ungs s t i ffen and become l es s compl i ant , res ul t i ng i n di ffi cul t y wi t h mechani cal vent i l at i on and s ubs equent hi gh peak ai rway pres s ures .
D. Clinical features and diagnosis
Pa g e 3 1 2
ABC Ambe r CHM Conve rte r Tria l ve rsion, http://w w w .proce sste x t.com/a bcchm.html
Sympt oms may devel op i mmedi at el y aft er t he i ns ul t but us ual l y are del ayed 24–48 hours .
1. Progres s i ve t achypnea us ual l y i s t he earl i es t s i gn, fol l owed by dys pnea.
2. Phys i cal fi ndi ngs oft en are abs ent or l i mi t ed t o bronchi al breat h s ounds and ral es .
3. Pul monary funct i on and bl ood gas s t udi es s how i ncreas ed mi nut e vent i l at i on, decreas ed l ung vol umes , and acut e res pi rat ory al kal os i s . P.65
4. Ches t radi ograph s hows pat chy, di ffus e bi l at eral , fl uffy i nfi l t rat es .
5. Cardi ac out put us ual l y i s i ncreas ed s omewhat , al t hough t ermi nal l y i t may decreas e and be accompani ed by met abol i c aci dos i s and t i s s ue hypoxi a.
E. Therapy
1. Oxygenation. The ul t i mat e goal of t herapy i s t o provi de adequat e t i s s ue oxygenat i on. Overal l t i s s ue oxygenat i on can be es t i mat ed from t he mi xed venous oxygen cont ent (CVO 2 ). In addi t i on, concomi t ant meas urement of cardi ac out put by t hermodi l ut i on may ai d i n t he correct i on of abnormal oxygen t rans port .
Pa g e 3 1 3
ABC Ambe r CHM Conve rte r Tria l ve rsion, http://w w w .proce sste x t.com/a bcchm.html o
o
a. Hypoxemi a can be correct ed by mai nt ai ni ng t he PaO 2 at approxi mat el y 60–80 mm Hg. Thi s res ul t s i n approxi mat el y a 90% oxygen s at urat i on, whi ch ens ures t hat t i s s ue oxygen needs are met as l ong as cardi ac out put and hemogl obi n l evel s are normal .
o
o
b. Mechanical ventilation i s requi red by mos t pat i ent s wi t h ARDS.
(1) PEEP commonl y i s us ed t o i ncreas e l ung vol ume (i .e., FRC), reduce i nt rapul monary s hunt , and i mprove [V wi t h dot above]/[Q wi t h dot above] rel at i ons hi ps . PEEP may caus e barot rauma or a reduced cardi ac out put . In pat i ent s whos e cardi ac out put i s compromi s ed, t he PaO 2 i ncreas es but oxygen del i very t o t he t i s s ues may decreas e. Therefore, i t i s i mport ant t o meas ure mi xed venous PaO 2 (MVO 2 ) when us i ng PEEP.
(2) A l arge mul t i cent ered Nat i onal Ins t i t ut es of Heal t h (NIH)–s upport ed t ri al s howed t hat t he us e of a l ow t i dal vol ume s t rat egy (e.g., 6 mL/kg) decreas ed mort al i t y compared wi t h one us i ng a l arger t i dal vol ume (12 mL/kg).
(3) Ot her met hods of vent i l at i on s uch as inverse-ratio, pressure release, and high-frequency ventilation may be us eful i n cert ai n s i t uat i ons .
Pa g e 3 1 4
ABC Ambe r CHM Conve rte r Tria l ve rsion, http://w w w .proce sste x t.com/a bcchm.html
(4) Some s t udi es have demons t rat ed t hat pl aci ng pat i ent s i n t he prone pos i t i on or us i ng i nhal ed ni t ri c oxi de may i mprove oxygenat i on, but nei t her t reat ment i mproves s urvi val .
2. Other measures. The underl yi ng di s eas e proces s mus t be t reat ed. In addi t i on, pat i ent s who requi re more t han 24–48 hours of mechani cal vent i l at i on s houl d recei ve nut ri t i onal s upport , preferabl y t hrough t he gas t roi nt es t i nal t ract .
3. Possible new treatments. Ot her novel t herapi es have been i nves t i gat ed for t reat ment of ARDS. None have s hown cons i s t ent , unequi vocal benefi t . They i ncl ude s urfact ant repl acement ; β-agoni s t s ; i nhal ed ni t ri c oxi de; cort i cos t eroi ds (gi ven aft er 3 days ); i buprofen; ket oconazol e (i nhi bi t i on of t hromboxane s ynt hes i s ); ant i endot oxi n ant i bodi es ; TNF-α ant i bodi es and IL-1 recept or ant agoni s t s ; and t herapi es l i mi t i ng fl ui d admi ni s t rat i on t o decreas e t he devel opment of ext ravas cul ar l ung wat er.
VII. Pulmonary Hypertension A. Definitions
1. Pul monary hypert ens i on i s a condi t i on charact eri zed by chroni c el evat i on of mean pul monary art ery pres s ure t o >25 mm Hg at res t , or >30 mm Hg wi t h exerci s e. The W orl d Heal t h Organi zat i on (W HO) has al s o recent l y defi ned pul monary hypert ens i on t o be pres ent when a s ys t ol i c pul monary art ery pres s ure i s >40 mm Hg, whi ch
Pa g e 3 1 5
ABC Ambe r CHM Conve rte r Tria l ve rsion, http://w w w .proce sste x t.com/a bcchm.html
corres ponds t o t ri cus pi d regurgi t at i on on a Doppl er echocardi ogram of 3–3.5 m/s ec.
2. Pul monary hypert ens i on i s furt her defi ned as t o whet her i t i s pri mary or s econdary i n ori gi n. o
o
a. Primary pulmonary hypertension (PPH) i s s us t ai ned el evat i on i n pul monary art ery pres s ure wi t hout an i dent i fi abl e caus e.
o
o
b. Secondary pulmonary hypertension i s s us t ai ned el evat i on of pul monary art ery pres s ure at t ri but abl e t o an underl yi ng caus e, s uch as emphys ema, parenchymal di s eas es as found i n s arcoi dos i s , i di opat hi c pul monary fi bros i s , human i mmunodefi ci ency vi rus (HIV) di s eas e, port al hypert ens i on, cocai ne i nhal at i on, or us e of appet i t e s uppres s ant drugs .
B. Incidence
1. True es t i mat es of t he i nci dence of pri mary or s econdary pul monary hypert ens i on are di ffi cul t t o obt ai n becaus e of t he vaguenes s of pat i ent s ympt oms , di agnos i s made rel at i vel y l at e i n t he pat i ent 's cours e of i l l nes s , and t he requi rement i n s ome cas es for i nvas i ve t es t s t o accurat el y di agnos e t he di s eas e. P.66
o
Pa g e 3 1 6
ABC Ambe r CHM Conve rte r Tria l ve rsion, http://w w w .proce sste x t.com/a bcchm.html
o
a. Current dat a s ugges t t hat t he i nci dence of PPH i n t he general popul at i on i s approxi mat el y 1–2 cas es per mi l l i on i ndi vi dual s .
o
o
b. The i nci dence of s econdary pul monary hypert ens i on i s even more di ffi cul t t o as cert ai n. In s ome di s orders , s uch as HIV-rel at ed di s eas e, approxi mat el y 2% of pat i ent s are found t o have as s oci at ed pul monary hypert ens i on. Approxi mat el y 0.5% of pat i ent s are found t o have port al hypert ens i on. In pat i ent s who us e appet i t e s uppres s ant drugs , t he l i kel i hood of devel opi ng pul monary hypert ens i on (odds rat i o, 6.3) was much great er i f t he pat i ent s us ed drugs for more t han 3 mont hs .
C. Pathogenesis
1. In pat i ent s wi t h PPH, t hree fact ors cont ri but e t o an el evat i on i n pul monary art ery pres s ure: vasoconstriction, remodeling of the vascular wall, and in situ thrombosis.
2. In pat i ent s wi t h s econdary pul monary hypert ens i on, t hes e t hree fact ors may coexi s t , i n conjunct i on wi t h a reduct i on i n pul monary capi l l ary bed s i ze (mas s i ve pl eural effus i ons wi t h vol ume overl oad, kyphos col i os i s ) or des t ruct i on of t he underl yi ng l ung parenchyma (advanced emphys ema, s arcoi dos i s , and chroni c nonres ol ved t hromboembol i c di s eas e).
[More i nformat i on onl i ne.]
3. Vas ocons t ri ct i on was fi rs t cons i dered t o be an i mport ant el ement i n t he pat hogenes i s of pul monary
Pa g e 3 1 7
ABC Ambe r CHM Conve rte r Tria l ve rsion, http://w w w .proce sste x t.com/a bcchm.html
hypert ens i on, becaus e vas odi l at i on was not ed i n res pons e t o admi ni s t rat i on of acet yl chol i ne. Moreover, evi dence of medi al hypert rophy of t he pul monary art ery wal l i ndi cat es vas ocons t ri ct i on i nduci ng art eri al s moot h mus cl e prol i ferat i on. Ot her el ement s cons i dered i mport ant i n t he pat hogenes i s of vas ocons t ri ct i on i ncl ude an i mbal ance of pros t acycl i n t o t hromboxane and i mpai red s ynt hes i s of endot hel i al -deri ved ni t ri c oxi de, whi ch act s as a l ocal vas orel axant . As vas ocons t ri ct i on remai ns s us t ai ned, i nt i mal and advent i t i al t i s s ue prol i ferat es .
4. Event ual l y, t hrombos i s may res ul t from i njury t o t he endot hel i um, abnormal fi bros i s , i ncreas ed procoagul ant act i vi t y, and underl yi ng pl at el et dys funct i on.
5. The hi s t opat hol ogy of pul monary hypert ens i on repres ent s a combi nat i on of pul monary art ery i njury and repai r. Al t hough s everal hi s t opat hol ogi c pat t erns are obs erved, none appears t o be pat hognomoni c for t he ent i t y. o
o
a. Pl exogeni c pul monary art eri opat hy i s t he mos t common l es i on; i t i s charact eri zed by medi al hypert rophy and fi brot i c i nt i mal l es i ons t hat may cont ai n organi zed t hrombi .
o
o
b. Thrombot i c pul monary art eri opat hy i s pres ent when organi zed mural t hrombi are s een, caus i ng i n s i t u t hrombos i s i n t he s et t i ng of an uni nvol ved art eri al wal l i n a pul monary art ery of normal di mens i ons .
o
Pa g e 3 1 8
ABC Ambe r CHM Conve rte r Tria l ve rsion, http://w w w .proce sste x t.com/a bcchm.html
o
c. Increas ed t hi cknes s of t he medi al s moot h wal l of t he pul monary art ery (e.g., i s ol at ed medi al hypert rophy) precedes t he format i on of pl exogeni c l es i ons and may be revers i bl e wi t h t reat ment .
D. Epidemiology
1. The mean age of pat i ent s devel opi ng PPH i s approxi mat el y 36 years , wi t h a s l i ght femal e t o mal e predomi nance (1.7 vs . 1.0).
2. No raci al predi l ect i on has ever been found.
3. Inves t i gat ors s pecul at e t hat underdi agnos i s and underreport i ng of pul monary hypert ens i on i s probabl y common and cont ri but es t o poor pat i ent out come by onl y di agnos i ng t he pat i ent duri ng t he l at er s t ages of t he di s eas e.
4. The Int ernat i onal Regi s t ry for Pri mary Pul monary Hypert ens i on report ed t hat 6% of t he enrol l ed pat i ent s had a fi rs t -degree rel at i ve wi t h t he di s eas e.
(More i nformat i on onl i ne.)
o
o
a. Fami l i al pul monary hypert ens i on i s i nheri t ed as an aut os omal domi nant t rai t , wi t h vari abl e but l ow penet rance.
o
o
b. Genet i c ant i ci pat i on (e.g., s ubs equent generat i ons devel op PPH at an earl i er age, and great er s everi t y) furt her affect s gene penet rance.
o
Pa g e 3 1 9
ABC Ambe r CHM Conve rte r Tria l ve rsion, http://w w w .proce sste x t.com/a bcchm.html
o
c. The gene for fami l i al PPH has been mat ched t o chromos ome 2q 31–32.
o
o
d. Furt hermore, t he bone morphogenet i c prot ei n recept or t ype II gene (BMP R2) has been i dent i fi ed as t he act ual gene for fami l i al pul monary hypert ens i on, and i t s product , t rans formi ng growt h fact or β-recept or, may be i mport ant i n modul at i ng pul monary vas cul ar t one.
o
o
e. In one s t udy, BMP R2 was found i n 26% of pat i ent s wi t h s poradi c, or nonfami l i al , PPH, s ugges t i ng t hat a s erol ogi c t es t may i dent i fy pat i ent s at ri s k for devel opi ng pul monary hypert ens i on.
E. Clinical Features Sympt oms as s oci at ed wi t h pul monary hypert ens i on are vague and nondes cri pt . Thi s res ul t s i n mos t pat i ent s havi ng s ympt oms for approxi mat el y 2 years before bei ng di agnos ed, al t hough 10% of pat i ent s have s evere s ympt oms for 3 years before t he di agnos i s i s act ual l y made.
1. Fat i gue appears t o be t he mos t common s ympt om and affect s approxi mat el y 60% of i ndi vi dual s .
2. Dys pnea occurs i n 60% of pat i ent s when t hey i ni t i al l y pres ent but i s encount ered by al l as t he di s eas e progres s es .
3. Syncope and angi na, part i cul arl y wi t h exert i on, are
Pa g e 3 2 0
ABC Ambe r CHM Conve rte r Tria l ve rsion, http://w w w .proce sste x t.com/a bcchm.html
l at e mani fes t at i ons of t he di s eas e, s ugges t i ng t he pres ence of s evere pul monary hypert ens i on caus i ng reduced cardi ac out put .
F. Diagnosis
1. PPH i s a di agnos i s of excl us i on. For t hi s reas on, pul monary funct i on t es t s , el ect rocardi ogram (ECG), ches t radi ograph, echocardi ogram, and vent i l at i on–perfus i on s can are t he i mport ant i ni t i al di agnos t i c t es t s when pul monary hypert ens i on i s s ugges t ed. o
o
a. Pul monary funct i on s t udi es are i mport ant t o eval uat e for t he pres ence of ot her caus es cont ri but i ng t o t he devel opment of pul monary hypert ens i on. In pat i ent s wi t hout underl yi ng parenchymal l ung di s eas e who have PPH, t here are general l y mi l d res t ri ct i ve changes but t here i s no evi dence of obs t ruct i on. The di ffus i ng capaci t y for carbon monoxi de (D LC O ) mi ght be s l i ght l y reduced.
o
o
b. The ECG may i ndi cat e ri ght vent ri cul ar enl argement or s how ri ght vent ri cul ar s t rai n.
o
o
c. A ches t radi ograph may s how enl argement of t he pul monary art eri es .
o
o
d. An echocardi ogram can es t i mat e pul monary art ery pres s ures , s how evi dence of t ri cus pi d regurgi t at i on, es t i mat e t he cont ract i l e s t at e of t he ri ght vent ri cul ar i n res pons e t o an el evat i on i n
Pa g e 3 2 1
ABC Ambe r CHM Conve rte r Tria l ve rsion, http://w w w .proce sste x t.com/a bcchm.html
ri ght -s i ded pres s ures , and al s o rul e out congeni t al val vul ar and l eft vent ri cul ar myocardi al di s eas e. o
o
e. Vent i l at i on–perfus i on l ung i magi ng i s i mport ant t o excl ude chroni c unres ol ved t hromboembol i c di s eas e. In pat i ent s wi t h PPH, i t may s how a peri pheral nons egment al pat chy was hout of radi oact i ve t racer.
P.67
2. In s el ect ed cas es , s l eep s t udy, ches t CT wi t h and wi t hout cont ras t , s erol ogi c s t udi es , and ri ght -heart cat het eri zat i on may be requi red. o
o
a. Ches t CT wi t h i nt ravenous cont ras t al s o may be us eful i n excl udi ng l arge cent ral art ery cl ot or confi rmi ng a caus e of s econdary pul monary hypert ens i on (e.g., i nt ers t i t i al l ung di s eas e, emphys ema).
o
o
b. Serol ogi c s t udi es are performed t o s creen for connect i ve t i s s ue di s eas es . A pos i t i ve ant i nucl ear ant i body t es t i s common i n t he s et t i ng of PPH; however, hi gher t i t ers or s peci fi c ant i body pat t erns s houl d rai s e t he i ndex of s us pi ci on for underl yi ng col l agen vas cul ar di s eas e t hat may i mport ant l y cont ri but e t o t he pat i ent 's pres ent at i on and prognos i s .
o
o
c. Ri ght -heart cat het eri zat i on i s cri t i cal i n t he
Pa g e 3 2 2
ABC Ambe r CHM Conve rte r Tria l ve rsion, http://w w w .proce sste x t.com/a bcchm.html
eval uat i on of pat i ent s wi t h s us pect ed PPH t o confi rm t he pres ence of t he di s eas e, t o as s es s t he magni t ude of hypert ens i on, and al s o t o eval uat e t he res pons e of t he pat i ent t o pharmacol ogi c t reat ment . It al s o may be us ed as a gui de i n s el ect cas es t o hel p deci de whet her s i ngl e- or doubl e-l ung t rans pl ant at i on i s requi red i n pat i ent s whos e condi t i on i s not res pondi ng t o pharmacol ogi c t herapy. o
o
d. W het her or not obs t ruct i ve s l eep apnea (OSA) caus es pul monary hypert ens i on has been debat ed. In pat i ent s wi t h pul monary hypert ens i on, a s l eep s t udy s houl d be done before a di agnos i s of PPH i s gi ven.
G. Prognosis
1. Hi s t ori cal l y, t he prognos i s for pat i ent s wi t h PPH has been poor, wi t h an es t i mat ed medi an s urvi val of 2.8 years , as not ed i n t he NIH regi s t ry report i ng on t he us e of convent i onal t reat ment . In t hes e regi s t ry dat a, s urvi val rat es at 1, 3, and 5 years were 68%, 48%, and 34%, res pect i vel y.
2. Newer dat a s ugges t t hat ant i coagul at i on may doubl e t he 3-year s urvi val rat e.
3. In pat i ent s whos e condi t i ons res ponds t o cal ci um channel bl ockers , t he 5-year s urvi val rat e approxi mat es 95%.
Pa g e 3 2 3
ABC Ambe r CHM Conve rte r Tria l ve rsion, http://w w w .proce sste x t.com/a bcchm.html
4. Fi ve-year s urvi val rat es i n pat i ent s wi t h New Y ork Heart As s oci at i on (NY HA) cl as s i fi cat i ons III and IV who are t reat ed wi t h epopros t enol are equal t o t hos e of mat ched cont rol s .
H. Therapy
1. Convent i onal medi cal t herapy for pul monary hypert ens i on has cent ered around t he us e of di uret i cs , di goxi n, ant i coagul ant s , and vas odi l at ors
2. Di uret i cs are frequent l y us ed t o t reat exces s i ve edema t hat compromi s es t he pat i ent 's condi t i on, es peci al l y when hepat i c conges t i on and as ci t es are pres ent . However, di uret i cs mus t be us ed wi t h caut i on i n pat i ent s wi t h hi gher ri ght -s i ded pres s ures t o avoi d s i gni fi cant reduct i ons i n cardi ac prel oad t hat coul d furt her reduce cardi ac out put .
3. Dat a demons t rat i ng t he effect i venes s of ant i coagul at i on are l i mi t ed. However, bas ed on t he poor prognos i s and evi dence of i n s i t u t hrombos i s i n pat i ent s wi t h pul monary hypert ens i on, ant i coagul at i on i s recommended t o achi eve an i nt ernat i onal normal i zed rat i o (INR) of 2–2.5 t i mes t hat of normal cont rol s .
4. Di goxi n i s recommended by s ome expert s ; however, i t i s s omewhat cont rovers i al . It i s al ways recommended for us e i n pat i ent s who have pul monary hypert ens i on wi t h evi dence of l eft vent ri cul ar dys funct i on. However, i t s rol e i n pat i ent s wi t h i s ol at ed ri ght vent ri cul ar dys funct i on s econdary t o pul monary hypert ens i on i n t he
Pa g e 3 2 4
ABC Ambe r CHM Conve rte r Tria l ve rsion, http://w w w .proce sste x t.com/a bcchm.html
abs ence of l eft vent ri cul ar dys funct i on i s uncl ear.
5. Vas odi l at ors are i mport ant i n t he t reat ment of pat i ent s wi t h PPH. The rat i onal e for t he us e of vas odi l at ors i s bas ed on t he premi s e t hat vas ocons t ri ct i on i s a promi nent feat ure of t he di s eas e. o
o
a. An acut e res pons e t o a vas odi l at or (adenos i ne, ni t ri c oxi de, or epopros t enol ) i s pres ent i n approxi mat el y 10% of pat i ent s wi t h PPH and may port end a bet t er prognos i s and i ndi cat e whi ch pat i ent s may res pond t o hi gh-dos e cal ci um channel bl ockers .
o
o
b. Cal ci um channel bl ockers us ed i ncl ude ni fedi pi ne (dos age range, 30–240 mg/day) and di l t i azem (120–900 mg/day).
o
o
c. Caut i on needs t o be exhi bi t ed when us i ng t hes e agent s becaus e t hey have durat i ons of 2–4.5 hours each, and may preci pi t ous l y reduce cardi ac out put .
o
o
d. Si de effect s i ncl ude s ys t emi c hypot ens i on, edema, and hypoxemi a, al l of whi ch need t o be i dent i fi ed under cl os e admi ni s t rat i on i n a s t ruct ured s et t i ng by experi enced cl i ni ci ans .
P.68
Pa g e 3 2 5
ABC Ambe r CHM Conve rte r Tria l ve rsion, http://w w w .proce sste x t.com/a bcchm.html
6. Thos e pat i ent s whos e condi t i on fai l s t o s how an i ni t i al favorabl e res pons e t o s hort -act i ng agent s may s how l ong-t erm res pons e t o epopros t enol , s ugges t i ng t hat fact ors ot her t han vas ocons t ri ct i on may be i mport ant i n t he pat hogenes i s of pul monary hypert ens i on, or t hat epopros t enol has an effect ot her t han bei ng a vas odi l at or, s uch as ant i embol i c or ant i -i nfl ammat ory effect s . o
o
a. Epopros t enol mus t be gi ven by cont i nuous i nt ravenous i nfus i on, becaus e i t has a s hort hal f-l i fe. It cannot be gi ven oral l y becaus e i t i s i nact i vat ed by t he aci di c envi ronment of t he s t omach.
o
o
b. It i s admi ni s t ered cont i nuous l y and i nt ravenous l y by a port abl e bat t ery-operat ed pump connect ed t o a permanent i ndwel l i ng cent ral venous cat het er. In a 3-mont h pros pect i ve, randomi zed t ri al , epopros t enol reduced pul monary art ery pres s ure, i ncreas ed cardi ac out put , and i mproved pat i ent s ' exerci s e t ol erance, qual i t y of l i fe, and s urvi val , es peci al l y i n t hos e who had NY HA cl as s III and IV s ympt oms . In addi t i on, benefi ci al l ong-t erm hemodynami c res pons es have been report ed.
o
o
c. The major s i de effect s of epopros t enol are rel at ed t o t he compl exi t i es i mpos ed by t he us e of a cont i nuous devi ce and i ndwel l i ng l i ne. Ot her s i de effect s i ncl ude jaw pai n, fl us hi ng, headache, di arrhea, art hral gi as , and cardi ac arrhyt hmi a.
o
Pa g e 3 2 6
ABC Ambe r CHM Conve rte r Tria l ve rsion, http://w w w .proce sste x t.com/a bcchm.html
o
d. Oral (berapros t ) and i nhal ed (i l opros t ) anal ogs of pros t acycl i n are bei ng s t udi ed i n an at t empt t o avoi d t he compl i cat i ons of cont i nuous admi ni s t rat i on whi l e mai nt ai ni ng t he hemodynami c and s urvi val benefi t s . Trepros t i ni l i s an anal og of pros t acycl i n t hat needs t o be admi ni s t ered cont i nuous l y, but s ubcut aneous l y. However, pai n and di s comfort at t he i nject i on s i t e appear t o s i gni fi cant l y l i mi t t he broad appl i cat i on of t hi s drug.
o
o
e. A new cl as s of oral agent s , endot hel i n ant agoni s t s , s hows promi s e i n promot i ng pul monary art ery vas odi l at i on and decreas i ng s moot h mus cl e prol i ferat i on. The abnormal i t y of great es t concern as s oci at ed wi t h bos ent an us e i s a mi l d but revers i bl e el evat i on i n l i ver enzymes , whi ch appears t o res pond t o s t oppi ng t he drug.
7. In pat i ent s i n whom pul monary hypert ens i on remai ns unaffect ed by t he us e of t he agent s des cri bed previ ous l y, l ung t rans pl ant at i on i s a cons i derat i on. Lung t rans pl ant at i on carri es wi t h i t an approxi mat el y 50% 5-year mort al i t y, i t i s expens i ve, and i t has i ncreas ed morbi di t y rel at ed t o t he drugs us ed for i mmune s uppres s i on, as wel l as t he t rans pl ant at i on procedure i t s el f. However, i n s el ect ed ci rcums t ances , i t i s an i mport ant benefi ci al t herapy.
VIII. Pulmonary Embolism A. Definition In pul monary embol i s m, a t hrombus ari s es el s ewhere i n t he body and mi grat es t o t he pul monary vas cul ar t ree, where i t caus es
Pa g e 3 2 7
ABC Ambe r CHM Conve rte r Tria l ve rsion, http://w w w .proce sste x t.com/a bcchm.html
obs t ruct i on. Nearl y al l pul monary embol i deri ve from deep venous t hrombos i s (s ee Chapt er 1 VIII A). Rarel y (e.g., i n s i ckl e cel l di s eas e), pul monary art eri al t hrombos i s occurs as a pri mary event wi t hout di s cerni bl e cl ot s el s ewhere.
B. Incidence Acut e pul monary embol i s m i s a major caus e of morbi di t y and mort al i t y i n t he Uni t ed St at es . Ful l y 50% of cas es of deep venous t hrombos i s are compl i cat ed by pul monary embol i s m. Each year, as many as 650,000 i ndi vi dual s s us t ai n pul monary embol i , an es t i mat ed 150,000 of whom di e as a res ul t . (Thi s mort al i t y fi gure has not changed duri ng t he l as t 25 years .)
C. Etiology
1. Site of thrombus formation. St as i s i n t he iliofemoral venous system wi t h s ubs equent deep venous thrombosis i s t he mos t common precurs or of pul monary embol i s m. Ot her common s i t es of t hrombus format i on i ncl ude t he prostatic and pelvic veins. Except i n drug abus ers , or t hos e pat i ent s wi t h i ndwel l i ng cat het ers , pul monary embol i general l y do not ori gi nat e i n t he upper ext remi t i es .
2. Predisposing factors. Condi t i ons t hat i ncreas e a pat i ent 's ri s k of venous t hrombos i s , and, t herefore, pul monary embol i s m, are di s cus s ed i n Chapt er 1 VIII A 2.
3. Precipitating factors. The fact ors t hat cont rol t he t et heri ng of a t hrombus t o t he wal l of a vei n and t he di s l odgi ng of a t hrombus i nt o t he ci rcul at i on are not wel l unders t ood. However, exerci s e and s t rai ni ng at defecat i on, wi t h cons equent changes i n venous fl ow and
Pa g e 3 2 8
ABC Ambe r CHM Conve rte r Tria l ve rsion, http://w w w .proce sste x t.com/a bcchm.html
pres s ure, are wel l -known preci pi t at i ng event s . P.69
D. Pathophysiology
1. Embolization. W hen a t hrombus breaks off from i t s s i t e of ori gi n, i t i s carri ed t hrough t he i nferi or vena cava and ri ght vent ri cl e t o t he pul monary art eri es , where i t l odges . Pul monary embol i may be s i ngl e or mul t i pl e and vary i n s i ze from mi cros copi c part i cl es t o l arge s addl e embol i t hat compl et el y bl ock t he major branches of t he pul monary art ery.
2. Hemodynamic consequences o
o
a. Obs t ruct i on of t he pul monary art eri es by t he embol us i ncreas es res i s t ance t o bl ood fl ow t hrough t he pul monary ci rcui t and i ncreas es ri ght vent ri cul ar aft erl oad. W hen more t han 50%–60% of t he pul monary perfus i on i s i mpeded, s evere pul monary hypert ens i on, ri ght vent ri cul ar s t rai n, and cardi ac fai l ure ens ue.
o
o
b. The embol i s m al s o may caus e i nt rapul monary refl exes and t he rel eas e of humoral s ubs t ances (e.g., hi s t ami ne, s erot oni n, pros t agl andi ns ), l eadi ng t o vas ocons t ri ct i on t hroughout t he l ungs . Thi s vas ocons t ri ct i ve effect furt her i ncreas es pul monary vas cul ar res i s t ance and t he work of t he ri ght vent ri cl e.
Pa g e 3 2 9
ABC Ambe r CHM Conve rte r Tria l ve rsion, http://w w w .proce sste x t.com/a bcchm.html o
o
c. Fewer t han 10% of cas es of pul monary embol i s m progres s t o pulmonary infarction becaus e t he l ung parenchyma has t hree s ources of oxygen (i .e., t he ai rways , t he bronchi al ci rcul at i on, and t he pul monary ci rcul at i on).
o
o
d. Recurrent pul monary embol i progres s i vel y occl ude t he pul monary vas cul ar bed and l ead t o chroni c progres s i ve pul monary hypert ens i on and, ul t i mat el y, cor pulmonale.
3. Pulmonary consequences o
o
a. The pri mary pul monary cons equence of an embol i s m i s [V with dot above]/[Q with dot above] mismatch.
(1) “was t ed― vent i l at i on (“dead s pace―) occurs i n t he l ung s egment s where t he vas cul ar s uppl y i s obs t ruct ed and perfus i on cannot occur.
(2) Convers el y, overperfus i on and di mi ni s hed vas cul ar res i s t ance i n ot her l ung s egment s caus e profound ri ght -t o-l eft i nt rapul monary s hunt i ng, wi t h i nadequat e oxygenat i on of a l arge port i on of perfus ed bl ood.
o
o
b. Ot her pul monary res pons es i ncl ude conges t i ve
Pa g e 3 3 0
ABC Ambe r CHM Conve rte r Tria l ve rsion, http://w w w .proce sste x t.com/a bcchm.html
at el ect as i s of t he i s chemi c s egment of t he l ung, refl ex bronchi ol ar and vas cul ar cons t ri ct i on, and t he l os s or mal funct i on of al veol ar s urfact ant .
E. Clinical features and diagnosis
1. Symptoms. Pat i ent s may compl ai n of dys pnea at res t and ches t pai n res embl i ng t hat of myocardi al i nfarct i on (MI). Syncope may occur as cardi ac out put decl i nes . Pul monary i nfarct i on may caus e pl euri t i c pai n and hemopt ys i s .
2. Physical examination o
o
a. Nearl y al l pat i ent s wi t h pul monary embol i s m have t achypnea and t achycardi a, and many have a l ow-grade fever.
o
o
b. In massive pulmonary embolism, t he s evere phys i ol ogi c cons equences are mani fes t ed as cyanos i s , peri pheral venous engorgement , and hepat i c conges t i on; t here may be evi dence of cor pul monal e. In submassive pulmonary embolism, t he l es s profound hemodynami c changes may be t rans i ent , s o t hat hypot ens i on, t achycardi a, and hypoxi a may not be obs erved at t he t i me of i ni t i al exami nat i on.
o
o
c. There i s dul l nes s t o percus s i on and decreas ed breat h s ounds over t he i nvol ved area of t he l ung. Fi ndi ngs are us ual l y normal over t he remai ni ng l ung s egment s . Occas i onal l y, a pl eural fri ct i on rub or
Pa g e 3 3 1
ABC Ambe r CHM Conve rte r Tria l ve rsion, http://w w w .proce sste x t.com/a bcchm.html
wheez i ng i s heard. o
o
d. Evi dence of deep venous t hrombos i s i s s een i n 50% of pat i ent s .
3. Chest radiography. Res ul t s are normal i n mos t pat i ent s . A few s how pl at e-l i ke at el ect as i s , a uni l at eral l y hi gh di aphragm, or a s mal l pl eural effus i on. Occas i onal fi ndi ngs i ncl ude a bul gi ng pul monary art ery and a l arge ol i gemi c l ung s egment . A wedge-s haped, pl eural -bas ed dens i t y i s t ypi cal of a pul monary i nfarct i on.
4. Electrocardiography. The ECG us ual l y i s not s peci fi c but may hel p di fferent i at e bet ween MI and pul monary embol i s m. o
o
a. The mos t common fi ndi ng i s s i nus t achycardi a wi t h or wi t hout premat ure at ri al and vent ri cul ar cont ract i ons . The mean P axi s commonl y s hi ft s t o t he ri ght when ri ght -s i ded pul monary obs t ruct i on i s s evere and t he S wave i n l ead I and t he Q and T waves i n l ead III are abnormal . P.70
o
o
b. Ri ght vent ri cul ar s t rai n may produce i nt ermi t t ent ri ght bundl e branch bl ock, P pul monal e (i .e., “peaked― P waves ), and marked cl ockwi s e rot at i on of t he ECG.
Pa g e 3 3 2
ABC Ambe r CHM Conve rte r Tria l ve rsion, http://w w w .proce sste x t.com/a bcchm.html
5. Blood gas analysis o
o
a. Hypoxemia (i ndi cat ed by a decreas e i n PaO 2 ), hypervent i l at i on (i ndi cat ed by a decreas e i n PaCO 2 ), and a mi l d acut e res pi rat ory al kal os i s (i ndi cat ed by a l ow PaCO 2 and s l i ght l y el evat ed pH) are t he cl as s i c changes s een i n pat i ent s wi t h pul monary embol i s m. However, t hey are nonspecific for pul monary embol i s m, and pul monary embol i s m may occur wi t hout t hes e changes .
o
o
b. A more s ens i t i ve i ndi cat or of abnormal gas exchange i s t he A–a DO 2 ; a normal gradi ent es s ent i al l y rul es out pul monary embol i s m.
6. Pulmonary radioisotope scanning o
o
a. In perfusion scanning, t he pat i ent 's bl ood i s l abel ed wi t h a radi oact i ve t racer. Poorl y perfus ed areas of t he l ung appear as rel at i vel y i nact i ve areas on t he s can. However, even t hough normal res ul t s vi rt ual l y excl ude t he di agnos i s , t he t es t i s not s peci fi c for pul monary embol i s m, becaus e pneumoni a and COPD al s o can produce s canni ng abnormal i t i es .
o
o
b. In xenon ventilation scanning, t he pat i ent i nhal es t he radi oact i ve t racer. Thi s procedure oft en i s performed i n conjunct i on wi t h perfus i on s canni ng t o i ncreas e t he s peci fi ci t y of t he t es t . Fi ndi ng wel l -vent i l at ed but poorl y perfus ed areas s ugges t s
Pa g e 3 3 3
ABC Ambe r CHM Conve rte r Tria l ve rsion, http://w w w .proce sste x t.com/a bcchm.html
pul monary embol i s m; fi ndi ng areas wi t h bot h perfus i on and vent i l at i on defect s s ugges t s parenchymal l ung di s eas e rat her t han pul monary embol i s m.
7. Spi ral (hel i cal ) ches t comput ed t omography wi t h i nt ravenous cont ras t i s anot her t echni que t o di agnos e pul monary embol i s m. Thi s t echni que i nvol ves cont i nuous movement of t he pat i ent t hrough t he s canner wi t h mul t i pl e rapi d s cans of t he t horax duri ng a s i ngl e breat h (Fi gure 2-11). Several s t udi es have report ed great er t han 95% s ens i t i vi t y and s peci fi ci t y t o di agnos e pul monary embol i s m; ot hers have report ed l ower val ues . Spi ral CT i s mos t s ens i t i ve for t hromboembol i s m i nvol vi ng t he mai n, l obar, and s egment al art eri es . Li mi t at i ons i ncl ude poor vi s ual i zat i on of t he peri pheral ves s el s i n t he upper and l ower l obes , and hori zont al l y ori ent ed ves s el s i n t he ri ght mi ddl e l obe and l i ngul a.
8. Pulmonary angiography i s t he t i me-honored gol d s t andard t es t for t he di agnos i s of pul monary embol i s m. It i s unequi vocal l y di agnos t i c i f embol i are vi s ual i zed. Al t hough i nvas i ve, t hi s t es t s houl d be performed when vent i l at i on and perfus i on s cans are equi vocal and al s o when t he ri s ks from l ong-t erm ant i coagul at i on t herapy are great er t han us ual .
9. W hen t he di agnos i s of pul monary embol i s m remai ns i n ques t i on des pi t e many di agnos t i c s t udi es , corroborative evidence of venous di s eas e i n t he l ower ext remi t i es s houl d be s ought . o
o
a. Noni nvas i ve met hods i ncl ude i mpedance
Pa g e 3 3 4
ABC Ambe r CHM Conve rte r Tria l ve rsion, http://w w w .proce sste x t.com/a bcchm.html
pl et hys mography, l eg and t hi gh s canni ng wi t h radi oi odi nat ed fi bri nogen, and Doppl er ul t ras onography. However, t hes e t echni ques can produce fal s e-pos i t i ve res ul t s , and t hey cannot be us ed t o eval uat e pel vi c vei ns .
FIGURE 2-11. Spi ral comput ed t omograph of t he ches t wi t h cont ras t s howi ng l arge cl ot (bl ack arrow) obs t ruct i ng ri ght mai n pul monary art ery.
P.71
o
o
b. Cont ras t venography i s hel pful i n di agnos i ng occl us i on of t he pel vi c, t hi gh, and l eg vei ns . Thi s i nvas i ve t echni que i s not wi t hout morbi di t y; however; i t has been as s oci at ed wi t h phl ebi t i s , hypers ens i t i vi t y react i ons , and l ocal pai n.
o
o
c. D-di mer i s a degradat i on product rel eas ed duri ng endogenous fi bri nol ys i s . The negat i ve predi ct i ve val ue of D-di mer has been report ed t o be 97%–98% i n s everal s t udi es . D-di mer t es t i ng i s compl i cat ed by l i mi t at i ons i n as s ay performance. D-di mer i s not cons i dered a s t andard t es t for pul monary embol i s m.
Pa g e 3 3 5
ABC Ambe r CHM Conve rte r Tria l ve rsion, http://w w w .proce sste x t.com/a bcchm.html
F. Therapy
1. Anticoagulants o
o
a. Unl es s cont rai ndi cat ed, t he drug of choi ce for document ed or s us pect ed pul monary embol i s m i s heparin, gi ven i n dos es t hat mai nt ai n t he part i al t hrombopl as t i n t i me (PTT) at 2 t o 212 t i mes normal . It i s preferabl y gi ven by cont i nuous i nt ravenous admi ni s t rat i on.
o
o
b. Low–molecular-weight heparin (LMW H) i s a t ype of fract i onat ed nat ural hepari n wi t h a mol ecul ar wei ght of 200–700 dal t ons (15,000 for s t andard hepari n). It i s gi ven s ubcut aneous l y. It i s an al t ernat i ve t o unfract i onat ed hepari n and may enabl e pat i ent s t o be di s charged earl i er, or i n s ome cas es , t o recei ve onl y out pat i ent t herapy.
(1) LMW H has a l onger hal f-l i fe t han nat ural hepari n.
(2) LMW H has recent l y demons t rat ed effi cacy i n t he prophyl axi s and management of venous t hrombos i s .
o
o
c. Aft er ant i coagul at i on wi t h hepari n or LMW H has been achi eved for a few days , t herapy i s changed t o warfarin, gi ven oral l y, i n dos es t hat mai nt ai n t he prot hrombi n t i me (PT) at 2–2.5 t i mes t he
Pa g e 3 3 6
ABC Ambe r CHM Conve rte r Tria l ve rsion, http://w w w .proce sste x t.com/a bcchm.html
normal val ue or t he INR at 2–3. The oral ant i coagul ant i s cont i nued for 3–6 mont hs .
2. Oxygen i s gi ven rout i nel y t o correct hypoxemi a, and bed rest ordi nari l y i s pres cri bed unt i l t he dys pnea and pai n res ol ve, aft er whi ch pat i ent s may ambul at e whi l e remai ni ng on ant i coagul ant t herapy.
3. T hrombolytic agents. Fi bri nol yt i c agent s are gi ven when rapi d l ys i s of cl ot s i s i mport ant . o
o
a. Becaus e t hrombol yt i c agent s i ncreas e t he ri s k of hemorrhage, t hey are res erved for us e when occl us i on has produced ri ght -s i ded heart fai l ure and hemodynami c i ns t abi l i t y.
o
o
b. Thrombol yt i c agent s appear t o provi de l ong-t erm phys i ol ogi c i mprovement i n t he pul monary vas cul ar bed.
4. Surgery o
o
a. Inferi or vena caval l i gat i on, cl i ppi ng, and pl i cat i on and percut aneous “umbrel l a― i ns ert i on, are s urgi cal remedi es wi t h modes t morbi di t y t hat precl ude embol i c recurrence for a s hort peri od.
(1) Col l at eral venous channel s i n t he pel vi s or l ower abdomen may devel op i n pat i ent s
Pa g e 3 3 7
ABC Ambe r CHM Conve rte r Tria l ve rsion, http://w w w .proce sste x t.com/a bcchm.html
wi t h chroni c vena caval obs t ruct i on and may fi nd rout es around t he obs t ruct i on t o t he pul monary art ery, event ual l y negat i ng t he effect i venes s of t he s urgi cal procedure.
(2) Vena caval occl us i on s houl d be res erved for pat i ent s i n whom embol i s m recurs , des pi t e adequat e ant i coagul at i on t herapy, and for pat i ent s i n whom ant i coagul ant s are cont rai ndi cat ed (e.g., pat i ent s wi t h act i ve bl eedi ng).
o
o
b. Embol ect omy remai ns an al t ernat i ve t reat ment for pat i ent s who cannot mai nt ai n effect i ve cardi ac out put . Occas i onal l y, embol ect omy i s l i fes avi ng, but t he overal l s urvi val rat e i s approxi mat el y 10%.
5. Prophylactic “ minidose― heparin therapy. Gi vi ng s mal l dos es of hepari n prophyl act i cal l y has been hel pful i n prevent i ng pul monary embol i s m i n cert ai n hi gh-ri s k pat i ent s , wi t h l i t t l e ri s k of hemorrhage. o
o
a. “Mi ni dos e― hepari n prophyl axi s i s part i cul arl y appl i cabl e for ol der pat i ent s who undergo l ower abdomi nal or pel vi c s urgery and who are on bed res t pos t operat i vel y. It al s o i s hel pful for obes e s urgi cal pat i ent s . The cus t omary regi men i s t o gi ve s ubcut aneous hepari n 2 hours before s urgery and t o cont i nue i t aft er s urgery unt i l t he pat i ent i s ambul at ory.
o
o
b. It al s o i s recommended for pat i ent s on
Pa g e 3 3 8
ABC Ambe r CHM Conve rte r Tria l ve rsion, http://w w w .proce sste x t.com/a bcchm.html
prol onged bed res t becaus e of s t roke, MI, cardi ac fai l ure, or cancer. o
o
c. “Mi ni dos e― hepari n has not been s hown t o be effect i ve for major ort hopedi c, pros t at i c, ocul ar, or neuros urgi cal procedures .
P.72
IX. Diseases of the Pleura A. Pleural effusion
1. Definition. A pl eural effus i on i s an abnormal accumul at i on of fl ui d i n t he pl eural s pace.
2. Etiology and pathogenesis o
o
a. In heal t hy pat i ent s , t he pl eural cavi t y cont ai ns a s mal l vol ume of l ubri cat i ng s erous fl ui d, formed pri mari l y by t rans udat i on from t he pari et al pl eura and abs orbed pri mari l y by t he capi l l ari es and l ymphat i cs . The bal ance bet ween format i on and removal of t hi s fl ui d may be compromi s ed by any di s order t hat i ncreas es t he pul monary or s ys t emi c venous pres s ure, l owers t he pl as ma oncot i c pres s ure, i ncreas es capi l l ary permeabi l i t y, or obs t ruct s t he l ymphat i c ci rcul at i on.
o
o
b. A pl eural effus i on may be a t rans udat e or an exudat e (Tabl e 2-4; s ee IX A 4 a).
Pa g e 3 3 9
ABC Ambe r CHM Conve rte r Tria l ve rsion, http://w w w .proce sste x t.com/a bcchm.html
(1) T ransudates are caus ed by el evat ed venous pres s ure or by decreas ed pl as ma oncot i c pres s ure; t he pri mary pat hol ogi c proces s does not di rect l y i nvol ve t he pl eural s urface.
(2) Exudates are caus ed by i ncreas ed permeabi l i t y of t he pl eural s urface (due t o i nfl ammat i on, t rauma, or di s eas e) or by obs t ruct i on of t he l ymphat i cs .
o
o
c. A pl eural effus i on may have a noni nfl ammat ory or an i nfl ammat ory caus e.
(1) Noninflammatory pleural effusions may occur i n any condi t i on t hat caus es as ci t es , obs t ruct i on of t he venous or l ymphat i c out fl ow from t he t horax, i s ol at ed l eft - or ri ght -s i ded conges t i ve heart fai l ure (CHF), or a s evere reduct i on i n t he pl as ma prot ei n concent rat i on.
(2) Inflammatory pleural effusions res ul t from i nfl ammat i on of s t ruct ures adjacent t o t he pl eural s urface.
(a) The s i t e of i nfl ammat i on us ual l y i s jus t beneat h t he vi s ceral pl eura wi t hi n t he l ung but occas i onal l y i s wi t hi n t he medi as t i num, di aphragm, or ches t wal l . Secondary i nfl ammat i on of l arger areas
Pa g e 3 4 0
ABC Ambe r CHM Conve rte r Tria l ve rsion, http://w w w .proce sste x t.com/a bcchm.html
of t he pl eural s urface may res ul t i n rapi d out pouri ng of exudat e.
(b) Removal of t he fl ui d by t he normal cl eari ng mechani s ms may be cons i derabl y ret arded by i nfl ammat ory obs t ruct i on of t he l ymphat i cs t hat drai n t he t horax.
3. Clinical features and diagnosis o
o
a. Symptoms res ul t from i nfl ammat i on of t he pari et al pl eura and compres s i on of t he l ung.
(1) Pleuritic pain occurs mos t commonl y wi t h i nfl ammat ory effus i ons and oft en i s accompani ed by a fri ct i on rub.
(a) The pai n commonl y i s a s harp, s t abbi ng s ens at i on t hat i s mi ni mal duri ng qui et res pi rat i on but i nt ens i fi es abrupt l y duri ng ful l i nfl at i on of t he l ungs .
TABLE 2-4 Causes of Pleural Effusions
Pa g e 3 4 1
ABC Ambe r CHM Conve rte r Tria l ve rsion, http://w w w .proce sste x t.com/a bcchm.html
Ca us es of ex ud ate s Mal i gn anc y (e. g., bro nch og eni c car ci n om a, l ym ph om a, me t as t at ic tu mo r) Infl
Pa g e 3 4 2
ABC Ambe r CHM Conve rte r Tria l ve rsion, http://w w w .proce sste x t.com/a bcchm.html
am ma t or y pro ces s es Inf ect i on s (e. g., t ub erc ul o sis , pn eu mo ni a ) Pul mo nar y em bol ic di s eas e Col l ag
Pa g e 3 4 3
ABC Ambe r CHM Conve rte r Tria l ve rsion, http://w w w .proce sste x t.com/a bcchm.html
en vas cul ar di s eas e (e. g., rhe um at o id art hri t is) Su bdi ap hra gm at i c pro ces s As bes t os is Pa ncr eat itis Hy
Pa g e 3 4 4
ABC Ambe r CHM Conve rte r Tria l ve rsion, http://w w w .proce sste x t.com/a bcchm.html
pot hyr oi d ism Tra um a Ca us es of tra ns ud ate s De cre as e d pl a sm a onc ot i c pre ssu re Ne phr ot i c s yn dro
Pa g e 3 4 5
ABC Ambe r CHM Conve rte r Tria l ve rsion, http://w w w .proce sste x t.com/a bcchm.html
me Ci rr hos is Inc rea s ed hyd ros t at ic pre ssu re Co ng es t i ve he art fai l ure
P.73
(b) Pl euri t i c pai n mus t be di fferent i at ed from t he pai n of ri b fract ure, cos t ochondri t i s , compres s i on of i nt ercos t al nerve root s , herpes zos t er, acut e bronchi t i s , and vari ous cardi ovas cul ar and es ophageal condi t i ons .
Pa g e 3 4 6
ABC Ambe r CHM Conve rte r Tria l ve rsion, http://w w w .proce sste x t.com/a bcchm.html
(2) Dyspnea can occur i f t he accumul at i on of pl eural fl ui d compres s es t he l ung and i nt erferes wi t h t he movement of t he di aphragm.
o
o
b. Physical signs. Fl ui d us ual l y accumul at es fi rs t at t he bas e of t he l ung, where t he earl i es t phys i cal s i gns are not ed.
(1) Aus cul t at i on us ual l y reveal s a dul l -t o-fl at percus s i on and reduced or abs ent breat h s ounds over t he area of t he effus i on. An area of bronchial breathing s omet i mes i s heard over t he adjacent compres s ed l ung and may be accompani ed by an al t ered voi ce qual i t y or egophony.
(2) The medi as t i num us ual l y s hi ft s away from t he s i de of a l arge effus i on unl es s t he medi as t i num has become fi xed i n pos i t i on by a t umor or a port i on of t he l ung on t he affect ed s i de has become compl et el y at el ect at i c.
o
o
c. Radiographic appearance
(1) Chest radiography. The earl i es t vi s i bl e s i gns of effus i on on a pl ai n-fi l m radi ograph are bl unt i ng of t he cos t ophreni c angl e and bl urri ng of t he pos t eri or di aphragm i n t he l at eral vi ew. A pos t eri or–ant eri or fi l m may
Pa g e 3 4 7
ABC Ambe r CHM Conve rte r Tria l ve rsion, http://w w w .proce sste x t.com/a bcchm.html
s how no abnormal i t y i f t here i s l es s t han 300 mL pl eural fl ui d. A l at eral decubi t us fi l m may hel p di fferent i at e free fl ui d from previ ous i nfl ammat ory adhes i ons .
(2) Diagnostic ultrasound may hel p l ocal i ze t he effus i on more accurat el y when bl i nd t horacent es i s i s di ffi cul t or uns ucces s ful .
o
o
d. Specialized diagnostic procedures
(1) Unl es s a caus e has been es t abl i s hed, t he pres ence of fl ui d i n t he pl eural cavi t y i s an i ndi cat i on for thoracentesis.
(a) The fl ui d s houl d be removed, t he gros s appearance not ed, and s peci mens s ent t o t he l aborat ory for exami nat i on.
(b) Rout i ne l aborat ory procedures i ncl ude meas uri ng t he t ot al prot ei n and l act at e dehydrogenas e (LDH) cont ent and exami ni ng t he s pun s peci men for cel l s . Bact eri ol ogi c and cyt ol ogi c exami nat i ons and anal ys i s for gl ucos e, amyl as e, and pH provi de furt her i nformat i on.
(2) Needle biopsy of t he pl eura i s performed t o det ermi ne t he caus e of an exudat e when repeat ed t horacent es es are nondi agnos t i c.
Pa g e 3 4 8
ABC Ambe r CHM Conve rte r Tria l ve rsion, http://w w w .proce sste x t.com/a bcchm.html
(3) W hen ordi nary meas ures fai l t o es t abl i s h a defi ni t i ve di agnos i s , thoracotomy or video-assisted thoracoscopy (VAT S) wi t h expl orat i on of t he l ung and bi ops y of t he i nvol ved areas of t he pl eural s urface may be es s ent i al for accurat e di agnos i s .
4. Differential diagnosis. Laborat ory dat a can hel p det ermi ne t he caus e of a pl eural effus i on. o
o
a. It i s us eful t o es t abl i s h whet her t he fl ui d i s an exudat e or a t rans udat e (s ee Tabl e 2-4; IX A 2 b). The pres ence of one or more of t he fol l owi ng cri t eri a i ndi cat e t he fl ui d i s an exudate:
(1) Pl eural fl ui d t o s erum prot ei n rat i o ≥0.5
(2) Pl eural fl ui d LDH great er t han t wo-t hi rds of t he upper l i mi t of normal for s erum
(3) Pl eural fl ui d t o s erum LDH rat i o ≥0.6
o
o
b. The pres ence of gross blood i n t he pl eural fl ui d i s mos t common when t he effus i on i s caus ed by t umor, t rauma, or pul monary i nfarct i on.
o
o
c. The pl eural fl ui d t o s erum glucose rat i o i s s omet i mes l ow when t he effus i on i s caus ed by
Pa g e 3 4 9
ABC Ambe r CHM Conve rte r Tria l ve rsion, http://w w w .proce sste x t.com/a bcchm.html
t ubercul os i s or t umor but us ual l y i s very l ow i n effus i ons t hat are caus ed by rheumat oi d art hri t i s . o
o
d. The pl eural fl ui d amylase l evel frequent l y i s el evat ed when t he effus i on i s at t ri but abl e t o pancreat i c di s eas e or rupt ure of t he es ophagus , and occas i onal l y i s el evat ed moderat el y i n mal i gnant effus i ons .
o
o
e. The pH of pl eural fl ui d us ual l y i s 7.3 or great er. Lower val ues occas i onal l y are s een i n t ubercul os i s and mal i gnant effus i ons . A pH of l es s t han 7.2 i n a parapneumoni c effus i on s ugges t s an empyema.
5. T herapy o
o
a. Treat ment mus t be di rect ed at t he di s eas e caus i ng t he effus i on. Appropri at e t herapy may cal l for ches t t ube pl acement , ant i bi ot i cs , ant i t ubercul ous t herapy, repeat ed t horacent es i s , or chemi cal pl eurodes i s t o el i mi nat e t he pl eural s pace. P.74
o
o
b. Dys pnea may be rel i eved by a t horacent es i s , but t hi s procedure carri es t he ri s k of pneumot horax (from pl eural punct ure) or cardi ovas cul ar col l aps e (from removi ng t oo much fl ui d t oo qui ckl y).
B. Empyema
Pa g e 3 5 0
ABC Ambe r CHM Conve rte r Tria l ve rsion, http://w w w .proce sste x t.com/a bcchm.html
1. Definition and clinical features. An empyema i s an accumul at i on of pus i n t he pl eural s pace; i t i s an occas i onal compl i cat i on of bot h bact eri al pneumoni a and l ung abs ces s . The fl ui d us ual l y i s t hi ck and has t he appearance of frank pus . As previ ous l y s t at ed, pl eural fl ui d wi t h a pH of l es s t han 7.2 s t rongl y s ugges t s an empyema.
2. T herapy. An empyema al mos t al ways requi res ches t t ube drai nage as wel l as ant i bi ot i c t herapy. If t he fl ui d i t s el f i s noni nfect ed, wi t h a rel at i vel y l ow W BC count and a pH of more t han 7.2, t he empyema may res ol ve wi t h s ys t emi c ant i mi crobi al t herapy. However, aft er s everal days wi t hout adequat e drai nage, mos t empyemas become l ocul at ed, s o t hat t ube drai nage i s l es s effect i ve.
C. Pneumothorax
1. Defi ni t i on. Pneumot horax i s an accumul at i on of ai r or gas i n t he pl eural s pace. If t he accumul at i on i s l arge enough, t he underl yi ng l ung parenchyma may become col l aps ed and funct i onl es s .
2. Etiology o
o
a. Pneumot horax i s a common medi cal probl em, frequent l y caus ed by trauma, i ncl udi ng t rauma due t o medi cal procedures . Primary spontaneous pneumothorax occurs wi t h t he rupt ure of bul l ae i n
Pa g e 3 5 1
ABC Ambe r CHM Conve rte r Tria l ve rsion, http://w w w .proce sste x t.com/a bcchm.html
an upper l obe. It occurs more frequent l y i n men t han i n women, and es peci al l y i n men 20–40 years of age. o
o
b. Spont aneous pneumot horax al s o may occur secondary t o l ung i nvol vement i n almost any lung disease but occurs mos t commonl y i n COPD, Pneumocystis carinii i nfect i on, cys t i c fi bros i s , and t ubercul os i s .
3. Clinical features and diagnosis o
o
a. The major symptoms of pneumot horax are pain and dyspnea. The pai n may be ei t her s harp and s evere or dul l and mi l d.
o
o
b. Physical examination s hows hyperres onance and decreas ed breat h s ounds over t he i nvol ved s i de.
o
o
c. Chest radiographs, i f obt ai ned duri ng expi rat i on, may hel p demons t rat e s mal l pneumot horaces , becaus e t hi s t echni que i ncreas es t he cont ras t bet ween t he l ung and t he pl eural s pace.
4. T herapy o
o
a. A s mal l s pont aneous pneumot horax oft en res ol ves by i t s el f. A more s evere pneumot horax cal l s for re-expans i on of t he l ung t hrough
Pa g e 3 5 2
ABC Ambe r CHM Conve rte r Tria l ve rsion, http://w w w .proce sste x t.com/a bcchm.html
pl acement of an i nt ercos t al ches t t ube and appl i cat i on of appropri at e negat i ve pres s ure. The t reat ment i s cont i nued for 24–48 hours aft er t he l ung i s re-expanded, s o t hat t he pl eural s pace s eal s and pl eural adhes i ons prevent recurrence. In s el ect cas es , s mal l er, l es s -compl i cat ed pn eumot horaces can be t reat ed wi t h pneumocent es i s (e.g., removal of ai r duri ng a t horacent es i s ) wi t hout pl acement of a ches t t ube. o
o
b. Spont aneous pneumot horax has a t endency t o recur, and s urgi cal t reat ment s houl d be cons i dered aft er t hree or more occurrences on a gi ven s i de. Surgery i nvol ves an open t horacot omy or VATS and abras i on of t he pl eural s urfaces , whi ch produces s ymphys i s of t he pari et al and vi s ceral pl eurae.
5. Complications. Al t hough pneumot horax i s a rel at i vel y beni gn condi t i on, s eri ous compl i cat i ons can res ul t . o
o
a. Bilateral simultaneous pneumothorax i s rare but can caus e rapi d deat h.
o
o
b. Pneumot horax may be accompani ed by hemorrhage i nt o t he pl eural s pace, whi ch res ul t s i n hemopneumothorax.
o
o
c. T ension pneumothorax i s a bui l d-up of pos i t i ve pres s ure wi t hi n t he pl eural s pace, whi ch rapi dl y produces s evere res pi rat ory and cardi ovas cul ar compromi s e. Pat i ent s undergoi ng pos i t i ve-pres s ure mechani cal vent i l at i on are part i cul arl y at ri s k.
Pa g e 3 5 3
ABC Ambe r CHM Conve rte r Tria l ve rsion, http://w w w .proce sste x t.com/a bcchm.html
(1) Tens i on pneumot horax pres umabl y res ul t s from a bal l -val ve mechani s m at t he s i t e of t he ai r l eak, whi ch al l ows ai r t o ent er but not l eave t he pl eural s pace.
(2) Thi s l eads t o progres s i ve col l aps e of t he l ung, a cont ral at eral s hi ft of t he medi as t i nal s t ruct ures , and reduced bl ood fl ow t o t he ri ght s i de of t he heart , i mpai ri ng cardi ovas cul ar funct i on as wel l as l ung funct i on. Prompt decompres s i on of t he i nvol ved pl eural s pace i s i ndi cat ed.
P.75
D. Chylothorax W hen t he t horaci c duct i s l acerat ed or obs t ruct ed by t rauma or t umor, l ymph may accumul at e i n t he pl eural s pace. Thi s condi t i on, t ermed chyl ot horax, i s i dent i fi ed by a murky appearance of t he fl ui d, demons t rat i on of fat dropl et s on s t ai ni ng wi t h Sudan III, and a t ot al neut ral fat cont ent of >0.5 g/dL.
E. Primary pleural neoplasia
1. Localized fibrous mesothelioma. Thi s uncommon t umor ari s es from t he pl eural s urface and mos t commonl y i s at t ached t o t he vi s ceral pl eura. o
o
a. Symptoms
Pa g e 3 5 4
ABC Ambe r CHM Conve rte r Tria l ve rsion, http://w w w .proce sste x t.com/a bcchm.html
(1) The l es i on may caus e ches t di s comfort and dys pnea i f i t becomes very l arge. However, mos t t umors are di s covered before t hes e s ympt oms devel op.
(2) The s yndrome of hypertrophic pulmonary osteoarthropathy, whi ch i s as s oci at ed wi t h art hral gi a of t he hands , ankl es , wri s t s , and knees and wi t h cl ubbi ng of t he fi ngers , may occur s econdary t o pl eural -bas ed t umors .
o
o
b. Diagnosis. Ches t radi ograph i ndi cat es a mas s l es i on. A pl eural effus i on occas i onal l y i s pres ent .
o
o
c. T herapy i s s urgi cal res ect i on, whi ch al s o rel i eves t he s ympt oms of t he hypert rophi c pul monary os t eoart hropat hy.
o
o
d. Prognosis. Mos t of t hes e t umors are beni gn, and pat i ent s have an excel l ent prognos i s . A few of t hes e t umors are mal i gnant but have favorabl e cours es .
2. Diffuse malignant mesothelioma o
o
a. Incidence. Thi s mal i gnant t umor occurs over a wi de age range, wi t h t he average age at ons et bei ng 55 years . The i nci dence i s i ncreas ed i n workers expos ed t o as bes t os ; general l y, t he mal i gnancy devel ops 20 or more years aft er expos ure.
Pa g e 3 5 5
ABC Ambe r CHM Conve rte r Tria l ve rsion, http://w w w .proce sste x t.com/a bcchm.html o
o
b. Symptoms and diagnosis. Ches t pai n and dys pnea are t he predomi nant s ympt oms . The ches t radi ograph may s how pl eural t hi ckeni ng, pl eural effus i on, or bot h. The di agnos i s i s di ffi cul t t o es t abl i s h by cyt ol ogi c exami nat i on; open pl eural bi ops y oft en i s neces s ary.
o
o
c. T herapy i nvol ves radi at i on t herapy or chemot herapy; res ul t s are uni forml y di s mal .
X. Chest Wall Disorders A. Etiology Ches t wal l di s orders may be ei t her mechani cal or neuromus cul ar i n ori gi n. They may caus e res pi rat ory dys funct i on and, i n s evere cas es , res pi rat ory fai l ure.
1. Mechanical disorders affect i ng t he ches t wal l i ncl ude kyphos i s , s col i os i s , obes i t y-as s oci at ed hypovent i l at i on, fi brot horax, t horacopl as t y, ankyl os i ng s pondyl i t i s , and ches t wal l t rauma.
2. Neuromuscular diseases affect i ng t he ches t wal l are pol yneuropat hi es , mot or s ys t em di s eas es , mus cul ar dys t rophi es , s pi nal cord i njuri es , mul t i pl e s cl eros i s , and myas t heni a gravi s .
B. Kyphoscoliosis Kyphoscoliosis, t he mos t common and bes t unders t ood ches t wal l di s eas e, i s us ed i n t hi s di s cus s i on as a prot ot ype for t he pat hophys i ol ogy, cours e, and management of al l ches t wal l di s orders .
Pa g e 3 5 6
ABC Ambe r CHM Conve rte r Tria l ve rsion, http://w w w .proce sste x t.com/a bcchm.html
1. Definition. Kyphos col i os i s i s a common s kel et al abnormal i t y charact eri zed by pos t eri or curvat ure (kyphosis) and l at eral curvat ure (scoliosis) of t he s pi ne. Thes e proces s es , al one or i n combi nat i on, decreas e t he vol ume and mobi l i t y of t he l ung and ches t wal l .
2. Incidence. Kyphos col i os i s affect s 1% of t he Uni t ed St at es popul at i on and occurs predomi nant l y i n femal es . The deformi t y i s cl i ni cal l y s i gni fi cant i n 2.3% of affect ed i ndi vi dual s .
3. Etiology i s not cl ear i n 80% of cas es . A major known caus e i s chi l dhood pol i omyel i t i s . Congeni t al kyphos col i os i s i s uncommon, but can pot ent i al l y res ul t i n s evere deformi t y and neurol ogi c abnormal i t i es .
4. Pathophysiology o
o
a. Lung vol ume i s reduced i n kyphos col i os i s becaus e t he ches t wal l i s di s t ort ed. The di s t ort i on al s o caus es s t i ffnes s of t he ches t wal l , i ncreas i ng t he work of breat hi ng and caus i ng decreas ed t ot al res pi rat ory compl i ance and reduct i on of FRC. The pres s ure–vol ume compl i ance curve of t he l ung i s nearl y normal , and forced expi rat ory fl ow i s pres erved rel at i ve t o l ung vol ume.
o
o
b. Gas exchange i s i mpai red i n marked kyphos col i os i s : al veol ar hypovent i l at i on occurs and PaCO 2 ri s es . The A–a DO 2 i s mi l dl y wi dened
Pa g e 3 5 7
ABC Ambe r CHM Conve rte r Tria l ve rsion, http://w w w .proce sste x t.com/a bcchm.html
becaus e of [V wi t h dot above]/[Q wi t h dot above] i nequal i t y, whi ch res ul t s from t he compres s i ve effect of at el ect as i s and i nadequat e peri odi c hyperi nfl at i on. P.76
o
o
c. Pul monary hypert ens i on event ual l y i s pres ent at res t as wel l as duri ng exerci s e i n pat i ent s wi t h moderat e ches t wal l deformi t y and no cl i ni cal s i gns of cardi ac dys funct i on.
5. Clinical features and diagnosis o
o
a. Symptoms
(1) Exertional dyspnea i s t he out s t andi ng res pi rat ory s ympt om. The ons et and s everi t y of dys pnea correl at e wi t h t he degree of t he s pi nal angul at i on, as meas ured on t he ches t fi l m. Hypovent i l at i on s upervenes i n t hos e pat i ent s whos e deformi t y i s s evere.
(2) Sequelae of prolonged arterial hypoxemia, i ncl udi ng pul monary hypert ens i on, ri ght vent ri cul ar dys funct i on, and cor pul monal e, may devel op as l at e mani fes t at i ons .
o
o
b. Chest radiograph. Ri bs on t he convex port i on of
Pa g e 3 5 8
ABC Ambe r CHM Conve rte r Tria l ve rsion, http://w w w .proce sste x t.com/a bcchm.html
t he s pi ne are wi del y s paced and rot at ed pos t eri orl y, caus i ng a charact eri s t i c hump. Ri bs on t he concave as pect are crowded and di s pl aced ant eri orl y and encroach on t he apex of t he s econdary curve. The pres ence of kyphos col i os i s can oft en make i nt erpret at i on of t he ches t x-ray di ffi cul t .
6. T herapy o
o
a. Earl y i dent i fi cat i on of kyphos col i os i s i n adol es cence i s t he key t o prevent i on of s ympt omat i c di s eas e. Corrective intervention s houl d be cons i dered when t he angul at i on i s great er t han 40 degrees . There are t wo forms of i nt ervent i on:
(1) Mechanical. A Mi l waukee brace can be appl i ed ext ernal l y duri ng t he earl y s t ages of t he di s eas e.
(2) Surgical. The Harri ngt on procedure, us i ng met al rods and focal s pi nal fus i on, can be performed, aft er whi ch t he pat i ent wears a pl as t er of Pari s jacket cas t for s everal mont hs . Surgery does not i mprove t he maxi mum breat hi ng capaci t y but may i mprove art eri al oxygen and oxygen des at urat i on. At bes t , s urgery appears t o pres erve what ever pul monary funct i on i s pres ent at t he t i me of i nt ervent i on.
o
Pa g e 3 5 9
ABC Ambe r CHM Conve rte r Tria l ve rsion, http://w w w .proce sste x t.com/a bcchm.html
o
b. Peri odi c hyperi nfl at i on wi t h i nt ermi t t ent pos i t i ve-pres s ure breat hi ng devi ces appears t o i ncreas e l ung compl i ance and PaO 2 i n out pat i ent s . In pat i ent s who devel op obs t ruct i ve s l eep apnea (OSA) becaus e of cervi cal s pi ne angul at i on, cont i nuous pos i t i ve-pres s ure breat hi ng (wi t h CPAP) duri ng s l eep i s us eful . In t hos e who devel op chroni c hypercapni c res pi rat ory fai l ure, noni nvas i ve pos i t i ve-pres s ure vent i l at i on (NPPV) or negat i ve-pres s ure vent i l at i on may be us eful .
7. Complications. Res pi rat ory fai l ure and cor pul monal e, t he major compl i cat i ons , res ul t from res pi rat ory i nfect i ons or i njudi ci ous us e of s edat i ves , or bot h.
C. Chest trauma
1. Blunt trauma. Bl unt ches t t rauma caus es i njury ei t her by di rect appl i cat i on of s udden force t o t he ches t wal l and i ndi rect t rans mi s s i on of t hes e forces t o t he i nt rat horaci c s t ruct ures or by s econdary vi s ceral des t ruct i on by ches t wal l s t ruct ures . o
o
a. Injury of ext rapul monary organs oft en accompani es bl unt ches t t rauma. Di s rupt i on of t he ches t wal l may caus e ri b fract ures , hemot horax, pneumot horax, and fl ai l ches t . Inert i al i njury may caus e rupt ure of t he bronchi al , di aphragmat i c, or great bl ood ves s el s .
o
o
b. Flail chest mos t commonl y res ul t s from mot or vehi cl e i njury or overzeal ous cardi ac res us ci t at i on.
Pa g e 3 6 0
ABC Ambe r CHM Conve rte r Tria l ve rsion, http://w w w .proce sste x t.com/a bcchm.html
The ches t wal l , or at l eas t one hemi t horax, i s rendered uns t abl e by mul t i pl e fract ures of t he ri bs , s t ernum, or cos t ochondral joi nt s .
(1) The i njured port i on moves paradoxically, t hat i s , i nward on i ns pi rat i on as t he i nt rapl eural pres s ure becomes s ubat mos pheri c and out ward on expi rat i on as t he i nt rapl eural pres s ure i ncreas es t oward at mos pheri c.
(2) Res pi rat ory fai l ure i s t reat ed wi t h vol ume-cycl ed mechani cal vent i l at i on, pai n cont rol , and oxygen s uppl ement at i on.
2. Penetrating trauma i s charact eri zed by punct ure or l acerat i on of t he ches t wal l and i nt rat horaci c fi s t ul ae. Vehi cul ar acci dent s and kni fe and mi s s i l e wounds are t he us ual caus es . Expl orat ory t horacot omi es are i ndi cat ed for pers i s t ent hemot horax and s ucki ng ches t wounds and t o det ermi ne t he l i kel i hood of medi as t i nal , di aphragmat i c, or cardi ac di s rupt i on.
XI. Mediastinal Diseases A. Mediastinal masses The medi as t i num i s di vi ded i nt o t hree part s : anterior/superior, middle, and posterior. Medi as t i nal mas s es may occur at any age and are charact eri s t i c of t he medi as t i nal P.77 compart ment i n whi ch t hey occur. The l at eral ches t radi ograph oft en i s t he mos t i mport ant i ni t i al di agnos t i c meas ure.
1. Masses in the anterior/superior mediastinum
Pa g e 3 6 1
ABC Ambe r CHM Conve rte r Tria l ve rsion, http://w w w .proce sste x t.com/a bcchm.html o
o
a. T hymomas are t he mos t common s uperi or medi as t i nal mas s es . Presentation frequent l y i nvol ves cough, ches t pai n, and s uperi or vena caval obs t ruct i on. Myas t heni a gravi s occurs i n approxi mat el y one t hi rd of pat i ent s wi t h a t hymoma. RBC apl as i a and hypogammagl obul i nemi a are ot her recogni zed but rare as s oci at i ons . Surgi cal exci s i on i s recommended.
o
o
b. Hodgkin' s disease and non-Hodgkin' s lymphomas may mani fes t as mas s es i n t he ant eri or or s uperi or medi as t i num.
o
o
c. Intrathoracic goiters may occur, part i cul arl y i n mi ddl e-aged women. They us ual l y are as ympt omat i c but may caus e s t ri dor, hoars enes s , or dys phagi a.
o
o
d. Dermoid cysts appear as dens e, homogeneous l obul ar s hadows i n t he ant eri or medi as t i num, oft en wi t h cal ci fi cat i ons i n t he wal l s . Teet h may be recogni zed wi t hi n t he t umor. Dermoi d cys t s us ual l y are as ympt omat i c unl es s i nfect i on or mal i gnant change devel ops .
2. Masses in the middle mediastinum o
o
a. Lymphomas may al s o appear i n t he mi ddl e medi as t i num.
o
Pa g e 3 6 2
ABC Ambe r CHM Conve rte r Tria l ve rsion, http://w w w .proce sste x t.com/a bcchm.html
o
b. Pleuropericardial cysts occur i n t he mi ddl e medi as t i num at t he ri ght cardi ophreni c angl e, charact eri s t i cal l y appeari ng as s moot h, s harpl y demarcat ed mas s es of uni form dens i t y.
o
o
c. Bronchogenic cysts and reduplication of the esophagus are rare caus es of mi ddl e medi as t i nal mas s es .
3. Masses in the posterior mediastinum. Neurogenic tumors are t he mos t common pos t eri or medi as t i nal t umors and charact eri s t i cal l y occur al ong t he paravert ebral border. Thes e t umors oft en are as ympt omat i c i n adul t s but may caus e ches t pai n wi t h s t ri dor, breat hl es s nes s , cough, and t racheal compres s i on. Horner's s yndrome and s pi nal cord compres s i on al s o may occur.
4. Aneurysms of the aorta may occur i n any compart ment dependi ng on t hei r l ocat i on wi t hi n t he aort a.
5. Mas s es i n t he anterior/superior compartment are more l i kel y t o be mal i gnant t han t hos e i n ot her compart ment s .
B. Mediastinitis
1. Acute mediastinitis i s a s evere, l i fe-t hreat eni ng i l l nes s t hat mos t oft en fol l ows rupt ure of t he es ophagus . It al s o may fol l ow vomi t i ng, dent al work,
Pa g e 3 6 3
ABC Ambe r CHM Conve rte r Tria l ve rsion, http://w w w .proce sste x t.com/a bcchm.html
endos copy, or ot her t rauma and i s charact eri zed by fever, ches t pai n, and vari abl e medi as t i nal enl argement . The di s eas e progres s es rapi dl y and requi res emergency medi cal and s urgi cal t reat ment .
2. Chronic mediastinitis and mediastinal fibrosis. Histoplasma s p. or, rarel y, ot her fungi or mycobact eri a, can produce a chroni c granul omat ous proces s i n t he medi as t i num, oft en wi t h ext ens i ve s car t i s s ue t hat cont ract s t o caus e narrowi ng of t he t rachea, bronchi , vena cava, pul monary art eri es , and pul monary vei ns . Medi as t i ni t i s t hat occurs wi t hout any known caus e i s referred t o as idiopathic mediastinal fibrosis.
C. Pneumomediastinum Pneumomediastinum i s t he pres ence of ai r i n t he medi as t i num. Ai r i s pres umed t o l eak from al veol i and t o di s s ect al ong bronchi t o t he hi l um, from whi ch i t may ent er t he medi as t i num. If pres s ure bui l ds i n t he medi as t i num, ai r may expand i nt o t he neck t i s s ues , produci ng s ubcut aneous emphys ema. However, i f t he medi as t i nal ai r i s confi ned, t he i ncreas i ng pres s ure may i nt erfere wi t h ci rcul at i on. W hen t hi s occurs , t racheos t omy us ual l y i s adequat e t herapy. Int ervent i on i s unneces s ary i n pat i ent s wi t hout ci rcul at ory probl ems .
XII. Diffuse Interstitial Lung Disease A. Definition Di ffus e i nt ers t i t i al l ung di s eas e i s a broad t erm for a group of rel at ed di s orders , whi ch are al l charact eri zed by di ffus e i nfl ammat ory al veol ar i nfi l t rat i on. Regardl es s of t hei r vari ous caus es , t hes e ent i t i es s hare cert ai n cl i ni cal , radi ographi c, pat hol ogi c, and phys i ol ogi c charact eri s t i cs . Al l begi n acut el y and progres s t o a chroni c condi t i on; t hat i s , a pot ent i al l y revers i bl e i nt ers t i t i al pneumoni t i s progres s es t o di ffus e pul monary fi bros i s . P.78
Pa g e 3 6 4
ABC Ambe r CHM Conve rte r Tria l ve rsion, http://w w w .proce sste x t.com/a bcchm.html
B. Etiology
1. In approxi mat el y 50% of cas es , i nt ers t i t i al l ung di s eas e occurs s pont aneous l y. The t ermi nol ogy for t hes e di s eas es i s vari ed. In Nort h Ameri ca, t he t erm idiopathic pulmonary fibrosis i s favored, whereas i n Great Bri t ai n, cryptogenic fibrosing alveolitis i s preferred.
2. The remai ni ng cas es are as s oci at ed wi t h many known or s us pect ed caus es (Tabl e 2-5). Envi ronment al or occupat i onal expos ure t o a vari et y of s ubs t ances i s wel l known t o i nduce i nt ers t i t i al pul monary fi bros i s (s ee XIII); t hes e s ubs t ances may act as i nci t i ng al l ergens or di rect l y t oxi c agent s . In ot her cas es , i nt ers t i t i al l ung di s eas e i s as s oci at ed wi t h connect i ve t i s s ue di s eas e, s arcoi dos i s , or chroni c hypers ens i t i vi t y pneumoni t i s . Many of t hes e di s eas es have been as s oci at ed wi t h ci garet t e s moki ng.
C. Pathophysiology and histology
1. Stages. The pat hol ogi c changes i n t hes e di s orders have a hi ghl y vari abl e t i me-cours e, dependi ng on t he degree of expos ure and on t he t ype of i njuri ous agent . The cl i ni cal cours e may run from a few weeks t o many years . o
o
a. Acute stage. The earl i es t s t age of di ffus e i nt ers t i t i al l ung di s eas e i s charact eri zed by acut e
Pa g e 3 6 5
ABC Ambe r CHM Conve rte r Tria l ve rsion, http://w w w .proce sste x t.com/a bcchm.html
damage t o capi l l ary and al veol ar epi t hel i al cel l s , l eadi ng t o i nt ers t i t i al and i nt ra-al veol ar P.79
edema and s ubs equent format i on of hyal i ne membranes . Thi s s t age may ei t her res ol ve compl et el y or progres s t o t he s t age of acut e i nt ers t i t i al pneumoni a.
TABLE 2-5 Causes of Diffuse Pulmonary Infiltration In hal ed su bst an ces Ga s es (ca dm iu m, me rcu ry) Mi n era l dus ts (s i l i ca
Pa g e 3 6 6
ABC Ambe r CHM Conve rte r Tria l ve rsion, http://w w w .proce sste x t.com/a bcchm.html
, as b es t os ) Ant i ge ns (ba ct e ri a, mo l ds , ani ma l pro t ei n) As pi r at e d fl ui d or for ei g n bo dy Dr ug the ra
Pa g e 3 6 7
ABC Ambe r CHM Conve rte r Tria l ve rsion, http://w w w .proce sste x t.com/a bcchm.html
py Bu s ul fan Bl e om yci n Ni t rof ura nt o in Gol d Cyc l op hos ph am i de Me t ho t re xat e Ra dia tio n the ra py Inf ect
Pa g e 3 6 8
ABC Ambe r CHM Conve rte r Tria l ve rsion, http://w w w .proce sste x t.com/a bcchm.html
ion Rec urr ent bac t eri al pn eu mo ni a Tu ber cul os i s Vi r al i nf ect i on s Ne opl asi a Bro nch oal veo l ar car ci n om a
Pa g e 3 6 9
ABC Ambe r CHM Conve rte r Tria l ve rsion, http://w w w .proce sste x t.com/a bcchm.html
Leu ke mi a Ly mp ho ma Ly mp ha ngi tic s pr ea d Me tab olic dis ea se Ure mi a Dis ea ses of un kn ow n eti olo
Pa g e 3 7 0
ABC Ambe r CHM Conve rte r Tria l ve rsion, http://w w w .proce sste x t.com/a bcchm.html
gy Sar coi dos is Col l ag en vas cul ar (co nn ect i ve tis s ue ) di s eas e Go od pas t ur e's s yn dro me Am yl o i do sis Idi op
Pa g e 3 7 1
ABC Ambe r CHM Conve rte r Tria l ve rsion, http://w w w .proce sste x t.com/a bcchm.html
at h ic pu lm on ary he mo sid ero sis Pul mo nar y al v eol ar pro t ei nos is Bro nch i ol i tis obl i t er ans – org ani zin g pn
Pa g e 3 7 2
ABC Ambe r CHM Conve rte r Tria l ve rsion, http://w w w .proce sste x t.com/a bcchm.html
eu mo ni a
o
o
b. Chronic stage. In many pat i ent s , t he di s eas e progres s es t o a chroni c s t age, i n whi ch ext ens i ve depos i t i on or al t erat i on of col l agen res ul t s i n wi des pread fi bros i s . In addi t i on, t hi s s t age i s marked by s moot h mus cl e hypert rophy and profound di s rupt i on of t he al veol ar s paces , whi ch are l i ned wi t h at ypi cal cuboi dal cel l s .
o
o
c. End stage. The di s eas e event ual l y progres s es unt i l t he l ung becomes “honeycombed.― In t hi s s t age, t he ent i re al veol ar and capi l l ary net work i s repl aced wi t h fi brous t i s s ue and di l at ed s paces , t he capi l l ary bed i s decreas ed, and t he i nvol ved l ung has no remai ni ng gas exchange funct i on.
2. Histologic classification o
o
a. Several t erms have been us ed i n an at t empt t o des cri be t he vari ous pul monary changes . A common cl as s i fi cat i on fol l ows , i n decreas i ng order of
Pa g e 3 7 3
ABC Ambe r CHM Conve rte r Tria l ve rsion, http://w w w .proce sste x t.com/a bcchm.html
occurrence:
(1) Us ual i nt ers t i t i al pneumoni t i s (UIP)
(2) Nons peci fi c i nt ers t i t i al pneumoni t i s (NSIP)
(3) Bronchi ol i t i s obl i t erans -organi zi ng pneumoni a (BOOP)
(4) Res pi rat ory bronchi ol i t i s as s oci at ed-i nt ers t i t i al l ung di s eas e (RB-ILD)
(5) Des quamat i ve i nt ers t i t i al pneumoni t i s (DIP)
(6) Lymphocyt i c i nt ers t i t i al pneumoni t i s (LIP)
(7) Acut e i nt ers t i t i al pneumoni t i s Acut e i nt ers t i t i al pneumoni t i s i s an acut e ARDS i l l nes s (al s o cal l ed Hamman-Ri ch s yndrome) and di ffers from t he ot her di s orders , whi ch are more chroni c.
o
o
b. Many aut hori t i es bel i eve t hat t hes e cl as s i fi cat i ons are s omewhat art i fi ci al and may repres ent vari ous s t ages or di fferent pat hways i n t he progres s i on from acut e t o end-s t age pul monary fi bros i s . The hi s t ol ogi c pat t ern obs erved at bi ops y
Pa g e 3 7 4
ABC Ambe r CHM Conve rte r Tria l ve rsion, http://w w w .proce sste x t.com/a bcchm.html
depends l argel y on t he s t age of t he di s eas e at whi ch t he s peci men i s obt ai ned. In addi t i on, becaus e t hi s i s a het erogeneous pat hol ogi c proces s , di fferent areas wi t hi n a gi ven s peci men may s how vari ed s t ages of anat omi c al t erat i on.
3. Effects on pulmonary function o
o
a. In t he earl y s t age of di s eas e, hypoxemi a and an i ncreas e i n t he A–a DO 2 occur wi t h exerci s e. As t he di s eas e progres s es , res t i ng hypoxemi a devel ops . The abnormal i t i es of gas exchange are al mos t cert ai nl y t he res ul t of [V with dot above]/[Q with dot above] abnormal i t i es . Al t hough D LC O us ual l y i s decreas ed as t he di s eas e progres s es , t hi s becomes s i gni fi cant i n caus i ng hypoxemi a onl y duri ng exerci s e, not at res t .
o
o
b. The vent i l at ory pat t ern becomes res t ri ct i ve l at e i n t he di s eas e and i s charact eri zed by a decreas e i n al l s ubdi vi s i ons of l ung vol ume. Lung compl i ance i s decreas ed, s hi ft i ng t he pres s ure–vol ume compl i ance curve downward and t o t he ri ght . Expi rat ory fl ow rat es and t he FEV 1 /FVC rat i o us ual l y are wel l pres erved.
o
o
c. The abnormal i t i es i n ai rway funct i on t es t s t hat are s een i n a number of pat i ent s wi t h pul monary fi bros i s i ndi cat e t hat t he peri pheral ai rways are s i gni fi cant l y i nvol ved i n t hi s pat hol ogi c proces s .
D. Clinical features
Pa g e 3 7 5
ABC Ambe r CHM Conve rte r Tria l ve rsion, http://w w w .proce sste x t.com/a bcchm.html
1. Symptoms of i nt ers t i t i al l ung di s eas e s een mos t frequent l y are dys pnea on exert i on and nonproduct i ve cough. Increas ed fat i gabi l i t y, fever, and wei ght l os s al s o are common.
2. Physical findings i ncl ude t achypnea, di gi t al cl ubbi ng, and l at e i ns pi rat ory dry crackl es . If t he di s eas e i s s evere, cyanos i s and evi dence of ri ght vent ri cul ar fai l ure al s o may be pres ent .
E. Diagnosis The cl i ni cal , radi ographi c, and phys i ol ogi c fi ndi ngs s t rongl y s ugges t t he di agnos i s of di ffus e i nt ers t i t i al l ung di s eas e. Defi ni t i ve di agnos i s requi res t i s s ue confi rmat i on, preferabl y by open-l ung bi ops y or VATS.
1. Laboratory findings, i ncl udi ng pul monary funct i on s t udi es (s ee XII C 3), us ual l y are nons peci fi c. Tes t s for ant i nucl ear ant i bodi es (ANAs ), rheumat oi d fact or, and i mmunogl obul i ns may be pos i t i ve.
2. Radiographic findings. Chest radiographs us ual l y s how a di ffus e ret i cul onodul ar pat t ern t hroughout bot h l ung fi el ds t hat i s oft en more pronounced at t he l ung bas es . Ches t CT, es peci al l y wi t h hi gh-res ol ut i on cut s , provi des more ext ens i ve dat a on t he ext ent of l ung i nvol vement and whet her fi bros i s or al veol i t i s i s t he predomi nant proces s (Fi gure 2-12). In s ome cas es , however, cl i ni cal evi dence of di s eas e may exi s t wi t hout radi ographi c confi rmat i on. P.80
Pa g e 3 7 6
ABC Ambe r CHM Conve rte r Tria l ve rsion, http://w w w .proce sste x t.com/a bcchm.html
FIGURE 2-12 Ches t CT i n a pat i ent wi t h IPF s howi ng peri pheral ret i cul ar abnormal i t i es (bl ack arrow) and t ract i on bronchi ect as i s (whi t e arrow). IPF, i di opat hi c pul monary fi bros i s .
3. Open-lung biopsy i s us eful i n det ermi ni ng t he s t age of t he di s eas e, t he appropri at e t herapy, and t he probabl e prognos i s . W het her t he di s eas e res ponds t o t herapy appears t o correl at e wel l wi t h pat hol ogi c evi dence of fi bros i s . Speci mens s howi ng act i ve cel l ul ar i nfi l t rat es and mi ni mal fi bros i s s ugges t a much bet t er prognos i s t han t hos e s howi ng ext ens i ve fi bros i s .
4. Becaus e t he hi s t ol ogi c changes are het erogeneous , transbronchial lung biopsy is of limited usefulness unl es s t here i s cl ear-cut evi dence of s arcoi dos i s , i nfect i on, or carci noma.
5. Bronchoalveolar lavage (BAL), wi t h anal ys i s of t he cel l ul ar el ement s ret ri eved, i s a di agnos t i c procedure t hat has been us ed t o di fferent i at e i nt ers t i t i al l ung di s eas e due t o an i nfi l t rat i ve proces s (whi ch i s as s oci at ed wi t h an i ncreas ed number of
Pa g e 3 7 7
ABC Ambe r CHM Conve rte r Tria l ve rsion, http://w w w .proce sste x t.com/a bcchm.html
pol ymorphonucl ear l eukocyt es ) from hypers ens i t i vi t y pneumoni t i s (whi ch i s as s oci at ed wi t h an i ncreas ed number of l ymphocyt es ). However, i t i s t oo nons peci fi c t o be us ed di agnos t i cal l y.
F. Therapy
1. Corticosteroids have been t he mai ns t ay of t herapy and are cl earl y i ndi cat ed when open-l ung bi ops y s hows an act i ve cel l ul ar proces s wi t hout ext ens i ve fi bros i s . Large dos es (e.g., predni s one 1 mg/kg/day) may be us ed i ni t i al l y, and phys i ol ogi c and radi ographi c paramet ers s houl d be moni t ored cl os el y. If t here i s i mprovement aft er 6 weeks , t he dos e s houl d be t apered gradual l y, wi t h frequent moni t ori ng t o det ect phys i ol ogi c rel aps e.
2. If no i mprovement occurs wi t h s t eroi ds al one, immunosuppressive agents may be advi s abl e, ei t her gi ven al one or i n combi nat i on wi t h s t eroi ds . Az at hi opri ne i s t he mos t wi del y us ed; cycl ophos phami de and chl orambuci l al s o have been us ed. Newer agent s t hat may affect t he devel opment of fi bros i s , s uch as i nt erferon gamma and col chi ci ne, are s omet i mes t ri ed. Ant i oxi dant s s uch as acet yl cys t ei ne are us e i n s ome pat i ent s .
G. Prognosis Int ers t i t i al l ung di s eas e has a vari abl e cours e.
1. Some cas es res ol ve or arres t s pont aneous l y or aft er removal of a known caus at i ve agent (e.g., a drug or envi ronment al fact or).
Pa g e 3 7 8
ABC Ambe r CHM Conve rte r Tria l ve rsion, http://w w w .proce sste x t.com/a bcchm.html
2. However, us ual i nt ers t i t i al pneumoni t i s i s a progres s i ve i nt ers t i t i al l ung di s eas e t hat can be an i ns i di ous , devas t at i ng di s eas e wi t h cons i derabl e morbi di t y and mort al i t y. The average l engt h of s urvi val aft er di agnos i s i s 4–5 years .
XIII. Occupational Lung Diseases A. Introduction Many res pi rat ory i l l nes s es are caus ed by i nhal at i on of i mpure ai r. To produce l ung di s eas e, an i njuri ous i nhal ant mus t :
1. Exi s t i n a s i ze and form t hat i s capabl e of reachi ng t he l ower res pi rat ory t ract P.81
2. Be depos i t ed on or abs orbed i nt o bronchi al or al veol ar s urfaces
3. Remai n i n t he res pi rat ory t ract for a s uffi ci ent peri od t o produce i njury
B. Pulmonary responses to mineral dusts Lung di s eas e caus ed by i nhal at i on of dus t i s t ermed pneumoconiosis.
1. Asbestos-related disease. Asbestos i s t he t erm appl i ed t o s everal nat ural l y occurri ng fi brous s i l i cat es , whos e fi bers may be l ong, curl ed, and fl exi bl e, or s t rai ght and bri t t l e. As bes t os fi bers may caus e s everal di s t i nct t ypes of l es i ons . o
Pa g e 3 7 9
ABC Ambe r CHM Conve rte r Tria l ve rsion, http://w w w .proce sste x t.com/a bcchm.html
o
a. Asbestosis. Thi s i s a di ffus e i nt ers t i t i al cel l ul ar and fi brot i c react i on of t he l ung t o i nhal ed as bes t os fi bers . Affect ed pat i ent s compl ai n of breat hl es s nes s , and phys i cal s i gns i ncl ude di gi t al cl ubbi ng and bas i l ar ral es . The ches t fi l m s hows s mal l l ungs cont ai ni ng hazy i nfi l t rat es compos ed of s mal l i rregul ar or l i near opaci t i es ; l ower l ung zones are more heavi l y affect ed. A res t ri ct i ve vent i l at ory i mpai rment and a reduced D LC O are t he expect ed abnormal i t i es .
o
o
b. Nonneoplastic pleural disorders. As bes t os may caus e pl euri s y wi t h effus i on, pl eural pl aques , and di ffus e pl eural t hi ckeni ng.
o
o
c. Cancer. Many years aft er expos ure (l at ency peri od), cancer can devel op i n pers ons expos ed t o as bes t os at s uffi ci ent concent rat i ons .
(1) Bronchogenic carcinoma i s a recogni zed cons equence of as bes t os expos ure, es peci al l y i n i ndi vi dual s who s moke.
(2) Malignant pleural mesotheliomas (s ee IX E 2) are rare t umors . They are us ual l y as bes t os -rel at ed but are not as s oci at ed wi t h s moki ng.
2. Silica-related disease. Free s i l i ca and s i l i cat es are abundant component s of t he eart h's crus t . To be i njuri ous t o t he l ungs , t hes e part i cl es mus t exi s t as
Pa g e 3 8 0
ABC Ambe r CHM Conve rte r Tria l ve rsion, http://w w w .proce sste x t.com/a bcchm.html
res pi rabl e aeros ol s . o
o
a. Silicosis i s a di ffus e fi brot i c react i on of t he l ungs t o i nhal at i on of free crys t al l i ne s i l i ca (s and, quart z). Inhal ed s i l i ca part i cl es are i nges t ed by al veol ar macrophages , whi ch s oon rupt ure, rel eas i ng cyt ot oxi c enzymes al ong wi t h t he engul fed s i l i ca part i cl es . The s i l i ca i s re-i nges t ed by ot her macrophages and t he cycl e cont i nues , s t i mul at i ng a l ocal fi brot i c react i on. The fi nal res ul t i s t he format i on of t he rel at i vel y acel l ul ar fibrous silicotic nodule t hat charact eri zes t hi s di s eas e.
(1) Simple silicosis. In t hi s s t age, t he ches t radi ograph s hows numerous s mal l , rounded opaci t i es (i s ol at ed nodul es ) s cat t ered t hroughout t he l ungs . Si mpl e s i l i cos i s us ual l y i s not as s oci at ed wi t h vent i l at ory i mpai rment .
(2) Complicated silicosis. If expos ure t o s i l i ca cont i nues , i s ol at ed nodul es may coal es ce t o form l arger mas s es of fi brot i c t i s s ue t hat di s t ort t he l ungs . Thi s progres s i on al s o may occur becaus e of mycobact eri al or mycot i c i nfect i ons . Compl i cat ed s i l i cos i s may l ead t o progres s i ve mas s i ve fi bros i s (PMF) and oft en produces a mi xt ure of res t ri ct i ve and obs t ruct i ve vent i l at ory i mpai rment .
o
o
b. Nonfibrotic effects. Si l i cat es s uch as t al c, kaol i n, ful l er's eart h, and bent oni t e can produce s i mpl e or compl i cat ed pneumoconiosis, wi t hout di ffus e pul monary fi bros i s .
Pa g e 3 8 1
ABC Ambe r CHM Conve rte r Tria l ve rsion, http://w w w .proce sste x t.com/a bcchm.html
3. Coal workers' pneumoconiosis (CWP, “ black lung―) o
o
a. Al t hough coal dus t i s l es s fi brot i c t han s i l i ca, CW P s hares wi t h s i l i cos i s t he radi ographi c appearance of s mal l , rounded opaci t i es i n t he s i mpl e s t age and l arge, congl omerat e mas s es i n t he compl i cat ed s t age. However, s i mpl e CW P onl y occas i onal l y devel ops i nt o compl i cat ed CW P.
o
o
b. Simple CWP has no charact eri s t i c funct i onal abnormal i t y. A chroni c bronchi t i s probabl y account s for mos t of t he res pi rat ory di s abi l i t y i n affect ed pat i ent s . Complicated CWP can l ead t o PMF, wi t h res t ri ct i ve vent i l at ory i mpai rment .
4. Beryllium-related disease. Beryl l i um compounds can produce bot h acut e and chroni c l ung di s eas e. The acut e form i s a di ffus e pneumoni t i s t hat can devel op i nt o pul monary edema. The chroni c form i s s t ri ki ngl y s i mi l ar t o s arcoi dos i s , wi t h granul omas t hroughout t he body; i t caus es progres s i ve l os s of res pi rat ory funct i on.
5. Pulmonary response to other mineral dusts. Some dus t s (e.g., t i n oxi de, i ron oxi de, bari um s ul fat e), when depos i t ed i n t he l ung, are cl eared i nt o aggregat i ons i n t he pul monary l obul es . Thes e dus t col l ect i ons caus e l i t t l e or no react i on and do not phys i cal l y i nt erfere wi t h vent i l at ory funct i on or perfus i on.
P.82
Pa g e 3 8 2
ABC Ambe r CHM Conve rte r Tria l ve rsion, http://w w w .proce sste x t.com/a bcchm.html
TABLE 2-6 Selected Examples of Hypersensitivity Pneumonitis (Extrinsic Allergic Alveolitis) Re sp on sibl Dis e or An de tig r en Far Sp me ore r's s l un of g
t he rm op hi l i c act i no my cet es in mo l dy
hay Bi r Ant d
i ge
fan ns
Pa g e 3 8 3
ABC Ambe r CHM Conve rte r Tria l ve rsion, http://w w w .proce sste x t.com/a bcchm.html
ci e fro r's m l un fea g
t he rs , exc ret a, or s er
um Mu Sp s hr ore oo s m
of
wor t he ker rm 's
op
l un hi l i g
c act i no my cet es in co mp
os t Mal Sp t
ore
wor s ker of 's
As
l un pe
Pa g e 3 8 4
ABC Ambe r CHM Conve rte r Tria l ve rsion, http://w w w .proce sste x t.com/a bcchm.html
g
rgil lus cla vat us in gra
in Gra Du in
st
wor der ker i ve 's
d
l un fro g
m t he gra in we evi
l Ba Th gas er s os mo is
phi lic act i no my cet es in s ug ar can
Pa g e 3 8 5
ABC Ambe r CHM Conve rte r Tria l ve rsion, http://w w w .proce sste x t.com/a bcchm.html
e res i du e Hu Th mi er di fi mo er phi (ai r l i c con act di t i i no on my er) cet l un es g
in hu mi di fi ers or ai r con di t i on ers
C. Pulmonary response to organic dusts (hypersensitivity pneumonitis, extrinsic allergic alveolitis)
1. Etiology and pathogenesis. The i nhal at i on of organi c dus t s (e.g., fungal s pores , t hermophi l i c act i nomycet es , and fragment s of ani mal and veget abl e mat t er) caus es a
Pa g e 3 8 6
ABC Ambe r CHM Conve rte r Tria l ve rsion, http://w w w .proce sste x t.com/a bcchm.html
di ffus e, granul omat ous pul monary parenchymal react i on known as hypersensitivity pneumonitis or extrinsic allergic alveolitis. Some common exampl es are l i s t ed i n Tabl e 2-6. Pat i ent s oft en have ant i bodi es agai ns t t he offendi ng s ubs t ances , and t he fi ndi ngs s ugges t a t ype III hypers ens i t i vi t y react i on, wi t h t i s s ue damage from t he depos i t i on of ant i gen–ant i body compl exes . Pat hol ogi cal l y, mononucl ear i nt ers t i t i al i nfi l t rat es and fi bros i s predomi nat e.
2. Clinical features. Several hours aft er expos ure, pat i ent s experi ence mal ai s e, fever, and chi l l s , wi t h ches t t i ght nes s and pers i s t ent dry cough. Radi ographs obt ai ned duri ng acut e at t acks s how pul monary i nfi l t rat es . Sympt oms abat e wi t hi n a few days but recur wi t h s ubs equent expos ures . Repeat ed expos ures may l ead t o a fi xed res t ri ct i ve l ung di s eas e.
3. T herapy i ncl udes avoi dance of expos ure and cort i cos t eroi d t reat ment for acut e at t acks .
D. Obstructive airway disease due to inhalants
1. Occupational asthma. Expos ure t o vari ous occupat i onal i nhal ant s can caus e occupat i onal as t hma, oft en wi t hout a demons t rabl e i mmunol ogi c mechani s m. Affect ed i ndi vi dual s may or may not be at opi c, and t he react i on us ual l y occurs s everal hours aft er expos ure. W hen as t hma res ul t s from a s i ngl e hi gh-l evel cont act i nhal at i on, i t i s cal l ed react i ve ai rways dys funct i on s yndrome (RADS). o
Pa g e 3 8 7
ABC Ambe r CHM Conve rte r Tria l ve rsion, http://w w w .proce sste x t.com/a bcchm.html
o
a. Etiologic agents
(1) Si mpl e i norgani c chemi cal s (e.g., pl at i num s al t s )
(2) Si mpl e organi c chemi cal s (e.g., di -i s ocyanat es , formal dehyde, pht hal i c anhydri des )
(3) Det ergent enzymes deri ved from Bacillus subtilis
(4) W ood dus t , es peci al l y wes t ern red cedar
(5) Fungal ant i gens and grai n weevi l ant i gens
(6) Ani mal dander and excret i ons
(7) Grai n and grai n cont ami nant s
o
o
b. Diagnosis and therapy. The di agnos i s i s s t rongl y s ugges t ed by a hi s t ory of as t hma s ympt oms t hat occur on workdays but not on weekends or duri ng vacat i on from work. Sympt oms oft en occur at work but del ayed react i ons may not occur unt i l aft er t he workday has ended. Di rect confi rmat i on by chal l enge t es t i ng i s t he mos t convi nci ng demons t rat i on of t he caus al rel at i ons hi p. Avoi dance of expos ure i s t he mos t
Pa g e 3 8 8
ABC Ambe r CHM Conve rte r Tria l ve rsion, http://w w w .proce sste x t.com/a bcchm.html
effect i ve t reat ment . Acut e at t acks may res pond t o s t andard as t hma medi cat i on.
2. Byssinosis i s occupat i onal as t hma i nduced by cot t on dus t ; i t i s s een i n t ext i l e workers . It i s uncert ai n whet her t he pat hogeni c mechani s m i s i mmunol ogi c or pharmacol ogi c. At fi rs t , t he affect ed worker experi ences ches t t i ght nes s and s hort nes s of breat h earl y i n t he work week but feel s wel l l at er i n t he work week. W i t h years of expos ure, t he s ympt oms may l as t l at er i nt o t he week, unt i l s ympt oms and s i gns of chroni c fi xed ai rway obs t ruct i on fi nal l y prevai l .
3. Industrial bronchitis. W hen chroni c obs t ruct i ve bronchi t i s i s caus ed by occupat i onal i nhal ant s , t he di s order i s di ffi cul t t o recogni ze becaus e t he s ympt oms of bronchi t i s (e.g., chroni c cough) P.83
are s o preval ent i n t he general popul at i on. Ident i fi cat i on of an occupat i onal i nhal ant requi res careful and ext ens i ve epi demi ol ogi c s t udi es .
E. Pulmonary response to irritant gases Irri t ant gas es i nfl ame t he res pi rat ory t ract and can caus e upper and l ower ai rway di s eas e. In hi gh concent rat i ons , t hey can caus e pul monary edema.
1. Such agent s i ncl ude ammoni a (NH 3 ), hydrochl ori c aci d (HCl ), s ul fur di oxi de (SO 2 ), ni t rogen di oxi de (NO 2 ), and phos gene (Cl 2 CO).
Pa g e 3 8 9
ABC Ambe r CHM Conve rte r Tria l ve rsion, http://w w w .proce sste x t.com/a bcchm.html
2. Oft en, t here i s a l at ent peri od of 12–24 hours before t he ons et of ches t s ympt oms .
3. The pul monary edema due t o any i rri t ant gas i s t reat ed wi t h s upport i ve meas ures and cort i cos t eroi ds . In s ome cas es , fol l ow-up s hows bronchi ol i t i s obl i t erans .
XIV. Pulmonary Diseases of Unknown Etiology Except for s arcoi dos i s , pul monary di s eas es of unknown et i ol ogy are encount ered i nfrequent l y by cl i ni ci ans .
A. Sarcoidosis
1. Definition. Sarcoi dos i s i s a mul t i s ys t em di s eas e charact eri zed by t he pres ence of noncas eat i ng granul omas i n vari ous organs . The l ungs are i nvol ved i n more t han 90% of report ed cas es . Pat i ent s wi t h s arcoi dos i s us ual l y pres ent wi t h medi as t i nal or hi l ar l ymphadenopat hy wi t h pul monary i nfi l t rat i on, combi ned wi t h cut aneous or ocul ar l es i ons . Les s common but i mport ant mani fes t at i ons i ncl ude peri pheral adenopat hy, eryt hema nodos um, art hri t i s , s pl enomegal y, hepat omegal y, hypercal cemi a, di ffus e or l ocal i zed CNS i nvol vement , and cardi omyopat hy. A cons i s t ent i mmunol ogi c feat ure i s depres s i on of del ayed-t ype hypers ens i t i vi t y.
2. Incidence. Sarcoi dos i s can occur i n ei t her s ex at any age but appears mos t commonl y i n t he t hi rd t o fi ft h decades of l i fe. In t he Uni t ed St at es , t he i nci dence of s arcoi dos i s i s 10- t o 18-fol d hi gher i n bl acks t han i n whi t es .
Pa g e 3 9 0
ABC Ambe r CHM Conve rte r Tria l ve rsion, http://w w w .proce sste x t.com/a bcchm.html
3. Possible etiologic factors. The noncas eat i ng epi t hel i oi d granul oma of s arcoi dos i s s ugges t s a t i s s ue res pons e t o s ome focal i ns ul t . o
o
a. Infectious agents. Cl i ni cal and pat hol ogi c s i mi l ari t i es have s ugges t ed a connect i on wi t h t ubercul os i s and ot her mycobact eri al di s eas e. However, a fai l ure t o i dent i fy any i nfect i ous agent s cons i s t ent l y and l ack of an epi demi ol ogi c as s oci at i on have made any i nfect i ous caus e unl i kel y.
o
o
b. Immunologic defects
(1) Pat i ent s wi t h s arcoi dos i s s how i mpai red cel l ul ar i mmuni t y charact eri zed by a compl et e s ki n anergy t o t ubercul i n and ot her common s ki n ant i gens . The l evel of ci rcul at i ng T l ymphocyt es i s decreas ed, pos s i bl y becaus e of s eques t rat i on i n t he l ung, becaus e BAL t ypi cal l y s hows marked i ncreas es i n t hes e cel l s .
(2) The s i gni fi cance of t hes e i mmunol ogi c abnormal i t i es i s unknown. They may repres ent a fundament al l y abnormal i mmunol ogi c res pons i venes s , or i t may be t hat t he i mmunol ogi c changes are s econdary phenomena and t hat t he pri mary pat hol ogi c proces s remai ns t o be di s covered.
Pa g e 3 9 1
ABC Ambe r CHM Conve rte r Tria l ve rsion, http://w w w .proce sste x t.com/a bcchm.html
(3) Humoral i mmuni t y i s normal , and s us cept i bi l i t y t o i nfect i on i s not i ncreas ed.
4. Pathology o
o
a. The fundament al l es i on of s arcoi dos i s i s a noncaseating granuloma. Thi s cl us t er of epi t hel i oi d cel l s i s i ndi s t i ngui s habl e from t he granul omas occurri ng i n ot her di s eas es s uch as fungal di s eas e, mycobact eri al di s eas e, and Hodgki n's di s eas e.
o
o
b. Gi ant cel l s frequent l y are pres ent and cont ai n s everal t ypes of i ncl us i ons . Lymphocyt es and rare pl as ma cel l s may be pres ent at t he peri phery of t he granul oma; neut rophi l s and eos i nophi l s are abs ent .
5. Clinical features vary cons i derabl y, dependi ng on t he s i t e and ext ent of i nvol vement . o
o
a. Pulmonary involvement
(1) Symptoms. Fat i gue and exert i onal dys pnea are common. Cough, i f pres ent , us ual l y i s nonproduct i ve. Hemopt ys i s i s rare. Ches t pai n occurs i nfrequent l y, and pl euri s y i s uncommon. P.84
Pa g e 3 9 2
ABC Ambe r CHM Conve rte r Tria l ve rsion, http://w w w .proce sste x t.com/a bcchm.html
(2) Pulmonary function testing. Res ul t s may be normal but us ual l y s how s ome i mpai rment of gas exchange and s ome evi dence of l ung res t ri ct i on wi t h reduced VC and D LC O . In many cas es , s mal l ai rway funct i on al s o i s abnormal .
(3) Chest radiography. Enl arged i nt rat horaci c l ymph nodes are s een, part i cul arl y earl y i n t he cours e of t he di s eas e. Parenchymal mani fes t at i ons vary from a fai nt i nt ers t i t i al i nfi l t rat e, t o wel l -devel oped di ffus e nodul ar i nfi l t rat es , t o varyi ng degrees of l ung fi bros i s , i ncl udi ng “honeycombi ng.― The radiographic staging of pul monary i nvol vement i n s arcoi dos i s i s as fol l ows :
(a) Stage 1: Bi l at eral hi l ar adenopat hy and normal l ung parenchyma
(b) Stage 2: Bi l at eral hi l ar adenopat hy and i nt ers t i t i al i nfi l t rat e
(c) Stage 3: Int ers t i t i al i nfi l t rat e onl y
(d) Stage 4: Fi bros i s
o
o
b. Systemic involvement
Pa g e 3 9 3
ABC Ambe r CHM Conve rte r Tria l ve rsion, http://w w w .proce sste x t.com/a bcchm.html
(1) Uveitis i s a common pres ent at i on and may progres s t o bl i ndnes s .
(2) A vari et y of i nfi l t rat i ve skin lesions occurs i n one t hi rd of pat i ent s and oft en port ends chroni c progres s i ve s arcoi dos i s . An except i on i s erythema nodosum, whi ch may occur earl y i n t he di s eas e and i s as s oci at ed wi t h a good prognos i s .
(3) Bone and joint involvement. Trans i ent pol yart hri t i s i s as s oci at ed wi t h eryt hema nodos um; a chroni c form of art hri t i s al s o occurs . Bone i nvol vement may produce cys t i c des t ruct i on and di s abi l i t y.
(4) Nervous system involvement may mani fes t as Bel l 's pal s y and ot her crani al neuropat hi es , peri pheral neuropat hi es , and (rarel y) granul omat ous meni ngi t i s .
(5) Cardiomyopathy mani fes t s as arrhyt hmi as and conduct i on di s t urbances t hat carry a hi gh ri s k for s udden deat h.
(6) Liver function abnormalities may occur.
(7) Disturbances in calcium metabolism (e.g., hypercal ci uri a, renal s t ones , and hypercal cemi a; s ee Chapt er 6 II V) occur i n up t o 25% of pat i ent s .
Pa g e 3 9 4
ABC Ambe r CHM Conve rte r Tria l ve rsion, http://w w w .proce sste x t.com/a bcchm.html
6. Diagnosis o
o
a. Sarcoi dos i s s houl d be s us pect ed i n pat i ent s wi t h medi as t i nal or hi l ar adenopat hy and i nt ers t i t i al l ung di s eas e (e.g., pul monary fi bros i s ). Eryt hema nodos um, uvei t i s , s ki n l es i ons , hypercal cemi a, mul t i s ys t em di s eas e, and granul omas of any organ s houl d al s o s ugges t s arcoi dos i s .
o
o
b. Di agnos t i c confi rmat i on requi res t i s s ue bi ops y s howi ng t ypi cal granul omas i n a pat i ent wi t h cons i s t ent cl i ni cal pres ent at i ons ; t rans bronchi al bi ops y i s oft en di agnos t i c. Becaus e s arcoi dos i s i s a di agnos i s of excl us i on, al l t i s s ue s ampl es s houl d be cul t ured t o rul e out i nfect i ous caus es and s houl d be s peci al l y s t ai ned for i dent i fi cat i on of fungal di s eas e.
7. Clinical course and prognosis. The cours e of s arcoi dos i s i s vari abl e. Granul omas may remai n unchanged i n t i s s ue for many years , may regres s , or may organi ze, res ul t i ng i n t i s s ue fi bros i s . Chroni c i nfl ammat i on and fi bros i s i n t he l ung caus e s eri ous s t ruct ural di s t ort i on and l os s of funct i on. o
o
a. In mos t pat i ent s , t he di s eas e regres s es wi t hi n 2 years and does not recur. Any t i s s ue des t ruct i on t hat occurs i s permanent but us ual l y caus es no major di s abi l i t y.
o
Pa g e 3 9 5
ABC Ambe r CHM Conve rte r Tria l ve rsion, http://w w w .proce sste x t.com/a bcchm.html o
b. In approxi mat el y 25% of pat i ent s , t he di s eas e i s more progres s i ve and caus es s eri ous di s abi l i t y. Mul t i s ys t em i nvol vement i s common, wi t h s ki n s arcoi dos i s and hypercal cemi a part i cul arl y promi nent . Approxi mat el y 5% of pat i ent s di e of res pi rat ory fai l ure.
8. T herapy. Cort i cos t eroi d admi ni s t rat i on i s t he pri nci pal t reat ment for s arcoi dos i s . However, a deci s i on mus t be made as t o whet her a pat i ent 's s ympt oms warrant t herapy, as t herapy can be hazardous . o
o
a. Sarcoi dos i s t hat carri es a t hreat of di s abi l i t y s houl d be t reat ed. Indi cat i ons i ncl ude s ympt omat i c pul monary di s eas e, uvei t i s , hypercal cemi a, cardi ac s arcoi dos i s , and neurol ogi c s arcoi dos i s .
o
o
b. The pat i ent mus t be as s es s ed peri odi cal l y t o det ermi ne whet her cont i nuat i on of t reat ment i s warrant ed. Cl i ni cal obs ervat i on, pul monary funct i on t es t i ng, and ches t radi ography frequent l y are us ed t o eval uat e t he effect i venes s of t he t herapeut i c regi men.
P.85
B. Goodpasture's syndrome (see also Chapter 6 I X G)
1. Definition. Goodpas t ure's s yndrome i s a progres s i ve aut oi mmune di s eas e of t he l ungs and ki dneys t hat
Pa g e 3 9 6
ABC Ambe r CHM Conve rte r Tria l ve rsion, http://w w w .proce sste x t.com/a bcchm.html
produces intra-alveolar hemorrhage and glomerulonephritis. The di s eas e i s rare, occurs at al l ages , and i s predomi nant i n mal es .
2. Pathogenesis and pathology o
o
a. Goodpas t ure's s yndrome i s caus ed by an ant i –gl omerul ar bas ement membrane (ant i -GBM) ant i body, us ual l y IgG, whi ch react s wi t h gl omerul ar and al veol ar bas ement membranes .
o
o
b. Li near depos i t i on of t he ant i body, charact eri s t i c of a t ype II hypers ens i t i vi t y react i on, occurs al ong t he bas ement membrane of gl omerul i , al veol i , and capi l l ari es .
(1) The pat hol ogi c res ul t i n t he l ung i s di ffus e capi l l ary l eakage and i nt ra-al veol ar hemorrhage but l i t t l e or no i nfl ammat i on.
(2) The renal l es i on i s a prol i ferat i ve gl omerul onephri t i s t hat progres s es t o renal fai l ure.
3. Clinical features. Pat i ent s us ual l y pres ent wi t h hemopt ys i s and dys pnea. However, renal fai l ure wi t hout pul monary compl ai nt s can be an i ni t i al fi ndi ng, and a hi s t ory of res pi rat ory i l l nes s oft en precedes t he ons et of pul monary hemorrhage.
Pa g e 3 9 7
ABC Ambe r CHM Conve rte r Tria l ve rsion, http://w w w .proce sste x t.com/a bcchm.html
4. Diagnosis o
o
a. Bi l at eral al veol ar i nfi l t rat es on ches t radi ograph, hypoxemi a, and a res t ri ct i ve pat t ern on pul monary funct i on t es t i ng are charact eri s t i c.
o
o
b. The di agnos i s i s confi rmed by demons t rat i on of ant i -GBM ant i body i n t he s erum or i n a bi ops y s peci men from t he ki dney or l ung.
o
o
c. Differential diagnosis i ncl udes W egener's granul omat os i s , s ys t emi c l upus eryt hemat os us (SLE), and i di opat hi c pul monary hemos i deros i s .
(1) Wegener' s granulomatosis (s ee XIV C) us ual l y affect s bot h t he upper and l ower res pi rat ory t ract , and l acks ant i -GBM ant i body.
(2) SLE (s ee Chapt er 10 VIII) i s di s t i ngui s hed from Goodpas t ure's s yndrome by t he abs ence of ant i -GBM ant i body and t he fi ndi ngs of free DNA, vari ous ANAs , and depres s ed s erum l evel s of compl ement .
(3) Idiopathic pulmonary hemosiderosis i s charact eri zed by repeat ed pul monary hemorrhage but no nephri t i s . Deat h caus ed by mas s i ve hemorrhage may occur at any t i me, but prol onged s urvi val wi t h or wi t hout s ympt oms of pul monary i ns uffi ci ency i s common. Idi opat hi c pul monary hemos i deros i s
Pa g e 3 9 8
ABC Ambe r CHM Conve rte r Tria l ve rsion, http://w w w .proce sste x t.com/a bcchm.html
has no known i mmune mechani s ms for pat hogenes i s , and no s ucces s ful t herapy has evol ved.
5. Prognosis and therapy o
o
a. Unt reat ed Goodpas t ure's s yndrome i s rapi dl y fat al as a res ul t of renal fai l ure or as phyxi a from pul monary hemorrhage.
o
o
b. Current l y, t he combi nat i on of pl as mapheres i s t o remove ci rcul at i ng ant i -GBM ant i bodi es and i mmunos uppres s i ve t herapy wi t h cort i cos t eroi ds and al kyl at i ng agent s appears t o gi ve t he bes t res ul t s . Hi gh-dos e cort i cos t eroi d t herapy oft en cont rol s epi s odes of l ung hemorrhage but not t he progres s i ve renal di s eas e.
o
o
c. Bi l at eral nephrect omy wi t h hemodi al ys i s or ki dney t rans pl ant at i on has been us ed t o cont rol end-s t age renal di s eas e.
C. Wegener's granulomatosis (see also Chapter 6 Part I: X N 3)
1. Definition. Thi s di s eas e i s t he prot ot ype of a group of rare di s orders charact eri zed by granul omat ous i nfl ammat i on and necros i s of t he l ung and ot her organs . Indi vi dual s of al l ages may be affect ed; mal es are affect ed more commonl y t han femal es . o
Pa g e 3 9 9
ABC Ambe r CHM Conve rte r Tria l ve rsion, http://w w w .proce sste x t.com/a bcchm.html
o
a. W egener ori gi nal l y des cri bed t he s yndrome as a des t ruct i ve granul omat ous i nfi l t rat i on of t he upper res pi rat ory t ract and l ung parenchyma combi ned wi t h gl omerul onephri t i s .
o
o
b. Today t he di s eas e i s recogni zed as a s ys t emi c vas cul i t i s wi t h a predi l ect i on for t he res pi rat ory t ract and ki dney. Ot her commonl y i nvol ved s i t es are t he s ki n, joi nt s , and peri pheral nerves . Invol vement of t he eyes , heart , and CNS al s o can occur.
o
o
c. A vari ant form affect s t he res pi rat ory t ract , chi efl y t he l ungs , whi l e s pari ng t he ki dney.
P.86
2. Pathogenesis and pathology o
o
a. The pat hogenet i c mechani s m i s t hought t o be an i mmunol ogi c i njury of ves s el s , wi t h s econdary i nfl ammat ory changes .
o
o
b. The pat hol ogi c l es i on i s an angi i t i s of s mal l ves s el s wi t h charact eri s t i c t i s s ue necros i s s urrounded by mononuclear i nfl ammat ory cel l s , formi ng noncas eat i ng granul omas .
(1) In t he l ung, t hi s proces s commonl y
Pa g e 4 0 0
ABC Ambe r CHM Conve rte r Tria l ve rsion, http://w w w .proce sste x t.com/a bcchm.html
res ul t s i n excavat i on and des t ruct i on of t he l ung parenchyma, caus i ng hemopt ys i s and pul monary i ns uffi ci ency.
(2) The renal l es i on i s a focal gl omerul onephri t i s t hat can progres s t o renal fai l ure.
3. Diagnosis. W egener's granul omat os i s i s i dent i fi ed by t he cl as s i c cl i ni cal t ri ad of upper and l ower res pi rat ory i nvol vement and gl omerul onephri t i s , s upport ed by a pos i t i ve ant i -neut rophi l i c cyt opl as mi c ant i body (ANCA) t es t and bi ops y of t he i nvol ved t i s s ue.
4. Prognosis and therapy o
o
a. The unt reat ed di s eas e i s fat al i n mos t pat i ent s wi t hi n 1 mont h t o s everal years . Some forms of t he di s eas e are as s oci at ed wi t h l onger s urvi val rat es , part i cul arl y t hos e t hat do not i nvol ve act i ve nephri t i s .
o
o
b. Correct di agnos i s i s cri t i cal becaus e of t he remarkabl e effi cacy of cytotoxic therapy. Cycl ophos phami de al one or wi t h cort i cos t eroi ds produces rapi d revers al of t he di s eas e, i f i nt roduced earl y i n t he cours e.
D. Histiocytosis X (eosinophilic granuloma of the lung)
Pa g e 4 0 1
ABC Ambe r CHM Conve rte r Tria l ve rsion, http://w w w .proce sste x t.com/a bcchm.html
1. Definition. Hi s t i ocyt os i s X i s a generi c t erm for a group of s ys t emi c di s orders charact eri zed by vari ous degrees of fi bros i s wi t h focal i nfi l t rat i ons of t i s s ue by nonmal i gnant hi s t i ocyt es and eos i nophi l s . The di s eas e can be l ocal i zed t o one area (e.g., bone or lung) or i t can be wi del y di s s emi nat ed. Eosinophilic granuloma (of bone or l ung) i s l ocal i zed; Letterer-Siwe disease and Hand-Schüller-Christian syndrome are wi des pread.
2. Incidence. The di s eas e i s rare, affect s men more commonl y t han women, and affect s chi l dren and young adul t s more commonl y t han ot her age-groups . An abnormal i t y of t he i mmune s ys t em i s s us pect ed.
3. Pathology. Prol i ferat i ng hi s t i ocyt es s how cyt opl as mi c i ncl us i ons , t he s o-cal l ed “X bodi es .― Pul monary hi s t i ocyt os i s X produces bi l at eral , ret i cul onodul ar i nfi l t rat es , wi t h a predi l ect i on for t he upper l obes and t ypi cal progres s i on t o cys t format i on, fi bros i s , and “honeycombi ng.―
4. Clinical features and diagnosis o
o
a. Findings may i ncl ude cough, ches t pai n, dys pnea, fever, s pont aneous pneumot horax, and a “honeycomb― appearance on ches t radi ography. Lyt i c bone di s eas e may be pres ent . A t ri ad of di abet es i ns i pi dus , exopht hal mos , and bone l es i ons i s s een occas i onal l y.
o
o
b. Pulmonary function testing i ndi cat es res t ri ct i on
Pa g e 4 0 2
ABC Ambe r CHM Conve rte r Tria l ve rsion, http://w w w .proce sste x t.com/a bcchm.html
and i mpai red gas exchange. In advanced cas es , s evere obs t ruct i on may domi nat e. o
o
c. Definitive diagnosis requi res bi ops y of i nvol ved t i s s ue or el ect ron mi cros copi c demons t rat i on of X bodi es i n BAL fl ui d.
5. T herapy and prognosis. Cort i cos t eroi ds are gi ven for pul monary mani fes t at i ons , but t hei r effi cacy i s uncert ai n. Surgery or radi at i on t herapy i s us ed for l ocal i zed bone di s eas e. The prognos i s i s vari abl e—s ome cas es res ul t i n deat h, but s pont aneous remi s s i ons are common.
E. Alveolar proteinosis
1. Definition. Al veol ar prot ei nos i s i s a rare di s eas e charact eri zed by mas s i ve accumul at i ons of a phos phol i pi d- and prot ei n-ri ch s ubs t ance i n al veol i . The i nt ers t i t i um us ual l y i s not i nvol ved, and t here i s no underl yi ng di s eas e or ot her organ i nvol vement . The di s order i s more common i n men t han i n women and has been des cri bed i n al l ages .
2. Pathology. The s ubs t ance i n t he al veol i i s cl os el y rel at ed t o pul monary s urfact ant and probabl y accumul at es as a res ul t of i mpai red cl earance. The i mpai red cl earance may be due t o defi ci ency or defect of granul ocyt e macrophage col ony-s t i mul at i ng fact or (GM-CSF) i n s ome cas es . Macrophages engorged wi t h t he s ubs t ance al s o are pres ent , but ot her i nfl ammat ory cel l s are l acki ng.
Pa g e 4 0 3
ABC Ambe r CHM Conve rte r Tria l ve rsion, http://w w w .proce sste x t.com/a bcchm.html
3. Clinical features o
o
a. Findings. Dys pnea, nonproduct i ve cough, pul monary ral es , and cyanos i s are common. P.87
o
o
b. Clinical course and prognosis
(1) Pat i ent s are predi s pos ed t o l ung i nfect i on, i ncl udi ng nocardi os i s and fungal i nfect i ons , pos s i bl y becaus e of a funct i onal i mpai rment of al veol ar macrophages .
(2) The di s eas e may progres s t o res pi rat ory i ns uffi ci ency and deat h, but s pont aneous res ol ut i on i s jus t as common. Pul monary fi bros i s has been des cri bed as a l at e compl i cat i on.
4. Diagnosis. Pul monary funct i on t es t i ng s hows a res t ri ct i ve vent i l at ory pat t ern and hypoxi a. Ches t radi ograph s hows an al veol ar i nfi l t rat e, us ual l y i n a bi l at eral peri hi l ar but t erfl y di s t ri but i on s i mi l ar t o t he pat t ern s een i n pul monary edema. Lung bi ops y or BAL i s neces s ary t o demons t rat e t he peri odi c aci d–Schi ff (PAS)–pos i t i ve mat eri al i n t he al veol i .
Pa g e 4 0 4
ABC Ambe r CHM Conve rte r Tria l ve rsion, http://w w w .proce sste x t.com/a bcchm.html
5. T herapy. Pat i ent s wi t h mi ni mal s ympt oms requi re no t herapy. For dys pnei c pat i ent s , whol e l ung l avage i s effect i ve and revers es t he phys i ol ogi c abnormal i t y. Cort i cos t eroi ds are cont rai ndi cat ed becaus e t hey i ncreas e t he ri s k of i nfect i on.
F. Bronchiolitis obliterans-organizing pneumonia (BOOP)
1. Definition and pathology. BOOP, al s o cal l ed crypt ogeni c organi zi ng pneumoni a (COP), i s a cl i ni copat hol ogi c s yndrome i nvol vi ng granul at i on t i s s ue wi t hi n s mal l ai rways and al veol ar duct s . It i s as s oci at ed wi t h chroni c i nfl ammat i on i n t he s urroundi ng al veol i .
2. Etiology. Vi ral i nfect i on, t oxi c i nhal at i on, rheumat oi d art hri t i s , and ot her col l agen vas cul ar di s eas es , as wel l as drugs , are s ome of t he pot ent i al caus es of BOOP.
3. Clinical features o
o
a. BOOP affect s bot h men and women. The mean age at pres ent at i on i s 58 years ; pat i ent s range i n age from 21 t o 80 years .
o
o
b. The di s eas e us ual l y pres ent s as a s ubacut e fl u-l i ke i l l nes s wi t h coughi ng, fever, mal ai s e, fat i gue, and wei ght l os s . Ins pi rat ory crackl es are frequent l y pres ent .
Pa g e 4 0 5
ABC Ambe r CHM Conve rte r Tria l ve rsion, http://w w w .proce sste x t.com/a bcchm.html
4. Diagnosis o
o
a. Chest radiography shows bi l at eral , peri pheral , wedged-s haped al veol ar opaci t i es .
o
o
b. Pulmonary function testing s hows a res t ri ct i ve defect .
o
o
c. Lung biopsy i s t he defi ni t i ve way t o make a di agnos i s .
5. T herapy. In t wo-t hi rds of pat i ent s , cort i cos t eroi d t herapy res ul t s i n rapi d and compl et e recovery.
XV. Sleep Apnea Syndrome A. Definition Sl eep apnea i s a di s order charact eri zed by repet i t i ve peri ods of apnea (i .e., ces s at i on of breat hi ng) occurri ng duri ng s l eep. A peri od of more t han 10 s econds wi t hout ai rfl ow i s cons i dered an apnei c epi s ode. Pat i ent s wi t h t hi s s yndrome can have hundreds of s uch epi s odes duri ng t he cours e of one ni ght 's s l eep. Thi s di s order can be demons t rat ed i n 9% of mi ddl e-aged women and 24% of mi ddl e-aged men.
B. Etiology and pathophysiology Sl eep apnea may be obs t ruct i ve, cent ral , or mi xed.
1. In obstructive sleep apnea, t rans i ent obs t ruct i on of t he upper ai rway, us ual l y t he oropharynx, prevent s i ns pi rat ory ai rfl ow. The obs t ruct i on res ul t s from l os s of t one i n t he pharyngeal mus cl es or t he geni ogl os s us mus cl es (whi ch normal l y caus e t he t ongue t o prot rude
Pa g e 4 0 6
ABC Ambe r CHM Conve rte r Tria l ve rsion, http://w w w .proce sste x t.com/a bcchm.html
forward from t he pos t eri or pharyngeal wal l ).
2. In central apnea, t here i s no dri ve t o breat he duri ng t he apnei c epi s ode; t hat i s , t here i s no s i gnal from t he res pi rat ory cent er t o i ni t i at e i ns pi rat i on. Rarel y, t he caus e i s a neurol ogi c di s order. W hy t he dri ve i s abs ent i n ot her i ndi vi dual s i s not known.
3. In mixed apnea, pat i ent s have epi s odes of bot h obs t ruct i ve and cent ral apnea. Pol ys omnography (s l eep s t udy) wi l l del i neat e t he di fferent forms of apnea pres ent i n any gi ven pat i ent .
C. Clinical features
1. Symptoms. Us ual l y, s l eep part ners not i ce pat i ent s ' probl ems . Loud snoring i s promi nent , and pat i ent s may t hras h about duri ng peri ods of obs t ruct i ve apnea. Duri ng t he dayt i me, t hey are overly somnolent and may s how pers onal i t y changes or s l owed ment at i on. P.88
2. Physical signs. In pat i ent s wi t h central apnea, moni t ori ng of t he ches t wal l mot i on reveal s no movement ; t hi s corres ponds t o ces s at i on of ai rfl ow and oxygen des at urat i on. In pat i ent s wi t h obstructive apnea, ches t wal l and abdomi nal movement can be det ect ed duri ng frui t l es s at t empt s t o move ai r t hrough t he obs t ruct ed ai rway.
Pa g e 4 0 7
ABC Ambe r CHM Conve rte r Tria l ve rsion, http://w w w .proce sste x t.com/a bcchm.html
3. Obes i t y hypovent i l at i on s yndrome (OHS) occas i onal l y may coexi s t wi t h OSA. The OHS i s charact eri zed by ext reme obes i t y and al veol ar hypovent i l at i on duri ng wakeful nes s .
D. Therapy
1. Central apnea. Treat ment i nvol ves res pi rat ory s t i mul ant s . A phreni c nerve pacemaker may be i mpl ant ed t o s t i mul at e t he di aphragm el ect ri cal l y.
2. Obstructive apnea. In t hos e pat i ent s requi ri ng i nt ervent i on, nas al CPAP i s t he mos t common t reat ment . Oral appl i ances or s urgery t o debul k t he pos t eri or pharynx may be requi red on occas i on. Rarel y, i n s evere cas es , t racheos t omy t o bypas s t he upper ai rway may be requi red. W ei ght l os s i s i ndi cat ed i n mos t pat i ent s , but wi t hout bari at ri c s urgery, i s rarel y s ucces s ful .
E. Complications Seri ous compl i cat i ons i ncl ude cardi ac arrhyt hmi as , pul monary hypert ens i on, and unexpl ai ned cor pul monal e. P.89
Study Questions/Answers and Explanations 1. A 32-year-old man working on a construction crew falls from the scaffolding and develops multiple long bone fractures. Four hours after presenting to the emergency department, he complains of increased shortness of breath, hypoxemia, and has diffuse infiltrates on his chest radiograph, consistent with development of acute respiratory distress syndrome. Which of the following findings is almost always present in patients who
Pa g e 4 0 8
ABC Ambe r CHM Conve rte r Tria l ve rsion, http://w w w .proce sste x t.com/a bcchm.html
present with ARDS? A A s mal l l ocal i zed mas s on ches t radi ograph B Reduced l ung compl i ance C Normal oxygenat i on wi t h i mpai red mi nut e vent i l at i on D Increas ed art eri al PCO 2 E Pul monary embol i s m Vi ew Ans wer 1. T he answer is B [VI C–D]. Acut e res pi rat ory di s t res s s yndrome (ARDS, “wet l ung―) begi ns wi t h a di s rupt i on of capi l l ary i nt egri t y, whi ch l eads t o ext ravas at i on of fl ui d, fi bri n, and prot ei n i nt o t he al veol i . As a res ul t , t he l ungs become wet and s t i ff (i .e., noncompl i ant ). Thi s condi t i on i s charact eri zed by s evere hypoxi a caus ed by ext reme vent i l at i on–perfus i on ([V wi t h dot above]/[Q wi t h dot above]) i mbal ance and s hunt i ng of bl ood i n t he fl ui d-fi l l ed areas of t he l ung. Cl i ni cal feat ures i ncl ude progres s i ve t achypnea; pat chy, di ffus e, fl uffy i nfi l t rat es on ches t radi ograph; i ncreas ed mi nut e vent i l at i on; and decreas ed l ung vol umes . There us ual l y i s an abs ence of s peci fi c phys i cal fi ndi ngs . 2. An 18-year-old girl complains of symptoms of an upper respiratory tract infection for 1 to 2 days, followed by an increase in shortness of breath and greater use of inhaled bronchodilators for treatment of her chronic stable asthma. She presents to the emergency room with increased use of accessory muscles of ventilation, tachypnea, and wheezing. Shortly after the symptoms of her asthmatic attack have resolved, pulmonary function testing is most likely to show which of the following? A Normal val ues for peak expi rat ory fl ow B Decreas ed l ung compl i ance C Increas ed res i dual vol ume (RV) D No change i n peak expi rat ory fl ow aft er i nhal at i on of bronchodi l at or E El evat ed FEV 1 /FVC rat i o Vi ew Ans wer 2. T he answer is C [III F 4 a–b]. Pat i ent s who have had a recent
Pa g e 4 0 9
ABC Ambe r CHM Conve rte r Tria l ve rsion, http://w w w .proce sste x t.com/a bcchm.html
as t hmat i c at t ack, even t hough as ympt omat i c, s t i l l have res i dual ai rfl ow obs t ruct i on t hat may t ake a coupl e of mont hs t o di s appear. Duri ng t hi s t i me, pat i ent s s t i l l res pond t o bronchodi l at ors but s how abnormal peak expi rat ory fl ow, i ncreas ed l ung compl i ance, and cont i nued mal di s t ri but i on of vent i l at i on. The FEV 1 /FVC rat i o i s ei t her normal or decreas ed (due t o obs t ruct i on) i n as t hma. It i s oft en i ncreas ed i n res t ri ct i ve l ung di s eas es . 3. A 16-year-old child presents with recurrent respiratory tract infections, poor weight gain, and breathlessness on exertion. Which of the following combinations of findings would provide a definite diagnosis of cystic fibrosis? A A fami l y hi s t ory of cys t i c fi bros i s : Abnormal pul monary funct i on B Abnormal pul monary funct i on: Pancreat i c i ns uffi ci ency C Pancreat i c i ns uffi ci ency: Hi gh el ect rol yt e concent rat i on and s weat D Hi gh el ect rol yt e, concent rat i on, and s weat : abnormal ches t radi ograph E Abnormal ches t radi ograph; fami l y hi s t ory of cys t i c fi bros i s Vi ew Ans wer 3. T he answer is C [IV B 5 a (1)–(2) (a)–(c)]. Alt hough ches t radi ography and pul monary funct i on t es t i ng s how abnormal i t i es , t he s weat t es t i s t he defi ni t i ve t es t for cys t i c fi bros i s . In vi rt ual l y al l cas es of cys t i c fi bros i s , s odi um and chl ori de concent rat i ons i n s weat are i ncreas ed s i gni fi cant l y, whereas concent rat i ons of t hes e el ect rol yt es are normal el s ewhere i n t he body. To make a di agnos i s of cys t i c fi bros i s , t hi s defect mus t be i dent i fi ed. The di agnos i s i s confi rmed by a pos i t i ve s weat t es t combi ned wi t h any one of t he fol l owi ng fi ndi ngs : a fami l y hi s t ory of cys t i c fi bros i s , obs t ruct i ve pul monary di s eas e, or pancreat i c i ns uffi ci ency. 4. A 19-year-old man presents with fatigue, dyspnea, and a mild nonproductive cough. Pulmonary function testing shows a mild decrease in the FVC and FEV 1 with a preserved FEV 1 /FVC ratio. D L C O is mildly decreased. Chest radiograph shows bilateral hilar adenopathy and mild interstitial infiltrates. Which of the following statements is true regarding his most likely diagnosis? A Open l ung or vi deo-as s i s t ed t horacos copi c (VATS) bi ops y wi l l be
Pa g e 4 1 0
ABC Ambe r CHM Conve rte r Tria l ve rsion, http://w w w .proce sste x t.com/a bcchm.html
requi red t o confi rm t he di agnos i s B Mos t pat i ent s experi ence a prol onged downhi l l cours e wi t h res pi rat ory fai l ure l i kel y wi t hi n 5 years C Mani fes t at i ons of t he di s eas e are confi ned t o t he l ungs D Cort i cos t eroi ds are t he pri mary t herapy E The pres ence of eryt hema nodos um at t he t i me of di agnos i s port ends a poor prognos i s . Vi ew Ans wer 4. T he answer is D [XIV A 5 a (1)–(3), b (1)–(7), 6–8]. The pres ence of fat i gue, dys pnea, and cough combi ned wi t h res t ri ct i ve defect on pul monary funct i on t es t i ng and hi l ar adenopat hy and i nt ers t i t i al i nfi l t rat es on ches t x-ray s t rongl y s ugges t s s arcoi dos i s . The di agnos i s of s arcoi dos i s frequent l y requi res bi ops y, but open l ung bi ops y i s us ual l y not requi red as t rans bronchi al bi ops y i s t ypi cal l y s uffi ci ent . Al t hough t he l ungs are i nvol ved i n 90% of cas es , al mos t any s ys t em can be i nvol ved i n s arcoi dos i s , i ncl udi ng cardi ac, neurol ogi c, s ki n, renal , and s kel et al s ys t ems . In mos t cas es , t he di s eas e regres s es wi t hi n 2 years , ei t her wi t h or wi t hout cort i cos t eroi ds , t he preferred t reat ment . In 25% t he di s eas e may progres s t o fi bros i s and di s abl i ng pul monary di s eas e. Eryt hema nodos um s i gni fi es a favorabl e prognos i s when pres ent . 5. A 36-year-old woman is seen for newly developed lower extremity edema. On further questioning, she admits to progressively worsening exercise tolerance for the past 2–3 years. She has had lightheadedness on occasion, and once had a syncopal episode while rushing for the bus. Her lungs are clear, and heart examination reveals a right ventricular heave and a right-sided fourth heart sound (S 4 ). She is presumed to have primary pulmonary hypertension (PPH), and further confirmatory testing is ordered. Which of the following statements about PPH in this patient is true? A If her pul monary art ery pres s ure and cardi ac out put res pond t o vas odi l at ors duri ng cardi ac cat het eri zat i on, t hen s he wi l l l i kel y res pond t o hi gh-dos e cal ci um channel bl ockers . B Echocardi ography provi des l i t t l e di agnos t i c i nformat i on and
Pa g e 4 1 1
ABC Ambe r CHM Conve rte r Tria l ve rsion, http://w w w .proce sste x t.com/a bcchm.html
s houl d not be performed unt i l l at e i n her work-up, i f at al l . P.90
C Ant i coagul ant t herapy i s cont rai ndi cat ed i n PPH due t o t he ri s k of bl eedi ng wi t h t hi s di s eas e. D If PPH i s confi rmed, her expect ed 5-year s urvi val i s excel l ent . E Syncope occurs earl y i n t he cours e of PPH when pul monary art ery pres s ures are s t i l l rel at i vel y l ow. Vi ew Ans wer 5. T he answer is A [VII E 3, F 1, G 1–2, H 5 a]. Syncope i s a l at e mani fes t at i on of PPH and s ugges t s s everel y el evat ed pul monary art ery pres s ures . The i ni t i al res pons e t o a vas odi l at or duri ng ri ght -heart cat het eri zat i on i s as s oci at ed wi t h a bet t er prognos i s and a favorabl e res pons e t o l ong-t erm cal ci um channel bl ockers . Echocardi ography can es t i mat e pul monary art ery pres s ure and as s es s ri ght vent ri cul ar funct i on and i s a us eful i ni t i al t es t t o confi rm t he di agnos i s of PPH. Ant i coagul ant t herapy has been s hown t o prol ong s urvi val and i s pres cri bed for nearl y al l pat i ent s wi t h PPH. The prognos i s for PPH i s general l y poor, wi t h a 5-year s urvi val rat e of approxi mat el y 34%. 6. Chronic obstructive pulmonary disease (COPD) is classified as emphysematous or bronchitic, depending on the pathologic changes that occur in the lung. Although these two COPD syndromes rarely exist as pure entities, they may be differentiated on the basis of their clinical presentation. Which of the following clinical features is common to both the emphysematous and bronchitic types of COPD? A Pol ycyt hemi a B Improved ai rfl ow wi t h bronchodi l at ors C Dys pnea D Chroni c cough E Hypercapni a Vi ew Ans wer 6. T he answer is C [II E 1, 2 a (1)–(2)]. Al l pat i ent s wi t h chroni c
Pa g e 4 1 2
ABC Ambe r CHM Conve rte r Tria l ve rsion, http://w w w .proce sste x t.com/a bcchm.html
obs t ruct i ve pul monary di s eas e (COPD) experi ence dys pnea t o s ome degree. The t wo cl as s i c t ypes of COPD—emphys emat ous and bronchi t i c—repres ent ext remes of t he s pect rum and rarel y are encount ered i n t hei r pure form i n cl i ni cal pract i ce. By defi ni t i on, i ndi vi dual s wi t h t he emphys emat ous t ype of COPD pres ent at a rel at i vel y ol der age (>60 years ). Thi s form of di s eas e i s charact eri zed by progres s i ve exert i onal dys pnea, wei ght l os s , l i t t l e or no cough, mi l d hypoxi a, hypocapni a, and onl y a mi l d i ncreas e i n ai rway res i s t ance (Raw) t hat s hows l i t t l e i mprovement wi t h bronchodi l at i on. Pers ons wi t h t he bronchi t i c t ype of COPD pres ent at a rel at i vel y young age. Thi s form i s charact eri zed by epi s odi c dys pnea, fl ui d ret ent i on, chroni c cough, s evere hypoxemi a, hypercapni a, pol ycyt hemi a, and an i ncreas e i n Raw t hat i mproves wi t h bronchodi l at i on. 7. A 70-year-old man complains of increased shortness of breath which has progressed over the previous 2 years. Chest radiographs show bilateral basilar interstitial infiltrates. T he patient is noted to have hypoxemia on ambulation. It is considered that the patient may have early manifestations of idiopathic pulmonary fibrosis, or usual interstitial pneumonitis (UIP). Pulmonary function data in this patient would most likely show: A Low l ung vol umes B A decreas e i n t he forced expi rat ory vol ume i n 1 s econd t o forced vi t al capaci t y (FEV 1 /FVC) rat i o C An i ncreas ed vi t al capaci t y (VC) D A decreas ed di ffus i ng capaci t y (D LC O ) E An i ncreas ed ai rways res i s t ance (Raw) Vi ew Ans wer 7. T he answer is A
[I F 2]. Res t ri ct i ve di s orders are
charact eri zed by l ow l ung vol umes . D LC O may or may not be decreas ed i n pul monary fi bros i s , a t ype of res t ri ct i ve di s eas e. A decreas e i n forced expi rat ory vol ume i n 1 s econd/forced vi t al capaci t y (FEV 1 /FVC) i s t he hal l mark of obs t ruct i ve, not res t ri ct i ve, di s eas e. Vi t al capaci t y (VC) and ai rway res i s t ance (Raw) are
Pa g e 4 1 3
ABC Ambe r CHM Conve rte r Tria l ve rsion, http://w w w .proce sste x t.com/a bcchm.html
decreas ed i n res t ri ct i ve l ung di s orders . 8. A 62-year-old woman with congestive heart failure (CHF) develops pneumonia and a large pleural effusion. T horacentesis is performed in an effort to establish whether the pleural effusion is due to CHF or pneumonia. Which of the following findings would indicate that the pleural effusion is due to CHF? A A prot ei n cont ent of 6 g/dL B A pH of 7.13 C A gl ucos e cont ent of 20 mg/dL D A l act at e dehydrogenas e (LDH) cont ent of 100 mg/dL (wi t h a s erum LDH l evel of 420 mg/dL) E A pl eural fl ui d t o s erum prot ei n rat i o of 0.7 Vi ew Ans wer 8. T he answer is D [IX A 2 (b), 4 (a); Tabl e 2-4]. W i t h t he except i on of t he l act at e dehydrogenas e (LDH) fi ndi ngs , al l of t he pl eural fl ui d fi ndi ngs l i s t ed i ndi cat e t he pres ence of an exudat e. Exudat es are caus ed by i nfl ammat i on or di s eas e of t he pl eural s urface or by l ymphat i c obs t ruct i on (e.g., due t o t ubercul os i s , l ung cancer, or pneumoni a). Trans udat es are caus ed by el evat ed s ys t emi c or pul monary venous pres s ure or by decreas ed pl as ma oncot i c pres s ure [e.g., due t o conges t i ve heart fai l ure (CHF) or nephrot i c s yndrome]. Therefore, i n es t abl i s hi ng t he et i ol ogy of a pl eural effus i on, i t i s us eful t o det ermi ne whet her t he fl ui d i s a t rans udat e or an exudat e. Thi s det ermi nat i on oft en can be made on t he bas i s of a chemi cal anal ys i s of t he pl eural fl ui d. A pl eural fl ui d prot ei n cont ent of more t han 2.9 g/dL and an LDH cont ent of more t han 250 mg/dL us ual l y i ndi cat e t he pres ence of an exudat e. In addi t i on, an exudat e us ual l y i s as s oci at ed wi t h a pl eural fl ui d t o s erum prot ei n rat i o of l es s t han 0.5 and a pl eural fl ui d t o s erum LDH rat i o of l es s t han 0.6. Pl eural fl ui d pH val ues bel ow 7.2 and a pl eural fl ui d gl ucos e cont ent of l es s t han 20 mg/dL al s o are as s oci at ed wi t h i nfl ammat ory effus i ons (exudat es ). P.91
P.92
Pa g e 4 1 4
ABC Ambe r CHM Conve rte r Tria l ve rsion, http://w w w .proce sste x t.com/a bcchm.html
Pa g e 4 1 5
ABC Ambe r CHM Conve rte r Tria l ve rsion, http://w w w .proce sste x t.com/a bcchm.html
Editors: Wolfsthal, Susan T itle: NMS Medicine, 6th Edition Copyri ght ©2008 Li ppi ncot t W i l l i ams & W i l ki ns > T able of Cont ent s > Chapt er 3 - Hemat ologic Diseases
Chapter 3
Hematologic Diseases Yanis Bellil Ann Zimrin
I. Red Blood Cell (RBC) Disorders (Online Figure 3-1)
Online Figure 3-1. Normal peri pheral bl ood s mear wi t h normal pl at el et s , l ymphocyt e, and mat ure neut rophi l . The red bl ood cel l s are uni form i n s i ze and s hape wi t h a di amet er t hat approxi mat es t he nucl eus of a l ymphocyt e. Not e t hat t he cent ral pal l or i s approxi mat el y one-t hi rd of a red bl ood cel l 's di amet er. (From Armi t age JO, ed. At l as of Cl i ni c al Hemat ol ogy. Phi l adel phi a: Li ppi ncot t W i l l i ams & W i l ki ns /Current Medi ci ne, Inc. 2004:6–3.)
A. Anemia caused by abnormal hemoglobin synthesis and iron metabolism Hemogl obi n, whi ch repres ent s 95% of t he t ot al compos i t i on of an
Pa g e 4 1 6
ABC Amber CHM Converter Trial version, http://www.processtext.com/abcchm.html
RBC, i s a mi xt ure of gl obi n and t he i ron-cont ai ni ng heme compound, protoporphyrin. Any abnormal i t y i n hemogl obi n s ynt hes i s or i ron met abol i s m res ul t s i n hemogl obi n-defi ci ent cel l s . As a rul e, s uch defi ci ent cel l s exhi bi t hypochromia (i .e., di mi ni s hed hemogl obi n concent rat i on) and microcytosis (i .e., di mi ni s hed s i ze). Bot h condi t i ons may be det ect ed us i ng t he RBC i ndi ces avai l abl e by cal cul at i on and t he Coul t er count er al ong wi t h an exami nat i on of a s t ai ned bl ood s mear. Di s orders caus i ng t hi s t ype of anemi a fal l i nt o four major cl as s es .
1. Iron deficiency anemia i s t he mos t common form of anemi a i n t he Uni t ed St at es , where 20% of adul t women are report ed t o be i ron defi ci ent . o
o
a. Etiology. Iron defi ci ency i s mos t commonl y caus ed by bl ood l os s when t he l os s of t he i ron component exceeds di et ary i nt ake of i ron. Exampl es i ncl ude gas t roi nt es t i nal bl ood l os s from an ul cer or a t umor and mens t rual bl ood l os s . Occas i onal l y, i n t he neonat e and young chi l d, new bl ood format i on and s ubs equent i ncreas ed i ron us e exceeds i ron i nt ake and res ul t s i n anemi a wi t hout concomi t ant bl ood l os s .
o
o
b. Clinical manifestations. The s ympt oms of i ron defi ci ency anemi a, l i ke al l anemi as , i ncl ude fat i gue and weaknes s . Sympt oms s peci fi c t o i ron defi ci ency may i ncl ude epi t hel i al changes s uch as bri t t l e nai l s and at rophi c t ongue. In addi t i on, t he underl yi ng pat hol ogy may domi nat e t he s ympt oms (e.g., pept i c ul cer).
o
c. Diagnosis. In t he appropri at e
Page 417
ABC Ambe r CHM Conve rte r Tria l ve rsion, http://w w w .proce sste x t.com/a bcchm.html
cl i ni cal s et t i ng (e.g., i n a young woman wi t h exces s i ve mens t rual bl ood l os s and anemi a), a s mear s howi ng hypochromi a and mi crocyt os i s i s adequat e for t he di agnos i s (
Onl i ne
Fi gure 3-2). W hen more s peci fi c t es t s are needed, a pos i t i ve di agnos i s can be made from l ow l evel s of ferri t i n, and a l ow s erum i ron l evel i n as s oci at i on wi t h an el evat ed t ot al i ron-bi ndi ng capaci t y. o
ONLINE FIGURE 3-2 Peri pheral bl ood s mear cons i s t ent wi t h i ron defi ci ency anemi a. There i s s i gni fi cant hypochromas i a and mi crocyt os i s . Not e t he s mal l s i ze of t he eryt hrocyt es as compared t o t he nucl ei of mat ure l ymphocyt es . (From Tkachuk DC, Hi rs chmann JV. Wi nt robe's At l as of Cl i ni c al Hemat ol ogy. Bal t i more: Li ppi ncot t W i l l i ams & W i l ki ns , 2007:1–4. ) o
o
d. T herapy. Treat ment us ual l y i nvol ves res t orat i on of t he body's i ron s t ores t o correct t he anemi a. As a rul e, oral i ron i n t he form of ferrous s ul fat e s uffi ces ; however, for pat i ent s who do not t ol erat e
Pa g e 4 1 8
ABC Ambe r CHM Conve rte r Tria l ve rsion, http://w w w .proce sste x t.com/a bcchm.html
t hi s form, ferrous gl uconat e and fumarat e are avai l abl e. In di ffi cul t cas es t hat are refract ory, ei t her phys i ol ogi cal l y or due t o an i nabi l i t y t o t ake oral i ron, parent eral i ron preparat i ons are gi ven. Ret i cul ocyt os i s occurs 7 days aft er appropri at e t reat ment ; aft er 3 weeks , t he hemogl obi n l evel i ncreas es s everal grams .
2. Anemia of chronic disease. Thi s mi l d-t o-moderat e anemi a i s as s oci at ed wi t h i nfl ammat ory di s eas es s uch as rheumat oi d art hri t i s , s eri ous i nfect i ons , and carci noma. o
o
a. Pathophysiology
(1) Anemi a of chroni c di s eas e i s charact eri zed by hemogl obi n l evel s of 8–10 g/dL, al t hough l ower l evel s are pos s i bl e. It i s unus ual for t he hemogl obi n l evel t o be l es s t han 7 g/dL.
(2) Pat i ent s wi t h t hi s anemi a have pl ent i ful i ron but di mi ni s hed i ron ut i l i zat i on by t he bone marrow. Therefore, inadequat e amount s of i ron are avai l abl e t o t he bone marrow for RBC format i on des pi t e adequat e body s t ores .
(3) Anot her i mport ant mechani s m for t hi s anemi a i s an i mpai red marrow res pons e t o eryt hropoi et i n. Hepci di n, a 25–ami no aci d pept i de, has been found t o pl ay a key rol e i n P.94
Pa g e 4 1 9
ABC Ambe r CHM Conve rte r Tria l ve rsion, http://w w w .proce sste x t.com/a bcchm.html
t he condi t i on by cont rol l i ng t he effl ux of i ron from s t orage s i t es . Hepci di n l evel s i ncreas e duri ng i nfl ammat i on, res ul t i ng i n decreas ed i ron concent rat i on i n t he bl ood. o
o
b. Diagnosis i s bas ed on confi rmat i on of t he fol l owi ng fi ndi ngs :
(1) The anemi a as s oci at ed wi t h chroni c i nfl ammat i on i s of moderat e degree and reveal s s l i ght hypochromi a and mi crocyt os i s .
(2) The mean corpus cul ar vol ume (MCV) i s 80–85 fL, and t he mean corpus cul ar hemogl obi n concent rat i on (MCHC) i s 30–32 g/dL.
(3) If s t ai ned, t he marrow reveal s pl ent i ful i ron s t ores . In addi t i on, s erum ferri t i n l evel s us ual l y are normal or el evat ed. Serum i ron i s l owered as i s t he t ot al i ron-bi ndi ng capaci t y, unl i ke t he cl i ni cal s i t uat i on wi t h i ron defi ci ency anemi a.
o
o
c. T herapy. Hemat i ni cs , i ncl udi ng i ron, are not effect i ve t reat ment for t hi s di s order.
(1) Correction of the underlying disease can l ead t o revers al of t he anemi a wi t hi n 1 mont h.
Pa g e 4 2 0
ABC Ambe r CHM Conve rte r Tria l ve rsion, http://w w w .proce sste x t.com/a bcchm.html
(2) Exogenous erythropoietin i s effect i ve i n cert ai n s i t uat i ons s uch as peri operat i vel y i n rheumat oi d pat i ent s undergoi ng joi nt repl acement s urgery, i n acqui red i mmunodefi ci ency s yndrome (AIDS), and in i nfl ammat ory bowel di s eas es . Becaus e t he i nfl ammat ory s t at e i nduces a rel at i ve hypos ens i t i vi t y t o eryt hropoi et i n, generous dos es may be needed.
3. Sideroblastic anemias o
o
a. Pathophysiology. Thes e anemi as , caus ed by di s orders i n t he s ynt hes i s of t he heme moi et y of hemogl obi n, are charact eri zed by t rapped i ron i n t he mi t ochondri a of nucl eat ed RBCs . Many of t he enzymes for prot oporphyri n s ynt hes i s are l ocat ed i n t he nucl eat ed RBC mi t ochondri a. Therefore, derangement s i n t hes e pat hways caus e i ron accumul at i on i n t he peri nucl ear mi t ochondri a, whi ch renders t hi s anemi a i t s charact eri s t i c morphol ogi c fi ndi ng of ringed sideroblasts. The defect i ve heme s ynt hes i s caus es di mi ni s hed hemogl obi n l evel s i n t hes e cel l s ; as a res ul t , t hi s cel l popul at i on i s hypochromi c and mi crocyt i c.
o
o
b. T ypes. There are t wo t ypes of s i derobl as t i c anemi as .
(1) Hereditary sideroblastic anemia. Thi s X-l i nked condi t i on i s due t o an abnormal i t y i n
Pa g e 4 2 1
ABC Ambe r CHM Conve rte r Tria l ve rsion, http://w w w .proce sste x t.com/a bcchm.html
pyri doxi ne (vi t ami n B 6 ) met abol i s m. It i s t hought t o be a congeni t al defect i n t he enzyme δ-aminolevulinic acid (ALA) synthetase.
(2) Acquired sideroblastic anemias are more common t han t he heredi t ary t ype. Lead, al cohol , and t he ant i bact eri al drug i s oni azi d caus e s i derobl as t i c anemi a by i nhi bi t i ng enzymes of prot oporphyri n s ynt hes i s ; however, many cas es are i di opat hi c.
o
o
c. Diagnosis. Thi s anemi a may be rel at i vel y s evere i n pat i ent s ol der t han 60 years and i s charact eri zed by hemogl obi n l evel s of 8–10 g/dL. Coult er count er reveal s normocyt i c or even macrocyt i c cel l s , but exami nat i on of t he bl ood s mear s hows a di morphi c popul at i on wi t h s ome very s mal l cel l s . The bone marrow reveal s eryt hroi d hyperpl as i a, and i ron s t ai ni ng demons t rat es t he ri nged s i derobl as t s . Iron s t udi es s how el evat ed ferri t i n l evel s and hi gh s erum i ron l evel s wi t h hi gh t rans ferri n s at urat i on. Oft en, s ome normal or s l i ght l y macrocyt i c cel l s are s een i nt ermi ngl i ng wi t h t he hypochromi c, mi crocyt i c cel l s , and a bl ood s mear reveal i ng s uch a condi t i on i s di agnos t i c of t hi s anemi a.
o
o
d. T herapy
(1) If a drug s uch as i s oni azi d, chl orampheni col , or al cohol i s i nvol ved, t he anemi a and s i derobl as t i c changes regres s wi t h di s cont i nuat i on of t he agent .
Pa g e 4 2 2
ABC Ambe r CHM Conve rte r Tria l ve rsion, http://w w w .proce sste x t.com/a bcchm.html
(2) Al l pat i ent s s houl d be gi ven a t ri al of pyridoxine i n hi gh dos es ; however, i n al l but t he heredi t ary cas es , t hi s us ual l y fai l s . Oft en, t hes e pat i ent s are t rans fus i on dependent . Acut e l eukemi a devel ops i n a port i on (10%) of pat i ent s wi t h acqui red i di opat hi c di s eas e. In t hes e pat i ent s , t he s i derobl as t i c anemi a i s a prel eukemi c s yndrome and i s cl as s i fi ed as a myel odys pl as t i c s yndrome.
(3) Effi cacy us i ng exogenous erythropoietin t o t reat t hi s condi t i on i s approxi mat el y 20%, and t hi s t herapy s houl d be t ri ed i n t rans fus i on-dependent pat i ent s .
4. T halassemias are genet i c di s orders charact eri zed by di mi ni s hed s ynt hes i s of one of t he gl obi n chai ns . Thes e di s eas es are due t o abnormal i t i es i n t he genes t hat are res pons i bl e for s ynt hes i s of t he gl obi n port i on of t he hemogl obi n mol ecul e. Thal as s emi as are named accordi ng t o t he defi ci ent chai n. o
o
a. T ypes
(1) α-T halassemias. Thes e di s orders are charact eri zed by defi ci ent α-chai n s ynt hes i s , us ual l y due t o del et i on of one or more of t he α-gl obi n genes from t he genome. There are four s uch P.95
Pa g e 4 2 3
ABC Ambe r CHM Conve rte r Tria l ve rsion, http://w w w .proce sste x t.com/a bcchm.html
genes and, t hus , four α-t hal as s emi as ; t hes e range from a mi l d, s ubcl i ni cal , as ympt omat i c anemi a t o a s evere anemi a t hat i s fat al i n ut ero. The α-t hal as s emi as are mos t preval ent i n As i an popul at i ons .
(2) β-T halassemias. Thes e di s orders are due t o t he abs ence or mal funct i on of t he β-gl obi n gene. In t he l at t er cas e, RNA i s pres ent but i n reduced amount s or i n defect i ve forms . The t wo β-gl obi n genes i n t he genome res ul t i n t wo di fferent forms of β-t hal as s emi a. β-T halassemia major, or Cooley' s anemia, i s a s evere di s eas e t hat appears i n chi l dhood. Pat i ent s wi t h t hi s di s eas e are t rans fus i on dependent . Pat i ent s wi t h β-thalassemia minor, a mi l d anemi a, are not t rans fus i on dependent and can l i ve ful l , normal l i ves . The β-t hal as s emi as are mos t preval ent i n i ndi vi dual s of Medi t erranean des cent , part i cul arl y t hos e from Greece and It al y.
o
o
b. Diagnosis. The t hal as s emi as s houl d be s us pect ed i n anemi c pat i ent s who reveal marked abnormal i t i es on bl ood s mear. Such abnormal i t i es i ncl ude mi crocyt os i s , hypochromi a, and poi ki l ocyt os i s (i .e., t he pres ence of bi zarrel y s haped RBCs ).
(1) α-T halassemia i s mos t di ffi cul t t o di agnos e when i t exi s t s i n t he carri er s t at e. Pat i ent s wi t h t he carri er form have a mi l d
Pa g e 4 2 4
ABC Ambe r CHM Conve rte r Tria l ve rsion, http://w w w .proce sste x t.com/a bcchm.html
mi crocyt i c, hypochromi c anemi a. There i s no exces s of t he non–β-hemogl obi ns becaus e al l chai ns have α component s . Therefore, s ophi s t i cat ed s t udi es are requi red for defi ni t i ve di agnos i s . In neonat es , however, t he di agnos i s can be made from cord bl ood t es t s t hat s how an i ncreas e i n Bart' s hemoglobin. W i t h t he advent of genet i c mappi ng t echnol ogy, t he pres ence or abs ence of α-gl obi n genes can be as cert ai ned t o make t he di agnos i s defi ni t i vel y.
(2) β-T halassemia res embl es i ron defi ci ency anemi a except t hat i ron i s pres ent i n t he marrow. The di agnos i s may be confi rmed i n s everal ways .
(a) Meas urement of mi nor hemogl obi n chai ns A 2 and F reveal s el evat i ons as t he eryt hron at t empt s t o compens at e for t he di mi ni s hed s ynt hes i s of β chai ns by maki ng exces s i ve γ and δ chai ns i ns t ead.
(b) Genet i c anal ys i s of hemogl obi n mRNA or genet i c mappi ng of t he gl obi n genes i s now avai l abl e.
o
o
c. T herapy i ncl udes chroni c red cel l t rans fus i ons and fol i c aci d s uppl ement at i on. The regul ar us e of i ron chel at i on t herapy has s l owed t he devel opment of i ron overl oad i n t hes e pat i ent s . In s el ect ed pat i ent s , bone marrow t rans pl ant at i on (BMT) has
Pa g e 4 2 5
ABC Ambe r CHM Conve rte r Tria l ve rsion, http://w w w .proce sste x t.com/a bcchm.html
been curat i ve, and t hi s form of t herapy l i kel y wi l l be us ed more frequent l y i n t he fut ure.
B. Macrocytic anemias Macrocytic anemias are charact eri zed by RBCs t hat exceed 100 µm. Three major mechani s ms are l i nked t o t he devel opment of macrocyt i c anemi a.
1. Accelerated erythropoiesis. Ret i cul ocyt es and young eryt hrocyt es are l arger t han normal ; t herefore, i ndi vi dual s wi t h l arge numbers of ret i cul ocyt es have l arge numbers of ci rcul at i ng cel l s of great s i ze. A ret i cul ocyt e count confi rms t he di agnos i s .
2. Increased membrane surface area. Pat i ent s wi t h exces s i ve pl as ma l i pi ds abs orb t hes e l i pi ds ont o RBC s urfaces , whi ch creat es an enl arged membrane s urface area and a macrocyt os i s i n exces s of 100 µm. Thi s condi t i on i s mos t common i n pat i ent s wi t h l i ver di s eas e and can be di agnos ed by a bl ood s mear t hat reveal s t he charact eri s t i c target cell of l i ver di s eas e (i .e., a round macrocyt e wi t h a redundant membrane). Li ver di s eas e caus es t hi s by di mi ni s hed hepat i c s ynt hes i s of l eci t hi n–chol es t erol acet yl t rans feras e (LCAT). Di mi ni s hed s ynt hes i s of LCAT res ul t s i n exces s pl as ma-free chol es t erol , whi ch i s abs orbed ont o RBC membranes .
3. Defective DNA synthesis i s t he mai n charact eri s t i c of t he cl as s i c megaloblastic anemias. In t hes e condi t i ons , eryt hroi d precurs ors cannot produce nucl ei c aci d, and s o nucl ear mat urat i on i s arres t ed. Cyt opl as mi c mat urat i on proceeds , however, res ul t i ng i n abnormal l y l arge cel l s . Thes e cel l s are l arger t han t hos e s een wi t h accel erat ed
Pa g e 4 2 6
ABC Ambe r CHM Conve rte r Tria l ve rsion, http://w w w .proce sste x t.com/a bcchm.html
eryt hropoi es i s , and t hey have an i ncreas ed membrane 3
s urface area. An MCV t hat exceeds 115 mm i s not uncommon. o
o
a. Etiology. Megal obl as t i c anemi as us ual l y are caus ed by a defi ci ency of ei t her vi t ami n B 1 2 or fol i c aci d.
(1) B 1 2 deficiency may res ul t from fi s h t apeworm i nfes t at i on, s t ri ct veget ari an (vegan) di et s , or i nt es t i nal bl i nd l oops wi t h bact eri al overgrowt h. The mos t common caus e, however, i s a l ack of t he i nt ri ns i c fact or neces s ary for vi t ami n B 1 2 abs orpt i on i nt o t he t ermi nal i l eum. Two forms of at rophi c gas t ri t i s have been des cri bed. Type A, perni ci ous anemi a, i s a t rue P.96
aut oi mmune gas t ri t i s i nvol vi ng t he fundus and body -
of t he s t omach and has t he gas t ri c pari et al cel l H /K +
-ATPas e as t he mol ecul ar t arget of t he aut oi mmune
proces s . Type B i s nonaut oi mmune, i nvol ves t he ent i re s t omach, and i s as s oci at ed wi t h Hel i c obac t er pyl ori i nfect i on.
(2) Folic acid deficiency i s caus ed by di et ary defi ci ency due t o i nadequat e i nt ake, i nadequat e abs orpt i on, or bot h. Thi s condi t i on i s mos t commonl y encount ered i n al cohol i cs .
Pa g e 4 2 7
ABC Ambe r CHM Conve rte r Tria l ve rsion, http://w w w .proce sste x t.com/a bcchm.html
(3) Drug-induced disorders of DNA synthesis. Cert ai n drugs us ed t o t reat cancer (e.g., met hot rexat e), bact eri al i nfect i ons (e.g., t ri met hopri m), and paras i t i c i nfect i ons (e.g., pyri met hami ne), as wel l as phenyt oi n, i nt erfere wi t h fol i c aci d met abol i s m and caus e megal obl as t i c anemi a and bone marrow changes . Thes e di agnos es can be made eas i l y from pat i ent hi s t ory.
o
b. Clinical manifestations. Pat i ent s wi t h megal obl as t i c anemi a have varyi ng degrees of anemi a as s oci at ed wi t h l arge RBCs . Becaus e nucl ei c aci d met abol i s m i s neces s ary for al l cel l ul ar el ement s i n bone marrow, whi t e bl ood cel l s (W BCs ) and pl at el et s are di mi ni s hed. The i neffect i ve eryt hropoi es i s and i nt ramedul l ary hemol ys i s as s oci at ed wi t h t hi s di s order oft en res ul t i n s erum lactate dehydrogenase (LDH) l evel s t hat exceed 500 uni t s /dL. Bot h RBCs and W BCs i n bone marrow reveal t he cl as s i c megal obl as t i c s i gn of i mmat ure, open nucl ei i n as s oci at i on wi t h mat ure cyt opl as mi c component s . Bl ood s mear s hows charact eri s t i c oval macrocyt es and hypers egment ed pol ymorphonucl ear l eukocyt es (PMNLs ) (
Onl i ne Fi gure 3-3).
o
ONLINE FIGURE 3-3 Peri pheral bl ood s mear from a pat i ent wi t h B 1 2 defi ci ency. Not e t he hypers egment ed neut rophi l s and macro-oval ocyt es (bl ack arrows ). (From Tkachuk DC, Hi rs chmann JV. Wi nt robe's At l as of Cl i ni c al
Pa g e 4 2 8
ABC Ambe r CHM Conve rte r Tria l ve rsion, http://w w w .proce sste x t.com/a bcchm.html
Hemat ol ogy. Bal t i more: Li ppi ncot t W i l l i ams & W i l ki ns , 2007:1–82. ) o
o
c. Differential diagnosis
(1) Serum l evel s of vi t ami ns as wel l as RBC fol i c aci d l evel s s houl d be meas ured t o det ermi ne whet her t he defi ci ency i s i n fol i c aci d or vi t ami n B 1 2 .
(2) Vi t ami n B 1 2 has a neurol ogi c funct i on; t herefore, when a macrocyt i c anemi a i s as s oci at ed wi t h neurol ogi c s ympt oms , part i cul arl y pos t eri or col umn s i gns and s ympt oms , vi t ami n B 1 2 defi ci ency s houl d be s us pect ed. The hal l marks of perni ci ous anemi a are macrocyt i c anemi a, neurol ogi c s ympt oms and s i gns , and atrophic glossitis.
(3) The Schilling test for t he pres ence of i nt ri ns i c fact or and i nt es t i nal funct i on can be performed t o di fferent i at e t he caus e of vi t ami n B 1 2 defi ci ency. However, cl i ni ci ans have become l es s ent hus i as t i c about t hi s t es t becaus e of i t s compl exi t y and occas i onal i naccuracy. Mos t pract i t i oners prefer t o us e more refi ned bi ochemi cal t es t i ng of t he puri ne s ynt het i c pat hways . Current l y, meas urement of met hyl mal oni c aci d l evel s (el evat ed) has repl aced t he Schi l l i ng t es t as t he di agnos t i c procedure of choi ce. However, t he Schi l l i ng t es t remai ns us eful for s t udy of t he
Pa g e 4 2 9
ABC Ambe r CHM Conve rte r Tria l ve rsion, http://w w w .proce sste x t.com/a bcchm.html
phys i ol ogy of vi t ami n B 1 2 . Serum ant i pari et al cel l ant i bodi es are a rel at i vel y s peci fi c t es t fi ndi ng for t he aut oi mmune t ype A vari ant (>90%). o
o
d. T herapy. Speci fi c t herapy i s det ermi ned by t he vi t ami n t hat i s mi s s i ng. Fol i c aci d al one s houl d never be gi ven i n an undi agnos ed cas e of macrocyt i c anemi a; fol i c aci d revers es hemat ol ogi c s i gns , but neurol ogi c degenerat i on cont i nues unabat ed.
(1) Pat i ent s wi t h perni ci ous anemi a due t o vi t ami n B 1 2 defi ci ency requi re l i fel ong t reat ment wi t h parent eral vi t ami n B 1 2 . If revers i bl e caus es are found (e.g., i nt es t i nal bact eri al overgrowt h), appropri at e meas ures may revers e t he defi ci ency and obvi at e t he need for permanent vi t ami n B 1 2 t herapy.
(2) Fol i c aci d defi ci ency i s t reat ed wi t h oral preparat i ons of fol i c aci d.
C. Normochromic, normocytic anemias Normochromic, normocytic anemias repres ent a vas t array of condi t i ons charact eri zed by normal cel l s i ze and hemogl obi n concent rat i on. Thes e anemi as are not rel at ed by common pat hogeni c mechani s ms ; classification is by the degree of marrow response to the anemia.
1. Anemia associated with impaired marrow response. The fol l owi ng anemi as are charact eri zed by normal or l ow ret i cul ocyt e count s .
Pa g e 4 3 0
ABC Ambe r CHM Conve rte r Tria l ve rsion, http://w w w .proce sste x t.com/a bcchm.html o
o
a. Hypoplastic, or aplastic anemia, i s an i nt ri ns i c marrow di s eas e charact eri zed by an abs ence of s t em cel l s . Al l myel oi d (deri ved from t he bone marrow) cel l l i nes are i nvol ved, wi t h a res ul t ant pancytopenia. Severe cas es of t hi s s eri ous di s eas e are as s oci at ed wi t h a hi gh mort al i t y rat e. Level s of s erum eryt hropoi et i n are us ual l y el evat ed proport i onat el y t o t he degree of anemi a. There i s l i t t l e effect i ve medi cal t reat ment . In young pat i ent s , BMT t echni ques are curat i ve i n cas es i n whi ch an appropri at e marrow donor i s avai l abl e.
o
o
b. Disorders characterized by infiltration of bone marrow (myel opht hi s i c anemi as ) i ncl ude myel oma, carci noma, and l eukoeryt hrobl as t os i s . Di s rupt i on of bone marrow archi t ect ure i s P.97
common; a bl ood s mear t hat reveal s i mmat ure W BCs and nucl eat ed RBCs i s a cl ue t o t he pres ence of t hes e condi t i ons . A bone marrow as pi rat e and bi ops y confi rm t he di agnos i s i n s uch cas es . o
o
c. Anemia due to diminished erythropoietin secretion i s t he anemi a of chronic renal failure. Eryt hropoi et i n i s a prot ei n–l i pi d mol ecul e requi red by t he marrow for adequat e RBC format i on. W i t h s evere ki dney di s eas e, t he eryt hropoi et i n s ecret ed by t he ki dneys i s l os t , and anemi a ens ues . The degree of anemi a roughl y correl at es wi t h t he degree of renal fai l ure.
Pa g e 4 3 1
ABC Ambe r CHM Conve rte r Tria l ve rsion, http://w w w .proce sste x t.com/a bcchm.html
(1) Proper at t ent i on t o i ron and fol i c aci d s t ores i s i mport ant i n t hi s group of pat i ent s , becaus e defi ci enci es i n t hes e nut ri ent s s econdari l y compl i cat e t he anemi a of renal fai l ure.
(2) Eryt hropoi et i n has been uni forml y effect i ve i n rai s i ng t he hemogl obi n t o normal or near normal l evel s i n t hes e pat i ent s , and t hi s i ncreas e i n hemogl obi n has t rans l at ed i nt o an enhanced qual i t y of l i fe.
o
o
d. Other anemias associated with hypoproliferation of bone marrow i ncl ude t hos e as s oci at ed wi t h hypot hyroi di s m, hypopi t ui t ari s m, and l i ver di s eas e.
2. Anemia associated with appropriately increased RBC production. The fol l owi ng anemi as are charact eri zed by an i ncreas ed number of ret i cul ocyt es . o
o
a. Anemia following hemorrhage. An i ncreas ed ret i cul ocyt e count i s t he normal marrow res pons e i n pat i ent s who bl eed ei t her overt l y (e.g., wi t h s urgery) or covert l y (e.g., i nt o t he gas t roi nt es t i nal t ract ) and have adequat e i ron s t ores . Thi s condi t i on can be confus ed wi t h hemol ys i s ; however, t he cl i ni cal s et t i ng (e.g., a pos t operat i ve pat i ent wi t h a l arge res ol vi ng hemat oma) oft en l eads t o confi rmat i on of t he di agnos i s .
o
Pa g e 4 3 2
ABC Ambe r CHM Conve rte r Tria l ve rsion, http://w w w .proce sste x t.com/a bcchm.html
o
b. Hemolytic anemias repres ent condi t i ons i n whi ch RBC s urvi val i s s hort ened. In mos t cas es , t he marrow i s i nt ri ns i cal l y normal ; t herefore, adequat e new RBCs can be made, and t he pat i ent s have el evat ed ret i cul ocyt e count s . Di agnos t i c of t hes e anemi as are s i gns of i ncreas ed RBC des t ruct i on (e.g., s hort ened RBC hal f-l i fe, el evat ed s erum LDH, reduced hapt ogl obi n) combi ned wi t h s i gns of accel erat ed marrow act i vi t y (e.g., el evat ed ret i cul ocyt e count s and eryt hroi d hyperpl as i a i n t he marrow). The di agnos i s of hemol ys i s s houl d be made fi rs t , and t he s peci fi c caus e of t he hemol ys i s s ought l at er. Hemol yt i c anemi a exi s t s i n hundreds of forms , whi ch are grouped as fol l ows :
(1) Hemolytic anemia due to factors extrinsic to the RBC
(a) Autoantibodies can at t ach t o t he RBC and caus e i t s des t ruct i on by t he ret i cul oendot hel i al s ys t em. A cl as s i c exampl e i s Coombs-positive hemolytic anemia due t o ei t her warm [i mmunogl obul i n G (IgG)] or col d (IgM) ant i bodi es . Thi s anemi a may be i di opat hi c or may ari s e as a compl i cat i on of col l agen di s eas e or l ymphoma. In s evere cas es , s t eroi ds and s pl enect omy may be requi red t o cont rol t he anemi a.
(b) Exogenous agents s uch as mal ari al organi s ms can render t he RBC vul nerabl e t o hemol ys i s .
Pa g e 4 3 3
ABC Ambe r CHM Conve rte r Tria l ve rsion, http://w w w .proce sste x t.com/a bcchm.html
(c) Abnormalities in the circulation can caus e premat ure des t ruct i on of RBCs . The fol l owi ng are exampl es of s uch abnormal i t i es , and di s orders as s oci at ed wi t h each condi t i on are al s o ci t ed.
(i) Li pi d abnormal i t i es (s pur-cel l anemi a i n advanced l i ver di s eas e)
(ii) Fi bri n depos i t i on i n t he mi crovas cul at ure wi t h s heari ng of RBCs [di s s emi nat ed i nt ravas cul ar coagul at i on (DIC) s yndrome]
(iii) RBC damage due t o t rauma from pros t het i c heart val ves
(2) Hemolytic anemia due to factors intrinsic to the RBC. Thes e di s orders i nvol ve congeni t al abnormal i t i es t hat render t he RBC vul nerabl e t o hemol ys i s .
(a) Membrane disorders i ncl ude hereditary spherocytosis, where a defect i n t he membrane s odi um–pot as s i um–ATPas e pump caus es RBC s wel l i ng. Thi s res ul t s i n t he charact eri s t i c fi ndi ng on bl ood s mear of s mal l , round, hyperchromi c RBCs wi t hout t he us ual cent ral pal l or (i .e.,
Pa g e 4 3 4
ABC Ambe r CHM Conve rte r Tria l ve rsion, http://w w w .proce sste x t.com/a bcchm.html
s pherocyt es ). Thes e cel l s are os mot i cal l y fragi l e and are des t royed i n t he s pl een. Spl enect omy us ual l y cont rol s t he anemi a, al t hough t he RBC defect remai ns .
(b) Hemoglobin disorders—hemoglobinopathies. Thes e di s eas es , of whi ch more t han 250 are known, are caus ed by
poi nt
mut at i ons i n t he DNA code rel at ed t o vari at i on i n a s i ngl e ami no aci d i n t he gl obi n chai ns . Such ami no aci d changes caus e a vari et y of s t ruct ural and funct i onal changes i n t he RBC. W hen ami no aci d changes occur i n t he i nner hydrophobi c s t ruct ural port i ons of t he hemogl obi n mol ecul e, uns t abl e hemogl obi n di s eas e wi t h l ow-grade hemol ys i s can res ul t (e.g., Hb Zuri ch). Ami no aci d changes i n t he heme–oxygen bi ndi ng areas of t he mol ecul e caus e changes i n oxygen affi ni t y and gi ve ri s e t o t he s o-cal l ed pol ycyt hemi c hemogl obi nopat hi es (e.g., Hb Pot omac).
The mos t commonl y
encount ered hemogl obi nopat hi es i nvol ve ami no aci d changes near t he s urface of t he gl obul ar hemogl obi n mol ecul e, whi ch predi s pos e t he hemogl obi n t o pol ymeri zat i on. Such pol ymeri zat i on res ul t s i n t he hemogl obi n becomi ng ri gi d, wi t h s ubs equent membrane and cel l s hape changes . Thes e affect ed RBCs become l i abl e t o hemol ys i s .
P.98
TABLE 3-1 Clinical Manifestations of Sickle Cell Anemia
Pa g e 4 3 5
ABC Ambe r CHM Conve rte r Tria l ve rsion, http://w w w .proce sste x t.com/a bcchm.html
R R R el el el at at at e e e d d d to to to C A In hr b cr o n e ni or a c m s H al e e A d m d S ol h u yt e sc ic si e A o pt n n ib e s, ili m Si ty ia ck to li In n fe g, ct a io n n d V a s oo cc lu
Pa g e 4 3 6
ABC Ambe r CHM Conve rte r Tria l ve rsion, http://w w w .proce sste x t.com/a bcchm.html
si o n N P P or ai n m nf e oc ul u yt cr m i c i s oc a es oc n
ca
e
l
m
se
ia
ps
is El C S ev er al at e m e br o d ov n bi as el l i r cu l a u la se bi r ps n ac i s a ci n d d e L nt D H G Ac O al ut s t ls e e to a o n n m
Pa g e 4 3 7
ABC Ambe r CHM Conve rte r Tria l ve rsion, http://w w w .proce sste x t.com/a bcchm.html
e d ye di ch l i t s e ro i s as ni e c ca rd io p ul m o n ar y di se as e (e .g ., ac ut e ch es t sy n dr o m es ,
Pa g e 4 3 8
ABC Ambe r CHM Conve rte r Tria l ve rsion, http://w w w .proce sste x t.com/a bcchm.html
co r p ul m o n al e) Pr ia pi s m S pl e ni c a ut oi nf ar ct io n Sk el et al ch a n g
Pa g e 4 3 9
ABC Ambe r CHM Conve rte r Tria l ve rsion, http://w w w .proce sste x t.com/a bcchm.html
es (e .g ., as e pt ic n ec ro si s of th e hi p) LDH, l act at e dehydr ogenas e.
(i) Hemoglobin S (Hb S). The mos t common hemogl obi nopat hy i s Hb S, whi ch caus es sickle cell anemia (
Onl i ne Fi gure 3-4). Thi s
di s order occurs i n 1% of Afri can Ameri cans . The di s eas e mani fes t at i ons res ul t from t hree major pat hophys i ol ogi c proces s es (Tabl e 3-1). The pol ymeri zed hemogl obi n s everel y deforms RBCs and res ul t s i n marked, chronic hemolysis. Thi s mani fes t s as s evere, chroni c anemi a (wi t h hemogl obi n of 5–10 g/dL); predi s pos i t i on t o apl as t i c cri s es as s oci at ed wi t h parvovi rus and ot her i nfect i ons ; el evat ed bi l i rubi n l evel s wi t h al mos t uni vers al gal l s t one di s eas e and frequent , chroni c ul cerat i on of t he l egs i n t he ankl e area. Spl eni c aut oi nfarct i on and di mi ni s hed s ynt hes i s of
Pa g e 4 4 0
ABC Ambe r CHM Conve rte r Tria l ve rsion, http://w w w .proce sste x t.com/a bcchm.html
ops oni z i ng i mmunogl obul i ns res ul t s i n frequent i nfect i ons , es peci al l y wi t h encaps ul at ed mi croorgani s ms (e.g., pneumococci , Haemophi l us , Sal monel l a s peci es ). Acute episodes of pain (“ painful crises―) are t he pri nci pal s ympt om and caus e of morbi di t y i n pat i ent s wi t h s i ckl e cel l di s eas e. Pai nful cri s es are t hought t o be t he res ul t of mi crovas cul ar occl us i on wi t h i nfarct i on res ul t i ng from l ocal hypervi s cos i t y as s oci at ed wi t h t he ri gi d, deformed and abnormal l y adhes i ve s i ckl e cel l s .
ONLINE FIGURE 3-4 Peri pheral bl ood s mear from a pat i ent wi t h s i ckl e cel l anemi a. Not e t he mul t i pl e s i ckl e and t arget cel l s . (From Tkachuk DC, Hi rs chmann JV. Wi nt robe's At l as of Cl i ni c al Hemat ol ogy. Bal t i more: Li ppi ncot t W i l l i ams & W i l ki ns , 2007:1–72. )
(ii) Ot her common hemogl obi nopat hi es t hat caus e l es s s evere s i ckl e s yndromes , i ncl ude hemogl obi n C (Hb C), hemogl obi n O (Hb O), and mi xt ures s uch as hemogl obi n SC (Hb SC).
Pa g e 4 4 1
ABC Ambe r CHM Conve rte r Tria l ve rsion, http://w w w .proce sste x t.com/a bcchm.html
(iii) Diagnosis i s confi rmed by hemogl obi n el ect rophores i s , whi ch demons t rat es t he charact eri s t i c changes i n mobi l i t y caus ed by s peci fi c ami no aci d changes .
(iv) T herapy, whi ch i s s t i l l not s at i s fact ory, cont i nues t o evol ve. Supportive medical care remai ns t he corners t one of effect i ve t herapy and i ncl udes t rans fus i ons when i ndi cat ed (e.g., duri ng hypopl as t i c cri s es ), proper fl ui d management and at t ent i on t o hydrat i on, and anal ges i cs for t he pai n of mi crovas cul ar occl us i on.
(v) Newer di rect i ons i n t herapy have moved t oward mani pul at i on of t he hemogl obi n mi x wi t hi n t he cel l t o di mi ni s h t he propens i t y t o pol ymeri z e. Hb F has been found t o have an i nhi bi t ory effect on s i ckl i ng. The cyt ot oxi c agent hydroxyurea has been t he mos t ext ens i vel y s t udi ed, and recent dat a s ugges t t hat enhanci ng Hb F l evel s us i ng hydroxyurea res ul t s i n l es s pai nful cri s es i n s el ect ed cas es .
See more i nformat i on onl i ne. Fi nal l y, BMT, whi ch res ul t s i n
phenot ypi c cure, has been report ed i n pat i ent s wi t h coexi s t i ng s i ckl e cel l anemi a and acut e l eukemi a as wel l as s t and-al one cas es . As t he t echnol ogy of marrow t rans pl ant at i on evol ves and becomes s afer, s uch t herapy may become an opt i on. Pat i ent s wi t h hemogl obi nopat hi es wi t h s i gni fi cant morbi di t y and mort al i t y rat es (e.g., s i ckl e cel l anemi a) are candi dat es for gene transfer therapy , a devel opi ng t echnol ogy.
(vi) Speci fi c complications are res pons i bl e for much of t he
Pa g e 4 4 2
ABC Ambe r CHM Conve rte r Tria l ve rsion, http://w w w .proce sste x t.com/a bcchm.html
mort al i t y i n s i ckl e cel l di s eas e. An i mport ant exampl e i s t he devel opment of acute chest syndrome. In t hi s s i t uat i on, whi ch us ual l y compl i cat es what appears t o be a rout i ne pai nful cri s i s , t he pat i ent devel ops a fever, frequent l y mani fes t s an unus ual l y bri s k l eukocyt os i s , and t hen demons t rat es profound hypoxi a. Soon t hereaft er, a di ffus e ches t radi ograph i nfi l t rat e i s evi dent , and t he cl i ni cal s yndrome evol ves i nt o P.99
an adul t res pi rat ory di s t res s s yndrome (ARDS)–l i ke s i t uat i on. Thi s compl i cat i on carri es a hi gh mort al i t y i f not qui ckl y recogni zed and t reat ed. The s yndrome us ual l y res ponds very wel l t o exchange t rans fus i ons t o l ower Hb S l evel s t o bel ow 50%. Ot her current l y accept ed i ndi cat i ons for RBC exchange t rans fus i on i ncl ude s t roke and cent ral nervous s ys t em (CNS) l es i ons , pri api s m, and s i ckl e cel l hepat i c s eques t rat i on cri s i s . The advent and i ncreas i ng eas e of t hi s t herapy has s ubs t ant i al l y reduced t he mort al i t y res ul t i ng from t hes e condi t i ons .
(c) Disorders of the cytoplasm and enzymes occur as congeni t al hemol yt i c anemi as . An RBC l acks a nucl eus and mi t ochondri a when i t l eaves t he marrow;
Pa g e 4 4 3
ABC Ambe r CHM Conve rte r Tria l ve rsion, http://w w w .proce sste x t.com/a bcchm.html
t herefore, an RBC mus t s urvi ve i t s 120-day l i fe s pan wi t h i t s gi ven compl ement of enzymes . A defi ci ent cel l i s hemol yzed earl i er t han a normal cel l .
(i) Glucose-6-phosphate dehydrogenase (G6PD) deficiency i s an ext remel y common X-l i nked di s order; more t han 150 s ubt ypes affect more t han 100 mi l l i on i ndi vi dual s worl dwi de. Defi ci ent i ndi vi dual s are vul nerabl e t o oxi dant s t res s , whi ch occurs wi t h i nfect i ons and wi t h cert ai n drugs (e.g., s ul fa drugs , qui ni ne). Such oxi dant s t res s res ul t s i n denat ured hemogl obi n or Hei nz bodi es , l eadi ng t o hemol ys i s of affect ed RBCs . A defi ci ency of G6PD al s o des t roys t he reduci ng capaci t y of t he RBCs . It i s bel i eved t hat G6PD prot ect s i ndi vi dual s from fal ci parum mal ari a; t hi s di s eas e i s mos t common i n endemi c areas . Diagnosis i s made by demons t rat i ng Hei nz bodi es duri ng an acut e hemol yt i c anemi a epi s ode or by meas uri ng abnormal l y l ow l evel s of enzyme i n t he s t eady s t at e. Enzyme l evel s s houl d not be meas ured duri ng a hemol yt i c epi s ode, when t hey may t emporari l y become more normal as a res ul t of des t ruct i on of ol d, ext remel y defi ci ent cel l s and t hei r repl acement by rel at i vel y G6PD-ri ch
Pa g e 4 4 4
ABC Ambe r CHM Conve rte r Tria l ve rsion, http://w w w .proce sste x t.com/a bcchm.html
ret i cul ocyt es and neocyt es .
(ii) Pyruvate kinase (PK) deficiency i s an aut os omal reces s i ve exampl e of an enzymopat hy. Thi s defi ci ency i s mos t common i n nort hern European popul at i ons . Pat i ent s may benefi t from s pl enect omy.
II. Disorders Associated with an Elevated Hematocrit Increas es i n hemat ocri t are caus ed by ei t her increased RBC mass or decreased plasma volume. The fol l owi ng di s cus s i on deal s excl us i vel y wi t h di s orders as s oci at ed wi t h abnormal el evat i on of hemat ocri t (i .e., hemat ocri t ≥55%).
A. Terminology The t erm polycythemia oft en i s us ed t o des cri be an i ncreas e i n t he number of RBCs , wi t h no reference t o fl uct uat i ons i n l eukocyt es and pl at el et s . However, t hi s condi t i on i s more accurat el y t ermed erythrocytosis. [There i s a condi t i on cal l ed polycythemia vera i n whi ch l eukocyt es and pl at el et s al s o i ncreas e i n number. See II C 2 b (2) (a) for a di s cus s i on of t hi s di s order.] Increas ed hemat ocri t s occur i n t wo ways .
1. Relative erythrocytosis refers t o an el evat i on of hemat ocri t due t o di mi ni s hed pl as ma vol ume; RBC mas s remai ns normal .
2. Absolute erythrocytosis refers t o an el evat i on of hemat ocri t due t o a t rue i ncreas e i n RBC mas s .
B. Pathophysiology
Pa g e 4 4 5
ABC Ambe r CHM Conve rte r Tria l ve rsion, http://w w w .proce sste x t.com/a bcchm.html
Bl ood vi s cos i t y i s di rect l y proport i onal t o hemat ocri t ; t herefore, an exces s i vel y el evat ed hemat ocri t can di mi ni s h t i s s ue bl ood fl ow, decreas e t i s s ue oxygen del i very, and i ncreas e cardi ac work. In ext reme cas es , t hi s can res ul t i n hypervi s cos i t y s yndromes .
C. Classification
1. Relative erythrocytosis exi s t s i n t wo forms . o
o
a. Stress erythrocytosis, or Gaisböck' s syndrome, occurs predomi nant l y i n mi ddl e-aged men. Thi s di s order us ual l y i s as ympt omat i c, al t hough i t may be as s oci at ed wi t h i ncreas ed cardi ovas cul ar di s eas e. It i s i mport ant t o di fferent i at e pat i ent s wi t h s t res s eryt hrocyt os i s from t hos e wi t h earl y and s ubt l e mani fes t at i ons of t he much more s eri ous condi t i on pol ycyt hemi a vera. Pat i ent s wi t h s t res s eryt hrocyt os i s requi re no t reat ment .
o
o
b. Erythrocytosis occurs secondary to known causes of contracted plasma volume (e.g., exces s i ve di ures i s ; nas ogas t ri c drai nage; s evere gas t roent eri t i s , es peci al l y i n i nfant s ; burns ). P.100
Thes e condi t i ons are apparent cl i ni cal l y; t herapy i ncl udes fl ui d and pl as ma repl acement wi t h t reat ment of t he underl yi ng condi t i on. o
o
2. Absolute erythrocytosis i s cl as s i fi ed accordi ng
Pa g e 4 4 6
ABC Ambe r CHM Conve rte r Tria l ve rsion, http://w w w .proce sste x t.com/a bcchm.html
t o t he mechani s m res pons i bl e for i ncreas ed RBC mas s .
a. Hypoxia
(1) Etiology. Caus es i ncl ude s evere l ung di s eas e, s evere heart fai l ure, cyanot i c heart di s eas e wi t h ri ght -t o-l eft cardi opul monary s hunt s , and abnormal hemogl obi ns wi t h i ncreas ed oxygen affi ni t y.
(2) Pathophysiology. The RBC mas s ri s es s econdary t o t i s s ue hypoxi a, whi ch caus es an i ncreas e i n renal eryt hropoi et i n and s ubs equent hemat ocri t el evat i on.
(3) Diagnosis may be apparent cl i ni cal l y, but bl ood gas anal ys i s s howi ng art eri al oxygen s at urat i on l es s t han 92%, a moderat el y el evat ed s erum eryt hropoi et i n l evel and P 5 0 anal ys i s (i .e., s t udi es of t he oxygen-rel eas i ng charact eri s t i cs of hemogl obi n) may be requi red t o confi rm t he di agnos i s .
(4) T herapy i s s omewhat conject ural , but phl ebot omy i s a favored t reat ment for pat i ent s wi t h hemat ocri t s t hat are pers i s t ent l y great er t han 55%.
Pa g e 4 4 7
ABC Ambe r CHM Conve rte r Tria l ve rsion, http://w w w .proce sste x t.com/a bcchm.html
b. Neoplasia
(1) Neoplastic erythropoietin sources caus e t he RBC mas s t o i ncreas e.
(a) Etiology. Caus es i ncl ude hypernephroma and renal cys t s ; s uch renal pat hol ogy account s for more t han 90% of t hi s t ype of eryt hrocyt os i s . Ot her t umors i ncl ude cerebel l ar hemangi obl as t oma, hepat oma, and ut eri ne fi broi ds .
(b) Diagnosis requi res radi ol ogi c demons t rat i on of t he appropri at e t umor wi t h i nt ravenous pyel ography, comput ed t omography (CT), or ul t ras ound t echni ques . Serum eryt hropoi et i n t es t i ng reveal s ext remel y el evat ed t i t ers from aut onomous eryt hropoi et i n s ecret i on by t hes e t umors .
(c) T herapy. Removal of t he t umor correct s t he hemat ocri t .
(2) Autonomous bone marrow
(a) In t he condi t i on polycythemia vera, t he bone marrow becomes
Pa g e 4 4 8
ABC Ambe r CHM Conve rte r Tria l ve rsion, http://w w w .proce sste x t.com/a bcchm.html
aut onomous and s ynt hes i zes cel l s i ndependent l y of eryt hropoi et i n l evel s . (Theoret i cal l y, eryt hropoi et i n l evel s s houl d be near zero i f meas ured accurat el y.) Pol ycyt hemi a vera repres ent s a t rue neopl as m of t he marrow s t em cel l s . Thi s has been found t o be as s oci at ed wi t h a mut at i on i n t he JAK2 prot ei n i n a majori t y of cas es .
(i) Diagnosis. Accordi ng t o t he Pol ycyt hemi a Vera St udy Group, di agnos i s i s confi rmed by t he pres ence of al l t hree of t he fol l owi ng major criteria or by t he fi rs t t wo major cri t eri a and any t wo of t he fol l owi ng minor criteria. The major criteria are el evat ed RBC mas s , art eri al oxygen s at urat i on exceedi ng 92%, and s pl enomegal y. The minor criteria are l eukocyt os i s , t hrombocyt os i s , el evat ed l eukocyt e al kal i ne phos phat as e (LAP), and el evat ed s erum vi t ami n B 1 2 l evel . JAK2 mut at i on anal ys i s , i f avai l abl e, can confi rm t he di agnos i s .
(ii) T herapy i nvol ves removal of RBCs , s uppres s i on of
Pa g e 4 4 9
ABC Ambe r CHM Conve rte r Tria l ve rsion, http://w w w .proce sste x t.com/a bcchm.html
marrow funct i on, or bot h. Phlebotomy removes RBCs and s houl d be performed t o l ower hemat ocri t t o t he 45% range. If us ed al one, t hi s i s t he s afes t t herapy. Marrow s uppres s i on i s needed when hemat ocri t cont rol requi res frequent phl ebot omy or when ot her cel l l i nes are el evat ed. Int erferon α has been s hown t o be an effect i ve agent t o cont rol myel oprol i ferat i on and s pl enomegal y. Radi oact i ve phos phorus , once commonl y us ed t o modul at e marrow act i vi t y, i s as s oci at ed wi t h an i ncreas ed i nci dence of l eukemi a and nonhemat ol ogi c cancers , and i s now us ed onl y rarel y. Chemot herapy wi t h t he ant i met abol i t e hydroxyurea i s us ed t o cont rol many cas es of pol ycyt hemi a vera. Few i f any l eukemogeni c or s econd mal i gnancy–i nduci ng effect s have been encount ered wi t h t hi s agent .
(iii) Survival i s meas ured i n years and i s 7–10 years i n mos t s t udi es . W i t h unt reat ed pol ycyt hemi a vera, however, s urvi val i s onl y 2–3 years . The major caus es of morbi di t y
Pa g e 4 5 0
ABC Ambe r CHM Conve rte r Tria l ve rsion, http://w w w .proce sste x t.com/a bcchm.html
and mort al i t y are t hromboembol i c and cardi ovas cul ar. Leukemi c t rans format i on occurs i n 5%–10% of pat i ent s .
(b) Myelofibrosis, a chroni c myel oprol i ferat i ve di s order, i s pat hophys i ol ogi cal l y rel at ed. It i s charact eri zed by s pl enomegal y, i mmat ure granul ocyt es and eryt hrocyt es i n t he P.101
bl ood, di s t ort ed t ear-drop–s haped RBC forms , and marrow fi bros i s . The di s eas e i s a monoclonal stem cell disease of pri mi t i ve hemat opoi et i c s t em cel l s . The fibrosis i s a s econdary event .
(i) Anemia and s i gns and s ympt oms of massive splenomegaly are t he hal l marks of t he di s eas e.
(ii) Mos t therapy is supportive. In s el ect ed cas es of t rue hypers pl eni s m and s ympt oms from mas s i ve s pl enomegal y, s pl enect omy i s benefi ci al .
III. White Blood Cell (WBC) Disorders
Pa g e 4 5 1
ABC Ambe r CHM Conve rte r Tria l ve rsion, http://w w w .proce sste x t.com/a bcchm.html
The W BCs i ncl ude l ymphocyt es , monocyt es , eos i nophi l s , bas ophi l s , and neut rophi l s (PMNLs ). Di s orders of W BCs can be cons i dered i n t erms of exces s i ve or reduced numbers of cel l s and i n t erms of funct i onal abnormal i t y.
A. Lymphocytes Lymphocytes exi s t i n marrow as wel l as i n t he l ymphoi d t i s s ue of t he body. Lymphocyt e funct i ons i ncl ude del ayed hypers ens i t i vi t y, whi ch i s performed by T l ymphocyt es (T cells), and ant i body product i on, whi ch i s performed by B l ymphocyt es (B cells) and pl as ma cel l s .
1. Lymphopenia refers t o a di mi ni s hed number of l ymphocyt es . o
o
a. Lymphopenia without significant immune deficiency i s s een i n many i l l nes s es t hat caus e el evat ed s erum cort i s ol l evel s s uch as acut e i nfect i ons and i nfl ammat ory s t at es . Chemot herapy, radi ot herapy, and Hodgki n's di s eas e al s o are as s oci at ed wi t h l ymphopeni a. In none of t hes e condi t i ons i s ant i body product i on s everel y affect ed.
o
o
b. Congenital lymphopenia with immune deficiency i s as s oci at ed wi t h s peci fi c i mmune defi ci ency s yndromes .
(1) Varieties
(a) B-cell deficiency
(i) Bruton' s agammaglobulinemia i s an X-l i nked di s eas e
Pa g e 4 5 2
ABC Ambe r CHM Conve rte r Tria l ve rsion, http://w w w .proce sste x t.com/a bcchm.html
charact eri zed by recurrent i nfect i ons wi t h encaps ul at ed organi s ms . It i s caus ed by defi ci ent quant i t i es of ops oni zi ng ant i body. Al t hough peri pheral l ymphocyt e count s may be normal , s peci fi c count i ng for B cel l s s hows t hei r abs ence. In addi t i on, l ymphoi d fol l i cl es reveal no germi nal cent ers , whi ch are t he B-cel l areas . Therapy cons i s t s of exogenous gamma gl obul i n and pl as ma vi a t rans fus i on.
(ii) Other B-cell deficiency states i ncl ude common vari abl e hypogammagl obul i nemi a and IgA defi ci ency.
(b) T -cell deficiency. T hymic hypoplasia (Di George s yndrome) i s t he prot ot ypi cal T-cel l defi ci ency s yndrome. Pat i ent s wi t h t hi s condi t i on have vari abl e t ot al l ymphocyt e count s but l ow numbers of T cel l s wi t h abs ent T-cel l funct i on. Recurrent fungal i nfect i ons are s een i n t hes e pat i ent s .
(c) Deficiency of both B and T cells. Di s orders charact eri zed by di mi ni s hed numbers of bot h B and T cel l s i ncl ude at axi a–t el angi ect as i a s yndrome, W i s kot t -Al dri ch s yndrome of i mmunodefi ci ency and t hrombocyt openi a, and s evere combi ned i mmunodefi ci ency di s eas e (SCID).
(2) Diagnosis. For al l of t he condi t i ons di s cus s ed, di agnos i s requi res t he cl i ni cal s et t i ng of repeat ed i nfect i ons combi ned wi t h t he fol l owi ng fi ndi ngs :
(a) Lymphocyt e count s of B and T cel l s , i ncl udi ng s urface s ubs et markers
Pa g e 4 5 3
ABC Ambe r CHM Conve rte r Tria l ve rsion, http://w w w .proce sste x t.com/a bcchm.html
(b) Meas urement of s peci fi c i mmunogl obul i n l evel s
(c) Demons t rat i on of t he abs ence of s peci fi c B-cel l areas (i .e., germi nal cent ers and pl as ma cel l s ) or T-cel l areas (i .e., t hymus and l ymph node medul l ary cords )
(3) T herapy. BMT has been curat i ve i n many of t hes e condi t i ons .
o
o
c. Acquired immunodeficiency syndrome (AIDS). For a di s cus s i on of AIDS, s ee Chapt er 8 VIII C 3.
2. Lymphocytosis i s defi ned as an exces s i ve number of 3
l ymphocyt es (i .e., >5000/mm ). The di fferent i al di agnos i s of abs ol ut e l ymphocyt os i s i s l i mi t ed. o
o
a. Infection. Cert ai n i nfect i ons caus e l ymphocyt os i s . In chi l dren, bot h pert us s i s and acut e i nfect i ous l ymphocyt os i s may caus e count s 3
t hat exceed 50,000/mm . In adul t s , l es s er el evat i ons are s een wi t h hepat i t i s and i nfect i ous mononucl eos i s . o
o
b. Hematopoietic disorders as s oci at ed wi t h l ymphocyt os i s i ncl ude acut e l ymphocyt i c l eukemi a (ALL), chroni c l ymphocyt i c l eukemi a (CLL), and
Pa g e 4 5 4
ABC Ambe r CHM Conve rte r Tria l ve rsion, http://w w w .proce sste x t.com/a bcchm.html
cert ai n l ymphomas .
(1) CLL. An adul t who i s ol der t han 50 years and who mani fes t s a mat ure l ymphocyt os i s mos t l i kel y has CLL. The cel l s as s oci at ed wi t h CLL are mat ure l ymphocyt es t hat accumul at e i n t he body.
(a) Diagnosis. A peri pheral bl ood s mear s howi ng a mat ure l ymphocyt os i s i s hi ghl y s ugges t i ve. Corroborat i ve fi ndi ngs i ncl ude marrow i nfi l t rat i on by mat ure l ymphocyt es , an enl arged s pl een, and l ymphadenopat hy. Lymphocyt e markers can al s o be det ermi ned. The t echni que of fl ow cyt omet ry i s di agnos t i c (
Onl i ne Fi gure 3-5).
ONLINE FIGURE 3-5 CLL. Not e t he mul t i pl e l ymphocyt es wi t h s pars e cyt opl as m and round t o s l i ght l y oval nucl ei . Damaged l ymphocyt es (“s mudge cel l s ―) are al s o pres ent . (From Armi t age JO, ed. At l as of Cl i ni c al Hemat ol ogy. Phi l adel phi a: Li ppi ncot t W i l l i ams & W i l ki ns /Current Medi ci ne, Inc. 2004:2–21. )
P.102
Pa g e 4 5 5
ABC Ambe r CHM Conve rte r Tria l ve rsion, http://w w w .proce sste x t.com/a bcchm.html
(b) Staging. The CLL t umor burden i s rel at ed t o cert ai n cl i ni cal fi ndi ngs , whi ch al s o have prognos t i c s i gni fi cance.
(i) Stage 0 i s charact eri zed by peri pheral l ymphocyt os i s onl y. The prognos i s for pat i ent s wi t h s t age 0 CLL i s excel l ent ; medi an s urvi val exceeds 10 years .
(ii) Stages 1 and 2. St age 1 i s charact eri zed by peri pheral l ymphocyt os i s and l ymphadenopat hy, and s t age 2 by t he pres ence of s pl enomegal y. Bot h s t ages 1 and 2 have i nt ermedi at e prognos es , wi t h a medi an s urvi val of 60 mont hs .
(iii) Stages 3 and 4. St age 3 i s charact eri zed by t he pres ence of anemi a, and s t age 4 by t he pres ence of t hrombocyt openi a. Bot h s t ages 3 and 4 s i gni fy marrow fai l ure and have poor prognos es , wi t h a medi an s urvi val <24 mont hs .
(iv) Ot her prognos t i c fact ors i ncl ude l ymphocyt e doubl i ng t i me, chromos omal and V i mmunogl obul i n gene s t at us , and CD38 expres s i on.
Pa g e 4 5 6
ABC Ambe r CHM Conve rte r Tria l ve rsion, http://w w w .proce sste x t.com/a bcchm.html
(c) T herapy for CLL.
(i) Early-stage CLL. As a rul e, s t ages 0, 1, and 2 CLL s houl d not be t reat ed.
(ii) Late-stage CLL i s t reat ed wi t h an al kyl at i ng agent (e.g., chl orambuci l ), or puri ne anal og (e.g., fl udarabi ne) al one or i n combi nat i on wi t h s t eroi ds .
(2) Cert ai n wel l -di fferent i at ed lymphomas al s o are charact eri zed by an exces s i ve number of l ymphocyt es i n t he bl ood. Thes e di s orders are s i mi l ar t o CLL.
(3) Acute lymphocytic leukemia (ALL) i s charact eri zed by mat urat i on arres t i n t he l ymphoi d l i ne wi t h t i s s ue i nfi l t rat i on by l ymphobl as t s . (Thi s di s eas e i s di s cus s ed i n more det ai l i n IV A.)
o
o
3. Monocytosis and monocytopenia. Monocyt es are phagocytic cells t hat are an i mport ant component of t he cel l ul ar i mmune s ys t em and t hat s ecret e many cyt oki nes , i ncl udi ng t umor necros i s fact or (TNF), al l of t he i nt erferons (IFNs ), granul ocyt e–macrophage col ony-s t i mul at i ng fact or (GM-CSF), and granul ocyt e col ony-s t i mul at i ng fact or (G-CSF).
Pa g e 4 5 7
ABC Ambe r CHM Conve rte r Tria l ve rsion, http://w w w .proce sste x t.com/a bcchm.html
a. Isolated monocytopenia does not occur. Monocyt openi a i n combi nat i on wi t h decreas es i n ot her cel l l i nes occurs wi t h apl as t i c anemi a, hai ry cel l l eukemi a, and s t eroi d us e.
b. Benign monocytosis can occur i n many i nfect i ous di s eas es [e.g., t ubercul os i s , s ubacut e bact eri al endocardi t i s (SBE), cyt omegal ovi rus (CMV)] and i nfl ammat ory di s eas es (e.g., rheumat oi d art hri t i s , s arcoi d). The fi ndi ng i s nons peci fi c.
c. Monocyt os i s can accompany es s ent i al l y al l of t he hemat ol ogi c and l ymphat i c mal i gnanci es .
B. Basophils, eosinophils, and neutrophils Di s orders i n t hes e cel l s al s o are cl as s i fi ed accordi ng t o fl uct uat i ons i n cel l numbers and t o funct i onal defi ci ency.
1. Basophils. An abnormal l y i ncreas ed number of bas ophi l s i s cal l ed basophilia. Thi s uncommon condi t i on us ual l y i s as s oci at ed wi t h t he myel oprol i ferat i ve s yndromes , part i cul arl y chroni c myel ogenous l eukemi a (CML).
2. Eosinophils. An abnormal l y i ncreas ed number of eos i nophi l s i s cal l ed eosinophilia. Thi s condi t i on i s more common t han bas ophi l i a and occurs mos t commonl y s econdary t o ot her di s eas e proces s es , i ncl udi ng: o
Pa g e 4 5 8
ABC Ambe r CHM Conve rte r Tria l ve rsion, http://w w w .proce sste x t.com/a bcchm.html
o
a. Neopl as i a (e.g., l ymphoma, Hodgki n's di s eas e)
o
o
b. Addi s on's di s eas e
o
o
c. Al l ergi c and at opi c di s eas e (mos t common caus e of eos i nophi l i a)
o
o
d. Col l agen vas cul ar di s eas e (e.g., necrot i zi ng vas cul i t i s )
o
o
e. Paras i t i c i nfes t at i on
o
o
f. Pri mary hypereos i nophi l i c s ympt oms are uncommon. Thes e s yndromes are charact eri zed by very hi gh (20,000–50,000) l evel s of s us t ai ned eos i nophi l i a, i nvas i ve t oxi c effect s t o t he heart and l ungs , and chromos omal cl onal aberrat i ons t ypi cal of mal i gnancy.
3. Neutrophils o
o
a. Neutrophilia i s an exces s i vel y i ncreas ed number of neut rophi l s i n t he bl ood. Caus es of neut rophi l i a i ncl ude t he fol l owi ng:
(1) Mos t cas es of neut rophi l i a res ul t from condi t i ons s uch as i nfect i on, t umor, s t res s , col l agen di s orders , and s t eroi ds ; t he
Pa g e 4 5 9
ABC Ambe r CHM Conve rte r Tria l ve rsion, http://w w w .proce sste x t.com/a bcchm.html
underl yi ng di s eas e may not be apparent cl i ni cal l y.
(2) In unus ual cas es , more t han 50,000 3
neut rophi l s /mm appear i n t he bl ood. Thes e s o-cal l ed leukemoid reactions can be di fferent i at ed from t he l eukemi as by t he abs ence of t he ci rcul at i ng bl as t forms and by t he fi ndi ng of el evat ed LAP val ues . P.103
(3) Neut rophi l i a al s o res ul t s from neoplastic marrow diseases s uch as pol ycyt hemi a vera and CML.
(a) Diagnosis of CML. CML i s s us pect ed i n pat i ent s wi t h exces s i ve W BC count s and s pl enomegal y. Peri pheral bl ood s mears reveal a s pect rum of cel l forms rangi ng from mat ure pol ymorphonucl ear neut rophi l s t o i mmat ure bl as t s . Several fi ndi ngs confi rm t he di agnos i s : t he pres ence of an abnormal marker chromos ome (t he Philadelphia chromosome) i n t he marrow precurs or cel l s and very l ow LAP l evel s . The fus i on prot ei n of t he abnormal gene i n CML i s det ect abl e, as i s t he abnormal genet i c mat eri al i t s el f. The fus i on prot ei n res ul t s i n a funct i onal , mut ant t yros i ne ki nas e t hat res ul t s i n a l ack of cont rol of bas i c
Pa g e 4 6 0
ABC Ambe r CHM Conve rte r Tria l ve rsion, http://w w w .proce sste x t.com/a bcchm.html
cel l ul ar proces s es s uch as prol i ferat i on rat e, adherence, and phys i ol ogi c deat h (apopt os i s ). Affect ed s t em cel l s mani fes t a prol i ferat i ve advant age over normal s t em cel l s , reduced adherence t o marrow s t roma, and decreas ed apopt os i s .
(i) CML i s now di agnos ed us i ng pol ymeras e chai n react i on (PCR) t echnol ogy t o demons t rat e t he pres ence of t hi s abnormal genet i c mat eri al . PCR i s abl e t o reveal t he abnormal chromos omal fus i on, even when t he marrow appears normal us i ng morphol ogy and rout i ne cyt ogenet i cs .
(ii) Furt her bi ochemi cal s t udi es have demons t rat ed t hat t he fus i on of chromos omal mat eri al of chromos omes 9 and 22 and i t s res ul t ant fus i on prot ei n caus e marrow s t em cel l s t o “t urn off†• t he apopt os i s pat hway of programmed cel l deat h. Thus , t hes e abnormal cel l s are “i mmort al i zed― and creat e an accumul at i on of marrow and peri pheral bl ood el ement s t hat are cl i ni cal l y recogni zed as CML.
(b) Prognosis of CML. The medi an s urvi val rat e for CML pat i ent s i s 3–4
Pa g e 4 6 1
ABC Ambe r CHM Conve rte r Tria l ve rsion, http://w w w .proce sste x t.com/a bcchm.html
years . Mos t pat i ent s di e duri ng t he s o-cal l ed blast crisis, when t hi s chroni c di s order convert s i nt o a hi ghl y mal i gnant vari et y of acut e l eukemi a.
(i) Hydroxyurea i s an effect i ve pal l i at i ve agent t hat reduces t he l eukemi a cel l burden i n t he chroni c phas e of CML. Pat i ent s obt ai n a cl i ni cal and hemat ol ogi c remi s s i on, but t he abnormal cyt ogenet i cs and fus i on prot ei n remai n and are demons t rabl e on s t udy of marrow and peri pheral bl ood.
(ii) α-Interferon (α-IFN) admi ni s t ered t hree t i mes weekl y al s o i nduces remi s s i on i n 70% of pat i ent s . In 40%–60% of cas es , i t can convert t he pat i ent 's marrow t o Phi l adel phi a chromos ome–negat i ve s t at us and may be curat i ve i n s ome pat i ent s .
(iii) Imat i ni b mes yl at e (Gl eevec) i s a des i gned BCR-ABL t yros i ne ki nas e i nhi bi t or t hat revers es t he effect s of t hat fus i on prot ei n. It i nduces remi s s i on i n >90% and major cyt ogenet i c revers al i n >50% of cas es , and has become t he i ni t i al t herapy of choi ce i n CML. Drug res i s t ance over t i me i s a probl em
Pa g e 4 6 2
ABC Ambe r CHM Conve rte r Tria l ve rsion, http://w w w .proce sste x t.com/a bcchm.html
(i .e., durabi l i t y of res pons e), Ot her t yros i ne ki nas e i nhi bi t ors (e.g., des at i ni b and MN-107) have s hown s i mi l ar act i vi t y and can be effect i ve i n pat i ent s who become refract ory t o i mat i ni b. The el uci dat i on of how abnormal , mal i gnant genet i c t rans format i on res ul t s i n di s eas e and t he abi l i t y t o s peci fi cal l y, pharmacol ogi cal l y i nt ervene i n t hat proces s marks a mi l es t one i n cancer t herapeut i cs .
(iv) Marrow transplantation i s curat i ve and i s t he t herapy of choi ce for younger pat i ent s (s ee
IV D 4).
o
o
b. Neutropenia i s an abs ol ut e decreas e i n t he number of ci rcul at i ng neut rophi l s . Neut ropeni a occurs rarel y as an earl y mani fes t at i on of i nt ri ns i c marrow di s eas e (e.g., acut e l eukemi a) but more commonl y s econdary t o exogenous s t i mul i .
(1) Infections. Cert ai n vi ral i nfect i ons (e.g., hepat i t i s , i nfl uenz a) and bact eri al i nfect i ons (e.g., t yphoi d fever) caus e neut ropeni a.
(2) Drugs (e.g., phenot hi azi nes , fl uoxet i ne, ant i t hyroi d medi cat i ons ) are as s oci at ed wi t h neut ropeni a and, i n s evere i ns t ances , can caus e agranul ocyt os i s . Agranul ocyt os i s i s charact eri zed by:
Pa g e 4 6 3
ABC Ambe r CHM Conve rte r Tria l ve rsion, http://w w w .proce sste x t.com/a bcchm.html
(a) Profoundl y l owered neut rophi l count s 3
(i .e., <500/mm )
(b) Severe pros t rat i on, hi gh fever, and oft en a necrot i c pharyngi t i s
(c) Bone marrow s howi ng maturation arrest (i .e., l arge numbers of i mmat ure W BC forms i n an ot herwi s e normal marrow) P.104
(d) Hi gh mort al i t y rat es unl es s t reat ed earl y and aggres s i vel y wi t h s upport i ve meas ures and pot ent bact eri ci dal ant i bi ot i cs . Us e of genet i cal l y engi neered CSF and ot her marrow growt h fact ors has been encouragi ng. W BC G-CSF has been s hown t o reduce febri l e days and hos pi t al days .
(3) The collagen vascular diseases [e.g., s ys t emi c l upus eryt hemat os us (SLE), rheumat oi d art hri t i s ] have been s hown t o caus e neut ropeni a vi a i mmune des t ruct i on of W BCs .
(4) Familial forms of neutropenia i ncl ude fami l i al beni gn chroni c neut ropeni a, cycl i c neut ropeni a, and chroni c i di opat hi c
Pa g e 4 6 4
ABC Ambe r CHM Conve rte r Tria l ve rsion, http://w w w .proce sste x t.com/a bcchm.html
neut ropeni a. Thes e di s orders have good prognos es , al t hough t hey are as s oci at ed wi t h i ncreas ed nui s ance i nfect i ons (e.g., boi l s ) when t he neut rophi l count i s l es s t han 3
500/mm . They res pond wel l t o G-CSF.
(5) Chemotherapeutic agents us ed i n t he t herapy of mal i gnant di s eas e are t he mos t common caus e of neut ropeni a. Chemot herapy-i nduced neut ropeni a can be markedl y amel i orat ed i n di ffi cul t cas es by t he judi ci ous us e of G-CSF.
o
o
c. Functional disorders of neut rophi l s i nvol ve a compromi s ed abi l i t y t o fi ght i nfect i on. Thes e condi t i ons are rare; affect ed i ndi vi dual s have recurrent i nfect i ons .
(1) Chédiak-Higashi syndrome i s an aut os omal reces s i ve di s order charact eri zed by al bi ni s m and i ncreas ed pyogeni c i nfect i on. In addi t i on, neut rophi l s and ot her granul e-cont ai ni ng cel l s (e.g., mel anos omes ) reveal gi ant , fus ed, peroxi das e-s t ai ni ng granul es wi t h decreas ed abi l i t y t o ki l l i nges t ed mi crobes .
(2) Chronic granulomatous disease (CGD) of chi l dhood i s an X-l i nked di s order i n neut rophi l met abol i s m charact eri zed by a defect i n neut rophi l -free radi cal format i on (as s oci at ed wi t h t he oxi dat i ve burs t and ki l l i ng act i vi t y i n neut rophi l s ), res ul t i ng i n s us cept i bi l i t y t o recurrent s uppurat i ve i nfect i ons . Thi s condi t i on oft en i s fat al , and t he di agnos i s i s confi rmed by t he pres ence of neut rophi l s t hat cannot oxi di ze ni t robl ue t et razol i um dye t o bl ue-bl ack from col orl es s . BMT has been effect i ve i n t hes e cas es .
Pa g e 4 6 5
ABC Ambe r CHM Conve rte r Tria l ve rsion, http://w w w .proce sste x t.com/a bcchm.html
IV. Acute Leukemias and Myelodysplasias The acut e l eukemi as are di s orders i n t he mat urat i on of hemat opoi et i c t i s s ue t hat are charact eri zed by t he pres ence of i mmat ure l eukocyt es i n t he marrow and peri pheral bl ood. The i mmat ure cel l s are arres t ed i n t he earl i es t phas es of di fferent i at i on and are referred t o as blasts. Myel odys pl as i a i s a rel at ed cl onal di s order of t he pl uri pot ent i al s t em cel l s , res ul t i ng i n i nadequat e, i neffect i ve eryt hropoi es i s , dys morphi c changes i n t he marrow, and peri pheral cyt openi as .
A. Acute leukemia classification and epidemiology It i s i mport ant bot h prognos t i cal l y and t herapeut i cal l y t o di s t i ngui s h t he l ymphocyt i c from t he nonl ymphocyt i c (myel ogenous ) l eukemi as . Cl as s i fi cat i on of cel l s i nvol ves s peci al hi s t ochemi cal s t ai ns (e.g., peroxi das e i n myel ogenous l eukemi a), marker enzymes (e.g., t ermi nal t rans feras e i n ALL), and s peci fi c cel l -s urface ant i geni c markers ; us e of t hes e met hods i n combi nat i on al l ows cl as s i fi cat i on t hat approaches 95% accuracy. Cyt ogenet i c anal ys i s , marker chromos ome defect s , and chromos omal bandi ng t echni ques have even furt her refi ned and made more accurat e cl as s i fi cat i on of acut e l eukemi as . In addi t i on, s uch chromos omal groupi ngs appear t o i dent i fy s ubgroups wi t hi n s peci fi c l eukemi a t ypes t hat have di fferent res pons es t o t herapy and di fferent prognos es . Current l y, acut e l eukemi as are cl as s i fi ed i nt o t wo major t ypes , acut e l ymphocyt i c l eukemi a (ALL) and acut e myel ogenous l eukemi a (AML) [
Onl i ne Tabl e 3-2].
ONLINE TABLE 3-2 Classification of the Acute Leukemias
Pa g e 4 6 6
ABC Ambe r CHM Conve rte r Tria l ve rsion, http://w w w .proce sste x t.com/a bcchm.html
A Fr F M e e L q at / u ur A e e L n s L cy S u bt y p e A c ut e M y el o g e n o u s L e u k e m ia (
Pa g e 4 6 7
ABC Ambe r CHM Conve rte r Tria l ve rsion, http://w w w .proce sste x t.com/a bcchm.html
A M L) M U 1: nc m o ye m lo m bl o as n ti c ( wi th o ut m at ur at io n) M C M 2: o os m m t ye m fr lo o e bl n q as
u
ti
e
c
nt
(
A
wi
M
th
L
m
Pa g e 4 6 8
ABC Ambe r CHM Conve rte r Tria l ve rsion, http://w w w .proce sste x t.com/a bcchm.html
at ur at io n) M U Bl 3: nc as pr o t s o m wi m m th ye o gi lo n a cy
nt
ti
gr
c
a n ul es DI C is a co m m o n co m pl ic at io
n M C Ti 4: o s s
Pa g e 4 6 9
ABC Ambe r CHM Conve rte r Tria l ve rsion, http://w w w .proce sste x t.com/a bcchm.html
m m u ye m e lo o in m n fi l o
tr
n
at
oc
io
yt
n
ic M U Ti 5: nc s s m o u o m e n m in oc o fi l yt n t r ic
at io
n M R Fo 6: ar r er e m yt
at
hr
io
ol
n
e
of
uk
m
e
ul
m
ti
ia
n uc le at e d R
Pa g e 4 7 0
ABC Ambe r CHM Conve rte r Tria l ve rsion, http://w w w .proce sste x t.com/a bcchm.html
B C bl as ts in b o n e m ar ro w M R Ac 7: ar ut m e e e
m
g
ye
ak
lo
ar
fi
yo
br
bl
os
as
is
ti c A c ut e L y m p h o
Pa g e 4 7 1
ABC Ambe r CHM Conve rte r Tria l ve rsion, http://w w w .proce sste x t.com/a bcchm.html
cy ti c L e u k e m ia ( A L L) Ac C Cl ut o as e m si ly m c m o ac p n ut h
e
o
le
bl
uk
as
e
ti
m
c
ia
le
of
uk
ch
e
il
m
d
ia
h o o d Bce
Pa g e 4 7 2
ABC Ambe r CHM Conve rte r Tria l ve rsion, http://w w w .proce sste x t.com/a bcchm.html
ll or ig in B U S ur nc m ki o al tt'm l, s m va n o cu e n ol o
at
pl
e
as
d
m
bl
s
as t fo r m s B-
(l
ce
e
ll
uk
or
e
ig
m
in
ia a n d ly m p h o
Pa g e 4 7 3
ABC Ambe r CHM Conve rte r Tria l ve rsion, http://w w w .proce sste x t.com/a bcchm.html
m a) W or se pr o g n os is A U Af d nc fe ul o ct t m s T- m yo ce o u ll n n le
g
uk
a
e
d
m
ul
ia
t m e n La rg e m e di as ti n al
Pa g e 4 7 4
ABC Ambe r CHM Conve rte r Tria l ve rsion, http://w w w .proce sste x t.com/a bcchm.html
m as se s a n d C N S in vo lv e m e nt CNS, cent ral nervou s s ys t em ; DIC, di s s em i nat ed i nt rava s cul ar coagul at i on; RBC, red bl ood cel l .
1. ALL i s common i n chi l dren: 85% of cas es of ALL occur
Pa g e 4 7 5
ABC Ambe r CHM Conve rte r Tria l ve rsion, http://w w w .proce sste x t.com/a bcchm.html
i n chi l dren, and 90% of l eukemi a t hat occurs i n chi l dren i s ALL. Convers el y, ALL i s not a common l eukemi a i n adul t s . Techni ques s uch as membrane s urface markers and ant i body det ect i on of s urface ant i gens have enabl ed i nves t i gat ors t o charact eri ze ALL s ubs et s . o
o
a. The mos t common ALL vari ant (compri s i ng 75% of cas es ) i s of B-cel l l i neage of nul l vari et y (i .e., t here i s abs ent ros et t e format i on). Thi s vari ant al s o expres s es t he common ALL ant i gen (CALLA) on t he cel l s urface.
o
o
b. T-cel l ALL and ot her l es s common vari et i es cons t i t ut e t he remai nder of cas es .
o
o
c. Mos t ALL vari et i es expres s t ermi nal deoxynucl eot i dyl t rans feras e (TdT), and s t ai ni ng for t hi s enzyme i s us eful i n di fferent i at i ng ALL from myel ogenous l eukemi as .
o
o
d. Di fferent i at i ng s ubt ypes has prognos t i c s i gni fi cance i n t hat , for exampl e, T-cel l vari et i es of ALL are more res i s t ant t o t herapy and have far wors e prognos es t han common vari et y B-cel l ALL.
2. AML, or acute nonlymphocytic leukemia (ANLL), i s common i n adul t s , and t he i nci dence i ncreas es wi t h age. Speci fi c envi ronment al ri s ks i ncl ude moderat e-t o-hi gh dos es of i oni zi ng radi at i on, chemi cal s s uch as benzene and pet rol eum product s , and pri or expos ure t o cyt ot oxi c chemot herapy agent s s uch as al kyl at i ng drugs (e.g., cycl ophos phami de, chl orambuci l ). The AML cel l of ori gi n probabl y ari s es at di fferent l evel s of hemat opoi es i s i n di fferent
Pa g e 4 7 6
ABC Ambe r CHM Conve rte r Tria l ve rsion, http://w w w .proce sste x t.com/a bcchm.html
pat i ent s , whi ch account s for t he cl i ni cal l y wel l -defi ned s ubt ypes of AML. In mos t cas es , t he AML cl one ari s es from mul t i pot ent i al precurs ors capabl e of di fferent i at i ng i nt o granul ocyt e, eryt hrocyt e, macrophage, or megakaryocyt e col ony-formi ng uni t s (CFUs ). Therefore, i n mos t pat i ent s , l ymphoi d and eryt hroi d l i neages are not i nvol ved i n t he l eukemi c proces s (
Onl i ne Fi gure 3-6).
ONLINE FIGURE 3-6 Acut e myel oi d l eukemi a. A. Not e t he myel obl as t s and cyt opl as mi c pi nk/red rod-l i ke s t ruct ure (Auer rod) i n t he upper ri ght corner of t he s l i de. B. Several exampl es of Auer rod morphol ogy. (From Tkachuk DC, Hi rs chmann JV. Wi nt robe's At l as of Cl i ni c al Hemat ol ogy. Bal t i more: Li ppi ncot t W i l l i ams & W i l ki ns , 2007:2–57& 2–6B. ) P.105
B. Clinical features Clinical features of acut e l eukemi a repres ent t he effect s of marrow i nfi l t rat i on by nonmat uri ng, funct i onl es s bl as t cel l s , i ncl udi ng
Pa g e 4 7 7
ABC Ambe r CHM Conve rte r Tria l ve rsion, http://w w w .proce sste x t.com/a bcchm.html
s ubs equent bone marrow fai l ure.
1. Physical findings i ncl ude fat i gue and pal l or (due t o anemi a); fever and i nfect i on (due t o neut ropeni a); and pet echi ae, purpura, and epi s t axi s (due t o t hrombocyt openi a). Infi l t rat i ve s ympt oms may i ncl ude s pl enomegal y, gi ngi val hypert rophy, and bone pai n.
2. Laboratory findings i ncl ude al mos t uni vers al pancyt openi a and ci rcul at i ng bl as t forms . Increas ed cel l t urnover res ul t s i n el evat ed uri c aci d l evel s . In promyel ocyt i c l eukemi a (M3), DIC i s us ual l y pres ent wi t h hypofi bri nogenemi a and el evat ed fi bri n s pl i t product s .
C. Diagnosis The di agnos i s of acut e l eukemi a i s confi rmed by t he fi ndi ng of bl as t i nfi l t rat i on of t he bone marrow. Speci al hi s t ochemi cal s t ai ns (e.g., peroxi das e, Sudan bl ack), enzyme markers (e.g., TdT), s urface ant i geni c markers , and chromos ome cyt ogenet i c s t udi es are performed t o i dent i fy t he s peci fi c s ubt ype of l eukemi a i nvol ved.
D. Therapy Ini t i al t herapy cons i s t s of chemot herapeut i c abl at i on of t he l eukemi c cel l l i ne. In AML, i t i s neces s ary t o abl at e al l normal marrow as wel l . Becaus e normal marrow has a s hort er generat i on t i me t han l eukemi c bl as t s , recovery wi t h normal marrow t i s s ue i s pos s i bl e. Thi s i ni t i al marrow abl at i on i s t ermed induction therapy and i s fol l owed by s everal cycl es of cons ol i dat i on t herapy.
1. ALL induction therapy. ALL bl as t s are i ni t i al l y more s el ect i vel y s ens i t i ve t o chemot herapy t han AML bl as t s . It oft en i s pos s i bl e t o des t roy ALL bl as t s wi t h s ome s pari ng of normal marrow. Thus , i nduct i on t herapy for ALL i s as s oci at ed wi t h l ower morbi di t y and mort al i t y rat es t han i t i s for AML. ALL i nduct i on i s general l y fol l owed by l es s i nt ens i ve mai nt enance t herapy.
See more
Pa g e 4 7 8
ABC Ambe r CHM Conve rte r Tria l ve rsion, http://w w w .proce sste x t.com/a bcchm.html
i nformat i on onl i ne. o
o
a. Combi nat i ons of vi ncri s t i ne, predni s one, ant hracycl i nes , and L-as paragi nas e are us ed t o obt ai n i ni t i al remi s s i ons .
o
o
b. Cons ol i dat i on t herapy and a peri od of l es s i nt ens i ve mai nt enance t herapy wi t h agent s s uch as mercapt opuri ne and met hot rexat e are effect i ve.
o
o
c. Treat ment of pot ent i al s anct uary s i t es s uch as t he CNS i s requi red i n many cas es of ALL.
o
o
d. BMT pl ays a rol e, us ual l y aft er i ni t i al rel aps e. Chemot herapy has excel l ent res ul t s i n ALL, far s uperi or t o t hos e i n AML, es peci al l y i n chi l dren younger t han 15 years i n whom t he cure rat e approaches 80%. In adul t s , compl et e remi s s i on rat es of 80% can be achi eved, us ual l y for l onger t han 2 years . However, onl y 15%–20% of t hes e pat i ent s remai n i n remi s s i on.
2.
AML induction therapy
requi res abl at i on of al l marrow
el ement s , bot h bl as t s and normal t i s s ue. The bes t avai l abl e regi men remai ns cyt os i ne arabi nos i de and an ant hracycl i ne ant i bi ot i c. More aggres s i ve admi ni s t rat i on of cycl es of chemot herapy aft er i nduct i on t o el i mi nat e res i dual l eukemi c cel l s does prol ong t he remi s s i on durat i on and i s benefi ci al . Dat a i ndi cat e t hat : o
o
a. Indi vi dual s wi t h AML have a 60%–70% compl et e res pons e rat e t o i nduct i on regi mens .
Pa g e 4 7 9
ABC Ambe r CHM Conve rte r Tria l ve rsion, http://w w w .proce sste x t.com/a bcchm.html o
o
b. W i t h varyi ng pos t i nduct i on regi mens , t he medi an res pons e durat i on i s 12–15 mont hs , wi t h 25%–35% of t hes e pat i ent s experi enci ng 24-mont h di s eas e-free s urvi val .
o
o
c. Of i ndi vi dual s wi t h AML, 15%–25% are al i ve 2 years aft er t hei r di agnos i s . Some of t hes e wi l l rel aps e and s ome wi l l be “cured.―
3.
Radiotherapy i s us ed i n ALL t o s t eri l i ze s anct uary s i t es of
l at e rel aps e. Such s i t es are found i n t he CNS and, pos s i bl y, i n t he t es t es .
4.
BMT has emerged as defi ni t i ve, curat i ve t herapy for acut e
l eukemi a. BMT us ual l y i s us ed aft er t he fi rs t remi s s i on has been obt ai ned. The goal of BMT i s permanent eradi cat i on of t he res i dual l eukemi a and prevent i on of rel aps e. Ul t ra–hi gh-dos e chemot herapy al one or wi t h radi ot herapy pri or t o BMT may be gi ven i n dos es t hat are not l i mi t ed by concerns of t oxi ci t y t o t he marrow, becaus e t he s ubs equent marrow i nfus i on and t rans pl ant at i on “res cues ― t he pat i ent from t he marrow t oxi ci t i es t hat t hes e t herapi es produce. o
o
a. Sui t abl e donors are a prerequi s i t e for s ucces s ful BMT. Allogeneic BMT us es s t em cel l s obt ai ned from human l eukocyt e ant i gen (HLA)–mat ched donors .
(1) Ident i cal t wi n donors are opt i mal but rare. The mos t common donors (35%) are
Pa g e 4 8 0
ABC Ambe r CHM Conve rte r Tria l ve rsion, http://w w w .proce sste x t.com/a bcchm.html
HLA-mat ched s i bl i ngs .
(2) Dat a cont i nue t o s how t hat younger pat i ent s do bet t er wi t h BMT, part i cul arl y regardi ng t he i nci dence of graft -vers us -hos t di s eas e (GVHD) and abi l i t y t o t ol erat e t hi s effect (s ee onl i ne IV D 4 c). Nevert hel es s , wi t h i ncreas i ng experi ence and t echnol ogi cal i mprovement s , t he upper age l i mi t for BMT cont i nues t o ri s e, and i t now exceeds 50 years i n many t rans pl ant at i on cent ers .
o
o
b. W hen no mat ched donors are avai l abl e, autologous BMT us i ng marrow i nfus i on of harves t ed and s t ored remi s s i on marrow s howi ng i mproved s urvi val rat es has accrued. A recent revi ew demons t rat ed a 48% s urvi val rat e at 4 years i n pat i ent s undergoi ng t rans pl ant at i on i n fi rs t remi s s i on. The major caus e of fai l ure i s recurrent l eukemi a aft er t rans pl ant at i on. Vari ous marrow purgi ng t echni ques t o remove l eukemi c cel l s are under s t udy.
o
o
c. GVHD i s a major obs t acl e encount ered i n marrow t rans pl ant at i on. Thi s react i on res ul t s from t he graft i ng of i mmunocompet ent donor T cel l s , whi ch res pond t o al l oant i gens expres s ed on hos t cel l s .
(1) Major t arget organs are s ki n (dermat i t i s ), gas t roi nt es t i nal t ract (di arrhea), and l i ver.
(2) Therapy may be prol onged and us ual l y
Pa g e 4 8 1
ABC Ambe r CHM Conve rte r Tria l ve rsion, http://w w w .proce sste x t.com/a bcchm.html
cons i s t s of s ome combi nat i on of predni s one, cycl os pori ne, and l ow-dos e az at hi opri ne.
d. Ot her i mport ant pos t t rans pl ant at i on fact ors i ncl ude t he t oxi ci t y of t he preparat i ve regi mens , i nt ers t i t i al pneumoni t i s , ot her i nfect i ons , graft reject i on (rare), and hepat i c veno-occl us i ve di s eas e.
5. Supportive therapy i s ext remel y i mport ant and s erves as a prot ot ype for s upport i ve t herapy i n ot her forms of neopl as i a. The fol l owi ng pri nci pl es appl y t o t he us e of chemot herapy regi mens as wel l as t o bone marrow abl at i on and t rans pl ant at i on. o
o
a. The hemoglobin level s houl d be mai nt ai ned above 8 g/dL by t rans fus i ons of packed RBCs . Leukodepl et ed and radi at ed bl ood i s us ed t o avoi d al l os ens i t i zat i on i n t he event of l at er BMT.
o
o
b. Prophyl act i c t rans fus i on of pl at el et concent rat es avoi ds s eri ous s pont aneous bl eedi ng when t he 3
pl at el et count decl i nes bel ow 10,000/mm , and pl at el et t rans fus i on i s us ed t o mai nt ai n pl at el et 3
count s above 10,000/mm . Thi s pract i ce has l owered t he i nci dence of fat al hemorrhage i n acut e l eukemi a from 80% t o l es s t han 20%. o
o
c. Control of infection i s a major det ermi nant of s urvi val for pat i ent s bei ng t reat ed for acut e l eukemi a.
Pa g e 4 8 2
ABC Ambe r CHM Conve rte r Tria l ve rsion, http://w w w .proce sste x t.com/a bcchm.html
(1) Infections are treated early and aggressively. Neut ropeni c pat i ent s (t hos e 3
wi t h PMNs <500/mm ) are s us cept i bl e t o al l organi s ms , es peci al l y gram-negat i ve rods and fungi (e.g., Candi da, As pergi l l us ).
(a) Ini t i al t emperat ure el evat i ons requi re t horough cl i ni cal eval uat i on, vi gorous cul t ures , and i mmedi at e and empi ri c t reat ment wi t h broad-s pect rum bact eri ci dal ant i bi ot i c combi nat i ons of cephal os pori ns ami nogl ycos i des vancomyci n, and s emi s ynt het i c peni ci l l i ns (e.g., i mi peni m).
(b) Secondary t emperat ure el evat i ons t hat occur aft er t reat ment wi t h pot ent broad-s pect rum ant i bi ot i cs may requi re t he empi ri c us e of ant i fungal agent s (e.g., amphot eri ci n B). Prevent i on of fungal i nfect i ons wi t h prophyl act i c t ri azol es (e.g., fl uconazol e) has proved effect i ve and has cont ri but ed t o di mi ni s hed morbi di t y and mort al i t y from fungal i nfect i ons .
See more i nformat i on onl i ne.
P.106
(c) At t ent i on t o el ect rol yt es and uri c aci d i s requi red i n t hos e pat i ent s wi t h hi gh cel l t urnover and i n t hos e recei vi ng mul t i pl e ant i bi ot i cs .
(i) Treat ment wi t h al l opuri nol and adequat e fl ui ds i s requi red for uri c aci d cont rol .
Pa g e 4 8 3
ABC Ambe r CHM Conve rte r Tria l ve rsion, http://w w w .proce sste x t.com/a bcchm.html
(ii) El ect rol yt e l evel s , es peci al l y t he pot as s i um l evel , mus t be mai nt ai ned i n pat i ent s bei ng gi ven ant i bi ot i cs .
(2) St ri ct hand-was hi ng and s ki n-care t echni ques , t he us e of l ong-t erm cent ral cat het ers t o avoi d peri pheral i ndwel l i ng l i nes , and t he pract i ce of s t ri ct rect al care decreas es t he l i kel i hood of i nfect i on.
d. Stimulation of marrow recovery us i ng growt h fact ors made by recombi nant DNA t echnol ogy i s fi ndi ng a rol e i n s upport i ve t herapy. Myel oi d CSFs res ul t i n more rapi d recovery of PMNL count s i n pat i ent s wi t h AML t reat ed wi t h ei t her chemot herapy or BMT. The rout i ne addi t i on of G-CSF t o t he t herapeut i c regi men once pat i ent s have recei ved and cl eared t hei r chemot herapy has res ul t ed i n a s hort eni ng of t he peri od of s evere neut ropeni a and reduced t he morbi di t y due t o i nfect i ous di s eas e.
E. Prognosis
1. The prognos i s for chi l dren wi t h ALL is very good; more t han 95% obt ai n compl et e remi s s i on. Approxi mat el y 70%–80% of pat i ent s are di s eas e free at 5 years and are l i kel y cured. If rel aps e occurs , s econd
Pa g e 4 8 4
ABC Ambe r CHM Conve rte r Tria l ve rsion, http://w w w .proce sste x t.com/a bcchm.html
compl et e remi s s i ons are pos s i bl e i n mos t cas es . Pat i ent s i n s econd remi s s i ons are candi dat es for BMT, wi t h 35%–65% probabi l i t y of l ong-t erm s urvi val .
2. The prognos i s for pat i ent s wi t h AML is poor. Among pat i ent s recei vi ng t he bes t care, us i ng current chemot herapy regi mens , 75% obt ai n compl et e remi s s i on and 25% di e. Furt hermore, t he us ual durat i on of remi s s i on i s onl y 12–18 mont hs . Al t hough s ome i nves t i gat ors cl ai m 20% cure rat es us i ng i nt ens i ve pos t remi s s i on t herapy, t here i s l i t t l e common experi ence wi t h s i gni fi cant cure rat es . In pat i ent s younger t han 50 years , BMT, bot h al l ogenei c and aut ol ogous , i s t he t reat ment of choi ce aft er obt ai ni ng a fi rs t compl et e remi s s i on Current res ul t s i ndi cat e t hat 50% of young pat i ent s wi t h AML who undergo al l ogenei c BMT experi ence prol onged di s eas e-free i nt erval s and may be cured.
F. Myelodysplastic syndrome (MDS) The myel odys pl as i as are cl onal di s orders charact eri zed cl i ni cal l y and morphol ogi cal l y by defect i ve and i neffect i ve hemat opoi es i s . They are t he res ul t of pat hol ogy i n t he hemat opoi et i c s t em cel l s and are t hus charact eri zed by cyt openi as of varyi ng degree i n red cel l , whi t e cel l , and megakaryocyt i c l i nes . Al l have a t endency t o t ermi nat e i n AML. A vari et y of chromos omal abnormal i t i es have been wel l defi ned i n t hes e s yndromes , t he mos t common bei ng l os s of chromos ome 5 (13%) and chromos ome 7 (5%), t ri s omy 8 (5%), and del et i ons of part s of chromos omes 17 and 20.
Cl as s i fi cat i on s chemes are s omewhat
cumbers ome and di s put ed, but t he reas onabl y es t abl i s hed FAB cl as s i fi cat i on i s s hown i n Onl i ne Tabl e 3-3.
ONLINE TABLE 3-3 FAB Classification of
Pa g e 4 8 5
ABC Ambe r CHM Conve rte r Tria l ve rsion, http://w w w .proce sste x t.com/a bcchm.html
Myelodysplastic Syndromes P er ip h er al Bl M o ar o ro d w S Fi Fi u n n bt di di y n n p g g e s s R C N ef yt or ra o m ct p al or e /h y ni yp a a er n of ce e at l l m l e ul i a as ar t wi o th n dy e sp ce l a ll st li ic
Pa g e 4 8 6
ABC Ambe r CHM Conve rte r Tria l ve rsion, http://w w w .proce sste x t.com/a bcchm.html
n ch e, a us n u g al es ly < a 5 n % e bl m as ia ts R C Pr ef yt es ra o e ct p nc or e e y ni of a a ri n of n e at g m le e i a as d wi t s i th o d ri n er n e o g ce bl e l l as d li t s i n fo d e, r er al m o m s, bl os m as t or
Pa g e 4 8 7
ABC Ambe r CHM Conve rte r Tria l ve rsion, http://w w w .proce sste x t.com/a bcchm.html
t s al e w th ay a s n a 1 n 5 e % m ia R C D ef yt ys ra o pl ct p as or e t i y ni c a a wi n of t h e at 5 m le % i a as â wi t €“ th t 2 ex w 0 ce o % s s ce bl bl l l as as l i t t s n fo es r m s R > 2 ef 5 0 ra % % ct bl â or as €“
Pa g e 4 8 8
ABC Ambe r CHM Conve rte r Tria l ve rsion, http://w w w .proce sste x t.com/a bcchm.html
y t 3 a fo 0 n r % e m bl m s as ia
t
wi
fo
th
r
ex
m
ce
s
ss
or
bl
pr
as
es
ts
e
in
nc
tr
e
a
of
ns
A
fo
u
r
er
m
ro
at
ds
io n C M U hr o p o n to ni oc 2 c yt 0 m es % ye > bl l o 1 as m à t o — fo n 1 r
Pa g e 4 8 9
ABC Ambe r CHM Conve rte r Tria l ve rsion, http://w w w .proce sste x t.com/a bcchm.html
9
oc 0 m yt /L s ic le uk e m ia (C M M L) FAB, French -Ameri can-Bri t i s h. DIC, di s s em i nat ed i nt rava s cul ar coagul at i on; HMW K, hi gh†“mol ec ul ar-w ei ght ki ni no gen; SD, s t anda rd devi at i
Pa g e 4 9 0
ABC Ambe r CHM Conve rte r Tria l ve rsion, http://w w w .proce sste x t.com/a bcchm.html
on.
G. Clinical features Clinical features of t he myel odys pl as i as al l rel at e t o bone marrow fai l ure.
1. Hi s t ory and phys i cal fi ndi ngs i ncl ude anemi a feat ures (progres s i ve fat i gue, dys pnea on exert i on, pal l or); neut ropeni a feat ures (frequent i nfect i ons ); and t hrombocyt openi a feat ures (bl eedi ng and brui s i ng). MDS part i cul arl y affect s t he el derl y (medi an age of ons et i s t he s event h decade), and MDS has repl aced i ron defi ci ency as t he mos t frequent caus e of anemi a i n t hi s age-group.
2. Laborat ory fi ndi ngs i ncl ude cyt openi as , wi t h anemi a bei ng t he mos t frequent pres ent i ng feat ure (90%).
H. Diagnosis A pat i ent wi t h cyt openi a wi l l have normal i ron, B 1 2 , and fol at e l evel s and wi l l be “refract ory― t o t herapy wi t h t hes e hemat i ni cs . Peri pheral s mear demons t rat es a vari et y of abnormal i t i es t hat are s ugges t i ve (t eardrop forms , Pel ger Huet hypogranul at ed hypol obul ar W BC forms , and abnormal pl at el et forms )(
onl i ne Fi gure 3-7). Event ual l y, exami nat i on of bone
marrow i s performed and s hows morphol ogi c abnormal i t i es i ncl udi ng
Pa g e 4 9 1
ABC Ambe r CHM Conve rte r Tria l ve rsion, http://w w w .proce sste x t.com/a bcchm.html
megal obl as t i c RBC, as ynchronous mat urat i on of cyt opl as m and nucl ei , ri nged s i derobl as t forms , mi cromegakaryocyt es , and exces s bl as t forms . Det ai l ed, refi ned cyt ogeni c s t udi es , i ncl udi ng PCR anal ys i s , wi l l reveal t he frequent chromos omal abnormal i t i es ci t ed previ ous l y.
Online Figure 3-7 Peri pheral bl ood s mear wi t h s i gni fi cant l eukocyt os i s . Not e t he mul t i pl e degrees of granul ocyt e mat urat i on i ncl udi ng bands and mat ure neut rophi l s as wel l as marked bas ophi l i a. (From Tkachuk DC, Hi rs chmann JV. Wi nt robe's At l as of Cl i ni c al Hemat ol ogy. Bal t i more: Li ppi ncot t W i l l i ams & W i l ki ns , 2007:4–10)
I. Therapy T herapy of MDS i s compl ex and rel at es t o t he many s yndromes t hat have an ext remel y vari abl e prognos i s .
1. Support i ve t herapy i s t he mos t wi del y offered s t rat egy. The us e of eryt hropoi et i n (EPO) and G-CSF t o amel i orat e anemi a and s evere neut ropeni a i s s upport ed by evi dence-bas ed t ri al s . Becaus e EPO i s effect i ve onl y i n t he mi nori t y, t rans fus i ons are us ed for s ympt omat i c anemi a. Aggres s i ve P.107
Pa g e 4 9 2
ABC Ambe r CHM Conve rte r Tria l ve rsion, http://w w w .proce sste x t.com/a bcchm.html
and judi ci ous us e of ant i bi ot i cs and pl at el et t rans fus i on t echni ques are s i mi l ar t o t hos e us ed i n AML s i t uat i ons .
2. Novel agent s are cont i nuous l y bei ng eval uat ed i n t hi s di ffi cul t pat i ent popul at i on. They i ncl ude l enal i domi de (Revl i mi d), an i mmunomodul ary agent , and az acyt i di ne, an ant i met abol i t e t hat i mpai rs DNA met hyl at i on.
3. In t he s mal l s ubs et of hi gh-ri s k, hi gh-mort al i t y pat i ent s of appropri at e age-group, BMT has been s ucces s ful l y us ed and i s t he onl y curat i ve t herapy avai l abl e.
J. Prognosis Prognosis i s vari abl e. As s t at ed, mos t MDS t ends t o t ermi nat e i n AML. Medi an s urvi val s range from 6 mont hs t o 6 years , dependi ng on a compl ex s et of vari abl es , i ncl udi ng number and s everi t y of cyt openi as ; pres ence or abs ence of cyt ogeni c abnormal i t i es , and percent age of myel obl as t s i n t he bone marrow. In es s ent i al l y al l s ubs et s , t he pres ence of MDS negat i vel y i mpact s s urvi val s i gni fi cant l y even when MDS i s not t he pri mary caus e of deat h.
V. Disorders of Coagulation and Hemostasis A. General considerations Hemos t as i s requi res an i nt act coagul at i on s ys t em of vas cul ar and t i s s ue component s , pl at el et s , and coagul at i on prot ei ns . Defi ci ency or di s eas e of any of t hes e component s may caus e ei t her s pont aneous or t rauma-rel at ed hemorrhage.
1. History. A careful hi s t ory provi des cl ues t o t he pat hogenes i s of bl eedi ng. Immedi at e, mucocut aneous bl eedi ng s ugges t s vas cul ar or pl at el et di s eas e; del ayed
Pa g e 4 9 3
ABC Ambe r CHM Conve rte r Tria l ve rsion, http://w w w .proce sste x t.com/a bcchm.html
deep-t i s s ue bl eedi ng and hemart hros i s s ugges t coagul at i on prot ei n defi ci ency. Genet i c t rans mi s s i ons of bl eedi ng di s orders (e.g., t he X-l i nked hemophi l i as ) al s o are el i ci t ed by hi s t ory, as i s i nges t i on of drugs (e.g., as pi ri n).
2. Physical findings ai d i n di fferent i at i on of bl eedi ng s yndromes . Mucocut aneous pet echi ae and purpura s ugges t pl at el et di s orders ; hemat omas and hemart hros i s s ugges t coagul opat hy.
3. Laboratory testing i s vi t al i n t he eval uat i on of bl eedi ng di s orders . Si ngl e t es t s rarel y provi de concl us i ve res ul t s , s o vari ous bat t eri es of t es t s have been devel oped. The coagul at i on cas cade i s s hown i n Fi gure 3-8 and t he di agnos i s of common bl eedi ng di s orders bas ed on commonl y us ed t es t s i s s hown i n Tabl e 3-4.
B. Disorders of blood vessels and vascular tissues The fol l owi ng bl eedi ng di s orders res ul t from pat hol ogy i n t he ves s el area i t s el f, wi t h s econdary l eakage of bl ood. Mos t of t hes e di s orders have as t hei r hal l mark a vi s i bl e and us ual l y pal pabl e s ki n l es i on. Tes t i ng performed on pat i ent s wi t h t hes e bl eedi ng di s orders reveal s normal coagul at i on.
1. Autoimmune (allergic) purpura (Henoch-Schönlein purpura) occurs mos t commonl y i n chi l dren and young adul t s . o
o
a. Etiology. The s yndrome i s as s oci at ed wi t h s t rept ococcal i nfect i ons and drugs (e.g., peni ci l l i n).
Pa g e 4 9 4
ABC Ambe r CHM Conve rte r Tria l ve rsion, http://w w w .proce sste x t.com/a bcchm.html o
o
b. Clinical signs. Perivascular inflammatory lesions wi t h s eros angui neous l eakage i nt o t he s ki n, s ubmucos a, and s eros a are t he hal l mark of t he di s eas e. The l es i ons are s ymmet ri c and pal pabl e and are us ual l y found on t he di s t al ext remi t i es . Bowel l es i ons may caus e gas t roi nt es t i nal s ympt oms ; joi nt l es i ons caus e art hri t i s .
o
o
c. T herapy and prognosis. No s peci fi c t herapy i s uni forml y hel pful . Prognos i s i s good, except i n t he 5%–10% of pat i ent s who devel op gl omerul onephri t i s .
2. Purpura associated with infections may be due t o embol i c occl us i on of t he mi crovas cul at ure (e.g., endocardi t i s ) or t o endot hel i al i njury by t he i nfect i ous agent (e.g., Ri c ket t s i a). Bi ops y and cul t ure of t he mat eri al may be hel pful .
3. Structural malformations of vessels and vascular tissues are as s oci at ed wi t h t he fol l owi ng condi t i ons : o
o
a. Scurvy i s caus ed by vi t ami n C defi ci ency; col l agen s ynt hes i s i s i mpai red as a res ul t of t hi s defi ci ency. Ves s el wal l s wi t h poor col l agen s upport are pl i abl e and eas i l y rupt ured.
(1) Phys i cal fi ndi ngs as s oci at ed wi t h s curvy i ncl ude peri fol l i cul ar pet echi ae, gi ngi val
Pa g e 4 9 5
ABC Ambe r CHM Conve rte r Tria l ve rsion, http://w w w .proce sste x t.com/a bcchm.html
bl eedi ng, and s ubperi os t eal hemorrhages . Bl eedi ng t i me us ual l y i s prol onged.
(2) Therapy wi t h 1 g/day of vi t ami n C rapi dl y correct s al l bl eedi ng.
P.108
FIGURE 3-8 The coagul at i on cas cade. Each coagul at i on fact or, when act i vat ed, act i vat es t he next fact or i n t he s eri es . (Fact ors are numbered i n order of t hei r di s covery, not i n order of act i vat i on.) In t he i nt ri ns i c s ys t em, fact or XII i s i ni t i al l y act i vat ed by an unknown mechani s m and s ubs equent l y by kal l i krei n duri ng t he cont act phas e. Al t ernat i vel y, fact or VII and t i s s ue fact or can i ni t i at e t he ext ri ns i c s ys t em. Int ri ns i c and ext ri ns i c pat hways each end wi t h act i vat i on of fact or X, s et t i ng off a fi nal ; common pat hway t hat ends wi t h format i on of t he fi bri n cl ot . Open arrow s i ndi cat e convers i on of a s ubs t rat e or a react ant t o a product . HMW K, hi gh–mol ecul ar-wei ght ki ni nogen; TF, t i s s ue fact or; PL,
Pa g e 4 9 6
ABC Ambe r CHM Conve rte r Tria l ve rsion, http://w w w .proce sste x t.com/a bcchm.html
phos phol i pi d. o
o
b. Hereditary hemorrhagic telangiectasia i s an aut os omal domi nant di s order as s oci at ed wi t h abnormal l y t hi n ves s el wal l s and i mpai red vas cul ar cont ract i l i t y. Such ves s el s are markedl y fri abl e, l i abl e t o burs t wi t h t rauma, and unabl e t o cont ract appropri at el y for pri mary hemos t as i s .
(1) Physical findings i ncl ude s mal l , nodul ar vi ol aceous l es i ons on t he l i ps , face, ears , t ongue, and gas t roi nt es t i nal mucos a; t hes e l es i ons bl anch on pres s ure. Bl eedi ng i s common, es peci al l y gas t roi nt es t i nal bl eedi ng and epi s t axi s , wi t h res ul t ant i ron defi ci ency anemi a.
TABLE 3-4 Presumptive Diagnosis of Common Bleeding Disorders by Primary Screening Tests P C r o e m Bl
s m
Pl e
u o
at e
m n
el di
pt Et
et n
iv io
C g
e lo
o Ti P
Di gi
u mT P a e nt e T T g s
Pa g e 4 9 7
ABC Ambe r CHM Conve rte r Tria l ve rsion, http://w w w .proce sste x t.com/a bcchm.html
n o si s D Pr N N T IT ec ol or or hr P; re o m m o dr a n al al m u s g
b g
e e
oc s
d d
yt o p e ni
a N Pr Pr N v â or ol ol or o €¦ mo o mn al n n al W g g
ill
e e
e
d d
br a n d' s di s e a s
e N Pr N N T Dr or ol or or hr u mo mmo g
Pa g e 4 9 8
ABC Ambe r CHM Conve rte r Tria l ve rsion, http://w w w .proce sste x t.com/a bcchm.html
al n al al m s ; g
b ur
e
oc e
d
yt m o ia p at h
y N N Pr N C H or or ol or o e mmo ma m al al n al g o g
ul p
e
o hi
d
p li at a h A y or of h in e tr m in o si p c hi p li at a h B; w fa a ct y or VI II a n
Pa g e 4 9 9
ABC Ambe r CHM Conve rte r Tria l ve rsion, http://w w w .proce sste x t.com/a bcchm.html
d lu p u sty p e in hi bi to rs N N Pr Pr C Li or or ol ol o v m m o o a er al al n n g di g g ul s e e o e d d p a at s h e; y vi of t a co m m in mK o d n ef or i c m ie ul nc t i y; pl D e IC
Pa g e 5 0 0
ABC Ambe r CHM Conve rte r Tria l ve rsion, http://w w w .proce sste x t.com/a bcchm.html
p ; at h h e w p a ar ys i n N N N Pr C F or or or ol o ac m m m o a to al al al n g r g ul VI e o I d p d at ef h ic y ie of nc e y xt ri n si c p at h w a y N N N N â H or or or or €¦ er mmmm
e
al al al al
di ta ry
Pa g e 5 0 1
ABC Ambe r CHM Conve rte r Tria l ve rsion, http://w w w .proce sste x t.com/a bcchm.html
te la n gi ec ta si a; al le rg ic p ur p ur a Adapt ed wi t h permi s s i on from Lee GR, et al : Wi nt robe's Cl i ni c al Hemat ol ogy, 9t h ed. Phi l adel phi a: Lea and Febi ger, 1993:1315. DIC, di s s emi nat ed i nt ravas cul ar coagul at i on; ITP, i di opat hi c
Pa g e 5 0 2
ABC Ambe r CHM Conve rte r Tria l ve rsion, http://w w w .proce sste x t.com/a bcchm.html
t hrombocyt op eni c purpura; PTT, part i al t hrombopl as t i n t i me; PT, prot hrombi n t i me.
o
P.109
o
(2) Diagnosis i nvol ves t he as s oci at i on of t hree fact ors : recurrent hemorrhage, mul t i pl e t el angi ect as es , and fami l i al occurrence.
(3) Improvement i n s upport i ve meas ures s uch as l ocal meas ures (nas al emol l i ent s ), EPO, eps i l on ami no caproi c aci d, and s afer parent eral i ron forms have i mproved qual i t y of l i fe and prognos i s i n t hes e pat i ent s .
o
o
c. Diminished collagen synthesis due t o steroid therapy res ul t s i n a s yndrome of vas cul ar fragi l i t y and s ki n bl eedi ng.
4. Miscellaneous vascular conditions o
o
a. Paraproteinemias, i ncl udi ng cryogl obul i nemi as
Pa g e 5 0 3
ABC Ambe r CHM Conve rte r Tria l ve rsion, http://w w w .proce sste x t.com/a bcchm.html
and amyl oi dos i s , are as s oci at ed wi t h s ki n bl eedi ng. Di agnos i s requi res demons t rat i on of t he paraprot ei n. o
o
b. Senile purpura occurs i n el derl y i ndi vi dual s as a res ul t of degenerat i on and l os s of dermal col l agen, el as t i n, and s ubcut aneous fat . Thi s di s order, whi ch i s t hought t o be caus ed by s heari ng i njury t o bl ood ves s el s from t he hypermobi l i t y of t he s ki n on t he t hi nned underl yi ng t i s s ue, i s charact eri zed by beni gn purpura of t he arms .
C. Disorders of platelets Pl at el et s pl ay a rol e i n t he pri mary arres t of bl eedi ng; abnormal i t i es i n t hes e hemos t at i c component s l ead t o hemorrhagi c di at hes i s . Pl at el et abnormal i t i es are cl as s i fi ed accordi ng t o di s orders of number and funct i on.
1. T hrombocytopenia (i .e., decreas ed numbers of pl at el et s ) i s t he most common cause of abnormal bleeding. o
o
a. General considerations
(1) W i t h a pl at el et count of l es s t han 3
100,000/mm , t here i s a pot ent i al i ncreas ed ri s k of bl eedi ng wi t h t rauma and cert ai n s urgi cal procedures . Mos t i ndi vi dual s experi ence an i ncreas ed t endency t o brui s e wi t h pl at el et count s of bet ween 30,000 and 3
10,000/mm . Format i on of s pont aneous pet echi ae wi t h bl eedi ng may occur wi t h 3
pl at el et count s l es s t han 10,000/mm , wi t h
Pa g e 5 0 4
ABC Ambe r CHM Conve rte r Tria l ve rsion, http://w w w .proce sste x t.com/a bcchm.html
s eri ous s pont aneous bl eedi ng (e.g., CNS) occurri ng wi t h pl at el et count s of l es s t han 3
5,000/mm .
(2) Thrombocyt openi a i s frequent l y an i ndi cat i on for a marrow exami nat i on, whi ch reveal s t he pres ence of t he pl at el et precurs ors , megakaryocytes. The abs ence of megakaryocyt es i ndi cat es pl at el et product i on probl ems . The pres ence of megakaryocyt es i ndi cat es ei t her peri pheral des t ruct i on of pl at el et s or, i n t he pres ence of s pl enomegal y, s pl eni c s eques t rat i on of pl at el et s .
o
o
b. Mechanisms of t hrombocyt openi a i ncl ude i mpai red pl at el et product i on, abnormal pl at el et pool i ng, and i ncreas ed peri pheral des t ruct i on. Vari ous exampl es , marrow fi ndi ngs , and t herapi es are s ummari zed i n Tabl e 3-5.
(1) Impaired platelet production
(a) Etiology
(i) Megakaryocyt es may be s el ect i vel y s uppres s ed by cert ai n agent s (e.g., t hi azi de di uret i cs , et hanol ).
(ii) Impai red pl at el et product i on i s as s oci at ed wi t h t he megal obl as t i c
Pa g e 5 0 5
ABC Ambe r CHM Conve rte r Tria l ve rsion, http://w w w .proce sste x t.com/a bcchm.html
hemat opoi es i s s een i n vi t ami n B 1 2 and fol at e defi ci enci es , as wel l as i n cas es of myel odys pl as t i c and prel eukemi c s yndromes .
(iii) A rare caus e i s amegakaryocyt i c t hrombocyt openi a caus ed by congeni t al defi ci ency of megakaryocyt e CFUs .
(b) Diagnosis i s confi rmed by a bone marrow s mear t hat reveal s marrow megakaryocyt i c hypopl as i a.
(c) T herapy i nvol ves removal of t he offendi ng agent , i f pos s i bl e, or t reat ment of t he underl yi ng di s eas e. Pat i ent s have es s ent i al l y normal pl at el et hal f-l i ves and s houl d be t rans fus ed wi t h exogenous pl at el et s i f t hey are t hrombocyt openi c and bl eedi ng. Thrombocyt openi a as s oci at ed wi t h vi t ami n B 1 2 or fol at e defi ci ency i s rapi dl y correct ed by t herapy wi t h t he defi ci ent vi t ami n.
(d) Associated conditions Impai red pl at el et product i on i s as s oci at ed wi t h apl as t i c anemi a, myel opht hi s i c proces s es (repl acement of marrow by t umor or fi bros i s ), and cert ai n rare congeni t al s yndromes (e.g., t hrombocyt openi a wi t h abs ent radi i ).
Pa g e 5 0 6
ABC Ambe r CHM Conve rte r Tria l ve rsion, http://w w w .proce sste x t.com/a bcchm.html
P.110
TABLE 3-5 Mechanisms of Thrombocytopenia Cli Ma nic Me rro al ch w Co T h ani Fin ndi er sm din tio ap s gs ns y Im Me Ind Re pai gak uce mo red ary d
val
pro ocy che of duc t es mi c off t i o red al l en n
uce y
of
d
di n
or g
pl a or phy ag t el abs s i c ent et s ent al l s ; y
s up
(e. por g., t i v che e mo t he rap y, rad i at i
Pa g e 5 0 7
ABC Ambe r CHM Conve rte r Tria l ve rsion, http://w w w .proce sste x t.com/a bcchm.html
on) Apl Mar as t row ic
t ra
an ns p em l an i a, t at par i on oxy sm al noc t ur nal he mo gl o bi n uri a, l eu ke mi a Inf Tre ect at i on me (e. nt g., of vi r i nf al
ect
he i on pat itis ,
Pa g e 5 0 8
ABC Ambe r CHM Conve rte r Tria l ve rsion, http://w w w .proce sste x t.com/a bcchm.html
cyt om eg al o vi r us , t ub erc ul o sis ) Ine Ab Me B 1 2 ffe nor gal or ct i ma obl fol ve l
as t at e
pro an i c
s up
duc d/o di s pl e tio r
ord me
n
dys er nt a
of
pl a (vi t t i o
pl a s t i am n t el c
in
et s me B 1 2 gak def ary i ci e ocy ncy t es , fol at e def i ci e ncy ) My Tre el o at
Pa g e 5 0 9
ABC Ambe r CHM Conve rte r Tria l ve rsion, http://w w w .proce sste x t.com/a bcchm.html
dys me pl a nt s i a of s
un
an der d
l yi
my ng el o di s pro ord l i fe er rat i ve di s ord ers En Inc Idi Se ha rea op e V nce s ed at h C 1 d
me i c
b
des gak t hr (3) t ru ary om ct i ocy boc on t es yt o of
pe
pl a
ni c
t el
pur
et s
pur
(i m
a Pos Pl a
mu neme di a t ed )
t -t r s m ans ap fus her i on es i pur s pur
Pa g e 5 1 0
ABC Ambe r CHM Conve rte r Tria l ve rsion, http://w w w .proce sste x t.com/a bcchm.html
a Dru Re g-i mo nd val uce of d
off
(e. en g., di n ri fa g mp ag i n, ent me t hi ci l l i n, s ul fon am i de s, ph eny t oi n, qui ni n e, qui ni d i ne , he par i n) Ly Tre
Pa g e 5 1 1
ABC Ambe r CHM Conve rte r Tria l ve rsion, http://w w w .proce sste x t.com/a bcchm.html
mp at ho me ma nt s
of un der l yi ng di s ord er St e roi ds
Im mu ne co mp l ex di s ord er (e. g., s ys te mi c l up us ery t he ma t os us )
Pa g e 5 1 2
ABC Ambe r CHM Conve rte r Tria l ve rsion, http://w w w .proce sste x t.com/a bcchm.html
En Inc Thr Pl a ha rea om s m nce s ed bot ap d
me i c
her
des gak t hr es i t ru ary om s ct i ocy boc on t es yt o of
pe
pl a
ni c
t el
pur
et s
pur
(no
a He Pl a
t im mu nerel at e d)
mo s m l yt i ap c
her
ure es i mi c s s yn dro me Di s Tre s e at mi me nat nt ed of i nt r un ava der s cu l yi l ar ng coa di s gul ord at i ers on
Pa g e 5 1 3
ABC Ambe r CHM Conve rte r Tria l ve rsion, http://w w w .proce sste x t.com/a bcchm.html
Di l Su ut i pp on ort i an ve d car di o pul mo nar y byp as s Spl Us en ual om l y eg not al y req ui r ed; s pl en ect om y ma y be nec es s ary occ as i on al l
Pa g e 5 1 4
ABC Ambe r CHM Conve rte r Tria l ve rsion, http://w w w .proce sste x t.com/a bcchm.html
y
(2) Abnormal platelet pooling res ul t s when pl at el et s are s eques t ered from t he ci rcul at i on. Splenic platelet sequestration i s t he mos t common caus e of abnormal pl at el et pool i ng.
(a) Pathophysiology. Normal l y, t he s pl een hol ds one-t hi rd of t he ci rcul at i ng pl at el et pool . As s pl enomegal y occurs , hi gher numbers of pl at el et s are s eques t ered and, t hus , are unavai l abl e for hemos t as i s . In very l arge s pl eens , as much as 90% of t he pl at el et pool may be s eques t ered; however, pl at el et s i n t he peri pheral ci rcul at i on do have normal s urvi val t i mes .
(b) Diagnosis of hypers pl eni s m i s s ugges t ed by a moderat e t hrombocyt openi a (pl at el et count s of 3
<40,000/mm are unus ual ), a bone marrow as pi rat e t hat reveal s adequat e
Pa g e 5 1 5
ABC Ambe r CHM Conve rte r Tria l ve rsion, http://w w w .proce sste x t.com/a bcchm.html
marrow megakaryocyt es , and evi dence of s i gni fi cant s pl eni c enl argement .
(c) Clinical features i n s uch cas es are domi nat ed by t he underl yi ng i l l nes s caus i ng t he s pl enomegal y (e.g., ci rrhos i s wi t h port al hypert ens i on).
(d) T herapy us ual l y i s not requi red, al t hough s pl enect omy may correct t he probl em. Trans fus ed pl at el et s are s eques t ered i n t he s ame rat i o and are l es s effect i ve t han i n hypoact i ve marrow s t at es .
(3) Increased peripheral destruction of pl at el et s i s t he most common form of thrombocytopenia. Condi t i ons i nvol vi ng i ncreas ed pl at el et des t ruct i on are charact eri zed by s hort ened pl at el et s urvi val and i ncreas ed numbers of marrow megakaryocyt es . Idiopathic thrombocytopenic purpura (IT P) i s t he prot ot ypi cal i mmune-medi at ed t hrombocyt openi a; no apparent exogenous caus es for pl at el et des t ruct i on exi s t . P.111
(a) Clinical features. The acute variant of ITP occurs i n chi l dren bet ween t he
Pa g e 5 1 6
ABC Ambe r CHM Conve rte r Tria l ve rsion, http://w w w .proce sste x t.com/a bcchm.html
ages of 2 and 6 years and oft en occurs aft er a nons peci fi c vi ral i l l nes s . The chronic variant occurs i n young adul t s , more commonl y i n young women. Al l ITP pat i ent s s how varyi ng degrees of t hrombocyt openi a, whi ch i n s ome acut e cas es i s s evere (i .e., as s oci at ed wi t h 3
pl at el et count s of <1000/mm ), and al l s how i ncreas ed megakaryocyt es . Ot her bl ood fi ndi ngs and cel l l i nes are normal . Pat i ent s have mucocut aneous bl eedi ng wi t h pet echi ae, purpura, mucos al bul l ae, and exces s i ve bl eedi ng aft er t rauma.
(b) Diagnosis requi res excl us i on of as s oci at ed i l l nes s es (e.g., SLE) and t hrombocyt openi a t hat i s i nduced by drugs (e.g., qui ni di ne). Pl at el et ant i body t echni ques are avai l abl e but remai n nons peci fi c and are not as hel pful ei t her di agnos t i cal l y or as di s eas e moni t ors as had been hoped.
(i) In pat i ent s who are younger t han 60 years of age wi t h ot herwi s e normal bl ood count s , bone marrow exami nat i on i s no l onger requi red. The cl i ni cal di agnos i s of ITP can be made as l ong as s pl enect omy i s not cont empl at ed (i n whi ch cas e a confi rmat ory marrow exami nat i on i s performed).
Pa g e 5 1 7
ABC Ambe r CHM Conve rte r Tria l ve rsion, http://w w w .proce sste x t.com/a bcchm.html
(ii) In pat i ent s ol der t han 60 years , a marrow exami nat i on s houl d be performed becaus e of t he i ncreas i ng i nci dence of myel odys pl as i a i n s uch pat i ent s .
(iii) New epi demi ol ogi c s t udi es have s hown a hi gh i nci dence of human i mmunodefi ci ency vi rus (HIV) and hepat i t i s C as caus es of t hrombocyt openi a, and many aut hori t i es s ugges t t es t i ng for t hem i n ITP pat i ent s . If appropri at e on cl i ni cal grounds , l upus t es t i ng al s o s houl d be performed. Thes e condi t i ons can mani fes t as apparent l y i s ol at ed t hrombocyt openi a.
(c) Clinical course. The acut e chi l dhood vari ant oft en runs i t s cours e and res ol ves wi t hi n 4–8 weeks ; t he adul t form i s more chroni c and demons t rat es rel aps es and remi s s i ons .
(d) T herapy for t he acut e chi l dhood form us ual l y i nvol ves prot ect i on from t rauma and, i n s ome cas es , a s hort cours e of s t eroi ds . Treat ment of adul t ITP i s more compl ex and prot ract ed.
(i) High doses of steroids produce compl et e remi s s i on and remai n t he
Pa g e 5 1 8
ABC Ambe r CHM Conve rte r Tria l ve rsion, http://w w w .proce sste x t.com/a bcchm.html
core t herapy for chroni c ITP.
(ii) St eroi ds i nduce remi s s i ons i n approxi mat el y t wo-t hi rds of cas es and i n 35% res ul t i n s us t ai ned remi s s i ons requi ri ng no furt her t reat ment .
(iii) Splenectomy may be neces s ary i n res i s t ant cas es and i s as s oci at ed wi t h compl et e remi s s i on. The need for s pl enect omy has di mi ni s hed i n recent years .
(iv) Refractory patients may requi re immunosuppressive therapy (e.g., wi t h azat hi opri ne and cycl ophos phami de). Al t hough platelet transfusions s houl d not be wi t hhel d i n ITP pat i ent s who are bl eedi ng, s uch exogenous t rans fus i ons are l es s effi caci ous t han i n ot her t hrombocyt openi c s t at es becaus e of t he s ame s hort s urvi val of t he pl at el et s .
(v) An effect i ve t herapy t hat can t emporari l y el evat e pl at el et count s i n an acut e cri s i s i s infusion with intravenous IgG. The i nfus ed IgG compet es for and s at urat es ret i cul oendot hel i al bi ndi ng s i t es , maki ng fewer of t hem avai l abl e for
Pa g e 5 1 9
ABC Ambe r CHM Conve rte r Tria l ve rsion, http://w w w .proce sste x t.com/a bcchm.html
pl at el et bi ndi ng and des t ruct i on. Thi s maneuver has al s o gai ned popul ari t y as a preoperat i ve t herapy i n ITP pat i ent s who requi re s urgery. Hyperimmune anti-Rh(D) globulin (e.g., s uch as t hat gi ven t o women aft er gi vi ng bi rt h t o Rh-pos i t i ve i nfant s ) has been found t o act i n a l i ke manner as an urgent t herapy i n ITP and can be us ed i n s i mi l ar s et t i ngs (i .e., pri or t o s urgery or when t hrombocyt openi a i s ext reme) wi t h excel l ent res ul t s . Thes e maneuvers have become t he i ni t i al t herapy i n mos t cas es of acut e and s evere ITP.
(e) Prognosis. The overal l prognos i s i s good; onl y 2%–3% of ITP pat i ent s di e aft er 5 years .
o
o
2. T hrombocytopathia i nvol ves pl at el et s t hat are adequat e i n number but unabl e t o funct i on properl y i n hemos t as i s and i n t he pri mary arres t of bl eedi ng.
a. Description. Thrombocyt opat hi a i s charact eri zed by:
(1) Pl at el et -t ype mucocut aneous bl eedi ng
Pa g e 5 2 0
ABC Ambe r CHM Conve rte r Tria l ve rsion, http://w w w .proce sste x t.com/a bcchm.html
(2) Normal pl at el et count s
(3) Demons t rat e abnormal i t i es i n pl at el et funct i on t es t i ng (e.g., aggregomet ry)
P.112
b. Etiology
(1) Drug-related platelet dysfunction i s t he mos t common caus e of abnormal pl at el et funct i on.
(a) Aspirin permanent l y acet yl at es pl at el et membranes , i mpai ri ng t he pl at el et pros t agl andi n s ynt hes i s [e.g., i mpai ri ng s ynt hes i s of t hromboxane A 2 (TXA 2 )] requi red for proper pl at el et funct i on. Such i mpai red pl at el et s may prol ong bl eedi ng t i mes and caus e brui s i ng and i ncreas e hemorrhage wi t h t rauma. The as pi ri n-i nduced pl at el et l es i on permanent l y al t ers ci rcul at i ng pl at el et s and l as t s unt i l t he pl at el et s are repl aced by new unaffect ed ones , us ual l y i n 3–7 days .
(b) Cl opi dogrel (Pl avi x) i s a new
Pa g e 5 2 1
ABC Ambe r CHM Conve rte r Tria l ve rsion, http://w w w .proce sste x t.com/a bcchm.html
and powerful ant i pl at el et agent t hat s el ect i vel y i nhi bi t s pl at el et recept or bi ndi ng of adenos i ne di phos phat e (ADP) and s ubs equent act i vat i on of t he IIB/IIIA compl ex. Li ke as pi ri n, t he effect i s i rrevers i bl e and l as t s for t he remai nder of t he expos ed pl at el et 's l i fe s pan.
(c) Other anti-inflammatory drugs (e.g., i ndomet haci n) caus e s i mi l ar dys funct i on but di ffer from as pi ri n i n t hat t hei r effect s are not permanent and di s appear when t he agent i s wi t hdrawn.
(2) Uremia-associated dysfunction i s caus ed by uremi c pl as ma di s aggregat i on of hi gh–mol ecul ar-wei ght pol ymers of fact or VIII t hat are requi red for proper pl at el et funct i on. In bl eedi ng uremi c pat i ent s , di al ys i s may evoke a res pons e. Maneuvers t o res t ore normal –mol ecul ar-wei ght forms of fact or VIII may t emporari l y correct t hi s dys funct i on. Thes e maneuvers i ncl ude admi ni s t eri ng fact or VIII exogenous l y as cryopreci pi t at e and el i ci t i ng endogenous i ncreas es i n fact or VIII by gi vi ng des mopres s i n (dDAVP).
(3)Congenital forms of platelet dysfunction i ncl ude Gl anz mann's t hrombas t heni a (defi ci ency of t he IIB/IIIa
Pa g e 5 2 2
ABC Ambe r CHM Conve rte r Tria l ve rsion, http://w w w .proce sste x t.com/a bcchm.html
gl ycoprot ei n) and gray pl at el et s yndrome (abs ence or reduct i on of al pha granul es ). Thes e ent i t i es -can be s ugges t ed by a hi s t ory of fami l i al bl eedi ng, al t hough von W i l l ebrand's di s eas e (a di s order of pl as ma prot ei ns ) i s a more common fami l i al coagul opat hy wi t h a s i mi l ar cl i ni cal pres ent at i on.
D. Disorders of the coagulation system Disorders of the coagulation system may be cl as s i fi ed as heredi t ary or acqui red. The heredi t ary forms us ual l y res ul t from defi ci ency of a s i ngl e coagul at i on prot ei n. Al t hough great l y vari abl e i n degree, t he cl i ni cal mani fes t at i ons of t hes e di s orders are s omewhat s i mi l ar. The acqui red forms , whi ch are more compl ex t han t he heredi t ary forms , us ual l y res ul t from mul t i pl e and mi xed defi ci enci es i n t he coagul at i on prot ei ns .
1. The hereditary coagulopathies are bes t exempl i fi ed by t he di s orders of fact or VIII. o
o
a. Hemophilia A i s t he mos t common heredi t ary coagul opat hy, account i ng for 68%–80% of s uch condi t i ons .
(1) Pathogenesis. Genet i cal l y, hemophi l i a A i s t rans mi t t ed as a cl as s i c X-l i nked reces s i ve t rai t —t he di s order i s carri ed by femal es and mani fes t ed i n mal es . Bl eedi ng res ul t s from t he abs ence of VIII
p ro
, t he
l ow–mol ecul ar-wei ght , or procoagulant portion of t he fact or VIII mol ecul e. The l arge ag
antigenic portion of t he mol ecul e, VIII , i s pres ent i n normal amount s . The genet i c l ocus for hemophi l i a has been el uci dat ed and s t udi ed. The mi l der forms (Tabl e 3-6) are charact eri zed by s i ngl e ami no aci d changes
Pa g e 5 2 3
ABC Ambe r CHM Conve rte r Tria l ve rsion, http://w w w .proce sste x t.com/a bcchm.html
t hat res ul t i n s ynt hes i s of abnormal fact or VIII
p ro
, ant i geni cal l y recogni zabl e but
funct i onal l y defect i ve. Such P.113
pat i ent s do not make ant i bodi es t o fact or VIII
p ro
in
res pons e t o concent rat e t herapy. Severe forms (<1%) are charact eri zed by gene del et i on, whi ch res ul t s i n abs ent ant i geni cal l y recogni zabl e fact or VIII prot ei n mat eri al . Exogenous fact or VIII gi ven as t reat ment i s s een as forei gn prot ei n and i s probl emat i c. Ant i -VIII
p ro
ant i bodi es form i n
20%–25% of cas es .
TABLE 3-6 Clinical and Laboratory Findings in Hemophilia A and Hemophilia B L P e a v rt el ia of l F T a h Cl ct r in o o ic S r m al e V b Pi v II o ct e I pl u ri o a r ty r st e
Pa g e 5 2 4
ABC Ambe r CHM Conve rte r Tria l ve rsion, http://w w w .proce sste x t.com/a bcchm.html
I X ( U / in d Ti L m ) e S 0 V H e â er e v €“ y m er 2 pr ar e
ol t h o ro n si g s e a d n d s p o nt a n e o u s bl e e di n g
Pa g e 5 2 5
ABC Ambe r CHM Conve rte r Tria l ve rsion, http://w w w .proce sste x t.com/a bcchm.html
s e v er e a n d fr e q u e nt ; cr ip pl in g co m m o n M 2 Pr H o â ol e d €“ o m er 5 n ar at
g th
e
e ro d si s a n
Pa g e 5 2 6
ABC Ambe r CHM Conve rte r Tria l ve rsion, http://w w w .proce sste x t.com/a bcchm.html
d s p o nt a n e o u s bl e e di n g in fr e q u e nt ; di s a bi lit y u nc o m
Pa g e 5 2 7
ABC Ambe r CHM Conve rte r Tria l ve rsion, http://w w w .proce sste x t.com/a bcchm.html
m o n; s e v er e bl e e di n g fr o m in ju ri e s a n d s ur g er y Mi 5 V H l d â ar e €“ i a m 2 bl ar 5 e th
Pa g e 5 2 8
ABC Ambe r CHM Conve rte r Tria l ve rsion, http://w w w .proce sste x t.com/a bcchm.html
ro si s a n d s p o nt a n e o u s bl e e di n g v er y u n u s u al ; u n s
Pa g e 5 2 9
ABC Ambe r CHM Conve rte r Tria l ve rsion, http://w w w .proce sste x t.com/a bcchm.html
u s p ec te d a n d s e v er e bl e e di n g fr o m in ju ri e s a n d s ur g er
Pa g e 5 3 0
ABC Ambe r CHM Conve rte r Tria l ve rsion, http://w w w .proce sste x t.com/a bcchm.html
y S 2 U Bl u 5 s e bc â u e l i €“ al di ni 4 l y n ca 9 n g l
or af m te al r m aj or tr a u m a or s ur g er y p o ss ib le ; di a g n o
Pa g e 5 3 1
ABC Ambe r CHM Conve rte r Tria l ve rsion, http://w w w .proce sste x t.com/a bcchm.html
si s of te n m is s e d Adapt ed wi t h permi s s i on from Lee GR, et al . Wi nt robe 's Cl i ni c al Hemat ol ogy, 9t h ed. Phi l adel phi a: Lea and Febi ger, 1993:14 28.
(2) Clinical features vary wi t h t he degree of defi ci ency and are s ummari zed i n Tabl e 3-6. The nat ure of hemorrhage i s deep-t i s s ue bl eedi ng wi t h deep hemat omas , hemart hros i s , and s i gni fi cant bl eedi ng aft er s t res s s uch as t rauma and s urgery. Repeat ed hemart hros i s
Pa g e 5 3 2
ABC Ambe r CHM Conve rte r Tria l ve rsion, http://w w w .proce sste x t.com/a bcchm.html
res ul t s i n s evere di s abl i ng art hropat hy, whi ch i s t he cl i ni cal hal l mark of s evere hemophi l i a A.
(3) Diagnosis. The cons t el l at i on of s pont aneous or unexpect ed hemorrhage, es peci al l y hemart hros i s , i n a mal e pat i ent wi t h an appropri at e fami l y hi s t ory i s s ugges t i ve. Laborat ory t es t s reveal a prol onged partial thromboplastin time (PT T ), whi ch i s i ndi cat i ve of defi ci enci es i n t he i nt ri ns i c t hrombopl as t i n s ys t em; ot her coagul at i on t es t s are normal . The di agnos i s i s confi rmed by fact or as s ay demons t rat i ng l ow l evel s of VIII
p ro
ag
and normal l evel s of VIII .
Det ect i on of t he abnormal gene i t s el f i s now pos s i bl e and cl i ni cal l y avai l abl e.
(4) T herapy i s wi t h VIII
p ro
t rans fus i on i n t he
form of ei t her cryopreci pi t at e or fact or VIII concent rat es . The format i on of VIII
p ro
ant i bodi es i s a di ffi cul t compl i cat i on i n pat i ent s wi t h s evere hemophi l i a. Act i vat ed fact or VII (NovoSeven) can be us eful for acut e bl eedi ng epi s odes i n pat i ent s wi t h i nhi bi t ors t o fact or VIII. o
o
b. von Willebrand' s disease (vW D) i s a het erogeneous di s order t hat al s o i nvol ves t he fact or VIII mol ecul e.
(1) Pathophysiology. Genet i cal l y, i t s t rans mi s s i on i s vari abl e, but i n i t s mos t
Pa g e 5 3 3
ABC Ambe r CHM Conve rte r Tria l ve rsion, http://w w w .proce sste x t.com/a bcchm.html
common form, vW D i s t rans mi t t ed as an aut os omal domi nant t rai t wi t h vari abl e penet rance. Thi s di s order i nvol ves heredi t ary defi ci ency or derangement of VIII Becaus e VIII
ag
ag
and VIII
p ro
.
s erves as a cofact or for
pl at el et adhes i on, pat i ent s mani fes t bot h coagul opat hy and abnormal pl at el et funct i on.
(2) Clinical features i ncl ude i mmedi at e, mucocut aneous (i .e., pl at el et -t ype) bl eedi ng due t o VIII
ag
defi ci ency and del ayed,
deep-t i s s ue, pos t -t rauma (i .e., coagul at i on-t ype) bl eedi ng due t o VIII
p ro
defi ci ency. As wi t h hemophi l i a A, t he cl i ni cal pi ct ure for vW D vari es wi t h t he degree of defi ci ency.
(3) Diagnosis i s s ugges t ed by abnormal bl eedi ng of a mi xed nat ure i n an i ndi vi dual wi t h an appropri at e fami l y hi s t ory. Laborat ory t es t s s how prol onged bl eedi ng t i me due t o decreas ed pl at el et funct i on and a frequent l y prol onged PTT due t o VIII
p ro
defi ci ency.
Meas urement s of fact or VIII demons t rat e ei t her a combi ned and equi val ent defi ci ency of VIII
ag
and VIII
p ro
or a derangement i n mul t i mer ag
mi xt ures of VIII . Ri s t ocet i n-i nduced pl at el et aggregat i on i s abnormal i n vW D. Ot her t es t s of pl at el et aggregat i on are normal .
(4) T herapy requi res t he repl acement and coordi nat i on of bot h t he ant i geni c and procoagul ant port i ons of fact or VIII.
Pa g e 5 3 4
ABC Ambe r CHM Conve rte r Tria l ve rsion, http://w w w .proce sste x t.com/a bcchm.html
Concent rat es are ri ch i n VIII ag
p ro
but l ow i n VIII
; t herefore, ei t her cryopreci pi t at e or pl as ma,
whi ch cont ai n bot h, may be us ed i n vW D. A vari et y of concent rat es ri ch i n VIII
ag
and von
W i l l ebrand's fact or have been devel oped. They can be vi ral l y cl eans ed by pas t euri zat i on or det ergent t echni ques and have vol ume advant ages when a vW D pat i ent needs more i nt ens e or prol onged t herapy. In mi l d cas es , i ncreas ed s ecret i on of VIII can be i nduced by gi vi ng dDAVP. o
o
c. Other hereditary coagulation disorders are uncommon. Di agnos i s requi res fact or anal ys i s t o demons t rat e t he s peci fi c defi ci ency.
(1) Hemophilia B (factor IX deficiency) i s i dent i cal t o hemophi l i a A i n i t s genet i c feat ures and cl i ni cal mani fes t at i ons . Therapy di ffers i n t hat ei t her pl as ma or a puri fi ed prot hrombi n compl ex (whi ch cont ai ns concent rat ed fact ors II, VII, IX, and X) i s us ed as a s ource for fact or IX.
(2) Factor XI deficiency i s an aut os omal reces s i ve coagul opat hy wi t h mi l der cl i ni cal mani fes t at i ons t han t hos e of t he hemophi l i as . PTT i s prol onged due t o l ow fact or XI l evel s . Pl as ma s erves as adequat e repl acement t herapy.
(3) Factor XII, prekallikrein, and high–molecular-weight kininogen
Pa g e 5 3 5
ABC Ambe r CHM Conve rte r Tria l ve rsion, http://w w w .proce sste x t.com/a bcchm.html
deficiencies are uni que i n t hat t hey caus e s i gni fi cant prol ongat i on of PTT yet no predi s pos i t i on t o hemorrhage. Di agnos i s i s s ugges t ed by abnormal PTT wi t h no hi s t ory of bl eedi ng, even wi t h t rauma. Fact or anal ys i s i s neces s ary t o demons t rat e t he defi ci ent fact or.
(4) Deficiencies of all other factors have been des cri bed but are rare.
P.114
2. The acquired coagulation disorders are more compl ex t han t he heredi t ary forms . The acqui red coagul opat hi es us ual l y i nvol ve mul t i pl e and mi xed fact or defi ci enci es and oft en are compl i cat i ons of ot her di s eas es . Coagul at i on t es t i ng s hows abnormal i t i es i n mul t i pl e pat hways ; oft en t he bl eedi ng s everi t y correl at es poorl y wi t h coagul at i on abnormal i t i es s een i n l aborat ory t es t i ng. o
o
a. Vitamin K–dependent factor deficiency
(1) Etiology
(a) Malabsorption of vi t ami n K can occur wi t h bi l i ary obs t ruct i on as wel l as wi t h i nt es t i nal di s eas e (e.g., s prue). Agai n, coagul opat hy merel y compl i cat es t he obvi ous cl i ni cal pi ct ure.
Pa g e 5 3 6
ABC Ambe r CHM Conve rte r Tria l ve rsion, http://w w w .proce sste x t.com/a bcchm.html
(b) Part i cul arl y i n i nt ens i ve care uni t s , nutritional deficiency occurs i n pat i ent s wi t h poor oral i nt ake of vi t ami n K and i n t hos e i n whom ant i bi ot i cs have removed t he gas t roi nt es t i nal fl ora t hat s erve as an al t ernat e s ource of vi t ami n K.
(c) Drugs can i nt erfere wi t h vi t ami n K met abol i s m, mos t s peci fi cal l y, t he vi t ami n K ant agoni s t warfarin, whi ch i s us ed t o t reat t hrombot i c di s eas es .
(2) Pathophysiology. The l i ver s ynt hes i zes fact ors II, VII, IX, and X. The fi nal pos t t rans l at i onal s t ep i n t hei r s ynt hes i s renders t hes e prot ei ns funct i onal and requi res γ-carboxyl at i on of a mi ni mum of 10 t ermi nal gl ut ami c aci d res i dues , wi t h vi t ami n K as a cofact or. Int erference i n t hi s mechani s m caus es funct i onal defi ci ency of t hes e cl ot t i ng prot ei ns , part i cul arl y i mpai ri ng Ca
2+
bi ndi ng of
t he prot ei ns at t he cl ot s i t e.
(3) Diagnosis. Becaus e many cl ot t i ng fact ors are defi ci ent , l aborat ory t es t i ng s hows prolonged PT T and prothrombin time (PT ). Speci fi c meas urement s demons t rat e t he defi ci ency of fact ors II, VII, IX, and X.
(4) T herapy for vi t ami n K–rel at ed coagul opat hy vari es wi t h i t s caus e.
Pa g e 5 3 7
ABC Ambe r CHM Conve rte r Tria l ve rsion, http://w w w .proce sste x t.com/a bcchm.html
(a) In cas es of mal abs orpt i on and nut ri t i onal defi ci ency, s uppl ement al (oft en parent eral ) vi t ami n K correct s t he coagul opat hy.
(b) In cas es of exces s warfari n, wi t hdrawal of t he offendi ng drug wi t h s uppl ement al vi t ami n K i s effi caci ous .
(c) In pat i ent s wi t h normal ga s t roi nt es t i nal t ract funct i on, oral admi ni s t rat i on i s more effect i ve (s peed and ext ent of recovery) t han s ubcut aneous admi ni s t rat i on. Furt hermore, s mal l dos es i n t he 1–3-mg range can be us ed.
o
o
b. Disseminated intravascular coagulation (DIC)
(1) Etiology. DIC i s a common acqui red coagul opat hy t hat occurs s econdary t o ot her di s eas e proces s es s uch as t he fol l owi ng:
(a) Act i vat i on of t he i nt ri ns i c coagul at i on pat hway by endot hel i al damage (e.g., i n gram-negat i ve s eps i s , meni ngococcemi a, and vi remi a)
(b) Act i vat i on of t he ext ri ns i c pat hway by abnormal ent ry of t i s s ue
Pa g e 5 3 8
ABC Ambe r CHM Conve rte r Tria l ve rsion, http://w w w .proce sste x t.com/a bcchm.html
t hrombopl as t i ns i nt o t he ci rcul at i on (e.g., i n obs t et ri c compl i cat i ons , carci nomat os i s , and mas s i ve t rauma)
(2) Pathophysiology. DIC i s i ni t i at ed by s t i mul i i n t he s ys t emi c ci rcul at i on t hat act i vat e t he coagul at i on mechani s m and caus e t he abnormal format i on of exces s i ve s ys t emi c t hrombi n. The t hrombi n, i n t urn, caus es ext ens i ve act i vat i on of coagul at i on i n t he mi croci rcul at i on, whi ch cons umes many coagul at i on moi et i es and act i vat es t he fi bri nol yt i c s ys t em s econdari l y.
(3) Clinical features vary dependi ng on t he bal ance bet ween i nt ravas cul ar coagul at i on and fi bri nol ys i s and fact or depl et i on.
(a) In fl ori d acut e cas es (e.g., amni ot i c fl ui d embol i s m), t he coagul opat hy i s domi nant , and t he major s ympt oms are bl eedi ng and s hock.
(b) In more chroni c cas es (e.g., carci nomat os i s ), t hrombos i s and cl ot t i ng may predomi nat e.
(c) Many cas es of DIC i nvol ve abnormal coagul at i on paramet ers but no bl eedi ng or cl ot t i ng, whereas ot her cas es have a mi xt ure of bot h bl eedi ng and cl ot t i ng compl i cat i ons .
Pa g e 5 3 9
ABC Ambe r CHM Conve rte r Tria l ve rsion, http://w w w .proce sste x t.com/a bcchm.html
(4) Diagnosis. Laborat ory t es t s reveal a compl i cat ed pi ct ure. Many coagul at i on fact ors are cons umed i n t he di ffus e cl ot t i ng proces s ; t hes e fact or defi ci enci es prol ong bot h PT and PTT. Pl at el et cons umpt i on res ul t s i n t hrombocyt openi a. Fi bri nogen defi ci ency ari s es from t hrombi n-medi at ed cl ot t i ng as wel l as pl as mi n-medi at ed fi bri nol ys i s . The s econdary fi bri nol ys i s i s demons t rat ed by t he pres ence of hi gh t i t ers of fi bri n degradat i on product s (FDPs ), i ncreas i ngl y bei ng meas ured by D-di mer as s ays . P.115
(5) T herapy i s di rect ed at addres s i ng t he underl yi ng “t ri gger.― Repl acement of fi bri nogen (wi t h cryopreci pi t at e) i s general l y res erved for fi bri nogen l evel s l es s t han 100 mg/dL. Repl acement of ot her defi ci enci es wi t h pl as ma i s res erved for acut e, cl i ni cal l y s i gni fi cant bl eedi ng. Hepari n i s res erved for pat i ent s wi t h t hrombot i c compl i cat i ons (e.g., s ki n i nfarct i on, acral gangrene, recogni zabl e ves s el t hrombos es ). Many pat i ent s requi re no s peci fi c coagul at i on t herapy.
o
o
c. Liver disease res ul t s i n a compl ex coagul opat hy i nvol vi ng many as pect s of cl ot t i ng.
(1) Li ver di s eas e res ul t s i n i mpai red s ynt hes i s of vi t ami n
Pa g e 5 4 0
ABC Ambe r CHM Conve rte r Tria l ve rsion, http://w w w .proce sste x t.com/a bcchm.html
K–dependent cl ot t i ng prot ei ns , fi bri nogen, ant i t hrombi n III, pl as mi nogen, and ot her prot ei n moi et i es .
(a) St udi es have s hown t hat t here i s a hi erarchy of coagul at i on funct i on defi ci t s . Prognos t i c dat a al s o rel at e t o s peci fi c defect s i n coagul at i on funct i on encount ered i n l i ver di s eas e.
(i) The earl i es t , mos t s ens i t i ve, mos t revers i bl e defect i s t he l oweri ng of vi t ami n K–dependent fact ors s uch as fact or VII. As hepat ocel l ul ar funct i on det eri orat es furt her, fi bri nogen l evel s become quant i t at i vel y l ower and at t i mes even qual i t at i vel y abnormal (dys fi bri nogenemi a).
(ii) The mos t s evere s i t uat i on, whi ch correl at es wel l wi t h poor regenerat i ve capaci t y and overal l mort al i t y, i s t he l oweri ng of fact or V t o l evel s l es s t han 50%. Many l i ver t rans pl ant at i on cent ers s t rongl y emphas i ze fact or V l evel s i n t rans pl ant at i on deci s i ons .
(b) Thes e s t udi es can be us ed t o rat e s everi t y of di s eas e, prognos i s , and urgency for l i ver t rans pl ant at i on.
o
Pa g e 5 4 1
ABC Ambe r CHM Conve rte r Tria l ve rsion, http://w w w .proce sste x t.com/a bcchm.html
o
(2) Impai red cl earance of FDPs and act i vat ed coagul at i on fact ors may res ul t i n a mi l d DIC-t ype condi t i on.
o
o
(3) Port al hypert ens i on may res ul t i n s pl enomegal y and exces s i ve pl at el et pool i ng wi t h t hrombocyt openi a.
o
o
(4) The accumul at i on of FDPs caus es i mpai red pl at el et funct i on (t hrombocyt opat hi a).
o
o
(5) T herapy. Sus t ai ni ng coagul at i on funct i on and cont rol l i ng hemorrhage i n pat i ent s wi t h advanced l i ver di s eas e i s one of t he mos t di ffi cul t s i t uat i ons i n coagul at i on medi ci ne. The number of defi ci ent prot ei ns and t he s hort hal f-l i ves of s ome (e.g., fact or VII, 4–6 hours ) render mos t concent rat es non-effect i ve. Fres h frozen pl as ma (FFP) has been us ed t radi t i onal l y, but , becaus e of vol ume compl i cat i ons and s hort T½, t he res ul t s are poor. Genet i cal l y engi neered act i vat ed fact or VII can be us ed for acut e bl eedi ng but i t s us e i s l i mi t ed by i t s s hort hal f-l i fe.
d. Pathologic inhibitors of coagulation o
o
(1) The lupus-type inhibitor i s t he mos t common coagul at i on i nhi bi t or. Al t hough fi rs t des cri bed i n pat i ent s wi t h SLE, l upus i nhi bi t ors mos t oft en occur wi t h ot her condi t i ons or i di opat hi cal l y. It i s charact eri zed by a prol onged PTT and t endency for
Pa g e 5 4 2
ABC Ambe r CHM Conve rte r Tria l ve rsion, http://w w w .proce sste x t.com/a bcchm.html
t hrombos i s . Such pat i ent s oft en requi re ant i coagul at i on chroni cal l y, al t hough t he preci s e durat i on and i nt ens i t y of ant i coagul at i on i n t he l upus i nhi bi t or hypercoagul abi l i t y s t at e remai n s omewhat cont rovers i al . o
o
(2) Specific inhibitors of coagul at i on are ant i bodi es wi t h s peci fi ci t y for s i ngl e coagul at i on prot ei ns . The mos t common i nhi bi t or i s fact or VIII ant i body, whi ch ari s es i n 20% of pat i ent s wi t h hemophi l i a who have recei ved fact or t herapy. The i di opat hi c vari et y occurs mos t oft en i n pat i ent s ol der t han 65 years and pres ent s as an expl os i ve coagul opat hy i n a previ ous l y normal pers on.
(a) Cl i ni cal l y, s uch ant i bodi es caus e profound bl eedi ng dys cras i as of s everi t y s i mi l ar t o t hat of congeni t al defi ci ency.
(b) Di agnos i s i s made by demons t rat i ng a s peci fi c fact or defi ci ency t hat i s not correct ed by admi ni s t rat i on of normal pl as ma (abnormal mi xi ng s t udy).
(c) Therapy i s di ffi cul t becaus e t he ant i body al s o i nact i vat es exogenous l y admi ni s t ered fact ors . St eroi ds and i mmunos uppres s i ve agent s have been us ed wi t h l i mi t ed s ucces s . The new act i vat ed fact or VII preparat i ons have been us ed wi t h s ucces s .
e. Other acquired coagulation disorders. Coagul opat hy
Pa g e 5 4 3
ABC Ambe r CHM Conve rte r Tria l ve rsion, http://w w w .proce sste x t.com/a bcchm.html
has been as s oci at ed wi t h amyl oi dos i s (fact or X defi ci ency), nephrot i c s yndrome (due t o renal was t i ng of coagul at i on prot ei ns , es peci al l y fact or IX), ext racorporeal ci rcul at i on (t hought t o act i vat e t he coagul at i on s ys t em part i al l y and caus e l ow-grade DIC), and mas s i ve t rans fus i ons (t he pat i ent hemorrhages normal bl ood, but i t i s repl aced wi t h bl ood bank–deri ved bl ood t hat i s poor i n coagul at i on fact ors and pl at el et s ).
VI. Abnormalities Related to Hypercoagulability and Increased Risk of Thrombosis A. Anatomic abnormalities of vessels and blood flow Di s orders t hat predi s pos e t he pat i ent t o t hrombos i s rel at ed t o anat omi c abnormal i t i es of ves s el s and bl ood fl ow, not t o i nt ri ns i c abnormal i t i es i n t he bl ood i t s el f, i ncl ude t he fol l owi ng:
1. Stasis can occur i n condi t i ons s uch as pos t operat i ve s t at es , ort hopedi c i njury, and neurol ogi c di s eas es . Pat hogenet i c fact ors i ncl ude venodi l at i on and l ocal i zed areas of reduced cl earance of act i vat ed procoagul ant fact ors . P.116
2. T urbulence (e.g., as s oci at ed wi t h aneurys ms ) can damage vas cul ar endot hel i um and expos e bl ood t o t he i nt eri or of t he ves s el wal l , faci l i t at i ng endot hel i al –pl at el et –coagul at i on prot ei n i nt eract i on.
Pa g e 5 4 4
ABC Ambe r CHM Conve rte r Tria l ve rsion, http://w w w .proce sste x t.com/a bcchm.html
3. T rauma t o bl ood ves s el s can damage and di s rupt ves s el wal l s . Tumors can di rect l y i nvade bl ood ves s el s or i ndi rect l y render t he endot hel i um procoagul ant . The vas cul i t i c s yndromes are anot her exampl e.
B. Intrinsic blood disorders Di s orders caus i ng abnormal i t i es of pl at el et s can al s o predi s pos e t o t hrombos i s .
1. Myeloproliferative diseases s uch as pol ycyt hemi a vera and es s ent i al (hemorrhagi c) t hrombocyt hemi a, 6
es peci al l y when t he pl at el et count exceeds 10 /mm
3
2. Paroxysmal nocturnal hemoglobinuria (PNH), whi ch i s as s oci at ed wi t h enhanced pl at el et react i vi t y and unus ual t hrombos es i nvol vi ng abdomi nal hepat i c vei ns
C. Specific hereditary disorders of enhanced thrombosis associated with abnormalities of plasma proteins
1. Abnormal fibrinogens (dys fi bri nogens ) t hat are t oo s ens i t i ve t o t hrombi n or t hat cl ot t oo t i ght l y and do not al l ow for phys i ol ogi c l ys i s by pl as mi n and abnormal plasminogens t hat are unabl e t o di s s ol ve cl ot s phys i ol ogi cal l y are rare.
2. Deficiencies of coagulation inhibitors o
Pa g e 5 4 5
ABC Ambe r CHM Conve rte r Tria l ve rsion, http://w w w .proce sste x t.com/a bcchm.html o
a. Antithrombin III deficiency can be quant i t at i ve or qual i t at i ve. The defect has vari abl e phenot ypi c penet rance but can caus e recurrent venous t hrombos i s i n young pat i ent s .
o
o
b. Protein C deficiency. Prot ei n C i s a vi t ami n K–dependent fact or s ynt hes i zed by t he l i ver, whi ch i nhi bi t s fact ors VIIIa and Va. The homoz ygot i c condi t i on caus es purpura ful mi nans i n neonat es . The het eroz ygot i c condi t i on res ul t s i n enhanced venous t hrombos i s and a pos s i bl e predi s pos i t i on t o s ki n necros i s when pl aced on warfari n wi t hout concomi t ant hepari n.
o
o
c. Activated protein C resistance [Factor V (Leiden)] i s a genet i c defect i n fact or V, whi ch renders i t refract ory t o t he i nhi bi t ory act i vi t y of prot ei n C. Thi s condi t i on i s now known t o be t he mos t common congeni t al hypercoagul abl e s t at e. Some s t udi es report a 20% i nci dence i n hypercoagul abl e s t at e work-ups . Dat a now demons t rat e t hat t hi s l es i on al one frequent l y i s not s uffi ci ent t o caus e t hrombos i s , and approxi mat el y 50% of known het eroz ygot es never experi ence t hrombos i s . The het eroz ygous condi t i on act s as a comorbi d ri s k fact or wi t h ot her event s s uch as chi l dbi rt h, l ong bone fract ure, oral cont racept i ve us e, and s urgery t o caus e t hrombos i s . Thi s “mul t i pl e hi t ― pat hophys i ol ogy appears t o be common i n mos t hypercoagul abl e s i t uat i ons .
o
o
d. Protein S deficiency. Prot ei n S i s a cofact or t hat
Pa g e 5 4 6
ABC Ambe r CHM Conve rte r Tria l ve rsion, http://w w w .proce sste x t.com/a bcchm.html
faci l i t at es fact ors VIIIa and Va i nhi bi t i on by prot ei n C. It s defi ci ency res ul t s i n a t hrombot i c di at hes i s s yndrome s i mi l ar t o prot ei n C defi ci ency. o
o
e. Abnormal prothrombin variant (G20210A) l evel s and el evat ed homocys t ei ne al s o i ncreas e t hrombos i s .
D. Specific acquired disorders of enhanced thrombosis associated with abnormalities of plasma proteins
1. Pregnancy and oral contraceptive agents are as s oci at ed wi t h s t at i s t i cal l y i ncreas ed i nci dence of s t roke, acut e myocardi al i nfarct i on (MI), and venous t hrombos i s . Pregnancy and oral cont racept i ves caus e el evat i on i n mos t procoagul ant prot ei ns and di mi nut i on of mos t fi bri nol yt i c and i nhi bi t or prot ei ns , t hus al t eri ng t he hemos t at i c bal ance i n favor of cl ot t i ng. In a s mal l but s i gni fi cant s ubpopul at i on, an addi t i onal l es i on (e.g., fact or V Lei den, l upus i nhi bi t or) i s found. o
o
a. T herapy. Hepari n t herapy i s us ed i n pregnancy. W arfari n mus t be avoi ded, becaus e i t cros s es t he pl acent a and i s t erat ogeni c. Low–mol ecul a r-wei ght hepari ns are qui t e effect i ve and are more conveni ent t o us e. The ant i coagul at i on peri od mus t ext end t hrough t o t he peri part um peri od, becaus e t he mont h aft er del i very may be t he peri od of hi ghes t ri s k.
Pa g e 5 4 7
ABC Ambe r CHM Conve rte r Tria l ve rsion, http://w w w .proce sste x t.com/a bcchm.html
2. Nephrotic syndrome i s as s oci at ed wi t h uri ne was t i ng of ant i t hrombi n III and, more i mport ant l y, hypoal bumi nemi a, whi ch may res ul t i n enhanced pl at el et aggregat i on. P.117
3. Lupus inhibitor (antiphospholipid antibody) syndrome. The pres ence of t he s o-cal l ed l upus i nhi bi t or i s as s oci at ed wi t h i ncreas ed t hrombos i s , es peci al l y i n pat i ent s wi t h SLE. Mos t cas es , however, are not as s oci at ed wi t h SLE or ot her underl yi ng condi t i ons . Lupus i nhi bi t or i s becomi ng one of t he more common di agnos abl e condi t i ons rel at ed t o hypercoagul abi l i t y, wi t h an i nci dence of t hrombot i c compl i cat i ons rangi ng from 9% t o 28% i n affect ed pat i ent s . o
o
a. Pathophysiology. The l upus i nhi bi t or, or ant i phos phol i pi d ant i bodi es , or bot h, i nt eract wi t h t he coagul at i on pat hway at t he X–V–Ca compl ex.
o
o
b. Clinical manifestations. Des pi t e prol onged l aborat ory cl ot t i ng t i mes , t hrombos i s (bot h art eri al and venous ), pl acent al dys funct i on, and recurrent mi s carri age are t he major s ympt oms and s i gns of t hi s di s order. Abnormal bl eedi ng does not occur.
o
o
c. Diagnosis. Laborat ory fi ndi ngs i ncl ude prol ongat i on of t he PT and more commonl y t he PTT t hat does not revers e when pat i ent pl as ma i s
Pa g e 5 4 8
ABC Ambe r CHM Conve rte r Tria l ve rsion, http://w w w .proce sste x t.com/a bcchm.html
mi xed wi t h normal pl as ma. Di agnos i s i s confi rmed by furt her demons t rat i ng t he i nhi bi t or react i on wi t hi n t he cl ot t i ng s ys t em us i ng t he t hrombopl as t i n t i s s ue i nhi bi t i on (TTI) t es t and di l ut e Rus s el l 's venom vi per as s ays . o
o
d. T herapy. Aggressive anticoagulation i ni t i al l y wi t h hepari n and fol l owed by warfari n i s i ndi cat ed i n cas es of l upus i nhi bi t or s yndrome accompani ed by cl i ni cal t hrombos i s . W arfari n ant i coagul at i on t o an i nt ernat i onal normal i zed rat i o (INR) of 2–3 has recent l y been s hown t o be adequat e.
4. Heparin-induced thrombocytopenia (HIT ) syndrome, i s as s oci at ed wi t h t hrombos i s of venous and art eri al s i t es i n pat i ent s us i ng hepari n. Thi s s yndrome has become more commonl y recogni zed. o
o
(a) Incidence i s 1%–5% of pat i ent s on hepari n t herapy. Mos t dat a s ugges t t hat t hi s i s more common i n s et t i ngs s uch as coronary art ery bypas s graft i ng (CABG) and ot her i nt ens e vas cul ar mani pul at i on s i t uat i ons .
o
o
(b) Pathogenesis i nvol ves hepari n-dependent ant i pl at el et IgG ant i body. The pl at el et undergoes an “act i vat i ng event ― (as wi t h vas cul ar s urgery), whi ch res ul t s i n expres s i on of PF4 on i t s membrane. The hepari n bi nds t o t he PF4, creat i ng a neoant i gen t o whi ch an IgG hepari n-dependent ant i body i s made. The ant i body act i vat es pl at el et s and caus es rel eas e of pl at el et mi cropart i cl es i nt o
Pa g e 5 4 9
ABC Ambe r CHM Conve rte r Tria l ve rsion, http://w w w .proce sste x t.com/a bcchm.html
t he ci rcul at i on, res ul t i ng i n dual hypercoagul abi l i t y, pl at el et hyperreact i vi t y, and pl as ma hypercoagul abi l i t y. o
o
(c) Diagnostic testing i s s ubopt i mal . Thus , HIT remai ns a cl i ni cal di agnos i s t hat mus t be managed when t here i s a s t rong cl i ni cal s us pi ci on. More s ens i t i ve and s peci fi c enzyme-l i nked i mmunos orbent as s ay (ELISA) t echni ques are avai l abl e but have a hi gh fal s e-pos i t i ve rat e. Serot oni n rel eas e as s ays are more s peci fi c but are not as readi l y avai l abl e. Thrombocyt openi a us ual l y precedes t hrombos i s , s o pl at el et count s s houl d be moni t ored i n hepari n pat i ent s and hepari n di s cont i nued i f t he pl at el et count s i gni fi cant l y decl i nes .
o
o
(d) T herapy i nvol ves t he us e of di rect t hrombi n i nhi bi t ors s uch as l epi rudi n and argat roban. Lepi rudi n, a recombi nant hi rudi n, i s excret ed by t he ki dney; ext reme caut i on i s requi red i n t he s et t i ng of el evat ed creat i ni ne and renal fai l ure. Argat roban, a s mal l pept i de t hat i nhi bi t s t he t hrombi n cat al yt i c s i t e, i s excret ed s ol el y by hepat i c met abol i s m; caut i on i s requi red i n t he s et t i ng of l i ver dys funct i on and el evat ed bi l i rubi n. As wi t h hi rudi n, t here i s no ant i dot e. Thes e agent s have markedl y i mproved t he management and prognos i s of HIT.
o
o
5. T hrombotic thrombocytopenic purpura (T T P)
TTP i s charact eri zed by a defi ci ency or i nhi bi t i on of t he von W i l l ebrand's fact or (vW F) cl eari ng prot eas e ADAMTS 13, l eadi ng t o
Pa g e 5 5 0
ABC Ambe r CHM Conve rte r Tria l ve rsion, http://w w w .proce sste x t.com/a bcchm.html
t he accumul at i on of l arge vW F mul t i mers i n ci rcul at i on. Thes e mul t i mers l ead t o abnormal pl at el et aggregat i on and depos i t i on wi t hi n vas cul ar beds . Al t hough al l organ s ys t ems may be i nvol ved, t he cl as s pent ad of TTP cons i s t s of t hrombocyt openi a, anemi a, ment al s t at us changes (s ympt oms may vary from confus i on t o s ei z ure), renal fai l ure, and fever. Al l fi ve abnormal i t i es are rarel y s een, but mi croangi opat hi c hemol yt i c anemi a wi t h s chi s t ocyt es on peri pheral s mear and t hrombocyt openi a are key t o maki ng t he di agnos i s
Onl i ne Fi gure 3-9.
o
Online FIGURE 3-9 Mi croangi opat hi c hemol yt i c anemi a. Not e t he red bl ood cel l fragment at i on, mi cros pherocyt es and profound t hrombocyt openi a. (From Tkachuk DC, Hi rs chmann JV. Wi nt robe's At l as of Cl i ni c al Hemat ol ogy. Bal t i more: Li ppi ncot t W i l l i ams & W i l ki ns , 2007:1–56C.)
b. Pl as mapheres i s t herapy adds t he mi s s i ng enzyme and al s o removes fact or VIII
ag
macroaggregat es . Pheres i s has res ul t ed i n a +
revers al i n mort al i t y from 80 % before pheres i s t o <20% now. Pl as ma i nfus i on i s l es s effect i ve t han pl as ma phores i s but can be hel pful i n management s i t uat i ons unt i l pl as mapheres i s can be i ni t i at ed. Laborat ory
Pa g e 5 5 1
ABC Ambe r CHM Conve rte r Tria l ve rsion, http://w w w .proce sste x t.com/a bcchm.html
meas urement s of pl at el et s and LDH are us ed t o as s es s res pons e t o t reat ment . P.118
c. T T P syndrome occurs i di opat hi cal l y but i s al s o s een as a compl i cat i on of advanced AIDS, drugs (e.g., ant i reject i on drugs , cycl os pori n), or i nfect i ons (e.g., t oxi geni c Es c heri c hi a c ol i O157).
o
o
6. Malignancy associated with chronic DIC (T rousseau' s syndrome) caus es enhanced t hrombos i s of many vari et i es . The mechani s m for enhanced t hrombos i s i s chroni c DIC i ni t i at ed by t umor cel l s rel eas i ng ei t her t hrombopl as t i c s ubs t ances , act i vat i ng fact or XII, or bot h.
a. Mi grat ory t hrombophl ebi t i s of s uperfi ci al vei ns i s uni forml y as s oci at ed wi t h chroni c DIC i n cl as s i c Trous s eau's s yndrome. Common neopl as ms are adenocarci nomas of l ung, pancreas , s t omach, and pros t at e. Thi s condi t i on requi res hepari n t herapy for cont rol .
b. Marant i c endocardi t i s i s anot her DIC vari ant ; i t res ul t s i n fi bri n veget at i on i n t he heart and recurrent embol i zat i on.
c. Rout i ne deep venous t hrombos i s and pul monary embol i s m are al s o s t at i s t i cal l y
Pa g e 5 5 2
ABC Ambe r CHM Conve rte r Tria l ve rsion, http://w w w .proce sste x t.com/a bcchm.html
as s oci at ed wi t h mal i gnancy. In one l arge s t udy, 7% of pat i ent s wi t h i di opat hi c di s eas e devel oped neopl as i a at 2 years compared wi t h 2.5% i n age-mat ched cont rol s .
E. Diagnostic and therapeutic approach to hypercoagulability
1. Diagnostic approach o
o
a. Cl i ni cal s et t i ngs i n whi ch i nves t i gat i on for a hypercoagul abl e s t at e i s i ndi cat ed i ncl ude:
(1) Fami l y hi s t ory of document ed t hromboembol i c event s
(2) Age younger t han 40 years at ons et of t hromboembol i c event s
(3) Recurrent t hromboembol i c event s
(4) Unus ual s i t es of t hrombos i s (e.g., mes ent eri c or cerebral vei ns )
(5) Res i s t ance t o s t andard t herapy for t hrombos i s
o
o
b. Tabl e 3-7 s ummari zes s everal caus es of hypercoagul abi l i t y.
Pa g e 5 5 3
ABC Ambe r CHM Conve rte r Tria l ve rsion, http://w w w .proce sste x t.com/a bcchm.html
2. T reatment depends on the underlying cause. o
o
a. In myel oprol i ferat i ve di s orders , t reat ment of t he underl yi ng di s eas e wi t h hydroxyurea and anagrel i de i s effect i ve.
o
o
b. Ves s el abnormal i t i es can be s urgi cal l y repai red i f found at angi ography.
o
o
c. Mos t congeni t al prot ei n defi ci ency s t at es , i ncl udi ng prot ei n C, are managed wi t h l i fel ong warfari n t herapy once a t hrombot i c event has occurred. No t reat ment i s gi ven t o as ympt omat i c carri ers , becaus e at l eas t 50% of t hes e i ndi vi dual s never devel op a t hrombos i s . Such P.119
i ndi vi dual s s houl d be gi ven aggres s i ve prophyl axi s , however, when i n hi gh-ri s k s i t uat i ons s uch as fract ure or s urgery.
TABLE 3-7 Hypercoagulable Syndromes
Pa g e 5 5 4
ABC Ambe r CHM Conve rte r Tria l ve rsion, http://w w w .proce sste x t.com/a bcchm.html
Inherited Forms Rel at i vel y Common Fact or V (Lei den) and prot ei n C res i s t ance Ant i t hrombi n III defi ci ency
Qua nt i t a t i ve
Qual i t at i ve
Prot ei n C defi ci ency Prot ei n S defi ci ency Prot hrombi n vari ant (G20210A) Hyperhomocys t ei ne mi a Acquired Forms Pregnancy (es peci al l y pos t part um peri od) Oral cont racept i ve and es t rogen us e Lupus ant i coagul ant s and ant i cardi ol i pi n ant i bodi es Mal i gnancy (chroni c DIC) Myel oprol i ferat i ve di s eas es (.pol ycyt hemi a vera,
Pa g e 5 5 5
ABC Ambe r CHM Conve rte r Tria l ve rsion, http://w w w .proce sste x t.com/a bcchm.html
es s ent i al t hrombocyt openi a) and PNH Nephrot i c s yndrome Hepari n-i nduced t hrombocyt openi a s yndrome Rare Abnormal fi bri nogens Pl as mi nogen defi ci ency
Qual i t at i ve
Qua nt i t a t i ve
Decreas ed rel eas e of pl as mi nogen act i vat or Increas ed l evel s of hi s t i di ne-ri ch gl ycoprot ei n DIC, di s s emi nat ed i nt ravas cul ar coagul at i on.
TABLE 3-8 Management of Hypercoagulable States According to Venous Thromboembolism (VTE) Risk Stratification
Pa g e 5 5 6
ABC Ambe r CHM Conve rte r Tria l ve rsion, http://w w w .proce sste x t.com/a bcchm.html
Moderate Risk
Carri ers of i nhe ri t ed t hro mbo phi l i as who have neve r expe ri enc ed a t hro mbo tic even t
Pat i ent s wi t h an acqu i red t hro mbo phi l i a who have
Pa g e 5 5 7
ABC Ambe r CHM Conve rte r Tria l ve rsion, http://w w w .proce sste x t.com/a bcchm.html
not expe ri enc ed a t hro mbo tic even t (e.g. , ant i phos phol i pi d ant i bodi es , mal i gnan cy)
Pat i ent s wi t h one t hro mbo tic even t as s o ci at ed wi t h
Pa g e 5 5 8
ABC Ambe r CHM Conve rte r Tria l ve rsion, http://w w w .proce sste x t.com/a bcchm.html
a prov ocat i ve reve rs i bl e com orbi d stim ul us (e.g. , l eg cas t i ng, hor mon al t her apy) Aggres s i ve VTE prophyl axi s when expos ed t o hi gh ri s k s i t uat i ons (e.g., pregnancy, prol onged i mmobi l i t y) High Risk
Two or mor e docu
Pa g e 5 5 9
ABC Ambe r CHM Conve rte r Tria l ve rsion, http://w w w .proce sste x t.com/a bcchm.html
men t ed VTE even ts
One VTE even t of l i fet hre at en i ng nat u re (e.g. , mas s i ve PE)
One docu men t ed VTE even t pl us
pres ence of mul t i pl e i nheri t ed defect
Pa g e 5 6 0
ABC Ambe r CHM Conve rte r Tria l ve rsion, http://w w w .proce sste x t.com/a bcchm.html
pres ence s peci fi cal l y of AT III defi ci ency , prot ei n C/S defi ci ency
pres ence of ant i phos p hol i pi d ant i body s yndrome or act i ve mal i gnanc y
i di opat hi c VTE event (?)
Indefi ni t e ant i coagul at i on us i ng an accept ed regi men PE, pul monary embol i s m. o
o
d. Lupus -i nhi bi t or pat i ent s who have mani fes t ed t hrombos i s are managed wi t h warfari n as l ong as t he i nhi bi t or i s pres ent . The amount of warfari n ant i coagul at i on cont i nues t o be cont rovers i al .
o
Pa g e 5 6 1
ABC Ambe r CHM Conve rte r Tria l ve rsion, http://w w w .proce sste x t.com/a bcchm.html o
e. Hypercoagul abi l i t y as s oci at ed wi t h cancer us ual l y requi res l ong-t erm s ubcut aneous hepari n t herapy, becaus e warfari n does not cont rol t he t hrombot i c di at hes i s i n t hes e cas es . W hen effect i ve t herapy for t he t umor i s avai l abl e, as i t i s for pros t at e cancer, i t t emporari l y res ol ves t he coagul opat hy.
o
o
f. Tabl e 3-8 s ummari zes t herapeut i c pri nci pl es for t he common hypercoagul abl e s t at es .
P.120
Study Questions/Answers and Explanations 1. An 83-year-old received a hip replacement in her left leg 2 months previously. At that time, a mild anemia of 10 g/dL was noted, and she had been started on iron therapy by her primary care physician. She has mild hypertension. T here was remote gastric surgery for unknown reasons 40 years ago. She denies symptoms of weakness or fatigue as well as shortness of breath with exertion. However, her ambulation has been somewhat limited by her hip surgery. Her appetite is “ good,― but her family reports her oral intake has been poor for some time. She denies all other symptoms. Physical examination reveals a thin-appearing female in no distress. T here is pallor of her conjunctival and oral mucous membranes. T he tongue is papillated except for the lateral edges, which are smooth. Heart examination reveals regular rhythm 96 beats/min with a Grade II systolic murmur along the left sternal border. Abdomen is soft without masses or hepatosplenomegaly. A well-healed midline laparotomy incision is present. Neurologic examination results were normal. Stool was Hemoccult negative. 3
Complete blood count revealed WBC of 5.8K/mm ; platelets
Pa g e 5 6 2
ABC Ambe r CHM Conve rte r Tria l ve rsion, http://w w w .proce sste x t.com/a bcchm.html 3
190K/mm . However, hemoglobin is 8.2 g/dL (hematocrit 25%) 3
with an MCV of 111 mm . Reticulocytes were 25,000/mm
3
(1.4%). An outpatient iron panel obtained after her hip surgery admission showed serum iron 80, transferrin 360, and ferritin of 50 µg/dL. Which of the following is the most appropriate management for this patient? A Increas e t he i ron dos age and add vi t ami n C t o her regi men. B Obt ai n vi t ami n B 1 2 and fol i c aci d l evel s and i ni t i at e t herapy wi t h bot h, pendi ng res ul t s . C Ini t i at e a cours e of parent eral i ron t herapy. D Obt ai n a bone marrow as pi rat e and i ni t i at e eryt hropoi et i n t herapy for myel odys pl as i as . Vi ew Ans wer 1. T he answer is B [I B 3 d]. The pat i ent 's fi ndi ngs s ugges t t he pres ence of at l eas t one of t he common nut ri t i onal anemi as —i ron defi ci ency, fol at e defi ci ency, or B 1 2 defi ci ency. Her di et s eems poor. In addi t i on, t here i s a hi s t ory of and evi dence for s ome t ype of a gas t ri c s urgery—ei t her t ot al or part i al res ect i on i n t he pas t . The mos t common anemi a aft er gas t ri c res ect i on i s i ron defi ci ency t hat res ul t s from removal of aci d-s ecret i ng cel l s , whi ch are needed t o reduce i ron t o i t s ferrous form, t he abs orbabl e form of i ron. In addi t i on, l os s of abs orpt i ve s urface and t he pres ence of s t omal eros i ons and bl eedi ng al s o cont ri but e t o t hi s pat hophys i ol ogy. However, t hi s pat i ent 's s erum ferri t i n does fal l wi t hi n t he normal range, whi ch s t rongl y s ugges t s t hat t he l ack of i ron i s not what i s i nt erferi ng wi t h bl ood format i on i n t hi s pat i ent at t hi s t i me. Furt hermore, t he pat i ent i s di s pl ayi ng macrocyt i c i ndi ces wi t h an 3
MCV of 111 mm , whi ch agai n al mos t as s uredl y means t hat i ron-defi ci ent eryt hropoi es i s charact eri zed by defi ci ent hemogl obi n s ynt hes i s and mi crocyt os i s are not t he dri vi ng force of t he anemi a i n t hi s s i t uat i on. Thus , ans wer A, i ncreas i ng i ron dos age and addi ng as corbi c aci d t o faci l i t at e abs orpt i on, al t hough bas ed on s ound phys i ol ogi c pri nci pl es i n t hi s i ns t ance, mi ght not be expect ed t o revers e t hi s anemi a. Ans wer C i s anot her i ron maneuver t hat can s omet i mes be requi red i n t rue mal abs orpt i ve s i t uat i ons . The newer
Pa g e 5 6 3
ABC Ambe r CHM Conve rte r Tria l ve rsion, http://w w w .proce sste x t.com/a bcchm.html
parent eral forms are l es s al l ergeni c t han t radi t i onal preparat i ons and have made t hi s t herapy s afer. However, t hi s s t i l l requi res a t i me-cons umi ng i nfus i on, and al l avenues of oral i ron t herapy s houl d be empl oyed before res ort i ng t o t hi s . In t hi s cas e, dat a do not poi nt t o i ron defi ci ency as t he pri mary l es i on here. Myel odys pl as i a (MDS) has become t he mos t common caus e for anemi a i n t he geri at ri c popul at i on and i s an i mport ant cons i derat i on i n t hi s s et t i ng. Typi cal l y, pat i ent s wi t h MDS mani fes t a s l i ght macrocyt os i s but onl y rarel y t o t he degree s een here. MDS can be cl as s i fi ed by s t udyi ng bone marrow fi ndi ngs , and i n approxi mat el y 25% of cas es t he anemi a res ponds t o pharmacol ogi c dos es of EPO. However, before t he di agnos i s of MDS can be made, revers i bl e caus es of macrocyt i c anemi a s uch as B 1 2 and fol i c defi ci ency mus t be excl uded, s o ans wer D i s not appropri at e for t hi s pat i ent . The appropri at e management for t hi s pat i ent i s ans wer B, obt ai ni ng B 1 2 and fol i c aci d l evel s and i ni t i at i ng t herapy pendi ng res ul t s . Her s i gni fi cant macrocyt os i s and s ubt l e phys i cal fi ndi ngs of t ongue at rophy s ugges t B 1 2 defi ci ency, i ncl udi ng t rue t ype A aut oi mmune gas t ri t i s i nvol vi ng t he fundus and body of t he s t omach, whi ch i s t he cl as s i cal Addi s oni an perni ci ous anemi a. Thi s pat i ent had s ome form of gas t ri c s urgery and a hi s t ory of PUD. Thus , s he may have B 1 2 defi ci ency as s oci at ed wi t h t ot al gas t ri c res ect i on, whi ch, al t hough l es s common t han i ron defi ci ency, affect s pat i ent s havi ng had t hat operat i on about a decade l at er, or t ype B H. pyl ori –as s oci at ed at rophi c gas t ri t i s (whi ch affect s fundus , body, and ant rum) rel at ed t o her PUD. A vari et y of excel l ent bi ochemi cal t es t s can det ermi ne whet her t here i s B 1 2 defi ci ency (s erum B 1 2 l evel by radi oi mmune as s ay, met hyl mal oni c aci d l evel s ) and whet her t here i s t rue aut oi mmune perni ci ous anemi a and i t s caus es (t he pres ence of ci rcul at i ng i nt ri ns i c fact or aut oant i bodi es ). Becaus e t here i s a pos s i bi l i t y of fol at e defi ci ency, a fol i c aci d l evel s houl d be obt ai ned. Becaus e ongoi ng B 1 2 defi ci ency–rel at ed neurol ogi c morbi di t y can progres s and become i rrevers i bl e, t herapy can and s houl d be i ni t i at ed wi t h parent eral B 1 2 whi l e wai t i ng for t he bi ochemi cal confi rmat i on. Thus , ans wer B i s t he mos t appropri at e
Pa g e 5 6 4
ABC Ambe r CHM Conve rte r Tria l ve rsion, http://w w w .proce sste x t.com/a bcchm.html
cours e of management here. 2. A 61-year-old woman is admitted for urgent cardiac catheterization. T he patient had experienced ischemic cardiac pain at an outside hospital and was treated in standard fashion including IV heparin, but she continued to decompensate. She was transferred for cardiac catheterization, which demonstrated triple-vessel coronary artery disease. She received full-dose intravenous heparin as well as nitrates for 3 days then was taken to the OR for CABG. T he patient had nonremarkable preoperative blood counts. On the third postoperative day, she was found to have right lower extremity venous thrombosis and was restarted on IV heparin with rapid attainment of therapeutic PT T . T he next day, the right foot became cool, cyanotic, and painful. Doppler study demonstrated extensive DVT in right proximal venous system and arterial occlusion in the right iliac artery. Blood studies showed hemoglobin 12 g/dL, WBC 21K, and platelets 33K. Blood smear was nonremarkable save for decreased platelets. Which of the following is the most likely diagnosis for these events? A Inadequat e hepari n dos age wi t h recurrent t hrombos i s B Thrombot i c t hrombocyt openi c purpura (TTP) C Hepari n-i nduced t hrombocyt openi a s yndrome (HIT) D Cardi ac embol i s m E Homocys t i nemi a Vi ew Ans wer 2. T he answer is C [VID 4]. Thi s expl os i ve s i t uat i on i s t ypi cal of t he hepari n-i nduced t hrombocyt openi a (HIT) wi t h t hrombos i s s yndrome. Hepari n i s a wi del y us ed ant i coagul ant acros s many i ndi cat i ons . In approxi mat el y 3% of cas es , pat i ent s devel op a cl i ni cal l y and bi ol ogi cal l y act i ve ant i body t o a hepari n–PF4 neo-ant i gen on t he pl at el et , whi ch i n t he furt her pres ence of hepari n caus es pl at el et act i vat i on, aggregat i on, t hrombocyt openi a, and rel eas e of t i s s ue fact or. If t he hepari n i s s t i l l cont i nued, paradoxi cal t hrombos i s as s oci at ed wi t h mas s i ve pl at el et aggregat es (“whi t e cl ot ―) may ens ue. The t i me cours e i s
Pa g e 5 6 5
ABC Ambe r CHM Conve rte r Tria l ve rsion, http://w w w .proce sste x t.com/a bcchm.html
us ual l y wi t h 5–10 days of hepari n expos ure, al t hough i n pat i ent s wi t h pri or expos ure t o hepari n, HIT may occur much fas t er. There i s a hi erarchy of i nci dence ri s k wi t h ful l -dos e i nt ravenous us e caus i ng more HIT t han s ubcut aneous and ot her l es s i nt ens e regi mens . However, al l forms and rout es can el i ci t t he react i on. In s t udi es , l ow-mol ecul ar-wei ght hepari n (LMW H) forms l es s commonl y i nci t e HIT. However, LMW H wi l l cros s react and t ri gger HIT i n s ens i t i zed pat i ent s . W hen HIT proceeds i nt o t hrombos i s , bot h art eri al and venous t hrombos i s can occur and can be l i mb- and l i fe-t hreat eni ng i n up t o 50% of cas es . W hen HIT i s s us pect ed, al l hepari ns by al l rout es (i ncl udi ng di al ys i s ) mus t be di s cont i nued. Al t ernat i ve ant i coagul ant s (hi rudi n, Argat roban) are now avai l abl e for pat i ent s who requi re ongoi ng ant i coagul at i on. HIT i s frequent l y a cl i ni cal di agnos i s i ni t i al l y—t he devel opment of t hrombocyt openi a wi t hout ot her obvi ous caus e i n a pat i ent on hepari n. There are confi rmat ory l aborat ory t es t s s uch as t he ELISA HIT ant i body as s ay. Thrombos i s can propagat e or recur i n hepari ni zed pat i ent s (ans wer A). However, t he devel opment of a s i gni fi cant new t hrombocyt openi a i n as s oci at i on wi t h t he cl ot s woul d not be expect ed and i s more t ypi cal of HIT. TTP (ans wer B) does mani fes t t he combi nat i on of t hrombos i s and t hrombocyt openi a. However, t he t hrombot i c as pect s of TTP are i n t he mi croci rcul at i on rat her t han i n l arge art eri es or vei ns ; and a cardi nal fi ndi ng i s mi croangi opat hi c hemol ys i s wi t h RBC fragment at i on and s chi s t ocyt es , whi ch were abs ent here. Cardi ac embol i s m (ans wer D) woul d not be expect ed t o i nvol ve t he venous s ys t em. Homocys t i nemi a (ans wer E) i s a hypercoagul abl e s yndrome t hat i ndeed can i nvol ve bot h art eri al and venous ci rcul at i ons wi t h premat ure and recurrent t hrombot i c event s . Thi s pat i ent 's pres ent at i on s houl d el i ci t a homocys t ei ne meas urement . However, t hrombocyt openi a woul d not be expect ed as part of homocys t i nemi a. 3. A 20-year-old African American man is admitted for management of sickle cell anemia. He started to experience pain in his joints and lower back, consistent with his usual sickle cell pain pattern, about 48 hours before admission. He has Percocet at home for such episodes, but has not received much relief
Pa g e 5 6 6
ABC Ambe r CHM Conve rte r Tria l ve rsion, http://w w w .proce sste x t.com/a bcchm.html
from them. He does well with his sickle cell anemia, requiring hospital admission about 2–3 times per year for pain management. He holds a full-time job in retail and is engaged to be married. His CBC in the receiving ward showed hemoglobin 3
7.9 g/dL and WBC 24,000/mm , which compare to known baseline levels of 7.8 and 14,000, respectively. Chest film was scheduled for first thing in the morning. You examine him on morning rounds and find him to be in mild respiratory distress. He has developed moderate pleuritic pain. Vital signs are temperature 38.6°C; blood pressure 110/70 mm Hg; pulse 110 beats/min and respirations 24 breaths/min. He has mild scleral icterus. Chest examination shows a few rales in both bases without signs of consolidation. Morning laboratory findings demonstrate a hemoglobin of 7.2 3
g/dL with a reticulocyte count of 230,000/mm and a WBC of 3
33,000/mm with a left shift. Bilirubin is 4 mg/dL, but other liver functions P.121 are normal. T he chest film shows small, bilateral infiltrates in the bases. An O 2 saturation on room air is 90%. What is the most appropriate definitive management for this patient? A Arrange for an exchange t rans fus i on. B Perform an ul t ras ound and CT s can of t he abdomen t o excl ude as cendi ng chol angi t i s . C Ini t i at e hepari n for acut e pul monary embol i s m. D Ini t i at e ant i bi ot i cs wi t h s peci fi c pneumococcus and s al monel l a coverage. E Ini t i at e hydroxyurea 1000 mg/day Vi ew Ans wer 3. T he answer is A [I C 2 b (2) (b) (vi )]. Thi s pat i ent i s mani fes t i ng cl ear fi ndi ngs cons i s t ent wi t h t he acut e ches t s yndrome, t he l eadi ng caus e of deat h and s econd mos t common caus e for hos pi t al i zat i on i n pat i ent s wi t h s i ckl e cel l anemi a. Typi cal fi ndi ngs demons t rat ed here i ncl ude fever, s hort nes s of breat h,
Pa g e 5 6 7
ABC Ambe r CHM Conve rte r Tria l ve rsion, http://w w w .proce sste x t.com/a bcchm.html
t achypnea, s t ri ki ng l eukocyt os i s , ches t i nfi l t rat es , and hypoxemi a. Ini t i at i ng event s can be bot h i nfect i ous or noni nfect i ous (e.g., bone marrow embol i s m from vas ooccl us i ve event s ), but once es t abl i s hed, t he pat hophys i ol ogy s eems s i mi l ar t o acut e res pi rat ory di s t res s s yndrome (ARDS). The s yndrome can res ul t i n res pi rat ory fai l ure wi t h requi rement for vent i l at i on and mort al i t y i n as many as 9% of cas es . The acut e ches t s yndrome (ACS) i s one of t he accept ed cri t eri a for exchange t rans fus i on of red cel l s , and t hat i s t he mos t appropri at e choi ce here. The procedure i s us ual l y eas i l y performed i n t he ICU s et t i ng and has excel l ent res ul t s , wi t h an 81% recovery even aft er mechani cal vent i l at i on, whi ch i s far s uperi or t o t hat of ot her pat i ent s wi t h ARDS. Si mpl e t rans fus i on may be adequat e i n mi l d cas es , but exchange i s preferred i n rapi dl y det eri orat i ng cas es as s een here. Ot her accept ed i ndi cat i ons for exchange t rans fus i on i n s i ckl e cel l pat i ent s i ncl ude neurol ogi c compl i cat i ons s uch as s t roke, pri api s m, and t he l es s common hepat i c s eques t rat i on cri s i s , whi ch caus es exqui s i t el y t ender and mas s i ve hepat omegal y and ext reme el evat i ons of bi l i rubi n. The pat i ent has had chol ecys t ect omy, whi ch i s us ual for an adul t s i ckl e cel l pat i ent . Thes e pat i ent s have l i fel ong hemol ys i s and uni vers al l y form bi l i rubi n gal l s t ones . The pri or chol ecys t ect omy, abs ence of abdomi nal s ympt oms , normal l i ver funct i on s t udi es , and onl y mi ni mal l y el evat ed bi l i rubi n are s t rong negat i ves for t he di agnos i s of as cendi ng chol angi t i s . The bi l i rubi n el evat i on i s qui t e cons i s t ent wi t h t he hemol yt i c anemi a pres ent . Thus , ans wer B woul d not be t he i ni t i al di agnos t i c or t herapeut i c pat hway of choi ce. Al t hough newer s t udi es have i ndi cat ed t hat ACS can be caus ed frequent l y by bone marrow fat embol i from i nfarct ed marrow, whi ch occurs duri ng an i ni t i al pai nful cri s i s , t he pat hophys i ol ogy of ACS i s qui t e di fferent from t hat of t radi t i onal t hromboembol i s m. The embol i are s mal l fat embol i , not cl ot s , and i ni t i at e ARDS vi a cyt oki ne and s urfact ant mechani s ms . In t he abs ence of DVT, whi ch was not s us pect ed epi demi ol ogi cal l y or cl i ni cal l y here, hepari n (choi ce C) woul d not be t he preferred t herapy. Pat i ent s wi t h homozygous s i ckl e cel l anemi a have been s hown t o
Pa g e 5 6 8
ABC Ambe r CHM Conve rte r Tria l ve rsion, http://w w w .proce sste x t.com/a bcchm.html
be predi s pos ed t o s eri ous i nfect i ons from encaps ul at ed organi s ms , part i cul arl y t he Pneumoc oc c us and Sal monel l a s peci es . Thes e pat i ent s have i ncreas ed i nci dence and s everi t y from s uch i nfect i ons , whi ch s houl d be i n t he di fferent i al of a febri l e s i ckl e cel l pat i ent . However, t hi s pat i ent al s o di s pl ays t he ent i re s pect rum of ACS fi ndi ngs , s o i s pas t t he poi nt of us i ng ant i bi ot i cs al one for t herapy. He requi res aggres s i ve pul monary meas ures (oxygen, bronchodi l at ors , et c.) and a t rans fus i on maneuver i n addi t i on t o any ant i bi ot i cs . Furt hermore, recent s t udi es have expanded t he uni vers e of i nfect i ous agent s s een wi t h ACS and s ugges t t he addi t i on of a macrol i de i n s uch cas es . Ans wer D i s i nadequat e on many count s as t herapy i n t hi s s i t uat i on. Hydroxyurea has been us ed c hroni c al l y i n s el ect ed homoz ygous s i ckl e cel l anemi a pat i ent s . Earl i er t heori es s ugges t ed t hat i t s abi l i t y t o rai s e HbF l evel s and reduce s i ckl i ng were t he bas i s for i t s us e. More recent dat a s ugges t t hat i t s rol e i s much more compl ex and i nvol ves l oweri ng W BC (an i ndependent mort al i t y predi ct or i n s i ckl e cel l pat i ent s ) and modul at i ng cyt oki ne l evel s i n pat i ent s , whi ch may be as i mport ant as or more s o t han HbF l evel s . Nonet hel es s , hydroxyurea requi res weeks t o have effect and t hus i s not an appropri at e acut e t herapy. 4. A 19-year-old man experiences moderate trauma while playing in an intramural football game. T hat evening he experiences worsening pain and swelling in the left thigh. On evaluation, he manifests marked swelling in the left thigh, hemoglobin of 9 g/dL, and PT T 56 seconds. MRI shows a large hematoma. T he most likely diagnosis is which of the following? A Uremi a B Hemophi l i a A C Therapy wi t h as pi ri n D Fact or XII defi ci ency E Idi opat hi c t hrombocyt openi c purpura (ITP) Vi ew Ans wer 4. T he answer is B [V D 1 a]. Hemophi l i a A i nvol ves coagul at i on fact or VIII
p ro
; pl at el et funct i on i s not affect ed i n t hi s di s eas e. Thi s
Pa g e 5 6 9
ABC Ambe r CHM Conve rte r Tria l ve rsion, http://w w w .proce sste x t.com/a bcchm.html
res ul t s i n a coagul at i on-t ype bl eedi ng s yndrome wi t h del ayed, deep t i s s ue– t ype bl eedi ng. Al t hough t he exact mechani s m i s uncl ear, uremi a i s bel i eved t o caus e pl at el et dys funct i on by t oxi ns t hat al t er t he fact or VIII ant i gen pol ymers requi red for normal pl at el et funct i on. As pi ri n caus es t hrombocyt opat hi a by i nhi bi t i ng s ynt hes i s of t hromboxane A 2 (TXA 2 ), a s t rong i nducer of pl at el et aggregat i on. Fact or XII defi ci ency res ul t s i n prol ongat i on of t he PTT but i s not as s oci at ed wi t h abnormal bl eedi ng. 5. A 29-year-old woman presents with new purpura and petechiae on both lower legs. She has also experienced menorrhagia in her last two menstrual cycles. On examination in addition to the purpura and petechiae above, several petechiae are noted on her tongue and hard palate. Blood count reveals hemoglobin 12 g/dL WBC 8000/cc, and platelets 9000/cc. Which of the following is an expected part of her diagnostic evaluation or management? A Bone marrow megakaryocyt es are general l y decreas ed. B Pl at el et -as s oci at ed i mmunogl obul i n G (IgG) i s di agnos t i c. C Spl enomegal y and ot her cyt openi as are us ual l y pres ent . D The pl at el et l i fe s pan i s prol onged. E IVIg i s an effect i ve t herapy. Vi ew Ans wer 5. T he answer is E [V C 1 b (3) (v)]. Except when t hrombocyt openi a has caus ed chroni c bl eedi ng wi t h i ron defi ci ency anemi a, i s ol at ed t hrombocyt openi a i s t he rul e i n i di opat hi c t hrombocyt openi c purpura (ITP). Spl enomegal y and ot her cyt openi as are not part of ITP, and t hei r pres ence s houl d bri ng t he di agnos i s i nt o doubt . ITP i s t he prot ot ypi cal t hrombocyt openi a due t o i ncreas ed peri pheral des t ruct i on. Pl at el et l i fe s pan i s markedl y s hort ened, and, i n res pons e, t he bone marrow megakaryocyt es are i ncreas ed. Pl at el et -as s oci at ed ant i bodi es can us ual l y be found on pl at el et s and i n pl as ma, al t hough t hei r s i gni fi cance remai ns cont rovers i al . Long-t erm t herapy i nvol ves IVIg, part i cul arl y i n t he acut e s et t i ng, cort i cos t eroi ds , and s pl enect omy. Immunos uppres s i on may be requi red i n refract ory cas es .
Pa g e 5 7 0
ABC Ambe r CHM Conve rte r Tria l ve rsion, http://w w w .proce sste x t.com/a bcchm.html
6. A 64-year-ol d man t hat you have been fol l owi ng for hypert ens i on and di abet es pres ent s for a rout i ne fol l ow-up. He has no compl ai nt s as i de from occas i onal exert i onal dys pnea. On exami nat i on you not i ce t hat hi s conjunct i vae and nai l beds are pal e. Hi s CBC i s s i gni fi cant for hemogl obi n of 9 g/dL wi t h an MCV of 74 fL, hi s peri pheral bl ood s mear i s s hown
onl i ne. Hi s chemi s t ri es ,
i ncl udi ng l i ver funct i on t es t s and bi l i rubi n, are normal . In addi t i on t o i ron s t udi es , whi ch of t he fol l owi ng i s t he next bes t s t ep t o eval uat e t hi s pat i ent 's anemi a? A Serum B 1 2 and fol at e l evel s B Bone marrow exami nat i on C Col onos copy D LDH and hapt ogl obi n E Bone s can Vi ew Ans wer 6. T he answer is C [I A 1 a]. Thi s pat i ent has a mi crocyt i c anemi a, mos t commonl y caus ed by occul t bl ood l os s . Furt her eval uat i on wi t h s erum i ron, ferri t i n, and t ot al i ron bi ndi ng capaci t y can confi rm t he di agnos i s . Once t he di agnos i s of i ron defi ci ency anemi a i s made, i t i s i mperat i ve t o fi nd t he underl yi ng et i ol ogy of bl ood l os s . In t hi s s et t i ng, eval uat i on for a GI mal i gnancy s uch as col on cancer i s t he appropri at e next s t ep i n management . Col onos copy i s t he t es t of choi ce. B 1 2 and fol at e defi ci enci es pres ent as megal obl as t i c anemi a wi t h macrocyt os i s (MCV of 100 fL or above). LDH and hapt ogl obi n t es t i ng are us ed when a hemol yt i c anemi a i s s us pect ed. Thes e anemi as are not as common, us ual l y normocyt i c t o s l i ght l y macrocyt i c and are as s oci at ed wi t h an el evat ed ret i cul ocyt e count . Hemol yt i c anemi as do not have t he charact eri s t i c fi ndi ngs of t hi s pat i ent 's peri pheral bl ood s mear. (Image from From Tkachuk DC, Hi rs chmann JV. W i nt robe's At l as of Cl i ni cal Hemat ol ogy. Bal t i more: Li ppi ncot t W i l l i ams & W i l ki ns , 2007:1-5, t op.) 7. A 28-year-old white woman is in the 30th week of gestation and notes persistent swelling and discomfort in the right calf. She is previously healthy without prior major medical problems.
Pa g e 5 7 1
ABC Ambe r CHM Conve rte r Tria l ve rsion, http://w w w .proce sste x t.com/a bcchm.html
She has used carrier contraception before her pregnancy. T here is no family history of venous thromboembolism (VT E). A Doppler study confirms DVT of the right lower extremity. Which of the following is most likely in this patient? A Ant i t hrombi n III defi ci ency B Prot ei n C defi ci ency C Fact or V (Lei den) defi ci ency D Hemophi l i a A E Trous s eau's s yndrome Vi ew Ans wer 7. T he answer is C [VI E 2; Tabl e 3-9]. One of t he newer devel opment s i n hypercoagul abi l i t y has been t he fi ndi ng of a s i t uat i on i n whi ch t he defect for t he coagul at i on i nhi bi t or agoni s t prot ei n C i s l ocat ed i n t he recept or mol ecul e rat her t han i n t he amount of funct i on of prot ei n C i t s el f. The defect i ve fact or V mol ecul e, whi ch i s named for t he pl ace i t was di s covered—Lei den, Bel gi um, i s t he abnormal prot ei n t hat i s i ns ens i t i ve t o t he phys i ol ogi c i nhi bi t i on by prot ei n C. Mos t expert s agree t hat t hi s fact or V defect may be t he mos t common di agnos abl e i nborn defect s as s oci at ed wi t h a hypercoagul abl e s t at e; s ome s t udi es found an i nci dence as hi gh as 20% i n hypercoagul abi l i t y workups . A common s et t i ng for VTE as s oci at ed wi t h Lei den i s a comorbi d ri s k fact or s uch as pregnancy or us e of oral cont racept i ves . Prot ei n C and ant i t hrombi n III defi ci enci es are t radi t i onal defect s i n t he i nhi bi t ors t hems el ves , ei t her due t o qual i t at i ve defect s i n t he i nhi bi t or or quant i t at i ve defi ci ency of t he mol ecul es . They have been found t o have a hi gher VTE pot ent i al t han V Lei den and are l es s commonl y as s oci at ed wi t h a s econd ri s k fact or. Trous s eau's s yndrome i s anot her known hypercoagul abl e s t at e, but i t i s acqui red rat her t han genet i c. The condi t i on i s t he res ul t of a form of di s s emi nat ed i nt ravas cul ar coagul at i on (DIC) caus ed by carci noma, es peci al l y muci n-produci ng adenocarci noma. Hemophi l i a, al t hough heredi t ary and caus ed by a vari et y of mol ecul ar l es i ons , i s as s oci at ed wi t h profound bl eedi ng rat her t han cl ot t i ng.
Pa g e 5 7 2
ABC Ambe r CHM Conve rte r Tria l ve rsion, http://w w w .proce sste x t.com/a bcchm.html
P.122
8. A 59-year-ol d, as ympt omat i c, whi t e mal e wi t h chroni c renal i ns uffi ci ency, hypert ens i on, and di abet es i s s een i n fol l ow-up aft er a heart cat het eri zat i on. He has no compl ai nt s and feel s wel l . On phys i cal exam you not i ce bi l at eral nont ender pos t eri or cervi cal and axi l l ary l ymphadenopat hy, and s pl enomegal y. The res t of hi s exami nat i on i s normal . A CBC reveal s a hemogl obi n of 12.8 g/dL, a pl at el et count of 190,000 and a W BC count of 41,000/uL. Hi s peri pheral bl ood s mear i s s hown
onl i ne (l ow-power on l eft ;
hi gh-power on ri ght ). W hat i s t he mos t l i kel y di agnos i s ? A Chroni c myel ogenous l eukemi a B Chroni c l ymphocyt i c l eukemi a C Acut e myel ogenous l eukemi a D Myel odys pl as t i c s yndrome D Leukemoi d react i on Vi ew Ans wer 8. T he answer is B [III 2 b 1]. CLL i s t he mos t common l eukemi a affect i ng ol der adul t s . In t he majori t y of cas es pat i ent s are as ympt omat i c but may pres ent wi t h fat i gue and weaknes s . Typi cal phys i cal exam fi ndi ngs i ncl ude l ymphadenopat hy and s pl enomegal y. A peri pheral bl ood s mear s howi ng mat ure s mal l l ymphocyt es cons t i t ut i ng t he majori t y of whi t e bl ood cel l s s ugges t s t he di agnos i s , whi ch can be confi rmed by fl ow cyt omet ry. In AML, l eukemi c cel l s are i mmat ure myel oi d bl as t s . Peri pheral bl ood s mears i n CML reveal a s pect rum of cel l forms rangi ng from i mmat ure bl as t s t o mat ure pol ymorphonucl ear neut rophi l s . The myel odys pl as t i c s yndromes are charact eri zed by defect i ve hemat opoi es i s whi ch l eads t o cyt openi as , anemi a bei ng t he mos t common. Leukemoi d react i ons are charact eri zed by myel oi d cel l s , not l ymphocyt es . (Image from Tkachuk DC, Hi rs chmann JV. W i nt robe's At l as of Cl i ni cal Hemat ol ogy. Bal t i more: Li ppi ncot t W i l l i ams & W i l ki ns , 2007:5-5)
Pa g e 5 7 3
ABC Ambe r CHM Conve rte r Tria l ve rsion, http://w w w .proce sste x t.com/a bcchm.html
9. A 62-year-old man has had an elevated hematocrit for at least 3 years. His past medical history and review of systems are negative, except for mild, well-controlled hypertension. His latest complete blood count reveals: hemoglobin, 18 mg/dL; 3
hematocrit, 56%; and WBC count, 17,500 mm , with platelets, 3
800,000 mm . On further investigation, which of the following findings is the most typical and expected? A Ri nged s i derobl as t s on bone marrow exami nat i on B Art eri al bl ood oxygen s at urat i on l es s t han 88% C Pres ence of a Phi l adel phi a chromos ome on cyt ogenet i c t es t i ng D Very l ow t o abs ent eryt hropoi et i n t i t er E Many Pel ger-Huët cel l s on peri pheral bl ood s mear Vi ew Ans wer 9. T he answer is D [II A 1; C 1, 2]. The very elevat ed hemogl obi n and hemat ocri t , whi ch s t rongl y correl at e wi t h t rue el evat i ons i n red bl ood cel l (RBC) mas s rat her t han pl as ma cont ract i on, are cons i s t ent wi t h pol ycyt hemi a. Pol ycyt hemi a t hen i s broken down i nt o aut onomous or pri mary pol ycyt hemi a vera, a s t em cel l di s eas e, where t he marrow i s aut onomous l y creat i ng t oo many cel l s , vers us react i ve or s econdary forms , where t he marrow i s res pondi ng t o i ncreas ed eryt hropoi et i n from s ome al t erat i on of normal phys i ol ogy. The el evat i ons of t he ot her cel l l i nes s ugges t pol ycyt hemi a vera, becaus e t hi s s t em cel l di s eas e i nvol ves al l marrow cel l l i nes . Ri nged s i derobl as t s and Pel ger-Huët cel l s are s een i n myel odys pl as i a, whi ch i s anot her condi t i on res ul t i ng from abnormal marrow cl onal s t em cel l s . However, i n t hi s condi t i on, cyt openi as rat her t han i ncreas es i n count s are expect ed. On occas i on, chroni c myel ogenous l eukemi a (CML), anot her marrow s t em cel l cl onal prol i ferat i ve di s eas e, may mani fes t wi t h el evat ed count s —s peci fi cal l y t he whi t e bl ood cel l (W BC) count and t o a l es s er ext ent , pl at el et s . In CML, a hemogl obi n and hemat ocri t el evat ed t o t hi s degree woul d be unus ual and are much more t ypi cal of pol ycyt hemi a vera. Hi gh hemogl obi ns and hemat ocri t s from s econdary caus es s uch as t i s s ue hypoxi a (as wi t h cert ai n cardi ac and pul monary di s eas es )
Pa g e 5 7 4
ABC Ambe r CHM Conve rte r Tria l ve rsion, http://w w w .proce sste x t.com/a bcchm.html
res ul t from phys i ol ogi c i ncreas es i n eryt hropoi et i n s een i n t hes e s t at es or (as wi t h renal t umors ) s pect acul arl y, pat hol ogi cal l y el evat ed l evel s s een i n paraneopl as t i c cas es of hi gh hemat ocri t s . The l ow oxygen s at urat i on coul d caus e a s econdary el evat ed hemogl obi n, but not t he el evat i ons i n t he ot her cel l l i nes mani fes t ed by t hi s pat i ent . T he res pons e opt i ons for I t ems 11–14 are t he s ame. Y ou w i l l be requi red t o s el ec t one ans w er for eac h i t em i n t he s et . QUEST IONS 10–13 A Prot ei n C defi ci ency B Paroxys mal noct urnal hemogl obi nuri a (PNH) C Lupus i nhi bi t or s yndrome D Trous s eau's s yndrome Mat c h eac h of t he fol l ow i ng c l i ni c al vi gnet t es w i t h t he mos t l i kel y di agnos i s . 10. A 30-year-old woman with a history of rash and arthritis has deep vein thrombosis; in addition, she has had a problem with recurrent abortions. 11. A 45-year-old man, who has been followed by his physician for 4 years for chronic iron deficiency anemia with unknown source of blood loss, has portal vein thrombosis. 12. A 60-year-old man had deep vein thrombosis of his left leg last month and is being appropriately managed with warfarin. He has symptoms of new clot formation in the right leg as well as a clot in his left forearm. 13. A 67-year-old woman with mitral stenosis is started on warfarin by her cardiologist. On the third day, painful red areas appear on her thigh and breast. Vi ew Ans wer 10–13. T he answers are: 10-C [VI D 3], 11-B [VI B 2], 12-D [VI D 6], 13-A [VI C 2 b]. T he lupus inhibitor is found in one third of patients with systemic lupus erythematosus (SLE). In these patients, the association with thrombosis risk is 10%–15%. In patients without SLE who display a similar inhibitor-type antibody, there is less risk of thrombosis, but the
Pa g e 5 7 5
ABC Ambe r CHM Conve rte r Tria l ve rsion, http://w w w .proce sste x t.com/a bcchm.html
risk is still far in excess of normal. Paroxysmal nocturnal hemoglobinuria (PNH) is a difficult diagnosis to make, and the disorder can masquerade as chronic iron deficiency (due to urine losses), chronic hemolysis, or both. PNH is associated with increased thrombotic risk and has a tendency to involve intra-abdominal veins. Recurrent thrombosis associated with adequate warfarin anticoagulation, especially when migratory and involving superficial veins, suggests the presence of chronic disseminated intravascular coagulation (DIC) with an occult adenocarcinoma (T rousseau' s syndrome). Warfarin can, in the early phases of administration, lower the vitamin K–dependent, short half-life protein inhibitor protein C more rapidly and create an imbalance with coagulation factors II, VII, IX, and X, thus affecting the hemostatic balance toward clotting. Characteristically, it occurs soon after the initiation of warfarin without concomitant administration of heparin.
Pa g e 5 7 6
ABC Ambe r CHM Conve rte r Tria l ve rsion, http://w w w .proce sste x t.com/a bcchm.html
Editors: Wolfsthal, Susan T itle: NMS Medicine, 6th Edition Copyri ght ©2008 Li ppi ncot t W i l l i ams & W i l ki ns > T able of Cont ent s > Chapt er 4 - Onc ologic Diseases
Chapter 4
Oncologic Diseases Evonne Fontanilla Katherine T kaczuk
I. U.S. Cancer Statistic and Screening Guidelines (Fi gures 4-1 and 4-2; Tabl e 4-1).
II. Tumor Growth Uncontrolled growth di s t i ngui s hes mal i gnant from normal cel l s . Mal i gnant cel l s are charact eri zed by a l os s of res pons i venes s t o growt h-i nhi bi t ory s i gnal s . More rapi dl y growi ng t umors may be more poorl y di fferent i at ed, but t hey may be more s ens i t i ve t o cert ai n ki nds of chemot herapy t reat ment s .
A. Patterns of tumor growth Cancer growt h fol l ows t wo general pat t erns .
1. Direct invasion of t he t i s s ue i n whi ch t he cancer aros e
2. Metastasis (di s s emi nat i on) t o di s t ant organs vi a t he bl oods t ream, l ymphat i c s ys t em, or by i nt racavi t ary ext ens i on o
o
a. Di fferent t ypes of tumors vary in metastatic potential. For exampl e, pri mary brai n cancers t end t o remai n l ocal i zed and rarel y s pread t o di s t ant foci . However, many carci nomas s uch as s mal l cel l
Pa g e 5 7 7
ABC Ambe r CHM Conve rte r Tria l ve rsion, http://w w w .proce sste x t.com/a bcchm.html
l ung cancer (SCLC) are al ready wi del y di s s emi nat ed i n mos t pat i ent s at t he t i me of di agnos i s . o
o
b. Each cancer al s o exhi bi t s a di s t i nct pattern of spread di ct at ed by t he bi ol ogy of t he t umor cel l –hos t i nt eract i on. For exampl e, common s i t es of met as t as es i n col on cancer i ncl ude t he l i ver, l ungs , and peri t oneal cavi t y, and common s i t es i n SCLC i ncl ude t he l ungs , l i ver, bones , and neuraxi s .
B. Tumor growth rate and prognosis Growt h rat e and pat t erns of s pread are i mport ant det ermi nant s of prognos i s i n t he abs ence of t reat ment and i n t he res pons e t o part i cul ar t herapeut i c modal i t i es s uch as s urgery, chemot herapy, or radi at i on t herapy.
1. Rapidly growing neoplasms s uch as acut e l eukemi as , SCLC, and l ymphomas are general l y hi ghl y res pons i ve t o chemot herapy and radi at i on t herapy. However, t here are numerous except i ons .
2. Slow-growing tumors s uch as l ow-grade s arcomas are l es s res pons i ve t o s ys t emi c t reat ment modal i t i es . Surgi cal res ect i on and radi at i on t herapy are more effect i ve t reat ment opt i ons for s uch neopl as ms .
3. Growt h rat es can be es t i mat ed us i ng flow cytometry t o det ermi ne t he percent age of t umor nucl ei i n t he S phas e of t he cel l cycl e. Immunohi s t ochemi s t ry can be us ed t o det ermi ne Ki -67, anot her refl ect i on of cel l prol i ferat i on. S-phas e det ermi nat i on may hel p predi ct t he prognos i s and pot ent i al need for adjuvant chemot herapy i n earl y s t age breas t cancer.
Pa g e 5 7 8
ABC Ambe r CHM Conve rte r Tria l ve rsion, http://w w w .proce sste x t.com/a bcchm.html
C. Genetic changes and cancer
1. Cancers are genetic diseases characterized by mutations or overexpression of genes that regulate vital cell processes. Oncogenes (cancer-promot i ng genes ) i nt erfere wi t h t he regul at ory cont rol s of t he cel l . Mos t oncogenes are deri ved from proto-oncogenes, normal genes t hat ordi nari l y regul at e cel l growt h proces s es . P.128
FIGURE 4-1 2006 Estimated U.S. Cancer Cases (Repri nt ed wi t h t he permi s s i on of t he Ameri can Cancer Soci et y, Inc. Al l ri ght s res erved.)
Pa g e 5 7 9
ABC Ambe r CHM Conve rte r Tria l ve rsion, http://w w w .proce sste x t.com/a bcchm.html
FIGURE 4-2 2006 Estimated U.S. Cancer Deaths (Repri nt ed wi t h t he permi s s i on of t he Ameri can Cancer Soci et y, Inc. Al l ri ght s res erved.) P.129
TABLE 4.1 Screening Guidelines for the Early Detection of Cancer in Asymptomatic People
Pa g e 5 8 0
ABC Ambe r CHM Conve rte r Tria l ve rsion, http://w w w .proce sste x t.com/a bcchm.html
Sit Recomm e endation Bre o Y ear as t ly mam mog rams are reco mme nded s t art i ng at age 40. The age at whi c h s cre eni n g s hou ld be s t op ped s hou ld be i ndi v i dua
Pa g e 5 8 1
ABC Ambe r CHM Conve rte r Tria l ve rsion, http://w w w .proce sste x t.com/a bcchm.html
l i zed by cons i deri ng t he pot e nt i al ri s ks and bene fi t s of s cre eni n g in t he cont ext of over al l heal th s t at us and l ong evi t y. o
Cl i ni cal brea st
Pa g e 5 8 2
ABC Ambe r CHM Conve rte r Tria l ve rsion, http://w w w .proce sste x t.com/a bcchm.html
exa m s hou ld be part of a peri odi c heal th exa m abou t ever y 3 year s for wom en in t hei r 20s and 30s and ever y year for wom en 40
Pa g e 5 8 3
ABC Ambe r CHM Conve rte r Tria l ve rsion, http://w w w .proce sste x t.com/a bcchm.html
and ol de r. o
Wo men s hou ld kno w how t hei r brea sts nor mal l y feel and repo rt any brea st chan ge pro mpt l y to t hei r heal th care prov i der
Pa g e 5 8 4
ABC Ambe r CHM Conve rte r Tria l ve rsion, http://w w w .proce sste x t.com/a bcchm.html
s. Brea st s el fexa m is an opt i on for wom en s t art i ng in t hei r 20s . o
Wo men at i ncre as ed ri s k (e.g. , fami ly hi s t ory, gene tic t end ency ,
Pa g e 5 8 5
ABC Ambe r CHM Conve rte r Tria l ve rsion, http://w w w .proce sste x t.com/a bcchm.html
pas t brea st canc er) s hou ld t al k wi t h t hei r doct ors abou t t he bene fi t s and l i mi t at i o ns of s t art i ng mam mog raph y s cre eni n g earl i er, havi
Pa g e 5 8 6
ABC Ambe r CHM Conve rte r Tria l ve rsion, http://w w w .proce sste x t.com/a bcchm.html
ng addi t i on al t es t s (i .e. , brea st ul t ra s oun d and MRI) , or havi ng mor e freq uent exa ms . Col Begi nni ng on at age &
50, men
rec and t u women m
s houl d begi n s creeni ng wi t h 1 of t he
Pa g e 5 8 7
ABC Ambe r CHM Conve rte r Tria l ve rsion, http://w w w .proce sste x t.com/a bcchm.html
exami nat i on s chedul es bel ow: o
A fecal occu lt bl oo d t es t (FOB T) or fecal i mm unoc hemi cal t es t (FIT ) ever y year
o
A fl exi bl e s i gm oi do s cop y (SFI G) ever
Pa g e 5 8 8
ABC Ambe r CHM Conve rte r Tria l ve rsion, http://w w w .proce sste x t.com/a bcchm.html
y 5 year s o
Ann ual FOB T or FIT and fl exi bl e s i gm oi do s cop y ever y 5 year s*
o
A doub l e-c ont r as t bari um ene ma ever y 5 year s.
Pa g e 5 8 9
ABC Ambe r CHM Conve rte r Tria l ve rsion, http://w w w .proce sste x t.com/a bcchm.html
o
A col o nos c opy ever y 10 year s *Co mbi n ed t es t i ng is pref erre d over ei t h er annu al FOB T or FIT, or FSIG ever y 5 year s, al on e. Peop
Pa g e 5 9 0
ABC Ambe r CHM Conve rte r Tria l ve rsion, http://w w w .proce sste x t.com/a bcchm.html
le who are at mod erat e or hi gh ri s k for col or ect a l canc er s hou ld t al k wi t h a doct or abou t a di ffe rent t es t i ng s che dul e . Pro The PSA s t a t es t and t e t he
Pa g e 5 9 1
ABC Ambe r CHM Conve rte r Tria l ve rsion, http://w w w .proce sste x t.com/a bcchm.html
di gi t al rect al exami nat i on s houl d be offered annual l y, begi nni ng at age 50, t o men who have a l i fe expect anc y of at l eas t 10 years . Men at hi gh ri s k (Afri can Ameri can men and men wi t h a s t rong fami l y hi s t ory of 1 or more fi rs t -degr ee rel at i ves di agnos e d wi t h pros t at e cancer at an earl y
Pa g e 5 9 2
ABC Ambe r CHM Conve rte r Tria l ve rsion, http://w w w .proce sste x t.com/a bcchm.html
age) s houl d begi n t es t i ng at age 45. For bot h men at average ri s k and hi gh ri s k, i nformat i on s houl d be provi ded about what i s known and what is uncert ai n about t he benefi t s and l i mi t at i on s of earl y det ect i on and t reat ment of pros t at e cancer s o t hat t hey can make an i nformed
Pa g e 5 9 3
ABC Ambe r CHM Conve rte r Tria l ve rsion, http://w w w .proce sste x t.com/a bcchm.html
deci s i on about t es t i ng. Ut e Cervix: rus Screeni ng s houl d begi n approxi m at el y 3 years aft er a woman begi ns havi ng vagi nal i nt ercours e, but no l at er t han 21 years of age. Screeni ng s houl d be done every year wi t h regul ar Pap t es t s or every 2 years us i ng l i qui d-bas ed t es t s . At or aft er age 30,
Pa g e 5 9 4
ABC Ambe r CHM Conve rte r Tria l ve rsion, http://w w w .proce sste x t.com/a bcchm.html
women who have had 3 normal t es t res ul t s i n a row may get s creened every 2 to 3 years . Al t ernat i v el y, cervi cal cancer s creeni ng wi t h HPV DNA t es t i ng and convent i o nal or l i qui d-bas ed cyt ol ogy coul d be performed every 3 years . However, doct ors may s ugges t a woman
Pa g e 5 9 5
ABC Ambe r CHM Conve rte r Tria l ve rsion, http://w w w .proce sste x t.com/a bcchm.html
get s creened more oft en i s s he has cert ai n ri s k fact ors , s uch as HIV i nfect i on or a weak i mmune s ys t em. W omen aged 70 years and ol der who have had 3 or more cons ecut i ve normal Pap t es t s i n t he l as t 10 years may choos e t o s t op cervi cal cancer s creeni ng . Screeni ng aft er
Pa g e 5 9 6
ABC Ambe r CHM Conve rte r Tria l ve rsion, http://w w w .proce sste x t.com/a bcchm.html
t ot al hys t erect omy (wi t h removal of t he cervi x) i s not neces s ary unl es s t he s urgery was done as a t reat ment for cervi cal cancer. Endomet rium: The Ameri can Cancer Soci et y recommen ds t hat at t he t i me of menopaus e al l women s houl d be i nformed about t he ri s ks and s ympt oms of
Pa g e 5 9 7
ABC Ambe r CHM Conve rte r Tria l ve rsion, http://w w w .proce sste x t.com/a bcchm.html
endomet ri al cancer, and s t rongl y encourag ed t o report any unexpect ed bl eedi ng or s pot t i ng t o t hei r phys i ci an s . Annual s creeni ng for endomet ri al cancer wi t h endomet ri al bi ops y begi nni ng at age 35 s houl d be offered t o women wi t h or at ri s k for heredi t ar y nonpol yp os i s col on cancer
Pa g e 5 9 8
ABC Ambe r CHM Conve rte r Tria l ve rsion, http://w w w .proce sste x t.com/a bcchm.html
(HNPCC). Ca For nce i ndi vi dual r-r s el a undergoi n t ed g peri odi c che heal t h cku exami nat i p
ons , a cancer-rel at ed checkup s houl d i ncl ude heal t h couns el i n g and, dependi n g on a pers on's age and gender, mi ght i ncl ude exami nat i ons for cancers of t he t hyroi d, oral cavi t y, s ki n, l ymph nodes , t es t es ,
Pa g e 5 9 9
ABC Ambe r CHM Conve rte r Tria l ve rsion, http://w w w .proce sste x t.com/a bcchm.html
and ovari es , as wel l as for s ome nonmal i g nant di s eas es . *Repri nt ed from Ameri can Cancer Soci et y. Cancer Fact s and Fi gures 2007. At l ant a: Ameri can Cancer Soci et y, Inc. Ameri can Cancer Soci et y gui del i nes for earl y cancer det ect i on are as s es s ed annual l y i n order t o i dent i fy whet her t here i s new s ci ent i fi c evi dence
Pa g e 6 0 0
ABC Ambe r CHM Conve rte r Tria l ve rsion, http://w w w .proce sste x t.com/a bcchm.html
s uffi ci ent t o warrant a reeval uat i on of current recommendat i ons . If evi dence i s s uffi ci ent l y compel l i ng t o cons i der a change or cl ari fi cat i on i n a current gui del i ne or t he devel opment of a new gui del i ne, a formal procedure i s i ni t i at ed. Gui del i nes are formal l y eval uat ed every 5 years regardl es s of whet her new evi dence s ugges t s a change i n t he exi s t i ng recommendat i ons . There are 9 s t eps i n t hi s
Pa g e 6 0 1
ABC Ambe r CHM Conve rte r Tria l ve rsion, http://w w w .proce sste x t.com/a bcchm.html
procedure, and t hes e “gui del i ne s for gui del i ne devel opment ― were formal l y es t abl i s hed t o provi de a s peci fi c met hodol ogy for s ci ence and expert judgment t o form t he underpi nni ng s of s peci fi c s t at ement s and recommendat i ons from t he Soci et y. Thes e procedures cons t i t ut e of del i berat e proces s t o ens ure t hat al l Soci et y recommendat i ons have t he s ame met hodol ogi c al and
Pa g e 6 0 2
ABC Ambe r CHM Conve rte r Tria l ve rsion, http://w w w .proce sste x t.com/a bcchm.html
evi dence-bas ed proces s at t hei r core. Thi s proces s al s o empl oys a s ys t em for rat i ng s t rengt h and cons i s t ency of evi dence t hat i s s i mi l ar t o t hat empl oyed by t he Agency for Heal t h Care Res earch and Qual i t y (AHCRQ) and t he U.S. Prevent i ve Servi ces Tas k Force (USPSTF). P.130
o
o
a. Oncogenes modi fy t he affect ed cel l , res ul t i ng i n t he acqui s i t i on of mal i gnant charact eri s t i cs s uch as changes i n s hape, aut ocri ne growt h fact or s ecret i on, l os s of cont act i nhi bi t i on, and anchorage i ndependence.
Pa g e 6 0 3
ABC Ambe r CHM Conve rte r Tria l ve rsion, http://w w w .proce sste x t.com/a bcchm.html
Oncogenes may funct i on i n a number of ways :
(1) They may caus e t he t umor cel l t o s ecret e mi t ogeni c growt h fact ors t hat t hen s t i mul at e growt h of t he s ame cel l , l eadi ng t o aut os t i mul at i on of cel l growt h, or autocrine growth.
(2) They may affect growt h fact or–i nduced s i gnal t rans duct i on.
(a) Typi cal l y, cel l s urface recept ors engaged by t hei r res pect i ve l i gands t rans mi t s i gnal s t hrough a s eri es of defi ned i nt racel l ul ar pat hways t hat cul mi nat e i n t he regul at i on of DNA s ynt hes i s or t rans cri pt i on.
(b) Mut at i ons or al t ered gene expres s i on i n any of t hes e pat hways may di s t urb s uch regul at ed s i gnal i ng, l eadi ng t o a l os s of regul at ed cel l growt h or funct i on. The prot ei n product s of s ome oncogenes s uch as ras mut at e s o t hat t hey cons t i t ut i vel y promot e cel l growt h i n t he abs ence of growt h fact or i nt eract i ons wi t h t hei r recept ors .
o
o
b. Si ngl e oncogenes t ypi cal l y do not i nduce t he t rans format i on of ful l y normal cel l s i nt o ful l y mal i gnant cel l s . Each oncogene induces only a subset of changes as s oci at ed wi t h ful l mal i gnancy. A s eri es of genet i c
Pa g e 6 0 4
ABC Ambe r CHM Conve rte r Tria l ve rsion, http://w w w .proce sste x t.com/a bcchm.html
changes i s t ypi cal l y requi red. o
o
c. Activation of proto-oncogenes
(1) Point mutations. A poi nt mut at i on i s a s i ngl e-bas e change i n a gene. A s i ngl e mut at i on i n codon 12 of t he ras gene caus es act i vat i on of ras , t he mos t commonl y act i vat ed oncogene i n s ol i d mal i gnanci es .
(2) Gene fusion. In chroni c myel ogenous l eukemi a (CML), t he abl prot o-oncogene i s t ypi cal l y fus ed wi t h a ful l y unrel at ed gene, t he BCR gene. The prot ei n encoded by t he BCR-abl hybri d (p210) has a novel s t ruct ure and funct i on.
(3) Amplification. Di fferences i n t he l evel of expres s i on of encoded prot ei ns al s o can caus e prot o-oncogenes t o become oncogeni c. For exampl e, i n chi l dhood neurobl as t oma, t he amount of t he N-myc gene i s i ncreas ed from a di pl oi d number t o many dozens per cel l . Nmyc gene ampl i fi cat i on i s as s oci at ed wi t h a poor prognos i s and i s a s i gni fi cant fact or i n t he aggres s i venes s of t hi s t umor.
2. T umor suppressor genes funct i on i n t he normal cel l t o res t ri ct or repres s cel l ul ar prol i ferat i on. The i nact i vat i on of t umor s uppres s or genes can promot e t umori genes i s .
o
Pa g e 6 0 5
ABC Ambe r CHM Conve rte r Tria l ve rsion, http://w w w .proce sste x t.com/a bcchm.html
o
a. Inact i ve al l el es of t umor s uppres s or genes may be acqui red i n t wo ways :
(1) Somat i c mut at i on
(2) Incorporat i on i n t he germ l i ne t o creat e a congeni t al predi s pos i t i on t o cancer (e.g., as i n ret i nobl as t oma)
o
o
b. Inact i vat i on of t umor s uppres s or genes i s as s oci at ed wi t h neurobl as t oma, SCLC, ki dney cancer, col on carci noma, W i l ms ' t umor, bl adder cancer, os t eobl as t oma, and l i ver cancer, among ot her mal i gnanci es .
o
o
c. Speci fi c mut at i ons i n t he BRCA1 gene i n affect ed women i ncreas e t he ri s k of earl y ons et breas t cancer. Res earch has recent l y s hown t hat t he BRCA1 gene, a heri t abl e s omat i c gene preval ent i n As hkenazi Jews , i s normal l y i nvol ved i n es t rogen recept or (ER) funct i on.
III. Cancer Staging For ful l s t agi ng i nformat i on, pl eas e s ee t he mos t recent edi t i on of t he AJCC Cancer St agi ng Handbook. At pres s t i me, t he mos t recent edi t i on i s t he Ameri c an Joi nt Commi t t ee on Canc er: AJCC Canc er St agi ng Handbook, 6t h ed. New Y ork: Spri nger-Verl ag, 2002.
Convent i onal t umor–node–met as t as i s (TNM) s t agi ng s ys t ems defi ne a s eri es of cat egori es or s t ages , each of whi ch defi nes a prognos t i c s t age of a t umor.
A. Objectives of staging Pa g e 6 0 6
ABC Ambe r CHM Conve rte r Tria l ve rsion, http://w w w .proce sste x t.com/a bcchm.html
1. Estimating prognosis. Hi gher s t ages are as s oci at ed wi t h progres s i vel y poorer prognos es .
2. Determining the best course of therapy. Accurat e s t agi ng i s neces s ary t o det ermi ne t he appropri at e s equence of l ocal , regi onal , and s ys t emi c t reat ment approaches .
3. Facilitating investigation. St agi ng provi des a uni form cl as s i fi cat i on s ys t em t o devel op uni form t reat ment cri t eri a and t o eval uat e t he cl i ni cal t ri al s l i t erat ure.
B. Staging systems St agi ng s ys t ems for al l s ol i d t umors are bas ed pri nci pal l y on t he anatomic extent of disease. Di fferences i n t umor bi ol ogy precl ude a uni vers al s t agi ng s ys t em appl i cabl e t o al l cancers .
1. The T NM classification system i s t he mos t wi del y us ed s t agi ng s ys t em for carci nomas and s arcomas . Thi s s ys t em as s i gns t he t umor t o a T level, whi ch corres ponds t o t he s i ze and ext ent of t he pri mary t umor; an N level, whi ch des cri bes t he degree of regi onal nodal i nvol vement ; and an M level, whi ch des cri bes t he pres ence and ext ent of di s t ant met as t as es . Si t e-s peci fi c defi ni t i ons of TNM s t ages have been compi l ed for a wi de range of neopl as ms .
2. Ol der cl as s i fi cat i on s ys t ems s t i l l are frequent l y us ed t o s t age l ymphomas and carci nomas of t he col on, rect um, pros t at e, and t es t i cl e.
Pa g e 6 0 7
ABC Ambe r CHM Conve rte r Tria l ve rsion, http://w w w .proce sste x t.com/a bcchm.html
C. Staging procedures The AJCC has defi ned t he fol l owi ng t hree l evel s of s t agi ng i nformat i on.
1. Clinical staging us es al l avai l abl e pretreatment data , i ncl udi ng di agnos t i c dat a obt ai ned from phys i cal exami nat i on, radi ol ogi c or nucl ear i magi ng t echni ques , l aborat ory t es t s , and endos copy or ot her i nvas i ve procedures .
2. Pathologic staging i s bas ed on dat a obt ai ned duri ng s urgery and from pat hol ogi c exami nat i on of t he res ect ed s peci men.
3. Retreatment staging may be us ed t o res t age t he t umor before addi t i onal t herapy, aft er fai l ure of i ni t i al t reat ment .
IV. Treatment Modalities A. General principle
1. Cancer t reat ment may be ei t her curative or palliative i n i nt ent . o
o
a. Curative t reat ment s ai m t o permanent l y eradi cat e t he t umor.
o
o
b. Palliative t reat ment s s eek t o opt i mi ze qual i t y of l i fe and, when pos s i bl e, t o prol ong l i fe when cure cannot be achi eved.
Pa g e 6 0 8
ABC Ambe r CHM Conve rte r Tria l ve rsion, http://w w w .proce sste x t.com/a bcchm.html
P.131
2. Succes s ful t reat ment programs t ypi cal l y i ncl ude one or more of t he fol l owi ng component s : s urgery, radi at i on t herapy, chemot herapy, and ot her noncyt ot oxi c s ys t emi c t herapi es .
3. Therapy mus t be cus t omi zed t o meet t he s peci fi c needs of i ndi vi dual pat i ent s . In many di s eas e pres ent at i ons , a mul t i di s ci pl i nary approach t hat coordi nat es t he effort s of t he pat hol ogi s t , s urgi cal oncol ogi s t , medi cal oncol ogi s t , radi at i on oncol ogi s t , and ot her s peci al i s t s i s des i rabl e.
4. Clinical trials, whi ch are es s ent i al t o progres s i n cancer t herapy, are i nt egrat ed i nt o t he rout i ne care of many cancer pat i ent s . Tri al s can be conduct ed t o i dent i fy new t reat ment opt i ons i n pat i ent s for whom no effect i ve t herapy i s avai l abl e or t o furt her refi ne exi s t i ng effect i ve t reat ment s t rat egi es . The cl i ni cal t ri al proces s t ypi cal l y i nvol ves t hree di s t i nct t ypes of t ri al s . o
o
a. Phase I trials us e new agent s or new combi nat i ons of agent s t o i dent i fy t oxi ci t i es and maximally tolerated doses, t o es t abl i s h a worki ng dos e for furt her s t udi es , and t o det ermi ne t he di s pos i t i on (pharmacokinetics) and bi ol ogi c effect s as t hey rel at e t o di s pos i t i on (pharmacodynamics). W hen pos s i bl e, s t udi es s houl d provi de a proof-of-concept t hat t he agent has t he des i red effect s on i t s put at i ve t arget at
Pa g e 6 0 9
ABC Ambe r CHM Conve rte r Tria l ve rsion, http://w w w .proce sste x t.com/a bcchm.html
dos es t o be us ed i n phas e II s t udi es . For each drug t es t ed, phas e I t ri al s t ypi cal l y i nvol ve fewer t han 25 pat i ent s wi t h advanced di s eas e. o
o
b. Phase II trials are des i gned t o eval uat e t he effi cacy of new agent s or drug combi nat i ons i n a part i cul ar s t age of di s eas e. Us ual l y, phas e II t ri al s are conduct ed i n 30–50 pat i ent s , wi t h earl y s t oppi ng rul es i f t he new t reat ment proves i neffect i ve.
o
o
c. Phase III trials. If phas e II t ri al s yi el d encouragi ng res ul t s , randomi zed phase III trials are conduct ed t o compare t he new t reat ment wi t h t he exi s t i ng s t andard of care. Phas e III t ri al s t ypi cal l y requi re hundreds t o t hous ands of pat i ent s .
B. Radiation therapy Fi ft y percent of al l cancer pat i ent s requi re radi at i on t herapy at s ome poi nt duri ng t he cours e of t hei r di s eas e.
1. Applications o
o
a. Radi at i on t herapy, ei t her al one or i n conjunct i on wi t h chemot herapy and s urgery, i s curat i ve i n s ome cancers (e.g., Hodgki n's di s eas e, head and neck cancer, cervi cal cancer, and rect al cancer).
o
o
b. Radi at i on t herapy al s o has i mport ant pal l i at i ve appl i cat i ons (e.g., rel i ef from met as t at i c bone pai n i n advanced breas t cancer).
Pa g e 6 1 0
ABC Ambe r CHM Conve rte r Tria l ve rsion, http://w w w .proce sste x t.com/a bcchm.html
2. Uses. Radi at i on t herapy can be us ed al one or i n combi nat i on wi t h chemot herapy or s urgery. o
a. In combination with chemotherapy.
Chemot herapy may be
us ed concurrent l y wi t h or before radi at i on t o enhance effi cacy.
(1) For exampl e, t he pyri mi di ne anal og 5-fl uorouraci l (5-FU), t he al kyl at i ng agent ci s pl at i n, gemci t abi ne, a nucl eos i de anal og t hat pri mari l y ki l l s cel l s undergoi ng DNA s ynt hes i s (S-phas e) and bl ocks t he progres s i on of cel l s t hrough t he G1/S-phas e boundary, and t he mi crot ubul e s t abi l i zer pacl i t axel (Taxol ) al l act as radi at i on s ens i t i zers when admi ni s t ered concurrent l y wi t h radi at i on t herapy.
(2) However, combi ned-modal i t y t herapy al s o can cont ri but e t o s evere t oxi ci t y react i ons . The radiation recall effect, an enhanced or re-act i vat ed res pons e i n a previ ous l y i rradi at ed area preci pi t at ed by t he concurrent admi ni s t rat i on of doxorubi ci n or met hot rexat e, i s a cl as s i c exampl e.
o
o
b. Surgery. Radi at i on t herapy may be us ed before surgery t o i nduce t umor regres s i on and faci l i t at e t he s urgi cal procedure. Radi at i on al s o may be us ed postoperatively t o reduce t he ri s k of l ocal recurrence. In mos t cancers , t he opt i mal t i mi ng of adjunct i ve radi at i on t herapy (e.g., preoperat i ve vers us pos t operat i ve) has not been det ermi ned.
Pa g e 6 1 1
ABC Ambe r CHM Conve rte r Tria l ve rsion, http://w w w .proce sste x t.com/a bcchm.html
3. Side effects. Radi at i on t herapy i s as s oci at ed wi t h l ocal s i de effect s , es peci al l y i n pat i ent s who are recei vi ng curat i ve-i nt ent hi gh-dos e radi at i on (e.g., for t he t reat ment of head or neck cancer). o
o
a. T ime course
(1) Acute effects, pri nci pal l y i nfl ammat i on, may occur wi t hi n days or weeks of t reat ment .
(2) Chronic effects s uch as fi bros i s and s carri ng may not be apparent unt i l mont hs or even years aft er t herapy.
o
o
b. The severity of advers e react i ons i s a funct i on of t he t reat ment s i t e, t he s i ze of t he radi at i on port , t he amount of radi at i on energy, and dos age vari abl es (e.g., t ot al dos e, radi at i on dos e per fract i on, and dos e rat e). The t oxi ci t y of hi gh-dos e radi at i on t herapy may be reduced by t he us e of t ai l ored, conformal radi at i on fi el ds . Si de effect s can be mi ni mi zed by:
(1) Accurat el y t arget i ng t he t umor area by us i ng s ophi s t i cat ed radi ol ogi c t echni ques [e.g., comput ed t omography (CT), magnet i c res onance i magi ng (MRI)]
(2) Di rect i ng radi at i on t o avoi d cri t i cal organs
Pa g e 6 1 2
ABC Ambe r CHM Conve rte r Tria l ve rsion, http://w w w .proce sste x t.com/a bcchm.html
wi t h l ow t ol erance t o radi at i on (e.g., t he s pi nal cord)
(3) Bl ocki ng normal t i s s ue from radi at i on
(4) Reduci ng t he t reat ment fi el d over t he cours e of t herapy
o
o
c. Systemic effects of radi at i on t herapy i ncl ude mal ai s e, fat i gue, anorexi a, and depres s ed bl ood count . Thes e general i zed s ympt oms are es peci al l y common i n pat i ent s t reat ed concurrent l y wi t h radi at i on and chemot herapy.
o
o
d. Skin reactions occur aft er hi gh-dos e t herapy di rect ed t o s i t es on or near t he s ki n s urface (e.g., ches t wal l aft er mas t ect omy) or femal e reproduct i ve organs (e.g., vul va). Acute reactions cons i s t of eryt hema, dry des quamat i on wi t h pruri t us , and moi s t des quamat i on. React i ons are l es s common aft er hi gh-energy phot on beam t herapy. Ret reat ment or t he us e of overl appi ng fi el ds may res ul t i n s evere and pers i s t ent s ki n react i ons .
(1) The affect ed area s houl d be kept as cl ean and dry as pos s i bl e. Addi t i onal t reat ment meas ures i ncl ude:
(a) Topi cal appl i cat i on of vi t ami n A and D oi nt ment , moi s t uri zers , baby oi l , or
Pa g e 6 1 3
ABC Ambe r CHM Conve rte r Tria l ve rsion, http://w w w .proce sste x t.com/a bcchm.html
corns t arch
(b) Topi cal appl i cat i on of cort i cos t eroi ds
(2) Pat i ent s s houl d avoi d i rri t at i ng cl ot hi ng and di rect expos ure t o t he s un.
o
o
e. React i ons of t he oral cavity and pharynx s uch as mucos i t i s , pai n, anorexi a, xeros t omi a, and dent al cari es occur aft er hi gh-dos e radi at i on t o t he head or neck area. P.132
(1) St ri ct at t ent i on t o dent al hygi ene, t opi cal anes t het i cs , art i fi ci al s al i va preparat i ons , and nut ri t i onal couns el i ng may be hel pful .
(2) Severe cas es of naus ea or gas t roi nt es t i nal i nt ol erance may requi re i ns ert i on of a gas t ri c feedi ng t ube or percut aneous gas t ros t omy.
o
o
f. Gastrointestinal reactions occur wi t h cumul at i ve radi at i on dos es exceedi ng 4000–5500 cGy.
(1) Esophagitis us ual l y s ubs i des i n 7–10 days but predi s pos es pat i ent s t o
Pa g e 6 1 4
ABC Ambe r CHM Conve rte r Tria l ve rsion, http://w w w .proce sste x t.com/a bcchm.html
s uperi mpos ed Candi da i nfect i ons . Treat ment meas ures i ncl ude us e of ant aci ds , a bl and di et , and t opi cal anes t het i cs .
(2) Radiation gastritis or enteritis may mani fes t as naus ea, vomi t i ng, di arrhea, abdomi nal pai n, anorexi a, or bl eedi ng. Treat ment meas ures i ncl ude: ant i emet i cs ; ant i di arrheal s ; a bl and, l ow-fat , l ow-res i due, hi gh-gl ut en di et ; and di et ary s uppl ement s .
(3) Rectal inflammation may res ul t i n bl eedi ng or pai n. A l ow-res i due di et , s t ool s oft eners , or s t eroi d enemas may gi ve rel i ef.
o
o
g. Radiation pneumonitis, charact eri zed by cough, dys pnea, and ches t pai n, us ual l y occurs 2–3 mont hs aft er del i very of radi at i on t o a s i gni fi cant l ung vol ume. Cort i cos t eroi ds are us ual l y effect i ve. St eroi ds s houl d be di s cont i nued gradual l y t o avoi d recurrence of s ympt oms .
o
o
h. Central nervous system (CNS) symptoms may occur duri ng t herapy or may not appear unt i l weeks or mont hs l at er.
(1) Acute symptoms accompanyi ng i nt racrani al i rradi at i on i ncl ude headache and s i gns of i ncreas ed i nt racrani al pres s ure (ICP); gas t roi nt es t i nal s ympt oms (i .e., naus ea and vomi t i ng) al s o may occur. Dexamet has one us ual l y provi des rapi d rel i ef.
Pa g e 6 1 5
ABC Ambe r CHM Conve rte r Tria l ve rsion, http://w w w .proce sste x t.com/a bcchm.html
(2) Delayed symptoms i ncl ude s hort -t erm memory l os s , vi s ual memory di s t urbances , and whi t e mat t er abnormal i t i es s uch as cal ci fi cat i ons and vent ri cul ar di l at i on. Thes e t oxi ci t i es are part i cul arl y common i n pat i ent s undergoi ng prophyl act i c crani al radi at i on for l i mi t ed-s t age SCLC.
(3) A somnolence syndrome charact eri zed by hypers omni a and fat i gue has been obs erved s everal weeks or mont hs aft er i nt racrani al i rradi at i on, es peci al l y i n pat i ent s who are recei vi ng concurrent i nt rat hecal chemot herapy.
o
o
i. Bone marrow suppression may fol l ow t he l arge-fi el d i rradi at i on us ed i n t he t reat ment of Hodgki n's di s eas e or pel vi c mal i gnanci es , es peci al l y i f pat i ent s are recei vi ng concurrent chemot herapy. Leukopeni a or t hrombocyt openi a may requi re s us pens i on of t herapy; however, anemi a i s rare. Hemogl obi n val ues bel ow 9 g/dL may requi re t rans fus i on.
o
o
j. In Hodgki n's di s eas e, medi as t i nal i rradi at i on i s as s oci at ed wi t h l ong-t erm t oxi ci t i es s uch as hypot hyroi di s m and coronary art ery di s eas e.
4. Newer T echniques: Int ens i t y modul at ed radi at i on t herapy (IMRT) and s t ereot act i c radi at i on t herapy are newer modal i t i es t hat mi ni mi ze damage t o normal t i s s ue whi l e del i veri ng hi gher dos es of radi at i on t o t he t arget
Pa g e 6 1 6
ABC Ambe r CHM Conve rte r Tria l ve rsion, http://w w w .proce sste x t.com/a bcchm.html
organs .
C. Chemotherapy
1. General comments o
o
a. Combi nat i ons of chemot herapy agent s us ed i n conjunct i on wi t h ot her cyt oreduct i ve t reat ment modal i t i es may i ncreas e cure rat es i n pat i ent s wi t h col orect al neopl as ms , es ophageal cancer, and breas t cancer.
o
o
b. Chemot herapy pl ays an i mport ant rol e i n t he pal l i at i ve t reat ment of pat i ent s wi t h advanced or met as t at i c cancers . In t hi s s et t i ng, t he s equent i al us e of modes t l y act i ve s i ngl e agent s may be preferabl e t o t he us e of more act i ve, but more t oxi c, combi nat i ons .
o
o
c. Recent i mprovement s i n drug devel opment and t he s creeni ng of new agent s have l ed t o t he i nt roduct i on of numerous new agent s , offeri ng pat i ent s more t reat ment opt i ons . Thes e advances have been coupl ed wi t h an i ncreas ed unders t andi ng of t umor bi ol ogy.
2. Uses. Chemot herapy s houl d be us ed onl y when pat i ent s benefi t from t he t reat ment . W hen cure i s t he object i ve, compl et e res pons es are general l y requi red, and cons i derabl e hos t t oxi ci t y i s accept abl e [e.g., t he
Pa g e 6 1 7
ABC Ambe r CHM Conve rte r Tria l ve rsion, http://w w w .proce sste x t.com/a bcchm.html
us e of bone marrow t rans pl ant at i on (BMT) for pat i ent s wi t h acut e l eukemi as ]. However, when pal l i at i on i s t he goal , accept abl e t oxi ci t y mus t be coupl ed wi t h an abs ence of di s eas e progres s i on. In general , i ndi vi dual s who exhi bi t an object i ve (e.g., compl et e or part i al ) res pons e t o chemot herapy l i ve bet t er and l onger t han do t hos e wi t h s t abl e or progres s i ve di s eas e. Meas urabl e di s eas e i s defi ned as t he pres ence of at l eas t one meas urabl e l es i on. If t he meas urabl e di s eas e i s res t ri ct ed t o a s ol i t ary l es i on, i t s neopl as t i c nat ure s houl d be confi rmed by cyt ol ogy/hi s t ol ogy. Meas urabl e l es i ons are defi ned as l es i ons t hat can be accurat el y meas ured i n at l eas t one di mens i on wi t h great es t di amet er ≥20 mm us i ng convent i onal t echni ques or ≥10 mm wi t h s pi ral CT s can. Nonmeas urabl e l es i ons are defi ned as al l ot her l es i ons , i ncl udi ng s mal l l es i ons (great es t di amet er <20 mm wi t h convent i onal t echni ques or <10 mm wi t h s pi ral CT s can), t hat i s , bone l es i ons , l ept omeni ngeal di s eas e, as ci t es , pl eural /peri cardi al effus i on, i nfl ammat ory breas t di s eas e, l ymphangi t i s cut i s /pul moni s , cys t i c l es i ons , and al s o abdomi nal mas s es t hat are not confi rmed and fol l owed by i magi ng t echni ques . Al l t arget l es i ons are eval uat ed by i magi ng s t udi es or phys i cal exam. (Tabl e 4-2) P.133
TABLE 4.2 Response Evaluation Criteria in Solid Tumors (RECIST)
Pa g e 6 1 8
ABC Ambe r CHM Conve rte r Tria l ve rsion, http://w w w .proce sste x t.com/a bcchm.html
Co Di s mp ap l et pe e
ara
Re nce s po of ns e al l (CR t ar ):
get l es i on
s Par At t i al l ea Re s t s po a ns e 30 (PR % ):
dec rea se in t he su m of t he LD of t ar get l es i on s, t ak
Pa g e 6 1 9
ABC Ambe r CHM Conve rte r Tria l ve rsion, http://w w w .proce sste x t.com/a bcchm.html
i ng as ref ere nce t he bas el i ne su m LD Pro At gre l ea ssi st ve a Di s 20 eas % e
i nc
(PD rea ):
se in t he su m of t he LD of t ar get l es i on s, t ak
Pa g e 6 2 0
ABC Ambe r CHM Conve rte r Tria l ve rsion, http://w w w .proce sste x t.com/a bcchm.html
i ng as ref ere nce t he sm al l es t su m LD rec ord ed sin ce t he t re at me nt sta rt e d or t he ap pe ara nce of on e or
Pa g e 6 2 1
ABC Ambe r CHM Conve rte r Tria l ve rsion, http://w w w .proce sste x t.com/a bcchm.html
mo re ne w l es i on s St a Nei bl e t he Di s r eas s uf e
fi ci
(SD ent ):
s hr i nk ag e to qu al i f y for PR nor s uf fi ci ent i nc rea se to qu al i f y for
Pa g e 6 2 2
ABC Ambe r CHM Conve rte r Tria l ve rsion, http://w w w .proce sste x t.com/a bcchm.html
PD, t ak i ng as ref ere nce t he sm al l es t su m LD sin ce t he t re at me nt sta rt e d Evalua tion of nontar get lesion s *C Di s om ap pl e pe t e ara Re nce
Pa g e 6 2 3
ABC Ambe r CHM Conve rte r Tria l ve rsion, http://w w w .proce sste x t.com/a bcchm.html
s po of ns e al l (CR no ):
nt a rge t l es i on s an d nor ma liz at i on of tu mo r ma rke r l ev
el *In Per co s i s mp t en l et ce e
of
Re on s po e ns e or /St mo abl re
Pa g e 6 2 4
ABC Ambe r CHM Conve rte r Tria l ve rsion, http://w w w .proce sste x t.com/a bcchm.html
e
no
Di s nt a eas rge e
t
(SD l es ):
i on (s ) an d/o r ma i nt en anc e of tu mo r ma rke r l ev el ab ove t he nor ma l lim
its *Pr Ap ogr pe es s ara
Pa g e 6 2 5
ABC Ambe r CHM Conve rte r Tria l ve rsion, http://w w w .proce sste x t.com/a bcchm.html
i ve nce Di s of eas on e
e
(PD or ):
mo re ne w l es i on s an d/o r un eq ui v oca l pro gre ssi on of exi sti ng no nt a rge t l es i on s
Pa g e 6 2 6
ABC Ambe r CHM Conve rte r Tria l ve rsion, http://w w w .proce sste x t.com/a bcchm.html
LD, l onges t di amet er of t arget l es i ons Tabl e obt ai n ed from t he Nat i on al Cancer Ins t i t u te (NCI)â €“Imag i ng Res po ns e Cri t eri a. ht t p:// i magi n g.canc er.gov/ cl i ni cal t ri al s /i magi n g/(acc es s ed Januar y 5,
Pa g e 6 2 7
ABC Ambe r CHM Conve rte r Tria l ve rsion, http://w w w .proce sste x t.com/a bcchm.html
2007)
3. Combination chemotherapy. Thi s approach has revol ut i oni zed t he t reat ment of pot ent i al l y curabl e or hi ghl y chemot herapy-res pons i ve cancers . In general , combi nat i ons of act i ve agent s are requi red t o cure pat i ent s wi t h advanced, chemot herapy-s ens i t i ve di s eas es s uch as acut e l eukemi a and t es t i cul ar cancer. Such combi nat i on regi mens cont ai n effect i ve drugs wi t h hi gh res pons e rat es for t he part i cul ar t umor t ype bei ng t reat ed. o
o
a. Combi nat i ons are s el ect ed bas ed on evi dence of t herapeut i c s ynergy or addi t i vi t y wi t h accept abl e hos t t oxi ci t y. W hen pos s i bl e, hi ghl y act i ve agent s wi t h nonoverl appi ng t oxi ci t y profi l es are chos en.
o
o
b. The us e of combi nat i ons of margi nal l y act i ve chemot herapeut i c agent s t ends t o be rel at i vel y i nact i ve but more t oxi c t han t he us e of s i ngl e agent s wi t h comparabl e effi cacy profi l es . However, i n chemot herapy-s ens i t i ve cancers , combi nat i on chemot herapy i s us ual l y far more effect i ve t han s i ngl e-agent t herapy.
Pa g e 6 2 8
ABC Ambe r CHM Conve rte r Tria l ve rsion, http://w w w .proce sste x t.com/a bcchm.html o
o
c. Combi nat i on chemot herapy may prevent or del ay t he devel opment of drug res i s t ance t hat commonl y devel ops when mal i gnant cel l s are expos ed t o a s i ngl e agent over a gi ven peri od.
4. Drug resistance. Thi s phenomenon may be ei t her innate or acquired. Cert ai n mal i gnanci es (e.g., pancreat i c cancer and hepat ocel l ul ar carci noma), whi ch rarel y res pond t o convent i onal chemot herapy agent s , exhi bi t i nnat e drug res i s t ance. Ot her cancers (e.g., SCLC, ovari an cancer, and breas t cancer) i ni t i al l y res pond wel l t o chemot herapy before t he emergence of drug-res i s t ant vari ant s t hat l ead t o di s eas e progres s i on. Several mechani s ms of drug res i s t ance have been i dent i fi ed. o
o
a. Inhibition of drug accumulation or active drug efflux. Mul t i drug res i s t ance medi at ed by t he MDR1 gene, whi ch encodes t he p-gl ycoprot ei n t rans port er, exempl i fi es t hi s phenomenon. MDR1 promot es t he effl ux of a vari et y of s t ruct ural l y unrel at ed nat ural product s s uch as doxorubi ci n, et opos i de, t axanes , and vi nca al kal oi ds . The MRP gene member of t he c-MOAT t rans port er fami l y appears t o be anot her exampl e of t hi s phenomenon. Some agent s s uch as met hot rexat e requi re pol ygl ut amyl at i on t o be ret ai ned i nt racel l ul arl y.
o
o
b. Overexpression of the drug target. Thi s t ype of drug res i s t ance i s s peci fi c t o s uch common drugs
Pa g e 6 2 9
ABC Ambe r CHM Conve rte r Tria l ve rsion, http://w w w .proce sste x t.com/a bcchm.html
as t he ant i met abol i t e met hot rexat e, i n whi ch res i s t ance t o t he ant i met abol i t e can res ul t from gene ampl i fi cat i on of t he drug's t arget , t he enzyme di hydrofol at e reduct as e. Si mi l arl y, overexpres s i on of t he enzyme ara-CTP deami nas e can account for res i s t ance t o cyt os i ne arabi nos i de i n acut e l eukemi a. Thes e fact ors form t he bas i s for hi gh-dos e t herapy wi t h met hot rexat e and cyt os i ne arabi nos i de; t he object i s t o s at urat e t he overexpres s ed enzymes . o
o
c. Accelerated repair of chemotherapy-induced cellular damage. Al kyl at i ng agent s cros s -l i nk wi t h DNA t o i nduce pot ent i al l y l et hal cel l ul ar damage. Thi s damage s t i mul at es t he act i vi t y of DNA repai r enzymes , whi ch can repai r l es i ons . Accel erat ed expres s i on or act i vi t y of t hes e repai r enzymes may promot e al kyl at or drug res i s t ance.
o
o
d. Inhibition of cell death (apoptosis) pathways. Evi dent l y, cel l damage act i vat es programmed cel l deat h (apopt os i s ) t o prot ect t he organi s m. In many mal i gnant cel l s , apopt os i s pat hways are dys regul at ed, s omet i mes becaus e of overexpres s i on of a part i cul ar ant i apopt ot i c pat hway component , or by t he l os s of a proapopt ot i c el ement . The exi s t ence of s uch “common pat hways ― for res pons e (or l ack of res pons e) t o l et hal cel l ul ar i njury may expl ai n t he i nnat e drug res i s t ance s een i n many mal i gnanci es .
5. Commonly used chemotherapeutic agents. The mos t oft en us ed chemot herapeut i c drugs and t hei r
Pa g e 6 3 0
ABC Ambe r CHM Conve rte r Tria l ve rsion, http://w w w .proce sste x t.com/a bcchm.html
mechani s ms of act i on, t herapeut i c us es , and t oxi ci t i es are di s cus s ed i n t he fol l owi ng s ect i ons (Tabl e 4-3).
TABLE 4-3 Classes of Common Chemotherapeutic Agents Used in Oncologic Disease Alk yla tin g Ag ent s Ni t rog en mu sta rds Me chl ore t ha mi ne Cyc l op hos ph am
Pa g e 6 3 1
ABC Ambe r CHM Conve rte r Tria l ve rsion, http://w w w .proce sste x t.com/a bcchm.html
i de I fos fa mi de Mel ph al a n Chl ora mb uci l P l at i nu m an al o gs Ci s pl a tin Car bo pl a tin Ox al i
Pa g e 6 3 2
ABC Ambe r CHM Conve rte r Tria l ve rsion, http://w w w .proce sste x t.com/a bcchm.html
pl a tin Ni t ros our eas Car mu sti ne (BC NU ) L om us t i ne (CC NU ) S t re pt o z oc in Al k yl s ul fon at e s
Pa g e 6 3 3
ABC Ambe r CHM Conve rte r Tria l ve rsion, http://w w w .proce sste x t.com/a bcchm.html
Bu s ul fan E t hy l en ei mi nes an d me t hy lm el a mi nes T hi o t ep a He xa me t hy lm el a mi ne T ri a z en es
Pa g e 6 3 4
ABC Ambe r CHM Conve rte r Tria l ve rsion, http://w w w .proce sste x t.com/a bcchm.html
Da car baz i ne (DT IC) Mic rot ub ule In hib ito rs Vi n ca al k al o i ds Vi n cri s tin e Vi n bl a sti ne Vi n ore l bi ne
Pa g e 6 3 5
ABC Ambe r CHM Conve rte r Tria l ve rsion, http://w w w .proce sste x t.com/a bcchm.html
T axa nes P acl i t ax el Do cet axe l E str am us t i ne An tibi oti cs Da un oru bi ci n Do xor ubi ci n (Ad ri a my
Pa g e 6 3 6
ABC Ambe r CHM Conve rte r Tria l ve rsion, http://w w w .proce sste x t.com/a bcchm.html
ci n ) Mi t oxa nt r on e Mi t om yci n-C Bl e om yci n An tim eta bol ite s Me t ho t re xat e 5 -Fl uor our aci l
Pa g e 6 3 7
ABC Ambe r CHM Conve rte r Tria l ve rsion, http://w w w .proce sste x t.com/a bcchm.html
(5FU) F l ox uri di n e (FU DR ) F l ud ara bi n e Mer cap t op uri ne (6MP ) Cyt ara bi n e (Ar a-C ) T hi o gu
Pa g e 6 3 8
ABC Ambe r CHM Conve rte r Tria l ve rsion, http://w w w .proce sste x t.com/a bcchm.html
ani ne (6TG) P ent os t at i n To poi so me ras e In hib ito rs T op oi s om era se I I ri n ot e can T op ot e can T
Pa g e 6 3 9
ABC Ambe r CHM Conve rte r Tria l ve rsion, http://w w w .proce sste x t.com/a bcchm.html
op oi s om era se II E t op os i de Mis cell an eo us Ag ent s P roc arb az i ne
6. T ypes of chemotherapeutic agents o
o
a. Alkylating agents. Chemical classes i ncl ude ni t rogen mus t ards , pl at i num anal ogs , ni t ros oureas , al kyl s ul fonat es (bus ul fan), et hyl enei mi nes and met hyl mel ami nes , and t ri azenes (dacarbazi ne).
(1) Mechanism of action
Pa g e 6 4 0
ABC Ambe r CHM Conve rte r Tria l ve rsion, http://w w w .proce sste x t.com/a bcchm.html
(a) Al kyl at i ng agent s react s t rongl y wi t h nucl eophi l i c s ubs t ances and form coval ent l i nkages . The pl at i num compl exes of t he pl at i num anal ogs bi nd t o DNA, formi ng i nt ras t rand cros s -l i nks . In addi t i on, t he pl at i num anal ogs al s o bi nd t o nucl ear and cyt opl as mi c prot ei ns . Ci s pl at i n behaves as a bi funct i onal al kyl at i ng agent .
(b) The t oxi c effect s of al kyl at i ng agent s are bel i eved t o be rel at ed t o free radi cal format i on and al kyl at i on of t he component s of DNA, RNA, and cel l ul ar prot ei ns . Thes e react i ons have profound effect s on DNA repl i cat i on and t rans cri pt i on and may caus e cyt ot oxi ci t y, mut agenes i s , and carci nogenes i s .
(c) Al kyl at i on can t ake pl ace i n prol i ferat i ng and nonprol i ferat i ng cel l s . Therefore, al kyl at i ng agent s are not cel l cycl e–s peci fi c. The prol i ferat i ng compart ment i s more s ens i t i ve t o t he ki l l i ng effect s of t hes e drugs .
(2) Drug resistance. Thi s may occur becaus e of decreas ed drug accumul at i on by res i s t ant cel l s , i ncreas ed pharmacol ogi c i nact i vat i on of al kyl at i ng agent s , i ncreas ed repai r of al kyl at i ng agent –i nduced damage, and i ncreas ed i nt racel l ul ar s ul fhydryl groups (e.g., free t hi ol s ), whi ch compet e wi t h ot her t arget s
Pa g e 6 4 1
ABC Ambe r CHM Conve rte r Tria l ve rsion, http://w w w .proce sste x t.com/a bcchm.html
(e.g., DNA) t o neut ral i ze t he drug i nt racel l ul arl y. Pat i ent s who fai l t o res pond t o t reat ment wi t h one al kyl at i ng agent oft en res pond t o anot her.
(3) T herapeutic uses and toxicities are s ummari zed i n Onl i ne Tabl es 4-4 t hrough 4-9
TABLE 4.4 Nitrogen Mustards Co Th m er mo ap
n
eut T o ic xici Dr Us tie ug es s Me Ho Bo chl dgk ne ore i n's ma t ha di s rro mi eas w ne e,
s up
(ni s cl pre t ro ero s s i ge s i n on, n
g
na
mu ag us e s t a ent a, rd) for vo pl e mi t ura i ng l
,
Pa g e 6 4 2
ABC Ambe r CHM Conve rte r Tria l ve rsion, http://w w w .proce sste x t.com/a bcchm.html
eff al o us i pec ons i a, ,
i nf
t op ert i i cal l i t y di l , ut e s cl s ol ero ut i s i n on g in
ag
my ent cos is fun goi des Cyc Ho Bo l op dgk ne hos i n's ma ph di s rro am eas w i de e,
s up
(Cy l ym pre t ox ph s s i an) om on, as , i nf chr ert i oni l i t y c
,
l ym al o ph pec ocy i a, t i c na
Pa g e 6 4 3
ABC Ambe r CHM Conve rte r Tria l ve rsion, http://w w w .proce sste x t.com/a bcchm.html
l eu us e ke a, mi vo a
mi t
(CL i ng L), , mu he l t i p mo le
rrh
my agi el o c ma cys ,
titi
bre s as t (w can hi c cer h ,
is
s m pre al l - ven cel l t ed l un by g
hi g
can h cer fl ui ,
d
ova i nt ri a ake n
)
can cer , pe di a t ri c
Pa g e 6 4 4
ABC Ambe r CHM Conve rte r Tria l ve rsion, http://w w w .proce sste x t.com/a bcchm.html
tu mo rs , bo ne ma rro w t ra ns p l an t at i on Ifo Sof Bo s fa t
ne
mi t i s ma de s ue rro (If s ar w ex) co s up ma pre s,
ssi
t es on, t i c i nf ul a ert i r
lity
can , cer al o ,
pec
ova i a, ri a vo n
mi t
can i ng cer , ,
cen
cer t ral
Pa g e 6 4 5
ABC Ambe r CHM Conve rte r Tria l ve rsion, http://w w w .proce sste x t.com/a bcchm.html
vi c ner al
vou
can s cer s ys ,
te
l ym m ph (C om NS) as t ox i ci t y (w hi c h ma y be red uce d by lim itin g hi g h sin gl e dos es , red uci ng nar cot
Pa g e 6 4 6
ABC Ambe r CHM Conve rte r Tria l ve rsion, http://w w w .proce sste x t.com/a bcchm.html
ic us e , red uci ng ant i na us e a ag ent s, an d ot h er CN S act i ve dru gs ) Mel My Bo ph el o ne al a ma ma n
,
rro
(Al ma w ker cro s up an) gl o pre bul s s i i ne on, mi i nf a,
ert i
for l i t y
Pa g e 6 4 7
ABC Ambe r CHM Conve rte r Tria l ve rsion, http://w w w .proce sste x t.com/a bcchm.html
me , rl y na us e us e d
a,
in
al o
ova pec ri a i a n an d bre as t can cer Chl CLL Bo ora ,
ne
mb ma ma uci l cro rro (Le gl o w uke bul s up ran i ne pre )
mi s s i a,
on,
l ym na ph us e om a, a
al o pec ia (ra re) , i nf ert i lity
Pa g e 6 4 8
ABC Ambe r CHM Conve rte r Tria l ve rsion, http://w w w .proce sste x t.com/a bcchm.html
, s ki n ras h, rar el y he pat ic t ox i ci t y
TABLE 4.5 Platinum Analogs Co Th m er mo ap
n
eut T o ic xici Dr Us tie ug es s Ci s Tes Re pl a t i c nal t i n ul a t ox (Pl r,
i ci t
at i ova y, nol ri a vo )
n,
mi t
he i ng ad , an he d
ari
Pa g e 6 4 9
ABC Ambe r CHM Conve rte r Tria l ve rsion, http://w w w .proce sste x t.com/a bcchm.html
nec ng k,
l os
l un s , g,
my
bl a el o dd s up er, pre cer s s i vi x on, ,
tin
an ni t d
us ,
es o per ph i ph ag era eal l can ne cer uro ;
pat
l ym hy, ph hai om r a; l os a
s
rad i at i on s en siti z er Car Ov My bo ari el o pl a an s up t i n an pre (Pa d
ssi
Pa g e 6 5 0
ABC Ambe r CHM Conve rte r Tria l ve rsion, http://w w w .proce sste x t.com/a bcchm.html
rap l un on; l at i g
vo
n) can mi t cer i ng ,
;
ot h mu er ch ci s l es pl a s t i n ne -s e phr ns i ot o t i v xi ci e
t y,
t u ne mo uro rs
pat hy, an d ot o t ox i ci t y t ha n ci s pl a
tin Ox Col Ne al i ore uro pl a ct a pat tin l
hy
can cer
Pa g e 6 5 1
ABC Ambe r CHM Conve rte r Tria l ve rsion, http://w w w .proce sste x t.com/a bcchm.html
TABLE 4.6 Nitrosoureas Co Th m er mo ap
n
eut T o ic xici Dr Us tie ug es s Car Bra Bo mu i n
ne
s t i t u ma ne mo rro (BC rs , w NU Ho s up )
dgk pre i n's s s i di s on, eas na e,
us e
mu a, l t i p vo le
mi t
my i ng el o , ma mi l ,
d
bo an ne d ma rev rro ers w
i bl
t ra e
Pa g e 6 5 2
ABC Ambe r CHM Conve rte r Tria l ve rsion, http://w w w .proce sste x t.com/a bcchm.html
ns p l i v l an er t at t ox i on i ci t y, rar e l un g an d ki d ney da ma ge Lo Ly Bo mu mp ne s t i ho ma ne ma rro (CC ,
w
NU bra s up )
in
pre
tu ssi mo on, rs , na col us e on a, can vo cer mi t i ng , der ma titi
Pa g e 6 5 3
ABC Ambe r CHM Conve rte r Tria l ve rsion, http://w w w .proce sste x t.com/a bcchm.html
s, al o pec i a, t ra ns i ent he pat ot o xi ci ty (us ual ly rar e an d mi l d) St r Pa Na ept ncr us e oz o eat a, ci n i c
vo
(Za i s l mi t nos et i ng ar) cel l , t u du mo od rs , en car al ci n ul c oi d ers s
,
Pa g e 6 5 4
ABC Ambe r CHM Conve rte r Tria l ve rsion, http://w w w .proce sste x t.com/a bcchm.html
ren al t ox i ci t y, mi l d bo ne ma rro w s up pre ssi on, gl u cos e i nt ol e ran ce du e to isl et cel l da ma ge, mi l d he
Pa g e 6 5 5
ABC Ambe r CHM Conve rte r Tria l ve rsion, http://w w w .proce sste x t.com/a bcchm.html
pat ic t ox i ci t y
TABLE 4.7 Alkyl Sulfonates Co Th m er mo ap
n
eut T o ic xici Dr Us tie ug es s Bu Chr Bo s ul oni ne fan c
ma
(My my rro l er el o w an) ge s up no pre us s s i l eu on, ke i nf mi ert i a,
lity
bo , ne pul ma mo
Pa g e 6 5 6
ABC Ambe r CHM Conve rte r Tria l ve rsion, http://w w w .proce sste x t.com/a bcchm.html
rro nar w
y
t ra fi br ns p os i l an s , t at s ki i on n hyp erp ig me nt a tio n
TABLE 4.8 Ethyleneimines and Methylmelamines Co Th m er mo ap
n
eut T o ic xici Dr Us tie ug es s Thi Ly Bo ot e mp ne pa ho ma (TS ma rro PA) ,
w
s up s up erfi pre ci al s s i
Pa g e 6 5 7
ABC Ambe r CHM Conve rte r Tria l ve rsion, http://w w w .proce sste x t.com/a bcchm.html
bl a on, dd i nf er ert i can l i t y cer , ,
na
bre us e as t a can an cer d ,
vo
ova mi t ri a i ng n
(ra
can re cer wi t ,
h
bo l ow ne dos ma es ) rro w t ra ns p l an t at i on , i nt r at h eca l t he rap y
Pa g e 6 5 8
ABC Ambe r CHM Conve rte r Tria l ve rsion, http://w w w .proce sste x t.com/a bcchm.html
for car ci n om at o us me ni n gi t i s He Ov Na xa ari us e me an a, t hy can vo l m cer mi t el a ,
i ng
mi s m , ne al l bo (He cel l ne xal l un ma en) g
rro
can w cer s up ,
pre
l ym s s i ph on, om i nf a,
ert i
en l i t y do , me mo t ri a od l
cha
can ng cer es ,
Pa g e 6 5 9
ABC Ambe r CHM Conve rte r Tria l ve rsion, http://w w w .proce sste x t.com/a bcchm.html
,
per
cer i ph vi c era al
l
can ne cer uro pat hy, par es t hes i as , at a xi a
TABLE 4.9 Triazenes Dru Th Co g
era mm pe on ut i Tox c
i ci t
Us i es es Da Mel Na car an us e baz om a, i ne a,
vo
(DT s ar mi t IC) co i ng ma , s,
ma
Ho l ai dgk s e,
Pa g e 6 6 0
ABC Ambe r CHM Conve rte r Tria l ve rsion, http://w w w .proce sste x t.com/a bcchm.html
i n's l ow di s -gr eas ad e
e fev er, fl ulik e s yn dro me , bo ne ma rro w s up pre ssi on, rar e he pat ic t ox i ci t y an d di a rrh ea
Pa g e 6 6 1
ABC Ambe r CHM Conve rte r Tria l ve rsion, http://w w w .proce sste x t.com/a bcchm.html
P.134
o
o
b. Microtubule inhibitors
(1) Mechanism of action
(a) Vinca alkaloids i nt erfere wi t h t he funct i on of mi crot ubul ar s pi ndl e prot ei ns , l eadi ng t o an arres t of t he cel l cycl e i n mi t os i s .
(b) Podophyllotoxins caus e met aphas e arres t by exert i ng a macromol ecul ar effect on DNA s ynt hes i s . They can i nduce s t rand breaks i n DNA by i nt eract i ng wi t h t opoi s omeras e II.
(c) T axanes s t abi l i ze mi crot ubul e pol ymeri zat i on and l ead t o a breakdown of mi t ot i c s pi ndl e format i on.
(2) T herapeutic uses and toxicities are s ummari zed i n Onl i ne Tabl e 4-10.
TABLE 4.10 Microtubule Inhibitors
Pa g e 6 6 2
ABC Ambe r CHM Conve rte r Tria l ve rsion, http://w w w .proce sste x t.com/a bcchm.html
C T o h m er m a o p n e T ut o ic xi U ci D s ti ru e e g s s Vinca alkaloi ds Vi Ac N nc ut e ri e ur s t l y ot i n m ox e p i ci ( h ty O oc (t nc yt h ov i c e i n l e fi r ) uk s t e si m g i a ns ,
ar
H e o lo d ss gk of
Pa g e 6 6 3
ABC Ambe r CHM Conve rte r Tria l ve rsion, http://w w w .proce sste x t.com/a bcchm.html
in d 's e di e se p as t e e, n ly d m o p n h re o fl m ex a, es K , a d p ec os re i 's as sa e rc d o di m st a al se ns at io n, a n d p ar es th
Pa g e 6 6 4
ABC Ambe r CHM Conve rte r Tria l ve rsion, http://w w w .proce sste x t.com/a bcchm.html
es ia s) , sc le ro si n g a g e nt , al o p ec ia , co ns ti p at io n Vi H B n o o bl d n as gk e ti in m n 's ar e di ro
Pa g e 6 6 5
ABC Ambe r CHM Conve rte r Tria l ve rsion, http://w w w .proce sste x t.com/a bcchm.html
(V s e w el as s u b e, p a l y pr n) m es p si h o o n, m n a, a t e us st e i c a, ul p ar er ca i p nc h er er ,
al
n n o e n ur â o €“ p s at m hy al , l
co
ce ns ll ti lu p n at g io ca n, nc a
Pa g e 6 6 6
ABC Ambe r CHM Conve rte r Tria l ve rsion, http://w w w .proce sste x t.com/a bcchm.html
er s c ,
le
K ro a si p n os g i 's a sa g rc e o nt m a Vi N B n o o or n n el â e bi €“ m n s ar e m ro (N al w av l
su
el ce p bi l l pr n l u es e) n s i g o ca n, nc p er er ,
ip
br h e er as al t n ca e
Pa g e 6 6 7
ABC Ambe r CHM Conve rte r Tria l ve rsion, http://w w w .proce sste x t.com/a bcchm.html
nc ur er o ,
p
l y at m hy p , h m o il m d a al o p ec ia , sc le ro si n g a g e nt T axan es P O Al ac va l e l i t ri rg ax a i c el n, re (T br ac ax e t i ol as o ) t , ns
Pa g e 6 6 8
ABC Ambe r CHM Conve rte r Tria l ve rsion, http://w w w .proce sste x t.com/a bcchm.html
lu ( n w g, hi a ch n m d ay h b e e a pr d ev a e n nt d e n d ec by k pr ca e nc m er e di ca ti o n wi th d ex a m et h as o n
Pa g e 6 6 9
ABC Ambe r CHM Conve rte r Tria l ve rsion, http://w w w .proce sste x t.com/a bcchm.html
e, B e n a dr yl , a n d ci m et id in e) , m ye lo su p pr es si o n, al o p ec ia , m
Pa g e 6 7 0
ABC Ambe r CHM Conve rte r Tria l ve rsion, http://w w w .proce sste x t.com/a bcchm.html
ya lg ia s, fl uli ke sy n dr o m e D O M oc va ye et ri l o ax a s u el n, p (T br pr ax e es ot as s i er t , o e) l u n, n al g, o a p n ec d ia h , e p a er d ip a h n er
Pa g e 6 7 1
ABC Ambe r CHM Conve rte r Tria l ve rsion, http://w w w .proce sste x t.com/a bcchm.html
d al n e ec d k e ca m nc a er re s q ui ri n g co rt i co st er oi d pr o p hy la xi s Es Pr M t r os ye a ta lo m te su us ca p t i nc pr n er es e
si
(E
o
Pa g e 6 7 2
ABC Ambe r CHM Conve rte r Tria l ve rsion, http://w w w .proce sste x t.com/a bcchm.html
m
n,
cy
n
t)
a us e a a n d vo m iti n g o
o
c. Antibiotics
(1) Mechanism of action. Ant i t umor
ant i bi ot i cs are nat ural s ubs t ances t hat i nhi bi t DNA and RNA s ynt hes i s . They behave as bot h phas e-s peci fi c and phas e-nons peci fi c agent s and exhi bi t a vari et y of effect s on di fferent phas es of t he cel l cycl e.
(2) T herapeutic uses and toxicities are s ummari zed i n
Tabl e
4-11.
TABLE 4.11 Antibiotics
Pa g e 6 7 3
ABC Ambe r CHM Conve rte r Tria l ve rsion, http://w w w .proce sste x t.com/a bcchm.html
Co Th m er mo ap
n
eut T o ic xici Dr Us tie ug es s Da Acu Bo un t e ne oru my ma bi ci el o rro n
ge w no s up us pre l eu s s i ke on; mi na a
us e
(A a; ML) vo mi t i ng ; al o pec i a; s cl ero sin g ag ent ; i nf
Pa g e 6 7 4
ABC Ambe r CHM Conve rte r Tria l ve rsion, http://w w w .proce sste x t.com/a bcchm.html
ert i lity ; car di o my op at h y Do Bre Bo xor as t ne ubi can ma ci n cer rro (Ad ;
w
ri a l ym s up my ph pre ci n om s s i )
as ; on; s ar na co us e ma a; s ; vo l un mi t g
i ng
can ; cer mu ;
cos
sto itis ma ; ch gas can t roi cer nt e ;
sti
t hy nal roi t ox
Pa g e 6 7 5
ABC Ambe r CHM Conve rte r Tria l ve rsion, http://w w w .proce sste x t.com/a bcchm.html
d
i ci t
can y; cer al o pec i a; i nf ert i lity ; car di o my op at h y; s cl ero sin g ag ent Mi t Bre My oxa as t el o nt r can s up on cer pre e
,
ssi
(D l ym on; HA ph rar D) om el y a; na AM us e L
a; vo mi t i ng
Pa g e 6 7 6
ABC Ambe r CHM Conve rte r Tria l ve rsion, http://w w w .proce sste x t.com/a bcchm.html
; al o pec i a; el e vat ed liv er fun ct i on t es ts; occ as i on al l y bl u e uri ne; car di o t ox i ci t y Mi t Bre Bo om as t ne yci can ma n
cer rro
(M ;
w
ut a can s up my cer pre
Pa g e 6 7 7
ABC Ambe r CHM Conve rte r Tria l ve rsion, http://w w w .proce sste x t.com/a bcchm.html
ci n of )
ssi
t he on; s t o s cl ma ero ch, s i n pa g ncr ag eas ent ,
;
col l un on, g, cer he vi x pat ,
i c,
an an d
d
l un ki d g; ney us e da d
ma
as ge; a
i nf
rad ert i i os l i t y ens ; i t i z he er mo wi t l yt i h
câ€
5-fl “ur uor em our i c aci l s yn for dro an me
Pa g e 6 7 8
ABC Ambe r CHM Conve rte r Tria l ve rsion, http://w w w .proce sste x t.com/a bcchm.html
al
;
can car cer di o t ox i ci t y Bl e Tes Mi l om t i c d yci ul a my n
r
el o
(Bl can s up en cer pre oxa ;
ssi
ne) he on; ad fev an er; d
al l
nec erg k
ic
can rea cer ct i ;
ons
l ym ; ph l un om g as ; t ox Ho i ci t dgk y; i n's s ki di s n eas cha e; ng cer es ; vi c al o al
pec
Pa g e 6 7 9
ABC Ambe r CHM Conve rte r Tria l ve rsion, http://w w w .proce sste x t.com/a bcchm.html
can i a; cer rar el y hyp ot e ns i on; s cl ero sin g ag ent o
o
d. Antimetabolites
(1) Mechanism of action. Mos t ant i met abol i t es are phas e-s peci fi c and act duri ng t he S phas e of t he cel l cycl e. Ant i met abol i t es i nt eract wi t h cel l ul ar enzymes by:
(a) Subs t i t ut i ng s ome met abol i t e t hat i s normal l y i ncorporat ed i nt o a key mol ecul e s uch as DNA or RNA, t hus i nt erferi ng wi t h cel l ul ar funct i on
(b) Compet i ng wi t h a normal met abol i t e for occupat i on of t he cat al yt i c s i t e of a key enz yme
Pa g e 6 8 0
ABC Ambe r CHM Conve rte r Tria l ve rsion, http://w w w .proce sste x t.com/a bcchm.html
(c) Compet i ng wi t h normal met abol i t es t hat act as i mport ant enzyme regul at ory s i t es or ot her i mport ant recept ors
(2) T herapeutic uses and toxicities are s ummari zed i n
Tabl e
4-12.
TABLE 4.12 Antimetabolites Co Th m er mo ap
n
eut T o ic xici Dr Us tie ug es s Me Acu Bo t ho t e ne t re l ym ma xat ph rro e
ocy w t i c s up l eu pre ke s s i mi on; a
mu
(AL cos L); i t i s os t ; eo di a ge rrh ni c ea; s ar na co us e
Pa g e 6 8 1
ABC Ambe r CHM Conve rte r Tria l ve rsion, http://w w w .proce sste x t.com/a bcchm.html
ma a; ;
rar
l ym e ph ki d om ney a; an bre d as t l i v ,
er
he da ad, ma nec ge. k,
(To
gas xi ci t ri c t i e ,
s
an are d
i nc
bl a rea dd s ed er wh can en cer us e s; d cho wi t ri o h car ot h ci n er om hi g a; hl y i nt r pro at h t ei eca n-b l
ou
us e nd
Pa g e 6 8 2
ABC Ambe r CHM Conve rte r Tria l ve rsion, http://w w w .proce sste x t.com/a bcchm.html
for dru car gs . ci n Hi g om h at o dos us es me s ho ni n ul d gi t i be s
gi v en wi t h i nt r ave no us l eu cov ori n, hyd rat i on an d al k al i ni z at i on of t he uri ne.
Pa g e 6 8 3
ABC Ambe r CHM Conve rte r Tria l ve rsion, http://w w w .proce sste x t.com/a bcchm.html
) 5-F Ca Bo l uo nce ne rou rs
ma
rac of
rro
il
t he w
(5- bre s up FU) as t pre ,
ssi
col on; on, mu s t o cos ma i t i s ch, ; pa di a ncr rrh eas ea; ,
hyp
he erp ad i g an me d
nt a
nec t i o k,
n;
es o rar ph e ag cer us , eb an el l d
ar
an at a us ; xi a ant ; i t u my mo oca
Pa g e 6 8 4
ABC Ambe r CHM Conve rte r Tria l ve rsion, http://w w w .proce sste x t.com/a bcchm.html
r
rdi
act al i vi t i s c y
he
is
mi
i nc a rea s ed wh en it is gi v en wi t h l eu cov ori n Fl o Col Bi l i xur on ary i di can s cl ne cer ero ;
sis
reg i on al per fus i on wi t h he pat
Pa g e 6 8 5
ABC Ambe r CHM Conve rte r Tria l ve rsion, http://w w w .proce sste x t.com/a bcchm.html
ic art ery Fl u Chr My dar oni el o abi c
s up
ne l ym pre (Fl ph s s i ud ocy on; ara t i c fl u)
l eu l i k ke e mi s yn a; dro l ym me ph ; om na a
us e a; al o pec
ia Mer Mai Bo cap nt e ne t op na ma uri nce rro ne t he w (6- rap s up MP y )
pre
for s s i acu on; t e na l eu us e ke a; mi s t o a
ma
Pa g e 6 8 6
ABC Ambe r CHM Conve rte r Tria l ve rsion, http://w w w .proce sste x t.com/a bcchm.html
titi s; rar el y ras h; liv er t ox i ci t y; i nt r ah ep at i c cho l es t as is Cyt Acu Bo ara t e ne bi n my ma e
el o rro
(ar ge w a-C no s up )
us pre l eu s s i ke on; mi na a
us e
(A a; ML) an ;
ore
ALL xi a
Pa g e 6 8 7
ABC Ambe r CHM Conve rte r Tria l ve rsion, http://w w w .proce sste x t.com/a bcchm.html
;
;
bl a s t o s t i ma c
titi
ph s ; as e fl uof
lik
chr e oni s yn c
dro
my me el o ; ge l i v no er us t ox l eu i ci t ke y; mi cer a; eb pre el l l eu ar ke dys mi fun a
ct i
on Thi AM Bo og L
ne
ua
ma
ni n
rro
e
w
(6-
s up
TG)
pre ssi
on Pe Ly Bo nt o mp ne
Pa g e 6 8 8
ABC Ambe r CHM Conve rte r Tria l ve rsion, http://w w w .proce sste x t.com/a bcchm.html
s t a ho ma t i n ma rro (Ni ;
w
pe hai s up nt ) ry
pre
cel l s s i l eu on; ke i nc mi rea a
s ed ren al t ox i ci t y; con jun ct i vi t i s; fev er; he pat ot o xi ci ty o
o
e. T opoisomerase I inhibitors
(1) Mechanism of action. Thes e agent s i nhi bi t t he funct i on of t opoi s omeras e I, an enzyme t hat regul at es t he unwi ndi ng of s upercoi l ed DNA.
Pa g e 6 8 9
ABC Ambe r CHM Conve rte r Tria l ve rsion, http://w w w .proce sste x t.com/a bcchm.html
(2) T herapeutic uses and toxicities are s ummari zed i n Onl i ne Tabl e 4-13.
TABLE 4.13 Topoisomerase Inhibitors Co Th m er mo ap
n
eut T o ic xici Dr Us tie ug es s Et o Tes Del pos t i c aye i de ul a d (VP r
l eu
-16 can kop )
cer eni ,
a,
s m mi l al l d cel l t hr l un om g
boc
can yt o cer pe ,
ni a
no , nâ rar €“s e ma na ll
us e
cel l a
Pa g e 6 9 0
ABC Ambe r CHM Conve rte r Tria l ve rsion, http://w w w .proce sste x t.com/a bcchm.html
l un an g
ore
can xi a cer , ,
di a
l ym rrh ph ea, om rar as , e s ar hyp co ot e ma ns i s,
on
(pe wi t di a h t ri c rap an i d d
i nf
Ka us i pos ons i 's ) Iri n Col Di a ot e ore rrh can ct a ea, (Ca l
my
mp can el o t os cer s up ar) ,
pre
no s s i nâ on, €“s al o ma pec ll
ia
cel l l un
Pa g e 6 9 1
ABC Ambe r CHM Conve rte r Tria l ve rsion, http://w w w .proce sste x t.com/a bcchm.html
g can cer To Ov My pot ari el o eca an s up n
can pre
(Hy cer s s i ca ,
on,
mt i s m an n) al l em cel l i a l un g can cer o
f. Miscellaneous agents. Procarbazi ne i s i n t hi s cat egory (Onl i ne Tabl e 4-14). o
TABLE 4.14 Miscellaneous Agents Co Th m er mo ap
n
eut T o ic xici Dr Us tie ug es s Pro Ho Ne car dgk uro baz i n's l og i ne di s i c
Pa g e 6 9 2
ABC Ambe r CHM Conve rte r Tria l ve rsion, http://w w w .proce sste x t.com/a bcchm.html
(M eas s y at u e; mp l an l ym t o e) ph ms om ; a
hyp ot e ns i ve rea ct i ons wi t h t ri c ycl i c ant i de pre ssa nt s an d s ub sta nce s wi t h hi g h t yr am i ne
Pa g e 6 9 3
ABC Ambe r CHM Conve rte r Tria l ve rsion, http://w w w .proce sste x t.com/a bcchm.html
con t en ts; di s ul fi ra m-l i ke rea ct i on wi t h al c oh ol an d dru gs in nos e dro ps , cou gh pre par at i ons , an d l oc
Pa g e 6 9 4
ABC Ambe r CHM Conve rte r Tria l ve rsion, http://w w w .proce sste x t.com/a bcchm.html
al an es t het i cs ; bo ne ma rro w de pre ssi on; na us e a; vo mi t i ng ; per i ph era l ne uro pat hy; l et har gy; ras h;
Pa g e 6 9 5
ABC Ambe r CHM Conve rte r Tria l ve rsion, http://w w w .proce sste x t.com/a bcchm.html
cen t ral ner vou s s ys te m t ox i ci t y
D. Biologic response modifiers Biologic response modifiers are agent s t hat al t er t he growt h charact eri s t i cs or ant i geni ci t y of a t umor, res ul t i ng i n a t herapeut i c enhancement of hos t res pons e. The mechani s m of act i on may be i mmunol ogi c or noni mmunol ogi c.
1. Classification. Bi ol ogi c res pons e modi fi ers are cl as s i fi ed accordi ng t o t he fol l owi ng funct i onal cat egori es . o
o
a. Agent s t hat restore, augment, or modify host immunologic mechanisms t hought t o be cri t i cal for t umor growt h and met as t as i s [e.g., baci l l e Cal met t e-Guéri n (BCG), i nt erferon-α (IFN-α)] P.135
o
o
b. Cel l s or cel l ul ar product s t hat have di rect cytotoxic or cytostatic effect s (e.g., donor l ymphocyt e i nfus i ons )
Pa g e 6 9 6
ABC Ambe r CHM Conve rte r Tria l ve rsion, http://w w w .proce sste x t.com/a bcchm.html o
o
c. Agent s t hat al t er metastatic potential or t hat affect t he initiation or maintenance of neoplastic transformation (e.g., ret i noi ds , mat ri x met al l oprot ei nas e i nhi bi t ors , angi ogenes i s i nhi bi t ors , cycl ooxygenas e-2 i nhi bi t ors , s i gnal t rans duct i on i nhi bi t ors )
2. BCG. An at t enuat ed s t rai n of Myc obac t eri um bovi s , BCG has demons t rat ed act i vi t y when admi ni s t ered l ocal l y i n di rect cont act wi t h t umor cel l s . o
o
a. T herapeutic indications. BCG t herapy i s used successfully in the treatment of recurrent superficial bladder cancers. Di rect i ns t i l l at i on of BCG i nt o t he bl adder aft er t rans uret hral res ect i on or ful gurat i on of vi s i bl e l es i ons s i gni fi cant l y reduces t he i nci dence of recurrent t umors as wel l as t he number of pat i ent s ul t i mat el y requi ri ng cys t ect omy for i nvas i ve bl adder cancer.
o
o
b. T oxicity. Local i rri t at i on and a fl u-l i ke i l l nes s , cons i s t i ng of fever and mal ai s e, i s common i n t he fi rs t 24 hours aft er admi ni s t rat i on.
3. Hematopoietic growth factors. Thi s fami l y of prot ei ns regul at es t he prol i ferat i on, di fferent i at i on, mat urat i on, and funct i on of bl ood cel l s . o
o
a. The interleukins (IL-1, IL-3, and IL-6) affect t ot i pot ent i al stem cells, or i mmat ure hemat opoi et i c
Pa g e 6 9 7
ABC Ambe r CHM Conve rte r Tria l ve rsion, http://w w w .proce sste x t.com/a bcchm.html
precurs ors . In addi t i on, t hes e cyt oki nes have wi des pread phys i ol ogi c effect s on l ymphocyt es and ot her organs . o
o
b. Colony-stimulating factors act pri mari l y on di fferent i at ed cel l s . Thes e fact ors i ncl ude eryt hropoi et i n (EPO), granul ocyt e col ony-s t i mul at i ng fact or (G-CSF), granul ocyt e macrophage col ony-s t i mul at i ng fact or (GM-CSF), macrophage col ony-s t i mul at i ng fact or (M-CSF), fl t -3 l i gand, and t hrombopoi et i n.
(1) EPO, s ynt hes i zed by t he ki dney, i s a major regul at ory fact or of red bl ood cel l (RBC) product i on. Int ravenous admi ni s t rat i on of EPO res ul t s i n a marked i ncreas e i n ret i cul ocyt e number and t he hemat ocri t .
(a) T herapeutic indications. Anemi a at t ri but abl e t o chroni c i l l nes s s uch as renal di s eas e and acqui red i mmunodefi ci ency s yndrome (AIDS) as wel l as myel odys pl as t i c s yndromes and anemi a from chemot herapy are as s oci at ed wi t h decreas ed EPO product i on.
(b) Side effects. Rarel y, EPO admi ni s t rat i on i s as s oci at ed wi t h an i ncreas e i n di as t ol i c bl ood pres s ure.
(2) GM-CSF pri mari l y s t i mul at es t he
Pa g e 6 9 8
ABC Ambe r CHM Conve rte r Tria l ve rsion, http://w w w .proce sste x t.com/a bcchm.html
product i on of neut rophi l s , eos i nophi l s , granul ocyt es , and monocyt es , whereas G-CSF s t i mul at es neut rophi l s and granul ocyt es .
(a) T herapeutic indications. Bot h G-CSF and GM-CSF reduce and s hort en granul ocyt e nadi rs from chemot herapy, has t en recovery aft er BMT, and hel p res t ore bone marrow funct i on i n di s eas e-rel at ed or i at rogeni c neut ropeni a. As a res ul t , pat i ent s experi ence fewer febri l e epi s odes and a reduced requi rement for ant i bi ot i cs aft er chemot herapy and BMT.
(b) Side effects. G-CSF may produce mi l d-t o-moderat e bone or mus cl e pai n or al t erat i ons i n bl ood chemi s t ry val ues . A few pat i ent s may devel op s pl enomegal y, headache, fl us hi ng, ches t di s comfort , cut aneous react i ons , or hypot ens i on.
4. Interferons (IFNs) are gl ycoprot ei ns t hat pos s es s ant i vi ral and ant i prol i ferat i ve act i vi t i es . They al s o enhance T-cel l and macrophage act i vi t y, s t i mul at e t he expres s i on of t umor-s peci fi c cel l s urface ant i gens , modi fy oncogene expres s i on, and promot e cel l ul ar di fferent i at i on. o
o
a. Two t ypes of IFN-α have been approved for t he t reat ment of hai ry-cel l l eukemi a and Kapos i 's s arcoma. IFN-α has al s o been us ed i n CML, non-Hodgki n's l ymphoma, renal cel l cancer, mycos i s
Pa g e 6 9 9
ABC Ambe r CHM Conve rte r Tria l ve rsion, http://w w w .proce sste x t.com/a bcchm.html
fungoi des , mel anoma, carci noi d t umors , and myel oma. o
o
b. T oxic reactions i ncl ude a fl u-l i ke i l l nes s , wi t h fever, chi l l s , mal ai s e, mi l d l eukopeni a, and el evat i on of hepat i c enzymes .
5. IL-2, a l ymphoki ne s ecret ed by act i vat ed T l ymphocyt es , modi fi es t he prol i ferat i on and funct i on of T and B cel l s and i s es s ent i al for t he growt h of T cel l s and peri pheral bl ood l ymphocyt es . IL-2 i s occas i onal l y effect i ve i n pat i ent s wi t h advanced mel anoma and renal cel l carci noma. In rare i ns t ances , t hes e res pons es are compl et e and very durabl e. Hi gh-dos e t herapy wi t h IL-2, al t hough very t oxi c (s ee IV D 5 b), i s requi red t o achi eve t hes e durabl e compl et e res pons es . The mechani s ms underl yi ng t hes e res pons es appear t o be t he act i vat i on and expans i on of T cel l s react i ve wi t h cancer cel l –s peci fi c ant i gens , l eadi ng t o t he es t abl i s hment of an effect i ve adapt i ve ant i t umor i mmune res pons e. IL-2 t herapy al s o s t i mul at es t he product i on of act i vat ed nat ural ki l l er (NK) cel l s , t ermed lymphokine-activated killer (LAK) cells, whi ch medi at e broad, non–major hi s t ocompat i bi l i t y compl ex (MHC)–res t ri ct ed t umor l ys i s . However, t he t herapeut i c rel evance of t hes e cel l s , whi ch rarel y i nfi l t rat e t umors , i s uncl ear. o
o
a. Indications. IL-2 t herapy i s i ndi cat ed i n pat i ent s wi t h advanced renal cel l carci noma and mel anomas .
o
o
b. Side effects. Hi gh-dos e IL-2 t herapy i s as s oci at ed wi t h s evere t oxi ci t y, i ncl udi ng a
Pa g e 7 0 0
ABC Ambe r CHM Conve rte r Tria l ve rsion, http://w w w .proce sste x t.com/a bcchm.html
capi l l ary l eak s yndrome, fl ui d ret ent i on, renal fai l ure, pul monary edema, s evere neurol ogi c s ympt oms , s hort nes s of breat h, el evat i ons i n s erum creat i ni ne and bi l i rubi n, anemi a, t hrombocyt openi a, di arrhea, s ki n ras hes , and fever. Al t hough hi gh-dos e t herapy frequent l y requi res i nt ens i ve care uni t –bas ed management , l ower dos es of IL-2 can be effect i vel y admi ni s t ered i n an out pat i ent s et t i ng wi t h equi val ent overal l cl i ni cal act i vi t y, but fewer durabl e compl et e remi s s i ons .
6. Monoclonal antibodies di rect ed agai ns t t umor ant i gens can be produced by convent i onal hybri doma t echnol ogy or by recombi nant engi neeri ng. Target s for monocl onal ant i bodi es i ncl ude ant i gens t hat are selectively expressed by mal i gnant cel l s [e.g., carci noembryoni c ant i gen (CEA)] or t hat exert i mport ant functional effects on mal i gnant cel l growt h or vi abi l i t y when t hey are engaged (e.g., HER2/neu, epi dermal growt h fact or recept or). o
o
a. Diagnostic uses. Monocl onal ant i bodi es t arget i ng di fferent i at i on ant i gens or ot her cancer-rel at ed ext racel l ul ar or i nt racel l ul ar s t ruct ures faci l i t at e hi s t opat hol ogi c di agnos i s . They have part i cul ar val ue i n s t agi ng l eukemi as and l ymphomas and i n i dent i fyi ng t he ori gi n of poorl y di fferent i at ed t umors . P.136
o
o
b. T herapeutic uses. Recent years have wi t nes s ed
Pa g e 7 0 1
ABC Ambe r CHM Conve rte r Tria l ve rsion, http://w w w .proce sste x t.com/a bcchm.html
t he emergence of s everal monocl onal ant i bodi es for t he t reat ment of vari ous mal i gnanci es . As t he concept of t arget ed t herapy gai ns more moment um, more monocl onal ant i bodi es and “s mal l mol ecul es ― agai ns t s peci fi c t arget s have become avai l abl e. Two ant i bodi es have had t he great es t i mpact on t he t reat ment of pat i ent s wi t h cancer t hus far.
(1) Tras t uzumab (Hercept i n) a monocl onal ant i body di rect ed agai ns t t he ext racel l ul ar domai n of HER2/neu oncogene–deri ved prot ei n, di rect l y i nhi bi t s breas t cancer cel l growt h and pot ent i at es t he effect s of cyt ot oxi c chemot herapeut i c agent s s uch as t axanes and pl at i num compounds . Tras t uzumab has been approved by t he U.S. Food and Drug Admi ni s t rat i on (FDA) for t he t reat ment of met as t at i c breas t cancer i n pat i ent s whos e t umors overexpres s HER2/neu, ei t her as a s i ngl e agent or i n conjunct i on wi t h t radi t i onal chemot herapeut i c agent s .
(2) Ri t uxi mab (Ri t uxan) i s a chi meri c monocl onal ant i body agai ns t t he CD20 ant i gen on t he s urface of mat ure B l ymphocyt es . Thi s agent i s approved by t he FDA for t he t reat ment of pat i ent s wi t h B-cel l non-Hodgki n's l ymphoma (NHL). Cl i ni cal t ri al s have s hown t hat , even i n previ ous l y t reat ed pat i ent s , s i ngl e-agent ri t uxi mab i s effect i ve i n i nduci ng remi s s i ons . However, t here i s no evi dence t hat t he overal l s urvi val of pat i ent s wi t h B-cel l NHL i s i mproved by t he us e of t hi s
Pa g e 7 0 2
ABC Ambe r CHM Conve rte r Tria l ve rsion, http://w w w .proce sste x t.com/a bcchm.html
ant i body. Ri t uxi mab i n conjunct i on wi t h chemot herapy i s now t he s t andard of care i n t he management of pat i ent s wi t h i nt ermedi at e and hi gh-grade l ymphoma.
(3) Ot her ant i bodi es are emergi ng as wel l . Bevaci zumab (Avas t i n) i s a recombi nant humani zed monocl onal ant i body agai ns t t he vas cul ar endot hel i al growt h fact or (VEGF). Thi s ant i -angi ogenes i s t arget has been s hown t o be effect i ve i n t he t reat ment of a number of mal i gnanci es , i ncl udi ng ki dney, col orect al , and l ung cancers . In combi nat i on wi t h chemot herapy, i t i s approved by t he FDA for t he t reat ment of advanced col orect al cancer, bas ed on recent l y report ed s t udi es . Thi s novel approach carri es t he promi s e of depri vi ng t he t umor of t he es s ent i al bl ood s uppl y i t needs for i t s growt h and perhaps met as t as es . The mechani s ms underl yi ng t he effi cacy of t hes e ant i bodi es are under i nves t i gat i on.
o
o
c. Other uses
(1) Ant i bodi es may be us ed ex vi vo t o purge aut ol ogous bone marrow of res i dual t umor cel l s before re-i nfus i on aft er hi gh-dos e cyt ot oxi c chemot herapy.
(2) Ant i bodi es al s o have pot ent i al ut i l i t y as vehi cl es for t he del i very of radi oi s ot opes and chemot herapeut i c agent s .
Pa g e 7 0 3
ABC Ambe r CHM Conve rte r Tria l ve rsion, http://w w w .proce sste x t.com/a bcchm.html
(a) Radiolabeled monoclonal antibodies. Two radi ol abel ed ant i bodi es have recent l y been approved by t he FDA for t he t reat ment of pat i ent s wi t h l ow-grade, refract ory NHL. Ibri t umomab t i uxet an (Zeval i n) and t os i t umomab (Bexxar) are capabl e of del i veri ng l et hal radi at i on dos es t o l ymphoma cel l s . Bot h agent s exhi bi t hi gh l evel s of ant i -t umor act i vi t y i n heavi l y pret reat ed pat i ent s . Zeval i n us es 131
90
Y t t ri um and Bexxar us es
Iodi ne as t he s ources of radi at i on.
Bot h can be gi ven on an out pat i ent bas i s wi t h mi ni mal ri s k t o ot her pat i ent cont act s .
(b) Chemoimmunoconjugates t hat cont ai n an ant i body react i ve wi t h CD33 and t he drug cal i cheami ci n exhi bi t s i gni fi cant act i vi t y i n chemot herapy-refract ory acut e myel ogenous l eukemi a (AML).
(c) Immunotoxins t hat cont ai n ant i body-bi ndi ng domai ns and cat al yt i c t oxi ns s uch as ri ci n A chai n or Ps eudomonas exot oxi n have undergone ext ens i ve cl i ni cal t es t i ng.
(3) Anti-idiotypic antibodies i nduce t he product i on of hos t ant i bodi es t hat mi mi c and ul t i mat el y recogni ze t umor-as s oci at ed cel l -s urface ant i gens . Thes e agent s are bei ng
Pa g e 7 0 4
ABC Ambe r CHM Conve rte r Tria l ve rsion, http://w w w .proce sste x t.com/a bcchm.html
i nves t i gat ed as pos s i bl e tumor vaccines.
7. Cancer vaccines may res ul t from new unders t andi ng of t umor i mmunol ogy and t he component s and regul at i on of t he human ant i t umor i mmune res pons e. Modern mol ecul ar bi ol ogy and progres s i n genet i c engi neeri ng have generat ed i nt eres t i n t hes e vacci nes . o
o
a. T umor cell vaccines, us ual l y l et hal l y i rradi at ed and t hen admi ni s t ered wi t h vari ous i mmunol ogi c adjuvant s , have not proved cons i s t ent l y benefi ci al but cont i nue t o be act i vel y s t udi ed, part i cul arl y as adjuvant t herapy i n hi gh-ri s k col on cancer. Genes for cyt oki nes s uch as GM-CSF or IL-2 or i mmune cos t i mul at ory mol ecul es s uch as B7.1 have been i nt roduced i nt o genetically modified tumor cell vaccines t o s t i mul at e l ocal i mmune res pons es and break P.137
i mmunol ogi c t ol erance t o ot herwi s e crypt i c t umor ant i gens . Puri fi ed or recombi nant l y produced t umor ant i gens can be admi xed wi t h adjuvant s t o i nduce s peci fi c i mmuni t y t o t hos e ant i gens . o
o
b. Typi cal l y, t he goal of cancer vacci ne effort s has been t o i nduce T -cell–mediated cytotoxic immune responses, becaus e t hes e s eem t o be t he mos t rel evant t o cancers bas ed on precl i ni cal model s . To t hi s end, peptide vaccines have been devel oped; t he pept i des bi nd t o cl as s I MHC mol ecul es and are recogni zed by defi ned T-cel l
Pa g e 7 0 5
ABC Ambe r CHM Conve rte r Tria l ve rsion, http://w w w .proce sste x t.com/a bcchm.html
recept ors whos e engagement medi at es t umor regres s i on. Thes e pept i des can be mi xed i n convent i onal adjuvant s or can be di rect l y l oaded ont o profes s i onal ant i gen-pres ent i ng cel l s s uch as dendritic cells. T-cel l recogni t i on epi t opes from a vari et y of ant i gens i dent i fi ed t o be t arget s i n regres s i ng mel anomas have been prepared as pept i de vacci nes , and pept i de vacci nes have been prepared from ot her pot ent i al t umor ant i gens i n ot her di s eas es . Dendri t i c cel l s al s o can be di rect l y l oaded wi t h whol e prot ei ns , or even t rans duced wi t h t he genet i c mat eri al from a defi ned gene or mal i gnant cel l t o s t i mul at e a cl as s I–dependent i mmune res pons e. o
o
c. Vacci nes des i gned t o s t i mul at e ant i body res pons es are under act i ve i nves t i gat i on as wel l i n a number of di s eas e s et t i ngs , us i ng defi ned i mmunogens as wel l as t he ant i -i di ot ype approach [s ee IV D 6 c (3)].
8. Angiogenesis inhibition, whi ch i s bas ed on t he obs ervat i on t hat neovas cul ari zat i on i s a neces s ary component of cancer growt h, has become an exci t i ng area of cancer i nves t i gat i on, cul mi nat i ng i n t he devel opment of Bevaci zumab [s ee IV D 6 b (3)]. Accordi ngl y, i nhi bi t i on of t he normal hos t res pons e t o angi ogeni c fact ors (e.g., VEGF, bas i c fi brobl as t growt h fact or, and angi ogeni n) el aborat ed by t umor cel l s s houl d exert a profound ant i prol i ferat i ve effect . In precl i ni cal model s , a number of s t rat egi es have exhi bi t ed promi s e, and earl y cl i ni cal t ri al s are bei ng conduct ed t o i dent i fy t he mos t promi s i ng s t rat egi es for fut ure i mpl ement at i on.
Pa g e 7 0 6
ABC Ambe r CHM Conve rte r Tria l ve rsion, http://w w w .proce sste x t.com/a bcchm.html
V. Lung Cancer A. Incidence/Epidemiology The frequency of l ung cancer i s i ncreas i ng rapi dl y. Ori gi nal l y a di s eas e t hat pri mari l y affl i ct ed men ol der t han age 60 years , l ung cancer has become t he s econd mos t common caus e of cancer i n women. The annual i nci dence of l ung cancer i n t he Uni t ed St at es i s 174,400 and i t i s t he l eadi ng caus e of cancer deat h i n bot h men and women.
B. Etiology/Risk Factors Epi demi ol ogi c s t udi es have l i nked l ung cancer t o t he fol l owi ng fact ors :
1. Cigarette smoking. Res earch has demons t rat ed a di rect correl at i on bet ween s moki ng and l ung cancer, wi t h i ncreas es i n dai l y ci garet t e cons umpt i on l eadi ng t o i ncreas es i n cancer i nci dence. “Pas s i ve s moki ng― i s as s oci at ed wi t h a s mal l but s i gni fi cant i ncreas e i n t he i nci dence of l ung cancer.
2. Industrial carcinogens. Expos ure t o beryl l i um, radon, hydrocarbons , mus t ard gas , ni ckel , radi at i on, and as bes t os has been l i nked t o l ung cancer. Ci garet t e s moki ng exacerbat es t he ri s k as s oci at ed wi t h s uch expos ure. Smoki ng and as bes t os expos ure i ncreas e t he ri s k 53-fol d.
3. Air pollutants (e.g., di es el exhaus t , pi t ch, t ar, ars eni c, chromi um, cadmi um, ni ckel ) i ncreas e t he ri s k of l ung cancer.
4. Existing lung damage. Adenocarci nomas of t he l ung
Pa g e 7 0 7
ABC Ambe r CHM Conve rte r Tria l ve rsion, http://w w w .proce sste x t.com/a bcchm.html
may devel op i n areas s carred by tuberculosis or ot her l ung condi t i ons as s oci at ed wi t h fibrosis. Thes e t umors are cal l ed scar carcinomas.
5. Pat i ent s wi t h lymphoma or malignancies of the head, neck, and esophagus have an i ncreas ed i nci dence of l ung cancer.
C. Screening At pres ent t here are no recommendat i ons from any organi zat i on endors i ng earl y l ung cancer det ect i on i n as ympt omat i c i ndi vi dual s at ri s k for l ung cancer. The Ameri can Cancer Soci et y (ACS), however, i n 2001 emphas i zed t he i mport ance of i nformed deci s i ons among i ndi vi dual s at ri s k who s eek t es t i ng. Indi vi dual s at hi gh ri s k for t he devel opment of l ung cancer i ncl ude t hos e who have s i gni fi cant expos ure t o t obacco s moke or occupat i onal expos ures . Such t es t i ng s houl d be done i n experi enced cent ers charact eri zed by mul t i di s ci pl i nary s peci al t y groups wi t h experi ence i n t es t i ng, di agnos i s , and fol l ow-up. Of cours e, t he bes t prevent i on i s s moki ng ces s at i on al l t oget her. P.138
D. Pathology There are four major vari et i es of l ung cancer: t hree t ypes are commonl y des cri bed as non–s mal l -cel l l ung cancer (non-SCLC), and one i s t ermed s mal l cel l l ung cancer (SCLC).
1. Squamous cell carcinomas (non-SCLC) were once t he mos t common t ype of non-SCLC, but t hey have become l es s common t han adenocarci nomas . Squamous cel l t umors t end t o ari s e cent ral l y near t he hi l um, where t hey pres ent as endobronchi al di s eas e or as a peri pheral l es i on.
Pa g e 7 0 8
ABC Ambe r CHM Conve rte r Tria l ve rsion, http://w w w .proce sste x t.com/a bcchm.html
2. Adenocarcinoma (non-SCLC) i s i ncreas i ng i n frequency, es peci al l y i n women. It i s t he mos t frequent l y di agnos ed form of non-SCLC i n bot h men and women. Thes e l es i ons are oft en peri pheral , occurri ng i n more di s t al ai rways . Bronchoalveolar cell cancer, a vari ant of adenocarci noma, ari s es i n t he al veol i and caus es l obul ar cons ol i dat i on on t he ches t radi ograph.
3. Large cell undifferentiated cancers (non-SCLC) account for 5%–10% of al l l ung cancers . Thes e t umors are us ual l y peri pheral l es i ons .
4. Small cell lung carcinoma (SCLC) ari s es from neural cres t neuroendocri ne or ami ne precurs or upt ake and decarboxyl at i on cel l s and progres s es rapi dl y wi t hout t reat ment . The average s urvi val t i me i n t he abs ence of t reat ment i s onl y 2–4 mont hs .
E. Clinical features
1. Local symptoms of i nt rat horaci c di s eas e i ncl ude cough, hemopt ys i s , obs t ruct i ve pneumoni a (due t o endobronchi al t umors ), ches t pai n, pl eural effus i on, hoars enes s (due t o recurrent l aryngeal nerve compres s i on by a medi as t i nal t umor), wheeze, s t ri dor, and s uperi or vena cava s yndrome (due t o obs t ruct i on of t he ves s el by a medi as t i nal t umor). Pat i ent s wi t h bronchoal veol ar cel l cancer may have a s evere cough product i ve of cl ear s put um.
2. Systemic manifestations i ncl ude:
Pa g e 7 0 9
ABC Ambe r CHM Conve rte r Tria l ve rsion, http://w w w .proce sste x t.com/a bcchm.html o
o
a. Anorexi a and wei ght l os s
o
o
b. Bone pai n from di s t ant met as t as es
o
o
c. Hepat omegal y, t endernes s , and fever caus ed by l i ver i nvol vement
o
o
d. CNS s i gns or s ei zures from brai n met as t as es or carci nomat ous meni ngi t i s
o
o
e. Hypercal cemi a from bone met as t as es or ot her humoral s ubs t ances
3. Pat i ent s wi t h SCLC may pres ent wi t h s ympt oms of ect opi c hormone product i on or ot her paraneoplastic syndromes. o
o
a. Ectopic adrenocorticotropic hormone (ACT H) secretion caus es hypokal emi a and mus cl e was t i ng.
o
o
b. Syndrome of inappropriate antidiuretic hormone secretion (SIADH) res ul t s i n hyponat remi a.
F. Diagnosis
1. Sputum cytology or bronchoscopy confi rms t he
Pa g e 7 1 0
ABC Ambe r CHM Conve rte r Tria l ve rsion, http://w w w .proce sste x t.com/a bcchm.html
di agnos i s i n pat i ent s wi t h endobronchi al di s eas e. Bronchos copy al s o as s es s es proxi mal endobronchi al t umor ext ens i on and t he s t at us of t he cont ral at eral l ung. It s houl d be performed i n al l pat i ent s wi t h cent ral l y l ocat ed t umors and i n s el ect ed pat i ent s wi t h peri pheral t umors , es peci al l y i f t hey are pot ent i al s urgi cal candi dat es .
2. T ransthoracic needle biopsy, gui ded by radi ography or CT, i s oft en neces s ary for t he di agnos i s of peri pheral l es i ons . The fal s e-negat i ve rat e i s 15%.
3. T ransbronchial needle aspiration of medi as t i nal nodes may obvi at e t he need for more i nvas i ve procedures .
4. T horacotomy or mediastinoscopy i s requi red i n approxi mat el y 5%–10% of pat i ent s . Thes e i nvas i ve procedures are es peci al l y us eful i n t he di agnos i s of SCLC, whi ch grows cent ral l y i n t he medi as t i num rat her t han endobronchi al l y. Medi as t i nos copy or medi as t i not omy al s o can be us ed t o as s es s t he res ect abi l i t y of medi as t i nal and hi l ar nodes .
5. Video-assisted transthorascopic surgery (VAT S) i s l es s i nvas i ve t han t horacot omy. VATS i nvol ves t hree or four i nci s i ons (1–2 cm) i n t he ches t wal l , di aphragm, l ung parenchyma, and medi as t i nal s t ruct ures . Vi s i bl e l es i ons may be removed by bi ops y.
6. Node biopsy i s us ed t o eval uat e s us pi ci ous s upracl avi cul ar or neck l ymph nodes .
Pa g e 7 1 1
ABC Ambe r CHM Conve rte r Tria l ve rsion, http://w w w .proce sste x t.com/a bcchm.html
7. CT scans of t he ches t , l i ver, brai n, and adrenal gl ands can es t abl i s h t he extent of metastatic disease and can provi de i mport ant i nformat i on about medi as t i nal node i nvol vement and ches t wal l i nvas i on. P.139
8. Radionuclide bone scans may be us ed t o rul e out met as t at i c di s eas e.
9. MRI of t he ches t i s mos t us eful for eval uat i ng s pread t o cardi ovas cul ar organs (e.g., heart , aort a, s uperi or vena cava).
10. Positron emission tomography (PET ) scan i s now wi del y us ed i n t he i ni t i al s t agi ng of pat i ent s wi t h non-SCLC. It provi des bet t er i nformat i on regardi ng t he i nvol vement of medi as t i nal nodes and can t herefore affect t he s t agi ng of pat i ent s . It al s o pl ays a rol e i n pos t t reat ment and fol l ow-up phas es of pat i ent care.
G. Prognosis
1. Non-SCLC. Key prognos t i c fact ors i ncl ude t he extent of tumor dissemination, performance status, and weight loss. o
o
a. Survi val rat es range from 40% t o 50% i n s t age I di s eas e and from 15% t o 30% i n s t age II di s eas e.
Pa g e 7 1 2
ABC Ambe r CHM Conve rte r Tria l ve rsion, http://w w w .proce sste x t.com/a bcchm.html o
o
b. Hi gher rat es are obs erved i n pat i ent s who undergo ext ens i ve medi as t i nal node s ampl i ng.
o
o
c. Pat i ent s wi t h met as t at i c di s eas e have medi an s urvi val s of l es s t han 1 year.
2. Limited SCLC. Long-t erm s urvi val rat es i n pat i ent s wi t h limited SCLC vary from 10% t o 50% aft er combi ned chemot herapy and radi at i on t herapy. Ext ens i ve di s eas e i s not curabl e, however, and medi an s urvi val does not exceed 1 year, des pi t e i ni t i al res pons i venes s t o chemot herapy.
H. Staging For ful l s t agi ng i nformat i on, pl eas e s ee t he mos t recent edi t i on of t he AJCC Canc er St agi ng Handbook.
1. Non-SCLC. Becaus e surgery is the only curative option, t he goal s of s t agi ng are t o eval uat e t umor res ect abi l i t y and t o det ermi ne t he ext ent of di s eas e s pread beyond t he hemi t horax.
2. SCLC o
o
a. St agi ng ai ms t o di fferent i at e bet ween l i mi t ed and ext ens i ve di s eas e.
(1) Limited disease i s t hat whi ch i s confi ned t o one hemi t horax; t he ent i re t umor can be encompas s ed i n a s i ngl e radi at i on t herapy
Pa g e 7 1 3
ABC Ambe r CHM Conve rte r Tria l ve rsion, http://w w w .proce sste x t.com/a bcchm.html
port al .
(2) Extensive disease i s charact eri zed by di s t ant met as t as es , s upracl avi cul ar node i nvol vement , or pl eural effus i ons .
o
o
b. St agi ng procedures i ncl ude ches t radi ography; CT of t he ches t , l i ver, and brai n, fol l owed by bone s cans wi t h pl ai n fi l m radi ography of s us pi ci ous areas ; and bi l at eral bone marrow bi ops i es or as pi rat es .
I. Therapy
1. Non-SCLC o
o
a. Pretreatment considerations
(1) A t umor i s res ect abl e i f thoracotomy demons t rat es t hat al l of t he t umor can be compl et el y exci s ed wi t h pat hol ogi cal l y negat i ve margi ns .
(2) Medi as t i nos copy or medi as t i not omy s houl d be us ed onl y i f pos i t i ve fi ndi ngs wi l l prevent a curat i ve t horacot omy.
o
o
b. T herapeutic modalities
Pa g e 7 1 4
ABC Ambe r CHM Conve rte r Tria l ve rsion, http://w w w .proce sste x t.com/a bcchm.html
(1) Surgery
(a) Operable tumors. If cardi opul monary s t at us i s compat i bl e wi t h l ung res ect i on, s urgi cal opt i ons i ncl ude l obect omy or pneumonect omy, dependi ng on t he ext ent of di s eas e. The s urgeon s houl d ai m t o remove t he t umor compl et el y wi t h adequat e margi ns of res ect i on whi l e cons ervi ng as much normal l ung t i s s ue as pos s i bl e.
(b) Advanced local cancer. Sel ect ed pat i ent s may undergo s urgi cal res ect i on wi t h or wi t hout ches t i rradi at i on.
(i) Pat i ent s wi t h s mal l t umors i n t he i ps i l at eral medi as t i nal nodes or l arge pri mary t umors wi t hout medi as t i nal node i nvol vement have 5-year s urvi val rat es as hi gh as 35% wi t h a combi nat i on of s urgery and radi at i on t herapy.
(ii) Pancoast' s tumor or t umors of t he s uperi or s ul cus al s o may be cured by a combi nat i on of s urgery and radi at i on t herapy.
(2) Radiation therapy
Pa g e 7 1 5
ABC Ambe r CHM Conve rte r Tria l ve rsion, http://w w w .proce sste x t.com/a bcchm.html
(a) Radi at i on t herapy reduces l ocal recurrences i n operabl e s t age II t umors .
(b) Pat i ent s who cannot t ol erat e s urgery becaus e of i ns uffi ci ent cardi opul monary res erve s houl d recei ve ches t i rradi at i on. Survi val rat es at 5 years range from 5% t o 20%.
(c) Definitive radiation therapy i s oft en us ed i n pat i ent s wi t h unresectable but localized cancers, wi t h l es s t han 5% l ong-t erm s urvi val . Palliative radiation therapy can rel i eve t he s ympt oms of pai n, hemopt ys i s , s uperi or vena cava s yndrome, or pneumoni t i s t hat are as s oci at ed wi t h obs t ruct i ve l es i ons . Sel ect ed pat i ent s wi t h met as t at i c di s eas e may al s o be t reat ed wi t h pal l i at i ve radi at i on.
(3) Chemotherapy
(a) An i nt ernat i onal s t udy of more t han 1800 pat i ent s wi t h non-SCLC who were t reat ed wi t h up t o 4 cycl es of a pl at i num-bas ed combi nat i on chemot herapy aft er res ect i on of P.140
t hei r pri mary t umors s howed t hat adjuvant chemot herapy i n pat i ent s wi t h res ect ed t umors
Pa g e 7 1 6
ABC Ambe r CHM Conve rte r Tria l ve rsion, http://w w w .proce sste x t.com/a bcchm.html
was benefi ci al . Res ul t s of t hi s s t udy s howed a 5% i mprovement i n overal l s urvi val . Several s t udi es have s hown t hat neoadjuvant chemot herapy i mproves s urvi val i n s t age III pat i ent s . In one s t udy, res earchers found t hat t wo cours es of ci s pl at i n and vi nbl as t i ne before radi at i on t herapy i n s t age III unres ect abl e pat i ent s was s uperi or t o radi at i on al one. The s urvi val rat es were 55% (vers us 40%) at 1 year, 26% (vers us 13%) at 2 years , and 23% (vers us 11%) at 3 years . In addi t i on, s everal s t udi es have s hown t hat concurrent chemoradi at i on t herapy i s s uperi or t o s equent i al t reat ment .
(b) Combi nat i on chemot herapy has a res pons e rat e of approxi mat el y 10%–30% in patients with metastatic non-SCLC. Res pons e rat es doubl e i n ambul at ory pat i ent s who have not l os t wei ght . The mos t commonl y us ed drug combi nat i ons i ncl ude:
(i) Carbopl at i n and pacl i t axel
(ii) Ci s pl at i n and et opos i de
(iii) Gemci t abi ne and ci s pl at i n or carbopl at i n
(iv) Iri not ecan and ci s pl at i n
Pa g e 7 1 7
ABC Ambe r CHM Conve rte r Tria l ve rsion, http://w w w .proce sste x t.com/a bcchm.html
(v) Vi norel bi ne and ci s pl at i n
(vi) Docet axel and carbopl at i n
2. SCLC o
o
a. Pretreatment considerations. W i t h limited disease, t he hi ghes t l ong-t erm s urvi val rat es (10%–50%) are obt ai ned wi t h concurrent radiation therapy and chemotherapy. Hyperfract i onat ed radi at i on t herapy combi ned wi t h chemot herapy may i mprove t reat ment out come.
o
o
b. T herapeutic modalities
(1) Chemotherapy (mai ns t ay of t reat ment ). For extensive disease, s i ngl e-agent or combination chemotherapy i s s t andard.
(a) Commonly used regimens i ncl ude et opos i de pl us ci s pl at i n or carbopl at i n.
(b) Ot her act i ve drugs i ncl ude i fos fami de, vi ncri s t i ne, pacl i t axel , t opot ecan, and i ri not ecan. Combi nat i on chemot herapy wi t h ci s pl at i n and i ri not ecan has s hown promi s e i n a recent phas e II t ri al .
Pa g e 7 1 8
ABC Ambe r CHM Conve rte r Tria l ve rsion, http://w w w .proce sste x t.com/a bcchm.html
(2) Radiation therapy. Prophylactic cranial irradiation s houl d be cons i dered i n res pons i ve pat i ent s wi t h l i mi t ed di s eas e. Radi at i on t herapy has onl y pal l i at i ve benefi t i n pat i ent s whos e condi t i ons have fai l ed t o res pond t o chemot herapy or who have brai n met as t as es .
J. Mesothelioma Thi s neopl as m ari s es from t he mes ot hel i al l i ni ng of t he pl eural and peri t oneal cavi t i es . It has a very poor prognos i s , wi t h a medi an s urvi val of 6–18 mont hs , and s o far no effect i ve t reat ment s have been i dent i fi ed. A very s el ect group of pat i ent s i s s urgi cal l y t reat ed, and even t hen t he overal l s urvi val i s poor. There i s a hi gh as s oci at i on bet ween as bes t os expos ure and t he devel opment of t hi s di s eas e. Smoki ng and as bes t os expos ure i ncreas es t he ri s k of devel opment of l ung cancer by 90-fol d. Several chemot herapeut i c agent s have been eval uat ed for t he t reat ment of t hi s condi t i on. Thes e i ncl ude Doxi l , gemci t abi ne, ci s pl at i n, and carbopl at i n. Mos t recent l y, mul t i t arget ed ant i fol at e pemet rexed (Al i mt a) has recei ved much at t ent i on as a pos s i bl e new and effect i ve agent i n t he t reat ment of mes ot hel i omas . The combi nat i on of t hi s agent wi t h a pl at i num agent i s s uperi or t o t he us e of pl at i num al one.
VI. Breast Cancer A. Incidence Breas t carci noma has i ncreas ed i n frequency over t he l as t s everal decades and onl y recent l y t he rat e decreas ed by approxi mat el y 7%, probabl y due t o decreas e i n t he us e of HRT i n pos t menopaus al women. It i s t he mos t common cancer di agnos ed i n women and s econd t o l ung cancer as caus e of deat h from cancer. The annual i nci dence rat e of breas t cancer i n t he Uni t ed St at es i n 2006 was 212,900. As many as one i n ei ght women wi l l devel op breas t cancer duri ng t hei r l i fet i me.
B. Risk factors Pa g e 7 1 9
ABC Ambe r CHM Conve rte r Tria l ve rsion, http://w w w .proce sste x t.com/a bcchm.html
1. Family history. At l eas t t wo heredi t ary pat t erns are charact eri s t i c of breas t cancer. o
o
a. Familial aggregation i s as s oci at ed wi t h a modes t i ncreas e i n ri s k and i s rel at i vel y common. The ri s k i s markedl y i ncreas ed among fi rs t -degree rel at i ves . P.141
o
o
b. T rue genetic pattern. Li nkage t o a s peci fi c gene wi t h hi gh penet rance account s for fewer t han 5% of al l cas es of breas t cancer. Cancer t ends t o occur at a younger age and i s more l i kel y t o be bi l at eral . Mul t i pl e fami l y members over t hree or more generat i ons are affect ed.
(1) p53 i s a t umor s uppres s or gene t hat appears i n carri ers and pat i ent s i n Li -Fraumeni fami l i es . The gene i s l ocat ed on chromos ome 17.
(2) Mut at i ons i n t he BRCA1 gene, l ocat ed on chromos ome 17q, are s een i n s ome pat i ent s wi t h a fami l y hi s t ory of breas t and ovari an cancer. Mut at i ons i n t he BRCA2 gene al s o confer i ncreas ed ri s k of breas t cancer.
Pa g e 7 2 0
ABC Ambe r CHM Conve rte r Tria l ve rsion, http://w w w .proce sste x t.com/a bcchm.html
2. Early menarche
3. Late menopause
4. Nulliparity or a first pregnancy after age 30 years
5. Prior history of invasive or noninvasive breast cancers [i nt raduct al or l obul ar carci noma i n s i t u (CIS)]
6. Age
7. Hormone replacement therapy (HRT ) i s as s oci at ed wi t h a modes t i ncreas e i n t he ri s k of breas t cancer.
8. Prolonged use of oral contraceptives before t he fi rs t pregnancy
9. Dietary factors [e.g., hi gh-fat di et (unproven), al cohol cons umpt i on]
10. Early exposure to ionizing radiation
C. Screening Rout i ne breas t s el f-exami nat i on, cl i ni cal breas t exami nat i on, and s creeni ng mammography have l ed t o earl i er det ect i on of breas t cancers . (For ful l s t agi ng i nformat i on, pl eas e s ee t he mos t recent edi t i on of t he AJCC Cancer St agi ng Handbook.)
1. Breast self-examination. Al l women s houl d be t aug
Pa g e 7 2 1
ABC Ambe r CHM Conve rte r Tria l ve rsion, http://w w w .proce sste x t.com/a bcchm.html
ht t he t echni que of breas t s el f-exami nat i on. Bet ween t he ages of 20 and 39 years , women s houl d undergo cl i ni cal breas t exami nat i on every 3 years and annual l y aft er age 40 years . Such exami nat i ons are bes t performed mont hl y, 5–7 days aft er t he mens t rual p eri od, when breas t s wel l i ng and fi brocys t i c changes are l es s l i kel y t o i nt erfere wi t h t he det ect i on of a l ump or mas s . More t han 80% of breas t cancers occur as pai nl es s mas s es .
2. Mammography. Al l women bet ween 35 and 40 years of age s houl d have a bas el i ne mammogram. o
o
(1) Dependi ng on t he pres ence of known ri s k fact ors , pat i ent s s houl d undergo mammography ei t her yearl y or every ot her year bet ween age 40 and 50 years , and yearl y aft er age 50 years .
o
o
(2) W omen wi t h ri s k fact ors for breas t carci noma s houl d have a yearl y mammogram at an earl i er age.
3. High-risk women. Al t hough t here are not s uffi ci ent dat a t o recommend a s peci fi c s urvei l l ance s t rat egy for hi gh-ri s k women, t he ACS gui del i nes recommend earl i er i ni t i at i on of s creeni ng, s creeni ng at s hort er i nt erval s and s creeni ng wi t h addi t i onal modal i t i es s uch as ul t ras ound or MRI. Femal e fami l y members at hi gh ri s k for breas t cancer, es peci al l y wi t h BRCA1 and BRCA2 mut at i ons , are advi s ed t o undergo mammographi c s creeni ng from t he age of 25 or 5 years pri or t o t he i ndex cas e.
D. Pathology
Pa g e 7 2 2
ABC Ambe r CHM Conve rte r Tria l ve rsion, http://w w w .proce sste x t.com/a bcchm.html
Breas t cancers are adenocarci nomas .
1. Classification. Mos t breas t cancers are i rregul ar mas s es or areas as s oci at ed wi t h mi crocal ci fi cat i ons . o
o
a. Infiltrating ductal carcinomas (70% of breas t cancers ) are charact eri zed by nes t s and cords of t umor cel l s s urrounded by a dens e col l agenous s t roma.
o
o
b. Infiltrating lobular carcinomas (10%–15% of breas t cancers ) are charact eri zed by abnormal cel l s cont ai ned wi t hi n a l obul e of t he breas t , but t hey haven't i nvaded t he s urroundi ng breas t t i s s ue.
o
o
c. Medullary carcinomas (5%–10% of breas t cancers ) are l arge bul ky t umors t hat have a l owgrade i nfi l t rat i ng t endency and are s urrounded by l ymphocyt es .
o
o
d. Inflammatory carcinomas (5% of breas t cancers ) i nvade t he dermal l ymphat i cs and caus e s ki n rednes s , i ndurat i on, warmt h, and an erys i pel oi d margi n.
o
o
e. Papillary carcinomas (1% of breas t cancers ) are l ow-grade, noni nvas i ve i nt raduct al l es i ons .
P.142
Pa g e 7 2 3
ABC Ambe r CHM Conve rte r Tria l ve rsion, http://w w w .proce sste x t.com/a bcchm.html
2. Hormone receptor status. Breas t cancer can be cl as s i fi ed accordi ng t o t he pres ence or abs ence of ERs and proges t erone recept ors (PGRs ), whi ch are cel l ul ar prot ei ns found by i mmunohi s t ochemi cal s t ai ni ng of t umor t i s s ue. The 20%–30% of breas t cancer pat i ent s wi t h HER2/neu oncogene gene ampl i fi cat i on and overexpres s i on have more aggres s i ve cancers and may have great er drug res i s t ance. o
o
a. ER-positive t umors are more common i n pos t menopaus al pat i ent s , whereas ER-negative t umors are more common i n premenopaus al pat i ent s .
o
o
b. Approxi mat el y 60% of pri mary breas t cancers have det ect abl e ERs .
E. Clinical features
1. Local symptoms i ncl ude a breas t l ump, s ki n t hi ckeni ng or al t erat i on, peau d'orange, di mpl i ng of t he s ki n, ni ppl e i nvers i on or crus t i ng (Paget di s eas e), and uni l at eral ni ppl e di s charge.
2. Pat i ent s may al s o pres ent wi t h s i gns and s ympt oms of metastatic disease.
F. Diagnosis
1. Pretreatment evaluation o
Pa g e 7 2 4
ABC Ambe r CHM Conve rte r Tria l ve rsion, http://w w w .proce sste x t.com/a bcchm.html
o
a. In addi t i on t o a medical history, physical examination, chest radiograph, and routine laboratory tests (e.g., bl ood count , l i ver and renal funct i on val ues , and s erum cal ci um), al l women wi t h newl y di agnos ed breas t cancer s houl d have a mammogram to detect multicentricity or bilateral involvement.
o
o
b. Radiologic tests may i ncl ude a bone s can and, i n advanced breas t cancer, a CT or MRI s can of t he l i ver, as di ct at ed by cl i ni cal pres ent at i on. If a bone s can s hows evi dence of met as t at i c di s eas e and radi ographs are negat i ve, a CT s can or MRI s can of t he bone or a bone bi ops y s houl d be performed t o det ermi ne t he correct t herapy.
o
o
c. Excisional biopsy t o obt ai n pat hol ogi c di agnos i s of breas t cancer i s i ndi cat ed onl y for pat i ent s who are unabl e t o undergo a core needle biopsy, whi ch i s us ual l y done under ul t ras ound or mammographi c gui dance (s t ereot act i c bi ops y).
o
o
d. Pat i ent s wi t h met as t at i c di s eas e may have el evat i on i n CA 15–3 or CEA tumor markers. The val ue of t hes e markers t o moni t or di s eas e s t at us remai ns cont rovers i al .
G. Prognosis Approxi mat el y 50% of pat i ent s wi t h operabl e breas t cancer devel op recurrent di s eas e unl es s t hey recei ve adjuvant chemot herapy or hormone t herapy. Prognos t i c fact ors i ncl ude:
Pa g e 7 2 5
ABC Ambe r CHM Conve rte r Tria l ve rsion, http://w w w .proce sste x t.com/a bcchm.html
1. T umor size. Thi s fact or i s t he mos t i mport ant predi ct or of recurrence and s urvi val . Tumors l arger t han 5 cm are as s oci at ed wi t h a decreas ed s urvi val rat e and an i ncreas ed ri s k of recurrence.
2. Axillary node status. Sevent y percent of pat i ent s wi t h negat i ve nodes are di s eas e-free at 10 years . Thi s fi gure decl i nes t o 40% of pat i ent s wi t h no more t han t hree pos i t i ve nodes and 15%–25% of pat i ent s wi t h four or more pos i t i ve nodes .
3. Histopathology. Poorl y di fferent i at ed t umors wi t h hi gh nucl ear grades have hi gher recurrence rat es .
4. Hormone receptor status. Of pri mary breas t cancers , 60%–70% expres s ERs , and 40%–50% expres s PGRs . Pat i ent s wi t h hormone recept or–pos i t i ve t umors have l ower rat es of recurrence and prol onged s urvi val rat es compared wi t h t hos e wi t h recept or-negat i ve t umors .
5. S-phase fraction and DNA index. The S-phas e fract i on (i .e., t he percent age of t umor cel l s i n t he S phas e of t he cel l cycl e) i s proport i onal t o t he t umor growt h rat e. Pat i ent s wi t h aneupl oi d t umors or hi gh S-phas e fract i ons , as det ermi ned by fl ow cyt omet ry, have a poor prognos i s compared wi t h t hos e wi t h s l ow-growi ng di pl oi d t umors .
6. Oncogenic expression. Expres s i on of t he HER-2/neu oncogene i s as s oci at ed wi t h a poorer prognos i s .
Pa g e 7 2 6
ABC Ambe r CHM Conve rte r Tria l ve rsion, http://w w w .proce sste x t.com/a bcchm.html
7. Other prognostic factors. In s ome s t udi es , t he fol l owi ng prognos t i c fact ors have been as s oci at ed wi t h a poorer prognos i s : cat heps i n-D, mut at ed p53, epi dermal growt h fact or recept or, and t umor growt h fact or-β (TGF-β).
H. Staging Please refer to the AJCC-X6 staging classification.
I. Therapy The pri mary goal of l ocal t herapy i s t o provi de opt i mal cont rol of t he di s eas e i n t he breas t and regi onal t i s s ues whi l e provi di ng t he bes t pos s i bl e cos met i c res ul t . Sys t emi c t herapy P.143 s houl d be gi ven t o pat i ent s at hi gh ri s k for met as t at i c di s eas e t o eradi cat e mi cromet as t as es . Pat i ent s s houl d be s een by a medi cal oncol ogi s t , radi at i on t herapi s t , and s urgeon t o det ermi ne t he bes t cours e of t reat ment , whi ch may i ncl ude s urgery, radi at i on t herapy, adjuvant chemot herapy, adjuvant endocri ne t herapy, or a combi nat i on of modal i t i es .
1. Surgery o
o
a. The opt i mal s urgi cal approach i s det ermi ned by t he fol l owi ng fact ors .
(1) Di s eas e s t age
(2) Tumor s i ze
(3) Tumor l ocat i on
Pa g e 7 2 7
ABC Ambe r CHM Conve rte r Tria l ve rsion, http://w w w .proce sste x t.com/a bcchm.html
(4) Breas t s i ze and confi gurat i on
(5) Number of t umors i n t he breas t
(6) Avai l abl e s urgi cal and radi ot herapeut i c t echni ques
(7) Pat i ent preference concerni ng breas t cons ervat i on
o
o
b. Procedures. In t he pas t , mos t pat i ent s underwent modi fi ed radi cal mas t ect omy. Recent dat a i ndi cat e t hat breas t cons ervat i on procedures s uch as l umpect omy pl us radi at i on al l ow adequat e l ocal cont rol of t he t umor and i mprove cos met i c out come i n s el ect ed cas es ; however, not al l pat i ent s are s ui t abl e candi dat es for breas t cons ervat i on (Tabl e 4-14).
(1) Modified radical mastectomy ent ai l s removal of t he breas t and axi l l ary cont ent s wi t h pres ervat i on of t he pect oral mus cl es . Pat i ent s may undergo breas t recons t ruct i on duri ng s urgery or at a l at er t i me.
(2) Partial mastectomy, or lumpectomy, i nvol ves exci s i on of t he t umor and an adjacent ri m of normal t i s s ue.
(a) Sentinel node biopsy (SNB) has
Pa g e 7 2 8
ABC Ambe r CHM Conve rte r Tria l ve rsion, http://w w w .proce sste x t.com/a bcchm.html
emerged as a us eful means for l i mi t i ng axi l l ary l ymph node di s s ect i on. It i s a mi ni mal l y i nvas i ve procedure where radi oact i ve dye and bl ue dye are i nject ed around t he t umor bed and t he i dent i fi ed s ent i nel l ymph node(s ) are exci s ed. Furt her axi l l ary di s s ect i on i s done onl y i f t he SNB i s pos i t i ve.
(b) Level I or II axillary dissection i s performed for adequat e s t agi ng and l ocal cont rol onl y i f t he SNB i s pos i t i ve. Level III dissection s houl d be cons i dered i n pat i ent s wi t h cl i ni cal l y pos i t i ve l ymph nodes .
2. Radiation therapy o
o
a. Patients treated with lumpectomy and SNB or full axillary dissection s houl d recei ve defi ni t i ve radi at i on t herapy t o t he whol e breas t wi t h a boos t t o t he t umor bed. Pat i ent s wi t h more t han t hree pos i t i ve l ymph nodes or ext ranodal ext ens i on of t he t umor s houl d al s o recei ve radi at i on t o t he l ymphat i c bed.
o
o
b. Patients undergoing mastectomy s houl d be cons i dered for pos t operat i ve radi at i on i f t hey have any of t he fol l owi ng ri s k fact ors for l ocal recurrence.
(1) Pri mary t umor >4 cm
Pa g e 7 2 9
ABC Ambe r CHM Conve rte r Tria l ve rsion, http://w w w .proce sste x t.com/a bcchm.html
(2) More t han t hree pos i t i ve axi l l ary nodes
(3) Tumors i nvol vi ng t he margi n of s urgi cal res ect i on, i nvas i on of pect oral fas ci a or mus cl e, or ext ranodal ext ens i on i nt o t he axi l l ary fat
o
o
c. In patients at high risk for distant metastases, radi at i on t herapy can be gi ven concurrent l y or del ayed unt i l t he compl et i on of adjuvant chemot herapy. The ri s k of i ps i l at eral arm l ymphedema i s i ncreas ed by pos t operat i ve axi l l ary radi at i on.
TABLE 4.15 Contraindications for Breast Conservation Surgery Lar ge tu mo r in a sm al l bre as t (i n cre as e s
Pa g e 7 3 0
ABC Ambe r CHM Conve rte r Tria l ve rsion, http://w w w .proce sste x t.com/a bcchm.html
lik el i ho od of po or cos me tic res ul t s) Su bar eol ar pri ma ry tu mo rs Mor e t ha n on e tu mo r in t he bre as t
Pa g e 7 3 1
ABC Ambe r CHM Conve rte r Tria l ve rsion, http://w w w .proce sste x t.com/a bcchm.html
Co nt r ai n di c at i ons to rad i at i on t he rap y Ad van ced di s eas e (i .e ., bey on d sta ge II) L arg e are as of i nt r
Pa g e 7 3 2
ABC Ambe r CHM Conve rte r Tria l ve rsion, http://w w w .proce sste x t.com/a bcchm.html
ad uct al di s eas e or mi c roc al ci fi c at i ons T um ors wi t h an ext ens i ve i nt r ad uct al co mp on ent (i .e ., 25 % of
Pa g e 7 3 3
ABC Ambe r CHM Conve rte r Tria l ve rsion, http://w w w .proce sste x t.com/a bcchm.html
t he pri ma ry tu mo r is in sit u an d t he re is at l ea st on e foc us of bre as t can cer t ha t is in sit u in nor ma
Pa g e 7 3 4
ABC Ambe r CHM Conve rte r Tria l ve rsion, http://w w w .proce sste x t.com/a bcchm.html
l bre as t tis s ue an d is s ep ara te fro m t he bre as t pri ma ry) P.144
3. Adjuvant chemotherapy. Thi s t echni que del ays or prevent s recurrence and i mproves s urvi val i n pat i ent s wi t h pos i t i ve axi l l ary nodes as wel l as i n s ome pat i ent s wi t h negat i ve axi l l ary nodes . Premenopaus al pat i ent s wi t h pos i t i ve axi l l ary nodes are mos t l i kel y t o benefi t from chemot herapy; s uch pat i ent s experi ence a 25%–30% reduct i on i n mort al i t y. o
o
a. Combination chemotherapy. Thi s approach i s commonl y us ed i n t he hi gh-ri s k adjuvant s et t i ng,
Pa g e 7 3 5
ABC Ambe r CHM Conve rte r Tria l ve rsion, http://w w w .proce sste x t.com/a bcchm.html
and 4–6 mont hs of t reat ment (4–8 cycl es of combi nat i on chemot herapy every 2–3 weeks ) i s us ual l y s uffi ci ent t o obt ai n maxi mum benefi t . In pat i ent s wi t h met as t at i c breas t cancer, t he us e of effect i ve s i ngl e agent s may be as effect i ve as t he us e of more act i ve but more t oxi c combi nat i on chemot herapy regi mens . Hi gh-dos e t herapy wi t h aut ol ogous s t em cel l res cue, whi ch does not i mprove t reat ment out comes i n women wi t h met as t at i c di s eas e, i s not s t andard t herapy i n t he hi gh-ri s k s et t i ng. Four t o ei ght cycl es of t herapy (dependi ng on t he regi men empl oyed) over 3 t o 6 mont hs are as effect i ve as l onger t reat ment peri ods . o
o
b. Drug regimens. Maxi mal l y t ol erat ed dos es s houl d be us ed unl es s s i gni fi cant t oxi ci t y devel ops .
(1) The mos t commonl y us ed adjuvant t herapy regi mens cont ai n s ome combi nat i on of t he agent s doxorubi ci n, cycl ophos phami de, met hot rexat e, and 5-FU. Taxanes are now i mport ant component s of adjuvant t herapy i n pat i ent s wi t h node-pos i t i ve di s eas e. The ant i -HER2-neu monocl onal ant i body, t ras t uzumab (Hercept i n) was recent l y FDA-approved for us e i n t he adjuvant s et t i ng for pat i ent s wi t h HER2/neu-pos i t i ve cancers .
(2) Pat i ent s at hi gher ri s k for devel opi ng recurrent or met as t at i c di s eas e are t ypi cal l y offered doxorubi ci n- or t axane-cont ai ni ng regi mens (6–8 cycl es ).
Pa g e 7 3 6
ABC Ambe r CHM Conve rte r Tria l ve rsion, http://w w w .proce sste x t.com/a bcchm.html
(3) Ot her active chemotherapeutic agents may be us ed t o pal l i at e pat i ent s wi t h met as t at i c di s eas e. Thes e i ncl ude t he t axanes [pacl i t axel (Taxol ) or docet axel (Taxot ere)], t he ant i -HER2/neu ant i body t ras t uzumab (Hercept i n), vi norel bi ne, gemci t abi ne, or capeci t abi ne as s i ngl e agent s or i n combi nat i on.
4. Adjuvant endocrine therapy o
o
a. Hormonal therapy. Bot h pre- and pos t menopaus al pat i ent s wi t h pos i t i ve hormone recept ors (ERs and/or PGRs ) are offered endocri ne t herapy aft er compl et i on of chemot herapy.
(1) The s el ect i ve es t rogen recept or modul at or (SERM) t amoxi fen and hormonal agent s cal l ed aromat as e i nhi bi t ors (AIs ; anas t rozol e, l et rozol e, and exemes t ane) are t he preferred agent s . AIs are FDA approved for p os t menopaus al women onl y and can be us ed up front for 5 years or s equent i al l y aft er 2–5 years of t amoxi fen t herapy for a t ot al of 5–10 years of adjuvant hormonal t herapy. Tamoxi fen and AIs del ay recurrence and i mprove s urvi val .
(2) Tamoxi fen adjuvant hormonal t herapy for 5 years s omet i mes i n combi nat i on wi t h l ut ei ni zi ng hormone–rel eas i ng hormone
Pa g e 7 3 7
ABC Ambe r CHM Conve rte r Tria l ve rsion, http://w w w .proce sste x t.com/a bcchm.html
(LH-RH) agoni s t s or oophorect omy are i ndi cat ed i n premenopaus al pat i ent s wi t h ER/PGR-pos i t i ve t umors .
(3) Pat i ent s wi t h ER-negat i ve t umors exhi bi t l i t t l e or no res pons e t o hormonal t herapy and t herefore i t i s not i ndi cat ed i n ei t her adjuvant or met as t at i c s et t i ng.
o
o
b. Hormonal therapy for metastatic breast cancer
(1) Hormonal t herapy i s appropri at e for pat i ent s wi t h s ubcut aneous met as t as es , l ymph node i nvol vement , pl eural effus i ons , bone met as t as es , and nonl ymphangi t i c l ung met as t as es . Mos t pat i ent s wi t h l i ver met as t as es , l ymphangi t i c di s eas e of t he l ung, peri cardi al met as t as es , or ot her pot ent i al l y l i fe-t hreat eni ng met as t as es are t reat ed i ni t i al l y wi t h chemot herapy t o obt ai n a rapi d res pons e, but s uch t reat ment deci s i ons mus t be cus t omi zed.
(1) Pat i ent s wi t h ER-pos i t i ve pri mary t umors exhi bi t res pons e rat es of at l eas t 30% t o hormone t herapy. If t he t umor cont ai ns bot h pos i t i ve ERs and PGRs , res pons e rat es are hi gher.
(3) Pos t menopaus al pat i ent s whos e hormone recept or s t at us i s unknown may res pond t o
Pa g e 7 3 8
ABC Ambe r CHM Conve rte r Tria l ve rsion, http://w w w .proce sste x t.com/a bcchm.html
hormone t herapy.
(4) Pat i ent s wi t h a previ ous res pons e t o hormonal t herapy may res pond t o di s cont i nuat i on of t he ori gi nal agent and s ubs t i t ut i on of a s econd agent .
(5) Ot her hormonal t herapi es /mani pul at i ons i ncl ude, ful ves t rant (Fas l odex), anas t rozol e (Ari mi dex), Let rozol e (Femara), exemes t ane (Aromas i n), meges t rol (Megace), fl uoxymes t erone (Hal ot es t i n), and l LH-RH ant agoni s t s .
o
o
c. The res ul t s of s t udi es of ovarian ablation by radiation, oophorectomy, or chemical ablation i n premenopaus al pat i ent s have been mi xed. Cert ai n s ubgroups may gai n l ong-t erm benefi t s .
P.145
5. Other types of breast cancer and treatment recommendations o
o
a. Intraductal breast cancer. Becaus e t he t umor i s noni nvas i ve (i .e., confi ned t o t he duct s ), careful pat hol ogi c revi ew can excl ude any ri s k of l ymph node i nvol vement or di s t ant met as t as es . The prognos i s i s excel l ent .
Pa g e 7 3 9
ABC Ambe r CHM Conve rte r Tria l ve rsion, http://w w w .proce sste x t.com/a bcchm.html
(1) Pat i ent s may be t reat ed by t ot al mas t ect omy or by l umpect omy fol l owed by radi at i on t o t he whol e breas t .
(2) Axi l l ary node di s s ect i on i s cont rovers i al ; mos t expert s bel i eve i t i s unneces s ary.
(3) Duct al carci noma i n s i t u (CIS) requi res defi ni t i ve s urgery. Tamoxi fen i s wi del y us ed i n t he pos t operat i ve s et t i ng.
o
o
b. Lobular CIS. Pat i ent s wi t h t hi s noni nvas i ve l es i on are at hi gher ri s k for devel opment of i nvas i ve cancer i n bot h breas t s . Treat ment opt i ons i ncl ude ri gorous obs ervat i on and fol l ow-up, and t amoxi fen prophyl axi s t herapy. Rarel y bi l at eral mas t ect omy i s performed for prophyl axi s .
o
o
c. Stage I and stage II disease. Mos t pat i ent s have t he opt i on of ei t her breas t cons ervat i on wi t h l umpect omy or modi fi ed radi cal mas t ect omy wi t h s ent i nel l ymph node bi ops y. Axi l l ary l ymph node di s s ect i on i s res erved for pat i ent s wi t h pos i t i ve s ent i nel l ymph node bi ops y. Pos t operat i ve radi at i on t herapy i s al s o avai l abl e for al l pat i ent s who had l umpect omy s urgery. Adjuvant t herapy s houl d be offered t o hi gh-ri s k s t age I pat i ent s and al l s t age II pat i ent s .
o
o
d. Stage III disease. Treat ment opt i ons are det ermi ned by t umor res ect abi l i t y.
Pa g e 7 4 0
ABC Ambe r CHM Conve rte r Tria l ve rsion, http://w w w .proce sste x t.com/a bcchm.html
(1) Patients with operable tumors are general l y t reat ed wi t h modi fi ed radi cal mas t ect omy and pos t operat i ve radi at i on t herapy. Thes e pat i ent s al s o recei ve preoperat i ve (neoadjuvant ) or pos t operat i ve adjuvant chemot herapy.
(2) Patients with inoperable stage III disease have a hi gh rat e of l ocal and di s t ant recurrence and poor s urvi val rat es .
(a) A combi ned-modal i t y approach i s requi red, us i ng s ys t emi c neoadjuvant chemot herapy i n addi t i on t o s urgery and radi at i on.
(b) In mos t cas es , aggres s i ve combi nat i on chemot herapy i s i ni t i at ed aft er bi ops y t o reduce t umor bul k, faci l i t at e l ocal t reat ment , and t reat di s t ant mi cromet as t as es .
VII. Gastrointestinal Cancers A. Carcinoma of the colon, rectum, and anus
1. Incidence. Cancers of t he col on and rect um occur i n 145,600 i ndi vi dual s annual l y i n t he Uni t ed St at es and account for 56,000 deat hs each year.
Pa g e 7 4 1
ABC Ambe r CHM Conve rte r Tria l ve rsion, http://w w w .proce sste x t.com/a bcchm.html
2. Etiology/ Risk Factors. Up t o 70% of pat i ent s have no i dent i fi abl e ri s k fact ors . However, genet i c and epi demi ol ogi c s t udi es have l i nked col on and rect al cancer t o t he fol l owi ng fact ors : o
o
a. Inherited predisposition. Fami l i al s yndromes s uch as adenomat ous pol ypos i s col i (APC), an aut os omal domi nant di s order caus ed by mut at i ons i n t he FAP gene on chromos ome 5, may l ead t o an i ncreas ed ri s k of col on cancer. In APC, cancers commonl y devel op i n adol es cence and young adul t hood, and t he i nci dence of col orect al neopl as ms i s nearl y 100% by age 50. Heredi t ary nonpol ypos i s col on cancer (HNPCC), whi ch i s al s o fami l i al , i s as s oci at ed wi t h a l ower but s i gni fi cant ri s k of cancer of t he col on and rect um. Mut at i ons i n t umor s uppres s or genes s uch as MCC, DCC, BRCA1, and p53 al s o confer hi gher ri s ks for col orect al neopl as ms .
o
o
b. Somatic mutation. The wel l -es t abl i s hed genet i c changes t hat accompany t he progres s i on from epi t hel i al hyperpl as i a t hrough pol yp format i on and event ual cancer devel opment form a cont i nuum. Thes e changes i ndi cat e an as s oci at i on among i nheri t ed predi s pos i t i ons (e.g., mut at ed p53) and acqui red mut at i ons i n ei t her t umor s uppres s or genes s uch as DCC or i n oncogenes s uch as ras .
o
o
c. Dietary influence. The hi gher i nci dence of col orect al neopl as ms i n i ndus t ri al i zed s oci et i es has l ed t o hypot hes es t hat hi gh-fat , l ow-fi ber di et s
Pa g e 7 4 2
ABC Ambe r CHM Conve rte r Tria l ve rsion, http://w w w .proce sste x t.com/a bcchm.html
t hat are defi ci ent i n cal ci um, s el eni um, fol at e, vi t ami ns D and E, and ot her t race el ement s predi s pos e i ndi vi dual s t o t he devel opment of col orect al neopl as ms . o
o
d. Preexisting inflammatory disease. Infl ammat ory bowel di s eas e, part i cul arl y ul cerat i ve col i t i s wi t h pancol i t i s , i s as s oci at ed wi t h a hi gher i nci dence of col orect al neopl as ms .
3. Screening o
o
a. Al l adul t s ol der t han 40 years wi t h s i gns or s ympt oms cons i s t ent wi t h col orect al neopl as ms s houl d undergo t es t i ng t o excl ude t he pres ence of a mas s l es i on i n t he col on or rect um. P.146
o
o
b. Al l average ri s k adul t s aged 50 years or ol der s houl d undergo one of t he fol l owi ng opt i ons : (i ) annual fecal occul t bl ood t es t (FOBT) or fecal i mmunochemi cal t es t (FIT), (i i ) fl exi bl e s i gmoi dos copy every 5 years , (i i i ) annual FOBT or FIT, pl us fl exi bl e s i gmoi dos copy, (i v) doubl e cont ras t bari um enema (DCBE) every 5 years , or (v) col onos copy every 10 years . Al l pos i t i ve t es t s s houl d be fol l owed up wi t h a col onos copy.
4. Pathology o
Pa g e 7 4 3
ABC Ambe r CHM Conve rte r Tria l ve rsion, http://w w w .proce sste x t.com/a bcchm.html
o
a. The l arge majori t y of col orect al neopl as ms are adenocarcinomas, and mos t are wel l or moderat el y di fferent i at ed. Poorl y di fferent i at ed neopl as ms are as s oci at ed wi t h poor prognos i s .
o
o
b. Squamous cell carcinomas can ari s e i n t he anus . Such neopl as ms di ffer from adenocarci nomas i n t erms of bi ol ogy and t herapy.
5. Clinical features o
o
a. Local symptoms i ncl ude a change i n bowel habi t s t hat may i nvol ve cons t i pat i on, di arrhea, and a change i n s t ool cal i ber, crampy abdomi nal pai n, and rect al bl eedi ng. Si gns of obs t ruct i on, perforat i on, or earl y s at i et y may occur. Tenes mus may be pres ent i n rect al pri mari es . Pat i ent s wi t h l ocal l y advanced rect al carci noma may pres ent wi t h s ympt oms of peri neal pai n.
o
o
b. Systemic manifestations are uncommon i n i ndi vi dual s wi t h l ocal i zed neopl as ms . W hen t he cancer has met as t as i zed t o di s t ant organs , t he mos t common mani fes t at i ons are:
(1) Anorexi a, fat i gue, and wei ght l os s
(2) Hepat omegal y, pai n, or fever due t o l i ver i nvol vement
Pa g e 7 4 4
ABC Ambe r CHM Conve rte r Tria l ve rsion, http://w w w .proce sste x t.com/a bcchm.html
6. Diagnosis o
o
a. Barium enema, flexible sigmoidoscopy, or colonoscopy are us ual l y requi red t o i dent i fy a s us pi ci ous mas s l es i on.
o
o
b. Biopsy of s us pi ci ous l es i ons i s needed t o es t abl i s h a di agnos i s .
o
o
c. W hen a di agnos i s of col orect al neopl as i a has been es t abl i s hed, addi t i onal s t udi es may be neces s ary bot h t o eval uat e t he ext ent of l ocal di s eas e and t o s creen for met as t at i c di s eas e.
(1) CT scans of the pelvis and endorectal ultrasound may be appropri at e for rect al neopl as ms .
(2) CT scans of the chest and abdomen can s creen for t he pres ence of i nt ra-abdomi nal (e.g., mes ent eri c or ret roperi t oneal l ymph node enl argement ), hepat i c, or pul monary met as t as es .
7. Prognosis. Five-year survival rate for stage 0–1 patients is >90%; for stage 2, 70%–85%; stage 3, 55%–70%; and stage 4, <10%. Cure rat es for l ymph node negat i ve T1–T3 l es i ons are 70%–90% and decreas e t o 35%–40% when regi onal l ymph nodes are i nvol ved. The cure rat es decreas e wi t h i ncreas i ng
Pa g e 7 4 5
ABC Ambe r CHM Conve rte r Tria l ve rsion, http://w w w .proce sste x t.com/a bcchm.html
numbers of i nvol ved regi onal l ymph nodes and i ncreas i ng T s i ze. o
o
a. Colorectal cancer. Key prognos t i c fact ors i ncl ude t he extent of tumor invasion, lymph node involvement, and the presence or absence of distant metastases. Mut at i ons i n p53 res ul t i n an advers e prognos i s , as does cl i ni cal pres ent at i on of bowel obs t ruct i on.
o
o
b. Anal cancer. Long-t erm s urvi val rat es i n pat i ent s wi t h localized squamous cell carcinoma of the anus are approxi mat el y 80% aft er combi ned chemot herapy and radi at i on t herapy. However, met as t at i c di s eas e i s not curabl e, des pi t e t he i ni t i al res pons i venes s of t hi s cancer t o chemot herapy.
8. Staging For ful l s t agi ng i nformat i on, pl eas e s ee t he mos t recent edi t i on of t he AJCC Cancer St agi ng Handbook.
9. T herapy o
o
a. Colorectal cancer
(1) Surgery
(a) Colon cancer. Treat ment of cancers above t he peri t oneal refl ect i on us ual l y
Pa g e 7 4 6
ABC Ambe r CHM Conve rte r Tria l ve rsion, http://w w w .proce sste x t.com/a bcchm.html
i nvol ves partial colectomy, unl es s t he ext ent of met as t at i c di s eas e and t he abs ence of col oni c obs t ruct i on neces s i t at e s ys t emi c chemot herapy [s ee VII A 9 a (3)]. Pri nci pl es of s urgi cal oncol ogy, i ncl udi ng t he es t abl i s hment of wi de s urgi cal margi ns , s houl d be fol l owed t o opt i mi ze pat i ent out come. In experi enced hands , l aparos copi c part i al col ect omy i s accept abl e.
(b) Rectal cancer. Treat ment i nvol ves ei t her abdomi noperi neal res ect i on or, i f t he l es i on i s i n t he hi gh rect um, l ow ant eri or res ect i on t hat pres erves s phi nct er funct i on and P.147
does not requi re col os t omy. Experi ment al approaches for l ow rect al t umors i ncl ude preoperat i ve chemot herapy and radi at i on fol l owed by s egment al rect um res ect i on wi t h format i on of a col oanal anas t omos i s .
(2) Radiation therapy
(a) Colon cancer. Pros pect i ve randomi zed t ri al s have not es t abl i s hed t he val ue of adjunct i ve radi at i on for enhancement of s urgi cal cures . Nevert hel es s , adjunct i ve radi at i on, frequent l y i n combi nat i on wi t h
Pa g e 7 4 7
ABC Ambe r CHM Conve rte r Tria l ve rsion, http://w w w .proce sste x t.com/a bcchm.html
chemot herapy, i s commonl y offered t o pat i ent s wi t h T 4 l es i ons or t hos e wi t h pos i t i ve operat i ve margi ns .
(b) Rectal cancer. Adjunctive radiation, in conjunction with chemotherapy, enhances cure rat es i n pat i ent s wi t h t rans mural or l ymph node–pos i t i ve rect al neopl as ms .
(3) Chemotherapy. 5-FU has l ong been t he mai ns t ay of chemot herapy for col orect al neopl as ms . Bi ochemi cal modul at i on of 5-FU t o enhance i t s mechani s m of act i on can be achi eved ei t her by admi ni s t eri ng t he agent by cont i nuous i nt ravenous i nfus i on, or by coadmi ni s t eri ng calcium leucovorin. Recent l y, anot her agent , irinotecan, has been approved for us e and appears t o have a degree of effi cacy s i mi l ar t o t hat of 5-FU. Anot her new agent , oxaliplatin, al s o s hows promi s e. Newl y avai l abl e oral formul at i ons bas ed on 5-FU s uch as capecitabine offer great er eas e of admi ni s t rat i on and cont i nuous expos ure t o t he drug, whi ch maxi mi zes ant i t umor propert i es .
(a) Adjuvant therapy for colon cancer. Surgery cures onl y 35%–40% of pat i ent s wi t h l ymph node–pos i t i ve (e.g., any T, N 1 or N 2 , M 0 ) col on cancer. Cure rat es i ncreas e t o approxi mat el y 50% when s uch pat i ent s are t reat ed for 6 mont hs wi t h 5-FU pl us cal ci um
Pa g e 7 4 8
ABC Ambe r CHM Conve rte r Tria l ve rsion, http://w w w .proce sste x t.com/a bcchm.html
l eucovori n. Cont i nuous i nfus i onal (CI) 5-FU regi mens are bei ng i ncreas i ngl y us ed bas ed on ongoi ng cl i ni cal dat a from s everal s t udi es , whi ch s how s uperi ori t y of t hi s t reat ment approach t o t he bol us admi ni s t rat i on.
(i) However, the role of adjuvant chemotherapy in the management of patients with lymph node–negative (e.g., T 3 N0 M 0 ) lesions remains controversial. Adjuvant chemot herapy i s not i ndi cat ed i n pat i ent s wi t h nont rans mural cancers i n t he abs ence of regi onal l ymph node i nvol vement . Recent prel i mi nary res ul t s from cl i ni cal t ri al s s ugges t a benefi t t o t he us e of chemot herapy i n t he adjuvant s et t i ng.
(ii) The effect i venes s of newer agent s s uch as i ri not ecan, oxal i pl at i n, or capeci t abi ne i s s l owl y emergi ng i n t he adjuvant t herapy s et t i ng. Combi nat i on regi mens s uch as FOLFOX4, whi ch i ncl ude oxal i pl at i n, 5-FU, and l eucovori n are reas onabl e al t ernat i ves t o 5-FU and l eucovori n regi mens for pat i ent s wi t h s t age 3 col on cancer. The Ameri can Soci et y of Cl i ni cal Oncol ogy panel recent l y concl uded t hat t he rout i ne us e of
Pa g e 7 4 9
ABC Ambe r CHM Conve rte r Tria l ve rsion, http://w w w .proce sste x t.com/a bcchm.html
adjuvant chemot herapy for pat i ent s wi t h s t age 2 di s eas e coul d not be recommended bas ed on t he revi ew of 37 randomi zed s t udi es t hat found no s t at i s t i cal l y s i gni fi cant s urvi val benefi t from pos t operat i ve chemot herapy.
(b) Adjuvant therapy for rectal cancer. The combi ned admi ni s t rat i on of radi at i on t herapy and 5-FU i mproves cure rat es . Typi cal l y, combi ned-modal i t y t herapy i s admi ni s t ered pos t operat i vel y, fol l owed by an addi t i onal 4 mont hs of chemot herapy al one. Excel l ent res ul t s are al s o obt ai ned when t he chemoradi at i on precedes s urgery.
(c) Pseudoadjuvant therapy following the resection of metastases. Rarel y, i ndi vi dual s pres ent wi t h met as t as es t hat are s ol i t ary or res t ri ct ed t o a s ol i t ary organ and are amenabl e t o ei t her res ect i on or s ome abl at i ve procedure s uch as cryos urgery. In pat i ent s who undergo hepat i c l obect omi es t o res ect met as t as es , t he combi ned us e of a s ys t emi c 5-FU–bas ed compound and i nt rahepat i c art eri al 5-fl uorodeoxyuri di ne (FUDR) i s as s oci at ed wi t h a s urvi val benefi t compared wi t h s urgery al one.
(d) Disseminated metastatic disease.
Pa g e 7 5 0
ABC Ambe r CHM Conve rte r Tria l ve rsion, http://w w w .proce sste x t.com/a bcchm.html
Mos t commonl y, met as t as es are di s s emi nat ed, and chemot herapy i s offered wi t h pal l i at i ve i nt ent . Bi ochemi cal l y modul at ed 5-FU (e.g., cont i nuous i nfus i on or 5-FU pl us cal ci um l eucovori n) i s t he s t andard of care for pat i ent s wi t h previ ous l y unt reat ed met as t at i c di s eas e. Chemot herapy wi t h oxal i pl at i n or i ri not ecan i s more act i ve t han 5-FU al one. Infus i onal 5-FU–bas ed s chedul es are bet t er t ol erat ed and are at l eas t as act i ve as bol us 5-FU–bas ed s chedul es and are emergi ng as t he preferred s chedul es of admi ni s t rat i on. P.148
(e) T he angiogenesis inhibitor bevacizumab (Avastin) i mproves bot h t he res pons e rat e and overal l s urvi val i n pat i ent s wi t h newl y di agnos ed met as t at i c col on cancer, i n conjunct i on wi t h cont emporary chemot herapy regi mens and i s FDA-approved for pat i ent s wi t h met as t at i c col orect al cancer.
(f) Cetuximab i s a chi meri zed IgG1 ant i body t o t he epi dermal growt h fact or (EGFR) and was recent l y approved by t he FDA for t he t reat ment of EGFR-pos i t i ve col orect al cancer t hat i s refract ory t o or
Pa g e 7 5 1
ABC Ambe r CHM Conve rte r Tria l ve rsion, http://w w w .proce sste x t.com/a bcchm.html
i nt ol erant of i ri not ecan chemot herapy.
(i) Capecitabine al one, or irinotecan in combination with 5-FU and calcium leucovorin, i mproves res pons e rat es and s urvi val . In met as t at i c di s eas e t hat i s refract ory t o chemot herapy wi t h 5-FU pl us cal ci um l eucovori n, i ri not ecan t reat ment i mproves res pons e rat es , t i me t o progres s i on, s urvi val , and qual i t y of l i fe i n compari s on wi t h ei t her no t herapy or cont i nuous -i nfus i on 5-FU. Oxaliplatin promi s es t o add anot her opt i on.
(ii) Ot her agent s t hat are i nfrequent l y us ed i ncl ude cont i nuous i nfus i ons of 5-FU, or s i ngl e-agent admi ni s t rat i on of mitomycin-C or nitrosoureas.
o
o
b. Anal cancer. Combined-modality chemotherapy and radiation therapy for localized cancers us ual l y i nvol ve 5-FU and mitomycin-C. The majori t y of pat i ent s wi t h l ocal i zed cancers are cured, and funct i on of t he anal s phi nct er i s pres erved.
B. Carcinoma of the pancreas
Pa g e 7 5 2
ABC Ambe r CHM Conve rte r Tria l ve rsion, http://w w w .proce sste x t.com/a bcchm.html
1. Incidence. Cancers of t he pancreas occur i n approxi mat el y 31,900 i ndi vi dual s annual l y i n t he Uni t ed St at es and account for more t han 31,300 deat hs each year.
2. Etiology. Al t hough s ci ent i s t s have found mut at i ons i n s ome put at i ve t umor s uppres s or genes , t he caus es of pancreat i c cancers remai n unknown. Nei t her pancreat i t i s nor di abet es i ncreas es t he ri s k of pancreat i c cancer. Caffei ne i nt ake remai ns cont rovers i al as a caus e of pancreat i c cancer. Caus es may i ncl ude: o
o
a. Cigarette smoking
o
o
b. Dietary carcinogens
o
o
c. Environmental carcinogens
3. Screening. There are no good s creeni ng t es t s for pancreat i c cancer. CA 19-9 i s el evat ed i n 70%–90% of pat i ent s wi t h pancreat i c cancer; however, i t i s not us eful as a s creeni ng t es t becaus e of l ow s peci fi ci t y.
4. Pathology. Mos t pancreat i c neopl as ms are adenocarci nomas , and mos t are poorl y or moderat el y di fferent i at ed. o
o
a. Adenocarcinomas can ari s e i n t he head, body, or t ai l of t he pancreas .
o
Pa g e 7 5 3
ABC Ambe r CHM Conve rte r Tria l ve rsion, http://w w w .proce sste x t.com/a bcchm.html
o
b. Neuroendocrine tumors s uch as carci noi d t umors or i s l et cel l t umors are l es s common but may be as s oci at ed wi t h hepat i c met as t as es . In cont ras t t o exocri ne t umors , t hes e mal i gnanci es may have rel at i vel y i ndol ent nat ural hi s t ori es . Thes e t umors may produce hormones t hat i nduce carcinoid syndrome, whi ch i s charact eri zed by fl us hi ng, hypot ens i on, di arrhea, and val vul ar heart di s eas e.
5. Clinical features o
o
a. Local symptoms may i ncl ude pai nl es s jaundi ce, epi gas t ri c ful l nes s , or s ympt oms of gas t ri c out l et obs t ruct i on. W hen adjacent nerves are i nvol ved, pat i ent s commonl y compl ai n of abdomi nal pai n t hat radi at es i nt o t he back.
o
o
b. Systemic manifestations are common and may i ncl ude:
(1) Anorexi a, wei ght l os s
(2) Hepat omegal y, pai n, or fever at t ri but abl e t o l i ver i nvol vement
(3) Bl eedi ng (di s order) mani fes t ed by ei t her recurrent s uperfi ci al t hrombophl ebi t i s (Trous s eau's s yndrome), deep t hrombophl ebi t i s , or di s s emi nat ed
Pa g e 7 5 4
ABC Ambe r CHM Conve rte r Tria l ve rsion, http://w w w .proce sste x t.com/a bcchm.html
i nt ravas cul ar coagul at i on (DIC)
6. Diagnosis. Pancreat i c cancer i s not ori ous l y di ffi cul t t o di agnos e, and pat i ent s frequent l y pres ent when t he cancer i s not res ect abl e for cure. Frequent l y us ed di agnos t i c modal i t i es i ncl ude t he fol l owi ng: o
o
a. CT scanning may i dent i fy a s us pi ci ous mas s l es i on.
o
o
b. Biopsy of s us pi ci ous l es i ons i s needed t o es t abl i s h a di agnos i s . Thi s may be accompl i s hed by endos copi c ret rograde chol angi opancreat ography (ERCP), fi ne-needl e as pi rat i on, or P.149
expl orat ory l aparos copy or l aparot omy. Some aut hori t i es have expres s ed concern t hat fi ne-needl e as pi rat i on t racks may be s i t es of t umor i mpl ant at i on. o
o
c. W hen a di agnos i s of pancreat i c cancer has been es t abl i s hed, addi t i onal s t udi es may be neces s ary t o eval uat e t he ext ent of l ocal di s eas e and t o s creen for met as t at i c di s eas e.
(1) Pat i ent s who are pot ent i al candi dat es for curat i ve-at t empt s urgery frequent l y undergo CT scans of the chest and abdomen, al ong wi t h angiography or MRI-based imaging techniques t o as s es s pat ency and i nt egri t y of key vas cul ar s t ruct ures s uch as t he s uperi or
Pa g e 7 5 5
ABC Ambe r CHM Conve rte r Tria l ve rsion, http://w w w .proce sste x t.com/a bcchm.html
mes ent eri c art ery.
(2) Bone scans and brain imaging are not indicated i n pat i ent s wi t h no s ympt oms of brai n or bone met as t as es .
7. Prognosis o
o
a. Surgically resectable cancer. St udi es have found t hat pat i ent s who undergo s urgery fol l owed by adjuvant chemoradi at i on may have l ong-t erm s urvi val as hi gh as 40%.
o
o
b. Locally advanced, unresectable cancer. Thes e pat i ent s cannot be cured, but t he us e of concurrent chemot herapy wi t h radi at i on t herapy i ncreas es average s urvi val t o approxi mat el y 1 year.
o
o
c. Metastatic cancer. The s urvi val of unt reat ed pat i ent s averages about 3 mont hs . Thi s i ncreas es t o al mos t 6 mont hs when gemci t abi ne chemot herapy i s us ed. Long-t erm s urvi val i s exceedi ngl y rare.
8. Staging. For ful l s t agi ng i nformat i on, pl eas e s ee t he mos t recent edi t i on of t he AJCC Cancer St agi ng Handbook.
9. T herapy o
Pa g e 7 5 6
ABC Ambe r CHM Conve rte r Tria l ve rsion, http://w w w .proce sste x t.com/a bcchm.html
o
a. Localized disease
(1) Surgery. Surgery i s t he t reat ment of choi ce for pat i ent s wi t h l ocal i zed pancreat i c cancer. W i t h an experi enced s urgeon, t he W hi ppl e procedure may be performed wi t h accept abl e operat i ve morbi di t y and mort al i t y. However, onl y a s mal l fract i on of pat i ent s who undergo t hi s operat i on or t ot al pancreat ect omi es are cured of pancreat i c cancer.
(2) Adjuvant chemotherapy and radiation therapy. Pos t operat i ve radi at i on t herapy admi ni s t ered i n conjunct i on wi t h 5-FU or gemcitabine chemotherapy has been s hown t o i mprove cure rat es .
o
o
b. Metastatic disease
(1) The approach t o met as t at i c di s eas e i s purel y pal l i at i ve. Narcot i c anal ges i cs or cel i ac pl exus bl ocks may pal l i at e pai n. Ei t her operat i ve bypas s or t he pl acement of bi l i ary s t ent s or cat het ers may pal l i at e obs t ruct i ve jaundi ce.
(2) Chemot herapy us i ng t he ant i met abol i t e gemcitabine i mproves qual i t y of l i fe and s urvi val , but few pat i ent s wi t h met as t at i c pancreat i c adenocarci noma s urvi ve beyond 1
Pa g e 7 5 7
ABC Ambe r CHM Conve rte r Tria l ve rsion, http://w w w .proce sste x t.com/a bcchm.html
year. 5-FU has l es s pal l i at i ve benefi t t han does gemci t abi ne. o
o
c. Neuroendocrine tumors. Many pat i ent s pres ent wi t h met as t at i c di s eas e t o t he l i ver. Carcinoid tumors may have an i ndol ent nat ural hi s t ory, and pat i ent s may be fol l owed for years before needi ng t herapeut i c i nt ervent i on. Chemot herapy wi t h 5-FU and streptozocin can provi de pal l i at i on. Locoregional approaches t o l i ver met as t as es s uch as chemoembol i zat i on or i nt rahepat i c art eri al chemot herapy may be appropri at e i n s el ect ed pat i ent s . Met as t at i c islet cell tumors are res pons i ve t o chemot herapy wi t h streptozocin and doxorubicin. Pat i ent s wi t h carcinoid syndrome may benefi t from t reat ment wi t h somatostatin analogs.
C. Carcinoma of the stomach
1. Incidence. Cancers of t he s t omach occur i n approxi mat el y 23,000 i ndi vi dual s annual l y i n t he Uni t ed St at es , and account for approxi mat el y 12,000 deat hs each year. St ri ki ng decreas es i n t he i nci dence of gas t ri c adenocarci nomas i n t he di s t al hal f have been not ed i n t he Uni t ed St at es s i nce t he earl y part of t he t went i et h cent ury; t he caus es of t hi s change are not known. The i nci dence of cancer of t he cardi a and gas t roes ophageal junct i on, however, has been ri s i ng rapi dl y.
2. Etiology. The et i ol ogy of s t omach cancers remai ns l argel y unknown. However, t he fol l owi ng condi t i ons have been i mpl i cat ed i n t he caus e of gas t ri c cancer:
Pa g e 7 5 8
ABC Ambe r CHM Conve rte r Tria l ve rsion, http://w w w .proce sste x t.com/a bcchm.html o
o
a. Precurs or conditions i ncl ude chroni c at rophi c gas t ri t i s , i nt es t i nal met apl as i a, perni ci ous anemi a, part i al gas t rect omy for beni gn di s eas e, Hel i c obac t er pyl ori i nfect i on, and gas t ri c adenomat ous pol yps . P.150
o
o
b. Family history. Pat i ent s wi t h heredi t ary nonpol ypos i s col orect al cancer (Lynch s yndrome) are at i ncreas ed ri s k.
o
o
c. T obacco use
o
o
d. Diet and vitamin deficiencies. Di et s t hat are hi gh i n s al t and ni t ros ami nes , s uch as ferment ed and s moked foods , are cons i dered ri s k fact ors for gas t ri c cancer. Defi ci enci es i n vi t ami ns A, C, and E; s el eni um; β-carot ene; and fi ber are al s o ri s k fact ors for cancer.
3. Screening. Current l y, t here are no recommendat i ons for s creeni ng i n as ympt omat i c i ndi vi dual s .
4. Pathology. The vas t majori t y of s t omach neopl as ms are adenocarci nomas . Mos t are poorl y or moderat el y di fferent i at ed. o
Pa g e 7 5 9
ABC Ambe r CHM Conve rte r Tria l ve rsion, http://w w w .proce sste x t.com/a bcchm.html o
a. Adenocarcinomas may ori gi nat e i n any part of t he s t omach. In t he l as t few decades , t here has been a ri s e i n t he i nci dence of adenocarci nomas ari s i ng i n or near t he gas t roes ophageal junct i on.
o
o
b. Lymphomas may ari s e i n t he s t omach.
5. Clinical features o
o
a. Local symptoms may i ncl ude abdomi nal pai n, earl y s at i et y, gas t roi nt es t i nal bl eedi ng, dys peps i a, dys phagi a, odynophagi a, or s ympt oms of gas t ri c out l et obs t ruct i on.
o
o
b. Systemic manifestations are common and may i ncl ude:
(1) Anorexi a, wei ght l os s , naus ea
(2) Fat i gue due t o anemi a
(3) Pai n due t o bone met as t as es
(4) Hepat omegal y, pai n, or fever due t o l i ver i nvol vement
(5) Bl eedi ng (di s order) mani fes t ed by ei t her recurrent s uperfi ci al t hrombophl ebi t i s (Trous s eau's s yndrome), deep
Pa g e 7 6 0
ABC Ambe r CHM Conve rte r Tria l ve rsion, http://w w w .proce sste x t.com/a bcchm.html
t hrombophl ebi t i s , or DIC
6. Diagnosis. Pat i ent s wi t h gas t ri c cancer may pres ent wi t h ulcers or gastrointestinal bleeding. Cancers mus t be di agnos ed and s t aged. o
o
a. An upper gastrointestinal series or upper gastrointestinal endoscopy es t abl i s hes t he pres ence of a mas s . Occas i onal l y, gas t ri c cancer mani fes t s as linitis plastica, wi t hout an obvi ous epi t hel i al l es i on, but wi t h a s ubmucos al t umor i nfi l t rat i ng t he organ.
o
o
b. Biopsy s houl d be performed i n al l gas t ri c ul cers , even t hos e t hat appear t o be beni gn (i f deemed s afe). Ul cers s houl d be fol l owed careful l y t o ens ure t hat compl et e heal i ng occurs . Typi cal l y, endos copi c bi ops y di agnos es t he pres ence of a gas t ri c cancer.
o
o
c. Commonl y us ed staging procedures i ncl ude t he fol l owi ng:
(1) CT scanning may i dent i fy peri gas t ri c or cel i ac l ymph node i nvol vement . In addi t i on, i t can i dent i fy t he pres ence of pul monary, medi as t i nal , l i ver, ret roperi t oneal l ymph node, or pel vi c met as t as es .
(2) Laparoscopy may be us ed preoperat i vel y t o eval uat e t he oment um, whi ch i s a common s i t e of met as t as i s t hat i s poorl y vi s ual i zed on
Pa g e 7 6 1
ABC Ambe r CHM Conve rte r Tria l ve rsion, http://w w w .proce sste x t.com/a bcchm.html
CT s cans .
(3) Bone scans and brain imaging are not indicated i n pat i ent s wi t h no s ympt oms of brai n or bone met as t as es .
7. Prognosis. Pat hol ogi c s t agi ng remai ns t he mos t i mport ant det ermi nant of prognos i s . Ot her negat i ve prognos t i c fact ors i ncl ude ol der age, wei ght l os s >10%, di ffus e vers us i nt es t i nal hi s t ol ogy, and l ocat i on of t he t umor proxi mal and ent i re s t omach i nvol vement carryi ng wors e prognos i s . o
o
a. Surgically resectable cancer. Cures res ul t i n t he majori t y of pat i ent s who undergo s urgery for earl y (e.g., T 1 N 0 M 0 ) l es i ons , but s urgi cal cure rat es decl i ne rapi dl y wi t h advanci ng di s eas e s t age.
o
o
b. Metastatic cancer. The average s urvi val of unt reat ed pat i ent s i s approxi mat el y 6 mont hs , and t reat ment wi t h combi nat i on chemot herapy i ncreas es t hi s t i me by onl y s everal mont hs .
8. Staging. For ful l s t agi ng i nformat i on, pl eas e s ee t he mos t recent edi t i on of t he AJCC Cancer St agi ng Handbook.
9. T herapy o
o
a. Surgery. Surgery i s t he t reat ment of choi ce for
Pa g e 7 6 2
ABC Ambe r CHM Conve rte r Tria l ve rsion, http://w w w .proce sste x t.com/a bcchm.html
pat i ent s wi t h l ocal i zed gas t ri c cancer. The probability of surgical cure is directly related to the stage of the cancer. Pat i ent s wi t h t rans mural , l ymph node–i nvol ved cancers have l ow s urgi cal cure rat es of onl y 15%.
(1) Becaus e gas t ri c cancer i s t ypi cal l y di agnos ed at rel at i vel y advanced s t ages i n t he Uni t ed St at es , rel at i vel y few pat i ent s are cured. More ext ens i ve s urgery, pi oneered i n Japan, has not P.151
been s hown defi ni t i vel y t o i mprove out comes i n pat i ent s wi t h l ocal i zed but advanced-s t age cancers .
(2) Adjuvant chemotherapy and chemotherapy. Pos t operat i ve radi at i on t herapy has not been s hown t o i mprove s urgi cal out comes i n pat i ent s who have undergone s urgery wi t h curat i ve i nt ent . However, adjuvant t herapy wi t h radi at i on t herapy pl us 5-FU chemot herapy i mproves s urvi val .
o
o
b. Palliative approach to metastatic disease. Narcot i c anal ges i cs or cel i ac pl exus bl ocks may reduce pai n. Ei t her operat i ve bypas s or t he pl acement of bi l i ary s t ent s or cat het ers may pal l i at e obs t ruct i ve jaundi ce. Al t hough chemot herapeut i c agent s s uch as 5-FU, doxorubi ci n, mi t omyci n-C, et opos i de, met hot rexat e,
Pa g e 7 6 3
ABC Ambe r CHM Conve rte r Tria l ve rsion, http://w w w .proce sste x t.com/a bcchm.html
pacl i t axel , docet axel , or ci s pl at i n have modes t s i ngl e-agent act i vi t y and may provi de pal l i at i on of s ympt oms , t hey have no meani ngful i mpact on s urvi val . Combi nat i on t herapy wi t h t wo or more of t hes e agent s may i mprove qual i t y of l i fe and s urvi val , but t he overal l i mpact i s not great . o
o
c. Additional use of chemotherapy. The t reat ment of gastric lymphomas t ypi cal l y i nvol ves chemot herapy. Aggres s i ve chemot herapy of t umors t hat res pond rapi dl y may res ul t i n gas t ri c perforat i on, however.
D. Carcinoma of the esophagus
1. Incidence. Cancers of t he es ophagus occur i n approxi mat el y 14,500 i ndi vi dual s annual l y i n t he Uni t ed St at es and account for more t han 13,700 deat hs each year. St ri ki ng i ncreas es i n t he i nci dence of es ophageal adenocarci nomas have been not ed i n t he Uni t ed St at es s i nce t he earl y part of t he t went i et h cent ury; t he caus es of t hi s change are not known.
2. Etiology. The et i ol ogy of es ophageal cancers remai ns l argel y unknown. Mos t adenocarcinomas ari s e i n Barrett' s esophagus, but rel at i vel y few i ndi vi dual s wi t h Barret t 's es ophagus devel op es ophageal mal i gnanci es . Gas t roes ophageal refl ux di s eas e (GERD), whi ch can be caus ed by obes i t y, may cont ri but e t o t he format i on of Barret t 's es ophagus . Alcohol abuse and tobacco use predi s pos e i ndi vi dual s t o s quamous cel l carci nomas .
Pa g e 7 6 4
ABC Ambe r CHM Conve rte r Tria l ve rsion, http://w w w .proce sste x t.com/a bcchm.html
3. Screening. Recommendat i ons for screening for Barrett' s esophagus include: o
o
a. For no dys pl as i a, endos copy every 2 t o 3 years
o
o
b. For l ow-grade dys pl as i a, endos copy every 6 mont hs for 12 mont hs and t hen yearl y.
o
o
c. For hi gh-grade dys pl as i a, es ophagect omy or t hree mont hl y endos copi es or phot odynami c t herapy.
4. Pathology. The vas t majori t y of es ophageal adenocarci nomas are poorl y or moderat el y di fferent i at ed. Adenocarcinomas t ypi cal l y ari s e i n or near t he gas t roes ophageal junct i on i n Barret t 's es ophagus . In t he l as t few decades , t he i nci dence of adenocarci nomas ari s i ng i n or near t he gas t roes ophageal junct i on has i ncreas ed.
5. Clinical features o
o
a. Local symptoms mos t commonl y i ncl ude dys peps i a, dys phagi a or odynophagi a, epi gas t ri c or ret ros t ernal pai n, and occas i onal l y gas t roi nt es t i nal bl eedi ng.
o
o
b. Systemic manifestations are common and may i ncl ude:
Pa g e 7 6 5
ABC Ambe r CHM Conve rte r Tria l ve rsion, http://w w w .proce sste x t.com/a bcchm.html
(1) Anorexi a, wei ght l os s
(2) Fat i gue due t o anemi a
(3) Pai n due t o bone met as t as es
(4) Hepat omegal y, pai n, or fever due t o l i ver i nvol vement
6. Diagnosis o
o
a. Bari um s wal l ow or upper gas t roi nt es t i nal endos copy es t abl i s hes t he pres ence of a mas s l es i on.
o
o
b. Typi cal l y, endoscopic biopsy di agnos es t he pres ence of a gas t ri c cancer.
o
o
c. Commonl y us ed staging studies i ncl ude t he fol l owi ng:
(1) CT scanning may i dent i fy medi as t i nal or cel i ac l ymph node i nvol vement . It can i dent i fy t he pres ence of pul monary, l i ver, ret roperi t oneal l ymph node, or pel vi c met as t as es .
(2) Endoscopic ultrasound may defi ne t he
Pa g e 7 6 6
ABC Ambe r CHM Conve rte r Tria l ve rsion, http://w w w .proce sste x t.com/a bcchm.html
ext ent of t umor i nvas i on i n t he es ophagus and t he pres ence of peri es ophageal l ymph node enl argement .
(3) Laparoscopy may be us ed preoperat i vel y t o eval uat e t he oment um, whi ch i s a common s i t e of met as t as i s t hat i s poorl y vi s ual i zed on CT s cans . P.152
(4) Bone scans and brain imaging are not indicated i n pat i ent s wi t h no s ympt oms of brai n or bone met as t as es .
7. Prognosis. Es ophageal cancer i s hi ghl y curabl e i n i t s earl i es t s t ages ; however, t he majori t y of pat i ent s pres ent wi t h l ocal l y advanced or met as t at i c di s eas e and many requi re concurrent neoadjuvant chemot herapy and i rradi at i on t herapy t o achi eve res ect abi l i t y. o
o
a. Surgically resectable cancer. Mos t pat i ent s who undergo s urgery for earl y (e.g., T 1 N 0 M 0 ) l es i ons are cured, but s urgi cal cure rat es decreas e rapi dl y as t he s t age of cancer i ncreas es .
o
o
b. Metastatic cancer. The s urvi val of unt reat ed pat i ent s averages about 6 mont hs , and chemot herapy i ncreas es average s urvi val by s everal mont hs i n res pons i ve pat i ent s .
Pa g e 7 6 7
ABC Ambe r CHM Conve rte r Tria l ve rsion, http://w w w .proce sste x t.com/a bcchm.html
8. Staging. For ful l s t agi ng i nformat i on, pl eas e s ee t he mos t recent edi t i on of t he AJCC Cancer St agi ng Handbook.
9. T herapy o
o
a. Localized cancer
(1) Surgery. Cure may res ul t from s urgery al one, chemot herapy and radi at i on t herapy, or chemot herapy and radi at i on t herapy fol l owed by s urgery. Surgery, ei t her al one or preceded by chemot herapy and radi at i on t herapy, i s t he mos t commonl y us ed curat i ve t reat ment modal i t y i n es ophageal di s eas e. The probabi l i t y of s urgi cal cure rel at es di rect l y t o t he s t age of t he cancer. A mi nori t y of pat i ent s wi t h t rans mural or l ymph node–i nvol ved cancers are cured by es ophagect omy. A curat i ve approach i s not us ual l y t aken for s upracl avi cul ar or cel i ac l ymph node met as t as es .
(2) Preoperative chemotherapy and radiation therapy. Preoperat i ve chemot herapy wi t h 5-FU and ci s pl at i n i n conjunct i on wi t h radi at i on t herapy has been s hown t o i mprove s urgi cal out comes i n pat i ent s who have undergone s urgery wi t h curat i ve i nt ent . W hen us ed al one, nei t her chemot herapy nor radi at i on t herapy i mproves s urgi cal out come i n ei t her preoperat i ve or pos t operat i ve
Pa g e 7 6 8
ABC Ambe r CHM Conve rte r Tria l ve rsion, http://w w w .proce sste x t.com/a bcchm.html
s et t i ngs . o
o
b. Metastatic disease. Treat ment i s pal l i at i ve.
(1) Radiation therapy in conjunction with chemotherapy wi t h agent s s uch as 5-FU or ci s pl at i n may pal l i at e l ocal s ympt oms caus ed by es ophageal obs t ruct i on.
(2) Esophageal stents and feeding tubes may mai nt ai n ent eral nut ri t i on and reduce t he ri s ks of as pi rat i on of es ophageal cont ent s .
(3) Narcotic analgesics may pal l i at e pai n.
(4) Other chemotherapeutic approaches may be us eful . Single-agent chemotherapy wi t h agent s s uch as 5-FU, et opos i de, i ri not ecan, pacl i t axel , docet axel , or ci s pl at i n may di s pl ay moderat e act i vi t y and may provi de pal l i at i on of s ympt oms but wi t hout meani ngful i mpact on s urvi val . Combination chemotherapy wi t h t wo or more of t hes e drugs may i mprove qual i t y of l i fe but has mi ni mal overal l i mpact on s urvi val .
E. Carcinoma of the hepatobiliary system
1. Incidence. Hepat obi l i ary cancers occur i n approxi mat el y 20,000 i ndi vi dual s annual l y i n t he Uni t ed St at es and account for more t han 16,000 deat hs each
Pa g e 7 6 9
ABC Ambe r CHM Conve rte r Tria l ve rsion, http://w w w .proce sste x t.com/a bcchm.html
year.
2. Etiology. Hepatocellular carcinoma i s as s oci at ed wi t h di s eas es t hat caus e chroni c l i ver i njury s uch as chronic active hepatitis caus ed by hepatitis B virus (HBV), hepatitis C virus (HCV), alcoholic cirrhosis, hemochromatosis, di et ary i ron overl oad, and W i l s on's di s eas e. Bi l i ary t ract neopl as ms s uch as cholangiocarcinoma and gallbladder cancer have i ncreas ed i nci dence i n i ndi vi dual s wi t h i nfl ammat ory bowel di s eas e.
3. Screening. Becaus e s i gns and s ympt oms do not us ual l y appear unt i l t he cancer i s i n i t s l at er s t ages , l i ver cancer i s s el dom di agnos ed earl y. However, i n hi gh-ri s k popul at i ons at ri s k for hepat ocel l ul ar cancer, recommendat i ons i ncl ude checki ng a α-fet oprot ei n (AFP) s erum t umor marker and ul t ras ound every 6 mont hs .
4. Pathology. Mos t hepat obi l i ary neopl as ms are adenocarci nomas .
5. Clinical features o
o
a. Local symptoms of hepatocellular carcinoma i ncl ude pain res ul t i ng from di s t ent i on of t he l i ver caps ul e, pal pabl e mas s i n t he ri ght upper quadrant , or abdomi nal s wel l i ng. Obstructive jaundice i s a charact eri s t i c pres ent at i on for cholangiocarcinoma. Jaundi ce may be a mani fes t at i on of gallbladder cancer.
Pa g e 7 7 0
ABC Ambe r CHM Conve rte r Tria l ve rsion, http://w w w .proce sste x t.com/a bcchm.html
P.153
o
o
b. Systemic manifestations are common and may i ncl ude:
(1) Si gns and s ympt oms of l i ver fai l ure s uch as edema, as ci t es , pruri t us , and port al hypert ens i on
(2) Anorexi a, wei ght l os s , naus ea and vomi t i ng
(3) Fat i gue due t o anemi a
(4) Pai n due t o bone met as t as es or peri t oneal met as t as es
(5) Dys pnea or cough due t o pul monary met as t as es
(6) Hepat omegal y, pai n, or fever due t o l i ver i nvol vement
6. Diagnosis. Hepatocellular carcinoma may be i ni t i al l y s us pect ed i n at -ri s k i ndi vi dual s by a ri s e i n the AFP serum tumor marker or by t he pres ence of mas s on a s creeni ng l i ver ultrasound. On occas i on, gallbladder carcinoma i s di s covered at chol ecys t ect omy for
Pa g e 7 7 1
ABC Ambe r CHM Conve rte r Tria l ve rsion, http://w w w .proce sste x t.com/a bcchm.html
s ympt omat i c, apparent l y beni gn gal l bl adder di s eas e. o
o
a. CT scans and MRI imaging may i dent i fy a mas s or mas s es i n t he l i ver. Typi cal l y, t he di agnos i s of hepat ocel l ul ar carci noma i s es t abl i s hed by a CT-gui ded or ul t ras ound-gui ded fi ne-needl e as pi rat i on bi ops y.
o
o
b. In pat i ent s who pres ent wi t h ri ght upper quadrant pain or jaundice, CT scan or ERCP may det ect ei t her a mas s , obs t ruct i on of bi l i ary fl ow, or a fi l l i ng defect i n a major bi l i ary radi cl e. Ei t her fi ne-needl e as pi rat i on bi ops y or ERCP-gui ded bi ops y can es t abl i s h a di agnos i s of cholangiocarcinoma.
o
o
c. Commonly used staging studies i ncl ude t he fol l owi ng:
(1) CT scanning may i dent i fy medi as t i nal or cel i ac l ymph node i nvol vement . It can i dent i fy t he pres ence of pul monary, l i ver, ret roperi t oneal l ymph node, or pel vi c met as t as es .
(2) Bone scans and brain imaging are not indicated i n pat i ent s wi t h no s ympt oms of brai n or bone met as t as es .
7. Prognosis o
Pa g e 7 7 2
ABC Ambe r CHM Conve rte r Tria l ve rsion, http://w w w .proce sste x t.com/a bcchm.html
o
a. Surgically resectable cancer. Surgi cal cure rat es decreas e rapi dl y wi t h i ncreas i ng di s eas e s t age.
o
o
b. Metastatic cancer. The average s urvi val of unt reat ed pat i ent s i s approxi mat el y 6 mont hs .
8. Staging. For ful l s t agi ng i nformat i on, pl eas e s ee t he mos t recent edi t i on of t he AJCC Cancer St agi ng Handbook.
9. T herapy o
o
a. Localized cancer
(1) Surgery
(a) In localized hepatobiliary neoplasms, s urgery al one may provi de a cure i n s ome i ns t ances . Some pat i ent s wi t h t echni cal l y unres ect abl e but l i ver-confi ned neopl as ms may be candi dat es for liver transplantation. Radi at i on t herapy and s t andard chemot herapy have no defi ned rol es i n t he management of hepat ocel l ul ar carci noma, al t hough chemot herapy i s occas i onal l y us ed t o reduce t umor s i ze and faci l i t at e curat i ve-at t empt s urgery.
Pa g e 7 7 3
ABC Ambe r CHM Conve rte r Tria l ve rsion, http://w w w .proce sste x t.com/a bcchm.html
(b) In biliary tract neoplasms, adjunct i ve 5-FU–bas ed chemot herapy pl us ext ernal beam or hi gh dos e-rat e i nt ral umi nal radi at i on i s commonl y us ed, al t hough pros pect i ve, randomi zed cl i ni cal t ri al s have not val i dat ed i t s ut i l i t y.
(2) Other locoregional approaches. Pat i ent s wi t h s mal l but unres ect abl e l es i ons may benefi t from t he i ns t i l l at i on of intralesional alcohol or chemoembolization, whi ch i nvol ves i nject i on of a s l urry of chemot herapeut i c agent s i n an oi l emul s i on di rect l y i nt o t he ves s el s uppl yi ng t he neopl as t i c l es i on. Nei t her of t hes e ext ens i vel y us ed approaches has been val i dat ed i n randomi zed cl i ni cal t ri al s . Radi ofrequency abl at i on of l es i ons and i ns t i l l at i on of radi ol abel ed mi cros pheres i n t he hepat i c t umors are t wo promi s i ng newer t echni ques t hat are under i nves t i gat i on.
(3) Metastatic disease. St andard chemot herapeut i c agent s are i neffect i ve i n pat i ent s wi t h advanced hepat ocel l ul ar carci noma. Admi ni s t rat i on of chemot herapy i s compl i cat ed by comorbi di t i es as s oci at ed wi t h l i ver fai l ure. No one chemotherapeutic agent or regimen has emerged as a standard of care in biliary neoplasms, but several agents have antitumor effects, and patients with metastatic disease may be treated with palliative intent. In addi t i on, stenting of
Pa g e 7 7 4
ABC Ambe r CHM Conve rte r Tria l ve rsion, http://w w w .proce sste x t.com/a bcchm.html
obs t ruct ed bi l e duct s , when feas i bl e, can pal l i at e jaundi ce. Narcotic analgesics may pal l i at e pai n. P.154
VIII. Gynecologic Malignancies A. Ovarian cancer
1. Incidence. Ovari an cancer devel ops i n 1 i n 68 women and i s t he mos t common caus e of deat h from gynecol ogi c mal i gnancy. Inci dence rat es are hi ghes t i n i ndus t ri al i zed nat i ons . In 2007, t he ACS es t i mat es t hat t here wi l l be about 22,430 new cas es of ovari an cancer di agnos ed t hat year i n t he Uni t ed St at es .
2. Risk factors for ovari an cancer i ncl ude: o
o
a. Nul l i pari t y
o
o
b. Fewer t han average number of pregnanci es or a hi s t ory of mi s carri age
o
o
c. Fami l y hi s t ory of ovari an cancer
o
o
d. Hi s t ory of endocri ne di s orders
o
o
e. Age
Pa g e 7 7 5
ABC Ambe r CHM Conve rte r Tria l ve rsion, http://w w w .proce sste x t.com/a bcchm.html o
o
f. Genetic ri s k fact ors i ncl udi ng, heredi t ary breas t and ovari an cancer s yndromes as s oci at ed wi t h BRCA1 and BRCA2 and wi t h heredi t ary nonpol ypos i s col orect al cancer s yndrome.
3. Screening. Ovari an cancer i s rarel y di agnos ed before t he di s eas e has reached an advanced s t age (s t age 3 or 4). o
o
a. Screeni ng i s recommended i n women wi t h known genet i c s yndromes s uch as BRCA1 and BRCA2 mut at i ons and Lynch II s yndrome, whi ch are as s oci at ed wi t h s i gni fi cant l y i ncreas ed ri s k of devel opi ng t hi s di s eas e; and for women wi t h a s t rong fami l y hi s t ory of ovari an cancer. Pel vi c ul t ras ound and CA 125 i n addi t i on t o pel vi c exam are ut i l i zed for s creeni ng.
o
o
b. Early detection. W omen wi t h average ri s k for ovari an cancer s houl d obt ai n rout i ne gynecol ogi c exami nat i ons . No exi s t i ng di agnos t i c modal i t y exhi bi t s s uffi ci ent s ens i t i vi t y and s peci fi ci t y t o permi t t he rel i abl e earl y det ect i on of s urgi cal l y curabl e cancers .
4. Pathology. The majori t y of ovari an cancers are epi t hel i al i n ori gi n. Thi s i ncl udes s erous , muci nous , endomet ri oi d, t rans i t i onal , and cl ear cel l ; wi t h cl ear cel l t ypes havi ng t he wors e prognos i s . Les s common are t he germ cel l and s t romal t umors .
Pa g e 7 7 6
ABC Ambe r CHM Conve rte r Tria l ve rsion, http://w w w .proce sste x t.com/a bcchm.html
5. Clinical features o
o
a. Local symptoms i ncl ude abdomi nal di s comfort , l ow back pai n, bl oat i ng, and abdomi nal di s t ent i on.
o
o
b. Sys t emi c manifestations i ncl ude:
(1) Virilization from germ cel l t umors
(2) Precocious puberty i n premenarchal women, amenorrhea i n women of reproduct i ve age, vaginal bleeding i n pos t menopaus al women can be caus ed by granul os e cel l t umors .
6. Diagnosis o
o
a. Initial evaluation
(1) Pelvic ultrasound det ect s s mal l l es i ons i n t he ovary, even t hos e t hat cannot be pal pat ed duri ng bi manual exami nat i on.
(2) Det ermi nat i on of l evel s of t he tumor marker CA 125 and an annual pelvic ultrasound are recommended for pat i ent s wi t h a fami l y hi s t ory of ovari an cancer.
o
Pa g e 7 7 7
ABC Ambe r CHM Conve rte r Tria l ve rsion, http://w w w .proce sste x t.com/a bcchm.html
o
b. Pretreatment evaluation
(1) Al l pat i ent s wi t h a new di agnos i s of ovari an cancer s houl d undergo abdomi nal and pel vi c CT s canni ng and/or MRI and a pel vi c ul t ras ound. Ches t radi ography, bari um enemas , cys t os copy, and fl exi bl e s i gmoi dos copy may be cons i dered, dependi ng on t he pres ent at i on of t he di s eas e.
(2) Rout i ne bl ood chemi s t ri es , i ncl udi ng al bumi n, magnes i um, and renal and l i ver funct i on t es t i ng, s houl d be performed. An el evat ed CA 125 i s found i n 80%–85% of pat i ent s wi t h di s eas e t hat i s not l ocal i zed t o t he ovari es . If a germ cel l t umor i s s us pect ed, t he fol l owi ng t umor markers s houl d be drawn: l act at e dehydrogenas e (LDH), AFP, and human chori oni c gonadot ropi n (hCG).
(3) A careful l aparot omy es t abl i s hes t he s t age and ext ent of t he di s eas e and may permi t t he reduct i on of t umor mas s es .
(a) The ent i re abdomi nal cont ent s s houl d be expl ored, and any s us pi ci ous l es i ons s houl d be removed or exami ned by bi ops y. P.155
Pa g e 7 7 8
ABC Ambe r CHM Conve rte r Tria l ve rsion, http://w w w .proce sste x t.com/a bcchm.html
(b) In addi t i on t o checki ng t he pri mary t umor for rupt ure or adherence, t he s urgeon s houl d not e t he amount and t ype of as ci t es (i f pres ent ). Sampl es of fl ui d s houl d be col l ect ed for cyt ol ogi c anal ys i s by peri t oneal was hi ng.
(c) The para-aort i c nodes i n t he regi on of t he renal hi l a s houl d be exami ned by bi ops y.
(d) The di aphragmat i c s urfaces s houl d be careful l y expl ored.
7. Prognosis o
o
a. The stage of disease i s t he mos t i mport ant prognos t i c fact or.
(1) Pat i ent s wi t h di s t ant met as t as es are rarel y cured, even aft er combi nat i on chemot herapy.
(2) Pat i ent s wi t h di s eas e l i mi t ed t o t he ovary may be cured wi t h s urgery al one.
(3) Pat i ent s wi t h mi ni mal s t age III di s eas e have an excel l ent prognos i s wi t h debul ki ng s urgery fol l owed by combi nat i on chemot herapy, es peci al l y i f t he res i dual t umor
Pa g e 7 7 9
ABC Ambe r CHM Conve rte r Tria l ve rsion, http://w w w .proce sste x t.com/a bcchm.html
mas s es meas ure l es s t han 2 cm i n di amet er. o
o
b. Histologic grade has great er prognos t i c s i gni fi cance t han t ype. Borderl i ne t umors , whi ch di s pl ay nucl ear charact eri s t i cs of mal i gnancy but l ack s t romal i nvas i on, behave i n an i ndol ent fas hi on and are not res pons i ve t o chemot herapy.
8. Staging. For ful l s t agi ng i nformat i on, pl eas e s ee t he mos t recent edi t i on of t he AJCC Cancer St agi ng Handbook.
9. T herapy o
o
a. Surgery
(1) Procedures
(a) Mos t pat i ent s undergo bilateral salpingo-oophorectomy and transabdominal hysterectomy. Al l gros s res i dual di s eas e s houl d be res ect ed i f pos s i bl e. In mos t cas es , i f a pat i ent has met as t at i c di s eas e out s i de t he abdomi nal and pel vi c areas , debul ki ng s urgery i s not i ndi cat ed.
(b) Becaus e ovari an cancer frequent l y s preads t hroughout t he peri t oneal cavi t y and us ual l y i s as s oci at ed wi t h oment al
Pa g e 7 8 0
ABC Ambe r CHM Conve rte r Tria l ve rsion, http://w w w .proce sste x t.com/a bcchm.html
and peri t oneal s eedi ng, t he ent i re abdomi nal and pel vi c cavi t i es s houl d be expl ored, a partial omentectomy performed, and t he paracol i c gut t ers i ns pect ed.
(2) Adjuvant therapy and follow-up
(a) Pat i ent s wi t h s t age IA or s t age IB di s eas e wi t h wel l -di fferent i at ed or moderat el y di fferent i at ed t umors requi re no addi t i onal adjuvant t herapy aft er s urgery. Ot herwi s e, pat i ent s wi t h s t age III and IV di s eas e s houl d recei ve adjuvant t herapy, whi ch us ual l y cons i s t s of chemotherapy with carboplatin and paclitaxel for six cycles postoperatively. Al t ernat i ves i ncl ude total abdominal radiation therapy or intraperitoneal radioisotopes. Advanced ovari an cancer (i .e., s t age III or IV) i s not curabl e by s urgery al one. However, reduction of bul ky cancer i s as s oci at ed wi t h an i mproved res pons e t o ei t her radi at i on or chemot herapy, i f t he l arges t res i dual mas s i s reduced t o l es s t han 2 cm i n di amet er. Les s ext ens i ve reduct i ons are not benefi ci al , even i f l arge amount s of t he t umor are removed.
(b) Second-look surgery. Surgi cal reexpl orat i on at t he concl us i on of chemot herapy can det ect remi s s i on,
Pa g e 7 8 1
ABC Ambe r CHM Conve rte r Tria l ve rsion, http://w w w .proce sste x t.com/a bcchm.html
as s es s res pons e, and al l ow furt her cyt oreduct i ve s urgery i n an at t empt t o prol ong s urvi val . Approxi mat el y 50% of pat i ent s wi t h cl i ni cal l y compl et e res pons es have s urgi cal l y document ed compl et e remi s s i ons . The val ue of t he rout i ne us e of t hi s procedure has not been es t abl i s hed. o
o
b. Chemotherapy. Sys t emi c chemot herapy res ul t s i n hi gh res pons e rat es i n pat i ent s wi t h ovari an cancer and i ncreas es t he l i kel i hood of a cure i n pat i ent s wi t h res ect abl e di s eas e.
(1) Drug regimens. The mos t commonl y us ed agent s are pacl i t axel , cycl ophos phami de, ci s pl at i n, and carbopl at i n. Few dat a s upport t he us e of doxorubi ci n-bas ed regi mens . Cont emporary regi mens commonl y us e pacl i t axel i n conjunct i on wi t h carbopl at i n or ci s pl at i n.
(a) Carbopl at i n (dos e-adjus t ed for renal funct i on t o area-under-t he-curve) i s equi val ent i n effi cacy t o ci s pl at i n and i s as s oci at ed wi t h l es s emes i s , peri pheral neuropat hy, and ot ot oxi ci t y, al t hough i t has an i ncreas ed rat e of bone marrow t oxi ci t y compared wi t h ci s pl at i n. Becaus e carbopl at i n can be admi ni s t ered eas i l y i n t he out pat i ent s et t i ng, i t i s general l y preferred over ci s pl at i n t o t reat ovari an cancer.
Pa g e 7 8 2
ABC Ambe r CHM Conve rte r Tria l ve rsion, http://w w w .proce sste x t.com/a bcchm.html
P.156
(b) Taxanes are as s oci at ed wi t h approxi mat el y 30% res pons e rat es i n pat i ent s who have fai l ed t o res pond t o ot her t herapi es . Taxane–pl at i num combi nat i ons i nduce compl et e cl i ni cal remi s s i on i n more t han 60% of women wi t h previ ous l y unt reat ed s t age III ovari an cancer.
(2) T reatment of residual disease after induction chemotherapy. Radi at i on t herapy has cons i s t ent l y fai l ed t o prol ong s urvi val and i s as s oci at ed wi t h unaccept abl e rat es of bowel t oxi ci t y.
(a) Intraperitoneal chemotherapy i s bas ed on t he s l ow peri t oneal cl earance of many chemot herapeut i c agent s rel at i ve t o t ot al body cl earance. Some s t udi es demons t rat e t hat i nt raperi t oneal chemot herapy i s advant ageous compared wi t h s ys t emi c chemot herapy i n t he t reat ment of ovari an cancer.
(i) Drugs are admi ni s t ered i n l arge vol umes (1–2 L) t hrough s emi permanent s ys t ems (e.g., cat het ers ).
Pa g e 7 8 3
ABC Ambe r CHM Conve rte r Tria l ve rsion, http://w w w .proce sste x t.com/a bcchm.html
(ii) Agent s t hat have been us ed i nt raperi t oneal l y i n ovari an cancer i ncl ude met hot rexat e, 5-FU, carbopl at i n, and ci s pl at i n.
(b) Salvage chemotherapy opt i ons i ncl ude t opot ecan, gemci t abi ne, and hexamet hyl mel ami ne. Bot h t opot ecan and gemci t abi ne are act i ve agent s undergoi ng t es t i ng i n pat i ent s wi t h chemot herapy-nai ve di s eas e. Hi gh-dos e t herapy wi t h aut ol ogous s t em cel l res cue i s feas i bl e, but i t has fai l ed t o s how s i gni fi cant i mprovement over convent i onal dos i ng.
(c) Radiation therapy. For pat i ent s wi t h advanced di s eas e, radi at i on t herapy i s an opt i on for pal l i at i on of s ympt oms . Succes s i s di rect l y rel at ed t o t he l ocat i on and vol ume of di s eas e at t he t i me of t herapy.
B. Cervical cancer
1. Incidence. Carci noma of t he cervi x was once one of t he mos t common caus es of deat h from cancer, but i nci dence rat es have decreas ed by one-hal f i n t he l as t 30 years . Cervi cal cancer affect s 9,700 women annual l y. o
o
a. Thi s decl i ne i n mort al i t y rat e i s l argel y at t ri but abl e t o t he i nt roduct i on of t he Papani col aou
Pa g e 7 8 4
ABC Ambe r CHM Conve rte r Tria l ve rsion, http://w w w .proce sste x t.com/a bcchm.html
(Pap) s mear for t he di agnos i s of dys pl as i a or CIS. o
o
b. Invas i ve cancer i s mos t frequent l y s een i n pat i ent s bet ween 45 and 55 years of age from l ower s oci oeconomi c groups .
2. Etiology o
o
a. Risk factors for cervi cal cancer i ncl ude:
(1) Earl y i ni t i al s exual act i vi t y
(2) Mul t i pl e s exual part ners
(3) Earl y age at fi rs t pregnancy and mul t i pl e pregnanci es
(4) Hi s t ory of venereal i nfect i on
(5) Oral cont racept i ve us e
o
o
b. Recent s t udi es have i mpl i cat ed i nfect i on wi t h human papi l l omavi rus (HPV), whi ch caus es common geni t al wart s , i n t he progres s i on of cervi cal cancer. Several oncogenes have been i mpl i cat ed as wel l , i ncl udi ng c-myc and ras .
3. Screening. The Pap smear i s us eful for det ect i ng
Pa g e 7 8 5
ABC Ambe r CHM Conve rte r Tria l ve rsion, http://w w w .proce sste x t.com/a bcchm.html
earl y l es i ons and has a s ens i t i vi t y of 90%–95%. o
o
a. As a prevent i ve meas ure, women s houl d begi n s creeni ng 3 years aft er t he ons et of vagi nal i nt ercours e or at 21 years , whi chever i s earl i er. W omen s houl d have a negat i ve t es t every 2 years i f us i ng l i qui d-bas ed cyt ol ogy or annual l y wi t h convent i onal cervi cal cyt ol ogy s mears unt i l t he age of 30. Aft er t he age of 30 and i f t hey have had t hree normal t es t s res ul t s cons ecut i vel y, t hey can cont i nue s creeni ng every 2–3 years . Screeni ng s houl d end at age 70 i f t hey have had no abnormal /pos i t i ve cyt ol ogy t es t s wi t hi n t he previ ous 10 years or earl i er i f a women has had a compl et e hys t erect omy.
o
o
b. Hemorrhage, i nfl ammat ory react i ons (e.g., a fungat i ng mas s ), or i nvas i ve cancer may res ul t i n fal s e-negat i ve s mears .
4. Pathology o
o
a. Squamous metaplasia i s t he precurs or of cervical intraepithelial neoplasia (CIN), whi ch i s s ubdi vi ded i nt o t hree grades accordi ng t o s everi t y. P.157
(1) Grade I (mi l d-t o-moderat e dys pl as i a)
Pa g e 7 8 6
ABC Ambe r CHM Conve rte r Tria l ve rsion, http://w w w .proce sste x t.com/a bcchm.html
(2) Grade II (moderat e-t o-s evere dys pl as i a)
(3) Grade III (s evere dys pl as i a and CIS)
o
o
b. CIS exhi bi t s cyt ol ogi c evi dence of neopl as i a wi t hout i nvas i on of t he bas ement membrane. Thi s s t age can pers i s t as l ong as 3–10 years before progres s i ng t o i nvas i ve cervi cal carci noma.
5. Clinical features o
o
a. Local symptoms i ncl ude: abnormal vagi nal bl eedi ng, vagi nal di s charge.
o
o
b. Systemic features i ncl ude:
(1) Fatigue and anemia s econdary t o chroni c bl eedi ng
(2) Pain in the lumbosacral or gluteal area s ugges t i ng hydronephros i s or i nvol vement of t he i l i ac or peri aort i c l ymph nodes
(3) Urinary or rectal symptoms
(4) Leg pain or edema res ul t i ng from l ymphat i c or venous bl ockage
Pa g e 7 8 7
ABC Ambe r CHM Conve rte r Tria l ve rsion, http://w w w .proce sste x t.com/a bcchm.html
6. Diagnosis o
o
a. Early detection. W omen wi t h epi t hel i al abnormal i t i es i n t he Pap s mear s houl d undergo cervical biopsy. Vi s i bl e l es i ons neces s i t at e bi ops y, regardl es s of Pap s mear fi ndi ngs . If t here are no vi s i bl e l es i ons , col pos copy (us i ng a bi nocul ar mi cros cope and l i ght s ource) reduces t he need for cervi cal coni zat i on.
o
o
b. Pretreatment evaluation s houl d i ncl ude ches t radi ograph, CT s cans of t he abdomen and pel vi s , compl et e bl ood count (CBC), l i ver and renal funct i on s t udi es , and a bari um enema i n pat i ent s wi t h s ympt oms i nvol vi ng t he col on or rect um.
(1) Becaus e of t he pos s i bi l i t y of upper ext ens i on of t he t umor, curettage of t he endocervi cal canal and endomet ri um i s recommended duri ng t he i ni t i al eval uat i on.
(2) Cystoscopy and rectosigmoidoscopy are performed i n s el ect ed pat i ent s wi t h advanced di s eas e.
7. Prognosis. The 5-year s urvi val rat e for s t age I cervi cal cancer i s 75%–90%; for s t age II, 50%–70%; for s t age III, 30%–35%; and for s t age IV, 10%–15%. o
Pa g e 7 8 8
ABC Ambe r CHM Conve rte r Tria l ve rsion, http://w w w .proce sste x t.com/a bcchm.html o
a. T umor volume i s an i mport ant prognos t i c fact or wi t hi n s t ages . Bot h t he grade and t umor vol ume wi t hi n a gi ven grade have prognos t i c s i gni fi cance. Hi s t ol ogi c t ype has l i t t l e s i gni fi cance, except for s ome unus ual vari ant s .
o
o
b. Lymph node metastases are as s oci at ed wi t h a poor prognos i s . Ot her poor prognos t i c fact ors i ncl ude tumor grade, depth of invasion, and lymphovascular space invasion.
8. Staging. For ful l s t agi ng i nformat i on, pl eas e s ee t he mos t recent edi t i on of t he AJCC Cancer St agi ng Handbook. Cl i ni cal s t agi ng i s oft en i naccurat e, and s t agi ng l aparot omy s t udi es have s hown t hat approxi mat el y 30% of pat i ent s have more ext ens i ve di s eas e t han i s s ugges t ed by cl i ni cal s t agi ng procedures .
9. T herapy. Cervi cal cancer i n mos t pat i ent s i s t reat ed wi t h surgery, radiation therapy, or a combination of both. Chemot herapy wi t h ci s pl at i n al s o may be pres cri bed as a radi at i on s ens i t i zer. Ci s pl at i n has document ed act i vi t y as a s i ngl e agent . o
o
a. T otal abdominal hysterectomy i s t he us ual t reat ment for CIS and CIN. In women who wi s h t o bear chi l dren, management wi t h cervi cal coni zat i on and careful fol l ow-up may be appropri at e. Therapy for s t age I mi croi nvas i ve cancer (i .e., an i nvas i on of <3 mm i n dept h) i s i dent i cal t o t hat for CIS. The ri s k of cent ral recurrence and l ymph node met as t as i s i s i ncreas ed when t he t umor exceeds
Pa g e 7 8 9
ABC Ambe r CHM Conve rte r Tria l ve rsion, http://w w w .proce sste x t.com/a bcchm.html
3–5 mm i n dept h; i n t hes e cas es , hys t erect omy and pel vi c l ymphadenect omy are i ndi cat ed. o
o
b. Radical hysterectomy, pelvic lymphadenectomy, and radiation therapy have s i mi l ar res ul t s i n s t age IB and s t age IIA cancers . Becaus e of t he 15%–25% i nci dence of l ymph node met as t as es , t reat ment cons i s t s of radi cal hys t erect omy and bi l at eral pel vi c l ymphadenect omy.
(1) Pos t s urgi cal compl i cat i ons i ncl ude bl adder and uret hral dys funct i on or fi s t ul as .
(2) Radi at i on t herapy (brachyt herapy and ext ernal beam radi at i on) may be admi ni s t ered aft er s urgery. Adjuvant radi at i on i s i ndi cat ed aft er s urgery i f t here i s mi cros copi c paramet ri al ext ens i on, l es i ons of more t han 4 cm, nodal met as t as es , l ymphat i c or vas cul ar i nvas i on, pos i t i ve margi ns , or grade III l es i ons .
P.158
o
o
c. Radiation therapy or surgery i s appropri at e for pat i ent s wi t h i nvas i on beyond t he cervi x but wi t hout di s t ant met as t at i c i nvol vement . If t here i s paramet ri al s pread (i .e., t he t umor ext ends t hrough t he pel vi c wal l or i nt o t he vagi na), radi at i on t herapy may be preferred.
Pa g e 7 9 0
ABC Ambe r CHM Conve rte r Tria l ve rsion, http://w w w .proce sste x t.com/a bcchm.html
C. Endometrial cancer
1. Incidence. Carci noma of t he endomet ri um i s t he mos t common gynecol ogi c mal i gnancy, account i ng for 6% of al l mal i gnant t umors i n women. The i nci dence of endomet ri al cancer has been s t eadi l y i ncreas i ng s i nce t he 1970s . Approxi mat el y 39,300 new cas es are di agnos ed annual l y, res ul t i ng i n approxi mat el y 6,400 deat hs .
2. Risk factors o
o
a. Obesity, diabetes, hypertension, polycystic ovarian disease, late onset of menopause, and increasing age are as s oci at ed wi t h an i ncreas ed ri s k of endomet ri al cancer.
o
o
b. Unopposed estrogen therapy al s o pos es an i ncreas ed ri s k. Proges t erone decreas es t he ri s k as s oci at ed wi t h t he us e of pos t menopaus al es t rogens . Us e of combi nat i on oral cont racept i ves appears t o decreas e ri s k.
o
o
c. Ot her ri s k fact ors i ncl ude t amoxi fen, chroni c anovul at i on, and i rregul ar mens es .
3. Screening. There are current l y no s creeni ng gui del i nes for earl y det ect i on i n as ympt omat i c peopl e.
Pa g e 7 9 1
ABC Ambe r CHM Conve rte r Tria l ve rsion, http://w w w .proce sste x t.com/a bcchm.html
4. Pathology. Mos t endomet ri al cancers are adenocarcinomas. Si xt y-s even percent are charact eri s t i c endomet ri al adenocarci nomas , 20% are endomet ri al adenocarci nomas wi t h focal areas of beni gn s quamous met apl as i a, and 13% are adenos quamous carci nomas .
5. Clinical features o
o
a. Local symptoms i ncl ude abnormal vaginal bleeding, whi ch i s s een i n approxi mat el y 90% of cas es . Ot her s ympt oms i ncl ude: prol onged or heavy mens es and i nt ermens t rual s pot t i ng i n premenopaus al women.
o
o
b. Systemic manifestations i ncl ude bowel obs t ruct i on, jaundi ce, as ci t es , and pai n.
6. Diagnosis o
o
a. Mos t women wi t h endomet ri al cancer have abnormal ut eri ne bl eedi ng.
o
o
b. Onl y 15%–20% of cas es of endomet ri al cancer are i dent i fi ed by Pap s mear. Di agnos t i c procedures i ncl ude bi ops y, fract i onal di l at at i on and curet t age (D&C), endomet ri al brus h, and jet -was h t echni ques .
o
o
c. Pretreatment evaluation i ncl udes rout i ne
Pa g e 7 9 2
ABC Ambe r CHM Conve rte r Tria l ve rsion, http://w w w .proce sste x t.com/a bcchm.html
hemat ol ogi c and bl ood chemi s t ri es , uri nal ys i s , ches t radi ograph, and occas i onal l y, hys t eros copy or hys t erography.
(1) CT and cystoscopy s houl d be performed i f t here i s evi dence of bl adder dys funct i on.
(2) Pat i ent s wi t h l ower gas t roi nt es t i nal s ympt oms s houl d undergo proctosigmoidoscopy and barium enema.
(3) Lymphangiography may be us ed t o defi ne i nvol vement of t he para-aort i c nodes i n hi gh-ri s k pat i ent s . It i s performed uncommonl y.
7. Prognosis o
o
a. Stage at diagnosis i s t he mos t i mport ant prognos t i c fact or. Fi ve-year s urvi val rat es are 86% for s t age I, 66% for s t age II, 44% for s t age III, and 16% for s t age IV di s eas e.
o
o
b. Ot her i mport ant prognos t i c fact ors i ncl ude t he extent of cervical and myometrial invasion, lymph node involvement (es peci al l y pel vi c or para-aort i c nodes ), and histologic grade. Pres ence of myomet ri al i nvas i on, vas cul ar s pace i nvas i on, and hi gh hi s t ol ogi c grade are among negat i ve prognos t i c fact ors . Of l es s er i mport ance are ut eri ne s i ze, pos i t i ve peri t oneal cyt ol ogy, cel l t ype, and
Pa g e 7 9 3
ABC Ambe r CHM Conve rte r Tria l ve rsion, http://w w w .proce sste x t.com/a bcchm.html
pat i ent age.
8. Staging. For ful l s t agi ng i nformat i on, pl eas e s ee t he mos t recent edi t i on of t he AJCC Cancer St agi ng Handbook. Sevent y-four percent of pat i ent s who are s een have s t age I di s eas e; 13%, s t age II di s eas e; 9%, s t age III di s eas e; and 3%, s t age IV di s eas e.
9. T herapy o
o
a. Treat ment meas ures i ncl ude surgery, radiation therapy, hormone therapy, and, occas i onal l y, chemotherapy.
(1) Early stage disease:
(i) Low ri s k (s t age IA, IB, IC grade 1): s urgery onl y wi t h t ot al abdomi nal hys t erect omy/bi l at eral s al pi ngo-oophorect omy (TAH/BSO). P.159
(ii) Int ermedi at e ri s k (s t age II, grade 1): TAH/BSO wi t h para-aort i c and s el ect i ve pel vi c l ymph node di s s ect i on. Pos t operat i ve adjuvant radi at i on t herapy maybe offered t o pat i ent s wi t h t umors wi t h deep myomet ri al i nvas i on, hi gh-grade hi s t ol ogy, cervi cal
Pa g e 7 9 4
ABC Ambe r CHM Conve rte r Tria l ve rsion, http://w w w .proce sste x t.com/a bcchm.html
i nvol vement , and vas cul ar s pace i nvas i on.
(iii) Hi gh ri s k (s t age IB or C hi gh grade, any s t age II or III): TAH/BSO and peri aort i c l ymph node s ampl i ng and pos t operat i ve radi at i on.
(iv) There are i ns uffi ci ent dat a t o recommend adjuvant chemot herapy at t hi s t i me.
(2) Metastatic Disease:
(i) Synt het i c proges t ogens are t he mos t commonl y us ed hormone t herapy agent s , wi t h res pons e rat es of approxi mat el y 35%. The probabi l i t y of res pons e depends on t he hi s t ol ogi c grade of t he t umor and t he pres ence of proges t erone recept ors . W el l -di fferent i at ed t umors have t he hi ghes t res pons e rat es .
(ii) Chemot herapy i s of l i t t l e benefi t , but s ome act i vi t y has been s een wi t h cycl ophos phami de, 5-FU, doxorubi ci n, and ci s pl at i n.
o
o
b. Specific treatment recommendations. Adenomat ous hyperpl as i a can be t reat ed wi t h hys t erect omy or hormone t herapy, dependi ng on
Pa g e 7 9 5
ABC Ambe r CHM Conve rte r Tria l ve rsion, http://w w w .proce sste x t.com/a bcchm.html
t he pat i ent 's des i re for chi l dren.
(1) Cont i nuous combi nat i on es t rogen and proges t erone cont racept i ve agent s or hi gh-dos e proges t ogens are us ed i n hormone t herapy. Ovul at i on can be i nduced wi t h cl omi phene.
(2) Pat i ent s mai nt ai ned on hormone t herapy s houl d be careful l y moni t ored and undergo rout i ne endomet ri al s ampl i ng.
IX. Cancer of Unknown Primary Site (CUPS) A. Incidence Pat i ent s wi t h CUPS have met as t at i c carci noma i n t he abs ence of a demons t rabl e pri mary s i t e. Approxi mat el y 2%–10% of new cancer pat i ent s have CUPS. Even at aut ops y, t he pri mary s i t e remai ns obs cure i n 15%–20% of t hes e pat i ent s .
B. Screening Becaus e t he pri mary s i t e i s unknown, t here are no i dent i fi abl e ri s k fact ors or s creeni ng recommendat i ons .
C. Pathology
1. Histologic subtypes o
o
a. Adenocarci noma i s t he mos t common s ubt ype, occurri ng i n 40%–77% of pat i ent s .
o
o
b. Squamous cel l carci nomas are pres ent i n 5%–15% of pat i ent s .
Pa g e 7 9 6
ABC Ambe r CHM Conve rte r Tria l ve rsion, http://w w w .proce sste x t.com/a bcchm.html o
o
c. Ot her cel l t ypes i ncl ude l ymphomas , germ cel l t umors , mel anomas , and s arcomas .
(1) Lymphoma can s omet i mes be di s t i ngui s hed from undi fferent i at ed carci noma by s peci al i mmunohi s t ochemi cal s t ai ns for l eukocyt e markers s uch as l eukocyt e common ant i gen.
(2) Metastatic small cell endocrine tumors can be i dent i fi ed by s t ai ni ng for S-100 or neuron-s peci fi c enol as e.
(3) Germ cell tumors may s t ai n pos i t i ve for AFP or hCG.
2. Li ght mi cros copy, el ect ron mi cros copy, and i mmunohi s t ochemi cal t echni ques may be us eful i n i dent i fyi ng t he hi s t ol ogi c s ubt ype or t i s s ue ori gi n. o
o
a. Vi s ual i zat i on of t he cel l ul ar archi t ect ure by light microscopy may i dent i fy adenocarci nomas , s quamous cel l carci nomas , mel anoma, s arcoma, or l ymphoma. However, becaus e CUPS i s oft en anapl as t i c, di fferent i at i on by l i ght mi cros copy i s not al ways pos s i bl e.
o
o
b. Immunohistochemical markers are i mport ant pat hol ogi c t ool s . For exampl e, a pos i t i ve kerat i n
Pa g e 7 9 7
ABC Ambe r CHM Conve rte r Tria l ve rsion, http://w w w .proce sste x t.com/a bcchm.html
s t ai n i mpl i es t hat t he t umor i s of epi t hel i al ori gi n. o
o
c. Electron microscopy det ect s intercellular bridging, charact eri s t i c of s quamous cel l carci nomas ; neurosecretory granules, charact eri s t i c of neuroendocri ne t umors ; melanosomes, charact eri s t i c of mel anoma; and intracellular structures s uch as myofi bri l s , charact eri s t i c of s arcomas .
D. Clinical features At pres ent at i on, mos t pat i ent s compl ai n of s ympt oms at met as t at i c s i t es . Nons peci fi c cons t i t ut i onal s ympt oms are al s o common: anorexi a, wei ght l os s , and fat i gue.
E. Diagnosis Becaus e pancreat i c, l ung, and col on cancers are t he mos t l i kel y pri mary s ources of undi fferent i at ed adenocarci noma, rout i ne di agnos t i c procedures i ncl ude a ches t radi ograph, fecal occul t bl ood t es t (accompani ed by col onos copy i n pos i t i ve cas es ), and abdomi nal CT. P.160
1. Women s houl d al s o undergo mammography, pelvic examination, and pelvic ultrasound.
2. Prostate-specific antigen (PSA) levels s houl d be det ermi ned i n men.
3. Pat i ent s wi t h s us pect ed germ cel l carci nomas s houl d al s o undergo β-hCG and AFP det ermi nat i ons .
Pa g e 7 9 8
ABC Ambe r CHM Conve rte r Tria l ve rsion, http://w w w .proce sste x t.com/a bcchm.html
F. Prognosis Medi an s urvi val i s onl y 5–6 mont hs aft er di agnos i s , al t hough t he range i s broad; 3%–5% of pat i ent s may be al i ve aft er 5 years .
G. Therapy Indi vi dual s , part i cul arl y men, wi t h CUPS pres ent i ng wi t h mi dl i ne-domi nant l es i ons (e.g., para-aort i c l ymphadenopat hy, medi as t i nal l ymphadenopat hy) and pul monary nodul es s houl d be t reat ed for poor prognos i s nons emi nomat ous germ cel l t umors . Treat ment vari es accordi ng t o t he s us pect ed pri mary s i t e but t ypi cal l y i ncl udes pal l i at i ve radi at i on t herapy or chemot herapy.
1. Slow-growing tumors may merel y be moni t ored at 4t o 8-week i nt erval s .
2. Chemot herapy i s an experi ment al opt i on for adenocarcinoma. W omen wi t h s us pect ed adenocarci noma i n an axi l l ary node s houl d be cons i dered t o have s t age II breas t cancer and be t reat ed accordi ngl y.
3. Pat i ent s wi t h squamous cell carcinoma metastatic to the neck from an unknown pri mary s i t e, but no di s t ant met as t as es , s houl d be t reat ed wi t h neck di s s ect i on and radi at i on t herapy t o t he neck.
4. Y oung men wi t h anaplastic tumors of the mediastinum or retroperitoneum may res pond t o t reat ment modal i t i es us ed for germ cel l t umors of t he t es t i s , es peci al l y i f AFP or β-hCG l evel s are el evat ed.
X. Cancers of the Kidney, Bladder, and Prostate Pa g e 7 9 9
ABC Ambe r CHM Conve rte r Tria l ve rsion, http://w w w .proce sste x t.com/a bcchm.html
A. Renal cell cancer
1. Incidence. Cancer of t he ki dney (hypernephroma) account s for 2%–3% of al l cancers . About 31,900 new cas es are di agnos ed each year. The rat i o of affect ed men t o women i s 2:1, wi t h an average age of 55–60 years at pres ent at i on.
2. Risk factors. Pat i ent s wi t h von Hi ppel -Li ndau di s eas e or acqui red pol ycys t i c ki dney di s eas e have a hi gher i nci dence of renal carci noma. Fami l i al t ypes of renal cel l carci noma have been as s oci at ed wi t h abnormal i t i es of chromos ome 3 and t he p53 gene. Ot her ri s k fact ors i ncl ude t obacco us e, hi gh cons umpt i on of fri ed or s aut éed meat s , obes i t y, and expos ure t o as bes t os and pet rol eum product s .
3. Screening. There are current l y no s creeni ng gui del i nes for earl y det ect i on i n as ympt omat i c peopl e.
4. Pathology o
o
a. Mos t renal t umors are cl ear cel l carci nomas (adenocarci nomas ); however, granul ar cel l t umors , mi xed t umors , and aggres s i ve carci nos arcomas (adenocarci nomas wi t h s arcomat ous degenerat i on) al s o occur. Rarel y, t umors ari s e i n renal cys t s .
o
o
b. Renal cancers are us ual l y extremely vascular, readily metastasizing t o t he l ung, bone, l i ver,
Pa g e 8 0 0
ABC Ambe r CHM Conve rte r Tria l ve rsion, http://w w w .proce sste x t.com/a bcchm.html
brai n, and ot her s i t es . Di s s emi nat i on al s o may occur when t umor embol i ent er t he renal vei n and i nferi or vena cava or by nodal s pread i n t he para-aort i c, paracaval , hi l ar, and medi as t i nal areas .
5. Clinical features o
o
a. Hemat uri a (40%–70% of pat i ent s )
o
o
b. Abdomi nal mas s wi t h fl ank pai n (20%–40% of pat i ent s )
o
o
c. W ei ght l os s (30% of pat i ent s )
o
o
d. Fever, mal ai s e, ni ght s weat s , or anemi a (15%–30% of pat i ent s )
o
o
e. Paraneopl as t i c s yndromes , i ncl udi ng hypercal cemi a, pol ycyt hemi a, or hypert ens i on
6. Diagnosis. In t he pas t , di agnos t i c procedures i ncl uded hemat ol ogi c s t udi es , i nt ravenous pyel ography (IVP), renal ul t ras ound, and, i n s ome cas es , a s el ect i ve renal art eri ogram or venacavogram. Now, ches t radi ographs , abdomi nal CT, and radi onucl i de bone s cans are us ed t o det ect t he pres ence of met as t as es . CT s cans are adequat e for i ni t i al di agnos i s and s t agi ng. MRI s cans are s uperi or for vi s ual i zi ng t umor or cl ot i n t he renal vei n and vena cava.
Pa g e 8 0 1
ABC Ambe r CHM Conve rte r Tria l ve rsion, http://w w w .proce sste x t.com/a bcchm.html
P.161
7. Prognosis. Radi cal nephrect omy yi el ds a 5-year s urvi val rat e i n 75% of pat i ent s wi t h l ocal i zed t umors . However, pat i ent s wi t h hi gh-grade s t age III l es i ons wi t h i nvas i on of major vas cul ar s t ruct ures have an overal l s urvi val rat e of onl y 30%–40%.
8. Staging. For ful l s t agi ng i nformat i on, pl eas e s ee t he mos t recent edi t i on of t he AJCC Cancer St agi ng Handbook.
9. T herapy o
o
a. Surgery
(1) Radical nephrectomy wi t h l ymph node di s s ect i on i s t he t reat ment of choi ce for s t age I, s t age II, and s t age III renal cel l carci nomas . Nephrect omy s houl d al s o be cons i dered for pat i ent s wi t h di s s emi nat ed di s eas e t o rel i eve hemat uri a, fl ank pai n, or paraneopl as t i c s yndromes .
(2) In cert ai n cas es , selective renal arterial embolization permi t s s urgi cal res ect i on or hal t s l i fe-t hreat eni ng hemat uri a.
Pa g e 8 0 2
ABC Ambe r CHM Conve rte r Tria l ve rsion, http://w w w .proce sste x t.com/a bcchm.html
(3) Pat i ent s wi t h one or t wo met as t at i c l es i ons , i ndol ent t umor growt h, and no evi dence of progres s i ve met as t as i s may be candi dat es for aggres s i ve surgical resection of metastases.
o
o
b. Radiation therapy has onl y l i mi t ed val ue becaus e mos t t umors are rel at i vel y radi ores i s t ant . However, radi at i on t herapy offers t he bes t pal l i at i ve opt i ons for pai nful bone met as t as es or nerve root compres s i ons .
o
o
c. Chemotherapy i s al s o rel at i vel y i neffect i ve, wi t h t rans i ent t umor regres s i on occurri ng i n fewer t han 5% of pat i ent s . Target ed t herapy wi t h agent s di rect ed agai ns t t he i nt racel l ul ar t yros i ne ki nas es are under act i ve i nves t i gat i on. Suni t i ni b (Sut ent ), whi ch t arget s pl at el et -deri ved growt h fact or recept or (PDGFR), Vas cul ar endot hel i al growt h fact or (VEGFR), KIT (CD 117), and FMS-l i ke t yros i ne ki nas e 3 (FLT-3)l and s orafeni b (Nexavar), a mul t i ki nas e i nhi bi t or, are t wo oral agent s t hat have demons t rat ed promi s i ng effi cacy and are now FDA-approved for t herapy of advanced renal cel l cancer.
o
o
d. Biologic response modifiers
(1) IFN-α yi el ds res pons e rat es of 13%–20% i n pat i ent s wi t h s oft t i s s ue or l ung met as t as es .
Pa g e 8 0 3
ABC Ambe r CHM Conve rte r Tria l ve rsion, http://w w w .proce sste x t.com/a bcchm.html
(2) Hi gh-dos e IL-2 t herapy yi el ds part i al or compl et e t umor regres s i on i n approxi mat el y 15% of cas es . However, t he t reat ment i s ext remel y t oxi c and requi res a hi gh l evel of s upport i ve care. Al t hough mos t res pons es are l i mi t ed i n durat i on, compl et e res pons es t end t o be durabl e.
B. Urothelial (transitional cell) carcinomas (bl adder, uret er, and renal pel vi s )
1. Incidence. Thes e t umors account for approxi mat el y 3% of t he cancer deat hs i n t he Uni t ed St at es . Bl adder carci nomas are t hree t i mes more common i n men t han i n women and us ual l y occur i n pat i ent s who are 50–70 years of age. The annual i nci dence i s approxi mat el y 57,400 cas es wi t h 12,500 deat hs and i ncl udes 80%–85% of pat i ent s t hat pres ent wi t h s uperfi ci al (e.g., T a , T 1 s , T 1 ) t umors wi t hout mus cl e-wal l i nvas i on at di agnos i s .
2. Risk factors. Bl adder cancers have been rel at ed t o tobacco us e as wel l as t o expos ure t o cert ai n chemi cal and bi ol ogi c carci nogens . o
o
a. Occupational carcinogens i n t he rubber, dye, pri nt i ng, and chemi cal i ndus t ri es have been i mpl i cat ed i n bl adder carci noma.
o
o
b. Saccharin has been proved t o caus e bl adder t umors i n ani mal s , but i t s rol e i n humans i s unproven.
Pa g e 8 0 4
ABC Ambe r CHM Conve rte r Tria l ve rsion, http://w w w .proce sste x t.com/a bcchm.html o
o
c. Schi s t os omi as i s of t he bl adder has been s t rongl y correl at ed wi t h s quamous cel l carci noma.
o
o
d. Ot her et i ol ogi c agent s i ncl ude cyclophosphamide, phenacetin, renal stones, and chronic infection.
o
o
e. Molecular alterations i n t he p53 and Rb t umor s uppres s or genes are i nvol ved i n 20%–40% of t umors , dependi ng on t he s t age and grade of t he pri mary t umor. The prognos i s wors ens when one or more of t hes e genes i s mut at ed.
3. Screening. There are current l y no s creeni ng gui del i nes for earl y det ect i on i n as ympt omat i c peopl e.
4. Pathology. Bl adder t umors are general l y of t rans i t i onal cel l ori gi n, wi t h t he except i on of t he s chi s t os ome-rel at ed vari ant , whi ch i s of s quamous cel l ori gi n.
5. Clinical features. Ei ght y-fi ve percent of bl adder cancer pat i ent s pres ent wi t h hematuria. Bladder irritability and infection are t he pres ent i ng s ympt oms i n t he remai ni ng 25%. P.162
Cons t i t ut i onal s ympt oms s uch as wei ght l os s , abdomi nal pai n, or bone pai n may be pres ent i n pat i ent s wi t h advanced
Pa g e 8 0 5
ABC Ambe r CHM Conve rte r Tria l ve rsion, http://w w w .proce sste x t.com/a bcchm.html
di s eas e.
6. Diagnosis o
o
a. Cystoscopy, wi t h bi manual pal pat i on and s ubs equent bi ops y, i s t he defi ni t i ve di agnos t i c procedure. Thi s t echni que i s nearl y 100% accurat e.
o
o
b. Radiologic procedures i ncl ude pel vi c and abdomi nal CT s cans , ches t radi ograph, bone s can, and ret rograde pyel ography for renal pel vi c or uret eral t umors .
o
o
c. Laboratory tests i ncl ude a CBC and chemi s t ry profi l e as wel l as uri ne cyt ol ogy and fl ow cyt omet ry for hi gh-ri s k pat i ent s .
7. Prognosis o
o
a. Major prognos t i c fact ors are t he t umor s t age at t i me of di agnos i s and degree of di fferent i at i on of t he t umor. Ot her advers e prognos t i c fact ors i ncl ude ol d age, expres s i on of p53, aneupl oi dy, t umor mul t i focal i t y, and pres ence of a pal pabl e mas s .
o
o
b. Fi ve-year rel at i ve s urvi val rat es for pat i ent s wi t h s uperfi ci al , mus cl e-i nvas i ve, and met as t at i c bl adder cancer are 95%, 50%, and 6%, res pect i vel y.
Pa g e 8 0 6
ABC Ambe r CHM Conve rte r Tria l ve rsion, http://w w w .proce sste x t.com/a bcchm.html
8. Staging. For ful l s t agi ng i nformat i on, pl eas e s ee t he mos t recent edi t i on of t he AJCC Cancer St agi ng Handbook.
9. T herapy o
o
a. Surgery
(1) Superficial lesions (s t ages T a , T 1 , and s omet i mes T 2 a ) are t reat ed wi t h endoscopic resection and fulguration, fol l owed by cys t os copy every 3 mont hs . Recurrent or mul t i pl e l es i ons are t reat ed wi t h i nt raves i cul ar i ns t i l l at i on of t hi ot epa, mi t omyci n, or BCG.
(a) The recurrence rat e aft er t rans uret hral res ect i on (TUR) i s 50%–70%. The recurrence rat e i s hi gher i f t he s i ze of t he t umor i s great er t han 3 cm, i f i t i s mul t i focal and as s oci at ed wi t h CIS, or i f i t i s a hi gh-grade T 1 l es i on.
(b) Even wi t h BCG prophyl axi s aft er TUR, mos t pat i ent s devel op recurrences , and s ome progres s t o mus cl e-i nvas i ve di s eas e.
Pa g e 8 0 7
ABC Ambe r CHM Conve rte r Tria l ve rsion, http://w w w .proce sste x t.com/a bcchm.html
(2) Recurrent cancer, diffuse transitional cell CIS, or stages T 2 and T 3 invasive cancers are t reat ed wi t h radical cystectomy.
(3) Tumors of t he renal pel vi s or uret er are t reat ed by removal of the affected kidney or ureter.
o
o
b. Combination chemotherapy
(1) In randomi zed s t udi es , combi nat i on chemot herapy wi t h gemci t abi ne and ci s pl at i n has been s hown t o be as effect i ve and much l es s t oxi c and bet t er t ol erat ed t han MVAC [met hot rexat e-vi nbl as t i ne-doxorubi ci n (Adri amyci n)-ci s pl at i n] i n pat i ent s wi t h advanced di s eas e. However, t hes e s t udi es were not powered t o det ect a s urvi val benefi t wi t h t hi s regi men.
(2) Newer combi nat i on regi mens under i nves t i gat i on i ncl ude gemci t abi ne pl us pacl i t axel , ci s pl at i n pl us pacl i t axel , and pacl i t axel pl us carbopl at i n.
C. Prostate carcinoma
1. Incidence. Cancer of t he pros t at e account s for 33% of al l cancers i n t he Uni t ed St at es . Approxi mat el y, 234,500 new cas es are di agnos ed each year. Inci dence rat es are hi gher among bl acks and i ncreas e wi t h age. The di s eas e i s much l es s common i n As i an men. At
Pa g e 8 0 8
ABC Ambe r CHM Conve rte r Tria l ve rsion, http://w w w .proce sste x t.com/a bcchm.html
aut ops y, t he i nci dence among men ol der t han age 50 years ranges from 14%–46%. In t he Uni t ed St at es , t he ri s k of deat h i s approxi mat el y 9%.
2. Risk factors. Ri s k fact ors i ncl ude age, family history, and race. Geographi c l ocat i on has al s o been i mpl i cat ed wi t h rat es of pros t at e cancer l owes t i n As i a and hi gher i n Scandi navi a and t he Uni t ed St at es .
3. Screening. Current l y t here i s i ns uffi ci ent evi dence regardi ng t he val ue of t es t i ng for earl y pros t at e cancer i n t he average-ri s k man. However, t he ACS recommends t hat t he PSA t es t and di gi t al rect al exam (DRE) s houl d be offered annual l y begi nni ng at age 50 years t o men of average ri s k whos e l i fe expect ancy i s at l eas t 10 years and at t he age of 40 for Afri can Ameri can men and for t hos e wi t h a fami l y hi s t ory of pros t at e cancer. P.163
4. Pathology. Nearl y al l pros t at e cancers are adenocarci nomas . o
o
a. Pros t at e adenocarci nomas are cl as s i fi ed as well-, moderately well-, or poorly differentiated, accordi ng t o t he Gleason prognostic classification system, whi ch as s i gns a grade bet ween 0 and 5 t o t he pri mary and s econdary di fferent i at i on pat t erns .
o
o
b. Mos t pros t at i c t umors ori gi nat e i n t he peri pheral zone; onl y 25% ori gi nat e i n t he cent ral zone.
Pa g e 8 0 9
ABC Ambe r CHM Conve rte r Tria l ve rsion, http://w w w .proce sste x t.com/a bcchm.html o
o
c. More t han 90% of di s t ant met as t as es i nvol ve bone, but s oft t i s s ue met as t as es al s o can occur i n t he nodes , l ung, and l i ver.
5. Clinical features. Pres ent i ng charact eri s t i cs once i ncl uded a pal pabl e nodul e (i n >50% of pat i ent s ), dys uri a, uri nary ret ent i on, t ermi nal hemat uri a, and uri nary dri bbl i ng, frequency, or urgency. Current l y, when PSA l evel s i nfl uence di agnos i s , nonpal pabl e (T1c) t umors are more common. Sys t emi c mani fes t at i ons of met as t at i c di s eas e i ncl ude bone pai n, wei ght l os s , and, rarel y, s pi nal cord compres s i on.
6. Diagnosis o
o
a. Diagnosis
(1) DRE remai ns t he di agnos t i c gol d s t andard for pros t at e carci noma, even t hough onl y 10% of t he pros t at i c t umors found as nodul es on rect al exami nat i on are s uffi ci ent l y l ocal i zed for cure.
(2) Pathologic examination of t i s s ue removed for t reat ment of obs t ruct i ve pros t at i c hypert rophy s hows t hat 10% of cas es are mal i gnant . The remai ni ng cas es , when found, are i n advanced s t ages ; oft en t hes e cancers are revealed in investigations for metastatic bone disease.
Pa g e 8 1 0
ABC Ambe r CHM Conve rte r Tria l ve rsion, http://w w w .proce sste x t.com/a bcchm.html
(3) PSA i s an es t abl i s hed di agnos t i c marker, but fal s e-pos i t i ve val ues may occur. If t he 3
PSA i s more t han 10 ng/mm , t hen 66% of t he pros t at e bi ops i es i ndi cat e pros t at e cancer. If 3
t he PSA i s bet ween 4 and 10 ng/mm , t hen 22% of t he bi ops i es wi l l be pos i t i ve. The percent age of free PSA (e.g., unbound t o pl as ma α 1 -ant i chymot ryps i n) may i ncreas e di agnos t i c s peci fi ci t y, but t hi s occurs at t he cos t of overal l s ens i t i vi t y.
(4) T ransrectal ultrasonography (T RUS) i s mos t us eful for eval uat i on of pros t at e s i ze and for t he preci s e bi ops y of l es i ons t hat are not pal pabl e. However, as a s ol e di agnos t i c t ool , i t i s i ns ens i t i ve and nons peci fi c.
o
o
b. Confirmation
(1) Needle biopsy, obt ai ned vi a t he rect um, peri neum, or uret hra, confi rms t he di agnos i s .
(2) Laboratory studies are us ed t o as s es s renal funct i on, whereas bone s cans , radi ographs , and pel vi c or abdomi nal CT confi rm t he pres ence of met as t at i c di s eas e.
7. Staging. For ful l s t agi ng i nformat i on, pl eas e s ee t he mos t recent edi t i on of t he AJCC Cancer St agi ng Handbook. St agi ng t echni ques are us ed al ong wi t h t he
Pa g e 8 1 1
ABC Ambe r CHM Conve rte r Tria l ve rsion, http://w w w .proce sste x t.com/a bcchm.html
hi s t ol ogi c grade and t he PSA l evel t o det ermi ne t he probabi l i t y t hat carci noma l i mi t ed t o t he pros t at e i s curabl e by s urgery or radi at i on. Cl i ni cal s t agi ng al one i s not accurat e. o
o
a. Stage T 1 A and T 1 B tumors are uns us pect ed cl i ni cal l y and are found at aut ops y or by exami ni ng t i s s ue removed for al l eged beni gn di s eas e. T 1 A ( well-differentiated wi t h t hree or fewer foci ) t umors have bet t er prognos es t han T 1 B (poorly differentiated, or more t han t hree foci ) t umors . T 1 C t umors , whi ch are nonpal pabl e, are det ect ed by el evat ed PSA l evel s . Pros t at e cancers are mos t commonl y di agnos ed at t he T 1 C s t age.
o
o
b. Stage T 2 tumors are neoplasms that are confined to the prostate gland. Thes e are t he cl as s i c nodul es found on rect al exami nat i on, whi ch t heoret i cal l y are curabl e by s urgery. Cl i ni cal l y pal pabl e l es i ons i n t he pros t at e are s t age T 2 A (i nvol vi ng one l obe) and s t age T 2 B (bi l at eral ).
o
o
c. Stage T 3 tumors are cancers t hat have s pread beyond t he pros t at e caps ul e or t o t he s emi nal ves i cl es , but not t o di s t ant s i t es . Thes e us ual l y cannot be cured by s urgery.
o
o
d. Stage T 4 tumors are cancers t hat have s pread from t he pros t at e area t o pel vi c l ymph nodes , bone, or el s ewhere. Approxi mat el y 50% of newl y di agnos ed cas es are s t age IV on pres ent at i on.
Pa g e 8 1 2
ABC Ambe r CHM Conve rte r Tria l ve rsion, http://w w w .proce sste x t.com/a bcchm.html
8. T herapy and prognosis are det ermi ned by t he cl i ni cal s t age of di s eas e, PSA l evel , and hi s t ol ogi c (Gl eas on) grade at di agnos i s . Comorbi di t i es and l i fe expect anci es al s o fact or i nt o t reat ment deci s i ons . o
o
a. Early disease i s t reat ed wi t h radi cal pros t at ect omy, ext ernal beam radi at i on, or i nt ers t i t i al i mpl ant at i on radi at i on t herapy. In s el ect ed cas es , no t reat ment (e.g., expect ant management ) i s appropri at e.
(1) Prostatectomy i s performed i n pat i ent s wi t h at l eas t a 10-year l i fe expect ancy. Pat i ent s wi t h cancers of s t age T 1 B , T 2 A , and T 2 B , and younger pat i ent s wi t h T 1 A di s eas e are candi dat es for s urgi cal cure. Up t o 16% of pat i ent s pres ent i ng wi t h T 1 A di s eas e, who are t hought t o have beni gn di s eas e, progres s .
(2) Nerve-sparing radical prostatectomy i s appropri at e for pat i ent s wi t h s mal l l es i ons . Thi s s urgi cal approach pres erves s exual funct i oni ng i n s ome men but i s as s oci at ed wi t h a 5%–15% rat e of pos t s urgi cal uri nary i ncont i nence and depends on t he expert i s e and t rai ni ng of t he s urgeon. P.164
(3) Radiation therapy i s us ed for ol der pat i ent s , t hos e wi t h ot her medi cal di s orders
Pa g e 8 1 3
ABC Ambe r CHM Conve rte r Tria l ve rsion, http://w w w .proce sste x t.com/a bcchm.html
or l arge pros t at i c l es i ons precl udi ng s urgery, and for men who wi s h a bet t er chance t o ret ai n normal s exual act i vi t y. W i t h modern t echni ques , i ncl udi ng conformal radi at i on t herapy, 70–75 Gy provides good l ocal cont rol wi t h accept abl e morbi di t y. Ext ernal beam radi at i on i s us eful i n pal l i at i ng met as t at i c bone di s eas e. St ront i um-89 or s amari um-153 may be us eful when admi ni s t e red i nt ravenous l y t o pal l i at e pai n i n pat i ent s wi t h wi des pread bone di s eas e. Novel t echni ques s uch as IMRT are us ed i n s el ect cent ers t o del i ver hi gh radi at i on dos e wi t h i mproved normal t i s s ue t ol erance. Di arrhea, rect al bl eedi ng, and proct i t i s are s omet i mes as s oci at ed wi t h t hi s form of t reat ment . o
o
b. Stage IV tumors cannot be cured, but s urvi val may be l ong (>5 years ) i n s ubgroups of pat i ent s wi t h l ow-grade t umors . The earl y us e of androgen s uppres s i on al s o may i mprove s urvi val .
(1) Endocrine therapy i s general l y t he i ni t i al mode of t reat ment .
(a) In t he pas t , orchiectomy and exogenous estrogens were us ed i nt erchangeabl y.
(i) Orchi ect omy i s preferred i n pat i ent s wi t h cardi ovas cul ar or t hrombot i c ri s k.
Pa g e 8 1 4
ABC Ambe r CHM Conve rte r Tria l ve rsion, http://w w w .proce sste x t.com/a bcchm.html
(ii) Pos t s urgi cal admi ni s t rat i on of exogenous hormones s uch as di et hyl s t i l bes t rol (DES; 1–3 mg dai l y) furt her s uppres s es t es t os t erone l evel s .
(b) More recent approaches have focus ed on t he LH-RH anal og leuprolide (Zol adex), ei t her al one or i n combi nat i on wi t h an ant i -androgen (flutamide, bi cal ut ami de, or ni l ut ami de). Aminoglutethimide or ketoconazole plus a corticosteroid are opt i ons for pat i ent s who fai l t o res pond t o pri mary hormone t reat ment .
(i) Thes e hormonal mani pul at i ons i nduce remi s s i ons i n approxi mat el y 50%–80% of pat i ent s , al t hough cures are rare.
(ii) Typi cal l y, pros t at e and s oft t i s s ue l es i ons regres s , aci d phos phat as e and PSA l evel s decl i ne t oward normal val ues , and bone pai n decreas es .
(iii) The average durat i on of t he i ni t i al hormone res pons e i s 9–18 mont hs .
Pa g e 8 1 5
ABC Ambe r CHM Conve rte r Tria l ve rsion, http://w w w .proce sste x t.com/a bcchm.html
(c) Ot her hormonal mani pul at i ons may produce t rans i ent res pons es .
(2) Chemotherapy has i mproved and i s des t i ned t o be us ed i n earl i er s t ages of di s eas e. The mos t frequent l y us ed agent s are es t ramus t i ne pl us ei t her vi nbl as t i ne, pacl i t axel , docet axel , or et opos i de. Mi t oxant rone pl us a cort i cos t eroi d has es t abl i s hed pal l i at i ve val ue beyond t hat of cort i cos t eroi ds al one, and t he FDA has approved t he us e of t hi s combi nat i on i n hormone-refract ory pros t at e cancer. Doxorubi ci n and cycl ophos phami de al s o may produce pal l i at i on. Treat ment opt i ons may be l i mi t ed for met as t at i c hormone refract ory pros t at e cancer becaus e of advanced age and addi t i onal co-morbi di t i es .
XI. Testicular Cancer A. Incidence Tes t i cul ar cancer i s t he mos t common mal i gnancy i n young men and account s for 1% of al l mal e cancers . Approxi mat el y 7,600 new cas es are det ect ed each year i n t he Uni t ed St at es . The average age at di agnos i s i s 32 years . Becaus e t hi s cancer can be cured wi t h vi gi l ant moni t ori ng and met i cul ous t herapy, appropri at e management i s es peci al l y i mperat i ve.
B. Risk factors The pres ence of a cryptorchid testicle and a prior history of t es t i cul ar cancer pres ent a ri s k.
C. Screening Becaus e of t he l ow preval ence of t es t i cul ar cancer, l i mi t ed accuracy of s creeni ng t es t s , t he U.S. Prevent i t i ve Servi ces Tas k Force does not recommend rout i ne s creeni ng for t es t i cul ar cancer i n
Pa g e 8 1 6
ABC Ambe r CHM Conve rte r Tria l ve rsion, http://w w w .proce sste x t.com/a bcchm.html
as ympt omat i c adol es cent and adul t mal es . The ACS, however, does recommend annual t es t i cul ar exams . Of not e, t es t i cul ar cancer aft er t he age of 40 i s uncommon.
D. Pathology Tes t i cul ar t umors are cl as s i fi ed as seminomatous or nonseminomatous.
1. Seminomas may be furt her s ubcl as s i fi ed as classical, anaplastic, or spermatocystic.
2. Four hi s t ol ogi c t ypes of nonseminomatous tumors are recogni zed. o
o
a. Embryonal carcinoma
o
o
b. T eratoma
o
o
c. Yolk sac carcinoma (al s o known as endodermal sinus tumor)
o
o
d. Choriocarcinoma
P.165
E. Clinical features
1. More t han 90%–95% of pat i ent s have a painless, solid testicular swelling. Occas i onal l y, pat i ent s wi t h
Pa g e 8 1 7
ABC Ambe r CHM Conve rte r Tria l ve rsion, http://w w w .proce sste x t.com/a bcchm.html
pai nful t es t i cul ar mas s es are erroneous l y di agnos ed as havi ng epi di dymi t i s or orchi t i s .
2. Para-aort i c l ymph node i nvol vement can mani fes t as ureteral obstruction, back pai n, or new-ons et vari cocel e.
3. Pat i ent s al s o may have abdominal complaints from an abdomi nal mas s or pul monary s ympt oms from mul t i pl e nodul es .
4. Gynecomastia and hyperthyroidism may occur i n pat i ent s wi t h el evat ed β-hCG l evel s . Uni l at eral gynecomas t i a may occur aft er s ucces s ful t reat ment as a cons equence of t es t i cul ar fai l ure and l ow t es t os t erone l evel s aft er chemot herapy.
F. Diagnosis
1. Scrotal ultrasound may s how a s us pi ci ous i nt rat es t i cul ar echogeni c focus . Ul t ras ound can al s o defi ne penet rat i on of t he s permat i c cord, epi di dymi s , and s crot um.
2. Radiologic tests may i ncl ude: o
o
a. Chest radiograph
o
o
b. CT scans of t he ches t , abdomen, and pel vi s , as wel l as t he brai n (i n pat i ent s wi t h neurol ogi c
Pa g e 8 1 8
ABC Ambe r CHM Conve rte r Tria l ve rsion, http://w w w .proce sste x t.com/a bcchm.html
s ympt oms ) o
o
c. Excretory urography (t o det ermi ne t he cours e of t he uret er)
o
o
d. Venacavogram
o
o
e. Bone scan (i f t here i s s kel et al pai n)
3. Inguinal exploratory surgery, i ncl udi ng hi gh l i gat i on of t he s permat i c cord and orchi ect omy, i s neces s ary for pat i ent s wi t h s us pi ci ous s crot al mas s es and no confi rmed di agnos i s . o
o
a. Vas cul ar cont rol mus t be achi eved before mani pul at i on of t he t umor.
o
o
b. Open bi ops y and s crot al expl orat i on are cont rai ndi cat ed becaus e of t he pos s i bi l i t y of t umor s pread.
4. Elevated blood levels of AFP or β-hCG are di agnos t i c for nons emi nomat ous germ cel l t umors . However, l evel s are normal i n many pat i ent s wi t h s emi nomas .
G. Prognosis For ful l s t agi ng i nformat i on, pl eas e s ee t he mos t recent edi t i on of t he AJCC Cancer St agi ng Handbook.
(Online T able 4-15)
Pa g e 8 1 9
ABC Ambe r CHM Conve rte r Tria l ve rsion, http://w w w .proce sste x t.com/a bcchm.html
1. Accordi ng t o t he Int ernat i onal Cons ens us Ri s k Cl as s i fi cat i on for germ cel l t umors , t he prognosis for bot h nons emi nomas and s emi nomas are cl as s i fi ed by t he pres ence of met as t as es and t he l evel of AFP, hCG, and LDH. Prognos i s i s s ubdi vi ded i nt o good, i nt ermedi at e, and poor.
2. Survival rates Expect ed 5-year s urvi val rat es for nons emi nomas for good, i nt ermedi at e, and poor prognos i s are 92%, 80%, and 48%, res pect i vel y. The expect ed 5-year s urvi val rat es for s emi nomas for good and i nt ermedi at e prognos i s are 86% and 72%, res pect i vel y. There i s no poor prognos i s cat egory for s emi nomas .
H. Staging For ful l s t agi ng i nformat i on, pl eas e s ee t he mos t recent edi t i on of t he AJCC Canc er St agi ng Handbook.
I. Therapy
1. Nonseminomatous tumors. In t he abs ence of advanced met as t as es requi ri ng i mmedi at e chemot herapy, mos t pat i ent s undergo orchi ect omy for defi ni t i ve t reat ment and hi s t ol ogi c anal ys i s of t he pri mary t umor. Approxi mat el y 30% of t hes e pat i ent s wi t h cl i ni cal s t age I nons emi nomat ous germ cel l t umors have occul t ret roperi t oneal l ymph node met as t as es . The ri s k i s hi gher (approxi mat el y 50%) i f t he pri mary t umor i s predomi nant l y embryonal carci noma and s hows vas cul ar i nvas i on. o
o
a. Clinical stage I disease (l i mi t ed t o t he t es t i s )
Pa g e 8 2 0
ABC Ambe r CHM Conve rte r Tria l ve rsion, http://w w w .proce sste x t.com/a bcchm.html
may be managed i n s everal ways :
(1) Retroperitoneal lymph node dissection, whi ch may be uni l at eral and t hus nerve-s pari ng wi t h pres ervat i on of ejacul at ory funct i on, i f nodes are negat i ve, may be neces s ary. Ot herwi s e, a bi l at eral procedure i s performed. Obs ervat i on wi t h ri gorous fol l ow-up, i ncl udi ng CT i magi ng and marker s t udi es , may be appropri at e. P.166
(2) Pri mary chemot herapy wi t h t wo cycl es of chemot herapy wi t h bl eomyci n, et opos i de, and ci s pl at i n, whi ch effect i vel y prevent s rel aps e, “overt reat s ― approxi mat el y 50% of pat i ent s .
(3) The t reat ment chos en depends on s everal fact ors , i ncl udi ng pat i ent age, rel i abi l i t y, feas i bi l i t y of cl os e fol l ow-up, ri s k of rel aps e (as es t i mat ed from pri mary t umor hi s t ol ogi c feat ures and CT s can), and need t o mai nt ai n fert i l i t y.
(a) In TN 1 -, TN 2 -, or M 0 -s t aged pat i ent s wi t h nons emi nomat ous cancer, t hos e wi t h s emi nomas and el evat ed AFP l evel s , or s emi nomas wi t h pers i s t ent β-hCG el evat i on aft er orchi ect omy, a bi l at eral ret roperi t oneal l ymph node di s s ect i on i s
Pa g e 8 2 1
ABC Ambe r CHM Conve rte r Tria l ve rsion, http://w w w .proce sste x t.com/a bcchm.html
performed.
(b) Obs ervat i on i s an opt i on for cl i ni cal N 0 pat i ent s .
o
o
b. Stage II disease i nvol ves s urgery or chemot herapy.
(1) Pat i ent s wi t h t umors l es s t han 3 cm i n di amet er may be t reat ed wi t h node di s s ect i on for curat i ve i nt ent .
(2) Pat i ent s wi t h ret roperi t oneal t umors more t han 5 cm i n di amet er s houl d recei ve chemot herapy.
(3) Adjuvant chemot herapy aft er compl et e res ect i on i s opt i onal . Pat i ent s who have proven ext ranodal ext ens i on, any node more t han 2 cm i n di amet er, or at l eas t s i x pos i t i ve nodes , have a rel aps e rat e aft er node di s s ect i on of great er t han 50%. W i t h adjuvant chemot herapy (e.g., t wo cycl es of et opos i de pl us ci s pl at i n), however, t he rel aps e rat e approaches zero.
2. Seminomatous tumors o
o
a. Stage I seminomas are t reat ed wi t h ret roperi t oneal and pel vi c radi at i on t herapy. In
Pa g e 8 2 2
ABC Ambe r CHM Conve rte r Tria l ve rsion, http://w w w .proce sste x t.com/a bcchm.html
recent years , t he pel vi c fi el d has not been us ed for t hi s purpos e at s ome cent ers , and obs ervat i on onl y, wi t h no radi at i on, has been feas i bl e i n s el ect ed pat i ent s . W i t h us e of t hi s approach, approxi mat el y 15% of pat i ent s rel aps e, but s al vage t herapy us i ng chemot herapy or radi at i on t herapy i s effect i ve i n al mos t al l cas es . In t he Uni t ed St at es , radi at i on t herapy remai ns t he s t andard approach. Aft er t reat ment , pat i ent s s houl d be reeval uat ed every mont h for t he fi rs t year and every 2 mont hs for a s econd year. o
o
b. Stage II seminomas. Al l pat i ent s wi t h s t age II s emi noma have ret roperi t oneal node met as t as es .
(1) Radiation therapy i s t he t reat ment of choi ce for pat i ent s wi t h N 1 and N 2 di s eas e. Becaus e of bone marrow t oxi ci t y from medi as t i nal radi at i on t herapy, prophyl act i c radi at i on t o t he medi as t i num i s no l onger i ndi cat ed.
(2) Pat i ent s wi t h N 3 ret roperi t oneal adenopat hy have a hi gh rel aps e rat e aft er radi at i on t herapy. Thus , t hes e pat i ent s s houl d recei ve chemotherapy i ns t ead. Ei t her t hree cycl es of bl eomyci n pl us et opos i de pl us ci s pl at i n or four cycl es of et opos i de pl us ci s pl at i n may be us ed.
o
o
c. Stage III or IV seminomas (and nonseminomatous germ cell tumors) are general l y
Pa g e 8 2 3
ABC Ambe r CHM Conve rte r Tria l ve rsion, http://w w w .proce sste x t.com/a bcchm.html
t reat ed wi t h ci s pl at i n i n combi nat i on wi t h et opos i de and bl eomyci n (BEP) at 3-week i nt erval s for t hree t o four cycl es . Four cycl es of et opos i de pl us ci s pl at i n have been s hown t o be equi val ent t o t hree cycl es of bl eomyci n pl us et opos i de pl us ci s pl at i n. Pat i ent s wi t h AJCC s t age IIIB or s t age IIIC di s eas e s houl d recei ve four cycl es of bl eomyci n pl us et opos i de pl us ci s pl at i n.
(1) Ot her chemot herapeut i c opt i ons for pat i ent s wi t h recurrent di s eas e i ncl ude i fos fami de, pacl i t axel , and vi nbl as t i ne.
(2) Surgi cal removal of res i dual mas s es pos t chemot herapy i s us ual l y recommended. Such mas s es cont ai n vi abl e cancer i n 10%–15% of cas es , fi bros i s or necros i s i n 40%–45%, and t erat omas i n 40%. Radi ol ogi c s t udi es cannot rel i abl y di s t i ngui s h bet ween t hes e pos s i bi l i t i es . Pat i ent s wi t h vi abl e res i dual t umors s houl d recei ve t wo addi t i onal cycl es of chemot herapy. Surgery al s o may be cons i dered as a “s al vage― t herapy for pat i ent s who are refract ory t o ci s pl at i n-bas ed chemot herapy, i f di s eas e i s confi ned t o one anat omi c s i t e and i s deemed res ect abl e.
(3) Hi gh-dos e chemot herapy wi t h carbopl at i n, et opos i de, and cycl ophos phami de may be combi ned wi t h an aut ol ogous BMT or peri pheral bl ood s t em cel l s upport i n s el ect ed pat i ent s wi t h chemot herapy-s ens i t i ve
Pa g e 8 2 4
ABC Ambe r CHM Conve rte r Tria l ve rsion, http://w w w .proce sste x t.com/a bcchm.html
rel aps es .
XII. Head and Neck Carcinomas A. Incidence Head and neck carci nomas account for 5% of t he cancers report ed each year i n t he Uni t ed St at es . Thes e t umors occur t hree t i mes more frequent l y i n men t han i n women. Pat i ent s are t ypi cal l y bet ween 50 and 60 years of age. The mos t frequent l y affect ed s i t es are t he oral cavi t y (40%), l arynx (25%), oropharynx and hypopharynx (15%), and s al i vary gl and (10%). P.167
B. Risk factors The fol l owi ng fact ors have been as s oci at ed wi t h an i ncreas ed i nci dence of head and neck carci nomas .
1. T obacco use. Ci garet t e s moki ng i s t he major caus e of head and neck cancer, and rel at i ve ri s k i ncreas es wi t h t he number of ci garet t es s moked per day. Chewi ng t obacco i s res pons i bl e for a recent i ncreas e i n t he i nci dence of oral cancer among young adul t s .
2. Alcohol consumption works i n combi nat i on wi t h ci garet t e s moki ng t o i ncreas e t he i nci dence of head and neck cancer 10- t o 40-fol d.
3. Nickel exposure i ncreas es t he ri s k of cancers of t he nas al cavi t y and paranas al s i nus .
4. Syphilis i s as s oci at ed wi t h an i ncreas ed i nci dence of cancer of t he t ongue.
Pa g e 8 2 5
ABC Ambe r CHM Conve rte r Tria l ve rsion, http://w w w .proce sste x t.com/a bcchm.html
5. Prolonged sun exposure i ncreas es t he i nci dence of l i p cancer.
6. The Epstein-Barr virus and cert ai n food dyes have been l i nked t o nas opharyngeal cancers i n As i ans .
C. Screening There are current l y no s creeni ng gui del i nes for earl y det ect i on i n as ympt omat i c peopl e. The U.S. Prevent at i ve Tas k Force does recommend couns el i ng for ces s at i on of t obacco us e and l i mi t at i on of al cohol i nt ake.
D. Pathology Mos t of t hes e t umors (95%) are s quamous cel l carci nomas , whi ch vary from wel l -di fferent i at ed vari et i es t o i nvas i ve, poorl y di fferent i at ed or undi fferent i at ed vari et i es . Nodal i nvol vement i s a fact or of t he pri mary t umor s i t e, s i ze, and degree of di fferent i at i on.
1. T umors of the salivary gland are mos t commonl y of t he mi xed t ype, al t hough adenoi d cys t i c carci nomas are not uncommon.
2. Nasopharyngeal cancers may be s quamous cel l cancers , l ymphoepi t hel i omas , or l ymphomas .
E. Clinical features Common s ympt oms i ncl ude dys phagi a, hoars enes s , head or neck pai n, ear pai n, and neck or head s wel l i ngs . Affect ed pat i ent s wi t h l arge l es i ons may pres ent wi t h ul cerat i ons or whi t e pat ches . Some pat i ent s pres ent wi t h adenopat hy i n t he abs ence of an obvi ous pri mary s ource.
F. Diagnosis
Pa g e 8 2 6
ABC Ambe r CHM Conve rte r Tria l ve rsion, http://w w w .proce sste x t.com/a bcchm.html
1. Detection o
o
a. T issue diagnosis rel i es on bi ops y of t he non-necrot i c port i on of t he t umor, t umor edge, and adjacent normal mucos a. Open bi ops y of t he neck nodes s houl d be avoi ded i f head and neck cancer of t he s quamous cel l t ype i s s us pect ed.
o
o
b. Radiologic studies may i ncl ude:
(1) Radi ographs of t he mandi bl e, s i nus , or nas opharynx
(2) CT and MRI s cans t o eval uat e t he nas opharynx, t hymus , oral cavi t y, and oral pharynx, and t o check nodal i nvol vement
(3) Bari um s wal l ow (i n pat i ent s wi t h evi dence of t umor encroachment on t he cervi cal es ophagus )
2. Pretreatment evaluation o
o
a. A t horough dental evaluation i s neces s ary before radi at i on t herapy t o hel p decreas e t he ri s k of os t eoradi onecros i s .
o
Pa g e 8 2 7
ABC Ambe r CHM Conve rte r Tria l ve rsion, http://w w w .proce sste x t.com/a bcchm.html o
b. If a mut i l at i ng s urgi cal procedure i s ant i ci pat ed, a prosthodontist s houl d be cons ul t ed t o pl an recons t ruct i ve cos met i c s urgery.
o
o
c. Many pat i ent s wi t h head and neck cancer are mal nouri s hed becaus e of t hei r i nabi l i t y t o s wal l ow and chew. Al l pat i ent s s houl d t herefore undergo a compl et e nutritional assessment, and every at t empt s houl d be made t o bri ng t he pat i ent i nt o pos i t i ve ni t rogen bal ance t o reduce t he compl i cat i ons of t reat ment .
G. Prognosis The mos t i mport ant det ermi nant of prognos i s i s s t age at di agnos i s . The 5-year s urvi val rat es for s t age I exceeds 80% but i s l es s t han 40% i n s t ages III and IV di s eas e. The pres ence of a pal pabl e l ymph node i n t he neck decreas es t he s urvi val rat e by 50% compared t o t he s ame T s t age wi t hout node i nvol vement . The l i fet i me ri s k of devel opi ng a new cancer i s 20%–40%.
H. Staging For ful l s t agi ng i nformat i on, pl eas e s ee t he mos t recent edi t i on of t he AJCC Canc er St agi ng Handbook. P.168
I. Therapy Smal l l es i ons can be t reat ed wi t h curat i ve s urgery and radi at i on t herapy. Pat i ent s wi t h advanced, unres ect abl e di s eas e can benefi t from radi at i on t herapy or combi ned-modal i t y approaches .
1. Surgery o
o
a. Pri mary t umors s houl d be exci s ed wi t h negat i ve
Pa g e 8 2 8
ABC Ambe r CHM Conve rte r Tria l ve rsion, http://w w w .proce sste x t.com/a bcchm.html
margi ns , whi ch may requi re s ki n fl aps . o
o
b. Pat i ent s wi t h neck l ymph node i nvol vement requi re radical neck dissection (i .e., di s s ect i on of t he s uperfi ci al and deep cervi cal fas ci a and cl os e l ymph nodes ; s t ernocl ei domas t oi d mus cl es ; i nt ernal , ext ernal , and jugul ar vei ns ; s pi nal acces s ory nerve; and s ubmandi bul ar gl ands ).
o
o
c. Pat i ent s wi t h cancers of t he oral cavi t y, oral pharynx, or s upragl ot t i c l arynx, and no evi dence of nodal i nvol vement are t reat ed wi t h a functional neck dissection for bet t er funct i onal and cos met i c res ul t s .
o
o
d. Surgery i s oft en as s oci at ed wi t h morbidity, i ncl udi ng cos met i c deformi t i es , s peech i mpedi ment s , as pi rat i on pneumoni a, s houl der droop, and pai n.
2. Radiation therapy. Mos t pat i ent s are t reat ed wi t h radi at i on over a 5- t o 6-week i nt erval . Si de effect s i ncl ude dry mout h, l os s of t as t e, mout h ul cers , os t eoradi onecros i s of t he mandi bl e (whi ch can be prevent ed by appropri at e dent al ext ract i ons , ant i bi ot i cs , and fl uori de admi ni s t rat i on), l aryngeal edema, and, rarel y, hypot hyroi di s m.
3. Chemotherapy for l ocal l y unres ect abl e or met as t at i c di s eas e may i ncl ude ci s pl at i n, 5-FU, met hot rexat e, and t axanes . o
Pa g e 8 2 9
ABC Ambe r CHM Conve rte r Tria l ve rsion, http://w w w .proce sste x t.com/a bcchm.html
o
a. New drugs s uch as i ri not ecan, pemet rexed, epi dermal growt h fact or recept or ant agoni s t s , and p53 gene t herapy (e.g., t axanes ) are bei ng t es t ed i n cl i ni cal t ri al s and s how promi s e.
o
o
b. Combination chemotherapy regimens have report ed hi gh res pons e rat es i n pat i ent s wi t h unres ect abl e di s eas e. Thes e pat i ent s exhi bi t res pons e rat es as hi gh as 90% aft er admi ni s t rat i on of ci s pl at i n and a cont i nuous i nfus i on of 5-FU.
XIII. Sarcomas A. Incidence Approxi mat el y 9500 new s arcomas are di agnos ed i n t he Uni t ed St at es each year, cons t i t ut i ng 1% of adul t mal i gnanci es .
B. Risk factors
1. Expos ure t o radiation
2. Expos ure t o chemicals (e.g., wood pres ervat i ves , herbi ci des , vi nyl chl ori de, or as bes t os )
3. Genetic abnormalities. For exampl e, pat i ent s wi t h von Reckl i nghaus en's di s eas e have a 10% l i fet i me ri s k of devel opment of a neurofi bros arcoma. Ot her caus es i ncl ude Gardner s yndrome, Li -Fraumeni s yndrome, and ret i nobl as t oma.
4. Pre-existing bone disease. Os t eos arcomas occur i n 0.2% of pat i ent s wi t h Paget 's di s eas e; however, mos t
Pa g e 8 3 0
ABC Ambe r CHM Conve rte r Tria l ve rsion, http://w w w .proce sste x t.com/a bcchm.html
os t eos arcomas occur i n pat i ent s younger t han 20 years of age.
C. Screening There are current l y no s creeni ng gui del i nes for earl y det ect i on i n as ympt omat i c peopl e.
D. Pathology The W orl d Heal t h Organi zat i on (W HO) cl as s i fi es mos t s oft t i s s ue s arcomas accordi ng t o t he pres umpt i ve t i s s ue of ori gi n; t hat i s , t he normal t i s s ues t he t umor mos t cl os el y res embl es . Exampl es i ncl ude t umors of adi pos e t i s s ue, s kel et al mus cl e, s moot h mus cl e, bl ood and l ymph ves s el s , crani al and peri pheral nerves ; peri vas cul ar t umors ; fi brohi s t i ocyt i c t umors ; fi brobl as t i c and myofi brobl as t i c t umors ; and ext ras kel et al cart i l agi nous and os s eous t umors .
E. Clinical features
1. Soft tissue sarcomas oft en mani fes t as a mass, swelling, or pain i n t he t runk or ext remi t i es .
2. Pat i ent s wi t h retroperitoneal tumors general l y experi ence weight loss or deep-seated pain.
3. Bleeding i s t he mos t common pres ent i ng feat ure of gynecologic and gastrointestinal sarcomas.
4. Mos t pat i ent s wi t h os t eos arcomas pres ent wi t h ei t her pain, a mass, or both.
P.169
F. Diagnosis Pa g e 8 3 1
ABC Ambe r CHM Conve rte r Tria l ve rsion, http://w w w .proce sste x t.com/a bcchm.html
1. Biopsy o
o
a. A rapi dl y growi ng mas s or one t hat exceeds 5 cm i n di amet er s houl d be regarded wi t h s us pi ci on, es peci al l y i f i t i s fi rm, deep, or fi xed.
o
o
b. Thes e l es i ons requi re a generous i nci s i onal bi ops y or an exci s i onal bi ops y; needl e bi ops y i s us ual l y i nadequat e.
o
o
c. The bi ops y s i t e mus t be s el ect ed careful l y t o al l ow for t he pos s i bi l i t y of l i mb-s pari ng s urgery.
2. Imaging procedures i ncl ude radi ographs , MRI, CT, and bone s cans . o
o
a. For s oft t i s s ue s arcomas , MRI i s s uperi or t o CT, provi di ng bet t er defi ni t i on of s oft t i s s ue pl anes and t umor margi ns .
o
o
b. MRI or CT and bone s canni ng are preferred for eval uat i ng bony i nvol vement .
o
o
c. MRI or CT s cans of t he l i ver i dent i fy hepat i c met as t as es i n t hos e pat i ent s wi t h vi s ceral s arcomas and i n t hos e wi t h ext remi t y s arcomas accompani ed by evi dence of abnormal hepat i c funct i on.
Pa g e 8 3 2
ABC Ambe r CHM Conve rte r Tria l ve rsion, http://w w w .proce sste x t.com/a bcchm.html
G. Prognosis Hi s t ol ogi c grade and t umor s i ze are t he mos t i mport ant prognos t i c fact ors .
1. Histologic grade i s det ermi ned by t he mi t ot i c rat e, nucl ear grade, ext ent of necros i s , nucl ear morphol ogy, and cel l ul ari t y.
2. T umor size i s an i ndependent prognos t i c fact or. Smal l (<5 cm), compl et el y exci s ed, l ow-grade l es i ons rarel y recur l ocal l y and have a l ow met as t at i c rat e.
3. Other factors. Mul t i vari ant anal ys i s i dent i fi ed t he fol l owi ng fact ors as bei ng as s oci at ed wi t h an i ncreas ed ri s k of l ocal recurrence: age ol der t han 53 years ; pres ent at i on wi t h recurrent di s eas e; hi gh-grade, pai nful mas s ; l i mb-s pari ng s urgery; and i nadequat e s urgi cal margi ns .
H. Therapy Surgery i s t he mai ns t ay of t herapy, s uppl ement ed wi t h radi at i on t herapy or chemot herapy.
1. Surgery o
o
a. Definitive resection may cons i s t of ext ens i ve s urgi cal exci s i on, i ncl udi ng a 2- t o 4-cm pat hol ogi cal l y document ed margi n of normal t i s s ue, or cons ervat i ve res ect i on wi t h document ed t umor-free margi ns , s uppl ement ed wi t h radi at i on t herapy. Re-exci s i on s houl d be cons i dered aft er bi ops y i f mi cros copi cal l y i nvol ved margi ns are
Pa g e 8 3 3
ABC Ambe r CHM Conve rte r Tria l ve rsion, http://w w w .proce sste x t.com/a bcchm.html
demons t rat ed. o
o
b. Limb-sparing surgery can be cons i dered i n t he majori t y of pat i ent s wi t h ext remi t y s arcomas .
(1) Thi s procedure i s as s oci at ed wi t h a 90% l ocal cont rol rat e and an overal l di s eas e-free s urvi val rat e of 60%, a s ucces s rat e t hat i s s i mi l ar t o t hat found wi t h amput at i on or radi cal res ect i on procedures .
(2) Preoperat i ve i nt ra-art eri al chemot herapy and radi at i on t herapy may faci l i t at e l i mb-s pari ng s urgery i n borderl i ne res ect abl e t umors .
o
o
c. Surgeons s houl d avoi d s eedi ng t he ent i re s urgi cal fi el d duri ng t he procedure. If s uch s eedi ng occurs , wi de exci s i on s houl d be fol l owed by 6600-cGy radi at i on t herapy.
o
o
d. Surgery offers t he chance of a cure for pat i ent s wi t h few (i .e., no more t han 10–15) pul monary met as t as es .
2. Radiation therapy may be us ed when t he t umor margi ns are l es s t han 2–4 cm or i f t umor s eedi ng has occurred. St udi es have s hown t hat a combi nat i on of l i mb-s pari ng s urgery and radi at i on t herapy i s as effect i ve as amput at i on.
Pa g e 8 3 4
ABC Ambe r CHM Conve rte r Tria l ve rsion, http://w w w .proce sste x t.com/a bcchm.html
3. Chemotherapy o
o
a. Adjuvant chemotherapy has no cl ear benefi t except i n young pat i ent s wi t h os t eos arcoma and adul t s wi t h ext remi t y s arcomas . A few s t udi es have report ed an i ncreas e i n di s eas e-free s urvi val rat es and overal l s urvi val rat es wi t h t he us e of doxorubi ci n-bas ed combi nat i on regi mens i n ext remi t y s arcomas . Doxorubi ci n al one appears t o be i neffect i ve.
o
o
b. Advanced sarcomas. Previ ous l y t reat ed pat i ent s may res pond t o doxorubi ci n (15%–35%), dacarbaz i ne (17%), or i fos fami de (20%–40%). Some s t udi es s ugges t t hat combi nat i on chemot herapy, part i cul arl y wi t h doxorubi ci n and i fos fami de, may yi el d i mproved res ul t s . The combi nat i on of gemci t abi ne pl us docet axel (Taxot ere) i s hi ghl y act i ve i n pat i ent s wi t h l ei omyos arcomas .
P.170
XIV. Multiple Myeloma (s ee Chapt er 6 Part I: X P)
A. Incidence Myel oma i s a neopl as m of t he pl as ma cel l s t hat are deri ved from B l ymphocyt es . It i s an uncommon neopl as m, account i ng for l es s t han 1%–2% of t he adul t cancers i n t he Uni t ed St at es . The i nci dence of myel oma i ncreas es wi t h advanci ng age and i s t wi ce as common i n bl acks . It i s es t i mat ed t hat 19,900 new cas es of mul t i pl e myel oma wi l l be di agnos ed i n 2007.
B. Etiology Pa g e 8 3 5
ABC Ambe r CHM Conve rte r Tria l ve rsion, http://w w w .proce sste x t.com/a bcchm.html
1. Al t hough no s peci fi c underl yi ng caus es have been proven, chroni c s t i mul at i on t o t he i mmune s ys t em may pl ay a rol e i n t he pat hogenes i s of myel oma. Int eres t i ngl y, IL-6 can promot e t he growt h of myel oma cel l s i n vi t ro.
2. Genet i c fact ors , as wel l as expos ure t o pet rol eum, as bes t os , l axat i ves , and radi at i on, al s o may be i mport ant fact ors .
3. Beni gn monocl onal gammopat hi es are s us pect ed precurs ors of mul t i pl e myel oma.
C. Screening There are current l y no s creeni ng gui del i nes for earl y det ect i on i n as ympt omat i c peopl e.
D. Pathology Mul t i pl e myel oma i s a t ype of mat ure B-cel l l ymphoi d charact eri zed by accumul at i on of mal i gnant pl as ma cel l s i n t he bone marrow compart ment .
E. Clinical features W i t h t he advent of aut omat ed l aborat ory bl ood t es t i ng, myel oma may be det ect ed earl i er.
1. The major feature of myel oma i s t he demons t rat i on of an abnormal monocl onal prot ei n (M protein) i n t he blood, urine, or both. Thi s M prot ei n us ual l y cons i s t s of ei t her one or a combi nat i on of heavy chai ns (IgG and IgA) and l i ght chai ns (κ and λ). o
Pa g e 8 3 6
ABC Ambe r CHM Conve rte r Tria l ve rsion, http://w w w .proce sste x t.com/a bcchm.html
o
a. M prot ei ns cons i s t i ng of t he whol e i mmunogl obul i n mol ecul es IgG and IgA account for 50% and 25% of t he cas es , res pect i vel y.
o
o
b. M prot ei ns cons i s t i ng of onl y l i ght chai ns account for 25% of cas es . In t hes e cas es , t he M prot ei n i s found onl y i n t he uri ne.
2. Complications i ncl ude: o
o
a. Infi l t rat i on of t he marrow by l arge numbers of pl as ma cel l s , whi ch are us ual l y abnormal
o
o
b. Weakness, fatigue, infection, and bleeding due t o marrow fai l ure
o
o
c. Osteolytic lesions res ul t i ng from myel oma-i nduced bone res orpt i on wi t h s ubs equent pai n and fract ure
o
o
d. Renal abnormalities due t o myel oma i nfi l t rat i on of t he ki dney, hypercal cemi a, t oxi c effect s of l i ght chai ns on t ubul es , amyl oi d depos i t i on, and hyperuri cemi a
o
o
e. Recurrent infections due t o acqui red hypogammagl obul i nemi a and l eukopeni a
o
Pa g e 8 3 7
ABC Ambe r CHM Conve rte r Tria l ve rsion, http://w w w .proce sste x t.com/a bcchm.html o
f. Hypercalcemia due t o myel oma-s t i mul at ed os t eocl as t act i vi t y
o
o
g. Hyperviscosity due t o a hi gh concent rat i on of t he M prot ei n, whi ch t ends t o aggregat e
F. Diagnosis The fol l owi ng feat ures mus t be demons t rat ed.
1. An abnormal M protein level (i .e., >3.5 g/dL) i n t he s erum, uri ne, or bot h
2. Marrow infiltration by pl as ma cel l s (i .e., >30% pl as ma cel l s on bone marrow bi ops y)
3. Additional supportive findings i ncl ude anemi a, os t eol yt i c s kel et al l es i ons , renal abnormal i t i es , and hypercal cemi a. The once-common s ympt oms of anemia and bone pain are s een l es s frequent l y.
4. Plasmacytoma on t i s s ue bi ops y. Becaus e marrow pl as macyt os i s occurs i n many chroni c i nfect i ons or i nfl ammat ory proces s es wi t hout an M component , t he major di ffi cul t y i n di fferent i al di agnos i s i s di s t i ngui s hi ng myel oma from monocl onal gammopat hy of unknown s i gni fi cance (MGUS).
G. Prognosis Thi s cl earl y depends on t he ext ent of myel oma at pres ent at i on. Pat i ent s wi t h s mol deri ng or s t age I myel oma may go many years wi t hout a need for t herapy, whereas pat i ent s wi t h s t age III myel oma and renal and ort hopedi c compl i cat i ons do poorl y. Mean s urvi val for pat i ent s requi ri ng t herapy i s 2–3 years .
Pa g e 8 3 8
ABC Ambe r CHM Conve rte r Tria l ve rsion, http://w w w .proce sste x t.com/a bcchm.html
P.171
H. Staging The l i near rel at i ons hi p bet ween t he eas i l y meas ured M prot ei n and t he cel l ul ar t umor burden forms t he bas i s for s t agi ng myel oma.
1. Stage I (l ow t umor burden). Pat i ent s have normal cal ci um and bone fi l ms as wel l as t he fol l owi ng bl ood dat a: o
o
a. Hemogl obi n l evel >10 g/dL
o
o
b. IgG l evel <5 g/dL
o
o
c. IgA l evel <3 g/dL
2. Stage II (i nt ermedi at e t umor burden). Pat i ent s have t he fol l owi ng bl ood dat a: o
o
a. Hemogl obi n l evel β 8.5–10 g/dL
o
o
b. IgG l evel = 5–7 g/dL
o
o
c. IgA l evel = 3–5 g/dL
Pa g e 8 3 9
ABC Ambe r CHM Conve rte r Tria l ve rsion, http://w w w .proce sste x t.com/a bcchm.html
3. Stage III (hi gh t umor burden). Pat i ent s have hypercal cemi a and os t eol yt i c l es i ons as wel l as t he fol l owi ng bl ood dat a: o
o
a. Hemogl obi n l evel <8.5 g/dL
o
o
b. IgG l evel >7 g/dL
o
o
c. IgA l evel >5 g/dL
I. Therapy
1. Pat i ent s i n whom a di fferent i al di agnos i s of MGUS cannot be excl uded and pat i ent s wi t h very l ow-grade smoldering myeloma s houl d be exami ned onl y at 3- t o 6-mont h i nt erval s for evi dence of di s eas e progres s i on.
2. Therapy s houl d be gi ven t o pat i ent s wi t h bone pai n, hypercal cemi a, renal fai l ure, bone marrow s uppres s i on, and s pi nal cord compres s i on. Res pons es are obt ai ned i n approxi mat el y t wo t hi rds of pat i ent s . o
o
a. Chemotherapy
(1) Agents
(a) Cl as s i cal l y, alkylating agents (e.g., melphalan) and prednisone are us ed, al t hough s ome phys i ci ans advocat e more
Pa g e 8 4 0
ABC Ambe r CHM Conve rte r Tria l ve rsion, http://w w w .proce sste x t.com/a bcchm.html
aggres s i ve regi mens t hat al s o cont ai n doxorubicin, vincristine, and nitrosoureas. The s uperi ori t y of mul t i drug regi mens over mel phal an and predni s one has not been demons t rat ed.
(i) The vincristine–doxorubicin (Adriamycin)–dexamethasone (VAD) regimen, i n whi ch vi ncri s t i ne and doxorubi ci n are admi ni s t ered as a cont i nuous i nfus i on wi t h pul s e dexamet has one, can be us ed as a pri mary or s al vage t herapy.
(ii) Thal i domi de, wi t h or wi t hout dexamet has one, has been s hown t o be effect i ve i n t he t reat ment of mul t i pl e myel oma. For pat i ent s who are not candi dat es for t herapy wi t h VAD, t hal i domi de and dexamet has one i s a reas onabl e al t ernat i ve. Recent l y t he FDA approved t he us e of a prot eas ome i nhi bi t or, bort ezomi b (Vel cade) as t herapy for pat i ent s wi t h mul t i pl e myel oma who had fai l ed at l eas t t wo pri or t herapi es .
(b) IFN has been us ed for mai nt enance t herapy.
(2) T oxicity. Therapy damages marrow and l eads t o l eukemi as and exces s i ve s econdary
Pa g e 8 4 1
ABC Ambe r CHM Conve rte r Tria l ve rsion, http://w w w .proce sste x t.com/a bcchm.html
marrow di s eas es i n l ong-t erm s urvi vors .
(3) Evaluation of effectiveness. W hen t herapy i s effect i ve, t he M prot ei n l evel s decl i ne. Therapy can be s t opped i f t he M prot ei n l evel s become normal or s t abi l i ze at 75% bel ow i ni t i al l evel s . Therapy can be res umed when di s eas e progres s i on occurs , al t hough res ul t s wi t h ret reat ment are not as good as t hos e wi t h i ni t i al t herapy.
o
o
b. Autologous transplantation has been performed i n many pat i ent s , mos t of whom have been rei nfus ed wi t h unpurged marrow. The compl et e remi s s i on rat e i s 20%–30%; 50% of pat i ent s who achi eve a compl et e remi s s i on are di s eas e-free at 4 years .
o
o
c. Supportive therapy i s ext remel y i mport ant i n t he management of myel oma and i ncl udes :
(1) Radiation therapy for l ocal bone di s eas e
(2) Hydration and proper management of hypercalcemia
(3) Orthopedic support and care
(4) Plasmapheresis for hypervi s cos i t y s yndrome
Pa g e 8 4 2
ABC Ambe r CHM Conve rte r Tria l ve rsion, http://w w w .proce sste x t.com/a bcchm.html
XV. Hodgkin's Disease A. Incidence In t he Uni t ed St at es , 7350 new cas es of Hodgki n's di s eas e are report ed each year. Thi s neopl as m has a charact eri s t i c bimodal age distribution.
1. A young adult peak occurs bet ween t he ages of 10 and 25 years and i s charact eri zed by equal i nci dence i n men and women, a preponderance of nodul ar s cl eros i s pat hol ogy, and a more beni gn cl i ni cal cours e.
2. A second adult peak occurs aft er age 50 years and i s charact eri zed by hi gh i nci dence among men, a preponderance of mi xed cel l ul ari t y, and a more aggres s i ve cl i ni cal cours e.
B. Etiology
1. Cl us t eri ng of cas es i n t i me and pl ace occurs i n Hodgki n's di s eas e, s ugges t i ng t hat viruses or environmental factors may play a role. However, t hi s cl us t eri ng i s s poradi c and has not been s ubs t ant i at ed by fi rm evi dence. St udi es have s ugges t ed an as s oci at i on wi t h Epstein-Barr virus i nfect i on. P.172
2. St at i s t i cal evi dence l i nks earl y ons et Hodgki n's di s eas e wi t h higher socioeconomic class, and an i ncreas ed i nci dence of Hodgki n's di s eas e among fami l y
Pa g e 8 4 3
ABC Ambe r CHM Conve rte r Tria l ve rsion, http://w w w .proce sste x t.com/a bcchm.html
members s ugges t s a genetic predisposition.
C. Screening There are current l y no s creeni ng gui del i nes for earl y det ect i on i n as ympt omat i c peopl e.
D. Pathology There are four major hi s t ol ogi c vari ant s of Hodgki n's di s eas e: nodul ar s cl eros i s , mi xed cel l ul ari t y, l ymphocyt e depl et ed, and l ymphocyt e domi nant . The mos t common hi s t i ol ogi c t ype i s nodul ar s cl eros i s . Of i nt eres t , however, i s t he fact t hat t he preci s e nat ure of t he t rul y mal i gnant cel l (t he bi nucl eat e gi ant cel l cal l ed t he Reed-Sternberg cell) remai ns a poi nt of cont rovers y, des pi t e new t echni ques t hat i ndi cat e t hat t hi s cel l i s more l i kel y deri ved from t he mononucl ear phagocyt e s ys t em t han from t rans formed l ymphocyt es .
E. Clinical features Hodgki n's di s eas e pat i ent s , es peci al l y young pat i ent s , us ual l y have asymptomatic swelling of a lymph node. The B s ubcl as s i fi cat i on i mpl i es , however, t hat Hodgki n's di s eas e may pres ent wi t h s uch systemic symptoms as fever, weight loss, and drenching sweats.
F. Diagnosis W orkup i ncl udes :
a. Thorough hi s t ory and phys i cal exami nat i on
b. Ches t radi ographs ; CT s cans of t he ches t , abdomen, and pel vi s .
c. Percut aneous bi l at eral bone marrow bi ops i es
Pa g e 8 4 4
ABC Ambe r CHM Conve rte r Tria l ve rsion, http://w w w .proce sste x t.com/a bcchm.html
d. PET s can
G. Prognosis Out come vari es mai nl y wi t h t he s t age of di s eas e and, t o a l es s er ext ent , wi t h hi s t ol ogy. Overal l , t here i s a 55%–60% 5-year s urvi val rat e wi t h al l cas es of Hodgki n's di s eas e.
1. Pat i ent s wi t h s t age I or s t age II di s eas e have a 5-year di s eas e-free s urvi val rat e exceedi ng 80%.
2. Pat i ent s wi t h s t age IIIA di s eas e have roughl y a 67% rat e of 5-year di s eas e-free s urvi val .
3. Pat i ent s wi t h s t age IIIA 2 , s t age IIIB, or s t age IV di s eas e, i f t reat ed wi t h chemot herapy, obt ai n remi s s i ons i n 80%–95% of cas es , wi t h more t han 50% of t hes e pat i ent s achi evi ng prol onged (i .e., >5 years ) di s eas e-free s urvi val .
H. Staging Hodgki n's di s eas e t ends t o s pread i n an orderl y fas hi on from node group t o node group. Thi s cont i guous nat ure i s i n marked cont ras t t o non-Hodgki n's l ymphomas , whi ch are mul t i cent ri c earl y i n t hei r devel opment . The modi fi ed Ann Arbor classification i s us ed for staging Hodgkin' s disease (for ful l s t agi ng i nformat i on, pl eas e s ee t he mos t recent edi t i on of t he AJCC Cancer St agi ng Handbook). W orkup i s bas ed on t he pri nci pl e t hat early-stage Hodgki n's di s eas e can be treated locally but late-stage Hodgki n's di s eas e requires systemic therapy. Therefore, pat i ent s are aggres s i vel y s t aged t o i ncl ude or excl ude s t age IIIA 2 , s t age IIIB, and s t age IV di s eas e. (Thi s s t agi ng i s far more ext ens i ve t han s t agi ng for non-Hodgki n's l ymphomas .)
I. Therapy Treat ment for Hodgki n's di s eas e cont i nues t o evol ve. However,
Pa g e 8 4 5
ABC Ambe r CHM Conve rte r Tria l ve rsion, http://w w w .proce sste x t.com/a bcchm.html
cert ai n pri nci pl es have been es t abl i s hed and s houl d be fi rml y adhered t o, becaus e even advanced disease is curable. Indi vi dual vari at i ons i n t reat ment s houl d be avoi ded and, i f us ed, s houl d be l i mi t ed t o wel l -cont rol l ed cl i ni cal t ri al s .
1. Stage I or II di s eas e i s t reat ed wi t h extended-field radiation therapy.
2. Stage IIIA 1 disease (pathologically staged) is treated with total nodal radiation therapy. Pat i ent s who rel aps e aft er radi at i on t herapy can be s ucces s ful l y gi ven chemotherapy. More t han 50% of s uch pat i ent s have l ong-t erm di s eas e-free s urvi val when s uch t echni ques are us ed. Cl i ni cal s t age IIIA 1 di s eas e (e.g., no s t agi ng l aparot omy) i s us ual l y t reat ed wi t h chemot herapy.
3. Stage IIIA 2 , IIIB, or IV di s eas e requi res systemic chemotherapy. Bone marrow t rans pl ant at i on has been us ed i n cas es of rel aps e. o
o
a. Regimens
(1) Many regi mens are avai l abl e. The cl as s i c mechlorethamine–vincristine (Oncovin)–procarbazine–prednisone (MOPP) regimen, whi ch i s gi ven for at l eas t s i x cycl es pl us t wo addi t i onal cycl es aft er compl et e remi s s i on i s document ed, was t he fi rs t curat i ve regi men. However, The doxorubicin (Adriamycin) –bleomycin–vinblastine–dacarbazine
Pa g e 8 4 6
ABC Ambe r CHM Conve rte r Tria l ve rsion, http://w w w .proce sste x t.com/a bcchm.html
(DT IC) [ABVD] regimen i s current l y t he fi rs t l i ne combi nat i on chemot herapy of choi ce. P.173
Thi s regi men has been s hown t o be more effect i ve and l es s t oxi c t han MOPP and as effect i ve as and l es s t oxi c t han t he MOPP/ABVD combi nat i ons .
(2) Ot her regi mens s uch as St anford V, whi ch us es a combi nat i on of mul t i pl e chemot herapy agent s and radi at i on t reat ment t o bul ky (>5 cm) l ymph nodes , are al s o i n us e. A phas e III randomi zed cl i ni cal t ri al of ABVD vers us St anford V i s underway
o
o
b. Principles
(1) Combi ned chemot herapy and radi at i on t herapy s houl d not be us ed rout i nel y unt i l furt her cl i ni cal t ri al s prove t he effect i venes s of s uch regi mens .
(2) Unl es s chemot herapeut i c agent s are gi ven i n ful l dos es and accordi ng t o pres cri bed s chedul es , t hei r effect i venes s can be s i gni fi cant l y compromi s ed.
o
o
c. Side effects. Bot h combi nat i on chemot herapy and t ot al nodal i rradi at i on are t oxi c and have many predi ct abl e s i de effect s , i ncl udi ng s evere naus ea and vomi t i ng, mucos i t i s , di arrhea, bone marrow
Pa g e 8 4 7
ABC Ambe r CHM Conve rte r Tria l ve rsion, http://w w w .proce sste x t.com/a bcchm.html
s uppres s i on, cardi ot oxi ci t y, hypot hyroi di s m, s t eri l i t y (i n s ome cas es ), and devel opment of s econdary marrow probl ems , i ncl udi ng acut e l eukemi a.
XVI. Non-Hodgkin's Lymphoma A. Incidence More t han 58,000 pat i ent s are di agnos ed wi t h non-Hodgki n's l ymphoma i n t he Uni t ed St at es each year. An i ncreas ed i nci dence i n ol der peopl e i s at t ri but abl e t o a ri s i ng i nci dence i n di ffus e l arge cel l l ymphoma. Non-Hodgki n's l ymphomas (es peci al l y CNS l ymphomas ) are more common i n pat i ent s wi t h acqui red i mmunodefi ci ency s yndromes and i n pat i ent s recei vi ng i mmunos uppres s i ve drugs s uch as t hos e wi t h ki dney and heart t rans pl ant s .
B. Etiology
1. Cytogenetic abnormalities s uch as chromos ome t rans l ocat i ons are commonl y obs erved i n l ymphoma cel l s .
2. Viral infection o
o
a. The Epstein-Barr virus has been l i nked t o Burkitt' s lymphoma, a di s eas e us ual l y found i n Afri ca.
o
o
b. An aggres s i ve T-cel l l eukemi a or l ymphoma occurs i n Japan and t he Cari bbean, and i s as s oci at ed wi t h human T l ymphot ropi c vi rus t ype I (HT LV-I) infection.
Pa g e 8 4 8
ABC Ambe r CHM Conve rte r Tria l ve rsion, http://w w w .proce sste x t.com/a bcchm.html
C. Screening There are current l y no s creeni ng gui del i nes for earl y det ect i on i n as ympt omat i c i ndi vi dual s .
D. Pathology A number of hi s t ol ogi c cl as s i fi cat i on s chemes are i n us e; t he one mos t wi del y us ed i n t he cl i ni cal l i t erat ure i s t he W HO cl as s i fi cat i on s ys t em. The W HO cl as s i fi cat i on for l ymphoi d neopl as ms i s found i n t he AJCC Cancer St agi ng Handbook.
1. The low-grade lymphomas are predomi nant l y B-cel l t umors . The intermediate-grade lymphomas i ncl ude bot h B- and s ome T-cel l l ymphomas , whereas immunoblastic lymphomas are predomi nant l y B-cel l t umors and lymphoblastic lymphomas are T-cel l t ypes . Mos t B-cel l t umors are monocl onal and produce ei t her a κ or a λ l i ght chai n i mmunogl obul i n.
2. Follicular small-cleaved cell lymphomas are t he mos t common hi s t ol ogi c t ype, account i ng for approxi mat el y 40% of cas es . Thes e pat i ent s have predomi nant l y s t age III or s t age IV di s eas e wi t h a hi gh i nci dence of bone marrow i nvol vement and an i ndol ent cours e, evol vi ng over many years .
3. Follicular mixed small-cleaved and large-cell lymphomas repres ent 20%–40% of pat i ent s . Large cel l s may account for as much as 25% of t he cel l popul at i on. The l ymph node archi t ect ure s hows di s t i nct nodul es , whi ch are us ual l y pres ent t hroughout t he node. Marrow i nvol vement i s common. Thi s s ubt ype i s i ndol ent but more aggres s i ve t han fol l i cul ar s mal l -cl eaved cel l l ymphoma.
Pa g e 8 4 9
ABC Ambe r CHM Conve rte r Tria l ve rsion, http://w w w .proce sste x t.com/a bcchm.html
4. Diffuse large-cell lymphomas are charact eri zed by l arge mal i gnant l ymphocyt es wi t h i ncreas ed nucl ear di amet ers . Thes e cel l s have cyt ol ogi c feat ures of l arge cl eaved or noncl eaved fol l i cul ar cel l s , wi t h nucl eol i , abundant cyt opl as m, and frequent mi t os es .
5. Immunoblastic lymphomas and ot her hi gh-grade non-Hodgki n's l ymphomas i ncl ude plasmacytoid, clear cell, and polymorphic cat egori es . Thes e s ubt ypes are rapi dl y fat al unl es s effect i ve t reat ment i s admi ni s t ered prompt l y.
P.174
E. Clinical features
1. Mos t pat i ent s are asymptomatic. Twent y percent of pat i ent s have fever, night sweats, or weight loss.
2. Pat i ent s wi t h i ndol ent l ymphomas may have waxing and waning adenopathy for s everal mont hs before di agnos i s , al t hough persistent nodal enlargement i s more common. Ext ranodal di s eas e mos t oft en i nvol ves t he s t omach, l ung, and bone, res ul t i ng i n s ympt oms charact eri s t i c of t he affect ed organ.
F. Diagnosis
1. A compl et e history and physical examination, wi t h part i cul ar emphas i s on al l l ymph node–beari ng areas ,
Pa g e 8 5 0
ABC Ambe r CHM Conve rte r Tria l ve rsion, http://w w w .proce sste x t.com/a bcchm.html
i ncl udi ng W al deyer's ri ng, as wel l as l i ver and s pl een s i z e.
2. Hematologic studies, i ncl udi ng a CBC, di fferent i al , pl at el et count , l i ver and ki dney funct i on s t udi es , LDH, and uri c aci d l evel s t udi es . Serum prot ei n el ect rophores i s rul es out hypogammagl obul i nemi a or a monocl onal gammopat hy.
3. Bone marrow biopsies and aspirates. An adequat e s urgi cal l ymph node biopsy exami ned by an experi enced pat hol ogi s t .
4. Radiologic studies, i ncl udi ng ches t radi ography; CT of t he ches t , abdomen, and pel vi s .
G. Prognosis Many pat i ent s who achi eve a compl et e res pons e, part i cul arl y t hos e wi t h di ffus e l arge cel l l ymphoma, remai n di s eas e-free for an ext ended peri od and wi l l be cured. Aggres s i ve combi nat i on chemot herapy regi mens cont ai ni ng doxorubi ci n have hi gh compl et e res pons e rat es rangi ng from 40% t o 80%. Prognos t i c fact ors (Int ernat i onal Prognos t i c Indi cat ors ) for l arge cel l l ymphoma i ncl ude i ncreas i ng pat i ent age, s t age, LDH l evel , performance s t at us , and i n ol der pat i ent s , t he pres ence or abs ence of ext ranodal di s eas e.
H. Staging The Ann Arbor s t agi ng s ys t em us ed t o cl as s i fy Hodgki n's di s eas e i s al s o us ed t o s t age non-Hodgki n's l ymphomas (s ee t he AJCC Cancer St agi ng Handbook).
I. Therapy Treat ment us ual l y requi res a mul t i di s ci pl i nary approach. Radi at i on t herapy and chemot herapy aft er s urgi cal bi ops y are t he mos t
Pa g e 8 5 1
ABC Ambe r CHM Conve rte r Tria l ve rsion, http://w w w .proce sste x t.com/a bcchm.html
common t reat ment modal i t i es .
1. Radiation therapy. Non-Hodgki n's l ymphomas are hi ghl y radi os ens i t i ve. o
o
a. In l ocal i zed di s eas e, radi at i on s houl d be t arget ed t o t he affect ed s i t e (4,000 cGy t o t he i nvol ved fi el d). The nodal s i t e and drai ni ng l ymphat i cs s houl d be i ncl uded i n t he radi at i on fi el d.
o
o
b. Radi at i on t herapy i s us ed palliatively i n di s s emi nat ed di s eas e or t o “cons ol i dat e― a compl et e res pons e t o chemot herapy i n areas of bul ky di s eas e.
o
o
c. Electron beam therapy has been us ed i n t he management of cut aneous l ymphomas s uch as t he earl y s t ages of mycos i s fungoi des .
o
o
d. Stage I indolent lymphomas. Long-t erm pat i ent fol l ow-up aft er i nvol ved or ext ended fi el d radi at i on t herapy for l ocal i z ed s t age I and s t age II l ow-grade l ymphoma reveal s a 10-year rel aps e-free s urvi val rat e of great er t han 50%, es peci al l y i n younger pat i ent s .
2. Chemotherapy o
o
a. Low-grade indolent lymphomas may not
Pa g e 8 5 2
ABC Ambe r CHM Conve rte r Tria l ve rsion, http://w w w .proce sste x t.com/a bcchm.html
requi re t reat ment for many years . W hen t herapy i s i ndi cat ed, chlorambucil or cyclophosphamide, wi t h or wi t hout predni s one, i s t he agent of choi ce. Ini t i al res ul t s wi t h hi gh-dos e chemot herapy fol l owed by aut ol ogous marrow t rans pl ant at i on are favorabl e, but s ucces s needs t o be meas ured by l ong-t erm fol l ow-up becaus e t hes e pat i ent s general l y have l ong-t erm s urvi val rat es , even wi t h l es s t oxi c t reat ment . The ant i -CD20 ant i body, ri t uxi mab, i s us eful i n rel aps ed pat i ent s , as i s t he nucl eos i de anal og, fl udarabi ne. o
o
b. Stage I or II intermediate and high-grade lymphomas oft en res pond t o combi nat i on chemot herapy and ri t uxi mab, wi t h or wi t hout radi at i on t herapy. Cure rat es approach 80%–90%.
o
o
c. Aggressive intermediate or high-grade lymphomas (e.g., l ymphobl as t i c or Burki t t 's l ymphoma) requi re i mmedi at e combi nat i on chemot herapy wi t h prot ocol s s i mi l ar t o t hos e us ed i n t he t reat ment of acut e l ymphobl as t i c l eukemi a (ALL). Prophyl act i c i nt rat hecal chemot herapy may al s o be gi ven for hi gh-grade l ymphomas . Salvage combination chemotherapy produces s econd compl et e or part i al remi s s i ons but i s rarel y curat i ve unl es s t he pat i ent undergoes BMT. P.175
(1) The most common regimen involves
Pa g e 8 5 3
ABC Ambe r CHM Conve rte r Tria l ve rsion, http://w w w .proce sste x t.com/a bcchm.html
cyclophospha mide–doxorubicin–vincristine (Oncovin)–prednisone (CHOP) plus rituximab. Aggres s i ve chemot herapy wi t h aut ol ogous s t em cel l s upport may i mprove s urvi val i n newl y di agnos ed hi gh-ri s k pat i ent s .
(2) Becaus e of encouragi ng res ul t s wi t h combi nat i on chemot herapy i n pat i ent s wi t h s t age III and s t age IV di s eas e fol l owed by radi at i on t herapy, randomi zed cl i ni cal t ri al s are underway t o compare chemot herapy al one and chemot herapy fol l owed by radi at i on t herapy.
(3) Colony-stimulating factors has t en granul ocyt e recovery and may permi t hi gher dos es and bet t er cure rat es .
(4) Radiolabeled monoclonal antibodies i bri t umomab t i uxet an (Zeval i n) and t os i t umomab (Bexxar) are now approved for s al vage t herapy i n pat i ent s who have fai l ed ri t uxi mab and convent i onal chemot herapy.
XVII. Paraneoplastic Syndromes A. Endocrine syndromes Endocrine syndromes res ul t i ng from ect opi c pol ypept i de hormone product i on are t he mos t common and bes t unders t ood of t he paraneopl as t i c proces s es . Many t umors produce more t han one bi ol ogi cal l y act i ve hormone, l eadi ng t o mul t i pl e endocri ne paraneopl as t i c s yndromes .
Pa g e 8 5 4
ABC Ambe r CHM Conve rte r Tria l ve rsion, http://w w w .proce sste x t.com/a bcchm.html
1. Criteria for es t abl i s hi ng ect opi c hormone s ecret i on by t umor cel l s i ncl ude: o
o
a. Increased hormone levels, as evi denced by radi oi mmunoas s ay or ot her t echni ques . However, s ome pat i ent s wi t h el evat ed hormone l evel s do not have cl i ni cal s ympt oms .
o
o
b. Decreased hormone levels after removal or treatment of the tumor
o
o
c. Persistent hormone elevation after removal of the normal gland t hat s ecret es t he normal hormone
o
o
d. An arteriovenous gradient for hormone levels acros s t he t umor vas cul ar bed
2. T ypes o
o
a. Ectopic growth hormone secretion has been det ect ed i n pat i ent s wi t h lung and gastric carcinomas. The el evat ed hormone l evel s may l ead t o hypert rophi c pul monary os t eoart hropat hy. Organomegal y may occur wi t h s l ow-growi ng carci noi d t umors .
o
o
b. Ectopic ACT H secretion was t he fi rs t paraneopl as t i c endocri ne s yndrome des cri bed i n t he
Pa g e 8 5 5
ABC Ambe r CHM Conve rte r Tria l ve rsion, http://w w w .proce sste x t.com/a bcchm.html
l i t erat ure.
(1) The mos t common tumors associated with ectopic ACT H production are SCLC and atypical carcinoids. Hi gh cort i s ol l evel s have al s o been des cri bed i n pat i ent s wi t h adenocarci noma and l arge-cel l carci noma of t he l ung, ot her carci noi d t umors , t hymoma, neural cres t t umors , medul l ary carci noma of t he t hyroi d, and bronchi al adenomas .
(2) Clinical features. Mos t pat i ent s have hypokalemia and metabolic alkalosis.
(a) Pat i ent s rarel y l i ve l ong enough for frank Cus hi ng's s yndrome t o devel op. However, di abet es , hypert ens i on, edema, mus cl e was t i ng, cent ral obes i t y, moon faci es , and s t ri ae may devel op i n t hos e wi t h ext remel y hi gh cort i s ol l evel s .
(b) Al t erat i ons i n ment al s t at us , fat i gue, and anorexi a caus ed by opi at e-l i ke pept i de fragment s are rare s ympt oms .
(3) Diagnosis i s confi rmed by a pl as ma ACTH 3
l evel of more t han 200 pg/mm , a pl as ma cort i s ol l evel of more t han 40 mg/dL wi t hout di urnal vari at i on, or a pos i t i ve dexamet has one s uppres s i on t es t .
B. Hematologic syndromes Pa g e 8 5 6
ABC Ambe r CHM Conve rte r Tria l ve rsion, http://w w w .proce sste x t.com/a bcchm.html
Hematologic syndromes may affect cel l ul ar bl ood el ement s , t he coagul at i on s ys t em, or ci rcul at i ng i mmunogl obul i ns .
1. Paraneoplastic syndromes of cellular blood components o
o
a. Erythrocytosis occurs i n renal cancer and hepat oma.
o
o
b. Pure red cell aplasia. Severe anemi a due t o a compl et e l ack of RBC product i on i s uncommon. However, 50% of pat i ent s wi t h pure red cel l apl as i a have thymoma. Many pat i ent s wi t h pure red cel l apl as i a al s o have ot her i mmunol ogi c abnormal i t i es s uch as hypogammagl obul i nemi a, paraprot ei nemi a, pos i t i ve ant i nucl ear ant i bodi es , or aut oi mmune hemol yt i c anemi a.
(1) Etiology. The caus e may rel at e t o effect s of s uppres s or T cel l s on RBC product i on, l eadi ng t o a s evere ret i cul ocyt openi c anemi a. P.176
(2) T herapy. In t hymoma, s urgery or radi at i on t herapy i s not uni forml y s ucces s ful i n revers i ng t he anemi a. Corticosteroids, splenectomy, and immunosuppressive therapy wi t h cycl ophos phami de may be us eful .
o
Pa g e 8 5 7
ABC Ambe r CHM Conve rte r Tria l ve rsion, http://w w w .proce sste x t.com/a bcchm.html
o
c. Autoimmune hemolytic anemia, or cold agglutinin disease, i s us ual l y caus ed by an i mmunogl obul i n produced by l ymphoma or chroni c l ymphocyt i c l eukemi a cel l s .
(1) RBC i ndi ces may be hi gh or l ow dependi ng on t he ret i cul ocyt e res pons e. If t he ret i cul ocyt e count i s hi gh, due t o normal marrow funct i on, t he i ndi ces are normal or hi gh. If i mmune hemol ys i s i s pres ent , mul t i pl e s pherocyt es are pres ent .
(2) T herapy wi t h prednisone (1–1.5 mg/kg/day) i s oft en effect i ve. Splenectomy i s al s o occas i onal l y us eful . In mos t cas es , however, nei t her s pl enect omy nor cort i cos t eroi ds are effect i ve for l ong peri ods unl es s t he underl yi ng mal i gnancy i s effect i vel y t reat ed.
o
o
d. Microangiopathic hemolytic anemia occurs wi t h muci n-produci ng adenocarci nomas , es peci al l y gas t ri c cancer.
(1) DIC can be di agnos ed by meas urement of fi bri nogen and fi bri n s pl i t product s .
(2) There i s no effect i ve treatment except t reat i ng t he underl yi ng t umor. Hepari n s houl d not be us ed unl es s t here i s concomi t ant DIC.
o
Pa g e 8 5 8
ABC Ambe r CHM Conve rte r Tria l ve rsion, http://w w w .proce sste x t.com/a bcchm.html
o
e. Granulocytosis occurs even i n t he abs ence of bone marrow i nvol vement . Ot her caus es of i nfl ammat i on or i nfect i on mus t be excl uded. The mos t common mal i gnanci es as s oci at ed wi t h granul ocyt os i s i ncl ude gas t ri c, l ung, and pancreat i c carci noma; mel anoma; CNS t umors ; Hodgki n's di s eas e; and l arge cel l l ymphomas .
o
o
f. T hrombocytosis occurs i n as many as one t hi rd of al l cancer pat i ent s . If es s ent i al t hrombocyt hemi a i s pres ent , hydroxyurea s houl d be gi ven t o keep 3
t he pl at el et count bel ow 500,000/mm . o
o
g. T hrombocytopenia i s oft en due t o s econdary effect s of chemot herapy, bone marrow i nvol vement , or radi at i on t herapy, but i s al s o s een i n s evere mi croangi opat hi c hemol yt i c anemi a and DIC. A s yndrome res embl i ng aut oi mmune t hrombocyt openi a has been des cri bed i n Hodgki n's di s eas e, l ymphoma, s ome t ypes of l eukemi a, and cert ai n s ol i d t umors . Thi s form of i di opat hi c t hrombocyt openi c purpura (ITP) i s t reat ed wi t h cort i cos t eroi ds , s pl enect omy, or i mmunos uppres s i ve drugs .
2. Paraneoplastic syndromes of the coagulation system o
o
a. Primary DIC occurs mos t frequent l y wi t h muci n-produci ng adenocarci nomas s uch as pancreat i c, gas t ri c, l ung, pros t at e, and col on cancers . Bot h acut e and chroni c DIC are as s oci at ed
Pa g e 8 5 9
ABC Ambe r CHM Conve rte r Tria l ve rsion, http://w w w .proce sste x t.com/a bcchm.html
wi t h t hrombophl ebi t i s , art eri al embol i , endocardi t i s , ci rcul at i ng i nhi bi t ors of coagul at i on, and abnormal ci rcul at i ng prot ei ns t hat may preci pi t at e hemorrhagi c compl i cat i ons .
(1) Acute DIC i s oft en obs erved i n acut e promyel ocyt i c l eukemi a, as t he res ul t of t he rel eas e of a procoagul ant cont ai ned i n t he abnormal granul es of t he l eukemi a promyel ocyt e.
(a) Clinical features. El evat i on of t he prot hrombi n t i me (PT) i s one of t he earl i es t det ect abl e s ympt oms al ong wi t h el evat i on of fi bri n s pl i t product s and l owered fi bri nogen l evel s . Furt her decreas es i n fi bri nogen l evel s and an i ncreas e i n t he part i al t hrombopl as t i n t i me (PTT) occur i n more s evere cas es .
(b) T herapy. Improved management of acut e DIC has l ed t o a bet t er s hort -t erm prognos i s for mos t pat i ent s .
(i) Meas ures i ncl ude aggressive transfusion support t o mai nt ai n t he pl at el et count bet ween 35,000 3
and 50,000/mm , intravenous vitamin K, and clotting factor replacement wi t h fres h-frozen pl as ma or cryopreci pi t at e i f t he fi bri nogen l evel i s l es s t han 75 mg/dL.
Pa g e 8 6 0
ABC Ambe r CHM Conve rte r Tria l ve rsion, http://w w w .proce sste x t.com/a bcchm.html
(ii) Heparin i s cont rovers i al but can be admi ni s t ered caut i ous l y t o refractory patients.
(2) Chronic DIC i s mos t common i n pat i ent s wi t h adenocarci nomas .
(a) Clinical features i ncl ude mi l d prol ongat i ons of t he PT, hi gh fi bri nogen l evel s , and el evat i on of t he fi bri n s pl i t product s . Pat i ent s oft en have t hrombophl ebi t i s or pul monary embol i .
(b) Heparin i s t he t reat ment of choi ce; thrombolytic therapy al s o may be cons i dered i f t here are no cont rai ndi cat i ons .
o
o
b. Syndromes associated with paraproteins
(1) Coagulopathy. Abnormal hemos t as i s and coagul at i on may devel op i n pat i ent s wi t h pl as ma cel l dys pl as i as s uch as mul t i pl e myel oma as a res ul t of t he effect s of paraprot ei ns on normal cl ot t i ng fact ors and pl at el et recept or funct i on. Paraprot ei ns can al s o i nhi bi t fi bri n P.177
Pa g e 8 6 1
ABC Ambe r CHM Conve rte r Tria l ve rsion, http://w w w .proce sste x t.com/a bcchm.html
monomer aggregat i on and act as i nhi bi t ors of fact or VIII. If pat i ent s have i nt ract abl e bl eedi ng, pl as mapheres i s may be requi red i n combi nat i on wi t h chemot herapy.
(2) Hyperviscosity. In mul t i pl e myel oma and macrogl obul i nemi a, pat i ent s wi t h a s erum vi s cos i t y great er t han 4 (rel at i ve t o wat er) may have s i gns of decreas ed bl ood fl ow, i ncl udi ng headaches , di zzi nes s , epi s t axi s , s ei zures , heari ng l os s , al t ered ment al s t at us , and cardi ac di s eas e. Treat ment cons i s t s of prompt pl as mapheres i s t o remove t he exces s i ve paraprot ei n and avoi d dehydrat i on, as wel l as t reat ment of t he underl yi ng pl as ma cel l dys cras i a.
C. CNS syndromes are rare The mos t common (and t he mal i gnanci es as s oci at ed wi t h t hem) are l i s t ed i n Tabl e 4-17.
TABLE 4.17 Central Nervous System Paraneoplastic Syndromes and Associated Malignancies
Pa g e 8 6 2
ABC Ambe r CHM Conve rte r Tria l ve rsion, http://w w w .proce sste x t.com/a bcchm.html
As soc iat ed Ma Sy lig nd na ro nc me y Li m Sm bi c al l enc cel l ep l un hal g i t i s can cer Su Sm bac al l ut e cel l cel l l un ul a g r
can
de cer ge , ner ova at i ri a on n can cer Su Ly bac mp ut e ho mo ma t or ne uro
Pa g e 8 6 3
ABC Ambe r CHM Conve rte r Tria l ve rsion, http://w w w .proce sste x t.com/a bcchm.html
pat hy Su Sm bac al l ut e cel l s en l un s or g y
can
ne cer uro pat hy Se Sm ns o al l ri m cel l ot o l un r
g
pol car yne ci n uro om pat a, hy Ho dgk i n's di s eas e Eat Lun on- g La can mb cer ert , my ova as t ri a he n ni a can
Pa g e 8 6 4
ABC Ambe r CHM Conve rte r Tria l ve rsion, http://w w w .proce sste x t.com/a bcchm.html
cer Pol Var ym i ed yos ; i t i s bre or as t der , ma l un t o g, my gas os i t roi t i s nt e sti nal , an d ova ri a n car ci n om as are mo st co mm on
XVIII. Oncologic Emergencies A. Spinal cord compression (SCC)
Pa g e 8 6 5
ABC Ambe r CHM Conve rte r Tria l ve rsion, http://w w w .proce sste x t.com/a bcchm.html
1. Etiology Mos t cas es of SCC are due t o met as t at i c t umors from pri mary breas t , l ung and pros t at e cancer; l ymphoma; mul t i pl e myel oma; renal and gas t roi nt es t i nal s t umors .
2. Clinical Features can i ncl ude back or radi cul ar pai n wi t h or wi t hout ot her s ympt oms s uch as mus cl e weaknes s ; acut e or s l owl y evol vi ng changes i n bowel or bl adder funct i on; or s ens ory l os s or aut onomi c dys funct i on. Thes e fi ndi ngs s houl d prompt a t horough phys i cal and neurol ogi c exami nat i on.
3. Diagnosis MRI wi t h gadol i ni um cont ras t i s t he gol d s t andard for di agnos i ng SCC becaus e of i t s hi gh s ens i t i vi t y and s peci fi ci t y.
4. T herapy The goal s of t herapy for SCC i ncl ude pai n cont rol , recovery of normal neurol ogi c funct i on, l ocal t umor cont rol , and avoi dance of compl i cat i ons . Al l pat i ent s who are s ympt omat i c and have an abnormal neurol ogi c exami nat i on s houl d recei ve s t eroi d t herapy i mmedi at el y. In addi t i on, s urgery and radi at i on oncol ogy cons ul t at i ons s houl d be purs ued i mmedi at el y upon cl i ni cal s us pi ci on of SCC i n a pat i ent wi t h mal i gnancy.
B. Superior vena cava syndrome (SVCS)
1. Etiology SVCS i s pri mari l y caus ed by di rect ext ri ns i c compres s i on of t he s uperi or vena cava (SVC) by t umors as s oci at ed wi t h bronchogeni c carci noma and non-Hodgki n l ymphoma affect i ng t he ant eri or medi as t i num. Ot her caus es i ncl ude met as t at i c di s eas e
Pa g e 8 6 6
ABC Ambe r CHM Conve rte r Tria l ve rsion, http://w w w .proce sste x t.com/a bcchm.html
t o t he medi as t i num, cent ral l i ne t hrombos i s , fi bros i ng medi as t i ni t i s , or a ret ros t ernal goi t er.
2. Clinical features Si gns and s ympt oms of SVCS may appear acut el y or chroni cal l y. Many pat i ent s wi l l exhi bi t neck or ches t wal l s uperfi ci al venous di s t ens i on, faci al and peri orbi t al edema, faci al pl et hora, cyanos i s , and upper ext remi t y edema. Ot her s ympt oms i ncl ude dys pnea, ort hopnea, cough, and hoars enes s .
3. Diagnosis Al t hough t he di agnos i s of SVCS can be made on phys i cal exami nat i on al one, noni nvas i ve i magi ng can al s o be us ed t o faci l i t at e t he di agnos i s . Such modal i t i es i ncl ude CT wi t h cont ras t , MRI, and Doppl er ul t ras onography exami nat i on of t he jugul ar or s ubcl avi an vei ns . P.178
4. T herapy If t here i s evi dence of res pi rat ory compromi s e or CNS i nvol vement , emergent radi at i on t herapy and ai rway s t abi l i zat i on mus t occur fi rs t . Nonemergent t reat ment cons i s t s of s ympt om rel i ef and t reat ment of t he underl yi ng mal i gnancy or i nfect i on caus i ng t he SVCS. If at al l pos s i bl e, i t i s i mport ant t o obt ai n t i s s ue pri or t o t reat ment i n order t o es t abl i s h t he di agnos i s .
C. Hypercalcemia
1. Etiology A l arge percent age of pat i ent s who pres ent wi t h hypercal cemi a have s ol i d t umor met as t as i s t o t he
Pa g e 8 6 7
ABC Ambe r CHM Conve rte r Tria l ve rsion, http://w w w .proce sste x t.com/a bcchm.html
bone. Ot her common mal i gnanci es as s oci at ed wi t h hypercal cemi a i ncl ude non–s mal l cel l l ung cancer, breas t cancer, mul t i pl e myel oma, and geni t ouri nary t umors .
2. Clinical features Pat i ent s may pres ent wi t h general i zed fat i gue, weaknes s , and dehydrat i on. Hypercal cemi a can al s o t arget s peci fi c organ s ys t ems s uch as CNS (ment al s t at us changes , s ei zures ), gas t roi nt es t i nal or geni t ouri nary t ract (naus ea/vomi t i ng, cons t i pat i on, i l eus ), and cardi ac (arrhyt hmi as ).
3. Diagnosis In hypercal cemi a of mal i gnancy, t he s erum i nt act parat hormone (i PTH) i s l ow or undet ect abl e and s erum parat hormone (PTH-RP) l evel s are el evat ed. Bot h 1, 25-di hydroxyergocal ci ferol and i norgani c phos phat e l evel s are l ow or normal .
4. T herapy Pat i ent s who are s ympt omat i c and found t o have s evere hypercal cemi a s houl d be hydrat ed vi gorous l y wi t h normal s al i ne pri or t o cons i derat i on of furos emi de. Once a pat i ent i s rehydrat ed and uri nary out put opt i mi zed, t he need for bi s phos phonat e t herapy wi t h zol edroni c aci d 4 mg i nt ravenous l y 15 mi nut es i nfus i on or pami dronat e or cal ci t oni n mus t be addres s ed. The mos t effect i ve t reat ment of hypercal cemi a, however, s houl d be di rect ed at t he underl yi ng mal i gnancy.
D. Tumor lysis syndrome (TLS)
1. Etiology Cel l deat h, wi t h s ubs equent rel eas e of i nt racel l ul ar cont ent s , i s a common occurrence aft er chemot herapy or i n t umors wi t h rapi d cel l t urnover. TLS
Pa g e 8 6 8
ABC Ambe r CHM Conve rte r Tria l ve rsion, http://w w w .proce sste x t.com/a bcchm.html
can occur i n t hes e s i t uat i ons , and i t can al s o be s een i n t he s et t i ng of t reat ment of l eukemi a or hi gh grade l ymphomas . Thi s phenomenon oft en res ul t s i n l i fe-t hreat eni ng l act i c aci dos i s , hyperuri cemi a, hyperkal emi a, hyperphos phat emi a, and hypocal cemi a.
2. Clinical features Many of t he s ympt oms of TLS are as s oci at ed wi t h t he el ect rol yt e abnormal i t i es . For i ns t ance, pat i ent s can devel op cardi ac arrhyt hmi as from s evere hyperkal emi a or hypocal cemi a. The hyperphos phat emi a and hyperuri cemi a of TLS may res ul t i n acut e renal fai l ure.
3. T herapy Correct i ve meas ures s houl d be di rect ed t oward t he met abol i c derangement s pres ent i n TLS. Oft en t hes e pat i ent s may requi re cl os er moni t ori ng ei t her on a t el emet ry uni t or i nt ens i ve care uni t i f t hey s houl d have s evere hemodynami c i ns t abi l i t y. The t reat ment of hyperuremi c acut e renal fai l ure fol l owi ng chemot herapy s houl d cons i s t s of al l opuri nol , l oop di uret i cs , i nt ravenous fl ui ds , and pos s i bl y recombi nant urat e oxi das e (ras buri cas e). It i s i mport ant t o i dent i fy i ndi vi dual s at ri s k for TLS pri or t o admi ni s t rat i on of chemot herapy. Thes e pat i ent s s houl d recei ve prehydrat i on, al l opuri nol , and met abol i c moni t ori ng for 24–28 hours aft er s t art i ng t herapy.
P.179
Study Questions/Answers and Explanations 1. A 55-year-old, postmenopausal woman presents for a yearly visit. A 1-cm nodule is palpated in the upper, outer quadrant of her right breast. Her most recent mammogram 6 months ago was negative. Her family history is significant for her mother
Pa g e 8 6 9
ABC Ambe r CHM Conve rte r Tria l ve rsion, http://w w w .proce sste x t.com/a bcchm.html
and a sister both diagnosed with breast cancer in their early to mid-50s. She has hypertension and hypercholesterolemia. What is the most appropriate next step in her management? A. Repeat mammogram and ul t ras ound now B. Repeat mammogram i n 6 mont hs C. Repeat cl i ni cal exami nat i on i n 1 mont h and refer for mammography i f t here i s an i ncreas e i n t he s i ze of t he nodul e D. Begi n t amoxi fen E. Refer her t o a s urgeon Vi ew Ans wer 1. T he answer is A [VI C]. A pal pabl e mas s duri ng t he phys i cal exami nat i on warrant s an i mmedi at e eval uat i on. Even t hough s he had undergone a mammography 6 mont hs ago, mos t oncol ogi s t s , radi ol ogi s t s , and s urgeons woul d repeat t hi s s t udy. In addi t i on, an ul t ras ound eval uat i on mi ght reveal whet her t hi s i s a cys t or a more s ol i d mas s . Repeat i ng t he mammogram i n 6 mont hs i s not a good opt i on, becaus e t hi s wi l l del ay t reat ment i f t he abnormal fi ndi ng i s i n fact a mal i gnancy. The s ame i s t rue for ans wer C. Vi rt ual l y no one wi l l begi n t herapy wi t hout a fi rm di agnos i s ; t herefore, ans wer D i s i ncorrect . 2. T he yearly mammogram in a 65-year-old postmenopausal woman shows an irregular area of microcalcification, which has grown in size compared with her mammogram from 2 years ago. She missed her mammogram last year. Physical examination is unrevealing, without lymphadenopathy or any nodularity in the breasts. You refer her to a surgeon. Eventually a 2-cm invasive ductal carcinoma is removed from her left breast. Sentinel lymph node biopsy shows two positive lymph nodes, and axillary lymph node dissection indicates five more positive lymph nodes. T he tumor expresses the estrogen receptor (ER+). Which of the following interventions would increase her chance of cure? A. Chemot herapy fol l owed by hormonal t herapy B. Radi at i on t herapy C. Tot al mas t ect omy D. Hormonal t herapy al one
Pa g e 8 7 0
ABC Ambe r CHM Conve rte r Tria l ve rsion, http://w w w .proce sste x t.com/a bcchm.html
E. Hi gh-dos e chemot herapy wi t h s t em cel l s upport Vi ew Ans wer 2. T he answer is A [VI H 3]. Thi s pat i ent i s at hi gh ri s k of recurrence by vi rt ue of havi ng di s eas e i n her l ymph nodes . Radi at i on t herapy or s urgi cal exci s i on of t he breas t cancer us ual l y provi de l ocal cont rol and provi de adequat e prot ect i on agai ns t recurrence of di s eas e i n t he t umor bed. However, mi cromet as t at i c di s eas e can onl y be addres s ed t hrough t he admi ni s t rat i on of s ys t emi c t herapy s uch as cyt ot oxi c chemot herapy. In pat i ent s wi t h hormonal l y s ens i t i ve t umors (e.g., es t rogen recept or pos i t i ve), hormonal t herapy has been s hown t o be very effect i ve i n t he prevent i on of di s eas e recurrence aft er chemot herapy and can be us ed as a s ol e t reat ment opt i on i n s el ect ed pat i ent s . Randomi zed cl i ni cal t ri al s have s hown t hat l umpect omy and radi at i on t herapy i s equal t o a t ot al mas t ect omy i n t erms of overal l s urvi val . For t umors t hat can be removed t ot al l y wi t h adequat e margi ns (>10 mm), l umpect omy offers bet t er cos met i c and ps ychol ogi cal res ul t s . It i s a l es s i nvas i ve s urgery and al l ows for fas t er recovery t i me. 3. An 18-year-old man presents with swollen lymph nodes in his neck for 2 weeks. He has been treated with antibiotics for 10 days, but the lymph nodes continue to grow. He has had a 10-lb. weight loss in the past 2 months and is experiencing drenching night sweats. He reports fatigue and inability to play basketball like he used to. He has not been sexually active in the last 1 year. T he next best course of action is: A. Changi ng t he ant i bi ot i cs B. CT s can of t he ches t C. Refer t o a s urgeon for l ymph node bi ops y D. Check t he HIV s t at us of t hi s pat i ent E. As s ure hi m t hat he has a s el f-l i mi t i ng vi ral i l l nes s Vi ew Ans wer 3. T he answer is C [XV H]. Any l ymph node t hat does not res pond t o t he us ual t reat ment , s uch as t wo cours es of di fferent ant i bi ot i cs , s houl d rai s e t he s us pi ci on of t he t reat i ng phys i ci ans . For eas i l y acces s i bl e l ymph nodes , t he preferred met hod of i nves t i gat i on i s a
Pa g e 8 7 1
ABC Ambe r CHM Conve rte r Tria l ve rsion, http://w w w .proce sste x t.com/a bcchm.html
ful l exci s i onal bi ops y. The exami nat i on of an i nt act l ymph node al l ows pat hol ogi s t s t o exami ne t he l ymph node archi t ect ure and arri ve at a defi ni t i ve di agnos i s . Al t hough a CT s can mi ght event ual l y be needed, i t s houl d not be t he fi rs t choi ce. Al s o, t he HIV s t at us of a pat i ent wi t h hi gh-ri s k behavi or needs t o be det ermi ned, but t here i s no i ndi cat i on from t he cas e pres ent at i on t hat t hi s pat i ent fal l s i n t hi s cat egory and i t s houl d not be t he fi rs t choi ce. 4. Eventually he is diagnosed with nodular sclerosing Hodgkin' s disease. His bone marrow biopsy is negative, and he is found to have disease in both sides of the diaphragm (stage III). T reatment of choice in the United States is: A. Chemot herapy wi t h t he ABVD regi men B. Radi at i on t o al l t he affect ed l ymph nodes C. Surgi cal res ect i on of al l t he affect ed l ymph nodes D. Treat ment wi t h ri t uxi mab E. Hi gh-dos e chemot herapy wi t h s t em cel l s upport Vi ew Ans wer 4. T he answer is A [XV I 3 a]. Thi s combi nat i on chemot herapy i s t he t reat ment of choi ce i n mos t part s of t he Uni t ed St at es . Ot her mul t i -agent chemot herapy regi mens are al s o accept abl e. Ongoi ng cl i ni cal t ri al s are compari ng effect i ve regi mens t o det ermi ne whet her t here are s uperi or al t ernat i ves t o t he ABVD regi men. There i s no i ndi cat i on t hat t he removal of al l i nvol ved l ymph nodes i s s urgi cal l y feas i bl e or t herapeut i c. Di s s emi nat ed Hodgki n's l ymphoma i s a s ys t emi c di s eas e and s houl d be t reat ed as s uch. In addi t i on, radi at i on t herapy i s res erved for cas es i n whi ch t here i s ei t her bul ky di s eas e or a focal area of t he body i s i nvol ved. Del i very of radi at i on t o mul t i pl e s i t es i s general l y not feas i bl e becaus e of t he t oxi ci t y of t hi s approach. Choi ce D, t reat ment wi t h ri t uxi mab, i s for non-Hodgki n's l ymphomas . 5. T he most serious long-term side effect of this treatment is: A. Conges t i ve heart fai l ure B. Peri pheral neuropat hy C. Myel odys pl as t i c s yndrome/acut e myel ogenous l eukemi a
Pa g e 8 7 2
ABC Ambe r CHM Conve rte r Tria l ve rsion, http://w w w .proce sste x t.com/a bcchm.html
D. Earl y myocardi al i nfarct i on E. Earl y ons et cat aract Vi ew Ans wer 5. T he answer is C [XV I c]. Cyt ot oxi c chemot herapy coul d l ead t o vari ous chromos omal abnormal i t i es t hat coul d res ul t i n acut e l eukemi a or myel odys pl as t i c s yndrome. The ri s k of s econdary myel oi d mal i gnanci es approaches 2% wi t h t he ABVD regi men, and i ncreas ed i nci dence of non-Hodgki n's l ymphoma i s not ed as wel l . A number of chemot herapeut i c agent s t hat al kyl at e DNA are t hought t o be i nvol ved i n t hi s proces s . In general , hi gh-dos e t herapy wi t h any of t hes e agent s i s as s oci at ed wi t h t he devel opment of AML/MDS. Conges t i ve heart fai l ure as a res ul t of expos ure t o doxorubi ci n (Adri amyci n) i s known t o occur. However, us ual l y t hi s 2
t oxi ci t y i s s een wi t h dos es exceedi ng 480 mg/m . The ri s k of myocardi al i nfarct i on at a young age s houl d not be an i s s ue i f radi at i on t o t he ches t and t horax i s not needed. Peri pheral neuropat hy i s al s o a manageabl e t oxi ci t y s een wi t h vi nca al kal oi ds s uch as vi ncri s t i ne. P.180
6. A 55-year-old African American man is seen in the emergency room with a 2-day history of hematuria, back pain, double vision, and altered mental status. He is an engineer with an office-based occupation with no exposure to chemicals or radiation. Family history is positive for an uncle with “ bone cancer.― He smokes half pack of cigarettes a day but does not drink alcohol. Laboratory evaluation in the emergency department shows an elevated total protein of 22 mg/dL, elevated gamma globulins, a serum creatinine of 2.5 mg/dL, and hemoglobin of 9 g/L. Blood smear shows roulette formation. You suspect: A. Acut e renal fai l ure of unknown et i ol ogy B. Hypervi s cos i t y s yndrome s econdary t o mul t i pl e myel oma C. A geni t ouri nary mal i gnancy
Pa g e 8 7 3
ABC Ambe r CHM Conve rte r Tria l ve rsion, http://w w w .proce sste x t.com/a bcchm.html
D. Drug t oxi ci t y E. Int ravas cul ar hemol ys i s Vi ew Ans wer 6. T he answer is B [XIV E 2]. Thi s chal l engi ng cas e requi res a cl os e exami nat i on of t he l aborat ory res ul t s and of t he pat i ent pres ent at i on i n an at t empt t o arri ve at a uni fyi ng di agnos i s t hat expl ai ns al l of t he obs erved abnormal i t i es . The el evat ed t ot al prot ei n and t he pres ence of roul et t es i n t he bl ood s mear are cl as s i c s i gns of paraprot ei nemi a. The hi s t ory of change i n ment al s t at us and doubl e vi s i on, i n a pat i ent wi t h el evat ed t ot al prot ei n, s houl d rai s e concerns for t he devel opment of hypervi s cos i t y s yndrome. Once t he pres ence of anemi a and renal i ns uffi ci ency has been es t abl i s hed, t he di agnos i s of mul t i pl e myel oma s houl d s t rongl y be cons i dered. A GU mal i gnancy us ual l y does not l ead t o an el evat ed t ot al prot ei n or changes i n t he bl ood s mear as di s cus s ed i n t hi s cas e. 7. Best management option at this point is: A. Pl as mapheres i s B. Trans fus i on of red bl ood cel l s C. Hemodi al ys i s D. Broad-s pect rum ant i bi ot i cs E. Support i ve care Vi ew Ans wer 7. T he answer is A [XIV I 2 c (4)]. Pl as mapheres i s offers t he onl y opt i on for reduci ng t he amount of prot ei n i n t he ci rcul at i on t o al l evi at e s ome of t he s ympt oms . Bl ood t rans fus i ons coul d be det ri ment al unt i l t he hypervi s cos i t y has been i mproved, becaus e bl ood t rans fus i ons wi l l rai s e bl ood vi s cos i t y. The ot her t wo opt i ons woul d not addres s t he i s s ue wi t h hi gh prot ei n cont ent i n t he ci rcul at i on. 8. A 75-year-old woman with metastatic breast cancer to her bones presents to the emergency department because she fell down at home after getting up from her chair. Her family states that she has been lethargic, nauseated, and notes that she has complained of dizziness and increasing thirst. She is found to be
Pa g e 8 7 4
ABC Ambe r CHM Conve rte r Tria l ve rsion, http://w w w .proce sste x t.com/a bcchm.html
orthostatic on examination.
Laboratory studies:
Blood urea nitrogen 42 mg/dL
T otal serum calcium 12.2 mg/dL
Serum creatinine 1.4 mg/dL
Serum albumin 2.8 g/dL
What is the most appropriate initial treatment?
A. Sl ow rehydrat i on wi t h hal f-normal s al i ne B. Int ravenous admi ni s t rat i on of a bi s phos phonat e C. Vi gorous rehydrat i on wi t h normal s al i ne D. Int ravenous admi ni s t rat i on of furos emi de al ong wi t h s al i ne rehydrat i on E. Int ravenous admi ni s t rat i on of cort i cos t eroi ds Vi ew Ans wer 8. T he answer is C [XVIII C 4]. The pat i ent has hypercal cemi a, t he mos t common met abol i c compl i cat i on of mal i gnancy. She i s profoundl y dehydrat ed, as wi t nes s ed by her ort hos t as i s and s ubs equent fal l at home. Ini t i al l y, t he pat i ent s houl d be adequat el y rehydrat ed wi t h normal s al i ne pri or t o admi ni s t rat i on of furos emi de. Admi ni s t rat i on of a bi s phos phonat e woul d not be appropri at e i n t he acut e s et t i ng becaus e of t he pat i ent 's vol ume depl et i on. For t he s ame reas on, whi l e cort i cos t eroi ds can be cons i dered an adjunct i ve t herapy i n a pat i ent wi t h a pot ent i al l y hormone-res pons i ve t umor, t he mai n pri ori t y i n t hi s s everel y dehydrat ed pat i ent s houl d be vol ume repl et i on.
Pa g e 8 7 5
ABC Ambe r CHM Conve rte r Tria l ve rsion, http://w w w .proce sste x t.com/a bcchm.html
9. A 50-year-old executive undergoes his first screening colonoscopy. He has been constipated for 1 year, relieved by laxatives. T here has been no blood in his stools. A fungating mass is seen in the sigmoid colon. Subsequent surgery shows a lesion that has penetrated through the muscularis propria. T hree regional lymph nodes are positive for adenocarcinoma. His best chance for cure would occur with which form of therapy? A. No furt her t herapy needed B. Adjuvant chemot herapy C. Radi at i on t reat ment D. Concurrent chemot herapy and radi at i on t herapy E. Immunot herapy Vi ew Ans wer 9. T he answer is B [VII 9 a (3) (a)]. Of t he opt i ons pres ent ed, onl y adjuvant chemot herapy woul d i ncreas e t he l i kel i hood of cure for a pat i ent who has had res ect i on of s t age III (e.g., l ymph node met as t as i s pos i t i ve, T a n y , N 1
or 2
, M 0 ) adenocarci noma of t he col on.
Thi s has been repeat edl y demons t rat ed i n a number of cl i ni cal t ri al s . Once cancer has been found i n t he l ymph nodes , t he pat i ent has a hi gh rat e of s ys t emi c di s s emi nat i on. The gui del i nes are l es s cl ear for t he management of pat i ent s wi t h t rans mural l y i nvas i ve cancers t hat do not met as t as i ze t o regi onal l ymph nodes (e.g., s t age II, T 3 N 0 M 0 ). Al t hough met a-anal ys i s s ugges t s a very s mal l advant age for t he us e of 5-fl uorouraci l –bas ed adjuvant chemot herapy, mos t aut hori t i es bas ed t reat ment recommendat i ons on t he bal ance of ant i ci pat ed benefi t wi t h pat i ent fi t nes s and t reat ment preferences . The prel i mi nary res ul t s of a randomi zed cl i ni cal t ri al empl oyi ng mul t i -agent chemot herapy wi t h 5-fl uorouraci l and oxal i pl at i n s hows a s urvi val advant age for t he combi nat i on i n pat i ent s wi t h bot h s t age II and s t age III di s eas es , and may change t he adjuvant t herapy paradi gm i n t he fut ure. 10. A 26-year-old black woman is back in your office after the return of her Pap screening. It showed a low-grade squamous intraepithelial lesion. T here was no inflammation found. She was
Pa g e 8 7 6
ABC Ambe r CHM Conve rte r Tria l ve rsion, http://w w w .proce sste x t.com/a bcchm.html
also found to be HIV-negative, and a Pap smear 2 years ago was negative. T he most appropriate next step in her treatment is: A. Bi ops y B. Coni z at i on C. Cryo- or l as er t herapy D. Ret urn (4 t o 6 mont hs ) for repeat Pap s mear E. Hys t erect omy Vi ew Ans wer 10. T he answer is A. [VIII B 6 a]. The correct ans wer i s bi ops y. Cervi cal cancer mort al i t y i s s o prevent abl e becaus e i t i s eas i l y reveal ed by Pap s mear. But i t i s onl y a s creeni ng t es t . If t he Pap s mear ret urns wi t h l ow grade s quamous i nt raepi t hel i al l es i on (or hi gh grade) t hen a punch bi ops y i s requi red for di agnos i s of CIN or i nvas i ve carci noma. If t he bi ops y t hen s hows CIN I (s l i ght dys pl as i a) t hi s may res ol ve, and does not requi re t reat ment . Fol l ow-up wi t h a repeat Pap s mear i n 4-6 mont hs . CIN II and III (moderat e and s evere dys pl as i a) are t reat ed wi t h abl at i ve t herapy, ei t her cryot herapy or l as er. If t he punch bi ops y ret urns s howi ng i nvas i ve carci noma, s t agi ng i s done, and t he pat i ent i s t reat ed wi t h hys t erect omy or radi at i on, or bot h. If t he bi ops y di d not make t he s t age of t he premal i gnancy cl ear, t hen coni zat i on i s i ndi cat ed. P.181
11. A 57-year-old postmenopausal woman has completed her adjuvant treatment for a stage II, ER-/PR-positive breast cancer. She is presenting for a discussion regarding initiation of hormonal treatment with tamoxifen. T he optimal duration of treatment with this agent is: A. 10 years B. 5 years C. 2 years D. Li fet i me E. Int ermi t t ent t reat ment for 5 years
Pa g e 8 7 7
ABC Ambe r CHM Conve rte r Tria l ve rsion, http://w w w .proce sste x t.com/a bcchm.html
Vi ew Ans wer 11. T he answer is B [VI I 4 a (1) (a-b)]. The opt i mal durat i on of t reat ment wi t h t amoxi fen i s 5 years . Large randomi zed cl i ni cal t ri al s have proved t hat s hort er t reat ment durat i on i s not as prot ect i ve as 5 years of t reat ment . Al s o, l onger durat i on of t reat ment (10 years ) i s as s oci at ed wi t h more s i de effect s s uch as t hromboembol i c phenomenon. 12. A 67-year-old man with a significant smoking history has had a persistent right upper lobe infiltrate. A CT scan shows a 3-cm solitary lesion with irregular borders. A fine-needle aspiration shows adenocarcinoma. Appropriate staging workup at this point includes: A. Bone marrow bi ops y B. PA and l at eral ches t radi ograms C. Lymphangi ogram D. CT s cans of t he ches t , abdomen, and pel vi s and a PET s can E. Vi deo-as s i s t ed t horaci c s urgery (VATS) Vi ew Ans wer 12. T he answer is D [V F]. The appropri at e s t agi ng of a pat i ent wi t h a di agnos i s of l ung cancer requi res al l t he t es t s ment i oned i n t hi s ans wer. The goal i s t o rul e out t he pres ence of gros s di s eas e i n al l t he areas where l ung cancer can pot ent i al l y met as t as i ze. Thi s i ncl udes l i ver, t he adrenal gl ands , medi as t i nal l ymph nodes , and cont ral at eral ches t . Rout i ne ches t fi l ms are not s uffi ci ent l y s ens i t i ve t o det ect s mal l er l ymph nodes i n t he hi l ar regi on. Nei t her bone marrow bi ops i es nor l ymphangi ograms are i ndi cat ed for s t agi ng of l ung cancer. In fact , l ymphangi ograms are no l onger rout i nel y performed. PET s cans are i ncreas i ngl y us ed i n t he i ni t i al s t agi ng of pat i ent s wi t h l ung cancer becaus e of t hei r s ens i t i vi t y i n det ect i ng mi ni mal di s eas e. Accurat e s t agi ng i nformat i on has a major i mpact on t he choi ce of t herapy for t hi s di s eas e. A pat i ent who, bas ed on i ni t i al CT s cans , i s cons i dered t o be a candi dat e for pot ent i al l y curat i ve s urgery mi ght be found t o have met as t at i c di s eas e by PET s can and t herefore be a candi dat e for pal l i at i ve chemot herapy.
Pa g e 8 7 8
ABC Ambe r CHM Conve rte r Tria l ve rsion, http://w w w .proce sste x t.com/a bcchm.html
13. A 48-year-old woman, smoker, with a history of Hodgkin' s disease at the age of 29, treated with chest radiation, has developed a squamous cell carcinoma of the lung. Staging workup shows only a 2.5-cm solitary lesion in the left upper lobe. Her FEV1 and FVC are 70% of the predicted values, respectively. T he best treatment option for her is: A. Radi at i on B. Chemot herapy wi t h carbopl at i n and Taxol C. Lobect omy D. Concurrent chemot herapy and radi at i on E. Radi at i on fol l owed by s urgery Vi ew Ans wer 13. T he answer is C [V I 1]. Treat ment of choi ce for s t age I l ung cancer i s s urgi cal res ect i on of t he t umor. Thi s can be accompl i s hed by a l obect omy or a wedge res ect i on of t he i nvol ved l ung s egment . Surgi cal res ect i on for s t age I or II non–s mal l cel l l ung cancer i s pot ent i al l y curat i ve, wi t h 5-year s urvi val of cl os e t o 70%. Nei t her chemot herapy al one nor concurrent chemoradi at i on t herapy i s appropri at e for a young ot herwi s e heal t hy pat i ent wi t h earl y s t age l ung cancer. 14. A 60-year-old man who is newly diagnosed with prostate carcinoma comes to see you. He underwent transrectal ultrasonography with a needle biopsy showing adenocarcinoma. Except for elevated PSA, he is currently asymptomatic. T he first test to order in staging for his prostate cancer is: A. Ful l body pl ai n fi l m x-rays B. Bone s can C. CT of head D. CEA l evel E. Repeat PSA now Vi ew Ans wer 14. T he answer is B [X C 6 b (2)]. The correct ans wer i s t o do a bone s can. Thi s i s t he fi rs t s t agi ng t es t done i n t he workup of pros t at e cancer. If abnormal i t i es are found on t he bone s can you t hen proceed t o do pl ai n fi l m x-rays of t he areas t o excl ude ot her
Pa g e 8 7 9
ABC Ambe r CHM Conve rte r Tria l ve rsion, http://w w w .proce sste x t.com/a bcchm.html
pos s i bl e caus es . Then you woul d do s urgi cal s t agi ng wi t h removal and exami nat i on of t he s urroundi ng nodes (oft en done wi t h pros t at ect omy).
Pa g e 8 8 0
ABC Ambe r CHM Conve rte r Tria l ve rsion, http://w w w .proce sste x t.com/a bcchm.html
Editors: Wolfsthal, Susan T itle: NMS Medicine, 6th Edition Copyri ght ©2008 Li ppi ncot t W i l l i ams & W i l ki ns > T able of Cont ent s > Chapt er 5 - Gast roint est inal Diseases
Chapter 5
Gastrointestinal Diseases Darryn Potosky Bruce Greenwald
I. Diseases of the Esophagus The es ophagus i s bas i cal l y an organ of t rans port , wi t h no s i gni fi cant abs orpt i ve or s ecret ory funct i on.
A. Features common to clinical disorders
1. Dysphagia, or di ffi cul t y i n s wal l owi ng, i s a s ympt om oft en des cri bed as a sticking sensation. o
o
a. Dysphagia for solids i ndi cat es an es ophageal obs t ruct i on as a res ul t of:
(1) Carci noma
(2) An es ophageal web or ri ng
(3) Beni gn es ophageal s t ri ct ure
o
o
b. Dysphagia for solids and liquids i ndi cat es an es ophageal abnormal i t y as a res ul t of mot or dys funct i on, s uch as :
Pa g e 8 8 1
ABC Ambe r CHM Conve rte r Tria l ve rsion, http://w w w .proce sste x t.com/a bcchm.html
(1) Scl eroderma
(2) Achal as i a
(3) Sympt omat i c di ffus e es ophageal s pas m (SDES)
o
o
c. T ransfer dysphagia i ndi cat es a di ffi cul t y i n i ni t i at i ng s wal l owi ng and i s oft en as s oci at ed wi t h:
(1) A neuromus cul ar di s order of t he pharynx or proxi mal es ophagus (e.g., aft er a cerebrovas cul ar acci dent )
(2) Proxi mal mus cl e weaknes s of t he pharynx or es ophagus (e.g., pol ymyos i t i s )
(3) Or ot her neuromus cul ar di s orders (myas t heni a gravi s , myot oni a dys t rophi ca, or Parki ns on's di s eas e).
2. Odynophagia, or pai n on s wal l owi ng, may be due t o: o
o
a. Motor disorders of t he es ophagus , es peci al l y achal as i a and SDES
o
o
b. Mucosal disruption caus ed by:
Pa g e 8 8 2
ABC Ambe r CHM Conve rte r Tria l ve rsion, http://w w w .proce sste x t.com/a bcchm.html
(1) Severe pept i c es ophagi t i s
(2) Severe i nfect i ons of t he es ophagus [candi dal es ophagi t i s , herpet i c es ophagi t i s , cyt omegal ovi rus (CMV)],
(3) Drug-i nduced es ophagi t i s (s ee I B 4 f)
(4) Radi at i on es ophagi t i s
(5) Inges t i on of l ye or ot her caus t i c agent s
3. Heartburn i s a s ubs t ernal burni ng s ens at i on t hat radi at es t owards t he mout h and may be i ni t i at ed by bendi ng forward.
B. Specific disorders
1. Reflux esophagitis i s caus ed by t he recurrent refl ux of gas t ri c cont ent s i nt o t he di s t al es ophagus . o
o
a. Etiology and pathogenesis
(1) Lower esophageal sphincter (LES) dysfunction. Normal l y, t he LES bl ocks refl ux of gas t ri c jui ce i nt o t he es ophagus . Refl ux es ophagi t i s i s t hought t o s t em from a defect i n t hi s LES mechani s m, s uch as :
Pa g e 8 8 3
ABC Ambe r CHM Conve rte r Tria l ve rsion, http://w w w .proce sste x t.com/a bcchm.html
(a) Decreas ed res t i ng LES t one P.186
(b) Prol onged or repeat ed i nt ermi t t ent t rans i ent rel axat i on of t he LES
(c) Trans i ent i ncreas e i n abdomi nal pres s ure
(2) Secondary causes of refl ux es ophagi t i s s houl d al ways be s us pect ed and correct ed i f pos s i bl e. The fol l owi ng condi t i ons appear t o decreas e LES t one:
(a) Pregnancy. Es peci al l y duri ng t he l as t t ri mes t er, heart burn may be s evere and probabl y i s caus ed by proges t erone's i nhi bi t ory effect s on t he LES.
(b) Drugs. Medi cat i ons t hat may decreas e t he LES t one as a s i de effect of s moot h mus cl e rel axat i on i ncl ude:
(i) Ant i chol i nergi c agent s
(ii) β 2 -Adrenergi c agoni s t s and t heophyl l i ne
Pa g e 8 8 4
ABC Ambe r CHM Conve rte r Tria l ve rsion, http://w w w .proce sste x t.com/a bcchm.html
(iii) Cal ci um channel –bl ocki ng agent s and ni t rat es
(c) Scleroderma. W eakeni ng of t he es ophageal s moot h mus cl e and t he LES regi on caus es s evere refl ux es ophagi t i s .
(d) Surgical vagotomy. Thi s procedure al s o may produce anat omi c al t erat i ons t hat l ead t o refl ux es ophagi t i s .
o
o
b. Clinical features
(1) Heartburn or acid regurgitation i s a s peci fi c s ympt om of gas t roes ophageal refl ux.
(2) Dysphagia i n t he es ophagi t i s pat i ent general l y i s for s ol i ds and may i ndi cat e a devel opi ng s t ri ct ure.
(3) Anemia may occur i f recurrent es ophageal bl eedi ng i s pres ent .
(4) Extra-esophageal manifestations i ncl ude recurrent l aryngi t i s , refl ux-i nduced as t hma, cough, atypical chest pain, and hi ccups .
o
o
c. Diagnosis.
Pa g e 8 8 5
ABC Ambe r CHM Conve rte r Tria l ve rsion, http://w w w .proce sste x t.com/a bcchm.html
(1) Barium swallow and upper gastrointestinal series. Thi s i s t he l eas t s ens i t i ve t es t . General l y, i t i s pos i t i ve onl y i n s evere gas t roes ophageal refl ux wi t h a very weakened LES or i n t he pres ence of es ophageal ul cerat i on.
(2) T wenty-four hour pH monitoring. Thi s meas ures t he number of epi s odes and l engt h of t i me t hat t he di s t al es ophageal pH i s l es s t han 4.
(3) Endoscopy with biopsy. Thi s procedure i s es peci al l y hel pful i n rul i ng out as s oci at ed Barret t 's es ophagus [s ee I B 1 e (5)]. However, eros i ve changes are s een i n onl y a mi nori t y of pat i ent s who undergo endos copy. A s quamous papi l l oma i s a s mal l beni gn pol yp at t he end of t he es ophagus , t hought t o be s econdary t o chroni c refl ux es ophagi t i s .
(4) Esophageal manometry. Thi s procedure i s us eful i n eval uat i ng LES pres s ure. Pres s ures cons i s t ent l y meas ured at l es s t han one-t hi rd of t he l ower l i mi t of normal us ual l y are as s oci at ed wi t h s i gni fi cant refl ux. Thi s t echni que i s es peci al l y us eful i n t he preoperat i ve eval uat i on of es ophageal refl ux when a fundopl i cat i on i s cont empl at ed.
o
o
d. T herapy
Pa g e 8 8 6
ABC Ambe r CHM Conve rte r Tria l ve rsion, http://w w w .proce sste x t.com/a bcchm.html
(1) Increasing the reflux barrier may be accompl i s hed by:
(a) Li fes t yl e modi fi cat i on s uch as el evat i ng t he head of t he bed 4–6 i nches , avoi di ng eat i ng for 3 hours before bedt i me, and avoi di ng fat t y or l arge-vol ume meal s
(b) Admi ni s t rat i on of al gi ni c aci d and ant aci d combi nat i ons
(c) Admi ni s t rat i on of drugs t hat i ncreas e LES t one (e.g., bet hanechol , met ocl oprami de, domperi done, ci s apri de)
(d) Ant i refl ux s urgery, es peci al l y Ni s s en fundopl i cat i on
(2) Decreasing gastric acid effects may be accompl i s hed by:
(a) Admi ni s t rat i on of ant aci ds
(b) Admi ni s t rat i on of hi s t ami ne 2 (H 2 )-recept or ant agoni s t s (e.g., ci met i di ne, famot i di ne, rani t i di ne, and ni zat i di ne)
(c) Admi ni s t rat i on of
Pa g e 8 8 7
ABC Ambe r CHM Conve rte r Tria l ve rsion, http://w w w .proce sste x t.com/a bcchm.html
hydrogen/pot as s i um prot on pump bl ockers (prot on pump i nhi bi t ors ) s uch as omeprazol e, l ans oprazol e, pant oprazol e, es omeprazol e, and rabeprazol e
(3) Maintaining LES pressure. The fol l owi ng agent s decreas e LES pres s ure and s houl d be avoi ded:
(a) Medi cat i ons (ant i chol i nergi cs , β-adrenergi cs , cal ci um channel †“bl ockers , and ni t rat es )
(b) Foods (chocol at e, fat s , and peppermi nt )
(c) Ot her (ni cot i ne and caffei ne)
P.187
o
o
e. Complications
(1) Benign esophageal stricture probabl y occurs i n a s mal l port i on of al l refl ux es ophagi t i s cas es and i s bes t di agnos ed by a bol us bari um s wal l ow, endos copy wi t h bi ops y, or cyt ol ogy.
(2) Esophageal ulceration may be
Pa g e 8 8 8
ABC Ambe r CHM Conve rte r Tria l ve rsion, http://w w w .proce sste x t.com/a bcchm.html
accompani ed by hemorrhage. However, t he pri mary s ympt om i s s evere and unrel ent i ng pai n.
(3) Reflux-induced laryngitis i s a common caus e of recurrent hoars enes s i n adul t s .
(4) Pulmonary aspiration i s a s eri ous s equel a of refl ux es ophagi t i s . Pat i ent s ol der t han 30 years of age who devel op repeat ed pneumoni a or as t hma s houl d be eval uat ed for es ophageal refl ux.
(5) Barrett' s esophagus refers t o a condi t i on i n whi ch col umnar epi t hel i um repl aces t he normal s quamous epi t hel i um of t he es ophagus , pos s i bl y as a res ul t of cont i nuous i nfl ammat i on. Thi s i s cons i dered a premal i gnant s t at e, wi t h a 40-fol d i ncreas ed ri s k of adenocarci noma.
2. Obstructive esophageal conditions o
o
a. Carcinoma
(1) Epidemiology. In t he Uni t ed St at es , t he i nci dence of adenocarci noma of t he es ophagus i s equi val ent t o t hat of s quamous cel l carci noma of t he es ophagus . Carci noma occurs predomi nant l y i n men and wi t h varyi ng i nci dence t hroughout t he worl d. The
Pa g e 8 8 9
ABC Ambe r CHM Conve rte r Tria l ve rsion, http://w w w .proce sste x t.com/a bcchm.html
popul at i on of t he Uni t ed St at es i s cons i dered at l ow ri s k; es ophageal cancer occurs i n onl y 4 of 100,000 i ndi vi dual s .
(2) Etiology. Cert ai n fact ors appear t o i ncreas e t he ri s k of es ophageal cancer.
(a) T obacco smoking, whi ch i ncreas es t he ri s k of s quamous cel l carci noma t wofol d t o fourfol d
(b) Alcohol consumption, whi ch has been s hown t o i ncreas e t he ri s k of s quamous cel l carci noma up t o 12 t i mes i n France. Al cohol and t obacco appear t o have an addi t i ve effect .
(c) Geographic factors. Inci dence l evel s , whi ch were found t o be 400 t i mes great er i n cert ai n regi ons i n Chi na and Iran, may be at t ri but abl e t o a di et t hat i ncl udes i ncreas ed amount s of pi ckl ed food, ni t ros ami nes , and mol ds as wel l as decreas ed amount s of s el eni um, fres h frui t s , and veget abl es .
See ot her fact ors onl i ne.
(d) Vitamin deficiency, es peci al l y vi t ami ns A and C, whi ch may be as s oci at ed wi t h an i ncreas ed ri s k of es ophageal cancer
(e) Lye ingestion, whi ch i s as s oci at ed wi t h t he devel opment of es ophageal cancer many years aft er expos ure
Pa g e 8 9 0
ABC Ambe r CHM Conve rte r Tria l ve rsion, http://w w w .proce sste x t.com/a bcchm.html
(f) Achalasia, whi ch may be as s oci at ed wi t h a 10% ri s k of s ubs equent carci nomas
(g) Barrett' s esophagus [s ee I B 1 e (5)]. Adenocarci noma event ual l y may devel op i n 10% of pat i ent s wi t h t hi s condi t i on.
(h) T ylosis–hyperkeratosis of the palms and soles. More t han 80% of pat i ent s wi t h t hi s aut os omal domi nant condi t i on devel op s quamous cel l carci noma of t he es ophagus .
(i) Celiac sprue
(j) Esophageal dysplasia as s oci at ed wi t h human papi l l oma vi rus (HPV)
(3) Clinical features
(a) Progressive dysphagia for solids i ndi cat es t he pres ence of an ongoi ng obs t ruct i ve l es i on. Us ual l y, when t he es ophageal l umen narrows t o 1.2 cm or l es s , a persistent dysphagia for s ol i d food i s obs erved.
(b) Pain us ual l y s i gni fi es ext ens i on of
Pa g e 8 9 1
ABC Ambe r CHM Conve rte r Tria l ve rsion, http://w w w .proce sste x t.com/a bcchm.html
t he t umor beyond t he wal l of t he es ophagus .
(c) Dysphagia for liquids, cough, hoarseness, and weight loss general l y are s ympt oms of advanced es ophageal carci noma.
(4) Diagnosis
(a) Endoscopy with biopsy and cytologic study, i f performed t oget her, es t abl i s hes a di agnos i s i n approxi mat el y 90% of cas es and i s t he di agnos t i c procedure of choi ce.
(b) Endoscopic ultrasound (EUS) i s us eful for s t agi ng es ophageal carci nomas .
(c) Computed tomography (CT ) and bronchoscopy s houl d be us ed t o eval uat e t he pres ence and ext ent of nodal met as t as es and bronchi al i nvas i on.
(d) Barium radiograph wi t h a bari um-coat ed bol us (e.g., bread or a mars hmal l ow) s houl d be performed when obs t ruct i ve dys phagi a i s s us pect ed.
Pa g e 8 9 2
ABC Ambe r CHM Conve rte r Tria l ve rsion, http://w w w .proce sste x t.com/a bcchm.html
(5) T herapy (s ee Chapt er 4 VII D 9)
o
o
b. Benign esophageal stricture may be a s equel a of prol onged refl ux es ophagi t i s . Heart burn may l es s en as s ol i d-food dys phagi a wors ens wi t h progres s i on of t he s t ri ct ure. Di agnos i s i s es t abl i s hed by a bol us bari um s wal l ow and endos copy. Treat ment general l y i s wi t h t apered bougi es or wi t h bal l oon di l at at i on cat het ers .
o
o
c. Esophageal webs s een i n t he upper one-third of t he es ophagus may be caus ed by a fai l ure of compl et e embryol ogi c recanal i zat i on. W ebs i n t hi s area al s o may be as s oci at ed wi t h i ron defi ci ency anemi a i n t he Plummer-Vinson (Paterson-Kelly) syndrome. Effect i ve t reat ment of P.188
t hi s s yndrome i ncl udes admi ni s t eri ng i ron for t he anemi a and us i ng an es ophageal bougi e t o fract ure t he webs . o
o
d. Esophageal rings mos t commonl y occur at t he squamocolumnar junction and are cal l ed Schatzki' s rings. Dys phagi a for s ol i ds oft en i s i nt ermi t t ent i n t hi s condi t i on, es peci al l y i f t he narrowes t poi nt of t he es ophagus meas ures bet ween 1.2 and 2 cm. Es ophageal bougi enage oft en i s effect i ve t herapy.
3. Esophageal motor disorders o
Pa g e 8 9 3
ABC Ambe r CHM Conve rte r Tria l ve rsion, http://w w w .proce sste x t.com/a bcchm.html
a. Oropharyngeal dysphagia (transfer dysphagia) i s a des cri pt i ve t erm appl i ed t o a di s order of t he neuromus cul ar apparat us of t he di s t al pharynx and upper es ophagus . Sympt oms i ncl ude di ffi cul t y i n i ni t i at i ng s wal l owi ng, nas al regurgi t at i on, and cough wi t h pul monary as pi rat i on.
See onl i ne for t ypes di s orders
as s oci at ed wi t h oropharyngeal dys phagi a
(1) Cerebrovascular accident (CVA), whi ch may be t he mos t common of t hes e di s orders and us ual l y i s at t ri but abl e t o t rans i ent brai ns t em edema
(2) Myasthenia gravis
(3) Myotonic dystrophy
(4) Polymyositis
(5) Bulbar poliomyelitis
(6) Parkinson' s disease
(7) Multiple sclerosis (MS)
(8) Amyotrophic lateral sclerosis
(9) Hypothyroidism
o
Pa g e 8 9 4
ABC Ambe r CHM Conve rte r Tria l ve rsion, http://w w w .proce sste x t.com/a bcchm.html o
b. SDES
(1) Pathology. LES abnormal i t i es (s i mi l ar t o achal as i a), wi t h i ncompl et e rel axat i on, have been des cri bed i n 30% of cas es , wi t h document ed progres s i on t o cl as s i c achal as i a. Thi s l eads mos t i nves t i gat ors t o t hi nk t hat SDES res ul t s from a neural defect .
(2) Clinical features
(a) Dysphagia for bot h s ol i ds and l i qui ds occurs .
(b) Odynophagia may occur, es peci al l y aft er i nges t i on of ext remel y hot or col d s ol i ds or l i qui ds .
(c) Spontaneous chest pain s i mi l ar t o angi na pect ori s may be evi dent . Noct urnal pai n, whi ch i s oft en des cri bed, i s rel i eved by t he s moot h mus cl e rel axant ni t rogl yceri n (t he s ame t reat ment pres cri bed for angi nal pai n). The abi l i t y of ni t rogl yceri n t o provi de rel i ef may furt her confus e t he di agnos i s .
(3) Diagnosis. Cl i ni cal evi dence i s t he bas i s of di agnos i s .
Pa g e 8 9 5
ABC Ambe r CHM Conve rte r Tria l ve rsion, http://w w w .proce sste x t.com/a bcchm.html
(a) Radiography may reveal a corks crew-s haped es ophagus .
(b) Esophageal manometry s houl d reveal :
(i) Hi gh-ampl i t ude, repet i t i ve, s i mul t aneous cont ract i ons i n approxi mat el y 30% of t he bas al s t at e or aft er ergonovi ne or edrophoni um s t i mul at i on (not a rout i nel y performed t es t ). (In a vari ant of SDES cal l ed t he nutcracker esophagus, hi gh-ampl i t ude prol onged cont ract i ons may be as s oci at ed wi t h ches t pai n.)
(ii) Several normal peri s t al t i c s equences t o di fferent i at e SDES from achal as i a
(iii) Incompl et e rel axat i on of t he LES i n approxi mat el y 30% of pat i ent s
(c) Balloon distention of the esophageal lumen can be us ed as a s t i mul us . A s mal l er-t han-normal amount of ai r or wat er i ns uffl at i on of t he bal l oon i ndi cat es a l ower t hres hol d for pai n.
Pa g e 8 9 6
ABC Ambe r CHM Conve rte r Tria l ve rsion, http://w w w .proce sste x t.com/a bcchm.html
(4) T herapy (s ucces s ful i n approxi mat el y 50% of cas es )
(a) Ant i chol i nergi c agent s
(b) Ni t rat es (s hort - or l ong-act i ng)
(c) Cal ci um channel –bl ocki ng agent s
(d) Es ophageal bougi enage
(e) Hydral azi ne t o decreas e peri s t al t i c ampl i t ude
(f) Surgery wi t h a l ongi t udi nal es ophageal myot omy (i n s evere, i ncapaci t at i ng cas es onl y)
o
o
c. Achalasia
(1) Epidemiology. Achal as i a occurs i n approxi mat el y 1 i n 100,000 i ndi vi dual s i n t he Uni t ed St at es and equal l y i n bot h s exes . The mos t common age of ons et i s bet ween 20 and 40 years .
(2) Pathology. A neural defect i s s ugges t ed by decreas ed gangl i on cel l s , wi t h fi bros i s and
Pa g e 8 9 7
ABC Ambe r CHM Conve rte r Tria l ve rsion, http://w w w .proce sste x t.com/a bcchm.html
s carri ng i n Auerbach's pl exus . Wallerian degeneration i s s ugges t ed by exami nat i on of vagal es ophageal fi bers . The dors al vagal nucl eus i s al s o abnormal . Supers ens i t i vi t y t o chol i nergi c s t i mul at i on and exogenous gas t ri n i s des cri bed.
(3) Clinical features
(a) Dys phagi a for s ol i ds and l i qui ds i n 95%–100% of pat i ent s
(b) W ei ght l os s i n 90% of pat i ent s
(c) Ches t pai n, whi ch i s s evere i n 60% of pat i ent s
(d) Noct urnal cough i n 30% of pat i ent s , i ndi cat i ng pos s i bl e overfl ow as pi rat i on of unempt i ed es ophageal cont ent s and, i n s uch cas es , t he need for i mmedi at e t reat ment
(e) Recurrent bronchi t i s or pneumoni a i n approxi mat el y 7%–8% of pat i ent s
P.189
(4) Diagnosis. Di agnos i s i nvol ves excl udi ng
Pa g e 8 9 8
ABC Ambe r CHM Conve rte r Tria l ve rsion, http://w w w .proce sste x t.com/a bcchm.html
mal i gnancy (i .e., carci noma or l ymphoma) at t he es ophagogas t ri c junct i on, whi ch may mi mi c achal as i a (“s econdary achal as i a― ). Secondary achal as i a may be charact eri zed by great er wei ght l os s , s hort er durat i on of s ympt oms , and l es s es ophageal di l at at i on i n an ol der pers on.
(a) Radiography may reveal a fl acci d, di l at ed, fl ui d-fi l l ed es ophagus wi t h a beak-like tapering over t he LES regi on. A CT s can of t he ches t and abdomen or an endos copi c ul t ras ound of t he es ophagogas t ri c junct i on may be needed t o rul e out s econdary (oft en mal i gnant ) achal as i a.
(b) Manometry i s t he mos t s ens i t i ve di agnos t i c met hod and s houl d reveal :
(i) Abs ence of normal peri s t al s i s i n t he ent i re es ophagus
(ii) El evat ed LES pres s ure
(iii) Incompl et e rel axat i on of t he LES, res ul t i ng i n a pers i s t ent obs t ruct i ng barri er aft er s wal l owi ng
(5) T herapy
Pa g e 8 9 9
ABC Ambe r CHM Conve rte r Tria l ve rsion, http://w w w .proce sste x t.com/a bcchm.html
(a) Drugs s uch as ni t rat es , ant i chol i nergi c agent s , β 2 -adrenergi c agoni s t s , and cal ci um channel -bl ocki ng agent s are effect i ve i n l es s t han 50% of pat i ent s .
(b) Pneumatic dilation i s effect i ve i n 70%–90% of pat i ent s , has a mort al i t y rat e of approxi mat el y 0.2%, and has a perforat i on rat e of roughl y 2%–3%.
(c) Endoscopic injection of botulinum toxin i nt o t he LES res ul t s i n decreas ed LES pres s ure and decreas ed dys phagi a i n 70%–80% of pat i ent s . Repeat ed i nject i ons may be neces s ary, and l ong-t erm effi cacy i s not ed i n l es s t han 50% of pat i ent s .
(d) Surgical therapy. The favored procedure i s a Heller myotomy, whi ch has a 65%–90% s ucces s rat e and a 3%–4% s urgi cal compl i cat i on rat e. For operat i ons t hat do not i ncorporat e an ant i refl ux procedure, t he rat e of pos t operat i ve refl ux may i ncreas e t o 25%–30% aft er s everal years . Laparos copi c t echni ques are favored.
o
o
d. Systemic sclerosis (scleroderma) i s a s ys t emi c col l agen vas cul ar di s eas e i nvol vi ng t he s ki n i n 98% of cas es . The es ophagus i s found t o be abnormal i n
Pa g e 9 0 0
ABC Ambe r CHM Conve rte r Tria l ve rsion, http://w w w .proce sste x t.com/a bcchm.html
75% of aut ops i es and i n 80% of cas es s t udi ed manomet ri cal l y.
(1) Pathology. The earl y es ophageal effect s are t hought t o be neural becaus e no anat omi c abnormal i t y of t he s moot h mus cl e can be i dent i fi ed at a t i me when marked weaknes s of t he es ophagus i s not ed. A l at e defect may i ncl ude a di s us e t ype of at rophy of t he ci rcul ar s moot h mus cl e el ement s of t he es ophagus ; t he l ongi t udi nal mus cul ar l ayers remai n i nt act .
(2) Clinical features
(a) Dys phagi a for s ol i ds and l i qui ds
(b) Severe heart burn i n 50% of pat i ent s
(c) Es ophageal s t ri ct ure i n 25% of l ong-t erm s urvi vors
(3) Diagnosis
(a) Radiography. Supi ne es ophagography may i ndi cat e poor es ophageal empt yi ng becaus e of an abs ence of peri s t al s i s .
(b) Manometry. Thi s procedure, whi ch i s t he mos t rel i abl e di agnos t i c t echni que,
Pa g e 9 0 1
ABC Ambe r CHM Conve rte r Tria l ve rsion, http://w w w .proce sste x t.com/a bcchm.html
reveal s :
(i) Decreas ed LES pres s ure
(ii) Very weak, l ow-ampl i t ude peri s t al t i c cont ract i ons i n t he di s t al s moot h mus cl e port i on (l ower t wo-t hi rds ) of t he es ophagus
(4) T herapy. The es ophageal di s order i s t reat ed wi t h ant i refl ux meas ures (s ee I B 1 d).
4. Other esophageal disorders o
o
a. Diverticula
(1) Zenker' s diverticulum i s a mucos al herni at i on (not a t rue di vert i cul um) above t he cri copharyngeal regi on. Obs t ruct i ve s ympt oms may occur i f t here i s i ncompl et e empt yi ng of t hi s di vert i cul um. Large di vert i cul a are t reat ed s urgi cal l y.
(2) T raction diverticula occur i n t he mi d and di s t al regi ons and are t hought t o be s econdary t o an adjacent i nfl ammat ory proces s s uch as t ubercul os i s .
(3) Epiphrenic diverticula occur i n t he di s t al
Pa g e 9 0 2
ABC Ambe r CHM Conve rte r Tria l ve rsion, http://w w w .proce sste x t.com/a bcchm.html
es ophagus , above t he LES, and oft en are as ympt omat i c. o
o
b. Infections. Bact eri al and vi ral s ources of es ophageal i nfect i on are common, but s everal i nfect i ous agent s are of part i cul ar i nt eres t . P.190
(1) Candidal esophagitis us ual l y occurs i n di abet i c pat i ent s , i mmunocompromi s ed hos t s (e.g., pat i ent s i nfect ed wi t h human i mmunodefi ci ency vi rus [HIV] or pat i ent s undergoi ng cancer chemot herapy or s t eroi d t reat ment ), and i n t hos e wi t h poor es ophageal empt yi ng (e.g., pat i ent s wi t h achal as i a or s evere s t ri ct ure). Odynophagi a i s a major s ympt om, and di agnos i s i s made by endos copy and cyt ol ogi c s t udi es . Treat ment i s wi t h nys t at i n, fl uconazol e, or, i n res i s t ant cas es , l ow dos es of amphot eri ci n B.
(2) Herpes simplex virus (HSV) may caus e es ophagi t i s i n i mmunocompromi s ed hos t s . The es ophagi t i s i s charact eri zed by rel at i vel y s mal l , i s ol at ed ul cers . Bi ops y of t he ul cerat i ng edges may s how charact eri s t i c mul t i nucl eat ed cel l s wi t h nucl ear i ncl us i ons . Treat ment i s wi t h acycl ovi r.
(3) CMV i nfect i on of t he es ophagus i s oft en
Pa g e 9 0 3
ABC Ambe r CHM Conve rte r Tria l ve rsion, http://w w w .proce sste x t.com/a bcchm.html
s een i n i mmunocompromi s ed pat i ent s and may produce very l arge ul cerat i ons of t he es ophagus . Int ranucl ear i ncl us i on bodi es are obs erved on bi ops y. Treat ment i s wi t h ganci cl ovi r.
(4) HIV esophagitis may res ul t i n a di ffus e i nfl ammat ory es ophagi t i s . Treat ment of i s ol at ed acqui red i mmunodefi ci ency s yndrome (AIDS) es ophagi t i s i s wi t h s t eroi ds .
o
o
c. Esophageal burns. Inges t i on of caus t i c agent s (i .e., s t rong al kal i or aci d) can caus e s eri ous es ophageal burns . Inges t i on of l ye or det ergent s s uch as chl ori ne bl each i s a common s ui ci dal ges t ure i n adul t s and a common acci dent i n chi l dren. Emergency endos copy s houl d be performed t o as s es s t he ext ent of damage. St eroi ds and broad-s pect rum ant i bi ot i cs are recommended i ni t i al l y i n management of es ophageal burns . Long-t erm s equel ae i n s urvi vors may i ncl ude es ophageal s t ri ct ure and es ophageal carci noma.
o
o
d. Esophageal tears. Thes e condi t i ons are s een mos t commonl y aft er vomi t i ng (75% of cas es ), s t rai ni ng, or coughi ng.
(1) A mucos al t ear (Mallory-Weiss syndrome) produces s i gni fi cant hemat emes i s aft er an i ni t i al nonbl oody vomi t us . Surgery i s requi red i n l es s t han 10% of t hes e cas es .
Pa g e 9 0 4
ABC Ambe r CHM Conve rte r Tria l ve rsion, http://w w w .proce sste x t.com/a bcchm.html
(2) A rupt ure of t he es ophagus (Boerhaave' s syndrome) us ual l y occurs above t he es ophagogas t ri c junct i on. Ai r i n t he l eft medi as t i nal regi on s ugges t s t he di agnos i s , and i mmedi at e s urgi cal i nt ervent i on i s neces s ary i f t he pat i ent i s t o s urvi ve.
o
o
e. Paraesophageal hernia. Unl i ke t he much more common and cl i ni cal l y i ns i gni fi cant hi at al herni a, a paraes ophageal herni a may l ead t o gas t ri c vas cul ar compromi s e. The es ophagogas t ri c junct i on t ravers es t he di aphragm i n t he appropri at e l ocat i on. The body of t he s t omach t hen t ravel s above t he di aphragm, and gas t ri c vol vul us wi t h i ncarcerat i on may occur. Surgery may be neces s ary t o al l evi at e s ympt oms of pai n, upper gas t roi nt es t i nal bl eedi ng, and i s chemi a.
o
o
f. Pill-induced esophagitis. Thi s condi t i on i s oft en caus ed by medi cat i ons s uch as oral bisphosphonates, as pi ri n and ot her nons t eroi dal ant i -i nfl ammat ory drugs (NSAIDs ), pot as s i um chl ori de t abl et s , i ron preparat i ons , qui ni di ne, t et racycl i nes , and ot her ant i bi ot i cs . Thes e medi cat i ons are oft en t emporari l y l odged i n t he es ophagus becaus e of ei t her i nadequat e l i qui d wi t h s wal l owi ng or a rel at i ve narrowi ng of t he es ophagus . Severe eros i ons and s t ri ct ures may devel op i n a mi nori t y of cas es . Treat ment i s s ympt omat i c.
II. Diseases of the Stomach A. Gastritis
Pa g e 9 0 5
ABC Ambe r CHM Conve rte r Tria l ve rsion, http://w w w .proce sste x t.com/a bcchm.html
1. Acute gastritis i s an i nfl ammat i on of t he gas t ri c mucos a, whi ch may be di ffus e or l ocal i zed and us ual l y i s s el f-l i mi t ed. o
o
a. Etiology
(1) Drugs t hat can damage t he mucos al barri er and l ead t o back-di ffus i on of aci d and peps i n i ncl ude:
(a) As pi ri n and NSAIDs
(b) Al cohol , whi ch may produce an addi t i ve effect wi t h as pi ri n
(2) Accidental ingestion of caustic substances s uch as s t rong al kal i (e.g., l ye), s t rong aci d [e.g., s ul furi c aci d (H 2 SO 4 ), HCl ], or fi xat i ves [e.g., formal dehyde, t ri ni t rophenol (pi cri c aci d)] can be fat al . Pat i ent s who s urvi ve t he i nges t i on of s uch corros i ves s us t ai n i njuri es t hat l eave cons i derabl e s cars and s ubs equent ant ral narrowi ng.
(3) Stress rel at ed t o s evere i l l nes s , es peci al l y i l l nes s i nvol vi ng many organ s ys t ems , caus es acut e gas t ri t i s . Is chemi a and gas t ri c aci d, even at normal l evel s , may be i nvol ved.
Pa g e 9 0 6
ABC Ambe r CHM Conve rte r Tria l ve rsion, http://w w w .proce sste x t.com/a bcchm.html
P.191
(4) Infections
(a) Helicobacter pylori i nfect i ous gas t ri t i s (s ee III C 4)
(b) Infections with CMV, herpesvirus, Mycobacterium avium complex (MAC), Candida, T reponema pallidum, and Mycobacterium tuberculosis have been as s oci at ed wi t h gas t ri t i s , es peci al l y i n i mmunocompromi s ed pat i ent s .
o
o
b. Clinical features (pres ent i n 70% of pat i ent s )
(1) Epigastric burning and pain, nausea, and vomiting
(2) Gastrointestinal bleeding, whi ch may be s evere and as s oci at ed wi t h hemat emes i s and s hock
o
o
c. Diagnosis. In mos t cas es , di agnos i s i s made on t he bas i s of endos copi c vi s ual i zat i on wi t h or wi t hout bi ops y. Conges t i on, fri abi l i t y, s uperfi ci al ul cerat i on, and pet echi ae frequent l y are s een i n t he gas t ri c mucos a.
Pa g e 9 0 7
ABC Ambe r CHM Conve rte r Tria l ve rsion, http://w w w .proce sste x t.com/a bcchm.html o
o
d. T herapy. Treat ment begi ns wi t h removal of offendi ng agent s . Ant aci ds , ant i vi ral and ant i fungal agent s , H 2 -recept or ant agoni s t s , prot on pump i nhi bi t ors , and s urface-act i ng agent s (e.g., s ucral fat e) are us eful as wel l . Pat i ent s wi t h acut e hemorrhagi c gas t ri t i s us ual l y res pond t o fl ui d or bl ood repl acement combi ned wi t h a regi men of ant aci ds , H 2 -recept or ant agoni s t s , and omepraz ol e or l ans opraz ol e, whi ch keep t he gas t ri c pH above 3.5. Surgery rarel y i s neces s ary for t hes e pat i ent s and i s as s oci at ed wi t h hi gh morbi di t y and mort al i t y rat es .
2. Chronic gastritis i s charact eri zed by a s uperfi ci al l ymphocyt e i nfi l t rat e i n t he l ami na propri a. o
o
a. Etiology. Chroni c gas t ri t i s can be caus ed by:
(1) Prol onged us e of al cohol , as pi ri n, and ot her i rri t at i ng drugs
(2) Radi at i on or t hermal i njury
(3) Immunol ogi c fact ors
(4) Infect i ons (e.g., H. pyl ori ) H. pylori i nfect i ous gas t ri t i s i s caus ed by a gram-negat i ve, s pi ral -s haped bact eri um t hat s urvi ves i n t he aci di c mi l i eu of t he s t omach by
Pa g e 9 0 8
ABC Ambe r CHM Conve rte r Tria l ve rsion, http://w w w .proce sste x t.com/a bcchm.html
produci ng ureas e, whi ch l i berat es ammoni a. Chroni c gas t ri t i s i nvol vi ng H. pyl ori has been as s oci at ed wi t h 80%–90% of duodenal ul cer pat i ent s (s ee III C 4) and wi t h 60%–70% of gas t ri c ul cer pat i ent s . o
o
b. T ypes
(1) Chronic type A gastritis i nvol ves t he fundus and body of t he s t omach; t he ant rum i s s pared. Thi s t ype of gas t ri t i s i s as s oci at ed wi t h pari et al cel l ant i bodi es , hi gh s erum gas t ri n l evel s , and perni ci ous anemi a.
(2) Chronic type B gastritis i nvol ves t he ant rum of t he s t omach; t he body and fundus are rel at i vel y s pared. Gas t ri n cel l ant i bodi es have been det ect ed i n s ome pat i ent s wi t h t hi s gas t ri t i s . More commonl y, refl ux of duodenal or bi l i ary s ecret i ons or H. pyl ori i nfect i ons are l i nked caus at i vel y t o t ype B gas t ri t i s .
o
o
c. Clinical features. Cl i ni cal evi dence may be l i mi t ed i n pat i ent s wi t h chroni c gas t ri t i s . Type A gas t ri t i s i s as s oci at ed wi t h hypochl orhydri a or achl orhydri a, whereas t ype B gas t ri t i s i s as s oci at ed wi t h normal aci d l evel s . Hypot hyroi di s m, di abet es mel l i t us , and vi t i l i go occur more frequent l y wi t h t ype A t han wi t h t ype B gas t ri t i s .
o
o
d. Clinical course. Dat a s ugges t t hat t hes e l es i ons may remai n unchanged for s everal years . Gas t ri c
Pa g e 9 0 9
ABC Ambe r CHM Conve rte r Tria l ve rsion, http://w w w .proce sste x t.com/a bcchm.html
at rophy devel ops i n approxi mat el y 50% of pat i ent s wi t h s uperfi ci al gas t ri t i s over 10–20 years . There i s an i ncreas ed as s oci at i on wi t h gas t ri c pol yps , gas t ri c ul cer, and gas t ri c cancer i n bot h t ypes of chroni c gas t ri t i s , wi t h t ype B bei ng as s oci at ed wi t h a hi gher i nci dence of gas t ri c cancer t han t ype A. o
o
e. T herapy. Treat ment us ual l y i s unneces s ary; however, condi t i ons as s oci at ed wi t h gas t ri t i s (e.g., perni ci ous anemi a, H. pyl ori i nfect i ons , hypot hyroi di s m, and di abet es ) s houl d be t reat ed accordi ngl y. Some i nves t i gat ors s ugges t yearl y gas t ri c cyt ol ogi c anal ys i s as a means of di agnos i ng an earl y cancer i n affect ed pat i ent s .
3. Special types of gastritis o
o
a. Eosinophilic gastroenteritis refers t o t he i nfi l t rat i on of eos i nophi l s i nt o t he gas t ri c ant rum, s mal l bowel , or bot h. Thi s i nfi l t rat i on, whi ch i s bel i eved t o be i mmunol ogi cal l y medi at ed, caus es t hi ckeni ng of t he i nt es t i nal wal l wi t h s ubs equent ant ral obs t ruct i on. Peripheral edema due to protein-losing enteropathy may occur. Peri pheral eos i nophi l i a is common. Di fferent i at i on from ot her i nfi l t rat i ve di s eas es (e.g., t ubercul os i s , s arcoi dos i s , l ymphoma, s yphi l i s , hi s t opl as mos i s , Crohn's di s eas e, and carci noma) may be di ffi cul t and depends on endos copi c bi ops y. Cort i cos t eroi d t herapy has been s ucces s ful i n provi di ng prol onged remi s s i on.
o
Pa g e 9 1 0
ABC Ambe r CHM Conve rte r Tria l ve rsion, http://w w w .proce sste x t.com/a bcchm.html o
b. Granulomatous gastritis i s an i nfi l t rat i ve di s eas e charact eri zed by noncas eat i ng granul omas , wi t h or wi t hout gi ant cel l s . Crohn's di s eas e, s arcoi dos i s , beryl l i um poi s oni ng, or i di opat hi c caus es may res ul t i n granul omat ous gas t ri t i s . Gas t ri c out l et obs t ruct i on i s common. Treat ment wi t h s t eroi ds or s urgery i s oft en requi red.
o
o
c. Hypertrophic gastritis i s an uncommon condi t i on as s oci at ed wi t h mas s i ve enl argement of t he gas t ri c fol ds .
(1) Clinical features. In i t s mos t ext reme form (i .e., Ménét ri er's di s eas e), t here i s hypos ecret i on of gas t ri c aci d, prot ei n l os s from t he s t omach, peri pheral edema, wei ght l os s , and abdomi nal pai n.
(2) Diagnosis. Endos copy and bi ops y, oft en wi t h a s uct i on apparat us , provi de t he di agnos i s .
(a) On bi ops y, gas t ri c mucous cel l s are hyperpl as t i c, and i nfl ammat ory cel l s are pres ent i n s ome pat i ent s .
(b) Lymphoma, amyl oi d i nfi l t rat i on, carci noma, and Zol l i nger-El l i s on s yndrome al s o can caus e l arge rugal fol ds and s houl d be excl uded.
Pa g e 9 1 1
ABC Ambe r CHM Conve rte r Tria l ve rsion, http://w w w .proce sste x t.com/a bcchm.html
(c) A t ype of hypers ecret ory hypert rophi c gas t ri t i s i s s i mi l ar t o Ménét ri er's di s eas e but i s as s oci at ed wi t h hi gh aci d out put and hyperpl as i a of pari et al gas t ri c cel l s .
(3) T herapy. Treat ment i ncl udes ant i chol i nergi c agent s , whi ch appear t o cl os e t he t i ght junct i ons bet ween cel l s and decreas e prot ei n l os s ; H 2 -recept or ant agoni s t s ; and s t eroi ds . Surgery (gas t ri c res ect i on) i s res erved for i nt ract abl e cas es .
B. Gastric neoplasms (see Chapter 4, “Oncology―)
1. T rue gastric lymphoma us ual l y occurs as a bul ky mas s as s oci at ed wi t h l arge, t hi ckened gas t ri c fol ds . Pai n i s t he mos t common pres ent i ng s ympt om. Hi s t ol ogi c exami nat i on us ual l y s hows di ffus e hi s t i ocyt i c non-Hodgki n's l ymphoma, but non-Hodgki n's l ymphomas of al l t ypes are more common t han Hodgki n's di s eas e i n t he s t omach. Gas t ri c l ymphoma, es peci al l y of t he P.192
mucos a-as s oci at ed l ymphat i c t i s s ue (MALT) t ype, has been s t rongl y as s oci at ed wi t h H. pyl ori i nfect i on. o
o
a. Diagnosis. Gas t ri c l ymphoma s houl d be di fferent i at ed from Ménét ri er's di s eas e, Zol l i nger-El l i s on s yndrome, and hypert rophi c gas t ri t i s . Di agnos i s i s confi rmed by bi ops y, ei t her
Pa g e 9 1 2
ABC Ambe r CHM Conve rte r Tria l ve rsion, http://w w w .proce sste x t.com/a bcchm.html
at endos copy wi t h a s uct i on apparat us or at s urgery. o
o
b. T herapy. Surgery and l ocal i zed radi at i on t herapy are t he general l y accept ed forms of t herapy, wi t h 5-year s urvi val rat es approachi ng 50% i n non-Hodgki n's l ymphoma pat i ent s whos e l ymphoma i s confi ned t o t he s t omach. Chemot herapy i s val uabl e i n pat i ent s wi t h s ys t emi c di s eas e. If H. pyl ori l ymphoma i s i dent i fi ed, t reat ment for t he H. pyl ori al one may l ead t o res ol ut i on.
2. Other gastric tumors o
o
a. Gastrointestinal stromal tumors (GIST S). The s t omach i s t he mos t common s i t e for GISTS, wi t h t he s mal l bowel bei ng t he s econd mos t frequent l ocat i on. GIST t umors cons t i t ut e 1%–3% of al l gas t ri c t umors and devel op from t he i nt ers t i t i al cel l s of Cajal (or pacemaker cel l s ). They may be beni gn or mal i gnant and pres ent wi t h GI bl eedi ng (40%), abdomi nal mas s (40%), or abdomi nal pai n (20%). Mal i gnancy i s oft en det ermi ned by t he s i ze (>4 cm) or t he number (>25) of mi t ot i c fi gures s een i n a hi gh-power fi el d (HPF). Treat ment i s wi t h s urgery or a t yros i ne ki nas e i nhi bi t or s uch as i mat i ni b mes yl at e (Gl eevec) or s uni t i ni b (Sut ent ).
o
o
b. Gastric malignancies s uch as fibrosarcomas, neurogenic sarcomas, carcinoids, and metastatic carcinomas (es peci al l y from breas t and l ung
Pa g e 9 1 3
ABC Ambe r CHM Conve rte r Tria l ve rsion, http://w w w .proce sste x t.com/a bcchm.html
carci nomas and mel anoma) mus t be di fferent i at ed from pri mary gas t ri c carci noma.
C. Disorders of gastric emptying
1. Pyloric stenosis o
o
a. Acquired pyloric stenosis occurs t rans i ent l y, as t he res ul t of edema from pept i c ul cer di s eas e, or chroni cal l y, as t he res ul t of pyl ori c s carri ng from recurrent ul cer di s eas e or neopl as m.
o
o
b. Congenital pyloric stenosis us ual l y mani fes t s i n i nfancy.
(1) Incidence. Congeni t al pyl ori c s t enos i s occurs i n approxi mat el y 2–4 of 1000 bi rt hs and us ual l y i s s een i n fi rs t born mal e chi l dren. There i s a fami l i al i nci dence.
(2) Clinical features. Pos t prandi al project i l e vomi t i ng of nonbi l i ous mat eri al , dehydrat i on, and wei ght l os s may occur. Vi s i bl e peri s t al s i s may be not ed, wi t h a mas s pal pat ed i n t he epi gas t ri um.
(3) Diagnosis. Radi ographi c evi dence i s t he bas i s of di agnos i s . A pl ai n fi l m s hows ai r i n t he s t omach, and a bari um s wal l ow confi rms t he di agnos i s .
Pa g e 9 1 4
ABC Ambe r CHM Conve rte r Tria l ve rsion, http://w w w .proce sste x t.com/a bcchm.html
(4) T herapy. Treat ment i s s urgi cal (Rams t edt operat i on) and cons i s t s of a myot omy of t he ci rcul ar mus cl e of t he pyl orus .
o
o
2. Gastric bezoars are col l ect i ons of nondi ges t i bl e s ubs t ances t hat s omet i mes form and cannot pas s t hrough t he pyl orus . Tri chobezoars are compos ed of hai r, and phyt obez oars are compos ed of pl ant fi bers . Bez oars general l y are s een i n pat i ent s who have undergone previ ous gas t ri c s urgery or i n ment al l y ret arded i ndi vi dual s who cons ume nondi ges t i bl e s ubs t ances . The s ympt oms i ncl ude t hos e of gas t ri c out l et obs t ruct i on and bl eedi ng from s uperfi ci al ul cerat i ons . It i s i mport ant t o excl ude a gas t ri c mas s or cancer, and t he di agnos i s can be es t abl i s hed endos copi cal l y. Bez oars s omet i mes can be enzymat i cal l y di s s ol ved wi t h papai n, acet yl cys t ei ne, or cel l ul as e; ot herwi s e, endos copi c or s urgi cal removal i s neces s ary.
o
o
3. Gastric diverticula occur on t he pos t eri or wal l of t he s t omach i n approxi mat el y 75% of cas es and us ual l y are wi t hi n 2 cm of t he es ophagogas t ri c junct i on. Unl es s t hey bl eed or perforat e t hes e congeni t al l es i ons are as ympt omat i c. Pseudodiverticula, s een mos t commonl y i n t he ant rum, are s carred remnant s of previ ous pept i c di s eas e.
o
o
4. Gastric volvulus may occur as a res ul t of weak l i gament ous at t achment s or may be s econdary t o a paraes ophageal herni a, an i nt ri ns i c gas t ri c l es i on,
Pa g e 9 1 5
ABC Ambe r CHM Conve rte r Tria l ve rsion, http://w w w .proce sste x t.com/a bcchm.html
or an adjacent mas s . Di agnos i s i s s upport ed by t he fi ndi ng of t wo s eparat e fl ui d l evel s i n t he l eft upper quadrant and by a l ack of bari um pas s age i nt o t he pyl orus . Therapy cons i s t s of t emporary nas ogas t ri c s uct i on. Recurrent or acut e vol vul us wi t h gas t ri c vas cul ar compromi s e may requi re s urgery. o
o
5. Gastroparesis i s a di s order of gas t ri c empt yi ng t hat i s not caus ed by an obs t ruct i on. The di agnos i s s houl d be made by a nucl ear s ol i d-phas e gas t ri c empt yi ng s t udy aft er mechani cal obs t ruct i on has been rul ed out by an upper gas t roi nt es t i nal s eri es or by endos copy. Gas t ropares i s mos t frequent l y i s caus ed by t ype 1 (i ns ul i n-dependent ) di abet es mel l i t us of l onger t han 10 years ' durat i on. Ot her condi t i ons as s oci at ed wi t h gas t ropares i s i ncl ude s ys t emi c s cl eros i s , pos t vagot omy s t at es , and t herapy wi t h ant i chol i nergi c agent s or narcot i cs . In di abet es , l os s of gas t ri c phas e III act i vi t y i s not ed on el ect ri cal recordi ngs , wi t h ot her s i gns of di abet i c vi s ceral neuropat hy oft en bei ng s een as wel l . Treat ment wi t h proki net i c agent s s uch as met ocl oprami de, domperi done, or eryt hromyci n has been effect i ve. El ect ri cal gas t ri c paci ng may be neces s ary i n res i s t ant cas es .
III. Peptic Ulcer Disease A. Introduction Pept i c ul cer di s eas e refers t o a group of di s orders of t he gas t roi nt es t i nal t ract . The di s orders al l i nvol ve areas of di s cret e t i s s ue des t ruct i on caus ed by aci d and peps i n. Pept i c ul cers occur mos t commonl y i n t he s t omach or proxi mal duodenum, l es s commonl y i n t he di s t al es ophagus , and rarel y i n t he s mal l i nt es t i ne. (Pept i c ul cers i n t he di s t al s mal l i nt es t i ne us ual l y are
Pa g e 9 1 6
ABC Ambe r CHM Conve rte r Tria l ve rsion, http://w w w .proce sste x t.com/a bcchm.html
as s oci at ed wi t h a Meckel 's di vert i cul um t hat cont ai ns gas t ri c mucos a.) In general , t he cl i ni cal feat ures and t reat ment of pept i c ul cer di s eas e are s i mi l ar regardl es s of l ocat i on, al t hough pept i c es ophagi t i s caus ed by refl ux of gas t ri c cont ent s has s ome uni que feat ures (s ee I B 1).
B. Incidence Pept i c ul cer di s eas e occurs more commonl y i n men t han i n women. Duodenal ul cers are t hree t i mes more common t han gas t ri c ul cers and occur approxi mat el y 10 years earl i er; t he peak i nci dence for duodenal ul cers i s at approxi mat el y 40 years of age, as oppos ed t o 50 years of age for gas t ri c ul cers . Duodenal ul cers have a 1-year rel aps e rat e of approxi mat el y 80% (much l ower i f as s oci at ed wi t h H. pyl ori t hat has been eradi cat ed). Pat i ent s wi t h gas t ri c ul cers have a 33% chance of devel opi ng s ubs equent duodenal ul cers .
C. Pathogenesis Aci d and peps i n are neces s ary for devel opment of ul cers . However, s everal fact ors , es peci al l y H. pyl ori i nfect i on, are t hought t o cont ri but e t o t he pat hogenes i s .
1. Social factors o
o
a. T obacco smoking i ncreas es t he ri s k of pept i c ul cer di s eas e. Smoki ng al s o rai s es t he morbi di t y and mort al i t y rat es and l owers t he heal i ng rat e for pept i c ul cers .
o
o
b. Drugs s uch as NSAIDs are i mpl i cat ed i n ul cer di s eas e, wi t h an ant i pros t agl andi n effect s ugges t ed as an underl yi ng fact or. Ul cers devel op i n approxi mat el y 30% of art hri t i s pat i ent s who t ake hi gh dos es of as pi ri n. St eroi ds al s o are t hought t o break t he mucos al barri er and may doubl e t he ri s k
Pa g e 9 1 7
ABC Ambe r CHM Conve rte r Tria l ve rsion, http://w w w .proce sste x t.com/a bcchm.html
of pept i c ul cer di s eas e. o
o
c. Alcohol compromi s es t he mucos al barri er and i ncreas es gas t ri c aci d s ecret i on.
2. Physiologic factors o
o
a. Gastric acid, al t hough es s ent i al for ul cer product i on, general l y i s meas ured at normal or decreas ed l evel s i n gas t ri c ul cer pat i ent s . Sl i ght l y el evat ed l evel s of gas t ri c aci d are not ed i n t he bas al and s t i mul at ed s t at es i n duodenal ul cer pat i ent s .
o
o
b. Serum gastrin l evel s are normal duri ng fas t i ng and i ncreas ed i n t he pos t prandi al s t at e i n duodenal ul cer pat i ent s . Bot h fas t i ng and pos t prandi al l evel s of s erum gas t ri n are hi gher t han normal i n gas t ri c ul cer pat i ent s .
P.193
3. Genetic factors o
o
a. First-degree relatives of gas t ri c ul cer pat i ent s have t hree t i mes t he ri s k of devel opi ng gas t ri c ul cers as t he general popul at i on. Si mi l arl y, t he ri s k of duodenal ul cer i s i ncreas ed i n t he fi rs t -degree rel at i ves of duodenal ul cer pat i ent s .
o
Pa g e 9 1 8
ABC Ambe r CHM Conve rte r Tria l ve rsion, http://w w w .proce sste x t.com/a bcchm.html
o
b. An i ncreas ed i nci dence of duodenal ul cer has been document ed among i ndi vi dual s wi t h bl ood group O, t hos e who demons t rat e el evat ed s erum l evel s of peps i nogen I, and t hos e who are nons ecret ors of bl ood group s ubs t ances .
4. Helicobacter pylori o
o
a. Microbiology. H. pyl ori has been i dent i fi ed on cul t ures of t he gas t ri c ant rum i n 90% of pat i ent s wi t h duodenal ul cer di s eas e or ant ral t ype B gas t ri t i s , and t he as s oci at i on for pat i ent s wi t h gas t ri c ul cer di s eas e i s 60%–70%.
o
o
b. Pathophysiology. H. pyl ori i s found on gas t ri c epi t hel i um and does not penet rat e t he cel l . If i t i s a pri mary offender, i t may act as a “barri er breaker,― al l owi ng aci d back-di ffus i on and pept i c ul cer di s eas e t o devel op. H. pyl ori el aborat es ammoni a, whi ch damages cel l s urfaces , and l i berat es a number of ot her i nfl ammat ory cel l recrui t i ng fact ors and adhes i on mol ecul es .
o
o
c. H. pylori detection. H. pyl ori l i berat es ureas e, and a bi ops y of t he gas t ri c ant rum may change a pH col or moni t or. A urea breat h t es t us i ng carbon 13
13 ( C)- or
14
C-l abel ed urea meas ures exhal ed
l abel ed carbon di oxi de. Suppl ement ed cul t ure medi um (s t arch, bl ood, or charcoal ) may be opt i mal under mi croaerophi l i c condi t i ons at 37°C. W art hi n-St arry s i l ver, Gi ems a, or hemat oxyl i n and eos i n s t ai ns may s how t he organi s m i n an
Pa g e 9 1 9
ABC Ambe r CHM Conve rte r Tria l ve rsion, http://w w w .proce sste x t.com/a bcchm.html
ext racel l ul ar l ocat i on. Ant i bodi es t o H. pyl ori det ect ed by enz yme-l i nked i mmunos orbent as s ay (ELISA) i ndi cat e act i ve or pri or i nfect i on. o
o
d. T reatment. Antibiotics are us ed t o t reat H. pyl ori i nfect i on. Regi mens t ypi cal l y i nvol ve combi nat i ons of an ant i bi ot i c (e.g., met roni dazol e, amoxi ci l l i n, t et racycl i ne preparat i ons , and cl ari t hromyci n), a bi s mut h-cont ai ni ng product , and a prot on pump i nhi bi t or (doubl e or t ri pl e t herapy). Treat ment i s for approxi mat el y 2 weeks , wi t h a 70%–90% res pons e rat e, dependi ng on t he regi men s el ect ed.
o
o
c. Outcome. St udi es have s hown a decreas ed rel aps e rat e i n duodenal ul cers t reat ed wi t h ant i bi ot i cs and bi s mut h as compared wi t h H 2 bl ockers .
5. Associated diseases. Some pat i ent s wi t h multiple endocrine neoplasia, type I (MEN I), pres ent wi t h gas t ri n-s ecret i ng t umors . Thi s probabl y account s for t he report ed as s oci at i on of duodenal ul cer di s eas e wi t h hyperparat hyroi di s m. Ot her di s eas es as s oci at ed wi t h gas t ri c ul cers i ncl ude ant ral at rophi c gas t ri t i s , rheumat oi d art hri t i s , chroni c obs t ruct i ve pul monary di s eas e (COPD), hepat i c ci rrhos i s , and chroni c renal fai l ure.
6. Psychosomatic factors i ncl ude chroni c anxi et y and “t ype A― pers onal i t y.
D. Clinical features Pa g e 9 2 0
ABC Ambe r CHM Conve rte r Tria l ve rsion, http://w w w .proce sste x t.com/a bcchm.html
1. Pain i s t he predomi nant s ympt om, al t hough i t may be abs ent i n 25% of gas t ri c ul cer pat i ent s . The pai n charact eri s t i cal l y i s des cri bed as an epi gas t ri c burni ng s ens at i on and may be accompani ed by bl oat i ng or naus ea. Eat i ng may exacerbat e t he pai n i n gas t ri c ul cer pat i ent s , whereas i n duodenal ul cer pat i ent s , t he pai n us ual l y i s di mi ni s hed by eat i ng, onl y t o recur 2–3 hours l at er. Pai n may awaken pat i ent s from s l eep, es peci al l y t hos e wi t h duodenal ul cers .
2. Upper gastrointestinal hemorrhage may be t he pres ent i ng s i gn of pept i c ul cer di s eas e, and anemi a from chroni c bl ood l os s may be s een.
3. Less common symptoms o
o
a. Repeated vomiting, whi ch may i ndi cat e gas t ri c out l et obs t ruct i on
o
o
b. Weight loss, whi ch i s s omewhat more common wi t h gas t ri c ul cer
E. Diagnosis Becaus e pat i ent s may compl ai n onl y of vague s ympt oms , a hi gh i ndex of s us pi ci on i s needed.
1. Radiography i s a us eful s creeni ng t ool ; however, an upper gas t roi nt es t i nal s eri es may mi s s up t o 30% of gas t ri c ul cers , and s carri ng of t he duodenal bul b from chroni c or recurrent ul cer di s eas e may make radi ographi c
Pa g e 9 2 1
ABC Ambe r CHM Conve rte r Tria l ve rsion, http://w w w .proce sste x t.com/a bcchm.html
i nt erpret at i on di ffi cul t . Doubl e-cont ras t t echni ques may i mprove P.194
di agnos t i c accuracy. Duodenal ulcers always are benign; however, gastric ulcers may be benign or malignant. Radi ographi c cri t eri a for beni gn gas t ri c ul cers i ncl ude: o
o
a. Ul cer crat er ext endi ng beyond t he gas t ri c wal l
o
o
b. Gas t ri c fol ds radi at i ng i nt o t he bas e of t he ul cer
o
o
c. Thi ck radi ol ucent col l ar of edema (Hampt on's l i ne) s urroundi ng t he ul cer bas e
o
o
d. Smoot h, regul ar, round, or ovoi d ul cer crat er
o
o
e. Pl i abl e and normal l y di s t ens i bl e gas t ri c wal l i n t he area of t he ul cer
2. Endoscopy may be us ed as t he pri mary di agnos t i c maneuver or t o confi rm a radi ographi c di agnos i s . Becaus e 5% of gas t ri c ul cers t hat occur i n t he Uni t ed St at es are mal i gnant , many aut hors advocat e endos copy wi t h mul t i pl e bi ops i es at t he margi n of t he ul cer and s i mul t aneous cyt ol ogi c brus hi ngs for t he eval uat i on of al l gas t ri c ul cers , wi t h s ubs equent endos copy i n 6–8 weeks t o document heal i ng.
Pa g e 9 2 2
ABC Ambe r CHM Conve rte r Tria l ve rsion, http://w w w .proce sste x t.com/a bcchm.html
3. Gastric acid analysis may hel p di s t i ngui s h beni gn from mal i gnant gas t ri c ul cers . Becaus e beni gn ul cers rarel y exi s t i n t he s et t i ng of achl orhydri a, t he abs ence of aci d s houl d prompt furt her work-up wi t h gas t ri c bi ops i es and cyt ol ogi c s t udi es . Al t hough gas t ri c ul cers i n t he pres ence of achl orhydri a nearl y al ways are mal i gnant , mos t mal i gnant ul cers occur i n s t omachs wi t h normal aci d s ecret i on.
4. H. pylori detection. (s ee III C 4 c)
F. Therapy Treat ment i s vi rt ual l y t he s ame for es ophageal , gas t ri c, and duodenal ul cers .
1. Intensive antacids, whi l e t hey have been s hown t o promot e t he heal i ng of gas t ri c and duodenal ul cers , are general l y of hi s t ori cal s i gni fi cance onl y.
2. H 2 -receptor antagonists (cimetidine, ranitidine, famotidine, and nizatidine), and proton pump inhibitors (omeprazole, esomeprazole, pantoprazole, lansoprazole, and rabeprazole) are t he mai ns t ay of t reat ment becaus e of pat i ent conveni ence, s us t ai ned aci d reduct i on, and i ncreas ed heal i ng rat es wi t h di mi ni s hed rel aps e rat es —particularly in combination with antibiotics if H. pylori is present.
3. Antibiotics are us ed t o t reat H. pyl ori i nfect i on (s ee III C 4 d). Treat ment i s us ual l y wi t h amoxi ci l l i n, doxycycl i ne wi t h met roni dazol e, or cl ari t hromyci n i n combi nat i on wi t h a prot on pump i nhi bi t or and a bi s mut h preparat i on for 2 weeks .
Pa g e 9 2 3
ABC Ambe r CHM Conve rte r Tria l ve rsion, http://w w w .proce sste x t.com/a bcchm.html
4. Dietary factors may be of s ome i mport ance. There i s no proof t hat bl and di et s promot e heal i ng i n pept i c ul cer di s eas e. In fact , mi l k may be harmful becaus e i t i ncreas es aci d s ecret i on, probabl y by cal ci um- and prot ei n-s t i mul at ed gas t ri n rel eas e. Caffei ne and al cohol s t i mul at e gas t ri c aci d s ecret i on and, t herefore, s houl d be res t ri ct ed i n acut e cas es . Decaffei nat ed coffee al s o may s t i mul at e aci d rel eas e. Ul cer pat i ent s who s moke s houl d be urged t o s t op s moki ng.
5. Other therapeutic agents and approaches o
o
a. Sucralfate i s a nons ys t emi c agent t hat , i n t he pres ence of an aci d pH, coat s t he ul cer bed and promot es heal i ng. Sucral fat e i s as effect i ve as H 2 -recept or ant agoni s t s and ant aci ds and has no s i gni fi cant s i de effect s .
o
o
b. Bismuth has bot h ul cer-i ns ul at i ng and peps i n-i nact i vat i ng propert i es but does not decreas e gas t ri c aci d product i on. Heal i ng rat es are s l i ght l y bet t er i n gas t ri c t han i n duodenal ul cer pat i ent s , and t he overal l rat e of advers e react i on i s l ow. In H. pyl ori –as s oci at ed di s eas e, bi s mut h may caus e t he organi s ms t o di s l odge from gas t ri c epi t hel i al cel l s . Mi l k and ant aci ds may i nt erfere wi t h i t s act i on and s houl d be avoi ded for 1 hour before and aft er i nges t i on of bi s mut h.
o
o
c. Prostaglandin E 2 (PGE 2 ) and PGF 2 may have a cyt oprot ect i ve effect on gas t ri c mucos a. Thes e
Pa g e 9 2 4
ABC Ambe r CHM Conve rte r Tria l ve rsion, http://w w w .proce sste x t.com/a bcchm.html
agent s al s o i ncreas e gas t ri c bl ood fl ow and decreas e gas t ri n-s t i mul at ed aci d s ecret i on. A PGE 2 anal og, misoprostol, has been s hown t o be effect i ve i n t reat i ng pept i c ul cer di s eas e and has been rel eas ed for us e i n prevent i ng gas t ri c ul cers i n pat i ent s t aki ng NSAIDs . The major s i de effect s of pros t agl andi ns are di arrhea and naus ea. o
o
d. T ricyclic antidepressants (e.g., doxepi n) have proved effect i ve i n t reat i ng pept i c ul cers , probabl y becaus e of t hei r H 2 -recept or ant agoni s t effect s .
6. Gastric irradiation decreas es aci d product i on for approxi mat el y 1 year and may pl ay a rol e i n t reat i ng recurrent di s eas e i n el derl y pat i ent s who cannot t ol erat e drugs or s urgery.
7. Surgery i s effect i ve t herapy for pept i c ul cer di s eas e and reduces recurrence rat es . o
o
a. Procedures. The mos t commonl y performed operat i on i s distal subtotal gastrectomy, wi t h wedge res ect i on of a gas t ri c ul cer i f one i s pres ent . Vagotomy with drainage (V + D) and vagotomy with antrectomy (V + A) are t he us ual procedures for compl i cat ed pept i c ul cer di s eas e. V + D i s as s oci at ed wi t h a hi gher recurrence rat e (approxi mat el y 7%–15%) t han V + A (3%), but i t i s as s oci at ed wi t h l es s pos t operat i ve wei ght l os s . A proxi mal s el ect i ve vagot omy appears t o l es s en pos t operat i ve compl i cat i ons .
o
Pa g e 9 2 5
ABC Ambe r CHM Conve rte r Tria l ve rsion, http://w w w .proce sste x t.com/a bcchm.html
o
b. Indications. The hi gh i nci dence of pos t operat i ve compl i cat i ons , regardl es s of t he t ype of procedure, has l i mi t ed t he rol e of s urgery t o treatment of complications (i ncl udi ng acut e emergenci es ) and intractable cases.
G. Complications
1. Hemorrhage occurs i n 20% of pat i ent s and i s t he mos t s eri ous compl i cat i on, havi ng a 10% mort al i t y rat e. If bl ood requi rement s exceed 3 U i n 24 hours for l onger t han 48–72 hours or i f i n-hos pi t al rebl eedi ng occurs , s urgi cal i nt ervent i on i s i ndi cat ed. Repeat ed bl eedi ng epi s odes occur i n approxi mat el y 30%–40% of cas es and may requi re s urgery.
2. Perforation occurs i n approxi mat el y 5%–10% of al l pept i c ul cers and i s far more common wi t h duodenal ul cers t han wi t h gas t ri c ul cers . Of ul cers t hat perforat e, 10% bl eed s i mul t aneous l y. Sympt oms and s i gns i ncl ude i nt ens e pai n, a ri gi d abdomen, decreas ed bowel s ounds , and di rect or rebound t endernes s . Thi s cat as t rophi c compl i cat i on i s confi rmed i n approxi mat el y 75%–85% of cas es by an erect abdomi nal radi ograph s howi ng free ai r under t he di aphragm. Mos t cas es requi re i mmedi at e s urgi cal i nt ervent i on, but s el ect ed pat i ent s have been t reat ed s ucces s ful l y wi t h nas ogas t ri c s uct i on and ant i bi ot i cs .
3. Gastric outlet obstruction occurs i n approxi mat el y 5%–10% of ul cer pat i ent s . Earl y s at i et y, epi gas t ri c ful l nes s , naus ea, and vomi t i ng of undi ges t ed food
Pa g e 9 2 6
ABC Ambe r CHM Conve rte r Tria l ve rsion, http://w w w .proce sste x t.com/a bcchm.html
(frequent l y i nges t ed s everal hours earl i er) s ugges t t he di agnos i s . W ei ght l os s i s common. Treat ment cons i s t s of nas ogas t ri c as pi rat i on for at l eas t 72 hours . Approxi mat el y 25%–40% of pat i ent s requi re s urgery becaus e 20%–40% of medi cal l y t reat ed pat i ent s have recurrent obs t ruct i on. P.195
4. Penetration i nt o an adjacent organ us ual l y i s a compl i cat i on of pos t eri or duodenal ul cers , wi t h penet rat i on i nt o t he pancreas . Pai n us ual l y i s s udden i n ons et and radi at es t o t he back. Serum amyl as e and l i pas e l evel s frequent l y are el evat ed. Treat ment i s s urgi cal .
H. Postsurgical complications
1. Stomal ulceration aft er s urgery may i ndi cat e an unrecogni zed hypers ecret ory s t at e (e.g., Zol l i nger-El l i s on s yndrome, ret ai ned gas t ri c ant rum, or i ncompl et e vagot omy). The di agnos i s of a s t omal ul cer i s bes t made by endos copy. Treat ment i s wi t h l ong-t erm prot on pump i nhi bi t ors , and repeat s urgery may be neces s ary.
2. Afferent loop obstruction i s a rare compl i cat i on. The pat i ent us ual l y compl ai ns of bl oat i ng and vomi t i ng of a cl ear or bi l i ous mat eri al approxi mat el y 30–60 mi nut es aft er eat i ng. The di agnos i s i s s ugges t ed by t he fai l ure of oral l y i nges t ed bari um t o ent er t he l oop or by t he
Pa g e 9 2 7
ABC Ambe r CHM Conve rte r Tria l ve rsion, http://w w w .proce sste x t.com/a bcchm.html
ret ent i on i ns i de t he l oop of t echnet i um-i mi nodi acet i c aci d (Tc-HIDA), whi ch i s cl eared by t he l i ver and bi l i ary s ys t em aft er i nt ravenous i nject i on but does not ent er t he gas t roi nt es t i nal t ract . Bact eri al overgrowt h can occur, and s urgi cal revi s i on of t he afferent l oop may be neces s ary.
3. Alkaline gastritis oft en i s s een endos copi cal l y i n pat i ent s who have undergone ant rect omy or s ubt ot al gas t rect omy. Gas t ri t i s oft en i s as ympt omat i c but may caus e naus ea, vomi t i ng, wei ght l os s , and epi gas t ri c pai n. Di agnos i s may be ai ded by meas urement of HIDA-l abel ed bi l e refl uxi ng back i nt o t he s t omach. Becaus e t he gas t ri t i s i s s econdary t o t hi s i ncreas ed refl ux of duodenal s ecret i ons i nt o t he s t omach, pat i ent s may requi re s urgery (a Roux-en-Y anas t omos i s ) t o di vert t hes e s ecret i ons furt her down t he gas t roi nt es t i nal t ract . Some cas es may be t reat ed effect i vel y wi t h s ubs t ances t hat bi nd bi l e aci ds (e.g., al umi num-cont ai ni ng ant aci ds and chol es t yrami ne) or wi t h s ucral fat e.
4. Dumping syndrome i s a nons peci fi c t erm t hat refers t o a vari et y of pos t prandi al s ympt oms . o
o
a. Early dumping syndrome occurs approxi mat el y 30 mi nut es aft er a meal and i s as s oci at ed wi t h di zzi nes s , fl us hi ng, di aphores i s , and pal pi t at i ons . Thes e s ympt oms have been as cri bed t o os mot i c s hi ft s of fl ui d or rel eas e of mas s i ve amount s of i nt es t i nal hormones as food empt i es rapi dl y from t he s t omach. The earl y dumpi ng s yndrome can be mi ni mi zed by decreas i ng t he carbohydrat e cont ent of meal s and by avoi di ng l i qui ds wi t h meal s .
Pa g e 9 2 8
ABC Ambe r CHM Conve rte r Tria l ve rsion, http://w w w .proce sste x t.com/a bcchm.html
Synt het i c s omat os t at i n has been us ed i n res i s t ant cas es . o
o
b. Late dumping syndrome occurs s everal hours aft er a meal and i s charact eri zed by di zzi nes s , weaknes s , and drows i nes s . Thi s s yndrome may be caus ed by react i ve hypogl ycemi a.
5. Nutritional problems o
o
a. Anemia occurs i n approxi mat el y 25% of pat i ent s aft er s urgery for pept i c ul cer di s eas e. Fact ors t hat may cont ri but e t o i ron defi ci ency i ncl ude l ow-grade bl ood l os s from al kal i ne pouch gas t ri t i s , di vers i on of i ron away from i t s preferent i al abs orpt i on s i t e (i .e., t he duodenum), and l ack of gas t ri c aci d needed for convers i on of i ron t o t he preferred form 3+
for abs orpt i on (i .e., Fe ). A l ack of i nt ri ns i c fact or l eads t o vi t ami n B 1 2 defi ci ency i n pat i ent s who have undergone a s ubs t ant i al gas t ri c res ect i on. o
o
b. Weight loss occurs i n approxi mat el y 50% of pos t operat i ve pat i ent s but i s not s evere unl es s a l arge gas t ri c res ect i on has been performed.
o
o
c. Significant steatorrhea us ual l y i ndi cat es a s econdary probl em (e.g., bact eri al overgrowt h or unmas ked cel i ac di s eas e), al t hough 50% of pat i ent s have an i ncreas e i n s t ool fat s econdary t o rapi d t rans i t and poor mi xi ng of food wi t h bi l e s al t s and pancreat i c enzymes .
o
Pa g e 9 2 9
ABC Ambe r CHM Conve rte r Tria l ve rsion, http://w w w .proce sste x t.com/a bcchm.html
o
d. Bone thinning may be due t o decreas ed abs orpt i on of vi t ami n D and cal ci um.
6. Gastric pouch cancer rat es are t wo t o four t i mes great er i n ul cer pat i ent s who have undergone s urgery t han i n pat i ent s whos e ul cers are t reat ed medi cal l y, es peci al l y 15–20 years aft er Bi l l rot h II gas t ri c res ect i on.
I. Zollinger-Ellison syndrome Zollinger-Ellison syndrome refers t o a non–β i s l et cel l t umor t hat produces gas t ri n and i s as s oci at ed wi t h gas t ri c aci d hypers ecret i on and pept i c ul cer di s eas e. The t umors are bi ol ogi cal l y mal i gnant i n 60% of cas es and mos t commonl y i nvol ve t he pancreas . Ot her t umor s i t es i ncl ude t he s t omach, duodenum, s pl een, and l ymph nodes . Tumor s i ze vari es from 2 mm t o 20 cm. Approxi mat el y 10% of t he pat i ent s wi t h Zol l i nger-El l i s on s yndrome have a res ect abl e l es i on.
1. Clinical features o
o
a. Pain from pept i c ul cer di s eas e i s common i n Zol l i nger-El l i s on s yndrome. In approxi mat el y 75% of cas es , t he ul cers are l ocat ed i n t he duodenal bul b. The remai ni ng cas es i nvol ve ul cers i n t he di s t al duodenum or jejunum or ul cers i n mul t i pl e l ocat i ons .
o
o
b. Diarrhea occurs i n approxi mat el y 50% of cas es becaus e of gas t ri c aci d hypers ecret i on. The hi gh aci d l evel s may damage t he s mal l i nt es t i nal mucos a, i nact i vat e pancreat i c l i pas e, and
Pa g e 9 3 0
ABC Ambe r CHM Conve rte r Tria l ve rsion, http://w w w .proce sste x t.com/a bcchm.html
preci pi t at e bi l e aci ds , caus i ng s t eat orrhea. The hi gh gas t ri n l evel s caus e i ncompl et e Na
+
and wat er
abs orpt i on and i ncreas e i nt es t i nal mot i l i t y. In addi t i on, t he vol ume of gas t ri c s ecret i on al one may caus e di arrhea. o
o
c. Endocrine abnormalities are common. Approxi mat el y 20% of t hes e pat i ent s have hyperparat hyroi di s m. Pi t ui t ary, adrenal , ovari an, and t hyroi d t umors al s o have been report ed wi t h Zol l i nger-El l i s on s yndrome. A di s t i nct s yndrome of pancreat i c, pi t ui t ary, and parat hyroi d t umors (i .e., MEN I) s hows an aut os omal domi nant pat t ern of i nheri t ance.
2. Diagnosis o
o
a. Gastrin levels. Pat i ent s demons t rat e el evat ed bas al -s t at e gas t ri n l evel s t hat do not i ncreas e 1 hour aft er a meal . Gas t ri n l evel s i ncreas e (rat her t han decl i ne) by 200 U aft er i nt ravenous s ecret i n admi ni s t rat i on and ri s e markedl y (rat her t han modes t l y) aft er i nt ravenous cal ci um admi ni s t rat i on.
o
o
b. Gastric acid output. Pat i ent s wi t h Zol l i nger-El l i s on s yndrome oft en have bas al gas t ri c aci d out put rat es of more t han 10 mEq/hour and bas al -t o-peak out put rat i os of great er t han 0.6.
o
o
c. Angiography. Becaus e gas t ri n-s ecret i ng t umors may be hi ghl y vas cul ar, angi ography may be hel pful .
Pa g e 9 3 1
ABC Ambe r CHM Conve rte r Tria l ve rsion, http://w w w .proce sste x t.com/a bcchm.html
3. T herapy o
o
a. Surgical treatment
(1) T otal gastrectomy i s t he t radi t i onal t herapy. The 10-year s urvi val rat e of 50% wi t h t hi s procedure i s t hought t o be at t ri but abl e pri mari l y t o t he s l ow-growi ng nat ure of t hi s l es i on. Mos t of t he l at e deat hs are caus ed by met as t at i c di s eas e.
(2) T umor localization, whi ch i nvol ves s ampl i ng gas t ri n l evel s t hrough cannul at i on of mul t i pl e pancreat i c and abdomi nal vei ns , may be us eful . Thi s t echni que offers t he hope of s urgi cal cure i n cas es of mul t i pl e pri mary t umors and i n cas es i nvol vi ng a t umor t hat i s t oo s mal l t o be vi s ual i zed by ordi nary means .
o
o
b. Medical treatment
(1) Proton pump inhibitors are t he medi cal t reat ment of choi ce and may requi re hi gh-dos e t herapy, dependi ng on t he res pons e t o aci d-s uppres s i ve t herapy us i ng gas t ri c anal ys i s for document at i on of aci d s uppres s i on.
(2) H 2 -recept or ant agoni s t s i n combi nat i on
Pa g e 9 3 2
ABC Ambe r CHM Conve rte r Tria l ve rsion, http://w w w .proce sste x t.com/a bcchm.html
wi t h anticholinergic agents have been us ed, es peci al l y i n conjunct i on wi t h a V + D procedure. Pat i ent s who have been unres pons i ve t o H 2 -recept or bl ockade may become res pons i ve as a res ul t of s urgery.
J. Other disorders of the stomach
1. Portal hypertensive gastropathy i s t he t erm gi ven t o di ffus e s ubmucos al di l at at i on of gas t ri c ves s el s t hat may rupt ure. It account s for upper gas t roi nt es t i nal bl eedi ng i n approxi mat el y 10% of pat i ent s wi t h port al hypert ens i on. A di ffus e but i rregul ar pat t ern of red s pot s or l i near s t reaks produces a charact eri s t i c pat t ern at endos copy. Treat ment wi t h β-bl ockade or bi cap (el ect rocaut ery) or l as er t herapy t o t he gas t ri c wal l may decreas e rebl eedi ng.
2. Dieulafoy' s ulcer i s a di ffi cul t -t o-di agnos e vas cul ar l es i on general l y s een i n t he gas t ri c fundus . Thi s ul cer may be a caus e of s i gni fi cant recurrent upper gas t roi nt es t i nal bl eedi ng. Men ol der t han 50 years of age are mos t commonl y affect ed. The l es i on appears as an expos ed art eri al defect wi t h no or mi ni mal mucos al ul cerat i on. Endos copy wi t h l as er or bi cap t reat ment i s effect i ve i n s t oppi ng t he bl eedi ng, but angi ography or s urgery i s oft en neces s ary becaus e of t he di ffi cul t y of i dent i fyi ng t he s mal l vas cul ar defect duri ng endos copy.
IV. Diseases of the Small Intestine A. Intestinal obstruction Intestinal obstruction i s a t erm us ed t o denot e fai l ure of pas s age of i nt es t i nal cont ent s and may be due t o mechani cal obs t ruct i on or adynami c i l eus .
Pa g e 9 3 3
ABC Ambe r CHM Conve rte r Tria l ve rsion, http://w w w .proce sste x t.com/a bcchm.html
1. Mechanical obstruction o
o
a. Etiology
(1) Extrinsic causes: Adhes i ons from pri or s urgery, i ncarcerat ed herni a, met as t at i c t umors , vol vul us , endomet ri os i s , and NSAID-i nduced s t ri ct ures (oft en mul t i pl e)
(2) Intramural causes: Hemat omas from t rauma, s t ri ct ures , and i nt ramural t umors
(3) Intraluminal causes: Epi t hel i al t umors (es peci al l y col oni c), i nt us s us cept i on, and forei gn bodi es
o
o
b. Clinical features
(1) Crampy pai n t hat waxes and wanes i n i nt ens i t y
(2) Hi gh-pi t ched bowel s ounds wi t h rus hes and t i nkl es
(3) Cons t i pat i on and obs t i pat i on
(4) Vomi t i ng, whi ch i s more promi nent i n
Pa g e 9 3 4
ABC Ambe r CHM Conve rte r Tria l ve rsion, http://w w w .proce sste x t.com/a bcchm.html
proxi mal i nt es t i nal obs t ruct i on
(5) Di s t ent i on, whi ch i s more promi nent i n di s t al i nt es t i nal obs t ruct i on
(6) Int es t i nal i s chemi a, l eadi ng fi rs t t o edema, t hen t o pet echi al hemorrhages , and fi nal l y t o necros i s and gangrene (Thi s i s s econdary t o i ncreas ed i nt ral umi nal pres s ure occurri ng aft er 6–12 hours of obs t ruct i on, when abs orpt i on ceas es and s ecret i on commences .)
o
o
c. Diagnosis us ual l y i s made wi t h pl ai n and upri ght abdomi nal radi ographs . Charact eri s t i c ai r-fl ui d l evel s exi s t above t he area of obs t ruct i on, and no ai r i s s een i n t he rect um. A bari um enema may be us eful i n di agnos i ng col oni c obs t ruct i on. Refl ux of bari um i nt o t he s mal l bowel al s o may be hel pful i n t he di agnos i s of l ow s mal l bowel obs t ruct i on.
o
o
d. T herapy
(1) Repl acement of fl ui d and el ect rol yt es
(2) Int es t i nal decompres s i on wi t h nas ogas t ri c s uct i on or s mal l bowel i nt ubat i on
(3) Surgery, whi ch us ual l y i s requi red for defi ni t i ve t reat ment of t he underl yi ng probl em
Pa g e 9 3 5
ABC Ambe r CHM Conve rte r Tria l ve rsion, http://w w w .proce sste x t.com/a bcchm.html
P.196
2. Adynamic, or paralytic, ileus i s a nonobs t ruct i ve l ack of propul s i on t hrough t he i nt es t i nal t ract . o
o
a. Etiology. Adynami c i l eus commonl y i s l i nked t o t he fol l owi ng condi t i ons :
(1) Recent abdomi nal s urgery, whi ch res ul t s i n i l eus t hat us ual l y i s t rans i ent (l as t i ng 2–3 days )
(2) El ect rol yt e i mbal ance, es peci al l y hypokal emi a
(3) Chemi cal or bact eri al peri t oni t i s
(4) Severe i nt ra-abdomi nal i nfl ammat i on s uch as pancreat i t i s and chol ecys t i t i s
(5) Sys t emi c i l l nes s s uch as pneumoni a
o
o
b. Clinical features. Phys i cal exami nat i on s hows a di s t ended abdomen and di mi ni s hed bowel s ounds .
o
o
c. Diagnosis. Radi ographs s how di ffus e i nt es t i nal gas and ai r i n t he rect um wi t hout ai r fl ui d l evel s .
Pa g e 9 3 6
ABC Ambe r CHM Conve rte r Tria l ve rsion, http://w w w .proce sste x t.com/a bcchm.html o
o
d. T herapy
(1) Bowel res t and pl acement of a nas ogas t ri c t ube
(2) Correct i on of underl yi ng caus es
(3) Int ravenous neos t i gmi ne (wi t h careful cardi ac moni t ori ng) may be us ed i n res i s t ant cas es wi t h careful cardi ac moni t ori ng.
B. Intestinal pseudo-obstruction Intestinal pseudo-obstruction i s a rare but i mport ant ent i t y charact eri zed by apparent l y recurrent epi s odes of mechani cal obs t ruct i on but wi t h no demons t rabl e s ource of obs t ruct i on.
1. Classification. o
o
a. Secondary pseudo-obstruction. Ps eudo-obs t ruct i on occurs s econdary t o many condi t i ons t hat affect ei t her t he s moot h mus cl e of t he gas t roi nt es t i nal t ract or t he neurol ogi c and hormonal cont rol of i nt es t i nal mot i l i t y.
(1) Underl yi ng di s eas es t hat i nvol ve t he s moot h mus cl e i ncl ude col l agen vas cul ar di s eas e (es peci al l y s cl eroderma), amyl oi dos i s , and myot oni c dys t rophy.
Pa g e 9 3 7
ABC Ambe r CHM Conve rte r Tria l ve rsion, http://w w w .proce sste x t.com/a bcchm.html
(2) Underl yi ng neurol ogi c di s eas es i ncl ude Chagas ' di s eas e, Parki ns on's di s eas e, and Hi rs chs prung's di s eas e.
(3) Underl yi ng endocri ne di s orders i ncl ude hypot hyroi di s m, di abet es mel l i t us , hypoparat hyroi di s m, and pheochromocyt oma.
(4) Drugs t hat depres s i nt es t i nal s moot h mus cl e funct i on i ncl ude phenot hi azi nes , t ri cycl i c ant i depres s ant s , gangl i oni c bl ockers , and cl oni di ne.
(5) Nont ropi cal s prue and ceroi d depos i t s i n t he bowel (mahogany bowel ) are rare caus es of ps eudo-obs t ruct i on.
o
o
b. Primary pseudo-obstruction. There are t wo forms of t hi s condi t i on.
(1) Heredi t ary hollow visceral myopathy i s a vacuol i zat i on and at rophy of i nt es t i nal s moot h mus cl e. Thi s di s order, whi ch i s t rans mi t t ed as an aut os omal domi nant t rai t , al s o affect s es ophageal and uri nary t ract s moot h mus cl e.
(2) An autonomic nervous system abnormality has been des cri bed i n s ome fami l i es wi t h pri mary ps eudo-obs t ruct i on. There may be a decreas e i n t ot al myent eri c pl exus neurons or neuronal eos i nophi l i c
Pa g e 9 3 8
ABC Ambe r CHM Conve rte r Tria l ve rsion, http://w w w .proce sste x t.com/a bcchm.html
i nt ranucl ear i ncl us i ons . Pos s i bl e s ympt oms i ncl ude ort hos t at i c hypot ens i on, at axi c gai t , dys art hri a, and abs ent deep t endon refl exes . o
o
2. Diagnosis. Ri gorous excl us i on of caus es of mechani cal obs t ruct i on wi t h document at i on of abnormal mot i l i t y i s neces s ary.
a. Esophageal manometry s howi ng normal or l ow LES pres s ure wi t h decreas ed ampl i t ude of peri s t al s i s di s t al l y i ndi cat es t he s moot h mus cl e vacuol i zat i on t ype of ps eudo-obs t ruct i on. Incompl et e rel axat i on of t he LES wi t h abs ent peri s t al s i s and repet i t i ve es ophageal cont ract i ons may i ndi cat e an aut onomi c nervous s ys t em abnormal i t y.
b. Radionuclide gastric emptying scans may s how del ayed empt yi ng.
c. Barium studies yi el d nons peci fi c fi ndi ngs . Mos t pat i ent s s how di l at ed areas of t he i nt es t i ne.
o
o
3. T herapy. Treat ment i s s upport i ve duri ng acut e exacerbat i ons . Chol i nergi c agent s and proki net i c agent s have been us ed wi t h l i mi t ed s ucces s , and s urgery s houl d be avoi ded. Home parent eral hyperal i ment at i on may be requi red for nut ri t i onal s upport . Int es t i nal s t as i s wi t h bact eri al overgrowt h s houl d be t reat ed wi t h ant i bi ot i cs .
Pa g e 9 3 9
ABC Ambe r CHM Conve rte r Tria l ve rsion, http://w w w .proce sste x t.com/a bcchm.html
C. Small bowel diverticula
1. Duodenal diverticula us ual l y are found i nci dent al l y duri ng an upper gas t roi nt es t i nal s eri es , at endos copy, or at aut ops y. They occur mos t frequent l y i n t he proxi mal duodenum, wi t hi n 1–2 cm of t he ampul l a of Vat er, and are as ympt omat i c i n mos t pat i ent s , al t hough duodenal di vert i cul a may rarel y caus e upper gas t roi nt es t i nal bl eedi ng. In s ome cas es , t he common bi l e duct empt i es di rect l y i nt o t he di vert i cul um, and common bi l e duct obs t ruct i on may occur becaus e of anat omi c i nt erference wi t h empt yi ng.
2. Jejunal diverticula probabl y are acqui red rat her t han congeni t al and us ual l y are as ympt omat i c. Jejunal di vert i cul a may l ead t o mal abs orpt i on s econdary t o bi l e s al t deconjugat i on when s t as i s i s s uffi ci ent t o al l ow an i ncreas e i n s mal l bowel bact eri a. Thi s l eads t o di arrhea, s t eat orrhea, wei ght l os s , and anemi a. Hypochl orhydri a or achl orhydri a al s o may be pres ent . Cont i nuous or al t ernat i ng ant i mi crobi al t herapy oft en correct s mal abs orpt i on, al t hough s urgi cal removal of mul t i pl e di vert i cul a or of a s i ngl e l arge di vert i cul um may be neces s ary i n refract ory cas es .
3. Meckel' s diverticulum i s a common congeni t al s t ruct ural defect t hat repres ent s t he remnant of t he vi t el l i ne duct . It i s found i n approxi mat el y 2% of aut ops i es and us ual l y i s l ocat ed i n t he t ermi nal i l eum wi t hi n 60 cm of t he i l eocecal val ve. Meckel 's di vert i cul a average 5–7 cm i n l engt h and may be qui t e l arge. Approxi mat el y one-t hi rd of Meckel 's di vert i cul a cont ai n
Pa g e 9 4 0
ABC Ambe r CHM Conve rte r Tria l ve rsion, http://w w w .proce sste x t.com/a bcchm.html
gas t ri c mucos a, whi ch may produce aci d. Di vert i cul i t i s , ul cerat i on, bl eedi ng, perforat i on, and obs t ruct i on are compl i cat i ons t hat requi re s urgi cal i nt ervent i on and frequent l y mi mi c t he s ympt oms of acut e appendi ci t i s . Becaus e of t he pres ence of gas t ri c mucos a, t he di agnos i s s omet i mes can be made by means of pert echnet at e s canni ng aft er H 2 -recept or bl ockade.
D. Diarrhea Diarrhea i s defi ned as an i ncreas e i n s t ool frequency and vol ume. The s t ool us ual l y i s l i qui d, and 24-hour out put exceeds 250 g. Pat i ent s may experi ence l ower abdomi nal crampy pai n and fecal urgency.
1. Classification. Pat hophys i ol ogi c cri t eri a are us ed t o cl as s i fy di arrhea as one of t hree di s t i nct t ypes . o
o
a. Secretory diarrhea
(1) Pathophysiology. Secret ory di arrhea occurs when t he s ecret i on of fl ui d and el ect rol yt es i s i ncreas ed or when t he normal abs orpt i ve capaci t y of t he bowel i s decreas ed.
(a) Agent s t hat act i vat e t he adenyl cycl as e–cycl i c adenos i ne 3′,5′-monophos phat e (cAMP) s ys t em i ncl ude chol era t oxi n, heat -l abi l e Es c heri c hi a c ol i t oxi n, Sal monel l a ent erot oxi n, and vas oact i ve i nt es t i nal pept i de (VIP).
Pa g e 9 4 1
ABC Ambe r CHM Conve rte r Tria l ve rsion, http://w w w .proce sste x t.com/a bcchm.html
(b) Agent s t hat probabl y do not act i vat e t he adenyl cycl as e–cAMP s ys t em i ncl ude heat -s t abl e E. c ol i t oxi n, a vari et y of ot her bact eri al ent erot oxi ns (e.g., t hos e produced by Cl os t ri di um perfri ngens , Ps eudomonas aerugi nos a, and Kl ebs i el l a pneumoni ae), cas t or oi l , and phenol pht hal ei n.
(c) Chroni c s ecret ory di arrhea i s s een i n t he pancreat i c chol era s yndrome wi t h VIP s ecret i on, i n medul l ary carci noma of t he t hyroi d gl and wi t h cal ci t oni n s ecret i on, i n carci noi d s yndrome wi t h s erot oni n s ecret i on, and i n vi l l ous adenoma of t he rect um.
(2) Diagnosis. Pers i s t ent di arrhea i n t he abs ence of food i nt ake or t he pres ence of a +
s t ool os mot i c gap [St ool os mol ari t y - 2(Na + +
K <].
(3) T herapy. Fl ui d and el ect rol yt e s upport s houl d be gi ven whi l e t he caus e of t he di arrhea i s bei ng det ermi ned. In general , di arrhea s econdary t o bact eri al ent erot oxi n i s s el f-l i mi t ed. Any cont ri but i ng exogenous agent mus t be wi t hdrawn.
o
o
b. Osmotic diarrhea i s caus ed by t he pres ence of nonabs orbabl e s ubs t ances i n t he i nt es t i ne, wi t h t he s econdary accumul at i on of fl ui d and el ect rol yt es . Such nonabs orbabl e s ubs t ances
Pa g e 9 4 2
ABC Ambe r CHM Conve rte r Tria l ve rsion, http://w w w .proce sste x t.com/a bcchm.html
i ncl ude l act os e i n a pat i ent wi t h l act as e defi ci ency, l axat i ves (e.g., magnes i um ci t rat e and s odi um phos phat e), and foods t uffs i n a pat i ent wi t h mal abs orpt i on. The di agnos i s i s s ugges t ed by t he abs ence of di arrhea aft er a 48- t o 72-hour fas t (wi t h concurrent i nt ravenous fl ui d repl acement ). There i s a s t ool os mot i c gap of great er t han 50. o
o
c. Abnormal intestinal motility caus es or cont ri but es t o t he di arrhea s een i n di abet es , i rri t abl e bowel s yndrome, pos t vagot omy s t at es , carci noi d s yndrome, and hypert hyroi di s m. Mechani s ms of abnormal i nt es t i nal mot i l i t y i ncl ude t he fol l owi ng:
(1) If s mal l bowel peri s t al s i s i s t oo rapi d, an abnormal l y l arge amount of fl ui d and part i al l y di ges t ed foods t uffs may be del i vered t o t he col on. P.197
(2) Ext remel y s l ow peri s t al s i s may al l ow bact eri al overgrowt h t o occur and bi l e s al t deconjugat i on t o caus e s econdary mal abs orpt i on.
(3) Rapi d col oni c mot i l i t y may not al l ow adequat e t i me for t he col on t o abs orb fl ui d del i vered t o t he cecum. (Normal l y, 90% of t he fl ui d i s abs orbed.)
Pa g e 9 4 3
ABC Ambe r CHM Conve rte r Tria l ve rsion, http://w w w .proce sste x t.com/a bcchm.html
2. Diagnosis o
o
a. T ests performed on s t ool s ampl es i ncl ude:
(1) Culture and sensitivity testing t o det ect a pat hogeni c bact eri al s t rai n. A pos i t i ve s t ool cul t ure i s found for 40% of pat i ent s who have whi t e bl ood cel l s (W BCs ) i n t he s t ool and fever.
(2) Microscopic examination t o i dent i fy ova and paras i t es (t hree s ampl es s houl d be s ent t o i ncreas e yi el d)
(3) Fecal occul t bl ood t es t i ng
(4) Sudan staining t o det ect fat dropl et s
(5) Fecal leukocyte t es t i ng or s t ool l act oferri n (Not e t hat t oxi geni c E. c ol i , vi rus es , Norwal k agent , and Gi ardi a l ambl i a are not i nvas i ve. Irri t abl e bowel s yndrome, mal abs orpt i on s yndrome, and l axat i ve abus e do not caus e pus i n t he s t ool .) St aphyl oc oc c us aureus , C. perfri ngens , and Ent amoeba hi s t ol yt i c a al s o may be pres ent wi t h fecal l eukocyt es .
(6) Tes t i ng for t he pres ence of Clostridium difficile toxin i n s t ool .
Pa g e 9 4 4
ABC Ambe r CHM Conve rte r Tria l ve rsion, http://w w w .proce sste x t.com/a bcchm.html o
o
b. Proctosigmoidoscopy al s o i s performed, es peci al l y t o excl ude or confi rm a di agnos i s of i nfl ammat ory bowel di s eas e.
3. Other causes of persistent diarrhea o
o
a. Irritable bowel syndrome (IBS) i s an i nt es t i nal di s order charact eri zed by chroni c abdomi nal pai n and al t ered bowel habi t s i n t he abs ence of any organi c caus e. It i s t he mos t commonl y di agnos ed gas t roi nt es t i nal condi t i on and compri s es 25%—50% of al l referral s t o gas t roent erol ogi s t s . Al t hough IBS can affect any i ndi vi dual , i t i s more common i n young pat i ent s and women, wi t h an es t i mat ed 2:1 femal e predomi nance i n Nort h Ameri ca.
(1) Pathophysiology. The exact pat hophys i ol ogy of IBS remai ns uncert ai n. Many bel i eve t here i s i nt erpl ay bet ween abnormal gas t roi nt es t i nal mot i l i t y (al t ered bas al cont ract i ons , prol onged mot or act i vi t y aft er meal s , abnormal res pons e t o s t res s ) i n t he s mal l bowel and col on, abnormal and hypers ens i t i ve vi s ceral nerve percept i on i n t he gut , and mucos al i mmune s ys t em act i vat i on. Res earchers are al s o expl ori ng t he pos s i bi l i t y of t he devel opment of IBS fol l owi ng a bout of i nfect i ous ent eri t i s . Al t hough i nves t i gat i ons are ongoi ng, no phys i ol ogi c or ps ychol ogi c abnormal i t y has been found t o be s peci fi c t o IBS.
Pa g e 9 4 5
ABC Ambe r CHM Conve rte r Tria l ve rsion, http://w w w .proce sste x t.com/a bcchm.html
(2) Clinical features. The pri mary compl ai nt s i n pat i ent s wi t h IBS are chroni c abdomi nal pai n and al t ered bowel habi t s . Abdomi nal pai n i s t ypi cal l y i nt ermi t t ent , vari abl e i n i nt ens i t y, and crampy i n nat ure. The pai n i s not us ual l y progres s i ve i n nat ure, as s oci at ed wi t h wei ght l os s or anorexi a, or found t o prevent pat i ent s from s l eepi ng. Bowel habi t s wi t h IBS are wi del y vari abl e. Compl ai nt s range from di arrhea or cons t i pat i on predomi nant , t o al t ernat i ng di arrhea and cons t i pat i on, t o normal s t ool s al t ernat i ng wi t h di arrhea and cons t i pat i on. Pat i ent s oft en compl ai n of ext reme urgency pri or t o a bowel movement fol l owed by a s ens e of i ncompl et e evacuat i on. Las t l y, t here i s al s o a s ens e of bl oat i ng and i ncreas ed abdomi nal gas i n pat i ent s wi t h IBS.
(3) Diagnosis i ncl udes careful hi s t ory and phys i cal l ooki ng for a compat i bl e s ympt om compl ex i n conjunct i on wi t h di agnos t i c t es t i ng t o rul e out organi c di s eas e. Gui del i nes recommend rout i ne l aborat ory t es t i ng, t he res ul t s of whi ch are us ual l y normal i n t hes e pat i ent s . St ool os mol al i t y, paras i t e exami nat i on, t hyroi d funct i on t es t s , s edi ment at i on rat e, and t es t i ng for cel i ac di s eas e s houl d be performed i f s ympt oms are cons i s t ent wi t h di arrhea predomi nant di s eas e. Fl exi bl e s i gmoi dos copy or col onos copy may be us ed i n pat i ent s ol der t han 40 t o excl ude neopl as m or mi cros copi c col i t i s , or younger pat i ent s t o excl ude i nfl ammat ory bowel
Pa g e 9 4 6
ABC Ambe r CHM Conve rte r Tria l ve rsion, http://w w w .proce sste x t.com/a bcchm.html
di s eas e.
(4) T herapy. Treat ment i ncl udes s t ool -bul ki ng agent s , ant i s pas modi cs , and pat i ent reas s urance. Effect i ve agent s i ncl ude t he s erot oni n 5-HT 3 ant agoni s t al os et ron for di arrhea and abdomi nal pai n, t he s erot oni n 5-HT 4 part i al agoni s t t egas erod, and t he chl ori de channel ant agoni s t l ubi pros t one for cons t i pat i on.
P.198
o
o
b. Lactase deficiency us ual l y i s not compl et e (i .e., pat i ent s have s ome enzyme act i vi t y), and pres ent i ng s ympt oms occur aft er pubert y. It affect s 60%–80% of bl acks and occurs t o a l es s er ext ent i n As i an and Medi t erranean popul at i ons . Sympt oms of l act as e defi ci ency i ncl ude abdomi nal bl oat i ng, crampi ng, and wat ery di arrhea aft er mi l k i nges t i on. Lact os e breat h t es t s meas ure i ncreas ed H
+
excret i on from col oni c bact eri al di ges t i on of l act os e. A rel at i ve l act as e defi ci ency may occur aft er an acut e epi s ode of vi ral ent eri t i s or i n as s oci at i on wi t h cel i ac di s eas e, W hi ppl e's di s eas e, or cys t i c fi bros i s . o
o
c. Incontinence i s a di s t urbi ng s ympt om t hat pat i ent s may fi nd di ffi cul t t o di s cus s wi t h a phys i ci an. It i s oft en not t rue di arrhea but i s as s oci at ed wi t h i nfl ammat ory di s eas es of t he anal canal (e.g., acut e gonococcal proct i t i s , Crohn's
Pa g e 9 4 7
ABC Ambe r CHM Conve rte r Tria l ve rsion, http://w w w .proce sste x t.com/a bcchm.html
di s eas e, and ul cerat i ve col i t i s ) or wi t h s ys t emi c neuromus cul ar di s eas es (e.g., di abet es mel l i t us and s cl eroderma). Incont i nence al s o may be a compl i cat i on of anal s urgery (e.g., fi s t ul ect omy and hemorrhoi dect omy). o
o
d. Laxative abuse, whi ch may be as s oci at ed wi t h ps ychi at ri c probl ems and a des i re t o l os e wei ght , i s an i ncreas i ng caus e of di arrhea. The t ype of l axat i ve i nges t ed det ermi nes t he cl i ni cal feat ures . Magnes i um s ul fat e, nonabs orbabl e s ugars (e.g., l act ul os e), and s odi um phos phat e res ul t i n os mot i c di arrhea. Di hydroxy bi l e s al t s , cas t or oi l , and di oct yl s odi um s ul fos ucci nat e (docus at e s odi um) caus e s ecret ory di arrhea. Surrept i t i ous l axat i ve us e i s di ffi cul t t o document except i n cas es of phenol pht hal ei n us e, i n whi ch al kal i ni zat i on of a s t ool or uri ne s ampl e by t he addi t i on of s odi um hydroxi de caus es t he col or of t he s ampl e t o change t o pi nk.
o
o
e. Systemic mastocytosis i s charact eri zed by mas t cel l prol i ferat i on i n t he s ki n, bones , l ymph nodes , and parenchymal organs . Ei ght y percent of affect ed pat i ent s have gas t roi nt es t i nal s ympt oms t hat i ncl ude naus ea, vomi t i ng, and recurrent epi s odes of di arrhea and abdomi nal pai n. Pruri t us , fl us hi ng, t achycardi a, hypot ens i ve epi s odes , and headaches al s o may be s een on an epi s odi c bas i s . Hepari n, al s o l i berat ed from mas t cel l s , can cont ri but e t o gas t roi nt es t i nal hemorrhage from pept i c ul cer di s eas e, whi ch i s rel at ed t o hi s t ami ne-i nduced hyperchl orhydri a.
Pa g e 9 4 8
ABC Ambe r CHM Conve rte r Tria l ve rsion, http://w w w .proce sste x t.com/a bcchm.html
(1) Diagnosis i s s ugges t ed when jejunal bi ops i es s how l arge numbers of mas t cel l s i n t he l ami na propri a, mus cul ari s mucos a, and s ubmucos a wi t h normal or mi l d vi l l ous at rophy. The cl as s i c dermat ol ogi c fi ndi ng i s t hat of urt i cari a pi gment os a as s oci at ed wi t h s ys t emi c mas t ocyt os i s .
(2) T herapy has been wi t h agent s t o bl ock bot h H 1 and H 2 recept ors , ant i chol i nergi cs , and, occas i onal l y, s t eroi ds
4. Infectious causes of di arrhea i ncl ude t he bact eri a l i s t ed i n Tabl e 5-1, whi ch caus e food poi s oni ng. Some organi s ms , s uch as C. di ffi c i l e and E. hi s t ol yt i c a, pri mari l y at t ach t o t he col on and rect um, res pect i vel y.
TABLE 5-1 Food Poisoning Syndromes S P I
o at
n
u h
c
rc o
u
e g
b
s e
at
of ni
O io S C c C r n y o M o g P m nt e m a e pt a c m ni ri o m h e s o m in a nt m d s at ni s
Pa g e 9 4 9
ABC Ambe r CHM Conve rte r Tria l ve rsion, http://w w w .proce sste x t.com/a bcchm.html
s io m n s Bacteria St 2 S M T S a â P, e o u p €“ S at xi d h 8 V, a n d yl h D n
e
o o
d
n
c ur
d
o
o s
ai
n
cc
ry
s
u
pr
et
s
o
;
a
d
in
ur
uc
te
e
ts
n
u
s
s
e v o m iti n g; n o th er a p y n e
Pa g e 9 5 0
ABC Ambe r CHM Conve rte r Tria l ve rsion, http://w w w .proce sste x t.com/a bcchm.html
e d e d in m o st ca s e s B 2 S R Ti E a â P, e s s ar c i €“ S h u l y l l 8 V, e e v u h S at i n o s o D e v m c ur
d a iti
er s
fri s i n
e
e o g,
u
d n la
s
ri
te
ce
r di ar rh e a; re co v er y wi
Pa g e 9 5 1
ABC Ambe r CHM Conve rte r Tria l ve rsion, http://w w w .proce sste x t.com/a bcchm.html
th in 2 4 h o ur s Cl 8 V, R T Pr o â S e o of s t €“ D h xi u ri 1
e n s
di 4
at
e
u h
e
di
mo
d
ar
p ur
m
rh
er s
e
e
fri
at
a
n g e n s Vi 6 V, S T O br h S al o ut i o o D, t xi br p ur F w n; e ar s
at t i a
a â
er s s ks
h €“
s u u
a 4
e e s
e d
af i n u
ma
o v al
o- ys
o a ly
ly
d si a
Pa g e 9 5 2
ABC Ambe r CHM Conve rte r Tria l ve rsion, http://w w w .proce sste x t.com/a bcchm.html
ti
o ss
c
n oc
u
ia
s
te d wi th in g e st io n of o ys te rs , cl a m s, a n d cr a b
s S 8 V, F Mi Di al â S o l d ar m €“ D, o t i rh o 4 F, d s s e n 8 P,
u a
Pa g e 9 5 3
ABC Ambe r CHM Conve rte r Tria l ve rsion, http://w w w .proce sste x t.com/a bcchm.html
el h H,
e wi
la o sy
in th
s ur s t
v lo
p s e
a w
ec
m
s i -g
ie
ic
o ra
s
di
n; d
s
p e
e
o fe
a
ss v
s
i b er
e
le ; t o ca xi rri n er st at e p o ss ib le ; s h o ul d n ot b e tr e
Pa g e 9 5 4
ABC Ambe r CHM Conve rte r Tria l ve rsion, http://w w w .proce sste x t.com/a bcchm.html
at e d P 1 S F T Tr at â D o o a h €“
o xi v
o 3
d n; el
g d
a t i er
e a
n s s 's
ni ys
d u di
c
w e ar
E
at i n rh
sc
er v e
h
a a;
er
s i pr
ic
o o
hi
n p
a
h
c
yl
ol
a
i
xi s or th er a p y wi th q ui n ol o
Pa g e 9 5 5
ABC Ambe r CHM Conve rte r Tria l ve rsion, http://w w w .proce sste x t.com/a bcchm.html
n e, a zi th ro m yc in , or ri f a xi m in H 1 S R T S e â P, a o e m €“ S w xi v or 7 D, b n; er rh d B e t i e a a
ef s s p
gi ys
; u ai
c
u e n
E.
n in a
c
pr v n
ol
oc a d
i,
e s i bl
s
ss o o
er
e n o
ot
d
d
y
m
y
p
il
di
e
k;
ar
O
w
rh
Pa g e 9 5 6
ABC Ambe r CHM Conve rte r Tria l ve rsion, http://w w w .proce sste x t.com/a bcchm.html
1
at
e
5
er
a;
7:
m
H
a
7
y b e a ss oc ia te d wi th h e m ol yt ic â €“ ur e m ic sy n dr o m e or T
Pa g e 9 5 7
ABC Ambe r CHM Conve rte r Tria l ve rsion, http://w w w .proce sste x t.com/a bcchm.html
T P; hi g h m or ta lit y ra te ; th er a p y is s u p p or ti v e; a nt ib io ti cs n ot
Pa g e 9 5 8
ABC Ambe r CHM Conve rte r Tria l ve rsion, http://w w w .proce sste x t.com/a bcchm.html
g e n er al ly re co m m e n d e d Vi 1 V, P T Li br â S o o fe i o €“ D or xi -t c 3
h n hr
h d
y
e
ol a
gi
at
er ys
e
e
a
n
ni
e
e
n g di ar rh e a; th er a p y
Pa g e 9 5 9
ABC Ambe r CHM Conve rte r Tria l ve rsion, http://w w w .proce sste x t.com/a bcchm.html
is in tr a v e n o u s re pl ac e m e nt of fl ui d a n d el ec tr ol yt e s; e pi d e
Pa g e 9 6 0
ABC Ambe r CHM Conve rte r Tria l ve rsion, http://w w w .proce sste x t.com/a bcchm.html
m ic oc cu rr e nc e S 1 S F T T hi â D, ec o h g €“ F, al xi er el 3 B â n; a la d
€“ t i p
s a
or s s y
p ys
al u wi
ec
s e th
ie
pr i n ci
s
e v pr
(
a a of
m
d; s i l o
il
fl i o x
d
e n ac
ca
s
in
s
if
e
st
s)
o ol cu lt ur e is p o si
Pa g e 9 6 1
ABC Ambe r CHM Conve rte r Tria l ve rsion, http://w w w .proce sste x t.com/a bcchm.html
ti v e or s e v er e in fe ct io n; g e n er al ly s el fli m it e d Cl 1 V, C T S o â H, a o e s t €“ R n xi v ri 4 E n n er di d
e
e
u a
d
C
m ys
fo
N
Pa g e 9 6 2
ABC Ambe r CHM Conve rte r Tria l ve rsion, http://w w w .proce sste x t.com/a bcchm.html
b
o
S
ot
d
sy
ul
s
m
in
pt
u
o
m
m s; v e nt il at or y s u p p or t n e e d e d; hi g h m or ta lit y ra
Pa g e 9 6 3
ABC Ambe r CHM Conve rte r Tria l ve rsion, http://w w w .proce sste x t.com/a bcchm.html
te C 2 S F Ti M a â D, ec s s o m €“ B al u s t p 8
â e co
yl d
€“ i n m
o a
or v m
b ys
al a o
a
s si n
ct
pr o ca
er
e n u
je
a
s
ju
d;
e
ni
p
of
et
b
s
ac te ri al di ar rh e a in th e U ni te d St at e s.
Pa g e 9 6 4
ABC Ambe r CHM Conve rte r Tria l ve rsion, http://w w w .proce sste x t.com/a bcchm.html
T h er a p y wi th er yt hr o m yc in fo r s e v er e ca s e s; m o st ca s e s d o
Pa g e 9 6 5
ABC Ambe r CHM Conve rte r Tria l ve rsion, http://w w w .proce sste x t.com/a bcchm.html
n ot re q ui re a nt ib io ti cs Cl ? S F T P o
D, ec o o
st
F, al xi s t
ri
B â n a
di
€“
nt
u
or
ib
m
al
io
di
s
ti
ffi
pr
c
ci
e
di
le
a
ar
d
rh e a; th er a p y wi th v a
Pa g e 9 6 6
ABC Ambe r CHM Conve rte r Tria l ve rsion, http://w w w .proce sste x t.com/a bcchm.html
nc o m yc in or m et ro ni d a z ol e Y 2 S F Ti M er 4 P, ec s s a s i â S al u y ni €“ D, â e b a 4 B €“ i n e e 8
or v s
nt h
al a e
er o
s si e
o ur
pr o n
c s
e n; wi
ol
a p th
iti
d; o p
c
p s s ol
a
et i b y s l e ar to th xi ri n ti s in
Pa g e 9 6 7
ABC Ambe r CHM Conve rte r Tria l ve rsion, http://w w w .proce sste x t.com/a bcchm.html
ch il dr e n; u s u al ly s el fli m it e d; a m in o gl yc o si d e s if s e v er e
Pa g e 9 6 8
ABC Ambe r CHM Conve rte r Tria l ve rsion, http://w w w .proce sste x t.com/a bcchm.html
Protozoa Gi u V W N M ar s ar at o o di u i a er n- s t a al bl
i n co
l a l y e;
v m
m1 M
a m
bl â or
si o
i a €“ S
v n
2 D
e ca
w <
mu
e P,
ec s
e bl
h e
ks o
a of
R at
ni w
a in
s at
n g
m er
g a
of b
e n
p or
1 d
at n
â fl
h e
€“ at
o in
4 ul
g fe
5 e
e ct
d nc
n io
a e
e u
ys
si s s di n ar ot rh co e ma pl i n et t h el e
Pa g e 9 6 9
ABC Ambe r CHM Conve rte r Tria l ve rsion, http://w w w .proce sste x t.com/a bcchm.html
y U u ni n te d d er St s t at o e o s; d th er a p y wi th m et ro ni d a z ol e u s u al ly is s uc ce ss fu
Pa g e 9 7 0
ABC Ambe r CHM Conve rte r Tria l ve rsion, http://w w w .proce sste x t.com/a bcchm.html
l Cr 7 S F In C y â D, ec t e o pt €“ P al rf m o 1
l y er m
s 0
co e o
p d
nt s nl
or a
a wi y
i d ys
m t h af
iu
i n i n fe
m
at t e ct e st s d in i fo al m o a m d b u or s n w or oc at pt o er i o m ; n pr s a o wi n m m d is ms e i n ec d g re p p t i at o o ie ol n nt s
s; ra re ly oc
Pa g e 9 7 1
ABC Ambe r CHM Conve rte r Tria l ve rsion, http://w w w .proce sste x t.com/a bcchm.html
cu rs in n or m al in di vi d u al s; n o ef fe ct iv e th er a p y is a v ai la bl e I s 7 F, F M T o d D, ec uc h
Pa g e 9 7 2
ABC Ambe r CHM Conve rte r Tria l ve rsion, http://w w w .proce sste x t.com/a bcchm.html
s a H, al o er p ys M l y s a or ?? â co al p a
€“ nt al y
b
S, a t e i s
el
D m ra t r
li
in ti i at o m e n et d in h fo s o o m pr d al i a l
m
n a -s d n ul w d fa at l a m er rg et e h b o o x w a el z ol e. R ar e in U ni te d St
Pa g e 9 7 3
ABC Ambe r CHM Conve rte r Tria l ve rsion, http://w w w .proce sste x t.com/a bcchm.html
at e s u nl ik e ot h er pr ot o z o al in fe ct io n s, e o si n o p hi li a m a y b
Pa g e 9 7 4
ABC Ambe r CHM Conve rte r Tria l ve rsion, http://w w w .proce sste x t.com/a bcchm.html
e pr e s e nt E 2- B, F P T nt 4 F, ec e h a w S, al n er m e D, l y et a o e P co ra p e ks v nt t i y b ?? ar a o i s a
ia m n m
hi
bl i n t h et
st
e at ro ro
ol
e u ni
yt
d g d
ic
fo h a
a
o mz d uc ol a o e n s or d al i o w la d at y o er er q ui n
ol Viruses R 2 V, F Ti S ot d D, o s s el a a F, o u fvi ys P d, e l i ru
w in m
Pa g e 9 7 5
ABC Ambe r CHM Conve rte r Tria l ve rsion, http://w w w .proce sste x t.com/a bcchm.html
s
at v i t er a e P s i d; er o s s n u o to p n xi p t o n or p
ti
er
v
s
e
o
tr
n
e at m e nt ; li v e or al v ac ci n e a v ai la bl
e N 2 F, F N S
Pa g e 9 7 6
ABC Ambe r CHM Conve rte r Tria l ve rsion, http://w w w .proce sste x t.com/a bcchm.html
or 4 D, o o el w â V, o ni fal €“ H d n l i k 2
v m
a 8
a it
g h
si e
e o
v d;
nt ur
e s
s
u p p or ti v e tr e at m e nt
M, mi l d; S, s evere; P, abdomi nal pai n; V, vomi t i ng; D, di arrhea; F, fever; B, bl ood i n s t ool ; H, headache; RE, res pi rat ory embarras s me nt ; CNS,
Pa g e 9 7 7
ABC Ambe r CHM Conve rte r Tria l ve rsion, http://w w w .proce sste x t.com/a bcchm.html
cent ral nervous s ys t em; TTP, t hrombot i c t hrombocyt op eni c purpura.
o
o
a. E. coli i s t he mos t common caus e of traveler' s diarrhea, whi ch i s t he oft en s evere di arrhea t hat occurs wi t hi n 2 weeks of a vi s i t t o a t ropi cal area. Travel er's di arrhea us ual l y i s s el f-l i mi t ed.
(1) A t oxi geni c, heat -l abi l e E. c ol i can act i vat e cAMP or cycl i c guanos i ne 3′,5′-monophos phat e (cGMP) t o caus e a s ecret ory di arrhea t hat may be bl oody. Speci es of Shi gel l a, Sal monel l a, and Campyl obac t er as wel l as E. hi s t ol yt i c a and G. l ambl i a are ot her known caus es of t ravel er's di arrhea.
(2) Tri met hopri m–s ul famet hoxaz ol e combi nat i ons , bi s mut h s ubs al i cyl at e, ri fa xi mi n, qui nol one ant i bi ot i cs , and oxyt et racycl i ne preparat i ons have been s hown t o be effect i ve i n prevent i on and t reat ment .
o
o
b. G. lamblia, a fl agel l at e prot ozoan, i s t he mos t
Pa g e 9 7 8
ABC Ambe r CHM Conve rte r Tria l ve rsion, http://w w w .proce sste x t.com/a bcchm.html
common caus e of water-borne infectious diarrhea i n t he Uni t ed St at es . Treat ment wi t h met roni dazol e or qui nacri ne us ual l y i s s ucces s ful . o
o
c. Viruses commonl y caus e acute self-limited diarrhea. Al t hough many di fferent vi rus es may caus e gas t roent eri t i s , caus es of vi ral gas t roent eri t i s t hat can be i dent i fi ed wi t h cert ai nt y are t he Norwalk agent (a parvovi rus ) and t he rotavirus.
o
o
d. Salmonella infection, or s al monel l os i s , may be hi ghl y vari abl e i n i t s pres ent at i on. Gas t roent eri t i s , t he mos t common form of s al monel l os i s , i s an acut e s el f-l i mi t ed di arrheal s yndrome. Ent eri c fever i s a s evere i l l nes s pri mari l y caus ed by Sal monel l a t yphi or Sal monel l a parat yphi but al s o i s s een wi t h i nfect i on by ot her t ypes of s al monel l a. Sal monel l a s ept i cemi a may be s een i n pat i ent s wi t h os t eomyel i t i s , mycot i c aneurys ms , or abs ces s es wi t h no evi dence or hi s t ory of gas t roi nt es t i nal di s eas e.
o
o
e. Shigella i nfect i on, or s hi gel l os i s , i s charact eri zed by acut e di arrhea wi t h fever and crampy abdomi nal pai n. If l eft unt reat ed, t he di s eas e progres s es t o a chroni c bl oody di arrhea wi t hout fever but wi t h wei ght l os s and debi l i t at i on, whi ch may l as t for weeks .
o
o
f. Campylobacter i nfect i on i s t he mos t common caus e of bact eri al di arrhea i n t he Uni t ed St at es .
Pa g e 9 7 9
ABC Ambe r CHM Conve rte r Tria l ve rsion, http://w w w .proce sste x t.com/a bcchm.html
Sympt oms i ncl ude di arrhea, whi ch may be bl oody, and fever. Di agnos i s i s made by pos i t i ve s t ool cul t ure but requi res s peci al medi a and handl i ng. Treat ment i s wi t h eryt hromyci n becaus e t he organi s m i s res i s t ant t o mos t ot her commonl y us ed ant i bi ot i cs . o
o
g. Cryptosporidium i nfect i on, or crypt os pori di os i s , i s a prot ozoal i nfect i on s een commonl y i n i mmunocompromi s ed pat i ent s s uch as t hos e wi t h AIDS. Thi s di arrheal s yndrome rarel y occurs i n normal i ndi vi dual s .
o
o
h. Isospora belli i nfect i on occurs s i mi l arl y t o crypt os pori di os i s . P.199
P.200
o
o
i. E. histolytica caus es bl oody dys ent ery wi t h fever.
o
o
j. Hemorrhagic E. coli serotype O157:H7 produces Shi ga t oxi n and caus es bl oody di arrhea.
5. Nonbacterial food poisoning o
o
a. Fish poisoning i s caus ed by i cht hyos arcot oxi ns (t oxi ns found i n t he fl es h of poi s onous fi s h).
Pa g e 9 8 0
ABC Ambe r CHM Conve rte r Tria l ve rsion, http://w w w .proce sste x t.com/a bcchm.html
(1) Ciguatera poisoning i s acqui red from cert ai n bot t om-dwel l i ng fi s h found i n t emperat e and t ropi cal coas t al zones . Inges t i on of s uch fi s h i s fol l owed i n 30 mi nut es t o 30 hours by naus ea, vomi t i ng, di arrhea, and pares t hes i a or numbnes s of t he l i ps , t ongue, and l i mbs . Treat ment i s s upport i ve.
(2) Scombroid poisoning i s acqui red from cert ai n fi s h s peci es (us ual l y t una, mackerel , bl uefi s h, herri ng, and boni t o) t hat are s us cept i bl e t o t he product i on of a heat -s t abl e t oxi n by t he act i on of Prot eus morgani i . Sympt oms res embl e t hos e of a hi s t ami ne react i on and i ncl ude fl us hi ng, headache, di zzi nes s , abdomi nal crampi ng, vomi t i ng, and di arrhea. The s ympt oms appear s oon aft er i nges t i on and l as t for 4–6 hours . Cooki ng does not i nact i vat e t he t oxi n once i t i s formed. Ant i hi s t ami nes may be us ed for s ympt omat i c rel i ef.
(3) T etraodon poisoning may res ul t from cons umpt i on of puffer fish. Thes e fi s h produce a neurot oxi n t ermed t et rodot oxi n, whi ch may caus e pares t hes i a of t he face and ext remi t i es wi t h naus ea, vomi t i ng, and di arrhea. Vent i l at or s upport may be neces s ary.
o
o
b. Mushroom poisoning occurs aft er i nges t i on of
Pa g e 9 8 1
ABC Ambe r CHM Conve rte r Tria l ve rsion, http://w w w .proce sste x t.com/a bcchm.html
any of t he 50 s peci es of mus hrooms known t o be t oxi c t o humans . Amani t a verna, Amani t a vi ros a, and Amani t a phal l oi des account for mos t cas es of mus hroom poi s oni ng. Naus ea, vomi t i ng, fever, di arrhea, and abdomi nal pai n devel op 6–24 hours aft er i nges t i on; 1–4 days l at er, hepat i c and renal i ns uffi ci ency may devel op wi t h s ubs equent coagul opat hy, heart fai l ure, convul s i ons , and coma. Mus hroom poi s oni ng has a mort al i t y rat e of 40%–90%.
(1) Diagnosis i s made by hi s t ory of i nges t i on or by det ect i on of mus hroom t oxi ns i n t he gas t ri c as pi rat e.
(2) T herapy i s s upport i ve. Thi oct i c aci d may be curat i ve and s houl d be admi ni s t ered prompt l y.
E. Malabsorption Malabsorption of food or nut ri ent s res ul t s from a defect at any s t ep of t he di ges t i ve proces s or i n any of t he organs t hat part i ci pat e i n normal di ges t i on. The cl i ni cal feat ures vary wi del y becaus e mal abs orpt i on may i nvol ve a s i ngl e nut ri ent or mul t i pl e nut ri ent s .
1. Etiology o
o
a. Maldigestion refers t o a defect ei t her i n i nt ral umi nal hydrol ys i s of t ri gl yceri des or i n mi cel l e format i on, whi ch res ul t s from t he fol l owi ng condi t i ons :
Pa g e 9 8 2
ABC Ambe r CHM Conve rte r Tria l ve rsion, http://w w w .proce sste x t.com/a bcchm.html
(1) Pancreatic insufficiency caus ed by chroni c pancreat i t i s , pancreat i c carci noma, or cys t i c fi bros i s
(2) Deficiency of conjugated bile salts becaus e of chol es t at i c or obs t ruct i ve l i ver di s eas e (e.g., chol angi ocarci noma)
(3) Bile salt deconjugation at t ri but abl e t o bact eri al overgrowt h i n bl i nd l oops (aft er Bi l l rot h II gas t rect omy) or i n jejunal di vert i cul a, or i n as s oci at i on wi t h ent erocol oni c fi s t ul ae or mot i l i t y di s orders (e.g., s cl eroderma, ps eudo-obs t ruct i on) P.201
(4) Inadequate mixing of gastric contents with bile salts and pancreatic enzymes as a res ul t of previ ous gas t ri c s urgery, es peci al l y Bi l l rot h II gas t rect omy
o
o
b. Intrinsic small bowel disease
(1) Celiac disease caus es fl at t eni ng of t he vi l l i and i nfl ammat ory cel l i nfi l t rat i on i n t he l ami na propri a (s ee IV E 4 c).
(2) Whipple' s disease, a s ys t emi c di s eas e
Pa g e 9 8 3
ABC Ambe r CHM Conve rte r Tria l ve rsion, http://w w w .proce sste x t.com/a bcchm.html
t hat i s i nfect i ous i n ori gi n, caus es mucos al damage and l ymphat i c obs t ruct i on (s ee IV E 4 d).
(3) Collagenous sprue refers t o t he depos i t i on of a col l agenous s ubs t ance i n t he l ami na propri a i n a pat i ent who ot herwi s e has t he cl i ni cal and hi s t ol ogi c feat ures of cel i ac di s eas e. Fi ft y percent of cas es of col l agenous s prue are res pons i ve t o s t eroi ds .
(6) Amyloidosis (ei t her pri mary or s econdary) may affect t he s mal l i nt es t i ne i n 70% of cas es by amyl oi d i nfi l t rat i on of t he s ubmucos a. Al t ered mot i l i t y t hat al l ows bact eri al overgrowt h al s o may cont ri but e t o mal abs orpt i on. Gas t roi nt es t i nal bl eedi ng (occul t ) may be obs erved i n approxi mat el y 25% of cas es . Di agnos i s i s made by bi ops y (us ual l y of a rect al val ve or s t omach) wi t h s peci al s t ai ns (e.g., Congo red). Treat ment pri mari l y i s s upport i ve. A t ri al of ant i bi ot i cs for bact eri al overgrowt h may be gi ven.
(7) Crohn' s disease may caus e mal abs orpt i on by mucos al damage, by mul t i pl e s t ri ct ures wi t h bact eri al overgrowt h, or as a res ul t of t he need for mul t i pl e bowel res ect i ons (s ee IV G).
o
o
c. Inadequate absorptive surface res ul t s from ext ens i ve s mal l bowel res ect i on, us ual l y for Crohn's di s eas e or vas cul ar compromi s e of t he s mal l
Pa g e 9 8 4
ABC Ambe r CHM Conve rte r Tria l ve rsion, http://w w w .proce sste x t.com/a bcchm.html
i nt es t i ne.
(1) Res ect i on of up t o 50% of t he s mal l i nt es t i ne i s wel l t ol erat ed i f t he remai ni ng bowel i s normal , and s urvi val i s pos s i bl e aft er more ext ens i ve res ect i on but requi res careful management . If t he proxi mal s mal l bowel i s res ect ed, cal ci um, fol i c aci d, and i ron may not be abs orbed; t et any may res ul t . If t he i l eum i s removed, bi l e aci d and vi t ami n B 1 2 abs orpt i on i s i mpai red great l y. Hepat i c dys funct i on, oxal at e ki dney s t ones , and i ncreas ed gas t ri c aci d s ecret i on are common compl i cat i ons of ext ens i ve bowel res ect i on. D-Lact i c aci dos i s i s a rare but l i fe-t hreat eni ng compl i cat i on.
(2) Ini t i al t herapy i ncl udes i nt ravenous fl ui d and el ect rol yt e admi ni s t rat i on or parent eral hyperal i ment at i on. Oral feedi ngs s houl d i ncl ude medi um-chai n t ri gl yceri de (MCT oi l ), fat -s ol ubl e vi t ami ns , and i ron. Int ramus cul ar vi t ami n B 1 2 i nject i ons and ant i di arrheal agent s (t o s l ow t he t rans i t t i me) may be needed. Chol es t yrami ne res i n may bi nd nonabs orbed bi l e s al t s and l es s en t he di arrhea; however, t hi s drug al s o depl et es t he t ot al body bi l e s al t pool and us ual l y i s not us ed i f more t han 100 cm of di s t al i l eum has been res ect ed.
o
o
d. Lymphatic obstruction
(1) Intestinal lymphangiectasia may be
Pa g e 9 8 5
ABC Ambe r CHM Conve rte r Tria l ve rsion, http://w w w .proce sste x t.com/a bcchm.html
pri mary (congeni t al ) or s econdary t o i nt es t i nal t ubercul os i s , W hi ppl e's di s eas e, t rauma, neopl as i a, or ret roperi t oneal fi bros i s . In advanced di s eas e, di l at ed l ymphat i c channel s rupt ure and l eak i nt o t he i nt es t i nal l umen, caus i ng l ymphopeni a, l ow s erum prot ei n l evel s , and mas s i ve peri pheral edema. Smal l bowel bi ops y reveal s t he charact eri s t i c di l at ed l ymphat i cs . Treat ment i s wi t h l ow-fat di et and MCT s uppl ement at i on.
(2) Intestinal lymphoma may mi mi c Crohn's di s eas e or adul t cel i ac di s eas e bot h cl i ni cal l y and radi ographi cal l y. Cl ues t o t he di fferent i al di agnos i s i ncl ude pers i s t ent fevers and a s hort durat i on of s ympt oms . Enl arged l ymph nodes or hepat os pl enomegal y may be found on phys i cal exami nat i on, and CT may s how enl arged ret roperi t oneal nodes . Di agnos i s oft en i s made onl y aft er s urgi cal bi ops y. Therapy i ncl udes l ocal res ect i on and radi at i on t herapy. Chemot herapy i s us ed for di s s emi nat ed di s eas e.
o
o
e. Multiple defects cont ri but e t o mal abs orpt i on i n t he fol l owi ng s et t i ngs :
(1) After gastrectomy. Mal abs orpt i on can res ul t aft er Bi l l rot h II gas t rect omy, when poor mi xi ng of gas t ri c cont ent s wi t h pancreat i c enzymes and s t as i s i n t he afferent l oop wi t h bact eri al overgrowt h are pres ent . Therapy i ncl udes s urgi cal correct i on of t he afferent
Pa g e 9 8 6
ABC Ambe r CHM Conve rte r Tria l ve rsion, http://w w w .proce sste x t.com/a bcchm.html
l oop and broad-s pect rum ant i bi ot i cs .
(2) Radiation enteritis. Thi s condi t i on i nt erferes wi t h t he bl ood s uppl y t o t he i nt es t i ne. Bact eri al overgrowt h al s o may occur s econdary t o t he radi at i on-i nduced i nt es t i nal s t ri ct ure. Lymphat i c obs t ruct i on due t o edema or fi bros i s al s o may be a part of t he s yndrome. P.202
(3) Diabetes mellitus. Al t ered gut mot i l i t y from di abet i c neuropat hy, bact eri al overgrowt h, and exocri ne pancreat i c i ns uffi ci ency al l have been i mpl i cat ed as mec hani s ms of di abet es mel l i t us –i nduced mal abs orpt i on.
o
o
f. Other causes of malabsorption
(1) Abetalipoproteinemia i s a rare di s eas e wi t h neurol ogi c mani fes t at i ons (e.g., at axi a, nys t agmus , i ncoordi nat i on, and ret i ni t i s pi gment os a), morphol ogi cal l y abnormal “s pi ny― red bl ood cel l s (RBCs ), l ow s erum chol es t erol and t ri gl yceri de l evel s , and l ow s erum bet al i poprot ei n l evel s . St eat orrhea occurs becaus e apoprot ei n B, whi ch i s neces s ary for normal chyl omi cron format i on, i s l acki ng i n i nt es t i nal cel l s . Fat i s found i n t he epi t hel i al cel l s on s mal l bowel bi ops y.
Pa g e 9 8 7
ABC Ambe r CHM Conve rte r Tria l ve rsion, http://w w w .proce sste x t.com/a bcchm.html
Treat ment wi t h MCT oi l bypas s es t he abs orpt i on defect . Fat -s ol ubl e vi t ami n s uppl ement at i on al s o may be requi red.
(2) Infections can caus e mal abs orpt i on and may be vi ral , bact eri al , or paras i t i c.
(a) Viral and bacterial enteritis may caus e t emporary mal abs orpt i on s econdary t o di s acchari das e defi ci ency, mucos al damage, or bot h.
(b) T ropical sprue i s an endemi c mal abs orpt i on di s order occurri ng i n t he t ropi cs . It i s t hought t o have an i nfect i ous et i ol ogy becaus e t ravel ers are s us cept i bl e t o t he di s eas e, and t reat ment wi t h t et racycl i ne us ual l y i s effect i ve.
(c) Ot her i nfect i ous caus es of mal abs orpt i on i ncl ude hookworm, t apeworm, s t rongyl oi di as i s , and Capi l l ari a phi l i ppi nens i s (roundworm) i nfect i on, whi ch are common out s i de t he Uni t ed St at es , and gi ardi as i s , whi ch i s rel at i vel y common i n t he Uni t ed St at es .
(3) Chronic intestinal ischemia may caus e mal abs orpt i on when t wo of t he t hree major i nt es t i nal ves s el s —t he cel i ac art ery, s uperi or mes ent eri c art ery, and i nferi or mes ent eri c
Pa g e 9 8 8
ABC Ambe r CHM Conve rte r Tria l ve rsion, http://w w w .proce sste x t.com/a bcchm.html
art ery—are occl uded (as can be s hown angi ographi cal l y). Sympt oms us ual l y are wei ght l os s and crampy pos t prandi al pai n, wi t h occas i onal bl oody di arrhea. Vas cul ar s urgery may res ul t i n i mprovement .
(4) Hypogammaglobulinemia may caus e mal abs orpt i on, es peci al l y i f i t i s as s oci at ed wi t h s erum i mmunogl obul i n (Ig)A, s erum IgG, or i nt es t i nal IgA defi ci ency. Int es t i nal and s erum IgM al s o may be i ncreas ed. Smal l bowel bi ops y s hows abs ence of pl as ma cel l s . Nodul ar l ymphoi d hyperpl as i a may be s een i n t he di s t al s mal l bowel on radi ograph, and G. l ambl i a may be det ect ed i n s t ool or duodenal as pi rat e. There i s an i ncreas ed ri s k of i nt es t i nal l ymphomas and gas t ri c cancer i n affect ed pat i ent s . Treat ment of gi ardi as i s wi t h met roni dazol e or qui nacri ne may i mprove abs orpt i on. Common vari abl e i mmunodefi ci ency i s t reat ed wi t h i nt ravenous gammagl obul i n, whi ch may res ul t i n a decreas e i n i nt es t i nal i nfect i ons .
(5) Metastatic carcinoid syndrome i s as s oci at ed wi t h an i ncreas ed product i on of 5-hydroxyt rypt ami ne (s erot oni n), whi ch caus es i ncreas ed gas t roi nt es t i nal mot i l i t y. Met hys ergi de, cyprohept adi ne, or s omat os t at i n may be us ed for t herapy i f s urgi cal res ect i on i s not pos s i bl e.
(6) Hypoparathyroidism may mani fes t as
Pa g e 9 8 9
ABC Ambe r CHM Conve rte r Tria l ve rsion, http://w w w .proce sste x t.com/a bcchm.html
s t eat orrhea. The mechani s m by whi ch parat hormone affect s fat abs orpt i on i s unknown, but vi t ami n D–dependent cal ci um abs orpt i on may pl ay a rol e.
(7) Drugs t hat may caus e mal abs orpt i on i ncl ude neomyci n, kanamyci n, and baci t raci n. Phenyt oi n caus es a s el ect i ve fol i c aci d mal abs orpt i on.
2. Clinical features are vari abl e. Pat i ent s may pres ent wi t h s ome or al l of t he fol l owi ng cl i ni cal mani fes t at i ons : o
o
a. Pas s age of abnormal s t ool s , whi ch are greas y, s oft , bul ky, and foul s mel l i ng and may fl oat i n t he t oi l et becaus e of t hei r i ncreas ed gas cont ent ; a fi l m of greas e or oi l dropl et s may be s een on t he s urface of t he wat er
o
o
b. W ei ght l os s , whi ch may be s evere and i nvol ve marked mus cl e was t i ng
o
o
c. Edema and as ci t es s econdary t o hypoal bumi nemi a
o
o
d. Anemi a s econdary t o al t ered abs orpt i on of i ron, vi t ami n B 1 2 , fol at e, or a combi nat i on of t hes e
o
o
e. Bone pai n or fract ures from vi t ami n D defi ci ency
o
Pa g e 9 9 0
ABC Ambe r CHM Conve rte r Tria l ve rsion, http://w w w .proce sste x t.com/a bcchm.html
o
f. Pares t hes i as or t et any from cal ci um defi ci ency
o
o
g. Bl eedi ng from vi t ami n K defi ci ency
3. Diagnosis i s bas ed on cl i ni cal evi dence, wi t h confi rmat i on by l aborat ory t es t s . o
o
a. Stool fat analysis. Thi s t es t may be qual i t at i ve or quant i t at i ve. A pos i t i ve Sudan s t ai n i ndi cat es excret i on of great er t han 15 g fat per day i n t he s t ool . A 72-hour fecal fat col l ect i on can be us ed t o quant i fy t he amount of fat abs orpt i on. Normal l y, an i ndi vi dual abs orbs 93%–95% of al l di et ary fat i nges t ed. Pancreat i c di s eas e oft en i s as s oci at ed wi t h fecal fat excret i on i n exces s of 20–30 g/day on a di et of 100 g/day of fat .
o
o
b. D-Xylose absorption testing. Becaus e D-xyl os e, a fi ve-carbon s ugar, does not requi re enzymat i c degradat i on or mi cel l e format i on for abs orpt i on, i t can be us ed t o meas ure i nt es t i nal mucos al i nt egri t y. Aft er a 25-g oral dos e, a 5-hour uri ne col l ect i on s houl d cont ai n at l eas t 4–5 g D-xyl os e. Al t ernat i vel y, a 2-hour s erum s ampl e may be us ed.
o
o
c. T esting for unabsorbed carbohydrate. Lowered s t ool pH occurs when unabs orbed carbohydrat es reach t he col on and bact eri al ferment at i on occurs . Thi s i s part i cul arl y common i n l act as e defi ci ency but al s o may be s een i n cel i ac di s eas e and s hort bowel s yndrome.
Pa g e 9 9 1
ABC Ambe r CHM Conve rte r Tria l ve rsion, http://w w w .proce sste x t.com/a bcchm.html o
o
d. Pancreatic function testing. Tes t i ng i nvol ves meas uri ng t he bi carbonat e and t ot al fl ui d out put from t he duodenum aft er s ecret i n s t i mul at i on. Meas uri ng fecal el as t as e can al s o be hel pful for eval uat i on pancreat i c exocri ne funct i on. It i s s ens i t i ve, s peci fi c, and i ndependent of pancreat i c enzyme repl acement t herapy.
o
o
e. Measurement of serum carotene levels. Becaus e vi t ami n A i s fat -s ol ubl e and t he s erum carot ene l evel i s a refl ect i on of vi t ami n A met abol i s m, a l ow s erum carot ene l evel wi t h normal vi t ami n A i nt ake may be a us eful s creeni ng t es t for fat mal abs orpt i on.
o
o
f. Bacterial overgrowth testing
(1) Direct culture of jejunal as pi rat es 5
3
yi el di ng great er t han 10 organi s ms /mm of as pi rat e i s cons i dered abnormal . The di agnos i s i s s t rongl y s ugges t ed by t he pres ence of fas t i di ous anaerobes (cl os t ri di a and bact eroi des ), facul t at i ve anaerobes (l act obaci l l i and ent erococci ), or col i forms .
14
(2) [ C]–D-xylose breath tests may be us eful . Bact eri al act i on on xyl os e rel eas es t he radi oact i ve i s ot ope,
14
CO 2 , whi ch, aft er
abs orpt i on can be det ect ed i n breat h s ampl es . Sens i t i vi t y and s peci fi ci t y approach 90%. Radi at i on expos ure i s s mal l .
Pa g e 9 9 2
ABC Ambe r CHM Conve rte r Tria l ve rsion, http://w w w .proce sste x t.com/a bcchm.html
(3) Measurement of tryptophan metabolites. El evat ed uri nary l evel s of i ndi can and 5-hydroxyi ndol eacet i c aci d are caus ed by i ncreas ed met abol i s m of t rypt ophan. Thi s t es t i s not s peci fi c, however, becaus e abnormal l evel s of t rypt ophan met abol i t es al s o are obt ai ned i n carci noi d s yndrome and W hi ppl e's di s eas e.
o
o
g. Small bowel radiography. Thi s procedure may be us eful , es peci al l y us i ng t he i nt ubat ed ai r cont ras t t echni que. Pool i ng or fl occul at i on of bari um does not occur as frequent l y as wi t h previ ous bari um t echni ques but , i f not ed, s ugges t s cel i ac di s eas e. Thi ck fol ds may be s een i n W hi ppl e's di s eas e, l ymphoma, amyl oi dos i s , radi at i on ent eri t i s , Zol l i nger-El l i s on s yndrome, and eos i nophi l i c ent eri t i s , and a ps eudo-W hi ppl e's appearance i s obs erved i n pat i ent s wi t h AIDS ent eropat hy.
o
o
h. Schilling test. Thi s t echni que i s us ed t o di agnos e vi t ami n B 1 2 mal abs orpt i on.
(1) An abnormal fi rs t -s t age Schi l l i ng t es t (admi ni s t rat i on of radi ol abel ed vi t ami n B 1 2 onl y) wi t h a normal s econd-s t age Schi l l i ng t es t (admi ni s t rat i on of a compl ex of vi t ami n B 12
and i nt ri ns i c fact or) i ndi cat es gas t ri c
defect s s uch as perni ci ous anemi a and l ack of i nt ri ns i c fact or caus ed by gas t ri c res ect i on.
Pa g e 9 9 3
ABC Ambe r CHM Conve rte r Tria l ve rsion, http://w w w .proce sste x t.com/a bcchm.html
The s econd-s t age t es t may be abnormal becaus e of bact eri al overgrowt h or res ect i on or i nfl ammat i on of t he t ermi nal i l eum, whi ch i s t he s i t e of abs orpt i on. Severe cel i ac di s eas e al s o may caus e an abnormal s econd-s t age t es t .
(2) In s evere pancreat i c i ns uffi ci ency, pancreat i c prot eas es are not pres ent i n s uffi ci ent quant i t y t o cl eave B 1 2 from gas t ri n R prot ei ns . In t he t hi rd s t age of t he t es t , l abel ed B 1 2 abs orpt i on i s i mproved aft er pancreat i c enzymes are gi ven oral l y t o t he pat i ent at t he t i me of t he t es t i ng procedure.
o
o
i. Small bowel biopsy. Thi s procedure i s es s ent i al for t he di agnos i s of many cas es of mal abs orpt i on. In properl y prepared s peci mens , normal vi l l ous crypt rat i os are 3:1 or 4:1. Fl at t eni ng of t he vi l l i wi t h i nfl ammat ory cel l i nfi l t rat i on i s charact eri s t i c of cel i ac di s eas e, but fl at t ened vi l l i al one may be s een i n i nfect i ous ent eri t i s , gi ardi as i s , l ymphoma, and bact eri al overgrowt h.
4. Characteristics and management of specific causes of malabsorption o
o
a. Pancreatic insufficiency may be caus ed by chroni c pancreat i t i s , pancreat i c carci noma, or cys t i c fi bros i s . Treat ment i s pancreat i c enzyme repl acement . H 2 -recept or ant agoni s t s may i ncreas e t he pot ency of enzymes when gi ven 1 hour before
Pa g e 9 9 4
ABC Ambe r CHM Conve rte r Tria l ve rsion, http://w w w .proce sste x t.com/a bcchm.html
meal s becaus e aci d may i nact i vat e t he exogenous enzymes . o
o
b. Bacterial overgrowth may be at t ri but abl e t o al t ered mot i l i t y (e.g., i n di abet es , amyl oi dos i s , i nt es t i nal ps eudo-obs t ruct i on), s mal l bowel di vert i cul a, s t ri ct ures (e.g., i n l ymphoma and P.203
Crohn's di s eas e), or bl i nd l oops aft er Bi l l rot h II gas t rect omy. Surgi cal correct i on of t he anat omi c probl ems may be cons i dered, but t reat ment wi t h ant i bi ot i cs frequent l y i s s ucces s ful . Ampi ci l l i n, amoxi ci l l i n and cl avul anat e pot as s i um, t et racycl i ne, or chl orampheni col may be us ed. Some pat i ent s requi re cont i nuous t herapy, and i n t hes e cas es , ant i bi ot i cs s houl d be rot at ed. o
o
c. Celiac disease (nontropical sprue)
(1) Al t hough t he etiology and pathogenesis are not ful l y unders t ood, i t i s cl ear t hat an abnormal s ens i t i vi t y t o gl ut en, a prot ei n component of wheat , caus es damage t o t he i nt es t i nal mucos a of t hes e pat i ent s . The i mport ance of genet i c fact ors i s demons t rat ed by abnormal s mal l bowel bi ops i es i n 10%–15% of fi rs t -degree rel at i ves of pat i ent s . The i nci dence may be as hi gh as 1 i n 120 t o 1 i n 150 of t he U.S. popul at i on. Cel i ac pat i ent s have proxi mal i nt es t i nal i nvol vement wi t h a rel at i ve s pari ng of t he di s t al i l eum.
Pa g e 9 9 5
ABC Ambe r CHM Conve rte r Tria l ve rsion, http://w w w .proce sste x t.com/a bcchm.html
(2) Clinical features i ncl ude di arrhea, s t eat orrhea, wei ght l os s , and abdomi nal bl oat i ng. Sympt oms may begi n i n chi l dhood and t hen l es s en, onl y t o reappear i n t he t hi rd t o s i xt h decades of l i fe. Onl y 30%–40% of adul t pat i ent s pres ent wi t h t he cl as s i c s ympt oms of di arrhea, abdomi nal bl oat i ng, wei ght l os s , and s t eat orrhea. Mos t of t he pat i ent s have i ns i di ous pres ent at i ons , i ncl udi ng i nfert i l i t y, abnormal l i ver funct i on t es t s , os t eoporos i s , anemi a, aut oi mmune di s eas es s uch as t ype I di abet es mel l i t us , rheumat oi d art hri t i s or s ys t emi c l upus eryt hemat os us . Iron defi ci ency anemi a unas s oci at ed wi t h gas t roi nt es t i nal bl ood l os s may be s een becaus e i ron i s preferent i al l y abs orbed i n t he more s everel y i nvol ved proxi mal s mal l i nt es t i ne. Unexpl ai ned cent ral nervous s ys t em (CNS) s ympt oms s uch as depres s i on, neurol ogi c defi ci t , or even s ei zures may be more common i n cel i ac pat i ent s .
(3) Diagnosis requi res s mal l bowel bi ops y, whi ch s hows vi l l ous at rophy, crypt hypert rophy, and cuboi dal change i n t he epi t hel i al cel l s . Infl ammat ory cel l i nfi l t rat i on i n t he l ami na propri a at endos copy may s how “s cal l oped fol ds ― i n t he proxi mal duodenum. Ant i -t i s s ue t rans gl ut ami nas e ant i body (IgA) i s current l y t he di agnos t i c ant i body of choi ce wi t h t he hi ghes t s ens i t i vi t y and s peci fi ci t y for cel i ac di s eas e. Al l
Pa g e 9 9 6
ABC Ambe r CHM Conve rte r Tria l ve rsion, http://w w w .proce sste x t.com/a bcchm.html
ant i bodi es may decreas e or di s appear wi t h t reat ment . Becaus e approxi mat el y 5%–7% of pat i ent s wi t h cel i ac di s eas e may be IgA defi ci ent , a t ot al IgA l evel may be neces s ary i f one i s rel yi ng on IgA cel i ac ant i bodi es for di agnos t i c purpos es . Ant i -endomys i al ant i bodi es are l es s s ens i t i ve but more s peci fi c.
(4) T herapy i s bas ed on wi t hdrawal of gl ut en from t he di et by el i mi nat i ng wheat , rye, barl ey, and oat s . Al t hough cl i ni cal res pons e t o gl ut en wi t hdrawal oft en i s dramat i c and may be s een i n a few days , hi s t ol ogi c recovery demons t rat ed on repeat s mal l bowel bi ops y may be del ayed for mont hs and, i n up t o 50% of pat i ent s , may never be demons t rat ed. In s everel y i l l pat i ent s , s t eroi ds may be of s hort -t erm benefi t .
(5) Complications
(a) Lymphoma or carcinoma
(b) Intestinal ulcers or strictures
(c) Dermatitis herpetiformis
(d) Collagenous sprue
Pa g e 9 9 7
ABC Ambe r CHM Conve rte r Tria l ve rsion, http://w w w .proce sste x t.com/a bcchm.html
(e) Increased incidence of autoimmune diseases
o
o
d. Whipple' s disease i s a s ys t emi c di s order mos t commonl y occurri ng i n mi ddl e-aged men.
(1) Etiology and pathogenesis. Numerous s mal l gram-pos i t i ve cocci are s een i n macrophages i n i ndi vi dual organs , and t he organi s m T ropheryma w hi ppel i i has been i s ol at ed as t he i nfect i ous agent . Mal abs orpt i on i s caus ed by mucos al damage and l ymphat i c obs t ruct i on.
(2) Clinical features depend on organ i nvol vement . In t he i nt es t i ne, peri odi c aci d–Schi ff (PAS)–pos i t i ve macrophages (i .e., macrophages t hat cont ai n baci l l i ) are found i n t he l ami na propri a. The mes ent eri c l ymph nodes , heart , s pl een, l ungs , and CNS al s o may be i nvol ved. Fever occurs i n one t hi rd t o onehal f of pat i ent s . Art hral gi a and art hri t i s are pres ent i n 60% of pat i ent s and may precede t he gas t roi nt es t i nal s ympt oms .
(3) Diagnosis i s made by i nt es t i nal bi ops y wi t h a PAS s t ai n. A s mal l bowel radi ograph may s how t hi ckened fol ds . In rare cas es , t he di s eas e i s focal and t he bi ops y s peci men i s normal .
(4) T herapy wi t h peni ci l l i n, ampi ci l l i n, or
Pa g e 9 9 8
ABC Ambe r CHM Conve rte r Tria l ve rsion, http://w w w .proce sste x t.com/a bcchm.html
t et racycl i ne i s requi red for at l eas t 4–6 mont hs and may be cont i nued i nt ermi t t ent l y (i .e., every ot her day) t hereaft er. The rel aps e rat e i s approxi mat el y 10%. P.204
F. Protein-losing enteropathy (PLE) Protein-losing enteropathy (PLE) refers t o t he exces s i ve l os s of s erum prot ei ns i nt o t he gas t roi nt es t i nal t ract . Three t ypes of di s orders may caus e PLE.
1. Mucosal ulceration caus es l eakage of prot ei n at t he ul cer s i t e and res ul t s from t he fol l owi ng condi t i ons : o
o
a. Mal i gnant di s eas e i nvol vi ng t he gas t roi nt es t i nal t ract
o
o
b. Mul t i pl e pept i c ul cers
o
o
c. Nongranul omat ous i l eojejuni t i s
2. Mucosal disease without ulceration but wi t h al t ered met abol i s m or cel l t urnover l eads t o i ncreas ed permeabi l i t y t o prot ei n. Such di s eas es i ncl ude: o
o
a. Ménét ri er's di s eas e (gi ant hypert rophi c gas t ri t i s )
Pa g e 9 9 9
ABC Ambe r CHM Conve rte r Tria l ve rsion, http://w w w .proce sste x t.com/a bcchm.html o
o
b. Cel i ac di s eas e
o
o
c. W hi ppl e's di s eas e
o
o
d. Infect i ous ent eri t i s
3. Lymph flow obstruction caus es i ncreas ed l ymphat i c pres s ure and prot ei n l eakage. Lymphat i c obs t ruct i on res ul t s from t he fol l owi ng condi t i ons : o
o
a. Lymphoma
o
o
b. Int es t i nal l ymphangi ect as i a
o
o
c. Cardi ac di s eas e s uch as cons t ri ct i ve peri cardi t i s and t ri cus pi d val ve di s eas e
o
o
d. C. phi l i ppi nens i s i nfect i on
G. Crohn's disease (regional enteritis) Crohn' s disease (regional enteritis) i s a chroni c granul omat ous di s eas e t hat may occur anywhere i n t he gas t roi nt es t i nal t ract from t he mout h t o t he anus . The i l eum mos t oft en i s i nvol ved, wi t h i l eocol i t i s i n more t han 50% of pat i ent s . The fi rs t peak of i nci dence occurs bet ween t he ages of 12 and 30 years ; a s econdary peak occurs at age 50 years .
1. Etiology
Pa g e 1 0 0 0
ABC Ambe r CHM Conve rte r Tria l ve rsion, http://w w w .proce sste x t.com/a bcchm.html o
o
a. Genetic factors appear t o pl ay a rol e, wi t h an i ncreas ed i nci dence of di s eas e not ed i n monoz ygot i c t wi ns and s i bl i ngs . Approxi mat el y 17% of pat i ent s wi t h Crohn's di s eas e have fi rs t -degree rel at i ves wi t h t he di s eas e. Men are affect ed more oft en t han women, and t he di s eas e i s more common among Jews . Compared wi t h t he general popul at i on, Jewi s h men have s i x t i mes t he ri s k of Crohn's di s eas e. An abnormal i t y i n chromos ome 16—t he NOD-2 gene—has been as s oci at ed wi t h fi bros t enos i ng s mal l bowel Crohn's di s eas e.
o
o
b. An immunologic mechanism i s t he mos t promi nent t heory. Abnormal numbers , s ubs et s , and funct i ons of T cel l s have been i dent i fi ed i n cas es of Crohn's di s eas e. Hyperres pons i ve i mmune funct i on, rat her t han abnormal funct i on, i s charact eri s t i c of an unres t ri ct ed res pons e t o i nfl ammat i on. Recent l y, a defect i n t he abi l i t y of i nt es t i nal epi t hel i al cel l s t o produce normal amount s of s uppres s or T (Ts ) cel l s (CD8 cel l s ) has been not ed. Lumi nal bact eri a or gut i nfect i on may i ni t i at e or propagat e t hi s res pons e.
2. Pathologic features o
o
a. Marked t hi ckeni ng of t he i nvol ved i nt es t i nal wal l wi t h t rans mural i nfl ammat i on
o
o
b. Enl arged and mat t ed mes ent eri c l ymph nodes
o
Pa g e 1 0 0 1
ABC Ambe r CHM Conve rte r Tria l ve rsion, http://w w w .proce sste x t.com/a bcchm.html
o
c. Focal granul omas i n 50% of s peci mens
o
o
d. Deep s erpi gi nous or l i near ul cerat i ons l eadi ng t o cobbl es t oni ng and fi s t ul a format i on
o
o
e. St ri ct ure format i on s econdary t o s carri ng
o
o
f. Al t ernat i ng areas of normal and i nvol ved mucos a
3. Clinical features are charact eri zed by peri odi c exacerbat i ons and remi s s i ons . o
o
a. Pain oft en i s col i cky, es peci al l y i n t he l ower abdomen, and may be i ncreas ed aft er meal s becaus e of t he obs t ruct i ve nat ure of t he pat hol ogi c proces s .
o
o
b. Systemic symptoms are common and i ncl ude fever, wei ght l os s , mal ai s e, and anorexi a.
o
o
c. Diarrhea i s t he us ual pres ent i ng s ympt om.
o
o
d. Intestinal obstruction i s t he pres ent i ng s ympt om i n approxi mat el y 25% of cas es .
o
o
e. Extraintestinal manifestations are numerous .
Pa g e 1 0 0 2
ABC Ambe r CHM Conve rte r Tria l ve rsion, http://w w w .proce sste x t.com/a bcchm.html
(1) Anemi a as wel l as growt h or s exual ret ardat i on probabl y i s at t ri but abl e t o i nadequat e cal ori c i nt ake.
(2) Hepat obi l i ary di s orders i ncl ude fat t y l i ver, peri chol angi t i s , nons peci fi c hepat i t i s , ci rrhos i s , and s cl eros i ng chol angi t i s . There i s an i ncreas ed ri s k of gal l s t ones . Li ver enzyme or l i ver bi ops y abnormal i t i es occur i n 50%â €“70% of pat i ent s wi t h Crohn's di s eas e.
(3) Renal di s orders i ncl ude ri ght uret eral obs t ruct i on s econdary t o cont i guous bowel i nvol vement and nephrol i t hi as i s . An i ncreas e i n cal ci um oxal at e s t ones i s caus ed by i ncreas ed oxal at e abs orpt i on, and an i ncreas e i n uri c aci d s t ones i s as cri bed t o i ncreas ed cel l t urnover and concent rat ed aci d uri ne.
(4) Peri pheral art hri t i s occurs i n 10%–12% of pat i ent s and ankyl os i ng s pondyl i t i s i n 2%–10%.
(5) Ski n probl ems i ncl ude eryt hema nodos um and, rarel y, pyoderma gangrenos um.
(6) Epi s cl eri t i s or uvei t i s may occur i n 3%–10% of pat i ent s .
o
o
f. Fistulas t o t he s ki n or ot her organs occur i n
Pa g e 1 0 0 3
ABC Ambe r CHM Conve rte r Tria l ve rsion, http://w w w .proce sste x t.com/a bcchm.html
approxi mat el y 20% of pat i ent s . Peri anal fi s t ul as or abs ces s es are es peci al l y common i n Crohn's col i t i s . Peri anal s ki n t ags are charact eri s t i c of col oni c Crohn's di s eas e.
4. Diagnosis i s bas ed on cl i ni cal s i gns and s ympt oms combi ned wi t h charact eri s t i c radi ographi c fi ndi ngs , i ncl udi ng deep (col l ar but t on) ul cerat i ons , l ong s t ri ct ured s egment s (s t ri ng s i gn), and s ki p areas . Col onos copy may be hel pful when t here i s col oni c i nvol vement , and bi ops i es may s how granul omas . Laborat ory s t udi es are not s peci fi c but may s how mul t i fact ori al anemi a, l eukocyt os i s , an i ncreas ed s edi ment at i on rat e, and evi dence of mal abs orpt i on or prot ei n l os s . The di fferent i al di agnos i s i ncl udes l ymphoma, t ubercul os i s , radi at i on ent eri t i s , and Y ers i ni a i nfect i on (es peci al l y i n acut e ent eri t i s ). P.205
5. T herapy i s s ympt omat i c. No s peci fi c t herapy or cure exi s t s . o
o
a. Supportive measures i ncl ude s hort -t erm, broad-s pect rum ant i bi ot i cs ; ant i di arrheal agent s ; bowel res t wi t h i nt ravenous fl ui d s upport (i .e., not hi ng by mout h); ent eral nut ri t i on wi t h t ube feedi ngs ; t ot al parent eral nut ri t i on; and vi t ami n s uppl ement at i on.
o
o
b. Medical treatment
Pa g e 1 0 0 4
ABC Ambe r CHM Conve rte r Tria l ve rsion, http://w w w .proce sste x t.com/a bcchm.html
(1) Sulfasalazine or 5-ami nos al i cyl i c aci d preparat i ons i n dos es of 3–4 g/day may be us ed al one or i n combi nat i on wi t h cort i cos t eroi ds t o t reat acut e di s eas e. Thi s may be more effect i ve i n Crohn's col i t i s . Et hyl cel l ul os e–coat ed oral 5-ami nos al i cyl i c aci d preparat i ons may be us eful i n s mal l bowel di s eas e.
(2) Vitamin B 1 2 i nject i ons are i ndi cat ed when i l eal di s eas e caus es mal abs orpt i on of t hi s vi t ami n.
(3) Oral calcium, vitamin D, or both may be hel pful i n pat i ent s wi t h cal ci um oxal at e s t ones by bi ndi ng oxal at e i n t he bowel and decreas i ng uri nary oxal at e.
(4) Metronidazole, quinolone antibiotics, or both may be effect i ve i n t reat ment of peri neal and peri anal fi s t ul as .
(5) Corticosteroids have been proven effect i ve by t he Nat i onal Cooperat i ve Crohn's Di s eas e St udy, es peci al l y i n pat i ent s wi t h s mal l bowel di s eas e. W hen remi s s i on i s obt ai ned, t he dos age s houl d be t apered gradual l y. Mai nt enance s t eroi ds i n pat i ent s i n remi s s i on do not prevent rel aps e. Budes oni de, a cort i cos t eroi d t hat i s i nact i vat ed by fi rs t -pas s met abol i s m t hrough t he l i ver, mi ni mi zes s ys t emi c s i de effect s and i s
Pa g e 1 0 0 5
ABC Ambe r CHM Conve rte r Tria l ve rsion, http://w w w .proce sste x t.com/a bcchm.html
effect i ve for i l eal and ri ght col on di s eas e.
(6) 6-Mercaptopurine, azathioprine, and methotrexate al s o have been us ed and are effect i ve i n prol ongi ng and i nduci ng remi s s i on.
(7) Infliximab, a chi meri c ant i body t hat bi nds t umor necros i s fact or (TNF), an i mport ant proi nfl ammat ory cyt oki ne i n Crohn's di s eas e, i s an effect i ve t reat ment met hod. Admi ni s t rat i on of t hi s agent res ul t s i n a dramat i c decreas e i n Crohn's di s eas e act i vi t y i ndex (CDAI) i n t wo t hi rds of pat i ent s and remi s s i on i n one t hi rd of al l pat i ent s . Thes e res ul t s may be obs erved wi t hi n jus t 4–6 weeks of t herapy.
o
o
c. Surgery may be neces s ary for recurrent i nt es t i nal obs t ruct i on, ent erocut aneous fi s t ul as , and perforat i on, as wel l as for growt h ret ardat i on t hat does not res pond t o i ncreas ed cal ori c i nt ake. The recurrence rat e aft er i ni t i al res ect i on may be as hi gh as 80% wi t hi n 15 years , es peci al l y wi t h i ni t i al s mal l bowel i nvol vement .
H. Small bowel tumors
1. Malignant tumors of t he s mal l i nt es t i ne are rare and i ncl ude adenocarcinomas, carcinoid tumors, lymphomas, and leiomyosarcomas. (GISTs —s ee
II B 2 a)
o
o
a. Etiology and pathogenesis. Some s mal l bowel mal i gnanci es ari s e de novo, but many are rel at ed
Pa g e 1 0 0 6
ABC Ambe r CHM Conve rte r Tria l ve rsion, http://w w w .proce sste x t.com/a bcchm.html
t o underl yi ng condi t i ons s uch as Crohn's di s eas e and cel i ac di s eas e. Rarel y t hes e mal i gnanci es ari s e from t he pol yps of Peut z-Jeghers s yndrome, fami l i al pol ypos i s , and Gardner's s yndrome. A part i cul ar t ype of l ymphoma cal l ed Medi t erranean l ymphoma i s endemi c t o t he Mi ddl e Eas t . Pat i ent s wi t h cel i ac di s eas e may have a 10%–15% i nci dence of s mal l bowel mal i gnancy, us ual l y l ymphoma. o
o
b. Pathology. Adenocarci noma, t he mos t common s mal l bowel t umor, i s es peci al l y common i n t he proxi mal s mal l bowel ; l ymphomas and carci noi d t umors pri mari l y occur i n t he appendi x and i l eum. Smal l bowel t umors may be met as t at i c from t he breas t , ki dney, ovary, and t es t i s as wel l as from mel anoma.
o
o
c. Clinical features. Bl eedi ng, obs t ruct i on, and mal abs orpt i on may occur. Carci noi d t umors of t he appendi x may occur as acut e appendi ci t i s but us ual l y are as ympt omat i c. Rarel y, appendi ceal carci noi d t umors are met as t at i c t o t he l i ver and, i n s uch cas es , may caus e carci noi d s yndrome, whi ch i s charact eri zed by fl us hi ng and di arrhea.
o
o
d. T herapy and prognosis. Surgery i s t he t reat ment of choi ce, but t he prognos i s i s poor, es peci al l y for adenocarci nomas . Lymphomas and l ei omyos arcomas have a bet t er prognos i s i f t hey are l ocal i zed t o a s mal l s egment of t he bowel . Radi at i on t herapy and chemot herapy are us ed pos t operat i vel y t o t reat s ys t emi c l ymphomas .
Pa g e 1 0 0 7
ABC Ambe r CHM Conve rte r Tria l ve rsion, http://w w w .proce sste x t.com/a bcchm.html
Carci noi d t umors grow s l owl y, and pat i ent s may s urvi ve for many years even i f t he di s eas e i s met as t at i c. Foregut carci noi ds res pond t o s t rept ozoci n i n approxi mat el y 50% of cas es . Somat os t at i n oft en decreas es t he fl us hi ng and di arrhea i n met as t at i c carci noi d t umors .
2. Benign tumors of t he s mal l i nt es t i ne i ncl ude adenomas, lipomas, and leiomyomas, whi ch may be as s oci at ed wi t h obs t ruct i on or bl eedi ng but us ual l y are as ympt omat i c.
I. Acute appendicitis Acute appendicitis i s a common and curabl e caus e of an acut e abdomen. Appendi ci t i s devel ops at any age and i n bot h s exes but mos t oft en occurs i n mal es bet ween 10 and 30 years of age.
1. Pathogenesis. It i s bel i eved t hat t he pri mary event i s an obs t ruct i on of t he appendi ceal l umen by a fecal i t h, i nfl ammat i on, forei gn body, or neopl as m. Aft er obs t ruct i on of t he l umen, i ncreas ed i nt ral umi nal pres s ure and i nfect i on may caus e appendi ceal necros i s and perforat i on.
2. Clinical features. Appendi ci t i s i s charact eri zed by pai n i n t he ri ght l ower quadrant , whi ch i ni t i al l y i s vague but becomes l ocal i zed t o McBurney's poi nt , peri t oneal s i gns , fever, and l eukocyt os i s i n t he range of 3
10,000–20,000/mm . Rect al t endernes s i s common i n pel vi c appendi ci t i s , and ret rocecal appendi ci t i s caus es ps oas mus cl e pai n on hi p ext ens i on. Pat i ent s at t he ext remes of age, great l y obes e pat i ent s , and pat i ent s t aki ng cort i cos t eroi ds may have nons peci fi c compl ai nt s and a rel at i vel y beni gn phys i cal exami nat i on. A hi gh
Pa g e 1 0 0 8
ABC Ambe r CHM Conve rte r Tria l ve rsion, http://w w w .proce sste x t.com/a bcchm.html
i ndex of cl i ni cal s us pi ci on mus t be mai nt ai ned i n t hes e cas es . P.206
3. Diagnosis o
o
a. Differential diagnoses. Pos s i bl e di agnos es i ncl ude acut e gas t roent eri t i s , mes ent eri c adeni t i s , Meckel 's di vert i cul um, and Crohn's di s eas e. In young women, ovari an t ors i on, rupt ured ovari an cys t , and pel vi c i nfl ammat ory di s eas e (PID) s houl d be cons i dered. In el derl y pat i ent s , di vert i cul i t i s , chol ecys t i t i s , i ncarcerat ed herni a, cecal carci noma, and mes ent eri c t hrombos i s s houl d be rul ed out .
o
o
b. Diagnostic modalities. In di ffi cul t -t o-di agnos e cas es , a barium enema may be us ed t o i dent i fy l ack of fi l l i ng of t he appendi x. A CT scan s hows a mas s -l i ke effect i n t he ri ght l ower quadrant . On ultrasound, a cl as s i c “bul l 's -eye― appearance of t he ri ght l ower quadrant i s bel i eved t o be rel at i vel y di agnos t i c of appendi ci t i s .
4. T herapy. Surgery s houl d be performed as earl y as pos s i bl e t o prevent perforat i on.
V. Diseases of the Colon A. Constipation
Pa g e 1 0 0 9
ABC Ambe r CHM Conve rte r Tria l ve rsion, http://w w w .proce sste x t.com/a bcchm.html
1. Simple constipation i s t he res ul t of del ayed t rans i t of i nt es t i nal cont ent s . The hi ghl y refi ned, l ow-fi ber di et s of i ndus t ri al i zed nat i ons probabl y cont ri but e t o t hi s probl em. Al t hough t he epi demi ol ogi c defi ni t i on of cons t i pat i on i s l es s t han t hree s t ool s per week, i ndi vi dual di fferences exi s t . Treat ment of s i mpl e cons t i pat i on i s di rect ed t oward i ncreas i ng i nt es t i nal bul k by i ncreas i ng di et ary fi ber cont ent wi t h frui t s , veget abl es , and bul ki ng agent s s uch as ps yl l i um hydrophi l i c col l oi ds , whi ch t rap wat er and el ect rol yt es wi t hi n t he bowel l umen. Long-t erm us e of pot ent l axat i ves s houl d be avoi ded becaus e t hey may res ul t i n des t ruct i on of col oni c i nt ramural nerve pl exus es and cat hart i c col on.
2. Constipation may occur wi t h a vari et y of di s eas es , i ncl udi ng ul cerat i ve proct i t i s , rect al fi s s ures or abs ces s es , and rect al s t ri ct ures as wel l as t he vari ed caus es of di ffus el y decreas ed i nt es t i nal act i vi t y di s cus s ed i n IV A and B. Irri t abl e bowel s yndrome, whi ch may pres ent as ei t her cons t i pat i on or di arrhea, i s di s cus s ed i n IV D 3 a.
3. Colonic inertia i s a mot i l i t y di s order of t he col on charact eri zed by poor propul s i on and i neffect i ve mi xi ng. W hen radi opaque markers are gi ven t o pat i ent s , t hey oft en remai n i n t he col on for s everal days . Thes e markers are di s t ri but ed t hroughout t he col on, i ndi cat i ng poor t ot al col oni c mot i l i t y. Outlet obstruction us ual l y occurs when a probl em wi t h i nt ernal or ext ernal s phi nct eri c rel axat i on devel ops ; when Si t z-Marks s t udi es are gi ven t o affect ed pat i ent s , t he markers col l ect i n t he rect um before defecat i on.
B. Colonic diverticula
Pa g e 1 0 1 0
ABC Ambe r CHM Conve rte r Tria l ve rsion, http://w w w .proce sste x t.com/a bcchm.html
Colonic diverticula are out pouchi ngs of t he mucos a onl y and, t herefore, are not t rue di vert i cul a. In t he Uni t ed St at es , col oni c di vert i cul a occur i n approxi mat el y 50% of i ndi vi dual s ol der t han 60 years of age.
1. Pathogenesis. In i ndus t ri al i zed nat i ons , col oni c di vert i cul a have been l i nked t o l ow-fi ber di et s . Di et s l ow i n fi ber and bul k are t hought t o caus e an i ncreas ed i nt ral umi nal pres s ure, part i cul arl y i n t he narrow s i gmoi d col on. Event ual l y, t he i ncreas ed i nt ral umi nal pres s ure caus es a mucos al herni at i on at t he s i t e of a perforat i ng art eri ol e carryi ng bl ood from t he s eros al s urface t o t he mucos a.
2. Clinical features. Sympt oms oft en are abs ent i n uncompl i cat ed col oni c di vert i cul a; however, pat i ent s may compl ai n of crampy abdomi nal pai n i n t he l eft l ower quadrant , wi t h al t ernat i ng di arrhea and cons t i pat i on. Oft en, t here i s rel i ef of t hes e s ympt oms aft er a bowel movement .
3. T herapy. Treat ment i s ai med at i ncreas i ng s t ool bul k wi t h hi gh-fi ber foods and hydrophi l i c col l oi ds , t hereby decreas i ng i nt ral umi nal pres s ure. Thi s res ul t s i n s ympt omat i c i mprovement , probabl y by regul at i ng bowel frequency.
4. Complications o
o
a. Diverticulitis occurs i n approxi mat el y 15%–25% of pat i ent s wi t h di vert i cul os i s . General l y, t here i s a mi croperforat i on (rarel y a free
Pa g e 1 0 1 1
ABC Ambe r CHM Conve rte r Tria l ve rsion, http://w w w .proce sste x t.com/a bcchm.html
perforat i on) wi t h a peri -di vert i cul ar abs ces s . Sympt oms i ncl ude l eft l ower quadrant abdomi nal pai n, fever, and cons t i pat i on. Treat ment wi t h ant i bi ot i cs , i nt ravenous fl ui ds , and bowel res t i s effect i ve i n mos t cas es . Met roni dazol e, ci profl oxaci n, or ampi ci l l i n s odi um/s ul bact am s odi um i s us ed mos t commonl y. Severel y i l l or P.207
t oxi c pat i ent s may requi re addi t i onal ant i bi ot i cs for adequat e coverage of Ps eudomonas and anaerobes . If an abs ces s occurs , a fi s t ul a t o t he bl adder or vagi na may devel op. o
o
b. Bleeding occurs i n approxi mat el y 5%–15% of cas es and us ual l y i s bri s k, pai nl es s , and not as s oci at ed wi t h s t rai ni ng. In mos t cas es , bl eedi ng s t ops s pont aneous l y wi t h onl y s upport i ve t herapy. Art eri ography or rapi d-s equence nucl ear s canni ng us i ng t echnet i um s ul fur col l oi d or t echnet i um-l abel ed eryt hrocyt es may l ocal i ze t he bl eedi ng port i on of t he col on and al l ow s egment al s urgi cal res ect i on, i f neces s ary.
C. Hirschsprung's disease Hirschsprung' s disease i s a congeni t al caus e of megacol on. Thi s condi t i on occurs i n 1 of 5000 l i ve bi rt hs and i s more common i n mal es .
1. Etiology. Hi rs chs prung's di s eas e i s caus ed by i ncompl et e caudad mi grat i on of neural cres t cel l s , whi ch renders t he i nt ernal anal s phi nct er and a vari abl e
Pa g e 1 0 1 2
ABC Ambe r CHM Conve rte r Tria l ve rsion, http://w w w .proce sste x t.com/a bcchm.html
s egment of t he rect um and s i gmoi d wi t hout i nnervat i on. The i nvol ved s egment cons t i t ut es a funct i onal obs t ruct i on, and t he normal proxi mal col on becomes di l at ed.
2. Clinical features. The di s eas e may occur i n i nfancy as meconi um i l eus , i nt es t i nal obs t ruct i on, or s evere cons t i pat i on, or i t may devel op i n l at er l i fe wi t h mi l der s ympt oms .
3. Diagnosis. Phys i cal exami nat i on i ndi cat es t he abs ence of s t ool i n t he rect um, and bari um enema s hows a narrowed (di s eas ed) s egment wi t h a di l at ed proxi mal col on (normal s egment ) i n approxi mat el y 75% of cas es . Anal manomet ry i s a good s creeni ng t es t for t hi s di s eas e and i ndi cat es a l ack of refl ex rel axat i on i n t he i nt ernal anal s phi nct er on rect al di s t ent i on. The abs ence of gangl i on cel l s on a ful l -t hi cknes s rect al bi ops y i s di agnos t i c.
4. T herapy. Treat ment i s s urgi cal .
D. Ulcerative colitis Ulcerative colitis i s a chroni c i nfl ammat ory di s eas e of t he col oni c mucos a and s ubmucos a. Ul cerat i ve col i t i s and Crohn's di s eas e s hare s ome feat ures and, des pi t e t hei r di s s i mi l ari t i es , oft en are pl aced t oget her under t he generi c headi ng of i nfl ammat ory bowel di s eas e. Ul cerat i ve col i t i s occurs i n 2–7 of 100,000 i ndi vi dual s , and femal es are affect ed more commonl y t han mal es . The major pe ak of i nci dence occurs bet ween t he ages of 15 and 30 years , wi t h a l es s er peak bet ween t he ages of 50 and 65 years .
1. Etiology and pathogenesis are s i mi l ar t o t hos e of
Pa g e 1 0 1 3
ABC Ambe r CHM Conve rte r Tria l ve rsion, http://w w w .proce sste x t.com/a bcchm.html
Crohn's di s eas e (s ee IV G). Fami l y members have an i ncreas ed ri s k of i nfl ammat ory bowel di s eas e; approxi mat el y 15%–17% of pat i ent s have a fi rs t -degree rel at i ve wi t h i nfl ammat ory bowel di s eas e. The i nci dence i s i ncreas ed t wofol d t o fourfol d i n Jews . W hereas s moki ng i ncreas es t he ri s k of Crohn's di s eas e, i t decreas es t he ri s k of ul cerat i ve col i t i s t o approxi mat el y hal f t hat of t he general popul at i on. There i s an i ncreas ed ri s k of ul cerat i ve col i t i s i n pat i ent s who have recent l y di s cont i nued s moki ng. For pat i ent s wi t h ul cerat i ve col i t i s pat i ent s who are HLA-B27 pos i t i ve t here i s a s t rong as s oci at i on wi t h art hri t i s , and es peci al l y wi t h ankyl os i ng s pondyl i t i s .
2. Pathology. The hal l marks of ul cerat i ve col i t i s are t he mi croabs ces s es of t he crypt s of Li eberkühn, whi ch are s een i n approxi mat el y 70% of cas es . The i nfl ammat ory res pons e general l y i s l i mi t ed t o t he mucos a. Macros copi c ul cerat i ons are not ed wi t h confl uence of t he i nfl ammat ory res pons e. Ps eudopol yps occur when normal mucos a i s i s ol at ed by s evere ul cerat i ons , but t here are no s ki p areas . The rect um and di s t al col on are i nvol ved mos t commonl y. Pancol i t i s i s s een i n 25% of cas es . Ul cerat i ve col i t i s wi t h rect al s pari ng i s ext remel y uncommon.
3. Clinical features are mi l d when t he di s eas e i s l i mi t ed t o t he rect um (ul cerat i ve proct i t i s ). Moderat e-t o-s evere s ympt oms may occur wi t h ext ens i ve di s eas e, part i cul arl y pancol i t i s , and i ncl ude bl oody di arrhea, wei ght l os s , fever, l eft l ower abdomi nal crampi ng pai n, and noct urnal pas s age of a s mal l vol ume of bl ood and mucus . Ful mi nant di s eas e occurs i n 15% of cas es . Ext ra-i nt es t i nal mani fes t at i ons , are s i mi l ar t o t hos e of
Pa g e 1 0 1 4
ABC Ambe r CHM Conve rte r Tria l ve rsion, http://w w w .proce sste x t.com/a bcchm.html
Crohn's di s eas e and are evi dent i n 10%–15% of pat i ent s . Ext ra-i nt es t i nal s i gns and s ympt oms i ncl ude nons ymmet ri cal l arge joi nt art hri t i s , s acroi l i i t i s (es peci al l y i n HLA-B27 pos i t i ve pat i ent s ), eryt hema nodos um, pyoderma gangrenos um, s cl eros i ng chol angi t i s , and uvei t i s .
4. Diagnosis i s bas ed on cl i ni cal pres ent at i on al ong wi t h t he excl us i on of i nfect i ous , paras i t i c, and neopl as t i c et i ol ogi c fact ors . Serol ogi c anal ys i s for peri nucl ear ant i -neut rophi l cyt opl as mi c ant i body (p-ANCA) may ai d i n di fferent i at i on bet ween ul cerat i ve col i t i s and Crohn's di s eas e (s ee IV G). Thes e ant i bodi es occur i n 60%–85% of pat i ent s wi t h ul cerat i ve col i t i s . o
o
a. Stool examination reveal s mucus , bl ood, and whi t e bl ood cel l s wi t hout paras i t es or bact eri al pat hogens . It i s i mport ant t o rul e out t he us ual caus es of dys ent ery, i ncl udi ng Sal monel l a, Shi gel l a, Campyl obac t er, pat hogeni c E. c ol i (es peci al l y E. c ol i O157:H7), amebi as i s , and C. di ffi c i l e i nfect i on.
o
o
b. Colonoscopy or proctosigmoidoscopy reveal s fri abi l i t y, edema, and hyperemi a of t he mucos a. Ul cerat i ons and a mucopurul ent exudat e may be pres ent . Is l ands of normal t i s s ue may have t he appearance of ps eudopol yps . Numerous bi ops y s ampl es s houl d be obt ai ned.
o
o
c. Barium enema s houl d not be performed i n s everel y i l l or t oxi c pat i ent s . If t he s ympt oms are s ubacut e, a bari um radi ograph aft er mi ni mal
Pa g e 1 0 1 5
ABC Ambe r CHM Conve rte r Tria l ve rsion, http://w w w .proce sste x t.com/a bcchm.html
preparat i on may s how a l ack of haus t ral marki ngs , fi ne s errat i ons (compat i bl e wi t h s mal l ul cerat i ons ), l arge ul cerat i ons , and ps eudopol yps .
5. T herapy vari es wi t h t he s everi t y and ext ent of di s eas e. o
o
a. In acut e fl ares , bowel rest with intravenous fluids may be us eful for s hort peri ods . Tot al parent eral nut ri t i on al l ows prol onged bowel res t wi t h repl et i on of vi t ami ns , mi neral s , el ect rol yt es , and cal ori es i n t he form of carbohydrat e, prot ei n, and fat . P.208
o
o
b. 5-Aminosalicylic acid and sulfasalazine have been s hown t o i nduce remi s s i on and decreas e rel aps e rat es i n ul cerat i ve col i t i s pat i ent s . Si de effect s i ncl ude headache, naus ea, ras h, and agranul ocyt os i s . 5-Ami nos al i cyl i c aci d enemas are effect i ve i n l eft -s i ded col i t i s .
o
o
c. Corticosteroids, whi ch are admi ni s t ered by enema (es peci al l y i n di s t al di s eas e s uch as ul cerat i ve proct i t i s ) or s ys t emi cal l y, may be effect i ve i n i nduci ng remi s s i on. Predni s one i s gi ven oral l y i n dos es of 20–60 mg/day.
o
o
d. Immunomodulating agents s uch as 6-mercapt opuri ne, az at hi opri ne, cycl os pori ne,
Pa g e 1 0 1 6
ABC Ambe r CHM Conve rte r Tria l ve rsion, http://w w w .proce sste x t.com/a bcchm.html
met hot rexat e, and hydroxychl oroqui ne have been t ri ed i n t he hopes of avoi di ng or del ayi ng emergent s urgery or as a s t eroi d-s pari ng agent i n s t eroi d-res i s t ant pat i ent s . In cl i ni cal t ri al s , bi ol ogi c t herapy wi t h i nfl i xi mab i nduces remi s s i on i n 40% of s evere cas es , and res pons e i n 60%–65%. o
o
e. Surgery requi res t he removal of t he ent i re col on wi t h t he creat i on of an i l eos t omy or creat i on of an i l eal J-pouch wi t h i l eoanal anas t omos i s . Surgery i s general l y res erved for t he fol l owi ng condi t i ons :
(1) Toxi c megacol on t hat i s unres pons i ve t o 24–72 hours of i nt ens i ve cons ervat i ve medi cal meas ures
(2) Perforat i on
(3) Mas s i ve hemorrhage t hat i s unres pons i ve t o cons ervat i ve t reat ment (rare)
(4) Carci noma
(5) Sus pect ed carci noma i n col oni c s t ri ct ures
(6) Growt h fai l ure i n adol es cent s , whi ch i s unres pons i ve t o cons ervat i ve t reat ment
(7) Dys pl as i a not ed on bi ops y at t he t i me of
Pa g e 1 0 1 7
ABC Ambe r CHM Conve rte r Tria l ve rsion, http://w w w .proce sste x t.com/a bcchm.html
s i gmoi dos copy or col onos copy, whi ch s houl d be performed rout i nel y for s creeni ng i n l ong-s t andi ng di s eas es
(8) Cure, es peci al l y aft er 10 years of di s eas e becaus e of t he i ncreas ed ri s k of cancer. The i l eoanal anas t omot i c procedure t hat s t ri ps t he col oni c mucos a and avoi ds t he need for a permanent col os t omy has become wi des pread.
6. Complications. The s ys t emi c compl i cat i ons not ed for Crohn's di s eas e (s ee IV G 3 e) oft en occur i n ul cerat i ve col i t i s . Addi t i onal compl i cat i ons occur wi t h ul cerat i ve col i t i s , whi ch us ual l y are not s een i n Crohn's di s eas e. o
o
a. T oxic megacolon refers t o an acut e di l at i on of t he col on (us ual l y t he t rans vers e port i on) t o a di amet er i n exces s of 6 cm. Thi s compl i cat i on of ul cerat i ve col i t i s probabl y i s at t ri but abl e t o s evere i nfl ammat i on, whi ch affect s l arge s egment s of t he col oni c mus cul at ure as wel l as neural cont rol of t he col on. Ant i chol i nergi c and ant i di arrheal medi cat i ons al s o may cont ri but e. Pat i ent s us ual l y are s everel y i l l , wi t h hi gh fever, abdomi nal pai n, and a marked l eukocyt os i s . Treat ment i s i nt ens i ve medi cal t herapy for 48–72 hours . Pat i ent s who do not res pond s houl d undergo an emergency t ot al col ect omy.
o
o
b. Carcinoma of t he col on i s as s oci at ed wi t h l ong-s t andi ng di s eas e of great ext ent (us ual l y pan col i t i s ). At 10 years , t he ri s k of carci noma i s 10% and may i ncreas e t o 20% at 20 years and 40% at
Pa g e 1 0 1 8
ABC Ambe r CHM Conve rte r Tria l ve rsion, http://w w w .proce sste x t.com/a bcchm.html
25–30 years . The mal i gnanci es oft en are mul t i cent ri c and aggres s i ve. St ri ct ured areas of t he col on pres ent a part i cul arl y di ffi cul t probl em becaus e of t he di ffi cul t y i n di fferent i at i ng i nt ens i ve i nfl ammat ory di s eas e from i s chemi c narrowi ng or carci noma. Regul ar col onos copi c exami nat i ons wi t h bi ops y s ampl es obt ai ned every 10–20 cm s houl d be performed i n pat i ent s who have had ul cerat i ve col i t i s for l onger t han 8–10 years . If hi gh-grade dys pl as i a i s not ed, a prophyl act i c t ot al col ect omy s houl d be cons i dered.
7. Prognosis. Mort al i t y rat es are approxi mat el y 20% i n t oxi c megacol on, wi t h hi gher rat es not ed i n pat i ent s ol der t han 60 years . Approxi mat el y 10% of pat i ent s do not experi ence a recurrent at t ack aft er t he i ni t i al ons et of di s eas e. Cont i nuous s ympt oms occur i n 10% of pat i ent s . Approxi mat el y 70%–80% of pat i ent s have recurrent remi s s i ons and rel aps es , and approxi mat el y 20% of pat i ent s event ual l y requi re t ot al col ect omy. Survei l l ance col onos copy wi t h bi ops y t o eval uat e for dys pl as i a s houl d be performed every 1–2 years aft er 8–10 years of pancol i t i s or 12–15 years of l eft -s i ded di s eas e.
E. Angiodysplasia Angiodysplasia refers t o s mal l vas cul ar abnormal i t i es , whi ch us ual l y are s een i n t he as cendi ng col on or cecum i n pat i ent s ol der t han 60 years of age. Invol vement of t he s mal l bowel or s t omach al s o has been report ed but i s l es s common. As s oci at i ons wi t h aort i c s t enos i s and chroni c renal i ns uffi ci ency have been report ed. P.209
Pa g e 1 0 1 9
ABC Ambe r CHM Conve rte r Tria l ve rsion, http://w w w .proce sste x t.com/a bcchm.html
1. Pathogenesis. Angi odys pl as i a i s bel i eved t o res ul t from obs t ruct i on of i nt es t i nal capi l l ari es and venul es as t hes e ves s el s pas s t hrough t he mus cul ari s .
2. Clinical features. Mos t pat i ent s are as ympt omat i c, but t he abnormal ves s el s are a common caus e of pai nl es s l ower gas t roi nt es t i nal bl eedi ng i n ol der i ndi vi dual s .
3. Diagnosis. Angi ographi c or col onos copi c demons t rat i on of i nt ral umi nal ext ravas at i on of bl ood duri ng t he acut e epi s ode may be us ed t o make t he di agnos i s .
4. T herapy. Bl eedi ng us ual l y can be managed cons ervat i vel y wi t h col onos copi c heat er probe or l as er t reat ment , but col on res ect i on occas i onal l y i s needed for recurrent or mas s i ve bl eedi ng.
F. Endometriosis Endometriosis i nvol ves t he col on i n approxi mat el y 10% of cas es . Sympt oms i ncl ude pai n or rect al bl eedi ng duri ng mens es . A bari um enema oft en reveal s ext ri ns i c compres s i on of t he rect os i gmoi d or des cendi ng col on. Treat ment i s hormonal . Rarel y, s urgery i s neces s ary t o al l evi at e obs t ruct i on, pai n, or recurrent bl eedi ng.
G. Tumors of the colon
1. Benign tumors. There are s everal hi s t ol ogi c t ypes of beni gn col oni c pol yps . Adenomatous polyps are cons i dered t o be precurs ors of adenocarci noma, and t he ri s k i ncreas es when t he pol yps are l arger t han 2 cm,
Pa g e 1 0 2 0
ABC Ambe r CHM Conve rte r Tria l ve rsion, http://w w w .proce sste x t.com/a bcchm.html
vi l l ous rat her t han t ubul ar, and s es s i l e rat her t han peduncul at ed. Approxi mat el y 5%–10% of i ndi vi dual s ol der t han age 40 years have col oni c pol yps , but mos t of t hes e pol yps are s mal l hyperplastic l es i ons t hat carry no mal i gnant pot ent i al . Ot her beni gn t umors i ncl ude l ei omyomas , l i pomas , and fi bromas . o
o
a. Clinical features. Smal l pol yps are as ympt omat i c. In l arge pol yps (>1 cm), rect al bl eedi ng may occur and may be mi cros copi c or macros copi c. Large pol yps may caus e s ympt oms of an i ncompl et e i nt es t i nal obs t ruct i on wi t h occas i onal crampy abdomi nal pai n.
o
o
b. Diagnosis. Imagi ng by col onos copy or radi ographi c s t udy. Bari um enema or CT s can i s pos s i bl e.
o
o
c. T herapy
(1) Therapy for peduncul at ed l es i ons i s col onos copi c removal wi t h s nare el ect rocaut ery. Ses s i l e l es i ons may requi re s urgi cal exci s i on.
(2) Becaus e carci noma occurri ng i n an adenomat ous pol yp may be focal , careful hi s t ol ogi c s ect i oni ng of t he ent i re pol yp, not jus t a bi ops y, i s neces s ary t o excl ude carci noma. If mal i gnancy i nvades t he s t al k of a pol yp, a s egment al res ect i on of t he col on i s i ndi cat ed t o rul e out l ymphat i c s pread.
Pa g e 1 0 2 1
ABC Ambe r CHM Conve rte r Tria l ve rsion, http://w w w .proce sste x t.com/a bcchm.html
(3) Synchronous pol yps occur i n 20% of cas es , and met achronous l es i ons occur i n approxi mat el y 30% of cas es . Therefore, ful l col on eval uat i on, general l y by col onos copy, mus t be performed. Fol l ow-up col onos copy i s i ndi cat ed and t he i nt erval bet ween s t udi es depends on compl et enes s of pol yp res ect i on, s i ze and number of pol yps removed, and whet her pol ys are found on s ubs equent exami nat i ons .
(4) Fi rs t -degree rel at i ves of a pat i ent wi t h col oni c pol yps or carci noma have approxi mat el y a fourfol d t o fi vefol d i ncreas ed ri s k of devel opment of a s i mi l ar l es i on. The dai l y us e of as pi ri n or ot her NSAIDs may be as s oci at ed wi t h decreas ed pol yp format i on.
2. Hereditary polyposis syndromes o
o
a. Familial adenomatous polyposis (FAP) i s an aut os omal domi nant s yndrome charact eri zed by adenomas of t he col on. Al l el i c l os s i n chromos ome 5q21-q22 was named t he adenomat ous pol ypos i s col i (APC) gene and can be det ect ed i n peri pheral bl ood l eukocyt es of pat i ent s wi t h fami l i al pol ypos i s of t he col on.
(1) Hundreds of adenomat ous pol yps of t he l arge i nt es t i ne or cancer of t he peri ampul l ary regi on of t he s mal l bowel i s not ed i n pat i ent s
Pa g e 1 0 2 2
ABC Ambe r CHM Conve rte r Tria l ve rsion, http://w w w .proce sste x t.com/a bcchm.html
and s houl d be eval uat ed peri odi cal l y wi t h es ophagogas t ri c duodenos copy.
(2) W hen os t eomas or s oft t i s s ue t umors (l i pomas , fi bromas , or des moi d t umors of t he mes ent ery) are pres ent , t hi s condi t i on i s cal l ed Gardner' s syndrome.
(3) Col oni c mal i gnancy devel ops by age 40 years i n 80%–90% of pat i ent s . A s ubt ot al res ect i on of t he col on wi t h cl os e s ubs equent obs ervat i on s houl d be performed by age 30 years . Sul i ndac t herapy has l ed t o regres s i on of pol yps i n s everal pat i ent s .
o
o
b. Peutz-Jeghers syndrome i s an aut os omal domi nant pol ypos i s s yndrome wi t h mucocut aneous pi gment at i on, part i cul arl y of t he buccal mucos a. The pol yps are hamart omas , not adenomas , whi ch carry a l ow ri s k for mal i gnant t rans format i on and may be pres ent i n t he s t omach and s mal l bowel as wel l as i n t he col on. Pat i ent s may have recurrent gas t roi nt es t i nal bl eedi ng.
o
o
c. T urcot syndrome refers t o pancol oni c adenomas wi t h mal i gnant CNS t umors . Thi s aut os omal domi nant pol ypos i s s yndrome has a hi gh ri s k of mal i gnancy.
o
o
d. Juvenile polyposis i s an aut os omal domi nant s yndrome wi t h gas t roi nt es t i nal bl eedi ng from pol yps of t he col on, s mal l bowel , and s t omach. The
Pa g e 1 0 2 3
ABC Ambe r CHM Conve rte r Tria l ve rsion, http://w w w .proce sste x t.com/a bcchm.html
ri s k of mal i gnancy i s s l i ght l y i ncreas ed i n l at er l i fe. o
o
e. Cronkhite-Canada syndrome i s a rare as s oci at i on of i nt es t i nal pol yps wi t h al opeci a, hyperpi gment at i on, and a l ack of fi ngernai l s . No concl us i ve i nheri t ance pat t ern has been not ed.
o
o
f. Cowden' s disease i s a rare aut os omal domi nant condi t i on charact eri zed by mul t i pl e hamart omas of t he face, ot her part s of t he s ki n (acral kerat os es ), or mout h. Breas t l es i ons (fi brocys t i c di s eas e or cancer) occur i n approxi mat el y 50% of pat i ent s . Thyroi d abnormal i t i es , i ncl udi ng goi t er or cancer, occur i n 10%–15% of pat i ent s . There i s no i ncreas ed ri s k of cancer as s oci at ed wi t h t he gas t roi nt es t i nal hamart omat ous pol yps .
3. Adenocarcinoma of t he col on. Thi s t ype of cancer has been s t eadi l y i ncreas i ng i n frequency i n t he Uni t ed St at es and ranks s econd t o l ung cancer i n men and t hi rd t o breas t cancer and l ung cancer i n women (overal l s econd i n men and women) as t he major l i fe-t hreat eni ng mal i gnancy. o
o
a. Epidemiology. The i nci dence of col orect al carci nomas i s i ncreas ed i n devel oped count ri es , es peci al l y t hos e wi t h a di et hi gh i n red meat and l ow i n fi ber. In t he Uni t ed St at es , t he i nci dence i s decreas ed i n Sevent h-Day Advent i s t s who pract i ce s t ri ct veget ari ani s m, whi ch al s o s ugges t s an as s oci at i on wi t h di et . The i nci dence of col orect al carci nomas i s hi gher i n as bes t os
Pa g e 1 0 2 4
ABC Ambe r CHM Conve rte r Tria l ve rsion, http://w w w .proce sste x t.com/a bcchm.html
P.210
workers , machi ni s t s , and fact ory woodworkers t han i n t he general popul at i on. Increas ed cal ci um and fol i c aci d i nt ake i n men and women, as wel l as pos t menopaus al hormone t herapy i n women, may be prot ect i ve. Dai l y us e of as pi ri n or ot her NSAIDs al s o may guard agai ns t t he devel opment of col on cancer. o
o
b. Etiology
(1) Dietary factors. Di et has been t he focus of mos t et i ol ogi c s t udi es . The i ncreas ed amount s of red meat and ani mal fat i n t he di et i n t he Uni t ed St at es promot e t he growt h of bact eri al s t rai ns t hat produce carci nogens i n t he col oni c l umen. Bi l e s al t s al s o may cont ri but e t o t hi s proces s . Vi t ami ns A, C, and E i n cert ai n foods may i nact i vat e t he carci nogens , and broccol i , t urni ps , and caul i fl ower i nduce benz pyrene hydroxyl as e, whi ch al s o may i nact i vat e i nges t ed carci nogens .
(2) Genetic factors. The rol e of genet i c fact ors i s demons t rat ed by FAP and by t he fact t hat fi rs t -degree rel at i ves of pat i ent s wi t h carci noma or pol yps have a t hreefol d t o fi vefol d i ncreas ed ri s k of col orect al carci noma. FAP account s for approxi mat el y 1% of col orect al cancer (s ee
V G 2onl ). Heredi t ary nonpol ypos i s
col orect al cancer (HNPCC) fami l y s yndromes account for 10% of al l col orect al cancer and i s an aut os omal domi nant s yndrome charact eri zed by an abnormal i t y i n mi s mat ch repai r genes ., One s houl d cons i der a genet i c caus e of col orect al cancer i f t he pat i ent
Pa g e 1 0 2 5
ABC Ambe r CHM Conve rte r Tria l ve rsion, http://w w w .proce sste x t.com/a bcchm.html
i s younger t han age 50 or has a fi rs t -degree rel at i ve wi t h a col on cancer s yndrome.
(a) HNPCC type I i s an aut os omal domi nant i nheri t ed form of col on cancer. HNPCC oft en devel ops before pat i ent s reach 45 years of age and frequent l y i nvol ves t he as cendi ng col on.
(b) HNPCC type II i s as s oci at ed wi t h carci noma of t he endomet ri um, ovary, uret er, renal pel vi s , s t omach, pancreas , and bi l i ary t ree and ot herwi s e i s s i mi l ar t o HNPCC t ype I.
(3) Other risk factors
(a) Ul cerat i ve col i t i s , es peci al l y pancol i t i s and di s eas e of great er t han 10 years ' durat i on (10% ri s k)
(b) Hi s t ory of col on cancer or adenoma (10% ri s k)
(c) Fami l y hi s t ory of col on cancer or adenomat ous pol yp
(d) Hi s t ory of femal e geni t al or breas t cancer
Pa g e 1 0 2 6
ABC Ambe r CHM Conve rte r Tria l ve rsion, http://w w w .proce sste x t.com/a bcchm.html
(e) Hi s t ory of juveni l e pol yps
(f) Immunodefi ci ency di s eas es
o
o
c. Clinical features. Si gns and s ympt oms vary, dependi ng on t he l ocat i on and s i ze of t he t umor. Tumors i n t he l eft col on, es peci al l y t hos e i n t he di s t al 25 cm, may mani fes t as obs t ruct i on. Ri ght col on t umors frequent l y pres ent as i ron defi ci ency anemi a and fat i gue. Ot her common s ympt oms i ncl ude a change i n bowel habi t , a decreas e i n s t ool s i ze, obvi ous bl ood i n t he s t ool , and crampy abdomi nal pai n. Met as t at i c di s eas e us ual l y i nvol ves t he l i ver; however, t he bone, l ung, and brai n al s o may be affect ed.
o
o
d. Screening (s ee Tabl e 5-2)
TABLE 5-2 Colorectal Cancer Screening Recommendations* Ris k for CR C Recommendations Av Col onos copy every era 10 years aft er age ge 50 ri s FOBT (× 3) yearl y k
aft er age 50 wi t h
pat FOBT i en Fl ex s i g or BE every
Pa g e 1 0 2 7
ABC Ambe r CHM Conve rte r Tria l ve rsion, http://w w w .proce sste x t.com/a bcchm.html
ts
3–5 years aft er
age 50 Inc FAP rea Fl ex s i g every 1 s ed year aft er pubert y ri s (cons i der t ot al k
col ect omy)
pat Genet i c couns el i ng i en for pat i ent and t s fami l y HN Col onos copy every PC 1–2 years aft er C
age 20 or 10 years earl i er t han t he younges t age of col on cancer di agnos i s i n t he fami l y (genet i c couns el i ng for
pat i ent and fami l y) Ul c Col onos copy every era 1–2 years aft er 8 t i v years of di s eas e i n e
pat i ent s wi t h
col i pancol i t i s or t i s col onos copy every 1–2 years aft er 15 years i n pat i ent s wi t h l eft -s i ded col i t i s 1s t Same s creeni ng as -De average ri s k gre pat i ent s , except e
begi n at age 40 or
rel 10 years earl i er at i t han t he 1s t -degree
Pa g e 1 0 2 8
ABC Ambe r CHM Conve rte r Tria l ve rsion, http://w w w .proce sste x t.com/a bcchm.html
ves rel at i ve's di agnos i s wi t h col on can cer Hi s
Col o
t or
nos c
y
opy
of
in
ad
1–
en
3
om
year
at o
s if
us
mul t
pol
i pl e
yps
(>3)
(va
aden
ri a
oma
bl e
s or
)
adva nced aden oma
1 or 2 s mal l (<1 cm) t ubu l ar aden
Pa g e 1 0 2 9
ABC Ambe r CHM Conve rte r Tria l ve rsion, http://w w w .proce sste x t.com/a bcchm.html
oma —c ol on os co py at 5†“10 year s Ad
Pat i
en
ent s
om
wi t h
at o
adva
us
nced
pol
l arg
yps
e (>1 cm) vi l l o us or noni nvas i ve mal i gnan t — col o nos c opy in 1– 3
Pa g e 1 0 3 0
ABC Ambe r CHM Conve rte r Tria l ve rsion, http://w w w .proce sste x t.com/a bcchm.html
year s (mul t i pl e >3)
Pat i ent s wi t h 1 s mal l (<1 cm) t ubu l ar aden oma —c ol on os co py in 3– 5 year s
FOBT, fecal occul t bl ood t es t i ng; FAP, fami l i al adenomat ous pol ypos i s ; HNPCC, heredi t ary nonpol ypos i s col orect al cancer s yndrome; Fl ex s i g, fl exi bl e s i gmoi dos copy;
Pa g e 1 0 3 1
ABC Ambe r CHM Conve rte r Tria l ve rsion, http://w w w .proce sste x t.com/a bcchm.html
CRC, col orect al cancer. *“Vi rt ual col onos copy― i s a radi ographi c comput eri zed col onography oft en us i ng t hree-di mens i onal t echni ques and may become an accept ed s creeni ng met hod o
o
e. Diagnosis
(1) Di agnos i s i s made by col onos copy or ai r cont ras t bari um enema demons t rat i on of pol yps or t umors fol l owed by endos copi c vi s ual i zat i on wi t h bi ops y and cyt ol ogi c s t udy. The ai r cont ras t bari um enema i s far more s ens i t i ve t han t he s i ngl e cont ras t exami nat i on. Des pi t e t hes e t echni ques , i t can be di ffi cul t t o di fferent i at e t he t umor from di vert i cul i t i s , beni gn s t ri ct ure, and Crohn's di s eas e.
(2) Carci noembryoni c ant i gen (CEA) det ermi nat i ons , al t hough not us eful for s creeni ng purpos es , may be us ed for peri odi c fol l ow-up i n pat i ent s wi t h a hi s t ory of carci noma of t he col on, wi t h an i ncreas i ng t i t er bei ng i ndi cat i ve of recurrent or met as t at i c di s eas e.
o
Pa g e 1 0 3 2
ABC Ambe r CHM Conve rte r Tria l ve rsion, http://w w w .proce sste x t.com/a bcchm.html
o
f. T herapy (s ee Chapt er 4 VII 9 a)
o
o
g. Prognosis. The overal l 10-year s urvi val rat e i s 45% and has not changed s i gni fi cant l y over t he l as t s everal years .
(1) Dukes' classification A and B. Cancer confi ned t o t he mucos a i s oft en det ect ed by s creeni ng and i s as s oci at ed wi t h an 80%–90% s urvi val rat e.
(2) Dukes' classification C. Cancer t hat i s l i mi t ed t o t he regi onal l ymph nodes i s as s oci at ed wi t h a 50%–60% s urvi val rat e. P.211
(3) Dukes' classification D. Cancer t hat has met as t as i zed t o di s t ant organs i s as s oci at ed wi t h a s urvi val rat e of l es s t han 25%.
H. Collagenous colitis Collagenous colitis i s a recent l y des cri bed s yndrome of chroni c wat ery di arrhea, es peci al l y s een i n mi ddl e-aged women. Laborat ory dat a are us ual l y normal , except 50% of pat i ent s may have an i ncreas e i n eryt hrocyt e s edi ment at i on rat e. Hypoal bumi nemi a and mi l d s t eat orrhea have been report ed. Col onos copi c exami nat i on fi ndi ngs are normal , but bi ops y i ndi cat es a t hi ck l ayer of s ubepi t hel i al col l agen depos i t i on, whi ch i s great er t han 15 µm i n
Pa g e 1 0 3 3
ABC Ambe r CHM Conve rte r Tria l ve rsion, http://w w w .proce sste x t.com/a bcchm.html
t hi cknes s (normal col l agen depos i t i on i s l es s t han 5 µm). Treat ment i s wi t h ant i di arrheal agent s , s ul fas al azi ne, s t eroi ds , or al l t hree. Microscopic colitis, marked by a l ymphocyt i c i nfi l t rat e, i s probabl y an earl y s t age of col l agenous col i t i s .
I. Pseudomembranous colitis Pseudomembranous colitis i s an acut e, pot ent i al l y s evere di s eas e of t he col on charact eri zed by exudat i ve pl aques t hat cover t he i nt es t i nal mucos a.
1. Pathogenesis. The di s eas e i s caus ed by an ent erot oxi n produced by C. di ffi c i l e, an anaerobi c bact eri um. It i s t hought t hat ant i bi ot i c t herapy may “s el ect out ― t he C. di ffi c i l e organi s m, al l owi ng prol i ferat i on and t oxi n product i on. Sympt oms begi n 3 days t o 4 weeks aft er i ni t i at i ng ant i bi ot i c t herapy. Vi rt ual l y al l ant i bi ot i cs have been as s oci at ed wi t h t hi s di s eas e, but cl i ndamyci n, fl uoroqui nol ones , and t he cephal os pori ns are t he mos t common offenders .
2. Clinical features. Si gns and s ympt oms i ncl ude wat ery di arrhea, crampy abdomi nal pai n, l ower abdomi nal t endernes s , and fever. Leukocyt os i s i s common. Dehydrat i on and el ect rol yt e di s t urbances may devel op i n s everel y i l l pat i ent s . Toxi c megacol on and col oni c perforat i on are rare, but s eri ous , compl i cat i ons t hat may requi re s urgi cal i nt ervent i on. Approxi mat el y 20% of pat i ent s rel aps e aft er pri mary t reat ment .
3. Diagnosis. Demons t rat i on of C. di ffi c i l e t oxi n i n t he s t ool or s i gmoi dos copi c vi s ual i zat i on of t he charact eri s t i c yel l ow-whi t e pl aques i n an eryt hemat ous and edemat ous mucos a s ugges t s t he di agnos i s . Bi ops y of t he pl aques s hows a muci nous , fi bri nous ,
Pa g e 1 0 3 4
ABC Ambe r CHM Conve rte r Tria l ve rsion, http://w w w .proce sste x t.com/a bcchm.html
pol ymorphonucl ear exudat e. Mos t pat i ent s have di s eas e t hroughout t he col on; however, t he di s eas e may be confi ned t o t he ri ght col on, and i n s uch cas es s i gmoi dos copi c fi ndi ngs are negat i ve. P.212
4. T herapy. The fi rs t s t ep i n t reat ment i s t o di s cont i nue unneces s ary ant i bi ot i cs , whi ch res ul t s i n i mprovement i n mos t pat i ent s . Al t hough met roni dazol e i s fi rs t -l i ne t herapy, vancomyci n s houl d be us ed i n s evere cas es . Rel aps e i s t reat ed wi t h a s econd cours e of t herapy. Mul t i pl e rel aps es can be t reat ed wi t h t aperi ng or pul s ed dos es of vancomyci n; or probi ot i cs i ncl udi ng Lac t obac i l l us or Sac c haromyc es boul ardi i , or chol es t yrami ne. Earl y dat a s ugges t ri faxi mi n t o be effect i ve. Severe rel aps e cas es have been t reat ed wi t h fecal t rans pl ant at i on or i nt ravenous i mmunogl obul i ns .
J. Cloacogenic carcinoma Cloacogenic carcinoma account s for 2.5% of al l anorect al carci nomas . It i s a carci noma of t he t rans i t i onal epi t hel i um of t he regi on of t he dent at e l i ne i n t he anal canal . It occurs at t he junct i on of t he ect oderm and ent odermal cl oaca—t he bl i nd caudal ext ens i on of t he hi ndgut . It i s more common i n women, wi t h a 3:1 rat i o, and i t i s mos t common i n t he 55- t o 70-year-ol d age group. Treat ment i s wi t h radi at i on and t hen s urgery.
K. Volvulus of the colon Volvulus of the colon general l y i nvol ves ei t her t he s i gmoi d col on (s l i ght l y more common) or t he cecal regi on. Si gmoi d vol vul us us ual l y occurs i n i ndi vi dual s ol der t han 60 years of age who l i ve i n nurs i ng homes , have CNS di s eas e, or t ake ant i mot i l i t y drugs . Men
Pa g e 1 0 3 5
ABC Ambe r CHM Conve rte r Tria l ve rsion, http://w w w .proce sste x t.com/a bcchm.html
are more s us cept i bl e, es peci al l y t hos e wi t h chroni c cons t i pat i on. Cecal vol vul us commonl y fol l ows previ ous s urgery. Acut e cas es requi re emergency col onos copy or bari um enema. Surgi cal res ect i on may be neces s ary i n 70%–90% of cas es .
L. Cytomegalovirus (CMV) colitis Cytomegalovirus (CMV) colitis i s oft en s een i n pat i ent s wi t h AIDS, s evere di abet es , renal fai l ure, or i nfl ammat ory bowel di s eas e. Bl oody di arrhea may occur becaus e of a deepl y ul cerat ed col on. Int ranucl ear i ncl us i on bodi es may be not ed on bi ops y. The ul cers of CMV col i t i s may l ead t o perforat i on. Ganci cl ovi r has been us ed for t reat ment i n s ome cas es .
M. Diversion colitis Diversion colitis i s col i t i s t hat devel ops i n s egment s of t he col on t hat have been di vert ed from t he fecal s t ream. The di vert ed col on i s eryt hemat ous , fri abl e, and may be as s oci at ed wi t h bl eedi ng. Al t hough t he gros s endos copi c appearance mi mi cs t hat of ul cerat i ve col i t i s , hi s t ol ogi c di fferent i at i on from ul cerat i ve col i t i s may al s o be di ffi cul t . Treat ment of di vers i on col i t i s has i ncl uded s hort -chai n fat t y aci ds or repl acement of t he col on i n cont i nui t y wi t h t he fecal s t ream. Di fferent i at i on bet ween di vers i on col i t i s and ul cerat i ve col i t i s i s neces s ary, becaus e t he l at t er condi t i on oft en fl ares up when re-anas t omos i s occurs .
VI. Diseases of the Rectum and Anus A. Ulcerative proctitis Ulcerative proctitis i s a l ocal i zed form of ul cerat i ve col i t i s , whi ch has a bet t er prognos i s and a great l y decreas ed ri s k of mal i gnancy.
1. Clinical features. Sympt oms i ncl ude di arrhea, rect al bl eedi ng, and t enes mus ; onl y rarel y do fever, wei ght l os s , and t he s ys t emi c compl i cat i ons of ul cerat i ve col i t i s occur.
Pa g e 1 0 3 6
ABC Ambe r CHM Conve rte r Tria l ve rsion, http://w w w .proce sste x t.com/a bcchm.html
2. Diagnosis. Ot her caus es of proct i t i s , es peci al l y i nfect i on, s houl d be rul ed out . The abs ence of i nfl ammat i on above t he rect um s houl d be veri fi ed by s i gmoi dos copy.
3. T herapy. Sul fas al azi ne, rect al cort i cos t eroi ds , and 5-ami nos al i cyl i c aci d enemas oft en are effect i ve t reat ment .
4. Outcome. In approxi mat el y 15%–20% of cas es , ul cerat i ve proct i t i s progres s es t o di ffus e ul cerat i ve col i t i s .
B. Infectious proctitis
1. Sexually transmitted diseases t hat caus e i nfect i ous proct i t i s i ncl ude s yphi l i s , gonorrhea, l ymphogranul oma venereum, and herpes s i mpl ex. Thes e di s eas es are es peci al l y common i n homos exual men, who may have mul t i pl e s i mul t aneous i nfect i ons (s ee Chapt er 8 VI D). o
o
a. Syphilis of t he rect um al mos t al ways i s pri mary s yphi l i s . The chancre, whi ch i s pai nl es s , appears 10–90 days aft er expos ure. Di agnos i s i s made by dark-fi el d exami nat i on of di s charge from t he chancre and by s erol ogi c t es t i ng, al t hough t he Venereal Di s eas e Res earch Laborat ori es (VDRL) t es t does not become pos i t i ve unt i l 1–2 weeks aft er t he appearance of t he chancre.
o
Pa g e 1 0 3 7
ABC Ambe r CHM Conve rte r Tria l ve rsion, http://w w w .proce sste x t.com/a bcchm.html o
b. Gonorrhea may be as ympt omat i c or may caus e rect al bl eedi ng and di arrhea. Di agnos i s i s made by cul t uri ng t he organi s m.
o
o
c. Lymphogranuloma venereum i s caus ed by one s t rai n of Chl amydi a t rac homat i s , a gram-negat i ve obl i gat e i nt racel l ul ar bact eri um. W hen l eft unt reat ed, t he acut e proct i t i s may devel op i nt o a chroni c des t ruct i ve i nfl ammat i on wi t h l at e s t ri ct ure format i on. The organi s m i s di ffi cul t t o cul t ure, al t hough cul t ure i n yol k s acs and t i s s ue cul t ures are pos s i bl e. Serol ogi c di agnos i s general l y i s more avai l abl e. Ti t ers of great er t han or equal t o 1:16 are hi ghl y s ugges t i ve of current i nfect i on.
o
o
d. Herpes simplex may caus e cons t i pat i on, hemat ochezi a, s evere anorect al pai n, t enes mus , and mucopurul ent di s charge from t he rect um. Bl adder dys funct i on wi t h i mpot ence may be pres ent . Rect al bi ops y s hows i nt ranucl ear i ncl us i ons . The s ympt oms s ubs i de s pont aneous l y but may recur.
2. Amebiasis (i .e., i nfect i on wi t h E. hi s t ol yt i c a) may pres ent as a di ffus e col i t i s or ext ra-i nt es t i nal di s eas e (e.g., meni ngi t i s , l i ver abs ces s ), or i t may be confi ned t o t he rect os i gmoi d col on, es peci al l y i n homos exual men. Sympt oms range from mi l d di arrhea t o bl oody dys ent ery. Di agnos i s i s made by demons t rat i ng t he organi s m i n t he s t ool or i n s i gmoi dos copi c bi ops y s peci mens of t he charact eri s t i c fl as k-s haped ul cers . Treat ment wi t h met roni dazol e or i odoqui none may be us eful .
Pa g e 1 0 3 8
ABC Ambe r CHM Conve rte r Tria l ve rsion, http://w w w .proce sste x t.com/a bcchm.html
C. Solitary rectal ulcer Solitary rectal ulcer i s a s yndrome cons i s t i ng of a s uperfi ci al ul cerat i on of unknown caus e combi ned wi t h pas s age of mucus or bl ood and dul l rect al pai n. The ul cers us ual l y are 2 cm i n di amet er on t he ant eri or rect al mucos al wal l and l ocat ed 7–10 cm from t he anal verge. They are mul t i pl e i n 25% of pat i ent s . A weaknes s i n t he rect al s l i ng mus cul at ure may be a cont ri but i ng caus e. Di agnos i s i s made by excl udi ng ot her caus es of rect al ul cers , i ncl udi ng i nfect i ons , i nfl ammat ory bowel di s eas e, and carci noma. Treat ment i s s upport i ve, wi t h aggres s i ve t reat ment of cons t i pat i on.
D. Hemorrhoids Hemorrhoids are di l at ed i nt ernal or ext ernal vei ns of t he hemorrhoi dal pl exus l ocat ed i n t he l ower rect um. In t he Uni t ed St at es , 60%–70% of t he popul at i on experi ence s ympt oms of hemorrhoi ds at P.213 s ome t i me. Si gns and s ympt oms i ncl ude peri anal pruri t us , rect al bl eedi ng (es peci al l y s mal l amount s on t he t oi l et t i s s ue or bri ght dropl et s i nt o t he t oi l et bowl ), anal pai n, and a pal pabl e mas s i n t he anal regi on. The di agnos i s i s made by anos copy or s i gmoi dos copy. Treat ment i s wi t h s t ool s oft eners , s upport i ve care wi t h heat or ant i -edema meas ures , or s urgery. Int ernal hemorrhoi ds may be t i ed wi t h rubber bandi ng or res ect ed wi t h l as er or bi cap el ect rocaut ery t echni ques .
E. Anal fissures, abscesses, and fistulas Anal fissures, abscesses, and fistulas are t ears , i nfect i ons , and hol l ow channel s from t he rect um t o t he peri anal s ki n, res pect i vel y. Local l y appl i ed heat , s i t z bat hs , and ant i bi ot i cs may be effect i ve. However, s urgi cal drai nage or exci s i on of an abs ces s or a fi s t ul ous t ract i s occas i onal l y neces s ary.
F. Pruritus Pruritus of t he peri anal s ki n has many caus es , i ncl udi ng i nfect i on
Pa g e 1 0 3 9
ABC Ambe r CHM Conve rte r Tria l ve rsion, http://w w w .proce sste x t.com/a bcchm.html
(bact eri al , fungal , or paras i t i c), l ocal i zed anorect al di s eas e (e.g., fi s t ul as , fi s s ures ), dermat ol ogi c di s eas es (e.g., ps ori as i s , eczema), poor hygi ene, di arrhea, and s ys t emi c di s eas es s uch as di abet es mel l i t us . The underl yi ng condi t i on s houl d be t reat ed. In addi t i on, l ocal care wi t h careful cl eani ng fol l owi ng defecat i on and ni ght l y appl i cat i on of hydrocort i s one cream may be hel pful i n cont rol l i ng s ympt oms .
G. Squamous cell carcinoma Squamous cell carcinoma of t he anus i s a rare mal i gnancy t hat mani fes t s as bl eedi ng, pai n, a mas s , and change i n bowel habi t s . Treat ment i s s urgi cal , and t he 5-year s urvi val rat e i s 60%.
VII. Diseases of the Pancreas A. Acute pancreatitis
1. Etiology o
o
a. Common causes. Approxi mat el y 70% of cas es of acut e pancreat i t i s t hat occur i n t he Uni t ed St at es are at t ri but abl e t o ei t her al cohol abus e or gal l s t ones .
(1) In al cohol i c pancreat i t i s , prot ei naceous pl ugs devel op i n t he pancreat i c duct s and cal ci fy i n t he body of t he pancreas , l eadi ng t o s t as i s and at rophy of di s t al s egment s . Al cohol i c pancreat i t i s i s mos t common i n men who have i nges t ed l arge amount s of al cohol over at l eas t 10 years .
(2) In gal l s t one pancreat i t i s , t he pas s age of
Pa g e 1 0 4 0
ABC Ambe r CHM Conve rte r Tria l ve rsion, http://w w w .proce sste x t.com/a bcchm.html
a common duct s t one (mul t i pl e s mal l s t ones , or even mi crol i t hi as i s ) may i ni t i at e refl ux of bi l i ary or i nt es t i nal cont ent s i nt o t he pancreat i c gl and. o
o
b. Les s common caus es of acut e pancreat i t i s
(1) Pos t operat i ve pancreat i t i s , whi ch may be s evere and i s es peci al l y common aft er hepat obi l i ary t ract s urgery
(2) Abdomi nal t rauma
(3) Hyperl i pi demi a, t ypes I and V (i ncreas ed chyl omi crons )
(4) Drugs s uch as azat hi opri ne, 6-mercapt opuri ne, es t rogens , t hi azi des , furos emi de, s ul fonami des , t et racycl i nes , cort i cos t eroi ds , val proi c aci d, pent ami di ne, 2–3 di deoxyi nos i ne, oct reot i de, and 5-ami nos al i cyl i c aci d
(5) Hypercal cemi a
(6) Uremi a
(7) Pept i c ul cer di s eas e, wi t h penet rat i on i nt o t he pancreas
Pa g e 1 0 4 1
ABC Ambe r CHM Conve rte r Tria l ve rsion, http://w w w .proce sste x t.com/a bcchm.html
(8) Cys t i c fi bros i s (i n rare cas es )
(9) Endos copi c ret rograde chol angi opancreat ography (ERCP)
(10) Vi ral i nfect i ons , es peci al l y mumps , coxs acki evi rus B
(11) Vas cul ar i ns uffi ci ency
(12) Pancreat i c cancer, caus i ng duct al obs t ruct i on, i ncl udi ng ampul l ary and duodenal l es i ons
(13) Heredi t ary pancreat i t i s , whi ch may be i nheri t ed i n an aut os omal domi nant pat t ern and carri es an i ncreas ed ri s k for devel opment of pancreat i c carci noma
(14) Pancreas di vi s um, i n whi ch t he mai n port i on of t he pancreas drai ns i nt o t he s mal l er acces s ory duct
(15) The bi t e of a cert ai n s corpi on (T i t yus t ri ni t at i s ), whi ch caus es i ncreas ed pancreat i c enzyme s ecret i on and may be as s oci at ed wi t h pancreat i t i s
(16) Idi opat hi c caus es
Pa g e 1 0 4 2
ABC Ambe r CHM Conve rte r Tria l ve rsion, http://w w w .proce sste x t.com/a bcchm.html
P.214
2. Clinical features o
o
a. Abdominal pain, whi ch oft en i s a s t eady or s evere pai n i n t he peri umbi l i cal regi on and may radi at e t o t he back
o
o
b. Nausea and vomiting, whi ch occur i n 70% of cas es
o
o
c. Abdominal tenderness, us ual l y wi t hout guardi ng or rebound
o
o
d. Diminished or absent bowel sounds
o
o
e. Epigastric fullness or mass, whi ch us ual l y i s found l at e i n t he cours e of t he di s eas e
o
o
f. Retroperitoneal bleeding, caus i ng a hemat oma at t he umbi l i cus (Cullen' s sign) or fl ank (T urner' s sign), s een i n hemorrhagi c pancreat i t i s
3. Diagnosis us ual l y i s bas ed on charact eri s t i c cl i ni cal pres ent at i ons , es peci al l y i n pat i ent s wi t h a hi s t ory of previ ous pancreat i t i s . o
Pa g e 1 0 4 3
ABC Ambe r CHM Conve rte r Tria l ve rsion, http://w w w .proce sste x t.com/a bcchm.html
o
a. El evat ed s erum amyl as e l evel s al mos t al ways exi s t duri ng an acut e at t ack but al s o may be caus ed by perforat ed ul cer, i nt es t i nal i nfarct i on, obs t ruct i on, or rupt ured ect opi c pregnancy.
o
o
b. El evat ed s erum l i pas e l evel s al s o are found i n acut e at t acks . Increas ed s erum t ryps i nogen l evel s may be hel pful i n di ffi cul t -t o-di agnos e cas es .
o
o
c. Abdomi nal radi ographs may s how a l ocal i zed i l eus (s ent i nel l oop) i n t he s mal l bowel regi on adjacent t o t he pancreas . CT s canni ng i s very hel pful for di agnos i s and s t agi ng of t he di s eas e. The s everi t y of t he pancreat i t i s can be di fferent i at ed i nt o edemat ous and necrot i zi ng pancreat i t i s on t he bas i s of t he CT s can.
4. T herapy i s s upport i ve and i ncl udes i nt ravenous admi ni s t rat i on of fl ui ds and anal ges i cs and bowel res t . Nas ogas t ri c s uct i on s houl d be us ed i f s evere vomi t i ng i s pres ent . In prot ract ed i l l nes s , ent eral t ube feedi ng or i nt ravenous hyperal i ment at i on may be us ed. If an i mpact ed gal l s t one caus es pancreat i t i s , t hen ERCP, s phi nct erot omy, and s t one removal i s t he t reat ment of choi ce.
5. Complications account for t he 10% mort al i t y rat e as s oci at ed wi t h acut e pancreat i t i s . o
o
a. Hemorrhagic pancreatitis i s cons i dered an ext ens i on of edemat ous pancreat i t i s caus ed by
Pa g e 1 0 4 4
ABC Ambe r CHM Conve rte r Tria l ve rsion, http://w w w .proce sste x t.com/a bcchm.html
chemi cal medi at ors (e.g., el as t as e), whi ch l eads t o ret roperi t oneal hemorrhage and wi des pread t i s s ue necros i s . Hemorrhagi c pancreat i t i s i s more common aft er t rauma, pos t operat i ve pancreat i t i s , and t he i ni t i al at t ack of acut e pancreat i t i s and may requi re peri t oneal l avage or s urgi cal i nt ervent i on for pl acement of drai ns . Bl ood may be pres ent i n t he peri t oneal cavi t y. The di agnos i s i s s ugges t ed by a decl i ni ng hemat ocri t i n a s everel y i l l pat i ent . o
o
b. Acute respiratory distress syndrome (ARDS) i s caus ed by i ncreas ed al veol ar capi l l ary permeabi l i t y and may caus e s evere hypoxi a requi ri ng mechani cal vent i l at i on.
o
o
c. Pancreatic abscess i s s ugges t ed when hi gh fever, el evat ed s erum amyl as e l evel s , and l eukocyt os i s pers i s t beyond 7–10 days . Gas s hadows i n t he regi on of t he pancreas may be vi s ual i zed by an abdomi nal fl at pl at e or by CT. Treat ment i ncl udes s urgi cal drai nage and ant i bi ot i cs .
o
o
d. Pancreatic pseudocyst refers t o a col l ect i on of fl ui d and debri s wi t hi n t he pancreas or i n a s pace l i ned by t he pancreas and ot her adjacent s t ruct ures . Di agnos i s i s made by ul t ras onography. Approxi mat el y 50% of ps eudocys t s (us ual l y s mal l er ones ) res ol ve s pont aneous l y. Large ps eudocys t s pers i s t i ng beyond 6–10 weeks requi re drai nage t o avoi d pot ent i al l y s eri ous compl i cat i ons s uch as hemorrhage and rupt ure.
o
Pa g e 1 0 4 5
ABC Ambe r CHM Conve rte r Tria l ve rsion, http://w w w .proce sste x t.com/a bcchm.html
o
e. Pancreatic ascites may occur becaus e of a l eaki ng ps eudocys t wi t h pancreat i c duct al des t ruct i on. The di agnos i s i s s ugges t ed by very hi gh s erum amyl as e l evel s i n peri t oneal fl ui d. Cons ervat i ve t herapy wi t h t ot al parent eral nut ri t i on and repeat ed paracent es i s may l ead t o res ol ut i on, but pancreat i c res ect i on may be neces s ary i n i nt ract abl e cas es .
B. Chronic pancreatitis Chronic pancreatitis res ul t s i n permanent s t ruct ural damage of pancreat i c t i s s ue.
1. Etiology. Mos t of t he caus es of acut e pancreat i t i s i n t he Uni t ed St at es al s o can res ul t i n chroni c pancreat i t i s . A not abl e except i on i s gal l s t ones , whi ch caus e onl y recurrent acut e at t acks of pancreat i t i s . Al cohol abus e account s for 90% of cas es of chroni c pancreat i t i s i n adul t s . Cys t i c fi bros i s i s t he mos t common caus e of chroni c di s eas e i n chi l dren.
2. Clinical features o
o
a. Pain, t he us ual pres ent i ng s ympt om, t ypi cal l y occurs i n t he epi gas t ri um aft er eat i ng and radi at es t o t he back.
o
o
b. Malabsorption occurs i n as s oci at i on wi t h steatorrhea and weight loss. P.215
Pa g e 1 0 4 6
ABC Ambe r CHM Conve rte r Tria l ve rsion, http://w w w .proce sste x t.com/a bcchm.html o
o
c. Jaundice occurs becaus e of edema and fi bros i s i n t he pancreat i c head and caus es obs t ruct i on of t he pancreat i c port i on of t he common bi l e duct .
o
o
d. Diabetes i s common; however, ket oaci dos i s , nephropat hy, and di abet i c vas cul ar di s eas e rarel y occur.
3. Diagnosis. The devel opment of cont i nuous pai n and s i gns of pancreat i c i ns uffi ci ency i n a pat i ent wi t h known recurrent pancreat i t i s i s s ugges t i ve, es peci al l y when at t ri but abl e t o al cohol i nges t i on. Speci fi c t es t s i ncl ude: o
o
a. Abdomi nal radi ographs , whi ch s how pancreat i c cal ci fi cat i on i n 30%–40% of cas es
o
o
b. Secret i n-s t i mul at i on t es t i ng wi t h duodenal i nt ubat i on and as pi rat i on, whi ch i ndi cat es a l ow bi carbonat e concent rat i on i n t he pancreat i c s ecret i on and l ow enzyme out put
o
o
c. ERCP, whi ch s hows di ffus e duct al di l at i on wi t h an i rregul ar, beaded (“chai n of l akes ―) appearance
4. T herapy. Treat ment i s ai med at cont rol l i ng t he mani fes t at i ons of t he di s eas e becaus e t he underl yi ng damage t o t he gl and i s permanent . In addi t i on, agent s t hat may promot e furt her damage (e.g., al cohol ) s houl d
Pa g e 1 0 4 7
ABC Ambe r CHM Conve rte r Tria l ve rsion, http://w w w .proce sste x t.com/a bcchm.html
be wi t hdrawn. o
o
a. Cont rol of pai n may requi re narcot i c anal ges i cs , but care mus t be t aken t o avoi d addi ct i on. W i t h abs t i nence from al cohol over t i me, s ome pat i ent s experi ence a l es s eni ng of pai n.
o
o
b. Repl acement of pancreat i c enzymes may be i ndi cat ed for t he t reat ment of s t eat orrhea or for t he rel i ef of pai n. Ant aci ds or H 2 -recept or ant agoni s t s may i ncreas e t he effect i venes s of oral enzyme preparat i ons .
o
o
c. Ins ul i n may be needed i n advanced cas es .
o
o
d. MCTs , whi ch are more eas i l y abs orbed t han l onger-chai n fat t y aci ds , are oft en gi ven.
o
o
e. Surgery i s a l as t res ort and general l y i s us ed for s evere pai n or recurrent , s evere at t acks . Subt ot al (80%) pancreat ect omy and t he Pues t ow procedure (i .e., anas t omos i s of t he pancreat i c duct l engt hwi s e t o a l oop of t he jejunum) are us ed mos t commonl y.
C. Neoplastic cystic lesions Neoplastic cystic lesions of t he pancreas i ncl ude s erous and muci nous cys t adenomas and adenocarci nomas . Thes e l es i ons are t rue cys t s of t he pancreas , wi t h mul t i pl e s mal l cys t s (s erous ) or l arge cys t s (muci nous ). Thes e are not s een t o communi cat e wi t h t he pancreat i c duct on ERCP. They do not cont ai n amyl as e, and t he s erum amyl as e l evel i s us ual l y normal (as di s cus s ed, t he s erum
Pa g e 1 0 4 8
ABC Ambe r CHM Conve rte r Tria l ve rsion, http://w w w .proce sste x t.com/a bcchm.html
amyl as e l evel i s el evat ed i n approxi mat el y 60%–75% of cas es of ps eudocys t s ). Cys t as s es s ment by endos copi c ul t ras ound wi t h fl ui d as pi rat i on can as s i s t i n del i neat i ng t he t ype of cys t pres ent . Treat ment of muci nous cys t adenomas i s s urgi cal removal —oft en wi t h compl et e cure.
D. Adenocarcinoma Adenocarcinoma of t he pancreas account s for more t han 90% of pancreat i c mal i gnanci es .
1. Epidemiology. The adenocarci noma i s duct al i n approxi mat el y 95% of pat i ent s . Thi s t umor, whi ch i ncreas ed i n i nci dence duri ng t he t went i et h cent ury, i s s econd t o col on carci noma as t he l eadi ng caus e of gas t roi nt es t i nal cancer–rel at ed deat h. Men are affect ed more commonl y t han women, and t he average age at pres ent at i on i s 55–65 years . Sevent y percent of adenocarci noma of t he pancreas occurs i n t he head of t he pancreas , wi t h 30% occurri ng i n t he body or t ai l .
2. Etiology. The ri s k of pancreat i c carci noma i s s i gni fi cant l y i ncreas ed i n pat i ent s wi t h heredi t ary pancreat i t i s , obese individuals, smokers, patients with chronic pancreatitis, and t he ri s k i s s l i ght l y i ncreas ed i n i ndi vi dual s wi t h di abet es mel l i t us . Chroni c pancreat i t i s i s al s o a known caus e of pancreat i c cancer.
3. Clinical features o
o
a. Common symptoms. Approxi mat el y 75% of pat i ent s have pai n t hat has been pres ent for 3–4 mont hs by t he t i me of di agnos i s . The pai n t ypi cal l y i s pos t prandi al epi gas t ri c or peri umbi l i cal
Pa g e 1 0 4 9
ABC Ambe r CHM Conve rte r Tria l ve rsion, http://w w w .proce sste x t.com/a bcchm.html
di s comfort , whi ch radi at es t o t he back and i s rel i eved by s i t t i ng up or bendi ng bot h knees . Jaundi ce i s pres ent i n approxi mat el y 65% of pat i ent s , and wei ght l os s occurs i n 60% of pat i ent s . Di arrhea and s t eat orrhea al s o are s omewhat common. The gal l bl adder may be pal pabl e (Courvoi s i er's s i gn) i n s ome pat i ent s . A pal pabl e epi gas t ri c mas s may be found on phys i cal exami nat i on. o
o
b. Les s common s ympt oms i ncl ude unexpl ai ned t hrombophl ebi t i s (Trous s eau's s i gn), depres s i on, t he new ons et of di abet es mel l i t us , or acut e pancreat i t i s .
P.216
4. Diagnosis. A hi gh i ndex of s us pi ci on oft en i s requi red i n pat i ent s wi t h cons t ant epi gas t ri c or peri umbi l i cal di s t res s . o
o
a. Laboratory tests are us ual l y nons peci fi c. Hi gh s erum amyl as e l evel s are found i n 25% of pat i ent s . CA 19–9, a t umor marker, has been as s oci at ed wi t h carci noma of t he pancreas . Al t hough not us eful as a s creeni ng t es t , i t has an approxi mat e 80% s ens i t i vi t y and 90% s peci fi ci t y for carci noma of t he pancreas and may be a marker for recurrent di s eas es or met as t as i s aft er pri mary res ect i on.
o
o
b. Spiral CT , magnetic resonance imaging (MRI),
Pa g e 1 0 5 0
ABC Ambe r CHM Conve rte r Tria l ve rsion, http://w w w .proce sste x t.com/a bcchm.html
or ultrasonography of t he pancreas demons t rat es a mas s i n 75%–80% of pat i ent s . Image-gui ded needl e bi ops y can provi de a hi s t ol ogi c di agnos i s . o
o
c. ERCP i s abnormal i n approxi mat el y 85%–90% of pat i ent s and general l y s hows a di s cret e s t ri ct ure i n t he mai n pancreat i c duct wi t h proxi mal di l at at i on. Thi s procedure al l ows bi ops i es t o be t aken.
o
o
d. EUS vi s ual i zes mos t t umors and can provi de i mport ant s t agi ng i nformat i on. EUS-gui ded fi ne needl e as pi rat i on provi des a hi s t ol ogi c di agnos i s .
o
o
e. Angiography may s how di s pl acement or encas ement of t he pancreat i c or duodenal art eri es . The venous phas e may be es peci al l y us eful i f t he s uperi or mes ent eri c vei n or s pl eni c vei n i s occl uded.
5. T herapy and prognosis (s ee Chapt er 4 VII B 7, 9)
E. Islet cell tumors Islet cell tumors account for 5% of pancreat i c adenocarci nomas . They may be mul t i cent ri c and t end t o grow more s l owl y t han t umors of duct ul ar ori gi n. Is l et cel l t umors frequent l y are as s oci at ed wi t h endocri ne adenomas i n t he pi t ui t ary and parat hyroi d gl ands (e.g., i n MEN I s yndrome).
1. Gastrinoma caus es t he Zol l i nger-El l i s on s yndrome
Pa g e 1 0 5 1
ABC Ambe r CHM Conve rte r Tria l ve rsion, http://w w w .proce sste x t.com/a bcchm.html
(s ee III I).
2. Insulinoma i s charact eri zed by i nappropri at el y hi gh i ns ul i n l evel s i n t he pres ence of hypogl ycemi a. Becaus e onl y 10%–15% of i ns ul i nomas are mal i gnant , s urgi cal res ect i on i s t he t reat ment of choi ce. Synt het i c s omat os t at i n may be an effect i ve t reat ment .
3. Glucagonoma i s found i n pat i ent s wi t h a s yndrome of di abet es mel l i t us , wei ght l os s , anemi a, and a charact eri s t i c ras h (mi grat ory necrol yt i c eryt hema). Mos t gl ucagonomas are mal i gnant , but s urgi cal debul ki ng may provi de s ympt omat i c i mprovement . St rept ozoci n i s t he mos t commonl y us ed chemot herapeut i c agent . Somat os t at i n may be us ed.
4. Somatostatinoma us ual l y occurs i n as s oci at i on wi t h t he t ri ad of di abet es , s t eat orrhea, and gal l s t ones . Approxi mat el y 50% of pat i ent s have a pos i t i ve fami l y hi s t ory of i s l et cel l t umors . The di agnos i s us ual l y i s made ei t her i nci dent al l y at s urgery for anot her probl em (e.g., chol ecys t i t i s ) or l at e i n t he cours e of t he di s eas e when met as t at i c di s eas e i s pres ent . St rept ozoci n t herapy has been effect i ve i n a s mal l number of pat i ent s .
5. VIPoma (al s o cal l ed pancreatic cholera, Verner-Morrison syndrome, and t he watery diarrhea, hypokalemia, and achlorhydria syndrome) i s a t umor of non-α, non-β i s l et cel l s t hat s ecret e VIP, whi ch caus es wat ery di arrhea. Sol i t ary l es i ons may be cured by s urgi cal res ect i on. Some i ndi vi dual s are res pons i ve t o cort i cos t eroi ds , and s t rept ozoci n has been us ed
Pa g e 1 0 5 2
ABC Ambe r CHM Conve rte r Tria l ve rsion, http://w w w .proce sste x t.com/a bcchm.html
s ucces s ful l y i n pat i ent s wi t h met as t at i c di s eas e. Synt het i c s omat os t at i n al s o may be us ed.
VIII. Diseases of the Biliary Tract A. Gallstones Gallstones are ext remel y common, occurri ng i n 15%–20% of t he popul at i on of t he Uni t ed St at es .
1. T ypes o
o
a. Cholesterol gallstones. Mos t gal l s t ones t hat occur i n wes t ern popul at i ons are compos ed pri mari l y of chol es t erol , whi ch i s t hought t o preci pi t at e from s upers at urat ed bi l e, es peci al l y at ni ght when bi l e i s concent rat ed i n t he gal l bl adder. For women, t he ri s k of chol es t erol gal l s t ones i ncreas es wi t h age, us e of oral cont racept i ves (at l eas t duri ng t he fi rs t 5 years of us e), or pregnancy. Rapi d wei ght l os s ; fami l y hi s t ory of di abet es mel l i t us ; i l eal di s eas e (Crohn's di s eas e), or i l eal res ect i on res ul t i ng i n a decreas ed bi l e s al t pool ; t he us e of agent s s uch as cl ofi brat e, ceft ri axone, or oct reot i de; or t ot al parent eral nut ri t i on al l may i ncreas e t he ri s k of chol es t erol gal l s t ones .
o
o
b. Pigmented gallstones. Compos ed pri mari l y of cal ci um bi l i rubi nat e, pi gment ed gal l s t ones are found i n pat i ent s wi t h chroni c hemol ys i s (e.g., s i ckl e cel l di s eas e) as wel l as i n As i an popul at i ons . In As i a, bi l i ary i nfect i on wi t h β-gl ucuroni das e–produci ng organi s ms l eads t o i ncreas ed amount s of poorl y s ol ubl e deconjugat ed bi l i rubi n i n bi l e.
Pa g e 1 0 5 3
ABC Ambe r CHM Conve rte r Tria l ve rsion, http://w w w .proce sste x t.com/a bcchm.html
2. T herapy. One-t hi rd t o one-hal f of pat i ent s wi t h gal l s t ones are as ympt omat i c and s houl d be t reat ed expect ant l y. Surgi cal removal of as ympt omat i c gal l s t ones i s unneces s ary, except i n di abet i c pat i ent s , i n whom t he ri s k of acut e chol ecys t i t i s wi t h compl i cat i ons i s hi gh.
B. Acute cholecystitis
1. Etiology. In 90%–95% of cas es , acut e chol ecys t i t i s i s caus ed by obs t ruct i on of t he cys t i c duct by an i mpact ed gal l s t one, whi ch l eads t o edema of t he gal l bl adder wal l wi t h s ubmucos al hemorrhage and mucos al ul cerat i on. Pol ymorphonucl ear i nfi l t rat i on i s a l at er event and probabl y i s at t ri but abl e t o t he l ow bact eri al count of t he obs t ruct ed gal l bl adder. Acal cul ous chol ecys t i t i s may occur s econdary t o s al monel l os i s , pol yart eri t i s nodos a, s eps i s , and t rauma.
2. Clinical features o
o
a. An at t ack of acut e chol ecys t i t i s s t art s wi t h crampy pai n i n t he epi gas t ri um or ri ght upper quadrant , whi ch may radi at e t o t he back near t he ri ght s capul ar t i p (bi l i ary col i c). The pai n i s t hought t o be generat ed by duct al obs t ruct i on and oft en i s pos t prandi al , t ypi cal l y s ubs i di ng wi t hi n s everal hours .
o
o
b. An el evat ed t emperat ure or W BC count , fever, naus ea, vomi t i ng, and i l eus al s o may be pres ent .
Pa g e 1 0 5 4
ABC Ambe r CHM Conve rte r Tria l ve rsion, http://w w w .proce sste x t.com/a bcchm.html
P.217
o
o
c. Ri ght upper quadrant t endernes s preci pi t at ed by deep i ns pi rat i on duri ng pal pat i on of t he ri ght upper quadrant i s known as Murphy' s sign.
o
o
d. Jaundi ce occurs i n 20% of pat i ent s and i s t hought t o be at t ri but abl e t o common duct s t ones or edema of t he common bi l e duct .
3. Diagnosis. The charact eri s t i c cl i ni cal pi ct ure, es peci al l y i n a pat i ent known t o have gal l s t ones , s ugges t s t he di agnos i s . o
o
a. Mos t gal l s t ones cons i s t of chol es t erol and are radi ol ucent ; 10%–15% of gal l s t ones cont ai n enough cal ci um t o appear radi opaque.
o
o
b. Al t hough gal l bl adder ul t ras onography can s how t he pres ence of s t ones (i .e., t he fl ui d-fi l l ed gal l bl adder appears l ucent , whereas t he s t ones wi t hi n i t are s ono-opaque and cas t s hadows ), t hi s t es t cannot be us ed t o demons t rat e cys t i c duct obs t ruct i on.
o
o
c. Fai l ure t o vi s ual i ze t he gal l bl adder duri ng radi onucl i de s canni ng aft er an i nt ravenous i nject i on of i mi nodi acet i c aci d (HIDA s canni ng) s t rongl y s ugges t s cys t i c duct obs t ruct i on. It may s how
Pa g e 1 0 5 5
ABC Ambe r CHM Conve rte r Tria l ve rsion, http://w w w .proce sste x t.com/a bcchm.html
gal l bl adder wal l t hi ckeni ng and peri chol ecys t i c fl ui d. Cys t i c duct obs t ruct i on i s al s o s ugges t ed when oral chol ecys t ography fai l s t o vi s ual i ze t he gal l bl adder, but chol ecys t ography i s not as rel i abl e as HIDA s canni ng.
4. T herapy o
o
a. Supportive treatment s houl d be provi ded i ni t i al l y, wi t h i nt ravenous fl ui d repl acement and nas ogas t ri c s uct i on for 24–48 hours . Lat er, t he gal l bl adder may be removed s urgi cal l y.
o
o
b. Laparoscopic cholecystectomy may al l ow gal l bl adder removal on a “s ame-day s urgery― bas i s i n many cas es . It i s general l y t he t reat ment of choi ce.
o
o
c. Other less commonly used therapies i ncl ude:
(1) Dissolution may be us ed t o di s s ol ve chol es t erol s t ones i n pat i ent s who are not s urgi cal candi dat es . Ursodeoxycholic acid or chenodeoxycholic acid may be us ed. If s everal s mal l s t ones are pres ent and fl oat i ng, a 50%–70% chance of di s s ol vi ng t he s t ones may be expect ed over a peri od of 12–24 mont hs .
(2) Lithotripsy may be t ri ed i f t he gal l bl adder i s funct i onal , t he s t one mas s i s l es s t han 3
Pa g e 1 0 5 6
ABC Ambe r CHM Conve rte r Tria l ve rsion, http://w w w .proce sste x t.com/a bcchm.html
cm, and t he pat i ent has no acut e s ympt oms (approxi mat el y 20%–25% of pat i ent s ).
(3) Percutaneous introduction of methylterbutaline ether al s o has been us ed i n pat i ent s who refus e chol ecys t ect omy.
5. Complications. Surgi cal i nt ervent i on general l y i s requi red. o
o
a. Empyema refers t o a pus -fi l l ed gal l bl adder. Pat i ent s may be t oxi c and are at hi gh ri s k for perforat i on.
o
o
b. Perforation
(1) Local i zed perforat i on occurs s everal days t o 1 week aft er t he ons et of acut e chol ecys t i t i s and l eads t o a peri chol ecys t i c abs ces s .
(2) Free perforat i on i nt o t he abdomi nal cavi t y, whi ch has a 25% mort al i t y rat e, occurs earl y i n t he cl i ni cal cours e, probabl y becaus e i nfl ammat i on i n t he earl y s t ages i s i ns uffi ci ent t o wal l off t he abs ces s .
(3) Perforat i on i nt o an adjacent organ may i nvol ve t he duodenum, jejunum, col on, or s t omach. If a l arge s t one i s pas s ed i nt o t he
Pa g e 1 0 5 7
ABC Ambe r CHM Conve rte r Tria l ve rsion, http://w w w .proce sste x t.com/a bcchm.html
l umen, i nt es t i nal obs t ruct i on (gal l s t one i l eus ) may res ul t . o
o
c. Emphysematous cholecystitis i s caus ed by gas -formi ng bact eri a (oft en cl os t ri di a, E. c ol i , or s t rept ococci ) i n t he gal l bl adder l umen and wal l . Men are affect ed more commonl y t han women, and 20%–30% of pat i ent s have di abet es mel l i t us . Earl y s urgi cal i nt ervent i on i s i ndi cat ed t o prevent perforat i on.
o
o
d. Postcholecystectomy syndrome refers t o abdomi nal pai n t hat pers i s t s aft er chol ecys t ect omy. The us ual caus e i s an i ni t i al l y mi s t aken di agnos i s , wi t h pai n pers i s t i ng from t he underl yi ng proces s (e.g., pancreat i c di s eas e and i rri t abl e bowel s yndrome). Some pat i ent s may have common duct s t ones .
C. Chronic cholecystitis Chronic cholecystitis i s a cl i ni cal t erm us ed t o des cri be a condi t i on of recurrent s ubacut e s ympt oms caus ed by gal l s t ones . Pat i ent s wi t h chroni c chol ecys t i t i s s how wi de vari abi l i t y i n t he t hi ckeni ng and fi bros i s of t he gal l bl adder wal l and i n t he i nfl ammat ory i nfi l t rat e. The di agnos i s i s bas ed on s ympt oms and l ow gal l bl adder eject i on fact or meas ured by a quant i t at i ve nucl ear hepat obi l i ary P.218 s can. Aft er ot her s ources of chroni c abdomi nal pai n (e.g., pept i c ul cer di s eas e, pancreat i t i s , and i rri t abl e bowel s yndrome) are rul ed out , a chol ecys t ect omy may be performed t o rel i eve s ympt oms .
D. Choledocholithiasis
Pa g e 1 0 5 8
ABC Ambe r CHM Conve rte r Tria l ve rsion, http://w w w .proce sste x t.com/a bcchm.html
1. Pathophysiology. Chol edochol i t hi as i s us ual l y occurs when a gal l s t one i s pas s ed i nt o t he common duct from t he gal l bl adder or when a gal l s t one t hat was mi s s ed duri ng operat i ve chol angi ography or common duct expl orat i on i s ret ai ned. Occas i onal l y, a s t one forms de novo i n t he common duct , es peci al l y when t here i s s t as i s from duct al obs t ruct i on.
2. Clinical features. Sympt oms frequent l y are i nt ermi t t ent and i ncl ude col i cky pai n i n t he ri ght upper quadrant . Fever, chi l l s , and jaundi ce accompani ed by el evat ed s erum l evel s of al kal i ne phos phat as e and t he t rans ami nas es i ndi cat e t he devel opment of chol angi t i s . Seps i s may res ul t from as cendi ng chol angi t i s , whi ch i s a cl os ed-s pace i nfect i on.
3. T herapy. Ant i bi ot i cs are gi ven as needed t o cont rol i nfect i on, but defi ni t i ve t reat ment cons i s t s of s urgi cal removal of t he s t one or endos copi c s phi nct erot omy and s t one ext ract i on.
E. Biliary dyskinesia Biliary dyskinesia (s phi nct er of Oddi dys funct i on) i s a cl i ni cal s yndrome of ri ght upper quadrant s ympt oms s i mi l ar t o chroni c cal cul ous chol ecys t i t i s .
1. Pathophysiology. An abnormal i t y of bi l i ary mot or funct i on i s propos ed, and manomet ri c fi ndi ngs may i ndi cat e el evat ed bas al s phi nct er of Oddi pres s ure (us ual l y 40 mm Hg great er t han i nt raduodenal pres s ure), a paradoxi cal cont ract i on of t he s phi nct er of Oddi aft er chol ecys t oki ni n i nject i on, or bot h.
Pa g e 1 0 5 9
ABC Ambe r CHM Conve rte r Tria l ve rsion, http://w w w .proce sste x t.com/a bcchm.html
2. T herapy. Pat i ent s may res pond t o s moot h mus cl e rel axant s (e.g., ni t rat es , cal ci um channel -bl ocki ng agent s ) or t o endos copi c or s urgi cal s phi nct erot omy of t he s phi nct er of Oddi .
F. Biliary stricture
1. Pathophysiology. Bi l i ary s t ri ct ure i s a narrowi ng of t he common bi l e duct general l y caus ed by s urgi cal i njury or s carri ng s ubs equent t o expl orat i on of t he common bi l e duct . Rarel y, t rauma or chol edochol i t hi as i s may res ul t i n a bi l i ary s t ri ct ure.
2. Clinical features. Pat i ent s us ual l y have i nt ermi t t ent obs t ruct i ve jaundi ce s everal weeks t o mont hs aft er bi l i ary t ract s urgery. Chol angi ography demons t rat es t he pres ence of a s moot h concent ri c narrowi ng of t he duct , wi t h proxi mal di l at at i on.
3. T herapy. The us ual t reat ment i s s urgi cal anas t omos i s of t he di l at ed proxi mal end of t he bi l e duct t o t he i nt es t i ne, but s ome pat i ent s may undergo percut aneous t rans hepat i c or endos copi c bal l oon di l at at i on wi t h bi l i ary s t ent pl acement .
G. Sclerosing cholangitis Sclerosing cholangitis, a rare di s eas e t hat caus es progres s i ve narrowi ng of t he bi l e duct s , general l y i s di agnos ed i n t he t hi rd or fourt h decade of l i fe and i s t hree t i mes more common i n men t han i n women.
1. Clinical features. Approxi mat el y 70% of pat i ent s
Pa g e 1 0 6 0
ABC Ambe r CHM Conve rte r Tria l ve rsion, http://w w w .proce sste x t.com/a bcchm.html
have i nfl ammat ory bowel di s eas e (us ual l y ul cerat i ve col i t i s ), but t he bi l i ary and i nt es t i nal di s eas es have i ndependent cl i ni cal cours es . The us ual pres ent i ng s ympt om i s pruri t us . There i s an i ncreas ed ri s k of chol angi ocarci noma i n s uch pat i ent s .
2. Diagnosis. Earl y di agnos i s i s pos s i bl e i n as ympt omat i c pat i ent s who s how marked el evat i on of s erum al kal i ne phos phat as e l evel s on rout i ne bi ochemi cal s creeni ng. ERCP or percut aneous chol angi ography s houl d es t abl i s h t he di agnos i s .
3. T herapy o
o
a. Medical treatment. Cort i cos t eroi ds , urs odeoxychol i c aci d, met hot rexat e, l ong-t erm ant i bi ot i cs , or varyi ng combi nat i ons of t hes e medi cat i ons have been us ed wi t h varyi ng s ucces s .
o
o
b. Surgical treatment. Surgi cal anas t omos i s of t he di s eas ed duct t o t he i nt es t i ne may be di ffi cul t or i mpos s i bl e, but l i ver t rans pl ant at i on i s a cons i derat i on i n many pat i ent s . Endos copi c or percut aneous di l at at i on of s t ri ct ures i s pos s i bl e i n s ome cas es .
H. Oriental cholangitis Oriental cholangitis i s a di s eas e of As i an popul at i ons or fi rs t -generat i on i mmi grant s from As i a. Inci dence rat es for men and women are equal . The et i ol ogi c fact ors may be bi l e s t as i s (oft en as s oci at ed wi t h paras i t i c i nfes t at i on) and a l ow-prot ei n di et . The
Pa g e 1 0 6 1
ABC Ambe r CHM Conve rte r Tria l ve rsion, http://w w w .proce sste x t.com/a bcchm.html
di s eas e i s charact eri zed by recurrent ri ght upper quadrant pai n, fever, jaundi ce, and bi l e duct –pi gment ed s t ones . Infect ed bi l e i s oft en found at s urgery. Treat ment i s wi t h s urgery and s urgi cal drai nage of t he bi l e duct s .
I. Cystic malformation of the bile ducts P.219
1. Choledochal cysts may pres ent as jaundi ce, chol angi t i s , or a l arge cys t fi l l ed wi t h numerous s t ones . Di agnos i s may be made by chol angi ography. Surgery i s us ed t o exci s e t he cys t or t o anas t omos e t he cys t t o t he i nt es t i ne.
2. Caroli' s disease i s charact eri zed by s accul ar di l at i on of t he i nt rahepat i c duct s , whi ch may be as s oci at ed wi t h ri ght upper quadrant pai n, chol angi t i s , or bot h becaus e of duct al s t one format i on. Hepat i c fi bros i s wi t h port al hypert ens i on may devel op, es peci al l y i n pat i ent s wi t h medul l ary s ponge ki dney. Recent l y, chol angi ocarci noma has been report ed i n congeni t al bi l i ary cys t s (up t o 30%), es peci al l y i f s t ones are pres ent i n t he cys t . Surgi cal decompres s i on occas i onal l y i s hel pful , and ant i bi ot i cs are us ed duri ng acut e epi s odes of chol angi t i s .
J. Tumors of the gallbladder
1. Adenocarcinoma of t he gal l bl adder i s a di s eas e of older women. The t umor affect s t hree t i mes as many
Pa g e 1 0 6 2
ABC Ambe r CHM Conve rte r Tria l ve rsion, http://w w w .proce sste x t.com/a bcchm.html
women as i t does men, and t he average age at di agnos i s i s 65–75 years . Al t hough mos t pat i ent s have as s oci at ed gal l s t ones , cancer devel ops i n l es s t han 1% of al l pat i ent s wi t h s t ones . Sympt oms general l y mi mi c t hos e of acut e or chroni c chol ecys t i t i s . On phys i cal exami nat i on, a mas s may be pal pabl e i n t he ri ght upper quadrant , and obs t ruct i ve jaundi ce may be s een s econdary t o l ocal s pread of t he t umor t o t he common bi l e duct . A cal ci fi ed gal l bl adder may be s een on abdomi nal radi ographs . An operat i on cons i s t i ng of chol ecys t ect omy, l ymph node di s s ect i on, and removal of a s mal l port i on of t he adjacent l i ver i s i ndi cat ed i f no obvi ous met as t at i c di s eas e i s found, but prognos i s general l y i s poor.
2. Benign tumors of t he gal l bl adder i ncl ude abnormal i t i es of t he mucos al l i ni ng (e.g., adenomat ous hyperpl as i a, chol es t erol os i s , and chol es t erol pol yps ), cys t i c changes i n t he gl ands , and papi l l ary adenomas . Thes e l es i ons us ual l y are as ympt omat i c. In s ome pat i ent s wi t h no ot her demons t rabl e caus es for abdomi nal pai n, however, chol ecys t ect omy has provi ded rel i ef of s ympt oms .
K. Tumors of the bile duct T umors of the bile duct Adenocarci noma of t he bi l e duct (chol angi ocarci noma) i s t ypi cal l y a di s eas e of older men and i s not as s oci at ed wi t h gal l s t ones .
1. Etiology. An i ncreas ed ri s k i s obs erved i n pat i ent s wi t h ul cerat i ve col i t i s who have s cl eros i ng chol angi t i s and i n pat i ent s expos ed t o benz ene or t ol uene deri vat i ves . Paras i t i c i nfect i on, es peci al l y wi t h Cl onorc hi s , of t he bi l i ary s ys t em has been l i nked t o t he hi gh rat e of chol angi ocarci noma i n As i an popul at i ons .
Pa g e 1 0 6 3
ABC Ambe r CHM Conve rte r Tria l ve rsion, http://w w w .proce sste x t.com/a bcchm.html
2. Pathology. Mos t t umors are of t he s ci rrhous or papi l l ary t ype. An ext ens i ve des mopl as t i c react i on may make di agnos i s di ffi cul t . Two-t hi rds of t he t umors are l ocat ed i n t he common bi l e duct or at t he bi furcat i on of t he common hepat i c duct (Klatskin tumors).
3. Clinical features. Jaundi ce, wi t h or wi t hout pai n, i s pres ent i n mos t pat i ent s , and wei ght l os s al s o i s common. Pruri t us may be s evere. Common duct t umors caus i ng obs t ruct i on di s t al t o t he cys t i c duct res ul t i n a pal pabl e gal l bl adder t hat i s not t ender.
4. Laboratory data. Serum al kal i ne phos phat as e l evel s are markedl y i ncreas ed, as are di rect and t ot al bi l i rubi n l evel s . Serum t rans ami nas e l evel s s how s mal l er i ncreas es and general l y are l es s t han 200 mg/dL.
5. Diagnosis. Di l at i on of t he i nt rahepat i c duct s i s s hown by ul t ras onography or CT, and percut aneous chol angi ography or ERCP fi ndi ngs general l y s ugges t t he di agnos i s . The di fferent i al di agnos i s i ncl udes pancreat i c carci noma, chol edochol i t hi as i s , bi l i ary s t ri ct ure, and s cl eros i ng chol angi t i s .
6. T herapy (s ee Chapt er 4 VI E 9)
IX. Diseases of the Liver A. Acute liver disease (Table 5-3)
Pa g e 1 0 6 4
ABC Ambe r CHM Conve rte r Tria l ve rsion, http://w w w .proce sste x t.com/a bcchm.html
1. Acute viral hepatitis, one of t he mos t common heal t h probl ems i n t he worl d, i s caus ed by any one of s everal vi rus es . o
o
a. Etiology
(1) Hepatitis A virus (HAV) i s an RNA vi rus t rans mi t t ed pri mari l y by t he fecal –oral rout e. Acut e i nfect i on i s ani ct eri c i n 50% of cas es . HAV does not l ead t o chroni c di s eas e or t o a carri er s t at e.
(2) Hepatitis B virus (HBV) i s a DNA vi rus t rans mi t t ed parent eral l y. Indi vi dual s at hi gh ri s k i ncl ude i nt ravenous drug abus ers , homos exual men, and t hos e expos ed t o bl ood and bl ood product s . Chroni c HBV di s eas e or a pers i s t ent carri er s t at e devel ops i n 1%–20% of pat i ent s worl dwi de dependi ng on t he mode of t rans mi s s i on and age at i nfect i on.
(3) Hepatitis C virus (HCV) i s an RNA vi rus t hat formerly accounted for 90% of pos t t rans fus i on hepat i t i s . The modes of t rans mi s s i on (parent eral , s exual , and, perhaps , peri nat al ) are s i mi l ar t o t hos e of HBV. Chroni c hepat i t i s devel ops i n up t o 85% of pat i ent s . Ci rrhos i s may devel op i n 20% of pat i ent s , wi t h an i ncreas ed ri s k of hepat ocel l ul ar carci noma. Genot ype I P.220
Pa g e 1 0 6 5
ABC Ambe r CHM Conve rte r Tria l ve rsion, http://w w w .proce sste x t.com/a bcchm.html
P.221
account s for mos t hepat i t i s C pat i ent s i n t he Uni t ed St at es and does not res pond as readi l y t o ant i vi ral /i nt erferon t herapy as ot her genot ypes .
TABLE 5-3 Comparison of Viral Hepatitis Types Delta Agent ( H H H H e H H e e e p e e p p p at p p at at at iti at at iti iti iti s iti iti s s s D s s A B C ) E G Ty R D R R R R pe N N N N N N of A A A A A A vir
(h
us
e p at iti s B re q ui re
Pa g e 1 0 6 6
ABC Ambe r CHM Conve rte r Tria l ve rsion, http://w w w .proce sste x t.com/a bcchm.html
d fo r re pl ic at io n) Inc 4 1 2 L V L ide 0 0 â o er o nc
€“ w y w
e
3
of
lo w
po siti ve ant ibo dy in the Uni ted Sta tes (% ) Inc 0 1 6 ? â â ide t o 0 0
€¦ €¦
nc e â â e
xt €“ €“
aft re 2 9 er m 0 0 blo el
Pa g e 1 0 6 7
ABC Ambe r CHM Conve rte r Tria l ve rsion, http://w w w .proce sste x t.com/a bcchm.html
od y tra ra nsf re usi on (% ) Inc 2 1 2 1 2 2 ub â â â â â â ati €“ €“ €“ €“ €“ €“ on 6 6 2 6 8 2 pe w m 4 m w 4 rio e o w o e w d
e nt e nt e e ks h e h ks e
s ks s ks Inf L D ? ? ? ? ect a ur ivit s t i n y
3 g
sta w h ge e e e p ks at of i t i in s oc B ul s at ur i o fa n ce to a 1 nt â ig €“ e
Pa g e 1 0 6 8
ABC Ambe r CHM Conve rte r Tria l ve rsion, http://w w w .proce sste x t.com/a bcchm.html
2 n w p e o e si ks t i af vi te ty r ja u n di ce oc cu rs Co F F F Pr F C mp ul ul ul e ul hr lica m m m s m o tio i n i n i n e i n ni ns a a a nt a c nt nt nt i n nt ca h h h 2 h rri e e e 0 e er p p p % p st at at at â at at i t i i t i i t i €“ i t i e s s s 5 s s; (r (r (r 0 i n fu ar ar ar % pr l e) e e of e m b b ca g i n ut ut s n a m m e a nt or or s nt h
Pa g e 1 0 6 9
ABC Ambe r CHM Conve rte r Tria l ve rsion, http://w w w .proce sste x t.com/a bcchm.html
e e of w e co co fu o p mml
m at
m m m e iti o o in n s n n a wi t h t h nt t h a a h 1 n n e 0 wi wi p % t h t h at â h h i t i €“ e e s 2 p p a 0 at at n % i t i i t i d/ m s s or or A) A) ch t a ; ; ro l i t ch ch ni y ro ro c ni ni ac c c ti ac ac v ti ti e v v h e e e h h p e e at p p iti at at s iti iti s s Me F P P P F P ch ec ar ar ar ec ar
Pa g e 1 0 7 0
ABC Ambe r CHM Conve rte r Tria l ve rsion, http://w w w .proce sste x t.com/a bcchm.html
ani al e e e al e sm â nt nt nt â nt of €“ er er er €“ er spr or al al al or al ea al
al
d Pr P H P ? G ? ev o e o
o
ent ol p ol
o
ion e at e
d
d iti d
h
i
y
s i
mB m
gi
mi
e
m
u mu
n
n mn
e
e u e s n s er e er u gl u mo m gl b gl o ul o b in b ul ; ul in v in ; ac v ci ac n ci at n io at n io n Ca R 1 Y ? ? Y
Pa g e 1 0 7 1
ABC Ambe r CHM Conve rte r Tria l ve rsion, http://w w w .proce sste x t.com/a bcchm.html
rri ar % e
e
er e, â s
s
sta i f €“ (5 te e 2 0 v 0 % er % of w p or at ld ie wi nt d s) e
(4) Delta hepatitis [hepatitis D virus (HDV)] i s caus ed by a s mal l , defect i ve RNA vi rus (del t a agent ) t hat i s i nfect i ous onl y i n t he pres ence of HBV i nfect i on, becaus e i t rel i es on HBV prot ei ns for repl i cat i on. It t herefore can compl i cat e acut e HBV i nfect i on but i s s een more commonl y as a “s uperi nfect i on― wi t h an i ncreas e i n abnormal l i ver funct i on t es t s i n a pat i ent wi t h chroni c HBV. HDV general l y has a chroni c, s evere cl i ni cal cours e.
(5) Hepatitis E virus i s a s mal l RNA vi rus (pos s i bl y a cal i ci vi rus ) t hat has been des cri bed i n cas es of acut e hepat i t i s i n Mexi co, As i a, and Afri ca. It has a s hort i ncubat i on peri od and i s probabl y wat erborne. Chol es t as i s i s rel at i vel y common.
(6) Hepatitis G virus i s an RNA vi rus wi t h a
Pa g e 1 0 7 2
ABC Ambe r CHM Conve rte r Tria l ve rsion, http://w w w .proce sste x t.com/a bcchm.html
genomi c organi zat i on s i mi l ar t o t hat of HCV. Hepat i t i s G i s t rans mi t t ed parent eral l y and account s for approxi mat el y 15% of non-A, non-B, and non-C cas es of chroni c hepat i t i s . It i s found more frequent l y i n i nt ravenous drug abus ers and pat i ent s on hemodi al ys i s .
(7) Other viruses t hat can caus e acut e hepat i t i s i ncl ude t he Eps t ei n-Barr vi rus , CMV, HSV, and t hos e caus i ng yel l ow fever and rubel l a.
o
o
b. Pathology. The l es i ons of acut e vi ral hepat i t i s are s i mi l ar regardl es s of et i ol ogy and i ncl ude mononucl ear cel l i nfi l t rat i on, cel l ul ar bal l ooni ng and necros i s , and condens ed cyt opl as m wi t h pyknot i c nucl ei (aci dophi l i c bodi es ).
o
o
c. Clinical features
(1) Mal ai s e, anorexi a, and fat i gue
(2) Art hri t i s and urt i cari a, whi ch are es peci al l y common i n HBV, are as cri bed t o ci rcul at i ng i mmune compl exes . Pol yart eri t i s nodos a or gl omerul onephri t i s may be s een i n HBV or HCV pat i ent s .
(3) Infl uenz a-l i ke s yndrome, whi ch i s es peci al l y common i n HAV
Pa g e 1 0 7 3
ABC Ambe r CHM Conve rte r Tria l ve rsion, http://w w w .proce sste x t.com/a bcchm.html
(4) Jaundi ce (wi t h dark uri ne or l i ght s t ool s ), whi ch i s s een i n 50% of cas es
(5) Hepat i c enl argement or t endernes s
(6) Spl enomegal y, whi ch occurs i n 20% of pat i ent s
(7) HCV onl y: concomi t ant porphyri a cut anea t arda, lichen planus, mi xed cryogl obul i nemi a, thyroiditis, or membranoprol i ferat i ve gl omerul onephri t i s . Immune compl ex di s eas e i s rel at i vel y common i n HCV pat i ent s .
o
o
d. Diagnosis. Acut e vi ral hepat i t i s i s di agnos ed bas ed on cl i ni cal feat ures as wel l as s uch l aborat ory fi ndi ngs as el evat ed l evel s of t he t rans ami nas es [i .e., as part at e ami not rans feras e (AST), al ani ne ami not rans ami nas e (ALT)], s erum bi l i rubi n, and s erum al kal i ne phos phat as e. The i ncreas e i n bi l i rubi n exceeds t he i ncreas e i n al kal i ne phos phat as e.
(1) In HAV, t he IgM ant i body i s el evat ed earl y i n t he cours e, fol l owed by an el evat i on of t he IgG ant i body i n 2–3 mont hs .
(2) In HBV, a pos i t i ve s urface ant i gen us ual l y i s di agnos t i c. However, becaus e t hi s i s an earl y fi ndi ng, i t may be neces s ary t o fol l ow
Pa g e 1 0 7 4
ABC Ambe r CHM Conve rte r Tria l ve rsion, http://w w w .proce sste x t.com/a bcchm.html
t he i ncreas e of IgM ant i core (HB c Ab) and l at er of ant i s urface ant i bodi es (HB s Ab)t o document acut e i nfect i on (Fi gure 5-1).
FIGURE 5-1 Typi cal cl i ni cal cours e of acut e hepat i t i s B vi rus (HBV) i nfect i on. Recombi nant and pool ed-pl as ma vacci nes agai ns t HBV do not t rans mi t act i ve hepat i t i s B s urface ant i gen (Hb s Ag) and do not cont ai n hepat i t i s B e ant i gen (Hb e Ag) or hepat i t i s core ant i gen. Cons equent l y, t he s erol ogi c profi l e of a vacci nat ed i ndi vi dual cons i s t s of hepat i t i s B s urface ant i body (ant i -HB s ) onl y. SGPT ALT, al ani ne ami not rans feras e; ant i -HB c , hepat i t i s B core ant i body; ant i -HB e , hepat i t i s B e ant i body.
(3) HCV can be di agnos ed by us i ng t echni ques t hat det ect vi ral RNA.
(4) HDV may be di agnos ed by an el evat ed del t a ant i body t i t er, oft en wi t h t he di s appearance of B s urface ant i gen from t he s erum. A pers i s t ent l y hi gh or s l owl y fal l i ng
Pa g e 1 0 7 5
ABC Ambe r CHM Conve rte r Tria l ve rsion, http://w w w .proce sste x t.com/a bcchm.html
hepat i t i s del t a ant i body i s s een i n chroni c s t at es . o
o
e. T herapy. Treat ment for acut e or act i ve di s eas e i s s upport i ve and i ncl udes i nt ravenous fl ui ds t o provi de hydrat i on, correct el ect rol yt e abnormal i t i es , and provi de cal ori c i nt ake i f naus ea and vomi t i ng are pres ent . Vi t ami n K s houl d be gi ven i f t he prot hrombi n t i me (PT) i s el evat ed. For chroni c cas es of HBV, t he prot eas e i nhi bi t or l ami vudi ne and adefovi r have been effect i ve i n cl eari ng act i ve vi ral repl i cat i on. For chroni c HCV, pegul at ed i nt erferon (PEG-IFN), i n combi nat i on wi t h ri bavi ri n, res ul t s i n s us t ai ned vi ral cl eari ng rat es of bet ween 30% and 70%, dependi ng on t he vi ral l oad and t he genomi c i dent i t y of t he vi rus (genot ype I res ponds l es s favorabl y).
o
o
f. Clinical course and complications. Nearl y al l cas es of acut e vi ral hepat i t i s are beni gn, wi t h mos t pat i ent s s howi ng normal res ul t s on l i ver funct i on t es t i ng by 8–10 weeks . However, compl i cat i ons may occur. P.222
(1) Fulminant hepatitis, a rare compl i cat i on of HAV and HCV, occurs i n 1%–2% of pat i ent s wi t h HBV. It i s an es peci al l y common compl i cat i on of del t a agent s uperi nfect i on i n pat i ent s wi t h chroni c hepat i t i s B ant i genemi a.
Pa g e 1 0 7 6
ABC Ambe r CHM Conve rte r Tria l ve rsion, http://w w w .proce sste x t.com/a bcchm.html
Pat i ent s us ual l y have progres s i ve jaundi ce, hepat i c encephal opat hy, and as ci t es . Hepat orenal s yndrome i s common. El evat ed PT i s an earl y s i gn. The i ni t i al l y el evat ed s erum t rans ami nas e l evel s l at er decl i ne, and l i ver s i ze decreas es as a res ul t of necros i s of t he l i ver parenchyma. The mort al i t y rat e vari es wi t h age and approaches 90%–100%, es peci al l y i n pat i ent s ol der t han 60 years of age.
(2) Chronic hepatitis i s a compl i cat i on of HBV and HCV, i n whi ch s erum t rans ami nas e l evel s are el evat ed for more t han 6 mont hs . Fi ft een percent of pat i ent s wi t h chroni c HCV may have hepat i t i s C ant i body but no act i vel y repl i cat i ng vi rus ; t hey are i ndi vi dual s who have recovered from acut e and chroni c HCV i nfect i on. Pat hol ogi cal l y, i nfl ammat i on, necros i s , and fi bros i s bri dgi ng port al areas or bet ween port al areas and cent ral vei ns occurs . The di s eas e may progres s t o ci rrhos i s . On phys i cal exami nat i on, pat i ent s may have s pl enomegal y, s pi der angi omat a, caput medus ae, and ot her s i gns of chroni c l i ver di s eas e. Li ver bi ops y i s neces s ary for di agnos i s .
(3) A chronic carrier state for hepat i t i s B s urface ant i gen exi s t s i n 0.2% of t he popul at i on of t he Uni t ed St at es . A carri er s t at e al s o exi s t s for HCV, becaus e bl ood donat ed by apparent l y normal i ndi vi dual s may t rans mi t t hi s di s eas e when t rans fus ed. Carri ers of HBV or HCV may have an i ncreas ed
Pa g e 1 0 7 7
ABC Ambe r CHM Conve rte r Tria l ve rsion, http://w w w .proce sste x t.com/a bcchm.html
ri s k of hepat oma.
(4) Cholestatic hepatitis may occur, part i cul arl y wi t h hepat i t i s E and rarel y wi t h HAV and HCV. Thi s condi t i on i s charact eri zed by t he al kal i ne phos phat as e l evel el evat ed di s proport i onat el y t o t he t rans ami nas e l evel . The cl i ni cal cours e i s t ypi cal of acut e vi ral hepat i t i s , but t he pres ent at i on mus t be di fferent i at ed from bi l i ary t ract obs t ruct i on.
(5) Aplastic anemia i s s een rarel y aft er acut e vi ral hepat i t i s . The mort al i t y rat e i s hi gh, and no t reat ment has proved effect i ve.
o
o
g. T herapy of acute disease. PEG-IFN al one or i n combi nat i on wi t h ri bavi ri n may cl ear hepat i t i s C vi rus aft er acut e i nfect i on (e.g., parent eral expos ure, bl ood t rans fus i on)
o
o
h. Prevention
(1) Immune serum globulin i s effect i ve when admi ni s t ered aft er expos ure t o HAV and al s o may be part i al l y prot ect i ve agai ns t HBV and HCV. P.223
(2) Hepatitis B immune globulin, whi ch i s
Pa g e 1 0 7 8
ABC Ambe r CHM Conve rte r Tria l ve rsion, http://w w w .proce sste x t.com/a bcchm.html
i mmunogl obul i n t hat cont ai ns hi gh t i t ers of ant i body t o HBV, conveys pas s i ve i mmuni t y and i s recommended aft er confi rmed expos ure t o HBV (e.g., from s ki n punct ure by a cont ami nat ed needl e).
(3) Vaccines
(a) HAV vaccine. An effect i ve vacci ne for prevent i on of HAV i s avai l abl e. It i s part i cul arl y recommended for i ndi vi dual s who t ravel t o endemi c areas or who have ot her forms of chroni c l i ver di s eas e. The vacci ne al s o may be effect i ve for s econdary prevent i on i n HAV hous ehol d cont act s .
(b) HBV vaccine. Thi s preparat i on of t he hepat i t i s B s urface ant i gen conveys act i ve i mmuni t y. It i s recommended for i ndi vi dual s at hi gh ri s k s uch as di al ys i s pat i ent s , medi cal pers onnel wi t h frequent expos ure t o bl ood product s , and i ndi vi dual s who are hepat i t i s B ant i body–negat i ve and who have had confi rmed expos ure t o HBV. Infant s are rout i nel y vacci nat ed.
2. Drug-induced liver disease may fol l ow expos ure t o vi rt ual l y any drug and mani fes t s as a vari et y of cl i ni cal s yndromes and hi s t ol ogi c fi ndi ngs (Tabl e 5-4). Any drug may i nduce l i ver di s eas e t hat overl aps t wo or more cat egori es of di s eas e mechani s ms .
Pa g e 1 0 7 9
ABC Ambe r CHM Conve rte r Tria l ve rsion, http://w w w .proce sste x t.com/a bcchm.html
TABLE 5-4 Agents of Drug-Induced Liver Disease Drugs Causing Immune -Mediat ed Reactio ns G r
C
a
h
n I r ul nf o Dr Dr
o la ni
ug ug
mm c
s
s
at m A
Ca Ca Vi o at ct usi usi r u o iv ng ng al s ry e h e
c
Alt p
h
er at h ol h Dir ed iti e e e ect Me s- p st p T o tab li at a at xici olis k iti si iti ty m e s s s Ace An H Al C A t a dro al l o hl ce mi ge ot p or t a no ns h ur pr m ph
a in o in
Pa g e 1 0 8 0
ABC Ambe r CHM Conve rte r Tria l ve rsion, http://w w w .proce sste x t.com/a bcchm.html
en
n ol m o e
a p zi h n e
e n Am Cor Is H C A i od t i c o y hl s aro os t ni dr or pi ne ero a al pr ri i ds zi a o n (?) d zi p n a e m id e As Es t O P Er Is pi ri rog x h yt o n
ens ac e hr ni ill n o a i n yl m zi b yc d ut i n a z o n
e Al c Et h P P E M oh an h h s t et ol
ol
e e ol h n n at yl yt yt e d oi oi
o
n n
p
a Car Int S Q Pr Ni
Pa g e 1 0 8 1
ABC Ambe r CHM Conve rte r Tria l ve rsion, http://w w w .proce sste x t.com/a bcchm.html
bo rav ul ui o t r n
en fo ni p of ous n di yl ur a n th a m e i o nt id
ur oi
e
ac n
s il t et t et V S T O rac rac al ul hi xy hl o ycl i pr fa a p ri d ne oi dr zi h e
c u d e ac g e ni id s s s at in
He avy me t al s Me t ho t re xat e Mu s hr oo m t ox i ns (ph al l oi d
Pa g e 1 0 8 2
ABC Ambe r CHM Conve rte r Tria l ve rsion, http://w w w .proce sste x t.com/a bcchm.html
in an d ph al l i n) Ni a ci n Ph os p hor us o
o
a. Direct toxicity by a chemi cal (e.g., carbon t et rachl ori de) or a met abol i t e (e.g., acet ami nophen) us ual l y repres ent s a dos e-rel at ed i njury. Ni aci n may caus e a dos e-rel at ed t oxi ci t y at great er t han 3 g/day i n crys t al l i ne form or 1–2 g/day i n s us t ai ned-rel eas e form. Acet ami nophen overdos e can be t reat ed wi t h N-acet yl cys t ei ne, whi ch bi nds t o t he t oxi c met abol i t e and provi des cys t ei ne for gl ut at hi one s ynt hes i s . Smal l dos es of acet ami nophen (3–4 g) i nges t ed wi t h al cohol may i nduce hepat i c necros i s marked by hi gh l evel s of hepat i c AST (3000–24,000 IU). Such cas es have a hi gh mort al i t y rat e.
o
o
b. Indirect toxicity may res ul t from i nt erference wi t h t he met abol i s m of bi l i rubi n (e.g., by es t rogens and androgens ) or wi t h prot ei n s ynt hes i s (e.g., by i nt ravenous t et racycl i ne, whi ch caus es mi croves i cul ar fat accumul at i on i n hepat ocyt es ).
o
Pa g e 1 0 8 3
ABC Ambe r CHM Conve rte r Tria l ve rsion, http://w w w .proce sste x t.com/a bcchm.html o
c. Immunologic drug reactions can caus e a vari et y of s yndromes , i ncl udi ng chol es t at i c jaundi ce (a s yndrome t hat mi mi cs acut e vi ral hepat i t i s ), a condi t i on wi t h a hi s t ol ogi c pi ct ure i ndi s t i ngui s habl e from chroni c act i ve hepat i t i s , and granul omat ous hepat i t i s . Ski n ras hes , eos i nophi l i a, and fever may be pres ent . In s ome cas es , i t has been pos t ul at ed t hat a drug or met abol i t e bi nds t o t he l i ver cel l membrane and act s as a hapten.
(1) Isoniazid caus es a cl i ni cal condi t i on s i mi l ar t o vi ral hepat i t i s and has been s hown t o be rel at ed t o a met abol i t e. P.224
(2) Halothane and ot her fl uori nat ed anes t het i cs frequent l y caus e onl y mi l d hepat i t i s or pos t operat i ve fever aft er fi rs t expos ure but may caus e ful mi nant hepat i t i s and deat h on re-expos ure.
(3) Chlorpromazine and chlorpropamide caus e chol es t at i c jaundi ce. An i nfl ammat ory i nfi l t rat e t hat frequent l y i ncl udes eos i nophi l s i s s hown on l i ver bi ops y.
(4) Phenytoin caus es a s erum s i cknes s –l i ke s yndrome, whi ch may res ul t i n mas s i ve hepat i c necros i s and deat h.
Pa g e 1 0 8 4
ABC Ambe r CHM Conve rte r Tria l ve rsion, http://w w w .proce sste x t.com/a bcchm.html
(5) Methyldopa may caus e t he cl i ni cal and hi s t ol ogi c fi ndi ngs of chroni c act i ve hepat i t i s , and s ome i nves t i gat ors bel i eve t hat a met abol i t e i s res pons i bl e.
(6) Excess intake of vitamin A (20,000–40,000 U/day) for years may caus e chol es t at i c hepat i c i njury. Vi t ami n A i s depos i t ed i n t he It o cel l s of t he l i ver, and l arge depos i t s compres s t he s i nus oi ds . Serum bi l i rubi n and al kal i ne phos phat as e l evel s may be el evat ed, prot hrombi n t i me may i ncreas e, and hepat i c enzyme act i vi t y may be rel at i vel y normal . Over t i me, ci rrhos i s may res ul t .
3. Alcoholic liver disease refers t o t he group of l i ver di s orders caus ed by acut e and chroni c al cohol i s m. Acut e effect s i ncl ude al cohol i c fat t y l i ver and al cohol i c hepat i t i s . Chroni c al cohol i s m i s a major caus e of ci rrhos i s of t he l i ver, whi ch i s di s cus s ed i n IX B 2 b. In t he Uni t ed St at es , al cohol i c l i ver di s eas e repres ent s t he fourt h mos t common caus e of deat h of adul t s 35–55 years of age. It appears t hat al cohol cons umpt i on of l es s t han 80 g/day i n men and 40 g/day i n (nonpregnant ) women general l y i s not as s oci at ed wi t h al cohol i c l i ver di s eas e. o
o
a. Alcoholic fatty liver occurs becaus e al cohol al t ers normal l i pi d met abol i s m. Mos t pat i ent s have hepat omegal y but ot herwi s e are as ympt omat i c unl es s t hey have ot her s ys t emi c probl ems rel at ed t o al cohol us e (e.g., pancreat i t i s and del i ri um t remens ). Laborat ory abnormal i t i es i ncl ude
Pa g e 1 0 8 5
ABC Ambe r CHM Conve rte r Tria l ve rsion, http://w w w .proce sste x t.com/a bcchm.html
i ncreas es i n γ-gl ut amyl t rans pept i das e, s erum t rans ami nas es , and al kal i ne phos phat as e. Hi s t ol ogi c exami nat i on s hows l arge-dropl et fat t y change i n t he l i ver. The prognos i s i s excel l ent for pat i ent s who compl et el y abs t ai n from al cohol cons umpt i on. o
o
b. Alcoholic hepatitis i s an acut e s yndrome t hat general l y occurs i n t he s et t i ng of heavy al cohol cons umpt i on. Many pat i ent s are report ed t o have i nges t ed more t han 100 g al cohol dai l y for more t han 1 year. (Approxi mat el y 100 g al cohol are cont ai ned i n 8 ounces of 100-proof whi s key, i n 30 ounces of wi ne, and i n ei ght 12-ounce cans of beer.) The rol e of decreas ed vi t ami n and prot ei n i nt ake i s cont rovers i al .
(1) Clinical features i ncl ude fever, jaundi ce, hepat omegal y, and l i ver t endernes s . As ci t es , encephal opat hy, and vari ceal bl eedi ng occas i onal l y are pres ent .
(2) Laboratory data i ncl ude l eukocyt os i s , 3
i ncreas ed AST (us ual l y <350 IU/mm ), el evat ed s erum bi l i rubi n, decreas ed s erum al bumi n, and a modes t i ncreas e i n s erum al kal i ne phos phat as e. Occas i onal l y, a chol es t at i c phas e i s pres ent wi t h marked el evat i ons i n t he al kal i ne phos phat as e and di rect bi l i rubi n. The ALT al mos t al ways i s l ower t han t he AST becaus e of decreas ed pyri doxi ne i nt ake and convers i on t o pyri doxal phos phat e. Al cohol -i nduced t hrombocyt openi a
Pa g e 1 0 8 6
ABC Ambe r CHM Conve rte r Tria l ve rsion, http://w w w .proce sste x t.com/a bcchm.html
i s pres ent i n 10% of pat i ent s .
(3) Diagnosis i s bas ed on cl i ni cal hi s t ory. Li ver bi ops y wi l l s how l arge-dropl et fat t y l i ver, pol ymorphonucl ear i nfi l t rat i on, al cohol i c hyal i ne (Mal l ory bodi es ), hepat ocyt e necros i s , and, occas i onal l y, s cl eros i s of cent ral vei ns .
(4) T herapy i s s upport i ve and i ncl udes a dai l y di et of 2500–3000 kcal wi t h s uppl ement al B vi t ami ns (es peci al l y t hi ami ne) and fol at e. Abs ol ut e abs t i nence from al cohol i s cruci al . Propyl t hi ouraci l (PTU) and cort i cos t eroi ds have a cont rovers i al t herapeut i c rol e but may be us eful i n s evere cas es .
4. Nonalcoholic fatty liver disease (NAFLD) or nonalcoholic steatohepatitis (NASH) des cri bes pat i ent s who l ack a hi s t ory of s i gni fi cant al cohol cons umpt i on but have l i ver bi ops y fi ndi ngs i ndi s t i ngui s habl e from al cohol i c hepat i t i s . Mos t cas es occur bet ween t he ages of 40 and 60 and t he di s eas e i s mos t common i n women. Al t hough t he et i ol ogy of NASH i s unknown, i t i s frequent l y as s oci at ed wi t h obes i t y, t ype 2 di abet es mel l i t us , and hyperl i pi demi a, al s o known as t he met abol i c s yndrome. NASH has al s o been des cri bed i n as s oci at i on wi t h t ot al parent eral nut ri t i on, abdomi nal s urgery, and medi cat i ons s uch as ami odarone and t amoxi fen. o
o
(1) Clinical course. NASH i s general l y cons i dered t o be a cl i ni cal l y s t abl e di s order wi t h a markedl y bet t er prognos i s t han al cohol i c hepat i t i s . In mos t
Pa g e 1 0 8 7
ABC Ambe r CHM Conve rte r Tria l ve rsion, http://w w w .proce sste x t.com/a bcchm.html
pat i ent s , t here i s l i t t l e change P.225
i n l i ver funct i on t es t s t hroughout t he cours e of t he di s eas e; however, i n s ome hi s t ol ogi c progres s i on occurs , and a s mal l fract i on of pat i ent s progres s t o end-s t age l i ver di s eas e. Unfort unat el y, no cl i ni cal or l aborat ory feat ures can predi ct progres s i on; one mus t rel y on l i ver bi ops y. o
o
(2) Diagnosis and laboratory data. Mos t pat i ent s wi t h NASH are as ympt omat i c, al t hough fat i gue, mal ai s e, and vague ri ght upper abdomi nal di s comfort may be pres ent . The mos t common pres ent at i on i s el evat ed l i ver funct i on t es t s . Hepat omegal y i s al s o a frequent fi ndi ng. Serum AST and ALT l evel s are el evat ed i n al mos t 90% of pat i ent s wi t h an AST/ALT rat i o us ual l y l es s t han 1. Al kal i ne phos phat as e i s l es s frequent l y el evat ed and hyperbi l i rubi nemi a i s uncommon. Li ver bi ops y i s t he onl y way t o confi rm or excl ude t he di agnos i s .
o
o
(3) T herapy. Current l y t here i s no proven effect i ve t reat ment for NASH. Modi fi cat i on of ri s k fact ors i ncl udi ng wei ght l os s , l i pi d cont rol , and i mproved di abet i c management are recommended. W ei ght reduct i on s houl d be gradual , s i nce rapi d wei ght l os s has been as s oci at ed wi t h wors eni ng of l i ver di s eas e. St udi es are ongoi ng wi t h t he hypogl ycemi c agent s met formi n, pi ogl i t azone, and ros i gl i t azone as pot ent i al t herapy.
B. Chronic liver disease
Pa g e 1 0 8 8
ABC Ambe r CHM Conve rte r Tria l ve rsion, http://w w w .proce sste x t.com/a bcchm.html
1. Chronic hepatitis mos t commonl y i s caus ed by vi ral i nfect i on or drugs . W hen not as s oci at ed wi t h ei t her of t hes e et i ol ogi es , chroni c act i ve hepat i t i s general l y i s t hought t o be i mmunol ogi cal l y medi at ed, al t hough an i mmunol ogi c mechani s m has not been proved. Thi s form of chroni c hepat i t i s i s oft en cal l ed aut oi mmune hepat i t i s . o
o
a. Clinical features i ncl ude mal ai s e, fat i gue, and vas cul i t i s . As t he di s eas e progres s es , mani fes t at i ons of chroni c l i ver di s eas e, i ncl udi ng as ci t es , encephal opat hy, and vari ceal bl eedi ng, domi nat e t he cl i ni cal pi ct ure.
o
o
b. Diagnosis i s bas ed on l i ver bi ops y t hat s hows pi ecemeal necros i s and bri dgi ng fi bros i s . The s erum t rans ami nas es are pers i s t ent l y el evat ed t o l evel s t hat oft en are 10 t i mes t he normal l evel s . Pos i t i ve ant i nucl ear ant i body (ANA) i s pres ent i n t wo-t hi rds of pat i ent s , and ant i –s moot h mus cl e ant i body i s found i n >80%.
o
o
c. T herapy wi t h hi gh-dos e cort i cos t eroi ds , azat hi opri ne, or bot h, i s benefi ci al i n t he i mmune-medi at ed t ype of chroni c hepat i t i s but us ual l y not i n t he drug- and vi rus -as s oci at ed di s eas es . IFN-α and ri bavi ri n have been s hown t o decreas e evi dence of HBV and HCV act i vi t y i n 30%–60% of cas es . Lami vudi ne and adefovi r may el i mi nat e or decreas e HBV act i vi t y, but recurrence may occur aft er wi t hdrawal of t herapy. Treat ment
Pa g e 1 0 8 9
ABC Ambe r CHM Conve rte r Tria l ve rsion, http://w w w .proce sste x t.com/a bcchm.html
of t he i di opat hi c aut oi mmune t ype of chroni c act i ve hepat i t i s i s cont i nued unt i l t he s erum t rans ami nas e l evel s decl i ne t o l es s t han t wi ce t he normal l evel s and a repeat l i ver bi ops y s hows res ol ut i on of t he i nfl ammat i on (general l y aft er more t han 1–2 years ).
2. Cirrhosis of the liver. Ci rrhos i s repres ent s a l at e s t age of progres s i ve hepat i c fi bros i s charact eri zed by di s t ort i on of t he hepat i c archi t ect ure and t he format i on of regenerat i ve nodul es . Regardl es s of t he caus e, i t i s general l y cons i dered t o be i rrevers i bl e i n i t s advanced s t ages , at whi ch poi nt t he onl y curat i ve opt i on i s l i ver t rans pl ant at i on. Pat i ent s wi t h ci rrhos i s are s us cept i bl e t o a vari et y of compl i cat i ons , and t hei r l i fe expect ancy i s markedl y reduced compared t o t hat of t he general popul at i on. o
o
a. Causes of cirrhosis.
(1) Alcohol-induced cirrhosis. Chroni c al cohol abus e caus es ci rrhos i s of t he l i ver (Laennec's ci rrhos i s ), whi ch, i n mos t cas es , i s t hought t o be a s equel a of al cohol i c hepat i t i s . The cl i ni cal feat ures of al cohol -i nduced ci rrhos i s refl ect i mpai red bl ood fl ow t hrough t he l i ver caus ed by obs t ruct i on by fi brot i c bands , res ul t i ng i n port al hypert ens i on, and by a decreas e i n hepat ocyt es avai l abl e for met abol i c funct i ons . Long-t erm al cohol us e al s o may be di rect l y t oxi c t o t he t es t i s , res ul t i ng i n t es t i cul ar at rophy and i mpot ence. Thes e effect s are compounded i n men by
Pa g e 1 0 9 0
ABC Ambe r CHM Conve rte r Tria l ve rsion, http://w w w .proce sste x t.com/a bcchm.html
i ncreas ed peri pheral l evel s of es t rogens , whi ch caus e s pi der angi omat a, gynecomas t i a, and pal mar eryt hema. Compl i cat i ons of al cohol -i nduced ci rrhos i s i ncl ude t he fol l owi ng condi t i ons :
(2) Viral (IX A 1, B 1)
(3) Nonalcoholic fatty liver disease (IX A 4)
(4) Autoimmune hepatitis (IX B P.226
(5) Primary biliary cirrhosis i s a di s eas e of unknown et i ol ogy. The us ual pat i ent age at di agnos i s i s 40–60 years , and 90% of pat i ent s are women.
(a) Pathogenesis, whi ch i s t hought t o be i mmunol ogi c, i nvol ves i nfl ammat ory des t ruct i on of s mal l i nt rahepat i c bi l i ary duct s . Earl y hi s t ol ogi c changes i ncl ude l ymphocyt i c i nfi l t rat i on and peri duct al granul oma format i on. Lat er i n t he di s eas e proces s , t he port al areas may s how an abs ence of duct s .
(b) Clinical features
Pa g e 1 0 9 1
ABC Ambe r CHM Conve rte r Tria l ve rsion, http://w w w .proce sste x t.com/a bcchm.html
(i) There i s a l ack of s ympt oms earl y i n t he cours e of t hi s di s eas e. However, an earl y di agnos i s oft en i s s us pect ed on t he bas i s of a marked i ncreas e i n t he al kal i ne phos phat as e l evel not ed on rout i ne bi ochemi cal s creeni ng.
(ii) The fi rs t s ympt om us ual l y i s pruritus, whi ch may be devas t at i ngl y s evere, es peci al l y at ni ght .
(iii) Jaundice occurs i n l at er s t ages of t he di s eas e, as do osteopenia (i n 25% of pat i ent s ) and xanthomas (i n approxi mat el y 10% of pat i ent s ).
(iv) In addi t i on, pri mary bi l i ary ci rrhos i s i s as s oci at ed wi t h s uch condi t i ons as Sjögren's s yndrome (i n 75% of pat i ent s ), t he pres ence of ant i t hyroi d ant i body (i n 25% of pat i ent s ), rheumat oi d art hri t i s (i n 5% of pat i ent s ), and t he CREST s yndrome (cal ci nos i s , Raynaud's phenomenon, es ophageal mot i l i t y dys funct i on, s cl erodact yl y, and t el angi ect as i a; i n 3% of pat i ent s ).
(c) Diagnosis i s made by t he
Pa g e 1 0 9 2
ABC Ambe r CHM Conve rte r Tria l ve rsion, http://w w w .proce sste x t.com/a bcchm.html
cons t el l at i on of i ncreas ed s erum chol es t erol , markedl y i ncreas ed al kal i ne phos phat as e (four t o s i x t i mes t he normal l evel ), i ncreas ed di rect bi l i rubi n, and t he pres ence of a pos i t i ve ant i mi t ochondri al ant i body, whi ch i s found i n more t han 90% of pat i ent s . The l i ver bi ops y s hows charact eri s t i c changes . Ext rahepat i c bi l i ary obs t ruct i on mus t be rul ed out .
(d) T herapy i s s upport i ve and i ncl udes admi ni s t rat i on of ant i pruri t i c agent s and s uppl ement at i on of vi t ami ns D and K and cal ci um, as wel l as bi s phos phonat es for prevent i on of os t eoporos i s . MCTs , whi ch do not requi re bi l e s al t mi cel l es for ads orpt i on, may be us ed as a di et ary s uppl ement . The pruri t us may res pond t o chol es t yrami ne, whi ch bi nds bi l e s al t s i n t he i nt es t i ne. Res ul t s have been achi eved wi t h cert ai n drugs (i .e., azat hi opri ne, chl orambuci l , col chi ci ne, and met hot rexat e). Perhaps t he mos t promi s i ng medi cal t reat ment i s urs odeoxychol i c aci d, whi ch may i mprove hepat i c s t ruct ure and funct i on i f i ni t i at ed before advanced s t ages of t he di s eas e. Int ravenous nal oxone t herapy may be us eful for s evere pruri t us . St andard defi ni t i ve s urgery i s l i ver t rans pl ant at i on, wi t h 5-year s urvi val rat es of 70% i n mos t s eri es .
Pa g e 1 0 9 3
ABC Ambe r CHM Conve rte r Tria l ve rsion, http://w w w .proce sste x t.com/a bcchm.html
(2) Secondary biliary cirrhosis us ual l y occurs aft er s everal years of bi l i ary t ract obs t ruct i on.
(a) Etiology i ncl udes common bi l e duct s t ones , common duct s t ri ct ures , chol angi ocarci noma, ampul l ary carci noma, s cl eros i ng chol angi t i s , and chroni c pancreat i t i s wi t h compres s i on of t he common duct as i t t ravers es t he pancreat i c head.
(b) Clinical features are s i mi l ar t o t hos e of pri mary bi l i ary ci rrhos i s but may be s upers eded by mani fes t at i ons of t he underl yi ng di s eas e. In addi t i on, pat i ent s may devel op chol angi t i s wi t h s haki ng chi l l s , fever, l eukocyt os i s , and jaundi ce. Ant i mi t ochondri al ant i bodi es us ual l y are not pres ent .
(c) T herapy i s ai med at rel i evi ng t he obs t ruct i on and i ncl udes s urgery, ext ernal drai nage, and pl acement of an i ndwel l i ng s t ent .
(6) Primary sclerosing cholangitis (VIII G)
(7) Ot her caus es i ncl ude Wilson' s disease (IX C 5), alpha-1-antitrypsin deficiency (IX C 1), hemochromatosis (IX C 3), and drug induced liver disease.
o
Pa g e 1 0 9 4
ABC Ambe r CHM Conve rte r Tria l ve rsion, http://w w w .proce sste x t.com/a bcchm.html
o
b. Vascular problems that mimic cirrhosis
(1) Budd-Chiari syndrome, or hepatic vein thrombosis, i s as s oci at ed wi t h hypercoagul abl e s t at es , pregnancy, t umors , abdomi nal t rauma, and us e of oral cont racept i ves . Pat i ent s us ual l y have as ci t es , and l i ver bi ops y s hows cent ri l obul ar conges t i on. Doppl er ul t ras ound exami nat i on may s how decreas ed or abs ent fl ow i n t he hepat i c vei n. At t empt s t o cat het eri ze t he hepat i c vei n oft en are uns ucces s ful , and t he mort al i t y rat e i s 50%–90%. In acut e Budd-Chi ari s yndrome (wi t hi n 1 week of ons et ), t reat ment wi t h t hrombol yt i c t herapy may be benefi ci al . Angi opl as t y wi t h s t ent i ng and t rans jugul ar i nt rahepat i c port os ys t emi c s hunt (TIPS) pl acement are al s o t reat ment s t o cons i der. Si de-t o-s i de port acaval s hunt s may prol ong s urvi val . Li ver t rans pl ant at i on may be requi red.
(2) Splenic vein thrombosis i s due t o abdomi nal t rauma, pancreat i t i s , and t umor. Al t hough t he port al vei n remai ns pat ent , gas t ri c vari ces devel op as s pl eni c vei n col l at eral s . Bot h es ophageal and gas t ri c vari ces devel op i n 15% of cas es . Di agnos i s i s made by angi ography, and t herapy wi t h s pl enect omy i s curat i ve.
(3) Veno-occlusive disease a di s eas e of s mal l hepat i c ves s el s , may fol l ow hepat i c
Pa g e 1 0 9 5
ABC Ambe r CHM Conve rte r Tria l ve rsion, http://w w w .proce sste x t.com/a bcchm.html
radi at i on (>3500 cGy), az at hi opri ne t herapy, or i nges t i on of cert ai n t ypes of Jamai can t ea. Aft er bone marrow t rans pl ant at i on (BMT), veno-occl us i ve di s eas e i s s een i n approxi mat el y 20% of pat i ent s . o
o
c. Complications of cirrhosis.
(1) Ascites
(a) Pathogenesis. Increas ed back-pres s ure i nt o capi l l ari es as wel l as decreas ed oncot i c pres s ure as t he res ul t of decreas ed al bumi n s ynt hes i s al l ows accumul at i on of a t rans udat i ve P.227
fl ui d i n t he peri t oneal cavi t y. In addi t i on, i ncreas ed ci rcul at i ng al dos t erone (pos s i bl y s econdary t o al t ered l i ver met abol i s m) cont ri but es t o Na
+
and wat er ret ent i on.
(b) Diagnosis. If fl ui d accumul at i on i s l arge, di agnos i s may be obvi ous on phys i cal exami nat i on. Ul t ras onography i s effect i ve for det ect i ng s mal l amount s of fl ui d. Paracent es i s , whi ch may requi re gui dance t hrough ul t ras onography, yi el ds a s t raw-col ored fl ui d wi t h l es s t han 2.5 g/dL prot ei n, a W BC count of l es s t han 3
300/mm (us ual l y mononucl ear), a normal gl ucos e l evel , and a l ow s erum amyl as e
Pa g e 1 0 9 6
ABC Ambe r CHM Conve rte r Tria l ve rsion, http://w w w .proce sste x t.com/a bcchm.html
l evel . A s erum al bumi n–as ci t es gradi ent (SAAG) great er t han 1.1 g/dL i s i ndi cat i ve of port al hypert ens i on.
(c) T herapy. Treat ment i s bas ed on s odi um res t ri ct i on Fl ui d res t ri ct i on may be neces s ary i f hyponat remi a devel ops . Al dos t erone ant agoni s t s (e.g., s pi ronol act one) and l oop di uret i cs (e.g., furos emi de) are us ed i f i ni t i al meas ures fai l . Large-vol ume paracent es i s can be us ed. If t here i s concern for hypot ens i on or fl ui d s hi ft s , al bumi n can be gi ven peri procedural l y. Some s t udi es s ugges t t hat 10 g of al bumi n for each 1 L of as ci t i c fl ui d removed i s effect i ve. Surgi cal s hunt i ng (Le Veen or Denver s hunt ) may be us eful i n recal ci t rant cas es , but bact eremi a and di s s emi nat ed i nt ravas cul ar coagul at i on (DIC) are pot ent i al compl i cat i ons of t hes e s hunt s . Pl acement of a TIPS t o connect t he hepat i c and port al vei ns can reduce port al hypert ens i on and rel i eve as ci t es .
(d) Complications. Res pi rat ory compromi s e and rupt ure of an umbi l i cal herni a may occur i n cas es of mas s i ve as ci t es . Infect i on of even a s mal l amount of fl ui d (e.g., due t o s pont aneous bact eri al peri t oni t i s ) can be fat al .
Pa g e 1 0 9 7
ABC Ambe r CHM Conve rte r Tria l ve rsion, http://w w w .proce sste x t.com/a bcchm.html
(2) Varices occur as a res ul t of t he devel opment of col l at eral ves s el s t hat bypas s t he obs t ruct ed l i ver. Vari ces are common i n t he es ophagus and s omewhat l es s common i n t he s t omach, duodenum, and hemorrhoi dal pl exus . Portal hypertensive gastropathy i nvol ves di l at i on of venous and capi l l ary ves s el s i n t he mucos a and s ubmucos a wi t h mi ni mal i nfl ammat ory i nfi l t rat e (s ee III J 1). Affect ed pat i ent s may s uffer bl eedi ng from t he mucos al l i ni ng of t he s t omach i t s el f. In mos t pat i ent s , port al decompres s i on res ul t s i n a decreas e of port al hypert ens i ve gas t ropat hy and ces s at i on of bl eedi ng.
(a) Diagnosis of es ophagogas t ri c vari ces may be s ugges t ed by an upper gas t roi nt es t i nal s eri es but i s bes t made by endos copy. Endos copy i s es s ent i al i n t he acut el y bl eedi ng pat i ent becaus e t he mort al i t y rat e i s hi gh (40%–50% for each epi s ode of bl eedi ng), and earl y t reat ment can be l i fes avi ng.
(b) Short-term therapy wi t h oct reot i de by cont i nuous i nt ravenous i nfus i on i s effect i ve i n 60%–70% of pat i ent s . Bal l oon t amponade wi t h a Sengs t aken-Bl akemore t ube cont rol s bl eedi ng i n 80% of pat i ent s , but i s as s oci at ed wi t h a ri s k of as pi rat i on and es ophageal rupt ure. Emergency s urgery has a 50% mort al i t y rat e. Endos copi c band l i gat i on or s cl erot herapy i s
Pa g e 1 0 9 8
ABC Ambe r CHM Conve rte r Tria l ve rsion, http://w w w .proce sste x t.com/a bcchm.html
effect i ve i n approxi mat el y 80%–90% of acut e cas es . A TIPS can s t op vari ceal bl eedi ng.
(c) Long-term therapy t o reduce t he chance of rebl eedi ng i ncl udes repeat endos copi c band l i gat i on or s cl erot herapy t o obl i t erat e al l vari ces and nons el ect i ve β-adrenergi c bl ockers (e.g., propranol ol and nadol ol ) t o reduce port al pres s ure. Surgi cal port al decompres s i on does not i ncreas e s urvi val rat es becaus e t here i s i ncreas ed encephal opat hy and l i ver fai l ure. Li ver t rans pl ant at i on i s res erved for good-ri s k al cohol -abs t i nent i ndi vi dual s .
(3) Portosystemic encephalopathy i s a revers i bl e neurol ogi c s yndrome charact eri zed by mood changes , confus i on, drows i nes s , di s ori ent at i on, and coma.
(a) Etiology. The pri mary caus e of hepat i c encephal opat hy i s uncl ear. El evat ed ammoni a l evel s are found i n t he bl ood. However, more recent l y, fal s e neurot rans mi t t ers and el evat ed l evel s of mercapt ans and fat t y aci ds have been i mpl i cat ed. Increas ed l evel s of endogenous benzodi azepi ne s ubs t ances may be not ed. In addi t i on, i ncreas ed l evel s of aromat i c ami no aci ds and decreas ed l evel s of branched-chai n
Pa g e 1 0 9 9
ABC Ambe r CHM Conve rte r Tria l ve rsion, http://w w w .proce sste x t.com/a bcchm.html
ami no aci ds are found i n t he bl ood, brai n, and uri ne. Secondary caus es of hepat i c encephal opat hy are t hought t o i ncl ude:
(i) Azot emi a, due t o i ncreas ed ni t rogen l oad
(ii) Increas ed product i on of ammoni a or i ncreas ed prot ei n l oad due t o cons t i pat i on, i ncreas ed di et ary prot ei n, or gas t roi nt es t i nal bl eed
(iii) Infect i on, whi ch l eads t o i ncreas ed t i s s ue cat abol i s m and i ncreas ed prot ei n l oad
(iv) Sedat i ves , whos e di rect depres s ant effect on t he brai n i s compounded by decreas ed hepat i c cat abol i s m of t he drugs P.228
(v) El ect rol yt e abnormal i t i es i ncl udi ng hypokal emi a and al kal os i s , whi ch l ead t o i mpai red renal excret i on of ammoni a and t o i ncreas ed t rans fer of ammoni a acros s t he bl ood–brai n barri er
Pa g e 1 1 0 0
ABC Ambe r CHM Conve rte r Tria l ve rsion, http://w w w .proce sste x t.com/a bcchm.html
(b) T herapy. Treat ment i ncl udes revers al of any of t he s econdary caus es . In addi t i on, t he fol l owi ng meas ures may be effect i ve.
(i) Lact ul os e effect i vel y decreas es col oni c pH and t raps ammoni um i on +
(NH 4 ) i n t he gas t roi nt es t i nal t ract . It al s o i s an effect i ve cat hart i c.
(ii) Ant i bi ot i cs s uch as met roni dazol e, ri faxi mi n, and neomyci n may decreas e i nt es t i nal fl ora t hat convert gas t roi nt es t i nal prot ei ns i nt o ammoni a.
(iii) Di et ary prot ei n s houl d be l i mi t ed t o l es s t han 40 g/day i f ot her meas ures are i neffect i ve.
(iv) The us e of branched-chai n ami no aci ds , al t hough academi cal l y appeal i ng, may not i mprove encephal opat hy.
(v) Predi s pos i ng fact ors (e.g., hypokal emi a, cons t i pat i on, and al kal os i s ) s houl d be correct ed.
Pa g e 1 1 0 1
ABC Ambe r CHM Conve rte r Tria l ve rsion, http://w w w .proce sste x t.com/a bcchm.html
(4) Hepatorenal syndrome i s a progres s i ve renal fai l ure t hat occurs i n pat i ent s wi t h s evere l i ver di s eas e. It i s a funct i onal renal fai l ure becaus e t he ki dneys are morphol ogi cal l y normal and funct i on wel l when t rans pl ant ed i nt o normal reci pi ent s . The mort al i t y rat e i s 90%–100%.
(a) Etiology. Al t hough t he exact mechani s m of hepat orenal s yndrome i s not known, s everal fact ors are i mpl i cat ed, i ncl udi ng:
(i) Afferent art eri ol ar vas ocons t ri ct i on, whi ch l eads t o i ncreas ed renal vas cul ar cons t ri ct i on
(ii) Rel at i ve s hunt i ng of bl ood from t he cort ex t o t he medul l a of t he ki dney
(iii) Decreas ed gl omerul ar fi l t rat i on rat e (GFR)
(iv) Decreas ed renal bl ood fl ow
(b) Diagnosis. The combi nat i on of 3
ol i guri a (i .e., uri ne out put <300 mm /24 h) wi t h i ncreas i ng bl ood urea ni t rogen (BUN) and creat i ni ne concent rat i ons i n a pat i ent wi t h s evere l i ver di s eas e i s
Pa g e 1 1 0 2
ABC Ambe r CHM Conve rte r Tria l ve rsion, http://w w w .proce sste x t.com/a bcchm.html
s ugges t i ve. Addi t i onal l aborat ory fi ndi ngs i ncl ude uri nary Na
+
concent rat i on of l es s t han 10 mEq/L and beni gn uri ne s edi ment . It i s i mport ant t o rul e out ot her caus es of ol i guri a [e.g., acut e t ubul ar necros i s (ATN), hypovol emi a, and uri nary t ract obs t ruct i on].
(c) T herapy. General l y, t reat ment i s uns ucces s ful . A fl ui d chal l enge s houl d be gi ven t o al l pat i ent s t o rul e out hypovol emi a. Succes s occas i onal l y has been report ed wi t h Le Veen or Denver s hunt s , port acaval s hunt s , or l i ver t rans pl ant at i on. No drugs have been s hown t o be benefi ci al .
(5) Coagulation defects us ual l y are at t ri but abl e t o decreas ed hepat i c s ynt hes i s of cl ot t i ng fact ors . In addi t i on, s pl enomegal y may cont ri but e t o t hrombocyt openi a.
(6) Hepatopulmonary syndrome i s a hypoxi c s t at e caus ed by i nt rapul monary vas cul ar s hunt i ng (effect i vel y, a ri ght -t o-l eft s hunt ). Exerci s e or s t andi ng-i nduced hypoxi a i s not ed, and cl ubbi ng may occur i n s evere cas es . Thi s s yndrome i s probabl y pres ent i n 30% of pat i ent s wi t h ci rrhos i s and may be s evere i n 10% of pat i ent s .
o
o
d. T herapy. The goal s of t reat i ng pat i ent s wi t h
Pa g e 1 1 0 3
ABC Ambe r CHM Conve rte r Tria l ve rsion, http://w w w .proce sste x t.com/a bcchm.html
ci rrhos i s i ncl ude: s l owi ng or revers i ng t he progres s i on of l i ver di s eas e, prevent i ng s uperi mpos ed i ns ul t s t o t he l i ver, prevent i ng and t reat i ng compl i cat i ons and det ermi ni ng t he appropri at enes s and opt i mal t i mi ng for l i ver t rans pl ant at i on. As l i ver t rans pl ant at i on i s t he defi ni t i ve t reat ment for decompens at ed ci rrhos i s , el i gi bl e pat i ent s s houl d be referred as earl y as pos s i bl e. El i gi bi l i t y i s det ermi ned by s everi t y of di s eas e, compl i cat i ons , and pot ent i al for s urvi val aft er t rans pl ant at i on, whi ch i s numeri cal l y cl as s i fi ed by t he model end-s t age l i ver di s eas e s core, or MELD s core. o
o
d. Cardiac cirrhosis i s a rare, l at e mani fes t at i on of s evere prol onged ri ght vent ri cul ar fai l ure, whi ch i s mos t oft en s een wi t h rheumat i c heart di s eas e (ei t her mi t ral or aort i c s t enos i s wi t h t ri cus pi d regurgi t at i on). Cons t ri ct i ve peri cardi t i s and s evere cardi omyopat hy al s o may be as s oci at ed wi t h cardi ac ci rrhos i s . Cl i ni cal feat ures i ncl ude an enl arged l i ver, as ci t es , and s pl enomegal y. PT oft en i s prol onged and precl udes t he us e of ant i coagul ant s i n t reat ment of t he val vul ar l es i on. Prognos i s depends on t he cours e of t he cardi ac di s eas e.
o
o
g. Nodular transformation of the liver i s a nonci rrhot i c caus e of port al hypert ens i on charact eri zed by t he unexpl ai ned devel opment of s mal l nodul es , es peci al l y i n t he peri hi l ar area. Recurrent vari ceal bl eedi ng may occur.
Pa g e 1 1 0 4
ABC Ambe r CHM Conve rte r Tria l ve rsion, http://w w w .proce sste x t.com/a bcchm.html
3. Liver abscess o
o
a. Pyogenic liver abscess us ual l y i s due t o bi l i ary t ract di s eas e, i ncl udi ng acut e chol ecys t i t i s and chol angi t i s . Ot her i nfect i ons (e.g., appendi ci t i s and di vert i cul i t i s ) as wel l as i nt ri ns i c hepat i c l es i ons al s o are i mport ant caus es . In 10% of cas es , t he caus at i ve fact or cannot be det ermi ned. Approxi mat el y 70% of abs ces s es cont ai n mi xed fl ora; t he mos t commonl y found organi s ms are anaerobes , E. c ol i , Kl ebs i el l a s peci es , St aphyl oc oc c us aureus , and s t rept ococci . P.229
(1) Clinical features. Fever, chi l l s , ri ght upper quadrant pai n, anorexi a, and naus ea may be pres ent . Pl euri t i c pai n occas i onal l y occurs , and wei ght l os s i s common. Tender hepat omegal y i s pres ent i n 50% of cas es . The al kal i ne phos phat as e l evel i s el evat ed i n approxi mat el y 80% of pat i ent s , jaundi ce i s pres ent i n approxi mat el y 33%, and bl ood cul t ures are pos i t i ve i n 40%. CT combi ned wi t h l i ver s canni ng, ul t ras onography, or bot h, can be us ed t o det ect an abs ces s great er t han 2 cm.
(2) Diagnosis. The cl i ni cal pres ent at i on s ugges t s t he di agnos i s , whi ch can be confi rmed by CT- or ul t ras onography-gui ded
Pa g e 1 1 0 5
ABC Ambe r CHM Conve rte r Tria l ve rsion, http://w w w .proce sste x t.com/a bcchm.html
as pi rat i on.
(3) T herapy. Ant i bi ot i cs , wi t h or wi t hout ext ernal drai nage, frequent l y are s ucces s ful and neces s ary for mul t i pl e s mal l abs ces s es . Treat ment us ual l y i s cont i nued for 4–6 weeks . Occas i onal l y, s urgi cal drai nage i s requi red.
o
o
b. Amebic liver abscess. Of t he s i x Ent amoeba s peci es found i n t he human col on, E. hi s t ol yt i c a i s t he onl y t rue pat hogen. In t he Uni t ed St at es , homos exual men and i ns t i t ut i onal i zed i ndi vi dual s are at great es t ri s k for amebi c l i ver abs ces s . Thi s di s eas e al s o i s common where di arrheal di s eas e due t o E. hi s t ol yt i c a i s endemi c.
(1) Clinical features. Ri ght upper quadrant pai n and fever may be pres ent . Pl euri t i c pai n, chi l l s , and ni ght s weat s al s o may be not ed. There i s a hi s t ory of i nt es t i nal amebi as i s i n 50% of pat i ent s .
(2) Laboratory data. More t han 50% of pat i ent s have el evat ed W BC count s 3
(>20,000/mm ), s erum t rans ami nas e l evel s , and s erum bi l i rubi n l evel s . Serum al kal i ne phos phat as e l evel s are abnormal i n approxi mat el y 80% of affect ed i ndi vi dual s .
(3) Diagnosis. Demons t rat i on of a fi l l i ng defect i n t he l i ver t hrough t he us e of
Pa g e 1 1 0 6
ABC Ambe r CHM Conve rte r Tria l ve rsion, http://w w w .proce sste x t.com/a bcchm.html
ul t ras onography, CT, or MRI i s s ugges t i ve of t he di agnos i s . As pi rat i on of a cys t i c cavi t y may reveal “anchovy pas t e― fl ui d wi t h t rophozoi t es . Serol ogi c t es t s (e.g., i ndi rect hemaggl ut i nat i on i nhi bi t i on and gel di ffus i on t es t s ) are pos i t i ve i n 95% of cas es .
(4) T herapy. Amebi ci des (e.g., met roni dazol e, chl oroqui ne, and di i odohydroxyqui n) may be effect i ve al one or may be combi ned wi t h CT or ul t ras ound-di rect ed as pi rat i on of t he abs ces s cavi t y. Li ver s canni ng s houl d be cont i nued unt i l heal i ng occurs .
(5) Complications. Rupt ure of t he cys t i nt o t he pl eural s pace, l ung, bowel , and ret roperi t oneum may occur. Rarel y, a cys t ext ends t o t he body s urface
o
o
c. Focal hepatic candidiasis i s an ent i t y cons i s t i ng of hepat i c and s pl eni c granul omas cont ai ni ng Candi da al bi c ans hyphae i n i mmunocompromi s ed hos t s . Mos t pat i ent s have previ ous l y recei ved cyt os i ne arabi nos i de for acut e l eukemi a.
(1) Clinical features i ncl ude a fever of unknown ori gi n i n i mmunocompromi s ed hos t s . Si gns of oropharyngeal candi di as i s may be pres ent , and t here may be ri ght upper quadrant pai n or t endernes s .
Pa g e 1 1 0 7
ABC Ambe r CHM Conve rte r Tria l ve rsion, http://w w w .proce sste x t.com/a bcchm.html
(2) Diagnosis i s made by liver biopsy. At l aparot omy or l aparos copy, s mal l whi t e nodul es l es s t han 5 mm i n wi dt h are s een. Li ver funct i on abnormal i t i es i ncl ude modes t bi l i rubi n and enz yme el evat i on wi t h an i ncreas e i n al kal i ne phos phat as e.
(3) T herapy i nvol ves s ys t emi c amphot eri ci n B, oft en i n conjunct i on wi t h fl uconazol e.
4. Hepatic cysts o
o
a. Solitary cysts, general l y found i n t he ri ght l obe of t he l i ver, us ual l y are as ympt omat i c but may caus e pai n and fever s econdary t o bl eedi ng, i nfect i on, or rupt ure.
o
o
b. Polycystic liver disease i s t he pres ence of mul t i pl e cys t s t hat range from s everal mi l l i met ers t o great er t han 10–15 cm i n di amet er. Li ke s ol i t ary cys t s , mos t cys t s i n pol ycys t i c l i ver di s eas e are as ympt omat i c except i n cas es i nvol vi ng hemorrhage, i nfect i on, or rupt ure. Renal cys t s are found i n 50% of pat i ent s ; cys t s al s o may be found i n t he pancreas , s pl een, and l ungs . Res ul t s of l i ver funct i on t es t i ng us ual l y are normal , al t hough mi l d el evat i on of s erum al kal i ne phos phat as e l evel s may be s een. Surgi cal as pi rat i on or decompres s i on occas i onal l y i s neces s ary.
o
o
c. Hydatid cysts are mos t common i n i ndi vi dual s who l i ve i n Greece, France, It al y, t he Mi ddl e Eas t ,
Pa g e 1 1 0 8
ABC Ambe r CHM Conve rte r Tria l ve rsion, http://w w w .proce sste x t.com/a bcchm.html
Sout h Ameri ca, and Icel and. Thi s di s eas e may be found el s ewhere i n t he des cendant s of i ndi vi dual s from t hes e regi ons .
(1) Etiology. Hydat i d cys t s are formed when t he i nfect i ng organi s m (i .e., Ec hi noc oc c us granul os us or Ec hi noc oc c us mul t i l oc ul ari s ) i s i nges t ed and t ravel s t hrough t he port al ci rcul at i on t o t he l i ver.
(2) Clinical features. The cys t us ual l y enl arges for 10–20 years aft er t he i ni t i al i nfect i on before becomi ng s ympt omat i c. There i s cal ci fi cat i on of a s ol i t ary cys t (s een i n 50% of pat i ent s ) and t he pres ence of daught er cys t s wi t hi n a l arger cys t .
(3) Diagnosis. Di agnos i s i s made by pos i t i ve compl ement fi xat i on or i ndi rect hemaggl ut i nat i on t es t s . Eos i nophi l i a i s occas i onal l y s een. Li ver bi ops y and as pi rat i on are not s ugges t ed becaus e l eakage may caus e fat al anaphyl axi s .
(4) T herapy. Treat ment may be s urgi cal . Recent l y, ul t ras ound-gui ded i nject i on of al cohol i nt o cys t cavi t i es has been us ed.
(5) Complications. Rupt ure, i nfect i on, hemorrhage, and s l ow l eakage caus i ng al l ergi c mani fes t at i ons may devel op.
o
Pa g e 1 1 0 9
ABC Ambe r CHM Conve rte r Tria l ve rsion, http://w w w .proce sste x t.com/a bcchm.html
d. Peliosis hepatis i s a rare condi t i on i nvol vi ng mul t i pl e bl ood-fi l l ed hepat i c cys t s .
See onl i ne for more i nformat i on. The
l i ver oft en has a mot t l ed bl ue appearance. Rupt ure wi t h bl eedi ng may be fat al . There i s an as s oci at i on bet ween t hi s condi t i on and t ubercul os i s , t herapy wi t h androgeni c s t eroi ds , and t he us e of oral cont racept i ves . There i s al s o an as s oci at i on wi t h AIDS pat i ent s , es peci al l y t hos e wi t h angi omat os i s (Roc henel l i a). Progres s i ve hepat omegal y wi t h l i ver fai l ure may occur. CT s canni ng may s how mul t i pl e defect s . Percut aneous l i ver bi ops y s hows charact eri s t i c changes but i s dangerous becaus e of t he vas cul ar nat ure of t he l es i ons .
5. Granulomatous hepatitis o
o
a. Etiology. Granul omat ous hepat i t i s mos t oft en i s s econdary t o s ys t emi c i nfect i ons (e.g., t ubercul os i s ), s arcoi dos i s , fungal i nfect i ons , s yphi l i s , and vi ral i nfect i ons (e.g., i nfect i ous mononucl eos i s , CMV i nfect i on, and vari cel l a). Q fever, paras i t i c di s eas es , MAC (es peci al l y i n HIV-i nfect ed pat i ent s ), Hodgki n's di s eas e, and beryl l i um t oxi ci t y al s o may caus e granul omat ous hepat i t i s . In addi t i on, granul omat ous hepat i t i s may be a mani fes t at i on of drug react i ons i nvol vi ng s ul fa drugs , hydral azi ne, or al l opuri nol . Occas i onal l y, no caus e can be found. P.230
o
o
b. Clinical features. Sympt oms i ncl ude weaknes s , fat i gue, markedl y i ncreas ed eryt hrocyt e s edi ment at i on rat e, and fever.
Pa g e 1 1 1 0
ABC Ambe r CHM Conve rte r Tria l ve rsion, http://w w w .proce sste x t.com/a bcchm.html o
o
c. Diagnosis. Li ver bi ops y i s us ed t o make t he di agnos i s .
o
o
d. T herapy. Treat ment of t he s econdary form i ncl udes wi t hdrawal of t he offendi ng agent and t reat ment of t he underl yi ng l es i on. The i di opat hi c form may res pond t o cort i cos t eroi d t herapy.
C. Systemic diseases with prominent liver involvement
1. α 1 -Antitrypsin deficiency i s a genet i c defect of t he gl ycoprot ei n t hat normal l y i nhi bi t s prot eol yt i c enzymes s uch as t ryps i n, chymot ryps i n, and el as t as e.
See onl i ne for more
i nformat i on. There are 24 al l el es i n t he prot eas e i nhi bi t or (Pi ) s ys t em. Ni net y percent of t he popul at i on of t he Uni t ed St at es i s genot ype Pi MM. The 22 genot ype i s homoz ygous for t he di s eas e s t at e, and pat i ent s have l es s t han 20% of normal s erum l evel s of α 1 -ant i t ryps i n. Indi vi dual s wi t h genot ype Pi MZ have approxi mat el y 50%–60% of normal l evel s . Homozygot es (Pi ZZ) us ual l y have l i ver di s eas e i n chi l dhood. Li ver di s eas e may devel op i n Pi SZ or Pi MZ het eroz ygot es , es peci al l y t hos e who s moke, and COPD may devel op i n t hes e i ndi vi dual s as adul t s (s ee Chapt er 2 II B). Di s eas e does not devel op i n al l i ndi vi dual s wi t h t he abnormal genot ypes . Di agnos i s i s made bas ed on a decreas ed α 1 -gl obul i n l evel obs erved on a prot ei n el ect rophores i s , a decreas ed α 1 -ant i t ryps i n l evel i n t he s erum, and by Pi t ypi ng. Li ver bi ops y s hows di as t as e-res i s t ant PAS-pos i t i ve gl obul es i n port al areas . There i s no effect i ve t herapy. Li ver t rans pl ant at i on may be us ed i n advanced cas es .
2. Amyloidosis i nvol ves t he l i ver i n 50% of cas es . Pat i ent s have hepat omegal y on phys i cal exami nat i on but
Pa g e 1 1 1 1
ABC Ambe r CHM Conve rte r Tria l ve rsion, http://w w w .proce sste x t.com/a bcchm.html
us ual l y are as ympt omat i c. Res ul t s of l i ver funct i on t es t i ng oft en are normal but may i ndi cat e a marked el evat i on of s erum al kal i ne phos phat as e.
3. Hemochromatosis i s an i nheri t ed di s order (t hought t o be aut os omal reces s i ve) i n whi ch i ncreas ed abs orpt i on of i ron l eads t o i ron depos i t i on i n t he l i ver, heart , pancreas , and ot her organs . Men are more commonl y affect ed t han women (at a rat i o of 8:1). In t he Uni t ed St at es , approxi mat el y 8%–9% of t he popul at i on are het eroz ygot es and 1 i n every 220 are homoz ygot es , maki ng hemochromat os i s one of t he mos t common genet i c l i ver di s eas es . o
o
a. Clinical features. Hepat omegal y, hyperpi gment at i on, and abnormal i t i es of t he cardi ac conduct i on s ys t em, t es t es , and joi nt s may occur. Fi ft y percent of t he pat i ent s have abdomi nal pai n. Ci rrhos i s and di abet es al s o may devel op.
o
o
b. Laboratory findings. El evat ed l evel s of s erum t rans ami nas es , i ncreas ed s erum i ron wi t h el evat ed percent s at urat i on (general l y >80%), and hi gh s erum l evel of ferri t i n are s een. The HFE gene i s l i nked t o HLA-A 3 l ocus on chromos ome 6. Approxi mat el y 85% of pat i ent s wi t h hemochromat os i s have t hi s C282Y mut at i on. Li ver bi ops y s hows i ron depos i t s i n bot h hepat ocyt es and Kupffer cel l s . Thi s fi ndi ng rul es out s econdary i ron overl oad (hemos i deros i s ) i n whi ch i ron i s depos i t ed i n Kupffer cel l s al one. A s ki n or i nt es t i nal bi ops y as wel l as an anal ys i s of fami l y members for el evat ed i ron, t ot al i ron-bi ndi ng capaci t y, or ferri t i n l evel s
Pa g e 1 1 1 2
ABC Ambe r CHM Conve rte r Tria l ve rsion, http://w w w .proce sste x t.com/a bcchm.html
al s o may be hel pful i n t he di fferent i al di agnos i s . Screeni ng fami l y members for t he el evat ed HLA hapl ot ypes may be us ed. CT or MRI of t he l i ver s hows charact eri s t i c changes . o
o
c. T herapy. Repeat ed phl ebot omy (us ual l y once or t wi ce weekl y for s everal mont hs or years ) decreas es t ot al body i ron s t ores , whi ch may be at 10 t i mes t he normal l evel . Phl ebot omy i s cont i nued unt i l anemi a devel ops or s erum i ron and ferri t i n l evel s normal i ze. Unt reat ed pat i ent s have an i ncreas ed ri s k of hepat oma.
o
o
d. Prognosis. If hemochromat os i s i s di agnos ed before ci rrhos i s devel ops , t he prognos i s wi t h t reat ment i s good (80% s urvi val at 15 years ). If ci rrhos i s or di abet es mel l i t us i s pres ent at t he t i me of di agnos i s , or i f t he i ron s t ores do not decreas e t o normal l evel s aft er 18 mont hs of t reat ment , t he prognos i s i s not as favorabl e. Pat i ent s wi t h hemochromat os i s and ci rrhos i s have approxi mat el y a 220-fol d i ncreas ed ri s k of l i ver cancer
4. Sarcoidosis. Approxi mat el y 70% of pat i ent s wi t h s arcoi dos i s have granul omas i n t he l i ver. Pat i ent s us ual l y do not have s ympt oms referabl e t o t he l i ver but may have i ncreas ed s erum al kal i ne phos phat as e l evel s . Fort y percent of pat i ent s have hepat omegal y. A l i ver bi ops y s howi ng noncas eat i ng granul omas may ai d i n t he di agnos i s .
5. Wilson' s disease i s an aut os omal reces s i ve di s eas e charact eri zed by exces s i ve copper depos i t i on, whi ch, i f unt reat ed,
Pa g e 1 1 1 3
ABC Ambe r CHM Conve rte r Tria l ve rsion, http://w w w .proce sste x t.com/a bcchm.html
may l ead t o ful mi nant hepat i c fai l ure. Copper may al s o be depos i t ed i n t he brai n, ki dney, and cornea; copper depos i t i ons i n t he cornea caus e Kayser-Fleischer rings.
See onl i ne for more
i nformat i on CNS di s eas e may be promi nent i f t he di agnos i s i s made i n adul t hood. Di agnos i s i s s ugges t ed by decreas ed s erum cerul opl as mi n l evel s and i ncreas ed uri nary copper excret i on (>100 µg/24 h); i t i s confi rmed by an i ncreas ed hepat i c copper concent rat i on i n a l i ver bi ops y s ampl e. Treat ment i s wi t h D-peni ci l l ami ne, t ri ent i ne, or zi nc i n combi nat i on wi t h a l ow-copper di et . Hepat i c t rans pl ant at i on has been us ed for ful mi nant hepat i c f ai l ure.
6. Liver disease of pregnancy o
a. Cholestasis us ual l y i s s een i n t he l as t t ri mes t er of pregnancy and i s beni gn. Pat i ent s may compl ai n of pruri t us and jaundi ce, and al l s ympt oms di s appear rapi dl y aft er del i very.
See onl i ne for
more i nformat i on The s yndrome i s t hought t o be medi at ed by es t rogens , proges t erone, or bot h, and s ubs equent us e of oral cont racept i ves or pregnancy may caus e a recurrence of s ympt oms . Recent l y, urs odeoxychol i c aci d t herapy has rel i eved pruri t us and al l owed t he del ay of del i very for great er fet al mat uri t y. o o
b. Acute fatty liver i s a s evere di s eas e us ual l y occurri ng i n a pri mi gravi da i n t he l as t t ri mes t er of pregnancy. Ful mi nant l i ver fai l ure may devel op, and an as s oci at i on wi t h t oxemi a has been report ed.
See onl i ne for more i nformat i on Prognos i s i s poor but i s
i mproved by prompt del i very. Pat hol ogi c changes i n t he l i ver i ncl ude s mal l -dropl et fat t y change s i mi l ar t o t hat s een i n fat t y l i ver i nduced by t et racycl i ne and val proi c aci d. o
o
c. HELLP syndrome (hemol ys i s , el evat ed l i ver enzymes , l ow pl at el et count ) i s oft en s een i n t he
Pa g e 1 1 1 4
ABC Ambe r CHM Conve rte r Tria l ve rsion, http://w w w .proce sste x t.com/a bcchm.html
t hi rd t ri mes t er of pregnancy. Thi s condi t i on i s as s oci at ed wi t h t oxemi a i n approxi mat el y 50% of pat i ent s . Abdomi nal pai n and vomi t i ng may be s evere. Treat ment i s del i very of t he i nfant . o
o
d. Hepatic rupture rarel y occurs aft er necros i s of t he l i ver i n ecl ampt i c pat i ent s i n t hei r l as t t ri mes t er. Pat i ent s wi t h hepat i c rupt ure have general l y been ol der and mul t i parous .
D. Inherited disorders of bilirubin metabolism
1. Gilbert' s syndrome i s an es s ent i al l y beni gn condi t i on t hat occurs i n approxi mat el y 7% of t he popul at i on of t he Uni t ed St at es . Decreas ed uri di ne di phos phat e (UDP) gl ucuronyl t rans feras e act i vi t y l eads t o mi l d unconjugat ed hyperbi l i rubi nemi a (us ual l y <3 mg/dL), whi ch i ncreas es aft er fas t i ng.
2. Crigler-Najjar syndrome exi s t s i n t wo forms : t ype I and t ype II. o
o
a. T ype I i s rare and i s charact eri zed by an abs ence of hepat i c UDP gl ucuronyl t rans feras e act i vi t y. Pat i ent s us ual l y di e i n i nfancy.
o
o
b. T ype II al s o i s rare and i s charact eri zed by markedl y di mi ni s hed hepat i c UDP gl ucuronyl t rans feras e act i vi t y, l eadi ng t o unconjugat ed hyperbi l i rubi nemi a i n t he range of 5–25 mg/dL.
Pa g e 1 1 1 5
ABC Ambe r CHM Conve rte r Tria l ve rsion, http://w w w .proce sste x t.com/a bcchm.html
Phenobarbi t al may be us ed t o i nduce mi cros omal enzyme act i vi t y. In mos t cas es , t here are no cl i ni cal s equel ae.
3. Rotor' s syndrome i s a rare aut os omal reces s i ve condi t i on. Impai red t rans port of conjugat ed bi l i rubi n out of t he hepat ocyt e l eads t o conjugat ed hyperbi l i rubi nemi a i n t he range of 2–10 mg/dL. Li ver bi ops y i s normal , and t he cl i ni cal cours e i s beni gn.
4. Dubin-Johnson syndrome i s s i mi l ar t o Rot or's s yndrome, except t hat l i ver bi ops y s hows t he accumul at i on of a dark pi gment wi t hi n hepat ocyt es . The el evat ed s erum bi l i rubi n may res pond s omewhat t o phenobarbi t al t herapy.
5. Alagille' s syndrome i s one of vari ous t ypes of fami l i al i nt rahepat i c chol es t as i s s yndromes . Al agi l l e's s yndrome i s an aut os omal domi nant s yndrome t hat i s general l y i dent i fi ed i n i nfant s younger t han 3 mont hs of age. It i s as s oci at ed wi t h congeni t al heart di s eas e (vent ri cul ar or at ri al s ept al defect s ), bony defect s (but t erfl y vert ebra or s pi na bi fi da), and renal or bi l i ary t ree anomal i es . There are cl as s i c chol es t at i c l aborat ory t es t fi ndi ngs , and few bi l e duct s are s een on l i ver bi ops y. Mos t pat i ent s have mi l d di s eas e, but ci rrhos i s devel ops l at er i n approxi mat el y 10% of t hes e pat i ent s .
E. Tumors of the liver
1. Benign tumors o
Pa g e 1 1 1 6
ABC Ambe r CHM Conve rte r Tria l ve rsion, http://w w w .proce sste x t.com/a bcchm.html
o
a. Hepatic adenomas us ual l y occur i n women of chi l dbeari ng age and are more common in t hos e who us e oral cont racept i ves . P.231
(1) Clinical features may be compl et el y abs ent . Some pat i ent s report ri ght upper quadrant ful l nes s . Occas i onal l y, s pont aneous rupt ure of t he adenoma l eads t o i nt ra-abdomi nal hemorrhage, whi ch i s fat al i n 25% of pat i ent s .
(2) Diagnosis i s made by demons t rat i on of a hepat i c mas s by CT and a col d s pot t hrough l i ver s canni ng. Res ul t s of l i ver funct i on t es t i ng are normal , and s erum α-fet oprot ei n (AFP) i s normal . Becaus e t he t umors are hypervas cul ar, l i ver bi ops y i s not s ugges t ed.
(3) T herapy i ncl udes di s cont i nui ng t he us e of oral cont racept i ves and moni t ori ng t umor s i ze t o document regres s i on. If regres s i on does not occur, t he t umor s houl d be removed s urgi cal l y t o prevent rupt ure.
o
o
b. Focal nodular hyperplasia al s o occurs pri mari l y i n women; t he t heory t hat t hi s l es i on i s as s oci at ed wi t h oral cont racept i ve us e i s cont rovers i al . The l es i on i s compos ed of cent ral connect i ve t i s s ue
Pa g e 1 1 1 7
ABC Ambe r CHM Conve rte r Tria l ve rsion, http://w w w .proce sste x t.com/a bcchm.html
wi t h radi at i ng s ept a, whi ch di vi de t he mas s i nt o nodul es . Li ver s canni ng may not s how an abnormal i t y becaus e al l t he el ement s of l i ver t i s s ue, i ncl udi ng Kupffer cel l s , are pres ent . However, CT and angi ography demons t rat e a hypervas cul ar mas s . There i s no pot ent i al for mal i gnant t rans format i on, and hemorrhage, rupt ure, and necros i s are rare. o
o
c. Hemangiomas are t he mos t common beni gn t umors of t he l i ver and are found at aut ops y i n 5%–7% of pat i ent s . W omen are affect ed more commonl y t han men.
(1) Clinical features us ual l y are abs ent . Large l es i ons may be as s oci at ed wi t h t hrombocyt openi a and hypofi bri nogenemi a, es peci al l y i n i nfant s . Hemangi omas al s o may be as s oci at ed wi t h t el angi ect as i a of ot her organs .
(2) Diagnosis i s made by angi ography, rapi d-s equence CT, MRI, or s i ngl e-phot on emi s s i on comput ed t omography (SPECT) l i ver s can. Abdomi nal radi ographs may s how cal ci fi cat i on. Li ver s canni ng s hows a col d s pot , whi ch ul t ras onography s hows t o be a s ol i d mas s . Hemangi omas us ual l y are s i ngl e but may be mul t i pl e.
(3) T herapy us ual l y i s not neces s ary. However, cort i cos t eroi d t herapy, radi at i on t herapy, and embol i zat i on al l have been
Pa g e 1 1 1 8
ABC Ambe r CHM Conve rte r Tria l ve rsion, http://w w w .proce sste x t.com/a bcchm.html
s hown t o be effect i ve i n decreas i ng t he s i ze of l arge hemangi omas .
2. Malignant tumors (s ee Chapt er 4 VII E)
X. Diseases of the Peritoneum, Mesentery, and Abdominal Vasculature A. Diseases of the peritoneum
1. Ascites refers t o t he accumul at i on of fl ui d i n t he peri t oneal cavi t y. o
o
a. Pathogenesis. The fol l owi ng mechani s ms l ead t o as ci t es format i on:
(1) Increased hydrostatic pressure, whi ch may be due t o ci rrhos i s , hepat i c vei n occl us i on (Budd-Chi ari s yndrome), i nferi or vena cava obs t ruct i on, cons t ri ct i ve peri cardi t i s , and conges t i ve heart fai l ure (CHF).
(2) Decreased colloid osmotic pressure, whi ch may res ul t from end-s t age l i ver di s eas e wi t h poor prot ei n s ynt hes i s , nephrot i c s yndrome wi t h prot ei n l os s , mal nut ri t i on, and prot ei n-l os i ng ent eropat hy (PLE).
(3) Increased permeability of peritoneal capillaries, whi ch may res ul t from t ubercul ous
Pa g e 1 1 1 9
ABC Ambe r CHM Conve rte r Tria l ve rsion, http://w w w .proce sste x t.com/a bcchm.html
peri t oni t i s , bact eri al peri t oni t i s , or mal i gnant di s eas e of t he peri t oneum.
(4) Leakage of fluid into the peritoneal cavity, l eadi ng t o bi l e as ci t es , pancreat i c as ci t es (us ual l y s econdary t o a l eaki ng ps eudocys t ), chyl ous as ci t es (s econdary t o l ymphat i c duct di s rupt i on due t o l ymphoma or t rauma), or uri ne as ci t es .
(5) Miscellaneous causes may i ncl ude myxedema, ovari an di s eas e (Mei gs ' s yndrome), or chroni c hemodi al ys i s .
o
o
b. Diagnosis. The pres ence of as ci t es us ual l y i s i ndi cat ed on phys i cal exami nat i on by abdomi nal di s t ent i on, a fl ui d wave, or s hi ft i ng dul l nes s . Abdomi nal ul t ras onography can rel i abl y det ect s mal l amount s of fl ui d. Paracent es i s can be performed wi t h or wi t hout gui dance by ul t ras onography, and t he as ci t i c fl ui d s houl d be anal yzed.
(1) From meas urement of t he al bumi n concent rat i on i n t he as ci t i c fl ui d and t he s erum al bumi n concent rat i on, a SAAG can be det ermi ned.
(a) If t he SAAG i s great er t han 1.1 g/dL, t he pat i ent has port al hypert ens i on (accuracy of approxi mat el y 97%), wi t h ci rrhos i s , cardi ac as ci t es , Budd-Chi ari
Pa g e 1 1 2 0
ABC Ambe r CHM Conve rte r Tria l ve rsion, http://w w w .proce sste x t.com/a bcchm.html
s yndrome, port al vei n t hrombos i s , veno-occl us i ve di s eas e, or fat t y l i ver of pregnancy.
(b) If t he SAAG i s l es s t han 1.1 g/dL, t he pat i ent may have peri t oneal carci nomat os i s , i nfect i on (e.g., peri t oni t i s or t ubercul os i s ), nephrot i c s yndrome, or pancreat i c or bi l i ary as ci t es .
(2) The amyl as e concent rat i on i s el evat ed i n pancreat i c as ci t es .
(3) The t ri gl yceri de concent rat i on i s el evat ed i n chyl ous as ci t es .
(4) Cyt ol ogi c fi ndi ngs are frequent l y pos i t i ve i n mal i gnancy.
(5) An abs ol ut e W BC count [general l y pol ymorphonucl ear l eukocyt es (PMNs )] great er 3
t han 250/mm i s s ugges t i ve of i nfect i on. W hen mononucl ear cel l s predomi nat e, t ubercul os i s or fungal i nfect i on i s l i kel y. P.232
(6) An RBC count great er t han 50,000/mm
3
denot es hemorrhagi c as ci t es , whi ch us ual l y i s
Pa g e 1 1 2 1
ABC Ambe r CHM Conve rte r Tria l ve rsion, http://w w w .proce sste x t.com/a bcchm.html
due t o mal i gnancy, t ubercul os i s , or t rauma. Hemorrhagi c pancreat i t i s , a rupt ured aort i c aneurys m, and a rupt ured hepat i c adenoma may caus e frank bl eedi ng i nt o t he peri t oneal cavi t y.
(7) Bact eri al i nfect i on i s document ed by cul t ure, as Gram s t ai ni ng as ci t i c fl ui d has l ow s ens i t i vi t y.
(8) A pH of l es s t han 7 s ugges t s bact eri al i nfect i on.
o
o
c. T herapy. Treat ment depends on t he underl yi ng caus e. Trans udat i ve as ci t es may be t reat ed wi t h +
bed res t , Na res t ri ct i on, and careful us e of di uret i cs . Paracent es i s of up t o 1 L of fl ui d may provi de rel i ef of acut e res pi rat ory embarras s ment s econdary t o t ens e as ci t es . Removal of l arge vol umes can l ead t o hypovol emi a and s hock. Int ravenous al bumi n can be us ed t o repl ace vol ume and prevent t hi s res pons e. A Le Veen or Denver s hunt may be us ed for i nt ract abl e or mal i gnant as ci t es , but t hes e s hunt s i nt roduce hi gh ri s ks of i nfect i on and DIC. The TIPS procedure al s o has been s hown t o be us eful i n t reat i ng refract ory as ci t es .
2. Bacterial peritonitis o
o
a. Pathogenesis
Pa g e 1 1 2 2
ABC Ambe r CHM Conve rte r Tria l ve rsion, http://w w w .proce sste x t.com/a bcchm.html
(1) Primary or spontaneous bacterial peritonitis us ual l y devel ops i n t he s et t i ng of preexi s t i ng as ci t es .
(2) Secondary or acute bacterial peritonitis us ual l y res ul t s from a perforat ed vi s cus , a rupt ured appendi x, an i nt es t i nal i nfarct i on, or ul cerat i ve col i t i s .
o
o
b. Clinical features
(1) Abdomi nal pai n, wi t h or wi t hout guardi ng and rebound
(2) Fever
(3) Leukocyt os i s
(4) Paral yt i c i l eus
o
o
c. Diagnosis
(1) Paracentesis ai ds i n det ermi ni ng whet her t he fl ui d i s exudat i ve or t rans udat i ve, has an el evat ed W BC count (a predomi nance of PMNs i s di agnos t i c), and can be cul t ured t o i dent i fy t he i nfect i ng organi s m. Inocul at i ng an anaerobi c cul t ure t ube wi t h fres hl y drawn as ci t i c fl ui d (i .e., at t he pat i ent 's beds i de)
Pa g e 1 1 2 3
ABC Ambe r CHM Conve rte r Tria l ve rsion, http://w w w .proce sste x t.com/a bcchm.html
and i nocul at i on of bl ood cul t ure bot t l es are oft en us eful . If t he i ni t i al as ci t i c fl ui d t ot al prot ei n cont ent i s l es s t han 1 mg/dL, pat i ent s have an i ncreas ed ri s k for s pont aneous bact eri al peri t oni t i s .
(2) Radiographic examination i ndi cat es free ai r under t he di aphragm i n t he pres ence of a perforat ed vi s cus , may s how a nons peci fi c i l eus , or may s how a haz y appearance cons i s t ent wi t h as ci t es .
o
o
d. T herapy
(1) Support i ve meas ures i ncl ude i nt ravenous admi ni s t rat i on of fl ui ds , correct i on of el ect rol yt e abnormal i t i es , and nas ogas t ri c s uct i on.
(2) Ant i mi crobi al t herapy for s pont aneous bact eri al peri t oni t i s i ncl udes a t hi rd-generat i on cephal os pori n t o cover gram-negat i ve organi s ms , pneumococcus , ot her s t rept ococci , and anaerobi c organi s ms .
(3) Surgi cal i nt ervent i on i s neces s ary i n cas es of s econdary bact eri al peri t oni t i s .
3. Other causes of peritonitis o
Pa g e 1 1 2 4
ABC Ambe r CHM Conve rte r Tria l ve rsion, http://w w w .proce sste x t.com/a bcchm.html o
a. Bile peritonitis
(1) Pathogenesis. Bi l e s pi l l age i nt o t he peri t oneal cavi t y (e.g., from a rupt ured gal l bl adder or gal l bl adder punct ure duri ng l i ver bi ops y) res ul t s i n a chemi cal peri t oni t i s .
(2) Clinical features. Severe abdomi nal pai n and s hock s econdary t o exudat i on of fl ui d from t he damaged peri t oneum may occur.
(3) T herapy. Treat ment i s s urgi cal aft er pat i ent s have been s t abi l i zed wi t h i nt ravenous vol ume repl acement and correct i on of el ect rol yt e abnormal i t i es .
o
o
b. Starch peritonitis
(1) Pathogenesis. Approxi mat el y 2–4 weeks aft er abdomi nal s urgery, granul omat ous peri t oni t i s devel ops i f t he peri t oneal cavi t y has been cont ami nat ed wi t h s urgi cal gl ove powder (s t arch), l i nt from s urgi cal drapes , part i cl es of s ut ure mat eri al , or t al c.
(2) Clinical features. Abdomi nal pai n, di s t ent i on, t endernes s , and ful l nes s may be pres ent .
(3) Diagnosis. Exami nat i on of t he as ci t i c fl ui d
Pa g e 1 1 2 5
ABC Ambe r CHM Conve rte r Tria l ve rsion, http://w w w .proce sste x t.com/a bcchm.html
under a pol ari zed mi cros cope s hows s t arch granul es t ermed maltese crosses. Laparos copy s hows s t uddi ng of t he peri t oneal s urface.
(4) T herapy. Cort i cos t eroi ds or NSAIDs are appropri at e.
o
o
c. Gonococcal peritonitis (Fitz-Hugh–Curtis syndrome) us ual l y i s s een i n young women and i s caus ed by an as cendi ng i nfect i on ori gi nat i ng i n t he pel vi s . Chl amydi a recent l y has been report ed t o caus e an i dent i cal s yndrome.
(1) Clinical features. Si gns and s ympt oms , whi ch mi mi c t hos e of acut e chol ecys t i t i s , i ncl ude abdomi nal pai n, fever, and ri ght upper quadrant peri t oneal s i gns . Occas i onal l y, a hepat i c fri ct i on rub i s pres ent .
(2) Diagnosis. Laborat ory t es t s s how an el evat ed W BC count and mi l d abnormal i t i es i n l i ver funct i on. Pel vi c exami nat i on may i ndi cat e adnexal t endernes s , and cul t ure of cervi cal mucus us ual l y i s pos i t i ve. Laparos copy s hows violin-string adhesions from t he l i ver t o ei t her t he ri ght adnexa or t he abdomi nal wal l .
(3) T herapy. Ceft ri axone i s admi ni s t ered. If chl amydi a al s o i s pres ent , t et racycl i ne i s gi ven.
Pa g e 1 1 2 6
ABC Ambe r CHM Conve rte r Tria l ve rsion, http://w w w .proce sste x t.com/a bcchm.html
4. Subphrenic abscess refers t o a col l ect i on of pus l ocat ed i nferi or t o t he di aphragm and above t he l i ver, s pl een, or s t omach. o
o
a. Pathogenesis. Abs ces s format i on us ual l y i s a compl i cat i on of di vert i cul i t i s , a rupt ured appendi x, a perforat ed ul cer, or an abdomi nal wound wi t h peri t oneal s oi l i ng. Occas i onal l y, an abs ces s i s s een aft er uncompl i cat ed abdomi nal s urgery.
o
o
b. Clinical features. Fever, l eukocyt os i s , and abdomi nal and s houl der pai n may occur.
o
o
c. Diagnosis. A radi ograph s howi ng el evat i on of one hemi di aphragm may s ugges t t he di agnos i s , whi ch us ual l y requi res demons t rat i on of t he abs ces s cavi t y by CT or ul t ras onography.
o
o
d. T herapy. Treat ment i nvol ves s urgi cal drai nage and broad-s pect rum ant i bi ot i cs t o combat gram-negat i ve as wel l as anaerobi c organi s ms .
5. T umors of the peritoneum o
o
a. Metastatic lesions are t he mos t common peri t oneal t umors . The pri mary l es i on us ual l y i s adenocarci noma of t he gas t roi nt es t i nal t ract , pancreas , or ovary. However, s arcomas , l ymphomas , l eukemi as , and carci noi d t umors al l may i nvol ve t he peri t oneum.
Pa g e 1 1 2 7
ABC Ambe r CHM Conve rte r Tria l ve rsion, http://w w w .proce sste x t.com/a bcchm.html
P.233
(1) Diagnosis. Paracent es i s s howi ng an exudat i ve fl ui d wi t h a moderat el y i ncreas ed l ymphocyt e count and pos i t i ve cyt ol ogi c fi ndi ngs are di agnos t i c. Needl e bi ops y of t he peri t oneum al s o may be us ed.
(2) T herapy. Treat ment i s di rect ed at t he underl yi ng mal i gnancy. Int raperi t oneal i nject i on of a s cl eros i ng agent occas i onal l y may be pal l i at i ve.
o
o
b. Mesothelioma i s s een mos t commonl y i n men ol der t han 50 years of age and i s as s oci at ed wi t h as bes t os expos ure.
(1) Clinical features i ncl ude abdomi nal di s t ent i on, abdomi nal pai n, naus ea, vomi t i ng, and wei ght l os s .
(2) Diagnosis requi res demons t rat i on of mal i gnant cel l s t hrough paracent es i s wi t h cyt ol ogi c t es t i ng, needl e bi ops y, or l aparot omy wi t h bi ops y.
(3) T herapy i ncl udes radi at i on t herapy, chemot herapy, or bot h, but pat i ent res pons e
Pa g e 1 1 2 8
ABC Ambe r CHM Conve rte r Tria l ve rsion, http://w w w .proce sste x t.com/a bcchm.html
us ual l y i s poor. o
o
c. Pseudomyxoma peritonei i s a rare condi t i on charact eri zed by t he pres ence of t hi ck gel at i nous mat eri al i n t he peri t oneal cavi t y.
(1) Pathogenesis. Thi s condi t i on res ul t s from t he rupt ure of ei t her an appendi ceal mucocel e or an ovari an muci nous cys t adenoma. Some i nves t i gat ors have report ed t he pres ence of l ow-grade mal i gnancy i n a hi gh percent age of t he underl yi ng t umors .
(2) Clinical features. Increas i ng abdomi nal gi rt h wi t hout s hi ft i ng dul l nes s i n an ot herwi s e heal t hy i ndi vi dual may occur.
(3) Diagnosis. Laparot omy i s oft en requi red.
(4) T herapy. Treat ment i nvol ves s urgi cal removal of t he muci nous mat eri al and underl yi ng t umor.
B. Diseases of the mesentery
1. Mesenteric panniculitis (mesenteric Weber-Christian disease) i s a rare condi t i on us ual l y s een i n ol der men, whi ch caus es i nfl ammat i on and fi bros i s of t he mes ent ery. o
Pa g e 1 1 2 9
ABC Ambe r CHM Conve rte r Tria l ve rsion, http://w w w .proce sste x t.com/a bcchm.html o
a. Pathogenesis i s t hought t o i nvol ve overgrowt h of normal fat t i s s ue i n t he mes ent ery wi t h s ubs equent degenerat i on, necros i s , and progres s i on t o fi bros i s and s car format i on. The i ni t i at i ng event may be i s chemi a, i nfect i on, or t rauma.
o
o
b. Clinical features i ncl ude crampy abdomi nal pai n, fever, wei ght l os s , naus ea, and vomi t i ng. Lymphat i c obs t ruct i on may devel op wi t h res ul t ant as ci t es , s t eat orrhea, and PLE.
o
o
c. Diagnosis requi res l aparot omy, whi ch s hows a t hi ckened fi brot i c mes ent ery wi t h fat necros i s and i nfi l t rat i on by foamy macrophages .
o
o
d. T herapy wi t h cort i cos t eroi ds or i mmunos uppres s i ve drugs has varyi ng res ul t s . In many pat i ent s , t he proces s appears t o be s el f-l i mi t ed, and t he prognos i s i s excel l ent .
2. Mesenteric cysts are congeni t al anomal i es of t he mes ent eri c l ymphat i c s ys t em, whi ch occur as s l owl y enl argi ng, pai nl es s , round, s moot h, mobi l e mas s es . Treat ment i s drai nage or exci s i on. Mes ent eri c cys t s are beni gn but rarel y may caus e s ympt oms becaus e of rupt ure, bl eedi ng, or t ors i on.
3. Mesenteric adenitis general l y i s s een i n chi l dren and young adul t s and mi mi cs acut e appendi ci t i s . o
Pa g e 1 1 3 0
ABC Ambe r CHM Conve rte r Tria l ve rsion, http://w w w .proce sste x t.com/a bcchm.html o
a. Etiology us ual l y i nvol ves a vi ral i nfect i on. However, many cas es are caus ed by Y ers i ni a.
o
o
b. Clinical features are abdomi nal pai n (whi ch may be s evere), naus ea, vomi t i ng, and fever. Some pat i ent s have addi t i onal evi dence of a vi ral i nfect i on (e.g., pharyngi t i s and myal gi a).
o
o
c. Diagnosis us ual l y i s made at l aparot omy for pres umed appendi ci t i s .
o
o
d. T herapy i ncl udes ant i bi ot i cs , i f Y ers i ni a i s i dent i fi ed, and s upport i ve care.
C. Diseases of the abdominal vasculature
1. Abdominal aortic aneurysm us ual l y mani fes t s as an as ympt omat i c pul s at i l e mas s , but s ome pat i ent s have abdomi nal pai n, back pai n, and l eg i s chemi a. The caus e us ual l y i s at heros cl eros i s . Leakage of bl ood i nt o s urroundi ng t i s s ues wi t h as s oci at ed abdomi nal , back, or fl ank pai n may precede overt rupt ure by s everal weeks . Rupt ure i nt o t he duodenum—occurring as mas s i ve gas t roi nt es t i nal hemorrhage—or i nt o t he abdomen may be cat as t rophi c. Treat ment i s s urgi cal , wi t h repl acement of t he aneurys m wi t h a s ynt het i c aort i c graft . Pos t operat i ve aort oent eri c fi s t ul as wi t h eros i on of t he graft i nt o t he duodenum (us ual l y i n t he s et t i ng of an i nfect ed graft ) may be s een s everal years aft er aneurys mect omy and may res ul t i n fat al bl eedi ng i f not recogni zed earl y.
Pa g e 1 1 3 1
ABC Ambe r CHM Conve rte r Tria l ve rsion, http://w w w .proce sste x t.com/a bcchm.html
2. Acute mesenteric ischemia i s a cl as s i c s yndrome of decreas ed bl ood s uppl y, us ual l y i nvol vi ng t he s uperi or mes ent eri c art ery. Pat i ent s have advanced art eri os cl erot i c cardi ovas cul ar di s eas e, oft en wi t h a hi s t ory of CHF, acut e myocardi al i nfarct i on (MI), cerebrovas cul ar di s eas e, or peri pheral vas cul ar di s eas e. Many pat i ent s have t aken s pl anchni c cons t ri ct i ng agent s s uch as di goxi n. Embol i c di s eas e (whi ch affect s 25% of pat i ent s ) oft en i s as s oci at ed wi t h uns t abl e cardi ac rhyt hms . Thrombos i s and nonoccl us i ve mes ent eri c i s chemi a are t he l eadi ng caus es of acut e mes ent eri c i s chemi c s yndromes , each account i ng for 25% of cas es . Mes ent eri c venous i nfarct i on and i nferi or mes ent eri c art eri al di s eas e account for t he res t of t he cas es . o
o
a. Clinical features. Sudden s evere peri umbi l i cal pai n i s t he mos t common s ympt om. A beni gn but hypoperi s t al t i c abdomen i s obs erved. Ant eropos t eri or (fl at pl at e) radi ograph of t he abdomen oft en s hows normal fi ndi ngs , but may s how s eparat i on of bowel l oops or “t humbpri nt i ng― (s ubmucos al hemorrhage and edema).
o
o
b. Diagnosis. An i ncreas ed W BC count (oft en 3
>20,000/mm ) s upport s t he di agnos i s , whi ch i s confi rmed by abdomi nal angi ography. Doppl er ul t ras onography al s o may s how decreas ed fl ow t hrough t he s uperi or mes ent eri c art eri al or cel i ac t ree. o
o
c. T herapy. Treat ment i s s urgi cal removal of t he
Pa g e 1 1 3 2
ABC Ambe r CHM Conve rte r Tria l ve rsion, http://w w w .proce sste x t.com/a bcchm.html
embol us or t hrombus , al t hough occas i onal l y ant i t hrombot i c agent s , bal l oon angi opl as t y of narrowed ves s el s , or bypas s s urgery i s us ed.
3. Chronic mesenteric ischemia us ual l y i s s een onl y when t here i s s i gni fi cant occl us i on of t wo of t he t hree major s pl anchni c art eri es . The s yndrome us ual l y i s s een i n ol der pat i ent s wi t h a hi s t ory of cardi ovas cul ar di s eas e. o
o
a. Clinical features i ncl ude i nt ermi t t ent crampy abdomi nal pai n occurri ng 15–30 mi nut es aft er eat i ng and l as t i ng s everal hours . Becaus e of t he as s oci at i on of pai n wi t h eat i ng, pat i ent s P.234
charact eri s t i cal l y become fearful of eat i ng and decreas e t hei r i nt ake t o t he poi nt of s ubs t ant i al wei ght l os s . Phys i cal exami nat i on frequent l y di s cl os es evi dence of peri pheral vas cul ar di s eas e, but t here are no s peci fi c fi ndi ngs i ndi cat i ng i nt es t i nal i s chemi a. The pres ence or abs ence of an abdomi nal brui t i s not hel pful . o
o
b. Diagnosis i s di ffi cul t and mus t be bas ed on s t rong cl i ni cal s us pi ci on combi ned wi t h angi ographi c demons t rat i on of s i gni fi cant narrowi ng (>50%) of t wo of t he t hree major s pl anchni c art eri es .
o
o
c. T herapy i s s urgi cal vas cul ar recons t ruct i on. Vas odi l at ors have not been s hown t o be effect i ve.
Pa g e 1 1 3 3
ABC Ambe r CHM Conve rte r Tria l ve rsion, http://w w w .proce sste x t.com/a bcchm.html
4. Ischemic colitis i s due t o a l ack of art eri al bl ood t o t he col on. Al t hough any port i on of t he col on may be affect ed, t he mos t common s i t e i s t he l eft col on and, i n part i cul ar, t he s o-cal l ed “wat ers hed area― at t he s pl eni c fl exure. Thi s area i s vul nerabl e becaus e i t i s t he s i t e where t he s uperi or mes ent eri c art eri al s uppl y ends and t he i nferi or mes ent eri c art eri al s uppl y begi ns . The rect um us ual l y i s s pared becaus e i t has a generous dual bl ood s uppl y. o
o
a. Clinical features i ncl ude bl oody di arrhea, l ower abdomi nal pai n, and occas i onal vomi t i ng. Infarct i on rarel y occurs . The ol der adul t who has a hi s t ory of heart di s eas e or abdomi nal aort i c aneurys m s urgery (wi t h l i gat i on of t he i nferi or mes ent eri c art ery) i s part i cul arl y s us cept i bl e.
o
o
b. Diagnosis i s s ugges t ed by negat i ve fi ndi ngs for ot her caus es of bl oody di arrhea i n t he el derl y popul at i on (i .e., pol yp, carci noma, di vert i cul os i s , and angi odys pl as i a). The W BC count may be 3
el evat ed t o approxi mat el y 20,000/mm . A fl at -pl at e radi ograph of t he abdomen may s how t humbpri nt i ng. Abdomi nal CT s can may s how s egment al t hi ckeni ng of t he bowel wal l . Pneumat os i s and gas i n t he mes ent eri c vei ns may be s een i n more advanced s t ages . Si gmoi dos copy or col onos copy s how mucos al i s chemi a. o
o
c. T herapy i s s upport i ve wi t h NPO i nt ravenous fl ui ds , bl ood repl acement , and ant i bi ot i cs t o
Pa g e 1 1 3 4
ABC Ambe r CHM Conve rte r Tria l ve rsion, http://w w w .proce sste x t.com/a bcchm.html
prevent s econdary i nvas i on. o
o
d. Prognosis general l y i s good. Lat e s t ri ct ures may devel op, whi ch coul d requi re bal l oon di l at at i on or s urgery.
5. Vasculitis. Invol vement of t he mes ent eri c ves s el s by pol yart eri t i s nodos a, l upus eryt hemat os us , or rheumat oi d vas cul i t i s mi mi cs art eri al embol i zat i on (caus i ng bowel i nfarct i on) or chroni c mes ent eri c i s chemi a. The di agnos i s i s s ugges t ed by t he s ys t emi c feat ures of t he di s eas e. Surgery i s requi red for acut e i nfarct i on. Ot herwi s e, medi cal t reat ment wi t h cort i cos t eroi ds , i mmunos uppres s i ve agent s , or bot h, frequent l y i s effect i ve.
6. Splenic infarction i s charact eri zed by s evere abdomi nal pai n i n young pat i ent s (<40 years ) who have pri mary hemat ol ogi c di s eas e (s i ckl e cel l di s eas e, l eukemi a, l ymphoma), or i n ol der pat i ent s (≥40 years ) who have embol i c di s eas es . An abs ces s may devel op wi t h hemorrhage or rupt ure. Di agnos i s i s s ugges t ed by CT or s pl een s can wi t h
99
Tc. Treat ment i s
s urgi cal . P.235
Study Questions/Answers and Explanations 1. A 30-year-old man has had difficulty swallowing both solids and liquids over the last 6 months. What is the most likely diagnosis? A Es ophageal carci noma B. Achal as i a
Pa g e 1 1 3 5
ABC Ambe r CHM Conve rte r Tria l ve rsion, http://w w w .proce sste x t.com/a bcchm.html
C. Schat z ki 's ri ngs D. Beni gn es ophageal s t ri ct ure E. Barret t 's es ophagus Vi ew Ans wer 1. T he answer is B [I A 1 b (2), B 2 c]. Es ophageal mot or di s orders s uch as achal as i a are charact eri zed by dys phagi a for bot h s ol i ds and l i qui ds . Obs t ruct i ve es ophageal condi t i ons s uch as carci noma, s t ri ct ure, and Schat zki 's ri ngs caus e dys phagi a for s ol i ds but al l ow free pas s age of l i qui ds . The dys phagi a as s oci at ed wi t h Schat zki 's ri ngs i s i nt ermi t t ent ; i n carci noma and s t ri ct ure, however, t he dys phagi a i s cons t ant . Barret t 's es ophagus i s t he repl acement of normal s quamous epi t hel i um wi t h col umnar epi t hel i um; t here i s no dys phagi a unl es s an ul cerat i on or s t ri ct ure compl i cat es t hi s condi t i on. 2. A 52-year-old man with a history of heartburn was diagnosed with carcinoma of the esophagus. What is the most likely cell type of this tumor? A Squamous cel l B. Oat cel l C. Trans i t i onal cel l D. Adenocarci noma E. Pri mary es ophageal mel anoma Vi ew Ans wer 2. T he answer is D [I B 2 a (1)]. Adenocarci noma of t he es ophagus has been t he s econd fas t es t ri s i ng cancer i n t he Uni t ed St at es , s econd onl y t o mal i gnant mel anoma. Pri mary cel l t ype i n a 52-year-ol d gent l eman wi t h a hi s t ory of refl ux pl aces hi m at great ri s k for adenocarci noma. Oat cel l and t rans i t i onal cel l carci nomas are not pri mary mal i gnanci es of t he es ophagus , and mal i gnant mel anoma woul d be a met as t at i c l es i on t hat , i n t hi s s et t i ng, woul d be cons i dered unl i kel y. 3. An 84-year-old woman was found to have an esophageal web (Plummer-Vinson syndrome). Which one of the following statements is correct? A They are caus ed by fol at e defi ci ency.
Pa g e 1 1 3 6
ABC Ambe r CHM Conve rte r Tria l ve rsion, http://w w w .proce sste x t.com/a bcchm.html
B. They are l ocat ed i n t he di s t al es ophagus . C. They caus e gas t roes ophageal refl ux. D. Treat ment i ncl udes es ophageal bougi enage. E. They res ul t i n el evat ed i ron s t ores i n t he bl ood. Vi ew Ans wer 3. T he answer is D [I B 2 c]. Es ophageal webs are s een i n t he upper t hi rd of t he es ophagus and may be caus ed by fai l ure of compl et e embryol ogi c recannul at i on or by mucos al prol i ferat i on s econdary t o i ron defi ci ency—t he Pl ummer-Vi ns on (Pat ers on-Kel l y) s yndrome. Becaus e of t he as s oci at ed i ron defi ci ency, t reat ment i ncl udes i ron s uppl ement at i on i n addi t i on t o fract uri ng t he webs wi t h an es ophageal bougi e. 4. A 54-year-old obese woman has chronic gastroesophageal reflux. Which of the following drugs are known to exacerbate her reflux esophagitis? A Chl orpropami de B. Met ocl oprami de C. Theophyl l i ne D. Acet ami nophen E. Omeprazol e Vi ew Ans wer 4. T he answer is C [I B 1 a (2) (b)]. Theophyl l i ne, a β-adrenergi c drug us ed as a bronchodi l at or for t reat i ng as t hma and chroni c bronchi t i s , i s a s moot h mus cl e–rel axi ng agent t hat exacerbat es refl ux es ophagi t i s . Ot her s moot h mus cl e–rel axi ng agent s t hat can exacerbat e gas t roes ophageal refl ux di s eas e i ncl ude di l t i azem, i s os orbi de di ni t rat e, and at ropi ne. Chl orpropami de, an oral hypogl ycemi c agent , has no effect on t he l ower es ophageal s phi nct er (LES). Met ocl oprami de i s a proki net i c agent t hat has cons t ri ct i ng effect s on t he LES and i mproves gas t ri c empt yi ng. Acet ami nophen has no di rect i rri t at i ng effect s on gas t roi nt es t i nal mucos a. Omeprazol e, a prot on pump i nhi bi t or, i s hel pful i n t he management of res i s t ant gas t roes ophageal refl ux di s eas e. 5. A 32-year-old man with HIV has pain on swallowing. Which of the following is the most likely cause?
Pa g e 1 1 3 7
ABC Ambe r CHM Conve rte r Tria l ve rsion, http://w w w .proce sste x t.com/a bcchm.html
A Scl eroderma B. Es ophageal vari ces C. Herpes s i mpl ex vi rus (HSV) i nfect i on D. Achal as i a E. Schat z ki 's ri ngs Vi ew Ans wer 5. T he answer is C [I A 2, B 4 b]. Odynophagi a, or pai n on s wal l owi ng, may be caus ed by mot or di s orders of t he es ophagus (e.g., di ffus e es ophageal s pas m) or mucos al di s rupt i on (e.g., as a res ul t of i nfect i on or drug-i nduced es ophagi t i s ). The mos t i mport ant i nfect i ous agent s are Candi da, herpes s i mpl ex vi rus (HSV), cyt omegal ovi rus (CMV), and human i mmunodefi ci ency vi rus (HIV), i nfect i ons t hat are commonl y s een i n i mmunocompromi s ed hos t s . Severe gas t roes ophageal refl ux wi t h ul cerat i ve es ophagi t i s and radi at i on es ophagi t i s can l ead t o s evere odynophagi a as wel l . Drugs t hat may caus e mucos al di s rupt i on i ncl ude pot as s i um chl ori de t abl et s , t et racycl i ne preparat i ons , cl i ndamyci n, qui ni di ne, as corbi c aci d, and i ron s ul fat e. Scl eroderma i s a mot or di s order t hat affect s t he s moot h mus cl e port i on of t he es ophagus , caus i ng weak, s i mul t aneous , i neffect i ve peri s t al s i s . Dys phagi a i s us ual l y t he onl y s ympt om. Es ophageal vari ces caus ed by port al hypert ens i on are general l y found i nci dent al l y at t he t i me of upper endos copy or when acut e upper gas t roi nt es t i nal bl eedi ng i s pres ent . Schat zki 's ri ngs are beni gn es ophageal s t ri ct ures pri mari l y s een i n t he di s t al es ophagus , i n whi ch dys phagi a i s t he onl y s ympt om pres ent . 6. A 23-year-old woman reports a 2–3-year history of postprandial lower abdominal discomfort and bloating with no specific food predilection. Physical examination and laboratory studies are normal. Celiac antibodies were negative. Which of the following statements concerning the most likely diagnosis is correct? A Lact as e defi ci ency i s t he preferred t erm. B. An underl yi ng neuromus cul ar or hormonal defect i s l i kel y wi t h vi s ceral hypers ens i t i vi t y. C. An underl yi ng i mmunol ogi c defect i s l i kel y.
Pa g e 1 1 3 8
ABC Ambe r CHM Conve rte r Tria l ve rsion, http://w w w .proce sste x t.com/a bcchm.html
D. The s yndrome may be a premal i gnant s t at e. E. Incont i nence i s a common cl i ni cal feat ure. Vi ew Ans wer 6. T he answer is B [IV D 3 a]. Irri t abl e bowel s yndrome, a common caus e of al t ernat i ng di arrhea and cons t i pat i on, i s a funct i onal di s order of mot i l i t y t hat probabl y i nvol ves a neuromus cul ar or hormonal defect . Lact as e defi ci ency i s a s eparat e ent i t y t hat may cont ri but e t o i rri t abl e bowel s yndrome but al s o may be t ot al l y unrel at ed. There i s no evi dence of an i mmunol ogi c defect , and t he s yndrome i s not cons i dered a premal i gnant s t at e. Incont i nence i s a s ympt om of al t ered anorect al phys i ol ogy t hat i s s een wi t h i nfl ammat ory di s eas es of t he anal canal and wi t h s ys t emi c neuromus cul ar di s orders s uch as di abet es or s cl eroderma. Al t hough i ncont i nence occas i onal l y occurs wi t h expl os i ve di arrhea, i ncont i nence i s not a common feat ure of i rri t abl e bowel s yndrome and s houl d s ugges t a s ys t emi c di s order. P.236
7. A 29-year-old internal medicine resident who had received recombinant hepatitis B vaccine most likely had which of the following immunologic markers? A Hepat i t i s B s urface ant i gen (HB s Ag) B. Hepat i t i s B core ant i body (ant i -HB c ) C. Hepat i t i s Be ant i body (ant i -HB e ) D. Hepat i t i s B s urface ant i body (ant i -HB s ) E. Ant i -HB c and ant i -HB s Vi ew Ans wer 7. T he answer is D [IX A 1 d; Fi gure 5-1]. The vacci ne agai ns t hepat i t i s B vi rus (HBV), i n ei t her t he recombi nant or t he pool ed pl as ma form, does not cont ai n hepat i t i s Be or hepat i t i s B core ant i gen (HB e Ag or HB c Ag, res pect i vel y). Therefore, t he ant i body produced i s s i mpl y t hat agai ns t hepat i t i s s urface ant i gen (HB s AG). Becaus e no act i ve HB s Ag i s t rans mi t t ed, t es t s for hepat i t i s B
Pa g e 1 1 3 9
ABC Ambe r CHM Conve rte r Tria l ve rsion, http://w w w .proce sste x t.com/a bcchm.html
s urface ant i genemi a are negat i ve. 8. A 51-year-old man with recurrent peptic ulcer disease had a fasting gastrin level of 1000. A presumptive diagnosis of Zollinger-Ellison syndrome was made. Which of the following organs is the most common site of origin of the tumor associated with this syndrome? A St omach B. Duodenum C. Lymph nodes D. Spl een E. Pancreas Vi ew Ans wer 8. T he answer is E [III I]. The Zol l i nger-El l i s on s yndrome i s a non–β i s l et cel l t umor t hat produces gas t ri n and i s as s oci at ed wi t h gas t ri c aci d hypers ecret i on and pept i c ul cer di s eas e. Tumors are bi ol ogi cal l y mal i gnant i n 60% of cas es , and t he mos t common s i t e i nvol ved i s t he pancreas . Tumor s i ze ranges from 2 mm t o 20 cm. 9. A 79-year-old male smoker with a history of coronary artery disease and peripheral vascular disease developed bloody diarrhea. No infectious pathogens were identified. Which part of the colon is most vulnerable to ischemic insult? A Spl eni c fl exure B. Cecum C. Rect um D. Si gmoi d col on E. Hepat i c fl exure Vi ew Ans wer 9. T he answer is A [X C 4]. Is chemi c col i t i s i s caus ed by a l ack of art eri al bl ood s uppl y t o t he col on. Al t hough any port i on of t he col on may be affect ed, t he mos t common s i t e i s t he l eft col on, part i cul arl y t he s o-cal l ed “wat ers hed― area at t he s pl eni c fl exure. 10. An otherwise healthy 28-year-old man has a 4-month history of epigastric discomfort and heartburn. Symptoms are usually
Pa g e 1 1 4 0
ABC Ambe r CHM Conve rte r Tria l ve rsion, http://w w w .proce sste x t.com/a bcchm.html
exacerbated postprandially, especially after eating spicy foods. T he patient denies dysphagia, weight loss, or decreased appetite. He does not take any medications. Physical examination is normal except for mild epigastric tenderness. Routine lab work is normal. Which of the following is most appropriate at this time? A Bari um s wal l ow B. Upper endos copy C. 24-hour ambul at ory es ophageal pH moni t ori ng D. Tri al of aci d s uppres s i ve t herapy E. Es ophageal manomet ry Vi ew Ans wer 10. T he answer is D [I B 1 d (2)]. Thi s pat i ent has t he cl as s i c fi ndi ngs and s ympt oms of uncompl i cat ed gas t roes ophageal refl ux di s eas e. A res pons e t o aci d s uppres s i ve t herapy i s t he bes t way t o confi rm t he di agnos i s , s i nce addi t i onal t es t i ng i s not i ndi cat ed i f t he pat i ent 's s ympt oms res ol ve wi t h t herapy. Furt her workup woul d be i ndi cat ed wi t h t he pat i ent had any s peci fi c warni ng s i gns (dys phagi a, wei ght l os s , odynophagi a) or di d not res pond t o aci d s uppres s i ve t herapy. 11. A 42-year-old IV drug abuser was found to have both hepatitis B and hepatitis C with chronic elevations of his hepatic transaminases. Which of the following epidemiologic statements is most appropriate? A Chroni c hepat i t i s wi l l devel op i n approxi mat el y 30%–50% of s uch pat i ent s . B. There i s a hi gh 1-year mort al i t y rat e aft er act i ve i nfect i on. C. Vacci ne prot ect i on i s avai l abl e t o prot ect agai ns t bot h di s eas es . D. The co-exi s t ence of HBV and HCV i s rare i n i nt ravenous drug abus ers . E. There i s an i ncreas ed ri s k for hepat oma. Vi ew Ans wer 11. T he answer is E [IX A 1]. Pat i ent s who are chroni c carri ers of hepat i t i s B vi rus (HBV) and t hos e wi t h chroni c act i ve HBV and hepat i t i s C vi rus (HCV) i nfect i on are at i ncreas ed ri s k for
Pa g e 1 1 4 1
ABC Ambe r CHM Conve rte r Tria l ve rsion, http://w w w .proce sste x t.com/a bcchm.html
devel opi ng hepat oma. HBV i s parent eral l y t rans mi t t ed, put t i ng i nt ravenous drug abus ers , homos exual men, and t hos e expos ed t o bl ood or bl ood product s at ri s k. There i s a 10% ri s k of chroni c di s eas e or becomi ng a chroni c carri er. HCV i s t rans mi t t ed t hrough parent eral , s exual , and perhaps peri nat al met hods and account s for 90% of cas es of pos t t rans fus i on hepat i t i s . Approxi mat el y 30%–50% of pat i ent s devel op chroni c hepat i t i s . Ful mi nant HBV i s as s oci at ed wi t h a hi gh mort al i t y rat e, whereas ful mi nant HCV rarel y occurs . Vacci nat i on provi des adequat e prot ect i on agai ns t HBV; however, current l y t here i s no HCV vacci ne. 12. A 32-year-old man recently returned from a ski vacation in New England. Nonbloody diarrhea developed toward the latter stages of his trip. Stool samples tested for fecal leukocytes were negative. Which of the following is the most likely diagnosis? A Shi gel l a i nfect i on B. Es c heri c hi a c ol i s erot ype O157:H7 i nfect i on C. Ul cerat i ve col i t i s D. Gi ardi a l ambl i a i nfect i on E. Col oni c i s chemi a Vi ew Ans wer 12. T he answer is D [IV D 4 b, e, j; V D; X C 4; Tabl e 5-1]. Gi ardi a l ambl i a i s t he mos t common caus e of wat erborne i nfect i ous di arrhea i n t he Uni t ed St at es . The organi s m preferent i al l y res i des i n t he upper s mal l i nt es t i ne, and i nfect ed pat i ent s may be as ympt omat i c, have mi l d di arrhea, or devel op a prol onged i l l nes s charact eri zed by mal abs orpt i on, di arrhea, bl oat i ng, and crampy abdomi nal pai n. Organi s ms s uch as Shi gel l a and Es c heri c hi a c ol i s erot ype O157:H7 are i nvas i ve pat hogens t hat can caus e fever, crampy abdomi nal pai n, and bl oody di arrhea. Ul cerat i ve col i t i s i s an i nfl ammat ory bowel di s eas e charact eri zed by bl oody di arrhea. Col oni c i s chemi a i s charact eri zed by t he acut e ons et of crampy abdomi nal pai n and bl oody di arrhea caus ed by a l ow-fl ow s t at e t o t he col on. P.237
Pa g e 1 1 4 2
ABC Ambe r CHM Conve rte r Tria l ve rsion, http://w w w .proce sste x t.com/a bcchm.html
13. A 22-year-old woman with changes in her bowel habits was found to have multiple discrete polyps. On family history, two first-degree relatives were noted to have colon cancer. In which of the following conditions are multiple polypoid lesions highly associated with malignancy? A Ul cerat i ve col i t i s B. Crohn's di s eas e C. Gardner's s yndrome D. Peut z -Jeghers s yndrome E. Juveni l e pol ypos i s Vi ew Ans wer 13. T he answer is C [V G 2 a (2)]. Gardner's s yndrome, charact eri zed by fami l i al adenomat ous pol ypos i s as s oci at ed wi t h os t eomas or s oft t i s s ue t umors , has an ext remel y hi gh ri s k for t he devel opment of col orect al cancer. Peut z-Jeghers s yndrome i s charact eri zed by mucocut aneous pi gment at i on of t he buccal mucos a and hamart omat ous pol yps i n t he s t omach, s mal l bowel , and col on. Thes e pol yps carry a very l ow ri s k for mal i gnant t rans format i on. Bot h ul cerat i ve col i t i s and Crohn's di s eas e do carry an i ncreas ed ri s k of col on cancer, but t hes e di s orders are not as s oci at ed wi t h adenomat ous col oni c pol yps . If mul t i pl e col oni c pol yps are s een i n ul cerat i ve col i t i s , t hey are general l y ps eudopol yps and are not neopl as t i c. Juveni l e pol ypos i s commonl y l eads t o gas t roi nt es t i nal bl eedi ng from pol yps of t he col on, s mal l bowel , and s t omach, and t he ri s k of mal i gnancy i s s l i ght l y i ncreas ed l at er i n l i fe. 14. A 32-year-old woman was found to have severe iron deficiency anemia. An upper endoscopy, colonoscopy, and video endoscopy were all normal. Family history revealed osteoporosis in her 39-year-old brother, which is currently being investigated. Her only gastrointestinal symptoms were those of occasional postprandial abdominal bloating with episodes of diarrhea, previously diagnosed as irritable bowel syndrome. Her mother has a known diagnosis of rheumatoid arthritis. All laboratory studies are normal. Which of the following is the most
Pa g e 1 1 4 3
ABC Ambe r CHM Conve rte r Tria l ve rsion, http://w w w .proce sste x t.com/a bcchm.html
likely diagnosis? A Ul cerat i ve col i t i s B. Crohn's di s eas e C. Lact os e i nt ol erance D. Irri t abl e bowel s yndrome E. Cel i ac di s eas e Vi ew Ans wer 14. T he answer is E [IV E 4 c]. Cel i ac (nont ropi cal ) s prue i s a di s eas e charact eri zed by abnormal s ens i t i vi t y t o gl ut en, a prot ei n component of wheat . Cel i ac pat i ent s have proxi mal i nt es t i nal i nvol vement wi t h rel at i ve s pari ng of t he di s t al i l eum. Iron i s abs orbed i n t he duodenum, and, t herefore, i ron defi ci ency anemi a not due t o bl ood l os s i s common i n t hi s s et t i ng. Crohn's di s eas e and ul cerat i ve col i t i s are act i ve i nfl ammat ory di s eas es t hat woul d be as s oci at ed wi t h changes on i nt es t i nal vi s ual i zat i on. Lact os e i nt ol erance woul d not be as s oci at ed wi t h i ron defi ci ency anemi a. A co-exi s t i ng aut oi mmune di s eas e i n her mot her (rheumat oi d art hri t i s ) and os t eoporos i s i n a young mal e are cl ues t hat s he mos t l i kel y has as s oci at ed fami l y members wi t h ei t her cel i ac di s eas e or ot her aut oi mmune di s orders . 15. Which of the following findings is likely to be found in this patient? A Promi nent vi l l i on s mal l i nt es t i ne bi ops y B. 3 g of D-xyl os e i n a 5-hour uri ne col l ect i on C. Hi gh carot ene l evel wi t h normal vi t ami n A i nt ake D. Four grams of fat on a 72-hour fecal fat col l ect i on E. Negat i ve Sudan s t ai n Vi ew Ans wer 15. T he answer is B [IV E 3 a, b, (onl i ne) e, i ]. Aft er a 25-g oral dos e of D-xyl os e, a 5-hour uri ne col l ect i on s houl d cont ai n at l eas t 5 g D-xyl os e. The fi ndi ng of l es s t han 4–5 g D-xyl os e i n t he s t ool i s i ndi cat i ve of t he mal abs orpt i on s yndromes s een wi t h cel i ac di s eas e. Fl at vi l l i wi t h i nfl ammat ory cel l i nfi l t rat i on on s mal l bowel bi ops y are charact eri zed by cel i ac di s eas e. The s erum carot ene l evel i s a refl ect i on of vi t ami n A met abol i s m. Becaus e vi t ami n A i s a
Pa g e 1 1 4 4
ABC Ambe r CHM Conve rte r Tria l ve rsion, http://w w w .proce sste x t.com/a bcchm.html
fat -s ol ubl e vi t ami n, a l ow s erum carot ene l evel wi t h normal vi t ami n A i nt ake may be us eful i n s creeni ng for fat mal abs orpt i on. A pos i t i ve Sudan s t ai n i s i ndi cat i ve of an underl yi ng mal abs orpt i ve proces s . However, t he gol d s t andard t es t for fat mal abs orpt i on i s a 72-hour s t ool col l ect i on for fecal fat . The coeffi ci ent of fat abs orpt i on i n t he s mal l i nt es t i ne i s 7%. As a res ul t , a pat i ent cons umi ng a 100-g fat di et s houl d have no more t han 7 g fat i n t he s t ool each day; more t han 7 g fat woul d be cons i s t ent wi t h a mal abs orpt i on s yndrome. 16. A 22-year-old college student was found to have right lower quadrant pain, fever, leukocytosis, and localization to McBurney' s point. On rectal examination, he had right lower quadrant tenderness. T he most likely diagnosis is: A Di vert i cul i t i s B. Ul cerat i ve col i t i s C. Appendi ci t i s D. Tubo-ovari an abs ces s E. Chol ecys t i t i s Vi ew Ans wer 16. T he answer is C [IV I]. Acut e appendi ci t i s mos t oft en occurs i n mal es bet ween t he ages of 10 and 30 years . Cl i ni cal feat ures i ncl ude ri ght l ower quadrant (McBurney's poi nt ) pai n, fever, and l eukocyt os i s . Di fferent i al di agnos es i ncl ude acut e gas t roent eri t i s , mes ent eri c adeni t i s , Meckel 's di vert i cul um, Crohn's di s eas e, ovari an t ors i on, rupt ured ovari an cys t , pel vi c i nfl ammat ory di s eas e (PID), and, i n el derl y pat i ent s , di vert i cul i t i s , chol ecys t i t i s , i ncarcerat ed herni a, and mes ent eri c t hrombos i s . 17. A 52-year-old man comes in for routine examination. He feels well, takes no medications, and has no family history of colorectal cancer. Physical examination and complete blood count are normal. He returns three fecal occult stool test cards, with one window testing positive for occult blood. What is the appropriate next step in management? A Repeat fecal occul t bl ood t es t i ng i n 3 mont hs B. Doubl e cont ras t bari um enema
Pa g e 1 1 4 5
ABC Ambe r CHM Conve rte r Tria l ve rsion, http://w w w .proce sste x t.com/a bcchm.html
C. Fl exi bl e s i gmoi dos copy D. Di gi t al rect al exami nat i on E. Col onos copy Vi ew Ans wer 17. T he answer is E [V G 3]. Col onos copy i s t he mos t appropri at e t es t for pat i ent s wi t h a pos i t i ve fecal occul t bl ood t es t . Annual fecal occul t bl ood t es t i ng, fol l owed by col onos copy i f t he t es t i s pos i t i ve, reduces col orect al cancer mort al i t y by as much as 33%. Col onos copy can det ect s ubt l e mucos al abnormal i t i es s uch as t el angi ect as i as , areas of i nfl ammat i on or ul cerat i on, and neopl as ms . In addi t i on mos t pol yps and s ome earl y s t age cancers can be removed duri ng t he procedure. 18. A 48-year-old woman was referred to a gastrointestinal clinic for recurrent nausea with occasional episodes of diarrhea alternating with constipation. A solid-phase gastric emptying study indicated markedly delayed gastric emptying. T he most likely explanation for gastroparesis in this setting is: A Chol i nergi c drug t herapy B. Duodenal ul cer C. Di abet es i ns i pi dus D. Scl eroderma E. Gas t ri c vari ces Vi ew Ans wer 18. T he answer is D [
onl i ne II C 5]. Gas t ropares i s i s a di s order
of gas t ri c empt yi ng and i s not as s oci at ed wi t h mechani cal obs t ruct i on. It i s mos t frequent l y as s oci at ed wi t h a great er t han 10-year hi s t ory of t ype 1 (i ns ul i n-dependent ) di abet es mel l i t us . Ot her condi t i ons as s oci at ed wi t h gas t ropares i s i ncl ude s ys t emi c s cl eros i s , pos t vagot omy s t at es , and t he us e of ant i chol i nergi c agent s . Proki net i c agent s (e.g., met ocl oprami de, domperi done, eryt hromyci n, and ci s apri de) have been us ed t o t reat gas t ropares i s . Gas t ri c vari ces have no effect on gas t ri c empt yi ng. P.238
Pa g e 1 1 4 6
ABC Ambe r CHM Conve rte r Tria l ve rsion, http://w w w .proce sste x t.com/a bcchm.html
19. A fourth-year medical student rotating on a radiology elective was presented an x-ray of the abdomen, which showed multiple air–fluid levels with dilated loops of small bowel, paucity of air in the colon, and no air in the rectum. T he radiology attending turned to the fourth-year medical student and asked which of the following clinical features would most likely be found in this patient. A Hypoact i ve bowel s ounds B. Pai n out of proport i on t o phys i cal exami nat i on C. Crampy abdomi nal pai n t hat waxes and wanes D. Di arrhea E. A fl at , ri gi d abdomen Vi ew Ans wer 19. T he answer is C [IV A 1]. The pat i ent has a mechani cal i nt es t i nal obs t ruct i on, as t he des cri pt i on of t he ai r–fl ui d l evel s i ndi cat es a mechani cal i nt es t i nal obs t ruct i on. Mechani cal i nt es t i nal obs t ruct i on may be t he res ul t of ext ri ns i c, i nt ramural , or i nt ral umi nal caus es . Sympt oms i ncl ude crampy abdomi nal pai n t hat waxes and wanes , obs t i pat i on or cons t i pat i on, naus ea and vomi t i ng, and abdomi nal di s t ent i on. Phys i cal exami nat i on of t he abdomen reveal s hi gh-pi t ched bowel s ounds and rus hes and t i nkl es , as wel l as marked abdomi nal di s t ent i on and t ympany on percus s i on. Pai n out of proport i on t o t he phys i cal exami nat i on i s mos t s ugges t i ve of acut e mes ent eri c i s chemi a. 20. A 50-year-old man undergoing screening colonoscopy with no associated symptoms was found to have a single small colonic polyp. Inquiring about his risk of cancer, which of the following circumstances indicates the greatest risk for cancer in an individual polyp? A W hen t hey are of t ubul ar hi s t ol ogy B. W hen t hey are as s oci at ed wi t h act i ve bl eedi ng C. W hen t hey are l arger t han 2 cm i n di amet er D. W hen t hey are peduncul at ed E. W hen pat i ent s are younger t han 50 years of age Vi ew Ans wer
Pa g e 1 1 4 7
ABC Ambe r CHM Conve rte r Tria l ve rsion, http://w w w .proce sste x t.com/a bcchm.html
20. T he answer is C [V G 1]. Adenomat ous pol yps t hat repres ent an i ncreas ed ri s k for adenocarci noma are great er t han 2 cm i n di amet er, vi l l ous rat her t han t ubul ar, and s es s i l e rat her t han peduncul at ed. There i s no as s oci at i on wi t h i ncreas ed ri s k rel at ed t o bl eedi ng or pat i ent age. 21. A 56-year-old patient with advanced alcoholic cirrhosis and known ascites is found to have abdominal pain, fever to 102°F, and a peripheral white blood cell count of 17,000 with a shift to the left. Which of the following statements regarding the primary diagnosis is correct? A It i s more l i kel y when as ci t i c fl ui d t ot al prot ei n exceeds 1 mg/dL. B. It devel ops i n t he s et t i ng of preexi s t i ng as ci t es . 3
C. The as ci t i c pol ymorphonucl ear count i s l es s t han 100 cel l s /mm . D. It i s oft en as s oci at ed wi t h as pergi l l os i s . E. It i s as s oci at ed wi t h a perforat ed vi s cus . Vi ew Ans wer 21. T he answer is B [X A 2]. The cl i ni cal feat ures of s pont aneous (pri mary) bact eri al peri t oni t i s , whi ch devel ops i n a s et t i ng of preexi s t i ng as ci t es , i ncl ude abdomi nal pai n, fever, l eukocyt os i s , and paral yt i c i l eus . The i ni t i al as ci t i c fl ui d t ot al prot ei n count i s 3
l es s t han 1 mm /dL. The abs ol ut e pol ymorphonucl ear count i n t he as ci t i c fl ui d i s general l y great er t han 250 cel l s /dL. Bact eri al peri t oni t i s as s oci at ed wi t h a perforat ed vi s cus i s s econdary bact eri al peri t oni t i s . 22. A 76-year-old man with a history of an acute myocardial infarction and peripheral vascular disease is seen in the emergency department for very severe abdominal pain, out of proportion to clinical findings. On physical examination, his abdomen is soft with hypoactive bowel sounds. Which of the following is associated with the most likely diagnosis? A A normal whi t e bl ood cel l (W BC) count B. Invol vement of t he i nferi or mes ent eri c art ery C. Cons t i pat i on D. A defi ni t i ve cl i ni cal pres ent at i on E. Lack of a s i gni fi cant medi cal hi s t ory
Pa g e 1 1 4 8
ABC Ambe r CHM Conve rte r Tria l ve rsion, http://w w w .proce sste x t.com/a bcchm.html
Vi ew Ans wer 22. T he answer is D [X C 2]. Thi s pat i ent has acut e mes ent eri c i s chemi a, whi ch i s a cl as s i c s yndrome charact eri zed by decreas ed bl ood s uppl y; us ual l y t he s uperi or mes ent eri c art ery i s i nvol ved. In general , pat i ent s have comorbi d condi t i ons s uch as at heros cl erot i c cardi ovas cul ar di s eas e, conges t i ve heart fai l ure (CHF), acut e myocardi al i nfarct i on (MI), cerebrovas cul ar di s eas e, or peri pheral vas cul ar di s eas e. Cl i ni cal pres ent at i on i s t he bas i s of t he di agnos i s . Sympt oms i ncl ude s udden, s evere peri umbi l i cal pai n wi t h a beni gn phys i cal exami nat i on (s ympt oms are out of proport i on t o t he phys i cal exami nat i on). Abdomi nal radi ographs may s how s eparat i on of bowel l oops or “t humbpri nt i ng.― The l eukocyt e 3
count i s general l y great er t han 20,000 cel l s /mm , and met abol i c aci dos i s i s pres ent . 23. A 32-year-old woman with Raynaud' s phenomenon has had heartburn and regurgitation for 2 years. What is the most likely mechanism for these symptoms in this case? A Pres ence of Hel i c obac t er pyl ori i n t he gas t ri c mucos a B. Decreas ed l ower es ophageal s phi nct er (LES) pres s ure C. Increas ed gas t ri c aci d s ecret i on D. Decreas ed peri s t al s i s i n t he upper t hi rd of t he es ophagus E. Es ophageal mus cul ar s pas m Vi ew Ans wer 23. T he answer is B [I B 3 d]. The mos t l i kel y mechani s m for gas t roes ophageal refl ux i n pat i ent s wi t h s cl eroderma i s decreas ed l ower es ophageal s phi nct er (LES) pres s ure. Pres ence of Hel i c obac t er pyl ori i n gas t ri c mucos a i s not a predi s pos i ng fact or for gas t roes ophageal refl ux. Al t hough i ncreas ed gas t ri c aci d s ecret i on may caus e and aggravat e gas t roes ophageal refl ux, i t i s not t he mechani s m i n t hi s cas e. Decreas ed es ophageal peri s t al s i s can be obs erved i n pat i ent s wi t h s cl eroderma but occurs i n t he l ower t hi rd of es ophagus . Es ophageal mus cl e s pas m i s not as s oci at ed wi t h gas t roes ophageal refl ux. 24. A 52-year-old man developed dysphagia for solids. An upper gastrointestinal series showed a mass lesion in the distal
Pa g e 1 1 4 9
ABC Ambe r CHM Conve rte r Tria l ve rsion, http://w w w .proce sste x t.com/a bcchm.html
esophagus. Endoscopy confirmed the presence of adenocarcinoma of the esophagus. Which of the following is the most important predisposing factor in his disease? A Achal as i a B. Pal mopl ant ar kerat os i s (t yl os i s ) C. Barret t 's es ophagus D. Cel i ac s prue E. Al cohol i nt ake Vi ew Ans wer 24. T he answer is C [I B 2 a, I B 1 e (5)]. Barret t 's es ophagus i s t he major predi s pos i ng fact or i n pat i ent s wi t h adenocarci noma of t he es ophagus . Smoki ng, al cohol i nges t i on, geographi c fact ors , achal as i a, and t yl os i s are i mport ant predi s pos i ng fact ors for s quamous cel l carci noma of t he es ophagus . Achal as i a and cel i ac s prue are as s oci at ed wi t h s quamous cel l carci noma of t he es ophagus . P.239
25. A 38-year-old man from T aiwan recently moved to this country and was found to have hepatitis B (HBV) on blood studies. Which of the following laboratory tests most reliably distinguish a chronic healthy carrier state from chronic active hepatitis B disease? A Serum hepat i t i s B s urface ant i gen (HB s AG) B. Serum hepat i t i s B core ant i body (ant i -HB c ) C. Serum ant i –s moot h mus cl e ant i body D. Li ver bi ops y E. Serum α-fet oprot ei n (AFP) Vi ew Ans wer 25. T he answer is D [IX A 1 f (2)]. The charact eri s t i c hi s t ol ogi c fi ndi ngs i n pat i ent s wi t h chroni c hepat i t i s are i nfl ammat i on of port al t ri ad wi t h pi ecemeal necros i s . Chroni c heal t hy carri ers of hepat i t i s B vi rus (HBV) have normal l i ver hi s t ol ogy. Hepat i t i s B s urface ant i gen (HB s AG) and core ant i body (ant i -HB c ) are det ect ed
Pa g e 1 1 5 0
ABC Ambe r CHM Conve rte r Tria l ve rsion, http://w w w .proce sste x t.com/a bcchm.html
i n heal t hy carri ers and pat i ent s wi t h chroni c hepat i t i s . Ant i –s moot h mus cl e ant i body and α-fet oprot ei n (AFP) are not markers for HBV. 26. A 35-year-old man with acquired immunodeficiency 3
syndrome (AIDS) with a CD4 count of 150 cells/mm has had jaundice and fever for 1 month. Liver function tests indicate the following:
T otal bilirubin: 3.2 mg/dL
Direct bilirubin: 2.7 mg/dL
Alkaline phosphatase: Elevated (three times normal)
AST and ALT : Normal
Liver ultrasound shows hepatomegaly with normal caliber biliary ducts, and liver biopsy indicates granulomatous liver disease. Which of the following is the most likely diagnosis? A Pol ycys t i c l i ver di s eas e B. Hepat i t i s C vi rus (HCV) C. Myc obac t eri um avi um-i nt rac el l ul are (MAI) D. Scl eros i ng chol angi t i s E. Hepat i t i s B vi rus (HBV) Vi ew Ans wer 26. T he answer is C [IX B 5]. In i mmunocompromi s ed pat i ent s , Myc obac t eri um avi um-i nt rac el l ul are (MAI) can caus e fever. Pres ence of granul oma i s hi ghl y s ugges t i ve of mycobact eri al i nfect i on. Pol ycys t i c l i ver di s eas e general l y does not caus e i ncreas ed l i ver t es t s . Hepat i t i s B vi rus (HBV) and hepat i t i s C vi rus (HCV) caus e necroi nfl ammat ory parenchymal l i ver di s eas e, and pat i ent s pres ent wi t h el evat ed as part at e ami not rans feras e (AST) and al ani ne ami not rans feras e (ALT). Al t hough s cl eros i ng chol angi t i s may occur
Pa g e 1 1 5 1
ABC Ambe r CHM Conve rte r Tria l ve rsion, http://w w w .proce sste x t.com/a bcchm.html
i n pat i ent s wi t h acqui red i mmunodefi ci ency s yndrome (AIDS), granul oma i s not t he us ual hi s t ol ogi c fi ndi ng i n t hes e i ndi vi dual s . 27. Results of hepatitis serology are as follows:
Positive hepatitis A IgG antibody
Negative hepatitis A IgM antibody
Positive hepatitis B surface antibody (anti-HB s )
Negative hepatitis B core antibody (anti-HB c )
Negative hepatitis C antibody
Which of the following is the most likely diagnosis? A Acut e hepat i t i s A vi rus (HAV) B. Acut e hepat i t i s B vi rus (HBV) C. Immuni t y agai ns t hepat i t i s C vi rus (HCV) D. Previ ous vacci nat i on agai ns t hepat i t i s B vi rus (HBV) E. Pas t expos ure t o hepat i t i s B vi rus (HBV) Vi ew Ans wer 27. T he answer is D [IX A 1 d]. The pres ence of hepat i t i s B s urface ant i body (ant i -HB s ) i n t he abs ence of hepat i t i s B core ant i body (ant i -HB c ) i ndi cat es previ ous vacci nat i on agai ns t hepat i t i s B (HBV). Pat i ent s wi t h acut e hepat i t i s A (HAV) have hepat i t i s A IgM ant i body, and t hos e wi t h acut e HBV are pos i t i ve for hepat i t i s B s urface ant i gen (HB s Ag). 28. A 35-year-old man with a history of IV drug abuse was found to have chronic fatigue and AST elevated 4 times with an ALT elevated at fivefold normal. Polyarteritis was subsequently diagnosed by vascular biopsy. T he most likely diagnosis is: A Hepat i t i s B
Pa g e 1 1 5 2
ABC Ambe r CHM Conve rte r Tria l ve rsion, http://w w w .proce sste x t.com/a bcchm.html
B. Hepat i t i s C C. Hepat i t i s A D. Hepat i t i s E E. Hepat i t i s G Vi ew Ans wer 28. T he answer is A [IX A]. Hepatitis B 29. Which of the following is indicative of active viral replication in the above patient? A Hepat i t i s B DNA l evel B. Hepat i t i s C DNA l evel C. Hepat i t i s C RNA l evel D. Hepat i t i s B RNA l evel E. Hepat i t i s C ant i body Vi ew Ans wer 29. T he answer is D [IX A]. Hepatitis B RNA level P.240
30. T he best means to protect his first-degree relatives in the absence of any prior evidence of hepatitis would be: A Vacci nat i on B. Pool ed i mmune gl obul i n (gamma gl obul i n) C. Avoi dance of hous ehol d cont act D. Avoi dance of s hel l fi s h i nges t i on E. Hepat i t i s i mmunogl obul i n Vi ew Ans wer 30. T he answer is A [IX A]. Vaccination Pol yart eri t i s nodos a or gl omerul onephri t i s may devel op i n pat i ent s wi t h hepat i t i s B vi rus (HBV). HBV pl aces i nt ravenous drug abus ers , homos exual men, and i ndi vi dual s expos ed t o bl ood and bl ood product s at hi gh ri s k for HBV, a DNA vi rus . Hepat ocel l ul ar carci noma, ci rrhos i s , and chroni c hepat i t i s may devel op i n pat i ent s wi t h HBV, HCV, and hepat i t i s D vi rus (HDV). HDV i s a s mal l , defect i ve RNA vi rus (del t a agent ). Cryogl obul i nemi a occurs i n
Pa g e 1 1 5 3
ABC Ambe r CHM Conve rte r Tria l ve rsion, http://w w w .proce sste x t.com/a bcchm.html
pat i ent s wi t h HCV onl y and not t hos e wi t h ot her forms of hepat i t i s . Ful mi nant hepat i t i s devel ops i n HAV, HBV, and HCV. W i t h hepat i t i s E vi rus i nfect i on, t he mort al i t y rat e i n pregnant women may be 10%–20%. Vacci nes for t he prevent i on of bot h HAV and HBV i nfect i ons are avai l abl e, whi ch are far more effect i ve i n prevent i ng t rans mi s s i on t han recommendat i ons t o avoi d cont act or expos ure t o hepat i t i s pat i ent s . Hepat i t i s B i mmune gl obul i n (HBIG) has been s hown t o decreas e t he s everi t y of hepat i t i s B but i s not as effect i ve as vacci nat i on i n prevent i ng hepat i t i s B.
Pa g e 1 1 5 4
ABC Ambe r CHM Conve rte r Tria l ve rsion, http://w w w .proce sste x t.com/a bcchm.html
Editors: Wolfsthal, Susan T itle: NMS Medicine, 6th Edition Copyri ght ©2008 Li ppi ncot t W i l l i ams & W i l ki ns > T able of Cont ent s > Chapt er 6 - Renal Diseases, Fluid and Elec t rolyt e Disorders, and Hypert ension
Chapter 6
Renal Diseases, Fluid and Electrolyte Disorders, and Hypertension Marc Brazie Donna Hanes
Part I: Renal Diseases I. Clinical Assessment of Renal Function
A. Urinalysis
1. Color. Uri ne normal l y i s yel l ow. o
o
a. Darkening on s t andi ng may be s een wi t h s ome di s eas es (e.g., porphyri a) and wi t h cert ai n drugs (e.g., met hyl dopa).
o
o
b. Red–orange–brown urine may be s een wi t h hemat uri a, hemogl obi nuri a, and myogl obi nuri a and wi t h cert ai n drugs (e.g., phenot hi azi nes ).
2. Chemistry. Qual i t at i ve chemi cal anal ys i s of uri ne i s performed wi t h commerci al l y avai l abl e dipsticks.
Pa g e 1 1 5 5
ABC Ambe r CHM Conve rte r Tria l ve rsion, http://w w w .proce sste x t.com/a bcchm.html o
o
a. Blood us ual l y i s not pres ent i n normal uri ne. Int act eryt hrocyt es , hemogl obi n, and myogl obi n al l produce pos i t i ve t es t res ul t s .
o
o
b. Glucose above 0.3 g/24 hr us ual l y i s not pres ent i n normal uri ne.
o
o
c. Ketone bodies are pres ent i n t he uri ne of heal t hy i ndi vi dual s onl y duri ng fas t i ng. Sodi um ni t roprus s i de reagent det ect s acet oacet at e but not β-hydroxybut yrat e.
o
o
d. Protein above 150 mg/24 hr us ual l y i s not pres ent i n normal uri ne. The di ps t i ck det ect s onl y al bumi n, not i mmunogl obul i ns or l i ght -chai n pol ypept i des , whi ch mus t be as s ayed us i ng aci d preci pi t at i on.
o
o
e. Bilirubin i s not pres ent i n normal uri ne. If el evat ed i n bl ood, wat er-s ol ubl e conjugat ed bi l i rubi n i s fi l t ered and pres ent i n uri ne.
o
o
f. Urine pH can be maxi mal l y aci di fi ed bel ow a pH of 5.0 and maxi mal l y al kal i ni zed above a pH of 7.5.
3. Concentration and dilution. Thes e val ues are meas ured by ei t her s peci fi c gravi t y (normal = 1.000–1.025) or os mol al i t y (normal = 50–1000 mOs m/kg uri ne). Many fact ors can affect uri ne
Pa g e 1 1 5 6
ABC Ambe r CHM Conve rte r Tria l ve rsion, http://w w w .proce sste x t.com/a bcchm.html
concent rat i on and di l ut i on.
4. Urinary sediment of formed el ement s i s prepared by cent ri fugat i on of uri ne at 2000 rpm for 10 mi nut es . The s edi ment from 12 mL of uri ne i s res us pended i n 1 mL of s upernat ant and i s exami ned mi cros copi cal l y. o
o
a. Crystals t hat are s een i n aci d uri ne i ncl ude cys t i ne and uri c aci d; t hos e found i n al kal i ne uri ne i ncl ude cal ci um phos phat e and cal ci um oxal at e.
o
o
b. Cells t hat are found i n vari ous di s eas e s t at es i ncl ude eryt hrocyt es , l eukocyt es , and epi t hel i al cel l s (i .e., renal t ubul ar, t rans i t i onal , or s quamous ).
o
o
c. Bacteria may be s een and are bes t confi rmed wi t h Gram s t ai ni ng of t he s edi ment .
o
o
d. Casts are cyl i ndri cal el ement s formed i n di s eas e s t at es as s oci at ed wi t h l ow i nt rarenal uri ne fl ow or heavy prot ei nuri a. The cas t i s a prot ei n coagul um, whi ch i s formed i n t he renal t ubul e and t raps any t ubul ar l umi nal cont ent s wi t hi n i t s mat ri x. Cas t s are named for t he el ement s recogni zed wi t hi n t hem s uch as :
(1) Red bl ood cel l (RBC) cas t
(2) W hi t e bl ood cel l (W BC) cas t
Pa g e 1 1 5 7
ABC Ambe r CHM Conve rte r Tria l ve rsion, http://w w w .proce sste x t.com/a bcchm.html
(3) Renal t ubul ar cel l cas t
(4) Granul ar cas t
(5) Hyal i ne cas t
(6) W axy cas t
B. Renal function testing
1. Glomerular filtration rate (GFR) i s a meas ure of t he amount of pl as ma ul t rafi l t rat e deri ved from bl ood i n a s peci fi ed peri od. (A normal GFR i s 115–125 mL/mi n.) In mos t ki dney di s eas es , t he GFR i s an accurat e i ndex of overal l renal funct i on.
2. Urine concentrating ability i s det ermi ned by meas uri ng uri ne os mol al i t y aft er 18–24 hours of wat er depri vat i on and agai n aft er t he admi ni s t rat i on of 5 uni t s of vasopressin. Under t hes e condi t i ons , uri ne reaches an os mol al i t y of 900 mOs m/kg (or a s peci fi c gravi t y of 1.023) i n 90% of normal i ndi vi dual s .
3. Urine diluting ability i s det ermi ned by meas uri ng uri ne os mol al i t y and vol ume 5 hours aft er a wat er l oad of 20 mL/kg body wei ght . In normal i ndi vi dual s , uri ne reaches an os mol al i t y of 100 mOs m/kg (or a s peci fi c
Pa g e 1 1 5 8
ABC Ambe r CHM Conve rte r Tria l ve rsion, http://w w w .proce sste x t.com/a bcchm.html
gravi t y of 1.003), and uri ne vol ume exceeds 80% of t he wat er l oad.
4. Renal urine acidification can be t es t ed by admi ni s t eri ng 100 mg ammoni um chl ori de per kg body wei ght t o decreas e pl as ma bi carbonat e concent rat i on bel ow 20 mEq/L. Uri ne normal l y aci di fi es (i .e., uri ne pH drops bel ow 5.5) under t hes e condi t i ons . Fas t i ng uri ne pH normal l y i s bel ow 5.5.
C. Radiography
1. Pl ai n fi l m radi ography, ul t ras onography, and comput ed t omography (CT) are us eful noni nvas i ve t echni ques for det ermi ni ng renal s i ze and t he pres ence of obs t ruct i on, s t ones , or mas s l es i ons .
2. Ot her modal i t i es s uch as Doppl er ul t ras onography may be us eful for det ermi ni ng bot h vas cul ar fl ow rat es and uri ne fl ow. Magnet i c res onance angi ography (MRA) i s anot her met hod t hat al l ows noni nvas i ve as s es s ment of art eri al funct i on.
3. Int ravenous urography and art eri ography may al s o hel p defi ne i nt rarenal morphol ogy.
D. Renal biopsy
Pa g e 1 1 5 9
ABC Ambe r CHM Conve rte r Tria l ve rsion, http://w w w .proce sste x t.com/a bcchm.html
1. Indications i ncl ude acut e renal fai l ure of unknown et i ol ogy or abnormal cours e, del ayed recovery from acut e renal fai l ure, and a poorl y funct i oni ng or det eri orat i ng renal al l ograft . Al s o, renal bi ops y occas i onal l y may be i ndi cat ed i n cas es of nephrot i c s yndrome, acut e gl omerul onephri t i s , and i n defi ni ng t he progres s i on of l upus nephri t i s .
2. Contraindications i ncl ude di as t ol i c bl ood pres s ure exceedi ng 100 mm Hg, i nfect i on at t he bi ops y s i t e, and abnormal bl ood coagul at i on.
II. Acute Renal Failure Thi s s udden, rapi d, but pot ent i al l y revers i bl e det eri orat i on i n renal funct i on i s s uffi ci ent t o caus e ni t rogenous was t e accumul at i on i n body fl ui ds .
A. Etiology Caus es may be prerenal , pos t renal , or parenchymal (Tabl e 6-1).
B. Clinical features
1. Azotemia. Ri s i ng bl ood urea ni t rogen (BUN) and s erum creat i ni ne l evel s are t he mos t readi l y avai l abl e l aborat ory s i gns of a decreas e i n GFR. Thes e bi ochemi cal changes may be i ndependent of cl i ni cal s ympt oms . Confoundi ng vari abl es t hat i nfl uence BUN and creat i ni ne mus t be cons i dered before renal fai l ure i s confi rmed. o
o
a. BUN l evel i s affect ed by rat es of urea product i on, a funct i on of t he amount of di et ary prot ei n or prot ei n breakdown (e.g., cat abol i c drugs or t i s s ue i njury), and by res orpt i on of gas t roi nt es t i nal or s oft -t i s s ue hemorrhage.
Pa g e 1 1 6 0
ABC Ambe r CHM Conve rte r Tria l ve rsion, http://w w w .proce sste x t.com/a bcchm.html o
o
b. Creat i ni ne l evel i s affect ed by endogenous creat i ni ne product i on (i ncreas ed by breakdown of mus cl e t i s s ue), by renal creat i ni ne s ecret i on (whi ch i s bl ocked by s uch drugs as ci met i di ne and t ri met hopri m), and by noncreat i ni ne chromogens (us ual l y drugs ) t hat caus e meas urement errors .
2. Derangement of urine volume o
o
a. Anuria (i .e., uri ne out put of <100 mL/day). Us ual l y an omi nous s i gn, anuri a oft en i ndi cat es ei t her compl et e art eri al occl us i on or s evere renal i njury. However, uri ne vol ume per s e confers very l i t t l e di agnos t i c s peci fi ci t y.
o
o
b. Oliguria (i .e., uri ne out put <500 mL/day). Such out put i s i ns uffi ci ent t o excret e t he dai l y os mol ar l oad. Al t hough mos t pat i ent s wi t h acut e renal fai l ure are ol i guri c, 25%–50% of s uch pat i ent s are not and produce more t han 800 mL of uri ne dai l y. P.246
TABLE 6-1 Causes of Acute Renal Failure
Pa g e 1 1 6 1
ABC Ambe r CHM Conve rte r Tria l ve rsion, http://w w w .proce sste x t.com/a bcchm.html
P at h Cl o a p ss h E ifi ys x c io a at lo m io g pl n y e Pr S G er ev as e er t r n e oi al ex nt t r es ac t i el n l u al l a bl r e vo e l u di m n e g d C e o pl n et g i o es n ti D ve ec h re e
Pa g e 1 1 6 2
ABC Ambe r CHM Conve rte r Tria l ve rsion, http://w w w .proce sste x t.com/a bcchm.html
as ar e t d fa re i l n ur al e p R er e fu n s i al o e n m R b e ol n us al ar te ri al o bs tr uc ti o n P In Ac os t r ut t r at e e u ur n b at al ul e * ar n o e
Pa g e 1 1 6 3
ABC Ambe r CHM Conve rte r Tria l ve rsion, http://w w w .proce sste x t.com/a bcchm.html
bs p t r hr uc o ti p o at n hy In St tr a ar g e h n or al n p ca el l c vi ul c us o Ki bs d tr n uc ey ti st o o n n Ur e et St er o o n p e, el cl vi ot c , o co bs m t r pr uc es
Pa g e 1 1 6 4
ABC Ambe r CHM Conve rte r Tria l ve rsion, http://w w w .proce sste x t.com/a bcchm.html
ti si o o n n Ur by et ex er t r al ar o e bs n t r al uc l y ti m o p n h Bl n a o d d d es er Pr -o os ut t a le ti t c o hy bs p t r er uc t r ti o o p n hy R Ac S e ut e n e ps al t u i s p b A
Pa g e 1 1 6 5
ABC Ambe r CHM Conve rte r Tria l ve rsion, http://w w w .proce sste x t.com/a bcchm.html
ar ul m e ar i n nc n o hy ec gl m ro yc al s i os s id N e e a p nt hr i b ot i o ox t i i ci cs ty , In ra t ri di ns oc ic o re nt n ra al s t di dy s e es as P es os Gl t s o tr m e er pt ul oc o oc n ca e l p gl
Pa g e 1 1 6 6
ABC Ambe r CHM Conve rte r Tria l ve rsion, http://w w w .proce sste x t.com/a bcchm.html
hr o iti m s er T ul u o b n ul e oi p nt hr er i t i st s i t i Dr al u n ge in p d hr uc iti e s d V W as e cu g lit e is n er 's gr a n ul o m at os is
Pa g e 1 1 6 7
ABC Ambe r CHM Conve rte r Tria l ve rsion, http://w w w .proce sste x t.com/a bcchm.html
*Mus t be bi l at er al , except in pat i en t s wi t h onl y one ki dney when it is uni l at e ral . o
o
c. Polyuria. Pat i ent s may have acut el y ri s i ng BUN and s erum creat i ni ne l evel s yet produce more t han 3 L of uri ne dai l y. Thi s condi t i on may repres ent a l es s s evere form of acut e renal fai l ure, wi t h pres ervat i on of s mal l amount s of gl omerul ar fi l t rat i on i n t he pres ence of t ubul ar damage. Pat i ent s wi t h part i al uri nary t ract obs t ruct i on frequent l y pres ent wi t h pol yuri a.
C. Diagnosis
1. Patient history. Acut e renal fai l ure us ual l y res ul t s from s everal , oft en s ynergi s t i c, renal i njuri es . A hi s t ory s houl d i ncl ude i nformat i on concerni ng: o
o
a. Recent s urgi cal and radi ographi c procedures
Pa g e 1 1 6 8
ABC Ambe r CHM Conve rte r Tria l ve rsion, http://w w w .proce sste x t.com/a bcchm.html o
o
b. Pas t and pres ent us e of medi cat i ons (e.g., ami nogl ycos i des or nons t eroi dal ant i -i nfl ammat ory drugs [NSAIDs ])
o
o
c. Al l ergi es
o
o
d. Underl yi ng chroni c renal di s eas e
o
o
e. Fami l y hi s t ory of renal di s eas e
o
o
f. Hi s t ory of voi di ng di ffi cul t i es (s ugges t i ve of obs t ruct i ve uropat hy), or s ympt oms of bl adder dys funct i on (e.g., hes i t ancy, urgency)
2. Physical examination. The phys i cal exami nat i on s houl d be organi zed t o paral l el t he di fferent i al di agnos i s . o
o
a. Prerenal failure i s s ugges t ed by cl i ni cal s i gns of:
(1) Int ravas cul ar vol ume depl et i on (e.g., ort hos t at i c changes i n bl ood pres s ure and pul s e, poor s ki n t urgor)
(2) Conges t i ve heart fai l ure (CHF)
o
Pa g e 1 1 6 9
ABC Ambe r CHM Conve rte r Tria l ve rsion, http://w w w .proce sste x t.com/a bcchm.html
o
b. Acute allergic interstitial nephritis i s s ugges t ed by eos i nophi l i a, eos i nophi l uri a, fever, and macul opapul ar ras h.
o
o
c. Lower urinary tract obstruction i s s ugges t ed by a s uprapubi c or fl ank mas s .
3. Urinalysis o
o
a. Sediment. Mi cros copi c exami nat i on of uri nary s edi ment provi des i nformat i on for t he di fferent i al di agnos i s .
(1) The pres ence of few formed el ement s or onl y hyaline casts i s s ugges t i ve of prerenal or pos t renal fai l ure.
(2) An abundance of erythrocytes i s uncommon i n t he abs ence of cal cul i , t rauma, i nfect i on, or t umor.
(3) An abundance of leukocytes may s i gni fy i nfect i on, i mmune-medi at ed i nfl ammat i on, or an al l ergi c react i on s omewhere i n t he uri nary t ract . P.247
Pa g e 1 1 7 0
ABC Ambe r CHM Conve rte r Tria l ve rsion, http://w w w .proce sste x t.com/a bcchm.html
(4) Eosinophiluria occurs i n up t o 95% of pat i ent s wi t h acut e al l ergi c i nt ers t i t i al nephri t i s . Hans el 's s t ai n oft en i s hel pful t o di s t i ngui s h eos i nophi l s from neut rophi l s i n uri ne.
(5) Brownish pigmented cellular casts and many renal t ubul ar epi t hel i al cel l s are obs erved i n 75% of pat i ent s wi t h acut e t ubul ar necros i s (ATN). Pi gment ed cas t s wi t hout eryt hrocyt es i n t he s edi ment from uri ne wi t h a pos i t i ve di ps t i ck for occul t bl ood i ndi cat e ei t her hemogl obi nuri a or myogl obi nuri a.
(6) RBC casts are vi rt ual l y pat hognomoni c for acut e gl omerul onephri t i s
(Onl i ne Fi gure 6-1).
ONLINE FIGURE 6-1 Red bl ood cel l cas t s (arrows ) wi t hi n ki dney
Pa g e 1 1 7 1
ABC Ambe r CHM Conve rte r Tria l ve rsion, http://w w w .proce sste x t.com/a bcchm.html
t ubul es . (From W i l l i ams G, Mal l i ck NP. Col or At l as of Renal Di s eas es . 2nd ed. Barcel ona: W ol fe Publ i s hi ng/Mos by-Y ear Book Europe Lt d, 1994:4.117. ) o
o
b. Culture. Uri ne cul t ure s houl d be performed i n al l pat i ent s .
o
o
c. Urine and blood chemistries. Several bi ochemi cal i ndi ces ai d i n eval uat i on. Mai nl y, t hes e t es t s di s t i ngui s h acut e ol i guri a due t o prerenal azot emi a from t hat due t o parenchymal renal di s eas e (ATN), on t he bas i s t hat renal t ubul ar funct i on i s pres erved i n t he former condi t i on and s everel y di s t urbed i n t he l at t er.
(1) The fractional excretion of sodium i s t he rat i o of t he uri ne-t o-pl as ma s odi um rat i o t o t he uri ne-t o-pl as ma creat i ni ne rat i o expres s ed as a percent age [(U N a /P N a )/(U C r /P C r ) × 100]. Val ues bel ow 1% s ugges t prerenal fai l ure, and val ues above 1% s ugges t ATN.
(2) Abnormal blood chemistries occas i onal l y ai d i n t he di agnos i s of renal fai l ure. A BUN-t o-s erum creat i ni ne rat i o above 20 i s common i n prerenal azot emi a.
4. Radiography o
Pa g e 1 1 7 2
ABC Ambe r CHM Conve rte r Tria l ve rsion, http://w w w .proce sste x t.com/a bcchm.html o
a. Ultrasonography i s t he met hod of choi ce for i dent i fyi ng t he pres ence of t wo ki dneys , for eval uat i ng ki dney s i ze and s hape, and for det ect i ng hydronephrosis or hydroureter. Renal cal cul i , abdomi nal aneurys ms , and renal vei n t hrombos i s s omet i mes are det ect ed by ul t ras onography.
o
o
b. Isotopic flow scans are margi nal l y us eful for eval uat i ng t he degree of renal perfus i on and t he pres ence of obs t ruct i ve uropat hy. Part i cul arl y us eful i s t he radi opharmaceut i cal agent diethylenetriamine pentaacetic acid (DT PA), whi ch i s excret ed onl y when t here i s free fl ow. Scanni ng us i ng hippurate i s us eful i n as s es s i ng whet her t ubul ar funct i on i s i nt act . Is ot opi c s cans are mos t hel pful i n eval uat i ng t he funct i on of t he renal al l ograft .
o
o
c. CT s cans are es peci al l y us eful i n eval uat i ng t he nat ure of cystic masses (i .e., beni gn or mal i gnant ).
o
o
d. Retrograde pyelography i s performed by i nject i ng cont ras t mat eri al i nt o t he uret eral ori fi ce duri ng cys t os copi c exami nat i on. Speci fi c i ndi cat i ons i ncl ude cert ai n cas es of s us pect ed obs t ruct i ve uropat hy i n whi ch i nt ervent i on t o rel i eve t he obs t ruct i on i s cont empl at ed.
5. Biopsy i s rel evant i n onl y a s el ect group of candi dat es , becaus e t he hi s t ol ogi c s everi t y and cl i ni cal
Pa g e 1 1 7 3
ABC Ambe r CHM Conve rte r Tria l ve rsion, http://w w w .proce sste x t.com/a bcchm.html
cours e of acut e renal fai l ure us ual l y do not correl at e wel l . It i s res erved for pat i ent s i n whom t he caus e of nephrot i c s yndrome i s s ought or i n whom an acut e i nfl ammat ory l es i on s uch as vas cul i t i s i s s us pect ed and requi res cyt ot oxi c t herapy for t reat ment . (Pat i ent s who fol l ow a cl as s i c l aborat ory and cl i ni cal cours e of ATN us ual l y do not benefi t from renal bi ops y.)
6. Cystoscopy i s i ndi cat ed i n al l cas es of uret hral obs t ruct i on and i n s ome cas es of uret eral obs t ruct i on.
D. Clinical course
1. Stages. Acut e renal fai l ure due t o ATN t ypi cal l y occurs i n t hree s t ages : azotemic, diuretic, and recovery. The i ni t i al , az ot emi c s t age can be ei t her ol i guri c or non-ol i guri c (Fi gure 6-2).
2. Morbidity and mortality. The occurrence of ol i guri a negat i vel y affect s morbi di t y and mort al i t y rat es . o
o
a. Gas t roi nt es t i nal bl eedi ng, s ept i cemi a, met abol i c aci demi a, and neurol ogi c abnormal i t i es are more common i n pat i ent s wi t h ol i guri a.
o
o
b. The ri s k of mort al i t y i s more t han t wo t i mes great er i n pat i ent s wi t h ol i guri a, al t hough ot her fact ors s uch as concomi t ant res pi rat ory or cardi ac fai l ure al s o i ncreas e mort al i t y dramat i cal l y.
3. Prognosis. Bot h t he s everi t y of t he underl yi ng
Pa g e 1 1 7 4
ABC Ambe r CHM Conve rte r Tria l ve rsion, http://w w w .proce sste x t.com/a bcchm.html
di s eas e and t he cl i ni cal s et t i ng i n whi ch acut e renal fai l ure occurs affect out come. For exampl e, t he mort al i t y rat e among pat i ent s wi t h ATN i s 60% when ATN i s a res ul t of s urgery or t rauma, 30% when i t occurs as a compl i cat i on of medi cal i l l nes s , and 10%–15% when pregnancy i s i nvol ved. Is chemi a-as s oci at ed ATN has nearl y t wi ce t he mort al i t y ri s k of nephrot oxi c ATN. In pat i ent s wi t h no compl i cat i ng fact ors who s urvi ve an P.248
epi s ode of acut e renal fai l ure, t he chance of compl et e recovery of ki dney funct i on i s 90%. Recent dat a s ugges t t hat t he us e of cert ai n t ypes of di al yzers i n pat i ent s who requi re di al ys i s may al s o i nfl uence prognos i s (s ee Part I: II E 4).
FIGURE 6-2 St ages of acut e t ubul ar necros i s . (Modi fi ed from
Pa g e 1 1 7 5
ABC Ambe r CHM Conve rte r Tria l ve rsion, http://w w w .proce sste x t.com/a bcchm.html
Thadhani R. Acut e renal fai l ure. N Engl J Med 1996;30:1448. )
E. Therapy
1. Preliminary measures o
o
a. Exclusion of reversible causes. Obs t ruct i on s houl d be rel i eved, nephrot oxi c drugs s houl d be wi t hdrawn, i nfect i on s houl d be t reat ed, and el ect rol yt e derangement s s houl d be correct ed.
o
o
b. Correction of prerenal factors. Int ravas cul ar vol ume and cardi ac performance s houl d be opt i mi zed.
o
o
c. Maintenance of urine output. Al t hough t he prognos t i c i mport ance of ol i guri a i s debat ed, management of pat i ent s wi t hout ol i guri a i s cl earl y eas i er t han management of pat i ent s wi t h ol i guri a. Hemodynami c paramet ers and i nt ravas cul ar vol ume s houl d be opt i mi zed. Loop di uret i cs are rarel y us eful t o convert t he ol i guri c form of ATN t o t he non-ol i guri c form. Sus t ai ned i nfus i ons of l oop-act i ve di uret i cs rat her t han bol us i nfus i ons are t he mos t effect i ve mechani s m for i ncreas i ng uri ne fl ow.
2. Conservative measures o
o
a. Management of fluids, electrolytes, and
Pa g e 1 1 7 6
ABC Ambe r CHM Conve rte r Tria l ve rsion, http://w w w .proce sste x t.com/a bcchm.html
anemia. Pat i ent s wi t h acut e renal fai l ure are cat abol i c and us ual l y l os e 0.3 kg of body wei ght dai l y. W ei ght gai n or s t abi l i t y us ual l y i ndi cat es s al t and wat er ret ent i on. P.249
(1) Tot al oral and i nt ravenous wat er admi ni s t rat i on s houl d equal dai l y sensible losses (vi a uri ne, s t ool , and nas ogas t ri c or s urgi cal t ube drai nage) pl us es t i mat ed insensible (i .e., res pi rat ory and dermal ) l os s es , whi ch us ual l y equal 600–800 mL/day.
(2) Combi ned di et ary and i nt ravenous s odi um and pot as s i um i nt ake s houl d not exceed t he meas ured 24-hour uri nary l os s es of t hes e el ect rol yt es .
(3) Sodi um bi carbonat e s houl d be admi ni s t ered i f aci demi a becomes s evere (i .e., i f s erum bi carbonat e concent rat i on drops bel ow 16 mEq/L).
(4) Oral phos phat e-bi ndi ng ant aci ds (e.g., calcium acetate) s houl d be gi ven i f t he s erum phos phat e concent rat i on exceeds 6.0 mg/dL.
(5) Magnes i um-cont ai ni ng drugs (e.g.,
Pa g e 1 1 7 7
ABC Ambe r CHM Conve rte r Tria l ve rsion, http://w w w .proce sste x t.com/a bcchm.html
magnes i um ci t rat e, magnes i um hydroxi de–cont ai ni ng ant aci ds ) s houl d be wi t hhel d.
(6) Eryt hropoi et i n and i ron s uppl ement s s houl d be admi ni s t ered when t he hemat ocri t fal l s bel ow 32%.
o
o
b. Dietary management. Adequat e cal ori c i nt ake i s es s ent i al for pat i ent s wi t h renal fai l ure. General l y, a di et t hat provi des 40–60 g of prot ei n and 35–50 kcal /kg l ean body wei ght i s s uffi ci ent . However, i n s ome pat i ent s , s evere cat abol i s m occurs , and prot ei n s uppl ement at i on t o achi eve 1.25 g of prot ei n/kg body wei ght i s requi red t o mai nt ai n ni t rogen bal ance.
3. Drug usage. Pat i ent s who devel op renal fai l ure abrupt l y s how onl y a 1.0 mg/dL/day i ncreas e i n s erum creat i ni ne becaus e endogenous creat i ni ne product i on remai ns cons t ant . Therefore, i t i s i mpos s i bl e t o cal cul at e appropri at e drug dos es bas ed on s erum creat i ni ne l evel unt i l a new s t eady s t at e i s achi eved. Meas urement of s erum drug l evel s oft en i s neces s ary for s afe drug us e.
4. Dialysis. Thi s procedure i s i ndi cat ed i n t he management of progres s i ve renal fai l ure t hat l eads t o s evere uremi a, i nt ract abl e aci demi a, hyperkal emi a, or vol ume overl oad. Cl i ni cal res earch has demons t rat ed t hat t he us e of art i fi ci al biocompatible membranes i mproves t he mort al i t y of pat i ent s wi t h acut e renal fai l ure who requi re di al ys i s . In addi t i on t o hemodi al ys i s
Pa g e 1 1 7 8
ABC Ambe r CHM Conve rte r Tria l ve rsion, http://w w w .proce sste x t.com/a bcchm.html
and peri t oneal di al ys i s , chronic venovenous hemofiltration (CVVH or CVVHD) i s a hi ghl y effect i ve form of renal repl acement t herapy. Thi s modal i t y ut i l i zes hi ghl y permeabl e membranes , whi ch al l ow t he di al ys i s or fi l t rat i on proces s es t o occur at very l ow hydros t at i c pres s ures and fl ows .
F. Complications of acut e renal fai l ure are s ummari zed i n
ONLINE Tabl e 6-2.
ONLINE TABLE 6-2: Complications of Acute Renal Failure Ma nif est ati Ac on ute s Co or mp Eti lica olo tio gie n s Int Inv rav ol v as c es ul a wei r
ght
ove gai rl o n, ad hyp ert ens i on
Pa g e 1 1 7 9
ABC Ambe r CHM Conve rte r Tria l ve rsion, http://w w w .proce sste x t.com/a bcchm.html
, el e vat ed cen t ral ven ous pre ssu re (as i nd i ca t ed by i nt ern al jug ul a r vei n di s t en sio n), an d pul mo nar y or
Pa g e 1 1 8 0
ABC Ambe r CHM Conve rte r Tria l ve rsion, http://w w w .proce sste x t.com/a bcchm.html
per i ph era l ed em a Hy De per vel kal ops em as ia
a
(i .e res .,
ul t
s er of um dec pot rea as s s ed iu
ren
m
al
con exc cen ret i t ra on t i o co n
mb
>5. i ne 5
d
mE wi t q/L h )
tis s ue nec ros is or
Pa g e 1 1 8 1
ABC Ambe r CHM Conve rte r Tria l ve rsion, http://w w w .proce sste x t.com/a bcchm.html
he mo l ys i s Hy Re po s ul nat t s re fro mi m a
exc
(i .e es s .,
i ve
s er wa um t er s od i nt iu
ake
m
in
con t he cen fac t ra e t i o of n
exc
<1 ret 35 ory mE fai l q/L ure ) Hy Re per s ul ph t s os p fro hat m em on ia
goi
(i .e ng .,
ph
Pa g e 1 1 8 2
ABC Ambe r CHM Conve rte r Tria l ve rsion, http://w w w .proce sste x t.com/a bcchm.html
s er os p um hor ph us os p i nt hat ake e
in
con t he cen fac t ra e t i o of n
exc
>5. ret 5
ory
mg fai l /dL ure )
or tis s ue nec ros
is Hy Re poc s ul al c t s em fro ia
m
(i .e dec .,
rea
s er s ed um 1,2 cal 5-h ci u ydr m
oxy
con vi t cen am
Pa g e 1 1 8 3
ABC Ambe r CHM Conve rte r Tria l ve rsion, http://w w w .proce sste x t.com/a bcchm.html
t ra i n tio D n
l ev
<8. el s 5
,
mg hyp /dL erp )
hos ph at e mi a, or hyp oal bu mi ne mi
a Hy Rar per el y cal occ ce urs mi dur a
i ng
(i .e t he .,
rec
s er ove um ry cal ph ci u as e m
fol l
con owi cen ng
Pa g e 1 1 8 4
ABC Ambe r CHM Conve rte r Tria l ve rsion, http://w w w .proce sste x t.com/a bcchm.html
t ra rha t i o bd n
om
>1 yol 0.5 ys i mg s -i /dL nd )
uce d acu te ren al fai l
ure Aci Is de as s mi oci a
at e
(i .e d .,
wi t
art h eri s ep al
sis
pH or <7. s ev 35) ere he art fai l ure Hy Do per es uri not ce req mi ui r
Pa g e 1 1 8 5
ABC Ambe r CHM Conve rte r Tria l ve rsion, http://w w w .proce sste x t.com/a bcchm.html
a
e t he rap y unl es s t he s er um uri c aci d con cen t ra tio n exc ee ds 15 mg
/dL Bl e Ma edi y ng occ ur s ec on dar y to pl a t el
Pa g e 1 1 8 6
ABC Ambe r CHM Conve rte r Tria l ve rsion, http://w w w .proce sste x t.com/a bcchm.html
et dys fun ct i on an d coa gul op at h y as s oci at e d wi t h s ep sis Sei Occ z ur as i es on al l y occ ur an d are rel at e d to ure
Pa g e 1 1 8 7
ABC Ambe r CHM Conve rte r Tria l ve rsion, http://w w w .proce sste x t.com/a bcchm.html
mi a Chr Ma oni y c
occ
ki d ur. ney Th di s e eas ma e
jori ty of pat i en ts wh o s ur vi v e acu te ren al fai l ure rec ove r fro m t he acu te i ns
Pa g e 1 1 8 8
ABC Ambe r CHM Conve rte r Tria l ve rsion, http://w w w .proce sste x t.com/a bcchm.html
ul t . Ho we ver , s ur vi vi ng pat i en t wi t h di a bet es me llit us or wi t h un der l yi ng ren al di s eas e are mo re lik
Pa g e 1 1 8 9
ABC Ambe r CHM Conve rte r Tria l ve rsion, http://w w w .proce sste x t.com/a bcchm.html
el y to dev el o p ren al fai l ure t ha t req ui r es di a l ys i s.
III. Chronic Kidney Disease Chroni c ki dney di s eas e i s defi ned as a s ubs t ant i al and i rrevers i bl e reduct i on i n renal funct i on t hat devel ops over a peri od of mont hs or years . The s t ages of chroni c ki dney di s eas e are out l i ned i n Tabl e 6-3.
A. Etiology
1. Prerenal causes i ncl ude s evere, l ong-s t andi ng renal art ery s t enos i s and bi l at eral renal art eri al embol i s m.
TABLE 6-3 Stages of Chronic Kidney Disease
Pa g e 1 1 9 0
ABC Ambe r CHM Conve rte r Tria l ve rsion, http://w w w .proce sste x t.com/a bcchm.html
E st i DmT e at r sc e e St ri d at a pt G m g io F e e n R nt 1 Ki > Di d 9 a n 0 g e
n
y
o
d
s
a
e
m
a
a
n
g
d
e
tr
wi
e
th
at
n
ca
or
u
m
s
al
e,
or
sl
in
o
cr
w
e
pr
a
o
s
gr
e
e
d
ss
Pa g e 1 1 9 1
ABC Ambe r CHM Conve rte r Tria l ve rsion, http://w w w .proce sste x t.com/a bcchm.html
G
io
F
n,
R
e v al u at e ri sk of ca rd io v a sc ul ar di s e a s
e 2 Ki 6 E d 0 st n â i e €“ m y 8 at d 9 e a
pr
m
o
a
gr
g
e
Pa g e 1 1 9 2
ABC Ambe r CHM Conve rte r Tria l ve rsion, http://w w w .proce sste x t.com/a bcchm.html
e
ss
wi
io
th
n
m il d d ec re a s e in G F R 3 Ki 3 E d 0 v n â al e €“ u y 5 at d 9 e a
a
m
n
a
d
g
tr
e
e
wi
at
th
co
m
m
o
pl
d
ic
er
at
at
io
e
n
Pa g e 1 1 9 3
ABC Ambe r CHM Conve rte r Tria l ve rsion, http://w w w .proce sste x t.com/a bcchm.html
d
s
ec re a s e in G F R 4 Ki 1 Pr d 5 e n â p e €“ ar y 2 e d 9 fo a
r
m
re
a
n
g
al
e
re
wi
pl
th
ac
s
e
e
m
v
e
er
nt
e
th
d
er
ec
a
re
p
a
y
s e in
Pa g e 1 1 9 4
ABC Ambe r CHM Conve rte r Tria l ve rsion, http://w w w .proce sste x t.com/a bcchm.html
G F R 5 Ki < In d 1 iti n 5 at e
e
y
re
fa
n
il
al
ur
re
e
pl ac e m e nt th er a p y
GFR, gl omerul ar fi l t rat i on rat e. P.250
2. Renal causes i ncl ude di abet es mel l i t us , hypert ens i on, chroni c gl omerul onephri t i s , chroni c t ubul oi nt ers t i t i al nephri t i s , s ys t emi c l upus eryt hemat os us (SLE), amyl oi dos i s , cys t i c di s eas es ,
Pa g e 1 1 9 5
ABC Ambe r CHM Conve rte r Tria l ve rsion, http://w w w .proce sste x t.com/a bcchm.html
neopl as i a, and human i mmunodefi ci ency vi rus (HIV).
3. Postrenal causes deri ve from l ong-s t andi ng uri nary obs t ruct i on.
B. Clinical features Pres ent i ng mani fes t at i ons are hi ghl y vari abl e. The fol l owi ng cons t el l at i on of s i gns and s ympt oms i s referred t o as uremia.
1. Neurologic signs of l et hargy, s omnol ence, confus i on, and neuromus cul ar i rri t abi l i t y devel op ei t her gradual l y or abrupt l y. As t eri xi s i s a t ypi cal fi ndi ng.
2. Cardiovascular signs of hypert ens i on, CHF, and peri cardi t i s al s o may be preci pi t ous .
3. Gastrointestinal signs, part i cul arl y anorexi a, naus ea, vomi t i ng, and a met al l i c t as t e, are very common.
4. Metabolic signs can ei t her be nons peci fi c (e.g., fat i gue, pruri t us , s l eep di s t urbances ) or be referabl e t o a s peci fi c defect (e.g., bone pai n from s econdary hyperparat hyroi di s m).
C. Diagnosis The i mport ant ai m of t he di agnos t i c approach i s t o es t abl i s h t he chroni ci t y of t he renal di s eas e as wel l as t he pot ent i al et i ol ogi es .
D. Therapy
1. Dietary restrictions are vi t al t o t he proper care of pat i ent s t o reduce s ympt oms and, pos s i bl y, ret ard t he
Pa g e 1 1 9 6
ABC Ambe r CHM Conve rte r Tria l ve rsion, http://w w w .proce sste x t.com/a bcchm.html
progres s i on of renal fai l ure. Di et ary prot ei n i s res t ri ct ed t o 0.6 g/kg l ean body wei ght i n earl y chroni c ki dney di s eas e, and di et ary s odi um i s res t ri ct ed t o 4 g/day unl es s res i dual uri ne out put obl i gat es great er dai l y l os s es . (In t hes e cas es , uri ne s odi um excret i on s houl d be meas ured and repl aced, but not exceeded, i n t he di et .) Di et ary i nt ake of pot as s i um, magnes i um, and phos phorus i s res t ri ct ed, and a fl ui d i nt ake l i mi t i s es t abl i s hed bas ed on dai l y l os s es . It s houl d be not ed t hat mos t pat i ent s wi t h chroni c renal fai l ure who do not yet requi re di al ys i s do not need s evere fl ui d res t ri ct i on.
2. Renal replacement therapy i s neces s ary for mai nt enance care of end-s t age renal di s eas e. o
o
a. Indications i ncl ude cl i ni cal uremi a, s evere azot emi a (i .e., GFR <15 mL/mi n), i nt ract abl e hyperkal emi a or aci demi a, and i nt ravas cul ar vol ume overl oad.
o
o
b. Modalities i ncl ude hemodi al ys i s , peri t oneal di al ys i s , and renal al l ograft t rans pl ant at i on.
E. Complications Vari ous di s orders ari s e i n t he cours e of chroni c renal fai l ure and duri ng l ong-t erm renal repl acement t herapy.
1. Hematologic disorders i ncl ude s evere anemi a and bl eedi ng.
Pa g e 1 1 9 7
ABC Ambe r CHM Conve rte r Tria l ve rsion, http://w w w .proce sste x t.com/a bcchm.html
2. Cardiovascular disorders i ncl ude hypert ens i on, peri cardi t i s , cardi omyopat hy, arrhyt hmi as , and CHF.
3. Neuromuscular disorders i ncl ude general i zed s ei z ures , confus i on, l et hargy, emot i onal l abi l i t y, myopat hy, peri pheral neuropat hy, and s yndromes rel at ed t o nerve compres s i on (e.g., carpal t unnel s yndrome).
4. Gastrointestinal disorders i ncl ude ul cers , gas t roduodeni t i s , col i t i s , and angi omas of t he ent i re gas t roi nt es t i nal t ract .
5. Endocrine disorders i ncl ude s econdary hyperparat hyroi di s m, cl i ni cal l y eut hyroi d hypot hyroxi nemi a, hyperprol act i nemi a, al t ered pi t ui t ary and gonadal funct i on (amenorrhea and i mpot ence), and gynecomas t i a.
6. Immune system disturbances i ncl ude l ymphocyt openi a, anergy, i ncreas ed s erum ant i compl ement act i vi t y, and abnormal monocyt e mot i l i t y. W het her pat i ent s may have i ncreas ed vul nerabi l i t y t o i nfect i ous di s eas es remai ns unproven.
7. Metabolic disorders i ncl ude renal os t eodys t rophy (os t ei t i s fi bros a and os t eomal aci a) and al t ered drug met abol i s m.
IV. Medical Complications of Renal Replacement Therapy
Pa g e 1 1 9 8
ABC Ambe r CHM Conve rte r Tria l ve rsion, http://w w w .proce sste x t.com/a bcchm.html
A. Introduction Renal repl acement t herapy i n pat i ent s wi t h chroni c renal fai l ure i s i ndi cat ed for uremi a (es peci al l y peri cardi t i s , neuropat hy, and os t eodys t rophy); i nt ract abl e hyperkal emi a, aci demi a, and CHF; ext racel l ul ar fl ui d vol ume overl oad t hat i s unres pons i ve t o di uret i cs ; and cert ai n i nt oxi cat i ons and poi s oni ngs . The choi ce of modal i t y (i .e., hemodi al ys i s , peri t oneal di al ys i s , or renal t rans pl ant at i on) and t he t i mi ng of i ni t i at i on of renal repl acement t herapy are bas ed on pat i ent age, underl yi ng di s eas es , compl i cat i ng medi cal condi t i ons , pat i ent preference and mot i vat i on, and pract i cal cons i derat i ons rel at i ng t o donor avai l abi l i t y and t o avai l abl e s i t es of peri t oneal or vas cul ar di al ys i s acces s .
B. Hemodialysis Thi s t ype of renal repl acement t herapy i nvol ves extracorporeal circulation of bl ood t hrough a di al ys i s membrane–cont ai ni ng uni t vi a a s urgi cal l y cons t ruct ed vas cul ar fi s t ul a or a t emporary or permanent ext ernal cat het er. Percut aneous punct ure and cannul at i on of t he vas cul ar acces s are requi red at each t reat ment . Bl ood and di al ys at e are s eparat ed by t he s emi permeabl e membrane, whi ch al l ows s ol ut es and wat er t o move from bl ood t o di al ys at e al ong el ect rochemi cal , hydros t at i c, and os mot i c pres s ure gradi ent s . Compl i cat i ons of hemodi al ys i s , wi t h vari ous mani fes t at i ons and et i ol ogi es , can devel op at any s t age of t he procedure (
Tabl e 6-4).
Online TABLE 6-4 Complications of Dialysis—Their Manifestations or Etiologies
Pa g e 1 1 9 9
ABC Ambe r CHM Conve rte r Tria l ve rsion, http://w w w .proce sste x t.com/a bcchm.html
Ma nif est ati Ac on ute s Co or mp Eti lica olo tio gie n s Im Co pro nt a per mi di a nat l yz i on er by pre pre par s er at i vat on i ve (fo rm al i n), ai r bu bbl es , or bac t eri a, em bol
Pa g e 1 2 0 0
ABC Ambe r CHM Conve rte r Tria l ve rsion, http://w w w .proce sste x t.com/a bcchm.html
ism , s ep sis , or me mb ran e rup t ur e Im Exc pro es s per cal wa ci u t er m, t re ma at gn me es i nt um , al u mi nu m, fl u ori de, or cop per ; im pro
Pa g e 1 2 0 1
ABC Ambe r CHM Conve rte r Tria l ve rsion, http://w w w .proce sste x t.com/a bcchm.html
per cl e ans i ng of mu ni ci pal wa t er s up pl i es , whi ch ma y l ea d to chl ora mi ne poi s on i ng an d s ev ere he mo l ys i s
Pa g e 1 2 0 2
ABC Ambe r CHM Conve rte r Tria l ve rsion, http://w w w .proce sste x t.com/a bcchm.html
Eq Po ui p wer me fai l nt ure fai l , ure ai r l ea ks , bl o od l os s fro m lin e s ep ara tio n, hyp oor hyp ert her mi a du e to im pro per ly
Pa g e 1 2 0 3
ABC Ambe r CHM Conve rte r Tria l ve rsion, http://w w w .proce sste x t.com/a bcchm.html
war me d di a l ys at e Al l Urt erg i ca ic
ri a,
rea an ct i ap ons hyl axi s in res po ns e to ma t eri al in t ub i ng or di a l yz er (e. g., ste ri l a nt s )
Pa g e 1 2 0 4
ABC Ambe r CHM Conve rte r Tria l ve rsion, http://w w w .proce sste x t.com/a bcchm.html
Va Bl e s cu edi l ar ng acc fro es s m pro pu bl e nct ms ure sit es , s ut ure lin es , an eur ys ma l di l at i on, en dov as c ul a r i nf ect i on Ant Loc i co al ag acc ul a es s nt bl e
Pa g e 1 2 0 5
ABC Ambe r CHM Conve rte r Tria l ve rsion, http://w w w .proce sste x t.com/a bcchm.html
co edi mp ng, l i ca gas t i o t roi ns nt e sti nal bl e edi ng Tra He ns f mo us i s i d on ero co s i s mp , l i ca he t i o pat ns i t i s Hy Ind pot uce ens d i on by acu te vol um e s hi ft s or t ru e vol um e
Pa g e 1 2 0 6
ABC Ambe r CHM Conve rte r Tria l ve rsion, http://w w w .proce sste x t.com/a bcchm.html
de pl e tio n; ace t at e in di a l ys i s fl ui d, whi ch ma y i nd uce vas odi l at i on; aut on om ic dys fun ct i on s ec on dar y
Pa g e 1 2 0 7
ABC Ambe r CHM Conve rte r Tria l ve rsion, http://w w w .proce sste x t.com/a bcchm.html
to ure mi c ne uro pat hy Car Dur di a i ng c
an
arr d hyt fol l hm owi i as ng t re at me nt ; pre ma t ur e ven t ri c ul a r con t ra ct i ons are co mm on es t
Pa g e 1 2 0 8
ABC Ambe r CHM Conve rte r Tria l ve rsion, http://w w w .proce sste x t.com/a bcchm.html
for m Di a He l ys i ad s
ach
dys e, eq na ui l i us e bri a, um vo mi t i ng , mu s cl e ach es , an d cra mp s
C. Peritoneal dialysis Thi s t ype of renal repl acement t herapy i nvol ves i ns t i l l at i on of 1–3 L s t eri l e di al ys at e i nt o t he peri t oneal cavi t y vi a a s urgi cal l y i mpl ant ed cat het er and drai nage of t he di al ys at e aft er a s peci fi ed dwell period. Frequent , bri ef exchanges (i .e., fort y-ei ght 1-hour exchanges ) may be performed weekl y i n-cent er. Longer exchanges (i .e., four 6-hour exchanges ) al s o are effect i ve and may be done on a cont i nuous ambul at ory bas i s dai l y. Even more frequent exchanges may be performed at night with an automated cycler. Mai nt enance peri t oneal di al ys i s i s as s oci at ed wi t h s peci fi c
Pa g e 1 2 0 9
ABC Ambe r CHM Conve rte r Tria l ve rsion, http://w w w .proce sste x t.com/a bcchm.html
compl i cat i ons .
1. Excessive removal of fluid may res ul t i n hypot ens i on, l i ght -headednes s , weaknes s , or s yncope.
2. Catheter-related complications i ncl ude occl us i on (us ual l y by fi bri nous debri s ), i nfect i on, mal pos i t i on, and, rarel y, fract ure.
3. Dialysate-related complications t hat devel op i f di al ys at e i s t oo rapi dl y i nfus ed or i nadequat el y warmed i ncl ude abdomi nal or back pai n, naus ea, and vomi t i ng.
4. Peritonitis i s t he mos t s eri ous compl i cat i on occurri ng i n chroni c peri t oneal di al ys i s pat i ent s . Infect i on devel ops by i nocul at i on t hrough or around t he cat het er or by cont ami nat i on of di al ys at e. o
o
a. Recurrent peri t oni t i s i s as s oci at ed wi t h hi gh morbi di t y rat es , frequent hos pi t al s t ays , and cons i derabl e expens e. Peri t oneal fi bros i s and l os s of di al ys i s effi ci ency can compl i cat e mul t i pl e recurrent i nfect i ons and may cons t i t ut e cri t eri a for wi t hdrawal from t hi s form of t herapy.
o
o
b. Treat ment of peri t oni t i s i s wi t h parent eral and i nt raperi t oneal ant i bi ot i cs .
o
o
c. Newer cat het er confi gurat i ons (y s et s ) have markedl y reduced t he i nci dence of di al ys i s -i nduced
Pa g e 1 2 1 0
ABC Ambe r CHM Conve rte r Tria l ve rsion, http://w w w .proce sste x t.com/a bcchm.html
peri t oni t i s .
D. Renal allograft transplantation Thi s procedure may be performed from a donor t o a reci pi ent i f t hes e i ndi vi dual s are histocompatible. Hi s t ocompat i bi l i t y i s meas ured by det ermi nat i on of human leukocyte antigen (HLA) t ypes , and blood group t ypes . Donor ki dneys may be from a l i vi ng rel at i ve or from a cadaver wi t h no evi dence of i nfect i ous di s eas e, s peci fi cal l y, bact eremi a, hepat i t i s , acqui red i mmunodefi ci ency s yndrome (AIDS), cyt omegal ovi rus (CMV), s yphi l i s , or mal ari a. Trans pl ant reci pi ent s are gi ven mai nt enance i mmunos uppres s i ve agent s (e.g., prednisone, mycophenolate mofetil, cyclosporine, tacrolimus, or rapamycin) t o prevent graft reject i on. Compl i cat i ons of t rans pl ant at i on ori gi nat e from s everal s ources .
1. Immunosuppressive disorders i ncl ude l eukopeni a (al kyl at i ng agent s ), hepat i t i s and vas o-occl us i ve di s eas e (azat hi opri ne), di arrhea (mycophenol at e mofet i l ), di abet es , obes i t y, cat aract s , and, pos s i bl y, pept i c ul cer di s eas e, avas cul ar necros i s of bone, and pancreat i t i s (predni s one). Nephrot oxi ci t y, t remors , hi rs ut i s m, and hypert ens i on may res ul t from us e of cycl os pori ne and t acrol i mus . Di abet es mel l i t us i s report ed wi t h t acrol i mus . Hyperl i pi demi a i s common wi t h rapamyci n.
2. Secondary hypertension may devel op from ext racel l ul ar fl ui d overl oad (predni s one), hi gh reni n s ecret i on from nat i ve ki dneys , vas cul ar s t enos i s of t he graft from anas t omot i c s t ri ct ure or ext ri ns i c compres s i on by l ymphocel e or uri noma, reject i on, recurrent gl omerul ar di s eas e, uret eral obs t ruct i on, or hypercal cemi a. Coi nci dent pri mary (es s ent i al ) hypert ens i on al s o may devel op.
Pa g e 1 2 1 1
ABC Ambe r CHM Conve rte r Tria l ve rsion, http://w w w .proce sste x t.com/a bcchm.html
3. Infection may occur at any t i me fol l owi ng t rans pl ant at i on by common pat hogens as wel l as by opport uni s t i c organi s ms . o
o
a. Common i nfect i ons i ncl ude uri nary t ract i nfect i on (60% of pat i ent s ), pneumoni a (20% of pat i ent s ), wound or cannul a i nfect i on, hepat i t i s , and s eps i s .
o
o
b. Uncommon i nfect i ons encount ered i n t rans pl ant reci pi ent s i ncl ude CMV-as s oci at ed pneumoni a, hepat i t i s , ret i ni t i s , encephal i t i s , or mononucl eos i s s yndrome; Crypt oc oc c us i nfect i on; Li s t eri a monoc yt ogenes meni ngi t i s (us ual l y occurri ng 6 mont hs pos t t rans pl ant at i on); Pneumoc ys t i s c ari ni i i nfect i on, and Legi onel l a pneumophi l a i nfect i on.
4. Rejection may be hyperacute (i mmedi at e and i nt raoperat i ve), acute (occurri ng 4–60 days fol l owi ng t rans pl ant at i on), or chronic (occurri ng >60 days fol l owi ng t rans pl ant at i on). o
o
a. Acute rejection i s as s oci at ed wi t h fever, decreas ed creat i ni ne cl earance, ol i guri a, s odi um ret ent i on, graft enl argement and t endernes s , hypert ens i on, and prot ei nuri a. Treat ment for acut e reject i on may i ncl ude hi gh-dos e cort i cos t eroi ds , ant i l ymphocyt e gl obul i n, monocl onal ant i bodi es di rect ed agai ns t cyt ot oxi c l ymphocyt es , and, occas i onal l y, t rans pl ant (graft ) nephrect omy.
o
Pa g e 1 2 1 2
ABC Ambe r CHM Conve rte r Tria l ve rsion, http://w w w .proce sste x t.com/a bcchm.html o
b. Chronic rejection, whi ch i s cl i ni cal l y l es s dramat i c, can be s us pect ed on t he bas i s of decreas ed creat i ni ne cl earance, i ncreas ed prot ei nuri a, hyperchl oremi c met abol i c aci dos i s , hypert ens i on, ol i guri a, wei ght gai n, and edema. About 5% of al l ograft s are l os t due t o chroni c reject i on t hat occurs wi t hi n 5 years of t rans pl ant at i on. Chroni c reject i on occurs i n t he majori t y of graft s wi t h t i me, and t here i s no t herapy.
5. Malignancy devel ops i n 2%–7% of t rans pl ant reci pi ent s , a rat e t hat i s 100 t i mes great er t han t he mal i gnancy rat e i n heal t hy, age-mat ched i ndi vi dual s . o
o
a. The majori t y of t umors (i n order of frequency) i nvol ve cancer of t he s ki n and l i ps , l ymphomas [es peci al l y of t he cent ral nervous s ys t em (CNS)], cervi cal carci noma, l ung carci noma, head and neck cancer, and col on carci noma.
o
o
b. The average t i me for mal i gnancy t o devel op i s 40 mont hs but may range from 1–158 mont hs . Lymphomas devel op s ooner (wi t hi n 27 mont hs of t rans pl ant at i on).
V. Proteinuria A. Definition Normal adul t s excret e l es s t han 150 mg of prot ei n i n a 24-hour peri od; small–molecular-weight proteins are t he major component . Uri nary prot ei n excret i on exceedi ng 300 mg/24 hours i s t ermed proteinuria. Al bumi n normal l y i s excret ed i n t he uri ne at a rat e of l es s t han 25 mg/day. Hi gher rat es s ugges t an abnormal i t y i n
Pa g e 1 2 1 3
ABC Ambe r CHM Conve rte r Tria l ve rsion, http://w w w .proce sste x t.com/a bcchm.html
gl omerul ar barri er funct i on, whi ch normal l y precl udes t he al bumi n mol ecul e from cros s i ng t he gl omerul ar bas ement membrane (GBM). Macroalbuminuria refers t o an excret i on rat e of al bumi n great er t han 300 mg/day, a condi t i on det ect abl e wi t h rout i ne s creeni ng met hods . Microalbuminuria refers t o a uri ne al bumi n excret i on rat e t hat exceeds 20 µg/mi nut e but i s l es s t han 200 µg/mi nut e (i .e., 30–300 mg/24 hours ). Smal l –mol ecul ar-wei ght P.251 prot ei ns are excret ed at an i ncreas ed rat e i f proxi mal reabs orpt i ve funct i on i s i mpai red (as i n tubular proteinuria).
B. Etiology
1. Orthostatic proteinuria refers t o an i ncreas e i n uri nary prot ei n t hat i s det ect abl e onl y when t he pat i ent has been s t andi ng. The 24-hour uri nary prot ei n out put t ends t o remai n cons t ant at about 0.5–2.5 g/24 hours , renal funct i on remai ns normal , and t he prognos i s i s excel l ent .
2. T ubulointerstitial nephritis i nvol ves t he excret i on of t ubul ar prot ei ns s uch as T amm-Horsfall protein and β 2 -microglobulin i n addi t i on t o al bumi n. Tubul oi nt ers t i t i al nephri t i s i s t ypi cal l y s een i n pat i ent s wi t h drug-i nduced di s eas e, chroni c i nfl ammat ory di s eas e (e.g., s arcoi dos i s ), or anal ges i c nephropat hy.
3. Glomerulonephritis t ypi cal l y produces al bumi nuri a (>2 g/24 hours ). Nephrot i c s yndrome (hypoal bumi nemi a, edema, and hyperl i pi demi a) occurs when prot ei n excret i on exceeds 3 g/24 hours .
C. Diagnosis
Pa g e 1 2 1 4
ABC Ambe r CHM Conve rte r Tria l ve rsion, http://w w w .proce sste x t.com/a bcchm.html
1. Urinalysis o
o
a. The mos t common s creeni ng t es t for prot ei nuri a i s t he uri ne di ps t i ck. and s ul fos al i cyl i c aci d preci pi t at i on (al bumi n, paraprot ei ns , i mmunogl obul i ns and amyl oi d). If pos i t i ve, charact eri zat i on of prot ei n (i .e., al bumi n vs . i mmunogl obul i n) can be performed wit h uri ne prot ei n el ect rophores i s .
o
o
b. Quant i fi cat i on of prot ei n i s eas i l y accompl i s hed us i ng a s pot uri ne prot ei n-t o-creat i ni ne rat i o. A rat i o >0.3 i s i ndi cat i ve of s i gni fi cant prot ei nuri a.
o
o
c. Li pi duri a i s s ugges t ed by oval fat bodi es on mi cros copi c s t udy.
2. Urine and blood chemistries i n pat i ent s wi t h macroal bumi nuri a s houl d i ncl ude quant i t at i ve prot ei n meas urement and uri ne prot ei n el ect rophores i s . El evat ed bl ood l i pi ds and hypoal bumi nemi a s upport a di agnos i s of nephrot i c s yndrome.
3. Biopsy i s i ndi cat ed i n t he eval uat i on of pat i ent s wi t h s i gni fi cant prot ei nuri a and nephrot i c s yndrome when no obvi ous caus e i s i dent i fi ed by noni nvas i ve means . Pat hol ogi c s t udy s houl d i ncl ude el ect ron mi cros copy, i mmunofl uores cence, and t he us e of s peci al s t ai ns (e.g., Congo red for amyl oi d).
Pa g e 1 2 1 5
ABC Ambe r CHM Conve rte r Tria l ve rsion, http://w w w .proce sste x t.com/a bcchm.html
D. Therapy
1. Orthostatic proteinuria. Treat ment i s not requi red.
2. T ubulointerstitial proteinuria. The underl yi ng di s order mus t be i dent i fi ed and t reat ed.
3. Glomerulonephritic proteinuria. The i mport ance of reduci ng prot ei nuri a cannot be overemphas i zed. It not onl y i ndi cat es underl yi ng gl omerul ar di s eas e, but i t can perpet uat e i t . Several t herapi es have proved us eful i n reduci ng prot ei nuri a and ret ardi ng progres s i on of renal di s eas e. Thes e i ncl ude angi ot ens i n-convert i ng enzyme (ACE) i nhi bi t ors , Angi ot ens i n recept or bi nders (ARBs ), l i pi d-l oweri ng agent s , aggres s i ve bl ood pres s ure cont rol , s moki ng ces s at i on, and di et ary prot ei n res t ri ct i on. In addi t i on, di uret i cs may be us eful for s ympt om rel i ef.
E. Complications The cons equences of hyperl i pi demi a (at heros cl eros i s and coronary art ery di s eas e), vi t ami n D defi ci ency (bone di s eas e), uri nary l os s of cert ai n prot ei ns i mpedi ng s pont aneous coagul at i on (t hrombos i s ), and marked s al t ret ent i on (mas s i ve edema, or anas arca) may res ul t . It has been s ugges t ed t hat pat i ent s wi t h nephrot i c s yndrome are more s us cept i bl e t o bact eri al i nfect i ons , part i cul arl y s pont aneous bact eri al peri t oni t i s s econdary t o pneumococcus i n chi l dren.
VI. Hematuria A. Definition Normal adul t s excret e 500,000–2,000,000 eryt hrocyt es per 24 hours , whi ch amount s t o l es s t han t hree eryt hrocyt es per hi gh-power fi el d (HPF) of res us pended uri nary s edi ment .
Pa g e 1 2 1 6
ABC Ambe r CHM Conve rte r Tria l ve rsion, http://w w w .proce sste x t.com/a bcchm.html
B. Etiology Caus es of hemat uri a are s ummari zed i n Tabl e 6-5. P.252
TABLE 6-5 Causes of Hematuria Cli nic al Eti Fe olo atu gy res Di f Gro fus s s e
or
Gl o mi c me ros rul cop on i c ep he hri t ma is
t uri
(e. a, g., ab SLE nor ,
ma
vas l cul i pro t i s ) t ei nur i a, red bl o
Pa g e 1 2 1 7
ABC Ambe r CHM Conve rte r Tria l ve rsion, http://w w w .proce sste x t.com/a bcchm.html
od cel l cas ts, dys mo rph ic red cel l s by ph as e -co nt r as t mi c ros cop y Foc Gro al
ss
Gl o or me mi c rul ros on cop ep i c hri t he is
ma
(e. t uri g., a IgA wi t ne ho phr ut
Pa g e 1 2 1 8
ABC Ambe r CHM Conve rte r Tria l ve rsion, http://w w w .proce sste x t.com/a bcchm.html
i t i s pro ,
t ei
t hi nur n
i a,
bas dys em mo ent rph me i c mb red ran cel l e
s
di s by eas ph e) as e -co nt r as t mi c ros cop y Va Gro s cu s s l ar or di s mi c eas ros e
cop ic he ma t uri a wi t ho ut
Pa g e 1 2 1 9
ABC Ambe r CHM Conve rte r Tria l ve rsion, http://w w w .proce sste x t.com/a bcchm.html
pro t ei nur i a, iso mo rph ic red cel l s by ph as e -co nt r as t mi c ros cop y Tu Is o mo mo rs
rph
(e. i c g., red hyp cel l ern s ep by hro ph ma as e ,
-co
bl a nt r dd as t er mi c
Pa g e 1 2 2 0
ABC Ambe r CHM Conve rte r Tria l ve rsion, http://w w w .proce sste x t.com/a bcchm.html
can ros cer cop )
y
Tra um a Ki d ney sto nes Sys te mi c coa gul op at h i es IgA, i mmun ogl obu l i n A; SLE, s ys t em ic l upus eryt he mat os us .
C. Diagnosis
1. Urinalysis o
Pa g e 1 2 2 1
ABC Ambe r CHM Conve rte r Tria l ve rsion, http://w w w .proce sste x t.com/a bcchm.html
o
a. Di ps t i ck t es t i ng cannot di fferent i at e hemat uri a from pi gment uri a (i .e., hemogl obi nuri a or myogl obi nuri a). A pos i t i ve orthotolidine test i n t he abs ence of mi cros copi cal l y det ect ed eryt hrocyt es pract i cal l y confi rms t he di agnos i s of pi gment uri a.
o
o
b. Uri ne cul t ure s houl d be performed rout i nel y.
2. Radiography o
o
a. Int ravenous urography can demons t rat e renal mas s es , cys t s , vas cul ar mal format i ons , papi l l ary necros i s , uret eral s t ri ct ure or obs t ruct i on by cal cul us , bl adder t umor, and uret eral devi at i on. CT s canni ng i s t he preferred modal i t y for as s es s ment of renal mas s es or cys t s for mal i gnant charact eri s t i cs .
o
o
b. Speci al s t udi es (e.g., angi ography, nucl ear s canni ng) occas i onal l y are of val ue i n del i neat i ng mas s l es i ons .
3. Biopsy occas i onal l y may as s i s t i n maki ng a di agnos i s of i mmunogl obul i n (Ig)A nephropat hy or i n charact eri zi ng t he l es i on of a pri mary gl omerul ar di s eas e.
4. Cystoscopy i s i ndi cat ed i n t he eval uat i on of hemat uri a when phys i cal exami nat i on, uri nal ys i s , and ot her i magi ng s t udi es fai l t o reveal t he caus e.
D. Therapy
Pa g e 1 2 2 2
ABC Ambe r CHM Conve rte r Tria l ve rsion, http://w w w .proce sste x t.com/a bcchm.html
1. The underl yi ng di s order mus t be i dent i fi ed and t reat ed.
2. Uri ne vol ume s houl d be mai nt ai ned t o prevent cl ot s and obs t ruct i on i n t he l ower uri nary t ract .
E. Complications
1. Iron deficiency anemia rarel y may compl i cat e chroni c, s i gni fi cant hemat uri a.
2. Lower urinary tract clots can i nduce obs t ruct i on.
VII. Nephrolithiasis A. Definition Renal calculi or stones ari s e due t o papi l l ary cal ci fi cat i on or preci pi t at i on i n uri ne of organi zed crys t al l i ne bodi es of cal ci um s al t s , uri c aci d, cys t i ne, or s t ruvi t e. The et i ol ogi es of nephrol i t hi as i s are gi ven i n Tabl e 6-6.
B. Clinical features Si gns and s ympt oms may vary cons i derabl y.
1. Occult passage of s mal l , as ympt omat i c s t ones may occur.
2. Hematuria accompani es s t one movement wi t hi n t he uri nary t ract 85% of t he t i me and may be mi cros copi c or gros s . Hemat uri a may occur wi t h or wi t hout pai n.
Pa g e 1 2 2 3
ABC Ambe r CHM Conve rte r Tria l ve rsion, http://w w w .proce sste x t.com/a bcchm.html
P.253
TABLE 6-6 Etiology of Nephrolithiasis Eti olo gy or As soc iat ed Sto Co ne ndi T y tio pe n Cal Hy ci u per m
par
ph at h os p yro hat i di e
sm , di s t al ren al t ub ul a r aci dos
Pa g e 1 2 2 4
ABC Ambe r CHM Conve rte r Tria l ve rsion, http://w w w .proce sste x t.com/a bcchm.html
is, i di op at h ic hyp erc al ci uri a, an d me dul l ar y s po ng e ki d ney Cal Idi ci u op m
at h
oxa i c l at hyp e
erc al ci uri a, exc es s di e t oxa
Pa g e 1 2 2 5
ABC Ambe r CHM Conve rte r Tria l ve rsion, http://w w w .proce sste x t.com/a bcchm.html
l at e, vi t am in C ab us e , sm al l bo wel di s eas es , pri ma ry hyp ero xal uri a, an d hyp erc al c em i a; 50 % of pat
Pa g e 1 2 2 6
ABC Ambe r CHM Conve rte r Tria l ve rsion, http://w w w .proce sste x t.com/a bcchm.html
i en ts hav e no i de nt i f i ab le ab nor ma lity Uri Per c
sis
aci t en d
tly con cen t ra t ed an d aci d uri ne, hyp eru ri c os u ri a, hyp eru ri c
Pa g e 1 2 2 7
ABC Ambe r CHM Conve rte r Tria l ve rsion, http://w w w .proce sste x t.com/a bcchm.html
em ia (i n go ut ) , an d exc es s di e t ar y pur i ne Cys Cys tin tin e
uri
a St r Uri uvi nar te y (t ri t ra pl e ct ph i nf os p ect hat i on e,
(ch
or ron ma i c gn or es i rec um urr – ent am )
Pa g e 1 2 2 8
ABC Ambe r CHM Conve rte r Tria l ve rsion, http://w w w .proce sste x t.com/a bcchm.html
mo by ni u ure mâ as e €“c -pr al ci od um uci ph ng os p bac hat t eri e) a s uc h as Pro t eu s, Pro vi d enc i a, Kl e bs i el l a, Ps e ud om on as , Ser rat i a, an d Ent
Pa g e 1 2 2 9
ABC Ambe r CHM Conve rte r Tria l ve rsion, http://w w w .proce sste x t.com/a bcchm.html
ero bac t er s pe ci e s
3. Frequency and dysuria are common compl ai nt s of pat i ent s wi t h s t ones l odged i n t he i nt raves i cal s egment of t he di s t al uret er and may be mi s t aken for t he s ympt oms of cystitis. Dys uri a al s o occurs duri ng t he pas s age of sludge.
4. Abdominal pain, tenesmus, and rectal pain may occur wi t h a s t one i n t he renal pel vi s and oft en are accompani ed by naus ea and vomi t i ng.
5. Renal colic, wi t h fl ank pai n radi at i ng t o t he i ngui nal l i gament , uret hra, l abi a, t es t i s , or peni s , i s t ypi cal of a s t one i n t he mi duret er.
C. Diagnosis
1. Patient history s houl d i dent i fy ot her fami l y members wi t h s t one di s eas e as wel l as t he pat i ent 's pas t and pres ent us e of drugs and vi t ami ns (part i cul arl y vi t ami ns A, C, and D).
Pa g e 1 2 3 0
ABC Ambe r CHM Conve rte r Tria l ve rsion, http://w w w .proce sste x t.com/a bcchm.html
2. Physical examination i s neces s ary t o di fferent i at e acut e renal col i c from ot her caus es of abdomi nal , pel vi c, and back pai n.
3. Urinalysis i s oft en us eful . o
o
a. Uri ne pH i s i nappropri at el y hi gh i n renal t ubul ar aci dos i s , favori ng cal ci um phos phat e s t one format i on. Low uri ne vol ume wi t h l ow uri ne pH i s a ri s k fact or for uri c aci d s t ones .
o
o
b. Crys t al s may be s een wi t h charact eri s t i c morphol ogi es , whi ch i ndi cat e t he compos i t i on of s t ones .
4. Urine and blood chemistries are cri t i cal t o t he met abol i c eval uat i on of t he pat i ent wi t h nephrol i t hi as i s . o
o
a. A bl ood s ampl e s houl d be exami ned for l evel s of el ect rol yt es , creat i ni ne, BUN, cal ci um, phos phat e, and uri c aci d.
o
o
b. A 24-hour uri ne col l ect i on s houl d be s t udi ed for uri ne vol ume and pH as wel l as l evel s of cal ci um, phos phat e, uri c aci d, oxal at e, creat i ni ne, s odi um, ci t rat e, and cys t i ne i n pat i ent s wi t h recurrent s t one di s eas e.
5. Radiography oft en confi rms t he di agnos i s .
Pa g e 1 2 3 1
ABC Ambe r CHM Conve rte r Tria l ve rsion, http://w w w .proce sste x t.com/a bcchm.html
Ul t ras onography (Fi gure 6-3) and non–cont ras t -enhanced CT have become t he modal i t i es of choi ce for t he di agnos i s of nephrol i t hi as i s .
6. Cystoscopy i s i ndi cat ed for t he det ect i on and removal of bl adder cal cul i and for t he removal of uret eral s t ones l odged near t he uret eroves i cal junct i on.
7. Stone analysis i s t he defi ni t i ve t ool for as cert ai ni ng t he s t at us of t he s t one (pas s ed or ret ai ned) and i t s compos i t i on. Effort s s houl d be made t o s t rai n uri ne and capt ure s t ones for chemi cal anal ys i s .
D. Therapy
1. Acute episodes of nephrol i t hi as i s s houl d be t reat ed wi t h ri gorous vol ume admi ni s t rat i on and pai n cont rol wi t h NSAIDs or narcot i cs .
2. Medi cal t herapy vari es dependi ng on s t one compos i t i on. In al l cas es , adequat e dai l y fl ui d i nt ake t o mai nt ai n a uri ne vol ume of more t han 2 L/day i s es s ent i al . For calcium oxalate stones (t he mos t common t ype), P.254
t herapy i ncl udes di et ary res t ri ct i on of oxal at e-ri ch food, el i mi nat i on of l arge dos es (i .e., >500 mg/day) of as corbi c aci d, and admi ni s t rat i on of hypocal ci uri c di uret i cs (t hi azi des or ami l ori de) or oral neut ral pot as s i um phos phat e. Oral admi ni s t rat i on of pot as s i um ci t rat e may be us eful i n i ncreas i ng t he uri nary excret i on of ci t rat e, a major uri nary
Pa g e 1 2 3 2
ABC Ambe r CHM Conve rte r Tria l ve rsion, http://w w w .proce sste x t.com/a bcchm.html
chel at or of i oni zed cal ci um and an i nhi bi t or of cal ci um oxal at e crys t al growt h. Cal ci um phos phat e, uri c aci d, and s t ruvi t e s t ones are covered onl i ne.
FIGURE 6-3 Ul t ras ound i mage demons t rat i ng a renal cal cul us (arrow). Not i ce t he s hadowi ng bel ow t he s t one. (From Daffner RH. Cl i ni cal Radi ol ogy: The Es s ent i al s . 3rd ed. Bal t i more: Li ppi ncot t W i l l i ams & W i l ki ns , 2007:9-28. )
3. Ext racorporeal or s urgi cal removal may be requi red i f s t one s i ze exceeds 5 mm. o
o
a. Calcium phosphate stones. Pri mary hyperparat hyroi di s m neces s i t at es prompt t reat ment wi t h parat hyroi dect omy, di s t al t ubul ar aci dos i s requi res i ndependent eval uat i on, and i di opat hi c hypercal ci uri a requi res di uret i cs (t hi azi des or ami l ori de) or oral neut ral pot as s i um phos phat e.
o
Pa g e 1 2 3 3
ABC Ambe r CHM Conve rte r Tria l ve rsion, http://w w w .proce sste x t.com/a bcchm.html
o
b. Uric acid stones. Therapy i ncl udes admi ni s t rat i on of oral s odi um bi carbonat e t o mai nt ai n an al kal i ne uri ne (i .e., pH >6) and, i n s el ect ed pat i ent s , res t ri ct i on of di et ary puri ne or admi ni s t rat i on of al l opuri nol .
o
o
c. Cystine stones. Sodi um bi carbonat e i s admi ni s t ered t o keep uri ne pH above 7.5, and acet azol ami de i s gi ven at bedt i me t o mai nt ai n uri ne al kal i ni t y duri ng t he ni ght . Uri ne out put s houl d be mai nt ai ned at more t han 4 L/day. Noncompl i ant pat i ent s and t hos e wi t h s evere or refract ory s t one di s eas e may be candi dat es for oral D-peni ci l l ami ne or i nt rarenal s t one di s s ol ut i on by al kal i ne or acet yl cys t ei ne i rri gat i on.
o
o
d. Struvite stones. Treat ment i s ai med at mai nt ai ni ng uri nary as eps i s , whi ch may requi re ant i bi ot i cs .
VIII. Urinary Tract Obstruction A. Introduction An obs t ruct i on i n t he uri nary t ract may occur at any poi nt bet ween t he renal t ubul es and t he uret hra. Uri nary obs t ruct i on may be acut e or chroni c, uni l at eral or bi l at eral , and part i al or compl et e. Chroni c uri nary obs t ruct i on oft en i s part i al and may be as ympt omat i c, part i cul arl y i n s l owl y progres s i ve cas es . The cons equences of uri nary obs t ruct i on i ncl ude s t ruct ural changes i n t he l ower uri nary t ract as a res ul t of i ncreas es i n pres s ure oppos i ng normal uri ne fl ow (obstructive uropathy), gros s di l at i on of t he cal yces and col l ect i ng s ys t em of t he affect ed ki dney (hydronephrosis), and, ul t i mat el y, renal parenchymal damage (obstructive nephropathy).
B. Etiology
Pa g e 1 2 3 4
ABC Ambe r CHM Conve rte r Tria l ve rsion, http://w w w .proce sste x t.com/a bcchm.html
The caus es of uri nary obs t ruct i on can be di vi ded i nt o mechanical caus es , whi ch may be i nt ri ns i c or ext ri ns i c, and functional caus es .
1. Intrinsic mechanical causes o
o
a. Intrarenal tubular obstruction res ul t s from preci pi t at i on of uri c aci d, s ul fonami de, or paraprot ei n crys t al s . Drugs s uch as i ndi navi r, and acycl ovi r may l ead t o i nt rat ubul ar preci pi t at i on of crys t al s .
o
o
b. Extrarenal pelvic or ureteral obstruction i s caus ed by cal cul us , t hrombus , papi l l ary necros i s , or t umor.
o
o
c. Structural lesions of the ureter or bladder i ncl ude s t ri ct ure, t umor, uret hral val ves , uret erocel es , and forei gn body.
P.255
2. Extrinsic mechanical causes o
o
a. Compression may be caus ed by:
(1) Pros t at i c hypert rophy or carci noma
(2) Ut eri ne prol aps e or t umor
Pa g e 1 2 3 5
ABC Ambe r CHM Conve rte r Tria l ve rsion, http://w w w .proce sste x t.com/a bcchm.html
(3) Ovari an abs ces s , cys t , or t umor
(4) Endomet ri os i s
(5) Pregnancy
(6) Ret roperi t oneal t umor, i nfect i on, l ymphadenopat hy, or fi bros i s
o
o
b. Surgical errors i ncl ude acci dent al uret eral l i gat i on.
3. Functional causes. Uret eral or bl adder dys funct i on res ul t s from myel odys pl as i a; i njury or congeni t al defect of t he s pi nal cord; t abes dors al i s ; di abet es mel l i t us ; mul t i pl e s cl eros i s (MS), and aut onomi c neuropat hy, i ncl udi ng drug-i nduced neuropat hy (e.g., due t o di s opyrami de).
C. Clinical features Si gns and s ympt oms vary dependi ng on t he s i t e of t he obs t ruct i on and t he s peed wi t h whi ch t he obs t ruct i on devel ops .
1. An absence of symptoms oft en occurs i n chroni c, s l owl y advanci ng obs t ruct i ve di s eas e. The cl i ni cal pi ct ure oft en i s overs hadowed by s i gns of t he pri mary di s eas e (e.g., i n a cas e of met as t at i c t umor or s urgi cal compl i cat i ons ) unt i l bi ochemi cal evi dence of renal i mpai rment devel ops .
Pa g e 1 2 3 6
ABC Ambe r CHM Conve rte r Tria l ve rsion, http://w w w .proce sste x t.com/a bcchm.html
2. Pain and renal enlargement (abdomi nal or fl ank mas s ) us ual l y are pres ent i n acut e obs t ruct i on. The pai n charact eri s t i cal l y i s a s t eady cres cendo, i s mos t s evere i n t he fl ank, and radi at es t oward t he i ps i l at eral t es t i s or l abi um.
3. Urinary symptoms predomi nat e i n obs t ruct i ve di s eas e of t he bl adder or uret hra. Hes i t ancy, decreas ed force of uri nary s t ream, uri nary frequency, and dri bbl i ng are common i n t he cont ext of obs t ruct i on.
4. Renal functional impairment t ypi cal l y i s expres s ed as t ubul ar defect s i n aci d and pot as s i um t rans port as wel l as defect i ve t ubul ar res pons i venes s t o hormone act i on. Cl i ni cal l y, hyperkal emi a, mi l d aci demi a, and pol yuri a precede azot emi a, whi ch may progres s t o renal fai l ure.
D. Diagnosis
1. Urinalysis vari es but may reveal i nappropri at el y di l ut e uri ne, hemat uri a (i n cas es of obs t ruct i on due t o cal cul us or t umor), or bact eri uri a. Becaus e i nfect i on oft en compl i cat es obs t ruct i on, caus i ng s eri ous det ri ment t o renal funct i on, uri ne cul t ure i s es s ent i al . Exami nat i on of t he uri nary s edi ment oft en s hows no abnormal i t y but may reveal crys t al s of uri c aci d or s ul fonami de.
2. Blood chemistries us ual l y are not di agnos t i c but are hel pful i n as s es s i ng t he s everi t y of i mpai red renal funct i on.
Pa g e 1 2 3 7
ABC Ambe r CHM Conve rte r Tria l ve rsion, http://w w w .proce sste x t.com/a bcchm.html
3. Radiography provi des t he cl i ni cal es s ent i al evi dence of obs t ruct i on. Ul t ras onography or CT rel i abl y det ect s evi dence of hydronephros i s s uch as cal yceal bl unt i ng and di l at i on of t he renal pel vi s , uret er, or bot h
(Onl i ne Fi gure 6-4). Doppl er
ul t ras onography may be effect i ve i n demons t rat i ng t he rat e of uri ne fl ow from each uret er. Int ravenous urography may fai l t o vi s ual i ze t he ki dneys i f t he GFR i s decreas ed s ubs t ant i al l y. Ret rograde urography occas i onal l y may hel p i dent i fy uni l at eral (part i cul arl y part i al ) uret eral obs t ruct i on. Nucl ear s canni ng of t he ki dney oft en i s s peci fi c enough t o confi rm t he di agnos i s .
FIGURE 6-4 Obs t ruct i ve uropat hy i n t wo pat i ent s as demons t rat ed by CT. A. CT i mage s hows di l at i on of t he uret er (U) as wel l as t he cal yces (arrows ). B. Image more di s t al l y s hows t he cal ci fi ed s t one i n t he uret er (arrow). C. CT i mage i n anot her pat i ent s hows mas s i ve cal yceal di l at i on (C). D. Image more di s t al s hows t he l arge obs t ruct i ng s t one i n t he uret er (arrow). The l ower pol e of t he ki dney i s enl arged. (From Daffner RH. Cl i ni cal Radi ol ogy: The Es s ent i al s . 3rd ed. Bal t i more: Li ppi ncot t W i l l i ams & W i l ki ns , 2007:9-13 )
E. Therapy
1. Relief of obstruction i s paramount and s houl d be appropri at e t o t he s t ruct ural nat ure of t he occl udi ng l es i on. Met hods i ncl ude pl acement of a t rans uret hral
Pa g e 1 2 3 8
ABC Ambe r CHM Conve rte r Tria l ve rsion, http://w w w .proce sste x t.com/a bcchm.html
(Fol ey) cat het er, s urgery, percut aneous nephros t omy, uret eral s t ent , and nephros copi c s t one removal .
2. Medical management fol l owi ng rel i ef of obs t ruct i on i s ai med at correct i ng pos t obs t ruct i ve di ures i s . Cont ri but i ng fact ors i ncl ude t he excret i on of s ol ut e (urea) t hat was ret ai ned duri ng t he peri od of obs t ruct i on and t he i mpai red concent rat i ng abi l i t y t hat us ual l y exi s t s i n t he recent l y obs t ruct ed ki dney. Management i nvol ves careful , adequat e fl ui d repl acement wi t h frequent as s es s ment of body wei ght , i nt ravas cul ar vol ume, and bl ood and uri ne el ect rol yt e concent rat i on.
P.256
F. Complications
1. Infection, part i cul arl y i n t he cont ext of obs t ruct i ng cal cul i , mus t be det ect ed and prompt l y t reat ed t o prevent ext ens i ve pyel onephri t i s , peri renal abs ces s , and s eps i s .
2. Hypertension occurs s econdary t o bot h i nt ravas cul ar vol ume expans i on and i s chemi c s t i mul at i on of reni n s ecret i on.
3. Polycythemia has been report ed i n as s oci at i on wi t h hydronephros i s and i s purport edl y due t o i ncreas ed eryt hropoi et i n rel eas e.
4. Persistent tubular defects may cont i nue beyond 1
Pa g e 1 2 3 9
ABC Ambe r CHM Conve rte r Tria l ve rsion, http://w w w .proce sste x t.com/a bcchm.html
year fol l owi ng rel i ef of obs t ruct i on. Impai red concent rat i ng abi l i t y and l i mi t ed excret i on of a pot as s i um l oad are t he mos t common defect s .
5. Chronic renal failure can devel op from obs t ruct i ve di s eas e, mos t commonl y wi t h l ongs t andi ng obs t ruct i on or wi t h compl i cat i ng uri nary t ract i nfect i on.
IX. Urinary Tract Infection See Chapt er 8 V F for coverage of uri nary t ract i nfect i ons .
X. Glomerular Disease A. Hereditary nephritis (Alport's syndrome)
1. Inheritance and incidence. Heredi t ary nephri t i s i s i nheri t ed as an X-l i nked or aut os omal domi nant t rai t wi t h vari abl e penet rance. Recent s t udi es have s hown t hat t he di s eas e i s caus ed by a defect i n one of t he genes encodi ng t he s ubuni t s of t ype IV col l agen, a bas ement membrane prot ei n.
2. Clinical features o
o
a. Hematuria, RBC casts, pyuria, proteinuria, and progressive renal failure occur wi t h vari abl e s everi t y. Renal fai l ure i s more common i n men.
o
o
b. High-frequency sensorineural hearing loss, oft en wi t hout cl i ni cal l y s i gni fi cant deafnes s , i s charact eri s t i c.
Pa g e 1 2 4 0
ABC Ambe r CHM Conve rte r Tria l ve rsion, http://w w w .proce sste x t.com/a bcchm.html
3. T herapy and prognosis. No t reat ment i s s ucces s ful i n s l owi ng or prevent i ng t he renal fai l ure, and prognos i s i s vari abl e. Occas i onal l y, affect ed pat i ent s who undergo renal t rans pl ant at i on devel op ant i gl omerul ar bas ement membrane (ant i -GBM) ant i body di s eas e (Goodpas t ure's s yndrome) i f t he Al port hos t recogni zes t he normal t ype IV col l agen of t he t rans pl ant ed ki dney as a “forei gn― ant i gen.
B. Minimal change disease
1. Incidence. Mi ni mal change di s eas e account s for t hree-fourt hs of cas es of i di opat hi c nephrot i c s yndrome i n chi l dren but onl y one-fourt h of cas es i n adul t s .
2. Pathology. Informat i on i s s cant and nons peci fi c. Fus i on of epi t hel i al foot proces s es i s s een wi t h el ect ron mi cros copy, but t hi s l es i on i s common t o al l prot ei nuri c s t at es .
3. Clinical features o
o
a. Nephrotic syndrome i s t he t ypi cal pres ent at i on by pat i ent s of al l ages .
o
o
b. Hypert ens i on occurs i n 10% of chi l dren and i n 35% of adul t s .
o
o
c. Hemat uri a i s uncommon.
o
Pa g e 1 2 4 1
ABC Ambe r CHM Conve rte r Tria l ve rsion, http://w w w .proce sste x t.com/a bcchm.html
o
d. Azot emi a devel ops i n 23% of chi l dren and i n 34% of adul t s .
4. T herapy o
o
a. Glucocorticoids. The remi s s i on rat e wi t h adequat e s t eroi d t reat ment i s 90% for bot h chi l dren and adul t s . Prol onged remi s s i on i s s een i n 10%–60% of pat i ent s ; however, rel aps e i s common and i s mul t i pl e i n 25%–50% of pat i ent s . Rel aps es t ypi cal l y are res pons i ve t o s t eroi ds , wi t h onl y 5% of i ni t i al l y s t eroi d-res pons i ve pat i ent s devel opi ng s t eroi d res i s t ance or dependence. Recent s t udi es have demons t rat ed t hat adul t s may have a poorer res pons e t o gl ucocort i coi ds t han previ ous l y bel i eved.
o
o
b. Cytotoxic agents. Drugs s uch as cycl ophos phami de and chl orambuci l have been effect i ve i n s t eroi d-res i s t ant and mul t i pl e rel aps i ng cas es . Occas i onal l y, s t eroi d-res i s t ant pat i ent s devel op s t eroi d s ens i t i vi t y fol l owi ng t reat ment wi t h cyt ot oxi c al kyl at i ng agent s . The pos s i bi l i t y of gonadal (chromos omal ) damage caus ed by t hes e drugs mus t be careful l y cons i dered. P.257
o
o
c. Other immunosuppressive agents. Current res earch demons t rat es t hat t he us e of agent s s uch as cycl os pori ne and mycophenol at e mofet i l i s
Pa g e 1 2 4 2
ABC Ambe r CHM Conve rte r Tria l ve rsion, http://w w w .proce sste x t.com/a bcchm.html
effect i ve i n refract ory cas es . o
o
d. ACE i nhi bi t ors and ARBs are al s o neces s ary for t he reduct i on of prot ei nuri a.
5. Prognosis. Mi ni mal change di s eas e i s as s oci at ed wi t h l ow mort al i t y rat es (i .e., 10% among adul t s and 1.5% among chi l dren), wi t h onl y 10% of deat hs caus ed by renal fai l ure.
C. Membranous glomerulonephritis (or glomerulopathy)
1. Etiology o
o
a. Primary (idiopathic) membranous glomerulonephritis account s for 30%–50% of cas es of i di opat hi c nephrot i c s yndrome i n adul t s but l es s t han 1% of cas es i n chi l dren.
o
o
b. Secondary membranous glomerulonephritis may occur wi t h a vari et y of underl yi ng condi t i ons , i ncl udi ng:
(1) Infection [e.g., chroni c hepat i t i s B vi rus (HBV) i nfect i on or hepat i t i s C vi rus (HCV) i nfect i on, s yphi l i s , mal ari a, s chi s t os omi as i s , and fi l ari as i s ]
Pa g e 1 2 4 3
ABC Ambe r CHM Conve rte r Tria l ve rsion, http://w w w .proce sste x t.com/a bcchm.html
(2) Rheumatic disease (e.g., SLE)
(3) Neoplasm (e.g., carci noma of t he l ung, col on, s t omach, breas t , and ki dney; non-Hodgki n's l ymphoma; l eukemi a; W i l ms ' t umor)
(4) Drug therapy (e.g., mercury, gol d, D-peni ci l l ami ne, capt opri l )
2. Pathology. The fi ndi ngs pres ent ed i n
Onl i ne Tabl e 6-7
charact eri ze t he s t ages of membranous gl omerul onephri t i s . Charact eri s t i c el ect ron mi cros cope fi ndi ngs can be s een i n Fi gure 6-5.
Online TABLE 6-7 Stages of Membranous Glomerulonephritis Ch ar act eri Sta sti ge cs I Nor ma l ap pe ara nce wi t h
Pa g e 1 2 4 4
ABC Ambe r CHM Conve rte r Tria l ve rsion, http://w w w .proce sste x t.com/a bcchm.html
lig ht mi c ros cop y; s ub epi t he l i al el e ct r onde ns e de pos its wi t h el e ct r on mi c ros cop II
y Spi kelik e pro jec tio ns
Pa g e 1 2 4 5
ABC Ambe r CHM Conve rte r Tria l ve rsion, http://w w w .proce sste x t.com/a bcchm.html
of bas em ent me mb ran e ma t eri al wi t h lig ht mi c ros cop y (be st vi s ual ize d wi t h a silv er im pre gn at i on sta
Pa g e 1 2 4 6
ABC Ambe r CHM Conve rte r Tria l ve rsion, http://w w w .proce sste x t.com/a bcchm.html
i n) ; var i ab le bas em ent me mb ran e t hi cke ni n g III Thi ck gl o me rul ar bas em ent me mb ran e (G BM ), wi t h a â€
Pa g e 1 2 4 7
ABC Ambe r CHM Conve rte r Tria l ve rsion, http://w w w .proce sste x t.com/a bcchm.html
œ mo t heat en †• or †œS wi s s -c he es e †• ap pe ara nce du e to enc i rcl em ent of im mu ne de pos its by
Pa g e 1 2 4 8
ABC Ambe r CHM Conve rte r Tria l ve rsion, http://w w w .proce sste x t.com/a bcchm.html
s pi kelik e pro jec tio ns of nor ma l GB M IV Thi cke ni n g of t he cap illa ry wal l, wi t h are as of s eg me nt a l or gl o
Pa g e 1 2 4 9
ABC Ambe r CHM Conve rte r Tria l ve rsion, http://w w w .proce sste x t.com/a bcchm.html
bal gl o me rul os c l er os i s; pos sib le t ub ul o i nt ers titi al fi br os i s
3. Clinical features. More t han 85% of adul t s pres ent 2
wi t h prot ei nuri a (>3 g/day/1.73 m body s urface area). The GFR us ual l y i s normal at di agnos i s and oft en remai ns normal for 4–5 years t hereaft er.
4. Clinical course. Membranous gl omerul onephri t i s has a vari abl e cours e. About 20%–30% of pat i ent s achi eve a l as t i ng s pont aneous remi s s i on, 20%–30% devel op vari abl e degrees of pers i s t ent prot ei nuri a and nonuremi c az ot emi a, and t he res t advance t o end-s t age renal di s eas e, us ual l y over a 5-year peri od. Mal e pat i ent s , t hos e wi t h heavy prot ei nuri a (>10 g/day), and t hos e who do not res pond wi t h a remi s s i on i n prot ei nuri a have
Pa g e 1 2 5 0
ABC Ambe r CHM Conve rte r Tria l ve rsion, http://w w w .proce sste x t.com/a bcchm.html
a wors e prognos i s .
5. T herapy. A combi nat i on of s t eroi ds and cyt ot oxi c agent s i n addi t i on t o ACE i nhi bi t or and ARB t herapy has proved effect i ve, but t he unpredi ct abl e out come of t he di s eas e makes t he eval uat i on of t herapy di ffi cul t . Recent randomi zed t ri al s fi nd t hat s t eroi ds al one s houl d not be us ed as pri mary t herapy. The us e of al t ernat i ng cycl es of chl orambuci l or cycl ophos phami de and predni s one i s effect i ve i n hal t i ng progres s i on t o end-s t age renal fai l ure i n pat i ent s wi t h nephrot i c s yndrome and mi l d renal i ns uffi ci ency. Pat i ent s t reat ed wi t h cyt ot oxi c drugs are more l i kel y t o experi ence remi s s i on of prot ei nuri a.
6. Complications. Probl emat i c condi t i ons may i nt ervene, caus i ng an abrupt decreas e i n renal funct i on. o
o
a. A hypercoagulable state exi s t s i n nephrot i c pat i ent s . Renal vei n t hrombos i s i s a recogni zed probl em; pul monary embol i s m and art eri al t hrombos i s al s o have been des cri bed.
o
o
b. Intravascular volume depletion s econdary t o vi gorous di uret i c admi ni s t rat i on l eads t o decreas ed renal bl ood fl ow.
o
o
c. Hypertension, obstruction, or infection may i mpai r fi l t rat i on.
Pa g e 1 2 5 1
ABC Ambe r CHM Conve rte r Tria l ve rsion, http://w w w .proce sste x t.com/a bcchm.html
FIGURE 6-5 El ect ron mi cros copi c fi ndi ngs of s ubepi t hel i al dens e depos i t s i n bas ement membrane i n a pat i ent wi t h membranous nephropat hy (From W i l l i ams G, Mal l i ck NP. Col or At l as of Renal Di s eas es . 2nd ed. Barcel ona: W ol fe Publ i s hi ng/Mos by-Y ear Book Europe Lt d, 1994:3.10. ) P.258
D. Mesangial proliferative glomerulonephritis
1. Etiology. The caus e i s unknown.
2. Pathology. The mes angi al prol i ferat i ve l es i on i s a gl obal and di ffus e i ncreas e i n t he number of mes angi al cel l s and i n t he mes angi al mat ri x.
3. Clinical features. As ympt omat i c prot ei nuri a or hemat uri a may occur. Al t hough 24-hour uri nary prot ei n excret i on can exceed 3.0 g, t he compl et e nephrot i c s yndrome i s i ncons i s t ent l y s een. One-t hi rd of pat i ent s are hypert ens i ve at t he t i me of di agnos i s . Creat i ni ne
Pa g e 1 2 5 2
ABC Amber CHM Converter Trial version, http://www.processtext.com/abcchm.html
cl earance i s reduced i n onl y 25% of pat i ent s at pres ent at i on. Serum compl ement component l evel s are normal .
4. Clinical course. The di s eas e has an ext remel y
vari abl e cours e.
5. T herapy. Hi gh-dos e (1–2 mg/kg/day)
gl ucocort i coi ds are report edl y effect i ve i n remi s s i on i nduct i on. Some s t eroi d fai l ures res pond t o t reat ment wi t h cycl ophos phami de or chl orambuci l .
E. Membranoproliferative glomerulonephritis
1. Incidence and etiology
o
o
a. Col l ect i vel y, t he di s orders i n t hi s group account for 41% of cas es of i di opat hi c nephrot i c s yndrome i n chi l dren and 30% of cas es i n adul t s . Mal es and femal es are affect ed equal l y.
o
o
b. Membranoprol i ferat i ve gl omerul onephri t i s may be i di opat hi c or s econdary t o SLE, cryogl obul i nemi a, or chroni c vi ral or bact eri al i nfect i on (e.g., hepat i t i s C, s yphi l i s , t ubercul os i s ).
2. Pathology. The t hree pat hol ogi c t ypes of membranoprol i ferat i ve gl omerul onephri t i s , each wi t h di s t i nct feat ures , are des cri bed i n
ONLINE Tabl e 6-8.
Page 1253
ABC Ambe r CHM Conve rte r Tria l ve rsion, http://w w w .proce sste x t.com/a bcchm.html
Online TABLE 6-8 Pathologic Types of Membranoproliferative Glomerulonephritis Fe T y atu pe res I Int act gl o me rul ar bas em ent me mb ran e, s ub en dot hel i al an d me s an gi a l de pos its,
Pa g e 1 2 5 4
ABC Ambe r CHM Conve rte r Tria l ve rsion, http://w w w .proce sste x t.com/a bcchm.html
sig ni fi can t me s an gi a l pro mi ne nce wi t h ma t ri x i nt erp os i tio n Im mu nof l uo res cen t pos itiv ity for im mu no
Pa g e 1 2 5 5
ABC Ambe r CHM Conve rte r Tria l ve rsion, http://w w w .proce sste x t.com/a bcchm.html
gl o bul in G (Ig G), co mp le me nt co mp ou nds (C1 q, C4, an d C2) , an d pro per II
di n Int
(al ra s o me cal l mb ed ran de ous ns e an de d
Pa g e 1 2 5 6
ABC Ambe r CHM Conve rte r Tria l ve rsion, http://w w w .proce sste x t.com/a bcchm.html
pos s ub it
epi
di s t he eas l i al e) de pos its (†œh um ps â €• ) in 50 % of cas es , me s an gi a l de pos its, mo der at e me s an gi a l pro mi ne
Pa g e 1 2 5 7
ABC Ambe r CHM Conve rte r Tria l ve rsion, http://w w w .proce sste x t.com/a bcchm.html
nce wi t h i nt erp os i tio n Im mu nof l uo res cen t pos itiv ity for IgG , C3, an d pro per di n III Fea t ur es of t ru e me mb
Pa g e 1 2 5 8
ABC Ambe r CHM Conve rte r Tria l ve rsion, http://w w w .proce sste x t.com/a bcchm.html
ran ous gl o me rul on ep hri t is (i .e ., wi t h s ub epi t he l i al de pos its) an d fea t ur es of t yp e I me mb ran opr ol i f era tiv
Pa g e 1 2 5 9
ABC Ambe r CHM Conve rte r Tria l ve rsion, http://w w w .proce sste x t.com/a bcchm.html
e gl o me rul on ep hri t is
3. Clinical features. The pres ent at i on i s hi ghl y vari abl e. Hypocompl ement emi a i s t he mos t charact eri s t i c l aborat ory fi ndi ng but i s not uni vers al l y pres ent . The degree of compl ement C3 depres s i on may be us ed as a rough gui de t o di s eas e act i vi t y.
4. Clinical course. Chroni c gl omerul onephri t i s wi t h end-s t age renal di s eas e devel ops i n mos t cas es . Rapi d progres s i on t o renal fai l ure wi t h edema and s evere hypert ens i on (acut e nephri t i s ) has been des cri bed.
5. T herapy. Treat ment i s not predi ct abl y effect i ve, al t hough occas i onal report s cl ai m s ome t herapeut i c benefi t . In chi l dren wi t h prot ei nuri a or i mpai red renal funct i on, hi gh-dos e s t eroi d t herapy s houl d be t ri ed and mai nt ai ned for 6–12 mont hs . In adul t s wi t h heavy prot ei nuri a (>3 g/day), t he antiplatelet agents di pyri damol e and as pi ri n may be us ed. However, evi dence does not defi ni t i vel y s upport t he effect i venes s of t hi s regi men.
F. Focal glomerulosclerosis
Pa g e 1 2 6 0
ABC Ambe r CHM Conve rte r Tria l ve rsion, http://w w w .proce sste x t.com/a bcchm.html
1. Incidence and etiology o
o
a. Focal gl omerul os cl eros i s account s for 30% of cas es of i di opat hi c nephrot i c s yndrome i n adul t s and i s t he mos t common caus e of s t eroi d-res i s t ant nephrot i c s yndrome i n chi l dren.
o
o
b. Thi s condi t i on i s report ed t o recur i n 30%–40% of renal al l ograft s wi t hi n 3 weeks t o 1 year fol l owi ng t rans pl ant at i on.
o
o
c. The et i ol ogy i s unknown. Focal gl omerul os cl eros i s i s s een occas i onal l y i n t he cont ext of AIDS, heroi n and ot her i nt ravas cul ar drug us e, and chroni c ves i couret eral refl ux, but t he caus al rel at i ons hi ps are uncert ai n.
2. Pathology. The hal l mark l es i on of focal and s egment al gl omerul os cl eros i s evol ves t hrough s everal s t ages , i ncl udi ng mi l d mes angi al promi nence, l os s of gl omerul ar cel l ul ari t y, and col l aps e of capi l l ary l oops .
3. Clinical features and diagnosis o
o
a. Nephrotic syndrome i s t he mos t common cl i ni cal pres ent at i on. Hypert ens i on and renal fai l ure occur i nfrequent l y i n chi l dhood but become more preval ent wi t h advanci ng age.
o
Pa g e 1 2 6 1
ABC Ambe r CHM Conve rte r Tria l ve rsion, http://w w w .proce sste x t.com/a bcchm.html o
b. There are no s peci fi c l aborat ory fi ndi ngs . Prot ei nuri a t ends t o be heavy (i .e., >15 g/24 hours ), and t he bi ochemi cal derangement s of nephrot i c s yndrome are accordi ngl y s evere.
4. T herapy o
o
a. Recent s t udi es s ugges t t hat prol onged gl ucocort i coi d t herapy (i .e., ≥6 mont hs ) may l ead t o remi s s i on of prot ei nuri a, al t hough no randomi zed t ri al s s upport t hi s recommendat i on.
o
o
b. Cycl os pori ne as wel l as ACE i nhi bi t ors and ARBs may be effect i ve i n reduci ng prot ei nuri a.
P.259
G. Goodpasture's syndrome
1. Definition. Goodpas t ure's s yndrome refers t o a group of i l l nes s es defi ned by t he fol l owi ng t ri ad of fi ndi ngs : glomerulonephritis (us ual l y cres cent i c), pulmonary hemorrhage, and anti-GBM antibody i n s erum. The renal and pul monary component s may be s evere or cl i ni cal l y s i l ent . The pres ence of t he ant i -GBM ant i body, however, has become t he es s ent i al feat ure of t he di agnos i s .
2. Clinical features and diagnosis o
Pa g e 1 2 6 2
ABC Ambe r CHM Conve rte r Tria l ve rsion, http://w w w .proce sste x t.com/a bcchm.html
o
a. Clinical presentation i s hi ghl y vari abl e.
(1) Generalized, systemic symptoms may precede organ-s peci fi c compl ai nt s . Fever and myal gi a are common.
(2) Renal involvement us ual l y i s i n t he form of rapi dl y progres s i ve renal fai l ure. Prot ei nuri a us ual l y i s mi l d, and t he uri nary s edi ment cont ai ns eryt hrocyt es and RBC cas t s . Thi s “nephri t i c― pi ct ure may be mi l d or s evere.
(3) Pulmonary manifestations i ncl ude radi ographi c i nfi l t rat es , hemopt ys i s , cough, and dys pnea. The l ung di s eas e us ual l y precedes ki dney di s eas e by a peri od of days t o weeks .
o
o
b. Laboratory findings
(1) The mos t i mport ant fi ndi ng i s evi dence of ci rcul at i ng i mmunogl obul i n G (IgG) ant i -GBM ant i body, whi ch i s pres ent i n more t han 90% of pat i ent s .
(2) The pat hol ogi c appearance of t he ki dney i s t ypi cal l y t hat of a cres cent i c, prol i ferat i ve gl omerul onephri t i s . Cres cent s i nvol ve 80%–100% of gl omerul i and are hi ghl y
Pa g e 1 2 6 3
ABC Ambe r CHM Conve rte r Tria l ve rsion, http://w w w .proce sste x t.com/a bcchm.html
cel l ul ar.
3. Clinical course. Li ke t he cl i ni cal pres ent at i on, t he cours e of di s eas e i s vari abl e, rangi ng from a mi nor recurrent pul monary hemorrhage t hat occurs for years —unt i l an abnormal uri nary s edi ment prompt s meas urement of ant i -GBM ant i body—t o abrupt -ons et , ful mi nant di s eas e, compl et e renal fai l ure, and a s phyxi at i on by mas s i ve pul monary bl eedi ng over a peri od of hours t o days .
4. T herapy. Several t reat ment met hods appear t o benefi t pat i ent s . Poor prognos t i c i ndi cat ors i ncl ude ol i goanuri a, s erum creat i ni ne above 6 mg/dL for many weeks , and advanced hi s t opat hol ogi c l es i ons . o
o
a. Hi gh-dos e predni s one s houl d be gi ven as an i ni t i al t herapy. Cycl ophos phami de s houl d al s o be admi ni s t ered t o pat i ent s under 55 years of age. A vari et y of s ucces s ful but uncont rol l ed cl i ni cal t ri al s have i nvol ved combi nat i ons of cort i cos t eroi ds and al kyl at i ng i mmunos uppres s i ve agent s .
o
o
b. Int ens i ve, dai l y pl as mapheres i s s houl d be admi ni s t ered for 14 days or unt i l ant i -GBM ant i body di s appears .
H. Idiopathic crescentic glomerulonephritis Indi vi dual s wi t h t hi s pat hol ogi cal l y defi ned ent i t y t ypi cal l y pres ent wi t h rapi d, progres s i ve det eri orat i on of renal funct i on. It i s i mperat i ve t o recogni ze t hat ot her l es i ons may i nduce t he cl i ni cal s yndrome of rapidly progressive glomerulonephritis (RPGN) (Tabl e 6-9). Thi s s ect i on cons i ders onl y t he i di opat hi c cas es (i .e.,
Pa g e 1 2 6 4
ABC Ambe r CHM Conve rte r Tria l ve rsion, http://w w w .proce sste x t.com/a bcchm.html
t hos e cas es not due t o ot her cres cent i c gl omerul ar di s eas es ). Idi opat hi c cres cent i c gl omerul onephri t i s may be cl as s i fi ed i nt o t hree ent i t i es : anti-GBM antibody disease (s ee Part I: X G); immune complex RPGN; and pauci-immune RPGN, i n whi ch gl omerul ar i nfl ammat i on and necros i s are pres ent but wi t hout i mmune depos i t s .
1. Incidence and etiology. Idi opat hi c cres cent i c gl omerul onephri t i s account s for about one-t hi rd of al l cas es of cres cent i c gl omerul onephri t i s . Mal es are affect ed t wi ce as oft en as femal es .
2. Clinical features. Pat i ent s pres ent wi t h abrupt -ons et renal fai l ure, wi t h rapi d l os s of renal funct i on (i n l es s t han 3 mont hs ); frequent l y normal bl ood pres s ure; and normal ki dney s i ze. Nons peci fi c s ympt oms (e.g., weaknes s , naus ea, cough, wei ght l os s , fever, myal gi a, art hral gi a) oft en announce t he di s eas e. Ext rarenal i nvol vement , wi t h t he except i on of l ung i nvol vement , i s rare. o
o
a. Renal manifestations. Approxi mat el y 50% of pat i ent s are ol i guri c and azot emi c at t he t i me of pres ent at i on.
o
o
b. Pulmonary manifestations. Trans i ent , mi l d pul monary i nfi l t rat es or hemopt ys i s i s s een i n one hal f of pat i ent s .
3. Diagnosis. There are no di agnos t i c l aborat ory fi ndi ngs . However, when i nt rarenal vas cul i t i s (i .e., pauci -i mmune gl omerul onephri t i s ) i s t he underl yi ng
Pa g e 1 2 6 5
ABC Ambe r CHM Conve rte r Tria l ve rsion, http://w w w .proce sste x t.com/a bcchm.html
caus e, t he antineutrophilic cytoplasmic P.260
antibody (ANCA) t es t i s pos i t i ve. The di agnos i s i s bas ed on t he di s covery of epi t hel i al cres cent s i n a majori t y of gl omerul i i n t he renal bi ops y s peci men.
TABLE 6-9 Causes of Acute Renal Failure with Crescentic Glomerulonephritis In pri ma ry glo me rul ar dis ea ses Pri ma ry (i di op at h i c) di ff us e cre s ce nt i c
Pa g e 1 2 6 6
ABC Ambe r CHM Conve rte r Tria l ve rsion, http://w w w .proce sste x t.com/a bcchm.html
gl o me rul on ep hri t is Typ e I: ant i -G BM ant i bo dy di s eas e wit ho ut pul mo nar y he mo rrh ag e Typ e II: im
Pa g e 1 2 6 7
ABC Ambe r CHM Conve rte r Tria l ve rsion, http://w w w .proce sste x t.com/a bcchm.html
mu ne co mp l ex di s eas e Typ e III: pa uci -i m mu ne gl o me rul on ep hri t is (A NC A-a sso ci a t ed ) Me s an gi o cap illa
Pa g e 1 2 6 8
ABC Ambe r CHM Conve rte r Tria l ve rsion, http://w w w .proce sste x t.com/a bcchm.html
ry gl o me rul on ep hri t is (es pec i al l y t yp e II) Me mb ran ous gl o me rul on ep hri t is wi t h or wi t ho ut s up eri mp
Pa g e 1 2 6 9
ABC Ambe r CHM Conve rte r Tria l ve rsion, http://w w w .proce sste x t.com/a bcchm.html
os e d ant i -G BM ant i bo dy di s eas e IgA ne phr op at h y (Be rge r's di s eas e) In ass oci ati on wit h inf ect iou s dis
Pa g e 1 2 7 0
ABC Ambe r CHM Conve rte r Tria l ve rsion, http://w w w .proce sste x t.com/a bcchm.html
ea ses Pos tst rep t oc occ al gl o me rul on ep hri t is Inf ect i ve en doc ard itis Occ ul t vi s cer al bac t eri al s ep sis Ot her i nf
Pa g e 1 2 7 1
ABC Ambe r CHM Conve rte r Tria l ve rsion, http://w w w .proce sste x t.com/a bcchm.html
ect i on s (e. g., he pat itis B) In as s oci at i on wi t h mu ltis ys t em di s eas es SLE Go od pas t ur e's s yn dro me (an t i -G
Pa g e 1 2 7 2
ABC Ambe r CHM Conve rte r Tria l ve rsion, http://w w w .proce sste x t.com/a bcchm.html
BM ant i bo dy di s eas e wi t h pul mo nar y he mo rrh ag e) He noc h-S chà ¶nl ei n pur pur aâ €”d iss em i na t ed vas cul i tis
Pa g e 1 2 7 3
ABC Ambe r CHM Conve rte r Tria l ve rsion, http://w w w .proce sste x t.com/a bcchm.html
We ge ner 's gra nul om at o sis Mi c ros cop ic pol yar t eri tis (hy per s en siti vi t y an gi i t is) Ot her var i an ts Cry oi mm un
Pa g e 1 2 7 4
ABC Ambe r CHM Conve rte r Tria l ve rsion, http://w w w .proce sste x t.com/a bcchm.html
ogl ob ul i ne mi a (mi xed , es s ent i al ) Rel aps i ng pol ych on dri t is Lun g can cer , l ym ph om a Ant i -G BM , ant i -gl
Pa g e 1 2 7 5
ABC Ambe r CHM Conve rte r Tria l ve rsion, http://w w w .proce sste x t.com/a bcchm.html
om eru l ar bas em ent me mb ran e; IgA , im mu no gl o bul in A; SLE , s ys te mi c l up us ery t he ma t os us .
4. Clinical course and prognosis. The prognos i s may be very bl eak, dependi ng on t he l evel of renal funct i on at
Pa g e 1 2 7 6
ABC Ambe r CHM Conve rte r Tria l ve rsion, http://w w w .proce sste x t.com/a bcchm.html
t he t i me of pres ent at i on. Renal fai l ure requi ri ng renal repl acement t herapy devel ops i n 3–6 mont hs i n more t han 50% of pat i ent s . However, recent dat a us i ng aggres s i ve i mmunos uppres s i ve regi mens have demons t rat ed t hat 50%–75% of pat i ent s may ent er remi s s i on.
5. T herapy. In ant i -GBM di s eas e, pl as mapheres i s and i mmunos uppres s i ves s houl d be us ed as des cri bed for Goodpas t ure's s yndrome (s ee Part I: X G). In i mmune compl ex gl omerul onephri t i s , t reat ment depends on t he i ndi vi dual caus at i ve di s order. In pauci -i mmune depos i t di s eas e, t reat ment i nvol ves pul s e met hyl predni s ol one and cycl ophos phami de.
I. Postinfectious glomerulonephritis Thi s acut e gl omerul onephri t i s occurs wi t h a vari et y of l ocal or s ys t emi c i nfect i ons . (Gl omerul onephri t i s as s oci at ed wi t h i nfect i ve endocardi t i s and vi s ceral abs ces s i s di s cus s ed i n Part I: X Q.) Pos t i nfect i ous gl omerul onephri t i s has been des cri bed as a s equel a of di s eas e caus ed by vi rus es , fungi , prot ozoa, and hel mi nt hs . However, t he prot ot ypi cal pos t i nfect i ous gl omerul onephri t i s i s poststreptococcal glomerulonephritis.
1. Incidence and etiology o
o
a. The di s eas e pri mari l y affect s s chool -aged chi l dren. The di s eas e i s rare before 2 years of age but has been report ed i n adul t s . Mal es are affect ed t wi ce as oft en as femal es .
o
o
b. Precedi ng i nfect i on wi t h nephri t ogeni c s t rai ns of group A β-hemol yt i c s t rept ococci (part i cul arl y t ype
Pa g e 1 2 7 7
ABC Ambe r CHM Conve rte r Tria l ve rsion, http://w w w .proce sste x t.com/a bcchm.html
12) i s t he rul e, al t hough pos i t i ve cul t ure of t he organi s m i s demons t rat ed i n l es s t han 20% of cas es at t he t i me of renal di s eas e. The s i t e of i nfect i on (i .e., s ki n or pharynx) appears t o vary wi t h t he geographi c area of s t udy. The l at ent peri od bet ween i nfect i on and cl i ni cal gl omerul ar di s eas e i s 7–15 days ; rarel y, i t i s as l ong as 3 weeks .
2. Pathology. Pos t s t rept ococcal gl omerul onephri t i s i s a di ffus e prol i ferat i ve di s eas e wi t h mes angi al and endot hel i al hypercel l ul ari t y. El ect ron-dens e depos i t s (s ubepi t hel i al “humps ―) and foot proces s P.261
fus i on are s een by el ect ron mi cros copy (Fi gure 6-6). Immunofl uores cence oft en i dent i fi es granul ar depos i t s of compl ement C3 al ong t he capi l l ary bas ement membrane.
FIGURE 6-6 El ect ron mi cros copi c fi ndi ngs of dens e s ubepi t hel i al depos i t s (humps ) i n a pat i ent wi t h pos t s t rept ococcal gl omerul onephri t i s . (From W i l l i ams G, Mal l i ck NP. Col or At l as of Renal Di s eas es . 2nd ed. Barcel ona: W ol fe Publ i s hi ng/Mos by-Y ear Book Europe Lt d, 1994:3.17.
Pa g e 1 2 7 8
ABC Ambe r CHM Conve rte r Tria l ve rsion, http://w w w .proce sste x t.com/a bcchm.html
)
3. Clinical features and diagnosis o
o
a. The t ypi cal cl i ni cal pres ent at i on i s a s udden ons et of hemat uri a and edema. Nephrot i c s yndrome devel ops i n l es s t han 15% of pat i ent s .
o
o
b. The charact eri s t i c, but not di agnos t i c, l aborat ory profi l e i s azot emi a, hypocompl ement emi a (CH 5 0 or C3), hemat uri a, l eukocyt uri a, and prot ei nuri a. Support i ng dat a i ncl ude el evat ed t i t ers of ant i s t rept ol ys i n O, ant i hyal uroni das e, and ant i -deoxyri bonucl eas e B ant i bodi es , al l of whi ch s ugges t precedi ng s t rept ococcal i nfect i on.
4. Clinical course and diagnosis o
o
a. The t ypi cal cours e of acut e di s eas e i s recovery, part i cul arl y among chi l dren. The acut e nephri t i s res ol ves wi t h amel i orat i on of edema and hypert ens i on 1–3 weeks aft er ons et . Prot ei nuri a may pers i s t for s everal mont hs , exacerbat ed by erect pos t ure and exerci s e. Mi cros copi c hemat uri a s i mi l arl y di s appears s l owl y over a peri od of s everal mont hs .
o
o
b. Long-t erm prognos i s i s cont rovers i al . Some pat i ent s advance t o end-s t age renal di s eas e. Fact ors as s oci at ed wi t h t hi s poor prognos i s are s evere ol i guri a or anuri a, cres cent s i n t he bi ops y
Pa g e 1 2 7 9
ABC Ambe r CHM Conve rte r Tria l ve rsion, http://w w w .proce sste x t.com/a bcchm.html
s peci men, pers i s t ent heavy prot ei nuri a, and rel at i vel y ol der age. Pers i s t ence of hypocompl ement emi a and progres s i ve renal fai l ure may, however, i ndi cat e an al t ernat i ve di agnos i s (e.g., membranoprol i ferat i ve gl omerul onephri t i s ).
5. T herapy. Hypert ens i on mus t be t reat ed aggres s i vel y, part i cul arl y i n chi l dren, who devel op fl ori d hypert ens i ve encephal opat hy at normal adul t bl ood pres s ures . Furos emi de or bumet ani de may l i kel y be requi red for t he underl yi ng edema-i nduci ng di s eas e. Ant i bi ot i c us e i s cont rovers i al , al t hough a 10-day cours e of peni ci l l i n i n nonal l ergi c pat i ent s i s s afe enough for rout i ne us e. Prophyl axi s fol l owi ng pos t s t rept ococcal gl omerul onephri t i s i s not i ndi cat ed becaus e recurrences are exceedi ngl y rare. Immunos uppres s i ve agent s or cort i cos t eroi ds have no t herapeut i c rol e.
J. IgA glomerulonephritis (Berger's disease)
1. Definition and incidence. The di s eas e i s charact eri zed by mes angi al depos i t s of IgA i n renal bi ops i es from pat i ent s wi t h recurrent hemat uri a but normal renal funct i on. The i nci dence of t hi s di s eas e vari es remarkabl y wi t h geographi c l ocat i on, and men are affect ed t hree t o four t i mes more oft en t han women.
2. Pathology. Fi ndi ngs are charact eri s t i c. o
o
a. Di ffus e, s omet i mes i rregul arl y di s t ri but ed IgA depos i t s are s een i n t he mes angi um. IgM or IgG al s o may be pres ent .
Pa g e 1 2 8 0
ABC Ambe r CHM Conve rte r Tria l ve rsion, http://w w w .proce sste x t.com/a bcchm.html o
o
b. Focal and s egment al gl omerul onephri t i s wi t h mes angi al prol i ferat i on i s common. Mes angi al promi nence may be t he onl y pat hol ogi c fi ndi ng.
P.262
3. Clinical features and diagnosis. Affect ed pat i ent s , who us ual l y are bet ween 20 and 40 years of age, mos t commonl y pres ent wi t h recurrent , oft en macros copi c hemat uri a but a normal GFR and normal t ubul ar funct i on. Bi ops y s peci mens from normal -appeari ng s ki n have i mmunofl uores cent pos i t i vi t y for IgA i n 50% of cas es . Meas uri ng s erum IgA l evel s i s not us eful .
4. Clinical course and prognosis. Cours e and out come are vari abl e. The 20-year s urvi val rat e i s about 50%. A mi nori t y of pat i ent s progres s t o renal fai l ure. Fact ors t hat predi ct a poor prognos i s i ncl ude advanced age at di s eas e ons et , heavy prot ei nuri a, hypert ens i on, and t he pres ence of cres cent s or s egment al s cl eros i s on renal bi ops y.
5. T herapy. Pat i ent s wi t h mi l d hi s t ol ogi c changes and prot ei nuri a of more t han 3 g/day s houl d recei ve predni s one for 4–6 mont hs , as wel l as ACE i nhi bi t or or ARB t herapy. Recent dat a s ugges t t hat fi s h oi l cont ai ni ng a number of ant i -i nfl ammat ory fat t y aci ds may be us eful i n s ome pat i ent s wi t h s l owl y progres s i ve di s eas e.
K. Henoch-Schönlein purpura
Pa g e 1 2 8 1
ABC Ambe r CHM Conve rte r Tria l ve rsion, http://w w w .proce sste x t.com/a bcchm.html
1. Definition. Thi s s ys t emi c di s eas e i s charact eri zed by purpura (whi ch may be s l i ght and go unnot i ced), art hri t i s , abdomi nal pai n, bl oody di arrhea, and nephri t i s .
2. Incidence. Henoch-Schönl ei n purpura pri mari l y affect s chi l dren. Cl i ni cal nephri t i s affect s 30% of pat i ent s , but al mos t al l pat i ent s have an abnormal ki dney bi ops y.
3. Pathology. The hi s t opat hol ogy ranges from mi l d, di ffus e mes angi al promi nence t o focal and s egment al prol i ferat i ve gl omerul onephri t i s on a background of di ffus e mes angi al prol i ferat i on. The hal l mark of Henoch-Schönl ei n purpura i s t he i nvari abl e i mmunofl uores cent pos i t i vi t y for IgA i n t he mes angi um.
4. Clinical features and diagnosis o
o
a. The cl i ni cal pres ent at i on oft en i s preceded by an i nfect i on [caus ed by a vi rus (e.g., herpes zos t er), mycopl as ma, or s t rept ococcus ], vacci nat i on, i ns ect bi t e, or drug admi ni s t rat i on. Ras h us ual l y devel ops earl y and evol ves from morbi l l i form t o purpuri c. The l egs and but t ocks are affect ed mos t commonl y. Art hri t i s t ypi cal l y i s mi l d and nondeformi ng. Gas t roi nt es t i nal bl eedi ng and pai n may domi nat e t he pres ent at i on.
o
o
b. Laborat ory fi ndi ngs are exceedi ngl y nons peci fi c,
Pa g e 1 2 8 2
ABC Ambe r CHM Conve rte r Tria l ve rsion, http://w w w .proce sste x t.com/a bcchm.html
al t hough el evat ed s erum IgA i s report ed frequent l y. Serum compl ement component l evel s us ual l y are normal .
5. Clinical course and prognosis o
o
a. The cl i ni cal cours e i s vari abl e. Pat i ent s wi t h recurrent purpura, heavy prot ei nuri a, and cl i ni cal l y s evere nephri t i s at t he t i me of pres ent at i on and pat i ent s whos e bi ops i es s how epi t hel i al cres cent format i on t end t o fare poorl y.
o
o
b. Among al l chi l dren wi t h Henoch-Schönl ei n purpura, t he 15-year s urvi val rat e i s 90%. By 10 years , however, 15% of t hes e pat i ent s have pers i s t i ng di s eas e and 8% have renal i mpai rment . Among adul t s , 50% heal compl et el y, 15% progres s t o renal fai l ure, and approxi mat el y 35% have pers i s t ent di s eas e.
6. T herapy. Several t reat ment met hods have been at t empt ed (e.g., i mmunos uppres s i on, s t eroi d t herapy, ant i coagul at i on) but wi t hout proven benefi t .
L. Diabetic nephropathy
1. Incidence. End-s t age renal di s eas e devel ops i n 30% of al l pat i ent s wi t h di abet es . Among pat i ent s wi t h juveni l e-ons et di abet es , 30% devel op renal di s eas e wi t hi n 20 years of t he ons et of di abet es . Among new pat i ent s cons i dered for mai nt enance renal repl acement t herapy (l argel y chroni c hemodi al ys i s ), at l eas t 40%
Pa g e 1 2 8 3
ABC Ambe r CHM Conve rte r Tria l ve rsion, http://w w w .proce sste x t.com/a bcchm.html
have chroni c renal fai l ure s econdary t o di abet es . Afri can Ameri can, Hi s pani c, and Nat i ve Ameri can i ndi vi dual s have a hi gher l i kel i hood of devel opi ng di abet i c nephropat hy t han Caucas i an pat i ent s who have di abet es .
2. Pathogenesis o
o
a. The evol ut i on of di abet i c nephropat hy i s s ympt omat i cal l y qui et unt i l l at e i n t he di s eas e proces s . Earl y i n di abet es , t he GFR oft en i s above normal . Thi s earl y hyperfi l t rat i on may be mos t s t ri ki ng i n t hos e pat i ent s who s ubs equent l y devel op gl omerul ar damage.
o
o
b. Ini t i al l y, microalbuminuria [t he l os s of s mal l amount s of prot ei n (range: 30–300 mg/day)] occurs , whi ch progres s es t o macroprot ei nuri a, fol l owed by azot emi a, and ul t i mat el y end-s t age renal di s eas e. The rapi di t y of t hi s cours e i s hi ghl y vari abl e.
P.263
3. Pathology. Two major pat hol ogi c l es i ons are as s oci at ed wi t h di abet es . o
o
a. Diffuse glomerulosclerosis i s uni forml y pres ent i n pat i ent s wi t h di abet i c nephropat hy. It i s charact eri zed by an eos i nophi l i c t hi ckeni ng of t he mes angi um and bas ement membrane due t o
Pa g e 1 2 8 4
ABC Ambe r CHM Conve rte r Tria l ve rsion, http://w w w .proce sste x t.com/a bcchm.html
accumul at i on of ext racel l ul ar mat ri x prot ei ns . o
b. Nodular glomerulosclerosis, al s o known as Kimmelstiel-Wilson syndrome, cons i s t s of round nodul es t hat are homogeneous at t he cent er and have ci rcumferent i al l ayeri ng of nucl ei
(Onl i ne Fi gure
6-7). Thes e nodul es oft en are mul t i pl e wi t hi n a gi ven gl omerul us and may be confl uent . Nodul ar gl omerul os cl eros i s i s s peci fi c for di abet es , but i t i s found i n onl y 50% of pat i ent s wi t h di abet i c nephropat hy. o
ONLINE FIGURE 6-7 Pat hognomi c nodul es (N) from a pat i ent wi t h di ffus e i nt ercapi l l ary gl omerul os cl eros i s caus ed by di abet es mel l i t us . (From W i l l i ams G, Mal l i ck NP. Col or At l as of Renal Di s eas es . 2nd ed. Barcel ona: W ol fe Publ i s hi ng/Mos by-Y ear Book Europe Lt d, 1994:4.5. )
4. Laboratory findings. Res ul t s are i ndi cat i ve of a s l owl y decl i ni ng GFR.
Pa g e 1 2 8 5
ABC Ambe r CHM Conve rte r Tria l ve rsion, http://w w w .proce sste x t.com/a bcchm.html
5. Clinical course. Fact ors t hat accel erat e renal det eri orat i on i ncl ude hypert ens i on, poor gl ycemi c cont rol , uri nary obs t ruct i on, i nfect i on, t he admi ni s t rat i on of nephrot oxi c drugs , and t he us e of i nt ravenous radi ocont ras t mat eri al .
6. T herapy o
o
a. Diabetic nephropathy s houl d be aggres s i vel y t reat ed.
(1) Ti ght cont rol of bl ood gl ucos e ret ards progres s i on of renal di s eas e, es peci al l y i n t hos e pat i ent s wi t h t he earl i es t l es i ons (mi croal bumi nuri a).
(2) Res t ri ct i on of di et ary prot ei n (i .e., t o <0.8 g/kg body wei ght /day) may s l ow t he devel opment of renal fai l ure once prot ei nuri a occurs and s houl d be i ni t i at ed earl y.
(3) St ri ct cont rol of hypert ens i on i s cruci al t o s l owi ng t he progres s i on of renal fai l ure. St udi es i ndi cat e t hat ACE i nhi bi t ors and ARBs pl ay an i mport ant rol e as pri mary t herapy for di abet i c pat i ent s wi t h renal dys funct i on (i ncl udi ng any degree of al bumi nuri a, hypert ens i on, or a reduced GFR).
(4) Avoi dance of nephrot oxi ns and
Pa g e 1 2 8 6
ABC Ambe r CHM Conve rte r Tria l ve rsion, http://w w w .proce sste x t.com/a bcchm.html
s urvei l l ance for uri nary i nfect i on or obs t ruct i on (neurogeni c bl adder) are prudent cons ervat i ve meas ures .
b. End-stage renal disease i s t reat ed us i ng renal repl acement t herapy when t he GFR i s <15 mL/mi nut e. o
o
(1) Hemodialysis i s t he mos t common form of renal repl acement empl oyed and i s t he mai ns t ay of management of end-s t age renal di s eas e.
o
o
(2) Good res ul t s may be achi eved us i ng cont i nuous ambul at ory peri t oneal di al ys i s i n pat i ent s wi t h di abet es . Ins ul i n may be gi ven i nt raperi t oneal l y, and i mproved di abet i c cont rol may be pos s i bl e.
o
o
(3) Pat i ent s who undergo t rans pl ant at i on of a ki dney from a l i vi ng rel at i ve have a bet t er 5-year s urvi val rat e t han t hos e who undergo chroni c hemodi al ys i s .
M. Lupus nephritis The mos t wi del y accept ed cl as s i fi cat i on i n us e for l upus nephri t i s i s t he W orl d Heal t h Organi zat i on (W HO) cl as s i fi cat i on. It combi nes t he us e of l i ght mi cros copi c (LM) fi ndi ngs wi t h i mmunofl uores cence (IF) and el ect ron mi cros copi c (EM) fi ndi ngs t o pres ent a uni form cl as s i fi cat i on s ys t em (Tabl e 6-10).
1. WHO classes o
o
a. WHO class II (mesangial glomerulonephritis)
Pa g e 1 2 8 7
ABC Ambe r CHM Conve rte r Tria l ve rsion, http://w w w .proce sste x t.com/a bcchm.html
may occur as t he earl i es t form of l upus nephri t i s .
TABLE 6-10 Original World Health Organization (WHO) Classification of Lupus Nephritis De scr Cla ipti ss on I Nor ma l gl o me rul i (by LM, IF, EM II
) Me s an gi a l gl o me rul on ep hri t is Nor mo cel l
Pa g e 1 2 8 8
ABC Ambe r CHM Conve rte r Tria l ve rsion, http://w w w .proce sste x t.com/a bcchm.html
ul a r me s an gi u m by LM but me s an gi a l de pos its by IF or EM Me s an gi a l hyp erc el l ul a ri t y wi t h me s an gi a l
Pa g e 1 2 8 9
ABC Ambe r CHM Conve rte r Tria l ve rsion, http://w w w .proce sste x t.com/a bcchm.html
de pos its by IF or EM III Foc al s eg me nt a l pro l i fe rat i ve gl o me rul on ep hri t is IV Di f fus e pro l i fe rat i ve gl o me rul on
Pa g e 1 2 9 0
ABC Ambe r CHM Conve rte r Tria l ve rsion, http://w w w .proce sste x t.com/a bcchm.html
ep hri t V
is Me mb ran ous gl o me rul on ep hri t is
EM, el ect ro n mi cros copy; IF, i mmun ofl uore s cence ; LM, l i ght mi cros copy. o
P.264
o
(1) Pathology. Bi ops y s hows mes angi al promi nence wi t h an i ncreas e i n mat ri x and i n t he number of mes angi al cel l s .
Pa g e 1 2 9 1
ABC Ambe r CHM Conve rte r Tria l ve rsion, http://w w w .proce sste x t.com/a bcchm.html
(2) Clinical features and diagnosis
(a) The compl et e s pect rum of cl i ni cal fi ndi ngs i s not ful l y known. Many pat i ent s are as ympt omat i c or pres ent wi t h onl y mi l d uri nary abnormal i t i es .
(b) Serol ogi c fi ndi ngs i ncl ude pos i t i ve fl uores cent ANA, mi l d ant i -DNA ant i body el evat i ons , and normal or mi l dl y decreas ed C3 and C4 l evel s .
(3) Clinical course and prognosis
(a) Uri nary abnormal i t i es may remi t , and t rans i t i on t o di ffus e prol i ferat i ve or membranous l upus nephri t i s occurs i n 15% of pat i ent s .
(b) Thi s mes angi al l es i on i s as s oci at ed wi t h cl i ni cal progres s i on onl y i f t here i s t rans i t i on t o a l es s favorabl e hi s t ol ogy.
o
o
b. WHO class III (focal proliferative glomerulonephritis) devel ops duri ng t he fi rs t year of cl i ni cal l upus i n 50% of pat i ent s .
(1) Pathology. The l es i on i s s harpl y del i neat ed s egment al endot hel i al and mes angi al cel l prol i ferat i on, whi ch us ual l y
Pa g e 1 2 9 2
ABC Ambe r CHM Conve rte r Tria l ve rsion, http://w w w .proce sste x t.com/a bcchm.html
affect s l es s t han 50% of al l gl omerul i .
(2) Clinical features and diagnosis
(a) Prot ei nuri a i s s een i n al mos t al l cas es ; however, nephrot i c s yndrome i s rare. Hemat uri a i s common, and mi l d renal i ns uffi ci ency i s s een occas i onal l y. Hypert ens i on i s not pres ent .
(b) Serol ogi c fi ndi ngs i ncl ude pos i t i ve fl uores cent ant i nucl ear ant i body (ANA) and modes t el evat i ons i n ant i -DNA ant i bodi es . Compl ement component s (C3 and C4) are at normal or decreas ed l evel s .
(3) Clinical course and prognosis
(a) Remi s s i on, as meas ured by ces s at i on of prot ei nuri a, i s s een i n about 50% of pat i ent s . Rel aps es commonl y occur wi t h ext rarenal fl ares of s ys t emi c l upus . Trans i t i on t o ot her forms of t he di s eas e (e.g., di ffus e prol i ferat i ve or membranous l upus nephri t i s ) occurs i n at l eas t 20% of pat i ent s .
(b) Renal fai l ure i s rare unl es s t he di s eas e progres s es t o di ffus e prol i ferat i ve l upus nephri t i s .
Pa g e 1 2 9 3
ABC Ambe r CHM Conve rte r Tria l ve rsion, http://w w w .proce sste x t.com/a bcchm.html
(c) The 5-year mort al i t y rat e i s 10%.
o
o
c. WHO class IV (diffuse proliferative glomerulonephritis) mos t commonl y devel ops wi t hi n t he fi rs t year of cl i ni cal l upus .
(1) Pathology. Mes angi al and endot hel i al cel l prol i ferat i on affect mos t gl omerul i wi t h varyi ng s everi t y. Capi l l ary l umi na are obl i t erat ed, and cres cent s affect up t o 30% of gl omerul i . Depos i t s of IgG, C3, C4, and C1q are di ffus e. IgA and IgM depos i t s al s o are s een frequent l y.
(2) Clinical features and diagnosis
(a) Prot ei nuri a and hemat uri a are uni vers al ; more t han 50% of pat i ent s pres ent wi t h nephrot i c s yndrome. Azot emi a, whi ch i s common earl y i n t he cours e of t he di s eas e, may be s evere. Hypert ens i on occurs commonl y.
(b) Serol ogi c fi ndi ngs i ncl ude pos i t i ve fl uores cent ANA, hi ghl y el evat ed ant i -DNA ant i bodi es , and depres s ed l evel s of C3 and C4. Cryogl obul i nemi a devel ops i n s ome cas es .
Pa g e 1 2 9 4
ABC Ambe r CHM Conve rte r Tria l ve rsion, http://w w w .proce sste x t.com/a bcchm.html
(3) Clinical course and prognosis
(a) Remi s s i on of t he nephrot i c s yndrome, whi ch i s s een i n 33% of pat i ent s , i s s omet i mes s us t ai ned. Trans i t i on t o mes angi al l upus nephri t i s occurs occas i onal l y i n as s oci at i on wi t h cl i ni cal remi s s i on.
(b) The 5-year mort al i t y rat e i s 50%. Deat h res ul t s from uremi a or act i ve s ys t emi c l upus , whi ch frequent l y i s compl i cat ed by i nfect i on. Hypert ens i on and renal fai l ure may occur as s equel ae even aft er l ong peri ods of cl i ni cal remi s s i on.
o
o
d. WHO class V (membranous glomerulonephritis) devel ops duri ng t he fi rs t year of cl i ni cal l upus i n about 50% of pat i ent s .
(1) Pathology. The hi s t opat hol ogi c pat t ern of membranous l upus nephri t i s i s very s i mi l ar t o t hat of i di opat hi c membranous gl omerul onephri t i s (s ee Part I: X C 2).
(2) Clinical features and diagnosis
(a) Prot ei nuri a i s s een i n al l pat i ent s , and hemat uri a i s common as wel l . Nephrot i c s yndrome i s s een at
Pa g e 1 2 9 5
ABC Ambe r CHM Conve rte r Tria l ve rsion, http://w w w .proce sste x t.com/a bcchm.html
pres ent at i on i n 50% of pat i ent s and ul t i mat el y occurs i n 80% of P.265
pat i ent s . Hypert ens i on and renal i ns uffi ci ency are rare at t he out s et of membranous l upus nephri t i s .
(b) Serol ogi c fi ndi ngs i ncl ude pos i t i ve fl uores cent ANA, normal or onl y mi l dl y el evat ed ant i -DNA ant i bodi es , and normal or decreas ed l evel s of C3 and C4.
(3) Clinical course and prognosis
(a) Remi s s i on from nephrot i c s yndrome i s s een i n 33% of pat i ent s , but rel aps es are common. Trans i t i on t o focal or di ffus e prol i ferat i ve l upus nephri t i s has been report ed but i s rare.
(b) The 5-year mort al i t y rat e i s 10% for pat i ent s who devel op hypert ens i on and renal i ns uffi ci ency duri ng pers i s t ent nephrot i c s yndrome.
2. Superimposed lesions In addi t i on t o t he major l es i ons des cri bed i n t he W HO cl as s i fi cat i on s ys t em, s everal s uperi mpos ed (s econdary) l es i ons may be s een.
Pa g e 1 2 9 6
ABC Ambe r CHM Conve rte r Tria l ve rsion, http://w w w .proce sste x t.com/a bcchm.html o
o
a. Glomerulosclerosis i s a s econdary l es i on t hat i s s een mos t commonl y i n di ffus e prol i ferat i ve l upus nephri t i s wi t h a prot ract ed cours e. Progres s i ve gl omerul os cl eros i s may be a caus e of renal fai l ure i n pat i ent s whos e s ys t emi c l upus remi t s .
o
o
b. Interstitial lupus nephritis us ual l y coexi s t s wi t h gl omerul ar di s eas e but may devel op al one.
(1) Pat hol ogi cal l y, t hi s l es i on i s charact eri zed by i nt ens e, mononucl ear i nt ers t i t i al i nfi l t rat i on, t ubul ar damage, and i nt ers t i t i al fi bros i s .
(2) IgG and C3 are i dent i fi ed i n peri t ubul ar capi l l ari es and i n t ubul ar bas ement membranes . Paral l el el ect ron-dens e depos i t s are s een.
(3) Cl i ni cal di s orders of t ubul ar funct i on (e.g., di s orders of pot as s i um excret i on, aci d excret i on, and uri ne concent rat i on and di l ut i on) are s een i n addi t i on t o vari abl e, nons el ect i ve prot ei nuri a.
o
o
c. Necrotizing vasculitis us ual l y compl i cat es di ffus e prol i ferat i ve l upus nephri t i s and pres ent s as rapi dl y accel erat i ng hypert ens i on and renal fai l ure. Hi s t opat hol ogi cal l y, an acel l ul ar necros i s of ves s el wal l s i s s een wi t h prot ei naceous occl us i ve t hrombi .
Pa g e 1 2 9 7
ABC Ambe r CHM Conve rte r Tria l ve rsion, http://w w w .proce sste x t.com/a bcchm.html
3. T herapy. Cri t eri a for t herapy are not ri gi dl y es t abl i s hed. The res pons e of membranous l upus nephri t i s t o t herapy vari es among report ed s eri es . Becaus e of t he poor prognos i s as s oci at ed wi t h di ffus e prol i ferat i ve l upus nephri t i s , t hi s l es i on current l y i s t reat ed—even i n cas es wi t h few cl i ni cal s i gns or s ympt oms of renal di s eas e. o
o
a. Glucocorticoid therapy i nvol ves vari ous regi mens . Induct i on t herapy wi t h oral predni s one (1–2 mg/kg/day) and pul s e i nt ravenous met hyl predni s ol one (1–2 g/day) has been des cri bed.
o
o
b. Cytotoxic drugs (e.g., cycl ophos phami de), when added t o s t eroi ds , oft en i nduce remi s s i on, pres erve or i mprove renal funct i on, or bot h.
N. Vasculitis
1. Introduction. The ki dney frequent l y i s i nvol ved i n s ys t emi c vas cul i t i s , al t hough t he act ual i nci dence i s unknown. The s pect rum of renal s yndromes as s oci at ed wi t h vas cul i t i s ranges from modes t “mi cros copi c― i nvol vement of art eri ol es , venul es , and capi l l ari es (a s yndrome referred t o as hypersensitivity vasculitis) t o ext ens i ve “cl as s i c― i nvol vement of medi um-s i zed ves s el s (a s yndrome referred t o as polyarteritis nodosa ).
2. Polyarteritis nodosa
Pa g e 1 2 9 8
ABC Ambe r CHM Conve rte r Tria l ve rsion, http://w w w .proce sste x t.com/a bcchm.html o
o
a. Etiology. Thi s t ype of vas cul i t i s may be pri mary (i di opat hi c) or s econdary t o drugs , vi ral i nfect i ons (e.g., HBV), or rheumat i c di s eas es (e.g., l upus , rheumat oi d vas cul i t i s ).
o
o
b. Pathology. The ki dneys s how a focal necrot i zi ng art eri t i s i n ves s el s rangi ng i n s i ze from t he renal art ery t o t he i nt erl obul ar vei ns .
o
o
c. Clinical features and diagnosis
(1) The cl i ni cal pres ent at i on oft en i s vague, cons i s t i ng of l ow-grade fever, myal gi a, art hral gi a, and wei ght l os s .
(2) The l aborat ory fi ndi ngs are numerous and, al t hough nons peci fi c, frequent l y s ugges t t he di agnos i s of pol yart eri t i s nodos a when cons i dered col l ect i vel y (s ee Tabl e 10-9).
(3) The di agnos i s can be confi rmed by renal angi ography, whi ch s hows mul t i pl e s mal l aneurys ms wi t h s egment al i nfarct i ons (Fi gure 6-8). Pos i t i ve s erum ANCA t i t ers have become a major di agnos t i c ai d i n vas cul i t i s . P.266
Pa g e 1 2 9 9
ABC Ambe r CHM Conve rte r Tria l ve rsion, http://w w w .proce sste x t.com/a bcchm.html
FIGURE 6-8 Angi ographi c demons t rat i on of mul t i pl e mi croaneurys ms charact eri s t i c of pol yart eri t i s nodos a. (Us ed wi t h permi s s i on from Das CJ, Pangt ey GS. Images i n cl i ni cal medi ci ne. Art eri al mi croaneurys ms i n pol yart eri t i s nodos a. N Engl J Med 2006;355:2574. ) o
o
d. Clinical course and prognosis. Progres s i on t o organ des t ruct i on or deat h i s t he expect ed out come.
o
o
e. T herapy. Us e of dai l y hi gh-dos e gl ucocort i coi ds and dai l y cycl ophos phami de (1–3 mg/kg/day) has i ncreas ed t he 1-year s urvi val rat e t o great er t han 80%.
3. Wegener' s granulomatosis (s ee al s o Chapt er 2 XIV C), whi ch affect s pat i ent s of al l ages but i s more common i n mi ddl e-aged men, i s a s peci al ki nd of vas cul i t i s (necrot i zi ng granul omat ous vas cul i t i s ) wi t h
Pa g e 1 3 0 0
ABC Ambe r CHM Conve rte r Tria l ve rsion, http://w w w .proce sste x t.com/a bcchm.html
renal i nvol vement . o
o
a. Pathology. The charact eri s t i c and di agnos t i c l es i on of necrot i zi ng vas cul i t i s and granul omat ous i nfl ammat i on i s mos t rel i abl y di s covered i n pul monary or upper ai rway bi ops y mat eri al . Oft en, a focal and s egment al necrot i zi ng vas cul i t i s and gl omerul i t i s are found wi t h few (i f any) i mmune depos i t s .
o
o
b. Clinical features and diagnosis
(1) The di s eas e affect s t he ki dney and upper res pi rat ory t ract , i ncl udi ng t he nos e, t hroat , and bronchi . Ul cerat i ve vas cul i t i c l es i ons , i ncl udi ng nas al s ept al perforat i on, are t he mos t recogni zabl e pres ent i ng s i gns .
(2) The hemat ol ogi c and s erol ogi c feat ures of W egener's granul omat os i s ext ens i vel y overl ap t hos e of pol yart eri t i s nodos a. ANCAs are pres ent i n t he s erum of mos t pat i ent s and may hel p i n di agnos i ng t he di s eas e and i n moni t ori ng res pons e t o t herapy.
o
o
c. Clinical course. W egener's granul omat os i s has a vari abl e cours e. Long-t erm remi s s i ons are s een occas i onal l y wi t h t herapy. Deat h us ual l y res ul t s from renal fai l ure, s eps i s , hemorrhage, or di s s emi nat ed i nt ravas cul ar coagul at i on (DIC).
o
Pa g e 1 3 0 1
ABC Ambe r CHM Conve rte r Tria l ve rsion, http://w w w .proce sste x t.com/a bcchm.html o
d. T herapy. Treat ment wi t h dai l y hi gh-dos e gl ucocort i coi ds and mont hl y cycl ophos phami de has i ncreas ed t he 1-year s urvi val rat e from l es s t han 20% t o great er t han 80%. Anecdot al report s i ndi cat e t hat t ri met hopri m-s ul famet hoxaz ol e may reduce t he rat e of recurrence.
O. Cryoglobulins and cryoglobulinemia
1. Cryoglobulins are prot ei ns t hat preci pi t at e at l ow t emperat ures and di s s ol ve on rewarmi ng. Three t ypes of cryogl obul i ns may be defi ned (Tabl e 6-11).
2. Cryoglobulinemia (i .e., pres ence of cryogl obul i ns i n t he bl ood) occurs i n a vari et y of cl i ni cal l y di s s i mi l ar condi t i ons . Renal di s eas e i s as s oci at ed pri mari l y wi t h t ypes I and II and probabl y has P.267
an i mmune compl ex–medi at ed pat hophys i ol ogy. Many pat i ent s wi t h mi xed cryogl obul i nemi a have an underl yi ng i nfect i on wi t h HCV.
TABLE 6-11 Cryoglobulins and Cryoglobulinemia
Pa g e 1 3 0 2
ABC Ambe r CHM Conve rte r Tria l ve rsion, http://w w w .proce sste x t.com/a bcchm.html
Cli nic al Fe T y atu pe res I: As s oci at e d wi t h he ma t ol ogi c ma lig na nci es Mo He noc avy l on pro al
t ei
cry nur ogl i a, ob he ul i ma ns t uri a, an d, occ
Pa g e 1 3 0 3
ABC Ambe r CHM Conve rte r Tria l ve rsion, http://w w w .proce sste x t.com/a bcchm.html
as i on al l y, an uri a Hi s t ol ogi c l es i on : us u al l y is a me mb ran opr ol i f era tiv e gl o me rul on ep hri t is II: As s
Pa g e 1 3 0 4
ABC Ambe r CHM Conve rte r Tria l ve rsion, http://w w w .proce sste x t.com/a bcchm.html
Mi x oci ed at e cry d ogl wi t ob h a ul i s yn ns dro t ha me t
of
i ncl i m ud mu e a nemo co noc mp l on l ex al
vas
co cul i mp t i s ; on ap ent pro wi t xi m h
at e
ant l y i bo 50 dy % act of i vi t pat y
i en
ag t s ai n hav st
e
pol ren ycl al on di s al
eas
Pa g e 1 3 0 5
ABC Ambe r CHM Conve rte r Tria l ve rsion, http://w w w .proce sste x t.com/a bcchm.html
IgG e Wi de s pe ct r um of cl i n i cal sig ns t ha t var y gre at l y in s ev eri t y Hy per t en sio n, az o te mi a, an d an uri
Pa g e 1 3 0 6
ABC Ambe r CHM Conve rte r Tria l ve rsion, http://w w w .proce sste x t.com/a bcchm.html
a, whi ch are po or pro gn os t ic sig ns En doc api l l ar y pro l i fe rat i on an d me s an gi a l pro mi ne nce (co mm on pat
Pa g e 1 3 0 7
ABC Ambe r CHM Conve rte r Tria l ve rsion, http://w w w .proce sste x t.com/a bcchm.html
hol ogi c fea t ur es ) III: Ma Mi x y ed be cry as s ogl oci ob at e ul i d ns wi t in h a whi var ch i et bot y h of co ot h mp er on di s ent eas s es , are wi t pol h ycl or on wi t al ho ut ren al di s eas e,
Pa g e 1 3 0 8
ABC Ambe r CHM Conve rte r Tria l ve rsion, http://w w w .proce sste x t.com/a bcchm.html
i ncl udi ng SLE , he pat itis B or C, an d s ys te mi c i nf ect i on s IgG, i mmun ogl obu l i n G; SLE, s ys t em ic l upus eryt he mat os us . o
o
a. Clinical features (s ee Tabl e 6-11)
o
Pa g e 1 3 0 9
ABC Ambe r CHM Conve rte r Tria l ve rsion, http://w w w .proce sste x t.com/a bcchm.html
o
b. Diagnosis i nvol ves t he det ect i on, charact eri zat i on, and quant i t at i on of cryogl obul i ns i n s erum.
o
o
c. T herapy i n i di opat hi c cas es i s not s t andardi zed. Immunos uppres s i ves and s t eroi ds are occas i onal l y effect i ve. Encouragi ng res ul t s have been obt ai ned wi t h pl as mapheres i s , part i cul arl y i n pat i ent s wi t h mi xed cryogl obul i nemi a. Ant i ret rovi ral t herapy may be effect i ve i n pat i ent s wi t h HCV-caus ed mi xed cryogl obul i nemi a.
P. Multiple myeloma (s ee Chapt er 4 XIV)
1. Definition. Mul t i pl e myel oma repres ent s a neopl as t i c
t rans format i on of a monocl onal B l ymphocyt e i nt o a pl as ma cel l , whi ch produces exces s i ve quant i t i es of i mmunogl obul i n or i mmunogl obul i n fragment (paraprot ei n). More t han 50% of affect ed pat i ent s di e of compl i cat i ons of renal fai l ure, and a much hi gher percent age of pat i ent s wi t h mul t i pl e myel oma have s ome form of renal i nvol vement .
2. Pathology. The many mechani s ms of renal i njury i n mul t i pl e myel oma have di fferent effect s on t he ki dney (s ee
Onl i ne
Tabl e 6-12).
Online TABLE 6-12 Mechanisms of Renal Injury in Multiple Myeloma
Pa g e 1 3 1 0
ABC Ambe r CHM Conve rte r Tria l ve rsion, http://w w w .proce sste x t.com/a bcchm.html
Me ch ani Eff sm ect of on Re the nal Kid Inj ne ury y Be Di r nce ect -Jo t ub nes ul a pro r t ei t ox nur i ci t ia
y Int rat ub ul a r obs t ru ct i on by cas t for ma tio
n Am Gl o yl o me i do rul
Pa g e 1 3 1 1
ABC Ambe r CHM Conve rte r Tria l ve rsion, http://w w w .proce sste x t.com/a bcchm.html
s i s ar an d t ub ul a r am yl o id de pos its Hy Re per nal cal vas ce oco mi ns t a
ri ct i on Cal ci u mâ €“p hos ph at e de pos itio
n Hy Acu per t e uri ura ce t e mi de a
pos
Pa g e 1 3 1 2
ABC Ambe r CHM Conve rte r Tria l ve rsion, http://w w w .proce sste x t.com/a bcchm.html
itio n an d t ub ul a r obs t ru ct i on Hy Va per s cu vi s l ar cos occ i t y l us i on Li g Gl o ht me cha rul in
ar
ne occ phr l us op i on at h y
3. Clinical features and diagnosis o
o
a. The cl i ni cal pres ent at i on of renal di s eas e i n mul t i pl e myel oma oft en i s s ubt l e. Anemi a and bone pai n i n t he pres ence of any form of abnormal uri nary fi ndi ng s houl d prompt eval uat i on for myel oma. Sl owl y progres s i ve renal i ns uffi ci ency i s
Pa g e 1 3 1 3
ABC Ambe r CHM Conve rte r Tria l ve rsion, http://w w w .proce sste x t.com/a bcchm.html
t ypi cal ; however, acut e renal fai l ure may be s een i n cert ai n ci rcums t ances (e.g., i n t he pres ence of hypercal cemi a). o
o
b. Many chemi cal abnormal i t i es commonl y occur. Ps eudohyponat remi a devel ops s econdary t o t he pres ence of l arge quant i t i es of paraprot ei n, al t eri ng t he nonaqueous phas e of pl as ma. The ani on gap i s l ow and occas i onal l y i s negat i ve becaus e of t he pos i t i ve charges on t he i mmunogl obul i n mol ecul es . Uri ne prot ei n concent rat i on i s i ncreas ed, refl ect i ng excret i on of t he huge paraprot ei n burden. As ment i oned previ ous l y, di ps t i ck meas urement for prot ei n i s i ns ens i t i ve t o i mmunogl obul i n and oft en yi el ds fal s e-negat i ve res ul t s . Thus , aci d preci pi t at i on wi t h s ul fos al i cyl i c aci d i s requi red.
4. T herapy and prognosis. Al t hough no s peci fi c t herapy exi s t s for t he renal di s eas e, chemot herapy for t he mal i gnancy, met i cul ous regul at i on of i nt ravas cul ar vol ume and el ect rol yt e s t at us , and di al ys i s (when neces s ary) may prol ong l i fe. Pl as mapheres i s may be us eful i n i mprovi ng renal funct i on i n pat i ent s wi t h acut e renal fai l ure and ci rcul at i ng l i ght chai ns .
Q. Glomerulonephritis in infective endocarditis Thi s di s eas e repres ent s t he prot ot ypi cal bact eri al i l l nes s t hat may l ead t o t he i nduct i on of gl omerul onephri t i s , pres umabl y t hrough an i mmune-compl ex mechani s m. Gl omerul onephri t i s i s t hought t o occur by s i mi l ar means i n vi s ceral abs ces s and i n i nfect i ons ari s i ng from
Pa g e 1 3 1 4
ABC Ambe r CHM Conve rte r Tria l ve rsion, http://w w w .proce sste x t.com/a bcchm.html
ext racorporeal ci rcul at i on devi ces (s hunt nephri t i s ). It i s pos s i bl e t hat any endovas cul ar i nfect i on can produce t hi s gl omerul onephri t i s .
1. Pathology. The hi s t ol ogi c s everi t y ranges from mi l d mes angi al prol i ferat i on t o s evere cres cent i c gl omerul onephri t i s .
2. Clinical features. About 15% of pat i ent s devel op renal i nvol vement . Noni mmune mechani s ms of i njury i ncl ude s ept i c embol i , i s chemi c ATN wi t h s evere CHF, and, i ndi rect l y, ant i bi ot i c nephrot oxi ci t y. Pat i ent s wi t h i mmune-compl ex gl omerul onephri t i s pres ent wi t h hemat uri a, RBC cas t s i n t he uri ne, az ot emi a, and hypocompl ement emi a.
3. T herapy. There i s no s peci fi c t reat ment for t he renal di s eas e. Except i n fai rl y advanced cas es , s ucces s ful t reat ment of t he underl yi ng i nfect i on us ual l y l eads t o res ol ut i on of t he renal di s eas e.
P.268
XI. Renal Cystic Disease A. Adult polycystic kidney disease
1. Definition, etiology, and incidence. Adul t pol ycys t i c ki dney di s eas e repres ent s t he mos t common caus e of renal fai l ure and deat h i n adul t s wi t h renal cys t i c di s eas e. It account s for approxi mat el y 5% of al l pat i ent s on mai nt enance di al ys i s .
Pa g e 1 3 1 5
ABC Ambe r CHM Conve rte r Tria l ve rsion, http://w w w .proce sste x t.com/a bcchm.html o
o
a. Inheri t ed as an aut os omal domi nant t rai t , adul t pol ycys t i c ki dney di s eas e achi eves 100% gene penet rance by t he t i me t he pat i ent i s 80 years of age. In t he majori t y of cas es , t he affect ed gene (pol ycys t i n-1) i s on chromos ome 16 (PKD1); however, i n a mi nori t y of fami l i es , t he genet i c defect (pol ycys t i n-2) res i des on chromos ome 4 (PKD2).
o
o
b. Men and women are affect ed equal l y. Fami l y hi s t ory i s pos i t i ve i n more t han 75% of cas es .
o
o
c. Adul t pol ycys t i c ki dney di s eas e mus t be di s t i ngui s hed from childhood (autosomal recessive) polycystic kidney disease (whi ch i s uni vers al l y fat al by t he t hi rd decade of l i fe) and t he congenital multicystic variant of renal dysplasia.
2. Clinical features o
o
a. The t ypi cal pres ent at i on of enl argi ng fl ank or abdomi nal mas s es , abdomi nal pai n, and s l owl y progres s i ve renal fai l ure becomes cl i ni cal l y evi dent by t he fourt h decade of l i fe, and renal repl acement t herapy becomes neces s ary wi t hi n 10 years of t he ons et of s ympt oms . As s oci at ed cl i ni cal fi ndi ngs may i ncl ude hypert ens i on, pol yuri a and noct uri a, eryt hrocyt os i s , and nephrol i t hi as i s .
o
Pa g e 1 3 1 6
ABC Ambe r CHM Conve rte r Tria l ve rsion, http://w w w .proce sste x t.com/a bcchm.html
o
b. The ons et of renal fai l ure occurs l at er i n t he mi nori t y of pat i ent s who have t he genet i c defect on chromos ome 4 (about 67 years as oppos ed t o about 54 years ) compared wi t h t he majori t y, who have t he defect on chromos ome 16.
o
o
c. Hepat i c cys t s occur i n 33% of cas es . However, l i ver i ns uffi ci ency i s rare, unl i ke i n t he chi l dhood form.
o
o
d. Int racrani al (berry) aneurys ms occur i n 12% of pat i ent s . In s ome s eri es , 6% of al l pat i ent s wi t h berry aneurys ms have adul t pol ycys t i c ki dney di s eas e.
3. Diagnosis. Di agnos i s i s made mos t eas i l y on t he bas i s of s peci fi c ul t ras onographi c fi ndi ngs . Coi nci dent fi ndi ngs i ncl ude hemat uri a, i mpai red uri ne concent rat i ng abi l i t y, and l ow-grade prot ei nuri a. Heavy prot ei nuri a, pers i s t ent hemat uri a, and pyuri a s houl d be i nves t i gat ed becaus e t hey rarel y occur i n uncompl i cat ed adul t pol ycys t i c ki dney di s eas e.
4. T herapy. Treat ment i s res t ri ct ed t o t he t herapy of end-s t age renal di s eas e as i t devel ops . Genet i c couns el i ng i s i mport ant , becaus e 50% of offs pri ng are affect ed. W omen wi t h adul t pol ycys t i c ki dney di s eas e are not at an i ncreas ed ri s k for fet al demi s e or hypert ens i on duri ng pregnancy except as cont ri but ed by exi s t i ng renal i ns uffi ci ency.
Pa g e 1 3 1 7
ABC Ambe r CHM Conve rte r Tria l ve rsion, http://w w w .proce sste x t.com/a bcchm.html
B. Nephronophthisis (medullary cystic disease)
1. Definition and incidence. Medul l ary cys t i c di s eas e i s t he mos t common caus e of end-s t age renal di s eas e i n chi l dren and adol es cent s .
2. Pathology. The ki dney i s s mal l i n medul l ary cys t i c di s eas e, whi ch di s t i ngui s hes t hi s di s eas e from pol ycys t i c and mul t i cys t i c di s eas es . Cys t s may be l ocat ed at t he cort i comedul l ary junct i on or i n t he medul l a. Acys t i c forms have been des cri bed. Int ers t i t i al fi bros i s i s promi nent , but cal ci fi cat i on does not occur.
3. Clinical features. Los s of uri ne concent rat i ng abi l i t y and fai l ure t o cons erve s odi um appropri at el y are al mos t i nvari abl e earl y s i gns of di s eas e. The i ni t i al pres ent at i on oft en i ncl udes pol yuri a, pol ydi ps i a, and enures i s , al t hough an az ot emi c pres ent at i on al s o i s common. Progres s i on t o renal fai l ure i s a cons t ant cl i ni cal feat ure.
4. T herapy. Treat ment i s ai med at mai nt ai ni ng s odi um and wat er homeos t as i s duri ng t he evol ut i on of di s eas e. Genet i c couns el i ng may be appropri at e i n di s eas e t hat i s cl earl y fami l i al . The pos s i bi l i t y of s ubcl i ni cal di s eas e i n s i bl i ngs mus t be cons i dered when a donor i s bei ng s el ect ed for t rans pl ant at i on.
C. Medullary sponge kidney
Pa g e 1 3 1 8
ABC Ambe r CHM Conve rte r Tria l ve rsion, http://w w w .proce sste x t.com/a bcchm.html
1. Definition and incidence. Medul l ary s ponge ki dney i s not a t rue cys t i c di s eas e but rat her an ect as i a of t he renal col l ect i ng t ubul e. Thi s common probl em i s i dent i fi ed i n 1 of every 200 urograms i n a l arge s eri es .
2. Prognosis. Out come i s excel l ent ; many pat i ent s have no det ect abl e i mpai rment of renal funct i on. Hypercal ci uri a i s common, as are s ubt l e defect s i n t he abi l i t y t o concent rat e and aci di fy t he uri ne. Nephrocal ci nos i s and nephrol i t hi as i s , of vari abl e s everi t y, are found i n 50% of cas es .
D. Simple renal cyst
1. Definition and incidence. The s i mpl e cys t i s t he mos t common renal cys t i c di s eas e; at l eas t one-hal f of al l i ndi vi dual s ol der t han 50 years of age have one or more macros copi c renal cys t s . Renal cys t s may be s ol i t ary or mul t i pl e and uni l at eral or bi l at eral .
2. Clinical features. Sympt oms are rare; s i mpl e cys t s us ual l y are di agnos ed duri ng pat i ent eval uat i on for ot her probl ems . Bl eedi ng and i nfect i on s t i mul at e t he cys t wal l t o t hi cken, and cal careous pl aques oft en form wi t hi n t he cys t wal l .
3. Clinical course. Si mpl e cys t s us ual l y are s t at i c, al t hough regres s i on may occur from one radi ographi c
Pa g e 1 3 1 9
ABC Ambe r CHM Conve rte r Tria l ve rsion, http://w w w .proce sste x t.com/a bcchm.html
as s es s ment t o t he next . Sol i t ary cys t s may undergo mal i gnant degenerat i on, al t hough t hi s fi ndi ng i s rare. Hemorrhagi c cys t s are more l i kel y t o cont ai n a neopl as m t han nonhemorrhagi c cys t s (i .e., i n up t o 30% of cas es as compared wi t h l es s t han 1% of cas es ). Multiple simple cysts may devel op i n end-s t age renal di s eas e i n pat i ent s who have undergone hemodial ys i s for l onger t han 7 years .
4. Diagnosis may be made by CT, ul t ras onography, urography, or angi ography. Cys t punct ure (for fl ui d as pi rat i on and cyt ol ogy) and cont ras t radi ol ogy s houl d be performed i n pat i ent s wi t h l arge cys t s wi t h abnormal ul t ras onographi c appearance. P.269
5. T herapy. In t he abs ence of i nfect i on or t umor, no s peci fi c t herapy i s i ndi cat ed for t hi s beni gn di s eas e.
XII. Tubulointerstitial Disease A. Acute interstitial nephritis
1. Definition. Acut e i nt ers t i t i al nephri t i s appears t o be a ki dney-bas ed hypers ens i t i vi t y react i on, us ual l y caus ed by a drug. Al t hough t he t rue i nci dence of acut e i nt ers t i t i al nephri t i s i s unknown, s everal hundred cas es have been formal l y report ed, and an i ncreas i ng awarenes s of t hi s di s eas e has come wi t h i ncreas ed cas e recogni t i on.
Pa g e 1 3 2 0
ABC Ambe r CHM Conve rte r Tria l ve rsion, http://w w w .proce sste x t.com/a bcchm.html
2. Etiology. Drugs i mpl i cat ed i n t he pat hogenes i s of acut e i nt ers t i t i al nephri t i s i ncl ude β-l act am ant i bi ot i cs (e.g., met hi ci l l i n, oxaci l l i n, and cephal ot hi n) and ot her ant i bi ot i cs (e.g., s ul fonami des ), NSAIDs (e.g., i buprofen, i ndomet haci n, fenoprofen, and t ol met i n), di uret i cs (e.g., t hi azi des and furos emi de), and many ot her unrel at ed drugs (e.g., phenyt oi n, ci met i di ne, s ul fi npyraz one, met hyl dopa, and phenobarbi t al ).
3. Clinical features. The classic pres ent at i on i s devel opment of acut e renal fai l ure wi t h fever, ras h, and eos i nophi l i a, yet onl y a mi nori t y of pat i ent s pres ent wi t h t hi s s ympt om t ri ad.
4. Diagnosis o
o
a. Urinalysis cl as s i cal l y s hows mi l d or no prot ei nuri a, mi cros copi c hemat uri a, pyuri a, and eos i nophi l uri a. Uri ne mus t be exami ned mi cros copi cal l y wi t h appropri at e s t ai ni ng met hods for t he pres ence of eos i nophi l uri a. Some pat i ent s wi t h acut e i nt ers t i t i al nephri t i s due t o NSAIDs pres ent wi t h nephrot i c s yndrome charact eri zed by uri nary prot ei n excret i on exceedi ng 3.0 g/24 hours .
o
o
b. Biopsy s hows pat chy, i rregul ar i nt ers t i t i al i nfi l t rat i on wi t h i nfl ammat ory cel l s . Monocyt es and l ymphocyt es are cons t ant fi ndi ngs . Eos i nophi l s may be abundant or compl et el y abs ent . Fi bros i s i s ext remel y unus ual and s houl d s ugges t underl yi ng or preexi s t i ng renal di s eas e. Rarel y, acut e i nt ers t i t i al nephri t i s may progres s t o chroni c i nt ers t i t i al nephri t i s , and fi bros i s may be
Pa g e 1 3 2 1
ABC Ambe r CHM Conve rte r Tria l ve rsion, http://w w w .proce sste x t.com/a bcchm.html
promi nent . Gl omerul i are normal or s how onl y mi l d mes angi al promi nence.
5. T herapy. Treat ment i ncl udes di s cont i nuat i on of t he et i ol ogi c drug and i ni t i at i on of s upport i ve meas ures (e.g., di et ary res t ri ct i ons , bl ood pres s ure management , and acut e di al ys i s ). The val ue of gl ucocort i coi d t herapy i s uncl ear; however, t he us e of s t eroi ds may be jus t i fi ed i n pat i ent s wi t h s evere or rapi dl y progres s i ve renal i ns uffi ci ency. In s uch pat i ent s , t he addi t i on of i mmunos uppres s i ve agent s s uch as cycl ophos phami de i s warrant ed. W hen i nt ers t i t i al nephri t i s i s as s oci at ed wi t h ci rcul at i ng ant i bodi es (a rare occurrence), t he addi t i on of pl as mapheres i s i s i ndi cat ed.
6. Prognosis. Out come i s excel l ent provi ded t hat t he offendi ng drug i s prompt l y wi t hdrawn. Recovery t i me vari es and may be prol onged i n pat i ent s wi t h ol i guri a and i n t hos e wi t h ext ens i ve i nt ers t i t i al cel l ul ar i nfi l t rat es . Temporary di al ys i s may be needed. Rarel y, pat i ent s progres s t o end-s t age renal di s eas e.
B. Chronic interstitial nephritis In general , t he cl i ni cal feat ures common t o t hes e i nt ers t i t i al di s eas es i ncl ude a rel at i ve pres ervat i on of gl omerul ar funct i on unt i l l at e i n t he di s eas e but an i mpai rment of t ubul ar funct i ons (e.g., uri ne concent rat i on, di l ut i on, aci di fi cat i on, and pot as s i um excret i on) earl y i n t he cours e of t he di s eas e.
1. Drug-related nephropathy o
o
a. Analgesic nephropathy i s t he prot ot ypi cal drug-rel at ed chroni c i nt ers t i t i al nephri t i s .
Pa g e 1 3 2 2
ABC Ambe r CHM Conve rte r Tria l ve rsion, http://w w w .proce sste x t.com/a bcchm.html
(1) Anal ges i c nephropat hy occurs more commonl y i n women t han i n men. Pat i ent s us ual l y are ol der t han 45 years of age and from l ow s oci oeconomi c cl as s es . Pat i ent s oft en compl ai n of frequent headaches or have coi nci dent ps ychi at ri c di s eas e.
(2) Int ravenous urography reveal s abnormal i t y i n more t han 90% of cas es , and papi l l ary necros i s i s s een i n more t han 50%. Hal f of t he pat i ent s are hypert ens i ve, and anemi a i s common and oft en out of proport i on t o t he degree of cl i ni cal l y apparent renal di s eas e.
(3) Several agent s have been i mpl i cat ed (e.g., acet ami nophen, phenacet i n, and as pi ri n), but none has been s peci fi cal l y proven cul pabl e. The ri s k of anal ges i c nephropat hy appears t o be i ncreas ed i n pat i ent s who us e more t han 3 g/day of s uch agent s . P.270
TABLE 6-13 Mechanisms for NSAID-Induced Disorders
Pa g e 1 3 2 3
ABC Ambe r CHM Conve rte r Tria l ve rsion, http://w w w .proce sste x t.com/a bcchm.html
NS AI D-I nd uc ed Dis Me or ch de ani r sm Ne Sev phr ere ot i i nt c
ers
s yn t i t i dro al me ne phr itis ; hi s t ol ogi cal l y nor ma l gl o me rul i De Re cre nal as e vas d
oco
Pa g e 1 3 2 4
ABC Ambe r CHM Conve rte r Tria l ve rsion, http://w w w .proce sste x t.com/a bcchm.html
GF ns t R
ri ct i on , es p eci al l y in pat i en ts wi t h pre exi sti ng ren al di s eas e, con ges tiv e he art fai l ure , or ci rr hos is;
Pa g e 1 3 2 5
ABC Ambe r CHM Conve rte r Tria l ve rsion, http://w w w .proce sste x t.com/a bcchm.html
pat i en ts t re at e d wi t h t ri a mt ere ne at par tic ul a r ri s k Pa Un pi l l kno ary wn nec ros is Ed Pri em ma a
ry ren al s od iu m ret ent
Pa g e 1 3 2 6
ABC Ambe r CHM Conve rte r Tria l ve rsion, http://w w w .proce sste x t.com/a bcchm.html
i on du e to pro sta gl a ndi n i nh i bi t i on , es p eci al l y in pat i en ts wi t h un der l yi ng con ges tiv e he art fai l ure
Pa g e 1 3 2 7
ABC Ambe r CHM Conve rte r Tria l ve rsion, http://w w w .proce sste x t.com/a bcchm.html
Hy Hy per por kal eni em ne ia
mi c hyp oal dos t er oni
sm GFR, gl omer ul ar fi l t rat i on rat e; NSAID, nons t e roi dal ant i -i n fl amm at ory drug.
(4) Treat ment of progres s i ve anal ges i c nephropat hy i s s upport i ve. Removal of t he i nci t i ng agent may arres t t he det eri orat i on of renal funct i on.
(5) NSAIDs can al s o be as s oci at ed wi t h ot her renal di s orders (Tabl e 6-13).
o
Pa g e 1 3 2 8
ABC Ambe r CHM Conve rte r Tria l ve rsion, http://w w w .proce sste x t.com/a bcchm.html o
b. Gold nephropathy i s a frequent and i mport ant compl i cat i on of parent eral gol d t herapy for rheumat oi d art hri t i s . Gol d accumul at i on l eads t o i mmune-compl ex membranous gl omerul onephri t i s and nephrot i c s yndrome. Ces s at i on of gol d t herapy at t he fi rs t s i gn of prot ei nuri a i s recommended and oft en res ul t s i n regres s i on of s i gns of renal di s eas e. It i s not yet known whet her oral gol d preparat i ons are equal l y nephrot oxi c.
o
o
c. Lithium nephrotoxicity may be i mport ant . Li t hi um carbonat e, us ed i n t he t reat ment of bi pol ar di s order, i s fi l t ered freel y and undergoes s i gni fi cant (i .e., 60%–70%) reabs orpt i on i n t he proxi mal t ubul es . Li t hi um t oxi ci t y res ul t s i n ant i di uret i c hormone (ADH)–unres pons i ve nephrogeni c di abet es i ns i pi dus , i ncompl et e di s t al renal t ubul ar aci dos i s , and, rarel y, az ot emi a.
2. T oxin-related nephropathy o
o
a. Lead nephropathy (saturnine gout) i s a wel l -recogni zed s equel a of chroni c l ead i nt oxi cat i on.
(1) The earl i es t cas es of l ead i nt oxi cat i on i nvol ved mi ners , pai nt manufact urers , and di s t i l l ers of “moons hi ne― l i quor. Lead poi s oni ng al s o has been report ed i n chi l dren who have i nges t ed l ead-bas ed pai nt . Indi vi dual s who recover from acut e l ead poi s oni ng occas i onal l y are found l at er t o be vi ct i ms of chroni c l ead–rel at ed renal
Pa g e 1 3 2 9
ABC Ambe r CHM Conve rte r Tria l ve rsion, http://w w w .proce sste x t.com/a bcchm.html
di s eas e.
(2) Cl i ni cal mani fes t at i ons of l ead nephropat hy i ncl ude a reduced GFR, reduced renal pl as ma fl ow (RPF), mi ni mal or no prot ei nuri a, normal uri nary s edi ment , gout y art hri t i s due t o hyperuri cemi a and l ow urat e cl earance, and, occas i onal l y, hypert ens i on, hyperkal emi a, and aci demi a.
(3) Treat ment i ncl udes removal of l ead expos ure and chel at i on t herapy wi t h s odi um or cal ci um ethylenediaminetetraacetic acid (EDT A) or D-penicillamine (i n appropri at e cas es ).
o
o
b. Cadmium nephropathy may l ead t o i nt ers t i t i al di s eas e. Cadmi um, a hi ghl y t oxi c byproduct of zi nc product i on, has numerous i ndus t ri al appl i cat i ons . Duri ng l ong-t erm expos ure, cadmi um accumul at es i n t he ki dney but bl ood and uri ne cadmi um concent rat i ons remai n normal . Cadmi um probabl y l eads t o end-s t age renal di s eas e, al t hough t he t rue i nci dence i s unknown.
o
o
c. Copper nephrotoxicity i s rare but occas i onal l y i s s een i n W i l s on's di s eas e. Cl i ni cal l y, copper nephropat hy may res embl e cadmi um nephropat hy (proxi mal t ubul ar di s eas e) or ATN. T rientine and D-penicillamine (when neces s ary) are t reat ment s of choi ce for copper chel at i on. Zinc acetate (Gal zi n or over-t he-count er forms of zi nc) are us ed for
Pa g e 1 3 3 0
ABC Ambe r CHM Conve rte r Tria l ve rsion, http://w w w .proce sste x t.com/a bcchm.html
mai nt enance or pres ympt omat i c pat i ent s . o
o
d. Mercury nephropathy may al s o l ead t o di s eas e. Mercury i s as s oci at ed wi t h s everal renal l es i ons , i ncl udi ng membranous and prol i ferat i ve gl omerul ar di s eas e wi t h nephrot i c s yndrome, proxi mal t ubul ar at rophy and Fanconi 's s yndrome wi t h t he devel opment of chroni c renal fai l ure, and ol i guri c ATN. Chel at i on t herapy wi t h British antilewisite (BAL, dimercaprol) and hemodi al ys i s may reduce mort al i t y i f i ni t i at ed prompt l y (i .e., wi t hi n 48 hours fol l owi ng expos ure).
3. Crystalline nephropathy o
o
a. Uric acid produces renal i njury i n t hree ways .
(1) Uric acid stones may devel op i n concent rat ed aci d uri ne.
(2) Acute uric acid nephropathy (acut e crys t al l i ne obs t ruct i on of renal t ubul es ) may accompany s udden or ext reme el evat i ons i n s erum uri c aci d (i .e., s erum l evel s >25 mg/dL), as occurs i n tumor lysis syndrome.
(3) Gouty nephropathy, a s yndrome of i nt ers t i t i al fi bros i s and decreas ed renal funct i on, may be rel at ed t o cort i cal mi crot ophi and a nephrot oxi c i nfl uence of hyperuri cemi a i n s ome gout y pat i ent s . Lead nephropat hy
Pa g e 1 3 3 1
ABC Ambe r CHM Conve rte r Tria l ve rsion, http://w w w .proce sste x t.com/a bcchm.html
(s at urni ne gout ) may account for a s i gni fi cant percent age of pat i ent s wi t h renal i ns uffi ci ency and gout . o
o
b. Oxalic acid al s o produces t ubul oi nt ers t i t i al di s eas e. El evat ed uri ne l evel s of oxal i c aci d may l ead t o t he format i on of cal ci um oxal at e s t ones or may mi mi c t he s yndrome of acut e uri c aci d P.271
nephropat hy (acut e crys t al l i ne obs t ruct i on). Primary hyperoxaluria i s an i nheri t ed di s eas e of oxal at e overproduct i on, whi ch t ermi nat es i n renal fai l ure wi t h ext ens i ve depos i t i on of oxal at e crys t al s t hroughout t he body (a condi t i on t ermed oxalosis). Ethylene glycol poisoning may l ead t o renal fai l ure i n part by t he hyperoxal uri a t hat res ul t s from t he met abol i s m of et hyl ene gl ycol t o oxal at e. The us e of methoxyflurane i n anes t hes i a has been l i nked t o an i ncreas ed oxal at e product i on wi t h res ul t ant nephrot oxi ci t y. Increas ed oxal at e abs orpt i on oft en i s s een fol l owi ng i l eojejunal bypas s s urgery for obes i t y and may l ead t o nephrocal ci nos i s . Mi l d hyperoxal uri a may res ul t from pyri doxi ne or t hi ami ne defi ci ency. o
o
c. Antiviral agents. Hi gh-dos e acyclovir and ganciclovir may l ead t o i nt rat ubul ar crys t al depos i t i on and acut e renal fai l ure. Indinavir, an ant i ret rovi ral agent , may al s o produce t hi s l es i on, becaus e i t s crys t al s are hi ghl y i ns ol ubl e. Hi gh-dos e methotrexate may i nduce i nt rat ubul ar crys t al depos i t i on as wel l , and hi gh-dos e sulfadiazine may l ead t o a s i mi l ar compl i cat i on
Pa g e 1 3 3 2
ABC Ambe r CHM Conve rte r Tria l ve rsion, http://w w w .proce sste x t.com/a bcchm.html
when uri ne i s part i cul arl y aci di c.
4. Miscellaneous nephropathies o
o
a. Amyloidosis (online Figure 6-9)
ONLINE FIGURE 6-9. Congo red-s t ai ned s ect i on s howi ng green bi refri ngence i n a pat i ent wi t h AA amyl oi d. (From W i l l i ams G, Mal l i ck NP. Col or At l as of Renal Di s eas es . 2nd ed. Barcel ona: W ol fe Publ i s hi ng/Mos by-Y ear Book Europe Lt d, 1994:4.78. )
1) Definition and classification. Amyl oi dos i s i s a di s order of unknown et i ol ogy, whi ch i nvol ves t he depos i t i on of eos i nophi l i c, amorphous mat eri al . The major cl as s i fi cat i ons
Pa g e 1 3 3 3
ABC Ambe r CHM Conve rte r Tria l ve rsion, http://w w w .proce sste x t.com/a bcchm.html
of amyl oi dos i s are primary amyloidosis (occurri ng wi t hout pre- or coexi s t i ng i l l nes s ), secondary amyloidosis (occurri ng i n t he pres ence of chroni c i nfl ammat ory di s eas e), and heredofamilial amyloidosis.
(2) Clinical features. Amyl oi d depos i t i on may be focal and res t ri ct ed t o t he ki dneys or s ys t emi c and general i zed. Prot ei nuri a i s uni vers al , wi t h nephrot i c s yndrome devel opi ng i n 76% of pat i ent s . Hypert ens i on occurs i n 50% of pat i ent s . Ki dney s i ze occas i onal l y i s i ncreas ed but decreas es wi t h advanced di s eas e.
(3) Clinical course. Progres s i ve det eri orat i on of renal funct i on i s t he rul e.
(4) Diagnosis. Bi ops y t hat demons t rat es amyl oi d prot ei n by green bi refri ngence wi t h Congo red s t ai n unequi vocal l y es t abl i s hes t he di agnos i s (s ee onl i ne Fi gure 6-9).
(5) T herapy. There i s no effect i ve t reat ment for pri mary amyl oi dos i s , al t hough alkylating agents and colchicine have been advocat ed. Treat ment of s econdary amyl oi dos i s i s l i mi t ed t o t he t reat ment of t he underl yi ng i nfl ammat ory di s eas e. Amyl oi dos i s recurs i n t he t rans pl ant ed ki dney.
o
o
b. Sarcoidosis (s ee al s o Chapt er 2 XIV A)
Pa g e 1 3 3 4
ABC Ambe r CHM Conve rte r Tria l ve rsion, http://w w w .proce sste x t.com/a bcchm.html
(1) Definition. Sarcoi dos i s i s a granul omat ous di s eas e of unknown et i ol ogy. Renal i nvol vement may be s econdary t o noncas eat i ng granul omat ous repl acement of t he renal i nt ers t i t i um.
(2) Clinical features. Renal s i ze us ual l y i s normal , and mi l d nons el ect i ve prot ei nuri a i s common. Hypercal cemi a, hypercal ci uri a, or bot h frequent l y compl i cat e s arcoi dos i s as a res ul t of i ncreas ed s ynt hes i s of 1,25-di hydroxychol ecal ci ferol [1,25-(OH) 2 D 3 ].
(3) Clinical course. The hypercal cemi a may i nduce acut e renal fai l ure, and t he hypercal ci uri a may l ead t o nephrocal ci nos i s or cal ci um nephrol i t hi as i s . Hypergl obul i nemi a, when pres ent , may be as s oci at ed wi t h di s t al renal t ubul ar aci dos i s . Al t hough gl omerul onephri t i s has been not ed among pat i ent s wi t h s arcoi dos i s , t he exi s t ence of a t rue s arcoi d gl omerul opat hy i s not ful l y es t abl i s hed.
(4) T herapy. St eroi ds are i ndi cat ed for t he management of hypercal cemi a.
C. Renal papillary necrosis
1. Definition. Renal papi l l ary necros i s res ul t s from i s chemi c necros i s of t he renal medul l a or renal papi l l ae.
Pa g e 1 3 3 5
ABC Ambe r CHM Conve rte r Tria l ve rsion, http://w w w .proce sste x t.com/a bcchm.html
There are t wo forms . The papi l l ary form i nvol ves t he ent i re papi l l a, whereas t he medul l ary form begi ns wi t h focal areas of i nfarct i on i n t he i nner medul l ary zone.
2. Etiologic factors. Condi t i ons as s oci at ed wi t h renal papi l l ary necros i s i ncl ude di abet es mel l i t us , uri nary t ract obs t ruct i on, s evere pyel onephri t i s , anal ges i c abus e, s i ckl e cel l hemogl obi nopat hy, ext reme hypoxi a and i nt ravas cul ar vol ume depl et i on i n i nfant s , and renal al l ograft reject i on.
3. Clinical features. The cl i ni cal pres ent at i on of renal papi l l ary necros i s vari es wi t h t he s t age and ext ent of di s eas e. Pat i ent s wi t h s i ckl e cel l t rai t may have compl et el y as ympt omat i c renal papi l l ary necros i s , whi ch i s di s covered i nci dent al l y duri ng urography for unrel at ed compl ai nt s . Infect i on frequent l y compl i cat es renal papi l l ary necros i s and l eads t o cl i ni cal pyel onephri t i s . The necrot i c papi l l ae may be s l oughed and produce t ypi cal uret eral col i c or uret eral obs t ruct i on. Azot emi a i s an uncommon pres ent i ng s i gn.
4. Clinical course. The cours e of renal papi l l ary necros i s i s a funct i on of t he underl yi ng di s eas e. End-s t age renal di s eas e may devel op, part i cul arl y among di abet i cs .
5. Diagnosis. Int ravenous urography can es t abl i s h t he di agnos i s of bot h forms of renal papi l l ary necros i s . Radi ographi cal l y, t he affect ed cal yces appear i rregul ar and fuz zy earl y i n t he di s eas e proces s . As t he l es i on progres s es , s eques t rat i on of t he necrot i c t i s s ue l eads t o s i nus format i on and t he appearance of a s i nus t ract or arc s hadow on t he urogram. In advanced renal papi l l ary
Pa g e 1 3 3 6
ABC Ambe r CHM Conve rte r Tria l ve rsion, http://w w w .proce sste x t.com/a bcchm.html
necros i s , t he s eques t rum may be s l oughed and s urrounded by cont ras t mat eri al —t he s o-cal l ed ri ng s i gn. Cal ci fi cat i on, cal i cect as i s , and medul l ary cavi t i es may be pres ent i n t hi s s t age.
6. T herapy. Treat ment i ncl udes rel i ef of obs t ruct i on, prevent i on and prompt eradi cat i on of i nfect i on, and cont rol of pai n (col i c). Surgery occas i onal l y i s neces s ary t o cont rol hemorrhage or t o rel i eve obs t ruct i on.
XIII. Renal Transport Defects
A. Meliturias Exces s i ve quant i t i es of sugars may gai n acces s t o t he uri ne becaus e of an i ncreas e i n fi l t ered l oad (as i n t he hypergl ycemi a of di abet es mel l i t us ) or fai l ure of appropri at e reabs orpt i on i n t he nephron.
1. Primary renal glycosuria i s an aut os omal reces s i ve di s order recogni zed from bi rt h by cons t ant gl ycos uri a i n t he abs ence of abnormal carbohydrat e met abol i s m (i .e., hypergl ycemi a, ket os i s , and ot her mel i t uri as ). Two vari ant s are des cri bed. Bot h are as s oci at ed wi t h a reduct i on i n ei t her t he amount of or affi ni t y for t he renal t rans port prot ei n for gl ucos e.
2. Other meliturias are recognized. o
o
a. Essential pentosuria (L-xylulosuria), an aut os omal reces s i ve defect i n t he met abol i s m of gl ucuroni c aci d, affect s pri mari l y Jews .
Pa g e 1 3 3 7
ABC Ambe r CHM Conve rte r Tria l ve rsion, http://w w w .proce sste x t.com/a bcchm.html
L-Xyl ul os uri a occurs s econdary t o a defi ci ency of ni cot i nami de–adeni ne di nucl eot i de phos phat e (NADP)–l i nked xyl i t ol dehydrogenas e. o
o
b. Essential fructosuria al s o i s an aut os omal reces s i ve error of met abol i s m, whi ch i s a res ul t of defect i ve phos phofruct oki nas e act i vi t y.
B. Aminoacidurias
1. Cystinuria i s an aut os omal reces s i ve defect i n t he t rans port of cys t i ne, l ys i ne, orni t hi ne, and argi ni ne. The l ow s ol ubi l i t y of cys t i ne account s for t he s ympt oms and compl i cat i ons of cys t i nuri a i n t hat i t predi s pos es t o t he format i on of renal cys t i ne s t ones , whi ch may l ead t o renal fai l ure. Therapy cons i s t s of a l i fel ong al kal i ne di ures i s t o prevent s t one format i on. D-Peni ci l l ami ne i s requi red i n many cas es (s ee Part I: VII D 3 c).
2. Dibasic aminoaciduria i s a s el ect i ve defect i n l ys i ne, orni t hi ne, and argi ni ne (but not cys t i ne) t rans port . Aut os omal reces s i ve and domi nant forms are recogni zed. Sympt oms i ncl ude ami no aci d–i nduced di arrhea, mal nut ri t i on, hyperammonemi a, and growt h and ment al ret ardat i on. Therapy cons i s t s of a l ow-prot ei n di et .
3. Iminoglycinuria i s a beni gn, aut os omal reces s i ve di s order of prol i ne, hydroxyprol i ne, and gl yci ne t rans port .
Pa g e 1 3 3 8
ABC Ambe r CHM Conve rte r Tria l ve rsion, http://w w w .proce sste x t.com/a bcchm.html
4. Hartnup disease i s an aut os omal reces s i ve defect i n t he t rans port of neut ral α-ami no aci ds . Pat i ent s wi t h Hart nup di s eas e have a reduced abi l i t y t o convert t rypt ophan t o ni aci n, res ul t i ng i n pel l agra—a s yndrome charact eri zed by phot os ens i t i ve eryt hema, cerebel l ar at axi a, neurops ychi at ri c s ympt oms , and del i ri um. Therapy i s oral ni cot i nami de.
C. Fanconi's syndrome
1. Definition. Fanconi 's s yndrome refers t o a col l ect i on of proxi mal t ubul ar defect s , whi ch may exi s t i n varyi ng number and degree of s everi t y and may be i nheri t ed or acqui red.
2. Etiology o
o
a. Inherited causes i ncl ude cys t i nos i s , Lowe's s yndrome, W i l s on's di s eas e, t yros i nemi a, gal act os emi a, gl ycogenos i s , and fruct os e i nt ol erance. P.272
o
o
b. Acquired causes i ncl ude t rans pl ant dys funct i on, myel oma, Sjögren's s yndrome, hyperparat hyroi di s m, pot as s i um depl et i on, amyl oi dos i s , nephrot i c s yndrome, i nt ers t i t i al nephri t i s , heavy met al t oxi ci t y, and out dat ed t et racycl i ne. Ifosfamide, a cycl ophos phami de-rel at ed drug, can i nduce a
Pa g e 1 3 3 9
ABC Ambe r CHM Conve rte r Tria l ve rsion, http://w w w .proce sste x t.com/a bcchm.html
Fanconi -l i ke s yndrome when i t i s us ed as an ant i neopl as t i c agent . Gl ycos uri a and ami noaci duri a have been report ed i n vi rt ual l y al l chi l dren t reat ed wi t h t hi s agent . o
o
c. An idiopathic form of Fanconi 's s yndrome al s o exi s t s .
3. Clinical features and course o
o
a. Sympt oms and s i gns i ncl ude gl ycos uri a, ami noaci duri a, phos phat uri a, bi carbonat uri a, vas opres s i n-res i s t ant pol yuri a, ri cket s or os t eoporos i s , s hort s t at ure, and uremi a.
o
o
b. The nat ural hi s t ory of Fanconi 's s yndrome depends heavi l y on t he cours e and prognos i s of t he underl yi ng di s eas e or di s eas es .
4. T herapy. Treat ment i s des i gned t o repl ace l os t uri nary s ol ut es and t o correct t he underl yi ng di s eas e or di s eas es . Phos phat e, vi t ami n D, and bi carbonat e s houl d be gi ven when i ndi cat ed by l aborat ory and cl i ni cal dat a.
XIV. Renal Vascular Disease A. Ischemic nephropathy Occl us i ve di s eas e of t he renal art eri al s ys t em encompas s es a broad s pect rum of cl i ni cal s yndromes and pat hophys i ol ogy. Art eri al bl ood fl ow may be i nt errupt ed by i n s i t u t hrombos i s or by embol i s m from di s t ant endovas cul ar s i t es . Occl us i on may be s udden and compl et e or gradual , wi t h res ul t ant funct i onal renal art ery s t enos i s .
Pa g e 1 3 4 0
ABC Ambe r CHM Conve rte r Tria l ve rsion, http://w w w .proce sste x t.com/a bcchm.html
1. Etiology o
o
a. Renal arterial thrombosis may devel op s pont aneous l y i n t he cont ext of at heros cl eros i s , aneurys m, art eri t i s , hypercoagul abl e s t at es , s i ckl e cel l di s eas e, and t hrombot i c mi croangi opat hy. Thrombos i s al s o may devel op as a compl i cat i on of ext ernal t rauma, i ns t rument at i on wi t h angi ography cat het ers , art eri al s urgery, and renal al l ograft t rans pl ant at i on.
o
o
b. Renal arterial embolism may be caus ed by a cl ot , t umor fragment , or i nfect i ous coagul um.
(1) Cardiac conditions t hat may l ead t o renal art eri al embol i s m i ncl ude a di l at ed l eft at ri um, art i fi ci al heart val ves , myocardi al i nfarct i on, i nfect i ve endocardi t i s , marant i c endocardi t i s , and myxoma.
(2) Noncardiac conditions t hat may caus e renal art eri al embol i s m i ncl ude at heromat ous pl aques and paradoxi cal , fat , or t umor embol i s m.
o
o
c. Cholesterol emboli may occur s pont aneous l y or fol l ow vas cul ar s urgery or angi ography. Acut e renal fai l ure may be t he onl y mani fes t at i on of di s eas e, or t here may be as s oci at ed eos i nophi l uri a. Occas i onal l y, s ki n l es i ons s uch as pet echi ae or l i vedo ret i cul ari s may be s een. Pat hol ogy of t he
Pa g e 1 3 4 1
ABC Ambe r CHM Conve rte r Tria l ve rsion, http://w w w .proce sste x t.com/a bcchm.html
s ki n or ki dney reveal s chol es t erol cl eft s i n s mal l and medi um-s i zed ves s el s . There i s no s peci fi c t herapy, but t he di s order may s pont aneous l y regres s , l eavi ng t he pat i ent wi t h adequat e res i dual ki dney funct i on. o
o
d. Progressive renal atherosclerosis affect s a l arge number (perhaps >20%) of i ndi vi dual s wi t h end-s t age renal fai l ure; t hey have angi ographi cal l y s i gni fi cant renal art ery s t enos i s . St udi es are underway t o det ermi ne t he opt i mal s t rat egi es for i dent i fyi ng and t reat i ng t hes e pat i ent s . The ri s ks of cont ras t -dye–as s oci at ed acut e renal fai l ure, chol es t erol embol i zat i on, and acut e renal art ery di s s ect i on have prevent ed many cl i ni ci ans from s ugges t i ng an aggres s i ve approach i n mos t pat i ent s . Int ervent i on i s i ndi cat ed i n t hos e wi t h s evere, uncont rol l abl e hypert ens i on, and i n t hos e pat i ent s wi t h di ffus e vas cul ar di s eas e wi t h cl earl y progres s i ve renal i ns uffi ci ency.
2. Clinical features and diagnosis o
o
a. Acute, complete renal arterial occlusion us ual l y mani fes t s as fl ank pai n, hemat uri a, fever, naus ea, t i s s ue necros i s [as evi denced by el evat ed l act at e dehydrogenas e (LDH) and as part at e t rans ami nas e (AST)], and acut e renal fai l ure. The di agnos i s i s confi rmed us i ng radi onucl i de s canni ng or angi ography. Bi l at eral occl us i on and occl us i on of a s ol i t ary funct i oni ng ki dney produce s evere anuri c acut e renal fai l ure.
o
Pa g e 1 3 4 2
ABC Ambe r CHM Conve rte r Tria l ve rsion, http://w w w .proce sste x t.com/a bcchm.html
o
b. Chronic or segmental occlusion produces s ympt oms and s i gns commens urat e wi t h t he degree of i s chemi c damage i ncl udi ng progres s i ve renal fai l ure.
P.273
3. T herapy o
o
a. Therapy for renal art eri al t hrombos i s i s s urgi cal removal of t he cl ot t o res t ore renal bl ood fl ow. Bes t res ul t s are obt ai ned when t he operat i on i s conduct ed wi t hi n 48–72 hours fol l owi ng t he ons et of di s eas e.
o
o
b. Therapy for renal art eri al embol i s m, whi ch us ual l y i s di ffus e and i nvol ves l arge numbers of s mal l er art eri al branches , i s ant i coagul at i on wi t h hepari n and res ol ut i on of t he underl yi ng focus of embol i .
o
o
c. Therapy for i s chemi c nephropat hy may i nvol ve angi opl as t y, s t ent pl acement , or s urgi cal revas cul ari zat i on. W hi l e t he rol e of each of t hes e t herapi es cont i nues t o be expl ored, t here i s l i t t l e evi dence t hat ki dney funct i on can be s ubs t ant i al l y pres erved or t hat acut e renal i ns uffi ci ency due t o i s chemi c nephropat hy can be revers ed.
B. Renal vein thrombosis Obs t ruct i on of renal venous drai nage by a cl ot may be caus ed by
Pa g e 1 3 4 3
ABC Ambe r CHM Conve rte r Tria l ve rsion, http://w w w .proce sste x t.com/a bcchm.html
ext ens i on of cl ot s i n t he vena cava, i nvas i on of t he renal vei n by t umor, s evere dehydrat i on i n i nfant s , renal amyl oi dos i s , and cert ai n gl omerul ar di s eas es as s oci at ed wi t h nephrot i c s yndrome, part i cul arl y membranous gl omerul onephri t i s . Renal vei n t hrombos i s general l y i s not a caus e of gl omerul ar di s eas e. Renal vei n t hrombos i s may devel op duri ng nephros i s becaus e of l os s of ant i coagul ant prot ei ns and procoagul ant deact i vat ors i n t he uri ne prot ei n.
1. Clinical features and diagnosis. Sl owl y evol vi ng renal vei n t hrombos i s may be compl et el y as ympt omat i c, whereas acut e renal vei n t hrombos i s may produce pai n, hemat uri a, cos t overt ebral angl e t endernes s , and, ul t i mat el y, s i gns of wors eni ng renal funct i on. The affect ed ki dney appears t o be enl arged when vi s ual i zed wi t h t he ai d of i nt ravenous urography. Sel ect i ve venography i s di agnos t i c. New t echni ques s uch as Doppl er ul t ras onography may be part i cul arl y us eful as a noni nvas i ve approach t o di agnos i s .
2. T herapy. Treat ment i s cont rovers i al , as i s t he bel i ef t hat renal vei n t hrombos i s predi s pos es t o pul monary embol i s m. Current t herapy i s l ong-t erm (3–6 mont hs ) ant i coagul at i on wi t h warfari n s odi um or l ow-mol ecul ar-wei ght hepari ns . Longer t reat ment i s recommended i f embol i c phenomena occur.
C. Renal artery stenosis In experi ment al ani mal s , i t has been cl earl y s hown t hat part i al reduct i on i n t he l umi nal s i ze of one or bot h renal art eri es produces renovas cul ar hypert ens i on, whi ch i s medi at ed i n mos t cas es by i ncreas ed reni n product i on wi t h res ul t ant act i vat i on of angi ot ens i n and al dos t erone. In humans , renal art ery s t enos i s i s a recogni zed caus e of renovas cul ar hypert ens i on. However, t he coi nci dence of radi ographi cal l y demons t rat ed renal art ery s t enos i s and cl i ni cal l y
Pa g e 1 3 4 4
ABC Ambe r CHM Conve rte r Tria l ve rsion, http://w w w .proce sste x t.com/a bcchm.html
demons t rat ed renovas cul ar hypert ens i on does not es t abl i s h a caus al rel at i ons hi p.
1. Etiology o
o
a. Medial fibromuscular dysplasia occurs more commonl y i n young women.
o
o
b. Adul t s ol der t han 50 years of age have renal artery atherosclerosis, whi ch i s t wi ce as common i n men as i n women.
o
o
c. Rare caus es i ncl ude T akayasu' s arteritis, arterial wall disease (e.g., hemat oma, di s s ect i ng aneurys m, and t umor), and external arterial compression due t o t umor, fi bros i s , or cys t .
2. Clinical features. Si gns and s ympt oms i ncl ude a nearl y cont i nuous abdomi nal or fl ank brui t , hypokal emi a, mi l d met abol i c al kal os i s , and as ymmet ri c ki dney s i ze. None of t hes e i s a cons t ant fi ndi ng, and oft en t here i s no feat ure t o di s t i ngui s h renal art ery s t enos i s from es s ent i al hypert ens i on.
3. Diagnosis. Di agnos t i c s t rat egi es vary accordi ng t o cl i ni cal s us pi ci on. o
o
a. MRA, or rarel y, t radi t i onal angi ography i s t he met hod of choi ce for a defi ni t i ve di agnos i s . St enot i c s egment s are rel i abl y i dent i fi ed by t hi s
Pa g e 1 3 4 5
ABC Ambe r CHM Conve rte r Tria l ve rsion, http://w w w .proce sste x t.com/a bcchm.html
s t udy. o
o
b. Duplex ultrasonography al l ows noni nvas i ve det ermi nat i on of renal bl ood fl ow. Thi s procedure i s operat or-dependent and may not be t echni cal l y feas i bl e i n al l pat i ent s .
o
o
c. Renal vein renin studies. Angi ographi c proof t hat renal art ery s t enos i s i s et i ol ogi cal l y i mport ant i s di ffi cul t t o obt ai n. Fi ndi ng t hat t he renal vei n reni n from t he affect ed s i de i s 1.5 t i mes great er t han t hat from t he unaffect ed s i de i s hel pful , but pat i ent s may res pond t o t reat ment wi t hout t hi s bi ochemi cal fi ndi ng.
4. T herapy. Therapeut i c opt i ons are ant i hypert ens i ve drugs , percut aneous t rans l umi nal angi opl as t y (PCTA) wi t h or wi t hout s t ent i ng, and s urgi cal repai r of t he affect ed ves s el .
P.274
D. Microangiopathy: hemolytic–uremic syndrome and thrombotic thrombocytopenic purpura (TTP) As members of t he di s eas e group t ermed mi croangi opat hi c hemol yt i c anemi a, hemol yt i c–uremi c s yndrome and TTP are s i mi l ar cl i ni cal s yndromes t hat s hare feat ures wi t h DIC, mal i gnant hypert ens i on, pos t part um renal fai l ure, s eps i s , and s ys t emi c s cl eros i s .
Pa g e 1 3 4 6
ABC Ambe r CHM Conve rte r Tria l ve rsion, http://w w w .proce sste x t.com/a bcchm.html
1. Clinical features o
o
a. T T P charact eri s t i cal l y mani fes t s as fever, mi croangi opat hi c hemol yt i c anemi a, t hrombocyt openi a, fl uct uat i ng neurol ogi c s i gns , purpura, and renal fai l ure. Gas t roi nt es t i nal i nvol vement (e.g., mucos al bl eedi ng and jaundi ce) i s common. In s ome cas es , t he et i ol ogi c agent i s an aut oant i body di rect ed t oward a prot eas e, whi ch cl eaves von W i l l ebrand's fact or.
o
o
b. Hemolytic–uremic syndrome i s pri mari l y a pedi at ri c di s order charact eri zed by mi croangi opat hi c hemol yt i c anemi a, t hrombocyt openi a, and acut e renal fai l ure. The di s order may occur i n epi demi cs and has been report ed t o fol l ow s hi gel l os i s . Hemol yt i c–uremi c s yndrome us ual l y has a s udden and dramat i c ons et , wi t h renal fai l ure t he domi nant cl i ni cal feat ure. As i n TTP, ot her organ s ys t ems may be i nvol ved. Renal funct i on ret urns i n mos t pat i ent s who recover from s ys t emi c di s eas e, but rel aps es have been report ed.
2. Diagnosis o
o
a. Laboratory findings are s i mi l ar i n bot h di s orders .
(1) Anemi a i s a cons t ant fi ndi ng, occurri ng i n as s oci at i on wi t h a vari et y of s t ruct ural l y damaged RBCs i n t he peri pheral ci rcul at i on. The ret i cul ocyt e count , fi bri n s pl i t product s ,
Pa g e 1 3 4 7
ABC Ambe r CHM Conve rte r Tria l ve rsion, http://w w w .proce sste x t.com/a bcchm.html
and i ndi rect bi l i rubi n l evel s are el evat ed. Serum LDH i s al s o el evat ed and i s t he mos t us eful cl i ni cal marker for fol l owi ng di s eas e progres s . Leukocyt os i s i s common, and t hrombocyt openi a i s s evere (i .e., <20,000 3
pl at el et s /mm ). The bone marrow s hows eryt hroi d hyperpl as i a wi t h adequat e or i ncreas ed megakaryocyt es . Azot emi a i s common, and t he degree of renal fai l ure i s charact eri s t i cal l y s evere. Recent l y, mi croangi opat hi c hemol yt i c anemi a has been found i n as s oci at i on wi t h t he ant i phos phol i pi d ant i body s yndrome.
(2) Uri nal ys i s s hows hemat uri a, pyuri a, hemogl obi nuri a, and granul ar cys t s .
(3) Mi crobi ol ogi c s t udi es may reveal verot oxi n-produci ng E. c ol i (s erot ype O157:H7) as t he caus e, part i cul arl y i n epi demi cs .
o
o
b. Renal biopsy findings al s o are s i mi l ar. Art eri ol es and s mal l art eri es are occl uded by eos i nophi l i c, hyal i ne t hrombi cont ai ni ng fi bri n and pl at el et aggregat es , whi ch caus e i mpres s i ve vas cul ar di l at i on. Mi croi nfarct s commonl y occur, but wi t hout i nfl ammat ory i nfi l t rat es or s i gns of vas cul i t i s . Renal l es i ons are focal and al mos t compl et el y confi ned t o t he art eri al s i de.
3. T herapy. Cont rol l ed t ri al s compari ng i ndi vi dual t reat ment programs have not been performed.
Pa g e 1 3 4 8
ABC Ambe r CHM Conve rte r Tria l ve rsion, http://w w w .proce sste x t.com/a bcchm.html
Therapeut i c met hods current l y i n us e i ncl ude ant i pl at el et drugs (e.g., as pi ri n, s ul fi npyrazone, and di pyri damol e), gl ucocort i coi ds , exchange t rans fus i on, pl as mapheres i s , and, rarel y, s pl enect omy. Mos t recent t ri al s have s ugges t ed t hat pl as ma exchange t herapy gi ves t he bes t t herapeut i c out come.
4. Prognosis. Out come has i mproved wi t h modern t herapy. Unt reat ed TTP i s al mos t uni vers al l y fat al wi t hi n 1 year. However, wi t h t he i ns t i t ut i on of earl y t herapy, 1-year s urvi val rat es may exceed 80%. Les s t han 5% of pat i ent s wi t h hemol yt i c–uremi c s yndrome di e wi t hi n 1 mont h.
E. Systemic sclerosis (scleroderma) Thi s general i zed di s t urbance of connect i ve and vas cul ar t i s s ue l eads t o fi bros i s of t he affect ed t i s s ue. Sys t emi c s cl eros i s may be a very l ocal i zed di s eas e (cal l ed morphea) or a l et hal , s ys t emi c di s eas e. Renal i nvol vement i s a common caus e of morbi di t y and deat h. The i nci dence of renal i nvol vement i n s ys t emi c s cl eros i s i s not s ol i dl y es t abl i s hed but , bas ed on aut ops y s eri es , i s es t i mat ed t o range from 42%–80%. Cl i ni cal evi dence of renal i nvol vement (i .e., azot emi a, hypert ens i on, and act i ve uri nary s edi ment ) i s s een i n about 45% of s ys t emi c s cl eros i s pat i ent s . In one s t udy, s ubt l e vas cul ar and hemodynami c abnormal i t i es were s een i n 80% of pat i ent s .
1. Clinical features and course o
o
a. Acute renal disease occurs i n t he cont ext of rapi dl y accel erat i ng general i zed di s eas e act i vi t y, wi t h promi nent mal i gnant hypert ens i on. Thi s i s commonl y referred t o as “s cl eroderma cri s i s .― Renal fai l ure can ens ue preci pi t ous l y i f t he bl ood
Pa g e 1 3 4 9
ABC Ambe r CHM Conve rte r Tria l ve rsion, http://w w w .proce sste x t.com/a bcchm.html
pres s ure i s uncont rol l ed. Pat hol ogi cal l y, acut e renal di s eas e i s s i mi l ar t o ot her mi croangi opat hi c di s eas es . The i nt erl obul ar art eri es P.275
s how marked i nt i mal t hi ckeni ng and mucoi d prol i ferat i on, whi ch may l ead t o cort i cal necros i s . Gl omerul ar changes us ual l y are mi l d and nons peci fi c, oft en cons i s t i ng of onl y mes angi al promi nence. Int ers t i t i al edema wi t h s ome mononucl ear i nfi l t rat e i s common. Unl i ke i s ol at ed mal i gnant hypert ens i on, s ys t emi c s cl eros i s does not pri mari l y affect art eri ol es but does produce advent i t i al fi bros i s . o
o
b. Chronic renal disease may be pres ent i n pat i ent s wi t h s ys t emi c s cl eros i s and l i t t l e or no cl i ni cal s i gns of renal i nvol vement . Ki dney s i ze us ual l y i s normal , and t he earl i es t s i gn of di s eas e i s prot ei nuri a, whi ch i s not ed i n 30% of pat i ent s . Nephrot i c s yndrome i s rare, and hemat uri a, uri nary cas t s , and pyuri a us ual l y are abs ent . Hypert ens i on compl i cat es chroni c renal di s eas e frequent l y (i .e., i n 25%–50% of pat i ent s ) and i s a harbi nger of i mpendi ng det eri orat i on of renal funct i on. Renal fai l ure occas i onal l y devel ops i n s ys t emi c s cl eros i s pat i ent s who have nei t her prot ei nuri a nor hypert ens i on.
2. T herapy o
o
a. T reatment of acute renal disease i nvol ves , t o a l arge degree, t he cont rol of accel erat ed
Pa g e 1 3 5 0
ABC Ambe r CHM Conve rte r Tria l ve rsion, http://w w w .proce sste x t.com/a bcchm.html
hypert ens i on. ACE i nhi bi t ors , ARBs , mi noxi di l , and ni t roprus s i de may be requi red. However, recent dat a s ugges t t hat overl y aggres s i ve l oweri ng of di as t ol i c bl ood pres s ure t o l es s t han 85 mm Hg may act ual l y i ncreas e t he ri s k of acut e renal fai l ure i n pat i ent s wi t h s cl eroderma. o
o
b. T reatment of chronic renal disease i s l es s cl ear-cut . It i s not known whet her s ome vas oact i ve t herapy duri ng earl y, nonazot emi c, nonhypert ens i ve s t ages of t he di s eas e i s prot ect i ve. Si mi l arl y, t he rol e of t reat ment for pat i ent s wi t h abnormal renal bi ops y s peci mens but no cl i ni cal renal di s eas e i s uncl ear.
F. Sickle cell nephropathy
1. Pathology o
o
a. Si ckl e cel l t rai t and s i ckl e cel l di s eas e are as s oci at ed wi t h a vari et y of renal compl i cat i ons . The renal medul l a i s rel at i vel y anoxi c and hyperos mol ar—fact ors t hat favor eryt hrocyt e s i ckl i ng. Mos t damage occurs i n t he renal papi l l ae.
o
o
b. Medul l ary i nfarct i on res ul t i ng from occl uded (s i ckl ed) ves s el s produces a s pect rum of t ubul ar di s orders , i ncl udi ng i mpai red uri ne concent rat i on. Becaus e t he i njury i s l ocat ed i n t he renal papi l l ae, t hes e pat i ent s behave as t hough t hey have been papi l l ect omi zed. Papi l l ary necros i s al s o i s s een. Pat i ent s wi t h s i ckl e cel l t rai t are affect ed l es s
Pa g e 1 3 5 1
ABC Ambe r CHM Conve rte r Tria l ve rsion, http://w w w .proce sste x t.com/a bcchm.html
s everel y t han t hos e wi t h s i ckl e cel l di s eas e.
2. Clinical features o
o
a. Impai red s ecret i on of pot as s i um and hydrogen i on occurs , and a frequent bi ochemi cal fi ndi ng i s hyperkal emi a wi t h a hyperchl oremi c (normal ani on gap) met abol i c aci dos i s (s ee Part II: IV D 2).
o
o
b. Hemat uri a repres ent s t he mos t dramat i c of t he renal abnormal i t i es i n s i ckl e cel l di s eas e. Al t hough i t i s us ual l y s el f-l i mi t ed, l i fe-t hreat eni ng exs angui nat i on occurs i n rare cas es .
o
o
c. Gl omerul ar di s eas e, i ncl udi ng nephrot i c s yndrome, has been document ed i n s i ckl e cel l di s eas e. Membranoprol i ferat i ve-l i ke l es i ons have been report ed, as has t ypi cal membranous gl omerul onephri t i s .
3. Clinical course. Al t hough frequent l y t he GFR i s s upranormal earl y i n t he cours e of s i ckl e cel l nephropat hy, gradual det eri orat i on of renal funct i on i s common. Progres s i on t o end-s t age renal di s eas e occurs i n s ome cas es .
4. T herapy o
o
a. Careful fl ui d management t o mai nt ai n adequat e i nt ravas cul ar vol ume, bot h duri ng cri s es and at
Pa g e 1 3 5 2
ABC Ambe r CHM Conve rte r Tria l ve rsion, http://w w w .proce sste x t.com/a bcchm.html
ot her t i mes , cl earl y i s i mport ant . Vol ume depl et i on i s i njuri ous t o renal funct i on and i s more l i kel y t o occur becaus e of t he uri ne-concent rat i ng defect . o
o
b. Pat i ent s who are prone t o hyperkal emi a or aci dos i s s houl d be advi s ed t o reduce t hei r di et ary i nt ake of pot as s i um and prot ei n.
o
o
c. Hemodi al ys i s i s us eful and does not i ncreas e t he number or s everi t y of cri s es or al t er t he t rans fus i on requi rement i n mos t pat i ent s .
o
o
d. Ki dney al l ograft s are s us cept i bl e t o s i ckl e cel l damage.
G. Radiation nephritis Hi gh dos es of i oni zi ng radi at i on are des t ruct i ve t o t he ki dney and uri nary t ract . Del i very of at l eas t 2000 rad (radi at i on-abs orbed dos e) over a peri od of s everal weeks can i nduce di s eas e. Radi at i on-i nduced nephri t i s mos t commonl y res ul t s from i nadvert ent expos ure duri ng radi ot herapy for abdomi nal or ret roperi t oneal t umor.
1. Immediate radiation nephrotoxicity res ul t s i n decreas ed renal bl ood fl ow, wi t h t ubul ar funct i on and bl ood pres s ure remai ni ng normal .
2. Acute radiation nephritis devel ops 6–12 mont hs aft er expos ure. Cl i ni cal s i gns are edema, hypert ens i on, headache, exert i onal dys pnea, anemi a, cyl i ndruri a,
Pa g e 1 3 5 3
ABC Ambe r CHM Conve rte r Tria l ve rsion, http://w w w .proce sste x t.com/a bcchm.html
prot ei nuri a, and mi cros copi c hemat uri a. Deat h occurs i n nearl y 50% of pat i ent s as a res ul t of s evere azot emi a and hypert ens i on. Pat i ent s who recover from t he acut e phas e may have s ome pers i s t ent prot ei nuri a.
3. Chronic radiation nephritis may fol l ow acut e radi at i on nephri t i s or devel op anew up t o 10 years fol l owi ng expos ure. Cl i ni cal s i gns are fai rl y nons peci fi c and i ncl ude fat i gue, noct uri a, hypert ens i on, hyperuri cemi a wi t h cl i ni cal gout , uremi a, anemi a, prot ei nuri a, cyl i ndruri a, and hypos t henuri a.
P.276
XV. The Kidney in Pregnancy
A. General physiologic effects
1. Under t he hormonal i nfl uence of pregnancy, renal s i ze i ncreas es by 1 cm or more (radi ographi cal l y); t he renal pel vi s , cal yces , and uret ers di l at e, as i n hydronephros i s ; t he GFR and RPF i ncreas e 25%–40%; a pri mary res pi rat ory al kal os i s devel ops ; and t he os mos t at res et s downward. Uri c aci d cl earance nearl y doubl es and renal excret i on of gl ucos e i ncreas es , whereas bl ood gl ucos e l evel s remai n normal .
2. Cl i ni cal l y, t he enl argement of t he col l ect i ng s ys t em s houl d not be mi s t aken for obs t ruct i on. The s erum
Pa g e 1 3 5 4
ABC Ambe r CHM Conve rte r Tria l ve rsion, http://w w w .proce sste x t.com/a bcchm.html
creat i ni ne and BUN val ues decreas e t o l es s t han 0.8 mg/dL and 13 mg/dL, res pect i vel y. The s erum bi carbonat e concent rat i on i s 4–5 mEq/L l ower t han i n t he pregravi d s t at e, and t he s erum os mol al i t y i s 10 mOs m/kg l ower, wi t h a corres pondi ng drop of 5 mEq/L i n s erum s odi um concent rat i on. The s erum uri c aci d i s reduced t o 3–4 mg/dL, and 24-hour uri ne gl ucos e may exceed 20 g at t erm i n t he abs ence of di abet es .
B. Urinary tract infections Urinary tract infections t hat occur i n pregnant women frequent l y are at t ri but abl e t o t he ri ch nut ri ent cont ent of t he uri ne and t o t he uri nary s t as i s res ul t i ng from uret eral di l at i on. (There i s s ome concern t hat as ympt omat i c bact eri uri a i ncreas es t he rat e of premat uri t y, part i cul arl y i f t here i s ki dney i nvol vement .)
1. Incidence o
o
a. Symptomatic bacteriuria repres ent s t he mos t common renal probl em s een by obs t et ri ci ans . Al t hough as ympt omat i c bact eri uri a occurs wi t h equal frequency i n pregnant and nonpregnant women (i .e., i n 4%–7% of al l women), cl i ni cal i nfect i on (i .e., cys t i t i s or pyel onephri t i s ) devel ops i n 20% of pregnant women.
o
o
b. Acute bacterial interstitial nephritis occurs i n 1%–2% of pregnant women, wi t h s i gns and s ympt oms t hat are comparabl e t o t hos e s een i n nonpregnant women. Lower t ract i nfect i on al s o pres ent s wi t h t ypi cal s ympt oms .
Pa g e 1 3 5 5
ABC Ambe r CHM Conve rte r Tria l ve rsion, http://w w w .proce sste x t.com/a bcchm.html
2. Laboratory findings and diagnosis. Res ul t s are t he s ame i n bot h pregnant and nonpregnant women.
3. T herapy. Al t hough an overwhel mi ng majori t y of pregnant women res pond wel l t o ant i bi ot i c t herapy wi t hout fet al morbi di t y, ant i bi ot i c t herapy mus t be chos en wi t h res pect t o pos s i bl e t oxi c effect s on t he fet us . o
o
a. Agents of choice. Ampicillin i s t he drug of choi ce when no al l ergy exi s t s . Cephalosporins are al s o s afe. Sul fonami des di s pl ace al bumi n-bound bi l i rubi n and may caus e kerni ct erus , and t et racycl i ne has obvi ous dent al and os s eous t oxi ci t y.
o
o
b. Duration of treatment. As ympt omat i c bact eri uri a s houl d be t reat ed for 10–14 days . Cl i ni cal pyel onephri t i s s houl d be t reat ed for 6 weeks , owi ng t o t he very hi gh rat e of rel aps e.
C. Acute renal failure Acute renal failure compl i cat es 1 i n 2000–5000 pregnanci es and has a bi modal pat t ern of occurrence. The firs t peak occurs i n t he fi rs t t ri mes t er and i s rel at ed t o s ept i c abort i on. The s econd peak occurs bet ween 34 and 40 weeks of ges t at i on and i s rel at ed t o preecl amps i a, hemorrhage, and i nt ravas cul ar vol ume depl et i on.
1. Clinical conditions o
Pa g e 1 3 5 6
ABC Ambe r CHM Conve rte r Tria l ve rsion, http://w w w .proce sste x t.com/a bcchm.html
o
a. Cortical necrosis i s a s peci al caus e of acut e renal fai l ure compl i cat i ng pregnancy. Acut e cort i cal necros i s account s for 5% of cas es of acut e renal fai l ure i n t he general popul at i on but 10%–30% of cas es among pregnant women. Cort i cal necros i s may compl i cat e any phas e of pregnancy, part i cul arl y wi t h abrupt i o pl acent ae. The necros i s can be pat chy or ext ens i ve. Some women recover vari abl e amount s of renal funct i on; however, mos t cas es ul t i mat el y progres s t o end-s t age renal di s eas e.
o
o
b. Idiopathic postpartum renal failure i s a rare caus e of renal fai l ure pecul i ar t o pregnancy. Affect ed pat i ent s pres ent s everal weeks aft er an uncompl i cat ed del i very wi t h renal fai l ure and s evere hypert ens i on. The exact et i ol ogy i s unknown; however, t he pat hol ogi c l es i on s t ri ki ngl y res embl es t he hemol yt i c–uremi c s yndrome/TTP compl ex. It i s unknown whet her a vi rus , ret ai ned pl acent al t i s s ue, or drugs i nduce t hi s condi t i on by derangi ng coagul at i on or endot hel i al cel l funct i on. Di l at at i on and curet t age (D&C) t o remove any pl acent al fragment s i s wort h cons i derat i on. Some pat i ent s i mprove wi t h ant i coagul at i on.
2. Prognosis. Pregnancy-rel at ed acut e renal fai l ure has a bet t er out l ook for recovery t han renal fai l ure t hat i s i nduced by medi cal or s urgi cal compl i cat i ons . Nonet hel es s , mat ernal mort al i t y i s s i gni fi cant , rangi ng from 10%–25%.
Pa g e 1 3 5 7
ABC Ambe r CHM Conve rte r Tria l ve rsion, http://w w w .proce sste x t.com/a bcchm.html
D. Hypertension Bl ood pres s ure decl i nes earl y i n pregnancy, reachi ng di as t ol i c l evel s t hat are 15 mm l ower t han prepregnancy l evel s by 22 weeks of ges t at i on. Bl ood pres s ure t hen ri s es gradual l y t o prepregnancy val ues by t erm. Thi s bl ood pres s ure drop i s accompani ed by a cons t ant cardi ac out put , whi ch s ugges t s decreas ed peri pheral res i s t ance as a mechani s m.
1. Clinical conditions. Hypert ens i ve di s orders of pregnancy are cl as s i fi ed for cl i ni cal purpos es as fol l ows : o
o
a. Preeclampsia–eclampsia (s ee Part I: XV E)
o
o
b. Chronic hypertension, whi ch, i n mos t women, i s es s ent i al hypert ens i on recogni zed before pregnancy. A s econdary caus e rarel y i s pres ent and may, as i n t he cas e of pheochromocyt oma, res ul t i n di s as t rous l y hi gh mat ernal mort al i t y. A s t andard eval uat i on for hypert ens i on s houl d be conduct ed i n women who have hi gh bl ood pres s ure and cont empl at e pregnancy.
o
o
c. Chronic hypertension with superimposed preeclampsia
o
o
d. Late or transient hypertension, whi ch, i n mos t cas es , occurs i n t he t hi rd t ri mes t er and res ol ves wi t hi n 10 days of del i very but t ends t o recur i n s ubs equent pregnanci es . Many of t hes e women may ul t i mat el y devel op es s ent i al hypert ens i on.
Pa g e 1 3 5 8
ABC Ambe r CHM Conve rte r Tria l ve rsion, http://w w w .proce sste x t.com/a bcchm.html
2. T herapy. The t reat ment of hypert ens i on duri ng pregnancy i s very di ffi cul t . Overl y ent hus i as t i c l oweri ng of mat ernal bl ood pres s ure may di mi ni s h ut eropl acent al bl ood fl ow, l eadi ng t o fet al compromi s e. Di as t ol i c bl ood pres s ures above 100 mm Hg i n nonecl ampt i c women are bes t t reat ed wi t h hydralazine, methyldopa, or calcium channel blockers. o
o
a. For many reas ons , s ome of whi ch remai n cont rovers i al , di uret i cs s houl d not be us ed rout i nel y.
o
o
b. ACE i nhi bi t ors and ARBs are cont rai ndi cat ed for pregnant women becaus e t hey reduce pl acent al bl ood fl ow and i nduce acut e renal fai l ure i n t he fet us , renal agenes i s , and ot her deformi t i es .
E. Preeclampsia–eclampsia (toxemia of pregnancy) Preeclampsia–eclampsia (toxemia of pregnancy) i s pri mari l y, but not excl us i vel y, a di s eas e of young pri mi paras .
1. Definition. Preecl amps i a i s a s yndrome of hypertension (frequent l y mal i gnant ), proteinuria, edema, and, i n i t s ext reme, a microangiopathic hemolytic anemia wi t h vas cul ar endot hel i al des t ruct i on. The hal l mark of t hi s di s eas e i s t he labile vasospasm, refl ect i ng a vas cul ar s ens i t i vi t y t o t he pres s or effect s of endogenous pept i des and cat echol ami nes . Bl ood pres s ure may fl uct uat e wi del y, but s us t ai ned 4- t o 6-hour peri ods of hypert ens i on are rel i abl e s i gns of
Pa g e 1 3 5 9
ABC Ambe r CHM Conve rte r Tria l ve rsion, http://w w w .proce sste x t.com/a bcchm.html
di s eas e. Preecl amps i a t hat i s as s oci at ed wi t h mat ernal convul s i ons and coma i s referred t o as eclampsia.
2. Pathology. The renal hi s t opat hol ogy i s gl omerul ar capi l l ary endot hel i os i s , wi t h s wel l i ng of capi l l ary endot hel i al cel l s i n t he abs ence of hypercel l ul ari t y. Vacuol i zat i on i s common.
3. Clinical features and diagnosis. The i ni t i al cl i ni cal pres ent at i on may be mi l d or s evere; however, s us t ai ned hypert ens i on newl y appeari ng i n t he t hi rd t ri mes t er of a fi rs t pregnancy i s a s ui t abl e cri t eri on for a pres umpt i ve di agnos i s of preecl amps i a. Unt reat ed, ful mi nant preecl amps i a progres s es rapi dl y t o mat ernal convul s i ons , anuri c renal fai l ure, and deat h.
4. T herapy i ncl udes hos pi t al i zat i on and bed res t , prompt del i very i f t he fet us i s mat ure, parent eral magnes i um s ul fat e for i mpendi ng convul s i ons , and careful t i t rat i on of bl ood pres s ure t o a di as t ol i c range of 95–105 mm Hg. Gangl i oni c bl ockers i nduce meconi um i l eus i n t he fet us and are t o be avoi ded. Di uret i cs are not recommended.
Part II: Fluid and Electrolyte Disorders I. Water Metabolism A. Normal physiology
1. Regulation of water intake. Increas ed t hi rs t i s t he normal res pons e t o wat er l os s . The neural cent er t hat cont rol s t he rel eas e of ADH i s anat omi cal l y cl os e t o t he
Pa g e 1 3 6 0
ABC Ambe r CHM Conve rte r Tria l ve rsion, http://w w w .proce sste x t.com/a bcchm.html
t hi rs t cent er. ADH rel eas e i s s t i mul at ed by i ncreas ed body fl ui d t oni ci t y or decreas ed effect i ve ci rcul at i ng vol ume. o
o
a. T onicity refers t o t he s hi ft of wat er t hrough bi omembranes produced by os mot i cal l y act i ve part i cl es s uch as gl ucos e and s odi um. Urea exert s vi rt ual l y no t oni ci t y becaus e i t eas i l y cros s es al l membranes and produces no os mot i c s hi ft of wat er.
o
o
b. Osmolality i s a funct i on of t he number of mol ecul es i n s ol ut i on i ndependent of effect s on wat er movement .
2. Regulation of water output o
o
a. Proximal tubular reabsorption. Of t he 200 L/day of wat er t hat i s fi l t ered at t he gl omerul us , 125 L i s reabs orbed i n t he proxi mal t ubul e.
o
o
b. Osmotic gradient formation in the medulla. Gl omerul ar fi l t rat e not reabs orbed i n t he proxi mal t ubul e ent ers t he l oop of Henl e, where i n t he t hi ck as cendi ng l i mb, act i ve s odi um chl ori de reabs orpt i on wi t hout wat er reabs orpt i on caus es di l ut i on of t he uri ne and i ncreas es t he concent rat i on of s ol ut es i n t he medul l ary i nt ers t i t i um.
o
o
c. Collecting tubular transport. W at er t hat reaches t he col l ect i ng t ubul e ei t her i s excret ed (i f ADH i s abs ent , caus i ng t he t ubul e t o be
Pa g e 1 3 6 1
ABC Ambe r CHM Conve rte r Tria l ve rsion, http://w w w .proce sste x t.com/a bcchm.html
i mpermeabl e t o wat er) or i s reabs orbed (i f ADH i s pres ent , caus i ng t he t ubul e t o be permeabl e t o wat er). Thus , ADH affect s t he os mol al i t y of uri ne, whi ch may range from 50 mOs m/kg t o 1200 mOs m/L.
B. Hyponatremia
1. Definition. Hyponat remi a refers t o s erum s odi um concent rat i on of l es s t han 135 mEq/L. The name, however, i s s omewhat mi s l eadi ng, becaus e hyponat remi a i s us ual l y a probl em of t oo much wat er, not t oo l i t t l e s odi um. In fact , t he s odi um cont ent of t he body may be i ncreas ed, decreas ed, or rel at i vel y unchanged. Hypot oni ci t y al ways i mpl i es hyponat remi a. The oppos i t e i s not al ways t rue: hyponat remi a can coexi s t wi t h i s ot oni ci t y, hypert oni ci t y, or hypot oni ci t y. o
o
a. Pseudohyponatremia (i s ot oni c hyponat remi a) i s a l aborat ory art i fact t hat occurs i n t he s et t i ng of ext reme hyperl i pi demi a or hyperprot ei nemi a. If t he l aborat ory us es an i ns t rument t hat report s s odi um cont ent per uni t vol ume of t ot al pl as ma rat her t han s odi um cont ent per vol ume of t he aqueous phas e, t hen s i gni fi cant el evat i ons i n pl as ma l i pi ds or γ-gl obul i ns can caus e t he report ed s odi um concent rat i on t o be art i fi ci al l y l ow. Thi s art i fact can be obvi at ed by us i ng an i on s el ect i ve el ect rode t hat meas ures s odi um i on concent rat i on i n t he aqueous phas e. P.277
o
Pa g e 1 3 6 2
ABC Ambe r CHM Conve rte r Tria l ve rsion, http://w w w .proce sste x t.com/a bcchm.html
o
b. Hypertonic hyponatremia res ul t s from t he s hi ft of wat er from t he i nt racel l ul ar fl ui d t o t he ext racel l ul ar fl ui d, whi ch i s caus ed by t he pres ence of os mot i cal l y act i ve part i cl es (e.g., gl ucos e) i n t he ext racel l ul ar fl ui d s pace. Serum s odi um concent rat i on i s reduced, but t he os mol al i t y of t he ext racel l ul ar fl ui d i s above normal .
o
o
c. T rue hyponatremia (hypot oni c hyponat remi a) occurs when t here i s exces s t ot al body wat er rel at i ve t o s ol ut e cont ent , and i s cl i ni cal l y s i gni fi cant when t he s erum s odi um concent rat i on i s l es s t han 125 mEq/L and t he s erum os mol al i t y i s l es s t han 250 mOs m/kg.
2. Etiology o
o
a. Decreased renal water excretion
(1) Decreased GFR. A decreas e i n t he fi l t ered l oad of wat er t o l es s t han 10% of normal res ul t s i n a cl i ni cal l y s i gni fi cant decreas e i n t he abi l i t y of t he ki dney t o excret e wat er.
(2) Increased proximal tubular reabsorption. An i ncreas e i n proxi mal t ubul ar reabs orpt i on of fi l t ered fl ui d from t he normal 65% t o more t han 90% may i mpai r t he capaci t y of t he ki dney t o excret e wat er. Increas ed proxi mal t ubul ar reabs orpt i on occurs
Pa g e 1 3 6 3
ABC Ambe r CHM Conve rte r Tria l ve rsion, http://w w w .proce sste x t.com/a bcchm.html
when t he ki dney i s hypoperfus ed (e.g., i n s t at es of exces s i ve fl ui d l os s from di arrhea or vomi t i ng). Decreas ed effect i ve renal perfus i on i n di s eas es s uch as CHF, ci rrhos i s , or nephrot i c s yndrome al s o s t i mul at es proxi mal t ubul ar reabs orpt i on. Thi s group of di s orders i s charact eri zed by a l ow uri ne s odi um concent rat i on, i ndi cat i ng i ncreas ed renal abs orpt i on of s odi um, hi gh BUN, and t he phys i cal fi ndi ng of ei t her t rue vol ume depl et i on or one of t he edemat ous condi t i ons .
(3) Increased collecting tubular reabsorption of water. Nonos mot i cal l y s t i mul at ed ADH s ecret i on i nduces s uch reabs orpt i on. Charact eri s t i cs of t hi s condi t i on i ncl ude rel at i vel y normal uri ne s odi um excret i on (i f i nt ake i s normal ), a hi gh uri ne os mol al i t y, and s i gns of body wat er expans i on res ul t i ng from exces s i ve ret ent i on of i nges t ed wat er.
o
o
b. Increased fluid intake. Fl ui d i nt ake i n exces s of 1 L/hour exceeds normal excret ory capaci t y and l eads t o hyponat remi a. Thi s s i t uat i on i s s een i n pat i ent s who are gi ven exces s i ve hypot oni c i nt ravenous fl ui ds and i n ps ychi at ri c pat i ent s who dri nk exces s i vel y.
o
o
c. Syndrome of inappropriate ADH secretion (SIADH) res ul t s from ADH rel eas e i n t he abs ence of i ncreas ed body fl ui d t oni ci t y or decreas ed effect i ve ci rcul at i ng vol ume. Caus es of SIADH are
Pa g e 1 3 6 4
ABC Ambe r CHM Conve rte r Tria l ve rsion, http://w w w .proce sste x t.com/a bcchm.html
l i s t ed i n Tabl e 6-14.
3. Clinical features. CNS dys funct i on may devel op as t he t oni ci t y of t he ext racel l ul ar fl ui d fal l s and wat er di ffus es down an os mot i c gradi ent i nt o t he brai n cel l s , l eadi ng t o cel l ul ar edema. Acut e P.278
hyponat remi a wi t h a decreas e i n s erum s odi um concent rat i on bel ow 125 mEq/L over a peri od of hours al mos t al ways i s as s oci at ed wi t h acut e CNS di s t urbances s uch as obt undat i on, coma, s ei zures , and deat h i f unt reat ed.
TABLE 6-14 Causes of SIADH Me Ca ch us ani e sm Tu Sev mo era r
l tu mo rs pro duc e an AD H-l i ke pe pt i
Pa g e 1 3 6 5
ABC Ambe r CHM Conve rte r Tria l ve rsion, http://w w w .proce sste x t.com/a bcchm.html
de, mo st not abl y sm al l cel l car ci n om a of t he l un g CN Exc S
es s
di s i ve eas AD e
H rel eas e ma y occ ur s ec on dar y to s ei
Pa g e 1 3 6 6
ABC Ambe r CHM Conve rte r Tria l ve rsion, http://w w w .proce sste x t.com/a bcchm.html
z ur es , cer ebr al t ra um a, bra in tu mo rs , or ps y chi at ri c di s t ur ba nce s. Na us e a an d vo mi t i ng al s o ma y
Pa g e 1 3 6 7
ABC Ambe r CHM Conve rte r Tria l ve rsion, http://w w w .proce sste x t.com/a bcchm.html
pro duc e exc es s i ve AD H rel eas e Pul SIA mo DH nar ma y
y
di s be eas s ee e
n wi t h pul mo nar y tu mo rs , i nf ect i on s, an d bro nch
Pa g e 1 3 6 8
ABC Ambe r CHM Conve rte r Tria l ve rsion, http://w w w .proce sste x t.com/a bcchm.html
os p as t ic di s eas e; t ho ug ht to be me di a t ed by J rec ept ors in t he pul mo nar y ci rc ul a tio n t ha t sti mu l at e
Pa g e 1 3 6 9
ABC Ambe r CHM Conve rte r Tria l ve rsion, http://w w w .proce sste x t.com/a bcchm.html
pi t ui t ary AD H rel eas e Adr Th en e al
l os
i ns s uffi of ci e gl u ncy coc ort i coi d i nh i bi t i on of AD H rel eas e res ul t s in exc es s i ve s ec
Pa g e 1 3 7 0
ABC Ambe r CHM Conve rte r Tria l ve rsion, http://w w w .proce sste x t.com/a bcchm.html
ret i on of AD H. In ad di t i on, pri ma ry adr en al i ns uffi ci e ncy i nv ol v i ng bot h gl u coc ort i coi d an d mi ner al o cor
Pa g e 1 3 7 1
ABC Ambe r CHM Conve rte r Tria l ve rsion, http://w w w .proce sste x t.com/a bcchm.html
tic oi d pro duc tio n ma y res ul t in ren al s od iu m wa sti ng, exa cer bat i ng t he hyp on at r em ia Dru Me gs di c at i ons ma y
Pa g e 1 3 7 2
ABC Ambe r CHM Conve rte r Tria l ve rsion, http://w w w .proce sste x t.com/a bcchm.html
act to pro duc e SIA DH by i nc rea sin g AD H rel eas e or by s en siti zin g t he ren al t ub ul e to t he eff ect s of AD
Pa g e 1 3 7 3
ABC Ambe r CHM Conve rte r Tria l ve rsion, http://w w w .proce sste x t.com/a bcchm.html
H. Exa mp l es i ncl ud e chl orp rop am i de , cl o fi br at e , t hi az i de di u ret i cs , an d ma ny CN S-a ct i ve dru gs Idi So op me
Pa g e 1 3 7 4
ABC Ambe r CHM Conve rte r Tria l ve rsion, http://w w w .proce sste x t.com/a bcchm.html
at h pat ic
i en ts, es p eci al l y el d erl y pat i en ts, ma y hav e no ap par ent rea s on for SIA DH , an d i nc rea s ed AD H rel
Pa g e 1 3 7 5
ABC Ambe r CHM Conve rte r Tria l ve rsion, http://w w w .proce sste x t.com/a bcchm.html
eas e in t he se pat i en ts ma y be du e sim pl y to adv anc i ng ag e Re Thi s et s os i s mo a s t a var t
i an t of SIA DH occ urri ng in
Pa g e 1 3 7 6
ABC Ambe r CHM Conve rte r Tria l ve rsion, http://w w w .proce sste x t.com/a bcchm.html
chr oni cal l y ill an d ma l no uri s he d pat i en ts in wh om t he s er um s od iu m is res et at a l ow val ue (t y pi c al l
Pa g e 1 3 7 7
ABC Ambe r CHM Conve rte r Tria l ve rsion, http://w w w .proce sste x t.com/a bcchm.html
y ab out 12 5 mE q/L ). Th es e pat i en ts are abl e to ma i nt ai n wa t er bal anc e, an d ap pro pri at e ly exc ret e a
Pa g e 1 3 7 8
ABC Ambe r CHM Conve rte r Tria l ve rsion, http://w w w .proce sste x t.com/a bcchm.html
wa t er l oa d ADH, ant i di u ret i c hormo ne; CNS, cent ral nervou s s ys t em ; SAIDH, s yndro me of i nappr opri at e ADH s ecret i on.
4. Diagnosis o
o
a. Physical findings. Exami nat i on may reveal :
(1) Vol ume depl et i on (e.g., i n cas es rel at ed t o drugs s uch as di uret i cs )
(2) Edema (e.g., i n cas es rel at ed t o ci rrhos i s
Pa g e 1 3 7 9
ABC Ambe r CHM Conve rte r Tria l ve rsion, http://w w w .proce sste x t.com/a bcchm.html
or CHF) o
o
b. Laboratory data
(1) Uri ne os mol al i t y: >50–100 mOs m/kg i n t he pres ence of pl as ma hypot oni ci t y
(2) Uri ne s odi um concent rat i on: hi gh when pl as ma vol ume i s expanded i n SIADH but l ow when effect i ve art eri al bl ood vol ume i s reduced, as i n edemat ous condi t i ons . A uri ne s odi um concent rat i on l es s t han 20 mEq/L s t rongl y argues agai ns t SIADH.
o
o
c. Water loading test. W hen an i nt ravas cul arl y vol ume-expanded i ndi vi dual i s gi ven 20 mL wat er/kg oral l y or i nt ravenous l y over a peri od of 20–40 mi nut es , t he normal res pons e i s excret i on of 80% of t hi s wat er l oad wi t hi n 4 hours and reduct i on of uri ne os mol al i t y t o bel ow 100 mOs m/kg. Fai l ure t o achi eve t hes e res ul t s s ugges t s an i mpai rment i n t he ki dney's abi l i t y t o excret e wat er.
5. T herapy o
o
a. Fluid restriction. Al l pat i ent s who are s everel y hyponat remi c s houl d reduce free wat er i nt ake t o approxi mat el y 800 mL/day.
o
Pa g e 1 3 8 0
ABC Ambe r CHM Conve rte r Tria l ve rsion, http://w w w .proce sste x t.com/a bcchm.html o
b. Inhibition of water reabsorption
(1) Demeclocycline. Thi s agent has been s hown t o al t er ADH-i nduced wat er fl ow i n t he col l ect i ng t ubul e. Thi s drug mus t be gi ven i n dos es of 600–1200 mg/day and requi res 4–5 days t o achi eve i t s peak act i on. Demecl ocycl i ne cannot be admi ni s t ered t o pat i ent s wi t h l i ver di s eas e, heart fai l ure, or ki dney di s eas e, becaus e i t may accumul at e t o t oxi c l evel s i n t hes e condi t i ons .
(2) Furosemide. Acut e admi ni s t rat i on of t hi s agent i n combi nat i on wi t h l arge amount s of s al i ne may l ead t o i ncreas ed wat er excret i on.
(3) T olvaptan. Thi s agent i s an oral vas opres s i n wi t h rel at i ve affi ni t y for t he V2 recept or, whi ch has been s hown t o i nduce a di ures i s wi t h proport i onal i t y more free-wat er t han s odi um l os s . Current s t udi es are eval uat i ng t he us e of t hi s drug i n mi l d t o moderat e hyponat remi a.
o
o
c. Hypertonic infusions. The i nfus i on of 3% s odi um chl ori de rapi dl y rai s es t he t oni ci t y of t he ext racel l ul ar fl ui d. Thi s i s rarel y done due t o t he ri s k of del et eri ous effect s , i ncl udi ng pul monary or cerebral edema, pont i ne demyel i nat i on, and s evere neurol ogi c damage. W hen us ed, s erum s odi um concent rat i on s houl d not i ncreas e fas t er t han 0.5 mEq/L/hour.
Pa g e 1 3 8 1
ABC Ambe r CHM Conve rte r Tria l ve rsion, http://w w w .proce sste x t.com/a bcchm.html
6. Complications o
o
a. Acute hyponatremia. Acut e reduct i on of s erum os mol al i t y can produce i nt racrani al hypert ens i on and brai n damage, part i cul arl y i f t he s erum s odi um concent rat i on fal l s bel ow 125 mEq/L over a peri od of hours .
o
o
b. Chronic hyponatremia. Brai n cel l s adapt t o chroni c hyponat remi a by l os s of net i nt racel l ul ar s ol ut e (pri mari l y pot as s i um chl ori de and organi c mol ecul es , t ermed os mol yt es ). Thi s adapt at i on, whi ch can occur over a peri od of a few days , l eads t o marked reduct i on i n t he degree of cel l s wel l i ng.
C. Hypernatremia
1. Definition. Hypernat remi a refers t o s erum s odi um concent rat i on t hat i s above normal . Cl i ni cal l y s i gni fi cant effect s are produced at s erum s odi um l evel s great er t han 155 mEq/L. Hypernat remi a al ways i mpl i es hypert oni ci t y of al l body fl ui ds , becaus e t he ri s e i n t he ext racel l ul ar fl ui d os mol al i t y obl i gat es movement of wat er from t he i nt racel l ul ar s pace, produci ng i ncreas ed i nt racel l ul ar os mot i c act i vi t y and cel l dehydrat i on.
2. Etiology o
o
a. Extrarenal causes
Pa g e 1 3 8 2
ABC Ambe r CHM Conve rte r Tria l ve rsion, http://w w w .proce sste x t.com/a bcchm.html
(1) Decreased fluid intake. Adequat e wat er i nt ake i s requi red t o mai nt ai n t he t oni ci t y of body fl ui ds i n t he face of cont i nuous wat er l os s es t hrough t he s ki n as wel l as l os s es t hrough t he uri ne and gas t roi nt es t i nal t ract . In cool envi ronment s , t hi s i nt ake equal s approxi mat el y 700 mL/day. If i nt ake i s l es s t han ext ernal l os s es , body fl ui d os mol al i t y ri s es . P.279
(2) Increased skin losses. Profus e s weat i ng may l ead t o exces s wat er l os s es t hrough t he s ki n. In addi t i on, burns and ot her wi des pread i nfl ammat ory l es i ons of t he s ki n may caus e marked fl ui d l os s es .
(3) Increased gastrointestinal losses. Di arrhea and prot ract ed vomi t i ng al s o may res ul t i n wat er defi ci t s .
o
o
b. Renal causes
(1) Osmotic diuresis. The pres ence of os mot i cal l y act i ve, nonreabs orbabl e s ol ut e i n t he gl omerul ar fi l t rat e prevent s wat er and s odi um reabs orpt i on and l eads t o i ncreas ed renal wat er l os s es . Hypergl ycemi a wi t h gl ycos uri a i s a common caus e of os mot i c di ures i s . Becaus e wat er l os s es are rel at i vel y great er t han s odi um l os s es , t he s erum s odi um
Pa g e 1 3 8 3
ABC Ambe r CHM Conve rte r Tria l ve rsion, http://w w w .proce sste x t.com/a bcchm.html
concent rat i on ri s es progres s i vel y duri ng os mot i c di ures i s .
(2) Decreased ADH effect
(a) Central diabetes insipidus (i .e., fai l ure of ADH s ynt hes i s or rel eas e) may occur i n t he fol l owi ng s et t i ngs :
(i) T umor. ADH defi ci ency may occur ei t her t hrough di rect i nvas i on of t he neurohypophys i s or t hrough i ncreas ed i nt racrani al pres s ure compres s i ng t he brai ns t em.
(ii) Histiocytosis. Hand-Schül l er-Chri s t i an di s eas e, i n part i cul ar, has a predi l ect i on for neurohypophys eal i nvol vement , produci ng ADH defi ci ency.
(iii) Sarcoidosis. The neurohypophys i s may be i nvol ved, produci ng di abet es i ns i pi dus .
(iv) T rauma. Cl as s i cal l y, aft er res ect i on of t he pi t ui t ary s t al k, a phas e of acut e ADH rel eas e i s fol l owed by a prol onged peri od of cent ral di abet es i ns i pi dus .
Pa g e 1 3 8 4
ABC Ambe r CHM Conve rte r Tria l ve rsion, http://w w w .proce sste x t.com/a bcchm.html
(b) Nephrogenic diabetes insipidus (i .e., fai l ure of renal wat er cons ervat i on des pi t e hi gh l evel s of pl as ma ADH) may occur i n t he fol l owi ng s et t i ngs .
(i) Renal disease. St ruct ural di s eas e i mpai rs t he i nt egri t y of t he renal medul l a and, t hereby, t he uri ne concent rat i ng abi l i t y.
(ii) Hypercalcemia. El evat i on of s erum cal ci um concent rat i on above 12 mg/dL may i mpai r uri ne concent rat i ng abi l i t y, mos t l i kel y as a res ul t of i nhi bi t i on of s odi um chl ori de reabs orpt i on i n t he t hi ck as cendi ng l i mb of Henl e's l oop, i ncreas ed medul l ary bl ood fl ow, di s s i pat i on of medul l ary hypert oni ci t y, and i nt erference wi t h ADH-medi at ed wat er fl ow i n t he medul l ary col l ect i ng t ubul e.
(iii) Hypokalemia. Reduct i on of s erum pot as s i um concent rat i on bel ow 3.5 mEq/L l eads t o a di rect s t i mul at i on of t hi rs t and a mi l d i mpai rment of uri ne concent rat i ng abi l i t y.
(iv) Lithium ingestion. Thi s act i on bl ocks ADH-s t i mul at ed os mot i c
Pa g e 1 3 8 5
ABC Ambe r CHM Conve rte r Tria l ve rsion, http://w w w .proce sste x t.com/a bcchm.html
wat er fl ow i n t he col l ect i ng t ubul e.
(v) Demeclocycline. Thi s t et racycl i ne ant i bi ot i c al t ers ADH-i nduced wat er fl ow t hrough a di rect effect on t he cel l membrane.
(vi) Sickle cell anemia. Reduced medul l ary bl ood fl ow produced by s i ckl i ng eryt hrocyt es wi t hi n t he vas a rect a al s o may i mpai r uri ne concent rat i ng abi l i t y.
(vii) Urinary tract obstruction and t he postobstructive state. Thes e condi t i ons are as s oci at ed wi t h nephrogeni c di abet es i ns i pi dus .
3. Clinical features o
o
a. CNS disorders. General i zed CNS depres s i on, i ncl udi ng obt undat i on, coma, and s ei zures , devel ops i n young chi l dren and el derl y pat i ent s . Int racerebral and s ubarachnoi d hemorrhage may occur i f s hri nkage of brai n vol ume l eads t o t ears i n t he bri dgi ng vei ns .
o
o
b. Extracellular volume depletion. Exces s i ve wat er l os s i n hypernat remi c s t at es may l ead t o t hi s condi t i on. Al t hough t he i nt racel l ul ar fl ui d account s for t wo-t hi rds of wat er defi ci t s , t he ext racel l ul ar
Pa g e 1 3 8 6
ABC Ambe r CHM Conve rte r Tria l ve rsion, http://w w w .proce sste x t.com/a bcchm.html
fl ui d vol ume al s o cont ract s mi l dl y. If l os s of wat er as wel l as s ol ut e occurs , t he cont ract i on i s more pronounced. However, i f t he et i ol ogy of t he hypernat remi a i s due t o exces s s al t i nt ake (e.g., hypert oni c s odi um bi carbonat e i nfus i on or s ea-wat er drowni ng), t he ext racel l ul ar fl ui d vol ume i ncreas es . o
o
c. Abnormal urine output. If t he ki dneys caus e wat er l os s es , pol yuri a (i .e., uri ne out put t hat i s i nappropri at el y hi gh gi ven t he l evel of pl as ma os mol al i t y or ext racel l ul ar fl ui d vol ume) may be pres ent . If t he ki dneys are normal and wat er l os s es are ext rarenal , uri ne vol ume t ypi cal l y i s reduced.
P.280
4. Diagnosis o
o
a. Dehydration test. Uri ne concent rat i ng abi l i t y may be t es t ed aft er overni ght dehydrat i on t o det ermi ne whet her a pat i ent has renal wat er was t i ng.
(1) W at er depri vat i on begi ns at 8:00 p.m. and l as t s 14 hours , aft er whi ch t he uri ne os mol al i t y s houl d exceed 800 mOs m/kg. The pat i ent t hen i s gi ven a s ubcut aneous dos e of ADH (5 U aqueous vas opres s i n). The uri ne os mol al i t y s houl d not be furt her i ncreas ed by t hi s maneuver.
Pa g e 1 3 8 7
ABC Ambe r CHM Conve rte r Tria l ve rsion, http://w w w .proce sste x t.com/a bcchm.html
(2) If t he uri ne os mol al i t y i s l es s t han 800 mOs m/kg aft er wat er depri vat i on or i f i t i ncreas es by great er t han 15% aft er ADH admi ni s t rat i on, s ome degree of ADH defi ci ency i s pres ent .
(3) If t he uri ne os mol al i t y does not exceed 300 mOs m/kg aft er wat er depri vat i on and t here i s no furt her i ncreas e aft er ADH admi ni s t rat i on, s ome form of nephrogeni c di abet es i ns i pi dus i s pres ent .
o
o
b. Plasma ADH assay. In nephrogeni c di abet es i ns i pi dus , t he uri ne os mol al i t y may not be a t rue refl ect i on of ADH rel eas e, and, t hus , pl as ma ADH l evel s s houl d be meas ured.
o
o
c. Assay of urine osmolality and composition
(1) It i s us eful t o meas ure t he s ol ut e compos i t i on of t he uri ne i n t he eval uat i on of pol yuri a. Uri ne os mol al i t y l es s t han 200 mOs m/L s ugges t s a pri mary defect i n wat er cons ervat i on. Uri ne os mol al i t y great er t han 200 mOs m/L duri ng pol yuri a s ugges t s an os mot i c di ures i s .
(2) Aft er meas uri ng uri ne os mol al i t y, t he uri ne s houl d be anal yzed for s odi um, gl ucos e, and urea t o det ermi ne t he et i ol ogy of t he di ures i s . A uri ne pH great er t han 6 may
Pa g e 1 3 8 8
ABC Ambe r CHM Conve rte r Tria l ve rsion, http://w w w .proce sste x t.com/a bcchm.html
i ndi cat e bi carbonat e di ures i s . o
o
d. Physical examination. Al t hough exami nat i on may be hel pful i n det ermi ni ng i f hypovol emi a or hypervol emi a i s pres ent , i t i s general l y not us eful i n det ermi ni ng t he et i ol ogy and pat hogenes i s of pol yuri a.
5. T herapy o
o
a. Free water may be admi ni s t ered oral l y, whi ch i s t he preferred rout e, or i nt ravenous l y as a 5% dext ros e s ol ut i on (D 5 W ). The dext ros e i s readi l y met abol i zed, l eavi ng behi nd free wat er. Infus i on of a fl ui d wi t h an os mol al i t y l es s t han 150 mOs m/L i s dangerous and may l ead t o acut e hemol ys i s at t he i nfus i on s i t e.
o
o
b. Vasopressin may be admi ni s t ered i n s everal di fferent forms . Current l y, t he agent of choi ce for t reat ment of ADH defi ci ency i s 1-deamino-8-D-arginine vasopressin (DDAVP or desmopressin), whi ch may be admi ni s t ered oral l y or as a nas al s pray every 12 hours .
o
o
c. T hiazide diuretics i mpai r di l ut i on i n t he di s t al nephron and s t i mul at e proxi mal t ubul ar reabs orpt i on of s odi um and wat er as a res ul t of vol ume depl et i on. The l at t er act i on reduces t he del i very of fl ui d t o t he di s t al nephron, t hereby reduci ng t he degree of pol yuri a. Thi azi des are us eful as adjunct t herapy i n pat i ent s wi t h
Pa g e 1 3 8 9
ABC Ambe r CHM Conve rte r Tria l ve rsion, http://w w w .proce sste x t.com/a bcchm.html
nephrogeni c di abet es i ns i pi dus . o
o
d. Other drugs s uch as cl ofi brat e, carbamazepi ne, and chl orpropami de enhance t he renal t ubul ar effect s of ADH and pos s i bl y cont ri but e t o t he s t i mul at i on of ADH rel eas e i n cert ai n s et t i ngs .
6. Complications. Di s eas es of wat er cons ervat i on are dangerous onl y i f pat i ent s are not al l owed acces s t o wat er. In s uch s et t i ngs , cel l ul ar dehydrat i on, CNS depres s i on, and s evere vol ume depl et i on may occur. The s eri ous mani fes t at i ons of acut e hypernat remi a are pri mari l y due t o brai n cel l s hri nkage. Brai n cel l s adapt t o chroni c hypernat remi a by net gai n of i nt racel l ul ar s ol ut e (mai nl y s odi um chl ori de and organi c os mol yt es s uch as myo-i nos i t ol , t auri ne, bet ai ne, and ot her met hyl ami nes ). Thi s adapt at i on, occurri ng over s everal days , l eads t o marked reduct i on i n t he degree of cel l s hri nkage.
II. Sodium Metabolism A. Normal physiology Sodi um i s t he pri mary os mot i c component of t he ext racel l ul ar fl ui d, and det ermi nes t he vol ume of t hat s pace and t he “ful l nes s ,― or effect i ve vol ume, of t he s ys t emi c ci rcul at i on. A l es s t han 1% change i n renal s odi um excret i on can produce major changes i n ext racel l ul ar fl ui d vol ume.
1. Renal handling. Sodi um i s freel y fi l t ered at t he gl omerul us , and t he majori t y mus t be reabs orbed t o mai nt ai n s odi um homeos t as i s . Al t hough onl y 10%–15% of t he gl omerul ar fi l t rat e i s reabs orbed P.281
Pa g e 1 3 9 0
ABC Ambe r CHM Conve rte r Tria l ve rsion, http://w w w .proce sste x t.com/a bcchm.html
i n t he di s t al t ubul e and col l ect i ng duct , t hi s s i t e i s t he major regul at or for det ermi ni ng fi nal uri ne s odi um compos i t i on.
2. Hormonal regulation. Many hormones may al t er t ubul ar handl i ng of s odi um, but none i s as wel l s t udi ed as al dos t erone, whi ch i s regul at ed by t he renin–angiotensin system. o
o
a. Reni n s ecret i on by t he ki dney i s s t i mul at ed by renal hypoperfus i on, adrenergi c s t i mul at i on, and ci rcul at i ng cat echol ami nes . Reni n i s rel eas ed from t he juxtaglomerular apparatus, whi ch i s l ocat ed bet ween t he afferent and t he efferent art eri ol es of t he gl omerul i .
o
o
b. Reni n i s an enz yme t hat cat al yzes t he convers i on of angiotensinogen t o t he decapept i de angiotensin I (i n pl as ma). Angi ot ens i n I i s convert ed t o t he oct apept i de angiotensin II (i n t he l ung and ki dney) by ACE. Angi ot ens i n II i s a pot ent vas ocons t ri ct i ve agent as wel l as a pot ent s t i mul us for i ncreas ed al dos t erone rel eas e from t he adrenal gl and.
o
o
c. Al dos t erone s t i mul at es s odi um reabs orpt i on i n t he cort i cal col l ect i ng duct .
o
d. Ot her hormones t hat regul at e s odi um handl i ng are l i s t ed i n Onl i ne Tabl e 6-15.
Pa g e 1 3 9 1
ABC Ambe r CHM Conve rte r Tria l ve rsion, http://w w w .proce sste x t.com/a bcchm.html
o
Online TABLE 6-15 Other Hormones and Their Reactions to Sodium Handling Act ion on So diu m Ho T r rm an on sp e ort At r Inh i al i bi t nat s ri ur ren et i al c
rea
pe bs o pt i rpt i de on (co l l ec tin g duc t) Do Int pa rar mi en ne al l y ge
Pa g e 1 3 9 2
ABC Ambe r CHM Conve rte r Tria l ve rsion, http://w w w .proce sste x t.com/a bcchm.html
ner at e d; i nh i bi t s pro xi m al t ub ul e s od iu m rea bs o rpt i on Pro Int s t a rar gl a en ndi al l ns y ge ner at e d; i nh i bi t t ub ul a r s od iu m
Pa g e 1 3 9 3
ABC Ambe r CHM Conve rte r Tria l ve rsion, http://w w w .proce sste x t.com/a bcchm.html
rea bs o rpt i on an d vas odi l at e t he ki d ney
B. Edema
1. Definition o
o
a. Edema general l y i s defi ned as an i ncreas e i n t he i nt ers t i t i al compart ment of t he ext racel l ul ar fl ui d.
(1) Normal l y, t he ext racel l ul ar fl ui d vol ume equal s approxi mat el y 14 L and account s for one-t hi rd of t he t ot al body wat er. About 25% of t he ext racel l ul ar fl ui d i s repres ent ed by pl as ma vol ume and i s cont ai ned wi t hi n t he ci rcul at i on. The ot her 75% or 11 L i s repres ent ed by t he i nt ers t i t i al fl ui d bet ween cel l s .
(2) If t he i nt ers t i t i al fl ui d vol ume i ncreas es by approxi mat el y 2 L, cl i ni cal l y evi dent edema
Pa g e 1 3 9 4
ABC Ambe r CHM Conve rte r Tria l ve rsion, http://w w w .proce sste x t.com/a bcchm.html
may res ul t ; edema may be observable (as swelling) or palpable (as pitting). o
o
b. Al t hough edema general l y i s a funct i on of i ncreas ed ext racel l ul ar fl ui d vol ume, i n s ome i ns t ances i ncreas ed t rans capi l l ary hydros t at i c pres s ure (e.g., as occurs i n t he port al ci rcul at i on i n ci rrhos i s ) al s o may cont ri but e t o edema.
2. Pathophysiology. Edema, or t he pat hol ogi c i ncreas e i n ext racel l ul ar fl ui d vol ume, pri mari l y i s a funct i on of exces s i ve renal t ubul ar reabs orpt i on of s odi um. Decreas ed renal perfus i on (e.g., as occurs i n CHF wi t h reduced cardi ac out put , i n ci rrhos i s wi t h reduced effect i ve art eri al bl ood vol ume, and i n t he nephrot i c s yndrome) i s t he proxi mat e caus e of t he i ncreas ed renal s odi um reabs orpt i on, whi ch repres ent s t he body's at t empt t o mai nt ai n adequat e effect i ve art eri al bl ood vol ume.
3. Etiology o
o
a. CHF. W hen cardi ac out put i s reduced, effect i ve art eri al bl ood vol ume i s decreas ed as wel l . The decreas e i n effect i ve art eri al bl ood vol ume t ri ggers t he rel eas e of reni n and al dos t erone, l eadi ng t o s t i mul at i on of di s t al t ubul ar reabs orpt i on of s odi um. In addi t i on, al t erat i ons i n renal hemodynami cs s t i mul at e an i ncreas e i n t he proxi mal t ubul ar reabs orpt i on of s odi um.
o
o
b. Cirrhosis. The pri mary caus e of s odi um ret ent i on
Pa g e 1 3 9 5
ABC Ambe r CHM Conve rte r Tria l ve rsion, http://w w w .proce sste x t.com/a bcchm.html
i n l i ver di s eas e may be as ci t es format i on as a res ul t of hi gh pres s ure i n t he port al ci rcul at i on. Port al hypert ens i on l eads t o i nt ravas cul ar fl ui d vol ume depl et i on and s econdary renal s odi um ret ent i on. W hen hypoal bumi nemi a occurs , effect i ve art eri al bl ood vol ume drops , s t i mul at i ng renal s odi um ret ent i on. Secondary hyperal dos t eroni s m i s common and i s caus ed by i nt ravas cul ar vol ume cont ract i on as wel l as i mpai red hepat i c cl earance of al dos t erone; t hes e fact ors l ead t o s t i mul at i on of di s t al t ubul ar reabs orpt i on of s odi um. A pri mary i ncreas e i n renal s odi um reabs orpt i on may occur i n ci rrhos i s . o
o
c. Nephrotic syndrome. Hypoal bumi nemi a l eads t o reduced effect i ve art eri al bl ood vol ume, as a res ul t of renal prot ei n l os s es , whi ch s t i mul at es renal t ubul ar reabs orpt i on of s odi um.
o
o
d. Chronic renal failure. W hen t he GFR fal l s t o l es s t han 10 mL/mi nut e, t he capaci t y of t he ki dney t o excret e t he t ypi cal di et ary s odi um l oad i s l i mi t ed, and edema may res ul t .
o
o
e. Excessive mineralocorticoid activity. Tumors of t he adrenal gl and and pi t ui t ary t umors t hat s ecret e l arge amount s of ACTH may be as s oci at ed wi t h marked s odi um ret ent i on.
4. Clinical features o
o
a. Peripheral edema. Sodi um ret ent i on may
Pa g e 1 3 9 6
ABC Ambe r CHM Conve rte r Tria l ve rsion, http://w w w .proce sste x t.com/a bcchm.html
mani fes t as s wel l i ng i n t he dependent regi ons of t he body. o
o
b. Pulmonary edema. If pul monary venous pres s ure acut el y ri s es above 18 mm Hg, pul monary edema may devel op.
P.282
5. Diagnosis o
o
a. On physical examination, peri pheral edema may be i dent i fi ed by t he pers i s t ence of an i ndent at i on fol l owi ng pal pat i on of t he s oft t i s s ues i n t he dependent areas . Pul monary edema i s i dent i fi ed by t he phys i cal fi ndi ngs of ral es or wheezes or by ches t radi ography.
o
o
b. Urine sodium assay reveal s a uri ne s odi um l evel t hat i s l es s t han s odi um i nt ake and t hat us ual l y i s s i gni fi cant l y l es s t han 20 mEq/L.
6. T herapy o
o
a. Dietary sodium restriction i s es s ent i al . A s odi um i nt ake of 2 g/day i s t he l owes t pract i cal i nt ake l evel t hat can be achi eved.
o
o
b. Diuretics are us eful for i ncreas i ng s odi um
Pa g e 1 3 9 7
ABC Ambe r CHM Conve rte r Tria l ve rsion, http://w w w .proce sste x t.com/a bcchm.html
excret i on.
(1) Loop diuretics s uch as furos emi de, t ors emi de, and bumet ani de are part i cul arl y effect i ve. Si de effect s of t hes e drugs i ncl ude i nt ravas cul ar vol ume depl et i on wi t h azot emi a, hyperuri cemi a, hypokal emi a, met abol i c al kal os i s , and hypomagnes emi a.
(2) T hiazide diuretics s uch as hydrochl orot hi azi de i nhi bi t s odi um reabs orpt i on i n t he di s t al convol ut ed t ubul e. They can be us ed effect i vel y, al t hough t hey are l es s pot ent t han l oop di uret i cs . Si de effect s are s i mi l ar t o t hos e of l oop di uret i cs .
(3) Potassium-sparing diuretics s uch as ami l ori de and t ri amt erene act pri mari l y t o bl ock s odi um reabs orpt i on and s econdari l y t o bl ock pot as s i um s ecret i on i n t he di s t al t ubul e. Us e of t hes e agent s may l ead t o pot as s i um ret ent i on and i ncreas ed s odi um excret i on.
(4) Aldosterone antagonists s uch as t he compet i t i ve agent s pi ronol act one or epl erenone al s o l ead t o pot as s i um ret ent i on and i ncreas ed s odi um excret i on.
III. Potassium Metabolism
A. Normal physiology
Pa g e 1 3 9 8
ABC Ambe r CHM Conve rte r Tria l ve rsion, http://w w w .proce sste x t.com/a bcchm.html
Pot as s i um i s t he pri mary cat i oni c component of t he i nt racel l ul ar fl ui d, whi ch cont ai ns approxi mat el y 4000 mEq pot as s i um. In compari s on, t he ext racel l ul ar fl ui d cont ai ns very l i t t l e pot as s i um—about 65 mEq. The rat i o of ext racel l ul ar t o i nt racel l ul ar pot as s i um concent rat i on i s an i mport ant det ermi nant of el ect ri cal act i vi t y i n exci t abl e membranes (e.g., t he cardi ac conduct i on s ys t em and s omat i c nerve endi ngs ). The normal di et ary pot as s i um i nt ake of 60–90 mEq/day mus t be excret ed by t he ki dney t o pres erve pot as s i um homeos t as i s . Al s o, di et ary pot as s i um i nt ake mus t be t aken up rapi dl y by cel l s i n preparat i on for renal excret i on; ot herwi s e, t he s erum pot as s i um rapi dl y ri s es t o l i fe-t hreat eni ng l evel s .
1. Extrarenal handling. Cel l ul ar upt ake of pot as s i um i s i nfl uenced by t he fol l owi ng ext rarenal fact ors : o
o
a. Insulin. Hi gh i ns ul i n l evel s s t i mul at e cel l ul ar upt ake of pot as s i um.
o
o
b. Epinephrine. Thi s β 2 -act i ve cat echol ami ne di rect l y s t i mul at es cel l ul ar upt ake of pot as s i um. Thi s act i on may be part i cul arl y i mport ant duri ng s evere exert i on, when s erum pot as s i um l evel s ri s e becaus e of mus cl e i s chemi a.
2. Renal handling. Mos t uri ne pot as s i um i s t he res ul t of di s t al t ubul ar s ecret i on. Several fact ors are known t o al t er pot as s i um s ecret i on. o
o
a. Aldosterone secretion. Al dos t erone di rect l y s t i mul at es pot as s i um s ecret i on and s odi um reabs orpt i on i n t he col l ect i ng t ubul e of t he ki dney.
Pa g e 1 3 9 9
ABC Ambe r CHM Conve rte r Tria l ve rsion, http://w w w .proce sste x t.com/a bcchm.html o
o
b. Sodium reabsorption. The del i very of fl ui d and s odi um t o t he col l ect i ng t ubul e al s o s t i mul at es pot as s i um s ecret i on. Thi s mechani s m account s for t he i ncreas ed pot as s i um s ecret i on caus ed by di uret i cs , whi ch act at more proxi mal s i t es i n t he nephron t o bl ock s odi um reabs orpt i on.
B. Hypokalemia
1. Definition. Hypokal emi a i s defi ned as s erum pot as s i um concent rat i on of l es s t han 3.5 mEq/L. Becaus e mos t of t he pot as s i um cont ent of t he body i s wi t hi n cel l s and cel l ul ar pot as s i um concent rat i on i s about 155 mEq/L, cel l ul ar pot as s i um can be s everel y depl et ed wi t hout caus i ng l arge changes i n s erum pot as s i um.
2. Etiology. Hypokal emi a can res ul t from ext rarenal or renal caus es . o
o
a. Extrarenal causes
(1) Dietary deficiency and gastrointestinal losses
(a) Inadequate dietary intake. Becaus e pot as s i um cons ervat i on i n t he ki dney i s l i mi t ed, a s evere reduct i on of i nt ake t o l es s t han 10 mEq/day for many days or weeks can l ead t o a l arge negat i ve
Pa g e 1 4 0 0
ABC Ambe r CHM Conve rte r Tria l ve rsion, http://w w w .proce sste x t.com/a bcchm.html
pot as s i um bal ance and hypokal emi a.
(b) Diarrhea. Becaus e t he pot as s i um cont ent of di arrheal fl ui d may be as hi gh as 100 mEq/L, di arrhea can l ead t o s evere pot as s i um depl et i on.
(c) Vomiting. Al t hough t he pot as s i um cont ent of vomi t us i s rel at i vel y s mal l , t he s econdary effect of i nt ravas cul ar vol ume depl et i on, whi ch produces s econdary hyperal dos t eroni s m, s t i mul at es renal pot as s i um excret i on.
(2) Potassium redistribution
(a) Insulin administration. A t herapeut i c or repl acement dos e of i ns ul i n can dri ve pot as s i um i nt o cel l s , produci ng acut e hypokal emi a.
(b) Epinephrine infusions. Epi nephri ne al s o can produce acut e hypokal emi a by an i ndependent act i on i nvol vi ng β 2 -recept ors .
(c) Folic acid and vitamin B 1 2 therapy. In pat i ent s wi t h megal obl as t i c anemi a, fol i c aci d and vi t ami n B 1 2 s t i mul at e cel l prol i ferat i on, t hus produci ng acut e hypokal emi a, becaus e pot as s i um i s us ed
Pa g e 1 4 0 1
ABC Ambe r CHM Conve rte r Tria l ve rsion, http://w w w .proce sste x t.com/a bcchm.html
i n cel l s ynt hes i s . Thi s effect al s o may be s een i n pat i ent s wi t h rapi dl y growi ng t umors .
(d) Alkalemia. In t he pres ence of exces s bas e, hydrogen i ons s hi ft out of cel l s i n exchange for pot as s i um.
(e) Hypokalemic periodic paralysis. In t hi s rare s yndrome, pot as s i um l evel s fal l acut el y—wi t hout a l os s of pot as s i um from t he body—pri or t o epi s odes of paral ys i s . Thi s s yndrome, whi ch i s commonl y as s oci at ed wi t h t hyroi d di s eas e i n As i ans , probabl y repres ent s a defect i n cat echol ami ne s ens i t i vi t y.
P.283
o
o
b. Renal causes. Any hyperact i vi t y of t he normal component s of renal pot as s i um excret i on can produce a negat i ve pot as s i um bal ance vi a i ncreas ed renal l os s es .
(1) Drug-induced renal losses
(a) Diuretics. Agent s t hat act proxi mal t o t he s i t e of pot as s i um s ecret i on s t i mul at e uri nary excret i on of pot as s i um by i ncreas i ng t he del i very of s odi um and fl ui d t o t he di s t al t ubul es .
Pa g e 1 4 0 2
ABC Ambe r CHM Conve rte r Tria l ve rsion, http://w w w .proce sste x t.com/a bcchm.html
(b) Penicillins. Carbeni ci l l i n, t i carci l l i n, and rel at ed drugs act as nonreabs orbabl e ani ons i n t he di s t al t ubul e and t hereby s t i mul at e pot as s i um s ecret i on. Si gni fi cant hypokal emi a i s commonl y s een.
(c) Aminoglycosides. Tubul ar defect s wi t h magnes i um was t i ng and s econdary pot as s i um was t i ng occas i onal l y may be s een i n pat i ent s t reat ed wi t h l arge dos es of gent ami ci n or rel at ed compounds .
(d) Amphotericin B. Thi s ant i fungal agent caus es damage t o t he api cal membrane of t he renal t ubul ar cel l , t hus i ncreas i ng pot as s i um l os s from t he cel l .
(2) Hormone-induced renal losses
(a) Primary hyperaldosteronism
(i) Primary adrenal adenomas are as s oci at ed wi t h hypokal emi a, hypert ens i on, and met abol i c al kal os i s .
(ii) Diffuse bilateral adrenal hyperplasia may be as s oci at ed wi t h
Pa g e 1 4 0 3
ABC Ambe r CHM Conve rte r Tria l ve rsion, http://w w w .proce sste x t.com/a bcchm.html
a mi l der hypokal emi a t han i s s een wi t h pri mary adrenal adenoma.
(iii) In ectopic ACT H syndrome, mas s i ve mi neral ocort i coi d i ncreas e and renal pot as s i um was t i ng may occur i n pat i ent s wi t h s mal l cel l l ung carci noma (a t umor t hat produces and s ecret es ACTH).
(iv) Exogenous mineralocorticoid. Licorice produced i n Europe (ani s e) cont ai ns glycyrrhetinic acid, whi ch prevent s convers i on of cort i s ol t o cort i s one i n t he renal di s t al t ubul e, t hereby s t i mul at i ng mi neral ocort i coi d recept ors and produci ng an al dos t erone-l i ke act i on. Inges t i on of t hi s agent may l ead t o hypokal emi a wi t h hypert ens i on and met abol i c al kal os i s . Cert ai n tobacco compounds al s o cont ai n gl ycyrrhet i ni c aci d and may caus e hypokal emi a.
(b) Secondary hyperaldosteronism
(i) Renin-secreting tumor. Thi s rare ent i t y, di agnos ed by art eri ography, i s charact eri zed by i nt rarenal t umors of t he juxt agl omerul ar apparat us . Severe
Pa g e 1 4 0 4
ABC Ambe r CHM Conve rte r Tria l ve rsion, http://w w w .proce sste x t.com/a bcchm.html
hypert ens i on and hypokal emi a may occur.
(ii) Renal artery stenosis may be as s oci at ed wi t h hypokal emi a and hypert ens i on as a res ul t of s econdary hyperal dos t eroni s m produced by hyperreni nemi a.
(iii) In malignant hypertension, s evere underperfus i on of t he ki dney may occur and may l ead t o hyperreni nemi a, s econdary hyperal dos t eroni s m, and hypokal emi a.
(iv) Disorders with reduced effective arterial blood volume produce onl y mi l d hypokal emi a des pi t e hyperreni nemi a and hyperal dos t eroni s m. Reduced t ubul ar fl ow rat e reduces pot as s i um s ecret i on. In CHF, s econdary hyperal dos t eroni s m may devel op, caus i ng mi l d hypokal emi a even i n t he abs ence of di uret i c us e. In cirrhosis, s evere hypokal emi a i s common becaus e of l ow i nt ake of pot as s i um and s econdary hyperal dos t eroni s m.
(3) Potassium loss due to primary renal tubular disorders
Pa g e 1 4 0 5
ABC Ambe r CHM Conve rte r Tria l ve rsion, http://w w w .proce sste x t.com/a bcchm.html
(a) Renal tubular acidosis i s oft en as s oci at ed wi t h pot as s i um was t i ng, whi ch may be s econdary t o s odi um depl et i on and met abol i c aci dos i s or di rect l y at t ri but abl e t o t ubul ar defect s i n pot as s i um cons ervat i on. Pot as s i um was t i ng i s a feat ure of di s t al (t ype I) as wel l as proxi mal (t ype II) renal t ubul ar aci dos i s of any et i ol ogy.
(b) Bartter' s syndrome and Gitelman' s syndrome are charact eri zed by renal pot as s i um was t i ng, met abol i c al kal os i s , and pol yuri a. Bl ood pres s ure us ual l y i s normal or reduced, but reni n and al dos t erone l evel s are very hi gh. In Bartter' s syndrome, t he pri mary defect l i es i n one of t he epi t hel i al t rans port prot ei ns i nvol ved i n t he reabs orpt i on of s odi um chl ori de, mos t commonl y t he +
+
furos emi de-s ens i t i ve Na –K –2Cl
-
cot rans port er i n t he as cendi ng l i mb of Henl e's l oop. In Gitelman' s syndrome, t he pri mary defect i nvol ves t he +
t hi azi de-s ens i t i ve Na –Cl
-
cot rans port er i n t he di s t al convol ut ed t ubul e. P.284
(c) Chronic magnesium depletion
Pa g e 1 4 0 6
ABC Ambe r CHM Conve rte r Tria l ve rsion, http://w w w .proce sste x t.com/a bcchm.html
produces a s yndrome of renal t ubul ar pot as s i um was t i ng wi t hout ot her as s oci at ed defect s i n i on t rans port . The pot as s i um was t i ng can be s evere and i s unres pons i ve t o pot as s i um repl et i on unt i l magnes i um defi ci t s have been correct ed.
(4) Potassium loss due to surreptitious diuretic use i s as s oci at ed wi t h a cl i ni cal pres ent at i on i dent i cal t o t hat of Bart t er's or Gi t el man's s yndrome, i ncl udi ng hypokal emi a, magnes i um was t i ng, met abol i c al kal os i s , hyperreni nemi a, and hyperal dos t eroni s m.
3. Clinical features o
o
a. Neuromuscular disorders. Pot as s i um depl et i on may caus e weaknes s and paral ys i s .
o
o
b. Cardiac disorders. Arrhyt hmi a, part i cul arl y i n t he pres ence of di gi t al i s i nt oxi cat i on, i s a hal l mark of s evere hypokal emi a.
o
o
c. Endocrine disorders. Hypokal emi a i s as s oci at ed wi t h abnormal i t i es i n pancreat i c i ns ul i n rel eas e. Gl ucos e i nt ol erance has been s hown t o wors en as a res ul t of di uret i c-i nduced hypokal emi a.
o
o
d. Polyuria. The pol yuri a of hypokal emi a i s a funct i on of pol ydi ps i a as wel l as i mpai red ADH
Pa g e 1 4 0 7
ABC Ambe r CHM Conve rte r Tria l ve rsion, http://w w w .proce sste x t.com/a bcchm.html
act i on.
4. Diagnosis o
o
a. Physical examination. The pres ence or abs ence of hypert ens i on i s a us eful di fferent i at i ng feat ure i n t he approach t o t he pat i ent wi t h hypokal emi a.
(1) If t he pat i ent i s hypert ens i ve, t he hypokal emi a may be caus ed by exces s i ve mi neral ocort i coi d act i vi t y. Becaus e many hypert ens i ve pat i ent s are t reat ed wi t h di uret i cs , any hypokal emi a coul d be a s i de effect of s uch t herapy.
(2) If t he pat i ent i s normot ens i ve, t he hypokal emi a repres ent s ei t her a gas t roi nt es t i nal or a pri mary renal l os s of pot as s i um.
o
o
b. Serum electrolyte assay. Thi s t es t rarel y i s us eful for eval uat i ng t he s peci fi c caus e of hypokal emi a. However, t he fi ndi ng of combi ned aci dos i s and hypokal emi a, whi ch s ugges t s renal t ubul ar aci dos i s , i s an except i on.
o
o
c. Urine potassium assay. Uri ne pot as s i um l evel s bel ow 20 mEq/L s ugges t ext rarenal pot as s i um l os s es , whereas l evel s exceedi ng 30 mEq/L s ugges t renal l os s es .
o
Pa g e 1 4 0 8
ABC Ambe r CHM Conve rte r Tria l ve rsion, http://w w w .proce sste x t.com/a bcchm.html
o
d. Renin–aldosterone axis assay
(1) Noninvasive tests. Several noni nvas i ve t ext s can be us ed t o det ermi ne whet her exces s i ve mi neral ocort i coi d act i vi t y i s due t o exces s i ve reni n product i on or t o a pri mary adrenal di s order (Tabl e 6-16).
(2) Invasive tests i ncl ude meas urement of bi l at eral renal venous reni n as wel l as adrenal venous al dos t erone and cort i s ol concent rat i ons . However, t hes e t es t s are rarel y performed, as t he di agnos i s can us ual l y be made by noni nvas i ve means .
TABLE 6-16 Renin–Aldosterone Axis Assay Sig nifi Me ca T e tho nc st d e Re 40 No ni n mg s t i s t i fur mu mu os e l at i l at i mi on on de of t es i s t
ren
ad i n mi l ev ni s el s
Pa g e 1 4 0 9
ABC Ambe r CHM Conve rte r Tria l ve rsion, http://w w w .proce sste x t.com/a bcchm.html
t er i n ed; upr t he i gh n
t
pl a pos sm itio a
n
ren i nd in
i ca
is
t es
me ren as u i n red s up in
pre
bot s s i h
on
s up i ne an d upr i gh t pos itio ns Al d 1â Lac os t €“2 k ero L
of
ne of
al d
s up s al os t pre i ne ero s s i are ne on i nf s up t es us e pre
Pa g e 1 4 1 0
ABC Ambe r CHM Conve rte r Tria l ve rsion, http://w w w .proce sste x t.com/a bcchm.html
t
d; s s i t he on n
to
pl a bel s
ow
ma nor al d ma os t l ero ma ne y is
i nd
me i ca as u t e red pri in
ma
bot ry h
al d
s up os t i ne ero an ne d
ove
upr rpr i gh od t
uct
pos i on itio ns Pl a Ser Val s m um ue a
ren gre
al d i n
at e
os t an r ero d
t ha
ne- al d n t o- os t 15
Pa g e 1 4 1 1
ABC Ambe r CHM Conve rte r Tria l ve rsion, http://w w w .proce sste x t.com/a bcchm.html
ren ero i s in
ne hi g
rat i l ev hl y o
el s s ug are ges me t i v as u e red of pri ma ry hyp era l do ste ron
ism Bi l (s e Hy at e e
per
ral †t en ren œT s i o al
es t n
ven †of ous •
al d
ren col os t i n; um ero adr n) no en
ma
al
is
ven
res
ous
po
al d
ns i
os t
ve
ero
to
ne
tu
Pa g e 1 4 1 2
ABC Ambe r CHM Conve rte r Tria l ve rsion, http://w w w .proce sste x t.com/a bcchm.html
an
mo
d
r
cor
re
tis
mo
ol
val
con
(hy
cen
per
t ra
t en
tio
sio
ns
n as s oci at e d wi t h bi l at e ral di s eas e is not )
P.285
o
o
e. Urine chloride assay. In cas es of mi neral ocort i coi d exces s , Bart t er's or Gi t el man's s yndrome, and di uret i c abus e, uri ne chl ori de l evel s t end t o be el evat ed i n t he pres ence of met abol i c al kal os i s and hypokal emi a. The abs ence of
Pa g e 1 4 1 3
ABC Ambe r CHM Conve rte r Tria l ve rsion, http://w w w .proce sste x t.com/a bcchm.html
el evat ed uri ne chl ori de l evel s i s hi ghl y s ugges t i ve of gas t roi nt es t i nal pot as s i um l os s es s uch as s urrept i t i ous vomi t i ng. o
o
f. Diuretic assay. If Bart t er's or Gi t el man's s yndrome i s s us pect ed, t he uri ne mus t be anal yzed for chl ori de and di uret i cs , i ncl udi ng l oop-act i ve agent s and t hi azi des , before a di agnos i s of a pri mary t ubul ar di s order can be es t abl i s hed. Such di uret i c as s ays are commerci al l y avai l abl e.
5. T herapy. In many cas es , hypokal emi a can be correct ed by admi ni s t rat i on of pot as s i um s al t s . o
o
a. Forms of potassium salts. Pot as s i um may be admi ni s t ered wi t h a vari et y of ani ons . Pot as s i um chl ori de i s t he preferred form of t herapy, becaus e many pat i ent s have concurrent chl ori de defi ci t s . In cas es of hypokal emi a wi t h coi nci dent renal t ubul ar aci dos i s , pot as s i um ci t rat e, pot as s i um l act at e, or pot as s i um gl uconat e may be gi ven.
o
o
b. Routes of administration. Pot as s i um may be gi ven i nt ravenous l y or oral l y.
(1) Int ravenous pot as s i um s ol ut i ons s houl d not exceed a concent rat i on of 60 mEq/L, and t he rat e of admi ni s t rat i on s houl d not exceed 60 mEq/hour. Normal l y, pot as s i um defi ci t s are on t he order of 300–1000 mEq. Thes e defi ci t s s houl d be repl aced s l owl y, over days , except when di gi t al i s i nt oxi cat i on or
Pa g e 1 4 1 4
ABC Ambe r CHM Conve rte r Tria l ve rsion, http://w w w .proce sste x t.com/a bcchm.html
l i fe-t hreat eni ng arrhyt hmi as are pres ent .
(2) Oral pot as s i um i s abs orbed effect i vel y and s houl d be s ubs t i t ut ed for i nt ravenous pot as s i um whenever pos s i bl e.
o
o
c. Chronic potassium therapy. Hypokal emi c pat i ent s recei vi ng di uret i cs s houl d be gi ven pot as s i um s uppl ement at i on t o mai nt ai n t he s erum pot as s i um l evel above 3.5 mEq/L. This l evel may be accompl i s hed wi t h oral pot as s i um s uppl ement at i on. Al t hough s ome foods are hi gh i n pot as s i um, i t i s di ffi cul t t o overcome t hes e defi ci t s s ol el y by i nges t i ng pot as s i um-ri ch food.
C. Hyperkalemia
1. Definition. Hyperkal emi a i s defi ned as s erum pot as s i um concent rat i on great er t han 5.5 mEq/L.
2. Etiology. Pseudohyperkalemia may be caus ed by rel eas e of pot as s i um from coagul at ed cel l s and pl at el et s aft er bl ood i s wi t hdrawn for anal ys i s . It can al s o occur i f t he pl at el et or whi t e bl ood cel l (W BC) count i s ext remel y hi gh, as i n myel oprol i ferat i ve di s orders . Meas urement of pl as ma pot as s i um i s requi red t o el i mi nat e t hi s art i fact . T rue hyperkalemia may res ul t from ext rarenal or renal caus es . o
o
a. Extrarenal causes
Pa g e 1 4 1 5
ABC Ambe r CHM Conve rte r Tria l ve rsion, http://w w w .proce sste x t.com/a bcchm.html
(1) Insulin deficiency. Hyperkal emi a i n di abet i c pat i ent s may be due t o a l ack of i ns ul i n and t o t he pres ence of as s oci at ed renal and adrenal abnormal i t i es .
(2) Cell lysis syndromes. Acut e cel l necros i s fol l owi ng ei t her chemot herapy or a mas s i ve crus hi ng i njury (rhabdomyolysis) produces hyperkal emi a by rapi d cel l ul ar rel eas e of pot as s i um.
(3) Succinylcholine therapy. The mus cl e rel axant s ucci nyl chol i ne may produce hyperkal emi a i n s us cept i bl e i ndi vi dual s wi t h general i zed mus cl e or neurol ogi c di s eas e.
(4) Hyperkalemic periodic paralysis. Thi s rare, fami l i al s yndrome t hat may be as s oci at ed wi t h an acut e s hi ft of ext racel l ul ar pot as s i um. [The hypokal emi c form of t hi s s yndrome, whi ch i s more common, i s di s cus s ed onl i ne Part II: III B 2 a (2) (e).]
(5) Hyperosmolality. Acut e i ncreas es i n ext racel l ul ar fl ui d os mot i c act i vi t y may produce a t rans cel l ul ar s hi ft of pot as s i um and res ul t i n hyperkal emi a. Thi s may occur wi t h admi ni s t rat i on of i nt ravenous cont ras t or gl ucos e.
(6) Acidosis. Mi neral aci dos i s , not organi c aci dos i s , may be as s oci at ed wi t h an acut e
Pa g e 1 4 1 6
ABC Ambe r CHM Conve rte r Tria l ve rsion, http://w w w .proce sste x t.com/a bcchm.html
s hi ft of pot as s i um from t he i nt racel l ul ar t o t he ext racel l ul ar fl ui d as hydrogen i ons ent er cel l s . o
o
b. Renal causes. The renal capaci t y t o excret e pot as s i um i s approxi mat el y 500–1000 mEq/day, whi ch i s 10–20 t i mes t he normal i nt ake. An i mpai rment of t he normal component s of renal pot as s i um excret i on may reduce t hi s excret ory capaci t y s o t hat normal i nt ake may produce hyperkal emi a.
(1) Severe renal failure. W hen t he GFR fal l s t o bel ow 10 mL/mi nut e, hyperkal emi a may occur, even wi t h normal i nt ake. At a GFR above t hi s l evel , hyperkal emi a i s not a res ul t of gl omerul ar i ns uffi ci ency per s e but i s t he res ul t of a s peci fi c di s order i n t ubul ar pot as s i um t rans port or an ext rarenal pot as s i um di s t urbance.
(2) Aldosterone insufficiency. Al dos t erone i s t he major hormonal det ermi nant of renal pot as s i um s ecret i on. P.286
(a) Acquired aldosterone deficiency. Thi s condi t i on may res ul t from renal di s eas e as s oci at ed wi t h reduced reni n product i on. (Recal l t hat reni n i s an
Pa g e 1 4 1 7
ABC Ambe r CHM Conve rte r Tria l ve rsion, http://w w w .proce sste x t.com/a bcchm.html
enzyme t hat cl eaves precurs or mol ecul es t o produce t he al dos t erone s ecret agogue, angi ot ens i n II.) Pri mary adrenal di s eas e al s o may be as s oci at ed wi t h reduced al dos t erone product i on. Al dos t erone defi ci ency due t o i mpai red reni n product i on or adrenal di s eas e may be produced by:
(i) Int ers t i t i al renal di s eas e
(ii) Lead nephropat hy
(iii) Di abet i c nephropat hy (i ns ul i n defi ci ency i n t hi s condi t i on may pot ent i at e hyperkal emi a)
(iv) Obs t ruct i ve uropat hy
(v) Angi ot ens i n ant agoni s t t herapy
(vi) Addi s on's di s eas e
(b) Inherited aldosterone deficiency. Several adrenal enzyme defect s as s oci at ed wi t h defi ci ency of t he 17- or 21-hydroxyl as e enzymes may be as s oci at ed wi t h al dos t erone defi ci ency.
(c) Drug-induced aldosterone
Pa g e 1 4 1 8
ABC Ambe r CHM Conve rte r Tria l ve rsion, http://w w w .proce sste x t.com/a bcchm.html
deficiency. NSAIDs and hepari n act t o reduce reni n s ecret i on or i nduce res i s t ance and may produce hyperkal emi a t hrough al dos t erone defi ci ency.
(3) Aldosterone resistance. The fol l owi ng condi t i ons are charact eri zed by t ubul ar defect s as s oci at ed wi t h el evat ed al dos t erone l evel s but i mpai red pot as s i um s ecret i on.
(a) Si ckl e cel l nephropat hy
(b) SLE
(c) Amyl oi dos i s
(d) Int ers t i t i al renal di s eas e
(e) Obs t ruct i ve uropat hy
(f) Heredi t ary al dos t erone res i s t ance
(g) Us e of t ri amt erene, ami l ori de, or s pi ronol act one
3. Clinical features o
o
a. Neuromuscular disorders. By al t eri ng
Pa g e 1 4 1 9
ABC Ambe r CHM Conve rte r Tria l ve rsion, http://w w w .proce sste x t.com/a bcchm.html
t rans membrane el ect ri cal pot ent i al , s evere hyperkal emi a may al t er mus cl e funct i on or neuromus cul ar t rans mi s s i on, l eadi ng t o s evere weaknes s or paral ys i s . o
o
b. Cardiac disorders. Cardi ac arrhyt hmi as may occur at any l evel above normal but general l y are not ed onl y when s erum pot as s i um concent rat i on exceeds 6 mEq/L. As s erum pot as s i um l evel ri s es , a s eri es of el ect rocardi ographi c (ECG) changes may be s een, i ncl udi ng:
(1) Prol ongat i on of t he P-R i nt erval
(2) T-wave peaki ng
(3) Prol ongat i on of t he QRS i nt erval
(4) Vent ri cul ar t achycardi as , vent ri cul ar fi bri l l at i on, and as ys t ol e
4. Diagnosis o
o
a. Elimination of pseudohyperkalemia. In vi t ro l ys i s of eryt hrocyt es , l eukocyt es , or pl at el et s can produce hyperkal emi a as a res ul t of i nt racel l ul ar pot as s i um rel eas e (ps eudohyperkal emi a). Al l hyperkal emi c pat i ent s s houl d be checked for ps eudohyperkal emi a by meas uri ng bot h pl as ma and s erum pot as s i um concent rat i ons and by i ns pect i ng
Pa g e 1 4 2 0
ABC Ambe r CHM Conve rte r Tria l ve rsion, http://w w w .proce sste x t.com/a bcchm.html
t he s erum for di s col orat i on s ugges t i ng hemol ys i s . The pres ence of a myel oprol i ferat i ve di s order may al s o produce ps eudohyperkal emi a. o
o
b. Urine potassium assay. Al t hough onl y a rough correl at i on exi s t s bet ween uri ne and s erum pot as s i um l evel s , hyperkal emi a i nduced by i ncreas ed i nt ake or i ncreas ed cel l l ys i s s houl d be as s oci at ed wi t h uri ne pot as s i um l evel s exceedi ng 50 mEq/L. Val ues l es s t han 30 mEq/L i n t he s et t i ng of hyperkal emi a s ugges t i mpai red renal s ecret i on of pot as s i um.
o
o
c. Renin–aldosterone axis assay. In cert ai n pat i ent s , eval uat i on of al dos t erone and reni n l evel s may hel p defi ne t he et i ol ogy of hyperkal emi a.
5. T herapy. Treat ment i s di vi ded i nt o acut e and chroni c phas es . o
o
a. Acute antagonism and redistribution
(1) Calcium. The i nt ravenous admi ni s t rat i on of 1–2 ampul es of cal ci um chl ori de acut el y ant agoni zes t he cardi ac effect s of hyperkal emi a. ECG changes may t rans i ent l y i mprove, but s erum pot as s i um l evel remai ns el evat ed. P.287
Pa g e 1 4 2 1
ABC Ambe r CHM Conve rte r Tria l ve rsion, http://w w w .proce sste x t.com/a bcchm.html
(2) Glucose and insulin. The i nt ravenous i nfus i on of 25 g (1 ampul e) of dext ros e pl us 15 uni t s of i ns ul i n l owers s erum pot as s i um wi t hi n 10–15 mi nut es .
(3) β 2 -Adrenergic agonists, gi ven by i nhal at i on or i nt ravenous l y, rapi dl y i nduce pot as s i um upt ake i nt o cel l s , but t hey are not uni forml y effect i ve.
o
o
b. Acute removal
(1) Diuretics. Furos emi de, bumet ani de, and, es peci al l y, acet azol ami de i ncreas e pot as s i um excret i on i n i ndi vi dual s wi t h adequat e renal funct i on.
(2) Cation-exchange resins. The admi ni s t rat i on of s odi um pol ys t yrene s ul fonat e bi nds pot as s i um i n t he gas t roi nt es t i nal t ract . About 2 mEq of s odi um are exchanged for every 1 mEq of pot as s i um removed, s o t hat a s ubs t ant i al s odi um l oad may res ul t . Sorbi t ol i s admi ni s t ered oral l y t o prevent s evere cons t i pat i on. Cat i on-exchange res i ns can remove 50–100 mEq of pot as s i um over a 6-hour peri od and may be gi ven oral l y or rect al l y.
o
o
c. Chronic removal. Aft er t he acut e removal of pot as s i um, pot as s i um homeos t as i s may be
Pa g e 1 4 2 2
ABC Ambe r CHM Conve rte r Tria l ve rsion, http://w w w .proce sste x t.com/a bcchm.html
mai nt ai ned wi t h any of t he fol l owi ng agent s .
(1) Diuretics. Furos emi de or acet azol ami de may be us ed i n combi nat i on wi t h fl udrocort i s one acet at e t o i ncreas e pot as s i um excret i on.
(2) Cation-exchange resins may be gi ven on a chroni c bas i s t o i ncreas e gas t roi nt es t i nal excret i on of met abol i s m.
IV. Acid–Base Metabolism
A. Normal physiology Aci d–bas e bal ance refers t o t he mai nt enance of t he hydrogen i on concent rat i on of body fl ui ds by t hree cont rol s ys t ems : body buffers (e.g., bi carbonat e), t he l ungs , and t he ki dneys . Becaus e hydrogen i ons (prot ons ) are hi ghl y react i ve part i cl es , even s l i ght changes i n hydrogen i on concent rat i on can caus e marked al t erat i ons i n phys i ol ogi c proces s es .
1. Hydrogen ion concentration and pH. The hydrogen i on concent rat i on of body fl ui ds i s l ow compared wi t h t he concent rat i ons of ot her i ons . It i s more conveni ent , t herefore, t o expres s t he concent rat i on as pH, or t he negative logarithm of hydrogen ion concentration. The pH of t he ext racel l ul ar fl ui d i s mai nt ai ned at about 7.4.
2. Generation and elimination of hydrogen ion. Normal met abol i c proces s es generat e l arge amount s of carboni c as wel l as noncarboni c (nonvol at i l e) aci ds , whi ch ent er
Pa g e 1 4 2 3
ABC Ambe r CHM Conve rte r Tria l ve rsion, http://w w w .proce sste x t.com/a bcchm.html
t he body fl ui ds and mus t be buffered and el i mi nat ed. o
o
a. Carbonic acid. Hydrogen i ons are produced t hrough compl et e oxi dat i on of gl ucos e and fat t y aci ds t o carboni c aci d. On dehydrat i on, carboni c aci d forms a vol at i l e end product (carbon di oxi de), whi ch can be el i mi nat ed by t he l ungs .
o
o
b. Nonvolatile acid. Approxi mat el y 1 mEq of nonvol at i l e aci d per ki l ogram of body wei ght i s produced dai l y and excret ed pri mari l y by t he ki dneys . Thi s aci d i s produced t hrough i ncompl et e met abol i s m of gl ucos e and fat t y aci ds t o organi c aci ds (e.g., acet oacet i c and β-hydroxybut yri c aci ds ) as wel l as met abol i s m of prot ei ns s uch as met hi oni ne and phos phoprot ei n t o s ul furi c and phos phori c aci ds , res pect i vel y.
3. Henderson-Hasselbalch equation. The bi carb -
onat e–carboni c aci d (HCO 3 –CO 2 ) s ys t em i s t he major buffer component of t he ext racel l ul ar fl ui d. Aci d–bas e di s t urbances oft en are charact eri zed i n t erms of changes i n ei t her t he bi carbonat e (bas e) or di s s ol ved carbon di oxi de (aci d) component of t hi s buffer pai r. The cl as s i c expres s i on of aci d–bas e s t at e i s bas ed on t he Henders on-Has s el bal ch equat i on, whi ch rel at es t hree vari abl es —pH, carbon di oxi de t ens i on (Pco 2 ), and -
pl as ma bi carbonat e concent rat i on ([HCO 3 ])—as wel l as t wo cons t ant s —pK and S—as :
where: pK = t he negat i ve l ogari t hm of t he di s s oci at i on cons t ant for carboni c aci d (6.1) and S = t he s ol ubi l i t y cons t ant
Pa g e 1 4 2 4
ABC Ambe r CHM Conve rte r Tria l ve rsion, http://w w w .proce sste x t.com/a bcchm.html
for carbon di oxi de i n pl as ma (0.03 mmol /L/mm Hg). Normal l y, -
t he pl as ma [HCO 3 ] i s 24 mmol /L and t he art eri al Pco 2 i s 40 mm Hg. Thus :
4. Respiratory regulation of arterial PCO 2 . By regul at i ng t he rat e of al veol ar vent i l at i on, t he l ungs may ret ai n or excret e carbon di oxi de and i n t hi s way regul at e t he “aci d― component of t he bi carbonat e buffer s ys t em.
5. Renal regulation of plasma bicarbonate content. There are t wo i mport ant as pect s of hydrogen i on met abol i s m i n t he ki dney: t he reabs orpt i on of bi carbonat e i on and t he s ecret i on of hydrogen i on. o
o
a. Reabsorption of bicarbonate ion. Approxi mat el y 4500 mEq of bi carbonat e are fi l t ered dai l y at t he gl omerul us , vi rt ual l y al l of whi ch are reabs orbed i nt o t he proxi mal t ubul es . The remai ni ng mi nut e port i on of t he fi l t ered bi carbonat e i s reabs orbed i nt o t he di s t al and col l ect i ng t ubul es . Proxi mal t ubul ar reabs orpt i on of bi carbonat e occurs i ndi rect l y by t he fol l owi ng proces s .
(1) Fi l t ered bi carbonat e i on t oget her wi t h s ecret ed hydrogen i on form carboni c aci d wi t hi n t he t ubul ar l umen. Sodi um i on reabs orpt i on i s l i nked t o t hi s s ecret i on of hydrogen i on t o mai nt ai n el ect roneut ral i t y.
Pa g e 1 4 2 5
ABC Ambe r CHM Conve rte r Tria l ve rsion, http://w w w .proce sste x t.com/a bcchm.html
(2) Carboni c anhydras e i n t he brus h border of t he proxi mal t ubul e cat al yzes t he dehydrat i on of carboni c aci d t o carbon di oxi de and wat er. Bei ng readi l y di ffus i bl e, carbon di oxi de di ffus es i nt o t he proxi mal t ubul ar cel l , where i nt racel l ul ar carboni c anhydras e cat al yzes i t s rehydrat i on t o carboni c aci d.
(3) The bi carbonat e i on formed by t he di s s oci at i on of carboni c aci d i s pas s i vel y reabs orbed i nt o t he peri t ubul ar bl ood al ong wi t h equi mol ar amount s of s odi um i on, whi ch i s act i vel y t rans port ed i nt o t he peri t ubul ar bl ood. The hydrogen i on formed by t he di s s oci at i on of carboni c aci d wi t hi n t he cel l s erves as a s ource of anot her hydrogen i on t o be s ecret ed. Thi s may occur i n exchange for s odi um i ons acros s t he api cal epi t hel i al membrane.
o
o
b. Addition of “ new― bicarbonate. In addi t i on t o cons ervi ng bi carbonat e, t he ki dneys add newl y s ynt hes i zed bi carbonat e t o t he pl as ma via secretion of hydrogen ion. Thi s proces s repl eni s hes t he bi carbonat e us ed t o buffer aci d produced by i ncompl et e met abol i s m of neut ral foods t uffs and by met abol i s m of nonvol at i l e aci d precurs ors i n t he di et .
(1) The addi t i on of new bi carbonat e does not i nvol ve t he bi carbonat e reabs orbed i nt o t he proxi mal t ubul e but , rat her, t he bi carbonat e
Pa g e 1 4 2 6
ABC Ambe r CHM Conve rte r Tria l ve rsion, http://w w w .proce sste x t.com/a bcchm.html
generat ed wi t hi n t he di s t al t ubul ar cel l vi a t he hydrat i on of carbon di oxi de and t he di s s oci at i on of carboni c aci d. Thi s proces s i s s i mi l ar t o t hat for t he reabs orpt i on of t he fi l t ered bi carbonat e; however, t he formed bi carbonat e i n t he cel l i s “new.―
(2) The renal cont ri but i on of new bi carbonat e i s accompani ed by t he excret i on of an equi val ent amount of aci d i n t he uri ne i n t he form of t i t rat abl e aci d, ammoni um i on, or bot h.
(a) T itratable acid formation and secretion. The exchange of hydrogen i on for s odi um i on convert s di bas i c s odi um phos phat e or s ul fat e i n t he gl omerul ar fi l t rat e i nt o monobas i c s odi um phos phat e or s ul fat e, whi ch i s excret ed i n t he uri ne as t i t rat abl e aci d. The hydrogen i on s ecret ed i nt o t he di s t al t ubul es , t herefore, can react wi t h fi l t ered phos phat e rat her t han fi l t ered bi carbonat e.
(b) Ammonia formation and secretion. Unl i ke phos phat e, ammoni a ent ers t he t ubul ar l umen by t ubul ar s ynt hes i s and s ecret i on, rat her t han fi l t rat i on. Ammoni a i s s ynt hes i zed i n t he proxi mal t ubul e as a product of gl ut ami ne met abol i s m, and t hen i t di ffus es acros s t he renal parenchyma t o be s ecret ed i nt o
Pa g e 1 4 2 7
ABC Ambe r CHM Conve rte r Tria l ve rsion, http://w w w .proce sste x t.com/a bcchm.html
t he l umen of t he col l ect i ng t ubul es . Vi rt ual l y al l of t he nonpol ar ammoni a t hat ent ers t he t ubul ar l umen i mmedi at el y combi nes wi t h hydrogen i on t o form ammoni um i on, whi ch i s nondi ffus i bl e becaus e i t i s l i pi d i ns ol ubl e. The renal excret i on of ammoni um i on res ul t s i n t he addi t i on of bi carbonat e t o t he pl as ma.
6. Concept of compensation. Compens at i on can be defi ned as t he phys i ol ogi c res pons e t o an al t erat i on i n ei t her t he respiratory or metabolic (renal ) component of aci d–bas e bal ance t o res t ore t he body pH val ue t oward normal . Physiologic compensation generally is not complete.
7. Concept of correction of metabolic acidosis. Compens at ory proces s es do not normal i ze t he l ow pH and s erum bi carbonat e concent rat i on i n met abol i c aci dos i s (e.g., i n di arrhea). However, t he normal ki dney can res t ore t hes e paramet ers t oward normal by i ncreas i ng net aci d excret i on i n t he uri ne (i .e., excret i on of exces s hydrogen i ons ). In l arge part , t he ki dney i ncreas es renal t ubul ar cel l product i on of ammoni a, t hereby i ncreas i ng excret i on of ammoni um i ons . The res ul t ant generat i on of new bi carbonat e i n t he di s t al t ubul e res t ores t he s erum bi carbonat e concent rat i on t oward t he normal range.
B. Respiratory acidosis
1. Definition. Increased blood PaCO 2 (i .e., >40 mm Hg)
Pa g e 1 4 2 8
ABC Ambe r CHM Conve rte r Tria l ve rsion, http://w w w .proce sste x t.com/a bcchm.html
and decreased blood pH (i .e., aci demi a) are charact eri s t i c.
2. Etiology. Res pi rat ory aci dos i s i s as s oci at ed wi t h a reduced capaci t y t o excret e carbon dioxide vi a t he l ungs . Caus es i ncl ude al l di s orders t hat reduce pul monary funct i on and carbon di oxi de cl earance. o
o
a. Primary pulmonary disease t hat i s as s oci at ed wi t h al veol ar–art eri al mi s mat ch may l ead t o ca rbon di oxi de ret ent i on, us ual l y as a l at e mani fes t at i on.
o
o
b. Neuromuscular disease. Any weaknes s of t he pul monary mus cul at ure t hat l eads t o reduced vent i l at i on (e.g., myas t heni a gravi s ) may produce carbon di oxi de ret ent i on.
o
o
c. Primary CNS dysfunction. Any s evere i njury t o t he brai ns t em may be as s oci at ed wi t h reduced vent i l at ory dri ve and carbon di oxi de ret ent i on.
o
o
d. Drug-induced hypoventilation. Any agent t hat caus es s evere depres s i on of CNS or neuromus cul ar funct i on may be as s oci at ed wi t h res pi rat ory aci dos i s .
3. Clinical features o
o
a. CNS disorders. Becaus e bl ood fl ow t o t he brai n
Pa g e 1 4 2 9
ABC Ambe r CHM Conve rte r Tria l ve rsion, http://w w w .proce sste x t.com/a bcchm.html
i s regul at ed by bl ood PaCO 2 , res pi rat ory aci dos i s i s as s oci at ed wi t h i ncreas ed bl ood fl ow t o t he brai n and i ncreas ed cerebros pi nal fl ui d (CSF) pres s ure. Thes e effect s may l ead t o a vari et y of s ympt oms of general i zed CNS depres s i on. o
o
b. Cardiac disorders. The aci demi a i n res pi rat ory aci dos i s i s as s oci at ed wi t h reduced cardi ac out put and pul monary hypert ens i on (effect s t hat may l ead t o cri t i cal l y reduced bl ood fl ow t o vi t al organs ).
4. Diagnosis o
o
a. Acute respiratory acidosis. Acut e carbon di oxi de ret ent i on l eads t o an i ncreas e i n bl ood PaCO 2 wi t h mi ni mal change i n pl as ma bi carbonat e cont ent . For each 10 mm Hg ri s e i n PaCO 2 , t he pl as ma bi carbonat e l evel i ncreas es by approxi mat el y 1 mEq/L and t he bl ood pH decreas es by approxi mat el y 0.08. Serum el ect rol yt e l evel s are cl os e t o normal i n i ndi vi dual s wi t h acut e res pi rat ory aci dos i s .
o
o
b. Chronic respiratory acidosis. Aft er 2–5 days , renal compens at i on (i .e., i ncreas ed hydrogen i on s ecret i on and bi carbonat e product i on i n t he di s t al nephron) occurs ; t hat i s , t he pl as ma bi carbonat e l evel s t eadi l y i ncreas es . Art eri al bl ood gas anal ys i s s hows t hat i n t he chroni c phas e of res pi rat ory aci dos i s , for each 10 mm Hg ri s e i n PaCO 2 , t he pl as ma bi carbonat e l evel i ncreas es by 3–4 mEq/L and t he bl ood pH decreas es by 0.03.
Pa g e 1 4 3 0
ABC Ambe r CHM Conve rte r Tria l ve rsion, http://w w w .proce sste x t.com/a bcchm.html
P.288
5. T herapy o
o
a. Correction of the underlying disorder. At t empt s s houl d be made t o correct mus cul ar dys funct i on or revers i bl e pul monary di s eas e, i f ei t her i s t he caus e of t he res pi rat ory aci dos i s . In t he cas e of drug-i nduced hypovent i l at i on, vi gorous at t empt s s houl d be made t o cl ear t he offendi ng agent from t he body.
o
o
b. Respiratory therapy. A bl ood PaCO 2 of more t han 60 mm Hg may be an i ndi cat i on for as s i s t ed vent i l at i on.
C. Respiratory alkalosis
1. Definition. Decreased blood PaCO 2 and an increased blood pH (al kal emi a) are charact eri s t i c.
2. Etiology. Res pi rat ory al kal os i s i s as s oci at ed wi t h exces s i ve el i mi nat i on of carbon di oxi de vi a t he l ungs . Caus es i ncl ude any di s order as s oci at ed wi t h i nappropri at el y i ncreas ed vent i l at ory rat e and carbon di oxi de cl earance. o
o
a. Anxiety (hys t eri cal hypervent i l at i on). Thi s condi t i on i s t he mos t common caus e of res pi rat ory
Pa g e 1 4 3 1
ABC Ambe r CHM Conve rte r Tria l ve rsion, http://w w w .proce sste x t.com/a bcchm.html
al kal os i s . o
o
b. Salicylate toxicity. Ini t i al l y, s al i cyl at e exces s caus es overs t i mul at i on of t he res pi rat ory cent er, res ul t i ng i n res pi rat ory al kal os i s . Met abol i c aci dos i s may devel op from t he s al i cyl at e l oad, whi ch enhances t he hypervent i l at i on.
o
o
c. Hypoxia. Any di s order as s oci at ed wi t h decreas ed oxygen t ens i on (Pao 2 ) of bl ood may l ead t o an i ncreas ed res pi rat ory rat e and, t hus , res pi rat ory al kal os i s .
o
o
d. Intrathoracic disorders. Any i nfl ammat ory or s pace-occupyi ng l es i on i n t he l ung may be as s oci at ed wi t h pri mary s t i mul at i on of vent i l at ory rat e, l eadi ng t o a l ow PaCO 2 . Such condi t i ons i ncl ude:
(1) Pul monary embol i s m
(2) Pneumoni a
(3) As t hma
(4) Pul monary fi bros i s
o
o
e. Primary CNS dysfunction. CNS di s orders t hat may be as s oci at ed wi t h i nappropri at e s t i mul at i on
Pa g e 1 4 3 2
ABC Ambe r CHM Conve rte r Tria l ve rsion, http://w w w .proce sste x t.com/a bcchm.html
of vent i l at i on i ncl ude:
(1) Cerebrovas cul ar acci dent (CVA)
(2) Tumor
(3) Infect i on
(4) Trauma
o
o
f. Gram-negative septicemia. An earl y mani fes t at i on of gram-negat i ve s ept i cemi a or bact eremi a i s a pri mary s t i mul at i on of vent i l at i on wi t h res pi rat ory al kal os i s . The mechani s m i s unknown.
o
o
g. Liver failure. The mos t common aci d–bas e di s order i n l i ver di s eas e i s pri mary res pi rat ory al kal os i s t hrough a di rect CNS effect of hyperammonemi a.
o
o
h. Pregnancy. Pri mary s t i mul at i on of vent i l at i on i s t ypi cal l y s een t hroughout pregnancy.
3. Clinical features. Acut e al kal emi a may be as s oci at ed wi t h s everal organ s ys t em di s orders . o
o
a. CNS disorders. A general i zed feel i ng of anxi et y may be pres ent and may progres s t o more s evere
Pa g e 1 4 3 3
ABC Ambe r CHM Conve rte r Tria l ve rsion, http://w w w .proce sste x t.com/a bcchm.html
obt undat i on and even precoma. o
o
b. Neuromuscular disorders. Acut e al kal emi a may produce a t et any-l i ke s yndrome, whi ch may be i ndi s t i ngui s habl e from t hat of acut e hypocal cemi a.
4. Diagnosis o
o
a. Acute respiratory alkalosis. Increas ed res pi rat ory rat e l eads t o a l os s of carbon di oxi de vi a t he l ungs , whi ch i n t urn i ncreas es t he bl ood pH. For each 10 mm Hg decreas e i n bl ood PaCO 2 acut el y, t he pl as ma bi carbonat e l evel decreas es by 2 mEq/L and t he bl ood pH i ncreas es by 0.08.
o
o
b. Chronic respiratory alkalosis. W i t hi n hours aft er an acut e decreas e i n art eri al PaCO 2 , hydrogen i on s ecret i on i n t he di s t al nephron decreas es , l eadi ng t o a decreas e i n pl as ma bi carbonat e. For each 10 mm Hg decreas e i n bl ood PaCO 2 chroni cal l y, t he pl as ma bi carbonat e l evel decreas es by 5–6 mEq/L and t he bl ood pH i ncreas es by onl y about 0.02. Serum chl ori de l evel al s o i s el evat ed.
5. T herapy. The pri mary goal of t herapy i s t o correct t he underl yi ng di s order. Us e of carbon di oxi de–enri ched breat hi ng mi xt ures or cont rol l ed vent i l at i on may be requi red i n cas es of s evere res pi rat ory al kal os i s (pH >7.6).
P.289
Pa g e 1 4 3 4
ABC Ambe r CHM Conve rte r Tria l ve rsion, http://w w w .proce sste x t.com/a bcchm.html
D. Metabolic acidosis
1. Definition. Met abol i c aci dos i s i s charact eri zed by a decreas ed bl ood pH and a decreas ed pl as ma bi carbonat e concent rat i on. Thi s condi t i on may be caus ed by one of t wo bas i c mechani s ms : t he l os s of bi carbonat e or t he accumul at i on of an aci d ot her t han carboni c aci d (e.g., l act i c aci d).
2. Etiology. The caus es of met abol i c aci dos i s may be di vi ded i nt o t hos e as s oci at ed wi t h a normal ani on gap and t hos e as s oci at ed wi t h an i ncreas ed ani on gap. T he anion gap reflects the concentrations of those anions that actually are present in serum but are not routinely assayed, i ncl udi ng negat i vel y charged pl as ma prot ei ns (mai nl y al bumi n), phos phat es , s ul fat e, and organi c aci ds (e.g., l act i c aci d). The ani on gap, meas ured i n mEq/L, repres ent s t he di fference bet ween t he concent rat i on of unmeas ured ani ons and cat i ons . It can be cal cul at ed as fol l ows : +
-
-
[eq] Anion gap = [Na ] – ([Cl ] + [HCO 3 ]) The normal val ue of t he ani on gap i s 8 ± 4 mEq/L. An i ncreas e repres ent s an i ncreas e i n one moi et y, us ual l y t he organi c aci ds . No change wi t h decreas es i n bot h pl as ma bi carbonat e concent rat i on and s erum pH s ugges t s a pri mary l os s of bi carbonat e or t he addi t i on of mi neral aci d. o
o
a. Metabolic acidosis with an increased anion gap
(1) Ketoacidosis. Thi s condi t i on refers t o a
Pa g e 1 4 3 5
ABC Ambe r CHM Conve rte r Tria l ve rsion, http://w w w .proce sste x t.com/a bcchm.html
s t at e of i ncreas ed ket oaci d format i on, whi ch l eads t o t i t rat i on of bi carbonat e and cons equent met abol i c aci dos i s . Ket oaci dos i s occurs as a compl i cat i on of di abet es mel l i t us , prol onged s t arvat i on, and prol onged al cohol abus e.
(2) Lactic acidosis. Decreas ed oxygen del i very t o t i s s ues res ul t s i n i ncreas ed l act at e product i on, wi t h accompanyi ng s evere met abol i c aci dos i s . Lact i c aci dos i s i s a charact eri s t i c feat ure of many condi t i ons as s oci at ed wi t h l ow t i s s ue perfus i on (e.g., s hock and s eps i s ).
(3) Renal failure. Met abol i c aci dos i s res ul t s from t he i nabi l i t y of t he ki dney t o excret e t he dai l y hydrogen i on l oad, deri ved from food and met abol i s m, as a res ul t of a decl i ne i n ammoni um excret i on res ul t i ng from decreas ed renal mas s . The aci dos i s of renal fai l ure i s charact eri zed by ei t her a normal or el evat ed ani on gap, dependi ng on t he s everi t y of t he decl i ne i n renal fi l t eri ng funct i on. In ei t her cas e, t he s erum bi carbonat e i s reduced due t o t he t i t rat i on by t he ret ai ned hydrogen i ons .
(a) The ani on gap i s el evat ed onl y when t he GFR i s s everel y reduced (<25 mm
3
/mi n) becaus e t he ani ons (s ul fat es and phos phat es ) t hat accompany t he ret ai ned hydrogen i ons cannot be fi l t ered.
Pa g e 1 4 3 6
ABC Ambe r CHM Conve rte r Tria l ve rsion, http://w w w .proce sste x t.com/a bcchm.html
(b) In mi l d-t o-moderat e renal fai l ure 3
(GFR = 25–60 mm /mi n), t hes e ani ons are freel y excret ed and do not accumul at e, res ul t i ng i n met abol i c aci dos i s wi t h normal ani on gap.
(4) Intoxication. The i nges t i on of a vari et y of chemi cal agent s may res ul t i n t he accumul at i on of organi c aci ds (e.g., l act i c aci d). Such i nt oxi cant s i ncl ude:
(a) Sal i cyl at e
(b) Met hanol
(c) Et hyl ene gl ycol
o
o
b. Metabolic acidosis with a normal anion gap (hyperchloremic metabolic acidosis) i s ei t her t he res ul t of renal or gas t roi nt es t i nal l os s es .
(1) Renal loss of bicarbonate may res ul t from t he fol l owi ng condi t i ons .
(a) Proximal tubular acidosis. Charact eri s t i cs i ncl ude decreas ed proxi mal t ubul ar reabs orpt i on of bi carbonat e l eadi ng t o exces s i ve uri nary excret i on of bi carbonat e. Caus es i ncl ude mul t i pl e myel oma, heavy met al
Pa g e 1 4 3 7
ABC Ambe r CHM Conve rte r Tria l ve rsion, http://w w w .proce sste x t.com/a bcchm.html
poi s oni ng, and W i l s on's di s eas e.
(b) Distal tubular acidosis. Charact eri s t i cs i ncl ude a decreas ed di s t al t ubul ar capaci t y for hydrogen i on s ecret i on and i nabi l i t y t o mai nt ai n an aci di c uri ne (uri ne pH oft en i s >6.0). Thi s defect l eads t o an i nabi l i t y t o generat e “new― bi carbonat e t hrough el i mi nat i on of prot ons buffered by ammoni a. Caus es i ncl ude heredi t ary di s orders , amphot eri ci n B t oxi ci t y, SLE, obs t ruct i ve uropat hy, Sjögren s yndrome, and ot her hypergl obul i nemi c condi t i ons .
(c) Hyperkalemic renal tubular acidosis. Hyperkal emi a, part i cul arl y t hat as s oci at ed wi t h hyporeni nemi c hypoal dos t eroni s m, i s charact eri zed by reduced ammoni a excret i on, reduced bi carbonat e product i on, and, t hus , t he i nabi l i t y t o buffer nonvol at i l e aci ds deri ved from t he di et . Aci dos i s (i .e., reduced pl as ma bi carbonat e) i s due t o reduced ammoni a product i on and, t herefore, reduced capaci t y t o excret e hydrogen i on and t o generat e “new†• bi carbonat e [s ee onl i ne Part II: IV A 5 a (3), b (2)]. P.290
Pa g e 1 4 3 8
ABC Ambe r CHM Conve rte r Tria l ve rsion, http://w w w .proce sste x t.com/a bcchm.html
(d) Moderate renal insufficiency. The decl i ne i n ammoni um excret i on res ul t i ng from decreas ed renal mas s caus es a decreas e i n net aci d excret i on and, t herefore, a decreas e i n s erum bi carbonat e concent rat i on. Recal l t hat when t he GFR i s great er t han 25 mL/mi n, ani ons s uch as s ul fat e and phos phat e are freel y excret ed i n t he uri ne and do not accumul at e, res ul t i ng i n a normal ani on gap.
(e) Carbonic anhydrase inhibition. Drugs s uch as acet azol ami de (a di uret i c) i nhi bi t t he act i on of carboni c anhydras e and t hereby reduce proxi mal t ubul ar reabs orpt i on of bi carbonat e.
(2) Gastrointestinal loss of alkali al s o produces t hi s s yndrome and may occur becaus e of:
(a) Di arrhea
(b) Pancreat i c fi s t ul as
(c) Uret eros i gmoi dos t omy
3. Clinical features. Si gns and s ympt oms us ual l y are rel at ed t o t he underl yi ng di s order. A bl ood pH of l es s
Pa g e 1 4 3 9
ABC Ambe r CHM Conve rte r Tria l ve rsion, http://w w w .proce sste x t.com/a bcchm.html
t han 7.2 may l ead t o reduced cardi ac out put . Aci dos i s al s o may be as s oci at ed wi t h res i s t ance t o t he vas ocons t ri ct i ve act i on of cat echol ami nes , res ul t i ng i n hypot ens i on. Kus s maul 's (deep and rhyt hmi c) res pi rat i on may be promi nent as t he vent i l at ory rat e i ncreas es i n res pons e t o t he fal l i n s erum pH.
4. Diagnosis o
o
a. Serum el ect rol yt e as s ay s hows a decreas ed bi carbonat e and a vari abl e chl ori de cont ent , dependi ng on whet her t he aci dos i s i s as s oci at ed wi t h a normal or an i ncreas ed ani on gap.
o
o
b. Art eri al bl ood gas anal ys i s al s o demons t rat es a decreas ed bi carbonat e l evel , wi t h a compens at ory decreas e i n bl ood PaCO 2 . Winters' formula predi ct s t hat i n pure met abol i c aci dos i s t he PaCO 2 s houl d be 1.5 t i mes t he bi carbonat e concent rat i on pl us 8 ± 2 mm Hg. Vari ance from t hi s predi ct ed res pons e t o pure met abol i c aci dos i s s ugges t s a compl i cat i ng res pi rat ory dys funct i on. (A l ower-t han-predi ct ed PaCO 2 s ugges t s pri mary res pi rat ory al kal os i s ; a hi gher-t han-predi ct ed PaCO 2 s ugges t s a di s order of pul monary funct i on, l eadi ng t o i nappropri at e carbon di oxi de ret ent i on.)
5. T herapy. Met abol i c aci dos i s may be t reat ed wi t h al kal i when t he bl ood pH i s l es s t han 7.2, wi t h i nt ravenous t herapy ai med at el evat i ng t he pH above t hi s poi nt . Sodi um bi carbonat e i s t he preferred al kal i . o
Pa g e 1 4 4 0
ABC Ambe r CHM Conve rte r Tria l ve rsion, http://w w w .proce sste x t.com/a bcchm.html
o
a. The requi red amount of bi carbonat e can be cal cul at ed on t he bas i s t hat bi carbonat e occupi es a s pace t hat account s for approxi mat el y 50% of body wei ght . Thus , t he amount of s odi um bi carbonat e needed t o rai s e t he pl as ma bi carbonat e from 13 mEq/L t o 20 mEq/L i s cal cul at ed as : 7 mEq/L × 0.5 × kg of body wei ght . Thi s number i s an approxi mat i on, and meas urement s of pl as ma bi carbonat e and bl ood pH mus t be repeat ed i n pat i ent s s o t reat ed. Thi s s pace i s much l arger i f t he met abol i c aci dos i s i s more s evere.
o
o
b. Underes t i mat i on of bi carbonat e requi rement s can occur i f bi carbonat e l os s es pers i s t (e.g., di arrhea) or i f ongoi ng aci d product i on i s s uffi ci ent l y rapi d t o cons ume admi ni s t ered bi carbonat e i n a bufferi ng react i on (e.g., l act i c aci dos i s ).
o
o
c. In t he pres ence of di s t al t ubul ar aci dos i s or renal i ns uffi ci ency, t he chroni c bi carbonat e requi rement i s 1 mEq/kg/day, whi ch i s equi val ent t o t he nonvol at i l e aci d product i on dai l y. However, i n proxi mal t ubul ar aci dos i s , t he bi carbonat e requi rement i s much great er (2–4 mEq/kg/day), becaus e of t he i neffect i ve reabs orpt i on of fi l t ered bi carbonat e.
E. Metabolic alkalosis
1. Definition. Increased blood pH and increased plasma bicarbonate concentration are charact eri s t i c.
Pa g e 1 4 4 1
ABC Ambe r CHM Conve rte r Tria l ve rsion, http://w w w .proce sste x t.com/a bcchm.html
2. Etiology. Increas ed pl as ma bi carbonat e l evel s res ul t from ei t her increased endogenous production of bicarbonate (i n t he s t omach or ki dney), wi t h reduced renal excret i on, or exogenous administration of bicarbonate or other alkali. Met abol i c al kal os i s depends on bot h t he fact ors t hat i ni t i at e al kal emi a (generat i on phas e) and t hos e t hat mai nt ai n i t (mai nt enance phas e). Becaus e t he renal capaci t y for bi carbonat e excret i on i s s everal t hous and mEq/day, i t i s cl ear t hat s ome i mpai rment i n renal bi carbonat e excret i on i s mandat ory for t he mai nt enance of met abol i c al kal os i s and a s us t ai ned ri s e i n pl as ma bi carbonat e. o
o
a. Excessive mineralocorticoid action on t he di s t al convol ut ed t ubul e and col l ect i ng t ubul e s t i mul at es hydrogen i on s ecret i on, t hereby rai s i ng t he pl as ma bi carbonat e l evel . Thi s occurs i n al l cas es of pri mary or s econdary hyperal dos t eroni s m. P.291
o
o
b. Vomiting generat es and mai nt ai ns met abol i c al kal os i s by s everal mechani s ms . Los s of gas t ri c hydrochl ori c aci d rai s es pl as ma bi carbonat e, and t he concomi t ant decreas e i n ext racel l ul ar fl ui d vol ume produced by vomi t i ng, reduces t he GFR, and i ncreas es t he rat e of proxi mal t ubul ar reabs orpt i on of s odi um and bi carbonat e t o mai nt ai n t he met abol i c al kal os i s . Secondary hyperal dos t eroni s m due t o ext racel l ul ar vol ume depl et i on al s o cont ri but es t o uri nary pot as s i um was t i ng.
Pa g e 1 4 4 2
ABC Ambe r CHM Conve rte r Tria l ve rsion, http://w w w .proce sste x t.com/a bcchm.html o
o
c. Diuretics. Inhi bi t i on of renal s odi um chl ori de reabs orpt i on l eads t o i ncreas ed fl ow rat e and, t herefore, i ncreas ed hydrogen i on s ecret i on i n t he di s t al convol ut ed t ubul e and col l ect i ng t ubul e. Increas ed hydrogen i on s ecret i on caus es i ncreas ed generat i on of bi carbonat e. The vol ume depl et i on produced by t he s odi um defi ci t s fol l owi ng di uret i c us e reduces t he GFR, s t i mul at es proxi mal t ubul ar reabs orpt i on of bi carbonat e, and mai nt ai ns met abol i c al kal os i s . Secondary hyperal dos t eroni s m due t o vol ume depl et i on caus es uri nary pot as s i um was t i ng and al s o cont ri but es t o mai nt enance of met abol i c al kal os i s by s t i mul at i ng hydrogen s ecret i on i n t he di s t al s egment s of t he nephron.
o
o
d. Administration of alkali, ei t her as s odi um bi carbonat e (e.g., duri ng cardi ac res us ci t at i on) or as organi c i ons (e.g., l act at e, ci t rat e, and acet at e, whi ch are met abol i cal l y convert ed t o bi carbonat e by hepat i c act i on), res ul t s i n an i ncreas ed pl as ma bi carbonat e l evel . However, unl es s renal reabs orpt i on of bi carbonat e i s s t i mul at ed, pl as ma bi carbonat e i s not s us t ai ned at an el evat ed l evel .
o
o
e. Rapid correction of hypercapnia. Fol l owi ng s us t ai ned res pi rat ory aci dos i s , renal bi carbonat e product i on i s el evat ed as a compens at ory event by a s t i mul at i on of hydrogen i on s ecret i on. If art eri al PCO 2 i s t hen acut el y reduced by mechani cal vent i l at i on, a t rans i ent s t at e of hyperbi carbonat emi a and el evat ed bl ood pH ens ues (a condi t i on t ermed posthypercapnic metabolic
Pa g e 1 4 4 3
ABC Ambe r CHM Conve rte r Tria l ve rsion, http://w w w .proce sste x t.com/a bcchm.html
alkalosis).
3. Clinical features. Si gns and s ympt oms general l y are domi nat ed by t he underl yi ng di s eas e s t at e. However, s ympt oms of t et any may be t he mos t pronounced cl i ni cal feat ures .
4. Diagnosis o
o
a. Serum electrolyte assay s hows an i ncreas ed bi carbonat e l evel and a decreas ed chl ori de l evel . Hypokal emi a i s a frequent fi ndi ng.
o
o
b. Arterial blood gas analysis reveal s an el evat ed bi carbonat e l evel and a compens at ory i ncreas e i n PaCO 2 val ue. Becaus e a decreas e i n vent i l at i on i s requi red t o el evat e PaCO 2 , hypoxi a al s o may res ul t .
o
o
c. Urinary chloride i s us eful i n t he di agnos i s of met abol i c al kal os i s . If ext racel l ul ar fl ui d vol ume cont ract i on i s not pres ent (e.g., due t o exces s i ve mi neral ocort i coi d act i vi t y) or i f renal reabs orpt i on of s odi um chl ori de i s i nhi bi t ed (e.g., due t o di uret i c us e), uri ne chl ori de l evel s are >15 mg/dL. If ext racel l ul ar fl ui d vol ume depl et i on i s pres ent (e.g., due t o vomi t i ng), uri ne chl ori de l evel t ypi cal l y wi l l be <15 mg/dL.
5. T herapy. Treat ment i nvol ves correct i on of t he underl yi ng di s eas e s t at e as wel l as reduct i on of renal avi di t y for bi carbonat e. The l at t er effect i s accompl i s hed
Pa g e 1 4 4 4
ABC Ambe r CHM Conve rte r Tria l ve rsion, http://w w w .proce sste x t.com/a bcchm.html
by ext racel l ul ar vol ume expans i on wi t h s odi um chl ori de–cont ai ni ng s ol ut i ons . o
o
a. The met abol i c al kal os i s caus ed by exces s i ve mi neral ocort i coi d act i vi t y i s hi ghl y dependent on pot as s i um depl et i on. Admi ni s t rat i on of pot as s i um chl ori de correct s t hi s di s order.
o
o
b. In i ndi vi dual s who have pos t hypercapni c met abol i c al kal os i s , judi ci ous admi ni s t rat i on of acet azol ami de or ot her i nhi bi t ors of proxi mal t ubul ar bi carbonat e reabs orpt i on i s an adjunct t o t herapy.
V. Calcium Metabolism
A. Normal physiology
1. Cal ci um exi s t s i n s erum i n t hree forms . About 40% of s erum cal ci um i s bound t o prot ei n, about 5%–15% i s compl exed wi t h ani ons s uch as ci t rat e and phos phat e, and t he remai ni ng port i on i s unbound, i oni zed cal ci um. The i oni zed component of s erum cal ci um i s t he mos t i mport ant cl i ni cal l y. For exampl e, hypoal bumi nemi a l owers s erum cal ci um by reduci ng t he prot ei n-bound component . The i oni zed cal ci um concent rat i on, however, i s unaffect ed by hypoal bumi nemi a, and t he pat i ent i s as ympt omat i c.
2. Ioni zed cal ci um homeos t as i s i s mai nt ai ned by a
Pa g e 1 4 4 5
ABC Ambe r CHM Conve rte r Tria l ve rsion, http://w w w .proce sste x t.com/a bcchm.html
bal ance of cal ci um i nput i nt o t he bl ood from t he gas t roi nt es t i nal t ract and bone and cal ci um out put from t he bl ood i nt o t he uri ne and l ower gas t roi nt es t i nal t ract . Cal ci um t rans port acros s t he gas t roi nt es t i nal t ract i s i nfl uenced s t rongl y by 1,25-dihydroxycholecalciferol [1,25-(OH) 2 D 3 , or calcitriol], whi ch i s t he act i ve met abol i t e of vi t ami n D. Parathyroid hormone (PT H) rai s es s erum cal ci um by i ncreas i ng cal ci um rel eas e from bone, by reduci ng renal excret i on of cal ci um, and by s t i mul at i ng renal act i vat i on of vi t ami n D t o cal ci t ri ol . Thi s i s t he s econd of t wo s t eps i n t he met abol i c act i vat i on of di et ary vi t ami n D. The fi rs t s t ep t akes pl ace i n t he l i ver, and t he product i s 25-hydroxycholecalciferol [25-(OH)D 3 , or calcifediol]. Serum cal ci um l evel i s t he pri nci pal regul at or of PTH rel eas e.
B. Hypocalcemia Hypocalcemia i s defi ned as a s erum cal ci um concent rat i on l es s t han 8.5 mg/dL. Hypoalbuminemia l owers t he t ot al s erum cal ci um by reduci ng t he prot ei n-bound component . General l y, t he t ot al s erum cal ci um l evel i s decreas ed 0.8 mg/dL for each 1 g/L decrement i n s erum al bumi n. However, PT H deficiency i s t he pri mary det ermi nant of hypocal cemi a. For a compl et e di s cus s i on of hypocal cemi a, s ee Chapt er 9 III B. P.292
C. Hypercalcemia Hypercalcemia res ul t s from di s orders t hat caus e ei t her i ncreas ed gas t roi nt es t i nal abs orpt i on or i ncreas ed bone res orpt i on of cal ci um. Normal s erum cal ci um l evel may reach 10.5 mg/dL i n men and 10.2 mg/dL i n women. Hi gher val ues may i ndi cat e t rue hypercal cemi a, but s erum prot ei n l evel al s o mus t be moni t ored t o confi rm t hat an i ncreas ed l evel of prot ei n does not expl ai n t he i ncreas ed t ot al s erum cal ci um. Hyperparathyroidism i s t he res ul t of overs ecret i on
Pa g e 1 4 4 6
ABC Ambe r CHM Conve rte r Tria l ve rsion, http://w w w .proce sste x t.com/a bcchm.html
of PTH, whi ch i n t urn caus es hypercal cemi a. A ful l di s cus s i on of hypercal cemi a can be found i n Chapt er 9 III A.
VI. Phosphate Metabolism
A. Normal physiology
1. Phosphate function. Phos phat e may be t he s i ngl e mos t i mport ant dynami c cons t i t uent requi red for cel l ul ar act i vi t y. Vi rt ual l y al l bodi l y funct i ons are powered by t he hi gh-energy phos phat e bonds of adenos i ne t ri phos phat e (ATP). In addi t i on, phos phat e i s t he major ani on and buffer of t he i nt racel l ul ar fl ui d. It s cont ri but ory rol e i n t he renal excret i on of hydrogen i on makes phos phat e an i mport ant cons t i t uent of aci d–bas e met abol i s m as wel l .
2. Phosphate distribution. About 85% of t he t ot al body s t ore of phos phat e i s i n bone. Phos phat e al s o i s found i n t he i nt racel l ul ar and ext racel l ul ar fl ui d compart ment s . Pl as ma phos phat e exi s t s pri mari l y i n t he form of i norgani c phos phat e, t he majori t y of whi ch i s free (not bound t o prot ei n). The i norgani c phos phat e cont ent of t he ext racel l ul ar fl ui d i s a pri me det ermi nant of i nt racel l ul ar i norgani c phos phat e, whi ch i s t he s ource of phos phat e for ATP. Int racel l ul ar phos phat e defi ci t s may res ul t i n reduced cel l energy product i on and, t herefore, general i zed cel l dys funct i on.
3. Phosphate homeostasis i nvol ves t he bal ance of phos phat e i nt ake and phos phat e out put (“ext ernal â€
Pa g e 1 4 4 7
ABC Ambe r CHM Conve rte r Tria l ve rsion, http://w w w .proce sste x t.com/a bcchm.html
• bal ance) as wel l as t he mai nt enance of normal phos phat e di s t ri but i on wi t hi n t he body (“i nt ernal ― bal ance). o
o
a. External phosphate balance. Normal di et ary i nt ake of phos phat e i s 1200 mg/day, whi ch i s provi ded pri mari l y by dai ry product s , and normal phos phat e excret i on i s 1200 mg/day (800 mg i n t he uri ne and 400 mg i n t he s t ool ). The gas t roi nt es t i nal t ract i s a pas s i ve component of ext ernal phos phat e bal ance, whereas renal phos phat e handl i ng i s cl os el y regul at ed.
(1) Normal l y, 90% of fi l t ered phos phat e i s reabs orbed i n t he proxi mal t ubul e, wi t h onl y a mi nut e port i on reabs orbed di s t al l y. The mai n regul at or of renal phos phat e handl i ng i s PTH. A hi gh PTH l evel i nhi bi t s phos phat e reabs orpt i on, and a l ow PTH l evel s t i mul at es it.
(2) PTH-i ndependent cont rol of renal phos phat e reabs orpt i on al s o i s exert ed by di et ary phos phat e cont ent and ot her hormones s uch as cal ci t oni n, t hyroi d hormone, and growt h hormone (GH). A decreas e i n phos phat e i nt ake s t i mul at es proxi mal t ubul ar reabs orpt i on of phos phat e.
o
o
b. Internal phosphate balance. Thi s , t oo, i s regul at ed, becaus e i nt racel l ul ar phos phat e l evel s are 200–300 mg/dL and ext racel l ul ar l evel s are 3–4 mg/dL. Increas ed i ns ul i n or β-adrenergi c
Pa g e 1 4 4 8
ABC Ambe r CHM Conve rte r Tria l ve rsion, http://w w w .proce sste x t.com/a bcchm.html
agoni s t l evel s , hydrogen i on s hi ft s , and i nt racel l ul ar met abol i c di s t urbances al l al t er t he phos phat e di s t ri but i on i n t he body.
B. Hypophosphatemia
1. Etiology. Hypophos phat emi a can res ul t from ext rarenal or renal l os s of phos phat e. One of t he mos t common caus es of hypophos phat emi a i s chronic severe alcoholism, whi ch l eads t o mos t of t he fol l owi ng feat ures . o
o
a. Extrarenal causes
(1) Dietary deficiency and gastrointestinal losses
(a) Inadequate dietary intake. Mos t food cont ai ns s ome phos phat e. Inadequat e di et ary i nt ake of phos phat e i s unus ual and occurs onl y under s peci fi c i at rogeni c ci rcums t ances .
(b) Antacid abuse. Large amount s of cal ci um s al t s (e.g., acet at e, carbonat e) and al umi num- or magnes i um-cont ai ni ng ant aci ds bi nd phos phat e i n t he gas t roi nt es t i nal t ract , i ncreas e gas t roi nt es t i nal phos phat e l os s es , and may produce hypophos phat emi a.
Pa g e 1 4 4 9
ABC Ambe r CHM Conve rte r Tria l ve rsion, http://w w w .proce sste x t.com/a bcchm.html
(c) Starvation. Duri ng prol onged s t arvat i on, cel l breakdown l i berat es phos phat e i nt o t he ext racel l ul ar fl ui d. The amount of phos phat e i n remai ni ng, i nt act cel l s , however, i s pres erved at normal l evel s . As uri nary pl us s t ool l os s of l i berat ed, ext racel l ul ar phos phat e exceeds di et ary i nt ake, negat i ve phos phat e bal ance occurs . Al t hough hypophos phat emi a does not fol l ow i mmedi at el y, s evere phos phat e defi ci t s may devel op on refeedi ng as cel l ul ar upt ake of phos phat e i s s t i mul at ed by new cel l growt h and macromol ecul e s ynt hes i s .
(2) Redistribution of body phosphate
(a) Increased glycolysis. Any condi t i on as s oci at ed wi t h i ncreas ed gl ycol ys i s wi t hi n cel l s caus es organi c phos phat e compounds t o accumul at e as t he phos phoryl at ed carbon res i dues i n t he Embden-Meyerhof pat hway, wi t h depl et i on of i nt racel l ul ar i norgani c phos phat e. Serum phos phat e l evel fal l s as phos phat e di ffus es i nt o cel l s , caus i ng hypophos phat emi a. Reduct i on of i nt racel l ul ar i norgani c phos phat e t hrough t hi s mechani s m may be dras t i c and l ead t o ATP depl et i on and cel l dys funct i on, s uch as rhabdomyol ys i s .
Pa g e 1 4 5 0
ABC Ambe r CHM Conve rte r Tria l ve rsion, http://w w w .proce sste x t.com/a bcchm.html
(b) Respiratory alkalosis. Hypervent i l at i on i s as s oci at ed wi t h reduced s erum phos phat e becaus e of i ncreas ed cel l ul ar upt ake of phos phat e.
(c) Sepsis. Hypophos phat emi a i s a known concomi t ant of gram-negat i ve s eps i s and may coexi s t (al t hough i ndependent l y) wi t h hypophos phat emi a becaus e of res pi rat ory al kal os i s .
o
o
b. Renal causes
(1) Excess PT H. Any form of hyperparat hyroi di s m res ul t s i n renal phos phat e was t i ng and hypophos phat emi a, provi ded t hat GFR i s not markedl y reduced.
(2) Primary renal tubular defects. Condi t i ons s uch as cys t i nos i s , heavy met al poi s oni ng, mul t i pl e myel oma, and W i l s on's di s eas e may be as s oci at ed wi t h general i zed proxi mal t ubul ar defect s (Fanconi 's s yndrome) and renal phos phat e was t i ng.
(3) Specific transport defects for phosphate. Thes e defect s have been des i gnat ed as hypophosphatemic vitamin
Pa g e 1 4 5 1
ABC Ambe r CHM Conve rte r Tria l ve rsion, http://w w w .proce sste x t.com/a bcchm.html
D–resistant rickets, whi ch may be fami l i al or s poradi c and exi s t i n bot h chi l d-ons et and adul t -ons et forms . In each of t hes e condi t i ons , decreas ed phos phat e t rans port i n t he proxi mal t ubul e produces exces s i ve renal phos phat e was t i ng.
(4) Glycosuria. Phos phat e and gl ucos e compet e for t rans port i n t he proxi mal t ubul e. Al l gl ycos uri c condi t i ons are as s oci at ed wi t h exces s i ve renal l os s es of phos phat e.
P.293
2. Clinical features o
o
a. Neurologic disorders. Cel l ul ar ATP defi ci ency may produce an encephal opat hy charact eri zed by obt undat i on, coma, and s ei zures . Peri pheral neuropat hy and Gui l l ai n-Barré s yndrome al s o have been des cri bed.
o
o
b. Hematologic disorders. Hemol yt i c anemi a, a rare compl i cat i on of profound hypophos phat emi a, i s due t o cel l ul ar ATP depl et i on and abnormal membrane i nt egri t y.
o
o
c. Muscular disorders. Dys funct i on of s kel et al mus cl e has been des cri bed and at t ri but ed t o ATP defi ci t s . Acut e rhabdomyol ys i s may be part i cul arl y preval ent i n al cohol i c pat i ent s who are acut el y
Pa g e 1 4 5 2
ABC Ambe r CHM Conve rte r Tria l ve rsion, http://w w w .proce sste x t.com/a bcchm.html
hypophos phat emi c. Paral ys i s of res pi rat ory mus cl es wi t h res pi rat ory fai l ure al s o may be s een. o
o
d. Bone disorders. Increas ed bone res orpt i on wi t h abnormal mi neral i zat i on occurs i n chroni c hypophos phat emi a.
3. Diagnosis. Hypophos phat emi a, unl es s due t o renal phos phat e was t i ng, caus es near-compl et e el i mi nat i on of phos phat e from t he uri ne. A uri ne phos phat e l evel of more t han 100 mg/L s t rongl y s ugges t s renal phosphate wasting. A l ow uri ne phos phat e l evel s ugges t s ant aci d-i nduced phos phat e depl et i on, gas t roi nt es t i nal l os s es , or i ncreas ed cel l ul ar upt ake of phos phat e. Gl ucos e i nfus i on wi t h s econdary i ns ul i n rel eas e i s t he caus e of hypophos phat emi a i n mos t hos pi t al i zed pat i ent s .
4. T herapy. Al l hypophos phat emi c pat i ent s s houl d be t reat ed. In general , t herapy i nvol ves correct i on of t he underl yi ng condi t i on, s uch as di s cont i nuat i on of gl ucos e i nfus i ons . In i ndi vi dual s wi t h s evere hypophos phat emi a and preexi s t i ng phos phat e depl et i on (e.g., al cohol i c pat i ent s ), s ympt omat i c hypophos phat emi a s houl d be t reat ed wi t h phos phat e s uppl ement at i on. Oral phos phat e i s preferred, and 1500–2000 mg/day may be gi ven i n di vi ded dos es . If a pat i ent i s comat os e or i s unabl e t o t ake oral phos phat e, i nt ravenous phos phat e may be admi ni s t ered t wi ce dai l y i n 250-mg dos es , i f s erum phos phat e i s l es s t han 1 mg/dL, provi ded t hat s erum phos phat e meas urement s are t aken at 12-hour i nt erval s . Infus i on of phos phat e mus t be di s cont i nued i f t he s erum phos phat e l evel ri s es t o 1.5 mg/dL.
Pa g e 1 4 5 3
ABC Ambe r CHM Conve rte r Tria l ve rsion, http://w w w .proce sste x t.com/a bcchm.html
5. Complications. The great es t danger of hypophos phat emi a l i es i n t he i njudi ci ous admi ni s t rat i on of i nt ravenous phos phat e. Acut e hypocal cemi a due t o t he format i on of cal ci um phos phat e may l ead t o s hock, acut e renal fai l ure, and deat h. For t hi s reas on, i nt ravenous phos phat e s houl d be admi ni s t ered onl y when s peci fi c cl i ni cal di s t urbances are cl earl y at t ri but abl e t o hypophos phat emi a.
C. Hyperphosphatemia
1. Etiology o
o
a. Renal failure. Becaus e t he ki dney i s t he mai n regul at or of s erum phos phat e l evel , renal fai l ure commonl y i s as s oci at ed wi t h hyperphos phat emi a. Thi s di s order i s not s een unt i l t he GFR has decreas ed t o at l eas t 25% of normal . Serum phos phat e l evel general l y does not exceed 10 mg/dL i n renal fai l ure. Val ues exceedi ng 10 mg/dL s ugges t an addi t i onal et i ol ogi c fact or.
o
o
b. Cell lysis syndromes
(1) Rhabdomyolysis. Acut e mus cl e breakdown of any et i ol ogy i s as s oci at ed wi t h t he rel eas e of cel l ul ar phos phat e and, t herefore, hyperphos phat emi a. Severe hyperphos phat emi a (i .e., s erum phos phat e concent rat i on >15 mg/dL) may be s een i n cas es as s oci at ed wi t h acut e renal fai l ure.
Pa g e 1 4 5 4
ABC Ambe r CHM Conve rte r Tria l ve rsion, http://w w w .proce sste x t.com/a bcchm.html
(2) T umor lysis syndrome. Mal i gnant di s orders as s oci at ed wi t h a hi gh s ens i t i vi t y t o chemot herapy or radi ot herapy res ul t i n rapi d cel l deat h from s uch t reat ment s . Thi s s yndrome may l ead t o mas s i ve rel eas e of phos phat e and ot her i nt racel l ul ar s ubs t ances i nt o t he ext racel l ul ar fl ui d. Severe hypocal cemi a, cardi ovas cul ar col l aps e, and renal fai l ure due t o cal ci um, urat e, and phos phat e depos i t i on i n t he ki dney have been des cri bed.
o
o
c. Exogenous phosphate administration. Any rout e (i .e., vi a i nt ravenous i nfus i on, by mout h, or vi a phos phat e enemas ) may res ul t i n s evere and unpredi ct abl e hyperphos phat emi a, es peci al l y i f t he GFR i s reduced.
o
o
d. Hypoparathyroidism. Becaus e t he l evel of PTH i s a key det ermi nant of t he rat e of renal phos phat e handl i ng, any condi t i on as s oci at ed wi t h parat hyroi d i ns uffi ci ency or a l ack of renal res pons e t o PTH may be charact eri zed by hyperphos phat emi a.
o
o
e. Miscellaneous causes. GH exces s , hypert hyroi di s m, s i ckl e cel l anemi a, and t umoral cal ci nos i s may be as s oci at ed wi t h hyperphos phat emi a.
P.294
Pa g e 1 4 5 5
ABC Ambe r CHM Conve rte r Tria l ve rsion, http://w w w .proce sste x t.com/a bcchm.html
2. Clinical features. Hypocal cemi a, hypot ens i on, and renal fai l ure may be s een i n s evere hyperphos phat emi a. Mi l der cas es , t ypi cal l y s een i n chroni c renal fai l ure, are as s oci at ed wi t h s econdary hyperparat hyroi di s m and renal os t eodys t rophy.
3. Diagnosis. Hyperphos phat emi a i n t he abs ence of renal i ns uffi ci ency i s due t o hypoparat hyroi di s m, cel l l ys i s , or t umoral cal ci nos i s . The et i ol ogi c di agnos i s of hyperphos phat emi a i s made on t he bas i s of t he pat i ent hi s t ory, phys i cal exami nat i on, and l aborat ory dat a.
4. T herapy. Acut e hyperphos phat emi a may be a medi cal emergency t hat requi res i mmedi at e t herapy. In cas es of t umor l ys i s s yndrome, acut e hemodi al ys i s may be neces s ary and i s an effect i ve t reat ment . Admi ni s t rat i on of l arge amount s of phos phat e-bi ndi ng res i ns may be us eful i n t he l ong-t erm t reat ment of hyperphos phat emi c condi t i ons .
VII. Magnesium Metabolism
A. Normal physiology Magnes i um i s t he s econd mos t abundant i nt racel l ul ar cat i on. (Pot as s i um i s t he mos t abundant .)
1. Magnesium distribution. More t han 50% of t he t ot al body s t ore of magnes i um i s i n bone, wi t h mos t of t he remai ni ng port i on found i n s oft t i s s ues , mai nl y mus cl e. Les s t han 1% of body magnes i um i s i n t he ext racel l ul ar
Pa g e 1 4 5 6
ABC Ambe r CHM Conve rte r Tria l ve rsion, http://w w w .proce sste x t.com/a bcchm.html
fl ui d, 20%–30% of whi ch i s bound t o prot ei n and t he res t exi s t i ng as free cat i on.
2. Magnesium homeostasis. Magnes i um i s abs orbed i nt o t he s mal l i nt es t i ne, but t hi s proces s i s pri mari l y unregul at ed. Duri ng di et ary depri vat i on of magnes i um, s t ool magnes i um l os s es res ul t i n hypomagnes emi a. The ki dney effi ci ent l y cons erves magnes i um duri ng di et ary depri vat i on and excret es any exces s magnes i um due t o exces s i ve i nt ake.
B. Hypomagnesemia
1. Definition. Cl i ni cal l y i mport ant hypomagnes emi a occurs when s erum magnes i um concent rat i on fal l s bel ow 1.0 mg/dL, al t hough i t has been propos ed t hat even mi l d degrees of hypomagnes emi a may be as s oci at ed wi t h a vari et y of cl i ni cal di s orders .
2. Etiology o
o
a. Extrarenal causes
(1) Dietary deficiency and gastrointestinal losses
(a) Inadequate dietary intake. Nut ri t i onal hypomagnes emi a may devel op aft er prol onged s t arvat i on as wel l as pos t operat i vel y.
Pa g e 1 4 5 7
ABC Ambe r CHM Conve rte r Tria l ve rsion, http://w w w .proce sste x t.com/a bcchm.html
(b) Malabsorption. General i zed mal abs orpt i on s yndrome, chroni c di arrhea, di ffus e bowel i njury, and chroni c l axat i ve abus e al l are as s oci at ed wi t h reduced gas t roi nt es t i nal abs orpt i on of magnes i um.
(2) Redistribution of body magnesium. Acut e cel l ul ar upt ake of magnes i um has been des cri bed i n i ndi vi dual s who are i n al cohol wi t hdrawal .
o
o
b. Renal causes
(1) Primary tubular disorders. A number of t ubul ar di s orders , i ncl udi ng Bart t er's s yndrome, renal t ubul ar aci dos i s , and pos t obs t ruct i ve di ures i s , are charact eri zed by a defect i n renal magnes i um cons ervat i on and hypomagnes emi a. Hypomagnes emi a al s o may devel op i n pat i ent s fol l owi ng renal t rans pl ant at i on. In a fami l i al form of renal magnes i um was t i ng, t he mol ecul ar mechani s ms i nvol ved have been i dent i fi ed.
(2) Drug-induced tubular losses. Di uret i cs s uch as t hi azi des and furos emi de t ypi cal l y produce vari ed degrees of hypomagnes emi a. Even s mal l dos es of t he chemot herapeut i c agent ci s pl at i n produce marked renal magnes i um was t i ng and cl i ni cal l y s evere hypomagnes emi a. Gent ami ci n and
Pa g e 1 4 5 8
ABC Ambe r CHM Conve rte r Tria l ve rsion, http://w w w .proce sste x t.com/a bcchm.html
amphot eri ci n B al s o may produce a t oxi c i njury t o t he renal t ubul e, wi t h magnes i um and pot as s i um was t i ng occurri ng i n t he abs ence of reduced GFR.
(3) Hormone-induced tubular losses. Hyperal dos t eroni s m or hypoparat hyroi di s m can be as s oci at ed wi t h renal t ubul ar magnes i um was t i ng and hypomagnes emi a.
(4) Ion- or nutrient-induced tubular losses. Becaus e cal ci um and magnes i um compet e for t rans port i n t he as cendi ng l i mb of t he l oop of Henl e, hypercal cemi a i s as s oci at ed wi t h reduced renal magnes i um t rans port . Phos phat e depl et i on, al cohol cons umpt i on, or bot h are as s oci at ed wi t h decreas ed renal reabs orpt i on of magnes i um, but t he mechani s ms are unknown.
3. Clinical features. Mus cl e t wi t chi ng, t remor, and mus cl e weaknes s are commonl y s een. Thes e phys i cal s i gns are due t o t he di rect effect of magnes i um on neuromus cul ar funct i on as wel l as t he hypocal cemi c effect of hypomagnes emi a. Severe chroni c hypomagnes emi a l eads t o decreas ed gl andul ar s ecret i on of PTH as wel l as i mpai red bone res pons e t o PTH, and bot h of t hes e effect s l ead t o hypocal cemi a. Al s o, hypomagnes emi a produces a defect i n renal pot as s i um reabs orpt i on, whi ch event ual l y produces pot as s i um depl et i on. Thus , al l of t he cl i ni cal s i gns of hypocal cemi a and hypokal emi a may be s een i n hypomagnes emi c pat i ent s . The cl i ni cal pres ent at i on of hypokal emi a and
Pa g e 1 4 5 9
ABC Ambe r CHM Conve rte r Tria l ve rsion, http://w w w .proce sste x t.com/a bcchm.html
hypomagnes emi a i ncl udes cardi ac arrhyt hmi as , part i cul arl y i n pat i ent s t aki ng di gi t al i s . P.295
4. T herapy. In mos t pat i ent s , a normal di et makes up for magnes i um defi ci t s . If ongoi ng l os s es occur, magnes i um s uppl ement at i on i s neces s ary. Even i n s evere magnes i um defi ci ency, however, 50% of an admi ni s t ered dos e of magnes i um i s excret ed i n t he uri ne. Sympt omat i c defi ci t s us ual l y amount t o 1–2 mmol /kg of body wei ght . o
o
a. If oral magnesium therapy i s requi red, magnes i um oxi de general l y i s t ol erat ed and has 25%–50% abs orpt i on. It i s gi ven four t i mes dai l y i n dos es of 250–500 mg.
o
o
b. In cas es of s evere hypomagnes emi a, parenteral magnesium therapy may be neces s ary.
C. Hypermagnesemia
1. Etiology. Becaus e t he ki dneys can excret e s everal t hous and mi l l i grams of magnes i um each day, hypermagnes emi a us ual l y i s i at rogeni c and occurs , i n a s us t ai ned fas hi on, onl y i n pat i ent s who have i mpai red renal funct i on and i nges t magnes i um as ei t her l axat i ves or ant aci ds . Acut e magnes i um i nt oxi cat i on may occur i n women who are t reat ed for t oxemi a of pregnancy wi t h i nt ravenous magnes i um s al t s t hat are admi ni s t ered at
Pa g e 1 4 6 0
ABC Ambe r CHM Conve rte r Tria l ve rsion, http://w w w .proce sste x t.com/a bcchm.html
an exces s i ve rat e. Mus cul ar paral ys i s can devel op at s erum magnes i um l evel s of 10 mg/dL.
2. T herapy. Cal ci um i on i s a di rect ant agoni s t of magnes i um and s houl d be gi ven t o pat i ent s who are s eri ous l y i l l wi t h magnes i um i nt oxi cat i on. Hemodi al ys i s may be requi red fol l owi ng ces s at i on of magnes i um t herapy.
Part III: Hypertension I. General Considerations A. Definition Hypert ens i on i s pres ent when t he s ys t ol i c bl ood pres s ure cons t ant l y exceeds 140 mm Hg or t he di as t ol i c bl ood pres s ure exceeds 90 mm Hg. Many s t udi es s ugges t t hat s ys t ol i c bl ood pres s ure el evat i on i s as , i f not more, s i gni fi cant t han di as t ol i c bl ood pres s ure el evat i on as a ri s k fact or for a vari et y of cardi ovas cul ar and renal di s eas es .
1. Primary hypertension i s hypert ens i on t hat has no known caus e. Pri mary hypert ens i on account s for 90%–95% of cas es of hypert ens i on.
2. Secondary hypertension i s at t ri but abl e t o a di agnos abl e di s eas e and account s for t he remai nder of cas es of hypert ens i on.
B. Diagnosis
1. Bl ood pres s ure s houl d be meas ured wi t h a loosely fitting cuff. The wi dt h of t he cuff s houl d equal at l eas t 50% of t he l engt h of t he upper arm. Ini t i al l y, bl ood pres s ure s houl d be determined in each arm t o be s ure
Pa g e 1 4 6 1
ABC Ambe r CHM Conve rte r Tria l ve rsion, http://w w w .proce sste x t.com/a bcchm.html
t hat art eri al obs t ruct i on i n t he upper ext remi t y i s not fal s el y l oweri ng t he di s t al arm art eri al pres s ure.
2. Bl ood pres s ure may be el evat ed at t i mes of s t res s ; i n fact , i n s ome pat i ent s , merel y bei ng i n a phys i ci an's offi ce may i nduce t rans i ent hypert ens i on. Multiple determinations over several visits and s ome form of home or workplace monitoring s houl d be conduct ed pri or t o i ni t i at i ng pharmacol ogi c t herapy i n hypert ens i ve pat i ent s .
C. Consequences of hypertension In general , t he mort al i t y rat e over 20 years among pat i ent s wi t h a s ys t ol i c bl ood pres s ure of great er t han 160 mm Hg or a dias t ol i c bl ood pres s ure of great er t han 100 mm Hg i ncreas es 100% i n t hos e who are unt reat ed.
1. Stroke. Pat i ent s wi t h a s ys t ol i c bl ood pres s ure of great er t han 160 mm Hg have a fourfol d i ncreas ed ri s k of s t roke i f unt reat ed.
2. Coronary artery disease. Pat i ent s wi t h a di as t ol i c pres s ure of great er t han 95 mm Hg have a more t han t wofol d i ncreas ed ri s k of coronary art ery di s eas e as compared wi t h normot ens i ve pat i ent s .
3. Congestive heart failure (CHF). Pat i ent s wi t h bl ood pres s ure of great er t han 160/95 mm Hg have a fourfol d i ncreas ed i nci dence of CHF. In 75% of pat i ent s wi t h CHF, hypert ens i on occurs P.296
Pa g e 1 4 6 2
ABC Ambe r CHM Conve rte r Tria l ve rsion, http://w w w .proce sste x t.com/a bcchm.html
at s ome t i me duri ng t he cours e of t hei r i l l nes s . Thi ckeni ng and hypert rophy of t he l eft vent ri cl e as a res ul t of hypert ens i on may produce CHF as a res ul t of di as t ol i c dys funct i on.
4. Chronic kidney disease. Hypert ens i on s i gni fi cant l y i ncreas es t he ri s k of chroni c ki dney di s eas es and end-s t age renal di s eas e. There i s unequi vocal evi dence t hat bl ood pres s ure reduct i on s l ows t he progres s i on of renal di s eas e.
II. Mechanisms of Hypertension The major mechani s ms of hypert ens i on are l i s t ed i n
Onl i ne
Tabl e 6-17.
online Table 6-17 Mechanisms of Hypertension Pri ma ry Hy pe rte nsi on Ab nor ma l car di a c an
Pa g e 1 4 6 3
ABC Ambe r CHM Conve rte r Tria l ve rsion, http://w w w .proce sste x t.com/a bcchm.html
d per i ph era l he mo dyn am i cs Im pai red pre ssu re nat ri ur es i s Bar om et e r res et t i ng Ab nor ma liti es in ren i nâ
Pa g e 1 4 6 4
ABC Ambe r CHM Conve rte r Tria l ve rsion, http://w w w .proce sste x t.com/a bcchm.html
€“a ngi ot e ns i nâ €“a l do ste ron e s ys te m Ab nor ma liti es in ot h er vas ore gul at o ry s ys te ms En dot hel in At r i al
Pa g e 1 4 6 5
ABC Ambe r CHM Conve rte r Tria l ve rsion, http://w w w .proce sste x t.com/a bcchm.html
nat ri ur et i c pe pt i de En dot hel iu mder i ve d rel axa tio n fac t or Se co nd ary Hy pe rte nsi on Re nov as c ul a r
Pa g e 1 4 6 6
ABC Ambe r CHM Conve rte r Tria l ve rsion, http://w w w .proce sste x t.com/a bcchm.html
hyp ert ens i on Re nal par enc hy ma l di s eas es En doc ri n ol o gi c cau s es Ora l con t ra cep tiv es Mi n era l oc ort i coi d exc
Pa g e 1 4 6 7
ABC Ambe r CHM Conve rte r Tria l ve rsion, http://w w w .proce sste x t.com/a bcchm.html
es s s yn dro me s Pri ma ry hyp era l do ste ron ism Gl u coc ort i coi d-r em edi abl e hyp era l do ste ron ism Exo ge no us hyp er
Pa g e 1 4 6 8
ABC Ambe r CHM Conve rte r Tria l ve rsion, http://w w w .proce sste x t.com/a bcchm.html
mi ner al o cor tic oi d ism Cu s hi ng s yn dro me Li d dl e s yn dro me Ph eoc hro mo cyt om a Mi s cel l an eo us cau s es
A. Primary hypertension
Pa g e 1 4 6 9
ABC Ambe r CHM Conve rte r Tria l ve rsion, http://w w w .proce sste x t.com/a bcchm.html
1. Abnormal cardiac and peripheral hemodynamics. Bl ood pres s ure i s t he product of cardi ac out put and t ot al peri pheral res i s t ance; t herefore, for hypert ens i on t o occur, t here mus t be an el evat i on i n cardi ac out put , t ot al peri pheral res i s t ance, or bot h. o
o
a. An abnormal i t y i n peri pheral res i s t ance i s a cont ri but i ng fact or i n mos t cas es of hypert ens i on.
o
o
b. Many pat i ent s wi t h hypert ens i on have ei t her pers i s t ent l y el evat ed cardi ac out put or el evat ed t ot al peri pheral res i s t ance earl y i n t he cours e of t he di s eas e.
2. Impaired pressure natriuresis o
o
a. In normal i ndi vi dual s , an el evat i on i n bl ood pres s ure l eads t o an al t erat i on i n i nt rarenal hemodynami cs and phys i cal forces t hat res ul t s i n nat ri ures i s . Thi s , i n t urn, caus es a di ures i s , a decreas e i n t ot al ext racel l ul ar vol ume, and a fal l i n bl ood pres s ure.
o
o
b. In pat i ent s wi t h es s ent i al hypert ens i on, t he ki dney fai l s t o res pond normal l y t o el evat ed art eri al pres s ure and nat ri ures i s i s i mpai red. The homeos t at i c abnormal i t y may ei t her caus e or hel p s us t ai n t he el evat ed art eri al pres s ure.
o
o
c. The caus e of t he fai l ure of normal pres s ure
Pa g e 1 4 7 0
ABC Ambe r CHM Conve rte r Tria l ve rsion, http://w w w .proce sste x t.com/a bcchm.html
nat ri ures i s i s unknown. Hormonal fact ors , abnormal i t i es i n t he s t ruct ure and act i vi t y of t rans port prot ei ns (e.g., t he s odi um–hydrogen exchanger i n t he renal t ubul e), and aut onomi c nervous s ys t em act i vi t y have al l been i mpl i cat ed experi ment al l y and on a cl i ni cal bas i s .
3. Baroreceptor resetting. In hypert ens i ve pat i ent s , barorecept ors i n t he carot i d art eri es and aort a are “res et ― s o t hat hi gher pres s ures are requi red t o exert an i nfl uence t oward l oweri ng bl ood pres s ure.
4. Abnormalities in the renin–angiotensin–aldosterone system. Abnormal i t i es i n each component of t hi s compl ex s ys t em may cont ri but e t o t he pat hogenes i s of hypert ens i on i n many pat i ent s wi t h hypert ens i on.
5. Abnormalities in other vasoregulatory systems o
o
a. Endothelin, a pept i de produced i n many organs and t i s s ues , i s a vas ocons t ri ct or wi t h pot ency s everal t i mes great er t han t hat of norepi nephri ne.
o
o
b. Atrial natriuretic peptide (ANP), deri ved from cardi ac mus cl e, may pl ay a rol e i n normal vas cul ar regul at i on as a vas odi l at or and may cont ri but e t o ext racel l ul ar vol ume homeos t as i s and bl ood pres s ure cont rol i n s t at es of mi neral ocort i coi d exces s by enhanci ng s odi um excret i on.
o
Pa g e 1 4 7 1
ABC Ambe r CHM Conve rte r Tria l ve rsion, http://w w w .proce sste x t.com/a bcchm.html o
c. Endothelium-derived relaxation factor (EDRF) (nitric oxide) i s a gas deri ved from argi ni ne met abol i s m t hat i s cruci al i n a number of vas oregul at ory phenomena. Defect s i n t hi s s ys t em are t hought t o cont ri but e t o t he hypert ens i on as s oci at ed wi t h pregnancy.
B. Secondary hypertension In 5%–10% of pat i ent s wi t h hypert ens i on, t hi s condi t i on i s s econdary t o an i dent i fi abl e di s order i n t he endocri ne s ys t em or t he aut onomi c nervous s ys t em.
1. Renovascular hypertension (s ee al s o Part I: XIV C) i s an i mport ant caus e of s econdary hypert ens i on. In t hi s di s order, hypert ens i on i s t he res ul t of a compl ex i nt erpl ay bet ween act i vat i on of t he reni n–angi ot ens i n–al dos t erone s ys t em and t he s ympat het i c nervous s ys t em. The pat t ern of endocri ne s t i mul at i on may al s o depend on whet her t he vas cul ar l es i ons are uni l at eral or bi l at eral . o
o
a. Ini t i al l y, hi gh angi ot ens i n II l evel s l ead t o vas ocons t ri ct i on and expanded ext racel l ul ar fl ui d vol ume as al dos t erone rel eas e i s s t i mul at ed.
o
o
b. W i t h t i me, vol ume expans i on may l ead t o s uppres s i on of t ot al renal reni n product i on s o t hat ci rcul at i ng reni n val ues may be normal des pi t e t he el evat ed bl ood pres s ure. Thi s may expl ai n why uns t i mul at ed pl as ma reni n l evel s are not a good i ndex of t he pres ence of renovas cul ar P.297
Pa g e 1 4 7 2
ABC Ambe r CHM Conve rte r Tria l ve rsion, http://w w w .proce sste x t.com/a bcchm.html
hypert ens i on i n al l cas es and why s ome pat i ent s (e.g., t hos e wi t h uni l at eral vas cul ar l es i ons ) may not even demons t rat e i ncreas ed reni n s ecret i on.
(1) The normal l evel s of reni n, whi ch are i nappropri at e i n t he pres ence of hypert ens i on and an expanded ext racel l ul ar fl ui d vol ume, s us t ai n t he hypert ens i on.
(2) In addi t i on, duri ng t hi s chroni c phas e, i ncreas ed s ympat het i c nervous s ys t em act i vi t y res ul t i ng from chroni c angi ot ens i n II s t i mul at i on cont ri but es t o hypert ens i on.
2. Renal parenchymal diseases. Hypert ens i on frequent l y accompani es a vari et y of renal di s eas es , hi ghl i ght i ng t he i mport ant cont ri but i on of renal endocri ne and excret ory funct i on i n t he regul at i on of bl ood pres s ure. o
o
a. Altered excretory function. Defect s i n t he renal excret i on of s al t and wat er no doubt cont ri but e t o t he pat hogenes i s of hypert ens i on i n pat i ent s wi t h advanced renal fai l ure. In pat i ent s mai nt ai ned on hemodi al ys i s or peri t oneal di al ys i s , modes t reduct i ons i n ext racel l ul ar fl ui d vol ume can produce a s t ri ki ng reduct i on i n art eri al bl ood pres s ure.
o
o
b. Altered renin–angiotensin–aldosterone activity. Is chemi c changes res ul t i ng from i nt rarenal s carri ng may act i vat e t he reni n–angi ot ens i n
Pa g e 1 4 7 3
ABC Ambe r CHM Conve rte r Tria l ve rsion, http://w w w .proce sste x t.com/a bcchm.html
s ys t em and cont ri but e t o hypert ens i on i n pat i ent s wi t h earl y or advanced renal fai l ure.
3. Endocrinologic causes o
o
a. Oral contraceptives. Thes e agent s caus e hypert ens i on i n approxi mat el y 5% of pat i ent s who us e t hem. Hypert ens i on occurs as a res ul t of es t rogen-i nduced i ncreas es i n angi ot ens i nogen s ynt hes i s i n t he l i ver. Reduced es t rogen l evel s or t he addi t i on of proges t erone may amel i orat e t hi s compl i cat i on.
o
o
b. Mineralocorticoid excess syndromes. Mi neral ocort i coi d exces s l eads t o hypert ens i on by i nduci ng s odi um and wat er ret ent i on, l eadi ng t o expans i on of t he ext racel l ul ar fl ui d vol ume. The hypert ens i on i s oft en accompani ed by hypokal emi a, becaus e mi neral ocort i coi ds promot e renal pot as s i um excret i on i n t he col l ect i ng duct of t he nephron. A vari et y of di s orders produce mi neral ocort i coi d exces s s t at es .
(1) Primary hyperaldosteronism. Tumors of t he adrenal gl and, whi ch are common on aut ops y, are funct i onal l y act i ve i n 1%–3% of pat i ent s , l eadi ng t o mi neral ocort i coi d exces s . Tumors may be uni l at eral or bi l at eral , or t he exces s may res ul t from a di ffus e bi l at eral hyperpl as i a of t he adrenal zona gl omerul os a.
Pa g e 1 4 7 4
ABC Ambe r CHM Conve rte r Tria l ve rsion, http://w w w .proce sste x t.com/a bcchm.html
(2) Glucocorticoid-remediable hyperaldosteronism. In rare pat i ent s , a defect i n adrenal devel opment l eads t o t he s ynt hes i s of al dos t erone i n t he adrenal gl and under t he i nfl uence of ACTH. A gene rearrangement al l ows expres s i on of ACTH-res pons i ve al dos t erone s ynt has e i n t he zona fas ci cul at a rat her t han t he zona gl omerul os a, l eadi ng t o t he product i on of l arge amount s of mi neral ocort i coi d hormone. Treat ment wi t h dexamet has one s uppres s es t hi s abnormal pat hway. Thi s di s order s houl d be s us pect ed i n pat i ent s wi t h a fami l y hi s t ory of hypert ens i on and hypokal emi a.
(3) Exogenous hypermineralocorticoidism. Some pat i ent s may devel op s t ri ki ng mi neral ocort i coi d exces s hypert ens i on from t he i nges t i on of glycyrrhetinic acid, a s ubs t ance found i n European l i cori ce and i n s ome forms of chewi ng t obacco. Gl ycyrrhet i ni c aci d bl ocks t he act i on of 11-bet a-hydroxys t eroi d dehydrogenas e (an enzyme t hat prevent s gl ucocort i coi d bi ndi ng t o recept ors i n t he renal di s t al t ubul e), res ul t i ng i n a hi gh mi neral ocort i coi d s t at e. Pat i ent s pres ent wi t h hypert ens i on, hypokal emi a, and l ow reni n and al dos t erone l evel s .
(4) Cushing syndrome. Gl ucocort i coi d exces s res ul t i ng from exogenous gl ucocort i coi d t herapy, a pi t ui t ary t umor, or an adrenal
Pa g e 1 4 7 5
ABC Ambe r CHM Conve rte r Tria l ve rsion, http://w w w .proce sste x t.com/a bcchm.html
adenoma produces hypert ens i on i n approxi mat el y 80% of pat i ent s wi t h Cus hi ng s yndrome. Hypert ens i on may occur becaus e cort i s ol has mi neral ocort i coi d-l i ke effect s and, t herefore, l eads t o t he ret ent i on of s odi um and wat er. However, many pat i ent s wi t h Cus hi ng s yndrome and hypert ens i on do not mani fes t a l ow reni n s t at e as woul d be expect ed i n pri mary mi neral ocort i coi di s m, i mpl yi ng t hat ot her mechani s ms may cont ri but e t o t he hypert ens i on as wel l .
(5) Liddle syndrome. Thi s fami l i al di s order i s t rans mi t t ed as an aut os omal domi nant form of hypert ens i on. It mani fes t s as hypert ens i on and hypokal emi a wi t h l ow reni n and al dos t erone l evel s . The caus e i s a genet i c defect i n t he bet a s ubuni t of t he s odi um channel i n t he api cal membrane of t he di s t al renal t ubul e. Thi s defect mi mi cs t he act i on of mi neral ocort i coi d P.298
hormones t o enhance s odi um ent ry i n t he di s t al nephron, t hereby enhanci ng s odi um abs orpt i on, i ncreas i ng pot as s i um excret i on, and expandi ng t he ext racel l ul ar fl ui d vol ume, l eadi ng t o an el evat ed art eri al bl ood pres s ure. o
o
c. Pheochromocytoma. Thi s t umor of t he adrenal medul l a i ncreas es t he s ecret i on of cat echol ami nes , l eadi ng t o hypert ens i on. Approxi mat el y 50% of pat i ent s have epi s odi c hypert ens i on; t he remai nder
Pa g e 1 4 7 6
ABC Ambe r CHM Conve rte r Tria l ve rsion, http://w w w .proce sste x t.com/a bcchm.html
have cons t ant hypert ens i on. o
o
d. Miscellaneous causes. Acromegaly, hyperparathyroidism, and hyperthyroidism al s o may produce hypert ens i on. Coarctation of the aorta, as not ed i n Chapt er 1 VII D, i s a correct abl e caus e of hypert ens i on.
III. Approach to The Hypertensive Patient The majori t y of pat i ent s wi t h hypert ens i on have pri mary hypert ens i on; and no furt her eval uat i on i s requi red. Secondary hypert ens i on s houl d be s us pect ed i n pat i ent s who devel op hypert ens i on at t he ext remes of age, pat i ent s wi t hout a fami l y hi s t ory of hypert ens i on, or pat i ent s wi t h hypert ens i on refract ory t o medi cal t herapy.
A. Renovascular hypertension
1. Renovas cul ar hypert ens i on secondary to atherosclerosis of the renal artery s houl d be s us pect ed i n pat i ent s wi t h t he fol l owi ng condi t i ons : o
o
a. Severe hypert ens i on as s oci at ed wi t h advanced hypert ens i ve ret i nopat hy
o
o
b. Hypert ens i on and s evere peri pheral vas cul ar di s eas e
o
o
c. Hypert ens i on and s udden det eri orat i on i n renal funct i on (wi t h or wi t hout t he recent i nt roduct i on of ACE i nhi bi t or t herapy)
o
Pa g e 1 4 7 7
ABC Ambe r CHM Conve rte r Tria l ve rsion, http://w w w .proce sste x t.com/a bcchm.html
o
d. Known di s pari t y i n renal s i ze or funct i on
2. Renovas cul ar hypert ens i on secondary to fibromuscular dysplasia s houl d be s us pect ed i n young women wi t h s evere hypert ens i on, s ys t ol i c and di as t ol i c abdomi nal brui t s , or a s t rong fami l y hi s t ory of hypert ens i on. Magnetic resonance angiography i s a us eful s creeni ng t es t i n t hos e pat i ent s wi t h a hi gh pri or probabi l i t y of di s eas e (s ee Part III: III A 1).
B. Hypermineralocorticoid states Hypermineralocorticoid states s houl d be s us pect ed i n pat i ent s wi t h pers i s t ent hypokal emi a and hypert ens i on. There i s no doubt t hat t hes e di s orders are underdi agnos ed becaus e many pat i ent s wi t h proven mi neral ocort i coi d exces s s t at es are normokal emi c. Nevert hel es s , pers i s t ent and unexpl ai ned hypokal emi a remai ns t he mos t t ypi cal s i gn.
1. Primary hyperaldosteronism. The di agnos i s res t s on fi ndi ng pat i ent s wi t h hypokal emi a and hypert ens i on and provi ng t hat hyperal dos t eroni s m caus es bot h di s orders . o
o
a. Pat i ent s who have unexpl ai ned hypokal emi a and hypert ens i on s houl d have t hei r 24-hour urinary excretion rate for aldosterone meas ured whi l e cons umi ng a 150-mEq s odi um di et . If t he al dos t erone val ue i s el evat ed and pl as ma reni n i s l ow, an abdominal CT scan s houl d be performed t o det ermi ne whet her a s urgi cal l y correct abl e (i .e., adenomat ous ) form of hyperal dos t eroni s m i s pres ent . Recent s t udi es s ugges t t hat a pl as ma al dos t erone–t o pl as ma reni n rat i o of great er t han 50 i s hi ghl y s peci fi c for pri mary
Pa g e 1 4 7 8
ABC Ambe r CHM Conve rte r Tria l ve rsion, http://w w w .proce sste x t.com/a bcchm.html
hyperal dos t eroni s m.
(1) Surgery rel i eves hypert ens i on and hypokal emi a i n over 70% of pat i ent s i f adenoma i s t he caus e. Al t hough s urgery does not cure hypert ens i on i n pat i ent s wi t h bi l at eral hyperpl as i a, t he hypokal emi a does remi t .
(2) Al t hough hypokal emi a may not be found i n s ome pat i ent s wi t h pri mary hyperal dos t eroni s m, pat i ent s wi t h hypokal emi a are more l i kel y t o have a pos i t i ve res pons e t o s urgery, s o t he fi ndi ng of s pont aneous hypokal emi a remai ns a reas onabl e s creeni ng t ool .
o
o
b. Pat i ent s wi t h a fami l y hi s t ory of hypert ens i on s houl d undergo a trial of glucocorticoid therapy t o det ermi ne whet her gl ucocort i coi d-remedi abl e hyperal dos t eroni s m i s pres ent [s ee Part II: III B 2 b (2)].
o
o
c. Selective adrenal venography and sampling for lateralizing aldosterone levels may be performed i n s ome pat i ent s .
2. Pheochromocytoma s houl d be s us pect ed i n hypert ens i ve pat i ent s who experi ence epi s odes of fl us hi ng, di aphores i s , wei ght l os s , and di arrhea. P.299
Pa g e 1 4 7 9
ABC Ambe r CHM Conve rte r Tria l ve rsion, http://w w w .proce sste x t.com/a bcchm.html
o
o
a. Measurement of urinary catecholamines and catecholamine metabolites i s us eful i n maki ng t he di agnos i s .
o
o
b. CT or MRI s houl d be us ed t o l ocat e t he t umor i n t he adrenal medul l a.
o
o
c. An
131
I met ai odobenzyl guani di ne (MIBG) s can
may be performed when t he s us pi ci on of di agnos i s i s hi gh but t he CT or MRI i s negat i ve.
3. Coarctation of the aorta s houl d be s us pect ed i n any young pat i ent wi t h hypert ens i on when t he bl ood pres s ure i n t he l eg i s at l eas t 20 mm Hg l ower t han t hat i n t he arm.
IV. Therapy for Hypertension A. Goals of therapy Treat ment of hypert ens i on i s ai med at reduci ng t he di as t ol i c bl ood pres s ure t o l es s t han 90 mm Hg and t he s ys t ol i c bl ood pres s ure t o l es s t han 140 mm Hg.
B. Indications for therapy It i s current l y bel i eved t hat al mos t al l pat i ent s wi t h a bl ood pres s ure of great er t han 140/90 mm Hg s houl d be t reat ed. In pat i ent s wi t h borderl i ne hypert ens i on, cont rovers y exi s t s about t he ri s ks vers us t he benefi t s of t herapy, but mos t cl i ni ci ans bel i eve t hat t he benefi t s of t herapy predomi nat e and t hat al l pat i ent s wi t h an el evat ed bl ood pres s ure s houl d recei ve t reat ment t o res t ore t he
Pa g e 1 4 8 0
ABC Ambe r CHM Conve rte r Tria l ve rsion, http://w w w .proce sste x t.com/a bcchm.html
val ues t o <130/80 mm Hg.
C. Types of therapy
1. Nonpharmacologic measures o
o
a. Sodium restriction al one may be s uffi ci ent t o l ower bl ood pres s ure i n many hypert ens i ve pat i ent s , part i cul arl y t hos e wi t h a hi gh s odi um i nt ake. Li mi t at i on of s odi um i nt ake t o no more t han 2 g/day has been s hown t o reduce bl ood pres s ure s i gni fi cant l y i n s us cept i bl e pat i ent s .
o
o
b. Weight reduction i n obes e pat i ent s al s o s i gni fi cant l y reduces bl ood pres s ure.
o
o
c. Limitation of alcohol consumption may be benefi ci al . Al cohol pot ent i at es t he act i on of cat echol ami nes and may exacerbat e hypert ens i on i n s us cept i bl e i ndi vi dual s .
2. Pharmacologic measures. Pat i ent compl i ance oft en i s bes t wi t h t herapeut i c regi mens t hat are s i mpl e t o fol l ow. In choos i ng an ant i hypert ens i ve regi men, cons i derat i on s houl d be gi ven not onl y t o s i de effect s , but al s o t o t he number of medi cat i ons and dos es per day t hat t he pat i ent mus t t ake. o
o
a. Diuretics, whi ch oft en are t he fi rs t l i ne of ant i hypert ens i ve t herapy, i ni t i al l y reduce ext racel l ul ar fl ui d vol ume. Aft er s everal mont hs , a
Pa g e 1 4 8 1
ABC Ambe r CHM Conve rte r Tria l ve rsion, http://w w w .proce sste x t.com/a bcchm.html
cont i nued reduct i on cannot be meas ured, yet bl ood pres s ure i s mai nt ai ned at a l ower l evel pri mari l y due t o decreas ed vas cul ar res i s t ance. The exact mechani s m whereby di uret i cs reduce peri pheral vas cul ar res i s t ance i s unknown.
(1) Agents
(a) T hiazide diuretics (e.g., hydrochlorothiazide, chlorthalidone, chlorothiazide, and metolazone) are more effect i ve ant i hypert ens i ve agent s t han loop diuretics (e.g., furos emi de, bumet ani de, t ors emi de, and et hacryni c aci d) i f s erum creat i ni ne i s <2 mg/dL.
(b) Potassium-sparing diuretics (e.g., amiloride and triamterene) act i n t he di s t al nephron and col l ect i ng t ubul e and are us eful i n t he t reat ment of hypermi neral ocort i coi d s t at es .
(c) Spironolactone and eplerenone are compet i t i ve i nhi bi t ors of al dos t erone.
(2) Side effects. Hypokal emi a, hypergl ycemi a, hyperl i pi demi a, hyperuri cemi a, hypercal cemi a (t hi azi des onl y), and prerenal azot emi a may occur. Therefore, peri odi c l aborat ory det ermi nat i on of pot as s i um and BUN l evel s i s i ndi cat ed. Uri c aci d s houl d be moni t ored peri odi cal l y i n pat i ent s wi t h a
Pa g e 1 4 8 2
ABC Ambe r CHM Conve rte r Tria l ve rsion, http://w w w .proce sste x t.com/a bcchm.html
hi s t ory of gout . Some s t udi es i mpl i cat e hypokal emi a caus ed by di uret i cs as an i mport ant ri s k fact or for unexpl ai ned s udden deat h, a cat as t rophi c compl i cat i on t hat can be avoi ded by pot as s i um s uppl ement at i on or us e of pot as s i um-s pari ng di uret i cs . o
o
b. β-Adrenergic blocking agents reduce cardi ac out put and reni n rel eas e. However, i t i s not cl ear t hat t hes e are t he pri mary mechani s ms by whi ch β-bl ockers reduce bl ood pres s ure. The combi nat i on of a β-bl ocker and a di uret i c agent reduces t he di as t ol i c bl ood pres s ure t o bel ow 90 mm Hg i n approxi mat el y 80% of pat i ent s wi t h mi l d-t o-moderat e hypert ens i on.
(1) Agents. Propranolol, nadolol, metoprolol, atenolol, timolol, betaxolol, carteolol, pindolol, carvedilol, acebutolol, and labetalol are among t he β-bl ockers t hat are current l y approved for t he t reat ment of hypert ens i on. (Labet al ol i s al s o an α-adrenergi c ant agoni s t .) P.300
(2) Side effects. Bronchos pas m, bradycardi a, a wors eni ng of exi s t i ng CHF, occas i onal i mpot ence, fat i gue, depres s i on, and ni ght mares may occur.
o
o
c. Centrally acting adrenergic antagonists i nhi bi t
Pa g e 1 4 8 3
ABC Ambe r CHM Conve rte r Tria l ve rsion, http://w w w .proce sste x t.com/a bcchm.html
s ympat het i c out fl ow from t he CNS by s t i mul at i ng cent ral α-adrenorecept ors , reduci ng peri pheral res i s t ance and bl ood pres s ure.
(1) Agents. Methyldopa and clonidine are t he commonl y us ed cent ral l y act i ng drugs .
(2) Side effects. Somnol ence, ort hos t at i c hypot ens i on, Coombs '-pos i t i ve hemol yt i c anemi a, i mpot ence, and hepat i c i njury are s i de effect s of met hyl dopa. W i t h cl oni di ne, t he major but rare s i de effect i s a rebound phenomenon t hat produces s evere hypert ens i on aft er wi t hdrawal of t he drug.
o
o
d. Peripherally acting sympathetic nerve antagonists caus e bl ood pres s ure t o fal l by reduci ng cat echol ami ne rel eas e from peri pheral s ympat het i c nerves .
(1) Agents. The mos t commonl y us ed drugs i n t hi s cat egory are res erpi ne and guanet hi di ne. Reserpine depl et es nerve s t orage ves i cl es of norepi nephri ne and, t hus , l i mi t s norepi nephri ne s ecret i on. Guanethidine di rect l y i nhi bi t s t he rel eas e of norepi nephri ne from adrenergi c neurons .
(2) Side effects. The major s i de effect of res erpi ne i s depres s i on; i t al s o may i ncreas e t he i nci dence of gas t ri c ul cerat i on. Ort hos t at i c hypot ens i on i s t he mos t common s i de effect
Pa g e 1 4 8 4
ABC Ambe r CHM Conve rte r Tria l ve rsion, http://w w w .proce sste x t.com/a bcchm.html
wi t h guanet hi di ne t herapy. o
o
e. α-Adrenergic blocking agents pharmacol ogi cal l y ant agoni ze norepi nephri ne's s t i mul at i on of adrenergi c recept ors , reduci ng bl ood pres s ure by reduci ng t ot al peri pheral res i s t ance. Prazosin, doxazosin, terazosin, and phenoxybenzamine are t he mos t commonl y us ed α-bl ockers .
o
o
f. Calcium channel antagonists reduce s moot h mus cl e t one, t hereby act i ng t o produce vas odi l at i on, by modul at i ng cal ci um rel eas e i n s moot h mus cl e. The s ubs equent reduct i on i n t ot al peri pheral res i s t ance reduces bl ood pres s ure. Cal ci um channel bl ockers may al s o reduce cardi ac out put by decreas i ng venous ret urn and t he i not ropi c s t at e.
(1) Agents. Current l y, t he di hydropyri di nes (i .e., nifedipine, nicardipine, isradipine, felodipine, nimodipine, amlodipine, and nitrendipine), and t he non-di hydropyri di nes (e.g., diltiazem, and verapamil) are approved for ant i hypert ens i ve t herapy.
(2) Side effects. Di hydropyri di nes may caus e headache, fl us hi ng, and peri pheral edema. Verapami l , and t o a l es s er ext ent , di l t i azem, have cardi odepres s ant act i ons (unl i ke t he di hydropyri di nes ) maki ng t hei r us e probl emat i c i n pat i ent s wi t h CHF.
o
Pa g e 1 4 8 5
ABC Ambe r CHM Conve rte r Tria l ve rsion, http://w w w .proce sste x t.com/a bcchm.html
o
g. Direct vasodilators di l at e art eri es and art eri ol es , reduci ng bl ood pres s ure by reduci ng t ot al peri pheral res i s t ance. They are part i cul arl y effect i ve when us ed wi t h a β-bl ocker t hat i nhi bi t s t he refl ex t achycardi a caus ed by di rect vas odi l at i on.
(1) Agents. Hydralazine and minoxidil are t he mos t commonl y us ed di rect vas odi l at ors .
(2) Side effects. The major s i de effect s of hydral azi ne are headache and a l upus -l i ke s yndrome; t he l at t er i s revers ed when t he drug i s di s cont i nued. The major s i de effect s of mi noxi di l are ort hos t at i c hypot ens i on and faci al hi rs ut i s m.
h. ACE inhibitors bl ock t he convers i on of angi ot ens i n I t o angi ot ens i n II (a vas ocons t ri ct or), t hus reduci ng t ot al peri pheral res i s t ance. In addi t i on, al dos t erone product i on i s decreas ed, reduci ng t he ret ent i on of s odi um and wat er. o
o
(1) Agents. Captopril, enalapril, fosinopril, benazepril, quinapril, ramipril, and lisinopril are us eful ACE i nhi bi t ors .
o
o
(2) Side effects. The mos t common s i de effect of ACE i nhi bi t ors i s cough, whi ch t ypi cal l y di s appears wi t h di s cont i nuat i on of t he drug. Hyperkal emi a may occas i onal l y occur as a res ul t of reduced
Pa g e 1 4 8 6
ABC Ambe r CHM Conve rte r Tria l ve rsion, http://w w w .proce sste x t.com/a bcchm.html
al dos t erone s ecret i on. Ot her s evere but uncommon s i de effect s i ncl ude l eukopeni a, ras h, and angi oedema. Becaus e angi ot ens i n II mai nt ai ns t he GFR duri ng s t at es of reduced renal bl ood fl ow (by mai nt ai ni ng a hi gh vas cul ar res i s t ance i n t he pos t gl omerul ar ves s el s ), t he us e of ACE i nhi bi t ors i n pat i ent s wi t h compromi s ed renal bl ood fl ow may l ead t o a form of acut e renal fai l ure. Reduct i on i n dos e or di s cont i nuat i on of drug i s us ual l y s uffi ci ent t o revers e t he fal l i n GFR.
i. Angiotensin II–type 1 receptor antagonists (angi ot ens i n recept or bl ockers ) are a cl as s of drugs t hat provi de t he benefi t s of bl ockade of t he angi ot ens i n s ys t em wi t hout s ome of t he annoyi ng s i de effect s , part i cul arl y cough. Thes e agent s appear t o offer al l of t he benefi t s of ACE i nhi bi t ors wi t h fewer s i de effect s . P.301
(1) Agents. Losartan, valsartan, irbesartan, candesartan, telmisartan, eprosartan, and olmesartan are avai l abl e.
(2) Side effects. Thes e are s i mi l ar t o t hos e of ACE i nhi bi t ors , except t hat cough has not been s een wi t h recept or bl ockers .
Pa g e 1 4 8 7
ABC Ambe r CHM Conve rte r Tria l ve rsion, http://w w w .proce sste x t.com/a bcchm.html
D. Therapeutic strategies
1. First-line therapy. Vari ous agent s are now cons i dered appropri at e for i ni t i al ant i hypert ens i ve t herapy. Many phys i ci ans s t i l l begi n t herapy wi t h a di uret i c, but β-bl ockers , ACE i nhi bi t ors , angi ot ens i n recept or bl ockers , or cal ci um channel bl ockers are al s o appropri at e i ni t i al drugs . At t he very bes t , monot herapy produces opt i mal cont rol of bl ood pres s ure i n onl y 75% of pat i ent s .
2. Second-line therapy. Submaxi mal dos es of t wo or t hree ant i hypert ens i ve agent s added i n a s t epwi s e fas hi on have been advocat ed as a s ucces s ful approach. The addi t i on of 12.5 or 25 mg hydrochl orot hi azi de wi l l pot ent i at e t he act i on of a number of ant i hypert ens i ve agent s , part i cul arl y t he ACE i nhi bi t ors and t he angi ot ens i n II–recept or ant agoni s t s . If t he bl ood pres s ure i s s t i l l uncont rol l ed, t he us e of mul t i pl e drug regi mens or t he addi t i on of di rect vas odi l at ors i s i ndi cat ed.
V. Hypertensive Crisis Thi s condi t i on i s defi ned as s evere hypert ens i on charact eri zed by a di as t ol i c bl ood pres s ure of great er t han 140 mm Hg. Bl ood pres s ure el evat i on t o t hi s degree can caus e vas cul ar damage, pul monary edema, encephal opat hy, ret i nal hemorrhages , renal damage, and deat h.
A. Diagnosis A di as t ol i c bl ood pres s ure of great er t han 140 mm Hg, fundus copi c fi ndi ngs of papi l l edema, changes i n neurol ogi c and ment al s t at us , and an abnormal renal s edi ment are t he hal l marks of hypert ens i ve cri s i s .
Pa g e 1 4 8 8
ABC Ambe r CHM Conve rte r Tria l ve rsion, http://w w w .proce sste x t.com/a bcchm.html
B. Therapy Immedi at e l oweri ng of t he bl ood pres s ure i s i ndi cat ed.
1. Infus i on of sodium nitroprusside i s a mai ns t ay of t herapy. o
o
a. The pat i ent 's bl ood pres s ure s houl d be moni t ored cons t ant l y s o t hat t he dos e can be adjus t ed t o mai nt ai n t he bl ood pres s ure wi t hi n t he des i red range. An exces s i ve reduct i on i n bl ood pres s ure can be revers ed rapi dl y by reduci ng t he dos e of t he i nfus i on.
o
o
b. Long-t erm admi ni s t rat i on of hi gh-dos e ni t roprus s i de can l ead t o cyani de i nt oxi cat i on. The l i kel i hood of t hi s compl i cat i on i ncreas es i n pat i ent s wi t h renal dys funct i on or aft er 3 days of t herapy.
o
o
c. β-Bl ockers s houl d be us ed i n conjunct i on wi t h ni t roprus s i de t o avoi d refl ex t achycardi a.
2. Labetalol, nicardipine, and nitroglycerine are al s o effect i ve i nt ravenous t herapeut i c agent s .
3. T he use of sublingual nifedipine has been shown to be dangerous (precipitating coronary ischemia or stroke), and it should not be a part of therapy.
P.302
Pa g e 1 4 8 9
ABC Ambe r CHM Conve rte r Tria l ve rsion, http://w w w .proce sste x t.com/a bcchm.html
Study Questions/Answers and Explanations 1. A 60-year-old man develops new-onset nephrotic syndrome. A percutaneous renal biopsy is contemplated to determine the nature of the glomerular disease. In which of the following situations is the renal biopsy absolutely contraindicated? A. The pat i ent has a di as t ol i c bl ood pres s ure of 120 mm Hg. B. The pat i ent has a s erum creat i ni ne l evel of 2.5 mg/dL (normal = 0.8–1.4 mg/dL). C. The pat i ent has di abet es mel l i t us . D. The pat i ent has heavy prot ei nuri a but no ot her uri nary abnormal i t y. E. The pat i ent has undergone a previ ous renal bi ops y. Vi ew Ans wer 1. T he answer is A [
Part I: I D 2]. Hypert ens i on i s an abs ol ut e
cont rai ndi cat i on t o performi ng a renal bi ops y becaus e t he i nci dence of s ubcl i ni cal bl eedi ng at t he s i t e i s hi gh (75%). In a hypert ens i ve pat i ent , t hi s bl eedi ng may l ead t o a major, l i fe-t hreat eni ng hemorrhage. Mi l d renal i ns uffi ci ency (as evi denced by a s mal l decreas e i n creat i ni ne cl earance), nephrot i c s yndrome wi t hout uri nary cas t s , and moderat el y advanced age are not cont rai ndi cat i ons t o performi ng a renal bi ops y. Mul t i pl e renal bi ops i es may be performed i n t he s ame pat i ent . A previ ous renal bi ops y i s not a cont rai ndi cat i on for s ubs equent bi ops i es . 2. A 71-year-old man in the surgical intensive care unit is seen for acute renal failure. After an operation for removal of gallstones, he had a persistent drainage from his biliary catheter associated with spiking fevers to 38.9°C. He has been taking gentamicin and ampicillin/sulbactam for the past 10 days. Over the last 4 days, his serum creatinine level has increased at a rate of 1 mg/dL/day, but his urine output of 1.5 L/day has not diminished. At no time has he had any hypotension during this hospitalization. Physical examination shows normal blood pressure and vital signs. Results of laboratory studies indicate a creatinine level of 7.1 mg/dL, and renal ultrasonography reveals no evidence of
Pa g e 1 4 9 0
ABC Ambe r CHM Conve rte r Tria l ve rsion, http://w w w .proce sste x t.com/a bcchm.html
obstruction. Which of the following is the most likely cause of the acute renal failure? A. Seps i s B. Trauma t o t he uret er duri ng s urgery C. Gent ami ci n nephrot oxi ci t y D. Acut e gl omerul onephri t i s E. Cephal ot hi n-i nduced acut e renal fai l ure Vi ew Ans wer 2. T he answer is C [Tabl e 6-1]. Approxi mat el y 5%–10% of pat i ent s t reat ed wi t h gent ami ci n devel op a non-ol i guri c form of acut e renal fai l ure. Al t hough t he pat i ent has recei ved normal dos es of gent ami ci n, i t s accumul at i on i n t he ki dney has produced a l at e form of acut e renal fai l ure. The s erum creat i ni ne l evel t hen ri s es whi l e an i nappropri at el y hi gh dos e i s mai nt ai ned. Thi s ri s e exacerbat es t he renal i ns uffi ci ency and prol ongs t he cours e of acut e renal fai l ure. The non-ol i guri c nat ure of t hi s pat i ent 's cl i ni cal condi t i on al s o i s a t ypi cal fi ndi ng i n gent ami ci n nephrot oxi ci t y. Al t hough t he pat i ent coul d have obs t ruct i ve uropat hy, t he negat i ve res ul t s of ul t ras onography s t rongl y i ndi cat e ot herwi s e. Cephal ot hi n can produce an acut e i nt ers t i t i al nephri t i s , but t he pat i ent 's cl i ni cal cours e i s much more compat i bl e wi t h t he more common drug-i nduced di s eas e of gent ami ci n nephrot oxi ci t y. Acut e gl omerul onephri t i s us ual l y i s as s oci at ed wi t h hypert ens i on and an act i ve uri nary s edi ment cont ai ni ng cas t s , prot ei n, and red bl ood cel l s (RBCs ). 3. A 39-year-old man who is experiencing labored breathing and mental obtundation comes to the emergency department. Physical examination is unremarkable. Laboratory studies show the following: serum sodium = 144 mEq/L, serum potassium = 3.7 mEq/L, serum chloride = 97 mEq/L, plasma bicarbonate -
concentration [HCO 3 ] = 16 mEq/L, arterial pH = 7.38, and PaCO 2
= 21 mEq/L. T hese findings indicate which of the following
acid–base disturbances? A. Res pi rat ory al kal os i s B. Met abol i c aci dos i s
Pa g e 1 4 9 1
ABC Ambe r CHM Conve rte r Tria l ve rsion, http://w w w .proce sste x t.com/a bcchm.html
C. Met abol i c aci dos i s and res pi rat ory al kal os i s D. Met abol i c aci dos i s and met abol i c al kal os i s E. No aci d–bas e di s t urbance Vi ew Ans wer 3. T he answer is C [Part II: IV C, D]. Al t hough t he art eri al pH i s wi t hi n t he normal range, t he pat i ent has t wo s eparat e aci d–bas e di s t urbances (met abol i c aci dos i s and res pi rat ory al kal os i s ) whi ch, t oget her, t end t o offs et t hei r effect s on pH. The l ow PaCO 2 i n t hi s cas e i s much l ower t han woul d be expect ed i f t he pat i ent had onl y res pi rat ory compens at i on. Thus , i f t he pat i ent has a s i ngl e aci d–bas e di s t urbance, namel y met abol i c aci dos i s , t hen t he expect ed PaCO 2 i s cal cul at ed us i ng W i nt ers ' formul a: -
Expect ed PaCO 2 = 1.5 × [HCO 3 ] + 8 ± 2 = 1.5 × 16 + 8 ± 2 = 32 ± 2 The meas ured PaCO 2 of l es s t han 32 mm Hg i ndi cat es t hat t he pat i ent al s o has a pri mary res pi rat ory di s t urbance, namel y res pi rat ory al kal os i s . 4. A 48-year-old man with a history of obesity, hypertension, and type 2 diabetes returns to your office for a routine follow-up visit. His body mass index (BMI) is 32, he consumes a low-fat diet but doesn' t follow the low sodium recommendations, and he gets little exercise. On physical examination, his blood pressure is 162/92 mm Hg., pulse 88 beats per minute, and he is comfortable. With the exception of mild retinopathy, his physical examination is normal. His hemoglobin A1c is 8, his blood urea nitrogen (BUN) is 24, and his serum creatinine is 1.5 mg/dL. His spot urine protein-to-creatinine ratio is 1.3. In addition to lifestyle modification, which of the following hypertensive agents should be initiated? A. Hydrochl orot hi azi de B. Li s i nopri l C. Fel odi pi ne D. Hydral azi ne
Pa g e 1 4 9 2
ABC Ambe r CHM Conve rte r Tria l ve rsion, http://w w w .proce sste x t.com/a bcchm.html
E. Cl oni di ne Vi ew Ans wer 4. T he answer is D [Part I: X L]. It i s cri t i cal t o recogni ze t hat i n a pat i ent wi t h di abet es and prot ei nuri a, i nhi bi t ors of t he reni n–angi ot ens i n–al dos t erone s ys t em [angi ot ens i n-convert i ng enz yme (ACE) i nhi bi t ors and angi ot ens i n recept or bl ockers (ARBs )] are t he mai ns t ay of t herapy. Al t hough t he ot her agent s are l i kel y t o l ower bl ood pres s ure, t hey wi l l not provi de addi t i onal renal prot ect i on. It i s l i kel y t hat t hi s pat i ent wi l l requi re more t han one bl ood pres s ure medi cat i on, and t he next agent s el ect ed s houl d be a t hi az i de, as i t act s s ynerget i cal l y wi t h t he ACE i nhi bi t ors and ARBs . 5. A 45-year-old man enters the hospital because of acute flank pain and red urine, which began at night and awakened him. Until this point, the man has been in good health. Physical examination in the P.303 emergency department is normal, but he does have hematuria, with essentially normal laboratory studies. Radiography of the abdomen reveals a stone in the right kidney. An abdominal CT scan shows the stone to be nonobstructing. Further laboratory studies demonstrate normal serum calcium and phosphate, and a subsequent urine culture is negative. Which of the following types of kidney stone is most likely to have caused this condition? A. Cal ci um oxal at e s t one B. Uri c aci d s t one C. Xant hi ne s t one D. St ruvi t e s t one E. Cys t i ne s t one Vi ew Ans wer 5. T he answer is A [Part I: VII A; Tabl e 6-6]. The pat i ent mos t l i kel y has i di opat hi c cal ci um oxal at e s t ones . Thi s concl us i on i s bas ed on t he fact t hat i t i s t he mos t common form of ki dney s t one, t he s t one i s radi opaque, and many of t he fi ndi ngs s ugges t t hat t he
Pa g e 1 4 9 3
ABC Ambe r CHM Conve rte r Tria l ve rsion, http://w w w .proce sste x t.com/a bcchm.html
ot her ans wers are wrong. Fi rs t , uri c aci d s t ones are radi ol ucent , s o a s t one woul d not have been s een on t he radi ographi c fl at pl at e pri or t o t he i nt ravenous urogram. Second, xant hi ne s t ones are ext remel y rare and are al s o radi ol ucent . Thi rd, s t ruvi t e s t ones are found onl y i n t he s et t i ng of a uri nary t ract i nfect i on. The abs ence of a pos i t i ve uri ne cul t ure s t rongl y s ugges t s t hat s t ruvi t e s t ones are not pres ent . Fourt h, al t hough cys t i ne s t ones coul d pres ent i n t hi s fas hi on even t hough t he pat i ent i s mi ddl e-aged, cys t i ne s t ones t ypi cal l y pres ent much earl i er and repres ent l es s t han 1% of t he s t ones anal yzed i n l arge s t udi es . Thus , on a purel y s t at i s t i cal bas i s , t he pat i ent i s unl i kel y t o have cys t i ne s t ones . However, al l pat i ent s wi t h ki dney s t one format i on who have not had t hei r s t one crys t al l ography anal yzed s houl d have a 24-hour cys t i ne det ermi nat i on t o be cert ai n t hat cys t i nuri a i s not pres ent . 6. A 70-year-old man with long-standing hypertension has developed more severe hypertension over a 6-month period. Despite antihypertensive medication, his blood pressure has risen from 140/90 mm Hg to 175/105 mm Hg. In addition, he had an episode of “ flash pulmonary edema― that led to his hospitalization for 1 week. He has continued to take his antihypertensive medication faithfully, but he has developed some intermittent claudication in his left leg. Although he has a long history of smoking, he has had no other medical problems. Which of the following is the most likely cause of this clinical disorder? A. Renal art ery s t enos i s B. Pheochromocyt oma C. Hyperal dos t eroni s m D. Pri mary wors eni ng of hypert ens i on E. Coarct at i on of t he aort a Vi ew Ans wer 6. T he answer is A [Part III: III A]. Renal art ery s t enos i s i s a common compl i cat i on i n pat i ent s wi t h l ong-s t andi ng hypert ens i on and peri pheral vas cul ar di s eas e. As many as 80% of pat i ent s wi t h t hi s combi nat i on of di s orders have renal art ery s t enos i s . It i s
Pa g e 1 4 9 4
ABC Ambe r CHM Conve rte r Tria l ve rsion, http://w w w .proce sste x t.com/a bcchm.html
t ypi cal l y at heros cl erot i c i n nat ure and i nvol ves t he os t i a of t he renal art eri es . “Fl as h pul monary edema― commonl y accompani es t hi s condi t i on and probabl y repres ent s acut e changes i n hemodynami c s t at us i nduced by an i ncreas ed s ens i t i vi t y t o t he adrenergi c nervous s ys t em. Ot her feat ures of t hi s di s order are wors eni ng of hypert ens i on as wel l as devel opment of azot emi a. Thi s cons t el l at i on of fi ndi ngs oft en neces s i t at es i nt ervent i on, wi t h ei t her percut aneous t rans l umi nal angi opl as t y (PCTA) or s urgery. 7. A 68-year-old woman with end-stage renal disease and a history of hypertension and systemic lupus erythematosus presents to the emergency department with complaints of sweats and palpitations, shortness of breath, and dizziness. Physical examination is remarkable for an anxious-appearing woman with a blood pressure of 148/62 mm Hg, pulse 120 beats per minute, and a temperature of 36°C. She has mild carotid bruits, a soft systolic ejection murmur with a regular rate and rhythm, and a few scattered crackles on examination. Laboratory studies are sent. Her electrocardiogram is shown below.
A. St art hemodi al ys i s B. Admi ni s t er i ns ul i n and gl ucos e C. Admi ni s t er cal ci um gl uconat e D. Gi ve epi nephri ne nebul i zers E. Admi ni s t er oral Kayexal at e res i n Vi ew Ans wer 7. T he answer is C [Part II: III C]. Thi s pat i ent has ECG changes and s ympt oms cons i s t ent wi t h hyperkal emi a as t he res ul t of mi s s ed di al ys i s . Al t hough each of t he i ndi vi dual res pons es i s accept abl e,
Pa g e 1 4 9 5
ABC Ambe r CHM Conve rte r Tria l ve rsion, http://w w w .proce sste x t.com/a bcchm.html
t he mos t i mport ant fi rs t s t ep i s t o admi ni s t er i nt ravenous cal ci um gl uconat e, whi ch wi l l s t abi l i ze t he myocardi al membranes . Thi s has an i mmedi at e effect . Ins ul i n and gl ucos e, as wel l as epi nephri ne, act rapi dl y, but t he effect di s s i pat es qui ckl y. Kayexal at e t akes t i me t o work t o reduce t he pot as s i um l oad. Hemodi al ys i s for an exi s t i ng pat i ent woul d be an accept abl e al t ernat i ve i f t he di al ys i s coul d be i ni t i at ed rapi dl y. 8. A 78-year-old man enters the hospital because of abnormalities of urination. T oday he is passing large amounts of urine; however, some days he passes no urine at all. He now has a blood pressure of 180/90 mm Hg, and, otherwise, his physical examination is normal. Laboratory studies show a blood urea nitrogen (BUN) of 120 mg/dL and a serum creatinine of 4.2 mg/dL. Urinalysis reveals a specific gravity of 1.010; urine that is negative for protein, glucose, ketone bodies, and blood; and an occasional P.304 white blood cell (WBC) per high-power field on microscopic examination. Which of the following is the most likely cause of the renal insufficiency? A. Obs t ruct i ve uropat hy B. Acut e gl omerul onephri t i s C. Acut e i nt ers t i t i al nephri t i s D. Acut e t ubul ar necros i s (ATN) E. Chroni c renal fai l ure of uns peci fi ed nat ure Vi ew Ans wer 8. T he answer is A [Part I: VIII A, C]. The i nci dence of pros t at i s m i n el derl y men i s s o great t hat i t mus t be cons i dered t he pri mary caus e of renal i ns uffi ci ency unt i l proven ot herwi s e. Thi s pat i ent 's hi s t ory i s cl as s i c i n t hat he had 1 or 2 days on whi ch he s eemed t o pas s no uri ne fol l owed by days of hi gh uri ne fl ow, a pat t ern t hat i s caus ed by t he gradual accumul at i on of l arge amount s of uri ne i n t he col l ect i ng s ys t em under pres s ure, whi ch event ual l y may overcome s ome degree of obs t ruct i on. The hi gh pres s ure i s
Pa g e 1 4 9 6
ABC Ambe r CHM Conve rte r Tria l ve rsion, http://w w w .proce sste x t.com/a bcchm.html
t rans mi t t ed back t o t he ki dneys and res ul t s i n renal i ns uffi ci ency. Acut e gl omerul onephri t i s and acut e i nt ers t i t i al nephri t i s are rul ed out by t he normal res ul t s of uri nal ys i s . The pos s i bi l i t y of acut e t ubul ar necros i s (ATN) s houl d be cons i dered, but no i nformat i on i n t he hi s t ory s ugges t s recent s urgery or nephrot oxi c drug i nt ake t hat woul d have produced ATN. The bes t way t o s creen for obs t ruct i ve uropat hy i s renal ul t ras onography, whi ch woul d demons t rat e di l at ed upper t ract cal yces . 9. A 47-year-old man enters the hospital with nephrotic syndrome. He reports that he initially experienced edema of his feet in the morning, but this condition rapidly progressed until the edema extended to his midcalves and lasted throughout the day. He has been in good health all his life and has not seen a physician in the last 5 years. On physical examination, blood pressure is 155/100 mm Hg, pulse is 88 bpm, respiratory rate is 15, and temperature is 37.0°C. Examination of the heart and lungs reveals no evidence of congestive heart failure (CHF), examination of the abdomen indicates mild ascites with normal hepatic size, and examination of the extremities reveals 4+ edema to the midcalf. Laboratory studies show a blood urea nitrogen (BUN) of 10 mg/dL and creatinine of 1.0 mg/dL. T he rest of the laboratory studies are unrevealing. Urinalysis reveals 4+ protein and one red blood cell (RBC) per high-power field. No RBC casts or other cellular elements are seen on urinalysis. T he 24-hour urine contains 9.6 g of protein. Which of the following is most likely to account for this clinical condition? A. Membranous nephropat hy B. Pos t s t rept ococcal gl omerul onephri t i s C. Lupus nephri t i s D. Amyl oi dos i s E. Di abet es mel l i t us Vi ew Ans wer 9. T he answer is A [Part I: X C; XII B 4]. Thi s pat i ent mos t l i kel y has membranous nephropat hy, whi ch i s t he mos t common caus e of
Pa g e 1 4 9 7
ABC Ambe r CHM Conve rte r Tria l ve rsion, http://w w w .proce sste x t.com/a bcchm.html
nephrot i c s yndrome i n pat i ent s i n t hi s age group. It i s charact eri zed by an abs ence of evi dence for a hi gh degree of gl omerul ar i nfl ammat i on. Thus , t he uri nal ys i s , whi ch reveal s l i t t l e i n t he way of cel l ul ar el ement s and heavy prot ei nuri a, s upport s t hi s di agnos i s . Pos t s t rept ococcal gl omerul onephri t i s and l upus gl omerul onephri t i s are much more l i kel y t o demons t rat e act i ve uri nary s edi ment s wi t h red bl ood cel l s (RBCs ) and RBC cas t s . In addi t i on, t hes e condi t i ons are oft en as s oci at ed wi t h s ys t emi c mani fes t at i ons of t he underl yi ng di s eas e; none are pres ent i n t hi s pat i ent . Amyl oi dos i s coul d gi ve a cl i ni cal pi ct ure s i mi l ar t o t hat des cri bed, but t he vas t majori t y of pat i ent s wi t h s ys t emi c amyl oi dos i s have s ys t emi c s i gns s uch as peri pheral neuropat hy, aut onomi c neuropat hy, or cardi ac di s eas e at t he t i me t hat t hey devel op nephrot i c s yndrome. Di abet i c nephropat hy i s unl i kel y becaus e of t he abs ence of hi s t ory. Fai l ure t o fi nd abnormal ret i nal fi ndi ngs on opht hal mos copi c exami nat i on woul d al s o make di abet i c nephropat hy an unl i kel y di agnos i s . 10. A 31-year-old man enters the hospital with acute onset of edema and hematuria. He had been well until 3 weeks earlier, when he had a sore throat and was treated with oral penicillin. Since that time, he has felt generally good but has noted the onset of dark, tea-colored urine and swelling of his legs. He denies any other symptoms, including rash, joint pain, chest pain, and the use of any medication but the penicillin. On physical examination, blood pressure is 160/110 mm Hg, pulse is 85 bpm, respiratory rate is 15, and temperature is 37.0°C. Other findings include marked peripheral edema extending to the midleg. Laboratory studies reveal a blood urea nitrogen (BUN) of 20 mg/dL, creatinine of 1.3 mg/dL, and normal electrolytes. T he serum complement level, including CH 5 0 and C4, is reduced by 50%, the anti-deoxyribonuclease B antibody and the antistreptolysin O titer are increased, and the anti-glomerular basement membrane (anti-GBM) antibody assay and antineutrophilic cytoplasmic antibody (ANCA) level are within normal limits. Urinalysis reveals 4+ protein, 4+ blood, and no glucose. Microscopic examination reveals three to four red
Pa g e 1 4 9 8
ABC Ambe r CHM Conve rte r Tria l ve rsion, http://w w w .proce sste x t.com/a bcchm.html
blood cell (RBC) casts per high-power field as well as many RBCs and white blood cells (WBCs). No bacteria are visible. Which of the following is the most likely cause of this acute renal syndrome? A. Pos t s t rept ococcal gl omerul onephri t i s B. Sys t emi c l upus eryt hemat os us (SLE) C. Goodpas t ure's s yndrome D. Immunogl obul i n A (IgA) nephropat hy E. Al l ergi c react i on t o peni ci l l i n Vi ew Ans wer 10. T he answer is A [Part I: X I]. The mos t l i kel y et i ol ogy of t hi s pat i ent 's acut e renal s yndrome i s pos t s t rept ococcal gl omerul onephri t i s . The fi ndi ng of a reduced s erum compl ement and el evat ed ant i s t rept ol ys i n O t i t er i n t he cl i ni cal s et t i ng of a s t rept ococcal i nfect i on s t rongl y s ugges t s t hi s caus e. Al t hough s ys t emi c l upus eryt hemat os us (SLE) may produce s ome as pect s of t he cl i ni cal pi ct ure, part i cul arl y t he nephri t i c renal s yndrome, t he abs ence of ot her s ys t emi c s i gns of SLE makes t hi s condi t i on hi ghl y unl i kel y. However, an ant i nucl ear ant i body (ANA) as s ay s houl d be performed t o ful l y excl ude t hi s di agnos i s . Goodpas t ure's s yndrome i s unl i kel y i n t he abs ence of a hi s t ory of pul monary abnormal i t i es or hemopt ys i s . Immunogl obul i n A (IgA) nephropat hy can pres ent as a s evere, acut e nephri t i c pi ct ure but s houl d not produce t he ot her s erol ogi c abnormal i t i es s een i n t hi s pat i ent . A react i on t o peni ci l l i n can res ul t i n an acut e renal i njury; however, l i ke IgA nephropat hy, i t i s not as s oci at ed wi t h t he compl ement and ant i s t rept ococcal ant i body t i t ers t hat are s een. Furt hermore, nephrot i c s yndrome woul d be an uncommon cons equence of peni ci l l i n react i on, whi ch much more t ypi cal l y pres ent s as an acut e i nt ers t i t i al i njury t o t he ki dney, wi t h renal i ns uffi ci ency, eos i nophi l s and ot her whi t e cel l el ement s i n t he uri ne, and mi ni mal prot ei nuri a. 11. A 64-year-old man enters the hospital because of renal insufficiency. Until 6 months earlier, when he developed persistent back pain, he was in good health. At that time, he was found to be severely anemic, and his blood urea nitrogen
Pa g e 1 4 9 9
ABC Ambe r CHM Conve rte r Tria l ve rsion, http://w w w .proce sste x t.com/a bcchm.html
(BUN) and creatinine levels were elevated (42 mg/dL and 4.6 mg/dL, respectively). He now undergoes further evaluation. He denies the use of any medications, any past history of renal injury, and any difficulty in voiding. He does complain of persistent weakness and easy fatigability, and his back pain has become more severe over the last 2 weeks. P.305
On physical examination, blood pressure is 120/80 mm Hg, pulse is 70 bpm, respiratory rate is 15, and temperature is 37.0°C. Major physical findings include severe pallor as well as clear evidence of muscle wasting. Urinalysis reveals 1+ protein on dipstick testing and 4+ on sulfosalicylic acid testing. Microscopic examination of the urine reveals an occasional broad cast and an occasional granular cast. Laboratory studies give the following results: BUN = 61 mg/dL, creatinine = 5.1 mg/dL, serum sodium = 141 mEq/L, serum potassium = 5.6 mEq/L, serum chloride = 101 mEq/L, serum bicarbonate = 14 mEq/L, serum calcium = 11.7 mg/dL, and serum phosphorus = 6.0 mg/dL. Which of the following is the most likely cause of this condition? A. Renovas cul ar di s eas e B. Thrombot i c renal di s eas e C. Mul t i pl e myel oma D. Sys t emi c l upus eryt hemat os us (SLE) E. Anal ges i c nephropat hy Vi ew Ans wer 11. T he answer is C [Part I: X P]. Mul t i pl e myel oma i s t he mos t l i kel y et i ol ogy. The combi nat i on of hypercal cemi a and acut e renal fai l ure rai s es t he pos s i bi l i t y of mul t i pl e myel oma as t he bone breakdown s econdary t o t umor i nvol vement rel eas es l arge amount s of cal ci um t o t he ext racel l ul ar fl ui d and hypercal cemi a ens ues . The renal fai l ure i n myel oma i s pri mari l y rel at ed t o hypercal cemi a
Pa g e 1 5 0 0
ABC Ambe r CHM Conve rte r Tria l ve rsion, http://w w w .proce sste x t.com/a bcchm.html
combi ned wi t h prot ei naceous cas t format i on wi t hi n t he renal t ubul es , produci ng a form of i nt rat ubul ar obs t ruct i on as wel l as a t ubul ar i nfl ammat ory l es i on. The major di agnos t i c cl ue i s t he fi ndi ng of a uri nary di ps t i ck t hat i s mi l dl y pos i t i ve for prot ei n i n t he uri ne but a s ul fos al i cyl i c aci d t es t t hat i s s t rongl y pos i t i ve. Di ps t i ck t es t i ng does not det ect t he negat i vel y charged l i ght -chai n prot ei ns , onl y t he al bumi n. The s ul fos al i cyl i c aci d t es t det ect s al l forms of prot ei ns . Renovas cul ar l es i ons and t hrombot i c renal di s eas e coul d pres ent wi t h t hi s pi ct ure, al t hough t hey s houl d not be as s oci at ed wi t h hypercal cemi a and s evere back pai n, and fi ndi ngs on exami nat i on of t he uri ne woul d not i ncl ude prot ei ns . Sys t emi c l upus eryt hemat os us (SLE) can, of cours e, be as s oci at ed wi t h s evere anemi a and joi nt mani fes t at i ons , but hypercal cemi a i s not part of t he pi ct ure. 12. A 21-year-old woman enters the hospital because of severe anemia and acute renal failure. T hree weeks prior to admission, she had delivered a normal, full-term infant, and had felt extremely weak and fatigued following delivery. Her physician notes that she is extremely pale. A complete blood count (CBC) reveals severe anemia as well as thrombocytopenia. Further laboratory screening reveals elevated blood urea nitrogen (BUN) and creatinine, and she is referred for admission. She denies any medication use postpartum and any previous medical problems similar to this one. Her laboratory studies were normal at the time of discharge from the hospital following delivery. On physical examination, vital signs are normal, and the only physical findings aside from her pallor are petechiae on the skin and multiple ecchymoses on the lower extremities. Laboratory studies reveal a hemoglobin of 6.3 mg/dL, a hematocrit of 18%, 3
a platelet count of 23,000/mm , BUN of 94 mg/dL, and creatinine of 9.1 mg/dL. T he serum antinucleophilic cytoplasmic antibody (ANCA) and serum antineutrophilic antibody (ANA) levels are normal. A peripheral blood smear reveals multiple schistocytes. Urinalysis reveals many red blood cell (RBC) casts. Which of the following is the most likely diagnosis? A. Hemol yt i c–uremi c s yndrome
Pa g e 1 5 0 1
ABC Ambe r CHM Conve rte r Tria l ve rsion, http://w w w .proce sste x t.com/a bcchm.html
B. Goodpas t ure's s yndrome C. Sys t emi c l upus eryt hemat os us (SLE) D. Idi opat hi c t hrombocyt openi c purpura E. A drug react i on Vi ew Ans wer 12. T he answer is A [Part I: XIV D 1 b]. The mos t l i kel y caus e of t hi s condi t i on i s hemol yt i c–uremi c s yndrome. Thi s s yndrome i s as s oci at ed wi t h pl at el et cons umpt i on as wel l as acut e renal i ns uffi ci ency. Thi s condi t i on may occur pos t part um and can be di agnos ed onl y by exami ni ng t he ent i re cl i ni cal pi ct ure. Pat i ent s wi t h s ys t emi c l upus eryt hemat os us (SLE) can pres ent wi t h t hrombocyt openi a and acut e renal fai l ure, but t he negat i ve ant i nucl ear ant i body (ANA) l evel i n t he abs ence of ot her s ys t emi c mani fes t at i ons makes t hi s ans wer unl i kel y. The fi ndi ng of s chi s t ocyt es on t he peri pheral s mear al s o s t rongl y s upport s t he di agnos i s of t he mi croangi opat hi c pi ct ure, whi ch expl ai ns t o a great er ext ent t he anemi a s een. The onl y drug-as s oci at ed caus e of renal i ns uffi ci ency t hat mani fes t s as s evere t hrombocyt openi a of t hi s degree as wel l as s evere mi croangi opat hi c hemol yt i c anemi a i s cycl os pori ne or peni ci l l ami ne t reat ment . Idi opat hi c t hrombocyt openi c purpura (ITT) i s not as s oci at ed wi t h concurrent renal di s eas e. The correct t herapy for t hi s condi t i on woul d be t o perform pl as ma exchange, pl as ma i nfus i ons , or bot h, as t he bes t way of l i mi t i ng t he i njury as s oci at ed wi t h hemol yt i c–uremi c s yndrome. The et i ol ogy of t hi s s yndrome remai ns uncl ear, al t hough i t can be as s oci at ed wi t h bact eri al i nfect i ons , part i cul arl y when i t occurs i n epi demi c form i n chi l dren. 13. A 69-year-old woman complains of easy fatigability, weight loss, and dizziness. Her medical history includes a remote history of tuberculosis. Physical examination reveals a blood pressure of 100/62 mm Hg and increased skin pigmentation. Laboratory studies reveal the following: serum sodium = 129 mEq/L, serum potassium = 6.1 mEq/L, serum chloride = 100 mEq/L, plasma -
bicarbonate concentration [HCO 3 ] = 21 mEq/L, glucose = 88 mg/dL, and blood urea nitrogen (BUN) = 30 mg/dL. Which of
Pa g e 1 5 0 2
ABC Ambe r CHM Conve rte r Tria l ve rsion, http://w w w .proce sste x t.com/a bcchm.html
the following diagnoses best fits these findings? A. Adrenal i ns uffi ci ency B. Chroni c renal fai l ure C. Adrenocort i cot ropi c hormone (ACTH)–s ecret i ng t umor D. Di s t al (t ype I) renal t ubul ar aci dos i s E. Syndrome of i nappropri at e s ecret i on of ant i di uret i c hormone (SIADH) Vi ew Ans wer 13. T he answer is A [Part II: I B]. The fi ndi ngs of l ow bl ood pres s ure, i ncreas ed s ki n pi gment at i on, hyperkal emi a, and hyponat remi a al l poi nt t o t he l i kel y di agnos i s of adrenal i ns uffi ci ency. Adrenocort i cot ropi c hormone (ACTH)–s ecret i ng t umors , whi l e t hey may produce s ki n pi gment at i ons , are us ual l y as s oci at ed wi t h hypert ens i on and hypokal emi c al kal os i s due t o exces s mi neral ocort i coi d s ecret i on. Hyperkal emi a due t o renal fai l ure i s s een when t he gl omerul ar fi l t rat i on rat e (GFR) i s bel ow 3
10–15 mm /mi n. The pat i ent 's bl ood urea ni t rogen (BUN) of 30 mg/dL s ugges t s onl y mi l dl y i mpai red renal funct i on. Di s t al (t ype I) renal t ubul ar aci dos i s i s charact eri zed by hypokal emi a (al t hough occas i onal cas es of hyperkal emi a have been des cri bed). 14. A 27-year-old man is referred for evaluation of hematuria. For the last 6 years, he has experienced painless gross hematuria two to three times per year. T he condition has always remitted spontaneously within 1–2 days, but on this occasion, bright-red blood has appeared in his urine for the last 8 days. H e denies any trauma to his kidneys or any other recent illnesses. Although he has a younger brother with sickle cell anemia, the man has been free from any symptoms of this abnormality. He has no other known family history or evidence of any other renal disease, including kidney stones and infection. P.306
Urinalysis reveals red-to-pink urine, with numerous red blood cells (RBCs) per high-power field, and no proteinuria on
Pa g e 1 5 0 3
ABC Ambe r CHM Conve rte r Tria l ve rsion, http://w w w .proce sste x t.com/a bcchm.html
orthotoluidine dipstick test. Which of the following is the most likely diagnosis at this point? A. Nephrol i t hi as i s B. Carci noma of t he ki dney C. Renal vei n t hrombos i s D. Pros t at i t i s E. Si ckl e cel l t rai t Vi ew Ans wer 14. T he answer is E [Part I: Tabl e 6-5; XIV F 2 b]. Si ckl e cel l t rai t , nephrol i t hi as i s , carci noma of t he ki dney, renal vei n t hrombos i s , and pros t at i t i s al l may be as s oci at ed wi t h hemat uri a. The fi ndi ng of gros s hemat uri a i n a young man wi t h a fami l y hi s t ory of s i ckl e cel l anemi a, however, s t rongl y s ugges t s t hat , i n t he abs ence of ot her s t i gmat a of s i ckl e cel l di s eas e, he s uffers from s i ckl e cel l t rai t . Thi s abnormal i t y commonl y i s as s oci at ed wi t h hemat uri a and i s caus ed by s i ckl i ng of red bl ood cel l s (RBCs ) i n t he ves s el s on t he s urface of t he renal pel vi s and papi l l a, l eadi ng t o s mal l areas of i nfarct i on and bl eedi ng. Renal t umor can produce bl eedi ng, but i t i s rarel y s o pers i s t ent i n t he abs ence of ot her s ympt oms . Renal vei n t hrombos i s can produce hemat uri a but us ual l y i n t he s et t i ng of s evere pai n and prot ei nuri a. Nephrol i t hi as i s al s o may produce rel at i vel y s i l ent hemat uri a i f t he s t one has been pres ent for a l ong peri od, but pai n t ypi cal l y i s as s oci at ed wi t h t hi s condi t i on. Uri nary t ract i nfect i on al s o can be as s oci at ed wi t h hemat uri a, but s ympt oms of i nfect i on (e.g., dys uri a, fever) s houl d be pres ent . 15. A 21-year-old man enters the hospital complaining that he has been passing dark reddish urine. He has recently recovered from a football-related knee injury incurred 3 months before, and the day before entering the hospital he engaged in vigorous physical activity for the first time since his knee injury. T his morning he awoke with sore, painful muscles and the aforementioned change in the character of his urine. Physical examination is essentially normal except for the painful muscles. Findings on urinalysis include red–brown color; pH of 5.0; specific gravity of 1.02; 3+ dipstick test for blood; and no
Pa g e 1 5 0 4
ABC Ambe r CHM Conve rte r Tria l ve rsion, http://w w w .proce sste x t.com/a bcchm.html
evidence of glucose, ketones, or bilirubin. Microscopic examination of the urine reveals occasional amorphous debris and three or four granular casts but no red blood cells (RBCs). Which of the following conditions most likely accounts for the urinalysis? A. Myogl obi nuri a B. Hemol yzed bl ood i n t he uri ne C. Inges t i on of foods t uffs cont ai ni ng red dye D. Uri nary t ract i nfect i on E. Renal t rauma Vi ew Ans wer 15. T he answer is A [Part I:
I A 1 b, 2 a, f; and II C 3 a (5).]
The fi ndi ng of dark reddi s h uri ne s ugges t s a number of underl yi ng condi t i ons . However, t he abs ence of red bl ood cel l s (RBCs ) on mi cros copi c exami nat i on of t he uri ne combi ned wi t h t he pos i t i ve di ps t i ck t es t for bl ood s t rongl y s ugges t s t he pos s i bi l i t y of myogl obi nuri a. Myogl obi n i s a pi gment t hat i s det ect ed by t he ort hot ol ui di ne reagent on t he di ps t i ck. Al t hough hemol yzed bl ood coul d be pres ent i n t he uri ne, a few RBCs s houl d be not ed, whereas none are s een i n t hi s pat i ent 's uri ne. Al t hough foods t uffs do cont ai n dyes t hat may col or t he uri ne, t hes e dyes do not produce pos i t i ve di ps t i ck t es t s for bl ood. Thi s cl i ni cal pi ct ure i s mos t cons i s t ent wi t h myogl obi nuri a caus ed by s udden, ext reme phys i cal exert i on by an i ndi vi dual who i s not wel l -condi t i oned. Such act i vi t y l eads t o mus cl e cel l breakdown and t he rel eas e of myogl obi n i nt o t he ci rcul at i on, wi t h i t s ul t i mat e fi l t rat i on by t he ki dney and appearance i n t he uri ne. 16. A 37-year-old man enters the hospital with a history of recent onset of hemoptysis and acute renal failure. Until 6 weeks ago, when he noted the onset of a cough which, over the ensuing 3 days, produced streaky, blood-tinged sputum, he had been in good health. His local physician ordered laboratory studies, which revealed a serum creatinine of 1.2 mg/dL. A chest radiograph taken at that time revealed bilateral fluffy infiltrates. T he patient was treated with antibiotics and followed
Pa g e 1 5 0 5
ABC Ambe r CHM Conve rte r Tria l ve rsion, http://w w w .proce sste x t.com/a bcchm.html
over the next 3 weeks; during that time, the lung picture worsened and the serum creatinine rose to 2.5 mg/dL. Now the man appears ill and in moderate distress. His blood pressure is 120/80 mm Hg, his pulse is 110 bpm, his respiratory rate is 22, and his temperature is 37.2°C. Examination of the chest shows bilateral rales and a few scattered wheezes, and examination of the heart reveals tachycardia but is otherwise normal. A chest radiograph reveals bilateral fluffy alveolar infiltrates. Laboratory examination shows a blood urea nitrogen (BUN) of 65 mg/dL, a creatinine of 4.3 mg/dL, and electrolytes within normal limits. T he hemoglobin level is 8.3 mg/dL and the hematocrit is 28%. Examination of the sputum reveals blood. Urinalysis reveals many red blood cell (RBC) casts. Serum antineutrophilic cytoplasmic antibody (ANCA) levels are negative, as are serum complement levels. T he serum antiglomerular basement membrane (anti-GBM) antibody titer is elevated to 1:64. Which of the following represents the most likely cause of this disorder? A. W egener's granul omat os i s B. Goodpas t ure's s yndrome C. Sys t emi c l upus eryt hemat os us (SLE) D. Mi cros copi c pol yart eri t i s nodos a E. Idi opat hi c cres cent i c gl omerul onephri t i s Vi ew Ans wer 16. T he answer is B [Part I: X G]. The mos t l i kel y et i ol ogy of t hi s di s order i s Goodpas t ure's s yndrome. W egener's granul omat os i s can pres ent wi t h a pul monary bl eedi ng s yndrome as s oci at ed wi t h acut e gl omerul onephri t i s , as s een i n t hi s pat i ent and as evi denced by t he hemopt ys i s as wel l as by t he act i ve uri nary s edi ment . However, t he negat i ve ant i neut rophi l i c cyt opl as mi c ant i body (ANCA) l evel and t he abs ence of a des t ruct i ve upper ai rway l es i on on phys i cal exami nat i on argue agai ns t W egener's granul omat os i s , and t he fi ndi ng of pos i t i ve ant i -gl omerul ar bas ement membrane (ant i -GBM) ant i bodi es s t rongl y poi nt s t o Goodpas t ure's s yndrome. Sys t emi c l upus eryt hemat os us (SLE) and mi cros copi c vas cul i t i s can pres ent
Pa g e 1 5 0 6
ABC Ambe r CHM Conve rte r Tria l ve rsion, http://w w w .proce sste x t.com/a bcchm.html
wi t h pul monary hemorrhage and renal i ns uffi ci ency, but nei t her i s as s oci at ed wi t h t he s erol ogi es s een i n t hi s pat i ent . Idi opat hi c cres cent i c gl omerul onephri t i s des cri bes an ent i t y s i mi l ar t o t hat s een i n t hi s pat i ent , but t he fi ndi ng of pos i t i ve s erum ant i -GBM ant i body t i t ers el i mi nat es gl omerul onephri t i s as an “i di opat hi c― ent i t y. 17. A 60-year-old man enters the hospital complaining of nausea, weakness, and confusion of 1 week' s duration. He has a long-standing history of hypertension and congestive heart failure (CHF) that has been treated with increasing amounts of diuretics and digoxin without apparent benefit. Physical examination indicates a blood pressure of 145/90 mm Hg (without orthostatic changes), jugular venous distention, bilateral basilar rales, and +2 bilateral ankle edema. Laboratory studies reveal the following: serum sodium = 120 mEq/L, blood urea nitrogen (BUN) = 93 mg/dL, glucose = 135 mg/dL, P.307 plasma osmolality = 252 mOsm/kg, and urine osmolality = 690 mOsm/kg. Which of the following is involved in the treatment of hyponatremia in this patient? A. 3% s odi um chl ori de i nfus i on B. 0.9% s odi um chl ori de i nfus i on C. 50 mg hydrochl orot hi azi de dai l y D. Sal t and wat er res t ri ct i on E. Demecl ocycl i ne Vi ew Ans wer 17. T he answer is D [Part II: I B 5]. W at er res t ri ct i on s houl d hel p produce a negat i ve wat er bal ance. Sal t res t ri ct i on i s al s o needed becaus e t he pat i ent has exces s t ot al s odi um burden; t hi s change s houl d al s o opt i mi ze t he res pons e t o di uret i cs . Hyponat remi a i n t he s et t i ng of an expanded ext racel l ul ar vol ume [e.g., conges t i ve heart fai l ure (CHF)] devel ops becaus e of di mi ni s hed renal abi l i t y t o excret e wat er i n t he face of cont i nui ng wat er i nges t i on. The renal di l ut i ng defect i s t he res ul t of i mpai red del i very of t ubul ar fl ui d t o
Pa g e 1 5 0 7
ABC Ambe r CHM Conve rte r Tria l ve rsion, http://w w w .proce sste x t.com/a bcchm.html
di s t al di l ut i ng nephron s egment s and i s preci pi t at ed by enhanced proxi mal t ubul ar reabs orpt i on. The l at t er i s caus ed by decreas ed renal perfus i on as a res ul t of i mpai red cardi ac out put . A decreas e i n effect i ve art eri al vol ume al s o s t i mul at es ant i di uret i c hormone (ADH) rel eas e, whi ch furt her i mpai rs uri nary di l ut i on. Ideal l y, t reat ment s houl d be di rect ed at i mprovi ng cardi ac funct i on. Di goxi n and di uret i cs have been admi ni s t ered wi t hout s i gni fi cant benefi t . The pat i ent cert ai nl y cannot t ol erat e any s al i ne i nfus i ons becaus e t hi s t reat ment woul d aggravat e hi s conges t i ve s ympt oms . Hydrochl orot hi azi de added t o hi s regi men may not be s uffi ci ent l y pot ent t o i mprove t he refract ory conges t i ve s ympt oms ; bes i des , hydrochl orot hi azi de i t s el f i mpai rs uri nary di l ut i on and may aggravat e t he hyponat remi a. Demecl ocycl i ne for t he t reat ment of hyponat remi a i s res erved for t he chroni c management of t he s yndrome of i nappropri at e ant i di uret i c hormone (SIADH). Becaus e of i t s pot ent i al nephrot oxi ci t y, demecl ocycl i ne i s cont rai ndi cat ed i n al l s t at es of az ot emi a, as i n t hi s cas e. 18. A 27-year-old woman presents with a 6-month history of polyarthritis, facial rash, and proteinuria. Urinalysis shows red blood cell casts and white blood cells. Which of the following findings are likely with further testing? A. Nodul ar gl omerul os cl eros i s B. Pos i t i ve fl uores cent ant i nucl ear ant i body (ANA) C. Gl omerul ar capi l l ary endot hel i os i s D. Necrot i zi ng granul omat ous vas cul i t i s E. Pos i t i ve Congo red s t ai ni ng for amyl oi d Vi ew Ans wer 18. T he answer is B [Part I: X M 1 a (2) (b)]. The pat i ent has s ys t emi c l upus eryt hemat os us (SLE). The gl omerul ar abnormal i t i es of SLE form a di s eas e s pect rum, wi t h four as s oci at ed major l es i ons : focal prol i ferat i ve, di ffus e prol i ferat i ve, membranous , and mes angi al forms of l upus nephri t i s . Serol ogi c s t udi es of t he four l es i ons det ermi ne di fferi ng l evel s of ant i -DNA ant i bodi es and compl ement component s , but common t o al l four are i mmunofl uores cent fi ndi ngs for ant i nucl ear ant i body (ANA). Nodul ar
Pa g e 1 5 0 8
ABC Ambe r CHM Conve rte r Tria l ve rsion, http://w w w .proce sste x t.com/a bcchm.html
gl omerul os cl eros i s i s a pat hol ogi c l es i on charact eri s t i c of di abet i c nephropat hy. Gl omerul ar capi l l ary endot hel i os i s i s t he pat hol ogi c fi ndi ng i n t he ki dney of pat i ent s wi t h preecl amps i a. Necrot i zi ng granul omat ous vas cul i t i s i s charact eri s t i c of W egener's granul omat os i s but may be s een i n ot her vas cul i t i c condi t i ons as wel l . Congo red s t ai ni ng for amyl oi d i s s een i n pri mary and s econdary forms of amyl oi dos i s , whi ch can caus e t he nephrot i c s yndrome i f t here i s ki dney i nvol vement . 19. A 57-year-old patient with a 13-year history of poorly controlled diabetes mellitus has developed a slow rise in serum creatinine and proteinuria over the last 18 months. Which of the following pathologic or laboratory findings is likely in this setting? A. Nodul ar gl omerul os cl eros i s B. Pos i t i ve fl uores cent ant i nucl ear ant i body (ANA) C. Gl omerul ar capi l l ary endot hel i os i s D. Necrot i zi ng granul omat ous vas cul i t i s E. Pos i t i ve Congo red s t ai ni ng for amyl oi d Vi ew Ans wer 19. T he answer is A [Part I: X L 3 b]. Al l l es i ons i n t he ki dneys of i ndi vi dual s wi t h di abet es mel l i t us are grouped under t he t erm di abet i c nephropat hy. The foremos t cl i ni cal feat ure of di abet i c gl omerul ar di s eas e i s overt prot ei nuri a, whi ch devel ops aft er a prol onged cours e of di abet es mel l i t us . Al t hough di ffus e gl omerul os cl eros i s i s t he mos t common l es i on (found i n 90% of al l di abet i c i ndi vi dual s wi t h overt nephropat hy) and i s correl at ed wi t h t he degree of prot ei nuri a and t he renal fai l ure, i t i s nei t her s peci fi c for nor di agnos t i c of di abet es . In cont ras t , t he ot her major renal l es i on as s oci at ed wi t h di abet es , nodul ar gl omerul os cl eros i s (Ki mmel s t i el -W i l s on s yndrome), i s bot h s peci fi c for and di agnos t i c of di abet i c gl omerul opat hy. A pos i t i ve ANA i s a t ypi cal feat ure of s ys t emi c l upus eryt hemat os us . Gl omerul ar capi l l ary endot hel i os i s i s t he pat hol ogi c fi ndi ng i n t he ki dney of pat i ent s wi t h preecl amps i a. Necrot i z i ng granul omat ous vas cul i t i s i s charact eri s t i c of W egener's granul omat os i s or ot her vas cul i t i c condi t i ons . Congo red s t ai ni ng
Pa g e 1 5 0 9
ABC Ambe r CHM Conve rte r Tria l ve rsion, http://w w w .proce sste x t.com/a bcchm.html
for amyl oi d i s s een i n pri mary and s econdary forms of amyl oi dos i s , whi ch can t ypi cal l y caus e t he nephrot i c s yndrome. 20. A 17-year-old woman in the third trimester of her first pregnancy has a blood pressure of 145/95 mm Hg and 2+ proteinuria. Further testing is most likely to reveal which of the following laboratory or pathologic findings? A. Nodul ar gl omerul os cl eros i s B. Pos i t i ve fl uores cent ant i nucl ear ant i body (ANA) C. Gl omerul ar capi l l ary endot hel i os i s D. Necrot i zi ng granul omat ous vas cul i t i s E. Pos i t i ve Congo red s t ai ni ng for amyl oi d Vi ew Ans wer 20. T he answer is C [
onl i ne Part I: XV E]. Preecl amps i a i s t he
t oxemi a of pregnancy; t hi s s yndrome i ncl udes hypert ens i on, edema, and prot ei nuri a. Ecl amps i a i ncl udes t he preecl ampt i c s pect rum of s ympt oms combi ned wi t h as s oci at ed mat ernal convul s i ons and coma. The pri mary pat hol ogi c renal feat ure i s gl omerul ar capi l l ary endot hel i os i s . A pos i t i ve ANA i s a t ypi cal feat ure of s ys t emi c l upus eryt hemat os us . Necrot i zi ng granul omat ous vas cul i t i s i s charact eri s t i c of W egener's granul omat os i s or ot her vas cul i t i c condi t i ons . Congo red s t ai ni ng for amyl oi d i s s een i n pri mary and s econdary forms of amyl oi dos i s , whi ch can caus e t he nephrot i c s yndrome i f t here i s ki dney i nvol vement . 21. A 34-year-old man who is a heavy smoker develops hematuria with red blood cell casts and 2+ proteinuria along with a chest x-ray that shows a nodular, cavitating lesion in the right middle lobe. He also has a necrotic-appearing lesion in his nares. Further testing is most likely to reveal which of the following laboratory or pathologic findings? A. Nodul ar gl omerul os cl eros i s B. Pos i t i ve fl uores cent ant i nucl ear ant i body (ANA) C. Gl omerul ar capi l l ary endot hel i os i s D. Necrot i zi ng granul omat ous vas cul i t i s E. Pos i t i ve Congo red s t ai ni ng for amyl oi d Vi ew Ans wer
Pa g e 1 5 1 0
ABC Ambe r CHM Conve rte r Tria l ve rsion, http://w w w .proce sste x t.com/a bcchm.html
21. T he answer is D [Part I: X N 3]. Necrot i zi ng granul omat ous vas cul i t i s (W egener's granul omat os i s ) i s charact eri zed by upper or l ower res pi rat ory t ract s ympt oms of necrot i zi ng vas cul i t i s and granul omat ous i nfl ammat i on. The ki dneys are i nvol ved i n approxi mat el y 50% of pat i ent s . Indi vi dual l y, t he vas cul i t i s and granul omat ous i nfl ammat i on are not s peci fi c for W egener's granul omat os i s ; bot h mus t be hi s t ol ogi cal l y evi dent for di agnos i s . The mos t recogni zabl e pres ent at i on of t he condi t i on i s t he occurrence of necrot i zi ng l es i ons . A pos i t i ve ANA i s a t ypi cal feat ure of s ys t emi c l upus eryt hemat os us . Gl omerul ar capi l l ary endot hel i os i s i s t he pat hol ogi c fi ndi ng i n t he ki dney of pat i ent s wi t h preecl amps i a. Congo red s t ai ni ng for amyl oi d i s s een i n pri mary and s econdary forms of amyl oi dos i s , whi ch can caus e t he nephrot i c s yndrome i f t here i s ki dney i nvol vement . 22. A 68-year-old man was diagnosed with multiple myeloma 1 year ago. Over the last 3 months he has developed severe lower extremity edema. Urinalysis shows 4+ proteinuria but no other abnormalities. T he serum creatinine is 1.7 mg/dL. Further testing is most likely to reveal which of the following laboratory or pathologic findings? A. Nodul ar gl omerul os cl eros i s B. Pos i t i ve fl uores cent ant i nucl ear ant i body (ANA) C. Gl omerul ar capi l l ary endot hel i os i s D. Necrot i zi ng granul omat ous vas cul i t i s E. Pos i t i ve Congo red s t ai ni ng for amyl oi d Vi ew Ans wer 22. T he answer is E [Part I: X P 2; onl i ne Tabl e 6-12]. One of t he mai n as s oci at i ons of mul t i pl e myel oma i s t he devel opment of pri mary amyl oi dos i s wi t h l i ght -chai n res t ri ct i on. Congo red s t ai ni ng i s rat her s peci fi c for i dent i fyi ng amyl oi dos i s . Pri mary amyl oi dos i s (AL) or s econdary amyl oi dos i s (AA) forms oft en affect t he ki dneys and t ypi cal l y caus e t he nephrot i c s yndrome wi t h progres s i ve renal i ns uffi ci ency. A pos i t i ve ANA i s a t ypi cal feat ure of s ys t emi c l upus eryt hemat os us . Gl omerul ar capi l l ary endot hel i os i s i s t he pat hol ogi c fi ndi ng i n t he ki dney of pat i ent s wi t h preecl amps i a. Necrot i zi ng
Pa g e 1 5 1 1
ABC Ambe r CHM Conve rte r Tria l ve rsion, http://w w w .proce sste x t.com/a bcchm.html
granul omat ous vas cul i t i s i s charact eri s t i c of W egener's granul omat os i s or ot her vas cul i t i c condi t i ons .
Pa g e 1 5 1 2
ABC Ambe r CHM Conve rte r Tria l ve rsion, http://w w w .proce sste x t.com/a bcchm.html
Editors: Wolfsthal, Susan T itle: NMS Medicine, 6th Edition Copyri ght ©2008 Li ppi ncot t W i l l i ams & W i l ki ns > T able of Cont ent s > Chapt er 7 - Allergic and Immunologic Disorders
Chapter 7 Allergic and Immunologic Disorders Mary E Bollinger Jefferson Lee
I. Overview of The Immune System Immune res pons es are generat ed by nat ural and adapt i ve mechani s ms t hat cons i s t of bot h cel l ul ar and humoral component s .
A. Natural immunity Natural immunity i s nons peci fi c; t hat i s , i t i s not i nfl uenced by previ ous ant i gen–ant i body i nt eract i ons .
1. Phagocytic cells, including polymorphonuclear leukocytes (PMNLs) [neutrophils and eosinophils], monocytes, and macrophages, form t he cel l ul ar component of t he nat ural i mmune res pons e. Thes e cel l s i nges t and des t roy pat hogeni c organi s ms and ot her forei gn mat eri al .
2. Natural killer (NK) cells are pot ent cyt ot oxi c cel l s whos e t arget s are not ant i gen-s peci fi c.
3. The complement system, a compl ex group of at l eas t 15 s erum prot ei ns , medi at es i nfl ammat ory react i ons by at t ract i ng granul ocyt es and macrophages , promot i ng
Pa g e 1 5 1 3
ABC Ambe r CHM Conve rte r Tria l ve rsion, http://w w w .proce sste x t.com/a bcchm.html
cel l –cel l i nt eract i ons neces s ary for ant i gen proces s i ng, and s t i mul at i ng t he l ys i s of envel oped vi rus es and bact eri a.
B. Adaptive immunity Adaptive immunity i s charact eri zed by di rect res pons es t o i ni t i al ant i gen pres ent at i on and memory (anamnestic) res pons es on re-expos ure.
1. Humoral immunity i nvol ves t he product i on of immunoglobulins by mature B lymphocytes. Ant i gen bi ndi ng t o membrane-bound i mmunogl obul i n t ri ggers t he di fferent i at i on of t hes e B cel l s i nt o mat ure pl as ma cel l s capabl e of s ecret i ng ant i gen-s peci fi c i mmunogl obul i n. o
o
a. Immunogl obul i ns are vi t al t o t he i mmune res pons e i n t hree pri mary ways .
(1) Immunogl obul i ns coat i nvadi ng organi s ms , t hereby i mpedi ng t hei r acces s t o t he hos t .
(2) Immunogl obul i n M (IgM) and IgG fi x compl ement (s ee I A 3).
o
o
(3) Immunogl obul i ns promot e ops oni zat i on, whi ch i ncreas es t he effi ci ency of phagocyt os i s .
b. Immunogl obul i ns are di vi ded i nt o fi ve cl as s es , or isotypes. o
o
(1) IgG, t he mos t preval ent s erum i mmunogl obul i n
Pa g e 1 5 1 4
ABC Ambe r CHM Conve rte r Tria l ve rsion, http://w w w .proce sste x t.com/a bcchm.html
i s ant i gen s peci fi c and has memory (anamnes t i c res pons e), meani ng t hat l evel s wi l l i ncreas e more rapi dl y wi t h s ubs equent ant i gen expos ure. Speci fi c IgG l evel s i ncreas e aft er expos ure t o an ant i gen as a res ul t of active i mmune res pons e (e.g., aft er expos ure t o a vi rus ) and as a res ul t of passive i mmune res pons e (e.g., aft er expos ure t o a vacci ne). In addi t i on t o memory, IgG i s ant i gen s peci fi c; for exampl e, mos t i ndi vi dual s wi l l onl y devel op vari cel l a once due t o devel opment of vari cel l a ant i body, whi ch i s s peci fi c and general l y l i fel ong. W i t h l at er expos ure, IgG l evel s wi l l i ncreas e s peci fi cal l y i n res pons e. Four IgG s ubcl as s es —IgG1, IgG2, IgG3, and IgG4—have been i dent i fi ed. o
o
(2) IgA i s t he pri nci pal i mmunogl obul i n i n s ecret ory fl ui ds , maki ng i t an i mport ant fi rs t -l i ne hos t defens e ant i body at s i t es of ant i gen ent ry. IgA exi s t s as a di mer l i nked t o an as s oci at ed s ecret ory component by a J chai n.
o
o
(3) IgM exi s t s i n s erum as a cl us t er of fi ve monomeri c uni t s l i nked by a J chai n and di s ul fi de bonds . It i s t he i ni t i al cl as s of ant i body produced as a pri mary res pons e t o ant i gens , wi t h l evel s i ncreas i ng wi t hi n XX of expos ure. IgM di ffers from IgG i n t hat i t does not mount an anamnes t i c res pons e and i s not ant i gen s peci fi c.
o
o
(4) IgD and IgE are monomeri c prot ei ns pres ent i n s erum i n t race amount s . IgE i s t he ant i body res pons i bl e for t ri ggeri ng t he al l ergi c react i on.
Pa g e 1 5 1 5
ABC Ambe r CHM Conve rte r Tria l ve rsion, http://w w w .proce sste x t.com/a bcchm.html
2. Cellular immunity i s medi at ed by T lymphocytes. o
o
a. Subpopulations of T cells have been defi ned on t he bas i s of funct i on and t he pres ence of charact eri s t i c s urface ant i gens .
(1) Cytotoxic T (T c) cells expres s t he CD8 s urface marker and are res pons i bl e for ki l l i ng cel l s t hat expres s forei gn ant i gens .
(2) Helper T (T h) cells enhance t he act i vi t y of B cel l s , macrophages , and ot her T cel l s . They expres s t he s urface marker CD4.
(3) Suppressor T (T s) cells are al s o CD8 pos i t i ve and i nhi bi t t he act i vi t y of ot her cel l s of t he i mmune s ys t em.
o
o
b. Vari ous t ypes of antigen-presenting cells interact with T cells, s t i mul at i ng t he rel eas e of lymphokines, i ncl udi ng i nt erl euki ns , granul ocyt e–macrophage col ony-s t i mul at i ng fact or (GM-CSF), t umor necros i s fact or (TNF), i nt erferon-γ (IFN-γ), and ot her fact ors . Thes e s ubs t ances regul at e vari ous as pect s of t he i mmune res pons e.
II. Pathogenesis of The IgE-Mediated Allergic Reaction Ant i gen-s t i mul at ed rel eas e of medi at ors by cros s -l i nki ng IgE
Pa g e 1 5 1 6
ABC Ambe r CHM Conve rte r Tria l ve rsion, http://w w w .proce sste x t.com/a bcchm.html
mol ecul es ont o IgE-s ens i t i zed mas t cel l s i s an IgE-mediated hypersensitivity reaction. Thes e react i ons are t he caus e of many al l ergi c di s orders , i ncl udi ng al l ergi c rhi ni t i s , urt i cari a, general i zed anaphyl axi s , i ns ect s t i ng s ens i t i vi t y, and s ome drug react i ons (s ee III–VII). Thes e react i ons al l requi re pri or expos ure t o t he ant i gen and s omet i mes s everal expos ures are needed for t he al l ergi c res pons e t o occur.
A. The IgE-mediated allergic response T he IgE-mediated allergic response can be di vi ded i nt o an earl y phas e and a l at e phas e. The earl y phas e i s domi nat ed by IgE, whereas t he l at e phas e i s domi nat ed by hel per T cel l s , whi ch t ake t i me t o mani fes t t hems el ves .
B. Initial exposure to antigen Initial exposure to antigen s t i mul at es t he product i on of s peci fi c IgE mol ecul es , whi ch bi nd t o hi gh-affi ni t y Fc recept ors on t he s urface of mas t cel l s . On re-expos ure, ant i gen cros s -l i nki ng of t hes e membrane-bound IgE mol ecul es res ul t s i n t he rel eas e of i nfl ammat ory medi at ors from t he mas t cel l and t he recrui t ment of hel per T cel l s . Exampl es of i nfl ammat ory medi at ors i ncl ude hi s t ami ne and pros t agl andi ns .
III. Allergic Rhinitis A. Definition Thi s i nfl ammat ory di s order of t he nas al mucos a i s charact eri zed by nas al bl ockage, rhi norrhea, s neezi ng, and pruri t us . It i s i ni t i at ed by mas t -cel l medi at ors rel eas ed duri ng t he IgE-medi at ed hypers ens i t i vi t y react i on. There are t wo major cl as s i fi cat i ons of al l ergi c rhi ni t i s .
1. Seasonal allergic rhinitis has peri odi c s ympt oms t hat occur onl y duri ng t he pol l i nat i ng s eas on of t he ant i gen t o whi ch t he pat i ent i s s ens i t i ve.
Pa g e 1 5 1 7
ABC Ambe r CHM Conve rte r Tria l ve rsion, http://w w w .proce sste x t.com/a bcchm.html
2. Perennial allergic rhinitis has cont i nuous or i nt ermi t t ent s ympt oms t hat occur year round.
B. Incidence Al t hough accurat e es t i mat es are di ffi cul t t o obt ai n, i t i s bel i eved t hat 10%–30% of adul t s and up t o 40% of chi l dren have al l ergi c rhi ni t i s , maki ng t hi s t he mos t common chroni c di s order of t he res pi rat ory t ract . There i s no apparent mal e or femal e predi l ect i on for al l ergi c rhi ni t i s , and no et hni c or raci al pat t erns have been i dent i fi ed. Sympt oms may begi n at any age but devel op i n mos t pat i ent s before age 20 years . A hi s t ory of al l ergi c di s orders i n t he i mmedi at e fami l y i s common.
C. Etiology Vari ous aeroal l ergens (e.g., s pores , pol l ens , and ani mal al l ergens ) t ri gger t he IgE-medi at ed hypers ens i t i vi t y react i on t hat underl i es al l ergi c rhi ni t i s .
1. Specific seasonal allergens have t ypi cal cal endar pat t erns , t hough t hi s can vary dependi ng on a l ocat i on's al t i t ude, l at i t ude, and predomi nant veget at i on. For exampl e, i n t he Nort heas t : o
o
a. T ree pollen s eas on runs from February t o May.
o
o
b. Grass pollen s eas on i s heavi es t i n May and June. P.314
o
o
c. Ragweed pollen s eas on runs from mi d-Augus t t i l l t he fi rs t fros t .
Pa g e 1 5 1 8
ABC Ambe r CHM Conve rte r Tria l ve rsion, http://w w w .proce sste x t.com/a bcchm.html o
o
d. Mold spores are wors t i n t i mes of damp weat her, es peci al l y wi t h wet l eaves on t he ground i n t he aut umn.
2. Specific perennial allergens o
o
a. Dust mites l i ve pri mari l y i n beddi ng and feed on des quamat ed s ki n. They produce al l ergeni c feces , whi ch can be a s i gni fi cant caus e of chroni c s ympt oms .
o
o
b. Epidermal antigens are produced pri mari l y by furry pet s s uch as cat s , dogs , and rabbi t s .
o
o
c. Indoor molds (i .e., s pores found i n homes ) i ncl ude As pergi l l us and Peni c i l l i um.
o
o
d. Other allergen sources i ncl ude cockroaches and mi ce.
o
o
e. Nonspecific irritants, whi ch are not t rue al l ergens , i ncl ude ci garet t e s moke, ai r pol l ut i on, perfumes , cooki ng odors , and chemi cal fumes .
D. Clinical features
1. Characteristic symptoms o
Pa g e 1 5 1 9
ABC Ambe r CHM Conve rte r Tria l ve rsion, http://w w w .proce sste x t.com/a bcchm.html
o
a. Sneezing—oft en wi t h paroxys ms of 15–20 s neezes i n qui ck s ucces s i on—i s charact eri s t i c and l i kel y t o occur i n t he earl y morni ng hours , but can vary t hroughout t he day and ni ght dependi ng on t he t ri gger.
o
o
b. Pruritus of t he nos e, pal at e, and pharynx i s common and may l ead t o t he “al l ergi c s al ut e s i gn,― from repeat ed pus hi ng up on t he end of t he nos e), caus i ng a t rans vers e nas al creas e.
o
o
c. A thin, watery nasal discharge us ual l y i s pres ent and i s as s oci at ed wi t h varyi ng amount s of nas al obs t ruct i on and pos t nas al drai nage. Mout h breat hi ng i s common due t o nas al obs t ruct i on.
o
o
d. Excess lacrimation as wel l as ocul ar pruri t us and s orenes s are common.
o
o
e. Loss of olfaction and taste may res ul t from chroni c s evere nas al conges t i on.
o
o
f. Otitis media, res ul t i ng from i mpai red drai nage of t he eus t achi an t ube, and sinusitis, res ul t i ng from i mpai red drai nage of t he paranas al s i nus es , s omet i mes occur.
2. Characteristic physical findings us ual l y are s een at t he t i me of maxi mal expos ure t o t he offendi ng ant i gens (e.g., duri ng t he hei ght of t he pol l en s eas on).
Pa g e 1 5 2 0
ABC Ambe r CHM Conve rte r Tria l ve rsion, http://w w w .proce sste x t.com/a bcchm.html o
o
a. Nasal findings
(1) The nas al cavi t y charact eri s t i cal l y cont ai ns t hi n nas al s ecret i ons , and t he mucos al s urface i s edemat ous , boggy, and us ual l y pal e or bl ui s h.
(2) Nas al pol yps may be s een but are not common i n al l ergi c rhi ni t i s . Thei r pres ence us ual l y s ugges t s cys t i c fi bros i s i n chi l dren and as pi ri n i nt ol erance i n adul t s and chi l dren.
(3) Evi dence of recent epi s t axi s i n t he ant eri or nas al vaul t may be s een i f a pat i ent 's nas al conges t i on has l ed t o frequent nos e-rubbi ng.
o
o
b. Conjunctival findings i ncl ude i nject i on and s wel l i ng, exces s l acri mat i on, granul ari t y, and, occas i onal l y, chemos i s . Infraorbi t al “s hi ners ― (i nfraorbi t al venous conges t i on) may devel op.
o
o
c. Oropharyngeal findings include mucus streaming down the posterior pharynx and cobblestoning of the lymphoid tissue of the same area.
o
o
d. Possible findings in children who have severe chronic nasal congestion i ncl ude broadened bony
Pa g e 1 5 2 1
ABC Ambe r CHM Conve rte r Tria l ve rsion, http://w w w .proce sste x t.com/a bcchm.html
dors um of t he nos e, narrowed pal at al arch, hal i t os i s , ret rognat hi c faci es , exces s gi ngi val and pharyngeal l ymphoi d t i s s ue, and dent al abnormal i t i es .
E. Evaluation
1. Diagnostic tests o
o
a. Accurat el y appl i ed skin tests wi t h pot ent and s peci fi c ant i gens are t he bes t di agnos t i c procedures t o i dent i fy t he ant i gens caus i ng IgE-medi at ed al l ergi c rhi ni t i s .
o
o
b. The radioallergosorbent test (RAST ) i s an al t ernat i ve t o s ki n t es t i ng and can be ordered by any pract i t i oner. It s s ens i t i vi t y and s peci fi ci t y have i mproved dramat i cal l y, and for many al l ergens i t i s as accurat e as s ki n t es t s . It i s rout i nel y us ed for pat i ent s wi t h ext ens i ve ecz emat oi d dermat i t i s , s i gni fi cant dermat ographi s m, or a compl i cat ed hi s t ory wi t hout corroborat i ve s ki n t es t s . IgG-bl ocki ng ant i bodi es and hi gh t ot al IgE l evel s may i nt erfere wi t h t he RAST.
o
o
c. An elevated IgE level i s pres ent i n onl y 30%–40% of pat i ent s wi t h al l ergi c rhi ni t i s . Cl i ni ci ans al s o mus t cons i der ot her pos s i bl e caus es of el evat ed IgE l evel s s uch as i mmunodefi ci ency di s orders (s ee VIII). Tot al s erum IgE s houl d not be rout i nel y us ed i n t he di agnos i s of al l ergi c rhi ni t i s . P.315
Pa g e 1 5 2 2
ABC Ambe r CHM Conve rte r Tria l ve rsion, http://w w w .proce sste x t.com/a bcchm.html
o
o
d. Peripheral eosinophilia may be s een but i s an inconsistent fi ndi ng i n pat i ent s wi t h al l ergi c rhi ni t i s .
o
o
e. A stained smear of nasal secretions at t he t i me of cl i ni cal l y act i ve di s eas e oft en s hows a hi gher number of eos i nophi l s , but t hi s fi ndi ng al s o may be s een i n pat i ent s wi t h eos i nophi l i c nonal l ergi c rhi ni t i s (s ee III E 2 c) or hyperpl as t i c s i nus i t i s , as wel l as i n normal i nfant s younger t han 6 mont hs of age.
2. Differential diagnosis for nonallergic causes of rhinitis o
o
a. Nas al conges t i on wi t h mi l d rhi norrhea s omet i mes i s s een i n pregnant women and i n pat i ent s wi t h hypothyroidism.
o
o
b. Vasomotor rhinitis i s a s yndrome charact eri zed by nas al bl ockage and rhi norrhea wi t hout evi dence of i mmunol ogi c or i nfect i ous nas al di s eas e. Res pons e t o medi cal t herapy oft en i s poor.
o
o
c. Sympt oms of eosinophilic nonallergic rhinitis are s i mi l ar t o t hos e of vas omot or rhi ni t i s ; however, i n eos i nophi l i c nonal l ergi c rhi ni t i s , nas al eos i nophi l i a i s pres ent , nas al pol yps are common,
Pa g e 1 5 2 3
ABC Ambe r CHM Conve rte r Tria l ve rsion, http://w w w .proce sste x t.com/a bcchm.html
and s ympt oms us ual l y res pond t o medi cal management . o
o
e. Infectious rhinitis oft en occurs i n pat i ent s wi t h underl yi ng al l ergi c rhi ni t i s and us ual l y i s as s oci at ed wi t h reddened nas al mucos a, t hi ck nas al s ecret i ons , s ore t hroat , cervi cal adenopat hy, and l ow-grade fever.
o
o
f. A s ympt om compl ex known as t he aspirin triad cons i s t s of nas al pol yps , chroni c s i nus di s eas e, and pot ent i al l y l i fe-t hreat eni ng as t hma. Sympt oms are exacerbat ed aft er admi ni s t rat i on of as pi ri n or ot her nons t eroi dal ant i -i nfl ammat ory drugs (NSAIDs ).
o
o
g. Anatomic abnormalities, t he mos t common of whi ch i s a part i al l y devi at ed s ept um, may caus e nas al obs t ruct i on. W hen uni l at eral di s charge wi t h a foul odor i s pres ent , obs t ruct i on by a forei gn body s houl d be s us pect ed.
o
o
h. Atrophic rhinitis i s s een i n ol der pat i ent s and i s oft en s econdary t o t rauma, s peci fi c i nfect i on, granul oma, or s urgery.
F. Therapy Management i s s t epwi s e and i nvol ves avoi dance of any offendi ng al l ergens and provocat i ve s ubs t ances , admi ni s t rat i on of s el ect i ve pharmacol ogi c agent s , and, i n s ome i ns t ances , i ni t i at i on of i mmunot herapy.
1. Avoidance. A fi rs t -l i ne t herapy. Thi s as pect of
Pa g e 1 5 2 4
ABC Ambe r CHM Conve rte r Tria l ve rsion, http://w w w .proce sste x t.com/a bcchm.html
management i s mos t s ucces s ful when a s i ngl e ant i gen (e.g., ani mal dander) i s res pons i bl e for s ympt oms . In pat i ent s wi t h mul t i pl e s ens i t i vi t i es , s ens i bl e envi ronment al precaut i ons are recommended and appear t o l i mi t s evere exacerbat i on of s ympt oms .
2. Pharmacologic agents o
o
a. H 1 -receptor antagonists (i .e., antihistamines) are us eful i n cont rol l i ng rhi norrhea and pruri t us but have l i t t l e cl i ni cal effect on nas al conges t i on. Sedat i on wi t h i mpai red s chool or job performance i s an undes i rabl e s i de effect of fi rs t -generat i on H 1 -recept or ant agoni s t s . Becaus e s econd-generat i on ant i hi s t ami nes are not as s oci at ed wi t h t hes e s i de effect s , t hey are us ual l y cons i dered fi rs t as as t he i ni t i al pharmacol ogi cal t reat ment for al l ergi c rhi ni t i s .
(1) First-generation H 1 -receptor antagonists
(a) Et hanol ami nes (e.g., di phenhydrami ne) are effect i ve but oft en produce s i gni fi cant s edat i on and at ropi ne-l i ke s i de effect s .
(b) Et hyl enedi ami nes (e.g., pyri l ami ne) al s o are effect i ve, produce l es s s edat i on t han et hanol ami nes , and have mi ni mal gas t roi nt es t i nal s i de effect s .
Pa g e 1 5 2 5
ABC Ambe r CHM Conve rte r Tria l ve rsion, http://w w w .proce sste x t.com/a bcchm.html
(c) Al kyl ami nes (e.g., chl orpheni rami ne) are effect i ve and produce mi ni mal t o modes t s edat i on.
(2) Second-generation H 1 -receptor antagonists
(a) Lorat adi ne, des l orat adi ne, cet i ri zi ne, and fexofenadi ne are admi ni s t ered once dai l y; t hey have not been as s oci at ed wi t h advers e cardi ovas cul ar effect s .
(b) Al l s econd-generat i on H 1 recept or ant agoni s t s l ack ant i chol i nergi c s i de effect s but may produce mi l d s edat i on i f gi ven at hi gher-t han-us ual dos es . Cet i ri zi ne wi l l produce s edat i on i n s ome pat i ent s at recommended dos ages .
(3) Intranasal H 1 -receptor antagonists. Azel as t i ne hydrochl ori de, whi ch i s approved for us e i n t he Uni t ed St at es , has an effi cacy equal t o t hat of oral H 1 - and H 2 -recept or ant agoni s t s . It al s o may reduce nas al bl ockage. Twent y percent of pat i ent s percei ve a bi t t er t as t e.
P.316
b. Adrenergic agonists (al s o known as
Pa g e 1 5 2 6
ABC Ambe r CHM Conve rte r Tria l ve rsion, http://w w w .proce sste x t.com/a bcchm.html
sympathomimetic medications) o
o
(1) Oral administration of α-adrenergi c agent (e.g., ps eudoephedri ne) i s effect i ve i n reduci ng nas al conges t i on but not rhi norrhea. Cent ral nervous s ys t em (CNS) s t i mul at i on may occur, as wel l as hypert ens i on and l os s of appet i t e. Short -t erm us e i s preferabl e (< 10 days ).
o
o
(2) T opical application of s hort -act i ng phenyl ephri ne or l ong-act i ng oxymet azol i ne decreas es nas al conges t i on, but regul ar admi ni s t rat i on for more t han 3–4 days res ul t s i n s evere rebound nas al conges t i on (rhi ni t i s medi cament os a).
c. Cromolyn sodium. A 4% s ol ut i on of cromol yn s odi um i s avai l abl e for t opi cal nas al us e. It mus t be a dmi ni s t ered frequent l y (3–6 t i mes dai l y) but i s effect i ve i n t reat i ng bot h s eas onal and perenni al al l ergi c rhi ni t i s . Becaus e s i de effect s are mi ni mal , cromol yn s odi um may at t i mes be preferred t o t opi cal nas al s t eroi ds , but i t i s l es s effect i ve. Pret reat ment before al l ergen expos ure may s i gni fi cant l y reduce nas al al l ergi c res pons e.
d. Corticosteroids o
o
(1) T opical agents. Hi ghl y effect i ve and rapi dl y met abol i zed t opi cal s t eroi d preparat i ons (e.g., momet as one furoat e monohydrat e, budes oni de, fl uni s ol i de acet at e, t ri amci nol one acet oni de, and
Pa g e 1 5 2 7
ABC Ambe r CHM Conve rte r Tria l ve rsion, http://w w w .proce sste x t.com/a bcchm.html
fl ut i cas one propi onat e) are benefi ci al i n t he t reat ment of s eas onal and perenni al al l ergi c rhi ni t i s . Thi s cl as s of medi cat i ons i s cons i dered t he t reat ment of choi ce. Thes e preparat i ons have mi ni mal s ys t emi c s i de effect s ; i rri t at i on and bl eedi ng rarel y occur, and Candi da overgrowt h i s very uncommon. o
o
(2) Systemic agents. Sys t emi c cort i cos t eroi ds s i gni fi cant l y i mprove s ympt oms duri ng t he hei ght of a s peci fi c pol l en s eas on. Thi s t reat ment i s rarel y i ndi cat ed, however, and us e of l ong-t erm oral cort i cos t eroi d t herapy for perenni al al l ergi c rhi ni t i s i s i l l -advi s ed becaus e of t he del et eri ous s i de effect s .
e. Intranasal anticholinergics. Iprat ropi um bromi de nas al s pray (0.03% and 0.06%) i s avai l abl e for t opi cal us e. Rhi norrhea but not ot her nas al s ympt oms are effect i vel y reduced. Si de effect s , al t hough mi ni mal (5%), i ncl ude drynes s and t rans i ent epi s t axi s .
f. Oral antileukotriene agents. Dat a s ugges t t hat t hes e medi cat i ons may be hel pful , and mont el ukas t s odi um has been approved for t hi s i ndi cat i on. Furt her s t udi es are needed t o defi ne t he rol e of t hes e agent s i n t he t reat ment of al l ergi c rhi ni t i s .
3. Immunotherapy. Such t reat ment may be i ndi cat ed i f pat i ent s cont i nue t o experi ence cl i ni cal l y s i gni fi cant s ympt oms aft er appropri at e envi ronment al avoi dance, and pharmacol ogi c meas ures have been t aken. Al t hough t reat ment i s t i me-cons umi ng and does not provi de
Pa g e 1 5 2 8
ABC Ambe r CHM Conve rte r Tria l ve rsion, http://w w w .proce sste x t.com/a bcchm.html
compl et e rel i ef for al l pat i ent s , i ncreas ed cos t s of ot her medi cat i ons makes i mmunot herapy rel at i vel y cos t -effect i ve.
IV. Urticaria and Angioedema A. Definitions
1. Urticaria (commonl y known as hives) i s a pruri t i c and t rans i ent s ki n erupt i on, wi t h i ndi vi dual l es i ons pres ent i ng as eryt hemat ous ci rcums cri bed papul es , or wheal s , oft en wi t h whi t e edemat ous cent ers . Les i ons are annul ar, round, fi gurat e, or confl uent . Repeat ed s crat chi ng oft en res ul t s i n t hi ckeni ng of t he s ki n, whi ch i s cal l ed lichenification. In acute urticaria, t he hi ves occur for l es s t han 6 weeks , and a t ri gger can be i dent i fi ed i n mos t cas es . In chronic urticaria, however, t he hi ves occur for great er t han s i x weeks , and t he t ri gger us ual l y remai ns unknown.
2. Angioedema may occur al one or joi nt l y wi t h urt i cari a. Angi oedema, or s wel l i ng, i s us ual l y of l onger durat i on t han urt i cari a, and oft en i nvol ves t he deeper s ubcut aneous t i s s ue—part i cul arl y i n l oos e areas around t he mout h, eyel i ds , and mal e geni t al i a. Angi oedema i n t he s ubmucos a of t he upper res pi rat ory t ract may l ead t o l aryngeal obs t ruct i on, whi ch repres ent s an ai rway emergency. Cl i ni cal l y s i gni fi cant caus es of angi oedema i ncl ude angi ot ens i n-convert i ng enzyme (ACE) inhibitors, NSAIDs, and heredi t ary defi ci enci es i n t he compl ement s ys t ems (al s o known as hereditary angioedema).
C. Clinical syndromes according to etiology
Pa g e 1 5 2 9
ABC Ambe r CHM Conve rte r Tria l ve rsion, http://w w w .proce sste x t.com/a bcchm.html
1. Idiopathic urticaria. Thi s t ype of urt i cari a l as t s s everal mont hs , and i n mos t cas es no s peci fi c caus e can be i dent i fi ed. Al s o known as chronic idiopathic urticaria . P.317
2. IgE-mediated urticaria. Many cas es of acute urticaria can be at t ri but ed t o an IgE-medi at ed react i on. The mos t common ant i gens el i ci t i ng t hi s res pons e are foods, latex, drugs, and venoms. Inhal ed pol l en may i nduce acut e urt i cari al l es i ons i n very s ens i t i ve i ndi vi dual s . Pol l en expos ure on t he s ki n al s o may i nduce urt i cari al -t ype l es i ons .
3. Complement-related disorders, al t hough rare, are wel l recogni zed. T wo major syndromes exi s t . o
o
a. Disorders associated with complement pathway activation. Connect i ve t i s s ue di s orders s uch as s ys t emi c l upus eryt hemat os us (SLE), hypocompl ement emi c vas cul i t i s , and s erum s i cknes s caus ed by drugs or bl ood product s are s omet i mes as s oci at ed wi t h chronic urticaria or angioedema. In t hes e di s orders , compl ement l evel s are oft en decreas ed but t hey al s o may be normal .
o
o
b. A deficiency in the inhibitor of the complement component C1 (C1-INH) may be heredi t ary (e.g., i n hereditary angioedema), i di opat hi c, or
Pa g e 1 5 3 0
ABC Ambe r CHM Conve rte r Tria l ve rsion, http://w w w .proce sste x t.com/a bcchm.html
as s oci at ed wi t h underl yi ng di s orders s uch as SLE or B-cel l l ymphomas .
4. Systemic diseases. Urt i cari a or angi oedema s omet i mes may be t he i ni t i al s i gn or a s equel a of an underl yi ng s ys t emi c i l l nes s . o
o
a. Acute infectious diseases i n chi l dren s uch as vi ral upper res pi rat ory i nfect i on or s t rept ococcal pharyngi t i s may be as s oci at ed wi t h acut e urt i cari a.
o
o
b. Systemic inflammatory disorders s uch as SLE, i nfl ammat ory bowel di s eas e, hepat i t i s B and C, macrogl obul i nemi a, and t he earl y phas e of ei t her hepat i t i s A or i nfect i ous mononucl eos i s may be as s oci at ed wi t h urt i cari a.
o
o
c. T hyroid disorders (wi t h or wi t hout hypot hyroi di s m) are s omet i mes as s oci at ed wi t h pers i s t ent i nt ract abl e urt i cari a. There i s a marked femal e predi l ect i on for t hi s condi t i on, and ant i t hyroi d ant i bodi es are commonl y s een i n affect ed pat i ent s .
5. Physical urticaria and angioedemas are i nduced by a phys i cal s t i mul us s uch as t emperat ure, s unl i ght , or phys i cal pres s ure. The pat hogeni c mechani s m for mos t of t hes e condi t i ons i s unknown. Feat ures of t he phys i cal urt i cari as and angi oedemas are des cri bed i n Tabl e 7-1.
D. Therapy
Pa g e 1 5 3 1
ABC Ambe r CHM Conve rte r Tria l ve rsion, http://w w w .proce sste x t.com/a bcchm.html
1. Prevention of further episodes i nvol ves avoi dance of known evocat i ve agent s (e.g., foods , medi cat i ons , and phys i cal fact ors ) and i dent i fi cat i on and management of underl yi ng medi cal di s orders .
2. Drug treatment of existing episodes us ual l y proceeds i n s t epwi s e fas hi on. o
o
a. H 1 -receptor blockers al one oft en are effect i ve. Fi rs t -generat i on ant i hi s t ami nes s uch as cyprohept adi ne, hydroxyz i ne, and di phenhydrami ne may be us ed, s o l ong as t he s i de effect of s edat i on can be s afel y t ol erat ed. Second-generat i on ant i hi s t ami nes s uch as l orat adi ne, des l orat adi ne, cet i ri zi ne, and fexofenadi ne may al s o be effect i ve t reat ment for urt i cari a, wi t hout t he s i de effect of s edat i on.
o
o
b. Adding an H 2 -receptor blocker s uch as ci met i di ne oft en has a s ynergi s t i c effect , becaus e H 2 recept ors make up approxi mat el y 15% of t he hi s t ami ne recept ors i n cut aneous bl ood ves s el s . H 2 -recept or bl ockers al one, however, are not effect i ve. Doxepi n, whi ch bl ocks bot h H 1 and H 2 recept ors , i s s omet i mes us eful i n t reat i ng refract ory urt i cari a.
o
o
c. Systemic steroids are res erved for s evere cas es . A s ki n bi ops y i s s ugges t ed before mai nt enance s t eroi d t herapy i s s t art ed.
o
Pa g e 1 5 3 2
ABC Ambe r CHM Conve rte r Tria l ve rsion, http://w w w .proce sste x t.com/a bcchm.html
o
d. Limited androgens s uch as danazol are effect i ve i n t reat i ng C1-INH defi ci ency, t hat i s , heredi t ary angi oedema.
V. Generalized Anaphylaxis A. Definition Anaphyl axi s i s an acut e l i fe-t hreat eni ng hypers ens i t i vi t y s yndrome. It affect s mul t i pl e organ s ys t ems and res ul t s from t he preci pi t ous and mas s i ve rel eas e of i nfl ammat ory medi at ors from bot h mas t cel l s and bas ophi l s . Sympt oms us ual l y begi n wi t hi n mi nut es of expos ure t o t he caus at i ve fact or and peak wi t hi n 1 hour, but can t ake up t o 4 hours for t he mos t s evere s ympt oms (s ee V D).
B. Incidence Exact i nci dence rat es are unknown, probabl y becaus e of i ncons i s t ent di agnos es of t he anaphyl axi s s yndrome. P.318
TABLE 7-1 Physical Urticarias and Angioedemas Cl in Di ic a al g F n St e o i at st T m u ic y ul r T p u e e e s s st
Pa g e 1 5 3 3
ABC Ambe r CHM Conve rte r Tria l ve rsion, http://w w w .proce sste x t.com/a bcchm.html
D St E St er ro x ro m ki a ki at n g n o g g g gr t h er t h a e at e p ske sk hi i n d i n s
tr
m
ip le re s p o n s e of L e wi s ( w h e al , fl ar e, a n
Pa g e 1 5 3 4
ABC Ambe r CHM Conve rte r Tria l ve rsion, http://w w w .proce sste x t.com/a bcchm.html
d e d e m a) , wi th oc ca si o n al p a ss iv e tr a n sf er to n or m al sk in ; p o
Pa g e 1 5 3 5
ABC Ambe r CHM Conve rte r Tria l ve rsion, http://w w w .proce sste x t.com/a bcchm.html
ss ib le i m m e di at epr e ss ur e a n gi o e d e m a; m a y fo ll o w e m ot io
Pa g e 1 5 3 6
ABC Ambe r CHM Conve rte r Tria l ve rsion, http://w w w .proce sste x t.com/a bcchm.html
n al st re ss or in fe ct io n D S D A el u e p a s t e pl y ai p, i c e n er at d e yt i o pr d h n e h e of s s e m pr ur a at e e vy o s s a pr u ur n e s e gi s s s u o ur w s i e e el n d
li g
e
n in
m
g er
a
ty t pi w ca ei ll g y ht
Pa g e 1 5 3 7
ABC Ambe r CHM Conve rte r Tria l ve rsion, http://w w w .proce sste x t.com/a bcchm.html
af s fe ct in g th e h a n d s or fe et ; di s a bi lit y s ec o n d ar y to di sc o m fo
Pa g e 1 5 3 8
ABC Ambe r CHM Conve rte r Tria l ve rsion, http://w w w .proce sste x t.com/a bcchm.html
rt C In In M h cr t e et ol e n h i n a s ac er s el h gi e y ol c i n pr i n ur co ur e t i re i t i s k ca b c i n ri o t r t e a d u st y nc ; t e al e m 1- x p t o er er 3- ci at m s ur m e e w ch (e h al .g e l e ., al n fe s , g v s e er o , m e et x i er m ci e s s e, ac h co
Pa g e 1 5 3 9
ABC Ambe r CHM Conve rte r Tria l ve rsion, http://w w w .proce sste x t.com/a bcchm.html
e m at p e a x ni p e o d s b ur y e, e s t xc re e ss ss ) iv e s w e at in g, la cr i m at io n, s al iv at io n, a n
Pa g e 1 5 4 0
ABC Ambe r CHM Conve rte r Tria l ve rsion, http://w w w .proce sste x t.com/a bcchm.html
d a b d o m in al cr a m pi n g C S L Ic ol u oc e d d al cu ur d i z b ti e e e ca n d ch ri e pr al a x ur l e (i p i t n di o u g o s s, e p ur er at e yt hi t o h c co e or l d m in
a,
h
a
er
n
it
d
e
s
Pa g e 1 5 4 1
ABC Ambe r CHM Conve rte r Tria l ve rsion, http://w w w .proce sste x t.com/a bcchm.html
d)
w el li n g in id io p at hi c fo r m ; p a p ul ar le si o n s ac co m p a ni e d b
Pa g e 1 5 4 2
ABC Ambe r CHM Conve rte r Tria l ve rsion, http://w w w .proce sste x t.com/a bcchm.html
y ch ill s, ar th ra lg ia , m y al gi a, a n d h e a d ac h e in in h er it e d fo r m
Pa g e 1 5 4 3
ABC Ambe r CHM Conve rte r Tria l ve rsion, http://w w w .proce sste x t.com/a bcchm.html
; p o ss ib le lif eth re at e ni n g e d e m a of u p p er or lo w er re s pi ra to ry
Pa g e 1 5 4 4
ABC Ambe r CHM Conve rte r Tria l ve rsion, http://w w w .proce sste x t.com/a bcchm.html
tr ac t Vi Pr S A br ol e p at o v pl or n er i c y g e at a e pr i o n d ur n gi oc i t of o cu u l a e p s b d at
or
e io
at
mn
or
a al
y
(h e
v
er x
or
e p
te
di o
x
ta s
to
ry ur
fo
or e
re
ac t o
ar
q vi
m
ui br re at d) i o n S E Pr E ol x o x ar p gr p ur o e o ti s ss s
Pa g e 1 5 4 5
ABC Ambe r CHM Conve rte r Tria l ve rsion, http://w w w .proce sste x t.com/a bcchm.html
ca ur i v ur ri e e e a to e to (s s xc s i x u or p t y n i a ec p or t i i fi e U o c s, V n U b li a V a g n w s ht d a e
li v
d
ch el
o
e e
n
ni n
w
fi gt
a
ca h
v
ti
el
o
e
n
n
of
gt
af
h
fe
tr
ct
ig
e
g
d
er
ar
in
e
g
a
th
s;
e
p
re
o
ac
ss
ti
ib
Pa g e 1 5 4 6
ABC Ambe r CHM Conve rte r Tria l ve rsion, http://w w w .proce sste x t.com/a bcchm.html
o
le
n)
pr e s e nc e of ot h er m et a b ol ic or i m m u n ol o gi c di s or d er s (i .e
Pa g e 1 5 4 7
ABC Ambe r CHM Conve rte r Tria l ve rsion, http://w w w .proce sste x t.com/a bcchm.html
., er yt hr o p oi et ic pr ot o p or p h yr ia a n d S L E) A E Pr A q x ur p u p i t pl a o u ic g s s at e ur s i o ni e o n c t o m of ur w et w t i at i
at
ca er m er
Pa g e 1 5 4 8
ABC Ambe r CHM Conve rte r Tria l ve rsion, http://w w w .proce sste x t.com/a bcchm.html
ri
e co
a
s m ac pr co e m ss p e a s ni of e v d ar b yi y n s g m te al m l
p
w er h at e ur al e s s re s e m bl in g th o s e of ch ol
Pa g e 1 5 4 9
ABC Ambe r CHM Conve rte r Tria l ve rsion, http://w w w .proce sste x t.com/a bcchm.html
in er gi c ur ti ca ri a SLE, s ys t emi c l upus eryt hem at os us ; UV, ul t ravi ol et .
C. Etiology A vari et y of mechani s ms , s ummari zed i n Tabl e 7-2, can i ni t i at e anaphyl act i c medi at or rel eas e.
1. IgE-mediated release. The bi ndi ng of IgE ant i bodi es ei t her t o compl et e ant i gens or t o ant i geni c det ermi nant s on hapt en–carri er compl exes can preci pi t at e t he rel eas e of i nfl ammat ory medi at ors . The mos t common caus es of IgE-medi at ed anaphyl axi s i ncl ude food (mos t not abl y peanuts, tree nuts, shellfish, and scaled fish), latex (es peci al l y i n health care workers, who have frequent expos ure), medications, insect venoms, and inhaled allergens.
2. Non–IgE-mediated release. Cert ai n s ubs t ances [e.g., opi at e anal ges i cs , radi ocont ras t medi a, NSAIDs
Pa g e 1 5 5 0
ABC Ambe r CHM Conve rte r Tria l ve rsion, http://w w w .proce sste x t.com/a bcchm.html
(as pi ri n)] can s t i mul at e degranul at i on of effect or cel l s di rect l y. Thi s i s cal l ed an anaphylactoid reaction.
3. Anaphylatoxin-mediated release. Si gni fi cant compl ement act i vat i on generat es l arge amount s of t he compl ement degradat i on product s C3a and C5a. Thes e fragment s bi nd t o mas t -cel l recept ors , t ri ggeri ng medi at or rel eas e.
D. Clinical Features and Diagnosis Cl i ni cal and l aborat ory fi ndi ngs charact eri s t i c of ext ens i ve medi at or rel eas e are t he bas i s for di agnos i s (Tabl e 7-3). P.319
TABLE 7-2 Etiologic Mechanisms of Anaphylaxis Ty pic al Me All ch er ani ge sm ns IgE Dru -m gs edi
S
at e ul f d
on
me am di a i de t or s rel
T
Pa g e 1 5 5 1
ABC Ambe r CHM Conve rte r Tria l ve rsion, http://w w w .proce sste x t.com/a bcchm.html
eas et r e
acy cl i n es P eni ci l l i ns Ce ph al o s po ri n s L oca l an es t het i cs Ins ect ven om s Al l erg en ext rac ts Ch ym
Pa g e 1 5 5 2
ABC Ambe r CHM Conve rte r Tria l ve rsion, http://w w w .proce sste x t.com/a bcchm.html
op ap ai n St r ept oki nas e Ins ul i n Foo d S hel l fi s h Nut s E ggs Mi l k An y foo d can cau se a rea ct i
Pa g e 1 5 5 3
ABC Ambe r CHM Conve rte r Tria l ve rsion, http://w w w .proce sste x t.com/a bcchm.html
on Di r Dru ect gs ma st
P ol y
cel l my me xi n di a B t or rel Opi eas at e e
s Va nco my ci n Ra di o con t ra st me
di a An Hu ap ma hyl n at o bl o xi n od -i n pro duc duc ed t s me
P
di a l as
Pa g e 1 5 5 4
ABC Ambe r CHM Conve rte r Tria l ve rsion, http://w w w .proce sste x t.com/a bcchm.html
t or ma rel
I
eas mm e
un
(vi ogl a
ob
co ul i mp ns le me Cry nt opr act eci i va pi t t i o at e n) s Ot No her ns t me ero cha i da ni s l ms ant i -i n fl a mm at o ry ag ent s Sul fi t e ad di t i ves Exe
Pa g e 1 5 5 5
ABC Ambe r CHM Conve rte r Tria l ve rsion, http://w w w .proce sste x t.com/a bcchm.html
rci s e Hor mo nes
1. Essential diagnostic findings i ncl ude at l eas t t wo of t he fol l owi ng s ympt oms : o
o
a. Bronchi al obs t ruct i on
o
o
b. Upper ai rway obs t ruct i on
o
o
c. Acut e hypot ens i on
o
o
d. Urt i cari a
o
o
e. Vomi t i ng or di arrhea
2. Risk factors for devel opi ng anaphyl axi s i ncl ude underl yi ng asthma or cardiac disease and nut allergy.
F. Therapy
Pa g e 1 5 5 6
ABC Ambe r CHM Conve rte r Tria l ve rsion, http://w w w .proce sste x t.com/a bcchm.html
1. Epinephrine. Thi s i s t he mos t effect i ve t herapy and s houl d be gi ven i mmedi at el y t o t reat anaphyl axi s . Int ramus cul ar i nject i on of epi nephri ne i s t he mos t effect i ve met hod of medi cat i on del i very. Ti mi ng i s cri t i cal —del ays i n epi nephri ne i nject i on are as s oci at ed wi t h i ncreas ed mort al i t y. Pat i ent s at ri s k of anaphyl axi s s houl d carry pen-l i ke epi nephri ne i nject ors wi t h t hem at al l t i mes .
2. Diphenhydramine (25–50 mg) i s gi ven i nt ravenous l y. Thi s t herapy wi l l not s t op an anaphyl act i c react i on but i s us ed as adjunct i ve t herapy t o epi nephri ne.
3. Corticosteroids (1–2 mg/kg) are admi ni s t ered i nt ravenous l y. Cort i cos t eroi ds are us ed t o decreas e t he l at er i nfl ammat ory res pons e.
4. Intravenous fluids, volume expanders, and pressor agents are us ed t o mai nt ai n bl ood pres s ure. Supplemental oxygen and even mechanical ventilation may be needed for res pi rat ory s upport . P.320
TABLE 7-3 Signs and Symptoms of Anaphylaxis
Pa g e 1 5 5 7
ABC Ambe r CHM Conve rte r Tria l ve rsion, http://w w w .proce sste x t.com/a bcchm.html
Or ga n or Or ga n Sy ste Ch m ar Aff act ect eri ed sti (% c In Sig div ns idu an als d Inv Sy olv mp ed to ) ms Ski Pru n
ri t u
(88 s %) Fl u s hi ng Urt i ca ri a An gi o ed em
Pa g e 1 5 5 8
ABC Ambe r CHM Conve rte r Tria l ve rsion, http://w w w .proce sste x t.com/a bcchm.html
a Eye Oc s
ul a
(16 r %) pru ri t u s Exc es s l ac ri m at i on Co nju nct i va l i nj ect i on Re Na s pi s al rat con ory ges s ys t i o te n m
Rhi nor
Up rhe per a ai r Co wa ug y
h
ed Ho
Pa g e 1 5 5 9
ABC Ambe r CHM Conve rte r Tria l ve rsion, http://w w w .proce sste x t.com/a bcchm.html
em ars a
en
(56 es s %) St ri dor Dy W h s pn eez ea i ng an Lar d
yng
wh eal eez ed i ng em (47 a %) Car W e di o akn vas es s cul Pal ar pi t s ys at i t e ons m
Tac
(33 hyc %) ard ia Hy pot ens i on Arr hyt hm i as Sh
Pa g e 1 5 6 0
ABC Ambe r CHM Conve rte r Tria l ve rsion, http://w w w .proce sste x t.com/a bcchm.html
ock Car di a c arr es t Ga Cra s t r mp oi n i ng t es Na t i n us e al
a
s ys Di a t e rrh m
ea
(30 Vo %) mi t i ng Ab do mi nal di s t en tio n Me t al l ic t as te
5. Pat i ent s on β-blockers may be refract ory t o t reat ment of anaphyl axi s . In t hi s cas e, glucagon can be gi ven i n addi t i on t o epi nephri ne t o provi de bot h
Pa g e 1 5 6 1
ABC Ambe r CHM Conve rte r Tria l ve rsion, http://w w w .proce sste x t.com/a bcchm.html
i not ropi c effect s and chronot ropi c effect s on t he heart by i ncreas i ng i nt racel l ul ar l evel s of cycl i c adenos i ne 3,'5'-monophos phat e, i ndependent of t he β-adrenergi c recept ors .
VI. Insect Sting Sensitivity St i ngs by a honeybee, yellow jacket, wasp, hornet, or fire ant may l ead t o s ens i t i zat i on and s ubs equent IgE-medi at ed hypers ens i t i vi t y react i ons .
A. Incidence Popul at i on s t udi es s how t hat up t o 25% of peopl e wi t h no hi s t ory of a s ys t emi c react i on aft er an i ns ect s t i ng nonet hel es s may be s ens i t i ve, as judged by pos i t i ve s ki n t es t s and t he pres ence of venom-s peci fi c IgE ant i bodi es . As many as 20% of t hes e i ndi vi dual s may have a s ys t emi c al l ergi c react i on i f s t ung agai n. It i s es t i mat ed t hat 75–100 deat hs occur i n t he Uni t ed St at es each year from i ns ect s t i ng react i ons .
B. Clinical features A nonallergic reaction t o an i ns ect s t i ng cons i s t s of t rans i ent s wel l i ng, pai n, and rednes s s urroundi ng t he i nject i on s i t e, al l of whi ch us ual l y s ubs i de i n 1–2 hours but can l as t up t o 48 hours . Allergic reactions vary from s cat t ered pat ches of urt i cari a t o s evere and fat al anaphyl axi s .
1. Large local reactions ext end a s i gni fi cant di s t ance from t he s t i ng s i t e ≥8 cm), peak i n 24 hours , and oft en t ake 5–7 days t o res ol ve.
2. Anaphylactic reactions occur wi t hi n a few mi nut es of t he s t i ng and may i nvol ve t he s ki n (urt i cari a, angi oedema, and pruri t us ), t he res pi rat ory t ract (l aryngeal edema and bronchos pas m), t he vas cul ar s ys t em (hypot ens i on), and t he gas t roi nt es t i nal s ys t em
Pa g e 1 5 6 2
ABC Ambe r CHM Conve rte r Tria l ve rsion, http://w w w .proce sste x t.com/a bcchm.html
(di arrhea, abdomi nal cramps , and naus ea). Repeat s t i ngs may caus e s i mi l ar, but more s evere s ympt oms . P.321
3. Delayed reactions i ncl ude s erum s i cknes s , vas cul i t i s , Gui l l ai n-Barré s yndrome, gl omerul onephri t i s , and myocardi t i s . The et i ol ogy of t hes e react i ons i s uncl ear.
C. Diagnosis
1. An accurat e history of t he s ympt oms and s i gns of a s ys t emi c al l ergi c react i on aft er a s t i ng i s neces s ary.
2. Specific antivenom IgE antibodies are det ect ed by pri ck or i nt radermal t es t s .
D. Therapy
1. Immediate therapeutic measures are s i mi l ar t o t hos e us ed i n t he t reat ment of anaphyl axi s from any caus e (s ee V F).
2. Prophylactic therapy o
o
a. Avoidance of i ns ect s t hat i nduce s ens i t i vi t y.
o
o
b. Al l at -ri s k i ndi vi dual s s houl d own and know how t o us e a self-injectable epinephrine kit.
Pa g e 1 5 6 3
ABC Ambe r CHM Conve rte r Tria l ve rsion, http://w w w .proce sste x t.com/a bcchm.html o
o
c. Indi vi dual s who have had an anaphyl act i c react i on t o an i ns ect s t i ng requi re specific venom immunotherapy, whi ch woul d t hen decreas e t hei r ri s k of anaphyl axi s t o t hat of t he general popul at i on.
VII. Drug Reactions A. Definitions Advers e drug react i ons can be cl as s i fi ed as predictable or unpredictable. Unpredi ct abl e res pons es i ncl ude hypersensitivity, or allergic, react i ons , whi ch cons t i t ut e 7%–10% of all adverse reactions.
1. T ype A or predictable responses mos t oft en i nvol ve known pharmacol ogi c act i ons , are dos e-dependent , and occur i n normal i ndi vi dual s . Nearl y 80% of advers e drug react i ons are t ype A, whi ch i ncl ude s i de effect s , overdos e/t oxi ci t y s econdary drug effect s , and drug–drug i nt eract i ons . The effect of ci profl oxaci n on warfari n us e [i ncreas i ng i nt ernat i onal normal i zed rat i o (INR)] and dys peps i a res ul t i ng from eryt hromyci n us e are bot h exampl es of t ype A res pons es .
2. T ype B or unpredictable responses t end t o occur i n predi s pos ed popul at i ons , are not rel at ed t o t he expect ed pharmacol ogi c act i on, and are mos t oft en i ndependent of dos e. T hree types of t ype B res pons es have been charact eri zed. o
o
a. Intolerance refers t o an advers e react i on t hat occurs at a s ubt herapeut i c dos e, for exampl e,
Pa g e 1 5 6 4
ABC Ambe r CHM Conve rte r Tria l ve rsion, http://w w w .proce sste x t.com/a bcchm.html
vomi t i ng at l ow dos es of t heophyl l i ne. o
o
b. Idiosyncrasy i s an unexpect ed and qual i t at i vel y abnormal res pons e, for exampl e, i s oni azi d-i nduced neuri t i s . Idi os yncrat i c react i ons may occur on a heredi t ary bas i s , for exampl e, hemol yt i c anemi a preci pi t at ed by qui nol ones i n a pat i ent wi t h gl ucos e-6-phos phat e dehydrogenas e (G6PD) defi ci ency.
o
o
c. Hypersensitivity res ul t s from an i mmunol ogi c react i on t hat t ri ggers an abnormal res pons e t o t he drug, for exampl e, peni ci l l i n-i nduced urt i cari a. Drug hypers ens i t i vi t y react i ons are cat egori zed accordi ng t o et i ol ogy, t ypi cal l y i nt o four t ypes .
(1) IgE-mediated reactions, al s o known as type I reactions, occur when a pharmacol ogi c agent or one of i t s met abol i t es bi nds wi t h drug-s peci fi c IgE ant i bodi es on mas t cel l or bas ophi l membranes , ul t i mat el y t ri ggeri ng t he rel eas e of medi at ors , whi ch l ead t o urticaria and angioedema, As not ed previ ous l y i n t hi s chapt er, bot h urt i cari a and angi oedema can occas i onal l y progres s t o anaphylaxis.
(2) Cytotoxic reactions, al s o known as type II reactions, res ul t from compl ement -medi at ed damage t o cel l membranes preci pi t at ed by t he bi ndi ng of IgM or IgG ant i bodi es t o drug ant i gens on cel l s urfaces . Thes e react i ons us ual l y i nvol ve bl ood cel l s , for exampl e, drug-i nduced
Pa g e 1 5 6 5
ABC Ambe r CHM Conve rte r Tria l ve rsion, http://w w w .proce sste x t.com/a bcchm.html
t hrombocyt openi a and hemol yt i c anemi a.
(3) Immune complex–mediated reactions, al s o known as type III reactions, devel op when s ol ubl e compl exes of IgM or IgG ant i bodi es and drug ant i gen are deposited in tissues. Compl ement act i vat i on occurs i n t he s ol ubl e phas e or when t he compl ex at t aches t o ves s el wal l s , l eadi ng t o i nfl ammat i on and s ubs equent t i s s ue i njury. Rash and fever are t he mos t common cl i ni cal mani fes t at i ons , but lymphadenopathy and arthralgia may al s o occur, for exampl e, s erum s i cknes s s yndrome caus ed by i nject i on of forei gn prot ei n or s erum.
(4) Cell-mediated reactions, al s o known as type IV reactions, requi re s ens i t i zed T l ymphocyt es t hat recogni ze a drug or i t s met abol i t e. The ant i gen–T-cel l i nt eract i on l eads t o a P.322
l ymphoki ne-i nduced i nfl ammat ory res pons e occurri ng 24–72 hours aft er drug admi ni s t rat i on. Contact dermatitis, for exampl e, from neomyci n, i s t he mos t common cl i ni cal exampl e of a cel l -medi at ed react i on. Drug-induced interstitial nephritis, for exampl e, from met hi ci l l i n, i s al s o an exampl e of a cel l -medi at ed react i on.
B. Clinical features Organ-s peci fi c s yndromes res ul t i ng from drug al l ergy i ncl ude t he fol l owi ng:
Pa g e 1 5 6 6
ABC Ambe r CHM Conve rte r Tria l ve rsion, http://w w w .proce sste x t.com/a bcchm.html
1. Dermatologic reactions are t he mos t common mani fes t at i ons of drug s ens i t i vi t y. o
o
a. Urticaria, occas i onal l y progres s i ng t o anaphyl axi s , can be preci pi t at ed by a vari et y of medi cat i ons ; t he mos t common offenders are peni ci l l i n, s ul fonami des , cephal os pori ns , and al l ergen ext ract s . Mos t urt i cari al react i ons are t he res ul t of IgE-medi at ed or di rect hi s t ami ne rel eas e.
o
o
b. Fixed drug eruptions, whi ch are mos t l i kel y at t ri but abl e t o a cel l -medi at ed react i on, may devel op aft er i nges t i on of t et racycl i ne, s ul fonami des , and peni ci l l i n. In t hi s react i on, di s cret e, nonpruri t i c l es i ons wi t h a macul ar t o bul l ous appearance occur at t he s ame pl ace each t i me t he medi cat i on i s t aken.
o
o
c. Photodermatitis i s charact eri zed by a bri ght eryt hemat ous erupt i on or eczemat oi d l es i ons i n areas expos ed t o ul t ravi ol et l i ght . Two pres ent at i ons are recogni zed.
(1) Phototoxicity as a res ul t of ul t ravi ol et l i ght expos ure occurs earl y i n drug t reat ment . Drugs i mpl i cat ed i n phot ot oxi c react i ons i ncl ude doxycycl i ne, coal t ar deri vat i ves , and ps oral ens .
(2) Photoallergy occurs 4–21 days aft er t he
Pa g e 1 5 6 7
ABC Ambe r CHM Conve rte r Tria l ve rsion, http://w w w .proce sste x t.com/a bcchm.html
i nges t i on of t he caus at i ve agent . It appears t o be a cel l -medi at ed proces s i n whi ch ul t ravi ol et l i ght i nduces a chemi cal al t erat i on of t he drug, l eadi ng t o t i s s ue s ens i t i zat i on and s ubs equent i njury. Phenot hi azi nes , gri s eoful vi n, and s ul fonami des are common offenders . o
o
d. Contact dermatitis devel ops 48–72 hours aft er t opi cal appl i cat i on of t he i mpl i cat ed medi cat i on and i s charact eri zed by a ves i cul ar T-cel l –medi at ed i nfl ammat ory react i on. Common preparat i ons caus i ng t hes e s ki n l es i ons are para -ami nobenzoi c aci d (PABA), neomyci n, and ant i hi s t ami nes .
o
o
e. Febrile mucocutaneous reactions are uncommon but may be s evere. Cyt ot oxi c, i mmune compl ex, and cel l -medi at ed react i ons al l cont ri but e t o t he pat hogenes i s of t hes e s yndromes .
(1) Stevens-Johnson syndrome, a s evere form of eryt hema mul t i forme, may mani fes t as papul ar, urt i cari al , ves i cul ar, or purpuri c l es i ons i nvol vi ng t wo or more mucos al s urfaces .
(2) T oxic epidermal necrolysis mani fes t s as epi t hel i al bul l ae wi t h s ubs equent des quamat i on. Fever and vi s ceral i nvol vement are common.
Pa g e 1 5 6 8
ABC Ambe r CHM Conve rte r Tria l ve rsion, http://w w w .proce sste x t.com/a bcchm.html
(3) Drugs i mpl i cat ed i n bot h s yndromes i ncl ude ri fampi n, phenobarbi t al , phenyt oi n, t ri met hopri m–s ul famet hoxaz ol e, and peni ci l l i ns .
2. Hepatic syndromes res ul t pri mari l y i n hepat ocel l ul ar changes , granul oma format i on, or chol es t as i s . Hydant oi n and hal ot hane, for exampl e, can caus e hepat ocel l ul ar damage, as can as pi ri n when admi ni s t ered t o chi l dren wi t h juveni l e rheumat oi d art hri t i s . Al l opuri nol , met hyl dopa, or s ul fonami des may caus e granul omat ous changes . Phenot hi azi nes , az at hi opri ne, and eryt hromyci n es t ol at e have been l i nked t o chol es t as i s and pai nl es s jaundi ce.
3. Renal involvement mos t oft en t akes t he form of acut e i nt ers t i t i al i nfl ammat i on. Met hi ci l l i n, s ul fonami des , and cephal os pori ns have been i mpl i cat ed i n t hi s react i on. Membranous gl omerul onephri t i s may occur aft er admi ni s t rat i on of capt opri l , gol d, peni ci l l ami ne, or probeneci d.
4. Pulmonary reactions i ncl ude pul monary i nfi l t rat i on and bronchos pas m. Pul monary i nfi l t rat i on has been report ed wi t h us e of ni t rofurant oi n, gol d compounds , and met hot rexat e. Drug-i nduced bronchos pas m may occur i n as t hmat i c pat i ent s aft er admi ni s t rat i on of as pi ri n or ot her NSAIDs . Exces s l eukot ri ene product i on, res ul t i ng from i nhi bi t i on of t he cycl ooxygenas e pat hway, pres umabl y i s res pons i bl e for t hi s react i on.
C. Diagnosis Becaus e s o few s peci fi c di agnos t i c l aborat ory t es t s are avai l abl e t o
Pa g e 1 5 6 9
ABC Ambe r CHM Conve rte r Tria l ve rsion, http://w w w .proce sste x t.com/a bcchm.html
confi rm drug al l ergy, obt ai ni ng an accurat e hi s t ory i s es s ent i al . Drug al l ergy s houl d be cons i dered as t he pos s i bl e caus e of al mos t any cl i ni cal s ympt om or s i gn becaus e drug al l ergy react i ons may mi mi c s o many ot her cl i ni cal condi t i ons .
1. Cl i ni cal feat ures t hat s ugges t drug hypers ens i t i vi t y o
o
a. Pri or expos ure t o t he drug
o
o
b. Ons et of s ympt oms wi t hi n 48 hours wi t h previ ous expos ure or 7 or more days aft er i ni t i at i ng drug t reat ment i f t here i s no pri or expos ure. P.323
o
o
c. Admi ni s t rat i on of t he drug at t he recommended dos e
o
o
d. Sympt oms and s i gns commonl y as s oci at ed wi t h known al l ergi c react i ons
o
o
e. Prompt di s appearance of t he s ympt oms aft er di s cont i nuat i on of t he i mpl i cat ed drug
2. Specific diagnostic tests o
o
a. Immediate skin tests for IgE-medi at ed react i ons (e.g., peni ci l l i n, i ns ul i n, and chymopapai n)
Pa g e 1 5 7 0
ABC Ambe r CHM Conve rte r Tria l ve rsion, http://w w w .proce sste x t.com/a bcchm.html o
b. Delayed skin tests (pat ch t es t i ng; a pres umed
o
ant i gen i s pl aced on t he s ki n, covered, and t hen reas s es s ed for a react i on at 48 hours aft er appl i cat i on) for cel l -medi at ed react i ons (e.g., PABA, ni ckel ) o
c. RAST testing for s peci fi c IgE ant i bodi es (e.g.,
o
peni ci l l i n) o
d. Coombs' antiglobulin test for cyt ot oxi c
o
react i ons caus i ng hemol yt i c anemi a
D. Reactions to selected drugs and biologic agents
1. Reactions to penicillin and i t s s emi s ynt het i c deri vat i ves are among t he mos t common advers e drug react i ons . From 1% t o 10% of pat i ent s recei vi ng t hes e drugs experi ence an al l ergi c res pons e. Monobact ams rarel y cros s -react wi t h peni ci l l i n, but carbapenems s houl d be cons i dered equal l y cros s -react i ve. Ni net y percent of s ens i t i zed i ndi vi dual s out grow t hi s al l ergy aft er 10 years . o
o
a. Diagnosis. W hen a β-l act am ant i bi ot i c i s t he onl y drug of choi ce for pat i ent s wi t h a hi s t ory s ugges t i ve of a peni ci l l i n al l ergy, skin testing i s recommended al t hough t es t i ng mat eri al s are l i mi t ed and des ens i t i zat i on may be requi red i n hi gh ri s k cas es . Pat i ent s wi t h a pos i t i ve s ki n t es t s houl d
Pa g e 1 5 7 1
ABC Ambe r CHM Conve rte r Tria l ve rsion, http://w w w .proce sste x t.com/a bcchm.html
not recei ve peni ci l l i n or s emi s ynt het i c peni ci l l i ns . Cephal os pori ns s houl d be admi ni s t ered t o s uch pat i ent s caut i ous l y, becaus e up t o 2% of pat i ent s who are al l ergi c t o peni ci l l i n have a cros s -react i on t o cephal os pori ns . Fi rs t -generat i on cephal os pori ns appear t o carry a great er ri s k t han s econd- or t hi rd-generat i on cephal os pori ns . o
o
b. T herapy. Des ens i t i zat i on t o peni ci l l i n-rel at ed I gE-medi at ed react i ons may be accompl i s hed by admi ni s t eri ng peni ci l l i n i n i ncrement s of i ncreas i ng s t rengt h over 4–6 hours . The des ens i t i zat i on procedure s houl d be carri ed out i n a cont rol l ed s et t i ng wi t h acces s t o appropri at e emergency t reat ment . The oral rout e, whi ch has been found t o be s afer t han t he parent eral rout e, i s preferred. Parent eral admi ni s t rat i on may be i ni t i at ed aft er t he compl et i on of oral des ens i t i zat i on. The refract ory, or des ens i t i zed, s t at e may be mai nt ai ned for weeks t o mont hs by admi ni s t eri ng t he drug at l eas t every 12 hours . Once t he pat i ent has di s cont i nued t he medi cat i on, prot ect i on i s l os t , and repeat i ng t he des ens i t i zat i on woul d be needed onl y i f t he drug needed t o be readmi ni s t ered.
2. Reactions to radiographic contrast dye occur i n 4%–8% of pat i ent s s t udi ed, wi t h anaphyl act oi d res pons es occurri ng i n as many as 1.7% of pat i ent s wi t h react i ons . Thi s i s caus ed by t he hi gh os mol ari t y of t he dye. Repeat anaphyl act oi d react i ons may occur i n 16%–44% of pat i ent s . Becaus e recent l y i nt roduced lower osmolarity contrast dyes appear t o be as s oci at ed wi t h a decreas ed ri s k of anaphyl act oi d react i ons , t hey are now t he agent s of choi ce,
Pa g e 1 5 7 2
ABC Ambe r CHM Conve rte r Tria l ve rsion, http://w w w .proce sste x t.com/a bcchm.html
part i cul arl y for t hos e pat i ent s wi t h a hi s t ory of previ ous al l ergi c react i on. There i s no as s oci at i on bet ween radi ocont ras t dye react i ons and s hel l fi s h or i odi ne s ens i t i vi t y. o
o
a. Pathogenesis. Anaphyl act oi d res pons es cl i ni cal l y mi mi c IgE-medi at ed anaphyl axi s but do not i nvol ve IgE or any ot her i mmune mechani s m.
o
o
b. Clinical features. Sympt oms of exces s vagal s t i mul at i on s uch as bradycardi a, hypot ens i on, naus ea, or vomi t i ng may occur. Thes e s ympt oms may be part i cul arl y s evere, s omet i mes cul mi nat i ng i n s hock, part i cul arl y i n pat i ent s t aki ng β-adrenergi c bl ockers and ACE i nhi bi t ors .
o
o
c. T herapy. Treat ment of an anaphyl act oi d react i on i s i dent i cal t o t hat for anaphyl axi s from ot her caus es (s ee V F). Vagal react i ons can be t reat ed wi t h at ropi ne. A pret reat ment prophylactic regi men of predni s one and di phenhydrami ne s i gni fi cant l y decreas es t he l i kel i hood of a repeat anaphyl act oi d react i on. Some expert s al s o recommend t he addi t i on of ephedri ne and an H 2 -s peci fi c ant i hi s t ami ne.
3. Insulin allergy and resistance o
o
a. Allergic reactions. Becaus e bovi ne i ns ul i n di ffers from human i ns ul i n by t hree ami no aci ds and porci ne i ns ul i n di ffers by onl y one ami no aci d, t he bovi ne-bas ed preparat i on i s more al l ergeni c.
Pa g e 1 5 7 3
ABC Ambe r CHM Conve rte r Tria l ve rsion, http://w w w .proce sste x t.com/a bcchm.html
Bovi ne i ns ul i n i s no l onger avai l abl e i n t he Uni t ed St at es . Human i ns ul i n, produced by recombi nant DNA t echnol ogy, i s much l es s al l ergeni c t han t he ani mal preparat i ons but rarel y may s t i l l preci pi t at e IgE-medi at ed hypers ens i t i vi t y react i ons . Cert ai n hi s t ocompat i bi l i t y ant i gens (HLA-DR2, HLA-DR3, and HLA-B7) may be as s oci at ed wi t h al l ergi c react i ons t o P.324
i ns ul i n, and as many as 50% of peopl e exhi bi t i ng i ns ul i n al l ergy al s o have experi enced al l ergi c react i ons t o ot her drugs .
(1) Local allergic reactions are charact eri zed by pruri t us , s wel l i ng, and mi l d eryt hema, us ual l y occurri ng duri ng t he fi rs t few mont hs of t herapy. Thes e react i ons oft en s ubs i de compl et el y aft er a few weeks ; i f not , ant i hi s t ami nes may be ei t her admi ni s t ered s ys t emi cal l y or mi xed wi t h t he i ns ul i n. Di vi di ng t he dos e among t wo or more i nject i on s i t es al s o may be effect i ve. Prol onged l ocal react i ons may res ul t i n l i poat rophy.
(2) Systemic allergic reactions are much l es s common t han l ocal react i ons (i .e., t hey occur i n <0.1% of pat i ent s ) and are t reat ed s i mi l arl y t o general i zed anaphyl axi s (s ee V F). If i ns ul i n t herapy i s rei ns t i t ut ed l es s t han 24 hours aft er t he s ys t emi c react i on, t he next dos e s houl d be reduced s o i t i s one-t hi rd t o one-s i xt h of t he ori gi nal , wi t h s l ow
Pa g e 1 5 7 4
ABC Ambe r CHM Conve rte r Tria l ve rsion, http://w w w .proce sste x t.com/a bcchm.html
i ncrement al i ncreas es as needed t o obt ai n met abol i c cont rol . A des ens i t i zat i on prot ocol mus t be i mpl ement ed i f pat i ent s requi re s ubcut aneous i ns ul i n and more t han 24 hours have pas s ed s i nce t he s ys t emi c react i on occurred. o
o
b. Resistance
(1) Nonimmunologic caus es of insulin resistance i ncl ude i nfect i on, s t res s , pregnancy, obes i t y, Cus hi ng's s yndrome, acromegal y, pheochromocyt oma, l eprechauni s m, and type A insulin resistance syndrome accompani ed by acant hos i s ni gri cans .
(2) Immunologic insulin resistance i s medi at ed by IgG ant i -i ns ul i n ant i bodi es and i s mani fes t ed by a need for i ncreas i ng dai l y dos es (up t o 200 U/kg i n adul t s and up t o 2 U/kg i n chi l dren). The pres ence of hi gh-t i t er IgG ant i bodi es t o i ns ul i n or t o t he i ns ul i n recept or can be confi rmed by a compet i t i ve bi ndi ng as s ay.
4. Protamine sulfate sensitivity. Prot ami ne s ul fat e i s a l ow-mol ecul ar-wei ght prot ei n pri mari l y us ed t o neut ral i ze hepari n, oft en duri ng cardi opul monary bypas s procedures , or t o act as a compl exi ng agent t o prepare neut ral prot ami ne Hagedorn (NPH) i ns ul i n. o
Pa g e 1 5 7 5
ABC Ambe r CHM Conve rte r Tria l ve rsion, http://w w w .proce sste x t.com/a bcchm.html
o
a. Clinical features. Fl us hi ng, urt i cari a, hypot ens i on or hypert ens i on, vent ri cul ar fi bri l l at i on, wheezi ng, noncardi ac pul monary edema, t rans i ent el evat i ons i n pul monary art ery pres s ure, t hrombocyt openi a, or neut ropeni a may devel op. Al l ergi c react i ons t o prot ami ne s ul fat e have been at t ri but ed t o IgE-medi at ed anaphyl act i c and compl ement -medi at ed anaphyl act oi d mechani s ms . Di abet i c pat i ent s who have us ed prot ami ne-cont ai ni ng i ns ul i ns are at i ncreas ed ri s k.
o
o
b. Diagnosis and therapy. Ski n t es t i ng i s not rel i abl e, but pret reat ment wi t h ant i hi s t ami nes and cort i cos t eroi ds may be hel pful for pat i ent s s us pect ed of bei ng at hi gh ri s k for al l ergi c react i ons . For hi gh-ri s k s urgi cal pat i ent s , al t ernat i ve revers al agent s s houl d be us ed.
5. Heterologous and homologous sera and vaccines o
o
a. Heterologous sera, whi ch are us ed i n t he management of gas gangrene, bot ul i s m, di pht heri a, and s ome s nake and s pi der bi t es , may evoke s erum s i cknes s or anaphyl axi s . Ant i l ymphocyt e gl obul i n produced i n hors es or rabbi t s may caus e anaphyl axi s and t hrombocyt openi a. OKT3, a muri ne monocl onal ant i body us ed t o i nhi bi t organ t rans pl ant reject i on, al s o can i nduce vari ous al l ergi c react i ons .
o
o
b. Homologous hyperimmune s era s uch as t hos e devel oped t o t reat rabi es , hepat i t i s B vi rus ,
Pa g e 1 5 7 6
ABC Ambe r CHM Conve rte r Tria l ve rsion, http://w w w .proce sste x t.com/a bcchm.html
t et anus , and cyt omegal ovi rus (CMV) l es s oft en caus e s erum s i cknes s . o
o
c. Mumps, measles, rubella, influenza, and yellow fever vaccines are grown on chi ck embryos , and anaphyl axi s aft er vacci nat i on i n egg-s ens i t i ve i ndi vi dual s has been report ed but i s ext remel y rare. The current recommendat i on i s t hat t he egg-al l ergi c pat i ent can be gi ven t he vacci ne wi t hout concern. In t he ext remel y s ens i t i ve egg-al l ergi c pat i ent who has anaphyl act i c react i ons , t es t i ng can be done. Thi s as s es s ment by bot h pri ck and i nt radermal t es t i ng s houl d be cons i dered before vacci nat i on. If a pat i ent t es t s pos i t i ve, t he vacci ne s houl d be admi ni s t ered us i ng a des ens i t i zat i on program.
6. Aspirin reactions us ual l y mani fes t as urt i cari a or wheez i ng. Chroni c rhi ni t i s , s i nus i t i s , and oft en i nt ract abl e s t eroi d-dependent as t hma devel op i n a s mal l , wel l -recogni zed s ubgroup of i ndi vi dual s bet ween t he ages of 30 and 60 years . In t hes e at -ri s k i ndi vi dual s , l i fe-t hreat eni ng s evere bronchos pas m may occur aft er as pi ri n i nges t i on. However, t hos e i n whom chroni c as t hma devel ops cannot i mprove t hei r as t hma by abs t i nence from as pi ri n or ot her NSAIDs . Becaus e t here are no i n vi vo or i n vi t ro t es t s for as pi ri n al l ergy, di agnos i s can be made onl y from t he medi cal hi s t ory and by as pi ri n chal l enge. Al t hough oral des ens i t i zat i on has been s ucces s ful i n s ome i ndi vi dual s , i t i s rarel y i ndi cat ed.
E. Other allergy syndromes
Pa g e 1 5 7 7
ABC Ambe r CHM Conve rte r Tria l ve rsion, http://w w w .proce sste x t.com/a bcchm.html
1. Allergic reactions to latex. Lat ex al l ergy has recent l y emerged as a s i gni fi cant cl i ni cal probl em. Nat ural l at ex i s found i n s urgi cal gl oves , cat het ers , bal l oons , condoms , and dent al el as t i c adhes i ves . o
o
a. Incidence. Heal t h care workers are part i cul arl y s us cept i bl e; up t o 7.4% of s urgeons and 5.6% of operat i ng room nurs es are al l ergi c t o l at ex. Chi l dren wi t h s pi na bi fi da and geni t ouri nary probl ems have a hi gh i nci dence of l at ex al l ergy.
o
o
b. Clinical features. Urt i cari a on cont act , rhi ni t i s , conjunct i vi t i s , and bronchos pas m aft er i nhal at i on are s ugges t i ve of l at ex al l ergy. Severe anaphyl axi s may ens ue duri ng s urgery, bari um enema, or dent al procedures .
o
o
c. Diagnosis. Lat ex al l ergy i s IgE-medi at ed, and t he di agnos i s i s bes t confi rmed by s erol ogi c IgE as s ays .
o
o
d. Prevention. Nonl at ex gl oves and t ubi ng are avai l abl e and s houl d be us ed duri ng s urgery i n pat i ent s wi t h known l at ex al l ergy. Premedi cat i on, ant i hi s t ami nes , and cort i cos t eroi ds are al s o us ed t o prevent s evere react i ons , but may not prevent react i ons .
3. ACE inhibitors. Thes e agent s exert t hei r effect s by pharmacol ogi c bl ockage of ki ni nas e II, whi ch l eads t o el evat ed l evel s of bradyki ni n and s ubs t ance P.
Pa g e 1 5 7 8
ABC Ambe r CHM Conve rte r Tria l ve rsion, http://w w w .proce sste x t.com/a bcchm.html o
o
a. Incidence. Chroni c cough devel ops i n 5%–20% of pat i ent s , and angi oedema occurs i n 0.1%–0.2% of pat i ent s t aki ng ACE i nhi bi t ors . Thes e medi cat i ons may al s o caus e anaphyl axi s . The occurrence of anaphyl axi s duri ng di al ys i s i s known t o be more frequent i n pat i ent s t aki ng ACE i nhi bi t ors .
o
o
b. Clinical features. A chroni c cough s t art s wi t hi n 1 week t o 6 mont hs aft er t he drug i s begun. The cough i s mos t oft en noct urnal and pri mari l y nonproduct i ve. Angi oedema oft en devel ops by 24 hours or wi t hi n 5–7 days aft er i ni t i at i on of t herapy. Severe l aryngeal edema and accompanyi ng s t ri dor may be l i fe-t hreat eni ng.
o
o
c. T herapy. Chroni c cough s ubs i des 1–4 weeks aft er t he medi cat i on i s s t opped. Subs t i t ut i on of anot her ACE i nhi bi t or i s not us eful . (For management of s evere angi oedema, s ee IV D.) Moderat e s ympt oms res ol ve gradual l y when t he drug i s di s cont i nued.
5. Sulfonamides and HIV-infected patients. A macul opapul ar erupt i on occurs i n up t o 3% of s ul fonami de reci pi ent s , s omet i mes fol l owed by eryt hema mul t i forme or St evens -Johns on s yndrome. In HIV-i nfect ed i ndi vi dual s who t ake s ul fonami des , 50% or more devel op a macul opapul ar erupt i on t hought t o be rel at ed t o reduced l evel s of gl ut at hi one reduct as e. No rel i abl e s ki n or i n vi t ro t es t s are avai l abl e. Oral
Pa g e 1 5 7 9
ABC Ambe r CHM Conve rte r Tria l ve rsion, http://w w w .proce sste x t.com/a bcchm.html
des ens i t i zat i on prot ocol s have s omet i mes been us ed s ucces s ful l y.
6. Minocycline-induced lupus. Mi nocycl i ne i s now frequent l y us ed i n t he l ong-t erm management of acne vul gari s . o
o
a. Clinical features. Mus cul os kel et al compl ai nt s , fever, wei ght l os s , and pl europul monary i nvol vement are charact eri s t i c, wi t h s ympt oms devel opi ng an average of 2 years aft er i ni t i at i on of drug t herapy. P.325
o
o
b. Diagnosis. Neurol ogi c, renal , and vas cul i t i c changes are unus ual , but hepat i c t rans ami nas e l evel s are commonl y el evat ed. A pos i t i ve ant i nucl ear ant i body (ANA) t i t er i s uni forml y pres ent , and ant i –doubl e-s t randed DNA and ant i neut rophi l i c cyt opl as mi c ant i body (ANCA) t es t s are al s o s omet i mes pos i t i ve.
o
o
c. T herapy. Sympt oms are s el f-l i mi t ed once t reat ment wi t h mi nocycl i ne i s s t opped.
F. Prevention of drug reactions
1. The concurrent admi ni s t rat i on of s everal medi cat i ons s houl d be avoi ded.
Pa g e 1 5 8 0
ABC Ambe r CHM Conve rte r Tria l ve rsion, http://w w w .proce sste x t.com/a bcchm.html
2. A det ai l ed hi s t ory of any previ ous drug react i ons s houl d be obt ai ned for al l pat i ent s t o gui de s el ect i on of a medi cat i on t hat i s unl i kel y t o cros s -react .
3. Pat i ent s wi t h document ed al l ergi c drug react i ons s houl d be i ns t ruct ed t o carry i dent i fi cat i on (e.g., a medi cal al ert bracel et ). Medi cal records for s uch pat i ent s s houl d be cl earl y marked.
4. Oral admi ni s t rat i on, when pos s i bl e, i s preferabl e t o parent eral admi ni s t rat i on, becaus e t he i nci dence of react i ons aft er oral admi ni s t rat i on i s much l ower.
5. Al l ergi c react i ons are unl i kel y t o be di s covered duri ng premarket i ng t ri al s , becaus e t he number of pat i ent s i s s mal l when compared wi t h t he i nci dence of react i ons , s o part i cul ar vi gi l ance i s i ndi cat ed when admi ni s t eri ng any new drug.
P.326
VIII. Immunodeficiency Disorders Thes e di s orders , whi ch may be ei t her hereditary or acquired, repres ent four distinct abnormalities of i mmune funct i on. The genes t hat encode for many of t he defect s t hat are res pons i bl e for t hes e di s orders have now been mapped t o s peci fi c chromos omes .
A Compl ement s ys t em defect s may affect ei t her t he ni ne compl ement component s or t hei r as s oci at ed regul at ory prot ei ns . The et i ol ogy, i nheri t ance pat t ern, cl i ni cal feat ures , and di agnos i s of t hes e s yndromes are s ummari zed i n Tabl e 7-4. Thi s i s t he rares t of t he i mmunodefi ci enci es .
Pa g e 1 5 8 1
ABC Ambe r CHM Conve rte r Tria l ve rsion, http://w w w .proce sste x t.com/a bcchm.html
B. Phagocyte disorders (Tabl e 7-5)
1. Common clinical features. Recurrent cut aneous and s i nopul monary i nfect i ons as wel l as chroni c cut aneous and oral i nfl ammat i on (oral ul cers ) may occur. Al t hough t he i nfl ammat ory res pons e may be del ayed, s ucces s i ve i nfect i ons wi t h St aphyl oc oc c us aureus , Ps eudomonas s peci es , Haemophi l us i nfl uenzae, and As pergi l l us are common and may be l i fe-t hreat eni ng.
2. T herapy. Management cons i s t s pri mari l y of t he di agnos i s , t reat ment , and prevent i on of pyogeni c i nfect i ons . However, s uch i nfect i ons are oft en refract ory t o t reat ment , des pi t e appropri at e ant i bi ot i c t herapy. o
o
a. A compl et e di agnos t i c work-up i s es s ent i al , i ncl udi ng t aki ng appropri at e cul t ures and bei ng es peci al l y vi gi l ant for t he devel opment of s uperfi ci al or vi s ceral mycos i s .
o
o
b. Prompt s urgi cal drai nage i s es s ent i al .
o
o
c. Bone marrow t rans pl ant at i on (BMT) has been us ed s ucces s ful l y i n s everal chi l dren t o correct l eukocyt e adhes i on defect s . BMT has al s o been us ed s ucces s ful l y t o effect l as t i ng amel i orat i on of chroni c granul omat ous di s eas e.
o
o
d. Daily trimethoprim-sulfa prophylaxis and I
Pa g e 1 5 8 2
ABC Ambe r CHM Conve rte r Tria l ve rsion, http://w w w .proce sste x t.com/a bcchm.html
FN-γ admi ni s t ered s ubcut aneous l y t hree t i mes per week res ul t s i n a reduced number of i nfect i ons and i s approved for us e i n chroni c granul omat ous di s eas e (CGD).
3. Genetic counseling s houl d be offered t o fami l i es of affect ed i ndi vi dual s .
TABLE 7-4 Complement Deficiency Disorders I n h e ri ta n P c h D e e ef / n e C ot ct h y Cl iv r pi in e o c ic C m E al o o x F Di m s p e a p o r at g o m e u n n e ss r o e Si io e si nt te n s s C A I L A
Pa g e 1 5 8 3
ABC Ambe r CHM Conve rte r Tria l ve rsion, http://w w w .proce sste x t.com/a bcchm.html
2 ut m u s s o p p e s ai u s s o re s - m m d li e al re k nt re m e of ce o s y t o ss v n ta i v al dr l e/ of o h 6 i
me
p m e; m m s ol u e yt n v ic e er co co e m m p pl pl y e e o m x g e e e nt s ni (C c H in fe
50
)*
ct io n s wi th gr a
Pa g e 1 5 8 4
ABC Ambe r CHM Conve rte r Tria l ve rsion, http://w w w .proce sste x t.com/a bcchm.html
m -p o si ti v e b ac te ri a C A I S A 4 ut m L s s o p E e s ai a s s o re n m md d e al re l u nt re m p of ce o u t o ss v s- ta i v al l i l e/ of k h 1 i
e e
9 m sym q m n ol u dr yt n o ic e m co co e; m m Sj pl pl à e e ¶ m x gr e
Pa g e 1 5 8 5
ABC Ambe r CHM Conve rte r Tria l ve rsion, http://w w w .proce sste x t.com/a bcchm.html
e e nt s n' (C s H sy n
50
)*
dr o m e; H e n oc hSc h à ¶ nl ei n sy n dr o m e C A I R A 3 ut m ec s s o p ur e s ai re s s o re nt m md s e al o e nt re p v of
Pa g e 1 5 8 6
ABC Ambe r CHM Conve rte r Tria l ve rsion, http://w w w .proce sste x t.com/a bcchm.html
ce s er t o ss o e ta i v ni i n l e/ z fe h 1 at ct e 9 io io m q n n ol s yt d ic u co e m t o pl p e y m o e g nt e (C ni H c or
50
)*
gr a m -n e g at iv e or g a ni s m
Pa g e 1 5 8 7
ABC Ambe r CHM Conve rte r Tria l ve rsion, http://w w w .proce sste x t.com/a bcchm.html
s C A I R A 5 ut m ec s s â o p ur e €“ s ai re s s C o re nt m 9 md me al fo e nt re r ni of ce m n t o s s at g t a i v i o oc l e/ n oc h C of ca e 5( t h l
m
9 e a ol q) m n yt ; e d ic C m g co 6, br o m C a n pl 7, n oc e C e oc m 9; at ca e C ta l
nt
9( ck i n (C 5) co fe H ; m ct C pl i o
50
)*
8( e n 1 x, s ; p, w i n 9 hi cr q) ch e ma
Pa g e 1 5 8 8
ABC Ambe r CHM Conve rte r Tria l ve rsion, http://w w w .proce sste x t.com/a bcchm.html
e s di e at d e s s u b sc ac e t e pt ri i b ol i l i ys t y is to s o m e vi ru s e s; rh e u m at ic di s or d er Pr X- I R Al o l i m ec t e p n p ur rn
Pa g e 1 5 8 9
ABC Ambe r CHM Conve rte r Tria l ve rsion, http://w w w .proce sste x t.com/a bcchm.html
er k ai re at di e re nt e n d d in p (P re o fe at ) ce p ct h ss s io w iv o n a e/ ni s y X z d h p at u e 1 io e m 1. n t o ol 2
p yt
3
y ic
â
o co
€“
g m
2
e pl
1.
ni e
1
c m or e g nt a ti ni t e s r m (A s P a H n d fu l m in a nt
50
); pr o p er di n
Pa g e 1 5 9 0
ABC Ambe r CHM Conve rte r Tria l ve rsion, http://w w w .proce sste x t.com/a bcchm.html
m le e v ni el n g oc oc ce m ia SLE, s ys t emi c l upus eryt hemat o s us . *Fol l owed by meas ureme nt of s peci fi c compl emen t component l evel s . P.327
TABLE 7-5 Disorders of Phagocyte Function
Pa g e 1 5 9 1
ABC Ambe r CHM Conve rte r Tria l ve rsion, http://w w w .proce sste x t.com/a bcchm.html
I n h e ri ta n P c h e e / n C ot h y Cl r pi in o c ic m E al Di o x F s s p e o o r at r m e u Dia d e ss r gn e Si io e osi r te n s s C U A O NB hr s b n T o u n s t es ni al or et t , c l y m of con gr X- al s y fi r a l i N m me n n A pt d ul k D o by o e P m che m d; H s mi l at m o b um o a xi y i ne
Pa g e 1 5 9 2
ABC Ambe r CHM Conve rte r Tria l ve rsion, http://w w w .proce sste x t.com/a bcchm.html
u y d a s ce s b a g nce di e s e an s in e 1 d e h fu y me a er nc e as u s i t t i ar re e e o ; me d n gr nt s a re a of s s n s up a ul ul ero ut t i o xi d o n m e, s g a ani o i n s on, m a (e an al b .g d H re s ., 2 O 2 ce e h . s s nc e (Pr i v e p en e of at at a or re i c l fo s , di a ur pi p gn ch ra ul os i ro t o m s m ry o i s o b n ma s ur ar de o s t y, by m b ly sa e y m mp s: st p lin
Pa g e 1 5 9 3
ABC Ambe r CHM Conve rte r Tria l ve rsion, http://w w w .proce sste x t.com/a bcchm.html
1 i
h g
q m n fet 2 ul o al 5, at d bl o 7 e e od. q d s) ) 1 p 1. h 2 a 3, g 1 oc 6 yt q e 2 s 4, X p 2 H C A I Fr Pre h ut m e s en à o p q ce © s ai u of di o re e gi a a m d nt nt k- al ch vi l ys Hi re e ra os o g ce m l
ma
a s s ot a l s i v a n gra hi e/ xi d nul s y 1 s , e es n q i n nt i n dr
t r er gra
o
ac i c nul
Pa g e 1 5 9 4
ABC Ambe r CHM Conve rte r Tria l ve rsion, http://w w w .proce sste x t.com/a bcchm.html
m
el b ocy
e
l u ac t es la te , r ri an b al d ac i n me t e fe l an ri ct os o ol i o me ys n s is s; in , p hai a ar r n t i s ha d al ft s . m oc (Pr o ul en v ar at a e a l m n di a e d gn nt cu os i of t a s n n is e e ma ut o de ro u by p s sa hi al mp l s bi l i n fr ni g o s fet m m al b ; bl o o p od.
Pa g e 1 5 9 5
ABC Ambe r CHM Conve rte r Tria l ve rsion, http://w w w .proce sste x t.com/a bcchm.html
n y ) e o md ar er ro m w a; m il d n e ut ro p e ni a; bl e e di n g te n d e nc y L A R D Sus e ut e el t ai u o d a ne k s uc y d oc o e e gra yt m d d nul
Pa g e 1 5 9 6
ABC Ambe r CHM Conve rte r Tria l ve rsion, http://w w w .proce sste x t.com/a bcchm.html
e al n u ocy a re e m t os d ce ut bi i s ; h s s ro l i dec e i v p ca rea s i e/ hi l o 2 l
s ed
co exp
n 1 ch rd res d q e s sio ef 2 m e n i c 2. ot p of i e 3 a ar CD nc
xi at 18
y
s i o on
(s
a n, pol
ur
n wi ys
fa
d t h by
ce
s s fl o
gl
pr u w
yc
e b cyt
o
a s om
pr
di e et r
ot
n q y
ei
g u
n)
e nt o m p h al iti s; co ld
Pa g e 1 5 9 7
ABC Amber CHM Converter Trial version, http://www.processtext.com/abcchm.html
sk in a b sc e ss e s; n ec ro ti zi n g d e e p s of t ti ss u e a b sc e ss e s NADPH,
Page 1598
ABC Ambe r CHM Conve rte r Tria l ve rsion, http://w w w .proce sste x t.com/a bcchm.html
reduced form of ni cot i na mi de– adeni ne di nucl eo t i de phos pha t e; NBT, ni t robl ue t et raz ol i um.
C. Antibody deficiency syndromes Antibody deficiency syndromes are charact eri zed by an i nabi l i t y t o produce ant i gen-s peci fi c ant i bodi es . Thes e di s orders have a vari et y of caus es , i ncl udi ng abnormal i t i es of t he s t ruct ural genes for t he heavy chai n of IgA, congeni t al i nfect i ons , and medi cat i ons . Tabl e 7-6 l i s t s t he mos t common humoral i mmunodefi ci ency di s orders .
1. Common clinical features. Mos t pat i ent s wi t h t hes e di s orders pres ent wi t h recurrent i nfect i ons caus ed by encaps ul at ed bact eri a s uch as St rept oc oc c us pyogenes , St rept oc oc c us pneumoni ae, and H. i nfl uenzae. Thes e pat i ent s have pri mari l y s i nopul monary i nfect i ons .
2. Diagnosis. Di agnos i s i s bas ed on: o
o
a. Quant i t at i ve determination of serum immunoglobulin levels—pri nci pal l y IgG, IgA, and IgM
o
Pa g e 1 5 9 9
ABC Ambe r CHM Conve rte r Tria l ve rsion, http://w w w .proce sste x t.com/a bcchm.html o
b. Functional immunoglobulin assays
(1) Det ermi nat i on of ant i bodi es t o naturally occurring immunogens (e.g., i s ohemaggl ut i ni ns , and ant i -A and ant i -B IgM ant i bodi es found i n i ndi vi dual s wi t h A, B, or O bl ood t ypes )
(2) Meas urement of ant i bodi es t o previously encountered antigens (e.g., St rept oc oc c us s peci es , vari cel l a vi rus , i nfl uenz a vi rus , or Eps t ei n-Barr vi rus )
(3) Det ermi nat i on of antibodies to prior immunizations (e.g., IgG1 ant i bodi es t o t et anus and di pht heri a t oxi ns or IgG1 ant i bodi es t o t he carbohydrat e caps ul e of H. i nfl uenzae and pneumococcus )
(4) As s es s ment of antibody production to newly introduced antigens (e.g., magni t ude of de novo res pons e 3–4 weeks aft er admi ni s t rat i on of t et anus t oxoi d or pneumococcal vacci nes )
3. T herapy o
o
a. The t reat ment of choi ce for mos t ant i body defi ci ency s yndromes i s replacement therapy wi t h i nt ravenous i mmunogl obul i n.
o
Pa g e 1 6 0 0
ABC Ambe r CHM Conve rte r Tria l ve rsion, http://w w w .proce sste x t.com/a bcchm.html
o
b. Pat i ent s wi t h s el ect i ve IgA defi ci ency are t reat ed as needed wi t h appropriate antibiotics for bact eri al i nfect i ons . Immunogl obul i n repl acement t herapy i s l i mi t ed t o t hos e i ndi vi dual s wi t h accompanyi ng IgG s ubcl as s defi ci ency.
P.328
TABLE 7-6 Humoral Immunodeficiency Disorders I n h e ri ta n P c h e e / n C ot h y Cl r pi in o c ic m E al Di o x F Di s s p e a o o r at g r m e u n d e ss r o e Si io e si r te n s s
Pa g e 1 6 0 1
ABC Ambe r CHM Conve rte r Tria l ve rsion, http://w w w .proce sste x t.com/a bcchm.html
X- X- D R T Li l i ys ec ot n n fu ur al k k nc re i e e t i nt m d d/ o s m a X n e u g q al v n a 2 g er o m 1. e e gl m 3 n in o a â e fe b gl €“ pr ct ul o 2 o io in b 2 d n < ul
uc s 1
in
t: d 0
e
[c u 0
m
yt e m
ia
o t o g/
(B
pl p d
ru
a y L;
to
s o s
n'
mg e
s)
ic e v t y ni er ro c el s i or y n g re e a d ki ni uc n s e a md s s , Ig e s t G,
Pa g e 1 6 0 2
ABC Ambe r CHM Conve rte r Tria l ve rsion, http://w w w .proce sste x t.com/a bcchm.html
(B ar Ig tk ti M )] n , g a at n 9 d â Ig €“ A 1 le 2 v m el o s; nt m h ar s k of e a d g d e; ec ch re ro a ni s c e m in e m ni at n ur g e o- B e ce nc l l e s p b h e al ar
Pa g e 1 6 0 3
ABC Ambe r CHM Conve rte r Tria l ve rsion, http://w w w .proce sste x t.com/a bcchm.html
iti in s g d s u ur e fa t o ce e i nt m er m o u vi n ru o s ; gl a o ut b oi ul m in m; u m n ar e k ar e th d ri d t i ec s re a s e in s ur fa ce re
Pa g e 1 6 0 4
ABC Ambe r CHM Conve rte r Tria l ve rsion, http://w w w .proce sste x t.com/a bcchm.html
ce pt or C 1 9, C 2 0, a n d C 2 1 ce ll s; a b s e nc e of pl a s m a ce ll s in ly
Pa g e 1 6 0 5
ABC Ambe r CHM Conve rte r Tria l ve rsion, http://w w w .proce sste x t.com/a bcchm.html
m p h oi d ti ss u e s L U A L R at n b e e e- k n s s d o n or s uc n o me e s w al v d et n B- er Ig h p ce e G, y at l l i n Ig p t e t e fe M o rn r ct , g ; m io a a e in n n m q al s d m u di d Ig a al ff u A gl s er e l e o e e to v b x nt p el ul di i a y s i n s t t i o (t e ri o g ot m b n e al i a ut t o ni i (a i o pl c m
Pa g e 1 6 0 6
ABC Ambe r CHM Conve rte r Tria l ve rsion, http://w w w .proce sste x t.com/a bcchm.html
l s n; a or m o hi s g u k g ma n n h a ni o o p ce s gl w er l l m o n ce s . s , b a nt H s t ul s a al ar i n co g f t i m m e of n a m of p g y o fi r at b b n s t i e et e v -d nt w < ar e s e 1 i a gr al e 0 bl e s n 0 e e o a m i
re h g g/
m la a e d m t i v s L) u v e 2 ; n e a 0 p o s b a o d wi n n s s ef t h or d i b ic s m 4 le i e el al 0 s l nc ec T- y i g y) t i ce e ht v l l ar d e ac s ; ec Ig t i gi re A v ar a
Pa g e 1 6 0 7
ABC Ambe r CHM Conve rte r Tria l ve rsion, http://w w w .proce sste x t.com/a bcchm.html
d at di s ef i o a e ic n si in ie a s; B nc n s ce y/ d pr l l li d u s k ec e- b el re l i e y a k ar s s e in u e syg sc d n s e IF dr ur pt N o fa i b -Î m ce i l i ³ e; i ty
n m
g
o m
e
d u
n
ul n
e
ar o
in
l y gl
cl
mo
a
p b
ss
h ul
II
oi i n
I
d ;
re
h i
gi
y m
o
p p
n
er ai
of
pl re
M
a d
H
si a
Pa g e 1 6 0 8
ABC Ambe r CHM Conve rte r Tria l ve rsion, http://w w w .proce sste x t.com/a bcchm.html
C
a; nt ly ib mo p d h y or re et s ic p ul o ar n ms al e ig s n a nc y; p er ni ci o u s a n e m ia ; la te d e v
Pa g e 1 6 0 9
ABC Ambe r CHM Conve rte r Tria l ve rsion, http://w w w .proce sste x t.com/a bcchm.html
el o p m e nt of a ut oi m m u n e di s or d er s S U Ar R R el n re ec e ec k s t ur d t i n e re uc v o d nt e e w m re d Ig n at s s A p ur pi er d at at ra u ef t e i o t o m i c rn n ry Ig i e ; of or A nc af Ig g l e y fe A- a v
Pa g e 1 6 1 0
ABC Ambe r CHM Conve rte r Tria l ve rsion, http://w w w .proce sste x t.com/a bcchm.html
ct b s t el s e ro s 1 ar i n ( in in te < 4 g st 5 0 ce i n m 0 l l al g/ â s; b d €“ t e ac L) 1 r te ; 0 m ri re 0 i n al d 0 al i n uc p bl fe e e oc ct d o k i o Ig pl i n n G e B- s ; 2 ( ce a or m l l ut Ig o di oi G s t ff m 4 co er m l e me u v m nt n el o ia e s n ti a in i
o n 2
mn d 0 m t o al % u pl l e of n a rg ca o s ic s d m di e ef a s s ;
Pa g e 1 6 1 1
ABC Ambe r CHM Conve rte r Tria l ve rsion, http://w w w .proce sste x t.com/a bcchm.html
i c ce or oc i e l l d ca nc s er s i y ca s ; o di p n n s a o al or bl s y m d e m il er of pt d )/ s o a s ec m b u re s n s c t i i n or e n 6 m pt g 0 al i b Ig % i t i ili A â e ty
€“ s
g
6 of
e
5 T-
n
% ce
e
of l l
in
ca fu
th
s nc
e
e ti
M
s; o
H
tr n
C
a
cl
n
a
si
ss
e
II
nt
I
d
re
ef
gi
ic
Pa g e 1 6 1 2
ABC Ambe r CHM Conve rte r Tria l ve rsion, http://w w w .proce sste x t.com/a bcchm.html
o
ie
n
nc
o
y
n
m
ch
a
ro
y
m
b
o
e
s
s
o
e
m
e
e
n
6
wi th ce rt ai n m e di ca ti o n s (d ip h e n yl h y d
Pa g e 1 6 1 3
ABC Ambe r CHM Conve rte r Tria l ve rsion, http://w w w .proce sste x t.com/a bcchm.html
a nt oi n, D -p e ni ci ll a m in e) C a n b e a ss oc ia te d wi th C el ia c Di s e a
Pa g e 1 6 1 4
ABC Ambe r CHM Conve rte r Tria l ve rsion, http://w w w .proce sste x t.com/a bcchm.html
s e IFN-γ, i nt erferonγ, MHC, major hi s t ocompa t i bi l i t y compl ex.
D. Cellular immunodeficiencies Cellular immunodeficiencies i ncl ude t hos e l i mi t ed t o defect s i n T-cel l funct i on, as wel l as combi ned di s orders affect i ng bot h cel l ul ar and humoral i mmuni t y (Tabl e 7-7).
1. Common clinical features. T-cel l defi ci enci es , whet her part i al or compl et e, t ypi cal l y res ul t i n recurrent i nfect i ons of great er s everi t y t han t hos e obs erved i n t he ant i body defi ci ency s yndromes . P.329
TABLE 7-7 T-Lymphocyte–Related Immunodeficiency Disorders
Pa g e 1 6 1 5
ABC Ambe r CHM Conve rte r Tria l ve rsion, http://w w w .proce sste x t.com/a bcchm.html
I n h e ri ta n P c h e e / n C ot h y Cl r pi in o c ic m E al Di o x F Di T s s p e a h o o r at g e r m e u n r d e ss r o a e Si io e si p r te n s s y Di D D A H B G el e n y M e et v o p T; or i o el m oc fe g n o al al t a e 2 p i e ce l sy2 m s m th n q e of i a y dr 1 nt t h ; m o 1; al e t o i c m s fi ca t a i e p el rd l
m
or d i a s pl
Pa g e 1 6 1 6
ABC Ambe r CHM Conve rte r Tria l ve rsion, http://w w w .proce sste x t.com/a bcchm.html
a d c er a di ef o u nt c ec ut m at oc t fl i
io
cu af o m n rr fe w m (s e ct t r u h nc i n ac n or e g t; o ts t n gl l i ru e o v ct o b e ur n ul d e at i n i s al u m of t e s pr th ta u o e n al v t h y; l y e ir h n m d y or e a p m nt n o al ) d pl ; fo a p ur s t o th ic ss p m ib h a le ar n fa y di i l n bl ur g e; e e h to al y m
Pa g e 1 6 1 7
ABC Ambe r CHM Conve rte r Tria l ve rsion, http://w w w .proce sste x t.com/a bcchm.html
p p a o er k uc t e e h lo s e ri p s , s ec l e m i fi a ; c di s a n h nt g or i b to t o th p d y hi y m l t af i c ru t e , mr p ; i ar l o m at w m h -g u yr ra ni oi d z d, e at a or i o n o n; d p d co p ec n or re ot t u a ru ni s nc s t e al i c d ca i n T rd fe ce
Pa g e 1 6 1 8
ABC Ambe r CHM Conve rte r Tria l ve rsion, http://w w w .proce sste x t.com/a bcchm.html
i a ct l l c io s; d n m ef s i t ec (o o t s cc g a e s i no st n i al m or ul re at cu e rr d e pr nt ol ) if er at io n m a y b e n or m al , d ec re
Pa g e 1 6 1 9
ABC Ambe r CHM Conve rte r Tria l ve rsion, http://w w w .proce sste x t.com/a bcchm.html
a s e d, or a b s e nt , d e p e n di n g o n d e gr e e of th y m ic d ef ic ie
Pa g e 1 6 2 0
ABC Ambe r CHM Conve rte r Tria l ve rsion, http://w w w .proce sste x t.com/a bcchm.html
nc y S Cl A S Pr B e a b ki of M v s s s n o T; er i c e i n u i n e al nc fe n t r co t y e ct d a m p of i o l y v bi e: al n m e n X- l
s, p n
e li a s h o d n d e o u i
k a p p s
m e pt s i e i m d i v s , ni m u re e p a; m n ce i
n cu u
o ss m e ta n d iv m u n o ef e/ u m e gl ic X n o o o i e q e ni u b nc 1 m a, s ul y 3. ec a a i n s y 1 h n n re n â a d er pl dr €“ ni di g ac o 1 s ar y; e m 3. m rh h m e 3 s ; e et e O v a, er nt th e st o ; er s t ar g e ty ig ti e x
Pa g e 1 6 2 1
ABC Ambe r CHM Conve rte r Tria l ve rsion, http://w w w .proce sste x t.com/a bcchm.html
p ia n n p e: l
g e er
a th b o i ut y y u m o ma s e s u g a nt o s ; e b al m fe 3 n g al w m or e re t h o m n ce y nt al e ss m h iti th i v oc s ; e er e/ yt m s a 2 e ar of p 0 s k ly y q or e m t r 1 H d p ia 3, a fa h l 2 s s i l oc i n q al ur yt pr 1 l' e e o 2, s t o s gr 1 co t h u e 4 rp ri b s s q u v p 1 s c e; o 3. l e s p 1 s e ul Cl v at a er i o ss e n ic o s; al p i ty p m
Pa g e 1 6 2 2
ABC Ambe r CHM Conve rte r Tria l ve rsion, http://w w w .proce sste x t.com/a bcchm.html
p or p e: t u ai M ni re ut s t d at i c i n i o i n vi n fe t r of ct o th io ly e n m γ s p ch (e h ai .g oc n ., yt of P e t h n pr e e ol IL u i f -2 m er re o at ce c y i v pt s t e or i s re O , s th C p er a o ty n n p di s e: d e; A a, s D v er A ar u (Z i c m A el i P- l a m
Pa g e 1 6 2 3
ABC Ambe r CHM Conve rte r Tria l ve rsion, http://w w w .proce sste x t.com/a bcchm.html
7 ); m 0 h u a y n n p o d o gl P pl o N a b P s t ul d ic in ef p l e i c er v i e i p el n h s cy er s ) al e ly v m er p el h y oi d d e t i pr ss e u ss e; e ch d o wi n th dr n o o d re ys s pl p a o si n
Pa g e 1 6 2 4
ABC Ambe r CHM Conve rte r Tria l ve rsion, http://w w w .proce sste x t.com/a bcchm.html
a; s p e o to ss s ib p i l i ec t y i fi of c d a e nt at i g h e b n y s a g e 2 y e ar s if u nt re at e d W X- Di Ec N E i s l i s z or ar k n or e m l y ot k g m al B t - e a a; Ig M Al d ni m G T
Pa g e 1 6 2 5
ABC Ambe r CHM Conve rte r Tria l ve rsion, http://w w w .proce sste x t.com/a bcchm.html
dr re z i c l e S i c ce at ro v pl h s s i o cy el e syiv n ti s; n n e/ of c el ec dr X ac t h e t o o p t i ro v m m 1 n m at y e 1. cy b e i n 2 t o oc d p 2 s k yt Ig at â el o A i e €“ et p l e nt 1 o e v s 1. n ni el wi 3 a a; s ; t h n re s l bl d cu i g e l a rr ht e ck e l y di of nt re n mp d g e y uc di mo e s br g d or a e Ig d n ni M er e- c l e s a in v s s fe el oc ct s ; ia io i te n m d s, p gl s t ai
Pa g e 1 6 2 6
ABC Ambe r CHM Conve rte r Tria l ve rsion, http://w w w .proce sste x t.com/a bcchm.html
yc ar re o ti d pr n h ot g u ei b m n y or s ; a al ac g re ce e s le 1 p ra y o te e n d ar s sy; e nt P t o h n p e e ol s i u ys s m ac a o ch n c y ar d st id ca i s e ta a a b n nt ol d i g is h e m er n of p s ; al e m l
svil
i
i r d-
m u to ms u in m
Pa g e 1 6 2 7
ABC Ambe r CHM Conve rte r Tria l ve rsion, http://w w w .proce sste x t.com/a bcchm.html
n fe o o ct d gl i o er o n at b s, e ul s t d i n ar e i s t i pr ot n e y g ss p in io e la n s ; t e of i n ch l y tr il m in d p si h h c o oc pl o yt at d; e el m pr et al ol a ig if b n er n a at or nc i v my e al (l re it y s y mp p o h n o s m e; a) m
Pa g e 1 6 2 8
ABC Ambe r CHM Conve rte r Tria l ve rsion, http://w w w .proce sste x t.com/a bcchm.html
i n ar 1 k 5 e %d of l y ca m s p e h s o p e ni a b y 6 y e ar s of a g e At A H Pr A In a ut y o b t r xi o p gr s a a s o e e v â o pl s s nt e €“ m a i v Ig n t e al s t e A o l a re i c ce i n u n ce t h re 7 s gi s s y b 0 g ec i v m el % a
Pa g e 1 6 2 9
ABC Ambe r CHM Conve rte r Tria l ve rsion, http://w w w .proce sste x t.com/a bcchm.html
t a e/ u l a of m s i 1 s ; r ca m a 1 l a at s a q ck a e gl 2 of xi s ; o 2. H a; Ig b 2 a oc G ul 3 s s ul 2 i n al oc s i n l ' ut u Ig s a bc G co n l a 2 rp e s s d u o d ef s c u ef i c le s ic ie s ; t e i e nc i n l a nc y; tr n y p i n gi i n o s i ec 3 s s c ta 5 ib C si % le D a; of g 4 re ca e a cu s n n rr e e d e s; th B- nt l o er ce s i w a ll n â p d o €“ y ef p m (f ec ul ol ut t s m ec ur
Pa g e 1 6 3 0
ABC Ambe r CHM Conve rte r Tria l ve rsion, http://w w w .proce sste x t.com/a bcchm.html
; o ul e) T- n ar . ce ar - Li ll y w m ch i n ei i t ro fe g e m ct ht x o i o Ig p s n M o o s; ; s m i n m ur al cr i l e tr e d to a a d Xr n s e a s l e pr y. oc d e at i n s s i o ci i o n d n af e of t e nc m r e it x- of o ra m g y al e i rr i g na n st di a i at nc m i o y ul n
at e d pr
Pa g e 1 6 3 1
ABC Ambe r CHM Conve rte r Tria l ve rsion, http://w w w .proce sste x t.com/a bcchm.html
ol if er at iv e re s p o n s e; s el ec ti v e d ec re a s e in C D 4 ce ll s; p er si
Pa g e 1 6 3 2
ABC Ambe r CHM Conve rte r Tria l ve rsion, http://w w w .proce sste x t.com/a bcchm.html
st e nt ly el e v at e d s er u m A F P ADA, adenos i ne deami nas e; AFP, α-fet oprot ei n; BMT, bone marrow t rans pl ant at i on; IL-2, i nt erl euki n-2; PNP, puri ne nucl eos i de phos phoryl as e. P.330
Pa g e 1 6 3 3
ABC Ambe r CHM Conve rte r Tria l ve rsion, http://w w w .proce sste x t.com/a bcchm.html
2. Diagnosis. A demons t rabl e decreas e i n t he number or funct i on of T cel l s i s t he bas i s of di agnos i s . Speci fi c T-cel l as s ays i ncl ude: o
o
a. Quantitative assessment of circulating lymphocytes. In normal i ndi vi dual s , 80%–85% of t he ci rcul at i ng l ymphocyt es are T cel l s . Val ues 3
bel ow 1500 mm are cons i dered abnormal . Immunofluorescence labeling wi t h T-cel l –s peci fi c monocl onal ant i bodi es i dent i fi es T-cel l s ubs et s (e.g., ant i -CD4 l abel s Th cel l s , and ant i -CD8 l abel s T-cel l popul at i ons ). o
o
b. Delayed hypersensitivity skin tests to previously encountered common antigens (e.g., Candi da al bi c ans , T ri c hophyt on, s t rept oki nas e–s t rept odornas e, mumps vi rus ). Pos i t i ve react i ons t o at l eas t t wo ant i gens i ndi cat e i nt act cel l ul ar i mmuni t y. The C. al bi c ans react i on can be bl unt ed or abs ent i n i mmunol ogi cal l y normal pat i ent s wi t h eczema.
o
o
c. Determination of in vitro responsiveness of cul t ured T cel l s t o specific antigens, mitogens (e.g., pokeweed, concanaval i n A, or hemaggl ut i ni n), or foreign cells
E. HIV HIV s el ect i vel y i nfect s Th cel l s , l eadi ng t o progres s i ve det eri orat i on of t he i mmune s ys t em and, ul t i mat el y, acquired immunodeficiency syndrome (AIDS). For a det ai l ed di s cus s i on of AIDS, refer t o Chapt er 8 VIII. Acquired immunodeficiency may al s o be s econdary t o mal nut ri t i on, di abet es , nephrot i c s yndrome, or
Pa g e 1 6 3 4
ABC Ambe r CHM Conve rte r Tria l ve rsion, http://w w w .proce sste x t.com/a bcchm.html
mal i gnancy/chemot herapy. P.331
Study Questions/Answers and Explanations 1. A 12-year-old boy is playing in the park near a trash can. He is stung by what he believes is a yellow jacket. He immediately has symptoms of urticaria and wheezing. T hese symptoms are treated in a local emergency department. He follows up in his primary care provider' s office 1 week later. T here is a question of the proper diagnosis of insect allergy. Which of the following findings provides the best evidence of insect sting hypersensitivity? A Pos i t i ve pri ck or i nt radermal s ki n t es t B Ext ens i ve l ocal react i on l as t i ng 5–7 days C Document ed evi dence of a s ys t emi c al l ergi c react i on D Speci fi c ant i venom i mmunogl obul i n E (IgE) ant i bodi es E General i zed urt i cari a i n t hi s pat i ent . Vi ew Ans wer 1. T he answer is C [VI C 1–2]. Cl i ni cal document at i on of a s ys t emi c al l ergi c react i on i s es s ent i al t o t he accurat e di agnos i s of i ns ect s t i ng hypers ens i t i vi t y. Typi cal l y, s i gns of anaphyl axi s devel op wi t hi n mi nut es aft er t he s t i ng. Subs equent pri ck or i nt radermal s ki n t es t i ng, s uppl ement ed as neces s ary by ant i venom i mmunogl obul i n E (IgE) l evel s [det ermi ned by radi oal l ergos orbent t es t (RAST)], provi des confi rmi ng evi dence of hypers ens i t i vi t y. A l arge l ocal react i on al one, whet her of s hort or prol onged durat i on, does not provi de s uffi ci ent evi dence of IgE-medi at ed hypers ens i t i vi t y. General i zed urt i cari a i n a chi l d younger t han 16 years of age i s not cons i dered anaphyl axi s and t hes e chi l dren do not have a great er ri s k t han t he general popul at i on of an anaphyl act i c react i on on t hei r next s t i ng. The onl y object i ve t es t i s a s erum β t rypt as e. Thi s t es t meas ures t rypt as e rel eas ed from mas t cel l s . W hen el evat ed, i t i s us eful , but i t i s not el evat ed i n al l anaphyl act i c react i ons .
Pa g e 1 6 3 5
ABC Ambe r CHM Conve rte r Tria l ve rsion, http://w w w .proce sste x t.com/a bcchm.html
2. A 25-year-old man presents to his primary care office with the following symptoms. He has clear rhinorrhea, ocular and nasal pruritus, nasal congestion, and sneezing. He knows that it is tree pollen season but has also recently acquired a pet cat for the first time. Which of the following tests is most useful in the diagnosis of the trigger for his allergic rhinitis? A Envi ronment al cat chal l enge B Meas urement of t ot al s erum i mmunogl obul i n E (IgE) l evel s C Peri pheral bl ood s mear for eos i nophi l s D Immedi at e hypers ens i t i vi t y punct ure s ki n t es t E St ai ned nas al s mear for eos i nophi l s Vi ew Ans wer 2. T he answer is D [III E 1]. An accurat el y appl i ed s ki n t es t i s t he mos t val uabl e t ool for i dent i fyi ng t he caus at i ve ant i gen i n al l ergi c rhi ni t i s , yi el di ng res ul t s i n approxi mat el y 15–20 mi nut es . Envi ronment al chal l enge t es t s are nei t her pract i cal nor avai l abl e t o mos t pat i ent s and may l ead t o s evere s ympt oms . Radi oal l ergos orbent t es t (RAST) has al mos t equal accuracy but i s more expens i ve. El evat ed i mmunogl obul i n E (IgE) l evel s are obs erved i n onl y 30%–40% of pat i ent s wi t h al l ergi c rhi ni t i s and may be s econdary t o ot her unrel at ed di s orders . Peri pheral eos i nophi l i a s een i n a peri pheral bl ood s mear i s an i ncons i s t ent fi ndi ng. Al t hough eos i nophi l s are us ual l y i dent i fi ed i n nas al s ecret i ons from pat i ent s wi t h al l ergi c rhi ni t i s , t hey are al s o det ect ed i n eos i nophi l i c nonal l ergi c rhi ni t i s and hyperpl as t i c s i nus i t i s . 3. A 4-year-old boy in your practice has been having many infections. You suspect that he may have an immune dysfunction. Quantitative immunoglobulins were sent and were normal. Which of the following immunodeficiency disorders is associated with normal immunoglobulin G (IgG) levels? A X-Li nked agammagl obul i nemi a (Brut on's ) B Di George s yndrome C Lat e-ons et hypogammagl obul i nemi a D At axi a-t el angi ect as i a
Pa g e 1 6 3 6
ABC Ambe r CHM Conve rte r Tria l ve rsion, http://w w w .proce sste x t.com/a bcchm.html
E Severe combi ned i mmunodefi ci ency Vi ew Ans wer 3. T he answer is B [Tabl es 7-6 and 7-7]. Al t hough reduced l evel s of i mmunogl obul i n A (IgA) or IgE may be s een i n pat i ent s wi t h Di George s yndrome, a T-cel l defi ci ency di s order, t he t ot al s erum i mmunogl obul i n l evel us ual l y i s normal and IgG l evel s are normal . In X-l i nked agammagl obul i nemi a (Brut on's ) and l at e-ons et hypogammagl obul i nemi a, IgG, IgM, and IgA l evel s are al l reduced, and t he t ot al i mmunogl obul i n l evel i s l es s t han 100 mg/dL. Pat i ent s wi t h at axi a-t el angi ect as i a have a defect i n t hei r DNA repai r mechani s m. One of t he cl i ni cal feat ures i s l ow IgA and IgG. Severe combi ned i mmunodefi ci ency res ul t s i n a decreas e i n al l i mmunogl obul i ns . 4. A 55-year-old woman comes into the emergency room complaining of a hoarse voice and states that, “ my mouth feels funny.― On history, she reports that she was diagnosed with hypertension several months ago. After an unsuccessful attempt at lifestyle modification to reduce her blood pressure, her doctor started her on some medication a few weeks ago, but she cannot remember the name of the medications. On exam, she has swollen lips and periorbital edema, and a definite hoarseness to the voice. What would be the most appropriate next step? A Di s cont i nue bl ood pres s ure medi cat i ons ; di s charge t o home, and advi s e her t o obt ai n al t ernat e medi cat i ons from her pri mary doct or B Recommend l orat adi ne and fol l ow-up wi t h her pri mary doct or wi t hi n one week t o eval uat e her s ympt oms C Al but erol and At rovent nebul i zer t herapy wi t h i nt ravenous s t eroi ds ; admi t t o medi ci ne fl oor for obs ervat i on D Subcut aneous epi nephri ne 0.5 mg 1:1000, i nt ravenous di phenhydrami ne, rani t i di ne, and s t eroi ds ; admi t t o i nt ens i ve care uni t for pos s i bl e ai rway emergency E None of t he above Vi ew Ans wer 4. T he answer is D [IV]. Thi s pat i ent has angi oedema, mos t l i kel y
Pa g e 1 6 3 7
ABC Ambe r CHM Conve rte r Tria l ve rsion, http://w w w .proce sste x t.com/a bcchm.html
from an ACE i nhi bi t or pres cri bed recent l y t o cont rol her bl ood pres s ure. The s wol l en l i ps and peri orbi t al edema are cl as s i c s ympt oms of angi oedema. The hoars enes s of t he pat i ent 's voi ce i s es peci al l y concerni ng, s i nce i t s ugges t s l aryngeal edema, whi ch coul d l ead t o an ai rway emergency. Gi ven t he ai rway i nvol vement , epi nephri ne s houl d be gi ven, al t hough t hi s i s mos t l i kel y non-IgE medi at ed. In addi t i on, H 1 and H 2 bl ockers s houl d be admi ni s t ered, t he l at t er provi di ng a s ynergi s t i c benefi t i n i mprovi ng angi oedema. Int ravenous s t eroi ds s houl d be us ed i n s evere cas es , s uch as t hi s one. The pot ent i al ai rway emergency warrant s ext remel y cl os e obs ervat i on, and admi s s i on t o an i nt ens i ve care uni t woul d be mos t appropri at e. P.332
5. Which one of the following adverse reactions is mediated by immunoglobulin G (IgG) autoantibodies? A Ins ul i n res i s t ance B Anaphyl axi s aft er i nges t i on of peanut s C Lat ex react i on D Sys t emi c react i on t o an i ns ect s t i ng E Ragweed-i nduced rhi ni t i s Vi ew Ans wer 5. T he answer is A [VII D 3 b]. Ins ul i n res i s t ance i s caus ed by i mmunogl obul i n G (IgG) aut oant i bodi es t hat bi nd t o i ns ul i n recept ors , t hereby i nhi bi t i ng t hei r funct i on. Thi s probl em us ual l y occurs i n women wi t h Sjögren's s yndrome, s ys t emi c l upus eryt hemat os us (SLE), or a l es s wel l -defi ned cl i ni cal s yndrome al s o s ugges t i ve of aut oi mmuni t y. Food-rel at ed anaphyl axi s , react i ons t o l at ex, s ys t emi c react i ons t o i ns ect s t i ngs , and ragweed-i nduced rhi ni t i s are al l IgE-medi at ed hypers ens i t i vi t y res pons es . 6. A 33-year-old man has an acute anaphylactic reaction to an intravenous drug while in the hospital. He is taking a β-adrenergic blockade drug. Which of the following therapeutic choices may be most useful in treating resistant hypotension?
Pa g e 1 6 3 8
ABC Ambe r CHM Conve rte r Tria l ve rsion, http://w w w .proce sste x t.com/a bcchm.html
A Subcut aneous aqueous epi nephri ne B Int ravenous t erbut al i ne C Int ravenous gl ucagon D Int ravenous ami nophyl l i ne E Int ravenous di phenhydrami ne Vi ew Ans wer 6. T he answer is C [V F 5]. Int ravenous gl ucagon has a pos i t i ve i not ropi c and chronot ropi c effect . In t hos e pat i ent s t aki ng β-bl ocker medi cat i ons , an even great er i ncreas e i n bl ood pres s ure may occur. Al t hough s ubcut aneous epi nephri ne i s al ways t he fi rs t l i ne of t herapy, i t may not be effect i ve i n pat i ent s us i ng a β-adrenergi c bl ockade drug. The ot her t herapeut i c agent s are rel at i vel y i neffect i ve i n t he face of β-adrenergi c bl ockade. 7. A 28-year-old woman with bronchial asthma is about to start a new job in a health care facility. She hears that health care workers are at a greater risk for developing a latex allergy. She is wondering whether any of her medical conditions make her at a higher risk. Which of the following clinical conditions is most commonly associated with latex allergy? A Bronchi al as t hma B Fi bros i ng al veol i t i s C Di abet es mel l i t us D Spi na bi fi da E Infl ammat ory bowel di s eas e Vi ew Ans wer 7. T he answer is D [VII E 1 a]. The hi ghes t preval ence of l at ex al l ergy occurs i n chi l dren wi t h s pi na bi fi da. Thi s appears t o be rel at ed t o earl y s urgi cal i nt ervent i on wi t h cons t ant expos ure t o t he nat ural l at ex i n cat het ers and t ubi ng. Surgeons and operat i ng room nurs es are at i ncreas ed ri s k of devel opi ng l at ex al l ergy. Bronchi al as t hma, fi bros i ng al veol i t i s , di abet es mel l i t us , and i nfl ammat ory bowel di s eas e are not commonl y as s oci at ed wi t h t he occurrence of l at ex al l ergy. QUEST IONS 8–11 Directions: T he res pons e opt i ons for I t ems 8–11 are t he s ame.
Pa g e 1 6 3 9
ABC Ambe r CHM Conve rte r Tria l ve rsion, http://w w w .proce sste x t.com/a bcchm.html
Y ou w i l l be requi red t o s el ec t one ans w er for eac h i t em i n t he s et . A Ant i gen avoi dance/envi ronment al cont rol s B Immunot herapy agai ns t known offendi ng ant i gens C Ant i hi s t ami ne preparat i on D Topi cal nas al s t eroi d For each of t he fol l owi ng pat i ent s wi t h al l ergi c rhi ni t i s , s el ect t he mos t appropri at e t herapeut i c i nt ervent i on. 8. A 34-year-old-woman with severe perennial allergic rhinitis loses 10–15 days of work each year as a result of secondary sinusitis. Immediate skin tests show significant positive reactions to house dust mite, Cladosporium, and grass and ragweed pollens. Various medications have been only partially successful in controlling symptoms. 9. A 20-year-old man has mild but persistent year-round symptoms of nasal congestion, rhinorrhea, and watery eyes. He has recurrent epistaxis. Immediate skin tests show positive reactions to house dust mite, Alternaria, and grass and ragweed pollens. 10. A 10-year-old girl has rhinorrhea and nasal pruritus after visiting a friend who has a kitten. 11. An 8-year-old boy regularly experiences moderately severe sneezing spells and watery eyes in May and June, September and October, and occasionally during January through March. Immediate skin tests show positive reactions to Alternaria, Cladosporium, house dust mite, and grass and ragweed pollens. He has used antihistamines with some persistent symptoms. Vi ew Ans wer 8–11. T he answers are: 8—B [III F 3], 9—C [III F 2 a–c], 10—A [III F 1], 11—D [III F 2 d (1)]. Immunotherapy, which significantly ameliorates symptoms and is used when allergic rhinitis is severe, when medical complications or intolerance to medication occurs, and when various types of medications have been only partially successful, would be recommended for the 34-year-old woman. Intranasal steroids are the most effective form of therapy, but
Pa g e 1 6 4 0
ABC Ambe r CHM Conve rte r Tria l ve rsion, http://w w w .proce sste x t.com/a bcchm.html
not an option here. T herefore, antihistamine preparations are the drugs of choice for the treatment of allergic rhinitis. T hese preparations usually control mild-to-moderate symptoms such as those experienced by the 20-year-old man and the 8-year-old boy; however, some patients experience intolerable side effects such as drowsiness. A second-generation antihistamine such as fexofenadine is less likely to cause such sedation. If an antihistamine preparation does not relieve symptoms or is not well tolerated, then intranasal therapy with corticosteroids or an H 1 blocker could be considered for the 8-year-old boy. Nasal cromolyn sodium, which in some cases is as effective as a decongestant in treating seasonal allergic rhinitis, could have been recommended for the 8-year-old boy with seasonal symptoms. T opical nasal steroids are the most effective preventive treatment of allergic rhinitis if symptoms persist despite environmental controls and p.r.n. antihistamines. Antigen avoidance is most successful when a single antigen can be identified as the causative agent. Because the 10-year-old girl developed symptoms only after being exposed to her friend' s kitten, it is likely that avoiding cats will alleviate her symptoms. Antigen avoidance is not feasible for all patients, however. For example, although a patient may be able to avoid animal dander indoors, the avoidance of multiple outdoor antigens is likely to prove difficult, if not impossible. Nonetheless, sensible environmental precautions help limit exacerbation of symptoms. P.333
P.334
P.335
Pa g e 1 6 4 1
ABC Ambe r CHM Conve rte r Tria l ve rsion, http://w w w .proce sste x t.com/a bcchm.html
QUEST IONS 12–14 Directions: T he res pons e opt i ons for I t ems 12–14 are t he s ame. Y ou w i l l be requi red t o s el ec t one ans w er for eac h i t em i n t he s et . A Acut e art hri t i s B Acut e anaphyl axi s C Chroni c cough D Bronchos pas m E Int ers t i t i al nephri t i s F Phot oal l ergi c react i on For each of t he fol l owi ng pat i ent s exhi bi t i ng an advers e drug react i on, s el ect t he mos t l i kel y cl i ni cal expres s i on. 12. A 22-year-old man, who is in good health except for recurrent sinusitis, elects to take 81 mg aspirin as anti-thrombosis therapy. Vi ew Ans wer 12. T he answer is D [III E 2 f]. T his man, an individual with recurrent sinusitis, also may have underlying chronic rhinitis and perhaps undiagnosed nasal polyposis. Bronchospasm, which may be prolonged and severe, typically begins within 30 minutes after aspirin ingestion. T he combination of asthma, nasal polyposis, and sinusitis is known as the aspirin triad. However, aspirin-induced anaphylaxis does not occur in patients with underlying sinusitis or polyposis and starts after two or more exposures to aspirin. 13. A 54-year-old woman with newly diagnosed hypertension starts enalapril at an appropriate dose. Vi ew Ans wer 13. T he answer is C [VII E 3]. Chronic cough has been reported in 5%–20% of patients who take angiotensin-converting enzyme (ACE) inhibitors. T he cough is usually nonproductive and nocturnal. Symptoms may begin as early as 1 week or as late as 6 months after drug use. ACE inhibitors also lead to development of angioedema in 0.1%–0.2% of patients, and in certain clinical situations, this group of medications is known to exacerbate anaphylaxis.
Pa g e 1 6 4 2
ABC Ambe r CHM Conve rte r Tria l ve rsion, http://w w w .proce sste x t.com/a bcchm.html
14. A 34-year-old woman is diagnosed with a dental infection and is started on penicillin. She has a history of a severe allergic reaction to an unknown antibiotic for a sexually transmitted disease (ST D) in college. Vi ew Ans wer 14. T he answer is B [VII D 1]. Penicillin can cause anaphylaxis, which is an acute life-threatening hypersensitivity reaction. In the past, the diagnosis of a penicillin allergy could be by history and skin testing; however, penicillin skin test materials are not available other than Pen G. If alternative antibiotics cannot be used, the patient should undergo penicillin desensitization in a monitored unit. Cephalosporins should be used with extreme caution in such patients, as up to 2% of patients who are allergic to penicillin can have a cross-reaction.
Pa g e 1 6 4 3
ABC Ambe r CHM Conve rte r Tria l ve rsion, http://w w w .proce sste x t.com/a bcchm.html
Editors: Wolfsthal, Susan T itle: NMS Medicine, 6th Edition Copyri ght ©2008 Li ppi ncot t W i l l i ams & W i l ki ns > T able of Cont ent s > Chapt er 8 - Infec t ious Diseases
Chapter 8
Infectious Diseases Devang Patel Janaki Kuruppu
I. General Principles of Human–Microbe Interaction The normal human body harbors a compl ex mi crobi al ecos ys t em. General l y, t hes e commensal organi s ms (referred t o as t he indigenous flora) are cons i dered t o be nonpat hogeni c. Infect i on and di s eas e may res ul t , however, when t he body i s chal l enged by a known pat hogen or when t he body's defens e s ys t em i s di s t urbed, al l owi ng uncont rol l ed growt h or i nvas i on by t he i ndi genous fl ora.
A. Normal human–microbe ecology Mi croorgani s ms can i nt eract wi t h humans i n t he fol l owi ng ways :
1. Most indigenous organisms seldom cause disease. Many of t he organi s ms normal l y found on t he s ki n and mucous membranes (e.g., St aphyl oc oc c us epi dermi di s , Corynebac t eri um s peci es ) are ubi qui t ous but may caus e di s eas e i n unus ual s et t i ngs (e.g., when hos t defens es are s i gni fi cant l y i mpai red or when art i fi ci al mat eri al s uch as a cat het er or a pros t het i c joi nt i s pres ent ).
2. Some indigenous organisms may cause disease in other body sites. Many bact eri a t hat normal l y exi s t i n one body s i t e may caus e di s eas e el s ewhere. For exampl e, α-hemol yt i c (vi ri dans ) s t rept ococci are commens al s i n t he oropharynx but can caus e
Pa g e 1 6 4 4
ABC Ambe r CHM Conve rte r Tria l ve rsion, http://w w w .proce sste x t.com/a bcchm.html
endocardi t i s i f t hey are i nocul at ed i nt o t he bl ood and s et t l e on a previ ous l y damaged heart val ve.
3. T ransient organisms may cause disease. The body's normal fl ora may al l ow t he t emporary growt h of cert ai n mi crobes , whi ch di s appear s pont aneous l y but may caus e di s eas e whi l e pres ent . For exampl e, pat i ent s wi t h i nvas i ve meni ngococcal di s eas e fi rs t have pharyngeal carri age of Nei s s eri a meni ngi t i di s , but onl y a t i ny fract i on of i ndi vi dual s wi t h meni ngococci i n t he pharynx ever devel op s ys t emi c di s eas e.
4. Pathogenic organisms usually cause disease. Mos t vi rus es , as wel l as Chl amydi a and Ri c ket t s i a, rarel y are i s ol at ed from humans except duri ng or fol l owi ng an acut e i l l nes s . Some pat hogeni c bact eri a i ncl ude Bruc el l a and Sal monel l a s peci es , Nei s s eri a gonorrhoeae, and Myc obac t eri um t uberc ul os i s .
B. Host defense mechanisms The human body has many ways of defendi ng i t s el f from pot ent i al l y pat hogeni c mi croorgani s ms .
1. Anatomic barriers are i nt egral i n prevent i ng i nfect i on. Thes e i ncl ude phys i cal barri ers s uch as i nt act s ki n and mucous membranes , as wel l as funct i onal barri ers s uch as t he mus cul ar prot ect i on of t he gl ot t i s and bl adder neck.
2. Cellular immunity i s expl ai ned i n Tabl e 8-1.
3. Humoral immunity i s expl ai ned i n Tabl e 8-1.
Pa g e 1 6 4 5
ABC Ambe r CHM Conve rte r Tria l ve rsion, http://w w w .proce sste x t.com/a bcchm.html
C. Fever One of t he mos t recogni zed feat ures of i nfect i ous di s eas es , regardl es s of t he s i t e of i nfect i on, i s t he el evat i on of body t emperat ure.
1. Normal regulation of body temperature. The normal core body t emperat ure i s approxi mat el y 37°C (±1°). Monocyt es s ecret e a pol ypept i de cal l ed interleukin-1 (IL-1), whi ch s t i mul at es t he hypot hal amus t o i ncreas e t he body's t emperat ure set point. The ri s e i n s et poi nt caus es al t erat i ons i n ci rcul at i on, met abol i s m, and pers pi rat i on, whi ch ul t i mat el y l ead t o a ri s e i n body t emperat ure. Temperat ure readi ngs above 38.3°C are cons i dered abnormal . Hypothermia (body t emperat ure °36°C) i s s omet i mes a res pons e t o overwhel mi ng i nfect i on, es peci al l y i n neonat es and t he el derl y.
TABLE 8-1 The Human Immune System Causes of
Opportunis
Diminished
tic
Component Source Function Function Cellular immunity Neut rophi l Bone Phagocyt os i s Genet i c marrow ; acut e
di s orders
i nfl ammat i on Chroni c
Organisms Endogenous fl ora; ent eri c
granul omat o baci l l i ; us di s eas e
Ps eudomona
Myel operoxi d s as e
aerugi nos a;
defi ci ency
Candi da
Acqui red
s peci es ;
caus es
As pergi l l us
Cyt ot oxi c
s peci es
t herapy
Pa g e 1 6 4 6
ABC Ambe r CHM Conve rte r Tria l ve rsion, http://w w w .proce sste x t.com/a bcchm.html
Leukemi a Apl as t i c anemi a Eos i nophi l
Bone
Modul at e
Drug react i on Idi opat hi c; None known
marrow hypers ens i t i cort i cos t eroi vi t y
d t herapy
react i ons t o mul t i cel l ul ar Monocyt e/macro Bone phage
paras i t es Rel eas e
marrow cyt oki nes ;
Cyt ot oxi c t herapy;
i nt eract wi t h l ymphoret i cul l ymphocyt es ar ; T l ymphocyt e
mal i gnancy
phagocyt os i s Thymus ; Modul at e Genet i c
Myc obac t eri
bone
um s peci es ;
act i vi t y of B di s orders ;
marrow l ymphocyt es , aut oi mmune fungi ; macrophages di s eas e; , and T
Li s t eri a
l ymphoret i cul s peci es ;
l ymphocyt es ar
Noc ardi a
mal i gnancy; s peci es AIDS; organ t rans pl ant at i on; cort i cos t eroi d t herapy Humoral immunity Ant i body Pl as ma Faci l i t at e cel l s
Compl ement
Li ver
Genet i c
Vi rus es ;
phagocyt os i s di s orders ;
pyogeni c
; i nact i vat e
mul t i pl e
bact eri a
t oxi ns
myel oma;
Enhance
s pl enect omy Genet i c Nei s s eri a
phagocyt os i s di s orders ;
s peci es
; di rect cel l
(es peci al l y
s evere l i ver
Pa g e 1 6 4 7
ABC Ambe r CHM Conve rte r Tria l ve rsion, http://w w w .proce sste x t.com/a bcchm.html
des t ruct i on
di s eas e
wi t h defi ci ency of compl ement component s (C5–C9)
AIDS, acqui red i mmunodefi ci ency s yndrome.
P.337
2. Conditions associated with fever. Al mos t any i nfect i ous proces s may be accompani ed by fever, al t hough t he abs ence of fever s houl d not excl ude t he cons i derat i on of an i nfect i on. Fever al s o may occur wi t h myocardi al i nfarct i on (MI), pul monary embol i s m, drug react i ons , aut oi mmune di s eas e, cancer (e.g., l ymphoma and renal cel l carci noma), or a vari et y of mi s cel l aneous i l l nes s es (e.g., i nfl ammat ory bowel di s eas e and s arcoi d). Al t hough cert ai n fever pat t erns are cl as s i cal l y as s oci at ed wi t h s peci fi c di s eas es (e.g. t ert i an fever wi t h Pl as modi um vi vax mal ari a) t he act ual t emperat ure pat t ern rarel y hel ps narrow t he range of et i ol ogi c fact ors t hat may be caus i ng t he fever.
3. Fever management. Antipyretic drugs [e.g., as pi ri n, acet ami nophen, nons t eroi dal ant i -i nfl ammat ory drugs (NSAIDs )] can modi fy a fever, but t hey s el dom compl et el y s uppres s a fever caus ed by i nfect i on. W hen fever i s ext reme (i .e., >42°C) or when t he accompanyi ng t achycardi a and ci rcul at ory changes are
Pa g e 1 6 4 8
ABC Ambe r CHM Conve rte r Tria l ve rsion, http://w w w .proce sste x t.com/a bcchm.html
poorl y t ol erat ed, i t i s wi s e t o t ry t o reduce body t emperat ure. However, i n mos t s et t i ngs , fever i s merel y uncomfort abl e.
4. Beneficial effects of fever. Fever does pl ay a rol e i n hos t defens e, becaus e many i nfect i ve mi crobes prefer normal body t emperat ure for opt i mal s urvi val and growt h. However, t he cl i ni cal s i gni fi cance of t hi s prot ect i on i s unknown. In addi t i on, s ome el ement s of t he i mmune s ys t em are more effi ci ent at hi gher t emperat ures , whereas ot hers are l es s effi ci ent . The net effect of fever on recovery from infect i on i s not known.
P.338
D. Microbial virulence factors Microbial virulence factors are i mport ant i n det ermi ni ng t he l i kel i hood of i nfect i on. To i nfect a hos t , mi croorgani s ms or t hei r product s mus t adhere t o hos t t i s s ue.
1. Some microbes invade host cells or breach barriers t o reach s us cept i bl e body s i t es and, al ong t he way, avoi d or overcome hos t defens es . In s ome s i t uat i ons (e.g., chi ckenpox or meas l es ), mi crobes can eas i l y i nfect ot herwi s e heal t hy i ndi vi dual s who have normal hos t defens es but no pri or expos ure t o t he mi crobe.
2. Some microbes have a special ability to produce a toxin or virulence factor t o caus e di s eas e. For exampl e, a t oxi n el aborat ed by s ome s t rai ns of St aphyl oc oc c us aureus [t oxi c s hock s yndrome t oxi n-1 (TSST-1)] i s res pons i bl e for toxic shock syndrome (TSS; s ee VII D 1 a).
Pa g e 1 6 4 9
ABC Ambe r CHM Conve rte r Tria l ve rsion, http://w w w .proce sste x t.com/a bcchm.html
3. In some situations, microbes can only cause disease in the presence of another pathogen. An exampl e i s delta hepatitis virus (HDV), whi ch cannot i nfect humans except i n t he pres ence of ongoi ng hepat i t i s B vi rus (HBV) i nfect i on, but , when pres ent , wors ens t he di s eas e cours e of HBV hepat i t i s .
4. Becaus e of t he s el ect i ve pres s ures i nduced by us i ng ant i mi crobi al agent s t o t reat or prevent i nfect i ons i n humans and ani mal s , microorganisms that are resistant to antibiotics may have a propensity for causing infections. Thi s i s es peci al l y t rue where ant i bi ot i c us e i s wi des pread, s uch as i n hos pi t al s and nurs i ng homes . Si mi l ar probl ems have appeared i n ambul at ory s et t i ngs , where ant i bi ot i c us e i s wi des pread. Thi s i s mos t not abl e i n chi l dren who recei ve repeat ed cours es of ant i bi ot i cs (e.g., for recurrent ot i t i s medi a) and harbor a more res i s t ant mi crobi al fl ora as a res ul t .
E. Epidemiologic considerations In addi t i on t o hos t and mi crobi al charact eri s t i cs , envi ronment al ci rcums t ances al s o hel p det ermi ne t he nat ure and s everi t y of an i nfect i on.
1. Contagious spread. Some i nfect i ous agent s are s pread from pers on t o pers on t hrough di rect phys i cal cont act (e.g., s yphi l i s ) or by i nfect i ous aeros ol s (e.g., t ubercul os i s ).
2. Vectors and fomites o
Pa g e 1 6 5 0
ABC Ambe r CHM Conve rte r Tria l ve rsion, http://w w w .proce sste x t.com/a bcchm.html o
a. Vect ors are ani mal s t hat act as hos t s or carri ers of a di s eas e wi t hout becomi ng i l l t hems el ves . Mos t vect ors are i ns ect s or art hropods . In general , i nfect i ons t hat are rel at ed t o ani mal s or t hei r product s (e.g., meat , mi l k, or eggs ) are cal l ed zoonoses.
o
o
b. Fomites are i nani mat e object s capabl e of s preadi ng i nfect i on. Fomi t es are mos t commonl y i dent i fi ed i n hos pi t al s , where enhanced pat i ent s us cept i bi l i t y and a hi gh concent rat i on of vi rul ent organi s ms coexi s t .
3. Geography. Some di s eas es occur excl us i vel y or at s ubs t ant i al l y great er frequency i n cert ai n areas . For exampl e, mos qui t o bi t es al mos t never res ul t i n t he t rans mi s s i on of mal ari a i n t he Uni t ed St at es . W hen cons i deri ng s uch a di s eas e i n a di agnos i s , a hi s t ory of t ravel t o or res i dence i n an endemi c area i s i mport ant . Durat i on of expos ure can al s o be i mport ant . For exampl e, fi l ari as i s l eads t o el ephant i as i s onl y aft er repeat ed expos ure t o t he mos qui t o vect or over a peri od of mont hs t o years .
4. Season. Many i nfect i ons occur more commonl y duri ng a part i cul ar t i me of t he year. Thi s may be becaus e of cert ai n act i vi t i es t hat mi ght expos e an i ndi vi dual t o ri s k and t hat are s eas onal i n nat ure (e.g., hunt i ng and fi s hi ng) or t o envi ronment al fact ors t hat favor t he growt h of t he mi crobe or i t s i ns ect vect or.
5. Institutions. Cert ai n s et t i ngs t hat bri ng t oget her s us cept i bl e i ndi vi dual s may al t er t he ri s k of i nfect i on.
Pa g e 1 6 5 1
ABC Ambe r CHM Conve rte r Tria l ve rsion, http://w w w .proce sste x t.com/a bcchm.html
Hos pi t al s and nurs i ng homes can ampl i fy and al t er t he nat ure of i nfect i ons among t he s i ck and t he el derl y, whereas day-care cent ers can have a s i mi l ar effect on t he young.
II. Use of Anti-Infective Therapy A. General principles Becaus e t he us e of ant i bi ot i cs can be l i fes avi ng, i t i s i mport ant t o recogni z e when t o i ni t i at e t reat ment . However, any t ype of medi cal i nt ervent i on pres ent s pot ent i al hazards , and t he i ndi s cri mi nat e us e of ant i bi ot i cs i s no except i on. In general , four ci rcums t ances prompt ant i mi crobi al t herapy.
1. Organism-based treatment o
o
a. W hen cul t ures or s t ai ns from a pat i ent demons t rat e a credi bl e mi croorgani s m, appropri at e ant i bi ot i c t reat ment i s i ni t i at ed. Not e t he fol l owi ng exampl es : P.339
(1) A wet mount of vagi nal s ecret i ons s howi ng T ri c homonas vagi nal i s
(2) Bl ood cul t ures t hat are pos i t i ve for St rept oc oc c us s angui s i n a pat i ent wi t h a hi s t ory of mi t ral val ve di s eas e and a fever
o
o
b. Techni ques for recogni zi ng s peci fi c ant i gens of
Pa g e 1 6 5 2
ABC Ambe r CHM Conve rte r Tria l ve rsion, http://w w w .proce sste x t.com/a bcchm.html
mi croorgani s ms al s o may be us ed t o hel p i ni t i at e t herapy. An exampl e i s t he fi ndi ng of crypt ococcal ant i gens i n t he cerebros pi nal fl ui d (CSF) i n a pat i ent wi t h chroni c meni ngi t i s . o
o
c. Is ol at i on of an organi s m al l ows i n vi t ro t es t i ng of ant i bi ot i c s us cept i bi l i t y. However, for many mi crobes , res i s t ance pat t erns are predi ct abl e enough t o permi t t reat ment wi t hout t hes e furt her t es t s .
2. Syndrome-based treatment i s i ni t i at ed when al l t hree of t he fol l owi ng condi t i ons appl y: o
o
a. The cl i ni cal pi ct ure s t rongl y s ugges t s s peci fi c organ di s eas e.
o
o
b. The t es t s needed t o make a mi crobi ol ogi c di agnos i s are not avai l abl e or pract i cal .
o
o
c. The mos t l i kel y caus at i ve organi s ms al l res pond t o t he s ame t reat ment .
(1) For exampl e, i f an ot herwi s e heal t hy young woman has pai nful uri nat i on and a pos i t i ve t es t for whi t e bl ood cel l s (W BCs ) i n t he uri ne, t reat ment can be i ni t i at ed for a uri nary t ract i nfect i on wi t hout even s ubmi t t i ng a uri ne s peci men for cul t ure.
Pa g e 1 6 5 3
ABC Ambe r CHM Conve rte r Tria l ve rsion, http://w w w .proce sste x t.com/a bcchm.html
(2) However, pat i ent s wi t h a poor res pons e t o i ni t i al t reat ment , rel aps e, or compl i cat i ng concomi t ant medi cal i l l nes s cannot be opt i mal l y managed wi t hout furt her di agnos t i c eval uat i on.
o
o
3. Empiric therapy i s gi ven when t he di agnos i s i s uncert ai n but cl i ni cal experi ence s ugges t s t hat t he pat i ent out come i n a part i cul ar s et t i ng i s i mproved wi t h ant i mi crobi al t herapy.
a. General l y, empi ri c t herapy i s i ni t i at ed whi l e awai t i ng t he res ul t s of di agnos t i c t es t s t o i dent i fy a s peci fi c caus at i ve organi s m.
b. Fever oft en devel ops as t he fi rs t s i gn of i nfect i on i n pat i ent s wi t h s evere neut ropeni a from cancer t herapy. Thes e pat i ent s may s uccumb t o t hei r i nfect i on before fi nal report s are recei ved from t he l aborat ory. Thus , ant i mi crobi al t herapy s houl d begi n at t he fi rs t s i gn of fever.
o
o
4. Prophylaxis i s us ed when a s peci fi c i nfect i on or compl i cat i on i s ant i ci pat ed and can be prevent ed. General l y, t hi s form of empi ri c t herapy i s res t ri ct ed t o a fi xed and bri ef peri od duri ng whi ch t here i s a ri s k of i nfect i on. Exampl es i ncl ude t he us e of peni ci l l i n before dent al procedures i n pat i ent s wi t h heart val ve di s eas e and t he us e of peri operat i ve ant i bi ot i cs i n s urgery. Longer durat i on of t he prophyl axi s l eads t o hi gher l i kel i hood of devel opi ng ant i mi crobi al t oxi ci t y or acqui s i t i on/s el ect i on of
Pa g e 1 6 5 4
ABC Ambe r CHM Conve rte r Tria l ve rsion, http://w w w .proce sste x t.com/a bcchm.html
res i s t ant fl ora.
B. Dosage and route (Online Figure 8-1)
ONLINE FIGURE 8-1 (A) Det ermi nat i on of mi ni mum i nhi bi t ory concent rat i on (MIC). (B) Ant i bi ot i c s us cept i bi l i t y t es t s . W hen cons i deri ng t he us e of ant i bi ot i cs , t he appropri at e dos age and rout e of admi ni s t rat i on mus t be es t abl i s hed.
1. Dosage o
o
a. Dos i ng i s bas ed on pharmacol ogi c and cl i ni cal dat a rel at ed t o t he s i ze and age of t he pat i ent , t he des i red l evel of t he drug i n t he t arget t i s s ue, t he drug's rat e of el i mi nat i on (oft en es t i mat ed by exami ni ng ki dney and l i ver funct i on), and t he
Pa g e 1 6 5 5
ABC Ambe r CHM Conve rte r Tria l ve rsion, http://w w w .proce sste x t.com/a bcchm.html
expect ed penet rat i on of t he drug i nt o t he i nfect ed t i s s ue. o
o
b. Gui del i nes for proper dos i ng are wi del y avai l abl e. Occas i onal l y dos age can be adjus t ed t hrough t he us e of bl ood or t i s s ue l evel s of t he drug. Thi s i s commonl y done, for exampl e, t o provi de a s afe and effect i ve cours e of vancomyci n t herapy i n a pers on wi t h reduced or abs ent ki dney funct i on.
2. Route. In t erms of comfort and conveni ence, oral t herapy i s us ual l y preferred. But parent eral l y admi ni s t ered (i .e., i nt ravenous or i nt ramus cul ar) drugs are us ual l y more rel i abl y abs orbed t han oral l y admi ni s t ered drugs . They are us ed i n t he fol l owi ng s i t uat i ons : o
o
a. In pat i ent s who cannot t ol erat e oral l y admi ni s t ered drugs becaus e of vomi t i ng, decreas ed peri s t al s i s , s wal l owi ng di ffi cul t y, or depres s ed ment al s t at e
o
o
b. In pat i ent s i n whom i ns t i t ut i on of t herapy i s urgent
o
o
c. W hen no oral form of t he drug exi s t s or when adequat e amount s of t he oral forms cannot be admi ni s t ered
Pa g e 1 6 5 6
ABC Ambe r CHM Conve rte r Tria l ve rsion, http://w w w .proce sste x t.com/a bcchm.html
C. Cost Thi s i s s ue has become more i mport ant wi t h i ncreas ed at t ent i on t o cos t cont ai nment . W hen al l ot her fact ors are equal , cos t may i nfl uence whi ch drug i s pres cri bed. W i t h i nt ravenous l y admi ni s t ered drugs , t he cos t of di l uent s and of l abor i nvol ved i n prepari ng and i nfus i ng t he drugs mus t be cons i dered. The cos t of admi ni s t rat i on i s l ower wi t h i nt ramus cul arl y gi ven drugs and i s general l y l owes t wi t h oral agent s . Newer drugs are oft en s t rongl y market ed t o pat i ent s and phys i ci ans , but are not neces s ari l y more effect i ve t han “ol der― drugs , whi ch may be avai l abl e i n generi c form.
D. Specific antibiotic spectrum The range or spectrum of mi croorgani s ms i nhi bi t ed or ki l l ed by an ant i bi ot i c i s an i mport ant cons i derat i on i n deci di ng whi ch drug t o us e. The res ul t s of i n vi t ro t es t i ng are hel pful i n s el ect i ng an ant i bi ot i c t hat i s mos t l i kel y t o be effect i ve agai ns t an i nfect i ve organi s m. The t wo mos t common ways t o pres ent t he res ul t s of i n vi t ro s us cept i bi l i t y t es t i ng are t o not e whet her t he organi s m bei ng t es t ed i s s us cept i bl e, i nt ermedi at e, or res i s t ant t o t he drug or t o not e t he concent rat i on of t he drug needed t o i nhi bi t t he organi s m (t hi s l at t er val ue i s cal l ed t he mi ni mum i nhi bi t ory concent rat i on, or MIC). Even when t he part i cul ar organi s m has not been t es t ed, i t may be as s umed—on t he bas i s of ei t her l ocal pat t erns of ant i bi ot i c s us cept i bi l i t y or general experi ence i n t reat i ng cert ai n s yndromes —t hat a gi ven drug i s l i kel y t o be effect i ve. However, s us cept i bi l i t y pat t erns meri t cl os e s crut i ny becaus e t hey change over t i me.
E. Concentration- or time-dependent effect
Pa g e 1 6 5 7
ABC Ambe r CHM Conve rte r Tria l ve rsion, http://w w w .proce sste x t.com/a bcchm.html
Some ant i mi crobi al s have no great er effect on bact eri al i nhi bi t i on or ki l l i ng when t he t i s s ue l evel s are rai s ed above a t hres hol d (s uch as t he MIC) needed t o s ee an effect . Thi s i s time-dependent act i on, s i nce t he benefi t of t he drug i s rel at ed t o t he amount of t i me t hat i t i s pres ent at or above t hi s t hres hol d l evel . β-Lact ams are, i n general , t i me-dependent ant i bi ot i cs . Ot her drugs are mos t effect i ve when t hey can reach hi gh peak l evel s or s pend prol onged peri ods at mul t i pl es of t he t hres hol d needed t o i nhi bi t or ki l l mi crobes , even i f t hei r concent rat i on fal l s bel ow t hat t hres hol d for a cons i derabl e peri od. Thi s i s cal l ed concentration-dependent act i vi t y and i s t ypi cal of ami nogl ycos i des .
F. Interpreting antimicrobial susceptibility tests I n vi t ro t es t i ng of ant i mi crobi al s us cept i bi l i t y can provi de us eful i nformat i on about t he s peci fi c mi crobe caus i ng an i nfect i on. For many bact eri a, t hes e t es t s can be accompl i s hed rapi dl y, wi t h res ul t s s omet i mes avai l abl e i n hours . However, the conditions used in susceptibility testing do not necessarily simulate conditions in the patient.
1. The res ul t s of s us cept i bi l i t y t es t i ng may s how t he mi croorgani s m t o be s us cept i bl e, i nt ermedi at el y s us cept i bl e, or res i s t ant t o t he drug bei ng t es t ed. Thi s i nformat i on pres uppos es t hat t he drug concent rat i on at t he s i t e of i nfect i on wi l l be s i mi l ar t o what us ual l y i s found i n s erum. It al s o as s umes t hat t he organi s m wi l l be i nhi bi t ed or ki l l ed at t he i nfect i on s i t e i f i t i s expos ed t o s uch a drug concent rat i on.
2. Anot her way t o as s es s s us cept i bi l i t y i s t o det ermi ne
Pa g e 1 6 5 8
ABC Ambe r CHM Conve rte r Tria l ve rsion, http://w w w .proce sste x t.com/a bcchm.html
t he act ual concent rat i on of ant i bi ot i c needed t o i nhi bi t t he mi croorgani s m. Thi s us ual l y i s done by expos i ng t he mi crobe t o varyi ng concent rat i ons of t he drug and obs ervi ng whi ch i s t he l owes t one t o i nhi bi t growt h [minimum inhibitory concentration (MIC)]. (onl i ne Fi gure 8-1). The l ower t he MIC, t he more s us cept i bl e t he organi s m. The Eps i l omet er t es t (E-t es t ) i s us ual l y a rel i abl e way t o obt ai n an MIC wi t hout t he i nconveni ence of prepari ng s eri al di l ut i ons of drug. In s ome i ns t ances , cl i ni cal experi ence s hows t hat a drug i s i neffect i ve des pi t e i n vi t ro dat a t hat s ugges t i t woul d work. For exampl e, cephal os pori ns are not us eful i n t reat i ng i nfect i ons caus ed by met hi ci l l i n-res i s t ant S. aureus , al t hough bl ood and t i s s ue l evel s i n exces s of t he MIC are eas i l y achi evabl e.
3. Di rect meas urement of s erum or t i s s ue l evel s of t he drug bei ng us ed may be des i rabl e s o t hat t he dos age can be adjus t ed t o avoi d t oxi ci t y and t o maxi mi ze t herapeut i c benefi t . Thi s i s es peci al l y t rue for pat i ent s who may have l es s predi ct abl e bl ood or t i s s ue l evel s of ant i bi ot i c becaus e of al t ered drug met abol i s m or excret i on s econdary t o renal or hepat i c dys funct i on. For exampl e, vancomyci n i s a t i me-dependent ant i bi ot i c, and l evel s s houl d be >8 µg/mL at al l t i mes . For mos t pat i ent s , s t andard dos i ng regi mens can achi eve t hi s goal , but pat i ent s wi t h fl uct uat i ons of renal funct i on or cert ai n forms of cont i nuous di al ys i s can have unpredi ct abl y l ow l evel s and may need t o be re-dos ed. Hi gh l evel s of vancomyci n have not been as s oci at ed cl earl y wi t h t oxi ci t y, but peak l evel s hi gher t han 40 µg/mL are unneces s ary.
G. Toxicity and side effects Al t hough l i fe-t hreat eni ng t oxi ci t y rarel y occurs wi t h ant i bi ot i c
Pa g e 1 6 5 9
ABC Ambe r CHM Conve rte r Tria l ve rsion, http://w w w .proce sste x t.com/a bcchm.html
t reat ment , phys i ci ans s houl d be fami l i ar wi t h t he s peci fi c ri s ks of each drug admi ni s t ered s o t hat s i de effect s can be ant i ci pat ed and mi ni mi z ed.
1. Allergic reactions, whi ch are s omet i mes fat al , can occur wi t h al mos t any pharmaceut i cal agent but are more common wi t h t he β-lactam antibiotics and s ul fonami des t han wi t h ot her ant i mi crobi al s .
2. Chloramphenicol, whi ch i s s el dom us ed i n t he Uni t ed St at es , i s as s oci at ed wi t h bone marrow suppression, i ncl udi ng i rrevers i bl e apl as t i c anemi a. It i s al s o as s oci at ed wi t h “Gray Baby s yndrome.―
3. Many drugs can caus e renal dysfunction, t he mos t promi nent bei ng t he aminoglycosides. Thi s t oxi ci t y i s revers i bl e and us ual l y of mi nor cl i ni cal s i gni fi cance, but ful l recovery may t ake weeks or mont hs . In rare i ns t ances , di al ys i s may be needed. In addi t i on, vari ous forms of amphotericin B are us ed i n t he t reat ment of fungal i nfect i ons . Careful at t ent i on t o t he pat i ent 's fl ui d s t at us may mi t i gat e t he revers i bl e nephrot oxi ci t y t hat i s oft en as s oci at ed wi t h t hi s agent . P.340
4. Abnormalities in blood coagulation (e.g., changes i n t he l evel s of cl ot t i ng fact ors or i n t he number or funct i on of pl at el et s ) have been as s oci at ed wi t h s everal β-lactam antibiotics and s omet i mes res ul t i n cl i ni cal l y s i gni fi cant bl eedi ng.
Pa g e 1 6 6 0
ABC Ambe r CHM Conve rte r Tria l ve rsion, http://w w w .proce sste x t.com/a bcchm.html
5. Ototoxicity i s an i nfrequent but pot ent i al l y debi l i t at i ng s i de effect of aminoglycosides. Ei t her audi t ory or ves t i bul ar i mpai rment can occur; oft en i t i s i rrevers i bl e.
6. Phototoxicity i s an exaggerat ed res pons e of t he s ki n t o s un expos ure. Thi s can res ul t i n s evere “s unburn†• and i s as s oci at ed wi t h us e of s ome fl uoroqui nol ones .
7. T oxicity to cardiac conduction can ari s e from any number of drugs (i ncl udi ng nonant i bi ot i c compounds ). The mos t common arrhyt hmi a i s as s oci at ed wi t h QT prol ongat i on. At t he ext reme end of QT prol ongat i on i s a s el f-s us t ai ni ng arrhyt hmi a cal l ed t ors ade de poi nt es , whi ch can be l i fe-t hreat eni ng. A number of ant i bi ot i cs (macrol i des and fl uoroqui nol ones ) have been as s oci at ed wi t h QT prol ongat i on, and t hei r act i ons are more l i kel y t o be s eri ous i f ot her medi cat i ons al s o prol ong t he QT i nt erval or i f t he pat i ent has a predi s pos i t i on t o a l ong QT t o begi n wi t h.
H. Adult immunization
1. It i s i mport ant for adul t s t o have i mmuni t y t o all of the childhood diseases o
o
a. For adul t s wi t h Hi s t ory of Incompl et e or Unknown Tet anus , Di pht heri a, or Pert us s i s Vacci nat i on, a s eri es of t hree Td
Pa g e 1 6 6 1
ABC Ambe r CHM Conve rte r Tria l ve rsion, http://w w w .proce sste x t.com/a bcchm.html
(t et anus /di pht heri a t oxoi d) vacci nes wi t h one of t he Td admi ni s t rat i ons i n t he s eri es repl aced by Tdap (t et anus /di pht heri a t oxoi d pl us acel l ul ar pert us s i s ) s houl d be admi ni s t ered. Boos t er dos es of t et anus s houl d be gi ven every 10 years t o al l adul t s but may be gi ven earl i er i f an i ndi vi dual has a t et anus -prone wound and t he previ ous dos e of t oxoi d was gi ven more t han 5 years earl i er. Pas s i ve i mmuni zat i on wi t h t et anus i mmunogl obul i n i s i ndi cat ed i f a wound has been ext ens i vel y cont ami nat ed wi t h di rt . Di pht heri a t oxoi d i s us ual l y co-admi ni s t ered wi t h t et anus . o
o
b. Ki l l ed pol i ovi rus vacci ne s houl d be gi ven t o adul t s who are not al ready i mmune, es peci al l y t hos e wi t h chi l dren who are due t o recei ve l i ve pol i ovi rus vacci ne. Al t hough pol i o may be eradi cat ed worl dwi de i n t he earl y years of t he t went y-fi rs t cent ury, ces s at i on of s t andard chi l dhood pol i o vacci nat i on may not fol l ow unt i l s everal years of eradi cat i on have el aps ed.
o
o
c. Becaus e rubel l a i n a pregnant woman can res ul t i n a devas t at i ng i nfect i on of t he fet us , al l women of chi l dbeari ng age s houl d be i mmune t o rubel l a. Serum t es t s for ant i body t o rubel l a are wi del y avai l abl e t o hel p i dent i fy adul t s who need vacci nat i on. Rubel l a i s a part of t he s t andard panel of chi l dhood vacci nes .
o
o
d. If t here i s no hi s t ory of mumps or meas l es i l l nes s or vacci nat i on, t he appropri at e l i ve vi rus vacci nes for t hes e i l l nes s es s houl d be gi ven.
Pa g e 1 6 6 2
ABC Ambe r CHM Conve rte r Tria l ve rsion, http://w w w .proce sste x t.com/a bcchm.html
Al t hough meas l es may rarel y occur i n vacci nat ed i ndi vi dual s , t hi s i s a rare event when t wo dos es of vacci ne have been appropri at el y admi ni s t ered.
2. Speci al vacci nat i on i s s ues concern t he elderly (aged 65 years and ol der) and chronically ill pat i ent s —i ncl udi ng human i mmunodefi ci ency vi rus (HIV)/acqui red i mmunodefi ci ency di s eas e (AIDS) —es peci al l y t hos e wi t h cardi ac or pul monary di s eas e. Al l s uch pat i ent s s houl d recei ve pneumococcal vaccination once and, i n t he appropri at e s eas on, s houl d have annual influenza vaccination. Re-i mmuni zat i on wi t h pneumococcal vacci ne may be cons i dered for t hos e i ndi vi dual s who had t hei r fi rs t dos e before t he age of 65, i f more t han 5 years has el aps ed s i nce t hat dos e. A new conjugat e vacci ne t o prevent pneumococcal i nfect i ons has been devel oped for chi l dren, and i t has reduced t he burden of pneumococcal di s eas e.
3. T ravelers t o count ri es where yellow fever i s endemi c (mos t of t ropi cal Afri ca and Sout h Ameri ca) s houl d recei ve t hi s vacci ne. T yphoid i mmuni zat i ons may be recommended for cert ai n t ravel ers . Hepatitis A vi rus (HAV) vacci ne i s recommended for t ravel ers who expect t o encount er poor s ani t ary faci l i t i es , es peci al l y t hos e res ul t i ng i n human fecal cont ami nat i on of food or wat er. Hepat i t i s A vacci ne has l argel y s uppl ant ed t he us e of i mmune s erum gl obul i n (ISG), whi ch has been i n s hort s uppl y. Di rect i nt ramus cul ar i nject i on of ISG i s s t i l l s ugges t ed i f t here i s not a wi ndow of at l eas t 2 weeks before a hi gh-ri s k expos ure t o hepat i t i s A vi rus .
4. Uni vers al i mmuni t y t o hepat i t i s B vi rus (HBV) and
Pa g e 1 6 6 3
ABC Ambe r CHM Conve rte r Tria l ve rsion, http://w w w .proce sste x t.com/a bcchm.html
varicella are current publ i c heal t h goal s . Al t hough i ndi vi dual s who are al ready i mmune by vi rt ue of pri or i nfect i on or vacci nat i on do not us ual l y benefi t from vacci nat i on, bot h vacci nes can be gi ven s afel y t o pers ons whos e expos ure hi s t ory i s unknown. Bot h vacci nes are l es s effect i ve i n i ndi vi dual s wi t h i mpai red i mmune s t at us . Becaus e vari cel l a vacci ne i s a l i ve vi rus vacci ne, i t s houl d be us ed caut i ous l y or not at al l i n pat i ent s wi t h known advanced i mmunodefi ci ency. o
o
a. Adul t s who have durabl e i mmuni t y t o vari cel l a-zos t er vi rus from a cl i ni cal bout of chi ckenpox i n chi l dhood do not need vacci nat i on.
o
o
b. Adul t s wi t h no hi s t ory of chi ckenpox or no s erol ogi c evi dence of i mmuni t y s houl d recei ve t wo dos es of vacci ne.
o
o
c. Adul t s wi t h no hi s t ory of HBV or vacci nat i on for i t s houl d recei ve t hree dos es of vacci ne i n an appropri at e s chedul e. Al t hough uni vers al admi ni s t rat i on of hepat i t i s A vacci ne i s not yet recommended, t here i s a formul at i on of hepat i t i s A and hepat i t i s B vacci ne combi ned t hat woul d reduce t he t ot al number of i nject i ons .
5. Asplenic individuals s houl d have pneumococcal and meningococcal vaccinations, even t hough prot ect i on from t hes e i nfect i ons i s i ncompl et e. In preparat i on for el ect i ve s pl enect omy, i mmuni zat i ons s houl d i deal l y be performed before s urgery, i f pos s i bl e.
Pa g e 1 6 6 4
ABC Ambe r CHM Conve rte r Tria l ve rsion, http://w w w .proce sste x t.com/a bcchm.html
6. HAV vacci ne i s hi ghl y effect i ve and s afe. It i s recommended i n areas wi t h unus ual l y hi gh rat es of HAV, for t ravel ers t o part s of t he worl d where HAV i s more common, and for peopl e wi t h chroni c HBV or chroni c hepat i t i s C vi rus (HCV) i nfect i ons .
7. Meningococcal vaccine has been recommended for col l ege-age s t udent s —es peci al l y t hos e l i vi ng i n dormi t ori es . There have been out breaks of s eri ous meni ngococcal i nfect i ons on col l ege campus es , al t hough t he number of cas es per year i s s mal l . The vacci ne i s wel l t ol erat ed but has a l i mi t ed durat i on of effect i venes s (a few years ) and does not prot ect agai ns t t ype b meni ngococcal i nfect i ons .
8. Human papilloma virus (HPV) vacci ne i s recommended for femal es age 9–26 t o prevent cancerous and precancerous cervi cal l es i ons . Durat i on of prot ect i on i s uncert ai n and i mmuni zat i on has not been s hown t o prevent di s eas e i n t hos e al ready i nfect ed.
III. Effective Use of The Microbiology Laboratory To maxi mi ze t he us eful nes s of any mi crobi ol ogy l aborat ory, i t i s neces s ary t o be fami l i ar wi t h i t s s peci fi c capabi l i t i es and procedures . The fol l owi ng are general gui del i nes .
A. Obtaining and handling specimens
1. Fresh specimens are superior to old ones. Thi s i s es peci al l y t rue when quant i t at i ve res ul t s are i mport ant (e.g., col ony count s of uri ne cul t ures ) or when t he t arget
Pa g e 1 6 6 5
ABC Ambe r CHM Conve rte r Tria l ve rsion, http://w w w .proce sste x t.com/a bcchm.html
organi s m i s fragi l e (e.g., prot ozoal t rophozoi t es i n s t ool ). If i mmedi at e del i very of a s peci men i s not pos s i bl e, proper mai nt enance of t he s peci men s houl d be ens ured unt i l i t can be proces s ed. o
o
a. Some bact eri a (e.g., N. gonorrhoeae) are s ens i t i ve t o col d, and s peci mens s houl d not be refri gerat ed.
o
o
b. Speci mens t o be cul t ured for anaerobes s houl d be mai nt ai ned i n a pre-reduced, oxygen-free envi ronment or i n a s yri nge wi t hout any ai r and wi t hout a needl e. They s houl d be t rans port ed t o t he l aborat ory for i mmedi at e pl at i ng.
2. Large specimens are better than small ones. W hen col l ect i ng s peci mens for mi crobi ol ogi cal cul t ure and s t ai ni ng, s ubmi t as much mat eri al as can be obt ai ned. W hen cul t uri ng a wound, us e pres s ure on t he wound t o expres s as much purul ent fl ui d as pos s i bl e from any fl uct uant areas , or as pi rat e fl ui d us i ng as ept i c t echni que. Somet i mes , mul t i pl e s peci mens are needed t o i dent i fy mi croorgani s ms (e.g., M. t uberc ul os i s ) t hat are s hed i nfrequent l y or i n s mal l numbers .
3. Biological hazards should be labeled and handled properly. Laborat ori es have i ns t i t ut ed universal precautions and t reat al l s peci mens as pot ent i al l y i nfect i ous . Some bact eri a and fungi may be es peci al l y haz ardous i n pure cul t ure (e.g., Bruc el l a, Franc i s el l a, and Coc c i di oi des ), and t he l aborat ory s houl d be not i fi ed i f t hes e organi s ms are s us pect ed cl i ni cal l y s o t hat appropri at e care can be t aken.
Pa g e 1 6 6 6
ABC Ambe r CHM Conve rte r Tria l ve rsion, http://w w w .proce sste x t.com/a bcchm.html
4. Newer techniques may bypass or accelerate culture results. Di rect DNA Probe t es t for C. t rac homat i s and N. gonorrhoeae can now be done wi t hout concern for mai nt ai ni ng bact eri al vi abi l i t y (neces s ary for cul t ure) and res ul t s are avai l abl e much fas t er t han wi t h cul t ure res ul t s . Si mi l arl y, cul t ure t es t i ng for mycobact eri a i s now fas t er t han before (pos i t i ve res ul t s back i n 10 days vs . 30 days ), and DNA probes can i dent i fy mycobact eri a i n l es s t han a day once cul t ures are pos i t i ve.
B. Interpreting negative and positive culture results W hen a di agnos i s res t s on i dent i fyi ng an organi s m obt ai ned from a cl i ni cal s peci men, i t i s cruci al t o recogni ze what repres ent s s i mpl e cont ami nat i on by ot her body fl ora or i nani mat e s ources . Laborat ory res ul t s are more credi bl e when a s peci men has been s t ai ned and s hown t o cont ai n t he appropri at e cel l t ype (e.g., neut rophi l or al veol ar macrophage i n s put um) and l acks evi dence of cont ami nat i on (e.g., s quamous epi t hel i al cel l s i n s put um or uri ne). W hen care and at t ent i on have been l avi s hed on a good s peci men, al mos t any pos i t i ve res ul t i s s i gni fi cant .
1. If ant i bi ot i cs have been admi ni s t ered, t he pot ent i al for pos i t i ve cul t ures i s reduced, and t he predi ct i ve val ue of negat i ve cul t ures al s o i s di mi ni s hed.
2. W hen cul t ures are t rul y negat i ve, t he us e of s peci al medi a and growt h condi t i ons s houl d be cons i dered. Suppl ement ary nut ri ent s or s uppres s i on of ot her
Pa g e 1 6 6 7
ABC Ambe r CHM Conve rte r Tria l ve rsion, http://w w w .proce sste x t.com/a bcchm.html
organi s ms may be neces s ary t o permi t growt h and i dent i fi cat i on.
3. Cul t ures t hat are repeat edl y pos i t i ve for a gi ven organi s m are more convi nci ng t han a s i ngl e pos i t i ve s peci men. Thi s i s es peci al l y t rue for bl ood cul t ures . Speci mens t aken from normal l y s t eri l e s i t es s uch as CSF and joi nt and pl eural fl ui ds us ual l y are rel i abl e when obt ai ned as ept i cal l y. However, i f t hey are cul t ure-pos i t i ve for s ki n or mucous membrane commens al s , repeat cul t ures may be requi red.
4. Corroborat i ve t es t s s uch as ant i gen det ect i on, s peci fi c nucl ei c aci d probes , and s eroconvers i on (i .e., t he devel opment of ant i bodi es t o a pat hogen) can es t abl i s h t he s i gni fi cance of a s i ngl e cul t ure-pos i t i ve s peci men or es t abl i s h a di agnos i s when cul t ures have not been obt ai ned or are negat i ve.
5. Quant i t at i ve t es t s s uch as vi ral l oad for HIV and quant i t at i ve bronchoal veol ar l avage (BAL) fl ui d cul t ures may be us eful i n as s es s i ng t he s everi t y of di s eas e or t he probabi l i t y t hat a pos i t i ve cul t ure repres ent s t rue i nfect i on.
IV. Risk Factors for Infection Somet i mes i t i s pos s i bl e t o i dent i fy pat i ent charact eri s t i cs t hat modi fy t he l i kel i hood or s everi t y of an i nfect i on. In general , t he i nt ens i t y or frequency of expos ure and t he degree of s us cept i bi l i t y correl at e wi t h pot ent i al hazard of i nfect i on. P.341
Pa g e 1 6 6 8
ABC Ambe r CHM Conve rte r Tria l ve rsion, http://w w w .proce sste x t.com/a bcchm.html
A. Diabetes In general , pat i ent s wi t h di abet es are more prone t o cert ai n i nfect i ons t han i ndi vi dual s wi t hout gl ucos e i nt ol erance.
1. Foot and lower leg ulcers, whi ch may become i nfect ed, are more l i kel y t o occur i n pat i ent s wi t h di abet es due t o neuropat hy or peri pheral vas cul ar di s eas e. Such s oft t i s s ue i nfect i ons are more l i kel y t o i nvol ve Gram-negat i ve rods , and t hey are more di ffi cul t t o cure t han t hos e occurri ng i n nondi abet i c i ndi vi dual s .
2. Candida infections, es peci al l y vul vovagi nal candi di as i s or t hrus h, are more l i kel y t o occur i n pat i ent s wi t h di abet es .
3. Urinary tract infections may be more common and more s evere i n di abet i c pat i ent s , and t he i ncreas ed morbi di t y probabl y i s rel at ed t o bl adder dys funct i on i n t hos e pat i ent s wi t h di abet i c neuropat hy. It i s oft en recommended t hat uri nary t ract i nfect i ons be more vi gorous l y t reat ed i n pat i ent s wi t h di abet es .
4. Some rare diseases occur al mos t excl us i vel y i n di abet i c pat i ent s . o
o
a. Malignant otitis externa i s a pai nful , rapi dl y progres s i ve, and l ocal l y des t ruct i ve di s eas e of t he ext ernal audi t ory canal t hat may ext end t o t he t emporal bone and t he brai n. Ps eudomonas aerugi nos a i s t he caus at i ve agent (s ee V B 1 a).
o
Pa g e 1 6 6 9
ABC Ambe r CHM Conve rte r Tria l ve rsion, http://w w w .proce sste x t.com/a bcchm.html
o
b. Rhinocerebral mucormycosis, a fungal i nfect i on t hat s t art s i n t he nos e or paranas al s i nus es , i s l ocal l y des t ruct i ve. It i s found i n pat i ent s wi t h s evere met abol i c aci dos i s (e.g., i n pat i ent s wi t h di abet es and ket oaci dos i s ).
o
o
c. Synergistic gangrene i s a s oft t i s s ue i nfect i on t hat oft en i s at t ri but abl e t o s t rept ococci and obl i gat e anaerobes . It may i nvol ve t he s ki n or t he underl yi ng fas ci al s t ruct ures and t ends t o progres s rel ent l es s l y unl es s t reat ed wi t h ext ens i ve s urgi cal débri dement .
o
o
d. Emphysematous cholecystitis i s a rare but s evere i nfect i on of t he gal l bl adder. Unl i ke ordi nary chol ecys t i t i s , i t requi res urgent ant i bi ot i c admi ni s t rat i on and cons i derat i on of s urgery. The cl i ni cal cl ues are s i mi l ar t o t hat of chol ecys t i t i s but t here i s oft en hi gh fever, t achycardi a, hypot ens i on, and ot her evi dence of s eps i s . X-rays , comput ed t omography (CT) s can, and ot her i magi ng s t udi es s how gas i n t he gal l bl adder wal l .
B. Alcoholism Acut e and chroni c al cohol us e can i mpai r neurol ogi c funct i on and decreas e t he effi cacy of neut rophi l s . In addi t i on, chroni c al cohol us e can res ul t i n s evere organ damage, es peci al l y t o t he l i ver.
1. The i nci dence of pneumococcal , as pi rat i on, and Gram-negat i ve baci l l ary pneumonias i s great er i n al cohol i cs t han i t i s i n t he general popul at i on. Tubercul os i s , anaerobi c l ung abs ces s , and empyema al s o are more common.
Pa g e 1 6 7 0
ABC Ambe r CHM Conve rte r Tria l ve rsion, http://w w w .proce sste x t.com/a bcchm.html
2. W hen as ci t es i s pres ent , al cohol i cs may devel op spontaneous bacterial peritonitis, a bact eri al i nfect i on of as ci t i c fl ui d t hat occurs i n t he abs ence of bowel perforat i on. Thi s i nfect i on, us ual l y caus ed by Gram-negat i ve aerobi c baci l l i or ent erococci , al s o occurs i n nonal cohol i c pat i ent s wi t h as ci t es (e.g., nephrot i c s yndrome).
C. Injection drug use Several i nfect i ons are more common i n i nject i on drug us ers t han i n comparabl e i ndi vi dual s who do not us e s uch s ubs t ances . Fever i n s uch drug us ers mus t be eval uat ed careful l y becaus e pyogeni c bact eri al i nfect i ons can progres s rapi dl y i f t reat ment i s del ayed.
1. Infections related to unsterile techniques. Inject i on drug us ers are prone t o a vari et y of s uppurat i ve compl i cat i ons becaus e, i n mos t cas es , t he preparat i on of t he drug i s not as ept i c, equi pment i s not s t eri l e, and s ki n cl eans i ng i s i nadequat e. o
o
a. Bacterial endocarditis, whi ch oft en occurs on t he t ri cus pi d val ve (a val ve rarel y i nfect ed i n ot her popul at i ons ), i s t he mos t s eri ous i nfect i on. Us ual l y, t he bact eri um res pons i bl e i s S. aureus .
o
o
b. Superficial skin infections are common, and i nfect i ous art hri t i s , us ual l y caus ed by P. aerugi nos a or S. aureus , i s s een frequent l y.
o
o
c. T etanus, al t hough rare, occurs more frequent l y, becaus e i nject i on drug us ers are more l i kel y t o
Pa g e 1 6 7 1
ABC Ambe r CHM Conve rte r Tria l ve rsion, http://w w w .proce sste x t.com/a bcchm.html
have i mproperl y cl eaned wounds .
2. Infections related to needle sharing. Some cont agi ous di s eas es can be t rans mi t t ed effi ci ent l y vi a t he s mal l amount of bl ood i n us ed needl es and s yri nges . The i nci dence of HBV, HCV, and HIV i nfect i on i s i ncreas ed i n i nject i on drug us ers who s hare needl es .
P.342
D. Occupational exposure
1. The ri s k of occupat i on-rel at ed i nfect i on may be unpredictable, as i n t he out breaks of Pontiac fever, whi ch were rel at ed t o t he aeros ol i zat i on of Legi onel l a pneumophi l a–cont ami nat ed wat er vi a ai r-condi t i oni ng duct s t hroughout an offi ce bui l di ng.
2. The ri s k of i nfect i on al s o may be i nfl uenced by fact ors out s i de t he i mmedi at e work envi ronment . For exampl e, t he ri s k of brucel l os i s among s l aught erhous e workers i s det ermi ned l argel y by t he number of l i ves t ock i nfect ed and t o a l es s er ext ent by t he degree of t he worker's expos ure t o t he bl ood of i nfect ed l i ves t ock.
3. W hen an i ncreas ed ri s k i s predictable, cert ai n prevent i ve meas ures can be i ns t i t ut ed (e.g., i mmuni zat i on of heal t h care workers agai ns t hepat i t i s B).
E. Internal prostheses W hen a pros t hes i s (s uch as art i fi ci al hi p or knee joi nt s and heart
Pa g e 1 6 7 2
ABC Ambe r CHM Conve rte r Tria l ve rsion, http://w w w .proce sste x t.com/a bcchm.html
val ves , ei t her mechani cal or bi ol ogi c mat eri al ) i s i ns ert ed i nt o t he body, t here i s a pos s i bi l i t y t hat an i nfect i on wi l l devel op at t he s i t e of i ns ert i on. Becaus e t he i mmune s ys t em react s t o any forei gn mat eri al , a pros t het i c organ i s s us cept i bl e t o i nfect i on by a great er number and vari et y of organi s ms t han i s t he nat i ve organ, even i f t hat organ i s damaged. In addi t i on, t he cl i ni cal pres ent at i on of t hes e i nfect i ons may be at ypi cal , and t he i nt erval bet ween s urgery and any mani fes t at i on of s eps i s may be l ong (e.g., many years ). In mos t cas es , bact eri a are i nocul at ed at t he t i me of s urgery, al t hough s econdary hemat ogenous or percut aneous s pread al s o occurs .
F. Indwelling catheters Des pi t e t he hi gh ri s k of i nfect i on wi t h t he us e of cat het ers , t here i s no i ndi cat i on for s ys t emi c ant i bi ot i c prophyl axi s before or duri ng cat het eri zat i on.
1. Li ke i nt ernal pros t hes es , cat het ers s uch as i nt ravenous or i nt ra-art eri al l i nes , bl adder cat het ers , and endot racheal t ubes offer s i t es of di mi ni s hed hos t res pons i venes s . Furt hermore, t hey provi de communi cat i on bet ween t he ext ernal envi ronment and t he ordi nari l y s t eri l e i nt ernal envi ronment of t he body.
2. In addi t i on t o i nt ri ns i c hos t fact ors , two important factors determine the likelihood of catheter-related infection: t he duration of catheterization and t he degree of cleanliness maintained during catheterization. Durat i on i s t he mos t i mport ant fact or i n t racheal and uret hral cat het eri zat i on. Bot h fact ors are i mport ant i n vas cul ar cat het eri zat i on.
More det ai l i s onl i ne.
o
o
a. For l ong-t erm i nt ravenous acces s wi t h a s urgi cal l y i mpl ant ed cat het er s uch as a Hi ckman s i l as t i c cat het er, t he ri s k of i nfect i on can be
Pa g e 1 6 7 3
ABC Ambe r CHM Conve rte r Tria l ve rsion, http://w w w .proce sste x t.com/a bcchm.html
reduced, al t hough not el i mi nat ed, by met i cul ous at t ent i on t o s t eri l e t echni que duri ng i ns ert i on and by avoi di ng i ns ert i on s i t es s uch as t he groi n t hat are prone t o mi crobi al cont ami nat i on. o
o
b. For t he pat i ent who needs i nt ravenous acces s for a s hort or an i nt ermedi at e peri od and has cat het ers pl aced i n t he arms , t he s i t es s houl d be changed every 48–72 hours . W hen a cent ral vei n cat het er i s pl aced for t emporary i nt ravenous acces s , t he maxi mal durat i on of t he cat het eri zat i on at a gi ven s i t e i s not abs ol ut e; however, t here i s a s ubs t ant i al ri s k of i nfect i on aft er a few days , and cat het ers s houl d be removed as s oon as pos s i bl e or at t he fi rs t s i gn of s eps i s . Long cat het ers can be i ns ert ed i n t he arm but have t hei r t i p i n t he vena cava. Thes e peri pheral l y i ns ert ed cent ral cat het ers (PICCs ) are us ed t o del i ver medi cat i ons t hat can i rri t at e peri pheral vei ns or caus e probl ems i f t he cat het er get s di s l odged.
o
o
c. Vas cul ar cat het ers general l y are mai nt ai ned wi t h a hi gh degree of care t o avoi d i nfect i on (e.g., as ept i c dres s i ng changes and t he appl i cat i on of ant i bi ot i c-cont ai ni ng oi nt ment s t o t he i ns ert i on s i t e); however, t he i mport ance of t hes e meas ures i n prevent i ng i nfect i on i s unknown. Mos t cat het er-rel at ed i nfect i ons s how l i t t l e or no evi dence of di s eas e at t he cat het er i ns ert i on s i t e, al t hough pus or s i gni fi cant rednes s at t he i ns ert i on s i t e i s us ual l y as s oci at ed wi t h i nfect i on of t he cat het er.
o
Pa g e 1 6 7 4
ABC Ambe r CHM Conve rte r Tria l ve rsion, http://w w w .proce sste x t.com/a bcchm.html o
d. Some cat het ers are i mpregnat ed wi t h s i l ver or wi t h ant i bi ot i cs , and t hes e are as s oci at ed wi t h fewer i nfect i ons . However, whet her t he rat e of s eri ous i nfect i on s uch as bact eremi a i s s i gni fi cant l y l ower wi t h t hes e cat het ers i s not cert ai n.
G. Granulocytopenia A s peci fi c ri s k t hat has been i dent i fi ed and careful l y s t udi ed i s rel at ed t o granulocytopenia, t he abs ence of adequat e numbers of ci rcul at i ng neut rophi l s . The ri s k of i nfect i on i s i nvers el y rel at ed t o t he durat i on and degree of granul ocyt openi a. Neut rophi l count s 3
great er t han 500/mm are adequat e prot ect i on agai ns t mos t opport uni s t i c i nfect i ons .
1. In mos t pat i ent s wi t h granul ocyt openi a, t he underl yi ng di s eas e i s a hematologic malignancy, al t hough reduced neut rophi l l evel s can be s een i n as s oci at i on wi t h apl as t i c anemi a, drug-i nduced agranul ocyt os i s , and cyt ot oxi c chemot herapy for non-hemat ol ogi c mal i gnanci es . Pat i ent s wi t h hemat ol ogi c mal i gnancy have an addi t i onal i nfect i on ri s k when t hey recei ve chemot herapy, whi ch al s o di s rupt s t he l i ni ng of t he gas t roi nt es t i nal t ract and permi t s bact eri al and fungal i nvas i on.
2. The i ndi genous fl ora of pat i ent s wi t h granul ocyt openi a i s l i kel y t o be al t ered duri ng hos pi t al i zat i on and wi t h t he admi ni s t rat i on of ant i bi ot i cs . The mi crofl ora mos t commonl y as s oci at ed wi t h i nfect i ons i n affect ed pat i ent s i s deri ved from t hei r i ndi genous fl ora. Bact eri a s uch as Es c heri c hi a c ol i and P. aerugi nos a, as wel l as fungi s uch as Candi da al bi c ans , are mos t oft en found.
Pa g e 1 6 7 5
ABC Ambe r CHM Conve rte r Tria l ve rsion, http://w w w .proce sste x t.com/a bcchm.html
3. Mol ds s uch as As pergi l l us s peci es may caus e s eri ous , oft en fat al , i nfect i ons i n pat i ent s wi t h granul ocyt openi a. Spores are i nhal ed from t he envi ronment .
4. Pers i s t ent fever i s t he hal l mark of i nfect i on i n pat i ent s wi t h neut ropeni a. Becaus e uncont rol l ed i nfect i on can res ul t i n rapi d cl i ni cal det eri orat i on, ant i mi crobi al s are gi ven i mmedi at el y aft er appropri at e cul t ure s peci mens are obt ai ned.
5. Granul ocyt es rel eas ed i nt o t he ci rcul at i on aft er t he us e of colony-stimulating factors s uch as fi l gras t i m are ful l y funct i onal . Thus , granul ocyt openi a aft er chemot herapy of s ol i d t umors i s oft en bri ef and unaccompani ed by s eri ous i nfect i on.
H. Immunosuppressive agents Thes e agent s are us ed t o t reat a vari et y of medi cal probl ems , s uch as prevent i on of reject i on of t rans pl ant ed organs , and cont rol of aut oi mmune di s orders . P.343
1. The expres s i on of t hi s i mmunodefi ci ency i s s een mos t l y i n t he i ncreas ed i nci dence of i nfect i on us ual l y cont rol l ed by cellular immunity (e.g., mycobact eri al , fungal , nocardi al , and cyt omegal ovi ral i nfect i ons ). St eroi ds al s o al t er s ome di agnos t i c t es t s , mos t not abl y t he expres s i on of del ayed hypers ens i t i vi t y.
2. St eroi ds are us ed wi del y i n t rans pl ant at i on procedures i nvol vi ng t he ki dney, heart , l ung, l i ver, and
Pa g e 1 6 7 6
ABC Ambe r CHM Conve rte r Tria l ve rsion, http://w w w .proce sste x t.com/a bcchm.html
bone marrow. In part , t he i nfect i ons experi enced by t hes e pat i ent s are rel at ed t o cort i cos t eroi ds and t he ot her drugs (i .e., cycl os pori ne, t acrol i mus , az at hi opri ne, and mycophenol at e mofet i l ) us ed t o prevent organ reject i on. Thes e l at t er drugs are s omet i mes referred t o as “s t eroi d s pari ng,― but t hi s refers t o t hei r us e i n reduci ng ri s k of s t eroi d-as s oci at ed s i de-effect s s uch as di abet es mel l i t us and os t eopeni a.
3. Inject abl e t umor necros i s fact or (TNF)–i nhi bi t i ng drugs are us eful i n t he t reat ment of rheumatoid arthritis and inflammatory bowel disease. However, t hes e drugs (es peci al l y i nfl i xi mab) have been as s oci at ed wi t h an exces s of cas es of t ubercul os i s (TB; i ncl udi ng ext rapul monary TB).
I. Neurologic deficits Neurol ogi c dys funct i on may predi s pos e pat i ent s t o i nfect i on i f s uch defi ci t s caus e body defens es t o be more eas i l y breached. The fol l owi ng are exampl es .
1. El i mi nat i on of t he gag refl ex, whet her by s t roke or coma, predi s pos es pat i ent s t o as pi rat i on of oral or gas t ri c cont ent s , res ul t i ng i n pneumonia.
2. Secondary i nfect i on of a hypoes t het i c l i mb. Diabetic neuropathy can res ul t i n s ki n ul cerat i on and i nfect i on.
3. Los s of neuromus cul ar cont rol of t he bl adder. Long-term catheterization (wi t h i t s at t endant compl i cat i ons , s uch as i nfect i on) may be neces s ary.
J. Age The l i kel i hood of acqui ri ng cert ai n i nfect i ons vari es cons i derabl y
Pa g e 1 6 7 7
ABC Ambe r CHM Conve rte r Tria l ve rsion, http://w w w .proce sste x t.com/a bcchm.html
wi t h age. In general , t he mat urat i on of t he i mmune s ys t em i s res pons i bl e for t he changi ng pat t ern of i nfect i ons i n earl y chi l dhood, and t he pres ence of ot her medi cal i l l nes s es account s for mos t of t he changes i n l at e adul t hood. Di mi ni s hed i mmune res pons e i n t he el derl y may predi s pos e t hem s l i ght l y t o i nfect i on. For exampl e, l ower t i t ers of ant i body t o i nfl uenz a vi rus are found aft er i mmuni zat i on i n t he el derl y t han i n younger adul t s .
K. Nosocomial infection Al t hough nos ocomi al i nfect i ons (i .e., i nfect i ons acqui red aft er 2 or more days i n t he hos pi t al ) can i nvol ve any body s i t e, t hey have s ome i mport ant common charact eri s t i cs .
1. Etiology and pathogenesis o
o
a. Residence in the hospital predi s pos es pat i ent s t o s ki n and mucos al col oni zat i on by mi crobi al fl ora di fferent from t hat found i n ambul at ory pat i ent s . Speci fi cal l y, ent eri c Gram-negat i ve rods (e.g., E. c ol i , Kl ebs i el l a) or P. aerugi nos a i n t he al i ment ary t ract may s pi l l over ont o t he s ki n or i nt o t he res pi rat ory t ree. The degree of i l l nes s i nfl uences t he l i kel i hood and rapi di t y of acqui ri ng hos pi t al fl ora.
o
o
b. Antimicrobials gi ven t o prevent or t reat i nfect i on may predi s pos e pat i ent s t o col oni zat i on and s ubs equent i nfect i on by hos pi t al fl ora.
o
o
c. Instrumentation (e.g., endot racheal t ubes and i nt ravenous cat het ers ) may bypas s s ome of t he nat ural hos t defens es .
o
Pa g e 1 6 7 8
ABC Ambe r CHM Conve rte r Tria l ve rsion, http://w w w .proce sste x t.com/a bcchm.html
o
d. Failure to observe appropriate infection control measures may permi t t he di s s emi nat i on of hos pi t al fl ora. The mos t i mport ant rout e for s uch di s s emi nat i on i s on t he hands of heal t h care workers .
o
o
e. Environmental factors, s uch as t hos e t hat permi t col oni zat i on of wat er by l egi onel l a or i mperfect ai r fi l t rat i on t hat al l ows for fungal s pores , may permi t i nfect i on t o occur at exces s rat es becaus e of t he i nherent l y decreas ed res i s t ance t o i nfect i on of s ome pat i ent s .
2. T herapy. The durat i on of hos pi t al i zat i on and t he l i kel i hood t hat t he i nfect i on was acqui red nos ocomi al l y i nfl uence t he us e of empi ri c ant i bi ot i cs t o t reat s us pect ed i nfect i ons . Knowl edge of t he ant i mi crobi al s us cept i bi l i t y pat t erns of nos ocomi al mi crobes i n one's i ns t i t ut i on can be es peci al l y hel pful .
3. Prevention
See onl i ne for det ai l s , i ncl udi ng res pi rat ory
precaut i ons .
P.344
o
o
a. Hands must be washed and gl oves changed bet ween pat i ent cont act s . Al cohol -bas ed gel s or creams are effect i ve and eas i er t o us e regul arl y t han i s t horough handwas hi ng before and aft er each pat i ent encount er.
o
Pa g e 1 6 7 9
ABC Ambe r CHM Conve rte r Tria l ve rsion, http://w w w .proce sste x t.com/a bcchm.html
o
b. Proper technique on insertion of devi ces s uch as endot racheal t ubes and i nt ravenous cat het ers i s es s ent i al . Prompt di s cont i nuat i on when t hey are no l onger needed may reduce t he burden of nos ocomi al i nfect i ons .
o
o
c. Universal precautions have been i ns t i t ut ed i n heal t h care faci l i t i es t o reduce t he ri s k of t rans mi s s i on of many pot ent i al l y hazardous mi crobes , i ncl udi ng bl ood-borne vi rus es s uch as HIV t ype 1 (HIV-1) and HBV. It i s ext remel y i mport ant t o be fami l i ar wi t h t he i ns t i t ut i onal pol i cy of t he heal t h care faci l i t y i n whi ch one works . The es s ent i al pri nci pl es of uni vers al precaut i ons are t o t reat each cl i ni cal s peci men as pot ent i al l y i nfect i ve, t o reduce s harp (e.g., needl es t i ck) expos ures , t o avoi d s pl as hes of body fl ui ds ont o mucous membranes , and t o di s pos e of pot ent i al l y hazardous s ubs t ances s afel y.
o
o
d. Protective isolation i s i nt ended t o prot ect pat i ent s from exogenous s ources of col oni zat i on or i nfect i on. Thi s meas ure i s us ual l y appl i ed t o pat i ent s wi t h ext ens i ve burns or wi t h ext reme neut ropeni a. There are many di fferent pract i ces of prot ect i ve i s ol at i on, but careful handwas hi ng and us e of gl oves are probabl y mos t i mport ant .
o
o
e. Contact isolation Cohort i ng pat i ent s col oni zed wi t h res i s t ant organi s ms t oget her. For i ns t ance, pat i ent s known t o be col oni zed wi t h met hi c i l l i n-res i s t ant S. aureus (MRSA) onl y s hare
Pa g e 1 6 8 0
ABC Ambe r CHM Conve rte r Tria l ve rsion, http://w w w .proce sste x t.com/a bcchm.html
s emi -pri vat e rooms wi t h s i mi l arl y col oni zed pat i ent s .
Respiratory precautions For s ome di s eas es , es peci al l y t ubercul os i s , reduction of exposure of health care workers and other patients t o aeros ol dropl et s exhal ed by an i nfect ed pat i ent i s i mport ant . In t he s et t i ng of document ed or s us pect ed i nfect i on by s uch organi s ms , a pri vat e room wi t h appropri at e vent i l at i on i s requi red. Reduct i on of t i me s pent i n t he room and fas t i di ous us e of mas ks by vi s i t ors and heal t h care workers can reduce ri s k of t rans mi s s i on. Such pat i ent s mus t wear mas ks when out of t hei r rooms (i .e. goi ng for t es t s l i ke ches t radi ographs or CT s cans ).
V. Specific Infections According to Body Site A. Central nervous system (CNS) infections
1. Meningitis o
o
a. Acute meningitis i s an i nfl ammat ory di s eas e i nvol vi ng t he arachnoi d l ayer of t he meni nges and t he fl ui d t hat ci rcul at es i n t he vent ri cl es and t he s ubarachnoi d s pace—t he CSF.
(1) Classification. (Tabl e 8-2) There are t wo major cl as s i fi cat i ons of meni ngi t i s : bacterial and aseptic. In bot h forms , fever, headache, and s t i ff neck occur.
Pa g e 1 6 8 1
ABC Ambe r CHM Conve rte r Tria l ve rsion, http://w w w .proce sste x t.com/a bcchm.html
(2) Etiology. (Tabl e 8-3).
(a) Bacterial meningitis. The causes vary with the age of the patient. E. c ol i and St rept oc oc c us agal ac t i ae occur mos t frequent l y i n i nfant s ; H. i nfl uenzae, whi ch us ed t o predomi nat e i n young chi l dren (2 mont hs t o 6 years ), has been pract i cal l y eradi cat ed by uni vers al vacci nat i on; N. meni ngi t i di s i s mos t common i n adol es cent s and young adul t s ; and S. pneumoni ae i s mos t common i n adul t s ol der t han 25 years . Li s t eri a monoc yt ogenes , whi ch ent ers t he body t hrough t he gas t roi nt es t i nal t ract , i s found mos t commonl y i n cancer pat i ent s and i mmunos uppres s ed i ndi vi dual s .
(b) Aseptic meningitis. Us ual l y a viral disease, as ept i c meni ngi t i s may al s o refl ect an inflammatory process adjacent to the meninges (e.g., cerebri t i s , brai n abs ces s , s i nus i t i s , or ot i t i s ). In part i al l y t reat ed bact eri al meni ngi t i s , t he CSF us ual l y has a pyogeni c nat ure, al t hough ant i bi ot i cs may modi fy t he di s eas e i n s uch a way as t o creat e an as ept i c pat t ern. Drug hypers ens i t i vi t y al s o may caus e an as ept i c meni ngi t i s (us ual l y wi t h a predomi nance of neut rophi l s i n t he W BCs ) and may occur rarel y wi t h
Pa g e 1 6 8 2
ABC Ambe r CHM Conve rte r Tria l ve rsion, http://w w w .proce sste x t.com/a bcchm.html
i buprofen or a vari et y of ot her agent s .
(3) Clinical features. W hen bact eri a caus e meni ngi t i s , t hi s mani fes t at i on us ual l y i s part of a systemic, bacteremic infection. An except i on occurs when bact eri a gai n acces s t o t he meni nges aft er t rauma or s urgery or vi a a bony defect (us ual l y i n t he t emporal area or t he cri bri form pl at e). Typi cal pres ent at i on i ncl udes fever, nuchal ri gi di t y, and al t ered ment al s t at us .
(a) W i t h di s eas e due t o H. i nfl uenzae or S. pneumoni ae, focal i nfect i on s uch as pneumonia or otitis al s o may be apparent .
(b) W i t h di s eas e due t o N. meni ngi t i di s , t here may be a charact eri s t i c s ys t emi c i nfect i on cons i s t i ng of a petechial or purpuric skin rash and hypotension, whi ch can devel op rapi dl y.
(c) W i t h as ept i c meni ngi t i s due t o echovi rus es or coxs acki e vi rus es , t here may be a charact eri s t i c rash resembling rubella or a vesicular or petechial rash.
(4) T herapy
(a) Bacterial meningitis. Antibiotics
Pa g e 1 6 8 3
ABC Ambe r CHM Conve rte r Tria l ve rsion, http://w w w .proce sste x t.com/a bcchm.html
have a s i gni fi cant i mpact on t he out come of t hi s di s eas e. W i t hout t reat ment , deat h i s al mos t cert ai n; wi t h t reat ment , however, t he mort al i t y rat e i s reduced t o approxi mat el y 20% of t hos e pat i ent s who are not mori bund at t he t i me of di agnos i s . Bact eri ci dal ant i bi ot i cs s houl d be gi ven i n dos ages t hat permi t t he drug t o achi eve ki l l i ng l evel s i n t he CSF. Becaus e t he bl ood–brai n barri er bl ocks vi rt ual l y al l drugs and reduces drug penet rat i on from t he bl ood i nt o t he CSF, maxi mum t ol erat ed s ys t emi c dos ages s houl d be gi ven even at t he end of t he t reat ment cours e (us ual l y a t ot al of 2 weeks ).
TABLE 8-2 Examination of Cerebrospinal Fluid in Suspected Meningitis W B C (c Gl
el
u P ls c r / o ot m s ei L e n ) B L In 1 a o cr 0 ct w e 0 e
a to
ri
s >
Pa g e 1 6 8 4
ABC Ambe r CHM Conve rte r Tria l ve rsion, http://w w w .proce sste x t.com/a bcchm.html
al
e 1 d 0 0 0 wi th n e ut ro p hi l d o m in a nc
e F L N 1 u o or â n w m €“ g t o al 5 al n t o 0 or i n 0 m cr al e a s e d T L In 5 B o cr â T w e €“
Pa g e 1 6 8 5
ABC Ambe r CHM Conve rte r Tria l ve rsion, http://w w w .proce sste x t.com/a bcchm.html
u
a 1
b
s 0
e
e 0
rc
d
ul a r Vi N N 5 r or or â al m m €“ al al 3 0 0 wi th ly m p h oc yt e pr e d o m in a nc e W BC, whi t e bl ood cel l .
P.345
Pa g e 1 6 8 6
ABC Ambe r CHM Conve rte r Tria l ve rsion, http://w w w .proce sste x t.com/a bcchm.html
TABLE 8-3 Organisms in Bacterial Meningitis Pr edi sp Im osi po Or
ng rta
ga
Fa nt
nis Ag cto Poi m e rs nts Nei Chi Fat s s e l dr
al
ri a en
if
me an
not
ni n d
t re
gi t i you
at e
di s ng
d
ad
ear
ul t
l y,
s
di ff us e pur pur ic ras h, req ui r es pro phy
Pa g e 1 6 8 7
ABC Ambe r CHM Conve rte r Tria l ve rsion, http://w w w .proce sste x t.com/a bcchm.html
l ax is for exp os e d con t ac ts Gro Ne Ma up on t er B
at e nal
St r s
i nf
ept
ect
oc o
i on
ccu s, Es c her ich ia c ol i Li s Ne Im t eri on mu a
at e nos
mo s ,
up
noc el d pre yt o erl s s i ge y
on
nes St r Al l Chr Mo ept ag oni s t oc o es , c
co
c c u el d i l l n mm s
erl es s on
Pa g e 1 6 8 8
ABC Ambe r CHM Conve rte r Tria l ve rsion, http://w w w .proce sste x t.com/a bcchm.html
pn y
,
cau
eu
as p s e
mo
l en i n
ni a
ia
ad
e
ul t
Ha Ad
s Pre
em ul t
do
op s
mi
hi l
nat
us
ed
i nfl
in
ue
chi l
nza
dre
e
n unt il uni ver s al vac ci n at i on
St a Al l Bac phy ag t er l oc es em oc c
i a,
us
for
aur
ei g
eus
n bo dy, en doc
Pa g e 1 6 8 9
ABC Ambe r CHM Conve rte r Tria l ve rsion, http://w w w .proce sste x t.com/a bcchm.html
ard itis Gra Al l Se m- ag ps i ne es , s , gat el d ne i ve erl uro rod y
s ur
s
ger y
(i) In adul t s , i ni t i al t herapy wi t h a combi nat i on of a broad-s pect rum cephal os pori n and vancomyci n i s us ed i ni t i al l y. Frequent l y, t hi s can be s i mpl i fi ed once t he res ul t s of cul t ures and s ens i t i vi t i es have been obt ai ned. Ampi ci l l i n i s s t i l l us ual l y effect i ve agai ns t S. pneumoni ae and vi rt ual l y al ways effect i ve agai ns t N. meni ngi t i di s and Li s t eri a.
(ii) If res i s t ant organi s ms are found, t he appropri at e drug (e.g., cefot axi me or vancomyci n) i s begun or cont i nued, and ot her agent s are s t opped. S. pneumoni ae res i s t ant t o peni ci l l i n (and s omet i mes cephal os pori ns ) i s bei ng i s ol at ed more frequent l y. In cri t i cal l y i l l pat i ent s wi t h s us pect ed or es t abl i s hed pneumococcal meni ngi t i s , vancomyci n s houl d be
Pa g e 1 6 9 0
ABC Ambe r CHM Conve rte r Tria l ve rsion, http://w w w .proce sste x t.com/a bcchm.html
added t o t he regi men unt i l s us cept i bi l i t y i nformat i on i s avai l abl e.
(iii) If pyogeni c meni ngi t i s i s pres ent , a s hort cours e of cort i cos t eroi ds s uch as dexamet has one s t art i ng before or with t he ant i bi ot i cs can prevent deat h and s ubs t ant i al morbi di t y i n adul t s or chi l dren.
(b) Aseptic meningitis. There i s no specific chemotherapy for t hi s condi t i on. Any underl yi ng di s eas e s houl d be t reat ed. If drug hypers ens i t i vi t y i s s us pect ed, t he offendi ng agent s houl d be s t opped.
o
o
b. Chronic meningitis
(1) Etiology. The mos t common caus es of chroni c meni ngi t i s are t ubercul os i s , crypt ococcal di s eas e, mal i gnancy, and s arcoi dos i s .
(2) Clinical features. Chroni c meni ngi t i s may have an i ndol ent pres ent at i on, wi t h s ympt oms s i mi l ar t o t hos e of acut e meni ngi t i s or wi t h al t ered ment at i on wi t h or wi t hout fever. CSF abnormal i t i es may progres s i f t he underl yi ng di s eas e i s unt reat ed. In many of t hes e
Pa g e 1 6 9 1
ABC Ambe r CHM Conve rte r Tria l ve rsion, http://w w w .proce sste x t.com/a bcchm.html
di s eas es , t he CSF gl ucos e l evel i s l ow. Chroni c meni ngi t i s s houl d be cons i dered when a l ow CSF gl ucos e l evel i s not ed i n pat i ent s who are found not t o have acut e bact eri al meni ngi t i s .
(3) T herapy. Treat ment i s di rect ed at t he underl yi ng di s eas e. [See VII C 3 and VIII G 1 (1) (c) for TB and crypt ococcal t reat ment .]
o
o
2. Encephalitis
a. Etiology. Mos t encephal i t i des are caus ed by vi rus es , s everal of whi ch are t rans mi t t ed by t he bi t es of i nfect i ve mos qui t oes .
b. Clinical features. Pat i ent s wi t h encephal i t i s us ual l y pres ent wi t h al t ered ment at i on, s ei zures , or bot h. The CSF may be normal or have an as ept i c pat t ern.
c. Diagnosis. Di agnos i s i nvol ves meas uri ng ri s i ng t i t ers of ant i body t o one of t he encephal i t i s vi rus es i n pat i ent s wi t h a compat i bl e cl i ni cal s yndrome. W hen herpes simplex encephalitis i s s us pect ed, earl y ant i vi ral t reat ment i s recommended. Meas urement of herpes s i mpl ex vi rus â €“s peci fi c nucl ei c aci d i n CSF has l argel y s uppl ant ed brai n bi ops y for di agnos i s . Succes s ful t reat ment of herpes encephal i t i s depends on i ni t i at i on of medi cat i ons before
Pa g e 1 6 9 2
ABC Ambe r CHM Conve rte r Tria l ve rsion, http://w w w .proce sste x t.com/a bcchm.html
neurol ogi c det eri orat i on becomes ext ens i ve. W es t Ni l e vi rus was fi rs t des cri bed as a caus e of encephal i t i s i n New Y ork i n 1999, but s i nce t hen t hi s mos qui t o-borne i nfect i on has been found t hrough much of Nort h Ameri ca. Pat i ent s wi t h W es t Ni l e encephal i t i s can have a P.346
compl et e recovery of funct i on, but many pat i ent s have s evere weaknes s duri ng t he i nfect i on and may recover onl y part i al l y i f at al l .
d. Differential diagnosis. T oxopl as ma gondi i can caus e encephal i t i s i n pat i ent s wi t h di mi ni s hed T-cel l funct i on (e.g., due t o AIDS or pos t t rans pl ant at i on i mmunos uppres s i ve t herapy). Some i nt oxi cat i ons and i mmune di s eas es [e.g., s ys t emi c l upus eryt hemat os us (SLE)] may have a pres ent at i on i ndi s t i ngui s habl e from encephal i t i s . Endocardi t i s al s o s houl d be cons i dered. Ot her vi rus es t o cons i der i ncl ude CMV, Eps t ei n-Barr vi rus (EBV), HIV, and t he zoonot i c encephal i c vi rus es s uch as W es t ern equi ne encephal i t i s and St . Loui s encephal i t i s vi rus . Syphi l i s , dug t oxi ci t y, and SLE s houl d al s o be cons i dered i n an i ndi vi dual wi t h acut e encephal i t i s .
e. T herapy. Treat ment of vi ral encephal i t i s i s supportive, wi t h t he except i on of acycl ovi r for herpes s i mpl ex encephal i t i s . Drug i nt oxi cat i on and i mmune di s eas es are
Pa g e 1 6 9 3
ABC Ambe r CHM Conve rte r Tria l ve rsion, http://w w w .proce sste x t.com/a bcchm.html
t reat abl e, and pat i ent s wi t h as s oci at ed encephal i t i s us ual l y res pond. o
o
3. Intracranial abscess. Suppurat i ve i nfect i ons can i nvol ve t he cont ent s of t he cal vari um, us ual l y by di rect s pread from an i nfect ed s i nus or ear.
a. Etiology. The bact eri ol ogy of i nt racrani al i nfect i ons refl ect s t he t ypes of organi s ms t hat caus e di s eas e i n more s uperfi ci al cont i guous s t ruct ures s uch as s t rept ococci and anaerobi c bact eri a. Abs ces s es can be l ocal i zed t o t he ext radural (al s o cal l ed epi dural ) or s ubdural s paces or i n t he brai n parenchyma. Rarel y, hemat ogenous s pread of bact eri a can gi ve ri s e t o i nt racrani al abs ces s es . Les s common agent s are T oxopl as ma, Noc ardi a, and Crypt oc oc c us s peci es , whi ch us ual l y are s een i n i mmunocompromi s ed pat i ent s wi t h reduced hel per T-cel l numbers or funct i on.
b. Diagnosis. Magnetic resonance imaging (MRI) (more s ens i t i ve) or CT (more wi del y avai l abl e and l es s expens i ve) of t he brai n i s hel pful i n maki ng t he di agnos i s . Earl y s cans may be equi vocal , but s t udi es repeat ed wi t hi n a few days are al mos t al ways pos i t i ve, es peci al l y wi t h t he us e of cont ras t . Defi ni t i ve di agnos i s requi res as pi rat i on for s t ai n and cul t ure.
c. T herapy. Bact eri al abs ces s es are t reat ed wi t h appropri at e antibiotics t hat penet rat e
Pa g e 1 6 9 4
ABC Ambe r CHM Conve rte r Tria l ve rsion, http://w w w .proce sste x t.com/a bcchm.html
brai n t i s s ue wel l . Thes e may i ncl ude peni ci l l i n, chl orampheni col , and met roni dazol e. The us ual t reat ment i s gi ven for i nfect i on by T oxopl as ma (e.g., pyri met hami ne, s ul fadi azi ne), Crypt oc oc c us (e.g., amphot eri ci n B, fl ucyt os i ne), or Noc ardi a (e.g., a s ul fa drug wi t h or wi t hout t ri met hopri m). Surgi cal exci s i on or decompres s i on i s not al ways needed, but s ome pat i ent s , es peci al l y t hos e wi t h l arge l es i ons or s l ow res pons e t o t reat ment , requi re serial aspirations or more aggres s i ve s urgi cal débri dement .
B. Head and neck infections
1. Otitis. Infect i ons of t he ear can i nvol ve any of t he ear's t hree major anat omi c areas —t he out er, mi ddl e, or i nner ear. o
o
a. Outer ear infections t end t o be mi nor i rri t at i ons of t he ext ernal audi t ory canal . Topi cal ant i bi ot i c t reat ment i s us ed for ext ernal ot i t i s . An except i on i s malignant otitis externa, a des t ruct i ve proces s mos t commonl y found i n di abet i c pat i ent s (s ee IV A 4 a). Mal i gnant ot i t i s ext erna requi res ant i bi ot i cs t hat are effect i ve agai ns t P. aerugi nos a i nfect i on pl us s urgi cal débri dement and drai nage.
o
o
b. Middle ear infection, or otitis media, t ypi cal l y i s a di s eas e of chi l dren and mani fes t s as ear pai n and reduced audi t ory acui t y. Ot os copy s hows a dul l , poorl y mobi l e t ympani c membrane wi t h or
Pa g e 1 6 9 5
ABC Ambe r CHM Conve rte r Tria l ve rsion, http://w w w .proce sste x t.com/a bcchm.html
wi t hout pus behi nd i t . The mos t common caus es are pneumococci , St rept oc oc c us pyogenes , Moraxel l a c at arrhal i s , and H. i nfl uenzae. In chroni c cas es , es peci al l y i f mul t i pl e cours es of ant i bi ot i cs have been gi ven, ent eri c Gram-negat i ve rods and anaerobes may be i nvol ved. Therapy for acut e ot i t i s medi a i s amoxi ci l l i n, amoxi ci l l i n wi t h cl avul ani c aci d, t ri met hopri m wi t h s ul famet hoxaz ol e, a macrol i de s uch as azi t hromyci n or cl ari t hromyci n, or an oral cephal os pori n. Cul t ures and appropri at e t herapy are s ugges t ed for chroni c ot i t i s medi a. o
o
c. Inner ear infections rarel y are caus ed by bact eri a. However, s everal vi rus es may be as s oci at ed wi t h a s yndrome of vertigo wi t h or wi t hout tinnitus. No t reat ment i s hel pful .
2. Sinusitis. (Fi gure 8-2) The paranas al s i nus es have cont i nuous expos ure t o t he ext ernal envi ronment vi a t he os t i a i n t he nos e. Under normal condi t i ons , hos t defens es mai nt ai n t he s t eri l e envi ronment of t he s i nus es . However, when mucoci l i ary cl earance i s i nt errupt ed becaus e of s t ruct ural or funct i onal abnormal i t i es , i nfect i on can s upervene. In addi t i on, pat i ent s wi t h mi d- t o l at e-s t age HIV i nfect i on are prone t o recurrent or chroni c s i nus i t i s . P.347
Pa g e 1 6 9 6
ABC Ambe r CHM Conve rte r Tria l ve rsion, http://w w w .proce sste x t.com/a bcchm.html
FIGURE 8-2 Si nus i t i s . o
o
a. Etiology. As i n mi ddl e ear i nfect i ons , virulent bacteria s uch as pneumococci and H. i nfl uenzae are mos t l i kel y t o caus e acut e di s eas e, and anaerobi c or ent eri c organi s ms are as s oci at ed wi t h more chroni c i nfect i ons . Bact eri al s i nus i t i s may fol l ow and mi mi c vi ral upper res pi rat ory i nfect i ons ; however, onl y t he bact eri al i nfect i ons go on t o devel op s uppurat i ve compl i cat i ons of t he CNS.
o
o
b. Diagnosis. Sinus radiography s hows mucos al t hi ckeni ng or opaci fi cat i on or ai r–fl ui d l evel s i n s i nus i t i s . Tendernes s and edema hel p l ocal i ze di s eas e t o t he s i nus es but are not pres ent i n al l pat i ent s . CT scanning i s more s ens i t i ve t han s i nus radi ography, but i t s houl d not be performed i n t he earl y s t ages of “col ds ,― becaus e mi nor s i nus abnormal i t i es are commonl y s een on CT s can i n pat i ent s wi t hout s i gni fi cant ongoi ng s i nus i t i s . Mos t cas es of s i nus i t i s are di agnos ed cl i ni cal l y, and i magi ng i s res erved for pat i ent s wi t h refract ory or
Pa g e 1 6 9 7
ABC Ambe r CHM Conve rte r Tria l ve rsion, http://w w w .proce sste x t.com/a bcchm.html
recurrent di s eas e. o
o
c. T herapy
(1) In mi l d cas es deconges t ant s al one are adequat e.
(2) Funct i onal endoscopic sinus surgery can be us eful i n pat i ent s wi t h repeat ed or pers i s t ent i nfect i on.
3. Odontogenic infections, t he mos t common i nfect i ons occurri ng i n t he oral cavi t y, us ual l y are l ocal and res pond t o s i mpl e meas ures s uch as drai ni ng abs ces s es , res t ori ng cari ous t eet h, and mai nt ai ni ng good oral hygi ene. Somet i mes , however, s oft t i s s ue i nfect i ons i n t he mout h can di s s ect t hrough t i s s ue pl anes and i nvol ve deeper s t ruct ures of t he face or neck. Exampl es are Ludwig' s angina, whi ch i s an i nfect i on ext endi ng t o t he fl oor of t he mout h, and retropharyngeal abscess, whi ch can t rack down t o t he medi as t i num. The i nvol ved bact eri a are s t rept ococci and i ndi genous oral anaerobes .
4. Eye infections. Normal l y, t he eyes are res i s t ant t o i nfect i ons . o
o
a. Conjunctivitis. Superfi ci al i nfect i ons of t he conjunct i va us ual l y are bact eri al or vi ral and res ol ve s pont aneous l y. An except i on i s gonorrheal conjunctivitis, a rare adul t di s eas e t hat mus t be t reat ed vi gorous l y.
Pa g e 1 6 9 8
ABC Ambe r CHM Conve rte r Tria l ve rsion, http://w w w .proce sste x t.com/a bcchm.html o
o
b. Keratitis. Becaus e t he t rans parency of t he cornea i s cruci al for vi s i on, di agnos t i c t es t s s uch as bact eri al and vi ral s mears and cul t ures are warrant ed. Ini t i al t herapy i s gui ded by t he cl i ni cal pi ct ure, becaus e a vari et y of organi s ms can caus e kerat i t i s , i ncl udi ng S. aureus , P. aerugi nos a, s t rept ococci , numerous fungi , and herpes s i mpl ex and vari cel l a-zos t er vi rus es . It i s i mport ant t o recogni ze kerat i t i s caus ed by herpes simplex virus s o t hat appropri at e ant i vi ral t herapy t o prevent bl i ndnes s can be i ns t i t ut ed.
o
o
c. Endophthalmitis. Bact eri a mos t commonl y caus e t hi s i nfect i on of t he i nt ernal s t ruct ures of t he eye, whi ch may fol l ow eye s urgery or i nfect i on el s ewhere i n t he body. Sys t emi c and t opi cal ant i bi ot i cs , s el ect ed on t he bas i s of cl i ni cal fi ndi ngs and Gram s t ai n of ocul ar mat eri al , are admi ni s t ered t o prevent i rrevers i bl e des t ruct i on of t he eye. However, t he prognos i s for normal vi s ual acui t y aft er endopht hal mi t i s i s poor.
P.348
C. Respiratory tract infections
1. Upper respiratory infections. Infect i ons i nvol vi ng t he nos e, t hroat , l arynx, ai rways , and adjacent s t ruct ures are t he mos t common caus es of morbi di t y i n t he Uni t ed St at es . o
Pa g e 1 6 9 9
ABC Ambe r CHM Conve rte r Tria l ve rsion, http://w w w .proce sste x t.com/a bcchm.html
o
a. Etiology. Upper res pi rat ory i nfect i ons al mos t i nvari abl y are vi ral .
o
o
b. Clinical features. Thes e “col ds ― and “fl us ― i nvol ve rhi norrhea, coryz a, cough, a s l i ght fever, and, s omet i mes , s ore t hroat . Duri ng s eas ons when i nfl uenz a i s epi demi c i n a communi t y, headache, cough, myal gi a, and a more marked t emperat ure el evat i on may s ugges t t he di agnos i s .
o
o
c. T herapy. Treat ment for “col ds ― i s s ympt omat i c. Infl uenz a A vi rus i nfect i ons may res ol ve more qui ckl y when an ant i vi ral s uch as amant adi ne, ri mant adi ne, os el t ami vi r, or zanami vi r i s admi ni s t ered wi t hi n 24–48 hours . Thes e medi cat i ons can be us eful i n prevent i ng i nfl uenz a i n hi gh-ri s k pat i ent s when i mmuni zat i on i s not feas i bl e or i s performed t oo l at e t o offer meani ngful prot ect i on.
o
d.
Prevention. Becaus e of t he ubi qui t y and ext reme i nfect i vi t y of
mos t epi demi c vi rus es , prevent i on of mos t upper res pi rat ory di s eas e i s di ffi cul t . Careful handwas hi ng i s a s i mpl e meas ure t o prevent t he s pread of upper res pi rat ory i nfect i ons . In many cas es , t i mel y admi ni s t rat i on of vacci ne devel oped agai ns t t he epi demi c s t rai ns t hat exi s t duri ng a part i cul ar s eas on al s o may prevent i nfl uenz a. Immuni zat i on s houl d be di rect ed at i ndi vi dual s who are at great es t ri s k for compl i cat i ons of upper res pi rat ory i nfect i on s uch as el derl y or chroni cal l y i l l i ndi vi dual s . Y earl y i mmuni zat i on has a favorabl e i mpact on i nfl uenz a morbi di t y and mort al i t y. In addi t i on, i mmuni z at i on of heal t hy adul t s may reduce abs ent eei s m at work, and vacci nat i on of heal t h care provi ders may prevent or s l ow
Pa g e 1 7 0 0
ABC Ambe r CHM Conve rte r Tria l ve rsion, http://w w w .proce sste x t.com/a bcchm.html
epi demi cs of i nfl uenz a i n nurs i ng homes and hos pi t al s .
2. Pharyngitis o
o
a. Etiology. The major et i ol ogi c agent s of pharyngi t i s , a common i l l nes s , are vi rus es , and S. pyogenes (group A s t rept ococci ).
o
o
b. Clinical features. Sore throat t hat occurs wi t h or wi t hout object i ve fi ndi ngs of eryt hema or exudat e on t he oropharynx or t ons i l s i s charact eri s t i c.
o
o
c. T herapy
(1) Treat ment for group A s t rept ococcal pharyngi t i s may accel erat e heal i ng and reduce s ympt oms , but t he major purpos e of t reat i ng t hi s di s order i s prevention of subsequent rheumatic fever, a rare s equel a.
(2) Vi ral pharyngi t i s does not i mprove wi t h any known chemot herapy.
(3) W het her t reat ment of mycopl as mal or chl amydi al pharyngi t i s i s benefi ci al i s not known.
3. T racheobronchitis i s an i nfect i on of t he ai rways . o
Pa g e 1 7 0 1
ABC Ambe r CHM Conve rte r Tria l ve rsion, http://w w w .proce sste x t.com/a bcchm.html
o
a. Etiology. Mos t cas es of t racheobronchi t i s are as s oci at ed wi t h i nfect i on by mycopl as ma or by viruses, s uch as i nfl uenz a or parai nfl uenz a vi rus (i n adul t s ) or res pi rat ory s yncyt i al vi rus (i n young chi l dren). However, bact eri a may pl ay a rol e i n pat i ent s wi t h chroni c obs t ruct i ve pul monary di s eas e (COPD) and cys t i c fi bros i s . H. i nfl uenzae, S. aureus , and P. aerugi nos a are bel i eved t o be pat hogeni c i n t hos e wi t h cys t i c fi bros i s (s ee Chapt er 2 IV B).
o
o
b. Clinical features and laboratory findings. Cough wi t h abundant , t hi ck s put um i s charact eri s t i c of t racheobronchi t i s . Fever, ches t pai n, and wheez i ng al s o may occur. If ral es or cons ol i dat i on i s found, pneumoni a i s s ugges t ed. The W BC count and ches t radi ograph are unchanged from bas el i ne i n uncompl i cat ed t racheobronchi t i s .
o
o
c. T herapy. Si mpl e s upport i ve meas ures us ual l y s uffi ce. Ant i bi ot i cs us ual l y are gi ven t o pat i ent s wi t h COPD exacerbat i on, al t hough t he benefi t of s uch t herapy i s modes t and mos t pronounced when al l t hree el ement s of exacerbat i on are pres ent : dys pnea, i ncreas ed vol ume of s put um, and i ncreas ed purul ence of s put um. Pat i ent s wi t h cys t i c fi bros i s undergo ches t phys i cal t herapy and recei ve ant i mi crobi al agent s accordi ng t o s put um bact eri ol ogy (s ee Chapt er 2 IV B 6 b).
4. Pneumonia o
o
a. Etiology. Caus es i ncl ude bact eri a, vi rus es , fungi ,
Pa g e 1 7 0 2
ABC Ambe r CHM Conve rte r Tria l ve rsion, http://w w w .proce sste x t.com/a bcchm.html
and paras i t es . Al t hough i dent i fi cat i on of t he s peci fi c caus e of pneumoni a i s i deal , many pat i ent s can be t reat ed i ni t i al l y on t he bas i s of cl i ni cal and demographi c feat ures . o
o
b. Clinical features and diagnosis
(1) Patient history may i ndi cat e an underl yi ng condi t i on. For exampl e, t he i nci dence of bact eri al pneumoni a i s i ncreas ed i n as s oci at i on wi t h COPD or al cohol i s m. Fever i s us ual l y, but not i nvari abl y, pres ent . Evi dence of ext ra fl ui d i n t he l ungs i s not ed as ral es or cons ol i dat i on on phys i cal exami nat i on and i nfi l t rat e on ches t radi ograph. In addi t i on, t he appearance of t he s put um can be us eful for maki ng t he appropri at e di agnos i s .
(2) Becaus e of t he vast differential diagnosis, i t i s hel pful t o cons i der pneumoni a i n t wo ways (recogni zi ng t hat a cons i derabl e overl ap i s pos s i bl e):
(a) W het her i t devel oped at home (community-acquired) or i n a hos pi t al or i ns t i t ut i on (hospital-acquired or nosocomial)
(b) W het her i t had a rapi d ons et wi t h chi l l s , fever, and cough (classical) or a more i ndol ent ons et (atypical)
Pa g e 1 7 0 3
ABC Ambe r CHM Conve rte r Tria l ve rsion, http://w w w .proce sste x t.com/a bcchm.html
P.349
o
o
c. Pneumonia syndromes (Tabl e 8-4)
(1) Classical community-acquired (CAP) pneumonia
(a) Etiology. Thi s s yndrome mos t frequent l y i s caus ed by S. pneumoni ae. However, H. i nfl uenzae, M. c at arrhal i s , and ent eri c Gram-negat i ve baci l l i al s o can caus e t hi s cl i ni cal pi ct ure.
(b) Diagnosis. Gram s t ai ni ng of expect orat ed s put um s hows l arge numbers of neut rophi l s (i .e., >25 per l ow-power fi el d) and few s quamous cel l s (<10 per l ow-power fi el d). The bact eri al fl ora may be mi xed, but oft en t here i s a predomi nance of one morphol ogi c t ype s uch as t he l ancet -s haped Gram-pos i t i ve cocci , whi ch appear i n pai rs i n pneumococcal pneumoni a.
(c) T herapy. W hen t he pres ent at i on i s t rul y cl as s i c, wi t h a s i ngl e s haki ng chi l l , rus t -col ored s put um, and a moderat e fever accompani ed by a Gram s t ai n s ugges t i ng pneumococci , peni ci l l i n us ed t o be t he drug of choi ce. Becaus e s o
Pa g e 1 7 0 4
ABC Ambe r CHM Conve rte r Tria l ve rsion, http://w w w .proce sste x t.com/a bcchm.html
many areas have a hi gh i nci dence of pneumococcal peni ci l l i n res i s t ance, an al t ernat i ve agent s uch as a macrol i de or fl uoroqui nol one act i ve agai ns t Gram-pos i t i ve bact eri a (e.g., gat i fl oxaci n, moxi fl oxaci n, or l evofl oxaci n) i s preferred as i ni t i al t reat ment .
TABLE 8-4 Classification of Pneumonias Comm unityAcquir ed Pneu monia (CAP) H o s A pi s ta pi lr A at c io q n ui P r n e e d T A
u P
y ty
m n
pi pi Vi o e c c r ni u al al al a m
Pa g e 1 7 0 5
ABC Ambe r CHM Conve rte r Tria l ve rsion, http://w w w .proce sste x t.com/a bcchm.html
o ni a Et M M A Gr P io o yc d a s lo s t o e m e g co pl n -n u y ma o e d m s vi g o o m ru at m n: a s i v o St p P e n re n ar or a pt e ai g s o u nf a a c m l u ni n o o e s d c c ni n m t y u a z s pi s e a fr ca p ( vi o l n W ru m p e al s g at u ki R ut h mn e A o o g s n g ni p pi a e a n ra er n Al e t o o s s u ry b wi o: m s y e t h H o nc s hi a ni yt fr g e a) i a o h mC l
m er
Pa g e 1 7 0 6
ABC Ambe r CHM Conve rte r Tria l ve rsion, http://w w w .proce sste x t.com/a bcchm.html
o hl vi m re p a ru o s i hi m s ut s t lu y
h a
s di
fl nc
in a
or e
fl p
a p
u n
at
e e
te
nz u
rn
a m
s
e, o M ni or a a e x el la c at ar rh al is Cl Cl M
R In
in a or
el cl
ic s s e
at u
al i c i n
e d
c al d
d e
o l y ol
to s
n wi e
E n
si t h nt
T ur
d s co
O si
e h ur
H n
r a s
u g
Pa g e 1 7 0 7
ABC Ambe r CHM Conve rte r Tria l ve rsion, http://w w w .proce sste x t.com/a bcchm.html
at ki e.
s h
io n M
e, o
n g y
al m
s ch al
te e
i l l gi
re s
s , a,
d
fe ar
m
v th
e
er ra
nt
, lg
al
pr i a
st
o m
at
d or
u
uc e
s,
t i pr
s
v o
ei
e m
z
co i n
ur
u e
e
g nt
s,
h,
C
pl
V
e
A
ur iti c p ai n G L L N Mi Si r ar e o x m a g s s or e i l m e di g d ar st n a a m t o ai u g ni i c C
Pa g e 1 7 0 8
ABC Ambe r CHM Conve rte r Tria l ve rsion, http://w w w .proce sste x t.com/a bcchm.html
n m n s ro A b o m bi P er s t s al of i c o n
b
e
s
ut
er
ro
v
p
e
hi
d
ls wi th pr e d o m in a nc e of o n e or g a ni s m C G P P L L h e at at o o e n ch ch b b
Pa g e 1 7 0 9
ABC Ambe r CHM Conve rte r Tria l ve rsion, http://w w w .proce sste x t.com/a bcchm.html
st er y y ar ar x al bi bi co or -r l y l a l a n m a l o t e t e s ul y b ra ra ol t i l ar l
l
id o
co i n i n at b n fi l fi l i o ar s t r t r n co ol at at m n id e e o s at s s s t ol io
li id
n
k at el i o y n in s ri g ht m id dl e or lo w er lo b
e T M M S Cl Br h ac ac u i n o e ro ro p d a r li li p a d
Pa g e 1 7 1 0
ABC Ambe r CHM Conve rte r Tria l ve rsion, http://w w w .proce sste x t.com/a bcchm.html
a d d or m s p e e t i yc p y or or v i n ec fl fl e t o t u u ca co ru or or re v m o o
er a
q q
a nt
ui ui
n ip
n n
a s
ol ol
er e
o o
o u
n n
b d
e e
e o
a
s m
s
in o
a
a n
n
d al
o
di (c
ut
t i ef
p
o e
at
n pi
ie
to m
nt
m e,
If
ac i
h
ro m
o
li ip
s
d e
pi
e n
ta
or e
li
fl m
z
u ,
e
or pi
d,
o p
a
q er
Pa g e 1 7 1 1
ABC Ambe r CHM Conve rte r Tria l ve rsion, http://w w w .proce sste x t.com/a bcchm.html
zi
ui ac
th
n ill
ro
ol i n
m
o -t
yc
n a
in
e z
+
o
ce
b
ft
ac
ri
ta
a
m
x
)
o
Â
n
±
e
m
or
ac
IV
ro
fl
li
u
d
or
e
o
fo
q
r
ui
co
n
v
ol
er
o
a
n
g
e
e of at y pi ca ls
CVA,
Pa g e 1 7 1 2
ABC Ambe r CHM Conve rte r Tria l ve rsion, http://w w w .proce sste x t.com/a bcchm.html
cardi ovas cul a r acci dent ; ETOH, et hyl al cohol ; IV, i nt ravenous .
P.350
(i) A macrol i de or fl uoroqui nol one i s us ed i n peni ci l l i n-al l ergi c pat i ent s when s ympt oms bl end i nt o t hos e s een i n t he at ypi cal form or when t he Gram s t ai n i s i nconcl us i ve.
(ii) If t he Gram s t ai n s hows Gram-negat i ve rods , t herapy appropri at e for Haemophi l us or ent eri c Gram-negat i ve baci l l i i s gi ven, us ual l y i n t he hos pi t al .
(iii) If Gram-negat i ve cocci appear s i ngl y or i n pai rs , t reat ment s houl d be di rect ed t oward M. c at arrhal i s , wi t h t ri met hopri m-s ul famet hoxaz ol e or a macrol i de.
Pa g e 1 7 1 3
ABC Ambe r CHM Conve rte r Tria l ve rsion, http://w w w .proce sste x t.com/a bcchm.html
(2) Atypical community-acquired pneumonia
(a) Etiology. Thi s s yndrome us ual l y i s caus ed by M. pneumoni ae or C. pneumoni ae, al t hough a vari et y of vi rus es may be res pons i bl e, i ncl udi ng adenovi rus , parai nfl uenz a vi rus , and res pi rat ory s yncyt i al vi rus .
(b) Diagnosis. At ypi cal CAP i s l es s s evere t han t he cl as s i cal form, wi t h a l es s di agnos t i c s put um. Sys t emi c s ympt oms (e.g., myal gi a, art hral gi a, and s ki n ras h) are more promi nent , and ches t compl ai nt s (e.g., pl euri t i c pai n and product i ve cough) are l es s marked i n at ypi cal pneumoni a t han i n cl as s i cal pneumoni a. Gram s t ai n s hows s ome neut rophi l s and a vari et y of bact eri al forms . Cul t ures s el dom are di agnos t i c. Ches t radi ographs us ual l y s how pat chy, bi l at eral i nfi l t rat es and l i t t l e or no pl eural effus i on (Fi gure 8-3).
(c) Differential diagnosis
(i) Aspiration pneumonia may res ul t from pul monary as pi rat i on of oral s ecret i ons , es peci al l y wi t h di mi ni s hed prot ect i on of t he ai rways (e.g., aft er uncons ci ous nes s , a s ei zure, or vocal cord paral ys i s ).
Pa g e 1 7 1 4
ABC Ambe r CHM Conve rte r Tria l ve rsion, http://w w w .proce sste x t.com/a bcchm.html
(ii) T uberculosis i s al s o a cons i derat i on i n a cas e of a s l owl y evol vi ng pul monary i nfect i on. However, t here us ual l y i s ot her evi dence of t ubercul os i s (e.g., radi ographi c fi ndi ngs , an expos ure hi s t ory, or a pos i t i ve aci d-fas t s mear of t he s put um) [s ee VII C].
(iii) Legionnaires' disease al s o may occur i n t hi s way (s ee VII E).
(d) T herapy. Pat i ent s wi t h at ypi cal pneumoni a oft en pres ent aft er t hey have had many days or over a week of s ympt oms . Effect i ve t reat ment may not bri ng i mmedi at e rel i ef but may furt her s hort en t he cours e of t he i l l nes s . Any of t he macrol i des or P.351
fl uoroqui nol ones i s appropri at e, and t he oral rout e i s adequat e. Therapy for ot her noncl as s i cal pneumoni as i nvol ves t he fol l owi ng meas ures :
Pa g e 1 7 1 5
ABC Ambe r CHM Conve rte r Tria l ve rsion, http://w w w .proce sste x t.com/a bcchm.html
FIGURE 8-3 Pneumoni a caus ed by Kl ebs i el l a pneumoni ae. Ches t x-ray reveal s ext ens i ve cons ol i dat i on of t he l eft l ower l obe. Cavi t y format i on i s apparent wi t hi n t he i nvol ved area (arrow). (From Engl eberg NC, Dermody T, Di Ri t a V. Sc haec ht er's Mec hani s ms of Mi c robi al Di s eas e. 4t h ed. Bal t i more: Li ppi ncot t W i l l i ams & W i l ki ns , 2006. )
(i) W hen aspiration pneumonia i s s us pect ed, an ant i bi ot i c wi t h cons i s t ent act i vi t y agai ns t oral anaerobes and s t rept ococci (e.g., peni ci l l i n or cl i ndamyci n) i s s ugges t ed.
(ii) T uberculosis al ways i s t reat ed wi t h combi nat i ons of ant i bi ot i cs (s ee VII C 3).
(iii) Legionnaires' disease us ual l y res ponds t o macrol i des or fl uoroqui nol ones , al t hough s ome s everel y i l l pat i ent s may requi re
Pa g e 1 7 1 6
ABC Ambe r CHM Conve rte r Tria l ve rsion, http://w w w .proce sste x t.com/a bcchm.html
hos pi t al i zat i on (s ee VII E 5).
(3) Classical (acute) hospital-acquired pneumonia
(a) Etiology. Thi s s yndrome can be caus ed by a vari et y of bact eri a. Pat i ent s oft en are granul ocyt openi c, pos t operat i ve, or i nt ubat ed. Becaus e t hes e pat i ent s us ual l y have pharyngeal col oni zat i on by ent eri c Gram-negat i ve aerobes , t he pneumoni a i s l i kel y t o i nvol ve t hes e organi s ms . Al l ri s k fact ors s houl d be t aken i nt o cons i derat i on when as s es s i ng t hes e pat i ent s .
(b) Diagnosis. Gram s t ai n and cul t ure of s put um are i mport ant i n det ermi ni ng t he organi s ms i nvol ved i n t hi s i nfect i on. Even t hough good res pi rat ory s peci mens can be hard t o obt ai n, i t i s wort h t he effort t o make a mi crobi al di agnos i s for pneumoni a pat i ent s ei t her i n or recent l y di s charged from t he hos pi t al . More i nvas i ve t es t s s uch as bronchos copy and l ung bi ops y may be needed t o confi rm t he di agnos i s .
(c) T herapy. W hen t he Gram s t ai n s ugges t s i nfl ammat i on and s hows abundant Gram-negat i ve rods , recommended t reat ment i s a β-l act am ant i bi ot i c wi t h act i vi t y agai ns t a wi de
Pa g e 1 7 1 7
ABC Ambe r CHM Conve rte r Tria l ve rsion, http://w w w .proce sste x t.com/a bcchm.html
vari et y of ent eri c Gram-negat i ve organi s ms , wi t h or wi t hout an ami nogl ycos i de. Knowl edge of s us cept i bi l i t y pat t erns i n t he hos pi t al can hel p i n choos i ng empi ri c t herapy. W hen cul t ure res ul t s are avai l abl e, more s peci fi c t herapy can be gi ven. In pat i ent s wi t h neut ropeni a, broad coverage (i ncl udi ng act i vi t y agai ns t P. aerugi nos a) i s us ed. If Legi onel l a i s s eri ous l y cons i dered, a macrol i de or fl uoroqui nol one s houl d be added. Hos pi t al s wi t h s i gni fi cant rat es of i nfect i on caus ed by met hi ci l l i n-res i s t ant S. aureus are s et t i ngs where t he us e of vancomyci n i s prudent when t he Gram s t ai n s ugges t s a s t aphyl ococcal i nfect i on or where t he s peci men coul d not be obt ai ned.
5. Lung abscess o
o
a. Etiology. Thi s i nfect i on may be t he res ul t of a pyogeni c pneumoni a caus ed by pat hogens s uch as S. aureus or S. pyogenes . More commonl y, however, l ung abs ces s occurs as a l at e s t age i n t he evol ut i on of an unt reat ed anaerobi c or mi xed fl ora pneumoni a, s uch as when oral or gas t ri c cont ent s ent er t he ai rway. Neurol ogi c i mpai rment of normal gl ot t al refl exes predi s pos es pat i ent s t o as pi rat i on and l ung abs ces s .
o
o
b. Clinical features. Approxi mat el y hal f of pat i ent s
Pa g e 1 7 1 8
ABC Ambe r CHM Conve rte r Tria l ve rsion, http://w w w .proce sste x t.com/a bcchm.html
are afebri l e, and s ome have l ong-s t andi ng cons t i t ut i onal compl ai nt s s uch as wei ght l os s . A foul -s mel l i ng s put um i s hi ghl y s ugges t i ve of anaerobi c i nfect i on. o
o
c. T herapy. Mos t l ung abs ces s es drai n i nt o t he t racheobronchi al t ree and can be cured wi t h appropri at e ant i mi crobi al t herapy, al t hough t he durat i on of t herapy oft en i s l ong.
6. T horacic empyema o
o
a. Etiology. Thi s i nfect i on al s o i s rel at ed t o underl yi ng pneumoni a. Any pyogeni c pneumoni a may gi ve ri s e t o a pl eural effus i on, whi ch may become i nfect ed. Rarel y, anaerobi c empyema i s t he onl y mani fes t at i on of t horaci c i nfect i on; pres umabl y, t hi s occurs as a s equel a t o an anaerobi c pneumoni a or l ung abs ces s , whi ch may not be apparent at t he t i me of t he empyema.
o
o
b. T herapy
(1) Noni nfect ed parapneumoni c effus i ons wi t h a rel at i vel y l ow W BC count and a pH above 7.2 us ual l y res ol ve wi t h s ys t emi c ant i mi crobi al t herapy.
(2) Infect ed effus i ons , or t hos e wi t h a very l ow pH or a hi gh W BC count , us ual l y requi re t horacos t omy drai nage.
Pa g e 1 7 1 9
ABC Ambe r CHM Conve rte r Tria l ve rsion, http://w w w .proce sste x t.com/a bcchm.html
D. Gastrointestinal infections
1. Food poisoning. Thes e s yndromes repres ent a di vers e col l ect i on of i nt oxi cat i ons or i nfect i ons as s oci at ed wi t h i nges t i on of food or beverage t hat has been cont ami nat ed wi t h pat hogeni c organi s ms , t oxi ns , or chemi cal s . Di s regardi ng t he purel y chemi cal ent i t i es s uch as heavy met al poi s oni ng, mus hroom poi s oni ng (s ee Chapt er 5 onl i ne IV D 5 b), and pol ychl ori nat ed bi phenyl (PCB) poi s oni ng, food poi s oni ng s yndromes can be di vi ded i nt o t hos e caus i ng gastrointestinal symptoms and t hos e caus i ng neurologic symptoms. In al l cas es of s us pect ed food poi s oni ng, s ome epi demi ol ogi c as s es s ment s houl d be done t o i dent i fy ot her i nfect ed i ndi vi dual s and t o report out breaks t o publ i c heal t h aut hori t i es .
P.352
o
o
a. Gastrointestinal food poisoning i s t he mos t fami l i ar t ype of food poi s oni ng (s ee Chapt er 5 IV D 4 and Tabl e 5-1). Di arrhea, naus ea, vomi t i ng, and abdomi nal pai n are t he mos t common mani fes t at i ons .
(1) Clinical categories us ual l y are bas ed on t he i ncubat i on peri od, t he food vehi cl e, and t he nat ure of t he gas t roi nt es t i nal compl ai nt .
(a) Preformed t oxi ns gi ve ri s e t o t he s hort es t -ons et s yndromes . For exampl e, t he ent erot oxi n-medi at ed s yndromes caus ed by s t aphyl ococci , Bac i l l us c ereus , and Cl os t ri di um perfri ngens mani fes t wi t hi n a few hours aft er i nges t i on of t he
Pa g e 1 7 2 0
ABC Ambe r CHM Conve rte r Tria l ve rsion, http://w w w .proce sste x t.com/a bcchm.html
cont ami nat ed food.
(b) Di s eas es t hat depend on t he growt h of bact eri a requi re a peri od of hours t o days t o mani fes t , and i t may be di ffi cul t t o rel at e t he i l l nes s t o a s peci fi c expos ure. Somet i mes cul t ures of vari ous l eft over foods are hel pful ; i n ot her cas es , careful recal l of foods cons umed i s di agnos t i c.
(2) T herapy. Al mos t al l forms of gas t roi nt es t i nal food poi s oni ng are s el f-l i mi t ed and remi t s pont aneous l y wi t hi n 1 or 2 days . Careful repl acement of fl ui d and el ect rol yt es may be needed for young chi l dren or t hos e adul t s wi t h s i gni fi cant underl yi ng medi cal di s eas e, but wat chful wai t i ng i s us ual l y s uffi ci ent .
o
o
b. Neurologic food poisoning i s rare. However, i t i s i mport ant t o es t abl i s h t he di agnos i s s o t hat appropri at e t herapy can be i ns t i t ut ed i mmedi at el y.
(1) Botulism. Thi s s yndrome i s an i nt oxi cat i on caus ed by a preformed t oxi n of Cl os t ri di um bot ul i num. Al t hough t he t oxi n i s des t royed by heat i ng, t he durat i on or degree of heat i ng may be i nadequat e. Oft en, a gas t roi nt es t i nal prodrome occurs before t he cl as s i c fl acci d paral ys i s s et s i n. Infant s have t he hi ghes t rat e of bot ul i s m, and i t i s charact eri zed by t he germi nat i on of C.
Pa g e 1 7 2 1
ABC Ambe r CHM Conve rte r Tria l ve rsion, http://w w w .proce sste x t.com/a bcchm.html
bot ul i num s pores i ns i de t he baby and el aborat i on of t he t oxi n di rect l y i nt o t he gas t roi nt es t i nal t ract . W hen adul t s devel op bot ul i s m from t hi s mechani s m of endogenous product i on of t oxi n, i t i s cal l ed hidden botulism.
(2) Fish poisoning (s ee Chapt er 5
onl i ne IV D 5 a)
(a) Consumption of contaminated shellfish can res ul t i n t wo t ypes of poi s oni ng: paral yt i c and neurot oxi c.
(i) Paralytic shellfish poisoning res ul t s from cont ami nat i on by neurot oxi n-produci ng di nofl agel l at es of t he genus Gonyaul ax. Thi s s yndrome i ncl udes pares t hes i as fol l owed by mus cl e weaknes s .
(ii) Neurotoxic shellfish poisoning res ul t s from s hel l fi s h cont ami nat i on by di nofl agel l at es of t he genus Gymnodi ni um. Thi s s yndrome i s charact eri zed by pares t hes i as wi t hout mus cl e weaknes s . Cons umpt i on of fl es hy fi s h can gi ve ri s e t o a s i mi l ar neurol ogi c s yndrome cal l ed ciguatera [s ee Chapt er 5
onl i ne IV D
5 a (1)].
(b) Consumption of certain contaminated fish s uch as t una and boni t o can res ul t i n an i l l nes s res embl i ng a hi s t ami ne react i on. Thi s s yndrome i s cal l ed scombroid.
Pa g e 1 7 2 2
ABC Ambe r CHM Conve rte r Tria l ve rsion, http://w w w .proce sste x t.com/a bcchm.html
(3) Monosodium glutamate intoxication (s omet i mes cal l ed Chinese restaurant syndrome). Thi s condi t i on i s charact eri zed by a burni ng and t i ght nes s i n t he upper body accompani ed by s ys t emi c s ympt oms s uch as fl us hi ng, di aphores i s , cramps , and naus ea.
(4) T herapy. Neurol ogi c food poi s oni ng s yndromes us ual l y res ol ve compl et el y but requi re pat i ent reas s urance and s upport i ve medi cal care. Support i ve meas ures may i ncl ude t racheal i nt ubat i on for pat i ent s wi t h bot ul i s m or paral yt i c s hel l fi s h poi s oni ng. Ident i fyi ng t he vehi cl e i s cruci al t o prevent furt her occurrences . Speci fi c ant i t oxi n t herapy i s avai l abl e for bot ul i s m and s houl d be admi ni s t ered as s oon as pos s i bl e aft er t he di agnos i s i s es t abl i s hed.
2. Peptic ulcer/gastroesophageal reflux disease (GERD). Caus at i ve agent i s oft en Hel i c obac t er pyl ori and mus t be eradi cat ed wi t h appropri at e t herapy (See Chapt er 5 III F 3).
3. Infectious diarrhea. Acut e di arrhea oft en can be at t ri but ed t o mi croorgani s ms . The occurrence of any i nfect i ous di arrhea s ugges t s a break i n opt i mal hygi eni c meas ures . The di s eas e i s t rans mi t t ed ei t her t hrough fecal cont ami nat i on of food or wat er or from human t o human vi a fomi t es or by s exual cont act (s ee Chapt er 5 IV D 4).
Pa g e 1 7 2 3
ABC Ambe r CHM Conve rte r Tria l ve rsion, http://w w w .proce sste x t.com/a bcchm.html
P.353
o
o
a. Etiology and pathogenesis. Det ermi ni ng t he s peci fi c et i ol ogi c agent of i nfect i ous di arrhea can be l abori ous and i s oft en uns ucces s ful . It may be us eful t o di s t i ngui s h i nvas i ve from noni nvas i ve proces s es .
(1) Invasive processes. Cert ai n agent s of acut e di arrhea (e.g., Shi gel l a, Sal monel l a, Ent amoeba, and Campyl obac t er) i nvade t he t ermi nal i l eum and col on, where t hey des t roy mucos al cel l s and caus e i nfl ammat i on. A bl oody di arrhea has been as s oci at ed wi t h s ome s t rai ns of E. c ol i t hat bear a t oxi n. Mos t of t hes e s t rai ns bel ong t o t he O157:H7 s erot ype of E. c ol i and can be cul t ured by a cl i ni cal l aborat ory i f t hey are al ert ed t o t he s us pi ci on of t hi s organi s m.
(1) Noninvasive processes. Ot her agent s of acut e di arrhea [e.g., ent erot oxi geni c E. c ol i , Vi bri o c hol erae (rarel y found i n t he Uni t ed St at es ), Crypt os pori di um, Gi ardi a i nt es t i nal i s , rot avi rus ] are ent erot oxi geni c and adhere t o mucos al cel l s i n t he s mal l i nt es t i ne, where t hey produce di arrheal t oxi ns .
o
o
b. Clinical features. Mani fes t at i ons vary wi t h res pect t o t he pres ence of abdomi nal pai n, cramps ,
Pa g e 1 7 2 4
ABC Ambe r CHM Conve rte r Tria l ve rsion, http://w w w .proce sste x t.com/a bcchm.html
and bl ood or mucus i n t he s t ool , as wel l as t o t he frequency and nat ure of t he bowel movement s .
(1) Diarrhea due to invasive agents general l y i s as s oci at ed wi t h rect al bl eedi ng, fever, and s ys t emi c s ympt oms s uch as headache.
(2) Diarrhea due to noninvasive agents us ual l y occurs wi t hout fever and i s as s oci at ed wi t h fewer s ys t emi c compl ai nt s . However, s ubs t ant i al fl ui d l os s from wat ery di arrhea i n t hes e pat i ent s may occur.
(3) Diarrhea due to enterohemorrhagic E. coli (O157:H7) i s us ual l y s el f-l i mi t ed, but i t can l ead t o hemol yt i c–uremi c s yndrome (HUS). Thi s i s a s eri ous i l l nes s t hat t ends t o occur mos t l y i n chi l dren and has a s ubs t ant i al mort al i t y rat e.
o
o
c. Diagnosis. It i s i mport ant t o di s t i ngui s h among t he vari ous i nfect i ous di arrheal di s eas es becaus e t hei r t herapi es are di fferent .
(1) Microscopic examination
(a) Fecal leukocytes. A us eful t es t i s t o exami ne t he s t ool for W BCs us i ng Gram s t ai n or preparat i ons s t ai ned wi t h met hyl ene bl ue.
Pa g e 1 7 2 5
ABC Ambe r CHM Conve rte r Tria l ve rsion, http://w w w .proce sste x t.com/a bcchm.html
(i) Invasive agents may s ecret e cyt ot oxi n or penet rat e t he mucos al wal l of t he bowel . Ei t her mechani s m may l ead t o t he pres ence of l eukocyt es i n t he s t ool . [The di arrhea as s oci at ed wi t h Cl os t ri di um di ffi c i l e, s omet i mes cal l ed pseudomembranous colitis (s ee Chapt er 5 V I), al s o i s as s oci at ed wi t h pus i n t he s t ool , as are s everal of t he s exual l y t rans mi t t ed proct i t i des and inflammatory bowel diseases (i .e., ul cerat i ve col i t i s and regi onal ent eri t i s or Crohn di s eas e).]
(ii) Noninvasive agents do not caus e t he mi grat i on of l eukocyt es i nt o t he bowel wal l and t he s t ool . However, s peci al t echni ques are needed t o i dent i fy Mi c ros pori di um or Crypt os pori di um i n t he s t ool , and t he l aborat ory may not perform t hes e t es t s rout i nel y.
(b) T rophozoites or cysts i n t he s t ool s ugges t t he pres ence of prot ozoa s uch as Ent amoeba hi s t ol yt i c a or G. i nt es t i nal i s .
(2) Stool culture
Pa g e 1 7 2 6
ABC Ambe r CHM Conve rte r Tria l ve rsion, http://w w w .proce sste x t.com/a bcchm.html
(a) The pres ence of fever or t he i dent i fi cat i on of fecal l eukocyt es s ugges t s an i nvas i ve pat hogen. W hen t he cl i ni cal s everi t y or t he epi demi ol ogi c s et t i ng makes t he preci s e di agnos i s i mport ant , t he s t ool can be cul t ured for t he mos t common of t hes e pat hogens (i .e., Shi gel l a, Sal monel l a, and Campyl obac t er). St ool cul t ures t ake at l eas t 2 days t o proces s , s o t herapeut i c deci s i ons are oft en made cl i ni cal l y.
(b) Eval uat i on of t he s t ool for prot ozoa (i .e., t he ova and parasite test) al s o i s recommended.
(c) C. di ffi c i l e produces an ent erot oxi n l eadi ng t o di arrhea, fever, and s t ool l eukocyt os i s . (In s evere cas es , a ps eudomembrane may be pres ent i n t he rect um or col on.) Di s eas e occurs duri ng or s hort l y aft er t he admi ni s t rat i on of ant i bact eri al agent s and i s es peci al l y common i n nosocomial diarrhea. Cul t ure and t oxi n as s ays are avai l abl e t o di agnos e C. di ffi c i l e i nfect i on.
(3) Proctosigmoidoscopy or colonoscopy. Thes e t echni ques may ai d i n t he di agnos i s of i nfl ammat i on-rel at ed di arrhea, es peci al l y by al l owi ng di fferent i at i on of i nfect i ous from noni nfect i ous caus es . P.354
Pa g e 1 7 2 7
ABC Ambe r CHM Conve rte r Tria l ve rsion, http://w w w .proce sste x t.com/a bcchm.html
(4) Biopsy. In cas es of pers i s t ent di arrhea i n whi ch l es s i nvas i ve t echni ques have fai l ed t o provi de a di agnos i s , biopsy of t he l arge bowel (for E. hi s t ol yt i c a) or t he s mal l bowel (for G. i nt es t i nal i s or Crypt os pori di um) may be di agnos t i c.
o
o
d. T herapy
(1) Fluid and electrolyte replacement i s paramount , regardl es s of t he caus al agent , and s houl d be adjus t ed accordi ng t o t he pat i ent 's degree of depl et i on.
(2) Speci fi c antimicrobial therapy may be neces s ary.
(a) Among t he bact eri al caus es of acut e di arrhea, onl y Shi gel l a i nfect i ons are rout i nel y t reat ed. The i nherent del ay i n di agnos i s makes i t unneces s ary t o t reat mos t Campyl obac t er, Sal monel l a, and E. c ol i i nfect i ons except i n s everel y i l l pat i ent s or t hos e wi t h i mpai red i mmuni t y. Mos t cas es i n previ ous l y heal t hy peopl e res ol ve wi t hout i nci dent wi t hi n a few days t o 1 week. W hen t reat ment i s i ndi cat ed, fl uoroqui nol ones are us ual l y effect i ve, al t hough i ncreas i ng
Pa g e 1 7 2 8
ABC Ambe r CHM Conve rte r Tria l ve rsion, http://w w w .proce sste x t.com/a bcchm.html
res i s t ance t o fl uoroqui nol ones has been not ed. Treat ment of ent erohemorrhagi c E. c ol i (O157:H7) i s not i ndi cat ed, and, under s ome condi t i ons , i t may exacerbat e t he di s eas e and l ead t o an i ncreas ed ri s k of HUS.
(b) Ant i prot ozoal agent s us ual l y are i ndi cat ed for gi ardi as i s and amebi as i s .
(c) Di s eas e as s oci at ed wi t h C. di ffi c i l e i s t reat ed wi t h ces s at i on of t he caus at i ve ant i bi ot i c (when pos s i bl e) and oral met roni dazol e. If t hi s t herapy fai l s , oral vancomyci n may be us ed. Approxi mat el y 20% of pat i ent s wi l l have recurrence aft er apparent l y s ucces s ful t reat ment , and t hey can be ret reat ed wi t h t he s ame regi men. A s mal l number of pat i ent s have frequent recurrences des pi t e good t herapy.
e. Prognosis. The pros pect for compl et e recovery from i nfect i ous di arrhea i s excel l ent for al l pat i ent s except t hos e who are s everel y i mmunocompromi s ed. In mos t cas es , a caus e i s not es t abl i s hed or even i nves t i gat ed. Di arrhea as s oci at ed wi t h C. di ffi c i l e i s normal l y eas y t o manage, but s ome pat i ent s can have poor out comes , i ncl udi ng deat h. Thi s i s al mos t al ways as s oci at ed wi t h s i gni fi cant underl yi ng di s eas e i ncl udi ng pri or organ t rans pl ant at i on and i mmune s uppres s i on.
Pa g e 1 7 2 9
ABC Ambe r CHM Conve rte r Tria l ve rsion, http://w w w .proce sste x t.com/a bcchm.html
E. Intra-abdominal infections
1. Peritonitis i s covered i n t he gas t roi nt es t i nal chapt er (s ee Chapt er 5 X A 2).
2. Postoperative intra-abdominal abscesses. Di agnos i s of t hes e i nfect i ons i s oft en di ffi cul t and needs t o be cons i dered i n s i t uat i ons of occul t fever. o
o
a. Etiology and pathogenesis
(1) Pos t operat i ve abs ces s es devel op mos t frequent l y when t he bowel has been breached i n an operat i ve procedure. However, t hey may occur when nonvi abl e t i s s ue or accumul at i ons of bl ood, s erum, uri ne, or bi l e are pres ent i n t he abdomi nal cavi t y.
(2) Al t hough t hes e i nfect i ons us ual l y occur wi t hi n s everal days of t he operat i ve procedure, t hey may be del ayed by weeks or mont hs . They t end t o devel op near t he anat omi c s i t e of t he s urgery, al t hough t he cont our of t he abdomen may al l ow i nfect ed fl ui d t o move t o ot her l ocat i ons .
o
o
b. Diagnosis. CT and ul t ras onography are t he mos t commonl y us ed t echni ques for l ocat i ng t hes e abs ces s es . Mos t i mport ant l y, t here mus t be a hi gh i ndex of s us pi ci on for pat i ent s who have prol onged
Pa g e 1 7 3 0
ABC Ambe r CHM Conve rte r Tria l ve rsion, http://w w w .proce sste x t.com/a bcchm.html
fever, abdomi nal pai n, or bot h aft er s urgery. o
o
c. T herapy. Once i nt ra-abdomi nal abs ces s es have been i dent i fi ed, they should be evacuated, ei t her by repeat s urgery or t hrough a t emporary fl exi bl e drai nage cat het er. Antimicrobial therapy s houl d be admi ni s t ered bas ed on t he res ul t s of Gram s t ai n and cul t ures . Anaerobes pl ay a major rol e i n i nt ra-abdomi nal abs ces s es and s houl d be s us pect ed when t he fl ui d i s foul -s mel l i ng or s hows pol ymi crobi al morphol ogy on Gram s t ai n. In s el ect i ng ant i mi crobi al t reat ment , t he onus i s t o prove t hat anaerobes are not pres ent .
3. Hepatic and splenic abscesses o
o
a. Etiology and pathogenesis. Bact eri al (pyogeni c) abs ces s es may devel op i n t he l i ver and s pl een, al t hough t hes e s i t es are rel at i vel y prot ect ed from i nfect i on. They may be s i ngl e or mul t i pl e and oft en are mul t i bact eri al . Hepat i c abs ces s es us ual l y are as s oci at ed wi t h bi l i ary di s eas e or port al vei n bact eremi a. Spl eni c abs ces s es are as s oci at ed wi t h s ys t emi c bact eremi a, hemogl obi nopat hy, t rauma, endocardi t i s , or i nt ravenous drug us e. P.355
o
o
b. Clinical features. Fever, chi l l s , and pai n near t he affect ed organ are common. However, cl i ni cal fi ndi ngs may be s ubt l e, maki ng l ocal i zat i on and
Pa g e 1 7 3 1
ABC Ambe r CHM Conve rte r Tria l ve rsion, http://w w w .proce sste x t.com/a bcchm.html
di agnos i s di ffi cul t . o
o
c. Diagnosis. Cros s -s ect i onal anat omi c i magi ng s uch as CT s canni ng, MRI, or ul t ras onography are us ual l y us ed t o make t he di agnos i s . Bl ood cul t ures s houl d al ways be obt ai ned. In t ravel ers t o areas endemi c for amoebi as i s or for i mmi grant s from t hos e areas , an eval uat i on for amoebi c abs ces s us i ng s erol ogi c t es t s or a cyt ol ogi c anal ys i s of t he abs ces s fl ui d can be very hel pful .
o
o
d. T herapy. T reatment is primarily surgical, wi t h exci s i on of abs ces s es from t he l i ver or s pl enect omy. Cat het er drai nage i s anot her opt i on for s el ect ed pat i ent s . Systemic antimicrobial therapy is mandatory. Somet i mes s mal l l i ver abs ces s es can be t reat ed wi t h ant i bi ot i cs al one and have a good out come.
4. Cholecystitis and cholangitis For furt her det ai l s s ee Chapt er 5 VIII B, C, as wel l as Chapt er 5 VIII G.
F. Urinary tract infections (UTIs)
1. Incidence. Thes e are t he mos t common bact eri al i nfect i ons encount ered i n cl i ni cal pract i ce and occur much more frequent l y i n women t han i n men. Infect i ons i n men, pregnant women, or chi l dren s houl d be cons i dered compl i cat ed. UTI i n s exual l y act i ve women i s general l y cons i dered uncompl i cat ed. o
o
a. UT Is are defined i n t erms of t he i nfl amed
Pa g e 1 7 3 2
ABC Ambe r CHM Conve rte r Tria l ve rsion, http://w w w .proce sste x t.com/a bcchm.html
uri nary s t ruct ure:
(1) Cystitis: t he bl adder
(2) Urethritis: t he uret hra
(3) Pyelonephritis: t he renal t ubul es and i nt ers t i t i um
(4) Prostatitis: t he pros t at e (men).
2. Etiology. Mos t UTIs are caus ed from s i mpl e i nocul at i on by Gram-negat i ve bact eri a duri ng i ns t rument at i on or duri ng s exual i nt ercours e. o
o
a. Agent s of i nfect i on
(1) Common agent s i ncl ude E. c ol i (mos t commonl y), Kl ebs i el l a, and Prot eus .
(2) Les s common caus es i ncl ude Gram-pos i t i ve cocci s uch as St aphyl oc oc c us s peci es (es peci al l y St aphyl oc oc c us s aprophyt i c us ) and ent erococci . Rare agent s : Noc ardi a, Ac t i nomyc et es , Bruc el l a, adenovi rus , and T orul ops i s .
o
o
b. Urethral inoculation i s very common i n women, part i cul arl y t hos e wi t h vagi nal or peri uret hral
Pa g e 1 7 3 3
ABC Ambe r CHM Conve rte r Tria l ve rsion, http://w w w .proce sste x t.com/a bcchm.html
col oni zat i on by vi rul ent bact eri a. Surgery, part i cul arl y cyt os copy, can cont ami nat e t he bl adder uri ne. o
o
c. Risk factors i ncl ude: obs t ruct i on wi t h res ul t ant uri nary s t as i s , ves i couret eral refl ux (may l ead t o as cendi ng i nfect i on), i ns t rument at i on (es peci al l y Fol ey cat het er), pregnancy (al t ered s moot h mus cl e funct i on), di abet es mel l i t us , or i mmune defi ci ency.
3. Clinical features o
o
a. In general , s ympt omat i c UTIs are charact eri zed by t he i rri t at i ve s ympt oms of uri nary frequency, urgency, and s uprapubi c pai n. Lower t ract (bl adder and uret hra) i nfect i on al s o i ncl udes dys uri a, burni ng, mal odorous or cl oudy uri ne, and cont i nence di ffi cul t i es . Fl ank pai n and fever s ugges t upper t ract (ki dney or uret er) i nvol vement .
o
o
b. Sympt oms of acute pyelonephritis (parenchymal renal i nfect i on) i ncl ude fl ank pai n, fever, mal ai s e, and s ympt oms of l ower t ract i nfect i on.
o
o
c. In el derl y or i ns t i t ut i onal i zed pat i ent s , septic shock pres ent s wi t h hypot hermi a, ment al s t at us al t erat i ons , s yncope, and coma.
o
d. The compos i t i on of t he uri nary s edi ment i n uri nary i nfect i ons i s charact eri zed by a l arge number of neut rophi l s and a vari abl e number of red cel l s . W BC cas t s may be obs erved i n ki dney
Pa g e 1 7 3 4
ABC Ambe r CHM Conve rte r Tria l ve rsion, http://w w w .proce sste x t.com/a bcchm.html
i nfect i ons
(s ee Onl i ne Fi gure 6-1).
4. Clinical syndromes o
o
a. There i s s t andard agreement t hat a bact eri al 5
count exceedi ng 10 organi s ms per mi l l i l i t er of uri ne combi ned wi t h i rri t at i ve voi di ng s ympt oms and pyuri a i ndi cat e s i gni fi cant di s eas e, not s i mpl e cont ami nat i on. However, t wo cl i ni cal l y i mport ant s i t uat i ons are not covered by t hi s defi ni t i on of uri nary t ract i nfect i ons .
(1) Pat i ent s with symptoms of t rue uri nary t ract i nfect i on but wi t h fewer t han 10
5
bact eri a per mi l l i l i t er of uri ne may refl ect earl y i nfect i on and s houl d be t reat ed wi t h ant i bi ot i cs . P.356
(2) Pat i ent s wi t h no s ympt oms of uri nary t ract i nfect i on but a uri ne bact eri al count 5
exceedi ng 10 /mL. Thi s condi t i on, whi ch i s t ermed asymptomatic bacteriuria, i s not rare and occurs mos t oft en i n women and i n ol der pat i ent s and s houl d not be t reat ed wi t h ant i bi ot i cs . o
o
b. Catheter-related bacteriuria i s an i nevi t abl e concomi t ant compl i cat i on of prol onged bl adder cat het eri zat i on. It can be del ayed by careful
Pa g e 1 7 3 5
ABC Ambe r CHM Conve rte r Tria l ve rsion, http://w w w .proce sste x t.com/a bcchm.html
as ept i c i ns ert i on, s i l ver-i mpregnat ed cat het ers , and s t ri ct cl os ed drai nage. However, aft er 1 or 2 weeks , s ome degree of col oni zat i on i s common, and 5
event ual l y, bact eri al count s exceedi ng 10 /mL of uri ne are reached. o
o
c. Prostatitis i s a compl i cat i on of many uri nary t ract i nfect i ons i n men. Acut e pros t at i t i s i s charact eri zed by peri neal pai n and i rri t at i ve voi di ng s ympt oms .
o
o
d. Urethritis i s oft en a s ympt om of a s exual l y t rans mi t t ed di s eas e (e.g., gonorrheal or chl amydi al i nfect i on). Copi ous uret hral di s charge i s di agnos t i c, but mi l dl y s ympt omat i c uret hri t i s can mi mi c a uri nary t ract i nfect i on.
5. Diagnosis. o
o
a. Urinalysis of fres h, uns pun uri ne s houl d be performed. However, quant i t at i ve uri ne cul t ures do not di s t i ngui s h bet ween upper and l ower t ract i nfect i ons .
1. Dipstick tests t o exami ne pres ence of enzymes s uch as t he l eukocyt e es t eras es may be us eful . Ident i fyi ng 1 bact eri um per hi gh-power fi el d (400X) i ndi cat e col ony growt h 5
on cul t ures >10 /mL.
2. Centrifuged sediment us ual l y reveal s
Pa g e 1 7 3 6
ABC Ambe r CHM Conve rte r Tria l ve rsion, http://w w w .proce sste x t.com/a bcchm.html
l eukocyt uri a (neut rophi l s ) and bact eri uri a.
3. Gram stain may charact eri ze t he offendi ng organi s m, al l owi ng more s peci fi c t herapy.
o
o
b. Routine urine culture is the definite method of 5
diagnosis. More t han 10 col oni es /mL s i gni fi es an i nfect i on t hat needs t reat ment . o
o
c. Examining the patient. W hen t o exami ne pat i ent s for anat omi c abnormal i t i es t o expl ai n UTIs i s cont rovers i al . Mos t evi dence s ugges t s t hat cys t os copy and urography s el dom s how t reat abl e caus es of i nfect i on; us e onl y for pat i ent s wi t h frequent recurrences and jeopardi zed ki dney funct i on. Men have a hi gher rat e of anat omi c compl i cat i ons (us ual l y pros t at e-rel at ed) t han women. Rect al exami nat i on may i ndi cat e enl argement or t endernes s of t he pros t at e, and purul ent di s charge may be expres s ed from t he uret hra aft er pros t at i c exami nat i on.
o
o
d. W hen bl ood cul t ures are pos i t i ve i n t he cont ext of a UTI, i nvol vement of t he ki dneys or pros t at e gl and i s i mpl i ed. Fever al s o s ugges t s renal i nvol vement .
6. T herapy. Ant i bi ot i c t reat ment i s us ual l y di rect ed at t he agent mos t l i kel y t o be res pons i bl e for t he i nfect i on. Correct i ve s urgery i s i ndi cat ed t o remove cal cul i or repai r obs t ruct i ve anat omi c l es i ons . o
Pa g e 1 7 3 7
ABC Ambe r CHM Conve rte r Tria l ve rsion, http://w w w .proce sste x t.com/a bcchm.html
o
a. E. coli–caused infections. E. c ol i has a propens i t y for caus i ng i nfect i ons i n ot herwi s e heal t hy i ndi vi dual s , s o admi ni s t er ant i bi ot i cs effect i ve agai ns t E. c ol i i n t he abs ence of cul t ures . Oral t ri met hopri m and s ul famet hoxaz ol e can be us ed al one or t oget her, as wel l as qui nol ones , t et racycl i nes , and s ome peni ci l l i n preparat i ons s uch as amoxi ci l l i n wi t h cl avul ani c aci d. Ampi ci l l i n al one i s l i kel y t o be i neffect i ve becaus e of t he l arge number of ampi ci l l i n-res i s t ant E. c ol i .
(1) A 3-day cours e of t herapy i s adequat e t o cure mos t i nfect i ons (es peci al l y cys t i t i s ) where uri ne drug l evel s s eem t o be t he mos t i mport ant det ermi nant of effect i venes s .
(2) Fai l ure t o cure i nfect i ons rapi dl y does not al t er t he effi cacy of fut ure t herapy. In rare i ns t ances , refract ory i nfect i on may requi re a l ong cours e of t herapy—s omet i mes up t o s everal mont hs —al t hough bri efer cours es (i .e., 2 weeks ) oft en are s ucces s ful .
o
o
b. Uncomplicated lower tract infection. Three-day t reat ment of t ri met hopri m–s ul famet hoxaz ol e (TMP-SMX) i s effect i ve and i nexpens i ve. Be aware of l ocal pat t erns of bact eri al s us cept i bi l i t y. Al t ernat i ves i ncl ude ni t rofurant oi n, and β-l act am ant i bi ot i cs , s uch as amoxi ci l l i n or cephal exi n.
o
o
c. Treat relapsing UT I and pyelonephritis for 14 days wi t h TMP-SMX, i f s t abl e; al t ernat i ves i ncl ude
Pa g e 1 7 3 8
ABC Ambe r CHM Conve rte r Tria l ve rsion, http://w w w .proce sste x t.com/a bcchm.html
fl uoroqui nol ones and amoxi ci l l i n or amoxi ci l l i n/cl avul ani c aci d. In t he pres ence of hi gh fever, hi gh whi t e bl ood cel l count , vomi t i ng, dehydrat i on, or evi dence of s eps i s , t he pat i ent s houl d be admi t t ed and t reat ed wi t h i nt ravenous ant i bi ot i cs unt i l s t abl e, and t hen can compl et e t herapy wi t h oral agent s as an out pat i ent . o
o
d. Asymptomatic bacteriuria. Treat ment i s i ndi cat ed for pat i ent s wi t h a known s t ruct ural or funct i onal abnormal i t y of t he ki dneys (i ncl udi ng renal t rans pl ant ) and t hos e who are pregnant .
o
o
e. Catheter-related bacteriuria may occur i n acut el y cat het eri zed pat i ent s and us ual l y res ol ves when t he cat het er i s removed. If not , or s ympt oms devel op wi t hi n 12–24 hours , t reat wi t h t he P.357
appropri at e ant i bi ot i c. Frequent bl adder cat het eri zat i ons (s everal t i mes dai l y) rat her t han us e of an i ndwel l i ng cat het er may l es s en t he ri s k of t hi s i nfect i on. o
o
f. Prostate infections. Us e ant i bi ot i cs t hat penet rat e and remai n act i ve i n pros t at e t i s s ue and fl ui d (e.g., t ri met hopri m, carbeni ci l l i n). Treat for at l eas t 14 days .
o
o
g. Recurrent UT I t ypi cal l y requi re TMP-SMX, met henami ne mandel at e, and s ul fi s oxazol e. Us i ng ci profl oxaci n may res ul t i n s t eri l e uri ne.
Pa g e 1 7 3 9
ABC Ambe r CHM Conve rte r Tria l ve rsion, http://w w w .proce sste x t.com/a bcchm.html
G. Skin and soft tissue infections Bact eri al , fungal , and vi ral i nfect i ons of t he s ki n and rel at ed s t ruct ures (e.g., hai r fol l i cl es , s weat gl ands ) are common. (See al s o Chapt er 12 XI.)
1. Bacterial skin infections us ual l y s t art i n areas of t rauma or previ ous di s eas e. o
o
a. Etiology. In ot herwi s e heal t hy i ndi vi dual s , Gram-pos i t i ve cocci s uch as S. pyogenes and S. aureus caus e mos t s ki n i nfect i ons . Immunocompromi s ed i ndi vi dual s are s ubject t o t he us ual Gram-pos i t i ve fl ora as wel l as a wi der vari et y of pat hogens , i ncl udi ng ent eri c Gram-negat i ve baci l l i and P. aerugi nos a.
o
o
b. Clinical features
(1) Cellulitis i s charact eri zed by rednes s , warmt h, and t endernes s of t he s ki n. It may i nvol ve a l i mi t ed area or may s pread wi del y and rapi dl y. Fever and l eukocyt os i s are common. [In s ome pat i ent s , a rapi dl y progres s i ve cut aneous i nfect i on (s t rept ococcal TSS) caus ed by group A s t rept ococci and charact eri zed by s i gni fi cant s ys t emi c feat ures can caus e s evere s ki n damage and even deat h.]
(2) Abscesses repres ent deeper, ci rcums cri bed i nfect i ons , whi ch oft en s t art i n
Pa g e 1 7 4 0
ABC Ambe r CHM Conve rte r Tria l ve rsion, http://w w w .proce sste x t.com/a bcchm.html
acces s ory s t ruct ures s uch as hai r fol l i cl es . They may be warm or of normal s ki n t emperat ure and frequent l y cont ai n pus . Fever i s mos t l i kel y t o be pres ent i n i ndi vi dual s wi t h l arge, mul t i pl e, or deep abs ces s es . Hi dradeni t i s occurs from i nfl ammat i on of s weat gl ands wi t h res ul t ant abs ces s ; oft en i t can be t reat ed wi t h warm compres s i on.
(3) Ulcers are not us ual l y t he res ul t of bact eri al i nfect i on al one but refl ect t i s s ue damage from i s chemi a or t rauma. Invari abl y, ul cers are col oni zed by bact eri a and may l ead t o deep s oft t i s s ue or bone i nfect i on. They us ual l y occur i n dependent areas (e.g., t he s acrum) or i n areas of poor bl ood fl ow or decreas ed s ens at i on (e.g., i n t he feet of a pat i ent wi t h di abet es ).
o
o
c. T herapy
(1) Cellulitis requires antibiotic management. Agent s act i ve agai ns t s t rept ococci and s t aphyl ococci us ual l y are effect i ve, i ncl udi ng s emi s ynt het i c peni ci l l i ns (e.g., nafci l l i n), cephal os pori ns , vancomyci n, cl i ndamyci n, and l i nezol i d.
(2) Abscesses should be drained, al t hough s ome rupt ure s pont aneous l y. Ant i bi ot i cs are s el dom needed unl es s t here i s accompanyi ng cel l ul i t i s .
Pa g e 1 7 4 1
ABC Ambe r CHM Conve rte r Tria l ve rsion, http://w w w .proce sste x t.com/a bcchm.html
(3) Ulcers us ual l y are managed by débridement and antibiotic therapy wi t h broad-s pect rum agent s act i ve agai ns t ent eri c Gram-negat i ve rods , Gram-pos i t i ve cocci , and anaerobes . Ski n graft i ng may be benefi ci al .
2. Fungal skin infections us ual l y are acqui red by expos ure of t he s ki n t o pat hogeni c fungi . Except i ons are cut aneous mani fes t at i ons of bl as t omycos i s or candi dal fungemi a. o
o
a. Clinical features. Fungal i nfect i ons t hat s t art i n t he s ki n fal l i nt o t hree groups .
(1) Dermatophytosis i s a s uperfi ci al i nfect i on of t he epi dermi s due t o dermat ophyt i c fungi (e.g., T ri c hophyt on, Mi c ros porum, and Epi dermophyt on s peci es ). At hl et e's foot and ri ngworm are exampl es . The s ki n us ual l y i s fl aky and may be s l i ght l y di s col ored but i s not frankl y pai nful .
(2) Candidiasis i s a red, t ender edemat ous ras h occurri ng i n moi s t body part s and caus ed by C. al bi c ans , oft en s een i n di abet i cs and i mmunos uppres s ed pat i ent s . Int ert ri go i n t he axi l l ary or i nframammary area i s an exampl e.
(3) Mixed bacterial flora and fungi (us ual l y Candi da s peci es ) can be i nvol ved i n s uperfi ci al i nfect i on. Thi s may be evi dent duri ng or aft er
Pa g e 1 7 4 2
ABC Ambe r CHM Conve rte r Tria l ve rsion, http://w w w .proce sste x t.com/a bcchm.html
admi ni s t rat i on of ant i bact eri al t herapy. o
o
b. Diagnosis. Pot as s i um hydroxi de preparat i ons and fungal cul t ures are t he foundat i on of di agnos i s .
o
o
c. T herapy
(1) Dermat ophyt os i s i s t reat ed wi t h t opi cal t herapy for l i mi t ed di s eas e or wi t h oral t herapy for ext ens i ve i nfect i on. Azoles (e.g., ket oconazol e, fl uconazol e) are mos t commonl y us ed. P.358
(2) Candi di as i s i s t reat ed t opi cal l y, but s peci al at t ent i on s houl d be gi ven t o keepi ng t he affect ed area cl ean and dry. Rarel y, s ys t emi c azol e t herapy i s requi red.
(3) Mi xed bact eri al and fungal i nfect i ons us ual l y do not requi re s peci fi c ant i fungal t herapy.
3. Viral skin infections may be cut aneous l y i nocul at ed, as i n t he cas e of herpes s i mpl ex, but more commonl y are a mani fes t at i on of s ys t emi c vi ral i nfect i on or an i mmune res pons e t o i nfect i on. An exampl e of t hi s i s varicella (chickenpox), i n whi ch t he vi rus i s acqui red
Pa g e 1 7 4 3
ABC Ambe r CHM Conve rte r Tria l ve rsion, http://w w w .proce sste x t.com/a bcchm.html
vi a t he res pi rat ory t ract and s preads t o t he s ki n aft er a vi remi a.
4. Deep soft tissue infections are rare but s eri ous i nfect i ons us ual l y caus ed by s t rept ococci , anaerobes , and Gram-negat i ve rods . There may be comparat i vel y l i t t l e abnormal i t y of t he s ki n i n s ome of t hes e i nfect i ons (e.g., fasciitis). Combi nat i ons of medi cal and s urgi cal t herapy us ual l y are us ed, but even s o, many pat i ent s s uccumb t o t hes e i nfect i ons . o
o
a. Gangrene i s an i nfect i on of t he s ki n and s oft t i s s ues caus ed by a mi xed anaerobi c and aerobi c bact eri al fl ora t hat us ual l y i ncl udes C. perfri ngens . It i s charact eri zed by cel l ul i t i s and gas i n t he s oft t i s s ues . Therapy i nvol ves s urgi cal débri dement and ant i bi ot i c t herapy (e.g., wi t h peni ci l l i n).
o
o
b. Fasciitis i s a rare i nfect i on bet ween t i s s ue pl anes . It us ual l y occurs pos t operat i vel y, aft er rupt ure of an abdomi nal vi s cus , or i n di abet i c pat i ent s . Pat hogeni c bact eri a i ncl ude mi xed anaerobes , s t rept ococci , and, occas i onal l y, Gram-negat i ve baci l l i .
H. Osteomyelitis Bone i nfect i ons can be cl as s i fi ed accordi ng t o t hei r pat hogenes i s . In general , bone i nfect i ons devel op i n t hree ways : by ext ens i on from a cont i guous i nfect i on, by di rect i nocul at i on duri ng s urgery or as a res ul t of t rauma, and by hemat ogenous s pread.
1. Osteomyelitis due to contiguous infection or inoculation. Bone i nfect i on s houl d be s us pect ed i n
Pa g e 1 7 4 4
ABC Ambe r CHM Conve rte r Tria l ve rsion, http://w w w .proce sste x t.com/a bcchm.html
pat i ent s wi t h a hi s t ory of trauma or surgery or wi t h obvi ous soft tissue infection overl yi ng bone, al t hough t he bone i nvol vement may be di ffi cul t t o prove. o
o
a. Etiology. Al mos t any bact eri um can be res pons i bl e, i ncl udi ng S. aureus , P. aerugi nos a, or anaerobes .
o
o
b. Clinical features. Sympt oms and s i gns may s i mpl y be t hos e of t he adjacent i nfect i on. Pai n i s common, and fever i s vari abl e.
o
o
c. Diagnosis. Thi s proces s may be di ffi cul t . Radi ographi c evi dence of os t eomyel i t i s l ags behi nd t he s ympt oms and pat hol ogi c changes by approxi mat el y 7–10 days . Al t hough bone s canni ng i s s ens i t i ve for os t eomyel i t i s , i t may not di s t i ngui s h bone i nfect i on from more s uperfi ci al s oft t i s s ue i nfect i on. The defi ni t i ve di agnos i s res t s on bone bi ops y and bact eri al cul t ure. An expens i ve but us eful t es t i s t he MRI s can, whi ch s hows changes i n t he bone marrow t hat can be det ect ed wel l before change i n pl ai n x-rays (Fi gure 8-4).
o
o
d. Clinical course and therapy. The cl i ni cal cours e gui des t he l engt h of t reat ment , but , general l y, long courses of antibiotics are neces s ary. Devi t al i zed bone, poor bl ood s uppl y, and adjacent i nfect i on may be reas ons for ext ended (mont hs ) t herapy and/or s urgi cal débri dement .
2. Hematogenous osteomyelitis. Thi s t ype of di s eas e
Pa g e 1 7 4 5
ABC Ambe r CHM Conve rte r Tria l ve rsion, http://w w w .proce sste x t.com/a bcchm.html
s houl d be s us pect ed i n febri l e pat i ent s who experi ence pai n and s wel l i ng over a bone but have no obvi ous s ource of i nfect i on. o
o
a. Etiology. The bact eri ol ogy i nvol ves l argel y S. aureus . However, Sal monel l a s peci es s eem t o be more i mport ant i n pat i ent s wi t h s i ckl e cel l di s eas e. Vert ebral os t eomyel i t i s al s o i s s omewhat di fferent i n t hat Gram-negat i ve baci l l i may be i nt roduced from t he uri nary t ract t hrough venous channel s .
o
o
b. Diagnosis. Radi ography and bone s canni ng are hel pful . The eryt hrocyt e s edi ment at i on rat e us ual l y i s el evat ed but t hi s i s qui t e a nons peci fi c fi ndi ng. Bone as pi rat i on and cul t ure are recommended for mi crobi ol ogi c di agnos i s . Bl ood cul t ures al s o s houl d be obt ai ned.
o
o
c. T herapy. Treat ment i s a prol onged cours e of antibiotics. Nonvi abl e bone may need t o be removed s urgi cal l y becaus e i t provi des a s i t e for pot ent i al rel aps e.
I. Intravascular infections and endocarditis Int ravas cul ar i nfect i ons mani fes t as bacteremia, or fungemia, dependi ng on t he t ype of i nfect i ve organi s m demons t rat ed i n t he bl ood. Such i nfect i ons may refl ect invasion or failure of containment at a localized site (e.g., bowel or l ung) or a primary infection of the blood vessels or the heart (e.g., endocardi t i s ).
1. Local infections l ead t o pos i t i ve bl ood cul t ures , wi t h a frequency dependent on t he s i t e and s everi t y of t he i nfect i on and on t he organi s m or organi s ms res pons i bl e
Pa g e 1 7 4 6
ABC Ambe r CHM Conve rte r Tria l ve rsion, http://w w w .proce sste x t.com/a bcchm.html
for t he i nfect i on. P.359
FIGURE 8-4 Magnet i c res onance i magi ng (MRI) s can of os t eomyel i t i s i n t he t horaci c s pi ne. In t he T1 i mage (l eft ), l os s of bony s t ruct ure i n a vert ebra i s s een (arrow). In t he T2 i mage (ri ght ), t he hi gh s i gnal i nt ens i t y due t o wat er mol ecul es s hows edema and i nfi l t rat e i n t he s ame vert ebra. (From Engl eberg NC, Dermody T, Di Ri t a V. Sc haec ht er's Mec hani s ms of Mi c robi al Di s eas e. 4t h ed. Bal t i more: Li ppi ncot t W i l l i ams & W i l ki ns , 2006. ) o
o
a. For exampl e, among hos pi t al i zed pat i ent s wi t h pneumococcal pneumoni a, 10%–25% have bact eremi a. The prognos i s for t hes e pat i ent s i s wors e t han for t hos e wi t hout bact eremi a becaus e
Pa g e 1 7 4 7
ABC Ambe r CHM Conve rte r Tria l ve rsion, http://w w w .proce sste x t.com/a bcchm.html
t hos e wi t h bact eremi a t end t o have more di ffus e l ung i nvol vement and more vi rul ent organi s ms . o
o
b. Even when bl ood cul t ures do not demons t rat e a part i cul ar organi s m, met as t at i c i nfect i on or di s s emi nat i on (e.g., crypt ococcal meni ngi t i s or mi l i ary t ubercul os i s ) s t rongl y s ugges t s bl ood-borne s pread.
2. Sepsis i s recogni zed as a s ys t emi c i nfl ammat ory res pons e s yndrome (SIRS) i n t he s et t i ng of an i nfect i on. SIRS i s defi ned i n a pat i ent wi t h t wo of t he fol l owi ng cri t eri a: o
o
1. Temperat ure >38°C or <36°C
o
o
2. Heart rat e >90 bpm
o
o
3. Res pi rat i ons >20/mi n or PaCO 2 <32 mm Hg OR
o
o
3
3
4. Leukocyt e count >12,000/mi n , <4000/mm , or >10% i mmat ure (band) cel l s .
o
o
a. Etiology. Seps i s can progres s t o s ept i c s hock, whi ch i ncl udes organ dys funct i on s econdary t o hyperfus i on. Sept i c s hock can be mul t i fact ori al , but one i dent i fi ed caus e i s endot oxi n (t he l i popol ys acchari de coat of Gram-negat i ve
Pa g e 1 7 4 8
ABC Ambe r CHM Conve rte r Tria l ve rsion, http://w w w .proce sste x t.com/a bcchm.html
organi s ms ). However, an i ndi s t i ngui s habl e cl i ni cal di s eas e can be caus ed by Gram-pos i t i ve bact eri a, vi rus es , and yeas t . o
o
b. T herapy. Antibiotic treatment i s cri t i cal for pat i ent recovery. Hi gh-dos e cort i cos t eroi d t herapy t o s uppl ement us ual s upport i ve meas ures does not i mprove s urvi val . Vari ous i mmunomodul at ors s uch as TNF i nhi bi t ors and ant i body t o endot oxi n have fai l ed t o i mprove t he prognos i s of s ept i c s hock. Act i vat ed prot ei n C (drot recogi n) has ant i coagul ant propert i es but al s o s eems t o bl ock s ome of t he des t ruct i ve pos i t i ve feedback l oops t hat promot e organ damage i n s hock. It i s us eful i n t he mos t s everel y i l l pat i ent s .
3. Catheter-related infections are s eri ous probl ems as s oci at ed wi t h hos pi t al i zat i on. Infect i on s el dom i s caus ed by t he i nfus i on of cont ami nat ed fl ui d. Rat her, t hes e i nfect i ons mos t commonl y occur at t he site of cannulation. o
o
a. Diagnosis. Di agnos i s i nvol ves demons t rat i on of ei t her l ocal s ki n i nfect i on at t he cannul at i on s i t e or pos i t i ve bl ood cul t ures and t he pres ence of t he s ame bact eri a i n s i gni fi cant numbers on P.360
a s emi quant i t at i ve cul t ure of t he cat het er. Somet i mes a vei n i nfect i on may not be apparent unt i l aft er t he cat het er has been removed. Infect ed vei ns are al mos t al ways cl ot t ed at t he s i t e of t he i nfect i on.
Pa g e 1 7 4 9
ABC Ambe r CHM Conve rte r Tria l ve rsion, http://w w w .proce sste x t.com/a bcchm.html o
o
b. T herapy. Removal of t he cat het er when l ocal i nfect i on i s pres ent i s neces s ary. W hen vas cul ar acces s i s cruci al and t he cat het er has been as ept i cal l y i ns ert ed i nt o t he vena cava (e.g., a Hickman catheter—a wi de-bore s i l as t i c cat het er us ed for chemot herapy, hyperal i ment at i on, or drawi ng bl ood), cons ervat i ve t herapy wi t h ant i bi ot i cs al one may cure t he i nfect i on. Fungal i nf ect i ons or t hos e caus ed by hi ghl y res i s t ant bact eri a us ual l y cannot be cured wi t hout removal of t he cat het er. Tunnel ed cat het ers t hat exhi bi t eryt hema and ot her s i gns of i nfl ammat i on al ong t he cat het er t ract al s o need t o be removed. Rarel y, an i nfect ed peri pheral vei n wi l l requi re s urgi cal management becaus e pus needs t o be drai ned.
4. Endocarditis us ual l y res ul t s from i nfect i on of t he cus p of a heart val ve, al t hough any part of t he endocardi um or any pros t het i c mat eri al i ns ert ed i nt o t he heart may be i nvol ved. o
o
a. Etiology. A vari et y of organi s ms may caus e endocardi t i s , al t hough bact eri a account for al mos t al l cas es . The s peci fi c agent of endocardi t i s depends on whi ch cardi ac s t ruct ures are affect ed.
(1) Infection of normal valves, whi ch i s rare, i s us ual l y as s oci at ed wi t h i nt ravenous drug us e. S. aureus i s t he mos t common pat hogen.
(2) Infection of previously damaged valves,
Pa g e 1 7 5 0
ABC Ambe r CHM Conve rte r Tria l ve rsion, http://w w w .proce sste x t.com/a bcchm.html
oft en from rheumat i c heart di s eas e, us ual l y i s at t ri but abl e t o vi ri dans s t rept ococci . Ot her agent s of endocardi t i s i n t hi s s et t i ng are ent erococci , S. aureus , and vari ous s mal l Gram-negat i ve rods cons t i t ut i ng part of t he normal oral fl ora known as t he Haemophi l us parai nfl uenzae, Haemophi l us aphrophi l us , Ac t i nobac i l l us ac t i nomyc et emc omi t ans , Cardi obac t eri um homi ni s , Ei kenel l a c orrodens , and Ki ngel l a ki ngae (HACEK) organi s ms .
(3) Infection of prosthetic valves i nvol ves s t aphyl ococci (bot h coagul as e-pos i t i ve and coagul as e-negat i ve) as t he mos t common agent s of earl y ons et di s eas e (occurri ng < 2 mont hs pos t operat i vel y). St rept ococci are t he mos t common agent s of l at e-ons et di s eas e (occurri ng >2 mont hs pos t operat i vel y).
o
o
b. Clinical features. Si gns and s ympt oms vary wi del y.
(1) Fever i s al mos t uni vers al . A new heart murmur i s hi ghl y s ugges t i ve but not al ways pres ent . Endocardi t i s i s one of t he mos t common caus es of fever of unknown ori gi n.
(2) Les s commonl y, embolic disease s uch as s t roke or s pl eni c art ery embol i s m and i nfarct i on i s evi dent . Mos t embol i are s mal l and may gi ve ri s e t o uncommon but di agnos t i cal l y hel pful phys i cal fi ndi ngs i ncl udi ng Roth' s spots, Osler' s nodes,
Pa g e 1 7 5 1
ABC Ambe r CHM Conve rte r Tria l ve rsion, http://w w w .proce sste x t.com/a bcchm.html
Janeway lesions, and conjunctival hemorrhage.
(3) A vari et y of cons t i t ut i onal s ympt oms s uch as myal gi a, back pai n, confus i on, or fat i gue may occur.
o
o
c. Diagnosis
(1) Blood cultures are cri t i cal and are pos i t i ve i n more t han 90% of cas es of endocardi t i s . (Previ ous us e of ant i bi ot i cs may l ower t hi s val ue.) Becaus e of t he cont i nuous bact eremi a of endocardi t i s , vi rt ual l y al l cul t ures are pos i t i ve, and i t i s rarel y neces s ary t o obt ai n more t han t hree or four cul t ures .
(2) For pat i ent s wi t h cul t ure-negat i ve endocardi t i s , t here i s l i t t l e i ncrement al val ue i n col l ect i ng s everal addi t i onal bl ood s ampl es for cul t ure. Somet i mes , t he mi crobi ol ogy l aborat ory can enhance i s ol at i on by us i ng special culture techniques, s uch as prol onged i ncubat i on t o i s ol at e s l ow-growi ng organi s ms (e.g., HACEK).
(3) Echocardi ography s houl d be performed i n s us pect ed endocardi t i s . t rans t horaci c echocardi ography (TTE) may reveal veget at i on or val vul ar i ns uffi ci ency s ugges t i ve of endocardi t i s , but t rans es ophageal
Pa g e 1 7 5 2
ABC Ambe r CHM Conve rte r Tria l ve rsion, http://w w w .proce sste x t.com/a bcchm.html
echocardi ography (TEE) has hi gher s ens i t i vi t y.
(4) Immune compl exes may caus e a glomerulonephritis, whi ch i s charact eri zed by el evat ed s erum creat i ni ne, hemat uri a, and cas t s i n t he uri ne, or rheumat ol ogi c mani fes t at i ons s uch as s t eri l e art hri t i s . The rol e of i mmune compl exes i n ot her as pect s of endocardi t i s i s not wel l unders t ood.
(5) Moderate anemia i s as s oci at ed wi t h endocardi t i s t hat has been pres ent for more t han 2 weeks .
P.361
o
o
d. T herapy. Treat ment has been careful l y s t udi ed. W hen endocardi t i s i s unt reat ed, i t i s al mos t uni forml y fat al . In general , pros t het i c val ve di s eas e i s more di ffi cul t t o t reat medi cal l y or s urgi cal l y.
(1) Antibiotic therapy al one provi des an excel l ent chance of cure for s t rept ococcal di s eas e on a nat i ve val ve and for s t aphyl ococcal di s eas e on t he t ri cus pi d val ve. The key i s t o provi de an adequat e dos e for a l ong enough peri od, us ual l y 4–8 weeks , dependi ng on t he organi s m.
(2) In medi cal fai l ures , valve replacement
Pa g e 1 7 5 3
ABC Ambe r CHM Conve rte r Tria l ve rsion, http://w w w .proce sste x t.com/a bcchm.html
may be a neces s ary adjunct t o ant i bi ot i c t herapy. Ot her i ndi cat i ons for val ve s urgery i ncl ude:
(a) Fungal endocardi t i s (an abs ol ut e i ndi cat i on)
(b) Conges t i ve heart fai l ure (CHF)
(c) Recurrent major embol i
(d) Inabi l i t y t o s t eri l i ze t he bl ood aft er 10–14 days
VI. Sexually Transmitted Diseases (STDs) A. Modes of transmission STDs us ual l y affect heal t hy adul t s . Mul t i pl e STDs can be pres ent at once. Mi croorgani s ms may be t rans mi t t ed duri ng i nt i mat e s exual rel at i ons i n t hree ways .
1. Cutaneous inoculation i s t he mos t common rout e for t he fi ve “cl as s i cal ― STDs (i .e., s yphi l i s , gonorrhea, chancroi d, l ymphogranul oma venereum, and granul oma i ngui nal e) as wel l as for herpes s i mpl ex. Appos i t i on of an i nfect ed s i t e t o a s us cept i bl e s i t e i n a part ner res ul t s i n a phys i cal t rans fer of mi croorgani s ms . Abras i on or t rauma may faci l i t at e t he i nfect i on of s ki n.
2. Blood-borne infection can be t rans mi t t ed by s exual act i vi t y. HBV, HCV, CMV, and HIV are al l t rans mi t t ed by i nocul at i on of mi cros copi c amount s of bl ood or s erum.
Pa g e 1 7 5 4
ABC Ambe r CHM Conve rte r Tria l ve rsion, http://w w w .proce sste x t.com/a bcchm.html
3. Enterically acquired infection may occur wi t h s exual act i vi t y becaus e t he anal area i s cl os e t o t he geni t al s or i s al s o us ed for s exual i nt eract i on. Shi gel l a, Ent amoeba, and HAV are exampl es of s exual l y t rans mi s s i bl e ent eri c di s eas es .
B. Urethritis
1. Etiology. Uret hri t i s i s cl as s i fi ed as gonococcal or nongonococcal. o
o
a. Gonococcal urethritis i s caus ed by N. gonorrhoeae. In al mos t al l cas es of gonococcal uret hri t i s , Gram s t ai n of t he uret hral di s charge s hows Gram-negat i ve i nt racel l ul ar di pl ococci .
o
o
b. Nongonococcal urethritis i s caus ed by C. t rac homat i s , Ureapl as ma ureal yt i c um, or s ome ot her, yet uni dent i fi ed, agent . (In 25% of cas es of gonococcal uret hri t i s , one of t hes e organi s ms al s o i s pres ent , and pat i ent s may have recurrent s ympt oms aft er t herapy for gonorrhea.)
2. Clinical features o
o
a. Dysuria i s obs erved i n mos t cas es of uret hri t i s , whet her gonococcal or nongonococcal .
o
o
b. Urethral discharge i s obs erved more frequent l y
Pa g e 1 7 5 5
ABC Ambe r CHM Conve rte r Tria l ve rsion, http://w w w .proce sste x t.com/a bcchm.html
i n men t han women and may be purul ent (us ual l y i n gonococcal di s eas e) or cl oudy and mucoi d (us ual l y i n nongonococcal di s eas e).
3. T herapy o
o
a. A vari et y of antibiotic regimens can be us ed t o t reat gonorrhea. However, res i s t ance pat t erns change, and up-t o-dat e recommendat i ons s houl d be s ought .
o
o
b. Many cl i ni ci ans recommend t hat t reat ment wi t h a t et racycl i ne or macrol i de fol l ow gonococcal t herapy t o el i mi nat e pos s i bl e s i mul t aneous nongonococcal uret hri t i s . A 7-day cours e of a t et racycl i ne or eryt hromyci n or a s i ngl e dos e of azi t hromyci n i s us ual l y s uffi ci ent , al t hough s ome pat i ent s experi ence a rel aps e wi t hi n a few weeks . Thes e pat i ent s us ual l y res pond t o anot her cours e of ant i bi ot i cs .
C. Pelvic inflammatory disease (PID) PID refers t o a compl ex of i nfect i ons i nvol vi ng t he ut erus , fal l opi an t ubes , or l i gament s of t he ut erus .
1. Etiology o
o
a. N. gonorrhoeae, C. t rac homat i s , or a mi xt ure of pel vi c anaerobes may be i nvol ved, al t hough i t i s di ffi cul t t o det ermi ne whi ch of t hes e i s res pons i bl e when i ns t i t ut i ng t herapy.
o
Pa g e 1 7 5 6
ABC Ambe r CHM Conve rte r Tria l ve rsion, http://w w w .proce sste x t.com/a bcchm.html
o
b. The pres ence of an i nt raut eri ne devi ce (IUD) may predi s pos e pat i ent s t o PID.
P.362
2. Clinical features and laboratory findings. The s ympt oms may be cont emporaneous wi t h mens t ruat i on and us ual l y cons i s t of l ower abdomi nal or pel vi c pai n and t endernes s on pal pat i on of t he cervi x, ut erus , or adnexa. Fever i s not neces s ari l y a s i gni fi cant feat ure. A cervi cal di s charge or pel vi c mas s may be pres ent , and t he W BC count may be normal or el evat ed.
3. Diagnosis o
o
a. It i s i mport ant t o obt ai n cul t ures or ot her di agnos t i c t es t s for N. gonorrhoeae and Chl amydi a.
o
o
b. Ul t ras onography of t he pel vi s may demons t rat e an adnexal mas s or abs ces s , whi ch s houl d be fol l owed careful l y.
4. T herapy. No s i mpl e t herapy i s effect i ve i n al l cas es of PID. Hos pi t al i zat i on i s s ugges t ed when pai n i s i ncapaci t at i ng or when parent eral t herapy i s gi ven. Combi nat i ons of ant i bi ot i cs are us ual l y needed t o cover t he t hree major cat egori es of pos s i bl e pat hogens .
5. Complications. The mos t s eri ous l ong-t erm
Pa g e 1 7 5 7
ABC Ambe r CHM Conve rte r Tria l ve rsion, http://w w w .proce sste x t.com/a bcchm.html
compl i cat i ons of PID are i nfert i l i t y, ect opi c pregnancy, and t he need for hys t erect omy.
D. Infectious proctitis
1. Etiology. Infect i ous proct i t i s can be caus ed by a vari et y of mi croorgani s ms . W hen anal s ex i s pract i ced, gonorrhea, s yphi l i s , chl amydi al i nfect i on, and herpes s houl d be cons i dered.
2. Clinical features and diagnosis. Proct al gi a (rect al pai n), a change i n bowel habi t s , and a mucoi d or bl oody anal di s charge bet ween bowel movement s s ugges t i nfect i ous proct i t i s . A s exual hi s t ory s houl d be obt ai ned t o ai d i n t he di agnos i s , and appropri at e di agnos t i c s t udi es (e.g., s i gmoi dos copy, cul t ure and Gram s t ai n of t he di s charge, bi ops y) s houl d be performed.
3. T herapy. W hen s peci fi c agent s of i nfect i ous proct i t i s can be i dent i fi ed, appropri at e ant i bi ot i c t reat ment s houl d be admi ni s t ered. In general , t he s ame regi mens us ed t o t reat t hes e pat hogens i n ot her body s i t es are effect i ve i n proct i t i s . However, for gonorrheal proct i t i s , cure rat es are l ower t han for gonorrheal uret hri t i s , and pos t -t herapy cul t ures s houl d be obt ai ned.
E. Syndromes of genital ulcers and lymphadenopathy
1. Incidence. Ul cerat i ve l es i ons of t he geni t al i a are common out pat i ent probl ems .
Pa g e 1 7 5 8
ABC Ambe r CHM Conve rte r Tria l ve rsion, http://w w w .proce sste x t.com/a bcchm.html
2. Etiology. There are many caus es , whi ch vary i n di fferent part s of t he worl d. In t he Uni t ed St at es , genital herpes i s t he mos t common caus e of geni t al ul cers , fol l owed by syphilis. Ot her caus es i ncl ude lymphogranuloma venereum, chancroid, and granuloma inguinale (donovanosis), al l of whi ch are uncommon i n t he Uni t ed St at es . P.363
3. Clinical features (Tabl e 8-5)
See Onl i ne Fi gures 8-5, 8-6, 8-7,
8-8, 8-9, 8-10 and 8-11 for col or phot os of t hes e condi t i ons .
ONLINE FIGURE 8-5 Geni t al herpes . (From Faro S. Sexual l y T rans mi t t ed Di s eas es i n Women. Phi l adel phi a: Li ppi ncot t W i l l i ams & W i l ki ns , 2003:CP14-A ).
Pa g e 1 7 5 9
ABC Ambe r CHM Conve rte r Tria l ve rsion, http://w w w .proce sste x t.com/a bcchm.html
ONLINE FIGURE 8-6 Pri mary s yphi l i s , peni l e chancre. (From Hal l JC. Sauer's Manual of Ski n Di s eas es . 8t h ed. Phi l adel phi a: Li ppi ncot t W i l l i ams & W i l ki ns , 1999:16-2A. )
ONLINE FIGURE 8-7 Secondary s yphi l i s . Macul opapul ar ras h on t he s ol e of t he foot of a pat i ent wi t h s econdary s yphi l i s . (From Faro S. Sexual l y T rans mi t t ed Di s eas es i n Women. Phi l adel phi a: Li ppi ncot t W i l l i ams & W i l ki ns , 2003:CP25 ).
Pa g e 1 7 6 0
ABC Ambe r CHM Conve rte r Tria l ve rsion, http://w w w .proce sste x t.com/a bcchm.html
ONLINE FIGURE 8-8 Lymphogranul oma venereum. (From Faro S. Sexual l y T rans mi t t ed Di s eas es i n Women. Phi l adel phi a: Li ppi ncot t W i l l i ams & W i l ki ns , 2003:CP22 ).
ONLINE FIGURE 8-9 Chancroi d. (From Kaufman RH, Faro S, Fri edri ch FG, et al . Beni gn Di s eas es of t he Vul va and Vagi na. St . Loui s : Mos by, 1994. )
Pa g e 1 7 6 1
ABC Ambe r CHM Conve rte r Tria l ve rsion, http://w w w .proce sste x t.com/a bcchm.html
ONLINE FIGURE 8-10 Granul oma i ngui nal e. (From Faro S. Sexual l y T rans mi t t ed Di s eas es i n Women. Phi l adel phi a: Li ppi ncot t W i l l i ams & W i l ki ns , 2003:CP3 ).
ONLINE FIGURE 8-11 Granul oma i ngui nal e. Donovan bodi es are vi s i bl e wi t hi n vacuol es of a mononucl ear cel l . Not e t he bi pol ar s t ai ni ng, whi ch i s at ypi cal . (From Faro S. Sexual l y T rans mi t t ed Di s eas es i n Women. Phi l adel phi a: Li ppi ncot t W i l l i ams & W i l ki ns , 2003:CP5. ) o
o
a. Genital herpes i ni t i al l y mani fes t s as i t chi ng and s orenes s fol l owed by t he appearance of eryt hema
Pa g e 1 7 6 2
ABC Ambe r CHM Conve rte r Tria l ve rsion, http://w w w .proce sste x t.com/a bcchm.html
and, event ual l y, t he devel opment of herpet i c ves i cl es . In i mmunocompromi s ed pat i ent s (e.g., t rans pl ant reci pi ent s , pat i ent s wi t h HIV), ves i cl es may be confl uent and l ead t o l arge ul cers . o
o
b. Syphilis, i n i t s pri mary form, mani fes t s as a pai nl es s , oft en s ol i t ary, chancre (ul cer) wi t h a hard, i ndurat ed bas e. Oral and vul var l es i ons may be s ubt l e, and oral l es i ons may be pai nful .
o
o
c. Lymphogranuloma venereum mani fes t s as nodes t hat are di s proport i onat el y l arge compared wi t h t he ul cers charact eri zed by a depres s i on bet ween t he i ngui nal and femoral nodes (groove sign).
o
o
d. Chancroid i s charact eri zed by mul t i pl e, pai nful ul cers wi t h ragged, undermi ned edges and s uppurat i ve i ngui nal nodes .
o
o
e. Granuloma inguinale i s a more i ndol ent i nfect i on charact eri zed by a pai nl es s , beefy-red l es i on wi t h ragged edges and l es s promi nent adenopat hy. It i s rare i n t he Uni t ed St at es . The cl i ni cal charact eri s t i cs and cours e of i l l nes s are more l i ke geni t al cancer t han ot her STDs .
4. Diagnosis (s ee Tabl e 8-5) o
o
a. W hen a genital ulcer i s not ed, a T zanck smear for herpes can hel p make a di agnos i s . However,
Pa g e 1 7 6 3
ABC Ambe r CHM Conve rte r Tria l ve rsion, http://w w w .proce sste x t.com/a bcchm.html
many l abs us e s hel l vi al s , whi ch produce res ul t s i n 1 day. Vi ral cul t ures are hel pful es peci al l y wi t h res i s t ant vi rus es . o
b. In al l cas es of geni t al ul cers , s erol ogi c t es t i ng
o
for s yphi l i s s houl d be performed. o
c. Di agnos i s of lymphogranuloma venereum i s
o
di ffi cul t .
TABLE 8-5 Sexually Transmitted Diseases C a u s Di at a Cl iv g in e n ic O o al Di r st Fi s g ic n e a T di a ni e n s s st g e m s s G H Di L e er re e ni p ct s i ta e i l
o
s mn
h si m â
Pa g e 1 7 6 4
ABC Ambe r CHM Conve rte r Tria l ve rsion, http://w w w .proce sste x t.com/a bcchm.html
er m u €” p pl n m e e of ul s x lu ti vi or pl ru e e, s sc v e e nc s i e; cu vi l o ra p l
u
cu s t l t ul ur ar e ; p ai nf ul S TrD L y e ar e p p kf s i hi o i e o li n ld n s e mâ m i c €” a ro u p sc s al o u l i p al d y; l y u R s m P ol
Pa g e 1 7 6 5
ABC Ambe r CHM Conve rte r Tria l ve rsion, http://w w w .proce sste x t.com/a bcchm.html
R it t e ar s t y; in d ur at e d; p ai nl e ss N o d e s â €” ru b b er y; n ot fl uc tu a nt Ly C C L m hl ul e
Pa g e 1 7 6 6
ABC Ambe r CHM Conve rte r Tria l ve rsion, http://w w w .proce sste x t.com/a bcchm.html
p a tu si h m re o o y ; n gr di s s a a er â n t r ol €” ul a o s o c gi m m h c al a o e l v mx p e at a a n is m p er
i n ul
e
at e
u
i o or
m
n v e si cl e N o d e s â €” la rg e; s u p p
Pa g e 1 7 6 7
ABC Ambe r CHM Conve rte r Tria l ve rsion, http://w w w .proce sste x t.com/a bcchm.html
ur at iv e C H Gr L h a a e a e m si nc m s t o ro o ai n i d p n; â hi cu €” l u l t ra s ur g d e g u (s e c r p d; e ec s yi i a of l
t;
m di e rt di y a lo n o e ki e n d g e N d) o d e s â €” te
Pa g e 1 7 6 8
ABC Ambe r CHM Conve rte r Tria l ve rsion, http://w w w .proce sste x t.com/a bcchm.html
n d er ; s u p p ur at iv e Gr C Bi L T a al o e et n y p s i ra ul m s y o cy o m wi n cl m at t h â i n a o Gi €” e; in b e la tr g a m rg i ui c t s e; m n er a s l et al i u or o h e m W wl o gr ri y pr a g a i n ht d m ul 's v â o s t a €“ m ai nc s at n i n ul is
g; fa ro m l l et
Pa g e 1 7 6 9
ABC Ambe r CHM Conve rte r Tria l ve rsion, http://w w w .proce sste x t.com/a bcchm.html
e h d o e x d a g z e ol s; e n ot in d ur at e d N o d e s â €” n ot pr o m in e nt RPR, rapi d pl as ma reagi n. o
o
d. Di agnos i s of chancroid i nvol ves el i mi nat i ng
Pa g e 1 7 7 0
ABC Ambe r CHM Conve rte r Tria l ve rsion, http://w w w .proce sste x t.com/a bcchm.html
ot her caus es of geni t al ul cers and i s ol at i ng Haemophi l us duc reyi from t he ul cers or s uppurat i ve nodes . o
o
e. Di agnos i s of granuloma inguinale i s confi rmed by demons t rat i on of Donovan bodi es i n edge s crapi ngs prepared wi t h Gi ems a or W ri ght 's s t ai n.
5. T herapy. Ident i fi cat i on and t reat ment of s exual cont act s i s al ways des i rabl e. o
o
a. Herpes infections are s el f-l i mi t ed but recurrent . Acycl ovi r, famci cl ovi r, or val acycl ovi r may s hort en t he cours e and reduce s ympt oms but do not affect t he nat ural hi s t ory of t hes e i nfect i ons . Pat i ent s wi t h frequent recurrences can us e l ower dos es of t hes e s ame medi ci nes as prophyl axi s .
o
o
b. Syphilis i s t reat ed wi t h varyi ng s chedul es of peni ci l l i n, dependi ng on t he s t age. Tet racycl i ne may be us eful for pat i ent s wi t h peni ci l l i n i nt ol erance or ot her s peci al requi rement s , but peni ci l l i n i s cl earl y preferred even i f i t requi res des ens i t i zat i on.
o
o
c. Lymphogranuloma venereum i s t reat ed wi t h a t et racycl i ne or a macrol i de.
o
o
d. Chancroid i s t reat ed wi t h azi t hromyci n, ceft ri axone, fl uoroqui nol ones , or eryt hromyci n. However, res i s t ance pat t erns of H. duc reyi are
Pa g e 1 7 7 1
ABC Ambe r CHM Conve rte r Tria l ve rsion, http://w w w .proce sste x t.com/a bcchm.html
vari abl e, and cul t ure s ens i t i vi t i es s houl d be obt ai ned i f pos s i bl e. o
o
e. Granuloma inguinale i s t reat ed wi t h a t et racycl i ne or t ri met hopri m–s ul famet hoxaz ol e.
6. Late complications o
o
a. Herpes simplex recurrences t end t o be mos t common earl y aft er acqui s i t i on but may cont i nue t o occur for many years and may become s evere i n i mmunocompromi s ed pat i ent s . Herpes al s o can compl i cat e part uri t i on and caus e devas t at i ng i nfect i on i n neonat es .
o
o
b. Syphilitic chancres heal s pont aneous l y aft er 1–2 weeks . However, i n t he s econdary phas e, s yphi l i s can mani fes t as a mul t i s ys t em di s eas e i ncl udi ng, but not l i mi t ed t o, l ymphadenopat hy, ras h (es peci al l y on t he pal ms and s ol es ), fever, pharyngi t i s , and meni ngi t i s . The mul t i s ys t em di s eas e res ol ves s pont aneous l y, and a l at ent , noni nfect i ve phas e ens ues . Mos t pat i ent s remai n s eropos i t i ve. In t ert i ary s yphi l i s , approxi mat el y 10% of pat i ent s devel op s eri ous l at e compl i cat i ons of t he aort a or t he CNS. The neurol ogi c l es i ons are vari ed but i ncl ude pupi l l ary di s t urbances , pos t eri or s pi nal col umn probl ems , and major cogni t i ve i mpai rment . Treat ment may hal t progres s i on but i s unl i kel y t o revers e damage, maki ng earl y di agnos i s and t reat ment cri t i cal .
o
Pa g e 1 7 7 2
ABC Ambe r CHM Conve rte r Tria l ve rsion, http://w w w .proce sste x t.com/a bcchm.html
o
c. Lymphogranuloma venereum may rarel y l ead t o geni t al or rect al s carri ng.
P.364
VII. Other Infectious Diseases and Syndromes A. Infections associated with adenopathy and splenomegaly Syndromes of adenopat hy (bot h l ocal and general ), s pl enomegal y, and fever are common medical probl ems . The di fferent i al di agnos i s s houl d i ncl ude t umors , rheumat i c di s eas es , and vas cul i t i s as wel l as s peci fi c i nfect i ous proces s es .
1. Generalized lymphadenopathy and fatigue are t he cl as s i c s i gns of infectious mononucleosis. o
o
a. Mononucl eos i s mos t commonl y i s caus ed by t he Epstein-Barr virus (EBV) and i s as s oci at ed wi t h s pl enomegal y, pharyngi t i s , and an at ypi cal l ymphocyt os i s .
o
o
b. CMV caus es a mononucl eos i s s yndrome t hat i s vi rt ual l y i ndi s t i ngui s habl e from EBV-i nduced mononucl eos i s .
o
o
c. Bot h i nfect i ons t end t o occur i n adol es cent s and young adul t s and may be s ubcl i ni cal .
Pa g e 1 7 7 3
ABC Ambe r CHM Conve rte r Tria l ve rsion, http://w w w .proce sste x t.com/a bcchm.html
(1) They are di s t i ngui s hed by s erol ogi c t es t s s uch as t he Monospot test, whi ch us ual l y demons t rat es t he pres ence of het erophi l e ant i body i n EBV i nfect i on and t he abs ence of het erophi l e ant i body i n CMV i nfect i on.
(2) Speci fi c ant i body t es t i ng for t he t wo vi rus es can confi rm t he di agnos i s i n equi vocal cas es .
(3) Bot h vi rus es can be t rans mi t t ed by i nt i mat e cont act i ncl udi ng, but not l i mi t ed t o, s exual i nt ercours e.
o
o
d. Some cas es of mononucl eos i s are caus ed by T oxopl as ma gondi i , but t hi s i nfect i on us ual l y i s s ubcl i ni cal or pres ent s as a mi l d “vi ral -t ype s yndrome― i n heal t hy adul t s .
o
o
e. Early HIV infection may mani fes t as fever, l ymphadenopat hy, and fat i gue, as wel l as ras h, neck s t i ffnes s , or ot her feat ures of a “vi ral i nfect i on.― Thi s s et of s ympt oms occurs i n 30%–60% of pat i ent s who experi ence acut e HIV i nfect i on. (Thi s i s s omet i mes cal l ed s eroconvers i on i l l nes s , becaus e ant i bodi es t o HIV become apparent at t he end of t hi s peri od.) Lymphadenopat hy al one or wi t h a vari et y of cons t i t ut i onal probl ems al s o occurs i n mi d t o l at e s t ages of HIV i nfect i on.
Pa g e 1 7 7 4
ABC Ambe r CHM Conve rte r Tria l ve rsion, http://w w w .proce sste x t.com/a bcchm.html
2. Fever, adenopat hy, and fat i gue al s o may be manifestations of secondary syphilis.
3. Splenomegaly out of proportion to adenopathy o
o
a. Thi s cl i ni cal pres ent at i on i s charact eri s t i c of onl y a few i nfect i ons s uch as mal ari a, s chi s t os omi as i s , and kal a-azar (vi s ceral l ei s hmani as i s ).
o
o
b. Thi s s yndrome al s o s houl d s ugges t a malignancy s uch as l ymphoma or Hodgki n's di s eas e, an infiltrative disease s uch as Gaucher's di s eas e, a congestive disease s uch as hepat i c ci rrhos i s , or a connective tissue disease s uch as SLE.
o
o
c. Local infections s uch as s pl eni c abs ces s and l eft -s i ded s ubphreni c abs ces s may mani fes t as a pal pabl e s pl een.
4. Localized adenopathy hel ps pi npoi nt a pot ent i al infection or tumor. o
o
a. A few s mal l (<8 mm) l ymph nodes i n t he i ngui nal , axi l l ary, or cervi cal regi on may be pres ent i n mos t heal t hy adul t s . If t he nodes are nont ender and fi rm but not rock hard and do not change over a peri od of weeks t o mont hs , t hey us ual l y do not warrant at t ent i on.
o
Pa g e 1 7 7 5
ABC Ambe r CHM Conve rte r Tria l ve rsion, http://w w w .proce sste x t.com/a bcchm.html
o
b. Lymphadenopat hy i n ot her areas i s unus ual wi t hout an obvi ous i nfect i on i n t he regi on drai ned by t hos e nodes .
o
o
c. Cut aneous i nocul at i on wi t h an i nfect i ve agent may res ul t i n a l ocal l es i on and regi onal adenopat hy. Exampl es are s porot ri chos i s (from expos ure t o Sporot hri x s c henc ki i , a pl ant -as s oci at ed fungus ) and cat -s crat ch di s eas e ( Bart onel l a hens el ae i nfect i on fol l owi ng cat bi t e or s crat ch).
B. Infections due to parasites Thes e i nfect i ons are more common i n t ropi cal cl i mat es and may be as s oci at ed wi t h eos i nophi l i a.
1. Mal ari a i s caus ed by a bl ood-borne paras i t e, Pl as modi um, whi ch i s endemi c t o much of t he t ropi cal and s ubt ropi cal worl d, res ul t i ng i n nearl y 3 mi l l i on deat hs per year. Of t he four t ypes of Pl as modi um s peci es (P. fal c i parum, P. oval e, P. mal ari ae, and P. vi vax), i nfect i on wi t h Pl as modi um fal c i parum i s t he mos t s evere and charact eri zed by fever, chi l l s , myal gi as , naus ea, vomi t i ng, or di arrhea. o
o
a. Diagnosis. Bl ood s mear s houl d be performed i n t hos e wi t h hi gh cl i ni cal s us pi ci on, s uch as i ndi vi dual s t ravel i ng from an endemi c regi on. Smear s houl d reveal paras i t es . P.365
o
Pa g e 1 7 7 6
ABC Ambe r CHM Conve rte r Tria l ve rsion, http://w w w .proce sste x t.com/a bcchm.html
o
b. T reatment. Due t o hi gh chl oroqui ne res i s t ance, agent s s uch as qui ni ne, s ul fadi ne, and pyri met hami ne are us ed i n addi t i on t o a new cl as s of ant i mal ari al drugs cl as s i fi ed as art emi s i ni n deri vat i ves .
o
o
c. Prophylaxis. Al l t ravel ers t o mal ari a-endemi c areas s houl d t ake prophyl axi s bas ed on t he res i s t ance pat t erns of mal ari a n t hat regi on.
2. Prot ozoa s uch as Ent amoeba and Gi ardi a are as s oci at ed wi t h di arrheal i l l nes s es .
3. Schi s t os omes (bl ood fl ukes ) are hel mi nt hs not l i mi t ed t o t he l umen of t he bowel and can produce eos i nophi l i a.
4. St rongyl oi des . St rongyl oi des s t erc oral i s i s t he mos t i mport ant hel mi nt h as s oci at ed wi t h eos i nophi l i a i n t he Uni t ed St at es . Sus pect s t rongyl oi di as i s i n pers ons who l i ve or have l i ved i n t ropi cal areas (or i n t he s out heas t ern Uni t ed St at es ) and who have pers i s t ent eos i nophi l i a wi t h or wi t hout cut aneous or gas t roi nt es t i nal compl ai nt s . St rongyl oi des l arvae can cros s t he bowel wal l , s o t hey may be as s oci at ed wi t h epi s odes of pol ymi crobi al bact eremi a i nvol vi ng t ypi cal bowel fl ora. Unt reat ed di s eas e can l as t for decades and wors en duri ng peri ods of i mmunos uppres s i on.
C. Tuberculosis Thi s i l l nes s remai ns a major medi cal probl em i n cert ai n i mmi grant and underpri vi l eged groups i n t he Uni t ed St at es as wel l as a wi des pread di s eas e t hroughout t he worl d.
Pa g e 1 7 7 7
ABC Ambe r CHM Conve rte r Tria l ve rsion, http://w w w .proce sste x t.com/a bcchm.html
1. Clinical syndromes o
o
a. Pulmonary tuberculosis i s t he mos t common form. In mos t adul t s , t hi s condi t i on i s charact eri zed by an i ncreas ed cough (pos s i bl y wi t h al t ered s put um), wei ght l os s , hemopt ys i s , and fat i gue. Mos t cas es of pul monary t ubercul os i s are bel i eved t o be a react i vat i on of M. t uberc ul os i s acqui red mont hs t o years earl i er rat her t han rei nfect i on or i ni t i al i nfect i on by t hi s bact eri um. However, rei nfect i on has been confi rmed i n pat i ent s bot h wi t h and wi t hout HIV i nfect i on. Pri mary di s eas e may res embl e bact eri al pneumoni a and s houl d be es peci al l y s us pect ed when a cl os e cont act has recent l y been di s covered t o have act i ve t ubercul os i s .
o
o
b. Extrapulmonary tuberculosis can devel op i n any organ, but t he mos t s eri ous l y affect ed are t he ki dneys , bones , and meni nges . Agai n, t he di agnos i s res t s on fi ndi ng M. t uberc ul os i s i n body fl ui d or t i s s ue. Onl y approxi mat el y 40% of pat i ent s wi t h ext rapul monary t ubercul os i s have cl i ni cal or radi ographi c evi dence of l ung i nvol vement at t he t i me of di agnos i s of t he ext rapul monary di s eas e.
2. Diagnosis. The di agnos i s i s bas ed on i dent i fi cat i on of aci d-fas t baci l l i —s peci fi cal l y, M. t uberc ul os i s , by mi cros copy, pol ymeras e chai n react i on (PCR), or cul t ure. However, t he di agnos i s may be di ffi cul t t o es t abl i s h, becaus e many pat i ent s have t oo few bact eri a t o be s een on di rect s t ai n, and i t t akes s everal weeks of i ncubat i on
Pa g e 1 7 7 8
ABC Ambe r CHM Conve rte r Tria l ve rsion, http://w w w .proce sste x t.com/a bcchm.html
for s peci mens t o grow. The ches t radi ograph us ual l y i s abnormal and s hows s i gns of a pri or expos ure t o M. t uberc ul os i s (e.g., cal ci fi ed or enl arged i nt rat horaci c l ymph nodes and i nfi l t rat es i n t he pos t eri or s egment of t he upper l obes ).
3. T herapy. Treat ment i s ai med at curi ng t he pat i ent who has a defi ni t e di agnos i s of t ubercul os i s . To be effect i ve, t he t herapy mus t i ncl ude at least four antimicrobial agents for induction, followed by at least two drug combinations for the remainder of treatment. Si ngl e-agent t reat ment has a hi gh ri s k of fai l ure becaus e of t he s el ect i on of drug-res i s t ant s t rai ns of t he i nfect i ng t ubercl e baci l l us . o
o
a. Becaus e of t hei r pot ency and rel i abi l i t y, isoniazid and rifampin are t he drugs of choi ce for t ubercul os i s . Ot her fi rs t -l i ne drugs i ncl ude et hambut ol , pyraz i nami de, and s t rept omyci n. Mos t aut hori t i es recommend s t art i ng four drugs i ni t i al l y t o provi de adequat e coverage, i n vi ew of pos s i bl e i s oni azi d or ri fampi n res i s t ance.
o
o
b. Second-l i ne drugs are us ed pri mari l y for pat i ent s who are i nt ol erant of t he fi rs t -l i ne agent s or who have drug-res i s t ant di s eas e. Thes e drugs i ncl ude et hi onami de, kanamyci n, and cycl os eri ne. Fl uoroqui nol ones have act i vi t y agai ns t M. t uberc ul os i s , whi ch woul d make t hem s eem t o be reas onabl e fi rs t -l i ne drugs , but t he need for l ong cours es and t he concern about s el ect i ng a res i s t ant bact eri al fl ora pus hes t hem t o s econd-l i ne s t at us .
o
Pa g e 1 7 7 9
ABC Ambe r CHM Conve rte r Tria l ve rsion, http://w w w .proce sste x t.com/a bcchm.html
o
c. The us ual durat i on of t reat ment i s 6–12 mont hs , dependi ng on t he pat i ent and t he regi men. Short er cours es have an unaccept abl y hi gh rat e of fai l ure.
P.366
4. Prevention o
a. Skin testing. The tuberculin skin test i s us ed wi del y t o s creen cert ai n hi gh-ri s k popul at i ons , part i cul arl y t hos e who have been expos ed t o an i nfect i ous i ndi vi dual . The t es t i nvol ves an i nt radermal i nject i on of t he purified protein derivative (PPD) of t ubercul i n
(s ee onl i ne for det ai l s ). Aft er 48–72 hours , t he
i nject i on s i t e i s exami ned for vi s i bl e and pal pabl e i ndurat i on. Rednes s wi t hout pal pabl e i ndurat i on does not i ndi cat e pos i t i vi t y. Becaus e of a pos s i bl e cros s -react i on aft er expos ure t o ot her mycobact eri a, a s i ngl e t ubercul i n s ki n t es t t o det ermi ne s ens i t i z at i on t o M. t uberc ul os i s i s cons i dered pos i t i ve onl y i f t he i ndurat i on at t he s ki n t es t s i t e meas ures at l eas t 15 mm i n di amet er i n i mmunocompet ent i ndi vi dual s , 10 mm i n s i ck pers ons wi t hout pri mary depres s i on of t hei r i mmune s ys t em, and 5 mm i n i mmunos uppres s ed i ndi vi dual s (e.g., organ t rans pl ant reci pi ent s or pat i ent s wi t h advanced HIV i nfect i on). Newer t es t s l ooki ng for di rect i mmune res pons i venes s of l ymphocyt es t o mycobact eri al component s can be done i n vi t ro and may yi el d l es s equi vocal and more rapi d res ul t s t han s ki n t es t s . o
o
b. T reatment of latent tuberculosis. Indi vi dual s wi t h a pos i t i ve PPD and no evi dence of act i ve di s eas e are cons i dered t o harbor l at ent t ubercul os i s , whi ch mus t be t reat ed t o prevent
Pa g e 1 7 8 0
ABC Ambe r CHM Conve rte r Tria l ve rsion, http://w w w .proce sste x t.com/a bcchm.html
react i vat i on. In many cas es , di s eas e can be prevent ed by admi ni s t eri ng i s oni azi d al one for 6–12 mont hs , at a dos age of 300 mg/day. Pyri doxi ne i s co-admi ni s t ered wi t h i s oni azi d t o prevent peri pheral neuropat hy. o
o
c. Immunization. Vacci nat i on of chi l dren and adul t s wi t h baci l l e Cal met t e-Guéri n (BCG) has been report ed t o reduce t he ri s k of acqui ri ng t ubercul os i s . BCG, a l i ve bact eri al vacci ne, s houl d not be us ed when t here i s known i mmunodefi ci e ncy. The PPD s ki n t es t can become pos i t i ve aft er BCG admi ni s t rat i on. The BCG vacci ne i s s el dom us ed i n t he Uni t ed St at es , but i t i s wi del y us ed i n ot her count ri es . Pri or BCG vacci nat i on does not al t er t he deci s i on t o us e chemoprophyl axi s .
o
o
d. Isolation. Becaus e of t he potential hazard of transmission of t ubercul os i s i n t he hos pi t al , i t i s i mport ant t o i dent i fy pot ent i al l y i nfect i ve pat i ent s and t o ens ure adequat e cont ai nment of t hei r i nfect i ous aeros ol s . Any pat i ent s us pect ed of act i ve t ubercul os i s s houl d be i s ol at ed wi t h negat i ve pres s ure vent i l at i on, and caregi vers s houl d wear mas ks unt i l t ubercul os i s i s rul ed out (l at ent t ubercul os i s does not requi re i s ol at i on).
D. Infections associated with diffuse rash and fever
1. T oxin-associated diseases o
Pa g e 1 7 8 1
ABC Ambe r CHM Conve rte r Tria l ve rsion, http://w w w .proce sste x t.com/a bcchm.html
o
a. T oxic shock syndrome (TSS) occurs when a s us cept i bl e i ndi vi dual i s col oni zed or i nfect ed by a s t rai n of S. aureus t hat produces a t oxi n (TSST-1). Thi s t oxi n i s a s uperant i gen t hat can s t i mul at e t he i mmune s ys t em i n a way t hat l eads t o s evere hypot ens i on and organ damage. Mos t adul t s have ant i body t o t hi s t oxi n and are t hus i mmune. Many cas es have been as s oci at ed wi t h t ampon us e i n young women, but current t ampons s eem t o be s afer, and mens t rual TSS i s cons i derabl y l es s common t han i t was i n t he earl y 1980s . Toxi n-produci ng S. pyogenes (group A s t rept ococci ) can caus e a very s i mi l ar i l l nes s .
(1) Clinical features
(a) TSS i s charact eri zed by fever, hypot ens i on, di arrhea, mucous membrane changes , and a di ffus e eryt hemat ous ras h wi t h des quamat i on on t he hands and feet . Mul t i s ys t em i nvol vement i s t he rul e, wi t h i nvol vement of gas t roi nt es t i nal , renal , hepat i c, hemat opoi et i c, and mus cul os kel et al organs .
(b) TSS vari es from a mi l d i l l nes s t o a l i fe-t hreat eni ng di s eas e. Hypot ens i on from fl ui d l os s and l ack of vas cul ar t one i s t he mos t omi nous prognos t i c s i gn. There may be recurrences unt i l prot ect i ve ant i body i s formed.
Pa g e 1 7 8 2
ABC Ambe r CHM Conve rte r Tria l ve rsion, http://w w w .proce sste x t.com/a bcchm.html
(2) T herapy. The t reat ment for acut e i l l nes s i s supportive, cons i s t i ng of fl ui ds and pres s ors . Ant i bact eri al t herapy i s gi ven t o prevent recurrence.
o
o
b. Scarlet fever i s an i l l nes s caus ed by i nfect i on wi t h t oxi geni c S. pyogenes (group A s t rept ococci ). It i s charact eri zed by fever, rough eryt hemat ous di ffus e ras h, mucous membrane eryt hema (i ncl udi ng s t rawberry t ongue), and l ocal s t rept ococcal i nfect i on (us ual l y i nvol vi ng t he s ki n). Les s s evere t han TSS, s carl et fever us ual l y res ponds t o s upport i ve meas ures and ant i s t rept ococcal t herapy.
2.
Non–t oxi n-medi at ed i l l nes s wi t h fever and ras h o
o
a. Adverse reactions to medications i ncl ude combi nat i ons of fever, ras h, and eos i nophi l i a. React i on s everi t y ranges from t ri vi al t o l i fe-t hreat eni ng.
(1) The mos t common offendi ng agent s i ncl ude ant i convul s ant s , oral hypogl ycemi cs , abacavi r for HIV di s eas e, and ant i bi ot i cs (es peci al l y β-l act ams and s ul fa drugs ). Unl i ke acut e hypers ens i t i vi t y react i ons , whi ch occur wi t hi n mi nut es t o hours of admi ni s t rat i on, t he ras h or fever of drug-i nduced react i ons us ual l y appears aft er a vari abl e i nt erval of unevent ful t herapy.
Pa g e 1 7 8 3
ABC Ambe r CHM Conve rte r Tria l ve rsion, http://w w w .proce sste x t.com/a bcchm.html
(2) The mos t s eri ous react i on, t oxi c epi dermal necrol ys i s (Stevens-Johnson syndrome), i s charact eri zed by di ffus e macul ar erupt i on and ext ens i ve i nvol vement of t he mucous membranes . If t he affect ed s ki n s l oughs , fat al i t y may occur becaus e of fl ui d derangement and s eps i s .
b. A number of viral exanthems (s ki n ras hes ) and enanthems (mucous membrane ras hes ) are wel l des cri bed. Mos t us ual l y affect chi l dren and unvacci nat ed i ndi vi dual s and are mi l d, each havi ng a charact eri s t i c appearance and cours e. Meas l es (rubeol a), eryt hema i nfect i os um, and coxs acki e vi rus are among t he bes t known.
c. Spotted fevers are caus ed by Ri c ket t s i a s peci es . In t he Uni t ed St at es , t he mos t wel l known i s Rocky Mountain spotted fever (RMSF), caus ed by Ri c ket t s i a ri c ket t s i i . RMSF, whi ch i s t rans mi t t ed by t he bi t e of t he dog t i ck (Dermac ent or s peci es ), mos t oft en occurs i n t he s out heas t ern part of t he count ry. Aft er a del ay of s everal days , t here i s headache, fever, and a ras h, whi ch s t art s peri pheral l y and moves cent ral l y. Infect i on i nvol ves t he vas cul ar endot hel i um and may be fat al i n 15%–20% of unt reat ed pat i ent s . Us ual l y, t et racycl i ne or chl orampheni col i s effect i ve i f gi ven earl y enough. A s i mi l ar s yndrome, us ual l y wi t hout ras h, can be caus ed by Ehrl i c hi a c haffeens i s , anot her t i ck-borne
Pa g e 1 7 8 4
ABC Ambe r CHM Conve rte r Tria l ve rsion, http://w w w .proce sste x t.com/a bcchm.html
ri cket t s i al pat hogen.
E. Legionnaires' disease
1. Etiology. Legi onnai res ' di s eas e i s a pneumoni a caus ed by Legi onel l a pneumophi l a, a Gram-negat i ve bact eri um t hat dwel l s i n warm aquat i c envi ronment s . Several l egi onel l al i ke organi s ms have been di s covered, whi ch produce s i mi l ar but di s t i nct di s eas e pat t erns .
2. Epidemiology o
o
a. Infect i on occurs when cont ami nat ed wat er i s aeros ol i zed and t hen i nhal ed (e.g., duri ng nebul i zer t reat ment s ). Some out breaks of l egi onnai res ' di s eas e have been connect ed t o t he ai rborne s pread of cont ami nat ed fl ui d from ai r-condi t i oni ng cool i ng t owers or from pot abl e wat er. P.367
o
o
b. Indi vi dual s who are part i cul arl y vul nerabl e t o i nfect i on i ncl ude ci garet t e s mokers , peopl e wi t h underl yi ng l ung di s eas e, and i mmunos uppres s ed i ndi vi dual s (e.g., t hos e recei vi ng s t eroi d t herapy).
3. Clinical features o
o
a. Fever occurs i n al mos t al l cas es , i s abrupt , and us ual l y i s as s oci at ed wi t h s haki ng chi l l s . A s udden
Pa g e 1 7 8 5
ABC Ambe r CHM Conve rte r Tria l ve rsion, http://w w w .proce sste x t.com/a bcchm.html
headache may precede t he rapi d i ncreas e i n t emperat ure. o
o
b. Cough i s a common s ympt om, whi ch i ni t i al l y i s nonproduct i ve but progres s es t o a product i ve cough t hat may be as s oci at ed wi t h s l i ght hemopt ys i s .
o
o
c. Les s common s ympt oms i ncl ude di arrhea, naus ea, vomi t i ng, and pl euri t i c pai n.
4. Diagnosis. The di agnos i s i s made by culturing the bacterium from i nfect ed body s i t es (e.g., l ung t i s s ue, pl eural fl ui d, or s put um) or by demons t rat i ng t he bact eri um by i mmunofl uores cent , nucl ei c aci d hybri di zat i on, or ant i gen det ect i on t echni ques . Uri nary ant i gen t es t i ng i s avai l abl e, but onl y for L. pneumophi l a Type 1. W hen L. pneumophi l a i nfect i on cannot be confi rmed by t hes e met hods , i ncreas i ng t i t ers of ant i bodi es from t he acut e phas e t o conval es cence can be di agnos t i c.
5. T herapy. The preferred t herapy for al l Legi onel l a i nfect i ons i s a macrolide (e.g., azi t hromyci n) or a fluoroquinolone (e.g., l evofl oxaci n).
F. Lyme disease
1. Etiology. Lyme di s eas e i s a mul t i s ys t em i nfect i on caus ed by a s pi rochet e, Borrel i a burgdorferi , whi ch i s t rans mi t t ed by t i ck bi t e.
Pa g e 1 7 8 6
ABC Ambe r CHM Conve rte r Tria l ve rsion, http://w w w .proce sste x t.com/a bcchm.html
2. Epidemiology. The i nci dence of Lyme di s eas e i s rel at ed t o t he pres ence of i t s vect or (us ual l y I xodes t i cks ) and i nfect ed wi l d mammal s s uch as deer and mi ce. In t he Uni t ed St at es , endemi c foci have expanded from i ni t i al s mal l areas i n New Engl and t o i ncl ude areas as far s out h as Georgi a as wel l as areas i n t he Mi dwes t and t he Paci fi c s t at es . Lyme di s eas e i s t he mos t common t i ck-borne i l l nes s i n t he Uni t ed St at es .
3. Clinical features. Lyme di s eas e i s di vi ded i nt o t hree phas es . The phas es may fol l ow each ot her cl os el y or be s eparat ed by peri ods wi t hout s ympt oms . o
o
a. The first phase i s charact eri zed by an enl argi ng eryt hemat ous ras h (erythema migrans) at t he s i t e of t he ori gi nal t i ck bi t e. There may be cent ral cl eari ng or a few s at el l i t e l es i ons . Pat i ent s are oft en cons t i t ut i onal l y i l l wi t h mal ai s e, headache, and mi l d fever.
o
o
b. The second phase i nvol ves t he heart (conduct i on abnormal i t i es , arrhyt hmi as ) or t he nervous s ys t em (crani al or peri pheral neuropat hi es or as ept i c meni ngi t i s ). New onset of Bell' s palsy (paral ys i s of crani al nerve VII) s houl d s ugges t t he pos s i bi l i t y of Lyme di s eas e.
o
o
c. The third phase, whi ch affect s onl y a few pat i ent s , cons i s t s of an ol i goart i cul ar art hri t i s or s ome pers i s t ent , mi l d neurops ychi at ri c di s t urbances .
Pa g e 1 7 8 7
ABC Ambe r CHM Conve rte r Tria l ve rsion, http://w w w .proce sste x t.com/a bcchm.html
4. Diagnosis. The di agnos i s i s di ffi cul t . In earl y i l l nes s , t he product i on of ant i body t o B. burgdorferi may not be det ect abl e. Convers el y, peopl e l i vi ng i n endemi c areas may have ant i body wi t hout any cl i ni cal i l l nes s . The organi s m may be demons t rat ed rarel y i n s ki n bi ops i es and can s omet i mes be cul t ured. Hi gh l evel s of i mmunogl obul i n M (IgM) ant i body and i ncreas i ng l evel s of IgG ant i body t o one or more B. burgdorferi ant i gens corroborat e cl i ni cal s us pi ci ons of Lyme borrel i os i s .
5. T herapy. Earl y antibiotic therapy s eems t o prevent di s eas e progres s i on for mos t pat i ent s . Tet racycl i nes are preferred for adol es cent s and adul t s ; peni ci l l i n i s an al t ernat i ve. For l at er-s t age di s eas e, hi gh dos ages of peni ci l l i n, t et racycl i ne (es peci al l y doxycycl i ne), and ceft ri axone have been s hown t o rel i eve s ympt oms . Some oral cephal os pori ns and azi t hromyci n have al s o been s hown t o be us eful . A vacci ne t hat had been market ed t o prevent Lyme di s eas e i s no l onger avai l abl e.
VIII. Retrovirus Infection of Humans A. Etiology Ret rovi rus es are s i ngl e-s t randed RNA vi rus es charact eri zed by t he pres ence of reverse transcriptase, an enz yme t hat us es t he vi ral RNA as a t empl at e t o make a copy of compl ement ary DNA for i nt egrat i on i nt o t he hos t cel l . P.368
1. Ret rovi rus es known t o caus e di s eas e i n humans are t he human T -cell lymphotropic viruses types I and II (HTLV-I and HTLV-II) and HIV.
Pa g e 1 7 8 8
ABC Ambe r CHM Conve rte r Tria l ve rsion, http://w w w .proce sste x t.com/a bcchm.html
2. HIV compri s es t wo t ypes : t ype 1 (HIV-1) and t ype 2 (HIV-2). HIV-1 caus es AIDS.
B. Epidemiology Al l of t hes e agent s can be t rans mi t t ed from pers on t o pers on vi a s exual act i vi t y or mi ngl i ng of bl ood (as occurs vi a bl ood t rans fus i on or s hari ng of bl ood-cont ami nat ed needl es ) and from mot her t o chi l d i n ut ero.
1. HT LV-I i s found mos t commonl y i n t he Cari bbean bas i n and i n s out hern Japan. Up t o 10% of t he popul at i on i n cert ai n vi l l ages may be i nfect ed.
2. HT LV-II i s not known t o have s uch marked geographi c cl us t ers , and i nci dence s eems t o be l ow worl dwi de.
3. HIV o
o
a. HIV-1 i s found worl dwi de. Becaus e i t has a l ong l at ent peri od and one pers on may i nfect many ot hers , t hi s vi rus has s pread expl os i vel y s i nce 1981, when i t was fi rs t i dent i fi ed. Mal e homos exual act i vi t y was t he predomi nant mode of t rans mi s s i on i n Nort h Ameri ca and Europe duri ng t he 1980s . Int ravenous drug us e and het eros exual t rans mi s s i on account for an i ncreas i ng s hare of new i nfect i ons , es peci al l y among women, (one of t he fas t es t growi ng groups t o be i nfect ed). In Afri ca, het eros exual t rans mi s s i on i s predomi nant .
o
Pa g e 1 7 8 9
ABC Ambe r CHM Conve rte r Tria l ve rsion, http://w w w .proce sste x t.com/a bcchm.html
o
b. HIV-2 i s found pri mari l y i n wes t Afri ca.
C. Clinical features
1. HT LV-I i s t he caus at i ve agent of adul t T-cel l l eukemi a–l ymphoma. Thi s unus ual mal i gnancy, whi ch i s as s oci at ed wi t h s ki n i nvol vement and hypercal cemi a, i s di ffi cul t t o t reat . HTLV-I al s o caus es HTLV-I as s oci at ed myel opat hy (HAM), al s o referred t o as t ropi cal s pas t i c parapares i s (TSP). HAM pres ent s wi t h i ns i di ous ons et of weaknes s or s t i ffnes s i n one or bot h l egs , back pai n, and uri nary i ncont i nence fol l owed by s l owl y progres s i ve and unremi t t i ng t horaci c myel opat hy. However, most people i nfect ed wi t h HTLV-I have no clinical illness even aft er years or decades of i nfect i on.
2. HT LV-II has not been cat egori cal l y as s oci at ed wi t h any i l l nes s .
3. HIV o
o
a. HIV-1 i nfect i on has t radi t i onal l y been di vi ded i nt o t hree overl appi ng s t ages .
(1) In early illness (s eroconvers i on s t age), wi t hi n a few weeks or mont hs aft er expos ure t o HIV, whol e vi rus and vi ral ant i gens and nucl ei c aci d can be found i n t he bl ood and body fl ui ds . Al t hough many pat i ent s are as ympt omat i c, approxi mat el y 40% of pat i ent s have a bri ef i l l nes s marked by headache,
Pa g e 1 7 9 0
ABC Ambe r CHM Conve rte r Tria l ve rsion, http://w w w .proce sste x t.com/a bcchm.html
fever, s ki n ras h, or l ymphadenopat hy, whi ch res ol ves s pont aneous l y wi t hi n a few weeks . Mos t peopl e who become i nfect ed wi t h HIV-1 devel op ant i body wi t hi n 1–6 mont hs of expos ure.
(2) A cl i ni cal latent period, duri ng whi ch t here i s no cl i ni cal i l l nes s but vi rus can be det ect ed and i s repl i cat i ng, occurs i n al mos t al l HIV-1–i nfect ed pat i ent s . Dependi ng on hos t and vi ral fact ors , t he vi ral l oad vari es wi del y, from <50 copi es /mL t o s everal mi l l i on. The vi rus al t ers t he hos t i mmune res pons e. The earl i es t i ndi cat i ons may be decl i ni ng numbers of T hel per (Th) cel l s (CD4), whi ch are t arget s of t he vi rus , and normal t o i ncreas ed numbers of T s uppres s or (Ts ) cel l s (CD8), l eadi ng t o an abnormal CD4:CD8 rat i o (normal rat i o of Th t o Ts cel l s i s 2:1). Some pat i ent s have mi nor vi ral i nfect i ons or more-frequent -t han-us ual recurrences of oral or geni t al herpes s i mpl ex, herpes zos t er, oral hai ry l eukopl aki a, mi l d fever, s weat s , wei ght l os s , and di arrhea. Common bact eri al i nfect i ons , s uch as s t rept ococcal pneumoni a and t ubercul os i s ) may occur wi t h i ncreas ed frequency at any poi nt i n HIV-1 i nfect i on, but t end t o become more common as t he di s eas e progres s es .
(3) Advanced symptomatic HIV-1 infection i s mani fes t ed as AIDS. HIV i nfect i on i s bes t t hought of as a cont i nuum, wi t h evi dence of progres s i ve i mmune depl et i on correl at i ng wi t h
Pa g e 1 7 9 1
ABC Ambe r CHM Conve rte r Tria l ve rsion, http://w w w .proce sste x t.com/a bcchm.html
i ncreas ed probabi l i t y of s peci fi c i nfect i on or mal i gnancy; nons peci fi c l ocal i zed or general i zed compl ai nt s s uch as fever, fat i gue, or ni ght s weat s ; or organ dys funct i on s uch as renal fai l ure or dement i a. Ol der defi ni t i ons of AIDS are mos t l y us ed t o det ermi ne el i gi bi l i t y for di s abi l i t y i ns urance rat her t han t o gui de t herapy. P.369
o
o
b. Monitoring of HIV-1 di s eas e i s hel pful i n es t abl i s hi ng prognos i s and adjus t i ng t reat ment . Vari ous quant i t at i ve as s ays of vi ral nucl ei c aci d predi ct t he rat e of progres s i on of di s eas e; t hos e pat i ent s wi t h t he l owes t vi ral l oads t end t o remai n s ympt om free and l i ve l onger t han t hos e wi t h hi gh vi ral l oads . W hen t reat ment i s effect i ve, t hes e as s ays s how a drop t o l ow or undet ect abl e viral load. If vi ral l oad has been very l ow and t hen i ncreas es , t reat ment may be fai l i ng.
o
o
c. HIV-2 i nfect i on fol l ows t he s ame general pat t ern as HIV-1, but much l es s i s known. The cl i ni cal cours e i s us ual l y mi l der or s l ower. Several wel l -des cri bed cas es of AIDS have been as s oci at ed wi t h HIV-2 i n t he abs ence of HIV-1. It wi l l probabl y be s hown t hat det ermi nant s of i mmunos uppres s i on are t he mos t s i gni fi cant predi ct ors of whet her opport uni s t i c i nfect i on or mal i gnancy wi l l occur.
D. Therapy
Pa g e 1 7 9 2
ABC Ambe r CHM Conve rte r Tria l ve rsion, http://w w w .proce sste x t.com/a bcchm.html
1. HT LV-I. Treat ment of HTLV-I–rel at ed mal i gnanci es wi t h convent i onal cancer chemot herapy has been di s appoi nt i ng. However, t he us e of ant i ret rovi ral s di rect ed agai ns t HIV-1 has res ul t ed i n s ome durabl e remi s s i ons . No t reat ment of as ympt omat i c HTLV-I i nfect i ons i s neces s ary.
2. HT LV-II. There i s no need t o t reat HTLV-II i nfect i on at any s t age.
3. HIV o
o
a. HIV-1. At all stages of illness, the goal of therapy is to suppress viral replication as measured by viral load. Improvement i n cl i ni cal and i mmune paramet ers may be del ayed but us ual l y fol l ows i n a mat t er of weeks t o mont hs . Chemot herapy may i nfl uence t he progres s i on of HIV-1 i nfect i on. Careful eval uat i on and t i mel y ant i ret rovi ral t herapy may del ay or prevent t he devel opment of s i gni fi cant compl i cat i ons . W hen t o s t art t herapy i s a compl ex ques t i on t hat depends on a number of fact ors . For pat i ent s wi t h symptomatic i nfect i on, t herapy s houl d be s t art ed as s oon as pos s i bl e. For pat i ent s wi t h rapi dl y progres s i ve i nfect i on (as det ermi ned by hi gh vi ral l oads and droppi ng CD4 count s ), earl y t herapy i s al s o appropri at e. W hat i s mos t uncl ear i s when t o i ni t i at e t herapy i n a pers on wi t h good i mmune funct i on. Earl y i nt ervent i on wi l l pres erve i mmune funct i on but can be t axi ng vi s -á-vi s s i de effect s and cos t . Int errupt i ons of t herapy are cont rovers i al , and recent s t udi es have s hown t hat
Pa g e 1 7 9 3
ABC Ambe r CHM Conve rte r Tria l ve rsion, http://w w w .proce sste x t.com/a bcchm.html
i nt errupt i ng ant i ret rovi ral t herapy may i ncreas e t he l i kel i hood of morbi di t y and mort al i t y. Therefore, s t ruct ured t reat ment i nt errupt i ons (STI) s houl d onl y be undert aken i n t he cont ext of a res earch s t udy.
(1) A vari et y of drugs have been s hown t o be effect i ve i n reduci ng t he vi ral l oad and i mprovi ng i mmune funct i on. Becaus e recommendat i ons change qui ckl y, t he deci s i on about whi ch t herapy t o us e s houl d be bas ed on current s t udi es . As of 2004, fi ve cl as s es of drugs for t he t reat ment of HIV-1 are avai l abl e: nucleoside reverse transcriptase inhibitors (NRT Is), nucleotide reverse transcriptase inhibitors, non-nucleoside reverse transcriptase inhibitors (NNRT Is), fusion inhibitors, and protease inhibitors (PIs).
(2) Evi dence i ndi cat i ng t hat combinations of agent s are much more effect i ve i s very s t rong. Al t hough s i ngl e agent s can be s hown t o have act i vi t y (as meas ured by reduci ng vi ral l oad and i ncreas i ng CD4 count s ), t here i s no occas i on t o us e t hem al one due t o al mos t uni vers al devel opment of res i s t ant vi rus t reat ed wi t h monot herapy or dual t herapy.
(3) Failure to adequately suppress viral replication or rebound vi ral repl i cat i on i s a poor prognostic sign. A re-eval uat i on of t reat ment i s warrant ed i n t hes e cas es .
Pa g e 1 7 9 4
ABC Ambe r CHM Conve rte r Tria l ve rsion, http://w w w .proce sste x t.com/a bcchm.html
(4) Resistance t o ant i ret rovi ral drugs occurs by mut at i onal change i n t he vi ral genome. Expos ure t o any of t he agent s i s predi ct abl y fol l owed by res i s t ance wi t hi n weeks t o mont hs unl es s vi ral repl i cat i on i s s ucces s ful l y s uppres s ed. Once res i s t ance occurs , i t i s very s t abl e and does not di s appear even wi t h drug ces s at i on. Res i s t ance t o vari ous ant i ret rovi ral drugs can be meas ured by genot ypi c anal ys i s (l ooki ng for known mut at i ons t hat confer s peci fi c res i s t ance) or phenot ypi c anal ys i s (aki n t o t he t es t s t o l ook for bact eri al drug res i s t ance). Bot h of t hes e met hods are expens i ve and s omewhat i nconcl us i ve, al t hough t hey can be hel pful i n gui di ng t herapy.
(5) Drug–drug interactions are common and pot ent i al l y s evere. Some ant i ret rovi ral s s uch as t he prot eas e i nhi bi t ors can have powerful effect s on hepat i c cyt ochrome enzymes . P.370
Thi s may res ul t i n unexpect edl y hi gh or l ow l evel s of ot her drugs t hat depend on l i ver met abol i s m.
(6) Direct drug toxicity i s common, al t hough us ual l y manageabl e. Aki n t o cancer chemot herapy, di fferent agent s t end t o have di fferent s i de effect s and t oxi ci t i es . Therefore
Pa g e 1 7 9 5
ABC Ambe r CHM Conve rte r Tria l ve rsion, http://w w w .proce sste x t.com/a bcchm.html
when as s embl i ng a combi nat i on of HIV-1 medi cat i ons one needs t o t ake i nt o account vari ous t oxi ci t i es . Some of t he mos t s evere t oxi ci t i es i ncl ude pancreat i t i s , peri pheral neuropat hy, s evere hypers ens i t i vi t y react i ons , l act i c aci dos i s , and dys l i pi demi a.
(7) Drug costs are hi gh, and t he cos t of moni t ori ng i s al s o cons i derabl e. W hereas effect i ve HIV-1 care s aves money i n t he l ong run (as compared wi t h t he cos t of hos pi t al i zat i ons , decreas ed product i vi t y, and earl y mort al i t y), i t can be di ffi cul t t o afford good HIV-1 care.
o
o
b. HIV-2. Treat ment for HIV-2 i nfect i on t ends t o mi rror t hat of HIV-1 and i s much l es s ext ens i vel y s t udi ed.
E. Prognosis The prognos i s of HIV-1 i nfect i on has changed radi cal l y wi t h t he awarenes s t hat s uppres s i ng vi ral repl i cat i on can al l ow for i mmune recons t i t ut i on. Before t he wi des pread us e of combi nat i on ant i vi ral t herapy (CART), al s o referred t o as hi ghl y act i ve ant i ret rovi ral t herapy (HAART), i ndi vi dual s who cont ract ed HIV-1 i nfect i on woul d s urvi ve on average 5–10 years wi t hout cl i ni cal s ympt oms (l at ent peri od). They woul d t hen experi ence a decl i ne i n i mmunocompet ence, l eadi ng al mos t i nexorabl y t o deat h. Now, wi t h careful medi cal management , HIV-1 can be cont rol l ed i n many i ndi vi dual s , al t hough t he durat i on of t hi s res pons e i s not yet known. Thi s has cons i derabl y i nfl uenced t he mort al i t y rat es of AIDS. Even for t hos e pat i ent s who are unwi l l i ng or unabl e t o fol l ow t he compl ex drug regi mens needed t o res t ore i mmune funct i on, i nt ermi t t ent ant i vi ral t herapy may provi de s ome benefi t , al t hough t he l ong-t erm prognos i s may not be good. W i t h t he many new
Pa g e 1 7 9 6
ABC Ambe r CHM Conve rte r Tria l ve rsion, http://w w w .proce sste x t.com/a bcchm.html
t reat ment modal i t i es i n devel opment for HIV-1–i nfect ed i ndi vi dual s , furt her prognos t i c changes may occur i n t he fores eeabl e fut ure.
F. Prevention
1. Screening. Recent l y, t he Cent ers for Di s eas e Cont rol and Prevent i on (CDC) has recommended t hat HI V s c reeni ng i s rec ommended for pat i ent s i n al l heal t h-c are s et t i ngs aft er t he pat i ent i s not i fi ed t hat t es t i ng w i l l be performed unl es s t he pat i ent dec l i nes (opt -out s c reeni ng). (The i t al i cs are di rect l y from t he CDC document : Morb Mort al W kl y Rep MMW R Sept ember 22, 2006; Vol . 55, No. RR-14). Previ ous l y, due t o concerns about s t i gmat i zat i on and di s cri mi nat i on, HIV t es t i ng has been res erved on an “opt -i n― bas i s , requi ri ng expl i ci t wri t t en cons ent , and ext ens i ve pret es t and pos t t es t couns el i ng. Thes e requi rement s are l oos ened i n t he current gui del i nes , wi t h hopes t hat more peopl e wi l l be encouraged t o know t hei r HIV s t at us and us e appropri at e prot ect i ve meas ures t o prevent t rans mi s s i on. HIV-1 s creeni ng t es t s are s ens i t i ve, s peci fi c, and wi del y avai l abl e for HIV-1 i nfect i on, but t hey may not det ect t he ant i body res pons e t o HIV-2. Home t es t i ng and poi nt -of-us e t es t i ng for HIV-1 are avai l abl e and accurat e, but s t andard t es t s s houl d al ways be done t o corroborat e pos i t i ve res ul t s .
2. Controlling spread of infection. Prevent i ve meas ures for cont rol l i ng t he s pread of HIV i ncl ude education about ri s k fact ors , sexual abstinence, us e of barrier precautions (condoms ), avoidance of childbirth i n women known t o be HIV-1 i nfect ed, needle-exchange programs, and universal testing of blood products.
Pa g e 1 7 9 7
ABC Ambe r CHM Conve rte r Tria l ve rsion, http://w w w .proce sste x t.com/a bcchm.html
Treat ment of HIV-1–i nfect ed pregnant women wi t h ant i ret rovi ral s s ubs t ant i al l y decreas es prenat al and peri part um t rans mi s s i on of HIV, as does del i very by ces arean s ect i on. Breas t feedi ng s houl d be avoi ded by women i nfect ed wi t h HIV-1. Suppres s i on of HIV-1 i n t he bl ood i s accompani ed by reduct i on of vi ral l oads on mucos al s urfaces s o i t s houl d al s o decreas e i nfect i vi t y. Thi s i n no way s houl d be cons t rued as a reas on t o reduce caut i ous nes s wi t h regard t o s afer s ex pract i ces and needl e s hari ng.
3. Public health. HIV-1 i nfect i on mus t be report ed t o t he s t at e or l ocal Depart ment of Heal t h, whi ch t hen report s t ot al numbers t o t he CDC. Pol i ci es vary wi del y from s t at e t o s t at e, and, t herefore, you mus t fami l i ari ze yours el f wi t h t he l ocal regul at i ons . Some s t at es report us i ng a name-bas ed procedure, whi l e ot hers us e anonymous or coded report i ng. Part ner not i fi cat i on requi rement s vary cons i derabl y among l egal juri s di ct i ons .
G. Complications Effect s of HIV-1 i nfect i on can res ul t from di rect damage caus ed by t he vi rus or from t he opport uni s t i c i nfect i ons or mal i gnanci es t hat accompany t he decl i ne of t he i mmune P.371 s ys t em i n l at e-s t age HIV-1 i nfect i on. In general , di rect effect s of HIV-1 i ncl ude fevers , ni ght s weat s , wei ght l os s , decreas ed l i bi do, cogni t i ve defi ci t s , and mus cl e was t i ng (Fi gure 8-12).
Pa g e 1 7 9 8
ABC Ambe r CHM Conve rte r Tria l ve rsion, http://w w w .proce sste x t.com/a bcchm.html
FIGURE 8-12 Boxpl ot of t he medi an (l i ne i ns i de t he box), fi rs t quart i l e (t op of box), and mean (as t eri s k). CD4+ l ymphocyt e count at t he t i me of t he devel opment of opport uni s t i c di s eas e. Can, C andi da es ophagi t i s ; CMV, cyt omegal ovi rus ; Crp, crypt os pori di os i s ; Cry, crypt ococcal meni ngi t i s ; DEM, acqui red i mmunodefi ci ency vi rus dement i a compl ex; HSV, herpes s i mpl ex vi rus i nfect i on; HZos , herpes z os t er; KS, Kapos i s arcoma; MAC, Myc obac t eri um avi um compl ex; NH, non-Hodgki n l ymphoma; PCP, pri mary pneumoni a; PML, progres s i ve mul t i focal l eukoencephal opat hy; Tox, T oxopl as ma gondi i encephal i t i s ; W S, was t i ng s yndrome. (Us ed wi t h permi s s i on from Moore RD, Chai s s on RE. Nat ural hi s t ory of opport uni s t i c di s eas e i n an HIV-i nfect ed urban cl i ni cal cohort . Ann I nt ern Med 1996;124:633–642. )
1. T ypes of complications o
o
a. Skin lesions
Pa g e 1 7 9 9
ABC Ambe r CHM Conve rte r Tria l ve rsion, http://w w w .proce sste x t.com/a bcchm.html
(1) Some l es i ons are rare i n i ndi vi dual s wi t hout HIV i nfect i on but are rel at i vel y common i n t hos e wi t h HIV-1 i nfect i on.
(a) Kapos i s arcoma, whi ch i s caus ed by a herpes vi rus (HHV-8)
(b) Eos i nophi l i c fol l i cul i t i s
(c) Di s s emi nat ed mol l us cum cont agi os um
(d) Baci l l ary angi omat os i s (Bart onel l a)
(e) Thrus h (Candi da)
(2) The fol l owi ng common di s orders are more s evere i n i ndi vi dual s wi t h HIV-1 i nfect i on.
(a) Ps ori as i s (i ncl udi ng ps ori at i c art hri t i s )
(b) Seborrhei c dermat i t i s
(c) Al opeci a
Pa g e 1 8 0 0
ABC Ambe r CHM Conve rte r Tria l ve rsion, http://w w w .proce sste x t.com/a bcchm.html
(d) Onychomycos i s (fungal i nfect i ons of t he nai l beds )
(e) Severe or recurrent geni t al candi dal i nfect i on
o
o
b. Lymphatic system. Lymph nodes are oft en enl arged, al t hough us ual l y t hi s i s not refl ect i ve of a s peci fi c i nfect i on or t umor. The enl arged nodes are charact eri zed on bi ops y as havi ng react i ve hyperpl as i a.
(1) Lymphomas oft en occur as ext ranodal di s eas e but may caus e nodal enl argement l at er.
(2) Several infectious agents can caus e l ymph node enl argement , i ncl udi ng T . pal l i dum, M. t uberc ul os i s , and Hi s t opl as ma c aps ul at um. Node as pi rat e or bi ops y i s us eful i n di s t i ngui s hi ng among t hes e organi s ms .
(3) Kaposi' s sarcoma may be found i n l ymph nodes .
P.372
o
o
c. Nervous system. Such i nvol vement may be di ffus e or focal . Some cogni t i ve or mot or dys funct i on i s common and t ends t o become more
Pa g e 1 8 0 1
ABC Ambe r CHM Conve rte r Tria l ve rsion, http://w w w .proce sste x t.com/a bcchm.html
s evere. Us ual l y t here i s evi dence of cerebral at rophy, and no opport uni s t i c agent can be found t o expl ai n t he dement i a.
(1) Meningitis i s caus ed mos t frequent l y by Crypt oc oc c us neoformans , a ubi qui t ous yeas t . Headache and neck s t i ffnes s range from mi l d or t rans i ent t o s evere, and t he CSF i s oft en normal or near normal wi t h regard t o cel l count , prot ei n l evel , gl ucos e concent rat i on, and general appearance. Crypt ococci are s een on Indi a i nk preparat i ons , and crypt ococcal ant i gen can be demons t rat ed i n s erum and CSF. Crypt oc oc c us i s eas i l y cul t ured from CSF and l es s commonl y from bl ood.
(a) As ept i c meni ngi t i s wi t h headache, nuchal ri gi di t y, and l ymphocyt i c pl eocyt os i s oft en occurs i n t he earl y s t age of HIV-1 i nfect i on and occas i onal l y i n l at er s t ages .
(b) Ot her forms of meni ngi t i s , al t hough rel at i vel y uncommon, occur more oft en t han i n t he general popul at i on.
(c) Cryptococcal disease can be prevent ed or del ayed by t he us e of fluconazole i n pat i ent s wi t h l at e-s t age HIV i nfect i on. Al t hough fl uconazol e i s s afe and prevent s or del ays t he occurrence of t hrus h, cont rol l ed s t udi es s how no s urvi val benefi t t o t hi s regi men
Pa g e 1 8 0 2
ABC Ambe r CHM Conve rte r Tria l ve rsion, http://w w w .proce sste x t.com/a bcchm.html
as compared wi t h t reat i ng fungal i nfect i ons as t hey occur. Pat i ent s wi t h crypt ococcal meni ngi t i s s houl d cont i nue t o recei ve ant i fungal t herapy unt i l t hei r 3
CD4 count s exceed 200/mm .
(2) Neuropathy may be a part of HIV-1 i nfect i on, an opport uni s t i c i nfect i on, or a s i de effect of t reat ment . For exampl e, vi ncri s t i ne, whi ch may be us ed for s ome AIDS-rel at ed mal i gnanci es , i nduces dos e-rel at ed peri pheral neuropat hy, as do cert ai n NRTIs s uch as zal ci t abi ne (ddC) and di danos i ne (ddI).
(3) Space-occupying lesions of the CNS are found wi t h s ome frequency i n HIV-1 i nfect i on. The pres ent at i on i s us ual l y one of focal neurol ogi c abnormal i t y (oft en accompani ed by s ei zures ) and an abnormal CT or MRI s can.
(a) T . gondi i , a prot ozoan paras i t e found worl dwi de, i s t he mos t common caus e of brain lesion i n pat i ent s who have AIDS. Many prophyl act i c regi mens for Pneumocystis carinii pneumonia (PCP) al s o can prevent cl i ni cal t oxopl as mos i s from occurri ng. However, i t i s not us ual l y advi s abl e t o add ant i -T oxopl as ma medi cat i ons t o t he prevent i ve regi men of an i ndi vi dual who i s i nt ol erant t o t he ki nd of PCP prophyl axi s t hat al s o prevent s t oxopl as mos i s .
Pa g e 1 8 0 3
ABC Ambe r CHM Conve rte r Tria l ve rsion, http://w w w .proce sste x t.com/a bcchm.html
(b) Lymphomas l ocat ed i n deep brai n s t ruct ures and progressive multifocal leukoencephalopathy (caus ed by t he JC vi rus ) are al s o di s proport i onat el y common.
(c) Syphilis can recrudes ce i n HIV-i nfect ed pers ons . Standard treatments are us ual l y effect i ve, but careful fol l ow-up i s i ndi cat ed. W hen t here i s a ques t i on of whet her a cure has been achi eved, a more i nt ens i ve regi men (e.g., t hat for t he t reat ment of neuros yphi l i s ) i s recommended.
o
o
d. Eye. HIV-1 may be as s oci at ed wi t h s ome di rect t oxi ci t y t o t he eye and opt i c nerve, but CMV retinitis i s t he mos t common and s eri ous ocul ar compl i cat i on of AIDS. The l es i ons are predomi nant l y ret i nal and s pare t he choroi d. They us ual l y progres s over weeks and can l ead t o bl i ndnes s . Invol vement of bot h eyes i s t he rul e, and CMV ret i ni t i s i s oft en part of a s ys t emi c i l l nes s i ncl udi ng predomi nant l y t he gas t roi nt es t i nal t ract . Seri ous CMV ocul ar di s eas e occurs i n l at e-s t age HIV i nfect i on when t he CD4 count i s very depres s ed. Treat ment of HIV t hat res ul t s i n i ncreas ed CD4 count s oft en s t ops progres s i on of di s eas e.
o
o
e. Upper alimentary tract
Pa g e 1 8 0 4
ABC Ambe r CHM Conve rte r Tria l ve rsion, http://w w w .proce sste x t.com/a bcchm.html
(1) The upper gas t roi nt es t i nal t ract i s mos t frequent l y affect ed by local candidal or herpes simplex infections, whi ch can ext end i nt o t he es ophagus , or by Kaposi' s sarcoma, whi ch can be found t hroughout t he i nt es t i ne. Endos copi c es ophageal brus hes or bi ops y can di s t i ngui s h bet ween t he t wo mos t common opport uni s t s .
(2) Vi s ual exami nat i on of t he oral cavi t y i s s uffi ci ent for i dent i fyi ng oral hairy leukoplakia, an Eps t ei n-Barr vi rus –rel at ed i nfect i on t hat predomi nant l y i nvol ves t he l at eral t ongue but may ext end t o ot her part s of t he oral cavi t y.
(3) Aphthous ulcers of t he mout h and es ophagus are common, and es ophageal ul cers can be l arge and pai nful .
P.373
o
o
f. Liver
(1) In t he l at e s t age of HIV-1 i nfect i on, t he l i ver oft en i s t he s i t e of opportunistic infections s uch as hi s t opl as mos i s , mycobact eri os i s [due t o M. t uberc ul os i s and Myc obac t eri um avi um-i nt rac el l ul are (MAI)], crypt ococcos i s , and CMV. Hepat i c candi di as i s , whi ch i s s omet i mes s een i n l eukemi a as part
Pa g e 1 8 0 5
ABC Ambe r CHM Conve rte r Tria l ve rsion, http://w w w .proce sste x t.com/a bcchm.html
of s ys t emi c fungal di s eas e, i s not s een i n AIDS.
(2) The s ame popul at i ons t hat are at ri s k for HIV-1 i nfect i on are al s o at ri s k for bl ood-borne hepat i t i s : HBV; HCV; HDV. HBV vaccine i s recommended for uni vers al us e. Becaus e t he vacci ne i s s afe and t he rout e of t rans mi s s i on of HBV i s s i mi l ar t o t hat of HIV, vacci nat i on of HIV-i nfect ed pers ons i s l ogi cal . Pel i os i s hepat i s i s a t reat abl e l i ver i nfect i on caus ed by B. hens el ae, t he agent of cat -s crat ch di s eas e.
o
o
g. Gastrointestinal tract distal to the esophagus
(1) HIV-1 can caus e an enteropathy di s t i nct from t he i nfect i ous di arrheas . Becaus e many meat s and ani mal product s s uch as mi l k and eggs can be cont ami nat ed wi t h bact eri a t hat caus e ent eri c i nfect i on, HIV-i nfect ed pers ons s houl d be advi s ed t o cons ume onl y pas t euri zed mi l k and t o eat wel l -cooked eggs and meat s .
(2) In addi t i on, Cryptosporidium s peci es , Microsporidium, Cyclospora, Isospora, and G. intestinalis are pot ent i al l y caus es of watery diarrhea and diffuse abdominal pain i n HIV-1–i nfect ed pat i ent s . The di s covery of Crypt os pori di um i n muni ci pal wat er s uppl i es i s worri s ome for pat i ent s wi t h advanced HIV i nfect i on. It i s not cl ear whet her t he us e of
Pa g e 1 8 0 6
ABC Ambe r CHM Conve rte r Tria l ve rsion, http://w w w .proce sste x t.com/a bcchm.html
wat er fi l t ers and bot t l ed wat er can s ubs t ant i al l y reduce t he ri s k of t hi s i nfect i on.
(3) The s t omach and s mal l bowel al s o may be s i t es of ori gi n of extranodal lymphoma.
(4) CMV colitis can be t he caus e of s evere abdomi nal pai n, di arrhea, and fever, and i t occas i onal l y can l ead t o perforat i on or megacol on.
o
o
h. Lungs. In AIDS, t he l ungs are t he mos t common t arget organ for s ympt omat i c di s eas e.
(1) Pneumocystis carinii pneumonia (PCP) devel ops i n mos t pat i ent s who do not recei ve prophyl axi s and whos e CD4 cel l count drops 3
bel ow 200/mm . In HIV-1–i nfect ed i ndi vi dual s , PCP has a s ubacut e pres ent at i on t hat del ays di agnos i s , oft en for weeks . By t he t i me pat i ent s come t o medi cal at t ent i on, t hey have fever, dry cough, and hypoxemi a. Ches t radi ographs may be normal or have i nt ers t i t i al marki ngs or fl uffy i nfi l t rat es . Us ual l y, di agnos i s i s made by exami ni ng s put um, BAL fl ui d, bronchi al was hi ngs or brus hi ngs , or l ung bi ops y s peci mens .
(a) Even aft er i ni t i at i on of t reat ment , PCP may progres s t emporari l y but ul t i mat el y yi el ds t o effect i ve t herapy i n 90% of pat i ent s . Short cours es of
Pa g e 1 8 0 7
ABC Ambe r CHM Conve rte r Tria l ve rsion, http://w w w .proce sste x t.com/a bcchm.html
s t eroi ds are oft en gi ven t o abort an earl y wors eni ng wi t h t he i ni t i at i on of t herapy.
(b) Every i ndi vi dual who has al ready had PCP s houl d recei ve s econdary prophyl axi s . Becaus e PCP i s a common compl i cat i on of mi d- t o l at e-s t age HIV i nfect i on, and becaus e prevent i ve t herapy i s s afe and effect i ve, i t has become general pract i ce t o offer PCP prevent i on t o every HIV-i nfect ed pers on when t he CD4 count fal l s t o bel ow 300 3
cel l s /mm . Fi rs t -l i ne prophyl axi s for PCP i s t ri met hopri m–s ul famet hoxaz ol e. For pat i ent s i nt ol erant of t hi s medi cat i on, daps one, cl i ndamyci n/pri maqui ne, or at ovaquone s houl d be cons i dered. Therapy i s l i fel ong unl es s i mmune funct i on i s i mproved by HIV-1 s uppres s i on.
(2) Bacterial pneumonias al s o occur more commonl y i n HIV-1–i nfect ed pat i ent s t han i n ot herwi s e heal t hy i ndi vi dual s . Al t hough t hes e pneumoni as may be more s evere t han i n i mmunocompet ent pat i ent s , t hey us ual l y res pond t o rout i ne ant i mi crobi al t herapy. Pneumococcus i s t he mos t common caus e of bact eri al pneumoni a i n adul t HIV-1–i nfect ed pat i ent s . Bact eri al pneumoni a can occur s i mul t aneous l y wi t h PCP. To prevent bact eri al pneumoni a i n HIV, earl y admi ni s t rat i on of pneumococcal vaccine i s recommended. A s i ngl e dos e s houl d be adequat e for a l i fet i me.
Pa g e 1 8 0 8
ABC Ambe r CHM Conve rte r Tria l ve rsion, http://w w w .proce sste x t.com/a bcchm.html
(3) Fungal pneumonia i s l es s common and i s us ual l y a part of s ys t emi c crypt ococcal , Coc c i di oi des , or Hi s t opl as ma i nfect i on. Awarenes s of expos ure t o t he agent s of endemi c mycos es (e.g., Coc c i di oi des or Hi s t opl as ma) i s cruci al t o make t he di agnos i s and t o i ni t i at e appropri at e t herapy.
(4) Viral pneumonia can be s evere i n HIV-1–i nfect ed pat i ent s . CMV, a herpes vi rus known t o caus e s evere l ung di s eas e i n ot her pat i ent s wi t h Th-cel l defi ci enci es (es peci al l y bone P.374
marrow t rans pl ant reci pi ent s ), may caus e a fat al pneumoni a al one or wi t h PCP i n HIV-1–i nfect ed pat i ent s .
(5) Mycobacterial disease i s common.
(a) Infect i on wi t h M. tuberculosis may appear rel at i vel y earl y i n HIV-1 i nfect i on. Al t hough i t has a hi gher propens i t y t o di s s emi nat e i n pat i ent s who have Th-cel l (CD4) depl et i on, i t us ual l y res ponds t o ant i mycobact eri al t herapy. Mul t i pl y res i s t ant (t o i s oni azi d and ri fampi n) M. t uberc ul os i s i s s t i l l uncommon, but t hes e s t rai ns are di ffi cul t t o t reat . In i mmunol ogi cal l y compet ent i ndi vi dual s ,
Pa g e 1 8 0 9
ABC Ambe r CHM Conve rte r Tria l ve rsion, http://w w w .proce sste x t.com/a bcchm.html
t he cl i ni cal res pons e may be onl y 60%, whereas i t i s cons i derabl y l ower i n HIV-i nfect ed pat i ent s . Prevent i on of t ubercul os i s i nvol ves t he fol l owi ng:
(i) Screening. Earl y s ki n t es t i ng i s mos t us eful for i dent i fyi ng i ndi vi dual s at ri s k for recrudes cence of previ ous l y acqui red i nfect i on, becaus e many pat i ent s become anergi c i n t he l at er s t ages of HIV i nfect i on. Onl y 5 mm of i ndurat i on i s needed t o qual i fy as a pos i t i ve s ki n t es t i n a pers on wi t h HIV i nfect i on. Aggres s i ve cont act t raci ng of pers ons who are known t o be i nfect i ous wi t h t ubercul os i s al s o can i dent i fy i ndi vi dual s at ri s k before t hey become i l l wi t h recent l y acqui red t ubercul os i s .
(ii) Prophylactic isoniazid reduces t he ri s k of recurrent t ubercul os i s when admi ni s t ered for 6–12 mont hs t o pat i ent s who have had pos i t i ve PPD s ki n t es t s .
(b) Nontuberculous mycobacteria, us ual l y MAI, may be found in s put um. In HIV-1–i nfect ed pat i ent s , t he l ungs are a fai rl y mi nor t arget organ for t hi s pat hogen, whi ch us ual l y i nfect s t he l i ver, s pl een, bl ood, and bone marrow.
Pa g e 1 8 1 0
ABC Ambe r CHM Conve rte r Tria l ve rsion, http://w w w .proce sste x t.com/a bcchm.html
Macrolides or rifabutin can be gi ven when t he CD4 count drops t o l es s t han 3
75 cel l s /mm .
(6) A noninfectious pneumonitis charact eri zed by l ymphocyt i c i nfi l t rat i on of t he l ungs has been s een frequent l y i n chi l dren wi t h HIV-1 i nfect i on and i s bei ng recogni zed more commonl y i n adul t s .
o
o
i. Cardiovascular system. Cardiomyopathy may be found i n HIV-1–i nfect ed pat i ent s .
o
o
j. Musculoskeletal system. Thi s s ys t em i s frequent l y i nvol ved i n al l s t ages of HIV-1 i nfect i on. Mani fes t at i ons are s el dom l i fe-t hreat eni ng but do caus e s ubs t ant i al morbi di t y. Except for AIDS-as s oci at ed art hri t i s , mos t of t hes e mani fes t at i ons are cl i ni cal l y s i mi l ar t o ent i t i es i n i ndi vi dual s wi t hout HIV-1 i nfect i on. However, t he i nci dence and s everi t y of t hes e mani fes t at i ons are great er i n HIV-1–i nfect ed pat i ent s and t end t o be wors e i n t he l at e s t age of HIV-1 i nfect i on.
(1) Articular manifestations
(a) Arthralgias occur i n up t o one t hi rd of HIV-1–i nfect ed i ndi vi dual s . The pai n i s i nt ermi t t ent and us ual l y affect s t he l arge joi nt s .
Pa g e 1 8 1 1
ABC Ambe r CHM Conve rte r Tria l ve rsion, http://w w w .proce sste x t.com/a bcchm.html
(b) Reiter' s syndrome has been report ed t o affect approxi mat el y 5% of HIV-1–i nfect ed homos exual men, al t hough i t s eems t o be rarer i n i ndi vi dual s who acqui re HIV-1 t hrough needl e s hari ng or het eros exual act i vi t y.
(c) In pat i ent s wi t h ps ori as i s , HIV-1 i nfect i on i ncreas es t he ri s k of devel opi ng psoriatic arthritis.
(d) AIDS-associated arthritis i s s evere and debi l i t at i ng. It affect s t he l arge joi nt s and produces onl y a mi l d s ynovi t i s . Int ra-art i cul ar s t eroi ds may gi ve cons i derabl e rel i ef.
(2) Muscular diseases
(a) Myalgias are common i n HIV-1 i nfect i on, es peci al l y i n t he earl y mononucl eos i s -l i ke i l l nes s as s oci at ed wi t h s eroconvers i on.
(b) Polymyositis wi t h proxi mal mus cl e weaknes s and el evat ed s erum l evel s of mus cl e enzymes [e.g., creat i ne ki nas e (CK)] occurs i n approxi mat el y 2% of HIV-1–i nfect ed i ndi vi dual s . It al s o may occur as a s i de effect of zi dovudi ne t herapy.
Pa g e 1 8 1 2
ABC Ambe r CHM Conve rte r Tria l ve rsion, http://w w w .proce sste x t.com/a bcchm.html
(3) Bone and joint infections are fai rl y uncommon i n HIV-1 i nfect i on, except among i nject i on drug us ers . The caus at i ve agent s are a mi xt ure of common organi s ms (s ee V H) and opport uni s t i c agent s s uch as mycobact eri a and fungi .
(4) Vari ous forms of vasculitis and connective tissue disease (e.g., Sjögren's s yndrome) s eem t o occur wi t h hi gher-t han-expect ed frequency.
o
o
k. Hematopoietic abnormalities. Thes e condi t i ons are common i n HIV-1 i nfect i on. P.375
(1) Immune thrombocytopenia may occur i n mi d- t o l at e-s t age HIV-1 i nfect i on and res embl es i di opat hi c t hrombocyt openi c purpura (ITP) or t hrombot i c t hrombocyt openi c purpura (TTP).
(2) Anemia i s common i n l at e HIV-1 i nfect i on. The pat t ern i s us ual l y one of chroni c di s eas e wi t h normochromi c, normocyt i c i ndi ces . Some ant i ret rovi ral t herapi es and ant i met abol i t es us ed t o prevent i nfect i ons may i nduce macrocyt i c anemi a. Serum l evel s of vi t ami n B 1 2 are oft en l ow, but vi t ami n B 1 2
Pa g e 1 8 1 3
ABC Ambe r CHM Conve rte r Tria l ve rsion, http://w w w .proce sste x t.com/a bcchm.html
t herapy does not i mprove hemat ol ogi c paramet ers . Many pat i ent s wi t h l ow l evel s of eryt hropoi et i n res pond t o eryt hropoi et i n repl acement t herapy.
(3) Neutropenia, al ong wi t h t he expect ed depl et i on of l ymphocyt es , may accompany HIV-1 i nfect i on. Thi s may be a s i gn of di s s emi nat ed bact eri al or opport uni s t i c i nfect i on, and bl ood and bone marrow cul t ure (i ncl udi ng mycobact eri al cul t ures ) can be hel pful . Ant i ret rovi ral and ant i met abol i t e t herapi es may cont ri but e t o neut ropeni a.
(4) Immunoglobulin disorders are common. The mos t frequent l y des cri bed i s a pol ycl onal i ncreas e i n gamma gl obul i n wi t h an i nabi l i t y t o produce novel i mmunogl obul i ns . Thus , pat i ent s wi t h advanced HIV-1 i nfect i on may res pond poorl y t o vacci nat i on and may not demons t rat e good s erol ogi c res pons es t o acut e i nfect i ons , es peci al l y when chal l enged duri ng t he l at e s t ages of HIV i nfect i on.
o
o
l. Endocrine system. A vari et y of endocri ne abnormal i t i es , i ncl udi ng t hyroi d and adrenal i ns uffi ci enci es , have been report ed i n HIV-1 i nfect i on.
o
o
m. Reproductive system
(1) Carci noma of t he ut eri ne cervi x and
Pa g e 1 8 1 4
ABC Ambe r CHM Conve rte r Tria l ve rsion, http://w w w .proce sste x t.com/a bcchm.html
dys pl as t i c changes occur more commonl y i n HIV-1–i nfect ed women.
(2) Fungal (candi dal ) vul vovagi ni t i s i s more frequent and may be caus ed by ant i fungal drug-res i s t ant s t rai ns .
(3) Herpes s i mpl ex i nfect i ons can become very s evere wi t h t he advanced i mmune depres s i on as s oci at ed wi t h HIV-1. Pat i ent s wi t h herpes geni t al i nfect i ons can have l arge, confl uent pai nful areas of i nvol vement .
2. T reatment of infections associated with HIV infection. (Tabl e 8-6) Therapeut i c regi mens may be t he s ame or more i nt ens i ve t han t hos e for t he s ame i nfect i on i n i mmunocompet ent i ndi vi dual s or pat i ent s wi t h ot her caus es of i mmune depres s i on. Many of t he i nfect i ons as s oci at ed wi t h HIV are pot ent i al l y chroni c; l i fel ong s econdary prophyl axi s and cl os e vi gi l ance are neces s ary t o cont rol recurrences . Recommendat i ons concerni ng t he t reat ment of t hes e condi t i ons are revi s ed cons t ant l y, and current i nformat i on s houl d al ways be s ought .
3. Prevention of complications o
o
a. Immune reconstitution. Prophyl axi s can be s t opped when i mmune funct i on i s res t ored. Some i ndi vi dual s experi ence t rans i ent wors eni ng of i nfect i ons as i mmune funct i on i mproves . Thi s can be a fai rl y s evere react i on t hat mi ght requi re
Pa g e 1 8 1 5
ABC Ambe r CHM Conve rte r Tria l ve rsion, http://w w w .proce sste x t.com/a bcchm.html
rei ns t i t ut i on of t herapy for t he previ ous l y qui es cent i nfect i on. Mos t of t he t i me i t i s mos t hel pful t o cont i nue ant i -ret rovi ral t herapy des pi t e t he cl i ni cal wors eni ng becaus e i mmune recons t i t ut i on event ual l y goes away. o
o
b. In general , prophyl axi s agai ns t t he common HIV-1–as s oci at ed i nfect i ons can be s t opped when t he CD4 count pas s es t he l evel at whi ch t he ri s k of t hat i nfect i on begi ns t o i ncreas e s i gni fi cant l y. Us ual l y t hi s i s done aft er at l eas t t wo CD4 meas urement s have been hi gh enough and when t he pat i ent i s cl i ni cal l y s t abl e. If t he CD4 drops , i t may be neces s ary t o res t art prophyl axi s .
P.376
P.377
TABLE 8-6 Treatment and Prevention of Infections Associated with HIV Infection
Pa g e 1 8 1 6
ABC Ambe r CHM Conve rte r Tria l ve rsion, http://w w w .proce sste x t.com/a bcchm.html
P r e v e nt Cl
io
in
n
ic
/
al
P
M
r
O a T o C r ni r p o g fe e h m a st at yl m ni at m a e s io e xi nt m n nt s s Bacterial St P A P Hi re n p n g pt e pr e h o u o u er c m pr m ra o o ia o te c c ni t e v t h u a a a a s
nt x n
p
ib
H
n
io
IV
e
ti
u
c
m o ni
Pa g e 1 8 1 7
ABC Ambe r CHM Conve rte r Tria l ve rsion, http://w w w .proce sste x t.com/a bcchm.html
a e M P St P O yc ul a P cc o m n D ur b o d a s a n ar n at c t ar d n hi er y, t h u g i u e er al h m xt a l y er t u ra p
ra
b p y
te
er ul
s
c m
th
ul o
a
o n
n
s i ar
in
s y
H
(C
IV
N
-
S,
e
G
v
I,
e
p
n
er
wi
it
th
o
C
ni
D
ti
4
s,
co
P
u
ot
nt
t'
>
s
5
Pa g e 1 8 1 8
ABC Ambe r CHM Conve rte r Tria l ve rsion, http://w w w .proce sste x t.com/a bcchm.html
di
0
s
0
e a s e, et c. ) M C Cl A C yc ol ar zi D o iti it th 4 b s , hr ro < a b o m5 c t ac m yc 0 er t e yc i n i u re i n 1 mm+ 2 a i a et 0 vi , h 0 u b a m mo mg -i n b w nt e ut e ra m ol e c ar
kl
el ro
y
lu w la , re h ; e M p yc at o iti b s
Pa g e 1 8 1 9
ABC Ambe r CHM Conve rte r Tria l ve rsion, http://w w w .proce sste x t.com/a bcchm.html
a ct er iu m a vi u m -c o m pl e x B B Er
B
ar ac yt
A
t o i l l hr
ca
n ar o
n
el y m
b
l a a yc
e
h n in
di
e gi ,
ffi
n o ot
cu
s mh
lt
el at er
to
a o m
di
e, s i ac
st
B. s ro
in
q (B l i
g
ui A) d
ui
nt B e
s
a ac s
h
n ill
fr
a ar
o
Pa g e 1 8 2 0
ABC Ambe r CHM Conve rte r Tria l ve rsion, http://w w w .proce sste x t.com/a bcchm.html
y
m
p
K
el
S
io si s h e p at iti s Viral H H A V Hi e e d ac g p p ef ci h at at o n er i t i i t i vi at i n s s , r, i o ci B ri l a n d vi s k m
e
ru of i v
nc
s ci u
e
rr di
in
h n
M
o e,
S
si te
M
s/ n h of e o p vi at r oc el lu
Pa g e 1 8 2 1
ABC Ambe r CHM Conve rte r Tria l ve rsion, http://w w w .proce sste x t.com/a bcchm.html
la r ca rc in o m a H H In A Hi e e te v g p p rf oi h at at er d er iti iti o s in s s , n- h ci C ri Î ar d vi s k ± i n e ru of /r g nc s ci i b n e rr a e i n h vi e IV o ri dl D si n e U s/
s
h e p at oc el lu la r ca rc in
Pa g e 1 8 2 2
ABC Ambe r CHM Conve rte r Tria l ve rsion, http://w w w .proce sste x t.com/a bcchm.html
o m a ( > H B V) H A S V H e cu u ac A p t e p ci V at s p n ca i t i el or at n s f- t i i o ca A li v n u vi m e
s
ru i t
e
s e
a
d
g
h
gr
e
a
p
v
at
at
iti
io
s,
n
n
of
o
ch
lo
ro
n
ni
g-
c
te
H
r
B
m
V
s
or
e
H
Pa g e 1 8 2 3
ABC Ambe r CHM Conve rte r Tria l ve rsion, http://w w w .proce sste x t.com/a bcchm.html
q
C
u
V
el
in
a
fe
e
ct io n
E O N N p ra o o st l
n n
ei h e e n- ai C B ry h ar l e e r u m vi k ot ru o h s pl er (E a a B ki p V) a y Bce ll ly m p h o m a s C R G N C yt et a o h o i n nc n ec
Pa g e 1 8 2 4
ABC Ambe r CHM Conve rte r Tria l ve rsion, http://w w w .proce sste x t.com/a bcchm.html
m iti ic e k e s lo
C
g C vi
M
al ol r
V
o iti
Ig
vi s
G
ru
to
s
a
(C
ss
M
e
V)
ss ri sk of re ac ti v at io n; a n n u al e y e e x a m s; C
Pa g e 1 8 2 5
ABC Ambe r CHM Conve rte r Tria l ve rsion, http://w w w .proce sste x t.com/a bcchm.html
D 4 co u nt u s u al ly lo w ( < 5 0) JC Pr S N Di vi o u o a ru gr p n g s e p e n s s or
o
iv ti
si
e v
s
m e;
ca
ul m
n
t i ai
b
fo n
e
ca t r
m
l
a
e
l e at
d
u m
e
k e
b
o nt
y
e is
M
nc H
RI
Pa g e 1 8 2 6
ABC Ambe r CHM Conve rte r Tria l ve rsion, http://w w w .proce sste x t.com/a bcchm.html
e A
,
p A
P
h R
C
al T
R
o to
fo
p re
r
at s t
JC
h or
in
y e
C
(P i
S
M m
F,
L) m
br
u
ai
n
n
oc
bi
o
o
m
p
p
sy
et e nc e H K I N M H a m o or V- p pr n e 8 o o e co si v
m
's e
m
s d
o
ar i
n
co m
in
mm
M
a u
S
(c n
M
ut e
Pa g e 1 8 2 7
ABC Ambe r CHM Conve rte r Tria l ve rsion, http://w w w .proce sste x t.com/a bcchm.html
a st n at e u o s u wi s th or H vi A sc A er R al T; ) D o x or u bi ci n ca n b e u s e d H G St N O S e a o ut V ni n n br ta d e e l
ar
a
ul d
ks
ce t r
ca
rs e
n
Pa g e 1 8 2 8
ABC Ambe r CHM Conve rte r Tria l ve rsion, http://w w w .proce sste x t.com/a bcchm.html
at
b
m
e
e
e
nt
xt e n si v e a n d di ffi cu lt to tr e at
V H A N Z er cy o V p cl n e o e s vi z r o st er , s hi n gl e
Pa g e 1 8 2 9
ABC Ambe r CHM Conve rte r Tria l ve rsion, http://w w w .proce sste x t.com/a bcchm.html
s H G St Fr P e a e V ni n q ta d u l
ar e
w d nt ar t r P ts e A C at P er m s vi e m ca nt e l / E ar R xc s ec i s s t a i o ur l
n v
ca
ei
nc
ll
er
a nc e; â € œ re ct al P A P s m e
Pa g e 1 8 3 0
ABC Ambe r CHM Conve rte r Tria l ve rsion, http://w w w .proce sste x t.com/a bcchm.html
ar â € • Viral H D H IV e A mA e R nt T ia E o si n o p hi li c fo lli cu lit is R e n al di s e a s e
Pa g e 1 8 3 1
ABC Ambe r CHM Conve rte r Tria l ve rsion, http://w w w .proce sste x t.com/a bcchm.html
C ar di o m y o p at h y V a sc ul iti s Ar th ri ti s T h yr oi d/ a dr e n al in s uf
Pa g e 1 8 3 2
ABC Ambe r CHM Conve rte r Tria l ve rsion, http://w w w .proce sste x t.com/a bcchm.html
f Fungal C T Fl M C a hr uc yc o n u o el m di s n e m d h a x o a V z tr n a ol oc fi r gi e h s t n
e in
al
s di
ca
N ca
n
ys t i
di
ta o
di
ti n
a
n of
si
s d
s
wi ec s re h- a a si n n d- g s C w D al 4 lo < w 2 0 0
Cr M A Fl y e m uc pt ni p o o n h n c gi ot a
Pa g e 1 8 3 3
ABC Ambe r CHM Conve rte r Tria l ve rsion, http://w w w .proce sste x t.com/a bcchm.html
o t i er z c c s i c ol u C in e s ut B (f n a Fl or e n uc s of e o ec or o n o mu a n a s z d n n ol ar s o e y d
pr
ul
o
e
p
s
h yl a xi
s) P P B B n n ac ac e e tr tr u u i
i
mmmm o o
fo
c y ni
r
st a
al
is
l
ji r
p
o
at
v
ie
e
nt
ci
s wi
Pa g e 1 8 3 4
ABC Ambe r CHM Conve rte r Tria l ve rsion, http://w w w .proce sste x t.com/a bcchm.html
th C D 4 < 2 0 0 Hi P It N S s t n ra o u o e co n s pl u n e p a ma
ec
s o z
t
m ni ol
in
a a e
p
c
at
a
ie
p
nt
s
s
ul
fr
at
o
u
m
m
O hi o a n d Mi ss is si p pi
Pa g e 1 8 3 5
ABC Ambe r CHM Conve rte r Tria l ve rsion, http://w w w .proce sste x t.com/a bcchm.html
Ri v er V al le ys C P A o n m cc e p id u h i o m ot i d o er e ni i c s a in i
B
m
Fl
m
uc
iti
o
s
n a z ol
e Parasitic T C P B Cl o N yr ac i n x S i
tr ic
o le m i
al
pl s i et m pr a o h
e
s n a
s
ms m
e
a R in
nt
g et e
at
o in +
io
Pa g e 1 8 3 6
ABC Ambe r CHM Conve rte r Tria l ve rsion, http://w w w .proce sste x t.com/a bcchm.html
n iti s
n
di s ul
ca
i
fa
n
di
b
a
e
zi
co
n
nf
e
u s e d wi th C N S ly m p h o m a; ri n ge n h a nc in g le si
Pa g e 1 8 3 7
ABC Ambe r CHM Conve rte r Tria l ve rsion, http://w w w .proce sste x t.com/a bcchm.html
o n s o n C T sc a Cr W P
n O
y at ar
cc
pt er o
ur
o y m
s
s di o
wi
p ar m
th
or rh yc
a
id e in
d
i u a (a
v
m
lt
a
Mi
h
nc
cr
o
e
o
u
d
s
g
di
p
h
s
or
p
e
id
o
a
iu
or
s
m
ef
e;
C
fi
b
yc
ca
e
lo
cy
st
s
)
tr
p
e
or
at
Pa g e 1 8 3 8
ABC Ambe r CHM Conve rte r Tria l ve rsion, http://w w w .proce sste x t.com/a bcchm.html
a
m
Is
e
o
nt
s
st
p
ra
or
te
a
g y is H A A R T to i m pr o v e i m m u n e st at u s
CNS, cent ral nervous s ys t em;
Pa g e 1 8 3 9
ABC Ambe r CHM Conve rte r Tria l ve rsion, http://w w w .proce sste x t.com/a bcchm.html
GI, gas t roi nt es t i nal ; HAART, hi ghl y act i ve ant i ret rovi r al t herapy; IgG, i mmunogl o bul i n G; IVDU, i nt ravenou s drug us ers ; MSM, Men who have s ex wi t h men. P.378
Study Questions/Answers and Explanations 1. A 70-year-old man is admitted to the hospital for an elective hernia repair. T he night before surgery, a nurse reports a rectal temperature of 38.1°C. After a careful examination shows no obvious source of infection, a single blood sample is sent for culture. T he surgery is uneventful, but 3 days later, Corynebacterium is identified on blood culture. Which of the following best explains this finding? A The pat i ent has bact eri al endocardi t i s caus ed by Corynebac t eri um. B W hi l e t aki ng t he rect al t emperat ure, t he nurs e i nadvert ent l y caus ed Corynebac t eri um bact eremi a.
Pa g e 1 8 4 0
ABC Ambe r CHM Conve rte r Tria l ve rsion, http://w w w .proce sste x t.com/a bcchm.html
C Toot h brus hi ng jus t before t he bl ood col l ect i on res ul t ed i n t rans i ent Corynebac t eri um bact eremi a. D Inadequat e s ki n preparat i on or carel es s handl i ng res ul t ed i n cont ami nat i on of t he bl ood cul t ure. E The l aborat ory mi s t ook Es c heri c hi a c ol i for Corynebac t eri um. Vi ew Ans wer 1. T he answer is D [I A; III B]. Inadequat e s ki n preparat i on or carel es s handl i ng res ul t s i n cont ami nat i on of t he bl ood cul t ure. Even t hough al l of t he ans wers are pos s i bl e, t he mos t l i kel y one i s s i mpl e cont ami nat i on; 2%–5% of bl ood cul t ures are cont ami nat ed by s ki n fl ora from t he pat i ent or t he phl ebot omi s t . Corynebac t eri um makes up a s mal l fract i on of oral and rect al fl ora and i s l es s l i kel y t o ent er t he bl ood from t hes e s i t es . Endocardi t i s caus ed by Corynebac t eri um i s rare and al mos t never i nvol ves nat i ve val ves . Es c heri c hi a c ol i and Corynebac t eri um are es s ent i al l y i mpos s i bl e t o confus e i n t he l aborat ory. 2. A 19-year-old man with acute nonlymphocytic leukemia is admitted to the hospital 2 weeks after his first round of chemotherapy. His temperature is 39.2°C, and physical examination shows no localized abnormalities. Chest radiograph shows a Hickman catheter with its tip in the right atrium. T he 3
white blood cell (WBC) count is 300/mm with no polymorphonuclear or band cells in the differential count. Blood cultures are obtained. Which of the following is the next step? A Ini t i at e ant i s t aphyl ococcal t reat ment for t he pos s i bi l i t y of Hi ckman cat het er–rel at ed bact eremi a. B Admi ni s t er broad-s pect rum ant i bi ot i cs wi t h excel l ent act i vi t y for ent eri c Gram-negat i ve rods and Ps eudomonas aerugi nos a. C Awai t res ul t s of bl ood cul t ures and ot her di agnos t i c t es t s becaus e i nfect i on coul d be caus ed by al mos t any mi croorgani s m. D Ini t i at e oral prophyl axi s t o prevent bact eri al and fungal i nfect i ons . E Admi ni s t er parent eral ant i fungal t herapy. Vi ew Ans wer 2. T he answer is B [II A 3]. The next s t ep i s t o admi ni s t er
Pa g e 1 8 4 1
ABC Ambe r CHM Conve rte r Tria l ve rsion, http://w w w .proce sste x t.com/a bcchm.html
broad-s pect rum ant i bi ot i cs wi t h act i vi t y agai ns t ent eri c Gram-negat i ve rods and Ps eudomonas aerugi nos a. The hei ght ened s us cept i bi l i t y of neut ropeni c pat i ent s t o di e qui ckl y of overwhel mi ng s eps i s makes earl y i nt ervent i on obl i gat ory. Even t hough many cl i ni ci ans i ncl ude ant i bi ot i c coverage for s t aphyl ococci , t he mos t common bact eri a encount ered are Gram-negat i ve rods . Ant i fungal t reat ment i s us ed i f ant i bact eri al t herapy fai l s t o cont rol t he fever or i f a s peci fi c fungal i nfect i on i s found. Prophyl axi s i s s t art ed before havi ng evi dence of act i ve i nfect i on. 3. A 20-year-old woman with a history of seizures for which she has taken phenytoin for 4 months has a fever of 38.7°C; she has felt febrile for 2 weeks. She has no respiratory or urinary symptoms. Physical examination findings are normal except for elevated body temperature. Chest radiograph, urinalysis, and complete blood count (CBC) findings are normal. Which of the following would be the best next step? A Admi t t he pat i ent for t oxi c s hock s yndrome (TSS). B Admi ni s t er acet ami nophen. C Treat occul t uri nary t ract i nfect i on wi t h ant i bi ot i cs . D Di s cont i nue phenyt oi n and pres cri be anot her ant i convul s ant . E Cool t he pat i ent wi t h a cool i ng bl anket . Vi ew Ans wer 3. T he answer is D [onl i ne VII D 2 a (1)]. The next bes t s t ep i s t o di s cont i nue t he phenyt oi n and pres cri be anot her ant i convul s ant . Of t he many drugs t hat caus e fever as an unwant ed s i de effect , ant i convul s ant agent s l ead t he l i s t . Prol onged fever can have many caus es ot her t han medi cat i on react i ons and may be di ffi cul t t o di agnos e. In ambul at ory pat i ent s , common bact eri al i nfect i ons are fai rl y wel l excl uded by normal fi ndi ngs on ches t radi ograph, uri ne, and bl ood t es t s . Toxi c s hock s yndrome (TSS) i s a mul t i s ys t em i l l nes s i n whi ch fever i s onl y one component . Al t hough juveni l e rheumat oi d art hri t i s affect s pat i ent s i n t hi s age group, as pi ri n woul d be more appropri at e t han acet ami nophen. Cool i ng bl anket s are us ed onl y i n s everel y i l l pat i ent s .
Pa g e 1 8 4 2
ABC Ambe r CHM Conve rte r Tria l ve rsion, http://w w w .proce sste x t.com/a bcchm.html
4. A sexually active 24-year-old woman known to be HIV-1–infected has had a fever for 2 days and has a productive cough. Chest radiographs show an infiltrate in the right lung. T wo weeks earlier, her helper T (T h) cell (CD4) 3
count was 510/mm . Gram stain of sputum shows many white blood cells (WBCs) and squamous epithelial cells with a mixed bacterial flora. T esting for nontreponemal antigen (rapid plasma reagin) is positive at two dilutions, and treponemal antigen testing is also positive. Which of the following is the most likely cause of the pneumonia? A St rept oc oc c us pneumoni ae B Pneumoc ys t i s c ari ni i C Cyt omegal ovi rus (CMV) D Myc obac t eri um avi um-i nt rac el l ul are (MAI) E Syphi l i s Vi ew Ans wer 4. T he answer is A [V C 4 b, c (1); VIII G 1 h (2)]. The mos t l i kel y caus e of t he pneumoni a i s St rept oc oc c us pneumoni ae. Al t hough P. c ari ni i i s t he mos t common s eri ous opport uni s t i c i nfect i on i n pat i ent s wi t h HIV-1 i nfect i on, t he product i ve cough, l ocal i zed i nfi l t rat e, and bri ef durat i on argue agai ns t P. c ari ni i pneumoni a (PCP). In addi t i on, t he Th cel l (CD4) count of more t han 500 3
cel l s /mm s ugges t s t hat s eri ous opport uni s t i c i nfect i ons s uch as PCP, cyt omegal ovi rus (CMV), or Myc obac t eri um avi um-i nt rac el l ul are (MAI) are unl i kel y for s ome t i me. Syphi l i s , whi ch may have been pres ent , al mos t never i nvol ves t he l ungs . 5. A 62-year-old man has right upper quadrant abdominal pain, nausea, and vomiting. Physical examination shows only guarding over the liver. Ultrasound examination confirms the diagnosis of gallstones P.379 without dilated bile ducts. T he man is allergic to penicillin (a rash developed after penicillin therapy for a sore throat). In addition to dietary changes, which of the following treatments would be
Pa g e 1 8 4 3
ABC Ambe r CHM Conve rte r Tria l ve rsion, http://w w w .proce sste x t.com/a bcchm.html
best? A No ant i bi ot i cs B Eryt hromyci n C Oral qui nol one D Tri met hopri m–s ul famet hoxaz ol e E Amoxi ci l l i n Vi ew Ans wer 5. T he answer is A [Chapt er 5???]. Uncompl i cat ed chol ecys t i t i s wi t hout obs t ruct i on of t he bi l i ary duct s or empyema of t he gal l bl adder i s bes t managed cons ervat i vel y. Al t hough s ome bact eri a may be found i n t he bi l e at t he t i me of chol ecys t ect omy, t here i s rarel y progres s i on t o i nfect i on. Eryt hromyci n es s ent i al l y has no act i vi t y for any of t he agent s as s oci at ed wi t h chol ecys t i t i s or chol angi t i s . Amoxi ci l l i n i s cont rai ndi cat ed for pat i ent s who are al l ergi c t o peni ci l l i n. Severel y i l l pat i ent s s houl d be hos pi t al i zed t o manage t he pos s i bi l i t y of s eri ous compl i cat i ons . 6. T wo weeks after emergency surgery for a perforated duodenal ulcer, a 39-year-old woman complains of fever and vague abdominal pain. Her only medication is ranitidine. On physical examination, she appears a little pale, has a temperature of 38°C, and has a slight fullness in the epigastrium. Computed tomography (CT ) scan of the abdomen shows an area of fluid collection measuring 3 × 3 × 8 cm in the left paracolic gutter. Which of the following would be the most effective next step? A Admi ni s t er ant i bi ot i c t herapy wi t h an agent hi ghl y effect i ve agai ns t aerobi c Gram-negat i ve rods (e.g., azt reonam). B Admi ni s t er ant i bi ot i c t herapy effect i ve agai ns t abdomi nal anaerobes (e.g., cl i ndamyci n). C Provi de cat het er drai nage of t he fl ui d col l ect i on and ant i bi ot i cs appropri at e for cul t ure res ul t s . D Admi ni s t er no t herapy unl es s bl ood cul t ures are pos i t i ve or t he col l ect i on changes i n s i ze. E Order a magnet i c res onance i magi ng (MRI) s can t o confi rm t he CT fi ndi ngs .
Pa g e 1 8 4 4
ABC Ambe r CHM Conve rte r Tria l ve rsion, http://w w w .proce sste x t.com/a bcchm.html
Vi ew Ans wer 6. T he answer is C [V E 2 c]. Cat het er drai nage of t he fl ui d col l ect i on and ant i bi ot i cs appropri at e for cul t ure res ul t s woul d be t he mos t effect i ve s t ep t o t ake next . The mos t l i kel y expl anat i on for t he cl i ni cal fi ndi ngs and t he comput ed t omography (CT) abnormal i t y i s a pos t operat i ve i nt ra-abdomi nal abs ces s . Thes e rarel y heal wi t hout drai nage. Surgi cal drai nage i s effect i ve, but a l es i on t hat can be reached s afel y wi t h a cat het er may be drai ned equal l y effect i vel y and more s afel y. Ant i bi ot i cs are us ual l y us ed as adjunct i ve t herapy. The fl ora of t hes e abs ces s es i s us ual l y mi xed wi t h ent eri c aerobes and anaerobes . Good cul t ures can be us eful i n refi ni ng t he exact t herapeut i c regi men. Magnet i c res onance i magi ng (MRI) i s a fi ne t es t , but t he CT s can has al ready l ed t o a di agnos i s . 7. A 64-year-old woman presents to the emergency department with abdominal pain and fever. She has had a long history of mild, intermittent dyspepsia that frequently has followed meals. She has had diabetes for 12 years, and although her response to oral hypoglycemic agents has been poor, she has refused to use insulin. At the time of presentation, she has been ill for 36 hours with vomiting, fever, and abdominal pain. On examination, she is febrile with a temperature of 38.9°C and has tachycardia of 124 beats/minute. She is mildly jaundiced. Bowel sounds are diminished, and tenderness and guarding are most marked in t he right upper quadrant. T oward the end of the examination, rigor is evident, and she has broken into a sweat. Which of the following is most likely to be her problem? A Pancreat i t i s wi t h abs ces s B Chol angi t i s C Perforat ed pept i c ul cer D Spl eni c abs ces s E Es ophageal refl ux Vi ew Ans wer 7. T he answer is B [V E 4]. Accordi ng t o t he pat i ent , s he has cert ai nl y had chol ecys t i t i s i n t he pas t . W i t h gal l s t ones i n t he gal l bl adder, t here i s al ways s ome ri s k of mi grat i on t o t he common
Pa g e 1 8 4 5
ABC Ambe r CHM Conve rte r Tria l ve rsion, http://w w w .proce sste x t.com/a bcchm.html
bi l e duct and s ubs equent obs t ruct i on of t he duct . The i nfect i on t hat may fol l ow obs t ruct i on of t he common bi l e duct may be very s eri ous , wi t h bact eremi a and s eps i s as common s equel ae. Pancreat i c abs ces s can mani fes t as abdomi nal pai n, but i t does not have t he as s oci at ed l ocal i zed ri ght upper quadrant fi ndi ngs as oft en as chol angi t i s . Perforat ed ul cers l ead t o peri t oni t i s . Spl eni c abs ces s es are al mos t al ways t he s equel ae of bact eremi a (e.g., wi t h endocardi t i s ). Refl ux es ophagi t i s can caus e s evere heart burn but does not res ul t i n s eps i s . 8. A 24-year-old man takes an exotic “ around the world― vacation to Europe, Africa, India, and China. On his holiday, he has frequent sexual encounters and does not use condoms. In addition, he is bitten by mosquitoes many times. On his return, he notes fever, and his physician finds generalized lymphadenopathy. Which of the following is least likely to account for both these symptoms? A Human i mmunodefi ci ency vi rus t ype 1 (HIV-1) B Infect i ous mononucl eos i s C Mal ari a D Syphi l i s E Cat -s crat ch di s eas e Vi ew Ans wer 8. T he answer is C [VII A, B]. Mal ari a i s charact eri zed by l ack of l ymphadenopat hy, al t hough s pl enomegal y and fever are charact eri s t i c. Al t hough general i zed enl argement of l ymph nodes i s a nons peci fi c fi ndi ng, i t oft en can l ead t o t he di agnos i s of a cl i ni cal ent i t y. W hen t he l ymphadenopat hy i s coupl ed wi t h fat i gue and an at ypi cal l ymphocyt os i s , i nfect i ous mononucl eos i s i s s ugges t ed. Thi s s yndrome mos t oft en i s caus ed by Eps t ei n-Barr vi rus (EBV), but a s i mi l ar cl i ni cal pi ct ure i s produced by cyt omegal ovi rus (CMV) i nfect i on. The i nfect i ons can be di s t i ngui s hed by us i ng s erol ogi c t es t s (e.g., Monos pot t es t ) or s peci fi c ant i bodi es . Secondary s yphi l i s al s o s houl d be cons i dered i n pat i ent s wi t h adenopat hy of s hort durat i on; t hi s can be confi rmed on t he bas i s of s erol ogi c t es t s and t he fi ndi ng of an appropri at e ras h. React i ve, hyperpl as t i c
Pa g e 1 8 4 6
ABC Ambe r CHM Conve rte r Tria l ve rsion, http://w w w .proce sste x t.com/a bcchm.html
l ymph nodes are common i n HIV-1 i nfect i on. For t hi s pat i ent , al l of t hes e are pos s i bi l i t i es , but t he mal ari a woul d not account for t he enl arged l ymph nodes . Cat -s crat ch di s eas e us ual l y caus es enl arged l ymph gl ands , wi t h s t el l at e mi croabs ces s es apparent hi s t ol ogi cal l y. 9. A 14-year-old girl from Pennsylvania goes to Wisconsin for a 2-week camping trip. T wo weeks after she returns, she notices a solitary circular rash on her left calf just above the area normally covered by her socks. Overall, she feels well, and there is gradual progression of the rash over the next week. Which of the following diseases endemic in Wisconsin or Pennsylvania did she most likely acquire? A Bl as t omycos i s B Lyme di s eas e C Cal i forni a encephal i t i s D Rocky Mount ai n s pot t ed fever (RMSF) E Syphi l i s Vi ew Ans wer 9. T he answer is B [VII F]. Onl y Lyme di s eas e and Rocky Mount ai n s pot t ed fever (RMSF) occur wi t h a ras h as a common feat ure. Cal i forni a encephal i t i s (whi ch i s s pread by mos qui t o) has few, i f any, s ki n mani fes t at i ons but does caus e a s yndrome of progres s i ve neurol ogi c det eri orat i on t hat res ol ves s pont aneous l y i n mos t cas es . Bl as t omycos i s , a di s eas e t hat ent ers t he body t hrough t he res pi rat ory t ract and may met as t as i ze t o ot her organs , al mos t never generat es a ci rcul ar ras h. RMSF has a ras h t hat us ual l y begi ns peri pheral l y but i s rarel y s ol i t ary. The cours e of di s eas e i s us ual l y fai rl y rapi d, i n cont ras t t o t he i ndol ent but progres s i ve l es i on t hat i s t ypi cal of eryt hema mi grans . Syphi l i s caus es a ras h but al mos t never i nvol ves onl y one s pot on t he l eg. P.380
10. Which of the following is the strongest indication to consider valve replacement surgery in a patient with infective endocarditis?
Pa g e 1 8 4 7
ABC Ambe r CHM Conve rte r Tria l ve rsion, http://w w w .proce sste x t.com/a bcchm.html
A Hemat uri a B Pos i t i ve cul t ures for St aphyl oc oc c us aureus on t he s econd day of t herapy C Spl i nt er hemorrhages and Os l er's nodes D Progres s i ve conges t i ve heart fai l ure (CHF) E Spl enomegal y Vi ew Ans wer 10. T he answer is D [V I 4 d]. Al t hough al l of t he ans wers refl ect as pect s of endocardi t i s , t he onl y abs ol ut e i ndi cat i ons for val ve repl acement are fungal endocardi t i s , conges t i ve heart fai l ure (CHF), val ve ri ng abs ces s , and fai l ure t o cl ear i nfect i on aft er a l ong cours e of ant i mi crobi al t herapy. Some aut hori t i es recommend val ve repl acement aft er mul t i pl e s i gni fi cant embol i . Mi nor embol i , an enl arged s pl een, and i mmunol ogi c phenomena are not caus e for val ve repl acement . 11. An 18-year-old woman takes a summer job working in a day-care center. A month after starting, she develops a rash over her whole body. T he lesions are small vesicles on a slightly erythematous base. Except for a slight cough and fever of 38°C, she feels well. She has proof of having received her childhood immunizations before she started school. Which of the following statements is most correct? A The pat i ent probabl y has an el evat ed t ot al whi t e bl ood cel l (W BC) count and a di fferent i al wi t h an abundance of neut rophi l s and band forms . B W i t hi n 1 week, t he pat i ent wi l l recover t ot al l y wi t h l i fel ong i mmuni t y and no furt her s equel ae. C The pat i ent probabl y devel oped an al l ergy t o s ome of t he fabri c i n t he t oys at t he day-care cent er. D The pat i ent may be i nfect i ous t o ot her fami l y members . E If t he pat i ent becomes pregnant 1 year or more l at er, s he wi l l requi re cl os e obs t et ri c obs ervat i on. Vi ew Ans wer 11. T he answer is D [Onl i ne II H 4]. Al t hough a vari cel l a vacci ne does exi s t , i t s i mpl ement at i on has not been uni vers al among ol der
Pa g e 1 8 4 8
ABC Ambe r CHM Conve rte r Tria l ve rsion, http://w w w .proce sste x t.com/a bcchm.html
chi l dren or adul t s . Thus , many chi l dren and adul t s are s t i l l s us cept i bl e t o vari cel l a. Sequel ae of vari cel l a i nfect i on are rare and i ncl ude pneumoni a, meni ngi t i s , and hepat i t i s as wel l as herpes z os t er (s hi ngl es ). The whi t e bl ood cel l (W BC) count i s normal or s l i ght l y depres s ed i n mos t of t he chi l dhood exant hems . Vari cel l a i n pregnancy can be s evere, but t hi s pat i ent wi l l be s afe once her ras h di s appears . 12. After a recent bad cold, an 18-year-old man has symptoms of sinusitis with nasal drainage and stuffiness. On examination, papilledema is evident. T he physician is convinced that the patient has a brain abscess. Which of the following statements is correct? A Magnet i c res onance i magi ng (MRI) s cans are us ual l y not hel pful . B The s ource of i nfect i on i s us ual l y cardi ac. C Ant i mi crobi al s act i ve agai ns t Ps eudomonas aerugi nos a are us ual l y a part of t he t reat ment . D Compl et e s urgi cal exci s i on i s us ual l y not needed. E The pat i ent i s l i kel y t o devel op bact eri al meni ngi t i s i n t he next 5 years . Vi ew Ans wer 12. T he answer is D [V A 3 c]. Cros s -s ect i onal i magi ng s t udi es [e.g., comput ed t omography (CT) and magnet i c res onance i magi ng (MRI)] are cri t i cal t o t he di agnos i s and management of i nt racrani al abs ces s . Surgery i s oft en undert aken t o obt ai n a s peci men for cul t ure or t o decompres s t he abs ces s . Tot al s urgi cal exci s i on i s rarel y requi red i n t he pat i ent who i s res pondi ng wel l t o medi cal t herapy. Al t hough occas i onal pat i ent s do have cardi opul monary s ources for i nfect i on, mos t pat i ent s have cont i guous i nfect i on of t he mi ddl e ear or, as i n t hi s cas e, t he s i nus . Meni ngi t i s can fol l ow t rauma t o t he cri bri form pl at e but rarel y fol l ows a brai n abs ces s . 13. A 12-year-old boy has had fever and bloody diarrhea for 2 weeks and has lost 4 pounds of body weight. T hree stool specimens have yielded no enteric pathogens on culture, and no ova or mature parasites are observed on direct examination. Which of the following is the best reason for his illness?
Pa g e 1 8 4 9
ABC Ambe r CHM Conve rte r Tria l ve rsion, http://w w w .proce sste x t.com/a bcchm.html
A Regi onal ent eri t i s B Gi ardi as i s C Travel ers ' di arrhea caus ed by ent erot oxi geni c Es c heri c hi a c ol i D Crypt os pori di os i s E Jejunal i nfarct i on Vi ew Ans wer 13. T he answer is A [V D]. Regi onal ent eri t i s , ul cerat i ve col i t i s , and a vari et y of i nvas i ve i nfect i ous pat hogens can caus e bl oody di arrhea. A s i ngl e s t ool s peci men may mi s s any of t hes e agent s . Gi ardi a l ambl i a and Crypt os pori di um us ual l y caus e an upper s mal l bowel l es i on t hat l eads t o wat ery di arrhea wi t hout fever. Travel ers ' di arrhea i s a t oxi n-medi at ed i nfect i on t hat al s o l eads t o mi ni mal i nfl ammat ory changes i n t he bowel . Bowel i nfarct s are ext remel y rare i n chi l dren. 14. A 26-year-old woman with no history of medical problems complains of 3 days of burning with urination and increased frequency. Urinalysis shows positive result for leukocyte esterase and nitrates on dipstick. T he microscopic examination shows 5–10 white blood cells and no bacteria. Which of the following is the next step? A W ai t for res ul t s before s t art i ng t herapy. B Treat wi t h Pyri di um, a uri nary anal ges i c, s i nce t here i s no bact eri uri a C Treat wi t h 3 days of t ri met hopri m-s ul famet hoxaz ol e D Treat for 7 days wi t h ci profl oxaci n E Treat for 14 days wi t h i nt ravenous cephal os pori n ant i bi ot i c Vi ew Ans wer 14. T he answer is C [V F 6 b]. The pat i ent 's pres ent at i on i s cons i s t ent wi t h uncompl i cat ed acut e uri nary t ract i nfect i on, and s he s houl d recei ve t reat ment . Cul t ures are not requi red t o confi rm t he di agnos i s , and abs ence of bact eri a on mi cros copi c exami nat i on does not rul e out UTI. The pres ence of W BC i n t he uri ne i s s t rong s ugges t i on of i nfect i on. Three days of t herapy are adequat e (and equi val ent t o a s even day cours e), and t ri met hopri m–s ul famet hoxaz ol e i s a good choi ce t hat i s
Pa g e 1 8 5 0
ABC Ambe r CHM Conve rte r Tria l ve rsion, http://w w w .proce sste x t.com/a bcchm.html
i nexpens i ve and wel l -t ol erat ed. However, i f rat es of res i s t ance i n t he communi t y are hi gh, us i ng ci profl oxaci n or anot her fl uoroqui nol one for 3 days woul d be advi s abl e. Int ravenous ant i bi ot i cs are not i ndi cat ed i n t he s et t i ng of uncompl i cat ed UTI. 15. A 70-year-old man has chronic lung disease. Which of the following immunizations should he receive annually? A Infl uenz a B Tet anus C Pneumococcal pneumoni a D Pert us s i s E Hepat i t i s B vi rus (HBV) Vi ew Ans wer 15. T he answer is A [
onl i ne II H 1]. Immuni t y t o pert us s i s i s
es t abl i s hed by i mmuni zat i on i n chi l dhood. Tet anus i mmuni t y i s mai nt ai ned by boos t ers every 10 years . A s i ngl e vacci nat i on for pneumococcal i nfect i on s houl d be gi ven t o al l pers ons wi t h chroni c cardi opul monary di s eas e, t o al l adul t s ol der t han age 60 years , and t o al l pat i ent s wi t h HIV i nfect i on. Re-i mmuni zat i ons may not be needed at al l , and cert ai nl y no more t han once every 5–7 years . However, t he i mmuni t y conferred by i nfl uenz a vacci ne i s s hort -l i ved and s t rai n s peci fi c. Y earl y i mmuni zat i ons for i nfl uenz a are s t rongl y recommended for t he el derl y and t hos e wi t h chroni c cardi opul monary di s eas es . Hepat i t i s B vi rus (HBV) vacci ne i s not rout i nel y i ndi cat ed i n t he el derl y, and vacci nat i on i s not needed for at l eas t 5 years . P.381
P.382
P.383
P.384
Pa g e 1 8 5 1
ABC Ambe r CHM Conve rte r Tria l ve rsion, http://w w w .proce sste x t.com/a bcchm.html
16. A 49-year-old man reports recent sexual contact with a commercial sex worker. About 9 days later, he experiences dysuria and has a very modest urethral discharge. Which of the following infectious agents most likely accounts for his symptoms? A Chl amydi a t rac homat i s i nfect i on B Syphi l i s C Tri chomoni as i s D Granul oma i ngui nal e E Es c heri c hi a c ol i Vi ew Ans wer 16. T he answer is A [VI B]. Chl amydi al i nfect i on i s one of t he mos t common caus es of uret hri t i s and frequent l y mani fes t s wi t h dys uri a. Syphi l i s and granul oma i ngui nal e do affect t he ext ernal geni t al s but t end t o s pare t he uret hral mucos a. Tri chomonas i s s omet i mes found i n t he mal e geni t al s , but i t i s al mos t al ways wi t hout s ympt oms . Es c heri c hi a c ol i can caus e a uri nary t ract i nfect i on but not a s exual l y t rans mi t t ed di s eas e (STD). 17. A 41-year-old man with a history of weight loss and hairy leukoplakia reports that a recent HIV-1 blood test came back 3
positive. His CD4 count is 204/mm , and his viral load is 58,000 copies/mL by reverse transcriptase polymerase chain reaction (moderately high). A purified protein derivative (PPD) skin test is negative for tuberculosis, and you find that he has already had a pneumococcal vaccination. Which of the following would be the next step? A W ai t unt i l he devel ops a more s i gni fi cant opport uni s t i c i nfect i on before begi nni ng ant i ret rovi ral t herapy. B Begi n z i dovudi ne at s t andard dos es and t ake t he pat i ent 's CD4 count and a vi ral l oad i n 6 weeks . C Begi n a combi nat i on regi men wi t h t hree act i ve agent s s uch as z i dovudi ne, l ami vudi ne, and efavi renz. D Begi n a cours e of fl uconazol e t o prevent t hrus h and ri fabut i n t o
Pa g e 1 8 5 2
ABC Ambe r CHM Conve rte r Tria l ve rsion, http://w w w .proce sste x t.com/a bcchm.html
prevent Myc obac t eri um avi um-i nt rac el l ul are (MAI) i nfect i on. E Refer t he pat i ent t o a dent i s t for management of t he hai ry l eukopl aki a. Vi ew Ans wer 17. T he answer is C [VIII D 3]. Sympt omat i c HIV-1 i nfect i on can be t reat ed us i ng combi nat i on t herapy wi t h exceedi ngl y good res ul t s . W i t h l uck, t he correct ant i ret rovi ral drug combi nat i on wi l l l ead t o an i mprovement i n t hi s pat i ent 's i mmune funct i on, and he mi ght be abl e t o avoi d al l chemoprophyl axi s i ndefi ni t el y. Del ay i n t reat ment now mi ght s pare hi m s ome drug s i de effect s , but he i s al mos t t o t he poi nt of devel opi ng s evere compl i cat i ons from HIV-1 i t s el f. Treat ment wi t h a s i ngl e drug s uch as zi dovudi ne i s not us eful becaus e of i t s l i mi t ed i mpact on vi ral burden and t he earl y emergence of res i s t ance. Al t hough prophyl axi s for Myc obac t eri um avi um-i nt rac el l ul are (MAI) and fungal i nfect i ons may reduce t he number of s uch i nfect i ons , i t need not be s t art ed unt i l i mmune dys funct i on has progres s ed cons i derabl y. Hai ry l eukopl aki a i s not s eri ous and wi l l s ubs i de wi t h i mproved i mmune funct i on. 18. When a patient needs a peripheral intravenous catheter for approximately 1 week, which of the following methods is the most important way of preventing catheter-related infections? A Us i ng a powerful ant i mi crobi al oi nt ment at t he junct i on of t he hub and t he s ki n B Changi ng t he cat het er s i t e every 48–72 hours C Gi vi ng s ys t emi c ant i bi ot i cs for t he ent i re durat i on of cat het er pl acement D Shavi ng t he s ki n before cl eans i ng i t E Us i ng t he l eg rat her t han t he arm Vi ew Ans wer 18. T he answer is B [IV
onl i ne F 2 b]. As s umi ng t hat al l
cat het ers are pl aced under t he bes t pos s i bl e condi t i ons , keepi ng t hem cl ean and dry i s al l t he dai l y care t hey need. Short pl as t i c cat het ers are prone t o i nfect i on over t i me, s o rout i ne repl acement i s recommended. Thi s rul e does not appl y t o cent ral cat het ers (pl aced di rect l y i nt o a cent ral vei n or t hreaded up al ong t he arm).
Pa g e 1 8 5 3
ABC Amber CHM Converter Trial version, http://www.processtext.com/abcchm.html
There i s no benefi t t o s havi ng t he s ki n, except t o provi de a bet t er s urface for t he adhes i ve t ape us ed t o s ecure t he cat het er. Int ravenous l i nes i n t he l eg, whi ch are t ri cki er t o mai nt ai n and have a hi gher rat e of i nfect i on, s houl d be avoi ded whenever pos s i bl e.
Page 1854
ABC Ambe r CHM Conve rte r Tria l ve rsion, http://w w w .proce sste x t.com/a bcchm.html
Editors: Wolfsthal, Susan T itle: NMS Medicine, 6th Edition Copyri ght ©2008 Li ppi ncot t W i l l i ams & W i l ki ns > T able of Cont ent s > Chapt er 9 - Endoc rine and Met abolic Diseases
Chapter 9 Endocrine and Metabolic Diseases Sonia Blome Daniel Lender
I. Disorders of the Pituitary Gland A. Anterior pituitary disease Anterior pituitary disease res ul t s from i ns uffi ci ent product i on of pi t ui t ary hormones (hypopi t ui t ari s m), exces s i ve product i on of pi t ui t ary hormones (acromegal y, Cus hi ng's di s eas e, or hyperprol act i nemi a), or t he l ocal effect s of pi t ui t ary t umors .
1. Pituitary tumors make up 10% of i nt racrani al t umors . Mos t are beni gn, but t hei r cont i nued s l ow growt h i n t he confi ned s el l ar and s upras el l ar areas may caus e s eri ous neurol ogi c damage. o
o
a. T ypes
(1) Pituitary adenomas are cl as s i fi ed by cel l t ype, bas ed on el ect ron mi cros copy and i mmunohi s t ochemi cal s t ai ni ng. Lactotroph tumors produce prol act i n, somatotroph tumors produce growt h hormone (GH), and corticotroph tumors produce adrenocort i cot ropi c hormone (ACTH).
Pa g e 1 8 5 5
ABC Ambe r CHM Conve rte r Tria l ve rsion, http://w w w .proce sste x t.com/a bcchm.html
(2) Craniopharyngiomas, t he mos t common t umors of t he hypot hal ami c–pi t ui t ary area i n chi l dren, ari s e from remnant s of cel l s from Rat hke's pouch.
(a) Thes e t umors us ual l y are l ocat ed above t he s el l a t urci ca, but t hey may produce changes wi t hi n t he s el l a i t s el f.
(b) They may be s ol i d or cys t i c, may cont ai n chol es t erol -ri ch fl ui d, and oft en cont ai n areas of cal ci fi cat i on.
(3) Meningiomas and metastatic tumors may i nvol ve t he hypot hal ami c–pi t ui t ary area.
o
o
b. Clinical features
(1) Excess hormone production by pi t ui t ary adenomas may l ead t o acromegaly (s ee I A 4), Cushing' s disease (s ee V B 1 a), or hyperprolactinemia (s ee I A 3).
(a) In rare cas es t hes e t umors may produce exces s t hyroi d-s t i mul at i ng hormone (TSH), caus i ng hypert hyroi di s m.
(b) Fol l i cl e-s t i mul at i ng hormone (FSH) and l ut ei ni zi ng hormone (LH) are frequent l y produced i n exces s by
Pa g e 1 8 5 6
ABC Ambe r CHM Conve rte r Tria l ve rsion, http://w w w .proce sste x t.com/a bcchm.html
pi t ui t ary t umors , but t hi s us ual l y does not res ul t i n a cl ear-cut cl i ni cal s yndrome.
(2) Insufficient hormone production, due t o compres s i on or des t ruct i on of pi t ui t ary and hypot hal ami c cel l s , produces t he s yndrome of hypopituitarism (s ee I A 2).
(3) Neurologic effects
(a) Optic nerve compression may occur. Pi t ui t ary t umors may pres s upward on t he i nferi or s urface of t he opt i c chi as m. Vi s i on l os s t ends t o occur fi rs t i n t he s uperi or t emporal quadrant s , wi t h bi t emporal hemi anopi a i n more advanced cas es .
(b) Headache i s common.
(c) Ot her neurol ogi c mani fes t at i ons s uch as ment al s t at us changes , crani al nerve abnormal i t i es , vomi t i ng, and papi l l edema are l es s common.
(4) Sens i t i ve i magi ng t echni ques [e.g., magnet i c res onance i magi ng (MRI)] can s how pi t ui t ary microadenomas (t umors <10 mm i n di amet er) i n 10% of normal i ndi vi dual s . If hyperprol act i nemi a or ot her hormone
Pa g e 1 8 5 7
ABC Ambe r CHM Conve rte r Tria l ve rsion, http://w w w .proce sste x t.com/a bcchm.html
abnormal i t i es are not pres ent and i f fol l ow-up s t udy s hows no progres s i ve enl argement , t hes e t umors s houl d be regarded as i nci dent al fi ndi ngs of no cl i ni cal s i gni fi cance. P.386
(5) Multiple endocrine neoplasia, type I (MEN I, Wermer syndrome) i s a s yndrome cons i s t i ng of t umors , oft en funct i oni ng, of t he pi t ui t ary, parat hyroi ds , and pancreat i c i s l et s .
o
o
c. Diagnosis
(1) Diagnostic imaging
(a) Microadenomas may be vi s ual i zed wi t h more s ens i t i ve procedures s uch as MRI with gadolinium enhancement or computed tomography (CT ).
(b) Skull radiographs may s how enl argement or di s t ort i on of t he s el l a when t umors are 10 mm or more i n di amet er (macroadenomas). Supras el l ar cal ci fi cat i on s ugges t s t he pres ence of a crani opharyngi oma.
(2) Hormone studies. Pi t ui t ary adenomas t hat s ecret e exces s GH, ACTH, or prol act i n can
Pa g e 1 8 5 8
ABC Ambe r CHM Conve rte r Tria l ve rsion, http://w w w .proce sste x t.com/a bcchm.html
be di agnos ed by meas uri ng t he pi t ui t ary hormones , or i n s ome cas es t he hormones produced by t arget organs , even i f t he adenoma i s t oo s mal l t o be vi s ual i zed by di agnos t i c i magi ng. o
o
d. T herapy
(1) Surgery i s i ndi cat ed for pi t ui t ary adenomas t hat produce neurol ogi c s ympt oms and for s ome t umors t hat caus e s yndromes as s oci at ed wi t h hormone overproduct i on.
(a) T ranssphenoidal pituitary microsurgery i s us ed for i nt ras el l ar t umors t hat have mi ni mal or no s upras el l ar ext ens i on. Smal l adenomas oft en can be removed wi t hout damage t o normal pi t ui t ary t i s s ue.
(b) T ransfrontal resection may be neces s ary for l arge t umors t hat ext end far out s i de t he s el l a t urci ca or compres s t he opt i c chi as m.
(2) Radiation therapy, us ed al one or i n conjunct i on wi t h s urgery, may decreas e t he s i ze of pi t ui t ary t umors and decreas e hormone product i on.
(3) Medical therapy
Pa g e 1 8 5 9
ABC Ambe r CHM Conve rte r Tria l ve rsion, http://w w w .proce sste x t.com/a bcchm.html
(a) Hormone replacement i s requi red i f hypopi t ui t ari s m i s pres ent .
(b) Dopami ne agoni s t s s uch as bromocriptine and cabergoline as wel l as s omat os t at i n anal ogs s uch as octreotide may decreas e t he s i ze and hormone product i on of cert ai n pi t ui t ary t umors [s ee I A 3 d (1) and I A 4 d (3)].
2. Hypopituitarism o
o
a. Etiology
(1) Pituitary tumors, mos t commonl y pi t ui t ary macroadenomas and crani opharyngi omas , may des t roy normal hypot hal ami c–pi t ui t ary t i s s ue.
(2) Sheehan' s syndrome i s hypopi t ui t ari s m caus ed by i nfarct i on of t he ant eri or pi t ui t ary gl and duri ng chi l dbi rt h. The pi t ui t ary gl and doubl es i n s i ze duri ng pregnancy, l argel y becaus e of hyperpl as i a of t he l act ot rophs . The bl ood s uppl y does not keep pace wi t h t he enl argement , however, and hypot ens i ve epi s odes duri ng a compl i cat ed del i very may l ead t o i nfarct i on.
Pa g e 1 8 6 0
ABC Ambe r CHM Conve rte r Tria l ve rsion, http://w w w .proce sste x t.com/a bcchm.html
(3) Surgery for t he removal of pi t ui t ary or ot her brai n t umors may damage t he hypot hal amus , t he pi t ui t ary gl and, or bot h.
(4) Less common causes of pi t ui t ary or hypot hal ami c des t ruct i on i ncl ude sarcoidosis, hemochromatosis, Hand-Schüller-Christian disease, tuberculosis, syphilis, and fungal infections.
o
o
b. Clinical features
(1) GH deficiency has di fferent effect s i n chi l dren and adul t s .
(a) In chi l dren, growth failure occurs , l eadi ng t o s hort s t at ure i n adul t hood (pituitary dwarfism).
(b) In adul t s , GH defi ci ency was once t hought t o have no i mport ant cl i ni cal effect s . It i s now recogni zed t hat GH defi ci ency caus es undes i red changes i n body compos i t i on, wi t h an increase in body fat and a decrease in lean body mass. Thes e changes may be accompani ed by decreas ed s t rengt h and exerci s e capaci t y.
(c) Ot her advers e effect s of GH defi ci ency may i ncl ude an i ncreas e i n
Pa g e 1 8 6 1
ABC Ambe r CHM Conve rte r Tria l ve rsion, http://w w w .proce sste x t.com/a bcchm.html
cardi ovas cul ar ri s k fact ors s uch as i ns ul i n res i s t ance and hyperl i pi demi a, at heros cl eros i s , i mpai red qual i t y of l i fe, and perhaps s hort ened l i fe expect ancy.
(2) Gonadotropin (LH and FSH) deficiency caus es amenorrhea and genital atrophy in women and loss of potency and libido in men. If adrenal androgens are defi ci ent as wel l , becaus e of concomi t ant ACTH defi ci ency, pubi c and axi l l ary hai r may be l os t , es peci al l y i n women. P.387
(3) T SH deficiency res ul t s i n t he s ympt oms and phys i cal changes of hypothyroidism (s ee II B 2 a,b).
(4) ACT H deficiency l eads t o adrenal insufficiency (s ee V C). Secondary adrenal i ns uffi ci ency (caus ed by pi t ui t ary di s eas e) di ffers i n s everal cl i ni cal mani fes t at i ons from pri mary adrenal i ns uffi ci ency (caus ed by adrenal di s eas e).
(a) Hyperpigmentation of t he s ki n and mucous membranes i s charact eri s t i c of pri mary adrenal di s eas e.
(i) It i s caus ed, i ndi rect l y, by t he
Pa g e 1 8 6 2
ABC Ambe r CHM Conve rte r Tria l ve rsion, http://w w w .proce sste x t.com/a bcchm.html
negat i ve-feedback s t i mul at i on of ACTH by l ow pl as ma cort i s ol l evel s . [ACTH and mel anocyt e-s t i mul at i ng hormone (MSH) are deri ved from t he s ame l arge precurs or mol ecul e (pro-opi omel anocort i n), s o when ACTH i s i ncreas ed, MSH i s i ncreas ed as wel l . MSH s t i mul at es mel anocyt es and caus es pi gment at i on.]
(ii) ACTH (and t herefore, MSH) l evel s are l ow i n s econdary adrenal i ns uffi ci ency; cons equent l y, hyperpi gment at i on i s not charact eri s t i c of t hi s condi t i on.
(b) Electrolyte changes (i .e., decreas ed s erum s odi um and i ncreas ed s erum pot as s i um l evel s ) are mi ni mal i n s econdary adrenal i ns uffi ci ency, becaus e al dos t erone product i on by t he adrenal cort ex (whi ch promot es s odi um ret ent i on) depends pri mari l y on reni n and angi ot ens i n (whi ch are undi s t urbed) rat her t han on ACTH.
(5) Prolactin deficiency may be res pons i bl e for t he pos t part um fai l ure of l act at i on i n Sheehan's s yndrome but ot herwi s e produces no cl i ni cal mani fes t at i ons .
(6) W i t h s l ow, progres s i ve des t ruct i on of
Pa g e 1 8 6 3
ABC Ambe r CHM Conve rte r Tria l ve rsion, http://w w w .proce sste x t.com/a bcchm.html
pi t ui t ary t i s s ue, failure of GH and gonadotropin secretion occurs earl y. W i t h cont i nui ng l os s of t i s s ue, TSH and fi nal l y ACTH and prol act i n l evel s fal l bel ow normal .
(7) Deficiency of individual pituitary hormones may occur. Is ol at ed GH defi ci ency and i s ol at ed gonadot ropi n defi ci ency are not uncommon, es peci al l y i n chi l dren. Is ol at ed defi ci enci es of TSH and ACTH are very uncommon.
o
o
c. Diagnosis
(1) Evaluation of target organ function i s oft en t he fi rs t s t ep i n t he di agnos i s of hypopi t ui t ari s m; t hi s condi t i on i s oft en s us pect ed becaus e of fai l ure of more t han one t arget organ (i .e., t hyroi d, adrenal gl ands , and gonads ). Tes t s of t hyroi d, adrenal , ovari an, and t es t i cul ar funct i on are des cri bed i n s ect i ons II, V, VI, and VII, res pect i vel y.
(2) Measurement of pituitary hormones. GH levels may be undet ect abl e under bas al condi t i ons i n normal i ndi vi dual s ; t herefore, provocat i ve maneuvers may be needed t o prove i nadequacy of hormone product i on.
(a) Insulin-induced hypoglycemia (t he insulin tolerance test) i s t he mos t cons i s t ent l y effect i ve t es t s t i mul us for
Pa g e 1 8 6 4
ABC Ambe r CHM Conve rte r Tria l ve rsion, http://w w w .proce sste x t.com/a bcchm.html
GH and can al s o s t i mul at e cort i s ol product i on. Regul ar i ns ul i n i s gi ven as an i nt ravenous bol us , and GH l evel s are meas ured aft er 30, 60, and 90 mi nut es .
(b) The pat i ent mus t be obs erved cl os el y duri ng t he t es t ; cent ral nervous s ys t em (CNS) s ympt oms of hypogl ycemi a requi re i mmedi at e i nt ravenous admi ni s t rat i on of gl ucos e. Thi s t es t s houl d not be performed i n pers ons ol der t han 65 years , or pers ons wi t h coronary di s eas e or a s ei zure di s order. The next mos t effect i ve s t i mul us for GH, aft er i ns ul i n-i nduced hypogl ycemi a, i s t he IV infusion of arginine and GH-releasing hormone.
(c) In pat i ent s wi t h panhypopi t ui t ari s m, provocat i ve t es t s may not be neces s ary t o di agnos e GH defi ci ency. If t hree or more ot her pi t ui t ary hormone defi ci enci es exi s t , t he probabi l i t y t hat GH i s al s o defi ci ent exceeds 95%. Thi s probabi l i t y i s even hi gher i f i ns ul i n-l i ke growt h fact or 1 (IGF-1) l evel s are l ow (al t hough one-t hi rd of GH-defi ci ent pat i ent s may have normal l evel s of IGF-1.)
o
o
d. T herapy
(1) The underlying cause of t he pi t ui t ary
Pa g e 1 8 6 5
ABC Ambe r CHM Conve rte r Tria l ve rsion, http://w w w .proce sste x t.com/a bcchm.html
i ns uffi ci ency (e.g., enl argi ng pi t ui t ary t umors , granul omat ous di s eas es ) s houl d be s ought and t reat ed, i f pos s i bl e.
(2) Hormone replacement
(a) GH administration can s t i mul at e growt h and i ncreas e t he ul t i mat e hei ght i n chi l dren wi t h i s ol at ed GH defi ci ency or panhypopi t ui t ari s m. Synt het i c human growt h hormone of recombi nant DNA ori gi n i s avai l abl e but mus t be gi ven by i nject i on and i s expens i ve. P.388
GH repl acement i s gi ven t o s ome adul t s wi t h GH defi ci ency; however, t hi s t reat ment mus t be cons i dered on a cas e-by-cas e bas i s .
(b) T hyroid hormone i s gi ven i n us ual repl acement dos es (s ee II B 4).
(c) Cortisol (hydrocort i s one) i s gi ven i n us ual repl acement dos es (s ee V C 4).
(d) Estrogen–progesterone combinations may be gi ven t o women, and t es t os t erone t o men, t o prevent or t reat t he mani fes t at i ons of hypogonadi s m (s ee VII A 3).
Pa g e 1 8 6 6
ABC Ambe r CHM Conve rte r Tria l ve rsion, http://w w w .proce sste x t.com/a bcchm.html
(e) Fert i l i t y i s cons i derabl y more di ffi cul t t o achi eve becaus e i t depends on t he preci s el y cont rol l ed admi ni s t rat i on of gonadotropins or gonadotropin-releasing hormone (GnRH).
(i) GnRH has been s ucces s ful i n res t ori ng ovul at i on i n women and s perm product i on i n men, but onl y i n cas es i n whi ch hypot hal ami c product i on of GnRH i s i mpai red but t he pi t ui t ary ret ai ns i t s abi l i t y t o s ecret e LH and FSH i n res pons e t o GnRH.
(ii) GnRH s t i mul at es LH and FSH product i on onl y i f i t i s admi ni s t ered i n a way t hat mi mi cs normal phys i ol ogi c s ecret i on; t hat i s , i t mus t be gi ven by regul ar pul s at i l e i nject i on every 90–120 mi nut es . (Cons t ant , rat her t han pul s at i l e, admi ni s t rat i on of GnRH has t he oppos i t e effect . It decreas es pi t ui t ary LH and FSH product i on).
3. Hyperprolactinemia. As hi gh as 50% of al l pi t ui t ary adenomas have been found t o s ecret e prol act i n. o
o
a. Etiology
Pa g e 1 8 6 7
ABC Ambe r CHM Conve rte r Tria l ve rsion, http://w w w .proce sste x t.com/a bcchm.html
(1) Prolactin-secreting pituitary adenomas (prolactinomas) are more common i n women t han i n men, us ual l y appeari ng duri ng t he reproduct i ve years and caus i ng mens t rual abnormal i t i es and gal act orrhea (the galactorrhea-amenorrhea syndrome). Men t end t o have l arger t umors at t he t i me of di agnos i s , whi ch us ual l y are s us pect ed becaus e of neurol ogi c i mpai rment and hypogonadi s m.
(2) Damage to the hypothalamus or pituitary stalk by t umors , granul omas , and ot her proces s es may prevent t he normal regul at ory effect of hypot hal ami c dopami ne on l act ot rope act i vi t y, res ul t i ng i n hypers ecret i on of prol act i n.
(3) Drugs t hat can i nhi bi t dopami ne act i vi t y and, t hus , i nt erfere wi t h i t s regul at i on of prol act i n s ecret i on i ncl ude ps ychot ropi c agent s (e.g., phenot hi azi nes , but yrophenones , and t ri cycl i c ant i depres s ant s ), ant i hypert ens i ves (e.g., met hyl dopa and res erpi ne), met ocl oprami de, ci met i di ne, and ot hers .
o
o
b. Clinical features
(1) Amenorrhea or mens t rual i rregul ari t y i s due t o t he i nhi bi t i on of hypot hal ami c GnRH product i on by prol act i n as wel l as t he di rect
Pa g e 1 8 6 8
ABC Ambe r CHM Conve rte r Tria l ve rsion, http://w w w .proce sste x t.com/a bcchm.html
effect s of t he prol act i n on t he ovari es .
(2) Galactorrhea i s a di rect res ul t of prol act i n exces s .
(3) Loss of potency and libido, wi t h l ow t es t os t erone l evel s , i s t he common endocri ne mani fes t at i on i n men.
o
o
c. Diagnosis
(1) Prolactin levels are el evat ed. A s erum prol act i n l evel great er t han 300 ng/mL s t rongl y s ugges t s t he pres ence of a prol act i noma. Funct i onal caus es of hyperprol act i nemi a s uch as drugs s el dom el evat e t he l evel above 100–200 ng/mL.
(2) CT scanning and MRI are us ed t o vi s ual i ze an adenoma.
o
o
d. T herapy. Treat ment of prol act i noma depends on t he s i ze of t he t umor and i t s mani fes t at i ons . A s mal l , non-enl argi ng t umor i n a woman wi t h i ns i gni fi cant gal act orrhea who does not des i re pregnancy may not requi re t reat ment . If pregnancy i s des i red, i f t he gal act orrhea or amenorrhea i s unaccept abl e, or i f t he t umor i s enl argi ng or caus i ng l ocal s ympt oms , t herapeut i c opt i ons i ncl ude surgery, administration of bromocriptine or another dopamine agonist, and radiation
Pa g e 1 8 6 9
ABC Ambe r CHM Conve rte r Tria l ve rsion, http://w w w .proce sste x t.com/a bcchm.html
therapy.
(1) Dopamine agonists (bromocri pt i ne, cabergol i ne, and pergol i depergol i de has been removed from t he market as of 3/2007 s o pel as e del et e) are remarkabl y effect i ve i n decreas i ng prol act i n l evel s , us ual l y t o normal , whi ch prompt l y rel i eves t he gal act orrhea and res t ores normal mens es and fert i l i t y; i t frequent l y reduces t umor s i ze as wel l .
(a) The s i de effect s of bromocri pt i ne i ncl ude naus ea, headache, di zzi nes s , and fat i gue.
(c) Becaus e of t he poor s urgi cal res ul t s i n pat i ent s wi t h l arge t umors , i ni t i al t reat ment wi t h bromocri pt i ne i s gi ven. If t he t umor s hri nks , t here i s a great er chance for s ucces s ful s urgery, or medi cal t reat ment al one may be cont i nued i ndefi ni t el y.
P.389
(2) T ranssphenoidal surgery i s cons i dered when pat i ent s cannot t ol erat e medi cal t herapy or have evi dence of compres s i ve s ympt oms . Large t umors wi t h s upras el l ar ext ens i on, however, us ual l y are not cured by s urgery.
Pa g e 1 8 7 0
ABC Ambe r CHM Conve rte r Tria l ve rsion, http://w w w .proce sste x t.com/a bcchm.html
(3) Radiation therapy may be us ed i n conjunct i on wi t h s urgery and dopami ne agoni s t s t o furt her reduce t umor s i ze and funct i on.
4. Acromegaly o
o
a. Etiology. Acromegal y i s caus ed by a pi t ui t ary adenoma t hat produces GH.
(1) In many cas es , t he adenoma i s l arge enough t o di s t ort t he s el l a t urci ca and can be s een on l at eral s kul l radi ographs ; i n ot her cas es , CT s can or MRI i s needed t o vi s ual i ze t he t umor, and i n a few cas es , no t umor can be vi s ual i zed.
(2) Immunohi s t ochemi cal s t ai ni ng s hows t hat t hes e adenomas are compos ed of s omat ot roph cel l s .
o
o
b. Clinical features. Exces s GH s ecret i on may caus e changes i n bone, s oft t i s s ues , and met abol i c proces s es .
(1) Bone and soft tissue changes
(a) In chi l dren, exces s GH s ecret i on may caus e i ncreas ed l i near growt h of l ong bones , res ul t i ng i n gigantism. Aft er
Pa g e 1 8 7 1
ABC Ambe r CHM Conve rte r Tria l ve rsion, http://w w w .proce sste x t.com/a bcchm.html
cl os ure of t he epi phys es at pubert y, t hes e changes cannot occur.
(b) In adul t s , s oft t i s s ue growt h and bone enl argement , es peci al l y i n t he acral areas of t he s kel et on, l ead t o diverse manifestations, many of which affect the patient' s appearance (Tabl e 9-1). Thes e changes are gradual and may not be obvi ous t o t he pat i ent or t he pat i ent 's fami l y unt i l t he pres ent appearance i s compared wi t h t hat on ol d phot ographs .
(2) Metabolic changes
(a) Decreased glucose tolerance, a res ul t of t he ant i -i ns ul i n act i ons of GH, i s common, al t hough overt di abet es occurs i n onl y 10% of pat i ent s wi t h acromegal y.
(b) A t endency t o devel op hyperphosphatemia i s caus ed by t he i ncreas ed t ubul ar reabs orpt i on of phos phat e t hat i s i nduced by GH.
o
o
c. Diagnosis. Cl i ni cal mani fes t at i ons rai s e t he s us pi ci on of acromegal y. Abnormal i t i es i n t he bl ood l evel s of GH, i ns ul i n-l i ke growt h fact or 1 (IGF-1 formerl y cal l ed s omat omedi n C), or bot h, confi rm t he di agnos i s .
Pa g e 1 8 7 2
ABC Ambe r CHM Conve rte r Tria l ve rsion, http://w w w .proce sste x t.com/a bcchm.html
(1) Becaus e gl ucos e s uppres s es GH i n normal s ubject s but not i n acromegal y, GH measurement after a glucose load may bes t di s t i ngui s h bet ween normal and acromegal i c s ubject s .
(2) IGF-1 i s a growt h fact or produced by t he l i ver under t he s t i mul at i on of GH. IGF-1 l evel s may be el evat ed i n pat i ent s wi t h acromegal y whos e GH l evel i s normal or equi vocal . P.390
El evat ed l evel s of IGF-1 provi de an addi t i onal i ndex of GH act i vi t y and furt her evi dence of t he di agnos i s .
TABLE 9-1 Skeletal and Soft Tissue Manifestations of Acromegaly Enl arg em ent of ha nds (es pec i al l y fi n ger
Pa g e 1 8 7 3
ABC Ambe r CHM Conve rte r Tria l ve rsion, http://w w w .proce sste x t.com/a bcchm.html
tip s) an d fee t Inc rea s ed ri n g, gl o ve, an d s ho e siz es Co ars eni ng of fac i al fea t ur es T hi c k s ki n fol
Pa g e 1 8 7 4
ABC Ambe r CHM Conve rte r Tria l ve rsion, http://w w w .proce sste x t.com/a bcchm.html
ds B row s an d nas ol a bi a l cre as e s Enl arg em ent of nos e Enl arg em ent of ma ndi bl e Pr og nat hi s m S pre
Pa g e 1 8 7 5
ABC Ambe r CHM Conve rte r Tria l ve rsion, http://w w w .proce sste x t.com/a bcchm.html
adi ng of t ee th Enl arg em ent of i nt ern al org ans H ear t, l un gs , liv er, s pl ee n, an d ki d ney s Ski n t hi cke ni n
Pa g e 1 8 7 6
ABC Ambe r CHM Conve rte r Tria l ve rsion, http://w w w .proce sste x t.com/a bcchm.html
g an d i nt ers titi al ed em a, wi t h sw el l i ng an d fi r mn es s of s of t tis s ue s Os t eo art hri t is Ent rap me nt
Pa g e 1 8 7 7
ABC Ambe r CHM Conve rte r Tria l ve rsion, http://w w w .proce sste x t.com/a bcchm.html
ne uro pat hi e s (es pec i al l y car pal t un nel s yn dro me ) Ra di o gra phi c cha ng es Enl arg em ent of sin us e s Tuf tin
Pa g e 1 8 7 8
ABC Ambe r CHM Conve rte r Tria l ve rsion, http://w w w .proce sste x t.com/a bcchm.html
g of di s t al ph al a ng es , cor tic al t hi cke ni n g
(3) Ini t i al l y, fas t i ng l evel s of GH and IGF-1 s houl d be meas ured. If t he GH l evel i s l es s t han 0.4 µg/L, and IGF-1 i s normal , acromegal y i s excl uded.
(4) If ei t her t he GH or IGF-1 l evel i s not l ow enough t o excl ude acromegal y, a 75-g oral gl ucos e t ol erance t es t s houl d be performed, wi t h GH meas urement s every 30 mi nut es for 2 hours . If t he GH l evel does not fal l bel ow 1.0 µg/L, a di agnos i s of acromegal y i s s ugges t ed; an el evat ed IGF-1 l evel i ncreas es t he l i kel i hood of t hi s di agnos i s .
(5) Fal s e-pos i t i ve GH res pons es (i .e., fai l ure of s uppres s i on) may occur aft er s urgery and i n pat i ent s wi t h di abet es mel l i t us , l i ver di s eas e,
Pa g e 1 8 7 9
ABC Ambe r CHM Conve rte r Tria l ve rsion, http://w w w .proce sste x t.com/a bcchm.html
ki dney di s eas e, and mal nut ri t i on. o
o
d. T herapy
(1) T ranssphenoidal pituitary adenomectomy caus es prompt normal i zat i on of GH l evel s i n mos t pat i ent s . Permanent cure i s common when t he adenoma i s s mal l but uncommon when t he t umor i s l arge and ext ends beyond t he s el l a t urci ca.
(2) Conventional radiation therapy l owers GH l evel s s l owl y; normal l evel s may not be reached unt i l 3–10 years aft er t reat ment , i f at al l .
(3) Octreotide, an anal og of s omat os t at i n, can be gi ven by s ubcut aneous i nject i on. It l owers GH l evel s i n many pat i ent s wi t h acromegal y and may be us eful i n pat i ent s i n whom s urgery and radi at i on t herapy have been uns ucces s ful .
(4) Pegvisomant
(a) Pegvisomant i s an anal og of GH t hat bi nds t o GH recept ors , bl ocki ng t he bi ndi ng of GH and t hus i nhi bi t i ng i t s act i ons . The res ul t i s a decreas e i n IGF-1, and l es s eni ng of t he effect s of exces s i ve GH i n acromegal y.
Pa g e 1 8 8 0
ABC Ambe r CHM Conve rte r Tria l ve rsion, http://w w w .proce sste x t.com/a bcchm.html
(b) Pegvi s omant i s gi ven by i nject i on once dai l y. It may be i ndi cat ed i n pat i ent s who have not res ponded adequat el y t o ot her t reat ment s .
B. Posterior pituitary disease
1. Diabetes insipidus. The t erm central diabetes insipidus i s us ed t o des cri be di s eas e due t o ant i di uret i c hormone (ADH) i ns uffi ci ency, and t he t erm nephrogenic diabetes insipidus i s us ed t o des cri be di s eas e due t o renal unres pons i venes s t o ADH. o
o
a. Etiology of cent ral di abet es i ns i pi dus
(1) Approxi mat el y 50% of cas es are idiopathic.
(2) Injury to the hypothalamic–pituitary area may res ul t from head t rauma, brai n t umors , and neuros urgi cal procedures .
(3) Less common causes i ncl ude sarcoidosis, syphilis, Hand-Schüller-Christian disease, and encephalitis.
o
o
b. Clinical features
Pa g e 1 8 8 1
ABC Ambe r CHM Conve rte r Tria l ve rsion, http://w w w .proce sste x t.com/a bcchm.html
(1) Polyuria, wi t h uri ne vol umes of 3–15 L dai l y, res ul t s from t he i nabi l i t y t o reabs orb free wat er and t o concent rat e uri ne i n t he abs ence of adequat e ADH.
(2) T hirst res ul t s , whi ch l ead t o increased fluid intake. A cons ci ous pat i ent wi t h a normal t hi rs t mechani s m and free acces s t o wat er wi l l mai nt ai n hydrat i on; t he di s eas e i n s uch a pat i ent i s an i nconveni ence rat her t han a t hreat t o l i fe. However, rapi d and l i fe-t hreat eni ng dehydrat i on may occur i n an i nfant or i n an uncons ci ous pat i ent .
(3) Laborat ory abnormal i t i es i ncl ude a dilute urine (os mol al i t y <200 mOs m/kg and s peci fi c gravi t y <1.005) and a hi gh-normal or s l i ght l y elevated plasma osmolality.
o
o
c. Differential diagnosis. In pat i ent s wi t h pol yuri a and di l ut e uri ne, cent ral di abet es i ns i pi dus mus t be di fferent i at ed from nephrogeni c di abet es i ns i pi dus and compul s i ve wat er dri nki ng.
(1) Nephrogenic diabetes insipidus i s a condi t i on i n whi ch t he renal t ubul es fai l t o res pond t o normal ci rcul at i ng l evel s of ADH.
(a) The condi t i on may be pri mary and fami l i al , s t art i ng i n i nfancy, or i t may occur l at er i n l i fe as a s econdary condi t i on i n as s oci at i on wi t h
Pa g e 1 8 8 2
ABC Ambe r CHM Conve rte r Tria l ve rsion, http://w w w .proce sste x t.com/a bcchm.html
hypokal emi a, hypercal cemi a, chroni c renal di s eas e, s i ckl e cel l anemi a, amyl oi dos i s , or t he us e of cert ai n drugs (e.g., l i t hi um, demecl ocycl i ne, and met hoxyfl urane).
(b) The cl i ni cal feat ures are t he s ame as t hos e caus ed by ADH defi ci ency. The di fference i s s een i n t he fai l ure of nephrogeni c di abet es i ns i pi dus t o res pond t o admi ni s t rat i on of ADH.
P.391
(2) Compulsive water drinking (psychogenic polydipsia) i s a pri mary ps ychi at ri c abnormal i t y t hat l eads t o pol yuri a and di l ut e uri ne. Di fferent i at i on from di abet es i ns i pi dus may be di ffi cul t . It i s mos t common i n young or mi ddl e-aged women who oft en have a hi s t ory of ps ychi at ri c di s orders .
o
o
d. Diagnosis
(1) Measurement of plasma osmolality. In unt reat ed pat i ent s , t hi s det ermi nat i on hel ps di s t i ngui s h t he caus es of pol yuri a. In di abet es i ns i pi dus , t he l os s of free wat er i s pri mary, and pl as ma os mol al i t y t ends t o be hi gh (280–310 mOs m/kg). In ps ychogeni c pol ydi ps i a, exces s i ve fl ui d i nt ake i s pri mary, and pl as ma os mol al i t y t ends t o be l ow
Pa g e 1 8 8 3
ABC Ambe r CHM Conve rte r Tria l ve rsion, http://w w w .proce sste x t.com/a bcchm.html
(255–280 mOs m/kg).
(2) Water deprivation test
(a) Method. Fl ui d i nt ake i s wi t hhel d unt i l uri ne os mol al i t y reaches a pl at eau (i .e., an hourl y i ncreas e of <30 mOs m/kg for 3 cons ecut i ve hours ). W hen uri ne os mol al i t y i s s t abl e, pl as ma os mol al i t y i s meas ured. Des mopres s i n, 2 µg (DDAVP, a s ynt het i c anal og of vas opres s i n) i s t hen i nject ed s ubcut aneous l y, and uri ne os mol al i t y i s meas ured agai n 1 hour l at er.
(b) Response. The res pons es t ypi cal of normal i ndi vi dual s and of pat i ent s wi t h part i al , compl et e, and nephrogeni c di abet es i ns i pi dus are s hown i n Tabl e 9-2. Pat i ent s wi t h part i al di abet es i ns i pi dus s how an i ncreas e i n uri ne os mol al i t y wi t h dehydrat i on, but t he i ncompl et enes s of t hei r res pons e i s demons t rat ed by a furt her i ncreas e aft er ADH i s i nject ed.
(3) Administration of hypertonic saline. Infus i on of a s ol ut i on (2.5% s odi um chl ori de gi ven i nt ravenous l y for 45 mi nut es at 0.25 mL/kg/mi n) aft er a wat er l oad (20 mL/kg i n 30–60 mi nut es ) caus es a s harp decreas e i n uri ne fl ow i n normal s ubject s becaus e of s t i mul at i on of ADH s ecret i on. Pat i ent s wi t h
Pa g e 1 8 8 4
ABC Ambe r CHM Conve rte r Tria l ve rsion, http://w w w .proce sste x t.com/a bcchm.html
di abet es i ns i pi dus cannot res pond t o t hi s s t i mul us . o
o
d. T herapy
(1) Desmopressin (DDAVP) can be admi ni s t ered oral l y, 0.1–1.2 mg dai l y i n 2 or 3 dos es . It al s o can be gi ven as a nas al s pray, and a parent eral preparat i on i s avai l abl e for us e i n acut el y i l l or pos t operat i ve pat i ent s .
(2) Chlorpropamide, an oral hypogl ycemi c agent , has t he addi t i onal effect of pot ent i at i ng t he act i on, t he s ecret i on, or bot h, of endogenous ADH. Thi s effect may be us ed i n t he t reat ment of di abet es i ns i pi dus .
(a) Pat i ent s who have at l eas t part i al ADH product i on oft en become as ympt omat i c when 250–500 mg of chl orpropami de i s t aken dai l y.
(b) Phys i ci ans and pat i ent s mus t wat ch for hypogl ycemi a, a pos s i bl e s i de effect .
(3) T hiazide diuretics have t he paradoxi cal effect of decreas i ng uri ne out put i n pat i ent s wi t h di abet es i ns i pi dus .
Pa g e 1 8 8 5
ABC Ambe r CHM Conve rte r Tria l ve rsion, http://w w w .proce sste x t.com/a bcchm.html
(a) The vol ume depl et i on i nduced by di uret i cs i ncreas es s odi um and wat er reabs orpt i on i n t he proxi mal t ubul e, t hereby bl unt i ng t he effect of t he defect i ve wat er abs orpt i on i n t he di s t al and col l ect i ng t ubul es .
(b) Thi azi des are onl y part i al l y effect i ve, decreas i ng uri ne vol ume by 30%–50%. However, unl i ke DDAVP, t hey are us eful for t reat i ng nephrogeni c di abet es i ns i pi dus becaus e t hei r act i on does not depend on di s t al t ubul ar res pons e t o ADH.
TABLE 9-2 Response to Water Deprivation Test In F cr ur e th a er s In e cr in e U a ri s Di n e a e in g O U n s ri o m n si ol e s al O
Pa g e 1 8 8 6
ABC Ambe r CHM Conve rte r Tria l ve rsion, http://w w w .proce sste x t.com/a bcchm.html
it y a b o v e 2 8 0 s mm O ol s al m it / y k in g R w e it s h p D o e n h s y e dr to at A io D n H N + or m al C - + o m
Pa g e 1 8 8 7
ABC Ambe r CHM Conve rte r Tria l ve rsion, http://w w w .proce sste x t.com/a bcchm.html
pl et e ce nt ra l di a b et es in si pi d us P + + ar ti al ce nt ra l di a b et es in si pi d us
Pa g e 1 8 8 8
ABC Ambe r CHM Conve rte r Tria l ve rsion, http://w w w .proce sste x t.com/a bcchm.html
N
- -
e p hr o g e ni c di a b et es in si pi d us ADH, ant i di u ret i c hormo ne. P.392
2. Syndrome of inappropriate secretion of ADH (SIADH) o
o
a. Etiology
Pa g e 1 8 8 9
ABC Ambe r CHM Conve rte r Tria l ve rsion, http://w w w .proce sste x t.com/a bcchm.html
(1) ADH product i on by malignant tumors, part i cul arl y oat cel l carci noma of t he l ung and carci noma of t he pancreas , was t he ori gi nal l y recogni zed caus e of SIADH.
(2) More commonl y, exces s ADH product i on by t he neurohypophys i al axi s or by di s eas ed t i s s ue i s caus ed by ot her di s eas e proces s es t hrough unknown mechani s ms . Thes e di s eas e proces s es i ncl ude pulmonary diseases (e.g., pneumoni a and t ubercul os i s ) and CNS disorders (e.g., s t roke, head i njury, and encephal i t i s ).
(3) Drugs (e.g., chl orpropami de, carbamazepi ne, vi ncri s t i ne, and cl ofi brat e) may s t i mul at e hypot hal ami c–neurohypophys eal ADH product i on.
o
o
b. Pathophysiology. ADH exces s caus es wat er ret ent i on and ext racel l ul ar fl ui d vol ume expans i on, whi ch i s t hen compens at ed for by i ncreas ed uri nary s odi um excret i on. Cl i ni cal l y, s i gni fi cant vol ume expans i on (i .e., edema or hypert ens i on) i s not pres ent , becaus e of t he nat ri ures i s . However, t he wat er ret ent i on and t he s odi um l os s bot h cont ri but e t o hyponatremia, whi ch is the hallmark of SIADH. If wat er i nt ake i s mi ni mi zed, t hi s s equence of event s does not occur, and s erum s odi um l evel s do not fal l .
o
o
c. Clinical features. Hyponatremia refers t o a
Pa g e 1 8 9 0
ABC Ambe r CHM Conve rte r Tria l ve rsion, http://w w w .proce sste x t.com/a bcchm.html
s erum s odi um l evel l es s t han 135 mEq/L.
(1) Sympt oms of l et hargy, confus i on, agi t at i on, headache, naus ea and vomi t i ng, and focal neurol ogi c abnormal i t i es are common when t he s odi um l evel decl i nes rapi dl y or when i t reaches a l evel l es s t han approxi mat el y 125 mEq/L.
(2) Sei zures and coma may occur wi t h more s evere hyponat remi a.
o
o
d. Diagnosis. The fol l owi ng condi t i ons are t he bas i s of di agnos i s .
(1) Hyponatremia i s pres ent , wi t h l ow s erum os mol al i t y.
(2) Daily urinary sodium excretion exceeds 20 mEq/L, des pi t e t he l ow s erum s odi um l evel s , and uri ne os mol al i t y i s hi gher t han s erum os mol al i t y. (Ot her caus es of hyponat remi a s uch as s odi um depl et i on caus e renal ret ent i on of s odi um, wi t h <20 mEq/L excret ed dai l y.)
(3) Condi t i ons t hat mi ght appropri at el y s t i mul at e ADH s ecret i on becaus e of vol ume depl et i on mus t be excl uded. Thes e i ncl ude adrenal i ns uffi ci ency, fl ui d l os s , edemat ous s t at es (e.g., heart fai l ure, nephros i s ,
Pa g e 1 8 9 1
ABC Ambe r CHM Conve rte r Tria l ve rsion, http://w w w .proce sste x t.com/a bcchm.html
ci rrhos i s ), and renal fai l ure. o
o
e. T herapy. The caus e of SIADH s houl d be t reat ed when pos s i bl e.
(1) Fluid restriction t o 500–1000 mL dai l y i s effect i ve i n i ncreas i ng t he s erum s odi um l evel and i s t he mai ns t ay of t reat ment . The l i mi t i ng fact or i s pat i ent adherence.
(2) If hyponat remi a i s s evere, hypertonic (3%) saline s houl d be admi ni s t ered t o el evat e t he s erum s odi um l evel above 120 mEq/L.
(a) Serum s odi um mus t not be i ncreas ed rapi dl y t o a l evel exceedi ng 125 mEq/L, or CNS damage may res ul t (s ee Chapt er 11 XI B 1).
(b) Sal t l oadi ng i s of onl y t emporary val ue becaus e t he addi t i onal s odi um i s s oon excret ed i n t he uri ne.
(3) If fl ui d res t ri ct i on cannot be enforced, 300 mg demeclocycline can be admi ni s t ered t hree or four t i mes dai l y. Demecl ocycl i ne i s an ant i bi ot i c wi t h t he us eful s i de effect of i nhi bi t i ng renal t ubul ar res pons e t o ADH.
II. Disorders of the Thyroid Gland
Pa g e 1 8 9 2
ABC Ambe r CHM Conve rte r Tria l ve rsion, http://w w w .proce sste x t.com/a bcchm.html
A. Thyroid function studies
1. Serum total thyroxine (T 4 ) determination meas ures t he t ot al bound (99.95%) and free (0.05%) T 4 i n t he ci rcul at i on. The s erum T 4 concent rat i on i s el evat ed i n hypert hyroi di s m and decreas ed i n hypot hyroi di s m. o
o
a. Serum T 4 concent rat i on i s i nfl uenced by thyroxine-binding globulin (T BG). Today free T 4 i s a more hel pful t es t .
(1) If t he bi ndi ng prot ei ns are el evat ed, t he t ot al T 4 concent rat i on i n t he bl ood i s hi gh, but t he concent rat i on of free T 4 (t he act i ve form of t he hormone at t he t i s s ue l evel ) remai ns normal . It i s regul at ed by t he normal l y funct i oni ng t hyroi d hormone–TSH feedback mechani s m.
(2) If t he concent rat i on of free T 4 i s normal , t he pat i ent i s eut hyroi d, and t he el evat ed l evel of t ot al T 4 i s mi s l eadi ng.
o
o
b. The convers e i s al s o t rue. TBG l evel s may be l owered by androgen t reat ment , ci rrhos i s , t he nephrot i c s yndrome, or congeni t al TBG defi ci ency. In t hi s cas e, t he concent rat i on of t ot al T 4 i s l ow, but t he pat i ent mai nt ai ns a normal l evel of free T 4 and i s eut hyroi d.
o
Pa g e 1 8 9 3
ABC Ambe r CHM Conve rte r Tria l ve rsion, http://w w w .proce sste x t.com/a bcchm.html
o
c. Therefore, t he t ot al T 4 concent rat i on al one i s not an adequat e t es t t o eval uat e t hyroi d funct i on. Ei t her t he T 4 concent rat i on mus t be meas ured i n conjunct i on wi t h a t es t t hat eval uat es prot ei n bi ndi ng [e.g., t he t ri i odot hyroni ne (T 3 ) upt ake t es t ] or free T 4 i t s el f mus t be meas ured.
2. Serum total T 3 determination meas ures t he concent rat i on of t he t ot al bound and free T 3 i n t he ci rcul at i on. The t ot al T 3 meas urement may gi ve t he s ame mi s l eadi ng res ul t s as t he t ot al T 4 meas urement i f t here i s an abnormal i t y i n bi ndi ng prot ei ns . T 3 hel ps di fferent i at e Graves ' di s eas e from ot her caus es of t hyrot oxi cos i s .
3. T 3 uptake test o
o
a. Method. Thi s t es t i s performed by combi ni ng i n a t ube t he pat i ent 's s erum, a known amount of radi ol abel ed T 3 , and an i ns ol ubl e bi nder of T 3 s uch as a s mal l pi ece of res i n. The bi ndi ng prot ei ns from t he pat i ent 's s erum and t he res i n compet e for t he l abel ed T 3 .
(1) If t he bi ndi ng prot ei ns are i ncreas ed, l es s T 3 bi nds t o t he res i n, and i f t he prot ei ns are decreas ed, more T 3 bi nds t o t he res i n.
(2) The res ul t , whi ch i s expres s ed as a percent age of l abel ed T 3 bound t o t he res i n, i s a meas ure of t he unoccupi ed bi ndi ng s i t es on
Pa g e 1 8 9 4
ABC Ambe r CHM Conve rte r Tria l ve rsion, http://w w w .proce sste x t.com/a bcchm.html
t he pat i ent 's t hyroi d hormone–bi ndi ng prot ei ns . o
o
b. Interpretation
(1) The T 3 res i n upt ake i s el evat ed i n hypert hyroi di s m. Thyroi d hormone i s i ncreas ed i n t he bl ood; t herefore, t he hormone bound t o prot ei n i s al s o i ncreas ed, l eavi ng fewer unoccupi ed bi ndi ng s i t es . Cons equent l y, t here i s i ncreas ed l abel ed T 3 bi ndi ng t o res i n. Convers el y, t he T 3 res i n upt ake i s decreas ed i n hypot hyroi di s m.
(2) In bot h i ns t ances , t he T 3 res i n upt ake vari es di rect l y wi t h changes i n t he t ot al T 4 and t ot al T 3 concent rat i ons and confi rms t he di agnos i s s ugges t ed by t he t ot al T 4 and t ot al T 3 l evel s .
(3) However, i f t he l evel s of t ot al T 4 , t ot al T 3 , or bot h, are i ncreas ed or decreas ed becaus e of abnormal i t i es of t he bi ndi ng prot ei ns , rat her t han hypot hyroi di s m or hypert hyroi di s m, t he T 3 res i n upt ake changes i n t he oppos i t e di rect i on. For exampl e, when bi ndi ng prot ei n i s i ncreas ed, t here i s an i ncreas e i n t ot al T 4 , as wel l as an i ncreas e i n unoccupi ed bi ndi ng s i t es t hat caus es a decreas e i n t he T 3 res i n upt ake.
Pa g e 1 8 9 5
ABC Ambe r CHM Conve rte r Tria l ve rsion, http://w w w .proce sste x t.com/a bcchm.html
4. Free T 4 index o
o
a. If t he t ot al T 4 l evel and T 3 res i n upt ake are known, an i ndex can be cal cul at ed t hat es t i mat es t he free T 4 l evel .
(1) The pat i ent 's T 3 res i n upt ake i s di vi ded by t he average normal T 3 res i n upt ake, and t he t ot al T 4 i s mul t i pl i ed by t hi s fract i on.
(2) The res ul t , whi ch i s cal l ed t he free T 4 i ndex, has approxi mat el y t he s ame normal range as t he t ot al concent rat i on of T 4 . Thi s proces s cons i ders t he effect s of abnormal i t i es of t hyroi d hormone–bi ndi ng prot ei ns on t he t ot al T 4 meas urement .
o
o
b. Example. A pat i ent has a t ot al T 4 of 15.0 µg/dL (normal = 4.5–12.5 µg/dL). If t he T 3 res i n upt ake i s 45%, t he free T 4 i ndex equal s 45% di vi ded by 30% (whi ch i s t he average normal upt ake) mul t i pl i ed by 15.0, or 22.5. Thi s res ul t s ugges t s a di agnos i s of hypert hyroi di s m. If t he T 3 res i n upt ake i s 15%, t he free T 4 i ndex i s 7.5 (15/30 × 15.0), s ugges t i ng t hat t he pat i ent i s eut hyroi d but has an i ncreas ed l evel of T 4 -bi ndi ng prot ei ns . P.393
Pa g e 1 8 9 6
ABC Ambe r CHM Conve rte r Tria l ve rsion, http://w w w .proce sste x t.com/a bcchm.html
TABLE 9-3 Symptoms of Hypothyroidism W e Mu akn s cl es s e ,
cra
l et mp har s gy, an d fat i gu e (†œs l ow i ng do wn †•) Dry W e s ki i gh n
t
an gai d
n
coa rs e hai r Puf Ho fy
ars
eye en l i d es s
Pa g e 1 8 9 7
ABC Ambe r CHM Conve rte r Tria l ve rsion, http://w w w .proce sste x t.com/a bcchm.html
s, fac e, an d ha nds ; sw ol l en l eg s Col Me d
nor
i nt rha ol e gi a ran ce Co He ns t ari i pa ng t i o l os n
s
5. Serum T SH measurement o
o
a. In pri mary hypothyroidism, meas uri ng t he s erum TSH concent rat i on i s a very s ens i t i ve t es t , becaus e i t us ual l y becomes el evat ed even before t hyroi d hormone l evel s decl i ne bel ow normal . TSH el evat i on i s caus ed by t he negat i ve feedback effect s of l ow t hyroi d hormone l evel s . In s econdary hypot hyroi di s m TSH wi l l be s uppres s ed or normal .
Pa g e 1 8 9 8
ABC Ambe r CHM Conve rte r Tria l ve rsion, http://w w w .proce sste x t.com/a bcchm.html o
o
b. In hyperthyroidism, t he el evat ed t hyroi d hormone concent rat i ons l ead t o s uppres s i on of s erum TSH t o l evel s bel ow normal . Thi s fi ndi ng i s a very s ens i t i ve i ndi cat i on of hypert hyroi di s m, becaus e TSH may be s uppres s ed even when t hyroi d hormone l evel s are not el evat ed above t he normal range.
6. Radioactive iodine uptake by t he t hyroi d gl and 24 hours aft er admi ni s t rat i on of t he i s ot ope i s i ncreas ed i n hypert hyroi di s m and decreas ed i n hypot hyroi di s m. Thi s t es t i s es peci al l y us eful i n det ect i ng forms of hypert hyroi di s m i n whi ch t he t hyroi d gl and i t s el f i s not s ynt hes i zi ng exces s hormone; t hat i s , t he hypert hyroi di s m as s oci at ed wi t h exogenous t hyroi d hormone admi ni s t rat i on, s ubacut e t hyroi di t i s , and ect opi c hormone product i on (e.g., caus ed by s t ruma ovari i ). In t hes e s i t uat i ons , bl ood hormone l evel s are hi gh, but t he radi oact i ve i odi ne upt ake i s l ow.
B. Hypothyroidism
1. Etiology o
o
a. Chronic autoimmune thyroiditis (s ee II D 2) i s t he mos t common caus e of s pont aneous hypot hyroi di s m i n t he Uni t ed St at es . Thi s condi t i on has a goi t rous form (Has hi mot o's di s eas e) and an at rophi c form; i n bot h forms ant i t hyroi d ant i bodi es are t ypi cal l y pres ent i n t he s erum.
o
Pa g e 1 8 9 9
ABC Ambe r CHM Conve rte r Tria l ve rsion, http://w w w .proce sste x t.com/a bcchm.html
o
b. Hypothyroidism frequently develops after the treatment of Graves' disease, and t he preval ence exceeds 50% i n pat i ent s t reat ed wi t h radi oact i ve i odi ne. However, hypot hyroi di s m al s o may occur aft er Graves ' di s eas e i s t reat ed by s ubt ot al t hyroi dect omy or ant i t hyroi d drugs .
o
o
c. Secondary hypothyroidism i s caus ed by any of t he condi t i ons t hat may affect t he hypot hal ami c–pi t ui t ary axi s and caus e hypopi t ui t ari s m (s ee I A 2 a).
o
o
d. Less common causes of hypot hyroi di s m i ncl ude congeni t al at hyreos i s , congeni t al bi ochemi cal defect s t hat prevent t hyroi d hormone product i on, and i ns ens i t i vi t y of t he t i s s ues t o t hyroi d hormone. Iodi ne defi ci ency i s an uncommon caus e of hypot hyroi di s m i n mos t hi ghl y devel oped count ri es , but i t i s common i n s ome areas of t he worl d.
2. Clinical features o
o
a. Symptoms (Tabl e 9-3)
(1) As met abol i s m s l ows becaus e of t he l es s ened effect s of t hyroi d hormone on t i s s ues , pat i ent s may experi ence weakness, lethargy, sleepiness and fatigue, and slowness of speech and thought.
Pa g e 1 9 0 0
ABC Ambe r CHM Conve rte r Tria l ve rsion, http://w w w .proce sste x t.com/a bcchm.html
(2) A puffy appearance, constipation, and a constant feeling of cold may be pres ent . Muscle cramps are common.
(3) Sl i ght -t o-moderat e weight gain refl ect s t he decreas ed met abol i s m, but mas s i ve wei ght gai n does not occur becaus e appet i t e t ends t o be di mi ni s hed.
(4) Edema of t he l arynx and mi ddl e ear may caus e voice changes and hearing loss i n s evere cas es .
(5) Exces s and irregular menstrual bleeding may be as s oci at ed wi t h anovul at ory cycl es .
o
o
b. Physical findings (Tabl e 9-4)
(1) Puffiness and nonpitting edema are caus ed by t he accumul at i on of muci nous mucopol ys acchari de-ri ch mat eri al i n t he t i s s ues . The t erm myxedema des cri bes t hi s phenomenon and i s s omet i mes us ed s ynonymous l y wi t h s evere hypot hyroi di s m.
TABLE 9-4 Physical Findings in Hypothyroidism
Pa g e 1 9 0 1
ABC Ambe r CHM Conve rte r Tria l ve rsion, http://w w w .proce sste x t.com/a bcchm.html
Thi Bra cke dyc ne ard d,
ia
puf fy fea t ur es Y el Sl o l ow w i s h ret ,
urn
dry of s ki de n
ep t en do n refl exe
s No Los npi s t t i of ng l at ed era em l a
por tio n of eye bro ws
Pa g e 1 9 0 2
ABC Ambe r CHM Conve rte r Tria l ve rsion, http://w w w .proce sste x t.com/a bcchm.html
Hy pot her mi a
P.394
TABLE 9-5 Effects of Hypothyroidism on Organ Systems Car Ner di o vou vas s cul s ys ar t e s ys m t e De m
cre
D as e ecr d eas me e
nt a
in
l
car fun di a ct i c
on
out Ps y
Pa g e 1 9 0 3
ABC Ambe r CHM Conve rte r Tria l ve rsion, http://w w w .proce sste x t.com/a bcchm.html
put chi P at ri eri c car cha di a ng l
es
eff (e. us i g., on ps y Re cho s pi s i s rat , ory de s ys pre te ssi m
on)
H Bl o ypo od ven Nor t i l a mo t i o chr n
om Pl i c
eur nor al
mo
eff cyt us i i c on an Ga em str ia oi n t es tin al t ra
Pa g e 1 9 0 4
ABC Ambe r CHM Conve rte r Tria l ve rsion, http://w w w .proce sste x t.com/a bcchm.html
ct C ons tip at i on
(2) The charact eri s t i c puffy, dul l appearance and t he slow return phase of t he Achi l l es and ot her deep tendon reflexes are perhaps t he mos t hel pful phys i cal fi ndi ngs i n s ugges t i ng hypot hyroi di s m.
o
o
c. Effects on organ systems. Al l organ s ys t ems are affect ed; s ome of t he mos t i mport ant changes are l i s t ed i n Tabl e 9-5.
o
o
d. Cretinism i s s evere hypothyroidism beginning in infancy. Cret i ni s m i s marked by ment al ret ardat i on and i mpai rment of phys i cal growt h and devel opment .
o
o
e. Myxedema coma may res ul t i f s evere hypot hyroi di s m goes unt reat ed. Thi s s eri ous condi t i on may occur gradual l y (over years ) or more acut el y i n res pons e t o preci pi t at i ng fact ors (e.g., i nfect i on, expos ure t o col d). The mort al i t y rat e i s 50%–75%. Hypot hermi a, hypogl ycemi a, s hock,
Pa g e 1 9 0 5
ABC Ambe r CHM Conve rte r Tria l ve rsion, http://w w w .proce sste x t.com/a bcchm.html
hypovent i l at i on, and i l eus may be pres ent i n addi t i on t o t he s everel y depres s ed s t at e of cons ci ous nes s .
3. Diagnosis o
o
a. Overt hypothyroidism i s s ugges t ed i n s evere cas es by t he charact eri s t i c s ympt oms and phys i cal fi ndi ngs ; however, mi l d cas es may es cape det ect i on unl es s l aborat ory t es t s are performed. Rout i ne l aborat ory s creeni ng i s es peci al l y recommended for newborns and women ol der t han 50 years of age. TSH l evel s s houl d al s o be meas ured i n el derl y pers ons wi t h nons peci fi c compl ai nt s , becaus e many hypot hyroi d s ympt oms s uch as fat i gue and cons t i pat i on may be mi s t aken for t he changes of agi ng.
(1) Serum T 4 and T 3 l evel s , as wel l as T 3 upt ake, are decreas ed.
(2) An i ncreas ed s erum concent rat i on of TSH i s t he earl i es t and mos t s ens i t i ve i ndi cat or of pri mary hypot hyroi di s m, and a s ens i t i ve TSH as s ay s houl d be us ed for s creeni ng. If t he TSH l evel i s el evat ed, t he di agnos i s may be confi rmed by t he fi ndi ng of a decreas ed s erum free T 4 or free T 4 i ndex.
o
o
b. Subclinical hypothyroidism i s a common condi t i on i n whi ch s erum TSH i s el evat ed, but s erum free T 4 or t he free T 4 i ndex i s normal rat her
Pa g e 1 9 0 6
ABC Ambe r CHM Conve rte r Tria l ve rsion, http://w w w .proce sste x t.com/a bcchm.html
t han decreas ed. Many affect ed pat i ent s progres s t o overt hypot hyroi di s m. The deci s i on t o t reat pat i ent s wi t h s ubcl i ni cal hypot hyroi di s m mus t be made on a cas e-by-cas e bas i s . A great er degree of TSH el evat i on, t he pres ence of s ympt oms t hat mi ght be caus ed by hypot hyroi di s m, and t he pres ence of ant i t hyroi d ant i bodi es are fact ors favori ng a deci s i on t o t reat .
4. T herapy o
o
a. T hyroid hormone preparations. Thyroi d ext ract deri ved from ani mal s ources and s ynt het i c preparat i ons cont ai ni ng bot h T 4 and T 3 have been us ed i n t he pas t and are s t i l l avai l abl e. However, s ynt het i c L-t hyroxi ne s odi um i s t he agent of choi ce.
(1) The admi ni s t ered T 4 i s s l owl y convert ed t o T 3 , and t he proport i ons of ci rcul at i ng T 4 and T 3 approxi mat e t hos e of eut hyroi d i ndi vi dual s .
(2) The peaks and val l eys of bl ood T 3 l evel s , whi ch are s een when exogenous T 3 i s gi ven, are avoi ded.
o
o
b. Initiation of treatment
(1) Pat i ent s wi t h s evere hypot hyroi di s m, ol der pat i ent s , and pat i ent s wi t h cardi ovas cul ar di s eas e may have an i ncreas ed
Pa g e 1 9 0 7
ABC Ambe r CHM Conve rte r Tria l ve rsion, http://w w w .proce sste x t.com/a bcchm.html
s ens i t i vi t y t o t hyroi d hormone and are at ri s k for acut e cardi ovas cul ar and ot her compl i cat i ons i f t he hypot hyroi di s m i s correct ed t oo qui ckl y. Therefore, t hes e pat i ent s s houl d be gi ven a very s mal l dos e of t hyroi d hormone i ni t i al l y P.395
(e.g., 25 µg of L-t hyroxi ne), whi ch i s i ncreas ed t o a ful l mai nt enance dos e duri ng a 6- t o 12-week peri od.
(2) Y ounger pat i ent s and pat i ent s wi t h l es s s evere hypot hyroi di s m may be s t art ed on a s l i ght l y hi gher dos e (50 µg of L-t hyroxi ne) and advanced t o a ful l repl acement dos e more qui ckl y (e.g., t he dos e may be rai s ed t o 100 µg i n 2 weeks and t o 125 µg or 150 µg i n anot her 2 weeks ).
o
o
c. Maintenance therapy. Mos t pat i ent s requi re approxi mat el y 0.7 t o 0.8 µg L-t hyroxi ne per pound of i deal body wei ght for phys i ol ogi c repl acement of t hyroi d hormone. W hen t hi s dos e i s t ol erat ed and s ympt oms of hypot hyroi di s m have res ol ved, t he dos e s houl d be furt her adjus t ed s o t hat s erum TSH i s mai nt ai ned i n t he normal range.
o
o
d. Myxedema coma has a hi gh mort al i t y rat e and must be treated rapidly, des pi t e t he ri s k as s oci at ed wi t h s udden hormone repl acement .
Pa g e 1 9 0 8
ABC Ambe r CHM Conve rte r Tria l ve rsion, http://w w w .proce sste x t.com/a bcchm.html
(1) Anci l l ary t reat ment i ncl udes t he t emporary us e of adrenal cort i cos t eroi ds and res pi rat ory s upport .
(2) L-Thyroxi ne i s gi ven i nt ravenous l y as a 500-µg bol us i nject i on, fol l owed by dai l y mai nt enance dos es .
C. Hyperthyroidism
1. Etiology o
o
a. Graves' disease (diffuse toxic goiter) is the most common cause of hyperthyroidism. It i s a form of aut oi mmune t hyroi d di s eas e i n whi ch an abnormal i mmunogl obul i n G (IgG) (t hyroi d-s t i mul at i ng i mmunogl obul i n) bi nds t o TSH recept ors on t he t hyroi d fol l i cul ar cel l s , caus i ng di ffus e enl argement of t he gl and and s t i mul at i on of t hyroi d hormone product i on. Graves ' di s eas e i s mos t common i n women bet ween t he ages of 20 and 50 years , al t hough ot hers may be affect ed.
o
o
b. T oxic nodular goiter (Plummer' s disease) and toxic thyroid adenoma are l es s common t han Graves ' di s eas e and us ual l y affect ol der i ndi vi dual s .
(1) Di s cret e areas of t he t hyroi d funct i on aut onomous l y, s ecret i ng exces s i ve amount s of t hyroi d hormone. The caus e i s unknown, but
Pa g e 1 9 0 9
ABC Ambe r CHM Conve rte r Tria l ve rsion, http://w w w .proce sste x t.com/a bcchm.html
act i vat i ng mut at i ons i n t he TSH recept or gene or i n t he s t i mul at ory G prot ei n t hat coupl e t he TSH recept or t o cycl i c adenos i ne monophos phat e (cAMP) format i on have been found i n many cas es .
(2) The pres ence of bot h normal t hyroi d t i s s ue and aut onomous l y funct i oni ng abnormal t i s s ue i s t he pat hognomoni c feat ure. Radionuclide scanning s hows t hat t he normal t i s s ue i s hypofunct i oni ng; t hi s occurs becaus e TSH i s s uppres s ed by t he exces s i ve l evel s of t hyroi d hormone produced by t he abnormal t i s s ue.
o
o
c. Subacute thyroiditis (s ee II D 1) may caus e t rans i ent hypert hyroi di s m.
o
o
d. Factitious hyperthyroidism may be caus ed by s urrept i t i ous i nges t i on of t hyroi d hormone by pat i ent s . Inadvert ent admi ni s t rat i on of exces s i ve dos es of t he hormone by phys i ci ans may have t he s ame res ul t .
o
o
e. Rare causes of hypert hyroi di s m i ncl ude exces s TSH product i on by pituitary tumors, teratomas of the ovary t hat produce t hyroi d hormone (struma ovarii), and overproduct i on of hormone by t he t hyroi d gl and fol l owi ng i odi ne i nges t i on, whi ch i s cal l ed jodbas edow.
2. Clinical features. Thyroi d hormone i ncreas es oxygen
Pa g e 1 9 1 0
ABC Ambe r CHM Conve rte r Tria l ve rsion, http://w w w .proce sste x t.com/a bcchm.html
cons umpt i on by t i s s ues , rai s i ng heat product i on and energy met abol i s m. It i nt eract s wi t h t he s ympat het i c nervous s ys t em i n a way t hat s eems t o i ncreas e t i s s ue s ens i t i vi t y t o cat echol ami nes and adrenergi c s t i mul i . In addi t i on, i t affect s prot ei n, fat , carbohydrat e, and vi t ami n met abol i s m (Tabl e 9-6). o
o
a. Metabolic changes i ncl ude an elevated basal metabolic rate and weight loss des pi t e i ncreas ed appet i t e and food i nt ake. Sweating and heat intolerance refl ect t he i ncreas ed heat product i on.
o
o
b. Cardiovascular effects
(1) The heart rat e i s i ncreas ed; sinus tachycardia i s common, wi t h rat es of 120 beat s /mi n or hi gher i n s evere cas es .
(2) Sys t ol i c bl ood pres s ure t ends t o be el evat ed and di as t ol i c bl ood pres s ure decreas ed, wi t h a wide pulse pressure.
(3) Myocardi al exci t abi l i t y i s i ncreas ed, and arrhyt hmi as [e.g., atrial fibrillation, premature ventricular contractions (PVCs)] may occur.
o
o
c. Gastrointestinal symptoms of l oos e s t ool s or diarrhea are common.
o
Pa g e 1 9 1 1
ABC Ambe r CHM Conve rte r Tria l ve rsion, http://w w w .proce sste x t.com/a bcchm.html o
d. Skin and hair changes. The skin is warm and moist becaus e of peri pheral vas odi l at i on and i ncreas ed s weat i ng. Fine, silky hair i s charact eri s t i c.
o
o
e. CNS effects i ncl ude emotional lability, restlessness, and fine tremor.
o
o
f. Muscle weakness and fatigue are common.
o
o
g. Ophthalmopathy
(1) Stare and lid lag (i .e., s l ow cl os i ng of t he upper l i d when t he eye moves downward, reveal i ng s cl era bet ween t he l i d and cornea) may occur i n any form of hypert hyroi di s m.
(2) True thyroid exophthalmos, however, i s s een onl y i n Graves ' di s eas e, occurri ng i n approxi mat el y 50% of cas es . The eye i s pus hed forward becaus e of muci nous and cel l ul ar infiltration of the orbit and extraocular muscles. There i s inflammation of the conjunctiva and s urroundi ng t i s s ues . Pat i ent s may compl ai n of tearing, eye irritation, pain, and double vision. In s evere cas es , vi s i on may be t hreat ened.
TABLE 9-6 Symptoms of Hyperthyroidism
Pa g e 1 9 1 2
ABC Ambe r CHM Conve rte r Tria l ve rsion, http://w w w .proce sste x t.com/a bcchm.html
Pal Mu pi t s cl at i e ons we akn es s an d fat i gu e Loo W a s e rm, s t o mo ol s i s t ,
s ki
di a n rrh Ner ea vou W e s ne i gh s s t l os s Fi n Em e
ot i
s i l k on y
al
hai l ab r
ilit
Fi n y e
Sw
t re eat mo i ng r
Pa g e 1 9 1 3
ABC Ambe r CHM Conve rte r Tria l ve rsion, http://w w w .proce sste x t.com/a bcchm.html
Re He s t l at es s i nt nes ol e s
ran ce
Am en orr he a, ol i go me nor rhe a
P.396
TABLE 9-7 Physical Findings in Hyperthyroidism Si n us t ac hyc ard ia Wi de pul se
Pa g e 1 9 1 4
ABC Ambe r CHM Conve rte r Tria l ve rsion, http://w w w .proce sste x t.com/a bcchm.html
pre ssu re Arr hyt hm i as (at ri al fi br illa tio n, PV Cs ) St a re an d lid l ag Exo pht hal mo s (on ly in Gra ves ' di s eas e)
Pa g e 1 9 1 5
ABC Ambe r CHM Conve rte r Tria l ve rsion, http://w w w .proce sste x t.com/a bcchm.html
Enl arg ed t hy roi d (di f fus el y or no dul ar) Thy roi d bru it Bri sk de ep t en do n refl exe s PV Cs , pre ma t ur e ven
Pa g e 1 9 1 6
ABC Ambe r CHM Conve rte r Tria l ve rsion, http://w w w .proce sste x t.com/a bcchm.html
t ri c ul a r con t ra ct i ons .
o
o
h. T hyroid storm i s a s udden exacerbat i on of t he s i gns and s ympt oms of hypert hyroi di s m.
(1) Preci pi t at i ng event s may i ncl ude i nt ercurrent i l l nes s , t rauma, s urgery, or chi l dbi rt h.
(2) The exacerbat i on of hypert hyroi di s m may be caus ed by an i ncreas e i n t he unbound fract i on of t hyroi d hormone, whi ch may occur wi t h s evere nont hyroi dal i l l nes s .
(3) Marked fever, t achycardi a, and agi t at i on are pres ent and may progres s t o s t upor and coma, wi t h vas cul ar col l aps e.
(4) The mort al i t y rat e i s 20%–40%.
3. Diagnosis o
Pa g e 1 9 1 7
ABC Ambe r CHM Conve rte r Tria l ve rsion, http://w w w .proce sste x t.com/a bcchm.html
o
a. Presenting symptoms of wei ght l os s , nervous nes s , pal pi t at i ons , mus cl e weaknes s , and di arrhea are charact eri s t i c.
o
o
b. Family history of t hyroi d di s eas e i s common.
o
o
c. Physical examination oft en reveal s a fi dget y, hyperki net i c pat i ent wi t h warm, moi s t s ki n; fi ne, s i l ky hai r; and a fi ne t remor of t he hands (Tabl e 9-7).
o
o
d. Laboratory studies s how an i ncreas e i n t he s erum concent rat i on of t ot al T 4 and free T 4 , s erum concent rat i on of t ot al T 3 and free T 3 , and T 3 res i n upt ake. The radi oact i ve i odi ne upt ake i s hi gh i n al l caus es of t hyrot oxi cos i s , wi t h t he except i on of t hyroi di t i s . Serum TSH l evel s are l ow.
4. T herapy. The adrenergic manifestations of hyperthyroidism (e.g., s weat i ng, t achycardi a, t remor) may be di mi ni s hed by β-blockers. Thes e drugs do not affect t hyroi d funct i on but provi de s ympt omat i c rel i ef unt i l t hyroi d hormone l evel s can be l owered t o normal . o
o
a. T reatment of Graves' disease. The mos t common met hods for t reat ment of Graves ' di s eas e are antithyroid drugs, subtotal thyroidectomy, and radioactive iodine.
(1) T hionamides
Pa g e 1 9 1 8
ABC Ambe r CHM Conve rte r Tria l ve rsion, http://w w w .proce sste x t.com/a bcchm.html
(a) Mechanism of action. Methimazole and propylthiouracil (PT U) i nhi bi t t he oxi dat i on of i odi de and t he coupl i ng of i odot yros i nes , t hereby decreas i ng t he s ynt hes i s of t hyroi d hormone. In addi t i on, PTU decreas es t he convers i on of T 4 t o T 3 i n peri pheral t i s s ues .
(b) Dosing. Ful l dos es (i .e., 30–40 mg met hi mazol e or 300–400 mg PTU) are gi ven dai l y unt i l t he pat i ent i s eut hyroi d.
(i) Al t hough bl ockade of hormone s ynt hes i s i s rapi d, cl i ni cal i mprovement occurs onl y aft er a few weeks or mont hs , becaus e a l arge pool of s t ored hormone cont i nues t o be rel eas ed from t he t hyroi d.
(ii) Aft er cl i ni cal i mprovement , t he dos e i s t apered t o t he l owes t dos e t hat mai nt ai ns eut hyroi di s m, and t he drug i s cont i nued for 1–2 years . Treat ment i s t hen di s cont i nued i n t he hope t hat a l as t i ng or permanent remi s s i on has occurred.
(iii) PTU i s t he drug of choi ce i n pregnancy.
Pa g e 1 9 1 9
ABC Ambe r CHM Conve rte r Tria l ve rsion, http://w w w .proce sste x t.com/a bcchm.html
(c) Drug toxicity
(i) Skin rash or joint pain occurs i n 3%–5% of pat i ent s , neces s i t at i ng a s wi t ch t o t he al t ernat i ve drug.
(ii) Agranulocytosis occurs i n fewer t han 0.5% of pat i ent s but i s l i fe-t hreat eni ng. For earl y det ect i on of agranul ocyt os i s , pat i ent s s houl d be i ns t ruct ed t o s t op t he drug i mmedi at el y i f fever, s ore t hroat , mout h ul cers , or ot her unexpl ai ned P.397
s ympt oms occur. Treat ment s houl d be res umed onl y aft er exami nat i on s hows a normal whi t e bl ood cel l (W BC) count .
(iii) Liver toxicity
(d) Advantages of medical treatment with antithyroid drugs
(i) Hos pi t al i zat i on, s urgery, and anes t hes i a are avoi ded.
(ii) There i s l es s l i kel i hood of t he occurrence of pos t t reat ment
Pa g e 1 9 2 0
ABC Ambe r CHM Conve rte r Tria l ve rsion, http://w w w .proce sste x t.com/a bcchm.html
hypot hyroi di s m t han i n pat i ent s t reat ed wi t h radi oact i ve i odi ne.
(e) Disadvantages of antithyroid drugs
(i) Permanent remi s s i on occurs i n fewer t han 50% of pat i ent s t reat ed.
(ii) Succes s ful t reat ment depends on pat i ent adherence, whi ch i s l es s of a probl em when t reat ment i s by s urgery or radi oact i ve i odi ne.
(3) Radioactive iodine
(a) Method of treatment. A s i ngl e dos e of i odi ne 131 (
131
I) caus es a decreas e i n
funct i on and s i ze of t he t hyroi d gl and i n 6–12 weeks . Approxi mat el y 75% of pat i ent s wi t h Graves ' di s eas e are made eut hyroi d by a s i ngl e dos e; t hos e who are s t i l l t hyrot oxi c aft er 12 weeks are gi ven a s econd dos e. Addi t i onal dos es can be gi ven i f needed. Event ual l y, al mos t al l pat i ent s are cured i n t hi s way.
(b) Advantages of radioactive iodine
(i) Hos pi t al i zat i on, s urgery, and anes t hes i a are avoi ded.
Pa g e 1 9 2 1
ABC Ambe r CHM Conve rte r Tria l ve rsion, http://w w w .proce sste x t.com/a bcchm.html
(ii) The rat e of cure approaches 100%.
(iii) Li t t l e pat i ent adherence i s requi red.
(c) Disadvantages of radioactive iodine
(i) Mul t i pl e t reat ment s may be needed.
(ii) Hypot hyroi di s m, t he t reat ment of whi ch requi res l ong-t erm pat i ent adherence, occurs i n approxi mat el y 10% of pat i ent s aft er 1 year and cont i nues t o devel op at a rat e of 2%–3% each year. Aft er 10–15 years , more t han 50% of pat i ent s are hypot hyroi d. Thi s compl i cat i on i s eas i l y t reat ed, however, wi t h a s i ngl e dai l y dos e of L-t hyroxi ne s odi um.
(iii) There i s a s l i ght ri s k of genet i c effect s (comparabl e i n magni t ude t o t he effect s of a bari um enema or i nt ravenous urogram) i n fut ure offs pri ng. However, no i ncreas e i n t he ri s k of l eukemi a, t hyroi d cancer, or ot her mal i gnanci es has been
Pa g e 1 9 2 2
ABC Ambe r CHM Conve rte r Tria l ve rsion, http://w w w .proce sste x t.com/a bcchm.html
found i n pat i ent s t reat ed wi t h radi oact i ve i odi ne.
(3) Subtotal thyroidectomy
(a) Advantages of surgery
(i) Cure of hypert hyroi di s m i s rapi d (once a eut hyroi d s t at e has been produced by ant i t hyroi d drugs ).
(ii) The s ucces s rat e i s hi gh; mos t pat i ent s are cured, and fewer become hypot hyroi d aft er s urgery t han aft er t reat ment wi t h radi oact i ve i odi ne.
(iii) Pat i ent adherence i s requi red for a s hort er peri od t han i t i s i n prol onged ant i t hyroi d drug t reat ment .
(b) Disadvantages of surgery
(i) Pat i ent s us ual l y mus t be hos pi t al i zed, and s urgi cal and anes t het i c ri s ks are i ncurred.
(ii) Surgi cal compl i cat i ons i ncl ude hypoparat hyroi di s m and recurrent
Pa g e 1 9 2 3
ABC Ambe r CHM Conve rte r Tria l ve rsion, http://w w w .proce sste x t.com/a bcchm.html
l aryngeal nerve paral ys i s .
(4) Choice of therapy
(a) Radi oact i ve i odi ne i s t he t reat ment of choi ce for mos t pat i ent s ol der t han 30–40 years and i s frequent l y us ed i n younger i ndi vi dual s .
(b) In younger pat i ent s , t he choi ce i s more di ffi cul t . In pat i ent s wi t h mi l d cl i ni cal mani fes t at i ons , s l i ght l y t o moderat el y el evat ed t hyroi d hormone l evel s , and an onl y moderat el y enl arged t hyroi d, a t ri al of ant i t hyroi d drugs i s reas onabl e, becaus e pat i ent s wi t h mi l d di s eas e have a bet t er chance for a l as t i ng remi s s i on.
(c) Surgery may be a bet t er choi ce for pat i ent s wi t h l arge goi t ers and s evere di s eas e and for pat i ent s who are unwi l l i ng t o t ake ant i t hyroi d drugs for a prol onged peri od.
o
o
b. T reatment of toxic nodular goiter and toxic adenoma. Becaus e ant i t hyroi d drug t herapy does not l ead t o permanent remi s s i on i n affect ed pat i ent s , t he opt i ons for t reat ment are s urgery and radi oact i ve i odi ne. P.398
Pa g e 1 9 2 4
ABC Ambe r CHM Conve rte r Tria l ve rsion, http://w w w .proce sste x t.com/a bcchm.html
(1) T hyroidectomy or removal of a hyperfunct i oni ng nodul e rapi dl y cures t he hypert hyroi di s m and rel i eves s ympt oms of pres s ure or t racheal or es ophageal obs t ruct i on t hat may be caus ed by a l arge goi t er.
(2) Radioactive iodine t reat ment requi res much l arger dos es i n pat i ent s wi t h t oxi c nodul ar goi t er t han i n pat i ent s wi t h Graves ' di s eas e becaus e t he affect ed t hyroi d cel l s are rel at i vel y radi ores i s t ant . However, becaus e t he unaffect ed t hyroi d cel l s are funct i onal l y s uppres s ed, t hey do not t rap
131
I and are
s pared t he effect s of radi at i on; t herefore, hypot hyroi di s m aft er radi oact i ve i odi ne t reat ment i s l es s common i n t hes e pat i ent s . o
o
c. T reatment of thyroid storm
(1) Cont rol t he adrenergi c s t i mul at i on wi t h β-bl ockers . Tradi t i onal l y propranol ol has been us ed, whi ch al s o i ncreas es t he peri pheral convers i on of T 4 → T 3 .
(2) Thi onami des are us ed fi rs t t o bl ock new s ynt hes i s of t hyroi d hormone.
(3) Iodi ne may be gi ven aft er t hi onami des are
Pa g e 1 9 2 5
ABC Ambe r CHM Conve rte r Tria l ve rsion, http://w w w .proce sste x t.com/a bcchm.html
admi ni s t ered t o i nhi bi t t he rel eas e of preformed hormones . If i odi ne i s gi ven pri or t o t hi oami des i t may provi de s ubs t rat e for furt her s ynt hes i s of t he hormone.
(4) Cort i cos t eroi ds are al s o us ed t o bl ock convers i on of T 4 → T 3 i n peri pheral t i s s ues .
D. Thyroiditis
1. Subacute thyroiditis (al s o cal l ed granul omat ous t hyroi di t i s or de Quervai n's t hyroi di t i s ) o
o
a. Etiology. The caus e of s ubacut e t hyroi di t i s general l y i s cons i dered t o be vi ral . Mumps and coxs acki evi rus , among ot hers , have been s us pect ed.
o
o
b. Clinical features
(1) Early symptoms. A prodrome of mal ai s e, upper res pi rat ory s ympt oms , and fever t hat l as t s 1 or 2 weeks may occur. Then, t he t hyroi d gl and becomes enl arged, fi rm, and t ender, wi t h pai n radi at i ng t o t he ears , neck, or arms .
(2) Hyperthyroidism. Leaki ng of t hyroi d hormone from damaged foll i cl es i nt o t he ci rcul at i on may l ead t o hypert hyroi di s m.
Pa g e 1 9 2 6
ABC Ambe r CHM Conve rte r Tria l ve rsion, http://w w w .proce sste x t.com/a bcchm.html
(3) Disease course. The t hyroi d pai n and hypert hyroi di s m s ubs i de i n a few weeks or mont hs . The gl and us ual l y ret urns t o normal s i ze; i f enl argement pers i s t s , chroni c aut oi mmune t hyroi di t i s s houl d be s us pect ed. A s el f-l i mi t ed peri od of hypothyroidism may occur aft er t he rel eas ed hormone has been met abol i zed but before new hormone product i on has res umed.
o
o
c. Diagnosis. W hen t he t hyroi d becomes acut el y s wol l en, t ender, and pai nful , es peci al l y i f s ympt oms of hypert hyroi di s m are pres ent , s ubacut e t hyroi di t i s i s s us pect ed. The di agnos i s i s confi rmed by a very low radioactive iodine uptake in the face of high serum T 4 and T 3 levels. The radi oact i ve i odi ne upt ake i s l ow becaus e t he fol l i cul ar cel l s are i njured and unabl e t o t rap i odi ne and becaus e t he hi gh l evel s of ci rcul at i ng t hyroi d hormone s uppres s TSH.
o
o
d. T herapy. Treat ment i s symptomatic, becaus e t he di s eas e i s s el f-l i mi t ed.
(1) As pi ri n, nons t eroi dal ant i -i nfl ammat ory drugs (NSAIDs ), and adrenal cort i cos t eroi ds (i n s evere cas es ) rel i eve t he pai n and t endernes s .
(2) β-Bl ocki ng drugs can be us ed t o rel i eve s ympt oms of hypert hyroi di s m.
Pa g e 1 9 2 7
ABC Ambe r CHM Conve rte r Tria l ve rsion, http://w w w .proce sste x t.com/a bcchm.html
2. Chronic autoimmune thyroiditis (Hashimoto' s thyroiditis) o
o
a. Etiology. Chroni c t hyroi di t i s i s a common aut oi mmune di s order t hat pri mari l y affect s women. Ant i t hyroi d ant i bodi es are pres ent i n mos t pat i ent s .
o
o
b. Clinical features
(1) T hyroid gland enlargement, t he mai n cl i ni cal mani fes t at i on, i s t he res ul t of aut oi mmune damage t hat l eads t o l ymphocyt i c i nfi l t rat i on, fi bros i s , and a weakened abi l i t y of t he t hyroi d t o produce hormone
(Onl i ne Fi gure 9-1).
ONLINE FIGURE 9-1 Pat hogenes i s of chroni c aut oi mmune t hyroi di t i s . T 3 , t ri i odot hyroni ne; T, t hyroxi ne; TSH, t hyroi d-s t i mul at i ng hormone. (Adapt ed from Adl i n EV. Endoc ri nol ogy Sc i enc e and Medi c i ne. Phi l adel phi a: Li ppi ncot t W i l l i ams & W i l ki ns , 2001:32. )
(2) Hypothyroidism i s pres ent i n many pat i ent s when t he di s eas e i s fi rs t di agnos ed
Pa g e 1 9 2 8
ABC Ambe r CHM Conve rte r Tria l ve rsion, http://w w w .proce sste x t.com/a bcchm.html
and may affect addi t i onal pat i ent s as t he di s eas e progres s es .
(3) Pain and tenderness of the gland s omet i mes occur, as i n s ubacut e t hyroi di t i s .
o
o
c. Diagnosis. Chroni c t hyroi di t i s i s s us pect ed i n any pat i ent wi t h a fi rm, nont oxi c goi t er; a hi gh t i t er of ant i t hyroi d peroxi das e ant i bodi es or ant i t hyrogl obul i n ant i bodi es , or bot h, i s confi rmat ory. Thyroi d funct i on t es t s us ual l y are normal unl es s t he pat i ent has hypot hyroi di s m.
o
o
d. T herapy. Treat ment wi t h L-t hyroxi ne s odi um oft en decreas es t he s i ze of t he goi t er and, t herefore, i s us eful even i n pat i ent s wi t h normal t hyroi d funct i on. If hypot hyroi di s m i s pres ent , t hi s t reat ment i s , of cours e, es s ent i al .
P.399
3. Painless thyroiditis (al s o cal l ed s i l ent t hyroi di t i s or pos t part um t hyroi di t i s ). Thi s s yndrome res embl es s ubacut e t hyroi di t i s i n s ome ways and chroni c t hyroi di t i s i n ot hers . o
o
a. Li ke s ubacut e t hyroi di t i s , pai nl es s t hyroi di t i s i s as s oci at ed wi t h t rans i ent , s el f-l i mi t ed hypert hyroi di s m, oft en wi t h t hyroi d gl and enl argement , and a l ow radi oact i ve i odi ne upt ake. Thyroi d pai n and t endernes s are abs ent , however.
Pa g e 1 9 2 9
ABC Ambe r CHM Conve rte r Tria l ve rsion, http://w w w .proce sste x t.com/a bcchm.html
(1) Recogni t i on of t hi s caus e of s el f-l i mi t ed hypert hyroi di s m i s i mport ant , becaus e, i n t he abs ence of t hyroi d pai n t o s ugges t t hyroi di t i s , t he s yndrome coul d eas i l y be mi s t aken for Graves ' di s eas e and be t reat ed i nappropri at el y.
(2) The l ow radi oact i ve i odi ne upt ake i s t he mos t us eful fi ndi ng for di s t i ngui s hi ng pai nl es s t hyroi di t i s from Graves ' di s eas e.
o
o
b. Li ke chroni c t hyroi di t i s , pai nl es s t hyroi di t i s i s charact eri zed by l ymphocyt i c i nfi l t rat i on of t he t hyroi d and i s cons i dered t o be an aut oi mmune di s eas e. However, al t hough ant i t hyroi d ant i bodi es may be pres ent , t he t i t ers are l ower t han i n chroni c t hyroi di t i s .
o
o
c. Pai nl es s t hyroi di t i s i s common aft er del i very; postpartum thyroiditis occurs aft er 5%–10% of pregnanci es .
4. Rare forms of thyroiditis o
o
a. Suppurative thyroiditis i s caus ed by pyogeni c bact eri al i nfect i on. Thi s t hyroi d condi t i on i s t reat ed wi t h ant i bi ot i cs and s urgi cal drai nage, i f neces s ary.
o
o
b. In Riedel' s struma (fi brous t hyroi di t i s ), fi brous
Pa g e 1 9 3 0
ABC Ambe r CHM Conve rte r Tria l ve rsion, http://w w w .proce sste x t.com/a bcchm.html
connect i ve t i s s ue repl aces normal t hyroi d t i s s ue and i nfi l t rat es s urroundi ng s t ruct ures . Surgery i s i ndi cat ed t o excl ude cancer and t o rel i eve t racheal compres s i on.
E. Thyroid nodules T hyroid nodules are pres ent i n 1% of i ndi vi dual s i n t hei r 20s and i n 5% of i ndi vi dual s i n t hei r 60s ; cancer i s found i n approxi mat el y 5% of t hes e nodul es . Thyroi d nodul es may be t rue adenomas , cys t s , l ocal i zed areas of chroni c t hyroi di t i s , col l oi d nodul es , hemorrhagi c necrot i c t i s s ue, or carci noma.
1. Diagnosis o
o
a. Risk assessment
(1) Radiation treatment of t he head or neck i n chi l dhood i s as s oci at ed wi t h an i ncreas ed preval ence of t hyroi d nodul es and t hyroi d cancer i n adul t l i fe.
(2) Sex. A hi gher percent age of nodul es are mal i gnant i n men t han i n women (al t hough nodul es are much more common i n women).
(3) Age. A hi gher percent age of nodul es are mal i gnant i n younger i ndi vi dual s (al t hough nodul es are much more common i n ol der i ndi vi dual s ). In chi l dren, 50% of nodul es are mal i gnant .
Pa g e 1 9 3 1
ABC Ambe r CHM Conve rte r Tria l ve rsion, http://w w w .proce sste x t.com/a bcchm.html
(4) Disease course. Mal i gnancy i s s ugges t ed by recent growt h of t he nodul e or by cont i nui ng growt h des pi t e s uppres s i ve t herapy wi t h L-t hyroxi ne. Mal i gnancy i s l es s l i kel y i f t he nodul e di s appears aft er as pi rat i on of cys t fl ui d, i f t he nodul e i s vi s i bl e as a “warm†• or “hot ― s pot on s ci nt i s can (i .e., as demons t rat ed by i t s upt ake of radi oact i ve i odi ne), or i f t he nodul e s hri nks wi t h s uppres s i ve t herapy.
(5) Physical examination
(a) Mal i gnancy i s s ugges t ed when t he nodul e i s fi xed i n pl ace and no movement occurs on s wal l owi ng.
(b) Unus ual l y fi rm cons i s t ency, i rregul ari t y of t he nodul e, or regi onal l ymph node enl argement al s o s ugges t s mal i gnancy.
(c) Mal i gnancy i s l es s l i kel y i f t here are mul t i pl e nodul es or i f t he nodul e i s l es s t han 1 cm i n di amet er.
o
o
b. Laboratory evaluation
(1) Check thyroid function tests.
Pa g e 1 9 3 2
ABC Ambe r CHM Conve rte r Tria l ve rsion, http://w w w .proce sste x t.com/a bcchm.html
(2) Obt ai n ultrasound. Thi s can oft en be combi ned wi t h fi ne-needl e as pi rat i on bi ops y.
(2) Fine-needle aspiration biopsy i s s afe and eas i l y performed i n an offi ce s et t i ng. Cel l s , not s ect i ons of t i s s ue, are obt ai ned and mus t be eval uat ed by a s ki l l ed cyt opat hol ogi s t . The overal l res ul t s and predi ct i ve val ues are s hown i n Tabl e 9-8.
(4) Radionuclide thyroid scintiscanning i dent i fi es t he nodul e as “hot ,― “warm,― or “col d.―
(a) Becaus e mos t cancers appear on s can as col d areas , onl y col d nodul es are cons i dered t o have a s i gni fi cant ri s k of mal i gnancy.
(b) Of al l nodul es , 70%–90% are col d, and mos t of t hes e are beni gn. Therefore, s canni ng may i ndi cat e a great l y reduced ri s k of mal i gnancy i n a nodul e t hat i s warm or hot , but i t does not yi el d much addi t i onal i nformat i on on t he ri s k of mal i gnancy i n a nodul e t hat i s col d.
2. Management. The goal of management i s s urgi cal removal of t he nodul es wi t h a hi gh probabi l i t y of mal i gnancy and careful obs ervat i on of t he ot hers , s omet i mes wi t h at t empt ed
Pa g e 1 9 3 3
ABC Ambe r CHM Conve rte r Tria l ve rsion, http://w w w .proce sste x t.com/a bcchm.html
P.400
s uppres s i on by L-t hyroxi ne. An al gori t hm for t he management of t hyroi d nodul es i s s hown i n Fi gure 9-2.
TABLE 9-8 Results of Fine Needle Aspiration Biopsy of Thyroid Nodules % Po Cyt
siti
olo % ve gic of for Fin No Ca din dul nc gs es er Be
75 1â
ni g
€“5
n Sus 20 20 pi ci ous or i nd et e rmi na nt Mal 5
97
i gn ant
F. Thyroid cancer
Pa g e 1 9 3 4
ABC Ambe r CHM Conve rte r Tria l ve rsion, http://w w w .proce sste x t.com/a bcchm.html
1. Epidemiology o
o
a. Thyroi d cancer i s common; i t i s found at aut ops y i n approxi mat el y 5% of pat i ent s wi t h no known t hyroi d di s eas e. However, deat h due t o t hyroi d cancer i s uncommon—approxi mat el y 1200 i ndi vi dual s di e of t hi s condi t i on each year i n t he Uni t ed St at es .
o
o
b. Thes e cont radi ct ory obs ervat i ons are expl ai ned bes t by t he behavi or of t hyroi d cancer. It i s us ual l y i ndol ent and t ends t o remai n l ocal i zed t o t he t hyroi d for many years , whi ch i s t he reas on for t he l ow mort al i t y rat e.
2. Etiology o
o
a. Radiation exposure. Inci dence of t hyroi d cancer i s i ncreas ed i n at omi c bomb s urvi vors and i n i ndi vi dual s who recei ved radi at i on t herapy t o t he neck (e.g., for enl arged t hymus or enl arged t ons i l s ) i n chi l dhood.
o
o
b. Genetic factors. One form of t hyroi d cancer, medul l ary carci noma, may be fami l i al .
o
o
c. T SH can i nduce t hyroi d cancer i n ani mal s and may s t i mul at e t he growt h of many human t hyroi d
Pa g e 1 9 3 5
ABC Ambe r CHM Conve rte r Tria l ve rsion, http://w w w .proce sste x t.com/a bcchm.html
cancers .
FIGURE 9-2 Al gori t hm for eval uat i on and management of pat i ent s wi t h a t hyroi d nodul e. (From Adl i n EV. Endoc ri nol ogy Sc i enc e and Medi c i ne. Phi l adel phi a: Li ppi ncot t W i l l i ams & W i l ki ns , 2001:48. Adapt ed from Maz zaferri EL. Management of a s ol i t ary t hyroi d nodul e. N Engl J Med 1993;328:553–559 ; and Si nger PA, Cooper DS, Dani el s GH, et al . Treat ment gui del i nes for pat i ent s wi t h t hyroi d nodul es and wel l -di fferent i at ed t hyroi d cancer. Arc h I nt ern Med 1996;156:2165–2172. ) P.401
3. T ypes. Thyroi d cancer may mani fes t as a s ol i t ary t hyroi d nodul e or, l es s commonl y, as mul t i pl e nodul es or a mas s i n t he neck. Thes e t umors occas i onal l y caus e
Pa g e 1 9 3 6
ABC Ambe r CHM Conve rte r Tria l ve rsion, http://w w w .proce sste x t.com/a bcchm.html
hoars enes s , s ympt oms of t racheal or es ophageal compres s i on (e.g., dys pnea, dys phagi a), or pai n. o
o
a. Papillary carcinoma, whi ch account s for 80% of al l t hyroi d cancer, affect s t he younges t age group—50% of pat i ent s are younger t han 40 years of age.
(1) The neopl as m cons i s t s of col umnar cel l s i n fol ds (t he papi l l ae). It t ends t o grow s l owl y, oft en remai ni ng l ocal i zed t o t he t hyroi d for years , and event ual l y s preads vi a t he l ymphat i c s ys t em t o ot her part s of t he t hyroi d and t o regi onal nodes .
(2) There are few recurrences aft er t reat ment , es peci al l y i n young pat i ent s wi t h s mal l pri mary t umors .
o
o
b. Follicular carcinoma, whi ch cons t i t ut es 10% of al l t hyroi d cancers , hi s t ol ogi cal l y may res embl e normal t hyroi d t i s s ue. Fol l i cul ar carci noma i s more mal i gnant t han papi l l ary cancer and oft en s preads t o bone, t he l ungs , and t he l i ver. The 10-year s urvi val rat e i s 50%.
o
o
c. Medullary carcinoma, whi ch account s for 5% of al l t hyroi d cancers , ari s es from t he parafol l i cul ar cel l s (or C cel l s ) of t he t hyroi d. It has a hyal i ne s t roma, whi ch may s t ai n for amyl oi d.
Pa g e 1 9 3 7
ABC Ambe r CHM Conve rte r Tria l ve rsion, http://w w w .proce sste x t.com/a bcchm.html
(1) Approxi mat el y 20% of t hes e carci nomas are fami l i al and may be a component of MEN II (Si ppl e's s yndrome), a s yndrome of medul l ary t hyroi d carci noma and pheochromocyt oma.
(2) Thi s t umor oft en produces cal ci t oni n and occas i onal l y produces ot her hormones .
(3) It i s more mal i gnant t han fol l i cul ar carci noma, wi t h bot h l ocal l ymphat i c and di s t ant hemat ogenous s pread.
o
o
d. Anaplastic carcinoma, whi ch account s for 5% of t hyroi d cancers , us ual l y affect s pat i ent s ol der t han 50 years of age and i s hi ghl y mal i gnant . It i nvades rapi dl y, met as t as i zes wi del y, and us ual l y caus es deat h wi t hi n a few mont hs .
4. T herapy. Papi l l ary, fol l i cul ar, and medul l ary carci noma us ual l y are t reat ed wi t h a combi nat i on of s urgery, s uppres s i on wi t h t hyroi d hormone, and radi oact i ve i odi ne. Anapl as t i c carci noma general l y i s t reat ed pal l i at i vel y. It may requi re s urgery t o rel i eve obs t ruct i on; chemot herapy may del ay deat h. o
o
a. Surgery
(1) Papi l l ary carci noma i s oft en t reat ed by t ot al t hyroi dect omy.
Pa g e 1 9 3 8
ABC Ambe r CHM Conve rte r Tria l ve rsion, http://w w w .proce sste x t.com/a bcchm.html
(2) Fol l i cul ar carci noma and more ext ens i ve papi l l ary t umors us ual l y are t reat ed by near-t ot al t hyroi dect omy; jus t enough t i s s ue i s l eft i n as s oci at i on wi t h t he pos t eri or caps ul e t o s pare t he parat hyroi d gl ands . Thi s more ext ens i ve procedure i s more l i kel y t o be compl i cat ed by hypoparat hyroi di s m, but i t i s fol l owed by l es s t umor recurrence.
o
o
b. Radioactive iodine therapy. Di fferent i at ed t hyroi d cancer (papi l l ary and fol l i cul ar) oft en accumul at es radi oact i ve i odi ne.
(1) Radi oi odi ne can be us ed t o abl at e any normal t hyroi d t i s s ue t hat remai ns aft er near-t ot al t hyroi dect omy; normal t hyroi d t i s s ue has great er affi ni t y for radi oi odi ne t han t umor t i s s ue, and t herefore l i mi t s t he amount of radi oi odi ne t hat woul d be t aken up by t umor.
(2) Aft er al l normal t hyroi d t i s s ue has been abl at ed, whol e-body s cans wi t h radi oi odi ne wi l l be more l i kel y t o reveal funct i oni ng met as t at i c t umor, whi ch can be t reat ed wi t h s ubs equent l arge dos es of radi oi odi ne.
o
o
c. Suppression therapy. Becaus e many t hyroi d cancers grow more rapi dl y wi t h TSH s t i mul at i on, TSH s houl d be s uppres s ed by t reat ment wi t h L-t hyroxi ne.
Pa g e 1 9 3 9
ABC Ambe r CHM Conve rte r Tria l ve rsion, http://w w w .proce sste x t.com/a bcchm.html
(1) Becaus e s uppres s i on of TSH produces a s t at e of s ubcl i ni cal hypert hyroi di s m, whi ch may decreas e bone dens i t y, i ncreas e cardi ac i rri t abi l i t y, and have ot her del et eri ous effect s , t he ext ent t o whi ch TSH s houl d be s uppres s ed i s not cl ear.
(2) A reas onabl e approach may be t o ai m for ful l s uppres s i on (undet ect abl e TSH) i n pat i ent s wi t h hi gh-ri s k t hyroi d cancer (l arge t umor, known met as t as es , et c.) and t o ai m for TSH i n t he l ow-normal range i n pat i ent s wi t h l ow-ri s k cancer.
III. Disorders of the Parathyroid Glands A. Primary hyperparathyroidism and hypercalcemia Pri mary hyperparat hyroi di s m i s t he res ul t of overs ecret i on of parat hyroi d hormone (PTH), whi ch i n t urn caus es hypercal cemi a. PTH i s i mport ant i n mai nt ai ni ng normal cal ci um homeos t as i s (Fi gure 9-3). P.402
Pa g e 1 9 4 0
ABC Ambe r CHM Conve rte r Tria l ve rsion, http://w w w .proce sste x t.com/a bcchm.html
FIGURE 9-3 Act i ons of parat hyroi d hormone (PTH) i n mai nt ai ni ng normal cal ci um homeos t as i s .
1. Epidemiology. Pri mary hyperparat hyroi di s m i s common, affect i ng approxi mat el y one i ndi vi dual i n every 1000 who are s creened. The di s eas e i s es peci al l y common i n mi ddl e-aged and el derl y women.
2. Etiology o
o
a. A s i ngl e parathyroid adenoma caus es 80%–90% of cas es , and hyperplasia of al l four gl ands caus es 10%–20% of cas es of pri mary hyperparat hyroi di s m. Parat hyroi d carci noma i s a rare caus e.
o
o
b. Predisposing factors
(1) Al t hough mos t cas es s eem t o ari s e wi t hout a known caus e, a hi s t ory of radiation to the neck i s pres ent i n 10% or more of pat i ent s .
(2) Fami l i al occurrence of parat hyroi d hyperpl as i a and of t he MEN s yndrome, whi ch oft en i nvol ves parat hyroi d adenomas or hyperpl as i a, i ndi cat es t hat genetic factors may be i mport ant .
o
o
3. Pathophysiology.
Pa g e 1 9 4 1
ABC Ambe r CHM Conve rte r Tria l ve rsion, http://w w w .proce sste x t.com/a bcchm.html
a. PTH rai s es t he s erum cal ci um l evel by s t i mul at i ng vi t ami n D product i on (whi ch i s i mport ant for gas t roi nt es t i nal abs orpt i on of cal ci um), by i ncreas i ng renal t ubul ar reabs orpt i on of cal ci um, by decreas i ng renal t ubul ar reabs orpt i on of phos phat e, and by promot i ng movement of cal ci um from t he bone.
b. Decreas ed ci rcul at i ng l evel s of i oni zed cal ci um s t i mul at e PTH product i on, t endi ng t o rai s e cal ci um l evel s back t o normal ; convers el y, i ncreas ed ci rcul at i ng l evel s of i oni zed cal ci um i nhi bi t PTH product i on. However, a parat hyroi d adenoma may funct i on aut onomous l y, produci ng exces s i ve PTH des pi t e hi gh s erum cal ci um l evel s and caus i ng t he abnormal i t i es of pri mary hyperparat hyroi di s m.
o
o
4. Clinical features. The rout i ne meas urement of s erum cal ci um l evel s i n mul t i channel s creeni ng t es t s has l ed t o earl y di agnos i s of pri mary hyperparat hyroi di s m. The di s eas e commonl y mani fes t s as mi l d as ympt omat i c hypercal cemi a, al t hough occas i onal l y pat i ent s are s een wi t h t he cl as s i c fi ndi ngs of advanced ki dney and bone di s eas e. Pat i ent s wi t h s erum cal ci um l evel s great er t han 11 or 12 mg/dL oft en have gas t roi nt es t i nal s ympt oms , neurol ogi c s ympt oms , or bot h.
a. Renal manifestations
Pa g e 1 9 4 2
ABC Ambe r CHM Conve rte r Tria l ve rsion, http://w w w .proce sste x t.com/a bcchm.html
(1) Al t hough PTH i ncreas es renal cal ci um reabs orpt i on, t he hypercal cemi a and res ul t i ng i ncreas ed gl omerul ar fi l t rat i on of cal ci um commonl y l ead t o hypercalciuria, whi ch may caus e t he format i on of urinary calculi.
(2) Chroni c hypercal cemi a may caus e depos i t i on of cal ci um wi t hi n t he renal parenchyma (nephrocal ci nos i s ) and event ual renal failure.
b. Skeletal manifestations. PTH exces s i ncreas es t he rat e of os t eocl as t i c bone res orpt i on and can l ead t o t he di s order of bone met abol i s m cal l ed osteitis fibrosa cystica.
(1) Symptoms i ncl ude bone pai n, fract ures , and areas of s wel l i ng and deformi t y l ocal i zed t o i nvol ved bones .
(2) There are areas of demineralization i n t he s kel et on. In s evere cas es , t here may be bone cysts and “brown tumors,― whi ch are l ocal i zed l es i ons cons i s t i ng of prol i ferat i ng os t eocl as t s , os t eobl as t s , and fi brous t i s s ue.
(3) Radi ographs may s how generalized
Pa g e 1 9 4 3
ABC Ambe r CHM Conve rte r Tria l ve rsion, http://w w w .proce sste x t.com/a bcchm.html
osteopenia, wi t h demi neral i zat i on of t he s kul l and ot her areas . Subperiosteal resorption of bone occurs i n t he phal anges and di s t al port i ons of t he cl avi cl es . Los s of t he l ami na dura around t he t eet h i s charact eri s t i c.
c. Gastrointestinal manifestations. Sympt oms rel at ed t o t he hypercal cemi a of hyperparat hyroi di s m i ncl ude anorexi a, wei ght l os s , cons t i pat i on, naus ea and vomi t i ng, and abdomi nal pai n. Pat i ent s wi t h hyperparat hyroi di s m may have an i ncreas ed i nci dence of pept i c ul cer di s eas e and pancreat i t i s .
d. Neurologic manifestations. Emot i onal changes and abnormal ment at i on may occur, whi ch al s o are rel at ed t o t he hypercal cemi a of hyperparat hyroi di s m. Fat i gue and mus cl e weaknes s are common.
o
o
5. Diagnosis
a. Laboratory findings
(1) Blood chemistry
(a) Elevation of the serum calcium level is the hallmark of primary hyperparathyroidism.
Pa g e 1 9 4 4
ABC Ambe r CHM Conve rte r Tria l ve rsion, http://w w w .proce sste x t.com/a bcchm.html
(b) The s erum phos phat e l evel i s l owered i n many, but not al l , cas es . P.403
TABLE 9-9 Causes of Hypercalcemia Pri Sa ma rco ry
i do
hyp s i s erp Be ara ni g t hy n roi fa di s mi l m
i al
Mal hyp i gn erc anc al c y
em W ia
i t h Hy bo per ne vi t me am t as i no t as s i s es D (e. Mi l g., k†bre “al
Pa g e 1 9 4 5
ABC Ambe r CHM Conve rte r Tria l ve rsion, http://w w w .proce sste x t.com/a bcchm.html
as t kal can i cer s yn ,
dro
my me el o Hy ma per ,
t hy
l ym roi ph di s om m a) Thi W az i i t h de out t he bo rap ne y me Im t as mo t as bi l i es z at (e. i on g., hyp ern ep hro ma ; pa ncr eat ic can cer
Pa g e 1 9 4 6
ABC Ambe r CHM Conve rte r Tria l ve rsion, http://w w w .proce sste x t.com/a bcchm.html
; s qu am ous cel l car ci n om a of t he l un g, cer vi x , an d es o ph ag us ; he ad an d nec k tu mo rs )
(c) The s erum al kal i ne phos phat as e l evel i s el evat ed onl y i n pat i ent s
Pa g e 1 9 4 7
ABC Ambe r CHM Conve rte r Tria l ve rsion, http://w w w .proce sste x t.com/a bcchm.html
wi t h s i gni fi cant bone di s eas e.
(2) Urine chemistry. Hypercalciuria i s common but , becaus e of t he cal ci um-reabs orbi ng act i on of PTH, approxi mat el y one-t hi rd of pat i ent s have normal uri ne cal ci um l evel s .
b. PT H assay. An el evat ed bl ood PTH l evel i n t he pres ence of hypercal cemi a i s s t rong evi dence for pri mary hyperparat hyroi di s m becaus e ot her caus es of cal ci um el evat i on t end t o s uppres s PTH l evel s . The immunoradiometric assay for intact PT H i s more s ens i t i ve and s peci fi c t han previ ous l y us ed as s ays .
c. Diagnostic imaging. Noni nvas i ve t echni ques s uch as ultrasonography, CT scanning, MRI, and isotope scans may demons t rat e parat hyroi d adenomas i n 60%–80% of cas es (Onl i ne Fi gure 9-4).
ONLINE FIGURE 9-4 Ses t ami bi s can t o l ocal i ze parat hyroi d adenomas . A. Immedi at e i mage; B. Del ayed i mage (t wo hours ). o
Pa g e 1 9 4 8
ABC Ambe r CHM Conve rte r Tria l ve rsion, http://w w w .proce sste x t.com/a bcchm.html
o
6. Differential diagnosis. Hypercal cemi a may be caus ed by ent i t i es ot her t han pri mary hyperparat hyroi di s m (Tabl e 9-9)
a. T umors
(1) Malignant tumors with bone metastases may caus e hypercal cemi a t hrough an i ncreas e i n bone res orpt i on due t o l ocal effect s and s omet i mes t hrough l ocal l y act i ng humoral s ubs t ances (e.g., os t eocl as t act i vat i ng fact or) produced by t he met as t at i c t umor.
(2) T umors that cause hypercalcemia in the absence of bone metastases do s o by produci ng PT H-related peptide (PT HrP), a humoral fact or t hat act s l i ke PTH and bi nds t o PTH recept ors but i s not meas ured by t he PTH radi oi mmunoas s ay. PTHrP may produce bi ochemi cal effect s l i ke t hos e of PTH, i ncl udi ng hypophos phat emi a and i ncreas ed uri nary nephrogenous cAMP. Humoral hypercal cemi a of mal i gnancy may be di fferent i at ed from pri mary hyperparat hyroi di s m by fi ndi ng an el evat ed l evel of PTHrP and a normal or l ow l evel of PTH.
TABLE 9-10 Indications for Surgery in
Pa g e 1 9 4 9
ABC Ambe r CHM Conve rte r Tria l ve rsion, http://w w w .proce sste x t.com/a bcchm.html
Primary Hyperparathyroidism Sy mp to ms rel at e d to hyp erc al c em ia Ser um cal ci u m l ev el s >1 mg /dL ab ove nor ma l ran ge Ag e
Pa g e 1 9 5 0
ABC Ambe r CHM Conve rte r Tria l ve rsion, http://w w w .proce sste x t.com/a bcchm.html
<5 0 yea rs Re nal ma ni f es t at i ons Re nal cal cul i Re duc ed cre at i ni n e cl e ara nce 24ho ur uri ne cal ci u m exc ret i
Pa g e 1 9 5 1
ABC Ambe r CHM Conve rte r Tria l ve rsion, http://w w w .proce sste x t.com/a bcchm.html
on >4 00 mg /dL Bo ne mi ner al de ns i ty >2. 5 SD s bel ow pe ak bo ne ma ss (t s co re) Pat i en t pre fer enc e
Pa g e 1 9 5 2
ABC Ambe r CHM Conve rte r Tria l ve rsion, http://w w w .proce sste x t.com/a bcchm.html
for s ur ger y or un wi l l i ng nes s /i na bi l i ty to un der go pro l on ge d fol l ow -up SD, sta nd ard dev i at i on. o
P.404
o
Pa g e 1 9 5 3
ABC Ambe r CHM Conve rte r Tria l ve rsion, http://w w w .proce sste x t.com/a bcchm.html
b. Sarcoidosis may caus e hypercal cemi a becaus e of product i on of 1,25-di hydroxyvi t ami n D 3 by granul omat ous t i s s ue. If no ot her fi ndi ngs i ndi cat e t he pres ence of s arcoi dos i s and t he PTH concent rat i on i s not el evat ed, a t herapeut i c t ri al may be hel pful . Serum cal ci um l evel s decl i ne wi t hi n 1 week of t he s t art of gl ucocort i coi d admi ni s t rat i on (e.g., 40 mg predni s one dai l y) i n mos t cas es of s arcoi dos i s , but are unaffect ed i n mos t cas es of pri mary hyperparat hyroi di s m.
c. Benign familial hypercalcemia (fami l i al hypocal ci uri c hypercal cemi a) i s an aut os omal domi nant di s order caus ed by an i nact i vat i ng mut at i on of t he gene encodi ng t he cal ci um-s ens i ng recept or. The parat hyroi d gl ands and t he renal t ubul es fai l t o recogni ze t he t rue concent rat i on of ext racel l ul ar cal ci um, l eadi ng t o overproduct i on of PTH and i ncreas ed renal reabs orpt i on of cal ci um.
(1) The res ul t i ng s yndrome cons i s t s of mi l d-t o-moderat e hypercal cemi a wi t h normal or s l i ght l y el evat ed s erum PTH l evel s and hypocal ci uri a. Pat i ent s remai n as ympt omat i c, wi t hout renal cal cul i , renal parenchymal damage, or bone di s eas e.
(2) Di agnos t i c cl ues are t he fami l i al occurrence and t he l ow (rat her t han hi gh)
Pa g e 1 9 5 4
ABC Ambe r CHM Conve rte r Tria l ve rsion, http://w w w .proce sste x t.com/a bcchm.html
uri nary cal ci um excret i on.
(3) Becaus e s urgi cal t reat ment i s not benefi ci al and i s not recommended, t hi s condi t i on s houl d be rul ed out i n pat i ent s wi t h apparent pri mary hyperparat hyroi di s m.
d. Vitamin D intoxication, whi ch i s us ual l y s een i n pat i ent s recei vi ng pharmacol ogi c dos es of t he vi t ami n for t he t reat ment of hypoparat hyroi di s m, res ul t s i n hypercal cemi a.
(1) The di agnos i s s houl d be apparent from t he pat i ent 's hi s t ory.
(2) If a s uffi ci ent l y rapi d fal l i n s erum cal ci um l evel s does not res ul t when vi t ami n D i nges t i on i s s t opped, gl ucocort i coi ds s houl d be gi ven. Gl ucocort i coi ds i nhi bi t t he act i on of vi t ami n D on i nt es t i nal cal ci um abs orpt i on and rapi dl y l ower s erum cal ci um l evel s .
e. Milk–alkali syndrome i s caus ed by t he i nges t i on of l arge quant i t i es of cal ci um and abs orbabl e al kal i . It i s charact eri zed by hypercal cemi a, s ys t emi c al kal os i s , and renal damage due t o nephrocal ci nos i s . Thi s s yndrome i s unl i kel y t o devel op unl es s cal ci um i nt ake exceeds 5 g cal ci um carbonat e
Pa g e 1 9 5 5
ABC Ambe r CHM Conve rte r Tria l ve rsion, http://w w w .proce sste x t.com/a bcchm.html
(2 g el ement al cal ci um) dai l y, whi ch i s approxi mat el y doubl e t he dos e us ual l y recommended for t he prevent i on or t reat ment of os t eoporos i s .
f. Other causes of hypercalcemia
(1) Hyperthyroidism. Thi s condi t i on may caus e hypercal cemi a becaus e of i ncreas ed bone t urnover. The di agnos i s i s us ual l y evi dent becaus e t he hypert hyroi di s m, not t he hypercal cemi a, i s t he pres ent i ng s yndrome.
(2) T hiazide diuretics. Thes e agent s decreas e uri nary cal ci um excret i on but rarel y caus e hypercal cemi a; t hey s houl d be avoi ded, however, i n pat i ent s wi t h hyperparat hyroi di s m.
(3) Prolonged immobilization. Becaus e of cont i nui ng bone res orpt i on i n t he abs ence of normal pos t ural s t i mul i for bone format i on, hypercal cemi a may occur. Thi s probl em i s part i cul arl y common i n chi l dren who are confi ned t o bed for l ong peri ods (e.g., i n a t ot al body cas t for t reat ment of mul t i pl e t raumat i c fract ures ).
(4) Paget' s disease. Increas ed bone t urnover and l ocal i zed bone t umors due
Pa g e 1 9 5 6
ABC Ambe r CHM Conve rte r Tria l ve rsion, http://w w w .proce sste x t.com/a bcchm.html
t o defect i ve regul at i on of bone met abol i s m may produce hypercal cemi a, es peci al l y duri ng peri ods of i mmobi l i zat i on.
(5) Recovery from acute renal failure. A s yndrome of hypercal cemi a may devel op duri ng t he recovery peri od aft er rhabdomyol ys i s and acut e renal fai l ure. Duri ng t he fi rs t 2–3 days aft er mus cl e i njury, t he i ni t i al mus cl e damage l eads t o l ocal cal ci um and phos phat e depos i t i on. W hen renal funct i on ret urns t o normal , cal ci um and phos phat e exi t from s i t es of mus cl e damage and ent er t he ci rcul at i on t o produce hypercal cemi a. Thi s defect t ypi cal l y occurs 2–3 weeks aft er t he acut e mus cl e i njury.
o
o
7. T herapy of primary hyperparathyroidism
a. Surgery
(1) Indications. As ympt omat i c pat i ent s wi t h mi ni mal el evat i on i n s erum cal ci um and no compl i cat i ons oft en have no progres s i on of t he di s eas e for 10 years or l onger. Such pat i ent s may be fol l owed cl os el y and cons i dered for s urgery onl y i f t hey devel op s peci fi c i ndi cat i ons (Tabl e 9-10).
Pa g e 1 9 5 7
ABC Ambe r CHM Conve rte r Tria l ve rsion, http://w w w .proce sste x t.com/a bcchm.html
(2) Initial surgical exploration. In mos t cas es , a single adenoma i s found and removed. If parathyroid hyperplasia i s found, t he s urgeon may remove t hree gl ands and part of t he fourt h, or al l parat hyroi d t i s s ue and t rans pl ant a port i on t o t he mus cl es of t he forearm or neck. The removal of addi t i onal parat hyroi d t i s s ue from t he t rans pl ant ed port i on may be accompl i s hed eas i l y i f neces s ary.
(3) Repeat surgical exploration. Approxi mat el y 10% of i ni t i al neck expl orat i ons fai l t o i ndi cat e abnormal parat hyroi d t i s s ue or cure t he di s eas e. Aft er t he pat i ent i s re-eval uat ed, addi t i onal i nvas i ve and noni nvas i ve l ocal i zi ng procedures are cons i dered before a s econd operat i on i s performed.
(4) Postoperative course. Trans i ent hypocal cemi a i s common aft er t he removal of a parat hyroi d adenoma becaus e t he remai ni ng normal gl ands are l i kel y t o have been s uppres s ed by l ong-s t andi ng hypercal cemi a.
(a) Pat i ent s us ual l y recover wi t hi n a few weeks .
(b) In t he occas i onal pat i ent wi t h s evere bone di s eas e, marked
Pa g e 1 9 5 8
ABC Ambe r CHM Conve rte r Tria l ve rsion, http://w w w .proce sste x t.com/a bcchm.html
i nt ract abl e hypocal cemi a may pers i s t for s everal mont hs , becaus e, when t he exces s PTH s t i mul us i s s uddenl y removed, t he demi neral i zed bone becomes avi d for cal ci um (t he “hungry bones †• s yndrome).
b. Medical therapy may be us ed i f no i ndi cat i ons for s urgery are pres ent (s ee Tabl e 9-10), i f ot her i l l nes s cont rai ndi cat es s urgery, or i f t he pat i ent refus es s urgery.
(1) Increased fluid intake and act i vi t y hel p t o mi ni mi ze hypercal cemi a.
(2) Oral phosphate i n dos es of 1–2 g dai l y oft en l owers t he s erum cal ci um l evel . The mai n compl i cat i on, ext ras kel et al cal ci fi cat i on, i s uncommon i f t hi s dos e i s not exceeded.
(3) Estrogen may l ower mi l dl y el evat ed s erum cal ci um l evel s t o normal , apparent l y by decreas i ng bone res orpt i on. Thi s t reat ment may be cons i dered i n pos t menopaus al women, who make up t wo-t hi rds of t he cas es of pri mary hyperparat hyroi di s m.
c. Emergency treatment i s neces s ary i f t he cal ci um l evel i ncreas es above 13–15 mg/dL
Pa g e 1 9 5 9
ABC Ambe r CHM Conve rte r Tria l ve rsion, http://w w w .proce sste x t.com/a bcchm.html
before t he adenoma can be removed or i f s urgi cal t reat ment i s refus ed or i s uns ucces s ful . Hypercal cemi a caus ed by di s eas es ot her t han hyperparat hyroi di s m (e.g., hypercal cemi a due t o mal i gnancy) al s o may be t reat ed by t he fol l owi ng met hods .
(1) Hydration with sodium diuresis. Four t o s i x l i t ers or more of i nt ravenous s al i ne dai l y, wi t h l arge dos es of furos emi de, i ncreas e renal cal ci um excret i on and reduce s erum cal ci um l evel s .
(2) Bisphosphonates s uch as pamidronate and z ol endri c aci d (Zomet a) bi nd t o hydroxyapat i t e i n bone and bl ock di s s ol ut i on of t hi s mi neral ; t hey al s o i nhi bi t os t eocl as t act i vi t y. The res ul t i ng i nhi bi t i on of bone res orpt i on produces a decl i ne i n s erum cal ci um l evel s over 1–7 days . Pami dronat e (60–90 mg) i s gi ven i nt ravenous l y i n a s i ngl e dos e over 24 hours ; t hi s can be repeat ed i n 7 days i f neces s ary.
(3) Plicamycin (formerl y cal l ed mi t hramyci n) i s an ant i neopl as t i c agent t hat l owers s erum cal ci um l evel s by i nhi bi t i ng os t eocl as t i c bone res orpt i on. A s i ngl e i nt ravenous dos e of 25 µg/kg admi ni s t ered over 4–6 hours may l ower t he s erum cal ci um l evel s for s everal
Pa g e 1 9 6 0
ABC Ambe r CHM Conve rte r Tria l ve rsion, http://w w w .proce sste x t.com/a bcchm.html
days . Thi s drug i s commonl y us ed i n t reat i ng hypercal cemi a due t o mal i gnancy.
(4) Gallium nitrate i s a pot ent i nhi bi t or of bone res orpt i on. It i s gi ven i n a dos e 2
of 200 mg/m /day by cont i nuous i nfus i on for 5 days . Pot ent i al nephrot oxi ci t y l i mi t s i t s us e t o pat i ent s wi t hout renal fai l ure.
(5) Calcitonin l owers s erum cal ci um l evel s by i nhi bi t i ng os t eocl as t i c bone res orpt i on. It i s t he fas t es t act i ng hypocal cemi c agent and i s very s afe, but i t i s l es s pot ent t han pl i camyci n, gal l i um ni t rat e, or pami dronat e, and i t s effect t ends t o decreas e aft er s everal days . Sal mon cal ci t oni n, 4 U/kg, i s gi ven i nt ramus cul arl y or s ubcut aneous l y every 12 hours .
(6) Glucocorticoids l ower s erum cal ci um l evel s i n pat i ent s wi t h s arcoi dos i s and vi t ami n D i nt oxi cat i on, and s omet i mes i n t hos e pat i ent s wi t h myel oma and hemat ol ogi c mal i gnancy. However, gl ucocort i coi ds do not l ower s erum cal ci um l evel s i n pat i ent s wi t h hyperparat hyroi di s m.
B. Hypoparathyroidism and hypocalcemia
Pa g e 1 9 6 1
ABC Ambe r CHM Conve rte r Tria l ve rsion, http://w w w .proce sste x t.com/a bcchm.html
1. Etiology o
o
a. Surgical removal of the parathyroid glands i s t he mos t common caus e of hypoparat hyroi di s m.
(1) Thi s may be an unavoi dabl e res ul t of radi cal neck di s s ect i on for cancer or a rare compl i cat i on of s ubt ot al t hyroi dect omy.
(2) T emporary hypoparathyroidism aft er neck s urgery i s not uncommon and may be due t o i s chemi c i njury t o t he gl ands . Recovery us ual l y occurs i n a few weeks or mont hs .
o
o
b. Idiopathic hypoparathyroidism of aut oi mmune et i ol ogy i s much l es s common. It us ual l y i s di agnos ed i n chi l dhood, may be fami l i al , and s omet i mes i s as s oci at ed wi t h adrenal i ns uffi ci ency and mucocut aneous candi di as i s .
2. Pathophysiology. PT H deficiency leads to hypocalcemia, the hallmark of hypoparathyroidism, t hrough t he s ame mechani s ms by whi ch i ncreas ed s ecret i on of PTH caus es hypercal cemi a P.405
(s ee III A 3). In addi t i on, t he decreas ed renal phos phat e cl earance l eads t o hyperphosphatemia, whi ch i s pres ent i n mos t cas es of hypoparat hyroi di s m.
Pa g e 1 9 6 2
ABC Ambe r CHM Conve rte r Tria l ve rsion, http://w w w .proce sste x t.com/a bcchm.html
3. Clinical features and diagnosis. Hypocal cemi a produces acut e s ympt oms rel at ed t o i ncreas ed neuromus cul ar i rri t abi l i t y. In addi t i on, l ong-t erm changes may occur as a res ul t of effect s on ect odermal t i s s ues and ect opi c cal ci um depos i t i on. o
o
a. Symptoms and signs
(1) Latent tetany
(a) Mi l d hypocal cemi a may caus e muscular fatigue and weakness as wel l as numbness and t i ngl i ng around t he mout h and i n t he hands and feet .
(b) Chvostek' s sign may be pos i t i ve (i .e., a t ap over t he faci al nerve i n front of t he ear el i ci t s a cont ract i on of t he faci al mus cl es and upper l i p). However, Chvos t ek's s i gn may be pos i t i ve i n 10% of normal i ndi vi dual s .
(c) T rousseau' s sign may be pos i t i ve; t hat i s , i nfl at i on of a bl ood pres s ure cuff on t he arm t o a pres s ure hi gher t han t he pat i ent 's s ys t ol i c pres s ure for 3 mi nut es el i ci t s carpal s pas m [fl exi on of t he met acarpophal angeal (MCP) joi nt s and ext ens i on of t he i nt erphal angeal joi nt s , wi t h drawi ng t oget her of t he fi ngers and adduct i on of t he t humb].
Pa g e 1 9 6 3
ABC Ambe r CHM Conve rte r Tria l ve rsion, http://w w w .proce sste x t.com/a bcchm.html
(2) Overt tetany. Severe hypocal cemi a caus es twitching and cramps of the muscles with carpopedal spasm. Laryngeal stridor and seizures may occur i n s evere cas es .
(3) Long-term effects of hypocalcemia
(a) Ectodermal changes i ncl ude at rophy, bri t t l enes s , and ri dgi ng of t he nai l s ; drynes s and s cal i ng of t he s ki n; and enamel defect s and hypopl as i a of t he t eet h.
(b) Calcification of the basal ganglia may occur and i s occas i onal l y as s oci at ed wi t h parki ns oni an s i gns and s ympt oms .
(c) Calcification of the optic lens may lead to cataract formation.
o
o
b. Laboratory abnormalities. Hypocalcemia and hyperphosphatemia are cons i s t ent l y pres ent i n hypoparat hyroi di s m. PTH l evel s , of cours e, are l ow, but cl i ni cal as s ays may not be s uffi ci ent l y s ens i t i ve t o di s t i ngui s h bet ween l ow and normal l evel s . An al bumi n l evel s houl d al ways be checked and t he s erum cal ci um correct ed for a l ow al bumi n. The correct i on formul a i s [4.0 – meas ured al bumi n] 0.8 + s erum cal ci um.
Pa g e 1 9 6 4
ABC Ambe r CHM Conve rte r Tria l ve rsion, http://w w w .proce sste x t.com/a bcchm.html
4. Differential diagnosis o
o
a. Pseudohypoparathyroidism
(1) Ps eudohypoparat hyroi di s m i s a heredi t ary di s eas e wi t h t wo di s t i nct areas of cl i ni cal expres s i on.
(a) Calcium metabolism is abnormal becaus e of end-organ res i s t ance t o t he act i on of PTH; t hat i s , t he ki dney and bone cannot res pond t o PTH, even t hough i t s concent rat i on i n t he s erum i s normal or i ncreas ed. The res ul t i s hypocal cemi a and hyperphos phat emi a, as are s een i n t rue hypoparat hyroi di s m.
(b) Developmental and skeletal abnormalities (Albright' s hereditary osteodystrophy) are pres ent .
(i) The mos t common abnormal i t i es are s hort s t at ure, s hort eni ng of t he met acarpal and met at ars al bones , and ment al defi ci ency.
(ii) The s kel et al abnormal i t i es s omet i mes occur i n t he abs ence of any di s order of cal ci um met abol i s m; t hi s has been cal l ed
Pa g e 1 9 6 5
ABC Ambe r CHM Conve rte r Tria l ve rsion, http://w w w .proce sste x t.com/a bcchm.html
“ps eudops eudohypoparat hyroi di s m.―
(2) If Al bri ght 's heredi t ary os t eodys t rophy i s not pres ent and i f hypocal cemi a i s not cl earl y pos t s urgi cal i n i t s ons et , t he di fferent i at i on bet ween t rue hypoparat hyroi di s m and ps eudohypoparat hyroi di s m may depend on l aborat ory t es t s .
(a) PTH l evel s may be s uffi ci ent l y el evat ed i n ps eudohypoparat hyroi di s m t o di s t i ngui s h t he s yndrome cl earl y from hypoparat hyroi di s m.
(b) Inject i on of PTH i s fol l owed prompt l y by i ncreas ed uri ne concent rat i ons of phos phat e and cAMP and, wi t hi n 1 or 2 days , by an i ncreas e i n s erum cal ci um i n pat i ent s wi t h hypoparat hyroi di s m. No res pons e occurs i n pat i ent s wi t h ps eudohypoparat hyroi di s m.
o
o
b. Hypoalbuminemia caus es a decreas e i n t he fract i on of s erum cal ci um t hat i s bound t o prot ei n and, t herefore, a decreas e i n t ot al s erum cal ci um; becaus e t he i oni zed fract i on of s erum cal ci um remai ns normal , however, t here are no cl i ni cal mani fes t at i ons of cal ci um defi ci ency. Thi s s houl d not be cons i dered a form of t rue hypocal cemi a. For each decrement i n s erum al bumi n of 1 g/L, t he s erum cal ci um i s expect ed t o decl i ne by approxi mat el y 0.8 mg/dL.
Pa g e 1 9 6 6
ABC Ambe r CHM Conve rte r Tria l ve rsion, http://w w w .proce sste x t.com/a bcchm.html o
o
c. Hypocal cemi a i n renal failure i s caus ed by many fact ors . Thes e i ncl ude renal phos phat e ret ent i on (wi t h res ul t ant hyperphos phat emi a), reduced product i on of 1,25-di hydroxyvi t ami n D 3 by t he di s eas ed ki dneys , and bone res i s t ance t o t he cal cemi c act i on of PTH.
o
o
d. Malabsorption as s oci at ed wi t h gas t roi nt es t i nal di s eas e may l ead t o i nadequat e cal ci um or vi t ami n D abs orpt i on and cons equent hypocal cemi a.
o
o
e. Vitamin D deficiency or res i s t ance t o t he act i ons of vi t ami n D may caus e hypocal cemi a t hrough decreas ed gas t roi nt es t i nal abs orpt i on of cal ci um.
o
o
f. Acute pancreatitis may l ead t o i nt ra-abdomi nal preci pi t at i on of cal ci um s oaps i n areas of fat necros i s . W het her t hi s expl ai ns why hypocal cemi a i s s omet i mes s een i n pat i ent s wi t h acut e pancreat i t i s i s uncert ai n.
o
o
g. Osteoblastic metastasis of pros t at e, breas t , or l ung cancer may produce hypocalcemi a, pres umabl y becaus e of rapi d bone upt ake of cal ci um.
o
o
h. Hypomagnesemia decreas es product i on of PTH and i nhi bi t s t he act i ons of PTH and vi t ami n D on bone, l eadi ng t o hypocal cemi a.
Pa g e 1 9 6 7
ABC Ambe r CHM Conve rte r Tria l ve rsion, http://w w w .proce sste x t.com/a bcchm.html
5. T herapy o
o
a. Hypoparathyroidism. PTH, whi ch mus t be i nject ed t wi ce dai l y, i s not commonl y us ed for l ong-t erm t herapy. Ins t ead, t reat ment wi t h supplemental calcium, combi ned wi t h vitamin D t o enhance i t s abs orpt i on, i s effect i ve i n correct i ng t he hypocal cemi a and hyperphos phat emi a of hypoparat hyroi di s m, even i n cas es caus ed by end-organ unres pons i venes s t o PTH (e.g., ps eudohypoparat hyroi di s m).
(1) Calcium supplementation. Us ual l y 1–2 g el ement al cal ci um i s gi ven dai l y.
(a) Commonl y us ed preparat i ons i ncl ude cal ci um gl uconat e (each 1-g t abl et cont ai ns 90 mg el ement al cal ci um), cal ci um carbonat e (each t abl et of Os -Cal 500 cont ai ns 500 mg cal ci um), and cal ci um gl ubi onat e (each t abl es poon of Neo-Cal gl ucon cont ai ns 345 mg cal ci um).
(b) The cal ci um i s gi ven i n t hree or four di vi ded dos es . The dos e can eas i l y be rai s ed or l owered t o regul at e s erum cal ci um l evel s .
(2) Vitamin D. Vitamin D 2 (ergocalciferol) has been us ed for years i n t he t reat ment of hypoparat hyroi di s m, but calcitriol has l argel y
Pa g e 1 9 6 8
ABC Ambe r CHM Conve rte r Tria l ve rsion, http://w w w .proce sste x t.com/a bcchm.html
repl aced i t .
(a) Calciferol
(i) Becaus e vi t ami n D 2 mus t be convert ed t o 1,25-di hydroxyvi t ami n D 3 t o be ful l y effect i ve (a convers i on t hat i s great l y i nhi bi t ed i n t he abs ence of PTH), l arge dos es are neces s ary. The average dai l y dos e i s 50,000 U, wi t h a range of 25,000–150,000 U, al t hough t he recommended di et ary al l owance i n normal pers ons i s onl y 400 U.
(ii) The ons et of act i on i s s l ow (1–2 weeks ), but t he effect may pers i s t for mont hs aft er admi ni s t rat i on i s s t opped.
(b) Calcitriol (1,25-dihydroxyvitamin D 3 ). The mai n advant age of cal ci t ri ol over vi t ami n D 2 i s t he fas t er ons et and ces s at i on of act i on, whi ch may l ead t o more preci s e cont rol of bl ood cal ci um l evel s . Cal ci t ri ol i s gi ven i n dos es of 0.25–2.0 µg/day.
P.406
o
o
b. Acute hypocalcemia. Treat ment of t hi s
Pa g e 1 9 6 9
ABC Ambe r CHM Conve rte r Tria l ve rsion, http://w w w .proce sste x t.com/a bcchm.html
condi t i on, whi ch may occur s hort l y aft er parat hyroi d res ect i on and may caus e s evere t et ani c s ympt oms , cons i s t s of i nt ravenous admi ni s t rat i on of cal ci um. Cal ci um gl uconat e (10%) i n a dos e of 1–2 g i s gi ven i nt ravenous l y i n approxi mat el y 10 mi nut es fol l owed by s l ow i nfus i on of 1 g cal ci um gl uconat e over t he next 6–8 hours .
IV. Disorders of Glucose Homeostasis A. Diabetes mellitus Thi s condi t i on i s characterized by hyperglycemia and ot her met abol i c derangement s t hat are caused by inadequate action of insulin on body t i s s ues , becaus e of ei t her reduced ci rcul at i ng l evel s of i ns ul i n or res i s t ance of t arget t i s s ues t o i t s act i ons .
1. Classification (Tabl e 9-11). Di abet es mel l i t us i s di vi ded i nt o t wo cat egori es —type 1 diabetes (formerl y cal l ed i ns ul i n-dependent di abet es ) and type 2 diabetes (formerl y cal l ed noni ns ul i n-dependent di abet es ). Syndromes wi t h feat ures from ei t her of t hes e cat egori es may be rel at ed t o t he s t age or s everi t y of t he di s eas e or t o ot her condi t i ons . Thes e s yndromes i ncl ude i mpai red fas t i ng gl ucos e, i mpai red gl ucos e t ol erance, ges t at i onal di abet es , previ ous abnormal i t y of gl ucos e t ol erance, pot ent i al abnormal i t y of gl ucos e t ol erance, and di abet es as s oci at ed wi t h cert ai n ot her di s eas es . o
o
a. T ype 1 diabetes affect s 5%–10% of di abet i c pat i ent s .
(1) In t ype 1 di abet es , not onl y i s i ns ul i n needed for opt i mal cont rol of bl ood gl ucos e, whi ch al s o may be t rue for pat i ent s wi t h t ype
Pa g e 1 9 7 0
ABC Ambe r CHM Conve rte r Tria l ve rsion, http://w w w .proce sste x t.com/a bcchm.html
2 di s eas e, but without exogenous insulin, patients are prone to the development of ketoacidosis. Thi s i s t hought t o refl ect a compl et e or al mos t compl et e abs ence of i ns ul i n i n pat i ent s wi t h t ype 1 di abet es , i n cont ras t t o t he part i al l ack of i ns ul i n and t he res i s t ance t o i ns ul i n charact eri s t i c of pat i ent s wi t h t ype 2 di abet es .
(2) Ot her key feat ures of t ype 1 di abet es are i t s occurrence i n chi l dren and young adul t s and i t s occurrence i n i ndi vi dual s who are l ean rat her t han obes e.
o
o
b. T ype 2 diabetes commonl y affect s overwei ght i ndi vi dual s ol der t han 40 years of age, al t hough wi t h t he ri s i ng preval ence of obes i t y i t i s i ncreas i ngl y bei ng di agnos ed i n chi l dren.
(1) Becaus e s ome i ns ul i n i s produced by t hes e pat i ent s , ketoacidosis does not occur.
(2) However, i ns ul i n t herapy may be neces s ary t o prevent s evere hypergl ycemi a.
TABLE 9-11 Major Types of Primary Diabetes Mellitus
Pa g e 1 9 7 1
ABC Ambe r CHM Conve rte r Tria l ve rsion, http://w w w .proce sste x t.com/a bcchm.html
T T y y p p e e 1 2 Di Di a a b b et et e e s s M M el el lit lit u u s s P 0. 2 re 2 % v %â al â €“ e €“ 4 n 0. % c 5 ; e %w ; o m m e e n n = > w m o e m n e n A U U
Pa g e 1 9 7 2
ABC Ambe r CHM Conve rte r Tria l ve rsion, http://w w w .proce sste x t.com/a bcchm.html
g susu e al al at l y l y o < > n 2 4 s 5 0 et ye ye ar ar s s G < > e 1 2 n 0 0 et % % ic of of s fi r fi r st st -d -d e e gr gr e e e e re re la la ti ti ve ve s s af af fe fe ct ct e e d; d; 5 9 0 0 %% co â
Pa g e 1 9 7 3
ABC Ambe r CHM Conve rte r Tria l ve rsion, http://w w w .proce sste x t.com/a bcchm.html
nc €“ or 1 d 0 a 0 nc % e co i n nc i d or e d nt a i c nc al e t in wi i d ns e nt ic al t wi ns H A N L ss o A oc n ia e te d wi th H L AD R 3,
Pa g e 1 9 7 4
ABC Ambe r CHM Conve rte r Tria l ve rsion, http://w w w .proce sste x t.com/a bcchm.html
H L AD R 4, H L AD Q A In N ut cr o oi e n m as e me u d ni pr ty ev al e nc e of a ut o a nt ib o di es to is
Pa g e 1 9 7 5
ABC Ambe r CHM Conve rte r Tria l ve rsion, http://w w w .proce sste x t.com/a bcchm.html
le t ce ll s a n d ot h er ti ss u es B U U o susu d al al y ly ly b le o ui a b ld n es e M K K et et et a os os b is is ol pr -r is o es m n is e; t a i n nt su; li in n su
Pa g e 1 9 7 6
ABC Ambe r CHM Conve rte r Tria l ve rsion, http://w w w .proce sste x t.com/a bcchm.html
pr l i o n d le uc ve ti ls o m n ay a b bs e e hi nt g h, n or m al , or lo w T In W re s u ei at l i g m n ht e
lo
nt
ss ; or al a g e nt s (e
Pa g e 1 9 7 7
ABC Ambe r CHM Conve rte r Tria l ve rsion, http://w w w .proce sste x t.com/a bcchm.html
.g ., su lf o ny lu re as , m et fo r m in , th ia zo li di n e di o n es ) or in su li n HLA,
Pa g e 1 9 7 8
ABC Ambe r CHM Conve rte r Tria l ve rsion, http://w w w .proce sste x t.com/a bcchm.html
human l eukoc yt e ant i ge n.
o
P.407
o
Online TABLE 9-12. Major Risk Factors for Type 2 Diabetes Fa mi l y hi s t or y of di a bet es (i .e ., par ent s or sib
Pa g e 1 9 7 9
ABC Ambe r CHM Conve rte r Tria l ve rsion, http://w w w .proce sste x t.com/a bcchm.html
lin gs wi t h di a bet es ) Ov erw ei g ht (B MI â ‰ ¥2 5 kg/ m2 ) Ha bi t ual phy sic al i na ct i vi t y Rac e/e t hn i ci t y
Pa g e 1 9 8 0
ABC Ambe r CHM Conve rte r Tria l ve rsion, http://w w w .proce sste x t.com/a bcchm.html
(e. g., Afri can Am eri can , Hi s pa ni c Am eri can , Nat i ve Am eri can , As i an Am eri can , an d Pac i fi c Is l an der )
Pa g e 1 9 8 1
ABC Ambe r CHM Conve rte r Tria l ve rsion, http://w w w .proce sste x t.com/a bcchm.html
Pre vi o us l y i de nt i f i ed IFG or IGT Hy per t en sio n â ‰ ¥1 40/ 90 mm Hg in ad ul t s) HD L cho l es t er ol â ‰ ¤3
Pa g e 1 9 8 2
ABC Ambe r CHM Conve rte r Tria l ve rsion, http://w w w .proce sste x t.com/a bcchm.html
5 mg /dL (0. 09 mm ol / L) an d/o r a t ri g l yc eri de l ev el â ‰ ¥2 50 mg /dL (2. 82 mm ol / L) Hi s t or y of GD M or
Pa g e 1 9 8 3
ABC Ambe r CHM Conve rte r Tria l ve rsion, http://w w w .proce sste x t.com/a bcchm.html
del i ve ry of a ba by wei ghi ng >9 lb Pol ycy sti c ova ry s yn dro me BM I, bo dy ma ss i nd ex; GD M, ges t at i on al
Pa g e 1 9 8 4
ABC Ambe r CHM Conve rte r Tria l ve rsion, http://w w w .proce sste x t.com/a bcchm.html
di a bet es me llit us ; HD L, hi g h-d ens ity lip opr ot e i n; IFG , im pai red fas tin g gl u cos e; IGT , im pai red gl u cos e
Pa g e 1 9 8 5
ABC Ambe r CHM Conve rte r Tria l ve rsion, http://w w w .proce sste x t.com/a bcchm.html
t ol er anc e. Us ed wi t h per mi s sio n fro m Scr ee ni n g for di a bet es . Di a bet es Car e 20 02; 25: 21 – 24. o
Pa g e 1 9 8 6
ABC Ambe r CHM Conve rte r Tria l ve rsion, http://w w w .proce sste x t.com/a bcchm.html o
c. Related syndromes
(1) Impaired fasting glucose or impaired glucose tolerance (see T able 9-13). Thi s i s a di s order of gl ucos e met abol i s m i n whi ch bl ood gl ucos e l evel s are hi gher t han t hos e of normal i ndi vi dual s but l ower t han t hos e of pat i ent s wi t h di abet es . The ri s k of devel opment of di abet es i s i ncreas ed i n affect ed i ndi vi dual s .
TABLE 9-13 Glucose Levels as Diagnostic Criteria for Diabetes Mellitus An y on e of the foll ow ing thr ee crit eri a mu st be me t to
Pa g e 1 9 8 7
ABC Ambe r CHM Conve rte r Tria l ve rsion, http://w w w .proce sste x t.com/a bcchm.html
ma ke a di a gn os i s of di a bet es me llit us . Th e di a gn os i s s ho ul d be con fi r me d by obt ai n i ng t he sa me res
Pa g e 1 9 8 8
ABC Ambe r CHM Conve rte r Tria l ve rsion, http://w w w .proce sste x t.com/a bcchm.html
ul t s (or fi n di n g an ot h er a bn or ma lity ) on a di ff ere nt day . Fas tin g pl a sm a gl u cos e l ev el â ‰
Pa g e 1 9 8 9
ABC Ambe r CHM Conve rte r Tria l ve rsion, http://w w w .proce sste x t.com/a bcchm.html
¥1 26 mg /dL (pr efe rre d t es t )* Pl a sm a gl u cos e l ev el â ‰ ¥2 00 mg /dL 2 ho urs aft er a 75g gl u cos e
Pa g e 1 9 9 0
ABC Ambe r CHM Conve rte r Tria l ve rsion, http://w w w .proce sste x t.com/a bcchm.html
l oa d (or al gl u cos e t ol era nce t es t)
â€
Ca s ua l pl a sm a gl u cos e l ev el (i .e ., wi t ho ut reg ard to foo d i nt ake
Pa g e 1 9 9 1
ABC Ambe r CHM Conve rte r Tria l ve rsion, http://w w w .proce sste x t.com/a bcchm.html
) 20 0 mg /dL or gre at e r (va lid onl y if sy mp to ms s uc h as pol yur i a, pol ydi ps i a, or wei ght l os s are pre s en
Pa g e 1 9 9 2
ABC Ambe r CHM Conve rte r Tria l ve rsion, http://w w w .proce sste x t.com/a bcchm.html
t) *A fas tin g pl a sm a gl u cos e l ev el >1 10 mg /dL but <1 26 mg /dL i nd i ca t es im pai re d fas tin g glu cos e.
Pa g e 1 9 9 3
ABC Ambe r CHM Conve rte r Tria l ve rsion, http://w w w .proce sste x t.com/a bcchm.html
â€
A
pl a sm a gl u cos e l ev el â ‰ ¥1 40 mg /dL but < 20 0 mg /dL 2 ho urs aft er a 75g gl u cos e l oa d i nd
Pa g e 1 9 9 4
ABC Ambe r CHM Conve rte r Tria l ve rsion, http://w w w .proce sste x t.com/a bcchm.html
i ca t es im pai re d glu cos e tol er an ce.
(2) Gestational diabetes. Di abet es or i mpai red gl ucos e t ol erance devel ops i n 2%–3% of pregnant , previ ous l y nondi abet i c women, mos t oft en i n t he l as t t ri mes t er of pregnancy.
(a) β-Cel l res erve i s apparent l y i nadequat e for t he i ncreas ed i ns ul i n requi rement s of pregnancy.
(b) Careful s creeni ng for ges t at i onal di abet es and i nt ens i ve t reat ment are es s ent i al becaus e of an i ncreas ed ri s k of neonat al morbi di t y.
(c) The gl ucos e t ol erance of mos t pat i ent s ret urns t o normal wi t hi n a few weeks aft er del i very, al t hough di abet es
Pa g e 1 9 9 5
ABC Ambe r CHM Conve rte r Tria l ve rsion, http://w w w .proce sste x t.com/a bcchm.html
devel ops i n many pat i ent s as l ong as 5–15 years l at er.
(3) Previous abnormality of glucose tolerance. Thi s t erm refers t o i ndi vi dual s wi t h normal gl ucos e l evel s who formerl y were gl ucos e i nt ol erant or di abet i c becaus e of pregnancy, i l l nes s , obes i t y, or medi cat i ons .
(4) Potential abnormality of glucose tolerance. Thi s refers t o i ndi vi dual s wi t h an i ncreas ed ri s k of fut ure di abet es becaus e of a hi s t ory of havi ng had l arge babi es (>9 pounds ), t he pres ence of di abet es i n an i dent i cal t wi n, or s i mi l ar fact ors .
(5) Di abet es or i mpai red gl ucos e t ol erance may occur secondary t o cert ai n di s eas es t hat affect t he product i on or act i on of i ns ul i n, s uch as chronic pancreatitis, Cushing' s syndrome, acromegaly, insulin receptor abnormalities, and ot hers .
(6) T he metabolic syndrome (i ns ul i n res i s t ance s yndrome, s yndrome X). Some met abol i c and pat hophys i ol ogi c abnormal i t i es t end t o cl us t er i n cert ai n pat i ent s . Ins ul i n res i s t ance may be a common abnormal i t y, perhaps t he pri mary one. Component s of t hi s s yndrome i ncl ude di abet es mel l i t us , obes i t y, hypert ens i on, dys l i pi demi a, and at heros cl eros i s .
Pa g e 1 9 9 6
ABC Ambe r CHM Conve rte r Tria l ve rsion, http://w w w .proce sste x t.com/a bcchm.html
2. Etiology (Onl i ne Tabl e 9-12). The caus e of di abet es mel l i t us i s unknown. Many et i ol ogi c fact ors are s us pect ed, wi t h major di fferences bet ween t hos e fact ors t hat are et i ol ogi c for t ype 1 and t ype 2 di abet es . o
o
a. T ype 1 diabetes. How t he fol l owi ng fact ors i nt eract t o caus e di abet es i s s pecul at i ve. For exampl e, a vi ral i nfect i on may t ri gger β-cel l des t ruct i on i n an i ndi vi dual wi t h genet i cal l y det ermi ned s us cept i bi l i t y t o s uch an i nfect i on and t o aut oi mmune react i vi t y t o i s l et -cel l ant i gens .
(1) Genetic factors
(a) Fi ft y percent of i dent i cal t wi ns of pat i ent s wi t h t ype 1 di abet es are di abet i c.
(b) There i s a s t rong as s oci at i on bet ween t ype 1 di abet es and cert ai n human l eukocyt e ant i gens (HLAs ) (s ee Tabl e 9-11).
(2) Autoimmune factors
(a) Ant i bodi es t o i s l et -cel l ant i gens (i ns ul i n, gl ut ami c aci d decarboxyl as e, and i ns ul i n-as s oci at ed prot ei n 2) can precede i t s ons et and are commonl y pres ent i n di abet i c pat i ent s s hort l y aft er
Pa g e 1 9 9 7
ABC Ambe r CHM Conve rte r Tria l ve rsion, http://w w w .proce sste x t.com/a bcchm.html
t he di s eas e i s di agnos ed, al t hough t hey us ual l y di s appear wi t hi n a few years .
(b) Ant i bodi es agai ns t ot her t i s s ues s uch as ant i t hyroi d ant i bodi es al s o are i ncreas ed.
(3) Environmental factors
(a) The concordance rat e for di abet es i n i dent i cal t wi ns i s 50% rat her t han 100%, whi ch woul d be t he rat e predi ct ed i f t he di s eas e were t ot al l y genet i c.
(b) Seas onal occurrence has been obs erved, wi t h an i ncreas ed di agnos i s of new cas es i n t he fal l and wi nt er.
o
o
b. T ype 2 diabetes
(1) Genetic factors are even more i mport ant et i ol ogi cal l y i n t ype 2 t han i n t ype 1 di abet es . The concordance rat e for di abet es i n i dent i cal t wi ns i s 90%–100%.
(2) Obesity i s a s i gni fi cant fact or—80% of pat i ent s wi t h t ype 2 di abet es are more t han 15% above t hei r i deal wei ght . Obes i t y i s as s oci at ed wi t h res i s t ance t o t he act i on of i ns ul i n bot h i n di abet i c and nondi abet i c
Pa g e 1 9 9 8
ABC Ambe r CHM Conve rte r Tria l ve rsion, http://w w w .proce sste x t.com/a bcchm.html
i ndi vi dual s ; t hi s res i s t ance i s caus ed mai nl y by abnormal i ns ul i n act i on beyond t he recept or.
(3) Genet i cal l y s us cept i bl e i ndi vi dual s may be unabl e t o s us t ai n t he i ncreas ed i ns ul i n product i on needed t o mai nt ai n carbohydrat e homeos t as i s i n t he face of i ns ul i n res i s t ance, wi t h res ul t i ng di abet es . Online T able 9-12 l i s t s ri s k fact ors for t ype 2 di abet es .
(4) Genetic defects of beta-cell function.
(a) Type 2 di abet es can be pres ent at a young age. It has been es t i mat ed t o repres ent 2% of pat i ent s wi t h t ype 2 di abet es . Thi s i s a di s t i nct group of i ndi vi dual s , not rel at ed t o t he epi demi c of obes i t y and di abet es , but t o an aut os omal domi nant t rans mi s s i on. They us ual l y have mi l d di s eas e. It has been cal l ed mat uri t y-ons et di abet es of t he young (MODY ). Thes e pat i ent s are qui t e het erogenous . There are members of t he fami l y t hat have t he defect but who do not devel op di abet es .
(b) Several genet i c abnormal i t i es have been i dent i fi ed; t ypes 3 and 2 account for 65% and 15% of cas es , res pect i vel y. Ot her s ubt ypes of t ype 1 and t ype 2
Pa g e 1 9 9 9
ABC Ambe r CHM Conve rte r Tria l ve rsion, http://w w w .proce sste x t.com/a bcchm.html
di abet es wi l l probabl y become more cl earl y defi ned i n t he fut ure.
(c) There i s a group of i ndi vi dual s who are di agnos ed wi t h aut oi mmune di abet es when t hey are adul t s and mi ght not requi re i ns ul i n i ni t i al l y. Phenot ypi cal l y t hey l ook l i ke pat i ent s wi t h t ype 2 di abet es . Thi s t ype of di abet es has been named l at ent aut oi mmune di abet es of adul t s , or LADA. It al s o has been cal l ed s l owl y progres s i ng i ns ul i n-dependent di abet es or 1.5 di abet es . It i s pres ent i n about 10% of newl y di agnos ed non–i ns ul i n-requi ri ng di abet i cs i n t he Uni t ed St at es . The pres ence of gl ut ami c aci d decarboxyl as e (GAD)-ant i bodi es hel ps t o make t he di agnos i s .
o
o
3. Pathophysiology
a. Levels of insulin
(1) In type 1 diabetes, s ome i ns ul i n may be produced for a few years aft er t he di s eas e i s di agnos ed, but i ns ul i n product i on event ual l y ceas es t ot al l y.
(2) In type 2 diabetes, i ns ul i n l evel s vary and oft en are s i mi l ar t o t he l evel s i n nondi abet i c i ndi vi dual s of s i mi l ar wei ght . However, t hes e i ns ul i n l evel s are
Pa g e 2 0 0 0
ABC Ambe r CHM Conve rte r Tria l ve rsion, http://w w w .proce sste x t.com/a bcchm.html
l ow when cons i dered i n rel at i on t o t he el evat ed bl ood gl ucos e concent rat i ons of di abet i c pat i ent s , and t hes e l evel s refl ect a decreas e i n β-cel l res pons i venes s t o gl ucos e.
b. Consequences of impaired insulin action. Impai red i ns ul i n act i on may be caus ed by i nadequat e i ns ul i n s ecret i on, by t arget -t i s s ue res i s t ance t o t he act i on of i ns ul i n, or bot h.
(1) Hyperglycemia. Ins ul i n i ncreas es t he s ynt hes i s of gl ycogen i n t he l i ver and i n mus cl e and i ncreas es t he upt ake of gl ucos e i n mus cl e and adi pos e t i s s ue. In t he abs ence of adequat e P.408
i ns ul i n act i on, hepat i c gl ucos e product i on i ncreas es (wi t h i ncreas ed gl ycogenol ys i s and i ncreas ed gl uconeogenes i s ) and peri pheral gl ucos e us e decreas es . The res ul t i s hypergl ycemi a.
(2) “ Glucose toxicity.― Hypergl ycemi a i s t hought t o i ni t i at e a s el f-perpet uat i ng cycl e i n whi ch el evat ed gl ucos e l evel s furt her i mpai r bot h t he abi l i t y of t he β cel l s t o produce i ns ul i n and t he act i on of i ns ul i n on peri pheral t i s s ues , l eadi ng t o a furt her ri s e i n s erum gl ucos e l evel s .
Pa g e 2 0 0 1
ABC Ambe r CHM Conve rte r Tria l ve rsion, http://w w w .proce sste x t.com/a bcchm.html
(3) Other metabolic derangements
(a) Ins ul i n normal l y act s as an anabol i c, energy s t orage–promot i ng agent . In addi t i on t o t he s t orage of gl ucos e as gl ycogen, i ns ul i n s t i mul at es t he format i on of fat t y aci ds from gl ucos e, t he es t eri fi cat i on of fat t y aci ds t o form t ri gl yceri des , and t he s t orage of ami no aci ds as prot ei n.
(b) Inadequat e i ns ul i n act i on on t arget t i s s ues caus es i nadequat e di s pos al of i nges t ed nut ri ent s and exces s i ve cons umpt i on of endogenous met abol i c fuel s . Bl ood fat t y aci ds and l i pi ds are i ncreas ed becaus e of decreas ed l i pogenes i s and i ncreas ed l i pol ys i s ; bl ood ami no aci ds are i ncreas ed becaus e of decreas ed prot ei n s ynt hes i s and i ncreas ed cat abol i s m of mus cl e prot ei n.
c. Levels of other hormones
(1) Glucagon l evel s are oft en el evat ed i n pat i ent s wi t h t ype 2 di abet es . Thi s may cont ri but e t o hypergl ycemi a t hrough t he act i on of gl ucagon i n s t i mul at i ng
Pa g e 2 0 0 2
ABC Ambe r CHM Conve rte r Tria l ve rsion, http://w w w .proce sste x t.com/a bcchm.html
gl ycogenol ys i s .
(2) Epinephrine, cortisol, and GH l evel s may be i ncreas ed duri ng peri ods of s t res s or poor di abet i c cont rol . Thi s may cont ri but e t o hypergl ycemi a t hrough t he ant i -i ns ul i n effect and di abet ogeni c act i on of t hes e hormones .
o
o
4. Clinical features
a. Polyuria and polydipsia us ual l y occur. The mos t common s ympt om of hypergl ycemi a i s i ncreas ed uri ne vol ume, whi ch i s caus ed by gl ucos e-i nduced os mot i c di ures i s . Increas ed fl ui d i nt ake i s a res pons e t o t he res ul t i ng dehydrat i on and t hi rs t .
b. Weight loss res ul t s from t he l os s of gl ucos e i n uri ne and t he cat abol i c effect s of t he decreas e i n i ns ul i n act i on, des pi t e i ncreas ed food i nt ake. General i zed weaknes s al s o refl ect s t he met abol i c derangement s .
c. Infections of the skin, vulva, and urinary tract are es peci al l y common i n uncont rol l ed di abet es , becaus e hypergl ycemi a decreas es res i s t ance t o i nfect i on.
d. Blurring of vision i s caus ed by changes i n t he s hape and refract i ve qual i t i es of t he opt i c
Pa g e 2 0 0 3
ABC Ambe r CHM Conve rte r Tria l ve rsion, http://w w w .proce sste x t.com/a bcchm.html
l ens t hat res ul t from hypergl ycemi a-i nduced os mot i c al t erat i ons . o
o
5. Diagnosis. Di abet es mel l i t us i s oft en s us pect ed becaus e of t ypi cal cl i ni cal mani fes t at i ons s uch as pol yuri a and unexpl ai ned wei ght l os s ; however, a defi ni t i ve di agnos i s i s bas ed on elevated glucose levels.
a. Glucose levels. Di abet es mel l i t us i s di agnos ed i f any one of t he t hree abnormal i t i es des cri bed i n Tabl e 9-13 i s pres ent . The di agnos i s s houl d be confi rmed by repeat i ng t he fas t i ng gl ucos e on a di fferent day.
b. Urine glucose levels. Gl ucos e appears i n t he uri ne onl y when t he renal t hres hol d of approxi mat el y 180 mg/dL i s exceeded.
(1) Thi s t hres hol d vari es wi del y and t ends t o i ncreas e wi t h age. Therefore, uri ne gl ucos e meas urement i s an i ns ens i t i ve and unrel i abl e t es t for di abet es .
(2) However, uri ne gl ucos e l evel s can be a rough gui de t o t he pres ence or abs ence of marked hypergl ycemi a and current l y i s not bei ng us ed t o manage pat i ent s .
Pa g e 2 0 0 4
ABC Ambe r CHM Conve rte r Tria l ve rsion, http://w w w .proce sste x t.com/a bcchm.html
c. Hemoglobin A1c i s not recommended for di agnos i s , but i s t he mos t effect i ve met hod for moni t ori ng t he t reat ment .
o
o
6. Acute complications of diabetes.
a. Diabetic ketoacidosis occurs i n pat i ent s wi t h t ype 1 di abet es whos e ci rcul at i ng i ns ul i n i s i ns uffi ci ent t o al l ow gl ucos e us e by peri pheral t i s s ue and t o i nhi bi t gl ucos e product i on and t i s s ue cat abol i s m. Increas ed l evel s of gl ucagon and hormones t hat i ncreas e i n res pons e t o s t res s (i .e., epi nephri ne, norepi nephri ne, cort i s ol , and GH) cont ri but e t o t he met abol i c derangement s . It can rarel y occur i n t ype 2 di abet es .
(1) Precipitating factors. Ket oaci dos i s may occur aft er s everal days of wors eni ng di abet i c cont rol or may appear s uddenl y wi t hi n a few hours . P.409
(a) Preci pi t at i ng fact ors i ncl ude any event t hat decreas es i ns ul i n avai l abi l i t y or caus es s t res s t hat i ncreas es t he need for i ns ul i n.
(b) Common fact ors are t he
Pa g e 2 0 0 5
ABC Ambe r CHM Conve rte r Tria l ve rsion, http://w w w .proce sste x t.com/a bcchm.html
omi s s i on of i ns ul i n dos es , i nfect i ons , i njuri es , emot i onal s t res s , exces s i ve al cohol i nges t i on, and i nt ercurrent i l l nes s .
(2) Pathophysiology (Fi gure 9-5)
(a) Hyperglycemia. Ins uffi ci ent i ns ul i n reduces peri pheral gl ucos e ut i l i zat i on and, t oget her wi t h gl ucagon exces s , i ncreas es hepat i c product i on of gl ucos e t hrough t he s t i mul at i on of gl uconeogenes i s and gl ycogenol ys i s and t he i nhi bi t i on of gl ycol ys i s . Prot ei n breakdown i n peri pheral t i s s ues provi des a fl ow of ami no aci ds t o t he l i ver as s ubs t rat e for gl uconeogenes i s . Hypergl ycemi a i s t he res ul t .
(b) Osmotic diuresis. Thi s condi t i on, whi ch res ul t s from t he el evat ed s erum gl ucos e (and ket one) l evel s , produces hypovolemia, dehydration, and loss of sodium, potassium, phosphate, and other substances in the urine. Vol ume depl et i on s t i mul at es cat echol ami ne rel eas e, whi ch furt her oppos es i ns ul i n act i on i n t he l i ver and cont ri but es t o l i pol ys i s .
Pa g e 2 0 0 6
ABC Ambe r CHM Conve rte r Tria l ve rsion, http://w w w .proce sste x t.com/a bcchm.html
(c) Ketogenesis. The l i pol ys i s t hat res ul t s from i ns ul i n l ack and cat echol ami ne exces s mobi l i zes free fat t y aci ds from t hei r s t ores i n adi pos e t i s s ue. Ins t ead of re-es t eri fyi ng t he i ncomi ng fat t y aci ds t o form t ri gl yceri des , t he l i ver s hi ft s i t s met abol i c pat hways t oward t he product i on of ket one bodi es .
(i) Gl ucagon i ncreas es t he hepat i c l evel of carni t i ne, whi ch enabl es fat t y aci ds t o ent er t he mi t ochondri a where t hey undergo β-oxi dat i on t o ket one bodi es .
(ii) Gl ucagon decreas es t he hepat i c cont ent of mal onyl coenzyme A, an i nhi bi t or of fat t y aci d oxi dat i on.
(d) Acidosis. The i ncreas ed hepat i c product i on of ket one bodi es (acet oacet at e and β-hydroxybut yrat e) exceeds t he body's abi l i t y t o met abol i ze or excret e t hem.
(i) The hydrogen i ons of t he ket one bodi es are buffered by
Pa g e 2 0 0 7
ABC Ambe r CHM Conve rte r Tria l ve rsion, http://w w w .proce sste x t.com/a bcchm.html
bi carbonat e, l eadi ng t o a decreas e i n s erum bi carbonat e and pH.
(ii) Art eri al carbon di oxi de t ens i on (PaCO 2 ) al s o decreas es becaus e of vent i l at ory compens at i on.
(iii) The ani on gap i ncreas es becaus e of t he el evat ed pl as ma l evel s of acet oacet at e and β-hydroxybut yrat e.
(iv) The res ul t i s met abol i c aci dos i s t hat i s as s oci at ed wi t h an i ncreas ed ani on gap.
(3) Clinical features and diagnosis
(a) Physical findings
(i) Rapid, deep breathing (Kussmaul' s respirations) occurs as t he body t ri es t o compens at e for met abol i c aci dos i s by i ncreas i ng carbon di oxi de excret i on.
Pa g e 2 0 0 8
ABC Ambe r CHM Conve rte r Tria l ve rsion, http://w w w .proce sste x t.com/a bcchm.html
FIGURE 9-5 Pat hogenes i s of di abet i c ket oaci dos i s . (From Adl i n EV. Endoc ri nol ogy Sc i enc e and Medi c i ne. Phi l adel phi a: Li ppi ncot t W i l l i ams & W i l ki ns , 2001:80. )
P.410
(ii) An odor of acetone i s oft en det ect ed on t he breat h.
(iii) Marked dehydrat i on i s common, wi t h dry skin and mucous membranes and poor skin turgor. Orthostatic hypotension may be pres ent becaus e of i nt ravas cul ar vol ume depl et i on.
(iv) Clouding of consciousness i s pres ent i n mos t cas es , and approxi mat el y 10% of pat i ent s are
Pa g e 2 0 0 9
ABC Ambe r CHM Conve rte r Tria l ve rsion, http://w w w .proce sste x t.com/a bcchm.html
comatose.
(b) Laboratory abnormalities
(i) Hyperglycemia. Serum gl ucos e l evel s i n ket oaci dos i s may be onl y s l i ght l y i ncreas ed, but more oft en t hey are markedl y el evat ed, averagi ng approxi mat el y 500 mg/dL. Renal funct i on affect s t he degree of hypergl ycemi a; gl ucos e l evel s are great l y el evat ed onl y when uri nary excret i on of gl ucos e i s l i mi t ed by vol ume depl et i on or renal abnormal i t i es .
(ii) Hyperketonemia. Serum l evel s of acet oacet at e, acet one, and β-hydroxybut yrat e are great l y i ncreas ed. The agent ni t roprus s i de, i n t he form of t abl et s or reagent s t ri ps , i s commonl y us ed t o meas ure s erum and uri ne ket one bodi es . It react s onl y wi t h acet oacet at e. If t he ot her ket one bodi es are i ncreas ed t o a much great er or much l es s er ext ent t han acet oacet at e, t he res ul t s of t he t es t may be
Pa g e 2 0 1 0
ABC Ambe r CHM Conve rte r Tria l ve rsion, http://w w w .proce sste x t.com/a bcchm.html
mi s l eadi ng.
(iii) Metabolic acidosis. T he serum bicarbonate level is low (us ual l y <10 mEq/L). The blood pH is low, and t he anion gap is increased.
(iv) Glycosuria and ketonuria. Uri nary l evel s of gl ucos e and ket one bodi es are i ncreas ed. The di agnos i s of di abet i c ket oaci dos i s can be made rapi dl y i f marked gl ycos uri a and ket onuri a are pres ent .
(v) Other laboratory findings. Serum potassium concentration may be i ncreas ed i ni t i al l y i n met abol i c aci dos i s becaus e of pot as s i um i on movement from t he i nt racel l ul ar t o t he ext racel l ul ar s pace. Lat er, t he s erum pot as s i um l evel i s l ow becaus e of bot h renal l os s es and t he movement of pot as s i um i ons back i nt o cel l s as t he aci dos i s i s correct ed. Serum sodium concentration t ends t o be l ow, mai nl y becaus e of di l ut i on as t he
Pa g e 2 0 1 1
ABC Ambe r CHM Conve rte r Tria l ve rsion, http://w w w .proce sste x t.com/a bcchm.html
os mot i c effect of t he hypergl ycemi a i ncreas es ext racel l ul ar wat er. Serum osmolality i s hi gh, us ual l y great er t han 300 mOs m/kg.
(4) T herapy. The t reat ment of di abet i c ket oaci dos i s has four mai n component s .
(a) Insulin i s admi ni s t ered t o i ncreas e gl ucos e us e i n t he t i s s ues , t o i nhi bi t t he fl ow of fat t y aci ds and ami no aci ds from t he peri phery, and t o count er t he effect s of gl ucagon on t he l i ver.
(i) Route of administration. If vol ume depl et i on and vas cul ar col l aps e are pres ent , poor t i s s ue perfus i on i mpai rs t he abs orpt i on of i nt ramus cul ar or s ubcut aneous i ns ul i n, and i ns ul i n i s us ual l y admi ni s t ered i nt ravenous l y. If no i nt ravenous acces s i s avai l abl e, t he i nt ramus cul ar rout e can be us ed.
(ii) Dosage. A pri mi ng dos e of 0.1 U/kg regul ar i ns ul i n i s gi ven i nt ravenous l y and i s fol l owed by t he i nfus i on of 0.1 U/kg/hr or approxi mat el y 5–10 U/hr. If t he s erum gl ucos e l evel does not decreas e (75–100 mg/dL/hr), t he s erum l evel of ket one bodi es does
Pa g e 2 0 1 2
ABC Ambe r CHM Conve rte r Tria l ve rsion, http://w w w .proce sste x t.com/a bcchm.html
not fal l , and s erum pH does not i ncreas e i n a few hours , t hen l arger dos es of i ns ul i n mus t be gi ven.
(iii) Aft er t he aci dos i s and hypergl ycemi a have res ol ved and t he uri ne has become free of ket one bodi es , t reat ment wi t h i nt ermedi at e-act i ng i ns ul i n i s res umed.
(b) Fluid replacement correct s t he dehydrat i on caus ed by gl ucos e-i nduced os mot i c di ures i s . The fl ui d defi ci t i n pat i ent s wi t h di abet i c ket oaci dos i s averages 3–5 L, whi ch mus t be prompt l y repl aced.
(i) Approxi mat el y 1 L normal s al i ne (0.9% NaCl ) i s gi ven each hour for t he fi rs t 2 hours , and t hen hal f-normal s al i ne (0.45% NaCl ) i s gi ven at a s l ower rat e. W hen t he s erum gl ucos e l evel fal l s t o 200–300 mg/dL, 5% or 10% gl ucos e i s i nfus ed t o prevent hypogl ycemi a.
(ii) Fl ui d repl acement l owers s erum gl ucos e l evel s , even wi t hout i ns ul i n, by i ncreas i ng uri ne fl ow (and hence, gl ycos uri a) and by decreas i ng t he l evel s of
Pa g e 2 0 1 3
ABC Ambe r CHM Conve rte r Tria l ve rsion, http://w w w .proce sste x t.com/a bcchm.html
cat echol ami nes and cort i s ol , whi ch were i ncreas ed by t he s t i mul us of vol ume depl et i on. It al s o i mproves t he aci dos i s by l oweri ng t he l act i c aci d. P.411
(c) Minerals and electrolytes must be replaced becaus e t hey are l os t vi a os mot i c di ures i s .
(i) Potassium may be needed t o repl ace l ow body s t ores . If i ni t i al s erum pot as s i um l evel s are el evat ed (due t o s evere aci dos i s ), repl acement i s del ayed; when t he l e vel s become normal or l ow, aft er t herapy has been i ni t i at ed, pot as s i um chl ori de i s i nfus ed at a rat e of 20–40 mmol /hr. Thi s can be gi ven as pot as s i um phos phat e or pot as s i um chl ori de, dependi ng on t he bl ood l evel s of cal ci um and phos phorus ; t oo much phos phat e repl acement may caus e hypocal cemi a.
(ii) Phosphate repl acement i s recommended, al t hough no benefi t of phos phat e admi ni s t rat i on has been demons t rat ed unl es s i t i s very
Pa g e 2 0 1 4
ABC Ambe r CHM Conve rte r Tria l ve rsion, http://w w w .proce sste x t.com/a bcchm.html
l ow. Approxi mat el y 10–20 mmol /hr may be gi ven as pot as s i um phos phat e, for a t ot al of 40–60 mmol .
(iii) Bicarbonate i s gi ven onl y when t he art eri al pH decl i nes bel ow 6.9 t o mai nt ai n t he pH above t hat l evel . Becaus e di abet i c ket oaci dos i s i s correct ed by fl ui ds and i ns ul i n, exces s i ve admi ni s t rat i on of bi carbonat e may res ul t i n rebound al kal os i s and hypokal emi a. Some phys i ci ans recommend bi carbonat e admi ni s t rat i on onl y when t he pH decreas es bel ow 7.1, when hemodynami cal l y uns t abl e, or i f hyperkal emi a wi t h el ect rocardi ographi c changes i s pres ent .
(d) T reatment of precipitating factors and complications
(i) Uri nary t ract i nfect i ons as wel l as ot her i nfect i ons mus t be i nves t i gat ed and t reat ed.
(ii) Meni ngi t i s , s t roke, and myocardi al i nfarct i on s houl d be cons i dered becaus e t hey may es cape det ect i on i n pat i ent s whos e s ens ori ums are cl ouded by
Pa g e 2 0 1 5
ABC Ambe r CHM Conve rte r Tria l ve rsion, http://w w w .proce sste x t.com/a bcchm.html
ket oaci dos i s .
(iii) If pat i ent s are uncons ci ous and have been vomi t i ng or have gas t ri c di l at i on, nas ogas t ri c as pi rat i on s houl d be performed.
(iv) If hypot ens i on pers i s t s des pi t e fl ui d repl acement , bl ood or pl as ma expanders s houl d be gi ven.
o
o
b. Hyperosmolar nonketotic coma i s l es s common t han ket oaci dos i s but has a much hi gher mort al i t y rat e. It occurs pri mari l y i n el derl y pat i ent s wi t h t ype 2 di abet es , oft en i n previ ous l y undi agnos ed i ndi vi dual s .
(1) Pathophysiology. Oft en a preci pi t at i ng fact or (e.g., i nfect i on, i ncreas ed gl ucos e i nges t i on, omi s s i on of i ns ul i n, or i nt ercurrent i l l nes s ) caus es i ncreas i ng hypergl ycemi a wi t hi n a few days or weeks . Os mot i c di ures i s , wi t hout adequat e fl ui d i nt ake, caus es dehydrat i on and progres s i ve decl i ne i n ment al s t at us . Ket oaci dos i s i s mi l d or abs ent , pres umabl y becaus e s uffi ci ent i ns ul i n i s pres ent t o i nhi bi t hepat i c ket ogenes i s .
(2) Clinical features
(a) The hyperglycemia t ends t o be more
Pa g e 2 0 1 6
ABC Ambe r CHM Conve rte r Tria l ve rsion, http://w w w .proce sste x t.com/a bcchm.html
marked t han i t i s i n ket oaci dos i s , wi t h average pl as ma gl ucos e concent rat i ons of approxi mat el y 1000 mg/dL.
(b) In t he abs ence of ket oaci dos i s , t he os mot i c di ures i s cont i nues for a l onger t i me before t he di agnos i s i s made and t herefore produces more s evere dehydration.
(c) Serum osmolality i s very hi gh, averagi ng approxi mat el y 360 mOs m/kg. The dehydrat i on and hyperos mol al i t y may caus e mental obtundation, seizures, and focal neurologic signs.
(d) Lact i c aci dos i s may be pres ent becaus e of t he hypovol emi a and i ndi cat es a wors e prognos i s .
(3) T herapy. Treat ment i s s i mi l ar t o t hat for di abet i c ket oaci dos i s . Fl ui d repl acement and revers al of t he hypergl ycemi a wi t h i ns ul i n are t he mai n goal s .
(a) Fluid replacement i n el derl y pat i ent s wi t h cardi ovas cul ar di s eas e requi res care t o avoi d vol ume expans i on, whi ch mi ght preci pi t at e heart fai l ure.
(b) Insulin s houl d not be s t art ed unt i l
Pa g e 2 0 1 7
ABC Ambe r CHM Conve rte r Tria l ve rsion, http://w w w .proce sste x t.com/a bcchm.html
fl ui d repl acement i s underway; a fal l i n pl as ma gl ucos e l evel s may wors en t he hypovol emi a and preci pi t at e s hock due t o t he l os s of t he i nt ravas cul ar vol ume-expandi ng effect of t he hypergl ycemi a. o
o
c. Hypoglycemic coma mus t be rapi dl y di fferent i at ed from di abet i c ket oaci dos i s and hyperos mol ar nonket ot i c coma, becaus e t herapy i s obvi ous l y qui t e di fferent .
(1) Etiology
(a) Hypogl ycemi a i n i ns ul i n-t reat ed di abet i c pat i ent s (“i ns ul i n s hock―) may be caus ed by excessive insulin dosage, delay in the ingestion of a meal, and excessive physical P.412
activity. Sul fonyl ureas may caus e hypogl ycemi c react i ons , but much l es s oft en t han does i ns ul i n.
(b) Pat i ent s wi t h t ype 1 di abet es may be s us cept i bl e t o hypogl ycemi a becaus e of insufficient levels of the counterregulatory hormones t hat normal l y l i mi t t he fal l i n s erum gl ucos e. The res pons e of gl ucagon t o hypogl ycemi a i s frequent l y i mpai red i n t hes e pat i ent s , and epi nephri ne product i on may
Pa g e 2 0 1 8
ABC Ambe r CHM Conve rte r Tria l ve rsion, http://w w w .proce sste x t.com/a bcchm.html
become i mpai red i f aut onomi c neuropat hy devel ops . (Epi nephri ne prevent s s evere hypogl ycemi a bot h by s t i mul at i ng gl ucos e product i on and by produci ng s ympt oms t hat al ert t he pat i ent t o hypogl ycemi a, prompt i ng rapi d gl ucos e i nges t i on.) o
o
(2) Pathophysiology and clinical features. Hypogl ycemi a produces s ympt oms t hrough t he fol l owi ng t wo mechani s ms :
(a) A fal l i n t he s erum gl ucos e concent rat i on s t i mul at es cat echol ami ne product i on and s ympat het i c nervous s ys t em out fl ow. Adrenergic stimulation t hen caus es s weat i ng, t achycardi a, pal pi t at i ons , t remul ous nes s , and mus cul ar weaknes s .
(b) Prol onged hypogl ycemi a depri ves t he CNS of i t s mai n s ource of fuel , gl ucos e. CNS symptoms of hypogl ycemi a us ual l y occur l at er t han t he adrenergi c s ympt oms , and t hey are pot ent i al l y more s eri ous . Ment al changes may progres s from somnolence and confusion t o coma. Headache, s l urred s peech, focal neurol ogi c s i gns , and s ei zures may occur.
o
o
(3) Diagnosis. The di agnos i s of hypogl ycemi a i s obvi ous i f s ympt oms of s weat i ng, pal pi t at i on, and t remul ous nes s occur at t he t i me of peak act i on of a recent i ns ul i n dos e. Pat i ent s l earn t o recogni ze t hi s react i on and t reat i t by dri nki ng orange jui ce or eat i ng candy. Les s obvi ous i s t he caus e of coma i n
Pa g e 2 0 1 9
ABC Ambe r CHM Conve rte r Tria l ve rsion, http://w w w .proce sste x t.com/a bcchm.html
di abet i c pat i ent s who are brought t o t he emergency depart ment . Cl ues t hat s ugges t hypogl ycemi a rat her t han ket oaci dos i s are t he hi s t ory of a mi s s ed meal or unus ual l y vi gorous exerci s e, t he fi ndi ng of profus e s weat i ng rat her t han dehydrat i on, and t he abs ence of Kus s maul 's res pi rat i ons . A fi ngers t i ck bl ood gl ucos e det ermi nat i on i s us eful for rapi d confi rmat i on of hypogl ycemi a. o
o
(4) T herapy
(a) Pat i ent s who are unabl e t o t ake gl ucos e oral l y are gi ven 50 mL 50% glucose intravenously over 3–5 mi nut es , fol l owed by a cons t ant i nfus i on of 5% or 10% gl ucos e.
(i) Some pat i ent s regai n cons ci ous nes s i mmedi at el y, ot hers more s l owl y.
(ii) Gl ucos e i nfus i on may have t o be mai nt ai ned duri ng t he expect ed durat i on of act i on of t he i ns ul i n or oral agent res pons i bl e for t he hypogl ycemi a. If t he hypogl ycemi a i s caus ed by chlorpropamide, t hi s may be s everal days .
(b) An i nt ramus cul ar i nject i on of 1 mg of glucagon may i ncreas e t he s erum gl ucos e l evel rapi dl y, al l owi ng pat i ent s t o regai n cons ci ous nes s and t ake oral gl ucos e. Teachi ng
Pa g e 2 0 2 0
ABC Ambe r CHM Conve rte r Tria l ve rsion, http://w w w .proce sste x t.com/a bcchm.html
pat i ent s ' fami l y members t o i nject gl ucagon may decreas e t he frequency of emergency depart ment vi s i t s .
(c) Aft er an epi s ode of i ns ul i n-i nduced hypogl ycemi a, t he i ns ul i n dos age, di et , or bot h, s houl d be readjus t ed t o prevent s ubs equent at t acks .
7. Chronic complications of diabetes. o
o
a. Pathogenesis
(1) Microvascular complications. Pat i ent s wi t h di abet es frequent l y devel op mi crovas cul ar di s orders i nvol vi ng t he s mal l bl ood ves s el s of t he eye, ki dney, and nerves . The Di abet es Cont rol and Compl i cat i ons Tri al , a 10-year randomi zed cont rol l ed t ri al i nvol vi ng 1441 pat i ent s wi t h t ype 1 di abet es , has s hown concl us i vel y t hat bet t er cont rol of s erum gl ucos e l evel s reduces t he i nci dence of ret i nopat hy, nephropat hy, and neuropat hy. Several mechani s ms have been propos ed t o expl ai n how el evat ed gl ucos e l evel s may caus e t hes e compl i cat i ons :
(a) Nonenz ymat i c gl ycos yl at i on of prot ei ns i n capi l l ary bas ement membranes and ot her t i s s ues , s i mi l ar t o t he proces s t hat produces gl ycos yl at ed hemogl obi n, may produce damage t o
Pa g e 2 0 2 1
ABC Ambe r CHM Conve rte r Tria l ve rsion, http://w w w .proce sste x t.com/a bcchm.html
t hes e t i s s ues t hat i s rel at ed t o t he bl ood gl ucos e l evel s .
(b) W hen gl ucos e l evel s are el evat ed, t he enzyme al dos e reduct as e convert s gl ucos e t o s orbi t ol , whi ch may caus e damage i n nerve cel l s , t he ret i na, and renal t i s s ue.
(2) Macrovascular complications. At heros cl erot i c di s eas e of t he medi um and l arge ves s el s , l eadi ng t o coronary, cerebrovas cul ar, or peri pheral vas cul ar di s eas e, i s as s oci at ed wi t h P.413
di abet es mel l i t us . Unl i ke t he mi crovas cul ar compl i cat i ons , however, i t i s al s o as s oci at ed wi t h t he l es s er degree of gl ucos e el evat i on known as i mpai red gl ucos e t ol erance. o
o
b. Diabetic retinopathy. Thi s condi t i on i s di rect l y rel at ed t o t he durat i on and s everi t y of di abet es . Preval ence i ncreas es from 3% at t he t i me t hat di abet es i s di agnos ed t o 20%–45% aft er 10 years . Of new cas es of bl i ndnes s i n adul t s , 20% are caus ed by di abet es .
(1) T ypes
(a) Background (simple,
Pa g e 2 0 2 2
ABC Ambe r CHM Conve rte r Tria l ve rsion, http://w w w .proce sste x t.com/a bcchm.html
nonproliferative) retinopathy makes up 90%–95% of al l cas es . Increas ed capi l l ary permeabi l i t y, vas cul ar occl us i on, and weaknes s of s upport i ng s t ruct ures l ead t o t he fi ndi ngs on fundus copi c exami nat i on of venous di l at at i on, exudat es , hemorrhages , and mi croaneurys ms .
(b) Proliferative retinopathy makes up 5%–10% of al l cas es . In res pons e t o vas cul ar occl us i on and i s chemi a, new ves s el s form on t he s urface of t he ret i na (neovas cul ari zat i on) and may grow i nt o t he vi t reous body of t he eye.
(i) Preret i nal or vi t reous hemorrhage may l ead t o cl ot ret ract i on and s car format i on, wi t h ret i nal det achment .
(ii) Vi t reous hemorrhage may caus e s udden bl i ndnes s .
(2) T herapy
(a) Background ret i nopat hy i s l es s l i kel y t o progres s i f di abet i c cont rol i s good. Annual s creeni ng for ret i nopat hy i s recommended, becaus e bl i ndnes s may be prevent ed by earl y t reat ment . Gl ucos e and bl ood pres s ure cont rol are very
Pa g e 2 0 2 3
ABC Ambe r CHM Conve rte r Tria l ve rsion, http://w w w .proce sste x t.com/a bcchm.html
i mport ant .
(b) Prol i ferat i ve ret i nopat hy may be t reat ed wi t h laser-beam photocoagulation, whi ch i s effect i ve i n obl i t erat i ng new ves s el s . Vitrectomy i s benefi ci al i n s el ect ed cas es .
o
o
c. Diabetic nephropathy. The renal l es i on t hat i s s peci fi c for di abet es i s intercapillary glomerulosclerosis (Kimmelstiel-Wilson disease). Ot her renal di s eas es as s oci at ed wi t h di abet es are papi l l ary necros i s , chroni c i nt ers t i t i al nephri t i s , and art eri os cl erot i c di s eas e.
(1) Incidence. Si gni fi cant renal di s eas e devel ops i n approxi mat el y 40% of pat i ent s wi t h t ype 1 di abet es and 20% of pat i ent s wi t h t ype 2 di abet es . Nearl y al l pat i ent s wi t h s evere gl omerul os cl eros i s al s o have ret i nopat hy. Al mos t 33% of new cas es of end-s t age renal di s eas e are caus ed by di abet es .
(2) Pathogenesis. Hypergl ycemi a may caus e i ncreas ed i nt ragl omerul ar pres s ure, l eadi ng t o damage t o t he bas ement membrane, depos i t i on of prot ei n i n t he mes angi um, gl omerul os cl eros i s , and renal fai l ure. Pathologic features of di abet i c gl omerul os cl eros i s i ncl ude an i ncreas e i n t he mes angi al mat ri x and i ncreas ed wi dt h of t he gl omerul ar bas ement membrane, hyal i ne
Pa g e 2 0 2 4
ABC Ambe r CHM Conve rte r Tria l ve rsion, http://w w w .proce sste x t.com/a bcchm.html
art eri os cl eros i s of t he afferent and efferent art eri ol es , and IgG and al bumi n depos i t s l i ni ng t he t ubul ar and gl omerul ar bas ement membranes . Di abet i c ki dneys t end t o be l arge, even when end-s t age renal di s eas e i s pres ent .
(3) Clinical features
(a) The fi rs t mani fes t at i on i s us ual l y proteinuria, whi ch oft en progres s es t o t he nephrotic syndrome.
(i) Microalbuminuria, whi ch can be det ect ed by s peci al t es t s , predi ct s t he l at er occurrence of renal fai l ure.
(ii) The pres ence of hypertension i s al s o as s oci at ed wi t h an i ncreas ed ri s k of renal fai l ure i n di abet i c pat i ent s .
(b) W hen renal fai l ure i s as s oci at ed wi t h nephrot i c range prot ei nuri a, t he progres s i on t o end-stage renal disease i s rapi d; t rans pl ant at i on or di al ys i s us ual l y becomes neces s ary wi t hi n 3 years .
(4) Prevention. Progres s i on of di abet i c nephropat hy can be prevent ed or del ayed by i nt ens i ve gl ycemi c cont rol and by cont rol of
Pa g e 2 0 2 5
ABC Ambe r CHM Conve rte r Tria l ve rsion, http://w w w .proce sste x t.com/a bcchm.html
coexi s t i ng hypert ens i on. Angi ot ens i n-convert i ng enzyme (ACE) i nhi bi t ors and angi ot ens i n recept or bl ockers (ARBs ) s l ow t he progres s i on of renal di s eas e by t hei r ant i hypert ens i ve effect and probabl y by an act i on i ndependent of bl ood pres s ure cont rol . A l ow-prot ei n di et (0.6–0.89 g prot ei n/kg body wei ght ) may al s o s l ow t he progres s i on of renal di s eas e. o
o
d. Diabetic neuropathy. Ol der pat i ent s wi t h a rel at i vel y l ong hi s t ory of di abet es and s evere hypergl ycemi a have an i ncreas ed i nci dence of t hi s common compl i cat i on. Accumul at i on of s orbi t ol i n Schwann cel l s , wi t h s ubs equent cel l damage, may pl ay a caus at i ve rol e. Sl owi ng of P.414
nerve conduct i on vel oci t y occurs , wi t h changes i n Schwann cel l funct i on and event ual s egment al demyel i nat i on and axonal degenerat i on.
(1) T ypes
(a) Peripheral polyneuropathy i s t he mos t common s yndrome.
(i) Di s t al , bi l at eral s ens ory changes i n t he l ower ext remi t i es predomi nat e. W eaknes s and upper ext remi t y i nvol vement are l es s frequent . Neuropat hi c ul cers of t he
Pa g e 2 0 2 6
ABC Ambe r CHM Conve rte r Tria l ve rsion, http://w w w .proce sste x t.com/a bcchm.html
feet are a common mani fes t at i on of di abet i c neuropat hy and are more common t han i s chemi c ul cers .
(ii) Symptoms i ncl ude pares t hes i as and pai n of t he feet . Exami nat i on may s how decreas ed refl exes , l os s of vi brat ory s ens e, and l os s of pai n s ens at i on.
(b) Autonomic neuropathy i s l es s common t han peri pheral pol yneuropat hy, but us ual l y i t i s s een i n pat i ent s who have peri pheral pol yneuropat hy. Postural hypotension i s a major mani fes t at i on. Ot her cl i ni cal l y i mport ant probl ems are s exual i mpot ence i n di abet i c men and uri nary ret ent i on wi t h abnormal bl adder funct i on. Abnormal gas t roi nt es t i nal mot i l i t y may res ul t i n del ayed gas t ri c empt yi ng (di abet i c gas t ropares i s ), cons t i pat i on, and di arrhea. The adrenergi c s ympt oms of hypogl ycemi a may be decreas ed or abs ent , l eadi ng t o del ayed recogni t i on and t reat ment of i ns ul i n react i ons .
(c) Les s common forms of di abet i c neuropat hy are radiculopathy, caus i ng l anci nat i ng pai n i n a s i ngl e dermat ome, and mononeuropathy, i nvol vi ng crani al nerves or proxi mal mot or nerves .
Pa g e 2 0 2 7
ABC Ambe r CHM Conve rte r Tria l ve rsion, http://w w w .proce sste x t.com/a bcchm.html
(2) T herapy
(a) Improved di abet i c cont rol may l es s en t he s ympt oms of peri pheral pol yneuropat hy.
(b) Sympt omat i c t reat ment of pai nful neuropat hy may be at t empt ed wi t h ami t ri pt yl i ne, phenyt oi n, carbamazepi ne, t opi cal caps ai ci n, gabapent i n, pregabal i n, or dul oxet i ne. Narcot i cs mi ght be needed.
o
o
e. Atherosclerosis. The i nci dence of at heros cl eros i s i s cons i derabl y i ncreas ed i n di abet i c pat i ent s ; t hat i s , di abet es , l i ke hypert ens i on, s moki ng, hyperl i pi demi a, obes i t y, and a pos i t i ve fami l y hi s t ory, i s a major ri s k fact or for t he devel opment of at heros cl eros i s .
(1) Coronary artery disease i s t wi ce as common i n di abet i c pat i ent s compared wi t h nondi abet i c pat i ent s . Smal l -ves s el di s eas e may cont ri but e t o myocardi al i s chemi a.
(2) Peripheral vascular disease, whi ch i s common i n di abet i c pat i ent s , i s mos t l i kel y t o affect t he l egs and feet . Smal l -ves s el di s eas e may pl ay a major rol e—i s chemi c changes i n a foot wi t h a normal pedal pul s e on exami nat i on i s t ypi cal of di abet es . Foot i nfect i ons , poorl y
Pa g e 2 0 2 8
ABC Ambe r CHM Conve rte r Tria l ve rsion, http://w w w .proce sste x t.com/a bcchm.html
heal i ng ul cers , and event ual gangrene, res ul t i ng i n amput at i on, are frequent compl i cat i ons .
8. T reatment of diabetes o
o
a. Goals of treatment
(1) Control of symptoms. The pol yuri a, wei ght l os s , i ncreas ed i nci dence of i nfect i ons , bl urri ng of vi s i on, and ot her s ympt oms of di abet es are rel at ed t o t he hypergl ycemi a. Ret urn of s erum gl ucos e l evel s t o normal bri ngs rel i ef of t hes e s ympt oms .
(2) Prevention of acute complications. Di abet i c ket oaci dos i s and nonket ot i c hypergl ycemi c coma are prevent ed by careful management of di abet es .
(3) Prevention of long-term complications. The Di abet es Cont rol and Compl i cat i ons Tri al and t he Uni t ed Ki ngdom Pros pect i ve Di abet es St udy cl earl y s howed t hat i nt ens i ve t herapy, wi t h l oweri ng of bl ood gl ucos e bel ow t he l evel s needed t o cont rol s ympt oms and prevent ket oaci dos i s , del ays or prevent s t he ons et of ret i nopat hy, nephropat hy, neuropat hy, and macrovas cul ar compl i cat i ons .
o
o
b. Diet. Before i ns ul i n was avai l abl e, s evere
Pa g e 2 0 2 9
ABC Ambe r CHM Conve rte r Tria l ve rsion, http://w w w .proce sste x t.com/a bcchm.html
res t ri ct i on of carbohydrat e i nt ake was neces s ary t o prol ong l i fe i n pat i ent s wi t h t ype 1 di abet es . In pat i ent s recei vi ng i ns ul i n, t he regularity, timing of carbohydrate intake, and i t s quant i t y are cruci al . It i s equal l y i mport ant t o avoid excessive fat intake, whi ch i ncreas es t he ri s k of at heros cl eros i s . In s eeki ng t o bal ance t hes e cons i derat i ons , phys i ci ans recommend t hat cal ori c i nt ake compri s e 12%–20% prot ei n, 50%–60% carbohydrat e, and 20%–30% fat .
(1) Specific objectives
(a) T ype 1 diabetes. The chi ef goal s of di et ary t reat ment are t o provi de adequat e cal ori es for growt h and act i vi t y and t o ens ure day-t o-day regul ari t y of food i nt ake s o t hat t he avai l abi l i t y of i ns ul i n i s coordi nat ed wi t h carbohydrat e i nt ake.
(b) T ype 2 diabetes. The chi ef goal i n mos t cas es i s t o at t ai n t he pat i ent 's i deal wei ght by means of cal ori c res t ri ct i on and regul ar exerci s e. Becaus e many pat i ent s become normogl ycemi c wi t h di et al one i f s i gni fi cant wei ght l os s i s achi eved, i ni t i al t reat ment s houl d emphas i ze t he i mport ance of di et . However, mos t pat i ent s are not abl e t o l os e enough wei ght t o cont rol gl ucos e l evel s t hrough di et al one.
Pa g e 2 0 3 0
ABC Ambe r CHM Conve rte r Tria l ve rsion, http://w w w .proce sste x t.com/a bcchm.html
(2) Diet calculation (T able 9-14)
(a) T he ideal body weight should be estimated.
(i) For men, t he es t i mat e i s 106 pounds for t he fi rs t 5 feet i n hei ght , and 6 pounds for each addi t i onal i nch.
(ii) For women, t he es t i mat e i s 100 pounds for t he fi rs t 5 feet i n hei ght , and 5 pounds for each addi t i onal i nch.
(iii) For heavy-framed i ndi vi dual s , 5–15 pounds may be added.
(b) The dat a i n Tabl e 9-14 can be us ed t o det ermi ne t he daily caloric need, whi ch vari es wi t h t he pat i ent 's act i vi t y l evel and need t o gai n or l os e wei ght .
(c) T he protein, carbohydrate, and fat intake should be determined by cal cul at i ng 20% of t he t ot al cal ori es as prot ei n (4 kcal /g), 50% as carbohydrat e (4 kcal /g), and t he remai ni ng 30% as fat (9 kcal /g).
Pa g e 2 0 3 1
ABC Ambe r CHM Conve rte r Tria l ve rsion, http://w w w .proce sste x t.com/a bcchm.html
(3) Additional suggestions
(a) Increas ed i nt ake of monouns at urat ed and pol yuns at urat ed fat s and reduced i nt ake of s at urat ed fat s and t rans -fat s are des i rabl e. Chol es t erol i nt ake s houl d not exceed 300–500 mg/day.
(b) For a bal anced ami no aci d cont ent , 50% of prot ei n s houl d be deri ved from t he meat exchange l i s t .
(c) Increas ed fi ber i nt ake, i n t he form of unproces s ed bran, cereal s , frui t s , and veget abl es , may l ower bl ood gl ucos e l evel s and decreas e t he need for i ns ul i n. P.415
TABLE 9-14 Daily Caloric Requirement*
Pa g e 2 0 3 2
ABC Ambe r CHM Conve rte r Tria l ve rsion, http://w w w .proce sste x t.com/a bcchm.html
Activit y Level M O D E R A T V S E E E L R D Y Y E A A N C C Bo T T T dy A I I Bui R V V ld Y E E Ob 2 3 3 es e 0 0 5 â €“ 2 5 Nor 3 3 4 ma 0 5 0 l Un 3 4 4 der 5 0 5 wei
â
ght
€“ 5
0 *Cal ori es (kcal )
Pa g e 2 0 3 3
ABC Ambe r CHM Conve rte r Tria l ve rsion, http://w w w .proce sste x t.com/a bcchm.html
requi red per ki l ogram of i deal body wei ght per day. Thes e es t i mat es are i nt ended to produce wei ght l os s i n obes e i ndi vi dual s , wei ght gai n i n underwei g ht i ndi vi dual s , and mai nt ena nce of wei ght i n normal i ndi vi dual s.
c. Oral antihyperglycemic agents (Tabl e 9-15)
Pa g e 2 0 3 4
ABC Ambe r CHM Conve rte r Tria l ve rsion, http://w w w .proce sste x t.com/a bcchm.html
(1) Sulfonylurea derivatives and meglitinides
(a) Mode of action. Thes e agent s , whi ch bi nd t o recept ors on t he β-cel l pl as ma membrane, res ul t i n cl os ure of pot as s i um channel s , openi ng of cal ci um channel s , and i nfl ux of cal ci um i nt o t he cel l . The end res ul t i s an i ncreas e i n i ns ul i n s ecret i on by β cel l s .
(b) Use. The s econd-generat i on s ul fonyl ureas l i s t ed i n Tabl e 9-15 have l argel y repl aced t he fi rs t -generat i on agent s (t ol but ami de, t ol azami de, acet ohexami de, and chl orpropami de). Repagl i ni de and nat egl i ni de have a mode of act i on l i ke t hat of t he s ul fonyureas but di ffer i n havi ng a fas t er ons et and s hort er durat i on of act i on. They mus t be t aken before each meal .
(c) Side effects. The pri mary s i de effect of s ul fonyl ureas i s hypogl ycemi a, whi ch may res ul t from exces s i ve dos i ng, drug i nt eract i ons , renal or hepat i c di s eas e, or i nadequat e food i nt ake. W i t h repagl i ni de and nat egl i ni de, hypogl ycemi a may be l es s l i kel y
Pa g e 2 0 3 5
ABC Ambe r CHM Conve rte r Tria l ve rsion, http://w w w .proce sste x t.com/a bcchm.html
becaus e of t he cl os e corres pondence bet ween t he peak drug effect and peak gl ucos e abs orpt i on aft er meal s .
(2) Metformin
(a) Mode of action. The mechani s m of act i on of t hi s bi guani de i s not wel l unders t ood, but met formi n decreas es hepat i c gl ucos e out put and i ncreas es peri pheral gl ucos e ut i l i zat i on. The drug does not affect β-cel l funct i on di rect l y.
(b) Use. The s t art i ng dos e i s 500 mg once dai l y wi t h breakfas t or s upper; t hi s can be rai s ed gradual l y t o a maxi mum of 2500 mg dai l y i n di vi ded dos es , wi t h meal s .
(c) Side effects. W hen met formi n i s us ed al one, hypogl ycemi a does not occur, becaus e t he drug does not s t i mul at e i ns ul i n s ecret i on.
(i) Gas t roi nt es t i nal s ympt oms of anorexi a, naus ea, abdomi nal di s comfort , and di arrhea are common, but t hey us ual l y di s appear or can be t ol erat ed i f t he i ni t i al dos e i s
Pa g e 2 0 3 6
ABC Ambe r CHM Conve rte r Tria l ve rsion, http://w w w .proce sste x t.com/a bcchm.html
l ow and i ncreas ed s l owl y.
(ii) Lact i c aci dos i s i s a s eri ous pot ent i al compl i cat i on, but i t i s rare unl es s pat i ent s have a predi s pos i ng condi t i on s uch as renal fai l ure, l i ver di s eas e, al cohol i s m, or any condi t i on t hat may caus e t i s s ue hypoxi a (e.g., heart fai l ure or pul monary di s eas e). Thes e condi t i ons are s t rong cont rai ndi cat i ons t o t he us e of met formi n. Lact i c aci dos i s has occurred when pat i ent s t aki ng met formi n have devel oped renal fai l ure aft er t he i nject i on of radi ocont ras t dyes . Therefore, met formi n s houl d be s t opped when s uch a procedure i s performed and not res umed unt i l normal renal funct i on i s document ed—24–48 hours l at er.
(3) T hiazolidinediones
(a) Mode of action. Ros i gl i t azone and pi ogl i t azone i ncreas e t he s ens i t i vi t y t o i ns ul i n i n mus cl e and fat , and t o a l es s er ext ent , i n t he l i ver. They act by s t i mul at i ng t he
Pa g e 2 0 3 7
ABC Ambe r CHM Conve rte r Tria l ve rsion, http://w w w .proce sste x t.com/a bcchm.html
peroxi s ome prol i ferat or-act i vat ed recept or γ, a nucl ear recept or t hat affect s i ns ul i n-res pons i ve genes .
(b) Side effects
(i) Hypogl ycemi a may occur, es peci al l y i f t hi azol i di nedi ones are us ed wi t h i ns ul i n or s ul fonyl ureas .
(ii) Li ver t oxi ci t y, whi ch coul d be fat al , has occurred i n a s mal l number of pat i ent s t reat ed wi t h t he fi rs t agent of t hi s cl as s (t rogl i t azone). Li ver funct i on t es t s s houl d be performed at bas el i ne and i f hi gher t han 1.5 t i mes normal , avoi d t hi s agent .
TABLE 9-15 Oral Antihyperglycemic Agents
Pa g e 2 0 3 8
ABC Ambe r CHM Conve rte r Tria l ve rsion, http://w w w .proce sste x t.com/a bcchm.html
U s u al St a M rt a in xi g m D al o D s ai e ly T i ( Dm m o in g s g / e of D d ( D r a m o u y g s g ) ) e S ul fo n yl ur e a s (s ec o n d
Pa g e 2 0 3 9
ABC Ambe r CHM Conve rte r Tria l ve rsion, http://w w w .proce sste x t.com/a bcchm.html
g e n er at io n) 1 8 O Gl
nc
i
e
m
or
e
t
pi
wi
ri
ce
d
d
e
ai ly 5 2 O
Gl
0 nc
ip
e
iz
or
id
t
e
wi ce d ai ly 2. 2 O
Gl 5 0 nc y
e
b
or
ur
t
id
wi
e
ce d ai
Pa g e 2 0 4 0
ABC Ambe r CHM Conve rte r Tria l ve rsion, http://w w w .proce sste x t.com/a bcchm.html
ly M e gl iti ni d e s 1. 1 1 R 5 6 5 e
m
p
in
a
ut
gl
e
in
s
id
b
e
ef or e e ac h m e al 1 3 1
N 8 6 5 at 0 0 m e
in
gl
ut
in
e
id
s
e
b ef
Pa g e 2 0 4 1
ABC Ambe r CHM Conve rte r Tria l ve rsion, http://w w w .proce sste x t.com/a bcchm.html
or e e ac h m e al Bi g u a ni d e s 5 2 Di M 0 5 vi et 0 0 d fo
0 e
r
d
m
d
in
o s e s, wi th m e al s
T hi a z
Pa g e 2 0 4 2
ABC Ambe r CHM Conve rte r Tria l ve rsion, http://w w w .proce sste x t.com/a bcchm.html
ol id in e di o n e s 4 8 O R
nc
o
e
si
or
gl
t
it
wi
a
ce
z
d
o
ai
n
ly
e 1 4 O Pi 5 5 nc o
e
gl
d
it
ai
a
ly
z o n e Î ± Gl uc o
Pa g e 2 0 4 3
ABC Ambe r CHM Conve rte r Tria l ve rsion, http://w w w .proce sste x t.com/a bcchm.html
si d a s e in hi bi to rs 2 3 Di A 5 0 vi ca
0 d
rb
e
o
d
s
d
e
o s e s at th e st ar t of m e al s 2 3 Di
Mi 5 0 vi gl
0 d
it
e
Pa g e 2 0 4 4
ABC Ambe r CHM Conve rte r Tria l ve rsion, http://w w w .proce sste x t.com/a bcchm.html
ol
d d o s e s at th e st ar t of m e al
s D 1 â O D 0 €“ nc P- 0
e
4
a
in
d
hi
a
bi
y
to rs
A dj
Si
u
ta
st
gl
fo
ip
r
ti
cr
n
e at in
Pa g e 2 0 4 5
ABC Ambe r CHM Conve rte r Tria l ve rsion, http://w w w .proce sste x t.com/a bcchm.html
in e cl e ar a nc e DDP-4, di pept i d yl pept i das e i nhi bi t or 4.
P.416
(iii) Ot her s i de effect s as s oci at ed wi t h t hi azol i di nedi ones i ncl ude wei ght gai n t hat i s part l y at t ri but abl e t o fl ui d ret ent i on, i ncreas ed pl as ma vol ume, edema, and pos s i bl e exacerbat i on of conges t i ve heart fai l ure.
(4) Dipeptidyl peptidase inhibitor 4 (DDP-4 inhibitors)
(a) Mode of action. In res pons e t o
Pa g e 2 0 4 6
ABC Ambe r CHM Conve rte r Tria l ve rsion, http://w w w .proce sste x t.com/a bcchm.html
a meal , t he gut rel eas es di fferent pept i des (i ncret i n) i nt o t he ci rcul at i on. One of t hes e i s cal l ed gl ucagon-l i ke pept i de 1 (GLP-1), whi ch enhances t he gl ucos e-dependent i ns ul i n s t i mul at i on, s uppres s es gl ucagon s ecret i on (t hi s decreas es hepat i c gl ucos e product i on), and regul at es gas t ri c empt yi ng. The hal f-l i fe of GLP-1 i s s hort , becaus e i t i s met abol i zed by t he DDP-4 enzyme. By i nhi bi t i ng t he effect of t hi s enzyme, t he hal f-l i fe and act i on of GLP-1 i s i ncreas ed. The dos e needs t o be adjus t ed for creat i ni ne cl earance.
(b) Side effects. Abdomi nal pai n, naus ea, and di arrheas can occur i n l es s t han 4% of pat i ent s . The ri s k of hypogl ycemi a i s very l ow.
(5) α-Glucosidase inhibitors
(a) Mode of action. Thes e drugs i nhi bi t t he act i on of enzymes i n t he brus h border of t he s mal l i nt es t i ne, decreas i ng t he convers i on of di s acchari des and ol i gos acchari des t o monos acchari des . Thi s del ays t he abs orpt i on of carbohydrat es , al l owi ng more t i me for i ns ul i n t o
Pa g e 2 0 4 7
ABC Ambe r CHM Conve rte r Tria l ve rsion, http://w w w .proce sste x t.com/a bcchm.html
act and bl unt i ng t he pos t prandi al ri s e i n pl as ma gl ucos e.
(b) Side effects. The s hi ft i ng of carbohydrat e di ges t i on t o t he di s t al s mal l bowel and col on t ends t o caus e s ympt oms of fl at ul ence, abdomi nal di s comfort , and di arrhea. To mi ni mi ze t hes e s ympt oms , t hes e drugs are s t art ed at very l ow dos es and i ncreas ed very gradual l y.
d. Insulin
(1) Preparations
(a) Ins ul i n preparat i ons are avai l abl e wi t h short (regul ar i ns ul i n, rapi d-act i ng i ns ul i n anal ogs ), intermediate, and long durations of action (Tabl e 9-16).
(b) Insulin lispro and insulin aspart have earl i er ons et of act i on and s hort er durat i on of act i on t han regul ar i ns ul i n. They t herefore can be i nject ed i mmedi at el y before meal s rat her t han 15–30 mi nut es before meal s , and are l es s l i kel y t o have prol onged act i on t hat coul d caus e hypogl ycemi a before t he next meal .
Pa g e 2 0 4 8
ABC Ambe r CHM Conve rte r Tria l ve rsion, http://w w w .proce sste x t.com/a bcchm.html
(c) Insulin glargine and det emi r are cons i dered “peakl es s ― i ns ul i ns becaus e t here i s l i t t l e vari abi l i t y i n i ns ul i n bl ood l evel s over 24 hours i f t he i ns ul i n i s i nject ed once dai l y. Some pat i ent s do not need t o t ake t hem t wi ce a day, but even i f t hat i s t he cas e, t here i s l es s ri s k of hypogl ycemi a. Thi s makes t hem excel l ent agent s t o provi de bas al i ns ul i n l evel s duri ng an ent i re 24-hour peri od.
TABLE 9-16 Insulin Preparations and Their Onset, Peak, and Approximate Duration of Action D u O
r
n
at
s P io et e n of a of A k A ct Ef ct T io fe io y n ct n p ( ( ( e h h h s ) ) ) F a
Pa g e 2 0 4 9
ABC Ambe r CHM Conve rte r Tria l ve rsion, http://w w w .proce sste x t.com/a bcchm.html
st -a ct in g R  2 5 e ½â â g -1 €“ €“ ul
4 8
ar h u m a n in s ul in In   3 s ¼½â ul
-1 €“
in
 5
li
½
s pr o, in s ul in a s p ar
Pa g e 2 0 5 0
ABC Ambe r CHM Conve rte r Tria l ve rsion, http://w w w .proce sste x t.com/a bcchm.html
t, in s ul in gl ul is in e In  1 6 h ¼â al
€“
e
2
d in s ul in , In te r m e di at eac ti n g N 1 4- 1 P â  2 H €“ ½ â h 3
€“
Pa g e 2 0 5 1
ABC Ambe r CHM Conve rte r Tria l ve rsion, http://w w w .proce sste x t.com/a bcchm.html
u
1
m
6
a n in s ul in L 1 8- 1 e â  4 nt €“ ½ â e 4
€“
in
1
s
8
ul in 7 Â 8 1 0 ½
2
% -1
â
N
€“
P
1
H
6
h u m a n, 3 0 % re g ul ar h
Pa g e 2 0 5 2
ABC Ambe r CHM Conve rte r Tria l ve rsion, http://w w w .proce sste x t.com/a bcchm.html
u m a n in s ul in L o n gac ti n g In â Fl 2 s €“ at 4 ul in gl ar gi n e In 2 1 2 s
1 4
ul
?
in d et e m ir NPH, i s ophane
Pa g e 2 0 5 3
ABC Ambe r CHM Conve rte r Tria l ve rsion, http://w w w .proce sste x t.com/a bcchm.html
i ns ul i n s us pens i on.
P.417
FIGURE 9-6 Ideal bas al /bol us i ns ul i n abs orpt i on pat t ern. (Adapt ed from Humes HD, DuPont HL, Gardner LB, et al . Kel l ey's T ext book of I nt ernal Medi c i ne. 4t h ed. Phi l adel phi a: Li ppi ncot t W i l l i ams & W i l ki ns , 2000. )
(d) NPH and s i mi l ar i nt ermedi at e-act i ng i ns ul i ns are avai l abl e i n premixed combinations wi t h regul ar i ns ul i n
Pa g e 2 0 5 4
ABC Ambe r CHM Conve rte r Tria l ve rsion, http://w w w .proce sste x t.com/a bcchm.html
and i ns ul i ns l i s pro and as part . Thes e combi nat i ons are 70% or 75% i nt ermedi at e i ns ul i n and 30% or 25% s hort -act i ng i ns ul i n.
(2) Insulin regimens (Figures 9–6 and 9–7)
(a) Physiologic insulin secretion cons i s t s of a rel at i vel y cons t ant , l ow rat e of product i on at ni ght and bet ween meal s (bas al i ns ul i n), and s hort burs t s of gl ucos e-s t i mul at ed i ns ul i n s ecret i on at meal t i me (bol us ).
(b) Thi s pat t ern of s ecret i on can be mi mi cked by t he us e of l ong-act i ng i ns ul i n t o provi de basal levels (e.g., i ns ul i n gl argi ne or i ns ul i n det emi r once dai l y or NPH i ns ul i n t wi ce dai l y), and s hort -act i ng i ns ul i n before each meal t o provi de prandial insulin coverage. Thes e regi mens enabl e i ns ul i n coverage t o be mat ched t o i ndi vi dual pat t erns of di et and act i vi t y, but t hey requi re four or more dai l y i nject i ons or an i ns ul i n pump.
(c) Premixed combinations of i nt ermedi at e- and s hort -act i ng i ns ul i n, for exampl e, i ns ul i n 70/30,
Pa g e 2 0 5 5
ABC Ambe r CHM Conve rte r Tria l ve rsion, http://w w w .proce sste x t.com/a bcchm.html
may be gi ven before breakfas t and before s upper. Thi s regi men provi des bas al and prandi al coverage wi t h onl y t wo dai l y i nject i ons , but t here i s l es s uni formi t y i n coverage and l es s fl exi bi l i t y i n dos age adjus t ment .
(d) If NPH and s hort -act i ng i ns ul i ns are gi ven t wi ce dai l y, t he eveni ng dos e of NPH may have i t s peak effect duri ng t he ni ght , caus i ng noct urnal hypogl ycemi a. If t he eveni ng dos e i s s pl i t , gi vi ng t he s hort -act i ng i ns ul i n before s upper and t he NPH at bedt i me, t he peak NPH effect may be more l i kel y t o occur duri ng an earl y morni ng ri s e i n gl ucos e l evel s , wi t h bet t er gl ucos e cont rol (but a need for t hree i ns t ead of t wo dai l y i nject i ons ).
(e) Portable infusion pump. The cl os es t cont rol of s erum gl ucos e l evel s i s achi eved by t he cons t ant i nfus i on of regul ar i ns ul i n t hrough a needl e pl aced s ubcut aneous l y i n t he abdomi nal wal l or t hi gh. A bas al rat e of approxi mat el y 12.5–15 mU/kg/hr i s s uppl ement ed wi t h pul s e dos es before each meal . The benefi t of t i ght er gl ucos e cont rol wi t h i nt ens i ve t herapy mus t be
Pa g e 2 0 5 6
ABC Ambe r CHM Conve rte r Tria l ve rsion, http://w w w .proce sste x t.com/a bcchm.html
bal anced agai ns t t he i ncreas ed ri s k of hypogl ycemi a and t he i nconveni ence of t he port abl e pump.
(3) Factors affecting insulin requirements
(a) Intercurrent illness or other stress. Ins ul i n needs may i ncreas e as a res ul t , perhaps becaus e of i ncreas ed l evel s of cat echol ami nes . A t emporary i ncreas e i n t he dos e of i ns ul i n may be needed.
(b) Exercise. The i ncreas ed gl ucos e ut i l i zat i on may caus e hypogl ycemi a unl es s t he i ns ul i n dos e i s reduced or ext ra carbohydrat e i s i nges t ed.
(c) Somogyi effect. Ins ul i n-i nduced hypogl ycemi a caus es rel eas e of count erregul at ory hormones s uch as epi nephri ne and gl ucagon; t hi s may t hen caus e rebound hypergl ycemi a. If t he caus e of t hi s hypergl ycemi a i s not recogni zed, t he i ns ul i n dos e may be i ncreas ed, l eadi ng t o even more s evere hypogl ycemi a. Hypogl ycemi a duri ng t he hours of s l eep may be an unrecogni z ed caus e of i ncreas ed morni ng fas t i ng gl ucos e l evel s ; i f t hi s i s t he cas e, a
Pa g e 2 0 5 7
ABC Ambe r CHM Conve rte r Tria l ve rsion, http://w w w .proce sste x t.com/a bcchm.html
decreas e i n t he i ns ul i n dos e may correct t he morni ng hypergl ycemi a.
(d) Dawn phenomenon. Earl y ri s e of cort i s ol , growt h hormone, and cat echol ami nes can i nduce morni ng hypergl ycemi a.
(4) Complications of insulin therapy
(a) Local allergy. Red, i t chy l umps may form at t he i nject i on s i t e mi nut es or hours aft er an i ns ul i n dos e. Thi s react i on t ends t o occur wi t hi n a few weeks of i ni t i al i ns ul i n t reat ment and us ual l y res ol ves i n a few weeks or mont hs .
(b) Systemic allergy. General i zed urt i cari a, angi oedema, and anaphyl axi s are rare but l i fe-t hreat eni ng react i ons t o i ns ul i n. Becaus e a ket os i s -prone pat i ent cannot s urvi ve wi t hout i ns ul i n, t hat pat i ent mus t be hos pi t al i zed and i ns ul i n des ens i t i zat i on mus t be performed. An i ni t i al i nt radermal dos e of 1/10,000 U i ns ul i n i s gi ven and i s i ncreas ed every 30 mi nut es .
(c) Antibody-mediated insulin resistance. Ins ul i n res i s t ance i s
Pa g e 2 0 5 8
ABC Ambe r CHM Conve rte r Tria l ve rsion, http://w w w .proce sste x t.com/a bcchm.html
oft en defi ned as a need for more t han 200 U dai l y, and i t i s more common i n pat i ent s who have been expos ed t o i ns ul i n i nt ermi t t ent l y. IgG i ns ul i n-bi ndi ng ant i bodi es i n t he s erum may caus e t he condi t i on.
(i) Ant i body-medi at ed res i s t ance i s oft en s el f-l i mi t ed, res ol vi ng wi t hi n 6 mont hs .
(ii) Caut i ous us e of gl ucocort i coi ds mi ght be neces s ary (t he s udden rel eas e of ant i body-bound i ns ul i n i n res pons e t o s t eroi d t herapy may caus e hypogl ycemi a).
(d) Lipodystrophy
(i) Lipohypertrophy. Local s wel l i ngs , compos ed of fi brous and fat t y t i s s ue, may occur at i ns ul i n i nject i on s i t es , perhaps becaus e of a l ocal l i pogeni c effect of i ns ul i n on t he fat cel l s . The s wel l i ngs may regres s i f human i ns ul i n i s us ed and t he s i t e of l i pohypert rophy i s avoi ded.
Pa g e 2 0 5 9
ABC Ambe r CHM Conve rte r Tria l ve rsion, http://w w w .proce sste x t.com/a bcchm.html
(ii) Lipoatrophy. Pi t s may form at i nject i on s i t es becaus e of t he di s appearance of s ubcut aneous fat . Thes e may s l owl y di s appear i f human i ns ul i n i s i nject ed i nt o t he peri met er of t he at rophi c area.
e. Novel therapies
(a) Incretin mimetics. They mi mi c t he enhancement of gl ucos e-dependent i ns ul i n s ecret i on by GLP-1. The onl y one approved s o far i s exenat i de. It s t i mul at es i ns ul i n rel eas e, decreas es hepat i c gl ucos e product i on by l oweri ng gl ucagon l evel , s uppres s es appet i t e, s l ows gas t ri c empt yi ng, and may have a rol e i n i s l et cel l regenerat i on. It i s gi ven t wi ce a day as a s ubcut aneous i nject i on. It has been approved t o be us ed i n combi nat i on wi t h met formi n, s ul fonyl ureas and t hi azol i di nendi ones . Tri al s are underway t o det ermi ne effi cacy and s afet y for t he combi nat i on wi t h i ns ul i n. Exenat i de's major s i de effect i s naus ea, whi ch us ual l y decreas es wi t h t i me. Hypogl ycemi a can occur i f us ed wi t h i n combi nat i on wi t h s ul fonyl ureas .
Pa g e 2 0 6 0
ABC Ambe r CHM Conve rte r Tria l ve rsion, http://w w w .proce sste x t.com/a bcchm.html
FIGURE 9-7. Bas al /bol us i ns ul i n abs orpt i on pat t ern wi t h s t andard i ns ul i n preparat i ons . (Adapt ed from Humes HD, DuPont HL, Gardner LB, et al . Kel l ey's T ext book of I nt ernal Medi c i ne. 4t h ed. Phi l adel phi a: Li ppi ncot t W i l l i ams & W i l ki ns , 2000. )
P.418
(b) Pramlintide. It i s a human anal og of amyl i n. Amyl i n i s co-s ecret ed wi t h i ns ul i n from t he β cel l , and i t s s ecret i on decreas es wi t h t i me. It s l ows gas t ri c empt yi ng, decreas es gl ucagon s ecret i on, and s uppres s es appet i t e. It i s gi ven as a s ubcut aneous i nject i on t hree t i mes a day before meal s . It s more common s i de effect i s naus ea, whi ch decreas es wi t h t i me. It has been approved t o be us ed i n combi nat i on wi t h i ns ul i n or oral agent s .
f. Evaluation of glucose control
Pa g e 2 0 6 1
ABC Ambe r CHM Conve rte r Tria l ve rsion, http://w w w .proce sste x t.com/a bcchm.html
(1) Home monitoring of capillary glucose levels, us i ng a gl ucos e met er and t es t s t ri ps , i s performed as oft en as neces s ary t o eval uat e gl ucos e cont rol . If s at i s fact ory preprandi al gl ucos e l evel s are at t ai ned, t he 2-hour pos t prandi al gl ucos e l evel s s houl d be meas ured.
(2) Goals of treatment s houl d be t o achi eve a fas t i ng pl as ma gl ucos e l evel bel ow 120 mg/dL, a pos t prandi al gl ucos e l evel bel ow 140–180 mg/dL, and an Hb A1 C
l evel bel ow 6%–7%.
(3) Glycosylated hemoglobin measurements
(a) The free ami no aci d groups of hemogl obi n and ot her body prot ei ns combi ne wi t h gl ucos e t o form a revers i bl e compound (Schi ff bas e), whi ch can t hen become a s t abl e gl ycos yl at ed prot ei n (Amadori rearrangement ). The ext ent of t hi s nonenzymat i c gl ycos yl at i on i s dependent on t he concent rat i on of gl ucos e i n bl ood; t hat i s , t he percent age of hemogl obi n t hat i s gl ycos yl at ed depends on t he bl ood gl ucos e l evel s t hat were pres ent duri ng t he l i fe s pan of t he current l y ci rcul at i ng red bl ood cel l s (RBCs ).
Pa g e 2 0 6 2
ABC Ambe r CHM Conve rte r Tria l ve rsion, http://w w w .proce sste x t.com/a bcchm.html
(b) The gl ycos yl at ed hemogl obi n l evel , t herefore, refl ect s t he degree of hypergl ycemi a duri ng t he precedi ng 6–12 weeks , and i t may be us eful i n es t i mat i ng t he average cont rol of s erum gl ucos e l evel s duri ng t hi s t i me.
f. Pharmacologic therapy of diabetes (Onl i ne Fi gure 9-8)
ONLINE FIGURE 9-8 Treat ment of t ype 2 di abet es mel l i t us . Fact ors t hat i nfl uence t he s el ect i on of s peci fi c agent s are des cri bed i n t he t ext (s ee IV A 8).
(1) T ype 1 diabetes. Pat i ent s wi t h t ype 1 di abet es are dependent on i ns ul i n t reat ment . Oral ant i hypergl ycemi c i ns ul i n s ens i t i zers agent s l i ke met formi n or t hi azol i di nedi ones coul d be us ed i n t he
Pa g e 2 0 6 3
ABC Ambe r CHM Conve rte r Tria l ve rsion, http://w w w .proce sste x t.com/a bcchm.html
obes e i ns ul i n-res i s t ant pat i ent . Int ens i ve i ns ul i n t herapy, wi t h mul t i pl e dai l y i nject i ons or t he port abl e i nfus i on pump, i s i ncreas i ngl y bei ng us ed.
(2) Gestational diabetes. If t hi s condi t i on cannot be cont rol l ed by di et , i ns ul i n mus t be us ed.
(3) T ype 2 diabetes
(a) Oral ant i hypergl ycemi c agent s s houl d be t ri ed fi rs t , al t hough pat i ent s who pres ent wi t h s evere hypergl ycemi a may requi re an i ni t i al peri od of i ns ul i n t reat ment t o achi eve prompt gl ycemi c cont rol . If cont rol cannot be obt ai ned wi t h oral agent s , i ns ul i n mus t be added.
(b) A s cheme for t he s el ect i on of oral agent s and i ns ul i n i n t he t reat ment of t ype 2 di abet es i s s hown i n Onl i ne Fi gure 9-8.
(i) Sul fonyl ureas (or megl i t i ni des ) or met formi n i s mos t commonl y chos en as i ni t i al t herapy. Met formi n may be es peci al l y us eful i n obes e pat i ent s becaus e i t does not caus e furt her wei ght gai n.
Pa g e 2 0 6 4
ABC Ambe r CHM Conve rte r Tria l ve rsion, http://w w w .proce sste x t.com/a bcchm.html
(ii) If t wo oral agent s are us ed, i t i s reas onabl e t hat one i s a promot er of i ns ul i n s ecret i on (s ul fonyl urea or a megl i t i ni de) and t he ot her a promot er of s ens i t i vi t y t o i ns ul i n (met formi n or a t hi azol i di nedi one).
(iii) Sul fonyl ureas (or megl i t i ni des ) and met formi n are us ual l y recommended for i ni t i al t reat ment becaus e t hey t end t o have great er hypogl ycemi c effi cacy as s i ngl e drugs t han t he t hi azol i di nedi ones . The α-gl ucos i das e i nhi bi t ors are t he l eas t pot ent of t hes e oral ant i hypergl ycemi c agent s and have a s econdary rol e i n t reat ment .
(c) If i ns ul i n mus t be added becaus e of fai l ure t o achi eve adequat e gl ucos e l evel s , i t i s common pract i ce t o cont i nue t he oral agent s al t hough t he dos e of s ul fonyl urea mi ght need t o be decreas ed i n order t o prevent hypogl ycemi a.
Pa g e 2 0 6 5
ABC Ambe r CHM Conve rte r Tria l ve rsion, http://w w w .proce sste x t.com/a bcchm.html
(i) Sul fonyl ureas or megl i t i ni des , becaus e t hey s t i mul at e gl ucos e-dependent i ns ul i n s ecret i on, may hel p provi de t he prandi al i ns ul i n needs . (If a s hort -act i ng i ns ul i n i s gi ven before meal s , or pre-mi xed i ns ul i n i s us ed, t he s ul fonyl ureas or megl i t i ni des s houl d be di s cont i nued).
(ii) Met formi n and t hi azol i di nedi ones , becaus e t hey i ncreas e i ns ul i n s ens i t i vi t y, may i ncreas e t he effect i venes s of exogenous i ns ul i n.
(d) W hen i t becomes neces s ary t o add i ns ul i n t herapy, NPH i ns ul i n, i ns ul i n gl argi ne, or i ns ul i n det emi r may be added at bedt i me t o t he oral agent s . A s t art i ng dos e of 10 uni t s may be i ncreas ed by 2 t o 6 uni t s every few days unt i l fas t i ng gl ucos e l evel s fal l bel ow 120 mg/dL (or unt i l noct urnal or dayt i me hypogl ycemi a occur).
(e) If pos t prandi al gl ucos e l evel s cannot be cont rol l ed, a s hort -act i ng i ns ul i n may need t o be added
Pa g e 2 0 6 6
ABC Ambe r CHM Conve rte r Tria l ve rsion, http://w w w .proce sste x t.com/a bcchm.html
before one or more meal s .
B. Hypoglycemia There i s no s i mpl e defi ni t i on of hypogl ycemi a. Gl ucos e l evel s l es s t han 45 or 55 mg/dL may be as s oci at ed wi t h hypogl ycemi c s ympt oms , but s ome normal i ndi vi dual s have gl ucos e l evel s l ower t han t hi s wi t hout s ympt oms aft er s everal days of fas t i ng or s everal hours aft er a gl ucos e l oad, wi t h res ul t ant s t i mul at i on of i ns ul i n s ecret i on. If t he di agnos i s i s i n doubt , Whipple' s triad (i .e., s ympt oms of hypogl ycemi a, l ow s erum gl ucos e l evel s , and rel i ef of t he s ympt oms when normogl ycemi a i s res t ored) may be us ed as a cri t eri on.
1. Insulinomas are rare t umors t hat ari s e from t he β cel l s of t he i s l et s of Langerhans . Mos t are s i ngl e, beni gn adenomas , but approxi mat el y 10% of t hes e t umors are mul t i pl e, and 10% are mal i gnant . They occur wi t h equal frequency i n t he head, t he body, and t he t ai l of t he pancreas . β-Cel l hyperpl as i a occas i onal l y may produce a s i mi l ar s yndrome. o
o
a. Clinical features. Ins ul i nomas produce exces s i ve quant i t i es of i ns ul i n, l eadi ng t o fasting hypoglycemia (i .e., a cat egory of hypogl ycemi a i n whi ch t he l owes t l evel s of s erum gl ucos e and t he mos t s evere s ympt oms occur aft er prol onged peri ods wi t hout food i nt ake).
(1) Sympt oms are mos t l i kel y t o occur i n t he earl y morni ng or l at e aft ernoon or aft er fas t i ng or exerci s e.
(2) The s ympt oms of hypogl ycemi a are t he
Pa g e 2 0 6 7
ABC Ambe r CHM Conve rte r Tria l ve rsion, http://w w w .proce sste x t.com/a bcchm.html
s ame as t hos e t hat res ul t from i ns ul i n overdos e i n di abet i c pat i ent s [s ee IV A 6 c (2)].
(3) Pat i ent s may gai n wei ght before t he di agnos i s i s made becaus e t hey l earn t o rel i eve or avoi d hypogl ycemi c s ympt oms by frequent l y s nacki ng on carbohydrat es .
o
o
b. Diagnosis (T able 9-17)
(1) An elevated serum insulin concentration when t he gl ucos e l evel i s l ow i s s t rong evi dence for t he pres ence of an i ns ul i noma, i f an exogenous i ns ul i n or s ul fonyl urea s ources are excl uded.
(a) In normal i ndi vi dual s , i ns ul i n l evel s fal l as gl ucos e l evel s fal l , and i ns ul i n l evel s become undet ect abl e at gl ucos e l evel s l es s t han 30 mg/dL.
(b) If t he s erum gl ucos e concent rat i on i s l es s t han 45 mg/dL, an i ns ul i n l evel hi gher t han 10 mU/mL i s abnormal .
(2) A prolonged fast, ext endi ng for 24–72 hours , may be neces s ary t o demons t rat e fas t i ng hypogl ycemi a wi t h i nappropri at el y hi gh i ns ul i n l evel s . Some s ugges t t he i nject i on of 1 mg of gl ucagon i nt ravenous l y at t he end of
Pa g e 2 0 6 8
ABC Ambe r CHM Conve rte r Tria l ve rsion, http://w w w .proce sste x t.com/a bcchm.html
t he fas t i f no hypogl ycemi a devel ops . A ri s e i n gl ucos e of 25 mg/dL or more poi nt s t o hypogl ycemi a-medi at ed by i ns ul i n. P.419
TABLE 9-17 Diagnostic Interpretation of the Results of a 72-hour Fast Serum
S e r u m β - Δ h S y e d r r u
S
o m
y
C P x gl
m Gl I - r y- u S pt u n p oi b c ul o c s e n ut o fo m o ul pt s yr s n Di s s in id ul at e yl a o e  e in e * in g r m µ p p mm s n si g U m m m g e o g / / ol ol ol / r si n d m / / / d u s s L L L L L L m
Pa g e 2 0 6 9
ABC Ambe r CHM Conve rte r Tria l ve rsion, http://w w w .proce sste x t.com/a bcchm.html
N N â < < < > < N or o ‰ 3 2 5 2. 2 o m
¥
al
4
â€
0
7 5
0
0 In Y â â â â â â N s
e ‰‰‰‰‰‰ o
ul s ¤ ¥ ¥ ¥ ¤ ¥ in
4 3 2 5 2. 2
o
5
m
0
7 5
0
a F Y â â < < â â N ac e ‰‰ 2 5 ‰‰ o ti s ¤ ¥ 0
¤ ¥
ti
4 3 0
2. 2
o
5 *
7 5
u s h y p o gl yc e m ia fr o m in s ul in
Pa g e 2 0 7 0
ABC Ambe r CHM Conve rte r Tria l ve rsion, http://w w w .proce sste x t.com/a bcchm.html
S Y â â â â â â Y ul e ‰‰‰‰‰‰ e fo s ¤ ¥ ¥ ¥ ¤ ¥ s n
4 3 2 5 2. 2
yl
5
ur
0
7 5
0
e ain d uc e d h y p o gl yc e m ia H Y â â < > â â N y e ‰‰ 2 5 ‰‰ o p
s ¤ ¤ 0
¤ ¥
o
4 3 0
2. 2
gl
5
7 5
yc e m ia m e di
Pa g e 2 0 7 1
ABC Ambe r CHM Conve rte r Tria l ve rsion, http://w w w .proce sste x t.com/a bcchm.html
at e d b y in s ul in -l i k e gr o w th fa ct or N Y â < < < > < N o e ‰ 3 2 5 2. 2 o n s ¤
0
â
4
0
€“
5
7 5
in s ul in m e di at e d
Pa g e 2 0 7 2
ABC Ambe r CHM Conve rte r Tria l ve rsion, http://w w w .proce sste x t.com/a bcchm.html
h y p o gl yc e m ia In N â < < < â â N a o ‰ 3 2 5 ‰‰ o d
¥
0
¤ ¥
v
4
0
2. 2
er
5
7 5
te nt fe e di n g d ur in g th e fa st N Y â < < < > < N o e ‰ 3 2 5 2. 2 o n s ¥
0
h
4
0
y
0
7 5
p
Pa g e 2 0 7 3
ABC Ambe r CHM Conve rte r Tria l ve rsion, http://w w w .proce sste x t.com/a bcchm.html
o gl yc e m ic di s or d er Meas urement s are made at t he t i me t he deci s i on i s made t o end t he fas t . Sequent i al s erum gl ucos e meas urement s i n t he hypogl ycemi c range fl uct uat e. Serum gl ucos e concent rat i ons 45 mg/dL at t he t i me a deci s i on i n made t o end t he fas t may ri s e t o as much as 56 mg/dL when t he fas t i s act ual l y ended approxi mat el y 1 hour l at er. Serum gl ucos e concent rat i on may be as l ow as 40
Pa g e 2 0 7 4
ABC Ambe r CHM Conve rte r Tria l ve rsion, http://w w w .proce sste x t.com/a bcchm.html
mg/dL duri ng prol onged fas t i ng i n normal women. To convert s erum gl ucos e val ues t o mmol /L, mul t i pl y by 0.056. To convert i ns ul i n val ues t o pmol /L, mul t i pl y by 6.0. Serum C-pept i de and proi ns ul i n were meas ured by i mmunomet ri c as s ay. In normal s ubject s l evel s of s erum i ns ul i n, C-pept i de, and proi ns ul i n concent rat i ons may be hi gher i f t he s erum gl ucos e concent rat i on i s 60 mg/dL. *Refers t o t he change i n s erum gl ucos e aft er t he admi ni s t rat i on of i nt ravenous gl ucagon. â€
Serum i ns ul i n
concent rat i ons may be very hi gh (>100 µU/mL) i n fact i t i ous
Pa g e 2 0 7 5
ABC Ambe r CHM Conve rte r Tria l ve rsion, http://w w w .proce sste x t.com/a bcchm.html
hypogl ycemi a from i ns ul i n. Reproduced wi t h permi s s i on from Servi ce FJ. Hypogl ycemi c di s orders . N Engl J Med 1995;332:1144. Copyri ght © 1995 Mas s achus et t s Medi cal Soci et y.
(3) Proinsulin, t he l arge precurs or mol ecul e of i ns ul i n, i s produced i n i ncreas ed amount s by i ns ul i nomas . Proi ns ul i n normal l y makes up 5%–20% of t he t ot al i ns ul i n i n bl ood t hat i s meas ured by radi oi mmunoas s ay; i n pat i ent s wi t h an i ns ul i noma, proi ns ul i n us ual l y exceeds 25%.
(4) Diagnostic imaging wi t h MRI, CT, ul t ras onography (i ncl udi ng endos copi c or i nt raoperat i ve ul t ras ound), and s el ect i ve art eri ography may be us ed t o i dent i fy and l ocal i ze an i ns ul i noma. Thes e t umors may be s mal l , however, averagi ng 1–2 cm, and t hey oft en cannot be vi s ual i zed.
o
o
c. T herapy
(1) Surgical removal of an adenoma or part i al pancreat ect omy for mul t i pl e adenomas
Pa g e 2 0 7 6
ABC Ambe r CHM Conve rte r Tria l ve rsion, http://w w w .proce sste x t.com/a bcchm.html
or β-cel l hyperpl as i a i s t he t reat ment of choi ce.
(2) Medical therapy i s res erved for pat i ent s whos e t umors cannot be compl et el y removed becaus e met as t at i c di s eas e exi s t s , previ ous s urgi cal at t empt s have fai l ed, or i l l nes s or pat i ent refus al makes s urgery i nfeas i bl e.
(a) Diazoxide i nhi bi t s t he rel eas e of i ns ul i n from β cel l s . A dos e of 200 mg dai l y, whi ch can be rai s ed as hi gh as 800 mg i f neces s ary, can prevent hypogl ycemi a i n pat i ent s wi t h an i noperabl e i ns ul i noma.
(b) Octreotide i nhi bi t s i ns ul i n and gl ucagon. Streptozocin i s an ant i bi ot i c t hat s peci fi cal l y des t roys β cel l s . It i s us ed t o t reat mal i gnant β-cel l t umors . Sys t emi c chemot herapy, i nt erferon-α, or chemoembol i zat i on coul d be t ri ed for mal i gnant t umors .
P.420
o
o
2. Factitious hypoglycemia i s caus ed by t he s urrept i t i ous s el f-admi ni s t rat i on of i ns ul i n or an oral hypogl ycemi c drug, mos t oft en by an i ndi vi dual who i s fami l i ar wi t h heal t h care, s uch as a nurs e or medi cal t echnol ogi s t , or by a di abet i c or rel at i ve of a di abet i c. Di fferent i at i on from hypogl ycemi a
Pa g e 2 0 7 7
ABC Ambe r CHM Conve rte r Tria l ve rsion, http://w w w .proce sste x t.com/a bcchm.html
caus ed by an i ns ul i noma may depend on s peci al s t udi es .
a. Serum C-peptide measurement i ndi cat es t he s ource of i ns ul i n s ecret i on [s ee IV B 1 b (3); Tabl e 9-17]. The l ow gl ucos e and hi gh i ns ul i n l evel s t hat are pat hognomoni c of an i ns ul i noma s houl d be accompani ed by i ncreas ed C-pept i de l evel s ; i f t he l at t er are l ow, i ndi cat i ng an exogenous s ource of t he hi gh i ns ul i n concent rat i on, t he di s eas e i s fact i t i ous .
b. Inges t i on of a s ul fonyl urea, however, s t i mul at es endogenous i ns ul i n product i on and t he C-pept i de l evel i s hi gh; t herefore, screening of blood or urine for sulfonylureas i s al s o neces s ary t o rul e out fact i t i ous di s eas e.
o
o
3. Extrapancreatic tumors may caus e hypogl ycemi a. Thes e us ual l y are l arge, i nt ra-abdomi nal t umors , mos t oft en of mes enchymal ori gi n (e.g., fi bros arcoma), al t hough t hey may be hepat i c carci nomas or ot her t umors . The mechani s m of hypogl ycemi a i s poorl y unders t ood; i ncreas ed us e of gl ucos e by s ome t umors and product i on by ot hers of an i ns ul i n-l i ke growt h fact or II (IGF-II) have been obs erved.
o
o
4. Ethanol-induced hypoglycemia occurs i n pat i ent s whos e gl ycogen s t ores are depl et ed becaus e of i nadequat e recent food i nt ake, us ual l y
Pa g e 2 0 7 8
ABC Ambe r CHM Conve rte r Tria l ve rsion, http://w w w .proce sste x t.com/a bcchm.html
12–24 hours aft er a bout of heavy dri nki ng.
a. The oxi dat i on of et hanol t o acet al dehyde and acet at e generat es reduced ni cot i nami de-adeni ne di nucl eot i de (NADH) and decreas es t he avai l abi l i t y of ni cot i nami de-adeni ne di nucl eot i de (NAD), whi ch i s needed for gl uconeogenes i s . W hen nei t her gl ycogenol ys i s nor gl uconeogenes i s i s avai l abl e t o mai nt ai n hepat i c gl ucos e product i on i n t he fas t i ng s t at e, hypogl ycemi a res ul t s .
b. Prompt recogni t i on and gl ucos e admi ni s t rat i on are es s ent i al becaus e t he mort al i t y rat e i s hi gher t han 10%.
o
o
5. Liver disease may res ul t i n i mpai rment of gl ycogenol ys i s and gl uconeogenes i s s uffi ci ent t o caus e fas t i ng hypogl ycemi a. Thi s i s s een i n ful mi nant vi ral hepat i t i s or acut e t oxi c l i ver di s eas e but not i n t he us ual , l es s s evere cas es of ci rrhos i s or hepat i t i s .
o
o
6. Other causes of fasting hypoglycemia i ncl ude cort i s ol defi ci ency, GH defi ci ency, or bot h, whi ch may occur i n adrenal insufficiency or hypopituitarism. Hypogl ycemi a may occur i n pat i ent s wi t h renal failure and heart failure; however, t he caus es are poorl y unders t ood.
o
o
7. “ Reactive hypoglycemia― may occur as a
Pa g e 2 0 7 9
ABC Ambe r CHM Conve rte r Tria l ve rsion, http://w w w .proce sste x t.com/a bcchm.html
res ul t of anot her, s mal l er group of di s orders , whi ch caus e hypogl ycemi a a few hours aft er t he i nges t i on of carbohydrat es . Ins ul i noma and t he ot her condi t i ons di s cus s ed previ ous l y mos t commonl y produce hypogl ycemi a i n t he fas t i ng s t at e.
a. Alimentary hypoglycemia occurs i n pat i ent s who have had a gas t rect omy or ot her s urgi cal procedure t hat l eads t o abnormal l y rapi d movement of food i nt o t he s mal l bowel . Rapi d abs orpt i on of carbohydrat e s t i mul at es exces s i ve i ns ul i n s ecret i on, caus i ng hypogl ycemi a s everal hours aft er a meal .
b. Reactive hypoglycemia of diabetes occurs i n an occas i onal pat i ent wi t h earl y di abet es who may have a l at e but exces s i ve rel eas e of i ns ul i n aft er a carbohydrat e-cont ai ni ng meal . The gl ucos e l evel i s el evat ed aft er 2 hours but t hen decreas es t o hypogl ycemi c l evel s 3–5 hours aft er t he meal .
c. “ Functional― hypoglycemia, al t hough commonl y di agnos ed i n pat i ent s wi t h chroni c fat i gue and anxi et y, i s probabl y a rare condi t i on. Hypogl ycemi a i s not a caus e of chroni c fat i gue, depres s i on, and l ack of energy.
(1) Clinical features. Hypogl ycemi a, wi t h adrenergi c s ympt oms s uch as s weat i ng and pal pi t at i ons , occurs 2–5 hours aft er a carbohydrat e-ri ch meal ,
Pa g e 2 0 8 0
ABC Ambe r CHM Conve rte r Tria l ve rsion, http://w w w .proce sste x t.com/a bcchm.html
pres umabl y becaus e of i ncreas ed i ns ul i n product i on or i ns ul i n s ens i t i vi t y.
(2) Diagnosis
(a) The overdi agnos i s of funct i onal hypogl ycemi a s t ems i n part from mi s i nt erpret at i on of t he 5-hour gl ucos e t ol erance t es t . One i n four normal i ndi vi dual s has a s erum gl ucos e l evel l es s t han 50 mg/dL 3–5 hours aft er t he nonphys i ol ogi c s t i mul us of 75–100 g gl ucos e, and s ome normal i ndi vi dual s have l evel s l es s t han 35 mg/dL, wi t hout P.421
s ympt oms . Thes e res pons es do not prove funct i onal hypogl ycemi a, and t he t es t s houl d not be done t o make t he di agnos i s of hypogl ycemi a.
(b) The di agnos i s depends on fi ndi ng hypogl ycemi a t hat coi nci des wi t h t he pat i ent 's t ypi cal s ympt oms and on fi ndi ng rel i ef of s ympt oms by carbohydrat e i nges t i on.
(3) T herapy cons i s t s of eat i ng four t o s i x s mal l meal s dai l y t hat are l ow i n carbohydrat e and hi gh i n prot ei n.
Pa g e 2 0 8 1
ABC Ambe r CHM Conve rte r Tria l ve rsion, http://w w w .proce sste x t.com/a bcchm.html
V. Disorders of the Adrenal Gland A. General considerations
1. Di s eas es of t he adrenal cortex are caus ed by t he exces s i ve product i on of cort i s ol (Cushing' s syndrome), al dos t erone (primary aldosteronism), and adrenal androgens (congenital adrenal hyperplasia), as wel l as by i nadequat e product i on of cort i s ol and al dos t erone (Addison' s disease).
2. Los s of t he adrenal medulla does not caus e i l l nes s , but catecholamine overproduction by a pheochromocytoma (a t umor of t he adrenal medul l a) caus es a charact eri s t i c hypert ens i ve s yndrome.
B. Cushing's syndrome Cushing' s syndrome i s caus ed by exces s i ve concent rat i ons of cort i s ol or ot her gl ucocort i coi d hormones i n t he ci rcul at i on.
1. Etiology o
o
a. The mos t common caus e of s pont aneous Cus hi ng's s yndrome i s bilateral adrenal hyperplasia (al s o known as Cushing' s disease). Bi l at eral adrenal hyperpl as i a i s caus ed by i ncreas ed s ecret i on of ACTH by a cort i cot roph adenoma of t he pi t ui t ary gl and. Exces s product i on of ACTH by t he pi t ui t ary gl and i s Cus hi ng's di s eas e. Cus hi ng's s yndrome i s a nons peci fi c des i gnat i on t hat refers t o i ncreas ed gl ucocort i coi d l evel s from any ori gi n.
o
Pa g e 2 0 8 2
ABC Ambe r CHM Conve rte r Tria l ve rsion, http://w w w .proce sste x t.com/a bcchm.html
o
b. Adrenal adenomas and adrenal carcinomas may caus e Cus hi ng's s yndrome.
o
o
c. Ectopic ACT H product i on by t umors s uch as oat cel l carci noma of t he l ung, carci noma of t he pancreas , bronchi al carci noi d t umors , and ot hers caus es adrenal hyperpl as i a and Cus hi ng's s yndrome.
o
o
d. Iatrogenic Cushing' s syndrome i s s een more oft en t han t he s pont aneous l y occurri ng s yndrome. It i s an expect ed compl i cat i on i n pat i ent s recei vi ng l ong-t erm gl ucocort i coi d t reat ment for as t hma, art hri t i s , and ot her condi t i ons .
2. Clinical features o
o
a. Central obesity i s caus ed by t he effect of exces s gl ucocort i coi d l evel s on fat di s t ri but i on. Fat accumul at es i n t he face, neck, and t runk, whereas t he l i mbs remai n t hi n. The “ moon face,― “ buffalo hump― (cervi cal fat pad), and supraclavicular fat pads cont ri but e t o t he “cus hi ngoi d― appearance of affect ed i ndi vi dual s .
o
o
b. Hypertension res ul t s from t he vas cul ar effect s of cort i s ol as wel l as ot her act i ons of t he hormone, i ncl udi ng s odi um ret ent i on.
o
Pa g e 2 0 8 3
ABC Ambe r CHM Conve rte r Tria l ve rsion, http://w w w .proce sste x t.com/a bcchm.html o
c. Decreased glucose tolerance i s common; 20% of pat i ent s have overt di abet es . Thi s i s a res ul t of t he i ncreas ed hepat i c gl uconeogenes i s and decreas ed peri pheral gl ucos e ut i l i zat i on caus ed by el evat ed l evel s of gl ucocort i coi d.
o
o
d. Symptoms of androgen excess (e.g., ol i gomenorrhea, hi rs ut i s m, and acne) may occur i n women wi t h Cus hi ng's di s eas e becaus e of s t i mul at i on by ACTH of adrenal androgen product i on.
o
o
e. Purple striae are l i near marks on t he abdomen, where t he t hi n, was t ed s ki n i s s t ret ched by underl yi ng fat .
o
o
f. Muscle wasting and weakness refl ect t he cat abol i c effect s of cort i s ol on mus cl e prot ei n.
o
o
g. Osteoporosis i s a frequent res ul t of cort i s ol exces s . It i s caus ed by i ncreas ed bone cat abol i s m and perhaps by t he i nhi bi t ory effect s of cort i s ol on col l agen s ynt hes i s and cal ci um abs orpt i on.
o
o
h. Susceptibility to bruising i s probabl y caus ed by enhanced capi l l ary fragi l i t y.
o
o
i. Psychiatric disturbances, es peci al l y depres s i on, are frequent res ul t s of cort i s ol exces s .
o
Pa g e 2 0 8 4
ABC Ambe r CHM Conve rte r Tria l ve rsion, http://w w w .proce sste x t.com/a bcchm.html o
j. Growth retardation i n chi l dren may be s evere.
3. Diagnosis. Serum and uri ne cort i s ol l evel s are el evat ed i n Cus hi ng's di s eas e and Cus hi ng's s yndrome, but overl ap wi t h normal val ues i s common, and el evat ed l evel s are s een i n normal pers ons at t i mes of s t res s . Therefore, t es t s of t he s uppres s i bi l i t y of cort i s ol and ot her s peci al t es t s are P.422
neces s ary. Fi gure 9-9 s hows a s cheme for t he di agnos i s of Cus hi ng's s yndrome t hat pos es t wo ques t i ons . Is Cus hi ng's s yndrome pres ent (Fi gure 9-9A)? If i t i s pres ent , what caus es i t s occurrence (Fi gure 9-9B)?
Pa g e 2 0 8 5
ABC Ambe r CHM Conve rte r Tria l ve rsion, http://w w w .proce sste x t.com/a bcchm.html
FIGURE 9-9 Al gori t hms t o det ermi ne t he di agnos i s and et i ol ogy of Cus hi ng's s yndrome. (A) St eps t o t ake t o det ermi ne whet her a pat i ent has Cus hi ng's s yndrome (of any caus e). (B) St eps t o t ake t o i dent i fy t he caus e of Cus hi ng's s yndrome (once t he di agnos i s has been made). ACTH, adrenocort i cot ropi c hormone; CRH,
Pa g e 2 0 8 6
ABC Ambe r CHM Conve rte r Tria l ve rsion, http://w w w .proce sste x t.com/a bcchm.html
cort i cot ropi n-rel eas i ng hormone; CT, comput ed t omography; MRI, magnet i c res onance i magi ng. o
o
a. Nonspecific laboratory abnormalities i ncl ude l eukocyt os i s , wi t h a rel at i vel y l ow percent age of l ymphocyt es and eos i nophi l s , and an el evat i on i n t he s erum gl ucos e l evel . P.423
o
o
b. The serum cortisol l evel i n normal i ndi vi dual s i s hi ghes t i n earl y morni ng and decreas es t hroughout t he day, reachi ng a l ow poi nt at about mi dni ght . Al t hough t he morni ng l evel may be i ncreas ed i n pat i ent s wi t h Cus hi ng's s yndrome, a l os s of t he normal di urnal vari at i on and an i ncreas e i n t he eveni ng l evel are more cons i s t ent fi ndi ngs . Sal i vary cort i s ol can be meas ured at ni ght .
o
o
c. The 24-hour urinary free cortisol excretion rate i s i ncreas ed i n mos t pat i ent s wi t h Cus hi ng's s yndrome. Thi s t es t i s t he mos t us eful i ndi cat or of dai l y cort i s ol s ecret i on.
o
o
d. ACT H measurement may hel p di fferent i at e t he caus es of Cus hi ng's s yndrome.
(1) ACTH l evel s are us ual l y hi gh-normal or s l i ght l y el evat ed i n pat i ent s wi t h Cus hi ng's di s eas e and may be markedl y el evat ed i n pat i ent s wi t h ect opi c ACTH product i on.
Pa g e 2 0 8 7
ABC Ambe r CHM Conve rte r Tria l ve rsion, http://w w w .proce sste x t.com/a bcchm.html
(2) W hen an aut onomous l y funct i oni ng adrenal t umor i s t he s ource of exces s cort i s ol s ecret i on, pi t ui t ary s ecret i on of ACTH i s s uppres s ed becaus e of t he hi gh l evel s of ci rcul at i ng cort i s ol , and t he ACTH l evel i s ext remel y l ow or undet ect abl e.
o
o
e. Low-dose dexamethasone suppression tests
(1) The overnight low-dose dexamethasone suppression test i s recommended as an i ni t i al s creeni ng procedure for any pat i ent s us pect ed of havi ng Cus hi ng's s yndrome. The pat i ent t akes 1 mg dexamet has one oral l y at 11:00 PM, and t he pl as ma cort i s ol l evel i s meas ured at 8:00 AM t he fol l owi ng day.
(a) The pl as ma cort i s ol l evel i s l es s t han 2 µg/dL i n mos t i ndi vi dual s , i ndi cat i ng normal s uppres s i on of ACTH and cort i s ol by t he dexamet has one. Becaus e t hi s t es t i s very s ens i t i ve, t he di agnos i s of Cus hi ng's s yndrome i s unl i kel y i n pat i ent s wi t h a normal res pons e.
(b) Pat i ent s wi t h Cus hi ng's s yndrome have cort i s ol l evel s great er t han 3 µg/dL; t he l evel s us ual l y exceed 10 µg/dL. Thi s res ul t i ndi cat es t hat furt her s t udy i s needed. (The t es t i s not very
Pa g e 2 0 8 8
ABC Ambe r CHM Conve rte r Tria l ve rsion, http://w w w .proce sste x t.com/a bcchm.html
s peci fi c; ment al or phys i cal s t res s may produce a fal s e-pos i t i ve res ul t .)
(2) In t he standard low-dose dexamethasone suppression test, t he pat i ent t akes dexamet has one 0.5 mg every 6 hours for 48 hours , s t art i ng at 8:00 AM (ei ght dos es ). The morni ng pl as ma cort i s ol drawn aft er 48 hours i s normal l y s uppres s ed t o l es s t han 2 µg/dL.
o
o
f. High-dose dexamethasone suppression tests
(1) In t he overnight high-dose dexamethasone suppression test, pl as ma cort i s ol i s meas ured at 8:00 AM on t wo cons ecut i ve days , and dexamet has one 8 mg i s t aken at 11:00 PM on t he fi rs t day. A decreas e i n t he pl as ma cort i s ol l evel of l es s t han 50% on t he s econd day i ndi cat es fai l ure of s uppres s i on. The hi gh-dos e t es t i s des i gned t o di s t i ngui s h bet ween t he t wo pri mary caus es of ACTH-dependent Cus hi ng's s yndrome.
(a) Pat i ent s wi t h Cus hi ng's di s eas e behave as t hough t hei r feedback res pons e t o gl ucocort i coi ds i s i nt act but s et at a hi gher-t han-normal l evel ; t hey res pond t o hi gh but not t o l ow dos es of dexamet has one.
(b) Pat i ent s wi t h ect opi c ACTH s ecret i on
Pa g e 2 0 8 9
ABC Ambe r CHM Conve rte r Tria l ve rsion, http://w w w .proce sste x t.com/a bcchm.html
produce ACTH aut onomous l y; t hei r cort i s ol l evel s are not s uppres s ed even by hi gh dos es of dexamet has one.
(2) In t he standard high-dose dexamethasone suppression test, 2 mg of dexamet has one i s t aken every 6 hours for 48 hours (ei ght dos es ). A fal l of 50% or more i n t he morni ng pl as ma cort i s ol at 48 hours i ndi cat es s uppres s i on.
o
o
g. In inferior petrosal sinus sampling, ACTH concent rat i ons are meas ured i n venous bl ood obt ai ned by cat het eri zat i on of t he i nferi or pet ros al s i nus es . Cort i cot ropi n-rel eas i ng hormone (CRH) may be i nject ed t o s t i mul at e ACTH s ecret i on. A hi gh concent rat i on of ACTH compared wi t h t hat i n peri pheral bl ood i ndi cat es t hat a pi t ui t ary adenoma i s t he caus e of ACTH exces s ; i f one s uperi or pet ros al s i nus has a hi gh ACTH concent rat i on whi l e t he ot her has a concent rat i on s i mi l ar t o peri pheral bl ood, t he adenoma i s more l i kel y t o be on t he s i de of t he pi t ui t ary corres pondi ng t o t he hi gher concent rat i on.
o
o
h. A combined standard low-dose dexamethasone suppression test and CRH test may be us ed when ot her t es t s have fai l ed t o di s t i ngui s h bet ween t rue Cus hi ng's s yndrome and t he “ps eudo-Cus hi ng's s yndrome― s omet i mes s een wi t h al cohol i s m or depres s i on.
(1) Aft er t he admi ni s t rat i on of 0.5 mg
Pa g e 2 0 9 0
ABC Ambe r CHM Conve rte r Tria l ve rsion, http://w w w .proce sste x t.com/a bcchm.html
dexamet has one every 6 hours for 48 hours , CRH i s i nject ed i nt ravenous l y, and pl as ma cort i s ol i s meas ured 15 mi nut es l at er. Pat i ent s wi t h Cus hi ng's s yndrome have cort i s ol l evel s great er t han 1.4 µg/dL, whereas pat i ent s wi t h “ps eudo-Cus hi ng's s yndrome― have l evel s l es s t han 1.4 µg/dL. P.424
(2) Thi s combi ned t es t makes us e of t he fol l owi ng obs ervat i ons : pat i ent s wi t h adrenal t umors or ect opi c ACTH product i on are res i s t ant t o s uppres s i on of cort i s ol by dexamet has one, whereas pat i ent s wi t h Cus hi ng's di s eas e have exaggerat ed res pons es t o CRH. By combi ni ng t hes e t es t s , i t i s pos s i bl e t o obt ai n a great er s eparat i on bet ween t he res ul t s s een i n Cus hi ng's s yndrome and t hos e s een i n ps eudo-Cus hi ng's s yndrome.
o
o
i. Radiographic findings
(1) Skull radiographs s how enl argement of t he s el l a t urci ca i n t he 10% of pat i ent s wi t h Cus hi ng's s yndrome who have macroadenomas, but t hey do not reveal mos t of t hes e t umors , whi ch are microadenomas averagi ng 5–6 mm i n di amet er.
Pa g e 2 0 9 1
ABC Ambe r CHM Conve rte r Tria l ve rsion, http://w w w .proce sste x t.com/a bcchm.html
(2) CT scans with injection of contrast medium det ect approxi mat el y 50% of t he pi t ui t ary adenomas t hat caus e Cus hi ng's di s eas e. MRI with gadolinium contrast, however, reveal s approxi mat el y 75% of t hes e t umors and i s t he met hod of choi ce.
(3) CT scans of the adrenal gland s how mos t adrenal t umors . Uni form enl argement of bot h adrenal gl ands s ugges t s an ACTH-dependent form of Cus hi ng's s yndrome, ei t her Cus hi ng's di s eas e or t he ect opi c ACTH s yndrome.
4. T herapy o
o
a. Adrenal adenomas can us ual l y be res ect ed compl et el y, oft en l aparos copi cal l y, wi t h cure of t he di s eas e. Cort i s ol repl acement may be needed for s everal mont hs t o a year pos t operat i vel y, unt i l t he remai ni ng normal adrenal t i s s ue (s uppres s ed by t he previ ous hi gh cort i s ol l evel s ) regai ns i t s abi l i t y t o produce cort i s ol .
o
o
b. Adrenal carcinoma i s oft en i noperabl e when fi rs t di agnos ed becaus e of met as t as es , us ual l y t o t he l i ver and t he l ungs . Mi t ot ane, met yrapone, and ami nogl ut et hi mi de are drugs t hat bl ock adrenal s t eroi d product i on, and t hey may rel i eve t he mani fes t at i ons of exces s cort i s ol product i on i n pat i ent s wi t h i noperabl e adrenal carci noma. Prol onged s urvi val of t hes e pat i ent s i s uncommon.
o
Pa g e 2 0 9 2
ABC Ambe r CHM Conve rte r Tria l ve rsion, http://w w w .proce sste x t.com/a bcchm.html
o
c. The ectopic ACT H syndrome can be cured by removal of t he t umor, but t hi s i s not pos s i bl e i n many cas es . The t umor caus i ng t he s yndrome, rat her t han t he Cus hi ng's s yndrome i t s el f, i s us ual l y t he pri mary probl em.
o
o
d. Cushing' s disease may be t reat ed i n s everal ways .
(1) T ranssphenoidal pituitary surgery is the treatment of choice. Even when t umors cannot be s een on CT s can or MRI, t rans s phenoi dal expl orat i on may di s cl os e a mi croadenoma. Surgery i s s ucces s ful i n 50%–95% of cas es and i s fol l owed by normal pi t ui t ary and adrenal funct i on as wel l as cure of Cus hi ng's di s eas e.
(2) Pituitary irradiation i s effect i ve i n many chi l dren, but i t cures fewer t han one-t hi rd of affect ed adul t s ; t he reas on for t he di fference i n res pons e bet ween chi l dren and adul t s i s uncl ear.
(3) Bilateral adrenalectomy cures Cus hi ng's di s eas e but l eaves t he pat i ent wi t h Addison' s disease and t he need for l i fel ong s t eroi d repl acement . In addi t i on, adrenal ect omy i s s omet i mes fol l owed by t he devel opment of Nelson' s syndrome, i n whi ch a pi t ui t ary adenoma undergoes rapi d growt h, perhaps becaus e i t i s no l onger i nhi bi t ed by
Pa g e 2 0 9 3
ABC Ambe r CHM Conve rte r Tria l ve rsion, http://w w w .proce sste x t.com/a bcchm.html
above-normal l evel s of cort i s ol .
(4) Medical therapy can be t ri ed i n a s el ect group of pat i ent s . Ket oconazol e, mi t ot ane, or ami nogl ut et hi mi de (t he l at t er t wo are more t oxi c) coul d be us ed.
C. Adrenal insufficiency
1. Etiology o
o
a. Primary adrenal insufficiency(Addison' s disease)
(1) Idiopathic atrophy of the adrenal cortex due t o an aut oi mmune proces s i s t he mos t common caus e of adrenal i ns uffi ci ency.
(2) T uberculosis may i nvol ve t he adrenal gl ands , wi t h des t ruct i on of bot h t he adrenal cort ex and medul l a.
(3) Iatrogenic causes
(a) Bilateral adrenalectomy for Cus hi ng's di s eas e res ul t s i n adrenal i ns uffi ci ency.
(b) Adrenal suppression following
Pa g e 2 0 9 4
ABC Ambe r CHM Conve rte r Tria l ve rsion, http://w w w .proce sste x t.com/a bcchm.html
prolonged steroid therapy may pers i s t for up t o 1 year or l onger.
(4) Acquired immunodeficiency syndrome (AIDS) s omet i mes l eads t o adrenal i ns uffi ci ency t hrough cyt omegal ovi rus (CMV) i nfect i on and ot her i nfect i ons of t he adrenal gl ands . P.425
(5) Adrenoleukodystrophy i s an X-l i nked di s order caus ed by a defi ci ency of very-l ong-chai n acyl CoA s ynt het as e. Thi s l eads t o an accumul at i on of very-l ong-chai n fat t y aci ds i n t he adrenal gl ands , caus i ng adrenal i ns uffi ci ency, and i n t he CNS, caus i ng a demyel i nat i ng s yndrome.
(6) Less common causes of adrenal des t ruct i on i ncl ude amyloidosis, fungal infections, syphilis, bilateral adrenal hemorrhage (es peci al l y i n pat i ent s recei vi ng ant i coagul ant s ), and metastatic malignancy.
o
o
b. Secondary adrenal insufficiency i s due t o pi t ui t ary di s eas e and res ul t s from any of t he caus es of hypopi t ui t ari s m (s ee I A 2 a).
2. Clinical features. The s ympt oms of adrenal i ns uffi ci ency are caus ed by bot h cort i s ol and al dos t erone
Pa g e 2 0 9 5
ABC Ambe r CHM Conve rte r Tria l ve rsion, http://w w w .proce sste x t.com/a bcchm.html
defi ci enci es . o
o
a. Cortisol deficiency
(1) Hyperpigmentation of the skin i s caus ed by i ncreas ed MSH act i vi t y t hat accompani es t he i ncreas ed pi t ui t ary s ecret i on of ACTH. The l at t er i s a feedback res pons e t o t he cort i s ol defi ci ency.
(a) Hyperpi gment at i on i s mos t not i ceabl e over expos ed areas , on mucous membranes , and i n s ki n creas es and s cars .
(b) In s econdary adrenal i ns uffi ci ency (whi ch i s caus ed by pi t ui t ary di s eas e), ACTH l evel s are l ow rat her t han el evat ed, and hyperpi gment at i on i s abs ent .
(2) Hypotension, oft en ort hos t at i c, i s caus ed by t he abs ence of t he pres s or effect of cort i s ol on vas cul ar t one and by a decreas e i n cardi ac out put . In Addi s on's di s eas e al dos t erone defi ci ency al s o pl ays a rol e.
(3) Gastrointestinal symptoms i ncl ude anorexi a, naus ea and vomi t i ng, and wei ght l os s .
Pa g e 2 0 9 6
ABC Ambe r CHM Conve rte r Tria l ve rsion, http://w w w .proce sste x t.com/a bcchm.html
(4) Hypoglycemia i s rel at ed t o decreas ed cort i s ol -i nduced gl uconeogenes i s .
(5) Mental symptoms may i ncl ude l et hargy and confus i on. Ps ychot i c mani fes t at i ons occur on occas i on.
(6) Intolerance to stress may occur. Pat i ent s who cannot i ncreas e t hei r cort i s ol out put i n res pons e t o s evere s t res s ri s k an acut e exacerbat i on of t he s ympt oms di s cus s ed above, wi t h l i fe-t hreat eni ng vas cul ar col l aps e.
o
o
b. Aldosterone deficiency
(1) Sodium loss res ul t s from reduced al dos t erone-medi at ed reabs orpt i on of s odi um i n t he di s t al renal t ubul es . Hypovolemia, decreased cardiac output, and decreased renal blood flow with azotemia as wel l as weaknes s , hypot ens i on, and wei ght l os s may be rel at ed t o s odi um depl et i on.
(2) Potassium retention caus ed by al dos t erone defi ci ency may l ead t o hyperkalemia and cardiac arrhythmias.
(3) Becaus e angiotensin II, rat her t han ACT H, has pri mary cont rol of al dos t erone p roduct i on, and t he reni n–angi ot ens i n s ys t em
Pa g e 2 0 9 7
ABC Ambe r CHM Conve rte r Tria l ve rsion, http://w w w .proce sste x t.com/a bcchm.html
i s not affect ed by ACTH defi ci ency, t here i s us ual l y no defi ci ency of al dos t erone i n s econdary adrenal i ns uffi ci ency. o
o
3. Diagnosis
a. ACT H response
(1) The normal adrenal gl and s harpl y i ncreas es i t s out put of cort i s ol when s t i mul at ed by ACTH; abs ence of t hi s res pons e i ndi cat es adrenal i ns uffi ci ency.
(2) ACT H test. The s erum l evel of cort i s ol i s meas ured before and 1 hour aft er an i nt ravenous or i nt ramus cul ar i nject i on of 0.25 mg (25 U) cos ynt ropi n, a s ynt het i c form of ACTH. The pl as ma cort i s ol s houl d reach a l evel of 20 µg/dL or hi gher.
o
o
b. Laboratory findings
(1) Nons peci fi c l aborat ory abnormal i t i es may i ncl ude hyponatremia, hyperkalemia, hypoglycemia, and an increased eosinophil count (gl ucocort i coi ds l ower t he eos i nophi l count ). Ches t radi ography may s how a small heart.
Pa g e 2 0 9 8
ABC Ambe r CHM Conve rte r Tria l ve rsion, http://w w w .proce sste x t.com/a bcchm.html
(2) Plasma cortisol, urinary free cortisol, and urinary 17-hydroxycorticosteroid levels are low. Bas el i ne l evel s , however, may overl ap wi t h t he val ues i n normal i ndi vi dual s , whi ch i s why ACTH t es t i ng i s neces s ary for a defi ni t i ve di agnos i s .
o
o
4. T herapy
a. Glucocorticoid replacement is needed in all patients.
(1) The us ual dos e of cort i s ol i s 10–30 mg dai l y. A hi gher dos e i s us ual l y gi ven i n t he morni ng and a s mal l er dos e i n t he eveni ng, t o mi mi c t he normal di urnal vari at i on. To avoi d t he P.426
pot ent i al harmful effect s of exces s cort i s ol , s uch as os t eoporos i s , t he dos e s houl d be no more t han i s neces s ary t o rel i eve t he mani fes t at i ons of cort i s ol defi ci ency.
(2) The dos e mus t be i ncreas ed duri ng t i mes of s t res s . Typi cal dos es woul d be t wi ce t he us ual dos e duri ng mi nor s t res s (e.g., common col d or dent al ext ract i on), 3–5 t i mes t he us ual dos e duri ng moderat e s t res s (e.g., i nfl uenz a or mi nor s urgery), and “s t res s dos es ― of 200–300 mg duri ng s evere s t res s (e.g.,
Pa g e 2 0 9 9
ABC Ambe r CHM Conve rte r Tria l ve rsion, http://w w w .proce sste x t.com/a bcchm.html
major s urgery or a s eri ous i nfect i on or i njury).
b. Mineralocorticoid replacement i s needed i n pat i ent s wi t h Addi s on's di s eas e. Fludrocortisone (Fl ori nef) i s gi ven i n a dai l y dos e of 0.05–0.2 mg. Pers i s t ence of l ow bl ood pres s ure, weaknes s , and l ow s erum s odi um and hi gh s erum pot as s i um l evel s s ugges t t hat a hi gher dos e of mi neral ocort i coi d i s needed; hypert ens i on, edema, or hypokal emi a s ugges t t hat t he dos e s houl d be decreas ed.
o
o
5. Adrenal crisis (addisonian crisis) i s an acut e, l i fe-t hreat eni ng compl i cat i on of Addi s on's di s eas e i n whi ch t he mani fes t at i ons of adrenal i ns uffi ci ency are great l y exaggerat ed.
a. Clinical features. Fever, vomiting, abdominal pain, altered mental status, and vascular collapse may occur i f Addi s on's di s eas e remai ns unt reat ed, or i t may occur i n a t reat ed pat i ent duri ng acut e s t res s i f addi t i onal gl ucocort i coi d repl acement i s not provi ded.
b. T herapy. Immedi at e i nt ravenous admi ni s t rat i on of 100 mg cort i s ol over 5–10 mi nut es s houl d be fol l owed by an addi t i onal 300 mg i n t he next 24 hours . Int ravenous s al i ne i s al s o needed, and mi neral ocort i coi d repl acement s houl d be provi ded i f hypot ens i on
Pa g e 2 1 0 0
ABC Ambe r CHM Conve rte r Tria l ve rsion, http://w w w .proce sste x t.com/a bcchm.html
and vol ume depl et i on pers i s t .
D. Primary aldosteronism
1. Etiology. Exces s i ve adrenal product i on of al dos t erone i s us ual l y caus ed by a s i ngl e small (0.5–3.0 cm) adrenal adenoma. Les s oft en (i .e., i n 20%–40% of cas es ), t here i s bi l at eral hyperpl as i a of t he adrenal cort ex.
2. Clinical features. Al dos t erone i ncreas es t he reabs orpt i on of s odi um and t he excret i on of pot as s i um and hydrogen i ons i n t he di s t al renal t ubul es . o
o
a. Sodium retention causes blood pressure elevation, whi ch i s t he chi ef cl i ni cal mani fes t at i on of t hi s s yndrome.
(1) The amount of s odi um and wat er t hat are ret ai ned i s l i mi t ed by compens at ory mechani s ms t hat i ncreas e renal s odi um excret i on i n res pons e t o ext racel l ul ar fl ui d vol ume expans i on; s odi um bal ance i s res t ored aft er 1–2 kg of fl ui d have accumul at ed.
(2) Al t hough t hi s amount of vol ume expans i on does not caus e edema, t he l ong-t erm i ncreas e i n cardi ac out put and, perhaps , ot her effect s of mi neral ocort i coi d exces s l ead t o hypert ens i on.
o
Pa g e 2 1 0 1
ABC Ambe r CHM Conve rte r Tria l ve rsion, http://w w w .proce sste x t.com/a bcchm.html o
b. Potassium loss causes hypokalemia, whi ch may produce muscle weakness, paresthesias, and tetany i n s evere cas es .
(1) Hypokalemic nephropathy may caus e pol yuri a.
(2) Metabolic alkalosis i s a res ul t of t he renal l os s of pot as s i um and hydrogen i ons .
3. Diagnosis o
o
a. Laboratory diagnosis
(1) Hypokalemia i n a hypert ens i ve pat i ent i s oft en t he cl ue t hat t ri ggers t he s earch for pri mary al dos t eroni s m, al t hough not al l pat i ent s wi t h al dos t eroni s m have a l ow s erum l evel of pot as s i um.
(2) Aldosterone mus t be meas ured under s t andardi zed condi t i ons becaus e i t i s affect ed by s odi um bal ance, di uret i cs , and ot her fact ors .
(a) Di uret i cs , ACE i nhi bi t ors , and vas odi l at ors s houl d be di s cont i nued at l eas t 2 weeks before s t udi es of al dos t erone (and reni n) are undert aken.
Pa g e 2 1 0 2
ABC Ambe r CHM Conve rte r Tria l ve rsion, http://w w w .proce sste x t.com/a bcchm.html
(b) Random al dos t erone meas urement s i n pat i ent s wi t h pri mary al dos t eroni s m may overl ap t hos e of normal i ndi vi dual s ; s odi um l oadi ng may be neces s ary t o di fferent i at e t he al dos t erone l evel s i n affect ed pat i ent s (whi ch are not s uppres s ed by a s odi um l oad) from t he l evel s i n normal i ndi vi dual s (whi ch are s uppres s ed by a s odi um l oad). Two of t he many ways t hat t hi s procedure can be performed are as fol l ows .
(i) The 24-hour urinary aldosterone excretion rate can be meas ured aft er t he pat i ent has i nges t ed more t han 250 mEq of s odi um dai l y for at l eas t 3 days . (Thi s can be ens ured by gi vi ng s odi um chl ori de t abl et s .) Sodi um l oadi ng may furt her l ower s erum l evel s of pot as s i um; t herefore, caut i on i s neces s ary i f t he pat i ent is P.427
hypokal emi c. An el evat ed al dos t erone l evel i n a 24-hour uri ne s ampl e t hat cont ai ns more t han 250 mEq of s odi um i ndi cat es hyperal dos t eroni s m.
(ii) Plasma levels of aldosterone can be meas ured aft er 2000 mL of
Pa g e 2 1 0 3
ABC Ambe r CHM Conve rte r Tria l ve rsion, http://w w w .proce sste x t.com/a bcchm.html
normal s al i ne have been i nfus ed over 4 hours . A val ue above 10 ng/dL i s cons i s t ent wi t h pri mary al dos t eroni s m. An eas i er, and maybe s afer t es t can be done by gi ve s odi um oral l y for 3 days aft er whi ch a 24-hour uri ne col l ect i on s houl d be done t o meas ure al dos t erne s ecret i on. A l evel great er t han 10 t o 14 mcg/day t oget her wi t h an uri nary s odi um of 250 mmol /day i s di agnos t i c of pri mary al dos t eroni s m. Severe hypert ens i on and conges t i ve heart fai l ure (CHF) are cont rai ndi cat i ons t o s al i ne i nfus i on.
(3) Plasma renin activity i s t he mos t us eful i ndi cat or of whet her el evat ed al dos t erone product i on i s pri mary or s econdary.
(a) Secondary aldosteronism i s caus ed by condi t i ons t hat ori gi nat e out s i de t he adrenal gl and and t hat reduce t he effect i ve art eri al bl ood vol ume, t hereby di mi ni s hi ng t he pres s ure or t ens i on s ens ed by t he juxt agl omerul ar cel l s .
(i) Such condi t i ons i ncl ude heart fai l ure, nephros i s , ci rrhos i s , vol ume depl et i on caus ed by di uret i cs , and renovas cul ar di s eas e.
Pa g e 2 1 0 4
ABC Ambe r CHM Conve rte r Tria l ve rsion, http://w w w .proce sste x t.com/a bcchm.html
(ii) Decreas ed pres s ure on t he juxt agl omerul ar cel l s s t i mul at es reni n rel eas e, whi ch i ncreas es angi ot ens i n II and, i n t urn, al dos t erone. Thus , the high aldosterone level is accompanied by increased renin activity.
(b) In primary aldosteronism, t he enhanced al dos t erone product i on i s caus ed by an adrenal abnormal i t y, not by i ncreas ed reni n act i vi t y; t he res ul t i ng vol ume expans i on s uppres s es reni n product i on. Thi s combi nat i on of increased aldosterone production and reduced renin activity can be caused only by primary aldosteronism, and i t i s a rel i abl e i ndi cat or of t hi s di agnos i s .
(c) Suppres s i on of reni n act i vi t y i s di agnos ed wi t h cert ai nt y onl y i f l evel s remai n l ow aft er mani pul at i ons t hat are known t o s t i mul at e reni n i n normal i ndi vi dual s s uch as di et ary s odi um res t ri ct i on, s everal hours of upri ght pos t ure, or furos emi de admi ni s t rat i on.
(4) The best screening test for primary aldosteronism i s meas urement of t he rat i o of pl as ma al dos t erone (ng/dL) t o pl as ma reni n act i vi t y (ng of angi ot ens i n I/mL/hr) i n a bl ood s ampl e drawn wi t h t he pat i ent i n an upri ght pos i t i on. Thi s rat i o i s i ncreas ed bot h by t he
Pa g e 2 1 0 5
ABC Ambe r CHM Conve rte r Tria l ve rsion, http://w w w .proce sste x t.com/a bcchm.html
el evat ed al dos t erone l evel and t he s uppres s ed reni n l evel charact eri s t i c of t he di s eas e. An al dos t erone-t o-reni n rat i o great er t han 25 i ndi cat es need for furt her s t udy. o
o
b. Adenoma versus hyperplasia. Pri mary al dos t eroni s m due t o an adenoma mus t be di s t i ngui s hed from pri mary al dos t eroni s m due t o hyperpl as i a, becaus e t he di s t i nct i on affect s t reat ment , whi ch i s us ual l y s urgi cal i n cas es of adrenal adenoma and medi cal i n bi l at eral hyperpl as i a.
(1) The bi ochemi cal changes of pri mary al dos t eroni s m—t he hypokal emi a, t he i ncreas ed al dos t erone l evel , and t he l ow reni n act i vi t y—are more pronounced i n cas es caus ed by a uni l at eral adenoma t han i n cas es caus ed by hyperpl as i a.
(2) The pl as ma al dos t erone concent rat i on may be meas ured at 8:00 AM, aft er 8 hours of recumbency, and agai n at noon, aft er 4 hours of ambul at i on.
(a) Level s are hi gher aft er ambul at i on i n normal s ubject s and i n pat i ent s wi t h bi l at eral hyperpl as i a, becaus e reni n and angi ot ens i n are s t i mul at ed by t he upri ght pos t ure and s ympat het i c out fl ow.
(b) However, pat i ent s wi t h uni l at eral
Pa g e 2 1 0 6
ABC Ambe r CHM Conve rte r Tria l ve rsion, http://w w w .proce sste x t.com/a bcchm.html
adrenal adenomas have a paradoxi cal fal l i n pl as ma al dos t erone l evel s , pres umabl y becaus e when reni n i s profoundl y s uppres s ed, al dos t erone i s i nfl uenced mai nl y by t he di urnal fal l i n ACTH.
(3) Adrenal vein aldosterone concentrations may be meas ured i n bl ood s ampl es obt ai ned by s el ect i ve cat het eri zat i on. A very hi gh l evel on one s i de i ndi cat es an adenoma; hi gh l evel s on bot h s i des i ndi cat e bi l at eral hyperpl as i a.
(4) CT scans and MRI s omet i mes s how al dos t erone-produci ng adenomas , but t hes e t umors may be s mal l and oft en cannot be vi s ual i zed.
(5) An al gori t hm for t he di agnos i s of pri mary al dos t eroni s m i s s hown i n Fi gure 9-10.
o
o
c. Ingestion of large amounts of natural black licorice s houl d be cons i dered i n t he di fferent i al di agnos i s .
4. T herapy o
o
a. Surgery
(1) Removal of a uni l at eral adenoma res ul t s i n cure of t he hypert ens i on i n approxi mat el y
Pa g e 2 1 0 7
ABC Ambe r CHM Conve rte r Tria l ve rsion, http://w w w .proce sste x t.com/a bcchm.html
60% of cas es and i mprovement i n anot her 25%. P.428
FIGURE 9-10 Al gori t hm for t he di agnos i s of pri mary al dos t eroni s m. CT, comput ed t omography; MRI, magnet i c res onance i magi ng. (From Adl i n EV, Endoc ri nol ogy Sc i enc e and Medi c i ne. Phi l adel phi a: Li ppi ncot t W i l l i ams & W i l ki ns , 2001:123. )
Pa g e 2 1 0 8
ABC Ambe r CHM Conve rte r Tria l ve rsion, http://w w w .proce sste x t.com/a bcchm.html
(2) In cont ras t , onl y 20%–50% of pat i ent s wi t h bi l at eral hyperpl as i a are i mproved by s urgery, even i f bi l at eral adrenal ect omy i s performed. Medi cal t herapy i s preferabl e. Epl erenone 50–100 mg dai l y can be us ed i n pat i ent s who devel op gynecomas t i a on s pi ronol act one.
o
o
b. Medical therapy. Spironolactone i nhi bi t s t he effect s of al dos t erone on t he renal t ubul e. A dos age of 200–400 mg dai l y correct s t he hypokal emi a and oft en correct s t he hypert ens i on.
E. Congenital adrenal hyperplasia
1. Etiology and pathophysiology. Congeni t al adrenal hyperpl as i a i s caus ed by a defect i n one of t he enzymes t hat are neces s ary for t he s ynt hes i s of cort i s ol . Cort i s ol defi ci ency s t i mul at es ACTH, whi ch caus es hyperpl as i a of t he adrenal cort ex and overproduct i on of what ever ACTH-dependent s t eroi ds are not affect ed by t he enz yme defi ci ency (mai nl y adrenal androgens ).
2. Clinical features o
o
a. Androgen excess i s caus ed by i ncreas ed adrenal product i on of dehydroepi andros t erone, andros t enedi one, and t es t os t erone.
(1) If pres ent duri ng fet al devel opment , t hi s di s order may caus e ambiguous genitalia i n femal e i nfant s . If androgen exces s i s
Pa g e 2 1 0 9
ABC Ambe r CHM Conve rte r Tria l ve rsion, http://w w w .proce sste x t.com/a bcchm.html
mani fes t ed i n t he pos t nat al peri od, i t may caus e virilization i n prepubert al gi rl s or i n young women. P.429
(2) In mal e i nfant s , t he cons equence of androgen exces s duri ng fet al devel opment i s macrogeni t os omi a. In t he pos t nat al peri od, t he cons equence i s precocious puberty.
o
o
b. The cort i s ol defi ci t us ual l y does not caus e major cl i ni cal mani fes t at i ons becaus e t he ACTH s t i mul at i on and adrenal hyperpl as i a mai nt ai n cort i s ol l evel s i n t he l ow-normal range, des pi t e t he enzyme defi ci ency.
o
o
c. Ot her mani fes t at i ons occas i onal l y occur, dependi ng on t he s peci fi c enzyme affect ed.
(1) 21-Hydroxylase deficiency account s for 95% of cas es of adrenal hyperpl as i a.
(a) In t he mi l d (s i mpl e vi ri l i zi ng) form, onl y t he androgen-exces s s ympt oms are of i mport ance.
(b) In t he s evere (s al t -l os i ng) form, t he product i on of al dos t erone i s i mpai red as wel l as t hat of cort i s ol ; mi neral ocort i coi d
Pa g e 2 1 1 0
ABC Ambe r CHM Conve rte r Tria l ve rsion, http://w w w .proce sste x t.com/a bcchm.html
defi ci ency l eads t o hyponat remi a, hyperkal emi a, dehydrat i on, and hypot ens i on.
(2) In 11-hydroxylase deficiency, deoxycort i cos t erone, a mi neral ocort i coi d, and adrenal androgens are overproduced. Thi s caus es hypert ens i on t hrough mechani s ms t hat are s i mi l ar t o t hos e caus i ng hypert ens i on i n pri mary al dos t eroni s m.
(3) In 17-hydroxylase deficiency, deoxycort i cos t erone i s overproduced, res ul t i ng i n hypert ens i on. However, becaus e 17-hydroxyl as e i s neces s ary for s ex s t eroi d s ynt hes i s , t here i s androgen defi ci ency as wel l as es t rogen defi ci ency. Thi s caus es t he devel opment of ambi guous geni t al i a i n mal e i nfant s and pri mary amenorrhea i n women.
3. Diagnosis. Concent rat i ons of adrenal androgens and precurs ors of cort i s ol are i ncreas ed i n bl ood and uri ne. The mos t us eful meas urement s are of blood testosterone, androstenedione, dehydroepiandrosterone, and 17-hydroxyprogesterone (a cort i s ol precurs or) as wel l as urinary 17-ketosteroids and pregnanetriol (a met abol i t e of 17-hydroxyproges t erone).
4. T herapy o
o
a. Medical therapy. Cortisol admi ni s t rat i on
Pa g e 2 1 1 1
ABC Ambe r CHM Conve rte r Tria l ve rsion, http://w w w .proce sste x t.com/a bcchm.html
s uppres s es t he overproduct i on of ACTH and adrenal androgens . In t he s al t -l os i ng s yndrome, mi neral ocort i coi d repl acement wi t h fl udrocort i s one may be neces s ary. o
o
b. Surgery. Recons t ruct i ve s urgery of t he ext ernal geni t al i a i n femal e i nfant s i s done i n t he fi rs t few years of l i fe.
F. Pheochromocytoma Thi s t umor i s deri ved from chromaffin cells, t he cel l s t hat s ynt hes i z e and s t ore cat echol ami nes . Locat ed pri mari l y i n t he adrenal medul l a, t hey al s o are l ocat ed i n s ympat het i c gangl i a and el s ewhere. The cel l s i n t he adrenal medul l a produce epi nephri ne and norepi nephri ne; t he ext ra-adrenal chromaffi n cel l s make onl y norepi nephri ne.
1. Epidemiology o
o
a. Incidence. Pheochromocyt oma i s found i n approxi mat el y 0.5% of pat i ent s wi t h s evere hypert ens i on and i n l es s t han 0.05% of al l hypert ens i ve pat i ent s . However, becaus e t hi s t umor may caus e a dramat i c and debi l i t at i ng s yndrome, oft en wi t h fat al compl i cat i ons i f undet ect ed, di agnos t i c effort s and awarenes s are requi red out of proport i on t o t he frequency of occurrence.
o
o
b. Familial occurrence. Pheochromocyt omas may occur s poradi cal l y or may occur as part of one of s everal fami l i al s yndromes .
Pa g e 2 1 1 2
ABC Ambe r CHM Conve rte r Tria l ve rsion, http://w w w .proce sste x t.com/a bcchm.html
(1) Multiple endocrine neoplasia type II (Sipple' s syndrome) i s charact eri zed by mul t i pl e pheochromocyt omas and medul l ary carci noma of t he t hyroi d; hyperparat hyroi di s m i s oft en pres ent .
(2) Neurofibromatosis and von Hippel-Lindau disease may be as s oci at ed wi t h pheochromocyt oma.
2. Pathology. Mos t pheochromocyt omas are s i ngl e t umors of t he adrenal medul l a. However, 10%–20% are l ocat ed out s i de of t he adrenal gl and, and 1%–3% are i n t he ches t or neck. Approxi mat el y 20% are mul t i pl e and 10% are mal i gnant .
3. Clinical features. The mani fes t at i ons of pheochromocyt oma (Tabl e 9-18) are caus ed by i ncreas ed l evel s of ci rcul at i ng cat echol ami nes . o
o
a. Hypertension i s paroxys mal i n approxi mat el y 50% of cas es and i s s us t ai ned i n t he remai nder. The di agnos i s i s oft en s ugges t ed by t he paroxysmal nature of the symptoms, caus ed by vari at i ons i n t he funct i on of t he t umor. At t acks t ypi cal l y l as t l es s t han 1 hour and may be preci pi t at ed by P.430
exerci s e, i nduct i on of anes t hes i a, uri nat i on (s ugges t i ng a pheochromocyt oma of t he bl adder), or pal pat i on of t he
Pa g e 2 1 1 3
ABC Ambe r CHM Conve rte r Tria l ve rsion, http://w w w .proce sste x t.com/a bcchm.html
abdomen.
TABLE 9-18 Manifestations of Pheochromocytoma Hy Pal per pi t t en at i s i o ons n He Ner ad vou ach s ne e
ss an d t re mo r
Sw W e eat i gh i ng t l os s o
o
b. Other features t hat s ugges t t he pres ence of a pheochromocyt oma are hyperglycemia, hypermetabolism, and postural hypotension i n a hypert ens i ve pat i ent .
4. Diagnosis. Pheochromocyt oma i s s us pect ed far more oft en t han i t i s di agnos ed. Many pat i ent s wi t h s ympt oms
Pa g e 2 1 1 4
ABC Ambe r CHM Conve rte r Tria l ve rsion, http://w w w .proce sste x t.com/a bcchm.html
of cat echol ami ne exces s prove t o have normal hormone l evel s . W i t h t he i ncreas ed us e of comput ed i magi ng, an i ncreas i ng number of as ympt omat i c pat i ent s wi t h pheochromocyt omas have been di agnos ed. o
o
a. The l evel s of urine catecholamines and t hei r met abol i t es are el evat ed i n mos t confi rmed cas es .
(1) The 24-hour urinary metanephrine excretion rate may be t he mos t us eful s creeni ng t es t , but t es t s of urinary free catecholamines (i .e., epi nephri ne and norepi nephri ne) and vanillylmandelic acid concent rat i ons are al s o of val ue. Cert ai n medi cat i ons and s t res s can gi ve fal s e-pos i t i ve res ul t s .
(2) St res s ful i l l nes s can rai s e cat echol ami ne l evel s t wofol d; great er t han t wofol d el evat i ons are more s ugges t i ve of pheochromocyt oma.
o
o
b. Serum catecholamine levels are vari abl e and are more di ffi cul t t o i nt erpret t han t he 24-hour uri ne meas urement s . Level s of plasma free metanephrines may prove t o be a s ens i t i ve i ndi cat or of a pheochromocyt oma.
o
o
c. The clonidine suppression test i s us eful i n pat i ent s wi t h mi l d cat echol ami ne el evat i on. Three hours aft er an oral dos e of 0.3 mg cl oni di ne, t he pl as ma l evel of norepi nephri ne i s l owered i nt o t he
Pa g e 2 1 1 5
ABC Ambe r CHM Conve rte r Tria l ve rsion, http://w w w .proce sste x t.com/a bcchm.html
normal range i n mos t normal i ndi vi dual s , but i t remai ns el evat ed i n pat i ent s wi t h pheochromocyt oma. o
o
d. CT scans or MRI of t he abdomen det ect as many as 90% of t hes e t umors becaus e t hey us ual l y are great er t han 1 cm i n di amet er.
o
o
e. Adrenal scanning wi t h
131
I i odobenzyl guani di ne
(MIBG) i s es peci al l y us eful for l ocal i zi ng ext ra-adrenal t umors .
5. T herapy o
o
a. Medical therapy
(1) The α- and β-adrenergi c bl ocki ng agent s are us eful for i noperabl e t umors and for preparat i on for s urgery.
(a) α-Adrenergic blocking agents rel i eve t he hypert ens i on and adrenergi c s ympt oms .
(i) Phenoxybenzamine i s gi ven oral l y, s t art i ng wi t h 10 mg t wi ce dai l y and i ncreas i ng t o 40 mg t wi ce dai l y, i f neces s ary.
Pa g e 2 1 1 6
ABC Ambe r CHM Conve rte r Tria l ve rsion, http://w w w .proce sste x t.com/a bcchm.html
(ii) Phentolamine can be gi ven i nt ravenous l y t o t reat acut e s evere el evat i ons i n bl ood pres s ure.
(iii) Prazosin, terazosin, or doxazosin al s o can be gi ven t o produce s us t ai ned α-adrenergi c bl ockade.
(b) β-Adrenergic blocking agents s houl d not be us ed al one, becaus e unoppos ed α-adrenergi c s t i mul at i on may l ead t o exacerbat i on of t he hypert ens i on. β-Bl ockers are s omet i mes us eful i n conjunct i on wi t h α-bl ockers .
(2) Metyrosine, whi ch i s an i nhi bi t or of t yros i ne hydroxyl as e, bl ocks t he format i on of norepi nephri ne and epi nephri ne and i s an al t ernat i ve agent for t he rel i ef of t he s ympt oms of pheochromocyt oma. It may be us ed when pat i ent s are i nt ol erant of t he adrenergi c-bl ocki ng agent s .
o
o
b. Surgery. Surgical removal of the pheochromocytoma is the treatment of choice. Careful expl orat i on of t he adrenal gl ands and t he peri aort i c s ympat het i c chai n s houl d be performed.
(1) Complications t hat frequent l y occur duri ng and aft er s urgery are ext reme s wi ngs i n bl ood pres s ure, cardi ac arrhyt hmi as , and
Pa g e 2 1 1 7
ABC Ambe r CHM Conve rte r Tria l ve rsion, http://w w w .proce sste x t.com/a bcchm.html
s hock. Thes e are caus ed by t he s udden removal P.431
of t he s ource of exces s cat echol ami ne product i on and by t he l ow bl ood vol ume t hat res ul t s from l ong-t erm cons t ri ct i on of t he vas cul ar compart ment .
(2) To prevent vas cul ar i ns t abi l i t y duri ng removal of a pheochromocyt oma, pat i ent s are t reat ed wi t h α-bl ockers t o mai nt ai n normal bl ood pres s ure for at l eas t 1 week before s urgery. β-Bl ockers may be added for a few days before s urgery, es peci al l y i f t achycardi a or anot her arrhyt hmi a i s pres ent .
VI. Female Reproductive Disorders Endocri ne di s orders t hat affect t he femal e reproduct i ve s ys t em us ual l y caus e mens t rual abnormal i t i es and i ncl ude t hos e di s orders i n whi ch menarche does not occur (primary amenorrhea) and t hos e di s orders t hat caus e ces s at i on of mens t rual peri ods aft er menarche (secondary amenorrhea). Androgen-exces s s yndromes are a common caus e of reproduct i ve abnormal i t i es t hat al s o are cons i dered i n t hi s s ect i on.
A. Primary amenorrhea (Table 9-19)
1. Gonadal dysgenesis (T urner' s syndrome). Thi s condi t i on occurs i n 1 i n 2500 l i ve femal e bi rt hs . o
o
a. Etiology and pathophysiology. Gonadal dys genes i s i s caus ed by a chromosomal abnormality t hat i s not fami l i al and i s not rel at ed
Pa g e 2 1 1 8
ABC Ambe r CHM Conve rte r Tria l ve rsion, http://w w w .proce sste x t.com/a bcchm.html
t o t he mot her's age. Pat i ent s have a chromat i n-negat i ve buccal s mear and a 45,X karyot ype. o
o
b. Clinical features
(1) Ovaries fail to develop; onl y bi l at eral s t reaks of connect i ve t i s s ue are pres ent , wi t hout germ cel l s . Es t rogen defi ci ency, caus ed by t he abs ence of ovari an t i s s ue, res ul t s i n sexual infantilism, wi t h abs ence of breas t devel opment and ot her s econdary s exual charact eri s t i cs , and i ncreas ed l evel s of LH and FSH.
(2) Somatic abnormalities are as s oci at ed wi t h gonadal dys genes i s .
(a) Mos t pat i ent s are s hort , bet ween 48 and 58 i nches i n hei ght .
(b) Ot her feat ures , pres ent i n varyi ng numbers of pat i ent s , i ncl ude a s hort , webbed neck, epi cant hal fol ds , l ow-s et ears , a s hi el d-l i ke ches t wi t h wi del y s paced ni ppl es , cubi t us val gus (wi de carryi ng angl e), and renal and cardi ac abnormal i t i es .
o
o
c. T herapy
Pa g e 2 1 1 9
ABC Ambe r CHM Conve rte r Tria l ve rsion, http://w w w .proce sste x t.com/a bcchm.html
(1) Estrogen therapy i nduces t he devel opment of s econdary s exual charact eri s t i cs . If es t rogen i s gi ven cycl i cal l y wi t h proges t erone, regul ar mens t rual bl eedi ng occurs , but fert i l i t y i s not pos s i bl e becaus e of t he abs ence of ovari es .
(2) GH therapy, i f s t art ed earl y enough, may add 2–4 i nches t o t he adul t hei ght .
(3) Removal of streak gonads may be neces s ary. Gonadal dys genes i s may occur i n pat i ent s wi t h s ex chromos ome mos ai ci s m i n whi ch one or more cel l l i nes bear a Y chromos ome. The frequency of gonadobl as t oma and ot her gonadal t umors i s i ncreas ed i n pat i ent s wi t h t hes e gonads , and t hei r prophyl act i c removal i s recommended.
2. T esticular feminization syndrome. Indi vi dual s wi t h t hi s s yndrome are genet i c mal es wi t h a 46,XY karyot ype, but t hey have normal femal e ext ernal geni t al i a and are rai s ed as gi rl s . o
o
a. Pathogenesis. The bas i c defect i s res i s t ance of t arget t i s s ues t o t he act i on of androgens . The fet al t es t es produce t es t os t erone, but becaus e t he wol ffi an duct s and geni t al t i s s ues cannot res pond t o t es t os t erone, femal e di fferent i at i on of t he ext ernal geni t al i a t akes pl ace. The fet al P.432
Pa g e 2 1 2 0
ABC Ambe r CHM Conve rte r Tria l ve rsion, http://w w w .proce sste x t.com/a bcchm.html
t es t es al s o produce mül l eri an duct –i nhi bi t i ng fact or, whi ch has i t s normal effect i n i nhi bi t i ng t he mül l eri an anl age, and s o t he fal l opi an t ubes , ut erus , and upper vagi na do not devel op.
TABLE 9-19 Causes of Primary Amenorrhea Ext ra Go go na na dal dal ca ca us us es es Go Hy na po dal pi t dys ui t ge ari nes s m is
Hy
(Tu po rne go r's na s yn dot dro rop me i c )
hyp
Tes og t i c on ul a adi r
sm
Pa g e 2 1 2 1
ABC Ambe r CHM Conve rte r Tria l ve rsion, http://w w w .proce sste x t.com/a bcchm.html
fe
Del
mi aye ni z d at i me on nar s yn che dro
C
me on Re ge s i s ni t t an al t
adr
ova en ry
al
s yn hyp dro erp me l as ia Ab nor ma liti es of t he ut e rus or vag i na o
o
b. Clinical features. The res ul t i s a phenot ypi c woman wi t h a vagi na t hat ends i n a bl i nd pouch, hypopl as t i c mal e duct s i ns t ead of t he fal l opi an
Pa g e 2 1 2 2
ABC Ambe r CHM Conve rte r Tria l ve rsion, http://w w w .proce sste x t.com/a bcchm.html
t ubes and ut erus , and t es t es l ocat ed i n t he abdomen, i ngui nal canal , or l abi a majora. Endogenous es t rogen s t i mul at es normal breas t devel opment at pubert y. The condi t i on i s s us pect ed when menarche fai l s t o occur or when a t es t i s i s fel t as an abdomi nal mas s , whi ch i s expl ored. o
o
c. T herapy. The t es t es are prone t o mal i gnant degenerat i on and s houl d be removed. Es t rogen t reat ment i s t hen gi ven t o mai nt ai n s econdary s exual charact eri s t i cs .
3. Resistant ovary syndrome. Inabi l i t y of t he ovari es t o res pond t o normal or i ncreas ed s t i mul at i on by gonadot ropi ns may be a res ul t of aut oi mmune des t ruct i on of t he ovari es or ot her condi t i ons .
4. Hypogonadotropic hypogonadism o
o
a. Panhypopituitarism due t o des t ruct i ve l es i ons of t he hypot hal ami c–pi t ui t ary area (s ee I A 2 a) caus es pri mary or s econdary amenorrhea, dependi ng on whet her t he probl em i s prepubert al or pos t pubert al i n ons et .
o
o
b. Isolated gonadotropin deficiency i s mos t oft en caus ed by defect i ve hypot hal ami c product i on of GnRH, us ual l y of unknown et i ol ogy. In Kallmann' s syndrome, t hi s defect i s as s oci at ed wi t h anos mi a.
5. Delayed menarche. Thi s di agnos i s s houl d be
Pa g e 2 1 2 3
ABC Ambe r CHM Conve rte r Tria l ve rsion, http://w w w .proce sste x t.com/a bcchm.html
cons i dered when mens t rual peri ods have not begun by 16 years of age. o
o
a. A di agnos i s of del ayed menarche, as oppos ed t o t hat of pri mary amenorrhea, can onl y be made i n ret ros pect , aft er s pont aneous mens t rual peri ods have begun. A fami l y hi s t ory of l at e pubert al devel opment s ugges t s t hat s pont aneous menarche may yet be expect ed.
o
o
b. If s evere ps ychol ogi cal s t res s i s caus ed by t he abs ence of s exual devel opment , i t may be neces s ary t o gi ve one or more 6-mont h cours es of es t rogen t herapy, wi t h l ong t reat ment -free peri ods t o obs erve whet her s pont aneous pubert y wi l l occur.
B. Secondary amenorrhea (Table 9-20)
1. Hypothalamic (al s o cal l ed “ psychogenic,― “ functional,― and “ idiopathic―) amenorrhea is t he most common form of nonphysiologic secondary amenorrhea. Obvi ous ps ychol ogi cal s t res s may or may not be pres ent . LH and FSH l evel s are l ow i n s ome cas es and normal i n ot hers . If t he hypot hal ami c-rel eas i ng hormone GnRH i s i nfus ed i n a phys i ol ogi c fas hi on (pul s e dos es every 90–120 mi nut es ), al l abnormal i t i es may be correct ed—ovari an fol l i cl es mat ure, ovul at i on t akes pl ace, a corpus l ut eum devel ops and funct i ons , and pregnancy may occur. Thi s s upport s t he cl i ni cal i mpres s i on t hat mos t cas es of funct i onal or i di opat hi c amenorrhea are caus ed by abnormal hypot hal ami c GnRH product i on.
Pa g e 2 1 2 4
ABC Ambe r CHM Conve rte r Tria l ve rsion, http://w w w .proce sste x t.com/a bcchm.html
TABLE 9-20 Causes of Secondary Amenorrhea Pre Ov gn ari anc an y
cau
Me s es no Pri pa ma us e ry Ut e ova ri n ri a e
n
cau fai l s es ure Int (†rau œp t eri re ne ma s yn t ur ech e i ae me (As no her pa ma us e n's †s yn •) dro Oo me ph )
ore
Hy ct o s t e my rec Ra
Pa g e 2 1 2 5
ABC Ambe r CHM Conve rte r Tria l ve rsion, http://w w w .proce sste x t.com/a bcchm.html
t o di a my t i o Hy n pot t he hal rap am y, i câ che €“p mo i t ui t he t ar rap y
y
cau Es t s es rog Hy en po exc pi t es s ui t Ov ari ari s m an Hy t u pot mo hal rs am Pro ic
l ac
(†t i n œp exc s yc es s ho Pi t ge ui t ni c ary †t u •) mo am rs en An orr dro
Pa g e 2 1 2 6
ABC Ambe r CHM Conve rte r Tria l ve rsion, http://w w w .proce sste x t.com/a bcchm.html
he ge a
n
Mal exc nut es s ri t i Pol on, ycy chr s t i oni c c
ova
i l l n ry es s s yn Exe dro rci s me e
Ov
Di s erp con rod t i n uct uat i on i on of of
adr
ora en l
al
con an t ra dro cep ge tiv n es Ov ari an tu mo rs P.433
Pa g e 2 1 2 7
ABC Ambe r CHM Conve rte r Tria l ve rsion, http://w w w .proce sste x t.com/a bcchm.html
2. Malnutrition may pl ay a rol e. Menarche s eems t o occur when a cri t i cal body wei ght i s reached, and mens t ruat i on oft en ceas es when t he wei ght of a woman whos e mens t rual cycl e was previ ous l y normal fal l s bel ow t hi s cri t i cal wei ght , whet her becaus e of food depri vat i on, chroni c i l l nes s , exces s i ve di et i ng, or anorexi a nervos a.
3. Exercise may be a fact or. Amenorrhea i s pres ent i n up t o 50% of femal e bal l et dancers , runners , and at hl et es . Exerci s e-rel at ed wei ght l os s i s at l eas t part i al l y res pons i bl e; t he ri s k of amenorrhea i s much hi gher i n women who have l os t more t han 10%–15% of t hei r body wei ght . Level s of LH, FSH, and es t rogen t end t o be l ow, s ugges t i ng a hypot hal ami c abnormal i t y.
4. “ Post-pill amenorrhea― refers t o a del ay of more t han 6 mont hs i n t he ret urn of mens es aft er t he di s cont i nuat i on of oral cont racept i ve us e. It occurs i n fewer t han 1% of oral cont racept i ve us ers . Ot her caus es of amenorrhea mus t be excl uded before cont racept i ve us e i s det ermi ned t o be t he caus e.
5. Primary ovarian failure (“ premature menopause―) i s s i mi l ar t o normal menopaus e; t hat i s , ovari an funct i on decl i nes , es t rogen l evel s decreas e, and gonadot ropi n l evel s i ncreas e. However, pri mary ovari an fai l ure occurs before 40 years of age. Aut oant i bodi es agai ns t ovari an ant i gens have been found i n s ome cas es .
Pa g e 2 1 2 8
ABC Ambe r CHM Conve rte r Tria l ve rsion, http://w w w .proce sste x t.com/a bcchm.html
6. Ovarian tumors (e.g., granul os a–t heca cel l t umors ) may i nhi bi t normal mens t rual cycl i ng by produci ng exces s i ve quant i t i es of es t rogen.
7. Prolactin excess i s a common caus e of s econdary amenorrhea (s ee I A 3 b).
8. Menopause o
o
a. Manifestations
(1) Menopaus e, or t ot al ces s at i on of mens t rual peri ods , occurs i n al l women when t he ovari es are depl et ed of fol l i cl es and ovul at ory funct i on and es t rogen product i on ceas e. The average age of menopaus e i n t he Uni t ed St at es i s 51 years .
(2) As fol l i cul ar s ecret i on of es t radi ol and i nhi bi n (a s uppres s or of gonadot ropi ns ) fal l s at menopaus e, FSH and LH l evel s ri s e. Es t radi ol l evel s remai n l ow, al t hough s ome es t radi ol i s formed by peri pheral convers i on of andros t enedi one produced by t he adrenal s and ovari es .
(3) The mos t promi nent s ympt oms of t he menopaus e i s t he hot flash, a s ens at i on of warmt h t hat may l as t up t o 5 mi nut es , s omet i mes fol l owed by s weat i ng and a col d s ens at i on. Headache, pal pi t at i ons , di zzi nes s ,
Pa g e 2 1 2 9
ABC Ambe r CHM Conve rte r Tria l ve rsion, http://w w w .proce sste x t.com/a bcchm.html
and weaknes s may accompany t he hot fl as h. Thes e epi s odes occur i rregul arl y and may i nt erfere wi t h s l eep. Hot fl as hes us ual l y di s appear wi t hi n a few years of menopaus e.
(a) The hot fl as h s eems t o be rel at ed t o a fal l i n es t rogen l evel s . Feedback s t i mul at i on of GnRH-produci ng cel l s i n t he hypot hal amus may affect nearby t hermoregul at ory cent ers .
(b) Es t rogen t herapy i s very effect i ve i n decreas i ng t he frequency and s everi t y of hot fl as hes .
(4) Ot her s ympt oms as s oci at ed wi t h t he menopaus e i ncl ude geni t ouri nary at rophy, vagi nal drynes s , uri nary s ympt oms , i ns omni a, and depres s i on.
o
o
b. Postmenopausal hormone replacement therapy. For many years i t was wi del y accept ed t hat repl acement of es t rogen aft er menopaus e had benefi t s t hat great l y out wei ghed i t s drawbacks . Recent s t udi es have changed t hi s percept i on, and t he i ndi cat i ons for pos t menopaus al hormone repl acement t herapy are bei ng recons i dered.
(1) Effects of estrogen on cardiovascular disease
Pa g e 2 1 3 0
ABC Ambe r CHM Conve rte r Tria l ve rsion, http://w w w .proce sste x t.com/a bcchm.html
(a) Many obs ervat i onal s t udi es have s hown t hat women who t ake es t rogen aft er menopaus e have l ower rat es of cardi ovas cul ar di s eas e. But l arge randomi zed cont rol l ed t ri al s have recent l y s hown t hat es t rogen does not prevent cardi ovas cul ar di s eas e, and may even, at t he begi nni ng of t reat ment , have s l i ght l y harmful effect s i n women who al ready have cardi ovas cul ar di s eas e.
(b) It appears t hat t he earl i er s t udi es di d not adequat el y account for s oci oeconomi c and educat i onal fact ors : t he women who t ook es t rogen al s o engaged i n ot her heal t h-promot i ng behavi ors s uch as s moki ng ces s at i on, exerci s e, bl ood pres s ure cont rol , and heal t hi er eat i ng. Thus es t rogen t herapy was as s oci at ed wi t h, but not t he caus e of, cardi ovas cul ar benefi t .
(2) Benefits of estrogen therapy In addi t i on t o rel i evi ng t he s ympt oms of hot fl as hes and geni t ouri nary at rophy, es t rogen repl acement i ncreas es bone mi neral dens i t y and reduces t he ri s k of os t eoporot i c fract ures . It al s o reduces t he ri s k of col orect al cancer. A very l arge s t udy, t he W omen's Heal t h Ini t i at i ve, found 5 fewer hi p fract ures and 6 fewer cas es of col orect al cancer per 10,000 women per year among t hos e t aki ng es t rogen.
Pa g e 2 1 3 1
ABC Ambe r CHM Conve rte r Tria l ve rsion, http://w w w .proce sste x t.com/a bcchm.html
(3) Risks of estrogen therapy Es t rogen i s known t o i ncreas e t he ri s k of breas t cancer, and t o affect t he cl ot t i ng mechani s m, t hereby i ncreas i ng t he ri s k of t hromboembol i c event s . The W omen's Heal t h Ini t i at i ve found 8 addi t i onal cas es of breas t cancer, 10 addi t i onal cas es of deep vei n t hrombos i s , and 8 addi t i onal cas es of pul monary embol i s m per 10,000 women per year i n t hos e t aki ng es t rogen. In ol der pat i ent s , es t rogen t herapy s eems t o i ncreas e t he ri s k for cardi ovas cul ar event s .
(4) Indications for hormone replacement therapy
(a) Becaus e t he pos s i bl e l ong-t erm harmful effect s of es t rogen repl acement now appear t o out wei gh i t s benefi t s , l ong-t erm hormone repl acement t herapy i s no l onger recommended i n mos t cas es .
(b) Es t rogen repl acement t herapy i s i ndi cat ed for s hort -t erm t reat ment of hot fl as hes i n t he peri menopaus al peri od, but t he s mal l es t dos e t hat rel i eves t hes e s ympt oms s houl d be us ed.
(5) Estrogen administration
(a) Es t rogen t herapy aft er menopaus e i ncreas es t he ri s k of ut eri ne cancer; t hi s
Pa g e 2 1 3 2
ABC Ambe r CHM Conve rte r Tria l ve rsion, http://w w w .proce sste x t.com/a bcchm.html
ri s k i s mi ni mi zed i f proges t erone i s gi ven al ong wi t h t he es t rogen.
(b) Es t rogen–proges t erone combi nat i ons can be gi ven cycl i cal l y or cont i nuous l y. A t ypi cal cycl i c regi men woul d be conjugat ed es t rogens 0.625 mg dai l y from day 1 t o day 25 each mont h, wi t h 5 or 10 mg medroxyproges t erone acet at e from day 16 t o day 25. Bl eedi ng i s expect ed a few days aft er t he hormones are s t opped.
(c) Combi ned t herapy al s o can be gi ven dai l y. A t ypi cal regi men woul d be conjugat ed es t rogens 0.625 mg and medroxyproges t erone acet at e 2.5 mg once dai l y. Thi s regi men has t he advant age of caus i ng amenorrhea aft er 6 mont hs or more i n a majori t y of women, but s ome women may cont i nue t o have unpredi ct abl e s pot t i ng.
(d) Ot her es t rogen preparat i ons i ncl ude oral es t radi ol and et hi nyl es t radi ol , and t rans dermal preparat i ons of es t radi ol . Many ot her proges t at i onal agent s are al s o avai l abl e.
(e) In women who have had a hys t erect omy, es t rogen can be gi ven dai l y wi t hout proges t erone, becaus e t here i s no ri s k of endomet ri al cancer.
Pa g e 2 1 3 3
ABC Ambe r CHM Conve rte r Tria l ve rsion, http://w w w .proce sste x t.com/a bcchm.html
C. Androgen excess syndromes
1. Polycystic ovary syndrome, a di s order of unknown et i ol ogy, i s charact eri zed by a chroni c l ack of ovul at i on, as s oci at ed wi t h s ympt oms of androgen exces s and oft en wi t h obes i t y. It i s pres ent i n 5%–10% of reproduct i ve-age women and i s one of t he mos t common caus es of i nfert i l i t y. P.434
TABLE 9-21 Diagnostic Criteria for Polycystic Ovary Syndrome Na tio nal Ins tit ute s of He alt h crit eri a (1 99 0)
Pa g e 2 1 3 4
ABC Ambe r CHM Conve rte r Tria l ve rsion, http://w w w .proce sste x t.com/a bcchm.html
Me ns t rua l i rre gul ari t y du e to an ovu l at i on or ol i goovu l at i on Evi de nce of cl i n i cal or bi o che mi c al hyp era ndr
Pa g e 2 1 3 5
ABC Ambe r CHM Conve rte r Tria l ve rsion, http://w w w .proce sste x t.com/a bcchm.html
og eni sm Cl i ni c al : hi r s ut ism , acn e, or ma l epat t er n bal di n g Bi och em i cal : hi g h s er um an dro ge n con
Pa g e 2 1 3 6
ABC Ambe r CHM Conve rte r Tria l ve rsion, http://w w w .proce sste x t.com/a bcchm.html
cen t ra tio ns Exc l us i on of ot h er cau s es of hyp era ndr og eni sm an d me ns t rua l i rre gul ari t y, s uc h as con ge ni t
Pa g e 2 1 3 7
ABC Ambe r CHM Conve rte r Tria l ve rsion, http://w w w .proce sste x t.com/a bcchm.html
al adr en al hyp erp l as i a, an dro ge n-s ecr et i ng tu mo rs , an d hyp erp rol act i ne mi a Ro tte rd am Cri ter ia by
Pa g e 2 1 3 8
ABC Ambe r CHM Conve rte r Tria l ve rsion, http://w w w .proce sste x t.com/a bcchm.html
the Eu ro pe an So cie ty of Hu ma n Re pr od uct ion an d Em bry olo gy /A me ric an So cie ty of Re pr od uct
Pa g e 2 1 3 9
ABC Ambe r CHM Conve rte r Tria l ve rsion, http://w w w .proce sste x t.com/a bcchm.html
ive Me dici ne (2 20 3) Tw o of t hr ee of t he fol l owi ng fi n di n gs are req ui r ed for di a gn os i s: Me ns t rua l i rre gul
Pa g e 2 1 4 0
ABC Ambe r CHM Conve rte r Tria l ve rsion, http://w w w .proce sste x t.com/a bcchm.html
ari t y du e to an ovu l at i on or ol i goovu l at i on Evi de nce of cl i n i cal or bi o che mi c al hyp era ndr og eni sm Cl i ni c al :
Pa g e 2 1 4 1
ABC Ambe r CHM Conve rte r Tria l ve rsion, http://w w w .proce sste x t.com/a bcchm.html
hi r s ut ism , acn e, or ma l epat t er n bal di n g Bi och em i cal : hi g h s er um an dro ge n con cen t ra tio ns Pol ycy
Pa g e 2 1 4 2
ABC Ambe r CHM Conve rte r Tria l ve rsion, http://w w w .proce sste x t.com/a bcchm.html
sti c ova ri e s (by ul t r as o un d) o
o
a. Pathophysiology (Online Figure 9-11). In t he cycl e s hown i n Onl i ne Fi gure 9-11, t he i ni t i at i ng event i s uncert ai n, and i t probabl y i s not t he s ame i n al l cas es . Propos ed pri mary defect s i ncl ude i ns ul i n res i s t ance, pri mary ovari an di s eas e, abnormal s ecret i on of LH by t he hypot hal ami c–pi t ui t ary axi s , and adrenal di s eas e.
Pa g e 2 1 4 3
ABC Ambe r CHM Conve rte r Tria l ve rsion, http://w w w .proce sste x t.com/a bcchm.html
ONLINE FIGURE 9-11 Propos ed pat hogenes i s of t he pol ycys t i c ovary s yndrome. The oval -s haped bl ocks repres ent cont ri but i ng fact ors ; t he rect angul ar bl ocks , key el ement s ; and t he unout l i ned words , i nt ermedi at e s t eps . FSH, fol l i cl e-s t i mul at i ng hormone; LH, l ut ei ni z i ng hormone.
(1) The ovary produces exces s androgeni c s t eroi ds , es peci al l y androstenedione. The andros t enedi one i s convert ed t o es t rone, an es t rogen, i n fat and ot her peri pheral t i s s ues . The i ncreas ed ci rcul at i ng and i nt raovari an l evel s of andros t enedi one and ot her androgens prevent t he mat urat i on of graafi an fol l i cl es , caus i ng anovulation.
(2) The i ncreas ed ci rcul at i ng l evel of es t rone has a pos i t i ve feedback effect on pi t ui t ary product i on of LH and a negat i ve effect on product i on of FSH. The increased LH level caus es hyperpl as i a of ovari an t hecal cel l s and s t roma and i ncreas ed androgen product i on. The decreased FSH level cont ri but es t o t he l ack of fol l i cl e mat urat i on.
(3) Obesity may enhance t he el evat ed l evel s of s ex s t eroi ds by decreas i ng s ex hormone–bi ndi ng gl obul i n (t hereby i ncreas i ng t he l evel of free t es t os t erone) and by i ncreas i ng t he peri pheral convers i on of andros t enedi one t o es t rone.
(4) As a res ul t of t he arrested follicle development, t he ovari es are enl arged, wi t h
Pa g e 2 1 4 4
ABC Ambe r CHM Conve rte r Tria l ve rsion, http://w w w .proce sste x t.com/a bcchm.html
t hi ckened caps ul es and many s mal l fol l i cul ar cys t s . St romal and t hecal hyperpl as i a are s een on mi cros copi c exami nat i on.
(5) Insulin resistance i s pres ent i n many women wi t h t he pol ycys t i c ovary s yndrome; obes i t y account s for s ome but not al l of t he i ns ul i n res i s t ance. Ins ul i n l evel s are el evat ed becaus e of t he t i s s ue res i s t ance t o i t s act i on. That t he hyperi ns ul i nemi a may be a pri mary caus e (or at l eas t a cont ri but i ng caus e) of t he i ncreas ed ovari an product i on of androgens i s s ugges t ed by t he cl i ni cal i mprovement t hat fol l ows t reat ment wi t h drugs t hat i ncreas e i ns ul i n s ens i t i vi t y and t hus ret urn s erum i ns ul i n l evel s t o normal .
(6) Abnormal i t i es of adrenal androgen production may be pres ent ; di s appearance of t he pol ycys t i c ovary s yndrome has fol l owed removal of an androgen-s ecret i ng adrenal adenoma i n s ome cas es .
o
o
b. Clinical features (T able 9-21)
(1) Infertility and menstrual abnormalities are t he res ul t of chroni c anovul at i on. Mos t pat i ent s have amenorrhea or ol i gomenorrhea. The prol onged, noncycl i c, unoppos ed es t rogeni c s t i mul at i on of t he endomet ri um may caus e funct i onal bl eedi ng and an i ncreas ed ri s k of endomet ri al carci noma.
Pa g e 2 1 4 5
ABC Ambe r CHM Conve rte r Tria l ve rsion, http://w w w .proce sste x t.com/a bcchm.html
(2) Androgen excess caus es oi l i nes s of t he s ki n, acne, and hi rs ut i s m i n mos t women wi t h t hi s s yndrome. Si gns of t rue vi ri l i zat i on (e.g., deepeni ng of t he voi ce, enl argement of t he cl i t ori s ) are rare.
(3) Obesity i s pres ent i n approxi mat el y 40% of pat i ent s .
o
o
c. Laboratory findings
(1) An increased LH-to-FSH ratio (≥2) i s a us eful di agnos t i c fi ndi ng. The LH l evel i s us ual l y el evat ed, and t he FSH l evel i s at t he l ow end of t he normal range (or t he l ow-normal range).
(2) Serum testosterone and androstenedione levels are us ual l y el evat ed. Increas ed l evel s of t he androgens of predomi nant l y adrenal ori gi n (i .e., dehydroepi andros t erone, dehydroepi andros t erone s ul fat e) are found l es s oft en.
(3) Serum estrone levels are us ual l y hi gh, and es t radi ol l evel s are normal .
(4) Prolactin l evel can be hi gh.
Pa g e 2 1 4 6
ABC Ambe r CHM Conve rte r Tria l ve rsion, http://w w w .proce sste x t.com/a bcchm.html
(5) In pat i ent s wi t h abnormal fasting glucose, an oral gl ucos e t ol erance t es t can be cons i dered.
o
o
d. T herapy. The goals of treatment are the relief of symptoms of androgen excess, the induction of ovulation and fertility, and the prevention of endometrial hyperplasia due t o exces s noncycl i c es t rogen s t i mul at i on.
(1) Androgen excess
(a) Metformin and thiazolidinediones, drugs t hat i ncreas e t he s ens i t i vi t y of t i s s ues t o i ns ul i n, have l owered t es t os t erone l evel s and i mproved t he s ympt oms of hyperandrogeni s m and mens t rual dys funct i on i n i ndi vi dual s wi t h pol ycys t i c ovary s yndrome. The effi cacy of metformin i n t hi s s yndrome makes i t a reas onabl e choi ce for i ni t i al t herapy, al t hough i t does not caus e i mprovement i n al l cas es .
(b) Spironolactone, an al dos t erone ant agoni s t t hat i s us ed pri mari l y for i t s di uret i c and ant i hypert ens i ve propert i es , has addi t i onal act i ons t hat make i t us eful i n t he t reat ment of hi rs ut i s m. Thi s agent decreas es ovari an and adrenal s ynt hes i s of androgens and i nhi bi t s androgen bi ndi ng t o recept ors i n hai r
Pa g e 2 1 4 7
ABC Ambe r CHM Conve rte r Tria l ve rsion, http://w w w .proce sste x t.com/a bcchm.html
fol l i cl es and ot her t arget t i s s ues . A dos e of 100 mg once or t wi ce dai l y i s oft en effect i ve. P.435
(c) Estrogen–progestin combinations may decreas e androgen l evel s by feedback i nhi bi t i on of pi t ui t ary LH product i on and by s t i mul at i on of hepat i c s ynt hes i s of s ex hormone–bi ndi ng gl obul i n, whi ch decreas es t he unbound fract i on of t es t os t erone.
(d) Glucocorticoids (e.g., predni s one 5.0–7.5 mg dai l y) may decreas e adrenal androgen product i on by s uppres s i ng ACTH. Thes e agent s may al s o l ower ovari an androgen s ecret i on, al t hough t he mechani s m i s unknown.
(e) The effect s of medi cal t herapy i n di mi ni s hi ng t he growt h of unwant ed faci al and body hai r are s el dom dramat i c and us ual l y t ake pl ace over a peri od of 3–6 mont hs . Mechanical methods of hair removal are us ual l y needed as wel l (e.g., s havi ng, el ect rol ys i s , l as er t reat ment , bl eachi ng, chemi cal depi l at ori es , and wax t reat ment s ). Vaniqa cream (efl orni t hi ne hydrochl ori de) i nhi bi t s an enz yme i n
Pa g e 2 1 4 8
ABC Ambe r CHM Conve rte r Tria l ve rsion, http://w w w .proce sste x t.com/a bcchm.html
s ki n, orni t hi ne decarboxyl as e, and may s l ow t he rat e of hai r growt h.
(2) Infertility. If met formi n does not res t ore normal ovul at ory mens t rual cycl es , ot her drugs may be us ed t o t reat i nfert i l i t y.
(a) Clomiphene citrate bl ocks t he bi ndi ng of es t rogen t o recept ors i n t arget t i s s ues . By bl ocki ng t he negat i ve feedback effect s of es t rogen on t he hypot hal amus and pi t ui t ary gl and, t hi s drug s t i mul at es LH and FSH product i on.
(i) If gi ven on t he fi ft h day t hrough t he ni nt h day aft er a mens t rual peri od i nduced by proges t erone, cl omi phene ci t rat e oft en s t i mul at es fol l i cl e mat urat i on and ovul at i on.
(ii) Ovul at i on can be i nduced wi t h cl omi phene ci t rat e i n approxi mat el y 80% of pat i ent s .
(b) Human menopausal gonadotropin has bot h FSH and LH bi oact i vi t y.
(i) It i s i nject ed dai l y unt i l i ncreas i ng s erum es t rogen l evel s and ul t ras onography of t he ovary i ndi cat e t hat fol l i cl e mat urat i on has
Pa g e 2 1 4 9
ABC Ambe r CHM Conve rte r Tria l ve rsion, http://w w w .proce sste x t.com/a bcchm.html
occurred.
(ii) Then human chori oni c gonadot ropi n (hCG), whi ch has pri mari l y LH act i vi t y, i s i nject ed t o i nduce ovul at i on. Becaus e t he ri s k of ovari an hypers t i mul at i on and of mul t i pl e ges t at i on i s hi gh, t hi s t herapy s houl d be res erved for res i s t ant cas es of i nfert i l i t y.
(c) GnRH, when gi ven i nt ravenous l y or s ubcut aneous l y i n pul s e dos es every 90–120 mi nut es , may i nduce ovul at i on wi t hout caus i ng ovari an hypers t i mul at i on.
(3) Chronic anovulation and abnormal menstrual bleeding. Unoppos ed noncycl i c s t i mul at i on of t he endomet ri um by es t rogen may caus e funct i onal bl eedi ng and may i ncreas e t he ri s k of endomet ri al cancer. Pers i s t ent endomet ri al prol i ferat i on can be i nt errupt ed ei t her wi t h proges t i n t reat ment (e.g., 10 mg dai l y of medroxyproges t erone acet at e for 10 days every 1–3 mont hs ) or wi t h cycl i c es t rogen–proges t i n t herapy.
2. Androgen-producing ovarian tumors are rare. Arrhenobl as t oma, t he mos t common of t hes e t umors , makes up l es s t han 1% of s ol i d ovari an t umors ; ot hers are hi l ar cel l t umors , adrenal res t t umors , and granul os a cel l t umors .
Pa g e 2 1 5 0
ABC Ambe r CHM Conve rte r Tria l ve rsion, http://w w w .proce sste x t.com/a bcchm.html o
o
a. Tes t os t erone l evel s t end t o be hi gher t han t hos e i n t he pol ycys t i c ovary s yndrome, and vi ri l i zat i on occurs more frequent l y.
o
o
b. Androgen l evel s are not s uppres s ed by t reat ment wi t h gl ucocort i coi ds or es t rogen–proges t i n combi nat i ons , as t hey oft en are i n t he pol ycys t i c ovary s yndrome.
o
o
c. Di agnos i s depends on det ect i on of t he t umor by pel vi c exami nat i on (t he majori t y are pal pabl e) and on di agnos t i c i magi ng t echni ques .
3. Hyperthecosis of t he ovary i s probabl y a s evere form of t he pol ycys t i c ovary s yndrome, but t he androgen exces s i s more evi dent . o
o
a. Diagnosis depends on t he hi s t ol ogi c fi ndi ng of l ut ei ni zed t hecal and s t romal cel l s .
o
o
b. Medical therapy i s not effect i ve, and oophorect omy may be neces s ary.
4. Adrenal tumors, ei t her adenomas or carci nomas , may produce exces s androgens wi t h or wi t hout exces s cort i s ol . Hi gh l evel s of adrenal androgens (uri nary 17-ket os t eroi ds , s erum dehydroepi andros t erone) t hat cannot be s uppres s ed by dexamet has one s ugges t t hi s di agnos i s ; 24-hour uri nary 17-ket os t eroi d l evel s great er
Pa g e 2 1 5 1
ABC Ambe r CHM Conve rte r Tria l ve rsion, http://w w w .proce sste x t.com/a bcchm.html
t han 50–100 mg s t rongl y s ugges t adrenal carci noma.
5. Congenital adrenal hyperplasia i s di s cus s ed i n t he s ect i on on di s orders of t he adrenal gl and (s ee V E). P.436
6. Idiopathic hirsutism i s a poorl y unders t ood but common condi t i on i n whi ch hi rs ut i s m occurs i n t he abs ence of marked hormone abnormal i t i es or mens t rual dys funct i on. o
o
a. The caus e of i di opat hi c hi rs ut i s m i s not known. It may have a fami l i al occurrence, and i t i s more common i n women of Medi t erranean ances t ry.
o
o
b. Increas ed res pons e of hai r fol l i cl es t o normal l evel s of t es t os t erone i s s us pect ed.
VII. Male Reproductive Disorders and Gynecomastia A. Hypogonadism Hypogonadism i n men affect s t wo s eparat e funct i ons —t he product i on of s permat ozoa by t he s emi ni ferous t ubul es and t he s ecret i on of t es t os t erone by t he Leydi g cel l s . The s emi ni ferous t ubul e defect caus es i nfert i l i t y; t he t es t os t erone defi ci ency l eads t o i nadequat e devel opment and mai nt enance of s econdary s exual charact eri s t i cs .
1. Physical and developmental effects
Pa g e 2 1 5 2
ABC Ambe r CHM Conve rte r Tria l ve rsion, http://w w w .proce sste x t.com/a bcchm.html o
o
a. Before puberty, t es t i cul ar fai l ure prevent s normal s exual devel opment .
(1) The peni s and t es t es remai n s mal l , and s permat ozoa are abs ent .
(2) Faci al and body hai r are s pars e.
(3) The voi ce remai ns hi gh-pi t ched, and mus cl e mas s and s t rengt h are di mi ni s hed.
(4) Increas ed growt h of l ong bones (becaus e of del ayed epi phys eal cl os ure) produces t he “eunuchoi dal habi t us ,― i n whi ch t he arm s pan i s more t han 2 i nches great er t han t he hei ght , and t he fl oor-t o-pubi c s ymphys i s di s t ance i s more t han 2 i nches great er t han t he s ymphys i s -t o-crown di s t ance.
o
o
b. After puberty, l os s of l i bi do and s exual pot ency may be t he fi rs t s ympt oms of t es t i cul ar fai l ure. Part i al regres s i on of s econdary s ex charact eri s t i cs may occur gradual l y, wi t h s l owi ng of faci al and body hai r growt h and decreas ed mus cl e mas s and bone dens i t y.
2. Clinical syndromes o
Pa g e 2 1 5 3
ABC Ambe r CHM Conve rte r Tria l ve rsion, http://w w w .proce sste x t.com/a bcchm.html o
a. Hypogonadotropic syndromes (Tabl e 9-22). The caus es of hypogonadi s m are di vi ded i nt o disorders of the hypothalamic–pituitary axis (hypogonadotropic hypogonadism) and disorders that originate with testicular damage, wi t h cons equent feedback s t i mul at i on of LH and FSH (hypergonadotropic hypogonadism).
(1) Hypogonadotropic (secondary) hypogonadism i s charact eri zed by defi ci ency of LH and FSH, wi t h res ul t i ng t es t os t erone defi ci ency and eunuchoi di s m. Kallmann' s syndrome i s a form of hypogonadot ropi c hypogonadi s m t hat i s as s oci at ed wi t h mi dl i ne defect s s uch as agenes i s of t he ol fact ory l obes , anos mi a, and cl eft pal at e. It i s more common i n men t han i n women. The bas i c hormonal defect i s i n t he hypot hal amus , rat her t han i n t he pi t ui t ary gl and; t hi s has been demons t rat ed by LH and FSH res pons e t o GnRH admi ni s t rat i on.
TABLE 9-22 Causes of Hypogonadism in Men
Pa g e 2 1 5 4
ABC Ambe r CHM Conve rte r Tria l ve rsion, http://w w w .proce sste x t.com/a bcchm.html
Hy Hy pe po rg go on na ad dot otr ro opi pic c sy sy nd nd ro ro me me s s Hy Kl i po nef pi t el t ui t er' ari s s m s yn Hy dro po me go Tes na t i c dot ul a rop r ic
ag
eu en nuc es i hoi s di s Tes m
tic
Kal ul a lm r an i nj n's ury
Pa g e 2 1 5 5
ABC Ambe r CHM Conve rte r Tria l ve rsion, http://w w w .proce sste x t.com/a bcchm.html
s yn Mu dro mp me s Del orc aye hi t i d
s
pu Ot ber her t y i nf ect i on s (e. g., go nor rhe a) Tra um a Sur ger y Ra di a tio n t he rap y Ca nce r che
Pa g e 2 1 5 6
ABC Ambe r CHM Conve rte r Tria l ve rsion, http://w w w .proce sste x t.com/a bcchm.html
mo t he rap y Cry pt o rch i di sm My ot o ni c dys t ro phy *Exces s produc t i on of ACTH by t he pi t ui t a ry gl and is Cus hi n g's di s eas e. Cus hi n g's s yndro me i s a nons pe
Pa g e 2 1 5 7
ABC Ambe r CHM Conve rte r Tria l ve rsion, http://w w w .proce sste x t.com/a bcchm.html
ci fi c des i gn at i on t hat refers to i ncreas ed gl ucoc ort i coi d l evel s from any ori gi n. o
P.437
o
(2) Delayed puberty i s a ret ros pect i ve di agnos i s . Pubert y may occur s pont aneous l y up t o about 20 years of age; unt i l t hi s age, t rue hypogonadot ropi c hypogonadi s m cannot be di agnos ed wi t h cert ai nt y unl es s as s oci at ed abnormal i t i es s uch as anos mi a are pres ent . The di agnos i s of del ayed pubert y i s s ugges t ed by a fami l y hi s t ory of l at e mat urat i on.
(a) If del ayed pubert y i s s us pect ed, a cours e of t herapy wi t h l ow dos es of t es t os t erone can be i ni t i at ed t o i nduce pubert al changes ; t rue pubert y may be i nduced by t hi s t reat ment .
Pa g e 2 1 5 8
ABC Ambe r CHM Conve rte r Tria l ve rsion, http://w w w .proce sste x t.com/a bcchm.html
(b) Tes t os t erone s houl d be gi ven for no more t han 6 mont hs at a t i me, wi t h 6 mont hs bet ween cours es , t o avoi d caus i ng epi phys eal cl os ure and l i mi t at i on of ul t i mat e hei ght and t o al l ow recogni t i on of t he ons et of s pont aneous pubert y, i f i t s houl d occur.
o
o
b. Hypergonadotropic syndromes (primary hypogonadism)
(1) Klinefelter' s syndrome, i n whi ch t he pres ence of t wo or more X chromos omes caus es congeni t al t es t i cul ar damage, occurs i n approxi mat el y 1 i n 400 mal e bi rt hs .
(a) Approxi mat el y 80% of pat i ent s have a 47,XXY karyotype.
(b) The t es t es are s mal l (<2 cm i n l engt h), wi t h hyal i ni zat i on of t he s emi ni ferous t ubul es and azoospermia.
(c) Leydi g cel l funct i on i s vari abl e. Tes t os t erone l evel s are defi ci ent , and eunuchoidism i s pres ent i n many, but not al l , cas es .
(d) Gynecomastia i s pres ent , and LH
Pa g e 2 1 5 9
ABC Ambe r CHM Conve rte r Tria l ve rsion, http://w w w .proce sste x t.com/a bcchm.html
and FSH l evel s are el evat ed, even (for unknown reas ons ) i n pat i ent s wi t hout t es t os t erone defi ci ency.
(e) Mental deficiency i s an as s oci at ed fi ndi ng i n 25% of pat i ent s .
(f) The onl y avai l abl e t reat ment i s testosterone replacement i n t hos e pat i ent s who requi re i t .
(2) T esticular agenesis i s recogni zed by fai l ure of pubert al devel opment and abs ence of t es t es i n t he s crot um or i n t he i ngui nal canal s . Los s of t he t es t es occurs aft er 7–14 weeks of ges t at i on, becaus e abs ence of t es t i cul ar hormones before t hi s s t age woul d res ul t i n a femal e phenot ype.
(3) Mumps orchitis affect s mai nl y germi nal cel l s ; i f t he di s eas e i s bi l at eral , i nfert i l i t y may res ul t , al t hough t hi s i s uncommon. Tes t os t erone product i on i s us ual l y uni mpai red.
(4) Cryptorchidism, es peci al l y i f i t i s bi l at eral , may be as s oci at ed wi t h hypogonadi s m becaus e t he undes cended t es t es are damaged by t rauma or t ors i on.
(a) An as s oci at i on bet ween
Pa g e 2 1 6 0
ABC Ambe r CHM Conve rte r Tria l ve rsion, http://w w w .proce sste x t.com/a bcchm.html
hypogonadi s m and crypt orchi di s m may exi s t becaus e t he crypt orchi di s m i s s omet i mes a cons equence of an i nt ri ns i c abnormal i t y i n t he t es t es .
(b) Treat ment wi t h hCG or GnRH may i nduce t es t i cul ar des cent i n s ome cas es .
(5) Myotonic dystrophy i s a s yndrome cons i s t i ng of myot oni a, cat aract s , and t es t i cul ar at rophy.
o
o
3. T herapy
a. T estosterone deficiency. Al t hough oral androgeni c s t eroi ds are avai l abl e, t hey do not provi de ful l y vi ri l i zi ng bl ood l evel s of mal e hormones . Treat ment of mal e hypogonadi s m i s t he i nject i on of 200–400 mg of a l ong-act i ng t es t os t erone preparat i on (e.g., Del at es t ryl or DEPO-Tes t os t erone) every 2–4 weeks , or t he us e of s ki n pat ches (Tes t oderm, Androderm) or a gel (AndroGel , Tes t i m) t o provi de t rans dermal t es t os t erone abs orpt i on.
b. Infertility. Sperm product i on and fert i l i t y cannot be i nduced i n i ndi vi dual s wi t h pri mary t es t i cul ar i njury. In hypogonadot ropi c hypogonadi s m, s permat ogenes i s can s omet i mes be brought about by provi di ng t he t es t es wi t h adequat e gonadot ropi c s t i mul at i on.
Pa g e 2 1 6 1
ABC Ambe r CHM Conve rte r Tria l ve rsion, http://w w w .proce sste x t.com/a bcchm.html
(1) Thi s can be done ei t her by i nject i ons t hree t i mes per week of hCG (whi ch has LH act i vi t y) and human menopaus al gonadot ropi n (whi ch has FSH act i vi t y) or by admi ni s t rat i on vi a port abl e i nfus i on pump of pul s e dos es of GnRH every 90–120 mi nut es .
(2) Bot h of t hes e met hods are expens i ve and i mpract i cal for l ong-t erm us e, but t hey have been us ed s ucces s ful l y i n s ome hi ghl y mot i vat ed men for t he s everal mont hs t hat are neces s ary t o i nduce s permat ogenes i s .
B. Gynecomastia Gynecomastia i s enl argement of t he mal e breas t . In t rue gynecomas t i a, fi rm, s omet i mes t ender, gl andul ar t i s s ue i s pres ent . The di s orders t hat caus e gynecomas t i a are us ual l y as s oci at ed wi t h i ncreas ed l evel s of es t rogens , decreas ed l evel s of androgens , or bot h. P.438
1. Pubertal gynecomastia i s not uncommon. At 12–15 years of age, approxi mat el y t wo-t hi rds of normal boys have s ome degree of gynecomas t i a, us ual l y a s mal l , fi rm s ubareol ar nodul e t hat di s appears i n mos t cas es wi t hi n 1–2 years . o
o
a. In t he occas i onal boy wi t h pers i s t ent breas t
Pa g e 2 1 6 2
ABC Ambe r CHM Conve rte r Tria l ve rsion, http://w w w .proce sste x t.com/a bcchm.html
enl argement , medi cal t reat ment wi t h t he ant i es t rogen tamoxifen may be t ri ed. o
o
b. If t hi s i s i neffect i ve, however, reduction mammoplasty mus t be cons i dered i f ps ychol ogi cal s t res s i s s evere.
2. Hypogonadism, ei t her pri mary (hypergonadot ropi c) or s econdary (hypogonadot ropi c), may be as s oci at ed wi t h gynecomas t i a.
3. Re-feeding after a period of starvation oft en l eads t o t rans i ent gynecomas t i a, whi ch may l as t for s everal mont hs . Renewed s ecret i on of previ ous l y i nhi bi t ed gonadot ropi ns and s ex s t eroi ds and decreas ed hormone i nact i vat i on by t he s t arved l i ver may be cont ri but i ng fact ors .
4. Liver disease, es peci al l y al cohol i c ci rrhos i s , i s a common caus e of gynecomas t i a. o
o
a. Es t rogen l evel s are i ncreas ed becaus e of accel erat ed convers i on of androgeni c precurs ors by peri pheral t i s s ues .
o
o
b. In addi t i on, al cohol i nhi bi t s t he t es t i cul ar product i on of t es t os t erone and t he pi t ui t ary product i on of gonadot ropi ns and i ncreas es hepat i c met abol i s m of t es t os t erone.
Pa g e 2 1 6 3
ABC Ambe r CHM Conve rte r Tria l ve rsion, http://w w w .proce sste x t.com/a bcchm.html
5. Chronic renal failure i s as s oci at ed wi t h gynecomas t i a, es peci al l y aft er t he s t art of hemodi al ys i s . The re-feedi ng phenomenon may pl ay a rol e, as may an i ncreas e i n t he rat i o of es t rogens t o androgens i n chroni c renal fai l ure.
6. Drugs o
o
a. Estrogens, commonl y us ed t o t reat pros t at i c carci noma, s t i mul at e t he breas t di rect l y.
o
o
b. Spironolactone, cimetidine, and digitalis al s o produce gynecomas t i a. They are bel i eved t o i nhi bi t androgen act i on by di s pl aci ng di hydrot es t os t erone from i t s i nt racel l ul ar recept or.
o
o
c. Marijuana bi nds t o es t rogen recept ors and may caus e gynecomas t i a t hrough a di rect es t rogeni c act i on.
o
o
d. Ot her drugs t hat may caus e t hi s probl em i ncl ude phenothiazines, tricyclic antidepressants, methyldopa, reserpine, and isoniazid.
7. T umors may caus e gynecomas t i a. o
o
a. Adrenal and testicular tumors may caus e gynecomas t i a t hrough t he product i on of es t rogen.
o
Pa g e 2 1 6 4
ABC Ambe r CHM Conve rte r Tria l ve rsion, http://w w w .proce sste x t.com/a bcchm.html o
b. T esticular choriocarcinomas may caus e gynecomas t i a t hrough t he s ecret i on of hCG, whi ch s t i mul at es t es t i cul ar es t rogen product i on.
o
o
c. Ot her mal i gnant t umors may caus e t he condi t i on t hrough t he ect opi c product i on of gonadot ropi ns .
8. Hyperthyroidism i ncreas es t he convers i on of androgens t o es t rogens i n t he peri pheral t i s s ues and i ncreas es t he ci rcul at i ng l evel of s ex hormone–bi ndi ng gl obul i n, whi ch rai s es t he es t rogen-t o-androgen rat i o. Thes e hormonal changes may caus e gynecomas t i a i n men wi t h hypert hyroi di s m.
VIII. Metabolic Bone Disease The met abol i c bone di s eas es i ncl ude os t eomal aci a, os t eoporos i s , os t ei t i s fi bros a cys t i ca, and ot her di s eas es . Os t ei t i s fi bros a cys t i ca i s di s cus s ed bri efl y i n t he s ect i on on hyperparat hyroi di s m (s ee III A 4 b).
A. Osteomalacia
1. Definition. Os t eomal aci a i s a skeletal abnormality i n whi ch t here i s inadequate mineralization of bone matrix. In chi l dren, t hi s us ual l y t akes t he form of ri cket s , caus ed by vi t ami n D defi ci ency. In adul t s , os t eomal aci a may be caus ed by many s peci fi c abnormal i t i es of cal ci um, phos phorus , and vi t ami n D met abol i s m.
2. Etiology o
Pa g e 2 1 6 5
ABC Ambe r CHM Conve rte r Tria l ve rsion, http://w w w .proce sste x t.com/a bcchm.html o
a. Vitamin D deficiency
(1) Defi ci ency of vi t ami n D caus es os t eomal aci a becaus e i t s mos t act i ve met abol i t e, 1,25-di hydroxyvi t ami n D 3 , i s es s ent i al for t he abs orpt i on of cal ci um and phos phat e from t he gas t roi nt es t i nal t ract . P.439
(2) Defi ci ency of vi t ami n D i s not rare i n t he Uni t ed St at es . Al t hough mos t Ameri cans obt ai n t he recommended di et ary al l owance of 400 U vi t ami n D from fort i fi ed foods , es peci al l y dai ry product s , di et ary defi ci ency may s t i l l occur becaus e of povert y, food faddi s m, eat i ng di s orders s uch as anorexi a nervos a, or l ack of s un expos ure i n el derl y, debi l i t at ed pat i ent s .
(3) Expos ure t o s unl i ght convert s 7-dehydrochol es t erol i n t he s ki n t o vi t ami n D 3 ; t hi s i s an i mport ant s ource of t he vi t ami n. Abs ence of s unl i ght may cont ri but e t o vi t ami n D defi ci ency.
o
o
b. Abnormal metabolism of, or response to, vitamin D
(1) Liver disease, when far advanced, may caus e os t eomal aci a by i nt erferi ng wi t h t he
Pa g e 2 1 6 6
ABC Ambe r CHM Conve rte r Tria l ve rsion, http://w w w .proce sste x t.com/a bcchm.html
normal hepat i c convers i on of vi t ami n D t o 25-hydroxyvi t ami n D 3 .
(2) Anticonvulsant drugs s uch as phenobarbi t al and phenyt oi n, i f t aken over a l ong peri od, may al t er t he met abol i s m of vi t ami n D by i nduci ng hepat i c mi cros omal enzymes . Os t eomal aci a and decreas ed s erum l evel s of 25-hydroxyvi t ami n D 3 have been des cri bed i n pat i ent s recei vi ng l ong-t erm t reat ment wi t h ant i convul s ant s .
(3) Vitamin D–dependent rickets type I i s an aut os omal reces s i ve di s order caus ed by i mpai red act i vi t y of renal 1α-hydroxyl as e, l eadi ng t o i nadequat e convers i on of 25-hydroxyvi t ami n D 3 t o 1,25-di hydroxyvi t ami n D 3 (cal ci t ri ol ). It i s t reat ed wi t h s mal l , phys i ol ogi c dos es (0.5–1.0 µg) of cal ci t ri ol .
(4) Vitamin D–dependent rickets type II, al s o aut os omal reces s i ve, i s caus ed by res i s t ance t o t he act i on of vi t ami n D becaus e of al t ered s t ruct ure or funct i on of t he cal ci t ri ol recept ors . To overcome t hi s res i s t ance, l arge, s upraphys i ol ogi c dos es of cal ci t ri ol , al ong wi t h cal ci um s uppl ement at i on, mus t be us ed.
o
o
c. Renal abnormalities
(1) Renal osteodystrophy may occur i n
Pa g e 2 1 6 7
ABC Ambe r CHM Conve rte r Tria l ve rsion, http://w w w .proce sste x t.com/a bcchm.html
pat i ent s wi t h chroni c renal fai l ure of any caus e. Bot h os t eomal aci a, caus ed by i mpai red renal product i on of 1,25-di hydroxyvi t ami n D 3 , and os t ei t i s fi bros a cys t i ca, caus ed by t he s econdary hyperparat hyroi di s m of renal fai l ure, are pres ent i n varyi ng degrees .
(2) X-linked hypophosphatemic rickets and autosomal dominant hypophosphatemic rickets are X-l i nked domi nant di s orders i n whi ch t he pri mary abnormal i t y i s renal l os s of phos phat e. They are caus ed by gene mut at i ons t hat affect t he proxi mal t ubul ar reabs orpt i on of phos phat e. Rel at i ve defi ci ency of cal ci t ri ol product i on i s al s o pres ent . Supraphys i ol ogi c dos es of cal ci t ri ol , t oget her wi t h phos phat e, may rai s e t he s erum phos phat e l evel , decreas e t he bony abnormal i t i es of ri cket s , and i ncreas e growt h.
(3) In Fanconi' s syndrome, renal t ubul ar defect s may l ead t o t he l os s of phos phat e as wel l as cal ci um, gl ucos e, and ami no aci ds , wi t h res ul t i ng os t eomal aci a.
o
o
d. Gastrointestinal disorders. Any di s eas e or s urgi cal procedure t hat l eads t o mal abs orpt i on and s t eat orrhea may reduce t he abs orpt i on of cal ci um, phos phat e, and vi t ami n D; os t eomal aci a may res ul t .
o
o
e. T umors s uch as hemangi operi cyt omas and gi ant cel l t umors of bone may produce a humoral
Pa g e 2 1 6 8
ABC Ambe r CHM Conve rte r Tria l ve rsion, http://w w w .proce sste x t.com/a bcchm.html
s ubs t ance t hat caus es phos phat uri a and os t eomal aci a; t he s yndrome (“ oncogenic osteomalacia―) i s cured by removal of t he t umor.
3. Pathophysiology o
o
a. The common defect i n t he vari ous di s eas es as s oci at ed wi t h os t eomal aci a i s t he lack of calcium and phosphorus for mineralization of bone matrix.
o
o
b. Rickets is caused by defective mineralization of bone before closure of the cartilaginous growth plates. Deformi t y occurs becaus e of pres s ure on weakened growt h pl at es and on t he abnormal l y s oft s haft s of t he l ong bones . Aft er cl os ure of t he growt h pl at es , onl y os t eomal aci a can occur, wi t h defect i ve mi neral i zat i on of mat ure l amel l ar bone.
o
o
c. Hi s t ol ogi cal l y, bone bi ops y s hows an exces s of unmi neral i zed bone mat ri x, whi ch i s s een as an i ncreas e i n t he vol ume and t hi cknes s of os t eoi d s eams coveri ng t he bone s urfaces .
4. Clinical features o
o
a. Pain and tenderness are common i n affect ed areas of t he s kel et on, es peci al l y t he s pi ne, ri bs , pel vi s , and l ower ext remi t i es .
Pa g e 2 1 6 9
ABC Ambe r CHM Conve rte r Tria l ve rsion, http://w w w .proce sste x t.com/a bcchm.html o
o
b. Muscle weakness i s common, affect i ng part i cul arl y t he proxi mal mus cl es of t he l egs .
o
o
c. Skeletal deformities and fractures occur i n s evere cas es . P.440
(1) The l ong bones may bow becaus e of t he s oft eni ng of t he s kel et on.
(2) Ri cket s i n chi l dren i s as s oci at ed wi t h wi deni ng of t he epi phys es ; s wel l i ng of t he wri s t s , knees , ankl es , and cos t ochondral joi nt s ; bowl egs ; and di s t urbances i n growt h.
o
o
5. Laboratory findings
a. Radiographs may s how decreased bone density and coars eni ng of t he t rabecul ar pat t ern. Looser' s zones are radi ol ucent bands t hat are perpendi cul ar t o t he peri os t eal s urface, caus ed by ps eudofract ures .
b. Al t hough l aborat ory abnormal i t i es depend on t he caus e and t he s everi t y of t he os t eomal aci a, t hey oft en i ncl ude low serum
Pa g e 2 1 7 0
ABC Ambe r CHM Conve rte r Tria l ve rsion, http://w w w .proce sste x t.com/a bcchm.html
phosphate, low or normal serum calcium, and increased serum alkaline phosphatase levels. o
o
6. T herapy
a. T reatment of the primary disorder i s s omet i mes pos s i bl e (e.g., correct i on of a bowel di s order caus i ng mal abs orpt i on or removal of a t umor caus i ng os t eomal aci a).
b. Vitamin D i s us ual l y t he mai ns t ay of t reat ment .
(1) In s i mpl e vi t ami n D defi ci ency, a phys i ol ogi c dos e of 400–800 U vi t ami n D dai l y may be al l t hat i s needed. In pat i ent s wi t h s evere defi ci ency, 100,000 U of ergocal ci ferol weekl y for 1 mont h mi ght be needed.
(2) Large dos es of cal ci t ri ol , whi ch does not requi re bi ochemi cal t rans format i on t o achi eve ful l act i vi t y, may be needed i n t he uncommon s yndromes caus ed by al t ered met abol i s m or act i on of vi t ami n D or renal was t i ng of phos phat e.
B. Osteoporosis
1. Definition. Os t eoporos i s i s a decreas e i n t ot al bone
Pa g e 2 1 7 1
ABC Ambe r CHM Conve rte r Tria l ve rsion, http://w w w .proce sste x t.com/a bcchm.html
vol ume, wi t h changes i n bone mi croarchi t ect ure, l eadi ng t o an i ncreas ed s us cept i bi l i t y t o fract ures . In t he Uni t ed St at es over 1 mi l l i on os t eoporot i c fract ures occur yearl y. Bot h i ncreas ed bone res orpt i on and decreas ed bone format i on have been obs erved.
2. Etiology o
o
a. Decreased bone mass at maturity
(1) Aft er reachi ng i t s peak at approxi mat el y 30 years of age, bone mas s decl i nes t hroughout t he remai ni ng years of l i fe. A l ow t ot al bone mas s at mat uri t y, a rel at i vel y rapi d rat e of bone l os s , or bot h, cont ri but e t o t he devel opment of “i nvol ut i onal ― os t eoporos i s .
(2) Genetic factors affect t he bone mas s at mat uri t y.
(a) Men and bl acks have great er peak bone mas s and l es s os t eoporos i s ; women and i ndi vi dual s of nort hern European ances t ry have l es s bone mas s at mat uri t y and more os t eoporos i s .
(b) A fami l i al t endency t oward os t eoporos i s has been obs erved.
o
Pa g e 2 1 7 2
ABC Ambe r CHM Conve rte r Tria l ve rsion, http://w w w .proce sste x t.com/a bcchm.html o
b. Calcium deficiency. Evi dence s ugges t s t hat cal ci um i nt ake i n Ameri can women i s l es s t han i s needed t o mai nt ai n cal ci um bal ance. As s oci at ed vi t ami n D defi ci ency i s very common.
(1) More t han 75% of women ol der t han 35 years of age fai l t o i nges t t he recommended dai l y al l owance of 1000 mg cal ci um. Al s o, cal ci um abs orpt i on decreas es i n l at er l i fe.
(2) The need t o mai nt ai n normal s erum l evel s of cal ci um may l ead t o i ncreas ed bone res orpt i on t hrough t he act i on of PTH. (The effect of PTH on bone i s t o i ncreas e t he rat e of cal ci um and phos phat e res orpt i on, and when s erum cal ci um l evel s are l ow, t he s ecret i on of PTH i ncreas es .)
o
o
c. Hormone changes. W hen es t rogen l evel s decreas e, whet her becaus e of ovari an di s eas e, oophorect omy, or normal menopaus e, t he rat e of bone l os s i s accel erat ed.
(1) Es t rogen defi ci ency may res ul t i n l es s s t i mul at i on of os t eobl as t i c act i vi t y and may i ncreas e t he s ens i t i vi t y of bone t o t he act i on of PTH.
(2) The i ncreas ed rat e of bone l os s pers i s t s for 5–10 years aft er menopaus e.
Pa g e 2 1 7 3
ABC Ambe r CHM Conve rte r Tria l ve rsion, http://w w w .proce sste x t.com/a bcchm.html
(3) Hypogonadi s m i n men i s al s o a caus e of bone l os s and i ncreas ed ri s k of fract ures .
3. Classification o
o
a. T wo types of os t eoporos i s have been des cri bed.
(1) Postmenopausal osteoporosis pri mari l y affect s women wi t hi n 15 years of menopaus e. The l os s of t rabecul ar bone i s accel erat ed, and fract ures of t he vert ebrae, whi ch cons i s t mai nl y of t rabecul ar bone, are common. P.441
(2) Senile osteoporosis affect s men and women ol der t han 75 years of age, caus i ng l os s of bot h cort i cal and t rabecul ar bone. Fract ures of t he hi p, whi ch i s l argel y cort i cal bone, occur, as do vert ebral fract ures .
o
o
b. Secondary osteoporosis may be as s oci at ed wi t h gl ucocort i coi d t herapy or s pont aneous Cus hi ng's s yndrome, mal abs orpt i on s yndromes or mal nut ri t i on, mul t i pl e myel oma, and prol onged i mmobi l i zat i on, among ot her condi t i ons .
4. Clinical features o
Pa g e 2 1 7 4
ABC Ambe r CHM Conve rte r Tria l ve rsion, http://w w w .proce sste x t.com/a bcchm.html o
a. Fractures
(1) Vertebral compression fractures t ypi cal l y affect T8 t o L3 and occur more commonl y i n women. They may caus e acut e back pai n t hat pers i s t s for s everal mont hs or may occur gradual l y and pai nl es s l y.
(2) Hip fractures, charact eri s t i cal l y i n t he neck and i nt ert rochant eri c regi ons of t he femur, are common i n bot h men and women ol der t han 65 years of age. Los s of funct i on frequent l y res ul t s , and, becaus e of compl i cat i ons , mort al i t y rat es may be as hi gh as 20% wi t hi n 1 year.
(3) The distal radius and ot her areas al s o may be t he s i t es of fract ures .
o
o
b. Pain and deformity. Back pai n may pers i s t l ong aft er an epi s ode of vert ebral fract ure becaus e of s pi nal deformi t y and al t erat i on of s pi nal mechani cs . Several i nches may be l os t from hei ght , and s evere kyphos i s may be t he res ul t of mul t i pl e vert ebral fract ures .
5. Diagnosis o
o
a. Radiography of the spine may i ndi cat e a decreas e i n bone dens i t y, wi t h accent uat i on of t he cort i cal out l i nes and promi nence of t he t rabecul ae.
Pa g e 2 1 7 5
ABC Ambe r CHM Conve rte r Tria l ve rsion, http://w w w .proce sste x t.com/a bcchm.html
(1) However, approxi mat el y 30% of bone t i s s ue mus t be l os t before t hes e abnormal i t i es appear on pl ai n radi ographs .
(2) W edge-s haped deformi t i es and compres s i on fract ures on s pi nal radi ographs al s o s ugges t t he di agnos i s of os t eoporos i s .
o
o
b. Dual-energy x-ray absorptiometry (DEXA), t he bes t met hod for eval uat i ng bone mi neral dens i t y, i s t he reference s t andard for t he di agnos i s of os t eoporos i s . The W orl d Heal t h Organi zat i on (W HO) has s ugges t ed t hat a di agnos i s of osteopenia be made when bone mi neral dens i t y i s reduced by more t han 1 s t andard devi at i on (SD) but l es s t han 2.5 SD bel ow t he mean bone dens i t y of young adul t s , and osteoporosis when i t i s reduced by 2.5 SD or more.
o
o
c. Screening for osteoporosis
(1) Preci s e gui del i nes are not avai l abl e, but many agree t hat DEXA of t he s pi ne and hi p s houl d be performed i n women ol der t han 65 years , and i n pos t menopaus al women younger t han 65 i f t hey have addi t i onal ri s k fact ors for os t eoporos i s .
(2) Risk factors for osteoporosis are l i s t ed i n Tabl e 9-23.
Pa g e 2 1 7 6
ABC Ambe r CHM Conve rte r Tria l ve rsion, http://w w w .proce sste x t.com/a bcchm.html
6. T herapy. W ei ght -beari ng exerci s e and adequat e i nt akes of cal ci um and vi t ami n D s houl d be encouraged i n everyone t o hel p prevent os t eoporos i s , and t hes e meas ures remai n i mport ant i n t he t reat ment of os t eoporos i s . The mos t pot ent pharmacol ogi c agent s are t he bi s phos phonat es and parat hyroi d hormone. Ot her t reat ment s t hat may be of s omewhat l es s er val ue are es t rogen repl acement t herapy, ral oxi fene, and cal ci t oni n. o
o
a. Calcium
(1) Pat i ent s wi t h os t eoporos i s s houl d i nges t at l eas t 1500 mg el ement al cal ci um dai l y.
(a) Becaus e t he average Ameri can t akes i n onl y approxi mat el y 500 mg cal ci um i n food, cal ci um s uppl ement s s houl d be gi ven. Cal ci um carbonat e t abl et s (e.g., 500 mg Os -Cal t wo or t hree t i mes dai l y) are us ual l y wel l t ol erat ed.
(b) Exogenous cal ci um may reduce t he rat e of bone l os s and t he fract ure rat e i n pat i ent s wi t h os t eoporos i s whos e us ual cal ci um i nt ake i s i nadequat e.
(2) Prevent i on of os t eoporos i s requi res at t ent i on t o cal ci um i nt ake i n heal t hy pers ons . Recommended dai l y cal ci um i nt ake i s
Pa g e 2 1 7 7
ABC Ambe r CHM Conve rte r Tria l ve rsion, http://w w w .proce sste x t.com/a bcchm.html
1000 mg i n women who are premenopaus al or t aki ng es t rogen repl acement and i n men younger t han 65. A hi gher i nt ake (1500 mg dai l y) i s recommended i n younger i ndi vi dual s who have not yet achi eved t hei r peak bone mas s , i n ol der men, and i n es t rogen-defi ci ent women. Achi evi ng t hes e goal s oft en requi res cal ci um s uppl ement at i on. o
o
b. Vitamin D. Pharmacol ogi c dos es (>1000 U dai l y) have not proved effect i ve i n t he t reat ment of os t eoporos i s . But phys i ol ogi c dos es (400–800 U dai l y) have decreas ed t he fract ure rat e i n P.442
el derl y pers ons who may be defi ci ent i n vi t ami n D becaus e of poor nut ri t i on and l ack of expos ure t o s unl i ght . Vi t ami n D s uppl ement at i on of 400–800 U dai l y s houl d be recommended i n s uch i ndi vi dual s and i n pat i ent s wi t h os t eoporos i s .
TABLE 9-23 Risk factors for Osteoporosis Per s on al hi s t or y of fra ct u re as
Pa g e 2 1 7 8
ABC Ambe r CHM Conve rte r Tria l ve rsion, http://w w w .proce sste x t.com/a bcchm.html
an ad ul t Hi s t or y of fra gi l i ty in a fi rs t -d egr ee rel at i ve Lo w bo dy wei ght (< 58 kg [12 7 l b] ) Cur ren t
Pa g e 2 1 7 9
ABC Ambe r CHM Conve rte r Tria l ve rsion, http://w w w .proce sste x t.com/a bcchm.html
ci g are tte sm oki ng Fe ma le s ex Es t rog en def i ci e ncy at an ear ly ag e (m en op aus e bef ore ag e 45 yea rs or
Pa g e 2 1 8 0
ABC Ambe r CHM Conve rte r Tria l ve rsion, http://w w w .proce sste x t.com/a bcchm.html
bi l at e ral ova ri e ct o my , pro l on ge d pre me no pa us a l am en orr he a (> 1 yea r) Wh ite rac e Ad van ce ag
Pa g e 2 1 8 1
ABC Ambe r CHM Conve rte r Tria l ve rsion, http://w w w .proce sste x t.com/a bcchm.html
e (> ag e 65) Li f el o ng l ow cal ci u m i nt ake Al c oh ol i sm Ina de qu at e phy sic al act i vi t y Rec urr ent fal l s De me
Pa g e 2 1 8 2
ABC Ambe r CHM Conve rte r Tria l ve rsion, http://w w w .proce sste x t.com/a bcchm.html
nt i a Im pai red eye sig ht des pi t e ad eq uat e cor rec tio n Po or he al t h/f rai l ty Me di c al con di t i ons : chr oni
Pa g e 2 1 8 3
ABC Ambe r CHM Conve rte r Tria l ve rsion, http://w w w .proce sste x t.com/a bcchm.html
c obs t ru ct i ve pul mo nar y di s eas e, gas t re ct o my , hyp erp ara t hy roi di s m, hyp og on adi sm , mu ltip le my el o
Pa g e 2 1 8 4
ABC Ambe r CHM Conve rte r Tria l ve rsion, http://w w w .proce sste x t.com/a bcchm.html
ma , cel i ac di s eas e Gl u coc ort i coi d t he rap y for >3 mo nt h s Ot her dru gs : ant i co nvu lsa nt s , Gn RH ag oni sts
Pa g e 2 1 8 5
ABC Ambe r CHM Conve rte r Tria l ve rsion, http://w w w .proce sste x t.com/a bcchm.html
, lith iu m, exc es s i ve dos es of t hy roi d hor mo ne Gn RH, go na dot rop i n-r el e as i ng hor mo ne. o
o
c. Bisphosphonates
(1) Thes e agent s bi nd t o hydroxyapat i t e i n bone and decreas e os t eocl as t i c bone
Pa g e 2 1 8 6
ABC Ambe r CHM Conve rte r Tria l ve rsion, http://w w w .proce sste x t.com/a bcchm.html
res orpt i on. Becaus e os t eobl as t i c bone format i on cont i nues for a t i me whi l e res orpt i on cavi t i es i n bone are fi l l ed i n, bone dens i t y may i ncreas e by 5%–10% over t he next 1–2 years . Aft er t hi s , t he mai n effect of ant i res orpt i ve agent s s uch as t he bi s phos phonat es i s t o prevent t he gradual decreas e i n bone dens i t y t hat mi ght ot herwi s e occur.
(2) Alendronate (Fosamax) i s very effect i ve i n i ncreas i ng bone mi neral dens i t y and i n decreas i ng t he fract ure rat e.
(a) The dos e i s 10 mg once dai l y or 70 mg once weekl y, whi ch i s equal l y effect i ve.
(b) The pri mary s i de effect s are gas t roi nt es t i nal s ympt oms , es peci al l y es ophageal i rri t at i on, and, i n a few cas es , es ophageal eros i on. To prevent t hes e compl i cat i ons and t o maxi mi ze drug abs orpt i on, pat i ent s mus t t ake al endronat e wi t h a ful l gl as s of wat er on ari s i ng i n t he morni ng; 30 mi nut es before t he fi rs t meal , beverage, or ot her medi cat i on; and t hey mus t not l i e down for 30 mi nut es aft er t hey s wal l ow t he pi l l .
(3) Risedronate (Actonel) i s s i mi l ar i n effect i venes s t o al endronat e but may be l es s
Pa g e 2 1 8 7
ABC Ambe r CHM Conve rte r Tria l ve rsion, http://w w w .proce sste x t.com/a bcchm.html
l i kel y t o caus e gas t roi nt es t i nal s i de effect s . The dos age i s 5 mg once dai l y or 35 mg once weekl y. o
o
d. Parathyroid hormone. W hen t he s kel et on i s expos ed t o cons t ant l y el evat ed bl ood l evel s of PTH, as i n pat i ent s wi t h pri mary hyperparat hyroi di s m, bone dens i t y i s l os t and fract ure ri s k i s i ncreas ed (see III A 4 b). But , paradoxi cal l y, when l ow dos es of PTH are i nject ed once every 24 hours , t he oppos i t e effect occurs : bone dens i t y i ncreas es and fract ure ri s k fal l s . Apparent l y t he i nt ermi t t ent expos ure of bone t o PTH favors bone format i on over bone res orpt i on.
(1) It i s pos s i bl e t hat PTH i ni t i al l y caus es res orpt i on of bone, whi ch rel eas es growt h fact ors s uch as IGF-1 and t rans formi ng growt h fact or bet a, whi ch t hen s t i mul at e os t eobl as t i c bone format i on. Thi s hypot hes i s i s favored by t he obs ervat i on t hat PTH l os es much of i t s bone-formi ng act i vi t y i f gi ven t oget her wi t h al endronat e, an i nhi bi t or of bone res orpt i on.
(2) PT H (teriparatide, Forteo) i s gi ven once dai l y i n a dos e of 20 mg by s ubcut aneous i nject i on. Si de effect s may i ncl ude hypercal cemi a (requi ri ng dos e reduct i on or di s cont i nuat i on P.443
of PTH), di zzi nes s , and l eg cramps . It has been
Pa g e 2 1 8 8
ABC Ambe r CHM Conve rte r Tria l ve rsion, http://w w w .proce sste x t.com/a bcchm.html
s hown t o prevent vert ebral fract ures but i t s rol e i n t he hi p i s uncl ear.
(3) The rol e of PTH i n t he t reat ment of os t eoporos i s i s not yet known. It s effect s on bone dens i t y are favorabl e compared wi t h ot her avai l abl e agent s , but i t i s expens i ve, i t requi res dai l y i nject i ons , and l ong-t erm experi ence i s l acki ng. It s houl d be cons i dered for us e i n pat i ent s wi t h s evere os t eoporos i s who are at hi gh ri s k for fract ures and who have not res ponded wel l t o ot her t reat ment s .
o
o
e. Estrogen. Es t rogen repl acement t herapy aft er menopaus e has been an i mport ant meas ure for t he prevent i on and t reat ment of os t eoporos i s . Recent fi ndi ngs , however, s ugges t t hat t he ri s ks as s oci at ed wi t h es t rogen t herapy, es peci al l y breas t cancer, t hromboembol i s m, and cardi ovas cul ar di s eas e, are great er t han previ ous l y t hought . Becaus e ot her very effect i ve t reat ment for os t eoporos i s i s avai l abl e, s uch as bi s phos phonat es and PTH, es t rogen repl acement t herapy i s as s umi ng a s econdary rol e i n t reat ment of os t eoporos i s .
o
o
f. Raloxifene. A group of compounds cal l ed s el ect i ve es t rogen recept or modul at ors (SERMs ) has been devel oped. Ral oxi fene, one member of t hi s group, has benefi ci al effect s on t he s kel et on s i mi l ar t o es t rogen (al t hough l es s pot ent )—reduces t he ri s k of vert ebral fract ures —but wi t hout t he undes i red es t rogeni c effect s on t he breas t s and ut erus .
Pa g e 2 1 8 9
ABC Ambe r CHM Conve rte r Tria l ve rsion, http://w w w .proce sste x t.com/a bcchm.html o
o
g. Calcitonin. Thi s hormone, whi ch i s s ecret ed by t he parafol l i cul ar cel l s of t he t hyroi d, i nhi bi t s os t eocl as t act i vi t y and decreas es t he rat e of bone l os s and fract ures i n os t eoporos i s . It may be gi ven by i nject i on or nas al s pray (Mi acal ci n). The nas al s pray del i vers 200 uni t s of cal ci t oni n, t he dai l y dos e, i n one puff; t he pat i ent al t ernat es nos t ri l s each day becaus e of pos s i bl e nas al i rri t at i on. It i s recommended i n pat i ent s wi t h s i gni fi cant pai n from an acut e os t eoporot i c fract ure. Tachyphyl axi s occurs wi t h chroni c us e.
P.444
Study Questions/Answers and Explanations 1. A 23-year-old man with gynecomastia is found to have a 47,XXY karyotype. T his patient probably also has which of the following conditions? A. Abnormal l i ver funct i on t es t s B. Low bl ood l evel s of l ut ei ni zi ng hormone (LH) and fol l i cl e-s t i mul at i ng hormone (FSH) C. Hi gh bl ood l evel s of es t rogen D. Azoos permi a E. Enl argement of t he t es t es Vi ew Ans wer 1. T he answer is D [VII A 2 b (1) (a)]. The 47,XXY karyot ype i ndi cat es Kl i nefel t er's s yndrome. El evat i on of gonadot ropi n l evel s , s mal l t es t es , and gynecomas t i a al s o are common fi ndi ngs i n Kl i nefel t er's s yndrome. 2. An 18-year-old woman is evaluated because she has never had a menstrual period. Pelvic examination is normal except that the vagina ends in a blind pouch. A karyotype is reported to be 46,XY. Which of the following diagnoses is most likely?
Pa g e 2 1 9 0
ABC Ambe r CHM Conve rte r Tria l ve rsion, http://w w w .proce sste x t.com/a bcchm.html
A. Congeni t al adrenal hyperpl as i a B. Turner's s yndrome C. Kal l mann's s yndrome D. Tes t i cul ar femi ni zat i on s yndrome E. Pol ycys t i c ovary s yndrome Vi ew Ans wer 2. T he answer is D [VI A 1 a, 2 b, C 1 b; VII A 2 a (1)]. Pat i ent s wi t h t he t es t i cul ar femi ni zat i on s yndrome have a normal mal e karyot ype and t es t es (l ocat ed i n t he abdomen or groi n) t hat produce t es t os t erone. However, becaus e t here i s res i s t ance of t he t i s s ues t o t he effect s of t es t os t erone, t he ext ernal geni t al i a devel op as femal e duri ng fet al l i fe. Congeni t al adrenal hyperpl as i a i s not as s oci at ed wi t h a s hort vagi na and bl i nd pouch, bot h of whi ch are t ypi cal of t es t i cul ar femi ni zat i on. Turner's s yndrome (gonadal dys genes i s ) i s charact eri zed by a 45,X karyot ype. Kal l mann's s yndrome i s more common i n men t han i n women and i s oft en accompani ed by anos mi a and cl eft pal at e. Pol ycys t i c ovary s yndrome us ual l y caus es s econdary, not pri mary, amenorrhea, and t hi s pat i ent has none of t he ot her t ypi cal cl i ni cal feat ures (e.g., oi l y s ki n, obes i t y, or hi rs ut i s m). 3. A 55-year-old woman has been treated for type 2 diabetes with isophane insulin suspension (NPH), 35 U once daily before breakfast. Home glucose measurements on a typical day are 7:00 AM (fasting), 238 mg/dL; 11:00 AM, 155 mg/dL; 4:00 PM, 128 mg/dL; and 8:00 PM, 125 mg/dL. Which of the following changes in insulin therapy would be reasonable? A. Increas i ng t he dos e of i ns ul i n B. Addi ng regul ar i ns ul i n t o t he dos e of NPH C. Gi vi ng t he dos e i n t he eveni ng i ns t ead of t he morni ng D. Addi ng a dos e of regul ar i ns ul i n before s upper E. Addi ng a s econd dos e of NPH i ns ul i n at bedt i me Vi ew Ans wer 3. T he answer is E [
IV A 8 d (4) (b)]. The pat t ern of gl ucos e
l evel s s ugges t s t hat t he dos e of i s ophane i ns ul i n s us pens i on (NPH) i s effect i ve duri ng t he day but does not ret ai n i t s effect unt i l t he
Pa g e 2 1 9 1
ABC Ambe r CHM Conve rte r Tria l ve rsion, http://w w w .proce sste x t.com/a bcchm.html
next morni ng. Gi vi ng more i ns ul i n i n t he morni ng or addi ng regul ar i ns ul i n woul d ri s k caus i ng hypogl ycemi a i n t he aft ernoon and eveni ng when gl ucos e l evel s are al ready at a s at i s fact ory l evel . However, a s econd dos e of NPH at bedt i me woul d be expect ed t o have i t s maxi mal act i on i n t he earl y morni ng, whi ch i s t he t i me t hat an i ncreas ed gl ucos e-l oweri ng effect i s des i red. 4. A 23-year old woman has noted a small amount of milk secretion from her nipples, and she has had no menstrual periods for the past 8 months. Serum prolactin is 78 ng/mL (normal, 5–25 ng/mL). Which test would be most sensitive for the diagnosis of a pituitary adenoma? A. Meas urement of s erum LH and FSH B. Vi s ual fi el d exami nat i on C. Comput ed t omography (CT) s can wi t h cont ras t i nject i on D. Magnet i c res onance i magi ng (MRI) wi t h gadol i ni um i nject i on E. Ins ul i n t ol erance t es t Vi ew Ans wer 4. T he answer is D [I A 1 b (3) (a), (4), c (1) (a)]. Magnet i c res onance i magi ng (MRI) wi t h gadol i ni um i s t he mos t s ens i t i ve t es t for t he det ect i on of pi t ui t ary adenomas . In fact , mi croadenomas may be s een i n as many as 10%–20% of normal women. Overs ecret i on of LH and FSH i s common, but many pi t ui t ary adenomas do not produce exces s i ve amount s of t hes e hormones . Impai red growt h hormone (GH) res pons e t o i ns ul i n-i nduced hypogl ycemi a or vi s ual fi el d changes woul d occur onl y i n t hos e t umors t hat are l arge enough t o i nt erfere wi t h normal pi t ui t ary funct i on or t o compres s t he opt i c chi as m. Comput ed t omography (CT) s cans s how very s mal l adenomas , but MRI wi t h gadol i ni um i s even more s ens i t i ve. 5. Successful treatment of primary hyperparathyroidism in a 60-year-old woman involves surgery to remove a single parathyroid adenoma. After surgery, she has a prolonged period of hypocalcemia, which requires continuous treatment with large doses of vitamin D and calcium. After 2–3 months, the need for vitamin D and calcium subsides, and she remains normocalcemic without treatment. T his woman probably had
Pa g e 2 1 9 2
ABC Ambe r CHM Conve rte r Tria l ve rsion, http://w w w .proce sste x t.com/a bcchm.html
which of the following conditions? A. Acci dent al des t ruct i on of t he ot her t hree parat hyroi d gl ands B. Removal of t he wrong parat hyroi d gl and C. Severe pancreat i t i s caus ed by her hyperparat hyroi di s m D. Unrecogni zed ps eudohypoparat hyroi di s m E. Severe bone di s eas e Vi ew Ans wer 5. T he answer is E [III A 7 a (4)]. W hen pri mary hyperparat hyroi di s m caus es os t ei t i s fi bros a cys t i ca, t he s udden correct i on of t he pri mary hyperparat hyroi di s m and cons equent removal of t he s ource of exces s i ve parat hyroi d hormone (PTH) al l ows t he s kel et on t o undergo rapi d repai r and remi neral i zat i on, whi ch creat es a marked but s el f-l i mi t i ng demand for cal ci um and phos phat e. If t he pat i ent had hypoparat hyroi di s m of t hi s s everi t y aft er s urgery, t he hypocal cemi a woul d probabl y have been permanent , and t he need for t reat ment woul d not have res ol ved. 6. A 32-year-old woman is found on gynecologic evaluation to have multiple ovarian cysts. Which of the following findings would lead you to make the diagnosis of polycystic ovary syndrome? A. Deepeni ng of t he voi ce, enl argement of t he cl i t ori s , and a hi gh t es t os t erone l evel B. Ol i gomenorrhea, obes i t y, a hi gh l ut ei ni zi ng hormone (LH) l evel , and a l ow fol l i cl e-s t i mul at i ng hormone (FSH) l evel C. Amenorrhea, acne, a l ow LH l evel , and a l ow FSH l evel D. Faci al hi rs ut i s m, acne, and i ncreas ed uri nary pregnanet ri ol , and 17-ket os t eroi ds E. Faci al hi rs ut i s m, normal mens t rual peri ods , and normal l evel s of FSH, LH, and t es t os t erone Vi ew Ans wer 6. T he answer is B [VI C 1]. Pol ycys t i c ovary s yndrome i s charact eri zed by a chroni c l ack of ovul at i on as s oci at ed wi t h s ympt oms of androgen exces s and oft en wi t h obes i t y. Severe vi ri l i z at i on and marked t es t os t erone el evat i on are more l i kel y t o be caus ed by an ovari an t umor or hypert hecos i s . Nei t her a l ow
Pa g e 2 1 9 3
ABC Ambe r CHM Conve rte r Tria l ve rsion, http://w w w .proce sste x t.com/a bcchm.html
l ut ei ni z i ng hormone (LH) l evel nor an i ncreas ed uri nary pregnanet ri ol concent rat i on i s charact eri s t i c of t he pol ycys t i c ovary s yndrome. Hi rs ut i s m wi t hout ot her cl i ni cal or l aborat ory abnormal i t i es us ual l y i s di agnos ed as “i di opat hi c hi rs ut i s m.― 7. A 58-year-old woman complains of fatigue, weight gain, and constipation. Examination shows puffy facial features and a slow return phase of the ankle reflex. Serum T SH is 52 mU/L (high), and free T 4 is 0.3 ng/dL (low). T reatment should be started with which of the following preparations? A. Thyroi d ext ract B. Thyrogl obul i n C. Thyroxi ne (T 4 ) D. Tri i odot hyroni ne (T 3 ) E. T 4 and T 3 Vi ew Ans wer 7. T he answer is C [II B 4 a]. Thyroxi ne (T 4 ) i s t he agent of choi ce. Thyroi d ext ract and t hyrogl obul i n cont ai n varyi ng proport i ons of t he t wo t hyroi d hormones , t hyroxi ne (T 4 ) and t ri i odot hyroni ne (T 3 ), maki ng i t di ffi cul t t o adjus t t he dos age preci s el y. Preparat i ons cont ai ni ng T 3 mus t be gi ven s everal t i mes dai l y t o mai nt ai n a normal bl ood l evel of T 3 becaus e T 3 has a s hort hal f-l i fe. However, T 4
has a l ong hal f-l i fe and i s convert ed t o T 3 i n t he l i ver and
el s ewhere; hypot hyroi d pat i ent s t aki ng t he opt i mal dos e of T 4 once dai l y have normal , s t abl e bl ood l evel s of bot h T 4 and T 3 . 8. A 45-year-old woman complains of nervousness, palpitations, and a 10-pound weight loss. Her thyroid gland is enlarged twofold, and her heart rate is 108 beats/minute. Free T 4 is 3.6 ng/dL (high), and T SH is undetectable. You recommend radioiodine therapy, and the patient asks you about possible complications. What is the most common complication of radioiodine therapy? A. Thyroi d s t orm B. Subacut e t hyroi di t i s C. Thyroi d cancer D. Hypot hyroi di s m
Pa g e 2 1 9 4
ABC Ambe r CHM Conve rte r Tria l ve rsion, http://w w w .proce sste x t.com/a bcchm.html
E. Leukemi a Vi ew Ans wer 8. T he answer is D [II C 4 a (3) (c) (i i )–(i i i )]. Hypot hyroi di s m i s pres ent i n 50% or more of pat i ent s t reat ed wi t h radi oi odi ne 10–15 years aft er t reat ment . Thyroi d s t orm and s ubacut e t hyroi di t i s are rare compl i cat i ons . No i ncreas ed i nci dence of t hyroi d cancer, l eukemi a, or ot her mal i gnanci es has been at t ri but ed t o radi oi odi ne t herapy. 9. A 60-year-old woman complains of headaches, and an MRI of the head is performed. T his study shows a 9-mm microadenoma of the pituitary gland. Which of the following hormones is most likely to be elevated? A. Growt h hormone (GH) B. Adrenocort i cot ropi c hormone (ACTH) C. Prol act i n D. Thyroi d-s t i mul at i ng hormone (TSH) E. Ins ul i n-l i ke growt h fact or 1 (IGF-1) Vi ew Ans wer 9. T he answer is C [I A 1 a (1), b (1), 4]. As many as 50% of al l pi t ui t ary adenomas have been found t o s ecret e prol act i n, and bl ood prol act i n l evel s s houl d be meas ured i n a pat i ent s us pect ed of havi ng a pi t ui t ary t umor. Acromegal y due t o growt h hormone (GH) exces s and Cus hi ng's di s eas e due t o adrenocort i cot ropi c hormone (ACTH) exces s are cons i derabl y l es s common, and overproduct i on of t hyroi d-s t i mul at i ng hormone (TSH) i s rare. Ins ul i n-l i ke growt h fact or I i s i ncreas ed i n acromegal y but i s not el evat ed i n mos t pat i ent s wi t h pi t ui t ary t umors . 10. A 19-year-old woman complains of nervousness, a 5-pound weight loss, tremors, palpitations, and sweating for the past 4 weeks. T he thyroid gland is slightly enlarged but not tender. T he total T 4 level is 15.3 µg/dL (normal, 4.5–12.5), and T 3 uptake is 38% (normal, 25–35), and the T SH is undetectable. T he best way to differentiate between the syndrome of painless thyroiditis and Graves' disease involves which of the following findings?
Pa g e 2 1 9 5
ABC Ambe r CHM Conve rte r Tria l ve rsion, http://w w w .proce sste x t.com/a bcchm.html
A. Thyroi d enl argement B. Low bl ood t hyroi d-s t i mul at i ng hormone (TSH) l evel s C. El evat ed bl ood t hyroxi ne (T 4 ) l evel s D. Low radi oact i ve i odi ne upt ake E. Tendernes s and pai n i nvol vi ng t he t hyroi d gl and Vi ew Ans wer 10. T he answer is D [II D 3 a (2)]. Low radi oact i ve i odi ne upt ake i s t he mos t us eful fi ndi ng for di s t i ngui s hi ng pai nl es s t hyroi di t i s from Graves ' di s eas e. Infl ammat i on and i njury t o t hyroi d cel l s , as wel l as a l ack of t hyroi d-s t i mul at i ng hormone (TSH), i nhi bi t radi oact i ve i odi ne upt ake i n pai nl es s t hyroi di t i s , whereas t he uni njured and i mmunogl obul i n-s t i mul at ed t hyroi d cel l s i n Graves ' di s eas e concent rat e radi oact i ve i odi ne at an i ncreas ed rat e. Enl argement of t he t hyroi d gl and and i ncreas ed bl ood l evel s of t hyroi d hormone, wi t h s uppres s i on of TSH, may occur i n bot h Graves ' di s eas e and pai nl es s t hyroi di t i s . The gl and i s not t ender or pai nful i n ei t her condi t i on. 11. A 23-year-old man is evaluated because of a diagnosis of hypogonadism. Which of the following findings would suggest primary testicular disease rather than hypothalamic or pituitary disease? A. Anos mi a B. Increas ed l evel s of fol l i cl e-s t i mul at i ng hormone (FSH) and l ut ei ni z i ng hormone (LH) C. Eunuchoi dal habi t us D. Los s of l i bi do and s exual pot ency E. Decreas ed s perm number and mot i l i t y Vi ew Ans wer 11. T he answer is B [VII A 1, 2]. Increas ed gonadot ropi n product i on i ndi cat es pri mary t es t i cul ar fai l ure wi t h negat i ve-feedback s t i mul at i on of t he hypot hal ami c–pi t ui t ary axi s . Anos mi a i s s omet i mes as s oci at ed wi t h hypot hal ami c fai l ure t o s ecret e gonadot ropi n-rel eas i ng hormone (GnRH). Hypogonadi s m, whet her caus ed by hypot hal ami c–pi t ui t ary di s eas e or t es t i cul ar di s eas e, i s as s oci at ed wi t h l os s of l i bi do and pot ency and
Pa g e 2 1 9 6
ABC Ambe r CHM Conve rte r Tria l ve rsion, http://w w w .proce sste x t.com/a bcchm.html
abnormal i t i es of s perm product i on. A eunuchoi dal habi t us res ul t s from cont i nued growt h of l ong bones due t o del ay i n t es t os t erone-i nduced epi phys eal cl os ure; t herefore, eunuchoi dal habi t us can res ul t from ei t her pri mary t es t i cul ar fai l ure or hypot hal ami c–pi t ui t ary di s eas e. 12. A 48-year-old man requires surgery to remove a tumor involving the hypothalamic area. T he pituitary stalk is damaged by the surgery. Which pituitary hormone might be expected to increase rather than decrease in serum concentration? A. Adrenocort i cot ropi c hormone (ACTH) B. Thyroi d-s t i mul at i ng hormone (TSH) C. Growt h hormone (GH) D. Prol act i n E. Lut ei ni zi ng hormone (LH) Vi ew Ans wer 12. T he answer is D [I A 3 a (2)]. Hypot hal ami c hormones reach t he ant eri or pi t ui t ary gl and t hrough t he port al ves s el s i n t he pi t ui t ary s t al k, and i njury t o t he s t al k may remove t he pi t ui t ary from t he i nfl uence of t he hypot hal amus . Mos t of t he hypot hal ami c fact ors are s t i mul at ory: ACTH product i on i s s t i mul at ed by cort i cot ropi n-rel eas i ng hormone (CRH), TSH by t hyrot ropi n-rel eas i ng hormone (TRH), growt h hormone by growt h hormone-rel eas i ng hormone (GHRH), and LH by gonadot ropi n-rel eas i ng hormone (GnRH). But t he mai n effect of t he hypot hal amus on prol act i n product i on i s i nhi bi t ory, t hrough t he act i on of dopami ne. W hen i njury t o t he pi t ui t ary s t al k prevent s dopami ne and ot her hypot hal ami c fact ors from reachi ng t he pi t ui t ary gl and i n hi gh concent rat i on, prol act i n product i on i ncreas es , i n cont ras t t o t he fal l i n l evel s of ot her pi t ui t ary hormones . 13. A 35-year-old woman has an MRI of the abdomen because of abdominal pain. Unexpectedly a 1-cm adenoma is seen in the left adrenal gland. In your evaluation of the patient, which of the following findings would be most compatible with a diagnosis of primary aldosteronism? A. Hyponat remi a
Pa g e 2 1 9 7
ABC Ambe r CHM Conve rte r Tria l ve rsion, http://w w w .proce sste x t.com/a bcchm.html
B. Aci dos i s C. Hypot ens i on D. Hyperkal emi a E. Suppres s ed pl as ma reni n act i vi t y Vi ew Ans wer 13. T he answer is E [V D 2, 3 a (3) (b)]. Plas ma reni n act i vi t y s uppres s i on i s a cl i ni cal feat ure of pri mary al dos t eroni s m. Exces s ci rcul at i ng l evel s of al dos t erone i ncreas e t he reabs orpt i on of s odi um, i n exchange for pot as s i um and hydrogen i ons , i n t he di s t al t ubul es . The res ul t i ng expans i on of ext racel l ul ar fl ui d vol ume caus es s uppres s i on of pl as ma reni n act i vi t y and event ual l y caus es hypert ens i on. The l os s of pot as s i um and hydrogen i ons caus es a t endency t oward met abol i c al kal os i s . Directions: T he response options for Items 14-16 are the same. You will be required to select one answer for each item in the set. A. Graves' disease B. Hypothyroidism C. Pregnancy D. Subacute thyroiditis E. Nontoxic goiter For each result of thyroid function tests, select the clinical condition with which it is most likely to be associated. 14. A 31-year-old woman complains of inability to sleep, weakness, heat intolerance, and sweating. T he free T 4 is elevated, and the radioiodine uptake is low. 15. A 26-year-old woman with some fatigue is found to have an elevated total T 4 level and a low T 3 resin uptake. 16. A 21-year-old woman has noted weight loss and palpitations. Her free T 4 is increased, and the radioiodine uptake at 24 hours is 62% (normal, 10%–30%). Vi ew Ans wer 14-16. T he answers are: 14-D [II D 1 c], 15-C [II A 1 a], and 16-A [II C 3 d]. In subacute thyroiditis, injured thyroid follicular cells release thyroid hormone, raising the blood level of
Pa g e 2 1 9 8
ABC Ambe r CHM Conve rte r Tria l ve rsion, http://w w w .proce sste x t.com/a bcchm.html
thyroxine (T 4 ). Radioactive iodine uptake is low, however, because the injured follicular cells are unable to trap iodine normally. Also, thyroid-stimulating hormone (T SH) is suppressed by the increased level of circulating thyroid hormone, and this further reduces the radioactive iodine uptake. In pregnancy, the high estrogen levels cause increased production of T 4 -binding globulin. T his raises the serum level of total T 4 and lowers the triiodothyronine (T 3 ) resin uptake. However, patients remain euthyroid, because the serum free T 4 level remains normal. In Graves' disease, the follicular cells trap increased amounts of iodine and produce increased amounts of thyroid hormone. T herefore, both the radioactive iodine uptake and the serum T 4 level are elevated. T his combination of findings indicates hyperthyroidism, caused either by Graves' disease or by toxic nodular goiter. Directions: T he response options for Items 17-21 are the same. You will be required to select one answer for each item in the set. A. Stimulation of an endocrine gland by autoimmune mechanisms B. Destruction of an endocrine gland by tumor, trauma, or infarction C. Destruction of an endocrine gland by autoimmune mechanisms D. Excessive production of hormone by an endocrine tumor E. Impaired sensitivity of peripheral tissues to normal circulating levels of a hormone For each case, select the primary pathologic process most likely to cause the patient' s disorder. 17. A 19-year-old woman has never had a menstrual period. On examination, it is found that her vagina ends in a blind pouch. She is a normal-appearing young woman, but her karyotype proves to be 46,XY. 18. A 55-year-old man complains of headaches, coarsening facial features, and increasing shoe size. His growth hormone and IGF-1 levels are increased.
Pa g e 2 1 9 9
ABC Ambe r CHM Conve rte r Tria l ve rsion, http://w w w .proce sste x t.com/a bcchm.html
19. A 25-year-old woman has lost 15 pounds recently and is irritable and tremulous. Her free T 4 level and radioiodine uptake are elevated. 20. A 40-year-old man has noticed darkening of his skin in the past year, as well as weakness and generalized aching of his joints. An ACT H test is performed: baseline plasma cortisol is 5 µg/dL, rising to 6 µg/dL 1 hour after ACT H injection. 21. A 38-year-old woman has surgery to remove a pituitary macroadenoma that was causing visual impairment. Postoperatively she developed amenorrhea and fatigue, with low levels of LH, FSH, free T 4 , and cortisol. Vi ew Ans wer 17–21. T he answers are: 17-E [VI A 2 a], 18-D [I A 4 a], 19-A [II C 1 a], 20-C [V C 1 a (1)], and 21-B [I A 2 a]. T he testicular feminization syndrome results from the inability of tissue to respond to testosterone and other androgens. If not stimulated by androgens, the fetal external genitalia develop as female organs. T herefore, a genetic male infant with testes and normal male testosterone levels is born with female external genitalia and is considered to be a normal female. T he usual cause of acromegaly is a growth hormone (GH)–secreting pituitary adenoma. Rarely, GH-releasing hormone production by an islet-cell adenoma may cause acromegaly. Graves' disease is caused by abnormal stimulation of the thyroid gland by thyroid-stimulating immunoglobulin. T his immunoglobulin G (IgG) antibody binds to receptors for thyroid-stimulating hormone (T SH). T his hormone then stimulates growth and hormone production by the thyroid follicular cells. Addison' s disease is most commonly caused by atrophy of the adrenal cortex. Antiadrenal antibodies are often present. Other evidence of autoimmunity such as antibodies against other tissues and the presence of other autoimmune diseases also are common findings. Hemorrhage into the adrenals and infectious
Pa g e 2 2 0 0
ABC Ambe r CHM Conve rte r Tria l ve rsion, http://w w w .proce sste x t.com/a bcchm.html
agents (e.g., tuberculosis) are less common causes of Addison' s disease. Pituitary tumors may compress normal tissue, impairing its function. Surgical removal of the tumor may further damage the hypothalamus and pituitary gland. Ischemic infarction at childbirth (Sheehan' s syndrome) and various destructive, infectious, and granulomatous lesions also cause hypopituitarism. 22. A 21-year-old woman presents to the emergency department complaining of abdominal pain. She states that she has been having increased urination, increased thirst, and a 10-lb weight loss. On examination she has a blood pressure of 80/60 mm Hg, rapid deep breaths, and dry mucous membranes. Which laboratory abnormality fits with the patient' s clinical presentation? A. Low pot as s i um B. Low hemat ocri t C. Ani on gap aci dos i s D. Low s erum os mol al i t y E. El evat ed s odi um Vi ew Ans wer 22. T he answer is C [IV A 6 a]. The pat i ent mos t l i kel y had di abet i c ket oaci dos i s (DKA), a condi t i on i n t ype 1 di abet es charact eri zed by a l ack of i ns ul i n t hat l eads t o hypergl ycemi a and ot her met abol i c derangement s . DKA res ul t s i n ani on gap met abol i c aci dos i s . The l i ver produces more ket one bodi es t han t he body can met abol i z e. The ani on gap refl ect s an el evat i on of t he acet oacet at e and β-hydroxybut yrat e i n t he pl as ma. Serum pot as s i um l evel s may be el evat ed i ni t i al l y, but oft en pat i ent s have a l ow body s t ore and requi re repl et i on. As t he aci dos i s correct s , pot as s i um wi l l s hi ft i nt o cel l s i n res pons e t o i ns ul i n. Pat i ent s may appear t o have an el evat ed hemat ocri t s econdary t o hemoconcent rat i on. Serum os mol al i t y i s oft en el evat ed and s erum s odi um i s di l ut ed s econdary t o os mot i c pul l of t he gl ucos e t hat s hi ft s fl ui d i nt o t he i nt ravas cul ar s pace.
Pa g e 2 2 0 1
ABC Ambe r CHM Conve rte r Tria l ve rsion, http://w w w .proce sste x t.com/a bcchm.html
23. Match the following medications with the appropriate mechanism of action or side effect. Some may be used more than once. A. Met formi n B. Sul fonyl ureas C. Thi az ol i di nedi ones (TZDs ) D. α-Gl ucos i de i nhi bi t ors 1 Increas e i ns ul i n by t he pancreat i c β cel l 2 Decreas e hepat i c gl ucos e out put 3 Si de effect s i ncl ude fl at ul ence, abdomi nal di s comfort , and di arrhea 4 Hypogl ycemi a mi ght res ul t 5 Increas es s ens i t i vi t y of mus cl e and fat t o i ns ul i n 6 Lact i c aci dos i s i s a s eri ous pot ent i al s i de affect Vi ew Ans wer 23. The ans wers are 1. B [IV A 8 c (1)], 2. A [IV A 8 c (2)], 3. D [IV A 8 c (2)], 4. B and C [IV A 8 c (3-4)], 5. C [IV A 8 c (3)], and 6. A [IVA 8 c (2)] Sul fonyl urea deri vat i ve bi nd t o recept ors on t he β cel l i n t he pancreas , i ncreas i ng i ns ul i n s ecret i on. Bi ndi ng of t he β-cel l recept ors cl os es pot as s i um channel s , opens cal ci um channel s , and creat es and i nfl ux of cal ci um i nt o t he cel l . Met formi n decreas es hepat i c gl ucos e out put . Hypogl ycemi a does not occur when met formi n i s us ed al one becaus e i t does not s t i mul at e i ns ul i n s ecret i on. The α-gl ucos i das e i nhi bi t ors del ay abs orpt i on of carbohydrat es unt i l t hey reach t he di s t al s mal l bowel and col on. Thi s res ul t s i n t he common s i de effect s of t hes e drugs : fl at ul ence, abdomi nal di s comfort and di arrhea. Bot h s ul fonyl ureas and t hi azol i di nedi ones can caus e hypogl ycemi a. Sul fonyl ureas s t i mul at e i ns ul i n s ecret i on and t hi azol i di nedi ones i ncreas e s ens i t i vi t y t o i ns ul i n i n t he peri pheral t i s s ue. Thi az ol i di nedi ones s uch as ros i gl i t azone and pi ogl i t azone i ncreas e s ens i t i vi t y t o i n i ns ul i n i n mus cl e and fat . Met formi n i s cont rai ndi cat ed i n pat i ent s wi t h renal fai l ure, l i ver
Pa g e 2 2 0 2
ABC Ambe r CHM Conve rte r Tria l ve rsion, http://w w w .proce sste x t.com/a bcchm.html
di s eas e, and al cohol i s m as i t may pot ent i al l y l ead t o l act i c aci dos i s . There have al s o been cas es of l act i c aci dos i s i n pat i ent s on met formi n aft er admi ni s t rat i on of i nt ravenous cont ras t dye for cert ai n procedures . Thi s may occur i f t he renal funct i on of t he pat i ent decl i nes aft er expos ure t o cont ras t . Met formi n s houl d be hel d 48–72 hours aft er a pat i ent recei ves cont ras t t o prevent t hi s compl i cat i on.
Pa g e 2 2 0 3
ABC Ambe r CHM Conve rte r Tria l ve rsion, http://w w w .proce sste x t.com/a bcchm.html
Editors: Wolfsthal, Susan T itle: NMS Medicine, 6th Edition Copyri ght ©2008 Li ppi ncot t W i l l i ams & W i l ki ns > T able of Cont ent s > Chapt er 10 - Rheumat ic Diseases
Chapter 10
Rheumatic Diseases Rebecca Manno Raymond Flores
I. Approach to the Patient with Joint Pain A. A thorough history and physical examination A thorough history and physical examination are t he corners t one i n t he eval uat i on of pat i ent s wi t h joi nt compl ai nt s (Fi gure 10-1).
1. Step 1. Is t he pai n joi nt -cent ered or l ocal i zed i n t he peri art i cul ar t i s s ues i nvol vi ng mus cl e, nerve, burs a, t endons , or l i gament s ?
2. Step 2. Is t he art hri t i s i nfl ammat ory or noni nfl ammat ory? An i mport ant cl ue i n maki ng t hi s di s t i nct i on i s t he durat i on of morni ng s t i ffnes s . Morni ng s t i ffnes s l as t i ng more t han 1 hour i s s ugges t i ve of an i nfl ammat ory art hri t i s (e.g., rheumat oi d art hri t i s ), whereas s t i ffnes s l as t i ng l es s t han 1 hour i s s ugges t i ve of a noni nfl ammat ory art hri t i s (e.g., os t eoart hri t i s ).
3. Step 3. How many joi nt s are affect ed? W hat i s t he pat t ern of joi nt i nvol vement ? The ans wers t o t hes e ques t i ons al s o provi de i mport ant di agnos t i c cl ues . For exampl e, i f t he pat t ern of joi nt i nvol vement i s as ymmet ri c, i t i s more l i kel y t hat t he pat i ent has a
Pa g e 2 2 0 4
ABC Ambe r CHM Conve rte r Tria l ve rsion, http://w w w .proce sste x t.com/a bcchm.html
s eronegat i ve s pondyl oart hropat hy, es peci al l y i f t here i s as s oci at ed i nfl ammat ory back pai n. If t he pol yart hri t i s i s s ymmet ri c, t he di fferent i al di agnos i s s houl d i ncl ude rheumat oi d art hri t i s , s ys t emi c l upus eryt hemat os us (SLE), pol ymyos i t i s , and s cl eroderma.
B. Laboratory tests (Online Tables 10-1 and 10-2) A vari et y of laboratory tests are hel pful i n maki ng rheumat ol ogi c di agnos es . Some are al s o us eful i n as s es s i ng di s eas e act i vi t y i n i ndi vi dual pat i ent s .
1. Acute phase reactants. A het erogeneous group of prot ei ns (e.g., fi bri nogen) i s s ynt hes i zed i n res pons e t o i nfl ammat i on. o
o
a. Erythrocyte sedimentation rate (ESR). Thi s i ndi rect nons peci fi c meas ure of s ys t emi c i nfl ammat i on i s cal cul at ed by obs ervi ng t he di s t ance i n mi l l i met ers t hat red bl ood cel l s (RBCs ) fal l i n a s peci fi c t ube i n 1 hour. A ri s e i n acut e phas e react ant s i ncreas es t he di el ect ri c cons t ant of pl as ma, res ul t i ng i n t he di s s i pat i on of i nt er-RBC repul s i ve forces . Thi s l eads t o i ncreas ed aggregat i on of RBCs , whi ch caus es t hem t o fal l fas t er, t hus el evat i ng t he eryt hrocyt e s edi ment at i on rat e. Us eful i n t he di agnos i s of t emporal art eri t i s and pol ymyal gi a rheumat i ca (PMR), i t can be hel pful i n as s es s i ng di s eas e act i vi t y i n t hes e condi t i ons as wel l as i n rheumat oi d art hri t i s . If t he ESR i s great er t han 100 mm/hour, i nfect i on and mal i gnancy s houl d be cons i dered i n t he di fferent i al di agnos i s .
o
Pa g e 2 2 0 5
ABC Ambe r CHM Conve rte r Tria l ve rsion, http://w w w .proce sste x t.com/a bcchm.html
o
b. C-reactive protein (CRP). The l i ver produces t hi s pent ameri c prot ei n as an acut e phas e react ant i n res pons e t o i nt erl euki n-6 (IL-6) and ot her cyt oki nes . C-react i ve prot ei n i s more s peci fi c t han t he ESR; i t ri s es and fal l s more qui ckl y. A hi ghl y s ens i t i ve CRP as s ay al s o may refl ect abnormal i t i es t hat l ead t o at herot hrombot i c event s and i s us ed as a predi ct or of cardi ovas cul ar ri s k.
2. Rheumatoid factor. Thi s ant i body t o t he Fc port i on of i mmunogl obul i n G (IgG) can be i n t he IgG or IgA cl as s , but i t i s us ual l y i n t he IgM cl as s . Onl y IgM rheumat oi d fact or i s rout i nel y meas ured by cl i ni cal l aborat ori es . Al t hough i t i s found i n 75%–85% of pat i ent s wi t h rheumat oi d art hri t i s , i t al s o occurs i n ot her condi t i ons charact eri zed by chroni c i mmune s t i mul at i on s uch as i nfect i on, mal i gnancy, and hypergl obul i nemi c s t at es . Rheumat oi d fact or i s al s o pres ent i n s ome normal i ndi vi dual s , t ypi cal l y t hos e ol der t han 70 years of age. Anti-cyclic citrullinated peptide (anti-CCP) i s an ami no aci d deri ved from argi ni ne res i dues and i s pres ent i n i nfl amed rheumat oi d art hri t i s s ynovi um. It has an equi val ent s ens i t i vi t y t o t hat of IgM rheumat oi d fact or (RF) i n di agnos i ng rheumat oi d art hri t i s but i s more s peci fi c and may be more us eful i n t he di agnos i s of earl y rheumat oi d art hri t i s .
3. Antinuclear antibody (ANA). Thes e ant i bodi es , whi ch are di rect ed agai ns t a vari et y of nucl ear ant i gens , are charact eri s t i c of a number of connect i ve t i s s ue di s orders . o
o
a. A pos i t i ve ANA i s found i n 95% of pat i ent s wi t h
Pa g e 2 2 0 6
ABC Ambe r CHM Conve rte r Tria l ve rsion, http://w w w .proce sste x t.com/a bcchm.html
SLE and, t o a l es s er ext ent , i n t hos e wi t h ot her condi t i ons s uch as di s coi d l upus eryt hemat os us , s cl eroderma, Sjögren's s yndrome, rheumat oi d art hri t i s , Raynaud's phenomenon, and vas cul i t i s . Ot her i nfl ammat ory di s orders as s oci at ed wi t h a pos i t i ve ANA i ncl ude chroni c act i ve hepat i t i s , i nt ers t i t i al pul monary fi bros i s , t ubercul os i s , human i mmunodefi ci ency (HIV) di s eas e, and mal i gnancy. Cert ai n medi cat i ons s uch as procai nami de, i s oni azi d, and hydral azi ne have been as s oci at ed wi t h ANA pos i t i vi t y and ant i hi s t one ant i bodi es . ANA s houl d not be performed as a s creeni ng t es t , becaus e i t i s pos i t i ve i n approxi mat el y 5% of t he normal popul at i on (us ual l y i n l ow t i t ers ). o
o
b. A pos i t i ve ANA mus t be i nt erpret ed i n l i ght of t he pat i ent 's age, s ex, and cl i ni cal and drug hi s t ory. The l evel of t he ANA t i t er i s al s o i mport ant ; t i t ers of 1:160 or great er are as s oci at ed wi t h great er cl i ni cal s i gni fi cance t han l ower t i t ers . Speci fi c pat t erns of fl uores cence are as s oci at ed wi t h cert ai n ant i bodi es (Onl i ne Tabl e 10-1).
ONLINE TABLE 10-1 Immunologic Specificity of Individual Antinuclear Antibodies (ANAs)
Pa g e 2 2 0 7
ABC Ambe r CHM Conve rte r Tria l ve rsion, http://w w w .proce sste x t.com/a bcchm.html
A nt ib o d y
Di
R
s
e
e
a
a
ct
s
ivi
e
ty
A
to
ss
A
o
nt P ci ig at at e te io n rn n A Ri S nt m p i b ; ec o h i fi dy o c t o m fo ds o r D g SL N e E, A n us ( e ef m o ul e us fo as
r
ur
m
e
o
d
ni
Pa g e 2 2 0 8
ABC Ambe r CHM Conve rte r Tria l ve rsion, http://w w w .proce sste x t.com/a bcchm.html
by
to
Fa
ri
rr
n
as
g
sa
di
y)
se as e ac ti vi ty , es p ec ia ll y lu p us n e p hr iti
s A H Dr nt o u i b m go o in dy g d t o e uc hi n e
Pa g e 2 2 0 9
ABC Ambe r CHM Conve rte r Tria l ve rsion, http://w w w .proce sste x t.com/a bcchm.html
st e d o o lu n us p es
us , b ut al so m ay b e pr es e nt in SL
E A S S nt p p i b ec ec o kl i fi dy e c t o d fo ex
r
tr
SL
ac
E
ta
b
bl
ut
e
pr
n
es
uc
e
le
nt
Pa g e 2 2 1 0
ABC Ambe r CHM Conve rte r Tria l ve rsion, http://w w w .proce sste x t.com/a bcchm.html
ar
in
a
o
nt
nl
ig
y
e
2
ns
5 %
S
of
m
p
it
at
h
ie nt
R
s
N
U
P
C T
S
D
S
C
A/
o
S
m
S
m
B
o n in Sj à ¶ gr e n' s sy n dr o
Pa g e 2 2 1 1
ABC Ambe r CHM Conve rte r Tria l ve rsion, http://w w w .proce sste x t.com/a bcchm.html
m e a n d SL E S S A as so ci at e d wi th p h ot o se ns iti vi ty S S B as so ci at e
Pa g e 2 2 1 2
ABC Ambe r CHM Conve rte r Tria l ve rsion, http://w w w .proce sste x t.com/a bcchm.html
d wi th n e o n at al lu p us a n d co n g e ni ta l h e ar t bl oc k A C C nt e R i b nt E o ro S dy m T t o er
Pa g e 2 2 1 3
ABC Ambe r CHM Conve rte r Tria l ve rsion, http://w w w .proce sste x t.com/a bcchm.html
ce e nt ro m er e RNP, ri bonu cl eopr ot ei n; UCTD, undi ffe rent i at ed connec t i ve t i s s ue di s eas e; CREST, l i mi t ed s cl erod erma wi t h c al ci nos is, R aynaud 's , e s ophag eal dys mo t i l i t y; s cl erod act yl y,
Pa g e 2 2 1 4
ABC Ambe r CHM Conve rte r Tria l ve rsion, http://w w w .proce sste x t.com/a bcchm.html
and t el angi ect as i a.
4. Complement. A meas urement of act i vi t y (CH50) or ant i gens (C3 or C4) can be us ed as an i ndi rect meas ure of i mmune compl ex format i on. Serum compl ement may be decreas ed as a res ul t of decreas ed product i on (heredi t ary defi ci ency or l i ver di s eas e) or i ncreas ed cons umpt i on [e.g., i n SLE, vas cul i t i s , mi xed cryogl obul i nemi a, s eps i s , or s ubacut e bact eri al endocardi t i s (SBE)]. Typi cal l y, l evel s of C3 and CH50 are l ow i n act i ve SLE. In pat i ent s wi t h angi oedema, a l ow C4 may be a cl ue t o t he di agnos i s of heredi t ary angi oedema (C1 es t eras e i nhi bi t or defi ci ency).
5. Antineutrophil cytoplasmic antibody (ANCA). Thi s ant i body, whi ch i s di rect ed agai ns t s peci fi c prot ei ns i n t he cyt opl as m of neut rophi l s , i s det ect ed by i ndi rect i mmunofl uores cence. Two ANCA pat t erns exi s t : cyt opl as mi c (c-ANCA) charact eri zed by di ffus e s t ai ni ng and peri nucl ear (p-ANCA) wi t h a peri nucl ear pat t ern. The prot ei n recogni zed by t he c-ANCA i s prot ei nas e-3, and t he prot ei n recogni zed by p-ANCA i s mos t commonl y myel operoxi das e. o
Pa g e 2 2 1 5
ABC Ambe r CHM Conve rte r Tria l ve rsion, http://w w w .proce sste x t.com/a bcchm.html o
a. c-ANCA i s s t rongl y as s oci at ed wi t h W egener's granul omat os i s , wi t h a s ens i t i vi t y of 30%–90% and a s peci fi ci t y of 98%. It has been s hown t o be hel pful i n fol l owi ng di s eas e act i vi t y.
o
o
b. p-ANCA i s as s oci at ed wi t h i di opat hi c cres cent i c gl omerul onephri t i s , Churg-St raus s s yndrome, mi cros copi c pol yangi i t i s , pol yart eri t i s nodos a, and ot her i mmunol ogi c di s orders .
6. Antiphospholipid antibody. Det ect i on i n s erum i nvol ves an enz yme-l i nked i mmunos orbent as s ay (ELISA) for IgG or IgM ant i cardi ol i pi n ant i bodi es , pos i t i ve l upus ant i coagul ant , or a fal s e-pos i t i ve Venereal Di s eas e Res earch Laborat ory (VDRL) t es t . A prol onged part i al t hrombopl as t i n t i me (PTT) or prot hrombi n t i me (PT) may be a cl ue t o t he pres ence of ant i phos phol i pi d ant i bodi es . Det ect i on s houl d be furt her eval uat ed by a mi xi ng s t udy, whi ch s houl d not correct wi t h a 1:1 mi x wi t h normal pl as ma. Ot her t es t s for t he l upus ant i coagul ant i ncl ude a di l ut e act i vat ed PTT, t he kaol i n cl ot t i ng t i me, and t he Rus s el l vi per venom t es t . The ant i -β 2 gl ycoprot ei n 1, whi ch can be pos i t i ve i n pat i ent s wi t h cl ot t i ng abnormal i t i es when ot her cl ot t i ng t es t s are normal , i s pos i t i ve i n pat i ent s wi t h ant i phos phol i pi d ant i body s yndrome. Ant i cardi ol i pi n ant i bodi es have al s o been i dent i fi ed i n pat i ent s wi t h i nfect i ons s uch as HIV and may be drug-i nduced, wi t h no i ncreas e i n t he i nci dence of cl ot t i ng.
7. Cryoglobulin. There are t hree major t ypes of cryogl obul i ns , whi ch are i mmunogl obul i ns or i mmunogl obul i n compl exes , t hat s pont aneous l y
Pa g e 2 2 1 6
ABC Ambe r CHM Conve rte r Tria l ve rsion, http://w w w .proce sste x t.com/a bcchm.html
preci pi t at e at l ow t emperat ures . T ype I i s as s oci at ed wi t h a s i ngl e monocl onal i mmunogl obul i n; type II i s as s oci at ed wi t h a mi xed cryogl obul i n wi t h a monocl onal component t hat act s as an ant i body agai ns t pol ycl onal IgG; and type III, al s o as s oci at ed wi t h mi xed cryogl obul i ns , i s more di ffi cul t t o det ect becaus e i t i s us ual l y pres ent i n s mal l quant i t i es and preci pi t at es more s l owl y. Mi xed cryogl obul i ns are frequent l y pres ent i n pat i ent s wi t h underl yi ng connect i ve t i s s ue di s eas es . In addi t i on, t hey have al s o been found i n i nfect i ons , l ymphoprol i ferat i ve di s eas es , l i ver di s eas es [hepat i t i s B vi rus (HBV) and hepat i t i s C vi rus (HCV)], and renal di s eas es s uch as prol i ferat i ve gl omerul onephri t i s .
8. Lyme borreliosis antibody testing. Meas urement us ual l y i nvol ves i ndi rect i mmunofl uores cence or ELISA. Becaus e of t he hi gh frequency of fal s e-pos i t i ve res ul t s , t he W es t ern bl ot as s ay i s ordered as a confi rmat ory t es t .
9. Human leukocyte antigen (HLA). A s t rong as s oci at i on of HLA-B27 has been not ed i n Caucas i an i ndi vi dual s wi t h s eronegat i ve s pondyl oart hropat hi es s uch as ankyl os i ng s pondyl i t i s . Unfort unat el y, HLA-B27 i s found i n 3% of heal t hy Afri can Ameri cans and 8% of heal t hy Caucas i ans ; t herefore, i t s di agnos t i c ut i l i t y i s l i mi t ed. HLA det ermi nat i ons of t he DR l ocus , s peci fi cal l y t he DR4 al l el es t hat mark more s evere di s eas e i n rheumat oi d art hri t i s , are current l y onl y res earch t ool s , but may have broader appl i cat i ons i n t he fut ure.
10. Synovial fluid analysis. Anal ys i s of t he s ynovi al fl ui d i s cri t i cal i n t he eval uat i on of art hri t i s . It i s hel pful
Pa g e 2 2 1 7
ABC Ambe r CHM Conve rte r Tria l ve rsion, http://w w w .proce sste x t.com/a bcchm.html
i n di s t i ngui s hi ng an i nfl ammat ory art hri t i s from a noni nfl ammat ory art hri t i s , and i t i s es s ent i al i n confi rmi ng t he pres ence or abs ence of gout or ps eudogout and i n t he eval uat i on of a pos s i bl e s ept i c joi nt . o
o
a. If onl y a s mal l amount of fl ui d i s obt ai ned, i t s houl d be s ent for Gram s t ai n and cul t ure. The l eukocyt e count can hel p cl as s i fy t he fl ui d i nt o groups (Onl i ne Tabl e 10-2). The fi ndi ng of a hemart hros i s s houl d rai s e s us pi ci on for t rauma, bl eedi ng di at hes i s (e.g., ant i coagul ant us e, hemophi l i a, t umors , or s curvy), and pi gment ed vi l l onodul ar s ynovi t i s .
ONLINE TABLE 10-2 Synovial Fluid Categorization A W
ss
B P o C e ci C rc at o e e u nt d nt a C ( g o C p e n at e of di e r P ti g mM o o mN n 3
ry ) s s N 0 <
Pa g e 2 2 1 8
ABC Ambe r CHM Conve rte r Tria l ve rsion, http://w w w .proce sste x t.com/a bcchm.html
or â 1 m €“ 0 al 2 ( 0 > 0 5 0 m o n oc yt e s) N 2 < O o 0 2 st ni 0 0 e nf â
o
l a €“
ar
m2
th
m0
ri
at 0
ti
or 0
s,
y
tr a u m
a In 2 2 R fl 0 0 A, a 0 â g m 0 €“ o m â 7 ut at €“ 0 , or 5
p
y 0,
s
Pa g e 2 2 1 9
ABC Ambe r CHM Conve rte r Tria l ve rsion, http://w w w .proce sste x t.com/a bcchm.html
0
e
0
u
0
d o g o ut , S L
E P > > S y 5 7 e ar 0, 0 pt th 0
ic
ro 0
ar
si 0
th
s
ri ti s; b ut ca n b e s e e n in g o ut ,
Pa g e 2 2 2 0
ABC Ambe r CHM Conve rte r Tria l ve rsion, http://w w w .proce sste x t.com/a bcchm.html
R A, a n d re ac ti v e ar th ri ti s RA, rheumat oi d art hri t i s , SLE, s ys t em l upus eryt hem at os us . o
o
b. Compens at ed pol ari zi ng mi cros copy i s an i nval uabl e t echni que for exami ni ng s ynovi al fl ui d. Needl e-s haped monos odi um urat e crys t al s are s t rongl y negat i vel y bi refri ngent , appeari ng yel l ow when t hei r l ong axi s i s paral l el t o t hat of t he compens at or. Cal ci um pyrophos phat e di hydrat e (CPPD) crys t al s are us ual l y rhomboi d-s haped wi t h bl unt ends , and t hey appear bl ue when t hei r l ong axi s i s paral l el t o t hat of t he compens at or.
Pa g e 2 2 2 1
ABC Ambe r CHM Conve rte r Tria l ve rsion, http://w w w .proce sste x t.com/a bcchm.html
II. Approach to the Patient with Low Back Pain (Online Figure 10-2 and Online Table 10-3) A. The low back comprises The l ow back compri s es fi ve l umbar vert ebrae, t he s acrum, t he coccyx, and t he i l i ac bones . The l ower nerve root s and l umbar s acral and pudendal pl exus es ari s e from t hes e areas . The s pi nal cord ends at L1, and bel ow t hi s l evel t he cauda equi na fi l l s t he canal down t o t he coccyx. In mos t pat i ent s wi t h l ow back pai n (LBP), t he caus e i s beni gn, and neurol ogi c i mpai rment does not occur. Des pi t e a t horough hi s t ory and phys i cal exami nat i on, an exact anat omi c s t ruct ure res pons i bl e for t he pai n oft en i s not i dent i fi ed; however, t he exami nat i on i s cruci al i n ens uri ng t hat t he caus e i s not s omet hi ng t hat requi res emergent eval uat i on or s peci fi c t reat ment (Onl i ne Fi gure 10-2). Fort unat el y, mos t cas es of acut e LBP i mprove wi t hi n a few days t o a few weeks .
Pa g e 2 2 2 2
ABC Ambe r CHM Conve rte r Tria l ve rsion, http://w w w .proce sste x t.com/a bcchm.html
ONLINE FIGURE 10-2 Approach t o t he pat i ent wi t h l ow back pai n. LBP, l ow back pai n; CBC, compl et e bl ood count ; ESR, eryt hrocyt e s edi ment at i on rat e.
1. Step 1. Is t he back pai n ori gi nat i ng from t he s pi ne and i t s s upport i ng t i s s ues or referred from a di s t ant s i t e? A l i s t of t he mos t common caus e of referred LBP i s s hown i n Onl i ne Tabl e 10-3.
Pa g e 2 2 2 3
ABC Ambe r CHM Conve rte r Tria l ve rsion, http://w w w .proce sste x t.com/a bcchm.html
ONLINE TABLE 10-3 Causes of Referred Low Back Pain Ret rop eri t on eal T um ors — l ym ph om as He ma to ma s †”es pec i al l y in ant i co ag ul a t ed pat i en
Pa g e 2 2 2 4
ABC Ambe r CHM Conve rte r Tria l ve rsion, http://w w w .proce sste x t.com/a bcchm.html
ts F i br os i s Re nal — pye l on ep hri t is, per i ne phr ic abs ces s, an d ne phr ol i t hi a sis Int raab do mi nal P
Pa g e 2 2 2 5
ABC Ambe r CHM Conve rte r Tria l ve rsion, http://w w w .proce sste x t.com/a bcchm.html
anc rea titi s Ch ol e cys titi s P erf ora t ed ul c er Ab do mi nal aor tic an eur ys m Pel vi c di s eas e P ros t at
Pa g e 2 2 2 6
ABC Ambe r CHM Conve rte r Tria l ve rsion, http://w w w .proce sste x t.com/a bcchm.html
eâ €”p ros t at itis , pro sta te can cer Ut e ri n eâ €”c arc i no ma , en do me t ri o sis
2. Step 2. Is t he pai n rel at ed t o t he cauda equina syndrome? Thi s s yndrome i s charact eri zed by LBP, bl adder dys funct i on (uri nary ret ent i on or fl ow i ncont i nence), s addl e anes t hes i a, l os s of s phi nct er t one, and l ower ext remi t y weaknes s . It may be as s oci at ed wi t h a t umor or a l arge cent ral di s c herni at i on and requi res an emergent neurol ogi c cons ul t at i on and magnet i c res onance i magi ng (MRI).
Pa g e 2 2 2 7
ABC Ambe r CHM Conve rte r Tria l ve rsion, http://w w w .proce sste x t.com/a bcchm.html
3. Step 3. Are ot her warning signs pres ent t o s ugges t an underl yi ng s ys t emi c di s eas e? In addi t i on t o t he warni ng s i gns for t he cauda equi na s yndrome, ques t i ons and exami nat i on s houl d be di rect ed wi t h t he cons i derat i on of a pot ent i al underl yi ng mal i gnancy, i nfect i ous proces s , or i nfl ammat ory s pondyl oart hropat hy.
4. Step 4. The pattern of pain radiation i s hel pful i n det ermi ni ng t he et i ol ogy. Mos t pat i ent s wi t h back pai n wi l l have pai n wi t hout radi at i on. The pai n i n t hi s cas e i s oft en referred t o as mus cul ar or l i gament ous “s t rai n,― al t hough a preci s e anat omi c di agnos i s oft en cannot be made. Pai n t hat radi at ed t o t he pos t eri or t hi gh has a di fferent di fferent i al t han pai n t hat radi at es t o t he ant eri or t hi gh, and careful cons i derat i on of t hes e ent i t i es (l i s t ed i n t he fl ow di agram) s houl d be purs ued when t aki ng t he hi s t ory and doi ng t he phys i cal exami nat i on. Pai n t hat radi at es al l t he way down t o t he foot i s mos t l i kel y at t ri but abl e t o a radi cul ar probl em s uch as herni at ed di s c. Oft en i n t hes e pat i ent s , t he foot di s comfort i s more bot hers ome t han t he back pai n.
5. Step 5. If t here i s no evi dence of an underl yi ng s ys t emi c di s eas e, referred pai n, or neurol ogi c compromi s e, t reat ment s houl d be ai med at pai n rel i ef wi t h ant i -i nfl ammat ory medi cat i ons . Narcot i cs s houl d be avoi ded. Dat a have s hown t hat t he addi t i on of mus cl e rel axant s can be us ed i n breaki ng t he pai n-s pas m-pai n cycl e. Bed res t i s not i ndi cat ed beyond t he fi rs t 1 or 2 days . Phys i cal t herapy al s o may be hel pful i n t he acut e and chroni c s et t i ngs .
Pa g e 2 2 2 8
ABC Ambe r CHM Conve rte r Tria l ve rsion, http://w w w .proce sste x t.com/a bcchm.html
6. Step 6. If t he pai n pers i s t s beyond 6 weeks , furt her eval uat i on i s i ndi cat ed. Dependi ng on t he hi s t ory and exami nat i on fi ndi ngs , furt her bl ood work s uch as a compl et e bl ood count (CBC), ESR, and al kal i ne phos phat as e may hel p rul e out s ys t emi c di s eas e or an underl yi ng i nfl ammat ory art hri t i s . Imagi ng s t udi es wi t h pl ai n x-rays may be hel pful but unfort unat el y are not very s ens i t i ve for i dent i fyi ng earl y met as t at i c di s eas e or i nfect i on. At any poi nt , i f t he i ndex of s us pi ci on i s hi gh for underl yi ng pat hol ogy or i f neurol ogi c abnormal i t i es are pres ent , furt her i magi ng s t udi es wi t h CT, MRI, or bone s can are i ndi cat ed. El ect romyogram (EMG) al s o may hel p i n del i neat i ng nerve i njury.
7. Nonorganic back pain i s oft en a di ffi cul t s i t uat i on t o deal wi t h. Red fl ags t hat may al ert t he cl i ni ci an t o t he pres ence of nonorgani c back pai n i ncl ude phys i cal exami nat i on fi ndi ngs t hat are i ncons i s t ent or change when t he pat i ent i s di s t ract ed.
B. Imaging plays an important role in the evaluation process
1. Plain x-rays offer t he advant age of bei ng readi l y avai l abl e and rel at i vel y i nexpens i ve. Ant eropos t eri or and l at eral vi ews are good for eval uat i ng al i gnment , and di s c and vert ebral body hei ght . They al s o can provi de a crude as s es s ment of bone dens i t y. Obl i que vi ews can hel p eval uat e for s pondyl ol ys i s . The major di s advant age i s t he i nabi l i t y t o eval uat e t he s oft t i s s ues , i n addi t i on t o t he fact t hat i t may not reveal any abnormal i t y i n t he earl y s t ages of an i nfect i ous proces s or mal i gnancy. As wi t h mos t of t he i magi ng modal i t i es , pl ai n x-rays may
Pa g e 2 2 2 9
ABC Ambe r CHM Conve rte r Tria l ve rsion, http://w w w .proce sste x t.com/a bcchm.html
i dent i fy abnormal i t i es t hat are not res pons i bl e for s ympt oms .
2. Computed tomography (CT ) scans are good for eval uat i ng t he bony archi t ect ure. They may be hel pful i n eval uat i ng nerve root i mpi ngement and s pi nal s t enos i s , es peci al l y i f rel at ed t o facet degenerat i ve changes .
3. Magnetic resonance imaging (MRI) may be bet t er t han CT for s pi nal i magi ng, gi ven t he bet t er s oft t i s s ue cont ras t and bet t er vi s ual i zat i on of t he l i gament s , vert ebral bone marrow, and t he cont ent s of t he s pi nal canal . One of t he s t rong advant ages i s t he abi l i t y t o det ect earl y met as t at i c di s eas e, and i t i s good at del i neat i ng t he ext ent of an i nfect i on. In addi t i on, MRI can vi s ual i ze t he i nt rat hecal and ext rat hecal nerve root s . Becaus e MRI cannot di rect l y vi s ual i ze bone i t i s l es s us eful i n eval uat i ng an acut e fract ure, es peci al l y of t he pos t eri or el ement s .
4. Bone scan i magi ng may be us eful i n det ect i ng occul t fract ures , i nfect i ons , and bony met as t as i s , and i n di fferent i at i ng t hes e changes from degenerat i ve changes . It al s o may be hel pful i n det ermi ni ng t he acui t y of a s us pect ed compres s i on fract ure. Tri pl e-phas e bone s canni ng i s rout i nel y us ed t o ai d i n t he di agnos i s of os t eomyel i t i s .
C. Common causes of back pain
1. Acute o
Pa g e 2 2 3 0
ABC Ambe r CHM Conve rte r Tria l ve rsion, http://w w w .proce sste x t.com/a bcchm.html
o
a. Strain i s a common di agnos i s gi ven t o acut e back pai n i n t he abs ence of a preci s e anat omi c di agnos i s . It i s us ual l y rel at ed t o overus e. The pai n i s t ypi cal l y l ocal i zed, i nvol vi ng mus cl es or l i gament s .
o
o
b. Disc herniation us ual l y res ul t s from abnormal forces on t he s pi ne. An i nfl ammat ory res pons e wi t h chemi cal i rri t at i on may occur i n addi t i on t o compres s i on of t he nerve root wi t h res ul t ant radi cul opat hy. The pai n i s us ual l y exqui s i t e, ext endi ng from t he back uni l at eral l y down one l eg. Dependi ng on t he exact l ocat i on of t he herni at i on, s l i ght vari at i ons i n l ower ext remi t y pai n, numbnes s , and t i ngl i ng occur.
o
o
c. Vertebral compression fracture us ual l y pres ent s wi t h acut e i nt ens e l ocal i zed pai n wi t h reduced s pi nal mot i on l as t i ng up t o 6 weeks . If t he pai n pers i s t s or i s as s oci at ed wi t h cons t i t ut i onal s ympt oms , furt her eval uat i on for mal i gnancy or myel oma s houl d be cons i dered.
2. Chronic o
o
a. Spinal stenosis i s a narrowi ng of t he neural canal , us ual l y t he res ul t of degenerat i ve changes s uch as bul gi ng di s c and hypert rophi c os t eoart hri t i s of t he facet joi nt s . As t he pat i ent at t empt s t o wal k, t he conges t i on i n t he affect ed regi on of t he s pi nal cord res ul t s i n compres s ed root s and i mpai red nerve funct i on. Sympt oms us ual l y i ncl ude
Pa g e 2 2 3 1
ABC Ambe r CHM Conve rte r Tria l ve rsion, http://w w w .proce sste x t.com/a bcchm.html
progres s i ve ambul at i on di ffi cul t i es wi t h as s oci at ed l eg heavi nes s , numbnes s , and as s oci at ed l eg or back pai n. Lower extremity claudication and t he pres ence of a stooped forward posture when wal ki ng are key fi ndi ngs . The phys i cal exami nat i on i s us ual l y not t hat remarkabl e except for t he abs ences of an S 1 refl ex. Ot her condi t i ons t o excl ude: di abet i c neuropat hy or vas cul ar cl audi cat i on.
(1) Central canal stenosis i s us ual l y t he res ul t of os t eophyt es on t he i nferi or art i cul ar proces s encroachi ng medi al l y or t he res ul t of l i gament um fl avum hypert rophy and annul ar bul gi ng.
(2) Neuroforaminal stenosis occurs when os t eophyt es on t he s uperi or art i cul ar proces s enl arge ant eri orl y and medi al l y, or os t eophyt es form at t he vert ebral margi n, t hereby encroachi ng on t he l at eral nerve root canal .
o
o
b. Degenerative arthritis and discogenic low back pain pres ent wi t h chroni c back pai n t hat i s wors e wi t h prol onged wei ght beari ng or exces s i ve us e. There are t ypi cal l y no radi cul ar fi ndi ngs .
o
o
c. Spondylolisthesis refers t o t he s ubl uxat i on of one vert ebra rel at i ve t o t he i nferi or vert ebra. In many i ns t ances t hi s i s an anat omi c x-ray fi ndi ng not neces s ari l y as s oci at ed wi t h pai n.
Pa g e 2 2 3 2
ABC Ambe r CHM Conve rte r Tria l ve rsion, http://w w w .proce sste x t.com/a bcchm.html o
o
d. Ankylosing spondylitis i s an i nfl ammat ory art hri t i s affect i ng t he axi al s kel et on and s acroi l i ac joi nt s . The mai n feat ure i s i nfl ammat ory back pai n. (See IV).
o
o
e. Discitis. A di s c s pace i nfect i on s houl d be cons i dered i n t he chroni cal l y i l l pat i ent or t he pat i ent wi t h a known s ys t emi c i nfect i on wi t h as s oci at ed back pai n. Prompt t reat ment i s i ndi cat ed t o prevent vert ebral col l aps e and t he format i on of an epi dural abs ces s .
III. Rheumatoid Arthritis A. Definition Rheumat oi d art hri t i s i s a chroni c i mmunol ogi cal l y medi at ed i nfl ammat ory di s order of unknown caus e t hat i s t ypi fi ed by s ynovi al cel l prol i ferat i on and i nfl ammat i on wi t h s ubs equent des t ruct i on of adjacent art i cul ar t i s s ue. The pres ent at i on i s charact eri zed by pol yart i cul ar, s ymmet ri cal joi nt i nvol vement , as wel l as charact eri s t i c ext ra-art i cul ar i nvol vement . Rheumatoid factor and anti-CCP frequent l y are pres ent i n t he s erum of affect ed i ndi vi dual s (s ee onl i ne I B 2).
B. Epidemiology
1. Prevalence and sex distribution. As many as 1% of adul t s may have rheumat oi d art hri t i s , dependi ng on t he cri t eri a us ed for di agnos i s . Cl i ni cal l y meani ngful forms of di s eas e are l es s common—0.5% of women and 0.1% of men have forms of t he i l l nes s t hat requi re ongoi ng t reat ment .
Pa g e 2 2 3 3
ABC Ambe r CHM Conve rte r Tria l ve rsion, http://w w w .proce sste x t.com/a bcchm.html
2. HLA associations. There i s an i ncreas ed preval ence of t he B-cel l al l oant i gen HLA-DR4 i n pat i ent s wi t h rheumat oi d art hri t i s . Evi dence al s o s ugges t s t hat s i mi l ar ami no aci d s equences coded by t he t hi rd hypervari abl e regi on of t he DR β chai n may expl ai n di s eas e as s oci at i on wi t h HLA-DR4, -DR1, -Dw4, -Dw14, and -Dw15. HLA-DR4 pos i t i vi t y al s o i s a marker for more s evere rheumat oi d art hri t i s .
3. Seropositivity for rheumatoid factor. Pat i ent s who have rheumat oi d fact or i n t hei r s erum appear t o have a di fferent i l l nes s from pat i ent s who are s eronegat i ve. Seropos i t i ve pat i ent s t end t o have more s evere di s eas e, more eros i ons , and more ext ra-art i cul ar feat ures . Ant i -CCP i s more s peci fi c t han rheumat oi d fact or and i s more l i kel y t o be pos i t i ve i n earl y RA.
P.452
Pa g e 2 2 3 4
ABC Ambe r CHM Conve rte r Tria l ve rsion, http://w w w .proce sste x t.com/a bcchm.html
FIGURE 10-1 Approach t o t he pat i ent wi t h joi nt pai n. The eval uat i on of chroni c pol yart hri t i s requi res a careful hi s t ory and phys i cal exami nat i on for i dent i fi cat i on of s ys t emi c feat ures t ypi cal of ot her i l l nes s es t hat can caus e art hri t i s . Sel ect ed l aborat ory and radi ographi c t es t i ng al s o can be hel pful . RA, rheumat oi d art hri t i s ; HIV, human i mmunodefi ci ency vi rus ; SLE, s ys t emi c l upus eryt hemat os us .
C. Etiology No s i ngl e fact or or agent i s known t o caus e rheumat oi d art hri t i s . Pres umabl y, an i ni t i al i ns ul t (pos s i bl y i nfect i ous ) i nt eract i ng wi t h t he hos t 's genet i cal l y es t abl i s hed i mmune res pons es det ermi nes whet her an i ni t i al s ynovi t i s i s s uppres s ed or perpet uat ed.
1. Extra-articular agent. The earl i es t i nfl ammat ory changes i n t he rheumat oi d joi nt i nvol ve i nfl ammat i on
Pa g e 2 2 3 5
ABC Ambe r CHM Conve rte r Tria l ve rsion, http://w w w .proce sste x t.com/a bcchm.html
and occl us i on of s mal l s ubs ynovi al ves s el s , s ugges t i ng t hat t he agent i s carri ed i n t he ci rcul at i on t o t he joi nt .
2. Infectious agent. An i nfect i ous et i ol ogy i s s ugges t ed becaus e vi rus -l i ke part i cl es oft en are pres ent i n s ynovi al bi ops i es earl y i n t he di s eas e cours e and becaus e pol yart hri t i s occurs i n as s oci at i on wi t h s everal human and ani mal bact eri al or vi ral i l l nes s es . However, no di rect evi dence of i nfect i on has been di s covered. Symmet ri cal i nfl ammat ory art hri t i s can occur i n pat i ent s who have parvovi rus or rubel l a vi rus i nfect i ons , al t hough t he joi nt fi ndi ngs are not t ypi cal l y pers i s t ent .
3. Genetic factors. A genet i c s us cept i bi l i t y t o al t ered i mmune res pons es probabl y i s i mport ant i n rheumat oi d art hri t i s . There i s no known as s oci at i on of HLA-A or HLA-B hapl ot ypes wi t h t he di s eas e, but a s i gni fi cant as s oci at i on exi s t s bet ween rheumat oi d art hri t i s and t he pres ence of HLA-DR4 and relat ed al l oant i gens of t he major hi s t ocompat i bi l i t y compl ex (MHC). The P.453
pres ence of t hes e and ot her genet i cal l y coded i mmune res pons e al l oant i gens may be i mport ant i n modul at i ng t he hos t 's cel l ul ar and humoral i mmune res pons es t o pot ent i al et i ol ogi c agent s .
4. Effects of Epstein-Barr virus on the immune response. Rheumat oi d art hri t i s pat i ent s have a defect i n t hei r abi l i t y t o regul at e B cel l s i nfect ed wi t h Eps t ei n-Barr vi rus . The vi rus may act as a pol ycl onal act i vat or of B-cel l aut oant i body product i on i n
Pa g e 2 2 3 6
ABC Ambe r CHM Conve rte r Tria l ve rsion, http://w w w .proce sste x t.com/a bcchm.html
rheumat oi d art hri t i s and, as s uch, may pl ay a rol e i n perpet uat i ng (not i ni t i at i ng) t he di s eas e.
D. Pathogenesis (Online Figure 10-3) An unknown et i ol ogi c agent (an exogenous one or an “al t ered†• endogenous one) i ni t i at es a nons peci fi c i mmune res pons e. Mos t evi dence s upport s t he hypot hes i s t hat RA i s a T-cel l –dri ven di s eas e. However, recent evi dence has demons t rat ed t he key rol e of B cel l s as wel l . Not l i s t ed on t he fi gure i s t he product i on of i mmunogl obul i n (RF) from B cel l s and act i vat i on of vas cul ar adhes i on mol ecul es . Once t he T cel l s are act i vat ed, t hey i nfi l t rat e t he s ynovi um, whi ch l eads t o vas cul ar and s ynovi al cel l prol i ferat i on (pannus format i on) and event ual res orpt i on of cart i l age and bone des t ruct i on. Immune-res pons e genes al s o may be i mport ant i n det ermi ni ng t he t ype, i nt ens i t y, and chroni ci t y of t he i mmune res pons e.
Pa g e 2 2 3 7
ABC Ambe r CHM Conve rte r Tria l ve rsion, http://w w w .proce sste x t.com/a bcchm.html
ONLINE FIGURE 10-3 Cyt oki ne s i gnal i ng pat hways i nvol ved wi t h i nfl ammat i on i n rheumat oi d art hri t i s . (Modi fi ed from Choy EH, et al . (2001). Cyt oki ne pat hways and joi nt i nfl ammat i on i n rheumat oi d art hri t i s . N Engl J Med 344: 907–916. )
1. Synovial cell interactions are i mport ant for mai nt enance of art i cul ar i nfl ammat i on. Int ercel l ul ar mes s ages are t rans mi t t ed by cytokines (s mal l prot ei ns t hat can ampl i fy and perpet uat e i nfl ammat i on i n t he rheumat oi d joi nt ). In general , cyt oki nes produced by macrophages and fi brobl as t s [IL-1, IL-6, granul ocyt e macrophage col ony-s t i mul at i ng fact or (GM-CSF), and t umor necros i s fact or-α (TNF-α)] are pres ent at hi gh
Pa g e 2 2 3 8
ABC Ambe r CHM Conve rte r Tria l ve rsion, http://w w w .proce sste x t.com/a bcchm.html
concent rat i ons i n t he rheumat oi d s ynovi um. Lymphokines produced by T cel l s [IL-2, IL-3, IL-4, and i nt erferon-γ (IFN-γ)] are pres ent at rel at i vel y l ow concent rat i ons , apparent l y s uppres s ed by s ubs t ances s ecret ed by macrophages . o
o
a. Macrophage–T cell. Macrophage and hel per +
T-cel l (Th) [CD4 T-cel l ] i nt errel at i ons hi ps are cent ral t o t he ampl i fi cat i on of t he i mmune res pons e. Macrophages proces s ant i gen and pres ent i t (i n as s oci at i on wi t h cl as s II MHC +
mol ecul es ) t o t he CD4 T cel l s , whi ch t hen become act i vat ed by t he i nt eract i on. Cert ai n bact eri al t oxi ns or ret rovi ral prot ei ns can funct i on as superantigens, bi ndi ng t o HLA mol ecul es or T-cel l recept ors di rect l y, pot ent i al l y ampl i fyi ng t he i nfl ammat ory res pons e. o
o
+
b. T h cell–B cell. Act i vat ed CD4 T cel l s s t i mul at e B-cel l prol i ferat i on and di fferent i at i on i nt o ant i body-produci ng cel l s . B cel l s cont ri but e hi ghl y t o i nfl ammat i on and furt her promot e T-cel l act i vat i on, part i al l y t hrough expres s i on of CD20. B cel l s are al s o res pons i bl e for produci ng rheumatoid factor, al t hough t hi s ant i body's exact rol e i n t he pat hogenes i s of RA remai ns uncl ear.
o
o
+
+
c. CD4 T cell–synovial cell. CD4 T cel l s produce s ol ubl e medi at ors (l ymphoki nes ) t hat can modul at e t he funct i on of s ynovi al l i ni ng cel l s , bot h macrophage-l i ke and fi brobl as t -l i ke. The fi brobl as t -l i ke l i ni ng cel l produces col l agenas e and pros t agl andi ns , and s t i mul at es t he growt h of
Pa g e 2 2 3 9
ABC Ambe r CHM Conve rte r Tria l ve rsion, http://w w w .proce sste x t.com/a bcchm.html
connect i ve t i s s ue; al l of t hes e effect s may be i mport ant i n t he des t ruct i ve effect s of t he s ynovi al pannus . o
o
d. Macrophage–endothelial cell. The i ngrowt h of capi l l ari es i s i mport ant t o t he propagat i on of s ynovi t i s and t he l at er growt h of t he pannus . Macrophages s i gnal capi l l ary endot hel i al cel l s t o mi grat e and repl i cat e by hepari n-bi ndi ng growt h fact ors .
2. Synovial fluid phase o
o
a. In cont ras t t o t he mononucl ear res pons e i n t he s ynovi um, t he neutrophil i s t he predomi nant cel l i n rheumat oi d s ynovi al fl ui d i nfl ammat i on. Numerous fact ors chemot act i c for neut rophi l s are pres ent i n t he i nfl amed joi nt [e.g., compl ement fragment s , l eukot ri ene B4 (LTB4), and i mmune compl exes wi t h rheumat oi d fact or]. Thes e neut rophi l s rel eas e oxygen-free radi cal s and hydrol yt i c enzymes t hat can des t roy cart i l age.
o
o
b. Bact eri al , mycobact eri al , and human heat shock proteins (HSPs ) s hare many ant i geni c s equences and may cros s -react wi t h col l agen or prot eogl ycan mol ecul es ; t herefore, an i nfect i on coul d generat e aut oi mmuni t y or l ocal i ze an i nfl ammat ory res pons e t o a joi nt .
3. Rheumatoid factor (s ee onl i ne I B 2). The s ynovi um produces i mmunogl obul i n, mos t of whi ch cons i s t s of IgM
Pa g e 2 2 4 0
ABC Ambe r CHM Conve rte r Tria l ve rsion, http://w w w .proce sste x t.com/a bcchm.html
and IgG rheumat oi d fact ors . Thes e i mmunogl obul i ns form compl exes i n s ynovi al fl ui d, whi ch act i vat e compl ement . Rheumat oi d fact or aggregat es al s o are i nges t ed by macrophages (whi ch s ecret e cyt oki nes ) and neut rophi l s (whi ch rel eas e di ges t i ve enz ymes ); bot h act i ons ampl i fy i nfl ammat i on.
4. Chronic proliferative lesion. A mas s of fi brobl as t i c, vas cul ar, and i nfl ammat ory cel l s (i .e., t he pannus) accumul at es at t he margi n of t he s ynovi al membrane–cart i l age border. The des t ruct i ve capaci t y of rheumat oi d art hri t i s i s as s oci at ed wi t h t he format i on of pannus and t he product i on of monoki nes s uch as TNF, IL-1, and met al l oprot ei nas es . Toget her t hes e abnormal i t i es l ead t o t he des t ruct i on of bone and cart i l age.
5. Joint destruction. Des t ruct i ve change i s unpredi ct abl e, and count eract i ng ant i -i nfl ammat ory cyt oki nes [e.g., t rans formi ng growt h fact or-β (TGF-β)] can downregul at e t he effect s of IL-1 and TNF-α, l eadi ng t o cart i l age repai r and i mmunos uppres s i on. Nat i ve IL-1 i nhi bi t ors perform s i mi l ar funct i ons . Thes e compens at ory mechani s ms oft en are overwhel med, and uni mpeded s ynovi al i nfl ammat i on and prol i ferat i on l ead t o l os s of cart i l age and bone as wel l as anat omi c di s t ort i on. Secondary degenerat i ve joi nt di s eas e res ul t s from t he cont i nued i nfl ammat i on and al t erat i ons i n bi omechani cal joi nt l oadi ng forces .
E. Clinical features
1. Synovitis
Pa g e 2 2 4 1
ABC Ambe r CHM Conve rte r Tria l ve rsion, http://w w w .proce sste x t.com/a bcchm.html o
o
a. Articular involvement. Fai rl y s ymmet ri cal , bi l at eral joi nt i nvol vement i s t ypi cal , oft en s pari ng t he di s t al i nt erphal angeal (DIP) joi nt s of t he hands . Met acarpophal angeal (MCP), proxi mal i nt erphal angeal (PIP), and wri s t joi nt i nvol vement i s s o common as t o be part of t he Ameri can Rheumat i s m As s oci at i on (ARA) revi s ed cri t eri a for di s eas e di agnos i s (Tabl e 10-4).
TABLE 10-4 The 1987 American Rheumatism Association Revised Criteria for the Classification of Rheumatoid Arthritis De Cri fini ter tio ion n 1. Mor Mor ni n ni n g g
stif
s t i f fne fne s s ss in an d aro un d t he joi
Pa g e 2 2 4 2
ABC Ambe r CHM Conve rte r Tria l ve rsion, http://w w w .proce sste x t.com/a bcchm.html
nt s , l as tin g at l ea st 1 ho ur bef ore ma xi m al im pro ve me 2.
nt At
Art l ea hri t s t is
t hr
of
ee
t hr joi ee nt or are mo as re s i m joi ul t nt an are eo as us l
Pa g e 2 2 4 3
ABC Ambe r CHM Conve rte r Tria l ve rsion, http://w w w .proce sste x t.com/a bcchm.html
y hav e ha d s of t tis s ue sw el l i ng or fl ui d (no t bo ny ove rgr ow th al o ne) obs erv ed by a phy s i ci an; t he 14
Pa g e 2 2 4 4
ABC Ambe r CHM Conve rte r Tria l ve rsion, http://w w w .proce sste x t.com/a bcchm.html
pos sib le are as are ri g ht or l eft PIP , MC P, wri st, el b ow, kne e, ank l e, an d MT P joi 3.
nt s At
Art l ea hri t s t is
on
of
e
ha are nd a
Pa g e 2 2 4 5
ABC Ambe r CHM Conve rte r Tria l ve rsion, http://w w w .proce sste x t.com/a bcchm.html
joi s w nt s ol l en (as def i ne d ab ove ) in a wri st, MC P, or PIP joi 4.
nt Si
Sy mu mm l t a et ri ne cal ous art i nv hri t ol v is
em ent of t he sa me joi nt are
Pa g e 2 2 4 6
ABC Ambe r CHM Conve rte r Tria l ve rsion, http://w w w .proce sste x t.com/a bcchm.html
as (as def i ne d in No. 2) on bot h sid es of t he bo dy (bi l at e ral i nv ol v em ent of PIP s, MC Ps , or MT Ps is acc ept
Pa g e 2 2 4 7
ABC Ambe r CHM Conve rte r Tria l ve rsion, http://w w w .proce sste x t.com/a bcchm.html
abl e wi t ho ut abs ol u te sy mm et r 5.
y) Su
Rh bcu eu t an ma eo t oi us d
no
no dul dul es es ove r bo ny pro mi ne nce s, or ext ens or s ur fac
Pa g e 2 2 4 8
ABC Ambe r CHM Conve rte r Tria l ve rsion, http://w w w .proce sste x t.com/a bcchm.html
es , or in jux t aart i cul ar reg i on s, obs erv ed by a phy s i ci 6.
an De
Ser mo um ns t rhe rat i um on at o of id
ab
fac nor t or ma l am ou nt s of s er um
Pa g e 2 2 4 9
ABC Ambe r CHM Conve rte r Tria l ve rsion, http://w w w .proce sste x t.com/a bcchm.html
rhe um at o id fac t or by any me t ho d for whi ch t he res ul t has be en pos itiv e in <5 % of nor ma l con t rol s ub jec ts
Pa g e 2 2 5 0
ABC Ambe r CHM Conve rte r Tria l ve rsion, http://w w w .proce sste x t.com/a bcchm.html
7.
Ra
Ra di o di o gra gra phi phi c c
cha
cha ng ng es es t yp i cal of rhe um at o id art hri t is on pos t er oa nt e ri or ha nd an d wri st rad i og rap hs , whi
Pa g e 2 2 5 1
ABC Ambe r CHM Conve rte r Tria l ve rsion, http://w w w .proce sste x t.com/a bcchm.html
ch mu st i ncl ud e ero sio ns or un eq ui v oca l bo ny dec al ci fi c at i on l oc al i z ed in or mo st ma rke d adj ace nt
Pa g e 2 2 5 2
ABC Ambe r CHM Conve rte r Tria l ve rsion, http://w w w .proce sste x t.com/a bcchm.html
to t he i nv ol v ed joi nt s (os t eo art hri t is cha ng es al o ne do not qu al i f y) For cl as s i fi cat i on purpos es , a pat i en t is s ai d t o have rheum at oi d art hri t i s i f he
Pa g e 2 2 5 3
ABC Ambe r CHM Conve rte r Tria l ve rsion, http://w w w .proce sste x t.com/a bcchm.html
or s he has s at i s fi ed at l eas t four of t hes e s even cri t eri a . Cri t eri a 1 t hroug h 4 mus t have been pres en t for at l eas t 6 weeks . Pat i en t s wi t h t wo cl i ni cal di agno s es are not excl ud ed. Des i gn at i on as
Pa g e 2 2 5 4
ABC Ambe r CHM Conve rte r Tria l ve rsion, http://w w w .proce sste x t.com/a bcchm.html
cl as s i c , defi ni t e, or probab le rheum at oi d art hri t i s is not t o be made. PIP, proxi m al i nt erph al ange al ; MCP, met ac arpoph al ange al ; MTP, met at a rs opha l angea l. (Repri nt ed from Arnet t FC, Edwort
Pa g e 2 2 5 5
ABC Ambe r CHM Conve rte r Tria l ve rsion, http://w w w .proce sste x t.com/a bcchm.html
h SM, Bl och DA, et al : Ameri c an Rheum at i s m As s oci at i on 1987 revi s e d cri t eri a for t he cl as s i fi cat i on of rheum at oi d art hri t i s. Art hri t i s Rheum 1988;3 1:315. ) o
o
b. T endon and ligament involvement. Synovi al l i ni ngs out s i de joi nt s can be i nvol ved as wel l .
(1) Palmar flexor tendinitis can caus e carpal
Pa g e 2 2 5 6
ABC Ambe r CHM Conve rte r Tria l ve rsion, http://w w w .proce sste x t.com/a bcchm.html
t unnel s yndrome.
(2) Rotator cuff tendinitis can caus e s houl der pai n and l i mi t at i on of mot i on.
(3) Atlantoaxial ligament involvement i n t he cervi cal s pi ne can l ead t o i ns t abi l i t y bet ween t he C1 and C2 vert ebrae and pot ent i al neurol ogi c compl ai nt s .
2. Extra-articular features (Tabl e 10-5) more oft en
exi s t i n pat i ent s who are s eropos i t i ve for rheumat oi d fact or and pat i ent s who have more s evere and es t abl i s hed di s eas e.
TABLE 10-5 Extra-articular Features of Rheumatoid Arthritis Ski n No dul es (20 %â €“2 5% of pat i en ts) Va s cu
Pa g e 2 2 5 7
ABC Ambe r CHM Conve rte r Tria l ve rsion, http://w w w .proce sste x t.com/a bcchm.html
liti s (pu rpu ra) Ey e Si c ca co mp l ex (10 %â €“1 5% of pat i en ts) Epi s cl eri t is Scl eri t is He art Per i ca rdi t is My oca
Pa g e 2 2 5 8
ABC Ambe r CHM Conve rte r Tria l ve rsion, http://w w w .proce sste x t.com/a bcchm.html
rdi t is (ra re) Val ve dys fun ct i on (ra re) Lu ng Pl e ura l eff us i on Int ers titi al fi br os i s No dul es Ne rve Ent rap me
Pa g e 2 2 5 9
ABC Ambe r CHM Conve rte r Tria l ve rsion, http://w w w .proce sste x t.com/a bcchm.html
nt (ca rpa l t un nel s yn dro me ) Va s cu liti s Di s t al s en s or y ne uro pat hy Mo no ne uri t is Blo od An em ia
Pa g e 2 2 6 0
ABC Ambe r CHM Conve rte r Tria l ve rsion, http://w w w .proce sste x t.com/a bcchm.html
of chr oni c di s eas e Thr om boc yt o sis Fel t y' s s yn dro me Me tab olis m Am yl o i do sis Ve sse ls Va s cu liti s S ki n
Pa g e 2 2 6 1
ABC Ambe r CHM Conve rte r Tria l ve rsion, http://w w w .proce sste x t.com/a bcchm.html
Ner ve Vi s cer a (ra re) o
o
a. Rheumatoid nodules are t he mos t common feat ures of ext ra-art i cul ar di s eas e and are found i n 20%–25% of pat i ent s . Thes e fi rm, s ubcut aneous mas s es t ypi cal l y are found i n areas of repet i t i ve t rauma (e.g., t he ext ens or s urfaces of t he forearm), al t hough t hey al s o can appear i n t he vi s cera (e.g., l ungs ). P.454
o
o
b. Eye involvement al s o i s common. Keratoconjunctivitis sicca i s s een i n 10%–15% of rheumat oi d art hri t i s pat i ent s who have a s econdary form of Sjögren's s yndrome (s ee XI). The oft en s ubt l e i nfl ammat i on of s cl eri t i s or epi s cl eri t i s occurs l es s commonl y.
o
o
c. Other organ involvement i s not ed i n Tabl e 10-5.
F. Diagnosis
Pa g e 2 2 6 2
ABC Amber CHM Converter Trial version, http://www.processtext.com/abcchm.html
Rheumat oi d art hri t i s i s a s us t ai ned, i nfl ammat ory pol yart hri t i s t hat t ypi cal l y i s s ymmet ri cal i n di s t ri but i on. It i s a di agnos i s of excl us i on of ot her forms of pol yart hri t i s , whi ch i t may i mi t at e. The pat i ent mus t have art hri t i s for at l eas t 6 weeks t o el i mi nat e vi ral s yndromes or ot her caus es of nons us t ai ned pol yart hri t i s . Fi ndi ng rheumat oi d fact or i n t he s erum i s us eful i n pat i ent s who have ot her feat ures of i nfl ammat ory pol yart hri t i s , but as many as 40% of pat i ent s wi t h rheumat oi d art hri t i s do not have t hi s marker i ni t i al l y. Ant i -CCP ant i body may be pres ent earl i er and i s more s peci fi c t han rheumat oi d fact or.
1. History. Pat i ent s wi t h rheumat oi d art hri t i s oft en have prolonged (>1 hour) morning stiffness. Pain in involved joints t ypi cal l y i s wors e i n t he morni ng. Constitutional complaints (wei ght l os s , anorexi a, and fat i gue) are common.
2. Physical examination. Cl as s i cal l y i nvol ved joi nt s are t he wrists and t he MCP and PIP joints of the hand; DIP joi nt s are t ypi cal l y s pared, as i s t he axi al s kel et on except for t he cervi cal s pi ne. Soft tissue swelling, rat her t han bony enl argement , i s t ypi cal around i nvol ved joi nt s , unl es s s econdary degenerat i ve changes have occurred; l i mi t at i on of joi nt mot i on and warmt h may be not ed. Rheumatoid nodules oft en are pres ent i n hi ghl y expres s ed di s eas e; t hey can be found over ext ens or promi nences , es peci al l y near t he ol ecranon. (
Onl i ne Fi gure 10-4
Page 2263
ABC Ambe r CHM Conve rte r Tria l ve rsion, http://w w w .proce sste x t.com/a bcchm.html
ONLINE FIGURE 10-4 Rheumat oi d art hri t i s . Severe ul nar devi at i on and met acarpophal angeal joi nt s ubl uxat i on i s charact eri s t i c of RA. (ACR Sl i de Col l ect i on of t he Rheumat i c Di s eas es , 3rd ed., 2004:9105040.)
3. Laboratory findings. The compl et e bl ood count (CBC) may reveal a normocytic, normochromic anemia of chroni c di s eas e, l eukocyt os i s , and t hrombocyt os i s . Thes e fi ndi ngs al ong wi t h an i ncreas ed ESR refl ect chroni c i nfl ammat i on. Rheumatoid factor and anti-CCP antibody (s ee onl i ne I B 2). Synovi al fl ui d fi ndi ngs refl ect mi l d-t o-moderat e i nfl ammat i on; l eukocyt e count s 3
are 5000–25,000/mm and cons i s t mai nl y of neut rophi l s .
4. Radiographic findings. Earl y charact eri s t i cs i ncl ude soft tissue swelling and l os s of bone i n peri art i cul ar areas (periarticular osteopenia). Si gns of s us t ai ned i nfl ammat i on i ncl ude l os s of bone at joi nt margi ns (erosions) and joint space narrowing as a res ul t of cart i l age l os s .
Onl i ne Fi gure 10-5
Pa g e 2 2 6 4
ABC Ambe r CHM Conve rte r Tria l ve rsion, http://w w w .proce sste x t.com/a bcchm.html
ONLINE FIGURE 10-5 Rheumat oi d art hri t i s . Thi s x-ray of t he wri s t s hows advanced res orpt i on of t he di s t al ul na and di ffus e des t ruct i ve art hri t i s of t he carpus . Thi s i s t ypi cal of s evere rheumat oi d art hri t i s . (ACR Sl i de Col l ect i on of t he Rheumat i c Di s eas es , 3rd ed., 2004:3505070.)
5. Differential diagnosis (s ee Fi gure 10-1). Becaus e rheumat oi d art hri t i s i s one of many i l l nes s es charact eri zed by chroni c pol yart i cul ar i nfl ammat i on, di agnos i s rel i es on excl udi ng ot her s uch P.455
i l l nes s es and s earchi ng for s ymmet ri cal peri art i cul ar s oft t i s s ue s wel l i ng and i nfl ammat ory charact eri s t i cs of rheumat oi d art hri t i s . o
o
a. Nonarticular disorders. Fi bromyal gi a i s a s yndrome of general i zed achi ng and t endernes s i n
Pa g e 2 2 6 5
ABC Ambe r CHM Conve rte r Tria l ve rsion, http://w w w .proce sste x t.com/a bcchm.html
s peci fi c s oft t i s s ue areas , wi t hout joi nt i nvol vement or i nfl ammat i on. Tendon, neurol ogi c, and vas cul ar compl ai nt s al s o may mi mi c joi nt pai n. o
o
b. Noninflammatory disorders
(1) Osteoarthritis us ual l y caus es bony rat her t han s oft t i s s ue s wel l i ng, and t he i nvol ved joi nt s t ypi cal l y are t he DIP and PIP joi nt s of t he hand, t he hi ps , and t he knees . The l umbar and cervi cal s pi ne can be i nvol ved as wel l . Cons t i t ut i onal and i nfl ammat ory compl ai nt s are abs ent , and s ynovi al fl ui d l eukocyt e 3
count s are l es s t han 2000/mm .
(2) Metabolic disorders (e.g., CPPD, hemochromat os i s , and W i l s on's di s eas e) caus e bony degenerat i ve change i n at ypi cal joi nt s (e.g., MCP joi nt s ).
o
o
c. Axial joint inflammation. Infl ammat i on of t he axi al s pi ne (es peci al l y t he s acroi l i ac joi nt s ) i s charact eri s t i c of t he s pondyl oart hropat hi es , and i nfl ammat ory back pai n due t o sacroiliitis s houl d be s ought . Infl ammat ory back pai n i s i ns i di ous , day-aft er-day pai n s t art i ng i n t he s acroi l i ac area and t ypi cal l y as s oci at ed wi t h prol onged morni ng s t i ffnes s . It i s wors ened wi t h res t and i mproved by exerci s e, t he oppos i t e of mechani cal l ow back pai n. The abs ence of s acroi l i ac joi nt i nvol vement does not rul e out t hes e di s orders , but i t s pres ence makes s pondyl oart hropat hy l i kel y.
o
Pa g e 2 2 6 6
ABC Ambe r CHM Conve rte r Tria l ve rsion, http://w w w .proce sste x t.com/a bcchm.html
o
d. Oligoarticular presentations. Cert ai n i l l nes s es mus t be cons i dered more s t rongl y when t he i ni t i al i nfl ammat ory pres ent at i on i nvol ves four or fewer joi nt s and i s as ymmet ri cal . Thes e di s orders i ncl ude crys t al di s eas es , i nfect i ous art hri t i s (e.g., Lyme di s eas e, gonococcemi a, endocardi t i s , and rheumat i c fever), and s pondyl oart hropat hi es (e.g., react i ve art hri t i s , ps ori at i c art hri t i s ).
o
o
e. Polyarticular presentations. It al s o i s i mport ant t o cons i der i nfl ammat ory di s orders t hat i ni t i al l y i nvol ve four or more joi nt s and are fai rl y s ymmet ri cal . Al t hough rheumat oi d art hri t i s i s t he prot ot ype, many ot her i l l nes s es mus t be di s t i ngui s hed, bas ed on cl i ni cal feat ures or organ i nvol vement not t ypi cal of rheumat oi d art hri t i s . Det ai l ed hi s t ory and phys i cal exami nat i on wi t h bas i c l aborat ory dat a are cri t i cal i n di s t i ngui s hi ng among di s orders t hat feat ure pol yart hri t i s .
(1) Other rheumatic diseases (e.g., l upus , s cl eroderma, pol ymyos i t i s /dermat omyos i t i s , PMR, and vas cul i t i s ) are di s t i ngui s hed by t he feat ures of t he pri mary i l l nes s .
(2) Viral disorders (e.g., rubel l a, HBV, and parvovi rus i nfect i on) are di s t i ngui s hed by a t ypi cal ras h, s erol ogi c markers , or organ i nvol vement .
(3) Malignancies may mani fes t as l ong bone pai n, di gi t al cl ubbi ng, and peri os t i t i s
Pa g e 2 2 6 7
ABC Ambe r CHM Conve rte r Tria l ve rsion, http://w w w .proce sste x t.com/a bcchm.html
mi mi cki ng pol yart hri t i s (hypert rophi c os t eoart hropat hy) or as paraneopl as t i c pol yart hri t i s .
(4) Sarcoidosis exhi bi t s medi as t i nal adenopat hy on ches t radi ograph and us ual l y eryt hema nodos um when i t i ncl udes a pol yart hri t i s .
(5) Amyloidosis i s as s oci at ed wi t h Congo red–pos i t i ve depos i t s i n t ypi cal organs , s ubcut aneous t i s s ue, and joi nt s .
G. Therapy In al l pat i ent s wi t h rheumat oi d art hri t i s , an at t empt i s made t o cont rol pai n and reduce i nfl ammat i on wi t hout caus i ng undes i rabl e s i de effect s . Pres ervat i on of joi nt funct i on and t he abi l i t y t o mai nt ai n l i fes t yl e are i mport ant l ong-t erm goal s .
1. Nonpharmacologic therapy o
o
a. Patient education. Educat i ng pat i ent s about t he di s eas e proces s i s part i cul arl y i mport ant i n chroni c di s eas es s uch as rheumat oi d art hri t i s , i n whi ch compl i ance wi t h i ns t ruct i ons and drug t reat ment i s cri t i cal t o t he out come.
(1) Description of the illness. The vari ous di s eas e cours es of rheumat oi d art hri t i s mus t be des cri bed, emphas i zi ng t hat mos t pat i ent s do wel l i f t hey are t reat ed appropri at el y. The chroni ci t y and i nt ermi t t ency of s ympt oms
Pa g e 2 2 6 8
ABC Ambe r CHM Conve rte r Tria l ve rsion, http://w w w .proce sste x t.com/a bcchm.html
mus t be di s cus s ed s o t hat pat i ent s unders t and t hat s pont aneous fl uct uat i ons i n an ext ended di s eas e cours e are normal . Pat i ent s mus t be educat ed about t he s ys t emi c nat ure of t he di s eas e proces s , s o t hat bot h t hey and t hei r fami l i es unders t and t hat fat i gue, mal ai s e, and wei ght l os s oft en accompany t hi s i l l nes s .
(2) Rest and exercise. Pat i ent s s houl d be advi s ed t o res t or s pl i nt acut el y i nvol ved joi nt s t o reduce i nfl ammat i on. Bri ef peri ods of bed res t may be us eful i n pat i ent s wi t h s evere P.456
pol yart i cul ar exacerbat i ons , and regul ar naps may hel p pat i ent s deal wi t h t he fat i gue of rheumat oi d art hri t i s . Convers el y, exerci s es t o s t rengt hen mus cl es s urroundi ng i nvol ved joi nt s s houl d be encouraged when t he art hri t i s i s under good cont rol . Al l joi nt s s houl d be put t hrough a ful l range of mot i on once dai l y t o prevent cont ract ures . o
o
b. Physical medicine
(1) Pat i ent s may benefi t from coordi nat i on of t hei r nonpharmacol ogi c t reat ment by physiatrists.
(2) Physical therapists can hel p pat i ent s s t rengt hen weakened mus cl e groups t o prot ect
Pa g e 2 2 6 9
ABC Ambe r CHM Conve rte r Tria l ve rsion, http://w w w .proce sste x t.com/a bcchm.html
damaged joi nt s . They can s how pat i ent s range-of-mot i on exerci s es t hat prevent joi nt cont ract ures .
(3) Occupational therapists can hel p pat i ent s obt ai n devi ces t o as s i s t t hem, can cons t ruct s pl i nt s for i nvol ved joi nt s , and can ai d i n rehabi l i t at i ng pat i ent s for act i vi t i es of dai l y l i vi ng and empl oyment .
2. Pharmacologic therapy. Nons t eroi dal ant i -i nfl ammat ory drugs (NSAIDs ) and cort i cos t eroi ds are oft en us ed t o provi de rel at i vel y prompt cont rol of pai n and i nfl ammat i on, but t hes e drugs do not al t er di s eas e progres s i on. Al t hough t he cours e of rheumat oi d art hri t i s can be vari abl e, mos t pat i ent s undergo a rel ent l es s progres s i ve cours e requi ri ng t he us e of di s eas e-modi fyi ng ant i rheumat i c drugs (DMARDs ). Cort i cos t eroi ds , whi ch are not vi ewed as fi rs t - or s econd-l i ne t herapi es , oft en are us ed i nt ra-art i cul arl y for di s eas e fl are-ups or oral l y t o hel p pat i ent s who are wai t i ng for a DMARD t o t ake effect . o
o
a. NSAIDs. As pi ri n i s t he prot ot ypi c drug of t hi s cl as s . Nonacet yl at ed s al i cyl at es al s o have been devel oped, whi ch caus e l es s s uppres s i on of pros t agl andi n s ynt hes i s .
(1) Mechanism of action. The pri mary mechani s m of act i on i s t he i nhi bi t i on of cycl ooxygenas e, wi t h a res ul t ant decreas e i n pros t agl andi n product i on. More recent dat a have s hown t hat cycl ooxygenas e exi s t s i n t wo
Pa g e 2 2 7 0
ABC Ambe r CHM Conve rte r Tria l ve rsion, http://w w w .proce sste x t.com/a bcchm.html
i s oforms : COX-1 and COX-2. COX-1 i s expres s ed cons t i t ut i vel y i n monocyt es /macrophages , t he cent ral nervous s ys t em (CNS), gas t ri c mucos a, ki dneys , and pl at el et s , where i t i s res pons i bl e for many of t he “hous ekeepi ng― act i vi t i es . However, COX-2 i s t i ght l y regul at ed and produced duri ng i nfl ammat i on. Mos t of t he avai l abl e t radi t i onal NSAIDs i nhi bi t bot h COX-1 and COX-2.
(2) Use. NSAIDs are us ed t o cont rol pai n and i nfl ammat i on by t he mechani s ms expl ai ned previ ous l y. Mos t pat i ent s us e t hem i n combi nat i on wi t h DMARDs .
(3) T oxicity. Becaus e t he majori t y of NSAIDs i nhi bi t bot h COX-1 and COX-2, t hey are more l i kel y t o produce gas t roi nt es t i nal ul cerat i on. Typi cal t oxi ci t i es i ncl ude dys peps i a, pept i c ul cers (pri mari l y of t he s t omach), hypert ens i on, renal dys funct i on, and bl eedi ng. Some pat i ent s are pl aced on mi s opros t ol or omeprazol e t o reduce t he ri s k of ul cers . Cl i ni cal hepat i t i s and bone marrow t oxi ci t y are very rare. The COX-2 agent s have been s hown t o have a l ower i nci dence of pept i c ul cerat i on and do not i nhi bi t pl at el et funct i on.
(4) COX-2 inhibitors. Cel ecoxi b i s a COX-2–s peci fi c i nhi bi t or. Thi s agent i s purport ed t o have a l ower i nci dence of gas t roi nt es t i nal ul cerat i on compared wi t h
Pa g e 2 2 7 1
ABC Ambe r CHM Conve rte r Tria l ve rsion, http://w w w .proce sste x t.com/a bcchm.html
t radi t i onal NSAIDs , and, becaus e of t he l ack of effect on pl at el et s , can be us ed i n pat i ent s on ant i coagul at i on. Cel ecoxi b mus t be us ed wi t h caut i on i n pat i ent s wi t h renal i ns uffi ci ency and i t al s o may caus e i ncreas ed fl ui d ret ent i on. Recent dat a have s ugges t ed an i ncreas ed cardi ovas cul ar mort al i t y i n pat i ent s recei vi ng rofecoxi b and val decoxi b, l eadi ng t o t he wi t hdrawal of t hes e agent s from t he market by t he U.S. Food and Drug Admi ni s t rat i on (FDA). Sul fa-al l ergi c pat i ent s s houl d not t ake cel ecoxi b. o
o
b. Corticosteroids. Thes e drugs have pot ent ant i -i nfl ammat ory effect s but equal l y pot ent and predi ct abl e t oxi ci t i es . They are us ed mos t commonl y i n rheumat oi d art hri t i s t o cont rol s eri ous ext ra-art i cul ar mani fes t at i ons (e.g., vas cul i t i s ).
(1) Systemic administration. In rare s i t uat i ons s uch as s evere progres s i ve di s eas e, predni s one dos es no hi gher t han 5–10 mg once dai l y i n t he morni ng may be us ed t o al l ow cont i nued funct i oni ng. Cont i nual at t empt s t o t aper t he dos age s houl d be made.
(2) Local instillation. Inject abl e cort i cos t eroi d preparat i ons can be i ns t i l l ed i nt o one or t wo joi nt s i nfl amed “out of phas e― wi t h ot her i nvol ved joi nt s . Thes e i nject i ons s houl d be performed onl y occas i onal l y, becaus e cart i l age l os s may res ul t from frequent i nject i ons i nt o t he s ame joi nt .
Pa g e 2 2 7 2
ABC Ambe r CHM Conve rte r Tria l ve rsion, http://w w w .proce sste x t.com/a bcchm.html
P.457
o
c. T raditional DMARDs. Di s eas e-modi fyi ng
o
ant i rheumat i c drugs are cri t i cal i n t he armament ari um i n t he t reat ment of rheumat oi d art hri t i s . Thes e agent s need t o be s t art ed earl y i n t he cours e of di s eas e (i deal l y wi t hi n 3 mont hs ). The hal l mark of t hes e agent s i s t hei r abi l i t y t o hal t progres s i on of di s eas e, s uch as t he devel opment of eros i ons . Commonl y us ed DMARDs i n t he t reat ment of rheumat oi d art hri t i s are l i s t ed i n Tabl e 10-6. Al t hough s i de effect s of many of t hes e medi cat i ons may be s eri ous , i n mos t i ns t ances t hey are mi l d, predi ct abl e, and can be managed. In mos t cas es , t he ri s k of ongoi ng progres s i ve di s eas e out wei ghs t he pot ent i al ri s k of medi cat i on t oxi ci t y. Each of t he agent s may be us ed al one or i n combi nat i on.
TABLE 10-6 DMARDS Used in the Treatment of Rheumatoid Arthritis M Si e d c e h Ef a fe M ni ct o s s ni A m / to g of P ri e A ot n nt ct e g
Pa g e 2 2 7 3
ABC Ambe r CHM Conve rte r Tria l ve rsion, http://w w w .proce sste x t.com/a bcchm.html
nt ia l T o xi io ci n ty H U M E y n ac y d k ul e r n ar e o o d x x w a a y n, m m c b a in hl ut g at o li e io r k
n
o el
e
q y
v
ui i n
er
n hi
y
e bi
6
s ts
m
ul l y
o
fa s
nt
te o
h
s
s
o
to
m
1
al
y
e
e
n
ar
zy
Pa g e 2 2 7 4
ABC Ambe r CHM Conve rte r Tria l ve rsion, http://w w w .proce sste x t.com/a bcchm.html
m e s a n d m ac ro p h a g e fu nc ti o n M In M C et hi y B h bi el C, ot t s o A r di s S e h u T, x y p A at dr pr L e of e T, ol s s cr at i o e e n, at re h i n d e in uc p e, t a at al
Pa g e 2 2 7 5
ABC Ambe r CHM Conve rte r Tria l ve rsion, http://w w w .proce sste x t.com/a bcchm.html
s ic b e, fi u t h br m er o i n e si e b s/ v y ci er i n rr y hi h m bi o o t i s i nt n s, h g p fo p ul r ur m 6 in o m e n o ( ar nt D y h N in s A) fl t h sya e nt m n h me e at v s i i o er s n/ y fi 6 br â o €“ si 8 s, w st e o e m ks
Pa g e 2 2 7 6
ABC Ambe r CHM Conve rte r Tria l ve rsion, http://w w w .proce sste x t.com/a bcchm.html
at . iti B s a s el in e C X R a n d h e p at iti s pr of il e L In Di S ef hi ar a lu bi rh m n ts e e o di a, a m h el s id y e M e dr v T o- at X or e b ot d ut at l i n
Pa g e 2 2 7 7
ABC Ambe r CHM Conve rte r Tria l ve rsion, http://w w w .proce sste x t.com/a bcchm.html
e v o d er b e e a h n s y z y el dr m i n o e e g s, C e al X n o R a p s ec e, i a th , er t e e ra b to y g in e hi ni bi ci ti ty n re g q p ui yr ri i
n
mg i d el in i e m ( in D at N io A) n
Pa g e 2 2 7 8
ABC Ambe r CHM Conve rte r Tria l ve rsion, http://w w w .proce sste x t.com/a bcchm.html
s y pr nt ot h oc e ol s i wi s th ch ol e st yr a m in e S In G C ul hi a B fa bi s t C s t s ro e al pr i n v a o t e er zi s t s t y n a in 2 e gl al â a i n €“ n to 4 di l e w n ra e s nc e a e, ks n h fo d e r ch m fi r e at s t m ol 3
Pa g e 2 2 7 9
ABC Ambe r CHM Conve rte r Tria l ve rsion, http://w w w .proce sste x t.com/a bcchm.html
ot o m a gi o xi c nt s cy h to s p th e e ni n a e s, v h er e y p 3 at m ot o o nt xi h ci s ty B , a co s nt el ra i n in e di G ca 6 te P d D if te s st ul i n fa g al le rg
Pa g e 2 2 8 0
ABC Ambe r CHM Conve rte r Tria l ve rsion, http://w w w .proce sste x t.com/a bcchm.html
ic A A M C z pr y B at o el C hi dr o e o u s v p g u er ri co p y n n pr 1 e v e â er s s €“ te io 2 d n, w to h e 6- e e m p ks er at wi ca ot t h pt o ch o xi a p ci n ur t y g in , e e ly s (6 m i n M p d P) h o s o s u pr e b ol a s if n e er d q at e u iv v e e er
Pa g e 2 2 8 1
ABC Ambe r CHM Conve rte r Tria l ve rsion, http://w w w .proce sste x t.com/a bcchm.html
nt di y ly s 1 co or â n d €“ v er 3 er s ; m t e ca o d ut nt in io h to n s t h wi t h i o t h er p co e ur nc af i n ur t e e re r n nt uc u le s ot e i d of e al s lo d p ec ur re i n a ol si n g d e n o v
Pa g e 2 2 8 2
ABC Ambe r CHM Conve rte r Tria l ve rsion, http://w w w .proce sste x t.com/a bcchm.html
o sy nt h e si s of p ur in e s C In R Cr y hi e e cl bi n at o t s al i n s T- i n i n p ce s e o l l uf e ri ac fi v n t i ci er e v e y at nc 2 i o y, w n a e n e e ks mu i a nt , il h st y a p bl er e
Pa g e 2 2 8 3
ABC Ambe r CHM Conve rte r Tria l ve rsion, http://w w w .proce sste x t.com/a bcchm.html
te th n e si n o m n o h nt y hl p y, er p t r er ic io h di o c si C s, B p C, ar LF e T st s h e si a s, gi n gi v al h y p er tr o
Pa g e 2 2 8 4
ABC Ambe r CHM Conve rte r Tria l ve rsion, http://w w w .proce sste x t.com/a bcchm.html
p h y; ca ut io n â €” m ul ti pl e m e di ca ti o n in te ra ct io n s DMARDs , di s eas emodi fyi n g ant i rheu mat i c drugs ;
Pa g e 2 2 8 5
ABC Ambe r CHM Conve rte r Tria l ve rsion, http://w w w .proce sste x t.com/a bcchm.html
ALT. al ani ne ami not ra ns feras e ; AST, as part at e ami not ra ns feras e ; CBC, compl et e bl ood count ; CXR, ches t x-ray; LFTs , l i ver funct i on t es t s ; MTX, met hot re xat e.
(1) Monotherapy
(a) Methotrexate i s cons i dered t he gol d s t andard for t he t reat ment of rheumat oi d art hri t i s , us ual l y gi ven as a weekl y oral dos e. At hi gher dos es , s ubcut aneous i nject i ons of met hot rexat e may be gi ven t o i mprove abs orpt i on and di mi ni s h s i de effect s . Concurrent dai l y fol i c aci d
Pa g e 2 2 8 6
ABC Ambe r CHM Conve rte r Tria l ve rsion, http://w w w .proce sste x t.com/a bcchm.html
admi ni s t rat i on hel ps prevent common s i de effect s , i ncl udi ng mucos al ul cers , dys peps i a, and cyt openi as . The us e of l eucovori n may be neces s ary i f t he s i de effect s do not res pond t o t he fol i c aci d. The s t art i ng dos e of met hot rexat e i s 7.5–10 mg weekl y, es cal at i ng t o a maxi mum of 25 mg weekl y. If a pat i ent has underl yi ng l i ver di s eas e or pul monary di s eas e or us es al cohol , met hot rexat e may not be appropri at e t o us e.
(b) Leflunomide act s s i mi l arl y t o met hot rexat e and requi res es s ent i al l y t he s ame moni t ori ng s t rat egy except t here i s no ri s k of pul monary t oxi ci t y; t hus , i t i s oft en P.458
us ed when met hot rexat e cannot be us ed, s uch as i n a pat i ent wi t h underl yi ng pul monary di s eas e.
(c) Sulfasalazine, al t hough not approved by t he FDA for us e i n rheumat oi d art hri t i s , has been us ed becaus e of i t s modes t effect i venes s and rat her l ow i nci dence of t oxi ci t y. In Europe i t i s t ypi cal l y us ed as a fi rs t -l i ne agent .
(d) Les s commonl y us ed agent s as
Pa g e 2 2 8 7
ABC Ambe r CHM Conve rte r Tria l ve rsion, http://w w w .proce sste x t.com/a bcchm.html
monot herapy i n t he t reat ment of RA i ncl ude gold, azathioprine, and hydroxychloroquine sulfate.
(2) Combination therapy i s us ed i f pat i ent s have had a s ubopt i mal res pons e t o monot herapy. Commonl y us ed combi nat i ons are l i s t ed bel ow.
(a) Methotrexate and hydroxychloroquine sulfate
(b) Methotrexate and hydroxychloroquine sulfate and sulfasalazine
(c) Methotrexate and leflunomide
(d) Methotrexate and cyclosporine
d. Biologic DMARDs. As t he pat hogenes i s of rheumat oi d art hri t i s and t he vari ous i mmune and i nfl ammat ory medi at ors have been i dent i fi ed, novel agent s i n t he t reat ment of RA have been devel oped. Thes e i ncl ude t he devel opment of t umor necros i s fact or (TNF) ant agoni s t s , ant i -cyt oki ne t herapy, and B-cel l modul at ors . The common bi ol ogi c DMARDs avai l abl e for us e are l i s t ed i n Tabl e 10-7.
Pa g e 2 2 8 8
ABC Ambe r CHM Conve rte r Tria l ve rsion, http://w w w .proce sste x t.com/a bcchm.html
TABLE 10-7 Biologic DMARDs Used in the Treatment of Rheumatoid Arthritis D o s e M a e n Si c d d h R e A a o Ef g ni ut fe e s in ct nt m e s Et R 2 Fl a ec 5 un o m li er m g k ce bi S e pt n Q s y a t m nt wi pt fu ce o si w m o e s, n e in pr kl je ot y ct ei or i o n 5 n bi 0 s i n m te d g re
Pa g e 2 2 8 9
ABC Ambe r CHM Conve rte r Tria l ve rsion, http://w w w .proce sste x t.com/a bcchm.html
s o ac t o nc t i s e o ol w n u e s, bl e i n e kl fe T y ct N
io
F
n
re
s,
ce
e
pt
x
or
ac
a
er
n
b
d
at
le
io
u
n
k
of
ot
h
ri
e
e
ar
n
t
e
fa
(L
il
T)
ur e, p o ss ib le d e
Pa g e 2 2 9 0
ABC Ambe r CHM Conve rte r Tria l ve rsion, http://w w w .proce sste x t.com/a bcchm.html
m y el in at io n, u n k n o w n lo n gte r m m al ig n a nc y ri sk , a ut oi m m
Pa g e 2 2 9 1
ABC Ambe r CHM Conve rte r Tria l ve rsion, http://w w w .proce sste x t.com/a bcchm.html
u n e p h e n o m e n a In C 3 In fl i hi m fu xi m g/ s i m er k o a ic g n b m IV m o o a n v y oc er b lo 2 e n h a al o s s a ur oc nt s i a ib e te o v d d er wi y y th t h 8 fe at w v bi e er n e , d ks n
Pa g e 2 2 9 2
ABC Ambe r CHM Conve rte r Tria l ve rsion, http://w w w .proce sste x t.com/a bcchm.html
s
a
T
u
N
s
F
e a, fl u s hi n g, h y p er te n si o n or h y p ot e n si o n; in fe ct io n,
Pa g e 2 2 9 3
ABC Ambe r CHM Conve rte r Tria l ve rsion, http://w w w .proce sste x t.com/a bcchm.html
a ut oi m m u n e p h e n o m e n a, e x ac er b at io n of C H F, p o ss ib le d
Pa g e 2 2 9 4
ABC Ambe r CHM Conve rte r Tria l ve rsion, http://w w w .proce sste x t.com/a bcchm.html
e m y el in at io n, u n k n o w n lo n gte r m m al ig n a nc y ri sk A R 4 In d ec 0 je al o m ct i
m g io
m bi S n
Pa g e 2 2 9 5
ABC Ambe r CHM Conve rte r Tria l ve rsion, http://w w w .proce sste x t.com/a bcchm.html
u n Q si m a e te a nt v re b h er ac u y ti m ot o a h n n er s , Ig w i n G e fe 1 e ct m k io o
n
n
s,
oc
lu
lo
p
n
u
al
s-
a
li
nt
k
ib
e
o
sy
d
m
y
pt
w
o
hi
m
ch
s,
bi
u
n
n
d
k
s
n
T
o
N
w
F
n lo
Pa g e 2 2 9 6
ABC Ambe r CHM Conve rte r Tria l ve rsion, http://w w w .proce sste x t.com/a bcchm.html
n gte r m m al ig n a nc y ri sk A Bi 1 In n n 0 je a d 0 ct ki s m i o nr t o g n a in S si te Q te rl d re e ai ac u ly ti ki
o
n
n
â
s,
€“
in
1
fe
re
ct
ce
io
pt
n
or
s,
s
u
(I
n
Pa g e 2 2 9 7
ABC Ambe r CHM Conve rte r Tria l ve rsion, http://w w w .proce sste x t.com/a bcchm.html
L-
k
1
n
R
o
A)
w n lo n gte r m m al ig n a nc y ri
sk Ri C 1 In t u hi 0 fe xi m 0 ct m er 0 i o a ic m n b m g s, o IV h n o y oc n p l o d ot n a e al ys n a 1 si nt + o i b 1 n,
Pa g e 2 2 9 8
ABC Ambe r CHM Conve rte r Tria l ve rsion, http://w w w .proce sste x t.com/a bcchm.html
o 5, ra d pr s y e h, w mm hi e al ch di i g bi ca n n te a d wi nc s th y C a D co 2 rt 0 ic o o n st B er ce oi ll d s A R IV In b ec d fe at o o ct ac m s i o e bi e n pt n b s , a y in nt w fu fu ei s i si g o o ht n n
re
pr
ac
ot
ti
ei
o
Pa g e 2 2 9 9
ABC Ambe r CHM Conve rte r Tria l ve rsion, http://w w w .proce sste x t.com/a bcchm.html
n
n
th
s
at
s
m
uc
o
h
d
a
ul
s
at
di
e
zz
s
in
T-
e
ce
ss
ll
a
ac
n
ti
d
vi
h
ty
e
b
a
y
d
bi
ac
n
h
di
e
n g C D 8 0 a n d C D 8 6
Pa g e 2 3 0 0
ABC Ambe r CHM Conve rte r Tria l ve rsion, http://w w w .proce sste x t.com/a bcchm.html
CHF, conges t i ve heart fai l ure; IV, i nt raven ous ; SQ, s ubcut an eous l y; TNF, t umor necros i s fact or.
(1) T NF inhibitors. TNF i s a pro-i nfl ammat ory cyt oki ne t hat i s s ynt hes i zed by a vari et y of cel l t ypes . Normal l y, s mal l amount s of TNF are pres ent , but overproduct i on t ri ggers a cas cade of i nfl ammat ory react i ons . Etanercept, infliximab, and adalimumab are FDA-approved for t he t reat ment of RA. Each of t hes e agent s has been s hown t o be effect i ve i n reduci ng s i gns and s ympt oms and i nhi bi t i ng t he progres s i on of s t ruct ural damage i n rheumat oi d art hri t i s .
(a) Use. TNF i nhi bi t ors may be us ed al one or i n combi nat i on (us ual l y wi t h met hot rexat e). P.459
Pa g e 2 3 0 1
ABC Ambe r CHM Conve rte r Tria l ve rsion, http://w w w .proce sste x t.com/a bcchm.html
(b) Safety considerations. Seri ous i nfect i ous and opport uni s t i c i nfect i ons (t ubercul os i s ) have been s een wi t h al l of t he TNF ant agoni s t s . TNF has been s hown t o be neces s ary i n granul oma format i on t hat i s cri t i cal i n t he cont rol of TB. Thus , i nhi bi t i on of TNF has been as s oci at ed wi t h react i vat i on of TB. Screeni ng for TB i s recommended before ini t i at i ng t reat ment wi t h t hes e agent s .
(2) Anti-cytokine therapy. IL-1RA (anakinra) cont rol s RA t hrough a mechani s m di fferent from t hat of t he TNF bl ocki ng agent s but has s i mi l ar s afet y concerns , i ncl udi ng i ncreas ed ri s k of i nfect i ons (not TB), neut ropeni a, and t he pot ent i al devel opment of mal i gnanci es . No dat a as yet s ugges t hi gher ri s k of demyel i nat i on or devel opment of conges t i ve heart fai l ure.
(3) Anti B-cell therapy. B cel l s pl ay a key rol e i n t he devel opment of i nfl ammat i on i n RA, i n addi t i on t o t he des t ruct i ve T cel l s . Therefore, an agent t hat can cont rol t he effect s of B cel l s may be benefi ci al t o t hes e pat i ent s . Ri t uxi mab i s a chi meri cal monocl onal
Pa g e 2 3 0 2
ABC Ambe r CHM Conve rte r Tria l ve rsion, http://w w w .proce sste x t.com/a bcchm.html
ant i body t hat bi nds CD20, a B-cel l s urface ant i gen. Thi s l eads t o a decreas e i n t he B-cel l popul at i on and has s hown benefi ci al effect s at cont rol l i ng i nfl ammat i on i n RA.
(4) T -Cell targeted therapy. CTLA4 Ig (abat acept ) prevent s CD28 from bi ndi ng t o i t s count er recept or CD80/CD86, caus i ng i nact i vat i on of pat hogeni c T cel l s .
e. Assessment of response. The effect i venes s of drug t herapy i s judged by as s es s i ng t he reduct i on i n a number of fact ors : morni ng s t i ffnes s , cons t i t ut i onal compl ai nt s , number of s wol l en and t ender joi nt s , and ESR. Somet i mes i mprovement i n anemi a of chroni c di s eas e and res ol ut i on of t hrombocyt os i s occurs . Indi ces t hat meas ure a pat i ent 's abi l i t y t o perform act i vi t i es of dai l y l i vi ng (heal t h as s es s ment ques t i onnai res ) al s o are us ed.
f. As new t herapi es are bei ng i nt roduced, t he t reat ment s t rat egy has been evol vi ng. In 2002, t he Ameri can Col l ege of Rheumat ol ogy revi s ed t he management gui del i nes .
o
o
3. Surgery. Art hropl as t i es or t ot al joi nt repl acement s may be appropri at e t o rel i eve pai n or hel p res t ore funct i on i n s t ruct ural l y damaged joi nt s .
Pa g e 2 3 0 3
ABC Ambe r CHM Conve rte r Tria l ve rsion, http://w w w .proce sste x t.com/a bcchm.html
H. Prognosis
1. Prognostic factors (Tabl e 10-8). Inabi l i t y t o cont rol
di s eas e act i vi t y and t he pres ence of s everal of t hes e i ndi cat ors s ugges t a poor prognos i s and t he need for more aggres s i ve t herapy, perhaps i ncl udi ng combi nat i ons of s econd-l i ne agent s and l ow-dos e oral gl ucocort i coi ds .
TABLE 10-8 Indicators of Poor Prognosis in Patients with Rheumatoid Arthritis Ma ny per sis t en tly i nfl am ed joi nt s Po or fun ct i on al sta t us (as cer
Pa g e 2 3 0 4
ABC Ambe r CHM Conve rte r Tria l ve rsion, http://w w w .proce sste x t.com/a bcchm.html
t ai ne d fro m he al t h-a sse ss me nt qu es t i on nai res ) Lo w for ma l ed uca tio n l ev el Rh eu ma t oi d fac
Pa g e 2 3 0 5
ABC Ambe r CHM Conve rte r Tria l ve rsion, http://w w w .proce sste x t.com/a bcchm.html
t or pos itiv ity HL ADR 4 pos itiv ity Ext raart i cul ar di s eas e Per sis t en tly el e vat ed acu te ph as e rea ct a nt s (e. g.,
Pa g e 2 3 0 6
ABC Ambe r CHM Conve rte r Tria l ve rsion, http://w w w .proce sste x t.com/a bcchm.html
ery t hr ocy te s ed im ent at i on rat e) Ra di o gra phi c evi de nce of ero sio ns HL A, hu ma n l eu koc yt e ant i ge n.
Pa g e 2 3 0 7
ABC Ambe r CHM Conve rte r Tria l ve rsion, http://w w w .proce sste x t.com/a bcchm.html
2. Mortality. Many pat i ent s wi t h rheumat oi d art hri t i s have a reas onabl y good prognos i s i f t hey res pond wel l t o t reat ment . Adequat e earl y res pons e t o t he us e of NSAIDs or ant i mal ari al s , wi t h or wi t hout cort i cos t eroi ds , i s a favorabl e prognos t i c s i gn. However, recent epi demi ol ogi c s t udi es s ugges t t hat pat i ent s wi t h s evere and pers i s t ent di s eas e have i ncreas ed mort al i t y rat es . In t hos e wi t h t he mos t hi ghl y expres s ed forms of rheumat oi d art hri t i s , mort al i t y rat es approach t hos e found i n s t age IV conges t i ve heart fai l ure (CHF) or s t age IV Hodgki n's di s eas e. The i ncreas e i n mort al i t y appears t o be t he res ul t of organ compromi s e caus ed by ext ra-art i cul ar feat ures (e.g., i nt ers t i t i al l ung di s eas e, cardi ac compl i cat i ons , and vas cul i t i s ), compl i cat i ons of drug t herapy, and i nfect i on.
P.460
IV. Spondyloarthropathies A. Unifying characteristics (Table 10-9) The s pondyl oart hropat hi es are a group of i nfl ammat ory art hri t i des di s t i nct from rheumat oi d art hri t i s , i ncl udi ng ankyl os i ng s pondyl i t i s , react i ve art hri t i s , ps ori at i c art hri t i s , and art hri t i s as s oci at ed wi t h i nfl ammat ory bowel di s eas e. Typi cal di s t i ngui s hi ng feat ures i ncl ude:
TABLE 10-9 Distinguishing Characteristics of Spondyloarthropathies
Pa g e 2 3 0 8
ABC Ambe r CHM Conve rte r Tria l ve rsion, http://w w w .proce sste x t.com/a bcchm.html
Axi al s ke l et on i nfl am ma tio n Ent hes is i nfl am ma tio n, oft en as y mm et ri c Ch ara ct e ri s t ic ext ras kel et a l fea
Pa g e 2 3 0 9
ABC Ambe r CHM Conve rte r Tria l ve rsion, http://w w w .proce sste x t.com/a bcchm.html
t ur es Uv ei t i s or con jun ct i vi t i s Ure t hri tis I nfl am ma t or y bo wel l es i on s P s or i as i s -l i ke ras hes As s
Pa g e 2 3 1 0
ABC Ambe r CHM Conve rte r Tria l ve rsion, http://w w w .proce sste x t.com/a bcchm.html
oci at i on wi t h HL Aâ €“B 27 Ab s en ce of rhe um at o id fac t or HL A, hu ma n l eu koc yt e ant i ge n.
1. Clinical features o
o
a. Skeletal
Pa g e 2 3 1 1
ABC Ambe r CHM Conve rte r Tria l ve rsion, http://w w w .proce sste x t.com/a bcchm.html
(1) Axial. As a group, t he s pondyl oart hropat hi es promi nent l y i nvol ve t he axi al s kel et on, part i cul arl y t he sacroiliac joints. W i t h t he except i on of t he cervi cal s pi ne, t he axi al s kel et on i s not commonl y i nvol ved i n rheumat oi d art hri t i s .
(2) Appendicular. Infl ammat ory art hri t i s of t he appendi cul ar s kel et on al s o occurs i n t hes e di s orders , but t he i nvol vement t ends t o be ol i goart i cul ar and as ymmet ri cal . In cont ras t , rheumat oi d art hri t i s us ual l y i s pol yart i cul ar and s ymmet ri cal .
(3) Enthesis. In bot h t he axi al and appendi cul ar s kel et ons , i nfl ammat i on of t endon and l i gament s i t es of at t achment t o bone i s common (e.g., cos t ochondri t i s , Achi l l es t endi ni t i s , and pl ant ar fas ci i t i s ). Tendon i nfl ammat i on i s l es s promi nent i n rheumat oi d art hri t i s .
o
o
b. Extraskeletal
(1) Nodules. Rheumat oi d nodul es are not found i n t he s pondyl oart hropat hi es .
(2) Internal organ involvement. The t ypi cal eye i nvol vement i n t he s pondyl oart hropat hi es (conjunct i vi t i s and ant eri or uvei t i s ), cardi ac
Pa g e 2 3 1 2
ABC Ambe r CHM Conve rte r Tria l ve rsion, http://w w w .proce sste x t.com/a bcchm.html
i nvol vement (aort i t i s ), and geni t ouri nary i nvol vement (uret hri t i s and pros t at i t i s ) are much di fferent from t he us ual ext ra-art i cul ar feat ures of rheumat oi d art hri t i s . o
o
c. Laboratory findings. Several cardi nal l aborat ory feat ures of chroni c i nfl ammat i on (i .e., anemi a, t hrombocyt os i s , and el evat ed gamma gl obul i n l evel s ) are not commonl y pres ent i n t he s pondyl oart hropat hi es as t hey are i n rheumat oi d art hri t i s . Al t hough t he ESR may be i ncreas ed, i t i s not a good meas ure of di s eas e act i vi t y. Rheumat oi d fact or t ypi cal l y i s abs ent .
o
o
d. Radiographic findings. The charact eri s t i c changes s een radi ographi cal l y are t hos e of peri os t eal new bone format i on at t he s i t e of t he ent hes opat hi c l es i ons (s ee IV A 3), bot h at axi al l ocat i ons (i n t he form of syndesmophytes) and appendi cul ar l ocat i ons . Al t hough eros i ve changes can occur, t hey occur mos t t ypi cal l y i n t he axi al s kel et on (i n t he hi ps , s acroi l i ac joi nt s , and s houl ders ).
o
o
e. T herapeutic response. Several s econd-l i ne agent s commonl y us ed i n t reat i ng rheumat oi d art hri t i s (e.g., gol d and hydroxychl oroqui ne) have no proven rol e i n t he t reat ment of s pondyl oart hropat hy, wi t h t he except i on of ps ori at i c peri pheral art hropat hy. Sul fas al azi ne and met hot rexat e are us ed for bot h rheumat oi d art hri t i s and t he appendi cul ar art hri t i s of t he s pondyl oart hropat hi es . TNF-i nhi bi t ors (et anercept ,
Pa g e 2 3 1 3
ABC Ambe r CHM Conve rte r Tria l ve rsion, http://w w w .proce sste x t.com/a bcchm.html
i nfl i xi mab, and adal i mumab) are now FDA-approved for t he t reat ment of ankyl os i ng s pondyl i t i s .
2. Genetic factors. The s t rong as s oci at i on of t he hi s t ocompat i bi l i t y ant i gen HLA-B27 wi t h cl i ni cal expres s i on of t he s pondyl oart hropat hi es provi des evi dence for genet i c t rans mi s s i on of t hes e di s orders . However, HLA-B27 i s found i n 8% of Caucas i ans , 3% of Afri can Ameri cans , and l es s t han P.461
1% of As i ans , and al l of t hes e i ndi vi dual s do not devel op s pondyl oart hropat hi es . Convers el y, s pondyl oart hropat hi es are found i n pat i ent s who are HLA-B27 negat i ve (10%–20%). o
o
a. Thi s rel at i ons hi p al s o i s a major reas on for groupi ng t hes e di s orders . The i ndependent correl at i on of HLA-B27 wi t h s peci fi c feat ures of s pondyl oart hropat hi es (e.g., s acroi l i i t i s , aort i t i s , and ant eri or uvei t i s ) expl ai ns bot h t he cl i ni cal overl ap among t hes e di s eas es and t hei r fami l i al cl us t eri ng.
o
o
b. The rol e of t he ant i gen i n di s eas e caus at i on i s not unders t ood. However, t rans geni c rat s can expres s HLA-B27 on cel l s urfaces and devel op cl i ni cal feat ures of s pondyl oart hropat hi es , s o t hi s gene product i s cl earl y i nvol ved i n di s eas e caus at i on or perpet uat i on.
o
o
c. Di s eas e s us cept i bi l i t y i s al s o as s oci at ed wi t h
Pa g e 2 3 1 4
ABC Ambe r CHM Conve rte r Tria l ve rsion, http://w w w .proce sste x t.com/a bcchm.html
ot her unknown genes , perhaps T-cel l recept or genes , account i ng for t he 10-fol d i ncreas e i n ri s k as s oci at ed wi t h HLA-B27 pos i t i vi t y i n s pondyl i t i s fami l i es .
3. Pathology o
o
a. The bas i c pat hol ogi c l es i on i n t he s pondyl oart hropat hi es i s an enthesopathy—an i nfl ammat i on occurri ng at t he s i t e where l i gament s and t endons at t ach t o bone. Thi s t ype of i nfl ammat i on expl ai ns t he frequency of s acroi l i i t i s , as cendi ng s pi nal l es i ons , and peri pheral t endon l e s i ons (e.g., Achi l l es t endi ni t i s ).
o
o
b. Al t hough i nfl ammat ory s ynovi t i s t hat i s i ndi s t i ngui s habl e from rheumat oi d s ynovi t i s can be s een i n t hes e i l l nes s es , i t i s not t ypi cal l y as wi des pread, chroni cal l y act i ve, and pot ent i al l y des t ruct i ve as i t i s i n rheumat oi d art hri t i s (ps ori at i c art hri t i s mut i l ans i s a not abl e except i on).
4. HIV-related spondyloarthropathic disease. Some HIV-pos i t i ve pat i ent s have s pondyl oart hropat hi c i l l nes s oft en wi t h feat ures of reactive arthritis or psoriatic arthritis. However, mos t commonl y, t hey have s ki n, joi nt , and t endon feat ures t hat prevent eas y cl as s i fi cat i on i nt o one i l l nes s or t he ot her, s ugges t i ng t hat t hes e s pondyl oart hropat hi c i l l nes s es have a common pat hogenes i s (s ee XIV A 1).
B. Specific disorders
Pa g e 2 3 1 5
ABC Ambe r CHM Conve rte r Tria l ve rsion, http://w w w .proce sste x t.com/a bcchm.html
1. Ankylosing spondylitis o
o
a. Definition. Ankyl os i ng s pondyl i t i s i s t he s pondyl oart hropat hy t hat i s mos t cl os el y as s oci at ed wi t h i nfl ammat i on of t he axi al s kel et on. Back pain and limited spinal mobility caus ed by sacroiliitis and vari abl e as cent of t he i nfl ammat i on up t he s pi ne domi nat e t he cl i ni cal expres s i on of t hi s di s eas e.
o
o
b. Epidemiology
(1) Prevalence. Ankyl os i ng s pondyl i t i s may be as common as 1 i n 1000 Caucas i an i ndi vi dual s , becaus e t he frequency of di s eas e paral l el s t he preval ence of t he HLA-B27 ant i gen i n t he popul at i on (s ee IV A 2). The frequency of ankyl os i ng s pondyl i t i s i s l ower i n Afri can Ameri can and As i an popul at i ons , paral l el i ng t he preval ence of t he ant i gen i n t hes e groups .
(2) Gender distribution. Ankyl os i ng s pondyl i t i s may be as preval ent i n women as i n men i f radi ographi c fi ndi ngs of s acroi l i i t i s are cons i dered t o be di agnos t i c of t he di s eas e. However, women t end t o have s omewhat mi l der di s eas e wi t h more peri pheral joi nt mani fes t at i ons .
Pa g e 2 3 1 6
ABC Ambe r CHM Conve rte r Tria l ve rsion, http://w w w .proce sste x t.com/a bcchm.html
(3) Familial aggregation. The ri s k of ankyl os i ng s pondyl i t i s i n an HLA-B27–pos i t i ve fami l y member of an affect ed proband i s 20%, as compared t o 1%–2% for t he general popul at i on of t hos e wi t h HLA-B27.
o
o
c. Etiology
(1) The major hi s t ocompat i bi l i t y ant i gen HLA-B27 occurs i n 90%–95% of whi t e pat i ent s wi t h ankyl os i ng s pondyl i t i s , but t he as s oci at i on i s l es s marked i n nonwhi t e popul at i ons (40%–50% of Afri can Ameri cans wi t h ankyl os i ng s pondyl i t i s have HLA-B27).
(2) Becaus e t he gene codi ng for t he expres s i on of HLA-B27 res i des on chromos ome 6, aut os omal t rans mi s s i on occurs . Thus , chi l dren of a proband who i s het eroz ygous for t he gene cont rol l i ng HLA-B27 product i on have a 50% probabi l i t y of event ual expres s i on of t he ant i gen on cel l s urfaces .
(3) The presence of HLA-B27 on cell membranes i s t hought t o be i mport ant i n t he caus at i on of ankyl os i ng s pondyl i t i s . The di s eas e i s s omehow caus ed or perpet uat ed by t he P.462
Pa g e 2 3 1 7
ABC Ambe r CHM Conve rte r Tria l ve rsion, http://w w w .proce sste x t.com/a bcchm.html
pres ence of a s hort ami no aci d s equence i n t he pept i de-bi ndi ng cl eft of t he HLA-B27 mol ecul e t hat i s abl e t o bi nd a uni que art hri t i s -caus i ng pept i de.
(a) Receptor theory
(Onl i ne Fi gure 10-6A). One current t heory
i s t hat t hi s marker i s a recept or for an envi ronment al fact or (e.g., bact eri al pept i de ant i gen, vi rus ), whi ch t hen can caus e di s eas e. The arthritogenic peptide theory i s a vari at i on i n whi ch an i mmune res pons e t o a bact eri al pept i de i s i ncreas ed becaus e t he pept i de (aft er i nt racel l ul ar proces s i ng) i s part i cul arl y wel l pres ent ed by HLA-B27.
ONLINE FIGURE 10-6 Human l eukocyt e ant i gen B27 (HLA-B27) as t he caus e of ankyl os i ng s pondyl i t i s . (A) Recept or t heory. A bact eri al pept i de or vi rus forms a compl ex ext ernal l y wi t h HLA-B27. The art hri t ogeni c pept i de t heory, a vari at i on of t he recept or t heory,
Pa g e 2 3 1 8
ABC Ambe r CHM Conve rte r Tria l ve rsion, http://w w w .proce sste x t.com/a bcchm.html
mai nt ai ns t hat t he bact eri al or vi ral pept i de i s proces s ed i nt racel l ul arl y and onl y pres ent ed on t he cel l s urface i n as s oci at i on wi t h t he HLA-B27 mol ecul e. (B) Mol ecul ar mi mi cry t heory. An i mmune res pons e i s mount ed agai ns t HLA-B27 (aut oi mmuni t y) or s uppres s ed agai ns t t he di s eas e-caus i ng pept i de (t ol erance) becaus e t he i mmune s ys t em cannot di s t i ngui s h bet ween t hei r s i mi l ar ami no aci d s equences .
(b) Molecular mimicry theory
(Onl i ne Fi gure 10-6B). A bact eri al
or ot her envi ronment al ant i gen pres ent ed on t he cel l s urface wi t h a di fferent HLA mol ecul e mi ght s hare s i mi l ar s equences wi t h t he HLA-B27 mol ecul e. If t hi s cl as s I compl ex (non–HLA-B27 pl us +
pept i de) i s recogni zed by t he cyt ot oxi c CD8 T cel l as HLA-B27, an i mmune res pons e can be mount ed agai ns t HLA-B27 (autoimmunity) or s uppres s ed agai ns t t he di s eas e-caus i ng pept i de (tolerance). Ei t her condi t i on coul d l ead t o cl i ni cal di s eas e expres s i on.
(c) T hymic selection theory.
HLA-B27 may funct i on at t he l evel
of t he t hymus by al l owi ng s el ect i on of arthritogenic T cells. o
o
d. Clinical features
(1) Disease onset. The di s eas e us ual l y devel ops i n t he s econd or t hi rd decade of l i fe.
(2) Disease course. The di s eas e begi ns wi t h t he gradual ons et of chroni c s acral backache, whi ch i s as s oci at ed wi t h prol onged morni ng s t i ffnes s and whi ch i mproves wi t h exerci s e. Mos t pat i ent s have prol onged, unremi t t i ng l ow back pai n for years . The di s eas e may be mi l d and caus e mi ni mal i nt erference wi t h funct i on,
Pa g e 2 3 1 9
ABC Ambe r CHM Conve rte r Tria l ve rsion, http://w w w .proce sste x t.com/a bcchm.html
or i t may be s evere and deformi ng.
(3) Manifestations of disease
(a) Axial skeletal involvement. Symmet ri cal i nfl ammat i on of t he s acroi l i ac joi nt s (s acroi l i i t i s ) i s t he mos t common pres ent at i on. Infl ammat i on and cons equent cal ci fi cat i on of t he s pi nal l i gament s and t he i nt ervert ebral zygapophys eal joi nt s can caus e l i mi t ed s pi nal mobi l i t y, and i nt ercos t al l i gament ent hes opat hy can caus e l i mi t at i on of ches t wal l expans i on.
(b) Peripheral joint involvement. Thi s feat ure i s t ypi cal of more s evere ankyl os i ng s pondyl i t i s . Eros i ve hi p and s houl der i nvol vement i s not uncommon and may be s evere. More di s t al s ynovi t i s i s l es s common, al t hough 35% of pat i ent s wi t h ankyl os i ng s pondyl i t i s have s ome evi dence of peri pheral joi nt di s eas e.
(c) Extraskeletal features. Cons t i t ut i onal compl ai nt s , fat i gue, and wei ght l os s are not as common i n ankyl os i ng s pondyl i t i s as i n rheumat oi d art hri t i s , but t hey may occur. Ant eri or uvei t i s occurs i n 25% of pat i ent s wi t h ankyl os i ng s pondyl i t i s , and s omet i mes occurs as an i s ol at ed cl i ni cal as s oci at i on
Pa g e 2 3 2 0
ABC Ambe r CHM Conve rte r Tria l ve rsion, http://w w w .proce sste x t.com/a bcchm.html
of HLA-B27. Aort i c root i nfl ammat i on can occur, us ual l y i n pat i ent s wi t h l ong-s t andi ng di s eas e; t hi s proces s can l ead t o aort i c val ve i ns uffi ci ency or, i f i t ext ends i nt o t he conduct i on s ys t em, compl et e heart bl ock. Upper l obe pul monary fi bros i s and chroni c pros t at i t i s are ot her uncommon ext ras kel et al feat ures . o
o
e. Diagnosis. Combi ni ng hi s t ori cal , phys i cal , and radi ographi c evi dence as wel l as excl udi ng mechani cal l ow back pai n, ot her s pondyl oart hropat hi es , and ot her i nfl ammat ory art hri t i des , al l ows for di agnos i s .
(1) Historical information
(a) Inflammatory versus mechanical low back pain. Infl ammat ory s acroi l i i t i s has a gradual ons et , i n earl y adul t hood, and i s pers i s t ent for more t han 3 mont hs ; t he pai n i s as s oci at ed wi t h prol onged earl y morni ng back s t i ffnes s and i s rel i eved by exerci s e and wors ened by res t . In cont ras t , t he ons et of mechani cal l ow back pai n us ual l y occurs l at er i n l i fe, wi t h s udden, s el f-l i mi t ed epi s odes t hat are wors ened by exerci s e and i mproved by bed res t .
(b) Familial association. Pat i ent s wi t h ankyl os i ng s pondyl i t i s oft en have ot her
Pa g e 2 3 2 1
ABC Ambe r CHM Conve rte r Tria l ve rsion, http://w w w .proce sste x t.com/a bcchm.html
affect ed fami l y members .
(c) Associated complaints. Evi dence of pri or i nfl ammat ory eye s ympt oms , recurrent ol i goart hri t i s , and i nfl ammat ory t endi ni t i s s houl d be s ought .
(2) Physical findings
(a) Musculoskeletal examination. Each s acroi l i ac joi nt s houl d be eval uat ed for t endernes s , l umbar s pi nal mobi l i t y s houl d be as s es s ed i n al l di rect i ons , and ches t expans i on s houl d be eval uat ed t o as s es s s everi t y of ches t wal l ent hes opat hi c l es i ons .
(b) General physical examination. Evi dence of as s oci at ed abnormal i t i es s houl d be s ought , i ncl udi ng ocul ar eryt hema, aort i c i ns uffi ci ency murmurs , and peri pheral art hri t i s and t endi ni t i s .
(3) Laboratory findings. The onl y charact eri s t i c l aborat ory abnormal i t y i n ankyl os i ng s pondyl i t i s i s t he vari abl e pres ence of HLA-B27 i n di fferent popul at i on groups . In general , t es t i ng for t hi s ant i gen i s not neces s ary for t he di agnos i s .
(4) Radiographic findings. Sacroi l i ac
Pa g e 2 3 2 2
ABC Ambe r CHM Conve rte r Tria l ve rsion, http://w w w .proce sste x t.com/a bcchm.html
i nvol vement i s bes t s een on an ant eropos t eri or radi ograph of t he pel vi s . Bl urred joi nt margi ns , peri art i cul ar s cl eros i s , eros i ons , and P.463
joi nt -s pace wi deni ng are charact eri s t i c, but t ot al joi nt obl i t erat i on i s t ypi cal of l ong-s t andi ng di s eas e. If t he i l l nes s i s more s evere or l ong-s t andi ng, as cendi ng s pi nal i nvol vement can occur, wi t h del i cat e fl owi ng cal ci fi cat i ons (s yndes mophyt es ) t hat bri dge t he i nt ervert ebral di s k, cul mi nat i ng i n a bamboo-spine appearance on a radi ograph.
(5) Differential diagnosis
(a) Nature of low back pain. As ment i oned previ ous l y, mechani cal l ow back pai n mus t be di fferent i at ed from i nfl ammat ory l ow back pai n [s ee IV B 1 e (1) (a)].
(b) Other disorders. Sacroi l i i t i s i s hi ghl y unus ual i n nons pondyl i t i c di s eas es . Charact eri s t i c s ki n l es i ons may s ugges t t hat a pat i ent wi t h s acroi l i i t i s has ps ori as i s or react i ve art hri t i s . Promi nent uret hri t i s i n as s oci at i on wi t h s acroi l i i t i s may al s o s ugges t react i ve art hri t i s , and promi nent bowel compl ai nt s wi t h s acroi l i i t i s may s ugges t
Pa g e 2 3 2 3
ABC Ambe r CHM Conve rte r Tria l ve rsion, http://w w w .proce sste x t.com/a bcchm.html
i nfl ammat ory bowel di s eas e. o
o
f. T herapy. Pat i ent educat i on and mul t i di s ci pl i nary t reat ment are i mport ant t herapeut i c component s i n ankyl os i ng s pondyl i t i s , jus t as t hey are i n rheumat oi d art hri t i s . Short -t erm goal s i nvol ve cont rol of pai n and reduct i on of i nfl ammat i on wi t hout caus i ng drug t oxi ci t y, and l ong-t erm goal s are prevent i on of pos t ural deformi t y and ret ent i on of empl oyment .
(1) Education
(a) Cigarette smoking. Indi vi dual s wi t h ent hes opat hi c ches t wal l res t ri ct i on or fi brot i c l ung di s eas e s houl d be di s couraged from s moki ng.
(b) Genetic counseling. Pat i ent s s houl d be made aware of t he fami l i al i nci dence of t he i l l nes s s o t hat i t can be di agnos ed earl y i n chi l dren and t reat ment of s pondyl i t i c s ympt oms begun.
(c) Protection of brittle spine. Pat i ent s wi t h ext ens i ve s pi nal i nvol vement mus t unders t and t hat mi ni mal t rauma can caus e a s pi nal fract ure. W eari ng a hard or s oft col l ar s houl d be recommended for aut omobi l e dri vi ng.
Pa g e 2 3 2 4
ABC Ambe r CHM Conve rte r Tria l ve rsion, http://w w w .proce sste x t.com/a bcchm.html
(2) Exercise. Spi nal ext ens i on exerci s es and correct pos t ure are i mport ant for prevent i on of deformi t y. Hard mat t res s es and s mal l cervi cal pi l l ows hel p prevent exces s i ve s pi nal fl exi on duri ng s l eep.
(2) Drug therapy. Al t hough many pat i ent s requi re DMARDS, NSAIDs are t he mai ns t ay of t reat ment . For pers i s t ent di s eas e act i vi t y, TNF i nhi bi t ors (et anercept , i nfl i xi mab, and adal i mumab) are now FDA-approved for t he t reat ment of ankyl os i ng s pondyl i t i s . They al l evi at e s ympt oms , i mprove funct i on, and may i mpact t he nat ural progres s i on of t he di s eas e. Local joi nt or peri t endi nous i ns t i l l at i on of a cort i cos t eroi d s omet i mes i s benefi ci al i n pat i ent s wi t h promi nent peri pheral di s eas e mani fes t at i ons not cont rol l ed by NSAIDs .
o
o
g. Prognosis. Mos t pat i ent s wi t h ankyl os i ng s pondyl i t i s remai n empl oyabl e and cont i nue t o funct i on wel l i n s oci et y. The progres s i on t o s evere, deformi ng di s eas e cannot be predi ct ed on t he bas i s of HLA-B27 s t at us or ot her cri t eri a. Pat i ent s wi t h s evere s pondyl i t i s t end t o have bri t t l e s pi nes , more cardi opul monary di s eas e, and ot her ext ras pi nal compl i cat i ons ; t hes e pat i ent s may have s hort ened l i fe s pans .
2. Reactive arthritis o
o
a. Definition. React i ve art hri t i s i s anot her
Pa g e 2 3 2 5
ABC Ambe r CHM Conve rte r Tria l ve rsion, http://w w w .proce sste x t.com/a bcchm.html
s pondyl oart hropat hy t hat i s s t rongl y as s oci at ed wi t h HLA-B27.
(1) React i ve art hri t i s i ni t i al l y referred t o t he cl as s i c t ri ad of nongonococcal uret hri t i s , conjunct i vi t i s , and art hri t i s t hat occurred aft er a di arrheal i l l nes s or geni t ouri nary i nfect i on, us ual l y wi t h Chl amydi a t rac homat i s . Many pat i ent s s t i l l have t he cl as s i c di s eas e t ri ad, but t he us e of HLA-B27 t ypi ng has al l owed “i ncompl et e― forms (wi t hout t he uret hri t i s or conjunct i vi t i s ) t o be i dent i fi ed.
(2) React i ve art hri t i s i s a predomi nant l y l ower ext remi t y ol i goart hri t i s t ri ggered by urethritis, cervicitis, or dysenteric infection. The vari abl e feat ures of t he t ypi cal s yndrome i ncl ude mucocutaneous lesions, inflammatory eye lesions, and sacroiliitis or peripheral arthritis.
o
o
b. Epidemiology
(1) Incidence. React i ve art hri t i s occurs i n 1%–3% of pat i ent s aft er nons peci fi c uret hri t i s and 0.2% of pat i ent s aft er dys ent ery out breaks caus ed by Shi gel l a fl exneri . HLA-B27 i s pres ent P.464
i n 75%–80% of cas es . Pat i ent s wi t h t he marker who devel op nons peci fi c uret hri t i s or S. fl exneri
Pa g e 2 3 2 6
ABC Ambe r CHM Conve rte r Tria l ve rsion, http://w w w .proce sste x t.com/a bcchm.html
dys ent ery have a 20%–25% chance of devel opi ng react i ve art hri t i s .
(2) Gender distribution. React i ve art hri t i s i s di agnos ed i n men much more oft en t han i n women, i n part becaus e cervi ci t i s i s l es s s ympt omat i c t han uret hri t i s . However, t he art hri t i s t ends t o be l es s s evere i n women as wel l .
o
o
c. Etiology and pathogenesis. Speci fi c i nfect i ons t ri gger t he cl i ni cal expres s i on of art hri t i s i n s us cept i bl e pat i ent s , i ncl udi ng chl amydi al and mycopl as mal uret hri t i s as wel l as dys ent eri c i nfect i ons caus ed by cert ai n s erot ypes of Shi gel l a, Sal monel l a, and Y ers i ni a. The pres ence of t he organi s m may not be neces s ary for l at er exacerbat i ons or chroni c act i vi t y of t he di s eas e, al t hough s ome pol ymeras e chai n react i on (PCR) and el ect ron mi cros copi c s t udi es of i nfl amed s ynovi um s ugges t t he pers i s t ence of chl amydi al organi s ms or fragment s i n react i ve art hri t i s . The unus ual s everi t y of react i ve art hri t i s i n pat i ent s wi t h acqui red i mmunodefi ci ency s yndrome (AIDS) s ugges t s t hat t he i nfl ammat ory res pons e does not +
requi re funct i oni ng CD4 cel l s .
(1) Exaggerated immune response. Concei vabl y, i nfect i ous ant i gens cros s -react wi t h s el f ant i gens s uch as HLA-B27 and s t i mul at e an exaggerat ed i mmune res pons e t hat can i ncl ude a noni nfect i ous art hri t i s .
Pa g e 2 3 2 7
ABC Ambe r CHM Conve rte r Tria l ve rsion, http://w w w .proce sste x t.com/a bcchm.html
(2) Suppressed immune response. It i s al s o concei vabl e t hat mol ecul ar mi mi cry bet ween HLA-B27 and bact eri al pept i des prevent s t he hos t i mmune s ys t em from recogni zi ng t he pat hogen, t hus al l owi ng di s s emi nat i on and product i on of di s eas e at wi del y vari ed s i t es . Evi dence t hat s ugges t s pers i s t ence of organi s ms or fragment s at s i t es of chroni c i nfl ammat i on s upport s t hi s t heory.
(2) Protected site of infection. Pers i s t ent s ubcl i ni cal geni t ouri nary or gas t roi nt es t i nal i nfect i ons (e.g., Chl amydi a, Shi gel l a) coul d caus e recurrent s heddi ng of organi s ms or bact eri al ant i gens t o joi nt s . Pers i s t ent s peci fi c IgA res pons es t o caus at i ve organi s ms and i mprovement wi t h s urgi cal s t ri ppi ng of t he uret hra i n venereal ons et of react i ve art hri t i s s upport t hi s t heory.
o
o
d. Clinical features
(1) Disease onset. React i ve art hri t i s begi ns mos t oft en i n young adul t hood. One t o t hree weeks aft er an epi s ode of uret hri t i s or dys ent ery, any of t he t ypi cal cl i ni cal feat ures of t he s yndrome can occur. The di s eas e oft en i s mi s di agnos ed becaus e t hes e feat ures t end t o occur s eri al l y rat her t han s i mul t aneous l y.
(2) Manifestations of disease
Pa g e 2 3 2 8
ABC Ambe r CHM Conve rte r Tria l ve rsion, http://w w w .proce sste x t.com/a bcchm.html
(a) Musculoskeletal
(i) Arthritis. A l ower ext remi t y ol i goart hri t i s i s t he mos t common joi nt pres ent at i on. The art hri t i s may be acut e and s el f-l i mi t ed, but i t i s more commonl y rel aps i ng or chroni c.
(ii) Enthesopathy. Infl ammat i on of t he t endons and l i gament s are as much a part of react i ve art hri t i s as t hey are of ankyl os i ng s pondyl i t i s . Pl ant ar fas ci i t i s and Achi l l es t endi ni t i s are mos t t ypi cal . Dactylitis (s aus age t oe), a l es i on i nvol vi ng bot h joi nt and t endon i nfl ammat i on i n t he s ame di gi t , i s al s o a common feat ure.
(iii) Sacroiliitis. As ymmet ri cal i nvol vement of s acroi l i ac joi nt s occurs i n approxi mat el y 20% of pat i ent s . Les s frequent l y, as ymmet ri cal as cendi ng s pi nal di s eas e occurs . The cons equent back pai n has t ypi cal i nfl ammat ory charact eri s t i cs , but onl y rarel y does t he as cendi ng s pi nal di s eas e l i mi t t horaci c or cervi cal s pi nal mobi l i t y.
(b) Genitourinary. Sympt omat i c or
Pa g e 2 3 2 9
ABC Ambe r CHM Conve rte r Tria l ve rsion, http://w w w .proce sste x t.com/a bcchm.html
as ympt omat i c urethritis i s ext remel y common. Chroni c prostatitis al s o i s common, affect i ng as many as 80% of pat i ent s i n s ome s eri es .
(c) Ocular. Bot h conjunctivitis and anterior uveitis oft en occur. The conjunct i val i nfl ammat i on i s an acut e, us ual l y s el f-l i mi t ed mani fes t at i on, whi ch may be recurrent . Ant eri or uvei t i s occurs i n more es t abl i s hed forms of di s eas e; i t may be chroni c and requi re t opi cal or s ys t emi c cort i cos t eroi ds t o prevent vi s ual det eri orat i on.
(d) Mucocutaneous. Fl eet i ng and pai nl es s oral ul cers are t he t ypi cal mucous membrane feat ures . Keratoderma blennorrhagica i s t he charact eri s t i c s cal i ng, pl aque-l i ke l es i on found anywhere on t he body, i ncl udi ng t he pal ms and s ol es . Thi s l es i on res embl es pus t ul ar ps ori as i s cl i ni cal l y and pat hol ogi cal l y. Circinate balanitis i s a pai nl es s , P.465
eryt hemat ous eros i on of t he gl ans peni s t hat may expand t o s urround t he uret hral ori fi ce. Each of t hes e s ki n l es i ons occurs i n approxi mat el y 20%–30% of pat i ent s wi t h react i ve art hri t i s , al t hough ci rci nat e bal ani t i s i s t he mos t common.
Pa g e 2 3 3 0
ABC Ambe r CHM Conve rte r Tria l ve rsion, http://w w w .proce sste x t.com/a bcchm.html
(e) Cardiovascular. Earl y cardi ovas cul ar changes i ncl ude t rans i ent pericardial rubs or first-degree heart block. In more s evere, l ong-s t andi ng di s eas e, an aortitis i dent i cal t o t hat s een i n ankyl os i ng s pondyl i t i s can caus e val vul ar i ns uffi ci ency or conduct i on s ys t em l es i ons .
o
o
e. Diagnosis
(1) General considerations. React i ve art hri t i s can be di ffi cul t t o di agnos e when t he ons et of vari ous cl i ni cal feat ures i s wi del y s eparat ed over t i me, but i t i s qui t e eas y i f arthritis, dysentery or urethritis, conjunctivitis, and mucocutaneous lesions appear s i mul t aneous l y. A t ent at i ve di agnos i s can be made when a s eronegat i ve as ymmet ri cal ol i goart hri t i s i s as s oci at ed wi t h any of t hes e ext ra-art i cul ar feat ures . Becaus e t hi s form of art hri t i s i s , s t ri ct l y s peaki ng, a “react i ve― art hri t i s , t he t emporal appearance of art hri t i s aft er uret hri t i s or a dys ent eri c i l l nes s i s es peci al l y convi nci ng. The mucocut aneous l es i ons , uret hri t i s , and cervi ci t i s , whi ch oft en are as ympt omat i c, mus t be s ought s peci fi cal l y whi l e t aki ng t he pat i ent hi s t ory and duri ng t he phys i cal exami nat i on.
(2) Laboratory findings. Ei ght y percent of whi t es wi t h react i ve art hri t i s have HLA-B27.
Pa g e 2 3 3 1
ABC Ambe r CHM Conve rte r Tria l ve rsion, http://w w w .proce sste x t.com/a bcchm.html
The s ynovi al fl ui d t ypi cal l y i s mi l dl y t o moderat el y i nfl ammat ory, wi t h neut rophi l predomi nance and no i mport ant di s t i ngui s hi ng charact eri s t i cs .
(3) Radiographic findings. As ymmet ri cal , ol i goart i cul ar eros i ons , joi nt s pace narrowi ng, and peri art i cul ar os t eopeni a can be s een radi ographi cal l y i n es t abl i s hed di s eas e. Peri os t eal new bone format i on i s a charact eri s t i c feat ure of react i ve art hri t i s , es peci al l y adjacent t o t he i ns ert i ons of t he Achi l l es t endon and pl ant ar fas ci a. Sacroi l i i t i s , i f i t occurs , t ypi cal l y i s as ymmet ri cal , and t he occas i onal pat i ent wi t h s pondyl oart hropat hy has as ymmet ri cal , l arge s yndes mophyt es at s cat t ered vert ebral l evel s .
(4) Differential diagnosis
(a) Diseases most likely to mimic acute reactive arthritis are gonococcal art hri t i s and ot her i nfect i ous art hropat hi es , even Lyme di s eas e. Thus , appropri at e t i s s ues s houl d be cul t ured and s erol ogi es obt ai ned. Crys t al -medi at ed art hri t i s (i .e., gout and ps eudogout ) and t he art hri t i s of rheumat i c fever s houl d be excl uded.
(b) Diseases most likely to mimic chronic recurring reactive arthritis i ncl ude ot her s pondyl oart hropat hi es ,
Pa g e 2 3 3 2
ABC Ambe r CHM Conve rte r Tria l ve rsion, http://w w w .proce sste x t.com/a bcchm.html
es peci al l y ps ori at i c art hri t i s and ankyl os i ng s pondyl i t i s . Cl os e at t ent i on t o t he s ymmet ry and s everi t y of s acroi l i ac and s pi nal i nvol vement , t o ext ras kel et al feat ures , and t o t he pres ence or abs ence of i nfect i ous t ri ggers may hel p di fferent i at e t hes e i l l nes s es . Many cas es of s o-cal l ed seronegative rheumatoid arthritis may act ual l y be cas es of react i ve art hri t i s . Typi cal cl i ni cal feat ures as wel l as radi ographi c evi dence of s acroi l i ac joi nt and peri os t i t i s s houl d be s ought . The pres ence or abs ence of HLA-B27 may be hel pful i n di agnos i ng part i cul arl y di ffi cul t cas es . o
o
f. T herapy
(1) Goals of treatment are es s ent i al l y t he s ame as i n ankyl os i ng s pondyl i t i s [s ee IV B 1 f (1)].
(2) Exercise. Pat i ent s s houl d be advi s ed t o res t t o l es s en i nfl ammat i on and t o perform appropri at e exerci s es t hat al l ow pres ervat i on of joi nt funct i on and prevent i on of cont ract ures .
(3) Drug therapy
(a) NSAIDs. Thes e agent s are t he
Pa g e 2 3 3 3
ABC Ambe r CHM Conve rte r Tria l ve rsion, http://w w w .proce sste x t.com/a bcchm.html
mai ns t ay of t reat ment . However, as pi ri n i s rel at i vel y i neffect i ve.
(b) Corticosteroids. Occas i onal i nt ra-art i cul ar i ns t i l l at i on may be us eful i n t he management of part i cul ar joi nt s t hat do not res pond t o t reat ment wi t h NSAIDs .
(c) Second-line agents. Bot h azat hi opri ne and met hot rexat e have been us ed t o cont rol part i cul arl y s evere and chroni c di s eas e, and s ul fas al azi ne i s us ed for art hri t i s unres pons i ve t o NSAIDs . The effect s of t hes e medi cat i ons on di s eas e cours e are not yet known. HIV t es t i ng s houl d be cons i dered for pat i ent s wi t h react i ve art hri t i s s uffi ci ent l y s evere t o requi re i mmunos uppres s i ve medi cat i ons .
(d) Antibiotics. Tri al s of t et racycl i ne-l i ke drugs , us ed for 3-mont h peri ods earl y i n di s eas e as s oci at ed wi t h uret hri t i s , s ugges t t hat ant i bi ot i c t reat ment may l es s en t he s everi t y and chroni ci t y of art hri t i s .
P.466
o
o
g. Prognosis. Chroni c or recurrent di s eas e appears t o be common. In 60%–80% of pat i ent s wi t h
Pa g e 2 3 3 4
ABC Ambe r CHM Conve rte r Tria l ve rsion, http://w w w .proce sste x t.com/a bcchm.html
react i ve art hri t i s , s kel et al compl ai nt s , ext ras kel et al compl ai nt s , or bot h, recur or become chroni c. Perhaps as many as 25% of pat i ent s are funct i onal l y di s abl ed by t hei r i l l nes s . Long-t erm probl ems wi t h aort i c regurgi t at i on and conduct i on di s t urbances are unus ual but i ncreas e wi t h t he durat i on of t he di s eas e. Pat i ent s who have HLA-B27 ant i gen have more s acroi l i i t i s and are more l i kel y t o have recurrent or chroni c di s eas e.
3. Psoriatic arthritis o
o
a. Definition. Any form of i nfl ammat ory art hri t i s as s oci at ed wi t h ps ori as i s i s cal l ed ps ori at i c art hri t i s . Rheumat oi d fact or general l y i s abs ent .
o
o
b. Epidemiology
(1) Ps ori at i c art hri t i s occurs much more commonl y i n pat i ent s who have a fi rs t -degree rel at i ve wi t h t he di s order.
(2) Ps ori as i s occurs t wo t o t hree t i mes more commonl y i n pat i ent s wi t h art hri t i s t han i n t he normal popul at i on. Convers el y, as many as 10%–20% of pat i ent s wi t h ps ori as i s may have an i nfl ammat ory art hri t i s .
o
o
c. Etiology. Al t hough a heredi t ary et i ol ogy i s apparent i n ps ori at i c art hri t i s , i t s charact eri s t i cs are not ful l y unders t ood. HLA-B27 i s hi ghl y
Pa g e 2 3 3 5
ABC Ambe r CHM Conve rte r Tria l ve rsion, http://w w w .proce sste x t.com/a bcchm.html
as s oci at ed wi t h s acroi l i i t i s i n ps ori as i s , and HLA-B27, -B13, and -DR7 are i ndependent l y as s oci at ed wi t h art hri t i s and ps ori as i s at an i ncreas ed frequency. Unknown environment al fact ors al s o may be i mport ant i n di s eas e expres s i on. o
o
d. Clinical features
(1) Disease onset. Mos t pat i ent s wi t h ps ori at i c art hri t i s devel op t he di s eas e i n t hei r l at e t hi rt i es t o earl y fort i es . Mos t pat i ent s devel op s ki n l es i ons before t he art hri t i s , but as many as 16% devel op i nfl ammat ory art hri t i s before t he ps ori as i s .
(2) Patterns of arthritis. Fi ve pat t erns of ps ori at i c art hropat hy have been di s t i ngui s hed, al t hough t he t ypi cal pat i ent oft en has t he s kel et al mani fes t at i ons of s everal of t hes e pat t erns . Peri pheral joi nt and t endon i nvol vement may be more s evere i n t he upper t han i n t he l ower ext remi t i es .
(a) DIP arthritis i s t he cl as s i c form of ps ori at i c art hri t i s . It i s s t rongl y as s oci at ed wi t h t ypi cal fi ngernai l abnormal i t i es as wel l as DIP joi nt rednes s , s oft t i s s ue s wel l i ng, and eros i ve changes s een radi ographi cal l y i n a cl i ni cal pi ct ure res embl i ng pri mary eros i ve os t eoart hri t i s .
Pa g e 2 3 3 6
ABC Ambe r CHM Conve rte r Tria l ve rsion, http://w w w .proce sste x t.com/a bcchm.html
(b) Asymmetrical oligoarthritis. Large and s mal l joi nt ol i goart hri t i s i s anot her cl i ni cal form of ps ori at i c art hri t i s . Sausage digits are common i n t hi s form as a mani fes t at i on of joi nt and ent hes i s i nvol vement of a s i ngl e phal anx.
(c) Symmetrical polyarthropathy. A fai rl y s ymmet ri cal pol yart hropat hy can occur i n conjunct i on wi t h ps ori as i s , and i t can be di ffi cul t t o di s t i ngui s h from rheumat oi d art hri t i s . As much as 25% of t hes e pat i ent s may be s eropos i t i ve for rheumat oi d fact or.
(d) Arthritis mutilans i s an unus ual l y des t ruct i ve form of ps ori at i c art hri t i s becaus e of t he s evere peri art i cul ar bone res orpt i on t hat occurs i n t he s mal l fi nger joi nt s . “Tel es copi ng di gi t s ― charact eri ze t he end s t age of t hi s uncommon form of art hri t i s . W i des pread joi nt ankyl os i s al s o can occur.
(e) Sacroiliitis. As many as 20% of pat i ent s wi t h ps ori at i c art hri t i s can have cl i ni cal or radi ographi c evi dence of s acroi l i i t i s , us ual l y as ymmet ri cal . As cendi ng s pi nal s yndes mophyt es occur i n an as ymmet ri cal and pat chy fas hi on.
(3) Extra-articular features
Pa g e 2 3 3 7
ABC Ambe r CHM Conve rte r Tria l ve rsion, http://w w w .proce sste x t.com/a bcchm.html
(a) Skin. Art hri t i s i s more l i kel y t o occur i n pat i ent s wi t h s evere s ki n i nvol vement t han i n t hos e wi t h mi l d ps ori as i s , al t hough i t can occur i n t he pres ence of l ocal i zed cut aneous l es i ons . Ski n and joi nt fl ares of di s eas e can occur i n as s oci at i on wi t h each ot her or i ndependent l y i n i ndi vi dual pat i ent s .
(b) Nails. Nai l abnormal i t i es mos t commonl y occur i n conjunct i on wi t h DIP joi nt l es i ons ; however, nai l changes are not ed more frequent l y i n al l of t he peri pheral forms of ps ori at i c art hri t i s i n compari s on t o ps ori as i s uncompl i cat ed by art hri t i s . Mul t i pl e pitting, t rans vers e depres s i ons , and onycholysis are t he charact eri s t i c abnormal i t i es found.
(c) Ocular. The onl y ot her ext ra-art i cul ar feat ures t hat occur commonl y i n pat i ent s wi t h ps ori at i c art hri t i s are conjunct i vi t i s (i n 20% of cas es ) and ant eri or uvei t i s (i n 10% of cas es ).
P.467
o
o
e. Diagnosis
(1) Clinical considerations. An as s oci at i on
Pa g e 2 3 3 8
ABC Ambe r CHM Conve rte r Tria l ve rsion, http://w w w .proce sste x t.com/a bcchm.html
s houl d be made bet ween t he t ypi cal s ki n and nai l l es i ons and t he joi nt and s pi nal i nvol vement t o di agnos e ps ori at i c art hri t i s . The s ki n mani fes t at i ons may be s ubt l e, s o part i cul ar care mus t be t aken t o check t he el bows , s cal p, groi n, navel , and but t ock cl eft .
(2) Laboratory findings. Laborat ory i ndi cat ors of chroni c di s eas e (e.g., anemi a, t hrombocyt os i s ) t ypi cal l y are abs ent . Pat i ent s us ual l y are s eronegat i ve for rheumat oi d fact or, al t hough 25% of pat i ent s wi t h s ymmet ri cal pol yart hri t i s are s eropos i t i ve.
(3) Radiographic findings
(a) DIP joint erosions can l ead t o joi nt -s pace narrowi ng, and proxi mal phal angeal bone res orpt i on can l ead t o a “penci l -i n-cup― abnormal i t y s een i n art hri t i s mut i l ans . Thi s s evere des t ruct i on al s o can occur i n more proxi mal joi nt s .
(b) Spinal involvement. As ymmet ri cal s acroi l i i t i s and as ymmet ri cal pat chy s yndes mophyt es are s een and are s i mi l ar t o t hos e s een i n react i ve art hri t i s .
(c) Enthesopathy. Cal ci fi cat i ons at t endon and l i gament i ns ert i ons occur.
Pa g e 2 3 3 9
ABC Ambe r CHM Conve rte r Tria l ve rsion, http://w w w .proce sste x t.com/a bcchm.html
(4) Differential diagnosis
(a) Reactive arthritis i s di s t i ngui s hed from ps ori at i c art hri t i s by i t s charact eri s t i c ext ra-art i cul ar feat ures and i t s t endency t o i nvol ve t he l ower ext remi t i es more t han t he upper ext remi t i es . In ps ori at i c art hri t i s , upper ext remi t y peri pheral joi nt i nvol vement i s more common.
(b) Rheumatoid arthritis. Pat i ent s wi t h ps ori as i s who pres ent wi t h s ymmet ri cal pol yart hri t i s , s eropos i t i vi t y for rheumat oi d fact or, and rheumat oi d nodul es may be di agnos ed wi t h rheumat oi d art hri t i s . Ot her overl appi ng pres ent at i ons of t he t wo di s eas es may be di ffi cul t t o di agnos e.
o
o
f. T herapy. Drug t reat ment for ps ori at i c art hri t i s i s s i mi l ar t o t hat for t he ot her s pondyl oart hropat hi es i n t hat NSAIDs are t he corners t one of t herapy. An i mport ant di fference: Gol d can be admi ni s t ered i nt ramus cul arl y t o t reat s us t ai ned mani fes t at i ons of peri pheral art hri t i s . Sul fas al azi ne al s o i s effect i ve. Met hot rexat e i s oft en t he fi rs t agent us ed i f NSAIDs are not s uffi ci ent ; i t has been part i cul arl y s ucces s ful i n cont rol l i ng bot h t he s ki n di s eas e and refract ory art hri t i s . Ot her cyt ot oxi c drugs (e.g., azat hi opri ne) al s o have been us ed i n t reat ment of s evere s ki n and joi nt di s eas e. Bi ol ogi cal agent s (TNF i nhi bi t ors ) are now
Pa g e 2 3 4 0
ABC Ambe r CHM Conve rte r Tria l ve rsion, http://w w w .proce sste x t.com/a bcchm.html
FDA-approved for t he t reat ment of ps ori at i c art hri t i s and pl aque art hri t i s . o
o
g. Prognosis. Mos t pat i ent s wi t h ps ori at i c art hri t i s , except for t he 5% or s o who devel op art hri t i s mut i l ans , can avoi d s i gni fi cant deformi t y and remai n empl oyed.
4. Enteropathic arthropathies o
o
a. Definition. Infl ammat ory art hri t i s as s oci at ed wi t h ei t her Crohn's di s eas e or ul cerat i ve col i t i s i s t ypi fi ed by a s eronegat i ve, mi grat ory pol yart i cul ar i nvol vement t hat waxes and wanes wi t h t he act i vi t y of t he bowel di s eas e. Sacroi l i i t i s and s pondyl i t i s al s o can occur i n affect ed i ndi vi dual s .
o
o
b. Clinical syndromes
(1) Peripheral arthritis. In 10%–20% of pat i ent s wi t h s evere ul cerat i ve col i t i s or Crohn's di s eas e—us ual l y t hos e wi t h ot her ext ra-i nt es t i nal mani fes t at i ons —a predomi nant l y l ower ext remi t y art hri t i s or t endi ni t i s occurs i n as s oci at i on wi t h fl are-ups of bowel di s eas e. The art hri t i s oft en occurs abrupt l y and us ual l y remi t s compl et el y wi t hi n weeks . There i s no as s oci at i on wi t h HLA-B27 i n t hi s peri pheral art hri t i s . Treat ment i s di rect ed at cont rol of t he bowel di s eas e, al t hough NSAIDs or l ocal or s ys t emi c cort i cos t eroi ds may hel p cont rol t he art i cul ar
Pa g e 2 3 4 1
ABC Ambe r CHM Conve rte r Tria l ve rsion, http://w w w .proce sste x t.com/a bcchm.html
compl ai nt s .
(2) Spondylitis. The s pondyl i t i s of i nfl ammat ory bowel di s eas e i s as s oci at ed wi t h HLA-B27 i n about 50% of pat i ent s . The radi ographi c fi ndi ngs are t ypi cal of t hos e found i n pri mary ankyl os i ng s pondyl i t i s (i .e., s ymmet ri cal s acroi l i ac joi nt changes and, l es s commonl y, as cendi ng s ymmet ri cal s pondyl i t i s wi t hout s ki n l es i ons ). Approxi mat el y 5% of pat i ent s wi t h i nfl ammat ory bowel di s eas e devel op s acroi l i i t i s or s pondyl i t i s , but t he act i vi t y of t hes e l es i ons and t hat of t he bowel di s eas e are i ndependent of one anot her. NSAIDs us ual l y are effect i ve i n cont rol l i ng s ympt oms but can caus e i ncreas ed bowel compl ai nt s ; s omet i mes onl y nonacet yl at ed s al i cyl at es can be t ol erat ed by t hes e pat i ent s .
P.468
o
o
c. Differential diagnosis. W hi ppl e's di s eas e can be confus ed wi t h ent eropat hi c art hropat hi es , becaus e mos t pat i ent s mani fes t promi nent bowel and joi nt compl ai nt s . In fact , s acroi l i i t i s occurs i n about 20% of cas es . However, W hi ppl e's di s eas e i s qui t e rare and mos t commonl y mani fes t s i n mi ddl e-aged men as wei ght l os s , s ki n hyperpi gment at i on, l ymphadenopat hy, fever, and s ympt oms of mal abs orpt i on.
V. Crystal-Related Joint Diseases A. Gout
Pa g e 2 3 4 2
ABC Ambe r CHM Conve rte r Tria l ve rsion, http://w w w .proce sste x t.com/a bcchm.html
1. Definition. Gout i s a di s order of puri ne met abol i s m t hat i s charact eri zed by serum uric acid elevation (hyperuri cemi a) and urate deposition i n art i cul ar or ext ra-art i cul ar t i s s ues . El evat i on of s erum uri c aci d al one i s not s uffi ci ent for t he di agnos i s of gout ; i n fact , onl y 10% of pat i ent s wi t h hyperuri cemi a devel op gout . Some unknown fact or predi s pos es pat i ent s t o urat e depos i t i on and art i cul ar i nfl ammat i on i n t he s et t i ng of s us t ai ned hyperuri cemi a.
2. Etiologic classification of hyperuricemia. Gout i s charact eri zed by ei t her epi s odi c or cons t ant elevation of serum uric acid concent rat i on above 7 mg/dL. Pat i ent s wi t h el evat ed s erum uri c aci d can be cl as s i fi ed as overproducers or underexcreters of uri c aci d, dependi ng on t he amount of uri c aci d excret ed duri ng a 24-hour peri od. Exces s i ve di et ary i nt ake of puri nes can cont ri but e t o hyperuri cemi a i n bot h t ypes of pat i ent s . o
o
a. Overproducers, who make up approxi mat el y 10% of t he gout popul at i on, excret e more than 750–1000 mg uri c aci d per day on an unres t ri ct ed di et . Thes e pat i ent s s ynt hes i ze great er t han normal amount s of uri c aci d de novo from i nt ermedi at es or vi a breakdown of puri ne bas es from nucl ei c aci ds . The defect caus i ng uri c aci d overproduct i on can be primary (as s oci at ed wi t h puri ne pat hway enzymat i c defect s ) or secondary (i ncreas ed cel l t urnover as s oci at ed wi t h al cohol us e, hemat ol ogi c mal i gnanci es , chroni c hemol ys i s , or cancer chemot herapy).
o
Pa g e 2 3 4 3
ABC Ambe r CHM Conve rte r Tria l ve rsion, http://w w w .proce sste x t.com/a bcchm.html
o
b. Underexcreters, who cons t i t ut e approxi mat el y 90% of t he gout popul at i on, excret e less than 700 mg uri c aci d per day.
(1) The mos t common caus es of decreas ed uri c aci d excret i on are drug effects (e.g., di uret i cs , al cohol , and l ow-dos e as pi ri n i nt erference wi t h t ubul ar handl i ng of urat e) and renal disease (e.g., chroni c renal fai l ure and l ead nephropat hy) [s ee Chapt er 6, Part I: XII B 2a]. Subt l e renal t ubul ar defect s i n urat e handl i ng al s o may be inherited and predi s pos e t o underexcret i on.
(2) Thi s group includes patients with combined defects, becaus e overproducers of uri c aci d al s o may be underexcret ers . The decreas ed renal excret i on of uri c aci d i s t he bas i s for hyperuri cemi a i n t hes e i ndi vi dual s .
(3) Mul t i pl e fact ors cont ri but e t o t he occurrence of gout i n cardiac, renal, and liver transplant recipients. Underl yi ng renal i ns uffi ci ency, us e of di uret i cs , and cycl os pori ne i mmunos uppres s i ve t herapy are t he mos t common predi s pos i ng fact ors . Affect ed pat i ent s t end t o have a rapi dl y progres s i ve form of gout wi t h t he format i on of ext ens i ve t ophi .
3. Associated conditions. The fol l owi ng condi t i ons occur more commonl y i n pat i ent s wi t h gout but are not known
Pa g e 2 3 4 4
ABC Ambe r CHM Conve rte r Tria l ve rsion, http://w w w .proce sste x t.com/a bcchm.html
t o be caus al . o
o
a. Obesity. Serum uri c aci d l evel ri s es wi t h body wei ght . Gout i s s i gni fi cant l y more common i n i ndi vi dual s who are more t han 15% overwei ght , part l y becaus e of decreas es i n urat e excret i on.
o
o
b. Diabetes mellitus. Impai red gl ucos e t ol erance i s common i n gout and may be a funct i on of obes i t y.
o
o
c. Hypertension. Al t hough hypert ens i on i s common i n pat i ent s wi t h gout , no i ndependent correl at i on exi s t s bet ween bl ood pres s ure and s erum uri c aci d l evel . Obes i t y probabl y i s res pons i bl e for a hi gh rat e of hypert ens i on i n pat i ent s wi t h gout .
o
o
d. Hyperlipidemias (t ypes II and IV). Increas ed pl as ma concent rat i ons of s ome l i pi ds are common i n pat i ent s wi t h gout ; however, di et , al cohol i nt ake, and body wei ght s eem t o be more i mport ant as s oci at i ons .
o
o
e. Atherosclerosis. Deat h i n pat i ent s wi t h gout i s commonl y at t ri but abl e t o cardi ovas cul ar or cerebrovas cul ar di s eas es . However, t he previ ous l y ment i oned ri s k fact ors t hat commonl y occur i n pat i ent s wi t h gout s eem t o expl ai n t he t endency for accel erat ed at herogenes i s .
P.469
Pa g e 2 3 4 5
ABC Ambe r CHM Conve rte r Tria l ve rsion, http://w w w .proce sste x t.com/a bcchm.html
4. Clinical stages of gout o
o
a. Asymptomatic hyperuricemia i s charact eri zed by an i ncreas ed s erum uri c aci d l evel i n t he abs ence of cl i ni cal evi dence of depos i t i on di s eas e (i .e., art hri t i s , t ophi , nephropat hy, or uri c aci d s t ones ).
(1) Hyperuricemia. The ri s k of acut e gout y art hri t i s or nephrol i t hi as i s i ncreas es as t he s erum uri c aci d concent rat i on i ncreas es . However, mos t pat i ent s do not devel op ei t her of t hes e condi t i ons .
(2) Hyperuricosuria. The ri s k of uri c aci d s t one format i on i n pat i ent s wi t h hyperuri cemi a i s mos t cl os el y rel at ed t o a uri nary uri c aci d excret i on exceedi ng 1000 mg/day. Thes e pat i ent s al s o are at ri s k for acute obstructive uropathy, a form of acut e renal fai l ure occurri ng mos t oft en fol l owi ng combi nat i on chemot herapy for cancer. Large puri ne l oads l ead t o s udden i ncreas es i n s erum uri c aci d l evel s , wi t h s ubs equent preci pi t at i on of uri c aci d crys t al s i n t he col l ect i ng t ubul es and uret ers .
o
o
b. Acute gouty arthritis, t he s econd s t age and pri mary mani fes t at i on of gout , i s an ext remel y pai nful , acut e-ons et art hri t i s .
Pa g e 2 3 4 6
ABC Amber CHM Converter Trial version, http://www.processtext.com/abcchm.html
(1) T ypical patient. Mos t pat i ent s (80%–90%) are mi ddl e-aged or el derl y men who have had s us t ai ned as ympt omat i c hyperuri cemi a for 20–30 years before t he fi rs t at t ack. W omen s eem t o be s pared unt i l menopaus e, perhaps vi a an es t rogen effect on uri c aci d cl earance. Ons et of acut e gout y art hri t i s i n t he t eens or t went i es i s unus ual and mos t oft en as s oci at ed wi t h a pri mary or s econdary caus e of uri c aci d overproduct i on.
(2) T ypical attack
(a) Presentation. A monoart i cul ar, l ower ext remi t y pres ent at i on i s mos t common wi t h 50% of pat i ent s experi enci ng t hei r fi rs t at t ack i n t he fi rs t met at ars ophal angeal (MTP) joi nt (cal l ed podagra) ( Onl i ne Fi gure 10-7). Many at t acks occur s uddenl y at ni ght , wi t h rapi d evol ut i on of joi nt eryt hema, s wel l i ng, t endernes s , and warmt h. Int ens e joi nt i nfl ammat i on can ext end i nt o t he s oft t i s s ues and mi mi c cel l ul i t i s or phl ebi t i s . Fever can occur i n s evere at t acks .
Page 2347
ABC Ambe r CHM Conve rte r Tria l ve rsion, http://w w w .proce sste x t.com/a bcchm.html
ONLINE FIGURE 10-7 Gout . Podagra i s t he cl as s i c pres ent at i on of gout , i nvol vi ng eryt hema and s wel l i ng of met at ars ophal angeal joi nt . (ACR Sl i de Col l ect i on of t he Rheumat i c Di s eas es , 3rd ed., 2004:9114040.)
(b) Course. At t acks us ual l y res ol ve i n a few days , al t hough s ome can ext end over s everal weeks . The joi nt us ual l y ret urns t o normal bet ween at t acks . Pol yart i cul ar i nvol vement can occur i n s ome cas es , and a t ypi cal progres s i on from monoart i cul ar t o pol yart i cul ar i nvol vement occurs by ext ens i on t o adjacent joi nt s .
(3) Pathogenesis of acute attacks (Onl i ne Fi gure 10-8)
Pa g e 2 3 4 8
ABC Ambe r CHM Conve rte r Tria l ve rsion, http://w w w .proce sste x t.com/a bcchm.html
ONLINE FIGURE 10-8 Pos t ul at ed pat hogenes i s of crys t al -i nduced joi nt i nfl ammat i on. Ig, i mmunogl obul i n; C3 and C4, compl ement component s C3 and C4; LP, l i poprot ei n.
(a) Sus t ai ned hyperuri cemi a or a rapi d fl uct uat i on i n t he l evel of s erum uri c aci d (as occurs i n t he s et t i ng of dehydrat i on, rapi d rehydrat i on, or wi t h t he i ni t i at i on of uri cos uri c t herapy), may l ead t o t he devel opment of microtophi (i .e., s mal l crys t al aggregat es ) i n t he s ynovi al membrane and cart i l age.
Pa g e 2 3 4 9
ABC Ambe r CHM Conve rte r Tria l ve rsion, http://w w w .proce sste x t.com/a bcchm.html
(b) Through s everal mechani s ms , mi crot ophi are di s rupt ed and crys t al s are rel eas ed i nt o t he joi nt s pace. Spont aneous urat e crys t al l i zat i on can al s o occur i n condi t i ons of urat e s upers at urat i on of joi nt fl ui d. (Trauma, joi nt t emperat ure changes , and fl uct uat i ons i n s erum or s ynovi al fl ui d uri c aci d concent rat i on are pot ent i al i ni t i at ors of t hes e proces s es .)
(c) Urat e crys t al s are coat ed by immunoglobulins and complement components. Thes e adherent prot ei ns enhance phagocyt os i s by neut rophi l s .
(d) Phagos omes i n t he crys t al -cont ai ni ng neut rophi l s fus e wi t h l ys os omes , and t he l ys os omal enzymes di ges t t he prot ei n coat i ng of t he crys t al s . The naked crys t al s t hen apparent l y di s rupt t he phagos omal membranes .
(e) Neut rophi l s are damaged by t he crys t al s , and l ys os omal enzymes are rel eas ed i nt o s ynovi al fl ui d, pot ent i at i ng i nfl ammat i on.
(f) Inflammatory mediators (e.g., IL-6) can be rel eas ed from s ynovi al macrophages expos ed t o uri c aci d crys t al s and may be res pons i bl e for
Pa g e 2 3 5 0
ABC Ambe r CHM Conve rte r Tria l ve rsion, http://w w w .proce sste x t.com/a bcchm.html
ext ens i on of i nfl ammat i on t o ot her joi nt s and i nt o s oft t i s s ues .
(g) Lipoproteins can ent er i nfl amed s ynovi al membranes and at t ach t o t he crys t al s , downregul at i ng t he i nfl ammat ory proces s .
o
o
c. Intercritical gout, t he t hi rd s t age of gout , i s an as ympt omat i c peri od aft er t he i ni t i al at t ack. Thi s s t age may be i nt errupt ed by new acut e at t acks .
(1) Recurrence of monoarticular attacks. About 7% of pat i ent s never experi ence a new at t ack of acut e gout y art hri t i s aft er t he fi rs t epi s ode. However, 62% experi ence a recurrence wi t hi n 1 year. Typi cal l y, pat i ent s are as ympt omat i c bet ween at t acks , but at t acks event ual l y become more frequent and abat e more gradual l y i f urat e depos i t i on remai ns unt reat ed over t i me.
(2) Disease progression. At t acks t end t o become pol yart i cul ar and more s evere over t i me. Some pat i ent s devel op a chroni c i nfl ammat ory art hri t i s wi t hout as ympt omat i c i nt erval s —a condi t i on t hat may be di ffi cul t t o di s t i ngui s h from rheumat oi d art hri t i s . Tophaceous gout t ypi cal l y occurs over a 10- t o 20-year peri od of unt reat ed urat e depos i t i on.
o
o
d. Chronic tophaceous gout, whi ch devel ops i n
Pa g e 2 3 5 1
ABC Ambe r CHM Conve rte r Tria l ve rsion, http://w w w .proce sste x t.com/a bcchm.html
unt reat ed pat i ent s , i s t he fi nal s t age of gout . The t ophus i s a col l ect i on of urat e crys t al mas s es s urrounded by i nfl ammat ory cel l s and vari abl e fi bros i s . Tophi t end t o devel op i n areas t hat are l ower i n t emperat ure, whi ch di mi ni s hes t he s ol ubi l i t y of t he crys t al s .
(1) T ypical locations of tophaceous deposits
(a) The pinna of the external ear i s a pot ent i al s i t e of t ophus devel opment , al t hough depos i t i on here i s uncommon.
(b) Other common locations are t he s urfaces of chroni cal l y i nvol ved joi nt s and s ubchondral bone as wel l as ext ens or s urfaces of t he forearms , ol ecranon burs ae, and t he i nfrapat el l ar and Achi l l es t endons .
(2) Pathogenesis of tophaceous gout
(a) Al t hough mi crot ophi may form i n joi nt s earl y i n t he urat e depos i t i on phas e, aggregat es t hat are l arge enough t o be pal pabl e or t o caus e anat omi c deformi t i es t ake years t o devel op. The rat e of t ophus format i on i s di rect l y rel at ed t o t he s everi t y and durat i on of hyperuri cemi a i n pat i ent s wi t h gout . Tophi do not occur i n pat i ent s wi t h as ympt omat i c hyperuri cemi a.
Pa g e 2 3 5 2
ABC Ambe r CHM Conve rte r Tria l ve rsion, http://w w w .proce sste x t.com/a bcchm.html
(b) Eros i on of cart i l age and adjacent s ubchondral bone occurs due t o di s pl acement of normal t i s s ue by t he t ophus and by t he i nfl ammat ory react i on to it.
5. Renal complications may ari s e at any s t age of gout , but nephrol i t hi as i s i s t he onl y common cl i ni cal pres ent at i on of renal i nvol vement . Prot ei nuri a and i mpai red abi l i t y t o concent rat e uri ne P.470
rel at ed t o urat e depos i t i on i n t he renal i nt ers t i t i um have been des cri bed i n gout pat i ent s (s ee Chapt er 6, Part I: XII B 3).
6. Diagnosis o
o
a. Acute gouty arthritis—laboratory findings
(1) Serum findings. The s erum uri c aci d val ue oft en i s not hel pful i n t he cl i ni cal di agnos i s of acut e gout . Serum uri c aci d concent rat i on i s normal i n at l eas t 10% of pat i ent s at t he t i me of an acut e at t ack, and an el evat ed s erum uri c aci d l evel i s not s peci fi c for acut e gout .
(2) Synovial fluid findings. The demons t rat i on of urate crystals, es peci al l y i nt racel l ul ar crys t al s , i n s ynovi al fl ui d i s
Pa g e 2 3 5 3
ABC Ambe r CHM Conve rte r Tria l ve rsion, http://w w w .proce sste x t.com/a bcchm.html
di agnos t i c. Thes e crystals characteristically are needle-shaped and negatively birefringent i n red-compens at ed, pol ari z ed l i ght , and t hey may be pres ent i n neut rophi l s duri ng an acut e at t ack. Synovi al fl ui d whi t e bl ood cel l (W BC) count s of 3
10,000–60,000/mm (predomi nant l y neut rophi l s ) al s o are common i n acut e at t acks . o
o
b. Chronic tophaceous gout
(1) Physical appearance. Tophi are fi rm, movabl e, and cream-col ored or yel l owi s h i f s uperfi ci al l y l ocat ed. If t hey ul cerat e, a chal ky mat eri al i s ext ruded.
(2) Radiographic findings. Tophaceous depos i t s appear as wel l -defi ned, l arge eros i ons (punched-out eros i ons ) of t he s ubchondral bone. Thes e eros i ons are mos t common at t he fi rs t MTP joi nt and at t he bas es and heads of phal anges ; however, any art i cul at i on can be affect ed. Typi cal gout y eros i ons have an overhanging edge of s ubchondral new bone format i on. Peri art i cul ar os t eopeni a i s abs ent .
(3) Aspiration. Tophi can be as pi rat ed and crys t al s demons t rat ed by pol ari zed mi cros copy.
Pa g e 2 3 5 4
ABC Ambe r CHM Conve rte r Tria l ve rsion, http://w w w .proce sste x t.com/a bcchm.html
7. T herapy. In al l s t ages of gout , s econdary caus es of hyperuri cemi a (e.g., medi cat i ons , obes i t y, exces s di et ary puri ne i nt ake, al cohol i nt ake, ot her di s eas e) s houl d be al t ered i f pos s i bl e. o
o
a. Asymptomatic hyperuricemia. In general , pat i ent s wi t hout evi dence of urat e depos i t i on do not requi re t reat ment ot her t han correct i on of underl yi ng caus es . Pat i ent s wi t h uri c aci d el evat i ons t hat chroni cal l y exceed 10 mg/dL have a great er t han 90% chance of developi ng acut e gout at s ome t i me, but mos t phys i ci ans wai t for an acut e at t ack t o begi n t reat ment .
o
o
b. Acute gouty arthritis. Drug t reat ment of t he acut e at t ack i s mos t effect i ve when s t art ed very earl y aft er s ympt oms begi n. As wi t h any i nfl ammat ory art hri t i s , res t or i mmobi l i zat i on of t he i nvol ved joi nt i s an i mport ant adjunct t o t reat ment .
(1) Colchicine, whi ch i nhi bi t s neut rophi l chemot axi s and i nfl ammat ory medi at or rel eas e by s uppres s i ng phos phol i pas e A 2 , has been us ed i nt ravenous l y t o t reat acut e at t acks . However, i t i s rarel y us ed t oday, as i t has been as s oci at ed wi t h i rrevers i bl e bone marrow s uppres s i on. Caut i on i s neces s ary i n el derl y pat i ent s and i n t hos e wi t h renal or hepat i c i mpai rment due t o i ncreas ed bone marrow t oxi ci t y and neuromyot oxi ci t y. Col chi ci ne can be us ed oral l y for acut e at t acks , but naus ea, vomi t i ng, and di arrhea oft en appear before t herapy i s s ucces s ful , s o t hat NSAIDs and
Pa g e 2 3 5 5
ABC Ambe r CHM Conve rte r Tria l ve rsion, http://w w w .proce sste x t.com/a bcchm.html
cort i cos t eroi ds are us ual l y preferred.
(2) NSAIDs oft en are us ed i n hi gh but qui ckl y t apered dos es t o t reat acut e at t acks . Any of t hes e agent s can be us ed, al t hough drugs t hat affect uri c aci d cl earance (e.g., s al i cyl at es , di fl uni s al ) s houl d be avoi ded, becaus e any fl uct uat i on i n s erum urat e can prol ong acut e at t acks . Caut i on i s warrant ed when us i ng NSAIDs i n t he pres ence of gas t roi nt es t i nal , hepat i c, or renal di s eas e. Indomet haci n, i f t ol erat ed, appears t o be t he mos t acut el y effi caci ous agent .
(3) Corticosteroids, oral and i nt ra-art i cul ar, can be us ed t o t reat acut e at t acks , part i cul arl y when NSAIDs are cont rai ndi cat ed.
(4) Drugs that alter serum uric acid concentrations (e.g., allopurinol, probenecid ) s houl d be avoi ded duri ng acut e at t acks becaus e rai s i ng or l oweri ng s erum uri c aci d can prol ong at t acks . In pat i ent s al ready t aki ng s uch drugs , t he dos age s houl d not be al t ered.
o
o
c. Intercritical gout. Prophyl act i c t reat ment wi t h s mal l dos es of col chi ci ne (0.6 mg once or t wi ce dai l y) or s mal l dos es of an NSAID can be us ed t o fores t al l new at t acks .
o
o
d. Chronic or tophaceous gout. Therapy for
Pa g e 2 3 5 6
ABC Ambe r CHM Conve rte r Tria l ve rsion, http://w w w .proce sste x t.com/a bcchm.html
chroni c gout cent ers on cont rol of hyperuri cemi a. Drugs t hat i ncreas e renal uri c aci d excret i on (uri cos uri c drugs ) or t hat decreas e uri c aci d product i on (xant hi ne oxi das e i nhi bi t ors ) are avai l abl e. The ai m of t hi s t reat ment i s t o reduce s erum urat e bel ow 6 mg/dL, t o al l ow reduct i on i n s erum s upers at urat i on by urat e and P.471
mobi l i zat i on of t i s s ue uri c aci d depos i t s . Pat i ent s who have had a recent acut e at t ack s houl d recei ve l ow-dos e col chi ci ne or an NSAID t o prevent new at t acks caus ed by fl uct uat i on i n s erum urat e.
(1) Uricosuric drugs (e.g., probenecid, sulfinpyrazone) can be us ed i n pat i ent s who excret e l es s t han 700 mg uri c aci d dai l y, who have normal renal funct i on, and who have no hi s t ory of uri nary s t ones .
(2) Xanthine oxidase inhibitors i ncl ude allopurinol, whi ch i s an anal og of hypoxant hi ne. Thi s drug i nhi bi t s de novo uri c aci d s ynt hes i s and compet i t i vel y i nhi bi t s xant hi ne oxi das e vi a enzymat i c convers i on t o oxypuri nol . Inhi bi t i on of uri c aci d s ynt hes i s wi t h xant hi ne oxi das e i nhi bi t ors i s preferred i n pat i ent s wi t h urat e excret i on great er t han 1000 mg/day, creat i ni ne cl earance l es s t han 30 mL/mi nut e, t ophaceous gout , or a hi s t ory of nephrol i t hi as i s . Dos ages are reduced i n t he pres ence of renal fai l ure t o avoi d t oxi ci t y. The mos t common s i de effect s of al l opuri nol are
Pa g e 2 3 5 7
ABC Ambe r CHM Conve rte r Tria l ve rsion, http://w w w .proce sste x t.com/a bcchm.html
dys peps i a, di arrhea, and ras h, whi ch occur i n 3%–10% of pat i ent s . A ful l -bl own hypers ens i t i vi t y s yndrome can occur rarel y, wi t h a mort al i t y rat e around 20%–30%.
(3) Import ant t reat ment i s s ues :
(a) It i s i mport ant t o us e col chi ci ne, NSAIDs , or s t eroi ds whi l e i ns t i t ut i ng al l opuri nol t herapy t o avoi d an acut e at t ack of gout i nduced by fl uct uat i on i n s erum urat e l evel s .
(b) Can us e col chi ci ne chroni cal l y i n pat i ent s unabl e t o t ol erat e al l opuri nol .
(c) Beware of us e of col chi ci ne i n pat i ent s wi t h renal i ns uffi ci ency, becaus e s ecret i on i s reduced and s evere t oxi ci t y may occur.
(d) May us e chroni c l ow-dos e s t eroi d t reat ment i n pat i ent s wi t h chroni c gout and chroni c renal i ns uffi ci ency.
B. Calcium pyrophosphate dihydrate (CPPD) deposition disease
1. Definition. Depos i t i on of CPPD crys t al s i n cart i l age and peri art i cul ar connect i ve t i s s ues can caus e a gamut
Pa g e 2 3 5 8
ABC Ambe r CHM Conve rte r Tria l ve rsion, http://w w w .proce sste x t.com/a bcchm.html
of art i cul ar mani fes t at i ons , rangi ng from as ympt omat i c depos i t i on t o acut e and chroni c i nfl ammat ory art hri t i s . The acut e form of CPPD depos i t i on di s eas e i s commonl y cal l ed pseudogout.
2. Etiologic classification o
o
a. Hereditary CPPD disease. A hi gh preval ence of CPPD di s eas e has been not ed i n many fami l i es , wi t h aut os omal domi nant t rans mi s s i on t he t ypi cal pat t ern. Secondary met abol i c as s oci at i ons wi t h CPPD di s eas e t ypi cal l y are not pres ent i n t hes e fami l i es .
o
o
b. Osteoarthritis. Chondrocal ci nos i s and CPPD crys t al s can occur as a res ul t of s evere os t eoart hri t i s . CPPD di s eas e al s o can caus e os t eoart hri t i s by damagi ng cart i l age.
o
o
c. CPPD disease associated with metabolic disorders. Correct i on of underl yi ng met abol i c di s orders , i f pos s i bl e, does not s eem t o al t er t he progres s i on of CPPD di s eas e. CPPD di s eas e occurs at a hi gher-t han-expect ed frequency i n as s oci at i on wi t h cert ai n di s eas es and condi t i ons . Pot ent i al abnormal i t i es of cal ci um, phos phorus , or cart i l age met abol i s m can expl ai n t he as s oci at i ons , whi ch i ncl ude:
(1) Hyperparat hyroi di s m
Pa g e 2 3 5 9
ABC Ambe r CHM Conve rte r Tria l ve rsion, http://w w w .proce sste x t.com/a bcchm.html
(2) Hemochromat os i s
(3) Hypot hyroi di s m
(4) Hypophos phat as i a
(5) Hypomagnes emi a
(6) W i l s on's di s eas e
3. Pathogenesis (s ee Onl i ne Fi gure 10-8) o
a.
Crystal deposition
(1) Site. The i ni t i al s i t e of CPPD depos i t i on appears t o be art i cul ar cart i l age s urroundi ng l acunae i n t he mi dzone. Lat er, depos i t i on occurs i n cl eft s of degenerat ed cart i l age and i n s cat t ered foci i n t he cart i l age mat ri x and s ynovi al membrane, event ual l y formi ng l arge crys t al l i ne mas s es .
(2) Process. An al t erat i on of cart i l age ground s ubs t ance, t he i oni c compos i t i on of t he mat ri x (i .e., cal ci um and pyrophos phat e), or a combi nat i on of bot h i s requi red for crys t al l i zat i on. Pos s i bl y, an al t ered condi t i on of t he mat ri x (e.g., removal of an i nhi bi t i ng agent or addi t i on of a nucl eat i ng agent )
Pa g e 2 3 6 0
ABC Ambe r CHM Conve rte r Tria l ve rsion, http://w w w .proce sste x t.com/a bcchm.html
al l ows crys t al s t o form i n t he mi croenvi ronment around t he chondrocyt e as pyrophos phat e i s rel eas ed from t he cel l . A cel l s urface ect oenzyme, nucl eos i de t ri phos phat e pyrophos phohydrol as e (NTP-PPH), generat es pyrophos phat e i n t he proces s of s cavengi ng ext racel l ul ar adenos i ne t ri phos phat e (ATP); t he i ncreas ed energy requi rement of great er prot eogl ycan s ynt hes i s i n os t eoart hri t i s t hus may l ead t o CPPD product i on. o
b.
Crystal-mediated joint damage. CPPD depos i t s s t i ffen t he
cart i l age, i mpai ri ng i t s wei ght -beari ng propert i es and accel erat i ng os t eoart hri t i c change. The acut e art hri t i c at t acks are bel i eved t o be i nduced by t he rel eas e of crys t al s from t he cart i l age i nt o t he joi nt s pace.
(1) Factors mediating crystal release
(a) Matrix loosening. CPPD crys t al s i n cart i l age exi s t i n equi l i bri um wi t h s ynovi al fl ui d cal ci um and pyrophos phat e concent rat i ons . Decreas es i n s erum cal ci um concent rat i on can l ead t o decreas es i n s ynovi al fl ui d pyrophos phat e l evel s and s ol ubi l i zat i on of joi nt CPPD crys t al s as equi l i bri um i s res t ored. Los s of margi nal CPPD depos i t s may caus e t he ent i re depos i t t o l oos en i n t he mat ri x, wi t h s ubs equent rel eas e of abundant CPPD crys t al s i nt o t he joi nt s pace. Thus , fl uct uat i ons of s erum cal ci um concent rat i on i n t he s et t i ng of acut e medi cal i l l nes s or duri ng t he
Pa g e 2 3 6 1
ABC Ambe r CHM Conve rte r Tria l ve rsion, http://w w w .proce sste x t.com/a bcchm.html
peri operat i ve peri od may i ni t i at e acut e crys t al rel eas e.
(b) Loss of matrix. Enzymat i c eros i on of cart i l age due t o an as s oci at ed i nfl ammat ory art hri t i s (e.g., i nfect i ous art hri t i s ) al s o may caus e crys t al rel eas e.
(c) Biomechanical forces. Impai red di s s i pat i on of wei ght -beari ng forces on cart i l age al s o may l ead t o crys t al l oos eni ng and rel eas e i nt o t he joi nt s pace.
(2) Factors affecting acute attacks
(a) Inflammatory response. Neut rophi l s are at t ract ed t o crys t al s coat ed wi t h IgG, compl ement , fi bronect i n, fi bri nogen, or ki ni nogen. Crys t al s are i nges t ed, caus i ng t he rel eas e of i nfl ammat ory medi at ors s uch as pros t agl andi ns and col l agenas e.
(b) Degree of response. CPPD crys t al s are s omewhat l es s i nfl ammat ory t han urat e crys t al s . General l y, fewer are rel eas ed i n t he acut e at t ack, t he crys t al s ads orb i nfl ammat ory prot ei ns l es s wel l , and CPPD crys t al s are not membranol yt i c.
o
Pa g e 2 3 6 2
ABC Ambe r CHM Conve rte r Tria l ve rsion, http://w w w .proce sste x t.com/a bcchm.html o
4. Clinical syndromes
a. Pseudogout account s for 25% of cas es of CPPD di s eas e. Acut e s wel l i ng, pai n, s t i ffnes s , and eryt hema devel op i n previ ous l y as ympt omat i c joi nt s . The knee i s mos t commonl y i nvol ved (50% of cas es ), but al mos t any s ynovi al joi nt , i ncl udi ng t he fi rs t MTP joi nt , can be i nvol ved. Spread t o adjacent joi nt s can occur, and preci pi t at i on of at t acks by acut e medi cal or s urgi cal i l l nes s i s P.472
common, occurri ng i n 10%–20% of cas es . Sys t emi c fi ndi ngs s uch as fever and l eukocyt os i s may be pres ent , es peci al l y i n el derl y pat i ent s . Joi nt s t ypi cal l y ret urn t o normal bet ween at t acks .
b. Pseudo–rheumatoid arthritis account s for 5% of cas es of CPPD di s eas e. In s ome pat i ent s , a s mol deri ng chroni c art hropat hy can occur. Subacut e epi s odes of pai n and s wel l i ng i n one or more joi nt s can be s uperi mpos ed on a more chroni c pi ct ure of prol onged morni ng s t i ffnes s , fat i gue, s ynovi al t hi ckeni ng, and progres s i ve deformi t i es .
c. Pseudo-osteoarthritis account s for 50% of cas es of CPPD di s eas e. Affect ed pat i ent s pres ent wi t h a cl i ni cal and radi ographi c pi ct ure s i mi l ar t o t hat of degenerat i ve joi nt di s eas e, al t hough about hal f have
Pa g e 2 3 6 3
ABC Ambe r CHM Conve rte r Tria l ve rsion, http://w w w .proce sste x t.com/a bcchm.html
s uperi mpos ed acut e at t acks of ps eudogout . Fl exi on cont ract ures are more common t han i n t ypi cal os t eoart hri t i s , and bi l at eral knee varus deformi t i es or i s ol at ed pat el l ofemoral art hri t i s may occur more commonl y t han i n os t eoart hri t i s . A s mal l percent age of t hes e pat i ent s may have s uch s evere joi nt des t ruct i on (e.g., of t he s houl ders or knees ) t hat t he cl i ni cal and radi ographi c appearance i s t hat of a neuropathic joint disorder, even i n t he abs ence of underl yi ng neurol ogi c di s eas e or apparent propri ocept i ve defi ci t .
d. Asymptomatic CPPD di s eas e occurs i n 20% of cas es . Thes e pat i ent s do not have joi nt pai n; t he di s eas e t ypi cal l y i s uncovered by t he fi ndi ng of as ympt omat i c chondrocal ci nos i s on radi ography. The preval ence of t hi s as ympt omat i c di s eas e, as wel l as ot her cl i ni cal l y evi dent forms of CPPD di s eas e, i ncreas es wi t h age.
o
o
5. Diagnosis. The fi ndi ng of t ypi cal crys t al s on s ynovi al fl ui d anal ys i s i s di agnos t i c of ps eudogout . Chondrocal ci nos i s s een on radi ography i s evi dence for a di agnos i s of CPPD depos i t i on. However, chondrocal ci nos i s may be pres ent i n pat i ent s who never devel op acut e ps eudogout .
a. Synovial fluid findings. Chunky, rhomboi d crystals that exhibit weakly positive birefringence i n red-compens at ed, pol ari z ed l i ght are t he hal l mark of t he acut e art hri t i s
Pa g e 2 3 6 4
ABC Ambe r CHM Conve rte r Tria l ve rsion, http://w w w .proce sste x t.com/a bcchm.html
s yndromes as s oci at ed wi t h CPPD di s eas e. Thes e may be i nt racel l ul ar (i n neut rophi l s ) or ext racel l ul ar, but t hey us ual l y are much l es s preval ent t han i n a t ypi cal gout -i nvol ved joi nt . Synovi al fl ui d l eukocyt os i s of 3
10,000–20,000 cel l s /mm (mos t l y neut rophi l s ) i s t ypi cal .
b. Radiographic findings
(1) Chondrocalcinosis. Cal ci fi cat i on of art i cul ar hyal i ne cart i l age, fi brocart i l age (mos t commonl y i n t he knee meni s ci , i nt ervert ebral di s k annul i , s ymphys i s pubi s , and wri s t t ri angul ar fi brocart i l age), s ynovi al membrane, t endons , and burs ae can occur, us ual l y i n a s t i ppl ed, l i near fas hi on.
(2) Osteoarthritis. Os t eoart hri t i c changes i n at ypi cal joi nt s (e.g., wri s t , el bow, and s houl der and MCP joi nt s ) s ugges t CPPD di s eas e. Subchondral bone cys t s may be more ext ens i ve i n radi ographs of joi nt s affect ed by CPPD di s eas e, and hook-s haped os t eophyt es charact eri s t i cal l y are pres ent wi t h MCP i nvol vement .
(3) Pseudoneuropathic joint. Radi ographi c fi ndi ngs t ypi cal of neuropat hi c joi nt di s orders , i ncl udi ng ext reme joi nt di s organi zat i on and bone
Pa g e 2 3 6 5
ABC Ambe r CHM Conve rte r Tria l ve rsion, http://w w w .proce sste x t.com/a bcchm.html
fragment s , can be found i n s evere cas es of CPPD di s eas e.
c. When to suspect CPPD
(1) Clinical or radiographic evidence of osteoarthritis i n joi nt s not us ual l y i nvol ved by os t eoart hri t i s s houl d s ugges t CPPD di s eas e as an al t ernat i ve di agnos i s .
(2) Attacks of acute inflammatory arthritis i n a s et t i ng of apparent os t eoart hri t i s s houl d s ugges t CPPD di s eas e.
(3) Acute arthritis occurring shortly after a medical illness or surgical procedure s houl d s ugges t a crys t al -medi at ed art hri t i s s uch as gout or ps eudogout .
(4) Radiographic changes more typical of osteoarthritis i n pat i ent s t hought t o have rheumat oi d art hri t i s s houl d s ugges t CPPD di s eas e as an al t ernat i ve di agnos i s .
(5) The presence of diseases commonly associated with CPPD disease (e.g., hyperparat hyroi di s m,
Pa g e 2 3 6 6
ABC Ambe r CHM Conve rte r Tria l ve rsion, http://w w w .proce sste x t.com/a bcchm.html
hemochromat os i s ) s houl d s ugges t CPPD di s eas e as a pos s i bl e caus e of any joi nt mani fes t at i ons . In peopl e younger t han 55 years of age, or i n t hos e wi t h recurrent , pol yart i cul ar di s eas e, an as s oci at ed met abol i c di s eas e i s more l i kel y. In t hes e pat i ent s , det ermi nat i ons of cal ci um, magnes i um, al kal i ne phos phat as e, cerul opl as mi n, and ferri t i n l evel s as wel l as l i ver and t hyroi d funct i on t es t s s houl d be cons i dered.
(6) Neuropathic joint presentations s houl d prompt i nves t i gat i on for CPPD di s eas e as wel l as pot ent i al l y as s oci at ed neurol ogi c di s orders .
P.473
o
o
6. T herapy
a. Acute attacks. Typi cal t reat ment i nvol ves as pi rat i on of i nfl ammat ory joi nt fl ui d, i nt ra-art i cul ar i nject i on of cort i cos t eroi d, and us e of NSAIDs .
b. Prophylaxis. No cl earl y effect i ve regi men i s avai l abl e, al t hough NSAIDs and, rarel y, l ow-dos e col chi ci ne are us ed.
C. Hydroxyapatite arthritis Hydroxyapat i t e crys t al s , t he t ypi cal compl exed form of cal ci um i n
Pa g e 2 3 6 7
ABC Ambe r CHM Conve rte r Tria l ve rsion, http://w w w .proce sste x t.com/a bcchm.html
bone, al s o can caus e s everal rheumat i c s yndromes .
1. Clinical syndromes o
o
a. Crystal deposition in osteoarthritis. Mi neral format i on i n cart i l age may be a res ul t of abnormal cart i l age met abol i s m i n more s evere forms of os t eoart hri t i s . Joi nt effus i ons i n t hes e pat i ent s cont ai n crys t al s of hydroxyapat i t e as oft en as t hey cont ai n CPPD crys t al s .
o
o
b. Calcific periarthritis. Hydroxyapat i t e depos i t i on i n burs ae and t endon s heat hs can caus e epi s odes of acut e i nfl ammat i on (i .e., peri art hri t i s and peri t endi ni t i s ), wi t h acut e at t acks of pai n, s wel l i ng, and eryt hema. Di s cret e cl umped depos i t s can be found radi ographi cal l y around t he s houl ders , great er t rochant ers , wri s t s , el bows , and di gi t s and i n ot her peri art i cul ar areas . Thes e depos i t s can be s hown t o di s i nt egrat e gradual l y on radi ographs t aken s everal weeks aft er t he acut e peri art hri t i s .
o
o
c. Destructive arthritis. Hydroxyapat i t e crys t al s can be as s oci at ed wi t h a chroni c des t ruct i ve art hropat hy, whi ch i s charact eri zed by eros i ve radi ographi c changes , l arge (us ual l y noni nfl ammat ory) effus i ons , prol i ferat i ve s ynovi t i s , s ynovi al mi neral depos i t i on, and peri art i cul ar l i gament ous i ns t abi l i t y. Thi s s yndrome occurs mos t oft en i n t he knee and s houl der (“ Milwaukee shoulder―) of el derl y pat i ent s . Synovi al fl ui d 3
anal ys i s s hows few W BCs (500–1000/mm ), wi t h
Pa g e 2 3 6 8
ABC Ambe r CHM Conve rte r Tria l ve rsion, http://w w w .proce sste x t.com/a bcchm.html
monocyt es predomi nat i ng. Hi gh concent rat i ons of neut ral prot eas es and col l agenas es s omet i mes are pres ent .
2. Diagnosis. Li ght mi cros copi c exami nat i on of s ynovi al fl ui d occas i onal l y reveal s browni s h gl obul es t hat are l arge cl umps of hydroxyapat i t e crys t al s . Is ol at ed crys t al s are t oo s mal l t o be s een wi t h ordi nary l i ght or pol ari zed l i ght mi cros copy. A cal ci um s t ai n, alizarin red S, can be us ed as a s creeni ng t es t for t he pres ence of hydroxyapat i t e or CPPD crys t al s i n effus i ons . As pi rat es from burs ae or t endon s heat hs may yi el d mi l ky or pas t y mat eri al t hat cont ai ns hi gh concent rat i ons of t hes e hydroxyapat i t e crys t al s .
3. Associated conditions. Di s orders of cal ci um and phos phorous met abol i s m s houl d be s ought i f mul t i pl e depos i t s are found.
4. T herapy. Mechani cal s pl i nt i ng and NSAIDs are us ed t o t reat acut e epi s odes of peri art hri t i s . NSAIDs oft en are us ed i n t he chroni c hydroxyapat i t e art hropat hy as wel l . Peri odi c as pi rat i on of t he l arge s ynovi al effus i ons t hat occur i n t he Mi l waukee s houl der may hel p pres erve l i gament ous i nt egri t y and remove des t ruct i ve enz ymes . Cort i cos t eroi d i nject i ons may hel p t reat acut el y s ympt omat i c joi nt s .
VI. Osteoarthritis A. Definition Os t eoart hri t i s i s t radi t i onal l y defi ned as a noni nfl ammat ory joi nt di s order charact eri zed by det eri orat i on of art i cul ar cart i l age and format i on of new bone at t he joi nt s urfaces and margi ns . Emergi ng
Pa g e 2 3 6 9
ABC Ambe r CHM Conve rte r Tria l ve rsion, http://w w w .proce sste x t.com/a bcchm.html
evi dence recogni zes t he i mport ance of bi omechani cal , bi ochemi cal , and cyt oki ne-medi at ed changes i n t he pat hogenes i s of os t eoart hri t i s .
B. Etiology Bot h s ys t emi c and l ocal fact ors cont ri but e t o t he devel opment of os t eoart hri t i s . Sys t emi c fact ors i ncl ude age, s ex, genet i cs , bone dens i t y, and nut ri t i onal el ement s , whi ch may make t he cart i l age more vul nerabl e t o i njury and l es s effi ci ent at repai r. Local bi omechani cal component s , i n combi nat i on wi t h s ys t emi c fact ors , have a s ynergi s t i c effect on t he proces s of cart i l age breakdown.
1. Age. The preval ence and i nci dence of os t eoart hri t i s i n al l joi nt s correl at es wi t h i ncreas i ng age. Agi ng i s as s oci at ed wi t h t he fol l owi ng changes : o
o
a. Decreas ed res pons i venes s of chondrocyt es t o growt h fact ors t hat s t i mul at e repai r
o
o
b. Increas ed l axi t y of l i gament s , whi ch makes t he joi nt s more uns t abl e and s us cept i bl e t o i njury P.474
o
o
c. Fai l ure of s hock abs orbers or prot ect ors of t he joi nt , whi ch i ncl udes decreas es i n mus cl e s t rengt h and neurol ogi c res pons es
o
o
d. Devel opment of a t hi nner ri m of noncal ci fi ed cart i l age, whi ch l eads t o an i ncreas ed s hear s t res s at t he bas al cart i l age l evel
Pa g e 2 3 7 0
ABC Ambe r CHM Conve rte r Tria l ve rsion, http://w w w .proce sste x t.com/a bcchm.html
2. Race. No cl ear raci al predi l ect i on has been document ed.
3. Sex. Gender appears t o be part i cul arl y i mport ant i n t he devel opment of erosive osteoarthritis of t he DIP and PIP joi nt s . Thi s vari ant i s 10 t i mes more common i n women t han men; i t i s aut os omal domi nant i n women and reces s i ve i n men.
4. Genetics. Res earchers have di s covered a defect i n t he gene codi ng for type II collagen synthesis, whi ch al l ows for earl y degenerat i on of t he t ype II col l agen. The pres ence of t hi s abnormal gene i s as s oci at ed wi t h a t ype of premat ure pol yart i cul ar os t eoart hri t i s and mi l d epi phys eal dys pl as i a s een i n s everal fami l i es .
5. Inflammation. Des pi t e t he noni nfl ammat ory nat ure of t he s ynovi al membrane and s ynovi al fl ui d i n os t eoart hri t i s , evi dence i ndi cat es t hat i nfl ammat ory medi at ors produced by s ynovi al t i s s ues and cart i l age pl ay a s i gni fi cant rol e. Cyt oki nes s uch as IL-1 as wel l as ot her i nfl ammat ory medi at ors s uch as ni t ri c oxi de and pros t agl andi ns are i mport ant i n t he aut odes t ruct i ve proces s es t hat promot e cart i l age degenerat i on. Thes e medi at ors , s eques t ered wi t hi n avas cul ar and aneural hyal i ne cart i l age, do not provoke t he cl as s i cal s i gns of i nfl ammat i on.
6. Obesity and abnormal stresses. An as s oci at i on exi s t s bet ween obes i t y and os t eoart hri t i s of t he knee, but i nt eres t i ngl y, t he s ame as s oci at i on does not exi s t
Pa g e 2 3 7 1
ABC Ambe r CHM Conve rte r Tria l ve rsion, http://w w w .proce sste x t.com/a bcchm.html
wi t h os t eoart hri t i s of t he hi p. Abnormal mechani cal l oadi ng cont ri but es t o di s eas e progres s i on by al t eri ng t he met abol i s m of os t eocyt es wi t hi n s ubchondral bone and chondrocyt es wi t hi n art i cul ar cart i l age. In addi t i on, abnormal bi omechani cal forces can i ncreas e t he expres s i on of mechanores pons i ve genes , wi t h t he res ul t ant rel eas e of prot eol yt i c enzymes , growt h fact ors , and medi at ors .
7. Neuropathy. Mus cl e t one around a joi nt modul at es t he forces of joi nt i mpact l oadi ng. If propri ocept i ve i nput t o t he joi nt i s i mpai red, abnormal mus cl e t one may res ul t i n os t eoart hri t i s by t rans ferri ng abnormal forces t o t he joi nt .
8. Deposition diseases (e.g., hemochromat os i s , ochronos i s , W i l s on's di s eas e, crys t al depos i t i on di s eas es ). Thes e condi t i ons , whi ch caus e depos i t i on of s ubs t ances i n t he cart i l age mat ri x, can res ul t i n di rect chondrocyt e i njury and premat ure os t eoart hri t i s .
C. Pathogenesis Os t eoart hri t i s i s a met abol i cal l y act i ve condi t i on, not a degenerat i ve one. Repai r and s ynt hes i s count eract t he des t ruct i ve proces s es unt i l t he repai r proces s es are overwhel med l at e i n t he di s eas e cours e.
1. Initial insult. Chondrocyt es i n hyal i ne cart i l age are res pons i bl e for mai nt ai ni ng t he ext racel l ul ar mat ri x by bal anci ng cat abol i c and anabol i c funct i ons . Synt hes i s of t he mat ri x i s s ynchroni zed wi t h t hei r degradat i on by prot eol yt i c enzymes . Two fact ors may cont ri but e t o t he i ni t i at i on of os t eoart hri t i s . o
Pa g e 2 3 7 2
ABC Ambe r CHM Conve rte r Tria l ve rsion, http://w w w .proce sste x t.com/a bcchm.html
o
a. A regi on of hi ghl y focal mechani cal s t res s
o
o
b. An i nt ri ns i c defect i n t he cart i l age mat ri x, as s i ngl e-bas e mut at i on i n t he t ype II procol l agen gene (COL2A1) renders col l agen fi bri l s prone t o fragment at i on and t hus earl y os t eoart hri t i s (s ee VI B 4)
2. Progression o
o
a. Early. Synt hes i s of prot eogl ycan and col l agen i s i ncreas ed, wi t h s ome degree of degradat i on.
o
o
b. Late. As os t eoart hri t i s devel ops furt her, prot eogl ycan s ynt hes i s decreas es , res ul t i ng i n a net l os s of mat ri x prot ei n. St ruct ural changes i n t he prot eogl ycan mol ecul es i ncl ude degradat i on of t he core prot ei n, reduct i on i n t he s i ze of t he prot eogl ycan aggregat es , and reduct i on i n t he hyal uroni c aci d cont ent . In addi t i on, s ynt hes i s of di fferent col l agen gene product s , whi ch al t er t he col l agen fi ber net work, charact eri zes t hi s l at e phas e.
3.
Factors involved o
o
a. Proteolytic enzymes. Chondrocyt es s ecret e degradat i ve enzymes (e.g., met al l oprot ei nas es ). Met al l oprot ei nas e act i vi t y i s cont rol l ed by i nhi bi t ors and act i vat ors . In os t eoart hri t i s , t he product i on of met al l oprot ei nas es exceeds t he
Pa g e 2 3 7 3
ABC Ambe r CHM Conve rte r Tria l ve rsion, http://w w w .proce sste x t.com/a bcchm.html
amount of s o-cal l ed t i s s ue i nhi bi t ors of met al l oprot ei nas e (TIMPs ). o
o
b. Cytokines and inflammatory mediators
(1) Catabolic: IL-1, IL-6, and IL-8; TNF; ni t ri c oxi de; and pros t agl andi n E 2 (PGE 2 ). IL-1 s t i mul at es t he product i on of prot eas es whi l e s uppres s i ng t he s ynt hes i s of prot eogl ycan and t ype II col l agen. In addi t i on, IL-1 i nduces chondrocyt e product i on of ot her det ri ment al cyt oki nes as wel l as ni t ri c oxi de and PGE 2 .
(2) Anabolic: growt h fact ors s uch as TGF-β and i ns ul i n-l i ke growt h fact or (IGF-1). TGF-β ant agoni zes t he act i on of IL-1, and IGF-1 i s t he major regul at or of prot eogl ycan s ynt hes i s .
D. Pathology
1. Cartilage changes o
o
a. Earl y i n os t eoart hri t i s , cart i l age changes i n color from bl ue t o yel l ow due t o l os s of prot eogl ycan.
o
o
b. Local i zed areas of softening are t he earl i es t pat hol ogi c changes i n os t eoart hri t i s .
o
o
c. Superfi ci al chipping and flaking of cart i l age
Pa g e 2 3 7 4
ABC Ambe r CHM Conve rte r Tria l ve rsion, http://w w w .proce sste x t.com/a bcchm.html
s i gni fi es more advanced di s eas e. o
o
d. Vert i cal fibrillations i n t he cart i l age i ndi cat e furt her progres s i on.
o
o
e. If t he heal i ng forces are overwhel med by des t ruct i ve forces , confl uent erosions event ual l y progres s t o ful l -t hi cknes s cart i l age l os s .
2. Bone changes o
o
a. New bone formation can occur under t he cart i l age (s een as eburnat i on on a radi ograph) or at t he joi nt margi n (s een as os t eophyt i c s purs on a radi ograph).
o
o
b. Subchondral cysts. Large ps eudocys t i c areas can form i n t he juxt a-art i cul ar bone due t o t rans mi s s i on of i ncreas ed mechani cal forces t o bone; pres umabl y, t hes e cys t s fai l t o heal becaus e of i mpai red perfus i on from s ubchondral mi crofract ures .
E. Classification (Table 10-10)
1. Primary osteoarthritis has no underl yi ng caus e for joi nt damage. It t ypi cal l y i nvol ves t he DIP and PIP joi nt s and t he fi rs t carpomet acarpal joi nt s . Knees , hi ps , and fi rs t MTP joi nt s oft en are i nvol ved as wel l as cervi cal and l umbar s pi ne facet joi nt s . The joi nt i nvol vement may be generalized or i t may occur i n an
Pa g e 2 3 7 5
ABC Ambe r CHM Conve rte r Tria l ve rsion, http://w w w .proce sste x t.com/a bcchm.html
i s ol at ed, sporadic fas hi on. Erosive os t eoart hri t i s i s a uni que s ubs et t hat occurs predomi nant l y i n mi ddl e-aged women and has aut os omal domi nant , s ex-i nfl uenced charact eri s t i cs . Epi s odi c eryt hema, s wel l i ng, and t endernes s occur i n i nvol ved joi nt s , es peci al l y t he DIP and PIP joi nt s of t he hands . Charact eri s t i c radi ographi c abnormal i t i es i ncl ude bone eros i ons and ankyl os i s , whi ch are unus ual i n t ypi cal os t eoart hri t i s . Diffuse idiopathic skeletal hyperostosis i s noni nfl ammat ory axi al and peri pheral ent hes i s hyperos t os i s mani fes t ed by radi ographi c “whi s keri ng― at t endon or l i gament i ns ert i on and fl owi ng os t eophyt es adjacent t o vert ebral di s ks .
2. Secondary osteoarthritis exhi bi t s an underl yi ng caus e for degenerat i ve joi nt di s eas e and may i nvol ve joi nt s not t ypi cal l y affect ed by pri mary os t eoart hri t i s (e.g., el bow, wri s t ). A pri mary met abol i c, i nfl ammat ory, or mechani cal proces s l eads s econdari l y t o os t eoart hri t i s (s ee Tabl e 10-10). P.475
TABLE 10-10 Classification of Osteoarthritis
Pa g e 2 3 7 6
ABC Ambe r CHM Conve rte r Tria l ve rsion, http://w w w .proce sste x t.com/a bcchm.html
Pri ma ry ost eo art hri tis (m ulti ple sit es) He ber de n's no des Ge ner al i z ed os t eo art hri t is †œE ros i ve †• os t
Pa g e 2 3 7 7
ABC Ambe r CHM Conve rte r Tria l ve rsion, http://w w w .proce sste x t.com/a bcchm.html
eo art hri t is Di f fus e i di op at h ic s ke l et al hyp ero sto sis Pri ma ry ost eo art hri tis (lo cal ) Cer vi c al s pi ne Hi p
Pa g e 2 3 7 8
ABC Ambe r CHM Conve rte r Tria l ve rsion, http://w w w .proce sste x t.com/a bcchm.html
Fi r st car po me t ac arp al joi nt Di s t al i nt erp hal an ge al joi nt s Lu mb ar s pi ne Kn ee Fi r st me t at ars op hal an
Pa g e 2 3 7 9
ABC Ambe r CHM Conve rte r Tria l ve rsion, http://w w w .proce sste x t.com/a bcchm.html
ge al joi nt Pro xi m al i nt erp hal an ge al joi nt s Se co nd ary ost eo art hri tis Co ng eni t al (e. g., hi p dys pl a sia )
Pa g e 2 3 8 0
ABC Ambe r CHM Conve rte r Tria l ve rsion, http://w w w .proce sste x t.com/a bcchm.html
De pos itio n di s eas e Oc hro nos is W il s on 's di s eas e He mo chr om at o sis Go ut Cal ci u m pyr op
Pa g e 2 3 8 1
ABC Ambe r CHM Conve rte r Tria l ve rsion, http://w w w .proce sste x t.com/a bcchm.html
hos ph at e de pos itio n di s eas e Ne uro pat hi c joi nt (e. g., di a bet es me llit us , s yp hi l i s) En doc ri n e/ me t ab ol i c (e.
Pa g e 2 3 8 2
ABC Ambe r CHM Conve rte r Tria l ve rsion, http://w w w .proce sste x t.com/a bcchm.html
g., acr om eg al y ) Os t eo nec ros is (es pec i al l y hi p s, kne es ) Inf ect i on (t u ber cul os i s) Infl am ma tio n (rh eu ma
Pa g e 2 3 8 3
ABC Ambe r CHM Conve rte r Tria l ve rsion, http://w w w .proce sste x t.com/a bcchm.html
t oi d art hri t is)
F. Clinical features
1. Symptoms vary wi t h t he joi nt i nvol ved and t he s everi t y of t he di s eas e. o
o
a. Pain. Mos t pat i ent s experi ence t he gradual ons et of a deep, achi ng pai n, whi ch wors ens wi t h act i vi t y and i s rel i eved by res t . W i t h more s evere di s eas e, pai n can occur even at res t and i nt erfere wi t h s l eep.
o
o
b. Morning stiffness i s bri ef (<30 mi nut es ) i n cont ras t t o t hat occurri ng i n i nfl ammat ory rheumat i c condi t i ons , whi ch general l y l as t s much l onger.
o
o
c. Gelling phenomenon refers t o t he s ens at i on of renewed s t i ffnes s i n os t eoart hri t i c joi nt s aft er prol onged i nact i vi t y.
Pa g e 2 3 8 4
ABC Ambe r CHM Conve rte r Tria l ve rsion, http://w w w .proce sste x t.com/a bcchm.html
2. Signs o
o
a. T enderness. Mi l d or moderat e t endernes s can be pres ent i n i nvol ved joi nt s .
o
o
b. Painful range of motion i n l arge joi nt s (e.g., knees , hi ps ) i s t he equi val ent of t endernes s i n s mal l joi nt s .
o
o
c. Crepitus (i .e., a gri ndi ng s ound or s ens at i on) can be fel t and s omet i mes heard when a joi nt i s put t hrough a ful l range of mot i on. Crepi t us i s caus ed by s urface i ncongrui t i es i n t he joi nt .
o
o
d. Warmth. Invol ved joi nt s us ual l y are cool but can feel warm wi t h fl are-ups of di s eas e act i vi t y.
o
o
e. Joint enlargement. Soft t i s s ue s wel l i ng may occur i f an effus i on i s pres ent . More commonl y, bone enl argement occurs i n t he form of os t eophyt es .
o
o
f. Deformity. Varus (medi al ) or valgus (l at eral ) angulation of joi nt s can occur l at e i n t he di s eas e. Gros s bone enl argement and joi nt s ubl uxat i on al s o can occur i n s evere di s eas e.
(1) Heberden' s nodes s peci fi cal l y refer t o enl argement of t he DIP joi nt s of t he hand.
Pa g e 2 3 8 5
ABC Ambe r CHM Conve rte r Tria l ve rsion, http://w w w .proce sste x t.com/a bcchm.html
(2) Bo uc ha rd' s no de s s pe ci fi cal l y ref er to enl arg em ent of t he PIP joi nt s of t he ha nd (O nl i ne Fi g ure 10-
Pa g e 2 3 8 6
ABC Ambe r CHM Conve rte r Tria l ve rsion, http://w w w .proce sste x t.com/a bcchm.html
9).
ONLINE FIGURE 10-9 Os t eoart hri t i s . Heberden's nodes are s een on t he di s t al i nt erphal angeal joi nt s and Bouchard's nodes are s een on t he proxi mal i nt erphal angeal joi nt s . (ACR Sl i de Col l ect i on of t he Rheumat i c Di s eas es , 3rd ed., 2004:9104030.)
G. Diagnosis Pat i ent hi s t ory combi ned wi t h phys i cal , l aborat ory, and radi ographi c fi ndi ngs form t he bas i s for di agnos i s .
1. Joint involvement o
o
a. The distribution of the involved joints s houl d s ugges t whet her t he os t eoart hri t i s i s pri mary or rel at ed t o an underl yi ng di s order (s ee VI E 1–2).
o
o
b. Joint swelling i s t ypi cal l y bony, s omet i mes wi t h s uperi mpos ed fluid.
2. Laboratory findings
Pa g e 2 3 8 7
ABC Ambe r CHM Conve rte r Tria l ve rsion, http://w w w .proce sste x t.com/a bcchm.html o
o
a. Hematologic findings. Res ul t s , i ncl udi ng t he ESR, are general l y normal .
o
o
b. Synovial fluid findings. Typi cal os t eoart hri t i c s ynovi al fl ui d i s s l i ght l y t urbi d, cont ai ns no crys t al s , and has a W BC count t hat i s onl y mi l dl y 3
i nfl ammat ory (i .e., <2000 cel l s /mm and <25% neut rophi l s ). P.476
3. Radiographic findings. Radi ographi c evi dence i s common aft er age 40 i n joi nt s t ypi cal l y affect ed by os t eoart hri t i s , and i t oft en i s as ympt omat i c. o
o
a. Findings typically present
(1) Joi nt s pace narrowi ng (due t o l os s of cart i l age)
(2) Subchondral s cl eros i s (i ncreas ed s ubchondral bone dens i t y)
(3) Margi nal os t eophyt es
(4) Subchondral cys t s
o
Pa g e 2 3 8 8
ABC Ambe r CHM Conve rte r Tria l ve rsion, http://w w w .proce sste x t.com/a bcchm.html
o
b. Findings typically absent
(1) Peri art i cul ar os t eopeni a
(2) Margi nal eros i ons (except i n t he di s t al DIP and PIP joi nt s i n t he eros i ve os t eoart hri t i s vari ant )
4. Differential diagnosis o
o
a. Monoarticular problems
(1) Periarticular abnormality. Pat i ent s may compl ai n of pai n i n a joi nt yet have i nvol vement of a peri art i cul ar s t ruct ure s uch as a t endon, l i gament , or burs a as t he real caus e of s ympt oms .
(2) Other causes. Bact eri al i nfect i ons and crys t al -medi at ed probl ems mus t al ways be cons i dered i f onl y one joi nt i s i nvol ved. Trauma, hemorrhage, and monoart i cul ar pres ent at i ons of i nfl ammat ory di s eas es al s o can be confus ed wi t h os t eoart hri t i s .
o
o
b. Polyarticular problems
(1) Inflammatory rheumatic disease. Sys t emi c compl ai nt s (e.g., anorexi a, wei ght
Pa g e 2 3 8 9
ABC Ambe r CHM Conve rte r Tria l ve rsion, http://w w w .proce sste x t.com/a bcchm.html
l os s , fat i gue, and fever), promi nent morni ng s t i ffnes s , and fi ndi ngs of i nfl ammat ory rheumat i c di s eas es s houl d be s ought .
(2) Soft tissue syndromes. Di s orders as s oci at ed wi t h regi onal achi ng (regi onal myofas ci al pai n) or general i zed achi ng (e.g., fi bromyal gi a, PMR) al s o s houl d be cons i dered i n pol yart i cul ar pres ent at i ons .
H. Therapy
1. Nonpharmacologic therapy o
o
a. General advice to patients. Joi nt overus e or repet i t i ve t rauma mus t be avoi ded. Weight loss may be benefi ci al i n art hri t i s of wei ght -beari ng joi nt s s uch as t he knees . Os t eoart hri t i c pai n i mproves wi t h res t , s o joint rest i s part i cul arl y i mport ant when pai n i s promi nent .
o
o
b. Supports
(1) A knee cage or brace s omet i mes i s us ed when knee l i gament ous i ns t abi l i t y coexi s t s wi t h os t eoart hri t i s .
(2) A soft cervical collar may be us ed for s ympt omat i c fl are-ups of cervi cal s pi ne os t eoart hri t i s .
Pa g e 2 3 9 0
ABC Ambe r CHM Conve rte r Tria l ve rsion, http://w w w .proce sste x t.com/a bcchm.html
(3) A lumbar corset (back brace) s omet i mes i s us ed t o but t res s s aggi ng abdomi nal or back mus cl es i n pat i ent s wi t h l ow back pai n.
(4) A cane may be hel pful i n s upport i ng a pat i ent wi t h uni l at eral hi p or knee os t eoart hri t i s .
(5) Arch supports (orthotics) or cus hi oned s hoes may decreas e t he t rans mi s s i on of wei ght -beari ng forces t o t he hi ps and knees .
o
o
c. Exercise. Isometric strengthening of s upport i ng mus cl es around joi nt s may be hel pful (e.g., quadri ceps -s et t i ng exerci s es i n knee art hri t i s ). Swi mmi ng and wat er aerobi cs are t he bes t form of aerobic exercise for a pat i ent wi t h os t eoart hri t i s of t he hi ps or knees ; runni ng s houl d be avoi ded by t hes e pat i ent s .
o
o
d. Heat/cold modalities. Appl i cat i on of moi s t heat or heat i ng pads , or even i ce, oft en can t emporari l y l es s en t he pai n of os t eoart hri t i s .
2. Pharmacologic therapy o
o
a. Analgesics
(1) T opical. Di rect appl i cat i on of capsaicin
Pa g e 2 3 9 1
ABC Ambe r CHM Conve rte r Tria l ve rsion, http://w w w .proce sste x t.com/a bcchm.html
(s ubs t ance P i nhi bi t or) t o t he s ki n overl yi ng a pai nful joi nt can rel i eve pai n.
(2) Systemic. Pai n-rel i evi ng medi cat i ons s uch as acetaminophen oft en are effect i ve i n moderat e-t o-hi gh dos es for mi l d-t o-moderat e os t eoart hri t i s . Narcot i cs s houl d be us ed onl y under ext enuat i ng ci rcums t ances .
P.477
o
o
b. NSAIDs. Pai n rel i ef wi t h l ow-t o-moderat e dos es of as pi ri n or ot her NSAIDs may be us eful . El derl y pat i ent s have more gas t roi nt es t i nal and renal s i de effect s from t hes e drugs and, t hus , s houl d be careful l y moni t ored when recei vi ng s uch t reat ment . The s el ect i ve COX-2 bl ocki ng agent s have been s hown t o have s i mi l ar effi cacy, but wi t h l es s gas t roi nt es t i nal t oxi ci t y (s ee II G 2).
o
o
c. Corticosteroids. Oral s t eroi ds have no pl ace i n t he management of os t eoart hri t i s . Occas i onal intra-articular i nject i ons of cort i cos t eroi ds may be of t emporary benefi t i n fl are-ups , but repeat ed us e of s t eroi ds carri es t he ri s k of pos s i bl e accel erat i on of t he di s eas e proces s .
o
o
d. Viscosupplementation therapy with intra-articular hyaluronic acid. Two formul at i ons of i nt ra-art i cul ar hyal uroni c aci d are avai l abl e for us e i n t he t reat ment of earl y (mi l d) os t eoart hri t i s
Pa g e 2 3 9 2
ABC Ambe r CHM Conve rte r Tria l ve rsion, http://w w w .proce sste x t.com/a bcchm.html
of t he knee. St udi es have s hown t hes e agent s t o be effect i ve i n modul at i ng pai n ori gi nat i ng from t he os t eoart hri t i c knee joi nt and have demons t rat ed t hat t hes e agent s may have a pos i t i ve effect on art i cul ar cart i l age bi ol ogy. o
o
e. Nutritional supplements. Gl ucos ami ne s ul fat e and chondroi t i n s ul fat e have been s hown i n s everal s t udi es t o decreas e s ympt oms of pai n and s t i ffnes s i n os t eoart hri t i s . A recent Nat i onal Ins t i t ut es of Heal t h (NIH)-s upport ed mul t i cent er t ri al of gl ucos ami ne hydrochl ori de and chondroi t i n s ul fat e fai l ed t o s how s ympt om benefi t . The rol e of t hes e agent s i n s l owi ng di s eas e progres s i on has not been es t abl i s hed.
o
o
f. Ongoing research
(1) Metalloproteinase inhibitors (s ee onl i ne VI C 3)
(2) Biological agents. Agent s s uch as proi nfl ammat ory cyt oki ne i nhi bi t ors , cyt oki ne s ol ubl e recept ors , and ant i -i nfl ammat ory cyt oki nes . Thes e agent s have pot ent i al benefi ci al t herapeut i c propert i es .
(3) Cartilage transplantation. Once perfect ed, t hi s t echni que l i kel y wi l l be res t ri ct ed t o younger pat i ent s wi t h cart i l age l es i ons i n t he abs ence of bony changes .
Pa g e 2 3 9 3
ABC Ambe r CHM Conve rte r Tria l ve rsion, http://w w w .proce sste x t.com/a bcchm.html
3. Surgery. In advanced di s eas e of t he knee or hi p, total joint replacement can be dramat i cal l y effect i ve i n al l evi at i ng pai n and res t ori ng funct i on. Angulation osteotomy i s s t i l l performed i n os t eoart hri t i s of t he knee t o t reat uni compart ment al di s eas e. The devel opment of new bi omat eri al s and expandi ng knowl edge on t he bi ol ogy of pros t hes i s l oos eni ng s houl d hel p l ower t he fai l ure rat e.
VII. Bacterial (Septic) Arthritis Bact eri al art hri t i s i s a s eri ous i nfl ammat ory art hri t i s of one or more joi nt s t hat can l ead t o rapi d joi nt des t ruct i on i f unt reat ed. Numerous organi s ms can caus e a s ept i c art hri t i s s yndrome.
A. Epidemiology (Table 10-11) Ot herwi s e heal t hy i ndi vi dual s can devel op bact eri al art hri t i s by di rect i nocul at i on or by bl ood-borne i nvas i on of a joi nt , but cert ai n peopl e are at hi gher ri s k.
1. Joint damage. Pat i ent s wi t h joi nt s damaged by os t eoart hri t i s , rheumat oi d art hri t i s , or ot her des t ruct i ve joi nt proces s es are more at ri s k for i nfect i on i n t he al ready damaged joi nt s . P.478
2. Immunosuppression. Pat i ent s wi t h i mmune defi ci enci es are at great er ri s k for bact eri al art hri t i s , part i cul arl y i f t hos e defect s i nvol ve neut ropeni a or i mpai red phagocyt os i s .
3. Inoculation. Repeat ed s ept i cemi a (e.g., i n t he s et t i ng of i nt ravenous drug abus e) pres ent s t he great es t
Pa g e 2 3 9 4
ABC Ambe r CHM Conve rte r Tria l ve rsion, http://w w w .proce sste x t.com/a bcchm.html
ri s k. Mos t medi cal procedures i nvol vi ng pot ent i al di rect i nocul at i on (e.g., joi nt as pi rat i on and i nject i on) are performed as ept i cal l y and onl y mi ni mal l y i ncreas e t he ri s k of organi s m i nt roduct i on i nt o t he joi nt .
4. Prosthetic joint. Repl acement of a joi nt wi t h a pros t hes i s removes t he normal defens es agai ns t joi nt i nfect i on, and pros t het i c mat eri al s are di ffi cul t t o s t eri l i ze once i nfect ed.
B. Etiology Infect i ous art hri t i s i s bes t cl as s i fi ed as gonococcal or nongonococcal. Gonococcal i nfect i ons account for at l eas t hal f of cas es of bact eri al art hri t i s i n adul t s , are much l es s des t ruct i ve, and are very res pons i ve t o t reat ment .
1. Neisseria gonorrhoeae. Thi s gram-negat i ve i nt racel l ul ar di pl ococcus caus es disseminated gonococcal infection (DGI). Typi cal l y s exual l y t rans mi t t ed, N. gonorrhoeae can caus e septic arthritis or a periarthritis–dermatitis syndrome.
2. Staphylococcus species. Cl us t ers of gram-pos i t i ve cocci on Gram s t ai n s ugges t a s t aphyl ococcal i nfect i on. o
o
a. Staphylococcus aureus i s t he mos t common nongonococcal caus e of s ept i c art hri t i s , whi ch t ypi cal l y ari s es from a s ki n s ource and can be es peci al l y rapi d i n caus i ng joi nt des t ruct i on.
o
o
b. Staphylococcus epidermidis al s o t ypi cal l y ari s es from a s ki n s ource. Al t hough us ual l y l es s
Pa g e 2 3 9 5
ABC Ambe r CHM Conve rte r Tria l ve rsion, http://w w w .proce sste x t.com/a bcchm.html
vi rul ent t han S. aureus , S. epi dermi di s i s becomi ng a more common caus e of joi nt i nfect i on i n pros t het i c joi nt s and i n t he s et t i ng of i nt ravenous drug abus e.
3. Streptococcus species o
o
a. Group A β-hemolytic streptococci remai n t he mos t common caus e of i nfect i on as s oci at ed wi t h gram-pos i t i ve chai ns on Gram s t ai n and ari s e from s ki n or res pi rat ory t ract s ources .
o
o
b. Non-group A streptococci ari s i ng from s ki n or uri nary t ract s ources are bei ng s een more oft en i n i mmunocompromi s ed pat i ent s , t hos e wi t h pros t het i c joi nt s , and t hos e who us e i nt ravenous drugs .
4. Nongonococcal gram-negative organisms o
o
a. Gram-negative enteric pathogens. Ski n col oni zat i on wi t h gram-negat i ve organi s ms i s a pot ent i al s ource of joi nt i nfect i on i n el derl y or i mmunocompromi s ed i ndi vi dual s , part i cul arl y t hos e i n hos pi t al s or chroni c care i ns t i t ut i ons . Uri nary t ract s ources al s o are common i n t hes e pat i ent s .
o
o
b. Haemophilus influenzae. Thi s gram-negat i ve coccobaci l l us remai ns a major res pi rat ory pat hogen i n i nfant s . The pot ent i al for di s s emi nat i on t o joi nt s i s great es t bet ween 6 mont hs and 2 years of age,
Pa g e 2 3 9 6
ABC Ambe r CHM Conve rte r Tria l ve rsion, http://w w w .proce sste x t.com/a bcchm.html
when i nfant s are wi t hout prot ect i ve mat ernal ant i bodi es and have not devel oped t hei r own. o
o
c. Neisseria meningitidis. Thi s gram-negat i ve i nt racel l ul ar di pl ococcus , s i mi l ar t o N. gonorrhoeae, ari s es from an upper res pi rat ory t ract s ource. It i s a much l es s common caus e of a di s s emi nat ed peri art hri t i s –dermat i t i s i nfect i on, whi ch i s t ypi cal l y charact eri zed by more s ki n l es i ons t han are s een i n DGI.
5. Anaerobic and polymicrobial infections are uncommon; t hey t ypi cal l y ari s e i n t he s et t i ng of pros t het i c joi nt i nfect i on or i n s everel y i mmunocompromi s ed pat i ent s .
C. Pathophysiology
1. Entry. Bact eri a ent er t he joi nt vi a t he bl oods t ream (s ept i cemi a) or di rect i nocul at i on (s ynovi al as pi rat i on, t rauma, or s urgery).
2. Early events. Once i ns i de t he joi nt , t he pres ence of bact eri a el i ci t s s ynovi al l i ni ng cel l hyperpl as i a and neut rophi l chemot axi s .
3. Acute destructive process. Synovi al cel l s and neut rophi l s rel eas e proteolytic enzymes, whi ch damage cart i l age and s ubchondral bone as wel l as ki l l t he bact eri a. Dyi ng bact eri a rel eas e lipopolysaccharides t hat can ei t her di rect l y degrade cart i l age or i ndi rect l y degrade i t by s t i mul at i ng t he rel eas e of IL-1 or
Pa g e 2 3 9 7
ABC Ambe r CHM Conve rte r Tria l ve rsion, http://w w w .proce sste x t.com/a bcchm.html
IL-1–like factors t hat caus e chondrocyt es t o rel eas e collagenases and prostaglandins.
4. Chronic destruction. Granul at i on t i s s ue act s s i mi l arl y t o rheumatoid pannus i n degradi ng cart i l age and bone.
D. Clinical features
1. Gonococcal arthritis. Pat i ent s wi t h DGI t ypi cal l y have geni t al i nfect i on wi t hout s ympt oms of art hri t i s or pel vi c i nfl ammat ory di s eas e; changes i n t he endomet ri um or cervi cal mucus may al l ow caus at i ve s t rai ns t o di s s emi nat e duri ng mens es . The l ack of geni t ouri nary compl ai nt s del ays t reat ment before di s s emi nat i on. o
o
a. Periarthritis–dermatitis syndrome. Mos t pat i ent s exhi bi t migratory or additive polyarthralgias for s everal days , fol l owed by fever, tenosynovitis, and oft en dermatitis. The s ki n l es i ons can be maculopapular or vesicular, wi t h t he ves i cl es becomi ng vesiculopustular over t i me. P.479
o
o
b. Monoarthritis. Twent y-fi ve percent t o 50% of pat i ent s wi t h DGI devel op an i nfect i ous mono- or ol i goart hri t i s wi t h purul ent joi nt effus i ons . Thes e “s ept i c joi nt s ― can occur wi t h or wi t hout precedi ng periarthritis–dermatitis syndrome.
Pa g e 2 3 9 8
ABC Ambe r CHM Conve rte r Tria l ve rsion, http://w w w .proce sste x t.com/a bcchm.html
2. Nongonococcal arthritis. Pat i ent s t ypi cal l y pres ent wi t h acut e pai n, t endernes s , s wel l i ng, and s evere l i mi t at i on of mot i on i n t he affect ed joi nt (s ). W armt h and eryt hema are s omet i mes pres ent , and s ome pat i ent s have evi dence of a s ource (e.g., s ki n, uri nary t ract , pharynx, l ung, cardi ac val ve) res pons i bl e for s ept i cemi c joi nt i nocul at i on.
E. Diagnosis
1. Making a presumptive diagnosis o
o
a. Gonococcal arthritis. Hi s t ory of recent s exual cont act s houl d be s ought and cul t ures from appropri at e s i t es (pharynx, s ki n l es i ons , joi nt fl ui d, bl ood, rect um, uret hra, and cervi x) t aken i n pat i ent s who have s us pi ci ous s ki n l es i ons or t enos ynovi t i s . A s exual l y act i ve adul t who has an acut e monoart hri t i s wi t hout crys t al s or ot her known caus e s houl d be cul t ured and t reat ed for DGI, unt i l ot her i nformat i on s ugges t s a more appropri at e cours e.
o
o
b. Nongonococcal septic arthritis. Synovi al fl ui d s houl d be as pi rat ed and s ent for Gram s t ai n, gl ucos e, W BC count and di fferent i al , and cul t ure and s ens i t i vi t y. A s ynovi al fl ui d W BC count great er 3
t han 50,000/mm wi t h more t han 90% neut rophi l s i s hi ghl y s ugges t i ve of bact eri al i nfect i on. Infect ed fl ui ds can s omet i mes have l es s s t ri ki ng W BC 3
el evat i ons (10,000–20,000/mm ).
Pa g e 2 3 9 9
ABC Ambe r CHM Conve rte r Tria l ve rsion, http://w w w .proce sste x t.com/a bcchm.html
2. Differential diagnosis. Ot her caus es of i nfect i ous and noni nfect i ous art hri t i s s houl d be cons i dered when maki ng a pres umpt i ve di agnos i s of bact eri al art hri t i s . Periarticular bone and soft tissue infections s houl d al s o be s ought . o
o
a. Other infections. Pat i ent s wi t h monoart i cul ar art hri t i s may have anot her i nfect i on.
(1) Lyme disease. Thi s condi t i on s houl d al ways be cons i dered i n pat i ent s l i vi ng i n endemi c areas who have a mono- or ol i goart i cul ar i nfl ammat ory art hri t i s , part i cul arl y t he knees . The art hri t i s may be accompani ed by one or more of t he dermat ol ogi c, cardi ac, or neurol ogi c feat ures of t he di s order.
(2) T uberculous or fungal arthritis. Thes e i nfect i ons general l y are more i ndol ent ; t hey t ypi cal l y are as s oci at ed wi t h l ower s ynovi al fl ui d W BC count s and l ower percent ages of neut rophi l s t han are s een i n bact eri al i nfect i ons . Synovi al bi ops y may be requi red for di agnos i s becaus e joi nt fl ui d cul t ures are frequent l y negat i ve.
o
o
b. Infection outside joints
(1) Osteomyelitis near a joi nt can caus e
Pa g e 2 4 0 0
ABC Ambe r CHM Conve rte r Tria l ve rsion, http://w w w .proce sste x t.com/a bcchm.html
fever and a s t eri l e, i nfl ammat ory effus i on mi mi cki ng s ept i c art hri t i s .
(2) Subcutaneous bursae next t o joi nt s (e.g., prepat el l ar, ol ecranon) can be i nfl amed or i nfect ed and mi mi c an i nfect i on of t he adjacent joi nt . The phys i cal appearance and l ocat i on of t he s wol l en burs a s houl d be di s t i nct i ve.
o
o
c. Inflammatory arthritis. Di s orders s uch as rheumat oi d art hri t i s , acut e rheumat i c fever (ARF), react i ve art hri t i s , and ps ori at i c art hri t i s can mani fes t as s evere monoart i cul ar i nvol vement l ooki ng l i ke an i nfect i on, s omet i mes wi t h W BC 3
count s exceedi ng 50,000/mm and more t han 90% neut rophi l s . In s ome i ns t ances , pat i ent s wi t h t hes e di s eas es mus t be t reat ed wi t h ant i bi ot i cs for 48 hours unt i l joi nt fl ui d cul t ures are known t o be negat i ve. Gout and ps eudogout can caus e acut e monoart hri t i s , s o s ynovi al fl ui d mus t be exami ned under pol ari zed l i ght mi cros copy for t he caus at i ve crys t al s .
F. Therapy
1. Antibiotics o
o
a. Choice of therapy
(1) Empiric treatment. Becaus e bact eri al art hri t i s i s s o rapi dl y des t ruct i ve, pres umpt i ve
Pa g e 2 4 0 1
ABC Ambe r CHM Conve rte r Tria l ve rsion, http://w w w .proce sste x t.com/a bcchm.html
ant i bi ot i c t herapy mus t be s t art ed before res ul t s of defi ni t i ve cul t ures are known. The part i cul ar ant i bi ot i c regi men i s t ai l ored t o t he organi s ms mos t l i kel y i n a part i cul ar i ndi vi dual (s ee Tabl e 10-11) and part i cul ar age group (Tabl e 10-12). In adul t s , gonococcal coverage i s appropri at e unl es s s exual cont act can be excl uded. Ot herwi s e, coverage for S. aureus and s t rept ococci t ypi cal l y i s i ncl uded. Gram-negat i ve coverage i s added i n pat i ent s who are i mmunocompromi s ed or who are i nt ravenous drug abus ers . P.480
TABLE 10-11 Epidemiology of Bacterial Arthritis Cl in ic Ri al O sk S rg F et a a ti ni ct n s or g m Pr R St io h a r e p jo u hy in m lo
Pa g e 2 4 0 2
ABC Ambe r CHM Conve rte r Tria l ve rsion, http://w w w .proce sste x t.com/a bcchm.html
t at c o d oi c c a d us m ar a a t h ur g ri t e e i s us , os te o ar th ri t is I Di S. m a a m b ur u et e n es us os m , u el gr p lit a pr us m es ,
-n
s i al e o co g n h at ol i v is e m ro ,
ds
ch ro ni
Pa g e 2 4 0 3
ABC Ambe r CHM Conve rte r Tria l ve rsion, http://w w w .proce sste x t.com/a bcchm.html
c re n al fa il ur e, cy to to xi c dr u gs , SL E, ca nc er , AI D S Jo D St in a a t m p as a hy pi g l o ra e c o ti d cc o or us n in e
Pa g e 2 4 0 4
ABC Ambe r CHM Conve rte r Tria l ve rsion, http://w w w .proce sste x t.com/a bcchm.html
or fl pi in a d je m er ct e m io d id n jo i s in , t S. (r a ar ur e) e us IV A S. dr xi a u al ur g jo e a i n us b t , us (a gr e cr a o m m -n io e cl g av at ic iv ul e ar ro ,
ds
st , er Ps n e oc u la d vi o
Pa g e 2 4 0 5
ABC Ambe r CHM Conve rte r Tria l ve rsion, http://w w w .proce sste x t.com/a bcchm.html
cu m la o r, n s a as cr a oi er li u ac gi ) n i n os vo a lv e m e nt Si Jo S ck i n al le t m ce i n o l l fe n ar ct el th io la ro n a p or n at os d hy t e St o a m p ye hy lit lo is co cc us sp
Pa g e 2 4 0 6
ABC Ambe r CHM Conve rte r Tria l ve rsion, http://w w w .proce sste x t.com/a bcchm.html
ec ie s, St re pt oc oc cu s p n e u m o ni a e, gr a m -n e g at iv e ro ds Pr P S. os er e t h i o pi et p d i c er er
Pa g e 2 4 0 7
ABC Ambe r CHM Conve rte r Tria l ve rsion, http://w w w .proce sste x t.com/a bcchm.html
jo at m in iv id t e is p S. er a i o ur d e P us os , t o gr p a er m at -n iv e e g p at er i v io e d ro ds or a n a er o b es AIDS, acqui r ed i mmun odefi ci ency s yndro
Pa g e 2 4 0 8
ABC Ambe r CHM Conve rte r Tria l ve rsion, http://w w w .proce sste x t.com/a bcchm.html
me; IV, i nt rave nous ; SLE, s ys t em ic l upus eryt he mat os us .
TABLE 10-12 Bacterial Arthritis: Age Groups and Common Organisms A g O e rg G a ro ni u s p m < H 2 a ye e ar m s o p hi lu s in fl u
Pa g e 2 4 0 9
ABC Ambe r CHM Conve rte r Tria l ve rsion, http://w w w .proce sste x t.com/a bcchm.html
e nz a e* , St a p hy lo co cc us a ur e us , gr o u p A st re pt oc oc ci , E nt er o b
Pa g e 2 4 1 0
ABC Ambe r CHM Conve rte r Tria l ve rsion, http://w w w .proce sste x t.com/a bcchm.html
ac te ri ac e a e 2 S. â a €“ ur 6 e ye us ar , s gr o u p A st re pt oc oc ci , H. in fl u e nz a e 6 S ye ex
Pa g e 2 4 1 1
ABC Ambe r CHM Conve rte r Tria l ve rsion, http://w w w .proce sste x t.com/a bcchm.html
ar u s al to ly a tr d a ul ns t m it te d: N ei ss er ia g o n or rh o e a e â€
N o n â €“ se xu al ly tr
Pa g e 2 4 1 2
ABC Ambe r CHM Conve rte r Tria l ve rsion, http://w w w .proce sste x t.com/a bcchm.html
a ns m it te d: S. a ur e us , gr o u p A st re pt oc oc ci , E nt er o b ac te ri ac e a
Pa g e 2 4 1 3
ABC Ambe r CHM Conve rte r Tria l ve rsion, http://w w w .proce sste x t.com/a bcchm.html
e *Y oun g chi l dre n at ri s k aft er l os s of mat ern al ant i bo dy. †In adults , N. gonorr hoeae is i mpl i ca t ed t wi ce as oft en as al l ot her agent s combi n ed.
(2) Directed therapy. Pos i t i ve res ul t s on s ynovi al fl ui d Gram s t ai n or pos i t i ve cul t ure res ul t s from bl ood, s ynovi al fl ui d, or ot her s ources al l ow more s peci fi c t reat ment of t he caus at i ve organi s m.
o
Pa g e 2 4 1 4
ABC Ambe r CHM Conve rte r Tria l ve rsion, http://w w w .proce sste x t.com/a bcchm.html
o
b. Route and duration of therapy
(1) Gonococcal arthritis. The gonococcus i s s o s ens i t i ve t hat 3 days of i nt ravenous ant i bi ot i c fol l owed by 7 more days of oral ant i bi ot i c us ual l y i s s uffi ci ent t reat ment .
(2) Nongonococcal arthritis. Ant i bi ot i cs t ypi cal l y are gi ven i nt ravenous l y for at l eas t 2 weeks i n nongonococcal bact eri al art hri t i s , and l onger i f cl i ni cal i mprovement i s s l ow. Oral medi cat i on i s gi ven for 2 addi t i onal weeks .
2. Drainage o
o
a. Closed-needle aspiration. Dai l y drai nage of any effus i on i s i mperat i ve t o remove organi s ms and i nfl ammat ory debri s , whi ch can des t roy cart i l age and s ubchondral bone. Cl os ed-needl e as pi rat i on us ual l y i s appropri at e, becaus e open s urgi cal drai nage prol ongs i mmobi l i zat i on and del ays ret urn of effect i ve funct i on.
o
o
b. Open surgical drainage. Hi p i nfect i ons are bes t handl ed by i mmedi at e open drai nage, es peci al l y i n chi l dren. Joi nt s t hat do not res pond t o needl e drai nage are t reat ed ei t her by open drai nage or by art hros copy.
o
Pa g e 2 4 1 5
ABC Ambe r CHM Conve rte r Tria l ve rsion, http://w w w .proce sste x t.com/a bcchm.html o
c. Arthroscopic drainage. Art hros copy i s an at t ract i ve al t ernat i ve t o open drai nage, becaus e l ys i s of adhes i ons and removal of i nfl amed s ynovi um can oft en be accompl i s hed wi t hout t he prol onged i mmobi l i zat i on of open drai nage.
3. Other ancillary measures. Continuous passive motion earl y i n t reat ment i s s omewhat more effect i ve t han compl et e i mmobi l i zat i on at prevent i ng l os s of cart i l age and s ubchondral bone. The avas cul ar cart i l age depends on joi nt mot i on for nut ri t i on. Weight-bearing and ambulation of involved joints should be avoided unt i l effus i ons are gone; ot herwi s e, postinfectious inflammatory arthritis can del ay recovery.
4. Response to treatment o
o
a. Trends i n t he fol l owi ng cl i ni cal and l aborat ory paramet ers al l ow as s es s ment of t herapeut i c progres s :
(1) Res ol ut i on of fever
(2) Res ol ut i on of s ynovi al effus i on
(3) Improvement i n joi nt pai n, t endernes s , and range of mot i on
(4) Res ol ut i on of l eukocyt os i s (bl ood, s ynovi al fl ui d)
Pa g e 2 4 1 6
ABC Ambe r CHM Conve rte r Tria l ve rsion, http://w w w .proce sste x t.com/a bcchm.html
(5) St eri l i t y of s ynovi al fl ui d cul t ures
o
o
b. Failure of improvement s houl d l ead t o recons i derat i on of di agnos i s , recons i derat i on of choi ce or dos age of ant i bi ot i c, and cons i derat i on of open or art hros copi c drai nage.
VIII. Systemic Lupus Erythematosus (SLE) A. Definition SLE i s a chroni c aut oi mmune di s order charact eri zed by mul t i s ys t em i nvol vement and cl i ni cal exacerbat i ons and remi s s i ons . Circulating immune complexes and autoantibodies caus e t i s s ue damage and organ dys funct i on. Mani fes t at i ons i nvol vi ng t he s ki n, s eros al s urfaces , CNS, ki dneys , and bl ood cel l s are part i cul arl y charact eri s t i c. P.481
B. Epidemiology
1. The overal l prevalence of SLE i s approxi mat el y 15–50 cas es per 100,000 popul at i on. The preval ence i n young women of chi l dbeari ng age i s approxi mat el y ei ght t o t en t i mes t hat i n men. Afri can Ameri can women are affect ed approxi mat el y t hree t i mes as oft en as Caucas i an women.
2. The frequency of occurrence of lupus i s hi gher i n t he rel at i ves of affect ed i ndi vi dual s t han i n t he general popul at i on, and t he di s eas e concordance rat e i n
Pa g e 2 4 1 7
ABC Ambe r CHM Conve rte r Tria l ve rsion, http://w w w .proce sste x t.com/a bcchm.html
i dent i cal t wi ns approaches 50%.
C. Etiology No s i ngl e caus e of l upus has been di s covered. Compl ex i nt errel at i ons hi ps among envi ronment al fact ors , genet i cal l y det ermi ned hos t i mmune res pons es , and hormonal i nfl uences probabl y are cri t i cal i n t he i ni t i at i on as wel l as t he expres s i on of t he di s eas e.
1. Environmental factors. Vi rus es and drugs or t oxi ns have been purs ued as caus at i ve agent s , but nei t her has been s hown t o caus e i di opat hi c SLE. Mi crobi al t oxi ns and vi ral (part i cul arl y ret rovi ral ) product s can funct i on as superantigens, bi ndi ng t o t he hel per T-cel l recept ors and MHC cl as s II mol ecul e compl exes nons peci fi cal l y. Bi ndi ng t o B-cel l MHC cl as s II mol ecul es mi ght act i vat e hel per T cel l s t o generat e autoimmune responses.
2. Genetic factors. Twi n and fami l y s t udi es s ugges t a genet i c predi s pos i t i on t o SLE. The di s eas e occurs commonl y i n fami l i es wi t h heredi t ary defi ci enci es of earl y compl ement component s . The hi s t ocompat i bi l i t y ant i gens HLA-DR2 and HLA-DR3 are pres ent much more commonl y i n SLE pat i ent s t han i n cont rol s . Some of t he HLA-DR3 as s oci at i on may be due t o l i nkage wi t h a del et i on for a C4a gene. Speci fi c combi nat i ons of hi s t ocompat i bi l i t y ant i gens may be as s oci at ed wi t h t he product i on of s peci fi c aut oant i bodi es (e.g., HLA-DR or HLA-DQ as s oci at i ons wi t h Ro and La ant i bodi es ).
3. Autoimmunity. Los s of t ol erance t o aut oant i gens i s cent ral t o t he pat hogenes i s of SLE, and genet i c t endenci es t oward t he devel opment of aut oant i bodi es , B-cel l hyperact i vi t y, and T-cel l dys funct i on are evi dent
Pa g e 2 4 1 8
ABC Ambe r CHM Conve rte r Tria l ve rsion, http://w w w .proce sste x t.com/a bcchm.html
i n pat i ent s wi t h t he di s eas e. The t endency t o devel op aut oi mmuni t y i n SLE i s not MHC-l i nked but may be caus ed by genes out s i de t he hi s t ocompat i bi l i t y l oci .
4. Apoptosis. Programmed cel l deat h, or apopt os i s , l eads t o t he orderl y repl acement of ol d cel l s i n al l organs , i ncl udi ng t he B and T l ymphocyt es of t he i mmune s ys t em. Defect s i n apopt os i s genes have been di s covered i n l upus mous e ki ndreds , res ul t i ng i n t he devel opment of aut oi mmuni t y, perhaps rel at ed t o fai l ure t o del et e aut oreact i ve T cel l s or aut oant i body-produci ng B cel l s .
5. Hormonal influences. Lupus i s predomi nant l y a di s eas e of women of chi l dbeari ng age, but hormonal fact ors probabl y are more i mport ant i n modul at i on of t he expres s i on of di s eas e t han i n caus at i on. Es t rogen may be a permi s s i ve fact or i n pol ycl onal B-cel l act i vat i on.
D. Pathogenesis Al l of t he cl i ni cal feat ures of SLE are mani fes t at i ons of cel l ul ar and humoral i mmune dys funct i on; however, atherosclerosis can be a s econdary effect of vas cul ar damage t hat l eads t o furt her organ i s chemi a.
1. Immune complexes. Ci rcul at i ng ant i gen–ant i body (i mmune) compl exes are depos i t ed i n bl ood ves s el s and t he renal gl omerul us , i ni t i at i ng a pat hol ogi c res pons e t hat damages t hes e t i s s ues . Thes e compl exes are charact eri s t i c feat ures of act i ve di s eas e, and t hei r s i ze, s ol ubi l i t y, concent rat i on, and compl ement -fi xi ng propert i es as wel l as ves s el hydros t at i c forces are i mport ant i n det ermi ni ng t i s s ue depos i t i on.
Pa g e 2 4 1 9
ABC Ambe r CHM Conve rte r Tria l ve rsion, http://w w w .proce sste x t.com/a bcchm.html
2. Reticuloendothelial dysfunction. The chroni c ci rcul at i on of i mmune compl exes s eems t o be i mport ant i n t hei r pat hogeni ci t y, as occurs i n a chroni c s erum s i cknes s react i on. At t i mes , t he abi l i t y of t he ret i cul oendot hel i al s ys t em t o remove i mmune compl exes from t he ci rcul at i on may be overwhel med. Pat i ent s wi t h C4a gene del et i ons more commonl y have SLE, perhaps as a res ul t of i mpai red i mmune compl ex cl earance.
3. Autoantibodies are produced ei t her i n t he s et t i ng of s peci fi c, ant i gen-dri ven, i nduct i on of aut oreact i ve l ymphocyt e cl ones or pol ycl onal act i vat i on of di fferent B-cel l l i neages . Thes e aut oant i bodi es can caus e t he fol l owi ng condi t i ons : o
o
a. T issue damage. Ant i bodi es t o RBCs , W BCs , or pl at el et s can caus e i mmune cyt openi as .
o
o
b. Cellular dysfunction. Ant i bodi es t o l ymphocyt es can i mpai r l ymphocyt e funct i on and i nt ercel l ul ar s i gnal i ng; ant i neuronal ant i bodi es cros s i ng a breached bl ood–brai n barri er can i mpai r neuron funct i on.
o
o
c. Immune complex formation. Compl exes of ant i bodi es and doubl e-s t randed DNA are i mport ant i n medi at i on of aut oi mmune renal di s eas e.
4. Lymphocyte dysfunction. B-cel l hyperact i vi t y, +
+
i mpai red CD8 cel l funct i on, and augment ed CD4 cel l act i vi t y are pres ent i n vari ous combi nat i ons i n l upus
Pa g e 2 4 2 0
ABC Ambe r CHM Conve rte r Tria l ve rsion, http://w w w .proce sste x t.com/a bcchm.html
pat i ent s , l eadi ng t o aut oant i body product i on and i ncreas ed generat i on of i mmune compl exes .
E. Pathology
1. Characteristic microscopic changes o
o
a. Hematoxylin bodies. Amorphous mas s es of nucl ear mat eri al bound wi t h i mmunogl obul i n can be found i n connect i ve t i s s ue l es i ons t hat become purpl e–bl ue when s t ai ned wi t h hemat oxyl i n. Neut rophi l s t hat i nges t t hes e bodi es i n vi t ro are cal l ed LE cells.
o
o
b. Fibrinoid necrosis. In SLE, i mmune compl exes of DNA, ant i body t o DNA, and compl ement may s t ai n wi t h eos i n (whi ch can s t ai n i mmune compl exes as wel l as fi bri n) i n ves s el wal l s and connect i ve t i s s ue, demons t rat i ng s o-cal l ed “fi bri noi d necros i s .―
o
o
c. Onion-skin lesions. Les i ons charact eri s t i c of SLE i n s pl eni c art eri es are cal l ed oni on-s ki n l es i ons becaus e of t he concent ri c depos i t i on of col l agen around t hem, pres umabl y formed as vas cul i t i c l es i ons heal .
2. T issue changes o
o
a. Skin. Al t hough s ome of t he mi l der s ki n l es i ons i n SLE have onl y nons peci fi c l ymphocyt i c i nfi l t rat i on
Pa g e 2 4 2 1
ABC Ambe r CHM Conve rte r Tria l ve rsion, http://w w w .proce sste x t.com/a bcchm.html
i n peri vas cul ar l ocat i ons i n t he dermi s , more t ypi cal l upus l es i ons s how dermal–epidermal junction deposits of immunoglobulins and compl ement as wel l as necros i s . Cl as s i c di s coi d l es i ons s how fol l i cul ar pl uggi ng, hyperkerat os i s , and t he l os s of s ki n appendages . Frank vas cul i t i c l es i ons al s o can occur i n s mal l dermal ves s el s . o
o
b. Kidney. Immune compl ex depos i t i on i n t he ki dney can l ead t o vari ous hi s t ol ogi c pi ct ures of i nfl ammat i on. A cardi nal charact eri s t i c of renal pat hol ogy i n SLE i s t he t endency of t hes e P.482
pat hol ogi c pi ct ures t o change over t i me, bas ed ei t her on changes i n di s eas e act i vi t y or t herapy. Bi ops y s peci mens are graded wi t h regard t o act i vi t y (act i ve i nfl ammat i on) and chroni ci t y (gl omerul ar s cl eros i s and fi brot i c i nt ers t i t i al change); t he mos t t reat abl e l es i ons have hi gh act i vi t y and l ow chroni ci t y.
(1) Mesangial disease refers t o mesangial hypercellularity caus ed by t he pres ence of i mmunogl obul i n depos i t s and i s t he mos t common renal pat hol ogi c l es i on i n SLE.
(2) Focal proliferative nephritis i nvol ves cel l ul ar prol i ferat i ve change onl y i n segments of glomeruli and i n less than 50% of glomeruli.
(3) Diffuse proliferative nephritis i nvol ves
Pa g e 2 4 2 2
ABC Ambe r CHM Conve rte r Tria l ve rsion, http://w w w .proce sste x t.com/a bcchm.html
cel l ul ar prol i ferat i on i n most of the segments of t he gl omerul us and i n more than 50% of glomeruli.
(4) Pure membranous nephritis cons i s t s of subepithelial gl omerul ar bas ement membrane i mmunogl obul i n depos i t s wi t hout gl omerul ar hypercel l ul ari t y (cal l ed wire looping on l i ght mi cros copy), al t hough pat i ent s may have overl appi ng combi nat i ons of prol i ferat i ve and membranous forms on bi ops y.
(5) Interstitial inflammation can al s o occur i n al l of t he above pat hol ogi c pi ct ures .
o
o
c. CNS. Large ves s el vas cul i t i c l es i ons can occur (al t hough t hey are uncommon) i n focal pres ent at i ons of t he di s eas e, but focal areas of peri vas cul ar s mal l ves s el i nfl ammat i on, mi croi nfarct i on, or mi crohemorrhages are more t ypi cal and do not correl at e wel l wi t h abnormal i t i es found on i magi ng s t udi es [comput ed t omography (CT) or magnet i c res onance i magi ng (MRI) s cans ] or t he neurol ogi c exami nat i on. The phospholipid antibody syndrome may be as s oci at ed wi t h t he s mal l ves s el occl us i ve l es i ons .
o
o
d. Vasculitis. Infl ammat ory l es i ons of capi l l ari es , venul es , and art eri ol es , caus ed by i mmune compl ex depos i t i on and vari abl e cel l ul ar i nfi l t rat i on, are res pons i bl e for much of t he t i s s ue des t ruct i on and damage s een i n SLE.
o
Pa g e 2 4 2 3
ABC Amber CHM Converter Trial version, http://www.processtext.com/abcchm.html
o
e. Other tissue lesions. Nons peci fi c mi l d s ynovi t i s and l ymphocyt i c i nfi l t rat i on of mus cl es occur frequent l y. Nonbact eri al endocardi t i s oft en i s pres ent but t ypi cal l y i s as ympt omat i c.
F. Clinical features and laboratory findings (Table 10-13)
1. Manifestations of SLE. Fat i gue, wei ght l os s , and fever are promi nent systemic complaints. o
a. Skin. The butterfly rash (i .e., faci al eryt hema over t he cheeks and nos e) and t he chroni c, pot ent i al l y s carri ng, discoid lesions (i .e., coi n-s haped l es i ons wi t h hyperemi c margi ns , cent ral at rophy, and depi gment at i on) are t he mos t cl as s i c. Les s commonl y, bul l ous and macul opapul ar erupt i ons can occur. Nons carri ng, ps ori as i form l es i ons (subacute cutaneous lupus) occur and are as s oci at ed wi t h t he Ro ant i body. Recurrent mucous membrane ul cerat i on, general i z ed or focal al opeci a, di gi t al vas cul i t i s , and phot os ens i t i vi t y al s o are pot ent i al dermat ol ogi c feat ures ( Onl i ne Fi gure 10-10). o
Page 2424
ABC Ambe r CHM Conve rte r Tria l ve rsion, http://w w w .proce sste x t.com/a bcchm.html
ONLINE FIGURE 10-10 Sys t emi c l upus eryt hemat os us . Thi s pat i ent has a cl as s i c mal ar ras h wi t h t el angi ect as i as . (ACR Sl i de Col l ect i on of t he Rheumat i c Di s eas es , 3rd ed., 2004: 9108040.) o
o
b. Nerve
(1) CNS. Focal or di ffus e neurol ogi c di s orders occur i n approxi mat el y 50% of pat i ent s . Generalized manifestations i ncl ude s evere headache, react i ve depres s i ons , ps ychos es , cogni t i ve di s t urbances , and s ei zures . Ps ychos i s i n s ome l upus pat i ent s correl at es wi t h t he pres ence of ant i body t o ri bos omal P prot ei n. Focal seizures al s o have been des cri bed, and hemi pares i s , crani al nerve defi ci t s , t rans vers e myel i t i s , and movement di s orders may pi npoi nt di s cret e areas of i nvol vement . Lumbar punct ure (LP), el ect roencephal ography (EEG), and CT s canni ng oft en are unreveal i ng. However, MRI s canni ng reveal s CNS l es i ons i n many pat i ent s , es peci al l y t hos e wi t h focal
Pa g e 2 4 2 5
ABC Ambe r CHM Conve rte r Tria l ve rsion, http://w w w .proce sste x t.com/a bcchm.html
pres ent at i ons .
(2) Peripheral nervous system. Some pat i ent s have s ens ory or s ens ori mot or neuropat hi es , and t hos e wi t h vas cul i t i s of t he vas onervorum may mani fes t as mononeuri t i s mul t i pl ex.
o
o
c. Heart. Sympt omat i c pericarditis occurs i n approxi mat el y 20% of pat i ent s wi t h SLE and peri cardi al effus i ons on echocardi ography i n as hi gh as 50%, but t amponade i s uncommon. Myocarditis (conduct i on abnormal i t i es , arrhyt hmi as , and CHF) i s l es s common and may be revers i bl e i f t reat ed prompt l y wi t h cort i cos t eroi ds . Al t hough coronary vessel vasculitis can occur i n ful mi nant cas es , premat ure at heros cl eros i s i n s t eroi d-t reat ed pat i ent s i s a more common caus e of myocardi al i nfarct i on (MI) i n pat i ent s wi t h l upus . Nonbacterial endocardial lesions (Li bman-Sacks endocardi t i s ) can be as s oci at ed wi t h embol i c CNS event s , val vul ar dys funct i on, t he ant i phos phol i pi d ant i body s yndrome, or i nfect i ve endocardi t i s . P.483
o
o
d. Lung. At s ome t i me i n t he di s eas e cours e, approxi mat el y 30% of SLE pat i ent s have s ympt omat i c pleuritis. Di aphragmat i c fi bros i s or di aphragm dys funct i on may mani fes t as “ shrinking lung syndrome,― wi t h a res t ri ct i ve pi ct ure on pul monary funct i on t es t i ng.
Pa g e 2 4 2 6
ABC Ambe r CHM Conve rte r Tria l ve rsion, http://w w w .proce sste x t.com/a bcchm.html
Parenchymal i nvol vement (lupus pneumonitis) can be di ffi cul t t o di s t i ngui s h from acut e i nfect i ons ; l upus i nfi l t rat es may be uni l at eral or bi l at eral , t end t o be fl eet i ng, occur wi t h act i ve di s eas e, and l ack purul ent s put um. Hemoptysis occurs as a feat ure of pul monary vas cul i t i s and t he P.484
acute pulmonary hemorrhage syndrome. Diffuse interstitial lung disease i s recogni zed, al bei t uncommonl y. Pulmonary hypertension as a res ul t of i s ol at ed pul monary vas cul ar i nvol vement al s o occurs . o
o
e. Gastrointestinal tract. Al t hough s ympt oms of naus ea, vomi t i ng, and abdomi nal pai n are common, di agnos t i c t es t i ng oft en i s unreveal i ng. Overt intestinal vasculitis can l ead t o bowel i nfarct i on, perforat i on, and hemorrhage. Lupus - or cort i cos t eroi d-rel at ed pancreatitis and revers i bl e gas t ri c damage or hepat i t i s i nduced by NSAIDs can occur as wel l .
o
o
f. Kidney. Mos t l upus pat i ent s have s ome cl i ni cal and pat hol ogi c evi dence of renal i nvol vement . Act i ve di s eas e oft en i s announced by abnormal i t i es i n t he uri nary s edi ment (i .e., RBCs , W BCs , or cel l ul ar cas t s formed i n t he abs ence of acut e bact eri al i nfect i on). Ot her cl i ni cal l aborat ory feat ures t ypi cal l y as s oci at ed wi t h act i ve renal di s eas e are el evat i ons i n s erum creat i ni ne and bl ood urea ni t rogen (BUN) l evel s , decreas ed l evel s of s erum compl ement component s or i ncreas ed t i t ers of ant i bodi es agai ns t doubl e-s t randed DNA.
Pa g e 2 4 2 7
ABC Ambe r CHM Conve rte r Tria l ve rsion, http://w w w .proce sste x t.com/a bcchm.html
The renal bi ops y oft en can ai d i n t reat ment deci s i ons and t he det ermi nat i on of prognos i s , al t hough t he bi ops y pat hol ogy may change wi t h di s eas e cours e or t herapy.
(1) Mesangial disease i s t he mos t common and mi l des t form of renal i nvol vement and may be as ympt omat i c. Many pat i ent s have mi l d prot ei nuri a or RBCs or W BCs on uri nal ys i s . Treat ment us ual l y i s not requi red.
(2) Focal proliferative nephritis oft en has a good prognos i s as wel l and t ypi cal l y requi res t reat ment wi t h cort i cos t eroi ds al one; however, i t s more s evere pres ent at i ons bl end wi t h di ffus e prol i ferat i ve nephri t i s cl i ni cal l y and prognos t i cal l y.
(3) Diffuse proliferative nephritis i s t he mos t s evere pat hol ogi c l es i on and us ual l y i s as s oci at ed wi t h hypert ens i on, s evere prot ei nuri a, and s ome degree of renal i ns uffi ci ency. The mos t s evere cas es can be as s oci at ed wi t h crescents and t he rapi d devel opment of s evere renal i ns uffi ci ency. Cort i cos t eroi ds and cyt ot oxi c agent s t ypi cal l y are requi red for pres ervat i on of renal funct i on.
(4) Membranous glomerulopathy cl as s i cal l y mani fes t s as l arge amount s of prot ei n i n t he uri ne and nephrot i c s yndrome, us ual l y wi t h rel at i vel y few cel l s i n t he uri ne. Cort i cos t eroi d t herapy may hel p cont rol prot ei n l os s . Sl owl y
Pa g e 2 4 2 8
ABC Ambe r CHM Conve rte r Tria l ve rsion, http://w w w .proce sste x t.com/a bcchm.html
progres s i ve renal i ns uffi ci ency may devel op over t i me; i t i s not cl ear whet her t he addi t i on of cyt ot oxi c agent s ret ards t hi s progres s i on. o
o
g. Muscle and bone. Arthralgia and symmetrical arthritis oft en are feat ures of acut e SLE, but t he rare joi nt deformi t i es (Jaccoud' s arthropathy) t hat occur are a funct i on of t endon or l i gament l axi t y rat her t han eros i ve joi nt di s eas e. Infl ammat ory mus cl e i nvol vement us ual l y i s s ubcl i ni cal , but cl i ni cal i nfl ammat ory myopat hy can occur.
o
o
h. Other. Phot os ens i t i vi t y can t ri gger s ys t emi c s ympt oms as wel l as s ki n mani fes t at i ons . Raynaud's phenomenon and s econdary Sjögren's s yndrome each occur i n approxi mat el y 25% of pat i ent s .
2. Laboratory findings o
o
a. Hematologic findings. Anemi a i s common duri ng act i ve di s eas e and more oft en i s t he anemi a of chroni c di s eas e t han hemol yt i c anemi a. Ant i bodi es t o l eukocyt es al s o occur, wi t h aut oi mmune l ymphopeni a a common feat ure of act i ve di s eas e; neut ropeni a i s l es s common. Ant i bodi es t o pl at el et s can caus e chroni c i mmune t hrombocyt openi a or more acut e fal l s i n t he pl at el et count wi t h act i ve di s eas e. El evat i on of t he eryt hrocyt e s edi ment at i on rat e i s common and correl at es wi t h di s eas e act i vi t y i n s ome pat i ent s .
Pa g e 2 4 2 9
ABC Ambe r CHM Conve rte r Tria l ve rsion, http://w w w .proce sste x t.com/a bcchm.html o
o
b. Coagulation parameters. Ant i bodi es t o t he phos phol i pi d component s of i ndi vi dual cl ot t i ng fact ors can i nt erfere wi t h coagul at i on t es t i ng, caus i ng prol ongat i on of t he PTT not correct abl e by t he addi t i on of normal pl as ma. Paradoxi cal l y, pat i ent s wi t h t he PTT prol ongat i on (t he “ lupus anticoagulant―) have a hi gher frequency of t hrombos i s t han bl eedi ng.
o
o
c. Serologic findings. Phos phol i pi d ant i bodi es al s o can caus e fal s e-pos i t i ve t es t res ul t s for s yphi l i s , more oft en by i nt erference wi t h reagi n [e.g., rapi d pl as ma reagi n (RPR) or VDRL] t es t i ng t han wi t h ant i t reponemal [e.g., fl uores cent t reponemal ant i body abs orpt i on (FTA-ABS)] t es t i ng.
o
o
d. Immunologic findings. Pat i ent s wi t h l upus commonl y have low complement component (C3 and C4) levels as a res ul t of i mmune compl ex act i vat i on; i n many pat i ent s , fal l s i n s erum compl ement l evel s paral l el di s eas e fl are-ups i f compl ement s ynt hes i s i s unchanged. P.485
(1) Hypergammaglobulinemia refl ect s B-cel l hyperact i vi t y. By far t he mos t s i gni fi cant i mmunol ogi c fi ndi ngs i n SLE pat i ent s are autoantibodies, s uch as ANA, ant i -DNA, and
Pa g e 2 4 3 0
ABC Ambe r CHM Conve rte r Tria l ve rsion, http://w w w .proce sste x t.com/a bcchm.html
ant i -Sm.
(2) ANAs. Approxi mat el y 99% of pat i ent s wi t h SLE have ANAs . Thes e ant i bodi es are det ect abl e by an i mmunofl uores cence t echni que t hat i nvol ves human epi t hel i al cel l l i nes (e.g., HEp-2 cel l s ). W hen t he t es t s erum i s appl i ed t o t he epi t hel i al cel l s t hat have been froz en and cut t o expos e nucl ear component s , t he pat i ent 's ANAs i nt eract wi t h t he nucl ear mat eri al , and t hi s i nt eract i on can be det ect ed by fl uores cence mi cros copy. A diffuse or homogeneous i mmunofl uores cent s t ai ni ng pattern i s mos t common i n SLE, al t hough speckled, nucleolar, and rim patterns al s o can be s een (s ee onl i ne I B 3).
G. Diagnosis Careful cons i derat i on of t he hi s t ori cal and phys i cal fi ndi ngs t hat s ugges t t hi s mul t i s ys t em di s eas e i s neces s ary. Sys t emi c i l l nes s wi t h charact eri s t i c ras h, pol yart hri t i s , and s eros i t i s i s a common pres ent at i on, but t he pos s i bi l i t y of l upus s houl d be ent ert ai ned even when pat i ent s pres ent wi t h s eemi ngl y i s ol at ed hemat ol ogi c cyt openi as , CNS di s eas e, or gl omerul onephri t i s . In s us pi ci ous s et t i ngs , phys i ci ans s houl d s eek l aborat ory evi dence of aut oi mmuni t y and at t empt t o excl ude ot her i l l nes s es .
1. Diagnostic criteria (s ee Tabl e 10-13). The 1997 ARA revi s ed cri t eri a for t he di agnos i s of SLE are us eful when t he di s eas e i s s us pect ed, and t he pres ence over t i me of any four of t he el even cri t eri a s t rongl y s ugges t s t he di agnos i s . Al t hough fi ndi ngs s uch as al opeci a, peri ungual vas cul i t i s , and l ow s erum compl ement l evel s are not among t he cri t eri a, t hey may be s upport i ve evi dence i n i ndi vi dual pat i ent s .
Pa g e 2 4 3 1
ABC Ambe r CHM Conve rte r Tria l ve rsion, http://w w w .proce sste x t.com/a bcchm.html
TABLE 10-13 Criteria for the Classification of Systemic Lupus Erythematosus De Cri fini ter tio ion n 1. Fi x Mal ed ar ery ras t he h
ma , fl at or rai s ed , ove r t he ma l ar em i ne nce s, t en di n g to s pa re
Pa g e 2 4 3 2
ABC Ambe r CHM Conve rte r Tria l ve rsion, http://w w w .proce sste x t.com/a bcchm.html
t he nas ol a bi a l fol 2.
ds Ery
Di s t he coi ma d
t ou
ras s h
rai s ed pat che s wi t h ad her ent ker at o tic s ca lin g an d fol l i cul ar pl u ggi
Pa g e 2 4 3 3
ABC Ambe r CHM Conve rte r Tria l ve rsion, http://w w w .proce sste x t.com/a bcchm.html
ng, at r op hi c s ca rri n g ma y occ ur in ol d er l es i on 3.
s Ski
Ph n ot o ras s en h s i t i as vi t a y
res ul t of un us u al rea ct i on to s un lig
Pa g e 2 4 3 4
ABC Ambe r CHM Conve rte r Tria l ve rsion, http://w w w .proce sste x t.com/a bcchm.html
ht , by pat i en t hi s t or y or phy s i ci an obs erv at i 4.
on Ora
Ora l or l
nas
ul c op ers har yng eal ul c era tio n, us u al l y pai nl e ss, obs erv
Pa g e 2 4 3 5
ABC Ambe r CHM Conve rte r Tria l ve rsion, http://w w w .proce sste x t.com/a bcchm.html
ed by a phy s i ci 5.
an No
Art ner hri t os i is
ve art hri t is i nv ol v i ng tw o or mo re per i ph era l joi nt s , cha rac t eri z ed by t en der
Pa g e 2 4 3 6
ABC Ambe r CHM Conve rte r Tria l ve rsion, http://w w w .proce sste x t.com/a bcchm.html
nes s, sw el l i ng, or eff us i 6.
on (a)
Ser Pl e os i uri t tis isâ €”c onv i nci ng hi s t or y of pl e uri t ic pai n or rub he ard by a phy s i ci an
Pa g e 2 4 3 7
ABC Ambe r CHM Conve rte r Tria l ve rsion, http://w w w .proce sste x t.com/a bcchm.html
or evi de nce of pl e ura l eff us i on OR (b) Per i ca rdi t isâ €”d ocu me nt e d by EC G or rub or evi de nce of per i ca rdi
Pa g e 2 4 3 8
ABC Ambe r CHM Conve rte r Tria l ve rsion, http://w w w .proce sste x t.com/a bcchm.html
al eff us i 7.
on (a)
Re Per nal s i s di s t en ord t er pro t ei nur ia gre at e r th an 0.5 g/d ay or gre at e r t ha t 3+ if qu ant ita tio n not
Pa g e 2 4 3 9
ABC Ambe r CHM Conve rte r Tria l ve rsion, http://w w w .proce sste x t.com/a bcchm.html
per for me d OR (b) Cel l ul ar cas tsâ €” ma y be red cel l , he mo gl o bi n , gra nul ar, t ub ul a r, or mi x 8.
ed (a)
Ne Sei uro z ur
Pa g e 2 4 4 0
ABC Ambe r CHM Conve rte r Tria l ve rsion, http://w w w .proce sste x t.com/a bcchm.html
l og es â ic
€”i
di s n ord t he ers abs enc e of off en di n g dru gs or kno wn me t ab ol i c der an ge me nt s ; e.g ., ure mi a, ket oac i do sis
Pa g e 2 4 4 1
ABC Ambe r CHM Conve rte r Tria l ve rsion, http://w w w .proce sste x t.com/a bcchm.html
, or el e ct r ol y te im bal anc e OR (b) Ps y cho sis — in t he abs enc e of off en di n g dru gs or kno wn me t ab ol i c der an
Pa g e 2 4 4 2
ABC Ambe r CHM Conve rte r Tria l ve rsion, http://w w w .proce sste x t.com/a bcchm.html
ge me nt s ; e.g ., ure mi a, ket oac i do sis , or el e ct r ol y te im bal anc 9.
e (a)
He He ma mo t ol l yt i ogi c c
an
di s em ord i aâ er €” wi t h ret i cul
Pa g e 2 4 4 3
ABC Ambe r CHM Conve rte r Tria l ve rsion, http://w w w .proce sste x t.com/a bcchm.html
ocy t os is OR (b) Leu kop eni aâ €”l es s t ha n 40 00/ mm 3
t ot al on tw o or mo re occ as i ons OR (c) Ly mp ho pe ni a
Pa g e 2 4 4 4
ABC Ambe r CHM Conve rte r Tria l ve rsion, http://w w w .proce sste x t.com/a bcchm.html
— l es s t ha n 15 00/ mm 3
on tw o or mo re occ as i ons OR (d) Thr om boc yt o pe ni a — l es s t ha n 10 0,0 00/ mm
Pa g e 2 4 4 5
ABC Ambe r CHM Conve rte r Tria l ve rsion, http://w w w .proce sste x t.com/a bcchm.html
3
in
t he abs enc e of off en di n g dru gs 10. (a) Im Ant mu i -D nol NA ogi : c
ant
di s i bo ord dy er t o nat i ve DN A in ab nor ma l tit er OR (b) Ant
Pa g e 2 4 4 6
ABC Ambe r CHM Conve rte r Tria l ve rsion, http://w w w .proce sste x t.com/a bcchm.html
i -S M: pre s en ce of ant i bo dy to SM nuc l ea r ant i ge n OR (c) Pos itiv e fi n di n g of ant i ph os p hol i pi d ant i bo di e
Pa g e 2 4 4 7
ABC Ambe r CHM Conve rte r Tria l ve rsion, http://w w w .proce sste x t.com/a bcchm.html
s bas ed on (1) an ab nor ma l s er um l ev el of IgG or Ig M ant i -c ard i ol i pi n ant i bo di e s, (2) a pos itiv e t es t
Pa g e 2 4 4 8
ABC Ambe r CHM Conve rte r Tria l ve rsion, http://w w w .proce sste x t.com/a bcchm.html
res ul t for l up us ant i co ag ul a nt us i ng a sta nd ard me t ho d, or (3) a fal s epos itiv e s er ol o gi c t es t for s yp hi l i
Pa g e 2 4 4 9
ABC Ambe r CHM Conve rte r Tria l ve rsion, http://w w w .proce sste x t.com/a bcchm.html
s kno wn to be pos itiv e for at l ea st 6 mo nt h s an d con fi r me d by T re po ne ma pal lid um im mo bi l i z at i on
Pa g e 2 4 5 0
ABC Ambe r CHM Conve rte r Tria l ve rsion, http://w w w .proce sste x t.com/a bcchm.html
or fl u ore s ce nt t re po ne ma l ant i bo dy abs orp tio n t es t 11. An Ant ab i nu nor cl e ma ar l ant t i t i bo er dy of ant i nu cl e ar ant i bo dy by
Pa g e 2 4 5 1
ABC Ambe r CHM Conve rte r Tria l ve rsion, http://w w w .proce sste x t.com/a bcchm.html
im mu nof l uo res cen ce or an eq ui v al e nt as s ay at any poi nt in tim e an d in t he abs enc e of dru gs kno wn to
Pa g e 2 4 5 2
ABC Ambe r CHM Conve rte r Tria l ve rsion, http://w w w .proce sste x t.com/a bcchm.html
be as s o ci a t ed wi t h †œd rug -i n duc ed l up us â €• s yn dro me a. Thi s cl as s i fi cat i on is bas ed on 11 cri t eri a . For t he purpos e of i dent i f yi ng pat i en ts in cl i ni cal
Pa g e 2 4 5 3
ABC Ambe r CHM Conve rte r Tria l ve rsion, http://w w w .proce sste x t.com/a bcchm.html
s t udi e s, a pers on mus t have SLE i f any four or more of t he 11 cri t eri a are pres en t, s eri al l y or s i mul t aneous l y, duri ng any i nt erva l of obs erv at i on. b. The modi fi cat i on s to cri t eri o n numbe r 10 were
Pa g e 2 4 5 4
ABC Ambe r CHM Conve rte r Tria l ve rsion, http://w w w .proce sste x t.com/a bcchm.html
made in 1997. ECG, el ect ro cardi o gram. Adapt e d from Tan EM, Cohen AS, Fri es JF, et al : The 1982 revi s e d cri t eri a for t he cl as s i fi cat i on of s ys t em ic l upus eryt he mat os us (SLE). Art hri t i s Rheum 1982;2
Pa g e 2 4 5 5
ABC Ambe r CHM Conve rte r Tria l ve rsion, http://w w w .proce sste x t.com/a bcchm.html
5:1271 –127 7, wi t h permi s s i on from t he Ameri c an Col l eg e of Rheum at ol og y. Adapt e d from Hochb erg MC: Updat i ng t he Ameri c an Col l eg e of Rheum at ol og y revi s e d cri t eri a for t he cl as s i fi cat i on of
Pa g e 2 4 5 6
ABC Ambe r CHM Conve rte r Tria l ve rsion, http://w w w .proce sste x t.com/a bcchm.html
s ys t em l upus eryt he mat os us [l et t er ]. Art hri t i s Rheum 1997;4 0:1725 , wi t h permi s s i on of t he Ameri c an Col l eg e of Rheum at ol og y.
2. Differential diagnosis. Phys i ci ans mus t be careful t o excl ude ot her chroni c rheumat i c di s eas es , es peci al l y rheumat oi d art hri t i s , overl ap s yndromes (i nfl ammat ory myopat hi es or s cl eroderma overl appi ng wi t h SLE), and vas cul i t i c s yndromes i n arri vi ng at t he di agnos i s of SLE. The fol l owi ng s yndromes al s o s houl d be cons i dered i n t he s et t i ng of pos s i bl e SLE: o
o
a. Undifferentiated connective tissue disease
Pa g e 2 4 5 7
ABC Ambe r CHM Conve rte r Tria l ve rsion, http://w w w .proce sste x t.com/a bcchm.html
(UCTD). Thi s di s order i s us ed t o des cri be pat i ent s wi t h cl i ni cal feat ures of s everal connect i ve t i s s ue di s eas es and hi gh t i t ers of ant i body t o U 1 RNP. Pat i ent s wi t h UCTD may have cut aneous feat ures of SLE, dermat omyos i t i s , or s cl eroderma; i nfl ammat ory mus cl e di s eas e; and a des t ruct i ve form of art hri t i s more t ypi cal of rheumat oi d art hri t i s . Furt hermore, t he s evere renal and CNS mani fes t at i ons of SLE us ual l y are not pres ent . W hen fol l owed for prol onged peri ods , mos t pat i ent s wi t h t hi s di s eas e more cl os el y res embl e pat i ent s wi t h s cl eroderma or SLE. o
o
b. Drug-induced lupus. Chroni c i nges t i on of s everal drugs can preci pi t at e a s yndrome of polyserositis, arthritis, and antihistone ANAs. The drugs mos t commonl y as s oci at ed i ncl ude hydral azi ne, procai nami de, peni ci l l ami ne, i s oni azi d, and phenyt oi n. Renal di s eas e i s rare i n t hi s s yndrome, and dermal and CNS feat ures are l es s common t han i n i di opat hi c SLE. Drug-i nduced SLE t ypi cal l y res ol ves on di s cont i nuat i on of t he drug. Hepat i c acet yl at i on of drugs s uch as hydral azi ne, i s oni azi d, and procai nami de i s apparent l y s l ow i n pat i ent s who devel op t hi s s yndrome. However, t hes e drugs t ypi cal l y are t ol erat ed wel l by pat i ent s wi t h s pont aneous SLE.
o
o
c. Discoid lupus. Pat i ent s can have t ypi cal s ki n mani fes t at i ons of SLE wi t hout s ys t emi c di s eas e. Fi ft een percent of t hes e pat i ent s have pos i t i ve ANAs . Thes e pat i ent s s houl d be cons i dered t o have SLE when ot her di s eas e feat ures are pres ent .
Pa g e 2 4 5 8
ABC Ambe r CHM Conve rte r Tria l ve rsion, http://w w w .proce sste x t.com/a bcchm.html
3. Associated syndromes o
o
a. Neonatal lupus syndrome can devel op i n i nfant s of mot hers (who may not have SLE) who have hi gh-t i t er IgG antibodies t o Ro. The mat ernal ant i bodi es apparent l y cros s t he pl acent a, bi nd t o fet al t i s s ue, and caus e i mmunol ogi c i njury. The mos t t ypi cal feat ures i ncl ude evanes cent l upus s ki n l es i ons , but t rans i ent t hrombocyt openi a or hemol yt i c anemi a can occur. The mos t s eri ous cl i ni cal pres ent at i on occurs i n pregnant women when t he ant i body t o Ro bi nds t o fet al cardi ac t i s s ue and caus es congeni t al heart bl ock, whi ch can requi re permanent paci ng. Mos t mot hers of affect ed i nfant s devel op a mi l d vers i on of s ome aut oi mmune di s eas e over t i me, oft en SLE.
o
o
b. Phospholipid antibody syndrome can occur as a mi mi c of SLE or as part of t he di s eas e. One-t hi rd t o one-hal f of l upus pat i ent s exhi bi t phos phol i pi d ant i body i f t es t ed, al t hough t he as s oci at ed cl i ni cal s yndrome occurs l es s commonl y. P.486
(1) Mani fes t at i ons mos t commonl y i ncl ude venous or arterial thromboses, s omet i mes of l arge ves s el s , and can al s o i ncl ude episodic thrombocytopenia. Pregnant patients may experience fetal death aft er t he fi rs t t ri mes t er or premat ure bi rt h, and t hes e
Pa g e 2 4 5 9
ABC Ambe r CHM Conve rte r Tria l ve rsion, http://w w w .proce sste x t.com/a bcchm.html
probl ems may recur i n s ucces s i ve pregnanci es . Some of t hes e pat i ent s exhi bi t pl acent al t hrombos i s , i nfarct i on, or i ns uffi ci ency, but t he caus e of fet al deat h i s not al ways cl ear.
(2) Tes t i ng for ant i bodi es t o phos phol i pi ds i s i nvol ved (s ee onl i ne I B 6).
(3) Management pri mari l y i nvol ves chronic anticoagulation therapy, t ypi cal l y wi t h warfari n, aft er t he fi rs t epi s ode of cl i ni cal t hrombos i s .
H. Therapy Treat ment mus t be i ndi vi dual i zed t o t he feat ures t hat a part i cul ar pat i ent exhi bi t s , and i t need not al ways i ncl ude cort i cos t eroi ds . Pat i ent s mus t unders t and t hat t he prognos i s i n t hi s chroni c di s eas e general l y i s bet t er t han t hey fear and t hat t hei r compl i ance wi t h medi cat i on regi mens and avoi dance of di s eas e preci pi t ant s (e.g., ul t ravi ol et l i ght and emot i onal s t res s ) oft en can favorabl y affect t he di s eas e cours e. Phys i ci ans mus t be al ert t o di s eas e fl are-ups rel at ed t o s urgery, ant ecedent i nfect i ons , or t he pos t part um peri od.
1. Topi cal sunscreens cont ai ni ng para-ami nobenzoi c aci d (PABA) or benz ophenones are effect i ve i n prot ect i ng t he one-t hi rd of l upus pat i ent s who are phot os ens i t i ve.
2. NSAIDs are us ed i n ful l ant i -i nfl ammat ory dos es for fever, joi nt compl ai nt s , and s eros i t i s . Mi l d el evat i on i n t rans ami nas e l evel s oft en devel ops i n SLE pat i ent s on t hes e drugs , and as ept i c meni ngi t i s has been report ed i n l upus pat i ent s on i buprofen, t ol met i n, and s ul i ndac.
Pa g e 2 4 6 0
ABC Ambe r CHM Conve rte r Tria l ve rsion, http://w w w .proce sste x t.com/a bcchm.html
3. Antimalarial drugs (e.g., hydroxychl oroqui ne and chl oroqui ne) oft en are us ed t o t reat fat i gue, s ki n di s eas e, and art hri t i s i n SLE. Ret i nal pi gment ary depos i t i on l eadi ng t o bl i ndnes s i s a rare compl i cat i on of ant i mal ari al t reat ment s i nce hydroxychl oroqui ne repl aced chl oroqui ne. Pat i ent s s houl d be s creened by an opht hal mol ogi s t every 6 mont hs t o 1 year.
4. Corticosteroids o
o
a. T opical preparations. Some of t he s ki n mani fes t at i ons are i mproved by t reat ment wi t h t opi cal gl ucocort i coi ds , al t hough di s coi d l es i ons us ual l y requi re addi t i onal t reat ment wi t h ant i mal ari al agent s .
o
o
b. Systemic corticosteroids
(1) Glucocorticoids i n varyi ng dos es are oft en requi red t o cont rol s evere mani fes t at i ons of SLE and l es s s evere s ympt oms when t hey are pers i s t ent and di s abl i ng. Thes e drugs s houl d be us ed caut i ous l y, becaus e l ong-t erm t reat ment us ual l y i s needed and t ypi cal s i de effect s ens ue. Chronic arthritis and serositis may requi re gl ucocort i coi ds i f NSAIDs are not s uffi ci ent . Severe hemolysis, l i fe-t hreat eni ng thrombocytopenia, pneumonitis, CNS or peripheral nervous system disease, cl i ni cal l y evi dent cardiac or skeletal muscle disease, renal disease, and vasculitis are t ypi cal i ndi cat i ons for s ys t emi c gl ucocort i coi ds .
Pa g e 2 4 6 1
ABC Ambe r CHM Conve rte r Tria l ve rsion, http://w w w .proce sste x t.com/a bcchm.html
Us ual l y t he drug dos age s el ect ed i s proport i onat e t o t he s everi t y of t he i l l nes s ; t he dos age i s t apered as mani fes t at i ons s ubs i de.
(2) “ Pulse― corticosteroids may be neces s ary. Large dos es of cort i cos t eroi ds s omet i mes are admi ni s t ered i nt ravenous l y for part i cul arl y s evere cas es of SLE. Seri ous renal and CNS manifestations have been t he t ypi cal i ndi cat i ons for pul s e t reat ment , but dangerous cardiopulmonary and hematologic involvement al s o mi ght warrant t hi s aggres s i ve t reat ment .
5. Cytotoxic agents (e.g., azat hi opri ne and cycl ophos phami de) s omet i mes are us ed t o t reat s evere, refract ory feat ures of l upus , part i cul arl y renal disease. Int ravenous “pul s e― cycl ophos phami de i s effect i ve for t he t reat ment of di ffus e prol i ferat i ve gl omerul onephri t i s and ot her acut e s evere mani fes t at i ons of l upus .
6. Mycophenolate mofetil i s a revers i bl e i nhi bi t or of i nos i ne monophos phat e dehydrogenas e and bl ocks prol i ferat i on of B and T cel l s . It has been us ed s ucces s ful l y i n renal al l ograft reject i on. It al s o has been us ed as an al t ernat i ve t o cycl ophos phami de for t he t reat ment of l upus nephri t i s .
7. Intravenous immunoglobulin may be effect i ve. Large dos es of i mmunogl obul i n have been gi ven i nt ravenous l y t o t reat refract ory mani fes t at i ons of l upus , part i cul arl y
Pa g e 2 4 6 2
ABC Ambe r CHM Conve rte r Tria l ve rsion, http://w w w .proce sste x t.com/a bcchm.html
immune-mediated thrombocytopenia or hemolytic anemia. Thi s t herapy i s expens i ve, oft en mus t be gi ven mont hl y, and cannot be given to IgA-deficient patients becaus e of t he ri s k of anaphyl axi s . P.487
I. Prognosis Out comes are cl earl y bet t er t oday t han i n t he pres t eroi d era; mi l der forms of di s eas e are recogni zed and, pres umabl y, appropri at e drug t reat ment i mproves morbi di t y and mort al i t y rat es . Renal disease and infectious complications are s t i l l major caus es of deat h, and promi nent CNS disease can l ead t o s evere di s abi l i t y. The mort al i t y rat e i s hi gher i n pat i ent s of l ower s oci oeconomi c s t at us and educat i onal at t ai nment , a charact eri s t i c common t o many chroni c i l l nes s es . St eroi d-rel at ed compl i cat i ons can be cri ppl i ng (e.g., avas cul ar necros i s of t he femoral head and os t eoporot i c vert ebral fract ures ) or fat al (e.g., premat ure coronary at heros cl eros i s ).
IX. Systemic Sclerosis (SSc) A. Definition Sys t emi c s cl eros i s (s cl eroderma) i s a connect i ve t i s s ue di s eas e charact eri zed by wi des pread small vessel obliterative disease and fibrosis of the skin (es peci al l y di s t al , di gi t al s ki n) and multiple internal organs, i ncl udi ng t he heart , l ungs , ki dneys , and gas t roi nt es t i nal t ract . The des cri pt i on bes t fi t s t he di ffus e form, but l ocal i zed forms al s o exi s t . Thes e l at t er forms i nvol ve pat ches of s ki n and s ubcut aneous t i s s ues but do not i ncl ude di gi t al s ki n i nvol vement , Raynaud's phenomenon, or i nt ernal organ changes .
B. Epidemiology SSci s a rel at i vel y rare di s eas e; fami l i al cl us t eri ng i s uncommon. The di s eas e i s t hree t o four t i mes more common i n women t han i n men. Coal mi ners are at a hi gher ri s k for t he di s eas e, pos s i bl y as a res ul t of expos ure t o s i l i ca dus t .
Pa g e 2 4 6 3
ABC Ambe r CHM Conve rte r Tria l ve rsion, http://w w w .proce sste x t.com/a bcchm.html
C. Etiology The et i ol ogy of SSci s unknown. In s ome cas es i t s eems l i kel y t hat an envi ronment al agent (e.g., t oxi n or vi rus ) caus es vas cul ar endot hel i al i njury, wi t h l at er i mmunol ogi c res pons es l eadi ng t o cont i nued endot hel i al damage and t i s s ue fi bros i s . Support i ve evi dence i ncl udes t he i ni t i at i on of a fi bros i ng s yndrome i n pat i ent s wi t h pol yvi nyl chl ori de expos ure, Spani s h “t oxi c oi l ― s yndrome, and L-t rypt ophan–rel at ed eos i nophi l i a myal gi a s yndrome. W het her t he i ni t i at i ng event i s envi ronment al or i mmunol ogi c, t he earl y l es i on i n SSc s eems t o be one of vascular endothelial damage, es peci al l y i n s mal l ves s el s .
D. Pathogenesis There i s evi dence of T-cel l act i vat i on i n t he bl ood, s ki n, and l ungs . Unregul at ed i mmunol ogi c proces s es appear t o be res pons i bl e for cont i nui ng vas cul ar damage and wi des pread dermal and internal organ fibrosis.
1. Vascular endothelial damage. Intimal hyperplasia of s mal l ves s el s i n s ki n and i nt ernal organs occurs at t he earl i es t s t age of di s eas e. Luminal narrowing from t hi s endot hel i al fi brot i c proces s can l ead t o t i s s ue i s chemi a; t hi s proces s i s enhanced by rel eas e of t he pot ent vas ocons t ri ct or, endot hel i n, from damaged ves s el s . Physiologic vasoconstrictive stimuli (e.g., col d, emot i on, pl at el et -deri ved t hromboxane A 2 , and s erot oni n) can res ul t i n furt her narrowi ng and s ympt omat i c Raynaud' s phenomenon i n s ki n or i nt ernal organs . Local rel eas e of t he phys i ol ogi c vas odi l at ors ni t ri c aci d and endot hel i um-deri ved rel axat i on fact or (EDRF) i s di mi ni s hed i n damaged ves s el s . In pat i ent s wi t h renal vas cul ar i nvol vement , i ncreas ed reni n–angi ot ens i n product i on can caus e a vi ci ous ci rcl e of vas ocons t ri ct i on, whi ch i s t he pres umed mechani s m for renal crisis i n t hi s i l l nes s .
Pa g e 2 4 6 4
ABC Ambe r CHM Conve rte r Tria l ve rsion, http://w w w .proce sste x t.com/a bcchm.html
2. T issue fibrosis may be caus ed by t he heal i ng of i s chemi c l es i ons from s mal l ves s el i njury and from i mmune proces s es caus i ng i ncreas ed fi brobl as t i c act i vi t y. Cytokines and growth factors s ecret ed by l ymphocyt es , monocyt es , and pl at el et s [e.g., pl at el et -deri ved growt h fact or (PDGF), TGF-β] l ead t o i ncreas ed s ecret i on of col l agen and ground s ubs t ance by fi brobl as t s wi t h t he cons equent devel opment of fi brot i c l es i ons . Mas t cel l s are pres ent i n i ncreas ed amount s i n s cl eroderma s ki n and may i nt eract wi t h l ymphocyt es i n cyt oki ne-dri ven fi brobl as t act i vat i on.
3. Autoantibodies. Pat i ent s wi t h s cl eroderma oft en demons t rat e ANAs ; t hei r rel at i ons hi p i n s cl eroderma pat i ent s t o di s eas e pat hogenes i s i s unknown, but t hes e ant i bodi es may hel p di s t i ngui s h s ubs et s of t he i l l nes s (s ee IX E 2).
E. Clinical features The degree of s ki n change and t he t ype and progres s i on of i nt ernal organ i nvol vement are di fferent i n t he limited form as compared t o t he diffuse form of SSc. Al s o, s pont aneous fl uct uat i ons i n di s eas e act i vi t y can occur.
1. Skin involvement. Ski n changes occur i n 95% of patients wi t h SSc. An earl y edematous phase of s mal l ves s el endot hel i al i njury and i ncreas ed permeabi l i t y may progres s t hrough an indurative phase as i ncreas i ng amount s of col l agen are produced i n t he s ubcut aneous t i s s ue. Epi dermal and s ki n appendage at rophy may occur i n l at e forms of t he di s eas e (atrophic phase) as t he s ki n becomes progres s i vel y bound t o underl yi ng t i s s ue. o
Pa g e 2 4 6 5
ABC Ambe r CHM Conve rte r Tria l ve rsion, http://w w w .proce sste x t.com/a bcchm.html
(1) Distribution. Changes mos t oft en begi n i n t he fi ngers and hands and may s pread t o i nvol ve more proxi mal t i s s ues , i ncl udi ng t he t runk and face. The l ower ext remi t i es oft en are l es s s everel y i nvol ved (
Onl i ne Fi gure 10-11).
o
ONLINE FIGURE 10-11 Sys t emi c s cl eros i s . Thi s pat i ent has faci al changes charact eri s t i c of SSc wi t h t i ght l i ps and t i ght , s hi ny s ki n over her cheeks and nos e. (ACR Sl i de Col l ect i on of t he Rheumat i c Di s eas es , 3rd ed., 2004:9110110.) o
o
(2) Associated features
(a) Raynaud' s phenomenon occurs i n 95% of patients wi t h SSc. Epi s odi c vas os pas m of t he damaged s mal l ves s el s i n t he di gi t s res ul t s i n a triphasic color change of t he i nvol ved area as bl ood fl ow ceas es (whi t e), ret urns s l uggi s hl y (bl ue), and exhi bi t s react i ve hyperemi a (red).
(b) T elangiectasias can occur i n i nvol ved areas as wel l as on mucous membranes .
Pa g e 2 4 6 6
ABC Ambe r CHM Conve rte r Tria l ve rsion, http://w w w .proce sste x t.com/a bcchm.html
(c) Subcutaneous calcifications can occur, es peci al l y i n fi nger t i ps . P.488
(d) Salt-and-pepper changes may occur, i n whi ch t he s ki n can become t aut , s hi ny, and i mmobi l e, and hyperpi gment ed areas can al t ernat e wi t h depi gment ed areas .
(e) Microcapillary abnormalities may be pres ent . The di s t al nai l bed prol i ferat es and t he capi l l ary bed becomes vi s i bl y abnormal , wi t h di l at i on, t ort uos i t y, and l os s of ves s el s . Thes e abnormal i t i es can be s een by exami ni ng t he proxi mal nai l beds wi t h wi de-fi el d mi cros copy.
(f) Skin ulcers are pos s i bl e. Di s t al di gi t al ul cers and fi nger t aperi ng occur due t o di s t al i nfarct i ons and cons equent l os s of di gi t al pul p, s omet i mes res ul t i ng i n i nfect i on.
2. Gastrointestinal tract o
o
(1) Esophageal dysfunction i s t he mos t common mani fes t at i on of i nt ernal organ i nvol vement . Esophageal motility dysfunction and reflux devel op as col l agen repl aces s moot h mus cl e i n t he
Pa g e 2 4 6 7
ABC Ambe r CHM Conve rte r Tria l ve rsion, http://w w w .proce sste x t.com/a bcchm.html
l ower t wo-t hi rds of t he es ophagus ; s t ri at ed mus cl e i n t he upper one-t hi rd of t he es ophagus i s rel at i vel y unaffect ed. Esophageal strictures can res ul t from t he cons t ant refl ux, and ulceration can occur at t he gas t roes ophageal junct i on. Bari um s wal l ow and es ophageal manomet ry may be hel pful i n document i ng di s eas e ext ent . Upper gas t roi nt es t i nal endos copy i s hel pful i n eval uat i ng t he ext ent of i nvol vement and pos s i bl e Barret t 's es ophagus [s ee Chapt er 5 I B 1 e (5)]. o
o
(2) Similar small bowel involvement l eads t o i nt es t i nal hypomot i l i t y and i nt ermi t t ent crampi ng, di arrhea, and bacterial overgrowth malabsorption syndrome. Wide-mouth diverticula can be s een i n t he t rans vers e and des cendi ng col on i n areas of pat chy mus cul ari s i nvol vement . Lower gas t roi nt es t i nal cont ras t radi ographi c s t udi es or mal abs orpt i on t es t i ng may demons t rat e s mal l bowel or col oni c i nvol vement .
3. Lung. W i des pread s mal l pul monary art eri al narrowi ng and fi brot i c change event ual l y can l ead t o isolated pulmonary hypertension. More commonl y, a fi brot i c prol i ferat i on i n t he peri bronchi al and peri al veol ar t i s s ues l eads t o progres s i ve interstitial lung disease. Pat i ent s wi t h i nt ers t i t i al l ung di s eas e pres ent wi t h s ympt oms of progres s i ve dys pnea on exert i on and demons t rat e a res t ri ct i ve pat t ern on pul monary funct i on t es t i ng, a bet t er di s eas e i ndi cat or t han ches t radi ographi c changes . Hi gh-res ol ut i on CT (HRCT) s canni ng of t he l ung can reveal t he charact er and di s t ri but i on of fi ne s t ruct ural abnormal i t i es not vi s i bl e on ches t radi ography. St udi es wi t h bronchoal veol ar l avage (BAL)
Pa g e 2 4 6 8
ABC Ambe r CHM Conve rte r Tria l ve rsion, http://w w w .proce sste x t.com/a bcchm.html
have s hown t hat a s i gni fi cant proport i on of pat i ent s wi t h SSc have an al veol i t i s , and aggres s i ve chemot herapy may be i ndi cat ed i n t hi s s ubs et of pat i ent s . Pl euri t i s i s uncommon i n SSc as compared wi t h ot her rheumat i c di s eas es (e.g., rheumat oi d art hri t i s , and SLE).
4. Heart. Cl i ni cal cardi ac i nvol vement can t ake s everal forms . Cl i ni cal l y, acut e and chroni c peri cardi t i s are unus ual , but effus i ons oft en can be s een on ul t ras onography and are as s oci at ed wi t h myocardi al i nvol vement . If i nt ers t i t i al myocardi al di s eas e i s ext ens i ve, frank cardiomyopathy can res ul t , produci ng CHF, arrhyt hmi as , and conduct i on di s t urbances . Angi na can be a res ul t of fi brot i c i nvol vement of s mal l myocardi al ves s el s . Ambul at ory el ect rocardi ogram (ECG) moni t ori ng or exerci s e s t res s t es t i ng can hel p t o uncover dangerous arrhyt hmi as or s ubt l e i s chemi a i n need of t reat ment .
5. Kidney. Sudden renal fai l ure (scleroderma renal crisis) can occur oft en as a combi nat i on of i nt erl obul ar art ery fi brot i c damage and s ome vas ocons t ri ct i ve s t i mul us (e.g., di ures i s , bl ood l os s , s urgery). Mas s i ve reni n–angi ot ens i n rel eas e i n res pons e t o decreas ed renal perfus i on wors ens t he vas ocons t ri ct i on, and acut e renal fai l ure can occur. Malignant hypertension and microangiopathic hemolytic anemia oft en accompany t hes e renal event s . Chronic renal failure and death occur unl es s emergent t reat ment res t ores renal perfus i on. Some but not al l pat i ent s demons t rat e new hypert ens i on, uri nary prot ei n, or el evat ed s erum creat i ni ne t es t i ng before acut e renal decompens at i on, s o t hes e feat ures s houl d be moni t ored peri odi cal l y.
Pa g e 2 4 6 9
ABC Ambe r CHM Conve rte r Tria l ve rsion, http://w w w .proce sste x t.com/a bcchm.html
6. Muscle. In many pat i ent s , a mi l d i ndol ent myopat hy mani fes t ed by mi nor enz yme el evat i ons and perhaps mi l d weaknes s occurs but does not requi re t reat ment . In s ome pat i ent s , an overt inflammatory myopathy i dent i cal t o pol ymyos i t i s can occur (“ overlap― presentation).
7. Joint and tendon. More t han 50% of pat i ent s wi t h s cl eroderma devel op s wel l i ng, s t i ffnes s , and pai n i n fi nger, wri s t , and knee joi nt s . Mi l d, s el f-l i mi t ed i nfl ammat ory art hri t i s can occur earl y i n t he di s eas e, but t ypi cal joi nt i nvol vement i s l i mi t ed t o s ynovi al fi bros i s and i mpai red range of P.489
mot i on due t o t he general i zed res t ri ct i on of t he fi brot i c proces s . Tendon s heat h i nvol vement i s not unus ual ; carpal t unnel s yndrome can res ul t from ext ens i ve t endon s heat h fi bros i s i n t he wri s t .
8. Nerve. Neurol ogi c i nvol vement t ypi cal l y i s l i mi t ed t o fi brot i c entrapment neuropathies of t he medi an and t ri gemi nal nerves .
F. Clinical syndromes Cl as s i fi cat i on of t he di s eas e i nt o limited and diffuse forms i s i mport ant becaus e of di fferences i n organ i nvol vement and, t hus , prognos i s . In general , t he more wi des pread vi s ceral i nvol vement of t he di ffus e form gi ves i t a poorer prognos i s t han t he l i mi t ed vari ant . Scl eroderma can al s o exi s t i n overlap forms, t he mos t di s t i nct of whi ch i s UCT D.
Pa g e 2 4 7 0
ABC Ambe r CHM Conve rte r Tria l ve rsion, http://w w w .proce sste x t.com/a bcchm.html
a. Diffuse scleroderma (SSc) i s t ypi fi ed by proximal skin involvement (s ki n proxi mal t o t he MCP joi nt s or forearms i n vari ous defi ni t i ons ) and t he pres ence of Scl-70 antibodies or antinucleolar ANAs. Vi s ceral organ i nvol vement i n t hi s form t ypi cal l y occurs earl i er t han i n t he l i mi t ed vari ant . Renal i nvol vement i s much more common t han i n t he l i mi t ed form, and pul monary fi bros i s occurs earl i er and more qui ckl y.
b. Limited scleroderma t ypi cal l y has s ki n i nvol vement l i mi t ed t o t he di s t al ext remi t i es and face, and al s o i s known as t he CREST variant (i nvol vi ng t he coexi s t ence of s ubcut aneous calcinosis, Raynaud' s phenomenon, e sophageal motility dysfunction, sclerodactyly, and telangiectasia). Anticentromere antibody i s mos t cl os el y as s oci at ed wi t h t hi s form, and vi s ceral i nvol vement i s us ual l y more s l owl y progres s i ve t han i n t he di ffus e form. One except i on i s t he earl y devel opment of pulmonary vascular hypertension i n t hi s form due t o obl i t erat i ve changes i n pul monary art eri ol es . The pul monary fi bros i s s o t ypi cal of t he di ffus e vari ant occurs more s l owl y i n t hi s form but can become s evere aft er s everal decades of di s eas e.
c. Undifferentiated connective tissue disease (UCT D) i s a rheumat i c s yndrome t hat can i ncl ude cl i ni cal feat ures of s cl eroderma, SLE, and pol ymyos i t i s i n as s oci at i on wi t h t he pres ence of hi gh t i t ers of antibody t o U 1 RNP. Mos t cas es evol ve i nt o more t ypi cal s cl eroderma or SLE over t i me.
G. Diagnosis
Pa g e 2 4 7 1
ABC Ambe r CHM Conve rte r Tria l ve rsion, http://w w w .proce sste x t.com/a bcchm.html
1. Clinical approach. A di agnos i s of s cl eroderma s houl d be ent ert ai ned i n t he pres ence of s ympt oms of Raynaud' s phenomenon, distal skin thickening, and visceral organ involvement. Phys i ci ans s houl d eval uat e di s t al nai l beds for t he s ugges t i ve capi l l ary abnormal i t y and at t empt t o document t he l ocat i on and ext ent of t he s ki n t hi ckeni ng t o di fferent i at e l ocal from general i zed forms . In addi t i on, t hey s houl d l ook for evi dence of i nt ernal organ change. Charact eri s t i c ANA t es t res ul t s al s o may be us eful i n cl as s i fyi ng pat i ent s . o
o
a. Raynaud' s phenomenon. The abnormal vas cul ar res pons e t o col d or emot i onal s t i mul i i s pres ent i n mos t pat i ent s wi t h t he di ffus e or l i mi t ed form of s cl eroderma, but mos t pat i ent s wi t h Raynaud's phenomenon do not have s cl eroderma or anot her form of connect i ve t i s s ue di s eas e. As many as 5%–10% of nons moki ng women i n popul at i on s urveys may have Raynaud's phenomenon; t ypi cal s creeni ng procedures for s cl eroderma woul d i ncl ude exami nat i on for fi nger edema, fi ngernai l -fol d capi l l ary abnormal i t i es , and ANAs . Pat i ent s wi t h none of t hes e feat ures are unl i kel y t o devel op s cl eroderma. Becaus e pat i ent s can have Raynaud's phenomenon i n as s oci at i on wi t h ot her connect i ve t i s s ue di s eas es (SLE or pol ymyos i t i s ), a cl i ni cal s earch for t hes e i l l nes s es s houl d be made as wel l .
o
o
b. Distal skin thickening. Such epi dermal t hi ckeni ng i s a di agnos t i c feat ure of bot h t he di ffus e and l i mi t ed forms of s cl eroderma. Pat i ent s wi t h di ffus e s cl eroderma al s o have more proxi mal i nvol vement , t ypi cal l y of t he upper arms , upper l egs , or t runk. Fewer t han 5% of pat i ent s may have
Pa g e 2 4 7 2
ABC Ambe r CHM Conve rte r Tria l ve rsion, http://w w w .proce sste x t.com/a bcchm.html
vi s ceral i nvol vement t ypi cal of s cl eroderma wi t hout s ki n t hi ckeni ng (scleroderma sine scleroderma). o
o
c. Laboratory findings. Di fferences i n ANA t es t i ng may di s cri mi nat e bet ween di ffus e and l i mi t ed forms of s cl eroderma. Pat i ent s wi t h t he di ffus e form oft en have Scl-70 antibodies or antinucleolar ANAs. Pat i ent s wi t h t he more l i mi t ed forms oft en have anticentromere antibodies.
o
o
d. Visceral involvement. Typi cal i nt ernal organ i nvol vement (s ee IX E 1) l ends furt her s upport t o s cl eroderma di agnos i s and hel ps di s cri mi nat e bet ween di ffus e and l i mi t ed forms .
P.490
2. Differential diagnosis. Ski n t hi ckeni ng can be s een i n ot her i l l nes s es bes i des s cl eroderma. Typi cal l y, Raynaud's phenomenon, charact eri s t i c di s t al hand i nvol vement , and vi s ceral organ changes of s cl eroderma are abs ent . o
o
a. Local scleroderma. Two forms of purel y l ocal di s eas e, morphea and linear scleroderma, have s i mi l ar cl i ni cal and pat hol ogi c appearance t o s cl erodermat ous s ki n. Thes e forms mani fes t as l ocal i zed fi brot i c pl aques (morphea) or as l ongi t udi nal bands (l i near s cl eroderma).
(1) Morphea may occur at any age (i t i s more
Pa g e 2 4 7 3
ABC Ambe r CHM Conve rte r Tria l ve rsion, http://w w w .proce sste x t.com/a bcchm.html
common i n chi l dhood) as a di s eas e charact eri zed by s mal l ci rcums cri bed s ki n l es i ons (guttate morphea) or l arger pat ches (morphea en plaque).
(2) Linear scleroderma occurs mos t commonl y i n chi l dren and young adul t s . Faci al i nvol vement (coup de sabre) can be ext remel y di s fi guri ng. The pri nci pal i mpact of l i near s cl eroderma can be i nt erference wi t h funct i on or growt h of underl yi ng mus cl e or bone l eadi ng t o ext remi t y was t i ng or cont ract ures .
o
o
b. Eosinophilic fasciitis. Pai n, s wel l i ng, and t endernes s devel op i n one or more ext remi t i es , s omet i mes aft er an epi s ode of vi gorous exerci s e. Indurat i on of i nvol ved s ki n and s ubcut aneous t i s s ue devel ops , but wi t hout Raynaud's phenomenon or s cl erodact yl y. Pat i ent s oft en have marked peri pheral bl ood eos i nophi l i a, and a ful l -t hi cknes s s ki n bi ops y (i ncl udi ng underl yi ng fas ci a and mus cl e) i s requi red t o demons t rat e t he deep fas ci al eos i nophi l s and chroni c i nfl ammat ory cel l i nfi l t rat e.
o
o
c. Eosinophilia–myalgia syndrome. A cl i ni cal s yndrome rel at ed t o t he i nges t i on of L-tryptophan dietary supplements can be confus ed wi t h s cl eroderma (due t o t he s ki n t hi ckeni ng i n s ome cas es ) and may be res pons i bl e for s ome cas es of eos i nophi l i c fas ci i t i s . Pat i ent s who t ake L-t rypt ophan l i kel y have i nges t ed a t oxi c
Pa g e 2 4 7 4
ABC Ambe r CHM Conve rte r Tria l ve rsion, http://w w w .proce sste x t.com/a bcchm.html
cont ami nant generat ed by t he s ynt het i c proces s , al t hough s ome may met abol i ze L-t rypt ophan abnormal l y. The corners t ones of di agnos i s are 3
eosinophilia (>1000/mm ) and severe myalgia found i n t he abs ence of ot her pos s i bl e caus es . Vari abl e feat ures i ncl ude s ki n ras hes and i ndurat i on, i nt ers t i t i al pul monary i nfi l t rat es , and pol yneuropat hy. El evat i on of mus cl e enzymes [al dol as e, not creat i ne ki nas e (CK)] and hepat i c enzymes al s o can occur.
H. Therapy
1. General. Di s eas e expres s i on i n i ndi vi dual pat i ent s i s ext remel y het erogenous . Speci fi c t herapy s houl d focus on ext ent of organ damage and rat e of progres s i on of di s eas e.
2. Specific disease features o
o
a. Skin involvement. D-Peni ci l l ami ne and col chi ci ne are l argel y s el ect i ve t herapi es of t he pas t . Mycophenolate mofetil and methotrexate may be us eful i n rapi dl y progres s i ng s ki n di s eas e.
o
o
b. Raynaud' s phenomenon. Pat i ent s s houl d unders t and t he need t o cover t hei r head, ears , hands , feet , and t runk i n col d weat her t o mi ni mi ze refl ex vas ocons t ri ct i on, and s moki ng s houl d be di s couraged. Calcium channel blocking agents are us eful vas odi l at i ng agent s i n t he t reat ment of s cl eroderma. Pat i ent s s houl d avoi d nons el ect i ve
Pa g e 2 4 7 5
ABC Ambe r CHM Conve rte r Tria l ve rsion, http://w w w .proce sste x t.com/a bcchm.html
β-bl ockers t hat can al s o cont ri but e t o vas ocons t ri ct i on. Intravenous prostaglandin infusions (epopros t enol ) have been us ed wi t h s ucces s for t reat i ng refract ory vas os pas t i c epi s odes and for heal i ng s ki n ul cerat i ons . o
o
c. Esophageal involvement. Refl ux es ophagi t i s , oft en s evere, requi res t he us e of prot on-pump i nhi bi t ors (e.g., omepraz ol e, l ans oprazol e). Ant i refl ux meas ures (e.g., head-of-bed el evat i on, frequent s mal l feedi ngs ) are hel pful as wel l .
o
o
d. Pulmonary involvement. For i nt ers t i t i al pul monary di s eas e, dai l y oral cyclophosphamide may be t he mos t effect i ve t herapy for al veol i t i s and progres s i ve pul monary fi bros i s . Tobacco s moki ng may be s ynergi s t i c i n pul monary i njury. Treat ment for t he pri mary pul monary hypert ens i on-l i ke s yndrome i ncl ude t he endot hel i n-1 recept or ant agoni s t , bos ent an; t he phos phodi es t eras e-5 i nhi bi t or, s i l denafi l ; and vari ous pros t agl andi n anal ogs (e.g., i l opros t , epopros t enol ). Ant i coagul at i on i n combi nat i on wi t h vas odi l at ors i mproves s urvi val , whi ch probabl y rel at es t o t he hi gh i nci dence of i n s i t u mi crot hrombi or pul monary embol i .
o
o
e. Lower gastrointestinal involvement. Bact eri al overgrowt h mal abs orpt i on may be i mproved by i nt ermi t t ent t reat ment wi t h broad-spectrum antibiotics (e.g., t et racycl i ne). Prokinetic agents (e.g., oct reot i de, ci s apri de) hel p cons t i pat i on and bl oat i ng rel at ed t o l ower gas t roi nt es t i nal mot i l i t y
Pa g e 2 4 7 6
ABC Ambe r CHM Conve rte r Tria l ve rsion, http://w w w .proce sste x t.com/a bcchm.html
dys funct i on. P.491
o
o
f. Muscle involvement. W hen i nfl ammat ory myos i t i s occurs i n overl ap s yndromes , corticosteroids are us ed, as i n pol ymyos i t i s . Met hot rexat e may be hel pful i n t reat i ng myos i t i s and joi nt di s eas e. NSAIDs may be us ed i n t reat i ng art i cul ar s ympt oms , as wel l , t aki ng care t o moni t or renal funct i on cl os el y.
o
o
g. Renal involvement. Aggres s i ve cont rol of hypert ens i on i s bes t . The angiotensin-converting enzyme (ACE) inhibitors (e.g., capt opri l and enal apri l ) are major advances i n cont rol l i ng bl ood pres s ure i n pat i ent s wi t h s cl eroderma and may hel p revers e t he angi ot ens i n-dependent vas ocons t ri ct i ve s t at e of s cl eroderma renal cri s i s . The avoi dance of hypovol emi a i n di ures i s and i n t he peri operat i ve s t at e i s i mport ant .
o
o
h. Cardiac involvement. Medi cat i ons us eful for cont rol of angi na, CHF, and arrhyt hmi as are us ed when t hes e compl i cat i ons occur.
X. Inflammatory Myopathies (Polymyositis and Dermatomyositis) A. Definition Pol ymyos i t i s i s an i di opat hi c i nfl ammat ory mus cl e di s eas e as s oci at ed wi t h promi nent proxi mal mus cl e weaknes s , mus cl e
Pa g e 2 4 7 7
ABC Ambe r CHM Conve rte r Tria l ve rsion, http://w w w .proce sste x t.com/a bcchm.html
enz yme el evat i ons , charact eri s t i c myopat hi c el ect romyogram (EMG) pat t erns , and i nfl ammat ory i nfi l t rat es on mus cl e bi ops y. W hen t hi s compl ex i s accompani ed by a charact eri s t i c ras h i t i s cal l ed dermatomyositis. Dermat omyos i t i s i s much more common t han pol ymyos i t i s i n chi l dren.
B. Epidemiology Pol ymyos i t i s i s a rare di s eas e, occurri ng i n approxi mat el y 1 i n 200,000 i ndi vi dual s , wi t h a peak i nci dence i n chi l dhood and i n l at e adul t hood. It i s t wi ce as common i n femal es as i n mal es and may be as s oci at ed wi t h mal i gnancy i n t he adul t -ons et form.
C. Etiology The bas i c caus e of pol ymyos i t i s i s unknown. However, i t i s bel i eved t hat an i ni t i at i ng vi ral i nfect i on and al t ered i mmune res pons es are pot ent i al l y i mport ant i n caus at i on and pat hogenes i s . Lymphocyt e-medi at ed mus cl e cel l damage i s t hought t o be t he cent ral pat hogenet i c fact or i n t hi s di s eas e, al t hough s mal l ves s el damage i s al s o an i mport ant fact or i n dermat omyos i t i s .
1. Infections can caus e acut e myos i t i s . o
o
a. Elevated titers of antibodies to picornaviruses (e.g., coxs acki evi rus B) have been found i n s ome juveni l e dermat omyos i t i s pat i ent s , and hi gh t i t ers of ant i bodi es t o T oxopl as ma gondi i have been found i n s ome adul t pol ymyos i t i s pat i ent s .
o
o
b. Is ol at i on of t hes e organi s ms from mus cl e of pat i ent s wi t h pol ymyos i t i s or dermat omyos i t i s has not been accompl i s hed, and ant i bi ot i c t reat ment for t oxopl as mos i s has not i mproved myos i t i s i n pat i ent s wi t h hi gh t i t ers of ant i body t o T . gondi i .
Pa g e 2 4 7 8
ABC Ambe r CHM Conve rte r Tria l ve rsion, http://w w w .proce sste x t.com/a bcchm.html
2. Autoimmunity o
o
a. Humoral. Mos t pat i ent s wi t h i nfl ammat ory myopat hi es demons t rat e aut oant i bodi es when human t umor cel l l i nes [i .e., human epi t hel i al cel l s (HEp-2)] are us ed as t es t i ng s ubs t rat e, but i t i s not known whet her t hes e ant i bodi es i nt erfere wi t h t he normal phys i ol ogi c funct i on of t hei r ant i gens . Molecular mimicry may be i mport ant i n pol ymyos i t i s /dermat omyos i t i s , becaus e aut oant i bodi es di rect ed agai ns t t RNA s ynt het as es are found i n s ome pat i ent s , perhaps res ul t i ng from mi mi cry bet ween a vi rus and an epi t ope on t he i nt racel l ul ar enzyme. Vas cul ar depos i t s of i mmune compl exes and compl ement are as s oci at ed wi t h endot hel i al cel l i njury and s mal l ves s el obs t ruct i on i n dermat omyos i t i s , es peci al l y i n juveni l e dermat omyos i t i s .
o
o
b. Cellular
(1) Polymyositis. Peri pheral bl ood l ymphocyt es from pol ymyos i t i s pat i ent s produce a l ymphot oxi n t hat i s cyt ot oxi c t o +
mus cl e cel l s . In addi t i on, cyt ot oxi c CD8 T cel l s are t he predomi nant cel l t ype found i n t he i nfl ammat ory i nfi l t rat e i n mus cl e, apparent l y at t acki ng mus cl e cel l s expres s i ng i ncreas ed numbers of cl as s I MHC-res t ri ct ed markers .
(2) Dermatomyositis. Some s t udi es s ugges t
Pa g e 2 4 7 9
ABC Ambe r CHM Conve rte r Tria l ve rsion, http://w w w .proce sste x t.com/a bcchm.html +
t hat CD4 T cel l s and B cel l s are predomi nant i n dermat omyos i t i s i nfi l t rat es , i n cont ras t t o +
t he predomi nant l y CD8 T-cel l i nfi l t rat es i n pol ymyos i t i s . Perhaps humoral mechani s ms are rel at i vel y more i mport ant i n dermat omyos i t i s t han i n pol ymyos i t i s .
D. Pathology The major s i t es of i nfl ammat i on are s kel et al mus cl e and, l es s commonl y, cardi ac mus cl e. Ski n i nvol vement i s a mi nor pat hol ogi c feat ure.
1. Inflammatory infiltrate. Lymphocyt es and pl as ma cel l s are t he predomi nant i nfl ammat ory cel l s , al t hough macrophages , eos i nophi l s , and neut rophi l s al s o can be +
s een i n mus cl e t i s s ue. CD8 cel l s i nfi l t rat e mus cl e fi bers +
i n pol ymyos i t i s . In dermat omyos i t i s , CD4 cel l s and B cel l s oft en are cl us t ered around s mal l bl ood ves s el s wi t hi n t he mus cl e.
2. Muscle fiber damage. Spot t y mus cl e fi ber necros i s and degenerat i on occur, wi t h l os s of cros s s t ri at i ons and vari at i on i n t he s i ze of s urvi vi ng fi bers . Increas ed numbers of mus cl e nucl ei and enhanced bas ophi l i c s t ai ni ng of fi bers i ndi cat e regenerat i on i n t he mi ds t of cel l deat h. Int ers t i t i al fi brot i c i nfi l t rat es occur i n chroni c cas es .
E. Clinical features and laboratory findings Infl ammat ory myopat hi es are cl i ni cal l y grouped i nt o s peci fi c s yndromes , dependi ng on s peci fi c organ mani fes t at i ons and res ul t s of l aborat ory t es t s .
1. Organ involvement
Pa g e 2 4 8 0
ABC Ambe r CHM Conve rte r Tria l ve rsion, http://w w w .proce sste x t.com/a bcchm.html o
o
a. Skin. The ras h of dermat omyos i t i s cons i s t s of eryt hemat ous pat ches , whi ch s omet i mes are s cal i ng or at rophi c and are di s t ri but ed over t he face, neck, upper ches t , and ext ens or s urfaces . Onl y s cat t ered i nfl ammat ory i nfi l t rat es i n t he dermi s are evi dent on bi ops y.
(1) Pat hognomoni c s ki n fi ndi ngs i ncl ude heliotrope rash (a vi ol et di s col orat i on and s wel l i ng of t he eyel i ds ) and Gottron' s sign (heaped-up eryt hemat ous papul es over t he MCP or PIP joi nt s ) (Onl i ne Fi gure 10-12).
ONLINE FIGURE 10-12 Dermat omyos i t i s . Thi s pat i ent has a hel i ot rope ras h i nvol vi ng t he eyel i ds . (ACR Sl i de Col l ect i on of t he Rheumat i c Di s eas es , 3rd ed., 2004:9109010)
(2) Ot her s i gni fi cant fi ndi ngs i ncl ude mechanic' s hands (roughened eryt hemat ous s ki n and hypert rophi c changes of t he pal ms and fi ngers ) as wel l as t he eryt hemat ous V sign (ant eri or ches t ) and shawl sign (neck and upper back)
(Onl i ne Fi gure
Pa g e 2 4 8 1
ABC Ambe r CHM Conve rte r Tria l ve rsion, http://w w w .proce sste x t.com/a bcchm.html
10-13).
P.492
ONLINE FIGURE 10-13 Dermat omyos i t i s . “Mechani c's hands ― i s charact eri s t i c of t hi s di s order. Demons t rat ed i s t he cracki ng of t he fi nger pads . (ACR Sl i de Col l ect i on of t he Rheumat i c Di s eas es , 3rd ed., 2004:9109060.) o
o
b. Lung. Chroni c i nt ers t i t i al l ung di s eas e can occur, es peci al l y i n as s oci at i on wi t h ant i s ynt het as e ant i bodi es . As pi rat i on pneumoni t i s and vent i l at ory i ns uffi ci ency al s o can occur [See d (1) bel ow].
o
o
c. Joint. A mi l d, s ymmet ri cal i nfl ammat ory art hri t i s occurs uncommonl y and rarel y i s des t ruct i ve; i t i s s een mos t oft en i n pat i ent s wi t h t he s ynt het as e ant i bodi es [s ee X E 2 b (2) (a)].
o
o
d. Muscle. Mos t pat i ent s have gradual but s t eady progres s i on of mus cl e weaknes s ; however, s ome
Pa g e 2 4 8 2
ABC Ambe r CHM Conve rte r Tria l ve rsion, http://w w w .proce sste x t.com/a bcchm.html
pat i ent s have s uch ful mi nant cours es t hat acute respiratory failure or myoglobinuric renal failure can ens ue.
(1) Skeletal muscle weakness i s t he pri mary mani fes t at i on of pol ymyos i t i s .
(a) Symmetrical, proximal, and upper and lower extremity weakness occurs , caus i ng di ffi cul t y wi t h ri s i ng from a chai r, s i t t i ng up i n bed, or combi ng hai r.
(b) Pharyngeal muscle involvement can l ead t o s wal l owi ng di ffi cul t i es and as pi rat i on, and res pi rat ory mus cl e dys funct i on can l ead t o res pi rat ory fai l ure.
(2) Cardiac muscle involvement i s not as common, but when i t occurs i t mani fes t s as cardi omyopat hy wi t h CHF, arrhyt hmi as , and conduct i on di s t urbances .
2. Laboratory findings o
o
a. Muscle enzymes. An i ncreas ed concent rat i on of enzymes t ypi cal l y pres ent i n s kel et al mus cl e i s promi nent i n pol ymyos i t i s . CK and aldolase are rout i nel y meas ured, and CK fract i onat i on may s ugges t myocardi al i nvol vement i f t he MB fraction (i .e., t he CK i s oenzyme found mai nl y i n
Pa g e 2 4 8 3
ABC Ambe r CHM Conve rte r Tria l ve rsion, http://w w w .proce sste x t.com/a bcchm.html
myocardi um but al s o i n regenerat i ng s kel et al mus cl e) i s i ncreas ed. However, i n mos t pat i ent s , CK el evat i on i s caus ed by t he MM band (i .e., t he fract i on mos t preval ent i n s kel et al mus cl e). Serum myoglobin l evel s are al s o el evat ed i n mos t pat i ent s and may be more s ens i t i ve t han CK l evel s i n s ome myos i t i s pat i ent s . Enzyme el evat i ons us ual l y correl at e wi t h act i vi t y of t he mus cl e di s eas e and are us ed as a paramet er t o eval uat e t reat ment res pons e. o
o
b. Autoantibodies. Mos t pat i ent s wi t h pol ymyos i t i s /dermat omyos i t i s (80%–90%) have ant i bodi es t o nucl ear or cyt opl as mi c ant i gens . Rout i ne ANA t es t i ng agai ns t HEp-2 cel l s i s performed; ot her myos i t i s -s peci fi c ant i bodi es requi re s peci al i zed t es t i ng (e.g., i mmunopreci pi t at i on).
(1) Nonspecific autoantibodies. ANAs , part i cul arl y speckled-pattern ANAs, are t he mos t common aut oant i bodi es i n t he i nfl ammat ory myopat hi es , occurri ng i n more t han 50% of pat i ent s . Ot her ant i bodi es al s o can be s een, i ncl udi ng Ro antibodies, La antibodies, PM/Scl antibodies, and Ku antibodies.
(2) Specific autoantibodies. Several aut oant i bodi es are formed onl y i n i nfl ammat ory myopat hi es ; t hey t arget cyt opl as mi c or nucl ear cons t i t uent s of myocyt es . The appearance of t hes e
Pa g e 2 4 8 4
ABC Ambe r CHM Conve rte r Tria l ve rsion, http://w w w .proce sste x t.com/a bcchm.html
ant i bodi es , whi ch s eem t o correl at e wi t h cl i ni cal pat t erns of i nvol vement , may be more us eful i n cl as s i fyi ng pat i ent s .
(a) Synthetase antibodies (anti-aminoacyl–tRNA synthetases) are di rect ed agai ns t cyt opl as mi c enzymes i nvol ved i n s peci fi c ami no aci d at t achment t o t RNA. Jo-1 antibody, found i n 20% of myos i t i s pat i ent s , i s t he mos t common, di rect ed agai ns t hi s t i dyl –t RNA s ynt het as e. Pat i ent s wi t h t hes e ant i bodi es may be cons i dered t o have t he “ant i s ynt het as e s yndrome,― whi ch i s as s oci at ed wi t h DR3 and DRw52 hapl ot ypes . Thi s condi t i on, whi ch us ual l y occurs i n t he s pri ng, i s charact eri zed by acut e ons et of art hri t i s , i nt ers t i t i al l ung di s eas e, fever, mechani c hands , and Raynaud's phenomenon. Affect ed pat i ent s have a 5-year s urvi val rat e of 70%.
(b) Signal recognition particle antibodies (SRP antibodies) recogni ze one of t he component s of t he SRP, a cyt opl as mi c prot ei n compl ex i nvol ved i n pol ypept i de t rans fer acros s t he endopl as mi c ret i cul um. Immunogenet i c dat a i ndi cat e a DR5 and DRw52 as s oci at i on. Pat i ent s t end t o have an acut e ons et of s evere myal gi a and s i gni fi cant cardi ac probl ems s uch as pal pi t at i ons . Prognos i s i s very poor, wi t h
Pa g e 2 4 8 5
ABC Ambe r CHM Conve rte r Tria l ve rsion, http://w w w .proce sste x t.com/a bcchm.html
a 5-year s urvi val rat e of onl y 25%.
(c) Mi-2 antibodies t arget a nucl ear prot ei n of unknown funct i on, i n cont ras t t o t he cyt opl as mi c prot ei ns of t he ot her myos i t i s -s peci fi c ant i bodi es . Thi s ant i body pat t ern i s found i n pat i ent s wi t h cl as s i c dermat omyos i t i s . The 5-year s urvi val i s nearl y 100%.
3. Clinical syndromes. The t radi t i onal cl as s i fi cat i on s cheme i s gi ven bel ow. o
o
a. Polymyositis. Charact eri s t i c feat ures i ncl ude upper and l ower ext remi t y proxi mal mus cul ar weaknes s and el evat ed mus cl e enzymes . Al l organ mani fes t at i ons l i s t ed i n X E 1 can occur, except t he s ki n fi ndi ngs . Pat i ent s wi t h pol ymyos i t i s t end t o have di s eas e t hat i s l es s res pons i ve P.493
t han dermat omyos i t i s t o t herapy. The t reat ment -res i s t ant , acut el y s evere di s eas e i s oft en as s oci at ed wi t h cardi ac mus cl e i nvol vement , a fi ndi ng more promi nent i n pat i ent s wi t h SRP antibodies. Pat i ent s wi t h pol ymyos i t i s who have PM/Scl antibodies t end t o do rel at i vel y wel l . o
o
b. Dermatomyositis. Charact eri s t i c feat ures i ncl ude t he mus cl e weaknes s , el evat ed mus cl e enzymes , and organ mani fes t at i ons s een i n pol ymyos i t i s , but t he di s t i nct feat ure of t hi s
Pa g e 2 4 8 6
ABC Ambe r CHM Conve rte r Tria l ve rsion, http://w w w .proce sste x t.com/a bcchm.html
s yndrome i s t he pres ence of skin involvement. In general , dermat omyos i t i s pat i ent s res pond reas onabl y wel l t o t herapy. o
o
c. Polymyositis or dermatomyositis associated with malignancy. Vi s ceral mal i gnanci es occur i n approxi mat el y 10%–25% of cas es , es peci al l y i n el derl y i ndi vi dual s wi t h l at e-ons et i nfl ammat ory myopat hy. The i nci dence of mal i gnancy i s t hought t o be hi gher i n pat i ent s wi t h dermat omyos i t i s t han i n t hos e wi t h pol ymyos i t i s . The mos t common mal i gnanci es are t hos e t hat are t ypi cal i n mi ddl e-aged or el derl y i ndi vi dual s (i .e., cancers of t he l ung, gas t roi nt es t i nal t ract , breas t , ut erus , and ovari es ). Removal of t he mal i gnancy occas i onal l y res ul t s i n remi s s i on of t he mus cl e di s eas e, but t he prognos i s i n general for t hes e pat i ent s i s poor; t hey di e of t hei r mal i gnancy or uncont rol l abl e mus cl e di s eas e.
o
o
d. Juvenile dermatomyositis. Mos t chi l dren have dermat omyos i t i s rat her t han pol ymyos i t i s . Lat e i n t he di s eas e cours e, young pat i ent s may devel op mus cl e cont ract ures from cal ci fi c depos i t s i n t he chroni cal l y damaged mus cl e. A wi des pread s mal l art ery and capi l l ary vas cul i t i s may l ead t o s ki n, mus cl e, and bowel i s chemi c l es i ons . Ot herwi s e, prognos i s i s s i mi l ar t o t hat i n adul t forms of i nfl ammat ory myopat hy.
o
o
e. Overlap syndromes. Mus cl e di s eas e t hat i s i dent i cal t o pol ymyos i t i s can occur i n SLE, rheumat oi d art hri t i s , s ys t emi c s cl eros i s , and
Pa g e 2 4 8 7
ABC Ambe r CHM Conve rte r Tria l ve rsion, http://w w w .proce sste x t.com/a bcchm.html
Sjögren's s yndrome. Pat i ent s oft en have mi l der mus cl e di s eas e t hat i s res pons i ve t o t herapy, and prognos i s i s det ermi ned by t he feat ures of t he underl yi ng di s eas e more t han t he mus cl e fi ndi ngs . o
o
f. Inclusion body myositis. Thi s s yndrome i s cons i dered a s ubs et of i nfl ammat ory myopat hi es +
becaus e of t he as s oci at ed CD8 -cel l cyt ot oxi ci t y di rect ed agai ns t mus cl e fi bers . The s yndrome t ends t o affect older men and has a gradual di s eas e ons et and progres s i on. Al t hough pat i ent s have mus cl e weaknes s , CK el evat i ons , and myopat hi c EMGs , t hey are di s t i ngui s hed from t ypi cal pol ymyos i t i s pat i ent s by havi ng distal as well as proximal weakness, t ypi cal absence of autoantibodies, and a characteristic muscle biopsy s howi ng vacuol ar changes and di s t i nct i ve el ect ron mi cros copi c fi ndi ngs . The cl i ni cal feat ures oft en are poorl y res pons i ve t o i mmunos uppres s i ve t herapy, but pat i ent s have an excel l ent 5-year s urvi val rat e becaus e of t he s l ow progres s i on of t he di s eas e.
F. Diagnosis
1. Approach o
o
a. Distinctive muscle findings. A cl i ni cal di agnos i s of i nfl ammat ory myopat hy t ypi cal l y i s cons i dered when pat i ent s pres ent wi t h proximal muscle weakness or increased muscle enzymes (e.g., CK, al dol as e). True proxi mal mus cl e weaknes s mus t be di s t i ngui s hed from general i zed fat i gue, di s t al
Pa g e 2 4 8 8
ABC Ambe r CHM Conve rte r Tria l ve rsion, http://w w w .proce sste x t.com/a bcchm.html
weaknes s more t ypi cal of neuropat hy, and pai n-rel at ed weaknes s (i .e., joi nt or t endon pai n t hat prevent s ful l mus cl e cont ract i on). Pat i ent s wi t h t rue proxi mal mus cl e weaknes s compl ai n of bei ng unabl e t o comb t hei r hai r or t o ri s e from a s quat t i ng or s i t t i ng pos i t i on. If t rue proxi mal weaknes s appears l i kel y from t he hi s t ory and phys i cal exami nat i on, CK t es t i ng i s performed and EMG and bi ops y of s ympt omat i c del t oi d or quadri ceps mus cl e s houl d be cons i dered. MRI may al s o be us eful i n l ocal i zi ng t he bes t bi ops y s i t e. The di agnos i s of pol ymyos i t i s i s l i kel y i f a pat i ent has t hree of t he fol l owi ng mus cl e cri t eri a:
(1) Charact eri s t i c proxi mal mus cl e weaknes s
(2) Infl ammat ory cel l i nfi l t rat e and myofi bri l degenerat i on on mus cl e bi ops y
(3) Increas ed mus cl e enzyme l evel s
(4) Myopat hi c EMG changes
o
o
b. Supportive findings
(1) The characteristic skin findings, i f found, are s upport i ve evi dence and s omet i mes precede cl i ni cal evi dence of mus cl e i nvol vement .
Pa g e 2 4 8 9
ABC Ambe r CHM Conve rte r Tria l ve rsion, http://w w w .proce sste x t.com/a bcchm.html
(2) Becaus e 90% of pol ymyos i t i s pat i ent s have aut oant i bodi es when t hei r s erum i s t es t ed agai ns t HEp-2 cel l s , t he pres ence of a pos i t i ve t es t for aut oant i bodi es al s o i s s upport i ve evi dence for t he di s eas e, part i cul arl y i f a myositis-specific antibody i s found.
P.494
2. Consideration of malignancy. A s earch for a mal i gnancy i s warrant ed i n mi ddl e-aged or el derl y pol ymyos i t i s /dermat omyos i t i s pat i ent s . Thorough phys i cal exami nat i on, i ncl udi ng a careful pel vi c eval uat i on i n women, ches t radi ograph, rout i ne hemat ol ogi c and bi ochemi cal t es t i ng, uri nal ys i s , and s t ool t es t i ng for occul t bl ood s houl d be performed. Abnormal i t i es found s houl d be purs ued t hrough addi t i onal t es t i ng.
3. Differential diagnosis. Ot her ent i t i es s houl d be cons i dered i n maki ng a di agnos i s of i nfl ammat ory myopat hy, es peci al l y when t he pres ent at i on i s s omewhat at ypi cal . Some di s orders can be el i mi nat ed on cl i ni cal grounds ; ot hers requi re l aborat ory t es t i ng, EMG, or mus cl e bi ops y. o
o
a. Endocrine disorders
(1) Hypothyroidism can mani fes t as proxi mal mus cl e achi ng, weaknes s , and mi l d-t o-moderat e CK el evat i on.
Pa g e 2 4 9 0
ABC Ambe r CHM Conve rte r Tria l ve rsion, http://w w w .proce sste x t.com/a bcchm.html
(2) Hyperthyroidism can mani fes t as di ffus e weaknes s ; CK l evel s t ypi cal l y are normal .
(3) Patients with Cushing' s syndrome may have l ower more t han upper ext remi t y proxi mal mus cl e weaknes s ; mus cl e enzymes are normal .
o
o
b. Drugs. Numerous drugs and t oxi ns can caus e mus cl e weaknes s and s omet i mes CK el evat i on; t hes e effect s us ual l y res ol ve when t he agent i s removed. Some of t he mos t common of t hes e agent s i ncl ude al cohol , chol es t erol -l oweri ng drugs (e.g., cl ofi brat e, gemfi broz i l , and l ovas t at i n), col chi ci ne, chl oroqui ne, cort i cos t eroi ds , D-peni ci l l ami ne, and zi dovudi ne.
o
o
c. Muscle diseases. Mus cul ar dys t rophi es and met abol i c mus cl e di s eas es can be di s t i ngui s hed by careful fami l y hi s t ory, di s t i ngui s hi ng pat t erns of weaknes s , and l ack of i nfl ammat i on on mus cl e bi ops y. Some requi re exerci s e t es t i ng wi t h venous l act at e det ermi nat i ons or el ect ron mi cros copi c exami nat i on of mus cl e.
o
o
d. Neurologic diseases. Earl y i n t he di s eas e cours e, i l l nes s es s uch as myas t heni a gravi s or amyot rophi c l at eral s cl eros i s (ALS) can mi mi c i nfl ammat ory myopat hy. The pres ence of ocul ar mus cl e i nvol vement or charact eri s t i c EMG or nerve conduct i on vel oci t y (NCV) fi ndi ngs can hel p i n
Pa g e 2 4 9 1
ABC Ambe r CHM Conve rte r Tria l ve rsion, http://w w w .proce sste x t.com/a bcchm.html
di agnos i ng t hes e di s eas es . o
o
e. Infections
(1) Bacterial. Some cas es of Lyme di s eas e are as s oci at ed wi t h myopat hy.
(2) Viral. Acut e and conval es cent s erol ogi es can di s t i ngui s h vi ral i l l nes s . Common caus es i ncl ude coxs acki evi rus , echovi rus , i nfl uenz a vi rus , and HIV.
(3) Parasitic. Serol ogi c t es t i ng can hel p di s t i ngui s h t oxopl as mos i s or t ri chi nos i s from pol ymyos i t i s .
o
o
f. Sarcoidosis. Infl ammat ory mus cl e i nvol vement , whi ch can be di s t i ngui s hed by charact eri s t i c organ i nvol vement and fi ndi ng noncas eat i ng granul omas on mus cl e bi ops y, may occur.
G. Therapy
1. Corticosteroids. Large dos es of cort i cos t eroi ds appear t o be effect i ve i n cont rol l i ng t he mus cl e di s eas e i n mos t pat i ent s . Predni s one us ual l y i s begun at a dos e of 60 mg/day and i s reduced gradual l y over s everal mont hs as mus cl e s t rengt h i mproves and CK l evel fal l s . Al t ernat e-day regi mens can be us ed t o prevent s t eroi d t oxi ci t y, but onl y aft er t he normal i zat i on of CK l evel and ret urn of mus cl e s t rengt h.
Pa g e 2 4 9 2
ABC Ambe r CHM Conve rte r Tria l ve rsion, http://w w w .proce sste x t.com/a bcchm.html
2. Immunosuppressive agents. Pat i ent s who do not res pond t o t he previ ous l y ment i oned cort i cos t eroi d s chedul es wi t hi n 3 mont hs are cons i dered t o be nonres ponders t o t reat ment . Frequent l y, t he addi t i on of met hot rexat e or az at hi opri ne al l ows cont rol of t he mus cl e di s eas e and gradual t aperi ng of t he s t eroi ds . The +
CD4 cel l -s peci fi c drug cycl os pori ne al s o has been effect i ve i n s ome pat i ent s who do not res pond t o general i mmunos uppres s i on. Tacrol i mus and mycophenol at e mofet i l may be effect i ve i n t reat i ng i nt ers t i t i al di s eas e and mus cl e i nfl ammat i on.
3. Physical therapy. W hen t he di s eas e proces s has been cl i ni cal l y cont rol l ed (i .e., CK l evel s normal i zed and mus cl e s t rengt h i mproved), mus cl e s t rengt heni ng exerci s e and aerobi c t rai ni ng can be i ni t i at ed. Mi l d exerci s e and pas s i ve s t ret chi ng t o prevent cont ract ures s houl d be us ed i n act i ve di s eas e.
4. Hydroxychloroquine. Thi s agent may hel p cont rol t he ras h of dermat omyos i t i s . P.495
5. Intravenous immunoglobulin. Thi s agent has been hel pful i n s ome s t eroi d-res i s t ant pat i ent s and i n pat i ent s pres ent i ng wi t h earl y, s evere di s eas e i ncl udi ng res pi rat ory i nvol vement /res pi rat ory fai l ure.
XI. Sjögren's Syndrome (SS) A. Definition
Pa g e 2 4 9 3
ABC Ambe r CHM Conve rte r Tria l ve rsion, http://w w w .proce sste x t.com/a bcchm.html
Sjögren's s yndrome (al s o cal l ed sicca syndrome) i s an i di opat hi c, aut oi mmune di s order charact eri zed by dry mout h (xerostomia) and dry eyes (keratoconjunctivitis sicca). Vari abl e lacrimal or salivary gland enlargement can occur (rel at ed t o l ymphocyt i c i nfi l t rat i on of l acri mal and s al i vary gl ands ).
B. Classification Sjögren's s yndrome i s di vi ded i nt o pri mary and s econdary forms . The t wo forms can be di s t i ngui s hed on t he bas i s of cl i ni cal feat ures and HLA as s oci at i ons .
1. Primary Sjögren' s syndrome has charact eri s t i c organ and exocri ne gl andul ar feat ures and a s i gni fi cant l y i ncreas ed as s oci at i on wi t h HLA-DR3. Pat i ent s can pres ent wi t h a vari et y of cl i ni cal mani fes t at i ons t hat are uncommon i n t he s econdary form, mai nl y due t o a wi der at t ack on exocri ne gl ands i n t he pri mary form. Speci fi c organ i nvol vement i s as fol l ows : o
o
a. Skin: dry s ki n and vagi na, Raynaud's phenomenon, and purpura (vas cul i t i s )
o
o
b. Lung: recurrent i nfect i ons and i nt ers t i t i al fi bros i s
o
o
c. Gastrointestinal tract: angul ar chei l i t i s , oral candi di as i s , beefy red t ongue, recurrent parot i t i s , dys phagi a, at rophi c gas t ri t i s , chroni c act i ve hepat i t i s , bi l i ary ci rrhos i s , and pancreat i t i s
o
o
d. Kidney: renal t ubul ar aci dos i s and i nt ers t i t i al nephri t i s
Pa g e 2 4 9 4
ABC Ambe r CHM Conve rte r Tria l ve rsion, http://w w w .proce sste x t.com/a bcchm.html o
o
e. Muscle: i ndol ent myos i t i s
o
o
f. Nerve: CNS i nvol vement (pos s i bl y vas cul i t i s , wi t h cl i ni cal mani fes t at i ons s i mi l ar t o CNS l upus ) and peri pheral neuropat hy
o
o
g. Hematologic system: s pl enomegal y wi t h neut ropeni a, l ymphomas , and ps eudol ymphomas
o
o
h. Joint: art hral gi as or mi l d i nfl ammat ory art hri t i s
o
o
i. Endocrine system: chroni c t hyroi di t i s [i ncreas ed t hyroi d-s t i mul at i ng hormone (TSH) l evel s and ant i t hyroi d ant i bodi es , perhaps i n as many as 50% of pat i ent s ]
o
o
j. Vasculitis (pol yart eri t i s -l i ke)
2. Secondary Sjögren' s syndrome occurs i n t he s et t i ng of anot her rheumat i c di s eas e (e.g., rheumat oi d art hri t i s , SLE, and s cl eroderma) and has an i ncreas ed as s oci at i on wi t h HLA-DR4. Pat i ent s us ual l y have s ympt oms t hat are l i mi t ed t o l acri mal and s al i vary gl and abnormal i t i es , al t hough t hey al s o have t ypi cal feat ures of a pri mary rheumat i c di s eas e.
C. Diagnosis
Pa g e 2 4 9 5
ABC Ambe r CHM Conve rte r Tria l ve rsion, http://w w w .proce sste x t.com/a bcchm.html
1. Clinical approach. Pat i ent s are mos t commonl y eval uat ed for Sjögren's s yndrome when t hey have compl ai nt s of dry eyes, dry mouth, or salivary gland enlargement. Pri mary Sjögren's s yndrome al s o s houl d be cons i dered i n t he s et t i ng of cutaneous vasculitis (purpura), CNS dysfunction s i mi l ar t o t hat whi ch occurs i n SLE (ei t her focal neurol ogi c abnormal i t i es or di ffus e feat ures s uch as depres s i on, ps ychos i s , or cogni t i ve dys funct i on), or t he cons pi cuous pres ence of ot her cl i ni cal feat ures l i s t ed i n XI B 1. o
o
a. Compl ai nt s of gritty eyes and dry mouth can be i nves t i gat ed i n t he fol l owi ng way:
(1) Ophthalmologic slit-lamp examination wi t h fl uores cei n or Ros e Bengal s t ai ni ng can be us ed t o exami ne for t he punct at e s t ai ni ng of kerat oconjunct i vi t i s . The Schirmer test, whi ch meas ures t he degree of t ear wet t i ng on fi l t er paper, i s a s upport i ve t es t ; l i mi t ed wet t i ng s ugges t s t hat kerat oconjunct i vi t i s found on s t ai ni ng i s s i cca-rel at ed.
(2) Lip biopsy of t he l ower l i p mucos a can be us ed t o s earch for charact eri s t i c l ymphocyt i c i nfi l t rat i on of t he mi nor s al i vary gl ands i n t hat l ocat i on.
o
o
b. Salivary gland enlargement can be eval uat ed wi t h an MRI of t he parot i d gl and. A nonhomogeneous dens i t y, whi ch can be di s t i ngui s hed from parot i t i s , t umor, and normal gl andul ar t i s s ue, may be evi dent .
Pa g e 2 4 9 6
ABC Ambe r CHM Conve rte r Tria l ve rsion, http://w w w .proce sste x t.com/a bcchm.html o
o
c. Salivary flow rate tests (sialometry) and radiographic studies (sialography) are s ens i t i ve but nons peci fi c t es t s and are not oft en us ed.
o
o
d. Many laboratory abnormalities occur and may offer s ugges t i ve evi dence; however, no s peci fi c l aborat ory t es t i s di agnos t i c. P.496
(1) T ests suggesting chronic inflammation. Anemi a of chroni c di s eas e, el evat ed ESR, hypergammagl obul i nemi a, and rheumat oi d fact or oft en are found i n Sjögren's s yndrome, whet her pri mary or s econdary.
(2) Autoantibody findings. ANAs are s een frequent l y i n Sjögren's s yndrome; ant i bodi es t o t he s mal l RNA prot ei n Ro are found i n 70% of pat i ent s and ant i bodi es t o t he prot ei n La are found i n 40% of pat i ent s . La ant i bodi es rarel y are found wi t hout Ro ant i bodi es , and t he pres ence of bot h i s rel at i vel y s peci fi c for Sjögren's s yndrome. Sal i vary duct ant i bodi es are common onl y i n t he s econdary form.
2. Differential diagnosis o
Pa g e 2 4 9 7
ABC Ambe r CHM Conve rte r Tria l ve rsion, http://w w w .proce sste x t.com/a bcchm.html o
a. Salivary gland enlargement al s o may be caus ed by a l ymphoi d or parot i d gl and neopl as m, granul omat ous i nfi l t rat i on (s arcoi dos i s ), al cohol us e, ci rrhos i s , s t arvat i on, di abet es , amyl oi dos i s , graft -vers us -hos t di s eas e (GVHD), hyperl i pi demi as , or i nfect i on (e.g., bact eri al i nfect i ons , mumps , HIV).
o
o
b. Dry mouth may be caus ed by t he us e of cert ai n drugs , i ncl udi ng t ri cycl i c ant i depres s ant s , phenot hi azi nes , and ant i hi s t ami nes .
o
o
c. Dry mouth and dry eyes are common i n t he geri at ri c popul at i on, and t hey oft en are pres ent wi t hout any ot her feat ures of Sjögren's s yndrome.
D. Therapy
1. General considerations o
o
a. Scrupulous oral hygiene i s i mport ant t o prevent rampant dent al cari es .
o
o
b. Pat i ent s wi t h pri mary Sjögren's s yndrome who demons t rat e s al i vary gl and or l ymphoi d t i s s ue enl argement mus t be cons i dered at risk for lymphoma, becaus e non-Hodgki n's l ymphomas occur at 40 t i mes t he normal rat e i n t hes e pat i ent s .
2. Symptomatic treatment
Pa g e 2 4 9 8
ABC Ambe r CHM Conve rte r Tria l ve rsion, http://w w w .proce sste x t.com/a bcchm.html o
o
a. Xerostomia. Avoi dance of drugs t hat dry out t he mout h and frequent dri nks of wat er or ot her l i qui ds t o keep t he mout h wet are t he us ual s ympt omat i c meas ures for t hi s condi t i on. Art i fi ci al s al i vas are commerci al l y avai l abl e, whi ch may hel p wi t h mout h wet t i ng and prevent i on of dent al cari es . Pi l ocarpi ne hydrochl ori de, a chol i nergi c agoni s t , i s avai l abl e for oral us e and can i ncreas e s ecret i on by t he exocri ne gl ands i ncreas i ng s al i vary fl ow.
o
o
b. Keratoconjunctivitis sicca. Art i fi ci al t ears (met hyl cel l ul os e) are us ed as oft en as needed t o keep t he eyes l ubri cat ed. Refract ory s ympt oms may res pond t o punct al i ns ert i on of s i l i cone pl ugs .
o
o
c. Features of severe disease. Progressive pneumonitis, vasculitis, neuropathy, and CNS involvement may requi re ant i -i nfl ammat ory t reat ment wi t h cort i cos t eroi ds and i mmunos uppres s i ve drugs .
E. Prognosis Out come for pat i ent s wi t h pri mary Sjögren's s yndrome depends on t he s everi t y of i nvol vement of organs ot her t han t he s al i vary gl ands , part i cul arl y t he CNS. Al s o, t he ri s k of l ymphoma i s hi gher i n more s evere cas es . Feat ures of t he pri mary rheumat i c di s eas e det ermi ne t he prognos i s for pat i ent s wi t h s econdary Sjögren's s yndrome.
XII. Vasculitis A. Definition Vas cul i t i c s yndromes are a group of cl i ni cal l y di s parat e di s orders
Pa g e 2 4 9 9
ABC Ambe r CHM Conve rte r Tria l ve rsion, http://w w w .proce sste x t.com/a bcchm.html
charact eri zed by necrosis and inflammation of blood vessel walls. Vas cul i t i s can exi s t as t he pri mary feat ure of an i di opat hi c condi t i on or as a s econdary mani fes t at i on of i nfect i ous , mal i gnant , or rheumat i c di s eas e.
B. Etiology Mos t of t he vas cul i t i c s yndromes are i di opat hi c. Hepat i t i s B s urface ant i gen (HBs Ag) and HCV have been found i n i mmune compl exes of s everal di fferent s yndromes , i ncl udi ng s ome cas es of pol yart eri t i s nodos a and es s ent i al mi xed cryogl obul i nemi a. HIV, parvovi rus , and cyt omegal ovi rus (CMV) have been as s oci at ed wi t h vas cul i t i c di s eas es , part i cul arl y pol yart eri t i s .
C. Pathogenesis Indi vi dual s yndromes oft en demons t rat e feat ures of di s ordered humoral and cel l ul ar i mmuni t y.
1. Disordered humoral immune response. In s ome vas cul i t i c s yndromes (e.g., l eukocyt ocl as t i c vas cul i t i s perhaps pol yart eri t i s nodos a), s ol ubl e i mmune compl exes form and depos i t i n ves s el wal l s , fi x compl ement , and at t ract i nfl ammat ory cel l s , whi ch caus e damage. Ant i bodi es may al s o react di rect l y wi t h neut rophi l s or endot hel i al cel l s t o caus e ves s el damage (e.g., W egener's granul omat os i s and ANCAs ). The s i t e and degree of damage are det ermi ned by many vari abl es , i ncl udi ng: o
o
a. Phys i cal and bi ochemi cal charact eri s t i cs of t he i mmune compl exes
o
o
b. Vari at i ons i n bl ood pres s ure (hydros t at i c forces )
o
Pa g e 2 5 0 0
ABC Ambe r CHM Conve rte r Tria l ve rsion, http://w w w .proce sste x t.com/a bcchm.html o
c. Changes i n bl ood ves s el s i ze and permeabi l i t y
o
o
d. Endot hel i al cel l adhes i ve propert i es
o
o
e. Effect i venes s of ret i cul oendot hel i al s ys t em removal of i mmune compl exes
o
o
f. Degree of t urbul ence i n ves s el s (e.g., i mmune compl exes depos i t at branch poi nt s )
o
o
g. Degree of pers i s t ence or recurrence of ant i gen expos ure (e.g., t oxi n vers us HBV)
o
o
h. Vari at i ons i n hos t i mmune res pons e (e.g., ANCA i n pat i ent s wi t h W egener's granul omat os i s ). An i nhal ed ant i gen or ot her i mmune s ys t em chal l enge may act i vat e neut rophi l s , whi ch i n t he pres ence of ANCA, degranul at e, recrui t T cel l s and monocyt es , and damage ves s el wal l s .
2. Disordered cellular immune response. Di s orders i nvol vi ng granul oma format i on are t he bes t exampl es of t he cont ri but i ons of t he cel l ul ar i mmune res pons e t o vas cul i t i c proces s es . Int eract i ons bet ween macrophages +
and CD4 T cel l s i n res pons e t o unknown ant i gens l ead t o granul oma format i on i n and around ves s el wal l s . In gi ant cel l art eri t i s , age-rel at ed ves s el degenerat i on i n l arge art eri es may expos e ant i gens t ypi cal l y hi dden t o t he i mmune s ys t em, whi ch t hen el i ci t a cel l ul ar i mmune res pons e; t here al s o may be an HLA-DR4 as s oci at i on wi t h t hi s di s eas e and prol i ferat i on of s peci fi c T-cel l
Pa g e 2 5 0 1
ABC Ambe r CHM Conve rte r Tria l ve rsion, http://w w w .proce sste x t.com/a bcchm.html
cl ones i n ves s el l es i ons .
D. Classification The current s ys t ems for cl as s i fyi ng vas cul i t i c s yndromes us e cl i ni cal feat ures , s i ze of t he i nvol ved ves s el , and t ype of cel l ul ar i nfi l t rat e t o s eparat e t he s yndromes (Tabl e 10-14). P.497
TABLE 10-14 Major Vasculitic Syndromes: Clinicopathologic Distinctions P at h ol o g Cl
y
in
c
ic
el
S al
lu
y F v la Di n e e r a d at ss in g r u el fil n o r si tr o m e z at si e s e e s H S S L S y ki m e ki p n al u n e in l
k bi
Pa g e 2 5 0 2
ABC Ambe r CHM Conve rte r Tria l ve rsion, http://w w w .proce sste x t.com/a bcchm.html
rs v (c oc o e ol a yt p n v pi oc s y si e l l l a ti m ar s t vi e i e i c ty nt s ; v v pr v a a e e sc sc d n ul ul o ul i t i iti m e s s i n s ; wi (s a ar t h er nt t e al u Vi ri l m s c ol l e s i er e s i ck al s ) o n in
n
e v
s
s s ol
at
; v
th
dr e
e
u m
s
g e
a
re nt
m
ac t y
e
t i pi
d
o ca
e
n ll
v
s; y
el
H m
o
e in
p
n i
m
Pa g e 2 5 0 3
ABC Ambe r CHM Conve rte r Tria l ve rsion, http://w w w .proce sste x t.com/a bcchm.html
oc m
e
h- al
nt
Sc a
al
h n
st
à d
a
¶ s
g
nl el
e
ei fn li p m ur i t p e ur d a; m ix e d e ss e nt ia l cr y o gl o b ul in e m ia
Pa g e 2 5 0 4
ABC Ambe r CHM Conve rte r Tria l ve rsion, http://w w w .proce sste x t.com/a bcchm.html
) P M S N Bi C ol ul m ec o or y t i al ro p t i a syl
t i s y co
rt s t a zi of s t e e n n i n er ri m d g v oi ti i l l m ar ol d s n e te v s; n e di ri e cy o ss u ti d to d M m s or t o o aj -s at g xi s or i z v a c a ar e e n a t e d s s Vi g ri ar el s c e al t e br er nt i s ri a al s ch e nc a a e s h n d m
p gi d
ic
oi o e
le
nt gr d
si
s a if
o
Si p n
n
mh o
s
ul y re
th
ta
s
at
n
p
s
e
o
p
o
n
ar
u
s
e
s
e
Pa g e 2 5 0 5
ABC Ambe r CHM Conve rte r Tria l ve rsion, http://w w w .proce sste x t.com/a bcchm.html
lu
le
n
si
g
o
s
n
a
s
n
at
d
v
s
ar
pl
io
e
u
e
s
n
st
N
a
o
g
d
e
o
s
s e sk in le si o n s u nc o m m o n Al Pr V Gr Bi le o ar a o r m yi n p
Pa g e 2 5 0 6
ABC Ambe r CHM Conve rte r Tria l ve rsion, http://w w w .proce sste x t.com/a bcchm.html
gi i n n ul s y c e g o of a nt (c m i n n al a at v gi l e pi o ol iti rg l l u v s i c ar s e (C , i e i n d h a s ; fi l or ur s t v t r g g- h e at a St m n e n, ra at ul s t y u i c e wi pi s s hi s ; t h ca di s t s e l l s or m o y e y al s i l u a L l
n n
s u ar o g e) n t e p g ri hi in e ls v s) ol v e m e nt Bl o o d e
Pa g e 2 5 0 7
ABC Ambe r CHM Conve rte r Tria l ve rsion, http://w w w .proce sste x t.com/a bcchm.html
o si n o p hi li a co m m o n WU S N L e p m ec u g p al ro n e er l
ti g
n a (a zi bi e n rt n o r' d er g p s l o i e gr s y g w s ; a (s r er v n i n a re ei ul u n s n o s ul pi s ) m bi o ra
a o
m to
s p
at ry
in sy
o tr
u u
si ac
p s
s t
p u
in
er al
v
a ly
ol
n n
Pa g e 2 5 0 8
ABC Ambe r CHM Conve rte r Tria l ve rsion, http://w w w .proce sste x t.com/a bcchm.html
v
d o
e
lo n
m
w di
e
er a
nt
ai g
Pr
r n
o
w o
m
a st
in
ys i c
e
F )
nt
oc c-
re
al A
n
n N
al
ec C
a
ro A
b
ti
n
zi
or
n
m
g
al
ar
iti
te
e
ri
s
ti s in lu n g s N ec ro ti zi n
Pa g e 2 5 0 9
ABC Ambe r CHM Conve rte r Tria l ve rsion, http://w w w .proce sste x t.com/a bcchm.html
g gl o m er ul o n e p hr iti s in ki d n e ys Gi P L Gi T a ol ar a e nt y g nt m c m e ce p el y ar l l or l al t e a al a gi ri n ar rt a e d t e e rh s ch ry ri e (e ro bi ti u s ni o s mp c p (t at ec m s y e ic ia o m a ll n p in y o
Pa g e 2 5 1 0
ABC Ambe r CHM Conve rte r Tria l ve rsion, http://w w w .proce sste x t.com/a bcchm.html
o 5 te n r 0 m uc al % p l e a O or ar rt cc al i n e ur ar fi l ri s t e t r ti i n ry at s) p ) e at
s
ie
in
nt
v
s
e
ol
ss
d
el
er
w
th
al
a
ls
n 5 0 Si g n s a n d sy m pt o m s of
Pa g e 2 5 1 1
ABC Ambe r CHM Conve rte r Tria l ve rsion, http://w w w .proce sste x t.com/a bcchm.html
cr a ni al ar te ry in v ol v e m e nt Hi g h er yt hr oc yt e s e di m e nt at io n ra te
Pa g e 2 5 1 2
ABC Ambe r CHM Conve rte r Tria l ve rsion, http://w w w .proce sste x t.com/a bcchm.html
T L L Gi A a ar ar a n k g g nt gi a e e ce o y v ar l l gr a e te a a s s s ri n p u' el e d h s cl s ch y a a (a ro C rt u or ni h e di t i c e ri ca c m s t ti t i ar o M s o ch n RI n ) o N O
n ec
cc
uc k
ur
l e ul
s
ar t r
in
in a
y
fi l s
o
tr o
u
at n
n
e o
g
s gr
(o
in a
ft
v p
e
e h
n
ss y
A
el
si
w
a
al
n)
ls
w
Pa g e 2 5 1 3
ABC Ambe r CHM Conve rte r Tria l ve rsion, http://w w w .proce sste x t.com/a bcchm.html
o m e n ANCA, ant i neut rophi l cyt opl as mi c ant i body; c-ANCA, cyt opl as mi c ANCA; MRI, magnet i c res onance i magi ng. P.498
1. Pat hol ogi c cl as s i fi cat i on by ves s el s i ze may be t he mos t cl i ni cal l y us eful : o
o
a. Large ves s el :
(1) Gi ant cel l (t emporal ) art eri t i s (GCA)
(2) Takayas u's art eri t i s
o
o
b. Medi um ves s el :
(1) Pol yart eri t i s nodos a (PAN)
Pa g e 2 5 1 4
ABC Ambe r CHM Conve rte r Tria l ve rsion, http://w w w .proce sste x t.com/a bcchm.html
(2) Kawas aki 's di s eas e
o
o
c. Smal l ves s el :
(1) Immune-compl ex medi at ed
(a) Hypers ens i t i vi t y vas cul i t i s
(b) Serum s i cknes s
(c) Mi xed es s ent i al cryogl obul i nemi a
(d) Henoch-Schönl ei n purpura (HSP)
(e) Urt i cari al vas cul i t i s
(2) ANCA-as s oci at ed (para-i mmune)
(a) W egener's granul omat os i s
(b) Churg-St raus s
(c) Mi cros copi c pol yangi i t i s (MPA)
2. Nons peci fi c hemat ol ogi c and i mmunol ogi c abnormal i t i es (e.g., anemi a, l eukocyt os i s , el evat ed ESR,
Pa g e 2 5 1 5
ABC Ambe r CHM Conve rte r Tria l ve rsion, http://w w w .proce sste x t.com/a bcchm.html
and hypocompl ement emi a) oft en exi s t i n many of t hes e s yndromes , but t hes e are not not ed i n Tabl e 10-14 unl es s t hey are s peci fi cal l y i mport ant for t he di agnos i s of a part i cul ar s yndrome. It i s i mport ant t o unders t and t hat t hes e s yndromes oft en bl ur and overl ap i n act ual pat i ent s .
E. Vasculitic syndromes
1. Hypersensitivity vasculitis (
onl i ne Fi gures 10-14 and 10-15
ONLINE FIGURE 10-14 Vas cul i t i s . Di gi t al i nfarct s can be caus ed by i nfl ammat i on and s ubs equent occl us i on of t he s mal l art eri es . Thi s can be caus ed by any s mal l ves s el vas cul i t i s . (ACR Sl i de Col l ect i on of t he Rheumat i c Di s eas es , 3rd ed., 2004:9105220.)
Pa g e 2 5 1 6
ABC Ambe r CHM Conve rte r Tria l ve rsion, http://w w w .proce sste x t.com/a bcchm.html
ONLINE FIGURE 10-15 Vas cul i t i s . Pal pabl e purpura, as i s s een here, can be pres ent i n s mal l ves s el vas cul i t i s from any caus e. (ACR Sl i de Col l ect i on of t he Rheumat i c Di s eas es , 3rd ed., 2004:9112120.) o
o
a. T ypical hypersensitivity vasculitis syndrome. Hypers ens i t i vi t y vas cul i t i s (al s o cal l ed leukocytoclastic vasculitis or cutaneous vasculitis ) i s i mmune compl ex–medi at ed i nfl ammat i on of s mal l ves s el s (art eri ol es , capi l l ari es , and venul es ). The caus e oft en i s uncl ear. In many cas es , hypers ens i t i vi t y vas cul i t i s s eems t o occur as an exaggerat ed i mmune res pons e t o a drug (e.g., peni ci l l i n) or infection (vi ral or bact eri al ant i gen), caus i ng a s el f-l i mi t ed i mmune compl ex vas cul i t i s . The s ki n al mos t al ways i s i nvol ved, wi t h pal pabl e purpura, urt i cari a, or ul cers . Pol yart i cul ar art hri t i s i s common. Di s t i nct organ i nvol vement may al l ow furt her cl as s i fi cat i on.
o
Pa g e 2 5 1 7
ABC Ambe r CHM Conve rte r Tria l ve rsion, http://w w w .proce sste x t.com/a bcchm.html o
b. Distinctive subsets
(1) Henoch-Schönlein purpura i s a s ys t emi c, s mal l ves s el vas cul i t i s t hat occurs pri mari l y i n chi l dren, oft en fol l owi ng a s t rept ococcal pharyngi t i s .
(a) Clinical features. Pal pabl e purpura, pol yart i cul ar art hral gi as or i nfl ammat ory art hri t i s , abdomi nal pai n or gas t roi nt es t i nal bl eedi ng due t o mes ent eri c ves s el i nvol vement , and i mmune compl ex–medi at ed gl omerul onephri t i s t ypi cal l y occur. A di s t i nct i ve feat ure i s IgA deposition i n t he vas cul ar l es i ons .
(b) Course. Mos t cas es of Henoch-Schönl ei n purpura res ol ve s pont aneous l y over a peri od of days t o weeks . Pat i ent s wi t h s evere renal or gas t roi nt es t i nal i nvol vement may requi re s t eroi ds .
(2) Serum sickness i s an i mmune compl ex vas cul i t i s t hat oft en fol l ows a drug (e.g., peni ci l l i n or s ul fonami de) or forei gn prot ei n expos ure by 7–10 days . Ant i l ymphocyt e gl obul i n now i s one of t he mos t common caus es . HBV art hri t i s –dermat i t i s s yndrome i s a s erum s i cknes s –l i ke res pons e t o Hbs Ag.
Pa g e 2 5 1 8
ABC Ambe r CHM Conve rte r Tria l ve rsion, http://w w w .proce sste x t.com/a bcchm.html
(a) Clinical features. Urt i cari a, purpura, art hri t i s , and art hral gi as are charact eri s t i c, and l ymphadenopat hy and i mmune compl ex gl omerul onephri t i s are common.
(b) Course. Res ol ut i on i s t ypi cal fol l owi ng removal of t he i nci t i ng agent .
(2) Hypocomplementemic urticarial vasculitis
(a) Clinical features. Pat i ent s wi t h t hi s form of vas cul i t i s exhi bi t urt i cari a, t ypi cal l y l as t i ng l onger t han 24 hours and, s omet i mes , art hri t i s , gl omerul onephri t i s , or gas t roi nt es t i nal i nvol vement . Hypocomplementemia i s a cons i s t ent feat ure, apparent l y rel at ed t o ant i bodi es di rect ed agai ns t C1q; t he degree of compl ement depres s i on t ypi cal l y paral l el s di s eas e act i vi t y. P.499
(b) Course. The di s eas e fol l ows a rel aps i ng, remi t t i ng cours e and may requi re cort i cos t eroi ds for t he more di s abl i ng mani fes t at i ons .
(4) Mixed essential cryoglobulinemia i s an
Pa g e 2 5 1 9
ABC Ambe r CHM Conve rte r Tria l ve rsion, http://w w w .proce sste x t.com/a bcchm.html
i di opat hi c di s order caus ed by col d-preci pi t at ed i mmunogl obul i n (cryogl obul i n) compl exes , whi ch produce a vas o-occl us i ve or i nfl ammat ory i njury. The s yndrome t ypi cal l y occurs i n pat i ent s who have t ype II (mi xed) cryogl obul i n. Many pat i ent s have been found t o have HCV or l es s oft en, HBV, i n t he i mmune compl exes .
(a) Clinical features. Recurrent at t acks of pal pabl e purpura, Raynaud's phenomenon, art hral gi a, and i mmune compl ex gl omerul onephri t i s may occur. Gas t roi nt es t i nal , hepat i c, and pul monary dys funct i on are occas i onal feat ures .
(b) Course. Severe renal i nvol vement i s t he mos t common reas on for t reat ment , whi ch may i ncl ude cort i cos t eroi ds , cyt ot oxi c agent s , or removal of t he cryo gl obul i n by apheres i s . IFN-α may hel p when t he di s eas e i s as s oci at ed wi t h HBV or HCV.
2. Polyarteritis nodosa i s a necrot i zi ng vas cul i t i s of s mal l and medi um-s i zed art eri es , whi ch mani fes t s as a mul t i s ys t em i l l nes s . Is chemi c art eri al l es i ons are caus ed by ves s el wal l i nfl ammat i on, i n s ome cas es as s oci at ed wi t h i mmune compl ex depos i t i on. HBs Ag i s pos i t i ve i n up t o 50% of pat i ent s . o
o
a. Clinical features. Constitutional complaints (e.g., fever, wei ght l os s , and anorexi a) are
Pa g e 2 5 2 0
ABC Ambe r CHM Conve rte r Tria l ve rsion, http://w w w .proce sste x t.com/a bcchm.html
common, and multiorgan ischemic dysfunction i s t he rul e. The l ungs and s pl een t ypi cal l y are s pared.
(1) Renal involvement i s mos t common and mani fes t s as s evere hypert ens i on, prot ei nuri a and act i ve s edi ment from gl omerul onephri t i s , or renal i ns uffi ci ency.
(2) Other common features i ncl ude art hral gi as , myal gi as , peri pheral nervous s ys t em abnormal i t i es (s ens ory pol yneuropat hi es , mononeuri t i s mul t i pl ex), and s ki n l es i ons (i nfarct i ons , nodos e l es i ons ). Ins i di ous CHF, di ffus e or focal CNS dys funct i on, and gas t roi nt es t i nal i nvol vement al s o can occur, dependi ng on t he s i ze of t he i s chemi c l es i ons .
(3) Microscopic polyarteritis i s a cl i ni cal vari ant t ypi fi ed by promi nent l ung i nvol vement and s egment al gl omerul onephri t i s , s omet i mes rapi dl y progres s i ve. The ves s el s i nvol ved t end t o be s mal l er (capi l l ari es , art eri ol es , and s mal l art eri es ). p-ANCA i s commonl y found (50%–80% of pat i ent s ) and abdomi nal angi ography i s t ypi cal l y negat i ve (becaus e t he s mal l er ves s el s are i nvol ved).
o
o
b. Course. Treat ment wi t h cort i cos t eroi ds , cyt ot oxi c agent s , or bot h i s neces s ary; t he i l l nes s t ypi cal l y i s fat al i f unt reat ed. IFN-α may hel p i n HBV-as s oci at ed cas es .
Pa g e 2 5 2 1
ABC Ambe r CHM Conve rte r Tria l ve rsion, http://w w w .proce sste x t.com/a bcchm.html
3. Allergic angiitis (Churg-Strauss disease) i s a granul omat ous vas cul i t i s t hat t ypi cal l y occurs i n pat i ent s wi t h as t hma. o
o
a. Clinical features. Al l ergi c angi i t i s i s t ypi cal l y a t ri phas i c di s eas e. Asthma occurs fi rs t , oft en as s oci at ed wi t h blood eosinophilia. T issue infiltrates, es peci al l y pulmonary ones , occur next ; eos i nophi l s are oft en pres ent i n t hes e l es i ons , s omet i mes i n granul omas . Vasculitis of s mal l or medi um-s i zed ves s el s occurs l as t , commonl y i nvol vi ng s ki n, nerve, or mus cl e l es i ons .
o
o
b. Course. The fart her apart i n t i me t he t hree phas es of i l l nes s occur, t he mi l der t he di s eas e; t he cl os er t oget her, t he more expl os i ve. Cat as t rophi c l ung, gas t roi nt es t i nal t ract , cardi ac, or nerve l es i ons may mandat e aggres s i ve i nt ravenous cort i cos t eroi d and cyt ot oxi c t herapy, but more i ndol ent forms are oft en eas i l y cont rol l ed wi t h moderat e cort i cos t eroi d dos es .
4. Wegener' s granulomatosis i s a granul omat ous , s mal l ves s el vas cul i t i s t hat t ypi cal l y i nvol ves t he upper and l ower res pi rat ory t ract and ki dney. o
o
a. Clinical features
(1) Respiratory tract. Pul monary feat ures t ypi cal l y i ncl ude upper ai rway compl ai nt s (e.g., s i nus i t i s , rhi ni t i s ) as wel l as l ower
Pa g e 2 5 2 2
ABC Ambe r CHM Conve rte r Tria l ve rsion, http://w w w .proce sste x t.com/a bcchm.html
ai rway s ympt oms (e.g., cough, s hort nes s of breat h, hemopt ys i s ). Cavi t ary or mul t i pl e i nfi l t rat es may be s een on ches t radi ography.
(2) Kidney. Renal i nvol vement i s charact eri zed by abnormal s edi ment or renal funct i onal i mpai rment .
(3) Organs. Ot her, l es s cl as s i cal feat ures i ncl ude s ki n l es i ons (pal pabl e purpura), art hri t i s , ocul ar or orbi t al i nfl ammat i on (t ypi cal l y wi t h propt os i s ), cardi ac l es i ons , and CNS or peri pheral nervous s ys t em i nvol vement . P.500
(4) Presence of c-ANCA. c-ANCA i s oft en found i n pat i ent s wi t h act i ve, mul t i organ di s eas e. In t hes e pat i ent s , t he pres ence of c-ANCA may be a rel at i vel y s peci fi c di agnos t i c t es t and may hel p moni t or di s eas e act i vi t y.
o
o
b. Course. Al t hough cort i cos t eroi ds may be us ed i ni t i al l y, cyt ot oxi c agent s s uch as cycl ophos phami de are uni forml y requi red t o prevent deat h. Met hot rexat e may pl ay a rol e i n mai nt ai ni ng di s eas e remi s s i on or i n l i mi t ed forms of t he di s eas e.
5. Giant cell (temporal) arteritis us ual l y i s a
Pa g e 2 5 2 3
ABC Ambe r CHM Conve rte r Tria l ve rsion, http://w w w .proce sste x t.com/a bcchm.html
granul omat ous i nfl ammat i on of t he carot i d art ery and i t s branches , but i t s omet i mes can i nvol ve t he vert ebral art ery or ot her aort i c branches . The pat hol ogy i s i ndi s t i ngui s habl e from t hat of Takayas u's art eri t i s . o
o
a. Clinical features. The di s eas e rarel y occurs i n pat i ent s younger t han age 50.
(1) Approxi mat el y 50% of pat i ent s have PMR (i .e., achi ng and s t i ffnes s of s houl der and hi p gi rdl es as s oci at ed wi t h an ESR >50 mm/hour, age >50 years , and cons t i t ut i onal compl ai nt s s uch as fever, mal ai s e, and wei ght l os s ), and mos t pat i ent s have feat ures rel at ed t o i s chemi a i n t he carot i d art ery regi on (i .e., headache, vi s ual s ympt oms , jaw cl audi cat i on, s cal p t endernes s , and neurol ogi c compl ai nt s ).
(2) Superficial temporal artery involvement i s common but oft en i s cl i ni cal l y s i l ent . Pot ent i al cl i ni cal feat ures i ncl ude t endernes s , nodul es , or eryt hema. Even when t he art ery i s cl i ni cal l y normal , t emporal art ery bi ops y t ypi cal l y reveal s pat hol ogy.
o
o
b. Course. Pat i ent s requi re s t eroi ds earl y and i n hi gh dos es t o prevent blindness, t he mos t s eri ous compl i cat i on of t hi s i l l nes s . Pat i ent s s houl d be s t art ed i mmedi at el y on at l eas t 1 mg/kg predni s one t o prevent bl i ndnes s . A t emporal art ery bi ops y may s t i l l be di agnos t i c up t o 1 week aft er t he i ni t i at i on of s t eroi ds . A cont ral at eral t emporal art ery i s neces s ary for di agnos i s i f t he ori gi nal
Pa g e 2 5 2 4
ABC Ambe r CHM Conve rte r Tria l ve rsion, http://w w w .proce sste x t.com/a bcchm.html
bi ops y s peci men i s negat i ve.
6. T akayasu' s arteritis i s a granul omat ous i nfl ammat i on of t he ves s el s of t he aort i c arch t hat i s charact eri zed pat hol ogi cal l y by a panart eri t i s cont ai ni ng mononucl ear and gi ant cel l s . Y oung, oft en As i an, women are at hi ghes t ri s k for t hi s di s order. o
o
a. Clinical features
(1) Generalized aching s i mi l ar t o PMR oft en occurs earl y i n t he i l l nes s . Feat ures of large artery ischemia devel op weeks t o mont hs l at er and may i ncl ude upper ext remi t y cl audi cat i on, angi na, and CHF from cardi ac or aort i c i nvol vement . Pul monary and mes ent eri c ves s el s al s o can be i nvol ved.
(2) Findings on physical examination i ncl ude art eri al brui t s , pul s e defi ci t s , and bl ood pres s ure di fferences bet ween ext remi t i es .
o
o
b. Course. Pat i ent s may requi re s t eroi ds earl y i n t he cours e of t he i l l nes s ; l at er, vas cul ar recons t ruct i on may be requi red for occl us i ve l es i ons . Cyt ot oxi c agent s are t ypi cal l y added i f s t eroi ds do not cont rol i nfl ammat ory di s eas e.
7. Other vasculitic syndromes do not fi t i nt o t he major cat egori es of vas cul i t i s due t o di s t i nct cl i ni cal or overl appi ng pat hol ogi c feat ures .
Pa g e 2 5 2 5
ABC Ambe r CHM Conve rte r Tria l ve rsion, http://w w w .proce sste x t.com/a bcchm.html o
o
a. Vasculitis as a secondary feature of a primary disease. Cert ai n di s eas es can exhi bi t vas cul i t i c i nfl ammat i on as a s econdary feat ure. The vas cul i t i s t ypi cal l y i s s mal l ves s el , cut aneous vas cul i t i s but s omet i mes can overl ap wi t h pol yart eri t i s -l i ke ves s el i nvol vement , es peci al l y i n rheumatoid arthritis and lupus. Dermal and CNS vas cul i t i c l es i ons are recent l y recogni zed s econdary feat ures of Sjögren' s syndrome. Hematologic malignancies and bact eri al and vi ral infections al s o can i ncl ude vas cul i t i c feat ures , us ual l y wi t h t he s ki n t he predomi nant organ i nvol ved.
o
o
b. Behçet' s syndrome. The pres ence of recurrent oral and geni t al ul cers defi nes t hi s s yndrome. Pat i ent s al s o may have eye i nfl ammat i on, pathergic skin lesions (l es i ons occurri ng at s i t es of s ki n i njury), and vas cul i t i s of t he CNS or ot her organs .
o
o
c. Kawasaki disease (mucocutaneous lymph node syndrome). Thi s febri l e i l l nes s of i nfant s and young chi l dren i s charact eri zed by conjunct i val i nject i on; di ffus e macul opapul ar ras h wi t h as s oci at ed edema, eryt hema, and event ual des quamat i on of t he hands and feet ; cracked l i ps ; “s t rawberry t ongue―; and cervi cal adenopat hy. Coronary vas cul i t i s can devel op i n 25% of pat i ent s and l ead t o aneurys m, MI, and s udden deat h. However, t he i nci dence of t hi s vas cul i t i s has great l y decreas ed s i nce t he advent of i nt ravenous i mmunogl obul i n t herapy, whi ch,
Pa g e 2 5 2 6
ABC Ambe r CHM Conve rte r Tria l ve rsion, http://w w w .proce sste x t.com/a bcchm.html
al ong wi t h as pi ri n, i s t he t reat ment of choi ce i n t hi s di s eas e. P.501
o
o
d. Isolated CNS vasculitis. Thi s condi t i on i s a granul omat ous i nfl ammat i on of s mal l or medi um-s i zed art eri es of t he brai n. Typi cal l y, pat i ent s do not have cl i ni cal or l aborat ory evi dence of i nfl ammat i on el s ewhere. Pres ent i ng feat ures oft en are combi nat i ons of di ffus e CNS compl ai nt s (headache, al t ered ment al s t at us , poor memory) and more focal ones (crani al nerve defect s , hemi pares i s ). A chroni c meni ngi t i s pi ct ure, wi t h pl eocyt os i s and i ncreas ed prot ei n, t ypi cal l y i s found on LP. An enhanced MRI s can of t he brai n i s a s ens i t i ve but nons peci fi c i ndi cat i on of pos s i bl e i nvol vement .
F. Diagnosis
1. Why suspect an underlying vasculitic process? Combi nat i ons of cl i ni cal feat ures s ugges t t he pos s i bi l i t y of a vas cul i t i s . o
o
a. Evi dence of a mul t i s ys t em di s eas e
o
o
b. Infect i ous et i ol ogi es t horoughl y excl uded
o
o
c. Laborat ory dat a may s ugges t an i nfl ammat ory
Pa g e 2 5 2 7
ABC Ambe r CHM Conve rte r Tria l ve rsion, http://w w w .proce sste x t.com/a bcchm.html
proces s o
o
d. Di s t i nct i ve cl i ni cal feat ures or pat t erns (Tabl e 10-15)
TABLE 10-15 Clinical Data Suggesting Vasculitis P h L ys a ic b al or E at x or a y m St Hi in u st at di or io e y n s C V N o e o n ss n st el s it fi p ut n e io di ci n n fi al g c c s te o Bl st mo s pl o (c
Pa g e 2 5 2 8
ABC Ambe r CHM Conve rte r Tria l ve rsion, http://w w w .proce sste x t.com/a bcchm.html
ai d hr nt pr o s es ni M suc al re i n ai /p fl s e ul a A se m n d m or ef at ex i ci i o i a t s n) Fe H A ve yp n r er e W te m ei ns i a g i o (c ht n hr l o (s o s s u ni D d c ru d di g e se e n as x o e) p ns T o et hr s ) o ur V m e es b (p s e oc re l
yt
s c t e os ri n i s
Pa g e 2 5 2 9
ABC Ambe r CHM Conve rte r Tria l ve rsion, http://w w w .proce sste x t.com/a bcchm.html
b d In e er cr d n e or es as ill s, e i ci n d t ) o er In d yt fe ul hr ct es oc io ,
yt
n br e H ui s e u t s di m S m a ki e n n nt i
fi at
m n io m di n u n ra n g te o s In d P cr ef al e i ci p as e a e nc bl d y e g vi p a ru ur m s p m (H ur a IV a gl ) In o
Pa g e 2 5 3 0
ABC Ambe r CHM Conve rte r Tria l ve rsion, http://w w w .proce sste x t.com/a bcchm.html
H fa b e rc ul p ti in at o s i t i ns T s Ul e B ce st or rs in C M di vi u c ru sc at s le in O fi g rg n p a di ot n n e is g nt c s ia h T l e e is mn c ic d h c er e o n m m es ia pl s , A ai cr b nt a n s m or A pi m n n al gi g re n N n a er al Ex v fu
Pa g e 2 5 3 1
ABC Ambe r CHM Conve rte r Tria l ve rsion, http://w w w .proce sste x t.com/a bcchm.html
t r e nc e fi t i m n o i t di n y n (â or g †‘ ja s cr w S e cl e at a ns i n u or i n di y/ e, ca m B t i ot U o or N) n n A Vi e b s u ur n al o or co p m m at al pl hi m ai es us nt Fo cl s ca e/ Tr l / l i a di ve ns ff r i e us e nt e nz is C y ch N m e S es m dy (â i c s f †‘
Pa g e 2 5 3 2
ABC Ambe r CHM Conve rte r Tria l ve rsion, http://w w w .proce sste x t.com/a bcchm.html
at u C t a nc K, ck t i A s o S St n T, ro T A ke e LT st ) is El T ec e tr n oc d ar er di n o es gr s a m T e st s p oi nt in g to w ar d s p e ci
Pa g e 2 5 3 3
ABC Ambe r CHM Conve rte r Tria l ve rsion, http://w w w .proce sste x t.com/a bcchm.html
fi c di a g n o s e s HI V (s ev er al sy n dr o m es ) H B s A g (2 0 % â €“ 3 0
Pa g e 2 5 3 4
ABC Ambe r CHM Conve rte r Tria l ve rsion, http://w w w .proce sste x t.com/a bcchm.html
% of p ol ya rt er iti s p at ie nt s) H e p at iti s C R N A A N A (S LE m or e li ke ly
Pa g e 2 5 3 5
ABC Ambe r CHM Conve rte r Tria l ve rsion, http://w w w .proce sste x t.com/a bcchm.html
) cA N C A ( W e g e n er 's gr a n ul o m at os is ) pA N C A ( m ic ro sc o
Pa g e 2 5 3 6
ABC Ambe r CHM Conve rte r Tria l ve rsion, http://w w w .proce sste x t.com/a bcchm.html
pi c p ol ya rt er iti s) Cr yo gl o b ul in s ( m ix e d es se nt ia l cr yo gl o b ul in e
Pa g e 2 5 3 7
ABC Ambe r CHM Conve rte r Tria l ve rsion, http://w w w .proce sste x t.com/a bcchm.html
m ia ) A nt iR o (S jà ¶ gr e n' s sy n dr o m e, SL E) ALT, al ani n e ami not rans fer as e; ANCA, ant i ne ut rophi l cyt opl as mi c
Pa g e 2 5 3 8
ABC Ambe r CHM Conve rte r Tria l ve rsion, http://w w w .proce sste x t.com/a bcchm.html
ant i bo dy; ANA, ant i nu cl ear ant i bo dy; AST, as part at e ami not rans fer as e; BUN, bl ood urea ni t roge n; CK, creat i n e ki nas e ; CNS, cent ral nervou s s ys t em ; SLE, s ys t em ic l upus eryt he mat os us
Pa g e 2 5 3 9
ABC Ambe r CHM Conve rte r Tria l ve rsion, http://w w w .proce sste x t.com/a bcchm.html
2. Confirming the diagnosis. If t he cl i ni cal eval uat i on s ugges t s a reas onabl e chance of vas cul i t i s , t he appropri at e approach t o maki ng a di agnos i s us ual l y l i es i n bi ops y of i nvol ved t i s s ue or vi s ceral angi ography; ANCA t es t i ng may be hel pful when mi cros copi c pol yart eri t i s or W egener's granul omat os i s are s us pect ed. o
o
a. Biopsy of a fresh lesion. Bi ops y from cl i ni cal l y i nvol ved t i s s ues i s t he preferred met hod of di agnos i ng vas cul i t i s ; i f i nvol ved, t he s ki n oft en i s t he eas i es t t i s s ue t o obt ai n.
(1) Di agnos i s of Wegener' s granulomatosis oft en rel i es on open l ung bi ops y demons t rat i on of granul omat ous vas cul i t i s , becaus e paranas al s i nus t i s s ue t ypi cal l y s hows nons peci fi c i nfl ammat i on and renal t i s s ue s hows onl y gl omerul onephri t i s .
(2) Di agnos i s of temporal arteritis requi res exami nat i on of a 3- t o 6-cm s egment of t he s uperfi ci al t emporal art ery i n mul t i pl e s ect i ons ; bi ops y of t he cont ral at eral s i de may be needed i f t he fi rs t bi ops y i s negat i ve and cl i ni cal s us pi ci on remai ns hi gh. P.502
(3) Bi ops y of l ept omeni nges and cl i ni cal l y i nvol ved brai n t i s s ue may be needed when isolated CNS vasculitis i s s t rongl y s us pect ed.
o
Pa g e 2 5 4 0
ABC Ambe r CHM Conve rte r Tria l ve rsion, http://w w w .proce sste x t.com/a bcchm.html
o
b. Visceral angiography. W hen pol yart eri t i s i s s us pect ed and appropri at e t i s s ue cannot be obt ai ned for bi ops y or when t he bi ops y i s unreveal i ng, abdomi nal t hree-ves s el angi ography may s how aneurys ms or art eri opat hy (di s cret e narrowi ng) s ugges t i ve of s mal l t o medi um-s i zed art ery vas cul i t i s . Aort i c arch and l arge ves s el angi ograms , carot i d ul t ras onography, and MRI s t udi es of t he ches t are us eful i n Takayas u's art eri t i s and gi ant cel l art eri t i s , becaus e t he i nvol ved ves s el s oft en cannot be s afel y s ubject ed t o bi ops y. CNS angi ograms may be warrant ed when i s ol at ed CNS vas cul i t i s i s s us pect ed.
o
o
c. Presence of ANCA. ANCA are t ypi cal l y IgG ant i bodi es di rect ed agai ns t cyt opl as mi c component s of neut rophi l s and monocyt es (s ee onl i ne I B 5).
3. Differential diagnosis. Numerous condi t i ons s houl d be cons i dered i n t he di fferent i al di agnos i s of vas cul i t i s . Import ant di s orders and t he t es t s us ed t o el i mi nat e t hem i ncl ude: o
o
a. Bacterial endocarditis (bl ood cul t ure)
o
o
b. Left atrial myxoma (t wo-di mens i onal or t rans es ophageal echocardi ogram)
o
o
c. Cholesterol embolism s yndrome (bi ops y s howi ng refract i l e crys t al s )
Pa g e 2 5 4 1
ABC Ambe r CHM Conve rte r Tria l ve rsion, http://w w w .proce sste x t.com/a bcchm.html o
d. T hrombotic diseases s uch as antiphospholipid
o
syndrome (ant i phos phol i pi d ant i body) and disseminated intravascular coagulation (DIC) or thrombotic thrombocytopenic purpura (TTP) [PT, PTT, pl at el et count , fi bri nogen, and fi bri n s pl i t product s ]
G. Therapy The management of pat i ent s wi t h vas cul i t i s i s compl ex. In al l pat i ent s wi t h known vas cul i t i c di s eas e, i t i s i mport ant t o record t he ext ent of di s eas e i ni t i al l y and t o moni t or organ i nvol vement (i n t erms of cl i ni cal , bi ops y, and angi ographi c evi dence). The t empo of di s eas e progres s i on al s o s houl d be gauged t o det ermi ne t he bes t t reat ment .
1. Antigen removal. Drugs t hat may be caus i ng vas cul i t i c proces s es s houl d be s t opped; pl as mapheres i s has been us ed i n at t empt t o remove known ant i gens (Hbs Ag and HCV) and unknown ant i gens (cryogl obul i n).
2. T reatment of primary disease. Cont rol of any pri mary rheumat ol ogi c, i nfect i ous , or mal i gnant proces s general l y i s t he mos t effect i ve way t o cont rol vas cul i t i s t hat i s a s econdary feat ure.
3. Immunosuppressive treatment. Vas cul i t i c s yndromes t hat are not rel at ed t o a removabl e ant i gen or t reat abl e pri mary di s order oft en s houl d be cont rol l ed wi t h i mmunos uppres s i ve agent s —ei t her corticosteroids, cytotoxic drugs, or a combi nat i on of bot h. Cyclosporine, worki ng s peci fi cal l y on act i vat ed +
CD4 T cel l s , has been us ed i n s ome vas cul i t i s di s eas es ,
Pa g e 2 5 4 2
ABC Ambe r CHM Conve rte r Tria l ve rsion, http://w w w .proce sste x t.com/a bcchm.html
part i cul arl y t o count eract eye i nvol vement i n Behçet 's s yndrome and s ome cas es of refract ory s mal l ves s el vas cul i t i s . o
o
a. Corticosteroids al one s houl d be at t empt ed as fi rs t -l i ne pharmacol ogi c t reat ment i n al l pat i ent s who requi re concomi t ant cyt ot oxi c t herapy, except t hos e wi t h W egener's granul omat os i s . Dai l y dos i ng us ual l y i s requi red, al t hough al t ernat e-day dos i ng may be s uffi ci ent once s ys t emi c feat ures of t he i l l nes s are cont rol l ed. Low-dose aspirin therapy i s oft en added t o t reat ment of vas cul i t i c di s orders t o count eract t he pot ent i al vaso-occlusive effect s of gl ucocort i coi ds .
o
o
b. Cytotoxic drugs (e.g., cycl ophos phami de)
(1) Wegener' s granulomatosis al mos t al ways i s fat al wi t hout cyt ot oxi c t herapy; t herefore, pat i ent s s houl d be s t art ed on a combi nat i on of one of t hes e agent s and cort i cos t eroi ds .
(2) Polyarteritis nodosa and allergic angiitis al s o oft en requi re cyt ot oxi c drugs i n addi t i on t o s t eroi ds for cont rol of di s eas e mani fes t at i ons , al t hough recent pros pect i ve s t udi es have s hown benefi t i n decreas i ng di s eas e recurrences but not i n prevent i ng mort al i t y.
(3) In ot her di s orders , cyt ot oxi c drugs
Pa g e 2 5 4 3
ABC Ambe r CHM Conve rte r Tria l ve rsion, http://w w w .proce sste x t.com/a bcchm.html
t ypi cal l y are added t o cort i cos t eroi d t herapy i f t he s t eroi d dos es cannot be eas i l y l owered wi t hout t he di s eas e fl ari ng.
4. Other drugs s uch as daps one, col chi ci ne, NSAIDs , hydroxychl oroqui ne, and H 1 - or H 2 -bl ocki ng ant i hi s t ami ni c agent s have been us ed, part i cul arl y i n refract ory s mal l ves s el vas cul i t i s .
5. Plasmapheresis, whi ch removes ant i bodi es and i mmune compl exes from pl as ma, has not been s hown t o i mprove out comes i n s ys t emi c necrot i zi ng vas cul i t i s (e.g., pol yart eri t i s and al l ergi c angi i t i s ). In mixed cryoglobulinemia, whet her or not i t i s rel at ed t o HBV or HCV i nfect i on, pl as mapheres i s may i mprove di s eas e feat ures , at l eas t t emporari l y.
P.503
XIII. Juvenile Idiopathic Arthritis (JIA) A. Definition Juveni l e i di opat hi c art hri t i s (JIA) i s a chronic inflammatory arthritis t hat begi ns before age 16 years . Before cons i deri ng a di agnos i s of JIA, art hri t i s s houl d be pres ent i n one or more joi nt s for more t han 6 weeks , and ot her rheumat i c di s eas es s houl d be excl uded.
B. Epidemiology Annual i nci dence may be as hi gh as 0.01%. Al t hough t he apparent HLA as s oci at i ons s ugges t t hat t he al t ered i mmune res pons es are genet i cal l y t rans ferabl e, fami l i al aggregat i on of cas es i s uncommon.
C. Etiology and pathogenesis
Pa g e 2 5 4 4
ABC Ambe r CHM Conve rte r Tria l ve rsion, http://w w w .proce sste x t.com/a bcchm.html
The s ame fact ors i mport ant i n t he devel opment of adul t rheumat oi d art hri t i s al s o appl y t o JIA (s ee III C and onl i ne D). Et i ol ogi c fact ors are unknown but may i ncl ude i nfect i ous agent s . Immune system dysfunction i s apparent i n t he prol ongat i on and mai nt enance of s ynovi t i s . A s ubs et of pat i ent s wi t h JIA have s el ect i ve IgA defi ci ency whi ch may be i mport ant i n di s eas e pat hogenes i s .
D. Pathology The synovial lesions cannot be di s t i ngui s hed hi s t ol ogi cal l y from t hos e i n adul t rheumat oi d art hri t i s . Infl ammat ory changes cont i guous t o t he growt h pl at e may l ead t o premat ure epi phys eal cl os ure and s hort ened l i mbs or di gi t s . Chroni cal l y i nvol ved joi nt s exhi bi t fibrous ankylosis more oft en t han i n adul t rheumat oi d art hri t i s . Pannus formation can occur, al t hough t ypi cal l y l at er i n t he di s eas e cours e t han i n adul t s wi t h rheumat oi d art hri t i s . As a cons equence, des t ruct i ve joi nt di s eas e al s o i s much l es s common i n JIA.
E. Classification Three pri mary s ubt ypes of JIA (i .e., s ys t emi c-ons et juveni l e art hri t i s , pauci art i cul ar art hri t i s , and pol yart i cul ar art hri t i s ) can be di s t i ngui s hed i n t he fi rs t 6 mont hs of i l l nes s . It i s i mport ant prognos t i cal l y and t herapeut i cal l y t o di s t i ngui s h bet ween s ubt ypes of i l l nes s . Import ant cl as s i fi cat i on feat ures are t he presence or absence of prominent systemic features and t he total number of involved joints.
1. Systemic-onset juvenile arthritis, al s o cal l ed Still' s disease, occurs i n approxi mat el y 10%–20% of pat i ent s and i s charact eri zed by an earl y pat t ern of prominent systemic complaints and extra-articular involvement. Boys are affect ed as commonl y as gi rl s , and a peak age of i nci dence i s not evi dent . o
o
a. Clinical features
Pa g e 2 5 4 5
ABC Ambe r CHM Conve rte r Tria l ve rsion, http://w w w .proce sste x t.com/a bcchm.html
(1) T ypical features of the early disease course are high spiking fevers and marked constitutional complaints. Overt art hri t i s may not be part of t he earl y cours e but devel ops wi t hi n weeks t o mont hs of t he ons et of i l l nes s . A charact eri s t i c nonpruritic, fleeting, maculopapular rash occurs i n 90% of pat i ent s and i s mos t apparent wi t h fever s pi kes .
(2) Common features of active disease i ncl ude l ymphadenopat hy, hepat os pl enomegal y, and peri cardi t i s . Accompanyi ng myocardi t i s i s rare.
(3) The mos t s eri ous mani fes t at i on of s ys t emi c-ons et juveni l e art hri t i s i s macrophage act i vat i on s yndrome. Act i vat ed macrophages i n mul t i pl e organs l eads t o l eukopeni a, t hrombocyt openi a, CNS and l i ver dys funct i on, and pos s i bl e mul t i organ fai l ure and deat h.
o
o
b. Laboratory findings
(1) Hematologic findings i ncl ude a s t ri ki ngl y el evat ed ESR, promi nent l eukocyt os i s , and t hrombocyt os i s , and moderat e-t o-s evere anemi a of chroni c di s eas e, al t hough t here oft en i s s t ri ki ng mi crocyt os i s .
Pa g e 2 5 4 6
ABC Ambe r CHM Conve rte r Tria l ve rsion, http://w w w .proce sste x t.com/a bcchm.html
(2) Serologic findings onl y rarel y i ncl ude rheumat oi d fact or and ANAs .
o
o
c. Disease course. Di s eas e fl are-ups are punct uat ed by rel at i vel y s ympt om-free i nt erval s . Pol yart i cul ar art hri t i s becomes evi dent at s ome poi nt i n t he fi rs t 6 mont hs of i l l nes s . Sys t emi c s ympt oms mos t oft en decreas e wi t hi n 9 mont hs aft er ons et .
(1) Approxi mat el y 50% of pat i ent s may begi n t o devel op s ympt oms of di s eas e t hat res embl e t he pol yart i cul ar art hri t i s s ubs et , wi t h progres s i ve joi nt i nvol vement det ermi ni ng di s eas e out come.
(2) The remai ni ng 50% of pat i ent s event ual l y recover compl et el y.
2. Polyarticular arthritis occurs i n approxi mat el y 30%–40% of pat i ent s and involves five or more joints i n t he fi rs t 6 mont hs of i l l nes s . Systemic features are not present. Gi rl s are affect ed P.504
much more oft en t han boys . Pol yart i cul ar art hri t i s can be furt her s eparat ed i nt o rheumatoid factor–positive or –negative subsets. Pat i ent s who are pos i t i ve for rheumat oi d fact or pres ent mos t oft en i n l at e chi l dhood; a peak age of i nci dence i s not evi dent for pat i ent s who are negat i ve for rheumat oi d fact or.
Pa g e 2 5 4 7
ABC Ambe r CHM Conve rte r Tria l ve rsion, http://w w w .proce sste x t.com/a bcchm.html o
o
a. Clinical features
(1) Typi cal l y, i nfl ammat ory pol yart i cul ar art hri t i s may have an acut e or a gradual ons et s i mi l ar t o t he pres ent at i on of adul t rheumat oi d art hri t i s . Symmet ri cal l arge and s mal l joi nt i nvol vement al s o i s t ypi cal . Promi nent feat ures may i ncl ude cervi cal s pi ne, and t emporomandi bul ar joi nt (TMJ) di s eas e.
(2) Rarel y, pat i ent s have s ympt oms and s i gns of ant eri or uvei t i s .
(3) Pat i ent s who are pos i t i ve for rheumat oi d fact or can have s ubcut aneous nodul es as i n adul t -ons et rheumat oi d art hri t i s .
o
o
b. Laboratory findings
(1) Hematologic findings frequent l y i ncl ude moderat e el evat i on of eryt hrocyt e s edi ment at i on rat e, l eukocyt e count , and pl at el et count . Pat i ent s us ual l y devel op a mi l d normochromi c, normocyt i c anemi a of chroni c di s eas e.
(2) Serologic findings i ncl ude rheumat oi d fact or i n 10%–20% and ANAs i n 20%–40% of pat i ent s .
Pa g e 2 5 4 8
ABC Ambe r CHM Conve rte r Tria l ve rsion, http://w w w .proce sste x t.com/a bcchm.html
P.505
o
o
c. Disease course. Pol yart i cul ar art hri t i s can be chroni c and pers i s t ent or can purs ue a more i nt ermi t t ent , rel aps i ng cours e.
(1) Pat i ent s who are pos i t i ve for rheumat oi d fact or are at great es t ri s k for chroni c, eros i ve, and s evere art hri t i s and s i gni fi cant di s abi l i t y. Thes e pat i ent s have di s eas e t hat i s very s i mi l ar t o adul t -ons et rheumat oi d art hri t i s .
(2) Pat i ent s who are negat i ve for rheumat oi d fact or l es s oft en have s evere di s eas e or di s eas e t hat l as t s i nt o adul t hood.
3. Pauciarticular arthritis occurs i n approxi mat el y 50% of pat i ent s and i nvol ves four or fewer joints i n t he fi rs t 6 mont hs of i l l nes s . Thi s pat i ent group i s compos ed of t hree s ubs et s . o
o
a. Oligoarthritis and anterior uveitis affect gi rl s more oft en t han boys , and peak i nci dence i s i n earl y chi l dhood.
(1) Clinical features
(a) Typi cal l y, t he art hri t i s i s
Pa g e 2 5 4 9
ABC Ambe r CHM Conve rte r Tria l ve rsion, http://w w w .proce sste x t.com/a bcchm.html
as ymmet ri cal , mi l d, and i nvol ves t he knee. Ot her peri pheral joi nt s al s o can be i nvol ved, but t he axi al s kel et on us ual l y i s s pared.
(b) Sys t emi c s ympt oms and s i gns are mi l d or abs ent .
(c) Pot ent i al l y s eri ous anterior uveitis unrel at ed t o art hri t i s act i vi t y can devel op i n up t o 25% of pat i ent s . Thi s us ual l y chroni c eye l es i on can be as ympt omat i c and can l ead t o blindness i f unrecogni zed or i nadequat el y t reat ed.
(2) Laboratory findings
(a) Hematologic findings us ual l y do not i ncl ude anemi a, t hrombocyt os i s , and l eukocyt os i s . The ESR i s normal or onl y mi ni mal l y el evat ed.
(b) Serologic findings i ncl ude ANA pos i t i vi t y i n 60% of pat i ent s , whi ch i dent i fi es t hos e at hi gher ri s k for chroni c uvei t i s . Rheumat oi d fact or t ypi cal l y i s not pres ent .
(3) Disease course. Mos t pat i ent s have pauci art i cul ar i nvol vement t hat i s manageabl e and not di s abl i ng. In approxi mat el y one-t hi rd
Pa g e 2 5 5 0
ABC Ambe r CHM Conve rte r Tria l ve rsion, http://w w w .proce sste x t.com/a bcchm.html
of pat i ent s , t he di s eas e event ual l y i nvol ves more t han four joi nt s , and i s furt her s ubdi vi ded as ext ended pauci -art i cul ar art hri t i s . o
o
b. Axial skeleton oligoarthritis predomi nant l y affect s boys , wi t h di s eas e ons et us ual l y begi nni ng i n l at e chi l dhood.
(1) Clinical features. As ymmet ri cal knee, ankl e, or mi d-t ars al art hri t i s i s mos t common, fol l owed by t he s acroi l i ac hi p joi nt s . Acut e ant eri or uvei t i s can occur as i n adul t s pondyl oart hropat hi es , but t hi s feat ure i s not t he chroni c and s i ght -t hreat eni ng form s een i n t he ot her pauci art i cul ar s ubs et .
(2) Laboratory findings
(a) Hematologic findings are not di s t i nct .
(b) Serologic findings i ndi cat e t hat 50% of t hes e pat i ent s have HLA-B27, but few have rheumat oi d fact or.
(2) Disease course. Many of t hes e pat i ent s devel op feat ures of ankyl os i ng s pondyl i t i s , ps ori at i c art hri t i s , or react i ve art hri t i s i n l at er l i fe.
o
Pa g e 2 5 5 1
ABC Ambe r CHM Conve rte r Tria l ve rsion, http://w w w .proce sste x t.com/a bcchm.html
o
c. Oligoarthritis with prominent dactylitis i s a t hi rd pres ent at i on s een mos t frequent l y i n gi rl s of any age. Psoriasis commonl y affect s t hes e pat i ent s and t hei r fami l i es .
F. Diagnosis
1. Difficulties in diagnosis. Di agnos i ng art hri t i s i n chi l dren may be di ffi cul t . Chi l dren may avoi d us i ng an i nvol ved joi nt i ns t ead of compl ai ni ng of pai n. They may res pond t o t he pai n of an i nfl amed joi nt by di s pl ayi ng i rri t abi l i t y, regres s i ve behavi or, or emot i onal wi t hdrawal . Once joi nt i nvol vement has been di s covered, di s eas e s houl d be pres ent for at l eas t 6 weeks before a di agnos i s of JIA i s s eri ous l y cons i dered. W i t hi n t he fi rs t 6 mont hs , an at t empt s houl d be made t o cl as s i fy pat i ent s i nt o a cl i ni cal s ubs et . o
o
a. The synovial fluid us ual l y i s mi l dl y i nfl ammat ory 3
(i .e., a W BC count of 10,000–20,000/mm ); however, t he number of W BCs pres ent may not paral l el di s eas e act i vi t y. Joi nt fl ui d cul t ure and anal ys i s are es peci al l y i mport ant i n more acut e pauci -art i cul ar forms t o excl ude i nfect i on (bact eri al or mycobact eri al ). o
o
b. Radiographic findings are nons peci fi c.
(1) Early findings may i ncl ude onl y s oft t i s s ue s wel l i ng and peri -art i cul ar demi neral i zat i on.
Pa g e 2 5 5 2
ABC Ambe r CHM Conve rte r Tria l ve rsion, http://w w w .proce sste x t.com/a bcchm.html
(2) Late findings i ncl ude epi phys eal changes (ei t her premat ure cl os ure or overgrowt h, dependi ng on epi phys eal act i vi t y at t he t i me of i nvol vement ) and art i cul ar eros i on or joi nt -s pace narrowi ng.
(3) Distinctive long bone periosteal elevation in leukemia may al l ow di fferent i at i on from JIA. Local i zed joi nt abnormal i t i es (e.g., os t eonecros i s and os t eochondri t i s ) may have di s t i nct radi ographi c pres ent at i ons .
(4) Cervical spine involvement, part i cul arl y wi t h ankyl os i s at C2–C3, i s common i n JIA.
2. Differential diagnosis o
o
a. A vari et y of genetic or inborn metabolic disorders as wel l as nonrheumat i c condi t i ons can s uperfi ci al l y res embl e JIA. It i s s peci fi cal l y i mport ant t o rul e out infectious etiologies (e.g., Lyme di s eas e and TB) and malignancies (e.g., l eukemi a) as caus es of chi l dhood art hri t i s .
o
o
b. Other rheumatic diseases (e.g., rheumat i c fever, SLE, and undi fferent i at ed connect i ve t i s s ue di s eas e) may requi re a peri od of obs ervat i on for charact eri s t i c ext ra-art i cul ar feat ures t o evol ve.
G. Therapeutic approach
Pa g e 2 5 5 3
ABC Ambe r CHM Conve rte r Tria l ve rsion, http://w w w .proce sste x t.com/a bcchm.html
1. Education of patients and parents about i nfl ammat ory art hri t i s i s i mport ant , and t he general l y favorabl e cours e of JIA s houl d be emphas i zed. Long-t erm goal s of s uppres s i on of di s eas e act i vi t y and prevent i on of deformi t y s houl d be i ns t i t ut ed, and chi l dren's ps ychol ogi cal and emot i onal devel opment s houl d not be negl ect ed.
2. The us e of appropri at e therapy i s i mport ant i n rel i evi ng pai n and mai nt ai ni ng funct i on. Parent s s houl d be urged t o perform act i ve rol es i n gi vi ng phys i cal t herapy and medi cat i ons , encouragi ng s chool at t endance, and mai nt ai ni ng chi l dren's abi l i t y t o be s el f-s uffi ci ent . o
o
a. Pharmacologic therapy
(1) Salicylates. Thes e agent s are no l onger t he pri mary drugs us ed i n t he t reat ment of JIA becaus e of concerns about t he pot ent i al preci pi t at i on of Reye's s yndrome by as pi ri n.
(2) Other NSAIDs. Current l y, naproxen and i buprofen are mos t oft en pres cri bed as i ni t i al t reat ment . Al t hough i ndomet haci n and nabumet one are not FDA-approved for chi l dren, t hey are al s o effect i ve.
(3) Second-line agents. Met hot rexat e, t he
Pa g e 2 5 5 4
ABC Ambe r CHM Conve rte r Tria l ve rsion, http://w w w .proce sste x t.com/a bcchm.html
mos t commonl y us ed s econd-l i ne agent , i s effect i ve. Oral gol d, hydroxychl oroqui ne, and D-peni ci l l ami ne al s o are now rarel y us ed i n t he t reat ment of juveni l e rheumat oi d art hri t i s . Sul fas al azi ne i s al s o us ed becaus e i t has rapi d ons et of effect , i nfrequent s eri ous t oxi ci t y, and rel at i vel y good effi cacy. A recent t ri al of l efl unomi de s howed t he agent t o be effect i ve.
(4) Biologic agents. Et anercept , a genet i cal l y engi neered fus i on prot ei n of recombi nant TNF and human IgG, i s approved for us e i n chi l dren. Thi s agent i s mos t oft en i ndi cat ed i n juveni l e rheumat oi d art hri t i s of a pol yart i cul ar cours e t hat i s unres pons i ve t o met hot rexat e.
(5) Corticosteroids
(a) In patients with systemic complaints who have life-threatening manifestations (e.g., peri cardi t i s ), moderat e-t o-hi gh-dos e s ys t emi c cort i cos t eroi ds may be requi red. P.506
(b) In nonambulatory children with polyarticular disease, a s mal l , every-ot her-day dos e of cort i cos t eroi ds may promot e wei ght beari ng. The cons equences of bei ng nonambul at ory i n
Pa g e 2 5 5 5
ABC Ambe r CHM Conve rte r Tria l ve rsion, http://w w w .proce sste x t.com/a bcchm.html
chi l dhood out wei gh pos s i bl e cort i cos t eroi d compl i cat i ons i n t hi s cl i ni cal s i t uat i on.
(c) Local cort i cos t eroi d i nject i ons oft en are effect i ve i n managi ng pauci-articular disease or more severely involved joints in polyarticular disease. In s ome chi l dren wi t h l i mi t ed pauci -art i cul ar di s eas e, i nt ra-art i cul ar s t eroi d t herapy at ons et may l ead t o compl et e remi s s i on.
(d) In t he t reat ment of chronic uveitis, s ys t emi c cort i cos t eroi ds may be needed i f i ni t i al t reat ment wi t h t opi cal or l ocal cort i cos t eroi ds and di l at i ng agent s i s not effect i ve. Recent cl i ni cal t ri al s wi t h i nfl i xi mab are very encouragi ng. Pat i ent s at hi gh ri s k for chroni c uvei t i s (ol i goart i cul ar s ubgroup) s houl d be eval uat ed every 3 mont hs t o det ermi ne t reat ment effect i venes s .
(6) Intravenous infusions of gamma globulin. Thi s agent has been us ed t o cont rol s evere s ys t emi c-ons et or pol yart i cul ar di s eas e.
o
o
b. Surgery. Correct i on of deformi t i es and t ot al joi nt repl acement s may be needed i n chroni c, s evere di s eas e. Jaw i mpl ant s have been us ed for mi crognat hi a.
o
Pa g e 2 5 5 6
ABC Ambe r CHM Conve rte r Tria l ve rsion, http://w w w .proce sste x t.com/a bcchm.html
o
c. Physical and occupational therapy. Such t reat ment i s es peci al l y i mport ant i n JIA. Chi l dren mus t l earn and pract i ce exerci s es t hat mai nt ai n mus cl e t one and prevent joi nt cont ract ures . Ni ght s pl i nt s may hel p mi ni mi ze t he evol ut i on of joi nt cont ract ures ; s eri al s pl i nt i ng may i mprove exi s t i ng joi nt cont ract ures .
H. Prognosis
1. Disability. Bet ween 50% and 75% of pat i ent s recover compl et el y by adul t hood. Approxi mat el y 10% devel op s evere funct i onal deformi t i es .
2. Specific complications o
o
a. Joint deformities. Pat i ent s wi t h pol yart i cul ar art hri t i s are mos t l i kel y t o devel op chroni c, eros i ve art hri t i s and s ubs equent joi nt deformi t i es , es peci al l y t hos e chi l dren wi t h rheumat oi d fact or, who res embl e pat i ent s wi t h adul t rheumat oi d art hri t i s .
o
o
b. Chronic uveitis. As much as 15% of pat i ent s wi t h ol i goart i cul ar di s eas e who have chroni c uvei t i s devel op s ome vi s ual i mpai rment , even i f t he probl em i s careful l y t reat ed.
o
o
c. Growth retardation
Pa g e 2 5 5 7
ABC Ambe r CHM Conve rte r Tria l ve rsion, http://w w w .proce sste x t.com/a bcchm.html
(1) General growth retardation can occur i n pat i ent s wi t h pers i s t ent , wi des pread i nfl ammat ory act i vi t y and i n t hos e t reat ed wi t h s ys t emi c cort i cos t eroi ds .
(2) Local growth abnormalities from t he i nfl ammat ory proces s can l ead t o micrognathia as wel l as t o leg and finger length discrepancies i n s ome pat i ent s .
3. Death. Macrophage act i vat i on s yndrome may be l i fe-t hreat eni ng, and, i n rare cas es , pat i ent s wi t h part i cul arl y s evere and prot ract ed di s eas e devel op progres s i ve amyloidosis or di e of s econdary i nfect i on. Rarel y, overwhel mi ng myocarditis i n s ys t emi c-ons et juveni l e rheumat oi d art hri t i s can l ead t o CHF and deat h.
XIV. Miscellaneous Syndromes A. Rheumatic manifestations of human immunodeficiency virus (HIV) Mus cul os kel et al compl ai nt s are rel at i vel y common duri ng t he cours e of HIV i nfect i on. The cl i ni cal s pect rum i s vari ed, rangi ng from art hral gi as t o di s t i nct rheumat i c di s orders . Art hral gi a and myal gi as , t he mos t common rheumat i c mani fes t at i ons , occur i n 10%–20% of pat i ent s , us ual l y wi t h t he i ni t i al i nfect i on. Ot her mani fes t at i ons can be cl as s i fi ed i nt o t he fol l owi ng groups :
1. Spondyloarthropathy (react i ve art hri t i s , ps ori at i c art hri t i s , and undi fferent i at ed s pondyl oart hropat hy). In young pers ons wi t h s eronegat i ve art hri t i s , i t i s i mport ant t o cons i der HIV.
Pa g e 2 5 5 8
ABC Ambe r CHM Conve rte r Tria l ve rsion, http://w w w .proce sste x t.com/a bcchm.html
2. HIV-associated arthritis. The us ual pres ent at i on i s an ol i goart i cul ar as ymmet ri c peri pheral art hri t i s affect i ng t he knees and ankl es . A s ymmet ri cal pol yart i cul ar eros i ve form, whi ch can al s o occur, may be di ffi cul t t o di s t i ngui s h from rheumat oi d art hri t i s . Thi s form i s us ual l y res pons i ve t o t reat ment wi t h NSAIDs .
3. Painful articular syndrome. Thi s poorl y unders t ood s yndrome, whi ch affect s 10% of HIV-i nfect ed pat i ent s , i s charact eri zed by s evere joi nt pai n of abrupt ons et t hat l as t s a few hours t o 2 days . Laborat ory s t udi es and i magi ng res ul t s us ual l y are normal . P.507
4. Septic arthritis. Thi s condi t i on i s us ual l y s een more frequent l y i n pat i ent s wi t h a hi s t ory of i nt ravenous drug abus e and i n hemophi l i acs . Bot h t he us ual and opport uni s t i c organi s ms are i mpl i cat ed. In i nt ravenous drug abus ers , t he mos t common pat hogens are S. aureus and St rept oc oc c us pneumoni ae. Of t he opport uni s t i c organi s ms , Candi da al bi c ans i s t he mos t common. Axi al joi nt s are affect ed more frequent l y t han peri pheral joi nt s .
5. Diffuse infiltrative lymphocytosis syndrome (DILS). Thi s s yndrome cons i s t s of dry eyes and mout h wi t h a pos i t i ve Schi rmer t es t and s al i vary gl and enl argement . DILS di ffers from cl as s i c Sjögren's s yndrome i n t hat i t us ual l y affect s mal es and i s not charact eri zed by art hri t i s or aut oant i body product i on (SS-A, SS-B, and rheumat oi d fact or).
Pa g e 2 5 5 9
ABC Ambe r CHM Conve rte r Tria l ve rsion, http://w w w .proce sste x t.com/a bcchm.html
6. Myopathies. Si gni fi cant mus cl e weaknes s may be t he pres ent i ng compl ai nt . Such condi t i ons may be t he res ul t of HIV myopat hy, z i dovudi ne myopat hy, HIV was t i ng s yndrome, rhabdomyol ys i s , or pyomyos i t i s .
7. Avascular necrosis and osteopenia/osteoporosis. Recent l y, avas cul ar necros i s has been report ed i n HIV pat i ent s . The et i ol ogy i s uncl ear. Os t eopeni a and os t eoporos i s l i kewi s e are newer fi ndi ngs i n t hi s pat i ent group. HIV t herapy may be i mpl i cat ed.
B. Hepatitis and cryoglobulinemia Bot h HBV and HCV can be l i nked wi t h art hri t i s . HBV i s as s oci at ed wi t h s udden-ons et s ymmet ri c art hri t i s occas i onal l y found wi t h urt i cari a. HCV i s oft en as s oci at ed wi t h t ype II cryogl obul i nemi a and can mani fes t as a combi nat i on of art hri t i s , pal pabl e purpura, and cryogl obul i nemi a.
C. Acute rheumatic fever (ARF) The di s order occurs fol l owi ng a group A s t rept ococcus (S. pyogenes ) pharyngeal i nfect i on. In one-t hi rd of pat i ent s , t he t ri ggeri ng i nfect i on i s s i l ent , but i t can be i dent i fi ed s erol ogi cal l y. The i ncubat i on peri od from pharyngi t i s t o i nfect i on i s 2–3 weeks . Accordi ng t o t he revi s ed Jones Cri t eri a (1992) for t he di agnos i s of ARF, i f a pat i ent has s upport i ve evi dence of precedi ng i nfect i on, t he pres ence of t wo major or one major and t wo mi nor mani f es t at i ons s ugges t s a hi gh probabi l i t y of ARF.
1. Major manifestations are cardi t i s , pol yart hri t i s , chorea, eryt hema margi nat um, and s ubcut aneous nodul es . The art hri t i s i s us ual l y mi grat ory, preferent i al l y affect i ng t he l arge joi nt s of t he l ower ext remi t i es . Pai n i n affect ed joi nt s may be out of proport i on t o phys i cal
Pa g e 2 5 6 0
ABC Ambe r CHM Conve rte r Tria l ve rsion, http://w w w .proce sste x t.com/a bcchm.html
exami nat i on fi ndi ngs of i nfl ammat i on.
2. Minor manifestations are art hral gi as , fever, el evat ed ESR, and a prol onged P-R i nt erval .
3. Supporting evidence of preceding group A streptococcal infection i ncl udes a pos i t i ve t hroat cul t ure, el evat ed or ri s i ng ant i s t rept ol ys i n-O t i t er, or rapi d s t rept ococcal ant i gen t es t .
D. Avascular necrosis Thi s condi t i on, whi ch i s al s o referred t o as osteonecrosis, i s us ed t o des cri be t he deat h of cel l ul ar component s of bone as t he res ul t of di mi ni s hed art eri al bl ood s uppl y. It may be i di opat hi c or occur i n a vari et y of s et t i ngs as s oci at ed wi t h cert ai n medi cat i ons (s t eroi ds and cyt ot oxi c agent s ), underl yi ng connect i ve t i s s ue di s eas es , hemat ol ogi c di s orders , i nfi l t rat i ve di s orders , embol i s m, al cohol , and t rauma. Invol vement of t he epi phys i s of l ong bones s uch as t he femoral heads i s us ual l y charact eri s t i c, but ot her bones can be affect ed. In t he earl i es t s t ages , di agnos i s can be made by MRI, but at l at er s t ages , fi ndi ngs can be i dent i fi ed on pl ai n fi l ms . Bl ood s t as i s , hypercoagul abi l i t y, and damage t o endot hel i al cel l s are i mport ant .
E. Fibromyalgia Thi s common noni nfl ammat ory condi t i on i s charact eri zed by di ffus e pai n. The et i ol ogy remai ns uncl ear, but affect ed pat i ent s t end t o have di s rupt i on of non–rapi d-eye movement (REM) s t age IV s l eep. In addi t i on t o di ffus e pai n, pat i ent s oft en report probl ems wi t h i ns omni a, i rri t abl e bowel , t ens i on headaches , mi grai nes , and depres s i on. Exami nat i on reveal s mul t i pl e t ender poi nt s (pai n on pal pat i on) over t he fol l owi ng mus cl e regi ons : s ubocci pi t al mus cl e i ns ert i on; t rapezi us ; s upras pi nat us ; gl ut eal ; great er t rochant er; l ow ant eri or cervi cal ; s econd cos t ochondral junct i on, 2 cm di s t al t o t he l at eral epi condyl es ; and t he medi al fat pad of t he knees , proxi mal
Pa g e 2 5 6 1
ABC Ambe r CHM Conve rte r Tria l ve rsion, http://w w w .proce sste x t.com/a bcchm.html
t o t he joi nt l i ne. Laborat ory i nves t i gat i on s houl d i ncl ude s t udi es t o rul e out met abol i c di s orders (e.g., hypot hyroi di s m and hypert hyroi di s m); el ect rol yt e di s t urbances (e.g., l ow magnes i um, cal ci um, or phos phorus ); l i ver di s eas e (e.g., vi ral hepat i t i s ); connect i ve t i s s ue di s eas e (e.g., PMR and SLE); and pri mary myopat hy as cl i ni cal l y i ndi cat ed. Treat ment i s ai med at i mprovi ng t he qual i t y of s l eep. Ami t ri pt yl i ne or l ow-dos e mus cl e rel axant s , i n addi t i on t o regul ar phys i cal exerci s e, are commonl y us ed. If depres s i on i s a major component , referral for ps ychi at ri c cons ul t at i on i s i ndi cat ed. P.508
F. Amyloidosis The t hree mai n forms of amyl oi d depos i t i on di s orders t hat pres ent wi t h an art hropat hy are AL amyl oi d, B2 mi crogl obul i n amyl oi d, and AA amyl oi d.
1. AL amyloid us ual l y mani fes t s as prot ei nuri a/renal fai l ure, a cardi omyopat hy, or a vari et y of neuropat hi c s ympt oms . Ot her fi ndi ngs i ncl ude edema, hepat omegal y, macrogl os s i a, and purpura. Peri -art i cul ar amyl oi d depos i t i on can pres ent as a ps eudoart hri t i s , but joi nt effus i ons wi t h amyl oi d fi bri l s may al s o be found. The charact eri s t i c fi ndi ng may be a s oft t i s s ue “s houl der pad s i gn.―
2. B2 microglobulin amyl oi d i s t he s econd mos t common form of amyl oi d depos i t i on. Mos t pat i ent s who devel op t hi s form of amyl oi dos i s have been on renal di al ys i s for prol onged peri ods and pres ent wi t h joi nt pai n, carpal t unnel s yndrome, and os t eonecros i s .
Pa g e 2 5 6 2
ABC Ambe r CHM Conve rte r Tria l ve rsion, http://w w w .proce sste x t.com/a bcchm.html
3. AA amyloid i s found i n pat i ent s wi t h rheumat oi d art hri t i s and fami l i al Medi t erranean fever. It i s al s o referred t o as a s econdary (react i ve) amyl oi d, becaus e i t can occur i n any chroni c i nfl ammat ory di s order, i ncl udi ng i nfect i ons , neopl as i a, or ot her rheumat i c condi t i ons .
G. Parvovirus B19 infection Parvovi rus or Fi ft h's di s eas e i s endemi c i n s chool -age chi l dren. It mani fes t s as fever, cons t i t ut i onal s ympt oms , and a “s l apped cheek― appearance wi t h s t ri ki ng eryt hema on t he chi l d's face. Adul t s t ypi cal l y pres ent wi t h art hral gi as or a s ymmet ri cal art hri t i s t hat may mi mi c rheumat oi d art hri t i s . Tes t i ng for Parvovi rus s erol ogy earl y i n t he cours e of t he di s eas e can be di agnos t i c (pos i t i ve IgM ant i body). The cours e i s s el f-l i mi t ed and res ponds t o ant i -i nfl ammat ory pai n medi cat i ons .
H. Relapsing polychondritis H. Rel aps i ng pol ychondri t i s i s a condi t i on charact eri zed by i nfl ammat i on of cart i l age. It can affect any cart i l age i n t he body, but mos t commonl y affect s t he cart i l agi nous port i on of t he nos e and ext ernal ear, pres ent i ng as s udden pai n, rednes s , and s wel l i ng wi t h event ual des t ruct i on i f unt reat ed. The et i ol ogy i s uncl ear, but aut oi mmune mechani s ms may be i nvol ved. It i s s omet i mes as s oci at ed wi t h ot her connect i ve t i s s ue di s orders as wel l as mal i gnanci es .
I. Auto-inflammatory diseases Thes e di s orders are del i neat ed mos t oft en by cycl es of cl i ni cal i nfl ammat i on wi t hout t he pres ence of ant i gen-s peci fi c T cel l s or charact eri s t i c aut oant i bodi es . Several of t hes e di s eas es previ ous l y have been cat egori zed as peri odi c fever s yndromes . Over t he l as t 5 years , t he genet i c bas i s for mos t of t hes e di s orders has been det ermi ned, and DNA s equenci ng t o det ect s peci fi c mut at i ons i s avai l abl e i n commerci al l aborat ori es (s ee onl i ne Tabl e 10-16).
ONLINE TABLE 10-16 AutoInflammatory
Pa g e 2 5 6 3
ABC Ambe r CHM Conve rte r Tria l ve rsion, http://w w w .proce sste x t.com/a bcchm.html
Diseases F e at
T
N
u
R H F
O
r F A I C MM e M P D AW I s F S S S S D M Je Sc D N N N aj wi ot ut o o o o s t i ch p p p r h, s , at at at A T h, Fr t e t e t e n ur Ir e rn rn rn c ki i s nc e s h h st h, ry Ar a b, Ar m e ni a n G A A A A A R e ut ut ut ut ut ar n o o o o o el et s s s s s y ic o o o o o a P m m m m m ut at al al al al al o te R D R d d s
Pa g e 2 5 6 4
ABC Ambe r CHM Conve rte r Tria l ve rsion, http://w w w .proce sste x t.com/a bcchm.html
r ec o ec o o o n e me mmm s s i n s s i n i n al iv a iv a a d e nt e nt nt o m in a nt C 1 1 1 1 1 1 h 6 2 2 q q q r p p q 4 4 4 o 1 1 2 4 4 4 m3 3 4 o s o m al L o c at io n G M T M CI CI CI e E N e A A A n F F v S S S e V R al 1 1 1 I
S o
n
FI n
v
A at
ol
(t e
v
y ki
e
p n
Pa g e 2 5 6 5
ABC Ambe r CHM Conve rte r Tria l ve rsion, http://w w w .proce sste x t.com/a bcchm.html
d
e a I s T e N F re ce pt or
) A 9 1 6 9 8 M g 0 0 m5 5 o e % 0 o % % st at b % nt < < wi O y b h 6 2 th n a y s m 0 in s g a m et e g e
Y 6 m
2 e di 0 2 a 0 n D 1 D 3 < 2 C u â a â 2 â o r €“ ys €“ 4 €“ nt at 3 t o 7 h 3 i n io d w d o d u n a e a ur a o of ys e ys s ys u A
ks
s
tt a c k s A P P P N N N b er ai ai o o o
Pa g e 2 5 6 6
ABC Ambe r CHM Conve rte r Tria l ve rsion, http://w w w .proce sste x t.com/a bcchm.html
d it n n n n n o o â â e e e m n €“ €“ in e n di al al o ar Ir n rh ri s e ta p a t i ec a o i fi n n c d (9
v
5
o
%
m
)
iti n
g Pl Fr Fr N N N N e e e o o o o u q q n n n n ri u u e e e e ti e e c nt nt C O Fr C M Ur Ur ut cc e o ac t i t i a a q m ul ca ca n s i u m o ri ri e o e o p al al o n nt n: a (n (n u al : m p ot ot s : t e ac ul pr pr er n ul ar ur ur ys d e
iti iti
i p er s ,
c) c)
el er p
Pa g e 2 5 6 7
ABC Ambe r CHM Conve rte r Tria l ve rsion, http://w w w .proce sste x t.com/a bcchm.html
oi yt a d h p er e ul yt m e h at s , e o p m u ur a s p p ur at a ch e s R M L Ol P P Pr h o oc i g ol ol o e n al o y y gr u ar i z s y ar ar e m th e m th th ss at ri d m ra ra i v ol t i m et l g l g e o s : y ri i a i a d gi l a al c s s - e c rg gi ar
la st
e a: t h
nc ru
jo ar ri
i n ct
in th ti
at i v
t s ra s
in e
(7 l g
g ch
5 ia
di a
%s
st n
)
al g li e m s, b m p a
Pa g e 2 5 6 8
ABC Ambe r CHM Conve rte r Tria l ve rsion, http://w w w .proce sste x t.com/a bcchm.html
ai s s n iv s e ca rt il a g e o v er gr o w th C A Fr A V N A o b e b er o nt nj s q s y n er u e u e fr e i o n nt e nt e
r
ct
nt
q
a
iv
(p
u
n
iti
ai
e
d
s
nf
nt
p
ul
o
)
st er io r u v ei ti s
Pa g e 2 5 6 9
ABC Ambe r CHM Conve rte r Tria l ve rsion, http://w w w .proce sste x t.com/a bcchm.html
L U C V N N U y nc o er o o nc mo m y n n o p m m co e e m h mo m
m
a o n m
o
d n
o
n
e
n
n
(c
o
er
p
vi
at
ca
h
l)
y C N N N C R N ol o o o o ar o d n n n n e n E e e e si x
st
p
e
o
nt
e
s u r e E x a c e r b at io n
Pa g e 2 5 7 0
ABC Ambe r CHM Conve rte r Tria l ve rsion, http://w w w .proce sste x t.com/a bcchm.html
s S N N N N C C e o o o o o o n n n n n mm s e e e e mm o
o o
ri
n n
n e u r al D e af n e ss C N N N N N C N o o o o o hr S n n n n n o e e e e e ni c m e ni n gi ti s A C 2 N N N N mo 5 o o o o yl m % n n n n oi m
e e e e
d o o n
Pa g e 2 5 7 1
ABC Ambe r CHM Conve rte r Tria l ve rsion, http://w w w .proce sste x t.com/a bcchm.html
si s L L â â â â â a o †“ †‘ †‘ †‘ †‘ b w S Ig W W W s C er D B B B 5 u ( C, C, C, a m> E E E in ty 1 S S S hi p 0 R R R bi e 0 to I I r T U in N / s F m er re L) o ce ; s pt m al or a fl ( y ui < b d n e s g/ n m or L) m al b el o w a g e 3 T C Et N ? ? ?I
Pa g e 2 5 7 2
ABC Ambe r CHM Conve rte r Tria l ve rsion, http://w w w .proce sste x t.com/a bcchm.html
r ol a o N N L e ch n n S S â at i c er e AI AI €“ m i n ce ef D D 1 e e pt fe s s a nt
, ct
nt
C iv
a
or e
g
ti ?
o
co St
ni
s t at
st
er i n
s
oi s d s ESR, eryt hrocyt e s edi ment at i on rat e; IgD, i mmunogl obul i n D; IL-1, i nt erl euki n-1; NSAIDs , nons t eroi dal ant i -i nfl ammat ory drugs ; TNF, t umor necros i s fact or; W BC, whi t e bl ood cel l .
1. Familial Mediterranean Fever (FMF). Thi s di s order us ual l y pres ent s before age 20 wi t h peri odi c 1- t o 3-day epi s odes of fever, pl euro-peri t oneal pai n, and art hri t i s . A majori t y of pat i ent s , part i cul arl y t hos e not t reat ed
Pa g e 2 5 7 3
ABC Ambe r CHM Conve rte r Tria l ve rsion, http://w w w .proce sste x t.com/a bcchm.html
wi t h col chi ci ne, devel op amyl oi dos i s .
2. T umor Necrosis Factor–Associated Periodic Syndrome (T RAPS). Thi s s yndrome al s o pres ent s pri mari l y before age 20, but pat i ent s experi ence l onger and l es s regul ar epi s odes of fever and al s o devel op t ender deep eryt hemat ous pat ches and pai nful conjunct i val i nject i on.
3. Hyper IgD Syndrome (HIDS). Cl i ni cal s ympt oms us ual l y s t art wi t hi n t he fi rs t 6 mont hs of l i fe. Al ong wi t h fever, t here i s s i gni fi cant abdomi nal pai n, oft en wi t h di arrhea, cervi cal l ymphadenopat hy, macul opapul ar ras h, and ol i goart hri t i s .
4. Familial Cold Auto-inflammatory Syndrome (FCAS). Sympt oms s t art duri ng i nfancy wi t h a s hort -l i ved (<24 hours ) macul opapul ar ras h, conjunct i val i nject i on, and i rri t abi l i t y wi t hout fever, al l of whi ch i s us ual l y t ri ggered by expos ure t o col d t emperat ures .
5. Muckle Wells Syndrome (MWS). Cl i ni cal mani fes t at i ons i ncl ude an urt i cari al -l i ke but not pruri t i c erupt i on, s evere di s t al l i mb pai ns , and t he gradual devel opment of s ens ori neural deafnes s .
6. Neonatal-Onset Multisystem Inflammatory Disease (NOMID). Thi s di s order not uncommonl y pres ent s duri ng t he neonat al peri od wi t h an urt i cari a-l i ke ras h, t he gradual devel opment of s evere art hropat hy wi t h ext ens i ve cart i l age overgrowt h, chroni c meni ngi t i s , and s ens ori neural heari ng l os s .
Pa g e 2 5 7 4
ABC Ambe r CHM Conve rte r Tria l ve rsion, http://w w w .proce sste x t.com/a bcchm.html
FCAS, MW S, and NOMID are now cons i dered t o be a fami l y of di s orders wi t h a s pect rum of progres s i ve cl i ni cal s everi t y and common mut at i ons of t he CI AS1 gene at t he q44 l ocus of chromos ome 1. P.509
Study Questions/Answers and Explanations 1. A 56-year-old women presents to your office with pain and stiffness in her hands. She states that the stiffness seems to last the entire morning. Which of the following joint findings is most suggestive of an inflammatory arthritis, rather than osteoarthritis, as the cause of her joint pain? A. Pai nful range of mot i on B. Crepi t us C. Bony art i cul ar enl argement D. Swel l i ng and warmt h E. Ins t abi l i t y Vi ew Ans wer 1. T he answer is D [IA]. A s wol l en and warm joi nt i s more l i kel y t o be affect ed by an i nfl ammat ory art hri t i s t han by os t eoart hri t i s . The pres ence of s ynovi al fl ui d i s more commonl y as s oci at ed wi t h i nfl ammat ory art hri t i s t han os t eoart hri t i s , and warmt h s ugges t s s ome degree of i nfl ammat i on. Os t eoart hri t i s t ypi cal l y i s as s oci at ed wi t h bony joi nt enl argement i n res pons e t o cart i l age and s ubchondral bone i njury. Pai nful joi nt range of mot i on, joi nt crepi t us , and joi nt i ns t abi l i t y coul d occur i n ei t her an i nfl ammat ory or os t eoart hri t i c joi nt probl em. 2. A 45-year-old man with a history of hypertension complains of left great toe pain of 24 hours' duration. He has had a low-grade fever and chills. He has no history of joint problems. T he examination is notable for a red, warm, swollen, left great toe. No other joints are involved. T here are no tophi. How is a definitive diagnosis made? A. Obt ai n an x-ray
Pa g e 2 5 7 5
ABC Ambe r CHM Conve rte r Tria l ve rsion, http://w w w .proce sste x t.com/a bcchm.html
B. Obt ai n fl ui d for s ynovi al anal ys i s C. Obt ai n a s erum uri c aci d l evel D. Obt ai n bl ood cul t ures E. Obt ai n HLA-B27 Vi ew Ans wer 2. T he answer is B [V A 6 a (2)]. The li kel y di agnos es are gout or ps eudogout . The onl y way t o make a defi ni t i ve di agnos i s i s t o perform a di agnos t i c art hrocent es i s and eval uat e t he fl ui d under t he pol ari zed mi cros cope for t he pres ence of crys t al s . X-rays i n pat i ent s wi t h l ong-s t andi ng s ympt oms can have eros i ons charact eri s t i c of gout , but t hey are us ual l y normal i n t hi s s et t i ng, wi t h t hi s bei ng hi s fi rs t at t ack. Hyperuri cemi a can be a ri s k fact or for gout , but not everyone wi t h hyperuri cemi a devel ops gout . In addi t i on, duri ng an acut e at t ack t he uri c aci d l evel may be fal s el y l ow. Thus , i n t hi s pat i ent wi t h acut e s ympt oms , t he uri c aci d l evel woul d not be t hat hel pful . The t oes can be i nvol ved i n t he pres ent at i on of a s pondyl oart hropat hy, but us ual l y as a s aus age di gi t . He has no ot her feat ures of s pondyl oart hropat hy. HLA-B27 t es t i ng woul d not be hel pful . 3. Which one of the following forms of juvenile idiopathic arthritis is most likely to be associated with serious eye complications? A. Pol yart i cul ar art hri t i s t hat i s s eropos i t i ve for rheumat oi d fact or B. Pol yart i cul ar art hri t i s t hat i s s eronegat i ve for rheumat oi d fact or C. Ol i go-art i cul ar art hri t i s wi t hout axi al s pi ne i nvol vement D. Ol i go-art i cul ar art hri t i s wi t h axi al s pi ne i nvol vement E. Sys t emi c-ons et juveni l e rheumat oi d art hri t i s Vi ew Ans wer 3. T he answer is C [XIII E]. Pat i ent s wi t h ol i goart i cul ar art hri t i s wi t hout axi al s pi ne i nvol vement are mos t l i kel y t o devel op chroni c and pot ent i al l y s evere ant eri or uvei t i s , whi ch can be cl i ni cal l y qui t e s ubt l e even as i t l eads t o progres s i ve vi s ual l os s . Up t o 25% of pat i ent s i n t hi s s ubs et may devel op ant eri or uvei t i s , and t he group t hat i s ant i nucl ear ant i body (ANA)–pos i t i ve appears t o be at hi ghes t ri s k. Pat i ent s wi t h axi al s pi ne i nvol vement can al s o devel op
Pa g e 2 5 7 6
ABC Ambe r CHM Conve rte r Tria l ve rsion, http://w w w .proce sste x t.com/a bcchm.html
ant eri or uvei t i s , but t hi s t ends t o be acut e, s el f-l i mi t ed, and eas i l y t reat abl e. 4. An 18-year-old woman comes to the emergency department complaining of severe right knee, right wrist, and left ankle pain. She has several skin lesions on her arms and legs; some are petechial and others are vesiculopustular. Physical examination also reveals tenderness and swelling of tendons around the involved joints but no actual joint swelling. Which of the following tests is most likely to yield the diagnosis? A. Pel vi c exami nat i on and cervi cal cul t ure B. Joi nt fl ui d as pi rat i on C. Ant i nucl ear ant i body (ANA) t es t i ng D. Rheumat oi d fact or t es t i ng E. St rept ococcal enzyme t es t i ng Vi ew Ans wer 4–5. T he answers are: 4-A [VII D 1], 5-C [VII F 1]. Thi s pat i ent has cl i ni cal feat ures s ugges t i ve of gonococcal peri art hri t i s –der mat i t i s s yndrome. In t hi s s et t i ng, t he cervi x woul d be t he mos t l i kel y s i t e for a pos i t i ve cul t ure. Joi nt fl ui d as pi rat i on coul d yi el d a pos i t i ve cul t ure, but t he pat i ent does not have joi nt effus i ons . Ant i nucl ear ant i body (ANA) t es t i ng woul d be hel pful i f t he fi ndi ngs were more cons i s t ent wi t h s ys t emi c l upus eryt hemat os us (SLE) t han wi t h gonorrhea. A pat i ent wi t h rheumat oi d art hri t i s woul d be unl i kel y t o have t he s ki n fi ndi ngs , s o rheumat oi d fact or i s not a hel pful t es t . Di fferent s ki n and art i cul ar fi ndi ngs woul d be pres ent i n rheumat i c fever, s o s t rept ococcal enzyme t es t i ng woul d be unl i kel y t o yi el d a di agnos i s . Becaus e gonorrhea i s t he mos t l i kel y di agnos i s , ant i bi ot i c t reat ment i s requi red. Cort i cos t eroi ds are cont rai ndi cat ed. Nons t eroi dal ant i -i nfl ammat ory agent s (NSAIDs ), l ocal s ki n care, and s pl i nt i ng of joi nt s may be us eful anci l l ary meas ures but do not t reat t he pri mary probl em of a bact eri al i nfect i on. 5. While awaiting results of laboratory testing, the patient in question 4 should receive which of the following treatments? A. Cort i cos t eroi ds
Pa g e 2 5 7 7
ABC Ambe r CHM Conve rte r Tria l ve rsion, http://w w w .proce sste x t.com/a bcchm.html
B. Nons t eroi dal ant i -i nfl ammat ory drugs (NSAIDs ) C. Ant i bi ot i cs D. Local care of s ki n l es i ons E. Spl i nt i ng of pai nful joi nt s Vi ew Ans wer 4–5. T he answers are: 4-A [VII D 1], 5-C [VII F 1]. Thi s pat i ent has cl i ni cal feat ures s ugges t i ve of gonococcal peri art hri t i s –dermat i t i s s yndrome. In t hi s s et t i ng, t he cervi x woul d be t he mos t l i kel y s i t e for a pos i t i ve cul t ure. Joi nt fl ui d as pi rat i on coul d yi el d a pos i t i ve cul t ure, but t he pat i ent does not have joi nt effus i ons . Ant i nucl ear ant i body (ANA) t es t i ng woul d be hel pful i f t he fi ndi ngs were more cons i s t ent wi t h s ys t emi c l upus eryt hemat os us (SLE) t han wi t h gonorrhea. A pat i ent wi t h rheumat oi d art hri t i s woul d be unl i kel y t o have t he s ki n fi ndi ngs , s o rheumat oi d fact or i s not a hel pful t es t . Di fferent s ki n and art i cul ar fi ndi ngs woul d be pres ent i n rheumat i c fever, s o s t rept ococcal enz yme t es t i ng woul d be unl i kel y t o yi el d a di agnos i s . Becaus e gonorrhea i s t he mos t l i kel y di agnos i s , ant i bi ot i c t reat ment i s requi red. Cort i cos t eroi ds are cont rai ndi cat ed. Nons t eroi dal ant i -i nfl ammat ory agent s (NSAIDs ), l ocal s ki n care, and s pl i nt i ng of joi nt s may be us eful anci l l ary meas ures but do not t reat t he pri mary probl em of a bact eri al i nfect i on. 6. A 50-year-old woman complains of a 2-month history of her hands becoming painful and turning white or blue in the cold; progressive skin tightness and thickening of fingers, hands, and forearms; shortness of breath on exertion; and a sensation of lower chest burning and food sticking on swallowing. Antibody testing shows the presence of antinuclear antibody (ANA) and elevated titers P.510 of antibody to Scl-70. Which of the following pathogenetic explanations best fits this patient' s illness? A. Infi l t rat i on of mucopol ys acchari des i nt o underl yi ng s ubepi t hel i al t i s s ues
Pa g e 2 5 7 8
ABC Ambe r CHM Conve rte r Tria l ve rsion, http://w w w .proce sste x t.com/a bcchm.html
B. Unregul at ed fi brobl as t i c col l agen s ynt hes i s C. Raynaud's phenomenon l eadi ng fi rs t t o i s chemi a and l at er t o t i s s ue fi bros i s D. Vas cul ar endot hel i al damage and i mmunol ogi cal l y medi at ed t i s s ue fi bros i s E. Carci nomat ous paraneopl as t i c proces s Vi ew Ans wer 6. T he answer is D [IX E]. Thi s pat i ent pres ent s wi t h charact eri s t i c feat ures of s cl eroderma, a chroni c i l l nes s i n whi ch unregul at ed i mmunol ogi c proces s es (perhaps t ri ggered by unknown envi ronment al ant i gens ) caus e s mal l ves s el endot hel i al damage and wi des pread dermal and i nt ernal organ fi bros i s . The s mal l ves s el endot hel i al damage l eads t o s econdary vas cul ar react i vi t y (Raynaud's phenomenon) and, pos s i bl y, i s chemi c t i s s ue damage. The i ncreas ed col l agen s ynt hes i s by t i s s ue fi brobl as t s , whi ch l eads t o wi des pread fi bros i s , i s not unregul at ed; rat her, i t i s caus ed by cyt oki ne and growt h fact or s ecret i on from l ymphocyt es , mas t cel l s , and pl at el et s . There i s no evi dence t hat pat i ent s wi t h s cl eroderma have t i s s ue mucopol ys acchari de i nfi l t rat i on or t hat t here are t umors res pons i bl e for paraneopl as t i c dermal fi bros i s . 7. Which of the following manifestations is more likely to be found in the diffuse form of systemic sclerosis than in the CREST variant? A. Es ophageal mot i l i t y dys funct i on B. Pul monary i nvol vement C. Di s t al s ki n t hi ckeni ng D. Renal di s eas e E. Tel angi ect as i as Vi ew Ans wer 7. T he answer is D [IX E 5]. Of t he cl i ni cal mani fes t at i ons l i s t ed, onl y renal di s eas e i s more l i kel y t o be found i n di ffus e s ys t emi c s cl eros i s t han i n t he CREST s yndrome, whi ch i nvol ves t he co-exi s t ence of s ubcut aneous cal ci nos i s , Raynaud's phenomenon, e s ophageal mot i l i t y dys funct i on, scl erodact yl y, and tel angi ect as i a. Al s o, pul monary i nt ers t i t i al fi bros i s t ypi cal l y wors ens fas t er i n t he
Pa g e 2 5 7 9
ABC Ambe r CHM Conve rte r Tria l ve rsion, http://w w w .proce sste x t.com/a bcchm.html
di ffus e form. Bot h forms are charact eri zed by es ophageal mot i l i t y dys funct i on, di s t al s ki n t hi ckeni ng, and Raynaud's phenomenon. Tel angi ect as i as al s o can occur i n bot h forms of s cl eroderma, al t hough t hey are more common and wi des pread i n t he CREST s yndrome. 8. Which of the following therapies is essential for treating polymyositis? A. Ant i mal ari al drugs B. Nons t eroi dal ant i -i nfl ammat ory drugs (NSAIDs ) C. Cort i cos t eroi ds D. Bed res t E. Aerobi c exerci s e Vi ew Ans wer 8. T he answer is C [X G 1]. Oral cort i cos t eroi ds are t he t ypi cal i ni t i al t reat ment for i nfl ammat ory myopat hy. Nons t eroi dal ant i -i nfl ammat ory agent s (NSAIDs ), ant i mal ari al drugs , bed res t , and aerobi c exerci s e are not recogni zed t reat ment s for pol ymyos i t i s , al t hough NSAIDs mi ght hel p as s oci at ed joi nt compl ai nt s and ant i mal ari al drugs have been us ed for t he s ki n feat ures of dermat omyos i t i s . Aerobi c exerci s e i s appropri at e for pat i ent s wi t h cont rol l ed di s eas e but mi ght be det ri ment al t o t hos e wi t h act i ve di s eas e. 9. A 32-year-old man presents with back pain with prominent stiffness lasting several hours, in addition to a painful swollen left ankle. T he examination shows limited motion of the spine and a left Achilles tendinitis. You are considering a spondyloarthropathy as a unifying diagnosis. Which of the following clinical features is typical of all spondyloarthropathies? A. Ent hes opat hi c i nfl ammat i on B. Uret hri t i s C. Ski n l es i ons D. Bowel i nfl ammat i on E. Oral ul cers Vi ew Ans wer 9. T he answer is A [IV A 3 a]. Ent hes opat hi c i nfl ammat i on (i .e.,
Pa g e 2 5 8 0
ABC Ambe r CHM Conve rte r Tria l ve rsion, http://w w w .proce sste x t.com/a bcchm.html
i nfl ammat ory l es i ons at l i gament ous , cart i l agi nous , and t endi nous at t achment s t o bone) i s a charact eri s t i c feat ure of al l s pondyl oart hropat hi es . Uret hri t i s and oral ul cers are t ypi cal of react i ve art hri t i s (Rei t er's s yndrome), and s ki n l es i ons are s een i n react i ve and ps ori at i c art hri t i s . Some pat i ent s wi t h i nfl ammat ory bowel di s eas e al s o have s ki n l es i ons (e.g., pyoderma gangrenos um), but t he mos t t ypi cal feat ure ot her t han rheumat i c compl ai nt s i s bowel i nfl ammat i on. 10. A 57-year-old previously well man has been ill for 2 months with fatigue, malaise, dyspnea on exertion, abdominal pain, and progressive numbness in his feet. He has lost 20 pounds during this period. Recently, he developed mild inflammatory polyarthritis of the hands and has physical signs suggesting a mononeuritis in the right median nerve distribution. Chest radiograph shows cardiomegaly and findings of early pulmonary edema. Which of the following is the most likely diagnosis? A. Hypers ens i t i vi t y vas cul i t i s B. Rheumat oi d art hri t i s C. Sys t emi c l upus eryt hemat os us (SLE) D. Pol yart eri t i s nodos a E. Churg-St raus s s yndrome Vi ew Ans wer 10. T he answer is D [XII D]. Thi s pat i ent has cons t i t ut i onal compl ai nt s (fat i gue, mal ai s e, and 20-l b wei ght l os s ) and has cl i ni cal evi dence of i mpai red funct i oni ng of mul t i pl e organs ; t hes e fi ndi ngs make vas cul i t i s a di s t i nct di agnos t i c pos s i bi l i t y. Mononeuri t i s i s an even more s peci fi c fi ndi ng, s ugges t i ng a s mal l t o medi um-s i zed vas cul i t i s of vas a nervorum. The ot her cl i ni cal fi ndi ngs are compat i bl e wi t h vas cul i t i c organ i mpai rment as wel l and are t ypi cal of pol yart eri t i s nodos a. Hypers ens i t i vi t y vas cul i t i s i s unl i kel y, becaus e pat i ent s wi t h t hi s di s order of t he very s mal l ves s el s al mos t al ways have di s t i nct s ki n fi ndi ngs (e.g., pal pabl e purpura) t o s upport t he di agnos i s . Bot h rheumat oi d art hri t i s and s ys t emi c l upus eryt hemat os us (SLE) are s ys t emi c i l l nes s es t hat can be compl i cat ed by a s mal l t o medi um-s i zed ves s el vas cul i t i s , but
Pa g e 2 5 8 1
ABC Ambe r CHM Conve rte r Tria l ve rsion, http://w w w .proce sste x t.com/a bcchm.html
t he onl y fi ndi ngs s upport i ng t hes e di agnos es are t he mi l d joi nt compl ai nt s . The abs ence of al l ergi c hi s t ory, eos i nophi l i a, and ches t radi ograph i nfi l t rat e di s t i nct from pul monary edema makes Churg-St raus s s yndrome unl i kel y. 11. A 64-year-old woman presents with gradually increasing right knee pain. T he pain is worse with weight bearing and relieved with rest. T he knee is cool to touch. T here is marked crepitus on range of motion, with a trace effusion. Which of the following would help confirm the diagnosis? A. Joi nt as pi rat i on B. Tri al of oral cort i cos t eroi ds C. X-ray wi t h wei ght -beari ng vi ews D. CBC, ESR E. Bone s can Vi ew Ans wer 11. T he answer is C [VI G 3]. The pat i ent has cl as s i c fi ndi ngs of os t eoart hri t i s of her knee. The bes t t es t t o confi rm t he di agnos i s woul d be an x-ray, whi ch us ual l y s hows joi nt s pace l os s , bony s cl eros i s , and degenerat i ve s purs . Al t hough t he pat i ent does have an effus i on, t he knee has no as s oci at ed warmt h; t hus , an as pi rat i on i n not neces s ary. If t here i s concern about a crys t al l i ne di s eas e or i nfect i on, t hen joi nt fl ui d anal ys i s woul d be hel pful . Laborat ory s t udi es s uch as CBC and ESR woul d not be i ndi cat ed i n t hi s pat i ent , becaus e t here are no cons t i t ut i onal s ympt oms , or i nfl ammat ory s i gns or s ympt oms . A bone s can wi l l l i kel y s how degenerat i ve changes , but i s more cos t l y and t i me cons umi ng. Onl y i f an occul t fract ure, avas cul ar necros i s , i nfect i on, mal i gnancy, or an i nfl ammat ory art hri t i s i s s us pect ed woul d a bone s can be hel pful . Oral s t eroi ds have no rol e i n t he t reat ment of t hi s di s order. 12. A 70-year-old man complains of increasing back pain. He has no other history of arthritis or preceding trauma. He can walk about 2 blocks then has to stop because of the low back pain with an associated numbness and tingling extending into his buttocks and left thigh. His examination shows a P.511
Pa g e 2 5 8 2
ABC Ambe r CHM Conve rte r Tria l ve rsion, http://w w w .proce sste x t.com/a bcchm.html
well-appearing man. His peripheral pulses were normal. T here was no spinal tenderness. His straight leg test was negative. His left lower extremity strength, range of motion, sensation, and reflexes were normal. What is the most likely diagnosis? A. Spondyl oart hropat hy B. Herni at ed di s k C. Spi nal s t enos i s D. Os t eoart hri t i s of t he l eft hi p E. Mus cul ar s t rai n Vi ew Ans wer 12. T he answer is C [onl i ne II C 2 a]. The pat i ent pres ent s wi t h cl as s i cal s ympt oms of s pi nal s t enos i s . Ps eudocl audi cat i on i s t he mos t common s ympt om, wi t h pai n wors ened by erect pos t ure, i mproved by forward fl exi on, and as s oci at ed numbnes s and weaknes s . Spondyl oart hropat hi es us ual l y pres ent i n younger i ndi vi dual s wi t h as s oci at ed i nfl ammat ory back pai n (pai n l as t i ng more t han 1 hour). The l ack of a pos i t i ve s t rai ght l eg t es t hel ps el i mi nat e s ci at i ca/herni at ed di s k. Al t hough underl yi ng os t eoart hri t i s of hi s hi p may be cont ri but i ng t o hi s l eft t hi gh di s comfort , i t does not expl ai n hi s back s ympt oms . In addi t i on, on exami nat i on he has good range of mot i on of hi s hi p, maki ng hi p pat hol ogy l es s l i kel y. Hi s s ympt oms are not cons i s t ent wi t h s i mpl e s t rai n, becaus e mos t s t rai n s yndromes i mprove i n 6–8 weeks . Directions: T he res pons e opt i ons for I t ems 13–15 are t he s ame. Y ou w i l l be requi red t o s el ec t one ans w er for eac h i t em i n t he s et . A. Rheumatoid arthritis B. Lyme disease C. Gonococcal arthritis D. Systemic lupus erythematosus (SLE) E. Polymyositis F. Sjögren' s syndrome G. Polymyalgia rheumatica (PMR) H. Reactive arthritis (Reiter' s syndrome) I. Pseudogout
Pa g e 2 5 8 3
ABC Ambe r CHM Conve rte r Tria l ve rsion, http://w w w .proce sste x t.com/a bcchm.html
For each of the following case descriptions, select the most appropriate diagnosis. 13. A 20-year-old woman complains of 2 weeks of fever, pleuritic chest pain, stiffness and swelling in wrists and metacarpophalangeal (MCP) and proximal interphalangeal (PIP) joints, an erythematous rash over both cheeks, and bilateral pretibial edema. 14. A 50-year-old man complains of a gritty sensation in his eyes and dry mouth, which he has experienced for several months. He has vague arthralgias in his hands and knees but only bulge signs (small amount of synovial fluid) in the knees on physical examination. He has scattered purpuric lesions over both calves and ankles. 15. An 80-year-old man complains of right knee pain and swelling. He has bony enlargement of the second and third metacarpophalangeal (MCP) joints bilaterally as well as wrist, proximal interphalangeal (PIP), and distal interphalangeal (DIP) joints. His knee is swollen, and range of motion is moderately limited by pain. A radiograph shows only flecks of calcium in the meniscal cartilage of the knee. Vi ew Ans wer 13–15. T he answers are: 13-D [VII F 1 and T able 10-13], 14-F [V B 4 a], 15-I [V B 5 b]. T his young woman has the sudden onset of a systemic febrile illness that includes findings suggestive of serositis (pleuritic chest pain), arthritis, and a facial skin rash (possibly a butterfly rash). New pretibial edema suggests the possibility of urinary protein loss from renal involvement (glomerulonephritis or nephrotic syndrome). Although laboratory data [including positive antinuclear antibodies (ANAs)] or radiographic support are necessary to confirm possible organ abnormalities, systemic lupus erythematosus (SLE) is the most likely cause of these findings. T his man likely has primary Sjögren' s syndrome. T he dry eyes and dry mouth suggest lacrimal and salivary gland involvement in this disorder, and further ophthalmologic evaluation or labial
Pa g e 2 5 8 4
ABC Ambe r CHM Conve rte r Tria l ve rsion, http://w w w .proce sste x t.com/a bcchm.html
salivary gland biopsy could confirm these suspicions. Arthralgias are a common feature of primary Sjögren' s syndrome; the arthritis of rheumatoid arthritis should have more actual joint findings (tenderness, swelling, or limitation of motion) to be considered as a primary disease accompanied by a secondary form of Sjögren' s syndrome. T he purpuric lesions on the calves suggest the possibility of a small vessel vasculitis, a common skin feature of primary Sjögren' s syndrome. T he man' s presentation is most consistent with a chronic degenerative arthritis and a superimposed acute inflammatory arthritis. T he degenerative, bony changes involve unusual joints [wrist, metacarpophalangeals (MCPs)] and include an unusual feature (chondrocalcinosis on knee radiograph). T hese findings are most compatible with calcium pyrophosphate deposition (CPPD) disease causing atypical degenerative arthritis, and the superimposed acute knee inflammation (pseudogout) is likely caused by release of the CPPD crystals into the knee joint. Finding positively birefringent crystals under red-compensated, polarized light examination of synovial fluid would be diagnostic. 16. A 25-year-old woman presents to your office with a month history of fatigue, generalized arthralgias, and photosensitivity. Her examination shows an erythematous rash sparing her nasolabial folds. Her lab tests indicated a normal CBC and comprehensive metabolic profile. Her ESR was elevated at 40, ANA was positive at 1:320 speckled, SSA was borderline positive. Her urinalysis was normal. T he best treatment now would be A. Hydroxychl oroqui ne B. NSAIDs C. St eroi ds D. Suns creen E. Cycl ophos phami de Vi ew Ans wer 16. T he answer is A [VII H3]. The pat i ent has SLE, wi t h t he mai n mani fes t at i ons bei ng fat i gue, art hral gi as , and t he mal ar ras h and
Pa g e 2 5 8 5
ABC Ambe r CHM Conve rte r Tria l ve rsion, http://w w w .proce sste x t.com/a bcchm.html
phot os ens i t i vi t y; al l of whi ch woul d res pond t o hydroxychl oroqui ne. Al t hough NSAIDs may i mprove her joi nt s ympt oms , t hey have no effect on t he ras h. Li kewi s e, s uns creen woul d be hel pful i n cont rol l i ng t he s ki n di s eas e and pos s i bl y prevent i ng di s eas e fl ares , but woul d not di rect l y i mprove her fat i gue or art hral gi as . There i s no evi dence of major organ i nvol vement ; t hus , s t eroi ds or cyt ot oxi c t herapy have no rol e i n t hi s pat i ent at t hi s poi nt . 17. A 35-year-old preschool teacher presents with a 2 -week history of malaise, low-grade fever, and a symmetric polyarthritis. Her examination shows synovitis affecting her wrists, MCPs, and PIPs bilaterally. Her CBC and comprehensive metabolic profile are normal. T he ANA and RF are negative. T he most likely diagnosis would be A. Parvovi rus B19 i nfect i on B. Pol ymyal gi c rheumat i ca (PMR) C. SLE D. Scl eroderma E. Os t eoart hri t i s Vi ew Ans wer 17. T he answer is A [XIV G]. Gi ven t hat t he pat i ent i s expos ed t o young chi l dren and has an acut e (l es s t han 6 week) s ymmet ri c pol yart hri t i s , Parvovi rus B19 i s t he mos t l i kel y di agnos i s at t hi s poi nt . Pol ymyal gi c rheumat i ca us ual l y pres ent s wi t h proxi mal s houl der and pel vi c gi rdl e pai n and s t i ffnes s and does not occur i n t hi s age group. SLE coul d be a cons i derat i on i f her s ympt oms were t o pers i s t , but t he fact t hat her ANA i s negat i ve makes SLE very unl i kel y. She has no feat ures of s cl eroderma. Os t eoart hri t i s woul d be unl i kel y bas ed on her age and t he fact t hat s he has i nfl ammat ory joi nt fi ndi ngs on exami nat i on (s ynovi t i s ). P.512
18. A 56- year-old man presents with a several month history of aching and stiffness of the neck, shoulder, and hip girdle; low grade fever; and malaise. Over the last few days, he has noted
Pa g e 2 5 8 6
ABC Ambe r CHM Conve rte r Tria l ve rsion, http://w w w .proce sste x t.com/a bcchm.html
a persistent right frontal headache and jaw claudication. Examination of the right temporal artery reveals a diminished pulse and slight tenderness. A stat erythrocyte sedimentation rate (ESR) returns 92 mm/hour (normal <20 mm/hour). Your most appropriate course of action would be the following: A. Tri al of an NSAID B. Tri al of predni s one @10 mg/day for 10 days C. Referral t o opht hal mol ogy cl i ni c t o s chedul e a t emporal art ery bi ops y D. Immedi at el y begi n predni s one at 1 mg/kg per day and refer t he pat i ent for a t emporal art ery bi ops y ASAP E. Immedi at el y obt ai n an enhanced MRI s can of t he brai n Vi ew Ans wer 18. T he answer is D [XII E 5 b]. The pat i ent pres ent s wi t h a cl as s i c pres ent at i on for PMR as s oci at ed wi t h gi ant cel l art eri t i s (GCA), s ugges t ed by t he pers i s t ent front al headache, t ender t emporal art ery and mos t s peci fi cal l y, jaw cl audi cat i on. The mos t appropri at e cours e of act i on i s t o t reat t he pat i ent wi t h hi gh-dos e predni s one (1mg/kg) t o prevent bl i ndnes s and obt ai n a t emporal art ery bi ops y as s oon as pos s i bl e t o document t he di agnos i s . A t emporal art ery bi ops y may s t i l l be di agnos t i c up t o 1 week aft er i ni t i at i on of s t eroi ds . A t ri al of an NSAID or l ow-dos e predni s one woul d be appropri at e for PMR al one, but t he pres ence of headache and jaw cl audi cat i on i n t hi s pat i ent s t rongl y s ugges t s underl yi ng GCA. Referral t o opht hal mol ogy cl i ni c and wai t i ng for t he res ul t s of a t emporal art ery bi ops y coul d l ead t o bl i ndnes s whi l e t he pat i ent remai ns unt reat ed. An enhanced MRI has no rol e i n t he di agnos i s of GCA i n t hi s pat i ent wi t h a cl as s i c hi s t ory. 19. A 48-year-old woman with established rheumatoid arthritis (RA) is referred to you for a second opinion regarding therapeutic options. She is concerned about treatment of her RA because of a recent positive tuberculin test noted on her yearly physical. She has no symptoms and her chest x-ray is clear. What DMARD(s) (disease modifying anti-rheumatic drug) has been associated with reactivation of T B?
Pa g e 2 5 8 7
ABC Ambe r CHM Conve rte r Tria l ve rsion, http://w w w .proce sste x t.com/a bcchm.html
A. Met hot rexat e B. Lefl unomi de C. Hydroxychl oroqui ne s ul fat e D. TNF i nhi bi t ors E. Met hot rexat e combi ned wi t h s ul fas al azi ne Vi ew Ans wer 19. T he answer is D [III G 2 d and T able 10-7]. Tumor necros i s fact or (TNF) has been s hown t o be neces s ary i n granul oma format i on t hat i s cri t i cal i n t he cont rol of TB. Thus , us e of TNF i nhi bi t ors (et anercept , i nfl i xi mab, and adal i mumab) have been as s oci at ed wi t h react i vat i on of TB. Screeni ng for TB i s recommended before i ni t i at i ng t herapy wi t h t hes e agent s . Treat ment wi t h ant i t ubercul ous medi cat i ons i s recommended for i ndi vi dual s who are PPD pos i t i ve pri or t o cons i deri ng t reat ment wi t h t hes e agent s .
Pa g e 2 5 8 8
ABC Ambe r CHM Conve rte r Tria l ve rsion, http://w w w .proce sste x t.com/a bcchm.html
Editors: Wolfsthal, Susan T itle: NMS Medicine, 6th Edition Copyri ght ©2008 Li ppi ncot t W i l l i ams & W i l ki ns > T able of Cont ent s > Chapt er 11 - Neurologic Disorders
Chapter 11
Neurologic Disorders Kerri Kissel Neil Porter
I. Approach to the Patient with a Neurologic Complaint A. Patient history The pat i ent hi s t ory i s t he corners t one of neurol ogi c as s es s ment .
1. Key questions. Ques t i ons t hat may hel p di rect t he pat i ent i nt ervi ew i ncl ude: o
o
a. W as t he ons et of s ympt oms gradual or s udden?
o
o
b. Are t he s ympt oms s t at i c, i nt ermi t t ent , or progres s i ve?
o
o
c. Has t he probl em remai ned l i mi t ed i n s cope, or have new feat ures been i nt roduced over t i me?
o
o
d. W hat concurrent probl ems does t he pat i ent have, and what medi cat i ons or drugs are bei ng us ed?
o
o
e. Is t here a fami l y hi s t ory of t he di s order or
Pa g e 2 5 8 9
ABC Ambe r CHM Conve rte r Tria l ve rsion, http://w w w .proce sste x t.com/a bcchm.html
predi s pos i ng condi t i ons ? o
o
f. W hat habi t s and t oxi n expos ures mi ght t he pat i ent have?
2. Review of symptoms. Dependi ng on t he cl i ni cal compl ai nt , a pat i ent s houl d be as ked whet her t here i s any hi s t ory of: o
o
a. Headache or t rauma t o t he head, neck, or s pi ne
o
o
b. Los s of cons ci ous nes s , convul s i ve act i vi t y, mood al t erat i ons , confus i on, or memory di s t urbances
o
o
c. Impai red or doubl e vi s i on, faci al numbnes s or weaknes s , i mpai red heari ng or s wal l owi ng, or abnormal s peech
o
o
d. Arm or l eg weaknes s or heavi nes s , s l ownes s of movement , al t ered l i mb s ens at i on, di s comfort or t i ngl i ng i n t he ext remi t i es
o
o
e. Cl ums i nes s , fal l i ng, or di zzi nes s
o
o
f. Bowel or bl adder di s t urbances or s exual dys funct i on
B. Neurologic examination From t he pat i ent hi s t ory, t he phys i ci an can generat e a s eri es of
Pa g e 2 5 9 0
ABC Ambe r CHM Conve rte r Tria l ve rsion, http://w w w .proce sste x t.com/a bcchm.html
di agnos t i c hypot hes es t hat can be t es t ed wi t h a focus ed neurol ogi c exami nat i on. Anat omi c l ocal i zat i on of t he pat hol ogy wi t hi n t he nervous s ys t em i s es s ent i al t o t hi s proces s (Fi gure 11-1).
1. Mental status. If t he pat i ent 's ment al s t at us i s abnormal , t he hi s t ory and t hos e component s of t he phys i cal exami nat i on t hat depend on pat i ent cooperat i on mus t be approached wi t hi n t he proper cont ext . For exampl e, i f t he pat i ent i s confus ed, t he s ens ory exami nat i on may be unrel i abl e. o
o
a. The pat i ent 's l evel of arous al , ori ent at i on, s hort - and l ong-t erm memory, affect (i .e., mood), concent rat i on and at t ent i on, fund of knowl edge, i ns i ght , judgment , and cons t ruct i onal abi l i t y s houl d be as s es s ed.
o
o
b. Li ngui s t i c abi l i t i es are eval uat ed by exami ni ng comprehens i on, repet i t i on, fl uency, nami ng, readi ng, and wri t i ng.
o
o
c. The i nt egri t y of ot her cort i cal funct i ons (e.g., graphes t hes i a, s t ereognos i s , t wo-poi nt di s cri mi nat i on, ri ght –l eft ori ent at i on, and negl ect ) s houl d be exami ned i f pari et al l obe dys funct i on i s s us pect ed.
2. Cranial nerves. Exami nat i on of crani al nerves II–XII i s neces s ary (Tabl e 11-1). o
o
a. In part i cul ar, vi s ual acui t y and fi el ds s houl d be
Pa g e 2 5 9 1
ABC Ambe r CHM Conve rte r Tria l ve rsion, http://w w w .proce sste x t.com/a bcchm.html
checked; t he opt i c nerve s houl d be exami ned; and abnormal i t i es of ocul ar mot i l i t y, i ncl udi ng nys t agmus and dys met ri a, s houl d be document ed (Tabl e 11-2). o
o
b. Abnormal i t i es of faci al s ens at i on (i ncl udi ng t he corneal refl ex) and movement al s o s houl d be i nves t i gat ed.
P.517
Pa g e 2 5 9 2
ABC Amber CHM Converter Trial version, http://www.processtext.com/abcchm.html
FIGURE 11-1 Summary of s ome of t he out s t andi ng neurol ogi c s i gns and s ympt oms t hat occur wi t h focal des t ruct i ve l es i ons i n t he ri ght or l eft cerebral hemi s phere as det ect ed on neurol ogi c exami nat i on. (A) Lat eral vi ew of t he l eft cerebral hemi s phere. (B ) Lat eral vi ew of t he ri ght cerebral hemi s phere. (Repri nt ed from NMS Neuroanat omy. Mal vern, PA: Harwal Publ i s hi ng, 1988:314 ).
TABLE 11-1 Twelve Cranial Nerves C N F N er u v n e ct io n I Ol S fa m ct el or l y II O Vi pt s i ic o n II O Ey I cu e lo m m ov ot e or m
Page 2593
ABC Ambe r CHM Conve rte r Tria l ve rsion, http://w w w .proce sste x t.com/a bcchm.html
e nt s IV Tr Ey oc e hl d e e ar pr es si o n ( w h e n a d d uc te d) V Tr Fa i g ci e al m se i n ns al at io n VI A Ey b e d a uc b e d
Pa g e 2 5 9 4
ABC Ambe r CHM Conve rte r Tria l ve rsion, http://w w w .proce sste x t.com/a bcchm.html
ns uc ti o n VI Fa Fa I ci ci al al m ov e m e nt VI V H II es e t i ar b in ul g oc oc hl e ar IX Gl P os al s o at p al h se ar ns yn at g io e n al X V P a al g at us al
Pa g e 2 5 9 5
ABC Ambe r CHM Conve rte r Tria l ve rsion, http://w w w .proce sste x t.com/a bcchm.html
m ov e m e nt XI S S pi h n o al ul ac d ce er ss sh or ru y g XI H T I yp o o n gl g os u sa e l
pr ot ru si o n
CN, crani al nerve. P.518
TABLE 11-2 Innervation of the Eye by
Pa g e 2 5 9 6
ABC Ambe r CHM Conve rte r Tria l ve rsion, http://w w w .proce sste x t.com/a bcchm.html
Its Six Nerves N In Cl u n in m er ic b v al er at Ef a io fe n n ct d
s
N
of
a
In
m
te
e
rr
of
u
N
pt
er
io
v
n
e
of N er v
e Effere nt C St Di N ri pl II at o I e pi (o d a, cu m ey l o us e m cl a ot e: b or s u d
Pa g e 2 5 9 7
ABC Ambe r CHM Conve rte r Tria l ve rsion, http://w w w .proce sste x t.com/a bcchm.html
n p uc er er t e ve i o d ) r, a m n e d di t u al rn ,
e
a d n d d o in w fe n ri Pt or os re i s ct (p i ; ar i n al fe ys ri i s or of o vo bl l i t iq io u n e al Le l i va d t o el r ev p at al i o p n)
Pa g e 2 5 9 8
ABC Ambe r CHM Conve rte r Tria l ve rsion, http://w w w .proce sste x t.com/a bcchm.html
e P br u a pi e l S di m la o te ot d h a m n us d cl fi e: xe p d u to pi l i ll g oc ht o Lo ns s s t ri of ct l e or ns Ci t h li ic ar ke y ni m n us g cl e C St Di N ri pl IV at o (t e pi
Pa g e 2 5 9 9
ABC Ambe r CHM Conve rte r Tria l ve rsion, http://w w w .proce sste x t.com/a bcchm.html
ro d a, ch m m l e us os ar cl t n e: s e er s u ve ve p re ) er o io n r lo o ok bl i n iq g u d e o w n a n d in ; ey e ex to rt e d; h e a d til te
Pa g e 2 6 0 0
ABC Ambe r CHM Conve rte r Tria l ve rsion, http://w w w .proce sste x t.com/a bcchm.html
d to si d e o p p os it e p ar al yz e d ey e C St Di N ri pl VI at o (a e pi b d a, d m m uc us os e cl t ns e: s e n l a ve er t e re ve ra o ) l
n
re l o ct ok u in
Pa g e 2 6 0 1
ABC Ambe r CHM Conve rte r Tria l ve rsion, http://w w w .proce sste x t.com/a bcchm.html
us g to si d e of p ar al ys is ; ey e tu rn e d in (a d d uc te d) C S H ar m or ot o n i d ot er s y h 's m m sy p us n at cl dr h e; o et s u m
Pa g e 2 6 0 2
ABC Ambe r CHM Conve rte r Tria l ve rsion, http://w w w .proce sste x t.com/a bcchm.html
ic p e n er (p er i o t o ve r s i ta s, rs m al i o a si n s, d h p e u m pi i f l l ac o ia di l la a to n r hi dr os is , va so di la ti o n) Affere nt C Fr Bl N o in II m d
Pa g e 2 6 0 3
ABC Ambe r CHM Conve rte r Tria l ve rsion, http://w w w .proce sste x t.com/a bcchm.html
(o re n pt t i es ic n s n a er ve ) C C A N or n V n es (t e t h ri al es g /c i a e o of m nj co i n u rn al nc e n ti a er va wi ve l
th
) af l o fe s s re of nt co s rn e al re fl ex Adapt e d from NMS Neuroa nat om
Pa g e 2 6 0 4
ABC Ambe r CHM Conve rte r Tria l ve rsion, http://w w w .proce sste x t.com/a bcchm.html
y. Mal ver n, PA: Harwal Publ i s hi ng, 1988:2 19.
3. Sensory system. Regi ons of abnormal t ouch, pai n (es t i mat ed by pi npri ck), t emperat ure, vi brat i on, and propri ocept i on s houl d be defi ned. o
o
a. Are t he fi ndi ngs confi ned t o one s i de of t he body, t he di s t ri but i on of one or more dermat omes , or t he t erri t ory of one or more peri pheral nerves ?
o
o
b. Are t he s ens ory changes found i n a “s t ocki ng–gl ove― di s t ri but i on?
4. Motor system o
o
a. The pat i ent 's strength s houl d be defi ned as i t pert ai ns t o i ndi vi dual mus cl es or groups of mus cl es . One convent i onal met hod of gradi ng mus cl e s t rengt h for purpos es of compari s on and des cri pt i on i s s hown i n Tabl e 11-3.
o
Pa g e 2 6 0 5
ABC Ambe r CHM Conve rte r Tria l ve rsion, http://w w w .proce sste x t.com/a bcchm.html o
b. A pronat or dri ft can be as s es s ed by havi ng pat i ent s ext end t hei r arms (pal m upward) wi t h t hei r eyes cl os ed. Any depres s i on or pronat i on i s s i gni fi cant .
o
o
c. The pres ence of atrophy, fasciculations, spasticity, and rigidity s houl d be not ed.
o
o
d. The pat i ent 's abi l i t y t o perform rapid alternating and other complex maneuvers s houl d be det ermi ned.
o
o
e. The pat i ent 's stance and gait s houl d be eval uat ed.
5. Coordination. Fi nger-t o-nos e and heel -t o-s hi n t es t i ng s houl d be performed. The phys i ci an s houl d l ook for Romberg' s sign (i .e., s wayi ng or fal l i ng when s t andi ng wi t h eyes cl os ed and feet cl os e t oget her).
6. Muscle stretch reflexes. The act i vi t y and s ymmet ry of t he brachi oradi al i s (C5, C6), bi ceps (C5, C6), t ri ceps (C7, C8), knee (L3, L4), and ankl e (S1, S2) refl exes s houl d be det ermi ned. The pres ence of t he Babinski response s houl d be as s es s ed wi t h pl ant ar s t i mul at i on.
TABLE 11-3 Medical Research Council of Great Britain Muscle Strength Grading Scale
Pa g e 2 6 0 6
ABC Ambe r CHM Conve rte r Tria l ve rsion, http://w w w .proce sste x t.com/a bcchm.html
Gr Eq ad uiv e ale nt Pa tie nt Abi lity 5/5 Nor ma l abi lity 4/5 Abi lity to ove rco me gra vi t y an d so me res ist anc e im pos ed by
Pa g e 2 6 0 7
ABC Ambe r CHM Conve rte r Tria l ve rsion, http://w w w .proce sste x t.com/a bcchm.html
t he exa mi ner 3/5 Abi lity to ove rco me gra vi t y onl y 2/5 Abi lity to mo ve wi t h gra vi t y el i mi nat ed 1/5 Onl y a fl i c ker of mo ve
Pa g e 2 6 0 8
ABC Ambe r CHM Conve rte r Tria l ve rsion, http://w w w .proce sste x t.com/a bcchm.html
me nt 0/5 Co mp l et e i na bi l i ty to mo ve P.519
C. Neurodiagnostic studies
1. Cerebrospinal fluid (CSF) evaluation o
o
a. Indications. St udy of t he CSF can provi de i nformat i on about i nt racrani al pres s ure (ICP) and i nfect i on, bl eedi ng, mal i gnancy, and s t eri l e i nfl ammat i on wi t hi n t he cent ral nervous s ys t em (CNS).
o
o
b. Specific measurements and assays
(1) Pressure. The openi ng pres s ure s houl d be det ermi ned. Pres s ure exceedi ng 180–200 mm H 2 O i s abnormal when a pat i ent i s rel axed and i n a l at eral decubi t us pos i t i on.
Pa g e 2 6 0 9
ABC Ambe r CHM Conve rte r Tria l ve rsion, http://w w w .proce sste x t.com/a bcchm.html
(2) Protein. An el evat ed CSF prot ei n l evel i s a nons peci fi c i ndi cat or of i nfl ammat i on or breakdown of t he bl ood–brai n barri er.
(3) Glucose. Hypoglycorrhachia (i .e., CSF gl ucos e <40 mg/dL or a s i mul t aneous CSF–bl ood gl ucos e rat i o of <0.6) s ugges t s i nfect i on or s t eri l e i nfl ammat i on.
(a) Rel at i vel y common caus es of hypogl ycorrhachi a i ncl ude bact eri al , fungal , or t ubercul ous i nfect i on; carci nomat ous meni ngi t i s ; and hypogl ycemi a.
(b) Les s common caus es i ncl ude mumps , herpes s i mpl ex vi rus (HSV) or zos t er i nfect i on, s ubarachnoi d hemorrhage (SAH), s arcoi dos i s , s yphi l i t i c meni ngi t i s , and s ys t emi c l upus eryt hemat os us (SLE).
(4) White blood cell (WBC) count. A W BC 3
count exceedi ng 5 cel l s /mm i s cons i dered abnormal .
(a) Excess neutrophils s ugges t acut e i nfect i on or, on occas i on, s t eri l e i nfl ammat i on.
Pa g e 2 6 1 0
ABC Ambe r CHM Conve rte r Tria l ve rsion, http://w w w .proce sste x t.com/a bcchm.html
(b) Excess mononuclear cells s ugges t a vi ral i nfect i on, an i ndol ent nonvi ral i nfect i ous proces s , or s t eri l e i nfl ammat i on.
(5) Blood. Bl ood may appear i n t he CSF as a res ul t of t he l ocal t rauma of a l umbar punct ure (LP) or by CNS hemorrhage from mul t i pl e caus es .
(a) A t raumat i c LP i s s us pect ed i f gros s bl ood exudes from t he needl e and t hen cl ears qui ckl y, or i f a l arge di s crepancy exi s t s bet ween t he number of red bl ood cel l s (RBCs ) i n t he fi rs t drops of CSF obt ai ned as compared wi t h a l at er al i quot .
(b) A t raumat i c LP s houl d not reveal a xant hochromi c (yel l ow-t i nged) CSF, becaus e s uffi ci ent t i me woul d not have el aps ed t o caus e breakdown of RBCs .
(6) Culture and Gram staining. Thes e s t udi es are i ndi cat ed t o eval uat e t he pos s i bi l i t y of i nfect i on. A CSF Venereal Di s eas e Res earch Laborat ory (VDRL) t es t i s i ndi cat ed i f CNS s yphi l i s i s a di agnos t i c cons i derat i on.
(7) Cytology. Cyt ol ogi c exami nat i on i s us eful i f mal i gnancy i s s us pect ed.
Pa g e 2 6 1 1
ABC Ambe r CHM Conve rte r Tria l ve rsion, http://w w w .proce sste x t.com/a bcchm.html
(8) Intrathecal immunoglobulin production. Thi s can be det ermi ned us i ng t he i mmunogl obul i n G (IgG) i ndex:
or t hrough CSF el ect rophores i s , whi ch reveal s t he pres ence of ol i gocl onal bands (di s cret e i mmunogl obul i n aggregat es ). An el evat ed IgG i ndex or ol i gocl onal bands are found i n CNS i nfl ammat ory di s orders s uch as mul t i pl e s cl eros i s (MS) and i nfect i ons . o
o
c. Contraindications t o t he performance of an LP
(1) A mass effect s uffi ci ent t o caus e di s t ort i on of t he l at eral or t hi rd vent ri cl es or a mi dl i ne s hi ft
(2) A posterior fossa mass
(3) A coagulopathy [e.g., a prot hrombi n t i me (PT) >3 s econds over cont rol or a pl at el et 3
count of < 50,000/mm ]
2. Electroencephalography (EEG) and evoked potentials (EPs) o
o
a. EEG. i s i ndi cat ed i n t he eval uat i on of seizure disorders, encephalopathies, sleep disorders,
Pa g e 2 6 1 2
ABC Ambe r CHM Conve rte r Tria l ve rsion, http://w w w .proce sste x t.com/a bcchm.html
and brain death. Prol onged vi deo EEG moni t ori ng i s t he gold standard for eval uat i ng s ei zure probl ems t hat are di ffi cul t t o di agnos e or t reat . o
o
b. EPs are repet i t i ve afferent s t i mul i pres ent ed t o t he eye, ear, peri pheral s ens ory nerves , or cerebral cort ex t hat caus e s t ereot ypi c wave forms t hat can be anal yzed by comput eri zed s i gnal averagi ng met hodol ogy. P.520
(1) Visual, brain stem–auditory, and somatosensory EPs can det ect l es i ons , whi ch are oft en cl i ni cal l y s i l ent , i n t he anat omi c pat hways s ubs ervi ng t hes e s ens ory s ys t ems .
(2) Motor EPs, whi ch can be el i ci t ed by t rans crani al magnet i c s t i mul at i on of t he mot or pat hways , provi de i nformat i on about t he i nt egri t y of t he mot or s ys t em.
3. Imaging studies o
o
a. Computed tomography (CT ) scanning. provi des an i mage of t he brai n t hat al l ows defi ni t i on of hemorrhage, edema, atrophy, mass lesions, and ventricular size.
Pa g e 2 6 1 3
ABC Ambe r CHM Conve rte r Tria l ve rsion, http://w w w .proce sste x t.com/a bcchm.html
(1) Int ravenous cont ras t can be admi ni s t ered t o defi ne regions where the blood–brain barrier is not intact.
(2) CT s canni ng can al s o vi s ual i ze the spinal cord and surrounding bony structures; t rans vers e i mages are es peci al l y wel l repres ent ed.
o
o
b. Magnetic resonance imaging (MRI) provi des excel l ent anatomic depiction of the brain, es peci al l y t he pos t eri or fos s a, and the spinal cord. Di s eas e of t he cerebral whi t e mat t er i s part i cul arl y wel l defi ned. Int ravenous cont ras t can det ect s i t es of a di s t urbed bl ood-brai n barri er. Di ffus i on wei ght ed i magi ng can det ect regi ons of cerebral i s chemi a, as refl ect ed by an area of decreas ed free wat er di ffus i on.
o
o
c. Single-photon emission computed tomography (SPECT ) provi des an i mage-bas ed es t i mat e of cerebral blood flow aft er i nt ravenous i nject i on of a radi oact i ve t racer.
o
o
d. Noninvasive vascular studies
(1) Duplex scanning provi des an ul t ras ound i mage of t he ext racrani al carot i d and vert ebral art eri es t oget her wi t h a Doppl er des cri pt i on of fl ow pat t erns .
Pa g e 2 6 1 4
ABC Ambe r CHM Conve rte r Tria l ve rsion, http://w w w .proce sste x t.com/a bcchm.html
(2) T ranscranial Doppler (T CD) defi nes i nt racrani al l arge art ery fl ow pat t erns .
(3) Magnetic resonance angiography (MRA) us es magnet i c res onance t echnol ogy t o i mage t he vas cul ar anat omy.
(4) Computed tomography angiography (CT A) us es comput ed t omography t echnol ogy wi t h i nt ravenous cont ras t admi ni s t rat i on t o i mage t he vas cul ar anat omy.
o
o
e. Angiography. The vas cul ar anat omy i s bes t defi ned wi t h cerebral angi ography. Thi s s t udy i s us eful for i dent i fyi ng an aneurys mal s ource of SAH, eval uat i ng occl us i ve cerebrovas cul ar di s eas e (es peci al l y i f s urgery i s cont empl at ed), defi ni ng vas cul i t i s , and as s es s i ng art eri ovenous mal format i ons .
4. Nerve conduction velocity (NCV) studies and electromyography (EMG) o
o
a. NCV studies. can pl ace peri pheral nerve di s eas e i nt o s ens ory, mot or, or s ens ori mot or cat egori es ; defi ne pri mari l y demyel i nat i ng or axonal dys funct i on; and i dent i fy s i t es of conduct i on bl ock. Thes e di s t i nct i ons hel p i n maki ng a di agnos i s .
o
o
b. EMG can di fferent i at e probl ems affect i ng mus cl e s o t hat t hey can be di vi ded i nt o broad cat egori es
Pa g e 2 6 1 5
ABC Ambe r CHM Conve rte r Tria l ve rsion, http://w w w .proce sste x t.com/a bcchm.html
s uch as denervat i on and myopat hy.
(1) The distribution of the observed changes hel ps det ermi ne whet her t he probl em i s myot omal (i .e., l i mi t ed t o a few nerve root s ) or di ffus e.
(2) The pattern of wave forms provi des i nformat i on pert ai ni ng t o ongoi ng mus cul ar denervat i on and rei nnervat i on.
II. Loss of Consciousness A. Syncope
1. Definition. Syncope refers t o a t rans i ent l os s of cons ci ous nes s t hat t ypi cal l y fol l ows i ns uffi ci ent bl ood s uppl y t o t he brai n for more t han a few s econds .
2. Clinical signs. Pat i ent s are t rans i ent l y unres pons i ve, wi t h di mi ni s hed mus cl e t one. A few general i zed t oni c s pas ms may occur, es peci al l y i f pat i ent s are prevent ed from l yi ng down.
3. Etiology. Cardi ac and ci rcul at ory caus es of s yncope are di s cus s ed i n Chapt er 1 IX. Mos t pat i ent s wi t h s yncope have a cardi ac or ci rcul at ory bas i s for t he event , al t hough neurol ogi c condi t i ons need t o be cons i dered i f t he di agnos i s remai ns el us i ve. o
o
a. Circulatory disturbances. are part i cul arl y
Pa g e 2 6 1 6
ABC Ambe r CHM Conve rte r Tria l ve rsion, http://w w w .proce sste x t.com/a bcchm.html
common.
(1) Vasovagal syncope, whi ch i s oft en s een i n young peopl e, i s commonl y as s oci at ed wi t h emot i onal s t res s , fear, or pai n. P.521
(2) Postprandial syncope, whi ch frequent l y affect s t he el derl y, oft en occurs fol l owi ng meal s i n whi ch al cohol has been cons umed.
(3) Syncope can occur i n di vers e s et t i ngs t hat have i n common a preceding Valsalva or straining maneuver t hat decreas es venous ret urn and promot es paras ympat het i c t one.
o
o
b. Cardiac output disturbances, ot her t han arrhyt hmi a or mechani cal obs t ruct i on, s houl d be cons i dered.
(1) Vasodepressor (neurocardiogenic) syncope i s caus ed by undue s t i mul at i on of afferent cardi ac mechanorecept ors becaus e of cardi ac di s t ent i on or s t renuous cont ract i ons . Thi s caus es a decreas e i n s ympat het i c act i vi t y and an i ncreas e i n paras ympat het i c act i vi t y t hat l eads t o vas odi l at at i on, bradycardi a, and s ubs equent hypot ens i on.
Pa g e 2 6 1 7
ABC Ambe r CHM Conve rte r Tria l ve rsion, http://w w w .proce sste x t.com/a bcchm.html
(2) Carotid sinus hypersensitivity can l ead t o bradyarrhyt hmi as and hypot ens i on. Becaus e carot i d s i nus hypers ens i t i vi t y i s pres ent i n many ol der men, i t s houl d be cons i dered res pons i bl e for s yncope onl y i f ot her caus es have been excl uded.
o
o
c. Hypoglycemia caus es a l ack of nut ri ent s uppl y t o t he brai n and can l ead t o s yncope. Hypogl ycemi a as a caus e of s yncope i s part i cul arl y l i kel y i n pat i ent s wi t h t ype 1 (i ns ul i n-dependent ) di abet es . Therapy i nvol ves t he admi ni s t rat i on of gl ucos e.
o
o
d. Neurologic disorders are rel at i vel y uncommon caus es of s yncope. Several condi t i ons are i mport ant becaus e t hey can l ead t o unres pons i venes s and are t herefore oft en cons i dered duri ng t he eval uat i on of a pat i ent wi t h t rans i ent unres pons i venes s .
(1) Seizures (s ee al s o IX A–B) are a caus e of unres pons i venes s t hat mus t be di fferent i at ed from s yncope.
(a) Pat i ent s are rarel y l i mp duri ng s ei zures . Many s ei zures caus e i nt ermi t t ent , rel at i vel y rhyt hmi c l i mb cont ract i ons (cl oni c act i vi t y) or s us t ai ned l i mb ext ens i on (t oni c act i vi t y).
(b) At oni c s ei zures are rare i n adul t s but do caus e s udden col l aps e. Abs ence
Pa g e 2 6 1 8
ABC Ambe r CHM Conve rte r Tria l ve rsion, http://w w w .proce sste x t.com/a bcchm.html
s ei zures , convers el y, do not res ul t i n a fal l .
(c) Becaus e s ome pat i ent s wi t h s yncope can exhi bi t i nvol unt ary movement s , t he pos s i bi l i t y of a pri mary s ei zure i s a cons i derat i on. However, i n mos t cas es , t he t oni c or myocl oni c act i vi t y t ends t o occur s everal s econds aft er cons ci ous nes s i s l os t and merel y refl ect s cerebral hypoperfus i on, not a pri mary s ei zure di s order.
(2) SAH can t rans i ent l y i ncreas e ICP, compromi s i ng gl obal cerebral perfus i on. Cl ues t o t he di agnos i s i ncl ude pers i s t ent headache, meni ngi s mus , and papi l l edema.
(3) Basilar artery migraine (a uni que t ype of mi grai ne wi t h aura) i s a rare caus e of unres pons i venes s . A hi s t ory of recurrent headache, recurrent epi s odes of unres pons i venes s , and as s oci at ed s ympt oms (e.g., vi s ual di s t ort i on and di zzi nes s ) s houl d l ead t o cons i derat i on of t hi s condi t i on. In mos t i ns t ances , ot her caus es of vert ebrobas i l ar i s chemi a s houl d be s ought before as s i gni ng a di agnos i s of bas i l ar art ery mi grai ne.
(4) Narcolepsy can caus e epi s odes of s l eep or cat apl exy t hat can be mi s t aken for s yncope.
Pa g e 2 6 1 9
ABC Ambe r CHM Conve rte r Tria l ve rsion, http://w w w .proce sste x t.com/a bcchm.html o
o
e. Psychogenic unres pons i venes s can be as s oci at ed wi t h anxi et y, pani c at t acks , or hypervent i l at i on, as wel l as s omat oform (convers i on) di s order.
4. Diagnosis. A hi s t ory and phys i cal exami nat i on oft en can provi de cl ues t o t he proper di agnos i s . If no cl ues are evi dent , i t i s general l y appropri at e t o proceed wi t h a cardi ovas cul ar eval uat i on as out l i ned i n Chapt er 1 IX D. o
o
a. Neurol ogi c t es t i ng frequent l y i ncl udes an EEG. Rarel y i s a CT s can di agnos t i c. In s ome pat i ent s , an MRI or an i magi ng s t udy of t he vas cul ar s ys t em can be i nformat i ve.
o
o
b. Upri ght t i l t t es t i ng, pos s i bl y wi t h i s oprot erenol i nfus i on, can provi de evi dence for a di agnos i s of vas odepres s or (neurocardi ogeni c) s yncope, es peci al l y i f t he charact eri s t i c hemodynami c changes occur i n l es s t han 15 mi nut es wi t hout i s oprot erenol i nfus i on.
5. T herapy. Treat ment depends on t he underl yi ng di agnos i s . Fi rs t -l i ne t herapy of neurocardi ogeni c s yncope i s t reat ment wi t h β-adrenergi c bl ockers .
B. Coma
1. Definition. Coma i s a s t at e i n whi ch a pat i ent i s unres pons i ve t o envi ronment al s t i mul i and unabl e t o
Pa g e 2 6 2 0
ABC Ambe r CHM Conve rte r Tria l ve rsion, http://w w w .proce sste x t.com/a bcchm.html
communi cat e i n any manner. Coma i s as s oci at ed wi t h ext ens i ve s t ruct ural or phys i ol ogi c damage t o bot h cerebral hemi s pheres or t o t he as cendi ng RAS i n t he di encephal on, mes encephal on, or pons . P.522
2. Etiology. The many caus es of coma can be broadl y grouped as s hown i n
Onl i ne Tabl e 11-4.
ONLINE TABLE 11-4 Causes of Coma Ca Po teg ssi ory ble Eti olo gic Fa cto rs Su Epi pra dur t en al t ori or al
s ub
(he dur mi s al ph he eri ma c)
to
l es ma i on Int s
rap
Pa g e 2 6 2 1
ABC Ambe r CHM Conve rte r Tria l ve rsion, http://w w w .proce sste x t.com/a bcchm.html
are nch ym al he mo rrh ag e Lar ge isc he mi c i nf arc tio n Tu mo r Ab s ce ss Tra um a Inf Po rat nt i ent ne ori or al
cer
l es eb i on el l s
ar
Pa g e 2 6 2 2
ABC Ambe r CHM Conve rte r Tria l ve rsion, http://w w w .proce sste x t.com/a bcchm.html
he ma to ma Ba sil ar art ery t hr om bos is Is c he mi c cer eb el l ar i nf arc tio n Tu mo r Ab s ce ss Di f Su fus bar e
ach
di s noi eas d
Pa g e 2 6 2 3
ABC Ambe r CHM Conve rte r Tria l ve rsion, http://w w w .proce sste x t.com/a bcchm.html
es , he me mo t ab rrh ol i c ag di s e ord Me ers ni n ,
gi t i
an s d
Enc
t ox ep i ns hal itis Hy dro cep hal us Dru gs (e. g., nar cot i cs , al c oh ol , bar bi t ura t es , an d
Pa g e 2 6 2 4
ABC Ambe r CHM Conve rte r Tria l ve rsion, http://w w w .proce sste x t.com/a bcchm.html
be nz o di a z ep i ne s) Hy poor hyp erg l yc em ia Is c he mi c or hyp oxi c enc ep hal op at h y Hy per car bi a My xed em a
Pa g e 2 6 2 5
ABC Ambe r CHM Conve rte r Tria l ve rsion, http://w w w .proce sste x t.com/a bcchm.html
Hy pot her mi a He pat ic or ren al fai l ure Thi am i ne def i ci e ncy Ps y cho ge ni c
3. Approach to the patient. Compl et e and rapi d as s es s ment i s cri t i cal for opt i mal care. o
o
a. Patient history. The phys i ci an s houl d as cert ai n t he fol l owi ng i nformat i on from s omeone cl os e t o t he pat i ent :
(1) Past medical status, es peci al l y i f t here i s a preexi s t i ng neurol ogi c, cardi ac, pul monary,
Pa g e 2 6 2 6
ABC Ambe r CHM Conve rte r Tria l ve rsion, http://w w w .proce sste x t.com/a bcchm.html
hepat i c, or renal condi t i on
(2) Prescription and over-the-counter drugs us ed by t he pat i ent
(3) Hi s t ory of drug abuse, i f appl i cabl e
(4) Recent patient complaints
(5) Details regarding the site where t he pat i ent was found (e.g., pres ence of empt y drug vi al s , evi dence of a fal l )
o
o
b. Physical examination. The examination s houl d be t horough. Ext remes of bl ood pres s ure, pul s e, or t emperat ure, abnormal breat hi ng pat t erns , evi dence of head or neck t rauma, and t he pres ence of meni ngi s mus s houl d be not ed careful l y. The s ki n s houl d be i ns pect ed for s i gns of t rauma or needl e t racks . Speci al at t ent i on s houl d be di rect ed t o t he pat i ent 's :
(1) Pupils. Pupi l l ary s i ze and react i vi t y are dependent on s ympat het i c and paras ympat het i c i nnervat i on. Brai ns t em refl exes s uch as t he pupi l l ary react i on t o l i ght offer cl ues t o t he l ocat i on of t he l es i on res pons i bl e for t he coma.
(a) Large, nonreactive pupils res ul t
Pa g e 2 6 2 7
ABC Ambe r CHM Conve rte r Tria l ve rsion, http://w w w .proce sste x t.com/a bcchm.html
from t he di s rupt i on of t he paras ympat het i c port i on of t he t hi rd crani al nerve, but may al s o be s een wi t h barbi t urat e overdos e.
(b) Small, reactive pupils res ul t from t he di s rupt i on of t he s ympat het i c pupi l l odi l at ory i mpul s es t hat ari s e i n t he hypot hal amus and cours e caudal l y t hrough t he peri aqueduct al gray mat t er and cervi cal s pi nal cord before t ravel i ng ros t ral l y wi t h t he i nt ernal carot i d art ery t oward t he eyes .
(c) Pinpoint pupils t hat are nonreact i ve t o l i ght may be s een wi t h narcot i c overdos e.
(2) Ocular motility. Anal ys i s of ocul ar mot i l i t y al l ows as s es s ment of damage t o t he brai ns t em and t he crani al nerves t hat cont rol eye movement .
(a) The eyes s houl d fi rs t be exami ned i n t he res t i ng pos i t i on for s pont aneous mot i on of t he eyebal l s . Al t hough t he eyes of comat os e pat i ent s may move s pont aneous l y, t hey do not fi xat e or t rack i n a purpos eful manner.
(b) If t he eyes are i mmobi l e, movement can be el i ci t ed t hrough t he
Pa g e 2 6 2 8
ABC Ambe r CHM Conve rte r Tria l ve rsion, http://w w w .proce sste x t.com/a bcchm.html
vestibulo-ocular reflex by movi ng t he pat i ent 's head s i de t o s i de (t he “ doll' s eyes― or oculocephalic maneuver) or by el evat i ng t he pat i ent 's head 30 degrees and i rri gat i ng t he ext ernal audi t ory canal wi t h i ce wat er. The former s houl d onl y be performed aft er a cervi cal i njury i s rul ed out .
(i) Conjugate deviation of t he eyes bi l at eral l y i mpl i es i nt act brai ns t em ci rcui t ry.
(ii) Failure of an eye to abduct i n res pons e t o t hes e maneuvers i mpl i es dys funct i on of pont i ne s t ruct ures or s i xt h nerve compromi s e.
(iii) Failure of an eye to adduct i mpl i es dys funct i on of t he medi al l ongi t udi nal fas ci cul us or ocul omot or nucl eus or nerve.
(iv) The pres ence of conjugate nystagmus away from t he s i de of i ce wat er i rri gat i on s ugges t s ps ychogeni c coma.
(3) Motor functions. Quadri pares i s , hemi pares i s , or monopares i s may occur i n comat os e pat i ent s .
Pa g e 2 6 2 9
ABC Ambe r CHM Conve rte r Tria l ve rsion, http://w w w .proce sste x t.com/a bcchm.html
(a) Quadriparesis and flaccidity s ugges t s pont i ne or medul l ary compromi s e or a hi gh cervi cal s pi nal cord i ns ul t .
(b) Decorticate posturing (i .e., l eg ext ens i on wi t h fl exi on of t he arm, wri s t , and fi ngers ) can be uni l at eral or bi l at eral and s ugges t s a hemi s pheral or di encephal i c l es i on.
(c) Decerebrate posturing (i .e., l eg and arm ext ens i on) al s o can be uni l at eral or bi l at eral and s ugges t s mi dbrai n or pont i ne compromi s e.
4. Clinical features. Once gl obal brai ns t em dys funct i on has devel oped, di fferent i at i on bet ween s uprat ent ori al and i nfrat ent ori al caus es of coma cannot be made wi t hout di agnos t i c t es t i ng unl es s a hi s t ory and s eri al obs ervat i ons of t he pat i ent 's cl i ni cal cours e can be document ed. o
o
a. Supratentorial causes of coma. are oft en charact eri zed by pat hol ogi c proces s es t hat res ul t i n swelling of a cerebral hemisphere. P.523
Pa g e 2 6 3 0
ABC Ambe r CHM Conve rte r Tria l ve rsion, http://w w w .proce sste x t.com/a bcchm.html
(1) Thi s mas s effect caus es a mi dl i ne s hi ft of t he affect ed hemi s phere t oward t he cont ral at eral s i de, compres s i on of t he i ps i l at eral t hi rd nerve as i t cours es near t he medi al t emporal l obe (uncus ), herni at i on of t he medi al t emporal l obe bel ow t he t ent ori al not ch (uncal herni at i on), di s t ort i on of t he mes encephal on, and herni at i on of t he ci ngul at e gyrus under t he mi dl i ne fal x (s ubfal ci al herni at i on).
(2) Typi cal l y, t here i s a progres s i ve cl i ni cal det eri orat i on charact eri zed by i ncreas i ng unres pons i venes s , devel opment of a t hi rd nerve pal s y i ps i l at eral t o t he s wol l en hemi s phere, and, ul t i mat el y, mi dbrai n compromi s e (refl ect ed by bi l at eral l y nonreact i ve, di l at ed pupi l s ).
o
o
b. Infratentorial causes of coma can be s us pect ed i f at axi a, mul t i pl e as ymmet ri c crani al nerve pal s i es , and uni l at eral or bi l at eral l i mb weaknes s or s ens ory l os s devel op before t he devel opment of more gl obal , s evere i mpai rment of brai ns t em funct i on (charact eri zed by nonreact i ve pupi l s , abs ent ocul ar mot i l i t y, and abs ent corneal and gag refl exes ).
o
o
c. Diffuse, toxic, or metabolic causes of coma can be s us pect ed i f pupi l l ary res pons es are i nt act , ocul ar mot i l i t y i s pres erved, corneal refl exes can be el i ci t ed, a gag refl ex i s pres ent , and l i mb movement i n res pons e t o noxi ous l ocal s t i mul i i s
Pa g e 2 6 3 1
ABC Ambe r CHM Conve rte r Tria l ve rsion, http://w w w .proce sste x t.com/a bcchm.html
obs erved. If pupi l l ary res pons i venes s remai ns even when ot her brai ns t em and l i mb funct i on i s l os t , a met abol i c caus e of coma s houl d be cons i dered. o
o
d. Psychogenic coma s houl d be s us pect ed i f t he pat i ent has a hi s t ory of ps ychi at ri c di s eas e or i f t he fi ndi ngs on phys i cal exami nat i on are nonphys i ol ogi c. Exampl es of nonphys i ol ogi c res pons es i n a “comat os e― pat i ent i ncl ude:
(1) The pres ence of nystagmus when t he pat i ent 's ears are i rri gat ed wi t h i ce wat er
(2) Advers i ve head and eye movement s
(3) Fai l ure of t he pat i ent 's arm, when hel d by t he exami ner over t he pat i ent 's face, t o fal l on t he face when rel eas ed by t he exami ner
(4) Res i s t ance t o havi ng t he eyel i ds opened
5. T herapy. Ideal l y, care of t he comat os e pat i ent i s i nt ert wi ned wi t h t he i ni t i al as s es s ment and t he devel opment of et i ol ogi c hypot hes es . o
o
a. Initial therapy. The “A, B, C's ― Mai nt ai ni ng an adequate airway, optimal ventilation, and appropriate blood pressure are pri ori t y concerns .
Pa g e 2 6 3 2
ABC Ambe r CHM Conve rte r Tria l ve rsion, http://w w w .proce sste x t.com/a bcchm.html
(1) If cervical fracture i s a pos s i bi l i t y, immobilization of the neck i s of great i mport ance.
(2) Endotracheal intubation may be i ndi cat ed t o prot ect t he ai rway.
(3) Blood samples for a compl et e bl ood count (CBC), el ect rol yt es , gl ucos e, renal and l i ver funct i on s t udi es , coagul at i on profi l es , bl ood gas es , and t oxi col ogy s houl d be obt ai ned.
(4) Intravenous thiamine (100 mg), one ampul e of D 5 0 W, and naloxone (0.4 mg) are oft en admi ni s t ered. Flumazenil can be gi ven i f benzodi azepi ne or hepat i c coma i s s us pect ed.
o
o
b. Management
(1) Imaging. If t he pat i ent 's general medi cal condi t i on permi t s , and i f t he caus e of coma i s not cl earl y cerebral anoxi a aft er cardi opul monary arres t or a drug overdos e, mos t pat i ent s s houl d have a brai n CT scan t o defi ne t he pres ence of an intracranial mass, cerebral edema, or hydrocephalus. Furt her management depends on t he et i ol ogy of t he coma.
Pa g e 2 6 3 3
ABC Ambe r CHM Conve rte r Tria l ve rsion, http://w w w .proce sste x t.com/a bcchm.html
(2) ICP evaluation and management
(a) ICP evaluation. Cons i derat i on of t he pat i ent 's ICP i s i nt i mat el y t i ed t o t he eval uat i on of coma. The i nt racrani al cavi t y has a fi ni t e vol ume and compl i ance.
(i) Normal l y, modes t vol ume addi t i ons t o t he i nt racrani al cont ent s (e.g., from a s mal l i nt raparenchymal hemat oma) caus e onl y a s mal l ri s e i n ICP.
(ii) W i t h progres s i ve i ncrement al i ncreas es t o t he i nt racrani al vol ume (e.g., from mas s i ve cerebral edema, a hemat oma, or a t umor), t he i nt racrani al compl i ance decreas es and t he ICP markedl y i ncreas es .
(iii) Becaus e cerebral perfus i on pres s ure i s t he res ul t of t he mean art eri al pres s ure mi nus t he ICP, an excessive rise of the ICP i s as s oci at ed wi t h impaired cerebral perfusion and progressive neurologic deterioration.
(b) ICP management. If a pat hol ogi c proces s as s oci at ed wi t h el evat ed ICP i s s us pect ed, emergency management
Pa g e 2 6 3 4
ABC Ambe r CHM Conve rte r Tria l ve rsion, http://w w w .proce sste x t.com/a bcchm.html
s houl d i ncl ude s t eps t o decreas e t he pres s ure, or at t he very l eas t , avoi d i ncreas i ng i t . If pos s i bl e, t he cerebral perfus i on pres s ure, or CPP, where CPP = MAP – ICP, s houl d be kept at great er t han 60 mm Hg and t he ICP at l es s t han P.524
20 mm Hg. Opt i mal management of i ncreas ed ICP oft en requi res di rect ICP moni t ori ng as wel l as det ermi nat i on of hemodynami c paramet ers .
(i) Pat i ent s can be hypervent i l at ed wi t h an Ambu bag before i nt ubat i on. Int ubat i on and endot racheal s uct i oni ng s houl d be performed careful l y t o mi ni mi ze el evat i on of ICP.
(ii) Fever and agi t at i on s houl d be mi ni mi zed.
(iii) The pat i ent 's head s houl d be el evat ed 30 degrees and kept i n mi dpos i t i on t o opt i mi ze venous drai nage.
(iv) Os mot i c t herapy i s us ed t o dehydrat e t he brai n and decreas e t he ICP. Pat i ent s are kept euvol emi c, and i nt ravenous
Pa g e 2 6 3 5
ABC Ambe r CHM Conve rte r Tria l ve rsion, http://w w w .proce sste x t.com/a bcchm.html
manni t ol or hypert oni c s al i ne i s admi ni s t ered t o achi eve a hyperos mot i c s t at e.
(v) Durotomy and hemicraniectomy can be used to decompress swollen brain.
(c) Prognosis. The prognos i s of coma i s general l y rel at ed t o t he caus e of t he coma, t he dept h of t he coma, and t he durat i on. In one s eri es , t he probabi l i t y of good or moderat e recovery was onl y 2% once t he pat i ent had remai ned i n coma for 14 days .
C. Vegetative state
1. Definitions o
o
a. The vegetative state i s charact eri zed by t he unawarenes s of s el f or ext ernal s t i mul i . Pat i ent s cannot i nt eract wi t h ot hers i n a meani ngful fas hi on. Aut onomi c funct i ons are rel at i vel y wel l mai nt ai ned, and a s l eep–wake cycl e exi s t s . Pat i ent s can s urvi ve wi t h medi cal and nurs i ng s upport .
o
o
b. A persistent vegetative state i s defi ned as a veget at i ve s t at e t hat pers i s t s for at l eas t 1 mont h aft er t he i ni t i al brai n i ns ul t . If, wi t h cont i nued obs ervat i on (us ual l y 3 mont hs for nont raumat i c
Pa g e 2 6 3 6
ABC Ambe r CHM Conve rte r Tria l ve rsion, http://w w w .proce sste x t.com/a bcchm.html
i njury), t here i s no meani ngful recovery, t he l i kel i hood of funct i onal recovery can be judged t o be ni l ; t he pat i ent can be s ai d t o be i n a permanent vegetative state.
2. T herapy. The fami l y and phys i ci an s houl d det ermi ne t he l evel of t reat ment appropri at e for t he pat i ent i n a pers i s t ent veget at i ve s t at e.
D. Brain death
1. Definition. Deat h i s recogni zed as occurri ng when t here i s i rrevers i bl e ces s at i on of al l brai n funct i on. A brai n i ns ul t s uffi ci ent t o caus e compl et e l os s of cerebral funct i on s houl d be document ed, i f pos s i bl e.
2. Approach to the patient o
o
a. Physical examination. Pat i ent s are compl et el y unres pons i ve t o ext ernal vi s ual , audi t ory, and t act i l e s t i mul i and are i ncapabl e of communi cat i on i n any manner.
(1) Pupi l l ary res pons es are abs ent , and eye movement s cannot be el i ci t ed by t he ves t i bul o-ocul ar refl ex or by i rri gat i ng t he ears wi t h col d wat er.
(2) The corneal and gag refl exes are abs ent , and t here i s no faci al or t ongue movement .
Pa g e 2 6 3 7
ABC Ambe r CHM Conve rte r Tria l ve rsion, http://w w w .proce sste x t.com/a bcchm.html
(3) The l i mbs are fl acci d, and t here i s no movement , al t hough pri mi t i ve wi t hdrawal movement s i n res pons e t o l ocal pai nful s t i mul i , medi at ed at a s pi nal cord l evel , can occur.
o
o
b. Apnea test. Pat i ent s have no res pi rat ory funct i on. An apnea t es t s houl d be performed t o as cert ai n t hat no res pi rat i ons occur at a PaCO 2 l evel of at l eas t 60 mm Hg. The oxygenat i on s houl d be mai nt ai ned as t he PaCO 2 i s al l owed t o ri s e. The i nabi l i t y t o devel op res pi rat i on i s cons i s t ent wi t h medul l ary fai l ure.
o
o
c. Exclusionary criteria. A di agnos i s of brai n deat h cannot be made i n t he s et t i ng of drug i nt oxi cat i on, hypot hermi a (defi ned as a core t emperat ure of <32°C), or s evere hypot ens i on (i .e., s hock).
o
o
d. Confirmatory tests. Thes e t es t s are us ual l y not neces s ary t o di agnos e brai n deat h but can be us ed i f doubt exi s t s or i f l ocal s t at ut es requi re t hem.
(1) An EEG does not demons t rat e any phys i ol ogi c brai n act i vi t y.
(2) T ests to assess cerebral blood flow fai l t o s how cerebral perfus i on.
o
o
e. Period of observation. Peri odi c eval uat i on i s
Pa g e 2 6 3 8
ABC Ambe r CHM Conve rte r Tria l ve rsion, http://w w w .proce sste x t.com/a bcchm.html
neces s ary before a di agnos i s of brai n deat h can be made, unl es s t here i s gros s evi dence of a nons urvi vabl e i ns ul t t o t he brai n.
(1) Two eval uat i ons (6 t o 12 hours apart ) are us ual l y s uffi ci ent t o s upport a di agnos i s of brai n deat h. P.525
(2) In t he pres ence of anoxi c brai n damage, 24 hours of obs ervat i on are appropri at e before decl ari ng brai n deat h.
III. Alteration in Behavior A. Delirium
1. Definition. Del i ri um i s a di s order of brai n funct i on affect i ng behavi or and caus i ng i mpai red at t ent i on and cogni t i on, mot or hyper- or hypoact i vi t y, al t ered s l eep–wake cycl es , and al t ered s t at es of arous al . It i s oft en acut e, revers i bl e, and s econdary t o a medi cal or neurol ogi c di s order.
2. Etiology. General i zed or focal caus es of cerebral dys funct i on are pot ent i al caus es of del i ri um. o
o
a. Generalized brain dysfunction. Caus es i ncl ude t he fol l owi ng:
Pa g e 2 6 3 9
ABC Ambe r CHM Conve rte r Tria l ve rsion, http://w w w .proce sste x t.com/a bcchm.html
(1) Drugs, i ncl udi ng ant i chol i nergi cs , ant i parki ns oni ans , anal ges i cs , ci met i di ne, di goxi n, benzodi azepi nes , ant i depres s ant s , and i l l i ci t s ubs t ances , may produce del i ri um. W i t hdrawal from al cohol , barbi t urat es , and benzodi azepi nes i s as s oci at ed wi t h del i ri um as wel l . The serotonin syndrome cons i s t s of del i ri um and di s ordered mot or and aut onomi c funct i on; i t res ul t s from overs t i mul at i on of s erot oni n recept ors . The neuroleptic malignant syndrome caus es del i ri um i n as s oci at i on wi t h fever, ri gi di t y, t remul ous nes s , and occas i onal l y myogl obi nuri a.
(2) Metabolic alterations, i ncl udi ng hypoxi a, hypercarbi a, hyponat remi a, uremi a, hepat i c fai l ure, hypergl ycemi a, hypogl ycemi a, fever, dehydrat i on, hypercal cemi a, myxedema, hypert hyroi di s m, porphyri a, ant i -t hyroi d ant i bodi es (Has hi mot o's encephal opat hy), and t hi ami ne and ni aci n defi ci enci es , can caus e del i ri um.
(3) Diffuse insults to the brain s uch as meni ngi t i s , encephal i t i s , fat embol i , and di s s emi nat ed i nt ravas cul ar coagul at i on (DIC) are as s oci at ed wi t h cogni t i ve i mpai rment .
(4) Nonconvulsive status epilepticus, i ncl udi ng abs ence or compl ex part i al s ei zures , may caus e del i ri um. Pos t i ct al pat i ent s may al s o be del i ri ous .
Pa g e 2 6 4 0
ABC Ambe r CHM Conve rte r Tria l ve rsion, http://w w w .proce sste x t.com/a bcchm.html
(5) Systemic infections s uch as a uri nary t ract i nfect i on, pneumoni a, or s eps i s .
o
o
b. Focal cerebral disease. Di fferent i at i ng a gl obal cerebral di s order from a focal brai n di s eas e t hat al s o may caus e al t ered behavi or pres ent s a cl i ni cal chal l enge. For exampl e, focal brai n di s eas e can caus e a s ubt l e aphas i a, whi ch may be mi s i nt erpret ed as del i ri um. Appropri at e l aborat ory and neurodi agnos t i c t es t s s houl d be performed bas ed on t he cl i ni cal pres ent at i on.
(1) Focal cerebral disease, t ypi cal l y caus ed by s t roke, i nvol ves t he nondomi nant t emporopari et al area, front al l obes , head of t he caudat e nucl eus , t hal amus (t he t op-of-t he-bas i l ar s yndrome), or occi pi t al l obes , whi ch may caus e bl i ndnes s . Pat i ent s who have a focal cerebral di s eas e may be agi t at ed and experi ence hal l uci nat i ons .
(2) Mass lesions al s o can caus e a confus i onal s t at e, es peci al l y i f t hey are l ocat ed i n t he front al l obes .
3. T herapy. The ai m of t herapy i s i dent i fi cat i on and t reat ment , when pos s i bl e, of t he caus es of del i ri um. An offendi ng agent may have t o be wi t hdrawn. Adequat e nut ri t i on s houl d be mai nt ai ned and t he s afet y of t he pat i ent ens ured. If neces s ary, s edat i on wi t h a l ow dos e of hal operi dol or a newer ant i ps ychot i c s uch as
Pa g e 2 6 4 1
ABC Ambe r CHM Conve rte r Tria l ve rsion, http://w w w .proce sste x t.com/a bcchm.html
ri s peri done, quet i api ne, or ol anzapi ne can be hel pful .
B. Dementia
1. Definition. Dement i a can be defi ned as a gl obal decl i ne i n cogni t i ve funct i on i n cl ear cons ci ous nes s . In es s ence, one s houl d be ext remel y caut i ous i n maki ng t he di agnos i s of dement i a i n a del i ri ous pat i ent .
2. Etiology. The caus es of dement i a are many. Ident i fi cat i on of a t reat abl e condi t i on mas queradi ng as a degenerat i ve proces s i s cri t i cal . o
o
a. Some causes of dementia i ncl ude Al zhei mer's di s eas e, Parki ns on's di s eas e, mul t i pl e cerebral i nfarct s , Hunt i ngt on's di s eas e, front ot emporal degenerat i on i ncl udi ng Pi ck's di s eas e, dement i a wi t h Lewy bodi es , human i mmunodefi ci ency vi rus (HIV) i nfect i on, and Creut zfel dt -Jakob di s eas e.
o
o
b. Potentially treatable conditions that can manifest as dementia i ncl ude depres s i on (ps eudodement i a), normal pres s ure hydrocephal us (NPH), s ubdural hemat oma, t umor, advers e drug effect s , t hyroi d di s eas e, vi t ami n B 1 2 defi ci ency, t hi ami ne defi ci ency, s yphi l i s , heavy met al P.526
i nt oxi cat i on, condi t i ons caus i ng hypers omni a (e.g., s l eep apnea s yndrome), chroni c meni ngi t i s , and W i l s on's di s eas e.
Pa g e 2 6 4 2
ABC Ambe r CHM Conve rte r Tria l ve rsion, http://w w w .proce sste x t.com/a bcchm.html
3. Alzheimer' s disease. Thi s condi t i on i s t he most common cause of chroni c dement i a. o
o
a. Definition. Al zhei mer's di s eas e i s a cl i ni copat hol ogi c ent i t y charact eri zed by progres s i ve memory l os s and ot her cogni t i ve defi ci t s . Ons et commonl y i s l at e i n l i fe, al t hough pat i ent s may be affect ed i n mi ddl e age.
(1) The di s eas e us ual l y ari s es s pont aneous l y, but genet i c fact ors have been i dent i fi ed. Fami l i al cas es have been as s oci at ed wi t h mut at i ons of t he genes for amyl oi d precurs or prot ei n, pres eni l i n 1, and pres eni l i n 2.
(2) There i s an as s oci at i on bet ween t he age of ons et of Al zhei mer's di s eas e and t he apol i poprot ei n E genot ype. Pat i ent s wi t h t he APOE4/4 genot ype have t he great es t ri s k for Al zhei mer's di s eas e at a gi ven age.
o
o
b. Prevalence. Al zhei mer's di s eas e i s a burgeoni ng publ i c heal t h probl em. It i s es t i mat ed t hat 60%–80% of dement ed pat i ent s have Al zhei mer's di s eas e. The preval ence i ncreas es s harpl y wi t h age, affect i ng 5%–15% of peopl e over age 65 and about t hree t i mes as many peopl e age 85 and ol der (t he fas t es t -growi ng s egment of t he popul at i on).
o
Pa g e 2 6 4 3
ABC Ambe r CHM Conve rte r Tria l ve rsion, http://w w w .proce sste x t.com/a bcchm.html o
c. Pathology. Al t hough t he caus e and pat hogenes i s are unknown, Al zhei mer's di s eas e has a charact eri s t i c pat hol ogy cons i s t i ng of intracellular neurofibrillary tangles and extracellular neuritic plaques.
(1) The t angl es are compos ed pri mari l y of abnormal l y phos phoryl at ed, mi crot ubul e-as s oci at ed t au prot ei ns .
(2) The amyl oi d prot ei n, Aβ, i s deri ved from amyl oi d precurs or prot ei n and i s depos i t ed i n s eni l e pl aques and bl ood ves s el s . The gene for amyl oi d precurs or prot ei n res i des on chromos ome 21 and may be i nvol ved i n fami l i al cas es .
(3) As s oci at ed pat hol ogi c proces s es di s t urb many neurot rans mi t t ers , part i cul arl y t he cholinergic system.
o
o
d. Diagnosis
(1) The clinical diagnosis of senile dementia of the Alzheimer' s type (SDAT ) can be made i f an ot herwi s e al ert pat i ent exhi bi t s progres s i ve memory l os s and ot her cogni t i ve defi ci t s s uch as di s ori ent at i on, l anguage di ffi cul t i es , i nabi l i t y t o perform compl ex mot or act i vi t i es , i nat t ent i on, vi s ual mi s percept i on, poor probl em-s ol vi ng abi l i t i es , i nappropri at e s oci al behavi or, and, occas i onal l y,
Pa g e 2 6 4 4
ABC Ambe r CHM Conve rte r Tria l ve rsion, http://w w w .proce sste x t.com/a bcchm.html
hal l uci nat i ons .
(a) The intellectual decline s houl d be pres ent i n t wo or more domai ns of cogni t i on and be document ed by cl i ni cal exami nat i ons s uch as t he mini-mental state examination. The ori gi nal exami nat i on was publ i s hed i n “Mi ni -Ment al St at e.― A Pract i cal Met hod for Gradi ng t he Cogni t i ve St at e of Pat i ent s for t he Cl i ni ci an. J Ps yc hi at r Res 1975;12(3):189–198. Tabl e 11-5 s hows s ampl e t es t i t ems and where t he t es t current l y may be obt ai ned.
TABLE 11-5 MMSE Sample Items Ori ent at i on to Ti me †œ Wh at is t he dat e?â €• Re
Pa g e 2 6 4 5
ABC Ambe r CHM Conve rte r Tria l ve rsion, http://w w w .proce sste x t.com/a bcchm.html
gi s t ra tio n â €œ Li s t en car efu l l y, I am goi ng to s ay t hr ee wor ds . Yo u s ay t he m bac k aft er I sto p. Re
Pa g e 2 6 4 6
ABC Ambe r CHM Conve rte r Tria l ve rsion, http://w w w .proce sste x t.com/a bcchm.html
ady ? Her e t he y are … AP PLE (pa us e ), PE NN Y (pa us e ), TA BL E (pa us e ). No w rep eat t ho se wor ds bac k
Pa g e 2 6 4 7
ABC Ambe r CHM Conve rte r Tria l ve rsion, http://w w w .proce sste x t.com/a bcchm.html
to me .†• [Re pe at up to 5 tim es , but s co re onl y t he fi rs t t ri a l .] Na mi ng †œ Wh at is t hi s ?â €• [Po
Pa g e 2 6 4 8
ABC Ambe r CHM Conve rte r Tria l ve rsion, http://w w w .proce sste x t.com/a bcchm.html
i nt to a pe nci l or pe n.] Re adi ng †œP l ea se rea d t hi s an d do wh at it s ay s .â €• [Sh ow exa mi ne e
Pa g e 2 6 4 9
ABC Ambe r CHM Conve rte r Tria l ve rsion, http://w w w .proce sste x t.com/a bcchm.html
t he wor ds on t he sti mu l us for m.] CL OS E YO UR EY ES †œR epr od uce d by s pe ci al per mi s sio n of t he Pu bl i s he
Pa g e 2 6 5 0
ABC Ambe r CHM Conve rte r Tria l ve rsion, http://w w w .proce sste x t.com/a bcchm.html
r, Ps y cho l og i cal As s es s me nt Re s ou rce s, Inc ., 16 20 4 Nor th Fl o ri d a Av en ue, Lut z, Fl o ri d a 33 54 9, fro
Pa g e 2 6 5 1
ABC Ambe r CHM Conve rte r Tria l ve rsion, http://w w w .proce sste x t.com/a bcchm.html
m t he Mi n i Me nt a l St a te Exa mi nat i on , by Mar s ha l Fol ste in an d Sus an Fol ste i n, Co pyr i gh t 19 75, 19
Pa g e 2 6 5 2
ABC Ambe r CHM Conve rte r Tria l ve rsion, http://w w w .proce sste x t.com/a bcchm.html
98, 20 01 by Mi n i Me nt a l LLC , Inc . Pu bl i s he d 20 01 by Ps y cho l og i cal As s es s me nt Re s ou rce s, Inc . Fur
Pa g e 2 6 5 3
ABC Ambe r CHM Conve rte r Tria l ve rsion, http://w w w .proce sste x t.com/a bcchm.html
t he r rep rod uct i on is pro hi b ite d wi t ho ut per mi s sio n of PA R, Inc . Th e MM SE can be pur cha s ed fro m PA
Pa g e 2 6 5 4
ABC Ambe r CHM Conve rte r Tria l ve rsion, http://w w w .proce sste x t.com/a bcchm.html
R, Inc . by cal l i ng (81 3) 96 8-3 00 3.â €•
P.527
(b) Formal neuropsychologic testing can confi rm t he cl i ni cal i mpres s i on and document progres s i on of t he di s eas e. Tes t s t hat addres s recal l (wi t h or wi t hout cues ) and del ayed recal l are es peci al l y s ens i t i ve for document i ng earl y memory i mpai rment .
(c) Ot her s ys t emi c and neurol ogi c di s eas es t hat coul d produce cogni t i ve decl i ne s houl d be abs ent .
(2) Differential diagnosis
(a) Pat i ent s wi t h pseudodementia
Pa g e 2 6 5 5
ABC Ambe r CHM Conve rte r Tria l ve rsion, http://w w w .proce sste x t.com/a bcchm.html
(depression) can exhi bi t many of t he feat ures of Al zhei mer's di s eas e. To compl i cat e mat t ers furt her, pat i ent s wi t h Al zhei mer's di s eas e may pres ent wi t h depres s i on. Ident i fi cat i on of pat i ent s wi t h ps eudodement i a i s i mport ant , becaus e t reat ment of t he depres s i on can res t ore cogni t i ve funct i on.
(i) A careful hi s t ory and neurops ychol ogi cal eval uat i on oft en can det ermi ne t he proper di agnos i s .
(ii) If doubt remai ns as t o t he rol e of depres s i on i n t he cl i ni cal pres ent at i on, appropri at e t reat ment for depres s i on i s warrant ed.
(b) Mild cognitive impairment
(i) Indi vi dual s wi t h mi l d cogni t i ve i mpai rment have a memory i mpai rment beyond t hat expect ed for normal agi ng. However, t hes e i ndi vi dual s do not yet meet cri t eri a for Al zhei mer's di s eas e.
(ii) Peopl e wi t h mi l d cogni t i ve i mpai rment evol ve t o Al zhei mer's di s eas e at a rat e of 10%–15% annual l y, compared wi t h 1%–2% annual l y for normal i ndi vi dual s .
Pa g e 2 6 5 6
ABC Ambe r CHM Conve rte r Tria l ve rsion, http://w w w .proce sste x t.com/a bcchm.html
(c) Other types of dementia
(i) Pick' s disease i s a front ot emporal dement i a charact eri zed by pers onal i t y changes , di s i nhi bi t i on, hyperoral i t y, and front ot emporal at rophy on i magi ng s t udi es . Hyperphos phoryl at ed t au prot ei n t hat accumul at es i n t he cerebral cort ex i s as s oci at ed wi t h t he di s eas e.
(ii) Dementia with Lewy bodies i s charact eri zed by cogni t i ve i mpai rment t hat can fl uct uat e, hal l uci nat i ons , and earl y parki ns oni an feat ures .
(iii) Frontotemporal dementia with parkinsonism linked to chromosome 17 (FT DP-17) i s charact eri zed by t he s ame behavi oral charact eri s t i cs as t he front ot emporal dement i a des cri bed for Pi ck's di s eas e. It i s as s oci at ed wi t h abnormal t au prot ei n.
o
o
d. T herapy
Pa g e 2 6 5 7
ABC Ambe r CHM Conve rte r Tria l ve rsion, http://w w w .proce sste x t.com/a bcchm.html
(1) Medical therapy i s us eful i n t reat i ng i ns omni a, agi t at i on, and depres s i on.
(a) In general , drugs s houl d i ni t i al l y be gi ven at a l ow dos e; t he dos e can be adjus t ed upward s l owl y as cl i ni cal l y i ndi cat ed.
(b) Medi cat i ons wi t h a s hort hal f-l i fe and few ant i chol i nergi c s i de effect s are bes t t ol erat ed.
(c) Chol i nes t eras e i nhi bi t ors s uch as donepezi l , gal ant ami ne, and ri vas t i gmi ne may i mprove cogni t i ve and behavi oral funct i on.
(2) Day care centers (i ncl udi ng day hos pi t al s ) and respite care are us eful adjunct s t o fami l y s upervi s i on of t he pat i ent wi t h Al zhei mer's di s eas e and ot her dement i ng di s orders .
4. Normal pressure hydrocephalus (NPH) o
o
a. Definition. NPH i s a condi t i on charact eri zed by a t ri ad of cognitive impairment, urinary incontinence, and gait apraxia (i .e., i mpai red ambul at i on wi t hout evi dence of pri mary mot or, s ens ory, or cerebel l ar dys funct i on).
o
Pa g e 2 6 5 8
ABC Ambe r CHM Conve rte r Tria l ve rsion, http://w w w .proce sste x t.com/a bcchm.html
o
b. Etiology. In mos t pat i ent s , t he caus e of NPH i s unknown. However, NPH can fol l ow SAH or meni ngi t i s , s omet i mes even years l at er.
o
o
c. Diagnosis. NPH s houl d be s us pect ed i n pat i ent s who pres ent wi t h t he cl i ni cal feat ures not ed i n III B 4 a. The fol l owi ng t es t s may hel p confi rm t he di agnos i s .
(1) Imaging studies
(a) CT or MRI reveal s vent ri cul ar enl argement wi t h rel at i vel y l i t t l e cort i cal at rophy (Fi gure 11-2).
(b) Cisternography i nvol ves i nject i ng a radi onucl i de i nt o t he l umbar t hecal s ac and t hen t aki ng s eri al det ermi nat i ons of t he fl ow pat t ern of t he radi oact i ve bol us . In t he pres ence of NPH, ci s t ernography demons t rat es pers i s t ent act i vi t y of t he radi onucl i de i n t he l at eral vent ri cl es aft er 48 hours .
P.528
Pa g e 2 6 5 9
ABC Ambe r CHM Conve rte r Tria l ve rsion, http://w w w .proce sste x t.com/a bcchm.html
FIGURE 11-2 A nonenhanced comput ed t omography (CT) s can s howi ng hydrocephal us cons i s t ent wi t h normal pres s ure hydrocephal us (NPH). Not e t he l ack of cort i cal at rophy (s ul cal effacement ).
(2) ICP monitoring for 24–48 hours can reveal t rans i ent pres s ure i ncreas es , i f t he di agnos i s i s i n doubt .
o
o
d. T herapy. Ins ert i on of a ventriculoperitoneal shunt can i mprove t he pat i ent 's condi t i on, es peci al l y i f performed wi t hi n 6 mont hs of t he ons et of t he probl em.
5. Creutzfeldt-Jakob disease. Thi s progres s i ve, degenerat i ve i l l nes s i s caus ed by prions (i .e., i nfect i ous prot ei naceous part i cl es ) and i s as s oci at ed wi t h a spongiform encephalopathy. o
Pa g e 2 6 6 0
ABC Ambe r CHM Conve rte r Tria l ve rsion, http://w w w .proce sste x t.com/a bcchm.html o
a. The gene for t he pri on prot ei n i s on chromosome 20; approxi mat el y 10% of cas es are heredi t ary. Il l nes s can devel op becaus e of infection or somatic and germ cell mutations. The CSF 14-3-3 prot ei n i s a marker for Creut zfel dt -Jakob di s eas e. “ Mad cow disease― probabl y repres ent s t rans mi s s i on of pri on di s eas e from i nfect ed cows t o humans vi a i nges t i on of bovi ne food product s .
o
o
b. Pat i ent s may exhi bi t myoclonus. The EEG oft en demons t rat es periodic discharges and an abnormal background rhyt hm.
o
o
c. Death us ual l y occurs wi t hi n s everal mont hs of t he ons et of t he di s eas e.
IV. Headache Many pat i ent s are concerned t hat t hei r headaches are caus ed by a l i fe-t hreat eni ng condi t i on s uch as a brai n t umor. Fort unat el y, t hi s i s rarel y t he cas e, but compl ai nt s of headache, whi ch are ext remel y common, al ways des erve furt her eval uat i on.
A. Etiology
1. Non-neurologic causes. Before as s umi ng t hat cephal i c di s comfort i s caus ed by an i nt racrani al di s order, t he phys i ci an s houl d cons i der t he pos s i bi l i t y of a non-neurol ogi c caus e. Di s orders of t he head and neck s uch as s i nus di s eas e, gl aucoma, dent al i nfect i ons , t emporomandi bul ar joi nt (TMJ) di s eas e, ear pat hol ogy, mus cul ar i njury, or cervi cal s pi ne probl ems can caus e headache.
Pa g e 2 6 6 1
ABC Ambe r CHM Conve rte r Tria l ve rsion, http://w w w .proce sste x t.com/a bcchm.html
2. Intracranial stimulation of pain-sensitive structures. Probl ems t hat affect t he meni nges or di s t ort t he l arger bl ood ves s el s caus e pai n. P.529
3. Life-threatening causes o
o
a. An intracranial mass caus es a headache t hat t ypi cal l y devel ops i ns i di ous l y and progres s i vel y wors ens .
(1) Clinical features. The pain i s unl i ke any t he pat i ent has experi enced and may awaken the person from s l eep. Occas i onal l y, t he headache i s wors e earl y i n t he day. W i t h t i me, associated symptoms (e.g., nausea, vomiting, exacerbation with lifting and straining) can devel op. On exami nat i on, evi dence of focal CNS disease i s t ypi cal l y apparent .
(2) T herapy. Treat ment i s di rect ed at t he underl yi ng l es i on.
o
o
b. A “ sentinel― SAH caus es t he apopl ect i c ons et of headache i n previ ous l y heal t hy i ndi vi dual s or t he s udden occurrence of headache t hat i s of uni que charact er i n a chroni c headache s ufferer (s ee al s o VIII C 1).
Pa g e 2 6 6 2
ABC Ambe r CHM Conve rte r Tria l ve rsion, http://w w w .proce sste x t.com/a bcchm.html
(1) Clinical features. The pos s i bi l i t y t hat a headache i s a res ul t of SAH i s s t rengt hened i f t he cephal i c di s comfort cannot be eas i l y at t ri but ed t o any of t he us ual caus es of head pai n. No neurol ogi c fi ndi ngs may be pres ent on exami nat i on, and meni ngi s mus may be abs ent .
(2) Diagnosis. Gi ven t he pot ent i al s eri ous nes s of t he condi t i on, pat i ent s s houl d have a cranial CT scan. If t hi s i s unreveal i ng, an LP document s t he pres ence of s ubarachnoi d bl eedi ng.
B. Headache syndromes
1. Migraine o
o
a. Etiology. The caus e of mi grai ne i s unknown, but s everal common preci pi t ant s have been obs erved.
(1) A family history of mi grai ne oft en exi s t s .
(2) Headaches can be rel at ed t o stress, altered sleep patterns, menses, oral contraceptives, alcohol use, caffeine withdrawal, monosodium glutamate (MSG) intake, and vari ous foodstuffs (e.g., chocol at e, nut s , aged chees es , and meat s cont ai ni ng ni t rat es ).
Pa g e 2 6 6 3
ABC Ambe r CHM Conve rte r Tria l ve rsion, http://w w w .proce sste x t.com/a bcchm.html
(3) Mi grai ne can devel op after seemingly minor head trauma; recogni t i on and t reat ment may prevent prol onged di s abi l i t y.
o
o
b. Pathophysiology. Hypot hes es cent er around t he i dea t hat a mi grai ne at t ack i s brought on by neurovas cul ar di s t urbances .
(1) The cl as s i c vasospasm–vasodilation theory aros e from cl i ni cal obs ervat i ons . Recent dat a s ugges t t hat ol i gemi a, s econdary t o a s l owl y s preadi ng area of neuronal depolarization (t he cort i cal depres s i on of Leão), occurs duri ng a headache prodrome and pers i s t s i nt o t he headache phas e. Hyperemi a occurs s ubs equent l y and can pers i s t aft er t he headache s ubs i des .
(2) Current t heory mai nt ai ns t hat dysfunction of t he trigeminovascular system, res ul t i ng i n t he peri vas cul ar rel eas e of s ubs t ance P and ot her neurot rans mi t t ers and i nfl ammat ory markers , l eads t o mi grai ne.
o
o
c. Migraine syndromes
(1) Migraine without aura (common migraine) i s an intermittent s yndrome charact eri zed by general i zed or hemi crani al pulsatile cephal i c di s comfort . Naus ea,
Pa g e 2 6 6 4
ABC Ambe r CHM Conve rte r Tria l ve rsion, http://w w w .proce sste x t.com/a bcchm.html
vomi t i ng, phot ophobi a, phonophobi a, and anorexi a may accompany t he headache.
(2) Migraine with aura (classic migraine) pres ent s wi t h an aura, oft en a vivid visual array of col ors i n a geomet ri c pat t ern i nvol vi ng one vi s ual hemi fi el d.
(a) The throbbing headache i s oft en contralateral to the visual display, and naus ea, vomi t i ng, phot ophobi a, phonophobi a, and anorexi a may be pres ent .
(b) Mi grai ne wi t h aura al s o can be as s oci at ed wi t h t rans i ent neurol ogi c defi ci t s s uch as vi s ual fi el d defi ci t s and hemi s ens ory l os s .
(c) On very rare occas i ons , s t roke i s a compl i cat i on of mi grai ne.
o
o
d. T herapy. Treat ment s houl d fi rs t i nvol ve removal of i nci t i ng agent s when pos s i bl e.
(1) Abortive therapy for migraine
(a) Ergotamine, avai l abl e i n oral , s ubl i ngual , nas al , and s uppos i t ory forms , i s a s erot oni n (5-HT 1 )–recept or agoni s t
Pa g e 2 6 6 5
ABC Ambe r CHM Conve rte r Tria l ve rsion, http://w w w .proce sste x t.com/a bcchm.html
t hat decreas es s ubs t ance P rel eas e at t he t ri gemi novas cul ar junct i on. Int ravenous ergot ami ne [di hydroergot ami ne (DHE 45)] al s o has proved t o be effi caci ous ; pret reat ment wi t h met ocl oprami de or prochl orperaz i ne prevent s naus ea. P.530
(b) Aspirin, nonsteroidal anti-inflammatory drugs (NSAIDs), and isometheptene can abort a mi grai ne. Anal ges i cs may be admi ni s t ered for s ympt omat i c rel i ef as wel l .
(c) T riptans, a fami l y of s erot oni n 5-HT 1 –recept or agoni s t s , are effect i ve.
(2) Prophylactic measures i ncl ude drug regi mens and changes i n pat i ent behavi or referabl e t o headache preci pi t ant s .
(a) Medications s uch as β-bl ockers , t ri cycl i c ant i depres s ant s , cal ci um channel bl ockers , NSAIDs , gabapent i n, t opi ramat e, or val proi c aci d may be us ed t o prevent mi grai nes . Prophyl act i c medi cat i ons , al t hough of s eemi ngl y di vers e t ypes , al l have a t endency t o al t er CNS s erot oni n act i vi t y.
Pa g e 2 6 6 6
ABC Ambe r CHM Conve rte r Tria l ve rsion, http://w w w .proce sste x t.com/a bcchm.html
(i) The choi ce of medi cat i on i s gui ded, i n part , by t he need t o avoi d or expl oi t a part i cul ar drug act i on (as i de from t he ant i headache effect ).
(ii) Ini t i al l y, a l ow dos e s houl d be admi ni s t ered, and t he t herapeut i c benefi t s and undes i rabl e s i de effect s s houl d be moni t ored as t he dos e i s i ncreas ed. The dos e can be i ncreas ed unt i l ei t her a benefi ci al res pons e i s achi eved or advers e s i de effect s devel op. A maxi mal dos e i s bes t mai nt ai ned for s everal weeks before concl udi ng t hat an agent i s not effect i ve.
(b) Biofeedback therapy may enabl e pat i ent s t o l es s en mi grai ne event s by hel pi ng t hem deal more effect i vel y wi t h s t res s .
2. A muscle contraction, or tension, headache i s charact eri zed by a band-l i ke di s comfort about t he head. o
o
a. Clinical features. Thi s t ype of headache oft en devel ops duri ng t he cours e of t he day and may be as s oci at ed wi t h emot i onal s t res s . Pos t eri or cervi cal and occi pi t al mus cl es are oft en t ender and may be i n s pas m. The di s t i nct i on bet ween t hi s t ype of headache and mi grai ne wi t hout aura can be
Pa g e 2 6 6 7
ABC Ambe r CHM Conve rte r Tria l ve rsion, http://w w w .proce sste x t.com/a bcchm.html
di ffi cul t . o
o
b. T herapy. Treat ment i nvol ves reas s urance, NSAIDs , mus cl e rel axant s , moi s t heat , and, on occas i on, ant i depres s ant drugs and ps ychot herapy.
3. Chronic daily headache o
o
a. Etiology. Pat i ent s wi t h mi grai ne or t ens i on headache can devel op chroni c dai l y headaches s pont aneous l y or as a res ul t of exces s i ve us e of anal ges i cs or ergot ami nes .
o
o
b. T herapy. Treat ment cons i s t s of withdrawal from excessive medications. Intravenous DHE 45 gi ven for 2–3 days can hel p break t he headache cycl e. Prophylactic migraine agents can hel p prevent a headache recurrence.
4. Cluster headache o
o
a. Clinical features. Cl us t er headaches are s evere peri orbi t al headaches , 30–90 mi nut es i n durat i on, t hat occur once or s everal t i mes dai l y over a peri od of s everal weeks or mont hs . The uni l at eral pai n may be accompani ed by i ps i l at eral l acri mat i on, conjunct i val i nject i on, nas al conges t i on, and Horner's s yndrome. The t ypi cal pat i ent i s a mi ddl e-aged man. Pat i ent s wi t h cl us t er headaches oft en pace, as oppos ed t o mi grai neurs , who s eek qui et , dark pl aces .
Pa g e 2 6 6 8
ABC Ambe r CHM Conve rte r Tria l ve rsion, http://w w w .proce sste x t.com/a bcchm.html o
o
b. T herapy
(1) Abortive and symptomatic treatment i ncl udes t he admi ni s t rat i on of 100% oxygen, ergot ami nes , anal ges i cs , or s umat ri pt an.
(2) Prophylactic therapy i ncorporat es l i t hi um, cal ci um channel bl ockers , or cort i cos t eroi ds .
5. T emporal (giant cell) arteritis (s ee al s o Chapt er 10 XII D 1) o
o
a. Clinical features. Pat i ent s over t he age of 50 years who compl ai n of a headache cent ered about one t empl e or l ocat ed i n t he occi pi t al area s houl d be eval uat ed for gi ant cel l art eri t i s . As s oci at ed s ympt oms i ncl ude vi s ual di s t urbances , jaw cl audi cat i on, fever, art hral gi as and myal gi as , and wei ght l os s . Pol ymyal gi a rheumat i ca (PMR) i s al s o pres ent i n approxi mat el y 50% of pat i ent s wi t h gi ant cel l art eri t i s .
o
o
b. Diagnosis. The eryt hrocyt e s edi ment at i on rat e i s t ypi cal l y great er t han 40 mm/hour, and t he C-react i ve prot ei n l evel i s el evat ed. Bi ops y of a t emporal art ery confi rms t he di agnos i s .
o
o
c. T herapy. Corticosteroid treatment can bri ng
Pa g e 2 6 6 9
ABC Ambe r CHM Conve rte r Tria l ve rsion, http://w w w .proce sste x t.com/a bcchm.html
rapi d rel i ef.
6. Benign (idiopathic) intracranial hypertension (pseudotumor cerebri) has no known caus e but i s as s oci at ed wi t h obes i t y, pregnancy, oral cont racept i ves , SLE, crani al venous s i nus t hrombos i s , and a hos t of ot her condi t i ons . P.531
o
o
a. Clinical features. The devel opment of a rel at i vel y cons t ant , general i zed headache i n pat i ent s wi t h a cl ear s ens ori um, papi l l edema, and an ot herwi s e normal neurol ogi c exami nat i on i s s ugges t i ve of beni gn i nt racrani al hypert ens i on. Vi s ual obs curat i ons can occur, and vi s ual l os s i s t he mos t s eri ous compl i cat i on.
o
o
b. Diagnosis. The di agnos i s i s s ugges t ed by a CT or MRI s can, whi ch i s normal . The di agnos i s can be confi rmed by fi ndi ng an el evat ed CSF openi ng pres s ure and an ot herwi s e normal CSF anal ys i s .
o
o
c. T herapy
(1) Vi s ual acui t y and fi el ds s houl d be moni t ored.
(2) Serial LPs can rel i eve t he s yndrome.
Pa g e 2 6 7 0
ABC Ambe r CHM Conve rte r Tria l ve rsion, http://w w w .proce sste x t.com/a bcchm.html
(3) Corticosteroids, acetazolamide, or furosemide may be admi ni s t ered.
(4) Refract ory di s eas e has been managed wi t h lumboperitoneal shunting of CSF or optic nerve sheath fenestration.
7. T rigeminal neuralgia (tic douloureux) i s a s yndrome t hat mos t oft en i s i di opat hi c but has been as s oci at ed wi t h MS, neopl as i a, and vas cul ar “l oops ― t hat i mpi nge on t he t ri gemi nal nerve. o
o
a. Clinical features. Li ght ni ng-qui ck, s evere faci al pai n, oft en as s oci at ed wi t h a t ri gger poi nt , i s s ugges t i ve of t ri gemi nal neural gi a. The pai nful jabs are us ual l y res t ri ct ed t o one or t wo di vi s i ons of t he t ri gemi nal nerve. There i s no l os s of faci al s ens at i on.
o
o
b. T herapy. Therapeut i c modal i t i es i ncl ude carbamazepine, baclofen, gabapentin, surgical intervention, and stereotactic radiation therapy.
8. Indomethacin-responsive headaches are charact eri zed by s evere, uni l at eral pai n t hat may be rel i eved by t reat ment wi t h i ndomet haci n. o
o
a. Chronic paroxysmal hemicrania. i s charact eri zed by pai nful , mul t i pl e at t acks (up t o 40
Pa g e 2 6 7 1
ABC Ambe r CHM Conve rte r Tria l ve rsion, http://w w w .proce sste x t.com/a bcchm.html
dai l y) t hat l as t from 2 mi nut es t o 2 hours wi t h noct urnal awakeni ngs and as s oci at ed aut onomi c feat ures . Al cohol can preci pi t at e t he at t acks . o
o
b. Episodic paroxysmal hemicrania i s charact eri zed by pai nful , mul t i pl e at t acks (6–30 dai l y) t hat l as t up t o 30 mi nut es and are as s oci at ed wi t h noct urnal awakeni ngs and ot her aut onomi c feat ures . Remi s s i ons l as t i ng mont hs t o years can occur.
9. Low pressure–volume headache i s charact eri zed by a headache t hat s ubs t ant i al l y wors ens when i n t he upri ght pos i t i on and convers el y i mproves wi t h a s upi ne pos t ure. o
o
a. Etiology. Pos t ural headaches t ypi cal l y occur aft er LP but can devel op s pont aneous l y or i n as s oci at i on wi t h a defect i n t he i nt egri t y of t he dura.
o
o
b. Diagnosis. MRI s hows general i zed dural enhancement . The cerebel l ar t ons i l s may have des cended bel ow t he l evel of t he foramen magnum, and t he brai n s t em may be “ki nked.―
o
o
c. T herapy. Treat ment cons i s t s of a blood patch (i nject i on of aut ol ogous bl ood i nt o t he l umbar epi dural s pace).
V. Weakness Many di s orders can caus e weaknes s . To pi npoi nt t he caus at i ve
Pa g e 2 6 7 2
ABC Ambe r CHM Conve rte r Tria l ve rsion, http://w w w .proce sste x t.com/a bcchm.html
di s order, t he phys i ci an mus t fi rs t det ermi ne whi ch part of t he nervous s ys t em i s di s eas ed.
A. Anatomic and functional approach W eaknes s can res ul t from dys funct i on at vari ous poi nt s i n t he cent ral (CNS) or peri pheral nervous s ys t em (PNS). Speci fi c l ocal i z at i on depends upon recogni t i on of t he pat t ern of weaknes s and as s oci at ed fi ndi ngs .
1. CNS disorders. Dys funct i on of t he pyrami dal t racks or upper mot or neurons i s cl as s i cal l y as s oci at ed wi t h “upper mot or neuron s i gns ― and “pyrami dal weaknes s .― o
o
a. Upper mot or neuron s i gns cons i s t of i ncreas ed refl exes , up-goi ng t oes , and i ncreas ed t one (s pas t i ci t y).
o
o
b. Pyrami dal weaknes s refers t o a pat t ern whereby t he ext ens ors are weaker t han t he fl exors i n t he upper ext remi t y, whereas t he fl exors are weaker t han t he ext ens ors i n t he l ower ext remi t y.
2. PNS disorders. Dys funct i on of t he mot or uni t (l ower mot or neurons and t he mus cl e fi bers t hey i nnervat e) i s as s oci at ed wi t h “l ower mot or neuron s i gns ― and “s i t e-s peci fi c weaknes s .― o
o
a. Lower mot or neuron s i gns refer t o reduced or abs ent refl exes , down-goi ng or mut e t oes , and decreas ed t one, as wel l as at rophy and fas ci cul at i ons (i nvol unt ary mus cl e t wi t ches ).
Pa g e 2 6 7 3
ABC Ambe r CHM Conve rte r Tria l ve rsion, http://w w w .proce sste x t.com/a bcchm.html
P.532
o
o
b. Si t e-s peci fi c weaknes s refers t o t he s peci fi c pat t erns as s oci at ed wi t h pat hol ogy al ong t he vari ous s i t es of t he mot or uni t .
(i) Lower mot or neuron di s eas e s uch as pol i o may produce weaknes s at vari ous s i t es .
(ii) Nerve root di s eas e wi l l produce weaknes s i n a “myot omal ― pat t ern aki n t o t he dermat omal s ens ory l os s .
(iii) Di s orders of t he neuromus cul ar junct i on general l y produce proxi mal weaknes s .
(iv) Di s orders of mus cl e (myopat hi es ) al s o produce proxi mal weaknes s .
B. Specific patterns of CNS pathology by anatomic location (Each pat t ern s houl d be as s oci at ed wi t h upper mot or neuron s i gns and pyrami dal weaknes s .)
1. Cortical (precent ral gyrus ) l es i ons may be di s t i ngui s hed by as s oci at ed changes on ment al s t at us exami nat i on. o
Pa g e 2 6 7 4
ABC Ambe r CHM Conve rte r Tria l ve rsion, http://w w w .proce sste x t.com/a bcchm.html o
a. Smal l l es i ons may be as s oci at ed wi t h monopl egi a and pos s i bl e s ens ory l os s i n s ame area.
o
o
b. Large hemi s pheri c l es i ons (e.g., MCA s t oke) s houl d produce a hemi pares i s , hemi s ens ory l os s , and a homonymous hemi anopi a. Aphas i a i s commonl y s een wi t h l eft -s i ded l es i ons whi l e negl ect may be s een wi t h ri ght -s i ded l es i ons .
2. Subcortical lesions (i n t he deep whi t e mat t er or pos t eri or l i mb of t he i nt ernal caps ul e) cl as s i cal l y produce a hemi pares i s wi t h t he pat t ern of face=arm=l eg, wi t h pos s i bl e hemi s ens ory l os s , but no vi s ual changes and onl y rare ment al s t at us changes
3. Midbrain l es i ons (i n t he cerebral peduncl es ) are rarel y s een i n i s ol at i on, but may caus e a s ubcort i cal pat t ern of weaknes s (face=arm=l eg) wi t h as s oci at ed t hi rd nerve dys funct i on or marked t remor.
4. Pontine l es i ons produce a s ubcort i cal pat t ern of weaknes s (face=arm=l eg), pos s i bl e hemi s ens ory l os s , and pos s i bl e crani al nerve defect s .
5. Medullary l es i ons are rarel y s een i n i s ol at i on; vas cul ar l es i ons us ual l y caus e a cont ral at eral hemi pares i s s pari ng t he face, i ps i l at eral t ongue devi at i on and cont ral at eral l os s of vi brat i on and propri ocept i on (medi al medul l ary s yndrome).
Pa g e 2 6 7 5
ABC Ambe r CHM Conve rte r Tria l ve rsion, http://w w w .proce sste x t.com/a bcchm.html
6. Spinal cord l es i ons (i nvol vi ng t he cort i cos pi nal t ract s ) caus e i ps i l at eral weaknes s bel ow t he l evel of t he l es i on.
C. Specific patterns of PNS pathology Speci fi c pat t erns of PNS pat hol ogy by anat omi c l ocat i on. (Each pat t ern may have s ome l ower mot or neuron s i gns , but “s i t e-s peci fi c― weaknes s .)
1. Motor neuron diseases (di s eas es of t he ant eri or horn cel l s uch as amyot rophi c l at eral s cl eros i s [ALS]) are pure mot or di s t urbances wi t h s evere general i zed weaknes s , was t i ng, hyporefl exi a, and fas ci cul at i ons .
2. Radiculopathies (nerve root di s orders ) produce weaknes s and s ens ory l os s i n t he di s t ri but i on of a nerve root (“myot omal ― weaknes s and “dermat omal ― s ens ory l os s res pect i vel y) as wel l as radi cul ar or s hoot i ng pai n.
3. Plexopathies (e.g., brachi al , l umbar, or s acral ) produce weaknes s , numbnes s , hyporefl exi a and pai n i n an ent i re l i mb.
4. Neuropathies general l y produce di s t al numbnes s , weaknes s and refl ex l os s .
5. Neuromuscular junction disorders s uch as myas t heni a gravi s are pure mot or di s orders charact eri zed by proxi mal weaknes s .
6. Myopathies (di s orders of mus cl e) are pure mot or
Pa g e 2 6 7 6
ABC Ambe r CHM Conve rte r Tria l ve rsion, http://w w w .proce sste x t.com/a bcchm.html
s yndromes us ual l y charact eri zed by proxi mal weaknes s , wi t h hyporefl exi a onl y i n very weak mus cl es .
VI. Disequilibrium and Dizziness A. Ataxia At axi a (gai t i ns t abi l i t y) can be caus ed by cerebel l ar dys funct i on or di s orders of t he mot or or s ens ory s ys t em.
1. Cerebellar dysfunction. In addi t i on t o cl ums i nes s and l i mb or t runcal i ns t abi l i t y, as mani fes t ed by i ncoordi nat i on and i mpai red s t ance and gai t , pat i ent s may exhi bi t hypot oni a and ocul ar dys met ri a. P.533
o
o
a. Acute cerebellar dysfunction. (e.g., s t roke, MS, neopl as i a) t ypi cal l y mani fes t s wi t h uni l at eral fi ndi ngs .
o
o
b. Caus es of rel at i vel y s ymmet ri c subacute or chronic cerebellar dysfunction i ncl ude al cohol i c cerebel l ar degenerat i on; drug i nt oxi cat i on (e.g., phenyt oi n); cerebel l ar or s pi nocerebel l ar degenerat i ons ; and paraneopl as t i c, i mmune-medi at ed degenerat i on.
2. Motor or sensory disorders. Pat i ent s wi t h s t rengt h i n t he 2/5–4/5 grade range (s ee Tabl e 11-3) may appear cl ums y, as may pat i ent s wi t h a s ens ory neuropat hy, dors al root gangl i on di s eas e, pos t eri or
Pa g e 2 6 7 7
ABC Ambe r CHM Conve rte r Tria l ve rsion, http://w w w .proce sste x t.com/a bcchm.html
col umn dys funct i on, or pari et al l obe di s eas e (pari et al at axi a).
B. Dizziness
1. Approach to the patient o
o
a. History. Obt ai ni ng a t horough hi s t ory ai ds i n pat i ent eval uat i on. By payi ng s peci al at t ent i on t o as s oci at ed s ympt oms , t he phys i ci an may be abl e t o i dent i fy pat hol ogy of t he i nner ear, ei ght h crani al nerve, or CNS as t he caus e of t he di zzi nes s .
o
o
b. Physical examination. The phys i ci an us es s everal s creeni ng t echni ques t o s earch for evi dence of s t ruct ural di s eas e.
(1) Nystagmus i s pres ent i n many pat i ent s who have vert i go. The nys t agmus can be obs erved s pont aneous l y (when t he pat i ent gazes s t rai ght ahead, l at eral l y, or vert i cal l y), or i t may be el i ci t ed by us i ng Frenz el l ens es t o bl ock vi s ual fi xat i on, by havi ng t he pat i ent s hake hi s or her head duri ng opht hal mos copy, or by coveri ng t he cont ral at eral eye duri ng opht hal mos copy (t o decreas e vi s ual fi xat i on).
(a) Horizontal or vertical nystagmus i s charact eri zed by a s l ow eye dri ft and a rapi d s hi ft i n t he oppos i t e di rect i on.
Pa g e 2 6 7 8
ABC Ambe r CHM Conve rte r Tria l ve rsion, http://w w w .proce sste x t.com/a bcchm.html
(b) T orsional nystagmus i s charact eri zed by a s l ow cl ockwi s e or count ercl ockwi s e rot at i on and a rapi d movement i n t he oppos i t e di rect i on.
(c) The di rect i on of nys t agmus refers t o t he di rect i on of t he fas t component of eye movement . Nys t agmus i s away from t he affect ed ear (i .e., s l ow phas e of eye movement i s t oward t he s i de of t he l es i on). It i s oft en of a mi xed hori zont al and t ors i onal t ype.
(2) Gaze mechanisms s houl d be t es t ed, i ncl udi ng saccadic speed and accuracy, and t he i nt egri t y of smooth pursuits. The pres ence of ocular dysmetria and visual acuity whi l e s haki ng t he head s houl d be not ed. Suppression of the vestibulo-ocular reflex (whi ch may be t es t ed by as ki ng t he pat i ent t o fi xat e on a t arget movi ng s ynchronous l y wi t h t he head) s houl d be not ed as wel l . The head thrust maneuver may demons t rat e abnormal correct i ve s accades .
(3) The phys i ci an s houl d check for t he pres ence of benign paroxysmal positional vertigo (BPPV) by performi ng t he Nyl en-Bárány maneuver (i .e., by hyperext endi ng t he pat i ent 's neck and rot at i ng t he pat i ent 's head l at eral l y as t he pat i ent i s rapi dl y moved from a s eat ed t o a s upi ne pos i t i on). Rot ary nys t agmus wi t h a
Pa g e 2 6 7 9
ABC Ambe r CHM Conve rte r Tria l ve rsion, http://w w w .proce sste x t.com/a bcchm.html
l i near component t hat appears aft er a s everal -s econd l at ency peri od s ugges t s BPPV.
(4) Vert i go t hat wors ens when pres s ure i s appl i ed t o t he t ragus may s ugges t a peri l ymphat i c fi s t ul a.
2. Selected causes of dizziness o
o
a. Labyrinthitis (vestibular neuronitis). i s an i nfl ammat ory condi t i on of t he i nner ear charact eri zed by t he acut e ons et of a s pi nni ng s ens at i on (vert i go), exacerbat ed by movement , and as s oci at ed wi t h naus ea and vomi t i ng. The condi t i on i s us ual l y fel t t o be vi ral or bact eri al i n ori gi n.
o
o
b. Ménière' s disease i s charact eri zed by epi s odi c vert i go accompani ed by nys t agmus , t i nni t us , fl uct uat i ng heari ng l os s , and aural di s comfort .
o
o
c. BPPV occurs when t he pat i ent moves hi s or her head (e.g., by rol l i ng over i n bed, l ooki ng up, or s t andi ng up). Thi s condi t i on i s us ual l y i di opat hi c i n nat ure but can be caus ed by t rauma, vi ral or i s chemi c i njury, or drug t oxi ci t y. A uni que phys i cal t herapy t echni que t hat can be us ed t o decreas e s ympt oms of BPPV has been devel oped.
o
o
d. Drug-induced vestibulopathy can be caus ed by gent ami ci n, s t rept omyci n, furos emi de, and
Pa g e 2 6 8 0
ABC Ambe r CHM Conve rte r Tria l ve rsion, http://w w w .proce sste x t.com/a bcchm.html
ci s pl at i n. P.534
o
o
e. Eighth cranial nerve disease can be as s oci at ed wi t h heari ng l os s . Caus es i ncl ude acous t i c neuroma, met as t at i c di s eas e, vas cul i t i s , and bas i l ar meni ngi t i s as s oci at ed wi t h i nfect i ous and i nfl ammat ory proces s es .
o
o
f. Lateral medullary syndrome (Wallenberg' s syndrome) i s due t o occl us i on of t he vert ebral or pos t eri or i nferi or cerebel l ar art ery.
(1) Pat i ent s oft en feel t hat t he ext ernal worl d i s “t i l t i ng― and compl ai n of feel i ng propel l ed t oward one s i de (us ual l y t he s i de of t he l es i on).
(2) Phys i cal exami nat i on can i ndi cat e nys t agmus (oft en hori zont al –t ors i onal and away from t he l es i on), i ps i l at eral at axi a, i ps i l at eral Horner's s yndrome (charact eri zed by pt os i s , mi os i s , and anhi dros i s ), and i ps i l at eral faci al s ens ory i mpai rment wi t h cont ral at eral body s ens ory l os s .
o
o
g. Cerebellar disease can be caus ed by s t roke, t umor, i nfect i on, or degenerat i ve and i nfl ammat ory di s eas e (s ee XVIII H). Paraneopl as t i c cerebel l ar degenerat i on i s as s oci at ed wi t h ant i -Y o ant i bodi es
Pa g e 2 6 8 1
ABC Ambe r CHM Conve rte r Tria l ve rsion, http://w w w .proce sste x t.com/a bcchm.html
(l eadi ng t o Purki nje cel l l os s ) and ant i -Ri (ant i neuronal ) ant i bodi es . Phys i cal exami nat i on i ndi cat es at axi a and pos t ural i ns t abi l i t y and many t ypes of nys t agmus . Ops ocl onus i s as s oci at ed wi t h ant i -Ri ant i bodi es . o
o
h. Brainstem lesions are t ypi cal l y as s oci at ed wi t h crani al nerve, mot or, s ens ory, and cerebel l ar dys funct i on.
o
o
i. Other causes of dizziness i ncl ude pos t ural hypot ens i on, hypervent i l at i on, hypogl ycemi a, hypot hyroi di s m, epi l eps y, mi grai ne, ps ychi at ri c di s orders (e.g., depres s i on, anxi et y, and s omat i zat i on di s orders ), and mul t i s ens ory i mpai rment (i .e., poor vi s i on and pos i t i onal s ens e).
3. T herapy o
o
a. Di zzi nes s may be rel i eved by treating the underlying disease. If t he underl yi ng di s eas e i s unknown or ot herwi s e unt reat abl e, acute symptomatic therapy i ncl udes t he us e of mecl i zi ne, prochl orperaz i ne, promet hazi ne, t opi cal s copol ami ne, and di azepam.
o
o
b. Vestibular and gait exercises may hel p habi t uat e t he ves t i bul ar pat hways t o t he abnormal i nfl uences of t he di s eas e.
o
o
c. Chronic, poorly defined causes of di zzi nes s can
Pa g e 2 6 8 2
ABC Ambe r CHM Conve rte r Tria l ve rsion, http://w w w .proce sste x t.com/a bcchm.html
be managed by wi t hdrawi ng t he pat i ent from drugs t hat affect t he ves t i bul ar s ys t em. Thi s act i on may al l ow a bet t er reeval uat i on of t he pat i ent . Treat ment wi t h a low dose of a benzodiazepine and vestibular and gait exercises may be effect i ve.
VII. Pain Syndromes W hen pat i ent s compl ai n of pai n, i t i s i mport ant t o expl ore whet her t he pai n i s new-ons et , l ong-s t andi ng, or i nt ermi t t ent , and whet her i t i s s t at i c or progres s i ve. Pat i ent s s houl d be as ked t o i dent i fy exacerbat i ng and al l evi at i ng fact ors as wel l as as s oci at ed s ympt oms s uch as weaknes s or numbnes s .
A. Pain originating from the lower back
1. Etiology o
o
a. Di agnos t i c cons i derat i ons i ncl ude l ocal mus cul ar s t rai n, t raumat i c condi t i ons of t he s pi ne, degenerat i ve and i nfl ammat ory art hri t i des , neopl as t i c and i nfect i ous proces s es , and nerve root i rri t at i on.
o
o
b. Neurol ogi c dys funct i on commonl y devel ops from i mpi ngement on nerve root s by di s k mat eri al , bony overgrowt h, or t hi ckened l i gament s .
2. Disorders associated with low back pain o
o
a. Sciatica. denot es a s yndrome of s harp pai n radi at i ng from t he l ow back t o t he but t ock, down
Pa g e 2 6 8 3
ABC Ambe r CHM Conve rte r Tria l ve rsion, http://w w w .proce sste x t.com/a bcchm.html
t he back of t he t hi gh t o t he cal f, and, at t i mes , over t he bot t om or t op of t he foot . Sci at i ca i s commonl y as s oci at ed wi t h di s k herni at i on but can res ul t from ot her condi t i ons t hat i rri t at e nerve root s L4, L5, or S1.
(1) Clinical features. Myot omal weaknes s and dermat omal numbnes s , al ong wi t h appropri at e mus cl e s t ret ch refl ex changes and pai n t hat i ncreas es when t he exami ner rai s es t he pat i ent 's ext ended l eg, can occur.
(2) Diagnosis. Pers i s t ent s ympt oms and neurol ogi c defi ci t s warrant i n-dept h i nves t i gat i on, whi ch commonl y i ncl udes t he us e of l umbos acral radi ographs , l umbar MRI, and EMG. CT myel ography may be i ndi cat ed for bet t er defi ni t i on of t he bony anat omy. P.535
(3) T herapy. Acut e l ow back pai n s houl d be eval uat ed careful l y. Treat ment s houl d be di rect ed at t he underl yi ng di s order i f pos s i bl e.
(a) Unl es s t umor or i nfect i on i s t he l i kel y caus e for s ci at i ca, i ni t i al t herapy s houl d i ncl udeavoi dance of s t renuous act i vi t y and t reat ment wi t h NSAIDs. If pat i ent s s how s i gns of a cauda equina syndrome (i .e., a fl acci d bl adder, rect al dys funct i on, and bi l at eral l ower mot or
Pa g e 2 6 8 4
ABC Ambe r CHM Conve rte r Tria l ve rsion, http://w w w .proce sste x t.com/a bcchm.html
neuron l eg weaknes s ), emergency surgery i s us ual l y requi red.
(b) Long-term therapy may be appropri at e. As t he acut e s yndrome remi t s , a regi men of back-strengthening exercises and weight loss (i f appl i cabl e) can be hel pful . Surgery (e.g., laminectomy) may be neces s ary t o rel i eve pers i s t ent s ympt oms .
o
o
b. Lumbar stenosis i s anot her common condi t i on res ul t i ng i n l ow back pai n. Pat i ent s oft en have a congeni t al l y s mal l l umbar canal . Over t i me, bony and l i gament ous overgrowt h and di s k prot rus i ons may furt her encroach on t he neural and vas cul ar cont ent s of t he canal and forami na.
(1) Clinical features. Pat i ent s devel op pai n and, occas i onal l y, sensory loss and weakness wi t h ambulation and prolonged standing, but t hey are rel at i vel y comfort abl e at res t . The neurol ogi c exami nat i on of res t i ng pat i ent s i s oft en qui t e normal .
(2) T herapy. It i s neces s ary t o di fferent i at e pai n caus ed by l umbar s t enos i s from t hat caus ed by vas cul ar i ns uffi ci ency. If s ympt oms are s evere, decompression surgery i s oft en purs ued for t reat ment of l umbar s t enos i s .
B. Pain originating from the neck
Pa g e 2 6 8 5
ABC Ambe r CHM Conve rte r Tria l ve rsion, http://w w w .proce sste x t.com/a bcchm.html
1. Etiology. Neck pai n i s oft en due t o t rauma (e.g., “whi pl as h―). Degenerat i ve and i nfl ammat ory bone and di s k di s eas e can l ead t o nerve root i rri t at i on and i s al s o a common s ource of neck pai n. Local i nfect i on s houl d be cons i dered for acut e s ympt oms ; a neopl as m may be t he cul pri t when s ubacut e s ympt oms exi s t .
2. Disorders associated with neck pain o
o
a. Cervical radiculopathy. Pai n can radi at e i n a dermat omal pat t ern wi t h a paral l el l os s of s ens at i on. W eaknes s may be i n a myot omal di s t ri but i on wi t h a corres pondi ng l os s of mus cl e s t ret ch refl exes .
o
o
b. Arthritis. The cervi cal radi cul opat hy as s oci at ed wi t h cervi cal art hri t i s (s pondyl os i s ) can oft en be t reat ed wi t h a soft cervical collar, NSAIDs, muscle relaxants, and, occas i onal l y, surgery.
C. Selected pain syndromes
1. Herpes zoster o
o
a. Acute syndrome. “ Shingles―. i s a pai nful condi t i on caus ed by t he act i vat i on of a l at ent herpes zos t er i nfect i on, whi ch affect s t he nerves t hat s uppl y t he s ki n. Al t hough mos t common i n t he el derl y, s hi ngl es can occur i n i mmunos uppres s ed
Pa g e 2 6 8 6
ABC Ambe r CHM Conve rte r Tria l ve rsion, http://w w w .proce sste x t.com/a bcchm.html
pat i ent s as wel l .
(1) Clinical features. The ras h, charact eri zed by pai nful ves i cl es on an eryt hemat ous bas e, may be preceded by di s comfort . The pai n oft en abat es when t he s ki n l es i ons heal but may pers i s t and l ead t o pos t herpet i c neural gi a.
(2) T herapy
(a) Corticosteroids may l es s en t he ri s k of pos t herpet i c neural gi a i n noni mmunos uppres s ed pat i ent s ol der t han 60 years .
(b) Famciclovir decreas es t he durat i on of pos t herpet i c neural gi a i n immunocompetent patients.
(c) Acyclovir s houl d be gi ven t o immunosuppressed patients t o has t en recovery.
(d) Analgesics, tricyclic antidepressants, gabapentin, and topiramate t reat pai n duri ng t he acut e phas e of t he i l l nes s .
o
o
b. Chronic syndrome
Pa g e 2 6 8 7
ABC Ambe r CHM Conve rte r Tria l ve rsion, http://w w w .proce sste x t.com/a bcchm.html
(1) Once t he ves i cl es have heal ed, res i dual di s comfort can be managed wi t h Gabapent i n, t opi cal Li docai ne, Pregabal i n, and caps ai ci n oi nt ment .
(2) Tri cycl i c ant i depres s ant s , carbamaz epi ne and ot her ant i convul s ant medi cat i ons , vapo-cool ant s pray, t rans cut aneous el ect ri cal nerve s t i mul at i on, and, occas i onal l y, s urgi cal i nt ervent i on may be effect i ve.
P.536
2. Complex regional pain syndrome (refl ex s ympat het i c dys t rophy; s ympat het i c mai nt ai ned pai n). Thi s chroni c pai nful condi t i on can occur i di opat hi cal l y or devel op aft er t rauma t o a l i mb. At t i mes t he s everi t y of t he i ni t i al i njury can be qui t e t ri vi al . o
o
a. Clinical features. The affect ed l i mb i s pai nful , wi t h al t ered s ens ory percept i ons , vas omot or t one, and t emperat ure. The s ki n i s oft en di s col ored.
o
o
b. Diagnosis. To make a di agnos i s , ot her l ocal condi t i ons t hat can caus e pai n mus t be el i mi nat ed. A di fferent i al nerve bl ock vi a i nfi l t rat i on of paravert ebral nerves wi t h varyi ng dos es of l ocal anes t hes i a hel ps di fferent i at e bet ween l ocal s omat i c nerve hyperexci t abi l i t y and t he aut onomi c s ympat het i c dys funct i on at t ri but abl e t o
Pa g e 2 6 8 8
ABC Ambe r CHM Conve rte r Tria l ve rsion, http://w w w .proce sste x t.com/a bcchm.html
s ympat het i c medi at ed pai n. Int ravenous phent ol ami ne bl ockade can al s o provi de evi dence for s ympat het i c medi at ed pai n. o
o
c. T herapy. Treat ment i ncl udes paravertebral sympathetic ganglion blocks and aggressive physical therapy. Admi ni s t rat i on of t ri cycl i c ant i depres s ant s , ant i convul s ant s , or t opi cal caps ai ci n or l i docai ne are us eful adjuvant t herapi es .
D. Chronic pain syndromes Nonmal i gnant pai n s yndromes , when chroni c, can be es peci al l y di ffi cul t t o eval uat e and manage.
1. Care s houl d be t aken t o as s es s pat i ent s ful l y for remedi al probl ems (e.g., l umbar s t enos i s ). An interdisciplinary team approach i s bes t .
2. Us e of antidepressants and supportive psychotherapy, coupl ed wi t h attempts to improve the patient' s functional status, can be benefi ci al . In s el ect ed i ns t ances , chronic opiate t herapy can be benefi ci al .
VIII. Stroke Thi s neurovas cul ar di s order i s t he mos t common neurol ogi c di s eas e caus i ng s eri ous morbi di t y and mort al i t y.
A. Introduction
1. Di s eas e of t he vas cul ar s ys t em can di s rupt bl ood s uppl y t o t he CNS, l eadi ng t o neuronal dysfunction.
Pa g e 2 6 8 9
ABC Ambe r CHM Conve rte r Tria l ve rsion, http://w w w .proce sste x t.com/a bcchm.html
St roke s yndromes can be broadl y cl as s i fi ed i nt o predomi nant l y ischemic or hemorrhagic processes. As ympt omat i c l es i ons i n ei t her cat egory of s t roke s yndrome can predi s pos e pat i ent s t o fut ure di s eas e.
2. Optimal management requi res preci s e di agnos i s for effect i ve t herapy. o
o
a. General principles of management. of s ympt omat i c pat i ent s i ncl ude moni t ori ng for s i gns of cl i ni cal det eri orat i on, ens uri ng adequat e oxygenat i on, mai nt ai ni ng euvol emi a, avoi di ng ext remes of bl ood pres s ure wi t hout exces s i vel y l oweri ng hi gh bl ood pres s ure, t reat i ng i nfect i on and fever, and us i ng normal s al i ne (rat her t han gl ucos e s ol ut i ons ) for i nt ravenous hydrat i on. Effort s s houl d be made t o prevent deep venous t hrombos i s and decubi t i , avoi d as pi rat i on, mai nt ai n nut ri t i on, and at t end t o bowel and bl adder funct i on.
o
o
b. Rehabilitation of pat i ent s wi t h pers i s t ent neurol ogi c defect s i s i mport ant .
(1) Mul t i di s ci pl i nary t herapy i nvol vi ng phys i cal , occupat i onal , and s peech t herapy can be purs ued on an i npat i ent or out pat i ent bas i s .
(2) Depres s i on occurs frequent l y and may be t reat ed medi cal l y.
B. Ischemic stroke
Pa g e 2 6 9 0
ABC Ambe r CHM Conve rte r Tria l ve rsion, http://w w w .proce sste x t.com/a bcchm.html
1. Introduction o
o
a. Etiology. Caus es i ncl ude cardi ogeni c embol i , ext racrani al and i nt racrani al l arge art ery di s eas e, s mal l art ery di s eas e, and vari ous s ys t emi c and hemat ol ogi c di s orders .
o
o
b. Clinical signs
(1) Impai red carot i d t erri t ory ci rcul at i on produces s ympt oms of cont ral at eral weaknes s and s ens ory l os s , aphas i a or negl ect s yndromes and i ps i l at eral t rans i ent monocul ar bl i ndnes s (amauros i s fugax). Typi cal l y, t he weaknes s or numbnes s i s great es t i n t he faci al area, l es s pronounced i n t he arm, and s t i l l l es s pronounced i n t he l eg.
(2) A di agnos i s of vert ebrobas i l ar di s eas e i s fai rl y cert ai n i f t here i s t rans i ent bi nocul ar bl i ndnes s or ot her bi l at eral vi s ual di s t urbances , di pl opi a, at axi a, quadri pares i s , or vert i go as s oci at ed wi t h ot her neurol ogi c s ympt oms .
o
o
c. T herapy. New t reat ment s for acut e i s chemi c s t roke are under i nves t i gat i on, dri ven by t he evi dence t hat t here i s at l eas t a 3- t o 6-hour “t herapeut i c wi ndow― duri ng whi ch
Pa g e 2 6 9 1
ABC Ambe r CHM Conve rte r Tria l ve rsion, http://w w w .proce sste x t.com/a bcchm.html
i nt ervent i on may l es s en brai n damage.
(1) T hrombolytic agents may be admi ni s t ered wi t h an accept abl e i nci dence of hemorrhagi c i nfarct i on and i nt raparenchymal hemat oma. A randomi zed, doubl e-bl i nd t ri al of i nt ravenous recombi nant tissue plasminogen activator (t-PA) admi ni s t ered no more t han 3 hours aft er t he ons et of an acut e i s chemi c s t roke at a dos e of 0.9 mg/kg (wi t h 10% of t he dos e gi ven as a bol us and t he remai nder i nfus ed over 1 hour) res ul t ed i n an i mproved cl i ni cal out come at 3 mont hs i n pat i ent s t reat ed wi t h t he act i ve drug. Pat i ent s t reat ed wi t h t -PA were more l i kel y t o s us t ai n a s ympt omat i c i nt racerebral hemorrhage duri ng t he P.537
fi rs t 36 hours ; however, t here was no s i gni fi cant di fference i n mort al i t y bet ween t reat ed and unt reat ed pat i ent s .
(a) Treat ment wi t h t -PA appears t o be benefi ci al for pat i ent s wi t h s mal l or l arge art ery occl us i ve di s eas e or cardi oembol i c s t roke. Becaus e t here i s s uch a narrow wi ndow of opport uni t y for t he admi ni s t rat i on of t -PA, rare pat i ent s des t i ned t o have a t rans i ent i s chemi c at t ack (TIA; s ee VIII B 2) wi l l be i nadvert ent l y t reat ed.
Pa g e 2 6 9 2
ABC Ambe r CHM Conve rte r Tria l ve rsion, http://w w w .proce sste x t.com/a bcchm.html
(b) Pat i ent s t o be t reat ed wi t h t -PA mus t meet s t ri ct cri t eri a t o mi ni mi ze t he ri s k of hemorrhagi c compl i cat i ons . In addi t i on, no ant i coagul ant s or ant i pl at el et agent s s houl d be admi ni s t ered for 24 hours aft er t -PA t reat ment , and bl ood pres s ure el evat i ons s houl d be t reat ed. Contraindications i ncl ude:
(i) A rapi dl y i mprovi ng neurol ogi c defi ci t or mi nor s ympt oms
(ii) A bas el i ne CT s can s howi ng evi dence of i nt racrani al hemorrhage
(iii) A s ys t ol i c bl ood pres s ure >185 mm Hg or a di as t ol i c bl ood pres s ure >110 mm Hg
(iv) A medi cat i on-i nduced or di s eas e-rel at ed coagul opat hy
(v) A hi s t ory of hemorrhagi c s t oke, recent s urgery, or anot her i nvas i ve procedure
(2) Al t hough not yet “s t andard of care,†• t here i s i ncreas i ng experi ence wi t h i nt ra-art eri al t hrombol ys i s and s everal
Pa g e 2 6 9 3
ABC Ambe r CHM Conve rte r Tria l ve rsion, http://w w w .proce sste x t.com/a bcchm.html
mechani cal approaches t o i nt ra-art eri al cl ot l ys i s for pat i ent s acut el y pres ent i ng wi t h l arge art ery t hromboembol i c occl us i ons . A vari et y of i magi ng t echni ques , i ncl udi ng CT and MR perfus i on s cans , MR di ffus i on, and CT or MR angi ography may as s i s t i n pat i ent s el ect i on.
2. T ransient ischemic attacks (T IAs) are s hort -l i ved neurol ogi c defi ci t s , t ypi cal l y l as t i ng mi nut es . A recent recommendat i on i s t hat t he maxi mal durat i on of a TIA be revi s ed downward t o 1 hour, i n cont radi s t i nct i on t o t he cl as s i c defi ni t i on of up t o 24 hours . Sympt oms are at t ri but abl e t o i s chemi a i n t he carot i d or vert ebrobas i l ar art eri al di s t ri but i ons . The di s t i nct i on bet ween a TIA and a s t roke i s arbi t rary, and even a bri ef s ympt omat i c epi s ode of cerebral i s chemi a, i n conjunct i on wi t h an abnormal di ffus i on wei ght ed i magi ng (DW I) s can, mi ght be cons t rued as repres ent i ng a s t roke. Bot h warrant compl et e eval uat i on t o det ermi ne t he underl yi ng pat hophys i ol ogy and decreas e t he ri s k of s ubs equent i s chemi c event s . o
o
a. Etiology. Al t hough TIAs oft en res ul t from at heros cl erot i c l arge ves s el di s eas e, ot her di agnos t i c pos s i bi l i t i es des erve cons i derat i on, i ncl udi ng cardi ogeni c embol i , aort i c arch at herot hrombot i c embol i , ot her l arge art ery di s orders s uch as di s s ect i on and fi bromus cul ar dys pl as i a, s mal l art ery di s eas e, hemat ol ogi c di s orders , and mi grai ne. Ot her di s eas e ent i t i es s uch as s ei zures , t umors , s ubdural hemat omas , and MS s omet i mes mas querade as a TIA.
o
Pa g e 2 6 9 4
ABC Ambe r CHM Conve rte r Tria l ve rsion, http://w w w .proce sste x t.com/a bcchm.html
o
b. Diagnosis. Di agnos t i c s t udi es s houl d i ncl ude a CBC, s yphi l i s s erol ogy, a coagul at i on profi l e, and a CT s can or MRI. A t rans t horaci c or t rans es ophageal echocardi ogram and 24-hour ambul at ory el ect rocardi ography (ECG) moni t ori ng may be i ndi cat ed.
(1) A dupl ex exami nat i on of t he ext racrani al carot i d art ery t erri t ory i s oft en i nformat i ve.
(2) TCD s t udi es , MRA, and CTA can provi de i ns i ght i nt o t he ext racrani al and i nt racrani al art eri al ci rcul at i on.
(3) Convent i onal i nt ervent i onal angi ography i s t he defi ni t i ve t es t t o out l i ne t he vas cul ar anat omy.
(4) Pat i ent s wi t h an unreveal i ng cardi ac eval uat i on who are t hought t o have an embol us may benefi t from a t rans es ophageal echocardi ogram.
(5) If cl i ni cal s us pi ci on exi s t s , an LP can be us ed t o eval uat e t he pos s i bi l i t y of CNS i nfl ammat i on.
o
o
c. T herapy. If at heros cl eros i s i s s us pect ed as t he caus e of t rans i ent cerebral i s chemi a, cont rol of ri s k fact ors for at heros cl eros i s i s es s ent i al (s ee Chapt er
Pa g e 2 6 9 5
ABC Ambe r CHM Conve rte r Tria l ve rsion, http://w w w .proce sste x t.com/a bcchm.html
1 III A 2). An eval uat i on for coronary art ery di s eas e i s al s o appropri at e.
(1) Extracranial carotid artery disease. Carot i d endart erect omy i s s uperi or t o medi cal t herapy i n t he prevent i on of i s chemi c s t roke i n pat i ent s experi enci ng a TIA i ps i l at eral t o an angi ographi cal l y demons t rat ed 50%–99% s t enos i s at t he i nt ernal carot i d art ery ori gi n.
(a) Pat i ent s wi t h l es s s evere s t enos i s s houl d be t reat ed wi t h as pi ri n. Cl opi dogrel or as pi ri n combi ned wi t h s l ow-rel eas e di pyri damol e are al s o effect i ve i n prevent i ng s t roke.
(b) Ri s k fact or management pert i nent t o at heros cl eros i s i s appropri at e.
(2) Intracranial large artery disease. A TIA can res ul t from l arge art ery s t enos i s or occl us i on. Recent evi dence s ugges t s t hat as pi ri n t herapy i s s afer t han, and as effect i ve as , ant i coagul at i on t herapy for s ympt omat i c i nt racrani al l arge art ery s t enos i s . P.538
(3) Other causes of T IA. Therapy s houl d be di rect ed at t he appropri at e pat hophys i ol ogi c proces s . For i ns t ance, ant i coagul ant s are
Pa g e 2 6 9 6
ABC Ambe r CHM Conve rte r Tria l ve rsion, http://w w w .proce sste x t.com/a bcchm.html
frequent l y us ed i n pat i ent s wi t h a s ource of cardi ogeni c embol i .
3. Cardiogenic embolic stroke o
o
a. Etiology. The mos t common caus e of embol i c s t roke i s nonval vul ar at ri al fi bri l l at i on.
(1) Ot her condi t i ons as s oci at ed wi t h cardi ogeni c embol i i ncl ude recent myocardi al i nfarct i on (MI), an aki net i c vent ri cul ar s egment , di l at ed cardi omyopat hy, a pros t het i c heart val ve, i nfect i ve and nonbact eri al t hrombot i c endocardi t i s , l eft heart myxoma, l eft at ri al s pont aneous cont ras t echo, and at ri al s ept al aneurys m.
(2) A pat ent foramen oval e (PFO) or at ri al s ept al defect predi s pos es pat i ent s t o paradoxi cal embol i , es peci al l y i f t here i s a document ed venous t hrombos i s . Pat i ent s younger t han 55 years of age wi t h a PFO and an as s oci at ed at ri al s ept al aneurys m may be at part i cul arl y hi gh ri s k for embol i c s t roke.
(3) Ot her cardi ac condi t i ons s uch as mi t ral val ve prol aps e or a hypoki net i c vent ri cul ar s egment are rarel y as s oci at ed wi t h a cardi ogeni c embol us .
o
o
b. Diagnosis. The di agnos i s i s mos t cert ai n i f t here
Pa g e 2 6 9 7
ABC Ambe r CHM Conve rte r Tria l ve rsion, http://w w w .proce sste x t.com/a bcchm.html
i s an abrupt ons et of neurol ogi c dys funct i on, an underl yi ng cardi ac condi t i on known t o predi s pos e t o embol i , s t rokes i n mul t i pl e vas cul ar t erri t ori es , hemorrhagi c art eri al i nfarct i on, s ys t emi c embol i , an abs ence of concurrent condi t i ons known t o caus e s t roke, and angi ography demons t rat i ng (pot ent i al l y t rans i ent ) ves s el occl us i ons i n t he abs ence of an i nt ri ns i c vas cul opat hy. Pat i ent s who have a cardi ac embol i s m rarel y pres ent wi t h al l of t hes e condi t i ons . o
o
c. T herapy. An i s chemi c i nfarct i on caus ed by a cardi ogeni c embol us may devel op i nt o a hemorrhagi c i nfarct i on, es peci al l y i f reperfus i on occurs or t he i nfarct i on i s l arge. Therefore, care mus t be t aken t o l es s en t he ri s k of parenchymal hemorrhage i n acut el y i l l pat i ent s whi l e s i mul t aneous l y t aki ng meas ures t o prot ect t hem from anot her embol i c s t roke.
(1) If t he pat i ent has had a rel at i vel y s mal l i s chemi c embol i c i nfarct i on, a CT s can s houl d be obt ai ned. If no bl ood i s pres ent , a cont i nuous i nfus i on of hepari n i s admi ni s t ered. A part i al t hrombopl as t i n t i me (PTT) great er t han t wi ce t he cont rol val ue s houl d be avoi ded t o mi ni mi ze t he ri s k of hemorrhagi c convers i on of t he i s chemi c i nfarct i on and cl i ni cal wors eni ng.
(2) If t he pat i ent has had a l arge i s chemi c embol i c i nfarct i on, ant i coagul at i on t herapy s houl d be wi t hhel d for 5–7 days . If t he
Pa g e 2 6 9 8
ABC Ambe r CHM Conve rte r Tria l ve rsion, http://w w w .proce sste x t.com/a bcchm.html
pat i ent i s recoveri ng and a s ubs equent CT s can reveal s no bl ood, hepari n may be admi ni s t ered.
(3) Subs equent t reat ment wi t h oral ant i coagul ant s depends on t he underl yi ng di s eas e proces s and t he pat i ent 's general condi t i on.
4. Large artery disease o
o
a. Aortic arch atheromas. or thrombi. Thes e t hrombi , whi ch are vi s ual i zed by t rans es ophageal echocardi ography, can embol i ze t o t he cerebral ci rcul at i on and caus e s t roke. Opt i mal management i s yet t o be defi ned but us ual l y i nvol ves ant i pl at el et or ant i coagul at i on t herapy, wi t h t he l at t er bei ng us ed for mobi l e, peduncul at ed t hrombi .
o
o
b. Asymptomatic cervical bruit and carotid stenosis. The combi nat i on of an i nt ernal carot i d art ery brui t and at heros cl eros i s at t he i nt ernal carot i d art ery ori gi n i s a marker for coronary art ery di s eas e as wel l as cerebrovas cul ar di s eas e. Approxi mat el y 2% of t hes e pat i ent s wi l l have an i s chemi c s t roke each year. Pat i ent s wi t h a hemodynami cal l y s i gni fi cant or progres s i ve s t enos i s are at i ncreas ed ri s k for cerebral i nfarct i on.
(1) Etiology. A mi dcervi cal brui t can be caus ed by a hyperdynami c ci rcul at i on (e.g., as i n anemi a, pregnancy, and t hyrot oxi cos i s ), an
Pa g e 2 6 9 9
ABC Ambe r CHM Conve rte r Tria l ve rsion, http://w w w .proce sste x t.com/a bcchm.html
ext ernal carot i d art ery s t enos i s , an i nt ernal carot i d art ery s t enos i s , or a venous hum.
(2) Diagnosis. Noni nvas i ve vas cul ar t es t i ng (s ee I C 3 d) i s hel pful i n det ermi ni ng whet her t he brui t ori gi nat es from at heros cl erot i c i nt ernal carot i d art ery di s eas e and whet her t he l es i on i s hemodynami cal l y s i gni fi cant . Angi ography i s us ual l y us ed i f t he pat i ent i s a s urgi cal candi dat e.
(3) T herapy
(a) Control of risk factors for atherosclerosis (s ee Chapt er 1 III A 2). Ant i pl at el et t herapy (as pi ri n, cl opi dogrel , or as pi ri n combi ned wi t h s l ow-rel eas e di pyri damol e) i s oft en pres cri bed, and a careful coronary art ery eval uat i on i s warrant ed. P.539
(b) Medical treatment. Pat i ent s wi t h l es s t han a 50% i nt ernal carot i d art ery s t enos i s s houl d be managed medi cal l y.
(c) Carotid endarterectomy
(i) Pat i ent s wi t h a 50%–99%
Pa g e 2 7 0 0
ABC Ambe r CHM Conve rte r Tria l ve rsion, http://w w w .proce sste x t.com/a bcchm.html
s t enos i s who undergo carotid endarterectomy have l es s of a ri s k of i ps i l at eral s t roke t han pat i ent s managed medi cal l y i f s urgery can be achi eved wi t h l es s t han a 3% ri s k of compl i cat i ons .
(ii) Prophylactic carotid endarterectomy for uni l at eral , s l i ght l y or moderat el y s t enot i c, as ympt omat i c i nt ernal carot i d art ery di s eas e before major cardi ac or vas cul ar s urgery i s usually inadvisable.
o
o
c. Ischemic infarction. An unders t andi ng of neuroanat omy i s es s ent i al t o l ocal i ze t he compromi s ed area of t he brai n and t o correl at e t hi s i nformat i on wi t h a l i kel y s i t e of vas cul ar di s eas e.
(1) Etiology
(a) Large artery occlusive disease can caus e i s chemi c i nfarct i on, ei t her by bei ng a s ource of art ery-t o-art ery embol i or by caus i ng hypoperfus i on di s t al t o a hemodynami cal l y s i gni fi cant vas cul ar s t enos i s . Large art ery di s eas e can i nvol ve t he ext racrani al or i nt racrani al port i ons of t he cerebrovas cul ar ci rcul at i on.
Pa g e 2 7 0 1
ABC Ambe r CHM Conve rte r Tria l ve rsion, http://w w w .proce sste x t.com/a bcchm.html
(b) Vas cul ar condi t i ons ot her t han at heros cl eros i s (e.g., arterial dissection, arteritis, T akayasu' s syndrome, fibromuscular dysplasia, and radiation-induced vasculopathy) s houl d be cons i dered.
(2) Diagnosis. Di agnos t i c s t udi es are purs ued t o defi ne t he vas cul ar anat omy. CTA, MRA, carot i d dupl ex, TCD, and convent i onal angi ography can be us ed. The s cope of t es t i ng i s det ermi ned, i n part , by t he cl i ni cal condi t i on of t he pat i ent and whet her carot i d endart erect omy or ant i t hrombot i c t herapy are pri mary t herapeut i c cons i derat i ons . The more accurat e t he defi ni t i on of t he underl yi ng caus e of t he i nfarct i on, t he more preci s e t he det ermi nat i on of prognos i s and t reat ment .
(3) T herapy
(a) Extracranial carotid artery disease. If t he pat i ent has s us t ai ned a mi nor i nfarct i on wi t h a funct i onal recovery and has a 50%–99% at heros cl erot i c s t enos i s of t he i ps i l at eral ori gi n of t he i nt ernal carot i d art ery, carot i d endart erect omy i s s uperi or t o medi cal t herapy for prevent i on of s ubs equent s t roke. If t he s t enos i s i s l es s t han 50%, t herapy wi t h ant i pl at el et drugs i s appropri at e.
Pa g e 2 7 0 2
ABC Ambe r CHM Conve rte r Tria l ve rsion, http://w w w .proce sste x t.com/a bcchm.html
(b) Intracranial large artery disease
(i) If t he i nfarct i on i s caus ed by a s evere s t enos i s or occl us i on of a l arge i nt racrani al art ery, recent evi dence s ugges t s t hat as pi ri n t herapy i s s afer t han, and equal l y effect i ve as , ant i coagul at i on t herapy.
(ii) Some phys i ci ans us e ant i coagul at i on agent s i f t he pat i ent has new s ympt oms .
5. Small artery disease. The i nfarct i ons res ul t i ng from s mal l art ery di s eas e t ypi cal l y are deep i n t he hemi s pheres or t he pont omes encephal i c regi on and are known as lacunae. The l es i ons are l es s t han or equal t o 15 mm i n di amet er. o
o
a. Etiology. The underl yi ng vas cul ar l es i on i s us ual l y hypert ens i on-as s oci at ed l i pohyal i nos i s . Di abet es mel l i t us i s al s o as s oci at ed wi t h l acunar i nfarct s . A s mal l at heromat ous pl aque bl ocki ng t he os t i um of an art eri ol e may, on occas i on, be t he caus e of s ome occl us i ons , as may i nfect i ous or s t eri l e i nfl ammat i on, cardi ogeni c embol i , and l arge art ery occl us i ve di s eas e.
o
o
b. Diagnosis
Pa g e 2 7 0 3
ABC Ambe r CHM Conve rte r Tria l ve rsion, http://w w w .proce sste x t.com/a bcchm.html
(1) Is chemi c event s from s mal l art ery di s eas e caus e s t ereot ypi c s yndromes s uch as pure mot or or s ens ory s t roke, s ens ori mot or s t roke, cl ums y hand–dys art hri a s yndrome, and at axi c hemi pares i s .
(2) Pat i ent s s houl d be s creened for t he pos s i bi l i t y of cardi ogeni c embol i , l arge art ery occl us i ve di s eas e, and hemat ol ogi c and i nfl ammat ory di s orders (s ee VIII B) i f t hey are not hypert ens i ve or i f t hei r hi s t ory, exami nat i on, or rout i ne di agnos t i c t es t s s ugges t a caus e ot her t han hypert ens i on.
o
o
c. T herapy i s di rect ed at cont rol l i ng hypert ens i on. If anot her condi t i on i s defi ned as t he caus e of t he i nfarct i on, appropri at e i nt ervent i on s houl d be purs ued. Ant i pl at el et t herapy may be admi ni s t ered t o decreas e t he l i kel i hood of s ubs equent i s chemi c s t roke.
6. Hematologic and systemic conditions. Thes e di s t urbances are as s oci at ed wi t h i s chemi c i nfarct i on. P.540
o
o
a. Associated conditions. Si ckl e cel l di s eas e, hypervi s cos i t y as s oci at ed wi t h pol ycyt hemi a and paraprot ei nemi as , and hypercoagul abi l i t y are condi t i ons as s oci at ed wi t h i s chemi c s t roke.
Pa g e 2 7 0 4
ABC Ambe r CHM Conve rte r Tria l ve rsion, http://w w w .proce sste x t.com/a bcchm.html o
o
b. Etiology. Hypercoagul abi l i t y i s as s oci at ed wi t h ant i phos phol i pi d ant i bodi es (i ncl udi ng ant i cardi ol i pi ns and t he l upus ant i coagul ant s yndrome), hyperhomocys t ei nemi a, defi ci ency of prot ei ns C and S, act i vat ed prot ei n C res i s t ance, ant i t hrombi n III defi ci ency, mal i gnancy, nephrot i c s yndrome, and pregnancy, as wel l as s everal ot her condi t i ons .
7. T he young ischemic stroke patient. Pat i ent s younger t han 45 years of age who pres ent wi t h s t roke are oft en di agnos t i c chal l enges . The pot ent i al et i ol ogi es are vas t and i ncl ude, but are not l i mi t ed t o, t he fol l owi ng condi t i ons : o
o
a. Drug (es peci al l y cocai ne) and al cohol abus e
o
o
b. Hypercoagul abl e s t at es
o
o
c. Cardi ogeni c embol i
o
o
d. Mi grai ne
o
o
e. Vas cul i t i s and ot her rare art eri al l es i ons
o
o
f. CNS i nfect i on, i ncl udi ng HIV-as s oci at ed condi t i ons
o
Pa g e 2 7 0 5
ABC Ambe r CHM Conve rte r Tria l ve rsion, http://w w w .proce sste x t.com/a bcchm.html
o
g. Cancer
o
o
h. Di s orders of homocys t ei ne met abol i s m
o
o
i. Fami l i al condi t i ons [e.g., neurofi bromat os i s (NF), von Hi ppel -Li ndau di s eas e)]
o
o
j. Pregnancy and t he pos t part um s t at e
8. T he deteriorating ischemic stroke patient o
o
a. Pathophysiology.
(1) The pat i ent 's condi t i on may det eri orat e as a res ul t of progres s i ve occl us i on of art eri es from cl ot propagat i on or art ery-t o-art ery embol i or becaus e of s ubs equent cardi ogeni c embol i . Det eri orat i on al s o may res ul t from exces s i ve l oweri ng of bl ood pres s ure or i nadequat e ant i coagul at i on t herapy.
(2) Hemorrhagi c i nfarct i on or a parenchymal hemat oma can occur s pont aneous l y or as a res ul t of t hrombol yt i c or ant i coagul at i on t herapy.
(3) If hepari n i s bei ng us ed as t reat ment , t he pos s i bi l i t y of hepari n-i nduced t hrombos i s
Pa g e 2 7 0 6
ABC Ambe r CHM Conve rte r Tria l ve rsion, http://w w w .proce sste x t.com/a bcchm.html
(wi t h as s oci at ed t hrombocyt openi a) s houl d be cons i dered.
(4) Cerebral edema can devel op, caus i ng s hi ft i ng of brai n s t ruct ures and an i ncreas e i n t he ICP.
o
o
b. Approach to the patient. Ideal l y, cont i nui ng effort s s houl d be made t o defi ne t he pat hophys i ol ogy of t he s t roke.
(1) The bl ood pres s ure and degree of hydrat i on s houl d be checked.
(2) A CT s can s houl d be obt ai ned t o defi ne mas s effect and hemorrhagi c compl i cat i ons .
(3) A hemat ocri t , pl at el et count , and cl ot t i ng profi l e s houl d be obt ai ned as appropri at e.
o
o
c. T herapy. Pat i ent s s houl d be confi ned t o bed, and ext remes of bl ood pres s ure s houl d be avoi ded.
(1) Appropri at e hydrat i on s houl d be mai nt ai ned wi t h normal s al i ne, and i ncreas es i n ICP s houl d be t reat ed as neces s ary.
(2) Cons i derat i on can be gi ven t o novel , i nt ervent i onal neuroradi ol ogi c t echni ques s uch
Pa g e 2 7 0 7
ABC Ambe r CHM Conve rte r Tria l ve rsion, http://w w w .proce sste x t.com/a bcchm.html
as i nt ra-art eri al t hrombol ys i s and angi opl as t y, dependi ng on t he degree of neurol ogi c i mpai rment , t he t i me cours e of t he i l l nes s , and t he avai l abi l i t y of res ources .
C. Hemorrhagic disorders
1. Subarachnoid hemorrhage (SAH) o
o
a. Etiology. The mos t common caus es of SAH are trauma and ruptured berry aneurysms.
(1) Ot her caus es i ncl ude coagul opat hi es , mycot i c aneurys m, art eri ovenous mal format i on, vas cul i t i s , and s ympat homi met i c drugs .
(2) Aneurysms may be fami l i al and are as s oci at ed wi t h pol ycys t i c ki dney di s eas e, coarct at i on of t he aort a, fi bromus cul ar dys pl as i a, moyamoya di s eas e, pol yart eri t i s nodos a, ps eudoxant homa el as t i cum, and Marfan and Ehl ers -Danl os s yndromes .
o
o
b. Diagnosis
(1) Clinical signs. Pat i ent s who have a rupt ured berry aneurys m compl ai n of “t he wors t headache of my l i fe,― but exami nat i on may not reveal many object i ve fi ndi ngs . Ot her
Pa g e 2 7 0 8
ABC Ambe r CHM Conve rte r Tria l ve rsion, http://w w w .proce sste x t.com/a bcchm.html
P.541
pat i ent s can pres ent wi t h meni ngi s mus , al t ered s t at es of arous al , and focal neurol ogi c fi ndi ngs . A part i al t hi rd nerve pal s y wi t h pupi l l ary di l at i on i s s ugges t i ve of a pos t eri or communi cat i ng art ery aneurys m.
(2) Diagnostic studies. A CT scan reveal s t he pres ence of s ubarachnoi d bl ood i n mos t pat i ent s (Fi gure 11-3). If t he i ndex of s us pi ci on for an SAH i s hi gh, but a CT s can i s unreveal i ng, an LP provi des t he proper di agnos i s . Convent i onal i nt ervent i onal angi ography i s neces s ary t o charact eri ze t he aneurys m.
o
o
c. T herapy. Treat ment of aneurys mal SAH i s ai med at controlling complications, whi ch i ncl ude re-bl eedi ng, vas os pas m l eadi ng t o del ayed i s chemi c s t roke, hyponat remi a, acut e or chroni c hydrocephal us , i nt raparenchymal and i nt ravent ri cul ar hemat oma, and cardi ac arrhyt hmi as . Unfort unat el y, t he mort al i t y rat e from aneurys mal rupt ure approaches 50%, al t hough recent s t udi es s ugges t i mproved out comes . Int ervent i onal neuroradi ol ogi c t echni ques are pl ayi ng a l arger rol e i n t he management of aneurys ms .
(1) Re-bleeding mos t frequent l y occurs i n t he fi rs t 48 hours aft er aneurys mal rupt ure. Early
Pa g e 2 7 0 9
ABC Ambe r CHM Conve rte r Tria l ve rsion, http://w w w .proce sste x t.com/a bcchm.html
intervention t o i s ol at e t he aneurys m s houl d be performed when pos s i bl e t o el i mi nat e t he t hreat of rebl eedi ng. Surgi cal “cl i ppi ng― or endovas cul ar t herapy, whi ch i nvol ves pl aci ng t hrombogeni c “coi l s ― i n t he aneurys m, can be us ed t o i s ol at e t he aneurys m.
(2) The cours e of vasospasm can be fol l owed wi t h TCD and CTA s t udi es , and t reat ed accordi ngl y.
(a) The devel opment of i s chemi c s t roke from vas os pas m i s l es s l i kel y when nimodipine, a cal ci um channel bl ocker t hat may decreas e s mal l art ery vas ocons t ri ct i on or provi de neuronal prot ect i on from i s chemi a, i s admi ni s t ered.
(b) Prophyl axi s agai ns t s ympt omat i c vas os pas m al s o i ncl udes mai nt ai ni ng pat i ent s i n a euvol emi c s t at e and avoi di ng hypot ens i on.
(c) If t he aneurys m has been i s ol at ed from t he ci rcul at ory s ys t em, s ympt omat i c vas os pas m may be t reat ed wi t h hypervolemic therapy, coupl ed wi t h a moderat e i ncreas e i n bl ood pres s ure.
(d) Refract ory vas os pas m may be
Pa g e 2 7 1 0
ABC Ambe r CHM Conve rte r Tria l ve rsion, http://w w w .proce sste x t.com/a bcchm.html
amenabl e t o angi opl as t y or s el ect i ve i nt ra-art eri al papaveri ne or cal ci um ant agoni s t i nfus i on.
FIGURE 11-3 A nonenhanced comput ed t omography (CT) s can demons t rat i ng a s ubarachnoi d hemorrhage (SAH). Not e t he bl ood i n t he bas al ci s t erns and t he s yl vi an fi s s ures . o
P.542
o
(3) Symptomatic hydrocephalus can be t reat ed wi t h a vent ri cul ar drai n, repeat ed LPs , or vent ri cul operi t oneal s hunt i ng, as di ct at ed by cl i ni cal ci rcums t ances .
2. Intraparenchymal hematoma o
Pa g e 2 7 1 1
ABC Ambe r CHM Conve rte r Tria l ve rsion, http://w w w .proce sste x t.com/a bcchm.html
o
a. Diagnostic considerations. Chroni c hypert ens i on i s oft en, but not i nvari abl y, as s oci at ed wi t h hemorrhage i n t he regi on of t he put amen, t hal amus , cerebel l um, and pons .
(1) An i nt raparenchymal hemorrhage t hat i s not accompani ed by a hi s t ory of hypert ens i on s houl d prompt a s earch for an underl yi ng caus e of bl eedi ng (e.g., coagul opat hy, aneurys m, art eri ovenous mal format i on, or t umor), part i cul arl y i f t he hemorrhage i s not l ocat ed i n a regi on of t he brai n t ypi cal l y as s oci at ed wi t h hypert ens i ve bl eedi ng.
(2) In el derl y pat i ent s , l obar hemat omas , es peci al l y i f mul t i pl e, may be i ndi cat i ve of amyl oi d angi opat hy. The APOE4 al l el e may be a ri s k fact or for amyl oi d angi opat hy and amyl oi d angi opat hy-rel at ed hemorrhage.
(3) Coagul opat hi es (es peci al l y when i nduced by t hrombol yt i c t herapy) and t he us e of drugs s uch as cocai ne and s ympat homi met i cs are as s oci at ed wi t h i nt raparenchymal hemat oma.
o
o
b. Diagnosis
(1) Clinical signs
(a) A large putaminal hematoma
Pa g e 2 7 1 2
ABC Ambe r CHM Conve rte r Tria l ve rsion, http://w w w .proce sste x t.com/a bcchm.html
caus es i mpai red cons ci ous nes s , cont ral at eral hemi pares i s and s ens ory l os s , and gaz e preference t o t he s i de of t he hemorrhage.
(b) T halamic hemorrhage can l ead t o i mpai red cons ci ous nes s ; cont ral at eral weaknes s and s ens ory l os s ; di mi ni s hed vert i cal gaz e; and s mal l , poorl y react i ve pupi l s (i .e., Pari naud's s yndrome).
(c) Pat i ent s wi t h cerebellar hemorrhage may pres ent wi t h di zzi nes s , i mpai red gai t and s t ance and l i mb at axi a. If t he hemat oma i s l arge, i mpai red cons ci ous nes s , crani al nerve pal s i es (i ncl udi ng eye movement abnormal i t i es ), and weaknes s can devel op.
(d) The cl as s i c s i gns of pontine hemorrhage i ncl ude coma, pi npoi nt react i ve pupi l s , i mpai red l at eral ocul ar mot i l i t y, and quadri pl egi a wi t h decerebrat e pos t uri ng. Smal l pont i ne hemorrhages caus e more res t ri ct ed pont i ne s yndromes .
(2) Diagnostic studies. A CT s can i s cent ral t o di agnos i s . A coagul at i on profi l e and drug t oxi col ogy s creen s houl d be performed. Angi ography may be appropri at e i n normot ens i ve pat i ent s and i n t hos e wi t h
Pa g e 2 7 1 3
ABC Ambe r CHM Conve rte r Tria l ve rsion, http://w w w .proce sste x t.com/a bcchm.html
hemorrhage at at ypi cal s i t es . o
o
c. T herapy
(1) Taki ng meas ures t o l ower t he el evat ed ICP as s oci at ed wi t h parenchymal hemat oma can i mprove out come.
(2) W i t h cerebel l ar hemat omas , s urgi cal res ect i on i s a l i fes avi ng meas ure. Surgi cal res ect i on of hemat omas at ot her s i t es i s recei vi ng i ncreas i ng at t ent i on.
(3) A coagul opat hy s houl d be t reat ed appropri at el y.
3. Arteriovenous malformation. Headaches , s ei zures , and i nt raparenchymal or, occas i onal l y, SAHs may res ul t . Therapeut i c i nt ervent i on may i ncorporat e mul t i pl e modal i t i es , i ncl udi ng s urgery, i nt ervent i onal radi ol ogy wi t h embol i zat i on, and s t ereot act i c radi os urgery.
IX. Seizures and Epilepsy A s ei z ure i nvol ves a sudden abnormality of brain electrical activity. Mani fes t at i ons of a s ei zure can i ncl ude i mpai rment or l os s of cons ci ous nes s and s ens ory, mot or, or behavi oral abnormal i t i es . The t erm epilepsy des cri bes a s yndrome charact eri zed by recurrent s ei z ures .
A. Classification Opt i mal management of s ei zure pat i ent s depends on proper cl as s i fi cat i on of t he s ei zure t ype. Sei zures can be cat egori zed as generalized or partial (focal).
Pa g e 2 7 1 4
ABC Ambe r CHM Conve rte r Tria l ve rsion, http://w w w .proce sste x t.com/a bcchm.html
1. Generalized seizures are charact eri zed by a s udden l os s of cons ci ous nes s . o
o
a. Generalized convulsive seizures. cons i s t of t oni c, cl oni c, or t oni c–cl oni c (grand mal ) mot or act i vi t y. General i zed convul s i ons can res ul t from a focal s ei zure di s order t hat has s pread, i nvol vi ng t he ent i re brai n. Postictal obtundation and confusion commonl y l as t mi nut es , and occas i onal l y, hours . The EEG oft en s hows general i zed s pi kes or s pi kes and as s oci at ed s l ow waves .
o
o
b. Aabsence seizures are charact eri zed by a bri ef, s t ari ng s pel l s wi t hout a pos t -i ct al s t at e. Typi cal l y, t he EEG demons t rat es general i zed 3-Hz s pi kes and as s oci at ed s l ow waves .
P.543
2. Simple partial (focal) seizures are not accompani ed by an i mpai rment of cons ci ous nes s . General i zed mot or s ei z ures may devel op s econdari l y. o
o
a. There may be i s ol at ed cl oni c or t oni c act i vi t y of a l i mb or t rans i ent al t ered s ens ory percept i ons .
o
o
b. The s ei zure act i vi t y may s pread over one s i de of t he body i n a Jacksonian march (e.g., t he
Pa g e 2 7 1 5
ABC Ambe r CHM Conve rte r Tria l ve rsion, http://w w w .proce sste x t.com/a bcchm.html
convul s i ve act i vi t y can s t art i n t he face, move t o t he i ps i l at eral arm and t hen t o t he l eg, and may evol ve i nt o a general i zed s ei zure). o
o
c. The EEG may s how a focal rhyt hmi c di s charge at t he ons et of a s i mpl e part i al s ei zure, but occas i onal l y, no i ct al act i vi t y i s det ect ed. Int eri ct al l y, focal s pi kes wi t h as s oci at ed s l ow waves are frequent l y pres ent .
3. Complex partial (focal) seizures are oft en charact eri zed by an aura fol l owed by impaired awareness. The EEG oft en s hows i nt eri ct al s pi kes or s pi kes wi t h as s oci at ed s l ow waves i n t he t emporal or front ot emporal areas and i ct al focal rhyt hmi c di s charges . General i zed mot or s ei zures may devel op s econdari l y. o
o
a. The aura may i nvol ve hallucinations (e.g., ol fact ory, vi s ual , audi t ory, or gus t at ory) and compl ex illusions (e.g., of havi ng experi enced a new event or of never havi ng experi enced a commonpl ace event ). However, pat i ent s frequent l y do not recal l t hei r aura.
o
o
b. Naus ea or vomi t i ng, focal s ens ory percept i ons , and focal t oni c or cl oni c act i vi t y may accompany a compl ex s ei zure.
o
o
c. Aft er t he aura, t here may be an epi s ode of i mpai red cons ci ous nes s , l as t i ng s econds t o s everal mi nut es , duri ng whi ch t i me aut omat i s ms may be obs erved. Ret urn t o bas el i ne cogni t i ve abi l i t i es can
Pa g e 2 7 1 6
ABC Ambe r CHM Conve rte r Tria l ve rsion, http://w w w .proce sste x t.com/a bcchm.html
t ake s everal mi nut es .
4. St at us epi l ept i cus i s defi ned as an epi s ode of repeat ed or ongoi ng s ei zure act i vi t y wi t h i mpai red arous al l as t i ng at l eas t 30 mi nut es . St at us epi l ept i cus may i nvol ve bot h convulsive (general i zed t oni c–cl oni c act i vi t y) and nonconvulsive (abs ence or compl ex part i al ) s ei zures . o
o
a. Pat i ent s experi enci ng convul s i ve s ei zures are at ri s k for hypoxi a, as pi rat i on, aci dos i s , hypot ens i on, hypert hermi a, myogl obi nuri a, hypogl ycemi a, and mul t i pl e phys i cal i njuri es .
o
o
b. Pat i ent s experi enci ng nonconvul s i ve s ei zures can appear del i ri ous . A fl uct uat i ng s ens ori um and s ubt l e aut omat i s ms or myocl oni c jerks are cl ues t o t he di agnos i s , and t he EEG i s confi rmat ory.
B. Etiology Tabl e 11-6 out l i nes s ome of t he many caus es of s ei zures . Several s ei z ure t ypes have a defi ned genet i c bas i s : aut os omal domi nant front al l obe epi l eps y (neuronal ni cot i ni c acet yl chol i ne recept or mut at i on) and aut os omal domi nant t emporal l obe epi l eps y wi t h audi t ory feat ures .
TABLE 11-6 Selected Causes of Seizures
Pa g e 2 7 1 7
ABC Ambe r CHM Conve rte r Tria l ve rsion, http://w w w .proce sste x t.com/a bcchm.html
Idi op at h ic Ge net ic pre di s pos itio n Me sia l te mp ora l s cl ero sis Me t ab ol i c ab nor ma liti es Hy po nat re
Pa g e 2 7 1 8
ABC Ambe r CHM Conve rte r Tria l ve rsion, http://w w w .proce sste x t.com/a bcchm.html
mi a Hy poor hyp erg l yc em ia Hy poc al c em ia Hy po ma gn es e mi a Ure mi a Va s cu l ar di s eas e
Pa g e 2 7 1 9
ABC Ambe r CHM Conve rte r Tria l ve rsion, http://w w w .proce sste x t.com/a bcchm.html
an d str oke I nfa rct i on, es p eci al l y cor tic al Va s cu l ar ma l for ma tio n Va s cu liti s Infl am ma t or y cau
Pa g e 2 7 2 0
ABC Ambe r CHM Conve rte r Tria l ve rsion, http://w w w .proce sste x t.com/a bcchm.html
s es S ys t em ic l up us ery t he ma t os us (SL E) Ne opl as i a Me t as t at ic an d pri ma ry bra in tu mo rs Inf ect
Pa g e 2 7 2 1
ABC Ambe r CHM Conve rte r Tria l ve rsion, http://w w w .proce sste x t.com/a bcchm.html
i on Me ni n gi t i s, abs ces s, an d enc ep hal itis De ge ner at i ve di s eas es Al z hei me r's di s eas e Tra um a Ecl
Pa g e 2 7 2 2
ABC Ambe r CHM Conve rte r Tria l ve rsion, http://w w w .proce sste x t.com/a bcchm.html
am ps i a Dru gs T he op hyl lin e L i do cai ne Coc ai n e Dru g an d s ub sta nce wi t hdr aw al Ant i co nvu lsa
Pa g e 2 7 2 3
ABC Ambe r CHM Conve rte r Tria l ve rsion, http://w w w .proce sste x t.com/a bcchm.html
nt me di c at i ons Be nz o di a z ep i ne s Al c oh ol Ps y cho ge ni c cau s es P.544
C. Diagnosis
1. Patient history and physical examination can ai d i n t he det ermi nat i on of whet her a s ei zure or s ome ot her t rans i ent event was res pons i bl e for t he pat i ent 's s ympt oms . o
Pa g e 2 7 2 4
ABC Ambe r CHM Conve rte r Tria l ve rsion, http://w w w .proce sste x t.com/a bcchm.html o
a. A family history of epilepsy or a history of febrile convulsions i s rel evant .
o
o
b. An accurate description of the event by an observer i s hel pful i n defi ni ng t he probl em.
o
o
c. Urinary incontinence, back pain (from a vert ebral compres s i on fract ure), myalgias, and oral lacerations are cl ues t o proper di agnos i s .
o
o
d. Fever, fatigue, stress, alcohol withdrawal, medications, and menses can provoke s ei zures .
2. Differential diagnosis. Ot her condi t i ons t hat may produce s udden l os s of cons ci ous nes s are di s cus s ed i n II A. Ps ychogeni c s ei zures s houl d be cons i dered i f pat i ent s exhi bi t nons t ereot ypi c event s , have an unexpect ed res i s t ance t o ant i epi l ept i c drugs , or have a ps ychi at ri c di s order.
3. Diagnostic studies o
o
a. The EEG i s cent ral t o t he eval uat i on of s ei zure pat i ent s . The bes t t echni que for ful l y charact eri zi ng a s ei zure di s order i s cont i nuous vi deo EEG moni t ori ng, but t hi s i s not us ual l y us ed as an i ni t i al di agnos t i c t es t . A normal EEG does not excl ude t he di agnos i s of epi l eps y.
o
o
b. An MRI s can i s t he mos t us eful modal i t y for
Pa g e 2 7 2 5
ABC Ambe r CHM Conve rte r Tria l ve rsion, http://w w w .proce sste x t.com/a bcchm.html
det ect i ng l es i ons t hat may caus e s ei zures . o
o
c. SPECT and positron emission tomography (PET ) can provi de addi t i onal i nformat i on t o hel p l ocal i ze a s ei zure focus .
o
o
d. Laboratory studies are i ndi cat ed t o eval uat e pot ent i al met abol i c or t oxi c caus es , i ncl udi ng gl ucos e, s odi um, cal ci um, bl ood urea ni t rogen (BUN), creat i ni ne (Cr), l i ver fucnt i on t es t s (LFTs ), and t oxi n s creen.
D. Therapy
1. General considerations. If a s ei zure i s t he s us pect ed di agnos i s , deci s i ons about t herapy depend on t he underl yi ng caus e. o
o
a. Correct i on of hyponat remi a, hypogl ycemi a, or drug i nt oxi cat i on may be al l t hat i s neces s ary.
o
o
b. Pat i ent s wi t h a neurol ogi c condi t i on known t o be as s oci at ed wi t h recurrent s ei zures oft en requi re medi cat i on.
o
o
c. Ant i convul s ant t herapy i s oft en not i ni t i at ed i n pat i ent s wi t h a s i ngl e, unprovoked convul s i on; a normal neurol ogi c exami nat i on; and a normal brai n i magi ng s t udy and EEG unl es s t hey experi ence a s econd s ei zure.
Pa g e 2 7 2 6
ABC Ambe r CHM Conve rte r Tria l ve rsion, http://w w w .proce sste x t.com/a bcchm.html
2. Medical therapy o
o
a. General principles. An at t empt i s us ual l y made t o prevent s ubs equent s ei zures by us i ng a single agent, t o l i mi t t oxi c effect s . The drug s houl d be admi ni s t ered i n progressive doses unt i l s ei zure cont rol has been achi eved or unt i l drug t oxi ci t y occurs . Onl y i f monot herapy fai l s s houl d a s econd drug be added. If cont rol i s t hen obt ai ned, t he fi rs t agent mi ght be careful l y wi t hdrawn.
o
o
b. Specific agents. The choi ce of medi cat i on s houl d be bas ed on t he s ei zure t ype, beari ng i n mi nd pos s i bl e cont rai ndi cat i ons and s i de effect s .
(1) Typi cal l y, general i zed convul s i ve, s i mpl e part i al , and compl ex part i al s ei zures are t reat ed wi t h carbamaz epi ne, phenyt oi n, val proi c aci d, t opi ramat e, l evet i racet am, l amot ri gi ne, or zoni s ami de.
(2) Val proi c aci d or et hos uxi mi de i s us ed for general i zed nonconvul s i ve s pel l s (abs ence s ei zures ).
(3) Val proi c aci d i s part i cul arl y effect i ve for cont rol l i ng juveni l e myocl oni c epi l eps y (a di s order charact eri zed by myocl oni c s ei zures ).
(4) Lamotrigine, gabapentin, tiagabine,
Pa g e 2 7 2 7
ABC Ambe r CHM Conve rte r Tria l ve rsion, http://w w w .proce sste x t.com/a bcchm.html
oxcarbazepine, and topiramate are adjunct i ve medi cat i ons for t he t reat ment of pat i ent s wi t h refract ory part i al s ei zures .
3. Surgical therapy. Pat i ent s refract ory t o medi cal cont rol of s ei zures may be candi dat es for s urgery t o cont rol t he epi l eps y. T emporal lobe resection, ablation of a cortical seizure focus, and corpus callosum sectioning are abl e t o reduce s ei zure frequency i n s ome pat i ent s who meet s peci fi c cri t eri a. Vagus nerve stimulation may hel p cont rol s ei zures .
4. Control of status epilepticus. General i zed convul s i ve s t at us epi l ept i cus i s a l i fe-t hreat eni ng condi t i on; t herefore, management of convul s i ve s t at us epi l ept i cus requi res maki ng cert ai n t hat t he P.545
ai rway i s unobs t ruct ed and mai nt ai ni ng adequat e oxygenat i on, bl ood pres s ure, and hydrat i on. Defi ni t i on of t he underl yi ng probl em i s es s ent i al . o
o
a. Glucose. and thiamine s houl d be admi ni s t ered aft er bl ood s ampl es for gl ucos e, el ect rol yt es , renal funct i on, ant i convul s ant drug l evel s , and t oxi col ogy have been obt ai ned.
o
o
b. Oft en, intravenous diazepam or lorazepam i s gi ven t o s t op t he convul s i ons . Loraz epam carri es l es s of a ri s k of res pi rat ory depres s i on or arres t and remai ns effect i ve for l onger peri ods .
Pa g e 2 7 2 8
ABC Ambe r CHM Conve rte r Tria l ve rsion, http://w w w .proce sste x t.com/a bcchm.html o
o
c. Admi ni s t rat i on of a benzodi azepi ne i s fol l owed by admi ni s t rat i on of phenytoin, fosphenytoin, or phenobarbital.
o
o
d. If convul s i ons cont i nue aft er l oadi ng dos es of phenyt oi n or fos phenyt oi n or phenobarbi t al have been admi ni s t ered, i nt ravenous midazolam, propofol, or pentobarbital can be gi ven i n a careful l y s upervi s ed s et t i ng, wi t h cont i nuous EEG moni t ori ng, unt i l t he s ei zure di s charges are el i mi nat ed from t he EEG.
o
o
e. Di mi ni s hed cardi ac out put , bradycardi a, and hypot ens i on oft en l i mi t t he dos e of i nt ravenous ant i convul s ant s . Fluid resuscitation and vasopressors may be us ed.
5. Psychosocial issues. The fol l owi ng i s s ues are i mport ant t o cons i der when managi ng epi l eps y. o
o
a. Pat i ent s may be depres s ed, have behavi oral di s t urbances , and oft en requi re vocat i onal s upport s ervi ces . Frequent l y, communi t y s upport s ervi ces are avai l abl e.
o
o
b. Revi ew of t he pat i ent 's driving status and work and pl ay envi ronment are neces s ary.
o
o
c. Fami l y and fri ends need t o be couns el ed as t o
Pa g e 2 7 2 9
ABC Ambe r CHM Conve rte r Tria l ve rsion, http://w w w .proce sste x t.com/a bcchm.html
how t o manage a convul s i on. o
o
d. W omen of chi l dbeari ng age s houl d be couns el ed about i s s ues rel at i ng t o pregnancy.
o
o
e. Pregnant women s houl d be mai nt ai ned on monot herapy at t he l owes t effect i ve dos e, gi ven t hat s ei zures may harm t he fet us , and t he ri s k of bi rt h defect s i ncreas es wi t h pol ypharmacy.
X. Movement Disorders A. Parkinson's disease
1. Pathogenesis o
o
a. Parki ns on's di s eas e i s charact eri zed by a degenerat i on of cel l s i n t he s ubs t ant i a ni gra, whi ch caus es a defi ci ency of dopami ne (a neurot rans mi t t er) i n t he CNS, l eadi ng t o a s eri es of changes i n mot or cont rol pat hways . The mechani s m behi nd t he degenerat i on of t hes e cel l s i s unknown, al t hough hypot hes es cent er about free radi cal damage and i mpai red mi t ochondri al oxi dat i ve funct i on.
o
o
b and c. Genet i cs . Major i ns i ght s i nt o t he genet i cs of Parki ns on's di s eas e are emergi ng. The SNCA gene codes for α-s ynucl ei n, t he PRKN gene codes for parki n, and t he UCHL1 gene codes for a ubi qui t i n hydroxyl as e. Several ot her chromos ome l oci have been as s oci at ed wi t h Parki ns on's di s eas e.
Pa g e 2 7 3 0
ABC Ambe r CHM Conve rte r Tria l ve rsion, http://w w w .proce sste x t.com/a bcchm.html o
o
c. A common feat ure emergi ng from t he genet i cs s t udi es i s t hat t he pat hophys i ol ogy of Parki ns on's di s eas e, as wel l as t hat of ot her neurodegenerat i ve di s eas es , may i nvol ve abnormal i t i es i n pat hways as s oci at ed wi t h ubi qui t i n-as s oci at ed prot ei n degradat i on.
2. Diagnosis o
o
a. Clinical symptoms and signs.
(1) Parki ns on's di s eas e pat i ent s oft en compl ai n of “s l owi ng down―; t hey have t roubl e dres s i ng, ari s i ng from a s eat ed pos i t i on, cl i mbi ng or des cendi ng s t ai rs , wri t i ng, and t urni ng over i n bed.
(2) On exami nat i on, rigidity and akinesia or bradykinesia are pres ent and a 3-Hz resting tremor and postural instability are oft en evi dent . Si gns are oft en as ymmet ri cal earl y i n t he di s eas e.
(3) Cogni t i ve i mpai rment devel ops i n more t han 50% of pat i ent s over t i me.
o
o
b. Differential diagnosis. The di agnos i s i s a cl i ni cal det ermi nat i on, al t hough, at t i mes , t es t i ng t o excl ude ot her ent i t i es pres ent i ng as
Pa g e 2 7 3 1
ABC Ambe r CHM Conve rte r Tria l ve rsion, http://w w w .proce sste x t.com/a bcchm.html
parki ns oni s m i s i ndi cat ed.
(1) Somet i mes serial observations are neces s ary t o det ermi ne whet her t he parki ns oni an s t at e i s t he harbi nger of anot her neurol ogi c i l l nes s ; t hi s i s of s peci al concern i n pat i ent s who do not have t he characteristic “ pill-rolling― or resting tremor of Parkinson' s disease or fai l t o res pond t o l evodopa t herapy.
(2) Condi t i ons t hat may produce parki ns oni an s ympt oms s i mi l ar t o t hos e found i n pat i ent s wi t h Parki ns on's di s eas e i ncl ude NPH, mul t i pl e s t rokes , hypot hyroi di s m, drug effect s [e.g., neurol ept i cs (dopami ne-bl ocki ng agent s ), met ocl oprami de, di l t i azem, and res erpi ne], W i l s on's di s eas e, anoxi c encephal opat hy, and i nt oxi cat i on [e.g., by carbon monoxi de, manganes e, or n-met hyl -4-phenyl -1,2,3,6-t et rahydropyri di ne (MPTP)].
(3) Rare neurol ogi c di s orders t hat may have parki ns oni an feat ures are s ummari zed i n o
Onl i ne Tabl e 11-7.
P.546
o
ONLINE TABLE 11-7 Key Features of Selected Conditions Causing Parkinsonism
Pa g e 2 7 3 2
ABC Ambe r CHM Conve rte r Tria l ve rsion, http://w w w .proce sste x t.com/a bcchm.html
Dis Dis or tin de gui r
shi ng Cli nic al Ch ar act eri sti
cs Pro Im gre pai s s i red ve ver s up t i c ran al ucl gaz ear e; pal ear sy ly (†axi œt al au ri gi op di t at h y y†wi t •) h pos t ur al i ns
Pa g e 2 7 3 3
ABC Ambe r CHM Conve rte r Tria l ve rsion, http://w w w .proce sste x t.com/a bcchm.html
t ab ilit y Mul Aut t i pl on e
om
s ys i c t e i ns ms uffi at r ci e op ncy hy ; (Sh cer y-D eb rag el l er ar s yn dys dro fun me ct i )
on; up per an d l ow er mo t or ne uro n dys fun ct i on
Pa g e 2 7 3 4
ABC Ambe r CHM Conve rte r Tria l ve rsion, http://w w w .proce sste x t.com/a bcchm.html
Di f De fus me e
nt i
Le a wy ear bo l y dy i n di s i l l n eas es s e
,
(†fl u œD ct u em at i ent ng ia
cog
wi t ni t i h
on,
Le vi s wy ual bo hal di e l uci s †nat •) i on s Cor As y t i c mm al - et ri bas c al
fi n
ga di n ngl gs i on on ic
exa
de mi ge nat
Pa g e 2 7 3 5
ABC Ambe r CHM Conve rte r Tria l ve rsion, http://w w w .proce sste x t.com/a bcchm.html
ner i on at i of on s en (†s or œt y au l os op s at h an y†d •) apr axi a; uni l at era l ri gi di t y; uni l at era l sti mu l us -s e ns i tiv e my ocl on us ; dys
Pa g e 2 7 3 6
ABC Ambe r CHM Conve rte r Tria l ve rsion, http://w w w .proce sste x t.com/a bcchm.html
t on ia Fro Ap nt o at h t e y, mp di s ora i nh l
i bi t
l ob i on ar , de an ge om ner i a, at i eff on ort (†ful œt s pe au ech op at h y†•) Hu Par nt i ki n ngt s on on' i an s
fea
di s t ur eas es e
pro mi ne nt in you
Pa g e 2 7 3 7
ABC Ambe r CHM Conve rte r Tria l ve rsion, http://w w w .proce sste x t.com/a bcchm.html
ng pat i en ts; fa mi l y hi s t or y, cho reo at h et o sis Ol i Cer vop eb ont el l oce ar reb dys el l fun ar ct i at r on; op aut hy* on om ic dys fun ct i on Ba Cal s al ci fi ga cat ngl i on
Pa g e 2 7 3 8
ABC Ambe r CHM Conve rte r Tria l ve rsion, http://w w w .proce sste x t.com/a bcchm.html
ia
vi s i
cal bl e ci fi on cat co i on mp ut e d to mo gra phy (CT ) s ca n Ne Aca uro nt h aca ocy nt h t es ocy i n t os we is
t per i ph era l bl o od sm
ear *May be t he s ame as mul t i pl
Pa g e 2 7 3 9
ABC Ambe r CHM Conve rte r Tria l ve rsion, http://w w w .proce sste x t.com/a bcchm.html
e s ys t em s at roph y.
3. T herapy. Parki ns on's di s eas e i s a progres s i ve di s eas e. Therefore, management prot ocol s vary dependi ng on t he pat i ent 's s ympt oms and t he ext ent of funct i onal i mpai rment . o
o
a. Early therapy. Several medi cat i ons are avai l abl e t o t reat Parki ns on's di s eas e.
(1) Carbidopa/levodopa combinations are t he mai ns t ay of t reat ment for Parki ns on's di s eas e. Thi s t reat ment s houl d begi n when t he di s eas e i mpai rs t he pat i ent 's funct i onal s t at us . A s us t ai ned rel eas e formul at i on of carbi dopa/l evodopa i s avai l abl e and provi des a more uni form cl i ni cal res pons e t han convent i onal dos i ng.
(a) Levodopa i s convert ed t o dopami ne by t he pres ynapt i c neuron and t herefore i ncreas es t he amount of neurot rans mi t t er avai l abl e t o t he pos t s ynapt i c dopami ne
Pa g e 2 7 4 0
ABC Ambe r CHM Conve rte r Tria l ve rsion, http://w w w .proce sste x t.com/a bcchm.html
recept or.
(b) Carbidopa bl ocks s ys t emi c convers i on of l evodopa t o dopami ne, t hereby decreas i ng t he undes i rabl e s ys t emi c effect s of l evodopa.
(2) Dopamine agonists are i ncreas i ngl y bei ng us ed at i ni t i al t herapy, es peci al l y i n younger pat i ent s . They may have s ome neuroprot ect i ve benefi t s . Drugs i ncl ude pramipexole, ropinirole, pergolide, and bromocriptine.
(3) Anticholinergics, whi ch i mprove t he chol i nergi c–dopami nergi c bal ance i n t he bas al gangl i a, are part i cul arl y hel pful i n t reat i ng t remor. However, t hey may cont ri but e t o cogni t i ve i mpai rment .
(4) Amantadine, whi ch i ncreas es t he avai l abi l i t y of dopami ne t o t he pos t s ynapt i c neuron, can be effect i ve earl y i n t he cours e of t he di s eas e or as an adjunct i ve t herapy l at er i n t he di s eas e cours e t o hel p “s moot h out ― mot or funct i on.
o
o
b. Advanced therapy. In t he l at er s t ages of t he di s eas e, t herapy i s di rect ed at opt i mi zi ng t he pat i ent 's funct i onal s t at us and avoi di ng advers e effect s of medi cat i on.
Pa g e 2 7 4 1
ABC Ambe r CHM Conve rte r Tria l ve rsion, http://w w w .proce sste x t.com/a bcchm.html
(1) Dopamine agonists. If t he t herapeut i c res pons e t o carbi dopa/l evodopa t herapy i s i nadequat e, or i f t he pat i ent cannot t ol erat e t he medi cat i on, pramipexole, ropinirole, pergolide, or bromocriptine may be admi ni s t ered.
(a) Thes e drugs are direct postsynaptic dopamine-receptor agonists.
(b) A combi nat i on of carbi dopa/l evodopa and a dopami ne agoni s t s eems t o be part i cul arl y effect i ve and i s oft en wel l t ol erat ed. Dopami ne agoni s t s hel p decreas e mot or fl uct uat i ons when us ed i n conjunct i on wi t h carbi dopa/l evodopa.
(2) Management with disease progression. Management of Parki ns on's di s eas e becomes i ncreas i ngl y di ffi cul t as t he di s eas e progres s es . “ Wearing off― effect s , dyskinesias, and wi de, random s wi ngs i n pat i ent mobi l i t y (“ on–off― phenomena) devel op. A s us t ai ned-rel eas e form of carbi dopa/l evodopa (al one or i n combi nat i on wi t h a dopami ne agoni s t ) can be us ed. Catechol O-methyltransferase inhibitors, whi ch i ncreas e t he s ynapt i c avai l abi l i t y of l evodopa by bl ocki ng i t s degradat i on, al s o can be us ed t o manage uns t abl e pat i ent s .
o
o
c. Ancillary therapy
Pa g e 2 7 4 2
ABC Ambe r CHM Conve rte r Tria l ve rsion, http://w w w .proce sste x t.com/a bcchm.html
(1) Ot her t herapeut i c maneuvers i ncl ude t he strategic reduction of medication i f pat i ent s are experi enci ng dys ki nes i a and judicious use of psychotropic agents t o t reat t he vari ous unt oward behavi oral cons equences of Parki ns on's di s eas e (e.g., i ns omni a, hal l uci nat i ons , agi t at i on).
(2) Dietary manipulations t hat redistribute or limit protein i nt ake duri ng t he day may i mprove t he effi cacy of l evodopa.
(3) Physical therapy and an exercise program hel p opt i mi ze mobi l i t y.
(4) Pallidotomy and deep brain stimulation offer new t herapeut i c opt i ons for refract ory Parki ns on's di s eas e pat i ent s .
(5) The rol e of s urgi cal i mpl ant s of dopami ne-cont ai ni ng cel l s for t he t reat ment of Parki ns on's di s eas e remai ns experi ment al .
B. Hyperkinetic disorders
1. T remor o
o
a. Benign essential tremor. i s charact eri zed by a pos t ure-rel at ed 5–9-Hz os ci l l at i on of t he hands
Pa g e 2 7 4 3
ABC Ambe r CHM Conve rte r Tria l ve rsion, http://w w w .proce sste x t.com/a bcchm.html
and forearms t hat i mpai rs performance of fi ne mot or t as ks .
(1) Thi s t ype of t remor i s oft en familial and may be accompani ed by titubation (head tremor).
(2) Cons umpt i on of al cohol may t emporari l y s uppres s t he t remor; s t res s , caffei ne, or s l eep depri vat i on may exacerbat e t he condi t i on.
(3) β-Adrenergic blocking agents and primidone are effect i ve t reat ment s .
P.547
o
o
b. An action (kinetic) tremor i s evi dent when pat i ent s move t hei r arms ; t here may be a rel at i vel y mi l d accompanyi ng pos t ural and i nt ent i on component . Treat ment wi t h clonazepam may be us eful .
2. Chorea des cri bes rapid, “ dance-like― distal limb and facial movements. Caus es i ncl ude hypert hyroi di s m, drugs (e.g., bi rt h cont rol pi l l s and l evodopa), Sydenham's chorea, pregnancy, SLE, ant i phos phol i pi d s yndrome, s t roke, porphyri a, W i l s on's di s eas e, Lyme di s eas e, Hunt i ngt on's di s eas e, and neuroacant hocyt os i s .
Pa g e 2 7 4 4
ABC Ambe r CHM Conve rte r Tria l ve rsion, http://w w w .proce sste x t.com/a bcchm.html
3. Athetosis general l y refers t o involuntary, slow, writhing, “ snake-like― limb movements. Caus es i ncl ude W i l s on's di s eas e, Hunt i ngt on's di s eas e, anoxi c encephal opat hy, t rauma, bi rt h cont rol pi l l s , and s everal rare heredi t ary di s orders .
4. Dystonia des cri bes slow, writhing, sustained and involuntary contractions of the proximal limb, trunk, and neck musculature. Dys t oni a i s as s oci at ed wi t h W i l s on's di s eas e, Parki ns on's di s eas e, Hunt i ngt on's di s eas e, t rauma, neuronal s t orage di s orders , encephal i t i s , drugs (e.g., neurol ept i cs , l evodopa), and ot her rare heredi t ary condi t i ons . o
o
a and b. Genet i cs . The T OR1A gene for aut os omal domi nant general i zed t ors i on dys t oni a codes for t ors i n A, an ATP-bi ndi ng and heat s hock prot ei n. Pat i ent s may res pond t o hi gh dos es of t ri hexypheni dyl .
o
o
b. Some pat i ent s wi t h aut os omal domi nant i di opat hi c dys t oni a are part i cul arl y res pons i ve t o carbi dopa/l evodopa t herapy. Thi s condi t i on i s as s oci at ed wi t h t he DY T 5 gene on chromos ome 14 t hat codes for guani ne t ri phos phat e (GTP) cycl ohydrol as e I; a defi ci ency of t hi s enzyme caus es l os s of dopami ne s ynt hes i s .
o
o
c. Focal dys t oni as s uch as wri t er's cramp, bl epharos pas m, s pas t i c dys phoni a, and t ort i col l i s can occur. Treat ment wi t h l ocal bot ul i num t oxi n i nfi l t rat i on can be benefi ci al .
Pa g e 2 7 4 5
ABC Ambe r CHM Conve rte r Tria l ve rsion, http://w w w .proce sste x t.com/a bcchm.html o
o
d. Some pat i ent s wi t h dys t oni a may exhi bi t chorea and at het os i s .
5. Hemiballismus des cri bes wild, flinging, principally proximal movements of t he arms or legs. It i s oft en caus ed by an i nfarct i n t he s ubt hal ami c nucl eus . Hal operi dol can decreas e t he i nvol unt ary movement s .
6. Blepharospasm can occur i n i s ol at i on or as part of a more wi des pread di s order s uch as Parki ns on's di s eas e, s t roke, or Mei ge's s yndrome (orofaci al dys t oni a). Bl epharos pas m can be of s uch s everi t y as t o caus e funct i onal bl i ndnes s . o
o
a. Many drugs have been t ri ed i n an at t empt t o cont rol t he probl em wi t h modes t effect .
o
o
b. Infi l t rat i on of bot ul i num t oxi n about t he eyes can provi de rel i ef by decreas i ng neuromus cul ar t rans mi s s i ons .
7. Neuroleptic-associated movement disorders repres ent a s pect rum of di s orders rel at ed t o t he acut e or chroni c admi ni s t rat i on of neurol ept i c medi cat i ons (al t hough for s el ect ed condi t i ons , ot her drugs are i mpl i cat ed as wel l ). o
o
a. Acute dystonia. t ypi cal l y occurs s hort l y aft er t he fi rs t few dos es of a neurol ept i c agent .
Pa g e 2 7 4 6
ABC Ambe r CHM Conve rte r Tria l ve rsion, http://w w w .proce sste x t.com/a bcchm.html
(1) Clinical signs i ncl ude uncont rol l abl e face, neck, t ongue, and eye mus cl e (ocul ogyri c cri s i s ) s pas ms .
(2) T herapy cons i s t s of t he admi ni s t rat i on of ant i chol i nergi cs or di phenhydrami ne.
o
o
b. Parkinsonism can devel op wi t h neurol ept i c us e. Therapy cons i s t s of decreas i ng t he dos e of t he neurol ept i c agent ; changi ng t o anot her neurol ept i c drug; admi ni s t eri ng an ant i chol i nergi c agent ; or us i ng amant adi ne, a carbi dopa/l evodopa preparat i on, or an “at ypi cal ― neurol ept i c s uch as cl ozapi ne, ri s peri done, or ol anzapi ne.
o
o
c. T ardive dyskinesia i s an al mos t cons t ant wri t hi ng movement of t he t ongue and oromandi bul ar area, whi ch may be accompani ed by bl epharos pas m, res pi rat ory grunt s , choreo-at het os i s , and t runcal hyperact i vi t y. Il l -fi t t i ng dent ures or an edent ul ous s t at e can caus e mout hi ng movement s t hat are mi s t aken for t ardi ve dys ki nes i a.
(1) Tardi ve dys ki nes i a i s us ual l y an advers e s i de effect of l ong-t erm us e of neurol ept i c agent s ; occas i onal l y ot her drugs s uch as amphet ami nes , ant i hi s t ami nes , and carbamazepi ne are caus al l y i mpl i cat ed.
Pa g e 2 7 4 7
ABC Ambe r CHM Conve rte r Tria l ve rsion, http://w w w .proce sste x t.com/a bcchm.html
(2) If a medi cat i on i s i mpl i cat ed as t he caus e of t he probl em, t he offendi ng agent s houl d be di s cont i nued i f pos s i bl e. Cl onaz epam, res erpi ne, t et rabenazi ne, and ot her drugs have been us ed t o t reat t ardi ve dys ki nes i a wi t h vari abl e s ucces s . Us e of an “at ypi cal ― neurol ept i c s uch as cl ozapi ne i s anot her opt i on.
o
o
d. Other neuroleptic-associated movement disorders i ncl ude t ardi ve dys t oni a, akat hi s i a (mot or res t l es s nes s ), and t he rabbi t s yndrome (rhyt hmi c l i p movement s ).
o
o
e. Neuroleptic malignant syndrome (NMS) i s an i di os yncrat i c react i on t o neurol ept i c agent s . It can al s o occur i n Parki ns on's di s eas e pat i ent s aft er t he abrupt di s cont i nuat i on of ant i parki ns oni an medi cat i ons . Dopami ne recept or bl ockade i s t hought t o be t he caus e of NMS. P.548
(1) Clinical signs i ncl ude al t ered ment at i on, hi gh fever, ri gi di t y, aut onomi c i ns t abi l i t y, hi gh creat i ne ki nas e (CK) l evel s , and myogl obi nuri a.
(2) T herapy for t hi s pot ent i al l y l et hal condi t i on i ncl udes hydrat i on, cool i ng bl anket s , ant i pyret i cs , dant rol ene, and
Pa g e 2 7 4 8
ABC Ambe r CHM Conve rte r Tria l ve rsion, http://w w w .proce sste x t.com/a bcchm.html
l evodopa/carbi dopa preparat i ons or bromocri pt i ne, al t hough t he us e of ant i parki ns oni an drugs i s cont rovers i al .
8. Meige' s syndrome (orofacial dystonia) i s an i di opat hi c condi t i on t hat has feat ures of dys t oni a and t ardi ve dys ki nes i a, part i cul arl y bl epharos pas m. o
o
a. Thi s di agnos i s cannot be made i f t he pat i ent has recent l y t aken neurol ept i c agent s or ot her drugs i mpl i cat ed as a caus e of t ardi ve dys ki nes i a.
o
o
b. No s i ngl e medi cat i on i s cons i s t ent l y effect i ve i n t reat i ng t hi s condi t i on; cl onaz epam i s a reas onabl e fi rs t -l i ne agent . Bot ul i num t oxi n i nfi l t rat i on can be us ed t o t reat t he bl epharos pas m.
9. Hemifacial spasm des cri bes l i ght ni ng-qui ck s pas ms of mus cl es i nnervat ed by t he faci al nerve. o
o
a. The condi t i on i s mos t oft en caus ed by i rri t at i on of t he faci al nerve by a vas cul ar “l oop.―
o
o
b. Rarel y, i t occurs aft er faci al nerve paral ys i s or i s as s oci at ed wi t h t umors , MS, or ot her i rri t at i ve proces s es .
o
o
c. Botulinum toxin infiltration, clonazepam, and carbamazepine have been us ed t o t reat hemifacial spasm; mi cros urgi cal decompres s i on
Pa g e 2 7 4 9
ABC Ambe r CHM Conve rte r Tria l ve rsion, http://w w w .proce sste x t.com/a bcchm.html
may be s ucces s ful .
10. T ics are bri ef, s t ereot ypi cal , i nvol unt ary movement s , s ounds , or s ens at i ons t hat occur wi t hi n t he cont ext of normal neurol ogi c funct i on. o
o
a. T ypes of tics.
(1) Simple motor tics are i s ol at ed movement s s uch as an eyebl i nk, s houl der s hrug, or faci al gri mace. Complex motor tics i ncl ude t ouchi ng, s mel l i ng, and jumpi ng.
(2) Simple phonic tics i ncl ude t hroat cl eari ng, s ni ffl i ng, and grunt i ng. Complex phonic tics i ncl ude t he repet i t i on of words and coprol al i a.
(3) Sensory tics oft en accompany mot or and phoni c t i cs and are charact eri zed by focal s ens at i ons of pres s ure, t i ckl e, warmt h, or col d.
o
o
b. T ourette' s syndrome
(1) Characteristics
(a) Mul t i pl e mot or and one or more phoni c t i cs (t hat has been pres ent at s ome t i me over t he cours e of t he i l l nes s )
Pa g e 2 7 5 0
ABC Ambe r CHM Conve rte r Tria l ve rsion, http://w w w .proce sste x t.com/a bcchm.html
(b) Ti cs t hat occur many t i mes a day, nearl y every day, for more t han 1 year
(c) Ti cs t hat change over t i me i n t hei r anat omi c l ocat i on, number, frequency, compl exi t y, t ype, and s everi t y
(d) Ons et of i l l nes s before age 21
(e) Abs ence of ot her condi t i ons t hat can caus e s i mi l ar, but i s ol at ed, s ympt oms (e.g., neurol ept i c drug effect s , s ei zures , and chorea)
(2) Etiology. Touret t e's s yndrome i s t hought t o be i nheri t ed i n a pol ygeni c manner. St ri at al dopami ne recept or s upers ens i t i vi t y may be res pons i bl e for t he cl i ni cal mani fes t at i ons .
(3) Associated behavioral disturbances. Obsessive–compulsive disorder and attention deficit hyperactivity disorder may occur.
(4) T herapy. Treat ment cons i s t s of educat i on and couns el i ng of t he pat i ent , fami l y, and ot her appropri at e part i es . Cl oni di ne, pi mozi de, or hal operi dol can be us ed t o manage di s abl i ng t i cs . Behavi oral
Pa g e 2 7 5 1
ABC Ambe r CHM Conve rte r Tria l ve rsion, http://w w w .proce sste x t.com/a bcchm.html
di s t urbances s houl d res pond t o appropri at e ps ychoact i ve medi cat i ons .
XI. Demyelinating Diseases A. Multiple sclerosis (MS) Mul t i pl e s cl eros i s (MS) i s charact eri zed by mul t i pl e foci of CNS demyel i nat i on. Pat i ent s ei t her experi ence cl i ni cal remi s s i ons (oft en fol l owed by rel aps es ) or chroni c, progres s i ve s ympt oms .
1. Pathophysiology. Al t hough t he caus e of MS remai ns unknown, a predomi nant t heory cont ends t hat MS i s an i mmunol ogi c di s order as s oci at ed wi t h CNS i mmunogl obul i n product i on and al t erat i on of T and B l ymphocyt es . The pathologic hallmark of MS i s i nfl ammat i on as s oci at ed wi t h areas of demyelination scattered about CNS white matter. Recent fi ndi ngs have al s o document ed axonal disruption.
2. Diagnosis o
o
a. Clinical signs.
(1) The di agnos i s i s mos t cert ai n i f neurol ogi c probl ems occur over an ext ended peri od and i nvol ve s everal whi t e mat t er pat hways . MRI fi ndi ngs can be us ed as evi dence of di s eas e di s s emi nat i on over t i me and anat omi c s pace.
(2) Several pat t erns emerge t hat are s ugges t i ve of MS, i ncl udi ng optic neuritis (a s i gn of whi ch i s t he Marcus -Gunn pupi l or afferent pupi l l ary defect ) and internuclear
Pa g e 2 7 5 2
ABC Ambe r CHM Conve rte r Tria l ve rsion, http://w w w .proce sste x t.com/a bcchm.html
ophthalmoplegia, ei t her i n i s ol at i on or i n as s oci at i on wi t h cort i cos pi nal t ract or cerebel l ar s i gns . Long-t erm fol l ow-up i ndi cat es t hat 74% of women and 34% of men who pres ent wi t h i s ol at ed opt i c neuri t i s ul t i mat el y devel op MS.
(3) Neurol ogi c s i gns t hat can be l ocal i zed t o a s i ngl e di s cret e area i n t he CNS s uch as t he brai ns t em or crani ocervi cal junct i on s houl d s ugges t an al t ernat e di agnos i s s uch as a t umor or art eri ovenous mal format i on.
o
o
b. Differential diagnosis. There i s no s peci fi c di agnos t i c marker for MS; t he phys i ci an needs t o excl ude ot her condi t i ons t hat can mas querade as MS. Among t hes e di s orders are s omat i zat i on di s order, SLE, brai ns t em or s pi nal vas cul ar mal format i on, Sjögren's s yndrome, Lyme di s eas e, HIV i nfect i on, vi t ami n B 1 2 defi ci ency, brai ns t em neopl as m, vas cul i t i s , s arcoi dos i s , and adrenomyel ol eukodys t rophy.
o
o
c. Diagnostic studies
(1) MRI i s an excel l ent t echni que for vi s ual i zi ng whi t e mat t er l es i ons . Al t hough i t s di agnos t i c s peci fi ci t y i s poor, t he cont ras t agent gadolinium-DT PA i ndi cat es areas of breakdown of t he bl ood–brai n barri er. Seri al MRI s t udi es s how whi t e mat t er l es i ons t hat may come and go wi t hout cl i ni cal mani fes t at i ons .
Pa g e 2 7 5 3
ABC Ambe r CHM Conve rte r Tria l ve rsion, http://w w w .proce sste x t.com/a bcchm.html
(2) Examination of the CSF can i ndi cat e a s t eri l e i nfl ammat i on wi t h a mi l d prot ei n el evat i on; modes t , predomi nant l y mononucl ear pl eocyt os i s ; an el evat ed IgG i ndex; ol i gocl onal bands ; and i ncreas ed myel i n bas i c prot ei n. Onl y occas i onal l y do al l of t hes e abnormal i t i es occur i n a s i ngl e pat i ent .
(3) Visual, brain stem–auditory, and somatosensory EPs and central motor conduction studies can demons t rat e cl i ni cal l y s i l ent di s rupt i on of whi t e mat t er t ract s .
3. T herapy. There i s no cure for MS. o
o
a. Corticosteroid therapy. may has t en maxi mal recovery from an acut e exacerbat i on. If opt i c neuri t i s i s t reat ed, hi gh dos es of i nt ravenous cort i cos t eroi ds are preferabl e t o l ower, oral dos es .
o
o
b. Interferon-β (IFN-β) t herapy decreas es t he frequency of rel aps es , es peci al l y moderat e and s evere at t acks . As judged by s eri al MRI s t udi es , di s eas e act i vi t y i s l es s ened wi t h IFN-β t reat ment . P.549
o
o
c. Glatiramer acetate al s o decreas es t he
Pa g e 2 7 5 4
ABC Ambe r CHM Conve rte r Tria l ve rsion, http://w w w .proce sste x t.com/a bcchm.html
frequency of rel aps es , es peci al l y for pat i ent s wi t h mi l d di s eas e. o
o
d. Ot herwi s e, t reat ment i s di rect ed at symptoms.
(1) Modafinil, amantadine, and pemoline can i mprove fatigue.
(2) Baclofen, tizanidine, and diazepam i mprove spasticity.
(3) A variety of agents may hel p urologic dysfunction, dependi ng on t he s peci fi c probl em.
o
o
e. Pat i ent s wi t h s evere di s eas e may res pond t o i mmunos uppres s i ve t herapy.
B. Central pontine myelinolysis
1. Etiology. Demyel i nat i on of t he pons and deep cerebral whi t e mat t er has been as s oci at ed wi t h t he excessively rapid correction of severe hyponatremia. Cent ral pont i ne myel i nol ys i s has been s een i n al cohol i cs , mal nouri s hed pat i ent s , and i n as s oci at i on wi t h di uret i c us e.
2. Diagnosis. Pat i ent s devel op impaired arousal, quadriparesis, and pseudobulbar signs. MRI or CT scan confi rms t he di agnos i s .
Pa g e 2 7 5 5
ABC Ambe r CHM Conve rte r Tria l ve rsion, http://w w w .proce sste x t.com/a bcchm.html
3. T herapy. Cons i s t s of s upport i ve meas ures .
Hyponat remi a s houl d be correct ed slowly, and hypernat remi a s houl d be avoi ded.
XII. Myelopathy and Other Spinal Cord Disorders A. Definition Myel opat hy refers t o a di s order of t he s pi nal cord. Speci fi c s yndromes occur i n rel at i on t o t he vari ous mot or and s ens ory t ract s affect ed by t he di fferent di s eas e proces s es (Fi gure 11-4).
B. Etiology Sel ect ed caus es of myel opat hy are l i s t ed i n
Onl i ne Tabl e 11-8.
ONLINE TABLE 11-8 Selected Causes of Myelopathy Ver t eb ral col um n di s ord ers T rau ma Cer vi c
Pa g e 2 7 5 6
ABC Ambe r CHM Conve rte r Tria l ve rsion, http://w w w .proce sste x t.com/a bcchm.html
al ste nos is Di s k pro t ru sio n Od ont oi d s ub l ux at i on Rh eu ma t oi d art hri t is Do wn s yn dro me Ne
Pa g e 2 7 5 7
ABC Ambe r CHM Conve rte r Tria l ve rsion, http://w w w .proce sste x t.com/a bcchm.html
opl as i a E pi d ura l s pi nal cor d co mp res sio n I nt r ad ura l ext ra me dul l ar y ma ss I nt r am ed ul l ary
Pa g e 2 7 5 8
ABC Ambe r CHM Conve rte r Tria l ve rsion, http://w w w .proce sste x t.com/a bcchm.html
ma ss Inf ect i on S pi n al epi dur al abs ces s Hu ma n im mu no def i ci e ncy vi r us (HI V) Hu ma n T-c el l l ym
Pa g e 2 7 5 9
ABC Ambe r CHM Conve rte r Tria l ve rsion, http://w w w .proce sste x t.com/a bcchm.html
ph ot r op hi c vi r us t yp e I (HT LVI) Mul t i pl e s cl ero sis Va s cu l ar di s eas e I nfa rct i on Va s cu l ar ma l for ma tio
Pa g e 2 7 6 0
ABC Ambe r CHM Conve rte r Tria l ve rsion, http://w w w .proce sste x t.com/a bcchm.html
n Me t ab ol i c di s eas es Vi t am in B12 def i ci e ncy (s u bac ut e co mb i ne d de ge ner at i on) Vi t am in E def i ci e ncy
Pa g e 2 7 6 1
ABC Ambe r CHM Conve rte r Tria l ve rsion, http://w w w .proce sste x t.com/a bcchm.html
Adr en om yel on eur op at h y Ra di a tio n eff ect s Syr i ng om yel ia Spi noc ere bel l ar de ge ner at i on
C. Clinical signs Di s eas e affect i ng t he s pi nal cord can pres ent wi t h pri nci pal l y “ long tract signs― or a combi nat i on of l ong t ract s i gns and
Pa g e 2 7 6 2
ABC Ambe r CHM Conve rte r Tria l ve rsion, http://w w w .proce sste x t.com/a bcchm.html
local radicular features.
1. Long tract motor signs res ul t from t he di s rupt i on of des cendi ng cort i cos pi nal fi bers , whi ch caus es weakness, spasticity, and hyperreflexia.
2. Impaired sensation res ul t s from di s ordered funct i on of as cendi ng s pi not hal ami c (pai n and t emperat ure) and dors al col umn pat hways (vi brat i on and propri ocept i on) (s ee Fi gure 11-4).
3. Bowel, bladder, and erectile function may be compromised.
4. Local radicular symptoms and signs include radiating pain, weakness, and sensory loss referabl e t o one or s everal myot omes and dermat omes . Thes e fi ndi ngs hel p defi ne t he ros t ral –caudal ext ent of a l es i on (Fi gure 11-5).
D. Selected conditions
1. Syringomyelia o
o
a. Definition. Syri ngomyel i a i s a condi t i on of unknown caus e res ul t i ng i n cavitation of the spinal cord. Syri ngomyel i a can occur i n i s ol at i on or i n as s oci at i on wi t h t he Arnold-Chiari malformation (i .e., des cent of t he cerebel l ar t ons i l s i nt o t he cervi cal s pi nal canal ).
Pa g e 2 7 6 3
ABC Ambe r CHM Conve rte r Tria l ve rsion, http://w w w .proce sste x t.com/a bcchm.html
FIGURE 11-4 Cros s s ect i on of t he s pi nal cord at C1. CST, cort i cos pi nal t ract ; DC, dors al col umn; STT, s pi not hal ami c t ract . o P.550 o
Pa g e 2 7 6 4
ABC Ambe r CHM Conve rte r Tria l ve rsion, http://w w w .proce sste x t.com/a bcchm.html
FIGURE 11-5 Topographi c rel at i ons hi p among nerve root s , s pi nal cord s egment s , and t he bodi es and s pi nous proces s es of t he vert ebrae, whi ch are i ndi cat ed by Roman numeral s . o
o
b. Diagnosis
(1) Signs of lower motor neuron dysfunction devel op on a s egment al bas i s and are accompani ed by upper motor neuron signs caudal t o t he cavi t y.
Pa g e 2 7 6 5
ABC Ambe r CHM Conve rte r Tria l ve rsion, http://w w w .proce sste x t.com/a bcchm.html
(2) Dermatomal loss of pain and temperature sensibility res ul t s from l ocal di s rupt i on of t he cros s i ng s pi not hal ami c fi bers by t he cavi t y (s yri nx). Impai red as cendi ng dors al col umn s ens i bi l i t y occas i onal l y occurs caudal t o t he cavi t y.
(3) MR I of t he s pi ne is t he opt im al dia gn ost ic tes t (
Onl i ne Fi g ure 116).
Pa g e 2 7 6 6
ABC Ambe r CHM Conve rte r Tria l ve rsion, http://w w w .proce sste x t.com/a bcchm.html o
ONLINE FIGURE 11-6 Nonenhanced T 1 -wei ght ed magnet i c res onance i magi ng (MRI) s can demons t rat i ng s yri ngomyel i a and an as s oci at ed Arnol d-Chi ari mal format i on. o
o
c. T herapy. Surgery i s occas i onal l y i ndi cat ed t o decompres s t he fl ui d-fi l l ed s pi nal cord cavi t y and t o perform a bi ops y of t he wal l t o eval uat e t he pos s i bi l i t y of a cavi t ary neopl as m. Otherwise, treatment is supportive.
2. T ransverse myelitis. Infl ammat i on of t he s pi nal cord can caus e an acut e myel opat hy. o
Pa g e 2 7 6 7
ABC Ambe r CHM Conve rte r Tria l ve rsion, http://w w w .proce sste x t.com/a bcchm.html o
a. Etiology. Al t hough many cas es of s egment al t rans vers e i nfl ammat i on are i di opat hi c, MS, SLE, and vari ous i nfect i ous agent s may caus e t rans vers e myel i t i s .
o
o
b. Diagnosis. Pat i ent s may experi ence l ocal i zed back or radi cul ar pai n, fol l owed by pri ckl i ng or burni ng s ens at i ons and progres s i ve weaknes s i n t he l egs . Bowel and bl adder di s t urbances are us ual l y pres ent .
o
o
c. T herapy. Corticosteroid treatment i s oft en advocat ed but i s of unproven val ue.
P.551
3. The Brown-Séquard syndrome i s caus ed by a l es i on of one s i de of t he s pi nal cord at a di s cret e l evel . There i s caudal i ps i l at eral upper mot or neuron weaknes s , upper mot or neuron s i gns , and l os s of propri ocept i on and vi brat ory s ens at i on as wel l as cont ral at eral l os s of pai n and t emperat ure s ens at i on.
4. Anterior spinal artery occlusion o
o
a. Etiology. Bl ockage of a radi cul ar art ery t o t he s pi nal cord can caus e an i s chemi c i nfarct i on.
(1) In many cas es , t he artery of Adamkiewicz, a branch of t he aort a s uppl yi ng
Pa g e 2 7 6 8
ABC Ambe r CHM Conve rte r Tria l ve rsion, http://w w w .proce sste x t.com/a bcchm.html
t he ant eri or t wo-t hi rds of t he l umbar s pi nal cord, i s compromi s ed.
(2) Thrombos i s can be caus ed by aort i c di s s ect i on, l ocal at heros cl eros i s , vas cul i t i s , and hypervi s cos i t y.
o
o
b. Diagnosis. Pat i ent s pres ent wi t h fl acci d, hyporefl exi c parapl egi a; i mpai red l ower ext remi t y pai n and t emperat ure s ens at i on; and compromi s ed bl adder and bowel funct i on. However, pos i t i on and vi brat i on s ens es are us ual l y pres erved.
o
o
c. T herapy. Treat ment of t he di s eas e res pons i bl e for t he s pi nal cord i nfarct i on.
XIII. Neuropathy A. Classification Neuropat hi es may be cl as s i fi ed by:
1. Course (acut e, s ubacut e, or chroni c)
2. T ype of symptoms and signs (s ens ory, mot or, aut onomi c, or any combi nat i on of t he t hree)
3. Presence of pain (hyperes t hes i a or dys es t hes i a)
4. Distribution (general i zed, focal , or mul t i focal )
Pa g e 2 7 6 9
ABC Ambe r CHM Conve rte r Tria l ve rsion, http://w w w .proce sste x t.com/a bcchm.html
5. NCV/EMG features (axonal or demyel i nat i ng)
B. Etiology Sel ect ed caus es of neuropat hy are l i s t ed i n Tabl es 11-9A and 11-9B.
C. Therapy Cont rol of t he underl yi ng di s eas e proces s i s cri t i cal .
1. If i mpai red s ens at i on renders t he pat i ent prone t o i njury, prot ect i ve meas ures s houl d be t aken.
2. W eaknes s (e.g., wri s t or foot drops ) cal l s for appropri at e s pl i nt i ng and phys i cal t herapy.
3. Aut onomi c i ns uffi ci ency i s di ffi cul t t o manage; ort hos t at i c hypot ens i on can be t reat ed wi t h agent s t hat expand bl ood vol ume (e.g., fl udrocort i s one) and i ncreas e vas cul ar t one (e.g., mi dodri ne).
4. Tri cycl i c ant i depres s ant s , carbamaz epi ne, phenyt oi n, and gabapent i n, can hel p pat i ent s wi t h pai n.
D. Selected syndromes
1. Compression neuropathies o
o
a. Pathophysiology.
(1) Nerves can be damaged by repeat ed wear agai ns t fi rm s urfaces , t ypi cal l y bone or fi brous
Pa g e 2 7 7 0
ABC Ambe r CHM Conve rte r Tria l ve rsion, http://w w w .proce sste x t.com/a bcchm.html
t i s s ue. Mot or and s ens ory l os s devel op, referent t o t he affect ed nerve.
(2) Common s i t es of compres s i on l eadi ng t o focal nerve dys funct i on i ncl ude t he median nerve at t he wri s t (carpal t unnel ), t he ulnar nerve at t he el bow, and t he peroneal nerve at t he fi bul ar head.
o
o
b. Etiology. The carpal tunnel syndrome can res ul t from repet i t i ve wri s t movement s , t rauma, carpal t unnel s t enos i s , art hri t i des (rheumat oi d art hri t i s and crys t al -i nduced s ynovi t i s ), di abet es mel l i t us , myxedema, pregnancy, birt h cont rol pi l l s , acromegal y, and i nfi l t rat i ve proces s es s uch as amyl oi dos i s .
o
o
c. Diagnosis. A pos i t i ve Ti nel 's s i gn (el i ci t i ng pares t hes i as by percus s i ng t he nerve at t he s i t e of compres s i on) may be s een cl i ni cal l y. An NCV s t udy reveal s evi dence of focal demyel i nat i on. If compres s i on i s s evere, evi dence of axonal i njury may appear (e.g., mus cl e was t i ng, denervat i on on t he EMG).
o
o
d. T herapy. T reatment of underlying conditions i s i mport ant . Spl i nt i ng oft en al l evi at es t he condi t i on, es peci al l y i f aggravat i ng maneuvers can be el i mi nat ed. Local cort i cos t eroi d i nject i ons may be benefi ci al . Surgical decompression of t he nerve i s neces s ary at t i mes .
P.552
Pa g e 2 7 7 1
ABC Ambe r CHM Conve rte r Tria l ve rsion, http://w w w .proce sste x t.com/a bcchm.html
TABLE 11-9A Selected Types and Causes of Neuropathy Cause s a s c c u h ut b r e a o c ni ut c e o r c Ne
h
ur
r
op
o
ath
ni
y Se
c Di T
ns o
a o
ry
b xi
ne
et n
uro
e s
pat
s Vi
hy
m ta el m lit in u B6 s in Ur t o e
Pa g e 2 7 7 2
ABC Ambe r CHM Conve rte r Tria l ve rsion, http://w w w .proce sste x t.com/a bcchm.html
m xi i a ca Al t i co o h n ol Sj a à b ¶ u gr s e e n' D s ef s y ic n i e dr nc o ie m s e Vi * ta P m ar in a s n B1 e , o B 6 pl , a B st 12
, ic
ni (a ac nt in iH H IV u H a
Pa g e 2 7 7 3
ABC Ambe r CHM Conve rte r Tria l ve rsion, http://w w w .proce sste x t.com/a bcchm.html
er nt e ib di o ta d ry y) n * e P ur ar o a p pr at ot hi ei e n s e Dr m u ia g Cr s y Vi o nc gl a o al b k ul al i n oi e d m s ia Ci A s m pl yl at oi in d P o h si e s
Pa g e 2 7 7 4
ABC Ambe r CHM Conve rte r Tria l ve rsion, http://w w w .proce sste x t.com/a bcchm.html
n L yt e oi pr n o 2 sy â € ², 3 â € ²Di d e o xy cy ti di n e Mo G CI t or ui D ne l l P uro ai L pat n- e hy B a ar d rà i n © to s y xi n ca dr t i o o
Pa g e 2 7 7 5
ABC Ambe r CHM Conve rte r Tria l ve rsion, http://w w w .proce sste x t.com/a bcchm.html
mn e M Di ul a ti b fo et ca e l s m m ot el or lit n u e s ur (p o ro p xi at mh al y is A ch nt e ib mo i c di n e e s ur t o o G p M1 at C h h y) ar Cr co iti tca M l
ar
Pa g e 2 7 7 6
ABC Ambe r CHM Conve rte r Tria l ve rsion, http://w w w .proce sste x t.com/a bcchm.html
ill ie n -T e o s s ot p h ol di y s n e e a ur s o e p at h y P or p h yr ia Se
Di CI
ns o
a D
ri m
b P/
ot o
et D
r
e A
ne
s D
uro
mS
pat
el
hy
lit u s Ur e m
â€
C h ar co tM
Pa g e 2 7 7 7
ABC Ambe r CHM Conve rte r Tria l ve rsion, http://w w w .proce sste x t.com/a bcchm.html
i a ar Cr i e i t i -T ca o l
ot
ill h n di e s ss e p a ol s y e n O e th ur er o h p er at e h di y ta V ry a n sc e ul ur iti o s p H at y hi p e ot s h M yr et oi ac di hr s o
Pa g e 2 7 7 8
ABC Ambe r CHM Conve rte r Tria l ve rsion, http://w w w .proce sste x t.com/a bcchm.html
mm Ly at m ic e le di u s k e o a d s ys e tr P o ar p a h pr y ot R ei ef n s e u mm i a di â€
Cr y o gl o b ul in e m ia P ar a
s e a s e A dr e n o m y el o n
Pa g e 2 7 7 9
ABC Ambe r CHM Conve rte r Tria l ve rsion, http://w w w .proce sste x t.com/a bcchm.html
n e e ur o o pl p a at st h ic y Dr Li u p g o s pr T ot o ei xi n n d s ef ic ie nc ie s S ar co id o si s Aut G Di H on ui a IV om l l b Vi ic
ai et nc
ne n- e ri uro B s s t pat ar m i n
Pa g e 2 7 8 0
ABC Ambe r CHM Conve rte r Tria l ve rsion, http://w w w .proce sste x t.com/a bcchm.html
hy rà el e © lit syu n s dr A o m m yl e oi P d or o p si h s yr F ia a m ili al d ys a ut o n o m ia HIV, human i mmunode fi ci ency vi rus ; CIDP, chroni c i nfl ammat ory
Pa g e 2 7 8 1
ABC Ambe r CHM Conve rte r Tria l ve rsion, http://w w w .proce sste x t.com/a bcchm.html
demyel i na t i ng pol yneuro pat hy; DADS, di s t al acqui red demyel i na t i ng s ymmet ri c neuropat h y. *Al s o, s ens ory neuronopa t hy. â€
Incl udi ng myel i n-as s oci at ed gl ycoprot e i n (MAG) ant i body and monocl on al gammopat hy.
2. Guillain-Barré syndrome o
o
a. Definition and etiology. Gui l l ai n-Barré s yndrome i s a predomi nant l y demyel i nat i ng mot or pol yneuropat hy t hat us ual l y occurs i n ot herwi s e
Pa g e 2 7 8 2
ABC Ambe r CHM Conve rte r Tria l ve rsion, http://w w w .proce sste x t.com/a bcchm.html
heal t hy i ndi vi dual s . The i l l nes s can fol l ow a nons peci fi c vi ral s yndrome or be as s oci at ed wi t h HIV i nfect i on, Campyl obac t er jejuni i nfect i on, hepat i t i s , i nfect i ous mononucl eos i s , Myc opl as ma pneumoni ae i nfect i on, vacci nat i on, s urgery, l ymphoma, or SLE. o
o
b. Diagnosis
(1) Clinical signs
(a) Cl as s i cal l y, pat i ent s pres ent wi t h progres s i ve weaknes s and arefl exi a. Progres s i on of t he di s eas e s houl d not ext end beyond 4–6 weeks .
(b) Generalized paralysis can devel op gradual l y or rel at i vel y acut el y, i mpedi ng res pi rat ory funct i on.
(c) Rel at i vel y mi nor s ens ory s i gns and s ympt oms occur; however, pat i ent s may compl ai n of pai nful ext remi t i es .
(d) The autonomic nervous system i s oft en i nvol ved. Invol vement of t he aut onomi c nervous s ys t em can l ead t o earl y mort al i t y as a res ul t of cardiac arrhythmias and wide swings in blood pressure.
P.553
Pa g e 2 7 8 3
ABC Ambe r CHM Conve rte r Tria l ve rsion, http://w w w .proce sste x t.com/a bcchm.html
TABLE 11-9B Selected Types and Causes of Neuropathy Classified by Other Features Ne Ca ur us op es ath y Dy Di a s es bet t he es t i c me ne l l i t uro us pat Al c hy oh ol ab us e HI V 2â €², 3â €²Di d eox ycy t i di ne Va
Pa g e 2 7 8 4
ABC Ambe r CHM Conve rte r Tria l ve rsion, http://w w w .proce sste x t.com/a bcchm.html
s cu liti s Ax Di a on bet al
es
ne me uro l l i t pat us hy Ure mi a Dru gs an d t ox i ns Cri tic al illn es s pol yne uro pat hy Va s cu liti s Par apr ot e
Pa g e 2 7 8 5
ABC Ambe r CHM Conve rte r Tria l ve rsion, http://w w w .proce sste x t.com/a bcchm.html
i ne mi a Cry ogl ob ul i ne mi a Vi t am in B12 def i ci e ncy Her edi t ar y ne uro pat hi e s De Gui my l l ai el i n-B nat arr i ng é ne s yn uro dro pat me hy CI
Pa g e 2 7 8 6
ABC Ambe r CHM Conve rte r Tria l ve rsion, http://w w w .proce sste x t.com/a bcchm.html
DP Par apr ot e i ne mi a* Her edi t ar y ne uro pat hi e s Mul Di a t i fo bet cal es (m me on l l i t on us eur Va i t i s s cu mu l i t i ltip s l ex Ly )
me
ne di s uro eas pat e hy Lep ros y Sar
Pa g e 2 7 8 7
ABC Ambe r CHM Conve rte r Tria l ve rsion, http://w w w .proce sste x t.com/a bcchm.html
coi dos is Her edi t ar y lia bi l i ty to pre ssu re pal sy Mal i gn ant i nfi l t ra t es HIV, human i mmun odefi ci ency vi rus ; CIDP, chroni c i nfl am mat ory demyel i nat i ng pol yne
Pa g e 2 7 8 8
ABC Ambe r CHM Conve rte r Tria l ve rsion, http://w w w .proce sste x t.com/a bcchm.html
uropat hy. *Incl u di ng myel i n -as s oci at ed gl ycopr ot ei n (MAG) ant i bo dy and monocl onal gammo pat hy.
(2) Diagnostic studies
(a) Exami nat i on of t he CSF s hows an el evat ed prot ei n and l es s t han 50 3
mononucl ear cel l s /mm ( albuminocytologic dissociation).
(b) The mot or nerve conduct i on vel oci t i es are t ypi cal l y s l owed.
(c) An abnormal l y s mal l compound mus cl e act i on pot ent i al ampl i t ude obt ai ned wi t h di s t al s t i mul at i on of a peri pheral nerve (a meas ure of t he i nt egri t y of t he mos t di s t al part s of t he
Pa g e 2 7 8 9
ABC Ambe r CHM Conve rte r Tria l ve rsion, http://w w w .proce sste x t.com/a bcchm.html
axonal port i on of t he nerve) i s as s oci at ed wi t h a poor prognos i s . o
o
c. T herapy
(1) Plasmapheresis can s hort en t he l engt h of t i me t hat pat i ent s are dependent on a res pi rat or and unabl e t o ambul at e. Cri t eri a t o i ni t i at e pl as mapheres i s i ncl ude t he i nabi l i t y t o wal k or rapi d progres s i on of t he di s eas e.
(2) Intravenous immunoglobulin treatment i s al s o effi caci ous and i s bet t er t ol erat ed t han pl as mapheres i s .
3. Diabetic neuropathy (s ee al s o Chapt er 9 IV A 7 d). Di abet es mel l i t us caus es s everal neuropat hi c s yndromes . The nerve i njury may be s econdary t o chroni c hypoxi a (rel at ed t o mi crovas cul ar di s eas e) t hat l eads t o axonal damage. o
o
a. T ypes.
(1) A predomi nant l y s ens ory, di s t al , s ymmet ri c, s mal l fi ber pol yneuropat hy can be dys es t het i c and i nvol ve pai n and t emperat ure modal i t i es more t han vi brat i on and pos i t i on s ens es .
(2) A predomi nant l y s ens ory, di s t al ,
Pa g e 2 7 9 0
ABC Ambe r CHM Conve rte r Tria l ve rsion, http://w w w .proce sste x t.com/a bcchm.html
s ymmet ri c, l arge-fi ber pol yneuropat hy may occur, affect i ng vi brat i on and pos i t i on modal i t i es .
(3) A s ens ori mot or neuropat hy can devel op.
(4) An aut onomi c neuropat hy or a mononeuropat hy or mononeuri t i s mul t i pl ex can occur. P.554
(5) Proxi mal di abet i c neuropat hy repres ent s i njury t o l arge nerves t hat caus es weaknes s and pai n; i t commonl y i nvol ves t he l umbos acral pl exus .
o
o
b. T herapy. As a group, pat i ent s wi t h bet t er bl ood gl ucos e cont rol have a l es s s evere pol yneuropat hy. However, i n i ndi vi dual pat i ent s , s ympt oms us ual l y do not res pond t o t i ght er bl ood gl ucos e cont rol .
4. Chronic inflammatory demyelinating polyneuropathy (CIDP). Thi s condi t i on, whi ch can be i di opat hi c or as s oci at ed wi t h a monocl onal gammopat hy, caus es s ens ori mot or neuropat hy. Ant i myel i n-as s oci at ed gl ycoprot ei n ant i bodi es may be det ect ed or t here may be an IgM monocl onal gammopat hy. Several vari ant s of CIDP have been defi ned bas ed on charact eri s t i c cl i ni cal pres ent at i ons , as s oci at ed monocl onal prot ei ns or ant i bodi es , and res pons e t o t herapy.
Pa g e 2 7 9 1
ABC Ambe r CHM Conve rte r Tria l ve rsion, http://w w w .proce sste x t.com/a bcchm.html o
o
a. Diagnosis. NCV s t udi es i ndi cat e s l owi ng, and t he CSF t ot al prot ei n i s el evat ed.
o
o
b. T herapy. CIDP i s res pons i ve t o cort i cos t eroi d t herapy, pl as mapheres i s , and i nt ravenous i mmunogl obul i n admi ni s t rat i on, as wel l as s everal i mmunos uppres s ant drugs .
XIV. Disorders of the Neuromuscular Junction A. Myasthenia gravis
1. Etiology. Ant i bodi es di rect ed agai ns t t he acet yl chol i ne recept or on t he mus cl e s urface caus e an i ncreas ed rat e of recept or des t ruct i on and l ead t o weaknes s .
2. Diagnosis o
o
a. Clinical signs. Pat i ent s are oft en young women or ol der men. Compl ai nt s of doubl e vi s i on, di ffi cul t y s wal l owi ng and s peaki ng, and l i mb weaknes s and fat i gue are common. On exami nat i on, pt os i s , eye movement abnormal i t i es , and proxi mal weaknes s may be s een. A t hymoma i s pres ent i n 10%–25% of pat i ent s .
o
o
b. Diagnostic studies
Pa g e 2 7 9 2
ABC Ambe r CHM Conve rte r Tria l ve rsion, http://w w w .proce sste x t.com/a bcchm.html
(1) Admi ni s t rat i on of intravenous edrophonium (a s hort -act i ng chol i nes t eras e i nhi bi t or) us ual l y produces a t rans i ent i mprovement i n s t rengt h i n pat i ent s s ufferi ng from myas t heni a gravi s . Pat i ent s wi t h res pi rat ory compromi s e or exces s i ve oral s ecret i ons s houl d not be gi ven edrophoni um becaus e t hey may be unabl e t o compens at e for t he i ncreas e i n s ecret i ons t hat occurs aft er admi ni s t rat i on of t hi s agent .
(2) Repetitive nerve stimulation studies can demons t rat e a decrement al res pons e of t he compound mus cl e act i on pot ent i al .
(3) Acetylcholine receptor antibodies can be det ect ed i n t he bl ood of 80%–90% of pat i ent s .
(4) A thoracic CT or MRI scan s hows a t hymoma, i f pres ent .
3. T herapy o
o
a. The mai ns t ay of t herapy i s admi ni s t rat i on of a cholinesterase inhibitor (e.g., pyri dos t i gmi ne).
o
o
b. T hymectomy can oft en l ead t o i mprovement ; t he pres ence of a t hymoma i s a defi ni t e i ndi cat i on
Pa g e 2 7 9 3
ABC Ambe r CHM Conve rte r Tria l ve rsion, http://w w w .proce sste x t.com/a bcchm.html
for s urgery. o
o
c. Corticosteroids, immunosuppressive agents, intravenous immunoglobulin, or plasmapheresis are effect i ve i n pat i ent s wi t h refract ory di s eas e.
B. Eaton-Lambert myasthenic syndrome
1. Pathophysiology. Thi s s yndrome res ul t s when ant i bodi es di rect ed agai ns t t he cal ci um channel s on t he pres ynapt i c membrane of t he neuromus cul ar junct i on i nt erfere wi t h t he cal ci um-medi at ed rel eas e of acet yl chol i ne ves i cl es i n res pons e t o nerve s t i mul at i on. Thi s s yndrome i s oft en as s oci at ed wi t h an underl yi ng mal i gnancy, es peci al l y small cell carcinoma of the lung, and aut oi mmune di s eas es .
2. Diagnosis o
o
a. Clinical signs. Pat i ent s compl ai n of weaknes s and fat i gue, have di mi ni s hed mus cl e s t ret ch refl exes , and may have i mpai red aut onomi c funct i on, l eadi ng t o dry mout h and poor vi s ual accommodat i on. Unl i ke myas t heni a, s t rengt h i mproves wi t h act i vi t y.
o
o
b. Diagnostic studies. Repet i t i ve nerve s t i mul at i on s t udi es s how an i ncrement al res pons e of t he compound mus cl e act i on pot ent i al .
3. T herapy. Di ami nopyri di ne, pl as mapheres i s ,
Pa g e 2 7 9 4
ABC Ambe r CHM Conve rte r Tria l ve rsion, http://w w w .proce sste x t.com/a bcchm.html
i nt ravenous i mmunogl obul i n, or i mmunos uppres s i ve t herapy may be hel pful . P.555
XV. Disorders of Muscle A. Muscular dystrophies
1. T ypes of muscular dystrophies o
o
a. Duchenne muscular dystrophy (DMD). i s an X-l i nked reces s i ve di s order caus ed by a defect i n t he dys t rophi n gene, whi ch codes for a mus cl e membrane prot ei n, dys t rophi n, t hat i s not det ect abl e i n pat i ent s wi t h DMD.
(1) Al t hough di s t al mus cl es are event ual l y affect ed as wel l , pat i ent s experi ence i ni t i al progres s i ve proxi mal mus cul ar weaknes s .
(2) Prednisone treatment s l ows progres s i on of weaknes s .
(3) Death us ual l y occurs i n t he t hi rd decade of l i fe, oft en as a res ul t of pneumoni a.
o
o
b. Becker' s muscular dystrophy (BMD) i s an X-l i nked reces s i ve di s order of mus cl e caus ed by a defect i n t he dys t rophi n gene, whi ch codes for a
Pa g e 2 7 9 5
ABC Ambe r CHM Conve rte r Tria l ve rsion, http://w w w .proce sste x t.com/a bcchm.html
mus cl e prot ei n, dys t rophi n, t hat i s al t ered or pres ent i n reduced quant i t y i n BMD. Pat i ent s wi t h BMD have a more benign course than those with DMD, with a 50% s urvi val rat e beyond age 50. o
o
c. Myotonic muscular dystrophy i s an aut os omal domi nant di s order l ocal i zed t o a gene, myot oni n-prot ei n ki nas e, l ocat ed on chromos ome 19. Pat i ent s wi t h myot oni c dys t rophy have an i ncreas ed number of CTG t ri nucl eot i de repeat s i n t he ki nas e gene. The s everi t y of t he di s eas e can i ncreas e i n s ucces s i ve generat i ons , and t he CTG repeat s i ncreas e proport i onat el y (a phenomenon known as “ant i ci pat i on―).
(1) Clinical signs i ncl ude a charact eri s t i c mus cl e myot oni a (i .e., pers i s t ent mus cl e act i vi t y i n res pons e t o cont ract i on or percus s i on), di s t al weaknes s , cat aract s , front al bal di ng, i mpai red i nt el l ect , hypers omni a, t es t i cul ar at rophy, cardi omyopat hy, mi t ral val ve prol aps e, and cardi ac conduct i on defect s .
(2) Death occurs i n t he fi ft h or s i xt h decade and t ypi cal l y i s at t ri but abl e t o res pi rat ory compromi s e or cardi ac arrhyt hmi a.
o
o
d. Facioscapulohumeral muscular dystrophy i s an aut os omal domi nant di s order charact eri zed by progres s i ve weaknes s about t he face, neck, upper t ors o, and proxi mal arms . The res pons i bl e gene i s l ocat ed on chromos ome 4.
Pa g e 2 7 9 6
ABC Ambe r CHM Conve rte r Tria l ve rsion, http://w w w .proce sste x t.com/a bcchm.html o
o
e. Limb-girdle muscular dystrophy i s act ual l y a group of aut os omal domi nant and reces s i ve di s orders charact eri zed by progres s i ve l os s of mot or s t rengt h of t he t runk and proxi mal l i mbs .
(1) The di s order repres ent s a group of condi t i ons wi t h di fferent genot ypi c ori gi ns and s i mi l ar phenot ype.
(2) Pat i ent s may have defect s i n t hei r dys t rophi n-as s oci at ed prot ei ns (e.g. members of t he dys t rophi n–gl ycoprot ei n compl ex) or ot her membrane-as s oci at ed prot ei ns .
2. Diagnosis. Pat i ent s t ypi cal l y have an elevated serum CK level. EMG demons t rat es a “myopat hi c― pat t ern (i .e., bri ef, s mal l ampl i t ude mus cl e pot ent i al s ). A muscle biopsy i s oft en i nformat i ve. Genetic studies are i ncreas i ngl y purs ued.
B. Acquired myopathy
1. Etiology. Mus cl e di s eas e can be caus ed by i nfl ammat ory, t oxi c, or met abol i c proces s es (Tabl e 11-10).
2. Diagnosis o
o
a. Clinical signs. A pat i ent hi s t ory and exami nat i on
Pa g e 2 7 9 7
ABC Ambe r CHM Conve rte r Tria l ve rsion, http://w w w .proce sste x t.com/a bcchm.html
may provi de cl ues t o t he di agnos i s .
(1) W eaknes s i s us ual l y proxi mal and s ymmet ri c. Di s eas e ons et can be acut e, s ubacut e, or chroni c.
(2) Swal l owi ng and breat hi ng can be compromi s ed, and myogl obi nuri a may res ul t from rapi d mus cl e des t ruct i on, l eadi ng t o renal i ns uffi ci ency.
o
o
b. Diagnostic studies. The s erum CK l evel may be el evat ed. The EMG reveal s a “myopat hi c― pat t ern. A mus cl e bi ops y i s oft en i nformat i ve.
3. Selected syndromes o
o
a. Corticosteroid myopathy. i s us ual l y caus ed by chroni c cort i cos t eroi d t herapy and i s as s oci at ed wi t h proxi mal mus cl e weaknes s and was t i ng. The s erum CK l evel i s normal , and t he EMG i s us ual l y unremarkabl e.
o
o
b. W i t h polymyositis, t he as s oci at ed i nfi l t rat i on of l ymphocyt es des t roys mus cl e fi ber (s ee Chapt er 10 IX D). P.556
Pa g e 2 7 9 8
ABC Ambe r CHM Conve rte r Tria l ve rsion, http://w w w .proce sste x t.com/a bcchm.html
TABLE 11-10 Selected Causes of Acquired Myopathy Pol ym yos itis I di o pat hi c As s oci at e d wi t h ot h er con nec tiv e tis s ue di s eas es , i nf ect i ou s ag
Pa g e 2 7 9 9
ABC Ambe r CHM Conve rte r Tria l ve rsion, http://w w w .proce sste x t.com/a bcchm.html
ent s (i n cl u di n g HI V), an d dru gs Der ma to my os i tis As s oci at e d wi t h ma lig na ncy Inc l us i on bo dy my
Pa g e 2 8 0 0
ABC Ambe r CHM Conve rte r Tria l ve rsion, http://w w w .proce sste x t.com/a bcchm.html
os i tis El e ct r ol y te di s ord ers Hy pok al e mi a Hy per kal em ia Hy per cal ce mi a Hy po ma gn es e mi
Pa g e 2 8 0 1
ABC Ambe r CHM Conve rte r Tria l ve rsion, http://w w w .proce sste x t.com/a bcchm.html
a Hy po ph os p hat em ia En doc ri n e di s ord ers Hy pot hyr oi d ism Hy per t hy roi di s m Cu s hi ng' s di s
Pa g e 2 8 0 2
ABC Ambe r CHM Conve rte r Tria l ve rsion, http://w w w .proce sste x t.com/a bcchm.html
eas e an d i at r og eni c cor tic os t ero id ad mi ni s t ra tio n Ad di s on' s di s eas e Acr om eg al y Dru gs (e.
Pa g e 2 8 0 3
ABC Ambe r CHM Conve rte r Tria l ve rsion, http://w w w .proce sste x t.com/a bcchm.html
g., †œs t at i ns †• εam i no cap roi c aci d, pro cai na mi de, zid ovu di n e, ph enc ycl i di n e, L-t ryp t op ha n) HI
Pa g e 2 8 0 4
ABC Ambe r CHM Conve rte r Tria l ve rsion, http://w w w .proce sste x t.com/a bcchm.html
V, hu ma n im mu no def i ci e ncy vi r us . o
o
c. Inclusion body myositis i s an i nfl ammat ory myopat hy charact eri zed by a res i s t ance t o cort i cos t eroi d t herapy and by di s t al and proxi mal weaknes s t hat may be as ymmet ri c.
(1) Diagnosis. Mus cl e bi ops y demons t rat es i nfl ammat i on and i ncl us i on bodi es (i ncl udi ng “ri mmed vacuol es ―) t hat cont ai n amyl oi d. Cyt ot oxi c T (Tc) cel l s are act i ve agai ns t a mus cl e ant i gen.
(2) T herapy. There i s no accepted treatment.
o
o
d. Polymyalgia rheumatica (PMR) i s not a myopat hy but can mas querade as one.
(1) Clinical signs
Pa g e 2 8 0 5
ABC Ambe r CHM Conve rte r Tria l ve rsion, http://w w w .proce sste x t.com/a bcchm.html
(a) Pat i ent s oft en appear weak, but t he major s ympt om i s painful (tender, aching, and stiff) muscles.
(b) Act i vi t i es s uch as as cendi ng s t ai rs may be di ffi cul t t o perform. On formal s t rengt h t es t i ng, pat i ent s may appear s l i ght l y weak, pres umabl y becaus e t he pai n prevent s maxi mal effort . It has been found t hat i f t he pai n can be rel i eved, s t rengt h i s pres erved.
(c) T emporal (giant cell) arteritis i s pres ent i n 15%–20% of pat i ent s .
(2) Diagnostic studies. The eryt hrocyt e s edi ment at i on rat e us ual l y i s s i gni fi cant l y el evat ed but may be normal . The s erum CK l evel i s normal , and t he EMG i s unremarkabl e. Mus cl e hi s t ol ogy i s normal ; bi ops y i s us ual l y not performed.
(3) T herapy. NSAID or corticosteroid therapy s houl d bri ng about a rapi d res ol ut i on of s ympt oms .
C. Nondystrophic myotonias (channelopathies) Nondystrophic myotonias (channelopathies) are charact eri zed by prol onged mus cl e rel axat i on aft er vol unt ary cont ract i on or mechani cal s t i mul at i on.
Pa g e 2 8 0 6
ABC Ambe r CHM Conve rte r Tria l ve rsion, http://w w w .proce sste x t.com/a bcchm.html
1. Clinical signs.
Onl i ne Tabl e 11-11 s ummari zes t he cl i ni cal
feat ures of t he nondys t rophi c myot oni as .
ONLINE TABLE 11-11 Clinical Features of the Nondystrophic Myotonias Sodi Chl um orid Cha
e
nnel Cha Dise nnel ase Dise s
ase
s H P Myo y a toni p r
a
e a Con rk m geni al y
ta
e ot m o ic ni a P C T B e o h e ri n o c o g m k F di e p e e c ni s r' at P ta e s n' u a r r e al
s
Pa g e 2 8 0 7
ABC Ambe r CHM Conve rte r Tria l ve rsion, http://w w w .proce sste x t.com/a bcchm.html
y si s P Y Y N Y er e e o e io s s
s
di c p ar al ys is P Y In N N ot e s o o a s o ss
m
iu
e
m
fa
-i
m
n
ili
d
e
uc
s
e d w e a k n e ss C N Y N N ol o e o o d-
s
in
Pa g e 2 8 0 8
ABC Ambe r CHM Conve rte r Tria l ve rsion, http://w w w .proce sste x t.com/a bcchm.html
d uc e d w e a k n e ss P O Y N N ar cc e o o a a s d si o o xi n ca al l m y ot o ni a Pr V V N R o ar ar o ar gr i a i a
e
e bl bl ss e e iv e w e a k
Pa g e 2 8 0 9
ABC Ambe r CHM Conve rte r Tria l ve rsion, http://w w w .proce sste x t.com/a bcchm.html
n e ss S N N N N ys o o o o te m ic in v ol v e m e nt G A A A A e D D D R/ n /1 /1 /7 7 et 7 7 ic tr a n s m is si o n/ ch ro m o s o
Pa g e 2 8 1 0
ABC Ambe r CHM Conve rte r Tria l ve rsion, http://w w w .proce sste x t.com/a bcchm.html
m e lo cu s Adapt ed from Pt acek LJ, Johns on KJ, Gri ggs RC: Genet i cs and phys i ol ogy of t he myot oni c mus cl e di s orders . N Engl J Med 1993;328(7 ):483. AD = aut os omal domi nant ; AR = aut os omal reces s i ve.
2. T herapy. Treat ment i ncl udes t he us e of qui ni ne, procai nami de, and phenyt oi n.
D. Metabolic myopathies Metabolic myopathies i ncl ude di s orders of carbohydrat e and l i pi d met abol i s m.
Pa g e 2 8 1 1
ABC Ambe r CHM Conve rte r Tria l ve rsion, http://w w w .proce sste x t.com/a bcchm.html
E. Myoglobinuria Myoglobinuria can be caus ed by crus h i njuri es , vas cul ar occl us i ons , i nfect i on, t oxi ns , drugs , met abol i c myopat hi es , hypert hermi a, and s evere i nfl ammat i on.
1. Diagnosis i s ai ded by document i ng an el evat ed s erum CK l evel and uri nary myogl obi n. The l at t er can be s us pect ed on t he fi ndi ng of dark (browni s h) uri ne and a pos i t i ve di ps t i ck for bl ood al ong wi t h a paradoxi cal abs ence of RBCs i n t he uri ne.
2. T herapy s houl d be di rect ed at t he underl yi ng caus e. Pat i ent s mus t be kept vi gorous l y hydrat ed t o prevent ki dney damage.
P.557
F. Acute quadriplegic myopathy Acute quadriplegic myopathy can devel op as a compl i cat i on of cri t i cal i l l nes s and t he s ys t emi c i nfl ammat ory res pons e s yndrome. There i s an as s oci at i on wi t h t he us e of cort i cos t eroi ds and nondepol ari zi ng neuromus cul ar bl ocki ng agent s . Treat ment i s s upport i ve.
XVI. Infection A. Meningitis, encephalitis, and neurologic complaints associated with HIV infection A. Meningitis, encephalitis, and neurologic complaints associated with HIV infection are di s cus s ed i n Chapt er 8.
B. Brain abscess
Pa g e 2 8 1 2
ABC Ambe r CHM Conve rte r Tria l ve rsion, http://w w w .proce sste x t.com/a bcchm.html
1. Etiology. An abs ces s can occur aft er neuros urgery or penet rat i ng head t rauma, i n as s oci at i on wi t h ot i t i s medi a or poor oral hygi ene, i n pat i ent s wi t h a bact eremi a, and i n i ndi vi dual s wi t h a pul monary art eri ovenous mal format i on or cardi ac ri ght -t o-l eft s hunt .
2. Diagnosis o
o
a. Clinical signs. Pat i ent s may pres ent wi t h headache, s ei zures , an al t ered s ens ori um, and focal neurol ogi c s ympt oms and s i gns .
o
o
b. Diagnostic studies. CT or MRI scanning can eas i l y det ect a brai n abs ces s , al t hough di fferent i at i on from a neopl as t i c l es i on can be di ffi cul t .
3. T herapy. Empi ri c t reat ment s houl d be di rect ed agai ns t aerobi c and mi croaerophi l i c gram-pos i t i ve s t rept ococci and anaerobes , i ncl udi ng Bac t eroi des fragi l i s . o
o
a. Antibiotic therapy. oft en i ncl udes peni ci l l i n, chl orampheni col , met roni dazol e, and cefot axi me or t ri met hopri m–s ul famet hoxaz ol e. The cl i ni cal s et t i ng can hel p defi ne t he mos t l i kel y pat hogen.
o
o
b. Therapy i s moni t ored by s eri al brai n i magi ng t echni ques .
Pa g e 2 8 1 3
ABC Ambe r CHM Conve rte r Tria l ve rsion, http://w w w .proce sste x t.com/a bcchm.html o
o
c. Oft en, surgical excision of t he abs ces s can be avoided. Stereotactic biopsy and drainage of an abscess occas i onal l y i s requi red.
C. A spinal epidural abscess A spinal epidural abscess i s us ual l y caus ed by hemat ogenous s eedi ng of an i nfect i ve organi s m and i s charact eri zed by l ocal pai n and t endernes s , fever, and neurol ogi c s i gns and s ympt oms appropri at e t o t he s i t e of t he i nfect i on.
1. Diagnosis can be ai ded by spinal MRI or CT scans and myelography.
2. Surgical drainage i s i ndi cat ed al ong wi t h antibiotic therapy.
D. Neurosyphilis
1. Stages and clinical signs o
o
a. Asymptomatic disease. CSF abnormal i t i es may be t he onl y s i gn of i nfect i on.
o
o
b. Acute syphilitic meningitis us ual l y devel ops wi t hi n 2 years of pri mary i nfect i on. Headache, meni ngi s mus , hydrocephal us , and crani al nerve pal s i es can occur.
o
o
c. Cerebrovascular (meningovascular) syphilis
Pa g e 2 8 1 4
ABC Ambe r CHM Conve rte r Tria l ve rsion, http://w w w .proce sste x t.com/a bcchm.html
mani fes t s mont hs t o years aft er pri mary i nfect i on. Affect ed pat i ent s pres ent wi t h i s chemi c s t rokes (oft en as s oci at ed wi t h headache) and behavi oral abnormal i t i es . o
o
d. General paresis devel ops one t o t wo decades aft er pri mary i nfect i on and i s charact eri zed by a progres s i ve dement i a.
o
o
e. T abes dorsalis al s o mani fes t s 10–20 years aft er pri mary i nfect i on wi t h l i ght ni ng pai ns , pares t hes i as , bl adder dys funct i on, gai t i ns t abi l i t y, Argyl l Robert s on pupi l s (i .e., i mpai red pupi l l ary l i ght react i on wi t h pres erved pupi l l ary cons t ri ct i on t o accommodat i on, perhaps as a res ul t of a mi dbrai n t egment al l es i on), arefl exi a (es peci al l y at t he ankl es ), and l os s of pos i t i on and vi brat i on s ens i bi l i t y.
2. Diagnostic studies. Pat i ent s wi t h HIV i nfect i on are at part i cul ar ri s k for neuros yphi l i s . o
o
a. Al mos t al l pat i ent s wi t h neuros yphi l i s have a reactive serum fluorescent treponemal antibody (FT A-ABS) test. Pat i ent s wi t h a nonreact i ve CSF FTA-ABS t es t do not have neuros yphi l i s .
o
o
b. The CSF VDRL t es t i s oft en react i ve i n neuros yphi l i s , but a nonreact i ve CSF VDRL t es t i s found i n 25-75% of pat i ent s wi t h neurol ogi c di s eas e years removed from t he pri mary i nfect i on. P.558
Pa g e 2 8 1 5
ABC Ambe r CHM Conve rte r Tria l ve rsion, http://w w w .proce sste x t.com/a bcchm.html
o
o
c. Other CSF findings i ncl ude a predomi nant l y mononucl ear pl eocyt os i s and an el evat ed prot ei n and IgG i ndex.
3. T herapy. Peni ci l l i n i s t he t reat ment of choi ce for neuros yphi l i s . Pat i ent s wi t h concurrent HIV i nfect i on devel op neuros yphi l i s earl i er and may be more res i s t ant t o t herapy t han i mmunocompet ent pat i ent s .
XVII. Primary CNS Tumors Al t hough t hes e t umors are rel at i vel y uncommon, t hey s eem t o be i ncreas i ng i n i nci dence.
A. Astrocytic neoplasms
1. As t rocyt omas are neopl as ms wi t h s l i ght hypercel l ul ari t y and pl eomorphi s m. Anaplastic astrocytomas are charact eri zed as havi ng moderat e cel l ul ari t y and pl eomorphi s m and s ome vas cul ar prol i ferat i on. o
o
a. Clinical signs. Pat i ent s wi t h as t rocyt omas and anapl as t i c as t rocyt omas t ypi cal l y pres ent wi t h focal hemi s pheri c neurol ogi c dys funct i on, convul s i ons , or headache.
o
o
b. T herapy. Anapl as t i c as t rocyt omas can be t reat ed wi t h surgery and radiotherapy. Chemotherapy may be hel pful .
Pa g e 2 8 1 6
ABC Ambe r CHM Conve rte r Tria l ve rsion, http://w w w .proce sste x t.com/a bcchm.html o
o
c. Prognosis. Out come i s i nvers el y rel at ed t o t he pat i ent 's age and t he pres ence of t umor necros i s . The s urvi val rat e aft er 2 years i s 38%–50%. Ot her predi ct ors of out come i ncl ude t he pat i ent 's funct i onal s t at us and t he amount of res i dual t umor aft er i ni t i al s urgery.
2. Glioblastoma multiforme has moderat e-t o-marked hypercel l ul ari t y, pl eomorphi s m, and necros i s ; vas cul ar prol i ferat i on may be pres ent . o
a. Clinical signs. Cl i ni cal feat ures are s i mi l ar t o t hos e of l es s aggres s i ve t umors . CT or MRI s cans t ypi cal l y s how an enhanci ng, i rregul ar mas s (
Onl i ne Fi gure 11-7).
o
ONLINE FIGURE 11-7 An enhanced T 1 -wei ght ed magnet i c
Pa g e 2 8 1 7
ABC Ambe r CHM Conve rte r Tria l ve rsion, http://w w w .proce sste x t.com/a bcchm.html
res onance i magi ng (MRI) s can demons t rat i ng a gl i obl as t oma mul t i forme. o
o
b. T herapy. Treat ment i ncl udes corticosteroid therapy (for edema reduct i on), surgical debulking, radiation therapy, and chemotherapy, bot h s ys t emi cal l y or l ocal l y admi ni s t ered.
o
o
c. Prognosis. The prognos i s of gl i obl as t oma mul t i forme i s poor, wi t h a s urvi val rat e of 10% aft er 24 mont hs .
B. Oligodendrogliomas
1. Clinical signs. Pat i ent s wi t h t hes e i nfi l t rat i ng t umors commonl y pres ent wi t h headache and convul s i ons ; focal neurol ogi c defi ci t s can devel op.
2. T herapy. Anapl as t i c ol i godendrogl i omas s eem t o be res pons i ve t o radiation therapy and chemotherapy. Surgery i s t he mai ns t ay of t reat ment .
3. Prognosis. The overal l medi an l engt h of s urvi val i s 53 mont hs .
C. Meningiomas Meningiomas are t umors t hat ari s e from t he meni nges and s l owl y enl arge, caus i ng a mas s effect t hat di s pl aces normal s t ruct ures . Angi obl as t i c meni ngi omas are l ocal l y i nvas i ve.
1. Clinical signs. Headache, s ei zures , and focal neurol ogi c s i gns can occur.
Pa g e 2 8 1 8
ABC Ambe r CHM Conve rte r Tria l ve rsion, http://w w w .proce sste x t.com/a bcchm.html
2. T herapy. Treat ment i s surgical resection; radiation therapy can be us ed for i nvas i ve t umors .
3. Prognosis. If t he ent i re t umor can be s urgi cal l y res ect ed, t he majori t y of pat i ent s do wel l . If t he ent i re t umor cannot be removed, t he pat i ent may experi ence recurrence of s ympt oms .
D. Schwannomas Schwannomas of t he ei ght h crani al nerve (acous t i c neuroma) t ypi cal l y ari s e from t he ves t i bul ar component of t he nerve. They can enl arge and di s pl ace s t ruct ures about t he cerebel l opont i ne angl e.
1. Clinical signs. Pat i ent s devel op di zzi nes s , heari ng l os s , and t i nni t us . A di mi ni s hed corneal refl ex may be a s i gn of t ri gemi nal nerve compromi s e by an enl argi ng mas s .
2. Diagnostic studies. Di agnos i s i s bes t made wi t h an enhanced MRI.
3. T herapy. Treat ment opt i ons i ncl ude surgical resection or stereotactic radiation therapy.
4. Prognosis. Smal l t umors can be s urgi cal l y cured. If res i dual t umor remai ns , recurrent s ympt oms can devel op, us ual l y years l at er.
P.559
Pa g e 2 8 1 9
ABC Ambe r CHM Conve rte r Tria l ve rsion, http://w w w .proce sste x t.com/a bcchm.html
E. Primary CNS lymphoma Primary CNS lymphoma i s i ncreas i ng i n i nci dence, es peci al l y i n i mmunocompromi s ed pat i ent s . Occas i onal l y, pri mary CNS l ymphoma pres ent s as meni ngeal l ymphomat os i s .
1. Diagnostic studies. Imaging studies s how one or more i nt ens el y enhanci ng l es i ons , t ypi cal l y i n a peri vent ri cul ar di s t ri but i on. The di agnos i s can be es t abl i s hed wi t h s t ereot act i c bi ops y.
2. T herapy. Al t hough i ni t i al t reat ment wi t h cort i cos t eroi ds can l ead t o a rapi d decreas e i n t he s i ze of t he mas s , ul t i mat e s urvi val depends on radi at i on t herapy and chemot herapy.
3. Prognosis. The devel opment of mul t i modal i t y t reat ment prot ocol s has ext ended pat i ent s urvi val for years , es peci al l y i n i mmunocompet ent pat i ent s .
XVIII. Hereditary Disorders
A. Wilson's disease (hepatolenticular degeneration) Thi s aut os omal reces s i ve di s eas e i s l ocal i zed t o an abnormal i t y of t he AT P7B gene on chromos ome 13. It may be as s oci at ed wi t h t he accumul at i on of copper i n t he brai n, l i ver, and ot her t i s s ues . Abnormal i t i es i n t he copper-bi ndi ng prot ei n cerul opl as mi n are us ual l y found.
Pa g e 2 8 2 0
ABC Ambe r CHM Conve rte r Tria l ve rsion, http://w w w .proce sste x t.com/a bcchm.html
1. Clinical signs. Pres ent i ng s i gns may i ncl ude hepat i c di s eas e; Kays er-Fl ei s cher ri ngs i n Des cemet 's membrane of t he cornea; or neurol ogi c s ympt oms : behavi oral probl ems (i ncl udi ng ps ychos i s ), movement di s orders (e.g., i ncoordi nat i on, t remor, mas ked faci es , dys t oni a, and at het os i s ); and hemol yt i c anemi a. There are al s o “pres ympt omat i c― pat i ent s who may pres ent wi t hout t hes e s ympt oms .
2. Diagnosis o
o
a. Di agnos i s i s made by fi ndi ng a l ow s erum cerul opl as mi n l evel (us ual l y), exces s i ve 24-hour uri ne copper excret i on, Kays er-Fl ei s cher ri ngs on eye exami nat i on wi t h a s l i t l amp, or i ncreas ed l evel s of hepat i c copper (by bi ops y).
o
o
b. MRI may reveal at rophy of t he caudat e and put amen wi t h i ncreas ed s i gnal i nt ens i t y on T 2 -wei ght ed i mages .
3. T herapy. Treat ment cons i s t s of copper chelation wi t h t ri ent i ne or D-peni ci l l ami ne, i f copper l evel s are hi gh enough. Zi nc may be us ed bot h as a mai nt enance t herapy or for pres ympt omat i c pat i ent s .
B. Neurofibromatosis (NF)
1. Neurofibromatosis type 1 (NF 1, von
Pa g e 2 8 2 1
ABC Ambe r CHM Conve rte r Tria l ve rsion, http://w w w .proce sste x t.com/a bcchm.html
Recklinghausen' s disease) i s an aut os omal domi nant di s order. The res pons i bl e gene i s l ocat ed on chromos ome 17, and t he gene product i s neurofi bromi n. o
o
a. Pat i ent s who meet t wo or more of t he fol l owi ng cri t eri a can be di agnos ed as s ufferi ng from NF 1.
(1) Neurofibromas (t wo or more, or one pl exi form neurofi broma)
(2) Café-au-lait macules (s i x or more meas uri ng 1.5 cm i n t hei r great es t di mens i on)
(3) Freckling i n t he axillary or inguinal areas
(4) Optic glioma
(5) Two or more iris hamartomas (Lisch nodules)
(6) Sphenoid dysplasia or thinning of the cortex of the long bones
(7) An immediate relative wi t h NF 1
o
o
b. Complications of NF 1 i ncl ude as t rocyt i c t umors , opt i c gl i oma, neurofi bros arcoma, compres s i ve peri pheral neuropat hi es , compres s i ve myel opat hy,
Pa g e 2 8 2 2
ABC Ambe r CHM Conve rte r Tria l ve rsion, http://w w w .proce sste x t.com/a bcchm.html
pheochromocyt oma, and s col i os i s . o
o
c. T herapy i s di rect ed at t he compl i cat i ons of t he di s eas e.
2. Neurofibromatosis type 2 (NF 2) i s an aut os omal domi nant di s order l ocal i zed t o chromos ome 22 and charact eri zed by bi l at eral acous t i c neurofi bromas .
C. von Hippel-Lindau disease Thi s aut os omal domi nant di s order i s l ocal i zed t o chromos ome 3. Cerebellar and retinal hemangioblastomas are charact eri s t i c.
1. Hemangi obl as t omas al s o may be found t hroughout t he CNS. The ki dneys , pancreas , and l i ver can harbor hemangi omas . Pheochromocyt omas may devel op.
2. Pol ycyt hemi a i s as s oci at ed wi t h cerebel l ar hemangi obl as t oma.
D. Osler-Weber-Rendu disease (hereditary hemorrhagic telangiectasia) Thi s aut os omal domi nant di s order i s as s oci at ed wi t h t wo genes ( HHT 1 and HHT 2). It i s charact eri zed by t el angi ect at i c s ki n l es i ons , CNS vas cul ar mal format i ons , and pul monary art eri ovenous fi s t ul ae. Brai n abs ces s es may occur, as may i s chemi c s t roke, s econdary t o a paradoxi cal embol us vi a a pul monary art eri ovenous fi s t ul a.
Pa g e 2 8 2 3
ABC Ambe r CHM Conve rte r Tria l ve rsion, http://w w w .proce sste x t.com/a bcchm.html
E. Tuberous sclerosis Thi s aut os omal domi nant di s order i s as s oci at ed wi t h t wo abnormal genes , T SC1 and T SC2.
1. Criteria for diagnosis i ncl ude mul t i pl e faci al angi ofi bromas , ungual fi bromas , ret i nal hamart oma, cort i cal t ubers , s ubependymal gl i al nodul es (oft en cal ci fi ed), and mul t i pl e renal angi omyol i pomas .
2. Some associated features are hypomel anot i c macul es , a s hagreen pat ch, mul t i cys t i c ki dneys , s ei z ures , and ment al ret ardat i on. Beni gn gi ant cel l as t rocyt omas can devel op, oft en near t he foramen of Monro.
F. Down syndrome (trisomy 21) Thi s genet i c di s order i s charact eri zed by ment al ret ardat i on, epi cant hal fol ds , Brus hfi el d s pot s on t he i ri s , a t rans vers e pal mar creas e, and cardi ac mal format i ons . By age 50, mos t pat i ent s devel op Al zhei mer's di s eas e. Myel opat hy may devel op becaus e of at l ant oaxi al di s l ocat i on.
G. Huntington's disease The gene t hat i s res pons i bl e for t hi s aut os omal domi nant di s order, whi ch i s l ocal i zed t o chromos ome 4, has an exces s of t ri nucl eot i de repeat s and encodes t he prot ei n hunt i ngt i n. Pat i ent s devel op a progres s i ve cogni t i ve decl i ne and choreoat het os i s bet ween 30 and 50 years of age. Depres s i on frequent l y occurs . At rophy of t he caudat e nucl eus i s charact eri s t i c.
Pa g e 2 8 2 4
ABC Ambe r CHM Conve rte r Tria l ve rsion, http://w w w .proce sste x t.com/a bcchm.html
H. Cerebellar atrophies Thes e condi t i ons are as s oci at ed wi t h mul t i s ys t em degenerat i ve proces s es i ncl udi ng abnormal ocul omot or funct i on, bul bar was t i ng and fas ci cul at i ons , upper mot or neuron s i gns , ext rapyrami dal dys funct i on, cogni t i ve decl i ne, vi s ual l os s , and peri pheral neuropat hy.
1. Etiology. Thes e di s eas es can be t rans mi t t ed as aut os omal domi nant or reces s i ve di s orders , or t hey may be s poradi c and pres umabl y rel at ed t o a new mut at i on.
2. T ypes. Numerous aut os omal domi nant s pi nocerebel l ar at axi as have been genet i cal l y i dent i fi ed. Mos t of t hes e at axi as are charact eri zed by an exces s number of t ri nucl eot i de repeat s and are i dent i fi ed as s pi nocerebel l ar at rophy t ype 1 (SCA-1), SCA-2, and s o on. o
o
a. The phenot ypi c–genot ypi c correl at i ons are oft en poor, maki ng i t di ffi cul t t o accurat el y predi ct a pat i ent 's genot ype from cl i ni cal eval uat i on al one.
o
o
b. Mol ecul ar genet i cs wi l l al l ow furt her genot ypi c cl as s i fi cat i on of t hes e di s orders .
3. Associated conditions. Cerebel l ar at rophi es have been as s oci at ed phenot ypi cal l y wi t h defi ci enci es of gl ut amat e dehydrogenas e, pyruvat e dehydrogenas e compl ex, and hexos ami ni das e; vi t ami n E defi ci ency; mi t ochondri al di s orders ; and el evat ed very l ong–chai n fat t y aci ds .
Pa g e 2 8 2 5
ABC Ambe r CHM Conve rte r Tria l ve rsion, http://w w w .proce sste x t.com/a bcchm.html
I. Peroxisome disorders The peroxi s ome i s a cel l ul ar organel l e t hat i s i nvol ved wi t h fat t y aci d oxi dat i on.
1. Adrenoleukodystrophy i s an X-l i nked di s order charact eri zed by progres s i ve i nt el l ect ual decl i ne, s pas t i ci t y, and vi s ual l os s . Adrenomyeloneuropathy, al s o an X-l i nked di s order, i s charact eri zed by a progres s i ve myel opat hy and neuropat hy. o
o
a. W hi t e mat t er changes are vi s i bl e on CT and MRI s cans i n adrenol eukodys t rophy.
o
o
b. Bot h condi t i ons res ul t i n exces s i ve l evel s of very l ong–chai n fat t y aci ds i n t he bl ood, perhaps as a res ul t of defect i ve β-oxi dat i on.
2. Refsum disease i s an aut os omal reces s i ve di s order charact eri zed by a s ens ori mot or neuropat hy, pi gment ary ret i nopat hy, at axi a, anos mi a, heari ng l os s , s ki n l es i ons , and el evat ed CSF prot ei n. o
o
a. Pathophysiology. There i s an exces s of phyt ani c aci d i n t he bl ood becaus e of a defect i n al pha-oxi dat i on of t hi s fat t y aci d.
o
o
b. T herapy. Di et ary cont rol of phyt ani c aci d i nt ake and pl as mapheres i s can hel p cont rol t he neurol ogi c
Pa g e 2 8 2 6
ABC Ambe r CHM Conve rte r Tria l ve rsion, http://w w w .proce sste x t.com/a bcchm.html
probl ems .
J. Mitochondrial disorders Mi t ochondri al DNA codes for component s of t he mi t ochondri al res pi rat ory chai n and oxi dat i ve phos phoryl at i on enzymat i c compl exes . Di s orders of mi t ochondri al DNA are t rans mi t t ed by nonmendel i an, mat ernal i nheri t ance. The mi t ochondri al encephal opat hi es and myopat hi es have ragged red fi bers i n mus cl e. The ragged red fi bers repres ent abundant abnormal mi t ochondri a t hat are demons t rat ed by us i ng a modi fi ed Gomori t ri chrome s t ai n. Pat i ent s may have s ens ori neural heari ng l os s , s hort s t at ure, or di abet es mel l i t us , and t hey oft en have el evat ed bl ood l act i c aci d l evel s .
1. Kearns-Sayre syndrome i s charact eri zed by progres s i ve ext ernal opht hal mopl egi a, pi gment ary ret i nopat hy, compl et e heart bl ock, CSF prot ei n l evel s great er t han 100 mg/dL, at axi a, and myopat hy. Del et i ons i n t he mi t ochondri al DNA are found.
2. Myoclonic epilepsy with ragged red fibers (MERRF) pres ent s wi t h myocl onus , epi l eps y, and at axi a. There i s a poi nt mut at i on i n t he mi t ochondri al genome.
3. Mitochondrial encephalomyopathy with lactic acidosis and stroke-like events (MELAS) i s charact eri zed by i nt ermi t t ent vomi t i ng and headaches , recurrent i s chemi c s t rokes , and s ei zures . Poi nt mut at i ons are pres ent i n t he mi t ochondri al DNA.
4. Leber' s hereditary optic neuropathy mani fes t s as
Pa g e 2 8 2 7
ABC Ambe r CHM Conve rte r Tria l ve rsion, http://w w w .proce sste x t.com/a bcchm.html
s ubacut e bi l at eral cent ral vi s i on l os s wi t h ret i nal mi croangi opat hy. It i s caus ed by s everal poi nt mut at i ons i n t he mi t ochondri al genome.
XIX. Toxic and Metabolic Disorders A. Vitamin B 12 deficiency Thi s vi t ami n defi ci ency res ul t s i n s ubacut e combi ned degenerat i on or SCD.
1. Pathophysiology. Depri vat i on of vi t ami n B 1 2 l eads t o demyel i nat i on and axonal degenerat i on, affect i ng t he peri pheral nerves , t he s pi nal cord (where t he pos t eri or and l at eral col umns are demyel i nat ed), and t he cerebrum.
2. Diagnosis o
o
a. Clinical signs. Neurol ogi c mani fes t at i ons i ncl ude cogni t i ve i mpai rment , di mi ni s hed pos i t i on and vi brat ory s ens at i on, upper mot or neuron s i gns wi t h abnormal gai t , and s ens ory peri pheral neuropat hy.
o
o
b. Diagnosis. Pat i ent s t ypi cal l y have anemi a, macrocyt os i s , and a l ow vi t ami n B 1 2 l evel . However, neurol ogi c di s eas e can occur wi t hout anemi a or macrocyt os i s . If t he di agnos i s remai ns s us pect i n a pat i ent wi t h a normal or margi nal l y decreas ed s erum vi t ami n B 1 2 l evel , t he fi ndi ng of el evat ed s erum met hyl mal oni c aci d and t ot al homocys t ei ne can confi rm t he pres ence of vi t ami n B 1 2 defi ci ency.
Pa g e 2 8 2 8
ABC Ambe r CHM Conve rte r Tria l ve rsion, http://w w w .proce sste x t.com/a bcchm.html
3. T herapy. Treat ment i s admi ni s t rat i on of cobalamin.
B. Acute intermittent porphyria
1. Pathophysiology. Thi s aut os omal domi nant di s order i s caus ed by a defect i n t he act i vi t y of uroporphyri nogen I s ynt het as e, whi ch l eads t o i ncreas ed act i vi t y of Δ-ami nol evul i ni c aci d s ynt het as e and el evat ed l evel s of Δ-ami nol evul i ni c aci d.
2. Diagnosis. Acut e i nt ermi t t ent porphyri a caus es del i ri u m; s ei zures ; and aut onomi c, s ens ory, and mot or neuropat hi es ; a Gui l l ai n-Barré–l i ke i l l nes s can devel op. Duri ng an acut e at t ack, t he Watson-Schwartz test i ndi cat es el evat ed l evel s of uri nary porphobi l i nogen.
3. T herapy. Hematin administration can decreas e cl i ni cal mani fes t at i ons . Preci pi t ant s s uch as barbi t urat es , phenyt oi n, s t arvat i on, and i nfect i on s houl d be avoi ded i n s us cept i bl e i ndi vi dual s .
C. Complications of alcohol abuse Al cohol may affect t he nervous s ys t em by i t s el f (i .e., al cohol i nt oxi cat i on, addi ct i on, or wi t hdrawal ) or i n t andem wi t h a nut ri t i onal defi ci ency. P.560
1. Complications from intoxication, addiction, and
Pa g e 2 8 2 9
ABC Ambe r CHM Conve rte r Tria l ve rsion, http://w w w .proce sste x t.com/a bcchm.html
withdrawal o
o
a. Acute alcohol intoxication. caus es del i ri um and i ncoordi nat i on.
(1) Severe i nt oxi cat i on can l ead t o met abol i c coma wi t h res pi rat ory depres s i on.
(2) Al cohol i c “ blackouts― are charact eri zed by t he i nabi l i t y t o form new memori es i n s pi t e of pres ervat i on of cons ci ous nes s .
o
o
b. Alcohol withdrawal caus es earl y s ympt oms of t remul ous nes s and hal l uci nos i s . Del i ri um t remens i s s een duri ng l at e wi t hdrawal from al cohol . Benzodi azepi nes may be us ed t o t reat al cohol wi t hdrawal s ympt oms .
o
o
c. Alcohol-related seizures (“ rum fits―) are bri ef, general i zed t oni c–cl oni c convul s i ons t hat frequent l y occur i n cl us t ers .
(1) Cl as s i cal l y, al cohol -rel at ed s ei zures were t hought t o occur pri mari l y i n t he 48 hours aft er wi t hdrawal from chroni c al cohol i nt ake. However, al cohol -rel at ed s ei zures al s o may be caus ed by chroni c al cohol i nt oxi cat i on and repres ent a t oxi c effect of al cohol on t he brai n, not a wi t hdrawal phenomenon.
Pa g e 2 8 3 0
ABC Ambe r CHM Conve rte r Tria l ve rsion, http://w w w .proce sste x t.com/a bcchm.html
(2) Typi cal l y, pat i ent s are al ert s hort l y aft er t he s ei zure. They do not have s ei zure di s charges on an i nt eri ct al EEG.
(3) T herapy s houl d be di rect ed at t he underl yi ng al cohol abus e. Ant i convul s ant medi cat i ons are not i ndi cat ed for t ypi cal al cohol -rel at ed s ei zures .
o
o
d. Al cohol abus e i s as s oci at ed wi t h cerebral atrophy and cognitive impairments and may predispose patients to stroke.
o
o
e. Al cohol i c pat i ent s may devel op a myopathy t hat i s acut e or chroni c and predomi nant l y affect s t he proxi mal mus cl es . Rhabdomyolysis can compl i cat e t he acut e di s order.
2. Complications related to nutritional deficiencies o
o
a. Wernicke' s encephalopathy. i s s een i n mal nouri s hed al cohol i c pat i ent s who have a thiamine deficiency. Ot her condi t i ons (e.g., hyperemes i s , renal di al ys i s , and mal nut ri t i on) are as s oci at ed wi t h t hi ami ne defi ci ency and can l ead t o W erni cke's encephal opat hy, even i n pat i ent s who do not abus e al cohol .
(1) Clinical signs. Pat i ent s t ypi cal l y are del i ri ous and have nys t agmus , eye movement
Pa g e 2 8 3 1
ABC Ambe r CHM Conve rte r Tria l ve rsion, http://w w w .proce sste x t.com/a bcchm.html
abnormal i t i es , and at axi a. Di s orders of cons ci ous nes s and hypot hermi a can occur.
(2) T herapy. Thi ami ne admi ni s t rat i on can res ol ve t he acut e i l l nes s .
o
o
b. Korsakoff psychosis i s a chroni c encephal opat hy t hat i s s een i n al cohol i c pat i ent s wi t h a t hi ami ne defi ci ency.
(1) Clinical signs. Feat ures i ncl ude ret rograde and ant erograde memory defi ci t s , apat hy, and i mpai red probl em-s ol vi ng abi l i t i es .
(2) T herapy. Pat i ent s may not i mprove wi t h abs t i nence from al cohol or t hi ami ne admi ni s t rat i on.
o
o
c. Sensory neuropathy i s caus ed by al cohol and oft en l eads t o dys es t hes i a. Al t hough t hi s axonal (al t hough s ome aut hors s ugges t demyel i nat i ve) neuropat hy i s probabl y rel at ed t o concurrent mal nut ri t i on, al cohol i t s el f may be t oxi c t o nerves .
o
o
d. Cerebellar degeneration may occur i n al cohol i cs . The ant eri or and s uperi or vermi s are part i cul arl y affect ed. Pat i ent s have di ffi cul t y wi t h gai t and s t ance; nys t agmus and arm at axi a are not promi nent .
D. Drug abuse
Pa g e 2 8 3 2
ABC Ambe r CHM Conve rte r Tria l ve rsion, http://w w w .proce sste x t.com/a bcchm.html
1. Us e of cocaine or “ crack― can res ul t i n s ei z ures , i s chemi c s t roke, s ubarachnoi d and i nt raparenchymal hemorrhage, and rhabdomyol ys i s . Hemorrhagi c s t rokes may be as s oci at ed wi t h an underl yi ng vas cul ar l es i on; defi ni t i on of t he vas cul ar anat omy i s us ual l y i ndi cat ed t o s earch for an aneurys m or art eri ovenous mal format i on.
2. Us e of heroin can caus e a met abol i c coma; mi os i s i s a charact eri s t i c fi ndi ng i n heroi n us ers . Heroi n has been as s oci at ed wi t h rhabdomyol ys i s , chroni c myopat hy, t rans vers e myel i t i s , and peri pheral neuropat hi es .
3. Inges t i on of phencyclidine (PCP) can caus e a wi de range of neurol ogi c di s t urbances rangi ng from acut e ps ychos i s t o coma. Nys t agmus , mi os i s , at axi a, myocl onus , dys t oni c pos t uri ng, general i zed ri gi di t y, dys ki nes i as , and s ei zures can occur.
4. Inges t i on of “ ecstasy― (an amphet ami ne anal og) can caus e del i ri um, hypert hermi a, and rhabdomyol ys i s . Thi s drug s el ect i vel y damages s erot oni nergi c neurons .
P.561
XX. Sleep Disorders A. Narcolepsy Narcolepsy i s charact eri zed by hypers omni a wi t h s hort l at ency peri ods for t he ons et of dayt i me s l eep and t he earl y devel opment
Pa g e 2 8 3 3
ABC Ambe r CHM Conve rte r Tria l ve rsion, http://w w w .proce sste x t.com/a bcchm.html
of rapi d eye movement (REM) s l eep. Pat i ent s awaken refres hed from s l eep at t acks .
1. Etiology. Narcol eps y i s probabl y an aut os omal domi nant di s order wi t h vari abl e penet rance; i t has been mapped t o chromos ome 6. The vas t majori t y of pat i ent s have ant i gens for HLA-DR2 and HLA-DQB1.
2. Clinical signs. Feat ures i ncl ude cataplexy (i .e., t rans i ent epi s odes of di mi ni s hed mus cl e t one), sleep paralysis, and vivid dreams at t he begi nni ng and end of s l eep.
3. T herapy. Met hyl pheni dat e, pemol i ne, or modafi ni l can di mi ni s h t he hypers omni a, and prot ri pt yl i ne or i mi prami ne can decreas e cat apl exy.
B. Sleep apnea Sleep apnea, or hypovent i l at i on duri ng s l eep, can be at t ri but abl e t o obs t ruct i ve caus es (e.g., obes i t y, a s mal l oropharynx) or t o nonobs t ruct i ve (CNS) caus es . Sl eep apnea i s commonl y “mi xed†• and repres ent s bot h CNS and obs t ruct i ve probl ems .
1. Clinical signs. Pat i ent s have dayt i me hypers omni a and can devel op noct urnal hypoxi a, pul monary hypert ens i on, and cardi ac arrhyt hmi as .
2. T herapy. Treat ment opt i ons i ncl ude wei ght l os s , cont i nuous pos i t i ve ai rway pres s ure (CPAP), uvul opal at opharyngopl as t y (UPPP), t racheos t omy, and s everal new, mi ni mal l y i nvas i ve s urgi cal t echni ques .
C. Periodic movements in sleep
Pa g e 2 8 3 4
ABC Ambe r CHM Conve rte r Tria l ve rsion, http://w w w .proce sste x t.com/a bcchm.html
Periodic movements in sleep i nvol ve fl exor cont ract i ons i n t he l egs . Pat i ent s can al s o experi ence s l eep myocl onus and dys t oni a and res t l es s l egs s yndrome whi l e awake.
1. Clinical signs. Pat i ent s are moment ari l y arous ed, and t herefore noct urnal s l eep qual i t y i s poor, l eadi ng t o dayt i me hypers omni a.
2. T herapy. Treat ment opt i ons i ncl ude carbi dopa/l evodopa combi nat i ons , dopami ne agoni s t s , gabapent i n, opi at es , and cl onaz epam.
XXI. Trauma A. Brain injury
1. A concussion i s a moment ary di s rupt i on of brai n funct i on aft er head i njury t hat res ul t s i n a bri ef l os s of cons ci ous nes s . o
o
a. Clinical signs. Aft er awakeni ng, pat i ent s compl ai n of headache, i mpai red memory, poor concent rat i on, bl urred vi s i on, t i nni t us , di zzi nes s , and naus ea.
o
o
b. Duration of symptoms. Sympt oms can pers i s t for days or weeks as t he postconcussion syndrome. Posttraumatic migraine can devel op but may be al l evi at ed by us i ng ant i mi grai ne agent s .
Pa g e 2 8 3 5
ABC Ambe r CHM Conve rte r Tria l ve rsion, http://w w w .proce sste x t.com/a bcchm.html
2. Severe head trauma can caus e brai n cont us i on, epi dural and s ubdural hemat oma, penet rat i ng brai n i njuri es , SAH, and CSF l eaks . o
o
a. Clinical signs. Progres s i ve ment al s t at us changes l eadi ng t o coma and focal neurol ogi c s i gns can devel op.
o
o
b. Diagnostic studies. Emergent CT s canni ng can hel p del i neat e t he ext ent of t he probl em.
o
o
c. T herapy. Cont rol of ICP and neuros urgi cal eval uat i on are i ndi cat ed as appropri at e.
3. A subdural hematoma can res ul t from acut e head t rauma. Pat i ent s pres ent wi t h an al t ered s ens ori um and focal neurol ogi c fi ndi ngs (e.g., hemi pares i s ). A s ubdural hemat oma i s al s o as s oci at ed wi t h mi nor t rauma and can pres ent wi t h s ubt l e s ubacut e or chroni c behavi oral changes and mi l d focal defi ci t s . At t i mes , no hi s t ory of t rauma i s el i ci t ed; t hi s i s es peci al l y t rue i n el derl y pat i ent s . o
a. Diagnostic studies. A CT s can i s very hel pful i n document i ng t he pres ence of a s ubdural hemat oma (
Onl i ne Fi gure 11-8).
Pa g e 2 8 3 6
ABC Ambe r CHM Conve rte r Tria l ve rsion, http://w w w .proce sste x t.com/a bcchm.html o
ONLINE FIGURE 11-8 A nonenhanced comput ed t omography (CT) s can s howi ng a chroni c s ubdural hemat oma. Not e t he mas s effect on t he l at eral vent ri cl e and t he abs ence of s ul ci . P.562
o
o
b. T herapy. Neuros urgi cal i nt ervent i on i s us ual l y i ndi cat ed for an acut e s ubdural hemat oma; a chroni c s ubdural hemat oma may not requi re s urgery.
B. Spinal cord injury Spinal cord injury can res ul t i n an acut e parapares i s or quadri pares i s . A cent ral cord s yndrome, whi ch can fol l ow a hyperext ens i on i njury t o t he cervi cal s pi nal cord, i s charact eri zed by l ower mot or neuron dys funct i on affect i ng t he cervi cal myot omes , mi l d s ens ory l os s i n t he arms , and a myel opat hy.
Pa g e 2 8 3 7
ABC Ambe r CHM Conve rte r Tria l ve rsion, http://w w w .proce sste x t.com/a bcchm.html
1. Diagnostic studies. Becaus e t he vert ebral col umn may not be s t abl e, pat i ent s s houl d be i mmobi l i zed whi l e a radi ographi c as s es s ment i s made.
2. T herapy. Hi gh-dos e met hyl predni s ol one t herapy gi ven wi t hi n 8 hours of i njury can i mprove out come.
P.563
Study Questions/Answers and Explanations 1. A 70-year-old man, a retired professor, complains of weakness and fatigue. Any physical activity is an effort, and he cannot find a comfortable position at rest. He has lost 5 pounds over the past month. Physical examination is normal. Which of the following diagnoses is most likely? A. Depres s i on B. Hypokal emi a C. Temporal (gi ant cel l ) art eri t i s D. Pol ymyal gi a rheumat i ca (PMR) E. Pol ymyos i t i s Vi ew Ans wer 1. T he answer is D [IV B 5 a; XV B 3 d (1)–(2)]. Pol ymyal gi a rheumat i ca (PMR) i s charact eri zed by mus cl e di s comfort , and pat i ent s oft en pres ent wi t h vague compl ai nt s , as i n t hi s cas e. By defi ni t i on, t he res ul t s of t he neuromus cul ar exami nat i on are normal . Depres s i on can caus e fat i gue and di mi ni s hed act i vi t y, but di s comfort i s l es s frequent l y a compl ai nt . A normal eryt hrocyt e s edi ment at i on rat e, whi ch i s el evat ed i n t he pres ence of PMR, woul d s upport a di agnos i s of depres s i on. Hypokal emi a can caus e mus cl e weaknes s , but di s comfort i s not a t ypi cal feat ure. Temporal (gi ant cel l ) art eri t i s can occur wi t h PMR, but t hi s pat i ent has no
Pa g e 2 8 3 8
ABC Ambe r CHM Conve rte r Tria l ve rsion, http://w w w .proce sste x t.com/a bcchm.html
compl ai nt of headache or s ympt oms referabl e t o vi s i on. Pol ymyos i t i s can caus e mus cl e di s comfort , but t here s houl d be accompanyi ng weaknes s . The s erum creat i ne ki nas e (CK) l evel i s oft en el evat ed. 2. A 29-year-old woman suddenly develops a left hemiparesis. T he patient experienced a deep venous thrombosis in her right leg 3 years ago. Which of the following conditions is the most likely cause of this patient' s deficit? A. Nonval vul ar at ri al fi bri l l at i on B. Lupus ant i coagul ant C. Mi t ral val ve prol aps e D. Mul t i pl e s cl eros i s (MS) E. As t rocyt oma Vi ew Ans wer 2. T he answer is B [VIII B 6 b]. The l upus ant i coagul ant (an ant i phos phol i pi d ant i body) i s as s oci at ed wi t h peri pheral venous t hrombos i s and i s chemi c (art eri al ) s t roke. In pat i ent s wi t h a hi s t ory of deep venous t hrombos i s , t he pos s i bi l i t y of a paradoxi cal embol us caus i ng a s t roke (vi a a ri ght -t o-l eft cardi ac s hunt ) s houl d al s o be cons i dered. Nonval vul ar at ri al fi bri l l at i on i s a common caus e of s t roke i n t he el derl y, but t here i s no reas on t o s us pect a rhyt hm di s t urbance i n t hi s pat i ent . Mi t ral val ve prol aps e has been as s oci at ed wi t h cardi ogeni c embol i and s t roke. However, a s earch for ot her caus es of s t roke s houl d al ways be made, becaus e mi t ral val ve prol aps e i s a rel at i vel y common ent i t y, and t he as s oci at i on wi t h s t roke i s weak. Mul t i pl e s cl eros i s (MS) and an as t rocyt oma can caus e neurol ogi c defi ci t s i n young pat i ent s ; however, t hes e condi t i ons are not as s oci at ed wi t h deep venous t hrombos i s unl es s t he pat i ent i s i mmobi l i z ed. Thes e di agnos es s houl d be cons i dered i n t he eval uat i on of pat i ent s wi t h hemi pares i s , but t he pat i ent 's hi s t ory can oft en s erve as a cl ue t o gui de di agnos t i c t hi nki ng. 3. An assembly-line worker complains of awakening at night with right-hand discomfort that resolves after several minutes. After 3 weeks of continuing symptoms, he seeks medical advice.
Pa g e 2 8 3 9
ABC Ambe r CHM Conve rte r Tria l ve rsion, http://w w w .proce sste x t.com/a bcchm.html
Examination discloses mild weakness of thumb abduction and diminished pain sensibility on the palmar aspect of the thumb and index finger. Which of the following diagnoses is most likely? A. Carpal t unnel s yndrome B. Cervi cal radi cul opat hy C. Refl ex s ympat het i c dys t rophy D. Tendi ni t i s E. Left mi ddl e cerebral art ery i s chemi c at t acks Vi ew Ans wer 3. T he answer is A [XIII D 1 a–b]. Peopl e s uch as as s embl y-l i ne workers and t ypi s t s are part i cul arl y prone t o carpal t unnel s yndrome becaus e t hei r dai l y act i vi t i es requi re repet i t i ve wri s t movement s , whi ch may, i n t i me, compres s t he medi an nerve, caus i ng neuropat hy. Awakeni ng at ni ght wi t h hand di s comfort i s a common compl ai nt , and t he fi ndi ngs on exami nat i on s upport t he di agnos i s . A cervi cal radi cul opat hy can caus e s ome of t hes e fi ndi ngs . However, t he hi s t ory i s more s ugges t i ve of carpal t unnel s yndrome. Pat i ent s wi t h a cervi cal radi cul opat hy oft en have a concurrent carpal t unnel s yndrome and vi ce vers a. The hi s t ory and exami nat i on are not s upport i ve of a di agnos i s of t endi ni t i s or refl ex s ympat het i c dys t rophy. Trans i ent i s chemi c at t acks (TIAs ) rarel y caus e di s comfort and are not as s oci at ed wi t h neurol ogi c defi ci t s aft er 24 hours have el aps ed. 4. A 73-year-old woman presents with a 6-month history of deteriorating gait and low back discomfort, which is exacerbated by walking. Examination is unremarkable except for hypoactive muscle stretch reflexes in the legs. Radiographs of the lumbosacral area show the expected degenerative changes associated with a woman of her age. Which of the following diagnoses is most likely? A. Acut e l umbar di s k herni at i on B. Lumbar s t enos i s C. Myopat hy D. Normal pres s ure hydrocephal us (NPH) E. Cervi cal s t enos i s
Pa g e 2 8 4 0
ABC Ambe r CHM Conve rte r Tria l ve rsion, http://w w w .proce sste x t.com/a bcchm.html
Vi ew Ans wer 4. T he answer is B [VII A 2 b]. Lumbar s t enos i s i s caus ed by degenerat i ve changes i n t he l umbos acral s pi ne, oft en i n as s oci at i on wi t h a congeni t al l y s mal l l umbos acral i nt ras pi nal s pace. The hi s t ory i s oft en t hat of vague l ow back di s comfort as s oci at ed wi t h s ubt l e fi ndi ngs on exami nat i on referabl e t o i mpi ngement on mot or and s ens ory root s . Di agnos i s can be made by t he charact eri s t i c fi ndi ng of an “hourgl as s ― appearance on magnet i c res onance i magi ng (MRI) s cans . An acut e di s k herni at i on i s charact eri zed by l ow back di s comfort and pai n ext endi ng i n a radi cul ar fas hi on down one or bot h l egs . Exami nat i on i s oft en cons i s t ent wi t h i mpi ngement on a s i ngl e s ens ory or mot or root . A myopat hy can caus e an i mpai red gai t ; l ow back di s comfort becaus e of weaknes s ; and hypoact i ve mus cl e s t ret ch refl exes , t ypi cal l y at t he knees . However, t hi s condi t i on i s much l es s common t han l umbar s t enos i s and t herefore i s not t he mos t l i kel y di agnos i s . Normal pres s ure hydrocephal us (NPH) caus es an apract i c gai t (i .e., di ffi cul t y i n wal ki ng i n s pi t e of an i nt act mot or, s ens ory, and cerebel l ar exami nat i on), cogni t i ve i mpai rment , and uri nary i ncont i nence. The cl i ni cal pi ct ure i s not cons i s t ent wi t h t hi s di agnos i s . Cervi cal s t enos i s can caus e a myel opat hy and res ul t ant gai t probl em. Becaus e t he pat i ent does not exhi bi t s i gns of a myel opat hy, t hi s di agnos i s i s unl i kel y. 5. A 45-year-old right-handed man complains that he has had difficulty holding and using a writing instrument for the past year. He notes the development of right-hand and forearm spasms only when writing. Physical examination is unremarkable. Which of the following diagnoses is most likely? A. Parki ns on's di s eas e B. Focal dys t oni a C. Carpal t unnel s yndrome D. Cervi cal radi cul opat hy E. Beni gn es s ent i al t remor Vi ew Ans wer 5. T he answer is B [X B 1; X B 4 XIII D 1 a–b]. W ri t er's cramp i s
Pa g e 2 8 4 1
ABC Ambe r CHM Conve rte r Tria l ve rsion, http://w w w .proce sste x t.com/a bcchm.html
a focal dys t oni a of unknown caus e. Pat i ent s devel op s ympt oms of cramps or s pas ms wi t h al t ered hand and arm pos t ure when at t empt i ng t he s peci fi c t as k of wri t i ng. Exami nat i on of t he pat i ent i s ot herwi s e normal . Mi crographi a i s a s ympt om of Parki ns on's di s eas e but i s us ual l y accompani ed by s i gns of ri gi di t y and bradyki nes i a, and oft en t remor. Carpal t unnel s yndrome i s caus ed by pres s ure on t he medi an nerve as i t ent ers t he hand vi a t he carpal t unnel . Medi an nerve dys funct i on l eads t o hand weaknes s and l os s of s ens i bi l i t y, whi ch can affect wri t i ng. A cervi cal radi cul opat hy can l ead t o hand numbnes s and weaknes s and hyporefl exi a. The exact di s t ri but i on of fi ndi ngs depends on t he nerve root s i nvol ved. A beni gn es s ent i al t remor i s charact eri zed by a di s t al upper ext remi t y t remor duri ng a t as k. There i s no accompanyi ng ri gi di t y or bradyki nes i a. Handwri t i ng i n part i cul ar may s uffer. Carpal t unnel s yndrome, cervi cal radi cul opat hy, and beni gn es s ent i al t remor are al l unl i kel y becaus e t he pat i ent 's exami nat i on s hows no s i gns or s ympt oms ot her t han di ffi cul t y wi t h wri t i ng. 6. A 24-year-old construction worker with a 2-year history of low back pain complains of an acute onset of bilateral leg weakness and incontinence. Which of the following treatments would be the best management tactic? A. Admi ni s t rat i on of nons t eroi dal ant i -i nfl ammat ory drugs (NSAIDs ) B. Emergency s urgery P.564
C. St ri ct bed res t D. Lumbar t ract i on E. Back exerci s es Vi ew Ans wer 6. T he answer is B [VII A 2 a (2)–(3)]. The pat i ent probabl y has an acut e di s k herni at i on caus i ng a cauda equi na s yndrome, an i ndi cat i on for emergency s urgery. Surgery s houl d be cons i dered as an el ect i ve i nt ervent i on for t hos e pat i ent s wi t h s ci at i ca,
Pa g e 2 8 4 2
ABC Ambe r CHM Conve rte r Tria l ve rsion, http://w w w .proce sste x t.com/a bcchm.html
non-di s abl i ng neurol ogi c defi ci t s , and di s k herni at i on (as demons t rat ed by appropri at e i magi ng t echni ques ) who fai l t o res pond t o 6 weeks of cons ervat i ve management . Nons t eroi dal ant i -i nfl ammat ory drugs (NSAIDs ) can be hel pful i n t he t reat ment of acut e, and pos s i bl y chroni c, l ow back pai n; however, t hey are not i ndi cat ed as pri mary t herapy for pat i ent s wi t h acut e, s evere neurol ogi c di s abi l i t y. Avoi dance of s t renuous act i vi t y i s very hel pful for t he t reat ment of acut e back pai n, but t hi s t herapy i s not appropri at e for t hi s pat i ent . Lumbar t ract i on i s probabl y not effect i ve for t he t reat ment of l ow back pai n. Pat i ent s t ypi cal l y cannot exerci s e duri ng t he fi rs t few days of acut e back pai n. However, as t he acut e pai n s ubs i des , an exerci s e program may hel p prevent fut ure probl ems . 7. A 32-year-old man presents with repetitive generalized motor convulsions that continue for 35 minutes until 2 mg of lorazepam are administered intravenously. T he next course of action should be to administer which of the following? A. Phenyt oi n i nt ravenous l y B. Carbamazepi ne oral l y C. Pent obarbi t al i nt ravenous l y D. Et hos uxi mi de oral l y E. Di az epam rect al l y Vi ew Ans wer 7. T he answer is A [IX D 2 b (1)–(2), 4 b, c]. Admini s t rat i on of i nt ravenous l orazepam s houl d be fol l owed by t he admi ni s t rat i on of phenyt oi n (or fos phenyt oi n) t o cont rol s t at us epi l ept i cus becaus e t he durat i on of act i on of l orazepam i s l i mi t ed. Therefore, unl es s t here i s a cont rai ndi cat i on, t he pat i ent s houl d recei ve a l oadi ng dos e of phenyt oi n (or fos phenyt oi n) i nt ravenous l y. Carbamazepi ne i s an effect i ve ant i convul s ant , but i t cannot be gi ven i nt ravenous l y or i nt ramus cul arl y. Therefore, a t herapeut i c l evel cannot be rapi dl y achi eved gi ven a drug hal f-l i fe of 8–12 hours . Int ravenous pent obarbi t al can be us ed t o cont rol repet i t i ve s ei z ures . However, becaus e t he pat i ent i s not current l y convul s i ng, i nduct i on of barbi t urat e coma i s not i ndi cat ed. Et hos uxi mi de i s
Pa g e 2 8 4 3
ABC Ambe r CHM Conve rte r Tria l ve rsion, http://w w w .proce sste x t.com/a bcchm.html
i ndi cat ed for t he t reat ment of abs ence s ei zures . Therefore, t hi s i s not an appropri at e t herapy for general i zed mot or convul s i ons . Rect al di azepam i s us ed t o abort s ei zures t emporari l y, es peci al l y i n chi l dren. 8. A 78-year-old woman complains of experiencing headaches and progressive confusion for the past month. She has a left hemianopia and cannot dress herself. A computed tomography (CT ) scan demonstrates a large, irregularly enhancing mass in the right parietal lobe. T here is no obvious systemic disease. Which of the following diagnoses is most likely? A. Brai n abs ces s B. Gl i obl as t oma mul t i forme C. Meni ngi oma D. Met as t as i s E. Cent ral nervous s ys t em (CNS) l ymphoma Vi ew Ans wer 8. T he answer is B [XVII A 2]. A l arge, i rregul arl y enhanci ng cent ral nervous s ys t em (CNS) mas s i n an el derl y pat i ent wi t hout s ys t emi c cancer i s hi ghl y s ugges t i ve of a gl i obl as t oma mul t i forme. However, a bi ops y i s neces s ary before a defi ni t i ve di agnos i s can be made. The pat i ent has no predi s pos i ng condi t i on for a brai n abs ces s s uch as poor dent i t i on or i nt ravenous drug abus e. A meni ngi oma can occur i n t he pari et al area and di s t ort t he brai n; however, a “convexi t y― meni ngi oma i s t ypi cal l y a homogeneous l y enhanci ng l es i on. In t he abs ence of s ys t emi c cancer, a brai n met as t as i s i s an unl i kel y caus e of t he pat i ent 's probl em. However, wi t hout a bi ops y, t he di agnos i s of met as t at i c di s eas e cannot be excl uded. CNS l ymphoma t ypi cal l y mani fes t s as a homogeneous l y enhanci ng mas s l es i on. Pat i ent s wi t h human i mmunodefi ci ency vi rus (HIV) and ot her i mmunocompromi s ed pat i ent s are prone t o CNS l ymphoma; i n ot her popul at i ons , CNS l ymphoma i s rare. 9. A 63-year-old woman develops intermittent dizziness. Examination shows a diminished right corneal reflex and mild hearing loss in the right ear. Which of the following diagnoses is
Pa g e 2 8 4 4
ABC Ambe r CHM Conve rte r Tria l ve rsion, http://w w w .proce sste x t.com/a bcchm.html
most likely? A. Cerebel l opont i ne angl e t umor B. Beni gn paroxys mal pos i t i onal vert i go (BPPV) C. Lat eral medul l ary s yndrome D. Méni ère's di s eas e E. Pont i ne i nfarct i on Vi ew Ans wer 9. T he answer is A [XVII D 1–3]. A cerebel l opont i ne angl e t umor s uch as a s chwannoma can caus e i nt ermi t t ent di zzi nes s . The t umor can ari s e from t he ei ght h crani al nerve, t hereby al s o affect i ng heari ng, and can pres s on t he t ri gemi nal nerve, caus i ng i mpai rment of t he corneal refl ex. Beni gn paroxys mal pos i t i onal vert i go (BPPV) caus es i nt ermi t t ent bri ef di z z i nes s t hat i s dependent on pos t ural changes . Nys t agmus i s charact eri s t i c, but t here are no ot her neurol ogi c defi ci t s . A l at eral medul l ary s yndrome us ual l y caus es cons t ant di zzi nes s t hat i s exacerbat ed wi t h movement . Al t hough t he corneal refl ex can be depres s ed, heari ng i s normal . Meni ere's di s eas e caus es i nt ermi t t ent di z z i nes s and heari ng l os s , but t he corneal refl ex i s not di mi ni s hed. A pont i ne i nfarct i on does not caus e i nt ermi t t ent s ympt oms ; heari ng l os s i s not expect ed as a res ul t . 10. A previously vigorous 80-year-old woman collapses when getting out of bed. Examination of her legs indicates bilateral weakness, loss of pain and temperature sensation, and areflexia. Her bladder is distended. T he remainder of the examination is unremarkable. Which of the following diagnoses is most likely? A. Gui l l ai n-Barré s yndrome B. Ant eri or cerebral art ery occl us i on C. Cauda equi na s yndrome D. Ant eri or s pi nal art ery occl us i on E. Thoraci c s pi nal cord compres s i on Vi ew Ans wer 10. T he answer is D [XII D 4 b]. Thrombos i s of t he caudal ant eri or s pi nal art ery l eads t o a fl acci d parapl egi a, l os s of pai n and
Pa g e 2 8 4 5
ABC Ambe r CHM Conve rte r Tria l ve rsion, http://w w w .proce sste x t.com/a bcchm.html
t emperat ure s ens at i on, and bowel and bl adder dys funct i on. The bl ood s uppl y t o t he caudal s pi nal cord ari s es from t he aort a vi a t he l umbar Art ery of Adamki ewi cz. The perfus i on t erri t ory of t he ant eri or s pi nal art ery i nvol ves t he ant eri or horn cel l s and t he pai n and t emperat ure pat hways . In t he Gui l l ai n-Barré s yndrome, whi ch rarel y devel ops s uddenl y, l os s of t emperat ure and pai n s ens at i on can occur but rarel y wi t hout a concurrent l os s of pos i t i on and vi brat i on s ens e. Sens ory l os s i s us ual l y l es s s evere t han mot or l os s . General i zed arefl exi a i s common. An ant eri or cerebral art ery t hrombos i s , es peci al l y when bi l at eral , caus es l eg weaknes s , but s ens ory l os s , when pres ent , i nvol ves al l modal i t i es . Upper mot or neuron s i gns are pres ent i n t he l eg. A cauda equi na s yndrome [VII A 2 (3) a] can caus e a fl acci d parapl egi a, but al l s ens ory modal i t i es may be compromi s ed becaus e s ens ory nerve root s , whi ch carry s ens ory fi bers of al l t ypes , are i nvol ved i n t he di s eas e proces s . Thoraci c s pi nal cord compres s i on i s oft en as s oci at ed wi t h back pai n and produces upper mot or neuron dys funct i on i n t he l egs . In t he acut e condi t i on, t he bl adder may be di s t ended, but wi t h t i me, i t s capaci t y i s t ypi cal l y reduced. 11. A 70-year-old man reports that over the past 2 months he has had progressive difficulty walking. Examination indicates distal (greater than proximal) weakness in the arms and legs and the absence of muscle stretch reflexes. Motor nerve conduction velocities (NCVs) are slowed. Which of the following diagnoses is most likely? A. Gui l l ai n-Barré s yndrome B. Lead poi s oni ng C. Chroni c i nfl ammat ory demyel i nat i ng pol yneuropat hy (CIDP) D. Amyot rophi c l at eral s cl eros i s (ALS) E. Pol ymyos i t i s Vi ew Ans wer 11. T he answer is C [XIII D D 4; Tabl es 11-9A and 11-9B]. Chroni c i nfl ammat ory demyel i nat i ng pol yneuropat hy (CIDP) i s charact eri zed by progres s i ve weaknes s t hat occurs over an ext ended t i me (e.g.,
Pa g e 2 8 4 6
ABC Ambe r CHM Conve rte r Tria l ve rsion, http://w w w .proce sste x t.com/a bcchm.html
>4 weeks ), abs ent mus cl e s t ret ch refl exes , and s l owed mot or nerve conduct i on vel oci t i es (NCVs ). Gui l l ai n-Barré s yndrome s houl d not caus e progres s i ve wors eni ng of s t rengt h beyond 4 weeks . Typi cal l y, onl y s ome mot or nerves demons t rat e s l owi ng, us ual l y i n a s egment al , rat her t han a general i z ed, fas hi on. Lead poi s oni ng can caus e a mot or neuropat hy i n chi l dren but onl y rarel y i n adul t s . Amyot rophi c l at eral s cl eros i s (ALS) caus es progres s i ve weaknes s , but mot or NCVs are unremarkabl e and mus cl e s t ret ch refl exes are oft en hyperact i ve. Pol ymyos i t i s caus es progres s i ve proxi mal (great er t han di s t al ) weaknes s and i s not as s oci at ed wi t h i mpai red mot or NCVs . Mus cl e s t ret ch refl exes are oft en pres erved i n proport i on t o t he mus cl e s t rengt h. 12. A 56-year-old man presents with a 2-year history of impotence and not feeling “ right.― Examination reveals masked facies, bradykinesia, rigidity, mild ataxia, and postural hypotension. T he diagnosis is which of the following? A. Parki ns on's di s eas e B. Subacut e combi ned degenerat i on P.565
C. Mul t i pl e s ys t ems at rophy D. Spi nocerebel l ar degenerat i on E. Vi t ami n E defi ci ency Vi ew Ans wer 12. T he answer is C [VI A 1 b; X A 2 a (2); XIX A 2 a; onl i ne Tabl e 11-7]. The s i gns and s ympt oms are mos t cons i s t ent wi t h mul t i pl e s ys t ems at rophy (Shy-Drager s yndrome). The earl y appearance of aut onomi c dys funct i on (i mpot ence and pos t ural hypot ens i on) and at axi a do not s upport t he di agnos i s of Parki ns on's di s eas e. Combi ned s ys t ems degenerat i on i s due t o vi t ami n B 1 2 defi ci ency. Parki ns oni an feat ures are not a feat ure of t he di s eas e. Spi nocerebel l ar degenerat i on i s charact eri zed by cerebel l ar dys funct i on and an accompanyi ng upper mot or neuron
Pa g e 2 8 4 7
ABC Ambe r CHM Conve rte r Tria l ve rsion, http://w w w .proce sste x t.com/a bcchm.html
s yndrome. Ext rapyrami dal and aut onomi c dys funct i on are not promi nent feat ures . Vi t ami n E defi ci ency can mas querade as s pi nocerebel l ar degenerat i on. 13. A patient with a subarachnoid hemorrhage (SAH) caused by a right anterior communicating artery aneurysm undergoes successful surgery 2 days after the hemorrhage. T hree days later, right arm weakness develops. Which of the following diagnoses is most likely? A. Hydrocephal us B. Meni ngi t i s C. Repeat hemorrhage D. Vas os pas m E. Hyponat remi a Vi ew Ans wer 13. T he answer is D [VIII C 1 c (2)]. Vas os pas m can devel op s everal days aft er an aneurys mal s ubarachnoi d hemorrhage (SAH). Pat i ent s pres ent wi t h progres s i ve weaknes s and al t erat i ons i n cons ci ous nes s . Earl y i n t he cours e, a comput ed t omography (CT) s can may not reveal an i s chemi c i nfarct i on. Hydrocephal us can occur i mmedi at el y aft er an SAH or weeks t o mont hs l at er. Sympt oms are t ypi cal l y nonfocal and, i f t he hydrocephal us devel ops acut el y, i t i s oft en accompani ed by a depres s ed l evel of cons ci ous nes s . Bact eri al meni ngi t i s can devel op aft er a crani ot omy. Typi cal l y, t here i s fever and i mpai red arous al . Focal s i gns can devel op but are rarel y t he pres ent i ng feat ure. Al t hough a repeat hemorrhage can occur aft er cl i ppi ng of an aneurys m i f t he aneurys m i s not compl et el y i s ol at ed from t he ci rcul at i on, i t i s unus ual for t hi s t o happen and pres ent wi t h a focal defi ci t , as oppos ed t o depres s ed cons ci ous nes s . Hyponat remi a, whi ch can devel op aft er SAH, can caus e an al t ered s ens ori um and s ei z ures but not uni l at eral weaknes s . 14. A 17-year-old high school varsity diver develops a headache, dizziness, left-sided arm and leg clumsiness, and loss of pain and temperature sensation in the left facial and right body areas after a practice session. Which of the following diagnoses is
Pa g e 2 8 4 8
ABC Ambe r CHM Conve rte r Tria l ve rsion, http://w w w .proce sste x t.com/a bcchm.html
most likely? A. Beni gn paroxys mal pos i t i onal vert i go (BPPV) B. Mul t i pl e s cl eros i s (MS) C. Vert ebral art ery di s s ect i on D. As t rocyt oma E. Labyri nt hi t i s F. Mi grai ne wi t h aura Vi ew Ans wer 14. T he answer is C [VI B 2 f]. The pat i ent has mani fes t at i ons of a l at eral medul l ary s yndrome. In a young pat i ent , who has s ubject ed hi ms el f t o s t renuous neck movement s , t he mos t l i kel y et i ol ogy i s vert ebral art ery di s s ect i on. Beni gn paroxys mal pos i t i onal vert i go (BPPV) caus es s udden epi s odes of di zzi nes s , t ypi cal l y wi t h changes i n pos i t i on. However, t here are no as s oci at ed neurol ogi c s ympt oms or s i gns ot her t han nys t agmus . Mul t i pl e s cl eros i s (MS) can caus e di zzi nes s and cl ums i nes s . However, t he cons t el l at i on of fi ndi ngs i n t hi s pat i ent , whi ch are referabl e t o a s i ngl e s i t e wi t hi n t he brai n and are of s udden ons et , make t he di agnos i s of MS l es s l i kel y. An as t rocyt oma can caus e di zzi nes s and uns t eadi nes s . Oft en t here i s a headache. The ons et i s t ypi cal l y i ns i di ous , and t he s ympt oms progres s i ve. Labyri nt hi t i s caus es s evere vert i go. Pat i ent s fi nd i t di ffi cul t t o move about and prefer t o remai n s t i l l . Gai t i ns t abi l i t y can occur becaus e of t he profound di z zi nes s i n t he abs ence of cerebel l ar defi ci t s . There are no neurol ogi c s i gns ot her t han nys t agmus . Mi grai ne wi t h aura i s as s oci at ed wi t h headache and focal neurol ogi c s ympt oms and s i gns . Unl es s pat i ent s have a hi s t ory s i mi l ar t o t hat des cri bed, t hey s houl d be eval uat ed for al t ernat i ve di agnos es s uch as art eri al di s s ect i on. Pat i ent s wi t h mi grai ne as s oci at ed wi t h focal neurol ogi c defi ci t s s houl d be eval uat ed for al t ernat e di agnos es before t he probl em i s at t ri but ed t o mi grai ne wi t h aura. 15. A 26-year-old woman presents with recurrent throbbing headaches accompanied by nausea, emesis, photophobia, and phonophobia. She has been treating herself with acetaminophen and ice packs to her forehead. A reasonable therapeutic
Pa g e 2 8 4 9
ABC Ambe r CHM Conve rte r Tria l ve rsion, http://w w w .proce sste x t.com/a bcchm.html
intervention might be to prescribe: A. Bacl ofen B. Ami t ri pt yl i ne C. Indomet haci n D. Cort i cos t eroi ds E. Sumat ri pt an Vi ew Ans wer 15. T he answer is B [IV B 1 d (1)–(2) (a)]. Migrai ne headaches res pond wel l t o t reat ment wi t h t ri cycl i c ant i depres s ant s s uch as ami t ri pt yl i ne. Thi s pat i ent has been unres pons i ve t o o ver-t he-count er acet ami nophen. As s umi ng s he i s havi ng frequent headaches t hat are i nt erferi ng wi t h her l i fes t yl e, i t i s reas onabl e t o begi n t reat ment wi t h a prophyl act i c mi grai ne medi cat i on. Bacl ofen i s not effect i ve for mi grai ne. Indomet haci n i s not commonl y pres cri bed for mi grai ne; t hough i t may benefi t an acut e headache epi s ode, i t i s not cons i dered a prophyl act i c medi cat i on. Cort i cos t eroi ds can be hel pful i n t he t reat ment of a s evere, acut e mi grai ne headache. However, cort i cos t eroi ds are not commonl y pres cri bed for mi grai ne prophyl axi s . Sumat ri pt an i s an effect i ve t reat ment for an acut e mi grai ne headache but i t i s not i ndi cat ed as a prophyl act i c mi grai ne t herapy. 16. A 76-year-old woman has been experiencing left temporal headaches for the prior 3 weeks. Over the prior 2 days she has had several brief episodes of cloudy vision in her left eye. On questioning, she notes a 5-pound weight loss over the past 2 weeks, which she attributes to discomfort when chewing. T herapy should be initiated with: A. Bacl ofen B. Ami t ri pt yl i ne C. Indomet haci n D. Cort i cos t eroi ds E. Sumat ri pt an Vi ew Ans wer 16. T he answer is D [IV B 5 c]. Thi s pat i ent l i kel y has gi ant cel l (t emporal ) art eri t i s . The t reat ment of choi ce for gi ant cel l art eri t i s
Pa g e 2 8 5 0
ABC Ambe r CHM Conve rte r Tria l ve rsion, http://w w w .proce sste x t.com/a bcchm.html
i s cort i cos t eroi ds . An eryt hrocyt e s edi ment at i on rat e and C-react i ve prot ei n as s ay s houl d be obt ai ned before begi nni ng cort i cos t eroi d t herapy, but wai t i ng for a t emporal art ery bi ops y s houl d not del ay i ni t i at i on of t herapy. Nei t her bacl ofen, ami t ri pt yl ene, i ndomet haci n, or s umat ri pt an are i ndi cat ed for gi ant cel l art eri t i s . 17. A 57-year-old man has a 2-month history of severe, daily headaches that involve the right frontotemporoparietal area. T he headaches typically last 60–90 minutes and occur once or twice daily. Over-the-counter medications have not provided relief. Chiropractic manipulation did not help. T he patient is desperate for relief because he cannot work during the headache episodes. A reasonable treatment strategy is to initiate: A. Bacl ofen B. Ami t ri pt yl i ne C. Indomet haci n D. Cort i cos t eroi ds E. Sumat ri pt an Vi ew Ans wer 17. T he answer is C [IV B 8]. The pat i ent 's hi s t ory i s s ugges t i ve of paroxys mal hemi crani a, an i ndomet haci n-res pons i ve di s order. In fact , a pos i t i ve res pons e t o t reat ment wi t h i ndomet haci n can confi rm a di agnos i s of paroxys mal hemi crani a. Bacl ofen, ami t ri pt yl ene, cort i cos t eroi ds , and s umat ri pt an are not cons i dered benefi ci al for paroxys mal hemi crani a. 18. A 53-year-old woman with the sudden onset of the worst headache of her life presents to the emergency department. She has a history of migraine but states that the current headache is not like her P.566 usual headaches. Results of her physical examination are unremarkable. T he initial preferred diagnostic test is a(an): A. Angi ogram
Pa g e 2 8 5 1
ABC Ambe r CHM Conve rte r Tria l ve rsion, http://w w w .proce sste x t.com/a bcchm.html
B. CT s can C. TCD D. MRI E. MR angi ogram Vi ew Ans wer 18. T he answer is B [VII C 1 b (2); IV A 3 b]. The hi s t ory i s hi ghl y s ugges t i ve of s ubarachnoi d hemorrhage (SAH). A brai n comput ed t omography (CT) s can i s t he bes t t es t t o s creen for i nt racrani al hemorrhage and s houl d be emergent l y obt ai ned t o document s ubarachnoi d bl ood. If t he s can i s normal and t he hi s t ory i s s ugges t i ve of SAH, t he woman s houl d undergo a l umbar punct ure (LP) t o s creen for bl ood before t he phys i ci an concl udes t hat s he does not have a SAH. Convent i onal angi ography i s neces s ary t o eval uat e pat i ent s wi t h SAH t o defi ne t he pres ence of an aneurys m and det ai l s of i t s anat omi c confi gurat i on. As s oci at ed abnormal i t i es s uch as vas os pas m can al s o be as s es s ed. However, an angi ogram s houl d not be t he i ni t i al di agnos t i c t es t i n a pat i ent wi t h a s us pect ed SAH. Trans crani al Doppl er (TCD) s t udi es can det ect cerebral art ery vas os pas m but not t he pres ence of an aneurys m. Vas os pas m t ypi cal l y devel ops a few days aft er SAH; t herefore, i t woul d not be expect ed t o be apparent at t he t i me of pat i ent pres ent at i on. Brai n MRI i s not as s ens i t i ve as brai n CT for t he det ect i on of acut e i nt racrani al hemorrhage. Therefore, brain MRI i s not cons i dered t he procedure of choi ce for t he acut e det ect i on of SAH. Al t hough magnet i c res onance (or comput ed t omography) angi ography (MRA or CTA, res pect i vel y) can document t he pres ence of an aneurys m, t hey are not as s ens i t i ve as convent i onal angi ography. Nei t her MRA nor CTA i s adequat e t o det ermi ne t he charact eri s t i cs of t he aneurys m ful l y and t o di rect t herapy. Therefore, MRA and CTA are not t he fi rs t t es t s t o be obt ai ned i n pat i ent s wi t h a s us pect ed SAH. 19. A 78-year-old man with a history of type 1 (insulin-dependent) diabetes mellitus, hypertension, hypercholesterolemia, and remote smoking presents with the
Pa g e 2 8 5 2
ABC Ambe r CHM Conve rte r Tria l ve rsion, http://w w w .proce sste x t.com/a bcchm.html
acute onset of aphasia. On examination, he has trouble expressing himself and has mild right facial and arm weakness. T here is a left carotid bruit. T he most definitive strategy for secondary stroke prevention will likely be: A. Acut e hepari n admi ni s t rat i on fol l owed by warfari n B. Ext racrani al i nt racrani al bypas s s urgery C. W arfari n and as pi ri n combi ned t herapy D. Carot i d endart erect omy E. As pi ri n monot herapy Vi ew Ans wer 19. T he answer is D [VIII B 4 c (1) (a), (3) (a)]. The pat i ent has a nondi s abl i ng s t roke. Hi s ri s k fact or profi l e and t he pres ence of an i ps i l at eral brui t s ugges t t hat t he mos t l i kel y caus e of hi s s t roke i s l eft i nt ernal carot i d art ery ori gi n s t enos i s . The defi ni t i ve i nt ervent i on t o prevent s ubs equent s t roke i s carot i d endart erect omy. Acut e hepari n admi ni s t rat i on and s ubs equent warfari n t herapy have not been demons t rat ed t o be effect i ve for s econdary s t roke prevent i on i n t hi s s et t i ng. Li kewi s e, t here i s no rol e for ext racrani al -i nt racrani al bypas s s urgery t o t reat ext racrani al i nt ernal carot i d art ery ori gi n s t enos i s . A combi nat i on of warfari n and as pi ri n has not been s hown t o be more effect i ve t han carot i d endart erect omy for s econdary s t roke prevent i on for a pat i ent s uch as t hi s . Al t hough as pi ri n i s appropri at e t herapy for s econdary s t roke prevent i on for s t roke due t o art eri al di s eas e, t reat ment wi t h as pi ri n s houl d not precl ude carot i d endart erect omy i n t he appropri at e s et t i ng. As pi ri n s houl d be admi ni s t ered aft er carot i d endart erect omy t o decreas e t he ri s k of pos t operat i ve s t roke. 20. A 32-year-old woman who is 1 week postpartum presents with progressive headache and confusion. On examination she is afebrile with a blood pressure of 110/65 mm Hg. Papilledema is noted. T he most likely diagnosis is: A. Ps eudot umor cerebri B. Pi t ui t ary apopl exy C. Bact eri al meni ngi t i s
Pa g e 2 8 5 3
ABC Ambe r CHM Conve rte r Tria l ve rsion, http://w w w .proce sste x t.com/a bcchm.html
D. Sagi t t al s i nus t hrombos i s E. Ecl amps i a Vi ew Ans wer 20. T he answer is D [IV B 6]. The pat i ent 's pres ent at i on s ugges t s s agi t t al s i nus t hrombos i s . Al t hough t hi s condi t i on i s as s oci at ed wi t h t he hypercoagul abl e s t at e of pregnancy, furt her t es t i ng i s i ndi cat ed t o eval uat e t he pos s i bi l i t y of an underl yi ng chroni c hypercoagul abl e condi t i on. Ps eudot umor cerebri (i di opat hi c i nt racrani al hypert ens i on) can pres ent i n as s oci at i on wi t h pregnancy, and al t hough i t i s charact eri zed by headache and papi l l edema, confus i on i s not a part of ps eudot umor cerebri . Pi t ui t ary apopl exy can occur pos t part um. It i s charact eri zed by headache and vi s ual dys funct i on. Papi l l edema and confus i on are not common s equel ae of pi t ui t ary apopl exy. Al t hough bact eri al meni ngi t i s caus es headache, confus i on, and papi l l edema, pat i ent s are t ypi cal l y febri l e. Ecl amps i a can caus e headache and confus i on as wel l as papi l l edema. However, t he pat i ent 's l ow bl ood pres s ure i s evi dence agai ns t t hi s di agnos i s . Al s o, t here i s no ment i on of prot ei nuri a or s ei z ures . 21. A 67-year-old woman with a history of hypertension, diabetes mellitus, and smoking presents to the emergency department at 8:30 AM with a mild expressive aphasia, right facial weakness, and mild right arm weakness. She had awakened at 7:00 AM and was speaking to her husband when her speech suddenly became difficult and weakness was noted. Her husband called 911, and she was transported to the hospital, where her blood pressure was 165/85 mm Hg. T he preferred treatment is: A. As pi ri n B. Hepari n C. W arfai n D. rt -PA E. Cl opi dogrel
Pa g e 2 8 5 4
ABC Ambe r CHM Conve rte r Tria l ve rsion, http://w w w .proce sste x t.com/a bcchm.html
Vi ew Ans wer 21. T he answer is D [VIII B 1 c (1)]. The pat i ent has had t he acut e ons et of a neurol ogi c defi ci t cons i s t ent wi t h a s t roke. She i s pres ent i ng for medi cal care wi t hi n 3 hours of s ympt om ons et . No cont rai ndi cat i ons t o i nt ravenous t hrombol yt i c t herapy wi t h rt -PA i s ment i oned. The pat i ent s houl d have an emergent brai n CT s can and i f no al t ernat e di agnos es are s ugges t ed (t umor, s ubdural hemat oma, et c.), i nt ravenous rt -PA s houl d be admi ni s t ered. As pi ri n i s s t andard of care i f t he pat i ent had pres ent ed more t han 3 hours aft er s ympt om ons et . There i s l i t t l e evi dence t hat hepari n or warfari n t herapy i s benefi ci al i n t hi s s et t i ng, and t here i s an i ncreas ed i nci dence of bl eedi ng wi t h t hes e ant i coagul ant s . Cl opi dogrel i s a t herapeut i c al t ernat i ve for s econdary s t roke prevent i on, but i t s us e s houl d not t ake precedence over t he admi ni s t rat i on of rt -PA. If rt -PA i s admi ni s t ered, as pi ri n admi ni s t rat i on s houl d be del ayed 24 hours . Al s o, i f l ow-dos e hepari n i s admi ni s t ered for deep venous t hrombos i s prophyl axi s , i t s us e s houl d al s o be del ayed for 24 hours before rt -PA us e. 22. A 62-year-old woman has a 2-month history of mild confusion. She occasionally has difficulty following a conversation. On the morning of presentation to the hospital, she developed 30 seconds of right face and arm twitching, followed by increased confusion. Examination shows difficulties in speech comprehension and a subtle right homonymous hemianopia. A brain CT scan shows a 2 to 3-cm ill-defined region of low density in the left parietal lobe. T he most likely diagnosis is: A. Mul t i pl e s cl eros i s B. St roke C. As t rocyt oma D. Abs ces s E. Met as t as i s Vi ew Ans wer 22. T he answer is C [XVII A 1 a]. The pat i ent pres ent s wi t h a
Pa g e 2 8 5 5
ABC Ambe r CHM Conve rte r Tria l ve rsion, http://w w w .proce sste x t.com/a bcchm.html
2-mont h hi s t ory of progres s i ve neurol ogi c defi ci t s . The CT s can s hows an area of l ow dens i t y, and t here i s no ment i on of mas s effect . The mos t l i kel y di agnos i s i s an as t rocyt oma, whi ch may not have much i n t he way of mas s effect or enhancement on CT s can. The progres s i ve hi s t ory i s agai ns t a s t roke. Mul t i pl e s cl eros i s rarel y pres ent s i n t hi s age group and woul d not be expect ed t o caus e a l arge area of decreas ed dens i t y on CT s can. An abs ces s or met as t as i s woul d have s ubs t ant i al mas s effect and enhancement on CT s can.
Pa g e 2 8 5 6
ABC Amber CHM Converter Trial version, http://www.processtext.com/abcchm.html
Editors: Wolfsthal, Susan T itle: NMS Medicine, 6th Edition Copyri ght ©2008 Li ppi ncot t W i l l i ams & W i l ki ns > T able of Cont ent s > Chapt er 12 - Dermat ologic Disorders
Chapter 12
Dermatologic Disorders Jeffrey Liu Jennifer Cooper
I. Structure and Function of Skin A. Structure The s ki n covers t he s ubcut aneous fat (adi pos e or panni cul us ) and cons i s t s of t wo l ayers : t he epi dermi s and t he dermi s .
1. Epidermis. The epi dermi s i s a s t rat i fi ed, s quamous epi t hel i um t hat i s 0.1 mm t hi ck. It produces kerat i n i nt ermedi at e fi l ament s and cont i nual l y di fferent i at es i nt o t he corni fi ed (horny) l ayer t hat provi des t he barri er funct i on t o t he s ki n. It renews i t s el f every 28 days and gi ves ri s e t o t he fol l owi ng s ki n appendages : hai r fol l i cl es , s ebaceous gl ands , s weat gl ands , and nai l s . o
a. Compartments
(onl i ne Fi gure
12-1)
Page 2857
ABC Ambe r CHM Conve rte r Tria l ve rsion, http://w w w .proce sste x t.com/a bcchm.html o
ONLINE FIGURE 12-1 Compart ment s of t he epi dermi s .
(1) Basal layer or stratum germinativum i s t he l ayer of undi fferent i at ed cel l s t hat cont act s t he bas ement membrane zone (BMZ).
(2) Spinous layer or stratum spinosum i s t he l ayer i n whi ch kerat i n fi l ament format i on occurs .
(3) Granular layer or stratum granulosum i s t he l ayer i n whi ch kerat i n fi l ament pai ri ng and as s embl y occurs .
(4) Cornified layer or stratum corneum, t he
Pa g e 2 8 5 8
ABC Ambe r CHM Conve rte r Tria l ve rsion, http://w w w .proce sste x t.com/a bcchm.html
fi nal di fferent i at i on product of t he epi dermi s , i s t he barri er l ayer. o
o
b. Cells
(1) Keratinocytes are t he epi t hel i al cel l s of t he epi dermi s t hat undergo t ermi nal di fferent i at i on.
(2) Melanocytes are neural cres t –deri ved cel l s t hat produce pi gment (mel ani n). They are di s t ri but ed al ong t he bas al cel l l ayer and t rans fer pi gment t o bas al kerat i nocyt es t o prot ect agai ns t ul t ravi ol et (UV) i rradi at i on.
(3) Langerhans cells are bone marrow–deri ved, dendri t i c, ant i gen-pres ent i ng cel l s di s t ri but ed wi t hi n t he epi dermi s t hat funct i on i n i mmune s urvei l l ance of t he s ki n.
o
o
c. Epidermal proteins
(1) Products of differentiation
(a) Keratins are i nt ermedi at e fi l ament prot ei ns and are cl as s i fi ed i nt o t wo t ypes , accordi ng t o mol ecul ar wei ght and charge. Type 1 kerat i ns pai r wi t h t ype 2 kerat i ns t o form a t i s s ue-s peci fi c kerat i n
Pa g e 2 8 5 9
ABC Ambe r CHM Conve rte r Tria l ve rsion, http://w w w .proce sste x t.com/a bcchm.html
fi l ament .
(b) Filaggrin i s a kerat i n-as s oci at ed prot ei n t hat forms a mat ri x for kerat i n fi l ament s .
(c) The terminal cell envelope encas es kerat i n fi l ament s . Toget her wi t h lipids, t he epi dermal prot ei ns provi de t he barri er funct i on of t he epi dermi s .
(2) Adhesion proteins
(onl i ne Fi gure 12-2)
online Figure 12-2 Adhes i on mol ecul es of t he epi dermi s .
(a) Desmosomes are mul t i prot ei n compl exes t hat s erve as poi nt s of at t achment bet ween kerat i nocyt es . They cont ai n i nt racel l ul ar and i nt ercel l ul ar
Pa g e 2 8 6 0
ABC Ambe r CHM Conve rte r Tria l ve rsion, http://w w w .proce sste x t.com/a bcchm.html
prot ei ns (des mogl ei ns , des mocol l i ns , and pl aki ns ).
(b) Hemidesmosomes are mul t i prot ei n compl exes t hat are l ocat ed on t he i nferi or s i de of bas al l ayer kerat i nocyt es and s erve as poi nt s of at t achment bet ween kerat i nocyt es and t he BMZ. They cont ai n i nt racel l ul ar and i nt ercel l ul ar prot ei ns (des mopl aki n, col l agen, and pl ect i n).
2. Dermis. The dermi s i s t he l ayer beneat h t he epi dermi s t hat provi des pl i abi l i t y and t ens i l e s t rengt h. It i s a mat ri x of fi brous and amorphous connect i ve t i s s ue t hat s upport s bl ood and l ymphat i c ves s el s , fi brobl as t s , s ki n appendages , and mas t cel l s . It prot ect s t he body from mechani cal i njury, bi nds wat er, ai ds i n t hermal regul at i on, and cont ai ns recept ors of s ens ory s t i mul i . o
o
a. Compartments
(1) The BMZ i s t he dermal–epidermal junction (DEJ) t hat forms t he i nt erface bet ween t he epi dermi s and t he dermi s .
(a) It cons i s t s of t wo l ayers : t he lamina lucida, whi ch l i es di rect l y beneat h t he bas al l ayer, and t he lamina densa, whi ch l i es di rect l y beneat h t he l ami na l uci da.
Pa g e 2 8 6 1
ABC Ambe r CHM Conve rte r Tria l ve rsion, http://w w w .proce sste x t.com/a bcchm.html
(b) It funct i ons t o anchor t he epi dermi s t o t he dermi s and provi de res i s t ance agai ns t ext ernal s heari ng forces .
(c) It s erves as a s emi penet rabl e barri er and i nfl uences t he pol ari t y of bas al cel l growt h and cyt os kel et on organi zat i on.
(2) The papillary dermis i s t he s uperfi ci al port i on of t he dermi s t hat l i es di rect l y bel ow t he BMZ.
(3) The reticular dermis i s t he deeper port i on of t he dermi s
o
o
b. Cells
(1) Endothelial cells are t he cel l s t hat form cut aneous bl ood ves s el s and l ymphat i c channel s .
(2) Fibroblasts s ecret e macromol ecul es and t he mat ri x component s of t he dermi s .
(3) Mast cells are bone marrow–deri ved cel l s t hat cont ai n preformed i nfl ammat ory medi at ors (e.g., hi s t ami ne and prot eas es ) and part i ci pat e i n a vari et y of bi ol ogi c res pons es , i ncl udi ng urt i cari a and defens e agai ns t paras i t es .
Pa g e 2 8 6 2
ABC Ambe r CHM Conve rte r Tria l ve rsion, http://w w w .proce sste x t.com/a bcchm.html o
o
c. Matrix
(1) Collagen i s a macromol ecul e t hat gi ves t ens i l e s t rengt h t o t he s ki n.
(2) Elastin i s a macromol ecul e t hat gi ves el as t i ci t y t o t he s ki n.
(3) Ground substance forms t he dermal mat ri x and cons i s t s of compl ex s ugars ( mucopolysaccharides) and prot ei ns ( proteoglycans).
o
o
d. Nerves and nerve receptors. Thes e s t ruct ures provi de s ens ory i nput t hroughout t he ent i re s ki n s urface.
3. Panniculus (adi pos e). The panni cul us i s t he s ubcut aneous fat l ayer t hat provi des t hermal i ns ul at i on. o
o
a. Hair follicles. Thes e epi dermal appendages ext end i nt o t he panni cul us .
o
o
b. Sebaceous glands. Thes e gl ands empt y i nt o hai r fol l i cl es , provi di ng l ubri cat i on t o t he horny l ayer.
o
Pa g e 2 8 6 3
ABC Ambe r CHM Conve rte r Tria l ve rsion, http://w w w .proce sste x t.com/a bcchm.html o
c. Sweat glands. The gl andul ar port i on i nvol ved i n s weat product i on ext ends i nt o t he panni cul us , connect i ng t o t he s ki n s urface t hrough epi t hel i um-l i ned duct s . Thes e gl ands part i ci pat e i n t hermoregul at i on and fl ui d and el ect rol yt e bal ance.
B. Functions
1. Barrier. The s t rat um corneum prot ect s agai ns t wat er l os s , ent rance of mi croorgani s ms , and envi ronment al t oxi ns . The deeper l ayers of t he s ki n prot ect from damage caus ed by UV i rradi at i on, mechani cal forces , and ext reme envi ronment al t emperat ures .
2. Fluid and electrolyte balance. Aut onomi c nerves i nnervat e s weat gl ands t hat s t i mul at e s weat product i on and as s i s t i n wat er and el ect rol yt e regul at i on.
3. T hermoregulation. The s ki n s erves as an out er s hel l t hat vari es bl ood fl ow and s weat s ecret i on t o regul at e heat exchange wi t h t he envi ronment t o mai nt ai n t he body's core t emperat ure.
4. Vitamin D metabolism. The s ki n i s a major s ource of nondi et ary vi t ami n D. UV i rradi at i on from s un expos ure convert s 7-dehydrochol es t erol t o vi t ami n D 3 (chol ecal ci ferol ), whi ch i s t hen hydroxyl at ed i n t he l i ver and t he ki dney t o form act i ve vi t ami n D (1,25-di hydroxychol ecal ci ferol ).
5. Sensory input. A neural net work provi des s ens ory i nput for t ouch, pai n, t emperat ure, i t ch, and mechani cal
Pa g e 2 8 6 4
ABC Ambe r CHM Conve rte r Tria l ve rsion, http://w w w .proce sste x t.com/a bcchm.html
s t i mul i .
6. Immune surveillance. The s ki n i s an i mport ant component of t he i mmune s ys t em and part i ci pat es i n bot h i nnat e and acqui red i mmuni t y. It s res i dent i mmune cel l s (Langerhans cel l s ); endogenous cyt oki nes (i nt erl euki n-1 [IL-1], IL-3, and IL-6); and growt h fact ors [granul ocyt e col ony s t i mul at i ng fact or (G-CSF), granul ocyt e macrophage col ony-s t i mul at i ng fact or (GM-CSF), macrophage colony-s t i mul at i ng fact or (M-CSF), and t umor necros i s fact or-α [TNF-α]) i nt eract wi t h ci rcul at i ng l ymphocyt es and macrophages . Thes e i mmune fact ors part i ci pat e i n a wi de range of phys i ol ogi c and pat hol ogi c proces s es .
II. Dermatologic Diagnosis A. General considerations The foundat i on of dermat ol ogi c di agnos i s rel i es on phys i cal exami nat i on of t he s ki n t o fi rs t i dent i fy t he morphol ogy of t he pri mary s ki n l es i on(s ). Bas ed on morphol ogy, a di fferent i al di agnos i s i s generat ed, and a fi nal di agnos i s i s det ermi ned by us i ng t he dermat ol ogi c hi s t ory and appropri at e l aborat ory s t udi es .
B. Examination of the skin
1. Inspection of t he s ki n defi nes morphol ogy, col or, di s t ri but i on, and confi gurat i on. o
o
a. Morphology. Pri mary l es i ons of t he s ki n are cl as s i fi ed bas ed on morphol ogy (Tabl e 12-1).
o
o
b. Distribution. The l ocat i on and ext ent of t he
Pa g e 2 8 6 5
ABC Ambe r CHM Conve rte r Tria l ve rsion, http://w w w .proce sste x t.com/a bcchm.html
pri mary erupt i on are i mport ant charact eri s t i cs i n defi ni ng t he di s eas e proces s .
(1) Local i zed: i nvol vement of l i mi t ed areas of t he s ki n
(2) Di ffus e: wi des pread i nvol vement of t he s ki n
(3) Bi l at eral : s ymmet ri cal i nvol vement
(4) Dermat omal : i nvol vement l i mi t ed t o s ki n over a dermat ome (e.g., herpes zos t er)
(5) Acral : i nvol vement of t he s ki n of t he di s t al ext remi t i es
o
c. Configuration
(Onl i ne Fi gures 12-3 t hrough 12-6). Several
feat ures of pri mary l es i ons are i mport ant i n defi ni ng t he di s eas e proces s . o
ONLINE FIGURE 12-3 Pri mary l es i on confi gurat i on: annul ar. (Court es y of St uart Les s i n, MD.)
Pa g e 2 8 6 6
ABC Ambe r CHM Conve rte r Tria l ve rsion, http://w w w .proce sste x t.com/a bcchm.html
ONLINE FIGURE 12-4 Pri mary l es i on confi gurat i on: l i near. (Court es y of St uart Les s i n, MD.) o
ONLINE FIGURE 12-5 Pri mary l es i on confi gurat i on: s erpi gi nous . (Court es y of St uart Les s i n, MD.)
Pa g e 2 8 6 7
ABC Ambe r CHM Conve rte r Tria l ve rsion, http://w w w .proce sste x t.com/a bcchm.html
ONLINE FIGURE 12-6 Pri mary l es i on confi gurat i on: pol ycycl i c. (Court es y of St uart Les s i n, MD.)
(1) Number (s i ngl e vers us mul t i pl e)
(2) Spat i al rel at i ons hi p (i .e., grouped)
(3) Shape (i .e., annul ar, l i near, s erpi gi nous , and pol ycycl i c)
2. Palpation det ermi nes t he t ext ure, cons i s t ency, and dept h of l es i ons , as wel l as t he t emperat ure, t endernes s , and qual i t y of eryt hema (i .e., bl anchabl e).
C. Dermatologic history
1. History of the eruption. The es s ent i al hi s t ori cal qual i fi ers for any cut aneous erupt i on i ncl ude t he fol l owi ng: o
o
a. Ons et and progres s i on
o
o
b. Durat i on and cours e
o
o
c. Sympt oms
o
o
d. Treat ment and res pons e
Pa g e 2 8 6 8
ABC Ambe r CHM Conve rte r Tria l ve rsion, http://w w w .proce sste x t.com/a bcchm.html
2. Medical history. A hi s t ory of medi cal probl ems focus ed on cut aneous fi ndi ngs , i ncl udi ng al l ergi es , s houl d be el i ci t ed.
3. Medication history. A det ai l ed medi cat i on hi s t ory i s vi t al , es peci al l y when eval uat i ng erupt i ons t hat may be t ri ggered by medi cat i ons . The medi cat i on hi s t ory s houl d i ncl ude pres cri pt i on and over-t he-count er (OTC) medi cat i ons , vi t ami ns , eye drops , and herbal s uppl ement s . P.573
P.574
TABLE 12-1 Terms Used in the Morphologic Description of Skin Lesions D e sc ri pt T e io rm n Appearance Pa S pul m e
al l, s
Pa g e 2 8 6 9
ABC Ambe r CHM Conve rte r Tria l ve rsion, http://w w w .proce sste x t.com/a bcchm.html
ol id , el e v at e d le si o n. P a p ul e s ar e g e n er al ly s m al le r th a n
Pa g e 2 8 7 0
ABC Ambe r CHM Conve rte r Tria l ve rsion, http://w w w .proce sste x t.com/a bcchm.html
1 c m in di a m et er Pla Br qu o e
a db a s e d p a p ul e th at oc cu pi e s a re la ti v
Pa g e 2 8 7 1
ABC Ambe r CHM Conve rte r Tria l ve rsion, http://w w w .proce sste x t.com/a bcchm.html
el y la rg e s ur fa ce ar e a in co m p ar is o n wi th it s h ei g ht a b o v e sk in
Pa g e 2 8 7 2
ABC Ambe r CHM Conve rte r Tria l ve rsion, http://w w w .proce sste x t.com/a bcchm.html
le v el > 1 c m Ma Fl cul at e
le si o n of v ar ia bl e si z e a n d s h a p e th at di ff er
Pa g e 2 8 7 3
ABC Ambe r CHM Conve rte r Tria l ve rsion, http://w w w .proce sste x t.com/a bcchm.html
s fr o m s ur ro u n di n g sk in b ec a u s e of it s co lo r, < 1 c m Pa Des cri pt i on of tch very l arge macul es or a t hi n but l arge pl aque >1 cm
Pa g e 2 8 7 4
ABC Ambe r CHM Conve rte r Tria l ve rsion, http://w w w .proce sste x t.com/a bcchm.html
No P dul al e
p a bl e, fi r m , ro u n d to s p h er oi d le si o n wi th a d e pt h of in v ol
Pa g e 2 8 7 5
ABC Ambe r CHM Conve rte r Tria l ve rsion, http://w w w .proce sste x t.com/a bcchm.html
v e m e nt gr e at er th a n th at of a p a p ul e Cy Epi t hel i al -l i ned st s ac t hat cont ai ns l i qui d or s emi s ol i d mat eri al (fl ui d, cel l s , and cel l product s ) Ve Fl sicl ui e
dfi l le d, el
Pa g e 2 8 7 6
ABC Ambe r CHM Conve rte r Tria l ve rsion, http://w w w .proce sste x t.com/a bcchm.html
e v at e d le si o n < 1 c m in di a m et er Bul Large ves i cl e la (>1 cm) Pu R stu ai le
s e d le si o n th at co nt ai
Pa g e 2 8 7 7
ABC Ambe r CHM Conve rte r Tria l ve rsion, http://w w w .proce sste x t.com/a bcchm.html
n s p ur ul e nt e x u d at e (i .e ., p u s) Wh Fl eal at -t o p p e d p a p ul e or pl a
Pa g e 2 8 7 8
ABC Ambe r CHM Conve rte r Tria l ve rsion, http://w w w .proce sste x t.com/a bcchm.html
q u e th at is ch ar ac te ri st ic al ly e v a n e sc e nt , di s a p p e ar in g wi th
Pa g e 2 8 7 9
ABC Ambe r CHM Conve rte r Tria l ve rsion, http://w w w .proce sste x t.com/a bcchm.html
in h o ur s (i .e ., hi v e) Er Ci rcums cri bed, osi s uperfi ci al on depres s i on res ul t i ng from l os s of al l or port i on of vi abl e epi dermi s Ulc Deeper er depres s i on res ul t i ng from des t ruct i on of epi dermi s and at l eas t upper (papi l l ary) dermi s Fis Li sur n e
e ar cr ac k in sk
Pa g e 2 8 8 0
ABC Ambe r CHM Conve rte r Tria l ve rsion, http://w w w .proce sste x t.com/a bcchm.html
in Fur Deep un necrot i zi ng cle form of fol l i cul i t i s wi t h pus accumul at i on. Several furuncl es may coal es ce t o form carbuncl e Ab Local i zed sce accumul at i on ss of purul ent mat eri al s o deep i n dermi s or s ubcut aneous t i s s ue t hat pus i s us ual l y not vi s i bl e on s urface of s ki n Sin Tract l eadi ng us from s uppurat i ve cavi t y t o s ki n s urface or bet ween cys t i c or abs ces s cavi t i es Atr E op pi hy d er m al
Pa g e 2 8 8 1
ABC Ambe r CHM Conve rte r Tria l ve rsion, http://w w w .proce sste x t.com/a bcchm.html
at ro p h y: th in ni n g of e pi d er m is a ss oc ia te d wi th d ec re a s e in n u m
Pa g e 2 8 8 2
ABC Ambe r CHM Conve rte r Tria l ve rsion, http://w w w .proce sste x t.com/a bcchm.html
b er of e pi d er m al ce ll s D er m al at ro p h y: u s u al ly m a ni fe st e d a s
Pa g e 2 8 8 3
ABC Ambe r CHM Conve rte r Tria l ve rsion, http://w w w .proce sste x t.com/a bcchm.html
d e pr e ss io n of sk in ; re s ul ts fr o m d ec re a s e in d er m al co n n ec ti v
Pa g e 2 8 8 4
ABC Ambe r CHM Conve rte r Tria l ve rsion, http://w w w .proce sste x t.com/a bcchm.html
e ti ss u e Scl Ci rcums cri bed er or di ffus e osi hardeni ng or s
i ndurat i on i n
s ki n Sc A ali b ng n or m al s h e d di n g or ac cu m ul at io n of st ra tu
Pa g e 2 8 8 5
ABC Ambe r CHM Conve rte r Tria l ve rsion, http://w w w .proce sste x t.com/a bcchm.html
m co rn e u m in p er ce pt ib le fl a k e s Pa Erupt i ons pul cons i s t i ng of os s cal i ng papul es qu am ou s Cr H ust ar s
d e n e d d e p
Pa g e 2 8 8 6
ABC Ambe r CHM Conve rte r Tria l ve rsion, http://w w w .proce sste x t.com/a bcchm.html
o si ts of dr ie d s er u m , bl o o d, or p ur ul e nt e x u d at e Ex S cor u iati p on er s
fi ci al
Pa g e 2 8 8 7
ABC Ambe r CHM Conve rte r Tria l ve rsion, http://w w w .proce sste x t.com/a bcchm.html
e xc a v at io n s of e pi d er m is th at m a y b e li n e ar or p u nc ta te a n d
Pa g e 2 8 8 8
ABC Ambe r CHM Conve rte r Tria l ve rsion, http://w w w .proce sste x t.com/a bcchm.html
re s ul t fr o m sc ra tc hi n g Lic T he hi nifi ck cat e ion ni n g of th e e pi d er m is re s ul ti n g
Pa g e 2 8 8 9
ABC Ambe r CHM Conve rte r Tria l ve rsion, http://w w w .proce sste x t.com/a bcchm.html
fr o m ch ro ni c sc ra tc hi n g Poi Combi nat i on of kil at rophy, od t el angi ect as i a, er and pi gment ary ma changes (hyperpi gment a t i on and hypopi gment at i on)
4. Social history. A det ai l ed s oci al hi s t ory oft en provi des di agnos t i c cl ues for cut aneous fi ndi ngs . The s oci al hi s t ory s houl d focus on s ki n care (habi t s and product s us ed), occupat i on, home envi ronment , s exual l y t rans mi t t ed di s eas es , and t ravel .
5. Family history. A fami l y hi s t ory i s i mport ant i n as s es s i ng s ki n di s eas e predi s pos i t i on and s us cept i bi l i t y, part i cul arl y for al l ergy, s ki n cancer, ps ori as i s , and rheumat i c di s eas e.
Pa g e 2 8 9 0
ABC Ambe r CHM Conve rte r Tria l ve rsion, http://w w w .proce sste x t.com/a bcchm.html
D. Dermatology laboratory tests
1. Direct microscopic examination. Preparat i ons of s ki n s crapi ngs from s cal i ng and bl i s t eri ng erupt i ons can provi de i mport ant di agnos t i c i nformat i on. P.575
o
o
a. Potassium hydroxide (KOH) preparation. KOH di ges t s cel l ul ar mat eri al and faci l i t at es t he vi s ual i zat i on of fungal hyphae, yeas t forms , and s cabi es mi t es .
o
o
b. T zanck smear. Gi ems a- or W ri ght -s t ai ned s crapi ngs from herpet i c ves i cl es reveal mul t i nucl eat ed gi ant cel l s .
o
o
c. Gram stain. Gram-pos i t i ve and gram-negat i ve bact eri a may be i dent i fi ed.
o
o
d. Darkfield microscopy. Vi s ual i zat i on of s pi rochet es (T reponema pal l i dum) from preparat i ons of pri mary chancre and s econdary l es i ons i s di agnos t i c of s yphi l i s .
2. Skin cultures. Bact eri al , vi ral , and fungal cul t ures can be obt ai ned from s ki n s wabs or s ki n bi ops i es .
Pa g e 2 8 9 1
ABC Ambe r CHM Conve rte r Tria l ve rsion, http://w w w .proce sste x t.com/a bcchm.html
3. Patch testing. Thi s met hod i s us ed for i dent i fyi ng compounds res pons i bl e for cont act dermat i t i s . Common and s us pect ed compounds are pl aced on t he s ki n under occl us i on (i .e., pat ch) for 48–72 hours . Induct i on of cut aneous i nfl ammat i on at s peci fi c pat ch s i t es i dent i fi es s ens i t i zi ng agent s .
4. Phototesting and photopatch testing o
o
a. Phototesting us es s peci al i zed l i ght s ources t hat del i ver s peci fi c ranges and quant i t i es of UV l i ght t o document phot os ens i t i vi t y i n i ndi vi dual s wi t h s us pect ed phot os ens i t i vi t y.
o
o
b. Photopatch testing combi nes pat ch t es t i ng and phot ot es t i ng t o i dent i fy phot os ens i t i zi ng compounds , whi ch produce a phot os ens i t i vi t y react i on when expos ed t o UV l i ght . Sus pect ed phot os ens i t i zi ng compounds are pat ch t es t ed wi t h and wi t hout UV expos ure.
5. Skin biopsy provi des t i s s ue for hi s t opat hol ogi c eval uat i on. o
o
a. Biopsy techniques
(1) Punch biopsy i s a commonl y us ed t echni que t o obt ai n 2–6-mm cyl i ndri cal s ki n s ampl es .
Pa g e 2 8 9 2
ABC Ambe r CHM Conve rte r Tria l ve rsion, http://w w w .proce sste x t.com/a bcchm.html
(2) T angential (shave) biopsy i s a rapi d t echni que us ed t o obt ai n s uperfi ci al s ki n s ampl es .
(3) Incisional biopsy i s us ed t o obt ai n l arge or deep (i ncl udi ng panni cul us ) s ki n s ampl es .
(4) Excisional biopsy i s us ed t o obt ai n an ent i re l es i on (e.g., s us pi ci ous pi gment ed l es i on) for hi s t ol ogi c revi ew.
o
o
b. Examination of biopsy tissue
(1) Light microscopy i s us ed t o eval uat e rout i ne (hemat oxyl i n and eos i n) and s peci al (e.g., peri odi c aci d–Schi ff) s t ai ns as wel l as i mmunohi s t ochemi cal (monocl onal ant i body) s t ai ns .
(2) Immunofluorescent microscopy i s us ed for i dent i fi cat i on of i mmunoreact ant s (e.g., aut oi mmune ant i bodi es ).
(3) Electron microscopy i s us ed for s peci al i zed s t udi es t hat requi re vi s ual i zat i on of s ubcel l ul ar s t ruct ures .
III. Dermatologic Therapy A. General considerations Topi cal t herapi es are frequent l y us ed i n t reat i ng dermat ol ogi c
Pa g e 2 8 9 3
ABC Ambe r CHM Conve rte r Tria l ve rsion, http://w w w .proce sste x t.com/a bcchm.html
di s eas es and oft en provi de al t ernat i ves t o s ys t emi c t herapi es . The chi ef advant ages of t opi cal t herapi es may i ncl ude hi gher effi cacy and l i mi t ed s ys t emi c t oxi ci t i es , whereas t he s el ect i on of t opi cal t herapi es and s ys t emi c t herapi es for dermat ol ogi c di s eas es i s gui ded by bas i c cl i ni cal management pri nci pl es .
1. Therapeut i c goal s s houl d be defi ned. o
o
a. Is t herapy i nt ended t o cure or amel i orat e (cont rol ) t he di s eas e?
o
o
b. Is t herapy i nt ended t o be s hort -t erm or l ong-t erm?
2. Therapy s houl d maxi mi ze t he benefi t -t o-ri s k rat i o and mi ni mi ze t oxi ci t y.
3. Pat i ent compl i ance s houl d be cons i dered and promot ed.
4. Cos t -effect i venes s s houl d be eval uat ed as part of t herapeut i c deci s i on maki ng.
B. Topical therapy
1. Two vari abl es are i nvol ved i n t he s el ect i on of t opi cal t herapy. Bot h t he medi cat i on and t he vehi cl e t o be us ed mus t be appropri at e for t he s peci fi c di s order bei ng t reat ed.
Pa g e 2 8 9 4
ABC Ambe r CHM Conve rte r Tria l ve rsion, http://w w w .proce sste x t.com/a bcchm.html
2. General l y, acut e i nfl ammat i on i s t reat ed wi t h aqueous dryi ng vehi cl es , and chroni c i nfl ammat i on i s t reat ed wi t h more l ubri cat i ng and moi s t uri zi ng preparat i ons (Fi gure 12-7).
3. Vehi cl e choi ce may be i nfl uenced by body s i t e. For exampl e, l ot i ons , s ol ut i ons , and s prays are more effect i ve i n hai ry areas (e.g., s cal p).
4. W at er, oi l , and part i cul at e compos i t i on of a vehi cl e det ermi ne i t s charact eri s t i cs . P.576
FIGURE 12-7 Vehi cl e s ui t abi l i t y for t opi cal s ki n t herapy i n t erms of cl i ni cal s et t i ng. o
o
a. Creams are emul s i ons of oi l , predomi nat el y i n wat er, t hus dryi ng t he s ki n out .
o
o
b. Oi nt ment s are emul s i ons of wat er,
Pa g e 2 8 9 5
ABC Ambe r CHM Conve rte r Tria l ve rsion, http://w w w .proce sste x t.com/a bcchm.html
predomi nat el y i n oi l , t hus more moi s t uri zi ng. o
c. Gel s are s emi s ol i d emul s i ons of al cohol or
o
acet one i n an organi c pol ymer (agar, gel at i n). o
d. Lot i ons are powders i n a wat er bas e.
o o
e. Sol ut i ons , s prays , and aeros ol s have a bas e of
o
wat er, al cohol , or propyl ene gl ycol .
5. Topi cal cort i cos t eroi ds are pres cri bed for a vari et y of i nfl ammat ory and pruri t i c condi t i ons . o
a. Pot ency. Hal ogenat ed cort i cos t eroi ds have t he great es t pot ency. Pot ency of t opi cal cort i cos t eroi ds i s meas ured by vas ocons t ri ct i ve capaci t y and may be cat egori zed as l ow, medi um, or hi gh
(Onl i ne Tabl e 12-2).
o
ONLINE TABLE 12-2 Potency of Topical Steroids
Pa g e 2 8 9 6
ABC Ambe r CHM Conve rte r Tria l ve rsion, http://w w w .proce sste x t.com/a bcchm.html
Co Co nc mp ent ou rat nd ion Low poten cy Hy 0.5 dro %, cor * t i s 1% on ,*; e
2.5
ace % t at e Mid-p otency Bet 0.0 am 25 et h % as o ne be nz o at e Bet 0.1 am % et h as o ne val era te Cl o 0.1 cor %
Pa g e 2 8 9 7
ABC Ambe r CHM Conve rte r Tria l ve rsion, http://w w w .proce sste x t.com/a bcchm.html
t ol on e pi v al a te De 0.0 s on 5% i de Fl u 0.0 me 3% t ha s on e pi v al a te Fl u 0.0 oci 25 nol % on e ace t on i de Fl u 0.0 ran 5% dre nol i de Hy 0.1 dro % cor tis on e but
Pa g e 2 8 9 8
ABC Ambe r CHM Conve rte r Tria l ve rsion, http://w w w .proce sste x t.com/a bcchm.html
yra te Hy 0.2 dro % cor tis on e val era te Tri 0.1 am %, ci n 0.5 ol o % ne ace t on i de High poten cy Am 0.1 ci n % oni de Bet 0.0 am 5% et h as o ne di p rop i on at e Cl o 0.0 bet 5%
Pa g e 2 8 9 9
ABC Ambe r CHM Conve rte r Tria l ve rsion, http://w w w .proce sste x t.com/a bcchm.html
as o l pro pi o nat e De 0.0 s ox 25 im % et a s on e Di fl 0.0 ora 5% s on e di a cet at e Fl u 0.0 oci 5% no ni d e Hal 0.1 ci n % oni de Hal 0.0 ob 5% et a s ol pro pi o nat e Mo 0.1
Pa g e 2 9 0 0
ABC Ambe r CHM Conve rte r Tria l ve rsion, http://w w w .proce sste x t.com/a bcchm.html
me % t as on e fur oat e *Avai l abl e over-t he-cou nt er. o
o
b. Us e
(1) Hi gh-pot ency t opi cal cort i cos t eroi ds are us ed for t he s hort -t erm t reat ment of acut e and s evere i nfl ammat ory erupt i ons (e.g., cl obet as ol ).
(2) Mi d-t o-l ow pot ency t opi cal s t eroi ds are us ed for t reat ment of mi l d t o moderat e i nfl ammat ory erupt i ons and for mai nt enance t herapy or prol onged t herapy (e.g., t ri amci nol one).
(3) Low-pot ency t opi cal s t eroi ds are t he onl y s t eroi ds t o be pres cri bed for t he face and i nt ert ri gi nous areas (s ki n fol ds of t he axi l l ae, i nframammary regi on, abdomi nal panus , and groi n) becaus e of t he i ncreas ed ri s k of l ocal s i de effect s i n t hes e areas (e.g., hydrocort i s one).
Pa g e 2 9 0 1
ABC Ambe r CHM Conve rte r Tria l ve rsion, http://w w w .proce sste x t.com/a bcchm.html
(4) In general , prol onged us e of t opi cal s t eroi ds s houl d be avoi ded. Tachyphyl axi s and s i de effect s are as s oci at ed wi t h prol onged t opi cal s t eroi d us e. If prol onged t opi cal s t eroi d t herapy i s requi red, t aperi ng t o s t eroi ds of a l ower pot ency i s advi s ed.
(5) Occl us i on i s an i mport ant adjuvant t o t opi cal t herapy and i ncreas es abs orpt i on by i ncreas i ng hydrat i on and t emperat ure. Mat eri al s s uch as pl as t i c (Saran) wrap and gl oves and s hower caps can be us ed as occl us i ve dres s i ngs .
(6) Twi ce-dai l y appl i cat i on i s general l y recommended becaus e of pat i ent compl i ance. Indi vi dual s are more l i kel y t o t reat t hei r s ki n i n t he morni ng and eveni ng when dres s i ng and undres s i ng. No compel l i ng cl i ni cal dat a demons t rat e an i ncreas e i n effi cacy wi t h a frequency of appl i cat i on of more t han t wi ce-dai l y appl i cat i on.
(7) See
onl i ne Tabl e 12-2 for compl et e s t eroi d pot ency l i s t i ng. o
o
c. Di s pens i ng
(1) Pres cri bi ng t he appropri at e quant i t y of t opi cal s t eroi ds i s i mport ant for bot h compl i ance and s afet y.
Onl i ne Tabl e 12-3
pres ent s gui del i nes for pres cri bi ng t opi cal s t eroi ds .
Pa g e 2 9 0 2
ABC Ambe r CHM Conve rte r Tria l ve rsion, http://w w w .proce sste x t.com/a bcchm.html
ONLINE TABLE 12-3 Guidelines for Dispensing Topical Steroids Am ou nt for Tw ice -D Am ail ou y nt Ap for plic On ati e
on
Ap for Ar plic 7 ea ati Da T r on ys eat (g) (g) ed * * Fac 1 15 e Sca 1
15
lp Ha 2
30
nds Ar 2
30
m Fee 3
45
t Leg 4 Ant 4
60 60
eri or
Pa g e 2 9 0 3
ABC Ambe r CHM Conve rte r Tria l ve rsion, http://w w w .proce sste x t.com/a bcchm.html
t ru nk Pos 4
60
t eri or t ru nk Ent 30 42 i re
5
bo
(1
dy l b.) *Amou nt s repres ent averag es for creams us ed in adul t popul a t i on.
(2) Mos t creams , oi nt ment s , and gel s are commerci al l y avai l abl e i n s mal l (15g or 30 g) or l arge (45 g or 60 g) t ubes .
(3) Mos t s ol ut i ons and l ot i ons are commerci al l y avai l abl e i n s mal l (30-mL) or l arge (60-mL) bot t l es .
o
o
d. Compl i cat i ons
Pa g e 2 9 0 4
ABC Ambe r CHM Conve rte r Tria l ve rsion, http://w w w .proce sste x t.com/a bcchm.html
(1) Local s i de effect s occur wi t h prol onged us e, es peci al l y on i nt ert ri gi nous areas and on t he face
(Onl i ne Tabl e 12-4).
ONLINE TABLE 12-4 Local Side Effects of Topical Steroids At r Hy op po hy pi g me nt a tio n Ca Ro ndi s ac da eas up l i k eri e nfe eru ct i pt i on on St ri Im ae, pai s t e red l l at wo e
un
ps e d ud he os c al i ars ng Mi l i Acn ari ei f a
or m eru
Pa g e 2 9 0 5
ABC Ambe r CHM Conve rte r Tria l ve rsion, http://w w w .proce sste x t.com/a bcchm.html
pt i on Tel Exa an cer gi e bat ct a i on s i a of ,
cut
pur an pur eo a,
us
an i nf d
ect
ery i on t he s , ma bac t eri al an d der ma t op hyt e i nf es t at i ons , s ca bi e s, or pe
Pa g e 2 9 0 6
ABC Ambe r CHM Conve rte r Tria l ve rsion, http://w w w .proce sste x t.com/a bcchm.html
di c ul o sis Peri oro faci al dermat itis
(2) Sys t emi c s i de effect s may res ul t from s ys t emi c abs orpt i on. Sys t emi c abs orpt i on may res ul t i n hypot hal amus –pi t ui t ary–adrenal (HPA) axi s s uppres s i on. Ri s k fact ors for HPA-axi s s uppres s i on i ncl ude i mpai red barri er funct i on, i ncreas ed s urface area of t reat ed s ki n, hi gh pot ency, and prol onged us e.
P.577
6. Topi cal ant i bi ot i cs are us eful agent s for t he t reat ment of s econdary bact eri al i nfect i on of s uperfi ci al wounds ; burns ; and s uperfi ci al , pri mary s ki n i nfect i ons (e.g., i mpet i go). Agent s i ncl ude mupi roci n cream or oi nt ment , s i l ver s ul fadi azi ne cream, pol ymyxi n B, baci t raci n oi nt ment (nonpres cri pt i on), and neomyci n oi nt ment (nonpres cri pt i on).
7. Topi cal ant i fungal s are pres cri bed for t he t reat ment of s uperfi ci al yeas t (e.g., candi di as i s ) and fungal (e.g., dermat ophyt e) i nfect i ons of t he s ki n. General l y, t he t opi cal
Pa g e 2 9 0 7
ABC Ambe r CHM Conve rte r Tria l ve rsion, http://w w w .proce sste x t.com/a bcchm.html
agent i s avai l abl e as a cream, but ot her formul at i ons i ncl ude powder, l acquer (for nai l s ), s pray, gel , s ol ut i on, vagi nal s uppos i t ory, and oral t roche. Topi cal ant i fungal agent s are l i s t ed in
Onl i ne Tabl e 12-5.
ONLINE TABLE 12-5 Topical Antifungal Agents F or m ul R at x A io / g n O e (s T nt ) C Polyen e N C, R ys O, x ta O t i S, n P, * V T, T Imida zoles Cl C, O ot L, T ri S, C m T, az V ol T
Pa g e 2 9 0 8
ABC Ambe r CHM Conve rte r Tria l ve rsion, http://w w w .proce sste x t.com/a bcchm.html
e Ec C R o
x
n az ol e K C, C et S = oc h R o
x;
n
S
az
h
ol
=
e
O T
C Mi C, O co L, T n S, C az P ol e O C, R xi L x co n az ol e S C, R ul S x co n az ol e
Pa g e 2 9 0 9
ABC Ambe r CHM Conve rte r Tria l ve rsion, http://w w w .proce sste x t.com/a bcchm.html
Allyla mines and nonaz ole ergost erol synthe sis inhibit ors A N R m L x or ol fi n e B C R ut
x
e n af in e H Cl N C, R af O, x tif P in e T C, O er S T bi
C
n
Pa g e 2 9 1 0
ABC Ambe r CHM Conve rte r Tria l ve rsion, http://w w w .proce sste x t.com/a bcchm.html
af in e Other agent s Ci C, R cl L, x o N pi L, ro S x h ol a m in e H C R al
x
o pr o gi n T C, O ol S, T n P C af ta te U C, O n P, T d O, C ec S yl e n
Pa g e 2 9 1 1
ABC Ambe r CHM Conve rte r Tria l ve rsion, http://w w w .proce sste x t.com/a bcchm.html
at e *Act i v e agai ns t yeas t ( Candi d a) onl y. C, cream; L, l ot i on; NL, nai l l acque r; O, oi nt me nt ; OS, oral s us pen s i on; P, powder ; S, s ol ut i o n/s pra y; Sh, s hamp oo; T, t roche; VT, vagi na
Pa g e 2 9 1 2
ABC Ambe r CHM Conve rte r Tria l ve rsion, http://w w w .proce sste x t.com/a bcchm.html
l t abl et ; Rx, pres cri pt i on; OTC, over-t he-cou nt er (nonpr es cri pt i on).
8. Topi cal ant i vi ral for human papi l l oma vi rus (HPV). Imi qui mod (5% cream) has been approved for t he t rea t ment of geni t al wart s . It i s an i mmune res pons e modi fi er t hat i nduces cyt oki nes , i ncl udi ng i nt erferon-α (IFN-α), at t he t reat ment s i t e.
C. Systemic therapy
1. Systemic corticosteroids are an avai l abl e opt i on i n dermat ol ogi c t herapy for s evere i nfl ammat ory di s eas es of t he s ki n t hat are not amenabl e t o t opi cal s t eroi d t herapy. o
o
a. Prednisone i s t he oral s t eroi d of choi ce of t he oral agent s us ed for t reat ment of dermat ol ogi c di s eas es .
o
o
b. There are a vari et y of t reat ment regi mens . For s uppres s i on of mos t i nfl ammat ory dermat os es i n
Pa g e 2 9 1 3
ABC Ambe r CHM Conve rte r Tria l ve rsion, http://w w w .proce sste x t.com/a bcchm.html
adul t s , an i ni t i al s t art i ng oral dos e of 40–60 mg predni s one i s commonl y us ed. For acut e condi t i ons , t he predni s one i s t apered over 10–20 days . If t he cours e of t herapy for s uppres s i on i s t oo s hort , a fl are and exacerbat i on may occur when prednis one i s di s cont i nued. o
o
c. Long-t erm oral us e requi res t aperi ng t o an al t ernat e-day dos e, whi ch decreas es t he frequency of mos t s i de effect s except cat aract s and os t eoporos i s .
2. Oral ant i hi s t ami nes are oft en us ed t o t reat pruri t us as s oci at ed wi t h a wi de range of s ki n di s eas es , part i cul arl y t hos e t hat are medi at ed by t ype I hi s t ami ne-medi at ed i mmune res pons es (e.g., urt i cari a). o
o
a. Compet i t i ve ant agoni s t s for hi s t ami ne H 1 recept ors are commonl y pres cri bed. Thes e agent s are met abol i zed i n t he l i ver. The s hort er-act i ng agent s l as t 3–6 hours , and t he l onger act i ng agent s l as t 12–24 hours .
o
o
b. Compl i cat i ons of ant i hi s t ami nes i ncl ude chol i nergi c s i de effect s (e.g., gl aucoma, cons t i pat i on, and xeros t omi a) and s edat i on. The l onger-act i ng ant i hi s t ami nes produce l es s s edat i on.
o
c. In recal ci t rant cas es of pruri t us , H 2 ant agoni s t s are added i n combi nat i on wi t h H 1 ant i hi s t ami nes . H 1 and H 2 recept or bl ockade al s o may be achi eved wi t h t he us e of doxepi n, a t ri cycl i c ant i depres s ant wi t h s t rong H 1 and H 2 ant i hi s t ami ne bl ocki ng
Pa g e 2 9 1 4
ABC Ambe r CHM Conve rte r Tria l ve rsion, http://w w w .proce sste x t.com/a bcchm.html
effect s . Bot h H 1 and H 2 ant i hi s t ami nes are avai l abl e i n pres cri pt i on and nonpres cri pt i on form
(onl i ne Tabl e 12-6).
o
ONLINE TABLE 12-6 Antihistamines Used in Dermatologic Treatments C D o o m s R p e x o ( / u mO n g T d ) C Shortacting H1 antag onists C 4 O hl
T
or
C
p h e ni ra m in e Di 2 O p 5, T h 5 C e 0 n
Pa g e 2 9 1 5
ABC Ambe r CHM Conve rte r Tria l ve rsion, http://w w w .proce sste x t.com/a bcchm.html
hy dr a m in e H 1 R yd 0, x ro 2 xy 5, zi 5 n 0, e 1 0 0 Longacting H1 antag onists C 5, R et 1 x i ri 0 zi n e D 1 R es 0 x lo ra ta di n e Fe 6 R xo 0, x fe 1
Pa g e 2 9 1 6
ABC Ambe r CHM Conve rte r Tria l ve rsion, http://w w w .proce sste x t.com/a bcchm.html
n 8 a 0 di n e Lo 1 O ra 0 T ta
C
di n e H2 antag onists Ci 2 O m 0 T et 0, C id 3 in 0 e 0, 4 0 0, 8 0 0 Fa 2 O m 0, T ot 4 C id 0 in e R 7 O a 5, T ni 1 C ti 5 di 0,
Pa g e 2 9 1 7
ABC Ambe r CHM Conve rte r Tria l ve rsion, http://w w w .proce sste x t.com/a bcchm.html
n 3 e 0 0 H 1 /H 2 Antag onist D 1 O ox 0, T e 2 C pi 5 n Rx, pres cri pt i on; OTC, over-t he-cou nt er.
IV. Acne and Rosacea A. Acne vulgaris
1. Definition. Acne vul gari s i s a common di s order of t he pilosebaceous unit (hai r fol l i cl e and s ebaceous gl ands ), l ocat ed pri mari l y on t he face and t runk. It i s mani fes t ed by fol l i cul ar comedones wi t h or wi t hout i nfl ammat ory papul es , pus t ul es , and nodul es .
2. Epidemiology. Acne vul gari s affect s 85%–100% of i ndi vi dual s t o s ome degree duri ng t hei r l i fet i me.
3. Etiology o
Pa g e 2 9 1 8
ABC Ambe r CHM Conve rte r Tria l ve rsion, http://w w w .proce sste x t.com/a bcchm.html
o
a. The obs t ruct i on of s ebaceous fol l i cl es i s due t o exces s i ve s ebum product i on by s ebaceous gl ands i n combi nat i on wi t h exces s i ve des quamat i on of t he fol l i cul ar epi t hel i um.
o
o
b. Infl ammat ory changes are l i nked t o t he pres ence of Propi oni bac t eri um ac nes , a res i dent , l i pophi l i c anaerobe. P. ac nes prol i ferat es i n t he mi croenvi ronment creat ed by exces s s ebum and des quamat ed fol l i cul ar cel l s . Thi s bact eri um produces chemot act i c fact ors and proi nfl ammat ory medi at ors t hat cont ri but e t o i nfl ammat i on.
4. Pathophysiology o
o
a. Acne may begi n i n t he prepubert al peri od, when i ncreas i ng amount s of adrenal androgens (dehydroepi andros t erone s ul fat e [DHEAS]) s t i mul at e s ebaceous gl and hyperpl as i a and i ncreas ed s ebum product i on. P.578
o
o
b. Abnormal fol l i cul ar di fferent i at i on i s coupl ed wi t h t hi s phenomenon, res ul t i ng i n t he devel opment of t he pri mary acne l es i ons , noni nfl ammat ory open and cl os ed comedones (“bl ackheads ― and “whi t eheads ,― res pect i vel y).
o
Pa g e 2 9 1 9
ABC Ambe r CHM Conve rte r Tria l ve rsion, http://w w w .proce sste x t.com/a bcchm.html o
c. Dermal i nfl ammat i on and fol l i cul ar rupt ure can l ead t o acne s carri ng.
5. Clinical features
(onl i ne Fi gure 12-8)
online Figure 12-8 Acne vul gari s . (Court es y of St uart Les s i n, MD.) o
o
a. Si t es of i nvol vement i ncl ude t he face and upper trunk.
o
o
b. Noninflammatory (comedonal) acne i s domi nat ed by open or cl os ed comedones wi t hout i nfl ammat ory l es i ons .
o
o
c. Mi l d inflammatory acne i s charact eri zed by i nfl ammat ory papul es and comedones .
o
o
d. Moderat e i nfl ammat ory acne i s charact eri zed by
Pa g e 2 9 2 0
ABC Ambe r CHM Conve rte r Tria l ve rsion, http://w w w .proce sste x t.com/a bcchm.html
comedones , i nfl ammat ory papul es , and pus t ul es . o
o
e. Nodulocystic acne i s charact eri zed by comedones , i nfl ammat ory papul es /pus t ul es , and i nfl ammat ory nodul es (>5 mm). Scarring i s oft en evi dent .
6. Diagnosis. The di agnos i s of acne vul gari s i s cl i ni cal .
7. T herapy. Treat ment i s di rect ed t oward t he pat hogeni c fact ors i nvol ved, i ncl udi ng fol l i cul ar dys kerat i ni zat i on, exces s s ebum product i on, and P. ac nes . The s everi t y of t he acne det ermi nes t he t ype and l evel of t herapy. o
o
a. T opical therapy
(1) T opical retinoids are comedol yt i c and ant i -i nfl ammat ory. The mos t commonl y pres cri bed t opi cal ret i noi ds i ncl ude adapal ene, t azarot ene, and t ret i noi n. They are us ual l y us ed i n combi nat i on wi t h ot her t reat ment s .
(2) T opical antibiotics t arget P. ac nes . Commonl y pres cri bed t opi cal ant i bi ot i cs i ncl ude eryt hromyci n and cl i ndamyci n.
(3) Benzoyl peroxide has ant i mi crobi al propert i es agai ns t P. ac nes . Benzoyl peroxi de i s t he act i ve i ngredi ent i n many OTC t opi cal
Pa g e 2 9 2 1
ABC Ambe r CHM Conve rte r Tria l ve rsion, http://w w w .proce sste x t.com/a bcchm.html
s ki n care product s ; i t i s avai l abl e i n a vari et y of formul at i ons , i ncl udi ng s oaps , was hes , l ot i ons , creams , and gel s . In addi t i on, i t i s avai l abl e by pres cri pt i on. Benzoyl peroxi de i s oft en combi ned wi t h t opi cal ant i bi ot i cs t o i mprove effi cacy and reduce ant i bi ot i c res i s t ance. o
o
b. Systemic therapy
(1) Oral antibiotics. Li pophi l i c ant i bi ot i cs are mos t effect i ve agai ns t P. ac nes . Tet racycl i ne, doxycycl i ne, and mi nocycl i ne are commonl y pres cri bed for i nfl ammat ory acne.
(2) Isotretinoin. Thi s s ys t emi c ret i noi d i s hi ghl y effect i ve i n t reat i ng s evere and nodul ocys t i c acne t hat i s recal ci t rant t o oral ant i bi ot i c and t opi cal t herapi es . It caus es normal i zat i on of epi dermal di fferent i at i on, depres s es s ebum product i on, i s ant i -i nfl ammat ory, and reduces l evel s of P. ac nes i n t he s ki n. Informed cons ent wi t h cont racept i on couns el i ng and bas el i ne l aborat ory t es t s (i ncl udi ng s erum pregnancy t es t ) are mandat ory before t he i ni t i at i on of t herapy. Mandatory participation in the iPLEDGEâ„¢ program is required for prescribers and patients receiving isotretinoin.
(3) Hormonal therapy. Thi s al t ernat i ve t o s ys t emi c ant i bi ot i cs and i s ot ret i noi n i n women
Pa g e 2 9 2 2
ABC Ambe r CHM Conve rte r Tria l ve rsion, http://w w w .proce sste x t.com/a bcchm.html
wi t h pers i s t ent acne i nvol ves t reat ment wi t h es t rogen or an ant i androgen. Oral cont racept i ves may be effect i ve. Combi nat i on oral cont racept i ve pi l l s (norges t i mat e–et hi nyl es t radi ol ) cont ai ni ng l ow l evel s of proges t at i onal compounds are preferred. Spi ronol act one, whi ch reduces androgen product i on, may be us ed al one or i n combi nat i on wi t h an oral cont racept i ve.
B. Rosacea
1. Definition. Ros acea i s a di s order of t he bl ood ves s el s and s ebaceous gl ands of t he face charact eri zed by eryt hema, t el angi ect as i a, fl us hi ng, and an i nfl ammat ory papul opus t ul ar erupt i on res embl i ng acne.
2. Epidemiology. Ros acea i s a common di s order t hat affect s approxi mat el y 14 mi l l i on adul t s i n t he Uni t ed St at es . It i s s een more frequent l y i n fai r-s ki nned i ndi vi dual s of European and Cel t i c des cent .
3. Etiology. The preci s e et i ol ogy of ros acea i s unknown. Cl i ni cal s i gns and s ympt oms res ul t from t he i nt eract i on of genet i c s us cept i bi l i t y wi t h envi ronment al t ri ggers (UV radi at i on, and t emperat ure changes ) and di et ary t ri ggers (hot dri nks , al cohol , and s pi cy foods ). P.579
4. Pathophysiology
Pa g e 2 9 2 3
ABC Ambe r CHM Conve rte r Tria l ve rsion, http://w w w .proce sste x t.com/a bcchm.html o
o
a. Vas cul ar l abi l i t y res ul t s i n i nt ermi t t ent faci al fl us hi ng, l eadi ng t o pers i s t ent rednes s and t el angi ect as i as t hat affect t he cent ral face and occas i onal l y t he eyes .
o
o
b. The pat hogenes i s of t he s ebaceous hyperpl as i a and t he i nfl ammat ory papul es and pus t ul es i s uncl ear. Mi t es of t he genus Demodex (i ndi genous t o human hai r fol l i cl es ) appear i n great er numbers , but t hei r et i ol ogi c rol e has not been es t abl i s hed.
5. Clinical features
(onl i ne Fi gure 12-9)
online Figure 12-9 Ros acea. (Court es y of St uart Les s i n, MD.) o
o
a. Fl us hi ng, eryt hema, and t el angi ect as i a can occur over t he cheeks , forehead, and chi n.
o
Pa g e 2 9 2 4
ABC Ambe r CHM Conve rte r Tria l ve rsion, http://w w w .proce sste x t.com/a bcchm.html
o
b. Infl ammat ory papul es and pus t ul es i nvol ve t he nos e, forehead, and t he cheeks , wi t h an abs ence of comedones and s carri ng.
o
o
c. Rhi nophyma i s a promi nence of s ebaceous gl ands of t he nos e t hat produces t hi ckened s ki n and di s fi gurement i n ext reme cas es .
o
o
d. Ocul ar ros acea produces ocul ar s i gns t hat may i ncl ude conjunct i val i nject i on, edema, chal azi on, and epi s cl eri t i s .
6. Diagnosis. The di agnos i s i s cl i ni cal . Overl appi ng feat ures of acne vul gari s may s omet i mes be pres ent , and i t may be di ffi cul t of di fferent i at e acne vul gari s and ros acea.
7. T herapy. Avoi dance of envi ronment al and di et ary t ri ggers i s an i mport ant s t rat egy t o reduce s i gns and s ympt oms . o
o
a. T opical antibiotics (e.g., t opi cal met roni dazol e) may be pres cri bed for mi l d di s eas e.
o
o
b. Systemic antibiotics (e.g., t et racycl i ne, doxycycl i ne, and mi nocycl i ne) are us ual l y effect i ve i n t reat i ng acnei form l es i ons but have l i mi t ed effect i venes s i n t reat i ng t he faci al eryt hema.
o
Pa g e 2 9 2 5
ABC Ambe r CHM Conve rte r Tria l ve rsion, http://w w w .proce sste x t.com/a bcchm.html o
c. Retinoids (e.g., i s ot ret i noi n) are pres cri bed as a one-t i me cours e (s i mi l ar t o acne t herapy) and may be an effect i ve t reat ment for s evere cas es . Al t ernat i vel y, l ong-t erm, l ow-dos e i s ot ret i noi n can be us ed.
V. Autoimmune Blistering Diseases: Pemphigus and Bullous Pemphigoid A. General considerations
1. Aut oi mmune bl i s t eri ng di s eas es are charact eri zed by
bl i s t eri ng of t he s ki n and mucous membranes as s oci at ed wi t h t he depos i t i on of aut oant i bodi es .
2. Local i zat i on of aut oant i bodi es t o epi t opes i n t he epi dermi s , BMZ, or dermi s defi nes t he s peci fi c cl i ni cal feat ures , hi s t opat hol ogy, and di rect i mmunofl uores cent s t ai ni ng pat t ern (onl i ne Tabl e 12-7).
ONLINE TABLE 12-7 Target Autoantibodies and Direct Immunofluorescence in Autoimmune Blistering Diseases
Pa g e 2 9 2 6
ABC Ambe r CHM Conve rte r Tria l ve rsion, http://w w w .proce sste x t.com/a bcchm.html
Di re ct I m m u A
n
ut
of
oi
lu
m
or
m
e
u
sc
n
e
e
n
Bl
c
is
e
te
(
ri
D
n
IF
g
)
Di Ig Fi s E n e pi di a to n s p g e e s P D In e es t e m m rc p o el hi gl l u g ei l a us n r vu 1, Ig
Pa g e 2 9 2 7
ABC Ambe r CHM Conve rte r Tria l ve rsion, http://w w w .proce sste x t.com/a bcchm.html
lg 3 G ar
st
is
ai ni n
g P D In e es t e m m rc p o el hi gl l u g ei l a us n r fo 1 Ig li
G
ac
st
e
ai
us
ni n
g P D In ar es t e a m rc n o el e gl l u o ei l a pl n r as 1, Ig ti 3 G c D st p es ai e m ni m o n p pl g hi ak a g in n
Pa g e 2 9 2 8
ABC Ambe r CHM Conve rte r Tria l ve rsion, http://w w w .proce sste x t.com/a bcchm.html
us 1, d 2 li E n nv e o ar pl B ak M in Z ,
C
p 3 er d ip e la p ki os n, i t pl s ec ti n B D Ig ul es G lo m a us o n p pl d e ak l i m in n p 1 e hi C ar g ol C oi l a 3 d g d e e n p X os VI i t
Pa g e 2 9 2 9
ABC Ambe r CHM Conve rte r Tria l ve rsion, http://w w w .proce sste x t.com/a bcchm.html
I s al o n g B M Z; o n e pi d er m al si d e of sa lt -s pl it sk in * Ci E Ig ca pi G t ri l i a ci gr n al i n d p C li e ol n
Pa g e 2 9 3 0
ABC Ambe r CHM Conve rte r Tria l ve rsion, http://w w w .proce sste x t.com/a bcchm.html
m la e p g ar hi e C g n 3 oi X d d VI e I p os it s al o n g B M Z; o n d er m al si d e of sa lt -s pl it sk in H C Ig
Pa g e 2 9 3 1
ABC Ambe r CHM Conve rte r Tria l ve rsion, http://w w w .proce sste x t.com/a bcchm.html
er ol G p la a es g n g e d es n l i ta X n t i VI e o I ar ni
C
s
3 d e p os it s al o n g B M Z; o n e pi d er m al si d e of
Pa g e 2 9 3 2
ABC Ambe r CHM Conve rte r Tria l ve rsion, http://w w w .proce sste x t.com/a bcchm.html
sa lt -s pl it sk in E C Ig pi ol G d la a er g n m e d ol n l i ys VI n is I e b
ar
ul
C
lo
3
sa
d
ac
e
q
p
ui
os
si
it
ta
s al o n g B M Z; o n d er
Pa g e 2 9 3 3
ABC Ambe r CHM Conve rte r Tria l ve rsion, http://w w w .proce sste x t.com/a bcchm.html
m al si d e of sa lt -s pl it sk in Li C Li n ol n e la e ar g ar Ig e Ig A n A d VI d er I e m C p at ol os os l a i t i s g al e o n n X g VI B I M Z a n d d
Pa g e 2 9 3 4
ABC Ambe r CHM Conve rte r Tria l ve rsion, http://w w w .proce sste x t.com/a bcchm.html
er m al si d e of sa lt -s pl it sk in B C Li ul ol n lo la e us g ar l u e Ig p n A us VI or er I gr yt
a
h
n
e
ul
m
ar
at
Ig
os
G
us
d e p os it s al
Pa g e 2 9 3 5
ABC Ambe r CHM Conve rte r Tria l ve rsion, http://w w w .proce sste x t.com/a bcchm.html
o n g B M Z a n d d er m al si d e of sa lt -s pl it sk in BMZ, bas em ent membr ane z one; Ig, i mmun ogl obu l i n. *Sal t -s
Pa g e 2 9 3 6
ABC Ambe r CHM Conve rte r Tria l ve rsion, http://w w w .proce sste x t.com/a bcchm.html
pl i t s ki n i s i ncuba t i on of s ki n bi ops y in 1 mol /L s al t before perfor mi ng t he DIF res ul t s in cl eava ge t hroug h t he l ami na l uci da of bas em ent membr ane.
3. Mos t aut oi mmune bl i s t eri ng di s eas es are i di opat hi c but may be as s oci at ed wi t h medi cat i ons , s uch as pemphi gus (s ee V B) and l i near IgA dermat os i s (s ee XII E 14), pregnancy (herpes ges t at i oni s ; s ee Tabl e 12-7), aut oi mmune di s eas e (bul l ous l upus eryt hemat os us ; Tabl e 12-8), or mal i gnancy (paraneopl as t i c pemphi gus ).
Pa g e 2 9 3 7
ABC Ambe r CHM Conve rte r Tria l ve rsion, http://w w w .proce sste x t.com/a bcchm.html
4. Two of t he mos t commonl y encount ered forms of aut oi mmune bl i s t eri ng di s eas es are pemphi gus and bul l ous pemphi goi d. Paraneopl as t i c pemphi goi d i s al s o rarel y s een.
B. Pemphigus
1. Epidemiology. The i nci dence of pemphi gus vari es from 1 per 1,000,000 t o 50 per 1,000,000 popul at i on dependi ng on t he t ype of pemphi gus and et hni ci t y. The ons et of pemphi gus i s t ypi cal l y young t o mi d adul t hood (<35 years of age) and affect s bot h s exes equal l y (s ee Tabl e 12-8).
TABLE 12-8 Comparison of Autoimmune Blistering Diseases Pa ra ne Bul Pe opl lou mp ast s hig ic pe us pe mp vul mp hig ga hig oid ris oid Av Av Ag g
g
es
ag ag var e
e
y
ons ons et , et ,
Pa g e 2 9 3 8
ABC Ambe r CHM Conve rte r Tria l ve rsion, http://w w w .proce sste x t.com/a bcchm.html
65 <3 5 Te Fl a Var ns e cci i ed bul d
bul
l ae bul l ae l ae Rar Oft Oft el y en en i nv i nv i nv ol v ol v ol v es es es mu mu mu cou cou cou s
s
s
me me me mb mb mb ran ran ran e
es es P.580
2. Etiology. Pemphi gus i s an i mmune-medi at ed di s eas e as s oci at ed wi t h t he product i on of IgG aut oant i bodi es t hat t arget i nt raepi dermal epi t opes . The preci s e caus e of aut oant i body product i on i s unknown. Cert ai n drugs may t ri gger pemphi gus i n s ome i ndi vi dual s . Exacerbat i ng drugs i ncl ude peni ci l l ami ne and angi ot ens i n-convert i ng enz yme i nhi bi t ors .
3. Pathophysiology. Pemphi gus i s medi at ed by IgG t arget i ng of des mogl ei n, a t rans membrane gl ycoprot ei n component of des mos omes . Des mos omes are
Pa g e 2 9 3 9
ABC Amber CHM Converter Trial version, http://www.processtext.com/abcchm.html
res pons i bl e for i nt ercel l ul ar adhes i on of kerat i nocyt es , and t hei r bi ndi ng by pemphi gus aut oant i bodi es res ul t s i n s eparat i on of kerat i nocyt es (acant hol ys i s ) and product i on of i nt raepi dermal bl i s t ers . o
o
a. Pemphigus vulgaris devel ops from IgG t arget i ng of des mogl ei n 3 (on kerat i nocyt es of mucous membranes and s ki n) and des mogl ei n 1 (on kerat i nocyt es of s ki n). Bl i s t eri ng res ul t s from s pl i t t i ng t hrough t he s uprabas al port i on of t he epi dermi s .
o
o
b. Pemphigus foliaceus devel ops from IgG t arget i ng des mogl ei n 1, res ul t i ng i n more s uperfi ci al bl i s t eri ng wi t hi n t he s ubcorneal port i on of t he epi dermi s .
4. Clinical features o
a. Pemphigus vulgaris i s charact eri zed by dys phagi a s econdary t o oral l es i ons (90% of cas es )
(Onl i ne Fi gure 12-10). Ski n bl i s t ers
are di s t ri but ed predomi nat el y on t he t runk and s cal p. They are fl acci d and break eas i l y (s ee Tabl e 12-8). o
Page 2940
ABC Ambe r CHM Conve rte r Tria l ve rsion, http://w w w .proce sste x t.com/a bcchm.html
ONLINE FIGURE 12-10 Oral pemphi gus vul gari s . (Court es y of St uart Les s i n, MD.) o
o
b. Pemphigus foliaceus i s charact eri zed by more crus t ed or denuded l es i ons (due t o t he s uperfi ci al nat ure of t he bl i s t eri ng) wi t h i nvol vement of t he face, s cal p, and t runk. Oral i nvol vement i s rare.
5. Diagnosis. Ins pect i on of t he s ki n and i dent i fi cat i on of pri mary l es i ons provi de a cl i ni cal di agnos i s . Skin biopsy i s requi red t o confi rm t he cl i ni cal i mpres s i on. o
o
a. Histology s hows i nt raepi dermal acant hol ys i s at t he s uprabas al (pemphi gus vul gari s ) or s ubcorneal (pemphi gus fol i aceus ) l evel .
o
o
b. Direct immunofluorescence (DIF) of peri l es i onal s ki n bi ops y s peci mens s hows IgG depos i t i on t hroughout t he i nt ercel l ul ar s paces bet ween kerat i nocyt es .
o
Pa g e 2 9 4 1
ABC Ambe r CHM Conve rte r Tria l ve rsion, http://w w w .proce sste x t.com/a bcchm.html o
c. Indirect immunofluorescence (IDIF) det ect s ci rcul at i ng IgG aut oant i bodi es i n t he pat i ent 's s erum. Serum t i t er l evel s us ual l y mi rror di s eas e act i vi t y.
6. T herapy o
o
a. Systemic corticosteroids are us ed t o provi de rapi d cont rol of t he bl i s t eri ng proces s .
o
o
b. Immunosuppressants are us ed i n combi nat i on wi t h cort i cos t eroi ds and as s t eroi d-s pari ng agent s for l ong-t erm cont rol and mai nt enance. Thes e agent s i ncl ude azat hi opri ne, mycophenol at e mofet i l , cycl ophos phami de, met hot rexat e, cycl os pori ne, i nt ravenous i mmune gl obul i n, and ant i bi ot i cs .
C. Bullous pemphigoid
1. Epidemiology. The exact i nci dence of bul l ous pemphi goi d i n t he Uni t ed St at es i s unknown. The report ed i nci dence i n Europe i s 6.6 cas es per mi l l i on annual l y. The average age at ons et i s 65 years .
2. Etiology. Bul l ous pemphi goi d i s an i mmune-medi at ed di s eas e as s oci at ed wi t h t he product i on of IgG aut oant i bodi es t hat t arget bas ement membrane epi t opes . The preci s e caus e of aut oant i body product i on i s unknown. Bul l ous pemphi goi d has been report ed t o be preci pi t at ed by phot ot herapy, radi at i on t herapy, and expos ure t o cert ai n medi cat i ons . Drugs t hat exacerbat e
Pa g e 2 9 4 2
ABC Ambe r CHM Conve rte r Tria l ve rsion, http://w w w .proce sste x t.com/a bcchm.html
bul l ous pemphi goi d i ncl ude furos emi de, nons t eroi dal ant i -i nfl ammat ory drugs (NSAIDs ), capt opri l , peni ci l l ami ne, and ant i bi ot i cs .
3. Pathophysiology. The IgG aut oant i bodi es of bul l ous pemphi goi d t arget des mopl aki n 1 (bul l ous pemphi goi d ant i gen 1) and col l agen XVII (bul l ous pemphi goi d ant i gen 2), whi ch are i nt racel l ul ar and t rans membrane component s , res pect i vel y, of hemi des mos omes . Hemi des mos omes are res pons i bl e for t he adhes i on of bas al kerat i nocyt es wi t h t he bas ement membrane. The bi ndi ng of ant i bodi es at t he bas ement membrane act i vat es compl ement and at t ract s i nfl ammat ory cel l s t o rel eas e prot eas es , l eadi ng t o s ubepi dermal bl i s t er format i on. (See onl i ne s t ruct ure and funct i on s ect i on.)
4. Clinical features o
a. Tens e bul l ae, wi t h or wi t hout eryt hema, us ual l y i nvol ve t he t runk and fl exural areas of t he s ki n
(onl i ne Fi gure 12-11.) (s ee
Tabl e 12-8). o
online Figure 12-11 Bul l ous pemphi goi d. (Court es y of St uart Les s i n, MD.) o
Pa g e 2 9 4 3
ABC Ambe r CHM Conve rte r Tria l ve rsion, http://w w w .proce sste x t.com/a bcchm.html
o
b. Mucos al i nvol vement rarel y occurs .
o
o
c. Pers i s t ent urt i cari al l es i ons s ubs equent l y devel op bul l ae.
P.581
5. Diagnosis o
o
a. Clinical inspection of t he s ki n and i dent i fi cat i on of pri mary l es i ons yi el d a cl i ni cal di agnos i s . Ski n bi ops y for hi s t ol ogy and DIF i s requi red t o confi rm t he cl i ni cal i mpres s i on.
(1) Histology s hows s ubepi dermal s pl i t wi t h an i nt act epi dermi s as t he bl i s t er roof. Eos i nophi l s oft en predomi nat e i n t he dermal i nfi l t rat e.
(2) DIF reveal s l i near IgG and C3 depos i t i on al ong t he DEJ. Bul l ous pemphi goi d can be di fferent i at ed from ot her s ubepi dermal aut oi mmune bl i s t eri ng di s eas es by i ncubat i ng t he s ki n bi ops y s ampl e i n 1 mol /L s al t before performi ng DIF. Thi s proces s i nduces cl eavage t hrough t he l ami na l uci da. DIF on s al t -s pl i t s ki n s hows IgG on t he bl i s t er roof (epi dermal s i de of s pl i t s ki n).
o
Pa g e 2 9 4 4
ABC Ambe r CHM Conve rte r Tria l ve rsion, http://w w w .proce sste x t.com/a bcchm.html o
b. IDIF can det ect ci rcul at i ng IgG aut oant i bodi es i n t he s erum of mos t pat i ent s . However, t i t er l evel s do not correl at e wi t h di s eas e act i vi t y.
6. T herapy. Mi l d cas es may be cont rol l ed wi t h ant i bi ot i cs and t opi cal s t eroi ds . Moderat e and s evere cas es requi re s ys t emi c cort i cos t eroi ds and s t eroi d-s pari ng i mmunos uppres s ant s , s uch as t hos e us ed t o t reat pemphi gus .
D. Paraneoplastic pemphigoid
1. Definition. Paraneopl as t i c pemphi gus i s an aut oi mmune bl i s t eri ng di s eas e charact eri zed by s evere mucos al i nvol vement and as s oci at i on wi t h an underl yi ng mal i gnancy.
2. Epidemiology. Paraneopl as t i c pemphi gus i s rare. Si nce i t s fi rs t des cri pt i on i n 1990, onl y 60+ cas es have been report ed.
3. Etiology. The mos t common mal i gnancy as s oci at ed wi t h paraneopl as t i c pemphi gus i s non-Hodgki n's l ymphoma.
4. Pathophysiology. An i mmune res pons e t o t umor ant i gens i s t hought t o generat e aut oant i bodi es t hat at t ack t he s ki n.
5. Clinical features o
Pa g e 2 9 4 5
ABC Ambe r CHM Conve rte r Tria l ve rsion, http://w w w .proce sste x t.com/a bcchm.html
o
a. Pai nful oral mucos al eros i ons t hat us ual l y pres ent as mucos i t i s wi t h crus t i ng are t he domi nant cl i ni cal feat ure.
o
o
b. Mucos al membranes of t he eyes , nos e, pharynx, t ons i l s , and geni t al i a may be affect ed.
o
o
c. Cut aneous erupt i on i s t ypi cal l y pol ymorphous . Combi nat i ons of bl i s t ers , ul cerat i ons , eryt hemat ous macul es and papul es , s cal y pl aques , urt i cari al pl aques , di ffus e eryt hroderma, and eros i ons have been report ed.
6. Diagnosis: Ski n bi ops y and DIF confi rm t he di agnos i s .
7. T herapy: Sympt omat i c rel i ef and t reat ment of s uperi nfect i on, i f pres ent , are neces s ary. o
o
a. Immunos uppres s i ve agent s are oft en i neffect i ve.
o
o
b. Treat ment of underl yi ng mal i gnancy may not res ul t i n s ki n cl eari ng.
VI. Cutaneous Reaction Patterns Each dermat ol ogi c di agnos i s i ncl uded i n t hi s s ect i on repres ent s a s peci fi c cut aneous react i on t hat can devel op i n res pons e t o mul t i pl e caus es . The cl i ni cal di agnos i s of each of t hes e cut aneous react i on pat t erns requi res a t horough i nves t i gat i on i nt o t he underl yi ng caus e t hat may vary from cas e t o cas e.
A. Erythema multiforme
Pa g e 2 9 4 6
ABC Ambe r CHM Conve rte r Tria l ve rsion, http://w w w .proce sste x t.com/a bcchm.html
1. Epidemiology. The t rue i nci dence of eryt hema mul t i forme i n t he Uni t ed St at es i s unknown. o
o
a. Erythema multiforme minor (l es s s evere form) may account for up t o 1% of dermat ol ogy offi ce vi s i t s .
o
o
b. Erythema multiforme major (more s evere form) occurs at an i nci dence of 0.6 t o 8.0 cas es per mi l l i on per year.
2. Etiology. Approxi mat el y 50% of cas es are i di opat hi c; no underl yi ng caus e i s i dent i fi ed. o
o
a. Infection. The mos t common caus es of eryt hema mul t i forme are i nfect i ous (vi ral , bact eri al , and fungal ). Herpes s i mpl ex vi rus (HSV) i s t he mos t common i nfect i ous et i ol ogy, fol l owed by Myc opl as ma pneumoni ae. Recurrent eryt hema mul t i forme (mi nor or major) can accompany recurrent HSV. P.582
o
o
b. Drug reaction. Anot her major caus e i s advers e drug react i ons as s oci at ed wi t h mal i gnancy, hormonal i mbal ances , col l agen vas cul ar di s eas es , and s arcoi dos i s .
Pa g e 2 9 4 7
ABC Amber CHM Converter Trial version, http://www.processtext.com/abcchm.html
3. Pathophysiology. Eryt hema mul t i forme i s an acut e hypers ens i t i vi t y react i on t o a vari et y of s t i mul i . It i s a cyt ot oxi c i mmunol ogi c react i on res ul t i ng i n epi dermal kerat i nocyt e necros i s as s oci at ed wi t h a dens e l ymphocyt i c i nfi l t rat e wi t hi n t he dermi s . Immune compl ex depos i t i on i s nons peci fi c. Subepi dermal bul l ae format i on occurs i n s evere forms .
4. Clinical features. Bas ed on t he s everi t y of feat ures and t he ext ent of mucous membrane i nvol vement , eryt hema mul t i forme may be cat egori zed i nt o mi nor and major forms . o
a. Erythema multiforme minor. Iris or target lesions are eryt hemat ous macul es or papul es wi t h concent ri c red borders and cent ral purpura or ves i cl es
(onl i ne Fi gure 12-12). Eryt hema
mul t i forme t ends t o be s ymmet ri c and have an acral di s t ri but i on, wi t h l es i ons i nvol vi ng t he pal ms and s ol es . Les i ons may i nvol ve ext remi t i es and face and coal es ce and become general i zed. Mi l d oral cavi t y bl i s t eri ng may be s een i n up t o 25% of cas es . Res ol ut i on us ual l y occurs wi t hi n 2 weeks . o
Page 2948
ABC Ambe r CHM Conve rte r Tria l ve rsion, http://w w w .proce sste x t.com/a bcchm.html
online Figure 12-12 Target l es i ons i n mul t i forme eryt hema. (Court es y of St uart Les s i n, MD.) o
o
b. Erythema multiforme major (Stevens-Johnson syndrome). Ski n fi ndi ngs are s i mi l ar t o eryt hema mul t i forme mi nor but are more s evere: <10% of s ki n area i s i nvol ved. Mout h, l i ps , and bul bar conjunct i vae are mos t commonl y affect ed, wi t h mucous membranes s everel y affect ed. Oral bul l ae break eas i l y and creat e eros i ons t hat readi l y become s econdari l y i nfect ed, oft en caus i ng pai n and bl oody, crus t ed l i ps .
o
o
c. T oxic epidermal necrolysis. Ski n l es i ons s i mi l ar t o eryt hema mul t i forme but i nvol vi ng >10% of t ot al s ki n s urface area. Int ravenous i mmune gl obul i n (IVIG) can be us ed as t reat ment . Des pi t e t reat ment , mort al i t y i s hi gh.
5. Diagnosis. Cl i ni cal i ns pect i on of t he s ki n and i dent i fi cat i on of pri mary s ki n l es i ons yi el ds a cl i ni cal di agnos i s . Ski n bi ops y confi rms t he cl i ni cal i mpres s i on. Eval uat i on of t he underl yi ng caus e s houl d accompany a cl i ni cal di agnos i s .
6. T herapy. Treat ment of t he underl yi ng caus e and wi t hdrawal of any offendi ng drugs i s t he fi rs t management s t ep. Support i ve care and s ympt omat i c rel i ef of pai nful and s econdari l y i nfect ed mucocut aneous l es i ons i s i ni t i at ed. Us e of s ys t emi c cort i cos t eroi ds i s cont rovers i al .
B. Lichen planus
Pa g e 2 9 4 9
ABC Ambe r CHM Conve rte r Tria l ve rsion, http://w w w .proce sste x t.com/a bcchm.html
1. Epidemiology. The i nci dence i s es t i mat ed at 1% of new pat i ent offi ce vi s i t s i n t he Uni t ed St at es .
2. Etiology. The exact et i ol ogy of l i chen pl anus i s unknown. Mos t cas es are i di opat hi c. A pos i t i ve fami l y hi s t ory i n a few cas es s ugges t s a genet i c predi s pos i t i on. As s oci at ed l i ver di s eas e, i ncl udi ng hepat i t i s C i nfect i on, chroni c act i ve hepat i t i s , and pri mary bi l i ary ci rrhos i s , as wel l as aut oi mmune di s orders s uch as ul cerat i ve col i t i s , dermat omyos i t i s , and myas t heni a gravi s , may occur. An advers e drug react i on (l i chenoi d drug react i on) may be t he caus e.
3. Pathophysiology. Li chen pl anus i s a cel l -medi at ed i mmunol ogi c react i on t arget i ng t he epi dermal kerat i nocyt es . A dens e l ymphocyt i c i nfi l t rat e at t he DEJ (l i chenoi d i nfi l t rat e) part i ci pat es i n t he des t ruct i on of bas al kerat i nocyt es (bas al vacuol i zat i on). Immunogenet i c predi s pos i t i on may pl ay a rol e i n t he pat hogenes i s . Human l eukocyt e ant i gen DR1 (HLA-DR1) and HLA-DR10 are as s oci at ed wi t h i di opat hi c l i chen pl anus , and HLA-B7 i s as s oci at ed wi t h fami l i al cas es .
4. Clinical features
(onl i ne Fi gure 12-13.) Mos t i di opat hi c cas es
are s el f-l i mi t i ng, l as t i ng 6 t o 18 mont hs . Ot her cas es can be chroni c and recurrent , part i cul arl y oral l i chen pl anus .
Pa g e 2 9 5 0
ABC Ambe r CHM Conve rte r Tria l ve rsion, http://w w w .proce sste x t.com/a bcchm.html
online Figure 12-13 Li chen pl anus . (Court es y of St uart Les s i n, MD.) o
o
a. Pol ygonal papul es are vi ol aceous (purpl e), pruri t i c, and fl at -t opped (l i chenoi d), us ual l y covered wi t h fi ne s cal es . Les i ons vary i n s i ze from 1 mm t o more t han 1 cm i n di amet er.
o
o
b. The di s t ri but i on of l es i ons t ypi cal l y i nvol ves fl exor s urfaces of t he upper ext remi t i es (wri s t s ), geni t al i a, and mucous membranes . Nai l s and s cal p may al s o be i nvol ved.
o
o
c. Oral l es i ons i nvol ve t he t ongue and buccal mucos a and may be as ympt omat i c pl aques di s pl ayi ng a whi t e ret i cul ar pat t ern (W i ckham s t ri ae). Eros i ve l es i ons are pai nful .
o
o
d. Nai l dys t rophy wi t h nai l pl at e t hi nni ng and ri dgi ng may be s een i n 10% of cas es . Nai l mat ri x
Pa g e 2 9 5 1
ABC Ambe r CHM Conve rte r Tria l ve rsion, http://w w w .proce sste x t.com/a bcchm.html
i nvol vement may l ead t o s carri ng of t he nai l bed (pt erygi um format i on). o
o
e. Scal p i nvol vement (l i chen pl anopi l ari s ) i s s een more oft en i n women and produces fol l i cul ar i nfl ammat i on, oft en res ul t i ng i n s carri ng al opeci a.
o
o
f. Les i ons res ol ve wi t h res i dual pos t i nfl ammat ory hyperpi gment at i on.
P.583
5. Diagnosis. Cl i ni cal i ns pect i on of t he s ki n and i dent i fi cat i on of pri mary s ki n l es i ons provi des a cl i ni cal di agnos i s . Ski n bi ops y confi rms t he cl i ni cal i mpres s i on. Eval uat i on of t he underl yi ng caus e and as s oci at ed di s orders s houl d accompany a cl i ni cal di agnos i s .
6. T herapy. Treat ment of an i dent i fi abl e underl yi ng caus e and withdrawal of any offending drugs are t he fi rs t management s t eps . Antihistamines and t opi cal s t eroi ds are effect i ve i n mi l d, s el f-l i mi t i ng cas es . Systemic corticosteroids are us ed acut el y t o s uppres s more s evere cas es . Long-t erm cont rol of chroni c cas es may requi re steroid-sparing immunosuppressants. T opical and systemic retinoids as wel l as topical and systemic cyclosporine have been s hown t o be act i ve i n s ome cas es .
C. Urticaria Urt i cari a (hi ves ) i s a common cut aneous react i on pat t ern requi ri ng i nves t i gat i on i nt o i t s underl yi ng et i ol ogy
(onl i ne Fi gure 12-14).
Pa g e 2 9 5 2
ABC Ambe r CHM Conve rte r Tria l ve rsion, http://w w w .proce sste x t.com/a bcchm.html
It i s covered i n det ai l i n Chapt er 7 IV.
online Figure 12-14 Urt i cari a. (Court es y of St uart Les s i n, MD.)
D. Pyoderma gangrenosum
1. Epidemiology. The i nci dence of pyoderma gangrenos um i n t he Uni t ed St at es i s es t i mat ed at approxi mat el y 1 per 100,000 peopl e each year.
2. Etiology. Pyoderma gangrenos um i s an ul cerat i ve s ki n di s order of unknown et i ol ogy. Several di s eas es are commonl y as s oci at ed wi t h pyoderma gangrenos um (Tabl e 12-9). No underl yi ng di s eas e as s oci at i on i s pres ent i n 50% of cas es .
3. Pathophysiology. The pat hophys i ol ogy of pyoderma gangrenos um i s poorl y unders t ood. It appears t o be an i mmune-medi at ed react i on i nvol vi ng an i nt ens e and des t ruct i ve neut rophi l i c i nfi l t rat e wi t hi n t he dermi s . The dermal i nfl ammat ory react i on caus es purul ent ul cerat i on. Immunocompl ex depos i t i on i s nons peci fi c.
4. Clinical features
(onl i ne Fi gure 12-15.)
Pa g e 2 9 5 3
ABC Ambe r CHM Conve rte r Tria l ve rsion, http://w w w .proce sste x t.com/a bcchm.html
online Figure 12-15 Pyoderma gangrenos um. (Court es y of St uart Les s i n, MD.) o
o
a. Ini t i al pres ent at i on may be eryt hemat ous t o vi ol aceous pai nful nodul es t hat qui ckl y ul cerat e.
o
o
b. Deep and purul ent ul cers wi t h hi ghl y i nfl ammat ory and t ender borders devel op.
o
o
c. Ul cer margi ns are t ypi cal l y undermi ned. Vi ol aceous borders overhang t he ul cer bed.
o
o
d. The l ower ext remi t i es are t ypi cal l y affect ed. Vari ant s may affect ot her areas .
o
o
e. Pathergy, t he devel opment of new l es i ons or aggravat i on of exi s t i ng ones aft er t rauma, may occur.
Pa g e 2 9 5 4
ABC Ambe r CHM Conve rte r Tria l ve rsion, http://w w w .proce sste x t.com/a bcchm.html
5. Diagnosis. Pyoderma gangrenos um i s a cl i ni copat hol ogi c di agnos i s of excl us i on and i s made aft er ot her caus es of cut aneous ul cerat i ons have been rul ed out . There i s no s peci fi c l aborat ory t es t , and t he hi s t opat hol ogy i s onl y s ugges t i ve, not di agnos t i c. A skin biopsy wi t h t i s s ue s ent for hi s t ol ogy and cul t ures i s requi red t o es t abl i s h a di agnos i s . Appropri at e l aborat ory s t udi es , i magi ng s t udi es , and di agnos t i c procedures (e.g., col onos copy or bone marrow biops y) s houl d be ordered t o i dent i fy any as s oci at ed di s eas es (s ee Tabl e 12-9).
6. Differential diagnosis o
o
a. Bact eri al i nfect i on (ps eudomonas or anaerobi c [cl os t ri di al ] i nfect i ons )
o
o
b. At ypi cal mycobact eri al i nfect i on
o
o
c. Deep fungal i nfect i on (Nort h Ameri can bl as t omycos i s )
o
o
d. Amebi as i s
o
o
e. Bromoderma and i ododerma
o
o
f. Rheumat oi d vas cul i t i s
Pa g e 2 9 5 5
ABC Ambe r CHM Conve rte r Tria l ve rsion, http://w w w .proce sste x t.com/a bcchm.html
TABLE 12-9 Diseases Associated with Pyoderma Gangrenosum Infl am ma t or y bo wel di s eas e: ul c era tiv e col i tis or Cro hn di s eas e Pol yar t hri tis (us ual ly sy mm
Pa g e 2 9 5 6
ABC Ambe r CHM Conve rte r Tria l ve rsion, http://w w w .proce sste x t.com/a bcchm.html
et ri c; ei t her s er on eg at i ve or s er op os i tiv e) He ma t ol ogi c di s eas es My el o id l eu ke mi as IgA mo noc l on al
Pa g e 2 9 5 7
ABC Ambe r CHM Conve rte r Tria l ve rsion, http://w w w .proce sste x t.com/a bcchm.html
ga mm op at h y My el o ma s (Ig A t yp e, pre do mi na nt l y) He pat ic di s eas es : he pat itis an d pri ma ry bi l i ary
Pa g e 2 9 5 8
ABC Ambe r CHM Conve rte r Tria l ve rsion, http://w w w .proce sste x t.com/a bcchm.html
ci rr hos is Col l ag en vas cul ar di s eas es : l up us ery t he ma t os us an d Sjà ¶gr en' s s yn dro me Ig, im mu no gl o bul i n.
Pa g e 2 9 5 9
ABC Ambe r CHM Conve rte r Tria l ve rsion, http://w w w .proce sste x t.com/a bcchm.html o
P.584
o
TABLE 12-10 Systemic Diseases Associated with Erythema Nodosum Bac t eri al i nf ect i on s (s t rep t oc occ al i nf ect i on s; mo st co mm on bac t eri al cau s e) Fun gal
Pa g e 2 9 6 0
ABC Ambe r CHM Conve rte r Tria l ve rsion, http://w w w .proce sste x t.com/a bcchm.html
i nf ect i on s (co cci di o i do my cos is; mo st co mm on fun gal i nf ect i on ) Pre gn anc y Sar coi dos is Infl am ma t or y
Pa g e 2 9 6 1
ABC Ambe r CHM Conve rte r Tria l ve rsion, http://w w w .proce sste x t.com/a bcchm.html
bo wel di s eas e Ho dg ki n' s di s eas e an d l ym ph om a Be hà §et 's di s eas e o
o
g. Brown recl us e s pi der bi t e
o
o
h. W egener's granul omat os i s
7. T herapy. Effect i ve t reat ment of t he as s oci at ed underl yi ng di s eas e mos t oft en res ul t s i n cont rol of pyoderma gangrenos um.
Pa g e 2 9 6 2
ABC Ambe r CHM Conve rte r Tria l ve rsion, http://w w w .proce sste x t.com/a bcchm.html o
o
a. Local wound care and dres s i ngs are ai med at gent l e cl eans i ng of t he ul cer bas e.
o
o
b. Surgical debridement or s urgi cal t herapy (s ki n graft i ng) i s cont rai ndi cat ed.
o
o
c. Int ral es i onal cort i cos t eroi d i nject i ons i nt o new nodul es or t he borders of s mal l ul cers may be effect i ve as an earl y i nt ervent i on.
o
o
d. Sys t emi c cort i cos t eroi ds are oft en requi red t o gai n rapi d cont rol of t he des t ruct i ve s ki n changes . Hi gh dos es of oral predni s one and i nt ravenous pul s ed met hyl predni s ol one are effect i ve. Ot her s ys t emi c t herapi es i ncl ude cycl os pori ne, t acrol i mus , mycophenol at e mofet i l , az at hi opri ne, cycl ophos phami de, chl orambuci l , daps one, t hal i domi de, and i nt ravenous i mmune gl obul i n.
E. Erythema nodosum
1. Epidemiology. The i nci dence i s unknown. Eryt hema nodos um occurs more frequent l y i n young adul t s and i n women.
2. Etiology. Thi s i mmune-medi at ed react i on may occur i n as s oci at i on wi t h s ys t emi c di s eas e (Tabl e 12-10) or adverse drug reaction (mos t commonl y oral cont racept i ves ), or i t may be idiopathic.
Pa g e 2 9 6 3
ABC Ambe r CHM Conve rte r Tria l ve rsion, http://w w w .proce sste x t.com/a bcchm.html
3. Pathophysiology. Eryt hema nodos um appears t o be del ayed-t ype hypers ens i t i vi t y react i on t arget i ng t he s ubcut aneous fat (panni cul us ). A l ymphohi s t i ocyt i c i nfi l t rat e devel ops , res ul t i ng i n nons uppurat i ve i nfl ammat ory nodul es . No ci rcul at i ng i mmunocompl exes have been found.
4. Clinical features
(onl i ne Fi gure 12-16.)
online Figure 12-16 Eryt hema nodos um. (Court es y of St uart Les s i n, MD.) o
o
a. A prodrome of fl u-l i ke s ympt oms , i ncl udi ng fever and art hral gi a, may occur.
o
o
b. Les i ons begi n as poorl y defi ned red, t ender nodul es t hat are 2–6 cm i n di amet er.
o
o
c. Les i ons are t ypi cal l y di s t ri but ed over t he
Pa g e 2 9 6 4
ABC Ambe r CHM Conve rte r Tria l ve rsion, http://w w w .proce sste x t.com/a bcchm.html
ant eri or l ower l eg but may appear on any s urface. o
o
d. Indi vi dual l es i ons devel op over 2 weeks , wi t h gradual s oft eni ng and a brui s e-l i ke appearance. The overl yi ng s ki n us ual l y des quamat es . Leg pai n and ankl e s wel l i ng may pers i s t for weeks .
o
o
e. Thi rt y percent of i di opat hi c cas es may l as t more t han 6 mont hs .
5. Diagnosis. Cl i ni cal i ns pect i on of t he s ki n and i dent i fi cat i on of pri mary s ki n l es i ons provi des a cl i ni cal di agnos i s . An i nci s i onal s ki n bi ops y wi t h adequat e s ampl i ng of t he s ubcut aneous fat confi rms t he cl i ni cal i mpres s i on and i s res erved for di agnos t i cal l y di ffi cul t cas es . Eval uat i on of an underl yi ng caus e and as s oci at ed di s orders s houl d accompany t he cl i ni cal di agnos i s .
6. T herapy. Treat ment of an i dent i fi abl e underl yi ng caus e and wi t hdrawal of any offendi ng drugs i s t he fi rs t management s t ep. In i di opat hi c or s el f-l i mi t ed cas es , s ympt omat i c rel i ef wi t h NSAIDs , compres s i on dres s i ngs , and l eg el evat i on i s us ual l y s at i s fact ory. In pers i s t ent cas es , s ys t emi c agent s s uch as col chi ci ne and l i qui d s upers at urat ed pot as s i um i odi de have proven effect i ve.
VII. dermatitis A. General considerations The t erm dermat i t i s can be defi ned as i nfl ammat i on of t he s ki n. It i s oft en us ed as a general t erm t o des cri be s ki n fi ndi ngs , s uch as “ras h―; however, i t al s o i s us ed i n t he di agnos es of s peci fi c cut aneous di s eas es . Three of t he mos t preval ent dermat i t i des are
Pa g e 2 9 6 5
ABC Ambe r CHM Conve rte r Tria l ve rsion, http://w w w .proce sste x t.com/a bcchm.html
des cri bed i n t hi s s ect i on. P.585
B. Atopic dermatitis
1. Definitions o
o
a. Atopic dermatitis (at opi c eczema or eczema) refers t o t he chroni c, rel aps i ng cut aneous mani fes t at i ons generat ed from pruri t us and s crat chi ng, as s oci at ed wi t h an underl yi ng al l ergi c predi s pos i t i on (at opy).
o
o
b. Atopy i s a heredi t ary predi s pos i t i on t oward devel opi ng hypers ens i t i vi t y react i ons , as s oci at ed wi t h el evat ed s erum i mmunogl obul i n E (IgE) l evel s , i ncl udi ng as t hma, al l ergi c rhi ni t i s , and at opi c dermat i t i s .
2. Epidemiology. At opi c dermat i t i s affect s as much as 12% of t he U.S. popul at i on.
3. Etiology. The preci s e et i ol ogy i s unknown. There i s a cl ear fami l i al pat t ern of i nheri t ance for at opi c dermat i t i s and for al l forms of at opy. St udi es s ugges t an aut os omal domi nant i nheri t ance pat t ern wi t h vari abl e penet rance. Genet i c s us cept i bi l i t y i nt eract s wi t h vari ous envi ronment al and di et ary t ri ggers .
Pa g e 2 9 6 6
ABC Ambe r CHM Conve rte r Tria l ve rsion, http://w w w .proce sste x t.com/a bcchm.html
4. Clinical features
(onl i ne Fi gure 12-17.)
online Figure 12-17 At opi c dermat i t i s . (Court es y of St uart Les s i n, MD.) o
o
a. At opi c dermat i t i s i s pri mari l y mani fes t ed i n i nfancy and chi l dhood. Approxi mat el y one-t hi rd of t he cas es pers i s t , t o s ome degree, i nt o adul t hood.
o
o
b. The flexor areas of the arms, legs, and neck are pri mari l y i nvol ved i n chi l dren and adul t s . Severe cas es may res ul t i n general i zed eryt hroderma.
o
o
c. Les i ons oft en become s econdari l y i nfect ed wi t h bact eri a from s crat chi ng and are charact eri zed by yel l ow crus t i ng wi t h i mpet i go.
Pa g e 2 9 6 7
ABC Ambe r CHM Conve rte r Tria l ve rsion, http://w w w .proce sste x t.com/a bcchm.html o
o
d. Hand and foot dermat i t i s may be t he onl y mani fes t at i on of at opi c dermat i t i s i n adul t s wi t h a hi s t ory of at opy. Fi ndi ngs i ncl ude eryt hema, s cal i ng or peel i ng, and fi s s uri ng of t he pal ms , s ol es , and fi ngers .
6. Diagnosis. At opi c dermat i t i s i s a cl i ni cal di agnos i s . Serum IgE l evel s are us ual l y el evat ed. A s ki n bi ops y may be hel pful i n confi rmi ng t he cl i ni cal di agnos i s and rul i ng out ot her di s orders .
7. T herapy o
o
a. Prevention i s effect i ve i n reduci ng acut e exacerbat i ons and compl i cat i ons .
(1) Exacerbat i ng t ri ggers s houl d be i dent i fi ed and avoi ded.
(2) Good s ki n care mai nt enance s houl d be pract i ced.
(a) Dai l y bat hi ng s houl d be recommended i n l ukewarm wat er, wi t h mi l d, moi s t uri zi ng s oaps .
(b) Us e of moi s t uri zers aft er bat hi ng i s recommended.
Pa g e 2 9 6 8
ABC Ambe r CHM Conve rte r Tria l ve rsion, http://w w w .proce sste x t.com/a bcchm.html o
o
b. T opical treatments
(1) T opical corticosteroids. Medi um- t o hi gh-pot ency t opi cal s t eroi ds t hat can be t apered t o l ow-pot ency t opi cal cort i cos t eroi ds are t he bes t t reat ment for acut e exacerbat i ons .
(2) T opical macrolide immunosuppressants. Pi mecrol i mus oi nt ment and t acrol i mus oi nt ment have been approved for t he t reat ment of at opi c dermat i t i s . Thes e cal ci neuri n i nhi bi t ors t hat are s i mi l ar t o cycl os pori ne are effect i ve s t eroi d-s pari ng t herapi es for t he t reat ment of acut e fl ares of at opi c dermat i t i s .
(3) Lubricants. The s ki n s houl d be moi s t uri zed wi t h emol l i ent s duri ng t reat ment of acut e fl ares as wel l as duri ng mai nt enance t herapy.
o
o
c. Systemic treatments
(1) Systemic corticosteroids. A s hort cours e of oral predni s one for t reat ment of acut e and s evere fl ares may be hel pful i n gai ni ng rapi d cont rol .
Pa g e 2 9 6 9
ABC Ambe r CHM Conve rte r Tria l ve rsion, http://w w w .proce sste x t.com/a bcchm.html
(2) Oral antibiotics. Ant i bi ot i cs are pres cri bed t o t reat s econdary i nfect i ons when pres ent .
(3) Antihistamines. The pruri t us of at opi c dermat i t i s i s not di rect l y medi at ed by hi s t ami ne, but H 1 and H 2 hi s t ami ne bl ockade may be effect i ve i n recal ci t rant cas es . The s edat i ve effect s of ant i hi s t ami nes t aken at bedt i me may provi de rel i ef from s crat chi ng duri ng s l eep and t hus reduce a perpet uat i ng fact or.
C. Allergic contact dermatitis
1. Definitions o
o
a. Allergic contact dermatitis i s i nfl ammat i on of t he s ki n i nduced by a del ayed-t ype hypers ens i t i vi t y (al l ergy) react i on t o a s peci fi c al l ergen t hat cont act s t he s ki n. P.586
o
o
b. Irri t ant cont act dermat i t i s i s cut aneous i nfl ammat i on i nduced by t he di rect cont act and t oxi c effect s of chemi cal s cont act i ng t he s ki n. Occupat i onal dermat i t i s refers t o al l ergi c cont act dermat i t i s or i rri t ant cont act dermat i t i s t hat occurs i n t he workpl ace.
Pa g e 2 9 7 0
ABC Ambe r CHM Conve rte r Tria l ve rsion, http://w w w .proce sste x t.com/a bcchm.html
2. Epidemiology. The i nci dence of bot h al l ergi c and
i rri t ant cont act dermat i t i s has been es t i mat ed t o be 13.6 cas es per 1000 popul at i on i n t he Uni t ed St at es . Surveys i ndi cat e t hat cont act dermat i t i s i s one of t he mos t frequent dermat ol ogi c di agnos es and repres ent s bet ween 5% and 9% of dermat ol ogy offi ce vi s i t s . Occupat i onal dermat i t i s repres ent s up t o 20% of al l report ed occupat i onal di s eas es i n t he Uni t ed St at es .
3. Etiology. Cut aneous cont act wi t h a s peci fi c al l ergen res ul t s i n al l ergi c cont act dermat i t i s i n s us cept i bl e i ndi vi dual s . Approxi mat el y 3000 chemi cal compounds have been document ed as s peci fi c caus es of al l ergi c cont act dermat i t i s .
Onl i ne Tabl e
12-11 l i s t s s ome common chemi cal s .
ONLINE TABLE 12-11 Common Allergens Frequently Inducing Allergic Contact Dermatitis Co mp ou nd s Co nta ini ng All All er er ge ge n n Uru Poi
Pa g e 2 9 7 1
ABC Ambe r CHM Conve rte r Tria l ve rsion, http://w w w .proce sste x t.com/a bcchm.html
s hi s on ol
i vy
oi l , (
poi
Rh s on us ) oak , an d poi s on su ma c Ni c Gol kel d s ul jew fat el r e y Co Ce bal me t
nt ,
di c me hl o t al ri d -pl e
at e d obj ect s, an d pai
nt s Ci n Fra na gra
Pa g e 2 9 7 2
ABC Ambe r CHM Conve rte r Tria l ve rsion, http://w w w .proce sste x t.com/a bcchm.html
mi c nce al c s oh ol Bal Foo s a ds , m
cos
of
me
Per t i c u
s, fra gra nce s, an d fl a vor i ng
s For Nai ma l l de pol hyd i s h e
, cos me tic s, pl a sti cs , an d gl u es
Pa g e 2 9 7 3
ABC Ambe r CHM Conve rte r Tria l ve rsion, http://w w w .proce sste x t.com/a bcchm.html
par Hai a
r
-Ph dye eny s , l en dye edi d am t ex i ne t i l e s, an d cos me tic s Mer Ru cap bb t ob er enz pro ot h duc i az t s ol e an d ad hes i ve s Thi Pre me s er ros vat al
i ve in con t ac t l en
Pa g e 2 9 7 4
ABC Ambe r CHM Conve rte r Tria l ve rsion, http://w w w .proce sste x t.com/a bcchm.html
s s ol ut i ons an d i nj ect abl es Ne To om pi c yci al n
ant i bi ot i cs
4. Pathophysiology. Smal l mol ecul es (hapt ens , <500 dal t ons ) bi nd carri er prot ei ns on epi dermal ant i gen-pres ent i ng cel l s (Langerhans cel l s ). Langerhans cel l s proces s ant i geni c mol ecul es and pres ent t hem t o CD4+ T l ymphocyt es (hel per T cel l s ), whi ch s t i mul at e s ecret i on of i nfl ammat ory medi at ors . Langerhans cel l s mi grat e from t he epi dermi s t o t he regi onal drai ni ng l ymph nodes and s t i mul at e prol i ferat i on and act i vat i on of ant i gen-s peci fi c T cel l s . Expanded numbers of ant i gen-s peci fi c T cel l s ci rcul at e i nt o t he s ki n and ampl i fy t he i nfl ammat ory react i on. Ini t i al s ens i t i zat i on may be acut e or chroni c. Us ual l y 10–14 days pas s aft er an i ni t i al acut e expos ure t o a s t rong cont act al l ergen before an i ndi vi dual becomes s ens i t i zed. Some i ni t i al expos ures may not res ul t i n cl i ni cal l y apparent al l ergi c cont act dermat i t i s . Once an i ndi vi dual i s s ens i t i zed t o a chemi cal , al l ergi c cont act dermat i t i s
Pa g e 2 9 7 5
ABC Ambe r CHM Conve rte r Tria l ve rsion, http://w w w .proce sste x t.com/a bcchm.html
devel ops wi t hi n hours t o s everal days of re-expos ure.
5. Clinical features
(onl i ne Fi gure 12-18.)
online Figure 12-18 Al l ergi c cont act dermat i t i s . (Court es y of St uart Les s i n, MD.) o
o
a. Acut e di s eas e i s charact eri zed by pruri t i c papul es and ves i cl es on an eryt hemat ous bas e.
o
o
b. Acut e expos ures res ul t i n cut aneous i nfl ammat i on and pruri t us l as t i ng 2–3 weeks .
o
o
c. The di s t ri but i on t ends t o be as ymmet ri cal and depends on t he nat ure of t he cont act . Li near or geomet ri c confi gurat i ons oft en are pres ent .
o
o
d. Chroni c al l ergi c cont act dermat i t i s , caus ed by chroni c expos ure t o al l ergen, may produce l i cheni fi ed, pruri t i c pl aques .
o
Pa g e 2 9 7 6
ABC Ambe r CHM Conve rte r Tria l ve rsion, http://w w w .proce sste x t.com/a bcchm.html o
e. Occas i onal l y, al l ergi c cont act dermat i t i s may general i ze and produce a di ffus e exfol i at i ve eryt hroderma.
6. Diagnosis. The di agnos i s i s us ual l y made on cl i ni cal grounds . Skin biopsy may s upport t he cl i ni cal di agnos i s i n di ffi cul t cas es . The i ni t i al s i t e of dermat i t i s oft en provi des t he bes t cl ues regardi ng t he pot ent i al caus e of di s eas e. Patch testing i s requi red t o confi rm t he di agnos i s and i dent i fy t he ext ernal chemi cal s t o whi ch t he pers on i s al l ergi c.
7. T herapy. Prevent i on i s t he mai ns t ay of t herapy. Ident i fi cat i on of t he offendi ng agent and avoi dance reduces ri s k for chroni c or recurrent dermat i t i s . o
o
a. T opical treatments
(1) Hi gh-pot ency topical corticosteroid creams are effect i ve when pres cri bed for 2-week cours es .
(2) Drying agents, s uch as cal ami ne l ot i on or cool compres s es (wi t h s al i ne or al umi num acet at e s ol ut i on), are hel pful i n s oot hi ng and dryi ng acut e, l ocal i zed ves i cul ar erupt i ons .
(3) Indi vi dual s wi t h wi des pread ves i cul ar erupt i ons may obt ai n rel i ef from cool oat meal bat hs .
o
Pa g e 2 9 7 7
ABC Ambe r CHM Conve rte r Tria l ve rsion, http://w w w .proce sste x t.com/a bcchm.html
o
b. Systemic treatments
(1) Corticosteroids. Severe or di ffus e di s eas e oft en requi res t reat ment wi t h a 2–3-week cours e of oral predni s one. Short (5-day) cours es of Sol u-Medrol “dos e packs ― are i nadequat e. W hen di s cont i nued, t hey us ual l y res ul t i n fl ari ng of s ympt oms .
(2) Oral H 1 antihistamines oft en are hel pful i n di mi ni s hi ng pruri t us .
D. Seborrheic dermatitis
1. Definition. Seborrhei c dermat i t i s (s eborrhea) i s a chroni c and recurrent papul os quamous di s order wi t h a di s t i nct i ve di s t ri but i on i nvol vi ng t he s cal p and cent ral face, ches t , axi l l ae, and groi n.
2. Epidemiology. Seborrhei c dermat i t i s i s a common di s order. P.587
3. Etiology o
o
a. The preci s e et i ol ogy i s unknown. An aberrant i mmune res pons e t o endogenous yeas t forms ( Pi t yros porum oval e) has been s pecul at ed.
Pa g e 2 9 7 8
ABC Ambe r CHM Conve rte r Tria l ve rsion, http://w w w .proce sste x t.com/a bcchm.html o
o
b. Tri ggers as s oci at ed wi t h fl ares i ncl ude s eas onal changes , t rauma (e.g., s crat chi ng), emot i onal s t res s , and medi cat i ons . Such medi cat i ons , whi ch may al s o i nduce s eborrhei c dermat i t i s , i ncl ude ps ychot ropi c medi cat i ons (e.g., chl orpromaz i ne, hal operi dol , and l i t hi um), benz odi azepi nes , ci met i di ne, et hi onami de, gol d, IFN-α, met hyl dopa, and ps oral en. In addi t i on, Parki ns on's di s eas e and acqui red i mmunodefi ci ency s yndrome (AIDS) are as s oci at ed wi t h fl ares .
4. Pathophysiology. Cut aneous i nfl ammat i on may res ul t from normal l evel s of P. oval e, rel eas i ng i nfl ammat ory free fat t y aci ds . P. oval e i s al s o abl e t o act i vat e t he al t ernat i ve compl ement pat hway i n affect ed i ndi vi dual s who have decreas ed humoral and cel l ul ar i mmune res pons es .
5. Clinical features. Seborrhei c dermat i t i s affect s adul t s but may be s een i n i nfant s (cradle cap). o
o
a. The s cal p i s us ual l y i nvol ved i n al l cas es . Di s eas e may vary from mi l d, pat chy s cal i ng (dandruff) t o wi des pread pruri t i c, eryt hemat ous , t hi ck, adherent greas y s cal es .
o
b. Di s eas e us ual l y moves from t he s cal p t o t he face, affect i ng t he eyebrows and nas ol abi al fol ds (T-zone)
(onl i ne Fi gure 12-19).
The ches t , axi l l ae, and groi n al s o may be affect ed.
Pa g e 2 9 7 9
ABC Ambe r CHM Conve rte r Tria l ve rsion, http://w w w .proce sste x t.com/a bcchm.html o
online Figure 12-19 Seborrhei c dermat i t i s . (Court es y of St uart Les s i n, MD.) o
o
c. A seborrheic blepharitis may occur, and rarel y, a general i zed exfol i at i ve eryt hroderma may devel op i n s evere cas es .
6. Diagnosis. Seborrhei c dermat i t i s i s us ual l y wel l recogni zed on cl i ni cal i ns pect i on and by hi s t ory.
7. T herapy. Mi l d cas es of s cal p i nvol vement (dandruff) res pond t o OTC s hampoos t hat cont ai n s al i cyl i c aci d, t ar, s el eni um, s ul fur, zi nc, or ket oconazol e. o
o
a. T opical corticosteroids are effect i ve for s hort -t erm t reat ment of acut e fl ares .
o
o
b. T opical antifungals are effect i ve for prol onged
Pa g e 2 9 8 0
ABC Ambe r CHM Conve rte r Tria l ve rsion, http://w w w .proce sste x t.com/a bcchm.html
t reat ment of t he s ki n.
VIII. Psoriasis A. Definition Ps ori as i s i s a chroni c, rel aps i ng s ki n di s eas e t hat mani fes t s as a papul os quamous erupt i on wi t h vari abl e cl i ni cal mani fes t at i ons , but t ypi cal l y occurs as i nfl ammat ory pl aques wi t h exces s i ve whi t e s cal i ng i n a predomi nat el y ext ens or di s t ri but i on.
B. Epidemiology Ps ori as i s affect s approxi mat el y 2%–3% of t he U.S. popul at i on, or 6.4 mi l l i on cas es . Approxi mat el y 200,000 new cas es occur annual l y. The medi an age at ons et i s 28 years .
C. Etiology Ps ori as i s appears t o be an aut oi mmune di s eas e wi t h a genet i c predi s pos i t i on. It i s mos t l y l i kel y i nheri t ed as a pol ygeni c, aut os omal domi nant di s eas e wi t h vari abl e penet rance. The preci s e t arget of t he i mmune res pons e i s unknown. HLA-B13, HLA-B17, HLA-BW 57, and HLA-CW 6 are mos t frequent l y as s oci at ed wi t h ps ori as i s . As many as one-t hi rd of pat i ent s report t hat one fami l y member i s affect ed. Ps ori as i s t ends t o fl are duri ng wi nt er mont hs and i mprove i n t he s ummer mont hs as a funct i on of s un (UV) expos ure. Drugs t hat exacerbat e ps ori as i s i ncl ude β-adrenergi c bl ockers , l i t hi um, and angi ot ens i n-convert i ng enzyme i nhi bi t ors .
D. Pathophysiology Ps ori as i s i s an i mmune-medi at ed di s eas e as s oci at ed wi t h a mi xed i nfl ammat ory dermal i nfi l t rat e t hat i ni t i at es and mai nt ai ns an i ncreas e i n t he prol i ferat i on rat e of epi dermal kerat i nocyt es i n affect ed areas . The i ncreas ed kerat i nocyt e t urnover rat e produces epi dermal t hi ckeni ng (acanthosis) and di s rupt i on of normal epi dermal di fferent i at i on, res ul t i ng i n s cal i ng. Di s eas e ons et and fl ares oft en fol l ow upper res pi rat ory i nfect i ons (e.g., s t rept ococcal pharyngi t i s ), s ugges t i ng t hat s uperant i gen act i vat i on of pat hogeni c T l ymphocyt es or mol ecul ar mi mi cry of cut aneous ant i gens pl ays a rol e i n pat hogenes i s .
Pa g e 2 9 8 1
ABC Ambe r CHM Conve rte r Tria l ve rsion, http://w w w .proce sste x t.com/a bcchm.html
E. Clinical features
1. Plaque-type psoriasis occurs as rai s ed eryt hemat ous pl aques covered wi t h a whi t e s cal e
(onl i ne Fi gure 12-20). Scrat chi ng t he
s cal e may l ead t o punct at e bl eedi ng (Aus pi t z s i gn).
online Figure 12-20 Ps ori as i s pl ague. (Court es y of St uart Les s i n, MD.)
2. The distribution of ps ori at i c l es i ons i ncl udes t he extensor surfaces of t he ext remi t i es (knees and el bows ), s cal p, and t runk (part i cul arl y t he pres acral area). P.588
3. Ps ori as i s may affect t he nails and caus e pi t t i ng of t he nai l pl at es , yel l ow di s col orat i on, and t hi ckeni ng and s eparat i on from t he nai l bed.
4. Les i ons oft en devel op at s i t es of cut aneous i njury or
Pa g e 2 9 8 2
ABC Ambe r CHM Conve rte r Tria l ve rsion, http://w w w .proce sste x t.com/a bcchm.html
t rauma (Köbner phenomenon).
5. Guttate (tear-shaped) psoriasis i s a cl i ni cal vari ant t hat devel ops rapi dl y, mos t oft en aft er a s t rept ococcal upper res pi rat ory i nfect i on. Mul t i pl e, s mal l (<1 cm) ps ori at i c l es i ons are di s t ri but ed on t he t runk and ext remi t i es .
6. Pustular psoriasis i s a t ypi cal l y s evere cl i ni cal vari ant t hat pres ent s wi t h s t eri l e pus t ul es appeari ng wi t hi n pl aques or di ffus el y. A pus t ul ar ps ori as i s fl are may be as s oci at ed wi t h hypocal cemi a.
7. Erythrodermic psoriasis i s a s evere cl i ni cal vari ant t hat pres ent s as a general i zed eryt hema wi t h ext ens i ve s cal i ng and exfol i at i on.
8. Psoriatic arthritis of varyi ng s everi t y affect s approxi mat el y 10% of ps ori at i c pat i ent s (s ee Chapt er 10 IV B 3).
F. Diagnosis Ps ori as i s i s us ual l y wel l recogni zed on cl i ni cal i ns pect i on of t he s ki n and us ual l y di s t i ngui s hed from ot her papul os quamous erupt i ons . Ski n bi ops y confi rms t he cl i ni cal i mpres s i on.
G. Therapy Treat ment i s t ai l ored t o t he s everi t y of ps ori as i s , whi ch i s us ual l y es t i mat ed by t he percent age of t he body s urface area i nvol ved. Topi cal t herapi es are us ed i n mi l d cas es , and phot ot herapy and s ys t emi c agent s are us ed for moderat e t o s evere cas es .
1. T opical agents i ncl ude kerat ol yt i c agent s (s al i cyl i c
Pa g e 2 9 8 3
ABC Ambe r CHM Conve rte r Tria l ve rsion, http://w w w .proce sste x t.com/a bcchm.html
aci d), coal t ar preparat i ons , t opi cal s t eroi ds , t opi cal vi t ami n D 3 anal ogs (cal ci pot ri ene), and t opi cal ret i noi ds . Phototherapy wi t h ps oral en pl us UV-A l i ght or UV-B l i ght i s very effect i ve i n cas es wi t h general i zed s ki n i nvol vement .
2. Systemic agents i ncl ude oral ret i noi ds (aci t ret i n), met hot rexat e (part i cul arl y for art hri t i s ), cycl os pori ne, and bi ol ogi c agent s . Bi ol ogi c agent s i ncl ude al efacept (a fus i on prot ei n t hat t arget s T-l ymphocyt e ant i gen mol ecul e CD2), et anercept (a fus i on prot ei n t hat t arget s t umor necros i s fact or), and efal i zumab (an ant i body t hat t arget s T-l ymphocyt e adhes i on mol ecul e CD11a). Systemic corticosteroids should be avoided becaus e of s evere rebound fl ares on wi t hdrawal .
IX. Pityriasis Rosea A. Definition Pi t yri as i s ros ea i s a beni gn, s el f-l i mi t ed papulosquamous exanthem wi t h a di s t i nct i ve di s t ri but i on i nvol vi ng t he trunk.
B. Epidemiology In chi l dren and young adul t s , t he es t i mat ed i nci dence i s 0.3%–3% of dermat ol ogy vi s i t s .
C. Etiology Pi t yri as i s ros ea appears t o be an infectious exanthem, but no defi ni t i ve et i ol ogi c agent has been i dent i fi ed. Cas e cl us t ers among cont act s have been report ed. There appears t o be a s eas onal predi l ect i on t o s pri ng, aut umn, and wi nt er. The rat e of recurrence i s l ow (3%). Drug-i nduced pi t yri as i s ros ea as s oci at ed wi t h s uch agent s as capt opri l , gol d, and D-peni ci l l ami ne has been des cri bed.
D. Pathophysiology An i ni t i al s ol i t ary l es i on (herald patch) devel ops i n 50%–90% of cas es , fol l owed by a s econdary crop of l es i ons t hat appear over
Pa g e 2 9 8 4
ABC Ambe r CHM Conve rte r Tria l ve rsion, http://w w w .proce sste x t.com/a bcchm.html
2–21 days . Les i ons s pont aneous l y regres s by 6 weeks .
E. Clinical features (onl i ne Fi gure 12-21.)
online Figure 12-21 Pi t yri as i s ros ea. (Court es y of Bri s t ol Myers Squi bb. Repri nt ed from Hal l JC. Sauer's Manual of Ski n Di s eas es , 8t h ed. Phi l adel phi a: Li ppi ncot t W i l l i ams & W i l ki ns , 1999:135. )
1. The herald patch t ypi cal l y meas ures 1–3 cm i n di amet er and us ual l y i s l ocat ed on t he t runk or proxi mal ext remi t i es . The pat ch i s oval or round and i s eryt hemat ous wi t h fi ne, peri pheral (col l aret t e) s cal i ng.
2. The secondary eruption i s s i mi l ar i n appearance but ranges i n s i ze from 0.5–1.0 cm. The l es i on i s s ymmet ri cal on t he t runk and fol l ows t he l i nes of
Pa g e 2 9 8 5
ABC Ambe r CHM Conve rte r Tria l ve rsion, http://w w w .proce sste x t.com/a bcchm.html
cl eavage of t he s ki n, produci ng a “ Christmas tree†• pat t ern on t he back.
3. Les i ons are us ual l y as ympt omat i c but may be pruri t i c.
P.589
F. Diagnosis Pi t yri as i s ros ea i s us ual l y wel l recogni zed on cl i ni cal i ns pect i on of t he s ki n and readi l y di s t i ngui s hed from ot her papul os quamous erupt i ons .
G. Therapy Pi t yri as i s ros ea i s a s el f-l i mi t ed di s eas e, and no s peci fi c t reat ment i s us ual l y neces s ary. Topi cal s t eroi ds may be us ed t o rel i eve pruri t us . UV radi at i on t herapy has been us ed t o has t en t he res ol ut i on of l es i ons .
X. Skin Cancers A. Basal cell carcinoma
1. Definition. Bas al cel l carci noma i s a neopl as t i c growt h of t he bas al cel l l ayer of epi dermi s .
2. Epidemiology. Bas al cel l carci noma i s t he mos t common t ype of cancer i n t he Uni t ed St at es . Approxi mat el y 1 mi l l i on new cas es occur each year.
3. Etiology. The mul t i fact ori al et i ol ogy i ncl udes genet i c s us cept i bi l i t y and envi ronment al expos ure t o UV l i ght (s unl i ght ).
Pa g e 2 9 8 6
ABC Ambe r CHM Conve rte r Tria l ve rsion, http://w w w .proce sste x t.com/a bcchm.html
4. Pathophysiology o
o
a. Neopl as t i c t rans format i on of bas al kerat i nocyt es produces downward growt h i nt o underl yi ng dermi s and deeper t i s s ues . Bas al cel l carci noma i s l ocal l y i nvas i ve wi t hout met as t at i c capaci t y.
o
o
b. Basal cell nevus syndrome (Gorl i n s yndrome) i s a rare aut os omal domi nant s yndrome wi t h germl i ne mut at i ons i n t he pt c h gene, res ul t i ng i n i ncreas ed bas al cel l carci nomas and devel opment al abnormal i t i es of t he s kel et al s ys t em, geni t ouri nary s ys t em, and cent ral nervous s ys t em.
5. Clinical features o
o
a. A pers i s t ent s ki n l es i on i s us ual l y i n t he 3–6-mm s i ze range and on s un-expos ed areas of t he s ki n.
o
b. Les i ons are fl es h col ored t o red, may be a pl aque or a papul e, and have a pearl y or t rans l ucent qual i t y
(onl i ne Fi gure 12-22).
Tel angi ect as i s may be pres ent .
Pa g e 2 9 8 7
ABC Ambe r CHM Conve rte r Tria l ve rsion, http://w w w .proce sste x t.com/a bcchm.html o
online Figure 12-22 Bas al cel l carci noma. o
o
c. Some l es i ons may be brown t o bl ack (pi gment ed bas al cel l carci noma).
o
o
d. The borders are rol l ed wi t h cent ral depres s i ons , ul cerat i on, or crus t i ng (rodent ulcer).
o
o
e. Les i ons l ocat ed on t he cent ral face and ears are at hi gh ri s k for recurrence.
o
o
f. Hi s t ol ogi c vari ant s i ncl ude morpheaform bas al cel l carci noma (s car-l i ke qual i t y and hi s t ol ogi c fi bros i s ) and mul t i cent ri c bas al cel l carci noma (i ndi s t i nct borders wi t h hi s t i ol ogi c s ki p areas ) and are at hi gh ri s k for recurrence.
6. Diagnosis. The di agnos i s i s made by cl i ni cal i ns pect i on of s ki n and s ki n bi ops y confi rmat i on.
Pa g e 2 9 8 8
ABC Ambe r CHM Conve rte r Tria l ve rsion, http://w w w .proce sste x t.com/a bcchm.html
7. T herapy o
o
a. Surgery. The cure rat e i s 90%–99%, dependi ng on t he t umor s i t e and s urgi cal t echni que. Surgi cal t echni ques i ncl ude t echni que exci s i on, Mohs mi crographi c s urgery, curet t age wi t h el ect rodes i ccat i on, and cryos urgery.
o
o
b. Radiation therapy. The cure rat e i s 93%–97%.
B. Squamous cell carcinoma
1. Definition. Squamous cel l carci noma i s a neopl as t i c growt h of t he s uprabas al cel l l ayer of epi dermi s .
2. Epidemiology. Squamous cel l carci noma i s t he s econd mos t common t ype of cancer i n t he Uni t ed St at es . Approxi mat el y 400,000 new cas es occur each year.
3. Etiology. The et i ol ogy i s mul t i fact ori al and i ncl udes genet i c s us cept i bi l i t y and envi ronment al expos ure t o UV l i ght (s unl i ght ).
4. Pathophysiology o
o
a. Neopl as t i c t rans format i on of s uprabas al kerat i nocyt es produces i nt raepi dermal (i n s i t u) foci l eadi ng t o t umor i nvas i on of underl yi ng dermi s and
Pa g e 2 9 8 9
ABC Ambe r CHM Conve rte r Tria l ve rsion, http://w w w .proce sste x t.com/a bcchm.html
deeper t i s s ues . UV-i nduced p53 gene mut at i ons are i dent i fi ed i n precurs or l es i ons (actinic keratosis), of whi ch 0.1%–1.0% progres s t o s quamous cel l carci noma i n s i t u. Squamous cel l carci nomas are l ocal l y i nvas i ve. Hi s t opat hol ogy of l es i ons progres s es from wel l di fferent i at ed t o poorl y di fferent i at ed. Met as t at i c progres s i on t o regi onal l ymph nodes occurs i n 1%–3% of cas es . P.590
o
o
b. Xeroderma pigmentosum i s an aut os omal reces s i ve s yndrome wi t h germ l i ne mut at i ons i n one of t he xeroderma pi gment os um nucl eot i de exci s i on repai r genes (XPA t hrough XPG), res ul t i ng i n phot os ens i t i vi t y and earl y ons et and i ncreas ed frequency of s ki n cancers , predomi nat el y s quamous cel l carci noma.
5. Clinical features
(onl i ne Fi gure 12-23.)
online Figure 12-23 Squamous cel l carci noma. (Court es y of St uart Les s i n, MD.) o
Pa g e 2 9 9 0
ABC Ambe r CHM Conve rte r Tria l ve rsion, http://w w w .proce sste x t.com/a bcchm.html o
a. Pers i s t ent l es i ons us ual l y are det ect ed on s un-expos ed areas of t he s ki n.
o
o
b. The red, s cal i ng pl aques have vari abl e degrees of fi rmnes s . Act i ni c kerat os es are s uperfi ci al wi t hout a pal pabl e component .
o
o
c. Large l es i ons (>1 cm) may be nodular wi t h ul cerat i on or s econdary crus t i ng.
o
o
d. A s ubs et of t hes e carci nomas are cons i dered hi gh ri s k for met as t as i s . Ri s k fact ors for met as t as i s i ncl ude l ocat i on on t he l ower l i p or ear, hi s t ol ogi c feat ures s uch as poor di fferent i at i on, i nvas i on more t han 4 mm i nt o t he dermi s , peri neural i nvas i on, occurrence i n i mmunos uppres s ed pat i ent s , and devel opment i n burn s i t es .
6. Diagnosis. The di agnos i s of s quamous cel l carci noma i s made by cl i ni cal i ns pect i on of s ki n and confi rmed by s ki n bi ops y.
7. T herapy o
o
a. Surgery. The cure rat e i s 90%–99% dependi ng on t he t umor s i t e and s urgi cal t echni que. Surgi cal t echni ques i ncl ude exci s i on, Mohs mi crographi c s urgery, curet t age wi t h el ect rodes i ccat i on, and cryos urgery.
o
Pa g e 2 9 9 1
ABC Ambe r CHM Conve rte r Tria l ve rsion, http://w w w .proce sste x t.com/a bcchm.html
o
b. Radiation therapy. The cure rat e i s 93%–97%. Prophyl act i c pos t operat i ve radi at i on t o t he s i t e may be i ndi cat ed i n hi gh-ri s k l es i ons .
C. Melanoma
1. Definition. Mel anoma i s t he neopl as t i c growt h of mel anocyt es .
2. Epidemiology o
o
a. Incidence. Cl os e t o 60,000 new cas es of mel anoma occur each year i n t he Uni t ed St at es .
o
o
b. Risk factors
(1) Genetic. Ten percent of cas es of mel anoma are fami l i al ; at l eas t one rel at i ve i s affect ed. From 1% t o 3% of cas es of mel anoma are heredi t ary; t wo or more fi rs t -degree rel at i ves are affect ed wi t h mel anoma and have a germ l i ne mut at i on i n t he CDKN2A gene.
(2) Previous melanoma. Pat i ent s who have had one mel anoma have a 5% probabi l i t y of a s econd pri mary mel anoma.
(3) Environmental. The ri s k of mel anoma
Pa g e 2 9 9 2
ABC Ambe r CHM Conve rte r Tria l ve rsion, http://w w w .proce sste x t.com/a bcchm.html
i ncreas es wi t h i ncreas ed UV expos ure, part i cul arl y wi t h an i ncreas ed number of s evere s unburns i n i ndi vi dual s younger t han 20 years of age.
(4) Pigmentation. The fol l owi ng fact ors are as s oci at ed wi t h an i ncreas ed ri s k of mel anoma.
(a) Fair complexion (readi l y s unburns and never t ans , bl ue eyes , red or bl onde hai r)
(b) Increased number of melanocytic nevi (moles) (>25)
(c) Pres ence of large melanocytic nevi (>6 mm)
(d) Pres ence of atypical (dysplastic) nevi (>5 mm wi t h varyi ng degrees of i rregul ar cl i ni cal feat ures [s ee X C 5]).
3. Etiology. The mul t i fact ori al et i ol ogy i ncl udes genet i c s us cept i bi l i t y and envi ronment al expos ure t o UV l i ght (s unl i ght ).
4. Pathophysiology. The i ni t i al s t ages of neopl as t i c t rans format i on res ul t i n upward (paget oi d) growt h of cyt ol ogi cal l y at ypi cal mel anocyt es wi t hi n epi dermi s (i n
Pa g e 2 9 9 3
ABC Ambe r CHM Conve rte r Tria l ve rsion, http://w w w .proce sste x t.com/a bcchm.html
s i t u). Tumor progres s i on res ul t s i n i nvas i on of underl yi ng dermi s and deeper t i s s ues . Regi onal l ymph node met as t as i s us ual l y precedes met as t as i s t o di s t ant s i t es , t ypi cal l y s ki n, l ung, brai n, and l i ver. In mos t cas es , BRAF gene mut at i ons are det ect ed i n earl y mel anoma t umor progres s i on.
5. Clinical features. The vas t majori t y of cut aneous mel anomas have uns t abl e cl i ni cal feat ures t hat change over t i me. Charact eri s t i c feat ures may be remembered us i ng t he mnemoni c “ABCD―
(onl i ne Fi gure 12-24).
online Figure 12-24 Mel anoma. (Court es y of St uart Les s i n, MD.) o
o
a. Asymmetry: a devi at i on i n t he overal l round t o oval confi gurat i on
o
o
b. Border: an i rregul ar ci rcumference of t he l es i on. The i ndi s t i nct margi ns may bl end i n t o t he fl es h-col ored background. P.591
o
o
c. Color: nonuni formi t y of pi gment at i on wi t h
Pa g e 2 9 9 4
ABC Ambe r CHM Conve rte r Tria l ve rsion, http://w w w .proce sste x t.com/a bcchm.html
vari at i ons i n col or, i ncl udi ng bl ack, bl ue, and red hues o
o
d. Diameter: general l y great er t han 6 mm for det ect i on
6. Diagnosis. The di agnos i s of mel anoma i s made by
cl i ni cal i ns pect i on of s ki n and confi rmed by exci s i onal bi ops y.
7. Staging
Refer t o Ameri can Joi nt Commi t t ee on Cancer
(AJCC) 2002 gui del i nes for ful l det ai l s
(Onl i ne Tabl e 12-12).
ONLINE TABLE 12-12 American Joint Committee on Cancer (AJCC) 2002 Tumor, Node, Metastasis (TNM) Classification and Stage Groupings for Melanoma Tu mo r (T ) Cla ssi fic ati on TX Pri ma
Pa g e 2 9 9 5
ABC Ambe r CHM Conve rte r Tria l ve rsion, http://w w w .proce sste x t.com/a bcchm.html
ry tu mo r can not be as s es s ed (e. g., s ha ve bi o ps y , reg res s ed pri ma ry) Ti s Mel an om a in sit u â ‰ ¤1. 0m
Pa g e 2 9 9 6
ABC Ambe r CHM Conve rte r Tria l ve rsion, http://w w w .proce sste x t.com/a bcchm.html
m T1 a: wi t ho ut ul c era tio n an d l ev el II/I II* b : wi t h ul c era tio n or l ev el IV or V* 1 .01 – 2.0 mm
Pa g e 2 9 9 7
ABC Ambe r CHM Conve rte r Tria l ve rsion, http://w w w .proce sste x t.com/a bcchm.html
T2 a: wi t ho ut ul c era tio n b : wi t h ul c era tio n 2 .01 – 4.0 mm T3 a: wi t ho ut ul c era tio n b : wi t h
Pa g e 2 9 9 8
ABC Ambe r CHM Conve rte r Tria l ve rsion, http://w w w .proce sste x t.com/a bcchm.html
ul c era tio n <4. 0 mm T4 a: wi t ho ut ul c era tio n b : wi t h ul c era tio n No de (N ) Cla ssi fic ati on
Pa g e 2 9 9 9
ABC Ambe r CHM Conve rte r Tria l ve rsion, http://w w w .proce sste x t.com/a bcchm.html
On e l ym ph no de N1 a: mi c ro me t as t as es (cl i ni c al l y occ ul t ) b : ma cro me t as t as es (cl i ni c al l y ap par
Pa g e 3 0 0 0
ABC Ambe r CHM Conve rte r Tria l ve rsion, http://w w w .proce sste x t.com/a bcchm.html
ent ) 2 – 3 l ym ph no des N2 a: mi c ro me t as t as es b : ma cro me t as t as es c : in t ra ns i t me t (s )/s at e llit
Pa g e 3 0 0 1
ABC Ambe r CHM Conve rte r Tria l ve rsion, http://w w w .proce sste x t.com/a bcchm.html
e(s ) wi t ho ut me t as t at ic l ym ph no des 4 or mo re l ym ph no des Me tas tas is (M ) Cla ssi fic ati on M1 a Di s
Pa g e 3 0 0 2
ABC Ambe r CHM Conve rte r Tria l ve rsion, http://w w w .proce sste x t.com/a bcchm.html
t an t s ki n, SQ, or l ym ph no de me t as t as es , nor ma l LD H M1 b Lun g me t as t as es , nor ma l LD H Al l ot h
Pa g e 3 0 0 3
ABC Ambe r CHM Conve rte r Tria l ve rsion, http://w w w .proce sste x t.com/a bcchm.html
er vi s cer al me t as t as es , nor ma l LD H M1 c An y di s t an t me t as t as es , el e vat ed LD H *Cl ark 's l ev el s :
Pa g e 3 0 0 4
ABC Ambe r CHM Conve rte r Tria l ve rsion, http://w w w .proce sste x t.com/a bcchm.html
l ev el II: i nv ad es t he pa pi l l ary der mi s Lev el III: i nv ad es to t he pa pi l l ary -re tic ul a r der ma l i nt erf ace Lev el
Pa g e 3 0 0 5
ABC Ambe r CHM Conve rte r Tria l ve rsion, http://w w w .proce sste x t.com/a bcchm.html
IV: i nv ad es t he ret i cul ar der mi s Lev el V: i nv ad es s ub cut an eo us tis s ue Mi c ro me t as t as es are di a gn os e d aft
Pa g e 3 0 0 6
ABC Ambe r CHM Conve rte r Tria l ve rsion, http://w w w .proce sste x t.com/a bcchm.html
er el e ct i ve or s en tin al l ym ph ad en ect om y. Ma cro me t as t as es are def i ne d as cl i n i cal ly det ect abl e l ym ph
Pa g e 3 0 0 7
ABC Ambe r CHM Conve rte r Tria l ve rsion, http://w w w .proce sste x t.com/a bcchm.html
no de me t as t as es con fi r me d by t he rap eut ic l ym ph ad en ect om y or wh en any l ym ph no de me t as t as is exh
Pa g e 3 0 0 8
ABC Ambe r CHM Conve rte r Tria l ve rsion, http://w w w .proce sste x t.com/a bcchm.html
i bi t s gro ss ext rac aps ul a r ext ens i on .
AJCC 2002 Stage Groupings for Cutaneous Melanoma Pathologic Stage
0 IA IB
Clinical Stage
Stage
Grouping*
Grouping**
Ti s , N0, M0 T1a, N0, M0 T1b, N0, M0
pTi s , N0, M0 pT1a, N0, M0 pT1b, N0, M0
T2a, N0, M0
pT2a, N0, M0
IIA
pT2a, N0, M0 T2b, N0, M0
pT2b, N0, M0
IIB
T3a, N0, M0 T3b, N0, M0
pT3a, N0, M0 pT3b, N0, M0
IIC III IIIA
T4a, N0, M0 pT4a, N0, M0 T4b, N0, M0 pT4b, N0, M0 Any T, N1–3, M0 –*** pT1-4a, N1a, M0 pT1-4a, N2a, M0 pT1-4b, N1a, M0
IIIB
–
pT1-4b, N2a, M0 pT1-4a, N1b, M0 pT1-4a, N2b, M0 pT1-4a/b, N2c, M0
Pa g e 3 0 0 9
ABC Ambe r CHM Conve rte r Tria l ve rsion, http://w w w .proce sste x t.com/a bcchm.html
IIIC
–
pT1-4b, N1b, M0 pT1-4b, N2b, M0 pT1-4b, N2c, M0
IV
Any T, Any N,
Any T, N3, M0 Any T, Any N,
Any M
Any M
*Cl i ni cal s t agi ng i ncl udes mi cros t agi ng of t he pri mary mel anoma and cl i ni cal /radi ol ogi c eval uat i on for met as t as es . By convent i on, i t s houl d be us ed aft er compl et e exci s i on of t he pri mary mel anoma wi t h cl i ni cal as s es s ment for regi onal and di s t ant met as t as es . **Pat hol ogi c s t agi ng i ncl udes mi cros t agi ng of t he pri mary mel anoma and pat hol ogi c i nformat i on about t he regi onal l ymph nodes aft er part i al or compl et e l ymphadenect omy. Pat hol ogi c St age 0 or St age 1A pat i ent s are t he except i on; t hey do not need pat hol ogi c eval uat i on of t hei r l ymph nodes . ***There are no s t age III s ubgroups for cl i ni cal s t agi ng. Us ed wi t h permi s s i on of t he Ameri can Joi nt Commi t t ee on Cancer (AJCC), Chi cago, Il l i noi s . The ori gi nal s ource for t hi s mat eri al i s t he AJCC Cancer St agi ng Manual , 6t h ed. (2002) publ i s hed by Spri nger-Verl ag New Y ork, Inc. o
Pa g e 3 0 1 0
ABC Ambe r CHM Conve rte r Tria l ve rsion, http://w w w .proce sste x t.com/a bcchm.html o
a. Four cl i ni cal s t ages are defi ned for mel anoma.
(1) St age I: s ki n i nvol vement wi t h a pri mary t umor <2 mm i n t hi cknes s
(2) St age II: s ki n i nvol vement wi t h a pri mary t umor >2 mm i n t hi cknes s
(3) St age III: any t umor wi t h l ymph node i nvol vement
(4) St age IV: met as t at i c di s eas e t o di s t ant s ki n and vi s cera
o
o
b. Sentinel lymph node mapping and biopsy i s a s urgi cal s t agi ng procedure t hat i dent i fi es t he mos t l i kel y l ymph node drai ni ng t he pri mary t umor. Thi s t echni que i s mos t oft en us ed i n mel anoma cas es wi t h pri mary t umor >1 mm i n t hi cknes s and no evi dence of l ymphadenopat hy or di s t ant met as t as i s .
8. T herapy o
o
a. Surgery
(1) Primary excision. Surgi cal exci s i on of pri mary t umor wi t h cl ear margi ns i s pri mary t herapy.
Pa g e 3 0 1 1
ABC Ambe r CHM Conve rte r Tria l ve rsion, http://w w w .proce sste x t.com/a bcchm.html
(2) Nodal dissection. El ect i ve node di s s ect i on i s cont rovers i al . Some i nves t i gat ors at t ri but e a prophyl act i c benefi t t o node di s s ect i on for l es i ons 1.50–3.99 mm i n dept h. Lymphadenect omy i s i ndi cat ed for pat i ent s i n whom adenopat hy devel ops i n t he abs ence of met as t at i c di s eas e.
o
o
b. Chemotherapy i s of l i t t l e benefi t and does not confer a s urvi val benefi t i n pat i ent s wi t h met as t at i c di s eas e. Drugs wi t h document ed act i vi t y i ncl ude dacarbazi ne, ci s pl at i n, and t he ni t ros oureas .
(1) Dacarbazine i s cons i dered t he bes t s i ngl e agent , wi t h a 15% res pons e rat e. Nei t her t amoxi fen nor IFN-α adds t o t he res pons e s een wi t h dacarbazi ne.
(2) The combination of dacarbazine, carmustine, cisplatin, and tamoxifen has been report ed t o have a 50% res pons e rat e. However, randomi zed cl i ni cal t ri al s have not confi rmed t hi s rat e.
o
o
c. Biologic response modifiers have modes t ant i t umor act i vi t y i n pat i ent s wi t h met as t at i c di s eas e.
(1) IFN-α has a 15%–20% res pons e rat e,
Pa g e 3 0 1 2
ABC Ambe r CHM Conve rte r Tria l ve rsion, http://w w w .proce sste x t.com/a bcchm.html
pri mari l y i n s oft t i s s ue and l ung met as t as es . Adjuvant IFN t herapy may i mprove t he di s eas e-free and overal l s urvi val rat es for s t age III pat i ent s and t hos e wi t h nodal met as t as es t hat are res ect ed.
(2) High-dose IL-2 has el i ci t ed l ong-t erm compl et e res pons es i n a s mal l proport i on of pat i ent s wi t h met as t at i c mel anoma. In phas e II cl i ni cal t ri al s , chemot herapy pl us IL-2 and IFN-α had hi gh res pons e rat es , but randomi zed phas e III cl i ni cal t ri al s have not confi rmed t hes e res ul t s .1(3) Ot her experi ment al procedures i ncl ude antitumor vaccines and monoclonal antibody therapy.
9. Prognosis. The Bres l ow dept h, whi ch i s t he t hi cknes s of t he mel anoma (dept h of i nvas i on), meas ured i n mi l l i met ers from t he s t rat um corneum t o t he deepes t penet rat i on of t he t umor, i s t hought t o be t he bes t prognos t i c i ndi cat or. Prognos t i c fact ors as s oci at ed wi t h i ncreas ed ri s k for mel anoma met as t as i s i ncl ude t he fol l owi ng: o
o
a. Pri mary t umor dept h of i nvas i on (>1 mm)
o
o
b. Pri mary t umor ul cerat i on
o
o
c. Vas cul ar or l ymphat i c i nvas i on
o
o
d. Age >65 years
Pa g e 3 0 1 3
ABC Ambe r CHM Conve rte r Tria l ve rsion, http://w w w .proce sste x t.com/a bcchm.html o
o
e. Pri mary t umor l ocat ed on t runk or head and neck
o
o
f. Mal e s ex
10. Prevention o
o
a. Primary prevention for mel anoma i nvol ves s un avoi dance. Recommendat i ons i ncl ude t he fol l owi ng: P.592
(1) Avoi d mi d-day s un (10:00 AM t o 2:00 PM) and art i fi ci al t anni ng.
(2) Seek s hade when pos s i bl e.
(3) Us e prot ect i ve cl ot hi ng (hat s , l ong s l eeves , and s ungl as s es ).
(4) Frequent l y appl y s uns creen wi t h a s un prot ect i on fact or (SPF) of 30 or hi gher.
o
o
b. Secondary prevention for mel anoma i nvol ves earl y det ect i on and s urvei l l ance. Recommendat i ons i ncl ude t he fol l owi ng:
Pa g e 3 0 1 4
ABC Ambe r CHM Conve rte r Tria l ve rsion, http://w w w .proce sste x t.com/a bcchm.html
(1) Perform regul ar, peri odi c s ki n s el f-exami nat i on.
(2) Have a s ki n exami nat i on and s creeni ng by a dermat ol ogi s t (hi gh-ri s k pat i ent s ).
(3) Have a bi ops y of cl i ni cal l y s us pi ci ous pi gment ed l es i ons .
D. Cutaneous T-cell lymphoma
1. Definition. Cut aneous T-cel l l ymphoma (CTCL) cons t i t ut es a cl onal prol i ferat i on of mal i gnant T l ymphocyt es (T cel l s ). CTCL begi ns as an i ndol ent l ymphoma i nvol vi ng t he s ki n but may progres s t o i nvol ve l ymph nodes , bl ood, and ot her vi s ceral organs . CTCL has t wo cl i ni cal vari ant s . o
o
a. Mycosis fungoides cl as s i cal l y evol ves s l owl y t hrough progres s i ve s t ages of cut aneous i nvol vement from t he pat ch and pl aque s t age (s t ages I–II), t hrough t he t umor or eryt hrodermi c s t age (s t age III), wi t h progres s i on t o ext racut aneous i nvol vement (s t age IV).
o
o
b. Sézary syndrome, t he more rapi dl y progres s i ng vari ant , pres ent s as s t age IV di s eas e wi t h di ffus e s ki n i nvol vement (eryt hroderma), l ymphadenopat hy, and l eukocyt os i s charact eri zed by t he pres ence of l ymphocyt es wi t h an at ypi cal
Pa g e 3 0 1 5
ABC Ambe r CHM Conve rte r Tria l ve rsion, http://w w w .proce sste x t.com/a bcchm.html
cerebri form nucl ear morphol ogy. Thes e l ymphocyt es are referred t o as Sézary cel l s .
2. Epidemiology. An es t i mat ed 1,000 new cas es of CTCL occur each year i n t he Uni t ed St at es .
3. Etiology. The et i ol ogy of CTCL i s unknown. It appears t o be an acqui red di s eas e wi t h no known ri s k fact ors , cas e cl us t eri ng, or genet i c predi s pos i t i on.
4. Pathophysiology. Infi l t rat i on of t he s ki n i s charact eri zed by t he pres ence of neopl as t i c T cel l s wi t hi n t he epi dermi s (epi dermot ropi s m) as s i ngl e cel l s or i n cl us t ers (Pautrier microabscess). Immunophenot ypi ng of t he neopl as t i c T cel l s s hows expres s i on of CD4 (hel per s ubs et ) ant i gen. The cons t el l at i on of i mmunol ogi c abnormal i t i es as s oci at ed wi t h CTCL has been cl os el y correl at ed t o aberrant Th-2 cyt oki ne expres s i on and regul at i on.
5. Clinical features o
a. Earl y ons et of l es i ons t ypi cal l y i nvol ve a “ bathing trunk― distribution, i ncl udi ng s un-prot ect ed areas : but t ocks , hi ps , axi l l ae, and women's breas t s
(onl i ne Fi gure 12-25).
Pa g e 3 0 1 6
ABC Ambe r CHM Conve rte r Tria l ve rsion, http://w w w .proce sste x t.com/a bcchm.html o
online Figure 12-25 Cut aneous T-cel l l ymphoma. (Court es y of St uart Les s i n, MD.) o
o
b. Les i ons progres s from pat ch, t o pl aque, and t o t umors or eryt hroderma, and t hey i nvol ve an i ncreas i ng percent age of t he s ki n s urface area.
(1) Patches are red, s cal i ng macul es wi t h vari abl e s i ze (1–15 cm), s hapes (round, oval , or cres cent ), and confi gurat i ons (s erpi gi nous or annul ar).
(2) Plaques are red, vari abl y i nfi l t rat ed pl aques t hat t ypi cal l y evol ve from pat ches .
(3) T umors are reddi s h t o vi ol aceous nodul es (1–15 cm) t hat oft en s uperfi ci al l y ul cerat e.
Pa g e 3 0 1 7
ABC Ambe r CHM Conve rte r Tria l ve rsion, http://w w w .proce sste x t.com/a bcchm.html
(4) Exfoliative erythroderma i s di ffus e rednes s and exfol i at i ve s cal i ng oft en as s oci at ed wi t h hyperkerat os i s of t he pal ms and s ol es and l ymphadenopat hy.
6. Diagnosis. The di agnos i s of CTCL i s made by cl i ni cal i ns pect i on of s ki n and confi rmed by s ki n bi ops y. Becaus e CTCL may be s l ow t o evol ve, s everal s ki n bi ops i es may be neces s ary t o di agnos e.
7. T herapy o
o
a. Skin-directed therapy. Thes e t herapi es are t ypi cal l y us ed i n earl y s t ages wi t h no evi dence of ext racut aneous i nvol vement . They may be combi ned wi t h s ys t emi c t herapi es i n advanced s t ages .
(1) Topi cal chemot herapy may be effect i ve. Mechl oret hami ne (ni t rogen mus t ard) or carmus t i ne are mi xed i nt o a t opi cal vehi cl e (wat er or oi nt ment ) and appl i ed di rect l y on t he s ki n.
(2) Topi cal ret i noi ds are us ed. Bexarot ene gel has been approved for t opi cal us e i n CTCL.
(3) Phot ot herapy i s us eful i n CTCL.
Pa g e 3 0 1 8
ABC Ambe r CHM Conve rte r Tria l ve rsion, http://w w w .proce sste x t.com/a bcchm.html
(4) El ect ron beam radi at i on t herapy has been s hown t o be very effect i ve i n res ol vi ng CTCL.
o
o
b. Systemic therapies are t ypi cal l y us ed i n cas es of evi dent ext racut aneous i nvol vement (bl ood, l ymph nodes , or vi s ceral organs ). P.593
(1) Bi ol ogi c res pons e modi fi ers are oft en us ed as s i ngl e agent s or i n combi nat i on.
(a) IFN-α
(b) Ext racorporeal phot opheres i s
(c) Ret i noi ds (bexarot ene)
(d) IL-2 fus i on t oxi n (deni l euki n di ft i t ox; t oxi n fus ed t o IL-2 t o s el ect i vel y expos e t oxi n onl y t o mal i gnant cel l s )
(2) Drugs us ed i n bot h s i ngl e-agent and combi nat i on chemot herapy of T-cel l non-Hodgki n's l ymphomas al s o are us ed t o t reat CTCL.
XI. Cutaneous Infections and Infestations
Pa g e 3 0 1 9
ABC Ambe r CHM Conve rte r Tria l ve rsion, http://w w w .proce sste x t.com/a bcchm.html
A. Bacterial skin infections Bacterial skin infections us ual l y s t art i n areas of t rauma or i mpai red barri er funct i on of t he s ki n. St aphyl oc oc c us aureus and St rept oc oc c us pyogenes are t he predomi nant organi s ms t hat caus e s ki n i nfect i on i n heal t hy, noni mmunocompromi s ed i ndi vi dual s .
1. Clinical presentations o
a. Impetigo i s a s uperfi ci al , i nt ra-epi dermal , hi ghl y cont agi ous ves i cul opus t ul ar, oft en oozi ng erupt i on. Les i ons have gol den crus t s
(onl i ne Fi gure 12-26). Frequency i s hi ghes t i n chi l dren. o
online Figure 12-26 Impet i go. b. Cellulitis i s an acut e s preadi ng i nfect i on of t he dermi s and s ubcut aneous t i s s ue mani fes t ed by warmt h, eryt hema, s wel l i ng, and t endernes s . S. pyogenes i nfect i ons are oft en as s oci at ed wi t h as cendi ng l ymphangi t i s
(onl i ne Fi gure 12-27).
Pa g e 3 0 2 0
ABC Ambe r CHM Conve rte r Tria l ve rsion, http://w w w .proce sste x t.com/a bcchm.html o
online Figure 12-27 Cel l ul i t i s . o
o
c. Furuncle i s a deep necrot i zi ng form of fol l i cul i t i s t hat occurs as a warm, t ender, eryt hemat ous , fl uct uant , purul ent nodul e cent ered on a hai r fol l i cl e. Several furuncl es may coal es ce, formi ng a carbuncl e.
o
o
d. Abscess i s a l ocal i zed accumul at i on of purul ent mat eri al deep i n t he dermi s or s ubcut aneous t i s s ue. The pus i s us ual l y not vi s i bl e on t he s urface of t he s ki n.
2. T herapy o
o
a. Impetigo. Compres s es t o dry t he area are neces s ary. Topi cal ant i bi ot i cs (e.g., mupi roci n) are us ed agai ns t Staphylococcus and Streptococcus. Sys t emi c ant i bi ot i cs al s o may be effect i ve.
o
o
b. Cellulitis. Sys t emi c ant i bi ot i cs are us ed agai ns t Staphylococcus and Streptococcus.
Pa g e 3 0 2 1
ABC Ambe r CHM Conve rte r Tria l ve rsion, http://w w w .proce sste x t.com/a bcchm.html o
o
c. Furuncles and abscesses. Les i ons s houl d be drai ned, and s ys t emi c ant i bi ot i c t herapy s houl d be us ed.
B. Fungal skin infections
1. Clinical presentations o
o
a. Dermatophytoses. Dermat ophyt es caus e a s uperfi ci al fungal i nfect i on of t he dead kerat i n of s ki n, hai r, and nai l s . Epi dermophyt on, Mi c ros porum, and T ri c hophyt on s peci es i nfect humans . Infect i on i s charact eri zed by eryt hemat ous and s cal i ng macul es wi t h act i ve rai s ed borders , oft en wi t h cent ral cl eari ng (ri ngworm).
(1) T inea capitis i s i nfect i on of s cal p hai r and produces al opeci a.
(2) T inea corporis i s i nfect i on of t he t runk and ext remi t i es (onl i ne Fi gure 12-28).
online Figure 12-28 Ti nea corpori s . (Court es y of St uart Les s i n,
Pa g e 3 0 2 2
ABC Ambe r CHM Conve rte r Tria l ve rsion, http://w w w .proce sste x t.com/a bcchm.html
MD.)
(3) T inea manuum and tinea pedis are i nfect i ons of t he pal ms , s ol es , and i nt erdi gi t al webs . The res ul t ant web s pace macerat i on oft en i s t he s i t e of ent ry for s econdary bact eri al cel l ul i t i s .
(4) T inea cruris i s i nfect i on of t he groi n.
(5) T inea barbae i s i nfect i on of t he beard area and neck. Mal es are t ypi cal l y affect ed.
(6) T inea faciale i s i nfect i on of t he face.
(7) T inea unguium (onychomycosis) i s i nfect i on of t he nai l and produces t hi ckened, yel l ow nai l s .
o
b. Candidiasis. Thi s s uperfi ci al yeas t i nfect i on (Candi da al bi c ans ) affect s moi s t i nt ert ri gi nous areas . Superfi ci al pus t ul es rapi dl y rupt ure, produci ng confl uent eryt hemat ous , moi s t eros i ons wi t h s cal i ng borders and s at el l i t e l es i ons on t he peri phery
(onl i ne
Fi gure 12-29).
Pa g e 3 0 2 3
ABC Ambe r CHM Conve rte r Tria l ve rsion, http://w w w .proce sste x t.com/a bcchm.html o
online Figure 12-29 Candi di as i s . (Court es y of St uart Les s i n, MD.) o
c. T inea versicolor. Endogenous yeas t forms (Malassezia furfur) caus e numerous , wel l -margi nat ed, fi nel y s cal y, oval -t o-round pi nk, hypo- or hyperpi gment ed macul es s cat t ered over t he t runk (onl i ne Fi gure 12-30). o
online Figure 12-30 Ti nea vers i col or. (Court es y of St uart Les s i n, MD.)
P.594
Pa g e 3 0 2 4
ABC Ambe r CHM Conve rte r Tria l ve rsion, http://w w w .proce sste x t.com/a bcchm.html
2. Diagnosis. KOH preparat i ons and fungal cul t ures of s cal es confi rm t he cl i ni cal di agnos i s . o
o
a. Dermatophytoses. Long-branched hyphae are apparent on KOH exami nat i on.
o
o
b. Candidiasis. Buddi ng s pores (ps eudohyphae) are apparent on KOH exami nat i on.
o
o
c. T inea versicolor. Spores and s hort hyphae (“s paghet t i and meat bal l s ―) are apparent on KOH exami nat i on.
3. T herapy o
o
a. Dermatophytoses are t reat ed wi t h t opi cal ant i fungal agent s . Ti nea capi t i s requi res oral gri s eoful vi n. Onychomycos i s res ponds bes t t o oral agent s (e.g., gri s eoful vi n, i t raconazol e, and t erbi nafi ne).
o
o
b. Candidiasis i s t reat ed wi t h t opi cal ant i fungal creams . Care s houl d be t aken t o keep areas of i nvol vement dry. Rarel y i s s ys t emi c t herapy needed.
o
o
c. T inea versicolor i s t reat ed wi t h t opi cal agent s . It al s o res ponds t o s i ngl e dos es of oral ket oconazol e.
Pa g e 3 0 2 5
ABC Ambe r CHM Conve rte r Tria l ve rsion, http://w w w .proce sste x t.com/a bcchm.html
C. Viral skin infections
1. Clinical presentations o
a. Herpes simplex virus (HSV) i nfect i ons caus e a prodrome of pai n precedi ng t he appearance of grouped ves i cl es on an eryt hemat ous bas e. Typi cal l y, HSV t ype 1 caus es herpes l abi al i s (col d s ores , fever bl i s t ers ), and HSV t ype 2 caus es geni t al herpes . HSV i nfect i ons t end t o be recurrent and may be exacerbat ed by s t res s , t he mens t rual cycl e, and i mmunos uppres s i on
(onl i ne Fi gure
12-31). o
online Figure 12-31 Herpes s i mpl ex vi rus i nfect i on. (Court es y of St uart Les s i n, MD.) o
b. Herpes zoster i nfect i ons (s hi ngl es ) caus e a prodrome of pai n radi at i ng al ong a dermat ome before an erupt i on of mul t i pl e eryt hemat ous ves i cl es , whi ch have a dermat omal di s t ri but i on. Herpes z os t er i nfect i ons , whi ch are ul t i mat el y caus ed by varicella-zoster virus (VZV), repres ent t he react i vat i on of l at ent vari cel l a vi rus (chi ckenpox) from t he dors al gangl i a of s pi nal and cent ral nerves
(onl i ne Fi gure 12-32).
Pa g e 3 0 2 6
ABC Ambe r CHM Conve rte r Tria l ve rsion, http://w w w .proce sste x t.com/a bcchm.html o
online Figure 12-32 Herpes zos t er i nfect i on. (Court es y of St uart Les s i n, MD.) o
c. Verruca vulgaris (warts) are caus ed by human papilloma virus (HPV) i nfect i on of t he epi dermi s . Fl es hed-col ored verrucous papul es t ypi cal l y occur on t he ext remi t i es
(onl i ne Fi gure 12-33).
o
online Figure 12-33 Verruca vul gari s .
(1) Verruca plana (flat warts) are 2–4-mm fl at -t opped fl es h-col ored papul es t hat affect
Pa g e 3 0 2 7
ABC Ambe r CHM Conve rte r Tria l ve rsion, http://w w w .proce sste x t.com/a bcchm.html
t he face.
(2) Condyloma acuminata are geni t al wart s t hat t end t o devel op caul i fl ower-l i ke macerat ed papul es .
2. Diagnosis o
o
a. T zanck smear det ect s mul t i nucl eat ed gi ant cel l s of bot h HSV and VZV. DIF and cul t ure of bl i s t ers i dent i fy HSV.
o
o
b. Skin biopsy confi rms t he cl i ni cal i mpres s i on of wart s i f needed and i dent i fi es l es i ons of bowenoi d papul os i s .
3. T herapy o
o
a. HSV and herpes zoster. Systemic antivirals (acycl ovi r, val acycl ovi r, and famci cl ovi r) reduce vi ral s heddi ng and t i me t o heal .
o
o
b. Warts. Therapy i ncl udes a vari et y of modal i t i es s uch as topical salicylic acid preparations, trichloroacetic acid, cryotherapy, laser therapy, curettage and electrodesiccation, and intralesional bleomycin or IFN-α. Addi t i onal t opi cal t herapi es for geni t al wart s i ncl ude podophyllin and imiquimod.
D. Rickettsial diseases
Pa g e 3 0 2 8
ABC Ambe r CHM Conve rte r Tria l ve rsion, http://w w w .proce sste x t.com/a bcchm.html
Ri cket t s i ae are pl eomorphi c bact eri a t hat are obl i gat e i nt racel l ul ar paras i t es . Trans mi t t ed t o humans by t i cks and mi t es , t hey caus e acut e s ys t emi c i nfect i ons t hat may be accompani ed by a ras h.
1. Clinical presentations. Rocky Mountain spotted fever. Ti ck-borne Rickettsia rickettsii caus es fever, myal gi as , headache, and a pet echi al ras h t hat t ypi cal l y begi ns around wri s t s and ankl es or t runk. Invol vement of t he pal ms and s ol es may occur aft er t he fi ft h day of s ympt oms .
2. Diagnosis. Serol ogy confi rms t he di agnos i s of ri cket t s i al di s eas es . Ski n bi ops y s peci mens of pet echi al l es i ons s how vas cul i t i s .
3. T herapy. Ri cket t s i al di s eas es are t reat ed wi t h ant i bi ot i c t herapy and s upport i ve t herapy. Doxycycl i ne i s t he drug of choi ce. Ri cket t s i al pox i s a s el f-l i mi t i ng di s eas e, and occas i onal l y ant i bi ot i cs may not be neces s ary.
P.595
E. Infestations
1. Clinical presentations o
a. Scabies. Thi s i nt ens el y pruri t i c and hi ghl y cont agi ous i nfes t at i on of t he epi dermi s i s caus ed by t he Sarcoptes scabiei mi t e. The mi t e burrows i nt o t he s ki n. The res ul t ant 1–3-mm burrows , papul es , ves i cl es , and s econdary excori at i ons are concent rat ed i n t he web
Pa g e 3 0 2 9
ABC Ambe r CHM Conve rte r Tria l ve rsion, http://w w w .proce sste x t.com/a bcchm.html
s paces of t he fi ngers , fl exor as pect s of t he wri s t s , ant ecubi t al fos s a, axi l l a, areol a and ni ppl es , umbi l i cus , but t ocks , and feet (onl i ne Fi gure 12-34). o
online Figure 12-34 Scabi es . (Court es y of St uart Les s i n, MD.) o
b. Pediculosis. Thi s i nfes t at i on of t he s ki n wi t h lice (ectoparasites) i s caus ed by Pediculus humanus capitis (head louse), Pediculus humanus corporis (body louse), and Pthirus pubis (pubic louse). Pedi cul os i s s preads from pers on t o pers on by cl os e phys i cal cont act or t hrough fomi t es (e.g., combs , cl ot hes , hat s , or l i nens ). Al l forms produce i t chi ng wi t h 2–4-mm eryt hemat ous papul es , excori at i ons , and oft en crus t i ng. Li ce and nits (eggs attached to hairs) are vi s i bl e on cl i ni cal i ns pect i on (Onl i ne Fi gures 12-35A and 12-35B). o
online Figure 12-35 A. Pubi c l i ce ni t s (mul t i pl e ni t s are at t ached
Pa g e 3 0 3 0
ABC Ambe r CHM Conve rte r Tria l ve rsion, http://w w w .proce sste x t.com/a bcchm.html
t o pubi c hai rs ). (Court es y of St uart Les s i n, MD.) B. Head l i ce ni t at t ached t o hai r s haft s een wi t h a hand l ens . (Court es y of Hi l ary Bal dwi n. Repri nt ed from Hal l JC. Sauer's Manual of Ski n Di s eas es , 8t h ed. Phi l adel phi a: Li ppi ncot t W i l l i ams & W i l ki ns , 1999:17. )
2. Diagnosis o
o
a. Scabies. Di agnos i s i s confi rmed wi t h mi cros copi c exami nat i on of s ki n s crapi ngs from pri mary l es i ons s howi ng s cabi es mi t es and eggs .
o
o
b. Pediculosis. Di agnos i s i s confi rmed wi t h mi cros copi c exami nat i on of ni t s and l i ce.
3. T herapy o
o
a. Scabies. Scabi ci des (e.g., permethrin) are pres cri bed for pat i ent s , hous ehol d members , and cl os e pers onal cont act s . Al l bed l i nen and cl ot hi ng s houl d be was hed i n hot wat er. Sympt omat i c t reat ment may i nvol ve ant i hi s t ami nes .
o
o
b. Pediculosis. Pedi cul i ci des (permethrin, pyrethrin, lindane shampoo or lotion) are appl i ed t opi cal l y and s houl d be reappl i ed i n 7 days . Al l cont act s s houl d be t reat ed. Bed l i nen, cl ot hi ng, and ot her mat eri al s (e.g., combs ) s houl d be was hed i n hot wat er.
XII. Cutaneous Drug Reactions
Pa g e 3 0 3 1
ABC Ambe r CHM Conve rte r Tria l ve rsion, http://w w w .proce sste x t.com/a bcchm.html
A. Definition A cut aneous drug react i on i s an advers e drug react i on i nvol vi ng t he s ki n t hat res ul t s from expos ure t o a medi cat i on by i nges t i on, t opi cal appl i cat i on, i nject i on, i ns t i l l at i on, i ns ert i on, or i nhal at i on.
B. Epidemiology As many as 5% of cours es of drug t herapy are compl i cat ed by advers e drug react i ons . Approxi mat el y 30% of hos pi t al i zed medi cal pat i ent s devel op at l eas t one advers e drug react i on duri ng t he cours e of t hei r hos pi t al i zat i on (3 mi l l i on each year i n t he Uni t ed St at es ), and 2%–3% of t hes e pat i ent s devel op s ki n react i ons (60,000–90,000 each year).
C. Etiology and pathophysiology Vari ous mechani s ms are res pons i bl e for di fferent t ypes of advers e drug react i ons .
1. Overdose. Us ual l y t he s ympt oms are an exaggerat i on of t he pharmacol ogi c act i on of t he drug (e.g., hemorrhage res ul t i ng from overdos age of ant i coagul ant s ).
2. Cumulation. Cumul at i ve effect s may occur aft er prol onged admi ni s t rat i on of s ome drugs (e.g., argyri a i n pat i ent s us i ng s i l ver-cont ai ni ng nos e drops ).
3. Pharmacologic side effects. Thes e are unwant ed but known pharmacol ogi c act i ons of drugs (e.g., al opeci a i n pat i ent s t aki ng cyt os t at i c drugs ).
4. Idiosyncrasy and intolerance
Pa g e 3 0 3 2
ABC Ambe r CHM Conve rte r Tria l ve rsion, http://w w w .proce sste x t.com/a bcchm.html o
o
a. Idiosyncrasy. Thi s qual i t at i vel y abnormal res pons e i s pres ent onl y i n cert ai n peopl e and i s not dependent on i mmunol ogi c mechani s ms .
o
o
b. Intolerance. An abnormal l y s mal l dos e of t he drug produces t he charact eri s t i c effect s of t he drug. Many of t hes e react i ons are a res ul t of di fferences i n enzymat i c cons t i t ut i on (e.g., hemol yt i c anemi a i n pat i ent s wi t h gl ucos e-6-phos phat e dehydrogenas e defi ci ency who t ake pri maqui ne).
5. Ecological imbalance. Reduci ng t he fl ora of one s peci es of mi croorgani s ms can res ul t i n overgrowt h of anot her (e.g., anogeni t al moni l i as i s fol l owi ng us e of broad-s pect rum ant i bi ot i cs ).
6. Exacerbation of existing latent or overt disease (e.g., preci pi t at i on of porphyri a by barbi t urat es )
7. Jarisch-Herxheimer reaction. Thi s res ul t s from t he admi ni s t rat i on of a drug hi ghl y effect i ve i n t he t reat ment of an exi s t i ng i nfect i on. The rel eas e of t oxi c s ubs t ances caus ed by t he des t ruct i on of t he s ens i t i ve mi croorgani s ms caus es exacerbat i ons of exi s t i ng l es i ons or t he devel opment of new ones (e.g., earl y s yphi l i s t reat ed wi t h peni ci l l i n).
8. Immunologic reactions o
Pa g e 3 0 3 3
ABC Ambe r CHM Conve rte r Tria l ve rsion, http://w w w .proce sste x t.com/a bcchm.html
o
a. IgE-dependent reactions. IgE mol ecul es fi xed t o s ens i t i zed mas t cel l s or bas ophi l s are cros s -l i nked by pol yval ent drug–prot ei n conjugat es , caus i ng rel eas e of bi ol ogi cal l y act i ve mat eri al s (e.g., hi s t ami ne, cyt oki nes , and chemoki nes ).
o
o
b. Immunocompl ex-dependent drug react i ons . Ant i gen–ant i body compl exes form 6 or more days aft er drug expos ure (“s erum s i cknes s ―) and t ypi cal l y caus e fever, art hri t i s , nephri t i s , neuri t i s , edema, and urt i cari al or papul ar ras h.
o
o
c. Cyt ot oxi c drug react i ons . Ant i gens or ant i gen–ant i body compl exes become at t ached t o cel l s urfaces , whi ch are damaged i n t he cours e of s ubs equent compl ement act i vat i on (e.g., hemol yt i c anemi a i nduced by peni ci l l i n).
o
o
d. Cel l -medi at ed drug react i ons . No ci rcul at i ng ant i bodi es are found i n t hi s t ype of i mmune res pons e (e.g., cont act drug hypers ens i t i vi t y).
D. Clinical features
1. Cut aneous drug react i ons vary i n t hei r pres ent at i on and i ncl ude a wi de array of pri mary l es i ons and s ki n fi ndi ngs (Onl i ne Tabl e 12-13).
ONLINE TABLE 12-13 Skin Manifestations
Pa g e 3 0 3 4
ABC Ambe r CHM Conve rte r Tria l ve rsion, http://w w w .proce sste x t.com/a bcchm.html
of Adverse Drug Reactions Acn Fi Pr e
x ur e it d u er s u pt io
n Al o Hi P pec rs ur ia
ut p i s ur
ma Bul H P la
y u p st er ul pi e g m e nt at io
n Di s H St col y ri ora p a tio o e n
pi g m e
Pa g e 3 0 3 5
ABC Ambe r CHM Conve rte r Tria l ve rsion, http://w w w .proce sste x t.com/a bcchm.html
nt at io n Ecz Li T em ch el a
e a n n pl gi a ec n ta u si s a
s Epi M Ur der ac t i ma ul ca l
e ri
nec
a
rol ys i s Ery N V t he ai a ma l
sc
mu d ul l t i f ys i t i or t r s me o p h y Ery P V t he a e ma p s i no ul cl
Pa g e 3 0 3 6
ABC Ambe r CHM Conve rte r Tria l ve rsion, http://w w w .proce sste x t.com/a bcchm.html
dos e e um Exf Phot ol i os e at i ns i t i ve vi t y der ma titi s
2. Some drugs are as s oci at ed wi t h an i ncreas ed frequency of a part i cul ar cut aneous react i on. However, a drug may caus e urt i cari a i n one pers on, vas cul i t i s i n anot her, and eryt hema mul t i forme i n anot her.
3. Some medi cat i ons produce cut aneous react i ons wi t h great er frequency (Tabl e 12-14).
E. Types of cutaneous drug reactions The vari ous cut aneous drug react i ons are frequent l y as s oci at ed wi t h a prot ot ype drug
(Onl i ne Tabl e 12-15).
ONLINE TABLE 12-15 Cutaneous Drug Reactions and Frequently Associated Drugs
Pa g e 3 0 3 7
ABC Amber CHM Converter Trial version, http://www.processtext.com/abcchm.html
Ass oci Dr ate ug
d
Re Dru act g(s ion ) Mor Pr bi l l ot i for ot m
y
eru p pt i e: on a m pi ci lli n Urt Pr i ca ot ri al ot eru y pt i p on e: p e ni ci lli n Li c Pr he ot noi ot d
y
eru p
Page 3038
ABC Amber CHM Converter Trial version, http://www.processtext.com/abcchm.html
pt i e: on g ol d Va Pr s cu ot l i t i ot s
y p e: al lo p ur in ol O th er s: th ia zi d e s a n d s ul fo n a m
Page 3039
ABC Amber CHM Converter Trial version, http://www.processtext.com/abcchm.html
id e s Ery Pr t he ot ma ot mu y ltif p or e: me s ul fo n a m id e s O th er s: b ar bi tu ra te s, h y d a nt oi
Page 3040
ABC Amber CHM Converter Trial version, http://www.processtext.com/abcchm.html
n s, a n d th ia zi d e s Exf Pr ol i ot at i ot ve y ery p t hr e: od di er p ma h e n yl h y d a nt oi n O th er s: s
Page 3041
ABC Amber CHM Converter Trial version, http://www.processtext.com/abcchm.html
ul fo n a m id e s, s al ic yl at e s, a n d a nt i m al ar ia ls Tox Pr ic
ot
epi ot der y ma p l
e:
nec al rol l o ys i p
Page 3042
ABC Amber CHM Converter Trial version, http://www.processtext.com/abcchm.html
s
ur in ol O th er s: b ar bi tu ra te s, h y d a nt oi n s, a n d s ul fo n a m id e
s Pi g O
Page 3043
ABC Ambe r CHM Conve rte r Tria l ve rsion, http://w w w .proce sste x t.com/a bcchm.html
me ra nt a l ry
co
al t nt era ra t i o ce ns pt iv e s (i nc re a s e d m el a ni n pr o d uc ti o n) . H e a vy m
Pa g e 3 0 4 4
ABC Ambe r CHM Conve rte r Tria l ve rsion, http://w w w .proce sste x t.com/a bcchm.html
et al s (s il v er ni tr at e d e p o si ti o n) A m io d ar o n e (d e p o si ti o n
Pa g e 3 0 4 5
ABC Amber CHM Converter Trial version, http://www.processtext.com/abcchm.html
of li p of u sc in ) Acn Pr ei f ot or ot m
y
eru p pt i e: ons co rt ic o st er oi d s O th er s: lit hi u m , di p h
Page 3046
ABC Amber CHM Converter Trial version, http://www.processtext.com/abcchm.html
e n yl h y d a nt oi n, a n d or al co nt ra ce pt iv e s Ery Pr t he ot ma ot no y dos p um e: or al co nt ra ce
Page 3047
ABC Ambe r CHM Conve rte r Tria l ve rsion, http://w w w .proce sste x t.com/a bcchm.html
pt iv e s O th er s: s ul fo n a m id e s, s al ic yl at e s, br o m id e s, a n d io di
Pa g e 3 0 4 8
ABC Amber CHM Converter Trial version, http://www.processtext.com/abcchm.html
d e s Fi x Pr ed ot dru ot g
y
eru p pt i e: on p h e n ol p ht h al ei n (c o m p o n e nt of O T C la x at
Page 3049
ABC Amber CHM Converter Trial version, http://www.processtext.com/abcchm.html
iv e s) O th er s: b ar bi tu ra te s, s ul fo n a m id e s, a n d p h e n ac et in Lup Pr us - ot
Page 3050
ABC Amber CHM Converter Trial version, http://www.processtext.com/abcchm.html
l i k ot e
y
s yn p dro e me s : h y dr al a zi n e a n d pr oc ai n a m id e Ph Pr ot o ot s en ot siti y vi t p y
e:
eru t h pt i i a ons z i d e
Page 3051
ABC Ambe r CHM Conve rte r Tria l ve rsion, http://w w w .proce sste x t.com/a bcchm.html
di ur et ic s O th er s: s ul fo n a m id e s, s ul fo n yl ur e a s, a n d p h e n ot
Pa g e 3 0 5 2
ABC Amber CHM Converter Trial version, http://www.processtext.com/abcchm.html
hi a zi n e s Dru Pr g-i ot nd ot uce y d
p
l i n e: ear v IgA a der nc ma o t os m es yc in O th er s: a m io d ar o n e, a m pi ci
Page 3053
ABC Ambe r CHM Conve rte r Tria l ve rsion, http://w w w .proce sste x t.com/a bcchm.html
lli n, a n d ca pt o pr il
1. Morbilliform eruption
(onl i ne Fi gure 12-36)
online Figure 12-36 Morbi l l i form drug erupt i on. (Court es y of St uart Les s i n, MD.) o
o
a. Thi s meas l es -l i ke erupt i on i s t he mos t common t ype of cut aneous drug react i on.
o
Pa g e 3 0 5 4
ABC Amber CHM Converter Trial version, http://www.processtext.com/abcchm.html o
b. The pri mary l es i on i s a mi nut e, eryt hemat ous macul e t hat oft en fades wi t h pres s ure. Les i ons form l arger areas of confl uence, caus i ng wi des pread and s ymmet ri c eryt hema.
2. Urticarial eruption o
o
a. Urt i cari a i s t he s econd mos t common t ype of cut aneous drug react i on (s ee Chapt er 7 IV).
o
o
b. Drugs may caus e urt i cari a by i mmunol ogi c (IgE-dependent or i mmune compl ex-dependent ) or noni mmunol ogi c (di rect hi s t ami ne l i berat i on) mechani s ms . P.596
TABLE 12-14 Drugs Frequently Associated with Cutaneous Reactions Re acti on Rat e (pe r Dr 100 ug 0) Tri 5 me 9
Page 3055
ABC Amber CHM Converter Trial version, http://www.processtext.com/abcchm.html
t ho pri ms ul fa me t ho xaz ol e Am 5 pi ci 2 llin Se 3 mi s 6 ynt het ic pe ni ci llin s Cor 2 tic 8 ot r opi n Ery 2 t hr 3 om yci n Sul 1 fi s 7 oxa z ol e
Page 3056
ABC Ambe r CHM Conve rte r Tria l ve rsion, http://w w w .proce sste x t.com/a bcchm.html
Pe 1 ni ci 6 llin G Ge 1 nt a 6 mi c in s ul fat e Pra 1 ct o 6 l ol Ce 1 ph 3 al o s po ri n s Qui 1 ni d 2 i ne
3. Lichenoid eruption. Drugs may caus e l i chen pl anus -l i ke erupt i ons (s ee VI B). Mucos al i nvol vement i s rare.
4. Vasculitis
(onl i ne Fi gure 12-37.)
Pa g e 3 0 5 7
ABC Ambe r CHM Conve rte r Tria l ve rsion, http://w w w .proce sste x t.com/a bcchm.html
online Figure 12-37 Vas cul i t i s (Henoch-Schönl ei n purpura). o
o
a. Vas cul i t i s occurs as pal pabl e purpura as a res ul t of bl ood ves s el damage and ext ravas at i ons of eryt hrocyt es i nt o t he dermi s .
o
o
b. Pat i ent s oft en pres ent wi t h vas cul i t i s i n dependent areas (e.g., l egs ).
o
o
c. Al l opuri nol i s t he prot ot ype caus al drug. Thi azi des and s ul fonami des al s o may caus e vas cul i t i s .
5. Erythema multiforme (s ee VI A). Sul fonami des are t he prot ot ype caus al drugs . Barbi t urat es , hydant oi ns , and t hi azi des al s o may caus e eryt hema mul t i forme.
6. Exfoliative erythroderma.
(onl i ne Fi gure 12-38.) Exfol i at i ve
eryt hroderma occurs as a general i zed eryt hema wi t h exfol i at i ve s cal i ng. Mucous membranes are us ual l y s pared.
Pa g e 3 0 5 8
ABC Ambe r CHM Conve rte r Tria l ve rsion, http://w w w .proce sste x t.com/a bcchm.html
online Figure 12-38 Exfol i at i ve dermat i t i s due t o a Di l ant i n drug erupt i on. (Repri nt ed from Hal l JC. Sauer's Manual of Ski n Di s eas es , 8t h ed. Phi l adel phi a: Li ppi ncot t W i l l i ams & W i l ki ns , 1999:239. )
7. T oxic epidermal necrolysis (TEN)
(onl i ne Fi gure 12-39). TEN
occurs as a ful mi nat i ng, general i zed s l oughi ng of s ki n wi t h compl et e necros i s of epi dermi s comparabl e t o s econd-degree burn. Mucous membranes are s everel y affect ed. Mort al i t y i s hi gh (s ee VI A 4 c).
online Figure 12-39 Toxi c epi dermal necrol ys i s . (Court es y of St uart Les s i n, MD.)
8. Pigmentary alterations. Thes e changes may occur i n res pons e t o a medi cat i on t hrough a vari et y of
Pa g e 3 0 5 9
ABC Ambe r CHM Conve rte r Tria l ve rsion, http://w w w .proce sste x t.com/a bcchm.html
mechani s ms . o
a. Increas ed mel ani n product i on oft en res ul t s i n macul ar hyperpi gment at i on of t he face t ermed melasma
(onl i ne Fi gure
12-40.) and i s oft en as s oci at ed wi t h oral cont racept i ve us e. o
online Figure 12-40 Mel as ma (drug-i nduced hyperpi gment at i on). (Court es y of St uart Les s i n, MD.) o
o
b. Depos i t i on of pi gment ed mat eri al s res ul t s from heavy met al s (e.g., s i l ver ni t rat e).
o
o
c. Depos i t i on of l i pofus ci n res ul t s from ami odarone.
9. Acneiform eruption
(onl i ne Fi gure 12-41).
Pa g e 3 0 6 0
ABC Ambe r CHM Conve rte r Tria l ve rsion, http://w w w .proce sste x t.com/a bcchm.html
online Figure 12-41 St eroi d acne. (Court es y of St uart Les s i n, MD.) o
o
a. Drug-i nduced acne has a rapi d ons et of acnei form l es i ons i n a s i mi l ar s t age of devel opment i nvol vi ng areas ot her t han t he face.
o
o
b. Acnei form erupt i ons may occur at any age.
10. Erythema nodosum (s ee VI E).
11. Fixed drug eruption
(onl i ne Fi gure 12-42.)
online Figure 12-42 Fi xed drug erupt i on. (Court es y of St uart Les s i n, MD.)
Pa g e 3 0 6 1
ABC Ambe r CHM Conve rte r Tria l ve rsion, http://w w w .proce sste x t.com/a bcchm.html o
o
a. A fi xed drug erupt i on produces a demarcat ed oval or ci rcul ar eryt hema t hat heal s wi t h res i dual hyperpi gment at i on. Les i ons t end t o be s ol i t ary but may be mul t i pl e.
o
o
b. Re-expos ure t o t he offendi ng drug caus es l es i ons t o devel op i n t he s ame l ocat i on.
12. Lupus-like syndrome
(onl i ne Fi gure 12-43). The cut aneous
fi ndi ngs of drug-i nduced l upus are us ual l y i ndi s t i ngui s habl e from s ys t emi c l upus eryt hemat os us .
online Figure 12-43 Drug-i nduced l upus . (Court es y of St uart Les s i n, MD.)
13. Photosensitivity eruptions
(onl i ne Fi gure 12-44).
Pa g e 3 0 6 2
ABC Ambe r CHM Conve rte r Tria l ve rsion, http://w w w .proce sste x t.com/a bcchm.html
online Figure 12-44 Drug-i nduced phot os ens i t i vi t y react i on. (Court es y of St uart Les s i n, MD.) o
o
a. Phot os ens i t i vi t y react i ons requi re t he pres ence of bot h l i ght and t he i nci t i ng drug.
o
o
b. Speci fi c wavel engt hs of l i ght known as t he “act i on s pect rum― (285–425 nm) are requi red t o i ni t i at e a phot os ens i t i ve drug react i on.
o
o
c. A phototoxic reaction (noni mmunol ogi c) res ul t s from di rect cel l ul ar i njury when a drug i s phot oact i vat ed by l i ght . P.597
Pa g e 3 0 6 3
ABC Ambe r CHM Conve rte r Tria l ve rsion, http://w w w .proce sste x t.com/a bcchm.html o
o
d. A photoallergic reaction (i mmune medi at ed) res ul t s when l i ght i nt eract s wi t h a drug t o produce an i mmunogeni c i nt ermedi at e or met abol i t e t hat s t i mul at es an i mmune res pons e.
14. Drug-induced linear IgA dermatosis (LAD)
(onl i ne Fi gure
12-45.)
online Figure 12-45 Drug-i nduced l i nea IgA dermat os i s . o
o
a. The het erogeneous pruri t i c l es i ons i ncl ude t ens e bul l ae. There i s a predomi nat el y ext ens or di s t ri but i on wi t h mucos al i nvol vement i n approxi mat el y 70% of cas es .
o
o
b. DIF of s ki n bi ops y s peci mens s hows l i near depos i t s of IgA al ong t he bas ement membrane t hat are i dent i cal t o t hos e found i n i di opat hi c cas es of LAD of adul t and chi l dhood.
o
Pa g e 3 0 6 4
ABC Ambe r CHM Conve rte r Tria l ve rsion, http://w w w .proce sste x t.com/a bcchm.html o
c. IgA aut oant i bodi es t arget col l agen XVII (BPAG 2) and t ype VII col l agen.
F. Diagnosis A cut aneous drug react i on s houl d be s us pect ed i n any wi des pread ras h of rapi d ons et .
1. It i s cri t i cal t o obt ai n a compl et e drug hi s t ory. o
o
a. As k s peci fi cal l y about t oni cs , l axat i ves , s edat i ves , t ranqui l i zers , pai n medi cat i ons , vi t ami ns , oral cont racept i ve pi l l s , eye drops , i nhal ant s , i mmuni zat i ons , and s uppos i t ori es .
o
o
b. In hos pi t al i zed pat i ent s , check t he pat i ent 's chart for medi cat i on orders , i ncl udi ng “prn― medi cat i ons ; “s t at ― orders ; anes t het i cs ; and di agnos t i c agent s (e.g., cont ras t medi a and radi oact i ve t racers ).
2. Det ermi ne whi ch drug i s t he mos t l i kel y caus al agent . o
o
a. Drugs s t art ed wi t hi n 1 week before t he appearance of t he erupt i on are mos t l i kel y caus al .
o
o
b. Drugs wi t h a hi gh frequency of cut aneous react i ons are mos t l i kel y caus al .
o
o
c. Drugs commonl y caus i ng t he morphol ogi c pat t ern pres ent cl i ni cal l y s houl d be s us pect ed.
Pa g e 3 0 6 5
ABC Ambe r CHM Conve rte r Tria l ve rsion, http://w w w .proce sste x t.com/a bcchm.html
3. A s ki n bi ops y may as s i s t i n maki ng a di agnos i s . Al t hough t here i s no s i ngl e hi s t ol ogi c fi ndi ng di agnos t i c of a drug erupt i on, ot her caus es of t he erupt i on may be rul ed out .
G. Therapy
1. The caus al drug s houl d be di s cont i nued i f pos s i bl e. Some cl i ni cal s i t uat i ons prevent di s cont i nuat i on of t he caus al drug.
2. Topi cal or s ys t emi c cort i cos t eroi ds (for rel at i vel y s evere erupt i ons ) may produce s ympt omat i c rel i ef.
3. Ant i hi s t ami nes may hel p rel i eve as s oci at ed pruri t us .
4. Topi cal care cons i s t s of s oot hi ng bat hs or compres s es and appropri at e dres s i ngs . W hen pres ent , s econdary i nfect i ons s houl d be t reat ed.
XIII. Selected Cutaneous Manifestations of Systemic Diseases A. General considerations The s ki n i s oft en affect ed by s ys t emi c di s eas es . Cut aneous mani fes t at i ons of s ys t emi c di s eas es may devel op concurrent l y as a di s eas e progres s es or may be one of t he earl y s i gns and s ympt oms l eadi ng t o an i ni t i al di agnos i s . Recogni t i on of as s oci at ed cut aneous mani fes t at i ons ai ds i n t he di agnos i s and management of many s ys t emi c di s eas es .
B. Stasis dermatitis
Pa g e 3 0 6 6
ABC Ambe r CHM Conve rte r Tria l ve rsion, http://w w w .proce sste x t.com/a bcchm.html
1. Definition. Stasis dermatitis i s a common i nfl ammat ory s ki n di s eas e t hat occurs on t he l ower ext remi t i es i n i ndi vi dual s wi t h chroni c venous i ns uffi ci ency. St as i s dermat i t i s i s t he earl i es t s ki n change as s oci at ed wi t h venous i ns uffi ci ency and may l ead t o venous l eg ul cerat i on and fi bros i s .
2. Epidemiology. The es t i mat ed preval ence of s t as i s dermat i t i s /l i podermat os cl eros i s i s approxi mat el y 6%–7% i n i ndi vi dual s ol der t han 50 years of age.
3. Etiology. Venous i ns uffi ci ency i s t he underl yi ng caus e of t he cut aneous changes s een i n s t as i s dermat i t i s /l i podermat os cl eros i s .
4. Pathophysiology o
o
a. Decreas ed compet ency of t he one-way val vul ar s ys t em i n t he deep venous pl exus of t he l egs res ul t s i n backfl ow of bl ood from t he deep venous s ys t em t o t he s uperfi ci al venous s ys t em, produci ng venous hypert ens i on. The l i nk bet ween venous hypert ens i on and cut aneous changes i s not preci s el y unders t ood.
o
o
b. Venous hypert ens i on produces i ncreas ed fl ow rat es and hi gh oxygen t ens i on, cont rary t o earl y t heori es t hat hypot hes i zed t hat an i ncompet ent venous s ys t em l ead t o pool i ng of bl ood and
Pa g e 3 0 6 7
ABC Amber CHM Converter Trial version, http://www.processtext.com/abcchm.html
reduced bl ood fl ow and oxygen t ens i on i n dermal capi l l ari es . P.598
o
o
c. Increas ed venous hydros t at i c pres s ure i ncreas es permeabi l i t y of t he dermal mi croci rcul at i on and res ul t s i n t he format i on of fi bri n cuffs around dermal capi l l ari es . However, fi bri n cuffs have not been found t o decreas e oxygen di ffus i on s i gni fi cant l y.
o
o
d. Venous hypert ens i on has been s hown t o res ul t i n l eukocyt e t rappi ng i n t he mi croci rcul at i on t hat may res ul t i n t he i ncreas ed rel eas e of i nfl ammat ory medi at ors and l eukocyt e s l udgi ng t hat cont ri but e t o t i s s ue i s chemi a.
5. Clinical features
(onl i ne Fi gure 12-46.)
Page 3068
ABC Ambe r CHM Conve rte r Tria l ve rsion, http://w w w .proce sste x t.com/a bcchm.html
online Figure 12-46 St as i s dermat i t i s . o
o
a. Acut e and chroni c changes , i ncl udi ng edema, vari cos i t i es , and di ffus e red-brown di s col orat i on repres ent i ng dermal depos i t s of hemos i deri n (from degraded, ext ravas at ed eryt hrocyt es ), may occur agai ns t a background of s ki n changes .
o
o
b. Cl i ni cal changes occur bi l at eral l y. The medi al ankl e i s mos t frequent l y and s everel y i nvol ved.
o
o
c. Earl y fi ndi ngs i ncl ude eryt hemat ous s cal i ng of a l ower ext remi t y.
o
o
d. Exudat i ve, weepi ng pat ches , and pl aques may be as s oci at ed wi t h s econdary i nfect i on.
o
Pa g e 3 0 6 9
ABC Ambe r CHM Conve rte r Tria l ve rsion, http://w w w .proce sste x t.com/a bcchm.html o
e. Ul cerat i on may occur and general l y t ends t o be medi al wi t h exudat i ve bas es . Heal i ng res ul t s i n s carri ng.
o
o
f. Li cheni fi cat i on and hyperpi gment at i on may occur as a cons equence of chroni c s crat chi ng and rubbi ng. Indurat i on, fi bros i s , and s carri ng may res ul t wi t h cons t ri ct i on around t he ankl es , produci ng an i nvert ed bowl i ng pi n appearance i n t he l ower l eg.
6. Diagnosis. Cl i ni cal i ns pect i on and venous Doppl er s t udi es es t abl i s h t he di agnos i s of s t as i s dermat i t i s /l i podermat os cl eros i s i n t he s et t i ng of venous i ns uffi ci ency. Ski n bi ops i es are rarel y i ndi cat ed but rul e out ot her di agnos es , es peci al l y when ul cerat i on i s pres ent .
7. T herapy o
o
a. Topi cal t herapy i s di rect ed at t he rel i ef of s i gns and s ympt oms . Treat ment s i ncl ude compres s es for weepi ng l es i ons , l ocal wound care for ul cerat i ons , and s hort -t erm us e of mi d-pot ency t opi cal cort i cos t eroi ds for reduci ng s ympt omat i c i nfl ammat i on and i t chi ng.
o
o
b. Obvi ous s uperfi ci al i nfect i ons (i mpet i gi ni zat i on) s houl d be t reat ed wi t h t opi cal mupi roci n or a s ys t emi c ant i bi ot i c wi t h act i vi t y agai ns t St aphyl oc oc c us and St rept oc oc c us s peci es .
o
Pa g e 3 0 7 0
ABC Ambe r CHM Conve rte r Tria l ve rsion, http://w w w .proce sste x t.com/a bcchm.html o
c. Mai nt enance t herapy requi res us e of moi s t uri zers t o prevent drynes s and breakdown of t he s ki n.
o
o
d. Compres s i on s t ocki ngs and el evat i on of l egs at res t are i mport ant mai ns t ays of mai nt enance and prevent i on. Art eri al i ns uffi ci ency i s oft en pres ent i n pat i ent s wi t h venous i ns uffi ci ency; t herefore, as s es s i ng t he pat i ent 's peri pheral art eri al ci rcul at i on wi t h a Doppl er s t udy i s advi s ed before recommendi ng compres s i on t herapy.
C. Cutaneous sarcoidosis
1. Definition. Sarcoi dos i s i s charact eri zed by noncas eat i ng epi t hel i oi d granul omas t hat pri mari l y affect t he l ungs . The l ymphat i c (es peci al l y pul monary l ymph nodes ), opht hal mi c, nervous , mus cul os kel et al , hepat i c, cardi ac, renal , and endocri ne s ys t ems are al s o affect ed. In addi t i on, t he s ki n i s oft en affect ed, and recogni t i on and eval uat i on of t hes e s ki n l es i ons oft en ai ds i n es t abl i s hi ng t he di agnos i s .
2. Epidemiology. The i nci dence of s arcoi dos i s i s 1–40 per 100,000 popul at i on i n t he Uni t ed St at es . It i s more preval ent among Afri can Ameri cans t han among Caucas i ans .
3. Etiology. The exact et i ol ogy of s arcoi dos i s has not been cl earl y defi ned. Immunogenet i c s us cept i bi l i t y and envi ronment al expos ure i nfl uence di s eas e expres s i on. Et i ol ogi c agent s may i ncl ude i nfect i ons , envi ronment al agent s , or aut oant i gens .
Pa g e 3 0 7 1
ABC Ambe r CHM Conve rte r Tria l ve rsion, http://w w w .proce sste x t.com/a bcchm.html
4. Pathophysiology. Impai red humoral and cel l ul ar res pons es have been document ed i n pat i ent s wi t h s arcoi dos i s . Chroni c ant i gen expos ure may l ead t o a chroni c Th1 cyt oki ne–dri ven res pons e, whi ch res ul t s i n granul oma format i on.
5. Clinical features
(onl i ne Fi gure 12-47.)
online Figure 12-47 Cut aneous s arcoi dos i s . o
o
a. Macul ar or papul ar s arcoi dos i s i s t he mos t common l es i on i n cut aneous s arcoi dos i s and i s charact eri zed by as ympt omat i c red-brown macul es and papul es oft en i nvol vi ng t he face, peri orbi t al , nas ol abi al fol ds , or ext ens or s urfaces .
o
o
b. Lupus pernio i s t he mos t di s t i nct i ve of s arcoi d s ki n l es i ons and i s charact eri zed by reddi s h t o vi ol aceous i ndurat ed pl aques and nodul es t hat us ual l y affect t he nos t ri l s , cheeks , ears , and l i ps . P.599
o
Pa g e 3 0 7 2
ABC Ambe r CHM Conve rte r Tria l ve rsion, http://w w w .proce sste x t.com/a bcchm.html
o
c. Pl aque s arcoi dos i s i s charact eri zed by round t o oval , red-brown t o vi ol aceous i nfi l t rat ed pl aques , whi ch are oft en di s t ri but ed s ymmet ri cal l y and have an annul ar appearance. The cent er of t he pl aques may be at rophi c or s cal y.
o
o
d. Subcut aneous nodul ar s arcoi dos i s (Dari er-Rous s y s arcoi dos i s ) i s charact eri zed by nont ender, fi rm, fl es h-col ored or vi ol aceous 0.5–2-cm nodul es t hat are commonl y found on t he ext remi t i es or on t he t runk.
o
o
e. Scars from previ ous t rauma may become i nfi l t rat ed and t ender wi t h a reddi s h or vi ol aceous col or.
o
o
f. Eryt hema nodos um i s a hypers ens i t i vi t y react i on t hat may devel op i n t he s et t i ng of s arcoi dos i s . It i s oft en as s oci at ed wi t h hi l ar l ymphadenopat hy, ant eri or uvei t i s , or pol yart hri t i s (Löfgren s yndrome).
6. Diagnosis. Cl i ni cal i ns pect i on and s ki n bi ops y demons t rat i ng di agnos t i c noncas eat i ng granul omas are t he bas i s of di agnos i s . Addi t i onal s ki n bi ops y s peci mens s houl d be s ent for s peci al s t ai ns and cul t ures t o rul e out i nfect i ous caus es of granul oma format i on, i ncl udi ng mycobact eri al and deep fungal i nfect i ons .
7. T herapy. Treat ment of underl yi ng mani fes t at i ons i s paramount .
Pa g e 3 0 7 3
ABC Ambe r CHM Conve rte r Tria l ve rsion, http://w w w .proce sste x t.com/a bcchm.html o
o
a. Topi cal or i nt ral es i onal cort i cos t eroi ds (t ri amci nol one acet oni de) may be us ed t o t reat cut aneous l es i ons .
o
o
b. To avoi d s carri ng, s ys t emi c i mmunos uppres s ant s are us ed for recal ci t rant l es i ons t hat do not res pond t o i nt ral es i onal cort i cos t eroi ds . Immunos uppres s ant s for cut aneous s arcoi dos i s i ncl ude met hot rexat e, azat hi opri ne, and chl orambuci l . Ot her report edl y us eful t herapeut i c agent s i ncl ude ant i mal ari al drugs (hydroxychl oroqui ne and chl oroqui ne), cycl os pori ne, oral i s ot ret i noi n, al l opuri nol , and t hal i domi de.
D. Dermatitis herpetiformis
1. Definition. Dermat i t i s herpet i formi s i s an aut oi mmune bl i s t eri ng s ki n di s eas e as s oci at ed wi t h gl ut en-s ens i t i ve ent eropat hy (GSE).
2. Epidemiology. The es t i mat ed frequency of dermat i t i s herpet i formi s i n t he Uni t ed St at es i s approxi mat el y 39 cas es per 100,000 popul at i on.
3. Etiology. Depos i t s of IgA i n t he s ki n l ead t o pruri t i c bl i s t eri ng i n pat i ent s wi t h GSE.
4. Pathophysiology o
Pa g e 3 0 7 4
ABC Ambe r CHM Conve rte r Tria l ve rsion, http://w w w .proce sste x t.com/a bcchm.html o
a. Pat hogenes i s i s as s oci at ed wi t h t he pres ence of GSE, an i ncreas ed expres s i on of HLA-A1, HLA-B8, HLA-DR3, and HLA-DQ2 hapl ot ypes and granul ar depos i t i on of IgA at t he DEJ of t he s ki n.
o
o
b. Sus cept i bl e i ndi vi dual s , as det ermi ned by HLA hapl ot ype, may devel op a cel l -medi at ed i mmune res pons e res ul t i ng i n T-cel l act i vat i on i n t he s mal l bowel mucos a exacerbat ed by a s ens i t i vi t y t o gl ut en, a prot ei n pres ent i n barl ey, rye, and wheat but not i n ri ce.
o
o
c. Cut aneous i nfl ammat i on i s predomi nat ed by a dermal i nfi l t rat e of neut rophi l s wi t h mi croabs ces s es format i on and progres s i on t o s ubepi dermal ves i cl e format i on t hrough t he l ami na l uci da of t he bas ement membrane zone. DIF s hows a granul ar depos i t i on of IgA.
5. Clinical features o
o
a. Ski n l es i ons are ext remel y pruri t i c groups of eryt hemat ous ves i cl es , mos t frequent l y l ocat ed on ext ens or s urfaces .
o
o
b. Les i ons oft en are pus t ul ar or crus t ed from excori at i on.
6. Diagnosis. Ski n bi ops y and DIF confi rm t he di agnos i s .
Pa g e 3 0 7 5
ABC Ambe r CHM Conve rte r Tria l ve rsion, http://w w w .proce sste x t.com/a bcchm.html
7. T herapy. Di ami nodi phenyl s ul fone and s ul fapyri di ne are t he pri mary drugs us ed i n t reat ment . Rel i ef of s ympt oms may be s een wi t hi n 24–48 hours of t he s t art of t herapy. Pat i ent s may choos e t o cont rol t he s ki n di s eas e wi t h a gl ut en-free di et .
E. Calciphylaxis
1. Definition. Cal ci phyl axi s i s t he phenomenon of vas cul ar cal ci fi cat i on and s ki n necros i s i n end-s t age renal di s eas e (ESRD).
2. Epidemiology. Cal ci phyl axi s i s ext remel y rare i n t he general popul at i on, affect i ng 1%–4% of pat i ent s wi t h ESRD. P.600
3. Etiology. The preci s e et i ol ogy i s uncert ai n, but hypercal cemi a, hyperphos phat emi a, el evat ed cal ci um-phos phat e product s , and s econdary hyperparat hyroi di s m as s oci at ed wi t h ESRD are cont ri but i ng fact ors . In t he abs ence of renal di s eas e, t here are cas e report s of occurrence of cal ci phyl axi s i n as s oci at i on wi t h pri mary hyperparat hyroi di s m, ci rrhos i s , and rheumat oi d art hri t i s .
4. Pathophysiology o
o
a. The pat hogenes i s i s poorl y defi ned. Vas cul ar cal ci fi cat i on i s a cons t ant fi ndi ng and may
Pa g e 3 0 7 6
ABC Ambe r CHM Conve rte r Tria l ve rsion, http://w w w .proce sste x t.com/a bcchm.html
s ens i t i ze t he vas cul ar mi croenvi ronment t o hypercoagul abi l i t y or promot e i nt i mal hyperpl as i a and vas cul ar occl us i on. o
o
b. Al t ernat i vel y, ext ens i ve endot hel i al cal ci fi cat i on and i nt i mal hyperpl as i a, whi ch are known t o compromi s e t he l umi nal s i ze of ves s el s i n cal ci phyl axi s , may res ul t i n vas cul ar occl us i on. Thes e mechani s ms remai n hypot het i cal and have not yet been proven t o l ead t o cal ci phyl axi s .
5. Clinical features o
o
a. Earl y l es i ons of cal ci phyl axi s devel op s uddenl y and progres s rapi dl y as nons peci fi c vi ol aceous mot t l i ng or as eryt hemat ous papul es , pl aques , or nodul es .
o
o
b. Les i ons progres s wi t h a s t el l at e purpuri c confi gurat i on wi t h cent ral necros i s and are ext remel y pai nful and t ender.
o
o
c. Les i ons may be s i ngul ar or numerous .
o
o
d. Les i ons may occur ei t her di s t al l y on t he l ower l egs or proxi mal l y i n areas of body fat , s uch as t he l ower abdomen, t hi ghs , and but t ocks .
6. Diagnosis. The di agnos i s i s confi rmed by i nci s i onal s ki n bi ops y wi t h adequat e s ampl i ng of t he s ubcut aneous
Pa g e 3 0 7 7
ABC Ambe r CHM Conve rte r Tria l ve rsion, http://w w w .proce sste x t.com/a bcchm.html
t i s s ue. Laborat ory s t udi es eval uat i ng renal , endocri ne, and coagul at i on s t at us s houl d be ordered.
7. T herapy. Local t reat ment i nvol ves wound care wi t h debri dement t o avoi d wound i nfect i on and s eps i s . Hyperbari c oxygen t herapy may be us eful .
F. Porphyria Cutanea Tarda
1. Definition. Porphyri a cut anea t arda (PCT) i s a phot os ens i t i ve bl i s t eri ng di s order t hat res ul t s from a heredi t ary or acqui red defi ci ency i n t he act i vi t y of t he hepat i c heme s ynt het i c enzyme uroporphyri nogen decarboxyl as e (URO-D).
2. Epidemiology. The exact i nci dence of PCT i n t he Uni t ed St at es i s not known but i s es t i mat ed t o be approxi mat el y 3 per 100,000 peopl e. PCT i s t he mos t common t ype of porphyri a.
3. Etiology. Germ l i ne mut at i ons i n t he gene encodi ng URO-D are det ect ed i n i ndi vi dual s affect ed wi t h fami l i al forms of PCT. Cl i ni cal expres s i on of bot h t he fami l i al and acqui red forms oft en requi res expos ure t o hepat ot oxi c agent s or condi t i ons i ncl udi ng et hanol , es t rogens , exces s i ron s t ores , hepat i t i s C vi rus , and human i mmunodefi ci ency vi rus (HIV).
4. Pathophysiology. Reduced hepat i c URO-D act i vi t y res ul t s i n overproduct i on of uroporphyri n (URO) and coproporphyri n (COPRO). Bot h URO and COPRO are wat er s ol ubl e and are excret ed i n el evat ed l evel s i n t he uri ne.
Pa g e 3 0 7 8
ABC Ambe r CHM Conve rte r Tria l ve rsion, http://w w w .proce sste x t.com/a bcchm.html
In addi t i on, t hey are phot oact i ve mol ecul es and abs orb energy from vi s i bl e l i ght and produce a phot ot oxi c react i on t hat res ul t s i n i ncreas ed mechani cal fragi l i t y of t he s ki n aft er s unl i ght expos ure.
5. Clinical features
(onl i ne Fi gure 12-48.)
online Figure 12-48 Porphyri a cut anea t arda. (Court es y of Texas Pharmaceut i cal s . Repri nt ed from Hal l JC. Sauer's Manual of Ski n Di s eas es , 8t h ed. Phi l adel phi a: Li ppi ncot t W i l l i ams & W i l ki ns , 1999:239. ) o
o
a. Fragi l i t y of s un-expos ed s ki n aft er mechani cal t rauma i s t he mos t common fi ndi ng. Thi s l eads t o eros i ons and t ens e bul l ae on t he dors al as pect s of t he hands , forearms , and face.
o
o
b. Heal i ng of crus t ed eros i ons and bl i s t ers l eaves s cars , mi l i a (t i ny s ubepi dermal kerat i nous cys t s ), and hyperpi gment ed and hypopi gment ed at rophi c pat ches .
o
Pa g e 3 0 7 9
ABC Ambe r CHM Conve rte r Tria l ve rsion, http://w w w .proce sste x t.com/a bcchm.html
o
c. Hypert ri chos i s and hyperpi gment at i on i s oft en vi s i bl e over t he t emporal and mal ar faci al areas .
o
o
d. Scl eroderma-l i ke changes may occur over t he preauri cul ar face, neck, ches t and t he back. Thes e s cl erodermoi d pl aques can devel op dys t rophi c cal ci fi cat i on.
o
o
e. A uri ne s ampl e i s oft en gros s l y di s col ored wi t h a t ea- or wi ne-col ored t i nt .
6. Diagnosis o
o
a. The di agnos i s i s confi rmed by a hi s t ory of phot os ens i t i vi t y, charact eri s t i c s ki n fi ndi ngs , a s ki n bi ops y for hi s t ol ogy and DIF (whi ch i s negat i ve for i mmunoreact ant s ), and el evat ed URO and COPRO l evel s i n a 24-hour uri ne col l ect i on. P.601
o
o
b. Any underl yi ng et i ol ogy or as s oci at ed di s order s houl d be eval uat ed wi t h l aborat ory s t udi es , i ncl udi ng hemat ol ogi c and i ron profi l es , l i ver funct i on t es t s , and s creeni ng for hepat i t i s and HIV.
7. T herapy o
Pa g e 3 0 8 0
ABC Ambe r CHM Conve rte r Tria l ve rsion, http://w w w .proce sste x t.com/a bcchm.html o
a. Exacerbat i ng fact ors s houl d be el i mi nat ed or mi ni mi zed. Such meas ures i ncl ude avoi dance of s unl i ght , el i mi nat i on of al cohol , and di s cont i nuat i on of es t rogen when pos s i bl e.
o
o
b. Therapeut i c phl ebot omy i s oft en s ucces s ful i n reduci ng exces s i ron s t ores i n t i s s ue, whi ch i s fol l owed by i mprovement of deregul at ed heme s ynt hes i s due t o URO-D i nhi bi t i on. One uni t of whol e bl ood i s removed weekl y every 2–3 weeks , as t ol erat ed by t he pat i ent .
o
o
c. W hen phl ebot omy i s cont rai ndi cat ed, l ow dos es of chl oroqui ne phos phat e or hydroxychl oroqui ne s ul fat e may be pres cri bed. Thes e drugs di s pl ace hepat i c i ron s t ores . Large dos es can caus e s evere hepat ot oxi ci t y.
P.602
Study Questions/Answers and Explanations 1. A middle-aged man who has recently begun taking captopril and hydrochlorothiazide for hypertension develops an itchy rash in sun-exposed areas. T he most likely cutaneous diagnosis is A. Urt i cari a B. Eryt hema mul t i forme C. Phot os ens i t i vi t y react i on D. Eryt hema nodos um E. Vas cul i t i s Vi ew Ans wer 1. T he answer is C [XII E 13]. Hydrochlorot hi azi de i s as s oci at ed wi t h a hi gh frequency of phot os ens i t i vi t y. Capt opri l , an angi ot ens i n-convert i ng enz yme i nhi bi t or, i s as s oci at ed wi t h a hi gh
Pa g e 3 0 8 1
ABC Ambe r CHM Conve rte r Tria l ve rsion, http://w w w .proce sste x t.com/a bcchm.html
frequency of papul os quamous s ki n react i ons . The cl i ni cal fi ndi ngs do not s upport t he di agnos es of urt i cari a, eryt hema mul t i forme, eryt hema nodos um, or vas cul i t i s . 2. An adolescent girl presents with a 1-year history of acne that she has been treating with over-the-counter (OT C) medications. Examination shows comedones and an occasional inflammatory papule on the face. She inquires about isotretinoin and would like to know whether it would be appropriate for her acne. You inform her that isotretinoin is indicated for the treatment of which of the following types of acne? A. Comedonal acne B. Mi l d i nfl ammat ory acne C. Moderat e i nfl ammat ory acne D. Nodul ocys t i c acne E. St eroi d acne Vi ew Ans wer 2. T he answer is D. [IV A 7 b (2)]. Is ot ret i noi n i s onl y i ndi cat ed for t he t reat ment of s evere and recal ci t rant nodul ocys t i c acne. Thi s agent s houl d not be pres cri bed for mi l der forms of acne. St eroi d acne res ponds t o t he di s cont i nuat i on of s ys t emi c s t eroi ds and t opi cal t ret i noi n. 3. A young man presents with acute allergic contact dermatitis to poison ivy. Which of the following formulations is likely to be the most therapeutically effective? A. Low-pot ency cort i cos t eroi d cream B. Hi gh-pot ency cort i cos t eroi d cream C. Mi d- t o hi gh-pot ency cort i cos t eroi d s ol ut i on D. Mi d- t o hi gh-pot ency cort i cos t eroi d oi nt ment E. Sys t emi c cort i cos t eroi d t reat ment . Vi ew Ans wer T he answers are: 3-B[III] Bot h t he pot ency and t he vehi cl e of a t opi cal cort i cos t eroi d mus t be appropri at e for t he s peci fi c cl i ni cal s et t i ng. Hi gh-pot ency t opi cal s t eroi ds are us ed for t he s hort -t erm t reat ment of acut e and s evere i nfl ammat ory erupt i ons . A cream has dryi ng effect s , and a hi gh-pot ency s t eroi d cream i s wel l s ui t ed for
Pa g e 3 0 8 2
ABC Ambe r CHM Conve rte r Tria l ve rsion, http://w w w .proce sste x t.com/a bcchm.html
t reat ment of a ves i cul ar al l ergi c cont act dermat i t i s . An oi nt ment has moi s t uri zi ng effect s , and a mi d- t o hi gh-pot ency s t eroi d oi nt ment i s hi ghl y effect i ve for t reat i ng dry, s cal y ps ori at i c pl agues . Sol ut i ons and l ot i ons are t he vehi cl e of choi ce for t he s cal p; t herefore, a mi d- t o hi gh-pot ency s t eroi d s ol ut i on i s effect i ve for t reat i ng a fl are of s cal p ps ori as i s . Low-pot ency t opi cal s t eroi ds are t he onl y s t eroi ds us ed on t he face. The dryi ng effect s of a cream are des i rabl e for a greas y, s cal i ng fl are of s eborrhei c dermat i t i s i n t hi s l ocat i on. Sys t emi c cort i cos t eroi ds are not neces s ary except i n more s evere and wi des pread dermat ol ogi c cas es . 4. A young woman develops an acute flare of plaque-type psoriasis on her trunk and extremities. Which of the following formulations is likely to be the most therapeutically effective? A. Low-pot ency cort i cos t eroi d cream B. Hi gh-pot ency cort i cos t eroi d cream C. Mi d- t o hi gh-pot ency cort i cos t eroi d s ol ut i on D. Mi d- t o hi gh-pot ency cort i cos t eroi d oi nt ment E Systemic corticosteroid treatment Vi ew Ans wer 4-D[III] Bot h t he pot ency and t he vehi cl e of a t opi cal cort i cos t eroi d mus t be appropri at e for t he s peci fi c cl i ni cal s et t i ng. Hi gh-pot ency t opi cal s t eroi ds are us ed for t he s hort -t erm t reat ment of acut e and s evere i nfl ammat ory erupt i ons . A cream has dryi ng effect s , and a hi gh-pot ency s t eroi d cream i s wel l s ui t ed for t reat ment of a ves i cul ar al l ergi c cont act dermat i t i s . An oi nt ment has moi s t uri zi ng effect s , and a mi d- t o hi gh-pot ency s t eroi d oi nt ment i s hi ghl y effect i ve for t reat i ng dry, s cal y ps ori at i c pl agues . Sol ut i ons and l ot i ons are t he vehi cl e of choi ce for t he s cal p; t herefore, a mi d- t o hi gh-pot ency s t eroi d s ol ut i on i s effect i ve for t reat i ng a fl are of s cal p ps ori as i s . Low-pot ency t opi cal s t eroi ds are t he onl y s t eroi ds us ed on t he face. The dryi ng effect s of a cream are des i rabl e for a greas y, s cal i ng fl are of s eborrhei c dermat i t i s i n t hi s l ocat i on. Sys t emi c cort i cos t eroi ds are not neces s ary except i n more s evere and wi des pread dermat ol ogi c cas es .
Pa g e 3 0 8 3
ABC Ambe r CHM Conve rte r Tria l ve rsion, http://w w w .proce sste x t.com/a bcchm.html
5. A middle-aged man develops an acute flare of scalp psoriasis. Which of the following formulations is likely to be the most therapeutically effective? A. Low-pot ency cort i cos t eroi d cream B. Hi gh-pot ency cort i cos t eroi d cream C. Mi d- t o hi gh-pot ency cort i cos t eroi d s ol ut i on D. Mi d- t o hi gh-pot ency cort i cos t eroi d oi nt ment E. Systemic corticosteroid treatment Vi ew Ans wer 5-C[III] Bot h t he pot ency and t he vehi cl e of a t opi cal cort i cos t eroi d mus t be appropri at e for t he s peci fi c cl i ni cal s et t i ng. Hi gh-pot ency t opi cal s t eroi ds are us ed for t he s hort -t erm t reat ment of acut e and s evere i nfl ammat ory erupt i ons . A cream has dryi ng effect s , and a hi gh-pot ency s t eroi d cream i s wel l s ui t ed for t reat ment of a ves i cul ar al l ergi c cont act dermat i t i s . An oi nt ment has moi s t uri zi ng effect s , and a mi d- t o hi gh-pot ency s t eroi d oi nt ment i s hi ghl y effect i ve for t reat i ng dry, s cal y ps ori at i c pl agues . Sol ut i ons and l ot i ons are t he vehi cl e of choi ce for t he s cal p; t herefore, a mi d- t o hi gh-pot ency s t eroi d s ol ut i on i s effect i ve for t reat i ng a fl are of s cal p ps ori as i s . Low-pot ency t opi cal s t eroi ds are t he onl y s t eroi ds us ed on t he face. The dryi ng effect s of a cream are des i rabl e for a greas y, s cal i ng fl are of s eborrhei c dermat i t i s i n t hi s l ocat i on. Sys t emi c cort i cos t eroi ds are not neces s ary except i n more s evere and wi des pread dermat ol ogi c cas es . 6. A middle-aged woman develops an acute flare of seborrheic dermatitis affecting the nasolabial folds of her face. Which of the following formulations is likely to be the most therapeutically effective? A. Low-pot ency cort i cos t eroi d cream B. Hi gh-pot ency cort i cos t eroi d cream C. Mi d- t o hi gh-pot ency cort i cos t eroi d s ol ut i on D. Mi d- t o hi gh-pot ency cort i cos t eroi d oi nt ment E. Sys t emi c cort i cos t eroi d t reat ment Vi ew Ans wer 6-A[III] Bot h t he pot ency and t he vehi cl e of a t opi cal cort i cos t eroi d
Pa g e 3 0 8 4
ABC Ambe r CHM Conve rte r Tria l ve rsion, http://w w w .proce sste x t.com/a bcchm.html
mus t be appropri at e for t he s peci fi c cl i ni cal s et t i ng. Hi gh-pot ency t opi cal s t eroi ds are us ed for t he s hort -t erm t reat ment of acut e and s evere i nfl ammat ory erupt i ons . A cream has dryi ng effect s , and a hi gh-pot ency s t eroi d cream i s wel l s ui t ed for t reat ment of a ves i cul ar al l ergi c cont act dermat i t i s . An oi nt ment has moi s t uri zi ng effect s , and a mi d- t o hi gh-pot ency s t eroi d oi nt ment i s hi ghl y effect i ve for t reat i ng dry, s cal y ps ori at i c pl agues . Sol ut i ons and l ot i ons are t he vehi cl e of choi ce for t he s cal p; t herefore, a mi d- t o hi gh-pot ency s t eroi d s ol ut i on i s effect i ve for t reat i ng a fl are of s cal p ps ori as i s . Low-pot ency t opi cal s t eroi ds are t he onl y s t eroi ds us ed on t he face. The dryi ng effect s of a cream are des i rabl e for a greas y, s cal i ng fl are of s eborrhei c dermat i t i s i n t hi s l ocat i on. Sys t emi c cort i cos t eroi ds are not neces s ary except i n more s evere and wi des pread dermat ol ogi c cas es . P.603
7. A 40-year-old woman develops painful ulcers in her mouth followed by a blistering eruption on the trunk. Examination shows erosions on the soft palate and pharynx as well as 0.5–1.5-cm crusted superficial erosions and flaccid bullae on the trunk. Skin biopsy reveals epidermal acantholysis and IgG intercellular staining. T he most likely diagnosis is A. Bul l ous pemphi goi d B. Pemphi gus vul gari s C. Paraneopl as t i c pemphi gus D. Eryt hema mul t i forme E. Porphyri a cut anea t arda Vi ew Ans wer 7. T he answer is B. [V B]. Pemphi gus vul gari s t ypi cal l y affect s mi ddl e-aged i ndi vi dual s and oft en pres ent s wi t h dys phagi a and oral i nvol vement . Bul l ae are fl acci d, and di rect i mmunofl uores cence s hows IgG i nt ercel l ul ar s t ai ni ng. Convers el y, bul l ous pemphi goi d rarel y i nvol ves oral mucos a and us ual l y pres ent s wi t h t ens e bul l ae wi t h l i near IgG and C3 al ong t he dermal –epi dermal junct i on.
Pa g e 3 0 8 5
ABC Ambe r CHM Conve rte r Tria l ve rsion, http://w w w .proce sste x t.com/a bcchm.html
Paraneopl as t i c pemphi gus i s charact eri zed by s evere oral i nvol vement and het erogeneous s ki n l es i ons wi t h hi s t ol ogi c and di rect i mmunofl uores cent feat ures of bot h pemphi gus and pemphi goi d. Eryt hema mul t i forme us ual l y i s charact eri zed by t arget -l i ke l es i ons on t he pal ms and s ol es and di s t i nct hi s t opat hol ogy. Porphyri a cut anea t arda i s charact eri zed by phot os ens i t i vi t y and t ens e bul l ae i n a phot o di s t ri but i on and i s as s oci at ed wi t h el evat ed uri nary porphyri ns . 8. A 35-year-old woman with a history of Crohn' s disease develops a painful nodule on her lower leg that soon ulcerates. Examination shows an extremely tender, deep, 8 × 10-cm ulceration with a purulent base and erythematous undermined borders. T he most likely diagnosis is A. Eryt hema nodos um B. St as i s dermat i t i s C. Pyoderma gangrenos um D. Vas cul i t i s E. Cal ci phyl axi s Vi ew Ans wer 8. T he answer is C. [VI D]. The cl i ni cal fi ndi ngs of a deep purul ent and pai nful ul cer are t ypi cal of pyoderma gangrenos um, whi ch oft en i s as s oci at ed wi t h i nfl ammat ory bowel di s eas e. Eryt hema nodos um i s not an ul cerat i ve proces s . St as i s dermat i t i s may be ul cerat i ve but not t o t he ext ent of pyoderma gangrenos um. Pal pabl e purpura i s t he hal l mark of vas cul i t i s (s evere cas es may res ul t i n ul cerat i on). Cal ci phyl axi s produces necrot i c s ki n l es i ons . 9. A middle-aged man presents with a red rash on the abdomen. Examination shows a 5-cm erythematous scaling plaque with central clearing. A potassium hydroxide preparation from a scraping of the scales shows branched hyphae. T he most accurate diagnosis is A. Candi di as i s B. Ti nea vers i col or C. Herpes s i mpl ex D. Ti nea capi t i s
Pa g e 3 0 8 6
ABC Ambe r CHM Conve rte r Tria l ve rsion, http://w w w .proce sste x t.com/a bcchm.html
E. Ti nea corpori s Vi ew Ans wer 9. T he answer is E. [XI B]. Dermat ophyt e i nfect i on of t he body (t i nea corpori s ) t ypi cal l y produces annul ar l es i ons t hat , on pot as s i um hydroxi de (KOH) preparat i on, s how branched hyphae. Candi di as i s produces pus t ul es t hat form coal es cent areas of eryt hema, t ypi cal l y i n i nt ert ri gi nous areas , and KOH exami nat i on s hows buddi ng s pores . Ti nea vers i col or produces confl uent , s cal i ng macul es on t he t runk, and KOH exami nat i on s hows s pores and nonbranched hyphae. Herpes s i mpl ex i s a bl i s t eri ng erupt i on, and a Tz anck s mear s hows mul t i nucl eat ed gi ant cel l s . Ti nea capi t i s i s a dermat ophyt e i nfect i on t hat affect s t he s cal p. 10. An elderly woman with Parkinson' s disease presents with a red scaling rash on the face that has waxed and waned for several months. Examination shows mild erythema with greasy scales predominantly over the eyebrows and nasolabial folds with extension onto the malar surfaces of the face. T he most likely diagnosis is A. Ps ori as i s B. Ti nea vers i col or C. Lupus eryt hemat os us D. Seborrhei c dermat i t i s E. Al l ergi c cont act dermat i t i s Vi ew Ans wer 10. T he answer is D. [VII D]. Greas y s cal es al ong t he T-zone of t he face are t ypi cal of s eborrhei c dermat i t i s . Fl ares of s eborrhei c dermat i t i s are s een i n pat i ent s wi t h Parki ns on's di s eas e. Ps ori as i s and t i nea vers i col or rarel y affect t he face. Lupus eryt hemat os us i s charact eri zed by a more promi nent mal ar eryt hema and does not s i gni fi cant l y i nvol ve t he eyebrows and nas ol abi al fol d. Al l ergi c cont act dermat i t i s i s t ypi cal l y charact eri zed by l i near feat ures t o i t s confi gurat i on. 11. A 55-year-old man presents with a chronic rash over the buttocks and hips that has been unresponsive to topical steroids. It has recently started to itch. Examination shows
Pa g e 3 0 8 7
ABC Ambe r CHM Conve rte r Tria l ve rsion, http://w w w .proce sste x t.com/a bcchm.html
6–12-cm erythematous, scaling plaques in a “ bathing trunk― distribution. Potassium hydroxide preparation is negative for evidence of a fungal infection. A skin biopsy indicates an atypical lymphocytic infiltrate with evidence of epidermotropism and Poutier microabscess formation. T he mostly diagnosis is A. Impet i go B. Ps ori as i s C. Cut aneous T-cel l l ymphoma (mycos i s fungoi des ) D. Ti nea corpori s E. At opi c dermat i t i s Vi ew Ans wer 11. T he answer is C. [X D]. A papul os quamous erupt i on i n a bat hi ng t runk di s t ri but i on can be s een wi t h cut aneous T-cel l l ymphoma, t i nea corpori s , or pos s i bl y ps ori as i s . The s ki n bi ops y s hows di agnos t i c changes wi t h at ypi cal l ymphocyt es i nfi l t rat i ng t he epi dermi s (epi dermot ropi s m) and formi ng cl us t ers wi t hi n t he epi dermi s (Paut ri er mi croabs ces s ). Impet i go t ypi cal l y has gol den crus t s as a predomi nant feat ure. At opi c dermat i t i s predomi nant l y affect s fl exor areas . 12. A 48-year-old woman with a fair complexion presents with a 4-mm pearly papule with a central crust located on the medial canthus of the right eye. A skin biopsy shows a well-circumscribed basal cell carcinoma. Which of the following clinical features makes the risk of recurrence of basal cell carcinoma high? A. Si z e of t he l es i on B. Fai r compl exi on C. Hi s t ol ogi c feat ures D. Locat i on of t he l es i on E. Gender Vi ew Ans wer 12. T he answer is D. [X, A]. The l ocat i on of a bas al cel l carci noma on t he medi al cant hus of t he eye det ermi nes where a l es i on i s at hi gh ri s k for recurrence. The cent ral face (eyes and nos e) and ears
Pa g e 3 0 8 8
ABC Ambe r CHM Conve rte r Tria l ve rsion, http://w w w .proce sste x t.com/a bcchm.html
are hi gh-ri s k areas for recurrence of bas al cel l carci noma. A fai r compl exi on i s as s oci at ed wi t h an i ncreas ed i nci dence of bas al cel l carci noma, bot h hi gh- and l ow-ri s k l es i ons , and i s not a cl i ni cal feat ure s peci fi c for hi gh-ri s k bas al cel l carci noma. Hi s t ol ogi c feat ures s uch as fi bros i s (morpheaform bas al cel l carci noma) and s ki p areas (mul t i cent ri c bas al cel l carci noma) are as s oci at ed wi t h hi gh ri s k of recurrence for bas al cel l carci noma. Gender i s not a fact or i n det ermi ni ng t he ri s k of bas al cel l carci noma. P.604
13. A 65-year-old man with a fair complexion presents with a 9-mm hyperkeratotic, erythematous plaque on the left jaw and no regional lymphadenopathy. A skin biopsy shows a poorly differentiated squamous cell carcinoma of skin with evidence of perineural invasion. Which of the following clinical features makes the risk of metastasis in squamous cell carcinoma high? A. Si z e of t he l es i on B. Fai r compl exi on C. Hi s t ol ogi c feat ures D. Locat i on of t he l es i on E. Gender Vi ew Ans wer 13. T he answer is C. [X, B]. Two hi s t ol ogi c feat ures of a s quamous cel l carci noma t hat are as s oci at ed wi t h hi gh ri s k for met as t as i s are poor di fferent i at i on and peri neural i nvas i on. Ot her fact ors s uch as l ocat i on on t he l ower l i p and s i ze great er t han 1 cm al s o cont ri but e t o met as t as i s . In t hi s pat i ent , t he l es i on i s on t he l eft jaw, and i t i s jus t l es s t han 1 cm i n di amet er. Gender and compl exi on do not i ncreas e t he ri s k for met as t as i s . 14. A mi ddl e-aged woman pres ent s wi t h an i t chy erupt i on over her arms and t runk s everal days aft er gardeni ng. Ski n exami nat i on s hows t he changes s een i n
The mos t l y l i kel y di agnos i s i s
Pa g e 3 0 8 9
ABC Ambe r CHM Conve rte r Tria l ve rsion, http://w w w .proce sste x t.com/a bcchm.html
Online Unnfig 12-Q14. (Courtesy of Stuart Lessin, MD.) A. Al l ergi c cont act dermat i t i s B. Herpes s i mpl ex i nfect i on C. Bul l ous pemphi goi d D. At opi c dermat i t i s E. Vas cul i t i s Vi ew Ans wer 14. T he answer is A. [VII C]. The phot o demons t rat es eryt hemat ous macul es and ves i cl es i n a di s t i nct i ve l i near confi gurat i on. Such a confi gurat i on i s t ypi cal of a cont act dermat i t i s . Grouped ves i cl es , t ypi cal of herpes s i mpl ex i nfect i on, are not s een. Tens e bul l ae, t ypi cal of bul l ous pemphi goi d, i s not pres ent . Fl exor i nvol vement , t ypi cal of at opi c dermat i t i s , i s not s een. There i s no evi dence of pal pabl e purpura, t ypi cal of vas cul i t i s . 15. An el derl y man pres ent s wi t h a bl i s t eri ng erupt i on over t he l eft t horaci c back, fl ank, and ches t . It devel oped s hort l y aft er he began t o experi ence i nt ens e burni ng and pai n i n t he affect ed areas . Exami nat i on s hows grouped ves i cl es on an eryt hemat ous bas e i n a dermat omal di s t ri but i on over t he l eft t ors o. A Tzanck s mear from one of t he ves i cl es s hows t he fi ndi ngs s hown i n t he
. The mos t l y
l i kel y di agnos i s i s
Pa g e 3 0 9 0
ABC Ambe r CHM Conve rte r Tria l ve rsion, http://w w w .proce sste x t.com/a bcchm.html
Online Unnfig 12-Q15. (Courtesy of Stuart Lessin, MD.) P.605
A. Scabi es mi t e i nfes t at i on B. Pemphi gus vul gari s as s oci at ed wi t h acant hol yt i c cel l s C. Herpes zos t er as s oci at ed wi t h mul t i nucl eat ed gi ant cel l s D. Herpes s i mpl ex as s oci at ed wi t h mul t i nucl eat ed gi ant cel l s E. Impet i go as s oci at ed wi t h gram-pos i t i ve cocci Vi ew Ans wer 15. T he answer is C. [XI C 1 a]. The phot o demons t rat es a Tzanck s mear (s t ai ned wi t h Gi ems a s t ai n) of s crapi ng from bl i s t ers s howi ng mul t i nucl eat ed gi ant cel l s . Mul t i nucl eat ed gi ant cel l s can be s een on Tzanck s mears of herpes zos t er and herpes s i mpl ex. The pat i ent has a t ypi cal pres ent at i on of herpes zos t er becaus e of t he cl i ni cal hi s t ory of neuri t i c pai n as s oci at ed wi t h bl i s t eri ng i n a dermat omal di s t ri but i on. Scabi es can be vi s ual i zed wi t h s crapi ng of web-s pace burrows from pat i ent s wi t h s cabi es . Acant hol yt i c cel l s can be vi s ual i zed when s crapi ng bl i s t ers of pemphi gus vul gari s . A Gram s t ai n of i mpet i go reveal s gram-pos i t i ve cocci of St aphyl oc oc c us or St rept oc oc c us s peci es . 16. A mi ddl e-aged woman compl ai ns of a s evere s unburn 1 day aft er goi ng t o t he beach and 2 days aft er i ni t i at i ng a cours e of oral s ul famet hoxaz ol e-t ri met hopri m for a bl adder i nfect i on. Ski n
Pa g e 3 0 9 1
ABC Ambe r CHM Conve rte r Tria l ve rsion, http://w w w .proce sste x t.com/a bcchm.html
exami nat i on s hows changes s hown i n t he
. The mos t l y l i kel y
di agnos i s i s
Online Unnfig 12-Q16. (Courtesy of Stuart Lessin, MD.) A. Li chen pl anus B. At opi c dermat i t i s C. Seborrhei c dermat i t i s D. Ps ori as i s E. Phot os ens i t i vi t y react i on Vi ew Ans wer 16. T he answer is E. [XII E 13]. The phot o demons t rat es an eryt hemat ous macul opapul ar erupt i on wi t h a s harp and di s cret e border, out l i ni ng t he V of t he neck, i ndi cat i ve of a phot odi s t ri but i on. The hi s t ory i ndi cat es t hat t he pat i ent was s t art ed on s ul famet hoxaz ol e, whi ch has a hi gh frequency of phot os ens i t i ve react i ons . The di s t ri but i on and cl i ni cal feat ures s een i n t he fi gure are not t ypi cal of l i chen pl anus , at opi c dermat i t i s , s eborrhei c dermat i t i s , or ps ori as i s . 17. A mi ddl e-aged woman not es an as ympt omat i c dark area on her t runk s everal weeks aft er t aki ng over-t he-count er (OTC) l axat i ves for cons t i pat i on. Ski n exami nat i on s hows t he changes s hown i n t he . The mos t l y l i kel y di agnos i s i s
Pa g e 3 0 9 2
ABC Ambe r CHM Conve rte r Tria l ve rsion, http://w w w .proce sste x t.com/a bcchm.html
Online Unnfig 12-Q17. (Courtesy of Stuart Lessin, MD.) A. Mel as ma B. Mel anocyt i c nevus (mol e) C. Fi xed drug erupt i on D. Mel anoma E. Ps ori as i s Vi ew Ans wer 17. T he answer is C. [XII E 11]. The phot o demons t rat es a wel l -ci rcums cri bed, s l at e gray, hyperpi gment ed area t ypi cal of a fi xed drug erupt i on. Phenol pht hal ei n, a common i ngredi ent of over-t he-count er (OTC) l axat i ves , i s as s oci at ed wi t h a hi gh frequency of fi xed drug erupt i ons . Mel as ma i s hyperpi gment at i on, t ypi cal l y on t he face. The hi s t ory and s l at e gray col or i s not t ypi cal of a mel anocyt i c nevus . The l es i on i s wel l ci rcums cri bed wi t h no feat ures of as ymmet ry or pi gment i rregul ari t y t hat can be s een i n mel anoma. Ps ori as i s does not produce pi gment ed l es i ons . P.606
18. A pregnant woman devel ops pai nful nodul es on her l egs . Ski n exami nat i on s hows t he changes s hown i n t he
. The mos t l y l i kel y
di agnos i s i s
Pa g e 3 0 9 3
ABC Ambe r CHM Conve rte r Tria l ve rsion, http://w w w .proce sste x t.com/a bcchm.html
Online Unnfig 12-Q18. (Courtesy of Stuart Lessin, MD.) A. St as i s dermat i t i s B. Cel l ul i t i s C. Vas cul i t i s D. Eryt hema nodos um E. Eryt hema mul t i forme Vi ew Ans wer 18. T he answer is D. [VI E]. The phot o demons t rat es i nfl ammat ory nodul es on t he l ower ext remi t y, whi ch i s t ypi cal of eryt hema nodos um. Pregnancy i s as s oci at ed wi t h eryt hema nodos um. St as i s dermat i t i s i s charact eri zed by confl uent changes about t he ankl es . Cel l ul i t i s t ypi cal l y affect s one ext remi t y wi t h confl uent eryt hema. Vas cul i t i s i s charact eri zed by pal pabl e purpura. Eryt hema mul t i forme i s charact eri zed by t arget l es i ons di s t ri but ed peri pheral l y. 19. W hat hi s t ol ogi c charact eri s t i c of t he l es i on pi ct ured i n t he
is
t he bes t prognos t i c i ndi cat or for t hi s pat i ent ?
Pa g e 3 0 9 4
ABC Ambe r CHM Conve rte r Tria l ve rsion, http://w w w .proce sste x t.com/a bcchm.html
Online Unnfig 12-Q19 A. Ul cerat i on B. Mi t os es per hi gh powered fi el d C. Degree of i nfl ammat ory i nfi l t rat e D. Bres l ow dept h E. Peri neural i nvas i on Vi ew Ans wer 19. T he answer is D [X C 9]. The pat i ent has a l es i on t hat i s cons i s t ent wi t h mel anoma. Al t hough al l t he fact ors l i s t ed affect t he prognos i s of pat i ent s wi t h mel anoma, t he bes t prognos t i c i ndi cat or i s t he t hi cknes s of t he mel anoma (i .e., dept h of i nvas i on) from t he s t rat um corneum t o t he deepes t penet rat i on of t he t umor, al s o known as t he Bres l ow dept h. P.607
20. A 26-year-old man comes to your office complaining of an itchy rash on his back that developed over the last 2 weeks. He reports the rash initially started as a small scaly raised oval-shaped lesion on his chest that progressed to symmetrically involve both sides of his back in a “ Christmas tree― like pattern. Which of the following is the most appropriate next step? A. Ini t i at e t opi cal ant i fungal t reat ment B. Ini t i at e t opi cal s t eroi d t reat ment C. Ini t i at e s ys t emi c s t eroi d t reat ment
Pa g e 3 0 9 5
ABC Ambe r CHM Conve rte r Tria l ve rsion, http://w w w .proce sste x t.com/a bcchm.html
D. No t reat ment for s el f-l i mi t ed condi t i on E. Perform s ki n bi ops y Vi ew Ans wer 20. T he answer is D [IX G]. The pat i ent has a ras h t hat i s cons i s t ent wi t h pi t yri as i s ros ea. Reas s urance t hat t he condi t i on i s s el f l i mi t ed i s t he mos t appropri at e next s t ep. Topi cal ant i fungal s , s t eroi ds , or s ys t emi c s t eroi d t reat ment are not warrant ed at t hi s t i me. Ski n bi ops y i s not i ndi cat ed at t hi s t i me ei t her.
Pa g e 3 0 9 6
ABC Ambe r CHM Conve rte r Tria l ve rsion, http://w w w .proce sste x t.com/a bcchm.html
Editors: Wolfsthal, Susan T itle: NMS Medicine, 6th Edition Copyri ght ©2008 Li ppi ncot t W i l l i ams & W i l ki ns > T able of Cont ent s > Case St udies in Clinic al Dec ision Making
Case
Studies in Clinical Decision Making Case 1 Recurrent Sinopulmonary Infections in a Child A 3-year-old boy is brought to your office for routine follow-up. It is noted that the child has another ear infection. On reviewing his chart, you note that the child has had many, many ear infections, several sinus infections, and three episodes of documented pneumonia. None of these infections has required hospitalization. T he child has always responded well to antibiotics.
Questions
What phys i c al exami nat i on fi ndi ngs w oul d make t he prac t i t i oner ques t i on w het her t hi s c hi l d w as i mmunol ogi c al l y normal ?
What i s t he normal number of i nfec t i ons for a c hi l d of t hi s age?
What are ot her di s eas es t hat c oul d be c aus i ng t he rec urrent i nfec t i ons ?
Discussion The chi l d pres ent ed herei n has a common probl em: recurrent
Pa g e 3 0 9 7
ABC Ambe r CHM Conve rte r Tria l ve rsion, http://w w w .proce sste x t.com/a bcchm.html
s i nopul monary i nfect i ons . The di fference bet ween a chi l d who i s havi ng mul t i pl e i nfect i ons and one who has i mmune defi ci ency i s s omet i mes s ubt l e. It i s normal for a chi l d of t hi s age t o have anywhere bet ween 8 and 12 i nfect i ons per year. Mos t of t hes e woul d occur duri ng t he wi nt er mont hs . If a chi l d i s pres ent l y i n daycare, t hen t hat number of i nfect i ons can ri s e t o as many as 16. Al s o, i t i s s omet i mes very di ffi cul t t o di s t i ngui s h bet ween a vi ral ot i t i s medi a and a bact eri al one; t hi s can l ead t o an overus e of ant i bi ot i cs . Several cl i ni cal s i gns may i ndi cat e t hat t he chi l d may have an i mmune defi ci ency and not jus t a normal number of i nfect i ons , i ncl udi ng abs ence of l ymphoi d t i s s ue. Thi s i s s een i n X-l i nked agammagl obul i nemi a (XLA). Therefore, t he chi l d wi t h XLA wi l l have vi rt ual l y no t ons i l l ar or adenoi dal t i s s ue. W hen t he exami ner t ri es t o pal pat e for l ymph nodes , t hey wi l l not be eas i l y found. Furt hermore, chi l dren wi t h t hi s di s eas e commonl y have very s evere ear i nfect i ons , and t he ot os copi c exami nat i on wi l l s how s carred and deformed eardrums , whi ch i s very di fferent from mos t chi l dren wi t h recurrent ear i nfect i ons . Pyogeni c encaps ul at ed bact eri a s uch as St rept oc oc c us pneumoni a, Haemophi l us i nfl uenzae, St aphyl oc oc c us aureus , and Ps eudomonas s peci es us ual l y caus e t hes e i nfect i ons . W i t h al l of t hes e organi s ms , ant i bodi es are i mport ant for ops oni z at i on and ki l l i ng of t he bact eri a. Las t l y, i f t he chi l d had many epi s odes of pneumoni a, t hey may have chroni c s carri ng i n t he ches t , but t hat i s an unus ual fi ndi ng and woul d not be i dent i fi ed by phys i cal exami nat i on but rat her by radi ol ogi c s t udi es . The di fferent i al di agnos i s for t hi s chi l d i ncl udes anat omi c abnormal i t i es of t he upper ai rway, gas t roes ophageal refl ux, and pot ent i al l y even at opy as t he caus e of recurrent ot i t i s medi a and s i nus i t i s . Al l ergi c rhi ni t i s woul d not commonl y caus e t hes e s ympt oms t o occur at a very young age. However, an anat omi c abnormal i t y or gas t roes ophageal refl ux can caus e t hi s chi l d t o have ear and s i nus i nfect i on from a very earl y age. Ot her t ypes of i mmune defi ci enci es al s o can caus e recurrent i nfect i ons . The mos t common woul d be a s evere combi ned i mmune defi ci ency, but s uch chi l dren al mos t al ways pres ent before 1 year of
Pa g e 3 0 9 8
ABC Ambe r CHM Conve rte r Tria l ve rsion, http://w w w .proce sste x t.com/a bcchm.html
l i fe. Ot her i mmune defi ci enci es al s o can pres ent at t hi s age and woul d need t o be eval uat ed (e.g., W i s kot t -Al dri ch and hyper IgE s yndrome). Cys t i c fi bros i s i s a cons i derat i on as wel l i n any chi l d wi t h recurrent s i nus and pul monary di s eas e. P.611
On careful examination, you note that the child has very scarred tympanic membranes. You also note that he has very small tonsils. In addition, you also review the chart carefully and note that this child has received antibiotics since the age of 1, almost on a monthly basis.
Question
What i mmunol ogi c and radi ol ogi c s t udi es w oul d be i ndi c at ed?
Discussion The s erol ogi c eval uat i on woul d cent er on t he chi l d's i mmunogl obul i ns . Thi s coul d be done ei t her i n a s t epwi s e fas hi on or as a comprehens i ve exami nat i on. The fi rs t s t ep woul d i nvol ve meas uri ng t he quant i t at i ve IgG, IgM, IgA, and IgE l evel s . Ot her t es t s t hat coul d be done woul d i nvol ve funct i onal ant i bodi es t o di pht heri a and t et anus t o eval uat e t he chi l d's abi l i t y t o mount an ant i body res pons e. A compl et e bl ood count wi t h di fferent i al as wel l as an el ect rol yt e panel and l i ver funct i on t es t s al s o woul d be i ndi cat ed. If t he i mmunogl obul i ns were abnormal , t hen T-cel l i dent i fi cat i on woul d be i ndi cat ed t o i dent i fy whet her B cel l s are pres ent . The radi ol ogi c s t udi es t hat coul d be performed woul d i ncl ude a ches t x-ray, l ooki ng for s carri ng from previ ous pneumoni as , a l at eral neck fi l m t o eval uat e adenoi dal t i s s ue, and pos s i bl y a bari um s wal l ow or mi l k s can t o i dent i fy whet her t he chi l d has gas t roes ophageal refl ux. T he laboratory values are available and indicate that the child
Pa g e 3 0 9 9
ABC Ambe r CHM Conve rte r Tria l ve rsion, http://w w w .proce sste x t.com/a bcchm.html
does have x-linked agammaglobulinemia: IgG levels of 150 mg/dL (normal = 600–1500 mg/dL), antibody titers to diphtheria and tetanus were undetectable, T -cell indices were normal, and no B-cells were detected. In consultation with a pediatric immunologist, infusions of IV gamma globulins are started. T he levels are measured, and a proper therapeutic value is obtained.
Questions
Shoul d t he c hi l d rec ei ve ot her medi c at i ons i n addi t i on t o t he I V gamma-gl obul i n?
Does t he c hi l d need i mmuni zat i ons , now t hat he i s rec ei vi ng gamma-gl obul i n?
Discussion Many chi l dren wi t h XLA wi l l have chroni c di s eas e not ed before t hei r ons et of t herapy wi t h IV gamma-gl obul i n. Some of t he pat i ent s wi l l not need any ot her t herapy ot her t han t he IV gamma-gl obul i n. Many chi l dren wi l l need chroni c broad-s pect rum ant i bi ot i cs . Thes e broad-s pect rum ant i bi ot i cs are rout i nel y rot at ed t o decreas e t he amount of res i s t ance t hat i s generat ed. The di s cont i nuat i on of ant i bi ot i cs us ual l y t ri ggers an i mmedi at e i nfect i on of t he s i nus es or ears . Immuni z at i ons are not onl y unneces s ary but i n s ome cas es cont rai ndi cat ed i n t hi s chi l d. The chi l d wi t h XLA has no abi l i t y t o generat e an ant i body res pons e; t herefore, no i mmuni zat i on wi l l be hel pful . In fact , i mmuni zat i ons wi t h l i ve vi rus es can be det ri ment al . A number of chi l dren wi t h XLA who recei ved l i ve pol i o vi rus may have devel oped ful mi nat e pol i o aft er t he i mmuni zat i on. It was t hi s popul at i on t hat was t he i mpet us for t he change from us i ng a l i ve pol i o vi rus t o t he ki l l ed vacci ne t hat i s current l y i n us e. T he child is doing well, and the family is concerned about other issues they need to face. T hey are worried that even with the
Pa g e 3 1 0 0
ABC Ambe r CHM Conve rte r Tria l ve rsion, http://w w w .proce sste x t.com/a bcchm.html
replacement therapy, his immune system is not normal.
Questions
Are t here ot her i nfec t i ons and ot her mani fes t at i ons t hat meri t c onc ern?
Are t hes e c hi l dren s us c ept i bl e t o ot her i nfec t i ons ?
Discussion Thes e chi l dren, before t he advent of IV gamma-gl obul i n, woul d commonl y s uccumb t o echovi rus or Coxs acki e vi rus i nfect i ons . They woul d devel op a chroni c meni ngoencephal i t i s t hat woul d be i ns i di ous and have progres s i ve neurol ogi c s ympt oms . Thes e i nfect i ons woul d go on t o be qui t e debi l i t at i ng, i f not fat al . Thes e chi l dren l es s commonl y are s us cept i bl e t o a vari et y of ot her ai l ment s , i ncl udi ng prot ei n-l os i ng ent eropat hy, mal abs orpt i on, neut ropeni a, al opeci a t ot al i s , and amyl oi dos i s . They al s o may have an aut oi mmune s eronegat i ve art hri t i s t hat i s not rel at ed t o an i nfect i on. The art hri t i s us ual l y affect s t he l arge joi nt s , caus i ng hydrart hros i s and l i mi t ed range of movement wi t hout joi nt pai n or des t ruct i on. P.612
Thes e chi l dren handl e mos t ot her i nfect i ons qui t e wel l . They are not more s us cept i bl e t o herpes i nfect i on or t ubercul os i s . W hen t hey do have t hes e i nfect i ons , t hey are no more s eri ous t han i nfect i ons t hat occur i n any normal pat i ent and are s el f-l i mi t ed. Pneumoc ys t i s c ari ni i has been obs erved i n a few pat i ent s wi t h XLA, but t hos e are us ual l y ext remel y debi l i t at ed pat i ent s .
Case 2 Oral Erosions and a Rash
Pa g e 3 1 0 1
ABC Ambe r CHM Conve rte r Tria l ve rsion, http://w w w .proce sste x t.com/a bcchm.html
A 29-year-old African American man presents to the emergency department with the chief complaint of painful mouth erosions that have developed over 3 days. T he initial symptom was a burning sensation of the left upper lip. T hen tender sores developed on the gums and progressed to involve the entire mouth. Swallowing has become difficult because of increased secretions and pain. A rash has erupted on the patient' s arms and legs within the past day. Before the onset of symptoms, the patient was in good health. He has no ongoing or past medical problems. He takes no medications, vitamins, or herbal supplements. He has no known allergies and has no history of atopy. He works as a security officer and denies participating in any activities that pose a high risk for human immunodeficiency virus (HIV) infection. T here is no family history of autoimmune disease or cancer. Review of systems is remarkable for a headache.
Question
What i s t he di fferent i al di agnos i s bas ed on t he hi s t ory of oral eros i ons and a ras h?
Discussion The di fferent i al di agnos i s of oral eros i ons i ncl udes i nfect i ous caus es . Herpes s i mpl ex vi rus (HSV) i nfect i on produces l ocal i zed bl i s t eri ng and s econdary eros i ons t hat may affect t he gi ngi va, buccal mucos a, and pal at e. It i s al s o t he mos t common i nfect i ous et i ol ogy of eryt hema mul t i forme. Hand-foot -and-mout h di s eas e (a coxs acki evi rus i nfect i on) produces bl i s t eri ng and eros i ons of t he pal at e, buccal mucos a, gi ngi va, and t ongue, as wel l as bl i s t ers on t he hands and feet . Aut oi mmune bl i s t eri ng di s eas es s uch as pemphi gus vul gari s and paraneopl as t i c pemphi gus i nvol ve t he oral mucos a and s ki n. An advers e react i on t o a s ys t emi c medi cat i on can affect t he mucous membranes . Eryt hema mul t i forme major and t oxi c epi dermal necrol ys i s are t he mos t s evere forms of an advers e drug
Pa g e 3 1 0 2
ABC Ambe r CHM Conve rte r Tria l ve rsion, http://w w w .proce sste x t.com/a bcchm.html
react i on. An al l ergi c cont act dermat i t i s t o oral hygi ene product s (e.g., t oot hpas t e, mout hwas h) can produce i nfl ammat ory changes and eros i ons of t he oral mucos a. Li chen pl anus i s as s oci at ed wi t h eros i ve oral l es i ons . Sys t emi c di s orders s uch as i nfl ammat ory bowel di s eas e, s ys t emi c l upus eryt hemat os us , and Behçet 's di s eas e may i nvol ve t he oral mucos a and caus e eros i ve l es i ons . T he patient appears to be in mild distress and has a temperature of 99°F (37.2°C). Physical examination of the skin shows grouped erythematous vesicles and pustules on the left upper lip. T he vermilion border is crusted with dried blood, and the anterior gingiva, buccal mucosa, and soft palate are bright red and eroded with serous exudate. T he arms and legs show 0.5–1.0-cm erythematous round macules without scaling. T he dorsum of the hands, the palms, and the soles have 1-cm erythematous target-like macules with a central violaceous hue. T here is shotty submental and cervical lymphadenopathy.
Question
What t es t s w oul d you perform t o narrow t he di fferent i al di agnos i s ?
Discussion Sampl es for a Tzanck s mear and HSV ant i body t es t s houl d be t aken from t he grouped ves i cul opus t ul es of t he l eft upper l i p t o i dent i fy t he pres ence of HSV. The upper l i p and oral mucos a s houl d be s wabbed, and bact eri al and vi ral cul t ures s houl d be obt ai ned t o confi rm an i nfect i ous et i ol ogy. A s ki n bi ops y from t he buccal mucos a and one of t he t arget -l i ke l es i ons s houl d be obt ai ned t o confi rm t he di agnos i s and rul e out ot her caus es of oral eros i ons , s uch as pemphi gus vul gari s . T he T zanck smear reveals multinucleated giant cells. Subsequent results demonstrate positive HSV type 1 cultures for the lip and skin biopsies of the buccal mucosa and hand characterized by histologic changes consistent with erythema
Pa g e 3 1 0 3
ABC Ambe r CHM Conve rte r Tria l ve rsion, http://w w w .proce sste x t.com/a bcchm.html
multiforme. P.613
Questions
What ac ut e t reat ment s houl d you provi de?
What are t he l ong-t erm t reat ment i s s ues ?
Discussion To reduce vi ral s heddi ng and i ncreas e heal i ng t i me, oral ant i vi ral t herapy s houl d be i ni t i at ed at appropri at e dos es for pri mary HSV i nfect i on. For oral pai n rel i ef, a t opi cal anes t het i c, s uch as vi s cous l i docai ne, s houl d be provi ded wi t h i ns t ruct i ons for t he pat i ent t o s wi s h and s pi t out up t o four t i mes a day. A s hort 10–14-day cours e of oral predni s one may provi de addi t i onal rel i ef of acut e i nfl ammat i on of t he eryt hema mul t i forme react i on; however, us e of cort i cos t eroi ds i n eryt hema mul t i forme remai ns cont rovers i al . The as s oci at ed cons t i t ut i onal s ympt oms (headache and l ow-grade fever), l ymphadenopat hy, and no hi s t ory of HSV i ndi cat e t hat t he i nfect i on i s pri mary and t hat t he pat i ent i s at ri s k for recurrent epi s odes . The pat i ent s houl d be educat ed regardi ng t he nat ural hi s t ory of herpes l abi al i s , i t s i nfect i vi t y, and t he s i gns and s ympt oms of recurrence. Becaus e of t he s evere nat ure of t he pat i ent 's react i on t o hi s i ni t i at e erupt i on, he s houl d be provi ded wi t h a pres cri pt i on for an oral ant i vi ral agent wi t h t he i ns t ruct i ons t o i ni t i at e t herapy at t he fi rs t s ympt oms of recurrence. If t he pat i ent devel ops frequent erupt i ons s uch t hat t he epi s odi c ant i vi ral t herapy proves t o be i neffect i ve i n s uppres s i ng eryt hema mul t i forme, t hen ongoi ng prophyl act i c dos es of ant i vi ral agent s s houl d be us ed for l ong-t erm s uppres s i on.
Case 3
Pa g e 3 1 0 4
ABC Ambe r CHM Conve rte r Tria l ve rsion, http://w w w .proce sste x t.com/a bcchm.html
Polyarthritis A 65-year-old woman presents to your office with a several-week history of pain and stiffness in her shoulders, hips, and hands. T his began 2 weeks after visiting her grandchildren. Within a few weeks, she noted swelling in her hands and feet in addition to generalized stiffness lasting most of the morning. She did have a low-grade fever, but denied any chills, sweats, or weight loss. She had no associated headache, jaw claudication, or visual changes. She denied photosensitivity, oral ulcers, sicca symptoms, Raynaud' s, serositis, history of miscarriages, or blood clots. She does have a family history of psoriasis.
Questions
Bas ed on her hi s t ory, w hat t ype of art hri t i s i s t hi s : i nfl ammat ory or noni nfl ammat ory?
How does t he pat t ern of joi nt i nvol vement hel p i n det ermi ni ng t he t ype of art hri t i s ?
What i s your di fferent i al di agnos i s ?
Discussion The pat i ent 's hi s t ory of morni ng s t i ffnes s l as t i ng more t han 1 hour s ugges t s t hat i nfl ammat ory art hri t i s i s t he caus e of her s ympt oms . At t hi s junct ure, t he art hri t i s has been pres ent onl y for a few weeks ; t herefore, i nfect i ous et i ol ogi es mus t be cons i dered. She does have a l ow-grade fever, but rarel y does a bact eri al art hri t i s pres ent i n a pol yart i cul ar fas hi on s uch as t hi s , unl es s t he pat i ent i s cri t i cal l y i l l or has a hi s t ory of i nt ravenous drug us e. Vi ral i nfect i ons may pres ent i n t hi s manner wi t h a new-ons et i nfl ammat ory art hri t i s . Of part i cul ar concern i n t hi s pat i ent woul d
Pa g e 3 1 0 5
ABC Ambe r CHM Conve rte r Tria l ve rsion, http://w w w .proce sste x t.com/a bcchm.html
be parvovi rus B19 i nfect i on, as s he has been expos ed t o young chi l dren. Parvovi rus B19 i nfect i on, or Fi ft h's di s eas e, i s endemi c among s chool -aged chi l dren. Chi l dren t ypi cal l y pres ent wi t h a ras h on t hei r cheeks gi vi ng t hem a “s l apped cheek― appearance i n conjunct i on wi t h a l ow-grade fever and mi l d cons t i t ut i onal s ympt oms . Adul t s i nfect ed wi t h parvovi rus B19 may devel op a fl u-l i ke i l l nes s and mi l d macul opapul ar ras h on t he ext remi t i es , wi t h art hral gi as and art hri t i s s een i n approxi mat el y 20% of pat i ent s . The art hri t i s i s s ymmet ri c and can be confus ed wi t h rheumat oi d art hri t i s (RA). Y oung t o mi ddl e-aged adul t women are at hi ghes t ri s k for t he art hropat hy. Di agnos t i c t es t i ng s oon aft er t he ons et of s ympt oms i s hel pful i n maki ng t hi s di agnos i s . A pos i t i ve IgM ant i body s ugges t s act i ve i nfect i on. A pos i t i ve IgG ant i body i s cons i s t ent wi t h pri or expos ure, but i t i s not hel pful i n t he di agnos i s gi ven t he hi gh preval ence of s eroconvers i on i n t he general popul at i on. Al t hough t he pres ence of fever rai s es t he i ndex of s us pi ci on for an i nfect i ous proces s , many pat i ent s wi t h noni nfect i ous i nfl ammat ory art hri t i s can have l ow-grade fevers . P.614
Ot her concerns i n t hi s pat i ent gi ven her age woul d i ncl ude a paraneopl as t i c s yndrome or pol ymyal gi a rheumat i ca (PMR), es peci al l y i n l i ght of t he proxi mal s houl der and pel vi c gi rdl e i nvol vement . PMR may rarel y have an as s oci at ed i nfl ammat ory art hri t i s of t he peri pheral joi nt s . Rheumat oi d art hri t i s , s ys t emi c l upus eryt hemat os us , s cl eroderma, and pol ymyos i t i s al l may pres ent wi t h an i nfl ammat ory s ymmet ri c pol yart hri t i s . Ps ori at i c art hri t i s , al t hough us ual l y pres ent i ng i n an as ymmet ri c ol i goart i cul ar fas hi on, may pres ent wi t h a s ymmet ri c pat t ern of joi nt i nvol vement , as can a crys t al l i ne art hri t i s s uch as gout (part i cul arl y i n el derl y women). On examination, the patient is in obvious discomfort related to her joints. Her temperature is 38°C. She has a small, slightly raised erythematous scaly rash in her hairline and around her
Pa g e 3 1 0 6
ABC Ambe r CHM Conve rte r Tria l ve rsion, http://w w w .proce sste x t.com/a bcchm.html
ears and in her umbilicus. No nail pits are present. T here are no other mucocutaneous lesions. T he temporal arteries have good pulsations and are nontender. T here is no adenopathy. T he cardiopulmonary and abdominal examinations are normal. T he musculoskeletal examination is notable for limitation of motion of her shoulders secondary to pain and stiffness. She has obvious synovitis (synovial thickening with tenderness) of her bilateral wrists, metacarpophalangeal (MCPs) joints, proximal interphalangeal (PIP) joints, ankles, and metatarsophalangeal (MT P) joints. T here are no “ sausage digits― and there is no evidence of an enthesopathy. She had mild warmth in both of her knees with moderate joint effusions. T here was no sclerodactyly or muscle atrophy or weakness.
Questions
How do t he fi ndi ngs on phys i c al exami nat i on gui de your di fferent i al di agnos i s ?
What i nformat i on s houl d you s eek from di agnos t i c t es t s at t hi s t i me?
Discussion The exami nat i on confi rms t he pres ence of a s ymmet ri c i nfl ammat ory pol yart hri t i s . Thus , t he di fferent i al i ncl udes RA; s ys t emi c l upus eryt hemat os us (SLE); pol ymyal gi a rheumat i ca; and l es s l i kel y i nfect i ous , ps ori at i c and crys t al l i ne art hri t i s . The pat i ent has a ras h t hat has t he appearance of ps ori as i s . Ps ori at i c art hri t i s devel ops i n onl y 5% of pat i ent s wi t h ps ori as i s . The ri s k of ps ori at i c art hri t i s i ncreas es wi t h a fami l y hi s t ory of s pondyl oart hropat hy or ext ens i ve nai l pi t t i ng. There are fi ve di fferent pat t erns of ps ori at i c art hri t i s . See Chapt er 10, Sect i on IV B 3 d (2). It i s pos s i bl e t hat t hi s pat i ent has a ps eudorheumat oi d pat t ern of ps ori at i c art hri t i s . The di fferent i al di agnos i s s t i l l i ncl udes vi ral i l l nes s s uch as parvovi rus B19, rheumat oi d art hri t i s , SLE, and mal i gnancy. It i s now unl i kel y
Pa g e 3 1 0 7
ABC Ambe r CHM Conve rte r Tria l ve rsion, http://w w w .proce sste x t.com/a bcchm.html
bas ed on t he phys i cal exami nat i on t hat t he pat i ent has s cl eroderma or an i nfl ammat ory myopat hy. At t hi s poi nt , a compl et e bl ood count (CBC) and a comprehens i ve met abol i c profi l e wi l l hel p el i mi nat e ot her s ys t emi c i l l nes s es s uch as a vi ral hepat i t i s and mal i gnancy. Sys t emi c meas ures of i nfl ammat i on s uch as an eryt hrocyt e s edi ment at i on rat e (ESR) or C-react i ve prot ei n (CRP) are nons peci fi c i ndi cat ors of i nfl ammat i on and are not hel pful i n t he di agnos i s , but may be hel pful i n as s es s i ng res pons e t o t reat ment . Parvovi rus s erol ogy s houl d be eval uat ed, gi ven her expos ure t o chi l dren. A rheumat oi d fact or (RF), ant i -CCP, and an ant i nucl ear ant i body (ANA) may be hel pful i n t he di agnos i s , gi ven t he s ymmet ri c pres ent at i on of her art hri t i s . It i s i mport ant t o keep i n mi nd t hat 15% of pat i ent s wi t h RA have a negat i ve RF. A s ynovi anal ys i s woul d be hel pful i n as s es s i ng t he degree of s ynovi al i nfl ammat i on and t o eval uat e t he pres ence of crys t al s . T he test results come back as follows: hemoglobin is 10.5 g/dL; the white blood cell (WBC) count is 8,300/µL, and the platelet count is 500,000/µL. T he Westergren erythrocyte sedimentation rate is 78 mm/hour. A comprehensive metabolic profile is normal. T he RF is positive at 349 IU/mL and anti-CCP was 80 Ul/mL. T he ANA is positive at 1:80 speckled. Parvovirus antibody IgG and IgM are negative. T he x-rays of her hands and wrist show mild periarticular osteopenia around her MCPs. No erosions are seen. Synovial fluid analysis indicates 18,000 WBC with 80% polymorphonuclear cells (PMNs) and 20% lymphs. No crystals were seen.
Questions
What i s t he di agnos i s ?
What addi t i onal di agnos t i c t es t s are appropri at e t o c onfi rm t he di agnos i s ?
Pa g e 3 1 0 8
ABC Ambe r CHM Conve rte r Tria l ve rsion, http://w w w .proce sste x t.com/a bcchm.html
What t herapeut i c approac h i s mos t appropri at e now ?
P.615
Discussion At t hi s poi nt , RA i s t he mos t l i kel y di agnos i s gi ven t he morni ng s t i ffnes s , s ymmet ri cal joi nt i nvol vement , t he pres ence of t he s erum RF, i nfl ammat ory s ynovi al fl ui d, and t he radi ographi c feat ure of peri art i cul ar os t eopeni a. There i s no evi dence of an as s oci at ed mal i gnancy, and PMR and ps ori at i c art hri t i s are l es s l i kel y gi ven t he pos i t i ve RF. Parvovi rus has been excl uded bas ed on negat i ve s erol ogi es . The pat i ent mus t have art hri t i s for a peri od of 6 weeks t o defi ni t i vel y di agnos e RA, t o ens ure t hat ot her vi ral or s el f-l i mi t ed ent i t i es are not cont ri but i ng t o t he pat i ent 's art hri t i s . In t he meant i me, t he pat i ent i s s t art ed on nons t eroi dal ant i -i nfl ammat ory drug (NSAID) t herapy and ret urns i n 4 weeks . On return 4 weeks later, the patient still has morning stiffness lasting most of the day. Her examination is notable for the presence of aggressive synovitis in her wrists, MCPs, PIPs, knee, ankle, and MT PJs. T he results of the tests are reviewed with the patient, and therapeutic approach is outlined.
Questions
What t herapi es s houl d be c ons i dered at t hi s junc t ure?
Shoul d s t eroi ds be gi ven?
Discussion Now t hat i t i s cl ear t hat t he art hri t i s has been pres ent for 6 weeks , i t i s i mport ant t hat a di s eas e-modi fyi ng ant i rheumat i c drug (DMARD) be s t art ed i mmedi at el y, as eros i ons occur earl y i n t he di s eas e cours e. The gol d s t andard i n t he t reat ment of RA i s
Pa g e 3 1 0 9
ABC Ambe r CHM Conve rte r Tria l ve rsion, http://w w w .proce sste x t.com/a bcchm.html
met hot rexat e. Before s t art i ng met hot rexat e, a bas el i ne hepat i t i s s creen and ches t x-ray s houl d be done. In addi t i on, i t i s becomi ng s t andard pract i ce t o obt ai n a t es t for t ubercul os i s [puri fi ed prot ei n deri vat i ve (PPD)], becaus e i f t he met hot rexat e i s i neffect i ve, an ant i –t umor necros i s fact or (TNF) agent may be cons i dered. There have been document ed cas es of react i vat i on of t ubercul os i s duri ng ant i -TNF t herapy. W hi l e on t he met hot rexat e, t he pat i ent mus t abs t ai n from al cohol and have l aborat ory work every 6–8 weeks t o moni t or for t oxi ci t y. A s hort cours e of l ow-dos e s t eroi ds may be hel pful i n cont rol l i ng s ome of t he s ympt oms whi l e wai t i ng for a DMARD t o work, but a l ong-t erm cours e of s t eroi ds has no rol e i n hal t i ng t he progres s i on of di s eas e and i s fraught wi t h many pot ent i al s i de effect s and l ong-t erm compl i cat i ons . The pat i ent el ect s t o s t art met hot rexat e t herapy. Fol i c aci d 1 mg dai l y i s pres cri bed t o di mi ni s h t he l i kel i hood of s t omat i t i s .
Case 4 Severe Headache A 62-year-old woman calls your office because of a severe headache that came on suddenly 1 hour ago. Her medical history is significant for headaches associated with menses, which ceased 12 years ago, and one or two throbbing headaches annually since menopause. She has hypertension controlled with an angiotensin-converting enzyme (ACE) inhibitor and beta-blocker, dyslipidemia treated with a statin, and she smokes one-half pack per day in spite of repeated urgings to stop smoking.
Questions
What are s ome pos s i bl e c aus es of headac he?
What ot her ques t i ons w oul d you l i ke t o as k t hi s pat i ent ?
Discussion
Pa g e 3 1 1 0
ABC Ambe r CHM Conve rte r Tria l ve rsion, http://w w w .proce sste x t.com/a bcchm.html
The caus es of headache are numerous . In a pat i ent wi t h headaches , i t i s us eful t o charact eri ze each t ype of headache and devel op a worki ng hypot hes i s as t o t he caus e of each headache. In t hi s pat i ent , t here are pot ent i al l y t hree t ypes of headache: peri mens t rual headaches , pos t menopaus al headaches , and t he s evere headache prompt i ng her t o s eek medi cal as s i s t ance at t hi s t i me. Head pai n i s caus ed by di s orders of t he head or neck, i ncl udi ng neurol ogi c di s orders . Probl ems as di vers e as gi ant cel l art eri t i s , acut e gl aucoma, s i nus i t i s , dent al abs ces s , t emporomandi bul ar joi nt dys funct i on, t ri gemi nal neural gi a, or cervi cal art hri t i s can caus e head di s comfort . Int racrani al caus es of headache i ncl ude cent ral nervous s ys t em (CNS) mas s l es i ons s uch as pri mary or met as t at i c t umors , abs ces s , hydrocephal us , meni ngi t i s , s uperi or s agi t t al s i nus t hrombos i s , s ubarachnoi d hemorrhage, and mi grai ne. P.616
One approach t o t he di fferent i al di agnos i s of headache i s t o i dent i fy acut e, l i fe-t hreat eni ng di s orders s uch as meni ngi t i s and s ubarachnoi d hemorrhage from chronic condi t i ons s uch as mi grai ne. Furt hermore, i t i s hel pful t o di fferent i at e condi t i ons t hat requi re s peci fi c, urgent i nt ervent i on s uch as gi ant cel l art eri t i s or acut e gl aucoma from more chroni c di s orders s uch as mi grai ne or cervi cal art hri t i s . Further history shows that the patient' s perimenstrual headaches were characterized by holocephalic throbbing discomfort that started 1–2 days before menses and lasted 2–3 days. She graded the headache pain as 7/10. T here was occasional nausea and photophobia. T he patient found relief with nonsteroidal anti-inflammatory medications. Her postmenopausal headaches occurred every 3–4 months and again were holocephalic and throbbing, but only lasted a few hours or an entire day. T he pain was 4–6/10 and was relieved with the same medications she had used for her perimenstrual headaches.
Pa g e 3 1 1 1
ABC Ambe r CHM Conve rte r Tria l ve rsion, http://w w w .proce sste x t.com/a bcchm.html
T he patient denied visual symptoms including diplopia, jaw claudication, nasal drainage, dental pain, jaw “ clicking,― facial pain, or neck discomfort. She denied weakness or numbness in any limb and did not think she had a fever. T he patient did state that she had never had a headache like her current one, which was holocephalic and graded as 9–10/10. T he pain was constant, and she had nausea and photophobia. She was gardening when the headache suddenly began and had momentarily collapsed to her knees. A dose of naproxen had not brought any relief.
Questions
What are your di agnos t i c c ons i derat i ons ?
What do you advi s e t he pat i ent t o do?
Discussion The pat i ent 's s ympt oms are not s i mi l ar t o her pri or headache hi s t ory, whi ch are cons i s t ent wi t h mi grai ne. There are no s ympt oms t o s ugges t l ocal ocul ar, s i nus , dent al , or mus cul os kel et al s ources of pai n. Of concern i s t he s udden ons et of s evere, at ypi cal head pai n. It i s not cl ear from t he pat i ent 's hi s t ory whet her t he col l aps e was due t o bri ef weaknes s , l os s of cons ci ous nes s , or bot h. The col l aps e, accompani ed by naus ea, s ugges t s a neurol ogi c probl em s uch as a s t roke s yndrome (vert ebrobas i l ar di s eas e, i nt racrani al hemorrhage) or i ncreas ed i nt racrani al pres s ure [hydrocephal us , s agi t t al s i nus t hrombos i s , s ubarachnoi d hemorrhage (SAH)]. Her l ack of focal neurol ogi c s ympt oms i s not s upport i ve of vert ebrobas i l ar di s eas e or an i nt racerebral hemat oma. The rapi d ons et of s ympt oms i s not cons i s t ent wi t h t he pres ent at i on of s agi t t al s i nus t hrombos i s . Subarachnoi d hemorrhage pres ent s as “t he wors t headache of my l i fe.― The headache i s t ypi cal l y of s udden ons et , hol ocephal i c, s evere, and can be accompani ed by naus ea and bri ef
Pa g e 3 1 1 2
ABC Ambe r CHM Conve rte r Tria l ve rsion, http://w w w .proce sste x t.com/a bcchm.html
l os s of cons ci ous nes s , probabl y refl ect i ng a t rans i ent i ncreas e i n i nt racrani al pres s ure. Phot ophobi a can occur, as can emes i s . Typi cal l y t here are no i mmedi at e focal neurol ogi c s ympt oms or s i gns , al t hough di pl opi a can refl ect damage t o t he ocul omot or nerve from t he mas s effect of a pos t eri or communi cat i ng art ery aneurys m. Smoki ng i s a ri s k fact or for SAH. Because of your concern about SAH, you advise the patient to call 911 and have an ambulance transport her to the hospital. You contact the hospital to inform them about the patient' s arrival and your diagnostic suspicion.
Questions
T o w hi c h as pec t s of t he phys i c al exami nat i on s houl d t he emergenc y depart ment phys i c i ans be part i c ul arl y al ert ?
What di agnos t i c t es t s are mos t es s ent i al t o t he pat i ent 's eval uat i on?
Discussion Pat i ent s pres ent i ng wi t h SAH frequent l y are qui t e hypert ens i ve becaus e of el evat ed cat echol ami ne l evel s . Al t hough pat i ent s wi t h SAH can have a l ow-grade fever, el evat ed t emperat ure s houl d rai s e concern for an i nfect i ve, rat her t han SAH-as s oci at ed, meni ngi t i s . If a pat i ent has i mpai red cons ci ous nes s , t here s houl d be concern about ai rway prot ect i on, and i nt ubat i on s houl d be cons i dered. P.617
The emergency depart ment phys i ci an s houl d exami ne t he head and neck for t he caus es of headache referred t o previ ous l y. Al t hough evi dence of meni ngeal i rri t at i on, s uch as Kerni g's and Brudz i ns ki 's s i gns , s houl d be l ooked for i n pat i ent s wi t h a pos s i bl e SAH, t hes e s i gns are frequent l y abs ent earl y i n t he cl i ni cal cours e. A cri t i cal det ermi nant of out come aft er SAH i s t he pat i ent 's l evel of
Pa g e 3 1 1 3
ABC Ambe r CHM Conve rte r Tria l ve rsion, http://w w w .proce sste x t.com/a bcchm.html
cons ci ous nes s on pres ent at i on, wi t h progres s i vel y i mpai red arous al bei ng as s oci at ed wi t h a wors e prognos i s . An ocul omot or pal s y i n an al ert pat i ent i s part i cul arl y s ugges t i ve of a pos t eri or communi cat i ng art ery aneurys m, and an abducens pal s y s ugges t s i ncreas ed i nt racrani al pres s ure (a “fal s e l ocal i zi ng s i gn―). Al ert or obt unded pat i ent s wi t h SAH rarel y have ot her focal neurol ogi c s i gns s uch as a hemi pares i s . The “gol d s t andard― for t he di agnos i s of SAH i s t he pres ence of s ubarachnoi d bl ood on a brai n comput ed t omography (CT) s can. If bl ood i s pres ent , t here i s no need t o perform a l umbar punct ure. A modes t proport i on of al ert pat i ent s wi t h SAH have an unremarkabl e CT s can; i n t hes e i ndi vi dual s , a l umbar punct ure i s i ndi cat ed t o document t he pres ence of bl eedi ng. Cl ot t i ng s t udi es [pl at el et count , prot hrombi n t i me, and act i vat ed part i al t hrombi n t i me (PTT)] are appropri at e t o s creen for a bl eedi ng di at hes i s . A s erum s odi um s houl d be obt ai ned, becaus e SAH pat i ent s are prone t o di s t urbances i n s odi um concent rat i ons . An el ect rocardi ogram (ECG) may s how fi ndi ngs s uch as peaked T waves or arrhyt hmi as , whi ch may repres ent cat echol ami ne-as s oci at ed cardi ac effect s . T he patient' s blood pressure was 185/95 mm Hg; she was afebrile. T he patient had no neck rigidity and a normal head and neck examination. She was alert and cognitively intact. Her neurologic examination was normal. A brain CT scan showed subarachnoid blood with mild dilation of the ventricles. Clotting studies, serum sodium concentration, and the ECG were normal.
Questions
What are t he next management s t eps ?
At t hi s t i me, t o w hat c ompl i c at i ons s houl d t he pat i ent 's phys i c i ans be al ert ?
Pa g e 3 1 1 4
ABC Ambe r CHM Conve rte r Tria l ve rsion, http://w w w .proce sste x t.com/a bcchm.html
Discussion Pat i ent s wi t h SAH s houl d be pl aced i n an i nt ens i ve care uni t . There s houl d be frequent neurol ogi c eval uat i ons . Ext remes of bl ood pres s ure s houl d be avoi ded. It i s hel pful t o cons i der t he rel at i ons hi p: cerebral perfus i on pres s ure = mean art eri al pres s ure – i nt racrani al pres s ure. A cerebral perfus i on pres s ure of 70–80 mm Hg i s des i rabl e. In an al ert pat i ent , who pres umabl y does not have a cri t i cal el evat i on of i nt racrani al pres s ure, a mean bl ood pres s ure of approxi mat el y 90–100 mm Hg can be t arget ed. Labet al ol i s a part i cul arl y effect i ve ant i hypert ens i ve agent i n t hi s s et t i ng becaus e i t does not caus e an el evat i on i n i nt racrani al pres s ure. Hydrat i on wi t h normal s al i ne i s appropri at e s o as t o mai nt ai n a euvol emi c s t at e. Mi l d s edat i on and anal ges i a for headache s houl d be cons i dered, and agent s t o prevent s t rai ni ng at s t ool can be pres cri bed. Ni modi pi ne, a cal ci um channel ant agoni s t , decreas es t he i nci dence of del ayed i s chemi c defi ci t s s econdary t o cerebral vas os pas m. Unfort unat el y, i t can caus e exces s i ve hypot ens i on, whi ch can exacerbat e cerebral i s chemi a i f not adequat el y t reat ed. Neuros urgi cal and radi ol ogi c cons ul t at i on s houl d be obt ai ned, and urgent four-ves s el angi ography i s neces s ary t o document al l aneurys ms t hat t he pat i ent may have. Compl i cat i ons at t hi s t i me i ncl ude t he devel opment of hydrocephal us at t ri but abl e t o di s rupt i on of CSF fl ow pat t erns by s ubarachnoi d (or i nt ravent ri cul ar) bl ood, aneurys mal rebl eedi ng, and hyponat remi a. A decreas ed s erum s odi um l evel can be caus ed by overhydrat i on, t he s yndrome of i nappropri at e s ecret i on of ant i di uret i c hormone, and cerebral s al t was t i ng. Thes e compl i cat i ons can caus e di mi ni s hed cons ci ous nes s , s ei zures , and, occas i onal l y, focal neurol ogi c s i gns . T he patient' s blood pressure is successfully controlled with labetalol. Nimodipine is well tolerated. A cerebral angiogram indicates a single anterior communicating artery aneurysm measuring 10 mm in its largest diameter. T he patient remains
Pa g e 3 1 1 5
ABC Ambe r CHM Conve rte r Tria l ve rsion, http://w w w .proce sste x t.com/a bcchm.html
alert and cognitively intact. T he patient, family, and physician team discuss treatment options to prevent rebleeding, including surgical “ clipping― or endovascular “ coiling― of the aneurysm. T he patient decides on surgical intervention, which is performed on the second hospital day. P.618
Questions
At t hi s t i me, t o w hat c ompl i c at i ons s houl d t he pat i ent 's phys i c i ans be al ert ?
What management s t rat egi es s houl d be purs ued?
Discussion An operat i ve compl i cat i on of ant eri or communi cat i ng art ery aneurys m s urgery i s cogni t i ve dys funct i on becaus e of i njury t o bas al forebrai n s t ruct ures . Aft er s urgery, pat i ent s are s t i l l at ri s k for hydrocephal us and hyponat remi a. W i t h s ucces s ful i s ol at i on of t he aneurys m from t he cerebral ci rcul at i on, t he ri s k of rebl eedi ng s houl d be ni l . Of concern at t hi s t i me i s t he devel opment of del ayed i s chemi c neurol ogi c defi ci t s becaus e of cerebral vas os pas m. Thi s compl i cat i on i s t hought t o be at t ri but abl e t o t he effect s of s ubarachnoi d bl ood on t he i nt racrani al art eri es ; t here i s vas ocons t ri ct i on wi t h as s oci at ed hi s t ol ogi c fi ndi ngs of i nt i mal hyperpl as i a and s moot h mus cl e prol i ferat i on. Pat i ent s can devel op i mpai red cons ci ous nes s and focal neurol ogi c defi ci t s . Seri al t rans crani al Doppl er (TCD) s t udi es can s how i ncreas i ng bl ood fl ow vel oci t i es , whi ch can be a harbi nger of s ubs equent cl i ni cal l y s i gni fi cant cerebral vas os pas m. Medi cal management t o prevent s ympt omat i c vas os pas m and s ympt omat i c cerebral i s chemi a i ncl udes ni modi pi ne and “t ri pl e H t herapy―: hemodi l ut i on, hypert ens i on, and hypervol emi a. Opt i mal oxygen t rans port at i on
Pa g e 3 1 1 6
ABC Ambe r CHM Conve rte r Tria l ve rsion, http://w w w .proce sste x t.com/a bcchm.html
occurs at hemat ocri t s i n t he 30–35 range. El evat i on of t he mean art eri al bl ood pres s ure by 10%–20%, t oget her wi t h hypervol emi a, can prevent or t reat cerebral i s chemi a caus ed by vas os pas m. T he patient is cognitively intact after surgery. Blood pressure and serum sodium levels are in an acceptable range. However, serial T CD studies show a progressive elevation of blood flow velocities in the middle and anterior cerebral arteries. T riple H therapy is begun. On the third postoperative day, the patient becomes obtunded, and a mild right hemiparesis develops despite optimal medical management. A brain CT scan shows a left frontal ischemic infarction; there is no evidence of new hemorrhage.
Questions
What i s t he probabl e c aus e of t he pat i ent 's new s t roke?
What management opt i ons mi ght be c ons i dered?
Discussion The pat i ent has s us t ai ned an i s chemi c i nfarct i on caus ed by cerebral vas os pas m. A cerebral angi ogram s houl d be obt ai ned t o defi ne t he ext ent of t he vas os pas m. Int ervent i onal neuroradi ol ogi c approaches t o t he t reat ment of medi cal l y refract ory s ympt omat i c vas os pas m i ncl ude angi opl as t y and i nt ra-art eri al i nfus i on of papaveri ne. T he patient is taken to the angiography suite. Images demonstrate diffuse vasospasm, worse in the left middle cerebral artery. Angioplasty is performed and successfully dilates the left middle cerebral artery. Several papaverine infusions are directed at the distal branches of the anterior and middle cerebral arteries. Over the next several days, the patient has a progressive decline in consciousness and develops a left hemiparesis. Repeat CT scans show multiple areas of ischemic infarction. A repeat angiogram shows severe diffuse vasospasm.
Pa g e 3 1 1 7
ABC Ambe r CHM Conve rte r Tria l ve rsion, http://w w w .proce sste x t.com/a bcchm.html
T wo weeks after her SAH, the patient has lost all evidence of brain function and is declared brain dead.
Comment Des pi t e many advances i n neurol ogi c i nt ens i ve care, aneurys mal SAH remai ns a di s eas e wi t h a mort al i t y rat e approachi ng 50%. Pat i ent s are at ri s k for many compl i cat i ons , and al t hough t hey may appear qui t e i nt act on pres ent at i on, t hei r cl i ni cal cours e can be peri l ous .
Case 5 Deep Venous Thrombosis A 28-year-old man consults his primary physician because of a sudden onset of shortness of breath and right pleuritic chest pain, having returned 3 days before the onset of symptoms from a business trip to T okyo. He has been in excellent health all his life and takes no medications. His medical P.619 history is negative, with the exception of tonsillectomy at age 10 without bleeding complications and an appendectomy at age 18, which was followed 10 days later by an episode of right calf tenderness and swelling that resolved spontaneously without specific therapy. Family history indicated that his father died suddenly at age 35 of a “ heart attack,― and his sister died suddenly of unknown cause at age 25 postpartum. On physical examination, he is a physically fit–appearing man complaining of right-sided chest discomfort on deep inspiration and moderate distress with shortness of breath. Vital signs showed blood pressure (BP) = 110/60 mm Hg; pulse = 114 beats/minute; respirations = 24 breaths/minute; temperature = 37.5°C. Examination of the heart indicates a summation gallop at the apex with a prominent P2 at the base. Auscultation of the lungs discloses a pleural friction rub heard in the right axilla. Remainder of the physical examination is negative, with the exception of tenderness on palpation of the left calf, with pain
Pa g e 3 1 1 8
ABC Ambe r CHM Conve rte r Tria l ve rsion, http://w w w .proce sste x t.com/a bcchm.html
elicited by dorsiflexion of the left foot. Subtle, but definite slight edema was present in the left ankle.
Questions
What i s your di fferent i al di agnos i s and t he mos t l i kel y expl anat i on for hi s s ympt oms ?
What i ni t i al di agnos t i c s t udi es s houl d be c arri ed out i mmedi at el y t o c onfi rm t he di agnos i s ?
Discussion A ches t x-ray was obt ai ned, whi ch s howed no abnormal i t y. An ECG s howed s i nus t achycardi a wi t h cl ockwi s e rot at i on and ri ght axi s devi at i on. An art eri al bl ood s ampl e was obt ai ned, whi ch s howed t he fol l owi ng res ul t s : PaO 2 = 63; PaCO 2 = 31; pH = 7.49, art eri al oxygen s at urat i on, 87%.
Questions
How do you i nt erpret t hes e fi ndi ng?
What di agnos t i c s t udi es s houl d be done next ?
Woul d you c ons i der i ns t i t ut i ng any form of t herapy bas ed on t he res ul t s at t hi s poi nt ?
Woul d you c ons i der obt ai ni ng any furt her di agnos t i c s t udi es before i ns t i t ut i ng t herapy?
Discussion The normal ches t x-ray es s ent i al l y excl udes pneumoni a or ot her
Pa g e 3 1 1 9
ABC Ambe r CHM Conve rte r Tria l ve rsion, http://w w w .proce sste x t.com/a bcchm.html
pul monary parenchymal pat hol ogy as t he caus e for t he pat i ent 's s ympt oms . The ECG s howed onl y nons peci fi c fi ndi ngs but i s cons i s t ent wi t h abnormal i t i es obs erved i n pul monary embol i c di s eas e. The art eri al bl ood gas s t udi es s how evi dence of oxygen des at urat i on, mi l d res pi rat ory al kal os i s , and an abnormal A-a gradi ent t hat , i n t he abs ence of pul monary parenchymal di s eas e, s t rongl y s ugges t s a vent i l at i on/perfus i on abnormal i t y s uch as i s obs erved i n acut e pul monary embol i s m (PE). The correct di agnos t i c procedure i s a vent i l at i on/perfus i on l ung s can or s pi ral CT s canni ng of t he ches t . However, before s endi ng t he pat i ent t o t he radi ol ogy depart ment , i t woul d be appropri at e t o i ns t i t ut e hepari n t herapy i mmedi at el y bas ed on a pres umpt i ve worki ng di agnos i s of PE, becaus e t he next embol us , i f mas s i ve, mi ght be fat al . However, before i ns t i t ut i ng hepari n t herapy, i t woul d be des i rabl e t o obt ai n s at i s fact ori l y col l ect ed bl ood s ampl es for prel i mi nary i nves t i gat i on of a t hrombophi l i c s t at e, becaus e t he res ul t s from s uch a workup are di ffi cul t or i mpos s i bl e t o i nt erpret aft er ant i coagul ant t herapy has been i ns t i t ut ed. Even i n t he face of acut e venous t hromboembol i c di s eas e, s ome of t he coagul at i on-bas ed s t udi es are not s ubject t o defi ni t i ve i nt erpret at i on and t herefore s houl d al s o be repeat ed at a l at er dat e, when t he pat i ent i s as ympt omat i c and not recei vi ng ant i coagul ant t herapy. At t hi s poi nt i n t he workup, venous bl ood s ampl es can be obt ai ned and properl y s t ored for t he fol l owi ng det ermi nat i ons pendi ng t he res ul t s of t he V/Q s can:
Prot ei n C
Prot ei n S
Prot hrombi n 20210
Fact or V Lei den
Pa g e 3 1 2 0
ABC Ambe r CHM Conve rte r Tria l ve rsion, http://w w w .proce sste x t.com/a bcchm.html
Lupus ant i coagul ant /ant i cardi ol i pi n ant i bodi es
Ant i t hrombi n III
Homocys t ei ne
P.620
Of t hes e s t udi es , homocys t ei ne, ant i cardi ol i pi n ant i bodi es , and genet i c t es t i ng for prot hrombi n 20210 and fact or V Lei den wi l l not be affect ed by t he acut e event or ant i coagul ant medi ci nes . In addi t i on, a bas el i ne prot hrombi n t i me and part i al t hrombopl as t i n t i me (PTT) s houl d be obt ai ned before t he i ns t i t ut i on of ant i coagul ant t herapy. Fi nal l y, an as s ay for D-di mer wi l l ai d i n di agnos i s of an acut e t hrombos i s . The V/Q s can s howed a l arge perfus i on defect i n t he ri ght mi ddl e l obe di s t ri but i on wi t h normal vent i l at i on, a res ul t cons i s t ent wi t h a hi gh probabi l i t y for PE. If t he V/Q s can were i ndet ermi nat e or s howed a l ow probabi l i t y of PE, t hen a pul monary angi ogram s houl d be done t o rul e i n or rul e out PE. Al t ernat i vel y a s pi ral CT s can of t he ches t can be obt ai ned t o document t he pres ence or abs ence of PE. Cons i derat i on al s o coul d be gi ven t o carryi ng out Doppl er ul t ras onography t o i nves t i gat e t he pos s i bi l i t y of deep venous t hrombos i s (DVT) as t he s ource of a PE, al t hough, i f t he res ul t s of t he V/Q s can or s pi ral CT defi ni t i vel y es t abl i s h a di agnos i s of PE, f urt her i nves t i gat i on for DVT i s unneces s ary. As s ay for D-di mer was markedl y el evat ed, 500 µg/dL (normal <25 µg/dL), furt her confi rmi ng t he pres ence of acut e t hrombos i s . An el evat ed D-di mer as s ay i s non-s peci fi c, however, and al t hough confi rmat ory, i t i s not di agnos t i c of PE. A normal D-di mer l evel vi rt ual l y excl udes t he pos s i bi l i t y of PE.
Question
Pa g e 3 1 2 1
ABC Ambe r CHM Conve rte r Tria l ve rsion, http://w w w .proce sste x t.com/a bcchm.html
Havi ng es t abl i s hed a di agnos i s of pul monary t hromboembol i s m, w hat furt her t herapy s houl d be i ns t i t ut ed?
Discussion Concurrent admi ni s t rat i on of hepari n t herapy combi ned wi t h an oral ant i coagul ant s uch as Coumadi n s houl d be i ns t i t ut ed. Hepari n s houl d be mai nt ai ned for at l eas t 5 days and warfari n for a vari abl e t i me t hereaft er (s ee bel ow).
Questions
What i s t he rat i onal e for c onc urrent admi ni s t rat i on of hepari n and oral ant i c oagul ant s ?
How are t hes e t w o t herapi es moni t ored?
Discussion The rat i onal e for concurrent admi ni s t rat i on of hepari n and oral ant i coagul ant s i s t he fol l owi ng: Hepari n i nduces i mmedi at e ant i coagul at i on by enhanci ng t he act i vi t y of endogenous ant i t hrombi n III, whereas vi t ami n K ant agoni s t s s uch as Coumadi n (whi ch i nhi bi t t he s ynt hes i s of fact ors VII, IX, and X; prot hrombi n; prot ei n C; and prot ei n S) requi re approxi mat el y 4–6 days t o become ful l y effect i ve bas ed on t he hal f-l i ves of t he vi t ami n K–dependent prot ei ns . However, by decreas i ng t he s ynt hes i s of prot ei n C and prot ei n S, Coumadi n act ual l y can i ni t i al l y enhance t he propens i t y t o t hromboembol i s m, res ul t i ng i n “Coumadi n-i nduced s ki n necros i s ― unl es s gi ven s i mul t aneous l y wi t h hepari n. Abrupt di s cont i nuat i on of hepari n t herapy wi t hout coverage by oral ant i coagul ant s can decreas e t he concent rat i on of ant i t hrombi n III and i s as s oci at ed wi t h a hi gh rel aps e rat e of t hrombos i s . Unfract i onat ed hepari n i s moni t ored by t he PTT, whereas warfari n
Pa g e 3 1 2 2
ABC Ambe r CHM Conve rte r Tria l ve rsion, http://w w w .proce sste x t.com/a bcchm.html
t herapy i s moni t ored by t he PT. Recent l y, t he us e of l ow-mol ecul ar-wei ght forms of hepari n (LMW H) i n pl ace of s t andard hepari n has i ncreas ed. LMW H can be dos ed on a wei ght bas i s wi t h no need t o moni t or PTT, whi ch i s a bi g advant age.
Questions
Why i s fol l ow -up s o i mport ant i n t he management of t hi s pat i ent ?
What s houl d be t he objec t i ve of fol l ow -up management ?
Discussion Thi s pat i ent devel oped DVT and PE at a young age, has a s ugges t i ve pri or pers onal hi s t ory of t hromboembol i s m, and has a s t rong fami l y hi s t ory s ugges t i ve of PE i n fi rs t -degree rel at i ves . Therefore, he i s a l i kel y candi dat e for di agnos i s of an i nheri t ed t hrombophi l i a, s uch as defi ci ency of prot ei n C, prot ei n S, ant i t hrombi n III, t he prot hrombi n 20210 pol ymorphi s m, or fact or V Lei den. St udi es t o i nves t i gat e t hes e pos s i bi l i t i es s houl d be confi rmed ei t her us i ng t he aforement i oned i ni t i al l y obt ai ned s peci mens or aft er a 6-mont h t o 12-mont h peri od of t reat ment wi t h oral ant i coagul ant s , fol l owed by a 1–2-week peri od off oral ant i coagul ant s t o rees t abl i s h a normal l evel of t hes e prot ei ns . If defi ci enci es of one or P.621 more of t hes e prot ei ns are es t abl i s hed, cons i derat i on s houl d be gi ven t o l ong-t erm prophyl axi s wi t h oral ant i coagul ant s . The pat i ent was t reat ed wi t h LMW H and di d wel l . For compl et enes s , a Doppl er ul t ras ound was performed and was pos i t i ve for DVT i n t he l eft l eg. Ini t i al hypercoagul abl e s t at e workup i ndi cat ed fact or V Lei den het eroz ygous s t at e by genet i c t es t i ng. The epi demi ol ogy of t hi s t hrombot i c event fi t s ni cel y i nt o t he current t hi nki ng, wherei n “mul t i pl e hi t s ― are res pons i bl e for mos t event s . In t hi s cas e,
Pa g e 3 1 2 3
ABC Ambe r CHM Conve rte r Tria l ve rsion, http://w w w .proce sste x t.com/a bcchm.html
hi s genet i c s us cept i bi l i t y (fact or V Lei den) combi ned wi t h unus ual venous s t as i s (prol onged pl ane ri de) t o t ri gger a t hromboembol i c compl i cat i on. Ot her t ri ggeri ng event s i ncl ude general anes t hes i a for >1 hour, i mmobi l i zat i on by cas t i ng of t he l eg, and pregnancy and t he pos t part um s t at e. Thi s pat i ent i s current l y on l ong-t erm Coumadi n and i s a candi dat e for i ndefi ni t e ant i coagul at i on becaus e he has an underl yi ng genet i c hypercoagul abl e s t at e and has experi enced mul t i pl e t hrombot i c event s .
Case 6 Newly Discovered Renal Failure A 62-year-old man visits his physician for a routine physical examination. He mentions that he has to get out of bed to urinate more frequently than he used to (two to three times each night) and that he has difficulty initiating and maintaining a urinary stream. In addition, he states that he has experienced mild shortness of breath on walking two to three flights of stairs and that his regular biweekly workouts at the gym have been much more difficult for him lately. Physical examination shows moderate pallor, a blood pressure of 150/105 mm Hg, a pulse of 80 beats/minute, and a respiratory rate of 18 breaths/minute. T here is trace pedal edema. Examination of the chest and heart shows no abnormalities. Examination of the abdomen shows slight fullness in the lower abdomen but neither tenderness nor pain. Laboratory work is obtained, including a complete blood count (CBC), electrolyte, glucose, calcium, and renal studies. T he results of these studies are as follows: blood urea nitrogen (BUN), 88 mg/dL; creatinine, 6.4 mg/dL; sodium, 137 mEq/L; potassium, 5.9 mEq/L; chloride, 112 mEq/L; bicarbonate, 16 mEq/L; hemoglobin, 8.7 g/dL.
Questions
What i s t he fi rs t det ermi nat i on you s houl d make w i t h
Pa g e 3 1 2 4
ABC Ambe r CHM Conve rte r Tria l ve rsion, http://w w w .proce sste x t.com/a bcchm.html
res pec t t o t hi s pat i ent ?
What addi t i onal i nformat i on mi ght hel p you make t hi s det ermi nat i on?
What i s t he mos t l i kel y c aus e of t hi s pat i ent 's renal i ns uffi c i enc y? Does t he pat i ent 's al t ered el ec t rol yt e and ac i d–bas e met abol i s m s upport t hi s di agnos i s ?
Discussion The det ermi nat i on of ons et of renal fai l ure i s cruci al t o det ermi ne t he proper t herapeut i c approach. Acut e renal fai l ure i mpl i es t he pot ent i al for revers i bi l i t y, and t herefore every effort mus t be made t o i dent i fy any ongoi ng i njuri ous agent s t hat advers el y affect t he ki dney. In addi t i on, acut e s upport t hrough s ome form of di al ys i s t herapy may be requi red t o s us t ai n t he pat i ent unt i l renal recovery can occur. Convers el y, t he pres ence of chroni c renal fai l ure s ugges t s t hat t he pat i ent has s us t ai ned i rrevers i bl e i njury. The pat i ent mus t t hen be t reat ed i n a way t o prevent furt her l os s of ki dney funct i on. Chroni c renal repl acement t herapy, ei t her t hrough di al ys i s s upport or t hrough t rans pl ant at i on, may be neces s ary. The det ermi nat i on of chroni ci t y of renal fai l ure can be di ffi cul t . In pat i ent s wi t h acut e renal fai l ure, a cl ear preci pi t at i ng event (e.g., expos ure t o nephrot oxi c agent s s uch as gent ami ci n, s eps i s , s hock) us ual l y can be i dent i fi ed. The det ermi nat i on of t he ons et of chroni c di s eas e i s more probl emat i c. The t i me of i ni t i at i on of s ubt l e s ympt oms cons i s t ent wi t h renal fai l ure (e.g., pruri t us ) may be hel pful i n det ermi ni ng t he ons et of t he di s eas e. Renal ul t ras ound provi des i nformat i on about ki dney s i ze, whi ch t ypi cal l y i s reduced i n chroni c ki dney di s eas e and normal i n acut e renal fai l ure. Except i ons i ncl ude di abet i c nephropat hy and amyl oi dos i s , whi ch may be as s oci at ed wi t h normal or enl arged ki dneys , and pol ycys t i c ki dney di s eas e, whi ch may be as s oci at ed wi t h ext remel y l arge ki dneys des pi t e coexi s t ent renal fai l ure. Chroni c ki dney di s eas e i s t ypi cal l y as s oci at ed wi t h s econdary hyperparat hyroi di s m;
Pa g e 3 1 2 5
ABC Ambe r CHM Conve rte r Tria l ve rsion, http://w w w .proce sste x t.com/a bcchm.html
s ubperi os t eal res orpt i on i s t ypi cal l y s een radi ographi cal l y. In addi t i on, general i zed bone demi neral i zat i on, l os s of bone mas s at t he acromi al –cl avi cul ar joi nt , or mot t l i ng of t he s kul l may be i mport ant fi ndi ngs . Anemi a i s t ypi cal l y s een i n chroni c ki dney di s eas e becaus e eryt hropoi et i n l evel s are reduced as ki dney mas s is P.622 reduced. However, anemi a may devel op rat her rapi dl y, and occas i onal l y, s evere anemi a i s s een i n bot h acut e and chroni c renal fai l ure; i t l i kel y expl ai ns t he dys pnea i n t hi s pat i ent . Thi s pat i ent mos t l i kel y has acut e renal i ns uffi ci ency as a res ul t of chroni c obs t ruct i ve uropat hy. Thi s concl us i on s t ems from t he hi s t ory of di ffi cul t y voi di ng; t he pres ence of bl adder ful l nes s on phys i cal exami nat i on; and t he fi ndi ngs of renal fai l ure, hyperkal emi a, and met abol i c aci dos i s . Ot her common caus es of acut e renal fai l ure i n t he ambul at ory s et t i ng i ncl ude t oxi c nephropat hy from medi cat i on us e and acut e gl omerul onephri t i s . In men, t he mos t common caus e of chroni c obs t ruct i ve nephropat hy i s pros t at i c hypert rophy, but bl adder cancer or pros t at i c cancer i s pos s i bl e as wel l . The pat i ent has hyperchl oremi c hypobi carbonat emi a wi t h as s oci at ed hyperkal emi a. Thi s cons t el l at i on of fi ndi ngs i s common i n pat i ent s wi t h chroni c ki dney di s eas e, part i cul arl y i n as s oci at i on wi t h obs t ruct i ve uropat hy. The hypobi carbonat emi a s ugges t s met abol i c aci dos i s but al s o coul d be caus ed by chroni c res pi rat ory al kal os i s . The pres ence of renal i ns uffi ci ency makes met abol i c aci dos i s t he mos t l i kel y caus e of t he hypobi carbonat emi a i n t hi s pat i ent . The hi gh chl ori de l evel refl ect s t he fact t hat el ect ri cal neut ral i t y mus t be mai nt ai ned by body fl ui ds , and as bi carbonat e l evel s fal l , preferent i al chl ori de reabs orpt i on wi t h s odi um occurs i n t he renal t ubul es . Becaus e t he ki dney i s unabl e t o generat e new bi carbonat e t o res t ore t he l evel s reduced by i nges t i on of di et ary precurs ors of vari ous mi neral aci ds , bi carbonat e l evel s fal l . A reduced bi carbonat e l evel i s t ypi cal of al l forms of renal fai l ure. The i ncreas e i n s erum pot as s i um i s an i mport ant i ndi cat i on of obs t ruct i ve uropat hy. Pot as s i um i s s ecret ed i n t he di s t al nephron
Pa g e 3 1 2 6
ABC Ambe r CHM Conve rte r Tria l ve rsion, http://w w w .proce sste x t.com/a bcchm.html
but requi res adequat e amount s of al dos t erone as wel l as a normal l y res pons i ve di s t al t ubul e pot as s i um t rans port s ys t em for normal excret i on. Chroni c obs t ruct i ve uropat hy i s as s oci at ed wi t h i mpai rment of pot as s i um excret i on, occas i onal l y s econdary t o reduced al dos t erone l evel s but more t ypi cal l y due t o a di rect t ubul ar abnormal i t y. Correct i on of t he underl yi ng obs t ruct i ve proces s l eads t o a normal i zat i on of pot as s i um excret i on and a ret urn of s erum pot as s i um l evel s t o t he normal range. Renal ultrasound is performed and confirms the diagnosis of obstructive uropathy. A Foley catheter is put in place.
Question
What c l i ni c al probl em s houl d be ant i c i pat ed aft er rel i ef of obs t ruc t i on?
Discussion On rel i ef of obs t ruct i on, i t i s t ypi cal for pat i ent s wi t h obs t ruct i ve uropat hy t o undergo a s t ri ki ng di ures i s . The s ource of t he di ures i s i s a combi nat i on of excret i on of previ ous l y ret ai ned s ol ut es and fl ui ds as wel l as a mi l d res i dual t ubul ar defect i n s odi um and wat er cons ervat i on. Thi s mas s i ve di ures i s t ypi cal l y remi t s once t he pat i ent 's BUN and creat i ni ne have l evel ed off. Di al ys i s , by ei t her hemodi al ys i s or peri t oneal di al ys i s , may be neces s ary i n pat i ent s wi t h renal i ns uffi ci ency s econdary t o obs t ruct i ve uropat hy, but at t empt s s houl d al ways be made t o correct t he obs t ruct i ve uropat hy before i ni t i at i ng hemodi al ys i s . The deci s i on t o perform di al ys i s mus t be made on an i ndi vi dual bas i s and defi es s i mpl e cat egori zat i on. Hemodi al ys i s i s i ndi cat ed i n pat i ent s wi t h peri cardi t i s . Conges t i ve heart fai l ure (CHF) al s o may be an i ndi cat i on for di al ys i s t herapy t o achi eve fl ui d removal . Ins t i t ut i on of di al ys i s s houl d not be made on t he bas i s of t he abs ol ut e BUN l evel al one, al t hough mos t cl i ni ci ans bel i eve t hat t he BUN s houl d be mai nt ai ned bel ow 150 mg/dL.
Case 7
Pa g e 3 1 2 7
ABC Ambe r CHM Conve rte r Tria l ve rsion, http://w w w .proce sste x t.com/a bcchm.html
Coma in a Diabetic Patient A 20-year-old college student is rushed to the emergency department by ambulance after being found comatose by her roommate in her dormitory room. She is known to have type 1 (insulin-dependent) diabetes mellitus that has been well controlled by diet and insulin.
Questions
What are your i mmedi at e c onc erns about t hi s pat i ent ?
What are pos s i bl e mec hani s ms of c oma i n a di abet i c pat i ent ?
What i s at t he t op of your di fferent i al di agnos i s ?
P.623
Discussion W hen t he pat i ent fi rs t arri ves i n t he emergency depart ment , an adequat e ai rway s houl d be ens ured, vi t al s i gns s houl d be meas ured t o as s es s ci rcul at ory s t at us , i nt ravenous acces s s houl d be es t abl i s hed, and bl ood s houl d be drawn for chemi s t ri es , art eri al bl ood gas meas urement s , and a fi ngers t i ck gl ucos e det ermi nat i on. Then addi t i onal i nformat i on s houl d be obt ai ned from t he pat i ent 's fri end. The caus es of coma t hat are di rect l y rel at ed t o di abet es are di abet i c ket oaci dos i s , hypogl ycemi a, and hyperos mol ar nonket ot i c coma. The l at t er occurs mai nl y i n el derl y pat i ent s wi t h t ype 2 (non–i ns ul i n-dependent ) di abet es , s o t he fi rs t di agnos t i c i mpres s i ons when t hi s pat i ent arri ves i n t he emergency depart ment are di abet i c ket oaci dos i s and hypogl ycemi a.
Pa g e 3 1 2 8
ABC Ambe r CHM Conve rte r Tria l ve rsion, http://w w w .proce sste x t.com/a bcchm.html
Di abet i c ket oaci dos i s occurs when t here i s a s evere i ns ul i n defi ci ency, whi ch caus es an i nabi l i t y t o ut i l i ze gl ucos e and l eads t o hypergl ycemi a, ket os i s , and aci dos i s . Hypogl ycemi a i n a di abet i c pat i ent i s us ual l y t he res ul t of an i mbal ance bet ween t he fact ors t hat l ower t he bl ood gl ucos e l evel , pri mari l y admi ni s t ered i ns ul i n, oral hypogl ycemi c agent s (i n pat i ent s wi t h t ype 2 di abet es ), and exerci s e, and t he fact ors t hat rai s e t he bl ood gl ucos e l evel , namel y food i nges t i on and hepat i c out put of gl ucos e. Di abet i c ket oaci dos i s and hypogl ycemi a s houl d be rul ed out before s eri ous cons i derat i on i s gi ven t o ot her caus es of coma s uch as drug overdos e, s ei zures , s t roke, or meni ngi t i s . T he roommate recalls that the patient has been complaining of an “ upset stomach― for 2 days, with some nausea, anorexia, mild diarrhea, and increasing abdominal pain. She seemed a bit groggy that morning and did not go to classes. Because she was unable to eat breakfast, she did not take her morning insulin dose. T he roommate returned from class at 3:00 PM and found the patient lying in bed, breathing deeply and unresponsive to questions.
Questions
What ri s k fac t ors does t hi s pat i ent have for t he devel opment of di abet i c ket oac i dos i s ?
What i s t he s i gni fi c anc e of eac h of t he pat i ent 's c ompl ai nt s ?
Discussion The apparent l y gradual ons et over a peri od of hours , t he precedi ng i l l nes s , and t he omi s s i on of an i ns ul i n dos e s t rongl y s ugges t di abet i c ket oaci dos i s rat her t han hypogl ycemi a as t he mos t l i kel y di agnos i s . Il l nes s es s uch as upper res pi rat ory i nfect i on or gas t roent eri t i s may i ncreas e t he need for i ns ul i n becaus e of s t res s -rel at ed i ncreas es i n cat echol ami nes , cort i s ol , and gl ucagon;
Pa g e 3 1 2 9
ABC Ambe r CHM Conve rte r Tria l ve rsion, http://w w w .proce sste x t.com/a bcchm.html
omi t t i ng i ns ul i n dos es becaus e of t he i l l nes s makes mat t ers wors e. The gas t roi nt es t i nal s ympt oms at t he ons et of i l l nes s s ugges t t hat gas t roent eri t i s was t he preci pi t at i ng event , al t hough t he i ncreas i ng abdomi nal pai n coul d have been caus ed al s o by t he devel opi ng ket oaci dos i s . W hy does abdomi nal pai n s omet i mes occur i n di abet i c ket oaci dos i s ? Gas t ri c di s t ent i on may be a fact or, and i t i s known t hat ket os i s from ot her caus es s uch as s t arvat i on may l ead t o gas t roi nt es t i nal s ympt oms s uch as anorexi a, naus ea, and vomi t i ng. But t he preci s e caus e of abdomi nal pai n i n di abet i c ket oaci dos i s i s not known. The groggi nes s t hat t he pat i ent experi enced earl i er i n t he day s ugges t s t hat t he proces s l eadi ng t o ment al obt undat i on had al ready s t art ed. The s ympt oms of di abet i c ket oaci dos i s are t ypi cal l y gradual i n ons et (over s everal hours ), whereas t he ment al changes of hypogl ycemi a commonl y have a rel at i vel y s udden ons et , whi ch i s us ual l y preceded by adrenergi c s ympt oms s uch as s weat i ng, t remor, and pal pi t at i ons . On examination, the patient is unresponsive, with dry skin and mucous membranes and rapid, deep respirations. Her blood pressure is 100/60 mm Hg supine, falling to 80/50 mm Hg when the head of the bed is raised. T he neck veins are collapsed when the patient is lying supine. Her pulse rate is 110 beats/minute, her respiratory rate is 24 breaths/minute, and her temperature is 37°C. She winces when moderate pressure is applied to her abdomen. T he deep tendon reflexes are hypoactive.
Questions
Do t hes e phys i c al fi ndi ngs ai d i n di fferent i at i ng bet w een di abet i c ket oac i dos i s and hypogl yc emi a?
What i s t he s i gni fi c anc e of t he pat i ent 's res pi rat ory pat t ern?
Discussion
Pa g e 3 1 3 0
ABC Ambe r CHM Conve rte r Tria l ve rsion, http://w w w .proce sste x t.com/a bcchm.html
Thes e phys i cal fi ndi ngs are hi ghl y s ugges t i ve of di abet i c ket oaci dos i s . Ins uffi ci ent i ns ul i n act i on makes gl ucos e unavai l abl e t o t he t i s s ues , and t he l i ver res ponds by produci ng ket ones as an al t ernat i ve fuel . The ri s i ng bl ood l evel s of gl ucos e and ket ones caus e an os mot i c di ures i s , produci ng dehydrat i on and P.624 i nt ravas cul ar vol ume depl et i on, wi t h ort hos t at i c hypot ens i on. The os mot i c di ures i s l eads t o uri nary l os s es of pot as s i um (vomi t i ng i ncreas es i t s l os s ) and ot her el ect rol yt es ; hypokal emi a caus es mus cl e weaknes s and decreas ed refl exes and res pi rat ory paral ys i s i f s evere. Abdomi nal pai n and t endernes s , perhaps caus ed by t he ket os i s , may be s evere. The rapi d, deep res pi rat i ons , cal l ed Kus s maul 's res pi rat i ons , are caus ed by s t i mul at i on of t he res pi rat ory cent er by aci dos i s . Thi s l eads t o res pi rat ory al kal os i s , whi ch part i al l y offs et s t he met abol i c aci dos i s . Blood glucose by fingerstick, determined soon after the patient' s arrival in the emergency department, is found to be greater than 400 mg/dL. Urine dipstick testing is strongly positive for glucose and ketones. T reatment is started with an intravenous infusion of normal saline solution at a rate of 1000 mL/hour.
Questions
What i s t he rat i onal e behi nd orderi ng t hes e l aborat ory t es t s ?
What i ni t i al s t eps s houl d be t aken at t hi s poi nt t o manage t hi s pat i ent 's c ondi t i on?
Discussion The el evat ed bl ood and uri ne gl ucos e l evel s and uri ne ket one l evel s confi rm t he di agnos i s of di abet i c ket oaci dos i s . The fi rs t pri ori t y i n t reat ment i s fl ui d repl acement , whi ch can be s t art ed whi l e t he i ni t i al l aborat ory res ul t s are awai t ed. An i nfus i on of normal s al i ne
Pa g e 3 1 3 1
ABC Ambe r CHM Conve rte r Tria l ve rsion, http://w w w .proce sste x t.com/a bcchm.html
(1 L/hour for t he fi rs t 2 hours ) s houl d be gi ven i f i nt ravas cul ar vol ume depl et i on i s s evere, as i ndi cat ed by t he ort hos t at i c hypot ens i on and decreas ed cent ral venous pres s ure (decreas ed neck vei n fi l l i ng). Once t he di agnos i s of di abet i c ket oaci dos i s i s confi rmed, i ns ul i n t herapy s houl d be s t art ed. Intravenous infusion of regular insulin is started at a rate of 5 U/hour. Initial laboratory results include the following serum findings: glucose level of 520 mg/dL; sodium level of 132 mEq/L; potassium level of 3.3 mEq/L; and normal phosphate, calcium, and chloride levels. Blood urea nitrogen (BUN) and creatinine are slightly elevated, as is serum amylase. T he white blood cell 3
(WBC) count is 14,500/mm . Arterial blood gas analysis indicates a pH of 7.20, bicarbonate of 8 mEq/L, and a pattern consistent with high anion gap metabolic acidosis.
Questions
How do t hes e l aborat ory val ues affec t your di fferent i al di agnos i s ?
What i s t he s i gni fi c anc e of t he el evat ed amyl as e?
What i s t he s i gni fi c anc e of t he l eukoc yt os i s ?
What proc es s es are i nc l uded i n your di fferent i al di agnos i s for hi gh ani on gap ac i dos i s ?
Discussion Thes e l aborat ory fi ndi ngs are t ypi cal of moderat el y s evere di abet i c ket oaci dos i s . The met abol i c aci dos i s i s caus ed by t he hepat i c product i on of ket one bodi es , whi ch mus t be buffered by bi carbonat e. Sodi um t ends t o be l ow becaus e of t he os mot i c effect of hypergl ycemi a, whi ch i ncreas es ext racel l ul ar wat er, t hereby
Pa g e 3 1 3 2
ABC Ambe r CHM Conve rte r Tria l ve rsion, http://w w w .proce sste x t.com/a bcchm.html
di l ut i ng s erum s odi um. Pot as s i um, al t hough i ni t i al l y hi gh becaus e of movement out of cel l s , fal l s t o l ow l evel s as uri nary l os s es occur. Al t hough s erum phos phat e may be normal i ni t i al l y, l arge uri nary l os s es oft en l ead t o depl et i on of t ot al body phos phat e, whi ch s houl d be repl aced. The BUN and creat i ni ne t end t o be s l i ght l y i ncreas ed becaus e of t he effect of vol ume depl et i on on renal funct i on. Serum amyl as e oft en i s i ncreas ed, but t hi s us ual l y i s at t ri but ed t o t rans i ent l eakage from t he s al i vary gl ands as wel l as t he pancreas and does not us ual l y i ndi cat e pancreat i t i s . Leukocyt os i s i s common i n di abet i c ket oaci dos i s ; i f i nfect i on i s not pres ent , t he i ncreas ed W BC count may be at t ri but ed t o dehydrat i on, x i ncreas ed gl ucocort i coi d act i vi t y and cat echol ami ne s ecret i on, whi ch occurs i n res pons e t o s t res s and may caus e l eukocyt os i s . The hi gh ani on gap i s caus ed by t he pres ence of an unmeas ured ani on, i n t hi s cas e ket one bodi es . Ot her i mport ant caus es of hi gh ani on gap aci dos i s i ncl ude renal fai l ure, al cohol i c ket oaci dos i s , l act i c aci dos i s , and t oxi c s ubs t ances s uch as s al i cyl at es , et hyl ene gl ycol , and met hanol . After 6 hours of treatment, the patient is awake, breathing comfortably at a rate of 18 respirations per minute, and able to respond to questions. T he serum glucose level is 210 mg/dL, the arterial blood pH is 7.34, and the serum bicarbonate level is 14 mEq/L. Urine ketones are now only weakly positive. P.625
Questions
What ot her t ypes of ac ut e c are mi ght t hi s pat i ent requi re?
Pa g e 3 1 3 3
ABC Ambe r CHM Conve rte r Tria l ve rsion, http://w w w .proce sste x t.com/a bcchm.html
What are s ome of t he pot ent i al c ompl i c at i ons of di abet i c ket oac i dos i s ?
Discussion Becaus e i ns ul i n t herapy i s s t i l l needed t o t reat t he res ol vi ng ket os i s and aci dos i s , gl ucos e s houl d now be added t o t he i nt ravenous fl ui ds t o prevent hypogl ycemi a. Pot as s i um and phos phat e s houl d be repl aced as i ndi cat ed by bl ood val ues . Des pi t e of careful t reat ment , pat i ent s wi t h di abet i c ket oaci dos i s have a mort al i t y rat e of 5%–10%. Deat h may be caus ed by overwhel mi ng i nfect i on, by i rrevers i bl e s hock, or by art eri al t hrombos i s caus i ng myocardi al i nfarct i on (MI) or s t roke. Cerebral edema i s a rare compl i cat i on t hat occas i onal l y occurs i n chi l dren, who appear t o be res pondi ng wel l t o t reat ment and may caus e deat h. A more rapi d decreas e i n gl ucos e l evel s i n bl ood t han i n t he cerebros pi nal fl ui d (CSF) caus es fl ui d t o ent er t he rel at i vel y hyperos mol ar CSF compart ment , l eadi ng t o i ncreas ed i nt racrani al pres s ure (ICP) and cerebral edema.
Case 8 Shortness of Breath in a Young Woman A 38-year-old African American woman comes to the emergency department with a chief complaint of shortness of breath on exertion that has progressively worsened in the last 6 months. Her symptoms have become so severe that she now has difficulty eating secondary to extreme dyspnea, and this has resulted in a 27-pound weight loss. She also notes a severe cough productive of white sputum but denies fevers, chills, or night sweats. Before the onset of symptoms (about 6 months ago), she was in good health.
Questions
What are your i mmedi at e c onc erns i n t hi s pat i ent ?
Pa g e 3 1 3 4
ABC Ambe r CHM Conve rte r Tria l ve rsion, http://w w w .proce sste x t.com/a bcchm.html
How does t he ons et of s ympt oms hel p formul at e a di fferent i al di agnos i s ?
What are t he mos t l i kel y di agnos t i c c ons i derat i ons ?
Discussion On i ni t i al eval uat i on of t hi s pat i ent i n t he emergency depart ment , care mus t be t aken t o ens ure t hat s he has an adequat e ai rway. Her vi t al s i gns mus t al s o be as s es s ed, wi t h s peci fi c at t ent i on t o res pi rat ory rat e, heart rat e, and t emperat ure. Suppl ement al oxygen s houl d be provi ded t o ens ure an oxygen s at urat i on of great er t han 94%. Bl ood s houl d be drawn for art eri al bl ood gas anal ys i s , compl et e bl ood count (CBC), and chemi s t ri es . A ches t radi ograph (pos t eroant eri or and l at eral vi ews ) and el ect rocardi ogram (ECG) al s o s houl d be ordered. The caus es of s hort nes s of breat h i n t hi s age group can i ncl ude acut e and chroni c i nfect i ons ; cardi ac mal format i ons , i ncl udi ng val vul ar dys funct i on; anemi a; and chroni c l ung di s eas e, i ncl udi ng i nt ers t i t i al l ung di s eas e. In general , cardi ac caus es are eas y t o excl ude i n t wo s i t uat i ons : (1) when t he pat i ent has no hi s t ory of heart probl ems at bi rt h (i .e., congeni t al ) or i n earl y chi l dhood, and (2) when t he cardi ac exami nat i on i s normal . In addi t i on, anemi a can be excl uded when t here i s no hi s t ory of cont i nui ng heavy mens t rual bl ood l os s or gas t roi nt es t i nal hemorrhage. The durat i on of t hi s pat i ent 's s ympt oms makes an acut e i nfect i on very unl i kel y and s ugges t s a chroni c i nfect i on or one of t he i nt ers t i t i al l ung di s eas es . However, t he l ack of a pers i s t ent fever, chi l l s , or recurrent ni ght s weat s makes chroni c i nfect i on l es s probabl e, al t hough acqui red i mmunodefi ci ency s yndrome (AIDS)–rel at ed i nfect i ons s uch as Pneumoc ys t i s c ari ni i pneumoni a (PCP) can pres ent wi t h a pauci t y of cons t i t ut i onal s ympt oms . Furt hermore, t hi s pat i ent i s at part i cul ar demographi c ri s k for t he devel opment of s arcoi dos i s , a chroni c granul omat ous di s order of unknown caus e t hat commonl y affect s young Afri can Ameri can women.
Pa g e 3 1 3 5
ABC Ambe r CHM Conve rte r Tria l ve rsion, http://w w w .proce sste x t.com/a bcchm.html
Ot her di agnos t i c cons i derat i ons i ncl ude pul monary l ymphangi ol ei omyomat os i s , hypers ens i t i vi t y pneumoni t i s , eos i nophi l i c granul oma, i di opat hi c pul monary fi bros i s , and i nt ers t i t i al l ung di s eas e s econdary t o connect i ve t i s s ue di s orders or vas cul i t i s . Pul monary l ymphangi ol ei omyomat os i s , a di s order charact eri zed by prol i ferat i on of at ypi cal s moot h mus cl e cel l s , l eads t o parenchymal l ung di s eas e, pl eural effus i ons , hemopt ys i s , and pneumot horax; however, i t i s exceedi ngl y rare. Hypers ens i t i vi t y P.626 pneumoni t i s al s o can occur i n younger i ndi vi dual s aft er expos ure t o a vari et y of occupat i onal and envi ronment al t ri ggers . Al t hough i t i s us ual l y s el f-l i mi t ed, i t can become chroni c. Eos i nophi l i c granul oma, a rare di s order, i s charact eri zed by abnormal prol i ferat i on of hi s t i ocyt es , l eadi ng t o i nfi l t rat i ve l ung di s eas e i n t he t hi rd and fourt h decades of l i fe. Thi s condi t i on, whi ch occurs predomi nant l y i n s mokers , i s al mos t excl us i vel y a di s eas e of whi t es . Idi opat hi c pul monary fi bros i s oft en caus es i ns i di ous , progres s i ve res pi rat ory i mpai rment , but i t i s uncommon i n pat i ent s younger t han 40 years ol d. Pul monary mani fes t at i ons of vas cul i t i s or connect i ve t i s s ue di s orders , however, are mos t common i n younger pat i ent s . T hree months before the woman entered the hospital, she had to stop working because of severe exercise limitation. She has no history of occupational exposure to dusts or chemicals. A married woman, she has been in a monogamous relationship for many years. A test for human immunodeficiency virus (HIV) 3 months ago was negative. She is a Jehovah' s Witness, denies use of tobacco or illicit drugs, and has lived in an urban neighborhood all her life. She denies skin rash, chest pain, or arthralgias but admits to seeing “ spots before her eyes― and having occasional nausea and vomiting.
Questions
How does t he pat i ent 's s oc i al and oc c upat i onal hi s t ory
Pa g e 3 1 3 6
ABC Ambe r CHM Conve rte r Tria l ve rsion, http://w w w .proce sste x t.com/a bcchm.html
as s i s t i n narrow i ng t he di fferent i al di agnos i s ?
What i s t he s i gni fi c anc e of t he pat i ent 's c ompl ai nt s ?
Discussion The s oci al and fami l y hi s t ory cont ri but es more t o rul i ng out ot her caus es of chroni c l ung di s eas e t han es t abl i s hi ng a defi ni t i ve di agnos i s . Speci fi cal l y, s arcoi dos i s i s more common among nons mokers , i n cont ras t t o eos i nophi l i c granul oma, whi ch i s al mos t excl us i vel y a di s eas e of s mokers . Furt hermore, t he pat i ent 's monogamous rel at i ons hi p, oppos i t i on t o bl ood t rans fus i on on rel i gi ous grounds , and abs t i nence from i nt ravenous drug us e, as wel l as t he negat i ve HIV t es t , put her at a much l ower ri s k for PCP i nfect i on, whi ch us ual l y occurs i n pat i ent s wi t h wel l -es t abl i s hed HIV di s eas e. The abs ence of a s i gni fi cant t ravel hi s t ory al s o hel ps el i mi nat e chroni c fungal i nfect i on as a di agnos t i c pos s i bi l i t y. Anot her pert i nent negat i ve fact or i s t he l ack of any s i gni fi cant envi ronment al or occupat i onal expos ures , whi ch makes hypers ens i t i vi t y pneumoni t i s l es s l i kel y. However, t hi s condi t i on cannot be excl uded compl et el y, becaus e cert ai n expos ures are qui t e s ubt l e or t emporal l y remot e; t hey can be mi s s ed wi t hout a ri gorous expos ure hi s t ory. A careful revi ew of s ys t ems i s cri t i cal t o eval uat e for di s orders t hat can affect mul t i pl e organ s ys t ems . Ocul ar i nvol vement can l ead t o phot ophobi a, t eari ng, pai n, and l os s of vi s i on. Gas t roi nt es t i nal i nvol vement can res ul t i n dys phagi a, abdomi nal pai n, jaundi ce, and naus ea and vomi t i ng. Infi l t rat i on of ot her organ s ys t ems can l ead t o s ki n ras hes , s yncope, i rregul ar heart rhyt hm, ches t pai n, poor uri ne out put , joi nt pai n, and l ymph node enl argement . Di s orders t hat are part i cul arl y prone t o mani fes t i n mul t i pl e organ s ys t ems i ncl ude t he connect i ve t i s s ue di s orders s uch as s ys t emi c l upus eryt hemat os us (SLE) and s cl eroderma, as wel l as t he vas cul i t i des , i ncl udi ng W egener's granul omat os i s . In addi t i on, s arcoi dos i s may occur i n al mos t any organ s ys t em of t he body. On examination, the woman is in mild distress secondary to
Pa g e 3 1 3 7
ABC Ambe r CHM Conve rte r Tria l ve rsion, http://w w w .proce sste x t.com/a bcchm.html
dyspnea. Her pulse is 104 beats/minute, her respiratory rate is 28 breaths/minute, her temperature is 35.8°C, and her blood pressure is 140/80 mm Hg. Her mucous membranes are dry. T here is jugular venous distention with no significant adenopathy. She is not using accessory muscles of respiration, and lung examination shows coarse breath sounds bilaterally without wheezing, rales, dullness to percussion, or egophony. Cardiac examination indicates tachycardia, with a 2/6 systolic murmur at the left sternal border and an S 3 at the right sternal border. T here is no hepatosplenomegaly. T he extremities show 1+ pitting edema without clubbing, and there is no skin rash. Neurologic examination is unremarkable.
Question
What i s t he s i gni fi c anc e of t he fi ndi ngs on phys i c al exami nat i on?
Discussion Al t hough many pat i ent s wi t h i nt ers t i t i al l ung di s eas e pres ent wi t h cl i ni cal l y s i l ent pul monary i nvol vement , s ome exhi bi t advanced l ung s carri ng. Phys i cal fi ndi ngs , whi ch are oft en abs ent earl y i n t he di s eas e, P.627 become apparent onl y when advanced pul monary hypert ens i on and cor pul monal e have devel oped. Thes e fi ndi ngs i ncl ude t he pres ence of a t ri cus pi d regurgi t at i on murmur, a ri ght -s i ded S 3 , a ri ght vent ri cul ar or s t ernal heave, and conges t i on of t he l i ver and l ower ext remi t i es . In addi t i on, pat i ent s wi t h i nt ers t i t i al l ung di s eas e general l y pres ent wi t h i ns pi rat ory ral es , or crackl es . However, t hes e are l es s l i kel y t o be heard i n pat i ent s wi t h s arcoi dos i s or ot her granul omat ous i nt ers t i t i al l ung di s eas es . In addi t i on, a careful s earch for any ext rapul monary mani fes t at i ons s houl d be undert aken.
Pa g e 3 1 3 8
ABC Ambe r CHM Conve rte r Tria l ve rsion, http://w w w .proce sste x t.com/a bcchm.html
T he patient' s initial laboratory findings include a pH of 7.44, a PaCO 2 of 36.2 mm Hg, and a PaO 2 of 52 mm Hg on room air. Blood urea nitrogen (BUN), creatinine, serum calcium, and serum albumin are normal. Chest radiographs (posteroanterior and lateral views) show bilateral reticulonodular interstitial infiltrates, with widening of the right paratracheal region and fullness of both hila, suggestive of mediastinal lymphadenopathy. Minimal cardiomegaly is also noted. An ECG shows normal sinus rhythm with right atrial enlargement. Spirometry and lung volume analysis obtained shortly thereafter indicate a forced vital capacity (FVC) of 1.25 L (43% predicted), a forced expiratory volume in 1 second (FEV 1 ) of 1.02 L (43% predicted), and an FEV 1 /FVC ratio of 82%. T he total lung capacity (T LC) is 1.26 L (44%), and the diffusing capacity (DL C O ) is 7.3 mL/min/mm Hg (29% predicted).
Questions
What i s t he s i gni fi c anc e of t he radi ographi c fi ndi ngs ?
How w oul d you i nt erpret t he pul monary func t i on t es t s ?
What furt her t es t s s houl d be performed t o hel p make t he di agnos i s ?
Discussion The radi ographi c fi ndi ngs confi rm t he pres ence of i nt ers t i t i al l ung di s eas e; i n fact , t hey are qui t e s ugges t i ve of pul monary s arcoi dos i s . Int rat horaci c l ymphadenopat hy, whi ch occurs i n 75%–90% of pat i ent s wi t h s arcoi dos i s , i s us ual l y pres ent i n t he bronchopul monary, t racheobronchi al , and parat racheal chai ns . Thi s cont ras t s wi t h l ymphangi ol ei omyomat os i s , hypers ens i t i vi t y pneumoni t i s , connect i ve t i s s ue di s orders , and vas cul i t i s , i n whi ch i nt rat horaci c l ymphadenopat hy i s di s t i nct l y uncommon. The l ung
Pa g e 3 1 3 9
ABC Ambe r CHM Conve rte r Tria l ve rsion, http://w w w .proce sste x t.com/a bcchm.html
parenchyma al s o may be i nvol ved i n s arcoi dos i s ; affect ed pat i ent s us ual l y demons t rat e bi l at eral ret i cul onodul ar i nfi l t rat es , al t hough a wi de vari et y of ot her parenchymal abnormal i t i es , i ncl udi ng bul l ous changes , al veol ar i nfi l t rat es , peri pheral nodul es , and fi bros i s or “honeycombi ng,― may be pres ent . The Int ernat i onal Congres s on Sarcoi dos i s has propos ed t he fol l owi ng radi ographi c cl as s i fi cat i on of t he di s eas e: St age 0, abs ence of abnormal i t i es ; s t age 1, l ymph node enl argement wi t hout parenchymal abnormal i t i es ; s t age 2, l ymph node enl argement and di ffus e parenchymal di s eas e; s t age 3, di ffus e parenchymal di s eas e wi t hout adenopat hy; and s t age 4, pul monary fi bros i s . The fi ndi ngs i n t hi s pat i ent woul d be cons i s t ent wi t h s t age 2 di s eas e. The pul monary funct i on t es t s i ndi cat e a markedl y reduced TLC, whi ch i s s ugges t i ve of s evere res t ri ct i ve l ung di s eas e wi t h a s i gni fi cant i mpai rment i n t he DLcoand res t i ng art eri al hypoxemi a. Thes e fi ndi ngs correl at e wi t h t he radi ographi c abnormal i t i es jus t des cri bed and i ndi cat e an ext ens i ve parenchymal proces s . However, ot her di ffus e i nt ers t i t i al proces s es can produce s i mi l ar phys i ol ogi c derangement s , l i mi t i ng t he s peci fi ci t y of t he t es t res ul t s . Of not e, ai rfl ow obs t ruct i on res ul t i ng from granul omat ous i nfi l t rat i on of t he ai rways may be evi dent i n s ome pat i ent s but i s abs ent i n t hi s pat i ent who has a normal FEV 1 /FVC rat i o. Maki ng a defi ni t i ve di agnos i s of s arcoi dos i s can be chal l engi ng. Ti s s ue di agnos i s , whi ch i s oft en purs ued, may be fal s el y pos i t i ve, es peci al l y i f l ymph node or l i ver bi ops y i s performed. W hen radi ographi c evi dence of parenchymal di s eas e i s pres ent , as i n t hi s pat i ent , bronchos copy wi t h t rans bronchi al bi ops y i s preferred; t he s ens i t i vi t y of t hi s met hod i s 85%–90%. In cert ai n pat i ent s , s ome noni nvas i ve s t udi es may be us eful . Thes e i ncl ude t he s erum angi ot ens i n-convert i ng enz yme (ACE) l evel , percent age of l ymphocyt es i n bronchoal veol ar l avage (BAL) fl ui d, and gal l i um l ung s canni ng. T he patient undergoes fiberoptic bronchoscopy and transbronchial lung biopsy. Review of the pathologic specimens indicates the presence of multiple noncaseating granulomas. Stains and cultures for acid-fast bacilli and fungi are negative.
Pa g e 3 1 4 0
ABC Ambe r CHM Conve rte r Tria l ve rsion, http://w w w .proce sste x t.com/a bcchm.html
T he serum ACE level is elevated at 111 U/L (range: 8–52). T he echocardiogram is significant for a severely dilated right ventricle with preserved left ventricular function and an estimated pulmonary artery systolic pressure of 90 mm Hg. Moderate tricuspid regurgitation is present. P.628
Questions
What i s t he s i gni fi c anc e of t he pat hol ogi c fi ndi ngs and t he el evat ed s erum ACE l evel ?
How do you i nt erpret t he res ul t s of ec hoc ardi ography?
What t reat ment opt i ons are avai l abl e?
Discussion The pres ence of noncas eat i ng granul omas i n t he correct cl i ni cal s cenari o i s vi rt ual l y pat hognomoni c for s arcoi dos i s . However, s i mi l ar granul omas may occur i n a wi de vari et y of ot her di s eas es , i ncl udi ng t ubercul os i s , fungal i nfect i on, mal i gnancy, beryl l i os i s , and forei gn body react i on. Al t hough t he ACE l evel s upport s t he di agnos i s , i t i s rel at i vel y nons peci fi c and may be el evat ed i n pat i ent s wi t h t ubercul os i s , hi s t opl as mos i s , l epros y, and HIV i nfect i on. Nevert hel es s , t he ACE l evel may be a us eful marker of di s eas e act i vi t y. It has been s hown t hat a progres s i ve decreas e i n t he ACE l evel may refl ect s pont aneous remi s s i on of di s eas e or res pons e t o t herapy. The res ul t s of echocardi ography are cons i s t ent wi t h t he pres ence of s evere pul monary hypert ens i on and cor pul monal e. Thes e are expect ed s equel ae of s evere, di ffus e parenchymal i nvol vement and chroni c hypoxemi a. The pres ence of s uch ext ens i ve di s eas e i n vi t al
Pa g e 3 1 4 1
ABC Ambe r CHM Conve rte r Tria l ve rsion, http://w w w .proce sste x t.com/a bcchm.html
organs s uch as t he l ungs and heart [and t he eyes or cent ral nervous s ys t em (CNS)] mandat es t he i ns t i t ut i on of defi ni t i ve t herapy. In pat i ent s wi t h as ympt omat i c di s eas e or i nvol vement of nonvi t al organs , t herapy may be wi t hhel d and pat i ent s cl os el y fol l owed. The rat i onal e behi nd t hi s approach i s t he obs ervat i on t hat 30%–50% of cas es s pont aneous l y remi t i n a peri od of up t o 3 years , wi t h an addi t i onal 20%–30% of cas es remai ni ng rel at i vel y s t abl e over t he s ame peri od. The mos t s ucces s ful t herapy for s arcoi dos i s has been t he admi ni s t rat i on of s ys t emi c cort i cos t eroi ds s uch as predni s one at a dos age of 20–60 mg by mout h dai l y. If pat i ent s res pond, t he t reat ment i s cont i nued at t he l owes t pos s i bl e dos e t o mai nt ai n remi s s i on of di s eas e for a peri od of s everal weeks or mont hs before s t oppi ng. Occas i onal l y, di s eas e recurs wi t h t aperi ng of t he s t eroi d dos e, and an upward adjus t ment i s requi red. Unfort unat el y, t he us e of s t eroi ds can l ead t o mul t i pl e unt oward effect s , i ncl udi ng t he devel opment of os t eoporos i s , cat aract s , di abet es , hypert ens i on, ps ychi at ri c di s t urbances , and i mmunos uppres s i on. Al t hough t here has been much effort t o devel op al t ernat i ve t reat ment regi mens , i ncl udi ng chl oroqui ne and met hot rexat e, t hes e have met wi t h very l i mi t ed s ucces s .
Case 9 Syncope A 64-year-old man is walking around a shopping mall with his wife when he suddenly loses consciousness and falls to the ground. An ambulance brings him to the emergency department. On arrival to the hospital, the man is awake and alert and complaining only of pain in his right elbow, which he apparently injured when he fell. He is placed on a cardiac monitor, and blood is drawn for laboratory work, which includes a complete blood count (CBC) and electrolyte, glucose, calcium, and cardiac enzyme studies.
Questions
Pa g e 3 1 4 2
ABC Ambe r CHM Conve rte r Tria l ve rsion, http://w w w .proce sste x t.com/a bcchm.html
What i s t he defi ni t i on of s ync ope?
What are s ome of t he c aus es of a s udden l os s of c ons c i ous nes s ?
Discussion Syncope i s a s udden, t emporary l os s of cons ci ous nes s caus ed by a l ack of cerebral perfus i on. The caus es of s udden l os s of cons ci ous nes s can be di vi ded i nt o t hree major cat egori es —cardi ovas cul ar, neurol ogi c, and met abol i c. Cardi ovas cul ar s yncope occurs when t he cardi ovas cul ar s ys t em fai l s t o mai nt ai n adequat e bl ood pres s ure for cerebral perfus i on. Inadequat e s t roke vol ume, i nadequat e heart rat e, or i nadequat e t ot al peri pheral res i s t ance al l coul d be caus es of cardi ovas cul ar s yncope. Impai rment i n s t roke vol ume s uffi ci ent l y enough t o caus e s yncope may be s een i n i s chemi a and myocardi al i nfarct i on (MI), dehydrat i on or hemorrhage, or as a res ul t of t achyarrhyt hmi as , whi ch i mpai r vent ri cul ar fi l l i ng. Mi t ral , aort i c, or i di opat hi c hypert rophi c s ubaort i c s t enos i s , at ri al myxoma, and pul monary embol i s m al s o may caus e s yncope by obs t ruct i ng cardi ac i nfl ow or out fl ow. Severe bradycardi a (i .e., heart rat e <40 beat s /mi nut e) s uch as occurs i n heart bl ock and s i ck s i nus s yndrome may caus e s yncope. In s ome ci rcums t ances , cardi ac out put i s adequat e, but t ot al P.629 peri pheral res i s t ance i s reduced, l eadi ng t o a decreas e i n bl ood pres s ure. Aut onomi c dys funct i on or t he us e of vas odi l at ors may reduce t ot al peri pheral res i s t ance t o t he ext ent t hat s yncope res ul t s . A common mani fes t at i on of s yncope i nduced by reduced t ot al peri pheral res i s t ance i s t he “vas ovagal fai nt ,― whi ch affect s approxi mat el y 50% of t he popul at i on at s ome poi nt duri ng t he cours e of a l i fet i me. Head t rauma, s t roke, or cerebrovas cul ar di s eas e may al l caus e l os s of cons ci ous nes s . Sudden i ncreas es i n i nt racrani al pres s ure (ICP)
Pa g e 3 1 4 3
ABC Ambe r CHM Conve rte r Tria l ve rsion, http://w w w .proce sste x t.com/a bcchm.html
can compromi s e cerebral perfus i on and caus e a l os s of cons ci ous nes s , even i n t he s et t i ng of normal s ys t emi c bl ood pres s ure. A cl as s i c exampl e i s a col l oi d cys t of t he t hi rd vent ri cl e t hat act s as a bal l val ve and caus es s udden obs t ruct i ve hydrocephal us . Onl y rarel y do di s orders of t he cerebrovas cul ar s ys t em compromi s e perfus i on t o s uch an ext ent as t o caus e s yncope. Therefore, s yncope s houl d not be vi ewed as a us ual mani fes t at i on of a t rans i ent i s chemi c at t ack (TIA). Rarel y, pat i ent s wi t h compromi s e of di s t al bas i l ar art ery ci rcul at i on experi ence a s udden l os s of cons ci ous nes s becaus e of i mpai red perfus i on of t he as cendi ng ret i cul ar act i vat i ng s ys t em. Sei z ures are commonl y cons i dered i n t he di fferent i al di agnos i s of s yncope. However, s ei zures i n adul t s rarel y caus e a l os s of cons ci ous nes s t hat i s not as s oci at ed wi t h repet i t i ve mot or act i vi t y. It s houl d be not ed t hat s ome pat i ent s wi t h s yncope at t ri but abl e t o cardi ac or ci rcul at ory fai l ure have s ome general i zed i nvol unt ary mot or act i vi t y res ul t i ng from i mpai red cerebral perfus i on, but t hi s mot or act i vi t y does not repres ent a pri mary convul s i ve event . Fi nal l y, i mpai red del i very of es s ent i al nut ri ent s t o t he brai n (e.g., as a res ul t of hypogl ycemi a or hypoxi a) can caus e a s udden l os s of cons ci ous nes s . However, met abol i c di s t urbances us ual l y wors en gradual l y and caus e obt undat i on before l os s of cons ci ous nes s ; t herefore, t he t rue defi ni t i on of s yncope i s not general l y met . Additional history indicates that the fall was not preceded by an aura and that the patient remained continent during the episode. He awoke spontaneously and was not confused on awakening. T he patient has had several previous episodes of light-headedness during exertion but says he never lost consciousness during those episodes. T he patient does not recall anything unusual about the day before his syncopal episode, except that he experienced mild angina that was immediately relieved by a single sublingual nitroglycerin tablet.
Questions
Pa g e 3 1 4 4
ABC Ambe r CHM Conve rte r Tria l ve rsion, http://w w w .proce sste x t.com/a bcchm.html
What i s s i gni fi c ant about t he ons et and rec overy of t hi s s ync opal epi s ode?
Bas ed on t he above l i mi t ed hi s t ory, w hat proc es s do you mos t s us pec t as a c aus e of t hi s pat i ent 's s ync ope?
Discussion The abrupt ons et of l os s of cons ci ous nes s i s charact eri s t i c of a cardi ac caus e. There were no warni ng s ympt oms t o s ugges t t he aura of a s ei zure; no di s cret e neurol ogi c s ympt oms charact eri s t i c of a TIA; and no di aphores i s , naus ea, and wooz i nes s s uch as woul d be as s oci at ed wi t h a vas ovagal or hypogl ycemi c epi s ode. That t he pat i ent remai ned cont i nent duri ng t he event s upport s a cardi ac or vas ovagal caus e, but t hi s i nformat i on i s probabl y not very hel pful i n deci s i on maki ng; t he cont i nence of a pat i ent duri ng a s ei zure i s more l i kel y rel at ed t o whet her t he pat i ent 's bl adder i s ful l at t he t i me of t he event . The l ack of confus i on on awakeni ng argues agai ns t a s ei zure epi s ode, al t hough pat i ent s wi t h at oni c s ei zures can very rapi dl y regai n normal cogni t i on. Pat i ent s wi t h a compl ex part i al s ei zure do have a peri od of confus i on oft en l as t i ng s everal mi nut es . An i mport ant ques t i on t o as k t he pat i ent 's wi fe i s whet her her hus band di s pl ayed any i nvol unt ary mot or act i vi t y whi l e uncons ci ous . If he remai ned l i mp, t hi s woul d argue s t rongl y agai ns t t he event bei ng a s ei zure. The pat i ent 's hi s t ory of s everal pri or epi s odes of l i ght -headednes s duri ng exert i on s ugges t s an underl yi ng abnormal i t y of t he cardi ovas cul ar s ys t em. In part i cul ar, t hi s hi s t ory i n an ol der pat i ent s houl d di rect at t ent i on t o t he heart . Two fact s rei nforce t hi s i mpres s i on: (1) t he pat i ent apparent l y has a hi s t ory of angi na and (2) he has been t aki ng ni t rogl yceri n. In s ummary, a cardi ac caus e of s yncope i s hi ghl y l i kel y i n t hi s pat i ent , becaus e of t he abrupt ons et of t he l os s of cons ci ous nes s and t he hi s t ory of angi na. The mos t l i kel y cardi ac caus es of s yncope, gi ven t hi s hi s t ory, i ncl ude i s chemi a, vent ri cul ar t achyarrhyt hmi a, or l eft vent ri cul ar out fl ow t ract obs t ruct i on.
Pa g e 3 1 4 5
ABC Ambe r CHM Conve rte r Tria l ve rsion, http://w w w .proce sste x t.com/a bcchm.html
Aut onomi c dys funct i on or vol ume depl et i on are l es s l i kel y. T he patient' s medical history is significant because of sporadic bouts of angina, for which he takes nitroglycerin, which he borrows from his wife. T he angina has not recently increased in frequency or intensity. T he patient has had a heart murmur for many years but has been told that it is not P.630 significant. He takes no regular medications. His father died of a heart attack at age 60, and his mother had a mild stroke at 70 but did not die until 10 years later.
Question
What i s t he s i gni fi c anc e of t he pat i ent 's angi na, fami l y hi s t ory, and heart murmur?
Discussion The pat i ent 's angi na s ugges t s t he pos s i bi l i t y of coronary art ery di s eas e, and furt her eval uat i on by ei t her s t res s t es t i ng or cardi ac cat het eri zat i on i s warrant ed t o as s es s t he degree and s everi t y of at heros cl eros i s . The s i gni fi cance of t he fami l y hi s t ory i s not very hel pful . Al t hough a “heart at t ack― at age 60 i n t he pat i ent 's fat her does s ugges t premat ure coronary di s eas e, many pat i ent s who are l abel ed as heart at t ack vi ct i ms have not t rul y s us t ai ned an MI. The pat i ent 's mot her's s t roke at age 70 mi ght repres ent at heros cl erot i c di s eas e (at a rel at i vel y advanced age) or a hemorrhagi c event . Cert ai nl y, t he pat i ent 's hi s t ory rai s es t he pos s i bi l i t y t hat at heros cl eros i s of t he coronary or cerebral art eri al ci rcul at i on i s caus i ng t he pat i ent 's s ympt oms . Al t hough s t roke or TIA due t o l arge art ery occl us i ve di s eas e rarel y i s a caus e of s yncope, s ome pat i ent s wi t h bi l at eral s evere i nt ernal carot i d art ery s t enos i s may have s yncopal event s . The s i gni fi cance of t he murmur i s uncl ear from t he hi s t ory al one, but i t s ugges t s t hat underl yi ng s t ruct ural cardi ac di s eas e coul d be
Pa g e 3 1 4 6
ABC Ambe r CHM Conve rte r Tria l ve rsion, http://w w w .proce sste x t.com/a bcchm.html
cont ri but i ng t o t he pat i ent 's s ympt oms . Al t hough i t i s very l i kel y t hat t hi s pat i ent has s uffered s yncope from a cardi ovas cul ar caus e, t he addi t i onal i nformat i on does not hel p s way our cl i ni cal judgment one way or t he ot her. It adds s ome “s oft ― feat ures s ugges t i ve of coronary art ery or val vul ar heart di s eas e, whi ch mi ght i n t urn be res pons i bl e for a cardi ac arrhyt hmi a l eadi ng t o s yncope. However, t hi s addi t i onal hi s t ory i s al s o ent i rel y cons i s t ent wi t h t he abs ence of s i gni fi cant heart di s eas e. Physical examination shows a well-developed, well-nourished white man in no acute distress. His vital signs include a pulse of 88 beats/minute, respiratory rate of 14 breaths/minute, blood pressure of 108/74 mm Hg, and a temperature of 37.2°C. T he patient' s skin is warm and dry, his pupils are equal and reactive to light and accommodation, and his lungs are clear. Cardiac examination indicates a normal S 1 , a soft S 2 , and an S 4 . He has a harsh, late-peaking systolic ejection murmur that is loudest in the aortic area and radiates to both carotids, which demonstrate delayed upstroke and low volume. T here are no carotid bruits and there is no jugular venous distention. Examination of the abdomen and extremities shows no abnormalities, with the exception of a bruise on the right elbow. Neurologic examination is completely within normal limits.
Questions
Whi c h of t hes e s i gns and s ympt oms are s i gni fi c ant ?
How does t hi s phys i c al exami nat i on i nfl uenc e t he di fferent i al di agnos i s ?
What s t udi es w oul d you l i ke t o obt ai n?
Discussion
Pa g e 3 1 4 7
ABC Ambe r CHM Conve rte r Tria l ve rsion, http://w w w .proce sste x t.com/a bcchm.html
The pat i ent 's appearance does not s ugges t a s ys t emi c i l l nes s . Obt ai ni ng s upi ne and upri ght bl ood pres s ure and pul s e det ermi nat i ons i s i mport ant i n l i ght of t he pat i ent 's rel at i vel y l ow bl ood pres s ure. The pat i ent does have a s l i ght l y hi gh pul s e, whi ch s ugges t s dehydrat i on. That hi s s ki n i s dry al s o rai s es t he pos s i bi l i t y of dehydrat i on, as wel l as aut onomi c fai l ure wi t h res ul t ant pos t ural hypot ens i on. Hi s pres erved pupi l l ary accommodat i on at t es t s t o pres ervat i on of paras ympat het i c nervous funct i on but l eaves open t he pos s i bi l i t y of s ympat het i c nervous s ys t em fai l ure. The hars h, l at e peaki ng, s ys t ol i c eject i on murmur, t he del ayed and reduced carot i d ups t rokes , and t he s oft S 2 are hi ghl y s ugges t i ve of aort i c s t enos i s , an i mport ant caus e of s yncope. The l ack of carot i d brui t s mus t be cons i dered wi t hi n t he cont ext of a l oud precordi al murmur t hat i s al ready radi at i ng i nt o t he neck; cri t i cal carot i d s t enos es can be mi s s ed becaus e of mas ki ng s ounds (as i n t hi s pat i ent ). Pat i ent s wi t h preoccl us i ve s t enos i s at t he i nt ernal carot i d art ery ori gi n may not have enough forceful perfus i on t o generat e an audi bl e brui t . The i nformat i on from t he exami nat i on di rect s at t ent i on t o t he heart . Of part i cul ar concern i s t he pos s i bi l i t y of s t ruct ural heart di s eas e as a caus e of i mpai red aort i c area out fl ow. Gi ven t he pat i ent 's hi s t ory, cons i derat i on al s o s houl d be gi ven t o t he pos s i bi l i t y of as s oci at ed coronary art ery di s eas e and, l es s l i kel y, carot i d art ery di s eas e. P.631
Us ual l y t he next s t udi es t hat are ordered i n pat i ent s s us pect ed of havi ng cardi ac di s eas e are t he el ect rocardi ogram (ECG) and t he ches t radi ograph. However, i n t hi s cas e nei t her i s l i kel y t o be very i nformat i ve. The ECG may s how evi dence of l eft vent ri cul ar hypert rophy, but s ome pat i ent s wi t h s evere aort i c s t enos i s fai l t o demons t rat e t hi s fi ndi ng. Thus , a negat i ve ECG s houl d not di s s uade one from t he di agnos i s . Typi cal l y, t he ches t radi ograph i n aort i c s t enos i s s hows a normal -s i zed heart , s omet i mes wi t h a boot -s haped confi gurat i on i ndi cat i ve of concent ri c l eft vent ri cul ar
Pa g e 3 1 4 8
ABC Ambe r CHM Conve rte r Tria l ve rsion, http://w w w .proce sste x t.com/a bcchm.html
hypert rophy. Agai n, t hi s fi ndi ng i s not s peci fi c. The mos t i mport ant s t udy t o be ordered at t hi s t i me i s an echocardi ogram wi t h Doppl er eval uat i on of t he aort i c val ve. The echocardi ogram demons t rat es t he concent ri c l eft vent ri cul ar hypert rophy t ypi cal of aort i c s t enos i s . Furt hermore, i t demons t rat es s evere res t ri ct i on of t he aort i c val ve l eafl et s . Bot h of t hes e fi ndi ngs are cons i s t ent wi t h t he di agnos i s but do not hel p quant i fy t he s everi t y of t he di s eas e. However, Doppl er eval uat i on of t he aort i c val ve can quant i fy t he t rans val vul ar aort i c gradi ent preci s el y. In mos t cas es t he echo Doppl er s t udy i s adequat e t o confi rm t he di agnos i s and t o arri ve at a deci s i on regardi ng s urgery. Becaus e t hi s pat i ent i s i n t he coronary di s eas e age group and becaus e of t he hi s t ory of angi na, cardi ac cat het eri zat i on t o confi rm t he aort i c val ve gradi ent and t o defi ne t he coronary anat omy wi t h coronary art eri ograms s houl d al s o be performed. An ECG shows left ventricular hypertrophy but no evidence of past or present ischemia. A chest radiograph is within normal limits, and a radiograph of the right elbow is also normal. Laboratory values are noncontributory. T he patient is sent for an echocardiogram with Doppler examination. T he echocardiogram shows a calcified aortic value and left ventricular hypertrophy, both consistent with calcific aortic stenosis. T he aortic valve gradient is 70 mm Hg. T he patient is admitted to the hospital for a cardiac catheterization and probable aortic valve replacement, because only 50% of patients with aortic stenosis who have syncope achieve a 3-year survival if left untreated.
Questions
What are s ome of t he c aus es of aort i c s t enos i s ?
What i s t he pat hophys i ol ogy underl yi ng t he phys i c al s i gns and s ympt oms t hat c harac t eri ze aort i c s t enos i s ?
Pa g e 3 1 4 9
ABC Ambe r CHM Conve rte r Tria l ve rsion, http://w w w .proce sste x t.com/a bcchm.html
Discussion Caus es of aort i c s t enos i s i ncl ude congeni t al , rheumat i c, and cal ci fi c condi t i ons . In t hi s pat i ent 's age group, cal ci fi c aort i c s t enos i s i s of part i cul ar concern. There i s no evi dence of a dynami c s ubval vul ar l eft vent ri cul ar out fl ow t ract obs t ruct i on, as may be s een wi t h hypert rophi c cardi omyopat hy. In aort i c s t enos i s , t he heart mus cl es force bl ood t hrough a s t enot i c val ve, whi ch res ul t s i n pres s ure overl oad on t he l eft vent ri cl e and s ubs equent concent ri c l eft vent ri cul ar hypert rophy. The cl i ni cal t ri ad of angi na, s yncope, and heart fai l ure i s t ypi cal . The angi na i s t he res ul t of i mpai red bl ood fl ow l i mi t i ng oxygen t o t he enl arged myocardi um. The s yncope can be caus ed by a decreas e i n t he heart 's abi l i t y t o i ncreas e cardi ac out put acros s t he s t enot i c val ve. Syncope al s o can occur becaus e of arrhyt hmi as res ul t i ng from cal ci fi cat i on wi t hi n t he cardi ac conduct i on s ys t em. The pat i ent 's l ow s ys t emi c bl ood pres s ure i s a cons equence of hi s i mpai red cardi ac out put . The di mi ni s hed S 2 i s t he res ul t of i mpai red val ve mot i on from t he aort i c di s eas e. The S 4 i s a res ul t of decreas ed l eft vent ri cul ar compl i ance. Thi s pat i ent i l l us t rat es t hat , wi t h an appropri at e hi s t ory and phys i cal exami nat i on, cl ues t o t he caus e of a pat i ent 's s yncope can be defi ned. Furt her eval uat i on can t hen be t ai l ored t o t he pat i ent . However, a caus e for s yncope i s not readi l y evi dent i n many pat i ent s . In t hes e s i t uat i ons , a revi ew of t he pat i ent 's hi s t ory may be i nformat i ve. For exampl e, di d t he s yncope occur aft er a meal or duri ng s t rai ni ng at defecat i on? In t he pat i ent wi t h recurrent s yncope, a Hol t er moni t or t o exami ne for arrhyt hmi as , a s t res s t es t t o t ry and i nduce t he arrhyt hmi a res pons i bl e for t he s yncope, aut onomi c t es t i ng t o exami ne t he i nt egri t y of pos t ural refl exes , and an echocardi ogram t o s earch for underl yi ng cardi ac pat hol ogy may be warrant ed. If arrhyt hmi as are t he s us pect ed caus e, el ect rophys i ol ogi c s t i mul at i on i n s el ect ed cas es may provoke t he caus at i ve arrhyt hmi a. If occl us i ve cerebrovas cul ar di s eas e i s s us pect ed, a carot i d dupl ex exami nat i on, magnet i c res onance angi ography (MRA), t rans crani al Doppl er s t udi es , or i nt ervent i onal
Pa g e 3 1 5 0
ABC Ambe r CHM Conve rte r Tria l ve rsion, http://w w w .proce sste x t.com/a bcchm.html
angi ography s houl d be cons i dered. If a neurol ogi c et i ol ogy i s s us pect ed, an el ect roencephal ogram (EEG) t o l ook for s ei zure act i vi t y and magnet i c res onance i magi ng (MRI) t o l ook for s t ruct ural brai n abnormal i t i es are warrant ed. P.632
Case 10 Breast Lump A 41-year-old white woman goes to her physician after discovering a lump in her left breast while doing a breast self-examination in the shower. She had thought in the past that she might have felt something in the same location, but now feels that the lump has gotten larger.
Question
What addi t i onal hi s t ori c al i nformat i on w oul d you l i ke t o have from t hi s pat i ent ?
Discussion Ei ght y percent of women wi t h breas t l umps fi nd t he l ump whi l e performi ng a breas t s el f-exami nat i on. As part of t hi s pat i ent 's hi s t ory, t he phys i ci an s houl d as k t he pat i ent about her ri s k fact ors for breas t cancer. One s houl d al s o as k t he fol l owi ng ques t i ons : How l ong has t he pat i ent has been aware of t he l ump? W hat was i t s ori gi nal s i ze? W hen di d i t i ncreas e i n s i ze, and by how much? It al s o mi ght hel p t o know whet her t he mas s changed i n s i ze i n conjunct i on wi t h t he pat i ent 's mens t rual peri od, becaus e beni gn di s eas e can regres s aft er t he mens t rual cycl e. If t he mas s i s not cl i ni cal l y s us pi ci ous , i t mi ght be appropri at e t o reeval uat e t he pat i ent aft er her next mens t rual cycl e when t he fl ow ends —i f t he mas s pers i s t s , i nt ervent i on i s neces s ary. Di d t he pat i ent experi ence any di s charge from t he ni ppl es ? If s o, was i t bl oody or cl ear? The
Pa g e 3 1 5 1
ABC Ambe r CHM Conve rte r Tria l ve rsion, http://w w w .proce sste x t.com/a bcchm.html
pres ence of breas t edema, di s col orat i on, or pai n i s i mport ant t o as cert ai n. Fi nal l y, t he pat i ent s houl d be as ked whet her s he has had any mammograms i n t he pas t . Further history reveals that the patient first discovered the lump in the upper outer quadrant of her left breast about 4 months ago, but she did not seek medical attention because she has always had large, “ lumpy― breasts. T he patient' s menstrual period does not seem to have affected the presence of the lump in any way. She has not experienced nipple discharge, breast edema or discoloration, or pain. T he patient has no previous medical or surgical history and takes no medications. One of her cousins died of breast cancer, but none of her other female relatives have had any cancer. Both of her parents are alive; her father has hypertension and coronary artery disease. T he patient, who has experienced no symptoms of menopause, first began to menstruate at age 12 and has had regular periods lasting 4 days on a 30-day cycle. She has one 7-year-old daughter. She uses a diaphragm for birth control. Although the patient had a baseline mammogram at age 35, she has not had one since then, and she has not seen a gynecologist for over 1 year.
Questions
What ri s k fac t ors does t hi s pat i ent have for devel opi ng breas t c anc er?
What di s eas es mi ght be i nc l uded i n your di fferent i al di agnos i s at t hi s poi nt ?
Discussion The mos t i mport ant ri s k fact ors for breas t cancer are fami l y hi s t ory—mat ernal and/or pat ernal hi s t ory of breas t cancer, mal e breas t cancer or ovari an cancer (es peci al l y i f t he pat i ent has a fi rs t -degree rel at i ve who devel oped premenopaus al breas t cancer),
Pa g e 3 1 5 2
ABC Ambe r CHM Conve rte r Tria l ve rsion, http://w w w .proce sste x t.com/a bcchm.html
a hi s t ory of fi brocys t i c di s eas e, a hi s t ory of previ ous breas t bi ops i es , earl y ons et of menarche, l at e ons et of menopaus e, a fi rs t pregnancy aft er t he age of 30, and us e of hormone repl acement t herapy by a pos t menopaus al pat i ent . A pat i ent 's l ack of ri s k fact ors for breas t cancer s houl d not di s s uade t he phys i ci an from performi ng t he neces s ary procedures t o rul e out mal i gnancy. Al t hough mos t breas t cancers are found i n t he upper out er quadrant , l ocat i on s houl d not change t he phys i ci an's approach t o excl udi ng mal i gnancy. A young, premenopaus al woman wi t h a breas t l ump coul d have a beni gn mas s s uch as a fi broadenoma or a cys t . Breas t l umps are beni gn i n 80% of cas es , but mal i gnancy mus t be rul ed out . If t he l ump i s i ndeed mal i gnant , t he pat i ent has a hi gher chance of l ymph node met as t as i s and mi cromet as t at i c di s eas e as t he t umor i ncreas es i n s i ze. Physical examination shows a well-developed, thin, white woman who is slightly anxious but in no acute distress. T he examination is normal except for the presence of a 2-cm mass in the upper, outer quadrant of the left breast. T he mass appears firm and somewhat mobile. T here is no obvious asymmetry between breasts, skin dimpling, or redness. No axillary nodes can be palpated, and there are no masses in the right breast. P.633
Question
What w oul d you do next t o eval uat e t hi s pat i ent ?
Discussion Al l pat i ent s wi t h a defi nabl e mas s s houl d have a mammogram; becaus e t hi s pat i ent has not had a mammogram i n 6 years , havi ng one now woul d probabl y be es peci al l y wort hwhi l e. A breas t l ump i s vi s i bl e on a mammogram al one i n 20% of cas es . However,
Pa g e 3 1 5 3
ABC Ambe r CHM Conve rte r Tria l ve rsion, http://w w w .proce sste x t.com/a bcchm.html
mammograms are oft en di ffi cul t t o i nt erpret i n premenopaus al pat i ent s or pos t menopaus al pat i ent s t aki ng es t rogen repl acement t herapy, whi ch may i ncreas e t he breas t t i s s ue dens i t y on mammogram; dens e (mammographi cal l y–whi t e) normal gl andul ar breas t t i s s ue may obs cure t umor mas s es of s i mi l ar dens i t y. W hen a woman goes t hrough menopaus e, t he mammogram has a gray background, becaus e t he breas t cons i s t s of mos t l y fi brofat t y t i s s ue. The gray s erves as an i deal background for i dent i fi cat i on of new breas t abnormal i t i es , whi ch appear whi t er i n cont ras t t o t he normal breas t t i s s ue. A negat i ve mammogram does not neces s ari l y i mpl y t hat a pat i ent 's mas s i s not cancerous . In addi t i on t o havi ng a mammogram, t hi s pat i ent s houl d be referred for an ul t ras ound of t he breas t whi ch can det ermi ne whet her t he mas s i s s ol i d or cys t i c and i f t he mas s i s cys t i c (s i mpl e cys t ) a needl e as pi rat i on of t he cys t t o eval uat e for t he pres ence of fl ui d. If t he mas s di s appears wi t h as pi rat i on of t he fl ui d, t hen i t i s a beni gn cys t (as s umi ng t he cyt ol ogy i s negat i ve). Les s t han 1% of pat i ent s i n whom t he mas s t ot al l y di s appears have mal i gnant cel l s s een on cys t as pi rat i on. A cys t i c l es i on s een on ul t ras ound exami nat i on s houl d be t he s ame s i ze as t he pal pabl e mas s . If t here are no i nt ernal echoes wi t hi n t he cys t , t hen a s ol i d l es i on cannot be rul ed out , and carci noma i s s t i l l a pos s i bi l i t y and a needl e bi ops y i s t hen recommended t o obt ai n t i s s ue for pat hol ogi cal eval uat i on. If t he mas s i s s ol i d on breas t ul t ras ound, t he cont ent s of t he needl e s houl d be pl aced on a s l i de and s ubmi t t ed for cyt ol ogy. Even i f t he cyt ol ogy i s beni gn, a mal i gnant l es i on s t i l l cannot be rul ed out and a core needl e bi ops y t o obt ai n more t i s s ue for di agnos i s s houl d be preformed. Majori t y of pat i ent s wi t h s us pi ci ous breas t l umps can undergo a core needle bi ops y under ul t ras ound or mammographi c gui dance (s t ereot act i c bi ops y). Very few pat i ent s wi t h s us pi ci ous breas t mas s es wi l l need an exci s i onal s urgi cal bi ops y t o di agnos e t hei r breas t cancer. Attempted fine-needle aspiration does not produce any fluid. A mammogram shows a well-defined 2.2-cm density in the left upper outer quadrant. No other densities or calcifications are
Pa g e 3 1 5 4
ABC Ambe r CHM Conve rte r Tria l ve rsion, http://w w w .proce sste x t.com/a bcchm.html
noted. An ultrasound of the density confirms a solid mass 2.2 cm diameter, which corresponds with the mammographic density. T he suspicious mammogram, coupled with the enlarging, palpable mass and the failure to aspirate fluid, lead the physician to recommend that the patient undergo a biopsy of the lesion. T he patient is referred to a radiology facility for an ultrasound-guided core needle biopsy of the suspicious lesion. T he pathology report confirms an infiltrating ductal carcinoma, grade 3, estrogen receptor positive (90%), progesterone receptor positive (50%) and Her2/neu positive (+3) by immunohistochemical (IHC) stains.
Questions
How w oul d you s t age t hi s pat i ent 's c anc er?
What w oul d be your preferred mode of t reat ment ?
What i s t he pat i ent 's prognos i s ?
Discussion Thi s pat i ent woul d be cl as s i fi ed as havi ng cl i ni cal s t age II di s eas e becaus e t he t umor meas ures bet ween 2 and 5 cm i n di amet er and t here are no fi xed nodes or di s t ant met as t as es . Mos t pat i ent s wi t h s t age I or II breas t cancer can undergo l umpect omy (breas t cons ervat i on s urgery) wi t h s ent i nel l ymph node bi ops y wi t h furt her axi l l ary di s s ect i on onl y i f t he s ent i nel l ymph node i s pos i t i ve for met as t as es . Radi at i on t herapy t o t he whol e breas t i s gi ven t o al l pat i ent s who had breas t cons ervat i on t herapy. The cure rat es as s oci at ed wi t h t hi s mode of t reat ment are equi val ent t o t hos e for a modi fi ed radi cal mas t ect omy, i f t he pat i ent meet s cert ai n cri t eri a. Becaus e t hi s part i cul ar pat i ent does not have mul t i cent ri c di s eas e i n t he breas t , more t han one breas t pri mary, or a t umor great er t han 5 cm i n di amet er, s he i s an opt i mal candi dat e for radi at i on
Pa g e 3 1 5 5
ABC Ambe r CHM Conve rte r Tria l ve rsion, http://w w w .proce sste x t.com/a bcchm.html
t herapy. P.634
Before t he deci s i on i s made t o perform a l umpect omy and axi l l ary di s s ect i on, t he pat i ent s houl d be s een by a radi at i on t herapi s t . If t he radi at i on t herapi s t feel s t hat t he pat i ent i s not an opt i mal candi dat e, t hen a modi fi ed radi cal mas t ect omy i s warrant ed. If t he pat i ent choos es t o have a modi fi ed radi cal mas t ect omy, s he has t he opt i on of undergoi ng breas t recons t ruct i on s urgery at t he t i me of t he mas t ect omy or at s ome fut ure t i me. Adjuvant hormonal t herapy, chemot herapy, and radi at i on t herapy (i .e., radi at i on t herapy for purpos es ot her t han breas t cons ervat i on) al s o may pl ay a rol e i n t reat i ng pat i ent s wi t h breas t cancer. Adjuvant chemot herapy i s al mos t al ways gi ven before adjuvant radi at i on t herapy. Pat i ent s who pres ent wi t h l arge breas t t umors (>5 cm i n di amet er) or wi t h unres ect abl e t umors may need neoadjuvant chemot herapy, whi ch means t hat t he chemot herapy i s gi ven fi rs t t o s hri nk t he t umor and t hen i s fol l owed by s urgery and radi at i on t herapy. Becaus e t he majori t y of breas t t umors res pond t o neoadjuvant chemot herapy t hi s approach can be very val uabl e as i t al l ows t he s urgeon t o compl et el y res ect t he t umor. Prognos i s vari es wi del y i n breas t cancer pat i ent s . Approxi mat el y 50% of pat i ent s wi t h operabl e breas t cancer devel op recurrent di s eas e unl es s t hey recei ve adjuvant chemot herapy or hormone t herapy. Prognos t i c fact ors i ncl ude t umor s i ze, axi l l ary node s t at us , t he hi s t opat hol ogy of t he t umor, hormone recept or s t at us (es t rogen and proges t erone), t he S-phas e fract i on and DNA i ndex, and oncogene expres s i on (Her2/neu). Mos t young premenopaus al women wi t h s t age II breas t cancer wi l l recei ve adjuvant chemot herapy and hormonal t herapy, i n addi t i on t o s urgery and radi at i on t herapy, i f t he t umor i s es t rogen or proges t erone pos i t i ve. W omen wi t h Her2/neu pos i t i ve breas t cancer wi l l al s o recei ve Tras t uz umab a monocl onal ant i body agai ns t Her2/neu.
Pa g e 3 1 5 6
ABC Ambe r CHM Conve rte r Tria l ve rsion, http://w w w .proce sste x t.com/a bcchm.html
Case 11 Acute Low Back Pain A 35-year-old man presents to the emergency department with a 2-day history of severe low back pain. T he pain is worse when he sits up and better when he lies flat on his back. He claims to be well, although he recently experienced polyarthralgias, chills, and sweating. He reports that he was hospitalized for some form of hepatitis 2 years ago; he says it resolved uneventfully and he does not know what caused it. He denies alcohol or drug abuse.
Questions
What are your i mmedi at e c onc erns about t hi s pat i ent 's l ow bac k pai n?
What c ondi t i ons mi ght c aus e referred bac k pai n?
What addi t i onal i nformat i on do you w ant t o know about t he pai n?
Discussion The hi s t ory of chi l l s and s weat s makes one cons i der an i nfect i ous proces s more s eri ous l y t han ot her pos s i bl e caus es of t hi s l ow back pai n. The man des cri bes t he pai n as s evere, whi ch makes i t more worri s ome. The fact t hat t he pai n i s wors e when s i t t i ng upri ght and bet t er wi t h recumbency i s not part i cul arl y hel pful i n s ugges t i ng a s peci fi c, pot ent i al l y s eri ous caus e of t hi s back compl ai nt . Pai n t hat i s i ns i di ous i n ons et , as s oci at ed wi t h prol onged morni ng s t i ffnes s , rel i eved by exerci s e, and wors ened by res t s ugges t s underl yi ng i nfl ammat ory back pai n, t ypi cal l y t he s acroi l i i t i s of t he s pondyl oart hropat hi es . In cont ras t , mechani cal l ow back pai n oft en i s s udden i n ons et , wors ened by exerci s e, and i mproved by res t .
Pa g e 3 1 5 7
ABC Ambe r CHM Conve rte r Tria l ve rsion, http://w w w .proce sste x t.com/a bcchm.html
Pai n t hat i s ri ppi ng or t eari ng and perhaps as s oci at ed wi t h abdomi nal compl ai nt s i s t ypi cal of an expandi ng abdomi nal aort i c aneurys m. Pai n as s oci at ed wi t h bl adder or bowel i ncont i nence and s addl e anes t hes i a s ugges t s a mi d-l i ne l umbar di s k herni at i on wi t h cauda equi na compres s i on; pai n as s oci at ed wi t h l eg s ens ory or mot or compl ai nt s s ugges t s l at eral di s k herni at i on and s pi nal nerve compres s i on; and pai n as s oci at ed wi t h gas t roi nt es t i nal or geni t ouri nary compl ai nt s s ugges t s a need t o i nves t i gat e an i nt ra-abdomi nal s ource as t he caus e. Cons t i t ut i onal compl ai nt s s uch as fever and chi l l s make an i nfect i on more l i kel y. Pai n i ncreas i ng wi t h recumbency (t he oppos i t e of t hi s pat i ent 's compl ai nt ) s ugges t s a pos s i bl e t umor; mi d-l i ne pai n s ugges t s a t umor, an i nfect i on, or a compres s i on fract ure. Ni net y percent of l ow back pai n i s caus ed by s el f-l i mi t ed bi omechani cal or s t rai n probl ems , but i t i s i mport ant t o l ook for feat ures t hat make an acut e medi cal condi t i on more l i kel y. There i s i mport ant i nformat i on s t i l l t o be obt ai ned about t he nat ure of t hi s pat i ent 's back pai n. P.635
On initial examination, the patient is writhing in discomfort and asking for intramuscular narcotics. His temperature is 38.3°C, and prominent needle tracks are observed on his hands and legs. Diffuse tenderness and guarding are noted on abdominal examination. Marked tenderness is noted over the entire low back paraspinal region, and prominent percussion tenderness is noted over several lumbar and sacral vertebrae. T he patient cooperates poorly with strength testing, but deep tendon reflexes and sensation are intact and anal wink and perianal sensations are normal. T he patient complains of hamstring tenderness when either leg is raised in a straightened, extended position.
Questions
Pa g e 3 1 5 8
ABC Ambe r CHM Conve rte r Tria l ve rsion, http://w w w .proce sste x t.com/a bcchm.html
How do t he fi ndi ngs on phys i c al exami nat i on hel p you i n t hi nki ng about your di fferent i al di agnos i s ?
What i nformat i on s houl d you s eek from l aborat ory t es t s and proc edures at t hi s t i me?
Discussion Several fi ndi ngs on phys i cal exami nat i on pl ace s el f-l i mi t ed mus cul os kel et al pai n far down t he di fferent i al di agnos i s l i s t . For one t hi ng, i t i s known t hat t he pat i ent i s l i kel y abus i ng i nt ravenous drugs , even t hough he deni ed t hi s i n t he hi s t ory. Narcot i c drug-s eeki ng behavi or makes t he phys i cal exami nat i on more di ffi cul t t o i nt erpret , becaus e exaggerat ed res pons es t o phys i cal exami nat i on maneuvers are common i n drug-abus i ng pat i ent s . In addi t i on, t he pat i ent has a fever and promi nent fi ndi ngs on abdomi nal and back pal pat i on as wel l as on s pi nal percus s i on. Int ravenous drug abus e and fever make an i nfect i ous proces s (e.g., epi dural abs ces s , paras pi nal or peri rect al col l ect i on, s ept i c s acroi l i i t i s ) much more l i kel y. No s peci fi c evi dence for an aort i c aneurys m exi s t s on abdomi nal exami nat i on, and t here i s no neurol ogi c evi dence for cauda equi na compres s i on or di s cogeni c nerve compres s i on. No evi dence has been pres ent ed for a pot ent i al referred s ource of pai n s uch as a penet rat i ng duodenal ul cer (gas t roi nt es t i nal s ource) or a renal i nfect i on or s t one (geni t ouri nary s ource). There i s no evi dence of a s pondyl oart hropat hy. A compl et e bl ood count (CBC) mi ght hel p wi t h eval uat i on of an i nfect i ous proces s , part i cul arl y i f l eukocyt os i s i s pres ent . Amyl as e woul d be i ncreas ed wi t h pancreat i t i s and bowel emergenci es (e.g., s mal l bowel obs t ruct i on). The eryt hrocyt e s edi ment at i on rat e i s nons peci fi c, but s i gni fi cant el evat i on al s o mi ght s ugges t an i nfect i on, part i cul arl y os t eomyel i t i s . Uri nal ys i s i s i mport ant for di s coveri ng a pot ent i al geni t ouri nary i nfect i on. Three s et s of pai red bl ood cul t ures s houl d be obt ai ned, becaus e bact eri al endocardi t i s can pres ent wi t h mus cul os kel et al compl ai nt s , and any bact eri al proces s i n t he l ow back mi ght be as s oci at ed wi t h bact eremi a. Pl ai n
Pa g e 3 1 5 9
ABC Ambe r CHM Conve rte r Tria l ve rsion, http://w w w .proce sste x t.com/a bcchm.html
radi ographs of t he l ow back and s acroi l i ac joi nt s qui t e l i kel y mi ght be normal , al t hough pat i ent s wi t h di s ki t i s can have i nt ervert ebral narrowi ng and vert ebral end-pl at e des t ruct i on; t hos e wi t h s acroi l i i t i s can have s acroi l i ac joi nt eros i ons and s cl eros i s . A magnet i c res onance i magi ng (MRI) i s t he bes t i magi ng modal i t y for eval uat i ng s oft t i s s ue or i nt ras pi nal col l ect i ons , part i cul arl y t o rul e out an epi dural abs ces s . T he test results come back as follows: hemoglobin is 11.3 g/dL, the white blood cell (WBC) count is 15,000/µL, and the platelet count is 500,000/µL. T he Westergren erythrocyte sedimentation rate is 100 mm/hour. Chemistries indicate only an elevated alkaline phosphatase; amylase is normal. Urinalysis is negative. Chest radiograph, lumbar spine radiograph, and pelvic radiographs are unrevealing. Several hours later, the patient continues to require high doses of intramuscular narcotic for pain control and still complains of severe low back pain. He says that he is having difficulty walking to the bathroom because of leg weakness. His maximum temperature is 38.9°C. Physical examination indicates continuing poor cooperation with strength testing, but he appears to be severely weak (3/5 on muscle strength testing) in all lower extremity muscle groups. He also has decreased sensation to pinprick from the toes to the navel and prominent increased deep tendon reflexes in the lower extremities with four beats of clonus bilaterally.
Questions
What i s t he mos t l i kel y di agnos i s at t hi s poi nt ?
What addi t i onal di agnos t i c t es t s are appropri at e t o c onfi rm a di agnos i s ?
Pa g e 3 1 6 0
ABC Ambe r CHM Conve rte r Tria l ve rsion, http://w w w .proce sste x t.com/a bcchm.html
What t herapeut i c approac h i s mos t appropri at e now ?
P.636
Discussion The neurol ogi c fi ndi ngs (l eg weaknes s , abdomi nal s ens ory l evel , i ncreas ed refl exes , and cl onus ) s ugges t s pi nal cord compres s i on wi t h feat ures of an upper mot or neuron l es i on. W hen t he hi s t ory of i nt ravenous drug abus e, fever, l ow back pai n, and progres s i ve neurol ogi c defi ci t i s added, t he di fferent i al di agnos i s narrows markedl y. The pat i ent 's s ens ory l evel t o about T12 s ugges t s t hat t he cord compres s i on i s occurri ng at t hi s l evel , probabl y from an epi dural abs ces s . Condi t i ons s uch as t rans vers e myel i t i s or an i s chemi c myel opat hy at T12 cord l evel or a mi d-l i ne herni at ed t horaci c di s k are s t i l l pos s i bl e, but t hes e di agnos es are l es s l i kel y t o expl ai n t he fever or t o be as s oci at ed wi t h i nt ravenous drug abus e. Gi ven t he hi gh i ndex of s us pi ci on for i nfect i on, empi ri c ant i bi ot i c admi ni s t rat i on s houl d be s t rongl y cons i dered. The bes t procedure for confi rmi ng a di agnos i s i s an MRI s can. Thi s t es t wi l l bes t di s cri mi nat e bet ween an i nt ri ns i c cord l es i on (e.g., t rans vers e myel i t i s ) and an ext ri ns i c compres s i on (e.g., due t o an abs ces s or herni at ed di s k). The t es t s houl d be done emergent l y, becaus e earl y s urgi cal i nt ervent i on i s cri t i cal t o pres ervat i on of l ower ext remi t y funct i on. T he emergency MRI scan shows an epidural abscess at T 12, and surgical débridement is carried out immediately. Staphylococcus aureus grows from one set of the blood cultures and the abscess.
Question
What remai ni ng t reat ment modal i t i es s houl d be us ed now ?
Discussion
Pa g e 3 1 6 1
ABC Ambe r CHM Conve rte r Tria l ve rsion, http://w w w .proce sste x t.com/a bcchm.html
The pat i ent s houl d be t reat ed for 4–6 weeks wi t h ant i bi ot i cs effect i ve agai ns t S. aureus , i ni t i al l y i nt ravenous l y. He s houl d be careful l y wat ched for feat ures of bact eri al endocardi t i s , des pi t e t he fact t hat onl y one bl ood cul t ure was pos i t i ve. Furt hermore, t he pat i ent s houl d recei ve i nt ens i ve phys i cal t herapy once t he acut e pai n s ubs i des s o t hat he can recover l ower ext remi t y s t rengt h and t he abi l i t y t o wal k.
Case 12 Gastrointestinal Bleeding A 54-year-old white man presents to his general practitioner' s office complaining of fatigue. He says that he tires easily and often feels light-headed and short of breath after climbing a single flight of steps or taking a short walk. He says that he is having difficulty at work because he is “ just not himself.―
Questions
What are s ome pos s i bl e c aus es of general i zed fat i gue and w eaknes s ?
What ot her ques t i ons w oul d you l i ke t o as k t hi s pat i ent ?
Discussion Fat i gue and weaknes s are common compl ai nt s t hat can be ps ychogeni c or phys i cal i n ori gi n. It i s i mport ant t o di fferent i at e bet ween t hes e broad et i ol ogi c cat egori es . Ps ychogeni c caus es cons i s t of anxi et y s t at es and depres s i on. Phys i cal caus es i ncl ude i nfect i ous di s eas e, met abol i c di s orders , bl ood dys cras i as , renal di s eas e, l i ver di s eas e, chroni c pul monary di s eas e, chroni c cardi ovas cul ar di s eas e, neopl as t i c di s eas es , and neuromus cul ar di s eas e. To narrow t he di agnos t i c pos s i bi l i t i es for t hi s pat i ent 's fat i gue and weaknes s , addi t i onal hi s t ory s houl d be obt ai ned. The pat i ent
Pa g e 3 1 6 2
ABC Ambe r CHM Conve rte r Tria l ve rsion, http://w w w .proce sste x t.com/a bcchm.html
s houl d be ques t i oned about whet her he has experi enced wei ght l os s , fever, chi l l s , ches t pai n, paroxys mal noct urnal dys pnea, ort hopnea, pedal edema, abdomi nal pai n, changes i n bowel habi t s , mel ena, hemat ochezi a, pol yuri a, pol ydi ps i a, pol yphagi a, i nt ol erance t o heat or col d, or i ns omni a. A pos i t i ve ans wer t o t hes e ques t i ons oft en, al t hough not al ways , i ndi cat es a phys i cal or organi c caus e of fat i gue and weaknes s . Further history indicates that the patient' s symptoms started about 2 months ago and have steadily worsened. He claims he was in excellent health until this time; in fact, he has not had a routine physical examination in approximately 2 years because he has felt healthy. T he patient takes no medications, except for the occasional use of acetaminophen or laxatives. He is an executive in a publishing company who smokes approximately half a pack of cigarettes per day and P.637 drinks a martini or two at lunch. He has no significant family history, except that an estranged older brother died after an abdominal operation for an unknown cause. T he patient denies chest pain or palpitations but has occasional shortness of breath and dyspnea on exertion, as described above. He has had no loss of appetite and even jokes that he can eat even on days when he is a bit “ irregular.― When asked about his constipation, he notes that he sometimes has difficulty passing his stool, but the stool is of normal consistency. He reports that his stools have seemed a bit darker lately, but he thought it might be due to laxatives he took for constipation; he has not noticed any bright red blood in the stool and denies hematemesis, nausea, vomiting, or diarrhea.
Question
Whi c h of t hes e s i gns and s ympt oms c onc ern you?
Discussion
Pa g e 3 1 6 3
ABC Ambe r CHM Conve rte r Tria l ve rsion, http://w w w .proce sste x t.com/a bcchm.html
Thi s pat i ent 's hi s t ory rai s es s everal poi nt s of concern. Hi s s ympt oms began 2 mont hs ago, meani ng t hey are chroni c compl ai nt s , and t hey have s t eadi l y wors ened—an omi nous s i gn. Acut e compl ai nt s coul d be at t ri but ed t o an acut e vi ral or s el f-l i mi t ed i l l nes s . That t he pat i ent was i n excel l ent heal t h unt i l 2 mont hs ago al s o i ndi cat es a change from a preexi s t i ng pat t ern. Sympt oms t hat have been pres ent for a l ong t i me (2–3 years or more) s ugges t a chroni c nonprogres s i ve i l l nes s s uch as i rri t abl e bowel s yndrome. However, i n a pat i ent who was previ ous l y s ympt om-free and i n excel l ent heal t h, a 2-mont h hi s t ory of s ympt oms t hat have s t eadi l y wors ened woul d s ugges t a new and pot ent i al l y s eri ous change t hat coul d be compat i bl e wi t h i nfl ammat ory bowel di s eas e, i nfect i ous s t at es (e.g., gi ardi as i s ), or even mal i gnant di s eas e i nvol vi ng t he gas t roi nt es t i nal t ract , part i cul arl y t he col on. The abs ence of medi cat i ons woul d el i mi nat e t he pos s i bi l i t y t hat a s i de effect of a drug (e.g., an ant i hypert ens i ve agent ) i s res pons i bl e for t he fat i gue and s hort nes s of breat h. Smoki ng i s cl earl y as s oci at ed wi t h numerous mal i gnanci es , i ncl udi ng bronchogeni c and pancreat i c carci noma, i n addi t i on t o i t s known cardi ovas cul ar and pul monary effect s . The fami l y hi s t ory of an abdomi nal operat i on for an unknown caus e rai s es t he s pect er of pos s i bl e col oni c carci noma, whi ch has a t wo- t o t hreefol d i ncreas ed i nci dence i n fi rs t -degree rel at i ves . Dys pnea on exert i on coul d s i gni fy a cardi ovas cul ar caus e but al s o coul d be as s oci at ed wi t h an anemi c s t at e. The “abnormal l y dark s t ool s ― mi ght be i ndi cat i ve of mel ena, whi ch woul d s ugges t t hat t he upper gas t roi nt es t i nal t ract (above t he l i gament of Trei t z) i s t he mos t l i kel y s ource of bl eedi ng; dark red or maroon-colored s t ool s t ypi cal l y i ndi cat e bl ood emanat i ng from t he ri ght col on. Physical examination shows a well-developed, overweight white man with a somewhat rapid respiratory rate (i.e., 20 respirations/min). Other vital signs show a pulse of 94 beats/minute, a blood pressure of 110/60 mm Hg, and a normal temperature. T he patient' s skin is pale, as are his mucous membranes. His lungs are clear on auscultation, and his heart
Pa g e 3 1 6 4
ABC Ambe r CHM Conve rte r Tria l ve rsion, http://w w w .proce sste x t.com/a bcchm.html
rhythm is regular, with normal first and second heart sounds (S 1 and S 2 ) and no third or fourth heart sound (S 3 or S 4 ). His abdomen is soft, without tenderness or obvious masses. His extremities show no cyanosis or edema; he has prolonged capillary refill. His rectal examination shows no palpable masses, but a dark, heme-positive stool is noted.
Questions
Bas ed on t hi s phys i c al exami nat i on, w hat do you s us pec t i s t he c aus e of t hi s man's fat i gue?
What are s ome of t he c aus es of mel ena?
What s ort s of di s orders produc e hemat oc hezi a?
How w oul d you proc eed i n w orki ng up t he heme-pos i t i ve s t ool ?
Discussion Phys i cal exami nat i on s hows an i ndi vi dual whos e s ki n and mucous membranes are pal e and who has heme-pos i t i ve s t ool . Thes e s i gns s ugges t t hat t he pat i ent has gas t roi nt es t i nal bl eedi ng and i s mos t l i kel y anemi c, pos s i bl y t he reas on he i s feel i ng fat i gued. Mel ena i s t he pas s age of dark, t arry s t ool s due t o t he pres ence of bl ood al t ered by i nt es t i nal jui ces . It i s mos t commonl y caus ed by upper gas t roi nt es t i nal bl eedi ng (e.g., duodenal ul cer di s eas e, gas t ri c ul cer di s eas e, hemorrhagi c gas t ri t i s , eros i ve es ophagi t i s , es ophageal vari ces or Mal l ory-W ei s s t ears , or P.638 vas cul ar ect as i a of t he s t omach). In an i ndi vi dual who i s not acut el y i l l , pept i c ul cer di s eas e woul d be t he mos t common caus e;
Pa g e 3 1 6 5
ABC Ambe r CHM Conve rte r Tria l ve rsion, http://w w w .proce sste x t.com/a bcchm.html
es ophageal vari ces or a Mal l ory-W ei s s t ear of t he di s t al es ophagus woul d be unl i kel y. Hemat ochez i a i s t he pas s age of bl ood from t he rect um, whi ch vari es i n col or from dark red or maroon (from bl eedi ng i n t he ri ght col on) t o bri ght red (as a res ul t of bl eedi ng from a more di s t al col oni c s ource or t he anal ri ng i t s el f). Hemat ochezi a may be caus ed by col oni c di vert i cul os i s and angi odys pl as i a, hemorrhoi ds , fi s s ures , col on pol yps , col on carci noma, ul cerat i ve col i t i s , i nfect i ous dys ent ery (es peci al l y Shi gel l a, Campyl obac t er, and amebi c col i t i s ), and i s chemi c col i t i s . Unl i ke mel ena, hemat ochezi a i s oft en a s i gn of neopl as t i c l es i ons of t he gas t roi nt es t i nal t ract ; t herefore, t he pat i ent s houl d be eval uat ed for t hes e di s orders . Occas i onal l y, an upper gas t roi nt es t i nal s ource (e.g., bri s k bl eedi ng from a pept i c ul cer) coul d l ead t o t he pas s age of bri ght red bl ood from t he rect um. General l y, t hi s occurrence i s s een i n a pat i ent who i s ot herwi s e hemodynami cal l y uns t abl e and woul d be i dent i fi ed by t he fi ndi ng of bl ood on t he pas s age of a nas ogas t ri c t ube i nt o t he s t omach. The i ni t i al workup for heme-pos i t i ve s t ool depends on t he pat i ent 's hi s t ory and phys i cal exami nat i on. If t he hi s t ory and exami nat i on are s ugges t i ve of an upper gas t roi nt es t i nal s ource, i ni t i al eval uat i on s houl d i ncl ude upper endos copy. However, i n mos t cas es of occul t gas t roi nt es t i nal bl eedi ng when a l ower gas t roi nt es t i nal s ource i s expect ed, col onos copy i s i ndi cat ed. If t hi s s t udy i s unreveal i ng, an upper gas t roi nt es t i nal s ource s houl d be purs ued, i ncl udi ng eval uat i on of t he s mal l bowel . Upper gas t roi nt es t i nal endos copy i s t he bes t i ni t i al fi rs t s t udy. If no upper gas t roi nt es t i nal s ource i s i dent i fi ed, s mal l bowel ent eros copy, s mal l bowel fol l ow-t hrough, or wi rel es s caps ul e endos copy i s i ndi cat ed. A nasogastric tube is inserted but does not reveal any evidence of blood. T he physician has an anoscope in his office, but anoscopy does not show any obvious lesions. In-office hemoglobin and hematocrit tests give values of 9.4 g/dL and 36%, respectively. T he physician decides to refer the patient immediately to a gastroenterologist for further workup of the
Pa g e 3 1 6 6
ABC Ambe r CHM Conve rte r Tria l ve rsion, http://w w w .proce sste x t.com/a bcchm.html
gastrointestinal bleeding.
Questions
Why does t he phys i c i an fi rs t l ook for evi denc e of upper gas t roi nt es t i nal bl eedi ng?
What are t he mos t c ommon c aus es of l ow er gas t roi nt es t i nal bl eedi ng i n t hi s age group?
Discussion The pat i ent 's hi s t ory s ugges t ed a dark s t ool but , on t he bas i s of t he hi s t ory, t he phys i ci an coul d not det ermi ne whet her t he s t ool was bl ack. Therefore, an upper gas t roi nt es t i nal s ource was s ought by t he pas s age of a nas ogas t ri c t ube. In approxi mat el y 75%–80% of cas es , an upper gas t roi nt es t i nal bl eedi ng s ource can be i dent i fi ed by bl ood i n t he nas ogas t ri c t ube. Occas i onal l y, bl eedi ng from a duodenal ul cer wi l l be s o s l i ght i t wi l l caus e a gas t ri c as pi rat e t o be negat i ve for bl ood. In t hi s pat i ent 's age group, t he t wo mos t common caus es of l ower gas t roi nt es t i nal bl eedi ng are di vert i cul os i s and angi odys pl as i a. The di fferent i al di agnos i s woul d al s o i ncl ude t hos e condi t i ons known t o caus e hemat ochezi a, as di s cus s ed earl i er. A bl eedi ng di at hes i s caus ed by a pri mary hemat ol ogi c s ource s uch as l eukemi a, t hrombocyt openi a, hemophi l i a, or di s s emi nat ed i nt ravas cul ar coagul at i on (DIC) s houl d al s o be cons i dered. T he gastroenterologist sees the patient immediately, and some laboratory tests are performed. A complete blood count (CBC) 3
shows a white blood cell (WBC) count of 7.2/mm , a platelet 3
count of 525,000/mm , and hemoglobin and hematocrit consistent with the previous values. Red blood cell (RBC) indices 3
show a mean corpuscular volume (MCV) of 70 µm and a mean corpuscular hemoglobin (MCH) of 25 pg. Serum iron and transferrin are decreased. Prothrombin time (PT ) and partial thromboplastin (PT T ) are normal, as are electrolyte, blood urea
Pa g e 3 1 6 7
ABC Ambe r CHM Conve rte r Tria l ve rsion, http://w w w .proce sste x t.com/a bcchm.html
nitrogen (BUN), and creatinine levels.
Questions
What t ype of anemi a do t hes e val ues s ugges t ?
Whi c h of t he previ ous l y ment i oned di fferent i al di agnos es c an be rul ed out by t he l aborat ory val ues ?
What c ondi t i ons are t he hi ghes t on your di fferent i al ?
What are your i mmedi at e management pl ans ?
P.639
Discussion The l aborat ory fi ndi ngs s ugges t a mi crocyt i c, hypochromi c anemi a. 3
The MCV of 70 mm and t he MCH of 25 pg are compat i bl e wi t h t hi s di agnos i s . An el evat ed pl at el et count i s oft en s een i n pat i ent s wi t h chroni c bl ood l os s . The di fferent i al di agnos i s of t hi s t ype of anemi a i ncl udes i ron defi ci ency caus ed by poor i nt ake or l ack of abs orpt i on, or chroni c bl ood l os s . The us ual s ource of chroni c occul t bl ood l os s i s from t he gas t roi nt es t i nal t ract . Iron chel at i on t herapy for l ead i nt oxi cat i on al s o may caus e t hi s t ype of anemi a. Congeni t al anemi a s uch as t hal as s emi a i s al s o as s oci at ed wi t h hypochromi c, mi crocyt i c anemi a. Iron defi ci ency s t at es are charact eri zed by a l ow s erum i ron, a hi gh t ot al i ron-bi ndi ng capaci t y, and a l ow ferri t i n l evel . The normal PT and PTT make a bleedi ng di at hes i s unl i kel y. Anemi a of chroni c renal i ns uffi ci ency woul d be rul ed out by t he normal BUN and creat i ni ne l evel s . The mos t l i kel y caus es of t hi s pat i ent 's probl em are t hos e i nvol vi ng chroni c gas t roi nt es t i nal bl ood l os s (i .e., col oni c pol yp, col oni c
Pa g e 3 1 6 8
ABC Ambe r CHM Conve rte r Tria l ve rsion, http://w w w .proce sste x t.com/a bcchm.html
neopl as m, or angi odys pl as i a). Of part i cul ar concern i s col oni c neopl as m, i n l i ght of t he pat i ent 's recent hi s t ory of a change i n bowel habi t s (cons t i pat i on). Immedi at e management goal s i ncl ude fl ui d repl acement , correct i ng hypovol emi a i f pres ent , moni t ori ng s eri al hemogl obi n and hemat ocri t val ues t o be cert ai n t he pat i ent does not have ongoi ng bl eedi ng, arres t i ng any act i ve hemorrhage, and prevent i ng recurrent hemorrhage. T he gastroenterologist arranges for a colonoscopy. During the colonoscopy, several polyps are noted in the descending portion of the colon. T he polyps are 1–2 cm in size, and most are pedunculated. T hese lesions are removed by snare cautery and sent to pathology for evaluation.
Questions
What ot her proc edures c oul d have been us ed t o i nves t i gat e t hi s probl em?
Why w as c ol onos c opy us eful i n t hi s c as e?
What w oul d have prevent ed c ol onos c opy from bei ng a us eful di agnos t i c t ool ?
Discussion If bl eedi ng were act i ve and profus e, a t echnet i um-l abel ed RBC s can, angi ography, or bot h coul d l ocat e a bl eedi ng s ource, es peci al l y i f t he bl ood l os s exceeded 0.2–1.0 mL/mi n. Col onos copy i s t he t es t of choi ce i n t hi s s i t uat i on, becaus e i t can be bot h di agnos t i c and t herapeut i c i f a col oni c pol yp i s di s covered. If unavai l abl e, fl exi bl e s i gmoi dos copy and hi gh-qual i t y ai r cont ras t bari um enema woul d have been an accept abl e means of eval uat i ng t he col on. In t hi s pat i ent , col onos copy was us eful becaus e t here was no
Pa g e 3 1 6 9
ABC Ambe r CHM Conve rte r Tria l ve rsion, http://w w w .proce sste x t.com/a bcchm.html
act i ve bl eedi ng t o l i mi t t ot al exami nat i on of t he col oni c mucos a. However, i f t he bowel l umen had been coat ed wi t h bl ood, a s mal l pol yp or art eri ovenous mal format i on coul d have been mi s s ed. Al s o, i f hemodynami c i ns t abi l i t y makes t he pat i ent a poor candi dat e for adequat e s edat i on, a t horough eval uat i on of t he col on may not be pos s i bl e. Pathology indicates villous adenomatous polyps with no foci of carcinoma. T he patient is reassured, treated with iron supplements, and told to return in 1 year for a follow-up colonoscopy. If unremarkable, a repeat colonoscopy at 3–5 years would be indicated.
Questions
What fac t ors about pol yps i nc reas e t he ri s k of adenoc arc i noma?
What w oul d t he t reat ment have been i f foc i of mal i gnant c el l s had been found?
Discussion Adenomat ous and vi l l ous adenomat ous pol yps are cons i dered precurs ors for adenocarci noma. The ri s k of adenocarci noma i n pat i ent s wi t h col oni c pol yps i ncreas es i n proport i on t o t he amount of vi l l ous t i s s ue pres ent i n t he pol yp. Pol yps t hat are s es s i l e or great er t han 2 cm i n di amet er al s o carry an i ncreas ed ri s k for devel opi ng i nt o carci noma. Management of a mal i gnant focus i n a col oni c pol yp depends on t he t ype of pol yp. If a pol yp i s peduncul at ed on a l ong s t al k and t he mal i gnant cel l s have not i nvaded t he s t al k, t he pat i ent coul d be cured by s i mpl e col onos copi c pol ypect omy. However, i f t he mal i gnant cel l s have i nvaded t he s t al k, or i f t he pol yp i s s es s i l e and t he mal i gnant cel l s reach t he margi ns of res ect i on, t he pat i ent woul d requi re a s egment al col oni c res ect i on. In ei t her cas e, fol l ow-up col onos copy i s i ndi cat ed on a peri odi c bas i s i n pat i ent s who have adenomat ous
Pa g e 3 1 7 0
ABC Ambe r CHM Conve rte r Tria l ve rsion, http://w w w .proce sste x t.com/a bcchm.html
col oni c pol yps , or i f a mal i gnancy i s i dent i fi ed i n any s uch l es i ons . P.640
Case 13 HIV Infection A 37-year-old man, who was diagnosed with human immunodeficiency virus type 1 (HIV-1) infection 8 years ago presents to the emergency department with a one month history of headaches and mild confusion, with worsening symptoms, and increasing photophobia. He has experienced good overall health since diagnosis despite a steady decline in his CD4 count. He cannot recall his most recent CD4 count, but thinks it was about 3
200 cells/mm 1 year ago. He had been treated with various combinations of antiretroviral therapy (ART ), but had never achieved viral suppression due to poor adherence. He finally stopped ART completely due to intolerable side effects, and had not been back to clinic in the past 9 months. Imaging studies and spinal fluid analysis confirmed the diagnosis of cryptococcal meningitis, and the patient responded well to antifungal therapy. His CD4 count was found to be 36 cells/mm
3
and viral load is >100,000 copies/mL. He returns to clinic for follow-up, and after reviewing the risks and benefits of ART , with emphasis placed on the need to strict adherence to taking his medications regularly, he agrees to resume ART .
Questions
What i s t he val ue of t he CD4 c ount i n moni t ori ng pat i ent s w ho are i nfec t ed w i t h HI V-1?
What are t he major neurol ogi c probl ems enc ount ered i n peopl e w i t h HI V-1 i nfec t i on?
Pa g e 3 1 7 1
ABC Ambe r CHM Conve rte r Tria l ve rsion, http://w w w .proce sste x t.com/a bcchm.html
Di s c us s t he i mport anc e of adherenc e t o ART and t he c ons equenc e of poor adherenc e?
Discussion Meas uri ng CD4 count and HIV-1 RNA l evel s (referred t o as “vi ral l oad―) have become s t andard met hods of moni t ori ng HIV-1 i nfect ed pat i ent s over t i me. The t wo paramet ers , CD4 and vi ral l oad, refl ect di s t i nct as pect s of t he pat i ent 's cl i ni cal s t at us . CD4 i s an i ndex of t he current s t at e of t he i mmune s ys t em, and i n t he nat ural cours e of i nfect i on, wi t hout ant i ret rovi ral t reat ment , t he 3
CD4 count wi l l decl i ne, at an average rat e of 50 cel l s /mm per year (t he rat e of decl i ne oft en accel erat es wi t h advanced di s eas e). As t he CD4 count fal l s bel ow cert ai n t hres hol ds , t he pat i ent 's ri s k for cert ai n opport uni s t i c i nfect i ons (OIs ) i ncreas es —for i ns t ance, 3
bel ow 200 cel l s /mm t he ri s k of devel opi ng Pneumoc ys t i s c ari ni i pneumoni a (PCP) i ncreas es markedl y, and prophyl axi s s houl d be i ni t i at ed. In cont ras t , t he vi ral l oad meas ures ongoi ng repl i cat i on of t he vi rus , and predi ct s t he rat e of progres s i on of di s eas e (t he hi gher t he vi ral l oad, t he more rapi dl y t he CD4 count wi l l decl i ne over t i me), and i s mos t commonl y us ed t o moni t or t he s ucces s of ART. The goal of ART i s s uppres s i on of vi ral repl i cat i on t o bel ow t he l evel of det ect i on on s ens i t i ve vi ral RNA as s ays (us ual l y t o t he l evel of <400 or <50 copi es /mL). It i s common for pat i ent s i nfect ed by HIV-1 t o experi ence a l ong peri od of cl i ni cal l at ency aft er i ni t i al i nfect i on. In t he abs ence of ART, act i ve vi ral repl i cat i on occurs duri ng al l phas es of t he di s eas e, but s ympt oms us ual l y do not become apparent unt i l t here has been a s ubs t ant i al reduct i on i n hos t i mmune funct i on. Int ervent i on wi t h ant i ret rovi ral drugs can prevent , del ay, or revers e t hi s i mmune det eri orat i on. Current t reat ment gui del i nes () recommend i ni t i at i on of ant i ret rovi ral t herapy (ART) for al l pat i ent s wi t h a CD4 <200 3
cel l s /mm , cons i deri ng ART for pat i ent s wi t h CD4 >200 cel l s /mm
3
3
and <350 cel l s /mm dependi ng on pat i ent readi nes s and ot her 3
fact ors , and for t hos e wi t h CD4 >350 cel l s /mm and vi ral l oad
Pa g e 3 1 7 2
ABC Ambe r CHM Conve rte r Tria l ve rsion, http://w w w .proce sste x t.com/a bcchm.html 3
>100,000 copi es /mm s ome cl i ni ci ans i ni t i at e t herapy, whi l e ot hers recommend deferral of t herapy. In t hos e pat i ent s wi t h a CD4 of 350 3
3
cel l s /mm and vi ral l oad <100,000 copi es /mm , ART s houl d be deferred. Several neurol ogi c i nfect i ons can compl i cat e HIV-1 i nfect i on; t he t wo mos t common condi t i ons are crypt ococcal meni ngi t i s and t oxopl as mi c encephal i t i s . Bot h of t hes e i nfect i ons occur rel at i vel y l at e i n t he cours e of HIV-1 i nfect i on, general l y wi t h CD4 count s 3
l ower t han 50–100 cel l s /mm . Pat i ent s wi t h crypt ococcal i nfect i on commonl y pres ent wi t h chroni c meni ngi t i s charact eri zed by headache, phot ophobi a, and di ffus e, oft en mi l d, neurol ogi c s ympt oms . Pat i ent s wi t h t oxopl as mos i s more oft en pres ent wi t h focal neurol ogi c fi ndi ngs or s ei zures . Dement i a s econdary t o HIV-1 i nfect i on i s al s o common l at e i n t he cours e of t he di s eas e. Les s common neurol ogi c compl i cat i ons i ncl ude progres s i ve mul t i focal l eukoencephal opat hy, cent ral nervous s ys t em (CNS) l ymphoma, neuros yphi l i s , Li s t eri a meni ngi t i s , herpes s i mpl ex encephal i t i s , and cyt omegal ovi rus (CMV) encephal i t i s . Succes s of ART i s di rect l y l i nked t o a pat i ent 's abi l i t y t o correct l y t ake t hei r ant i ret rovi ral medi cat i ons as di rect ed on a regul ar s chedul e. It has been s hown t hat t he l i kel i hood of vi ral s uppres s i on drops P.641 off s i gni fi cant l y when adherence fal l s bel ow 95% (a pat i ent mus t t ake >95% of t hei r dos es —t hi s means t hat on a once-dai l y regi men, a pat i ent can mi s s onl y one dos e i n a mont h). Poor adherence t o ART l eads t o expos ure of repl i cat i ng vi rus t o s ubt herapeut i c pl as ma l evel s of ant i ret rovi ral drug, whi ch, i n conjunct i on wi t h t he hi gh mut at i on rat e of t he vi rus , l eads t o t he devel opment of res i s t ant vi ral s t rai ns . Thus , before s t art i ng a pat i ent on ART, i t i s cruci al t o adequat el y educat e t he pat i ent about HIV pat hogenes i s and t he way t he ART drugs work, emphas i z i ng t he mechani s m of res i s t ance devel opment , and warni ng about s i de-effect s and barri ers t o adherence. An i mport ant cons i derat i on before s t art i ng ART i s t o as s es s t he pat i ent 's
Pa g e 3 1 7 3
ABC Ambe r CHM Conve rte r Tria l ve rsion, http://w w w .proce sste x t.com/a bcchm.html
unders t andi ng and l i kel i hood of adherence, becaus e t he cons equences of devel opi ng res i s t ant vi rus predi s pos e t o i ncreas i ng compl exi t y i n des i gni ng a s ucces s ful ART regi men. T he patient was placed on the following medications: trimethoprim–sulfamethoxazole daily; fluconazole daily; abacavir, lamivudine, and lopinavir/ritonavir He returns to clinic for routine followup after 6 months on therapy. During this office visit, the patient says he is less peppy than usual, and reports watery diarrhea and crampy abdominal pain for the past 3-4 weeks. He thinks he may have had some fevers over the past 2 weeks, but he has not taken his temperature.
Questions
What i s t he purpos e of eac h of t he medi c at i ons t he pat i ent i s c urrent l y t aki ng?
What i s t he di fferent i al di agnos i s of w at ery di arrhea i n advanc ed HI V i nfec t i on?
What di agnos t i c t es t s c an be done t o det ermi ne t he c aus e of w at ery di arrhea?
Discussion Thi s pat i ent i s recei vi ng a number of medi cat i ons t hat are common t o pat i ent s wi t h HIV i nfect i on. Abacavi r, l ami vudi ne, and l opi navi r/ri t onavi r are four of t he growi ng l i s t of ant i ret rovi ral drugs avai l abl e for s uppres s i on of HIV vi ral repl i cat i on. Tri met hopri m–s ul famet hoxaz ol e (TMP/SMX) i s us ed t o prevent P. c ari ni i i nfect i on, but i t i s al s o part l y effect i ve i n prevent i ng s ympt omat i c t oxopl as mos i s and ot her common bact eri al i nfect i ons . Thi s s t rat egy i s cal l ed “pri mary prophyl axi s ― s i nce t he pat i ent has no hi s t ory of document ed PCP or t oxopl as mos i s , but i s 3
at ri s k for t hes e i nfect i ons wi t h a CD4 count of <100 cel l s /mm . The
Pa g e 3 1 7 4
ABC Ambe r CHM Conve rte r Tria l ve rsion, http://w w w .proce sste x t.com/a bcchm.html
fl uconazol e i s gi ven t o prevent a recurrence of t he pat i ent 's previ ous l y document ed crypt ococcal meni ngi t i s . In cont ras t t o t he us e of TMP/SMX, t he us e of fl uconazol e i n t hi s pat i ent i s an exampl e of “s econdary prophyl axi s ,― t o prevent recurrence of di s eas e i n s omeone wi t h document ed pri or i nfect i on. Becaus e t he rat e of s ympt omat i c recurrence of crypt ococcal meni ngi t i s i s hi gh, fl uconazol e s houl d be gi ven unt i l t he pat i ent 's i mmune funct i on 3
i mproves s ubs t ant i al l y (i .e. CD4 >200 cel l s /mm s us t ai ned over s everal mont hs ). W at ery di arrhea i s a common compl i cat i on of HIV i nfect i on. Many cas es are s el f-l i mi t ed, and no s peci fi c et i ol ogy i s det ermi ned. However, a wi de vari et y of pat hogens have been as s oci at ed wi t h t hi s s yndrome. The mos t common i n pat i ent s wi t h l ow CD4, and perhaps t he hardes t t o t reat , i s Crypt os pori di um parvum. Ot her prot oz oans t hat can caus e di arrhea of t hi s nat ure are Gi ardi a i nt es t i nal i s , mi cros pori di a, and I s os pora bel l i . Common bact eri al agent s of wat ery di arrhea i ncl ude Es c heri c hi a c ol i , Sal monel l a, and Campyl obac t er. W hen a pat i ent i s undergoi ng or has recent l y compl et ed a cours e of ant i bact eri al t herapy, di arrhea s econdary t o t he t oxi n produced by Cl os t ri di um di ffi c i l e s houl d al ways be cons i dered. Di s s emi nat ed i nfect i on wi t h Myc obac t eri um avi um-i nt rac el l ul are, CMV, and hi s t opl as ma can al s o caus e col i t i s and di arrhea. Di agnos t i c t es t s for wat ery di arrhea s houl d be ordered when t he di arrhea i s pers i s t ent or ext remel y s ympt omat i c. Us ual l y, s t ool s peci mens are s ent for mi cros copi c as s es s ment of prot ozoa (oft en cal l ed t he “ova and paras i t es ―—O & P—t es t ). The l aborat ory s houl d be not i fi ed t hat crypt os pori di a and ot her unus ual prot oz oa are bei ng s ought . Fecal l eukocyt es and rout i ne s t ool cul t ures s houl d al s o be ordered. In t he appropri at e cl i ni cal cont ext (recent ant i bi ot i c expos ure), t oxi n t es t s for C. di ffi c i l e s houl d be ordered as wel l . If t hes e noni nvas i ve t es t s do not yi el d a res ul t , furt her eval uat i on coul d ent ai l endos copi c s t udi es of t he l ower gas t roi nt es t i nal t ract , obt ai ni ng bi ops i es t o t es t for i nvas i ve MAI, CMV, or fungal i nfect i on, or as pi rat i on of t he upper i nt es t i nal cont ent s . A defi ni t i ve di agnos i s can be el us i ve i f t he fi rs t few s t ool
Pa g e 3 1 7 5
ABC Ambe r CHM Conve rte r Tria l ve rsion, http://w w w .proce sste x t.com/a bcchm.html
t es t s are negat i ve. On examination, the patient looks alert but pale. His temperature is 38°C, but otherwise, his vital signs are normal. T he patient has lost 6 pounds since his last visit, 6 weeks earlier. Otherwise, there are no focal abnormalities. Laboratory testing shows a hemoglobin of 8 g/dL and a white blood cell (WBC) 3
count of 3400 cells/mm with a normal differential. T he mean 3
corpuscular volume (MCV) is 89 µm , and the serum vitamin B 1 2 level is at the lower limit of normal. T he patient' s P.642 3
CD4 count is 48 cells/mm and the viral load is 45,000 copies/mL. Chemistries show mild elevation of aspartate aminotransferase (AST ) and alanine aminotransferase (ALT ).
Questions
Of w hat s i gni fi c anc e i s fever i n i ndi vi dual s w i t h advanc ed HI V i nfec t i on?
What i s t he di fferent i al di agnos i s of t he anemi a?
What i s t he s i gni fi c anc e of t he w ei ght l os s ?
Why has t hi s pat i ent not res ponded t o ant i ret rovi ral t herapy?
Discussion Fever i s a nons peci fi c fi ndi ng i n pat i ent s wi t h advanced HIV i nfect i on. The devel opment of new fever s houl d prompt a careful hi s t ory and phys i cal eval uat i on. In addi t i on t o t he l arge s pect rum of i nfect i ons t hat can produce a fever, a number of noni nfect i ous proces s es s uch as mal i gnanci es and drug react i ons s houl d be
Pa g e 3 1 7 6
ABC Ambe r CHM Conve rte r Tria l ve rsion, http://w w w .proce sste x t.com/a bcchm.html
cons i dered. If t he pat i ent hi s t ory, phys i cal exami nat i on, ches t radi ographs , and rout i ne l aborat ory t es t s do not s ugges t an organ-s peci fi c abnormal i t y, di s s emi nat ed i nfect i ons s uch as M. avi um-i nt rac el l ul are (MAI), hi s t opl as mos i s , CMV, and HIV i t s el f s houl d be cons i dered as caus es of t he fever. Anemi a i s common i n pat i ent s wi t h advanced HIV i nfect i on and has mul t i pl e pot ent i al et i ol ogi es . Mi crocyt i c anemi a s houl d prompt t he us ual eval uat i on for i ron defi ci ency, and s ources of bl ood l os s s uch as gas t roi nt es t i nal bl eedi ng. Normocyt i c anemi a i s frequent l y due t o chroni c i nfl ammat i on, but can be caus ed by bone marrow i nfi l t rat i on wi t h i nfect i ons s uch as MAI, Hi s t opl as ma, CMV, and parvovi rus B19, i n addi t i on t o ot her pat hogens . Macrocyt i c anemi a may refl ect nut ri t i onal defi ci enci es and are commonl y as s oci at ed wi t h HIV-rel at ed drug t herapy—z i dovudi ne, t ri met hopri m-s ul famet hoxaz ol e, and ganci cl ovi r can caus e macrocyt i c anemi a. In many cas es of drug-rel at ed anemi a, t he anemi a i s mi l d and onl y bears wat chi ng, but s omet i mes pat i ent s are gi ven l eucovori n (fol i ni c aci d) t o combat t he fol at e depl et i on brought on by zi dovudi ne or t ri met hopri m-s ul famet hoxaz ol e. The s erum l evel s of vi t ami n B 1 2 can be depres s ed i n pat i ent s wi t h acqui red i mmunodefi ci ency s yndrome (AIDS), but t here i s rarel y a res pons e t o i nject i ons of vi t ami n B 1 2 . W ei ght l os s i s a s i gn of act i ve di s eas e as s oci at ed wi t h HIV i nfect i on. In many pat i ent s , no obvi ous mal abs orpt i on, i nfect i on, or mal i gnancy i s pres ent t o account for wei ght l os s , s o effort s are di rect ed at i ncreas i ng t he l ean body wei ght . Prompt i dent i fi cat i on and t reat ment of underl yi ng i nfect i ons , as wel l as nut ri t i onal s uppl ement s and hormonal mani pul at i ons are t he mos t common i nt ervent i ons . Many i nfect i ons are as s oci at ed wi t h wei ght l os s , but i n t he abs ence of mal abs orpt i on or organ dys funct i on, di s s emi nat ed i nfect i ons s houl d be cons i dered fi rs t . The pat i ent fai l ure t o s uppres s vi ral repl i cat i on wi t h hi s ant i ret rovi ral agent s may have s everal caus es . Fi rs t , and mos t l i kel y, due t o hi s pri or ART expos ure and poor adherence, he has a hi gh l i kel i hood of harbori ng res i s t ant vi rus . Res i s t ance t es t i ng, had i t been performed before s t art i ng t he current regi men, may not
Pa g e 3 1 7 7
ABC Ambe r CHM Conve rte r Tria l ve rsion, http://w w w .proce sste x t.com/a bcchm.html
have reveal ed res i s t ance mut at i ons , becaus e “wi l d-t ype― vi rus oft en out -grows t he res i s t ant s t rai ns i n t he abs ence of t herapy, l eadi ng t o fal s e-negat i ve t es t i ng. Res i s t ance t es t i ng at t hi s t i me, whi l e t he pat i ent i s on hi s current regi men, can be very hel pful i n deci di ng on an al t ernat e regi men. Second, one mus t al ways cons i der whet her a pat i ent i s adheri ng t o hi s regi men, and i f he mi s s es dos es frequent l y, t hi s wi l l l ead t o i ncompl et e vi ral s uppres s i on and emergence of res i s t ant vi ral s t rai ns . Thi rd, he may have been i nfect ed wi t h a s t rai n of HIV-1 t hat was al ready res i s t ant t o one or more of t he ant i ret rovi ral drugs . Blood cultures for mycobacteria are reported as positive 2 weeks later. T he patient' s fever has persisted, and he has lost 3 more pounds. He still has no localizing complaints but continues to have intermittent diarrhea and abdominal pain. Repeat complete blood count (CBC) and chemistry tests show no change.
Question
Di s c us s t he i mpl i c at i ons of t he bl ood c ul t ure res ul t s .
What t reat ment i s i ndi c at ed?
Coul d t hi s i nfec t i on have been prevent ed?
Discussion The pos i t i ve bl ood cul t ure for mycobact eri a s t rongl y s ugges t s t hat t he pat i ent has di s s emi nat ed M. avi um-i nt rac el l ul are i nfect i on (MAI), a commonl y occurri ng l at e compl i cat i on of HIV i nfect i on, P.643 3
es peci al l y i n pat i ent s who have CD4 count s <50 cel l /mm . MAI account s for t he wei ght l os s , fevers , anemi a, and di arrhea s een i n t hi s pat i ent , as wel l as t he el evat ed l i ver t rans ami nas e l evel s .
Pa g e 3 1 7 8
ABC Ambe r CHM Conve rte r Tria l ve rsion, http://w w w .proce sste x t.com/a bcchm.html
Pat i ent s wi t h MAI i nfect i on have i nfi l t rat i on of many organs (wi t h very l i t t l e t i s s ue react i on), wi t h frequent hi gh t i t ers of mycobact eri a i n t he bl ood and bone marrow. Many affect ed pat i ent s experi ence a part i al or compl et e revers al of wei ght l os s when t he bact eri al i nfect i on i s brought under cont rol . Treat ment of MAI us ual l y i ncl udes a macrol i de ant i bi ot i c (cl ari t hromyci n or az i t hromyci n) + et hambut ol ± ri fabut i n. Ri fabut i n mus t be us ed wi t h caut i on i n pat i ent s t aki ng ant i ret rovi ral s , due t o s i gni fi cant drug i nt eract i ons . Many cl i ni ci ans i ns t i t ut e MAI prophyl axi s wi t h azi t hromyci n when 3
t he CD4 count i s l es s t han 50 cel l s /mm , al t hough t here i s concern of devel opi ng macrol i de res i s t ance wi t h expos ure t o a s i ngl e agent . T he patient is started on clarithromycin, and ethambutol. T he fevers and diarrhea resolve over the next few weeks, his energy level improves, and he is able to return to work full time. He returns to the clinic 4 weeks later to discuss changing his antiretroviral regimen. Resistance testing shows multiple thymidine-analog mutations (T AMs) along with the M184V mutation associated with lamivudine resistance; the K103M mutation, conferring cross-resistance to the entire class of NNRT I agents; and in the PI sequence, he has only one mutation, L90M. Based on these results, a regimen of T enofovir + Emtricitabine (both drugs are available combined in one tablet) given once daily along with Lopinavir/ritonavir given twice daily. When the patient returns in 4 weeks, viral load testing reveals a drop to 3,330 copies/mL. CD4 count is 61 3
cells/mm .
Question
I s t he res pons e t o t he new regi men adequat e?
What i s t he pat i ent 's prognos i s ?
Discussion
Pa g e 3 1 7 9
ABC Ambe r CHM Conve rte r Tria l ve rsion, http://w w w .proce sste x t.com/a bcchm.html
A decl i ne i n vi ral l oad of 1 l og i s expect ed at 4 weeks . becaus e t hi s pat i ent s t art ed t hi s regi men wi t h a vi ral l oad of 45,000. 1 l og woul d be 4,500, s o he i s current l y meet i ng vi ral s uppres s i on goal s . W e expect t hat he s houl d be s uppres s ed t o l es s t han 50 copi es /mL by 6 mont hs on t herapy t o confi rm s ucces s ful and s us t ai ned ART. However, at every cl i ni c vi s i t , adherence mus t be ques t i oned and emphas i z ed. Offeri ng t he s i mpl es t regi men, wi t h t he l eas t pi l l -burden and t he mi ni mum frequency of dos i ng per day wi l l opt i mi z e t herapy, but wi t h i ncreas i ng res i s t ance, t hes e goal s of s el ect i ng t herapy become i ncreas i ngl y di ffi cul t . For t hi s reas on, s ome cl i ni ci ans wi l l defer t herapy i n pat i ent s who pers i s t ent l y demons t rat e i nabi l i t y t o adhere t o a regi men, t o prevent t he accumul at i on of mut at i ons i n t he vi rus popul at i on. If t he pat i ent can mai nt ai n adherence t o hi s regi men, hi s prognos i s i s good, des pi t e hi s cont i nued l ow CD4 count . Key poi nt s i n cari ng for HIV-i nfect ed pat i ent s over t i me i s t o moni t or careful l y for new OIs , encourage adherence, and mi ni mi ze pi l l -burden t o t he ext ent pos s i bl e. As s oon as t hi s pat i ent 's CD4 count ri s es above 3
200cel l s /mm on t wo cons ecut i ve det ermi nat i ons over 6 mont hs , mos t prophyl act i c agent s can be s afel y di s cont i nued. However, keep i n mi nd t hat recons t i t ut i on of t he CD4 count can be del ayed even wi t h s ucces s ful vi ral s uppres s i on, es peci al l y i n t hos e pat i ent s who had s evere i mmunos uppres s i on when s t art i ng on t herapy. HIV care i s an ongoi ng rel at i ons hi p bet ween cl i ni ci an and pat i ent , and t he pat i ent mus t as s ume res pons i bi l i t y for hi s care i n part ners hi p wi t h hi s cl i ni ci an. The cl i ni ci an, i n t urn, mus t be s ens i t i ve t o t he “drug-fat i gue― whi ch occurs i n many pat i ent who mus t t ake HIV medi cat i ons regul arl y, and do everyt hi ng pos s i bl e t o as s i s t pat i ent s i n s ucces s ful l y remai ni ng on t herapy.
Case 14 Dyspnea A 58-year-old man calls an ambulance complaining that he “ cannot get enough air― and that he is “ suffocating.†• When the paramedics arrive, they find that the man is alert and oriented but that he cannot speak more than one or two
Pa g e 3 1 8 0
ABC Ambe r CHM Conve rte r Tria l ve rsion, http://w w w .proce sste x t.com/a bcchm.html
words at a time because of shortness of breath. On the way to the emergency department, the paramedics insert an intravenous line and administer fluids. T hey also give the man oxygen at a rate of 15 L/minute via mask. P.644
Questions
How w oul d you defi ne dys pnea?
What are your i mmedi at e c onc erns about t hi s pat i ent ?
What are s ome of t he mec hani s ms t hat produc e dys pnea?
Discussion From hi s compl ai nt s and phys i cal appearance, t hi s man appears t o be s ufferi ng from s ympt oms of dys pnea, a condi t i on t hat i s defi ned as any uncomfort abl e awarenes s of breat hi ng. It i nvol ves bot h t he percept i on of an abnormal s ens at i on and a react i on t o t hat percept i on. Dys pnea has many caus es , mos t of whi ch are cardi ac or pul monary i n ori gi n. The overri di ng concern i n a pat i ent wi t h dys pnea i s t o rul e out t he pos s i bi l i t y of s eri ous cardi ac or pul monary caus es of t he s ympt oms . Eval uat i on s houl d begi n wi t h a check of vi t al s i gns , i ncl udi ng bl ood pres s ure, pul s e, res pi rat ory rat e, and t emperat ure, t o begi n t o narrow t he l i s t of pot ent i al major cardi ac or pul monary caus es of dys pnea. Condi t i ons t hat are as s oci at ed wi t h s ympt oms of dys pnea i ncl ude pul monary vas cul ar di s eas e [e.g., pul monary embol i s m, pul monary hypert ens i on s econdary t o chroni c obs t ruct i ve pul monary di s eas e (COPD)], i nt ers t i t i al l ung di s eas e, as t hma, neuromus cul ar di s eas e, ches t wal l di s eas e, and conges t i ve heart fai l ure (CHF)
Pa g e 3 1 8 1
ABC Ambe r CHM Conve rte r Tria l ve rsion, http://w w w .proce sste x t.com/a bcchm.html
i ncl udi ng t hat caus ed by t he vari ous t ypes of heart di s eas e i ncl udi ng val vul ar di s eas e, hypert ens i on, cardi omyopat hy, and pul monary edema. Al t hough ches t pai n woul d be a more common pres ent i ng s ympt om of myocardi al i nfarct i on (MI) t han dys pnea woul d be, MI s houl d be cons i dered and rul ed out wi t h cardi ac enz yme s t udi es and an el ect rocardi ogram (ECG). Anot her concern i n t hi s cas e (i .e., a new pat i ent wi t hout a known caus e of dys pnea) i s t he admi ni s t rat i on of oxygen at a hi gh fl ow rat e (i .e., 15 L/mi nut e), whi ch may be t he wrong approach i f t hi s man has obs t ruct i ve pul monary di s eas e wi t h carbon di oxi de ret ent i on. Del i very of i ns pi red oxygen at a hi gh concent rat i on can wors en carbon di oxi de ret ent i on. On arrival at the emergency department, the man is still short of breath and breathing at a rate of 28 respirations/minute. His other vital signs show a pulse of 110 bpm, a blood pressure of 150/85 mm Hg, and a temperature of 38.1°C. Initial history reveals that 6 days ago the patient developed an upper respiratory infection with rhinorrhea and a cough productive of thick yellow sputum. He has been short of breath for the last few days, but his symptoms worsened in the last several hours. He notes that he is more short of breath with minimal exertion and often begins to cough as a result of any effort. He denies chest pain or tightness. T he patient' s past medical history is significant because of two previous hospital admissions for similar episodes. He also has a history of hypertension, for which he is supposed to take a mild diuretic once a day; however, he ran out of the drug and has not yet refilled his prescription. He uses an inhaler as needed but denies other medications or drug allergies. T he patient says that he has smoked two packs of cigarettes per day for 30 years and that he drinks three to four beers per night. He is not married and recently lost his job in construction because he tires too easily.
Questions
Pa g e 3 1 8 2
ABC Ambe r CHM Conve rte r Tria l ve rsion, http://w w w .proce sste x t.com/a bcchm.html
How do t hes e des c ri pt i ons of t he pat i ent 's s ympt oms hel p you i n t hi nki ng about your di fferent i al di agnos i s ?
How c an you di fferent i at e bet w een c ardi ac and pul monary c aus es of dys pnea?
Discussion The s ubs equent hi s t ory i s more s ugges t i ve of an acut e i l l nes s s uperi mpos ed on an underl yi ng chroni c proces s (e.g., chroni c l ung di s eas e wi t h CHF or upper res pi rat ory i nfect i on exacerbat i on) t han of an acut e proces s (e.g., acut e MI, pul monary embol i s m). The di fferent i al di agnos i s at t hi s poi nt i ncl udes obs t ruct i ve pul monary di s eas e, as t hma, CHF, i nfect i on, and a t umor (a pos s i bi l i t y i n a pat i ent wi t h COPD and a hi s t ory of ci garet t e s moki ng). The phys i ci an s houl d al s o keep i n mi nd t he pos s i bi l i t y of work-rel at ed expos ures s uch as as bes t os , as t hma-s ens i t i zers , and ot her occupat i onal pul monary t oxi ns . Al t hough t he chroni ci t y of s ympt oms poi nt s more t oward a chroni c t han an acut e proces s , t he pos s i bi l i t y of an acut e MI s houl d s t i l l be rul ed out . Recurrent pul monary embol i s m al s o i s a pos s i bi l i t y at t hi s poi nt , and vent i l at i on-perfus i on s canni ng wi t h l ower-ext remi t y Doppl er s t udi es may be i ndi cat ed. P.645
Physical examination shows a well-developed, slightly overweight, white man in moderate respiratory distress with vital signs as noted above. His skin is moist without cyanosis, his neck shows mildly dilated jugular veins, his lungs reveal a prolonged expiratory phase with expiratory wheezes in all lung fields and poor respiratory effort, and examination of his heart reveals a prominent S 3 but no murmurs. Examination of his abdomen is unremarkable, revealing a slightly enlarged liver, and his extremities show 2+ peripheral edema.
Pa g e 3 1 8 3
ABC Ambe r CHM Conve rte r Tria l ve rsion, http://w w w .proce sste x t.com/a bcchm.html
Questions
What do t hes e phys i c al s i gns s ugges t ?
What di s eas e proc es s w oul d be at t he t op of your di fferent i al di agnos i s now ?
What l aborat ory t es t s or s t udi es w oul d you l i ke t o obt ai n?
Discussion The expi rat ory wheez i ng and i ncreas ed res pi rat ory effort not ed on phys i cal exami nat i on of t hi s pat i ent s ugges t t he pres ence of obs t ruct i ve l ung di s eas e. In addi t i on, t here i s an S 3 gal l op and peri pheral edema. The S 3 and di l at ed jugul ar vei ns s ugges t l eft -s i ded heart fai l ure, and t he enl arged l i ver and peri pheral edema s ugges t ri ght -s i ded heart fai l ure. The wheez es may be as s oci at ed wi t h ei t her obs t ruct i ve l ung di s eas e or CHF. At t hi s poi nt , i t i s es s ent i al t o di fferent i at e cardi ac from pul monary caus es of t hi s man's di s t res s t o pl an an effect i ve t reat ment . A ches t radi ograph al ong wi t h an ECG, cardi ac enz yme s t udi es , and a compl et e bl ood count (CBC) wi t h di fferent i al woul d hel p det ermi ne whet her t he pat i ent has ei t her or bot h underl yi ng condi t i ons . T he emergency department physician calls for the patient' s old records, orders some laboratory tests (including arterial blood gas studies on oxygen, a CBC, a serum electrolyte panel, a cardiac enzyme analysis, a sputum culture, a “ stat― ECG, and a portable radiograph). T he ECG suggests some right axis deviation and sinus tachycardia but no ischemic changes. T he results of the arterial blood gas studies are returned promptly and show a pH of 7.44, a PaO 2 of 81 mm Hg, a PaCO 2 of 50 mm Hg, and an O 2 saturation of 95% on 3 L FIO 2 by nasal cannula.
Pa g e 3 1 8 4
ABC Ambe r CHM Conve rte r Tria l ve rsion, http://w w w .proce sste x t.com/a bcchm.html
Question
What s ort of pat t ern do t he art eri al bl ood gas fi ndi ngs s ugges t ?
Discussion The art eri al bl ood gas fi ndi ngs repres ent adequat e oxygenat i on (normal PaO 2 = 80 mm Hg or above) wi t h mi l d hypercarbi a (normal PacO 2 = 40 mm Hg) and a normal pH. The normal pH s ugges t s a chroni c proces s . T he patient' s chest radiograph suggests a mild prominence of the pulmonary vascular markings. Otherwise, there are no radiographic changes.
Question
Whi c h di agnos es are l es s l i kel y bec aus e of t hes e radi ographi c res ul t s ?
Discussion The radi ographi c fi ndi ngs make t he pos s i bi l i t y of pos t obs t ruct i ve pneumoni t i s , t umor, or pneumoni a much l es s l i kel y and are more compat i bl e wi t h a COPD exacerbat i on wi t h s ome mi l d CHF. There i s no mas s t o s ugges t a t umor and t here are no i nfi l t rat es t o s ugges t pneumoni a. Because the patient continues to have respiratory distress and is very uncomfortable without the oxygen, he is admitted to the hospital. His old records reveal that his previous admissions were for exacerbations of bronchitis. He is started on fluids, oxygen, diuretics, and a broad-spectrum antibiotic.
Questions
Pa g e 3 1 8 5
ABC Ambe r CHM Conve rte r Tria l ve rsion, http://w w w .proce sste x t.com/a bcchm.html
What are your goal s for t herapy for t hi s pat i ent ?
How c an you prevent s i mi l ar epi s odes from happeni ng agai n?
P.646
Discussion The goal s for t hi s pat i ent are t o cont rol t he exacerbat i on of hi s bronchi t i s wi t h ant i bi ot i cs and bronchodi l at ors and t o ret urn hi m t o bas el i ne res pi rat ory s t at us . Peak fl ow meas urement s s houl d be obt ai ned before and aft er bronchodi l at ors are admi ni s t ered, and t he pos s i bi l i t y of us i ng cort i cos t eroi ds al ong wi t h t he rout i ne bronchodi l at or t herapeut i c regi men s houl d be ent ert ai ned. The pos s i bi l i t y of underl yi ng heart di s eas e al s o s houl d be addres s ed. An echocardi ogram woul d be i ndi cat ed at t hi s t i me t o det ermi ne i f t here i s any cardi ac funct i onal abnormal i t y. The mos t l i kel y di agnos i s here i s chroni c bronchi t i s exacerbat ed by an upper res pi rat ory i nfect i on wi t hout pneumoni a, wi t h an underl yi ng pos s i bi l i t y of mi l d CHF. Fut ure prophyl axi s s houl d i ncl ude i mmuni z at i on agai ns t i nfl uenz a as wel l as pneumococcal vacci ne. In addi t i on, t he pos s i bi l i t y of ent eri ng a pul monary rehabi l i t at i on program s houl d be di s cus s ed wi t h t he pat i ent .
Pa g e 3 1 8 6